Kiran Bank Po Reasoning Chapterwise Solved Papers 1999 Till Date (English Medium) 9821874015, 9821643815

Kiran Bank Po Reasoning Chapterwise Solved Papers 1999 Till Date (English Medium) (3716) 10900+Objective Questions With

263 124 28MB

English Pages [1041] Year 2022

Report DMCA / Copyright

DOWNLOAD FILE

Polecaj historie

Kiran Bank Po Reasoning Chapterwise Solved Papers 1999 Till Date (English Medium)
 9821874015, 9821643815

Table of contents :
Cover
Title
Copyrights
About The Book
Contents
Untitled
1 : ALPHABET
2 : SERIES
3 : ANALOGY
4 : CLASSIFICATION
5 : CODING/DECODING
6 : BLOOD RELATIONSHIP
7 : SYMBOLS & NOTATIONS
8 : DISTANCE & DIRECTION
9 : RANKING/ARRANGEMENT
10 : INPUT
11 : SYLLOGISM
12 : PROBLEM SOLVING
13 : ARGUMENTATION
14 : ASSUMPTION
15 : CONCLUSIONS/ INFERENCES
16 : COURSES OF ACTION
17 : DATA SUFFICIENCY
18 : DATA ANALYSIS
19 : CAUSE & EFFECT
20 : MISCELLANEOUS

Citation preview

n K ir a

’s

Chapterwise Questions with explanation of Previous Years’ Exams conducted by Nationalised Banks, Gramin Banks, IBPS (PO/MT/SO, RRBs Officer), SBI & SBI Associates, RBI (Grade ‘B’), NABARD (Grade–‘A’) & Insurance Sector (AAO, AO etc.)

FOR FRANCHISEE CONTACT :

RU-67, PITAMPURA, DELHI-110034, Ph : 27345258, 27342249, Fax : 27345258 BPRE–1

Delhi : RU-67, Opposite Power House, Pitampura, Delhi-110034, Ph. : 9821874015, 9821643815 Email : [email protected] www.kiranprakashan.com

For the books & magazines of Kiran Prakashan contact your nearest booksellers. For detailed inform ation log on our w ebsite : www.kiranprakashan.com

© KIRAN INSTITUTE OF CAREER EXCELLENCE PVT. LTD. (KICX) NEW EDITION The copyright of this book is entirely with the Kiran Institute of Career Excellence Pvt. Ltd.The reproduction of this book or a part of this will be punishable under the Copyright Act. All disputes subject to Delhi jurisdiction. Every possible effort has been made to ensure that the information contained in this book is accurate at the time of going to press, and the publishers and authors cannot accept responsibility for any errors or omissions, however caused. No responsibility for loss or damage occasioned to any person acting, or refraining from action, as a result of the material in this publication can be accepted by the editor, the publisher or any of the authors. Reviewed by : Think Tank of PRATIYOGITA KIRAN, KIRAN PRAKASHAN & KICX Assistance : ● Rakesh Kumar ● Govind Pd. Singh ● Sanket Sah ● Auranjeb Khan Design & Layout by : KICX COMPUTER SECTION, New Delhi.

Must Read

Price :

1234567890123456789012345678901212345678901234567890123456789012123456789012 1234567890123456789012345678901212345678901234567890123456789012123456789012 1234567890123456789012345678901212345678901234567890123456789012123456789012 1234567890123456789012345678901212345678901234567890123456789012123456789012 1234567890123456789012345678901212345678901234567890123456789012123456789012

USEFUL FOR ALL COMPETITIVE EXAMS

555

Price :

299

BPRE–2

Buy Today

Price :

295

About the Book......✍ The significance of past never diminishes, either in effect or substance. To forget this moot point becomes suicidal sometimes. A minute observation of the past and accumulated experiences help us in shaping our present. The positive and compendious conclusions drawn from the past become our pioneer in charting new ways and teach us to rectify our remedies. Our present is, thus, beautified. Where there is beauty, attraction cannot be negated. It is in this way that the foundation of a concrete future is further solidified. Aspirations turn into realities. This is a realistic reflection. If the past is forgotten by marginalizing this realism, the truth of past surfaces as a path strewn with pebbles. If we preserve and cherish these pebbles of the past and make correct use of them, they become the linchpin of an edifice called success. Thus, the intricacies of our path become effortless and our destination gets more and more intelligible. Kiran Prakashan Pvt. Ltd., which has been at your service for decades and helping you with the multitude of relevant offerings, has made a meaningful effort to put together these invaluable and very useful pebbles in the form of a book, titled Kiran’s BANK PO REASONING Chapterwise Solved Papers. It is indeed a momentous occasion to present the Kiran’s BANK PO REASONING Chapterwise Solved Papers, based on the new syllabus and pattern announced by the Institute of Banking Personnel Selection (IBPS), SBI, RBI NABARD and Insurance sector. Kiran Prakashan has been keeping you updated with all the relevant information related to these very changes. This Book is another example of the commitment we feel towards our readers, imparting completeness in their search for a better and secure future. The book before you, Kiran’s BANK PO REASONING Chapterwise Solved Papers, is a compilation of chapterwise questions of Reasoning that have been asked in the various competitive examinations by Nationalised Banks, Gramin Banks, IBPS (PO/MT/SO RRBs Officer), SBI, RBI (Grade‘B’), NABARD GRADE–‘A’ & Insurance Sector (AAO, AO etc.). This book has been conceived as a holistic treatment and single solution to all the difficulties that a candidate encounters while appearing at the Nationalised Banks, Gramin Banks, IBPS, SBI, RBI NABARD & Insurance Sector examinations. The Think-Tank of Kiran Prakashan has taken into account the needs and difficulties of candidates and made an attempt to simplify the subject matter by de-constructing every thread and each pattern. Not only have the questions asked in the different examinations conducted by Banks Exams been compiled at one place, but also they have been compartmentalized topically and made easy to grasp and digest with the help of comprehensive explanations. This is a rare collection of more than 10900 questions and their respective explanations. It wouldn’t be bragging if we say, do master these questions and their explanations, you will automatically find yourself at ease with most of what surfaces in form of examination questions. In other words, master this book sincerely; success will itself become the mistress. Kiran’s BANK PO REASONING Chapterwise Solved Papers has been parceled out into 24 Chapters, (20 verbal + 4 Non-verbal) each dealing with bulk of questions and explanations. The Chapters are : VERBAL REASONING : Alphabet, Series, Analogy, Classification, Coding/ Decoding, Blood Relationship, Symbols & Notations, Distance & Direction, Ranking/ Arrangement, Input, Syllogism, Problem Solving, Argumentation, Assumption, Conclusions/Inferences, Courses of Action, Data Sufficiency, Data Analysis, Cause & Effect, Miscellaneous. NON-VERBAL REASONING : Series, Analogy, Classification, Miscellaneous. An attempt has been made to impart completeness to this work which you will find more and more indispensable, the more you get acquainted with it. BPRE–3

Explanations to the questions serve in some way as highly directed study material. We have ensured not to bombard you with bulky study materials which sometimes prove futile if a candidate is facing severe crunch of time. Johann Wolfgang von Goethe, considered the supreme genius of modern German literature, said once: “What is not started today is never finished tomorrow.” And, we have turned the key on with the Book titled Kiran’s BANK PO REASONING Chapterwise Solved Papers, where facts have been consolidated and refined and presented in the easy-to-digest objective question format which have actually been already asked in the different levels and formats of competitive examinations. Scientifically it has been proved that you tend to remember anything for long if that has been asked from you and you found yourself in a testing situation. So, this book has been scientifically weaved on the principle of ‘test to prepare. Some Important Facts ● Always use pen and paper to solve Reasoning. ● Study the facts and formulae of every topic. ● Always try to understand the root/source of the formulae & Tricks. ● Give a subjective approach to your study, otherwise objective approach will serve no objective. ● Comprehend the application of formulae and Tricks. ● A proper use makes the solution easy whereas an improper use hardens it. ● Solve questions of varied kind and degree. ● Go ahead from easy to harder questions. ● Believe in complete solution of questions. ● Practice makes a man perfect. ● Reasoning needs continuous practice. ● While preparing, underline the important points. ● This will help you during revision. ● It is perilous walking on a tight rope. ● So, give a wide dimension to your study. ● Most importantly, think positive, do positive. On making a microscopic appraisal of the above given explanations, you will discover that they are not merely explaining a specific question but they are more in the nature of introducing you to a subject or a theme. You will find yourself at ease in similar other questions as well. In short, this unique compilation of more than 10900 questions will serve you in more than one way. Last, but not the least, there is no shortcut to success. Only hard work and perseverance pays rich dividends in the long term. So, it is desirable on your part to make the best out of this unique offering before you. Let us remind you that the questions you find in this book will not only help you prepare for the Banks PO/Management Trainees, Specialist Officer Examinations, they will equip you with the much required knowledge and insight in ‘cracking’ other examinations as well. Hugh Nibley, the famous American author and Mormon apologist said: “Only if you reach the boundary will the boundary recede before you. If you don’t, if you confine your efforts, the boundary will shrink to accommodate itself to your efforts. If you can only expand your capacities by working to the very limit.” Our researchers have taken painstaking efforts in the direction of accommodating and expanding to the limit. The guiding principle has been to have an eye on whatever is significant, and when you do have such a microscopic eye, there is little that can be expected to have been ignored. So, here is our new, novel and unique offering to you, Kiran’s BANK PO REASONING Chapterwise Solved Papers. Read it, incorporate and digest it to reach the limits of life. See to know what it is all about. With the Best Wishes (Publisher) Satyanarayan Prasad Email : [email protected]

BPRE–4

CONTENTS S.No.

CHAPTERS

Total No. of Questions

Page No.

1.

Alphabet

[No. of Questions 247 (132+115*)]

BPRE–11

2.

Series

[No. of Questions 229 (165+64*)]

BPRE–26

3.

Analogy

[No. of Questions 151 (95+56*)]

BPRE–38

4.

Classification

[No. of Questions 162 (75+87*)]

BPRE–47

5.

Coding/Decoding

[No. of Questions 861 (569+292*)]

BPRE–54

6.

Blood Relationship

[No. of Questions 509 (444+65*)]

BPRE–113

7.

Symbols & Notations

[No. of Questions 972 (666+306*)]

BPRE–160

8.

Distance & Direction

[No. of Questions 300 (265+35*)]

BPRE–234

9.

Ranking/Arrangement

[No. of Questions 2093 (1558+535*)]

BPRE–272

10.

Input

[No. of Questions 546 (391+155*)]

BPRE–438

11.

Syllogism

[No. of Questions 955 (672+283*)]

BPRE–498

12.

Problem Solving

[No. of Questions 1539 (1371+168*)]

BPRE–657

13.

Argumentation

[No. of Questions 170 (92+78*)]

BPRE–807

14.

Assumption

[No. of Questions 280 (124+156*)]

BPRE–825

15.

Conclusions/Inferences

[No. of Questions 363 (184+179*)]

BPRE–843

16.

Courses of Action

[No. of Questions 371 (118+253*)]

BPRE–873

17.

Data Sufficiency

[No. of Questions 470 (317+153*)]

BPRE–893

18.

Data Analysis

[No. of Questions 254 (201+53*)]

BPRE–965

19.

Cause & Effect

[No. of Questions 227 (131+96*)]

BPRE–992

20.

Miscellaneous

[No. of Questions 267 (213+54*)]

BPRE–1013

[TOTAL NO. OF QUESTIONS 10966 (7783+3183*)] * No. of questions available online/visit kicx.in

BPRE–5

Chapterwise Distribution of Questions asked in the Nationalised Banks, Gramin Banks, IBPS (PO/MT/SO) Exams Held from 2011 to 2021

1

3

4

1



1

1

1



1



1













1





3.

Analogy







2

















1





4.

Classification





1

















1







5.

Coding/Decoding







5

3

6

5

6

5

4





5



4

6.

Blood Relationship











1

3

4



1



3

2

5

5

7.

Symbols & Notations









5



6

5

5

3

3



5

5



8.

Distance & Direction

1







1

1





1





3





1

9.

Ranking/Arrangement



18

13

12

6

10

12

7

11

11

9

16

10

10

10 —

IBPS Bank PO/MT CWE-III, 26.10.2013

IBPS Bank PO/MT CWE (Prelim Exam) 04.12.2021





IBPS RRBs Officer CWE (Prelim Exam) 31.12.2020

IBPS Bank PO/MT CWE (Prelim Exam), 12.10.2019



2

IBPS Specialist Officer (Agriculture) CWE 29.10.2017

1



Corporation Bank SO (Marketing) Exam, 22.02.2014





IBPS Specialist Officer CWE, 17.03.2013

2

Series

IBPS RRBs Officer CWE, 09.09.2012

Alphabet

2.

CHAPTERS

IBPS Bank PO/MT CWE, 17.06.2012

1.

S. No.

IBPS Bank PO/MT CWE, 18.09.2011

IBPS Bank PO/MT CWE (Prelim Exam), 14.10.2018

IBPS Bank PO/MT CWE (Main Exam), 18.11.2016

IBPS Specialist Officer (IT) CWE 14.02.2016

IBPS Bank PO/MT CWE–V (Main Exam) 31.10.2015

Bank of Baroda Junior Management Grade/ Scale-I Exam, 18.04.2015

REASONING

VERBAL REASONING

10. Input

5

4



3

5





4

5

5

5







11. Syllogism

2

7

5

5

5

4

6

5

5

3

5

3



5

4

12. Problem Solving

10







5

6

7

8

13

10

17

5

10

15

10

13. Argumentation







3









1

2











14. Assumption

5

1



2

2

2



















15. Conclusions/Inferences

5

5

5

7

2



1

1

2













16. Courses of Action











10







2











17. Data Sufficiency

5

5





5



5

5



3

5









18. Data Analysis





3

5

5

5









1

3







19. Cause & Effect

5





6

1

2

2

1

1

3

1









20. Miscellaneous









5

1







3

3









NON-VERBAL REASONING 21. Series

5

5

10

























22. Analogy































23. Classification





5

























24. Miscellaneous































50

50

40

50

50

50

50

50

50

50

50

35

35

40

35

TOTAL NUMBER OF QUESTIONS

BPRE–6

Chapterwise Distribution of Questions asked in the SBI Exams Held from 2011 to 2019.

SBI PO (Prelim) Exam, 09.06.2019

SBI PO (Prelim) Exam, 08.07.2018

SBI PO Phase-II (Main) Exam, 04.06.2017

SBI PO (Preliminary) Exam 07.05.2017

SBI PO Online (Main) Exam, 31.07.2016

SBI PO Phase–I (Pre.) Online Exam 27.06.2015

SBI Associate PO Online Exam, 30.11.2014

SBI Management Executive Exam, 23.02.2014

CHAPTERS

SBI PO Exam, 28.04.2013

S. No.

SBI Associate Banks PO Exam, 07.08.2011

REASONING

VERBAL REASONING 1. Alphabet





1











2

1

2. Series



















2

3. Analogy



















1

4. Classification

1















1



5. Coding/Decoding

6

5





5

1

1







6. Blood Relationship







3

3

1

1



2

3

7. Symbols & Notations 8. Distance & Direction

3 —

10 —

5 —

6 —

5 2

5 —

5 1

— 9

4 4

4 —

9. Ranking/Arrangement

12

5

5

11

10

5

6

10

9

10

10. Input

3

5



6



5



3

4



11. Syllogism



5

5

6

5

5







4

12. Problem Solving





5

3

5

10

21

12

9

10

13. Argumentation

2

2







3









14. Assumption

1

1

5









1





15. Conclusions/ Inferences 16. Courses of Action

6 —

1 1

5 5

— 2

— —

— —

— —

3 —

— —

— —

17. Data Sufficiency

5

5

4

5



5









18. Data Analysis

6

5

10

5



5



6





19. Cause & Effect







1







1





20. Miscellaneous







2













21. Series

5

5

















22. Analogy 23. Classification

— —

— —

— —

— —

— —

— —

— —

— —

— —

— —

24. Miscellaneous





















50

50

50

50

35

45

35

45

35

35

NON-VERBAL REASONING

TOTAL NUMBER OF QUESTIONS

BPRE–7

Chapterwise Distribution of Questions asked in the RBI Officer Grade–B and NABARD Officer Grade–A Exams Held from 2011 to 2021.

RBI Officer in Grade ‘B’ Phase-I Exam, 16.08.2018

RBI Grade ‘B’ Officer Exam, 09.11.2019

RBI Officer Grade ‘B’ Phase-I Exam, 06.03.2021





1



1



2

1

10







1







1

1



3.

Analogy

1





















4.

Classification

1





1















5.

Coding/Decoding



6

8

5

5



5



5



5

6.

Blood Relationship





5

2





1



4



10

7.

Symbols & Notations

10

5

5

5

5

5

4



5





8.

Distance & Direction

15





1







3

3

4

2

9.

Ranking/Arrangement

5



8

12

13

10

5

9

10

9

21

Input

5

6

5

2

5

5

5





5



11.

Syllogism



6

6

6

5

5

6

3



4

5

12.

Problem Solving

5



8

11

17

8

16

4

15

20

2

13.

Argumentation

5



1

1



1

4



1

1



14.

Assumption

10

5













1

1



RBI Officer Grade ‘B’ Phase-I Exam, 04.09.2016 (Shift-II)



1

NABARD A.M. Online Exam, 15.05.2016

2

Series

RBI Officer Grade ‘B’ Phase-I Exam, 21.11.2015

Alphabet

2.

RBI Officer Grade ‘B’ Phase-I Exam, 03.08.2014

1.

S. No. CHAPTERS

RBI Grade–B Officer Exam, 06.02.2011

NABARD A.M. Online Exam, 06.08.2017

NABARD Officer Grade ‘A’ Online Exam, 01.03.2015

RBI Officer Grade ‘B’ Online Exam, 25.08.2013

REASONING

VERBAL REASONING

10.

15.

Conclusions/ Inferences 5

7

1



1

1

2



3

1



16.

Courses of Action











1

1

1

2



17.

Data Sufficiency

10



3

2

5

3

5



5

2



18.

Data Analysis



17

7



1

1











19.

Cause & Effect



3

3

2

1

1

2



2

1

5

20.

Miscellaneous



5









3



2

8



21.

Series























22.

Analogy























23.

Classification























24.

Miscellaneous























80

60

60

50

60

40

60

20

60

60

60

5

NON-VERBAL REASONING

TOTAL NUMBER OF QUESTIONS

BPRE–8

Chapterwise Distribution of Questions asked in the Insurance Officer Exams Held from 2012 to 2019.

LIC AAO Main Exam, 28.06.2019

LIC AAO Prelim Exam, 05.05.2019

NICL A.O. Exam, 05.06.2017

LIC AAO Online Exam, 06.03.2016

NIACL Administrative Office (AO) Online Exam, 12.01.2015

NICL (GIC) Administrative Officer Exam, 15.12.2013

LIC Assistant Administrative Officer Exam, 12.05.2013

CHAPTERS

United India Insurance AAO Exam, 26.05.2013

S. No.

United India Insurance AAO Exam, 03.06.2012

REASONING

VERBAL REASONING 1. Alphabet

3

3















2. Series

2

1



1











3. Analogy

14





2











4. Classification 5. Coding/Decoding

2 —

— 7

— 6

7 3

— —

— —

— 6

— —

— —

6. Blood Relationship



3

1

2

2

2

4



1

7. Symbols & Notations

5







6



2

4

3

8. Distance & Direction







1

2

2

3

3

1

9. Ranking/Arrangement

5



1

3

6

10

10

10

10

10. Input

5

5





5









11. Syllogism

6

6

5



8

5



5

3

12. Problem Solving 13. Argumentation

— 5

1 —

— —

— —

4 —

11 —

10 —

10 —

10 —

14. Assumption

5

















15. Conclusions/ Inferences



4

6













16. Courses of Action





1













17. Data Sufficiency



5

3



5







2

18. Data Analysis

5

15

7



11









19. Cause & Effect









1









20. Miscellaneous

3





6







3



NON-VERBAL REASONING 21. Series



















22. Analogy



















23. Classification



















24. Miscellaneous



















60

50

30

25

50

30

35

35

30

TOTAL NUMBER OF QUESTIONS

BPRE–9

C 

ALPHABET

1

ALPHABET

QUESTIONS FROM 1999 TO 2010 ARE AVAILABLE ONLINE NATIONALISED BANKS & IBPS PO/MT/SO EXAMS 1. If all the vowels in the word ‘SAINTLY’ are rearranged in the alphabetical order from left to right, followed by the consonants in the alphabetical order from left to right, the position of which of the following five alphabets will remain unchanged ? (1) S (2) T (3) L (4) A (5) Y (BOB Junior Management Grade/Scale–I Exam. 18.04.2015)

2. It is possible to make only one meaningful English word with the third, fourth, seventh and the eighth letters of the word ‘VIDEOGRAPHY’, which would be the second letter of that word from the right end ? If more than one such word can be formed, give ‘X’ as the answer. If no such word can be formed, give Z as your answer. (1) D (2) A (3) R (4) X (5) Z (BOB Junior Management Grade/Scale–I Exam. 18.04.2015)

3. How many such pairs of letters are there in the word ‘MACHINERY’ each of which has as many letters between them in the word (in both the forward and backward directions), as they have between them in the English alphabetical series ? (1) Three (2) Two (3) None (4) One (5) More than three (BOB Junior Management Grade/Scale–I Exam. 18.04.2015)

4. How many such pairs of letters are there in the word ‘LENGTH’, each of which has as many letters between them in the word (in both forward and backward directions) as they have between them in the English alphabetical series ?

(1) None (2) One (3) Two (4) Three (5) More than three (IBPS RRBs Officer Scale–I & II CWE 13.09.2015)

5. If each vowel in the word ‘REMAINS’ is changed to next alphabet according to the English alphabetical series and each consonant is changed to previous letter, how many alphabet will be there in the English alphabetical series between the letters which are at the extreme ends of the new word thus formed ? (1) None (2) One (3) Two (4) Three (5) Four (IBPS RRBs Officer Scale–I & II CWE 13.09.2015)

6. If in the word EQUALITY, the positions of first and the fifth letters are interchanged, similarly the positions of the second and the sixth letters are interchanged and so on, which letter will be third from the right end? (1) I (2) U (3) Q (4) E (5) L (IBPS Bank PO/MT CWE–V Main Exam. 31.10.2015)

7. How many such pairs of letters are there in the word REFRESHING each of which has as many letters between them (in both forward and backward directions) in the word as they have in the English alphabet? (1) Three (2) One (3) Two (4) None (5) More than three (IBPS Bank PO/MT CWE–V Main Exam. 31.10.2015)

8. If in the word ISOLATE, all the consonants are replaced by the previous letter in the alphabet and all the vowels are replaced by the next letter and then all the letters are arranged alphabetically, which letter will be third from the right end?

BPRE–11

(1) Q (2) P (3) Other than those given as options (4) N (5) B (IBPS Bank PO/MT CWE–V Main Exam. 31.10.2015)

9. How many meaningful English words can be made from the letters IMET, using all the letters but each letter only once in each word? (1) None (2) One (3) Three (4) Four (5) Two (IBPS Bank PO/MT CWE–V Main Exam. 31.10.2015)

10. How many meaningful English words can be formed with the letters ARILT using each letter only once in that word ? (1) None (2) One (3) Two (4) Three (5) More than three (Punjab & Sind Bank PO Exam. 23.01.2011)

Directions (11–12) : Study the following information to answer the given questions : (IBPS Bank PO/MT CWE 18.09.2011)

In a five letter English word (which may or may not be a meaningful English word), there are two letters between L and P. S is not placed immediately next to L. There is only one letter between S and A. S is towards the right of A. S is not placed immediately next to E. 11. Which of the following is correct with respect to the word thus formed? (1) E is at one of the extreme ends of the word (2) P is not placed immediately next to A (3) There are two letters between A and E in the word thus formed (4) P is placed second to the right of E (5) None is correct 12. Whick of ihe following words will be formed based on the given conditions?

ALPHABET (1) SPAEL (2) PEALS (3) LEAPS (4) SEPAL (5) LAPSE 13. How many such pairs of letters are there in the word ‘SUBSTANCE’ each of which has as many letters between them in the word (in both forward and backward directions) as in the English alphabet ? (1) None (2) One (3) Two (4) Three (5) More than three (IBPS Specialist Officer CWE 11.03.2012)

14. How many meaningful English words, not ending with ‘D’ can be made with the third, the fifth, the seventh and the ninth letters of the word ‘STEADFAST’ using each letter only once in each word? (All letters are counted from left to right) (1) None (2) One (3) Two (4) Three (5) More than three (IBPS Specialist Officer CWE 11.03.2012)

15. How many such pairs of letters are there in the word BARGAINED, each of which has as many letters between them in the word (in both forward and backward directions) as they have between them in the English alphabetical series ? (1) Two (2) Three (3) None (4) One (5) More than three (IBPS RRBs Office Assistant CWE 09.09.2012)

16. How many such pairs of letters are there in the word ADJUST ING each of which has as many letters between them in the word as in the English alphabet? (1) None (2) One (3) Two (4) Three (5) More than three (Corporation Bank SO (Marketing) Exam, 22.02.2014)

17. How many meaningful English words can be formed with the letters DERI, using all the letters but each letter only once in each word ? (1) One (2) Two (3) Three (4) More than three (5) None 18. How many meningful English words can be formed with the

letters AEHJLO, using all the letters but each letter only once in each word ? (1) One (2) Two (3) Three (4) More than three (5) None 19. If it is possible to make only one meaningful English word with the first, third, eighth and ninth letters of the word TRANSFORMS, which of the following will be the second letter of that word ? if no such word can be formed, give ‘X’ as the answer and if more than one such word can be formed, give ‘Y’ as the answer. (1) T (2) R (3) M (4) X (5) Y 20. How many such pairs of letters are there in the word SOLUTION, each of which has as many letters between them in the word (in both forward and backward directions) as they have between them in the English alphabetical series ? (1) None (2) One (3) Two (4) Three (5) More than three 21. How many such pairs of letters are there in the word ‘PAINTED’ each of which has as many letters between them in the word in both forward and backward directions, as there are between them in the English alphabetical series? (1) More than three (2) Two (3) Three (4) None (5) One (IBPS Specialist Officer (Marketing) CWE 01.02.2016)

22. How many meaningful English words can be made from the letters NOEC using each letter only once in each word ? (1) None (2) One (3) Two (4) Three (5) More than three 23. How many such pairs of letters are there in the word SCHEDULE (in both forward and backward directions), each of which has as many letters between them in the word as in the English alphabetical series ? (1) None (2) One (3) Two (4) Three (5) More than three (IBPS RRBs Officer Scale-I CWE, 06.09.2014)

BPRE–12

24. If all the vowels in the word ‘SAINTLY’ are rearranged in the alphabetical order from left to right, followed by the consonants in the alphabetical order from left to right, the position of which of the following five alphabets will remain unchanged ? (1) S (2) T (3) L (4) A (5) Y 25. It is possible to make only one meaningful English word with the third, fourth, seventh and the eighth letters of the word ‘VIDEOGRAPHY’, which would be the second letter of that word from the right end ? If more than one such word can be formed, give ‘X’ as the answer. If no such word can be formed, give Z as your answer. (1) D (2) A (3) R (4) X (5) Z 26. How many such pairs of letters are there in the word ‘MACHINERY’ each of which has as many letters between them in the word (in both the forward and backward directions), as they have between them in the English alphabetical series ? (1) Three (2) Two (3) None (4) One (5) More than three (Bank of Baroda Junior Management Grade/Scale-I Exam, 18.04.2015)

27. How many such pairs of letters are there in the word ‘SHARPLY’, each of which has as many letters between them in the word (in both forward and backward directions), as there are between them in the English alphabetical series ? (1) None (2) One (3) Two (4) More than three (5) Three (IBPS Specialist Officer (IT) CWE 14.02.2016)

28. How many such pairs of letters are there in the word ‘HONESTY’ each of which has as many letters between them in the word (in both forward and backward directions) as they have between them in the English alphabetical series?

ALPHABET (1) More than three (2) None (3) Two (4) One (5) Three (SIDBI Officer Online Exam.24.02.2016)

29. How many such pairs of letters are there in the word CRITICAL, each of which has as many letters in the word (in both forward and backward directions) as they have between them in English alphabetical series? (1) More than three (2) One (3) None (4) Three (5) Two (United Bank of India PGDBF Manipal Exam,07.08.2016)

30. How many such pairs of letters are there in the word BACTERIA each of which has as many letters between them (both in forward and backward directions) in the word as they have in the English alphabetical series ? (1) More than three (2) None (3) Two (4) Three (5) One (Bank of Baroda Exam, 25.09.2016)

31. If it is possible to make only one meaningful English word with the first, second, third and fifth letters of the word TRANSFORM’, which would be the second letter of that word from the left end? If more than one such word can be formed, give ‘X’ as your answer. If no such word can be formed, give ‘Z’ as your answer. (1) T (2) X (3) Z (4) A (5) R (Bank of Baroda Exam, 25.09.2016)

32. How many such pairs of letters are there in the word WEIGHTS each of which has as many letters between them in the word (in both forward and backward directions) as there are in the English alphabetical series ? (1) One (2) More than three (3) Two (4) Three (5) None (Bank of Maharashtra PO Exam, 26.10.2016)

33. If only one meaningful English word can be made with the first, the fourth, the ninth and the tenth letters of the word CORRUPTION, using each letter only once, then which of the following is the third letter of the word

from the left? If no such word can be formed then your answer is ‘X’. If more than one such word can be formed then your answer is ‘Z’. (1) X (2) Z (3) R (4) O (5) C

38. The position of how many alphabet will remain the same if each of the alphabet in the word NETWORK are rearranged in alphabetical order from left to right ? (1) Three (2) Two (3) None (4) One (5) More than three

(IBPS RRBs Officer CWE (Pre.) Exam, 14.11.2016 (Shift-I))

(IBPS RRBs Officers CWE (Prelim Exam) 10.09.2017)

34. How many such pairs of letters are there in the word ASTROLOGICAL, each of which has as many letters between them in the word (in both forward and backward directions) as they have in the English alphabetical series ? (1) One (2) None (3) Three (4) More than three (5) Two (IBPS SO (Agriculture) Exam, 29.01.2017)

35. The positions of how many alphabet will remain the same if each of the alphabet in the word CLIMATE are arranged in alphabetical order from left to right? (1) One (2) Two (3) Three (4) None (5) More than three (IBPS RRBs Officers CWE (Prelim Exam) 09.09.2017 Ist Sitting)

36. If it is possible to make only one meaningful English word with the second, the third, the fifth and the eighth letters of the word MNEMONICS using each letter once, when counted from left to right. Which of the following will be the second letter of the word so formed from the left end? If no such word can be formed, give X as your answer. If more than one such word can be formed, give Z as your answer. (1) M (2) N (3) E (4) X (5) Z (IBPS RRBs Officers CWE (Prelim Exam) 09.09.2017 IInd Sitting)

37. The position of how many alphabet will remain the same if each of the alphabet in the word COMPANY are rearranged in alphabetical order from left to right? (1) None (2) Two (3) Three (4) One (5) More than three (IBPS RRBs Officers CWE (Prelim Exam) 09.09.2017 IInd Sitting)

BPRE–13

39. If it is possible to make only one meaningful English word with the second, the third, the sixth and the ninth letters of the word REDUCTION using each letter only once, when counted from left to right, which of the following will be the fourth letter of the word so formed from the left end. If no such word can be formed, give X as your answer. If more than one such word can be formed, give Z as your answer. (1) E (2) T (3) X (4) D (5) Z (IBPS RRBs Officers CWE (Prelim Exam) 10.09.2017)

40. The positions of how many alphabet will remain at the same position if each of the alphabet in the word HOLIDAY are rearranged in alphabetical order from left to right? (1) More than three (2) Two (3) None (4) One (5) Three (IBPS RRBs Officer CWE (Prelim Exam) 16.09.2017)

41. If it is possible to make only one meaningful English word with the first, the second, the fourth and the eighth letters of the word STEROIDAL using each letter of the word only once, when counted from left to right, which of the following will be the fourth letter of the word so formed from the right end. If no such word can be formed give X as your answer. If more than one such word can be formed, give Z as your answer. (1) X (2) S (3) R (4) Z (5) T (IBPS RRBs Officer CWE (Prelim Exam) 16.09.2017)

42. In the word PATRONISE, the letter immediately before each vowel is replaced with the next al-

ALPHABET phabet (as per the English alphabetical order) and all others remain unchanged. Which of the following letters will appear twice in the word thus formed? (1) Only T (2) Both T and O (3) Both S and O (4) Only Q (5) Only E (IBPS SO (IT Officer) CWE (Prelim Exam) 30.12.2017)

43. If it is possible to make only one meaningful English word with the fourth, the eighth, the ninth and the eleventh fetters from the left of the word EXAMINATION using all the letters but each letter only once, which would be the second letter of the word from the left end? If more than one such word can be formed, give ‘Z’ as your answer. If no such word can be formed, give ‘Y’ as your answer. (1) Z (2) T (3) Y (4) M (5) I (IBPS SO (IT Officer) CWE (Prelim Exam) 30.12.2017)

44. How many such pairs of letters are there in the word CHAMPION each of which has as many letters between them in the word (in both forward and backward directions) as they have between them in the English alphabetical order? (1) More than Four (2) One (3) Two (4) Four (5) Three (IBPS SO (Law Officer) CWE (Prelim Exam) 31.12.2017)

45. If it is possible to make only one meaningful English word with the third, fifth, tenth and eleventh letters (when counted from left to right) of the word MAGISTERIAL (using each letter only once), which of the following will be the fourth letter of the word from the left end? If no such word can be formed give ‘X’ as your answer. If more than one such word can be formed give ‘Z’ as you answer. (1) L (2) G (3) X (4) S (5) Z (IBPS SO (Law Officer) CWE (Prelim Exam) 31.12.2017)

46. In the word SPHERICAL, if each vowel is changed to the next letter as per English alphabetical order

and each consonant is changed to the previous English alphabetical order and the word thus formed is arranged in an alphabetical order from left to right, which of the following will be fifth letter of the word from the right? (1) Q (2) O (3) G (4) K (5) J (IBPS SO (Law Officer) CWE (Prelim Exam) 31.12.2017)

47. Four of the following five are alike in a certain way based on the English alphabetical order and thus they form a group. Which is the one that does not belong to the group? (1) GAEF (2) TSWX (3) WQUV (4) PJNO (5) RLPQ (IDBI Bank PO Exam 29.04.2018)

48. The positions of how many alphabet will remain the same if each of the alphabet in the word INDULGES is rearranged in alphabetical order from left to right? (1) Two (2) None (3) Four (4) One (5) Three (IDBI Bank PO Exam 29.04.2018)

49. How many such pairs of letters are there in the word SYSTEMATIC each of which has as many letters between them in the word (in both forward and backward direction) as in the English alphabet? (1) None (2) One (3) Two (4) Three (5) More than three (IDBI Bank PO Exam 29.04.2018)

50. How Many such pairs of letters are there in the word ‘TRANSFER’, each of which has as many letters between them in the word (in both forward and backward directions) as they have between them in the English alphabet? (1) None (2) One (3) Three (4) More than three (5) Two (IBPS Bank PO/MT CWE-VII (Prelim Exam) 15.10.2017)

51. How many such pairs of letters are there in the word TELEPATHY each of which has as many letters between them in the word (in both forward and backward directions), as they have between

BPRE–14

them in the English alphabetical series ? (1) One (2) Two (3) None (4) Three (5) More than three (IBPS Bank PO/MT CWE (Main Exam) 26.11.2017)

52. How many such pairs of letters are there in the word CAUTION each of which has as many letters between them in the word (in both forward and backward direction) as they have between them in the English alphabetical order? (1) Two (2) Three (3) Four (4) One (5) More than four (IBPS RRBs Officer CWE (Prelim Exam) 11.08.2018)

53. If the second, fourth, seventh and eighth letters of the word “FRACTION” are combined to form a meaningful word, then what will be the 3rd letter from the left in the word so formed. If more than one meaningful word is formed then the answer is X, if no such word is formed then answer is Z? (1) O (2) X (3) R (4) Z (5) C (IBPS RRBs Officer CWE (Prelim Exam) 18.08.2018)

54. How many such pairs of letters are there in the word ‘CREDIT’, each of which has as many letters between them in the word (in both forward and backward directions) as they have between them in the English alphabetical series ? (1) Three (2) None (3) More than three (4) Two (5) One (Indian Bank PO Prelim Exam, 06.10.2018)

55. If it is possible to make only one meaningful English word with the first, the sixth, the ninth and the tenth letters (When counted from left to right) of the word ‘JUNCTIONAL’, using each letter only once, which would be the third letter of that word from the left end? If more than one such word can be formed, give ‘X’ as the answer. If no such word can be formed, give Z as your answer.

ALPHABET (1) Z (3) X (5) A

(2) I (4) L

(IBPS Bank PO/MT CWE (Prelim Exam) 14.10.2018)

56. How many such pairs of letters are there in the word ‘TRANSFER’, each of which has as many letters between them in the word (in both forward and backword directions) as they have between them in the English alphabet? (1) None (2) One (3) Three (4) Two (5) More than three (IBPS Bank PO/MT CWE (Prelim Exam) 21.10.2018)

57. How many such pairs of letters are there in the word WEIGHTS each of which has as many letters between them (in both forward and backward directions) as there are in the English alphabetical series? (1) One (2) More than three (3) Two (4) Three (5) None (Canara Bank PO Exam, 23.12.2018)

58. How many such pairs of letters are there in the word ‘MISCONDUCT’ each of which has as many letters between them in the word as in the English alphabet? (1) One (2) None (3) Two (4) Three (5) More than three (IBPS Specialist Officer CWE (Prelim Exam) 31.01.2019)

59. If it is possible to make only one meaningful four letter English word with the first, the fourth, the fifth and the ninth letters of the word EDUCATION using each letter only once, when counted from left to right, which of the following will be the third letter of the word so formed from the left end? If no such word can be formed give X as your answer. If more than one such word can be formed, give Z as your answer. (1) N (2) X (3) C (4) E (5) Z 60. The positions of how many alphabet will remain the same if each of the alphabet in the word CHAMBER are arranged as per the English alphabetical order from left to right ? (1) Three (2) None (3) Two (4) One (5) More than three

61. If it is possible to make only one meaningful word with the 5th, 6th, 7th and 9th letters of the word UNIVERSITY using each letter only once, which of the following will be the third letter of that word? If no such word can be made, your answer is ‘V’ and if more than one such word can be formed, then the answer is ‘Y’. (1) S (2) T (3)V (4) Y (5) R IBPS RRBs Officer CWE (Prelim) Exam, 04.08.2019

62. How many such pairs of letters are there in the word ‘EDUCATION’, each of which has as many letters between them in the word (in both forward and backward directions) as they have between them in the English alphabetical series? (1) One (2) Two (3) Three (4) Four (5) More than four IBPS RRBs Officer CWE (Prelim) Exam, 17.08.2019

63. If all the letters of the word ‘OFFICER’ are arranged in alphabetical order from left to right in such a way that vowels are arranged first followed by consonants, then how many letters are there between O and R after the arrangement (repetition of letters should also be considered)? (1) Two (2) One (3) None (4) Three (5) Four IBPS Bank PO/MT CWE (Prelim) Exam, 12.10.2019

64. If in the word COLGATE all the letters are arranged as per the English alphabetical order, then how many letters will be there in the English alphabetical order between the fourth letter from the left and second letter from the right of the word thus formed? (1) Three (2) Nine (3) Seven (4) Five (5) Four IBPS Bank PO/MT CWE (Prelim) Exam, 19.10.2019

65. How many such pairs of letters are there in the word DECOMPOSE, each of which has as many letters between them in the word (in both the forward and backward directions) as in the English alphabetical series? (1) More than three (2) None (3) Two (4) Three (5) One IBPS Bank PO/MT CWE (Prelim) Exam, 30.11.2019

66. How many such pairs of letters are there in the word ‘QUOTIENT’, each of which has as many letters between them in the word (in both forward and backward directions) as they have between them in the English alphabet? (1) None (2) One (3) Two (4) Three (5) More than three (IBPS RRBs Officer CWE Prelim Exam, 13.09.2020)

Directions (67–69) : Study the following information carefully and answer the questions given below : (IBPS RRBs Officer CWE Prelim Exam, 07.08.2021)

67. How many meaningful English words can be formed from the letter–clusters NELCAC and TEURO ? (1) One (2) Two (3) Three (4) Four (5) Five 68. How many meaningful English words can be formed from the letter -clusters RATST and PALAH ? (1) Two (2) Three (3) Four (4) Five (5) Six 69. How many meaningful English words can be formed from the letter -clusters NIOTP and ADERY ? (1) Two (2) Three (3) Four (4) Five (5) Six

Directions (70 – 74) : Study the following three Sequences 1, 2 and 3 carefully and answer the questions given below : (IBPS RRBs Officer CWE Prelim Exam, 07.08.2021)

1 SDIQSJAUHOABCOADEGEIWGBCDEFUIGOI 2 ADEGSDIQSJAUHOABCOEIWGBCDEFUIGOI 3 2 6832393584593284764572971569763859

BPRE–15

ALPHABET 70. Which of the following statements is correct regarding the Sequence –1? (1) Only eleven alphabet are at prime numbered positions (2) Only six alphabet are at perfect square numbered positions (3) Only eight alphabet are immediately followed by a Consonant (4) Only 15 alphabet are immediately preceded by a Vowel (5) Other than those given as options 71. Which of the following statements is correct regarding the Sequence – 3? (1) Only 14 even numbers are immediately preceded by an even number (2) Only 12 odd numbers are immediately followed by an odd number (3) Only six perfect square numbers are immediately followed by a prime number (4) Only seven perfect square numbers are immediately followed by an odd number (5) Other than those given as options 72. Which of the following statements is correct regarding the Sequence –2 ? (1) Only 15 vowels are immediately followed by a Consonant (2) Only five Consonants are immediately preceded by a Vowel (3) Only eight vowels are immediately followed by a Consonant (4) Only 10 Consonants are immediately followed by a Vowel (5) Other than those given as options 73. Which of the following statements is correct regarding the Sequence – 3 ? (1) Only 11 even numbers are at odd numbered positions (2) Only seven odd numbers are at even numbered positions (3) Only five even numbers are at odd numbered positions (4) Only 12 odd numbers are at even numbered positions (5) Other than those given as options

74. Which of the following statements is correct regarding the Sequence – 1? (1) Only seven Consonants are at the even numbered positions (2) Only eight vowels are at the odd numbered positions (3) Only five Consonants are at the even numbered positions (4) Only six Vowels are at the odd numbered positions (5) Other than those given as options 75. How many such pairs of letters are there in the word SIGNIFICANT, each of which has as many letters between them in the word (in both the forward and backward directions) as in the English alphabetical series ? (1) One (2) None (3) More than three (4) Two (5) Three (IBPS Bank PO/MT CWE Prelim Exam, 04.12.2021)

SBI PO EXAMS 1. If it is possible to make only one meaningful English word with the second, the fifth, the seventh and the ninth letters of the word SUBSTANTIAL, which of the following will be the second letter of that word ? If no such word can be formed, give ‘#’ as the answer and if more than one such word can be formed, give ‘@’ as the anwer. (1) U (2) T (3) N (4) # (5) @ 2. How many such pairs of letters are there in the word CHARGED, each of which has as many letters between them in the word as in the English alphabetical series (in both forward and backward directions) ? (1) None (2) One (3) Two (4) Three (5) More than three 3. Each consonant in the word BLARING is replaced with the previous letter and each vowel is replaced with next letter in the English alphabetical series and the new letters are rearranged

BPRE–16

alphabetically. Which of the following will be the third letter from the right end ? (1) K (2) B (3) M (4) J (5) F (SBI Specialist Officer (Law Officer : MMGS Scale-II) Online Exam, 19.04.2014)

4. If it is possible to make only one meaningful English word with the second, third, sixth and ninth letters of the word REGULATOR (using each letter only once in the words) which would be the second letter of the word from right ? If more than one such word can be formed give ‘Y’ as the answer. If no such word can be formed give ‘Z’ as your answer. (1) A (2) G (3) Z (4) Y (5) R (SBI Assistant Manager (System) Exam, 17.01.2016)

5. How many such pairs of letters are there in the word IMPROVISER, each of which has as many letters between them (in both and forward directions) as there are in English alphabetical series? (1) None (2) More than three (3) Three (4) One (5) Two SBI PO (Prelim Exam), 08.07.2018 (Shift-II))

6. If only one meaningful English word can be formed using the third, fourth, fifth and eighth letters of the word BARGAINED, then what will be the second letter from left of the word so formed? (1) No such word can be formed (2) R (3) A (4) E (5) More than one such words can be formed SBI PO (Prelim Exam), 08.07.2018 (Shift-II))

7. If only one meaningful English word can be formed using the first, second, sixth and seventh letters from the left of the word MARGINED (using each letter only once), then what will be the second letter from the left of the word so formed ? (1) E (2) More than one such words can be formed (3) M (4) N (5) No such word can be formed SBI PO Prelim Exam, 14.06.2019

ALPHABET 8. How many such pairs of LETTERS are there in the word NATIONALS each of which has as many letters between them in the word (in both forward and backward directions) as they have between them in the English alphabetical order ? (1) One (2) Four (3) Two (4) More than four (5) Three SBI PO Prelim Exam, 14.06.2019

9. If it is possible to make only one meaningful English word using the first, the fifth, the sixth and the eighth letters (when counted from left to right) of the word FLOATING (using each letter only one), which of the following will be the second letter of the word so formed from the right end? If no such word can be formed give X as your answer. If more than one such word can be formed give Z as your answer. (1) X (2) T (3) I (4) Z (5) F SBI PO Prelim Exam, 09.06.2019

RBI GRADE–B/ NABARD GRADE–A OFFICER EXAMS 1. The positions of the first and the fourth letters of the word PLANET are interchanged, similarly, the positions of the second and fifth letters and third and sixth letters are interchanged. In the new arrangement thus formed, how many letters are there between the letter which is second from the right and the letter which is fourth from the left according to the English alphabetical order ? (1) None (2) One (3) Two (4) Three (5) Four (RBI Grade-B Officer Exam. 06.02.2011)

2. The positions of how many alphabets will remain unchanged if each of the alphabets in the word WORTHY is arranged in alphabetical order from left to right ? (1) None (2) One (3) Two (4) Three (5) More than three (RBI Grade-B Officer Exam. 06.02.2011)

3. How many such pairs of letters are there in the word REASON, each of which has as many letters between them in the word (in both forward and backward directions) as they have between them in the English alphabetical series ? (1) None (2) One (3) Two (4) Three (5) More than three

English alphabetical series and all the consonants are changed to the previous alphabet in the English alphabetical series, how will the word be written ? (1) BADHLMPQSV (2) BAFJLMPQSV (3) BAFJLNQQSV (4) BAFJKOQRSV (5) BAFJNOPSUV

(RBI Grade-B Officer Exam. 06.02.2011)

(RBI Officer Grade ‘B’ Phase-I Exam, 04.09.2016 (Shift-II))

4. If it is possible to make only one meaningful word with the second, third, eighth and ninth letters of the word ‘CONFL-ICTED’, which would be the second letter of the word from the left? If more than one such word can be formed, give ‘A’ as the answer. If no such word can be formed, give ‘Z’ as your answer. (1) A (2) T (3) O (4) N (5) Z (RBI Grade-B Officer Exam. 06.02.2011)

5. How many such pairs of letters are there in the word PRELIMINARY, each of which has as many letters between them in the word (in both forward and backward directions) as they have in the English alphabetical series ? (1) One (2) None (3) Three (4) More than three (5) Two (RBI Officer Grade ‘B’ Phase-I Exam. 21.11.2015)

6. If all the letters in the word ‘REGULATION’ are arranged in English alphabetical order from left to right and then all the vowels are changed to the next alphabet in the English alphabetical series and all the consonants are changed to the previous alphabet in English alphabetical series, how will the word be written? (1) BFFJKOQQSV (2) ZFFJKONSSV (3) ZDHHMONSUT (4) BFHUKMPORV (5) BFFJKMPQSV (RBI Officer Grade ‘B’ Phase-I Exam, 04.09.2016 (Shift-I))

7. If all the letters in the word ‘TAMBOURINE’ are arranged in English alphabetical order from left to right and all the vowels are changed to next alphabet in the

BPRE–17

8. If it is possible to make only one meaningful English word with the second, the seventh, the eighth and the tenth letters of the word RAINFOREST (when counted from left to right) using each letter only once, which of the following will be the second letter of the word so formed from the right end? If no such word can be formed give X as your answer. If more than one such word can be formed give Z as your answer. (1) Z (2) X (3) A (4) T (5) R (RBI Assistant Manager Online Exam 25.03.2017)

9. If in the word WORKPLACE, first all the vowels are arranged in alphabetical order from left to right and then all the consonants are arranged in alphabetical order from left to right, and then each letter is changed to the next letter in the English alphabetical series, which of the following is the fifth letter from the left? (1) Q (2) M (3) L (4) F (5) P (RBI Assistant Manager Online Exam 25.03.2017)

10. How many such pairs of letters are there in the word BECOMING each of which has as many letters between them in the word (in both forward and backward directions) as they have between them in the English alphabetical series? (1) One (2) More than three (3) Three (4) None (5) Two (RBI Assistant Manager Online Exam 25.03.2017)

ALPHABET 11. How many such pairs of letters are there in the word LANGUAGE each of which has as many letters between them in the word (in both forward and backward directions) as there are in English alphabetical order? (1) Three (2) One (3) More than three (4) Two (5) None (RBI Officers in Grade ‘B’ Phase-I Exam. 16.08.2018)

12. If only one meaningful English word can be made with the fourth, the fifth, the ninth and the eleventh letters of the word NOURISHMENT (when counted from left to right), using each letter only once, then which of the following will be the second letter of the word from the left end? If no such word can be formed then your answer is ‘X’. If more than one such word can be formed then your answer is ‘Z’. (1) X (2) E (3) Z (4) T (5) R (RBI Officers in Grade ‘B’ Phase-I Exam. 16.08.2018)

13. How many such pairs of letters are there in the word GUARDIAN each of which has as many letters between them in the word (in both forward and backward directions) as they have between them in the English alphabetical order? (1) More than three (2) Three (3) None (4) One (5) Two 14. If it is possible to make only one meaningful four letter English word using the third, the fourth, the seventh and and eleventh letters (when counted from left to right) of the word TRADITIONAL (using each letter only once), which of the following will be the third letter of the word so formed from the left end? If no such word can be formed, mark X as your answer. If more than one such word can be formed mark Z as your answer. (1) A (2) I (3) Z (4) L (5) X RBI Grade ‘B’ Officer Exam, 09.11.2019

Directions (15 – 19) : Study the following two Sequences carefully and answer the questions given below : (RBI Officers Grade ‘B’ Phase-I Exam, 06.03.2021)

1 45729715697638592683239358459328476 2 OABCOEIWGBCDEFUIGOIADEGSDIQSJAUH 15. In the Sequence-1, how many such odd numbers are at the odd numbered positions, each of which is immediately followed by a perfect square number ? (1) None (2) Two (3) One (4) Three (5) More than three 16. In the Sequence-2, how many such Vowels are there, each of which is immediately preceded by a Vowel and also immediately followed by a Consonant ? (1) Seven (2) Three (3) Four (4) Two (5) Five 17. In the Sequence-1, how many such even numbers are at odd numbered positions, each of which is immediately followed by an odd number ? (1) Five (2) Two (3) Four (4) More than five (5) Three 18. In the Sequence-2, if it is possible to make a meaningful English word with the second, the eighth, the eighteenth, the twenty-fifth, the twenty eighth, and the thirty second letters, mark the last letter of that word as your answer. If more than one such word can be formed, mark ‘X’ as the answer and if no such word can be formed, mark ‘Y’ as the answer. (1) W (2) H (3) O (4) X (5) Y 19. In the Sequence-2, how many Consonants at even numbered positions are immediately followed by a Vowel ? (1) One (2) Three (3) Two (4) Four (5) Five Directions (20–24) : In each of the following questions, how many meaningful English words can be formed with the letters of the two letter-clusters separately and using each letter only once in each word ? (RBI Officers Grade ‘B’ Phase-I Exam. 06.03.2021)

BPRE–18

20. MIREP ; SERUL (1) None (2) (3) Two (4) (5) Four 21. SECAE ; WOBEL (1) Four (2) (3) One (4) (5) Five 22. VETNE ; SINEM (1) Two (2) (3) Three (4) (5) One 23. LACEP ; SAFET (1) Four (2) (3) Six (4) (5) Three 24. LEMAS ; COHUT (1) Three (2) (3) Five (4) (5) Six

One Three

Three Two

Four None

Five Two

Four Two

INSURANCE EXAMS 1. How many meaningful words can be formed by using any two letters of the word NOT ? (1) One (2) Two (3) Three (4) Four (General Insurance Corporation Exam. 11.12.2011) 2. How many meaningful English words can be formed with the letters ENCO, using each letter only once in each word ? (1) None (2) One (3) Two (4) Three (5) More than three (United India Insurance AO Exam. 26.05.2013)

3. How many such pairs of letters are there in the word COAST (in both the forward and backward direction), each of which has as many letters between them in the word as they have between them in the English alphabetical series ? (1) None (2) One (3) Two (4) Three (5) More than three (United India Insurance AO Exam. 26.05.2013)

ALPHABET 4. How many meaningful English words can be formed from the first, the fifth, the eighth and the tenth letters of the word CREATIVITY, using each letter only once in each word ? (1) None (2) One (3) Two (4) Three (5) More than three (United India Insurance AO Exam. 26.05.2013)

5. Only one meaningful word can be formed by rearranging the letters of the given jumbled word. Find out that word. (1) MUTAMOSPHPC (2) PHIKANTHCOPIST (3) HEPPOCANLUS (4) UPPHPOTAOMIS (NICL (GIC) AO (Finance) Exam, 08.09.2013)

6. If all the vowels in the word ‘GUIDANCE’ are rearranged in the alphabetical order from left to right, followed by the consonants in the alphabetical order from left to right, the position of which of the following five alphabet given below will remain unchanged? (1) N (2) I (3) G (4) D (5) A (OICL Specialist Officer (Finance) Exam. 03.05.2015)

7. It is possible to make only one meaningful word with the sixth, eighth, ninth and tenth letters of the word ‘DOCUMENTARY’, which would be the third letter of that word from left end? If more than one such words can be formed, give ‘X’ as the answer. If no such words can be formed, give Z as your answer. (1) X (2) R (3) Z (4) A (5) T (OICL Specialist Officer (Finance) Exam. 03.05.2015)

8. How many such pairs of letters are there in the word ‘PLAT FORMS’ each of which as many letters between them in the word (in both forward and backward directions), as they have between them in the English alphabetical series? (1) Three (2) None (3) One (4) Two (5) More than three (OICL Specialist Officer (Finance) Exam. 03.05.2015)

SHORT ANSWERS

S A

1. (5)

NATIONALISED BANKS & IBPS PO/MT/SO EXAMS

A I N T L I L N S T

Y Y

2. (4)

1 2 3 4 5 6 7 8 9 10 11

1. (5) 5. (1) 9. (4) 13. (4) 17. (2) 21. (3) 25. (4) 29. (5) 33. (3) 37. (2) 41. (4) 45. (5) 49. (3)

2. (4) 6. (3) 10. (3) 14. (2) 18. (5) 22. (3) 26. (1) 30. (4) 34. (4) 38. (3) 42. (2) 46. (5) 50. (3)

3. (1) 7. (1) 11. (4) 15. (5) 19. (5) 23. (4) 27. (3) 31. (2) 35. (4) 39. (5) 43. (5) 47. (2) 51. (2)

4. (3) 8. (2) 12. (3) 16. (4) 20. (4) 24. (1) 28. (5) 32. (2) 36. (5) 40. (2) 44. (5) 48. (4) 52. (1)

V I D E O G R A P H Y

53. (3) 57. (2) 61. (4) 65. (1) 69. (2) 73. (4)

54. (4) 58. (4) 62. (5) 66. (4) 70. (1) 74. (2)

55. (2) 59. (5) 63. (4) 67. (3) 71. (4) 75. (3)

56. (3) 60. (4) 64. (3) 68. (1) 72. (3)

12 5 14 7 20 8 L E N G T H

Specified letters ⇒ D, E, R, A Meaningful words ⇒ DARE, DEAR, READ 3. (1)

2. (4) 6. (5)

8 9 14 5 18 25 H I N E R Y

4. (3)

5. (1)

R

E

M

I

A

–1 +1 –1 +1 +1 Q

SBI PO EXAMS 1. (3) 5. (3) 9. (5)

3 C

13 1 M A

3. (1) 7. (2)

4. (4) 8. (2)

RBI GRADE–B/NABARD GRADE–A OFFICER EXAMS

F

L

B Q

N

S

–1 –1

J

M

R

R

17 18 No letter 6. (3) 1 2 3 4 5 6 E Q U A L I According to question, L

I

T

Y

E

Q

U

7 T

8 Y

A

1. (4) 5. (5) 9. (3)

2. (5) 6. (5) 10. (5)

3. (2) 7. (2) 11. (4)

4. (1) 8. (1) 12. (3)

13. (2)

14. (3)

15. (3)

16. (5)

7. (1)

17. (4)

18. (1)

19. (2)

20. (4)

21. (1)

22. (3)

23. (2)

24. (5)

18 5 6 18 5 19 8 9 14 7 R E F R E S H I N G 8. (2)

3rd from the right end

INSURANCE EXAMS 1. (3) 5. (4)

2. (3) 6. (2)

3. (3) 7. (1)

4. (2) 8. (4)

B

−1

+1

B

−1

B

+1

B B

K

P

R

J

B

E

T

A

L

O

S

I +1

−1

B

+1

S

B F

Alphabetical order of letters : B

F

J

K

P

R

S

EXPLANATIONS NATIONALISED BANKS & IBPS PO/MT/SO EXAMS BPRE–19

3rd from the right end 9. (4) Meaningful words ⇒ TIME, EMIT, ITEM, MITE

ALPHABET 10. (3) Meaningful words ⇒ TRAIL, TRIAL (11–12) : L P L PS L APS LEAPS 11. (4) P is placed second to the right of E. 12. (3) The word is LEAPS. 13. (4)

19 21 2 19 20 1 14 3 S U B

S T

A N

22. (3) Meaningful words ⇒ CONE, ONCE 23. (4)

19 S

3 C

5 E

S ¯ A

24. (1)

5

C E

8 H

4 D

32. (2)

23 W 33. (3)

A I N T L ¯ ¯ ¯ ¯ ¯ I L N S T

Y ¯ Y

Specified letters ⇒ C, R, O, N Meaningful Word ⇒ CORN Its third letter from the left ⇒ R 34. (4)

Five letter from the left

2 B

1 18 7 A R G

1 9 14 5 A I N E

4 D

3 C

8 9 14 5 18 25 H I N E R Y

4

D

10 21 19 20 9

J

U

S

T

I

14

7

N

G

17. (2) Meaningful words ⇒ RIDE, DIRE 18. (5) No meaningful English word can be formed with the letters AEHJLO. 19. (5)

1 2 3 4 5 6

7 8

9 10

T R A N S F O R

M S

19 S

8 H

1 A

18 R

16 P

12 L

25 Y

8 H

15 O

14 N

5 E

19 20 25 S T Y

21 20 U T

9 15 14 I O N

3 18 9 C R I

20 T

9 I

3 C

1 A

12 L

1 A

9 14 20 I N T

5 E

4 D

20 T

35. (4)

18 R

15 O

12 15 L O

7 G

9 I

3 C

C L I M A A C E I L

1 A

12 L

T E M T

2 B

1 A

31. (2)

1 2 M N

3 4 5 6 7 8 E M O N I C

9 S

37. (2)

C O M P A N Y A C M N O P Y

N E T W O R E K N O R T 39. (5)

H O L 40. (2)

3 C

20 T

5 E

18 R

9 I

1

2

T

R

1 A

3 4 5 6 7 8 9 A N S F OR M

Specified letters ⇒ T, R, A, S Meaningful words ⇒ AR TS, TSAR, TARS, RATS, STAR

BPRE–20

K W

DENT

21. (3)

16 P

19 S

1 2 3 4 5 6 7 8 9 R E D U C T I O N Specified letters ⇒ E, D, T and N Meaningful words ⇒ TEND,

29. (5)

30. (4)

15 12 O L

1 A

38. (3)

28. (5)

Specified letters ⇒ T, A, R, M Meaningful Words ⇒ MAR T, TRAM 20. (4)

19 S

10 N

Specified letters ⇒ N, E, O, C Meaningful Words ⇒ CONE, ONCE

27. (3) 1

2 3 4 5678 9 C OR R UP T I O

36. (5)

13 1 M A

16. (4)

A

1

25. (4)

Specified letters ⇒ D, E, R, A Meaningful words ⇒ DARE, DEAR, READ 26. (1)

Meaningful Word ⇒ DATE 15. (5)

7 8 20 19 G H T S

5 E

V I D E O G R A P H Y

S T E A D F A S T

9 I

21 12 U L

1 2 3 4 5 6 7 8 9 10 11 14. (2) 1 2 3 4 5 6 7 8 9

5 E

A D H I L 1

41. (4)

I D A

S

Y

O Y

2 3 4 5 6 7 8 9 T E R O I D A L

Specified letters ⇒ S, T, R, A Meaningful words ⇒ STAR, ARTS, RAST, RATS, TARS, TSAR 42. (2) P A T R O N I S E

↓ ↓ ↓ ↓ ↓ ↓ ↓ ↓ ↓ Q A T S O O I T E Both O and T appear twice.

ALPHABET 43. (5)

60. (4)

52. (1)

1 2 3 4 56 7 8 EX A M I N A T

9 10 11 I O N

Specified letters ⇒ M, T, I and N Meaningful Word ⇒ MINT Its second letter from the left end ⇒I 44. (5)

3 C

8 H

3 C

1 A

21 U

20 T

9 I

15 O

53. (3)

2

1

F R

4 5 6 7 C A T I O



P

H

E

R

I

C

A

L

3rd letter

G

F

Q

J

B

B

K

B B F G J K O Q R 5th letter from the right

M

B

E

R

A

C

E

H M

R

B

from the left

9

10

T

Y

G –6 W –6 P R

–6 –6

A +4 E +1 F Q +4 U +1 V +4 +1 O N J +4 +1 Q P L

3 18 5 4 9 20 C R E D I T

5 E

1 2 3 4 5 6 7 8 9 10 J U N C T I O N A L

4 21 3 D U C

63. (4)

Specified Letters ⇒ J, I, A L Meaningful Word ⇒ JAIL Its third letter from the left ⇒ I 56. (3)

20 18 T

R

1

14 19

6

5

18

A

N

F

E

R

S

57. (2)

S + 4 W +1 X

23 5 9 7 8 W E I G H

48. (4)

I N D U L G E S D E G I L N S U

7

8

R

S

I

1 20 9 15 14 A T I O N

55. (2)

O F F I C E R E I O C F F R

There are three letters – C, F and F – between O and R.

C O L G A T E A C E G L O T

4th letter from left ⇒ G 2nd letter from right ⇒ O

GHIJKLMNO 7 Letters 65. (1)

But,

T –1

6

E

Specified letters ⇒ E, R, S, T Meaningful Words ⇒ ERST, RETS, REST, TRES 62. (5)

64. (3)

47. (2)

5

54. (4)

–1 –1 –1 +1 –1 +1 –1 +1 –1 O

A

N

Meaningful word ⇒ CORN

Specified letters ⇒ G, S, A, L Meaningful Words ⇒ GALS, LAGS, SLAG 46. (5)

C H

61. (4) 1 2 3 4 U N I V

Specified letters ⇒ R, C, O, N

1 13 16 9 15 14 A M P I O N

1 2 3 4 5 6 7 8 9 10 11 M A G I S T E R I A L

R

8

3

45. (5)

S

14 N

20 19 T S

5

3

15

13

D E

C

O

M

4

58. (4)

16 15 19

P

O

5

S E

49. (3)

19 25 19 20 5 13 1 20 9 3 S Y S T E M A T I C 20 18 1 14 19 6 5 18 50. (3) T R A N S F E R 51. (2)

20 T

5 E

12 L

5 E

16 P

1 A

20 T

8 H

25 Y

13 9 19 3 15 14 4 21 3 20 M I S C O N D U C T

17 21 15 20 9 Q U O T I

59. (5)

1

2

66. (4)

3

4

5

6

7

8

9

E D U C A

T

I

O N

Specified letters ⇒ E, C, A and N Meaningful words ⇒ ACNE, ANCE, CANE

BPRE–21

5 14 20 E N T

67. (3) NELCAC ⇒ CANCEL TEURO ⇒ OUTER ; ROUTE 68. (1) RATST ⇒ START PALAH ⇒ ALPHA 69. (2) NIOTP ⇒ PITON; POINT ADERY ⇒ READY

ALPHABET 2. (4)

70. (1)

1 2 3 4 5 6 7 8 9 10 11 12 13 14 15 16 S D I Q S J A U H O A B C O A D

72

93 28 97

47 6 97

156

SD

AB

O

EF

GBCD

A DE

45

FU

six even numbers are immediately preceded by an even number. ● Only 10 odd numbers are immediately followed by an odd numbers. ● Only seven perfect square numbers are immediately followed by a Prime Number.

GO

A

B

7

2

9

7

1

U

I

● Only eight vowels are immediate-

diately followed by a vowel. 73. (4)

numbered positions. 74. (2) Only nine Consonants are at the even numbered positions. ● Only eight vowels are at the odd numbered positions. 75. (3)

7

14 9

6

9

3

1

14 20

S

G

N

F

I

C

A

N

I

I

M

M

Q

F

F

J

K

1

2

3

4 5

6

7 8

9

R

E

G U L

A

T O

R

Specified Letters ⇒ E, G, A, R Meaningful Words ⇒ GEAR, RAGE 5. (3)

9 13 16 18 15 22 9 19 5 18 I M P R O V I S E R

1 2 3 4 5 6 7 8 9 B A R G A I N E D

6

9

7

6

Specified letters ⇒ R, G, A, E Meaningful Words ⇒ GEAR; RAGE, AGER, GARE

3

8

5

9

SBI PO EXAMS 1. (3)

7. (2) 1 2 3 4 5 6 7 8 M A R G I N E D Specified letters ⇒ M, A, N, E Meaningful words ⇒ MANE; MEAN; NAME; AMEN; 8. (2) 14

1

20

9

15 14

1

N

A

T

I

O

A

N

12 19 L

S

1 2 3 4 5 6 7 8 9 10 11 S U B S T A N T I A L Specified letters

19 9

J

6. (5)

● Only 10 even numbers at the odd

numbered positions.

Q

ly followed by a consonant. ● Only eight consonants are imme-

5

● Only 12 odd numbers at the even

B

3rd from right

Vowel Consonant

5

K

4. (4)

19 20 21 22 23 24 25 26 27 28 29 30 31 32 33 34 35 4

4 D

OI

1 2 3 4 5 6 7 8 9 10 11 12 13 14 15 16 17 18 2 6 8 3 2 3 9 3 5 8 4 5 9 3 2 8 4 7

6

5 E

A

IW

GS DI QS JA

I

7 G

Now,

HO AB CO EIWGBC DE

63859

18 R

IQ SJA

COE

U7 IG

1 A

B L A R I N G –1 –1 +1 –1 +1 –1 –1

Consonant Vowel

● Only

72. (3)

EG

UH

93 58

71. (4) 2 6 8 3 2 3

45

AD

8 H

3. (1)

17 18 19 20 21 22 23 24 25 26 27 28 29 30 31 32 E G E I W G B C D E F U I G O I Prime Numbers : 2, 3, 5, 7, 11, 13, 17, 19, 23, 29, 31 Perfect Square Numbers : 1, 4, 9, 16, 25 ● 16 alphabet are immediately followed by a Consonant. ● 14 alphabet are immediately preceded by a Vowel.

3 C

⇒ U, T, N and I

Meaningful word

⇒UNIT

9. (5)

1 234 5 F LOA T

6 7 8 I N G

Specified letters ⇒ F, T, I and G Meaningful word ⇒ GIFT Its second letter from the right ⇒F

T

BPRE–22

ALPHABET 10. (5)

RBI GRADE–B/NABARD GRADE–A OFFICER EXAMS 1. (4) P N

L A N E T P

E L

2 B

5 E

3 C

15 O

13 M

9 I

14 N

7 G

14 N

7 G

21 U

1 A

7 G

5 E

11. (4)

T A

L M N O P

12 L

2. (5) W O R T H Y H O R T W Y

1 A

12. (3)

5 6 7 8 9 10 11 I SHM E N T

1 2 3 4 N OU R

3. (2) 18 5 1 19 15 14 R E A S O N 4. (1) 1 2 3 4 5 6 7 8 9 10 CO N F L I C T E D Meaningful Words ⇒ NOTE, TONE 5. (5)

16 18 5 12 9 13 9 14 1 18 25 P R E L I M I N A R Y

Specified letters ⇒ R, I, E, T Meaningful words ⇒ RITE, TIER, TIRE 13. (2) 7 G

21 U

1 A

18 R

4 D

5 I

4 D

9 I

1 A

14 N

20. (4) MIREP ⇒ PRIME SERUL ⇒ LURES; RULES 21. (1) SECAE ⇒ CEASE WOBEL ⇒ BELOW; BOWEL; ELBOW 22. (3) VETNE ⇒ EVENT SINEM ⇒ MINES; MIENS 23. (2) LACEP ⇒ PLACE SAFET ⇒ FATES; FEAST; FEATS; FESTA 24. (5) LEMAS ⇒ MALES; MEALS; LAMES; ALMES COHUT ⇒ COUTH; TOUCH

INSURANCE EXAMS 1. (3) Meaningful words : NO, ON, TO. Hence, three meanigful words can be formed by using any two letters of the formed word NOT. 2. (3) Meaningful words ⇒ CONE, ONCE

14. (3) 6. (5) R E G U L A T I O N English alphabetical order : A

E

+1 +1

B

G –1

F

I

L

+1 –1

F

J

N –1

K

O +1

M

R –1

P

T –1

Q

U +1

S

V

7. (2)

T A M B O U R I N E M N O R T U

A B E I

+1 –1 +1 +1 –1 –1 +1 –1 –1 +1 B A F J

L M P Q S V

8. (1)

1 R

2 A

3 4 5 6 I N F O

7 R

8

9 E S

1 T

2 R

3 A

7 8 I O

9 10 11 N A L

Specified letters ⇒ A, D, I and L Meaningful words ⇒ DIAL, LAID 15. (3)

OAB

CO

UIG

O

EIW IAD

GBCDEF EGSDIQSJ

↓ 5th letter from the left

15 O

1 A

19 S

20 T

4. (2)

6. (2)

G U

I

D A

A E

I

U C D G N

N C E

AUH 1 4 5

9 6 9

729715 19

Specified Letters ⇒ A, R, E, T Meaningful words ⇒ RATE, TEAR, TERA 9. (3) W O R K P L A C E According to question, A E O C K L P R W Next letter in the English alphabetical series B F P D L M Q S X

3

3. (3) C

1 2 3 4 5 6 7 8 9 10 C R E A T I V I T Y 15 45729715697638 5 9 2683239358459328476 Meaningful Word ⇒ CITY 16. (5) 5. (4) Meaningful Word Vowel Vowel Consonant ⇒ HIPPOPOTAMUS

17. (4)

10 T

6 T

76385926 8 3 27

21 2 3 33

9358 4 5 9328 4 7 6 18. (1) Specified letters ⇒ A, W, O, D, S, H Meaningful word ⇒ SHADOW 19. (2) 14 4 12 OAB C O EIWGBC D E F U

IGOIADEGSDIQSJAUH

BPRE–23

7. (1) 1 2 3 4 5 D O C U M

6 E

7 N

8 T

9 A

10 R

11 Y

Specified letters ⇒ E, T, A, R Meaningful words ⇒ RATE, TEAR 8. (4)

16 12 1 20 6 15 18 13 19 P L A T F O R M S

❐❐❐

ALPHABET

MODEL EXERCISES 1. How many meaningful words can be formed with the letters EAML? (1) 1

(2) 3

(3) 4

(4) 2

(5) None of these 2. If the second, third, fifth, eight and ninth letters of the word CONTEMPLATION are combined to form a meaningful word, what will be the middle letter of that word? If more than one such words can be formed, your answer is X and if no such word can be formed, your answer is Y. (1) X

(2) O

(3) A

(4) Y

(5) None of these 3. If only the consonants in the word MEAT are changed in such a way that each of them becomes the next letter in the English alphabet and the remaining letters are kept unchanged, then how many meaningful words can be formed with the new set of letters using each letter only once in a word? (1) None

(2) Two

(3) Three

(4) One

(5) None of these 4. Which letter should be the ninth letter to the left of the ninth letter from the right, if the first half of the alphabet of English are reversed ? (1) D

(2) F

(3) E

(4) I

(5) None of these 5. If the second, third, fifth, eighth and ninth letters of the word CONTEMPLATION are combined to from a meaningful word, what will be the middle letter of the word? If more than one such word can be formed, your answer is X and if no such words can be formed your answer is Y. (1) A

(2) O

(3) X

(4) Y

(5) None of these

6. How many such pairs of letters are there in the word INSTRUCTION which have as many letters between them in the word as, in the English alphabet? (1) Two

(2) One

(3) Three

(4) Four

(5) None of these 7. What is the number of the different letters which occur in the word PANCHATANTRA? (1) 7

(2) 6

(3) 9

(4) 12

(5) None of these 8. How many such pairs of letters are there in the word ‘HONESTY’ each of which has as many letters between them in the word (in both forward and backward directions) as they have between them in the English alphabetical series? (1) More than three (2) None

(3) Two

(4) One

(5) Three

Directions (9 – 13): Study the following three series (ROWs) carefully and answer the questions given below : (1) A D E G S D I Q S J A U H O A BCOEIWGBCDEFUIG OI (2) 2 6 8 3 2 3 9 3 5 8 4 5 9 3 2 8 4764572971569763 859 (3) S D I Q S J A U H O A B C O A DEGEIWGBCDEFUIG OI 9. With respect to Row – 3, which of the following statement is CORRECT? (1) There are 7 alphabet terms occupy even number positions (2) There are 8 alphabet terms occupy odd number positions

BPRE–24

(3) There are 5 alphabets terms occupy even number positions (4) There are 6 alphabet terms occupy odd number positions (5) None of these 10. With respect to Row – 2, which of the following statements is CORRECT? (1) There are 10 even numbers at odd number positions (2) There are 7 odd numbers at even number place value (3) There are 5 even numbers at odd number positions (4) There are 12 odd numbers at even number positions (5) None of these 11. With respect to Row – 1, which of the following statement is CORRECT? (1) There are 15 vowels immediately a consonant (2) There are 5 consonants immediately preceded by a vowel (3) There are 8 vowels immediately followed by a consonant (4) There are 10 consonants immediately followed by a vowel (5) None of these 12. With respect to Row – 2, which of the following statement is CORRECT? (1) There are 14 even numbers immediately preceded by even numbers (2) There are 12 odd numbers immediately followed by an odd number (3) There are 6 perfect square numbers immediately followed by a prime number (4) There are 7 perfect square number immediately followed by an odd number (5) None of these

ALPHABET 13. With respect to Row – 3, which of the following statement is CORRECT?

10. (4)

SHORT ANSWERS

(1) There are 7 alphabets in prime number positions

1.(2)

2.(2)

3.(1)

4.(3)

5.(2)

6.(4)

7.(1)

8.(5)

(2) There are 10 alphabets in perfect square number positions

9.(2)

10.(4)

11.(3)

12.(4)

13.(1)

14.(4)

15.(3)

16.(3)

(3) There are 8 alphabets immediately followed by a consonant (4) There are 15 alphabets immediately preceded by a vowel

EXPLANATIONS

(1) Two

(2) One

(3) None

(4) Three

(1)

ADEGSDIQSJAUHOA BCOEIWGBCDEFUIG OI 12. (4) 2683239358459328 4764572971569763 859

Lame and male can be formed. 2. (2) The meaningful word that can be formed is Alone. The middle

13. (1) SDIQSJAUHOABCOA DEGEIWGBCDEFUIG OI

letter is O. 3. (1) Consonants in the word ‘Meat’ are M and T. Next alphabet of M is N and for T it is U. The new

14. (4)

word is NEAU. 4. (3) M L K J I H G F E D C B A N OPQRSTUVWXYZ

22 15 3 1

2 21 12 1 18 25

V

B U

O C A

L

A R

Y

5. (2) Here, the specified letters are O, N, E, L and A. The meaningful

(5) More than three 15. How many such pairs of letters are there in the word FEATURE, each of which has as many letters between them in the word (in both forward and backward directions) as they have between them in the English alphabetical series ?

11. (3)

1. (2) Three meaningful word Meal,

(5) None of these 14. How many such pairs of letters are there in the word VOCABULARY each of which has as many letters between them in the word (in both forward and backward directions) as they have between them in the English alphabetical series?

2683239358459328 4764572971569763 859

word formed with these letters is ALONE. Hence, the required middle letter of the word is ‘O’.

15. (3)

6 F

5 E

1 20 21 18 5 A T U R E

6 F

5 E

1 20 21 18 A T U R

6. (4) According to question,

16. (3)

Two

5 E

(2) One So, there are four such pairs.

(3) More than three (4) Three

7. (1) In the word PANCHATANTRA,

16. How many such pairs of letters are there in the word FEATURE, each of which has as many letters between them in the word (in both the forward and backward directions) as they have between them in the English alphabetical series ?

❑❑❑

different letters are A, C, H, N, P, R and T. So, total 7 letters.

(5) None

CLEAR YOUR DOUBTS

8. (5)

8 H

15 O

14 N

5 E

19 20 25 S T Y

(1) None (2) Three (3) More than three (4) One (5) Two

9. (2) SDIQSJAUHOABCOA DEGEIWGBCDEFUIG

[email protected]

OI

BPRE–25

SERIES

2

SERIES

QUESTIONS FROM 1999 TO 2010 ARE AVAILABLE ONLINE NATIONALISED BANKS & IBPS PO/MT/SO EXAMS 1. What will come in the place of the question mark? G D B A ? (1) E (2) F (3) C (4) H (5) A (Bank Of Baroda PO Exam. 13.03.2011)

2. Which of the following will come next in the following series ? 91982987398764987 6559876546987654 (1) 7 (2) 8 (3) 3 (4) 2 (5) 4 (IBPS RRBs Office Assistant CWE 09.09.2012)

3. Which of the following will come in the place of the question mark (?) in the following series based on the English alphabetical order ? ZA BY XC DW ? (1) VF (2) EU (3) UE (4) EV (5) VE (IBPS RRBs Office Assistant CWE 09.09.2012)

4. What should come next in the following number series ? 45321545321445321 345321? (1) 1 (2) 4 (3) 2 (4) 5 (5) None of these (IDBI Bank Officer Exam. 16.09.2012)

5. Which of the following groups of alphabets should replace the blank spaces so that the group of alphabets, given in bold, follow a logical pattern from the preceding and the following group of alphabets ? bw — yza dstuv — — opqre (1) x, e, d (2) x, c, f (3) v, e, f (4) x, c, d (5) x, e, n (Corporation Bank SO (Marketing) Exam, 22.02.2014)

6. What should come next in the following letter series ?

AN CQ (1) JZ (3) IY (5) LZ

ET

GW ? (2) IZ (4) KA

(IBPS RRBs Officer Scale-I CWE, 06.09.2014)

7. Each odd digit in the number 42793681 is substituted by the previous lower digit and each even digit is substituted by the next higher digit and the digits so obtained are rearranged in ascending order, which of the following will be the third digit from the left end after the rearrangement ? (1) 5 (2) 6 (3) 3 (4) 8 (5) 7 (BOB Manipal School of Banking Officer Online Exam, 14.08.2014)

11.

The following series is based on the English alphabet. Which of the following will come in the place of the question mark (?) in the given series ? BDZ CFY DHX EJW ? (1) GKV (2) FLU (3) FMW (4) FLV (5) GLU (Bank of Maharashtra PO Exam, 26.10.2016)

12. How many such pairs of digits are there in the number 67594138 (both in forward and backward directions), each of which has as many digits between them in the number as in the arithmetical series? (1) Two (2) None (3) One (4) Three (5) More than three (IBPS RRBs Officer CWE (Pre.) Exam, 14.11.2016 (Shift-I))

8. The positions of how many digits will remain the same if the digits in the number 69852731 are rearranged in the ascending order within the number from left to right? (1) One (2) More than three (3) Three (4) Two (4) None

13. The following series is based on the English alphabet. Which one of the following will come in place of the question mark (?) in the given alphabetical series? XBF UDK RFO OHR ? (1) LKU (2) MKS (3) LJT (4) MJS (5) LJV

(SIDBI Officer Online Exam.24.02.2016)

14. How many such pairs of digits are there in the number 31748296 (both in forward and backward directions), each of which has as many digits between them as in the arithmetical series? (1) Three (2) More than three (3) One (4) None (5) Two

9. If all the digits of number 643915827 are rearranged in ascending order from left to right, the position of how many digits will remain unchanged? (1) More than three (2) One (3) None (4) Three (5) Two (United Bank of India PGDBF Manipal Exam,07.08.2016)

10. What should come in the place of question–mark (?) in the following English alphabetical series? A E G K M Q S ? (1) W (2) U (3) X (4) V (5) Z (Bank of Maharashtra PO Exam, 26.10.2016)

BPRE–26

(IBPS RRBs Officer CWE (Pre.) Exam, 14.11.2016 (Shift-I))

(IBPS RRBs Officer CWE (Pre.) Exam, 14.11.2016 (Shift-II))

15. How many such pairs of digits are there in the number 71864392 (both in forward and backward directions), each of which has as many digits between them in the number as in the arithmetic series?

SERIES (1) More than three (2) None (3) Three (4) One (5) Two (IBPS RRBs Officer CWE (Pre.) Exam, 14.11.2016 (Shift-III))

16. What will come next in the place of question-mark (?) in the following series based on the English alphabetical order? AZ CX FU JQ ? (1) PL (2) OM (3) LO (4) PM (5) OL (IBPS RRBs Officers CWE (Prelim Exam) 09.09.2017 IInd Sitting)

17. What will come in the place of (?) in the following series based on the English alphabetical order ? AN CP ER GT ? (1) IV (2) JU (3) HW (4) HV (5) IX (IBPS RRBs Officers CWE (Prelim Exam) 10.09.2017)

18. What will come in place of question mark (?) in the following series based on the English alphabetical order? AG–4, CI–8, EK–12, GM–16, ? (1) IO–20 (2) JM–24 (3) KO–20 (4) IO–24 (5) IN–22 (IBPS RRBs Officer CWE (Prelim Exam) 16.09.2017)

19. What will come in place of the question mark (?) in the given series? AZ-4 BY-6 DW-10 GT 16 (?) (1) KP – 20 (2) JO – 20 (3) KN – 24 (4) KP – 24 (5) JO – 22 (IBPS SO (IT Officer) CWE (Prelim Exam) 30.12.2017)

20. What will come in place of (?) in the following series based on English alphabetical order and numbers? YC6 XD 8 VF11 SI 15 ? JR26 (1) ON 21 (2) PN 19 (3) PO 19 (4) OM 20 (5) PM 20 (IBPS SO (Law Officer) CWE (Prelim Exam) 31.12.2017)

21. What should come in place of question mark (?) in the following series based on the English alphabet? BED EIG HMJ KQM ? (1) PUN (2) OUQ (3) NUQ (4) NUP (5) NUR (IBPS Bank PO/MT CWE-VII (Prelim Exam) 15.10.2017)

22. How many pair of digits have same number of digits between them in the number “573814269” as in the numeric series? (1) Five (2) Four (3) Six (4) Three (5) More than six (IBPS RRBs Officer CWE (Prelim Exam) 18.08.2018)

23. What should come in place of question mark (?) in the following series based on the English alphabet? BED EIG HMJ KQM ? (1) PUN (2) OUQ (3) NUQ (4) NUP (5) NUR (IBPS Bank PO/MT CWE (Prelim Exam) 21.10.2018)

24. The following series is based on the English alphabet. Which of the following will come in the place of the question mark (?) in the given series? BDZ CFY DHX EJW ? (1) GKV (2) FLU (3) FMW (4) FLV (5) GLU (Canara Bank PO Exam, 23.12.2018)

25. In a number 86143259 if all digits are arranged in ascending order from left to right within the number then how many digits will remain on the same position? (1) One (2) None (3) Three (4) Two (5) Four (IBPS Specialist Officer CWE (Prelim Exam) 31.01.2019)

26. What will come next in place of (?) in the following series based on the English alphabetical order? ZBY XDW UGT QKP ? (1) LPK (2) MOL (3) LOK (4) MPL (5) LKK IBPS RRBs Officer CWE (Prelim) Exam, 04.08.2019

27. How many such pairs of digits are there in the number 2485163 each of which has as many digits between them in the number (in both forward and backward directions) as they have between them in arithmetic numerals ? (1) None (2) More than three (3) Three (4) Two (5) One IBPS RRBs Officer CWE (Prelim) Exam, 04.08.2019

28. If in the number 918755389401, positions of the first and the sec-

BPRE–27

ond digits are interchanged, positions of the third and fourth digits are interchanged and so on till the positions of the 11th and 12th digits are interchanged, which digit will be 7th from the right end? (1) 1 (2) 0 (3) 9 (4) 3 (5) 5 IBPS Bank PO/MT CWE (Prelim) Exam, 12.10.2019

29. If ‘1’ is subtracted from the first, third, fifth and seventh digits and ‘2’ is added to the second, fourth, sixth and eight digits (when counted from left to right) of the number ‘37824156’, how many odd digits would be in the new number ? (1) More than three (2) Three (3) Two (4) One (5) None (IBPS SO Prelim Exam, 28.12.2019)

30. How many such pairs of digits are there in the number ‘62195738’ each of which has as many digits between them in the number (in both forward and backward directions) as they have between them in the arithmetic numerals? (1) Seven (2) Four (3) Six (4) More than seven (5) Five (Indian Bank Specialist Officer SO Exam, 08.03.2020)

SBI PO EXAMS 1. What will come at the place of question-mark (?) in the following alphabetical series ? AG IN PT ? (1) VZ (2) VY (3) WZ (4) UX (5) UY 2. What will come at the place of question-mark (?) in the following number series ? 12341253412563412 56? (1) 6 (2) 5 (3) 7 (4) 3 (5) 2 (SBI Specialist Officer (Law Officer : MMGS Scale-II) Online Exam, 19.04.2014)

3. How many such digits are there in the number 836257419, each of which is as far away from the beginning of the number as when the digits are rearranged in ascending order within the number?

SERIES (1) None (2) One (3) Two (4) Three (5) More than three (SBI Management Executive Exam, 23.02.2014)

4. The positions of how many digits will remain same if the digits in the number 49276381 are arranged in the ascending order within the number from left to right ? (1) None (2) Three (3) Two (4) One (5) More than three (SBI Assistant Manager (System) Exam, 17.01.2016)

5. What will come in place of question mark (?) in the following series based on the English alphabetical order and numbers? YC – 12 VF-16 SI-21 PL-27 (?) JR-42 (1) MN-34 (2) OR-33 (3) MO-32 (4) NP-32 (5) MO-34 SBI PO Prelim Exam, 09.06.2019

6. How many such pairs of digits are there in the number 74925816 each of which has as many digits between them in the number (in both forward and backward directions) as they have between them in arithmetic numerals ? (1) One (2) Two (3) Four (4) Three (5) More than four SBI PO Prelim Exam, 09.06.2019

7. If ‘1’ is added to each even digit and ‘2’ is subtracted from each odd digit of the number ‘75826349’, then how many digits will not appear more than once in the resultant number? (1) Three (2) One (3) More than three (4) None (5) Two SBI PO Prelim Exam, 09.06.2019

RBI GRADE–B/ NABARD GRADE–A OFFICER EXAMS Directions (1–3) : The following questions are based on the alphabetical series given below: CLRTBQSMAPDINFJ KGYX (RBI Grade-B Officer Exam. 06.02.2011)

1. Four of the following five are alike in a certain way and so form a group. Which is the one that does not belong to the group? (1) LBT (2) SPA

(3) IJF (4) PID (5) BMS 2. If ‘CT’ is related to ‘RQ’ and ‘AI’ is related to ‘DF’ in a certain way, to which of the following is ‘SP’ related to, following the same pattern ? (1) MD (2) DN (3) AD (4) AI (5) DF 3. What will come in place of the question mark in the following series? CR LB TM SI ? (1) PK (2) DK (3) DG (4) NX (5) PG 4. Which of the following will come in the given series in the place of question mark? W V U T 8 7 6 5 $ % # @ V U T 7 6 5 % # @ U T 6 5 ? (1) % (2) # (3) T (4) @ (5) 5 (RBI Officers in Grade ‘B’ Phase-I Exam. 16.08.2018)

INSURANCE EXAMS Directions (1–5) : In each of the following letter series, some of the letters are missing which are given in that order as one of the alternatives below it. Choose the correct alternative. (United India Insurance AAO Exam.03.06.2012)

1. f – h i g h – j h i j – i j – l – k l m (1) gikkj (2) jijlk (3) fihjk (4) ihjfk 2. m – o – o m o – n – n o n o – (1) nomon (2) mnmon (3) nnmmm (4) monno 3. q – r – s s s q r – r r – s – q (1) rssqr (2) srqsq (3) rqqrs (4) qsrsq 4. – j i x h – i j h y y – i x – x – j h y (1) jihxy (2) yxjhi (3) hiyxj (4) xjihy 5. w – q w – q – l y – l – z (1) lxwwq (2) qwlxw (3) wlwqx (4) xqlww Directions (6–10) : In each of the following letter series, some of the letters are missing which are given in that order as one of the alternatives below it. Choose the correct alternative. (United India Insurance AAO Exam.03.06.2012)

6. f – h i g h – j h i j – i j – l – k l m (1) gikkj (2) jijlk (3) fihjk (4) ihjfk 7. m – o – o m o – n – n o n o –

BPRE–28

8.

9.

10.

11.

(1) nomon (2) mnmon (3) nnmmm (4) monno q–r–sssqr–rr–s–q (1) rssqr (2) srqsq (3) rqqrs (4) qsrsq –jixh–ijhyy–ix–x–jhy (1) jihxy (2) yxjhi (3) hiyxj (4) xjihy w–qw–q–ly–l–z (1) lxwwq (2) qwlxw (3) wlwqx (4) xqlww What should come next in the following number series ? 02462302464502466702 ? (1) 4 (2) 2 (3) 6 (4) 8 (5) 5 (United India Insurance AO Exam. 26.05.2013)

12. Which number is wrong in the given series ? 1, 9, 25, 50, 81 (1) 1 (2) 25 (3) 50 (4) 81 (NICL (GIC) AO (Finance) Exam, 08.09.2013)

13. Which one set of letters when sequentially placed at the gaps in the given letters series shall complete it ? ab _ _ baa _ _ ab _ (1) baabb (2) aabab (3) aabaa (4) aaaaa (NICL (GIC) Administrative Officer Exam, 08.09.2013)

14. A, D, H, M, ?, Z (1) T (2) G (3) N (4) S 15. b, e, d, f, ?, h, j, ?, l (1) i m (2) m i (3) i n (4) j m (NICL (GIC) AO (Finance) Exam, 15.12.2013)

16. How many 7’s are there in the following series which are preceded by 6, which is not preceded by 8 ? 87678675679761677 688697687 (1) Two (2) Nil (3) One (4) Three (NICL (GIC) Administrative Officer Exam, 15.12.2013)

17. How many such digits are there in the number ‘6384257’ in which the digits are same as in the number series (both from backward and forward directions)? (1) None (2) One (3) Two (4) Three (5) More than three (LIC AAO Prelim Exam. 04.05.2019)

SERIES 45321 5

4. (3)

SHORT ANSWERS

14. (5) 3

NATIONALISED BANKS & IBPS PO/MT/SO EXAMS 1. (3)

2. (3)

3. (5)

4. (3)

5. (2) 9. (2) 13. (3) 17. (1) 21. (4) 25. (4) 29. (1)

6. (2) 10. (1) 14. (5) 18. (1) 22. (5) 26. (1) 30. (1)

7. (3) 11. (4) 15. (1) 19. (4) 23. (4) 27. (3)

8. (4) 12. (5) 16. (5) 20. (4) 24. (4) 28. (5)

3. (3) 7. (5)

2. (4)

2. (3) 6. (1) 10. (1) 14. (4)

3. (3) 7. (3) 11. (1) 15. (1)

+2

+2

+2

+3

+3

9 –1

3

3 –1

6

8 +1

+1

1

1, 2, 3, 4, 5, 6 3. (5) The pairs of opposite letters are given. The order of letters is changed every time. Thus,

A N 18. (1)

5

6

7

8

9

Y

+6

G

K

Q

+6

S

W

+6

B

ZA, BY, XC, DW, VE.

E

4

1

3

8

13. (3)

F

+5

U D K

–3 +2 +4

R F O

–3 +2 +3

O H R

BPRE–29

+2 +2

I

+4

8

R

E

+2

12

T

+2 +2

K

G

+2

+4

+2

I

+2

V

+2

G

I

+2

M

O

+4

16

20

+1

B

–1

+2

D

–2

+3 –3

+4

G

K

–4

–1

–2

X

+1

+2

D

+2

+3

8

V F 11

–3 +3 +4

–4

S

O

+4

I 15

M

+5

20

–5

J

+5

R

+6 26

23. (4)

12. (5)

9

+2

E

+2

5 7 3 8 1 4 2 6 9

B +1 C +1 D +1 E +1 F +2 +2 H +2 J +2 L D F –1 –1 –1 W –1 V X Z Y

5

C

P

+2

22. (5)

(4)

7

+2

C

B +3 E +3 H +3 K +3 N E +4 I +4 M +4 Q +4 U D +3 G +3 J +3 M +3 P

+6

M

+2

21. (4)

10. (1)

+2

2

P T W Y +2 6 +4 10 +6 16 +8 24 4 20. (4) C

+6

+2

+4

Z

6

B

9

19. (4)

6 4 3 9 1 5 8 2 7 1 2 3 4 5 6 7 8 9

X

3

L

4

9. (2)

–3

4

Q –5

G

0

Std 4 Std 5

3

6

U –4

6 9 8 5 2 7 3 1

6

8

X –3

A

–1

1 2 3 5 6 7 8 9

+6

2. (3) The sereis proceeds as illustrated below : 9, 98, 987, 9876, 98765, 987654, 987654

7 –1

2

E

1

Z –2

8. (4)

NATIONALISED BANKS & IBPS PO/MT/SO EXAMS Std 2 Std 3

6

O

3rd from left

4. (2)

C

9

J +5

A

11.

Std 1

2 +1

0

EXPLANATIONS

A

2

F +4

+2

5 3 6 8 2 7 9 Ascending order of digits

A

B

8

17. (1)

+3

+1

4. (2) 8. (3) 12. (3) 16. (4)

4

C +3

+3 N  → Q   → T   → W   → Z

4. (3)

3. (3)

7

A +2

7. (3)

2. (3) 6. (5)

D

16. (5)

2

A  → C   → E   → G   → I

INSURANCE EXAMS

1. (3) G

45321

6. (2)

RBI GRADE–B/ NABARD GRADE–A OFFICER EXAMS

1. (1) 5. (1) 9. (2) 13. (3) 17. (5)

15. (1) 7

bw x y z a, d s t u v c , f o p q r e

SBI PO EXAMS

1. (4)

45321 3

5. (2)

4

1. (2) 5. (5)

1

45321 4

–3 +2 +2

L

D

+3

+3

E

+4

+4

I

+3

G

+3

H M J

+3

K

+4

M

T

B +1 C +1 D +1 E +1 +2 +2 +2 J +2 F H 24. (4) D –1 –1 –1 –1 W Z Y X

8 1

6 2

1 3

4 3 4 5

2 6

5 8

U

+3

25. (4)

J

N

+4

Q

+3

+3

9 9

P

F L V

SERIES 26. (1)

7. (5)

–2

Z

X

+2

B

U

+3

D

–2

Y

–3

G

–3

W

T

–4

–5

Q

+4 –4

L

+5

K

P

–5

P

K

27. (3)

7 –2

5 –2

5

4

8

5

1

6

3

28. (5) 9

1

8

7 5

1

9

7

8

5

5 3

8 9

4 0

1

8

3 4

9

1

0

5

7th from the right end ⇒ 5 7

8

+2

2

6

+1

9

+1

3

3 –2

7

1

4 +1

9

ix h /x

–2

5

7

RBI GRADE–B/ NABARD GRADE–A OFFICER EXAMS

2

–1

9

+2

7

4 –1

4

1

5

+2

3

–1

3

2

1

9

5

−1

+3

−1

+3

8

−1

r s s q

−1

ix h /x

+2 A  → D

+8

A

I

+7

G

N

+7

P

+6

2. (3) 1 1 1

2 2 2

5 5

3 3 6

1

2

5

6

+1

4. (2) 4

6 3

2 4

5 5

7 4 6 7

1 8

9 9

12 +4

16 +5

21 +6

V U T V U T

P

–3

M

L

+3

O

27 +7

8 7 6 5 7 6 5 6 5

/z 23 45 67

02 4 12. (3) (1)2 = 1 (3)2 = 9 (5)2 = 25

.............

(9)2 = 81 13. (3)

$ % # @ % # @ # @

1. (1) f g h i /g h i j / h i j k –3

J

+3

R

34 +8

l q

11. (1) 0246 0246 0246

42

ly/

(7)2 = 49

a / ab a

14. (4)

S +7 Z

A +3 D +4 H +5 M +6 +3 15. (1) b f +3 j +3

INSURANCE EXAMS

5. (5) S –3 I +3

jhy

ab a / a ba / a b

4 9 2 7 6 3 8 1 1 2 3 4 6 7 8 9 V –3 F +3

+4

U T

4. (3)

Y –3 C +3

W

7 ....

3. (3)

8 3 1 2

+3

+2 +3 +4 +5 R  → B   → M   → I   → G

Y

4 4 3

+2

C  → L   → T   → S   → D

V

+5

T

w

+2 → I P  3. (3)

+6

i

10. (1) w l q / w x q / w

+2 → A S 

1. (2)

y j i x h / x i j h y /y j

9. (2)

+2 I  → F Similarly,

SBI PO EXAMS

/

8. (3) q r r q /s s s q /r q r r /

B  → M   → S

8

/z

m n o /n o m

P  → I   → D +3

l q

7. (3) m n o / n o m /o m n /

+2 → R 2. (4) C   +2 T  → Q

3

w

ly/

i j k l/ j k l m

−1

+2

jhy

6. (1) f g h i /g h i j / h i j k

I  → J   → F

+2

i

5. (1) w l q / w x q / w

→ T 1. (4) L  → B  

6

4

7

+3

y j i x h / x i j h y /y j

4. (2)

S  → P   → A

Odd digits ⇒ 9, 7, 3, 3 30. (1) 6

+1

3

29. (1) –1

2

Digits 9 and 1 appear only once.

2

3

8

e

–1 d –1 h

i m –1

l

/ 16. (4) 87678 67 5 67

9761

i j k l/ j k l m

67 2. (3) m n o / n o m /o m n /

7688697687

17. (5)

m n o /n o m

6. (5)

7

4

9

2

5

8

1

6

3. (3) q r r q /s s s q /r q r r /

r s s q

BPRE–30

6

3

8

4

2

5

7

❐❐❐

SERIES

MODEL EXERCISES 1. Fill in the next number in the series 2

3 4 5 , , , ,? 5 5 5 5 25

(1) (3)

7 25 6 25 5

(2) (4)

6 125 6 5 5

(5) None of these 2. Which combination of alphabets would come in the position of the question mark in the following sequence? ABP, CDQ, EFR, ? (1) GHS (2) GHT (3) HGS (4) GHR (5) None of these 3. Which of the following will come next in the series given below? nsi, org, pqe, qpc, ? (1) pqa (2) rqd (3) aor (4) roa (5) None of these Directions (4-7) : In these questions, a series is given which may consist of number, alphabets or a mixture of both. you have to identify from a choices given against each question, the next term in the series. 4. 6, 13, 28, 59, ? (1) 111 (2) 122 (3) 113 (4) 114 (5) None of these 5. QPO, SRQ, UTS, WVU, ? (1) XVZ (2) ZYA (3) YXW (4) VWX (5) None of these 6. 1, 5, 13, 25, 41, ? (1) 61 (2) 57 (3) 51 (4) 63 (5) None of these 7. 3, 10, 29, 66, 127, ? (1) 164 (2) 187 (3) 218 (4) 216 (5) None of these Directions (8-18) : Find the next term in the given series in each of these questions. 8. 198, 194, 185, 169, .... (1) 136 (2) 144 (3) 112 (4) 92 (5) None of these

9. 6, 9, 7, 10, 8, 11, ..... (1) 12 (2) 13 (3) 9 (4) 14 (5) None of these 10. 5, 6, 8, 9, 11, ... (1) 15 (2) 12 (3) 17 (4) 20 (5) None of these 11. 35, 30, 25, 20, 15, 10, .... (1) 15 (2) 10 (3) 5 (4) 2 (5) None of these 12. 0, 2, 6, 12, 20, .... (1) 38 (2) 30 (3) 45 (4) 60 (5) None of these 13. 5, 7, 9, 11, 13, .... (1) 15 (2) 10 (3) 8 (4) 6 (5) None of these 14. 125, 80, 45, 20, ..... (1) 8 (2) 12 (3) 10 (4) 5 (5) None of these 15. 198, 202, 211, 227, ..... (1) 210 (2) 212 (3) 252 (4) 27 (5) None of these 16. What is the next term in the following series? 2, 15, 41, 80, ? (1) 132 (2) 121 (3) 111 (4) 120 (5) None of these 17. Complete the series— 10, 18, 34, ?, 130, 258 (1) 32 (2) 60 (3) 68 (4) 66 (5) None of these 18. What is the next letter in the following series? a, c, b, d, e? (1) h (2) g (3) j (4) i (5) None of these Directions (19-23) : For these questions, what is the missing element in the sequence represented by the question mark?

BPRE–31

19. A, G, L, P, S, ? (1) X (2) Y (3) W (4) U (5) None of these 20. P3C, R5F, T8I, V12L, ? (1) Y1170 (2) X17M (3) X170 (4) X160 (5) None of these 21. 625, 5, 125, 25, 25, ?, 5 (1) 125 (2) 5 (3) 25 (4) 625 (5) None of these 22. 2, 12, 30, 56, ?, 132, 182 (1) 116 (2) 76 (3) 90 (4) 86 (5) None of these 23. 1, 1, 2, 6, 24, ?, 720 (1) 100 (2) 104 (3) 108 (4) 120 (5) None of these 24. The sum of the series 12 + 22 + 32 + 42 + ... + 152 is (1) 1080 (2) 1240 (3) 1460 (4) 1620 (5) None of these 25. Three numbers are in GP. Their sum is 28 and their product is 512. The numbers are (1) 6, 9 and 13 (2) 4, 8 and 16 (3) 2, 8 and 18 (4) 2, 6 and 18 (5) None of these 26. If the second term of a geometric progression is 2 and the sum of the series to infinity is 8, then the first term is (1) 5 (2) 2 (3) 4 (4) 1 (5) None of these 1 1 27. If b − a + b − c

=

1 1 + , then a, a c

b, c form a/an (1) arithmetic progression (2) geometric progression (3) harmonic progression (4) All of the above (5) None of these 28. If the nth term of an AP is 4n + 1, then the common difference is (1) 3 (2) 4 (3) 5 (4) 6 (5) None of these

SERIES Directions (29-30) : In each of these questions given below, there is a series with one number missing as shown by question mark. This term is given as one of the alternatives. Mark the correct alternative. 29. 4, 9, 25, ?, 169, 289, 361 (1) 49 (2) 74 (3) 89 (4) 91 (5) None of these 30. 2, 5, 10, 17, 26, 37, ? (1) 61 (2) 58 (3) 50 (4) 72 (5) None of these Directions (31-34) : In these questions, some of the letters are missing. The missing letters are given in the proper sequence as one of the alternatives given under each question. Find the correct choice. 31. –c – c a – ab – bc – (1) abcac (2) babca (3) ccabb (4) bcabb (5) None of these 32. rst – – rs – uv– stu – rst – – (1) rstrsts (2) uvtrstu (3) uvtrsuv (4) uvtrvuv (5) None of these 33. ab...abb...bba...b (1) aba (2) bba (3) bab (4) baa (5) None of these 34. Which term of the series 5, 10, 20, 40 .... is 1280? (1) 10th (2) 9 th th (3) 8 (4) 11th (5) None of these Directions (35-37) : A missing term in the series given in each of these questions is marked by a question mark (?). Choose the correct term to complete the series? 35.

1 3 5 7 , , , ,? 2 4 8 16

(1) 9/32 (2) 10/17 (3) 11/34 (4) 12/35 (5) None of these 36. ABD, DGK, HMS3 MTB, SBL, ? (1) ZKW (2) ZKU (3) ZAB (4) XKW (5) None of these

37. Choose the term that will continue the following series : P 3 C, R 5 F, T 8 I, V 12 L, ? (1) Y 17 O (2) X 17 M (3) X 17 O (4) X 16 O (5) None of these Directions (38-39) : In each of the following number series, two terms have been put within brackets. Mark your answer as (1) If the first bracketed term is right and second is wrong (2) If both the bracketed terms are right (3) If the first bracketed term is wrong and second is right (4) If both the bracketed terms are wrong >? (5) None of these 38. 4, 6, 10 (12), 16, (14), 22 39. 3, 10, 29, (66), (127), 218 40. (2), 5, (12), 25, 41, 61 Directions (41-48) : In each of the following questions, various terms of a series are given with one term missing as shown by 0). Choose the missing term. 41. QPO, SRQ, UTS, WVU, (?) (1) XVZ (2) YXW (3) ZYA (4) VWX (5) None of these 42. 19, 2, 38, 3, 114, 4, (?) (1) 228 (2) 352 (3) 256 (4) 456 (5) None of these 43. YEB, WFD, UHG, SKI, (?) (1) QOL (2) TOL (3) QGL (4) QNL (5) None of these 44. AZ, CX, FU, (?) (1) IR (2) JQ (3) IV (4) KP (5) None of these 45. 2Z5, 7Y7, 14X9, 23W11, 34V13, (?) (1) 27U24 (2) 45U15 (3) 47U15 (4) 47V14 (5) None of these 46. Complete the series. D-4, F-6, H-8, J-10, ?, ? (1) K-12, M-13 (2) L-12, N-14 (3) L-12, M-14 (4) K-12, M-14 (5) None of these

BPRE–32

Directions (47-48) : Complete the following series. 47. J2Z, K4X, L7V, ?, N16R, O22P (1) I 11 T (2) LI IS (3) M11 T (4) L 11T (5) None of these 48. gfe — ig — eii — fei —gf — ii (1) e i f g i (2) f i g i e (3) i f g i e (4) i f i g e (5) None of these 49. Which of the following will come in place of the question mark (?) in the following sequence? 6C7, 8F10, 11J14, 15O19, ? (1) 25U20 (2) 20U25 (3) 20U24 (4) 19U25 (5) None of these Directions (50-53) : Complete the series. 50. CAT, FDW, IGZ, ? (1) KJA (2) KTC (3) LHD (4) LJC (5) None of these 51. (2, 3), (3, 5), (5, 7), (7, 11), (11, 13), ? (1) (13, 15) (2) (15, 16) (3) (13, 17) (4) (13, 19) (5) None of these 52. HBGQ, LFKU, PJOY, ?, XRWG (1) TMRB (2) TNSC (3) SMTD (4) TQTB (5) None of these 53. MHZ, NIW, OKT, PNQ, ? (1) RRN (2) QRN (3) QRM (4) QQN (5) None of these Directions (54-57) : Complete the series. 54. A GMS → CIOU then ? (1) BHNT bhnt (2) ABCD ascd (3) BHNT djpv (4) bhnt DJPV (5) None of these 55. 48, 24, 72, 36, 108, ? (1) 115 (2) 110 (3) 121 (4) 54 (5) None of these 56. K-ll, M-13, P-16, T-20, ? (1) V-22 (2) U-21 (3) Y-25 (4) W-25 (5) None of these 57. B0R, E3U, G9Y, J18D..... (1) E3P (2) L30J (3) H9N (4) G3U (5) None of these

SERIES Directions (58-59) : Identify the letter-series that follows the given rule. 58. Number of letters skipped in between adjacent letters in the series decreases by one. (1) EJNQST (2) CINRTU (3) BGKNPR (4) LQUXAP (5) None of these 59. In the following series, the number of letters skipped in between the adjacent letters are in ascending order, ie, 1, 2, 3, 4. Which one of the following letter groups does not obey this rule? (1) GILPU (2) DFIMR (3) CEHLQ (4) HJMPT (5) None of these Directions (60-64) : Complete the series. 60. X, U, S, P, N, K ? (1) I (2) M (3) L (4) Q (5) None of these 61. FDS, JHW, NLA, ? (1) QND (2) RQD (3) RPE (4) SOF (5) None of these 62. 4, 10, 22, 46, ? (1) 56 (2) 76 (3) 66 (4) 94 (5) None of these 63. 9, 15, 23, 33, ? (1) 44 (2) 38 (3) 36 (4) 45 (5) None of these 64. C-2, E-3, G-4, I-5, ? (1) H-6 (2) K-6 (3) J-8 (4) L-7 (5) None of these Directions (65-67) : Complete the series by replacing the missing term indicated by ? 65. KM5, IP8, GS11, EV14, ‘?’ (1) BY17 (2) BX17 (3) CY17 (4) CY18 (5) None of these 66. 2A11, 4D13, 12G17, ? (1) 36J21 (2) 36 I 19 (3) 48J21 (4) 48J23 (5) None of these 67. ABD, DGK, HMS, MTB, SBL, ? (1) ZAB (2) XKW (3) ZKU (4) ZKW (5) None of these

Directions (68-73) : In each of these questions complete the series by replacing the ‘?’. 68. D-4, F-6, H-8, J-10, ? (1) K-12 (2) M-14 (3) L-12 (4) M-13 (5) None of these 69. 3F, 6G, 11I, 18L, ? (1) 21O (2) 25N (3) 27 P (4) 27Q (5) None of these 70. A, G, L, P, S, ? (1) U (2) W (3) X (4) Y (5) None of these 71. AI, BJ, CK, ? (1) DL (2) DM (3) GH (4) LM (5) None of these 72. 6, 11, 21, 36, 56, ? (1) 42 (2) 51 (3) 81 (4) 91 (5) None of these 73. 1, 9, 17, 33, 49, 73, ? (1) 97 (2) 98 (3) 99 (4) 100 (5) None of these Directions (74-78) : Complete the series by replacing the ‘?’ mark. 74. BC6, EFG, 210, IJK? (1) 1000 (2) 190 (3) 990 (4) 999 (5) None of these 75. 2A11, 4D13, 12G17, ? (1) 36J21 (2) 48J21 (3) 38 I 19 (4) 48J23 (5) None of these 76. 44, 40, 34, ? 16, 4 (1) 26 (2) 21 (3) 28 (4) 19 (5) None of these 77. 5, 9, 16, 29, 54, 103, ? (1) 103 (2) 94 (3) 102 (4) 200 (5) None of these 78. In which of the following series is the number of letters skipped between adjacent letters decreasing by one? (1) LPRU (2) LPSU (3) LPTV (4) LRPU (5) None of these Directions (79-80) : Complete the series by replacing ‘?’ mark.

BPRE–33

79. F6T20 H8V22 ? L12Z26 (1) J10X22 (2) H12X24 (3) H10X22 (4) J10X24 (5) None of these 80. C24W4 D23V5 E22U6 ? (1) F22T10 (2) F21T7 (3) F23T8 (4) F22T7 (5) None of these 81. Complete the series 162, 54, 18, 6, ? by replacing the question mark ‘?’ (1) 2 (2) 3 (3) 4 (4) 5 (5) None of these Directions (82-85) : Complete the series. 82. 5G7, 7H10, 10114, 14J19, ....... (1) 16K20 (2) 17K21 (3) 18K21 (4) 19K25 (5) None of these 83. J15K M21N ...... S39T V51W (1) N24P (2) P27Q (3) P29Q (4) P25Q (5) None of these 84. D23F H19J L17N ......... T11V (1) P15R (2) P14R (3) P13R (4) P12R (5) None of these 85. A70B D65F H60J ...... P50R (1) K55L (2) L55M (3) L55N (4) L55P (5) None of these Directions (86-89) : Complete the series by replacing the ‘?’ mark. 86. 4A, 12D, 28G, 60J, 124M, ‘?’ (1) 248Q (2) 252P (3) 256T (4) 246S (5) None of these 87. P3C, R5F, T8I, V12L, ? (1) Y17O (2) X17M (3) X17O (4) X160 (5) None of these 88. K8K, N13E, Q20Z, T31Y, ? (1) Z37Z (2) Z38Q (3) W44S (4) X41X (5) None of these 89. A729, G343, ?, S27, Yl (1) L64 (2) N75 (3) Q100 (4) M125 (5) None of these

SERIES Directions (90-93) : Complete the series by replacing the ‘?’ mark. 90. XZ 16 BD, FH 64 JL, NP 144 RT, VX 256 ZB, ? (1) DF 400 HJ (2) CE 400 HJ (3) DF 515 HJ (4) DF 324 HJ (5) None of these 91. 15 ABC 28, 18 DEF 32, 21 GHI 36, ?, 27 MNO 44 (1) 23 JKL 38 (2) 25 JKL 42 (3) 24 JKL 40 (4) 26 JKL 43 (5) None of these 92. AL20, CQ28, FU40, JX56, ? (1) HZ68 (2) OZ76 (3) LM86 (4) FZ68 (5) None of these 93. A3T, D7R, G16P, J35N, M74L, ? (1) R126P (2) P153J (3) N102L (4) Q163R (5) None of these Directions (94-98) : A sequence of groups of letters and numbers is given with one term missing as shown by (?). Choose the missing term out of the given alternatives. 94. 512 BCD 65, 343 EFG 50, 216 HIJ 37, ?, 64 NOP 17 (1) 180 KLM 30 (2) 191 KLM 33 (3) 125 KLM 26 (4) 98 KLM 26 (5) None of these 95. 17 YXW 65, 26 VUT 126, 37 SRQ 217, ?, 65 MLK 513 (1) 49 NOP 342 (2) 48 PON 341 (3) 50 PON 343 (4) 50 PON 344 (5) None of these 96. ZY 2505 BA, XW 1604 DC, VU 903 FE, ?, RQ 101 JI (1) TS 402 HG (2) ST 402 GH (3) TS 402 GH (4) ST 402 HG (5) None of these 97. 49 BCD 54, 60 EFG 67, 75 HIJ 84, ?, 117 NOP 130 (1) 92 KLM 103 (2) 909 KLM 98 (3) 94 KLM 105 (4) 96 KLM 116 (5) None of these

98. 101 ZA 82, 65 YB 50, 37 XC 26, ?, 5 VE 2 (1) 17 WD 10 (2) 20 WD 18 (3) 15 WD 8

(4) 12 WD 6

(5) None of these Directions (99–107) : Complete the series by replacing the question Mark (?) : 99. B0R, E3U, G9Y, J18D, ? (1) E30P

(2) H27N

(3) L30J

(4) G33U

(5) None of these 100. A729, G343, ?, S27, Y1

SHORT ANSWERS 1.(3)

2.(1)

3.(4)

4.(2)

5.(3)

6.(1)

7.(3)

8.(2)

9.(3)

10.(2)

11.(3)

12.(2)

13.(1)

14.(4)

15.(3)

16.(1)

17.(4)

18.(2)

19.(4)

20.(3)

21.(1)

22.(3)

23.(2)

24.(2)

25.(2)

26.(3)

27.(3)

28.(2)

29.(1)

30.(3)

31.(2)

32.(2)

33.(3)

34.(2)

35.(1)

36.(1)

(1) L64

(2) M75

37.(3)

38.(2)

39.(2)

40.(2)

(3) Q100

(4) M125

41.(2)

42.(4)

43.(1)

44.(2)

45.(3)

46.(2)

47.(3)

48.(3)

49.(2)

50.(4)

51.(3)

52.(2)

53.(2)

54.(3)

55.(4)

56.(3)

57.(2)

58.(1)

59.(4)

60.(1)

61.(3)

62.(4)

63.(4)

64.(2)

(2) FZ68

65.(3)

66.(4)

67.(4)

68.(3)

(4) HZ68

69.(3)

70.(1)

71.(1)

72.(3)

73.(1)

74.(3)

75.(4)

76.(1)

77.(4)

78.(2)

79.(4)

80.(2)

(5) None of these 101. Q331T, U442X, Y553A, ? (1) Z665D

(2) C664C

(3) B666D

(4) B664F

(5) None of these 102. RL12, TQ20, WU32, AX48, ? (1) EZ68 (3) GZ68

(5) None of these 103. V2R,Y8T,B15V, E76X, ? (1) F350Z

(2) G452Z

81.(1)

82.(4)

83.(3)

84.(3)

(3) H355B

(4) H355Z

85.(3)

86.(2)

87.(3)

88.(3)

89.(4)

90.(1)

91.(3)

92.(2)

93.(2)

94.(3)

95.(4)

96.(1)

97.(3)

98.(1)

(5) None of these 104. Select the missing number from the given responses. 9, 8 , 10, 16, 11, ?, 12, 64 (1) 28

(2) 36

(3) 25

(4) 32

101.(2)

99. (3) 100. (4)

102.(2) 103. (4) 104. (4)

105. (4) 106. (3) 107. (2)

105. Select the missing number from the given responses.

EXPLANATIONS

3, 4, 7, 11, 18, 29, ? (1) 31

(2) 39

(3) 43

(4) 47

106. Select the missing number from the given responses. 16, 18, 21, 25, 30, 36, ? ,,.. (1) 38

(2) 61

(3) 43

(4) 51

107. Select the missing number from the given responses. 121, 144, 289, 324, 529, 576, __ ? (1) 961

(2) 841

(3) 900

(4) 1320

BPRE–34

1. (3) Numerator of each fraction increases by 1 and denominator of each fraction is multiplied by

5. Hence, the next fraction would

be

6 25 5

.

2. (1) First and second letter move two steps in forward direction while the third letter moves one step in alphabetical order. 3. (4) 1st letter moves +1 step, second letter moves –1 step. IIIrd letter moves –2 steps. Hence, the next group of letters would be r o a.

SERIES 4. (2) The series progresses with a difference of 7, 7 × 2 + 1 = 15, 15 × 2 + 1 = 31, 31 × 2 + 1 = 63 Hence, the next digit of the series would be 59 + 63 = 122 5. (3) This question can be solved by clear observation of the first letter of each group. First letter of each group progresses with a difference of one letter Q (R) S (T) U (V) W (X) Y. Hence, next letter will be Y. 6. (1) The series progresses with a difference of + 4, + 8, + 12, +16, +20. 7. (3) Difference of each consecutive term of the series is 7, 19, 37, 61, 91. The series of difference moves with the following rule +12, +18, +24, +30. Hence, the next number would be 127 + 91 = 218 8. (2) The sequence progresses with the following difference – 22, –32, –42, –52. 9. (3) First, third, fourth and fifth terms are consecutive natural numbers. 10. (2) The sequence progresses with the following difference +1, +2, +1, +2, +1. Hence, the next number would be 12. 11. (3) The series moves with a difference of –5. 12. (2) The series moves with a difference of +2, +4, +6, +8, +10. 13. (1) The series moves with a difference of +2. 14. (4) The series moves with a difference of –45, –35, –25, –15. 15. (3) The series moves with the difference of + 22, + 32, + 42, + 52. 16. (1) The series follows the following pattern +13, +26, +39, + 52. Hence, the next number will be 80+ 52 = 132 17. (4) The pattern of the series is +8, +16, +32, +64, +128. 18. (2) a(b)c / b(c)d / e(f)g Hence, the next letter of the series would be g. 19. (4) The letters moves +6, +5, +4, +3, +2 steps in alphabetical orders. 20. (3) First letter moves two steps, second letter moves two steps, three steps, four steps .... successively and third letter moves third steps in forward direction. 21. (1) The missing number is 125.

22. (3) The series progresses with the difference +10, +18, +26, +34, +42, +50, .... 23. (2) The difference of the series is × 2, × 2, × 3, × 4, × 5, × 6, .... 24. (2) The sum of the squares of the first n natural numbers is given by n n + 1 2n + 1

b

gb

g

6

=

15 15 + 1 30 + 1

b

gb

g

6

= 1240 25. (2) Let the three numbers of GP be a, ar, ar2. Given, a + ar + ar2 = 28 and ∴ a3r3 = (51)2 ar = 8, a + ar2 = 20 ⇒ 8r2 – 20r + 8 = 0 ⇒ r = 2,r = –2 If r = 2, a = 4 Therefore, the three numbers are 4, 8, 16. 26. (3) Let the series of GP be

32. (2) rst/uv/rst/uv/rst/uv/rst/uv 33. (3) abb /abb /abb /abb 34. (2) The given series is GP. nth term of the series n–1 = ar n–1 1280 = 5 × (2) n–1 256 = (2) ⇒ (2)8 = (2)n–1 n–l=8⇒n=9 35. (1) Numerator increases by 1 and denominator is double that of the preceding number. Hence, the

9 . 32 36. (1) First letter moves +3, +4, +5, +6 ... steps in the alphabetical order, second letter moves +5, +6, +7: +8, ... steps and third letter moves +7, +8, +9, + 10, ... steps in the alphabetical order. 37. (3) First letter moves +1 step in alphabetical order, number moves +2, +3, +4, +5 steps in numerical order and last letter moves +3 steps in alphabetical order. 38. (2) next fraction would be

a , a , ar , ar 2..... a = 2, r a /r S = 1−r ∞



a r 1- r

b g

39. (2) =8

2

⇒ 4r – 4r + 1 = 0 ⇒ r=

a 1 ⇒ = Ist term = 4 r 2

1 1 1 1 27. (3) b − c − c = a − b − a 2ac ⇒ b = a +c 1 1 1 , , are in AP. a b c ⇒ a, b, c are in HP. 28. (2) nth term = a+ (n – 1) d = 4n + 1 (a – d) + nd = 1 + 4n ⇒ (a – d) = 1 and d = 4 ∴ a = 5 and d = 4 29. (1) The series is square of consecutive prime numbers. 30. (3) The given series is 12 + 1, 22 + 1, 32 + 1, 42 + 1, 52 + 1, 62 + 1, 72 + 1. Hence, the required number is 50. 31. (2) bca/cab/abc/bca.

3 10 29 (1)3 + 2 (2)3 + 2 (3)3 + 2 (66) (127) 218 ( 4 )3 + 2 (5)3 + 2 (6)3 + 2

40. (2)

41. (2)



BPRE–35

42. (4) The series is alternate one ×2

43. (1)

44. (2)

×3

×4

and

SERIES 45. (3) 55. (4)

66. (4) ×

1 2

×3

×

1 2

×3

×

1 2

56. (3) +2

46. (2) Each alphabetical letter is 2 place ahead in the alphabetical order. 47. (3)

+3

+2

+4

+3

+5

+4

67. (4)

+5

57. (2) +2

+2

+2

+2

68. (3) +2

48. (3) gfeii / gfeii / gfeii / gfeii 49. (2) +2

+3

+4

+1

+2

+4

+2

+2

+3

+4

69. (3)

59. (4)

+3 +3

+2

58. (1)

+5

+5

+7

+9

70. (1)

And C + 3 F + 4 J + 5 O + 6U 50. (4)

60. (1) The series follows the pattern

+6

+1

71. (1) 51. (3) (2, 3), (3, 5), (5,7), (7,11), (11,13) This is the series twin prime number. So, next term will be (13,17). 52. (2)

+4

+4

+4

+4

+4 +4

+1

+3

+1

+1

+1

+1

+2

72. (3)

+4 +4

+4

62. (4) 53. (2)

+6

+12

+24

+48

+6

+8

+10

+12

63. (4)

Similarly,

+4

61. (3)

First letter of each subsequent groups has three alphabets in between. The method is same for second and third letter of each group.

54. (3)

+5

64. (2) C – 2 → (3 – 2) = 1 E – 3 → (5 – 3) = 2 G – 4 → (7 – 4) = 3 I – 5 → (9 – 5) = 4 K – 6 → (11 – 6) = 5 Letter position in alphabet minus the number gives a series of natural number starting from 1. 65. (3)

BPRE–36

+5

+10

+15

+20

+25

73. (1)

+8

+8

+16

+16

+24

74. (3) The pattern is BC = 2 × 3 = 6 EFG = 5 × 6 × 7 = 210 LIK = 9 × 10 × 11 = 990 75. (4) The pattern is +2

×2

+4

×3

+6

×4

76. (1) 77. (4) The pattern is 5×2–1=9 9 × 2 – 2 =16 ... so on ∴ 103 × 2 – 6 = 200 78. (2) The required series is L+4=P P+3=S S+2=U

+24

SERIES 90. (1)

79. (4)

100. (4)

+2

+2

+2

+2

+2

+2

+6 A +6 G +6 M +6 S 27 729 343 125

+2

(9)3

91. (3) 80. (2)

+3

15

+1

ABC

+4

28 92. (2)

+3

18

+3

21

DEF

+1

GHI

+1

32

+4

36

+4

24

+3

27

JKL

+1

M NO

+4

44

40

(5)3

(3)

3

3

(1)

101. (2) +4

Q

+4

U

+111

T

Y

+111

442

+4

553

+3

X

A

+4

C

+111

664

+2

C

102. (2) +2

+3

+5

+4

+4

+5

R  → T  → W  → A  →

82. (4) +3

+4

+5

+6

F +2 L  → Q  → U  →X  → Z +8 +12 +16 +20 12  → 20 →32 →48 → 68

93. (2)

83. (3) +6

(7) 3

331

81. (1)

+2

Y 1

+8

+10

+3

103. (4)

+12

3

3

3

+3

+6

+7

+61

+279

+2

+2

+2

+2

+ + + V  → Y  →B  → E  →

–2

–4

–2

+1

+1

–11

–9

+1

+1

–13

–15

104. (4) There are two alternating series :

9

95. (4)

85. (3) –5

Z

R  → T  →V  →X  →

84. (3) –4

H

2  → 8  →15 →76  → 355

94. (3)

–5

–5

–5

+9

+11

+13

+15

–1

–1

–1

–1

10 +1

8 86. (2)

+1 16

12 +1

32

64

96. (1)

×2

×2

×2

105. (4) 3 + 4 = 7 4 + 7 = 11 11 + 18 = 29

87. (3)

18 + 29 = 47

97. (3) 49

88. (3)

11

+11

BCD \ 54

+1

+ 13

+15

60

EFG

+1

67

+ 17

+19

75 HIJ

+1

84

+ 21

94 KLM 105

+23 +1 +25

117 NOP 130

98. (1)

106. (3)16 + 2 = 18 18 + 3 = 21 21 + 4 = 25 25 + 5 = 30 30 + 6 = 36 36 + 7 = 43

In the second series, each term is increased by next prime number. 89. (4)

99. (3) +3

+2

+3

+3

→ E   → G  → J   → B  +3

+6

+9

+12

0  → 3   → 9  →18  → +3

+4

+5

+6

R  → U   → Y   → D   →

BPRE–37

L 30 J

107. (2) 121 = 11 × 11 144 = 12 × 12 (12 + 5)2 = 17 × 17 = 289 18 × 18 = 324 (18 + 5)2 = 23 × 23 = 529 24 × 24 =576 (24 + 5)2 = 29 × 29 = 841 ❐❐❐

ANALOGY

3

ANALOGY

QUESTIONS FROM 1999 TO 2010 ARE AVAILABLE ONLINE NATIONALISED BANKS & IBPS PO/MT/SO EXAMS Directions (1–3) : Study the following information to answer the given questions : (Indian Bank PO Exam. 02.01.2011 (Ist sitting)

‘TRAVEL’ is related to ‘UDKUSB’ and ‘CORNER’ is related to ‘MDQDPS’. 1. ‘SURVEY’ is related to ____. (1) UDXSTV (2) UXDTSV (3) TVSUDX (4) UDXTVS (5) UDXVTS 2. ‘GROUPS’ is related to ____. (1) TORHSP (2) TOHRSP (3) TORPHS (4) ROTHSP (5) ROTPHS 3. Which of the following pairs of words have the same relationship as FAN : HEAT ? (1) Water : Drink (2) Light : Night (3) Teach : Student (4) Air : Breathe (5) Food : Hunger (Punjab & Sind Bank PO Exam. 23.01.2011)

Directions (4 – 5) : Study the following information to answer the given questions : (IBPS Specialist Officer CWE Exam.17.03.2013)

‘TRAVEL’ is related to ‘UDKUSB’ and ‘CORNER’ is related to ‘MDQDPS’. 4. ‘SURVEY’ is related to ____. (1) UDXSTV (2) UXDTSV (3) TVSUDX (4) UDXTVS (5) UDXVTS 5. ‘GROUPS’ is related to ____. (1) TORHSP (2) TOHRSP (3) TORPHS (4) ROTHSP (5) ROTPHS 6. In a certain code TEMPORAL is written as OLDSMBSP, how is CONSIDER written in that code?

(1) RMNBSFEJ (2) BNMRSFEJ (3) RMNBJEFS (4) TOPDQDCH (5) None of these IBPS RRBs Officer CWE (Prelim) Exam, 17.08.2019

7. In a certain code language ‘LEAGUE’ is written as ‘NHEIXI’ and ‘RETAIL’ is written as ‘THXCLP’. How will ‘DAMAGE’ be written in that code language ? (1) ECPBIH (2) GDPDJH (3) FDQCJI (4) FCOCIG (5) HEQEKI IBPS Bank PO/MT CWE (Prelim) Exam, 12.10.2019

8. In a certain code Language, POMADE is written as WLTZKV and MOTLEY is written as TLASVF. How will HOSTEL be written in that code language ? (1) NLYAVR (2) SLZAVO (3) PLABVT (4) OLZAVS (5) LOAZSV IBPS Bank PO/MT CWE (Prelim) Exam, 30.11.2019

9. In a certain code if the word ‘‘IMPOSE’’ is coded as “HNOPRF”, and ‘‘ARDENT’’ is coded as “ZSCFMU”, then the ‘‘HORDES’’ will be coded as “__________”. (1) GPQCFR (2) GPSEDR (3) GPQEDT (4) IPSCFR (5) IPQCDT (IBPS SO Prelim Exam, 28.12.2019)

10. In a certain code language, ‘RIGHT’ is written as ‘UGJKW’ and ‘HOUSE’ is written as ‘KMSVC’. How will ‘POINT’ be written in that code language? (1) SRLQW (2) SMGQW (3) NMGLR (4) RRKPV (5) TNHRV (IBPS RRBs Officer CWE Prelim Exam, 13.09.2020)

BPRE–38

SBI PO EXAMS 1. In a certain code language, FORTUNE is coded as SPGVFOV and JOURNAL is coded as VPKTMBO. How is PROFILE coded in the same code language ? (1) PQQGDKH (2) PSQHFMJ (3) PSQDEMJ (4) NQOHFMJ (5) NQOHDKH SBI PO Prelim Exam, 09.06.2019

RBI GRADE–B/ NABARD GRADE–A OFFICER EXAMS 1. ‘Writing’ is related to ‘Pen’ in the same way as_____? (1) ‘Singing’ is related to ‘Music’ (2) ‘Stitching’ is related to ‘Needle’ (3) ‘Dancing’ is related to ‘Exercise’ (4) ‘Carpenter’ is related to ‘tools’ (5) ‘Paper’ is related to ‘book’ (RBI Grade-B Officer Exam. 06.02.2011)

Directions (2–3) : Study the following information and answer the given questions : (RBI Grade-B Officer’s Exam. 18.12.2011)

‘PROJECT’ is related to ‘CEOPR T’, ‘PLANE’ is related to ‘ELNP' and ‘WIN’ is related ‘NW’. 2. Following the same pattern, ‘ORGANISED’ will be related to_____ (1) ADEGOIRS (2) ADEGIOSR (3) ADEGIORS (4) ADEGBORS (5) Cannot be determined 3. Following the same pattern ‘DISTANCE’ will be related to_____ (1) ACDEINST (2) ACDEINS (3) ACDINST (4) ACDNSTI (5) Cannot be determined

ANALOGY

INSURANCE EXAMS Directions (1–5) : In each of the following questions, there is a certain relation between two given numbers on one side of :: and one number is given on a another side of :: while another number is to be found from the given alternatives, having the same relation with this number as the numbers of the given bear. Choose the best alternative. (Oriental Insurance Company Exam.08.04.2012)

1. 11 : 145 :: 14 : ? (1) 155 (2) 198 (3) 226 (4) 238 2. 3 : 36 :: 5 : ? (1) 130 (2) 138 (3) 145 (4) 148 3. 12 : 5 :: ? : 27 (1) 78 (2) 81 (3) 83 (4) 87 4. 6 : 108 :: 8 : ? (1) 220 (2) 238 (3) 278 (4) 320 5. 8 : 48 :: 14 : ? (1) 151 (2) 168 (3) 171 (4) 188 Directions (6–9) : Match the questions given below with (1), (2), (3) and (4) on the basis of relationship among items. The items may not be in the same order. (United India Insurance AAO Exam.03.06.2012)

6. Face : Nose : Mouth (1) Beverage : Coffee : Tea (2) Oak : Pine : Eucalyptus (3) Mountain : Snow : Forest (4) Accident : Injury : Pain 7. Evaporation : Cloud : Rain (1) Beverage : Coffee : Tea (2) Oak : Pine : Eucalyptus (3) Mountain : Snow : Forest (4) Accident : Injury : Pain 8. Physics : Chemistry : Botany (1) Beverage : Coffee : Tea (2) Oak : Pine : Eucalyptus (3) Mountain : Snow : Forest (4) Accident : Injury : Pain 9. Insect : Mosquito : Flies (1) Beverage : Coffee : Tea (2) Oak : Pine : Eucalyptus (3) Mountain : Snow : Forest (4) Accident : Injury : Pain

10. What is ralated to Lapse in the same way as Session is related to Conclude ? (1) Agency (2) Policy (3) Permit (4) Leave (United India Insurance AAO Exam.03.06.2012)

11. If is related to Condition in the same way as But is related to (1) Disapproval (2) Negation (3) Contradiction (4) Strengthen (United India Insurance AAO Exam.03.06.2012)

12. Clock is related to Time in the same way as Vehicle is related to which of the following ? (1) Road (2) Journey (3) Fuel (4) Passenger (United India Insurance AAO Exam.03.06.2012)

13. Bread is related to Wheat in the same way as Brick is related to (1) Clay (2) Building (3) Fire (4) Cement (United India Insurance AAO Exam.03.06.2012)

14. Drummer is related to Orchestra in the same way as Minister is related to (1) Voter (2) Constituency (3) Cabinet (4) Department (United India Insurance AAO Exam.03.06.2012)

Directions (15–19) : In each of the following questions, choose that set of numbers from the four alternative sets, that is similar to the given set. (United India Insurance AAO Exam.03.06.2012)

15. Given set : (91, 77, 143) (1) (109, 99, 119) (2) (117, 63, 187) (3) (119, 99, 187) (4) (169, 121, 127) 16. Given set : (826, 574, 952) (1) (764, 386, 854) (2) (842, 654, 928) (3) (576, 478, 754) (4) (682, 196, 934)

BPRE–39

17. Given set : (112, 136, 162) (1) (65, 84, 105) (2) (90, 128, 136) (3) (36, 52, 70) (4) (96, 102, 162) 18. Given set : (48, 84, 128) (1) (33, 65, 105) (2) (36, 70, 112) (3) (22, 52, 90) (4) (56, 96, 144) 19. Given set : (208, 439, 945) (1) (329, 515, 738) (2) (417, 658, 705) (3) (528, 645, 913) (4) (538, 725, 813) Directions (20–23) : Match the questions given below with (1), (2), (3) and (4) on the basis of relationship among items. The items may not be in the same order. (United India Insurance AAO Exam.03.06.2012)

20. Face : Nose : Mouth (1) Beverage : Coffee : Tea (2) Oak : Pine : Eucalyptus (3) Mountain : Snow : Forest (4) Accident : Injury : Pain 21. Evaporation : Cloud : Rain (1) Beverage : Coffee : Tea (2) Oak : Pine : Eucalyptus (3) Mountain : Snow : Forest (4) Accident : Injury : Pain 22. Physics : Chemistry : Botany (1) Beverage : Coffee : Tea (2) Oak : Pine : Eucalyptus (3) Mountain : Snow : Forest (4) Accident : Injury : Pain 23. Insect : Mosquito : Flies (1) Beverage : Coffee : Tea (2) Oak : Pine : Eucalyptus (3) Mountain : Snow : Forest (4) Accident : Injury : Pain 24. What is ralated to Lapse in the same way as Session is related to Conclude ? (1) Agency (2) Policy (3) Permit (4) Leave (United India Insurance AAO Exam.03.06.2012)

25. If is related to Condition in the same way as But is related to (1) Disapproval (2) Negation (3) Contradiction (4) Strengthen (United India Insurance AAO Exam.03.06.2012)

ANALOGY 26. Clock is related to Time in the same way as Vehicle is related to which of the following ? (1) Road (2) Journey (3) Fuel (4) Passenger (United India Insurance AAO Exam.03.06.2012)

27. Bread is related to Wheat in the same way as Brick is related to (1) Clay (2) Building (3) Fire (4) Cement (United India Insurance AAO Exam.03.06.2012)

28. Drummer is related to Orchestra in the same way as Minister is related to (1) Voter (2) Constituency (3) Cabinet (4) Department (United India Insurance AAO Exam.03.06.2012)

Directions (29–33) : In each of the following questions, choose that set of numbers from the four alternative sets, that is similar to the given set. (United India Insurance AAO Exam.03.06.2012)

29. Given set : (91, 77, 143) (1) (109, 99, 119) (2) (117, 63, 187) (3) (119, 99, 187) (4) (169, 121, 127) 30. Given set : (826, 574, 952) (1) (764, 386, 854) (2) (842, 654, 928) (3) (576, 478, 754) (4) (682, 196, 934) 31. Given set : (112, 136, 162) (1) (65, 84, 105) (2) (90, 128, 136) (3) (36, 52, 70) (4) (96, 102, 162) 32. Given set : (48, 84, 128) (1) (33, 65, 105) (2) (36, 70, 112) (3) (22, 52, 90) (4) (56, 96, 144) 33. Given set : (208, 439, 945) (1) (329, 515, 738) (2) (417, 658, 705) (3) (528, 645, 913) (4) (538, 725, 813)

34. Select the missing characters from the given responses ?

SHORT ANSWERS

BADC : FHEG :: NPMO : ? (1) RQTS (2) NTRC (3) TRQS (4) RTQS 35. Select the missing characters from the given responses.

NATIONALISED BANKS & IBPS PO/MT/SO EXAMS

AGMSY, CIOUA, EKQWC, ?, IOUAG, KQWCI (1) GMSYE (2) FMSYE (3) GNSYD (4) FMYES

1. (4)

2. (1)

3. (5)

4. (4)

5. (1)

6. (1)

7. (3)

8. (4)

9. (3)

10. (2)

SBI PO EXAMS

(NICL (GIC) AO (Finance) Exam, 08.09.2013)

36. Select the related word from the given alternatives. Physician : Treatment : : Judge : ? (1) Punishment (2) Judgement (3) Lawyer (4) Court 37. Select the related letters from the given alternatives. ABCD : WXYZ : : EFGH : ? (1) STUV (2) ZYXW (3) VUTS (4) WXZY 38. Select the related word from the given alternatives. SAFE : SECURE : : PROTECT : ? (1) guard (2) lock (3) sure (4) conserve 39. Select the related letters from the given alternatives. MASTER : OCUVGT : : LABOUR : ? (1) NCDQWT (2) NDERWT (3) NBCRWT (4) NEDRWT (NICL (GIC) Administrative Officer Exam, 08.09.2013)

40. Influenza : Virus :: Typhoid:? (1) Bacillus (2) Parasite (3) Protozoa (4) Bacteria 41. Two items or words bear a particular relation with each other. Third item bears the same relation (which first and the second bear) with one of the answer choices. The candidates are to pick up that word or item. Degree Relationship Warm : Hot : : (1) Glue : Paste (2) Bright: Genius (3) Climate : Weather (4) Leaders : People (NICL (GIC) Administrative Officer Exam, 15.12.2013)

BPRE–40

1. (2)

RBI GRADE–B/ NABARD GRADE–A OFFICER EXAMS 1. (2)

2. (3)

3. (5)

INSURANCE EXAMS 1. (3)

2. (3)

3. (1)

4. (4)

5. (2)

6. (3)

7. (4)

8. (2)

9. (1)

10. (2)

11. (3)

12. (2)

13. (1)

14. (3)

15. (3)

16. (4)

17. (3)

18. (2)

19. (3)

20. (3)

21. (4)

22. (2)

23. (1)

24. (2)

25. (3)

26. (2)

27. (1)

28. (3)

29. (3)

30. (4)

31. (3)

32. (2)

33. (3)

34. (1)

35. (1)

36. (2)

37. (1)

38. (1)

39. (1)

40. (4)

41. (2)

EXPLANATIONS

NATIONALISED BANKS & IBPS PO/MT/SO EXAMS (1-3): –1 –1 –1 T R A V E L

U D K U S B +1 +1 +1

ANALOGY 5. (1) –1 –1 –1 C O R N E R

M O T L E Y M D Q D P S +1 +1 +1

G R O U P S

1. (4)

S U R V E Y

+7

T O R H S P +1 +1 +1

6. (1)

–1 –1 –1

T U D X T V S +1 +1 +1

E

L

+7

The code has been generated on the following pattern :

P

O

R

A

L

+1 D

S

M

B

Code of Vowel = Opposite Letter Therefore,

S

P

H O S T E L

O L Z A V S +7

Therefore, +7

2. (1)

C

G R O U P S

O

N

S

I

D

–1

–1 –1 –1

R T O R H S P +1 +1 +1

M

N

B

S

+7

F

E

J

9. (3)

I M P O S E

L E A G U E

N H E I X I +2 +3 +4 +2 +3 +4

H N O P R F –1 +1 –1 +1 –1 +1

–1 –1 –1

T H X C L P +2 +3

U D K U S B +1 +1 +1

+4 +2 +3 +4

A R D E N T

Z S C F M U –1 +1 –1

Therefore,

+1

D A M A G E

–1 –1 –1 C O R N E R

+7

R

7. (3)

R E T A I L

T R A V E L

E

+1

3. (5) When it is hot, fan circulates air to keep us cool. Similarly, we eat food to satiate our hunger. (4-5) :

F D Q C J I

–1

+2 +3 M D Q D P S +1 +1 +1

+1

+4 +2

+3 +4

P O M A D E

–1 –1 –1

Therefore, H O R D E S

8. (4)

4. (4)

S U R V E Y

+7

Consonant + 7 = Code

M

–1 O

T L A S V F +7

–1 –1 –1

W L T Z K V +7

–1 +1 –1

+7

U D X T V S +1 +1 +1

G P Q E D T

+1 +7

–1 +1

BPRE–41

ANALOGY 10. (2)

R I G H T

RBI GRADE–B/ NABARD GRADE–A OFFICER EXAMS

U G J K W +3 –2 +3 +3 +3

1. (2) Pen is used to write something. Similarly, stitching is done by needle.

The code has been generated on the following pattern : Consonant + 3 ⇒ Code

P R O J E C T

The middle letter is deleted and the remaining letters are arranged according to the English alphabetical series.

Vowel –2 ⇒ Code

H O U S E

(2 –3) :

K M S V C +3 –2 –2 +3 –2

C E O P R T P L A N E ⇒E L NP W I N ⇒NW

2. (3) O R G A N I S E D A

D E G

I

O R S

3. (5) D I S T A N C E

S M G Q W +3 –2 –2 +3 +3

P O I N T

There is no middle letter.

INSURANCE EXAMS 1. (3) 11 × 13 = 143; 143 + 2 = 145 Similarly, 14 × 16 = 224; 224 + 2 = 226

SBI PO EXAMS

2. (3) 36 ⇒ 3 × 6 = 18

1. (2)

F

O

R

+2

+1 S J

P O

U

R N +2

P

K

N

E

+1

V F

G

+1 V

T U

M

8. (2) Physics, Chemistry and Botany are various branches of science. Similarly, Oak, Pine and Eucalyptus are different types of tree. 9. (1) Mosquito and Flies are insects. Similarly, Coffee and Tea are beverages.

Similarly, when a policy is ceased to be in operation it is said that policy has been lapsed. 11. (3) If denotes condition. Similarly, but denotes contradiction. 12. (2) Clock shows time. Clock is used to know the time. Similarly, Vehicle is used for journey. 13. (1) Wheat is the raw material of bread. Bread is made up of wheat. Similarly, brick is made up of clay. 14. (3) Drummer is an integral part of an orchestra. Similarly, Minister is an integral part of Cabinet. 15. (3) Given set

18 is completely divisible by 3.

13 × 7 = 91;

Similarly,

7 × 11 = 77;

145 ⇒ 1 × 4 × 5 = 20

11 × 13 = 143

20 is completely divisible by 5.

Similarly, 119 = 17 × 7;

O

A

V

15 =3 3. (1) 12 + 3 = 15; 5

L

78 + 3 = 81;

B

81 =3 27

O 4. (4)

Therefore,

9 × 11 = 99; 11 × 17 = 187 16. (4) Given set

Similarly,

+1

T

7. (4) Clouds are formed by evaporation of water and clouds cause rain. Similarly, accident causes injury and in turn, injury is painful.

10. (2) When the session ends it is said that session is concluded.

Thus,

Therefore,

6. (3) Nose and mouth are parts of face. Similarly, we find forest and snow on mountains.

108 = 18 6

8 + 2 + 6 = 16 5 + 7 + 4 = 16 9 + 5 + 2 = 16 Similarly, 6 + 8 + 2 = 16 1 + 9 + 6 = 16

P

R

O

+2

+1 P

F

S

Q

I

L

E

+1

H F

M

J

320 = 40 8 5. (2) 8 × 6 = 48 Similarly, 14 × 12 = 168

BPRE–42

9 + 3 + 4 = 16 17. (3) Given set 112 + 24 = 136 136 + 26 = 162

ANALOGY Similarly, 36 + 16 = 52 52 + 18 = 70 18. (2) Given set 48 + 36 = 84 84 + 44 = 128 Similarly, 36 + 34 = 70 70 + 42 = 112 19. (3) Given set

119 = 17 × 7; G +2 I +2 K +2 M +2 O +2 Q +2 S +2 U +2 W +2 +2 +2 +2 Y A C

30. (4) Given set 8 + 2 + 6 = 16 5 + 7 + 4 = 16

9 + 3 + 4 = 16 112 + 24 = 136

913 – 645 = 268

136 + 26 = 162

117 × 2 + 34 = 234 + 34 = 268

Similarly,

24. (2) When the session ends it is said that session is concluded. Similarly, when a policy is ceased to be in operation it is said that policy has been lapsed. 25. (3) If denotes condition. Similarly, but denotes contradiction. 26. (2) Clock shows time. Clock is used to know the time. Similarly, Vehicle is used for journey.

37. (1) A B C D

M S Y E

+2 +2 +2 +2 +2

I O U A G

+2 +2 +2 +2 +2

K Q W C I

W X Y Z

Pairs of opposite Letters Therefore,

36 + 16 = 52 52 + 18 = 70

E F G H

32. (2) Given set

S T U V

48 + 36 = 84 84 + 44 = 128 Similarly, 36 + 34 = 70

38. (1) Safe means protected from danger and harm, secure. Protect means to guard, to keep safe from harm, injury. 39. (1)

70 + 42 = 112 33. (3) Given set 439 – 208 = 231; 945 – 439 = 506 231 × 2 + 44 = 462 + 44 = 506

M A S T E R

Similarly, 645 – 528 = 117;

+2 +2 +2 +2

913 – 645 = 268 117 × 2 + 34 = 234 + 34 = 268 34. (1)

B A D C

F H E G +3 +4 +5

Similarly,

R Q T S +3

29. (3) Given set 13 × 7 = 91; 7 × 11 = 77; 11 × 13 = 143

+4 +4 +5

BPRE–43

Similarly, L A B O UR

+4

N P M O

O C U V G T +2 +2

27. (1) Wheat is the raw material of bread. Bread is made up of wheat. Similarly, brick is made up of clay. 28. (3) Drummer is an integral part of an orchestra. Similarly, Minister is an integral part of Cabinet.

G

31. (3) Given set

645 – 528 = 117;

erages

E

36. (2) The work of physician is treatment. Similarly, the work of Judge is to deliver judgement.

Similarly,

231 × 2 + 44 = 462 + 44 = 506

23. (1) Mosquito and Flies are insects. Similarly, Coffee and Tea are bev-

C

9 + 5 + 2 = 16

1 + 9 + 6 = 16

22. (2) Physics, Chemistry and Botany are various branches of science. Similarly, Oak, Pine and Eucalyptus are different types of tree.

+2

11 × 17 = 187

6 + 8 + 2 = 16

21. (4) Clouds are formed by evaporation of water and clouds cause rain. Similarly, accident causes injury and in turn, injury is painful.

+2

A

945 – 439 = 506

20. (3) Nose and mouth are parts of face. Similarly, we find forest and snow on mountains.

+2

9 × 11 = 99;

439 – 208 = 231;

Similarly,

35. (1)

Similarly,

N C D Q W T +2 +2 +2 +2 +2 +2

40. (4) The causative agent of influenza is virus. Similarly, the causative agent of typhoid is bacteria. 41. (2) Second is more intense form ❐❐❐ of the first.

ANALOGY

MODEL EXERCISES 1. ‘Offhand’ is related to ‘Premeditation’ in the same way as ‘above board’ is related to (1) Integrity (2) Honesty (3) Guide (4) Competition (5) None of these 2. 5 : 7 : : ? : 28 (1) 20 (2) 14 (3) 56 (4) 26 (5) None of these Directions (3-7) : Fill in the blanks in these questions from among the alternatives (1) to (4) 3. A D G D I N I P ... (1) V (2) W (3) X (4) Y (5) None of these 4. B E I N T ... (1) A (2) S (3) U (4) V (5) None of these 5. Z W S P L I E ... (1) D (2) F (3) K (4) B (5) None of these 6. ABA : ECE : : ...... : ...... (1) LML : NON (2) IDI : OFO (3) PQP : STS (4) CDC : GEG (5) None of these 7. CG : EI : : FJ : ... (1) LM (2) IJ (3) GK (4) JK (5) None of these Directions (8-9) : From the sets of numbers in the four alternatives given below, which is the most similar to the given set. 8. Given Set : (6, 15, 28) (1) (50, 59, 71) (2) (46, 56, 66) (3) (60, 69, 72) (4) (60, 69, 82) (5) None of these 9. Given Set : (81, 77, 69) (1) (56, 52, 44) (2) (64, 61, 53) (3) (92, 88, 79) (4) (75, 71, 60) (5) None of these Directions (10-14) : There is a blank space in each of these questions in which only one of the four alternatives given under each question satisfies the same relationship as is

found between the two terms on the other side of the sign : :. Find the correct alternative to fill in the blank space. 10. Shoe : Leather : : (1) Bus : Conductor (2) Train : Wagon (3) Highway : Asphalt (4) Medicine : Doctor (5) None of these 11. Condone : Offence : : (1) Overlook : Aberration (2) Error : Omission (3) Mitigate : Penitence (4) Conviction : Criminal (5) None of these 12. Dinosaur : Dragon : : (1) Evolution : Revelation (2) Gorilla : Soldier (3) Snow : Ice (4) Primeval : Medieval (5) None of these 13. Deterioration : : Rust : : (1) Recession : Inefficiency (2) Depression : Unemployment (3) Promulgation : Legislation (4) Iron : Water (5) None of these 14. Telephone : Ring :: (1) Door : Knock (2) Gate : Open (3) Door : Wood (4) Lock : Key (5) None of these Directions (15-16) : Find the set which is most similar to the given set. 15. Given set : (6, 15, 28) (1) (46, 56, 66) (2) (50, 59, 71) (3) (60, 69, 72) (4) (60, 69, 82) (5) None of these 16. Given set: (81, 77, 69) (1) (64, 61, 53) (2) (56, 52, 44) (3) (92, 88, 79) (4) (75,71, 60) (5) None of these 17. Three of the following four are alike in a certain way and so form a group. Which one does not belongs to that group ? (1) – (2) + (3) % (4) = (5) None of these Directions (18-19) : Answer the question independent of each other. 18. Off-hand is related to PERFUNCTORY in the same away as Above Board is related to

BPRE–44

(1) guide (2) honesty (3) integrity (4) competition (5) None of these 19. Which one is different from the remaining three? (1) GIJK (2) DFGH (3) CEFG (4) ABCD (5) None of these 20. Three of the following four are alike in certain way and so form a group. Which is one that does not belong to the group? (1) 71 (2) 73 (3) 77 (4) 67 (5) None of these 21. Unscramble the letters in the given words and find the odd one out. (1) UMRSME (2) EIWNTR (3) PIGRSN (4) LCUOD (5) None of these Directions (22-25) : Each of these questions consists of a pair of words. Establish the relationship among the given pair and identify the pair that best illustrates a similar relationship from amongest the alternatives. 22. Kick : Football (1) Mop : Sweep (2) Dust : Rage (3) Wash : Dishes (4) Throw : Ring (5) None of these 23. Rectangle : Pentagon (1) Triangle : Rectangle (2) Diagonal : Perimeter (3) Side : Angle (4) Circle : Square (5) None of these 24. Inherit : Acquire (1) Instinct : Habbit (2) Learn : Discover (3) Loss : Gain (4) Hierarchial : Succession (5) None of these 25. Oak : Coniferous (1) Fish : sea (2) Animal : Carnivore (3) Tree : Grove (4) Chimpanzee : Ape (5) None of these Directions (26–29) : Each of these questions consists of a pair of words bearing a certain relationship. From amongst the given alternatives, pick up the pair that best illustrates a similar relationship.

ANALOGY 26. Presumption : Certainty (1) Falsehood : Truth (2) Hearsay : Authenticity (3) Theorem: Proof (4) Hunch: Guess (5) None of these 27. Coal : Thermal (1) Power : Energy (2) Bulb : Light (3) Air : Atmosphere (4) Water : Hydel (5) None of these 28. Court: Justice (1) Police : Crime (2) Teacher : Study (3) Doctor : Sickness (4) Auditor : Accuracy (5) None of these 29. Relaxation : Work (1) Play : Cheat (2) Lunch : Dinner (3) Smile : Laugh (4) Fresh : Stale (5) None of these 30. Select the related letters from the given alternatives. ABCD : WXYZ : : EFGH : ? (1) STUV (2) ZYXW (3) VUTS (4) WXZY 31. Select the related word from the given alternatives. SAFE : SECURE : : PROTECT :? (1) guard (2) lock (3) sure (4) conserve 32. Select the related letters from the given alternatives. MASTER : OCUVGT : : LABOUR : ? (1) NCDQWT (2) NDERWT (3) NBCRWT (4) NEDRWT 33. Influenza : Virus :: Typhoid:? (1) Bacillus (2) Parasite (3) Protozoa (4) Bacteria 34. Two items or words bear a particular relation with each other. Third item bears the same relation (which first and the second bear) with one of the answer choices. The candidates are to pick up that word or item. Degree Relationship Warm : Hot : : (1) Glue : Paste (2) Bright: Genius (3) Climate : Weather (4) Leaders : People

35. Find out the number at the place of question mark 6:5::8:? (1) 2 (2) 4 (3) 6 (4) 10 36. Select the realated number from the given alternatives : M×N : 13×14 : F×R : ? (1) 14×15 (2) 5×17 (3) 6×18 (4) 7×19 37. Select the related number from the given alternatives. 16 : 56 :: 32 : ? (1) 96 (2) 112 (3) 120 (4) 128 38. In a certain code language, INVERSE is written as ZRMKIWV. How will MAGICAL be written in that code language? (1) JDPNODF (2) LFRPQFH (3) KEPNOEG (4) LFQOPFH (5) KEQOPEG 39. In a certain code language PURSUE is written as NXPQXH and FIESTA is written as DLHQRD. How will SCREAM be written in that code language ? (1) QAPCYK (2) QAPBXJ (3) PZOBXJ (4) QAPHDK (5) VFUCYP 40. In a certain code language, GOLDEN is written as JLOGVQ and GLOBES is written as JOLEVV. How will SEASON be written in that code language ? (1) VVZVLQ (2) VHDVRQ (3) WIEVSR (4) VEZVRK (5) VZVVQL

SHORT ANSWERS 1.(4)

2.(4)

3.(2)

5.(2)

6.(4)

7.(5)

8.(4)

9.(1)

10.(2)

11.(1)

12.(3)

13.(2)

14.(1)

15.(4)

16.(2)

17.(3)

18.(2)

19.(4)

20.(3)

21.(4)

22.(4)

23.(1)

24.(1)

25.(1)

26.(2)

27.(4)

28.(2)

29.(4)

30.(1)

31.(1)

32.(1)

33.(4)

34.(2)

35.(3)

36.(3)

37.(2)

38.(5)

39.(4)

40.(1)

BPRE–45

4.(1)

EXPLANATIONS 1. (4) Above board is related to competition. 2. (4) 5 : 7 : : x : 28 x = 26 –2

–2

+3 +3 3. (2) A ¾ ¾ ¾® D ¾ ¾ ¾® G +5 +5 D ¾¾ ¾® I ¾ ¾ ¾® N +7 +7 I ¾¾ ¾® P ¾ ¾ ¾® W

4. (1) The sequence is following the pattern like + 3, + 4, + 5, + 6, + 7. 5. (2) The sequence is moving backword in alphabetical order with a difference – 3, – 4, – 5, – 6, ....... and so on. 6. (4) In ABA, middle letter B is + 1 of A in alphabetical order whereas in ECE, middle letter C is –1 of letter E in alphabetical order. 7. (5) CG : EI : : FJ : HL +2 +2

+2 +2

8. (4) 6 + 9 = I5, I5 + 13 = 28 60 + 9 = 69, 69 + 13 = 82 9. (1) 81–4 = 77,77–8 = 69 56–4 = 52,52–8 = 44 10. (2) Shoe is made by leather and Train is made by wagon. 11. (1) Condone and offence have a contradictory relationship as an offence cannot be condoned. The same relationship is exhibited by relationship Overlook : Aberration. 12. (3) Dinosaur and Dragon fall in the same category as Snow and Ice. 13. (2) Deterioration is caused by rusting as depression is caused by unemployment. 14. (1) Ring is related with telephone and knock is related with door. 15. (4) The difference between first and second numbers is 9 and that between second and third numbers is 13. 16. (2) The difference between first and second numbers is 4 and that between second and third number is 8. 17. (3) – , + and = are mathematical operators while % is not. 18. (2) According to question Board is related to honesty 19. (4) In others, there is a gap of one letter between the first and the second letters of the group.

ANALOGY 20. (3) 77 does not belong to that group because others are prime number while 77 is not a prime number. 21. (4) LCUOD is odd one because other letters when rearranged provide name of seasons like summer, winter and spring. 22. (4) Kick is related to football same as throw is related to Ring 23. (1) Rectangle is a four sided figure while pentagon is a five sided figure, ie, one side more. Similarly, triangle has one more side than rectangle. 24. (1) Inherit is related to Acquire same as Instinct is related to Habbit 25. (1) Oak is related to coniferous same as Fish is related to sea. 26. (2) Presumption (Noun) means “the action of supposing something to be true”. Certainty (Noun) means “a thing that is certain.” Thus, Presumption is a lower degree word than that of Certainty. Hearsay (Noun) means “things one has heard from another person, which may or may not be true.” Authenticity (Noun) means “the quality of being authentic”. 27. (4) Coal is used to generate thermal power. Similarly, water is used to produce hydel power. 28. (2) Court is a place where trials or other law cases are held and justice is delivered. Similarly, teacher facilitates study. 29. (4) Relaxation is the antonym of work. Similarly, fresh is the antonyms of stale. 30. (1) A B C D

W X Y Z

32. (1)

39. (4)

M A S T E R

O C U V G T +2 +2

P U R S U E

N X P Q X H –2 +3 –2 –2 +3 +3

F I E S T A

D L H Q R D –2 +3 +3 –2 –2 +3

+2 +2 +2 +2

Similarly, L A B O UR

N C D Q W T +2 +2 +2 +2 +2 +2

33. (4) The causative agent of influenza is virus. Similarly, the causative agent of typhoid is bacteria. 34. (2) Second is more intense form of the first. 35. (3)

6 3;3+2=5 2

The code has been generated on the following pattern : Consonant – 2 = Code Vowel + 3 Code Therefore,

S C R E A M

Q A P H D K –2 –2 –2 +3 +3 –2

8 4;4+2=6 2 36. (3) M × N

13 × 14

Position number in the English alphabetal series. Therefore, F × R 6 × 18

40. (1)

J L O G V Q

G O L D E N +3

+3 16 37. (2) 16 × 3 + 2 ⇒ 48 + 8 = 56 Similarly,

+3 J O L E V V

G L O B E S +3

32 2 ⇒ 96 + 16 = 112 38. (5) 32 × 3 +

Pairs of opposite Letters Therefore, E F G H

S T U V

I

N

V

+4 Z

+3

E

M

R

S

E

+4

K

I

W

V

Therefore, 31. (1) Safe means protected from danger and harm, secure. Protect means to guard, to keep safe from harm, injury.

M

A

I

G

C

+6 E

The code has been generated on the following pattern : Consonant + 3 = Code Vowel = Opposite letter Therefore, V V Z V L Q

S E A S O N +3

+4 K

+3 +3

+6 R

+3

Q

O

A

L

+4 P

BPRE–46

+3 +3

E

G

❑❑❑

CLASSIFICATION

4

CLASSIFICATION

QUESTIONS FROM 1999 TO 2010 ARE AVAILABLE ONLINE NATIONALISED BANKS & IBPS PO/MT/SO EXAMS Directions (1– 2) : Four of the following five are alike in a certain way and hence form a group. Which is the one that does not belong to the group? (Bank Of Baroda PO Exam, 13.03.2011)

1. (1) Asia (2) Antarctica (3) Africa (4) Alaska (5) Australia 2. (1) Trophy (2) Medal (3) Certificate (4) Gift (5) Prize Directions (3–5) : Four of the following five are alike in a certain way based upon the order in which these appear in the English alphabetical series and hence form a group. Which is the one that does not belong to that group? (Bank Of Baroda PO Exam. 13.03.2011)

3. (1) MPQO (2) DGHF (3) RUTV (4) VYZX (5) JMNL 4. (1) AMCL (2) WIVK (3) GVIU (4) XDZC (5) HRJQ 5. (1) AZMN (2) BYEF (3) CXUV (4) FUNO (5) IRDV Directions (6–7) : Four of the following five are alike in a certain way and so form a group. Which is the one that does not belong to that group? (Bank Of Baroda PO Exam. 13.03.2011)

6. (1) BAH (2) EDG (3) CBF (4) HGA (5) DFE 7. (1) ED (2) DC (3) BA (4) EG (5) CB Directions (8-12) : Four of the following five are alike in a certain way and hence form a group. Which is the one that does not belong to the group? (Q.11–13 are based upon the English alphabetical series) (Allahabad Bank PO Exam. 17.04.2011)

8. (1) (3) (5) 9. (1) (3) (5)

Succeed Triumph Win Fair Indifferent Just

(2) Victory’ (4) Compete (2) Impartial (4) Unbiased

10. (1) KP (2) BY (3) DW (4) HU (5) GT 11. (1) JLNK (2) TVXU (3) ACEB (4) PRTQ (5) GJKH 12. (1) GIJF (2) OQRN (3) KMNL (4) UWXT (5) CEFB Directions (13–15) : Four of the following five are alike in a certain way (based on the English alphabetical series) and hence form a group. Which is the one that does not helong to that group? (Indian Overseas Bank PO Exam. 22.05.2011)

13. (1) ABX (2) ABCDV (3) ABCDEFT (4) ABCW (5) ABCDES 14. (1) DNFL (2) KRIT (3) HUJS (4) BQDO (5) JWLU 15. (1) CXIG (2) AZLJ (3) DWVT (4) BZQO (5) GUKI Directions (16–18) : Four of the following five are alike in a certain way and hence form a group. Which is the one that does not belong to the group ? (Indian Overseas Bank PO Exam. 22.05.2011)

16. (1) Eye (2) Kidney (3) Heart (4) Lungs (5) Ear 17. (1) Unique (2) Exclusive (3) Rare (4) Distinct (5) Extinct 18. Four of the following five are alike in a certain way and hence form a group. Which of the following does not belong to that group ? (1) Think (2) Reflect (3) Speak (4) Consider (5) Ponder (IBPS RRBs Office Assistant CWE Exam. 09.09.2012)

Directions (19-21) : Four of the following five are alike in a certain way (based on the position of alphabets in the English alphabetical series) and hence form a group. Which is the one that does not belong to that group? (IDBI Bank Officer Exam.16.09.2012)

BPRE–47

19. (1) KLNI (2) UVXS (3) CDFA (4) FGID (5) QROT 20. (1) ABCDEFGR(2) ABX (3) ABCDEU (4) ABCW (5) ABCDEFT 21. (1) DEI (2) FGM (3) BCE (4) IJR (5) EFK 22. Four of the following five are alike in a certain way based on the English alphabetical order and so form a group. Which is the one that does not belong to that group ? (1) RVXU (2) TXZW (3) KOPN (4) DHJG (5) LPRO (Bank of Maharashtra PO Exam, 26.10.2016)

23. Four of the following five are alike in a certain way (based on their positions of alphabets in the English alphabetical series) and hence form a group. Which is the one that does not belong to that group? (1) BFD (2) EIH (3) KOM (4) TXV (5) LPN (IBPS RRBs Officer CWE (Pre.) Exam, 14.11.2016 (Shift-II))

24. Four of the following five are alike in a certain way (based on their positions of alphabet in the English alphabetical series) and hence form a group. Which is the one that does not belong to that group? (1) RQT (2) NPL (3) FHD (4) KMI (5) WYU (IBPS RRBs Officer CWE (Pre.) Exam, 14.11.2016 (Shift-III))

25. Four of the following five are alike in a certain way as per the English alphabetical order and so they form a group. Which is the one that does not belong to that group? (1) HFBD (2) PNJL (3) XWTV (4) OMIK (5) YWSU (IBPS Bank PO/MT CWE (Prelim Exam) 14.10.2018)

CLASSIFICATION 26. Four of the following five are alike in a certain way based on the English alphabetical order and so they form a group. Which is the one that does not belong to that group ? (1) RVXU (2) TXZW (3) KOPN (4) DHJG (5) LPRO (Canara Bank PO Exam, 23.12.2018)

27. Four of the following five are alike in a certain way based on their positions in the English alphabetical order and thus they form a group. Identify the one which does not belong to the group. (1) ABZY (2) JKQP (3) LMPN (4) DEWV (5) GHTS (Indian Bank Specialist Officer SO Exam, 08.03.2020)

SBI PO EXAMS Directions (1-3) : Four of the following five are alike in a certain way and hence form a group. Which is the one that does not belong to the group? (SBI Associate Banks PO Exam. 07.08.2011)

1. (1) Break (2) Change (3) Split (4) Divide (5) Separate 2. (1) Train (2) Instruct (3) Educate (4) Advocate (5) Coach 3. (1) Extend (2) Higher (3) Upward (4) Rise (5) Ascend 4. Four of the following five are alike in a certain way and hence form a group. Which is the one that does not belong to the group ? (1) JN (2) CG (3) WZ (4) LP (5) PT (SBI Specialist Officer (Law Officer : MMGS Scale-II) Online Exam, 19.04.2014)

5. Four of the following five are alike in a certain way and hence form a group. Which is the one that does not belong to the group ? (1) PTSQ (2) UYXV (3) INMJ (4) KONL (5) DHGE (SBI Specialist Officer (Law Officer : MMGS Scale-II) Online Exam, 19.04.2014)

Directions (6–7) : Choose the odd one out from the given set of words or letters (SBI Assistant Manager (System) Exam, 17.01.2016)

6. (1) SXV (3) GLJ (5) RWU

(2) AFD (4) ORT

7. (1) ONRQ (2) HIKL (3) UTXW (4) BAED (5) KJNM 8. Four of the following five are alike in a certain way based on the English alphabetical series, and hence they form a group. Which one of the following does not belong to the group? (1) QSZ (2) IGM (3) DBH (4) HFL (5) TRX SBI PO (Prelim Exam), 08.07.2018 (Shift-II))

9. Four of the following five are alike in a certa in way based on their positions in the English alphabetical order and hence they form a group. Which one of the following does not belong to the group? (1) JHN (2) PRZ (3) IGM (4) ECI (5) USY SBI PO (Prelim Exam), 14.06.2019

RBI GRADE–B/ NABARD GRADE–A OFFICER EXAMS Directions (1–2) : Four of the following five are alike in a certain way and hence form a group. Which is the one that does not belong to the group?

INSURANCE EXAMS Directions (1-5) : Choose the odd numerical pair in each of the following questions : (Oriental Insurance Company Exam.08.04.2012)

1. (1) 13 – 22 (2) 24 – 76 (3) 16 – 52 (4) 17 – 62 2. (1) 6 – 16 (2) 18 – 48 (3) 21 – 56 (4) 27 – 76 3. (1) 39 – 77 (2) 51 – 119 (3) 33 – 88 (4) 52 – 91 4. (1) 26 – 4 (2) 226 – 14 (3) 274 – 16 (4) 82 – 8 5. (1) 2 – 4 (2) 4 – 8 (3) 6 – 18 (4) 8 – 32 Directions (6–9) : In each of these questions some numbers are given in rows (1), (2), (3) and (4). In three rows the numbers are ralated to each other, while the numbers in one row hold no relation. Identify that row. (United India Insurance AAO Exam.03.06.2012)

1 , 3, 4 2 (2) 6, 15, 6, 8

6. (1) 3, 7

(3) 1

(4) 2, 5, 2, 2

(RBI Grade-B Officer’s Exam. 18.12.2011)

1.

2.

3.

(1) Compare (2) Conflict (3) Distinguish (4) Contrast (5) Differentiate (1) Recollect (2) Retrieve (3) Regain (4) Recall (5) Realise Four of the following five are alike in a certain way (based on the English alphabetical series) and hence form a group. Which is the one that does not belong to that group ? (1) JOLN (2) TYWV (3) CHFE (4) NSQP (4) GLJI (Nabard Officer Grade ‘A’ Online Exam. 01.03.2015)

4. Four of the following five are alike in a certain way and so form a group. Which one of the following does not belong to the group? (1) KJIL (2) MLKN (3) DBCE (4) YXWZ (5) TSRU (RBI Assistant Manager Online Exam 25.03.2017)

BPRE–48

3 1 1 ,3 ,1 ,2 2 2 4

7. (1) 1

1 1 3 , , ,1 2 2 4

(2) 3, 1, 1 (3)

2 3

1 ,2 2

3 1 3 1 , , , 4 4 8 2

1 3 1 ,1 , 8 4 2 8. Four of the following five are alike in a certain way and hence form a group. Which is the one that does not belong to the group ? (1) SUXV (2) MORP (3) DFIG (4) CEGF (5) JLOM (4) 1,

(United India Insurance AAO Exam.03.06.2012)

9. Four of the following five are alike in a certain way and hence form a group. Which is the one that does not belong to the group?

CLASSIFICATION (1) KIF (3) HFL (5) MKH

(2) QOL (4) WUR

(United India Insurance AAO Exam.03.06.2012)

Directions (10–11) : In each of these questions some numbers are given in rows (1), (2), (3) and (4). In three rows the numbers are ralated to each other, while the numbers in one row hold no relation. Identify that row. (United India Insurance AAO Exam.03.06.2012)

1 , 3, 4 2 (2) 6, 15, 6, 8

10. (1) 3, 7

(3) 1

1 1 3 ,3 ,1 ,2 2 2 4

(4) 2, 5, 2, 2

11. (1) 1

1 1 3 , , ,1 2 2 4

(2) 3, 1, 1

(3)

2 3

1 ,2 2

3 1 3 1 , , , 4 4 8 2

1 3 1 ,1 , 8 4 2 Find the odd letters from the given alternatives. (1) DWHS (2) BYDW (3) CWFS (4) EVJQ (4) 1,

12.

(NICL (GIC) AO (Finance) Exam, 08.09.2013)

13.

Find the odd word from the given alternatives. (1) Talking (2) Walking (3) Sleeping (4) Running (NICL (GIC) AO (Finance) Exam, 08.09.2013)

14.

Find the odd word from the given alternatives. (1) Ladder (2) Staircase (3) Bridge (4) Escalator (NICL (GIC) AO (Finance) Exam, 08.09.2013)

15. Choose the pair in which the words are differently related from the rest. (1) Beans : Pulses (2) Rice : Cereals (3) Tea : Beverages (4) Legumes : Nodules (NICL (GIC) AO (Finance) Exam, 15.12.2013)

16. Choose the word which is different from the rest. (1) Wool (2) Honey (3) Silk (4) Wax (NICL (GIC) AO (Finance) Exam, 15.12.2013)

17. Choose the word which is different from the rest. (1) Acharya Kripalani (2) C.V. Raman (3) Rajagopalachari (4) Radhakrishnan (NICL (GIC) AO (Finance) Exam, 15.12.2013)

18. Choose the word which is different from the rest. (1) Chicken (2) Snake (3) Swan (4) Crocodile (NICL (GIC) AO (Finance) Exam, 15.12.2013)

19. Find that pair of numbers which is not related to the other three pairs of numbers due to lack of common property. (1) 47,59 (2) 42,29 (3) 57,69 (4) 73,61 (NICL (GIC) Administrative Officer Exam, 15.12.2013)

20. In each of the following questions find the odd word from the given responses. (1) Mouth Organ (2) Electric Guitar (3) Keyboard (4) Sonata (NICL (GIC) Administrative Officer Exam, 15.12.2013)

21. Find the odd one out. (1) Cathedral (2) Mosque (3) Monastery (4) Temple (NICL (GIC) Administrative Officer Exam, 15.12.2013)

22. Unscramble the letters in the given words and find the odd one (1) PLPAE (2) RAORCT (3) AUVAG (4) NOONI (NICL (GIC) Administrative Officer Exam, 15.12.2013)

23. Find the odd one out (1) Pistol (2) Sword (3) Gun (4) Rifle (NICL (GIC) Administrative Officer Exam, 15.12.2013)

24. Find the odd letters from given alternatives: (1) PQXZ (2) BCQN (3) ABDF (4) MNPR (NICL (GIC) Administrative Officer Exam, 15.12.2013)

BPRE–49

25. In the following question four pairs of words are given out of which words in three pairs are related in the same way. Find the pair which is differently related? (1) Lyric, Ode (2) Sky, Earth (3) Head, Legs (4) Newspaper, Journalist (NICL (GIC) Administrative Officer Exam, 15.12.2013)

26. Four of the following five follow a certain pattern. Which one of the following does not follow the pattern? (1) ECA (2) QOM (3) GEC (4) FBD (5) MKI (LIC AAO Prelim Exam. 04.05.2019)

SHORT ANSWERS

NATIONALISED BANKS & IBPS PO/MT/SO EXAMS 1. (4) 5. (5) 9. (3) 13. (5) 17. (5)

2. (4) 6. (5) 10. (4) 14. (2) 18. (3)

3. (3) 7. (4) 11. (5) 15. (3) 19. (5)

4. (2) 8. (4) 12. (3) 16. (3) 20. (1)

21. (5) 25. (3)

22. (3) 26. (3)

23. (2) 27. (3)

24. (1)

SBI PO EXAMS 1. (2) 5. (3) 9. (2)

2. (4) 6. (4)

3. (1) 7. (2)

4. (3) 8. (1)

RBI GRADE–B/ NABARD GRADE–A OFFICER EXAMS 1. (2)

2. (5)

3. (1)

4. (3)

INSURANCE EXAMS 1. (2) 5. (3) 9. (3) 13. (3)

2. (4) 6. (2) 10. (2) 14. (3)

3. (1) 7. (4) 11. (4) 15. (4)

4. (3) 8. (4) 12. (3) 16. (1)

17. (1) 21. (3) 25. (4)

18. (1) 22. (4) 26. (4)

19. (2) 23. (2)

20. (4) 24. (3)

CLASSIFICATION

EXPLANATIONS

NATIONALISED BANKS & IBPS PO/MT/SO EXAMS 1. (4) Except Alaska, all others ore continents. Alaska is a mountain range. 2. (4) Except Gift, all others are given to winner only. 3. (3) +3

+1

–2

+3

+1

–2

+3

–1

+2

+3

+1

–2

M  → P   → Q   → O D  → G   → H   → F R  → U   → T  → V V  → Y   → Z   → X +3

+1

–2

J  → M   → N   → L +2

4. (2) A

M

C

L –1

–1 W

I

V

9. (3) Indifferent conveys different meaning from the other words. 10. (4) Except HU, all others are pairs of opposite letters. H and S are opposite letters. 11. (5) +2

–4

+1

–4

+2

+1

–2

+2

+1

+2

+1

O  → Q   → R  → N K  → M   → N   → L –4

U  → W   → X   → T –4

C  → E   → F   → B 13. (5) X ⇒ 24, 24 + 2 = 26 V ⇒ 22, 22 + 4 = 26 T ⇒ 20, 20 + 6 = 26 W ⇒ 23, 23 + 3 = 26 S ⇒ 19, 19 + 5 = 24 –2 N

F

U

+2

–1 K

+2 X

D

Z

C

R

H

R

+1

U

J

S

B

Q

20. (1) The sum of the position numbers of the last two letters in each letter group, except in ABCDEFGR, is 26. BX ⇒ 2 + 24 = 26; EU = 5 + 21 = 26; CW = 3 + 23 = 26; FT ⇒ 6 + 20 = 26 But, GR ⇒ 7 + 18 = 25

+1

(5) 4 6 5 D F E –2

7. (4) Except in EG, in all others, the first student is to immediate right of the second student. 8. (4) Compete is different from the other words.

+1

+4

+1

+6

+1

+2

21. (5) D   → E  → I

F  → G  → M B  → C   → E

D

O

+1

+5

E  → F  → K 5 is odd number and other than Even number So, Ans is (5) 22. (3) +

+

+4

+2

+

+

+4

+2

–3

4 1 –2 K   → O   → P  → N –3

L

+4 –2 23. (2) B   → F  → D

+4 –2 → X   → V T 

U

+2 15. (3) Except in DWVT, in all others there is at least one Vowel. 16. (3) Except Heart, all others are found in pair in humans. 17. (5) Except Extinct, all others words convey more or less similar meaning. Extinct (Adjective) means ‘no longer in existence’, ‘no longer active’.

BPRE–50

+

+4 –2 → O   → M K 

–2 W

+8

I  → J  → R

+

+2

J

+5

4 2 –3 → P   → R  → O L  

+1

(4) 8 7 1 H G A

–3

Q  → R   → O   → T

→ J   → G D  → H   –2

6. (5) (1) 2 1 8 (2) 5 4 7 B A H E D G (3) 3 2 6 C B F

+1

–5

T  → X   → Z   → W

+2

5. (5) Except in IRDV, in all others the first two letters are pairs of opposite letters and the last two letters are consecutive letters.

+2

F  → G   → I  → D

4 2 –3 → V   → X  → U R  

Q –1

+1

T

–2 H

J

I

–2

–1 +2

+1

–5

–5

C  → D   → F  → A

L

+2 I

+2

+2

+1

14. (2) D

+2 V

+1

+2

G  → I   → J  → F

+1

+1

U  → V   → X   → S

–3

→ L  → N  → K J  +2 +2 –3 → U T → V → X  +2 +2 –3 → B A → C  → E  +2 +2 –3 → Q P → R → T  +3 +1 –3 → K  → H G → J   12. (3) +2

–5 +1 +2 → L   → N  → I 19. (5) K  

K +2

G

+2

18. (3) Speak is different from the other words. Except ‘Speak’, all other words signify abstract actions.

+4 –2 L  → P  → N +

4 –1 But, E   → I   → H –1

+3

+2

–4

+2

–4

→ Q  → T 24. (1) R   N  → P   → L → H  → D F  +2

–4

→ I K → M  +2

–4

W  → Y  → U

CLASSIFICATION 25. (3) H P O

–2

F

–2

N

–2

M

–4

+2

B

–4

+2

J

–4

+2

I

→ N 4. (3) J 

L

+4 → G C 

K

+4 L  → P

–2 –4 +2 Y W S U But, –1 –3 +2 X W T V 26. (3) R +4 V +2 X –3 U

T D

+4 +4

X H

+4

+2

Z

+2

J

+2

–3 –3

B

+4 P  → T

But, +3 W  → Z –1 –2 +4 → S   → Q 5. (3) P  → T   –1

–2

+4

–1

–2

+4

–1

–2

U  → Y   → X   → V

G

K  → O   → N   → L

–3

Z

+4

W

L P R O But, K +4 O +1 P –2 N 27. (3) A

+4

D

D  → H   → G   → E

But, +5 –1 –3 → N   → M   → J I  +5

–2

+5

–2

+5

–2

+3

+2

→ X  → V 6. (4) S 

Y

A  → F  → D G  → L  → J

Pairs of opposite letters.

O  → R   → T +5

–2

R  → W  → U

J

K

Q

P

–1

+4

–1

O  → N  → R   → Q +1

+2

+1

H  → I   → K   → L –1

7. (2)

D

E

W

V

–1

+4

–1

–1

+4

–1

–2

G +6 M +6 H B +6 L F +6 R X

B  → A   → E   → D

I

D –2 H –2

H

T

S

But,

L

M

O P

T But, Q +2 9. (2) J

N

The opposite letter of L is O.

SBI PO EXAMS 1. (2) ‘Change’ is different from the other words. Except the word Change all other words show fragementation 2. (4) ‘Advocate’ is different from the other words. Except the word Advocate all other words imply some kind of directions, dissemination of knowledge and skill. 3. (1) ‘Extend’ is different from the other words. Except the word Extend all other words indicate altitude.

–1

K  → J  → N   → M

8. (1)

G

+4

U  → T  → X   → W

I E

–2 S –2 –2

+7

G C

–2 –2

U But, +2 P

T C N G 4. (3)

Z

H

S

Conflict (Verb) means “to be very different,” “to be in opposition”. Distinguish (Verb) means “to recognise the difference between people or things”. Contrast (Verb) means “to compare two people or things and show the difference between them”. Differentiate (Verb) means “to see or show that two things are different.” 2. (5) Recollect (Verb) means “to be able to remember something from the past”. Retrieve (Verb) means “to get something back”. Regain (Verb) means “to get something back again after losing it”. Recall (Verb) means “to remember”. Realise (Verb) means “to become aware of or accept something as a fact”. Thus, ‘Realise’ is different from other words. 3. (1) +2 –3 +5 J O L N

+6 +6

N M I Y

+6 +6

+5 +5 +5 +5

+8

S L

–2 –2 –2

W F Q J

–1 –1 –1 –1

V E P I

–1

–1

+3

→ L   → K   → N M  –2 +1 +2 → B   → C   → E D  –1

+3

→ X   → W   → Z Y  Z

RBI GRADE–B/ NABARD GRADE–A OFFICER EXAMS 1. (2) ‘Conflict’ is different from other words. Compare (Verb) means “to examine people or things to see how they are alike and how they are different”, “to judge one thing and measure it against another thing”.

BPRE–51

H

–2

–1 –1 +3 → J   → I   → L K 

–1

R

Y

–1 –1 +3 → S   → R   → U T 

INSURANCE EXAMS 1. (2) 22 – 13 = 9 76 – 24 = 52 52 – 16 = 36 62 – 17 = 45 The difference between the two numbers in all the pairs except 76 – 24, is multiple of 9. 2. (4) 16 – 6 = 10 48 – 18 = 30 56 – 21 = 35 76 – 27 = 49

CLASSIFICATION The difference between the two numbers in all the pairs except 27– 76, is multiple of 5. 3. (1) Except the number pair 39 – 77, in each of all the other number pairs the two numbers are the multiples of a same certain number. 4. (3) 26 = (5)2 + 1 4=5–1 226 = (15)2 + 1 14 = 15 – 1 274 = (17)2 – 15 16 = 17 – 1 82 = (9)2 + 1 8=9–1 5. (3) 2 × 2 = 4 4×2=8 8 × 4 = 32 But, 6 × 3 = 18 3 is an odd number. 6. (2) In all the four options the first and the third numbers from the left are the same. In Option (2) the second and the fourth numbers are whole numbers. 7. (4) Arrange the numbers in ascending order and find out the difference between the two consecutive numbers. This will reveal the pattern of numbers. Option (1)

1 , 2, 3 2 Option (3) 1, 1

1 3 1 3 , , , 4 8 2 4 ⇒ 0.25, 0.375, 0.50, 0.75 Option (4) 1 1 3 , , 1, 1 2 8 4 ⇒ 0.125, 0.75, 1, 1.5 8. (4) +3

+

+

–2

→ U   → X   → V S  2 3 –2  → O   → R  → P M  +2

+3

+2

+2

+

+



–2 3 → O    →L Q  +6

–2

→ F  → L H  –2

–3

→ U   → R W  –2 –3 → K   → H M 

10. (2) In all the four options the first and the third numbers from the left are the same. In Option (2) the second and the fourth numbers are whole numbers. 11. (4) Arrange the numbers in ascending order and find out the difference between the two consecutive numbers. This will reveal the pattern of numbers. Option (1)

1 1 3 , , 1, 1 2 2 4 ⇒ 0.50, 0.75, 1, 1.5 Option (2) 1 , 2, 3 2 Option (3) 1, 1

1 3 1 3 , , , 4 8 2 4 ⇒ 0.25, 0.375, 0.50, 0.75 Option (4) 1 1 3 , , 1, 1 2 8 4 ⇒ 0.125, 0.75, 1, 1.5 12. (3) D W Pair of Opposite Letters. H S B Y

1 1 3 , , 1, 1 2 2 4 ⇒ 0.50, 0.75, 1, 1.5 Option (2)

+2

−3

–2

→ I   → F 9. (3) K  

–2

→ F   → I   → G D  –1

→ E   → G   → F C  2 3 –2  → L   → O  → M J 

D

W

E

V

J

Q

The opposite letter of C is X. The opposite letter of F is U. 13. (3) All except sleeping, denote activities. 14. (3) Bridge is different from the other three. Ladder is a structure for climbing up and down. Staircase is a set of stairs and its supporting structure. Escalator is a moving staircase carrying people up or down between floors or different levels. Bridge is a structure of wood, iron, concrete etc. built to provide a way across a river, road, railway etc.

BPRE–52

15. (4) Except in Legumes – Nodules, in all other pairs second denotes the class to which the first belongs. 16. (1) Except wool, all others can be obtained from insects. Wool is obtained from sheep and goat. 17. (1) Except Acharya Kripalani all others are Bharat Ratna Awardees. 18. (1) All except chicken can live in water. 19. (2) Except in the number-pair 4229, in all other pairs, the difference between the two numbers is 12. 59 – 47 = 12; 69 – 57 = 12; 73 – 61 = 12 But, 42 – 29 = 13 20. (4) Sonata is piece of music composed for one instrument or two, one of which is usually the piano. Except sonata, all others are musical instruments or part of a musical instrument. 21. (3) Monastery is a place or building where Buddhist monk lives as a community. All others are places of worship. 22. (4) PLPAE ⇒ APPLE RAORCT ⇒ CARROT AUVAG ⇒ GUAVA NOONI ⇒ ONION Onion is a round white or reddish vegetable with many layers inside each other. 23. (2) Sword is different from the other three. Sword is a weapon with a long thin metal blade and a protected handle. Pistol is a type of small gun. 24. (3) Except ABDF, in all other letter groups all the four letters are consonants. 25. (4) In all others the two terms denote the two extremies. An ode is a poem addressed to a person or thing or celebrating some special event. 26. (4)

E Q G M

–2 –2 –2 –2

C O E K

–2 –2 –2 –2

A M C I

But,

F

–4

B

+2

D ❐❐❐

CLASSIFICATION

MODEL EXERCISES 1. Three of the following are alike in a certain way and form a group. Find the odd one out. (1) Bird (2) Insect (3) Aeroplane (4) Kite (5) None of these Directions (2-5) : In each of these questions, four terms are given. While three of them are identical in some way, one is different from the rest. Select the odd one as your answer. 2. (1) T (2) Z (3) Q (4) H (5) None of these 3. (1) Hat (2) Bag (3) Purse (4) Basket (5) None of these 4. (1) UNICEF (2) IMF (3) WHO (4) SAARC (5) None of these 5. (1) 5 8 7 8 (2) 6 4 8 2 (3) 5 7 8 8 (4) 9 7 4 8 (5) None of these Directions (6-8) : In each of these questions there are four words with the letters jumbled up. Three of them are alike. Find the odd one out.



6. (1) CIRE (3) LOUDSC

(2) NAIR (4) RNUTHDE

(5) None of these 7. (1) FIWE

(2) FLAMEE

(3) BUSHDNA (4) OMAWN (5) None of these 8. Find the odd one out. (1) Grams

(2) Litres

(3) Tonnes

(4) Qminatals

(5) None of these 9. Which of the following cannot be a number of the series 1, 8, 27, 64, 125, ...? (1) 216

(2) 729

(3) 512

(4) 1000

(5) None of these

SHORT ANSWERS 1.(2)

2.(3)

3.(1)

4.(4)

5.(2)

6.(1)

7.(3)

8.(2)

9.(1)

EXPLANATIONS 1. (2) Except insects, all other fly in the sky. 2. (3) Q occupies the odd position (17) in the alphabetical order whereas other occupy the even position. 3. (1) All other objects are used for carrying purpose. 4. (4) Except SAARC, all other are agency of UNO. 5. (2) The sum of digits of all other numbers is 28. 6. (1) If all the letters are arranged then words formed are CRIE, RAIN, CLOUDS, THUNDER. Except CIRE all other belong to the same category. 7. (3) Rearranged words are Wife, Female, Husband woman. Hence, the Husband does not belong to this category. 8. (2) Litre is the unit of measurement for liquid 9. (1) Given numbers are cubes of natural numbers. ❑❑❑

SALIENT FEATURES ❖ A sincere attempt to use reader friendly lucid language to make understanding easy. ❖ Teach yourself approach adopted in shaping this book. ❖ A pious effort to put simple solutions and much needed short-cut methods. ❖ A long list of chapters and a wide variety of questions. ❖ Special emphasis on concept-building. ❖ Detailed discussion on basic concepts of each topic. ❖ Fundamental concepts/formulae illustrated through upgraded questions. ❖ Problem solving techniques through MCQs. ❖ Large number of Multiple Choice Questions (more then 7000) with detailed explanations. ❖ Exercises with a plenty of questions asked in various competitive exams with proper highlight of name of exam and year. ❖ Illustration of more than one method solution to many questions. ❖ Distribution of each exercise in differently oriented categories/ sections. ❖ Every attempt made to encourage to know quantitative aptitude, Price avoid rote learning. 635 ❖ Model Practice Sets for various exams. ❖ Select questions asked in previous exams concerned in each set. Enriched concepts on Algebra, Sequence and Series (Progression), Permutation and Combination, Quadratic Equations and Inequations, Probability, Geometry, Trigonometry, Mensuration and most importantly Data Sufficiency, Analysis and Interpretation.

BPRE–53

CODING / DECODING

5

CODING/DECODING

QUESTIONS FROM 1999 TO 2010 ARE AVAILABLE ONLINE NATIONALISED BANKS & IBPS PO/MT/SO EXAMS Directions (1-5) : Study the following information to answer the given questions : (Indian Bank PO Exam. 02.01.2011 (Ist sitting)

In a certain code ’8 2 9' means ‘how art thou,’ ‘9 5 8’ means ‘thou art good’ and ‘1 5 8 7 3’ means ‘thy good and thou bad’. 1. What may be the possible code for ‘thy’ ? (1) 1 or 7 (2) 7 (3) 3 (4) 5 (5) 1 or 7 or 3 2. What is the code for ‘thou’ ? (1) 9 (2) 8 (3) 2 (4) 5 (5) None of these 3. What is the code for ‘how’ ? (1) 5 (2) 8 (3) 2 (4) 9 (5) None of these 4. Which of the following may possibly be the code for ‘thou no good’ ? (1) 5 0 8 (2) 7 8 0 (3) 5 0 7 (4) 7 8 0 (5) None of these 5. What is the code for ‘how good thou art’ ? (1) 7 5 8 9 (2) 8 2 9 5 (3) 7 1 8 3 (4) 8 7 9 5 (5) Cannot be determined Directions (6–10) : Study the following information to answer the given questions : (Corporation Bank PO Exam. 16.01.2011)

In a certain code ‘a friend of mine’ is written as ‘4 9 1 6’ ‘mine lots of metal’ is written as ‘ 3 1 0 9’ and ‘a piece of metal’ is written as ‘7 1 6 3’. 6. What is the code for ‘piece’ ? (1) 3 (2) 6 (3) 1 (4) 7 (5) Cannot be determined 7. What does ‘9’ stand for ? (1) of (2) mine (3) friend (4) lots (5) metal

Which of the following may represent ‘a pleasure of mine’ ? (1) 6309 (2) 5216 (3) 9216 (4) 3694 (5) 5041 9. What does ‘0’ stand for ? (1) mine (2) metal (3) of (4) lots (5) a 10. ‘8 7 3’ would mean (1) a metal piece (2) metal for friend (3) piece of advise (4) friend of mine (5) large metal piece Directions (11–14) : Study the following information to answer the given questions : 8.

(Corporation Bank PO Exam. 16.01.2011)

In a certain code, ‘ze lo ka gi’ is a code for ‘must save some money’,‘fe ka so ni’ is a code for ‘he made good money’, ‘ni to da so’ is a code for ‘he must be good’ and ‘we so ze da’ is a code for ‘be good save grace’. 11. Which of the following is the code of ‘must’ ? (1) so (2) da (3) lo (4) ni (5) Cannot be determined 12. What does the code ‘ze’ stand for ? (1) some (2) must (3) be (4) grace (5) save 13. Which of the following is the code of ‘good’ ? (1) so (2) we (3) ze (4) lo (5) fe 14. ‘grace of money’, may be coded as (1) ka da fe (2) we ka so (3) ja da we (4) ka we yo (5) ja ka ze 15. In a certain code ‘PLANT’ is written as ‘$@2★©’ and ‘YIELD’ is written as ‘β64@%’. How is ‘DELAY’ written in that code ?

BPRE–54

(1) β4★2% (2) β4@2% (3) %42@β (4) %4@2β (5) None of these (Punjab & Sind Bank PO Exam. 23.01.2011)

Directions (16-20) : In each of the following questions, a group of letters is to be coded by number/symbol codes as per the codes given below and the conditions that follow. In each question, a group of letters is given followed by groups of number/symbol code numbered (1), (2), (3) and (4). The correct combination of codes is your answer. If none of the four combinations is correct, your answer is (5), i.e. ‘None of these’. (Punjab & Sind Bank PO Exam. 23.01.2011) Letter P D A F L H M R K T B E U Code 3 6 # 8 $ 4 2 7 @ 9 5 1 %

Conditions (i) If the first letter is a vowel and the last letter is a consonant, both are to be coded as ‘0’. (ii) If the first letter is a consonant and the last letter is a vowel, both are to be coded as’©’. (iii) If the first as well as the last letters are vowels, both are to be coded as the code for first letter. 16. KEDURT (1) @16%7@ (2) 916%7@ (3) 916%79 (4) @16$79 (5) None of these 17. EDRTMP (1) 167923 (2) 167921 (3) 067920 (4) 067923 (5) None of these 18. FHRAMU (1) 847#28 (2) %47#28 (3) 847#2% (4) ©47#2© (5) None of these 19. ULTPHE (1) %$934% (2) 1$9341 (3) %$9341 (4) 1$934% (5) None of these 20. HKEDLB (1) 5@16$4 (2) 4@16$5 (3) ©@16$4 (4) 0@16$5 (5) None of these

CODING / DECODING Directions (21–28) : Study the following information to answer the given questions : (UCO Bank PO Exam. 30.01.2011)

In a certain code ‘it is rush hour traffic’ is written as ‘sa le do mi ru’ ‘go to school’ is written as ’be no pa’, ‘one hour to go’ is written as ‘mi fi pa be’, ‘rush to one’ is written as ‘fi be sa’ and ‘traffic is fine’ is written as ‘ga ru do’. 21. ‘ru be wa’ would mean— (1) is to way (2) traffic is to (3) way is traffic (4) to traffic way (5) Cannot be determined 22. What is the code for ‘school’ ? (1) do (2) ru (3) be (4) pa (5) None of these 23. What is the code for ‘fine’ ? (1) ga (2) ru (3) pa (4) do (5) None of these 24. Which of the following represents ‘school hour go fine’ ? (1) pa be fi ga (2) no mi ra pa (3) pa no ga mi (4) ga no mi le (5) None of these 25. ‘mi fi le’ would mean — (1) it one to (2) to rush one (3) rush hour it (4) it one hour (5) None is correct 26. What does ‘sa’ stand for? (1) rush (2) traffic (3) it (4) is (5) None of these 27. Which of the following may represent ‘traffic is for one hour’ ? (1) fi ye no mi ru (2) fi le do mi ru (3) fi ye do mi ru (4) fi so do mi ro (5) None of these 28. What does ‘do’ stand for? (1) hour (2) ‘is’ or ‘traffic’ (3) it (4) is (5) None of these Directions (29–33) : Study the following information to answer the given questions: (Bank Of Baroda PO Exam. 13.03.2011)

In a certain code, ‘more money in market’ is written as ‘zo li aa to’; ‘share in market profit’ is written as ‘vo to je li’; ‘making more profit now’ is written as ‘su je zo ka’; ‘now the market gains’ is written as ‘do li yo su’.

29. Which of the following does ‘vo’ stand for? (1) profit (2) in (3) share (4) market (5) in or profit 30. What is the code for ‘making’ ? (1) ka (2) su (3) je (4) zo (5) Cannot be determined 31. Which of the .following is the code for ‘gains’ ? (1) su (2) li (3) yo (4) do (5) yo or do 32. Which of the following can be the code for ‘the more you share’ ? (1) do yo zo vo (2) vo wi zo do (3) vo zo wi bu (4) yo je vo wi (5) su vo zo do 33. ‘to ka li aa’ is a code for which of the following? (1) share more in market (2) now share more gains (3) the gains in market (4) the gains in profit (5) making money in market Directions (34-37) : Study the following information to answer the given questions : (Allahabad Bank PO Exam. 17.04.2011)

In a certain code ‘her idea has merit’ is written as ‘fo la ‘bu na’, ‘merit list has been displayed’ is written as ‘jo ke la si na’ and ‘her name displayed there’ is written as ‘ya si bu zo’, ‘name in merit list’ is written as ‘na ya go ke’. 34. What does ‘ke’ stand for? (1) been (2) has (3) merit (4) name (5) list 35. What is the code for ‘idea’ ? (1) fo (2) la (3) bu (4) na (5) Either bu or na 36. Which of the following represents ‘name has been displayed’ ? (1) ya la ke si (2) jo si ya la (3) si jo ke na (4) bu ya ke la (5) ya si jo zo 37. What does ‘zo’ stand for? (1) there (2) displayed (3) name (4) her (5) Cannot be determined 38. Which of the following may represent ‘her name is there’ ? (1) zo ya go wo (2) bu ya zo go (3) zo ya bu ke (4) ya zo wo bu (5) wo go zo ya

BPRE–55

39. What is the code for ‘in’ ? (1) na (2) ya (3) go (4) ke (5) Cannot be determined Directions (40–45) : Study the following information to answer the given questions: (Indian Overseas Bank PO Exam. 22.05.2011)

In a certain code ‘for profit order now’ is written as ‘ho ja ye ga’ ‘right now for him’ is written as ‘ga ve ja se’. ‘place order for profit’ is written as ‘ga bi ho ye’ and ‘only in right order’ is written as ‘ve du ye zo’. 40. What is the code for ‘him’ ? (1) ga (2) ve (3) ja (4) se (5) Cannot be determined 41. What does ‘bi’ stand for? (1) profit (2) order (3) place (4) for (5) now 42. ‘fo ve du’ could be a code for which of the following? (1) in right spirits (2) only in profit (3) order only him (4) place in right (5) order only now 43. What is the code for ‘profit’ ? (1) ye (2) ga (3) bi (4) ja (5) ho 44. Which of the following may represent ‘only for now’ ? (1) ja bi zo (2) du zo ga (3) zo ga ja (4) zo ga ye (5) du bi ja 45. What is the code for ‘order’ ? (1) ye (2) ga (3) bi (4) ja (5) ho 46. In a certain code ‘TEAMWORK’ is written as ‘NBFUJQNV’ and ‘SOME’ is written as ‘PTDL’. How is ‘PERSON’ written in that code? (1) QDOOPT (2) QDOMNR (3) SFQMNR (4) SFQOPT (5) None of these (IBPS Specialist Officer CWE Exam. 11.03.2012)

47. In a certain code ‘BASKET’ is written as ‘5$3%#1’ and ‘TRIED’ is written as ‘14★#2’. How is ‘SKIRT’ written in that code? (1) 3%★41 (2) 3★%41 (3) 3%#41 (4) 3#4%1 (5) None of these (IBPS Specialist Officer CWE Exam. 11.03.2012)

CODING / DECODING Directions (48-52) : Study the following information to answer the given questions : (IDBI Bank Officer Exam.16.09.2012)

In a certain code “new banking systems” is coded as “ss tp na”, “officer in uniform” is coded as “or mu at”, “new bank officer” is coded as ”or bk na” and “systems in bank” is coded as “bk at ss”. 48. What does the code “bk” stand for? (1) new (2) systems (3) officer (4) in (5) None of these 49. What will the code “ss mu” stand for ? (1) banking officer (2) new uniform (3) uniform banking (4) in systems (5) uniform systems 50. How will “new officer” be coded? (1) or na (2) tp na (3) na at (4) tp or (5) ss at 51. How will “bank officer in uniform” be coded ? (1) ss na at or (2) bk at or mu (3) ss na bk at (4) at mu ss or (5) bk ss mu na 52. What is the code in “in” ? (1) ss (2) or (3) at (4) mu (5) None of these Directions (53-57) : Study the following information to answer the given questions : (IBPS Specialist Officer CWE Exam.17.03.2013)

In a certain code ’8 2 9' means ‘how art thou,’ ‘9 5 8’ means ‘thou art good’ and ‘1 5 8 7 3’ means ‘thy good and thou bad’. 53. What may be the possible code for ‘thy’ ? (1) 1 or 7 (2) 7 (3) 3 (4) 5 (5) 1 or 7 or 3 54. What is the code for ‘thou’ ? (1) 9 (2) 8 (3) 2 (4) 5 (5) None of these

55. What is the code for ‘how’ ? (1) 5 (2) 8 (3) 2 (4) 9 (5) None of these 56. Which of the following may possibly be the code for ‘thou no good’ ? (1) 5 0 8 (2) 7 8 0 (3) 5 0 7 (4) 7 8 0 (5) None of these 57. What is the code for ‘how good thou art’ ? (1) 7 5 8 9 (2) 8 2 9 5 (3) 7 1 8 3 (4) 8 7 9 5 (5) Cannot be determined Directions (58-62) : Study the following information carefully and answer the given questions :

62. Which of the following is code of “serial” ? (1) pu (2) to

63.

64.

(Indian Overseas Bank PO Online Exam, 01.09.2013)

In a certain code language : “only in serial order” is written as “ve pu na to”. “order in the state” is written as “li ve su pu”. “the logical idea only” is written as “su na ri jo”. “in idea or theory” is written as “zt jo bk pu”. 58. Which of the following is the code of ‘theory’ ? (1) zt (2) bk (3) jo

(4) pu

(5) Either ‘zt’ or ‘bk’ 59. The code ‘li ri to ve’ may represent (1) serial order theory only (2) only idea state order (3) state logical serial order (4) serial theory state the (5) only the idea logical 60. Which of the following may represent “logical idea is only order”? (1) jo na ri ge ve (2) ve na ri jo pu (3) ri ve na zt bk (4) bk to pu jo ve (5) na ve su li pu 61. Which of the following is the code of “logical” ? (1) su (2) jo (3) na

(4) ri

(5) None of these

BPRE–56

65.

(3) ve (4) su (5) Cannot be determined In a certain code language the word COSTLY is written as WORVMF and the word PRAISE is written as CVGDPS. How will the word SOCCER be written in that code language ? (1) PHAFMV (2) BJWQRA (3) PGAENU (4) AKXPSB (5) None of these In a certain code language ‘he was not absent’ is written as ‘ja si fe ra’; ‘absent case was not’ is written as ‘ga fe je ra’ and ‘no one was absent’ is written as ‘da ge ra fe’. What does ‘si’ stands for ? (1) was (2) not (3) one (4) he (5) case In a certain code language DHIE is written as WSRV and AEFB is written as ZVUY. How will GKLH be written in that code language ? (1) SQNR (2) TQMP (3) TRDO (4) TPOS (5) None of these

(IBPS Bank PO/MT CWE-III, 26.10.2013)

66. In a certain code CONQUER is written as MNBRQDT. How is STEAMER written in that code ? (1) DRSBQDL (2) DSRBLDQ (3) DSRZQDL (4) DSRBQDL (5) None of these Directions (67–71) : Study the following information to answer the given questions : (Corporation Bank SO (Marketing) Exam, 22.02.2014)

In a certain code ‘it is rush hour traffic’ is written as ‘sa le do mi ru’ ‘go to school’ is written as ’be no pa’, ‘one hour to go’ is written as ‘mi fi pa be’, ‘rush to one’ is written as ‘fi be sa’ and ‘traffic is fine’ is written as ‘ga ru do’. 67. Which of the following represents ‘school hour go fine’ ? (1) pa be fi ga (2) no mi ra pa (3) pa no ga mi (4) ga no mi le (5) None of these

CODING / DECODING 68. ‘mi fi le’ would mean — (1) it one to (2) to rush one (3) rush hour it (4) it one hour (5) None is correct 69. What does ‘sa’ stand for? (1) rush (2) traffic (3) it (4) is (5) None of these 70. Which of the following may represent ‘traffic is for one hour’ ? (1) fi ye no mi ru (2) fi le do mi ru (3) fi ye do mi ru (4) fi so do mi ro (5) None of these 71. What does ‘do’ stand for? (1) hour (2) ‘is’ or ‘traffic’ (3) it (4) is (5) None of these 72. If in a certain code language PROUD is coded as OQPTC, NEW is coded MFV, RED is coded as QFC and TIMES is coded as SHNDR, then how is ALONE coded ? (1) BMNOF (2) ZMNOD (3) BKQNF (4) ZKPMD (5) ZNQRJ Directions (73–77) : Study the following information carefully and answer the questions given below : (BOB Manipal School of Banking Officer Online Exam, 14.08.2014)

In a certain code language, ‘capital cities are crowded’ is written as ‘ju sh pi be’ ‘crowded cities create chaos’ is written as ‘sh be nt ro’ ‘huge industry create capital’ is written as ‘db ju nt ka’ ‘industries are huge chaos’ is written as ‘ka pi ro db’ 73. What would be the code for “crowded cities have huge industry” ? (1) sh be ro ka db (2) pi ka ro sh be (3) ka be db sh ck (4) ju db ka sh pi (5) ka sh be nt db 74. What is the code for ‘chaos’ in that code language ? (1) ro (2) db (3) pi (4) ka (5) be

75. What would be the code for “huge cities create chaos” ? (1) ka be nt ro (2) nt ro db sh (3) be db ro nt (4) ro sh ka nt (5) Cannot be determined 76. What would be the code for “capitals are chaos” ? (1) ju sh pi (2) pi ro ju (3) be ro pi (4) nt ju pi (5) sh ju be 77. What is the code for “huge industry” ? (1) ka pi (2) db ro (3) db ka (4) ka db (5) ka ro Directions (78-82) : Study the following information carefully and answer the questions given below : (IBPS RRBs Officer Scale-I CWE, 06.09.2014)

In a certain code language, ‘rural and urban divide’ is coded as ‘na ku zu la’ ‘gap in rural infrastructure’ is coded as ‘kt la vm pi’ ‘urban planning more important’ is coded as ‘ti na cu bu’ ‘more divide than gap’ is coded as ‘pi cu dm zu’ 78. What is the code for ‘and’ ? (1) zu (2) na (3) ku (4) la (5) Cannot be determined 79. What is the code for ‘rural divide’ ? (1) zu la (2) la dm (3) pi zu (4) ku la (5) Cannot be determined 80. What is the code for ‘gap’ ? (1) dm (2) vm (3) zu (4) pi (5) pi or cu 81. Which of the following may possibly be the code for ‘infrastructure gap divide rural and urban planning’ ? (1) bu ku na zu pi la cu (2) vm la zu pi na cu ku (3) kt bu zu pi ti vm la (4) la ku vm kt ti bu na (5) Cannot be determined 82. What is the code for ‘than’ ? (1) pi (2) dm (3) cu (4) zu (5) ti

BPRE–57

Directions (83–87) : Study the following information carefully and answer the questions given below : (Bank of Baroda Junior Management Grade/ Scale-I Exam, 18.04.2015)

In a certain code language, ‘many residents of city’ is written as ‘ja nu pa la’ ‘city with many choices’ is written as ‘ko la nu si’ ‘choices made by residents’ is written as ‘pa mx ko tr’ ‘made good choices here’ is written as ‘vk rp tr ko’ (All the codes are two letter codes only) 83. What is the code for ‘with’ in the given code language ? (1) tr (2) la (3) ko (4) si (5) nu 84. What is the code for ‘made by residents’ in the given code language ? (1) mx pa tr (2) ko mx pa (3) tr mx ko (4) la pa tr (5) mx tr vk 85. What is the code for ‘choices’ in the given code language ? (1) si (2) vk (3) la (4) nu (5) ko 86. Which of the following may represent ‘here for good’ in the given code language ? (1) la rp wq (2) xz vk si (3) rp uy vk (4) vk rp ja (5) vk rp la 87. What does the code ‘nu’ stand for in the given code language? (1) of (2) Either ‘city’ or ‘many’ (3) with (4) Either ‘city’ or ‘good’ (5) good Directions (88–92) : Study the following information carefully and answer the questions given below : (BOB Junior Management Grade/Scale–I Exam. 18.04.2015)

In a certain code language, ‘many residents of city’ is written as ‘ja nu pa la’ ‘city with many choices’ is written as ‘ko la nu si’ ‘choices made by residents’ is written as ‘pa mx ko tr’ ‘made good choices here’ is written as ‘vk rp tr ko’ (All the codes are two letter codes only)

CODING / DECODING 88. What is the code for ‘with’ in the given code language ? (1) tr (2) la (3) ko (4) si (5) nu 89. What is the code for ‘made by residents’ in the given code language ? (1) mx pa tr (2) ko mx pa (3) tr mx ko (4) la pa tr (5) mx tr vk 90. What is the code for ‘choices’ in the given code language ? (1) si (2) vk (3) la (4) nu (5) ko 91. Which of the following may represent ‘here for good’ in the given code language ? (1) la rp wq (2) xz vk si (3) rp uy vk (4) vk rp ja (5) vk rp la 92. What does the code ‘nu’ stand for in the given code language? (1) of (2) Either ‘city’ or ‘many’ (3) with (4) Either ‘city’ or ‘good’ (5) good Directions (93–97) : Study the following information carefully and answer the questions given below : (IBPS RRBs Officer Scale–I & II CWE 12.09.2015)

In a certain code language, ‘few organic farming techniques’ is written as ‘li gs da cr’ ‘fertilizer products few available’ is written ‘fo pz nb gs’ ‘organic waste into fertilizer’ is written ‘nb cr pt mk’ ‘disposal of farming waste’ is written as ‘hu mk li yu’ (All codes are two letter codes only) 93. What will be the code for ‘few waste’ in the given code language? (1) gs li (2) pt da (3) da mk (4) Other than those given as options (5) mk gs 94. What is the code for ‘organic’ in the given code language? (1) gs (2) cr (3) da (4) pt (5) Other than those given as options

95. In the given code language, what does the code ‘yu’ stand for? (1) farming (2) techniques (3) either ‘of’ or ‘disposal’ (4) waste (5) either ‘into’ or ‘few’ 96. If ‘waste management techniques’ is coded as ‘ax da mk’ in the given code language, then how will ‘farming fertilizer management’ be coded as? (1) ax nb cr (2) li ax pt (3) gs li nb (4) nb li ax (5) Other than those given as options 97. What is the code for ‘available’ in the given code language? (1) either ‘pz’ or ‘fo’ (2) either ‘nb’ or ‘mk’ (3) li (4) hu (5) Other than those given as options Directions (98–102) : Study the following information carefully and answer the questions given below : (IBPS Bank PO/MT CWE–V (Preliminary) 03.10.2015)

In a certain code language, ‘festival for woman only’ is written as ‘pa ge bo xu’ ‘provide peace to women’ is written as ‘wr dl nj ge’ ‘women like to celebrate’ is written as ‘ge ct fx wr’ ‘celebrate peace in festival’ is written as ‘dl bo sv ct’ (All codes are two letter codes only) 98. What may be the possible code for ‘provide idea’ in the given code language ? (1) fx hy (2) xu bo (3) hy nj (4) nj xu (4) wr fx 99. What is the code for ‘celebrate’ in the given code language ? (1) ct (2) wr (3) sv (4) dl (5) fx 100. In the given code language what does the code ‘pa’ stand for ? (1) peace (2) either ‘for’ or ‘only’ (3) either ‘women’ or ‘to’ (4) celebrate (5) festival

BPRE–58

101. What is the code for ‘women’ in the given code language ? (1) bo (2) xu (3) ct (4) Other than those given as options (5) ge 102. If ‘peace to mind’ is coded as ‘zg wr dl’ in the given code language, then what is the code for ‘mind in festival’ ? (1) zg bo dl (2) dl zg sv (3) zg nj wr (4) bo sv zg (5) sv wr bo Directions (103–107) : Study the following information carefully and answer the questions given below : (IBPS Bank PO/MT CWE–V (Preliminary) 04.10.2015)

In a certain code language, ‘paint your house red’ is written as ‘ri fm ew cu’ ‘gate of red colour’ is written as ‘lb ew op sa’ ‘house of your choice’ is written as ‘sa cu ri nk’ ‘gate with red paint’ is written ‘gy op ew fm’ (All codes are two letter codes only) 103. What is the code for ‘red’ in the given code language ? (1) Other than those given as options (2) sa (3) gy (4) ew (5) fm 104. What may be the possible code for ‘gate crash’ in the given code language ? (1) jx op (2) ri op (3) lb jx (4) op lb (5) jx ri 105. In the given code language, what does the code ‘cu’ stand for ? (1) paint (2) either ‘of’ or ‘colour’ (3) choice (4) with (5) either ‘house’ or ‘your’ 106. What is the code for ‘paint’ in the given code language ? (1) gy (2) fm (3) sa (4) op (5) ri 107. If ‘colour with canvas’ is coded as ‘hv lb gy’ in the given code language, then what is the code for ‘canvas of choice’?

CODING / DECODING (1) hv nk ew (2) ri sa nk (3) nk hv sa (4) sa ew hv (5) sa ri hv Directions (108–112) : Study the following information carefully and answer the questions given below : (IBPS Bank PO/MT CWE–V (Preliminary.) 04.10.2015)

In a certain code language, ‘committee to review papers’ is written as ‘es fr re pt’ ‘review meeting in morning’ is written as ‘ch ba mo fr’ ‘meeting to appoint members’ is written as ‘re dv ch gi’ ‘appoint chairman in review’ is written as ‘mo gi fr yu’ (All the codes are two letter codes only) 108. What is the code for ‘morning’ in the given code language? (1) mo (2) yu (3) ch (4) Other than those given as options (5) ba 109. In the given code language, what does the code ‘pt’ stands for? (1) appoint (2) either ‘papers’ or ‘committee’ (3) morning (4) review (5) either ‘for’ or ‘members’ 110. What is the code for ‘review call’ in the given code language? (1) dv lq

(2) lq gi

(3) lq fr

(4) gi es

(5) fr dv 111. What is the code for ‘to’ in the given code language? (1) mo (3) gi

(2) fr (4) re

(5) dv 112. If ‘appoint new members’ is coded as ‘dv wz gi’ in the given code language, then what is the code for ‘new chairman meeting’? (1) ch wz yu (2) yu mo wz (3) fr es wz (4) ch yu fr (5) wz ch es Directions (113–117) : Study the following information carefully and answer the questions given below : (IBPS Bank PO/MT CWE–V (Preliminary) 10.10.2015 Ist Sitting)

In a certain code language, ‘offer prayers to god’ is written as ‘bi gv oc st’ ‘prayers for school assembly’ is written as ‘tm oc da pu’ ‘school offer special education’ is written as ‘nh mk tm gv’ ‘assembly must to school’ is written as ‘da st rx tm’ (All the codes are two letter codes only) 113. What is the code for ‘must’ in the given code language ? (1) da (2) Other than those given as options (3) rx (4) tm (5) st 114. What is the code for ‘education’ in the given code language? (1) Either ‘mk’ or ‘nh’ (2) Either ‘tm’ or ‘gv’ (3) mk (4) nh (5) tm 115. If ‘school to home’ is coded as ‘aj tm st’ in the given code language, then how ‘home for god’ will be coded ? (1) pu gv aj (2) bi aj oc (3) da bi st (4) aj bi pu (5) bi mk rx 116. What may be the possible code for ‘school must offer training’ in the given code language ? (1) rx gv mk tm (2) tm rk rx gv (3) oc gv rx tm (4) st gv oc bi (5) gv da nh pu 117. What does the code ‘da’ stand for in the given code language? (1) school (2) to (3) prayers (4) Other than those given as options (5) assembly Directions (118–122) : Study the following information carefully and answer the questions given below : (IBPS Bank PO/MT CWE–V (Preliminary) 10.10.2015)

In a certain code language, ‘always follow your passion’ is written as ‘ke ag mo jp’. ‘great passion for music’ is written as ‘mo bu sc nd’. ‘music always on mind’ is written as ‘fi sc ag lw’. ‘follow music on twitter’ is written as ‘ty jp fi sc’. (All codes are two-letter codes only)

BPRE–59

118. What is the code for ‘follow’ in the given code language (1) ke (2) jp (3) Other than those given as options (4) fi (5) sc 119. What is the code for ‘mind’ in the given code language ? (1) bu (3) lw (5) ty

(2) ag (4) ke

120. In the given code language, what does the code ‘nd’ stand for ? (1) either ‘for’ or ‘great’ (2) music (3) mind (4) always (5) either ‘music’ or ‘on’ 121. If ‘music always help’ is coded as ‘ag hr sc’ in the given code language, then what is the code for ‘help your twitter’? (1) ke ty bu

(2) hr tw ag

(3) hr ke sc

(4) ty ke hr

(5) bu ty hr 122. What may be the possible code for ‘divine passion’ in the given code language? (1) mo ag (2) bu mo (3) xy ag (4) xy bu (5) mo xy Directions (123–127) : Study the following information carefully and answer the questions given below : (IBPS RRBs Officer Scale–I & II CWE 13.09.2015)

In a certain code language, ‘challenges for rural education’ is written as ‘vx pr bt ze’ ‘find measures for problems’ is written as ‘ws dl ze ho’ ‘experts find challenges difficult’ is written as ‘bt ka mu dl’ ‘education difficult in villages’ is written as ‘xq cg pr ka’ (All codes are two letter codes only) 123. If ‘experts find solutions’ is written as ‘dl ny mu’ in the given code language, then what will ‘rural solutions difficult’ be coded as ? (1) ny vs ka (2) ny pr dl (3) ka mu ny (4) Other than those given as options (5) vx ka xq

CODING / DECODING 124. What is the code for ‘villages’ in the given code language ? (1) ws (2) mu (3) either ‘xq’ or ‘cg’ (4) Other than those given as options (5) either ‘pr’ or ‘ze’ 125. What will be the code for ‘find education’ in the given code language ? (1) ka pr (2) pr dl (3) ze ka (4) dl ws (5) Other than those given as options 126. What is the code for ‘challenges’ in the given code language ? (1) cr (2) Other than those given as options (3) ka (4) ze (5) bt 127. In the given code language, what does the code ‘ho’ stand for ? (1) either ‘measures’ or ‘problems’ (2) either ‘for’ or ‘experts’ (3) rural (4) find (5) difficult 128. In a certain code, PARTICLE is written as USBQFMDJ and GENERATE is written as FOFHFUBS, how is DOCUMENT written in that code? (1) VDEPUONF (2) VDPENFUQ (3) VDPEUOFN (4) PEUVDNOF (5) OFNVDUEP (IBPS Bank PO/MT CWE–V Main Exam. 31.10.2015)

Directions (129-133) : Study the following information carefully and answer the questions given below : (IBPS Bank PO/MT CWE–V Main Exam. 31.10.2015)

In a certain code language, ‘capital cities are crowded’ is written as ‘ju sh pi be’ ‘crowded cities create chaos’ is written as ‘sh be nt ro’ ‘huge industries create capital’ is written as ‘db ju nt ka’ ‘industries are huge chaos’ is written as ‘ka pi ro db’ (All the codes are two letter codes) 129. What is the code for ‘create’ in the given code language? (1) sh (2) db (3) nt (4) ro (5) pi

130. What does ‘ro’ stand for in the given code language? (1) chaos (2) capital (3) huge (4) create (5) are 131. What does ‘ka’ stand for in the given code language? (1) either ‘cities’ or ‘crowded’ (2) cities (3) create (4) either ‘huge’ or ‘industries’ (5) chaos 132. Which of the following possibly means ‘crowded metro cities’ in the given code language? (1) sh be ju (2) sh be ka (3) sh ka nt (4) ka nt pi (5) sh un be 133. What is the code for ‘capital’ in the given code language? (1) pi (2) ju (3) sh (4) be (5) db 134. In a certain code HEALING is written as BFIKHOJ. How is BEDTIME written in that code? (1) EFCSJNF (2) EFCSFNJ (3) EFCUFNS (4) CFESFNJ (5) FEUCNSF (IBPS Specialist Officer (Marketing) CWE 01.02.2016)

Directions (135–139) : Study the following information carefully and answer the questions given below : (IBPS Specialist Officer (Marketing) CWE 01.02.2016)

In a certain code language, ‘urban people prefer cars’ is written as ‘ve fm ab eg’ ‘profit for urban areas’ is written as ‘ab ep zi so’ ‘people demand for hike’ is written as ‘zi qr cd we’ ‘hike in profit margin’ is written as ‘al nj ep cd’ (All codes are two letter codes only) 135. What will be the possible code for ‘urban food demand’ in the given code language? (1) qr ab nj (2) qr cr ab (3) nj qr cd (4) qr al nj (5) zi ve nj 136. What is the code for ‘areas’ in the given code language? (1) ab (2) zi (3) ep (4) qr (5) so

BPRE–60

137. What does the code ‘nj’ stand for in the given code language? (1) Either ‘hike’ or ‘people’ (2) Either ‘in’ or ‘margin’ (3) profit (4) hike (5) demand 138. In the given code language, if ‘small’ is coded as ‘wy’, then how will ‘prefer small cars’ be coded as? (1) wy eg ab (2) fm ve wy (3) eg wy fm (4) ab eg fm (5) ab wy eg 139. What will be the code for ‘hike’ in the given code language? (1) cd (2) zi (3) Other than those given as options (4) qr (5) nj Directions (140–144) : Study the following information carefully and answer the questions given below : (IBPS Specialist Officer (IT) CWE 14.02.2016)

In a certain code language, ‘gloves are found there’ is written as ‘vg xa ze co’ ‘two pairs of gloves’ is written as ‘sh yc ti xa’ ‘found two new friends’ is written as ‘wd vg nu sh’ ‘all packets are new’ is written as ‘om co nu qk’ (All codes are two letter codes only) 140. What does the code ‘ti’ stand for in the given code language? (1) either ‘two’ or ‘new’ (2) are (3) gloves (4) either ‘of’ or ‘pairs’ (5) two 141. What will be the code for ‘found’ in the given code language ? (1) ze (2) wd (3) sh (4) yc (5) vg 142. What is the code for ‘are’ the given code language ? (1) ze (2) co (3) qk (4) om (5) ti 143. What will be the possible code for ‘new writer gloves’ in the given code language ? (1) xa nu dy (2) xa dy vg (3) ze xa wd (4) ze sh om (5) nu ti xa

CODING / DECODING 144. In the given code language, if ‘blue’ is coded as ‘af’, then how will ‘all blue packets’ be coded as ? (1) nu vg af (2) co qk om (3) af om qk (4) om af co (5) af co nu 145. In a certain code language, ‘PLOTS’ is written as ‘NJSRQ’, ‘SNACK’ is coded as ‘QLEAI’, how is FUDGE written in that code? (1) DSHEC

(2) DWHKJ

(3) DWHEG

(4) DWHEC

(5) DWHKG (SIDBI Officer Online Exam.24.02.2016)

Directions (146–150) : Study the following information carefully and answer the questions given below : (SIDBI Officer Online Exam.24.02.2016)

In a certain code language, ‘robots can become lawyers’ is written is ‘ju ac th eg’. ‘all doctors can check’ is written as ‘bd np ju mo’ ‘many doctors and lawyers’ is written as ‘np rt qs ac’ ‘machine check of robots’ is written as ‘eg ik ux bd’ (Note : All codes are two letter codes only) 146. What does the code ‘ac’ stand for in the given code language? (1) robots (2) many (3) lawyers (4) become (5) of 147. If ‘how and many’ is written as ‘qs ws rt’ then what will be the code for ‘how can’? (1) np qs (2) rt ju (3) qs rt (4) ws ju (5) bd ws 148. What is the code for ‘machine’ in the given code language? (1) rt (2) Either ‘eg’ or ‘rt’ (3) bd (4) eg (5) Either ‘ik’ or ‘ux’ 149. What is the code for ‘robots’ in the given code language? (1) ux (2) eg (3) ju (4) Either ‘eg’ or ‘ju’ (5) Either ‘ju’ or ‘ac’

150. What may be the possible code for ‘will become doctor’ in the given code language? (1) th zi np (2) zi ac bd (3) np th ux (4) zi np bd (5) bd ju ux 151. In a certain code language ‘TOASTER’ is written as ‘VQCRRCP’ and ‘RODENTS’ is written as ‘TQFDLRQ’. How will ‘PHANTOM’ be written in that code language? (1) RJCMVQO (2) NFYORMK (3) RJCMRMK (4) NFYOVQO (5) Other than those given as options (United Bank of India PGDBF Manipal Exam,07.08.2016)

Directions (152–157) : Study the following information carefully and answer the questions given below : (United Bank of India PGDBF Manipal Exam,07.08.2016)

In a certain code language, ‘space shuttle launch tomorrow’ is written as ‘va ki lo na’ ‘launch the space programme’ is written as ‘dc na bk ki’ ‘tomorrow at the station’ is written as ‘ep dc mj va’ ‘follow the station directives’ is written as ‘pa dc su ep’ (All the codes are two letter codes only) 152. What does the code ‘bk’ stand for in the given code language? (1) launch (2) space (3) the (4) programme (5) Either ‘space’ or ‘launch’ 153. What will be the code for ‘space launch station’ in the given code language? (1) ep na ki (2) ki pa lo (3) ep dc ki (4) na su va (5) mj va bk 154. What is the code for ‘tomorrow’ in the given code language? (1) ki (2) mj (3) va (4) dc (5) Either ‘va’ or ‘lo’ 155. In the given language if ‘rocket’ is coded ‘pw’, then how will ‘at the rocket station’ be coded as? (1) bk lo pw na (2) pw mj ep dc (3) su pa ki pw (4) dc pw va bk (5) pw ep su va

BPRE–61

156. What will be the possible code for ‘missiles launch tomorrow’ in the given code language? (1) ki va pa (2) na su mj (3) na lo ki (4) ki dc ep (5) va na rt 157. As per the given information the code for which of the following words cannot be known definitely? (1) Only ‘launch’ (2) Only ‘space’ (3) Only ‘programme’ (4) Both ‘space’ and ‘launch’ (5) Both ‘station’ and ‘shuttle’ Directions (158–162) : Study the the following information carefully and answer the questions given below : (Bank of Maharashtra PO Exam, 26.10.2016)

In a certain code language : ‘solving major political issues’ is written as ‘za ke uc pi’ ‘some issues rather important’ is written as ‘nt ah za mh’ ‘important political intervention required’ is written as ‘cv nt pi wb’ ‘solving rather than raising’ is written as ‘sr mh rd uc’ 158. What is the code for ‘major’ in the given code language ? (1) nt (2) za (3) uc (4) pi (5) ke 159. Which of the following may represent ‘some important deals’ in the given code language ? (1) nt mh vl (2) ah nt mh (3) nt pi ah (4) nt ds ah (5) nt za ah 160. What does the code ‘mh’ stand for in the given code language ? (1) raising (2) rather (3) than (4) solving (5) intervention 161. ‘wb kp cv’ could be code for which of the following in the given code language ? (1) required useful intervention (2) issues required intervention (3) required major raising (4) same intervention required (5) raising intervention intentionally 162. What is the code for ‘political’ in the given code language ? (1) wb (2) cv (3) pi (4) za (5) nt

CODING / DECODING 163. In a certain code language, ‘send the tests’ is coded as ‘al vx se’ and ‘all tests solved’ is coded as ‘se pg nb’. How will ‘tests’ be coded as in the given code language? (Note-all codes are two letter codes only) (1) nb (2) vx (3) Either ‘nb’ or ‘pg’ (4) se (5) Either ‘al’ or ‘pg’ (IBPS RRBs Officer CWE (Pre.) Exam, 14.11.2016 (Shift-II))

164. In a certain code language, ‘give me call’ is coded as ‘jo kl mx’ and ‘call for me’ is coded as ‘mx jo st’. How will ‘for’ be coded as in the given code language? (Note : all codes are two letter codes only) (1) Either ‘mx’ or ‘jo’ (2) Either ‘jo’ or ‘kj’ (3) kj (4) mx (5) st (IBPS RRBs Officer CWE (Pre.) Exam, 14.11.2016 (Shift-III))

Directions (165–168) : Read the given information carefully and answer the questions given below : (IBPS Bank PO/MT CWE (Main) Exam, 18.11.2016

In a certain code language, ‘enjoy your trip immensly’ is written as ‘U s j 16 F o k p 25 J n n f o t m 25 z p v 18” ‘economic journey has a good feeling’ is written as ‘B k p v s o f 2 5 G f f m j o 7 H p p 4 I b 19 F d p o p n j 3.’ ‘People like to travel abroad’ is written as U s b w f 1 2 Q f p q m 5 B c s p b 4 U 1 5 M j l 5’ 165. Which among the following is the code for ‘wish you a happy journey’ ? (1) k p v s o f 25 20 21 x j t 8 B I b q q 25 (2) I b q q 25 B X j t 8 Z p 21 K p v s o f 25 (3) B k p u s p f 26 I b q q 25 x j t 82 p 21 (4) v h r 8 x n 21 I n t q m d 25 G z 0 0 25 B (5) None of these 166. What will be the code for ‘make an economic trip’ ? (1) ssj21 zm Lyj5 Dbnmnh3 (2) Usj16 B14 Nbj5 Fdpopj3 (3) B14 Usj16 Nb15 Fdpopnj3 (4) Fdpopnj3 B14 Nbl5 Utj16 (5) None of these

167. Which among the following is the code of ‘Ljo4 Hp4 Bm12 J19 U15’ ? (1) God is kind of all (2) God makes all kind. (3) God is kind to all (4) Cannot be determined (5) None of these 168. Which among the following is the code for ‘keep smiling always’ ? (1) Bmxbz24 Lff16 Tmjnj07 (2) Lff16 Bmxbz22 Tnjm07 (3) Tnjmj07 Bmxbz19 Lff16 (4) Tnjnj07 Bnxbz24 Lff17 (5) None of these Directions (169–173) : Study the following information carefully and answer the questions given below : (Indian Bank PO (Pre.) Exam, 21.01.2017 (Ist Sitting))

In a certain code language, ‘plot for all persons’ is written as ‘fn bo dl sw’ ‘find the hidden plot’ is written as ‘dl et ga nu’ ‘try and find out’ is written as ‘ga yc mp zh’ ‘for the lock out’ is written as ‘nu mp fn rv’ (All codes are two-letter codes only) 169. If ‘try the key’ is coded as ‘nu ka yc’ in the given code language, then how will ‘key and lock’ be coded as? (1) ka bo zh (2) zh ga ka (3) Other than those given as options (4) bo rv ga (5) rv zh ka 170. In the given code language, what does the code ‘sw’ stand for? (1) Either ‘for’ or ‘find’ (2) Either ‘persons’ or ‘all’ (3) hidden (4) for (5) plot 171. Which of the following may represent the code for ‘find friends’ in the given code language? (1) bo ga (2) fn ga (3) ga cl (4) et bo (5) cl et 172. What is the code for ‘for hidden plot’ in the given code language? (1) et fn dl (2) fn dl bo (3) ga bo fn (4) mp rv et (5) dl et ga 173. What is the code for ‘out’ in the given code language? (1) fn (2) nu (3) rv (4) mp (5) Other than those given as options

BPRE–62

Directions (174–176) : Study the following information carefully and answer the questions given below : (Indian Bank PO (Pre.) Exam, 21.01.2017 (2nd Sitting))

‘make your own food’ is coded as ‘ri fu ja tu’ ‘way for your bus’ is coded as ‘cp wy ri md’. ‘make way for bus’ is coded as ‘fu wy md cp’ ‘own food no way’ is coded as ‘md ne tu ja’ (All the codes are two letter codes only) 174. How is ‘bus’ coded in the given code language ? (1) cp (2) wy (3) ‘cp’ or ‘wy’ (4) fu (5) md 175. If ‘our own food’ is coded as ‘ja su tu’, then which of the following could be the possible code for ‘make our way’ ? (1) md tu su (2) fu su cp (3) md su ri (4) su fu md (5) Other than those given as options 176. What does the code ‘no’ stand for in the given code language ? (1) wy (2) md (3) ja (4) tu (5) ne Directions (177–181) : Study the following information carefully and answer the questions given below : (IBPS RRBs Officers CWE (Prelim Exam) 09.09.2017 Ist Sitting)

In a certain code language, ‘cheque cleared from bank’ is written as ‘ru dg te ac’ ‘went to bank yesterday’ is written as ‘mx vl ib ru’ ‘yesterday received new cheque’ is written as ‘oj dg vl hp’ ‘new deposit from account’ is written as ‘sf hp nz te’ (All codes are two letter codes only) 177. In the given code language, what does the code ‘mx’ stand for? (1) cleared (2) either ‘bank’ or ‘new’ (3) either ‘to’ or ‘went’ (4) yesterday (5) bank

CODING / DECODING 178. Which of the following may represent the code for ‘dishonoured cheque’ in the given code language? (1) eq ac (2) dg eq (3) ac oj (4) oj ru (5) ru dg 179. What is the code for ‘cleared yesterday’ in the given code language? (1) ac dg (2) dg ru (3) ib vl (4) ru ib (5) vl ac 180. If ‘deposit received cash’ is coded as ‘oj yi sf’ in the given code language, then how will ‘new cash account’ be coded as? (1) yi nz hp (2) hp ib nz (3) dg hp vl (4) ib dg vl (5) nz yi dg 181. What is the code for ‘bank’ in the given code language? (1) vl (2) ac (3) oj (4) ru (5) hp Directions (182–186) : Study the following information carefully and answer the questions given below : (IBPS RRBs Officers CWE (Prelim Exam) 09.09.2017 IInd Sitting)

In a certain code language, ‘expecting five times profit’ is written as ‘vb yh nu ji’ ‘profit increases every year’ is written as ‘de lo ji km’ ‘every business expecting boom’ is written as ‘sz km wa yh’ ‘boom required this year’ is written as ‘pg cq lo sz’ (All codes are two letter codes only) 182. In the given code language, what does the code ‘km’ stand for? (1) every (2) business (3) boom (4) increases (5) year 183. In the given code language, what does the code ‘de yh’ stand for? (1) increases this (2) increases boom (3) boom profit (4) this expecting (5) expecting increases 184. Which of the following may represent the code for ‘boom market’ in the given code language? (1) lo km (2) oj sz (3) yh oj (4) km yh (5) sz lo

185. What is the code for ‘required’ in the given code language? (1) sz (2) either ‘sz’ or ‘lo’ (3) wa (4) either ‘pg’ or ‘cq’ (5) nu 186. If ‘five year profit’ is coded as ‘lo ji nu’ in the given code language, then ‘times for business’ may be coded as? (1) sz rj vb (2) wa nu rj (3) xl vb wa (4) xa wa cq (5) vb cq wa Directions (187–191) : Study the following information carefully and answer the questions given below : (IBPS RRBs Officers CWE (Prelim Exam) 10.09.2017)

In a certain code language, ‘chat requires more patience’ is written as ‘ zi mx ub cv’ ‘patience in every situation’ is written as ‘wp ko dl mx’ ‘every chat saved online’ is written as ‘ub dl ha tg’ ‘situation saved from crisis’ is written as ‘tg rq se ko’ (All codes are two letter codes only) 187. If ‘more chat saved’ is coded as ‘ub zi tg’ in the given code language, then ‘requires every possibility’ may be coded as ? (1) qt cv dl (2) dl qt ko (3) cv dl mx (4) ko ur qt (5) mx ur cv 188. What is the code for ‘crisis’ in the given code language ? (1) tg (2) ha (3) either ‘se’ or ‘rq’ (4) ko (5) either ‘ko’ or ‘zi’ 189. Which of the following may represent the code for ‘no patience’ in the given code language ? (1) cv wp (2) mx bj (3) bj zi (4) wp mx (5) mx se 190. In the given code language, what does the code ‘ko’ stand for ? (1) in (2) more (3) from (4) situation (5) every 191. In the given code language, what does the code ‘ha wp’ stand for ? (1) chat in (2) online from (3) every chat (4) from every (5) in online

BPRE–63

Directions (192–196) : Study the following information carefully and answer the questions given below : (IBPS Bank PO/MT CWE-VII (Prelim Exam) 14.10.2017)

In a certain code language, “with all caring friends” is written as “bs up ck lq” “all the time dancing” is written as “av tn lq og”. “friends dancing and singing” is written as “tn dz up ry” “caring and loving people” is written as “dz xn bz eh” (Note : All the codes are two letter codes only) 192. How may “all singing song” be coded as in the given code language? (1) ry sf lq (2) ry bs up (3) av lq ry (4) sf tn ry (5) ry lq eh 193. What is the code for “time” in the given code language? (1) Either ‘lq’ or ‘tn’ (2) up (3) tn (4) Either ‘av’ or ‘og’ (5) Either ‘xn’ or lq’ 194. If “people dancing together” is codes as “tn gi xn”, then how will “loving friends together” be coded as in the given code language ? (1) av dz up (2) eh gi dz (3) gi up lq (4) eh dz up (5) gi up eh 195. What does the code ‘ck’ stand for in the given code language ? (1) friends (2) all (3) with (4) singing (5) caring 196. What does ‘bs dz’ stand for in the given code language? (1) and with (2) and caring (3) caring with (4) with the (5) and the Directions (197–201) : Study the following information carefully and answer the questions given below : In a certain code language, (IBPS SO (IT Officer) CWE (Prelim Exam) 30.12.2017)

‘give me your number’ is coded as ‘tm ct ar bp’ ‘highest number in English’ is coded as ‘bp dg sk mi’

CODING / DECODING ‘test your English skills’ is coded as ‘tm fo ve dg’ ‘skills in pencil sketches’ is coded as ‘nu ve ky sk’ (Note : All codes are two letter codes only) 197. What is the code for ‘English’ in the given code language? (1) dg (2) tm (3) nu (4) ky (5) ve 198. What does the code ‘ct stand for in the given code language? (1) test (2) Either ‘give’ or ‘me’ (3) Either ‘your’ or ‘number’ (4) number (5) highest 199. What may be the possible code for ‘your hidden skills’ in the given code language? (1) mi tm fo (2) ve fo tm (3) ve rb sk (4) ve rb tm (5) rb tm ky 200. If in the given code language, ‘colour of pencil’ is coded as ‘nu xg hf’, what will be the code for ‘number of sketches’? (1) bp xg ve (2) hf fo xg (3) fo tm hf (4) ky tm nu (5) Cannot be determined 201. What does the code ‘fo’ stand for in the given code language? (1) None of the given options (2) your (3) test (4) skills (5) English Directions (202–204) : Study the following information carefully and answer the questions given below : (IBPS SO (Law Officer) CWE (Prelim Exam) 31.12.2017)

In a certain code language, ‘deposited cheque today’ is coded as ‘fx tb jw’ ‘today open locker’ is coded as ‘tb qs ka’ ‘branch open tomorrow’ is coded as ‘ka dm rc’ ‘withdraw money tomorrow’ is coded as ‘rc yb gp’ (Note : All the codes are two letter codes only) 202. What does the code ‘rc ka’ stand for in the given code language? (1) open tomorrow (2) branch cheque (3) branch tomorrow (4) many cheque (5) open cheque

203. What may be the possible code for ‘cheque deposited in branch’ in the given code language ? (1) dm tb rc gp (2) jw ka qs fx (3) tb rc jw dm (4) dm fx jw ve (5) fx rc ka jw 204. In the given code language, if ‘withdraw from’ is coded as ‘gp km’, then how will ‘money from bank’ be possibly coded as in the given code language? (1) uh yb rc (2) dm km yb (3) uh ka yb (4) yb km uh (5) km tb rc Directions (205–208) : Study the the following information carefully and answer the questions given below : (IDBI Bank PO Exam 29.04.2018)

In a certain code ‘here is tunnel she stop’ is written as ‘isa kin ha ti la’, ‘he goes through tunnel’ is written as ‘nit ti pi sit’, ‘she goes here often’ is written as ‘sit la tin isa’ and ‘tunnel is far through here’ is written as ‘ha nit la ti fa’ 205. Which of the following is the code for ‘tunnel’ ? (1) nit (2) ti (3) la (4) na (5) None of these 206. What does code ‘sit’ stand for? (1) tunnel (2) through (3) he (4) Cannot be determined (5) goes 207. ‘he is often’ can be coded as __________. (1) sa pi la (2) fa tin ha (3) pi ha tin (4) kin tip sit (5) None of these 208. Which of the following is the code for ‘goes far stop here’? (1) sit la fa kin (2) la fa pi tin (3) ha kin sit nit (4) Cannot be determined (5) None of these Directions (209–213) : Study the following information carefully and answer the questions given below : (IBPS Bank PO/MT CWE-VII (Prelim Exam) 15.10.2017)

In a certain code, ‘detail event year this’ is written as ‘bi gv oc st’

BPRE–64

‘event revised of awaited’ is written as ‘tm oc da pu’ ‘of detail results first’ is written as ‘nh mk tm gv’ ‘awaited great year of’ is written as ‘da st rx tm’ (All codes are two letter codes only) 209. In the given code language, what does the code ‘mk’ stand for? (1) either ‘detail’ or ‘year’ (2) this (3) of (4) Either ‘first’ or ‘results’ (5) great 210. What is the code for ‘awaited’ in the given code language? (1) da (2) rx (3) st (4) tm (5) Other than those given as options 211. What is the code for ‘event’ in the given code language? (1) nh (2) gv (3) oc (4) pu (5) rx 212. What may be the possible code for ‘awaited revised’ in the given code language? (1) ve be (2) da nh (3) nh ve (4) bi da (5) pu da 213. If ‘of year home’ is written as ‘aj tm st’ in the given code language, then what is the code for ‘home revised this’ ? (1) aj gv nh (2) pu aj bi (3) nh bi pu (4) bi da aj (5) aj oc pu 214. In a certain code language, CLIMATE is coded as JMDNFUB and BROUGHT is coded as PSCVUIH. How is FACTORY coded as per the given code language? (1) DBGSZSP (2) DBGUZSP (3) BBEUXTQ (4) GBDUPSZ (5) BBESQTX (IBPS RRBs Officer CWE (Prelim Exam) 11.08.2018)

Directions (215–219) : Study the following information carefully and answer the questions given below : In a certain code language : ‘you have other choices’ is coded as ‘lo se ca pr’ ‘they have old options’ is coded as ‘tu bd pr nk’ ‘manager wants old choices’ is coded as ‘af lo bd gi’

CODING / DECODING ‘employees wants new options’ is coded as ‘nk gi hm jv’ (Note : all the given codes are two letter codes only) (IBPS RRBs Officer CWE (Prelim Exam) 11.08.2018)

215. What is the code for ‘options’ in the given code language? (1) bd (2) nk (3) pr (4) tu (5) gi 216. What will be the possible code for ‘you are manager’ in the given code language? (1) af se wq (2) ca se af (3) se af jv (4) pr jv wq (5) jv ca lo 217. What is the code for ‘new’ in the given code language? (1) gi (2) se (3) Either ‘ca’ or ‘af’ (4) Either ‘hm’ or ‘jv’ (5) nk 218. What is the code for ‘have’ in the given code language? (1) lo (2) Either ‘se’ or ‘ca’ (3) pr (4) Either ‘nk’ or ‘se’ (5) ca 219. What does the code ‘tu’ stand for in the given code language? (1) have (3) options (3) old (4) wants (5) they Directions (220–224) : Study the following information carefully and answer the questions given below : In a certain code language, ‘confused about meeting friends’ is written as ‘de cv mo st’ ‘friends asked over phone’ is written as ‘ji mo np bq’ ‘phone call about meeting’ is written as ‘st zr de ji’ ‘confused people over think’ is written as ‘np ha cv fu’ (All codes are two letter codes only) (Indian Bank PO Exam, 06.10.2018)

220. If in the given code language, ‘think noble’ is coded as ‘ha wy’ then ‘noble people call’ will be coded as_____________. (1) zr wy fu (2) wy np cv (3) cv fu de (4) de ji np (5) fu de wy

221. In the given code language, what does the code ‘st’ stand for? (1) friends (2) either ‘meeting’ or ‘about’ (3) confused (4) phone (5) either ‘friends’ or ‘over’ 222. What is the code for ‘asked’ in the given code language? (1) np (2) ji (3) bq (4) mo (5) de 223. Which of the following may represent the code for ‘friends buy phone’ in the given code language? (1) mo ji zr (2) np pz zr (3) cv pz ha (4) pz ji mo (5) ji cv pz 224. What is the code for ‘confused’ in the given code language? (1) de (2) mo (3) np (4) fu (5) cv Directions (225-229) : Study the following information carefully and answer the questions given below: In a certain code language, ‘place satellite in orbit’ is written as ‘jo ki la si’ ‘orbit moon in circle’ is written as ‘bp la mi jo’ ‘circle earth through satellite’ is written as ‘si dv tu bp’ ‘moon navigation gone through’ is written as ‘js mi dv hm’ (All the codes are two letter codes only) (IBPS Bank PO/MT CWE (Prelim Exam) 21.10.2018)

225. What is the possible code for ‘place through committee’ in the given code language? (1) mi ki hm (2) kl dv jo (3) to dv tu (4) ty dv ki (5) dv mi ki 226. What is the code for ‘satellite’ in the given code language? (1) la (2) ki (3) si (4) bp (5) jo 227. What is the code for ‘earth’ in the given code language? (1) mi (2) si (2) dv (4) bp (5) tu 228. What does the code ‘hm’ stand for in the given code language? (1) earth

BPRE–65

(2) Either ‘navigation’ or ‘gone’ (3) Either ‘moon’ or ‘through’ (4) through (5) moon 229. Which of the following may stand for ‘jo wk la’ in the given code language? (1) launch in orbit (2) orbit the moon (3) orbit in circle (4) satellite in navigation (5) orbit moon closely Directions (230–234) : Study the following information carefully and answer the questions given below: In alphabetical series A - Z letter except vowels is assigned a different number from 1-6 (for example -B is coded as 1, C-2..... H-6) and again those numbers get repeated (for example - J-1, K-2...... and so on). Also each vowel is coded with different letters - p, q, r, s and t respectively. As per the code language, ECONOMIC ROW is written as “@2s5s4r@ &s&” and WE WIN is written as “q6 &r&”. Besides the above example, following operations are to be applied for coding the words given in the questions below. (i) If first letter is consonant and last letter is vowel then the codes of both of them will be interchanged. (ii) If both first and last letters are vowels then they are to be coded as $. (iii) If the first letter of a word is vowel and the last letter is consonant then both are to be coded as @. (iv) If both first and last letters are consonants then they are to be coded as &. (If the word does not satisfy the conditions given above then the letters of that word are to be coded as per the directions given above). (IBPS Bank PO/MT CWE (Main Exam) 18.11.2018)

230. What is the code for “IMAGE”? (1) &p54& (2) $4p5$ (3) rp54q (4) s612r (5) None of these 231. What is the code for “ROAR”? (1) 2sp2 (2) $ps$ (3) &s5$ (4) &sp& (5) None of these

CODING / DECODING 232. What is the code for “OASIS”? (1) sp3r3 (2) &p3r& (3) sp&r5 (4) @p3r@ (5) None of these 233. What is the code for “INSURANCE”? (1) &53t2p52& (2) r53t2p52q (3) $53t2p52$ (4) @53t2p52@ (5) None of these 234. What can be the word for “&p6q2&”? (1) Reader (2) Rotten (3) Papers (4) Pokers (5) Reality 235. How is “home” written in a certain code language? Statement I: “go to home” is written as tsa la da “and” on the home” is written as “ni da ka” Statement II: “way to market” is written as “sh ma pi” and “way to home” is written as “da pi ma” (1) Data in either statement I alone or statement II alone is sufficient to answer the question (2) Data in only statement II is sufficient to answer the question (3) Data in only statement I is sufficient to answer the question (4) Data in statement I and statement II together are not sufficient to answer the question (5) Data in statement I and statement II together are sufficient to answer the question (IBPS Bank PO/MT CWE (Main Exam) 18.11.2018)

Directions (236-240): Study the following information carefully and answer the questions given below : In a certain code language., “Every human has virtues” is written as “&a18 ?u23 +v11 @i45” “Honesty is the best policy” is written as “>h10 /o55 # e73 =s19/ o55” “Ethical moral society” is written as “$o19 *t39 $o3” “Dynamism and values of culture“ is written as” h10 $o19 (2) @i45 $o13 (3) ?u23 $o19 (4) ?u23 $o13 (5) &a18 *t39 237. What is the code for “Best Culture”? (1) >h10 h10 $o19 (3) #e73 $o13 (4) >h10 $o13 (5) = s19 *t39 240. What is the code for “Every Dynamism”? (1) @i45 *f29 (2) ?u23 h10 Moral ⇒ $o13 240. (3) Every ⇒ +v11 Dynamism ⇒ &y49 241. (3) P E O P L E R H S T O G +2 +3 +4 +4 +3 +2 U S I E N U S P E A K S +2 +3 +4 +4 +3 +2 Therefore, U H G S Q F S E C O N D +2 +3 +4 +4 +3 +2 (242-246) :

E $

P A P E R S Condition (iv) is applicable 235. (1) From statement I go

to

on

the

home

sa

home From statement II

ni

da

way to market

sh

ma pi

way to

da

home

virtues @i45 best policy #e73 /o55

Dynamism and values of culture &y 49 h10 society $o19

la

da ka

pi ma

BPRE–90

for

dance

prize

awarded

vb

dr

cs

for

the

best

choreographer

lt

vb

ua

mq

the

prize

ph

ez

medal

awarded

on

cs

yn

kl

distribution fj

ph

held lt

distribution

ez

CODING / DECODING 242. (3) best ⇒ mq prize ⇒ ph Now, choreographer ⇒ ua for ⇒ vb 243. (1) dance ⇒ dr The code for ‘show’ may be ‘xw’. 244. (2) awarded ⇒ cs 245. (5) lt ⇒ the 246. (4) medal ⇒ yn distribution ⇒ ez yesterday ⇒ og Now, yesterday ⇒ og held ⇒ fj on ⇒ kl (247–251) : ● The code for each word consists of three elements : a symbol, a letter and a number. ● There are two types of symbols : $ and %. The symbol denotes number of letters in the given word. If the number of letters is even, symbol $ has been used and if the number of letters is odd, symbol % has been used. ● The letter in the code is based on the opposite letter of the second letter from the right end. ● The number in the code is the position number of the third letter from the left end. Earth : (i) Number of letters : 5 (Odd) ⇒ % (ii) Second letter from the right end ⇒ t Its opposite letter is G. (iii) Third letter from the left end ⇒ r Its position number is 18. Thus, code for Earth is % G18. The codes for the other words are as follows : Laughs ⇒ $S21 With ⇒ $G20 Flowers ⇒ %I15

Become What

You

(252–256) :

$Z1 %L21 %E12

Reach

Your

Own Stars

$F21 %D14 %I1

247. (2) Wisdom : (i) Number of letters = 6 (Even) ⇒ $ (ii) Second letter from the right end = O Its opposite letter is L. (iii) Third letter from the left end ⇒ S = 19 Therefore, Wisdom ⇒ $L19 248. (4) $Z1 ⇒ What

went

for

picnic

rm

ub

az

enjoyed

yesterday qc

picnic

with

friends

dv

rm

fh

nk

Believe

$N3

%X1

249. (1) Imagination : (i) Number of letters = 11 (Odd) ⇒ % (ii) Second letter from the right end ⇒ O Its opposite letter is L. (iii) Third letter from the left end ⇒ a ⇒ 1 Therefore, Imagination ⇒ %L1 250. (3) % ⇒ Odd number of letters I ⇒ Its opposite letter is R. 3⇒C Therefore, %I3 ⇒ Victory 251. (1) Dreams : (i) Number of letters = 6 (Even) ⇒ $ (ii) Second letter from the right end ⇒ m Its opposite letter is N. (iii) Third letter from the right end ⇒ e = 5 Humanity : (i) Number of letters = 8 (Even) ⇒ $ (ii) Second letter from the right end ⇒ t Its opposite letter is G. (iii) Third letter from the left end ⇒ m = 13 Therefore, Dreams ⇒ $N5 Humanity ⇒ $G13

yesterday

friends

tl

xe

met

for

wj

tl

met together

movie there io

252. (1) nk ⇒ enjoyed/with 253. (2) went ⇒ ub movie ⇒ io Now, friends ⇒ dv there ⇒ wj 254. (3) together ⇒ xe for ⇒ qc The code for ‘dinner’ may be ‘vi’. 255. (4) met ⇒ tl 256. (5) rm ⇒ picnic ‘ef’ may stand for ‘in’.

BPRE–91

az

dv

qc

CODING / DECODING (257–261) :

(267–271) :

the

box

is

filled

jr

sm

aq

eb

Live Live

only

one

filled

can

eb

il

kw

vc

can

is

only

brown

sm

vc

po

il

the

board

is

box

sm

dh

aq

jr

257. (3) dh ⇒ board 258. (3) filled ⇒ eb 259. (3) il ⇒ can/only 260. (5) one ⇒ kw brown ⇒ po

Like

A

Good

constructed high

A

King

Last

King

jx

ve

gl

fu

rise

in

the

sky

mo

gl

ba

hy

constructed

in

recent

years

dn

cz

ve

ba

over

the

past

years

rl

dn

ps

mo

mu

pe

ka

su

ka

pe

ke

na

da ku

ke

ra

mu

su

267. (4) Good Day ⇒ da ku The code for ‘Was’ may be ‘pa’. 268. (5) Good ⇒ da/ku King ⇒ ke 268. (3) A ⇒ su Day ⇒ da What ⇒ de Now, What ⇒ de Good ⇒ ku King ⇒ ra 270. (1) na ⇒ Be 271. (2) ka ⇒ Live (272 – 276) : pay

charge

government function

building

ra

pe

Today

Day

officers

rise

frame

tax

13

60

33 11

army

50

protect

policy

85

tax

72

84 60

tax

15 60

84 17

public

pay

police

protect

public

85

84 20

officers

execute function

13

33 23

politicians frame policy protect

country

272. (1) 84 ⇒ public 50 ⇒ frame 12 ⇒ country 273. (4) 11 ⇒ government 38 ⇒ charge 84 ⇒ public 274. (5) politicians ⇒ 18 pay ⇒ 72 officers ⇒ 33 275. (2) army ⇒ 15 protect ⇒ 85 police ⇒ 20 276. (3) government ⇒ 11 officers ⇒ 33 pay ⇒ 72

BPRE–92

38 72

public

army

262. (4) constructed ⇒ ve 263. (4) sky ⇒ hy The code for ‘blue’ may be ‘gx’. 264. (2) recent ⇒ cz rise ⇒ gl 265. (3) ba ⇒ in 266. (5) years ⇒ dn high ⇒ fu over ⇒ ps Now, the ⇒ mo past ⇒ rl building ⇒ jx

Last

Like

Like

Be

is ⇒ sm The code for ‘blue’ may be ‘ng’. 261. (5) the ⇒ aq/jr (262–266) :

Today

50 17

18

15

85

12

CODING / DECODING 277. (3)

3

H O U S

6

4

2

8

E S

L U A W I W

9 +4 NV

F T D L H F Condition (iv) is applicable. 278. (1)

5 K

2 L

1 G

4 D

3

NV +4 NV +4

7

F

The code has been generated on the following pattern : Consonant + 4 ⇒ Code Vowel ⇒ Next Vowel (NV) as code Therefore,

K

Condition (iv) is applicable. 279. (4)

3

9

2

6

4

8

A

B

L

T

D

A

D E C I

D E

H I G O H I +4 NV +4

Condition (ii) is applicable. 280. (5)

NV

2

7

9

6

5

4

D

R

B

T

K

D

+4 NV

SBI PO EXAMS

Condition (iii) is applicable. 281. (2)

7

2

5

6

(1–6) : a lw a ys crea te n ew id ea s

3

8

ideeaass a n d id

cre ate though ts

A L K T F A Condition (ii) is applicable. (282–284) : B:A+B ●A ●A | B : A – B ●A $ B : A × B ●A @ B : A ÷ B 282. (2) (S|C) $ 100 @ C ⇒ (S – C) × 100 ÷ C 283. (1) MA @ CA $ 100

n ew

and

tfa

thho u g h ts and

insights

gi m a

ri

m a jo b a ki

b etter solution s

sha g i

ri

ri

to

tfa

tfa

1. (3) The code for ‘ideas’ is ‘gi’ 2. (4) The code ‘fa’ stands for ‘and’ 3. (2) fa ⇒ and lo ⇒ may be code for innovate ba ⇒ create 4. (2) The code for ‘new’ is ‘ri’ 5. (4) insights ⇒ jo always ⇒ sha better ⇒ ki/to

MA × 100 CA 284. (5) (P 2 | P 1) @ P 1 $ 100

6. (1) The code for ‘thoughts’ is ‘ma’ (7–11) :





n ethw

ba

economics

bP2 – P1g × 100

is

not

money

ka

la

ho

ga

P1

285. (4) L O C A T E

demand

and

supply

economics

mo

ta

pa

ka

money

makes

only

part

zi

la

ne

ki

economics

zi

mo

ka

ta

P U G E X I +4 NV +4 NV

demand

makes supply

+4 NV

7. (5) The code for ‘money’ is ‘la’.

BPRE–93

CODING / DECODING 20. (3) we ⇒ 2 skip ⇒ & this ⇒ $ 21. (1) skip ⇒ & 22. (2) can ⇒ 4 skip ⇒ & that part ⇒ 3 9

8. (5) The code for ‘supply’ is either ‘mo’ or ‘ta’ 9. (1) demand ⇒ ‘mo’ or ‘ta’ only ⇒ ‘ne’ or ‘ki’ The code for ‘more’ may be ‘xi’ 10. (2) and ⇒ pa money ⇒ la The code for ‘work’ may be ‘tu’ 11. (4) The code for ‘makes’ is ‘zi’ 12. (4) I N S T E A D

HMRUFBE –1 –1 –1 +1 +1 +1 +1

(23-24) :

(I) she likes apples

pic

sip

dip

(II) parrot

dip

pic

tif

tif

sip

dip

likes apples lots

(III) she likes

parrots

nit

23. (3) The code for ‘parrot’ is ‘tif’ 24. (2) (I) she likes apples (II) parrot likes apples lot

OGQBTFE

pic sip dip dip pic tif nit

The code for ‘parrot’ may be ’tif’ or ’nit’.

–1 –1 –1 +1 +1 +1 +1

(I) she

likes apples parrots

(II) she likes

pic sip dip tif sip dip

The code for parrot is ‘tif’. Thus, statement (II) is not necessary. (25–28) :

Therefore,

CNTCMFT –1 –1 –1 +1 +1 +1 +1

no

no

game played

rules

ro

sa

bi

ka

now

ka

za

bi

te

no

rules

given

ro

fo

ce

ka

now

we

were

given

ge

te

fo

li

13. (2) now ⇒ te 14. (4) game ⇒ bi 15. (1) ce ⇒ why 16. (3) rules ⇒ ro 17. (2) fo ⇒ given 18. (5) we ⇒ ‘ge’ or ‘li’ played ⇒ za simple ⇒ sa can we we that skip

create

lend to

part

that part

& 4 2 $

&

2

9 3 3

5 &

9

loans

more opportunity

banks for

vi

so

tk vi

all

loans

la hi

py

tk

rn

so

mn

dr

py

py

so

fd

py

tk

rn

so

25. (5) The code for ‘lend’ is either ‘tk’ or ‘vi’. 26. (3) Either ‘hi’ or ‘fd’ is the code of ‘for’. 27. (1) create more opportunity ⇒ la mn dr to ⇒ rn apply for ⇒ hi fd, loans ⇒ so 28. (4) banks ⇒ py (29–32) : all

banks

lend

loans

lend to

loans

all

create more opportunity

apply banks for

skip this

19. (5) that ⇒ 3 or 9

loans

banks

skip

was

lend

apply

why

(18-22) :

banks

banks

(13-18) : simple game

all

loans

vi

so

tk vi la hi

mn

dr

py

so

29. (5) The code for ‘lend’ is either ‘tk’ or ‘vi’. 30. (3) Either ‘hi’ or ‘fd’ is the code of ‘for’. 31. (1) create more opportunity ⇒ la mn dr to ⇒ rn apply for ⇒ hi fd loans ⇒ so 32. (4) banks ⇒ py

BPRE–94

py fd

CODING / DECODING (33–37) :

good

time

invest

to

money

only

work

buy

good

buy

stuff

bo

nj

kw

sy

ta

ge

mr

money

ta

fp

mr

ux

only

kw

and

and

sy time

bo

rd

fp

46. (4) shouted ⇒ bu ate ⇒ sh The code for ‘and’ may be ‘gy’. 47. (1) carrot ⇒ vg (48–52) : economic

growth

growth developing

33. (3) to ⇒ nj 34. (1) buy good ⇒ kw bo 35. (4) only ⇒ fp time ⇒ sy and money ⇒ ta mr 36. (2) stuff ⇒ rd 37. (5) invest ⇒ ge time ⇒ sy to ⇒ nj work ⇒ ux

conditions

kids is

teaching kids

down

ho je

ms bp kx cn dr

trend

bp

risk ⇒ zy

kj

te hy

tough

dp

op

cs mb

cs

re

mb gr

huge efforts

like

gi te ku ho

developing ⇒ ho

dp

huge

task required

economic conditions agenda

te

risk ⇒ zy Therefore, economic ⇒ ms

teaching

is

heart

country

cn fa ms

48. (4) slows ⇒ fa

(38–42) : efforts required in

developing

of

down

slows

hy kj

re

ba

down ⇒ cn 49. (3) conditions ⇒ bp of country ⇒ ku gi 50. (5) growth ⇒ te The code for ‘improve’ may be ‘cs’. 51. (2) kx ⇒ like/trend 52. (1) agenda ⇒ je 53. (4)

job

38. (2) kj ⇒ required/efforts 39. (4) teaching ⇒ dp

requires

expertise in

expertise

la nu si

area

li bo

la

40. (1) kids ⇒ cs/mb

requires

41. (5) in ⇒ te task ⇒ ba 42. (3) tough ⇒ op heart ⇒ gr The code for ‘quite’ may be ‘uc’.

rescue bugs carrot

shouted for all ate

the all

rescue

pr

co

54. (4) out of order

ja bi gf

order of court

st gf ja

out in court

dc st bi

ly bu

bugs

ke

mt co

rx

carrots

vg

rx ke

sh

for pretty Cinderella

inspection

si dm bo

inspection ⇒ dm

(43–47) : Cinderella

area

ly

pr

vg

The code for ‘order’ is either ‘ja’ or ‘gf’. 55. (5)

as

economic growth

43. (3) pr ⇒ Cinderella/for 44. (2) the ⇒ mt pretty ⇒ as

growth is

registered

registered

expected expected

qi number

Thus, the code for ‘economic’ is ‘jo’.

45. (5) bugs ⇒ either ‘ke’ or ‘rx’

BPRE–95

ve jo qi lo mn

lo ve pr

CODING / DECODING 56. (3) C R Y S T A L

E U C X Z H T +2 +3 +4

60. (4) i n s u r a n c e ↓ ↓ ↓ ↓ ↓ ↓ ↓ ↓ ↓ $ 5 3 w 2 c 5 2 $ Condition (ii) is applicable. (61–65) : (i) T

+5 +6

h e → @ h 08 e e t i n g → l e 48

M

+7 +8

Therefore,

F I G M E N T

H L K R K U B +2

a

d

journed→$

a

b

ruptly→#

(ii) S

i

nce→%

i 24

I

n

dia→%

n 24

g

a

ined→–

I

n

ternet→#

(iii) S

c

ientists→&

h

a

ve→*

d

i

b

a

cteria→#

(iv) C

u

rrent→l

d 80

b 63

+3 +4 +5 +6 +7

a 35 n 63 c 99

+8

a 15

(57–60) :

1

2

3

4

5

6

B

C D

F

G

H

J

K

L

M N

P

Q R

S

T

W

X

Z

Y

V

A, E, I, O, U ⇒ g, w, q, c, k e g

l 3

e g

c 2

t 4

i k

o q

s t a r t 3 4 c 2 4

n 5

l 3

n e 5 g

i k

v 5

e g

w 6

u

n

d

e

r

p

r

o

c

e

s

s

w

5

3

g

2

6

2

q

2

g

3

3

57. (4) g l o b a ↓ ↓ ↓ ↓ ↓ & 3 q 1 c Condition (iv) is applicable. 58. (1) t i t l e ↓ ↓ ↓ ↓ ↓ g k 4 3 4 Condition (i) is applicable. 59. (3) a d v i s e ↓ ↓ ↓ ↓ ↓ ↓ @ 3 5 k 3 g Condition (iii) is applicable.

l ↓ &

r ↓ @

s c o v er e d → @

a 63 u 48

e

c

onomic→#

s

c

enario→#

f

i

ne→*

i 99

c 63 c 63

i 15

61. (3) gained ⇒ – a 35 62. (5) Since ⇒ % i 24 meeting ⇒ l e 48 fine ⇒ * i 15 63. (2) & c 99 ⇒ scientists @ i 99 ⇒ discovered # n 63 ⇒ internet @ h o 8 ⇒ the 64. (1) economic ⇒ # c 63 internet ⇒ # n 63 65. (5) current ⇒ l u 48 (66–68) : The code has been generated by taking the immediate next letter of the first letter and writing the number of letters in the word. Entire ⇒ F6 Money ⇒ N5 Board ⇒ C5 Perfect ⇒ Q7 Sleeve ⇒ T6

BPRE–96

CODING / DECODING Washing ⇒ X7 World ⇒ X5 Stories ⇒ T7 Moving ⇒ N6 Partly ⇒ Q6 Falls ⇒ G5 Objects ⇒ P7 66. (1) Radio +1↓ S5 67. (3) Rising +1↓ S6 +1↓ Normal +1↓ O6 68. (5) They +1↓ U4 Forward +1↓ G7 T –1↓ S Number of letters ⇒ 5 South or Stone

(69-72) :

(73–77) : (D + F + N + S) – (E + E + E)

⇒ (4 + 6 + 14 + 19) – (5 + 5 + 5) ⇒ 43 – 15 = 28 (F + F + C + T) – (E + E)

⇒ (6 + 6 + 3 + 20) – (5 + 5) ⇒ 35 – 10 = 25 73. (4) (S + P + C + L) – (E + I + A) = (19 + 16 + 3 + 12) – (5 + 9 + 1) = 50 – 15 = 35 ⇒ D 74. (1) (C + L + G + Y) – (E + O + O) = (3 + 12 + 7 + 25) – ( 5 + 15 + 15) = 47 – 35 = 12 ⇒ A 75. (5) (C + N + M + Y) – ( E + O + O) = (3 + 14 + 13 + 25) – ( 5 + 15 + 15) = 55 – 35 = 20 ⇒ E 76. (3) (Q + T + N) – (E + U + A + I + O) = (17 + 20 + 14) – (5 + 21 + 1 + 9 + 15) = 51 – 51 = 0 ⇒ C 77. (1) (N + T + N + L) – (A + I + O + A) = (14 + 20 + 14 + 12) – (1 + 9 + 15 + 1) = 60 – 26 = 34 ⇒ A

RBI GRADE–B/NABARD GRADE–A OFFICER EXAMS (1–6) : Weapons hidden in town ¾® white black yellow red Ready weapons for attack ¾® hidden

roam

around

the

hills

mu

pa

jl

ko

hills

on

left

side

el

rf

pa

nt

left

to

roam

alone

jl

dg

el

hv

for

grey

indigo red green

own safety ¾® silver grey

violet white

own town under attack ¾® violet blue indigo black

1. (3) ready ⇒ green 2. (1) silver ⇒ safety 3. (5) black ⇒ town; ‘pink’ may be code for ‘risk’. yellow ⇒ in 4. (2) attack ⇒ indigo 5. (1) hidden ⇒ white; weapons ⇒ red ‘orange’ may be code for ‘all’. 6. (4) own ⇒ violet demand and supply market ¾® pa ni de re

alone

for

the

journey

bx

ko

oy

dg

Market needs more demand ¾® de ja ni fe

(7–12) : Supply demand is 69. (4) hills ⇒ pa 70. (3) for ⇒ oy alone ⇒ dg discovery ⇒ sz Now, discovery ⇒ sz to ⇒ hv journey ⇒ bx 71. (1) el ⇒ left ko ⇒ the 72. (2) side ⇒ rf/nt

related ¾® le de re ab

More related to economics ¾® ka ha ab ja

7. (5) The code for ‘economics’ is either ‘ka’ or ‘ha’. 8. (2)

re

ab

ni

supply related market 9. (4) more ⇒ ja 10. (1) market ⇒ ni needs ⇒ fe more ⇒ ja The code for ‘customers’ may be ‘sa’.

BPRE–97

CODING / DECODING 11. (2) needs ⇒ fe 12. (1) demand ⇒ de needs ⇒ fe supply ⇒ re (13–17) :

23. (1) ab ⇒ honour 24. (3) fq mz vh

economy

receiving

very

very

essence of

fast

money

of

banks in industry

fast

va

economy

su gy

in banks

bt ks

jo ni pa

pa

lo

ks bt

su

13. (3) su ⇒ of 14. (1) It is clear that the code for ‘essence’ is ‘lo’. 15. (4) essence ⇒ lo of ⇒ su money ⇒ gy 16. (5) economy ⇒ ‘jo’ or ‘pa’ in ⇒ ‘bt’ or ‘ks’ industry ⇒ dm 17. (1) fast ⇒ va (18-20) :

work

is

is

work

work

for

for

important today

money

tx

there

now

ne

joy

only

oj

only

joy

oj

ne

zu

ht

yd

an

yd

fq

army

worth defense

defense by army

25. (5) received highest ⇒ jo la rainfall ⇒ oa Therefore, rainfull ⇒ oa for ⇒ pb The code for agriculture may be ‘zs’. (26–30) :

jo

va

dm

dc

8

2

9

how

art

thou

9

5

8

thou

art

good

5

8

good

and

1

26. 27. 28. 29.

ka ka

mu

un

ft

ne

ft

ds

un

18. (1) money ⇒ ds 19. (3) money ⇒ ds is ⇒ ka important today ⇒ tx zu 20. (5) now there ⇒ ht mu is ⇒ ka work ⇒ ne (21–25) :

7 3

thy

thou bad

(5) (2) (3) (1)

The possible code for ‘thy’ is ‘1’, ‘3’ or ‘7’. thou ⇒ 8 how ⇒ 2 thou ⇒ 8 good ⇒ 5 The code for ‘no’ may be ‘0’.

30. (3) F

U

$

%

K

I

E

L

#

L

T

E

Therefore,

L

I

F

E

6



$

#

(31–35) :

jo

gi

la

sn

highest

civilian

award

received

tn

la

pb

jo

received

for

highest

bravery

dc

ab

sn

tn

bravery

award

an

honour

fq

mz

dc

yd

an

army

worth

defense

economy

work

related

oj

mv

bx

st

bx

oj

dy

employment

today

pk

employment

for

only

el

pk

fd

growth

related

el

dy

gm

work

21. (2) for ⇒ pb civilian ⇒ gi The code for ‘nights’ may be ‘kr’. 22. (4) Civilian ⇒ gi bravery ⇒ tn for ⇒ pb award ⇒ sn The code for ‘army cannot be definitely known.

and

and

is

growth

today

31. (4) related ⇒ mv people ⇒ xd only ⇒ fd Now, people ⇒ xd for ⇒ zn The code for ‘decision’ may be ‘kz’.

BPRE–98

zn mv

CODING / DECODING 32. (3) pk ⇒ employment 33. (1) economy ⇒ st is ⇒ gm The code for ‘boosting’ may be ‘rc’. 34. (2) and work

(45–49) :

⇒ oj bx

work today also important ⇒ qr en oj dy

economics

rt

is important today

qv

ai

nl dm

zu

rt

oy

to know is

study

study

hard

prepare

medium

ai

mj sp

sp

oy tk nl

mj

36. (4) mj ⇒ prepare 37. (3) medium ⇒ tk economics ⇒ ai 38. (1) economics

is

contacts

added

qv ai

important today

nl

dm

early

after

creation

require

to know

economics

contacts

early

35. (5) growth ⇒ el today ⇒ dy (36–40) : prepare

updated

morning

creation

require

updated

tp

rj uk

important

information now

de

45. (1) contacts ⇒ uk 46. (5) morning ⇒ cr require ⇒ de The code for ‘freshness’ may be ‘zq’. 47. (2) early ⇒ tp after ⇒ rj symptoms ⇒ by Therefore, by ⇒ symptoms ra ⇒ added vs ⇒ expertise 48. (4) information ⇒ at/gw 49. (3) updated ⇒ bt creation ⇒ mb 50. (4) C H A R I T

Thus, the code for ‘today’ is ‘qv’. study ⇒ oy 39. (5) hard ⇒ zu 40. (2)

is

distance

to

hd lh dm

fg

work

fertilizer products

organic

waste

disposal of

H

S

A

C

T

O

B

G

T

X

I

S

T

O

+1

li

gs

da

few

available

fo

pz

nb

gs

into

fertilizer

nb

cr

pt

mk

hu

mk

waste

X

H R

Y

Q

N

Therefore,

techniques

farming

Y

–1

D

T farming

R

+1

nl ba

The code for ‘know’ would be ‘rt’. The code for ‘distance’ would be ‘ba’. The code for ‘only’ may be ‘ez’ (41-44) : few organic

D

F

material ⇒ cr

sp

I

cr

Li

(51–55) :

yu

41. (4) few ⇒ gs waste ⇒ mk 42. (2) organic ⇒ cr 43. (3) waste ⇒ mk techniques ⇒ da Therefore the code for ‘managment’ is ‘ ax’ Now, farming ⇒ li fertilizer ⇒ nb management ⇒ ax 44. (1) available ⇒ fo/pz

BPRE–99

R

Y

Q

N

–1 J

I

T

mb ra

X

tp

at gw bt

–1

B

cr uk

mb vs de

expertise

+1

and more

bt

friends

make

life

easy

gk

nj

om

wp

good

to

have

friends

ew

ir

nj

uc

good

form

for

life

zy

gk

ir

th

easy

form

of

bonding

ls

th

fv

om

CODING / DECODING friends ⇒ nj make ⇒ wp life ⇒ gk easy ⇒ om good ⇒ ir to have ⇒ ew uc form ⇒ th for ⇒ zy of bonding ⇒ ls fv 51. (1) gk ⇒ life 52. (5) friends ⇒ nj of ⇒ ls/fv have

years

for

zy

hb

uc

53. (2) several

years

hb ad

Now, to ⇒ ew several ⇒ ad 54. (4) good ⇒ ir 55. (3) bonding ⇒ ls/fv form ⇒ th The code for ‘promote’ may be ‘yi’

way

for

your

bus

cp

wy

ri

md

make

way

for

bus

fu

wy

md

cp

own

food

no

way

md

ne

tu

ja

61. (3) bus ⇒ cp/wy 62. (3) own food ⇒ tu ja our ⇒ su Now, make ⇒ fu our ⇒ su way ⇒ md 63. (5) no ⇒ ne (64–68) :

payment bank got license (56–90) :

ak

sm

license

applied

to

authorities

li

tc

sm

zh

yet dj

to

receive

ge

work

result

hard

tg

za

kp

trial

win

smart

fu

lo

cx

result

out

election

mn

dw

za

bring

policy

politics

wd

pk

by

work

smart

policy

tg

lo

wd

voters

jq

mn

gt

ap

dj

payment tc

vb

list bank

receive

new

status

ge

nr

ak

fo

56. (1) payment ⇒ dj status ⇒ nr deleted ⇒ kw bank ⇒ ak account ⇒ yx Now, deleted new account ⇒ kw fo yx 57. (3) tc ⇒ to ap ⇒ got 58. (5) license ⇒ sm The code for ‘renewed’ may be ‘oq’. 59. (2) applies ⇒ zh/li 60. (4) license ⇒ sm got ⇒ ap new status ⇒ nr fo

(61–63) :

make

your

ri

fu

own food ja

tu

out

smart

election

list

jq

lo

dw

voters

smart

win

lo

fu

gt

bring democracy election 64. (5) voters ⇒ gt win ⇒ fu election ⇒ dw 65. (3) lo ⇒ smart jq ⇒ list gt ⇒ voters 66. (1) smart ⇒ lo voters ⇒ gt hard ⇒ kp democracy ⇒ ri 67. (4) work ⇒ tg 68. (2) bring ⇒ pk politics ⇒ by out ⇒ mn trial ⇒ cx

BPRE–100

pk

dw ri

CODING / DECODING C

INSURANCE EXAMS 1. (3) The common letters in VIBRANT and RENT are RNT. Therefore, RNT ⇒ w s p Now, E ⇒ m T⇒p A⇒f h and g may be codes for X and L. 2. (4)

C R A F T ⇒ q w x n g

your trip customised good

make

trip be

interesting

customised

trip always good me

happy

de la

pa

ni

ha ni

ta

de

ta

ka

ni

fa

ka ro

pa na

6. (3) The code far ‘always’ is ‘fa’. 7. (1) pa ⇒ make ni ⇒ trip de ⇒ your ha ⇒ be 8. (4) good ⇒ ka trip ⇒ ni

always ⇒ fa interesting ⇒ la 9. (2) The code for ‘make’ is ‘pa’. 10. (5) ‘ta’ represents ‘customised’.

D

M

–1

O

–1

Therefore, J

L

I

–1

K

–1

12. (4)

F

R

A

M

+1

Q

E

B

L

D

O

A

L

C

+1

K

E

–1

B

–1

M A S T E R ⇒ bn p q v x SE COND ⇒abjnow Now, MATR⇒pqvx COND⇒ajow In the code for NORTH, there would be two letters each from the codes for MATR and COND while one letter would be different. Therefore, NORTH ⇒ amvwx (6–10) : your

P

–1

SCO PE ⇒hslmp P ⇒ h or m UR ⇒ je/ja/ea E ⇒ s/l/p Therefore, IMPURE ⇒ s a j m r g 4. (4) COLUMN ⇒ a i o q r w BIRTH ⇒ g m t v x The letters L and N are common in COLUMN and LENGTH. Similarly, the letters T and H are common in BIRTH and LENGTH. Thus, in the code for LENGTH, there would be two letters each from the codes for COLUMN and BIRTH while two letters would be different. Therefore, LENGTH ⇒ kmn rvw 5. (3)

B

–1 –1

N

FA T H ER ⇒ px q w k n Therefore, C ⇒ g The code for SCRIPT ⇒ lnq g v f 3. (2) D E C O U R ⇒ j p s l e a

make

E

11. (2)

K

–1

P

J

B

Therefore, P

R

–1 Q

(13-16) :

I

D

+1 O

he

E

–1

J

needs

D

C

some

help

tu

some

le

needs medicine and some

know now

now

and where

13. (2) da ⇒ medicine tu ⇒ some ja ⇒ he 14. (4) le ⇒ needs 15. (5) he ⇒ ja know ⇒ pa where ⇒ ‘jo’ or ‘ze’

tu

si

go

jo ze

si

ki

16. (3) help ⇒ gi 17. (1) now ⇒ gi medicine ⇒ da help ⇒ gi The code of ‘could’ may be ‘pi’.

4

5

1

come to college

7

3

0

college

5

college the

was fun

Clearly, ‘7’ stands for ‘college’. 19. (3) M = 13; 13 – 4 = 9 E = 05; 05 – 4 = 1 K = 11; 11 – 4 = 7 L = 12; 12 – 4 = 8 F = 06; 06 – 4 = 2 J = 10; 10 – 4 = 6 Therefore, I = 09; 09 – 4 = 5 G = 07; 07 – 4 = 3 H = 08; 08 – 4 = 4 E = 05; 05 – 4 = 1 D = 04; 04 – 4 = 0

BPRE–101

da

ki

1

was in

le

tu pa gi

7

the

ja

help

4

18. (3) in

gi

7 9

CODING / DECODING 20. (4) P

28. (2) his ⇒ me experiences ⇒ li The code for ‘working’ may be ‘kj’. (29–32) :

A R E N T

↓ ↓ ↓ ↓ ↓ ↓ B D F G J K C H I L D R E N

↓ ↓ ↓ ↓ ↓ ↓ ↓ ↓

smart

M O X Q U F G J Therefore, R E P R I N T

coaching

summer

mo

in summer

classes

ph rd

es na

for students

jt

gk

mo

↓ ↓ ↓ ↓ ↓ ↓ ↓

F G B F X J K 21. (4) The word has been divided into two equal halves. Then, the letters of each part have been written in reverse order.

S U B S T I

T U T I O N

coaching smart

for

badminton

classes

na

students

managed well

29.

(5) na ⇒ for/students

30.

(1) summer ⇒ mo

jt

es

cz gk

bt

ph xl

(4) badminton ⇒ bt classes ⇒ gk 32. (3) well ⇒ cz/xl (33–38) :

31.

I T S B U S Similarly,

N O I T U T

D I S T R I

B U T I O N

I R T S I D

N O I T U B

22. (2) 1 2 13 12 1 3 1 21 3 ↓ ↓ ↓ ↓ ↓ ↓ ↓ ↓ ↓ A B M L A C A U C 1 5 20 ↓ ↓ ↓ A E T 12 13 2 1 ↓ ↓ ↓ ↓ L M B A 1 21 3 21 ↓ ↓ ↓ ↓ A U C U 23. (3) The letters have been written in the reverse order in the code. KINDLE ⇒ ELDNIK Therefore, EXOTIC ⇒ CITOXE 24. (3) A ⇒ 1 = 1 × 2 – 1 B⇒3=2×2–1 E⇒9=5×2–1 T ⇒ 39 = 20 × 2 – 1 B E A T ↓ ↓ ↓ ↓ 3 9 1 39 (25—28) : shared

his

Valuable

valuable

contribution of

experiences are his

shared

experiences

contribution of

teacher

teacher

success

sa

dy

li

ha

ro

li

sa

ps

night

cheek

black

two

kind

study

cap

two

lab

night

cheek

work

actor cap

stage

task study

stage

39

35

36

97

lie

92

56

25

84

cap

35

16

56

35

77

36

83

56

47

night kind

glass

33. (3) 47 ⇒ glass 34. (1) cap ⇒ 56 black ⇒ 97 two ⇒ 84 35. (2) 84 ⇒ two 25 ⇒ lie 92 ⇒ study/kind 36. (4) task, night, cheek, black, stage, two, cap, lie, lab and glass. 38. (3) M I C R O

W I D E S

me ha

kt

dy

me

ro

task

kt

cn

Therefore, S T A I N S

25. (3) dy ⇒ teacher 26. (1) contribution ⇒ ha/kt 27. (4) sa ⇒ valuable

70 83

39

84

42 70

92

37. (5) lie ⇒ 25 stage ⇒ 70 Sum = 25 + 70 = 95 two ⇒ 84 cap ⇒ 56 Sum = 84 + 56 = 140 Required ratio = =

+1 –1 +1 –1 +1

N

+1 –1 +1 –1 +1 +1 –1 +1 –1 +1 –1

X

T

95 140

19 = 19 : 28 28

H D Q P

H E D T

S

B H O R

❐❐❐

BPRE–102

CODING / DECODING

MODEL EXERCISES 1. If CIGARETTE is coded as GICERAETT, then DIRECTION will be coded as (1) IRDCTIONE (2) NOIETCRID (3) RIDTCENOI (4) NORTECDII (5) None of these 2. If DELHI can be coded as CCIDD. How would you code BOMBAY? (1) AMJXVS (2) MJXVSU (3) AJMTVT (4) WXYZAX (5) None of these 3. In a certain code, PROSE is written as PPOQE. How would LIGHT be written in that code? (1) LIGFT (2) LGGHT (3) LGGFT (4) JIEHR (5) None of these 4. If Z = 52 and ACT is equal to 48, then BAT will be equal to (1) 39 (2) 41 (3) 44 (4) 46 (5) None of these 5. BRIDGE is written as EULGJH in a certain code. How will FRUIT be written in that code? (1) IUXLW (2) IVLXW (3) IUWXL (4) IUXVT (5) None of these 6. ENGLAND is written as 1234526 and FRANCE as 785291. How will GREECE be written in this coding scheme? (1) 381191 (2) 381911 (3) 394132 (4) 562134 (5) None of these 7. If table is called chair, chair is called cot, cot is called pot and Pot is called filter. Where does a person sit? (1) Chair (2) Cot (3) Pot (4) Filter (5) None of these Directions (8-13) : Read the following and answer these questions : In a certain code language, 481 means sky is blue. 246 means sea is deep and 698 means sea looks blue. 8. What number is the code for ‘sky’? (1) 4 (2) 8 (3) 1 (4) 9 (5) None of these 9. What number is the code for ‘is’? (1) 8 (2) 1 (3) 4 (4) 2 (5) None of these

10. What number is the code for ‘looks’? (1) 4 (2) 8 (3) 6 (4) 9 (5) None of these 11. What number is the code for ‘sea’? (1) 2 (2) 8 (3) 6 (4) 4 (5) None of these 12. What number is the code for ‘blue’? (1) 8 (2) 6 (3) 1 (4) 9 (5) None of these 13. What number is the code for ‘deep’? (1) 4 (2) 2 (3) 6 (4) 1 (5) None of these 14. In a certain code, a number 13479 is written as AQFJL and 2568 is written as DMPN. How is 396824 written in that code? (1) QLPNMJ (2) QLPNMF (3) QLPMNF (4) QLPNDF (5) None of these 15. In a certain code, CALANDER is written as CLANAEDR. How is CIRCULAR written in that code? (1) ICCRLURA (2) CRIUCALR (3) CRIUCLRA (4) CARIUCLR (5) None of these 16. In a certain code, RATIONAL is written as RTANIOLA. How would TRIBAL be written in that code? (1) TRIALB (2) TIRALB (3) TIRLBA (4) TIRABL (5) None of these 17. In a certain code, CHAIR is written as EGCHT. How is AUDIT written in that code? (1) CTFHV (2) CSFHV (3) BTFHV (4) CTEHV (5) None of these 18. In a certain code, CERTAIN is coded as XVIGZRM. How is MUNDANE coded in that language? (1) NFMWZMV (2) VMZWMFN (3) NFMWZMX (4) NFMXZMV (5) None of these Directions (19-23) : In a defence message, GET AWAY, FIRE BACKWARDS, MOVE SLOW is coded as BEN CDCI, QHOE PCTL DCOXU, ZMWE VFMD.

BPRE–103

Based on this coding scheme, spot the codes for the words given in Questions. 19. OVER (1) MWOE (2) MWZO (3) MWEO (4) MWED (5) None of these 20. DEADLY (1) XECXEI (2) XENXFI (3) XECXEI (4) XECXFI (5) None of these 21. REWARD (1) OEDNXE (2) OTDCOX (3) OEDCOX (4) OEDCOU (5) None of these 22. GREAT (1) BOENC (2) BOEQN (3) BOECN (4) BOEHC (5) None of these 23. If SMOOTH is coded as 135579, ROUGH as 975531 and HARD as 9498, then SOFT will be coded as (1) 1527 (2) 1347 (3) 4998 (4) 8949 (5) None of these Directions (24-25) : In a secret way of writing, INDIA HAS WON FIRST GOLD MEDAL is written as JEZJU OUV SEF PJDVB REQZ ITZUQ. Bearing the method in mind, pick up the code from the answer choices for each given word. 24. FOOT (1) PEEB (2) SEEZ (3) BEEB (4) CEEA (5) None of these 25. DIESEL (1) ZJTVTQ (2) ZJEVTQ (3) BZTETV (4) ZJTQVT (5) None of these 26. If the code of ABCDEF is ZYXWVU, then what is the code for PASS? (1) KZHH (2) KHZZ (3) KMHH (4) WZHH (5) None of these 27. In a certain code, CAT is written as SATC and DEAR is written as SEARD. How would SING be written in that code? (1) GNISS (2) SINGS (3) SGNIS (4) BGINS (5) None of these

CODING / DECODING 28. In a certain code language, 134 means Good and Tasty, 478 means see good pictures and 729 means pictures are faint. Which of the following numerical symbols stands for see? (1) 1 (2) 2 (3) 7 (4) 8 (5) None of these 29. If BRIDGE is written as EULGJH in a certain code, how will FRUIT be written in that code? (1) IUXLW (2) IVLXW (3) IUWXL (4) IUXVT (5) None of these 30. In a certain code, EASE is written as GUCG. How is CUT written in that code? (1) UWE (2) VWE (3) EWU (4) CWF (5) None of these 31. In the following sequence or instructions, 1 stands for Run, 2 stands for Stop, 3 stands for Go, 4 stands for Sit and 5 stands for Wait. If the sequence is continued, which instruction will come next? 44545345314531245 453453 (1) Wait (2) Sit (3) Stop (4) Run (5) None of these 32. In a certain code language, ROAD is written as URDG. How is SWAN written in that code? (1) UXDQ (2) VZCQ (3) VZCP (4) VZDQ (5) None of these 33. PROMISE is coded as 1234567. What should be the code for MISER? (1) 45672 (2) 43672 (3) 76543 (4) 14572 (5) None of these Directions (34-36) : If EFGHIJK is the coded letter representing VUTSRQP, then choose the right code for the words given in the capital letters from the answer choices (1), (2), (3) and (4). 34. HIGH (1) STRS (2) SRTS (3) RSTR (4) RJHR (5) None of these 35. ZERO (1) BUHN (2) AVIM (3) AVIL (4) AVTI (5) None of these

36. GROUP (1) TILFK (3) TILEL (2) TILGH (4) TILHG (5) None of these 37. If NOR is coded as 236, then how should REST be coded in the same code language? (1) 61967 (2) 51958 (3) 61978 (4) 61858 (5) None of these 38. If ‘light’ is called ‘morning’, ‘morning’ is called ‘dark’ ‘dark’ is called ‘night’, ‘night is called ‘sunshine’ and ‘sunshine’ is called ‘dusk’, when do we sleep? (1) Dusk (2) Dark (3) Night (4) Sunshine (5) None of these 39. If A + B = C, D – C = A and E – B – C, then what does D + F stands for? Provide your answer in letter terms as well as in number terms. (1) J & 10 (2) A & 1 (3) C & 3 (4) Q & 17 (5) None of these 40. In a certain code language, ‘3a, 2b, 7c’ means ‘Truth is Eternal’, ‘7c, 9a, 8b, 3a’ means ‘Enmity is not Eternal’ and ‘9a, 4d, 2b, 8b’ means Truth does not perish’. Which of the following means ‘Enmity’ in that language? (1) 3a (2) 7c (3) 8b (4) 9a (5) None of these 41. In a certain code language, ‘po ki top ma’ means ‘Usha is playing cards’, ‘kop ja ki ma’ means Asha is playing tennis’, ‘ki top sop ho’ means ‘they are playing football’ and ‘po sur kop’ means ‘cards and tennis’. Which word in that language means Asha’? (1) ja (2) ma (3) kop (4) top (5) None of these 42. If VICTORY is coded as YLFWRUB, then how can SUCCESS be coded? (1) VXEEIVV (2) VXFFHVV (3) VYEEHVV (4) None of these (5) None of these 43. If in a certain language, TRIANGLE is coded SQHZMFKD, then which word would be coded as DWZLOKD?

BPRE–104

(1) EXAMPLE (2) DISMISS (3) FIGMENT (4) DISJOIN (5) None of these 44. In a certain code language, ‘nee muk pic’ means ‘grave and concern’, ‘ill dic so’ means ‘every body else’ and ‘tur muk so’ means ‘body and soul’. Which of the following would mean ‘every concern’? (1) dic pic (2) pic nee (3) ill nee (4) Cannot be determined (5) None of these 45. If ACELDNRA stands for CALENDAR, what does LEGIBIEL stands for? (1) LIEGIBLE (2) ELIGIBLE (3) BIGEELIC (4) BCIGEELI (5) None of these 46. In a certain code ROAD is written as URDG. How is SWAN written in that code? (1) UXDQ (2) VZDQ (3) VXDQ (4) VZCQ (5) None of these 47. If table is called chair, chair is called cot, cot is called pot and pot is called filter, where does a person sit? (1) Pot (2) Cot (3) Chair (4) Filter (5) None of these 48. In a certain code, ‘OVER’ is written as ‘$#%* and VIST is written as ‘#+×–. How is SORE written in that code? (1) ×$*% (2) %×$* (3) ×*$% (4) *$*× % (5) None of these 49. In a certain code language, ‘ne ri so’ means ‘good rainy day’, ‘si ne po’ means ‘day is wonderful’ and ‘ri jo’ means ‘good boy’. Which of the following means ‘rainy’ in the code? (1) ne (2) si (3) ri (4) so (5) None of these 50. In a certain code, ‘PEAK’ is written as ‘3512’ and DINE is written as ‘6895’. How is KIND written in that code? (1) 2396 (2) 2896 (3) 2986 (4) 2596 (5) None of these 51. In a certain code, ‘TERMINAL’ is written as SDQLJOBM, How is CREDIBLE written in that code?

CODING / DECODING (1) BQDCJCMF (2) BQDCHAKD (3) DSFEJCMF (4) DSFEHAKD (5) None of these 52. In a certain code, TEACHER’ is written as VGCEJGT. How is CHILDREN written in that code? (1) EJKNEGTP (2) EGKNFITP (3) EJKNFGTO (4) EJKNFTGP (5) None of these 53. In a certain code, ‘BASIC’ is written as ‘DDULE’. How is ‘LEADER’ written in that code? (1) NHCGGU (2) OHDGHU (3) NGCFGT (4) OGDFHT (5) None of these 54. As ‘COUNSEL is to ‘BITIRAK’ so also ‘GUIDANCE’ is to (1) FOHYZJBB (2) HOHYBJBA (3) FPHZZKAB (4) FORHYZJBB (5) None of these 55. If in a certain language, NEOMAN is coded as OGRQFT, which word will be coded as ZKCLUP ? (1) YIZHPJ (2) XIAJSN (3) YJBKTO (4) YIAQKJ (5) None of these 56. The letters L, M, N, O,P Q, R, S and T in their order are substituted by nine integers 1 to 9 in that order. 4 is assigned to P. The difference between P and T is 5. The difference between N and T is 3. What is the integer assigned to N ? (1) 6 (2) 5 (3) 4 (4) 7 (5) None of these 57. In a coded writing, ‘come at once father very ill’ is written as ‘XLNV ZG LMXY UYGSVI EVIB ROO’. What is the corresponding code for ‘mother’? (1) NOGSVI (2) NOTSVI (3) NLGSVI (4) NLGSTI (5) None of these 58. If ‘CONSTABLE’ is coded as 91, what will be the code number for ‘STABLE’? (1) 97 (2) 59 (3) 79 (4) 75 (5) None of these 59. If 3, 1, 11, 5 represents CAKE; 6, 1, 9 12 is FAIL, what represents FIRST ? (1) 9, 18, 6, 19, 20 (2) 6, 18, 9, 19, 20 (3) 6, 9, 18, 19, 20

(4) 18, 19, 20, 9, 6 (5) None of these 60. If 7, 15, 15, 4 is GOOD and 2, 5, 20, 20, 5, 18 is BETTER, what numbers represent BEST? (1) 2, 5, 19, 20 (2) 2, 19, 4, 20 (3) 2, 4, 19, 20 (4) 2, 5, 20, 19 (5) None of these Directions (61–65) : Study the following information carefully and answer the questions given below : In a certain code language, ‘more money in market’ is written as ‘zo li aa to’ ‘share in market profit’ is written as ‘vo to je li’ ‘making more profit now’ is written as ‘su je zo ka’ ‘now the market gains’ is written as ‘do li yo su’ 61. What is the code for ‘the share gains money’ in the given code language ? (1) to zo yo do (2) to li su do (3) aa do vo yo (4) yo do ka aa (5) vo je li aa 62. What is the code for ‘making’ in the given code language ? (1) ka (2) su (3) li (4) Either ‘je’ or ‘zo’ (5) Either ‘su’ or ‘ka’ 63. Which of the following may represent ‘loss in share market’ in the given code language ? (1) li je vo to

(2) su li ka vo

(3) vo li zo aa (4) am li vo to (5) do li vo to 64. What does the code ‘yo’ stand for in the given code language ? (1) now (2) the (3) gains

(4) market

(5) Either ‘the’ or ‘gains’ 65. What is the code for ‘now more profit’ in the given code language ? (1) je ka li (2) je su zo (3) to je su (4) zo aa do (5) yo su zo

BPRE–105

Directions (66–70) : Study the following information carefully and answer the questions given below : In a certain code language, ‘economy and wealth balance’ is written as ‘gh mk ru st’ ‘wealth of nations depleting’ is written as ‘tl zm ak gh’ ‘taxes balance nations better’ is written as ‘dj ru zm pn’ ‘better to revive economy’ is written as ‘br ht dj st’ (All codes are two letter codes only) 66. What does the code ‘tl’ stand for in the given code language? (1) and (2) Either ‘of’ or ‘depleting’ (3) Either ‘nations’ or ‘taxes’ (4) to (5) taxes 67. Which of the following may possibly represent ‘revive now’ in the given code language? (1) zm ht (2) bt zm (3) ht co (4) br ht (5) br dj 68. What is the code for ‘economy’ in the given code language? (1) mk (2) Either ‘mk’ or ‘ru’ (3) gh (4) dj (5) st 69. What is the code for ‘balance’ in the given code language? (1) st (2) gh (3) ak (4) ru (5) zm 70. Which of the following may represent ‘better generate wealth’ in the given code language? (1) fs gh dj (2) dj mk gh (3) gh pn st (4) gh fs mk (5) xs dj ak 71. In a certain code language, ‘PLOTS’ is written as ‘NJSRQ’, ‘SNACK’ is coded as ‘QLEAI’, how is FUDGE written in that code? (1) DSHEC

(2) DWHKJ

(3) DWHEG

(4) DWHEC

(5) DWHKG Directions (72–76) : Study the following information carefully and answer the questions given below : In a certain code language, ‘robots can become lawyers’ is written is ‘ju ac th eg’.

CODING / DECODING ‘all doctors can check’ is written as ‘bd np ju mo’ ‘many doctors and lawyers’ is written as ‘np rt qs ac’ ‘machine check of robots’ is written as ‘eg ik ux bd’ (Note : All codes are two letter codes only) 72. What does the code ‘ac’ stand for in the given code language? (1) robots (2) many (3) lawyers (4) become (5) of 73. If ‘how and many’ is written as ‘qs ws rt’ then what will be the code for ‘how can’? (1) np qs (2) rt ju (3) qs rt (4) ws ju (5) bd ws 74. What is the code for ‘machine’ in the given code language? (1) rt (2) Either ‘eg’ or ‘rt’ (3) bd (4) eg (5) Either ‘ik’ or ‘ux’ 75. What is the code for ‘robots’ in the given code language? (1) ux (2) eg (3) ju (4) Either ‘eg’ or ‘ju’ (5) Either ‘ju’ or ‘ac’ 76. What may be the possible code for ‘will become doctor’ in the given code language? (1) th zi np (2) zi ac bd (3) np th ux (4) zi np bd (5) bd ju ux 77. In a certain code language, “oka peru”, means “fine cloth”, “meta lisa” means “clear water” and “dona lisa peru” means “fine clear weather”. What is the code for the word “weather” in that language? (1) meta (2) oka (3) dona (4) lisa (5) peru Directions (78–79): Study the following information carefully and answer the questions given below: In a certain code language “Live Free Die Well” is written as “bg ph su md”

“Right To Free Life” is written as “va su oh ke” “To Live Make Home” is written as “tl ke ph rn” “Make Your Life Well” is written as “bg ri oh tl” (All the codes are two lettered code only) 78. What is the code for “free life”? (1) bg tl (2) su oh (3) ph ri (4) va ke (5) rn md 79. What does the code “rn ph” stands for ? (1) Home live (2) Well make (3) Your home (4) Life right (5) Make free Directions (80–84) : Study the following information carefully and answer the questions given below : In a certain code language, “stay close to moral values” is written as “cap tap map rap lap” “the way to go college” is written as “xip rap lol tip dap” “college gate stay close” is written as “tap nap xip map” “go the moral college” is written as “dap lap xip lol” “stay close go away” is written as “ tap dap zip map” 80. What is code for “the moral college”? (1) lap zip rap (2) zip tip cap (3) zip lol lap (4) tap rap map (5) lap xip lol 81. What does the code “lap zip rap” stand for? (1) moral college stay (2) gate away to (3) college moral value (4) to away moral (5) close the gate 82. What is the code for “stay moral” if the code for “close” is “tap”? (1) map lap (2) tap map (3) lap rap (4) rao lap (5) tap xip 83. What are the code and word for “tip to lap college”? (1) moral rap close zip (2) stay go away the

BPRE–106

(3) way rap moral xip (4) close cap college xip (5) None of these 84. Which of the following may represent “you go to college”? (1) map dap xip rip (2) jap xip dap rap (3) dap xip tap rap (4) rap xip tap map (5) rap xip dap lol Directions (85–86) : Study the following information carefully and answer the questions given below : Seven alphabet are arranged from bottom to top in eight boxes numbered 1 to 8. The lowermost box is numbered 1, the box just above it is numbered 2 and so on till the topmost box is numbered 8. T is in an odd numbered box and B is in an even numbered box. The word NET is arranged from bottom to top but not serially. The word LATE is arranged from top to bottom but not serially. The word NANO is arranged from top to bottom but not serially. The word NO is arranged serially from top to bottom. One N is exactly between E and O. There is only one box above the alphabet L. B is exactly between alphabet T and E. 85. What is code for BEAT ? (1) 5341 (2) 4365 (3) 3546 (4) 4563 (5) 4152 86. What does the number group “235764” denote ? (1) BALENO (2) BEATEN (3) NETBAN (4) NETLAB (5) LENTAB Directions (87 - 91): Study the following information carefully and answer the questions given below : In a certain code language, “army protects country” is written as “12 15 85” “politicians frame policy” is written as “50 1 18” “officers execute function” is written as “13 33 23” “police protect public” is written as “ 85 8 20” “public pay tax” is written as “7 8 60” “tax protects policy” sis written as “85 60 1” “frame public army” is written as “50 15 8”

CODING / DECODING “fovernment function tax” is written as “13 60 11” “officers charge pay” is written as “38 7 33” 87. What can be the code for “government officers pay”? (1) 18 – 12 – 33 (2) 11 – 85 – 60 (3) 23 – 13 – 60 (4) 11 – 33 – 7 (5) 18 – 20 – 1 88. What can be the code for “army protects police”? (1) 7 – 38 – 11 (2) 50 – 8 – 12 (3) 15 – 85 – 20 (4) 33 – 13 – 18 (5) 11 – 85 – 12 89. What can be the code for “politicians pay officers”? (1) 38 – 12 – 8 (2) 1 – 33 – 18 (3) 11 – 60 – 13 (4) 15 – 20 – 1 (5) 18 – 7 – 33 90. What can be the words for “11 38 8”? (1) Police frame tax (2) Government charge public (3) Politicians protect tax (4) Officers pay public (5) Army frame charge 91. What can be the words for “8 50 12”? (1) Policy charge police (2) Army execute tax (3) Public frame country (4) Government protect public (5) Police pay Government Directions (92–96) : Study the following information carefully and answer the questions given below : The following table represents some words and their codes. The codes are assigned on the basis of some pattern based on addition and subtraction.

Word OFFENCE DEFENCE Code 40 16 Columns P, Q, A, X and Y show some numerical values which form the codes for the words asked. You have to mark the appropriate column as your answer.

P Q A X Y 52 44 29 61 26 45 20 22 9 50 89 28 25 66 44 76 87 8 35 40 6 57 84 31 37 92. What is the code for “VOWEL”? (1) P (2) Q (3) A (4) X (5) Y 93. What is the code for “OXYGEN”? (1) P (2) Q (3) A (4) X (5) Y 94. What is the code for “BIOLOGY”? (1) P (2) Q (3) A (4) X (5) Y 95. What is the code for “AWARDS”? (1) X (2) Q (3) A (4) P (5) Y 96. What is the code for “CHARITY”? (1) P (2) Q (3) A (4) X (5) Y Directions (97 – 101) : Study the following information carefully and answer the questions given below : In a certain code language “wealth support yoga” is written as “50 12 9” “health is wealth” is written as “10 7 12” “yoga helps improvement” is written as “15 23 9” “teacher helps wealth” is written as “15 16 12” “teachers are guides” is written as “20 16 11” “wealth is support” is written as “50 12 10” “yoga guides balance” is written as “20 1 9” “happy is health” is written as “7 100 10” “bliss brings happy” is written as “100 25 5” “teachers are bliss” is written as “11 16 25” 97. What is the code for “Yoga is wealth”? (1) 15 50 1 (2) 9 10 12

BPRE–107

(3) 5 25 100 (4) 7 25 100 (5) 9 50 15 98. What is the code for “Happy health is bliss”? (1) 50 9 12 7 (2) 5 7 25 20 (3) 100 23 11 16 (4) 100 7 10 25 (5) 9 15 50 100 99. The code “50 5 1” stands for : (1) Improvement are bliss (2) Teacher is help (3) Support bring balance (4) Happy helps yoga (5) Guide is happy 100. The code “9 1 7” stands for (1) Guide support wealth (2) Wealth helps health (3) Bliss is teacher (4) Guide are happy (5) Yoga balance health 101. What is the resultant of “bliss + support + help + is + happy”? (1) 110 (2) 150 (3) 143 (4) 200 (5) 86

SHORT ANSWERS 1.(3) 5.(1) 9.(3) 13.(2) 17.(1) 21.(3) 25.(1) 29.(1) 33.(1) 37.(3) 41.(1) 45.(2) 49.(4) 53.(1) 57.(3) 61.(3) 65.(2) 69.(4) 73.(4) 77.(3) 81.(4) 85.(2) 89.(5) 93.(1) 97.(2) 101.(4)

2.(1) 6.(1) 10.(4) 14.(4) 18.(1) 22.(3) 26.(1) 30.(2) 34.(2) 38.(4) 42.(2) 46.(2) 50.(2) 54.(1) 58.(2) 62.(1) 66.(2) 70.(1) 74.(5) 78.(2) 82.(1) 86.(4) 90.(2) 94.(3) 98.(4)

3.(3) 7.(2) 11.(3) 15.(2) 19.(3) 23.(1) 27.(2) 31.(4) 35.(3) 39.(1) 43.(1) 47.(2) 51.(1) 55.(1) 59.(3) 63.(4) 67.(3) 71.(1) 75.(2) 79.(1) 83.(3) 87.(4) 91.(3) 95.(4) 99.(3)

4.(4) 8.(3) 12.(1) 16.(3) 20.(4) 24.(1) 28.(4) 32.(4) 36.(1) 40.(3) 44.(4) 48.(1) 52.(4) 56.(1) 60.(1) 64.(5) 68.(5) 72.(3) 76.(1) 80.(5) 84.(2) 88. (3) 92.(5) 96.(3) 100.(5)

CODING / DECODING

EXPLANATIONS 1. (3) CIG is coded as GIC, ARE is coded as ERA and TTE is coded as ETT. Therefore, DIRECTION is coded as RIDTCENOI. 2. (1) Each successive letter of the word DELHI is moved one, two, three, four, five steps backward in alphabetical or der. Hence, BOMBAY will be coded as AMJXVS. 3. (3) Second and fourth letters of the word PROSE are moved two steps in backward direction in alphabetical order. Hence, LIGHT can be coded as LGGFT.

P ¯ P

R

O ¯ O

–2 ¯

P

S

E ¯ Q E

–2 ¯

Then

L ¯ L

I

G H T ¯ –2¯ ¯ G F T

–2 ¯

G

4. (4) The alphabetical position of each letter is doubled. ACT =1 × 2 + 3 × 2 + 20 × 2 = 48 BAT = 2×2+ 1 × 2+ 20 × 2 = 46 5. (1) Each letter of the word is moved three steps in forward direction in alphabetical order. Therefore, FRUIT will be coded as IUXLW. R → 8, E → 1, 6. (1) Codes for G → 3, C → 9, E → 1. Hence, GREECE will be coded as 381191. 7. (2) A person sits on chair, here, chair is cot. So, the person sits on cot.

(8–13) :

4

8

1 ¾®

sky is blue

2

4

6 ¾®

sea is deep

6

9

8 ¾®

sea looks blue

8. (3) Code for Sky = 1 9. (3) Code for Is = 4 10. (4) Code for Looks = 9 11. (3) Code for Sea = 6 12. (1) Code for Blue = 8 13. (2) Code for Deep = 2 14. (4) 396824 will be coded as QLPNDF. 15. (2) Keeping first and last letter as fixed, rest two adjacent letters are reversed. Hence, CIRCULAR will be coded as CRIUCALR. 16. (3) R A T I O N A L — R T A N I O L A 12345678— 132 64587 Hence, TRIBAL will be coded as TIRLBA. 17. (1) First, third and fifth letters move two steps in forward direction, whereas second and fourth letters move one step in backward direction in alphabetical order.

18. (1) C is third from beginning, X is third from last, E is fifth from beginning & V is fifth from the last and so on. Hence, MUNDANE will be coded as NFMWZMV. 19. (3) Code for O → M, V→W, E→ E, R → O. Hence, the code of OVER is MWEO. 20. (4) Codes for D → X, E → E, A → C, D → X,L → F, Y → I. Hence, the code for DEADLY will be XECXFI. 21. (3) REWARD will be coded as OEDCOX. 22. (3) Codes for G→ B, R→ O, E→ E, A → C and T → N. Hence, GREAT will be coded as BOECN. 23. (1) According to question, code for SOFT will be 1527. 24. (1) Codes for F→ P, O → E, T→ B. Hence, code for FOOT will be PEEB. 25. (1) Code for DIESEL is ZJTVTQ. 26. (1) A is first alphabet and Z is last alphabet, B is second from start and Y is second last and so on. Hence, word PASS will be coded as KZHH, 27. (2) Letter S is placed in the beginning for each coding, then first letter is placed at last place, keeping all other letters as it. In the same way, SING will be coded as SINGS. 28. (4) On comparing all the three sentences, we get good = 4, pictures = 7 and see = 8. Hence, see stands for 8. 29. (1) Each letter of the word BRIDGE is moved three steps in forward direction in the alphabetical order. Hence, FRUIT will be coded as IUXLW. 30. (2) The 1st and 4th letters are interchanged. Then, second and third letters are interchanged. After this, each letter is moved two steps in forward direction. Hence, CUT will be coded as VWE. 31. (4) The sequence proceeds as below 4, 45, 453, 4531, 45312, 45, 453, 4531 Hence, the next instruction will be run. 32. (4) Each letter of the word ROAD moves three steps in forward direction in alphabetical order. 33. (1) Codes for P → 1, R→ 2, O → 3, M → 4, I → 5, S →6 and E→ 7. Hence, code for MISER will be 45672. 34. (2) HIGH will be coded as SRTS. 35. (3) ZERO will be coded as AVIL. 36. (1) GROUP will be coded as TILFK. 37. (3) Difference between alphabetical position of N and O is 1 as it is between 2 and 3 and that of 0 & R is 3 as it is between 3 and 6. Hence, REST will be coded as 61978. 38. (4) We sleep in the night. Night is called Sunshine. Hence, we sleep in Sunshine. 39. (1) D’s position in alphabetical order = 4, F’s position = 6. Hence, D + F = 10 which is the position of J. 40. (3) Comparing all the statements “Enmity” will be coded as 8b. 41. (1) ‘Asha’ is coded as ‘ja’, 42. (2) Each letter is coded three steps forward in alphabetical order. Hence, SUCCESS will be coded as VXFFHVV. 43. (1) Each letter of the alphabet moves one step backward inthe alphabetical order.

BPRE–108

CODING / DECODING B

44. (4) nee muk pic ® grave and concern ill

dic

so ® every body else

tur muk so ® body concern ® nee/pic every ® ill/dic

and soul

L

46. (2) R O A D +3 +3 +3 +3 U R D G Similarly,

G

M

I

N

A

L

B B B B B B B B

51. (1) −1 −1 −1 −1 +1 +1 +1 +1 S D Q L J O B M D

I

B

L

E

B B B B B B B B

−1 −1 −1 −1 +1 +1 +1 +1 D

T

E

A

C C

J H

C E

M

F

R

B B B B B B B

52. (4) +2 +2 +2 +2 +2 +2 +2 V G C E J G T C

H

I

L

D

R

E

N

B B B B B B B B

+2 +2 +2 +2 +2 +2 +2 +2 E

J

K

E

R

C

G

G

U

I

A

N

C

D

E

O

H

Y

Z

J

B

B

55. (1) The pattern is

47. (2) A person sits on a chair. Since, ‘chair’ is called ‘cot’. Our answer is ‘cot’. 48. (1) O V E R → $ # % * V I S T→# + × – ∴S O R E→× $ * % 49. (4) ne ri so → good rainy day ..(i) si ne po → day is wonderful (ii) ri jo → good boy ...............(iii) From (i) and (ii), day → ne From (i) and (iii), good → ri ∴ rainy → so 50. (2) P E A K → 3 5 1 2 D I N E →68 9 5 ∴K→2 I →8 N→9 D→6

Q

H

U

F

V Z D Q

B

D

B B B B B B B B

S W A N

E

A

–1 −6 −1 −5 −1 −4 −1 −3

+3 +3 +3 +3

R

C

54. (1) The pattern is C–1=B O–6=I U–1=T N–5=I S–1=R E–4=A L–1=K

E L I G I B L E

C

E

N

L E G I B I E L

R

I

B B B B B B

C A L EN D A R

E

S

+2 +3 +2 +3 +2 +3

45. (2) A C E L D N R A

T

A

B B B B B

53. (1) +2 +3 +2 +3 +2 D D U L E

N

F

T

G

P

N +1

E

O

M

A

N

B +2 B +3 B +4 B +5 B +6 B O

G

Z –1

K –2

Y

R

C –3

I

Q

Z

L –4

F

U –5

H

T

P –6

P

J

either

Y –1

Z

I –2

K

Z –3

H –4

C

P –5

L

J –6

U

P

56. (1) P is 4 T – P = 5 = 9 – 4 = 5, so T = 9 T – N = 3 = 9 – 6 =3, so N = 6 → XLNV 57. (3) come → ZG at → LMXY once → UYGSVI father → EVIB very → ROO ill → NLGSVI mother (in Reverse order) 58. (2) The alphabetical position of letters is as follows C=3 S = 19 O = 15 T = 20 N = 14 A=1 S = 19 B=2 T = 20 L = 12

BPRE–109

CODING / DECODING 68. (5) economy ⇒ st

E =5 59

A=1

69. (4) balance ⇒ ru 70. (1) better ⇒ dj

B=2 L = 12

wealth ⇒ gh The code for ‘generate’ may be ‘fs’.

E=5 91

71. (1) P L O T S

N J S R Q

59. (3) Letters are coded according to their place in English alphabet. So, first → 6, 9, 18, 19, 20 60. (1) Letters are coded according to their place in English alphabet. So, BEST → 2, 5, 19, 20 (61–65) :

–2 –2 +4 –2 –2

S N A C K more

money in market

zo li

aa

to

Q L E A I –2 –2

share in market profit

vo to je

li

+4 –2

making

more profit

now

su je

–2

zo ka Similarly,

now the

market gains

do li yo su

F U D G E

D S H E C –2

61. (3) the gains ⇒ do yo share ⇒ vo money ⇒ aa 62. (1) making ⇒ ka 63. (4) in ⇒ to share ⇒ vo

–2 +4 –2 –2 (72–76) :

market ⇒ li The code for ‘loss’ may be ‘am’. 64. (5) yo ⇒ ‘the’ or ‘gains’ 65. (2) now ⇒ su more ⇒ zo profit ⇒ je (66–70) : economy

and wealth

wealth

of

taxes better

nations

balance nations to

revive

robots

all doctors

balance

gh

mk

ru

st

depleting

tl

zm

ak

gh

better

dj

ru

zm

pn

economy

br

ht

dj

st

66. (2) tl ⇒ of/depleting 67. (3) The code for ‘revive’ is ‘br’ or ‘ht’. The code for ‘now’ may be ‘co’

can

become can

lawyers

check

ju ac th eg bd

np

ju mo

many doctors and lawyers

np rt qs ac

machine

eg

check of

robots

72. (3) ac ⇒ lawyers 73. (4) and many ⇒ rt qs how ⇒ ws how can ⇒ ws ju 74. (5) machine ⇒ ik/ux 75. (2) robots ⇒ eg 76. (1) become ⇒ th doctors ⇒ np The code for ‘will’ may be ‘zi’.

BPRE–110

ik ux bd

CODING / DECODING 77. (3) oka

80. (5) the ⇒ lol moral ⇒ lap college ⇒ xip 81. (4) lap ⇒ moral zip ⇒ away rap ⇒ to 82. (1) stay ⇒ map moral ⇒ lap 83. (3) tip ⇒ way to ⇒ rap lap ⇒ moral college ⇒ xip 84. (2) go ⇒ dap to ⇒ rap college ⇒ xip

peru

fine

cloth

meta

lisa

clear

water

dona

lisa

peru

fine

clear

weather

The code for “you” may be “jap”. (85–86) :

(78–79) : Live

Free

Die

Well

bg

ph

su

md

Right

To

Free

Life

va

su

oh

ke

To

Live

Make

Home

tl

ke

ph

rn

Make

Your

Life

Well

bg

ri

oh

tl

78. (2) Free ⇒ su Life ⇒ oh 79. (1) rn ⇒ Home ph ⇒ Live (80–84) :

stay

close

cap

tap

the

way

xip

college tap go dap

to

moral

map

rap

to

rap

go

gate nap the lap

stay xip moral xip

N L A T B E N 0

85. (2) B

E

A

T

4 86. (4) 2

3 3

6 5

5 7 6

4

N

E

T

L

B

values lap

college tip

lol

8 7 6 5 4 3 2 1

dap

close map college lol

stay

close

go

away

tap

dap

zip

map

A

(87-91) :

Army

Protects

Country

12

15

85

Politicians

Frame

Policy

50

1

18

Officers

Execute

Function

13

33

23

Police

Protects

Public

85

8

20

Public

Pay

Tax

7

8

60

Tax

Protects

Policy

85

60

1

Frame

Public

Army

50

15

8

Tax

13

60

11

Pay

38

7

33

Government Function Officers

BPRE–111

Charge

CODING / DECODING 87. (4) Government ⇒ 11 Officers ⇒ 33 ⇒7 Pay 88. (3) Army ⇒ 15 Protects ⇒ 85 Police ⇒ 20 89. (5) Politicians ⇒ 18 pay ⇒7 Officers ⇒ 33 90. (2) 11 ⇒ Government 38 ⇒ Charge 8 ⇒ Public 91. (3) 8 ⇒ Public 50 ⇒ Frame 12 ⇒ Country (92–96) : Rule–1 : When a word begins with a Vowel, the code can be generated by taking the sum of place values of the letters at the odd numbered positions in the word. 1 2 3 4 5 6 7 O F F E N C E Required sum = O + F + N + E = 15 + 6 + 14 + 5 = 40 Rule–2 : When a word begins with a consonant, the code can be generated by taking the difference of the resultants obtained in the following manner : 1 2 3 4 5 6 7 D E F E N C E Sum of place values of odd numbered letters =D+F+N+E = 4 + 6 + 14 + 5 = 29 Sum of place values of even numbered letters =E+E+C = 5 + 5 + 3 = 13 Required difference = 29 – 13 = 16 92. (5) 1 2 3 4 5 V O W E L (V + W + L) – (O + E) ⇒ (22 + 23 + 12) – (15 + 5) ⇒ 57 – 20 = 37 (Y) 93. (1) 1 2 3 4 5 6 O X Y G E N Required sum = O + Y + E = 15 + 25 + 5 = 45 (P) 94. (3) 1 2 3 4 5 6 7 B I O L O G Y (B + O + O + Y) – (I + L + G) ⇒ (2 + 15 + 15 + 25) – (9 + 12 + 7) ⇒ 57 – 28 = 29 (A) 95. (4) 1 2 3 4 5 6 A W A R D S Required sum = A + A + D = 1 + 1 + 4 = 6 (P)

96. (3) 1 2 3 4 5 6 7 C H A R I T Y (C + A + I + Y) – (H + R + T)

⇒ (3 + 1 + 9 + 25) – (8 + 18 + 20) ⇒ 38 – 46 = – 8 (A) (Ignore the sign) (97 – 101) :

yoga

support

wealth health

is

wealth

yoga

helps

improvement

15

9

10

7

12

15

23

9

15

16

12

16

teacher

helps

teacher

are

guides

20

is

support

50

wealth

12

11 10

12

wealth

yoga

guides balance

happy bliss

is

health

brings happy

teachers are

bliss

1

20 7

9

100 10

100 11 16

25 5 25

97. (2) yoga ⇒ 9 is ⇒ 10 wealth ⇒ 12 98. (4) happy ⇒ 100 health ⇒ 7 is ⇒ 10 bliss ⇒ 25 99. (3) 50 ⇒ support 5 ⇒ brings 1 ⇒ balance 100. (5) 9 ⇒ yoga 1 ⇒ balance 7 ⇒ health 101. (4) bliss + support + help + is happy

⇒ 25 + 50 + 15 + 10 + 100 ⇒ 200

BPRE–112

ppp

BLOOD RELATIONSHIP

6

BLOOD RELATIONSHIP

QUESTIONS FROM 1999 TO 2010 ARE AVAILABLE ONLINE NATIONALISED BANKS & IBPS PO/MT/SO EXAMS Directions (50–51) : Read the following information carefully and answer the questions which follow : (Corporation Bank PO Exam. 16.01.2011)

If ‘A × B’ means ‘A is son of B’. If ‘A + B’ means ‘A is daughter of B’. If ‘A ÷ B’ means ‘A is wife of B’. If ‘A – B’ means ‘A is father of B’. 1. What will come in the place of the question mark, to establish that Q is mother of N in the expression : ‘ N + O – P ? Q’ (1) + (2) × (3) – (4) ÷ (5) Either (1) or (2) 2. Which of the following relations are true based upon the relations given in the equation : ‘S ÷ T × V – W ÷ Y’ ? (1) T is brother of Y (2) S is daughter-in-law of W (3) S is daughter-in-law of Y (4) Y is daughter of V (5) None is true 3. If ‘B × C’ means ‘B is the daughter of C’, ‘B + C’ means ‘B is the husband of C’ and ‘B – C’ means ‘B is the sister of C’, then what does ‘M + N – P × Q’ mean ? (1) M is the brother-in-law of Q (2) M is the uncle of Q (3) M is the son-in-law of Q (4) Q is the mother-in-law of M (5) None of these (Punjab & Sind Bank PO Exam. 23.01.2011)

Directions (4–6) : Read the following information carefully and answer the questions which follow : (Bank Of Baroda PO Exam. 13.03.2011)

If If If If

‘A × B’ means ‘A is father of B’. ‘A + B’ means ‘A is wife of B’. ‘A ÷ B’ means ‘A is daughter ‘of B’. ‘A – B’ means ‘A is son of B’. 4. How is L related to Q in the expression ‘L ÷ M × Q – P ÷ Q’ ? (1) Granddaughter (2) Niece

(3) Daughter-in-law (4) Daughter (5) Cannot be determined 5. What will come in the place of the question mark, to establish that Q is the nephew of T in the expression : ‘Q ? R ÷ S × T ? (1) + (2) × (3) – (4) ÷ (5) Either – or ÷ 6. Which of the following relations are true based upon the relations given in the equation : ‘A – B × C + D – E’? (1) C is mother of A (2) E is wife of B (3) D is brother of A (4) E is mother-in-law of C (5) None is true 7. D said, “A’s father is the only brother of my sister’s son.” How is A’s father related to D ? (1) Cousin (2) Nephew (3) Aunt (4) Data Inadequate (5) None of these (Punjab & Sind Bank PO Exam. 23.01.2011)

8. B is the father of Q. B has only two children. Q is the brother of R. R is daughter of P. A is the granddaughter of P. S is the father A. How is S related to Q ? (1) Son (2) Son-in-law (3) Brother (4) Brother-in-law (5) None of these (IDBI Bank Officer Exam.16.09.2012)

9. If ‘B × C’ means ‘B is the daughter of C’, ‘B + C’ means ‘B is the husband of C’ and ‘B – C’ means ‘B is the sister of C’, then what does ‘M + N – P × Q’ mean ? (1) M is the brother-in-law of Q (2) M is the uncle of Q (3) M is the son-in-law of Q (4) Q is the mother-in-law of M (5) None of these (Corporation Bank SO (Marketing) Exam, 22.02.2014)

BPRE–113

Directions (10–12) : Study the following information carefully and answer the questions given below : (BOB Manipal School of Banking Officer Online Exam, 14.08.2014)

A is the brother of B. L is the sister of Q. K is the son of B and L. M is the sister of K. 10. How A is related to L ? (1) Father (2) Brother (3) Nephew (4) Brother–in–law (5) Cannot be determined 11. How Q is related to M ? (1) Uncle (2) Aunt (3) Sister (4) Brother (5) Cannot be determined 12. M is son of P. P is mother of G. G is father of L. L is son of Q. How is L related to M ? (1) Uncle (2) Brother (3) Nephew (4) Son (5) Cannot be determined Directions (13–14) : Study the following information carefully and answer the questions given below : J is mother of P. P is brother of K. K is mother of V. Q is brother of V. G is father of Q. S is father of P. 13. Which of the following statements is/are true on the basis of given information ? (1) K is wife of G. (2) K is daughter of S. (3) G is father of V. (4) S is husband J. (5) All are true 14. Who among the following is grandmother of V ? (1) S (2) P (3) J (4) K (5) G Directions (15–16) : Study the following information carefully and answer the questions given below : (IDBI Bank Officer Exam, 22.08.2014)

A is brother of B. G is brother of H. A is married to sister of G. Q is son of A. B is son of K. H is daughter of M. M is wife of T.

BLOOD RELATIONSHIP 15. How is Q related to G ? (1) Nephew (2) Paternal Uncle (3) Maternal Uncle (4) Brother (5) Son 16. How is T related to A ? (1) Father (2) Father-in-law (3) Brother-in-law (4) Brother (5) Cannot be determined. Directions (17–21) : Study the following information carefully and answer the questions given below : (SIDBI Officer Exam, 03.09.2014)

‘P @ Q’ means ‘P is mother of Q’. ‘P + Q’ means ‘P is wife of Q’. ‘P $ Q’ means ‘P is brother of Q’. ‘P % Q’ means ‘P is father of Q’. ‘P & Q’ means ‘P is husband of Q’. 17. Which of the following means A is mother-in-law of D ? (1) A + M % L + D (2) B $ A @ D % K & Q (3) A @ T + N % D $ L (4) A + H % M $ K % D (5) None of these 18. In the expression ‘K & R @ T $ D & M @ S % G’ who among the following is father-in-law of M ? (1) R (2) T (3) K (4) S (5) G 19. Which of the following means H is mother of L ? (1) T & H @ N + L % D (2) K % H + R % L $ N (3) H + J % N & L @ T (4) L $ R % H @ T & V (5) None of these 20. In the expression ‘K % R $ T & D @ H + S’ how S is related to D? (1) Brother (2) Uncle (3) Cousin (4) Son-in-law (5) None of these 21. In the expression ‘S + D $ P % H & V @ T’, how P is related to T ? (1) Grandmother (2) Uncle (3) Mother (4) Father (5) Grandfather Directions (22–24) : Study the following information carefully and answer the questions given below : (SIDBI Officer Exam, 03.09.2014)

‘P $ Q’ means ‘P is father of Q’. ‘P + Q’ means ‘P is son of Q’. ‘P @ Q’ means ‘P is sister of Q’. ‘P % Q’ means ‘P is wife of Q’ P & Q’ means ‘P is husband of Q’.

22. In the expression ‘M + K $ T @ N’ how is M related to N ? (1) Sister (2) Cousin (3) Brother (4) Paternal Uncle (5) None of these 23. Which of the following expressions represents the relation ‘V is mother of D’ ? (1) V % Q $ R @ D (2) V % Q @ R $ D (3) D + T @ J $ V (4) V @ F $ D % M (5) None of these 24. In the expression ‘E + H @ K $ B’ how is B related to E ? (1) Brother (2) Sister (3) Cousin (4) Cannot be determined (5) None of these Directions (25-26): Study the following information carefully and answer the questions given below : (IBPS RRBs Officer Scale-I CWE, 06.09.2014)

H has two sons A and Y. A is married to M. M is the mother of P. N is daughter-in-law of H. S is motherin-law of H. 25. Who among the following is the uncle of P ? (1) H (2) A (3) Y (4) M (5) None of these 26. Who among the following is the wife of Y ? (1) N (2) H (3) P (4) S (5) M Directions (27–29) : Study the following information carefully and answer the questions given below : (Bank of Baroda Junior Management Grade/Scale-I Exam, 18.04.2015)

P @ Q means P is father of Q. P + Q means P is husband of Q. P $ Q means P is brother of Q. P % Q means P is mother of Q. P & Q means P is sister of Q. 27. How is B related to E in this expression : ‘A @ B % C & D + E’ ? (1) Grandmother (2) Granddaughter (3) Mother–in–law (4) Aunt (5) Daughter–in–law

BPRE–114

28. What should come in place of the question mark (?) to establish that C is the aunt of E in the following expression ? A%B+C&D?E (1) $ (2) & (3) + (4) Either + or & (5) Either @ or % 29. Which among the following options is true if the expression ‘A + B & C @ D % E $ F’ is definitely true ? (1) E is the sister–in–law of A. (2) F is daughter–in–law of C. (3) B is the aunt of D. (4) A is the uncle of E. (5) C is the uncle of A. Directions (30–32) : Study the following information carefully and answer the questions given below : (IBPS RRBs Officer Scale–I & II CWE 12.09.2015)

S is the sister of B. K is the brother of B. ● K is the son of L. L is married to Y. Y is the daughter of J. ● Y has only one daughter. J is married to C. ● K is the brother of D. D is married to T. 30. If P is the brother of C, then how is J related to P? (1) Brother (2) Cousin (3) Sister (4) Uncle (5) Cannot be determined 31. How is T related to S? (1) Brother–in–law (2) Cousin (3) Sister–in–law (4) Sister (5) Brother 32. If Z is the daughter of T, then how is B related to Z? (1) Uncle (2) Father (3) Aunt (4) Cannot be determined (5) Mother Directions (33–34) : Study the following information carefully and answer the questions given below : ●

(IBPS Bank PO/MT CWE–V (Preliminary) 03.10.2015)

R is married to U. U is the mother of L. L is the sister of D. U has only one daughter. D is married to J. K is the son of J. F is the mother of J.

BLOOD RELATIONSHIP 33. How is D related to F ? (1) Cannot be determined (2) Daughter (3) Daughter-in-law (4) Son-in-law (5) Son 34. How is R related to K ? (1) Cannot be determined (2) Grandfather (3) Grandmother (4) Father (5) Uncle Directions (35–39) : Study the following information carefully and answer the questions given below : (IBPS Bank PO/MT CWE–V (Preliminary) 03.10.2015)

Ten persons — J, K, L, M, N, O, P, Q, R and S — are sitting around a circular table facing the centre with equal distances between each other (but not necessarily in the same order). Each one of them is also related to M in some way or the other. Only two persons sit between Q and L. M sits second to the left of Q. Only three persons sit between L and M’s sister. M’s son sits second to the right of M’s sister. Only one person sits between M’s son and S. J sits to the immediate right of R. R is neither the son nor the mother of M. S is an immediate neighbour of M’s mother. Only three persons sit between M’s mother and M’s brother. M’s daughter sits second to the left of M’s brother. M’s father is not an immediate neighbour of M. M’s wife sits third to the right of K. L is to the right of Q. Only four persons sits between M and M’s father. 35. Who sits second to the right R ? (1) M’s brother (2) M (3) R (4) N (5) M’s daughter 36. How many persons sit between K and L, when counted from the left of K ? (1) Six (2) One (3) None (2) Two (5) Four 37. Which of the following statements is true with respect to the given information ? (1) R sits second to the right of M’s wife.

K is the brother of J. J has only one daughter. J is the mother of L. L is the sister of T. B is the father of T. T is married to R. 40. How is J related to R ? (1) Cannot be determined (2) Sister-in-law (3) Mother (4) Aunt (5) Mother-in-law 41. How is T related to K ? (1) Niece (2) Cannot be determined (3) Nephew (4) Son (5) Daughter Directions (42–46) : Study the following information carefully and answer the given questions :

Only three persons sit between N’s wife and L. N’s son sits second to the right of N’s father. Only two persons sit between N’s father and N’s daughter. 42. Who amongst the following is the son of J ? (1) M (2) P (3) K (4) O (5) Q 43. How many persons sit between N and K, when counted from the left of K ? (1) Five (2) One (3) Four (4) None (5) Three 44. Who sits to the immediate right of Q ? (1) N’s sister (2) N (3) N’s wife (4) K (5) J 45. Which of the following statements is true with respect to the given information ? (1) All the given options are true (2) P sits to the immediate left of J. (3) N’s mother sits to the immediate left of N. (4) M is the mother-in-law of Q. (5) N is an immediate neighbour of his father. 46. How is J related to K ? (1) Sister (2) Uncle (3) Father (4) Sister-in-law (5) Daughter Directions (47–48) : Study the following information carefully and answer the questions given below :

(IBPS Bank PO/MT CWE–V (Preliminary) 04.10.2015)

(IBPS Bank PO/MT CWE–V (Preliminary.) 04.10.2015)

Eight persons, J, K, L, M, N, O, P and Q are sitting around a circular table facing the centre with equal distances between each other (but not necessarily in the same order). Each one of them is also related to N in some way or the other. K sits third to the left of N. Only one person sits between N and Q. N’s sister sits to the immediate right of Q. Only two persons sit between N’s sister and N’s mother. J sits to the immediate right of N’s mother. P sits to the immediate right of M. N’s brother sits third to the right of P. N’s wife sits second to the left of N’s brother.

D is the father of A. D is married to P. P is the mother of J. P has only one daughter. J is married to U. U is the son of L.

(2) K is an immediate neighbour of R. (3) M sits second to the left of L (4) All the given options are true. (5) S is the daughter of L. 38. How is K related to R ? (1) Son-in-law (2) Uncle (3) Brother (4) Niece (5) Daughter 39. Who amongst the following is the wife of M ? (1) J (2) L (3) O (4) Q (5) N Directions (40–41) : Study the following information carefully and answer the questions given below : (IBPS Bank PO/MT CWE–V (Preliminary) 04.10.2015)

BPRE–115

47. How is J related to L? (1) Daughter (2) Granddaughter (3) Cannot be determined (4) Niece (5) Daughter–in–law 48. How is A related to U? (1) Cannot be determined (2) Brother–in–law (3) Brother (4) Sister (5) Sister–in–law

BLOOD RELATIONSHIP Directions (49-50) : Study the following information carefully and answer the questions given below : (IBPS Bank PO/MT CWE–V (Preliminary) 10.10.2015 Ist Sitting)

U is the mother of D. S is the sister of D. L is the father of S. L has only one daughter. M is the daughter of S. P is the daughter of D. 49. If R is married to S, then how is R related to U ? (1) Grandson (2) Nephew (3) Son-in-law (4) Uncle (5) Cannot be determined 50. How is S related to P ? (1) Aunt (2) Sister (3) Mother (4) Niece (5) Grandmother Directions (51–52) : Study the following information carefully and answer the questions given below :

Only three persons sit between M and N’s husband. A sits second to the right of N’s husband. N’s mother sits to the immediate right of C. C is not the husband of N. N’s daughter sits third to the right of P. D sits to the immediate left of N’s brother. 53. How many persons sit between P and N, when counted from the left of N? (1) One (3) Two (5) Three

54. Which of the following statements is true with respect to the given information? (1) A is the son of D. (2) C is an immediate neighbour of D.

(IBPS Bank PO/MT CWE–V (Preliminary) 10.10.2015)

Q is the sister of T. T is the mother of D. T has only one son. D is the brother of J. J is married to M. Y is the daughter of M. 51. How is J related to Q ? (1) Son (2) Cannot be determined (3) Daughter (4) Niece (5) Nephew 52. How is Y related to T ? (1) Daughter (2) Daughter-in-law (3) Cannot be determined (4) Niece (5) Granddaughter Directions (53–56) : Study the following information carefully and answer the questions given below :

(3) D sits third to the left of B. (4) All the given options are true. (5) A sits second to the right of N’s husband. 55. Who sits to the immediate left of P? (1) B

(2) M

(3) N’s sister

(4) N’s husband

(5) A 56. Who amongst the following is the son of N ? (1) P

(2) B

(3) D (5) O

(4) C

57. How is B related to A ? (1) Grandmother

(IBPS Bank PO/MT CWE–V

(2) Son-in-law

(Preliminary) 10.10.2015)

(3) Grandson

Eight persons — A, B, C, D, M, N, O and P — are sitting around a circular table facing the centre with equal distances between each other (but not necessarily in the same order). Each one of them is also related to N in some way or the other. Only three persons sit between B and N. Only one persons sits between N and P. N’s father sits to the immediate right of P. N’s sister sits third to the right of N’s father. Only one person sits between N’s sister and N’s son. M sits third to the left of N’s son.

(2) Five (4) None

(4) Uncle (5) Wife Directions (58–59) : Study the following information carefully and answer the questions given below : (IBPS RRBs Officer Scale–I & II CWE 13.09.2015) ● ● ● ●

K is the brother of J. J is the mother of Y. Y is the sister of T. T is married to Q. S is the father of J. S has only one daughter. S is married to R. K is the brother of D. U is the father-in-law of D.

BPRE–116

58. How is D related to Y ? (1) Cannot be determined (2) Mother (3) Uncle (4) Father (5) Aunt 59. If Y is married to P, then how is S related to P ? (1) Cannot be determined (2) Sister (3) Brother-in-law (4) Sister-in-law (5) None of these 60. If U is the father of C, then how is D related to C ? (1) Cousin (2) Husband (3) Wife (4) Cannot be determined (5) Brother 61. R is sister of M. M is brother of H. D is mother of K. K is brother of M. How is R related to D ? (1) Daughter (2) Mother (3) Other than those given as options (4) Sister (5) Data Inadequate (IBPS Bank PO/MT CWE–V Main Exam. 31.10.2015)

62. Pointing to a woman, Mr. Suresh said, she is the daughter of my grandfather’s only daughter. How is Suresh related to the woman? (1) Cousin (2) Brother (3) Other than those given as options (4) Uncle (5) Cannot be determined (IBPS Bank PO/MT CWE–V Main Exam. 31.10.2015)

Directions (63-64) : Study the following information carefully and answer the questions given below : (IBPS Bank PO/MT CWE–V Main Exam. 31.10.2015)

J is the father of T. P is the brother of J. L is the mother of V. V is the brother of T. T is mother of S. T is the daughter–in–law of W. 63. How is J related to S? (1) Uncle (2) Brother (3) Grand–father (4) Cousin (5) Father 64. How is W related to P? (1) Son (2) Cannot be determined

BLOOD RELATIONSHIP (3) Grandson (4) Aunt (5) Uncle Directions (65–67) : Study the following information carefully and answer the questions given below : (SIDBI Officer Online Exam.24.02.2016)

M is the father of G. P is the wife of G. M is married to C. L is sibling of G. S is the father–in–law of L. K is the only child of S. 65. How is G related to K? (1) Cousin (2) Father (3) Cannot be determined (4) Brother–in–law (5) Nephew 66. If K is the mother of R, then how is L related to R? (1) Father (2) Son–in–law (3) Brother–in–law (4) Son (5) Cannot be determined 67. How is C related to P? (1) Mother (2) Mother–in–law (3) Sister–in–law (4) Grandmother (5) Niece Directions (68–71) : Study the following information carefully and answer the questions given below : (United Bank of India PGDBF Manipal Exam,07.08.2016)

‘A × B’ means ‘A is the son of B’. ‘A + B’ means ‘A is the daughter of B’. ‘A ÷ B’ means ‘A is the wife of B’. ‘A – B’ means ‘A is the father of B’. 68. In the epxpression ‘S × T ÷ V × W ÷ Y’, how T is related to Y? (1) Daughter (2) Mother (3) Daughter – in – law (4) Aunt (5) Granddaughter 69. In the expression ‘P ÷ Q × R – T’, how P is related to T? (1) Sister – in – law (2) Aunt (3) Sister (4) Mother (5) Daughter 70. Which of the following expressions establishes that K is the grandson of D? (1) K – P × S + D ÷ N (2) K + P ÷ S – D × N (3) S – P + N + D ÷ K (4) K × P ÷ S × D – N (5) Other than those given as options

71. In the expression ‘H – R × C + T ÷ W × M’, how C is related to W? (1) Niece (2) Cousin (3) Daughter (4) Mother (5) Cannot be determined Directions (72–74) : Study the following information carefully and answer the questions given below : (IBPS Bank PO/MT CWE (Pre Exam), 16.10.2016 (First Sitting))

T is the daughter of P. P is the father of L. L is the only son of A. B is the daughter-in-law of A. W is the son of B. 72. How is P related to B ? (1) Father-in-law (2) Borther (3) Son-in-law (4) Father (5) Brother-in-law 73. If Y is the husband of T, how is L related to Y ? (1) Brother (2) Nephew (3) Son-in-law (4) Son (5) Brother-in-law 74. How is A related to W ? (1) Grandfather (2) Grandson (3) Granddaughter (4) Grandmother (5) Uncle Directions (75–77) : Study the following information carefully and answer the questions given below : (IBPS Bank PO/MT CWE-VI (Pre Exam), 16.10.2016 (Second Sitting)) ● ● ● ●

75.

76.

77.

D is daughter of N. E is wife of N. G is sister of D. C is married to G. N has no son. K is mother of E. Q is only daughter of C. How Q is related to D? (1) Daughter (2) Cousin (3) Niece (4) Sister-in-law (5) Cannot be determined How N is related to K? (1) Brother-in-law (2) Cousin (3) Brother (4) Sister (5) Son-in-law How many daughters does N have? (1) Two (2) Three (3) One (4) Cannot be determined (5) None of these

BPRE–117

Directions (78–80) : Study the following information carefully and answer the questions given below : (IBPS Bank PO/MT (Pre.) Exam, 23.10.2016)

A is the mother of B. B is the sister of C. D is the son of C. E is the brother of D. F is the mother of E. G is the granddaughter of A. H has only two children- B and C. 78. How is F related to H? (1) Son-in-law (2) Daughter-in-law (3) Father-in-law (4) Granddaughter (5) Niece 79. How is C related to E? (1) Father (2) Son (3) Mother (4) Cousin (5) Grandfather 80. Who is mother of G? (1) C (2) B (3) F (3) Either B or F (5) Either C or F Directions (81–83) : Study the following information carefully and answer the questions given below : (Bank of Maharashtra PO Exam, 26.10.2016)

A%B means A is brother of B A$B means A is husband of B A@B means A is the sister of B A#B means A is mother of B As per the given expression N@P%T#M$K#J@H 81. How is H related to M ? (1) Either ‘brother’ or ‘sister’ (2) Father (3) Either ‘nephew’ or ‘ niece’ (4) Either ‘son’ or ‘daughter’ (5) Uncle 82. How is K related to T ? (1) Daughter-in-law (2) Sister (3) Daughter (4) Mother (5) Niece 83. How is P related to M ? (1) Brother (2) Father-in-law (3) Father (4) Uncle (5) Nephew Directions (133–86) : Study the following information carefully and answer the questions given below : (IBPS RRBs Officer CWE (Pre.) Exam, 14.11.2016 (Shift-I))

S is the husband of D. A is the brother of D. A is the only son of B. D is the sister of Q. R is married to Q. M is the father of R. N is the daughter of Q.

BLOOD RELATIONSHIP 84. If V is the grandfather of N, then how is B related to R? (1) Uncle (2) Mother-in-law (3) Grandmother (4) Aunt (5) Father-in-law 85. How is S related to Q? (1) Father (2) Grandfather (3) Brother-in-law (4) Uncle (5) Nephew 86. How is D related to N? (1) Mother (2) Sister-in-law (3) Cousin (4) Mother-in-law (5) Aunt Directions (87–89) : Study the following information carefully and answer the questions given below : (IBPS RRBs Officer CWE (Pre.) Exam, 14.11.2016 (Shift-II))

L is the only child of K. R is married to L. S is the sister of R. S is the only daughter of B. J, the father of B has only two children. Q is the daughter of J. 87. How is J related to S? (1) Grandfather (2) Brother-in-law (3) Uncle (4) Cousin (5) Father-in-law 88. If J has only one daughter, then how is B related to L? (1) Nephew (2) Niece (3) Father-in-law (4) Brother (5) Mother-in-law 89. How is Q related to R? (1) Daughter-in-law (2) Grandmother (3) Niece (4) Aunt (5) Mother-in-law Directions (90–92) : Study the following information carefully and answer the questions given below : (IBPS RRBs Officer CWE (Pre.) Exam, 14.11.2016 (Shift-III))

K and M are the children of G. G is married to R. S is the sister of G. A is the only son of R. P is the son of K.

90. How is M related to P? (1) Father (2) Uncle (3) Aunt (4) Grandmother (5) Brother-in-law 91. How is S related to R? (1) Sister-in-law (2) Daughter-in-law (3) Niece (4) Daughter (5) Granddaughter 92. If S does not have any sister, then how is G related to P? (1) Uncle (2) Grandfather (3) Father-in-law (4) Aunt (5) Grandmother 93. If Neha says, Anuradha’s father Ramesh is the only son of my Father-in-law Mahesh, then how Bindu, who is sister of Anuradha is related to Mahesh ? (1) Daughter (2) Wife (3) Daughter-in-law (4) Niece (5) None of these (IBPS Bank PO/MT CWE (Main) Exam, 18.11.2016

Directions (94–96) : Study the following information carefully and answer the questions given below : (Indian Bank PO (Pre.) Exam, 21.01.2017 (Ist Sitting))

Q is the brother of P. K is the mother of P. Q is married to C. C is the daughter-in-law of L. P is the mother of N. N is the sister of V. 94. How is K related to C? (1) Sister (2) Daughter-in-law (3) Mother (4) Daughter (5) Mother-in-law 95. How is L related to N? (1) Grandfather (2) Uncle (3) Grandson (4) Son (5) Son-in-law 96. If N has only one brother as sibling, how is V related to Q? (1) Father (2) Nephew (3) Uncle (4) Son (5) Brother

BPRE–118

Directions (97–99) : Study the following information carefully and answer the questions given below : (Indian Bank PO (Pre.) Exam, 21.01.2017 (2nd Sitting))

L is the mother of P. P is married to Q, who is the father of D. E is the brother of D. M is the father of X, who is the sister of P. X is married to Y, who have a daughter B and a son A. 97. How is M related to D ? (1) Grandmother (2) Grandfather (3) Uncle (4) Father (5) Other than those given as options 98. How is A related to E ? (1) Sister (2) Nephew (3) Cousin (4) Uncle (5) Other than those given as options 99. How is Y related to L ? (1) Son-in-law (2) Nephew (3) Brother (4) Uncle (5) Other than those given as options Directions (100–102) : Study the following information carefully and answer the questions given below : (IBPS Bank PO/MT CWE-VII (Prelim Exam) 14.10.2017)

D is the father of only F and E. D has only one son. E is married to G. G is the son-in-taw of M. H is the only son of G. K and J are the children of E. L is married to K. 100. How is M related to J? (1) Grandmother (2) Aunt

(3) Mother

(4) Niece

(5) Granddaughter

101. How is H related to L? (1) Brother (2) Brother-in-law (3) Son-in-law (4) Nephew (5) Uncle 102. How is F related to K? (1) Grandfather (2) Either ‘brother’ or ‘sister’ (3) Aunt (4) Either ‘nephew’ or ‘niece’ (5) Uncle

BLOOD RELATIONSHIP Directions (103–106) : Study the following information carefully and answer the questions given below :

Directions (110–111) : Study the following information carefully and answer the questions given below :

(IBPS SO (Law Officer) CWE (Prelim Exam) 31.12.2017)

(IBPS Bank PO/MT CWE-VII (Prelim Exam) 15.10.2017)

Z is married to Y. Z is the father of F. F is married to E. T is the mother of E. P, the sister of E is the only daughter of S and is unmarried. S has only two children. J is the brother-in-law of E. 103. How is J related to Y? (1) Brother (2) Nephew (3) Son (4) Uncle (5) Father 104. If T is the daughter of X, then how is P related to X? (1) Niece (2) Granddaughter (3) Either ‘sister’ or ‘sister-in-law’ (4) Grandmother (5) Either ‘daughter’ or ‘aunt’ 105. If J is the grandson of M, then how is M related to Y? (1) Cannot be determined (2) Mother-in-law (3) Father (4) Mother (5) Father-in-law 106. How is S related to F? (1) Grandfather (2) Brother-in-law (3) Brother (4) Father-in-law (5) Uncle Directions (107–109) : Study the following information carefully and answer the questions given below :

R is the sister of Q. M is the father of R. V is the son of Q. C is the maternal grandfather of V. M does not have married daughter. 110. How is R related to V? (1) Uncle (2) Aunt (3) Mother (4) Cannot be determined (5) Nephew 111. If B is married to Q, then how is B related to M? (1) Grandson (2) Son-in-law (3) Son (4) Daughter-in-law (5) Cannot be determined Directions (112-113) : In the following questions, the symbols #, &, @, * , $, % and © are used with the following meanings as illustrated below. Study the following information and answer the given questions : M # N means M is the son of N. M @ N means M is the daugther of N. M © N means M is the father of N. M $ N means M is brother of N. M * N means M is the husband of N. M & N means M is the daughterin-law of N. M % N means M is the wife of N

(IDBI Bank PO Exam 29.04.2018)

P and B are the only children of G. B is married to K, the only daughter of H and V. S is the only son of B. J is the grandfather of S. J has only one son. P is unmarried. 107. How is S related to G? (1) Grandson (2) Son (3) Nephew (4) Brother (5) Uncle 108. How is P related to J? (1) Brother (2) Son (3) Daughter (4) Niece (5) Nephew 109. If X is the brother-in-law of B, how is X related to H? (1) Father-in-law (2) Son (3) Cousin (4) Nephew (5) Uncle

(IBPS Bank PO/MT CWE (Main Exam) 26.11.2017)

112. If D & B % S © G % E © Q $ D then how is S related to D as there are only three generations? (1) Grandfather (2) Son-in-law (3) Daughter (4) Daughter-in-law (5) Grandson 113. If G & B * S @ W © F % O © Z, then how is Z related to S in the given arrangement? (1) Son (2) Sister (3) Cousin (4) Niece or Nephew (5) Daughter

BPRE–119

Directions (114–116) : Study the following information carefully and answer the questions given below : F is the husband of G. K is the mother-in-law of G. H is the father of F. M is the mother of H. P is the mother of K and B. (IBPS RRBs Officer CWE (Prelim Exam) 18.08.2018)

114. If Y is the father of H then how is Y related to M? (1) Mother (2) Father (3) Sister (4) Brother (5) Husband 115. How is P related to F? (1) Grandfather (2) Aunt (3) Mother (4) Grandmother (5) Wife 116. How is B related to H? (1) Sister (2) Brother (3) Can’t be determined (4) Husband (5) Wife Directions (117–119) : Study the following information carefully and answer the questions given below : B is married to D. P is the brother of B. R is the father-in-law of D. Q is the mother of R and G. K is the only daughter-in-law of Q. M is the only daughter of K. 117. If K has no brother-in-law, then how is G related to P? (1) Uncle (2) Grandmother (3) Sister (4) Father-in-law (5) Aunt 118. How is Q related to M? (1) Grandmother (2) Mother-in-law (3) Sister-in-law (4) None of those given as options (5) Aunt 119. How is B related to G? (1) Daughter-in-law (2) Nephew (3) Either ‘son-in-law’ or ‘daughter-in-law’ (4) Either ‘son’ or ‘daughter’ (5) Grandson

BLOOD RELATIONSHIP Directions (120–121) : In these questions, four sentences are given. Decide which sentence(s) is/are correct with regard to grammar, meaning and usage and select your answer accordingly. (Indian Bank PO Online Prelim Exam 06.10.2018)

120. (1) The newly introduced flights between the two cities have been so designed allowing them to serve the business and leisure travellers alike. (2) All are correct. (3) New flights introduced between the two cities are designed to cater to business and leisure travellers alike. (4) The design of the newly introduced flights is such that they cater to business and leisure travellers alike. (5) The newly introduced flights have been designed in such a way so as to cater to business and leisure travellers alike. 121. (1) The Prime Minister has instructed the Ministries to submit a detailed note on the projects and programmes undertaken by them. (2) Ministries have been instructed by the Prime Minister to submit a detailed note on the projects and programmes undertaken by them. (3) The instructions to submit a detailed note on the projects and programmes undertaken have been given by the Prime Minister to the ministries. (4) A detailed note on the projects and programmes undertaken has to be submitted by the ministries as per the instructions of the Prime Minister. (5) All are correct. Directions (122–124) : Study the following information carefully and answer the questions given below : J is the father of S. J has only one child. D is sister-in-law of S. K is the only sibling of D. D is unmarried. M is mother of K. H is grandson of M. L is sister of H. (IBPS Bank PO/MT CWE (Prelim Exam) 14.10.2018)

122. How is L related to D? (1) Mother (2) Sister-in-law (3) Daughter (4) Niece (5) Aunt 123. If T is grandfather of L then how is K related to T? (1) Son-in-law (2) Niece (3) Nephew (4) Daughter-in-law (5) Cannot be determined 124. How is J related to H? (1) Father (2) Father-in-law (3) Brother (4) Grandfather (5) Uncle Directions (125–126) : Study the following information carefully and answer the questions given below: R is the sister of Q. M is the father of R. V is the son of Q. C is the maternal grandfather of V. M does not have married daughter. (IBPS Bank PO/MT CWE (Prelim Exam) 21.10.2018)

125. How is R related to V? (1) Uncle (2) Aunt (3) Mother (4) Cannot be determined (5) Nephew 126. If B is married to Q, then how is B related to M? (1) Grandson (2) Son-in-law (3) Son (4) Daughter-in-law (5) Cannot be determined Directions (127–129) : Study the following information carefully and answer the questions given below. A % B means A is brother of B A $ B means A is husband of B A @ B means A is sister of B A # B means A is mother of B As per the given symbols N@P%T#M$K#J@H (Canara Bank PO Exam, 23.12.2018)

127. How is H related to M ? (1) Either ‘brother’ or ‘sister’ (2) Father

BPRE–120

(3) Either ‘nephew’ or ‘niece’ (4) Either ‘son’ or ‘daughter’ (5) Uncle 128. How is K related to T ? (1) Daughter-in-law (2) Sister (3) Daughter (4) Mother (5) Niece 129. How is P related to M ? (1) Brother (2) Father-in-law (3) Father (4) Uncle (5) Nephew 130. Arjit said, pointing towards a picture, “That picture is of sister of grandson of father of my maternal uncle”. How is lady in the picture related to Arjit? (1) Father’s sister (2) Mother’s sister (3) Cousin (maternal brother) (4) Cousin (maternal sister) (5) None of these IBPS RRBs Officer CWE (Prelim Exam) 17.08.2019

Directions (131–132) : Study the following information carefully and answer the questions given below : IBPS Bank PO/MT CWE (Prelim) Exam, 12.10.2019

There are eight family members namely P, Q, R, S, T, U, V and W. There are three married couples in the family. P is married to V. V is the father of R. R is the sister of U. T is the niece of U. Q is the father of T. S is the brother-in-law of P. U is the husband of W. 131. How is V related to W? (1) Mother-in-law (2) Father-in-law (3) Daughter-in-law (4) Sister-in-law (5) Cannot be determined 132. How many male members are there in the family? (1) Two (2) One (3) Three (4) Four (5) None of these

BLOOD RELATIONSHIP 133. A is married to B. A is the daughter of R. J is the daughter-in-law of R. G is the son of J. How is G related to B? (1) Nephew (2) Grandson (3) Brother-in-law (4) Son-in-law (5) Brother IBPS Bank PO/MT CWE (Prelim) Exam, 19.10.2019

Directions (134–136) : Study the following information carefully and answer the questions given below : (IBPS SO Prelim Exam, 28.12.2019)

Q is the father of B. Q is married to G. B is the brother of S. M is the only daughter of G. F is the only daughter of R and M. Only one of the children of Q is married. X is the only aunt of F. 134. How is Q related to R? (1) Daughter (2) Son-in-law (3) Brother-in-law (4) Father (5) Father-in-law 135. If A is a grandchild of G, then how is A related to S? (1) Sister (2) Niece (3) Nephew (4) Aunt (5) Brother 136. How is X related to M? (1) Brother (2) Daughter-in-law (3) Brother-in-law (4) Sister-in-law (5) Sister 137. There are six persons in a family namely A, B, C, D, E and F, all of them are related to each other in some or the other way. B is the only daughter of C. E is the sister-in-law of B, but is not married to F. F is the son of A. A has three children. C is the wife of A. How is D related to B? (1) Sister (2) Sister-in-law (3) Brother (4) Mother (5) Father (IBPS RRBs Officer CWE Prelim Exam, 13.09.2020)

138. Raksha is daughter of Reena. Reena is mother-in-law of Roma who is wife of Rakshit. Roshan is father of Rakshit and Ritik. How is Roma related to Raksha? (1) Cousin (2) Mother-in-law (3) Niece (4) Sister-in-law (5) Aunt (IBPS Bank PO/MT CWE Prelim Exam, 03.10.2020)

Directions (139–143) : In the following questions some symbols are used with the following meaning as illustrated below : (IBPS RRBs Officer CWE Prelim Exam, 31.12.2020) ●

X © Y ⇒ X is the parent of Y.



X & Y ⇒ X is the son-in-law of Y.



X ! Y ⇒ X is the brother of Y.



X : Y ⇒ X is the paternal grandfather of Y.



X > Y ⇒ X is the mother of Y.



X ^ Y ⇒ X is the sister of Y.



X @ Y ⇒ X is the child of Y.



X + Y ⇒ X is the niece or nephew of Y.



X * Y ⇒ X is the husband of Y.



X ? Y ⇒ X is the daughter of Y.



X # Y ⇒ X is the son of Y.



X = Y ⇒ X is the mother-in-law of Y.



X $ Y ⇒ X is the father of Y.



X % Y ⇒ X is the wife of Y.

A family consists of eight members - E, J, K, L, Q, W, X and Y – with four male members and four female members. Consider the following statements and answer the questions given below : ● W ? E = J $ L ● L + Y + Q ! E ● X ^ Y # K © W ● W belongs to the second generation and is unmarried. 139. How is J related to Y ? (1) Brother-in-law (2) Brother (3) Son (4) Sister-in-law (5) Father-in-law

BPRE–121

140. How is W related to L ? (1) Nephew (2) Niece (3) Aunt (4) Daughter (5) Cousin 141. How is X related to Q ? (1) Son-in-law (2) Father-in-law (3) Mother-in-law (4) Nephew (5) Niece 142. How is K related to L ? (1) Maternal aunt (2) Paternal uncle (3) Cousin (4) Grandfather (5) Granddaughter 143. How is Y related to E ? (1) Cousin (2) Son (3) Niece (4) Nephew (5) Sister-in-law Directions (144 –148) : Study the following information carefully and answer the questions given below : (IBPS RRBs Officer CWE Prelim Exam, 07.08.2021)

Q is the son of D. P is the son of M. W is the mother-in-law of Y. P is the son-in-law of Z. T is the mother of J. Y is the father of U. T is the daughter-in-law of B. Y has only one son and only one daughter. U is the daughter of T. T is the sister of D. D is married to P. B is the grandfather of J. 144. Who among the following do/ does not belong to the third generation ? I. J II. T III. Q IV. M V. U (1) Only I (2) Only IV (3) Only II and IV (4) Only IV and V (5) Only I, II and IV 145. How is W related to T? (1) Mother-in-law (2) Maternal Uncle (3) Brother-in-law (4) Son (5) Mother

BLOOD RELATIONSHIP 146. If M is the wife of R, how R is related to Q? (1) Paternal grandfather (2) Maternal grandfather (3) Paternal uncle (4) Maternal uncle (5) Cousin brother 147. How is D related to Y ? (1) Sister (2) Niece (3) Daughter-in-law (4) Sister-in-law (5) Aunt 148. How is U related to Z? (1) Great granddaughter (2) Granddaughter (3) Daughter-in-law (4) Daughter (5) Sister-in-law Directions (149–153) : Study the following information carefully and answer the questions given below : (IBPS Bank PO/MT CWE Prelim Exam, 04.12.2021)

There are nine members in a family viz., A, B, C, D, E, F, G, H and J. There are four married couples in the family and each married couple has at least one child. J is not the son of B. There are two sons-in-law in the family. B is mother-in-law of C. F is the son-in-law of D. A and H are blood related and belong to the second generation. F and B are not blood related but belong to the second generation. E is the eldest grandchild in the family. H is blood related to G. B does not have blood relation with anyone in the family except E who is a female. D has one daughter and one son. G is the grandmother of E. A is aunt of E. 149. Who among the following is NOT a female member in the family ? (1) G (2) D (3) A (4) B (5) E 150. How is J related to B’s spouse ? (1) Brother-in-law (2) Son-in-law (3) Father-in-law (4) Nephew (5) Grandson 151. How is E related to A ? (1) Niece (2) Daughter (3) Son (4) Nephew (5) Cousin 152. How is G related to A ? (1) Great grandmother (2) Grandmother (3) Mother (4) Maternal aunt (5) Paternal aunt

153. How is J related to E ? (1) Maternal uncle (2) Paternal aunt (3) Son-in-law (4) Brother-in-law (5) Cousin brother

SBI PO EXAMS Directions (1–4) : Study the following information carefully and answer the questions given below : (SBI Management Executive Exam, 19.09.2014)

‘P @ Q’ means ‘P is sister of Q’. ‘P + Q’ means ‘P is brother of Q’. ‘P $ Q’ means ‘P is husband of Q’. ‘P % Q’ means ‘P is mother of Q’. ‘P & Q’ means ‘P is father of Q’. 1. Which of the following indicates that ‘D is niece of A’\ (1) A + M % R $ D @ V (2) B & D $ H + A % T (3) A $ F % W @ D % M (4) A + K & D @ L $ Q (5) None of these 2. What will come in the place of question - mark (?) in the expression ‘A + B @ C & D ? E’ if it indicates that ‘C is father - in law of E’? (1) & (2) % (3) $ (4) + (5) @ 3. Which of the following can be the correct conclusion drawn from the expression ‘A % B @ C + D $ E % F’ ? (1) D is daughter of (2) C is paternal uncle of F (3) A is father of C and E (4) A has two daughters and only one son. (5) None of these 4. Which of the following can be the correct conclusion drawn from the expression \ ‘B % C @ D + E & F’ ? (1) E is sister of C and D (2) C is aunt of E (3) D is daughter of B (4) B has two daughters (5) B is grandmother of F. Directions (5–7) : Study the following information carefully and the questions given below : (SBI Associates PO Online Exam, 29.11.2014)

M is the mother of B. A is the husband of M. N is the only brother of B. C is married to N. Q is the only child of C. N does not have any sister. J is the father of A.

BPRE–122

5. If A does not have any grandson, then how is Q related to B? (1) Cannot be determined (2) Sister–in–law (3) Daughter–in–law (4) Niece (5) Nephew 6. How is A related to C ? (1) Uncle (2) Cannot be determined (3) Husband (4) Nephew (5) Brother–in–law 7. How is B related to J ? (1) Father (2) Nephew (3) Brother–in–law (4) Brother (5) Grandson Directions (8–10) : Study the following information carefully and answer the questions given below : (SBI Associates PO Online Exam, 30.11.2014)

A and Y are brothers of K. Y is the son of P and S. P is the daughter of X. M is the father in law of S. Q is the son of X. 8. If J is brother of X, then how is J related to Q ? (1) Uncle (2) Nephew (3) Cannot be determined (4) Brother-in-law (5) Son-in-law 9. How is Y related to M ? (1) Nephew (2) Father (3) Brother-in-law (4) Grandson (5) Brother 10. How is K related to Q ? (1) Cannot be determined (2) Niece (3) Daughter (4) Nephew (5) Son-in-law Directions (11–14) : Study the following information carefully and answer the questions given below : (SBI Management Executive Exam. 19.09.2014)

‘P @ Q’ means ‘P is sister of Q’. ‘P + Q’ means ‘P is brother of Q’. ‘P $ Q’ means ‘P is husband of Q’. ‘P % Q’ means ‘P is mother of Q’. ‘P & Q’ means ‘P is father of Q’. 11. Which of the following indicates that ‘D is niece of A’\ (1) A + M % R $ D @ V (2) B & D $ H + A % T (3) A $ F % W @ D % M (4) A + K & D @ L $ Q (5) None of these

BLOOD RELATIONSHIP 12. What will come in the place of question - mark (?) in the expression ‘A + B @ C $ D ? E’ if it indicates that ‘C is father - in law of E’? (1) & (2) % (3) $ (4) + (5) @ 13. Which of the following can be the correct conclusion drawn from the expression ‘A % B @ C + D $ E % F’ ? (1) D is daughter of (2) C is paternal uncle of F (3) A is father of C and E (4) A has two daughters and only one son. (5) None of these 14. Which of the following can be the correct conclusion drawn from the expression \ ‘B % C @ D + E & F’ ? (1) E is sister of C and D (2) C is aunt of E (3) D is daughter of B (4) B has two daughters (5) B is grandmother of F. Directions (15–17) : Study the following information carefully and the questions given below : (SBI Associates PO Online Exam. 29.11.2014)

M is the mother of B. A is the husband of M. N is the only brother of B. C is married to N. Q is the only child of C. N does not have any sister. J is the father of A. 15. If A does not have any grandson, then how is Q related to B ? (1) Cannot be determined (2) Sister–in–law (3) Daughter–in–law (4) Niece (5) Nephew 16. How is A related to C ? (1) Uncle (2) Cannot be determined (3) Father–in–law (4) Nephew (5) Brother–in–law 17. How is B related to J ? (1) Father (2) Nephew (3) Brother–in–law (4) Brother (5) Grandson Directions (18–20) : Study the following information carefully and answer the questions given below : (SBI Associates PO Online Exam. 30.11.2014)

A and Y are brothers of K. Y is the son of P and S. P is the daughter of X. M is the father in law of S. Q is the son of X. 18. If J is brother of X, then how is J related to Q ? (1) Uncle (2) Nephew (3) Cannot be determined (4) Brother-in-law (5) Son-in-law 19. How is Y related to M ? (1) Nephew (2) Father (3) Brother-in-law (4) Grandson (5) Brother 20. How is K related to Q ? (1) Cannot be determined (2) Niece (3) Daughter (4) Nephew (5) Son-in-law Directions (21–23) : Study the following information carefully and answer the questions given below : (SBI PO Phase–I (Preliminary) Online Exam. 20.06.2015)

S is the daughter of U. V has only two children– S and Y. Y is married to D. P is the brother of B. V has only two daughters. J is the mother of U. J is married to L. P is married to S. V is the son of T. 21. Who among the following is the sister–in–law of B ? (1) Y (2) S (3) U (4) J (5) T 22. Who among the following is the father of U ? (1) J (2) T (3) V (4) L (5) None of these 23. How is V related to P ? (1) Father–in–law (2) Mother–in–law (3) Father (4) Mother (5) Brother Directions (24–26) : Study the following information carefully and answer the questions given below : (SBI PO Phase–I (Preliminary) Online Exam. 21.06.2015)

J is the sister of T. T is married to D. D is the father of R. M is the son of H. T is the motherin-law of H. D has only one son and no daughter. J is married to B. L is the daughter of B.

BPRE–123

24. How is L related to T ? (1) Niece (2) Sister (3) Cannot be determined (4) Daughter (5) Mother 25. How is M related to D ? (1) Nephew (2) Uncle (3) Brother (4) Son (5) Grandson 26. How is J related to R ? (1) Sister (2) Aunt (3) Mother-in-law (4)Mother (5) Cannot be determined Directions (27 – 30) : Study the following information carefully and answer the questions given below : (SBI PO Phase–I (Preliminary) Online Exam. 27.06.2015)

T is the sister of D. D is married to P. P is the son of M. ● T is the mother of J. Y is the father of U. ● Y has only one son and only one daughter. ● U is the daughter of T. Q is the son of D. 27. How is P related to T ? (1) Brother (2) Cannot be determined (3) Brother-in-law (4) Cousin brother (5) Uncle 28. How is J related to D ? (1) Son (2) Niece (3) Son-in-law (4) Nephew (5) Daughter ●

29. How is Q related to M ? (1) Son-in-law (2) Grandson (3) Nephew (4) Son (5) Cannot be determined 30. Introducing a boy, a girl said, “He is the son of the daughter of the father of my uncle.” How is the boy related to the girl ? (1) Brother (2) Uncle (3) Nephew (4) Son-in-law (5) Brother-in-law (SBI Assistant Manager (System) Exam, 17.01.2016)

Directions (31–33) : Study the following information and answer the questions given below : (SBI PO Online (Pre.) Exam, 02.07.2016 (Ist Shift))

Q is the sister of M. P is wife of M. P has only one son R. S is mother of P. S is married to T. T has only one son and only one daughter

BLOOD RELATIONSHIP 31. If P is sister of J, how is J related to M? (1) Sister–in–law (2) Cannot be determined (3) Brother (4) Brother–in–law (5) Uncle 32. As per the given information, how is M related to S? (1) Son (2) Son–in–law (3) Niece (4) Nephew (5) Daughter–in–law 33. As per the given information, how is R related to Q? (1) Nephew (2) Cannot be determined (3) Uncle (4) Niece (5) Aunt 34. N is the sister of T. T is the only son of Y. Y is the mother of K. J is the daughter of K. J is the sister of M. D is married to K. If X is the wife of M, how D is related to X? (1) Father (2) Father–in–law (3) Mother (4) Mother–in–law (5) Cannot be determined (SBI PO Online (Pre.) Exam, 02.07.2016 (IInd Shift))

Directions (35–37) : Study the following information carefully and answer the questions given below : (SBI PO Phase-I (Pre.) Online Exam, 03.07.2016 (Ist Shift))

D is the mother of S. S is the sister of T. T is the mother of R. R is the only son of J. J is the father of U. U is married to K. 35. How is J related to D? (1) Brother–in –law (2) Son–in–law (3) Grandson (4) Son (5) Nephew 36. If R is the father of X, then how is K related to X? (1) Father (2) Aunt (3) Uncle (4) Father–in–law (5) Mother 37. How is S related to R? (1) Sister–in–law (2) Mother–in–law (3) Grandmother (4) Aunt (5) Mother

Directions (38–40) : Study the following information carefully and answer the questions given below : (SBI PO Phase-I (Pre.) Online Exam, 03.07.2016 (IInd Shift))

L is the sister of D. D is the mother of U. U is married to T. T is the only son of Z. Z is the son of R. R is the husband of Q. 38. If Z is the father of X, then how is X related to U? (1) Brother (2) Sister-in-law (3) Cousin (4) Brother-in-law (5) Sister 39. How is U related to L? (1) Son-in-law (2) Daughter (3) Daughter-in-law (4) Niece (5) Nephew 40. How is Q related to T? (1) Sister-in-law (2) Mother-in-law (3) Grandmother (4) Mother (5) Aunt Directions (41–43) : Study the following information carefully and answer the questions given below : (SBI PO Online (Preliminary) Exam, 10.07.2016)

S is the mother of D. K is the brother of D. K is the only son of M. M is the son of U. U is the husband of T. T is the mother of Y. 41. If D is married to X, then how is X related to M? (1) Son–in–law (2) Daughter–in–law (3) Son (4) Niece (5) Daughter 42. How is T related to K? (1) Mother (2) Aunt (3) Granddaughter (4) Grandmother (5) Mother–in–law 43. How is S related to Y? (1) Sister (2) Cousin (3) Mother–in–law (4) Niece (5) Sister-in-law 44. V is the brother of M and N. P is the mother of N. R is the father of V. Which of the following alternatives cannot be said to be definitely true? (1) R is the brother of M (2) R is the husband of P

BPRE–124

(3) V is the son of P (4) P is the mother of V (5) M is either son or daughter of R (SBI PO Online Main Exam, 31.07.2016)

45. S is the only son of V. V is married to R. M is the daughter of R. R is the grandmother of A. How is S definitely related to A? (1) Uncle (2) Cannot be determined (3) Father (4) Grandfather (5) Cousin (SBI PO Online (Preliminary) Online Exam. 06.05.2017)

46. C is the mother-in-law of Z. Z is the only child of P. P is the grandfather of S. S is the niece of T. If C has only one daughter, how is T definitely related to Z? (1) Daughter-in-law (2) Brother-in-law (3) Sister (4) Cannot be determined (5) Sister-in-law (SBI PO Online (Preliminary) Online Exam. 06.05.2017)

47. Q is the daughter of A. J is the brother of Q. J is the son of R. J is the father of S. If it is given that A is mother of Q, then what is the relation of R with respect to S? (1) Father (2) Mother-in-law (3) Mother (4) Father-in-law (5) Grandfather (SBI PO Online (Preliminary) Exam, 07.05.2017)

Directions (48–49) : Study the following information carefully and answer the questions given below : Ten persons P, Q. R, S, T, U, V, W, X and Z are there in a family of three generations. There are three married couples. S is the sister-in-law of R but not married to P. P is the son of V. W is the brother of T. Z is the father-in-law of Q. U is the mother of X and married to the brother of R. Q has only one sister, who is unmarried. X has no sibling. Z has only one grandson and no girl child. T is the youngest person of the family. (SBI PO Phase-II (Main) Exam 05.08.2018)

48. What is the ratio of male members to female members of the family respectively? (1) 5:4 (2) 1:1 (3) 3:2 (4) 2:3 (5) None of these

BLOOD RELATIONSHIP 49. How is S related to T? (1) Mother (2) Father (3) Uncle (4) Aunt (5) Grandmother 50. P is the brother of Q and R. S is R’s mother. T is P’s father. Which of the following statements cannot be definitely true? (1) T is Q’s father (2) S is P’s mother (3) P is S’s son (4) Q is T’s son (5) None of these SBI PO (Prelim Exam), 08.07.2018 (Shift-I))

Directions (51–52) : Study the following information carefully and answer the questions given below : Q is the mother of G. M is the father of Q. The only sibling of M is the father of W. J is the only sister-inlaw of M. R is the mother-in-law of J. R does not have any grandson. SBI PO (Prelim Exam), 08.07.2018 (Shift-II))

51. If K is the only uncle of Q, then how is K related to W? (1) Father-in-law (2) Uncle (3) Grandfather (4) Brother-in-law (5) Father 52. Which of the following statements is true with respect to the given information? I. Q is the daughter-in-law of R. II. W is the aunt of G. III. J is the mother-in-law of Q. (1) Both I and III (2) Both I and II (3) Both II and III (4) Only II (5) Only III Directions (53–55) : Study the following information carefully and answer the questions given below : U is the daughter-in-law of J. J is the father of only L and P. R is the niece of P. P’s spouse has no sibling. M is the grandfather of R. T is the only child of M. SBI PO Prelim Exam, 09.06.2019

53. How is U related to R? (1) Mother (2) Sister-in-law (3) Sister (4) Mother-in-law (5) Aunt 54. If J has only one son, then how is T related to L? (1) Husband (2) Wife

(3) Brother:in-law (4) Brother (5) Sister-in-law 55. How is J related to T ? (1) Brother (2) Father-in-law (3) Uncle (4) Grandfather (5) Grandson Directions (56–60) : Study the following information carefully and answer the questions given below : There are nine members A, B, C, D, E, F, G, H and I in a family. There are 4 couples and each couple has at least one child. ● G is the grandmother of E who calls A as aunt. ● I is not the son of B and there are two son in laws in the family. ● B is the mother in law of C and F is the son in law of D. ● A and H are blood related and belong to 2nd generation whereas F and B are not blood related but belong to 2nd generation. ● E is the eldest grandchild in the family and H is blood related to G. ● B does not have blood relation with anyone in the family except E who is a female. ● D has one daughter and one son. SBI PO Main Exam, 20.07.2019

56. How is B related to G? (1) Daughter (2) Mother–in–law (3) Sister–in–law (4) Aunt (5) Daughter–in–law 57. How is I related to E? (1) Maternal uncle (2) Paternal aunt (3) Son–in–law (4) Cousin brother (5) Brother–in–law 58. How is G related to A? (1) Great grand mother (2) Grand mother (3) Mother (4) Maternal aunt (5) Paternal aunt 59. How is E related to A? (1) Niece (2) Son (3) Nephew (4) Daughter (5) Cousin 60. How is I related to B’s spouse? (1) Brother–in–law (2) Nephew (3) Father–in–law (4) Son–in–law (5) Grandson

BPRE–125

RBI GRADE–B/ NABARD GRADE–A OFFICER EXAMS Directions (1-5) : Study the following information carefully and answer the given questions. (RBI Grade-B Officer’s Exam. 18.12.2011)

Eight family members A, B, C, D, E, F, G and H are sitting around a circular table, facing the centre but not necessarily in the same order. ● F, the wife of D is sitting third to right of C. ● A is the son of H. A is sitting second to left of D. D is not an immediate neighbour of either F or C. No male is an immediate neighbour of D. ● G sits second to left of D’s son. Only two persons sit between H and A’s brother. Neither C nor D is the brother of A. ● D’s son and the wife of D’s son are immediate neighbours of each other. ● F is the mother of H. F is not an immediate neighbour of B and G. ● G is the sister of E. 1. Who amongst the following is D’s son ? (1) E (2) G (3) A (4) B (5) Cannot be determined 2. Who sits second to the left of G ? (1) A’s brother (2) G’s mother (3) D (4) B’s father (5) A’s aunt 3. How many people sit between A and his brother ? (1) None (2) One (3) Two (4) Three (5) Four 4. Who amongst the following sits exactly between H and F ? (1) D’s wife (2) D’s son (3) C (4) B (5) A 5. Who amongst the following is the brother of A ? (1) E (2) G (3) A (4) B (5) Cannot be determined 6. Based on the given arrangement, how is A related to D ? (1) Grandfather (2) Son (3) Grandson (4) Daughter-in-law (5) Cannot be determined

BLOOD RELATIONSHIP 7.

8.

9.

Four of the following five are alike in a certain way based on the given arrangement and so form a group. Which is the one that does not belong to that group ? (1) B (2) C (3) H (4) G (5) F Which of the following statements regarding H is definitely correct? (1) H is a male (2) H is the cousin of C (3) Both the immediate neighbours of H are males (4) H is the daughter-in-law of D (5) H is the father of A Read the following information carefully and answer the question which follows : A is the brother of B. B is the sister of T. T is the mother of P. If it is provided that R is the grandfather of P, how would T be related to R ? (1) Daughter (2) Granddaughter (3) Sister (4) Wife (5) Daughter or Daughter-in-law (RBI Grade-B Officer’s Exam. 18.12.2011)

Directions (10-14) : Study the following information carefully and answer the questions given below : (RBI Officer Grade ‘B’ Phase-I Exam, 03.08.2014)

‘P © Q’ means ‘Q is the brother of P’. ‘P # Q’ means ‘P is the daughter of Q’. ‘P = Q’ means ‘Q is the sister of P’. ‘P £ Q’ means ‘P is the son of Q’. ‘P ★ Q’ means ‘P is the father of Q’. ‘P @ Q’ means ‘P is the mother of Q’. 10. What does the expression ‘P @ R = S © T £ V’ ? (1) V is the husband of P (2) R is the son of V (3) R is the daughter of V (4) V is the wife of P (5) None of these 11. Which of the following indicates that ‘C is the paternal uncle of D’? (1) C £ V # N @ L © D (2) C £ V £ L @ N © D (3) D £ L £ N @ V © C (4) D £ N # V @ L © C (5) None of these

12. Which of the following can be the correct conclusion drawn from the expression ‘L = M # N © P ★ Q’ ? (1) Q is the grandson of M (2) L is the uncle of N (3) N is the uncle of Q (4) Q is the niece of N (5) None of these 13. Which of the following can be correct conclusion drawn from the expression ‘Q £ N @ S © M = P’ ? (1) S is the brother of P (2) N has two sons and two daughters (3) S is the sister of Q (4) P is the sister of Q (5) None of these 14. Which of the following indicates that ‘Q is the daughter of N’ ? (1) Q ★ P # C @ N @ V (2) N ★ P # C @ Q @ V (3) M @ N # R ★ Q (4) M © Q = V # N (5) None of these Directions (15–16) : Study the following information carefully and answer the questions given below : (Nabard Officer Grade ‘A’ Online Exam. 01.03.2015)

T is the mother of L. L has only one brother, Q. Q is the son of Z. V is the only child of Q. M is the son-in-law of Q. Q is the brother of A. 15. Which of the following is/are required to establish that L is the uncle of V ? (1) Z has only one daughter (2) L is married to B (3) T is the mother of both L and A (4) No extra information is required as the relation can be established from the given information. (5) T has only four children. 16. Which of the following statements is true based on the given relationships ? (1) M is the grandson of Z (2) L is the aunt of M. (3) None of the given options is true (4) A is the son of T (5) V is the granddaughter of Z. 17. P is the only sister of M and N. B is the brother-in-law of M. None of the sons of W are married. W is the mother of N. W is married to Y. V, the wife of T has a daughter, W.

BPRE–126

How is B related to T? (1) Grandson (2) Grandson in-law (3) Granddaughter (4) Nephew (5) Son-in-law (RBI Officer Grade ‘B’ Phase-I Exam, 04.09.2016 (Shift-I))

18. T is the father of M and P. P is the only daughter of V. M is married to N. A and B are children of M. How is V related to B ? (1) Grandmother (2) Uncle (3) Aunt (4) Sister (5) Grandfather (RBI Officer Grade ‘B’ Phase-I Exam, 04.09.2016 (Shift-II))

Directions (19–23) : Study the following information carefully and answer the questions given below : (RBI Officer Grade ‘B’ Phase-I Exam 17.06.2017)

Seven people — P, Q, R, S, T, U and V — are related to each other in some or the other way. Each person is of a different age, but not necessarily in the same order. (Note : It is assumed that the husband is older than the wife.) T is older than V but younger than U. P is the mother of Q. R is the sister-in-law of P. R is unmarried. T is the daughter-in-law of S. S is the eldest member of the family. U is the father of only R and V. U does not have any sibling. V does not have any son. R is older than V. The third eldest member of the family is 54-years old. The youngest member of the family is 4 years old. 19. If in the given arrangement, Q + V = 31, then what is the possible age of R? (1) 27 (2) 56 (3) 23 (4) 59 (5) 30 20. If P is 24 years old, what will be the sum of ages of P and T? (1) 28 (2) 78 (3) 64 (4) 49 (5) 75 21. How is T related to Q? (1) Grandmother (2) Granddaughter (3) Aunt (4) Mother-in-law (5) Mother

BLOOD RELATIONSHIP 22. If M is married to S, then how is S related to R? (1) Grandmother (2) Granddaughter (3) Grandfather (4) Uncle (5) Cannot be determined 23. Who amongst the following is/are younger than V? (1) Both P and U (2) Only Q (3) Only P (4) Both P and Q (5) No one Directions (24–27) : Study the following information carefully and answer the questions given below : M and B are the children of R. Y is married to B. G is the father of Y. B has only one son. K is the only nephew of M. X is the grandfather of K. V is the only daughter of X. A is the daughter of V. (RBI Officers in Grade ‘B’ Phase-I Exam. 16.08.2018) 24. How is R related to A? (1) Cannot be determined (2) Father-in-law (3) Grandmother (4) Mother-in-law (5) Aunt 25. How is Y related to V? (1) Cousin (2) Sister-in-law (3) Brother-in-law (4) Nephew (5) Aunt 26. If F is married to G, then how is F related to B? (1) Sister-in-law (2) Mother (3) Aunt (4) Grandmother (5) Mother-in-law 27. If J is married to V, then how is J related to K? (1) Cannot be determined (2) Father-in-law (3) Father (4) Uncle (5) Grandfather Directions (28–32) : Study the following information carefully and answer the questions given below : (RBI Officers Grade ‘B’ Phase-I Exam. 06.03.2021) There are seven members –A, B, C, D, E, F and G – in a family. There are only three female members and

three married couples. One couple has no child. Each member of the family spent different amounts in a month. F is the aunt of A and she spent the highest amount. The amount spent by E is just less than that spent by A’s uncle, who spent the fourth highest amount. C spent Rs. 4500, the second highest amount. B is the only son of G, who spent the third highest amount. A is not a female and he spent an amount just less than D’s husband. C is the father of two children of different genders and one of them is D. E’s mother -in-law spent the third highest amount. 28. How is E related to B ? (1) Son (2) Sister-in-law (3) Mother-in-law (4) Father (5) Brother-in-law 29. Who among the following did spend less amount than B ? (1) Only E and A (2) Only A and D (3) E, A and D (4) Only A (5) Only E and D 30. Four of the following five are alike in a certain way and hence they form a group. Which one does not belong to that group ? (1) B (2) D (3) E (4) C (5) F 31. How is G related to A ? (1) Paternal grandfather (2) Maternal grandfather (3) Maternal uncle (4) Paternal uncle (5) Father 32. How is A related to F ? (1) Nephew (2) Son (3) Brother-in-law (4) Uncle (5) Husband Directions (33–37) : In the following questions, some symbols are used with the certain meanings as illustrated below. (RBI Officers Grade ‘B’ Phase-I Exam. 06.03.2021) Study the following information carefully and answer the questions given below : ● X @ Y - X is the child of Y. ● X © Y - X is the parents of Y ● X & Y - X is the son-in-law of Y. ● X ! Y - X is the brother of Y.

BPRE–127

X = Y - X is the cousin of Y. X $ Y - X is father of Y. ● X % Y - X is the wife of Y. ● X > Y - X is the mother of Y. ● X ^ Y- X is the sister of Y. ● X + Y- X is the niece or nephew of Y. ● X * Y- X is the husband of Y. ● X ? Y - X is the daughter of Y. ● X # Y - X is the son of Y. (33–35) : G & Y * M > D ! H ? Y ● M has only one daughter. 33. How is G related to M’s only daughter ? (1) Brother-in-law (2) Father (3) Uncle (4) Husband (5) Brother 34. How is D related to Y ? (1) Son (2) Father (3) Brother (4) Uncle (5) Cousin 35. If T is the father of G, how H is related to T ? (1) Son-in-law (2) Daughter-in-law (3) Daughter (4) Cousin (5) Cannot be determined (36–37) : P ? T $ W + H > F 36. How is W related to H ? (1) Daughter (2) Sister-in-law (3) Nephew (4) Niece (5) Cannot be determined 37. How is T related to F ? (1) Father (2) Aunt (3) Uncle (4) Sister (5) Brother ● ●

INSURANCE EXAMS Directions (1–5) : Read the following information to answer the given questions : (United India Insurance AAO Exam.03.06.2012)

(i) A, B, C, D, E and F are six family members. (ii) There is one Doctor, one Lawyer, one Engineer, two Students and one Housewife. (iii) There are two married couples in the family. (iv) F, who is a Doctor, is father of E. (v) A is a Student and her husband is not a Lawyer.

BLOOD RELATIONSHIP (vi) B is grandmother of D and is a Housewife. (vii) C is father-in-law of A and is an Engineer. (viii) D is sister of E. 1. How is E related to B ? (1) Grand-daughter (2) Grand-son (3) Grand-daughter or Grandson (4) Son-in-law 2. Which of the following is one of the pairs of married couples ? (1) F A (2) F B (3) C F (4) F D 3. Who is Doctor ? (1) D (2) E (3) A’s husband (4) C’s wife 4. How many female members are there in the family ? (1) Two (2) Two or Three (3) Three (4) Three or Four 5. Which of the following statements is definitely true ? (1) F is the grandmother of D. (2) F is the father of the Lawyer. (3) B is the mother of the Student. (4) D is the sister of the Lawyer. Directions (6–10) : Read the following information to answer the given questions : (United India Insurance AAO Exam.03.06.2012)

(i) A, B, C, D, E and F are six family members. (ii) There is one Doctor, one Lawyer, one Engineer, two Students and one Housewife. (iii) There are two married couples in the family. (iv) F, who is a Doctor, is father of E. (v) A is a Student and her husband is not a Lawyer. (vi) B is grandmother of D and is a Housewife. (vii) C is father-in-law of A and is an Engineer. (viii) D is sister of E. 6. How is E related to B ? (1) Grand-daughter (2) Grand-son (3) Grand-daughter or Grandson (4) Son-in-law 7. Which of the following is one of the pairs of married couples ? (1) F A (2) F B (3) C F (4) F D

8. Who is Doctor ? (1) D (2) E (3) A’s husband (4) C’s wife 9. How many female members are there in the family ? (1) Two (2) Two or Three (3) Three (4) Three or Four 10. Which of the following statements is definitely true ? (1) F is the grandmother of D. (2) F is the father of the Lawyer. (3) B is the mother of the Student. (4) D is the sister of the Lawyer. Directions (11–14) : Following questions are based on the information given below : (United India Insurance AO Exam. 26.05.2013)

(1) (2) (3) (4) 11.

‘P × Q’ means ‘P is father of Q’. ‘P – Q’ means ‘P is sister of Q’. ‘P + Q’ means ‘P is mother of Q’. ‘P ÷ Q’ means ‘P is brother of Q’. In the expression B + D × M ÷ N, how is M related to B ? (1) Granddaughter (2) Son (3) Grandson (4) Granddaughter or Grandson (5) None of these 12. Which of the following represents ‘J is son of F’ ? (1) J ÷ R – T × F (3) J + R – T × F (3) J ÷ M – N × F (4) Cannot be determined (5) None of these 13. Which of the following represents ‘R is niece of M’ ? (1) M ÷ K × T – R (2) M – J + R – N (3) R – M × T ÷ W (4) Cannot be determined (5) None of these 14. H is the father of J. J is the brother of L. N is the wife of L. K is the son of N. L and J are children of H. P is the wife of H. K and Z are grandchildren of P. Who is the uncle of K ? (1) J (2) H (3) Z (4) Cannot be determined (5) None of these (LIC Assistant Administrative Officer (AAO) Exam. 12.05.2013)

BPRE–128

15.

Vinod introduces Vishal as the son of the only brother of his father’s wife. How is Vinod related to Vishal ? (1) Cousin (2) Brother (3) Son (4) Uncle (NICL (GIC) AO Exam, 08.09.2013)

16. A’s son B is married with C whose sister D is married to E the brother of B. How D is related to A? (1) Sister (2) Daughter-in-law (3) Sister-in-law (4) Cousin (NICL (GIC) AO (Finance) Exam, 15.12.2013)

17. ‘D’ is mother-in-law of ‘M’ who is sister-in-law of ‘A’. ‘P’is father of ‘Q’ the only brother of ‘A’. How is ‘D’ related to ‘A’? (1) Mother-in law (2) Aunt (3) Mother (4) Wife 18. A girl introduced a boy as the son of the daughter of the father of her uncle. The boy is girl’s (1) Uncle (2) Nephew (3) Brother (4) Son (NICL (GIC) AO Exam, 15.12.2013)

Directions (19–21) : Study the following information carefully and answer the questions given below : (NIACL Administrative Officer (AO) Exam, 10.01.2015)

A @ B means A is father of B; A + B means A is son of B; A $ B means A is daughter of B; A % B means A is mother of B; A & B means A is husband of B. 19. Which among the following options is true if the expression ‘P + Q % R $ S + T & W’ is definitely true ? (1) Q is the only daughter of T. (2) P is the grandson of S. (3) R is the granddaughter of W. (4) T is the father–in–law of P. (5) W is the grandmother of Q. 20. How is V related to P in the expression ‘P & S % T @ U + V’ ? (1) Grandmother (2) Daughter–in–law (3) Grandson (4) Wife (5) Daughter

BLOOD RELATIONSHIP 21. What should come in place of the question mark, to establish that P is the mother–in–law of T in the expression ? P%Q+R@S?T (1) @ (2) either & or % (3) $ (4) & (5) + Directions (22-24) : Study the following information carefully and answer the questions given below :

26. What will come in the place of the question mark, if it is provided that M is the grandmother of F in the expression : ‘F * R < S ? M’ ? (1) > (2) < (3) + (4) * (5) Cannot be determined Directions (27–28) : Study the following information carefully and answer the questions given below :

(NIACL Administrative Officer (AO) Online Exam, 11.01.2015)

(LIC Assistant Administrative Officer (AAO) Online Exam. 22.03.2015)

‘A @ B’ means ‘A is daughter of B’ ‘A + B’ means ‘A is husband of B’ ‘A $ B’ means ‘A is mother of B’ ‘A % B’ means ‘A is father of B’ ‘A & B’ means ‘A is son of B’ 22. How is Q related to T in the expression ‘P & Q % R + S $ T ? (1) Grandfather (2) Uncle (3) Father (4) Brother (5) Cousin 23. Which among the following options is true if the expression ‘P % Q + R $ S $ T & V’ is definitley true ? (1) S is grandmother of T. (2) V is daughter-in-law of R (3) V is husband of S. (4) P is Great Grandfather of S (5) R is Great Grandmother of T. 24. What should come in the place of the question mark (?) to establish ‘R is the husband of T’ in the expression ‘P + Q $ R % S ? T ? (1) @ (2) & (3) % (4) Either @ or & (5) $ Directions (25-26) : Read the following information carefully and answer the questions given below :

B is the father of W and T. R is the son of W. R is the only brother of C. M is the mother of C. M is the daughter-in-law of Z. Z has only one daughter and only one son. 27. Which of the following is definitely true based upon the given relationships? (1) None of the given options is definitely true. (2) W is the son-in-law of Z. (3) W has only one daughter. (4) T is the aunt of C. (5) T is married to M. 28. Which of the following is/are required to establish that C is the daughter of M? (1) No extra information is required as the relation can be established from the given information. (2) Z is the mother of T. (3) M has only one son. (4) R is the grandson of B. (5) T is unmarried Directions (29–30) : Study the following information carefully and answer the questions given below :

(NIACL Administrative Officer (AO) Online Exam, 12.01.2015)

‘A * B’ means A is the son of B. ‘A + B’ means A is the father of B. ‘A> B’ means A is the daughter of B. ‘A < B’ means A is the wife B. 25. Which of the following pairs of people represents first cousins with regard to the relations given in the expressions, if it is provided that A is the sister of J : ‘L > V < J + P’ and ‘S * A < D + F < E + K’. (1) LP (2) SP (3) SK (4) SF (5) Cannot be determined

(LIC Assistant Administrative Officer (AAO) Online Exam. 05.03.2016)

A is the mother of both U and T. T is the sister of J. J is the son of M. P is married to U. U is the daughter-inlaw of K. 29. How is P related to M ? (1) Son-in-law (2) Nephew (3) Son (4) Cannot be determined (5) Cousin 30. If R is the husband of K, then how is K related to P ? (1) Cannot be determined (2) Mother-in-law (3) Grandmother (4) Sister-in-law (5) Mother

BPRE–129

Directions (31–32) : Study the following information carefully and answer the questions given below : (LIC Assistant Administrative Officer (AAO) Online Exam. 06.03.2016)

L has only two daughters- U and P. P is married to K. K is the father of S. S is the son-in-law of Y. U is married to Q. 31. If Z is the daughter-in-law of P, then how is Z related to S? (1) Mother (2) Mother-in-law (3) Sister (4) Niece (5) Wife 32. How is Q related to L? (1) Brother-in-law (2) Brother (3) Son-in-law (4) Nephew

(5) Son

Directions (33–34) : Study the following information carefully and answer the questions given below : (UIICL A.O. Exam 12.06.2016)

33.

If P is the sister of K, then how is K related to S ? (1) Grandmother (2) Daughter (3) Grandson (4) Granddaughter

(5) Daughter-in-law How is T related to B ? (1) Father-in-law (2) Niece (3) Daughter (4) Son-in-law (5) Daughter-in-law 35. A is wife of B. D is brother of A. P and Q are the children of E, who is wife of D. How is B related to Q ? (1) Father (2) Grandmother (3) Uncle (4) Father-in-law (5) Cousin

34.

(NICL A.O. Exam 05.06.2017)

Directions (36-38) : Study the following information carefully and answer the questions given below : There are seven members, four men and three women in a family. They are A, B, C, D, E, F and G. B is an Athlete and father of E. F is the paternal grandfather of E and is a Boxer. C, who is a wrestler, is daughter-in-law of D. G is E’s uncle and is an Advocate. There is one Architect, one Journalist and one Pilot in the family. C is not E’s mother. D is not married to B. There are three married couples in the family. (NICL A.O. Exam 05.06.2017)

BLOOD RELATIONSHIP 36. If F is the grandfather of a Pilot, what is his daughter-in-law’s profession ? (1) Journalist (2) Boxer (3) Advocate (4) Cannot be determined (5) Wrestler 37. If D is the Pilot and the Journalist is not a female, then what is E’s profession ? (1) Architect (2) Journalist (3) Boxer (4) Wrestler (5) Athlete 38. If the Pilot does not have any child, then who is the Pilot ? (1) A (2) C (3) E (4) Either A or B (5) Either B or C Directions (39–40) : Study the following information carefully and answer the questions given below : (GIC Officer Online Exam 15.05.2017)

K is the mother of M. M is the brother of D. D is the daughter of R. S is the mother of N and R. N is the mother of Q. 39. How is Q related to R? (1) Either son or daughter (2) Grandson (3) Daughter-in-law (4) Son-in-law (5) Either nephew or niece 40. How is M related to S? (1) Grandson (2) Uncle (3) Nephew (4) Father (5) Son 41. P is the daughter of Q. P and S are the only two children of R. S is married to V. R has only two daughters. W is the mother of Q. T is married to P. W is married to X. R is the son of Y. How is S related to W? (1) Son-in-law (2) Daughter-in-law (3) Mother (4) Granddaughter (5) Son (LIC AAO Main Exam, 28.06.2019)

SHORT ANSWERS

NATIONALISED BANKS & IBPS PO/MT/SO EXAMS 1. (2) 5. (3) 9. (3) 13. (5)

2. (3) 6. (5) 10. (4) 14. (3)

3. (3) 7. (2) 11. (5) 15. (1)

4. (1) 8. (4) 12. (3) 16. (2)

17. (1) 21. (5) 25. (3) 29. (3) 33. (4) 37. (5) 41. (3) 45. (2) 49. (3) 53. (1) 57. (3) 61. (1) 65. (4) 69. (1) 73. (5) 77. (1) 81. (4) 85. (3) 89. (4) 93. (5) 97. (2) 101. (2) 105. (1) 109. (2) 113. (4) 117. (5) 121. (5) 125. (2) 129. (4) 133. (1) 137. (3) 141. (5) 145. (5) 149. (2) 153. (5)

18. (3) 22. (3) 26. (1) 30. (5) 34. (2) 38. (4) 42. (5) 46. (4) 50. (1) 54. (5) 58. (3) 62. (5) 66. (1) 70. (4) 74. (4) 78. (2) 82. (1) 86. (5) 90. (3) 94. (5) 98. (3) 102. (5) 106. (4) 110. (2) 114. (5) 118. (1) 122. (4) 126. (4) 130. (4) 134. (5) 138. (4) 142. (4) 146. (1) 150. (4)

19. (2) 23. (1) 27. (3) 31. (3) 35. (2) 39. (1) 43. (3) 47. (5) 51. (4) 55. (4) 59. (5) 63. (3) 67. (2) 71. (3) 75. (3) 79. (1) 83. (4) 87. (1) 91. (1) 95. (1) 99. (1) 103. (3) 107. (1) 111. (4) 115. (4) 119. (2) 123. (5) 127. (4) 131. (2) 135. (3) 139. (1) 143. (2) 147. (4) 151. (1)

20. (4) 24. (3) 28. (5) 32. (1) 36. (3) 40. (5) 44. (1) 48. (2) 52. (5) 56. (2) 60. (2) 64. (2) 68. (3) 72. (1) 76. (5) 80. (3) 84. (2) 88. (3) 92. (2) 96. (2) 100. (1) 104. (2) 108. (3) 112. (1) 116. (3) 120. (2) 124. (4) 128. (1) 132. (4) 136. (4) 140. (3) 144. (3) 148. (2) 152. (3)

SBI PO EXAMS 1. (4) 5. (4) 9. (4) 13. (2) 17. (5) 21. (2) 25. (5) 29. (2) 33. (1) 37. (4) 41. (1) 45. (2) 49. (4) 53. (5) 57. (4)

2. (3) 6. (3) 10. (1) 14. (5) 18. (1) 22. (4) 26. (2) 30. (1) 34. (2) 38. (2) 42. (4) 46. (2) 50. (4) 54. (1) 58. (3)

3. (2) 7. (5) 11. (4) 15. (4) 19. (4) 23. (1) 27. (3) 31. (4) 35. (2) 39. (4) 43. (5) 47. (5) 51. (5) 55. (2) 59. (1)

4. (5) 8. (1) 12. (3) 16. (3) 20. (1) 24. (1) 28. (4) 32. (2) 36. (3) 40. (3) 44. (1) 48. (4) 52. (4) 56. (5) 60. (2)

RBI GRADE–B/ NABARD GRADE–A OFFICER EXAMS BPRE–130

1. (4) 5. (1) 9. (5) 13. (4) 17. (2) 21. (1) 25. (2) 29. (4) 33. (2) 37. (3)

2. (2) 6. (3) 10. (1) 14. (2) 18. (1) 22. (3) 26. (5) 30. (3) 34. (3)

3. (2) 7. (1) 11. (3) 15. (1) 19. (5) 23. (4) 27. (4) 31. (4) 35. (4)

4. (5) 8. (3) 12. (5) 16. (5) 20. (2) 24. (3) 28. (3) 32. (4) 36. (1)

INSURANCE EXAMS 1. (2) 5. (2) 9. (3) 13. (2) 17. (3) 21. (4) 25. (2) 29. (1) 33. (3) 37. (2) 41. (4)

2. (1) 6. (2) 10. (2) 14. (1) 18. (3) 22. (1) 26. (4) 30. (5) 34. (5) 38. (3)

3. (3) 7. (1) 11. (3) 15. (1) 19. (3) 23. (3) 27. (4) 31. (5) 35. (3) 39. (5)

4. (3) 8. (3) 12. (5) 16. (2) 20. (2) 24. (4) 28. (3) 32. (3) 36. (4) 40. (1)

EXPLANATIONS

NATIONALISED BANKS & IBPS PO/MT/SO EXAMS 1. (2) N + O → N is daughter of O. O – P → O is father of P. P × Q → P is son of Q. Thus, Q is mother of N. 2. (3) S ÷ T → S is wife of T. T × V → T is son of V. V – W → V is father of W. W + Y → W is daughter of Y. T is son of Y. S is sister-in-law of W. S is daughter-in-law of Y. Y is wife of V. 3. (3) M + N ⇒ M is the husband of N. N – P ⇒ N is the sister of P. P × Q ⇒ P is the daughter of Q. Deductions N and P are daughter of Q. P is sister-in-law of M. M is son-in-law of Q. 4. (1) L ÷ M → L is daughter of M. M × O → M is father of O. O – P → O is son of P. P ÷ Q → P is daughter of Q. P is wife of M. P is mother of L and O. Therefore, L is granddaughters of Q.

BLOOD RELATIONSHIP 5. (3) Q – R → Q is son of R. R ÷ S → R is daughter of S. S × T → S is father of T. R is sister of T. Therefore, Q is nephew of T. 6. (5) A – B → A is son of B. B × C → B is father of C. C + D → C is wife of D. D – E → D is son of E. B is father of A and C. A is brother of C. A is brother-in-law of D. C is sister of A The sex of E is not known. 7. (2) D’s sister’s son means nephew of D. A’s father is also nephew of D. 8. (4) Q is son of B. R is the sister of Q. P is the mother Q and R. S is the husband of R. S is the brother-in-law of Q. 9. (3) M+N⇒M is the husband of N. N – P ⇒ N is the sister of P. P × Q ⇒ P is the daughter of Q. Deductions N and P are daughter of Q. P is sister-in-law of M. M is son-in-law of Q. (10–12) : B and L are couple. A is brother of B. Therefore, A is brother–in–law of L. L is mother of K and M. L is sister of Q. The gender of Q is not known. 10. (4) A is brother of B. B is husband of L. Therefore, A is brother–in–law of L. 11. (5) L is sister of Q. L is mother of M. The gender of Q is not known. Therefore, Q is maternal uncle or aunt of M. 12. (3) P is mother of G and M. M is brother of G. L is son of G. Therefore, L is nephew of M. 13. (5) All the statements are true. 14. (3) J is mother of K. K is mother of V. So, J is grandmother of V.

(15-16) : A is husband of H. A is brother-in-law of G. Q is son of A and H. T is husband of M. T is father of G and H. 15. (1) Q is son of H and H is mother of Q. G is brother of H. So, Q is nephew of G. 16. (2) T is father of H. H is wife of A. So, T is father-in-law of A. 17. (1) Option (1) A + M ⇒ A is wife of M. M % L ⇒ M is father of L. L + D ⇒ L is wife of D. L is daughter of A and M. L is wife of D. So, A is mother-in-law of D. Option (2) B $ A ⇒ B is brother of A. A @ D ⇒ A is mother of D. D % K ⇒ D is father of K. K & Q ⇒ K is husband of Q. A is mother of D. Option (3) A @ T ⇒ A is mother of T. T + N ⇒ T is wife of N. N % D ⇒ N is father of D. D $ L ⇒ D is brother of L. T is mother of D and L. So, A is grandmother of D. Option (4) A + H ⇒ A is wife of H. H % M ⇒ H is father of M. M $ K ⇒ M is brother of K. K % D ⇒ K is father of D. So, A is grandmother of D. 18. (3) K & R ⇒ K is husband of R. R @ T ⇒ R is mother of T. T $ D ⇒ T is brother of D. D & M ⇒ D is husband of M. M @ S ⇒ M is mother of S. S % G ⇒ S is father of G. So, K is father-in-law of M. 19. (2) Option (1) T & H ⇒ T is husband of H. H @ N ⇒ H is mother of N. N + L ⇒ N is wife of L. L % D ⇒ L is father of D. L is husband of N. So, H is mother-in-law of L. Option (2) K % H ⇒ K is father of H. H + R ⇒ H is wife of R.

BPRE–131

20.

21.

22.

23.

24.

R % L ⇒ R is father of L. L $ N ⇒ L is brother of N. H is mother of L and N. Option (3) H + J ⇒ H is wife of J. J % N ⇒ J is father of N. N & L ⇒ N is husband of L. L @ T ⇒ L is mother of T. So, H is mother-in-law of L. Option (4) L $ R ⇒ L is brother of R. R % H ⇒ R is father of H. H @ T ⇒ H is mother of T. T & V ⇒ T is husband of V. So, H is niece of L. (4) K % R ⇒ K is father of R. R $ T ⇒ R is brother of T. T & D ⇒ T is husband of D. D @ H ⇒ D is mother of H. H + S ⇒ H is wife of S. So, S is son-in-law of D and T. (5) S + D ⇒ S is wife of D. D $ P ⇒ D is brother of P. P % H ⇒ P is father of H. H & V ⇒ H is husband of V. V @ T ⇒ V is mother of T. P is grandfather of T. (3) M + K ⇒ M is son of K. K $ T ⇒ K is father of T. T @ N ⇒ T sister of N. So, M is brother of N. (1) Option (1) V % Q ⇒ V is wife of Q. Q $ R ⇒ Q is father of R. R @ D ⇒ R is sister of D. V is mother of D and R. Option (2) V % Q ⇒ V is wife of Q. Q @ R ⇒ Q is sister of R. R $ D ⇒ R is father of D. Q is a male, so he cannot be sister of R. Option (3) D + T ⇒ D is son of T. T @ J ⇒ T is sister of J. J $ V ⇒ J is father of V. So, V is cousin of D. Option (4) V @ F ⇒ V is sister of F. F $ D ⇒ F is father of D. D % M ⇒ D is wife of M. So, V is paternal aunt of D. (3) E + H ⇒ E is son of H. H @ K ⇒ H is sister of K. K $ B ⇒ K is father of B. Hence, B is cousin of E.

BLOOD RELATIONSHIP (25-26) : A is the husband of M. A is the father of P. N is the wife of Y. S is grandmother of Y and A. 25. (3) Y is the uncle of P. 26. (1) N is the wife of Y. 27. (3) A @ B ⇒ A is father of B. B % C ⇒ B is mother of C. C & D ⇒ C is sister of D. D + E ⇒ D is husband of E. D is the son of B. B is mother–in–law of E. 28. (5) A % B ⇒ A is mother of B. B + C ⇒ B is father of C. C & D ⇒ C is sister of D. If D is father or mother of E, then C would be aunt of E. D @ E ⇒ D is father of E. D % E ⇒ D is mother of E. 29. (3) A + B ⇒ A is husband of B. B & C ⇒ B is sister of C. C @ D ⇒ C is father of D. D % E ⇒ D is mother of E. E $ F ⇒ E is brother of F. D is daughter of C. B is sister of C. So, B is aunt of D. 30. (5) J is married to C. There is no information about the gender of either J or C. Therefore, J is either brother–in– law of P. 31. (3) S is the sister of D. T is the wife of D. Therefore, T is the sister–in–law of S. 32. (1) D is the father of Z. B is the brother of D. Therefore, B is the uncle of Z. (33–34) : U is the mother of L and D. R is the father of L and D. D is the husband of J. K is the son of D and J. F is the mother-in-law of D. 33. (4) F is the mother of J. J is the wife of D. Therefore, D is the son-in-law of F. 34. (2) D is the father of K. R is the father of D. Therefore, R is the grandfather of K.

(35–39) : M's sister S

M J

M's motherQ

M's wife M's R brother

M's son L M's father

K M's daughter

35. (2) M sits second to the right of R. 36. (3) L is an immediate neighbour of K. 37. (5) R sits to the immediate left of M’s wife. K is an immediate neighbour of L. M sits second to the left of Q. S is daughter of L. 38. (4) K is niece of R. 39. (1) J is the wife of M. (40–41) : J is the sister of K. L is the daughter of J and B T is the son of J and B. T is the husband of R. 40. (5) R is the wife of T. J is the mother of T. Therefore, J is the mother-in-law of R. 41. (3) K is the brother of J. T is the son of J. J is the mother of T. Therefore, T is nephew of K. (42–46) : N’s brother K N’s son N’s father Q O

45. (2) N’s mother P sits second to the right of N. M is sister of Q. N is an immediate neighbour of L (N’s sister) and M (N’s daughter). 46. (4) J is the wife of N. K is the brother of N. Therefore, J is the sister-in-law of K. (47–48) : D is the father of A and J. P is the wife of D. J is the wife of U. J is the sister of A. A is the son of D and P. J is the daughter–in–law of L. 47. (5) J is the wife of U. U is the son of L. Therefore, J is the daughter–in– law of L. 48. (2) A is the brother of J. J is the wife of U. Therefore, A is brother–in–law of U. (49-50) : U is the mother of D and S. L is the father of D and S. U is the wife of L. D is the son of L and U. S is the mother of M. D is the father of P. 49. (3) S is the daughter L and U. Therefore, S is the wife of R. Thus, R is the son-in-law of U. 50. (1) P is the daughter of D. S is the sister of D. Therefore, S is the aunt of P. (51–52) : J is the daughter of T.

L N’s sister

J N’s wife N

P N’s mother M N’s daughter

42. (5) J is the wife of N. Q is the son of J and N. 43. (3) Four persons - O, J, P and M - sit between K and N when counted from the left of K. 44. (1) N’s sister L sits to the immediate right of Q.

BPRE–132

T is the mother of D and J. J is the wife of M. Y is the daughter of J and M. 51. (4) J is the daughter of T. Q is the sister of T. Therefore, J is the niece of Q. 52. (5) Y is the daughter of J. J is the daughter of T. Therefore, Y is the granddaughter of T.

BLOOD RELATIONSHIP (53–54) :

N’s Husband D

N’s son B

N’s Mother O

C N’s Sister

P N’s Brother A N’s Father

N

M N’s Daughter

53. (1) When counted from the left of N, only one person sits between P and N.

63. (3) T is the daughter of J. T is the mother of S. Therefore, J is the grand father of S. 64. (2) T is the daughter – in – law of W. P is the brother of J. J is the father of T. P is the uncle of T. The sex of W is not known. (65–67) : C is the wife of M. M is the father of G and L. C is the mother of G and L. L is married to K. 65. (4) G is the brother of L.

54. (5) A is the father-in-law of D. C is an immediate neighbour of O and M.

66.

D sits to the immediate right of B. 55. (4) N’s husband D sits to the immediate left of P.

67.

56. (2) B is the son of N. 57. (3) B is the son of N. A is the father of N. Therefore, B is the grandson of A. (58-61) : J is the mother of T and Y. S is husband of R. D and K are sons of S. 58. (3) D is the brother of J. J is the mother of Y. Therefore, D is the maternal uncle of Y. 59. (5) Y is married to P. P is the husband of Y. S is the grandfather of Y. Therefore, S is the grandfatherin-law of P. 60. (2) D is a male. Therefore, C is the wife of D. D is the husband of C. 61. (1) K is brother of H, M and R. D is mother of H, M and R. R is daughter of D. 62. (5) Only daughter of Suresh’s grandfather means aunt or mother of Suresh. Therefore, Suresh is either brother or cousin or that woman. (63-64) : T is the daughter of J and L. V is the son of J and L. J is the husband of L.

68.

69.

70.

K is married to L. Therefore, G is the brother–in– law of K. (1) K is the mother of R. Then, K is the wife of L. Therefore, L is the father of R. (2) P is the wife of G. C is the mother of G. Therefore, C is the mother–in– law of P. (3) S × T → S is the son of T. T ÷ V → T is the wife of V. V × W → V is the son of W. W ÷ Y → W is the wife of Y. T is the wife of V and V is the son of Y. Therefore, T is the daughter in– law of Y. (1) P ÷ Q → P is the wife of Q. Q × R → Q is the son of R. R – T → R is the father of T. Q is the brother of T. Q is the husband of P. Therefore, P is the sister–in–law of T. (4) Option (1) K – P → K is the father of P. P × S → P is the son of S. S + D → S is the daughter of D. D ÷ N → D is the wife of N. S is the daughter of D. K is the husband of S. Therefore, K is the son–in–law of D. Option (2) K + P → K is the daughter of P. Thus, K cannot be a grandson.

BPRE–133

Option (3) S – P → P is the father of P. P + N → P is the daughter of N. N + D → N is the daughter of D. D ÷ K → D is the wife of K. Therefore, K is the husband of D. Option (4) K × P → K is the son of P. P ÷ S → P is the wife of S. S × D → S is the son of D. D – N → D is the father of N. S is the father of K. D is the father of S. Therefore, K is the grandson of D. 71. (3) H – R → H is the father of R. R × C → R is the son of C. C + T → C is the daughter of T. T ÷ W → T is the wife of W. W × M → W is the son of M. Therefore, C is the daughter of W. (72–74) : P is the father of L and T. A is the mother of L and T. A is the wife of P. B is the wife of L. W is the son of L and B. 72. (1) P is the father of L. B is the wife of L. Thereofore, P is the father-in-law of B. 73. (5) T is the sister of L. Therefore, L is the brother-in-law of Y. 74. (4) A is the Grandmother of W. (75–77) : N is the father of D and G. E is the mother of D and G. C is the husband of G. E is the daughter of K. C is the father of Q. 75. (3) Q is the daughter of C and G. D is the sister of G. Therefore, Q is the niece of D. 76. (5) E is the wife of N. K is the mother of E. Therefore, N is the son-in-law of K. 77. (1) N has two daughters — D and G. (78–80) : B is the daughter of A. C is the son of A. F is the wife of C. D and E are sons of C and F. H is the husband of A.

BLOOD RELATIONSHIP 78. (2) F is the wife of C. C is the son of A and H. Therefore, F is the daughter-inlaw of H. 79. (1) C is the father of E. 80. (3) G is the granddaughter of A and H. B and C are children of A and H. G is the daughter of B or C. F is the wife of C. Therefore, either B or F is the mother of G. (81–83) : N @ P ⇒ N is the sister of P. P % T ⇒ P is the brother of T. T # M ⇒ T is the mother of M. M $ K ⇒ M is the husband of K. K # J ⇒ K is the mother of J.

(87–89) : S is the sister of R. R is the brother of S. R is the husband of L. J is the father of B and Q. Q is the sister of B. 87. (1) J is the father of B. S is the daughter of B. Therefore, J is the grandfather of S. 88. (3) J is the father of B and Q. Q is the only daughter of J. Therefore, B is the son of J. R is the husband of L. R is the son of B. Therefore, B is the father–in–law of L.

J @ H ⇒ J is the sister of H. 81. (4) K is the mother of J and H. M is the father of J and H. The gender of H is not known. Therefore, H is either son or daughter of M. 82. (1) T is the mother of M. M is the husband of K. Therefore, K is the daughter-inlaw of T. 83. (4) P is the brother of T. T is the mother of M. Therefore, P is the maternal uncle of M. (84–86) : S is the husband of D. D is the sister of A. D is the daughter of B. Q is the daughter of B. R is the husband of Q. N is the daughter of Q and R. 84. (2) N is the daughter of Q and R. R is the son of M. M is the paternal grandfather of N. Therefore, V is the maternal grandfather of N. Q is the wife of R. Q is the daughter of B. B is the mother of Q. Therefore, B is the mother–in– law of R. 85. (3) S is the husband of D. D is the sister of Q. Therefore, S is the brother–in– law of Q. 86. (5) N is the daughter of Q. D is the sister of Q. Therefore, D is the aunt of N.

89. (4) R is the son of B. Q is the sister of B. Therefore, Q is the aunt of R. (90–92) : A is the son of G and R. K and M are daughters of G and R. S is the sister–in–law of R. 90. (3) K and M are sisters. P is the son of K. Therefore, M is the aunt of P. 91. (1) G is married to R. S is sister of G. Therefore, S is the sister–in–law of R. 92. (2) If S has no sister, then G is brother of S. G is married to R. R is the wife of G. G is the father of K and M. P is the son of K. Therefore, G is the grandfather of K. 93. (5) Ramesh is the son and Neha is the daughter-in-law of Mahesh. Anuradha and Bindu are daughters of Ramesh and Neha. Therefore, Bindu is the grand daughter of Mahesh. The family tree will be as follows : Mahesh Father-in-law

Only son

Ramesh

Spouse

Neha

Father Anuradha Sister

Bindu

BPRE–134

(94–96) : Q is the brother of P. K is the mother of P and Q. C is the wife of Q. L is the father of P and Q. P is the daughter of K and L. N is the daughter of P. 94. (5) C is the wife of Q. K is the mother of Q. Therefore, K is the mother-in-law of C. 95. (1) L is the father of P. P is the mother of N. Therefore, L is the grandfather of N. 96. (2) N has only one brother as sibling. Therefore, V is the brother of N. P is the mother of N and V. Q is the brother of P. Therefore, V is the nephew of Q. (97–99) : P is the wife of Q. E is the son of P and Q. L is the mother-in-law of Q and Y. M is the father of P and X. M is the husband of L. X is the wife of Y. B is the daughter of X and Y. A is the son of X and Y. 97. (2) M is the father of P. P is the mother of D. Therefore, M is the maternal grandfather of D. 98. (3) A is the son of X and Y. E is the son of P and Q. P and X are the sisters. Therefore, A is the Cousin of E. 99. (1) Y is the husband of X. L is the mother of X. Therefore, Y is the son-in-law of L. (100–102) : D is the father of only F and E. E is married to G and G is the son-in-law of M. Therefore, E is the wife of G. E is the daughter of D and M. E is the sister of F. F is the son of D and M. D is the husband of M. M is the mother of E and F. H is the only son of E and G. K and J are daughters of E and G. L is married to K. L is the husband of K.

BLOOD RELATIONSHIP 100. (1) J is the daughter of E and G. M is the mother of E. E is the mother of J. Therefore, M is the grandmother (maternal) of J. 101. (2) H is the brother of J and K. K is the wife of L. Therefore, H is the brother-inlaw of L. 102. (5) F is the brother of E. E is the mother of K. Therefore, F is the maternal uncle of K. (103–106) : Z is the husband Y. Z is father-in-law of E. Y is mother of F. Y is mother-in-law of E. T is mother-in-law of F. P is the daughter of S and T. E is the husband of F. S is father-in-law of F. J is the son of Z and Y J is the brother of F. 103. (3) J is the son of Z and Y. 104. (2) P is the daughter of S and T. T is the daughter of X. Therefore, P is the grand daughter of X. 105. (1) J is the son of Z and Y. J is the grandson of M. The gender of M is not given. Again, it is not clear whether J is paternal or maternal grandson of M. Therefore, it cannot be deter mined, how is M related to Y. 106. (4) S is father-in-law of F. (107–109) : B is the husband of K. B is the brother of P. J is the father of B and P. G is the mother of B and P. P is the daughter of J and G. 107. (1) G is the mother of B. S is the son of B. Therefore, S is grandson of G. 108. (3) P is the daughter of J. 109. (2) B is the husband of K. K is the daughter of H and V. Therefore, X is the son of H. (110–111) : R is the sister of Q. M is the father of Q and R. V is the son of Q. V is the grandson (maternal) of C. M does not have married daughter. Therefore, Q is the son of M.

110. (2) R is the sister of Q. Q is the father of V. Therefore, R is the aunt of V. 111. (4) If B is married to Q, B would be wife of Q. M is the father of Q. Therefore, B is the daughter-inlaw of M. 112. (1) D & B D is the daughter-in-law of B. B is the wife of S. B%S SG S is the father of G. G%E G is the wife of E. E is the father of Q. E Q Q$D Q is the brother of D. G is the mother of D and Q. S is the father of G. Therefore, S is grandfather of D. G is the daugh113. (4) G & B ter-in-law of B. B is the husband of B*S S. S is the daughter of S@W W. W©F W is the father of F. F%O F is the wife of O. O is the father of Z. O©Z F is the sister of S and mother of Z. The gender of Z is not known. Therefore, Z is either nephew or niece of S. (114–116) : H is husband of K. M is mother-in-law of K. P is mother of K and B. 114. (5) M is mother of H. If Y is father of H, Y is husband of M. 115. (4) K is mother of F. P is mother of K. Therefore, P is grandmother (maternal) of F. 116. (3) B is sibling of K. K is mother of F. H is father of F. The gender of B is not known. (117–119) : R is the father-in-law of D. R is the father of B and P. Q is the mother of R and G. K is the only daughter-in-law of Q.

BPRE–135

Therefore, K is the wife of R. K is the mother of B, M and P. M is the only daughter of K. B and P are sons of K and R. D is the wife of B. 117. (5) K is the wife of R. Q is the mother of R and G. K has no brother-in-law. Therefore, G is sister-in-law of K. R is father of B, M and P. G is the sister of R. Therefore, G is the aunt of P. 118. (1) M is the daughter of R. R is the son of Q. Q is the mother of R. Therefore, Q is the grandmother of M. 119. (2) B is the son of R. G is the sister of R. Therefore, B is the nephew of G. 120. (2) All are correct 121. (5) All are correct (122–124) : J is the father of S. S is the only child of J. M is mother of D and K. H and L are children of K. S is married to K. 122. (4) L is daughter of K and S. D is sister of K. Therefore, L is niece of D. 123. (5) The gender of K is not known. If T is paternal grandfather of L, K is son of T. If T is maternal grandfather of L, K is daughter of T. 124. (4) S is the only child of J. S is married to K. J is the father of S. H is son of K and S. Therefore, J is grandfather of H. (125-126) : M is the father of Q and R. R is unmarried daughter of M. V is son of Q. Q is father of V. C is father-in-law of Q. 125. (2) R is aunt of V. 126. (4) Q is son of M. B is wife of Q. Therefore, B is daughter-in-law of M.

BLOOD RELATIONSHIP 135. (3) A is grandchild of G. S is son of G. It is given that only one of the children of G and Q is married. M is daughter of G and Q. M is wife of R. Again, F is the only daughter of M and R. Therefore, A is the son of M and R and nephew of S. 136. (4) F is daughter of M and R. X is the only aunt of F. Therefore, X is the sister-in-law of M. 137. (3) A has three children. F is the son of A and C. B is only daughter of A and C. E is sister-in-law of B and F. Therefore, D is son of A and C. D is the husband of E. Thus, D is brother of B. 138. (4) Roma is wife of Rakshit. Reena is mother of Raksha and Rakshit. Therefore, Roma is sister-in-law of Raksha.

(139–143) :

140. (3) W is the sister of X is the mother of L. Therefore, W is the aunt of L. 141. (5) X is the daughter of E. E the is mother of X. Q is the brother of E.

Therefore, X is niece of Q. 142. (4) K is father of X. X is mother of L. Therefore, K is grandfather of L. 143. (2) Y is son of E and K.

Brother

C

Wife-Husband

K

Da

ug

n So

h te

r

Q

Husband-Wife

Daughter

J

X

Y

W

L

(i) W ? E : W is daughter of E. E = J : E is the mother-in-law of J. J $ L : J is the father of L. (ii) L + Y : L is the niece or nephew of Y. Y + Q : Y is the niece or nephew of Q. Q ! E : Q is brother of E. (iii) X ^ Y : X is sister of Y. Y # K : Y is the son of K. K © W : K is the parent of W. 139. (1) J is husband of X. Y is brother of X. Therefore, J is brotherin-law of Y.

(144–148) :

J

144. (3) J, U and Q belong to the third generation. 145. (5) W is the mother of T and D. 146. (1) Q is the son of D and P. P is the son of M. If M is the wife of R, R would be father of P and paternal grandfather of Q.

BPRE–136

Wife–Husband

P

Son

u Da U

M

Z

Son

te r ug h Da D

ug h T

gh

te r

Husband–Wife

S on

Y

Wife–Husband

te r

W

Da

B Son

(127–129) : N is sister of P. N@P P%T P is brother of T. T is mother of M. T#M M$K M is husband of K. K is mother of J. K#J J is sister of H. J@H 127. (4) K is mother of J and H. M is the father of J and H. The gender of H is not known. Therefore, H is either son or daughter of M. 128. (1) T is mother of M. K is wife of M. Therefore, K is daughter-in-law of T. 129. (4) P is brother of T. T is mother of M. Therefore, P is maternal uncle of M. 130. (4) Father of Arjits’s maternal uncle means maternal grandfather of Arjit. Grandson of Arjit’s maternal grandfather means cousin (maternal brother) of Arjit. Sister of cousion (maternal brother) of Arjit means cousin (maternal sister) of Arjit. (131–132) : ● P is wife of V. R is daughter of P and V. U is son of P and V. ● T is daughter of Q and niece of U. ● W is wife of U. ● S is brother of V. 131. (2) V is father of U. U is the husband of W. Therefore, V is the father-in-law of W. 132. (4) Male members are : V, U, Q and S. 133. (1) A is wife of B. B is son-in-law of R. G is son of J. J is daughter-in-law of R. Therefore, G is nephew of B. (134 – 136) : l Q is the husband G. l B and S are sons of Q and G. l M is the only daughter of Q and G. l R is the husband of M. l F is the only daughter of R and M. l X is the only aunt of F. 134. (5) Q is father of M. M is wife of R. Therefore, Q is the father-in-law of R.

Q 147. (4) D and T are sisters. Y is the husband of T. Therefore, D is the sister-in-law of Y. 148. (2) U is daughter of Y and T. T is the daughter of W and Z. Therefore, U is granddaughter of Z.

BLOOD RELATIONSHIP (149–153) :

(8–10) :

Wife–Husband

th er

n

n so X

3. (2) A % B ⇒ A is mother of B. B @ C ⇒ B is sister of C. C + D ⇒ C is brother of D. D $ E ⇒ D is husband of E. E % F ⇒ E is mother of F. D is son of A. C is brother of D, the father of F. A is mother of B, C and D. A has one daughter and two sons. 4. (5) B % C ⇒ B is mother of C. C @ D ⇒ C is sister of D. D + E ⇒ D is brother of E. E & F ⇒ E is father of F. E is brother of C and D. C is sister of E. D is son of B. (5–7) :

B

hushand

N

brother wife Q

child

5. (4) A is grandfather of Q. Q is granddaughter of A. B is the brother of N. N is the father of Q. Therefore, Q is niece of B. 6. (3) A is Husband of C. 7. (5) B is grandson of J.

BPRE–137

father-inlaw M M

son

F

Q M

an sb hu

d

8. (1) X is the mother of Q. J is the brother of X. Therefore, J is the maternal uncle of Q. 9. (4) P is the daughter of M. Y is the son of P. Therefore, Y is the grandson of M. 10. (1) K is the child of P. P is the mother of K. Q is the brother of P. The gender of K is not known. Therefore, K is either nephew or niece of Q. 11. (4) Option (1) A + M ⇒ A is brother of M. M % R ⇒ M is mother of R. R $ D ⇒ R is husband of D. D @ V ⇒ D is sister of V. Therefore, A is brother of mother - in- law of D. Option (2) B & D ⇒ B is father of D. D $ H ⇒ D is husband of H. H + A ⇒ H is brother of A. A % T ⇒ A is mother of T. D is a male.

mother M

S

M

er

C

SBI PO EXAMS

Father

son

husband

ug ht

Wife–Husband

A is the sister of H. Therefore, E is the niece of A. 152. (3) G is the mother of A. 153. (5) J is the son of F and A. E is the daughter of H and B. A is the sister of H. Therefore, J is the cousin brother of E.

A

P

F

149. (2) Female members in the family : G, A, B and E. 150. (4) B is the wife of H. J is the son of F and A. A is the sister of H. Therefore, J is the nephew of H. 151. (1) E is the daughter of H and B.

J

Y M

B

da

E

M/F brother

M A

Husband–Wife

H

J

1. (4) Option (1) A + M ⇒ A is brother of M. M % R ⇒ M is mother of R. R $ D ⇒ R is husband of D. D @ V ⇒ D is sister of V. Therefore, A is brother of mother - in- law of D. Option (2) B & D ⇒ B is father of D. D $ H ⇒ D is husband of H. H + A ⇒ H is brother of A. A % T ⇒ A is mother of T. D is a male. Option (3) A $ F ⇒ A is husband of F. F % W ⇒ F is mother of W. W @ D ⇒ W is sister of D. D % M ⇒ D is mothe of M. A is father of D. Option (4) A + K ⇒ A is brother of K. K $ D ⇒ K is father of D. D @ L ⇒ D is sister of L. L $ Q ⇒ L is husband of Q. So, D is niece of A. 2. (3) A + B ⇒ A is brother of B. B @ C ⇒ B is sister of C. C & D ⇒ C is father of D. D $ E ⇒ D is husband of E. So, C is father-in-law of E.

K

br o

So

A

Daughter

Husband–Wife Son

F

D

r Da

ug hte

G

Option (3) A $ F ⇒ A is husband of F. F % W ⇒ F is mother of W. W @ D ⇒ W is sister of D.

C

D % M ⇒ D is mothe of M. A is father of D. Option (4) A + K ⇒ A is brother of K. K $ D ⇒ K is father of D. D @ L ⇒ D is sister of L. L $ Q ⇒ L is husband of Q. So, D is niece of A.

BLOOD RELATIONSHIP 12. (3) A + B ⇒ A is brother of B. B @ C ⇒ B is sister of C. C & D ⇒ C is father of D. D $ E ⇒ D is husband of E. So, C is father-in-law of E. 13. (2) A % B ⇒ A is mother of B. B @ C ⇒ B is sister of C. C + D ⇒ C is brother of D. D $ E ⇒ D is husband of E. E % F ⇒ E is mother of F. D is son of A. C is brother of D, the father of F. A is mother of B, C and D. A has one daughter and two sons. 14. (5) B % C ⇒ B is mother of C. C @ D ⇒ C is sister of D. D + E ⇒ D is brother of E. E & F ⇒ E is father of F. E is brother of C and D. C is sister of E. D is son of B. (15–17) : A is the father of B and N. B is the son of A and M. C is the wife of N. J is grandfather of B and N. J is father–in–law of M. A is father–in–law of C. M is mother–in–law of C. A is grandfather of Q. M is grandmother of Q. 15. (4) A is grandfather of Q. Q is granddaughter of A. B is the brother of N. N is the father of Q. Therefore, Q is niece of B. 16. (3) A is father–in–law of C. 17. (5) B is grandson of J. (18-20) : P is the sister of Q. Q is the brother of P. P is the mother of Y, A and K. P is the wife of S. M is the father of P and Q. X is the mother of P and Q. 18. (1) X is the mother of Q. J is the brother of X. Therefore, J is the maternal uncle of Q. 19. (4) P is the daughter of M. Y is the son of P. Therefore, Y is the grandson of M.

20. (1) K is the child of P. P is the mother of K. Q is the brother of P. The gender of K is not known. Therefore, K is either nephew or niece of Q. (21–23) : S is the daughter of U and V. Y is also daughter of U and V. Y is the wife of D. P is the husband of S. S is the sister–in–law of B. V is the husband of U. L is the husband of J. L is the father of U. 21. (2) S is the sister–in–law of B. 22. (4) L is the father of U. 23. (1) P is the husband of S. V is the father of S. Therefore, V is the father–in–law of P. (24–26) : T is wife of D. T is mother of R. J is wife of B. H is wife of R. L is the daughter of B and J. 24. (1) J is the mother of L. T is the sister J. Therefore, L is the niece of T. 25. (5) M is the son of H and R. R is the son of D. Therefore, M is the grandson of D. 26. (2) J is the sister of T. T is mother of R. Therefore, J is the anut of R. (27 – 29) : D is sister of T. D is wife of P. Y is husband of T. J is the son of T and Y. U is the daughter of T and Y. 27. (3) P is the husband of D. T is the sister of D. Therefore, P is the brother-in-law of T. 28. (4) J is the son of T and Y. D is the sister of T. Therefore, J is the nephew of D. 29. (2) Q is the son of D and P. P is the son of M. Therefore, Q is the grandson of M. 30. (1) Father of girl’s uncle means grandfather of the girl. Daughter of girl’s grandfather means aunt of the girl. The boy is the son of girl’s aunt. Therefore, the boy is cousin brother of that girl.

BPRE–138

(31-33) : M is the husband of P. P is the sister-in-law of Q. R is the son of M and P. S is the mother-in-law of M. T is the husband of S. P is the only daughter of S and T. 31. (4) P is the only daughter of S and T. Therefore, J is the brother of P. M is the husband of P. Thus, J is the brother–in–law of M. 32. (2) M is the son–in–law of S. 33. (1) R is the only son of M and P. Q is the sister of M. M is the father of R. Therefore, R is nephew of Q. 34. (2) Y is the mother of K, N and T. K is the daughter of Y. K is the mother of J and M. D is the father of J and M. X is the wife of M. Therefore, D is the father–in–law of X. (35–37) : S and T are daughters of D. T is the wife of J. T is the mother of R and U. U is the daughter of J and T. U is the wife of K. 35. (2) T is the daughter of D. J is the Husband of T. Therefore, J is the son–in–law of D. 36. (3) R is the brother of U. U is the wife of K. Therefore, K is the uncle of X. 37. (4) S is the sister of T. R is the son of T. Therefore, S is the aunt of R. (38–45) : U is the wife of T. Z is the son of Q and R. Q is the mother of Z. 38. (2) Z is the father of T. T is the husband of U. T is the only son of Z. Therefore, X is the daughter of Z. Thus, X is the sister–in–law of U. 39. (4) U is the daughter of D. L is the sister of D. Therefore, U is the niece of L. 40. (3) T is the son of Z. Z is the son of Q and R. Q is the mother of Z. Therefore, Q is the grandmother of T.

BLOOD RELATIONSHIP ● ●

Male members : Z, R, P and W Female members : V, U, Q, S, T and X. Husband–Wife

(Male) Z

V (Female) So n

n So

53. (5) U is the wife of P. R is the niece of P. Therefore, U is aunt of R. 54. (1) If J has only one son, L is daughter of J. T is husband of L. 55. (2) J is father-in-law of T. (56–60) : Wife–Husband

er ht

W

U

ug

Son

T

Daughter X

S is the unmarried sister of Q. 48. (4) Male members = 4 Female members = 6 Required ratio = 4 : 6 = 2 : 3 49. (4) Q is the mother of T. S is the sister of Q. Therefore, S is the aunt of T. 50. (4) S is mother of P, Q and R. T is father of P, Q and R. S is wife of T. P is son of S and T. The gender of Q is not known. (51–52) : M is the father of Q. J is the only sister-in-law of M. J is the mother of W. R is the mother of M. 51. (5) Q is the daughter of M. If K is the only uncle of Q, then K is the father of W. 52. (4) Q is the granddaughter of R. R is mother of M and that of father or W. R has no any grandson. Therefore, W is granddaughter of R. Q is the cousin of W and mother of G. Thus, W is the aunt of G. J is the wife of M’s brother. Q is the daughter of M. Therefore, J is the aunt of Q. (53–55) : ● J is the father of only L and P. ● R is the niece of P. ● R is the daughter of L. ● M is the grandfather of R. ● Therefore, M is father-in-law of L. ● T and L are couple. ● U is the wife of P.

BPRE–139

ter

Da ug h

Wife

D

n

Wife Q

G

P

So

R

Da

(41–43) : S is the mother of D and K. D is the daughter of M and S. M is the husband of S. U is the father of M and Y. 41. (1) D is the daughter of M and S. Therefore, X would be the son– in–law of M. 42. (4) T is the mother of M and Y. K is the only son of M. Therefore, T is the grandmother of K. 43. (5) S is the wife of M. M is the brother of Y. Therefore, S is the sister–in–law of Y. 44.(1) V is the brother of M and N. P is the mother of M, N and V. R is the father of M, N and V. P is the wife of R. R is the husband of P. M is either son or daughter of R. 45. (2) R is the wife of V. M is the daughter of R and V. S is the only son of R and V. R is the grandmother of A. Therefore, S is either father or maternal uncle of A. 46. (2) C is the mother-in-law of Z. C has only one daughter. Therefore, Z is the son-in-law of C. Z is the son of P. P is the grandfather of S. S is the niece of T. Therefore, S is the daughter of Z. P has only one child Z. Therefore, T is the son of C. Thus, T is the brother-in-law of Z. 47. (5) J is the son of A and R. J is father of S. A is mother of J and Q. R is the husband of A. Therefore, R is the grandfather of S. (48–49) : ● Z is the father-in-law of Q and Z has no girl child. P is the son of V. Thus, V is the mother of P and R. Z is the father of P and R. ● U is the mother of X and is married to P. ● S is the sister-in-law of R. Q has only one unmarried sister. Therefore, Q is the wife of R. ● W is the brother of T. T is the youngest in the family. W is the son of Q and R. T is the daughter of Q and R. ● Z has only one grandson. Therefore X is daughter of P and U.

Husband–Wife

Husband–Wife

A

F Son I

H

B Daughter Wife–Husband E

C

56. (5) B is daughter–in–law of D and G. 57. (4) I is cousin brother of E. 58. (3) G is mother of A. 59. (1) E is niece of A. 60. (2) I is son of A. H is husband of B. A is sister of H. I is nephew of H.

RBI GRADE–B/ NABARD GRADE–A OFFICER EXAMS 1. (4) B is the son of D 2. (2) H is mother of A, E and G. H sits second to left of G. 3. (2) One person sits between A and his brother E. 4. (5) A sits exactly between H and F. 5. (1) E is the brother of A. 6. (3) A is grandson of D. 7. (1) Except B, all others are females. 8. (3) H is a female. H is sister-in-law of C. H is daughter of D. H is mother of A. 9. (5) T is the mother of P. R is grandfather of P. Therefore, T is either daughter or daughter-in-law of R. 10. (1) P @ R → P is the mother of R. R = S → S is the sister of R. S © T → T is the brother of S. T £ V → T is the son of V. V is the husband of P. T is the brother of R and S. R is child of P and V. The sex of R is not known.

BLOOD RELATIONSHIP 11. (3) Option (1) C £ V → C is the son of V. V # N → V is the daughter of N. N @ L → N is the mother of L. L © D → D is the brother of L. N is the mother of D, L and V. D is the brother of V. So, C is the nephew of D. D is maternal uncle of C. Option (2) C £ V → C is the son of V. V £ L → V is the son of L. L @ N → L is the mother of N. N © D → D is the brother of N. L is the mother of D, N and V. D is paternal uncle of C. C is the nephew of D. Option (3) D £ L → D is the son of L. L £ N → L is the son of N. N @ V → N is the mother of V. V © C → C is the brother of V. N is grandmother of D. N is mother of C, L and V. C is the paternal uncle of D. Option (4) D £ N → D is the son of N. N # V → N is the daughter of V. V @ L → V is the mother of L. L © C → C is the brother of L. V is the mother of C, L and N. N is the sister of C and L. So, C is the maternal uncle of D. 12. (5)L = M → M is the sister of L. M # N → M is daughter of N. N © P → P is the brother of N. P ★ Q → P is the father of Q. P is the uncle of L and M. Q is the cousin of L and M. L is the child of N. N is either uncle or aunt of Q. Q is either niece or nephew of N. 13. (4) Q £ N → Q is the son of N. N @ S → N is the mother of S. S © M → M is the brother S. M = P → P is the sister of M. M and Q are two sons of N. P is the daughter of N. The sex of S in not known. P is the sister of M, Q and S. 14. (2) Option (1) Q ★ P → Q is the father P. P # C → P is the daughter C. C @ N → C is the mother of N. N @ V → N is the mother of V. Q is a male.

Option (2) W is the mother of M, N and P. B is the husband of P. T is the father of W. B is the son–in–law of W and Y. Therefore, B is the grandson–in– law of T. 18. (1) P is the daughter of T and V. V is the mother of M and P. M is the husband of N. V is the grandmother of B. (19–23) : • U>T>V • P is the mother of Q. Therefore, P is a female. • R is the unmarried sister-in-law of P. Therefore, R is the sister of husband of P. • U is the father of only R and V. Therefore, V is the father of Q. P is the wife of V. • R is older than V. • V does not have any son. It means Q is the daughter of P and V. S>U>T>R>V>P>Q





54 years 4 years 19. (5) Q + V = 31 ⇒ V = 31 – 4 = 27 years The age of R is more than 27 years but less than 54 years. 20. (2) P + T = 24 + 54 = 78 years 21. (1) T is the daughter -in-law of S. Therefore, T is the wife of U. U is the grandfather of Q. Thus, T is the grandmother of Q.

22. (3) S is the eldest member of the family. Therefore, M is the wife of S. R is the daughter of U. U is the son of M and S. Therefore, S is the grandfather of R. 23. (4) P and Q are younger than V. (24–27) : V is the only daughter of X. X is the grandfather of K. K is the son of B and Y. Thus, M and B are sons of X. R is the mother of B, M and V. X is the father of B, M and V. B is the husband Y. G is the father–in–law of B. A is granddaughter of X and R. 24. (3) A is the daughter of V. V is the daughter of R and X. R is the mother of V. Therefore, R is grandmother of A. 25. (2) Y is the wife of B. V is the sister of B. Therefore, Y is sister–in–law of V. 26. (5) If F is married to G, F would be mother of Y. Y is the wife of B. Therefore, F would be mother– in–law of B. 27. (4) K is the son of B and Y. V is the sister of B. If J is married to V, then J would be uncle of K.

(28–32) :

In the term of amount spent : F>C>G>B>E>A>D

Rs. 4500

BPRE–140

BLOOD RELATIONSHIP 28. (5) E is the husband of D. B is the brother of D. Therefore, E is the brother-in-law of B. 29. (3) E, A and D spent amount less than that of B. 30. (4) Except C, the other four belong to the second generation. 31. (2) A is the son of D and E. D is the daughter of G and C. G is the father of D. Therefore, G is maternal grandfather of A. 32. (1) A is the son of D and E. D is the sister of B. F is the wife of B. So, A is the nephew of F. (33–37) : ● G & Y : G is son-in-law of Y. ● Y * M : Y is the husband of M. ● M > D : M is the mother of D. ● D ! H : D is the brother of H. ● H ? Y : H is the daughter of Y. Y

M

H

G

D

33. (4) M’s only daughter is H. G is husband of H. 34. (1) D is the son of Y and M. 35. (2) H is daughter-in-law of T. (36–37) : ● P ? T : P is the daughter of T. ● T $ W : T is the father of W. ● W + H : W is the niece or nephew of H. ● H > F : H is the mother of F. T is the father of P and W. 36. (5) The gender of W is not known. 37. (3) T is the uncle of F.

INSURANCE EXAMS (1–5) : Member Sex Profession Relationship A Female Student Wife of F B Female Housewife Grandmother of D and E; Wife of C C Male Engineer Father-in-law of A; Husband of B D Female Lawyer Sister of E; Daughter of A and F E Male Student Brother of D F Male Doctor Father of E and D; Husband of A

1. (2) E is grandson of B. 2. (1) Married Couples : BC and FA. 3. (3) F, husband of A, is a doctor. 4. (3) A, B and D are female members. 5. (2) F is father of D and D is a lawyer. B is Grandmother of E. (6–10) : Member Sex Profession Relationship A Female Student Wife of F B Female Housewife Grandmother of D and E; Wife of C C Male Engineer Father -in-law of A; Husband of B D Female Lawyer Sister of E; Daughter of A and F E Male Student Brother of D F Male Doctor Father of E and D; Husband of A

6. (2) E is grandson of B. 7. (1) Married Couples : BC and FA. 8. (3) F, husband of A, is a doctor. 9. (3) A, B and D are female members. 10. (2) F is father of D and D is a lawyer. B is Grandmother of E. 11. (3) B + D means B is mother of D. D × M means D is father of M. M ÷ N means M is bother of N. Therefore, M is grandson of B. 12. (5) Option (1) J ÷ R means J is brother of R R – T means R is sister of T. T × F means T is father of F. Therefore, J is uncle of F. Option (2) J + R means J is mother of R. J is a female. Option (3) J ÷ M means J is brother of M. M – N means M is sister of N. N × F means N is father of F. Therefore, J is uncle of F. 13. (2) Option (1) M ÷ K means M is brother of K. K × T means K is father of T. T – R means T is sister of R. The sex of R is not clear. R is either nephew or niece of M. Option (2) M – J means M is sister of J. J + R means J is mother of R. R – N means R is sister of N. Therefore, R is niece of M 14. (1) P is the mother of J and L. K is the son of L and N. J is the brother of L. Therefore, J is the uncle of K.

BPRE–141

15. (1) Only brother of Vinod’s father’s wife means the maternal uncle of Vinod. Vishal is the son of Vinod’s maternal uncle. So, Vinod is cousin of Vishal. 16. (2) D is the wife of B. So, D is the daughter-in-law of A. 17. (3) M is sister-in-law of A. P is the only brother of A. So, M is the wife of P. Therefore, D is mother of A and P. 18. (3) The father of girl’s uncle means the Grandfather of the girl. Daughter of the girl’s grandfather is the aunt of the girl. Thus, the boy is the brother of that girl. 19. (3) P + Q ⇒ P is the son of Q. Q % R ⇒ Q is the mother of R. R $ S ⇒ R is the daughter of S. S + T ⇒ S is the son of T. T & W ⇒ T is the husband of W. Q is the mother of P and R. S is the husband of Q. S is the father of P and R. T is the father–in–law of Q. W is the mother of S. P is the mother–in–law of Q. R is the grandfather of T and W. 20. (2) P & S ⇒ P is the husband of S. S % T ⇒ S is the mother of T. T @ U ⇒ T is the father of U. U + V ⇒ U is the son of V. V is the wife of T. T is the son of P. Therefore, V is the daughter–in– law of P. 21. (4) P % Q ⇒ P is the mother of Q. Q + R ⇒ Q is the son of R. R @ S ⇒ R is the father of S. S & T ⇒ S is the husband of T. R is the father Q and S. P is the mother of Q and S. Therefore, P is the mother–in–law of T. 22. (1) P & Q ⇒ P is son of Q. Q % R ⇒ Q is father of R. R + S ⇒R is husband of S. S $ T ⇒ S is mother of T. R is father of T. So, Q is grandfather of T. 23. (3) P % Q ⇒ P is father of Q. Q + R ⇒ Q is husband of R. R $ S ⇒ R is mother of S. S $ T ⇒ S is mother of T. T & V ⇒ T is son of V. S is mother of T. V is son-in-law of R. P is Grandfather of S. R is Grandmother of T.

BLOOD RELATIONSHIP 24. (4) Father Daughter

Husband P

+

Q

$

R

%

Mother

S

&

T

Son

25. (2) L > V ⇒ L is daughter of V. V < J ⇒ V is wife of J. J + P ⇒ J is father of P. V is mother of L and P. S * A ⇒ S is son of A. A < D ⇒ A is wife of D. D + F ⇒ D is father of F. F < E ⇒ F is wife of E. E + K ⇒ E is father of K. D is father of S and F. F is daughter of A and D. F is mother of K. A is sister of J. J is father of L and P. A is mother of S and F. So, S and P are first cousins. 26. (4) F * R ⇒ F is son of R. R < S ⇒ R is wife of S. S * M ⇒ S is son of M. Therefore, M may be grandmother of F. (27–28) : M is the mother of C and R. W is the husband of M. Z is the mother of W and T. Z is the wife of B. W is the son of B and Z. T is the daughter of B and Z. 27. (4) W is the son of the Z. R is the son of W. R is the only brother of C. So, C may be brother or sister of R. T is the sister of W. C is the child of W. Therefore, T is the aunt of C. W is married to M. 28. (3) R is the son of M and W. C is also the child of M and W. If M has only one son (R), then C would be the daughter of M. (29–30) : A is the mother of J, T and U. M is the father of J, T and U. M is the husband of A. T and U are daughters of A and M. J is the son of A and M. 29. (1) P is the husband of U. U is the daughter of A and M. Therefore, P is the son-in-law of M.

30. (5) P is the son of K. If R is the husband of K, then K is the mother of P. (31-32) : P and U are daughters of L. K is the husband of P. S is the son of K and P. Q is the husband of U.

33.

(3) S is the mother of J. J is the father of K and P. K is the son of J and T. Therefore, K is the grandson of S.

34.

(5) T is the wife of J. B is the father of J. Therefore, T is the daughter-inlaw of B.

31. (5) P is the mother of S. Therefore, S is the husband of Z, i.e. Z is the wife of S. 32. (3) U is the daughter of L. Q is the husband of U.

35.

(3) B is the husband of A. D is the brother of A. D is the brother-in-law of B. D is the husband of E.

Therefore, Q is the son-in-law of (33–34) : B is the husband of S. J is the son of B and S.

P and Q are the children of D and E. Therefore, B is the uncle of Q.

J is the husband of T. (36–38) : Member

Sex

A

Female

Profession

B

Male

Athlete Wrestler

Relationship Wife of B;

C

Female

D

Female

E F

Male Male

Boxer

G

Male

Advocate

Father of E; Son of F and D; Husband of A; Wife of G; Daughter-in-law of F and D Wife of F; Mother of B and G; Son of B and A; Father of B and G; Husband of D; Husband of C;

The three married couples are :

(i) B and A (ii) G and C (iii) F and D 36. (4) F is the paternal grandfather of E. If E is Pilot, then the other daughter-in-law of F would be Architect or Journalist. 37. (2) If D is Pilot, and the Journalist is not a female, then E would be a Journalist and A would be an Architect. 38. (3) B and G are the sons of F and D. E is son of A and B. G and C have no child. G is an Advocate and C is a Wrestler. Therefore, E is a Pilot. 39. (5) N is the sister of R. N is the mother of Q. The gender of Q is not known. Therefore, Q is either nephew or niece of R. 40. (1) M is the son of R. R is the son of S. Therefore, M is the grandson of S. 41. (4) S is daughter of R and Q. W is mother of Q. Q is mother of S. Therefore, S is granddaughter of W. ❐❐❐

BPRE–142

BLOOD RELATIONSHIP

MODEL EXERCISES 1. Anil, introducing a girl in a party, said, ‘She is the wife of the grandson of my mother’. How is Anil related to the girl? (1) Father (2) Grandfather (3) Husband (4) Father-in-law (5) None of these 2. Pointing to a photograph, Arun said “she is the mother of my brother’s son’s wife’s daughter”. How is Arun related to the lady? (1) Cousin (2) Daughter-in-law (3) Uncle (4) Sister (5) None of these Directins (3-6) : In a family of six, C is the sister of F, B is the brother of E’s husband, D is the father of A and grandfather of F. There are two fathers and three brothers in the family. 3. How is D related to B? (1) Father (2) Mother (3) Brother (4) Indeterminate (5) None of these 4. Who is C’s mother? (1) E (2) B (3) F (4) D (5) None of these 5. Which of the following are not siblings? (1) A & B, A & C (2) B & A, C & F (3) C & F (4) A & C (5) None of these 6. If F is married to an outsider G, how is B related to G ? (1) Uncle (2) Aunt (3) Sister in law (4) Son (5) None of these Directions (7-10) : Study the following information carefully to answer these questions. Adhir Mishra has three children— Urmila, Raghu and Sumit. Sumit married Roma, the eldest daughter of Mr and Mrs Mohan. The Mohans married their youngest daughter to the eldest son of Mr and Mrs Sharma and they

had two children—Sandeep and Shaifali. The Mohan have two more children—Roshan and Bimla, both elder to Sheila. Sohan and Shivendar are sons of Sumit and Roma. Leela is the daughter of Sandeep. 7. How is Mrs Mohan related to Sumit? (1) Aunt (2) Mother-in-law (3) Mother (4) Sister-in-law (5) None of these 8. What is the surname of Sohan? (1) Sharma (2) Mohan (3) Mishra (4) Raghu (5) None of these 9. What is the surname of Leela? (1) Sharma (2) Mohan (3) Mishra (4) Sohan (5) None of these 10. How is Shivendar related to Roma’s father? (1) Son-in-law (2) Cousin (3) Son (4) Grandson (5) None of these Directions (11-13) : Read the following information to answer these questions. (i) A × B means A is the brother of B. (ii) A + B means A is the mother of B. (iii) A ÷ B means A is the son of B. (iv) A–B means A is the husband of B. 11. Which of the following would mean M is the father of N? (1) M ÷ N + O (2) M × O – N (3) M + O ÷ N (4) M – O + N (5) None of these 12. Which of the following is definitely true, if N is the son of M ? (1) M + N × O (2) M + N – O (3) N ÷ M (4) All the three are true (5) None of these 13. Which of the following would mean M is the aunt of N? (1) M × O ÷ N (2) N + O × M (3) N ÷ M + O (4) All of these (5) None of these

BPRE–143

14. X told Y, “Though I am the son of your father, you are not my brother.” How is X related to Y? (1) Sister (2) Son (3) Daughter (4) None of these (5) All of these Directions (15-17) : Study the following information to answer the questions. A family consists of seven members P, Q, R, S, T, U and V. While one of them is a student, die other six follow different professions of Architect, Lawyer, Professor, Manager, Doctor and Engineer. There are two married couples in the family. P and U are Architect and Doctor respectively. S, the father of Q and the grandfather of V, is an Engineer. No lady in the family is an Engineer or a Lawyer. V, who is a Student, is the grandson of T. 15. Which of the following is one of the married couples? (1) QR (2) QU (3) PR (4) Data inadequate (5) None of these 16. How is R related to V? (1) Mother (2) Aunt (3) Sister (4) Data inadequate (5) None of these 17. What is T’s profession? (1) Professor (2) Manager (3) Professor or Manager (4) Doctor (5) None of these Directions (18-22) : Study the following information carefully to answer these questions. There are six persons A, B, C, D, E and F. C is the sister of F. B is the brother of E’s husband. D is the father of A and grandfather of F. There are two fathers, three brothers and a mother in the group. 18. Who is the mother? (1) A (2) B (3) D (4) E (5) None of these 19. Who is E’s husband? (1) B (2) C (3) A (4) F (5) None of these

BLOOD RELATIONSHIP 20. How many male members are there in the group ? (1) One (2) Two (3) Three (4) Four (5) None of these 21. How is F related to E? (1) Uncle (2) Husband (3) Son (4) Daughter (5) None of these 22. Which of the following is a group of brothers? (1) ABF (2) ABD (3) BFC (4) BDF (5) None of these Directions (23-26) : Read the following information to answer these questions. • In a family of six persons, A, B, C, D, E and F, there are two married couples. • D is grandmother of A and mother of B. • C is the wife of B and mother of F. • F is the granddaughter of E. 23. What is C to A? (1) Grandmother (2) Mother (3) Cannot be determined (4) Wife (5) None of these 24. How many male members are there in the family? (1) Three (2) Four (3) Cannot be determined (4) Two (5) None of these 25. Which of the following is true ? (1) A is the sister of F (2) D has two grandsons (3) B has two daughters (4) None of these (5) All of these 26. Who among the following is one of the couples? (1) DE (2) EB (3) Cannot be determined (4) CB (5) None of these Directions (27-29) : Study the information given below to answer these questions. • There is a family of 5 persons A, B, C, D and E.

• They are working as a doctor, a teacher, a trader, a lawyer and a farmer. • B, an unmarried teacher, is the daughter of A. • E, a lawyer, is the brother of C. • C is the husband of the only married couple in the family. • A, a fanner, is a father of two sons and an unmarried daughter. ˆ Daughter-in-law of A is a doctor. 27. Which of the following is a group of female members in the family? (1) D and E (2) B and D (3) A, C and E (4) B and C (5) None of these 28. Which of the following is the married couple? (1) C and D (2) A and B (3) A and D (4) B and D (5) None of these 29. Which of the following is a group of male members in the family? (1) B and D (2) A, B and C (3) A, C and E (4) A, C and D (5) None of these 30. A boy goes to see a film and finds a man who is his relative. The man is the husband of the sister of his mother. How is the man related to the boy? (1) Brother (2) Nephew (3) Uncle (4) Husband (5) None of these 31. Rahul’s mother is the only daughter of Monika’s father. How is Monika’s husband related to Rahul? (1) Uncle (2) Father (3) Grandfather (4) Brother (5) None of these Directions (32-36) : Study the following information to answer these questions. There are six persons A, B, C, D, E and F, C is the sister of F, B is the brother of E’s husband. D is the father of A and grandfather of F. There are two fathers, three brothers and a mother in the group. 32. Who is the mother? (1) A (2) D (3) B (4) E (5) None of these 33. Who is E’s husband? (1) B (2) A (3) C (4) F (5) None of these

BPRE–144

34. How many male members are there in the group? (1) One (2) Three (3) Two (4) Four (5) None of these 35. How is F related to E? (1) Uncle (2) Son (3) Husband (4) Daughter (5) None of these 36. Which of the following is a group of brothers? (1) ABF (2) BFC (3) ABD (4) BDF (5) None of these Directions (37-39) : Study the information below to answer these questions. Seeta, Rajinder and Surinder are children of Mr and Mrs Aggarwal. Renu, Raja and Sunil are children of Mrs and Mr Malhotra. Sunil and Seeta are a married couple and Ashok and Sanjay are their children. Geeta and Rakesh are children of Mr and Mrs Gupta. Geeta is married to Surinder and has three children named Rita, Sonu and Raju. 37. How is Rajinder related to Ashok? (1) Father in-law (2) Brother-in-law (3) Cousin (4) Maternal Uncle (5) None of these 38. What is Sanjay’s surname? (1) Gupta (2) Malhotra (3) Aggarwal (4) Surinder (5) None of these 39. Renu is Sanjay’s (1) Sister-in-law (2) Sister (3) Cousin (4) Aunty (5) None of these 40. When Anuj saw Manish, he recalled, “He is the son of the father of my daughter’s mother.” Who is Manish to Anuj? (1) Brother (2) Brother-in-law (3) Cousin (4) Uncle (5) None of these 41. Saroj is mother-in-law of Vani who is sister-in-law of Deepak. Rajesh is father of Ramesh, the only brother of Deepak. How is Saroj related to Deepak ? (1) Wife (2) Aunt (3) Mother-in-law (4) Mother (5) None of these

BLOOD RELATIONSHIP 42. Mohan is the son of Arun’s father’s sister. Prakash is the son of Reva who is the mother of Vikas and grandmother of Arun. Pranab is the father of Neela and the grandfather of Mohan. Reva is the wife of Pranab. How is the wife of Vikas related to Neela? (1) Sister (2) Sister-in-law (3) Niece (4) Brother (5) None of these 43. Ajay is the brother of Vijay. Mili is the sister of Ajay. Sanjay is the brother of Rahul and Mehul is the daughter of Vijay. Who is Sanjay’s uncle? (1) Data inadequate (2) Mehul (3) Rahul (4) Ajay (5) None of these 44. Ravi is the son of Aman’s father’s sister. Sahil is the son of Divya who is the mother of Gaurav and grandmother of Aman. Ashok is the father of Tanya and grandfather of Ravi. Divya is the wife of Ashok. How is Gaurav’s wife related to Tanya? (1) Sister (2) Niece (3) Sister-in-law (4) Data inadequate (5) None of these 45. Introducing Rajesh, Neha said, “His brother’s father is the only son of my grandfather”. How is Neha related to Rajesh? (1) Sister (2) Daughter (3) Mother (4) Nice (5) None of these 46. Deepak said to Nitin, “That boy playing with the football is the younger of the two brothers of the daughter of my father’s wife.” How is the boy playing football related to Deepak? (1) Cousin (2) Brother (3) Son (4) Brother-in-law (5) None of these 47. Amit is the son of Rahul. Sarika, Rahul’s sister, has a son Sonu and a daughter Rita. Raja is the maternal uncle of Sonu. What is the relationship of Raja with Rita? (1) Maternal Uncle: (2) Brother (3) Uncle (4) Can’t be determined (5) None of these

48. Ramakrishna said, “This girl is the wife of the grandson of my mother.” How is Ramakrishna related to the girl? (1) Father (2) Father-in-law (3) Grandfather (4) Daughter-in-law (5) None of these 49. Showing a photograph to a friend, Ravi says, “She is the granddaughter of the elder brother of my father.” How is this girl related to Ravi ? (1) Niece (2) Sister (3) Aunt (4) Sister-in-law (5) None of these 50. I. P × Q means P is the brother of Q. II. P + Q means P is the father of Q. III. P–Q means P is the sister of Q. Which of the following represents X is the uncle of M? (1) X + Y – M (2) X × Y + M (3) X + Y × M (4) X – Y + M (5) None of these 51. Varun is the brother of Meghna and Nikhil. Prerna is the mother of Nikhil. Rakesh is the father of Varun. Which of the given statements cannot be said to be definitely true? (1) Rakesh is the brother of Meghna (2) Rakesh is the husband of Prerna (3) Varun is the son of Prerna (4) Prerna is the mother of Varun (5) None of these 52. A, B, C, D, E, F and G are members of a family consisting of 4 adults and 3 children, two of whom, F and G are girls, A and D are brothers and A is a doctor. E is an engineer married to one of the brothers and has two children. B is married to D and G is their child. Who is C? (1) G’s father (2)F’s father (3) E’s daughter (4) A’s son (5) None of these 53. Pointing to Abhishek, Seema said, “His father is the only son of my grandfather.” How is Seema related to Abhishek ? (1) Sister (2) Mother (3) Niece (4) Daughter (5) None of these

BPRE–145

54. Kartik is the father of Heena, Uma is the mother of Tanya. The sister of Heena and Tanya is Neha. Which of the following statements is definitely not true? (1) Uma is the mother of Neha (2) Heena is the sister of Neha (3) Uma has three daughters (4) Tanya is the son of Kartik (5) None of these 55. M, N, O and P are all distinct individuals. M is the daughter of N. N is the son of O and O is the father of P. If B is the son of N and B has one brother, D, then which of the following statements is/are true? I. M is the sister of D. II. D and B are brothers. III. O is the grandfather of D. (1) I only (2) 1 and III only (3) III only (4) All of these (5) None of these 56. Pointing out to the man on the stage, Madhu said that he is the brother of the daughter of the wife of my husband. How is the man on the stage related to Madhu? (1) Son (2) Husband (3) Cousin (4) Nephew (5) None of these 57. X introduces Y saying, “He is the husband of the grand daughter of the father of my father.” How is Y related to X? (1) Brother (2) Son (3) Brother-in-law (4) Son-in-law (5) None of these 58. While going to his office, Tejveer meets Rakesh who is related to Tajveer because Tajveer is Deepak’s father who is married to Garima. Garima is the daughter of Rakesh. Garima has a daughter named Isha. How is Tejveer related to Isha? (1) Uncle (2) Father-in-law (3) Grandfather (4) Grandmother (5) None of these

BLOOD RELATIONSHIP 59. Pointing to a lady in a photograph, Dev said, “This woman is my sister’s father’s son-in-law’s wife.” How is Dev related to the woman? (1) Wife (2) Sister (3) Mother (4) Can’t say (5) None of these Directions (60-62) : Study the information below to answer the questions. ‘A’, ‘B’, ‘C’, ‘D’, ‘E’, ‘F’ and ‘G’ are seven members of a family. There are two married couples among them, belonging to two different generations. Each member of the family has a different choice of food ie, Chinese, Continental, Rajasthani, Punjabi, South Indian, Gujarati and Marathi. The grandfather in the family likes Gujarati food. None of the ladies likes Continental or Rajasthani food. ‘D’ is the son of ‘A’ and ‘A’ likes Chinese food. ‘G’ is ‘C’s daughter-in-law and ‘C’ likes South Indian meals. ‘B’ is the grandfather of ‘F’ and ‘F’ likes Punjabi food. ‘C is the mother of ‘E’ and ‘E’ likes Continental food. 60. Which particular food is liked by ‘D? (1) Rajasthani (2) Gujarati (3) Marathi (4) Continental (5) None of these 61. Which is the food combination of the two married women? (1) Punjabi and Marathi (2) South Indian and Punjabi (3) Marathi and South Indian (4) Rajasthani and South Indian (5) None of these 62. How is ‘F’ related to ‘E’? (1) Sister (2) Daughter (3) Aunt (4) Data inadequate (5) None of these 63. Pointing towards a person in a photograph, Monika said, “He is the son of the father of my sister’s brother.” How is that person related to Monika? (1) Cousin (2) Mother (3) Father (4) None of these (5) All of these

64. Akash’s grandmother is married to Nirmal’s brother. Sahil’s mother is the granddaughter-in-law of Nirmal’s father-in-law. What is Akash to Sahil? (1) Cousin (2) Uncle (3) Nephew (4) Brother (5) None of these 65. Ankit is the grandson of Mohan whose brother is married to the grandmother of Ankush. How is Ankush related to Ankit? (1) Uncle (2) Nephew (3) Cousin (4) Brother (5) None of these 66. Raja is the son of Varun’s father’s sister. Sameer is the son of Ganga who is the mother of Jai and the grandmother of Varun. Nitin is the father of Varsha and the grandfather of Raja. Ganga is the wife of Nitin. How is Raja related to Ganga? (1) Son (2) Grandson (3) Nephew (4) Data inadequate (5) None of these Directions (67–69) : Read the following information to answer these questions. I. P, Q, R, S, T and U are the six members of a family. II. There is one Doctor, one Advocate, one Engineer, one Teacher, one Student and one Housewife among them. III. There are two married couples in the family. IV. U, who is an Advocate, is father of P. V. Q is a Teacher and is mother of R. VI. S is grandmother of R and is a Housewife. VII. T is the father of U and is a Doctor. VIII. R is the brother of P. 67. Which of the following statements is definitely true ? (1) U is father of the Engineer (2) P is the Engineer (3) T is father of the Teacher (4) R is brother of the Student (5) None of these

BPRE–146

68. How many female members are there in the family ? (1) Three only (2) Two or three (3) Two only (4) Three or four (5) None of these 69. How is P related to S ? (1) Either granddaughter or grandson (2) Grandmother (3) Grandson (4) Granddaughter (5) None of these Directions (70–72) : Read the following information to answer these questions. In a family, there are 10 members. G and N are a married couple while K is the husband of X. In this family, there are four housewives and four working husbands while other members are non-working. The husband in the last generation is a doctor and the husband of S is a teacher. L is a married woman but her husband is not P. Q and K are associated with the profession of engineer and accountant but K is not an accountant. P is the brother of T and both have a sister O. N is the daughter of L and X is the mother of P. 70. The family consists of how many generations ? (1) 6 (2) 1 (3) 4 (4) 2 (5) None of these 71. Who is the husband of S ? (1) T (2) K (3) L (4) P (5) None of these 72. Which of the following statements is not true ? (1) Husband of X is an engineer. (2) Second generation members are S, P, T and 0. (3) The number of female members in the family is 5. (4) The accountant and his wife belong to the 4th generation. (5) None of these Directions (73–75) : Read the following information to answer these questions : Six members of a family G, H, I, J, K and L are Accountant, Clerk, Lawyer, Jeweller, Doctor and Engineer, but not in the same order.

BLOOD RELATIONSHIP I. Doctor is the grandfather of L who is an Accountant. II. Clerk J is married to G. III. I, who is a Jeweller, is married to the Lawyer. IV. H is the mother of L and K. V. There are two married couples in the family. 73. What is the profession of K ? (1) Doctor (2) Clerk (3) Engineer (4) Accountant (5) None of these 74. How many male members are there in the family ? (1) Two (2) Three (3) Four (4) Cannot be determined (5) None of these 75. How is G related to K ? (1) Wife (2) Father (3) Grandmother (4) Grandfather (5) None of these Directions (76–78) : Read the following information to answer these questions : There are six children playing football namely G, H, I, J, K and L. G and K are brothers. L is the only sister of K. I is the only son of G’s uncle. H and J are the daughters of the brother of I’s father. 76. How is J related to G ? (1) Sister (2) Niece (3) Cousin (4) Uncle (5) None of these 77. How is I related to L ? (1) Cousin (2) Son (3) Uncle (4) Brother (5) None of these 78. How many male players are there ? (1) One (2) Three (3) Four (4) Five (5) None of these 79. How is the son-in-law of my wife’s only sister related to my wife’s brother ? (1) Cousin (2) Nephew (3) Uncle (4) Son-in-law (5) None of these 80. ‘B’, the son of ‘A’, was wedded to C whose sister ‘D’ was married to ‘E’. If E is the brother of ‘B’, how is D’ related to ‘A’ ?

(1) Sister (2) Daughter (3) Daughter-in-law (4) Cousin (5) None of these 81. Krishna is the mother-in-law of Bindu who is the sister-in-law of Ahsaan. Dharmender is the father of Sandeep, the only brother of Ahsaan. How is Krishna related to Ahsaan? (1) Wife (2) Aunt (3) Mother-in-law (4) Mother (5) None of these Directions (82–84) : Study the following information carefully and answer the questions given below ‘A × B’ means ‘A is father of B’ ‘A + B’ means ‘A is wife of B’ ‘A ÷ B’ means ‘A is daughter of B’. ‘A – B’ means ‘A is son of B’. 82. How is L related to Q in the expression ‘L ÷ M × O – P ÷ Q’ ? (1) Daughter-in-law (2) Aunt (3) Granddaughter (4) Sister (5) Cannot be determined 83. What should come in the place of question - mark (?) to establish that Q is nephew of T in the expression ‘Q ? R ÷ S × T’ ? (1) – (2) + (3) Either ‘–’ or ‘÷’ (4) × (5) ÷ 84. Which among the following options is true if the expression ‘A – B × C + D – E’ is definitely true ? (1) C is the brother of A (2) D is the father of A (3) E is the brother of B (4) C is daughter-in-law of E (5) E is father-in-law of C. Directions (85–87) : Study the following information carefully and answer the questions given below : M is the father of G. P is the wife of G. M is married to C. L is sibling of G. S is the father–in–law of L. K is the only child of S.

BPRE–147

85. How is G related to K? (1) Cousin

(2) Father

(3) Cannot be determined (4) Brother–in–law (5) Nephew 86. If K is the mother of R, then how is L related to R? (1) Father (2) Son–in–law (3) Brother–in–law (4) Son (5) Cannot be determined 87. How is C related to P? (1) Mother (2) Mother–in–law (3) Sister–in–law (4) Grandmother (5) Niece Directions (88-89) : Study the following information carefully to answer the given questions : ‘P + Q’ means ‘P is the sister of Q’. ‘P @ Q’ means ‘P is the wife of Q’. ‘P $ Q’ means ‘P is the son of Q.’ ‘P % Q’ means ‘P is the mother of Q’. 88. What is the relation between ‘J and A’ in the expression ‘A@F$M%J+T’ ? (1) J is the mother-in-law of A (2) A is the aunt of J (3) J is the sister-in-law of A (4) A is the husband of J (5) None of these 89. What will come in the place of question–mark, if it is provided that ‘J is the daughter-in-law of T’ in the expression ‘J%B ? K$T’? (1) @ (2) % (3) + (4) $ (5) % or + Directions (90–92) : Study the following information carefully and answer the questions given below : P @ Q means P is father of Q. P + Q means P is husband of Q. P $ Q means P is brother of Q. P % Q means P is mother of Q. P & Q means P is sister of Q. 90. How is B related to E in this expression : ‘A @ B % C & D + E’ ? (1) Grandmother (2) Granddaughter (3) Mother–in–law (4) Aunt (5) Daughter–in–law

BLOOD RELATIONSHIP 91. What should come in place of the question mark (?) to establish that C is the aunt of E in the following expression ? A%B+C&D?E (1) $ (2) & (3) + (4) Either + or & (5) Either @ or % 92. Which among the following options is true if the expression ‘A + B & C @ D % E $ F’ is definitely true ? (1) E is the sister–in–law of A. (2) F is daughter–in–law of C. (3) B is the aunt of D. (4) A is the uncle of E. (5) C is the uncle of A. 93. Read the following information carefully and answer the question which follows : If ‘A × B’ means ‘A is father of B’ If ‘A + B’ means ‘A is daughter of B’. If ‘A ÷ B’ means ‘A is son of B’ If ‘A – B’ means ‘A is sister of B’. What will come in the place of the question mark, to establish that P is the son-in-law of S in the expression : ‘P × Q + R – T?S (1) + (2) × (3) – (4) ÷ (5) Either + or ÷ Directions (94–98) : Read the information carefully and answer the following questions : If A + B means A is the father of B If A × B means A is the sister of B If A $ B means A is the wife of B If A % B means A is the mother of B If A ÷ B means A is the son of B 94. What should come in place of the question mark (?) to establish that J is the brother of T in the expression ? J÷P%H?T%L (1) × (2) ÷ (3) $ (4) Either ÷ or × (5) Either + or ÷ 95. Which among the given expressions indicate that M is the daughter of D ? (1) L % R $ D + T × M (2) L + R $ D + M × T (3) L % R % D + T ÷ M (4) D + L $ R + M × T (5) L $ D ÷ R % M ÷ T

96. Which among the following options is true if the expression ‘I + T % J × L ÷ K’ is definitely true ? (1) L is the daughter of T (2) K is the son-in-law of I (3) I is the grandmother of L (4) T is the father of J (5) J is the brother of L 97. Which among the following expressions is true if Y is the son of X is definitely false ? (1) W % L × T × Y ÷ X (2) W + L × T × Y ÷ X (3) X + L × T × Y ÷ W (4) W $ X + L + Y + T (5) W % X + T × Y ÷ L 98. What should come in place of the question mark (?) to establish that T is the sister-in-law of Q in the expression ? R%T×P?Q+V (1) ÷ (2) % (3) × (4) $ (5) Either $ or × 99. Pointing to a woman Nirmal said, “She is the daughter of my wife’s grandfather’s only child”. How is the woman related to Nirmal ? (1) Wife (2) Sister-in-law (3) Sister (4) Data inadequate (5) None of these 100. E is the son of A. D is the son of B. E is married to C. C is B’s daughter. How is D related to E? (1) Father-in-law (2) Cannot be determined (3) Uncle (4) Brother-in law (5) Daughter Direction (101–105): In the following questions, some symbols are used with the following meanings as illustrated below : X+Y X is the niece or nephew of Y where X is opposite to Y X is the husband of Y X*Y where X is second to the left of Y X#Y X is the son of Y where X is immediate left of Y X is the son-in-law of X&Y Y where X is immediate right of Y

BPRE–148

X^Y

X is the maternal aunt of Y where X is second to the right of Y X is the mother of Y X>Y where X is second to the left of Y X?Y X is the daughter of Y where X is second to the left of Y X is the brother of Y X!Y where X is immediate right of Y X$Y X is father of Y where X is third to the right of Y X is the wife of Y where X%Y X is immediate left of Y X is the child of Y X@Y where X is second to the right of Y X=Y X is the cousin of Y where X is third to the right of Y X is the parent of Y X©Y where X is third to the left of Y. X÷Y X is the father in law of Y where X is immediate right of Y Study the Conclusions based on the given statement and select the appropriate answer. Give answer (1) if only Conclusion I is true Give answer (2) if only Conclusion II is true Give answer (3) if either Conclusion I or Conclusion II is true Give answer (4) if neither Conclusion I nor Conclusion II is true Give answer (5) if both the Conclusion I and Conclusion II are true There are 8 family members sitting around a round table facing the centre. Consider the following coding: D?M*Z©B+G!M÷Q$C# B^X 101. Statement: With respect to D and Z Conclusions : I. D is sitting second to the right of Z II. D is the daughter of Z 102. Statement: With respect to M and C Conclusions : I. C is the grandson of M II. M is sitting opposite to C

BLOOD RELATIONSHIP 103. Statement: With respect to X and B Conclusions : I. X is sitting second to the left of B II. B is the niece of X 104. Statement: With respect to G and Q Conclusions : I. G and Q are blood related II. Q is sitting second to the right of G 105. Statement: With respect to X and M Conclusions : I. X is sitting third to the right of M II. X is the granddaughter of M, if there is only one grandson in the family 106. Reena is the mother-in-law of Roma who is wife of Rakshit. Rakshit whose father is Roshan, is the father of Ritik. Raksha is the daughter of Reena. How is Raksha related to Roma? (1) Cousin sister (2) Mother in law (3) Niece (4) Sister in law (5) Aunt Directions (107–111) : In the following questions, some symbols have been used with the following meanings as illustrated below : ● X ! Y → X is the brother of Y and X is to the immediate right of Y. ● X $ Y → X is father of Y and X is an second to the left of Y. ● X % Y → X is the wife of Y and X is an immediate neighbour of Y (either or sides). ● X @ Y → X is the child of Y and X is second to the right of Y. ● X = Y → X is the cousin of Y and X is third to the right of Y. ● X © Y → X is the parent of Y and X is third to the left of Y. ● X + Y → X is the niece or nephew of Y and X is opposite to Y. ● X * Y → X is the husband of Y and X is second to the left of Y. ● X # Y → X is the son of Y and X is to the immediate left of Y. ● X & Y → X is the son-in-law of Y and X is to the immediate right of Y.

X ^ Y → X is the sister of Y and X is to the immediate left of Y. ● X > Y → X is the mother of Y and X is second left of Y. 107. If C @ M * Q ^ G > Z are in a circular seating arrangement and C is facing the centre, who among the following is second to the left of Q? (1) Q’s Sister (2) Q’s Mother (3) Q’s Husband (4) Q’s Brother in law (5) Q’s Daughter 108. If J * G > K & F > D are in a circular seating arrangement and D is facing away from the centre, who among the following is third to the right of G? (1) G’s Daughter in law (2) G’s Son (3) G’s Husband (4) G’s Sister (5) G’s Daughter 109. If S % M ! Z $ K = B are in a circular seating arrangement and all are facing the centre, who among the following is sitting with S? (1) S’s Brother in law (2) S’s Nephew or Niece (3) S’s Husband (4) S’s Son (5) Either 1 or 2 110. If J ^ B > N # D © G are in a circular seating arrangement and all are facing the centre, who among the following is sitting third to the right of J? (1) J’s Brother (2) J’s Nephew (3) J’s Father (4) J’s Sister (5) J’s Niece 111. If R $ Z & K > C ^ J are in a circular seating arrangement and all are facing the centre, who among the following is sitting second to the right of J? (1) J’s Brother in law (2) J’s father in law (3) J’s Mother ●

(4) J’s Niece (5) J’s Husband

BPRE–149

Directions (112–116) : In the following questions, some symbols are used with the following meanings as illustrated below : X ! Y → X is the brother of Y where X is to the immediate right of Y ● X $ Y → X is father of Y where X is second to the left of Y ● X % Y → X is the wife of Y where X is to the immediate left neighbour of Y ● X @ Y → X is the child of Y where X is second to the right of Y ● X = Y → X is the cousin of Y where X is third to the right of Y ● X © Y → X is the parent of Y where X is third to the left of Y ● X + Y → X is the niece or nephew of Y where X is opposite to Y ● X * Y → X is the husband of Y where X is second to the left of Y ● X # Y → X is the son of Y where X is immediate left of Y ● X & Y→ X is the son-in-law of Y where X is to the immediate right of Y ● X ^ Y → X is the sister of Y where X is to the immediate left of Y ● X > Y → X is the mother of Y where X is second to the left of Y ● X ? Y → X is the daughter of Y where X is second to the left of Y (112–114) : Statement : ●

M*T©H!B>D%F@Q*Z 112. Who among the following is facing away from the centre in the arrangement? (1) F’s mother (2) H’s sister (3) T’s husband (4) D’s father-in-law (5) B’s mother 113. Who is sitting second to the right of D in the arrangement? (1) D’s mother (2) D’s uncle (3) D’s Son (4) D’s Grandfather (5) D’s son-in-law 114. Who among the following is sitting third to the right of T in the given arrangement? (1) T’s daughter-in-law (2) T’s grandson (3) T’s Son (4) T’s husband (5) T’s brother-in-law

BLOOD RELATIONSHIP (115–116) : Statement : K$L!G>Q?B#U*T 115. Who among the following is facing away from the centre? (1) Q’s grandmother (2) L’s grandfather (3) G’s Uncle (4) K’s wife (5) G’s husband 116. Who among the followingt is sitting second to the right of K in the arrangement? (1) K’s daughter (2) K’s granddaughter (3) K’s son (4) K’s son-in-law (5) K’s nephew 117. Mr. X the father K travels from north to south 5 km to reach Bus stand where he meets his daughter in law D. T the husband of D travels from D’s house towards east 8 km where he meets his brother B and T takes right 4 km to reach his uncle P’s house which is located near Maitri Mall. If Q the wife of P is maternal aunt of B, then which male child of the family stays near bus stand related to B? (1) Son (2) Uncle (3) Cousin (4) Nephew (5) Brother Directions (118 – 122): Consider the following relationships and answer the questions given below : T is the sister of D. D is married to P. P is the son of M. B is the grandfather of J. T is the mother of J. Y is the father of U. Y has only one son and only one daughter. U is the daughter of T. Q is the son of D. P is the son in law of Z. W is the mother in law of Y. T is the daughter in law of B. 118. How is U related to D ? (1) Daughter (2) Mother (3) Son in law (4) Cousin (5) Niece 119. How is Z related to J ? (1) Father in law

(2) Aunt (3) Mother (4) Grandfather (5) Uncle 120. How is D related to Y ? (1) Cousin sister (2) Nephew (3) Sister (4) Sister-in-law (5) Brother 121. How is Q related to T ? (1) Paternal uncle (2) Maternal aunt (3) Nephew (4) Brother (5) Sister 122. How is W related to T ? (1) Mother in law (2) Maternal uncle (3) Brother in law (4) Son (5) Mother Directions (123–127) : In the following questions, some symbols are used with the following meanings as illustrated below : ● X&Y X is the son-in-law of Y where X is to the immediate right of Y. X is sister of Y where ● X ^ Y X is to the immediate left of Y ● X!Y X is brother of Y where X is to the immediate right of Y ● X$Y X is father of Y where X is second to the left of Y X is the wife of Y ● X % Y where X is an immediate neighbour of Y (either or sides) ● X @ Y X is the child of Y where X is second to the right of Y X is the cousin of Y ● X = Y where X is third to the right of Y ● X © Y X is the parent of Y where X is third to the left of Y X is the niece or neph● X+Y ew of Y where X is opposite Y ● X * Y X is the husband of Y where X is second to the left of Y ● X#Y X is son of Y where X is to the immediate left of Y ● X?Y X is mother of Y where X is to the immediate left of Y Give answer (1) if only Conclusion I is true Give answer (2) if only Conclusion II is true

BPRE–150

Give answer (3) if either Conclusion I or Conclusion II is true Give answer (4) if neither Conclusion I nor Conclusion II is true Give answer (5) if both the Conclusion I and Conclusion II are true 123. Statement : M © Z @ T ! H $ B and only one person is facing away from the centre Conclusions : I. M is second to the right of H II. T is maternal uncle to B 124. Statement : G % M $ K @ G ^ P © Q and only one person is facing away from the centre Conclusions : I. M is an immediate neighbour of both Q and G II. G is aunt of Q 125. Statement : X @ M $ P @ K % M ! Z and only one person is facing away from the centre Conclusions : I. X is sitting exactly between Z and K II. X is either nephew or niece of K 126. Statement : G $ L & T * Q ? D and all the persons are facing the centre Conclusions : I. Q is second to the right of T II. L is son–in–law of Q 127. Statement : E & F ! T @ M % G and all are facing the centre Conclusions : I. F is second to the left of M II. G is father of T Directions (128–132) : In the following questions, some symbols are used with the following meanings as illustrated below : ● X $ Y → X is father of Y where X is second to the left of Y ● X % Y → X is the wife of Y where X is immediate neighbour of Y (either side) ● X + Y → X is the niece or nephew of Y where X is opposite to Y. ● X * Y → X is husband of Y where X is second to the left of Y ● X # Y → X is the son of Y where X is to the immediate left of Y ● X & Y → X is the son-in-law of Y where X is to the immediate right of Y ● X ^ Y → X is the sister of Y where X is to the immediate left of Y

BLOOD RELATIONSHIP X ! Y → X is the brother of Y where X is to the immediate right of Y ● X = Y → X is the cousin of Y where X is third to the right of Y ● X @ Y → X is the child of Y where X is second to the right of Y ● X © Y → X is the parent of Y where X is third to the left of Y 128. If H ! T $ L @ B © D in a circular seating arrangement is true and all are facing the centre, who among the following are sitting together? (1) H and L’s father (2) H and L’s mother (3) L and D (4) L’s father and D’s mother (5) H and D 129. If K ^ L + J ! X % I © Y in a circular seating arrangement is true and all are facing the centre, how K is related to the person who is sitting to the immediate left of Y? (1) Father (2) Mother (3) Niece (4) Uncle (5) Brother 130. If F © D % J @ H ! B % F in a linear seating arrangement is true and all are facing the north, how F is related to the person at right end of the line? (1) Father–in–law (2) Uncle (3) Husband (4) Nephew (5) Mother 131. If P @ L ! O * N = M in a linear seating arrangement is true and all are facing the north, how O is related to the person at right end of the line? (1) Mother (2) Nephew (3) Uncle (4) Brother–in–law (5) Aunt 132. If Q * Z ^ L @ T @ M in a linear seating arrangement is true and all are facing the EAST, how Z is related to the person at middle of the line? (1) Cousin (2) Grand daughter (3) Son in law (4) Daughter (5) Son ●

Directions (133 – 137) : Study the following information carefully and answer the questions given below: There are 11 members in a family. There are 4 couples and each couple has at least one child. L is the wife of J. B is the daughter of X. K is the son of V Q is the wife of T.G is niece of V. T is the maternal grandfather of K who is husband of D. J is the brother of V. V is the daughter of Q. X is the son in law of Q. L is not blood related with anyone in the family except G. B is paternal aunt of W. 133. How is J related to K? (1) Father (2) Grandson (3) Mother-in-law (4) Grandmother (5) Uncle 134. How is B related D? (1) Brother (2) Nephew (3) Aunty (4) Sister-in-law (5) Grandmother 135. How is Q related to L? (1) Co sister (2) Brother in law (3) Father (4) Mother in law (5) Paternal aunt 136. How is X related to W? (1) Maternal uncle (2) Grandfather (3) Maternal uncle (4) Nephew (5) Brother-in-law 137. How is G related to B? (1) Cousin (2) Sister (3) Brother (4) Nephew (5) Brother-in-law 138. A family consists of six members. B is the son of C but C is not the mother of B. A and C are a married couple. E is the brother of C. D is the daughter of A. F is the brother of B. How many children does A have? (1) One (2) Two (3) Three (4) Four (5) Either three or four Directions (139–143) : Study the following information carefully and answer the questions given below : Vijay is the son of Suman. Rajesh is the son-in-law of Prakash. Ritu is

BPRE–151

the sister of Suman. Suman is married to Rajesh. Sundar has only one son and only one daughter. Malathi is the daughter of Ritu. Kamala is the mother-in-law of Sundar. Ritu is the daughter-in-law of Raghu. Rajesh is the son of Manju. Raghu is the grandfather of Karan. Ritu is the mother of Karan. Sundar is the father of Malathi. 139. How is Vijay related to Ritu? (1) Son (2) Brother (3) Nephew (4) Uncle (5) Father 140. How is Prakash related to Rajesh? (1) Father (2) Brother (3) Uncle (4) Father-in-law (5) Brother-in-law 141. How is Malathi related to Suman? (1) Aunt (2) Niece (3) Sister (4) Mother-in-law (5) Daughter 142. How is Sundar related to Suman? (1) Brother-in-law (2) Father-in-law (3) Uncle (4) Son (5) Son-in-law 143. How is Raghu related to Karan? (1) Father-in-law (2) Son-in-law (3) Brother (4) Nephew (5) Grandfather Direction (144 – 148) : In the following questions, some symbols are used with the following meanings as illustrated below. Read the following instructions carefully and answer the questions given below : ● X © Y : X is the parent of Y. ● X & Y : X is the son-in-law of Y. ● X ! Y : X is the brother of Y. ● X : Y : X is the paternal grandfather of Y. ● X > Y : X is the mother of Y. ● X ^ Y : X is the sister of Y. ● X @ Y : X is the child of Y. ● X + Y : X is the niece or nephew of Y. ● X * Y : X is the husband of Y. ● X ? Y : X is the daughter of Y. ● X # Y : X is the son of Y. ● X = Y : X is the mother-in-law of Y.

BLOOD RELATIONSHIP There are nine family members. Six are male members and three are female members. P?A*E©F+H!A→W$R# F^U#G 149. How is P related to R? (1) Father (2) Grandfather (3) Mother (4) Grandmother (5) Aunt 150. How is A related to G? (1) Maternal uncle (2) Father-in-law (3) Cousin brother (4) Nephew (5) Sister-in-law 151. How is F related to H? (1) Daughter-in-law (2) Son-in-law (3) Sister-in-law (4) Nephew (5) Niece 152. How is W related to P? (1) Sister (2) Brother (3) Nephew (4) Brother-in-law (5) Grandson 153. How is E related to H? (1) Grandson (2) Granddaughter (3) Sister-in-law (4) Daughter-in-law (5) Daughter

69.(1) 73.(3) 77.(1) 81.(4) 85.(4) 89.(4) 93.(5) 97.(4) 101.(2) 105.(5) 109.(5) 113.(4) 117.(4) 121.(3) 125.(4) 129.(3) 133.(5) 137.(1) 141.(2) 145.(3) 149.(5) 153.(3)

70.(3) 74.(4) 78.(2) 82.(3) 86.(1) 90.(3) 94.(1) 98.(4) 102.(5) 106.(4) 110.(2) 114.(3) 118.(5) 122.(5) 126.(5) 130.(1) 134.(4) 138.(3) 142.(1) 146.(5) 150.(2)

71.(4) 75.(4) 79.(4) 83.(1) 87.(2) 91.(5) 95.(2) 99.(4) 103.(1) 107.(3) 111.(3) 115.(1) 119.(4) 123.(1) 127.(5) 131.(3) 135.(4) 139.(3) 143.(5) 147.(3) 151.(5)

72.(4) 76.(3) 80.(3) 84.(4) 88.(3) 92.(3) 96.(2) 100.(4) 104.(4) 108.(3) 112.(4) 116.(3) 120.(4) 124.(5) 128.(4) 132.(4) 136.(2) 140.(4) 144.(4) 148.(4) 152.(4)

EXPLANATIONS 1. (4) The grandson of Anil’s mother is son of Anil and wife of Anil’s son is daughter in law of Anil, thus Anil is father-in-law of girl. 2. (2) Lady is daughter-in-law of Arun. (3-6) :

SHORT ANSWERS

B

1.(4) 5.(4)

2.(2) 6.(1)

3.(1) 7.(2)

4.(1) 8.(3)

9.(1) 13.(5)

10.(4) 14.(4)

11.(4) 15.(4)

12.(4) 16.(4)

17.(3) 21.(3)

18.(4) 22.(1)

19.(3) 23.(2)

20.(4) 24.(3)

25.(4) 29.(3)

26.(1) 30.(3)

27.(2) 31.(2)

28.(1) 32.(4)

33.(2)

34.(4)

35.(2)

36.(1)

37.(4) 41.(4)

38.(2) 42.(2)

39.(4) 43.(1)

40.(2) 44.(3)

45.(1) 49.(1)

46.(2) 50.(2)

47.(1) 51.(1)

48.(2) 52.(4)

53.(1) 57.(3)

54.(4) 58.(3)

55.(4) 59.(2)

56.(1) 60.(1)

61.(3) 65.(3)

62.(4) 66.(2)

63.(4) 67.(1)

64.(1) 68.(2)

BPRE–152

C F

Sister

M

Grand father M Father M F D A

M/F

Father E F

Husband



Brother

X $ Y : X is the father of Y. X % Y : X is the wife of Y. There are eight members in a family and male and female members are equal in number. Statement : X ^ Y # K © W ? E = J$L+Y+Q!E W is one of the unmarried members in the family. 144. How is W related to J? (1) Niece (2) Nephew (3) Aunt (4) Sister-in-law (5) Daughter 145. How is L related to K? (1) Son-in-law (2) Daughter-in-law (3) Granddaughter (4) Son (5) Nephew 146. How is Q related to X? (1) Father (2) Mother (3) Paternal aunt (4) Mother-in-law (5) Maternal uncle 147. If T is W’s son, then how is Y related to T’s father? (1) Nephew (2) Niece (3) Brother-in-law (4) Son-in-law (5) Brother 148. If L is the grandchild of H, then how is X related to H’s husband? (1) Sister (2) Brother (3) Father-in-law (4) Daughter-in-law (5) Aunt Directions (149 – 153) : In the following questions, some symbols are used with the meanings as illustrated below: X + Y : X is the niece or nephew of Y X * Y : X is the husband of Y X # Y : X is the son of Y X & Y : X is son-in-law of Y X ^ Y : X is the maternal aunt of Y X > Y : X is the mother of Y X → Y : X is father in law of Y X ? Y : X is the daughter of Y X ! Y : X is the brother of Y X $ Y : X is the father of Y X % Y : X is the wife of Y X @ Y : X is the child of Y X = Y : X is the cousin of Y X © Y : X is the parent of Y ●

On the basis of information given in the question, we conclude that A and E are married couplex and parents of F & C. D is the father of A and B and grandfather of F & C. 3. (1) D is the father of B. 4. (1) E is the wife of A and mother of C. 5. (4) Clearly, A and C are not sibling. 6. (1) B is the uncle of F, hence he will be uncle of the spouse of B.

BLOOD RELATIONSHIP (7-10) : On the basis of information given in the question, we can draw the following family tree Adhin Mishra married Mr & Mrs Mohan Urmila Raghu Sumit Sons Sohan

Roma Roshan Bimla Shiela (Eldest) (youngest) married Shivendar

Eldest son of Mr Shrama Sandeep

Shaifali

23. (2) C is the mother of A. 24. (3) Sex of A is not known. Hence, number of male members cannot be determined. 25. (4) Again, since sex of A is not known, hence none of the statements is correct. 26. (1) DE is one of the couples. (27-29) :

Leela (Daughter)

7. (2) Mrs Mohan is Sumit’s motherin-law. 8. (3) Surname of Sohan is Mishra because Sohan is son of Sumit and Sumit is son of Mishra. 9. (1) Surname of Leela is Sharma because Leela is grand daughter of Mr sharma. 10. (4) Shivendar is grandson of Roma’s father(Mr Mohan). 11. (4) M– O =M is the husband of O. O + N = O is the mother of N. From these two, we conclude that M is the father of N. 12. (4) M + N × O = M is the mother of N and N is the brother of O ⇒ N is son of M. M + N – O ⇒ N is the son of M. N ÷M ⇒ N is the son of M. Hence, all the three statements deliver the same conclusion. So, all the three are true. 13. (5) M × O ÷ N = M is brother of O and O is the son of N, hence this statement is not true. N + O × M = N is the mother of O and O is the brother of M. ⇒ N is the mother of M. N ÷ M + O = N is the son of M and M is mother of O = M is mother of N. Hence, none of the statements concludes M is the aunt of N. 14. (4) X is son of Y’s father and Y is not brother of X, it means that Y is sister of X. Hence, X has to be brother of Y. (15-17) :

15. (4) Data are inadequate to answer the question. 16. (4) On the basis of information given in the question, we can not establish relationship of R & V. 17. (3) T may be s professor or a manager. (18-22) : On the basis of information given in the question, we can draw the family tree as below Ist generation D (Male) IInd gerneration B Brother A ⇔ E (Male) (Male) (Female) Sister IIIrd generation C (Female)

20. (4) There are four male members in the group, 21. (3) F is the son of E. 22. (1) ABF is the group of brothers. (23-26) : On the basis of information given in the question, the following family tree is prepared. Wife

Mother

Father G.F G.S

B M

G. Father

th Fa

A M/F

er

Wife

C

F Mother

BPRE–153

E M G.Daughter

f

F (Male)

From the above chart it is clear that A and E are married couple. D is the grandfather of C and F and father of A and B. 18. (4) E, wife of A is the mother in the family. 19. (3) A is the husband of E.

F D

m

(Male, Farmer) A

F

F

Daughter

Son

Wife D (Female, Doctor) E (Male,Lawyer)

C Male, Trader)

B Female, Teacher)

27. (2) Band D are a group of female mumbers in the family 28. (1) C and D are the married couple in the family. 29. (3) A, C and E are a group of male members in the family. 30. (3) The Sister of one’s mother is one’s maternal aunt. Hence, the man is the husband of the boy’s maternal aunt. 31. (2) Monika’s father’s only daughter is Monika. So, Monika is the mother of Rahul. Monika’s husband is the father of Rahul. (32-36) : D B

A

E

C

F

D and A are fathers. A, B and F are brothers. 32. (4) E is the mother. 33. (2) A is E’s husband. 34. (4) There are four male members D, A, B and F. 35. (2) F is the son of E. 36. (1) A, B and F are brothers. 37. (4) Rajinder is the brother of Seeta and Ashok is the son of Seeta.

BLOOD RELATIONSHIP Hence, Rajinder is the maternal uncle of Ashok. 38. (2) Sunil is the son of Mr. and Mrs. Malhotra. Sanjay is the son of Sunil. Hence, Sanjay’s surname is Malhotra. 39. (4) Renu is the sister of Sunil and Sanjay is the son of Sunil, so, Renu is the aunty of Sanjay. 40. (2) Daughter’s mother means wife. Father of wife means father-in-law. Son of Father -in-law means brother-in-law. So, Manish is brother-in-law of Anuj. 41. (4)

According to figure, Gourav’s wife is Tanya’s sister in law.

Father Deepak

M =

Father

Br

51. (1)

Wife

o th

er

Daughter Nitin

47. (1) Rahul

sister

Sarika Brother Raja

Ritu

Father-in-law

Reva (Wife)

Neela

From the above chart, it is clear that Vikas is the brother of Neela. Therefore, wife of Vikas will be sister-in-law of Neela. Ajay

Vijay

Mili

Elder Brother

Friend

Ravi

Niece

Grand daughter

50. (2) Check by options,

Nikhil

O N

Sister

P

Rahul M

The data given is inadequate to find the solution. 44. (3)

Sahil

Uma

Tanya is the daughter of Kartik. According to question, though sex of Heena and Tanya is not given, but option (4) is the most correct answer. 55. (4) I. M is the sister of D. II. D and B are brothers. III. O is the grandfather of D.

Mehul

Divya

= Female

Tanya

Option (2) =

Sanjay

= Male

G

Heena

Father

43. (1)

D = B

C

49. (1)

Mohan

Arun

Brother

Wife = Grandson

Son

Son

A

Kartik

Daughter

Prakash

Nikhil

So, C is A’s son. Option (4). 53. (1) Grandfater’s only son is father so the father of Abhishek and Seema is same. Hence, Seema is the sister of Abhishek. 54. (4)

Ramakrishna

Vikash

=

Engineer Doctor

Mother

Son

Megha

F

42. (2)

Son

Varun

48. (2)

Vani

Pranab (Husband)

Prerna

It is clear that Rakesh is the father of Megha. So, statement (1) cannot be definitely true. 52. (4) Draw the family diagram E

Daughter

Son Sonu

Wife

Sister in-law

Rakesh Brother

Deepak

Mother

Mother in-law

Uncle

46. (2)

Saroj

Ramesh

Y = Father

45. (1) Rajesh’s brother’s father is Rajesh’s father. The only son of grandfather is father. So, Neha is the sister of Rajesh.

Maternal u ncle Wife

Rajesh

Brother = X

Ashok

Gaurav

Father = X

M

Y

Tanya

Not possible as father cannot be a sister. Option (2)

BPRE–154

B

D

So, all three statements are true. 56. (1) Madhu’s husband daughter is her daughter and brother of her daughter is her son. 57. (3) Granddaughter of father of my father is my sister. So, Y is the brother-in-law of X.

BLOOD RELATIONSHIP 58. (3)

(67-69) : Tejveer

Rakesh

Garima

Deepak Isha

So, Tejveer is the grandfather of Isha. 59. (2) Dev’s sister’s father is Dev’s father. His father’s son in low’s wife is his sister. So, that woman is Dev’s sister. (60-62) :

B Gujrati E Continental

Male Female Married Couple

C South Indian A Chinese D Rajasthani

G Marathi F Punjabi

60. (1) D likes Rajasthani food. 61. (3) Two married women are G and C and their choice of food is Marathi and South Indian. 62. (4) As the sex of F in not known, so the relationship cannot be established. 63. (4) Father of Monika’s sister’s brother is Monika’s father. So, the son of Monika’s father is her brother. 64. (1) Sahil is the grandson of Nirmal. Akash is the grandson of Nirmal’s brother. So, Akash is cousin of Sahil. 65. (3) Ankit is the grandson of Mohan. Ankush is the grandson of Mohan’s brother. So, Ankush is the cousin of Ankit. 66. (2)

Nitin

Varsha

Ganga

Sameer

Jai

Raja

Varun is the son of Sameer or Jai. Raja is the grandson of Ganga.

Person

Sex

Profession

P

M/F

Q

Female

R

Male

S

Female

T

Male

Engineer/Student Child of Q and U; Grandchild of T and S Teacher Mother of P and R; Wife of U; Daughter-in-law of T and S Engineer/Student Son of Q and U; Grandson of T and S; Brother of P Housewife Grandmother of P and R; Mother of U; Wife of T; Mother-in-law of Q. Doctor Father of U; Father-in-law of Q; Husband of S; Grandfather of P and R.

U

Male

Advocate

Relationship

Father of P and R; Husband of Q.

Married Couples; Q and U; S and T. 67. (1) Either P or R is an Engineer. U is father of P and R. T is the father-in-law of the Teacher, Q. 68. (2) The sex of P is not known. Therefore, there are two or three female members in the family. 69. (1) P is either granddaughter or grandson of S. (70-72) : Husband Wife K–Engineer X P–Teacher S

Q–Accountant L G–Doctor N P is the brother of T. O is the sister of P and T. K is the father of O, P and T. X is the mother of O, P and T. L is the mother of N. 70. (3) The family consists of four generations. 71. (4) P is the husband of S. 72. (4) The Accountant Q and his wife L belong to the third generation.

(73-75) : Member

Sex

Profession

Relationship

G

Male

Doctor

H

Female

Lawyer

I

Male

Jeweller

J

Female

Clerk

K

M/F

Engineer

L

M/F

Accountant

Husband of J; father of I; Grandfather of K and L Mother of K and L; Wife of I; Daughter-in-law of G and J Father of K and L; Son of G and J; Husband of H Wife of G; Mother of I; Grandmother of K and L Child of H and I; Grandchild of G and J Child of H and I; Grandchild of G and J

73. (3) K is an Engineer. 74. (4) The sex of K and L is not known. 75. (4) G is the Grandfather of K. (76–78) : G and K are brothers. L is the sister of G and K. I is the cousin of G, K and L. H and J are cousins of I as well as G, K and L. Male Members : G, K and I Female Members : H, J and L 76. (3) J is the cousin of G. 77. (1) I is the cousin of L.

BPRE–155

78. (2) There are three male players: G, K and I. 79. (4) Son-in-law of my wife’s sister would be also son-in-law of my wife’s brother. 80. (3) B is the brother of E. B and E are sons of A. D is the wife of E. Therefore, D is the daughter-inlaw of A. 81. (4) Bindu is the sister-in-law of Ahsaan. Bindu is the wife Sandeep. Krishna is the mother of Ahsaan and Sandeep.

BLOOD RELATIONSHIP 82. (3) L ÷ M ⇒ L is daughter of M. M × O ⇒ M is father of O. O – P ⇒ O is son of P. P ÷ Q ⇒ P is daughter of Q. P is mother of L. Therefore, L is granddaughter of Q. 83. (1) R ÷ S ⇒ R is daughter of S. S × T ⇒ S is father of T. If Q is the son of R, then Q would be nephew of T. Therefore, Q – R ⇒ Q is son of R. 84. (4) A – B ⇒ A is son of B. B × C ⇒ B is father of C. C + D ⇒ C is wife of D. D – E ⇒ D is son of E. A is brother of C. D is husband of C. B is father-in-law of D. A is brother-in-law of D. C is daughter-in-law of E. The gender of E is not known. (85–87) : C is the wife of M. M is the father of G and L. C is the mother of G and L. L is married to K. 85. (4) G is the brother of L. K is married to L. Therefore, G is the brother–in– law of K. 86. (1) K is the mother of R. Then, K is the wife of L. Therefore, L is the father of R. 87. (2) P is the wife of G. C is the mother of G. Therefore, C is the mother–in– law of P. 88. (3) A @ F ⇒ A is wife of F. F $ M ⇒ F is son of M. M % J ⇒ M is mother of J. J + T ⇒ J is the sister of T. F is the husband of A. J is the sister of F. So, J is the sister-inlaw of A. 89. (4) J % B ⇒ J is the mother of B. B $ K ⇒ B is the son of K. K $ T ⇒ K is the son of T. Therefore, J is dauther-in-law of T. 90. (3) A @ B ⇒ A is father of B. B % C ⇒ B is mother of C. C & D ⇒ C is sister of D. D + E ⇒ D is husband of E. D is the son of B. B is mother–in–law of E. 91. (5) A % B ⇒ A is mother of B. B + C ⇒ B is father of C. C & D ⇒ C is sister of D. If D is father or mother of E, then C would be aunt of E. D @ E ⇒ D is father of E. D % E ⇒ D is mother of E.

92. (3) A + B ⇒ A is husband of B. B & C ⇒ B is sister of C. C @ D ⇒ C is father of D. D % E ⇒ D is mother of E. E $ F ⇒ E is brother of F. D is daughter of C. B is sister of C. So, B is aunt of D. 93. (5) P × Q means P is father of Q. Q + R means Q is daughter of R. R – T means R is sister of T. It is clear that P is husband of R. If we establish that T is either son or daughter of S, then P would be the son-in-law of S. T + S means T is daughter of S . T ÷ S means T is son of S. 94. (1) J ÷ P ⇒ J is the son of P. P % H ⇒ P is the mother of H. H × T ⇒ H is the sister of T. Therefore, P is the mother of J, H and T. J is the brother of T. 95. (2) Option (1) L % R ⇒ L is the mother of R. R $ D ⇒ R is the wife of D. D + T ⇒ D is the father of T. T × M ⇒ T is the sister of M. The gender of M is not known. M is either son or daughter of D. Option (2) L + R ⇒ L is the father of R. R $ D ⇒ R is the wife of D. D + M ⇒ D is the father of M. M × T ⇒ M is the sister of T. It is clear that M is the daughter of D. Option (3) L % R ⇒ L is the mother of R. R % D ⇒ R is the mother of D. D + T ⇒ D is the father of T. T ÷ M ⇒ T is son of M. D is husband of M. Option (4) D + L ⇒ D is the father of L. L $ R ⇒ L is the wife of R. R + M ⇒ R is the father of M. M × T ⇒ M is the sister of T. M is the granddaughter of D. Option (5) L $ D ⇒ L is the wife of D. D ÷ R ⇒ D is the son of R. R % M ⇒ R is the mother of M. M ÷ T ⇒ M is the son of T. M is the brother of D. 96. (2) I + T ⇒ I is the father of T. T % J ⇒ T is the mother of J. J × L ⇒ J is the sister of L. L ÷ K ⇒ L is the son of K. L is the son of K and hence Option (1) is incorrect. T is the wife of K. So, K is the son-in-law of I.

BPRE–156

I is the grandfather of L and hence Option (3) is incorrect. T is the mother of J and hence Option (4) is incorrect. J is the sister of L and hence Option (5) is incorrect. 97. (4) Option (1) W % L ⇒ W is the mother of L. L × T ⇒ L is the sister of T. T × Y ⇒ T is the sister of Y. Y ÷ X ⇒ Y is the son of X. Option (2) W + L ⇒ W is the father of L. L × T ⇒ L is the sister of T. T × Y ⇒ T is the sister of Y. Y ÷ X ⇒ Y is the son of X. Option (3) X + L ⇒ X is the father of L. L × T ⇒ L is the sister of T. T × Y ⇒ T is the sister of Y. Y ÷ W ⇒ Y is the son of W. X is the father of L, T and Y. Y is the son of X. Option (4) W $ X ⇒ W is the wife of X. X + L ⇒ X is the father of L. L + Y ⇒ L is the father of Y. Y + T ⇒ Y is the father of T. So, Y is the grandson of X. Option (5) W % X ⇒ W is the mother of X. X + T ⇒ X is the father of T. T × Y ⇒ T is the sister of Y. Y ÷ L ⇒ Y is the son of L. So, Y is the son of X and L. 98. (4) R % T ⇒ R is the mother of T. T × P ⇒ T is the sister of P. P $ Q ⇒ P is the wife of Q. So, T is the sister-in-law of Q. 99. (4) Only child of Nirmal’s wife’s grandfather means father or mother of Nirmal’s wife. Daughter of Nirmal’s wife’s father or mother means Nirmal’s wife or sister of Nirmal’s wife. 100. (4) E is husband of C. C is daughter of B. D is son of B. D is brother of C. Therefore, D is brother–in–law of E. (101–105) : D?M D is daughter of M. M*Z M is husband of Z. Z©B Z is the parent (Mother) of B. B+G B is the niece or nephew of G. G is brother of M. G!M M÷Q M is the father-in-law of Q. Q$C Q is father of C. C is son of B. C#B

BLOOD RELATIONSHIP B X.

X

B is the maternal aunt of

Z X

G

M

C

B

Q D

101. (2) D is sitting fourth to the left or right (opposite) of Z. D is daughter of M and Z. 102. (5) C is son of B and Q. Q is son-in-law of M. Therefore, C is maternal grandson of M. 103. (1) B is maternal aunt of X. 104. (4) G is brother of M. M is father-in-law of Q. Q is sitting second to the left of G. 105. (5) Both the Conclusions are true. 106. (4) Roma is wife of Rakshit. Rakshit is son of Reena and Roshan. Raksha is daughter of Reena and Roshan. Raksha is sister-in-law of Roma. 107. (3) C @ M C is the child of M. M is the husband of M*Q Q. Q ^G Q is sister of G. G>Z G is mother of Z. M is second to the left of G. M is the husband of Q.

G Q

M

Z

D

F

G

C

108. (3) J * G J is the husband of G. G>K G is mother of K. K is the son-in-law of K&F F. F>D F is mother of D. J is third to the right of G. J is the husband of G.

J

K

109. (5) S % M S is the wife of M. M!Z M is brother of Z. Z$K Z is father of K. K=B K is cousin of B. Z or K is sitting with S.

(112–114) : M is the husband of M*T T. T is the parent of H. T©H H!B H is brother of B. B is mother of D. B>D D%F D is the wife of F. F@Q F is the child of Q. Q*Z Q is the husband of Z. T is the mother of H. F is the son of Q and Z. Q is the father-in-law of D.

B Z

H

D

T

K/S

B

Q

F M M

Z/S

110. (2) J ^ B J is sister of B. B is mother of N. B>N N#D N is the son of D. DG D is the parent of G. N is sitting third to the right of J. J is the aunt of N. N is the nephew of J.

G B

N

J

D

111. (3) R $ Z R is the father of Z. Z is the son-in-law of Z&K K. K>C K is mother of C. C^J C is the sister of J. K is sitting second the right of J. K is the mother of C and J.

Z

T B

Q

U K

C

J

112. (4) Q is facing away from the centre. Q is the father-in-law of D 113. (4) M is sitting second to the right of D. B is the mother of D. B is the daughter of M. Therefore, M is grandfather of D. 114. (3) H is sitting third to the right of T. H is the son of T. (115–116) : K is father of L. K$L L!G L is brother of G. G>Q G is mother of Q. Q is the daughter of Q?B B B#U B is the son of U. U*T U is the husband of T. Q is the daughter of B and G. B is the son of U and T. T is the mother of B.

R

BPRE–157

L

K

G

115. (1) T is facing away from the centre. T is grandmother of Q. 116. (3) L is sitting second to the right of K. L is son of K.

BLOOD RELATIONSHIP 117. (4) X 5 km. 8 km.

Bus Stand D

B T 4 km.

Father

Sister

X

B

P

Maitri Mall

125. (4) X @ M X is the child of M. M$P M is the father of P. P is the child of K. P@K K%M K is the wife of M. M is brother of Z. M!Z X is the child of K. K is the mother of X. Z

129. (3)

Y

Husband

M K

T

(Father) (Mother) (Brother) I X J

L

K (Sister) J

Y

X

K

I

P

D

Son

Daughter–in–law

118. (5) T is sister of D. U is daughter of T. D is wife of P. Therefore, U is niece of D. 119. (4) B is grandfather of J. T is mother of J. Y is husband of T. Z is father of T and husband of W. Therefore, Z is grandfather of J. 120. (4) D is sister of T. Y is husband of T. Therefore, D is sister-in-law of Y. 121. (3) Q is son of D. Q is nephew of T. 122. (5) W is mother of T. M is the parent 123. (1) M © Z of Z. Z@T Z is the child of T. T!H T is brother of H. H$B H is father of B. T is paternal uncle of B.

X K 126. (5) G $ L G is the father of L. L&T L is the son-in-law of T. T is husband of Q. T*Q Q?D Q is mother of D. L is the son–in–law of Q and T. L

F (Father)

Sister (Brother) B H (Mother)

(Daughter) D (Wife)

F Q

B

J (Son) (Husband)

H

D

G D 127. (5) E & F E is the son-inlaw of F. F!T F is the brother of T. T@M T is the child of M. M%G M is the wife of G. T is the child of G and M. G is the father of T.

F is father–in–law of J. 131. (3)

(Father) (Brother) L O (Husband)

N (Wife)

P M

O

L

N

P

G

Z

T

T

M

M

M is second to the right of H. 124. (5) G % M G is wife of M. M$K M is father of K. K is the child of G. K@G G is the sister of P. G^P P©Q P is parent of Q. G is the aunt of Q.

O is uncle of P. 132. (4) M

T

E

F 128. (4) (Brother) (Father) H T

B (Mother)

Q (Husband)

Z (Wife)

(Sister)

M L (Child)

K

D (Child)

Q

D

T Q

P

L

Z M

J

T

H

B

L

K is the niece of J. 130. (1)

G

B, T

H

BPRE–158

L Z is the daughter of T.

L

BLOOD RELATIONSHIP

(144–148) : H ^ Y : X is the sister of Y. Y # K : Y is the son of K. K © W : K is the parent of W. W ? E : W is the daughter of E. E = J : E is the mother-in-law of J. J $ L : J is the father of L. L + Y : L is the niece or nephew of Y. Y + Q : Y is the niece or nephew of Q. Q ! E : Q is the brother of E.

147. (3) W is sister of Y. Therefore, Y is brother-in-law of T’s father. 148. (4) L is granddaughter of H. H is the paternal grandmother of L. Therefore, X is daughter-in-law of H’s husband. (149 – 153) : ● P ? A : P is the daughter of A. ● A * E : A is the husband of E.

Da ug ht er

n So

Suman

139. (3) Ritu is sister of Suman. Suman is mother of Vijay. Vijay is son of Suman. Therefore, Vijay is nephew of Ritu. 140. (4) Prakash is father of Suman. Suman is wife of Rajesh. Therefore, Prakash is father–in–law of Rajesh. 141. (2) Malathi is daughter of Ritu. Ritu is sister of Suman. Therefore, Malathi is niece of Suman. 142. (1) Sundar is husband of Ritu. Ritu is sister of Suman. Therefore, Sundar is brother–in–law of Suman. 143. (5) Raghu is father of Sundar. Sundar is father of Karan. Therefore, Raghu is grandfather of Karan. ●E

© F : E is the parent of F. (E is the mother of F). ● F + H : F is niece or nephew of H. ● H ! A : H is the brother of A. ●A W : A is the father -in-law of W. ● W $ R : W is the father of R. ● R # F : R is the son of F. ● F ^ U : F is the maternal aunt of U. ● U # G : U is the son of G.

Brother

H

Da Husband-Wife

P

Son

L

G

U

E

Sister

r te gh

Daughter

W

144. (4) X is the wife of J. W is the sister of X. Therefore, W is the sister-in-law of J. 145. (3) K is father of X. X is mother of L. L is daughter of X. K is the maternal grandfather of L. L is granddaughter of K. 146. (5) Q is brother of E. E is mother of X. Therefore. Q is maternal uncle of X.

Husband-Wife

A

F

Wife-Husband

W

Son

Y

Rajesh

Vijay

Malathi

ug ht er

Da ugh ter X

Wife – Husband Son

Son

Father

Ritu

er ht

K

Manju

u Da

er ht ug Son

Husband-Wife

Wife-Husband

Karan

Prakash

ug Da

E

Husband – Wife

Sundar

Da

J

Brother

Wife – Husband

Kamala

er ht ug Da

Q

Raghu

Son

(139–143) :

Mother

133. (5) J is the brother of V. K is the son of V. Therefore, J is the uncle of K. 134. (4) K is husband of D. B is daughter of X. Q is wife of T. X is son-in-law of Q. T is maternal grandfather of K. Therefore, B is sister-in-law of D. 135. (4) Q is the wife of T. L is the wife of J. J is the son of Q Therefore, Q is the mother-inlaw of L. 136. (2) B is paternal aunt of W. B is daughter of X. X is father of B. Therefore, X is grandfather of W. 137. (1) G is niece of V. V is daughter of Q. X is son-in-law of Q. B is daughter of X. Therefore, G is cousin of B. 138. (3) C is the father of B, D and F. C is the husband of A. Therefore, A has three children.

R

149. (5) P is sister of F. F is mother of R. Therefore, P is aunt of R. 150. (2) A is father of P. P is wife of G. Therefore, A is father-in-law of G. 151. (5) F is daughter of A. H is brother of A. Therefore, F is niece of H. 152. (4) W is husband of F. P is sister of F. Therefore, W is brother-in-law of P. 153. (3) E is wife of A. H is brother of A. Therefore, E is sister-in-law of H. ppp

BPRE–159

SYMBOLS & NOTATIONS

7

SYMBOLS & NOTATIONS

QUESTIONS FROM 1999 TO 2010 ARE AVAILABLE ONLINE NATIONALISED BANKS & IBPS PO/MT/SO EXAMS Directions (1-5) : In the following questions, the symbols @, #, %, $ and © are used with the following meaning as illustrated below : (Punjab & Sind Bank PO Exam. 23.01.2011)

‘P # Q’ means ‘P is neither greater than nor equal to Q’. ‘P © Q’ means ‘P is neither equal to nor smaller than Q’. ‘P % Q’ means ‘P is neither smaller than nor greater than Q’. ‘P $ Q’ means ‘P is not smaller than Q’. ‘P @ Q’ means ‘P is not greater than Q’. Now in each of the following questions assuming the given statements to be true, find which of the three conclusions I, II and III given below them is/are definitely true and give your answer accordingly. 1. Statements : R @ D, D © W, B $ W Conclusions : I. W # R II. B © D III. W $ R (1) None is true (2) Only I is true (3) Only III is true (4) Only either I or III is true (5) All are true 2. Statements : H $ V, V % M, K © M Conclusions : I. K © V II. M @ H III. H © K (1) Only I and III are true (2) Only II and III are true (3) Only I and II are true (4) All are true (5) None of these

3. Statements : K # T, T $ B, B @ F Conclusions : I. F $ T II. K # B III. T $ F (1) None is true (2) Only I is true (3) Only I and II are true (4) Only II and III are true (5) All are true 4. Statements: Z # F, R @ F, D © R Conclusions : I. Z # R II. F # D III. D © Z (1) None is true (2) Only I is true (3) Only III is true (4) Only either I or III is true (5) All are true 5. Statements : M © R, R % D, D @ N Conclusions : I. M © N II. N $ R III. M © D (1) Only I and II are true (2) Only II and III are true (3) Only I and III are true (4) All are true (5) None of these Directions (6–10) : In the following questions, the symbols @, $, #, © and % are used with the following meaning as illustrated below : (IBPS Specialist Officer CWE Exam. 11.03.2012)

‘P $ Q’ means ‘P is not smaller than Q.’ ‘P © Q’ means ‘P is neither greater than nor equal to Q.’ ‘P # Q’ means ‘P is neither smaller than nor equal to Q.’ ‘P % Q’ means ‘P is not greater than Q.’

BPRE–160

‘P @ Q’ means ‘P is neither greater than nor smaller than Q.’ Now in each of the following questions assuming the given statements to be true, find which of the four conclusions I, II, III and IV given below them is/are definitely true and give your answer accordingly. 6. Statements : R # J, J $ D, D @ K, K % T Conclusions : I. T # D II. T @ D III. R # K IV. J $ T (1) Only either I or II is true (2) Only III is true (3) Only III and IV are true (4) Only either I or II and III are true (5) None of these 7. Statements : T % R, R $ M, M @ D, D © H Conclusions : I. D % R II. H # R III. T © M IV. T % D (1) Only I is true (2) Only I and IV are true (3) Only I and II are true (4) Only II and IV are true (5) None of these 8. Statements : M @ B, B # N, N $ R, R © K Conclusions : I. K # B II. R © B III. M $ R IV. N © M (1) Only I and III are true (2) Only I and II are true (3) Only II and IV are true (4) Only II, III and IV are true (5) None of these 9. Statements : F # H, H @ M, M © E, E $ J Conclusions : I. J © M II. E # H III. M © F IV. F # E

SYMBOLS & NOTATIONS (1) Only I and II are true (2) Only II and III are true (3) Only I, II and III are true (4) Only II, III and IV are true (5) None of these 10. Statements : D % A, A @ B, B © K, K % M Conclusions : I. B $ D II. K # A III. M # B IV. A © M (1) Only I, II and IV are true (2) Only I, II and III are true (3) Only II, III and IV are true (4) Only I, III and IV are true (5) All I, II, III and IV are true 11. Which of the following symbols should be placed in blank spaces respectively (in the same order from left to right) in order to complete the given expression in such a manner that both ‘W > R’ as well as ‘E > S’ definitely hold true? W__ E __ A __ R __ S (1) >, =, >, > (2) >, >, =, > (3) , =, > (4) , > (5) >, , = (IBPS RRBs Officer Scale–I & II CWE 12.09.2015)

12. Which of the following symbols should be placed in the blank spaces respectively (in the same order from left to right) in order to complete the given expression in such a manner that both ‘A > I’ as well as ‘V > T’ definitely hold true ? A_V_I_C_T (1) >, =, (2) >, , = (3) >, >, >, = (4) , > (5) >, S>T Conclusions : I. P > T II. T < Q 19. Statement : LP Conclusions : I. O < M II. P < N 20. Statement : A > B, B > C = D < E Conclusions : I. C < A II. D < B 21. Statement : H > J =K, K > L, L > T, T < V, Conclusions : I. K > T II. L < H 22. Statement : A < B = C, D > C = E Conclusions : I. E > A II. A < D 23. Which of the following symbols should replace the question mark in the given expression in

SYMBOLS & NOTATIONS order to make the expressions ‘J > M’ as well as ‘N < K’ definitely true? J>K?L=M>N (2) < (1) < (4) = (3) > (5) > (UCO Bank PO Exam. 30.01.2011)

24. In which of the following expressions will the expressions ‘D > B’ as well as ‘C > F’ be definitely true ? (1) A > B > C > D = F (2) A < B < C = D > F (3) A < B < C < D > F (4) A < B > C = D > F (5) None of these (UCO Bank PO Exam. 30.01.2011)

25. Which of the following symbols should replace the question mark in the given expression in order to make the expressions ‘I > L’ as well as ‘M > K’ definitely true? I>J>K?L (2) < (3) < (4) = (5) Either < or < (Bank Of Baroda PO Exam. 13.03.2011)

26. Which of the following symbols should be placed in the blank spaces respectively (in the same order from left to right) in order to complete the given expression in such a manner that ‘S > P’ definitely holds true but ‘S = P’ does not hold true? P ___ Q ___ R ___ S (2) , >, > (3) >, C (1) A > C (3) D > B (4) A > D (5) None is true 28. If the expressions ‘E < J < H > Z’, ‘H < Y’ and ‘E > F’ are true, which of the following conclusions will be definitely false? (1) F < Y (2) Y > E (2) F < H (4) J < Y (5) All are true

29. Which of the following symbols should replace the question mark in the given expression in order to make the expressions ‘K < H’ as well as ‘M > J’ definitely true? H>I=J ? K (3) < (4) Either < or < (5) = 30. In which of the following expressions will the expression ‘P > S’ be definitely false ? (1) P > Q > R = S (2) S < R < Q < P (3) R = P > Q > S (4) S > Q > R < P (5) S < Q < R < P 31. Which of the following symbols should be placed in the blank spaces respectively (in the same order from left to right) in order to complete the given expression in such a manner that ‘N < K’ definitely holds true ? K ___ L ___ M ___ N (2) (4) >, >, < (3) >, =, < (5) None of these Directions (32–36) : In these questions, relationship between different elements is shown in the statements. These statements are followed by two conclusions. (Indian Overseas Bank PO Exam. 22.05.2011)

Mark answer (1) if only conclusion I follows. Mark answer (2) if only conclusion II follows. Mark answer (3) if either conclusion I or II follows. Mark answer (4) if neither conclusion I nor II follows. Mark answer (5) if both conclusions I and II follow. 32. Statements : A > B = C; B < D < E Conclusions : I. D > A II. E > C 33. Statements : L > U > K; Z < U < R Conclusions : I. L > Z II. K < R 34. Statements : YR>I Conclusions : I. J > I II. Y < R

BPRE–162

35. Statements : V > K > M = N; M > S; T < K Conclusions : I. T < N II. V = S 36. Statements : F< X < A; R < X < E Conclusions : I. F < E II. R < F Directions (37-41) : In these questions, relationships between different elements is shown in the statements. These statements are followed by two conclusions. (IBPS RRBs Office Assistant CWE Exam. 09.09.2012)

Give answer (1) if only conclusion I follows. Give answer (2) if only conclusion II follows. Give answer (3) if either conclusion I or conclusion II follows. Give answer (4) if neither conclusion I nor conclusion II follows. Give answer (5) if both conclusions I and II follow. 37. Statements : G < F = L < J; J < K = H Conclusions : I. H = G II. G < H 38. Statements : P U > V > W; X < Y = W > Z Conclusions : I. Z > U II. W < T 40. Statements : K < L < M < N; M < O < P Conclusions : I. P > K II. N > O 41. Statements : B < A < C; A > D < E Conclusions : I. B < E II. C > E Directions (42–46) : In each of these questions, relationship between different elements is shown in the statements . The statements are followed by two conclusions. (IDBI Bank Officer Exam.16.09.2012)

Give answer (1) if only conclusion I is true.

SYMBOLS & NOTATIONS Give answer (2) if only conclusion II is true. Give answer (3) if either conclusion I or II is true. Give answer (4) if neither conclusion I or II is true. Give answer (5) if both conclusions I and II are true. 42. Statements : E=AF=C Conclusions : I. E < B II. C < D 43. Statements : P > T = U ≤ W; Q ≥ R ≥ W Conclusions : I. T = R II. U < R 44. Statements : P > T = U ≤ W; Q ≥ R ≥ W Conclusions : I. P ≥ R II. U > Q 45. Statements : R≥S>W=X;Y≤R=P Conclusions : I. P > X II. Y ≤ X 46. Statements : C = D ≥ E < F; G < I ≤ C Conclusions : I. D > G II. E < I Directions (47-48) : In these questions, relationship between different elements is shown in the statements. These statements are followed by two conclusions. (IBPS Specialist Officer CWE Exam.17.03.2013)

Mark answer (1)

If Only conclusion I follows. (2) Only conclusion II follows. (3) Either conclusion I or II follows. (4) Neither conclusion I nor II follows. (5) Both conclusions I and II follow. 47. Statement : E < F < G = H > S Conclusions : I. G > S II. F < H 48. Statement : P < Q < W = L Conclusions : I. L > P II. Q < L Directions (49-53) % In these questions relationship between different elements is shown in the statements. The statements are followed by two conclusions. (Indian Overseas Bank PO Online Exam, 01.09.2013)

Give answer (1) if only Conclusion I is true. Give answer (2) if only Conclusion II is true. Give answer (3) if either Conclusion I or II is true. Give answer (4) if neither Conclusion I nor II is true. Give answer (5) if both Conclusions I and II are true. (59-50) : Statements : R > T < M = Z; C > T > B 49. Conclusions I. Z > C II. B < Z 50. Conclusions I. B < C II. R > C (51-52) : Statements : P > Q > T; Q < R; T > A 51. Conclusions : I. A < Q II. A < P 52. Conclusions : I. T < P II. R > A 53. Statements : P > T = U < W; Q>R>W Conclusions I. T = R II. U < R Directions (54–58) : In these questions, relationship between different elements is shown in the statements. The statements are followed by two conclusions. (BOB Manipal School of Banking Officer Online Exam, 14.08.2014)

Give answer (1) if only Conclusion I is true. Give answer (2) if only Conclusion II is true. Give answer (3) if either Conclusion I or II is true. Give answer (4) if neither Conclusion I nor II is true. Give answer (5) if both Conclusions I and II are true. (54–55) : Statements R=QM=E;I R II. E < Z 55. Conclusions I. Q < E II. M > Z 56. Statements PM;R M II. F < M (57–58) : Statements V>M=T>X;RS 57. Conclusions I. R < V II. S < X 58. Conclusions I. X < R II. V > S Directions (59–64) : In these questions, relationships between different elements is shown in the statements. These statements are followed by two conclusions. (IDBI Bank Officer Exam, 22.08.2014)

Give answer (1) if only conclusion I follows. Give answer (2) if only conclusion II follows. Give answer (3) if either conclusion I or conclusion II follows. Give answer (4) if neither conclusion I nor conclusion II follows. Give answer (5) if both conclusions I and II follow. (199–200) : Statements : F>A I 60. Conclusions : I. I < H II. H > N 61. Statements : FK Conclusions : I. E > F II. M > U 62. Statements : SU>T T II. D < O (63–64) : Statements : N>J=M;S>B>R O II. S > R 64. Conclusions : I. N = M II. N > M Directions (65-69) : In these questions, relationship between different elements is shown in the statements. The statements are followed by two conclusions. (SIDBI Officer Exam, 03.09.2014)

SYMBOLS & NOTATIONS Give answer (1) if only Conclusion I is true. Give answer (2) if only Conclusion II is true. Give answer (3) if either Conclusion I or II is true. Give answer (4) if neither Conclusion I nor II is true. Give answer (5) if both Conclusions I and II are true. 65. Statements : S = T < A = N > D; B E II. C = Z (68-69) : Statements : T > R = E < Q < L; CZ 68. Conclusions : I. R > C II. L > Z 69. Conclusions : I. L > C II. Z < T Directions (70–74) : In these questions, relationship between different elements is shown in the statements. The statements are followed by two conclusions. (IBPS RRBs Officer Scale-I CWE, 06.09.2014)

Give answer (1) if only Conclusion I is true. Give answer (2) if only Conclusion II is true. Give answer (3) if either Conclusion I or II is true. Give answer (4) if neither Conclusion I nor II is true. Give answer (5) if both Conclusions I and II are true. 70. Statement PE Conclusions I. K < L II. P < E (71-72): Statements P>R=A D (73-74) : Statements C > R > A = S < H; R < P < Q 73. Conclusions I. C > S II. P < C 74. Conclusions I. H < R II. R < Q Directions (75–80) : In these question relationships between different elements is shown in the statements. These statements are followed by four Conclusions numbered I, II, III and IV. Study the statements and Conclusions carefully and select the correct answer : (IBPS Bank PO/MT CWE-IV 18.10.2014)

75. Statement : Y>R>S=T H > T < C; Y>T>N Conclusions : I. H > Y II. S > C III. C = Y IV. C > N (1) Only Conclusions I and IV are true. (2) Only Conclusion III is true. (3) Only Conclusions II and IV are true. (4) Only Conclusion IV is true. (5) Only Conclusions I and II are true. 77. Statement : K>H Y II. L < A III. H < A IV. K > R

BPRE–164

(1) Only Conclusions I, II and III are true. (2) Only Conclusions II and IV are true. (3) None of the Conclusion is true (4) Only Conclusions II and III are true. (5) Only Conclusions II, III and IV are true. 78. Statements : D > M > U > E; NJ Conclusions : I. D > E II. E < J III. M > N IV. D > J (1) Only Conclusions I, II and III are true. (2) Only Conclusions II, III and IV are true. (3) Only Conclusions I, III and IV are true. (4) None of the Conclusions is true. (5) All the Conclusions I, II, III and IV are true. 79. Statement : P>L=U>CS Conclusions : I. P > C II. K < L III. S < U IV. C < L (1) Only Conclusions I and II are true. (2) Only Conclusions III and IV are true. (3) Only Conclusions I and IV are true. (4) Only Conclusions I, II and IV are true. (5) Only Conclusions II, III and IV are true. 80. Statements : K > G = M < T; U U IV. G < U (1) Only Conclusions I and III are true. (2) Only Conclusions III and IV are true.

SYMBOLS & NOTATIONS (3) Only Conclusions I, III and IV are true. (4) Only Conclusions III and IV are true. (5) Only Conclusions II and III are true. Directions (81–86) : In these questions, relationship between different elements has been shown in the statements. The statements are followed by two conclusions numbered I and II. Study the conclusions based on the given statements and decide which of the following conclusions is definitely true ? (Bank of Baroda Junior Management Grade/Scale-I Exam, 18.04.2015)

Give answer (1) if either Conclusion I or Conclusion II is true Give answer (2) if both Conclusion I and Conclusion II are true Give answer (3) if only Conclusion II is true Give answer (4) if neither Conclusion I nor Conclusion II is true Give answer (5) if only Conclusion I is true (81-82) : Statement : M>O>L>T=E>D 81. Conclusions : I. D < O II. M > E 82. Conclusions : I. T < O II. T = O 83. Statement : BD 87. Conclusions : I. D < O II. M > E 88. Conclusions : I. T < O II. T = O 89. Statement : BK=H>N>AM>T Conclusions : I. K > T II. R > M 95. Statements : S R; L > Q 96. Conclusions : I. P > S II. I > R 97. Conclusions : I. L < R II. E > Q (98–99) : Statements : G > R > E = A < T < S; D S 99. Conclusions : I. J > E II. J = E

SYMBOLS & NOTATIONS 100. Statements : A>B>C S; C > A 101. Conclusions I. L > K II. P < S 102. Conclusions I. C > P II. P = C 103. Statements J>U>N=K M > N; K E > D; Q>N D II. A < D (108–109) : Statements P < U = N < C > H > S; K > C 108. Conclusions I. P < C II. U > H 109. Conclusions I. K > U II. U = K

111. Conclusions : I. A > F II. R < K 112. Conclusions : I. Y > K II. F < Y (113-114) : Statements : BS; D>C C II. E = C 114. Conclusions : I. D > B II. E > S 115. Statement : A>B>CS>M I Directions (111-115) : In each of the following questions, a relationship between different elements is shown in the statements. The statements are followed by two Conclusions numbered I and II. Study the Conclusions based on the given statements and select the appropriate answer. (IBPS Bank PO/MT CWE–V (Preliminary) 10.10.2015 Ist Sitting)

Give answer (1) if neither Conclusion I nor Conclusion II is true. Give answer (2) if either Conclusion I or Conclusion II is true Give answer (3) if only Conclusion I is true. Give answer (4) if only Conclusion II it true Give answer (5) if both the Conclusion I and Conclusion II are true (111-112) : Statements : FK; Y>E

BPRE–166

Give answer (1) if both the Conclusion I and Conclusion II are true Give answer (2) if either Conclusion I or Conclusion II is true Give answer (3) if neither Conclusion I nor Conclusion II is true Give answer (4) if only Conclusion I is true Give answer (5) if only Conclusion II is true (116–117) : Statements C < R < E < A = M; Y>E

%

116. Conclusions : I.

M>R

II. Y > A 117. Conclusions : I. C = Y II. C < Y (118–120) : Statements % B < L < A = M > E > S; L>W>J 118. Conclusions : I. L < S II. E > W

SYMBOLS & NOTATIONS 119. Conclusions : I. J < M II. J = M 120. Statement % C>H>O>K=E D II. E < C Directions (121–123) : In each of the following questions, a relationship between different elements is shown in the statements. The statements are followed by two Conclusions numbered I and II. Study the Conclusion(s) based on the given statements and select the appropriate answer. (IBPS RRBs Officer Scale–I & II CWE 13.09.2015)

Give answer (1) if both the Conclusion I and Conclusion II are true Give answer (2) if neither Conclusion I nor Conclusion II is true Give answer (3) if only Conclusion I is true Give answer (4) if either Conclusion I or Conclusion II is true Give answer (5) if only Conclusion II is true 121. Statements : S < P < U > N; U > B; L < S Conclusions : I. B > P II. L < U 122. Statements : A > G > O > N < Y; O > S > R Conclusions : I. R < A II. Y > S 123. Statements : M > O > C > K = E < D; J > C; O < Z Conclusions : I. J > E II. K < Z 124. Which of the given expression is definitely true if the expression ‘Q > U < I < E = T > S’ is definitely true ? (2) U < T (1) Q > T (3) E > S (4) E > Q (5) I > S (IBPS RRBs Officer Scale–I & II CWE 13.09.2015)

Directions (125-129) : In each of the following questions, relationship between different elements is shown in the statement. These statements are

followed by two conclusions numbered I and II. Read both the statements and select the appropriate answer. (IBPS Bank PO/MT CWE–V Main Exam. 31.10.2015)

Give answer (1) if both the Conclusions I and II are true Give answer (2) if either Conclusion I or Conclusion II is true Give answer (3) if neither Conclusion I nor Conclusion II is true Give answer (4) if only Conclusion I is true Give answer (5) if only Conclusion II is true (125-126) : Statements % V > M = T > X; R < T > S 125. Conclusions : I. R < V II. S < X 126. Conclusions : I. X < R II. V > S 127. Statements % P < E < R > F; E > M; R < T Conclusions : I. T > M II. F < M 128. Statements % R = Q < I > M = E; I < Z Conclusions : I. Q < E II. M > Z 129. Statements % N = D < H > R; V > H < J Conclusions : I. V > D II. R < J Directions (130-134) : In each of the following questions, a relationship between different elements is shown in statements. The statements are followed by two Conclusions numbered I and II. Study the Conclusions based on the given statements and select the appropriate answer : (IBPS Specialist Officer (IT) CWE 14.02.2016)

Give answer (1) if only Conclusion I is true Give answer (2) if either Conclusion I or Conclusion II is true Give answer (3) if only Conclusion II is true Give answer (4) if neither Conclusion I nor Conclusion II is true Give answer (5) if both the Conclusion I and Conclusion II are true

BPRE–167

130. Statements L < Q < R = S; R > H > P Conclusions I. P < S II. L < P 131. Statements W > A = S > U < K; U > Y Conclusions I. K > W II. Y < W 132. Statements Y < U < W > S; M > W > D Conclusions I. Y < M II. S < D (133–134) : Statements S < P = Q < J; Q < Y; P < L 133. Conclusions I. J > L II. S < J 134. Conclusions I. Y > S II. S = Y Directons (135–140) : In each of the following questions, relationship between different elements is shown in the statements. The statements are followed by two Conclusions numbered I and II. Study the Conclusions based on the given statements and mark the appropriate answer : (SIDBI Officer Online Exam.24.02.2016)

Give answer (1) if both the Conclusion I and Conclusion II are true Give answer (2) if either Conclusion I or Conclusion II is true Give answer (3) if neither Conclusion I nor Conclusion II is true Give answer (4) if only Conclusion I is true Give answer (5) if only Conclusion II is true 135. Statements AS Conclusions I. A < S II. A = S (136–137) : Statements C > R < E = T; R < Y; U > E 136. Conclusions I. C > Y II. U > Y 137. Conclusions I. U > R II. T < U

SYMBOLS & NOTATIONS (138–139) : Statements P > R = B < S; CZ 138. Conclusions I. Z = P II. S < Z 139. Conclusions I. C < S II. C = S 140. Statements D < Q < N > W; Q > P Conclusions I. P = D II. N > P Directions (141–144) : In these questions, relationship between different elements is shown in the statements. The statements are followed by two conclusions numbered I and II. Study the conclusions based on the given statements and select the appropriate answer. (Bank of Baroda Exam, 25.09.2016)

Give answer (1) if only concluson I is true Give answer (2) if both conclusion I and II are true Give answer (3) if either conclusion I or II is true Give answer (4) if only conclusion II is true Give answer (5) if neither conclusion I nor II is true (141–142) : Statements : G < A < M = E > S; Q > E < O; D D II. M < O 143. Statements : A < B = C < D < E > F; XZ Conclusions : I. A > Z II. X < F 144. Statements : L = A < M > P; A > J = S < R Conclusions : I. M > J II. R > L

145. Which of the following expressions will be definitely false if the given expressions ‘D < I < F = C > U > L’; ‘T > C’ are definitely true? (1) L < F (2) D < T (4) C < D (3) C > I (5) T > L (Bank of Baroda Exam, 25.09.2016)

146. Which of the following symbols should replace the question mark (?) in the given expression in order to make the expressions ‘V > Y’ as well as ‘I < L’ definitely true? L>E=V?I>N=G>Y (1) < (2) Either ‘>’ or ‘=’ (3) Either ‘=’ or ‘ (5) < (Bank of Baroda Exam, 25.09.2016)

Directions (147–151) : In these questions, relationship between different elements is shown in the statement. The statements are followed by two conclusions. Study the conclusions based on the given statement and select the appropriate answer. (IBPS Bank PO/MT CWE (Pre Exam), 16.10.2016 (First Sitting))

Give answer (1) if either Conclusion I or Conclusion II is true Give answer (2) if both the Conclusion I and Conclusion II are true Give answer (3) if only Conclusion I is true Give answer (4) if only Conclusion II is true Give answer (5) if neither Conclusion I nor Conclusion II is true (147-148) : Statements L < A = M < P; A < C < T; M>O>R 147. Conclusions I. O < P II. C > L 148. Conclusions I. T < M II. A > R 149. Statements D > U = S > T; O = P < S Conclusions I. D > P II. P = D (150–151) : Statements T > I > L > E: I < N < B; N>S>D

BPRE–168

150. I. II. 151. I. II.

Conclusions N>E T E > R; L > Q 152. Conclusions : I. P > S II. I > R 153. Conclusions : I. L < R II. E > Q (154–155) : Statements: G > R < E = A < T < S; D < A < J 154. Conclusions : I. T > D II. R > S 155. Conclusions : I. J > R II. J = R 156. Statements : A>B>C R II. N > F (158–159) : Statements : Q < U = E > N; R > A > E; T Q (160–161) : Statements : R < A > C > E; A > N > T; C R II. T < E Directions (162–166) : In these questions, relationship between different elements is shown in the statement(s). The statement(s) are followed by two Conclusions numbered I and II. Study the Conclusions based on the given statement(s) and select the appropriate answer. (IBPS RRBs Officer CWE (Pre.) Exam, 14.11.2016 (Shift-I))

Give answer (1) if either Conclusion I or Conclusion II is true Give answer (2) if neither Conclusion I nor Conclusion II is true Give answer (3) if only Conclusion I is true Give answer (4) if both Conclusions I and II are true Give answer (5) if only Conclusion II is true 162. Statements : P < E < T < R; T > K

Conclusions : I. K > P II. R > K 163. Statements : X < W; A > C > H = W Conclusions : I. C > X II. A > W 164. Statements : J W Directions (167–171) : In these questions, relationship between different elements is shown in the statement(s). The statement(s) are followed by two conclusions numbered I and II. Study the conclusions based on the given statements and select the appropriate answer. (IBPS RRBs Officer CWE (Pre.) Exam, 14.11.2016 (Shift-II))

Give answer (1) if neither Conclusion I nor Conclusion II is true Give answer (2) if either Conclusion I or Conclusion II is true Give answer (3) if both the Conclusion I and Conclusion II are true Give answer (4) if only Conclusion I is true Give answer (5) if only Conclusion II is true (167–168) : Statements : M < A > N; E < A < G 167. Conclusions : I. M < E II. G > N 168. Conclusions : I. N < E II. G > M (169–170) : Statements : L > Y > A < R; P < A 169. Conclusions : I. P < L II. R > P 170. Conclusions : I. L > P II. A > R

BPRE–169

171. Statements : W>Q=U>I C Directions (172–176) : In these questions, relationship between different elements is shown in the statement(s). The statement(s) are followed by two Conclusions numbered I and II. Study the Conclusions based on the given statements and select the appropriate answer. (IBPS RRBs Officer CWE (Pre.) Exam, 14.11.2016 (Shift-III))

Give answer (1) if both the Conclusion I and Conclusion II are true Give answer (2) if either Conclusion I or Conclusion II is true Give answer (3) if neither Conclusion I nor Conclusion II is true Give answer (4) if only Conclusion I is true Give answer (5) if only Conclusion II is true 172. Statements : T = V < R < B; T < D Conclusions : I. D > R II. B < T (173–174) : Statements : R < O > E; Y > O > C 173. Conclusions : I. Y > R II. E < C 174. Conclusions : I. Y = E II. Y > E 175. Statement : P > R = S > Q; M < R Conclusions : I. M < P II. Q < M 176. Statements : HI>N=C>R Conclusions : I. C < H II. M > R Directions (177–179) : In the following questions, the symbols ≠, , % and @ are used with the following meaning : (IBPS Bank PO/MT CWE (Main) Exam, 18.11.2016

P ≠ Q means ‘P is neither greater nor equal to Q’. ● P  Q means ‘P is neither equal to nor smaller than Q’.



SYMBOLS & NOTATIONS P % Q means ‘P is neither smaller nor greater than Q’. ● P $ Q means ‘P is not smaller than Q’. ● P @ Q means ‘P is not greater than Q’. 177. Statements : Z ≠ F, R @ F, D © R Conclusions : I. Z ≠ R II. F ≠ D III. D % Z (1) Only Conclusions I and III are true. (2) Only Conclusions II and III are true. (3) Only Conclusions I is true. (4) All are true (5) None of these 178. Statements : R @ D; D © W; B $ W Conclusion : I. W ≠ R II. B © D III. W $ R (1) Only Conclusions I, II and III are true. (2) Only Conclusions I and III are true. (3) None is true. (4) Only Conclusions II and III are true. (5) None of these 179. Statements : H $ V, V % M, K © M Conclusions : I. K © V II. M @ H III. H © K (1) Only Conclusions II and III are true. (2) Only Conclusions I and II are true. (3) Only Conclusion II is true. (4) All are true (5) None of these Directions (180–184) : In these questions, relationship between different elements is shown in the statements. The statements are followed by two Conclusions numbered I and II. Read the Conclusions based on the statements and select the appropriate answer : ●

(Indian Bank PO (Pre.) Exam, 21.01.2017 (Ist Sitting))

Give answer (1) if either Conclusion I or Conclusion II is true Give answer (2) if neither Conclusion I nor Conclusion II is true

Give answer (3) if both the Conclusion I and Conclusion II are true Give answer (4) if only Conclusion I is true Give answer (5) if only Conclusion II is true (180–181) : Statements: T > R > A > C < K; N < E < C > S > D 180. Conclusions : I. S > A II. K < E 181. Conclusions : I. D < T II. N < R 182. Statements : S = L < U = M; Z > R>C=M Conclusions : I. Z > S II. S = Z (183–184) : Statements : Y < E < L = O > W; S > P > L > I < T 183. Conclusions : I. E < P II. I > E 184. Conclusions : I. Y < T II. S > W 185. Which of the following expressions will be definitely false if the given expression ‘J > N > U = Y < R < B < O < F’ is definitely true ? (1) F > B (3) U > B (5) U < F

(2) Y < N (4) Y < J

(Indian Bank PO (Pre.) Exam, 21.01.2017 (2nd Sitting))

186. In which of the given expressions does the expression S < H and T < I definitely hold true ? (1) (2) (3) (4) (5)

S S S S S

> < < <
= < >

T, T, T, T, T,

P P P P P

> > < <
=

H, H, H, H, H,

H H H H H

I K>X T < A; E < T ; R > D = T Conclusions : I. E < A II. R > S 189. Statement % R < A > B; C > A; F < B Conclusions : I. C > R II. F < A 190. Statement % L>K>XC>DE Conclusions : I. A > E II. A E Directions (192–194) : In each of the following questions, a statement is given followed by two sets of conclusions numbered I and II. These statements show relationship between different elements. You have to assume the statement to be true and then decide which of the given conclusions logically follows from the information given in the statement. (IBPS SO (Agriculture) Exam, 29.01.2017)

Give answer (1) if either Conclusion I or Conclusion II is true Give answer (2) if only Conclusions I is true Give answer (3) if both the Conclusion I and Conclusion II are true

SYMBOLS & NOTATIONS Give answer (4) if only Conclusion II is true Give answer (5) if neither Conclusion I nor Conclusion II is true (192–193) : Statements : D < W > U > I; Y < U; Y > O 192. Conclusions : II. O > U I. I < D 193. Conclusions : I. O < W II. I < Y 194. Statements : N > E = R < X; W < R; S > X Conclusions : I. S > N II. W < N 195. In which of the following expressions does the expression ‘H > F’ hold definitely true? (1) (2) (3) (4) (5)

E>F>R>B= C>H N>H>A=CF H>R>M=PF F D; X < R < Z 199. Conclusions : I.

P N; Z > A = U Conclusions : I. R > N

II. G < Z

Give answer (2) if only Conclusion II is true

207. Conclusions :

Give answer (3) if both the Conclusion I and Conclusion II are true

208. Statements :

Give answer (4) if either Conclusion I or Conclusion II is true Give answer (5) if neither Conclusion I nor Conclusion II is true 201. Statements :

II. T > L

(202–203) : Statements : P < I = N < K > S; X > K < Y II. P = Y

203. Conclusions : II. S < X

204. Statements : F > W = S < C; Y < W Conclusions : I. C > Y

II. F > C

205. Statements : K=ME Conclusions : I. A < E

Conclusions : I. W > K

II. E > U

209. Statements : Conclusions :

II. Z > K

BPRE–171

II. L > Z

210. Statements : R>B>F>T>N=W Conclusions : I. F > W

202. Conclusions :

I. I < S

W > A = L > E; K < L < U

I. J > L

Conclusions :

I. P < Y

II. U = N

J > X > E = Z; L > E

A < N = K > L; N < O = T I. O > A

I. U > N

II. R < T

Directions (211–215) : In each of the following questions, relationship between different elements is shown in the statements. The statements is followed by two Conclusions numbered I and II. Study the Conclusions based on the given statement and select the appropriate answer. (IBPS RRBs Officer CWE (Prelim Exam) 16.09.2017)

Give answer (1) if both the Conclusion I and Conclusion II are true Give answer (2) if neither Conclusion I nor Conclusion II is true

SYMBOLS & NOTATIONS Give answer (3) if either Conclusion I or Conclusion II is true Give answer (4) if only Conclusion I is true Give answer (5) if only Conclusion II is true 211. Statements : D > C < G > H; B > C Conclusions : I. H > B II B < D 212. Statements : NA Conclusions : I. C > A II. N < D (213–214) : Statements : S = T < R < A > Y; Z > P = R 213. Conclusions : I. Z > Y II. S < Z 214. Conclusions : I. P < A II. P = A 215. Statements : P > A > C = E; K < C < U Conclusions : I. E < U

II. P > K

Directions (216–217) : In these questions, relationship between different elements is shown in the statements. The statements are followed by two Conclusions numbered I and II. Study the conclusions based on the given statements and select the appropriate answer. (IBPS Bank PO/MT CWE-VII (Prelim Exam) 14.10.2017)

Give answer (1) if neither Conclusion I nor Conclusion II is true Give answer (2) if only Conclusion I is true Give answer (3) if either Conclusion I or Conclusion II is true Give answer (4) if both the Conclusion I and Conclusion II are true Give answer (5) if only Conclusion II is true 216. Statements B < O = L < D; P > C > A = L Conclusions I. P = B II. B < P 217. Statements P > O < L < E; S < O > A = K Conclusions I. E < S II. K < P

Directions (218–222) : In each of the following questions, relationship between different elements is shown in the statements. The statements are followed by two Conclusions numbered I and II. Study the Conclusions based on the given statement and select the appropriate answer. (IBPS RRBs Officer CWE (Main Exam) 05.11.2017)

Give answer (1) if either Conclusion I or Conclusion II is true Give answer (2) if neither Conclusion I nor Conclusion II is true Give answer (3) if both the Conclusion I and Conclusion II are true Give answer (4) if only Conclusion I is true Give answer (5) if only Conlcusion II is ture (218–219) : Statements : T > R > A > C < K; ND 218. Conclusions : I. S > A II. K < E 219. Conclusions : I. D < T II. N < R 220. Statements : S = L < U = M; Z > R > C = M Conclusions : I. Z > S II. S = Z (221–222) : Statements : Y < E < = O > W; S>P>L>I E 222. Conclusions : II. S > Ww. I. Y < T Directions (223–227) : In each of the following questions, relationship between different elements is shown in the statements. The statements are followed by two Conclusions numbered I and II. Study the conclusions based on the given statements and select the appropriate answer. (IBPS SO (IT Officer) CWE (Prelim Exam) 30.12.2017)

Give answer (1) if both the Conclusion I and Conclusion II are true Give answer (2) if either Conclusion I or Conclusion II is true Give answer (3) if only Conclusion I is true

BPRE–172

Give answer (4) if only Conclusion II is true Give answer (5) if neither Conclusion I nor Conclusion II is true 223. Statement : P>R>O=S S II. O < Q 224. Statement : RC E I. C < V 226. Statements : C < D < L < F > G; K > L > M Conclusions : I. C < M II. K > G 227. Statements : J < L < P < E; K > P > M Conclusions : I. M > E II. J < K Directions (228–230) : In each of the following questions, relationship between different elements is shown in the statements. The statements are followed by two Conclusions numbered I and II. Study the Conclusions based on the given statement and select the appropriate answer. (IBPS SO (Law Officer) CWE (Prelim Exam) 31.12.2017)

Give answer (1) if either Conclusion I or Conclusion II is true Give answer (2) if only Conclusion I is true Give answer (3) if only Conlcusion II is ture Give answer (4) if both the Conclusion I and Conclusion II are true Give answer (5) if neither Conclusion I nor Conclusion II is true 228. Statements : B < O > C > Z; Conclusions : I. C < V 229. Statements : G > O < A < L; Conclusions : I. L > S

T>O Z SM=E II. E < G

SYMBOLS & NOTATIONS 230. Statements : H < A = F < T; J = A > L > E Conclusions : I. E > H II. L < T 231. What should come in place of δ and # respectively in the expression R > O > U > M δ Y; G > U # F < E that the expression E > Y definitely holds true? (1) >, = (3) >, < (5) >,
,
M > A < Y < G; Q>A>D>U>Z 232. Conclusions : I. M > U II. M = U 233. Conclusions : I. S > Q II. Z < G (234–235) : Statements : L W Directions (236–240) : Study the following arrangement carefully and answer the questions given below : (IDBI Bank PO Exam 29.04.2018)

R 4 3 % M@ K E F 5 A # J N 1 8 U ©D B P 6 I W 7 δ Q * Z

236. If all the symbols are dropped from the above arrangement, which of the following will be fourth to the left of ninth from the left end? (1) E

(2) K

(3) M

(4) 3

(5) None of these 237. If all the numbers are dropped from the above arrangement, which of the following will be seventh to the right of eighteenth from the right end? (1) J

(2) #

(3) U

(4) N

(5) None of these 238. How many such consonants are there in the above arrangement, each of which immediately preceded by a symbol and immediately followed by a letter? (1) None

(2) One

(3) Two

(4) Three

(5) More than three 239. Four of the following five are alike in a certain way based on their positions in the above arrangement and so they form a group. Which is the one that does not belong to that group? (1) J A 1

(2) 3 R %

(3) 8 © 1

(4) # N A

(5) δ W Q 240. How many such numbers are there in the above arrangement, each of which is immediately followed by a letter but not immediately preceded by a symbol? (1) None

(2) One

(3) Two (4) More than three (5) Three Directions (241–245) : In each of the following questions, relationship between different elements is shown in the statements. The statements are followed by two Conclusions numbered I and II. Study the conclusions based on the given statements and select the appropriate answer. (IBPS Bank PO/MT CWE-VII (Prelim Exam) 15.10.2017) & (IBPS Bank PO/MT CWE (Prelim Exam) 21.10.2018)

BPRE–173

Give answer (1) if either Conclusion I or Conclusion II is true Give answer (2) if neither Conclusion I nor Conclusion II is true Give answer (3) if both the Conclusion I and Conclusion II are true Give answer (4) if only Conclusion I is true Give answer (5) if only Conclusion II is true (241–242) : Statements L > I = N > P; I > R > K; N N II. I < Z (243–244) : Statements S > A = N > D; A > L > E; M M II. A = M 245. Statements : P > V > R < E < Y; G>E>N Conclusions I. P > N II. V > Y Directions (246–249) : In each of the following questions, relationship between different elements is shown in the statements. The statements are followed by two Conclusions numbered I and II. Study the Conclusions based on the given statement and select the appropriate answer. IBPS RRBs OFFICER CWE (PRELIM EXAM), 11.02.2018

Give answer (1) if both the Conclusion I and Conclusion II are true Give answer (2) if either Conclusion I or Conclusion II is true Give answer (3) if only Conclusion I is true Give answer (4) if only Conclusion II is true Give answer (5) if neither Conclusion I nor Conclusion II is true (246–247) : Statements : F > I < J < N; Z < I > T = C

SYMBOLS & NOTATIONS 246. Conclusions : I. II. 247. I.

N J > K = M 248. Conclusions : I. G > J II. G < F 249. Conclusions : I. K > S II. E < F Directions (250–254) : Study the following information carefully and answer the questions given below: There is a 5 × 5 matrix which can produce signals which in turn help in illumination of some bulbs. The rows of the matrix are denoted by @, %, &, £ and ¥ from bottom to top in the same order and the columns are denoted by A, B, C, D and E from left to right. ¥ row contains numbers which are consecutive multiples of ‘9’ starting from ‘36’ £ row contains numbers which are consecutive multiples of ‘13’ & row contains numbers which are consecutive multiples of ‘8’ starting from ‘32’ % row contains numbers which are consecutive multiples of ‘7’ @ row contains numbers which are consecutive multiples of ‘11’ Note: If the starting number of a multiple is not given; then start the row from the same number. Example: If a row contains numbers which are consecutive multiples of ‘3’, then the row might be as follows:

3

6

9 12 15

The matrix helps in producing signals which is a string of numbers. There are four bulbs – P, Q, R and S. Based on the outcomes of the string one of the bulb blinks. Conditions for blink: I. If outcome of the string is below 80, the bulb Q blinks.

II. If outcome of the string is between 90 and 100, the bulb S blinks. III. If outcome of the string is between 101 and 120, then bulb P blinks. IV. If outcome of the string is between 121 and 200, the bulb R blinks. V. If none of the above condition follows, no bulb blinks. Four outcomes of the string: I. If all the numbers of the string are even numbers then, outcome is obtained by multiplying unit digits of all the two digit numbers. II. If a prime number is followed by another prime number then, outcome is obtained by sum of all the two digits numbers. III. If a prime number is preceded by a perfect square then, outcome is obtained by multiplying tenth place digits of all the numbers. IV.If there is only one prime number in the string, then outcome is obtained by addition of remaining numbers leaving the prime number (IBPS Bank PO/MT CWE (Main Exam) 18.11.2018)

250. If the string is %B £D, @A &C, then which of the following bulb blinks? (1) P (2) Q (3) R (4) S (5) None of these 251. If string is @A £A, ¥B, &D. then which of the following bulb blinks? (1) P (2) Q (3) R (4) S (5) None of these 252. If string is ¥E @D, %B £B, then which of the following bulb blinks? (1) Q (2) R (3) S (4) P (5) None of these 253. If string is &E ¥D, £A @A, then which of the following bulb blinks? (1) P (2) Q

BPRE–174

(3) S

(4) R

(5) None of these 254. If string is @E &D, £E £A, then which of the following bulb blinks? (1) P (2) Q (3) R (4) S (5) None of these Directions (255–257) : In each of the following questions, relationship between different elements has been shown in the statements. The statements are followed by two Conclusions numbered I and II. Study the Conclusions and decide which of the Conclusions is definitely true. Give answer (1) if either Conclusion I or Conclusion II is true Give answer (2) if only Conclusion I is true Give answer (3) if both the Conclusion I and Conclusion II are true Give answer (4) if only Conclusion II is true Give answer (5) if neither Conclusons I nor Conclusion II is true (Canara Bank PO Exam, 23.12.2018)

255. Statements W > V < T < P > Q; T > K > J > N; K > Z > H = D Conclusions : I. D < P II. T > D 256. Statements H < B < E < V; E = H > K > N; N N 257. Statements R > K < N > J = H; K < D < T = W; L > F > H = P Conclusions : I. N > P II. K < W Directions (258–262) : In each of the following questions, relationship between different elements is shown in the statements. The statements are followed by two Conclusions numbered I and II. Study the Conclusions based on the given statement and select the appropriate answer.

SYMBOLS & NOTATIONS Give answer (1) if only Conclusion I is true Give answer (2) if only Conclusion II is true Give answer (3) if either Conclusion I or Conclusion II is true Give answer (4) if neither Conclusion I nor Conclusion II is true Give answer (5) if both the Conclusion I and Conclusion II are true (IBPS Specialist Officer CWE (Prelim Exam) 31.01.2019)

258. Statements : W > U > P = J < K < Q < V; S>U>T Conclusions : I. P < V II. V > T 259. Statements : D > F > E = J > K < C < B; E D II. H > G 261. Statements : K = F < G < D < E > M; W > K; E > X Conclusions : I. G < X II. M > F 262. Statements : Q = S < N < P < R > M; W > Q; R > X Conclusions : I. W > P II. R > Q Directions (263–267) : In the following questions, relationships between different elements are given in different statements. The statements are followed by two Conclusions numbered I and II. Choose the correct answer accordingly : Give answer (1) if only Conclusion I is true Give answer (2) if only Conclusion II is true Give answer (3) if either Conclusion I or Conclusion II is true

Give answer (4) if neither Conclusion I nor Conclusion II is true Give answer (5) if both the Conclusion I and Conclusion II are true (IBPS RRBs Officer CWE Prelim Exam, 17.08.2019)

263. Statements : P = E; Q > P; V < Q Conclusions :

Conclusions : I. G < C II. H = A 269. Statements : M Z II. Y = Z

I. A > C

271. Statements :

II. A = C (265-266) : Statements : M > A > B = Q < P < J < Y; Z>A>X 265. Conclusions : I. B < Y

D < E = F > G > H; K > F Conclusions : I. D < H II. K > G 272. Statements : R > I = N > P; Y > R > K;

II. X > J 266. Conclusions : I. Z = Q II. Z > Q 267. Statements : G < R = A < S; T < R Conclusions : I. G < S II. S > T Directions (268–272) : In the following questions, relationships between different elements are given in the different statements. The statements are followed by two Conclusions numbered I and II. Choose the correct answer accordingly. Give answer (1) if only Conclusion I is true Give answer (2) if only Conclusion II is true Give answer (3) if either Conclusion I or Conclusion II is true Give answer (4) if neither Conclusion I nor Conclusion II is true Give answer (5) if both the Conclusion I and Conclusion II are true (IBPS Bank PO/MT CWE Prelim Exam, 12.10.2019)

268. Statements : F < G < D; D < H > C; F=C M > D Conclusions : I.

W>D

II. J > T

SYMBOLS & NOTATIONS 274. Statements : (1) B < D < E (2) D = F > G (3) A = B > C Conclusions : I. C < G II. F > A 275. Statements : (1) Y < T = W (2) D = Y > G (3) X > M < D Conclusions : I. X = Y is a possibility II. D < W is not possible 276. Statements : (1) D = M > Y (2) B < D < E (3) N = B > X Conclusions : I. X > Y II. M > N Directions (277–281) : In these questions, certain relationship between different elements is shown in the statements. The statements are followed by two Conclusions numbered I and II. Study the Conclusions based on the given statements and select the appropriate answer: Give answer (1) if only Conclusion I is true Give answer (2) if only Conclusion II is true Give answer (3) if either Conclusion I or Conclusion II is true Give answer (4) if neither Conclusion I nor Conclusion II is true Give answer (5) if both the Conclusion I and Conclusion II are true (IBPS RRBs Officer Scale-I CWE Main Exam, 13.10.2019)

277. Statements : K > U > D < R = M < B; X=A>K Conclusions : I. X > D II. R > A 278. Statements : L = A > Q > B = V > X; P>B=R Conclusions : I. P > L II. R > X

279. Statements : Y > H < E = Q < L = K; H>B=M Conclusions : I. M < Q II. B < Q 280. Statements : P < X > Q = D > H > Z; C>X=M Conclusions : I. M > H II. C > H 281. Statements : B < A < T = E > D > F; Y=X>T Conclusions : I. Y > D II. X < F Directions (282–285) : In each of the questions below are shown relationship between different elements in the statements followed by two Conclusions numbered I and II. Study the Conclusions based on the select appropriate answer. Give answer (1) if only Conclusion I is true Given answer (2) if only Conclusion II is true Give answer (3) if either Conclusion I or Conclusion II is true Give answer (4) if neither Conclusion I nor Conclusion II is true Give answer (5) if both the Conclusion I and Conclusion II are true (Indian Bank Specialist Officer SO Exam, 08.03.2020)

282. Statements : T > R > U > V; M > P = T; U < Q Conclusions : I. R < M II. Q < T 283. Statements : J > A = N > F; R < Y < N Conclusions : I. J > Y II. J = Y 284. Statements : P>M>D>S;DB

BPRE–176

Conclusions : I. P > R II. B < D 285. Statements : A = U < S; G > S > T Conclusions : I. A < T II. G >U Directions (286–290) : In each of the following questions, certain relationships between different elements are shown in the statements. The statements are followed by two Conclusions numbered I and II. Read the Conclusions carefully and decide which of the Conclusions is/are definitely true. Give answer (1) if only Conclusion I is true Given answer (2) if only Conclusion II is true Give answer (3) if either Conclusion I or Conclusion II is true Give answer (4) if neither Conclusion I nor Conclusion II is true Give answer (5) if both the Conclusion I and Conclusion II are true (IBPS RRBs Officer CWE Prelim Exam, 13.09.2020)

286. Statements : C > A = T; S < E = T Conclusions : I. A > E II. C > S 287. Statements : F < U < N; D > H < U = B Conclusions : I. H < N II. H = N (288–289) : Statements : GO>W>I G II. W < N 290. Statements : R>M>T Q II. Q > M

SYMBOLS & NOTATIONS Directions (291–295) : In each of the following questions, certain relationship between different elements is shown in the statements. The statements are followed by two Conclusions numbered I and II. Read the Conclusions based on the given statements and select the appropriate answer. Give answer (1) if only Conclusion I is true. Give answer (2) if only Conclusion II is true. Give answer (3) if either Conclusion I or Conclusion II is true Give answer (4) if neither Conclusion I nor Conclusion II is true Give answer (5) if both the Conclusion I and Conclusion II are true (IBPS Bank PO/MT CWE Prelim Exam, 03.10.2020)

291. Statements : A>B=C; Q=P>S>B=D>Y Conclusions : I. Q > B II. Y < C 292. Statements : W > K = B; NT=D>H>M Conclusions : I. B > D II. T < M 293. Statements : P < D < U = G > B; Y P 294. Statements : T>J L; D > Q = M; Q>R>V Conclusions : I. L < M II. G > V Directions (296–300) : In the following questions, the symbols @, +, ©, % and # are used with the following meaning as illustrated below :

‘A @ B’ means ‘A is not greater than B’. ‘A + B’ means ‘A is neither smaller than nor equal to B’. ‘A © B’ means ‘A is not smaller than B’. ‘A % B’ means ‘A is neither greater than nor equal to B’. ‘A # B’ means ‘A is neither greater than nor smaller than B’. Now in each of the following questions, assuming the given statements to be true, find which of the two Conclusions numbered I and II is/are definitely true ? Give answer (1) if only Conclusion I is true. Give answer (2) if only Conclusion II is true. Give answer (3) if both the Conclusion I and Conclusion II are true Give answer (4) if either Conclusion I or Conclusion II is true Give answer (5) if neither Conclusion I nor Conclusion II is true (IBPS RRBs Officer CWE Prelim Exam, 31.12.2020)

296. Statements : M + H; K % M; G # H Conclusions : I. H # K II. M © G 297. Statements : E @ F; D % E; T © F Conclusions : I. D % F II. T © E 298. Statements : T # Y; Y % L; G © L Conclusions : I. L + T II. G © T 299. Statements : G + U; L @ U; M © G Conclusions : I. M # U II. M + U 300. Statements : Z + U; P © W; W @ U Conclusions : I. Z + W II. P % U

BPRE–177

SBI PO EXAMS 1. Which of the following symbols should replace the question mark in the given expression in order to make the expressions ‘A > D’ as well as ‘F ≥ C’ definitely true ? A>B≥C?D≤E=F (1) > (2) < (4) = (3) < (5) Either = or > (SBI Associate Banks PO Exam. 07.08.2011)

2. In which of the following expressions will the expressions ‘R < P’ as well as ‘S > Q’ be definitely true ? (1) P > Q = R < T < S (2) S > T > R > Q < P (3) Q > R < T > P > S (4} S > T > R > Q > P (5) None of these (SBI Associate Banks PO Exam. 07.08.2011)

Directions (3–7) : Read each statement carefully and answer the following questions : (SBI Probationary Officer Exam 28.04.2013)

3. Which of the following expressions will be true if the expression R > O = A > S < T is definitely true ? (1) O > T (2) S < R (3) T > A (4) S = O (5) T < R 4. Which of the following symbols should replace the question mark (?) in the given expression in order to make the expressions ‘P > A’ as well as ‘T < L’ definitely true ? P > L ? A> N = T (2) > (1) < (3) < (4) > (5) Either < or < 5. Which of the following symbols should be placed in the blank spaces respectively (in the same order from left to right) in order to complete the given expression in such a manner that makes the expressions ‘B > N’ as well as ‘D < L’ definitely true ? B___L___O___N___D (1) =, =, >, > (2) >, >, =, > (3) >, , =, =, > (5) >, =, >, >

SYMBOLS & NOTATIONS 6. Which of the following should be placed in the blank spaces respectively (in the same order from left to right) in order to complete the given expression in such a manner that makes the expression ‘A < P’ definitely false ? ___ < ___ < ___ > ___ (1) L, N, P, A (2) L, A, P, N (3) A, L, P, N (4) N, A, P, L (5) P, N, A, L 7. Which of the following symbols should be placed in the blank spaces respectively (in the same order: from left to right) in order to complete the given expression in such a manner that makes the expression ‘F > N’ and ‘U > D’ definitely false ? F___O___U___N____D (1) (5) >, >, =, < Directions (8–13) : In each of the following questions, relationship between different elements is shown in the statements. The statements are followed by two Conclusions numbered I and II. Study the conclusions based on the given statement and select the appropriate answer. (SBI Associates PO Online Exam. 29.11.2014)

(8–9) : Statements : A < B < C = D; K > J > C 8. Conclusion I : C < K Conclusion II : B < D (1) Only Conclusion I is true (2) Only Conclusion II is true (3) Neither Conclusion I nor Conclusion II is true (4) Either Conclusion I or II is true (5) Both Conclusions I and II are true 9. Conclusion I : A > J Conclusion II : K > B (1) Only Conclusion I is true (2) Only Conclusion II is true (3) Neither Conclusion I nor Conclusion II is true (4) Either Conclusion I or II is true (5) Both Conclusions I and II are true

(10–11) : Statements : B>L>A>N N Conclusion II : L < K (1) Only Conclusion I is true (2) Only Conclusion II is true (3) Neither Conclusion I nor Conclusion II is true (4) Either Conclusion I or II is true (5) Both Conclusions I and II are true 11. Conclusion I : L > N Conlusion II : L = N (1) Either Conclusion I or II is true (2) Only Conclusion II is true (3) Neither Conclusion I nor Conclusion II is true (4) Only Conclusion I is true (5) Only Conclusions I is true (12–13) : Statements : A>B>C S; L > I; T < Z 19. Conclusions I. I < O II. Z > S 20. Conclusions I. Z > I II. S < I 21. Statements GU>P O 22. Statements A > L < P < B; P > M Conclusions I. B > M II. A > M (23–24) : Statements I > M = P > R; B < P > A 23. Conclusions I. A < I II. I = A 24. Conclusions I. A < R II. B < I Direction (25-29) : In these questions, relationship between different elements is shown in the statements. The statements are followed by two conclusions. (SBI Probationary Officer Online Exam, 21.06.2014)

Give answer (1) if only Conclusion I is true. Give answer (2) if only Conclusion II is true. Give answer (3) if either Conclusion I or II is true. Give answer (4) if neither Conclusion I nor II is true. Give answer (5) if both Conclusions I and II are true.

25. Statements : B > C = D > X; E < X; Z > D Conclusions : I. B > E II. Z > B 26. Statements : E>F>G O; T > M < P Conclusions : I. T > K II. P > O (28-29) : Statements : B > O = K > L; D > K > S 28. Conclusions : I. O < D II. S < L 29. Conclusions : I. L > D II. B > S Directions (30–34) : In these questions, relationships between different elements is shown in the statements. These statements are followed by two conclusions. (SBI Probationary Officer Online Exam, 28.06.2014)

Give answer (1) if only conclusion I follows. Give answer (2) if only conclusion II follows. Give answer (3) if either conclusion I or conclusion II follows. Give answer (4) if neither conclusion I nor conclusion II follows. Give answer (5) if both conclusions I and II follow. 30. Statements : A>B=CX>W R II. N < P 33. Statements : T>M=BF>R Conclusions : I. M > R II. F < T 34. Statements : O=P T = U > V; W < X; T < X < Y Conclusions : I. S > X II. Y > S 38. Statements : T > U > V; W > V; P < V Conclusions : I. P < T II. W < T 39. Statements : A > B < M = Z; N > B > Q Conclusions : I. Z > N II. Q < Z 40. Statements : L > M > O; M < N; O > F Conclusions : I. F < M II. F < L Directions (41–46) : In each of the following questions, relationship between different elements is shown in the statements. The statements are followed by two Conclusions numbered I and II. Study the conclusions based on the given statement and select the appropriate answer. (SBI Associates PO Online Exam, 29.11.2014)

(41–42) : Statements : A < B < C = D; K > J > C 41. Conclusion I : C < K Conclusion II : B < D (1) Only Conclusion I is true (2) Only Conclusion II is true

SYMBOLS & NOTATIONS (3) Neither Conclusion I nor Conclusion II is true (4) Either Conclusion I or II is true (5) Both Conclusions I and II are true 42. Conclusion I : A > J Conclusion II : K > B (1) Only Conclusion I is true (2) Only Conclusion II is true (3) Neither Conclusion I nor Conclusion II is true (4) Either Conclusion I or II is true (5) Both Conclusions I and II are true (43–44) : Statements : B>L>A>N N Conclusion II : L < K (1) Only Conclusion I is true (2) Only Conclusion II is true (3) Neither Conclusion I nor Conclusion II is true (4) Either Conclusion I or II is true (5) Both Conclusions I and II are true 44. Conclusion I : L > N Conlusion II : L = N (1) Either Conclusion I or II is true (2) Only Conclusion II is true (3) Neither Conclusion I nor Conclusion II is true (4) Only Conclusion I is true (5) Only Conclusions I is true (45–46) : Statements : A>B>CS 47. Conclusions : I. A > S II. B < K (1) Either Conclusion I or II is true (2) Both Conclusions I and II are true (3) Neither Conclusion I nor II is true (4) Only Conclusion II is true (5) Only Conclusion I is true 48. Conclusions : I. B < N II. S < N (1) Either Conclusion I or II is true (2) Both Conclusions I and II are true (3) Neither Conclusion I nor II is true (4) Only Conclusion II is true (5) Only Conclusion I is true (49-50) : Statement : K>L=M>N=O

O (1) Either Conclusion I or II is true

BPRE–180

(2) Both Conclusions I and II are true (3) Neither Conclusion I nor II is true (4) Only Conclusion II is true (5) Only Conclusion I is true (51-52) : Statements : XZ 51. Conclusions : I. Z < K II. Y < W (1) Either Conclusion I or II is true (2) Both Conclusions I and II are true (3) Neither Conclusion I nor II is true (4) Only Conclusion II is true (5) Only Conclusion I is true 52. Conclusions : I. X > J II. K > Y (1) Either Conclusion I or II is true (2) Both Conclusions I and II are true (3) Neither Conclusion I nor II is true (4) Only Conclusion II is true (5) Only Conclusion I is true Directions (53–58) : In these questions, relationship between different elements is shown in the statements. The statements are followed by two conclusions numbered I and II. Study the conclusions based on the given statement and select the appropriate answer. (SBI Associates PO Online Exam. 30.11.2014)

(53-54) : Statement : BK>S 53. Conclusions : I. A > S II. B < K (1) Either Conclusion I or II is true (2) Both Conclusions I and II are true (3) Neither Conclusion I nor II is true (4) Only Conclusion II is true (5) Only Conclusion I is true 54. Conclusions : I. B < N II. S < N

SYMBOLS & NOTATIONS (1) Either Conclusion I or II is true (2) Both Conclusions I and II are true (3) Neither Conclusion I nor II is true (4) Only Conclusion II is true (5) Only Conclusion I is true (55-56) : Statement : K>L=M>N=O

O (1) Either Conclusion I or II is true (2) Both Conclusions I and II are true (3) Neither Conclusion I nor II is true (4) Only Conclusion II is true (5) Only Conclusion I is true (57-58) : Statements : XZ 57. Conclusions : I. Z < K II. Y < W (1) Either Conclusion I or II is true (2) Both Conclusions I and II are true (3) Neither Conclusion I nor II is true (4) Only Conclusion II is true (5) Only Conclusion I is true 58. Conclusions : I. X > J II. K > Y (1) Either Conclusion I or II is true (2) Both Conclusions I and II are true (3) Neither Conclusion I nor II is true (4) Only Conclusion II is true (5) Only Conclusion I is true

Directions (59–63) : In each of the following questions, relationship between different elements is shown in the statements. The statements are followed by two Conclusions numbered I and II. Study the Conclusions based on the given statements and select the appropriate answer : (SBI PO Phase–I (Preliminary) Online Exam. 20.06.2015)

Give answer (1) if only Conclusion I is true. Give answer (2) if only Conclusion II is true Give answer (3) if either Conclusion I or Conclusion II is true Give answer (4) if neither Conclusion I nor Conclusion II is true Give answer (5) if both the Conclusions I and II are true. (59–60) : Statements Y < K < D = S ; D < V < O ; G>D V II. Y < Q 60. Conclusions I. K < O II. G = V 61. Statements DZ=R>M H Directions (64 – 68) : In each of the following questions, relationship between different elements is shown in the statements. The statements are followed by two Conclusions numbered I and II. Study the Conclusions based on the given statement(s) and select the appropriate answer. (SBI PO Phase–I (Preliminary) Online Exam. 21.06.2015)

Give answer (1) if only Conclusion I is true Give answer (2) if only Conclusion II is true

BPRE–181

Give answer (3) If either Conclusion I or Conclusion II is true Give answer (4) if both the Conclusion I and Conclusion II are true Give answer (5) if neither Conclusion I nor Conclusion II is true. (64 – 65) : Statements L = P < W < V < K > Q; B < L; K=M 64. Conclusions I. B < V II. M > P 65. Conclusions I. L > Q II. W = M 66. Statements R < U = B < S; B < X Conclusions I. X > R II. X = R (67–68) : Statements C > U < S < T = O > D > Y; Z=O

D II. S < P 68. Conclusions I. Z > Y II. C < O Directions (69– 73) : In each of the following questions, relationship between different elements is shown in the statements. The statements are followed by two conclusions numbered I and II. Study the conclusions based on the given statements and select the appropriate answer. (SBI PO Phase–I (Preliminary) Online Exam. 27.06.2015)

Give answer (1) if only Conclusion I is true Give answer (2) if neither Conclusion I nor Conclusion II is true Give answer (3) if only Conclusion II is true Give answer (4) if both Conclusion I and Conclusion II are true Give answer (5) if either Conclusion I or Conclusion II is true (69 – 70) : Statements M>A>B=Q J

SYMBOLS & NOTATIONS 70. Conclusions I. Z = Q II. Z > Q 71. Statements G < R = A < S; T < R Conclusions I. G < S II. S > T (72 – 73) : Statements P = U < M < K < I > N; D > P; I>C 72. Conclusions I. M < C II. N > U 73. Conclusions I. D > K II. I > P Directions (74 – 78) : In these questions, relationship between different elements is shown in the statements. The statements are followed by two conclusions numbered I and II. Study the conclusions based on the given statement and select the appropriate answer. (SBI Assistant Manager (System) Exam, 17.01.2016)

Give answer (1) if both the Conclusions I and II are true Give answer (2) if neither Conclusion I nor Conclusion II is true Give answer (3) if either Conclusion I or Conclusion II is true Give answer (4) if only Conclusion I is true Give answer (5) if only Conclusion II is true (74–75) : Statements Z < H = N < C > M > S; G=K>C 74. Conclusions I. K > H II. H = K 75. Conclusions I. Z < C II. G > S (76–77) : Statements B < L < A = M > E; Q > M < O; D < L 76. Conclusions I. Q > D II. E < O 77. Conclusions I. L < E II. B < Q

78. Statements : F>I>S>C V > W; X < Y = W >Z Conclusions : I. Z < U II. W< T 82. Statements : K < L < M < N; M < O < P Conclusions : I. P > K II. N > O 83. Statements : B < A < C; A > D < E Conclusions : I. B < E II. C > E Directions (84–88) : In these questions, relationship between different elements is shown in the statements. The statements are followed by two Conclusions numbered I and II. Study the Conclusion based on the statements and select the appropriate answer : (SBI PO Online (Pre.) Exam, 02.07.2016 (IInd Shift))

Give answer (1) if both the Conclusion I and Conclusion II are true Give answer (2) if only Conclusion I is true

BPRE–182

Give answer (3) if neither Conclusion I nor Conclusion II is true Give answer (4) if only Conclusion II is true Give answer (5) if either Conclusion I or Conclusion II is true 84. Statement : J>V>D D 85. Statements : B > E > U < F; L=A A (86–87) : Statement : L>G>OQ Y Directions (89–93) : In each of the following questions, relationship between different elements is shown in the statements. The statements are followed by two conclusions numbered I and II. Study the conclusions based on the given statement(s) and select the appropriate answer. (SBI PO Phase-I (Pre.) Online Exam, 03.07.2016 (Ist Shift))

Give answer (1) if either Conclusion I or Conclusion II is true. Give answer (2) if neither Conclusion I nor Conclusion II is true Give answer (3) if only Conclusion I is true Give answer (4) if both Conclusion I and Conclusion II are true Give answer (5) if only Conclusion II is true 89. Statements : G > R > O > S; Y < O < F Conclusions : I. G > Y II. S < F

SYMBOLS & NOTATIONS 90. Statement : W>I=P>E>S Conclusions : I. S < W II. S = W (91–92) : Statements : R T II. C < Q 93. Statements : R < O < A < M; L = D > A Conclusions : I. R < L II. D > O Directions (94–98) : In these questions, relationship between different elements is shown in the statements. The statements are followed by two Conclusions numbered I and II. Study the Conclusions based on the statements and select the appropriate answer. (SBI PO Phase-I (Pre.) Online Exam, 03.07.2016 (IInd Shift))

Give answer (1) if only Conclusion I is true Give answer (2) if only Conclusion II is true Give answer (3) if either Conclusion I or Conclusion II is true Give answer (4) if neither Conclusion I nor Conclusion II is true Give answer (5) if both Conclusion I and Conclusion II are true (94–95) : Statements : M R 97. Statements : R > I > S > K; V > O > S Conclusions : I. O < R II. K > V

98. Statements : D>R>A S = T > U; S > A > V Conclusions I. A < R II. V < U (100-101) : Statements B>E>A>T=H J > I Conclusions I. R = S II. S < R 103. Statements DE Conclusions I. O < E II. L > D Directions (104–107) : In these questions, relationship between different elements is shown in the statements. The statements are followed by two Conclusions numbered I and II. Study the Conclusions based on the given statements and select the appropriate answer : (SBI PO Online Main Exam, 31.07.2016)

BPRE–183

Give answer (1) if only Conclusion I is true Give answer (2) if only Conclusion II is true Give answer (3) if both the Conclusion I and Conclusion II are true Give answer (4) if either Conclusion I or Conclusion II is true Give answer (5) if neither Conclusion I nor Conclusion II is true 104. Statements : K > G > P; I P 105. Statements : M > N = O; Y=Q>P>O>R Conclusions : I. P > R II. Y = N 106. Statements : K > P > N < S; N=X>D Conclusions : I. K > N II. D < S 107. Statements : H > K > L < M; L A > L < E; C > L > O > N (2) P < A < L > E; C > L < O < N (3) P = A > L = E; C = L > O < N (4) P > A > L > E; C < L < O < N (5) P = A > L < E; C < L > O > N (SBI PO Online (Preliminary) Online Exam. 30.04.2017)

109. What should come in place of $ and # respectively in the expression: P > A $ R < O < T; S < L < A # M, so that the expression T > M definitely holds true? (1) >, < (2) T; P>R G; IO>R>T Conclusions : I. C < O II. G > T 113. What should come in place of $ and # respectively in the expressions V > U > L > T $ R; M > O # V > P, so that the expression R < M definitely holds true? (1) >, > (2) L>A R > E < A < P; G>I>C A 116. Conclusions : I. I > R II. I = R 117. In which of the following expression does the expression K > N definitely holds false ? (1) Q < U = B < K; N, > (2) >, < (3) >, = (4) =, > (5) R’ and ‘B < C’ holds definitely true? (1) B < C = A > K = R (2) C = K > B < R > A (3) C > B > A > K = R (4) B = K < C < R = A (5) None of these (SBI PO Online (Preliminary) Exam, 07.05.2017)

BPRE–184

Directions (120–122) : In these questions, relationship between different elements is shown in the statements. The statements are followed by two Conclusions numbered I and II. Study the Conclusions based on the given statements and select the appropriate answer : (SBI PO Online (Preliminary) Exam, 07.05.2017)

Give answer (1) if neither Conclusion I nor Conclusion II is true. Give answer (2) if both the Conclusion I and Conclusion II are true.11 Give answer (3) if only Conclusion I is true. Give answer (4) if only Conclusion II is true. Give answer (5) if either Conclusion I or Conclusion II is true. 120. Statements : M > U > L < N; L > Y > A Conclusions : I. Y < N II. Y = N 121. Statements : J > A > D = E; L < A < M Conclusions : I. M < J II. J > L 122. Statements : M < K > L = Y; P < T > M Conclusions : I. P > Y II. T < L Directions (123–124) : Study the following information carefully and answer the question given below : There is a 5 × 5 matrix which can produce signals which in turn help in illumination of some bulbs. The rows of the matrix are denoted by @, %, &, £ and ¥ from bottom to top in the same order and the columns are denoted by A, B, C, D and E from left to right : ¥ row contains numbers which are consecutive multiples of ‘9’ starting from ‘36’. £ row contains numbers which are consecutive multiples of ‘13’ & row contains numbers which are consecutive multiples of ‘8’ starting from ‘32’. % row contains numbers which are consecutive multiples of ‘7’ @ row contains numbers which are consecutive multiples of ‘11’

SYMBOLS & NOTATIONS Note : If the starting number of a multiple is not given; then start the row from the same number. Ex : If a row contains numbers which are consecutive multiples of ‘5’. Then the row might be as follows:

5 10 15 20 25 The matrix helps in producing signals which is a string of numbers. There are four bulbs, P, Q, R and S. Based on the outcomes of the string one of the bulb blinks. Conditions for blink : I. If outcome of the string is below 80, the bulb Q blinks. II. If outcome of the string is between 90 and 110, the bulb S blinks. III. If outcome of the string is between 125 and 150, then bulb P blinks. IV. If outcome of the string is between 175 and 200, the bulb R blinks. V. If none of the above condition follows, no bulb blinks. Three outcomes of the string : I. If all the numbers of the string are even numbers then, outcome is obtained by multiplying unit digits of all the two digit numbers. II. If a prime number is followed by another prime number then, outcome is obtained by sum of all the two digits numbers. III. If a prime number is preceded by a perfect square then, outcome is obtained by multiplying tenth place digits of all the numbers. (SBI PO Phase-II (Main) Exam 05.08.2018)

123. If Y = £B & E, £A @D, then which of the following bulb blinks? (1) R (2) P (3) Q (4) S (5) None blinks 124. If X = ¥C @ D £B @B, then which of the following bulb blinks? (1) R (2) P (3) Q (4) S (5) None blinks Directions (125–129) : In each of the following questions, relationship between different elements is shown in the statement(s). The statements are fol-

lowed by two Conclusions numbered I and II. Study the conclusions based on the given statements and select the appropriate answer. Give answer (1) if both the Conclusion I and Conclusion II are true Give answer (2) if neither Conclusion I nor Conclusion II is true Give answer (3) if either Conclusion I or Conclusion II is true Give answer (4) if only Conclusion I is true Give answer (5) if only Conclusion II is true SBI PO (Prelim Exam), 08.07.2018 (Shift-I))

125. Statements : P < Q < S = T, R = Q < U, V > U Conclusions : I. P > V II. V > T 126. Statements : U > X = Y, Y < Z < S, T=W>Z Conclusions : I. T < U II. S > U 127. Statements : A > P = S > T, V < B = T > X Conclusions : I. A > X II. P < B 128. Statements : S > U > V, Y < U < Z, ZW Conclusions : I. S < Z II. X > Y 129. Statements : P < X < Y < Q, S > Y < T, P = V > R Conclusions : I. V < S II. T > R Directions (130–133) : In each of the following questions, relationship between different elements is shown in the statements. The statements are followed by two Conclusions numbered I and II. Study the Conclusions based on the given statement and select the appropriate answer. Give answer (1) if either Conclusion I or Conclusion II is true. Give answer (2) if neither Conclusion I nor Conclusion II is true. Give answer (3) if both the Conclusion I and Conclusion II are true.

BPRE–185

Give answer (4) if only Conclusion I is true. Give answer (5) if only Conclusion II is true. SBI PO (Prelim Exam), 08.07.2018 (Shift-II))

(130–131) : Statements : H < O < L < D; G = D > S; P P II. P < D 132. Statement : BN=R>Q Conclusions : I. P < Q II. K > S

SYMBOLS & NOTATIONS 136. Statements : R = A > N = K; O>N=M>U=Y Conclusions : I. R = Y II. Y < R 137. Statements : J < K > L > M > N; X>L J

RBI GRADE–B/ NABARD GRADE–A OFFICER EXAMS 1. Which of the following expressions is true if the expression ‘C > R = O < S = T > M’ is definitely true ? (1) O < M (2) C > M (3) R > M (4) C > S (5) T > R (RBI Officer Grade ‘B’ Phase-I Exam. 21.11.2015)

2.

Which of the following symbols should replace the question mark in the given expression in order to make the expression ‘O £ K’ definitely true but ‘M > I’ definitely wrong ? I (5) ³

(RBI Grade-B Officer’s Exam. 18.12.2011)

Directions (3–7) : In the following questions, the symbols δ, #, %, @ and ★ are used with the following meaning as illustrated below : (RBI Officer Grade ‘B’ Exam. 25.08.2013)

‘P # Q’ means ‘P is neither greater than nor smaller than Q’. ‘P δ Q’ means ‘P is not smaller than Q’. ‘P @ Q’ means ‘P is neither smaller than nor equal to Q’. ‘P ★ Q’ means ‘P is not greater than Q’. ‘P % Q’ means ‘P is neither greater than nor equal to Q’. Now in each of the following questions assuming the given statements

to be true, find which of the three conclusions I, II and III given below them is/are definitely true and give your answer accordingly. 3. Statements : D % F, F @ H, H ★ N Conclusions : I. N @ F II. D % N III. H % D (1) None is true (2) Only I is true (3) Only II is true (4) Only III is true (5) Only I and II are true 4. Statements : B δ D, D % T, T ★ M Conclusions : I. B @ T II. M @ D III. B @ M (1) Only I is true (2) Only II is true (3) Only III is true (4) Only II and III are true (5) None of these 5. Statements : K # W, M @ W, R δ M Conclusions : I. K % M II. W % R III. R @ K (1) Only I and II are true (2) Only I and III are true (3) Only II and III are true (4) All I, II and III are true (5) None of these 6. Statements : M @ K, K δ T, T # J Conclusions : I. J # K II. M @ J III. J % K (1) Only I is true (2) Only II is true (3) Only III is true (4) Only either I or III is true (5) Only either I or III and II are true 7. Statements : R ★ N, N % B, B # T Conclusions : I. B @ R II. T @ N III. R % T (1) Only I and II are true (2) Only I and III are true (3) Only II and III are true (4) All I, II and III are true (5) None of these

BPRE–186

Directions (8-13) : In these questions, relationship between different elements is shown in the statements. These statements are followed by two conclusions. (RBI Officer Grade ‘B’ Online Exam. 25.08.2013)

Mark answer (1) if Only conclusion I follows Mark answer(2) if Only conclusion II follows Mark answer (3) if Either conclusion I or II follows Mark answer (4) if Neither conclusion I nor II follows Mark answer (5) if Both conclusions I and II follow 8. Statement : A ≥ B ≤ C, C < D, D > F Conclusions : I. F > B II. A > D 9. Statement : X > Y ≥ Z, Q = Y, P > X Conclusions : I. Z < P II. P > Q 10. Statement : L ≥ I, H > I > J , K < J Conclusions : I. H > L II. L > K 11. Statement : O ≥ P = Q, R < P, S < Q Conclusions : I. R < S II. O > S 12. Statement : D ≥ E > F = G, E = H < J Conclusions : I. J > D II. G < J 13. Statement : J ≥ R > Z, R > F < W, B > J Conclusions : I. J > F II. B > W Directions (14-18) : In these questions relationship between different elements is shown in the statements. The statements are followed by two conclusions. (RBI Officer Grade ‘B’ Online Exam. 25.08.2013)

Give answer (1) if only Conclusion I is true. Give answer (2) if only Conclusion II is true. Give answer (3) if only Conclusion I or II is true. Give answer (4) if neither Conclusion I not II is true. Give answer (5) if both Conclusions I and II are true.

SYMBOLS & NOTATIONS 14.

Statements : E > F = G; I = T; T < G Conclusions : I. I < E II. I = E 15. Statements : G > H < T; I > F; H > J Conclusions : I. J < G II. F < H 16. Statements : V > W < X; X < Y; Z > X Conclusions : I. Z > V II. Y > W 17. Statements : M > N > P; O > P; S < P Conclusions : I. S < M II. O < M 18. Statements : A > E > F; G > F; M > A Conclusions : I. M > E II. G < A Directions (19-23) : In these questions, relationship between different elements is shown in the statements. (RBI Officer Grade ‘B’ Phase-I Exam, 03.08.2014)

The statements are followed by two conclusions. Give answer (1) if only Conclusion I is true. Give answer (2) if only Conclusion II is true. Give answer (3) if either Conclusion I or II is true. Give answer (4) if neither Conclusion I nor II is true. Give answer (5) if both Conclusions I and II are true. 19. Statement : V=IE Conclusions : I. A > E II. L > V (20-21) : Statements : P > H = J > R; H < I < T 20. Conclusions : I. J > T II. P > T 21. Conclusions : I. R < T II. T > H (22-23) : Statements : C > H < M < R = A; Z > M > Y

22. Conclusions % I. Z > C II. Y < A 23. Conclusions % I. Y > C II. Z > A Directions (24–27) : In each of the following questions, a statement is given followed by two sets of conclusions numbered I and II. These statements show relationship between different elements. You have to assume the statement to be true and then decide which of the given conclusions logically follows from the information given in the statement. (RBI Officer Grade ‘B’ Phase-I Exam. 21.11.2015)

Give answer (1) if either Conclusion I or Conclusion II is true Give answer (2) if only Conclusions I is true Give answer (3) if both the Conclusion I and Conclusion II are true Give answer (4) if only Conclusion II is true Give answer (5) if neither Conclusion I nor Conclusion II is true (24–25) : Statements : B < U > S > G; W < S; W > M 25. Conclusions : I. G < B II. M > S 25. Conclusions : I. M < U II. G < W 26. Statements : K > B = N < U; T < N; P > U Conclusions : I. P > K II. T < K 27. In which of the following expressions does the expression ‘Q > Z’ hold definitely true? (1) E > Z > R > B = C > Q (2) N > Q > A = C < I > Z (3) Q > R > M = P < A > Z (4) Z < S = B < L < O < Q (5) Z = T < D < Y = X < Q Directions (28–32) : In each of the following questions, relationship between different elements is shown in the statements. The statements are followed by two conclusions numbered I and II. Study the conclusions based on the given statements and select the appropriate answer. (Nabard Officer Grade ‘A’ Online Exam. 01.03.2015)

BPRE–187

Give answer (1) if only Conclusion I is true Give answer (2) if only Conclusion II is true Give answer (3) if either Conclusion I or II is true Give answer (4) if neither Conclusion I nor II is true Give answer (5) if both Conclusions I and II are true (29–29) : Statement : M≥A>L=T≥E D II. E ≥ M 29. Conclusions : I. E < A II. M ≤ D (30–31) : Statements : C ≥ R = A < W ≤ L; S ≤ A 30. Conclusions : I. R < L II. L > S 31. Conclusions : I. C > S II. S = C 32. Statement : V≤O P ≥ S (2) M ≥ S = P < F (3) Q < M < F = P (4) P = A < R < M (5) None of these (RBI Grade-B Officer Exam. 11.10.2009)

Directions (33–38) : In each of the following questions, relationship between different elements is shown in the statements. The statements are followed by two Conclusions numbered I and II. Study the Conclusions based on the given statements and mark the appropriate answer : (NABARD Assistant Manager Exam, 15.15.2016)

Give answer (1) if both Conclusion I and Conclusion II are true Give answer (2) if neither Conclusion I nor Conclusion II is true Give answer (3) if only Conclusion I is true Give answer (4) if only Conclusion II is true Give answer (5) if either Conclsion I or Conclusion II is true

SYMBOLS & NOTATIONS (33–34) : Statements : Q > U > I < C = K > L; G < L; C < H 33. Conclusions : I. I < L II. C > G 34. Conclusions : I. Q > K II. H < U (35–36) : Statements : M < O = N < K < E < Y = S; DB

Conclusions :

(3) Y < T

M B

37. Conclusions : I.

E>O

(4) Z > V

(5) V< G

36. Conclusions : I.

48. Conclusions

I. P < T II. T = P 44. Which of the following expressions will be definitely false if the given expression ‘G > H = I > V < Y < Z < T, is definitely true? (1) I < G (2) T < V

II. D < S

38. Statements : R < A = I > N; S > A Conclusions : I. S > N II. N = S Directions (39–43) : In these questions, relationship between different elements is shown in the statements. The statements are followed by two conclusions numbered I and II. Study the conclusions based on the given statement and select the appropriate answer. (RBI Officer Grade ‘B’ Phase-I Exam, 04.09.2016 (Shift-I))

Give answer (1) if neither Conclusion I nor Conclusion II is true. Give answer (2) if either Conclusion I or Conclusion II is true Give answer (3) if only Conclusion I is true Give answer (4) if both Conclusion I and Conclusion II are true Give answer (5) if only Conclusion II is true (39–40) : Statements : C > V < R = N > T > Q; Y > N < A 39. Conclusions : I. Q > V II. Q < Y 40. Conclusions : II. A > Q I. C > Y (41–42) : Statements : P > R < U < M < V; T < U; L < M 41.Conclusions : I. L < V II. T < L 42. Conclusions : I. V > T II. T < P 43. Statements : R < Q < T = M < N = S; T > F > G; M = D > P

45. Which of the following symbols should replace the question mark(s) in the given expression in order to make the expressions ‘H < R’ as well as ‘D > M’ definitely true? D>I>H=S?M

(5) = (RBI Officer Grade ‘B’ Phase-I Exam, 04.09.2016 (Shift-I))

Directions (46– 50) : In these questions, relationship between different elements is shown in the statements. The statements are followed by two conclusions numbered I and II. Study the conclusions based on the given statements and select the appropriate answer. (RBI Officer Grade ‘B’ Phase-I Exam, 04.09.2016 (Shift-II))

Give answer (1) if neither Conclusion I nor II is true Give answer (2) if only Conclusion II is true Give answer (3) if either Conclusion I or II is true Give answer (4) if both Conclusions I and II are true Give answer (5) if only Conclusion I is true Statements P < L < A = N > D > I; K < A < Q 46. Conclusions KP

(1) T < E (3) E > C (5) R > T

(2) T > U (4) N < U

(RBI Officer Grade ‘B’ Phase-I Exam, 04.09.2016 (Shift-II))

51. Which of the following symbols should replace the question mark (?) in the given expression in order to make the expressions ‘M < Z’ and ‘Y < B’ definitely true ? M M; R > O > M (2) T > L < M; R < M < O (3) T > L > M; R > O > M (4) T > L > M; R < O < M (5) T > L = M; R < O < M (RBI Officer Grade ‘B’ Phase-I Exam 17.06.2017)

53. What should come in place of (?) in the expression P ? U = E > J; E ? M < W respectively so that the expression W > P definitely holds true? (2) >, = (1) =, < (3) =, = (4) >, < (5) >, < (RBI Officer Grade ‘B’ Phase-I Exam 17.06.2017)

(46–47) :

I.

I. R > Y II. J < R 49. Conclusions I. M < Y II. Y > K 50. Which of the following expressions will be definitely false if the given expression ‘U > B = N > T < C < R < E’ is definitely true ?

II. K < D

(48–49) : Statements K < A < U > M > J; A > Y; R > U

BPRE–188

Directions (54–57) : In these questions, relationship between different elements is shown in the statements. The statements are followed by two Conclusions numbered I and II. Study the Conclusions based on the given statements and select appropriate answer. (RBI Officer Grade ‘B’ Phase-I Exam 17.06.2017)

SYMBOLS & NOTATIONS Give answer (1) if both the Conclusion I and Conclusion II follow Give answer (2) if either Conclusion I or Conclusion II follows Give answer (3) if neither Conclusion I nor Conclusion II follows Give answer (4) if only Conclusion I follows Give answer (5) if only Conclusion II follows (54-55) : Statements : B < R < U > S > H; Y P = E; P Y II. U > R 59. Conclusions : I.

U>C

II. T < Y

60. Statements : P > R = B < S; C < B > Z

Conclusions : I. S > Z II. S < P 61. Statements : D < Q < N > W; Q > P Conclusions : I. N = W II. N > W Directions (62–66) : In each of the following questions, relationship between different elements is shown in the statements. The statements are followed by two Conclusions numbered I and II. Study the Conclusions based on the given statement and select the appropriate answer. (RBI Assistant Manager Online Exam 25.03.2017)

Give answer (1) if eithr Conclusion I or Conclusion II is true. Give answer (2) if neither Conclusion I nor Conclusion II is true. Give answer (3) if both Conclusion I and Conclusion II are true. Give answer (4) if only Conclusion I is true. Give answer (5) if only Conclusion II is true. 62. Statement : JD Conclusions : I. J < E II. I > D (63–64) : Statements : O < K > N = L > A; K < C < R; L O II. A > Q 64. Conclusions : I. O < C II. O = C 65. Statements : C > U < W = Z; U > O Conclusions : I. C > O II. Z > O 66. Statement : W A II. I > P 71. Statements : R < S < T < U; U = V > X > Y Conclusions : I. Y > R II. Y < U 72. What will come in place of ‘@’ and ‘#’ respectively in the following expression so that both ‘S = W’ and ‘P > J’ will definitely hold true? P > Q > R > S; R @ J < T # W (2) , = (5) >, < (SEBI Assistant Manager Exam. 17.11.2018)

Directions (73–74) : In each of the following questions, relationship between different elements is shown in the statements. The statements are followed by two Conclusions numbered I and II. Study the Conclusions based on the given statement and select the appropriate answer.

SYMBOLS & NOTATIONS Give answer (1) if only Conclusion I is true. Give answer (2) if only Conclusion II is true Give answer (3) if either Conclusion I or Conclusion II is true. Give answer (4) if neither Conclusion I nor Conclusion II is true. Give answer (5) if both Conclusion I and Conclusion II are true. (SEBI Assistant Manager Exam. 17.11.2018)

73. Statements : M = P < V < C > K; T > C = L Conclusions : I. M < L II. T > K 74. Statements : F > Y > N = E > H; N < W Conclusions : I. F > W II. H < Y 75. In which of the following expressions does the expressions S < H and T < I definitely hold true? (1) S > W < T, P > W < H, H < I (2) S < W > T, P > W < H, H > I (3) S < W = T, P < W < H, H < I (4) S < W < T, P < W > H, H < I (5) S < W > T, P < W = H, H > I (NABARD Grade ‘A’ Manager Exam, 15.06.2019)

76. In which of the following expressions does the expression E > A definitely hold true? (1) A < B > C; E > D > C (2) A = B > C; E > D > C (3) A > B = C; E > D > C (4) A < B < C; E > D < C (5) A = B < C; E > D > C (NABARD Grade ‘A’ Manager Exam, 15.06.2019)

77. What should come in place of @ and # respectively in the given expression S @ R > Q = P; S < O # T, so that the expression P < T definitely holds true? (1) >, < (2) >, > (3) =, > (4) (5) T’ definitely true ? H>M=K?T>J (1) < (2) < (4) = (3) > (5) Either = or > (LIC Assistant Administrative Officer (AAO) Exam. 2.05.2013)

Directions (17–20) : In these questions, relationship between different elements is shown in the statements. These statements are followed by two conclusions : (United India Insurance AO Exam. 26.05.2013)

Give answer (1) if only conclusion I follows. Give answer (2) if only conclusion II follows. Give answer (3) if either conclusion I or II follows. Give answer (4) if Neither conclusion I nor II follows. Give answer (5) if both conclusions I and II follow. 17. Statements : O>P>Q>R P 18. Statements : A < B < C; B = D > E > F; G < A Conclusions : I. D > A II. A < E

BPRE–191

(19–20) : Statements : H > I = J > K; L < K; I < M < N 19. Conclusions : I. H > M II. N > H 20. Conclusions : I. M < I II. H > K Directions (21–25) : In these questions, relationship between different elements is shown in the statements. The statements are followed by conclusions : (LIC Assistant Administrative Officer (AAO) Exam. 12.05.2013)

Give answer (1) if only conclusion I is true. Give answer (2) if only conclusion II is true. Give answer (3) if either conclusion I or II is true. Give answer (4) if neither conclusion I nor II is true. Give answer (5) if both conclusions I and II are true. 21. Statements : AC=D>E; GH Conclusions : I. A > H II. B > F 23. Statements : P>Q>R>SU>V Conclusions : I. Q < V II. P > S 24. Statements : J>K>L; NM Conclusions : I. K > M II. J > N 25. Statements : A=B>C=D>E; GH Conclusions : I. G = A II. G < A Directions (26–31) : In these questions, relationship between different elements is shown in the statements. The statements are followed by two conclusions numbered I and II. Study the conclusions based on the given statement and select the appropriate answer. (NIACL Administrative Officer (AO) Exam, 10.01.2015)

SYMBOLS & NOTATIONS 26. Statements : T < D < S < Y ; D > R > L ; S L (1) Either conclusion I or II is true (2) Neither conclusion I nor II is true (3) Both conclusion I and II are true (4) Only conclusion I is true (5) Only conclusion II is true 27. Statements : T < D < S < Y ; D > R > L ; S N (1) Either conclusion I or II is true (2) Neither conclusion I nor II is true (3) Both conclusions I and II are true (4) Only conclusion I is true (5) Only conclusion II is true 28. Statements : AMU=H;J>T 36. Conclusions : I. K > O II. O > H (1) Neither Conclusion I nor II is true (2) Both Conclusions I and II are true (3) Only Conclusion II is true (4) Either Conclusion I or II is true (5) Only Conclusion I is true 37. Conclusions : I. K > T II. J < O (1) Neither Conclusion I nor II is true (2) Both Conclusions I and II are true (3) Only Conclusion II is true (4) Either Conclusion I or II is true (5) Only Conclusion I is true Directions (38-43) : In these questions, relationship between different elements is shown in the statements. (NIACL Administrative Officer (AO) Online Exam, 12.01.2015)

The statements are followed by two conclusions. Give answer (1) if only Conclusion I is true. Give answer (2) if only Conclusion II is true. Give answer (3) if either Conclusion I or II is true. Give answer (4) if neither Conclusion I nor II is true. Give answer (5) if both Conclusions I and II are true. 38. Statements R>W;WK;K>J;J B

SYMBOLS & NOTATIONS 40. Statements D=T;TN Conclusions : I. H < D II. H > D 41. Statements HK;K=J Conclusions : I. J < E II. W > K 43. Statements RT Conclusions : I. R < T II. T < M Directions (44-49) : In each of the following questions, relationship between different elements is shown in the statements. The statements are followed by two Conclusions numbered I and II. Study the Conclusions based on the given statement and select the appropriate answer. (LIC Assistant Administrative Officer (AAO) Online Exam. 22.03.2015)

Give answer (1) if neither Conclusion I nor Conclusion II is true Give answer (2) if either Conclusion I or Conclusion II is true Give answer (3) if only Conclusion II is true Give answer (4) if both the Conclusion I and Conclusion II are true Give answer (5) if only Conclusion I is true (44-45) : Statements : B > E > A < L; T > A > S 44. Conclusions : II. L > T I. T > B 45. Conclusions : I. B > S II. L > S (46–47) : Statements : L = I > N < E; N > S 46. Conclusions : I. S < E II. E > L 47. Conclusions : I. S = L II. L > S 48. Statements : V < E > B = H > N; B < T Conclusions : I. T > V II. N < T Directions (49 – 53) : In each of the following questions, relationship between different elements is shown in the question. The statements are fol-

(1) (2) (3) (4)

lowed by conclusions. Study the Conclusions based on the given statements and select the appropriate answer. (OICL Specialist Officer (Finance) Exam. 03.05.2015)

Give answer (1) if both the Conclusions I and II are true Give answer (2) if either Conclusion I or Conclusion II is true Give answer (3) if only Conclusion I is true Give answer (4) if only Conclusion II is true Give answer (5) if neither Conclusion I nor Conclusion II is true (49–50) : Statements : P < U = R > T; R < A; P > M 49. Conclusions : II. T < A I. M < T 50. Conclusions : II. P < T I. A > P (51–52) : Statements : A > B = C < S; Q > C > R 51. Conclusions : II. S > R I. Q > B 52. Conclusions : II. Q > S I. R < A 53. Statements : K < I < H > D > R; Z < H Conclusions : I. Z < K II. Z > R 54. If ‘×’ means ‘addition’ ‘–’ means ‘division’, ‘÷’ means ‘subtraction ’ and ‘+’ means ‘multiplication’, then which of the following equations is correct ? (1) 16 + 5 – 10 × 4 ÷ 3 = 9 (2) 16 – 5 × 10 ÷ 4 + 3 = 12 (3) 16 + 5 ÷ 10 × 4 – 3 = 9 (4) 16 × 5 ÷ 10 ÷ 4 – 3 = 19

If ‘–’ stands for division ‘+’ for multiplication, ‘÷’ for subtraction and ‘×’ for addition, then which one of the equations is correct ? (1) 19 + 5 – 4 × 2 ÷ 4 = 11 (2) 19 × 5 – 4 + 2 ÷ 4 = 16 (3) 19 + 5 + 4 – 2 × 4 = 13 (4) 19 × 5 ÷ 4 + 2 – 4 = 22 (NICL (GIC) AO (Finance) Exam, 08.09.2013)

56.

If ‘–’ stands for division, ‘+’ stands for subtraction, ‘÷’ stands for multiplication, ‘×’ stands for addition, then which one of the following equations is correct?

BPRE–193

– – – –

2 2 2 2

+ + + +

4 4 4 4

÷ ÷ ÷ ÷

5 5 5 5

× × × ×

6 6 6 6

= = = =

44 21 341 36

(NICL (GIC) AO Exam, 15.12.2013)

Directions (57–61) : In each of the following questions, relationship between different elements is shown in the statements. The statements are followed by two conclusions numbered I and II. Study the conclusions based on the given statements and select the appropriate answer. (UIICL A.O. Exam 12.06.2016)

Give answer (1) if either Conclusion I or Conclusion II is true Give answer (2) if neither Conclusion I nor Conclusion II is true Give answer (3) if only Conclusion I is true Give answer (4) if both Conclusion I and Conclusion II are true Give answer (5) if only Conclusion II is true (57–58) : Statements : C < H < I > L; D > I; L > N 57. 58. 59.

60.

61.

(NICL (GIC) AO (Finance) Exam, 08.09.2013)

55.

70 70 70 70

62.

Conclusions I. D > L II. N < L Conclusions I. D > C II. C = D Statements P > A = R < E < N; R > T Conclusions II. T < P I. A > N Statements M > U = S > T; R < S > D Conclusions I. M > D II. T > R Statements H < E < R > O; R < S < M Conclusions II. H < S I. M > O If M > N < O < P and T < O > C > Z are true, then which of the following options is definitely true ? (1) M > O (2) C = T (3) O = Z (4) T < P (5) N > Z (NICL A.O. Exam 05.06.2017)

63.

If X > Y > Z and A > B > Y < M < T are true, which of the following options is definitely false ? (1) Z < X (2) T < X (3) A > Z (4) B > Z (5) Z < T (NICL A.O. Exam 05.06.2017)

SYMBOLS & NOTATIONS Directions (64–68) : In each of the following questions, relationship between different elements is shown in the statements. The statements are followed by two Conclusions numbered I and II. Study the Conclusions based on the statements and select the appropriate answer : Give answer (1) if only Conclusion I is true Give answer (2) if only Conclusion II is true Give answer (3) if either Conclusion I or Conclusion II is true Give answer (4) if neither Conclusion I nor Conclusion II is true Give answer (5) if both the Conclusion I and Conclusion II are true (LIC AAO Prelim Exam. 04.05.2019)

(64–65) : Statements : A≥M≥H≥K≥L L II. B ≥ K 65. Conclusions : I. A ≥ B II. B < K (66–67) : Statements : P < A ≤ R = K; S ≤ R ≤ N 66. Conclusions : I. N > P II. K < N 67. Conclusions : I. K > A II. A < K 68. Statements : M ≤ E ≤ L ≤ T > S; L > V Conclusions : I. V ≥ S II. S > V Directions (69–72) : In each of the following questions, relationship between different elements is shown in the statements. The statements are followed by two Conclusions numbered I and II. Study the Conclusions based on the given statement and select the appropriate answer. Give answer (1) if only Conclusion I is true Give answer (2) if only Conclusion II is true Give answer (3) if either Conclusion I or Conclusion II is true Give answer (4) if neither Conclusion I nor Conclusion II is true Give answer (5) if both the Conclusion I and Conclusion II are true (LIC AAO Prelim Exam. 05.05.2019)

69. Statements : H < A = L < P; B = A > K = Y Conclusions : I. Y > H II. K < P 70. Statements :

I. II. 71.

I. II. 72.

G ≤ U = Y; F ≥ C ≥ T = Y Conclusions : F=G G H < L < E; M < H > A = N Conclusions : N E Directions (73 – 75) : In the following questions, relationships between different elements are shown in different statements. The statements are followed by two Conclusions numbered I and II. Read the Conclusions based on the statements and select appropriate answer. Give answer (1) if only Conclusion I is true. Give answer (2) if only Conclusion II is true. Give answer (3) if both the Conclusion I and Conclusion II are true. Give answer (4) if either conclusion I or Conclusion II is true. Give answer (5) if neither Conclusion I nor Conclusion II is true. (LIC AAO Main Exam, 28.06.2019)

73. Statements : (1) P > Q = W (2) Q > O > U (3) O > V < W Conclusions : I. P > U II. V < Q 74. Statements : (1) Y > U > V < O (2) J > G > H = Y Conclusions : I. G > O II. O > G 75. Statements : (1) P > Q > F > I (2) Y > F > U Conclusions : I. P > Y II. P > U

BPRE–194

SHORT ANSWERS

NATIONALISED BANKS & IBPS PO/MT/SO EXAMS 1. (4) 5. (2) 9. (2) 13. (3) 17. (5) 21. (1) 25. (4) 29. (5) 33. (5) 37. (3) 41. (4) 45. (1) 49. (2) 53. (4) 57. (1) 61. (5) 65. (2) 69. (5) 73. (1) 77. (1) 81. (2) 85. (1) 89. (5) 93. (5) 97. (5) 101. (3) 105. (1) 109. (2) 113. (2) 117. (5) 121. (5) 125. (4) 129. (1) 133. (3) 137. (1) 141. (1) 145. (4) 149. (1) 153. (5) 157. (2) 161. (1) 165. (5) 169. (3) 173. (4) 177. (5) 181. (3) 185. (3) 189. (4) 193. (2) 197. (2)

2. (3) 6. (4) 10. (5) 14. (1) 18. (2) 22. (5) 26. (4) 30. (4) 34. (1) 38. (2) 42. (2) 46. (1) 50. (1) 54. (5) 58. (2) 62. (4) 66. (4) 70. (4) 74. (2) 78. (3) 82. (1) 86. (4) 90. (3) 94. (1) 98. (2) 102. (2) 106. (5) 110. (1) 114. (4) 118. (3) 122. (3) 126. (5) 130. (1) 134. (2) 138. (3) 142. (2) 146. (2) 150. (3) 154. (2) 158. (4) 162. (5) 166. (3) 170. (4) 174. (2) 178. (3) 182. (1) 186. (3) 190. (5) 194. (4) 198. (3)

3. (1) 7. (1) 11. (1) 15. (4) 19. (4) 23. (5) 27. (2) 31. (1) 35. (4) 39. (2) 43. (4) 47. (5) 51. (1) 55. (4) 59. (2) 63. (5) 67. (1) 71. (5) 75. (2) 79. (3) 83. (5) 87. (2) 91. (1) 95. (4) 99. (4) 103. (5) 107. (3) 111. (1) 115. (3) 119. (2) 123. (1) 127. (4) 131. (3) 135. (3) 139. (2) 143. (5) 147. (2) 151. (5) 155. (4) 159. (5) 163. (4) 167. (5) 171. (1) 175. (4) 179. (2) 183. (2) 187. (3) 191. (2) 195. (5) 199. (1)

4. (1) 8. (3) 12. (3) 16. (2) 20. (5) 24. (2) 28. (5) 32. (2) 36. (1) 40. (1) 44. (4) 48. (1) 52. (5) 56. (1) 60. (1) 64. (3) 68. (5) 72. (2) 76. (4) 80. (5) 84. (3) 88. (1) 92. (4) 96. (3) 100. (1) 104. (4) 108. (3) 112. (5) 116. (4) 120. (5) 124. (2) 128. (3) 132. (1) 136. (3) 140. (5) 144. (1) 148. (4) 152. (3) 156. (1) 160. (3) 164. (2) 168. (5) 172. (3) 176. (5) 180. (2) 184. (5) 188. (1) 192. (5) 196. (4) 200. (5)

SYMBOLS & NOTATIONS 201. (3) 205. (5) 209. (4) 213. (5) 217. (5) 221. (4) 225. (3) 229. (4) 233. (4) 237. (3) 241. (3) 245. (2) 249. (1) 253. (4) 257. (3) 261. (4) 265. (1) 269. (5) 273. (3) 277. (4) 281. (1) 285. (2) 289. (4) 293. (1) 297. (3)

202. (4) 206. (2) 210. (3) 214. (3) 218. (2) 222. (5) 226. (5) 230. (3) 234. (3) 238. (4) 242. (5) 246. (4) 250. (1) 254. (3) 258. (1) 262. (2) 266. (3) 270. (3) 274. (5) 278. (2) 282. (1) 286. (2) 290. (4) 294. (4) 298. (1)

203. (2) 207. (1) 211. (2) 215. (4) 219. (3) 223. (1) 227. (4) 231. (5) 235. (1) 239. (1) 243. (4) 247. (2) 251. (3) 255. (2) 259. (2) 263. (4) 267. (5) 271. (2) 275. (1) 279. (5) 283. (3) 287. (1) 291. (5) 295. (2) 299. (2)

204. (1) 208. (3) 212. (1) 216. (3) 220. (1) 224. (2) 228. (2) 232. (5) 236. (2) 240. (5) 244. (1) 248. (5) 252. (2) 256. (4) 260. (5) 264. (1) 268. (4) 272. (2) 276. (2) 280. (1) 284. (4) 288. (3) 292. (1) 296. (5) 300. (1)

SBI PO EXAMS 1. (4) 5. (4) 9. (2) 13. (3) 17. (2) 21. (4) 25. (1) 29. (4) 33. (4) 37. (4) 41. (5) 45. (4) 49. (5) 53. (5) 57. (2) 61. (5) 65. (5) 69. (1) 73. (3) 77. (5)

2. (1) 6. (3) 10. (1) 14. (4) 18. (5) 22. (1) 26. (4) 30. (2) 34. (5) 38. (1) 42. (2) 46. (3) 50. (4) 54. (2) 58. (4) 62. (2) 66. (3) 70. (5) 74. (3) 78. (2)

3. (2) 7. (3) 11. (4) 15. (1) 19. (5) 23. (3) 27. (5) 31. (5) 35. (4) 39. (2) 43. (1) 47. (5) 51. (2) 55. (5) 59. (2) 63. (4) 67. (2) 71. (4) 75. (1) 79. (3)

4. (4) 8. (5) 12. (4) 16. (3) 20. (1) 24. (2) 28. (1) 32. (1) 36. (1) 40. (1) 44. (4) 48. (2) 52. (4) 56. (4) 60. (1) 64. (4) 68. (1) 72. (2) 76. (1) 80. (2)

81. (5) 82. (1) 83. (4) 85. (1) 86. (3) 87. (2) 89. (5) 90. (3) 91. (1) 93. (4) 94. (1) 94. (2) 97. (4) 98. (4) 99. (2) 101. (3) 102. (4) 103. (4) 105. (1) 106. (3) 107. (5) 109. (2) 110. (2) 111. (1)

84. (4) 88. (2) 92. (2) 96. (5) 100. (1) 104. (2) 108. (4) 112. (5)

113. (1) 117. (3) 121. (4) 125. (2) 129. (1) 133. (2) 137. (3)

114. (2) 118. (2) 122. (1) 126. (2) 130. (1) 134. (2)

115. (1) 119. (3) 123. (3) 127. (4) 131. (5) 135. (4)

116. (5) 120. (5) 124. (1) 128. (5) 132. (4) 136. (5)

NATIONALISED BANKS & IBPS PO/MT/SO EXAMS (1-5) :

RBI GRADE–B/ NABARD GRADE–A OFFICER EXAMS 1. (5) 5. (4) 9. (5) 13. (1) 17. (1) 21. (5) 25. (2) 29. (1)

2. (3) 6. (5) 10. (5) 14. (3) 18. (1) 22. (2) 26. (4) 30. (5)

3. (1) 7. (4) 11. (2) 15. (1) 19. (5) 23. (4) 27. (5) 31. (3)

4. (2) 8. (4) 12. (2) 16. (2) 20. (4) 24. (5) 28. (4) 32. (2)

33. (4) 37. (1) 41. (3) 45. (5) 49. (1) 53. (3) 57. (5) 61. (1) 65. (3) 69. (1) 73. (5) 77. (1)

34. (4) 38. (5) 42. (3) 46. (2) 50. (2) 54. (2) 58. (5) 62. (5) 66. (4) 70. (4) 74. (2)

35. (2) 39. (5) 43. (2) 47. (5) 51. (2) 55. (3) 59. (3) 63. (2) 67. (2) 71. (2) 75. (3)

36. (3) 40. (5) 44. (2) 48. (4) 52. (4) 56. (4) 60. (4) 64. (1) 68. (1) 72. (4) 76. (5)

INSURANCE EXAMS 1. (1) 5. (4) 9. (1) 13. (2) 17. (4) 21. (2) 25. (3) 29. (3) 33. (4) 37. (5) 41. (5) 45. (4) 49. (4) 53. (5) 57. (4) 61. (4) 65. (4) 69. (2) 73. (3)

EXPLANATIONS

2. (4) 6. (4) 10. (4) 14. (1) 18. (1) 22. (5) 26. (5) 30. (2) 34. (1) 38. (4) 42. (4) 46. (5) 50. (3) 54. (1) 58. (1) 62. (4) 66. (5) 70. (3) 74. (4)

3. (3) 7. (1) 11. (4) 15. (4) 19. (4) 23. (2) 27. (2) 31. (5) 35. (4) 39. (1) 43. (2) 47. (2) 51. (1) 55. (4) 59. (5) 63. (2) 67. (2) 71. (1) 75. (2)

BPRE–195

4. (3) 8. (2) 12. (1) 16. (3) 20. (2) 24. (5) 28. (5) 32. (3) 36. (3) 40. (2) 44. (1) 48. (3) 52. (3) 56. (2) 60. (3) 64. (4) 68. (3) 72. (5)

#⇒< ⇒> % ⇒ = $ ⇒ ≥ @⇒ ≤

1. (4) R @ D ⇒ R < D D©W⇒D>W B$W⇒B>W Therefore, RW D : Not True III. W $ R ⇒ W > R : Not True W is either smaller or greater than or equal to R. 2. (3) H $ V ⇒ H > V V%M⇒V=M K©M⇒K>M Therefore, H > V = M < K Conclusions I. K © V ⇒ K > V : True II. M @ H ⇒ M < H : True III. H © K ⇒ H > K : Not True 3. (1) K # T ⇒ K < T T$B⇒T>B B@F⇒B B < F Conclusions I. F $ T ⇒ F > T : Not True II. K # B ⇒ K < B : Not True III. T $ F ⇒ T > F : Not True 4. (1) Z # F ⇒ Z < F R@F⇒RR Therefore, ZR Z : Not True 5. (2) M © R ⇒ M > R R%D⇒R=D D@N⇒DR=D N : Not True II. N $ R ⇒ N > R : True III. M © D ⇒ M > D : True

SYMBOLS & NOTATIONS (6 – 10) :

$ ⇒ ≥ ⇒ < # ⇒ > % ⇒ ≤ @⇒ = 6. (4) R # J ⇒ R > J J$D⇒J>D D@K⇒D=K K%T⇒K< T Therefore, R > J > D = K < T Conclusions : I. T # D ⇒ T > D : Not True II. T @ D ⇒ T = D : Not True Either I or II is true. III. R # K ⇒ R > K : True IV. J $ T ⇒ J > T : Not True 7. (1) T % R ⇒ T < R R$M⇒R>M M@D ⇒M=D D©H ⇒D M = D < H Conclusions : I. D % R ⇒ D < R : True II. H # R ⇒ H > R : Not True III. T © M ⇒ T < M : Not True IV. T % D ⇒ T < D : Not True 8. (3) M @ B ⇒ M = B B#N⇒B>N N $ R ⇒N >R R©K⇒R N > R < K Conclusions : I. K # B ⇒ K > B : Not True II. R © B ⇒ R < B : True III. M $ R ⇒ M > R : Not True IV. N © M ⇒ N < M : True 9. (2) F # H ⇒ F > H H@M⇒H=M M © E ⇒M < E E$J⇒E>J Therefore, F > H = M < E > J Conclusions : I. J © M ⇒ J < M : Not True II. E # H ⇒ E > H : True III. M © F ⇒ M < F : True IV. F # E ⇒ F > E : Not True 10. (5) D % A ⇒ D < A A@B⇒A=B B©K⇒B A : True III. M # B ⇒ M > B : True IV. A © M ⇒ A < M : True

11. (1) W > E = A > R > S W > R : True E > S : True 12. (3) A > V > I > C = T A > I : True V > T : True (13– 14) : (i) P © Q ⇒ P >/ Q ⇒ P < Q (ii) P # Q ⇒ P ≥/ Q ⇒ P < Q (iii) P @ Q ⇒ P >/ Q, P Q

(v) P $ Q ⇒ P Q

©Þ< #Þ
$Þ> 13. (3) A N⇒A>N S$N⇒S>N S W ⇒S > W W©R⇒W < R Now, A > N < S > W < R Conclusions I. R $ A ⇒ R > A : Not True R ⇒ S > R : Not True II. S A ⇒ S > A : Not True III. S IV. W © A ⇒ W < A : Not True 14. (1) D # E ⇒ D < E E©T⇒E E : True III. A IV. T # A ⇒ T < A : True 15. (4) O $ B ⇒ O > B B@S⇒B=S S#E⇒S S : True II. R III. B # R ⇒ B < R : True IV. B # E ⇒ B < E : True 16. (2) D © S ⇒ D < S S P⇒S>P P$I⇒P>I P @ T⇒P=T Now, D < S > P = T > I Conclusions I. P # D ⇒ P < D : Not True II. P $ T ⇒ P > T : Not True III. T # S ⇒ T < S : True IV. I © D ⇒ I < D : Not True

BPRE–196

17. (5) U @ N ⇒ U = N N$F⇒N>F F$A⇒F>A A R ⇒A>R Now, U = N > F > A > R Conclusions I. A @ U ⇒ A = U : Not True R ⇒ N > R : True II. N III. R © U ⇒ R < U : Not True IV. U A ⇒ U > A : Not True U is either greater than or equal to A. Therefore, either I or IV is true. 18. (2) P > Q = R > S > T Conclusions : I. P > T : Not True II. T < Q : True 19. (4) L < M < N > O > P Conclusions : I. O < M : Not Ture II. P < N : Not True 20. (5) A > B > C = D > E Conclusions : I. C > A : True II. D < B : True 21. (1) H > J = K > L > T < V Conclusions : I. K > T : True II. L < H : Not True 22. (5) A < B = C = E < D Conclusions : I. E > A : True II. A < D : True 23. (5) J > K > L = M > N J > M and N < K 24. (2) A > B > C > D = F B > D and C > F A B and C > F 25. (4) I > J > K = L < N = M Now, I > L and M > K 26. (4) P < Q < R < S Now, S > P 27. (2) E > C is true. 28. (5) E < J < H > Z HF F N Now, N < K

SYMBOLS & NOTATIONS 32. (2) A > B = C < D < E Conclusions : I. D > A : Not true II. E > C : True 33. (5) L > U > Z R>U>K Conclusions : I. L > Z : True II. K < R : True 34. (1) Conclusions : I. J > I : True II. Y < R : Not true 35. (4) T < K > M = N V>K>M=N>S Conclusions : I. T < N : Not true II. V = S : Not true 36. (1) F < X < E FR Conclusions : I. F < E : True II. R < F : Not true 37. (3) G < F = L < J < K = H or, G < F = L < J < K = H or, G = F = L = J = K = H Therefore, G is either smaller than H or equal to H. 38. (2) P < R < S < T > U Conclusions : I. U < R : Not True II. T > P : True 39. (2) T > U > V > W = Y > Z T>U>V>W=Y>X Conclusions : I. Z > U : Not True II. W < T : True 40. (1) K < L < M < N KA>D E : Not True 42. (2) E = A < D > B > F = C Conclusions : I. E < B : Not True II. C < D : True 43. (4) P > T = U < W < R < Q Conclusions : I. T = R : Not True T < R or T = R II. U < R : Not True U < R or U = R

44. (4) Conclusions : I. P > R : Not True II. U > Q : Not True 45. (1) Y < R = P > S > W = X Conclusions : I. P > X : True II. Y < X : Not True 46. (1) G < I < C = D > E < F Conclusions : I. D > G : True II. E < I : Not True 47. (5) E < F < G = H > S Conclusion I : G > S : True Conclusion II : F < H : True 48. (1) P < Q < W = L Conclusion I : L > P : True Conclusion II : Q < L : Not True 49. (2) Conclusions : I. Z > C : Not True II. B < Z : True 50. (1) Conclusions : I. B < C : True II. R > C : Not True (51–52) : P>Q>T QA P>Q>T>A P>QQ>T>A 51. (1) Conclusions : I. A < Q : True II. A < P : Not True 52. (5) Conclusions : I. T < P : True II. R > A : True 53. (4) P > T = U < W Q>R>W P>T=UM;RR SX RX V>M=T>S 57. (1) Conclusions : I. R < V : True II. S < X : Not True 58. (2) Conclusions : I. X < R : Not True II. V > S : True (59–60) : F>AA>I I N : Not true 61. (5) F < U < E = L M>E>K F T < D Conclusions : I. S > T : Not true II. D < O : Not true (63–64) : N>J=M S>B>RB>R R : True 64. (3) Conclusions : I. N = M : Not true II. N > M : Not true Either I or II is true. 65. (2) S = T < A = N > D; BS=TTT N : True 77. (1) Statements K>H U > E; N J EJ D>M>U>N D>M>U>J Conclusions I. D > E : True II. E < J : Not True III. M > N : True IV. D > J : True 79. (3) Statement P>L=U>CS Conclusions I. P > C : True II. K < L : Not True III. S < U : Not True IV. C < L : True 80. (5) Statements K>G=MU UO>L>T=E>D M>O>L>T=E>D M=O=L=T=E=D

BPRE–198

81. (2) Conclusions I. D < O : True II. M > E : True 82. (1) Conclusions I. T < O : Not True II. T = O : Not True T is either smaller than or equal to O. Therefore, either Conclusion I or Conclusion II is true. 83. (5) B < C = D < X < Y < Z Conclusions I. B < X : True II. Z < C : Not True (84–85) : RO>L>T=E>D M>O>L>T=E>D M=O=L=T=E=D 87. (2) Conclusions I. D < O : True II. M > E : True 88. (1) Conclusions I. T < O : Not True II. T = O : Not True T is either smaller than or equal to O. Therefore, either Conclusion I or Conclusion II is true. 89. (5) B < C = D < X < Y < Z Conclusions I. B < X : True II. Z < C : Not True (90 – 91) : R O > N ; Y>L;Q>C Y>LO>N F=A N > A < S ; N>M>T R>K=H>N>M>T Conclusions I. K > T : True II. R > M : True 95. (4) S < P < C = D < L > K ; D K : No True (96–97) : SR L>Q QR 96. (3) Conclusions : I. P > S : True II. I > R : True 97. (5) Conclusions : I. L < R : Not True II. E > Q : Not True (98–99) : G>R>E=A C < D < E < F Conclusions : I. A > E : Not True II. C < F : True (101–102) : PS C>A P U > N = K < E < D Conclusions I. E < D : Not True II. D > N : True (104–105) : IK FC C : Not True II. E = C : Not True E is either greater than or equal to C. Therefore, either Conclusion I or Conclusion II is true. 114. (4) Conclusions : I. D > B : Not True II. E > S : True 115. (3) A > B > C < D < E < F Conclusions : I. C < F : True II. A > E : Not True (116–117) : C K = E < D Conclusions : I. O > D : Not True II. E < C : True 121. (5) S < P < U > N U>B L N < Y O>S>R A>G>O>S>R SN S : Not True 123. (1) M > O > C > K = E < D J>C OC>K=E Z>O>C>K=E Conclusions : I. J > E : True II. K < Z : True 124. (2) Q > U < I < E = T > S Q > T : Not True U < T : True E > S : E is either greater than or equal to S. Therefore, it is not true. E > Q : Not True I > S : Not True (125-126) : V > M = T > X RS V>M=T>R SX RX V>M=T>S 125. (4) Conclusions I. R < V : True II. S < X : Not True 126. (5) Conclusions I. X < R : Not True II. V > S : True 127. (4) P < E < R > F E>M R R V>H D : True II. R < J : True 130. (1) L < Q < R = S R>H>P P U < K U>Y W>A=S>U>Y Conclusions I. K > W : Not True II. Y < W : True 132. (1) Y < U < W > S M>W>D YN=G>Y Now, V > Y : True I < L : True (147–148) : L P : Not True II. P = D : Note True D is either greater than or equal to P. Therefore, either Conclusion I or Conclusion II is true. (150–151) : T>I>L>E ID N>I>L>E T>I D : Not True

(152–153) : SR L>Q QR 152. (3) Conclusions : I. P > S : True II. I > R : True 153. (5) Conclusions : I. L < R : Not True II. E > Q : Not True (154–155) : G > R < E = A < T < S D E : Not True II. C < F : True 157. (2) F < D > R N

    E TN>T C R : Not True II. T > E : Not True 162. (5) P < E < T < R T>K PC>H=W A>C>H=W>X Conclusions : I. C > X : True II. A > W : True 164. (2) J < L < B < S > Y < M Conclusions : I. J < M : Not True II. L > Y : Not True (165–166) : B I < T < C Conclusions : I. W > T : Not True II. Q > C : Not True 172. (3) T = V < R < B TT=V R : Not True II. B < T : Not True (173–174) : RE Y>O>C RE 173. (4) Conclusions : I. Y > R : True II. E < C : Not True 174. (2) Conclusions : I. Y = E : Not True II. Y > E : Not True Y is either greater than or equal to E. Therefore, either Conclusion I or Conclusion II is true. 175. (4) P > R = S > Q MR>M MQ Conclusions : I. M < P : True II. Q < M : Not True 176. (5) H < M > I > N = C > R Conclusions : I. C < H : Not True II. M > R : True (177–178) : ● P ≠ Q ⇒ P >/ Q ⇒ P < Q ● P © Q ⇒ P Q ● P % Q ⇒ P / Q ⇒ P = Q ● P $ Q ⇒ P Q ● P @ Q ⇒ P >/ Q ⇒ P < Q 177. (5) Z ≠ F ⇒ Z < F R@F⇒RR ⇒ RR D©W⇒D>W ...(ii) ...(iii) B$W⇒B>W Conclusions : I. W ≠ R ⇒ W < R : Not True (Comparison not possible) II. B © D ⇒ B > D : Not True (Comparison not possible) III. W $ R ⇒ W > R : Not True (Comparison not possible) ...(i) 179. (2) H $ V ⇒ H > V V%M⇒V=M ...(ii) K©M⇒K>M ...(iii) Combining the statements (i), (ii) and (iii); ... (iv) K>M=V V : True II. M @ H ⇒ M < H : True III. H © K ⇒ H > K : Not True (Comparison not possible) (180–181) : T>R>A>CC>S EA>C>S>D T>R>A>C>E>N 180. (2) Conclusions : I. S > A : Not True II. K < E : Not True 181. (3) Conclusions : I. D < T : True II. N < R : True 182. (1) S = L < U = M Z>R>C=M S=LI W : True 185. (3) J > N > U = Y < R < B < O< F F > B : True Y < N : True Y < J : True U < F : True But, U > B : Not True U is smaller than B. 186. (3) Option (1) S>WWE=RW Conclusions I. S > N : Not True II. W < N : True 195. (5) Option (1) E>F>R>B=C>H ⇒ F>H Option (2) N>H>A=CF No relation can be established between H and F. Option (3) H>R>M=PF

    No relation can be established between H and F. Option (4) FE Conclusions I. W > K : True II. E > U : Not True 209. (4) J > X > E = Z L>E J>X>EE=Z Conclusions I. J > L : Not True II. L > Z : True 210. (3) R > B > F > T > N = W Conclusions I. F > W : True II. R < T : Not True 211. (2) D > C < G > H B>C B>CH D>C B : Not True II. B < D : Not True 212. (1) N < Q < C = D > S > A Conclusions : I. C > A : True II. N < D : True (213–214) : S = T < R < A > Y Z>P=R Z>P=RY S=T A > C = E KK Conclusions : I. E < U : True II. P > K : Not True 216. (3) B < O = L < D P>C>A=L BCD T>R>A>C>E>N

    218. (2) Conclusions : I. S > A : Not True II. K < E : Not True 219. (3) Conclusions : I. D < T : True II. N < R : True 220. (1) S = L < U = M ZC=M S=LI R > O = S < A = Q Conclusions I. P > S : True II. O < Q : True 224. (2) R < A < C = E < D or, R < A < C = E < D or, R < A < C = E < D or, R < A < C = E < D or, R = A = C = E = D Conclusions I. R < D : Not True II. R = D : Not True R is either smaller than or equal to D. Therefore, either Conclusion I or Conclusion II is true. 225. (3) V > E = N > I > C < E Conclusions I. C < V : True II. N > E : Not True 226. (5) C < D < L < F > G K>L>M CLG Conclusions I. C < M : Not True II. K > G : Not True

    BPRE–204

    227. (4) J < L < P < E K>P>M MC T>O>C>Z Conclusions I. C < V : True II. T > Z : Not True 229. (4) G > O < A < L SM=E S S : True II. E < G : True 230. (3) H < A = F < T J=A>L>E HL>E L O > U > M δ Y G>U#FO>U>M>Y G>UU>M>Y (232 – 233) : S>M>AD>U>Z M>A>D>U S>M>AY>A>D>U>Z 232. (5) Conclusions : I. M > U : Not True II. M = U : Not True M is either greater than or equal to U. Therefore, either Conclusion I or Conclusion II is true. 233. (4) Conclusions : I. S > Q : Not True II. Z < G : True (234 – 235) : LI>MT>L>B>K W>P IE A>L>M or, A > L > M or, A > L > M or, A = L > M or, A = L = M or, A > L = M 243. (4) Conclusions I. S > E : True II. L < S : Not True 244. (1) Conclusions I. A > M : Not True II. A = M : Not True A is either greater than or equal to M. Therefore, either Conclusion I or Conclusion II is true. 245. (2) P > V > R < E < Y G>E>N P>V>RN Conclusions I. P > N : Not True II. V > Y : Not True (246–247) :

    → W   → Q δ  But,

    F>I>T=C TP I>R>K NP L>I> R I=N>R>K I=N N : Not True II. I < Z : True

    P6I

    I. T = J : Not True II. J > T : Not True J is either greater than or equal to T. Therefore, either Conclusion I or Conclusion II is true. (248–249) : SK=M F>J>K=MK=M=E 248. (5) I. II. 249. (1)

    Conclusions : G > J : Not True G < F : Not True Conclusions :

    I. K > S : True II. E < F : True

    BPRE–205

    13 is a Prime Number (Outcome IV) ∴ Outcome = 55 + 56 + 65 = 176 (Condition IV) Therefore, bulb R blinks.

    SYMBOLS & NOTATIONS 255. (2) W > V < T < P > Q ; T>K>J>N; K>Z>H=D DN; NP N>J=H>P Conclusions I. N > P : True II. K < W : True 258. (1) W > U > P = J < K < Q < V S>U>T TP=J E = J > K < C < B EKE=J Conclusions : I. A = C : Not True II. G > J : True 260. (5) D < I = K < H G M W>K E>X G F : Not True 262. (2) Q = S < N < P < R > M W>Q R>X W>Q=S P : Not True 264. (1) A > B > C < D Conclusions : I. A > C : True II. A = C : Not True 265. (1) M > A > B = Q < P < J < Y Z>A>X XB=Q A > B = Q Conclusions : I. Z = Q : Not True II. Z > Q : Not True Z is either greater than of equal to Q. Therefore, either Conclusion I or Conclusion II is true. 267. (5) G < R = A < S TG Conclusions I. D < H : Not True II. K > G : True

    BPRE–206

    272. (2) R > I = N > P Y>R>K N D J>W=G>M=T Conclusions : I. W > D = True II. J > T = True 274. (5) G < F = D > B > C A=B A = Not True 275. (1) X > M < D = Y or, X > M < D = Y or, X = M = D = Y or, X > M < D = Y or, X = M < D = Y D=Y Y N=B Y = Not True II M > N : True 277. (4) K > U > D < R = M < B X=A>K X>K>U>D A>K>U>D D : Not True II. R > A : Not True 278. (2) L = A > Q > B = V > X P>B=R L=A>Q>B

    X Conclusions I. P > L : Not True II. R > X : True 279. (5) Y > H < E = Q < L = K H>B=M B=M H > Z C>X=M C>X=M>Q=D>H Conclusions I. M > H : True II. C > H : Not True 281. (1) B < A < T = E > D > F Y=X>T Y=X>T=E>D>F Conclusions I. Y > D : True II. X < F : Not True 282. (1) T > R > U > V M>P=T UP=T>R T>R>U A = N > F RY Conclusions I. J > Y : Not True II. J = Y : Not True J is either greater than or equal to Y. Therefore, either Conclusion I or Conclusion II is true. 284. (4) P > M > D > S DB P>M>DS>T AT U A > T SA>T=E>S Conclusions I. A > E : Not True II. C > S : True 287. (1) F < U < N D>H I < N Conclusions I. I < L : Not True II. L = I : Not True L is either greater than or equal to I. Therefore, either Conclusion I or Conclusion II is true. 289. (4) G < L > O > W > I < N Conclusions I. O > G : Not True II. W < N : Not True 290. (4) R > M > T < Q = S Conclusions I. R > Q : Not True II. Q > M : Not True 291. (5) Q > P > S > B B=C=D>Y Conclusions I. Q > B : True II. Y < C : True 292. (1) B = K > T = D T=D>H>M Conclusions I. B > D : True II. T < M : Not True 293. (1) D < U = G < L P D : True II. Y > P : Not True 294. (4) T > J < G < A = D > C Conclusions I. J < D : Not True II. T < C : Not True 295. (2) L < D > Q = M G=D>Q>R>V Conclusions I. L < M : Not True II. G > V : True (296–300) : (i) A @ B ⇒ A > B : A < B (ii) A + B ⇒ A < B : A > B (iii) A © B ⇒ A

    B:A>B

    (iv) A % B ⇒ A > B : A < B (V) A # B ⇒ A > B. A B : A = B 296. (5) M + H : M > H K%M:K G : Not True

    BPRE–207

    297. (3) E @ F : E < F D%E:DF DG>U>L Conclusions : I. M # U : M = U : Not True II. M + U : M > U : True 300. (1) Z + U : Z > U P©W:P>W W@U:WW B > C

    =

    D D : True F > C : True 2. (1) P > Q = R Q=R O = A > S < T O > T : Not True S < R : True T > A : Not True S = O : Not True as O > S T < R : Not True 4. (4) P > L



    A>N=T

    P > A : True T=N D Now, B > N and D < L

    SYMBOLS & NOTATIONS 6. (3) A < L < P > N 7. (3) F < O = U = N < D Now, F < N and U < D (8–9) : AC AA>N N : True II. L < K : Not True 11. (4) Conclusions I. L > N : True II. L = N : Not True (12–13) : A>B>CCC=D=E=F 12. (4) Conclusions I. C < F : Not True II. C = F : Not True F is either greater than or equal to C. Therefore, either Conclusion I or II is true. 13. (3) Conclusions I. A > D : Not True II. B < E : Not True (14-15) : ● P % Q ⇒ P Q ● P

    Q ⇒ P >/ Q ⇒ P < Q

    ● P $ Q ⇒ P Q ● P # Q ⇒ P >/ Q; P ≠ Q Therefore, P < Q ● P @ Q ⇒ P >/ Q; P R : True IV. D @ M ⇒ D = M : Not True 15. (1) R $ B ⇒ R > B B@A⇒B=A A%K⇒A>K K#M⇒KB=A>K A : Not True II. K $ B ⇒ K > B : Not True III. A % R ⇒ A > R : Not True IV. K # R ⇒ K < R : True 16. (3) D # M ⇒ D < M M$R⇒M>R R@J⇒R=J W%J⇒W>J Therefore, DR=J R : True II. M $ J ⇒ M > J : True III. R % D ⇒ R > D : Not True IV. W % M ⇒ W > M : Not True 17. (2) W @ T ⇒ W = T T$N⇒T>N N#F⇒NF Therefore, W=T>N N : True III. T $ F ⇒ T > F : Not True IV. V @ N ⇒ V = N : Not True 18. (5) B % K ⇒ B > K K#D⇒K K : True (19-20) : O=LS L>I TI

    BPRE–208

    I P < S Conclusions : I. G < U : Not True II. S > O : Not True 22. (1) A > L < P < B P>M A>L

    M M M : Not True (23-24) : I>M=P>R B

    A I>M=P>B I>M=P>A AR BR 23. (3) Conclusions : I. A < I : Not True II. I = A : Not True A is either smaller than or equal to I. 24. (2) Conclusions : I. A < R : Not True II. B < I : True 25. (1) B > C = D > X > E B>C=D E : True II. Z > B : Not True 26. (4) E > F > G < H < I < J Conclusions : I. G < E : Not True II. J > F : Not True 27. (5) K < L < M > N > O KO Conclusions : I. T > K : True II. P > O : True (28–29) : B>O=K>L D>K>S B>O=K>S D>K=O>L SL

    SYMBOLS & NOTATIONS 28. (1) Conclusions : I. O < D : True II. S < L : Not True 29. (4) Conclusions : I. L > D : Not True II. B > S : Not True 30. (2) A > B = C < M < D < E Conclusions I. A > D : Not True II. E > B : True 31. (5) Z = Y > X > W < V < T Conclusions I. Z > W : True II. W < T : True 32. (1) N < M < O > P = Q > R Conclusions I. O > R : True II. N < P : Not True 33. (4) T > M = B < Q > F > R Conclusions I. M > R : Not True II. F < T : Not True 34. (5) O = P < Q < R = S < T Conclusions I. T > Q : True II. O < S : True 35. (4) J < U < M > P < R Conclusions : I. J < R : Not True II. M < J : Not True 36. (1) T > R > E = Q NY T>R>N NE=Q E=QY T>R>Y Conclusions : I. N < T : True II. Y < Q : Not True 37. (4) S > T = U > V W T < X < Y W U > V W>V PU>VU>V>P Conclusions : I. P < T : True II. W < T : Not True

    39. (2) A > B < M = Z N>B>Q N> B < M = Z Q M > O M < N; O > F L>M>O>F N>M>O>F L>MN N : True II. L < K : Not True 44. (4) Conclusions I. L > N : True II. L = N : Not True (45–46) : A>B>CCC=D=E=F 45. (4) Conclusions I. C < F : Not True II. C = F : Not True F is either greater than or equal to C. Therefore, either Conclusion I or II is true. 46. (3) Conclusions I. A > D : Not True II. B < E : Not True (47–48) : BK>S 47. (5) Conclusions : I. A > S : True II. B < K : Not True 48. (2) Conclusions : I. B < N : True II. S < N : True (49–50) : K>L=M>N=O O : True (51-52) : XZ X J : Not True II. K > Y : True (53 – 54) : BK>S 53. (5) Conclusions : I. A > S : True II. B < K : Not True 54. (2) Conclusions : I. B < N : True II. S < N : True (55–56) : K>L=M>N=O

    O : True (57-58) : XZ X J : Not True II. K > Y : True (59–60) : Y S Conclusions : I. S < W : True II. S = W : Not True (91–92) : R T : Not True II. C < Q : Not True 93. (4) R < O < A < M L=D>A R A < F < T Conclusions : I. R < T : Not True II. D > F : Not True 99. (2) R > S = T > U S>A>V R>S>A VE>A>T=HJ>I S G > P IK>G>P Conclusions : I. I < G : Not True II. W > P : True 105. (1) M > N = O Y=Q>P>O>R M>N=O P > N < S N=X>D D K > L < M LLAL>C C < P : Not True 109. (2) P > A < R < O < T L G IO>R>T CR>T Conclusions : I. C < O : Not True II. G > T : Not True 113. (1) V > U > L > T > R M>O>V

    O>V>U>L>T>R R < M : True 114. (2) S < P < A > R > O E>L>AR>ECEEC>R 115. (1) Conclusions : I. N > P : Not True II. G > A : Not True

    SYMBOLS & NOTATIONS 116. (5) Conclusions : I. I > R : Not True II. I = R : Not True I is either greater than or equal to R. Therefore, either Conclusion I or Conclusion II is true. 117. (3) Option (1) Q U > V Y M > N X>LL>M>N JL J : Not True

    RBI GRADE–B/ NABARD GRADE–A OFFICER EXAMS 1. (5) C > R = O < S = T > M O < M : Not True C > M : Not true R > M : Not true C > S : Not true T > R : True 2. (3) I < J < K = M > N = O Now, O < K : True M > I : True #⇒=

    (3–7) :

    δ ⇒≥

    @⇒ >

    ∗ ⇒≤ %⇒
    H H ★ N⇒H < N Therefore, D < F > H < N Conclusions : I. N @ F ⇒ N > F : Not True II. D % N ⇒ D < N : Not True III. H % D ⇒ H < D : Not True

    4. (2) B δ D ⇒ B > D D%T⇒D T : Not True II. M @ D ⇒ M > D : True III. B @ M ⇒ B > M : Not True 5. (4) K # W ⇒ K = W M@W⇒M>W R δM ⇒R>M Therefore, K=W K KδT⇒K>T T # J ⇒T = J Therefore, M>K>T=J Conclusions : I. J # K ⇒ J = K : Not True II. M @ J ⇒ M > J : True III. J % K ⇒ J < K : Not True J is either smaller than or equal to K. Therefore, either Conclusion I or III follows. 7. (4) R ★ N ⇒ R < N N%B⇒N B < C < D > F Conclusions : I. F > B : Not True II. A > D : Not True 9. (5) P > X > Y = Q > Z Conclusions : I. Z < P : True II. P > Q : True 10. (5) L > I; H > I > J > K Conclusions : I. H > L : True II. L > K : True 11. (2) O > P = Q > R; O > P = Q > S Conclusions : I. R < S : Not True II. O > S : True 12. (2) D > E = H > F = G D>E=H D : Not True II. G < J : True 13. (1) B > J > R > Z B>J>R>F F : True II. B > W : Not True 14. (3) E > F = G > T = I Conclusions : I. I < E : Not True II. I = E : Not True I is either smaller than or equal to E. Therefore, either Conclusion I or II follows. 15. (1) G > H < T I>F H>J G>H>J JW W : True 17. (1) M > N > P O>P S

    N>PN>P>S O>P>S Conclusions : I. S < M : True II. O < M : Not True 18. (1) M > A > E > F < G Conclusions : I. M > E : True II. G < A : Not True 19. (5) V = I < T < A = L > Z > E Conclusions : I. A > E : True II. L > V : True (20-21) : P>H=J> R HR 20. (4) Conclusions : I. J > T : Not True II. P > T : Not True 21. (5) Conclusions : I. R < T : True II. T > H : True

    SYMBOLS & NOTATIONS (22-23) : C>HY C>HG W < S; W > M MM WG 24. (5) Conclusions I. G < B : Not True II. M > S : Not True 25. (2) Conclusions I. M < U : True II. G < W : Not True 26. (4) K > B = N < U T < N; P > U K>B=NT Conclusions I. P > K : Not True II. T < K : True 27. (5) Option (1) E>Z>R>B=C>Q ⇒ Z>Q Option (2) N>Q>A=CZ No relation can be established between Q and Z. Option (3) Q>R>M=PZ No relation can be established between Q and Z. Option (4) ZU>IL>G Q>U>I K : Not True II. H < U : Not True (36–37) : M < O = N < K < E < Y = S; DB MA S>A=I>N Conclusions : I. S > N : Not True II. N = S : Not True S is either greater than or equal to N. Therefore, either Conclusion I or Conclusion II is true.

    BPRE–214

    (39–40) : C>VT>Q Y>NN>T>Q C>VT>Q 39. (5) Conclusions I. Q > V : Not True II. Q < Y : True 40. (5) Conclusions I. C > Y : Not True II. A > Q : True (41–42) : P>R V : True V < G : True 45. (5) D > I > H = S = M < P < R Now, H < R : True D > M : True (45–46) : P< LD>I KY R>U YJ YJ KY 48. (4) Conclusions I. R > Y : True II. J < R : True 49. (1) Conclusions I. M < Y : Not True II. Y > K : Not True 50. (2) U > B = N > T < C < R < E (1) T < E : True (2) T > U : False (3) E > C : True (4) N < U : True (5) R > T : True 51. (2) (i) M < N < Y < P = Z < E = B M < Z : True Y < B : True (ii) M < N < Y < P < Z < E = B M < Z : True Y < B : True 52. (4) Option (1) T>L>M R>O>M T>L>MLM R>O>M T>L>MM RM>O>R Therefore, R < T : True

    Option (5) T>L=M RO>R Therefore, R < T : Not True 53. (3) P = U = E > J E=MRR R = B ≤ S CZ P>R=B>Z Z W Q>P DP Conclusions : I. N = W : Not True II. N > W : Not True N is either greater than or equal to W. Therefore, either Conclusion I or Conclusion II is true. 62. (5) J < O < I > N = E > D Conclusions : I. J < E : Not True II. I > D : True (63–64) : ON=L>A K U < W = Z U>O C>U>O O O : True 66. (4) W < Y < I < B < F Conclusions : I. W < F : True II. W = F : Not True (67–68) : S=UX>Y R R : Not True II. Y < U : True 72. (4) P > Q > R > S R@JQ>R>J ⇒ P > J is True S K : True 74. (2) F > Y > N = E > H NY>N W : Not True II. H < Y : True 75. (3) Option (1) S > W < T ; P > W < H; H < I S>WW W < H; H > I S D > C ⇒A=B>C B = C; E > D > C ⇒A>B=C D < C ⇒A D > C ⇒A=B R > Q = P; S < O < T ⇒T>O>S>R>Q=P P < T : True

    INSURANCE EXAMS (1–5) :

    + ⇒> – ⇒≥ × ⇒= ÷ ⇒< = ⇒≤

    1. (1) M × X ⇒ M = X L÷O⇒LM L=X⇒L M = X > L < O Conclusions : I. N + L ⇒ N > L : True II. X ÷ O ⇒ X < O : Not True

    BPRE–216

    2. (4) S = F ⇒ S < F K÷S⇒KB Therefore, R > B = K < S < F Conclusions : I. F + K ⇒ F > K : True II. K ÷ R ⇒ K < R : True 3. (3) H = V ⇒ H < V A×Z⇒A=Z H–Z⇒H>Z F+V⇒F>V Therefore, F > V > H > Z = A Conclusions : I. A × H ⇒ A = H : Not True II. H + A ⇒ H > A : Not True H is either greater than or equal to A. 4. (3) I + O ⇒ I > O R–I⇒R>I Z×R⇒Z=R J÷Z⇒J I > O Conclusions : I. Z × I ⇒ Z = I : Not True II. Z + I ⇒ Z > I : Not True Z is either greater than or equal to I. 5. (4) Q ÷ R ⇒ Q < R C+D⇒C>D F–R⇒F>R D×F⇒D=F Therefore, C > D = F > R > Q Conclusions : I. Q ÷ C ⇒ Q < C : True II. C + F ⇒ C > F : True 6. (4) There are 32 elements each in both the series. The 7th to the right of 17th element of the from the left means 24th from the left. As series has been reversed the 24th from left means 24th fromthe right in the original series, i.e., 9th from the left in the original series. 9th from the left in Series I ⇒ R Sixth to the right of 13th element from the right means 7th from the right. Now, 3rd to the left of the 7th from the right means 10th from the right. But, the last 16 elements have been written in the reverse order in Series II. Therefore, required element would be 7th from the right in the original sequence, i.e., (11).

    SYMBOLS & NOTATIONS 7. (1) Series–I 0+4+8+5+0+0+2+3+6 + 5 = 33 Series–II 5 + 5 + 0 + 11 + 22 + 0 + 12 + 24 + 5 + 5 = 89, 33 + 89 = 122 8. (2) Series–I

    Number Number Letter There is only one such combination :

    23R

    Number Symbol Letter There is no such combination. 9. (1) y ⇒ z ; z ⇒ (17) There are 33 elements between z and (17). Midway between z and (17) ⇒ ª 10. (4) –4

    N

    –1

    P

    –6

    ¥

    –1

    U

    –1

    2

    +1

    ¤

    –4

    M

    –2

    3

    +5

    7

    –5

    n

    Å

    –19

    1

    P

    +2

    6

    11. (4) There are 32 elements each in both the series. The 7th to the right of 17th element of the from the left means 24th from the left. As series has been reversed the 24th from left means 24th fromthe right in the original series, i.e., 9th from the left in the original series. 9th from the left in Series I ⇒ R Sixth to the right of 13th element from the right means 7th from the right. Now, 3rd to the left of the 7th from the right means 10th from the right. But, the last 16 elements have been written in the reverse order in Series II. Therefore, required element would be 7th from the right in the original sequence, i.e., (11). 12. (1) Series–I 0+4+8+5+0+0+2+3+6 + 5 = 33 Series–II 5 + 5 + 0 + 11 + 22 + 0 + 12 + 24 + 5 + 5 = 89 13. (2) Series–I

    Number Number Letter There is only one such combination :

    23R

    Number Symbol Letter There is no such combination. 14. (1) y ⇒ z z ⇒ (17) There are 33 elements between z and (17). Midway between z and (17) ⇒ ª 15. (4)

    o

    Series–II

    o

    Series–II

    –4

    N

    –1

    P

    –6

    ¥

    –1

    U

    –1

    2

    +1

    ¤

    –4

    M

    –2

    3

    +5

    7

    –5

    Å

    1

    P

    +2

    6

    16. (3) H > M = K > T > J ⇒ H > T and J < M 17. (4) O > P > Q > R R P : Not True 18. (1) A < B < C AE>F GF Conclusions : I. D > A : True II. A < E : Not True (19–20) : H>I=J>K>L N>M>I=J>K>L H>I=J K : True 21. (2) A < B < C < D = E < F > G Conclusions : I. G < C : Not True II. F > B : True 22. (5) A = B > C = D > E ; GH A=B>C>H A=B>C=D>E=F Conclusions : I. A > H : True II. B > F : True 23. (2) P > Q > R > S < T > U > V Conclusions : I. Q > V : Not True II. P > S : True 24. (5) J > K > L ; NM

    BPRE–217

    n

    –19

    J>K>L>M J>K>L>N Conclusions : I. K > M : True II. J > N : True 25. (3) A = B > C = D > E ; GH A=B>C=D>E=F>G Conclusions : I. G = A : May be True II. G < A : May be True G is either equal to A or smaller than A. Therefore, either Conclusion I or II is definitely true. 26. (5) T < D < S < Y ; D > R > L ; S A : Not True II. X < P : True (33–34) : L>M>N Z : Not True II. M = Z : Not True M is either greater than or equal to Z. Therefore, either Conclusion I or II is true. 34. (1) Conclusions I. L < P : Not True II. O > L : Not True 35. (4) S = T > U < R < E U>M>Y S=T>U>M S=T=U=M S=T>U=M S =T>U>M S=T=U>M Conclusions I. S > M : Not True II. M = S : Not True S is either greater than or equal to M. Therefore, either Conclusion I or II is true. (36–37) : K>J>MU=H;J>T H=UT 36. (3) Conclusions I. K > O : Not True II. O > H : True 37. (5) Conclusions I. K > T : True II. J < O : Not True 38. (4) R > W < F = Z Conclusions I. F < R : Not True II. Z < W : Not True 39. (1) B > K > J < M Conclusions I. J < B : True II. M > B : Not True 40. (2) D = T < H > N Conclusions I. H < D : Not True II. H > D : True 41. (5) H < N < K < D Conclusions I. D > N : True II. H < K : True 42. (4) W > E > K = J Conclusions I. J < E : Not True II. W > K : Not True 43. (2) R < M = B > T Conclusions I. R < T : Not True II. T < M : True

    (44–45) : B>E>AA>S B>E>AAE>A>S L>A>S 44. (1) Conclusions I. T > B : Not True II. L > T : Not True 45. (4) Conclusions I. B > S : True II. L > S : True (46–47) : L=I>NS SS L=I>N>S L=I=N=S L=I>N=S L=I=N>S 46. (5) Conclusions I. S < E : True II. E > L : Not True 47. (2) Conclusions I. S = L : Not True II. L > S : Not True L is either greater than or equal to S. Therefore, either Conclusion I or Conclusion II is true. 48. (3) V < E > B = H > N BN Conclusions I. T > V : Not True II. N < T : True (49 – 50) : PT RM M

    R>T PB=CC>R A>B=CC B : True II. S > R : True 52. (3) Conclusions : I. R < A : True II. Q > S : Not True 53. (5) K < I < H > D > R Z R : Not True 54. (1)

    ×⇒+ –⇒÷ ÷⇒– +⇒×

    Option (1) 16 + 5 – 10 × 4 ÷ 3 = 9 ⇒ 16 × 5 ÷ 10 + 4 – 3 = 9 ⇒8+4–3=9 Option (2) 16 – 5 × 10 ÷ 4 + 3 = 12 ⇒ 16 ÷ 5 + 10 – 4 × 3 = 12 ⇒

    16 + 10 – 12 ≠ 12 5

    Option (3) 16 + 5 ÷ 10 × 4 – 3 = 9 ⇒ 16 × 5 – 10 + 4 ÷ 3 = 9 ⇒ 80 – 10 +

    4 ≠9 3

    Option (4) 16 × 5 ÷ 10 ÷ 4 – 3 = 19 ⇒ 16 + 5 – 10 – 4 ÷ 3 = 19 ⇒ 16 + 5 – 10 +

    4 ≠ 19 3

    –⇒÷ +⇒×

    55. (4) ÷ ⇒ – × ⇒ +

    Option (1) 19 + 5 – 4 × 2 ÷ 4 = 11 ⇒ 19 × 5 ÷ 4 + 2 – 4 = 11 ⇒ 19 ×

    5 + 2 – 4 ≠ 11 4

    Option (2) 19 × 5 – 4 + 2 ÷ 4 = 16 ⇒ 19 + 5 ÷ 4 × 2 – 4 = 16 ⇒ 19 +

    5 × 2 – 4 = 16 4

    ⇒ 19 +

    5 – 4 ≠ 16 2

    SYMBOLS & NOTATIONS Option (3) 19 + 5 + 4 – 2 × 4 = 13 ⇒ 19 × 5 × 4 ÷ 2 + 4 = 13 ⇒

    380 + 4 ≠ 13 2

    Option (4) 19 × 5 ÷ 4 + 2 – 4 = 22 ⇒ 19 + 5 – 4 × 2 ÷ 4 = 22 ⇒ 19 + 5 – 4 ×

    2 = 22 4

    ⇒ 19 + 5 – 2 = 22

    56. (2)

    −⇒÷ +⇒− ÷⇒× ×⇒+

    70 – 2 + 4 ÷ 5 × 6 ⇒ 70 ÷ 2 – 4 × 5 + 6 ⇒ 35 – 20 + 6 ⇒ 41 – 20 = 21 (57–58) : CI L>N CL>N 57. (4) Conclusions I. D > L : True II. N < I : True 58. (1) Conclusions I. D > C : Not True II. C = D : Not True D is either greater than or equal to C. Therefore, either Conclusion I or Conclusion II is true. 59. (5) P > A = R < E < N R>T P>A=R>T Conclusions I. A > N : Not True II. T < P : True 60. (3) M > U = S > T RD M>U=S>D RT Conclusions I. M > D : True II. T > R : Not True 61. (4) H < E < R > O RR>O HZ T O : Not True (2) C = T : Not True (3) O = Z : Not True (4) T < P : True (5) Z < T : Not True 63. (2) X > Y > Z A>B>YZ A>B>YV V V : Not True V may be greater or smaller than S or equal to S. Therefore, either Conclusion I or Conclusion II is true. 69. (2) H < A = L < P B=A>K>Y

    BPRE–219

    HK>Y KC>T=Y GA=N MOE Conclusions : I. J < H : True II. G > E : True 73. (3) P > Q = W Q>O>U O>VQ>O>U P>Q>O>V Conclusions : I. P > U : True II. V < Q : True 74. (4) Y > U > V < O J>G>H=Y G>H=Y>U>V O II. O > G G may be smaller or greater than O or equal to O. Therefore, either Conclusion I or Conclusion II is true. 75. (2) P > Q > F > I Y>F>U P>Q>F>U P>Q>F Y : Not True II. P > U : True ❐❐❐

    SYMBOLS & NOTATIONS

    MODEL EXERCISES Directions (1-4) : In an imaginary language, the digits 0, 1, 2, 3, 4, 5, 6, 7, 8 and 9 are substituted by a, b, c, d, e, f, g, h, i an j. And 10 is written as ba. 1. (cd + ef) × be is equal to (1) 684 (2) 816 (3) 916 (4) 1564 (5) None of these 2. dc × f – (bf – d) × d is equal to (1) bce (2) bcf (3) abe (4) abb (5) None of these 3. baf ÷ bt × d is equal to (1) d (2) be (3) cb (4) df (5) None of these 4. bee+ fg – (ca × h / be) is equal to (1) bhc (2) bib (3) bja (4) bic (5) None of these Directions (5–7) : Read the following information to answer these questions : I. P Ψ Q means P is mother of Q. II. P ε Q means P is sister of Q. III. P $ Q means P is father of Q. IV. P # Q means P is brother of Q. 5. Which of the following means D is definitely daughter of A ? (1) A $ B # C # D (2) C Ψ A $ D ε B (3) A Ψ C $ B ε D (4) B Ψ A $ C # D (5) None of these 6. Which of the following means R is brother of T? (1) R Ψ S # U $ T (2) U Ψ R # S # T (3) U Ψ R ε S # T (4) K # R $ S ε T (5) None of these 7. Which of the following means A is nephew of C ? (1) D # C $ B # A ε E (2) A # B $ D ε E $ C (3) C # D $ B # A $ E (4) B Ψ E # C $ E ε A (5) None of these Directions (8–12) : Study the following arrangement carefully and answer the questions given below : M4PA3%R5#EJ2$DF1 U H B @ 9 T W I 8 K N 6 © V 7 ZQ

    8. If all the symbols in the above arrangement are dropped, which of the following will be the seventeenth from the left end ?

    9.

    10.

    11.

    12.

    (1) 1 (2) F (3) 9 (4) B (5) None of these Which of the following is the eighth to the right of the twentieth from the right end of the above arrangement ? (1) T (2) % (3) 2 (4) 6 (5) None of these How many such consonants are there in the above arrangement each of which is immediately preceded by a number but not immediately followed by another consonant ? (1) None (2) One (3) Two (4) Three (5) More than three Four of the following five are alike in a certain way based on their positions in the above arrangement and so form a group. Which is the one that does not belong to that group ? (1) 2DE (2) 9BW (3) PM3 (4) 6VK (5) R#J How many such numbers are there in the above arrangement each of which is immediately preceded by a consonant and immediately followed by a symbol ? (1) None (2) One (3) Two (4) Three (5) More than three

    13. Which of the following symbols should be placed in the blank spaces respectively (in the same order from left to right) in order to complete the given expression in such a manner that both ‘D > S’ as well as ‘E < B’ definitely holds true? B __ A __ S __ E __ D (1) >, >, , =, >, < (3) >, >, >, < (4) >, =, >, < (5) Other than those given as options 14. In which of these expressions ‘S

    BPRE–220

    > V’ be definitely false? (1) S > P > Q = G > R > V (2) P < A < S < T; V > O > T (3) V < A < L = R < S (4) S > C > F < H; V < F (5) S > T = O > P; V < J = P Directions (15 – 19): Study the following statements and all the Conclusions and then decide which of the given Conclusions logically follows from the given statements. A&B A%B A#B A@B A$B A!B A>B

    All As are Bs Some As are Bs Some As are not Bs No A is B No A being B is a possibility Some As being Bs is a possibity All As being Bs is a possibility.

    Give answer (1) if only Conclusion I is true Give answer (2) if only Conclusion II is true Give answer (3) if either Conclusion I or Conclusion II is true Give answer (4) if neither Conclusion I nor Conclusion II is true Give answer (5) if both the Conclusion I and Conclusion II are true 15. Statement : L & G # K @ M Conclusions : I. All Ks being Ls is a possibility II. Some Ks are Ls 16. Statement : Z % T # P $ Y Conclusions : I. Some Zs are Ts II. Some Zs being Ps is a possibility 17. Statement : Q $ B % L & M Conclusions : I. Some Qs are Ls II. Some Ms are Qs 18. Statement : P ! K & R % Y Conclusions : I. Some Ys are Ks II. All Ks are being Ps is a possibility 19. Statement : J % A @ N & Q Conclusions : I. Some As are being Qs is a possibility II. Some Ns being Js is a possibility Directions (20-24) : Study the following information carefully and read all the conclusions and then decide which of the given conclusions logically follows from the given statements.

    SYMBOLS & NOTATIONS A&B A%B A#B A@B A*B

    All As are Bs Some As are Bs Some As are not Bs No As are Bs All As being Bs is not possible but some As being Bs is a possibility. A$B Some As being Bs is not possible but No As being Bs is a possibility. A!B Some As being Bs is a possibility. Give answer (1) if only Conclusion I is true Give answer (2) if only Conclusion II is true Give answer (3) if either Conclusion I or Conclusion II is true Give answer (4) if neither Conclusion I nor Conclusion II is true Give answer (5) if both the Conclusion I and Conclusion II are true 20. Statement : G@M%K&F Conclusions : I. Some Ms are Fs. II. Some Fs are not Gs. 21. Statement : T*G&L@F Conclusions : I. All Gs are Ts. II. Some Fs are not Gs. 22. Statement : Z!B&K#G Conclusions : I. Some Gs being Z is a possibility. II. Some Bs not being Gs is a possibility. 23. Statement : If K $ E % Y & G Conclusions : I. Some Ks are not Gs. II. Some Bs are Gs. 24. Statement : F@L%U&J Conclusions : I. Some Fs being Us is a possibility. II. Some Js not being Ls is a possiblity. Directions (25–29) : Study the following information carefully and read all the Conclusions and then decide which of the given Conclusions logically follows from the given statements. A # B All As are Bs A % B Some As are Bs A & B Some As are not Bs A * B No A is B A @ B Some As being Bs is a possibility.

    Give answer (1) if only Conclusion I is true Give answer (2) if only Conclusion II is true Give answer (3) if either Conclusion I or Conclusion II is true Give answer (4) if neither Conclusion I nor Conclusion II is true Give answer (5) if both the Conclusion I and Conclusion II are true 25. Statement : K&F#G@L Conclusions : I. All Fs being Ls is a possibility. II. Some Ls are not Ks. 26. Statement : T&Z%M#G Conclusions : I. Some Ts are not Gs. II. Some Gs are Zs. 27. Statement : Q*D%B@K Conclusions : I. Some Ds not being Ks is a possibility. II. Some Bs are not Qs. 28. Statement : G#Z&K*P Conclusions : I. Some Ks are Gs. II. Some Ks are not Gs. 29. Statement : G&L&Z#M Conclusions : I. Some Ms are Ls. II. No G is M. Directions (30–34) : In each of the following questions, relationship between different elements is shown in the statements. The statements are followed by two Conclusions numbered I and II. Study the Conclusions based on the given statement(s) and select the appropriate answer : Give answer (1) if Only Conclusion I is true Give answer (2) if Only Conclusion II is true Give answer (3) if Either Conclusion I or Conclusion II is true Give answer (4) if Neither Conclusion I nor Conclusion II is true Give answer (5) if Both Conclusion I and Conclusion II are true 30. Statements : S < B < G < E < W; A > G Conclusions : I. G > B II. A = W

    BPRE–221

    31. Statements : P < B < D = K < L; M > D Conclusions : I. L > D II. P = M (32–33) : Statements : W < N > J = D > M; D < X 32. Conclusions : I. N > X II. X > M 33. Conclusions : I. W > M II. D < N 34. Statements : B>N>J V < W = M < S; X = A > B Conclusions I:H>S II : W > A 49. Statements : Q = P > S > B = D > Y; A > B = C Conclusions I:Q>B II : Y < C 50. Statements : X > U < D = P < G = K; U > Q = M Conclusions II : M < P I:D H > M; W > K = B Conclusions I:B>D II : T < M 52. Statements : A < B < C = D > E > F; X = Y > C Conclusion II : D > A I:X>E Directions (53–57) : In the following questions, relationships between different elements are given in three different statements. The statements are followed by two Conclusions numbered I and II. Choose the correct answer accordingly : Give answer (1) if only Conclusion I is true Give answer (2) if only Conclusion II is true Give answer (3) if both the Conclusion I and Conclusion II are true Give answer (4) if either Conclusion I or Conclusion II is true Give answer (5) if neither Conclusion I nor Conclusion II is true 53. Statements : 1. P < D < U 2. U = G > B 3. Y < G < L Conclusions : I. L > D II. Y > P 54. Statements : 1. F < H < J 2. J < D = C 3. K = C < G Conclusions : I. F < K II. H < G

    SYMBOLS & NOTATIONS 55. 1. 2. 3.

    Statements : J G C K 2. Q < L < M 3. N > K > J Conclusions : I. P > J II. J = P 57. Statements : 1. G = D > L 2. D > Q = M 3. Q > R > V Conclusions : I. L < M II. G > V Directions (58 – 62) : Study the following information and Conclusions carefully and decide which of the given Conclusions logically follows from the given statements : A&B A%B A#B A@B A*B A!B

    All As are Bs. Some As are Bs. Some As are not Bs. No A is B. Some As being Bs is a possibility. All As being Bs is a possibility.

    Give answer (1) if only Conclusion I follows Give answer (2) if only Conclusion II follows Give answer (3) if either Conclusion I or Conclusion II follows Give answer (4) if neither Conclusion I nor Conclusion II follows Give answer (5) if both the Conclusion I and Conclusion II follow 58. Statement : H#K&M@G Conclusions : I. All Ms are Hs. II. All Hs are Ms. 59. Statement : Z%Y&L&J Conclusions : I. Some Zs are Js. II. All Ys being Js is a possibility. 60. Statement : E*M#G@N Conclusions : I. All Gs are Es. II. Some Gs are not Es.

    61. Statement : D&E%F#G Conclusions : I. Some Ds being Gs is a possibility. II. No Gs being Fs is a possibility. 62. Statement : J@T%K&X Conclusions : I. Some Js are not Xs. II. Some Ts are Xs. Directions (63 – 67) : Study all the Conclusions and then decide which of the given Conclusions logically follows from the given statements. A#B A%B A&B A*B A@B A!B

    All As are Bs Some As are Bs Some As are not Bs No A is B Some As being Bs is a possibility. All As being Bs is a possibility.

    63. Statement : K@Z*Y!P (1) Some Ps are Ks (2) Some Zs are Ys (3) Some Ks are not Zs (4) No Z is P. (5) None of these 64. Statement : M%G#L&B (1) Some Bs being Ms is a possibility. (2) All Ls are Bs (3) All Gs are Bs (4) All Bs are Gs (5) No M is B. 65. Statement : Q!T*J%L (1) Some Qs are Js (2) All Ls are Js (3) Some Ts are Ls (4) Some Qs are Ls (5) Some Ls are not Ts 66. Statement : C%F!M#T (1) Some Cs are Ts (2) Some Fs being Ms is a possibility (3) All Ts are Ms (4) Some Cs are Ms (5) Some Ms are Ts 67. Statement : H%L&B#N

    BPRE–223

    (1) Some Ns are Bs. (2) All Ls are Ns (3) Some Hs are Ls (4) Either 1 or 2 (5) Either 2 or 3 Directions (68 – 72) : Study the following information and all the Conclusions carefully and then decide which of the given Conclusions logically follows from the given statements. A&B A%B A#B A@B A*B A$B

    All As are Bs Some As are Bs Some As are not Bs No A is B Some As being Bs is a possibility. All As being Bs is a possibility.

    68. Which is correct, if L & G # K @ M is true? (1) All Ks are Ls (2) All Ls are Ks (3) Some Gs are Ls (4) Some Ms are Ls (5) Some Gs are not Ms 69. Which is correct, if Q $ B % L & M is true? (1) Some Qs are Ms (2) All Bs being Ls is a possibility (3) Some Bs being Ms is a possibility (4) All Bs are Qs (5) (2) and (3) 70. Which is correct, if Z * T # P $ Y is true? (1) Some Zs are not Ys (2) All Ts are Ps (3) Some Ts are Ys (4) Some Ps are not Zs (5) Some Ys are Ps 71. Which is/are correct, if P $ K & R % Y is true? (A) Some Ps are Ys (B) All Ks are Ps (C) Some Ps being Ys is a possibility (D) Some Rs are Ps (E) All Ps being Ks is a possibility (1) A and E are true (2) C, D, and E are true (3) A, C and E are true (4) B, C and D are true (5) B, D and E are true 72. Which is correct, if J % A @ N & Q is true?

    SYMBOLS & NOTATIONS (1) (2) (3) (4)

    No Js are Qs No Qs are As Some Ns are not Qs Some Ns being Js is a possibility (5) Some As are not Qs Directions (73 – 77) : Study all the Conclusions and then decide which of the given conclusions logically follows from the given statements. A&B A%B A#B A@B A*B A$B

    All As are Bs Some As are Bs Some As are not Bs No A is B Some As being Bs is a possibility. All As being Bs is a possibility.

    73. K * T @ M & J (1) Some Ts are not Ks is a possibility (2) All Ts are Ks (3) Some Ms are not Js (4) Either (1) or (2) (5) None of the above 74. N % B & Q $ T (1) Some Ts being Bs is a possibility. (2) Some Qs being Ns is a possibility. (3) No T is N. (4) 2 and 3 (5) 1 and 2 75. H * Z # C @ L (1) All Cs being Hs is a possibility. (2) Some Hs are not Ls (3) Some Zs are not Ls (4) 1, 2 and 3 (5) None of the above 76. P & Q # R % S (1) Some Ps are Ss (2) Some Rs are Qs (3) All Qs being Rs is a possibility. (4) 1 and 3 (5) None of the above 77. X $ Y & Z % A (1) Some Zs being Xs is a possibility. (2) Some Ys being As is a possibility. (3) Some Zs not being Ys is a possibility. (4) 1 and 2 (5) None of the above

    5. (2) Option (1) A $ B : A is the father of B. B # C : B is the brother of C. C # D : C is the brother of D. The sex of D is not known. D is either son or daughter of A. Option (2) C Ψ A : C is the mother of A. A $ D : A is the father of D. D ε B : D is the sister of B. Thus, D is the daughter of A. 6. (2) Option (1) R Ψ S : R is the mother of S. R is a female. Option (2) U Ψ R : U is the mother of R. R # S : R is the brother of S. S # T : S is the brother of T. So, R is the brother of T. 7. (3) Option (1) D # C : D is the brother of C. C $ B : C is the father of B. B # A : B is the brother of A. A ε E : A is the sister of E. So, A is the daughter of C. Option (2) A # B : A is the brother of B. B $ D ; B is the father of D. D ε E : D is the sister of E. E $ C : E is the father of C. B is the father of D and E. B is the grandfather of C. So, A is the grandfather of C. Option (3) C # D : C is the brother of D. D $ B : D is the father of B. B # A : B is the brother of A. A $ E : A is the father of E. So, A is the nephew of C.

    SHORT ANSWERS 1. (2) 5. (2) 9. (1) 13. (5) 17. (4) 21. (5) 25. (1) 29. (4) 33. (4) 37. (1) 41. (3) 45. (1) 49. (1) 53. (1) 57. (2) 61. (5) 65. (5) 69. (5) 73. (4) 77. (4)

    2. (1) 6. (2) 10. (2) 14. (2) 18. (2) 22. (5) 26. (2) 30. (4) 34. (2) 38. (5) 42. (2) 46. (5) 50. (3) 54. (3) 58. (4) 62. (2) 66. (2) 70. (5) 74. (5)

    3. (3) 7. (3) 11. (5) 15. (1) 19. (4) 23. (1) 27. (2) 31. (1) 35. (2) 39. (5) 43. (4) 47. (4) 51. (2) 55. (5) 59. (5) 63. (3) 67. (1) 71. (2) 75. (1)

    4. (3) 8. (3) 12. (4) 16. (5) 20. (5) 24. (2) 28. (3) 32. (2) 36. (1) 40. (1) 44. (5) 48. (2) 52. (1) 56. (4) 60. (3) 64. (1) 68. (3) 72. (4) 76. (5)

    EXPLANATIONS 1. (2) (cd + ef) × be = (23 + 45) × 12 = 68 × 12 = 816 2. (1) dcxf – (bf – d) × d = 32 × 5 – (15 – 3) × 3 = 160 – 12 × 3 = 160 – 36 = 124 = bee 3. (3) baf ÷ bt × d =105 –15 × 3 = 7 × 3 = 21 = cb 4. (3) bee+ fg –

    FG ca × h IJ H be K

    = 200 –10 –190 = bja.

    8. (3) According to question, the new sequence would be

    M 4 P A 3 R 5 E J 2 D F 1 U H B 9 T W I 8 K N 6 V 7 Z Q 17th from the left end 9. (1) Eighth to the right of the twentieth from the right end means twelfth from the right the right end, i.e., T. 10. (2)

    Number Constant

    Vowel or Number or Symbol

    There is only one such combination : 11. (5)

    4PA

    12. (4) +2

    +2

    –2

    +4

    →#   → J R  → B   → W 9  +

    –2 4 → M   →3 P  +2

    –4

    → V   → K 6  +

    2 –4  → D  → E 2 

    BPRE–224

    Consonant

    Number Symbol

    Such combinations are :

    R5#

    ;

    J2$

    13. (5) Option (1) B>A>S S : True E < B : Not True Option (2) B>A=S>E S : Not True E < B : Not True Option (3) B>A>S>E S : Not True E < B : True Option (4) B>A=S>E S : Not True E < B : True 14. (2) Option (1) S>P>Q=G>R>V Clearly, S > V Option (2) PP VT=O>P=J>V Clearly, S > V

    Gs

    K

    VI No P being Y is a

    M P

    VI

    Y

    After combining venn diagrams II and IV, we get:

    VII After combining venn diagrams II and V, we get:

    Gs, Ls Ks

    Zs Ts VII

    Ps

    Venn diagram VII supports the Conclusion I. Some Zs are 16. (5) Z % T Ts. This is the Conclusion I.

    Zs Zs

    Ts

    or

    I

    Ts

    VIII Venn diagram VIII supports the Conclusion II. 17. (4) Q $ B No Q being B is a possibility

    II

    or

    Gs

    Ls

    Q

    B

    Ts

    I

    Zs

    B%L L&M

    or

    Ls

    Ps

    P $ Y possibility

    All Ls are Gs.

    Some Bs are Ls. All Ls are Ms.

    Some Bs are Ls. Gs

    I

    II Some Gs are not Ks. Gs

    Gs

    Ts

    V No K is M.

    K@M

    15. (1) L & G

    G#K

    Ks

    Ks

    III T#P Ps.

    Some Ts are not

    Ts

    Ts

    Ps or

    Ks

    Ps

    Ps

    or

    III

    Ps IV

    or

    All Ls are Ms. I + A ⇒ I-type of Conclusion ‘‘Some Bs are Ms’’. 18. (2) P ! K Some Ps being Ks is a possibility.

    IV

    or

    BPRE–225

    Ks

    V

    Ks

    or I

    II

    SYMBOLS & NOTATIONS or

    or 21. (5) T * G

    Ks

    T G (or)

    Ps

    III K&R All Ks are Rs. R%Y Some Rs are Ys. Venn diagram II supports the Conclusion II. Some Js are 19. (4) J % A As. Js

    Js

    B

    K Z G

    B

    Z K G

    As

    G&L

    G L

    L@F

    L

    F

    Main diagram : 23. (1) K $ E

    T

    G L F

    or,

    As

    I

    II

    or

    K E

    E%Y

    E Y

    Y&G

    Y

    G

    Main diagram :

    G L F

    or

    K K Y G

    or,

    24. (2) F @ L

    G T L F

    As Js

    22. (5) Z ! B & K # G

    Z B

    Z!B

    or

    A@N

    No A is N.

    A

    B&K

    B K

    K#G

    K G or

    N K IV

    N&Q

    K

    G

    Main diagram :

    All Ns are Qs.

    Qs

    Z

    Ns

    Main diagram :

    B

    F L U J 25. (1)K & F Some Ks are not Fs. F#G All Fs are Gs. G@L Some Gs being Ls is a possibility. Venn diagrams of ‘‘Some Ks are not Fs:

    Ks

    B K G

    Ks

    Fs

    or

    Qs

    I

    Z

    II or

    B

    VI

    V

    K

    or

    K 20. (5) G @ M

    M%K K&F Main diagram :

    F

    G M M K B

    K F

    Z K

    III Venn diagrams of ‘‘All Fs are Gs’’: Gs

    or

    Fs

    G K F

    Fs

    or

    or

    Ns

    G or

    L

    U J

    U&J Z

    F L U

    L%U

    III

    M

    or

    G T

    Z

    B

    K G

    BPRE–226

    IV

    Fs or Gs V

    SYMBOLS & NOTATIONS Venn diagrams of ‘‘Some Gs being Ls is a possibility’’ :

    M#G

    All Ms are Gs.

    Gs

    Gs Gs

    Ls

    or

    Gs

    Ls

    Ms VII

    VI

    VII

    Ls

    All Ms are Gs.

    Gs

    VIII After combining venn diagrams IV and VIII, we get :

    Ls Gs

    I + A ⇒ I–type of Conclusion ‘‘Some Zs are Gs.’’ Conclusion II is the Converse of it. 27. (2) Q * D No Q is D. D%B Some Ds are Bs.

    Ds Ds

    Bs

    or

    Fs

    IX

    or

    Venn diagram IX supports the Conclusion I. Some Ts are not 26. (2) T & Z Zs.

    Bs Ds

    Ts or

    III

    Zs

    B@K Some Bs being Ks is a possibility.

    II

    I or

    Bs Bs

    T

    Bs

    II

    I

    Zs

    Some Ds are Bs.

    VIII

    Some Zs are Ms.

    or

    Ts

    N o Q is D .

    or

    Ms

    Z

    Ks

    or

    IV

    Ks

    V or

    III Z%M

    Ks

    Some Zs are Ms.

    Bs

    Zs Zs

    Ms

    or

    Ms V

    IV

    Venn diagram VII contradicts the Conclusion I.

    VI After combining the venn diagrams III and VI, we get :

    E + I ⇒ O1 – type of Conclusion. “Some Bs are not Qs”. This is the Conclusion II. All Gs are Zs. 28. (3) G # Z Z&K Some Zs are not Ks. No K is P. K*P Conclusions I and II form Complementary Pair. Therfore, either Conclusion I or II follows. 29. (4) G & L Some Gs are not Ls. Some Ls are not Zs. L&Z Z#M All Zs are Ms. 30. (4) S < B < G < E < W A>G A>GD P X : Not True II. X > M : True 33. (4) Conclusions : I. W > M : Not True II. D < N : Not True 34. (2) B > N > J < Y = D < V < S Conclusions : I. J < S : Not True II. S > D : True 35. (2) M % G : Some Ms are Gs.

    Ms

    Ms Gs

    Gs or

    or

    Ms

    Ks Bs

    Zs

    Ds

    I

    II Gs

    Ms or

    VI VII

    BPRE–227

    III G # L : All Gs are Ls.

    SYMBOLS & NOTATIONS Ls Gs

    Ls or

    Gs V IV L & B : Some Ls are not Bs. Ls

    Ls Bs

    Bs

    I + E ⇒ O - type of Conclusion ‘‘Some Ks are not Ys’’. This is the Conclusion I. 37. (1) C % F : Some Cs are Fs.

    Cs

    L

    B

    Cs

    Ns

    III

    Some Ls are not Bs.

    or

    Ms Fs

    A + O ⇒ No Conclusion.

    Some Ms are Gs.

    Ms or

    IV

    or

    Ks Zs

    Ts Ms

    Bs

    VII VIII Venna diagram V supports the Conclusion I. After combining venn diagrams II and V, we get

    Fs V

    Hs Ls Bs

    M # T : All Ms are Ts.

    I + A = I - type of Conclusion ‘‘Some Ms are Ls’’. This is the Conclusion II. 36. (1) K @ Z : Some Ks being Zs is a possibility.

    Ns

    Bs

    F ! M : All Fs being Ms is a possibility.

    Zs

    B

    or B # N : All Bs are Ns.

    or

    All Gs are Ls.

    V

    VI

    All Gs are Ls.

    Ks

    or

    L II

    Fs

    VIII

    Ls Bs

    Bs IV

    Cs Fs

    Fs I

    VII

    or

    Ls

    or

    or

    VI

    L & B : Some Ls are not Bs.

    Ts or

    VI

    Ms VII

    Some Cs are Fs.

    IX Venn diagram IX supports the Conclusion II. 39. (5) Q ! T : All Qs being Ts is a possibility. Ts Qs

    II

    I

    I + A ⇒ I - type of Conclusion ‘‘Some Cs are Ms’’. (Possibility)

    Zs Ks

    or

    III Z * Y : No Z is Y. Z

    All Fs are Ms.

    Y

    IV Y ! P : All Ys being Ps is a possibility. Ps Ys

    Some Cs are Ms.

    No T is J.

    I + A ⇒ I - type of Conclusion ‘‘Some Cs are Ts’’. (Possibility) This is the Conclusion I. 38. (5) H % L : Some Hs are Ls.

    Hs Ls

    Ls or

    V

    I

    Some Ks are Zs.

    II Ls

    Hs or

    No Z is Y.

    All Qs are Ts.

    A + E ⇒ E - type of Conclusion

    All Ms are Ts.

    Hs

    I T * J : No T is J. J % L : Some Js are Ls.

    ‘‘No Q is J’’. This Conclusion supports the Conclusion II.

    No T is J

    Some Js are Ls.

    E + I ⇒ O1 - type of Conclusion ‘‘Some Ls are not Ts’’. This is the Conclusion I. 40. (1) B ! M + G % L B!M

    III

    BPRE–228

    B M

    SYMBOLS & NOTATIONS Main diagram

    M G

    M+G

    C

    D L

    G

    G%L

    K

    44. (5) D ! Z $ L % U

    Main diagram

    B G

    M

    L

    P Z

    Z#D

    Z D or Z

    P Z D

    Alternate-1

    P H Z D possibility Alternate-2

    P H Z D possibility 42. (2) Q @ U % Z + M Q@U

    Q U

    U%Z

    U

    Z+M

    Z M

    Z

    Main diagram

    Z U

    M

    U

    M

    Z Q

    possibility 43. (4) K $ D + M % C K$D

    K D

    D+M

    D M

    M%C

    Z L L

    M C

    U

    U Z

    Main diagram

    Q

    Z$L

    D

    P$Z

    H

    D Z

    Main diagram

    P

    H

    D!Z

    L%U

    41. (3) H % P $ Z # D H%P

    M

    D

    L

    45. (1) J © K J is father of K. K@B K is the daughter of B. B is the daughter-inB&M law of M. M!Z M is the brother of Z. Z is the husband of T. Z*T Conclusions J is the son of M. I. J # M It is true. M is mother of B. II. M $ B M is the father-in-law of B. T is the son of M. 46. (5) T # M M@B M is the daughter of B. B is the brother of K. B!K K is the wife of H. K%H H is the father of L. H©L Conclusions K is the mother of L I. K $ L It is true. II. M $ T M is the mother of T. It is true. C is the daugh47. (4) C & B ter-in-law of B. B is the husband of A. B*A A is the father of H. A©H H is the brother of G. H!G G is the father of K. G©K Conclusions I. H * K H is the husband of K. It is not true. H is the uncle of K. K is the mother of B. II. K $ B It is not true. 48. (2) B > H > V < W = M < S X=A>B A>B>H>V S = Not True II. W > A = Not True 49. (1) Q = P > S > B = D > Y A>B=C B=C=D>Y Conclusions I. Q > B = True II. Y < C : True 50. (3) X > U < D = P < G = K U>Q=M Q=M H > M W>K=B B=K>T=D Conclusions I. B > D : Not True II. T < M : Not True 52. (1) A < B < C = D > E > F X=Y>C X>C=D>E Conclusions I. X > E : True II. D > A : True 53. (1) P < D < U = G > B P J P=M=K=J or, P = M > K > J or, P = M = K > J or, P = M > K = J Conclusions : I. P > J : Not True II. J = P : Not True P is either greater than or equal to J. Therefore, either Conclusion I or Conclusion II is true.

    SYMBOLS & NOTATIONS 57. (2) L < D > Q = M G=D>Q>R>V Conclusions : I. L < M : Not True II. G > V : True 58. (4) H # K : Some Hs are not Ks. K & M : All Ks are Ms. M @ G : No M is G.

    E % F : Some Es are Fs.

    Es Es

    Fs

    Fs or

    IV

    III or

    All Ks are Ms.

    Some Ts are Ks.

    Es A + E ⇒ E–type of Conclusion ‘‘No K is G.’’ 59. (5) Z % Y : Some Zs are Ys. Y & L : All Ys are Ls. L & J : All Ls are Js.

    All Ks are Xs.

    V F # G : Some Fs are not Gs.

    Fs Fs

    Gs

    Gs or

    Some Zs are Ys.

    VII

    VI or

    All Ys are Ls. I + A ⇒ I–type of Conclusion ‘‘Some Zs are Ls.’’

    Fs VIII

    After combining Venn diagrams II, V and VII, we get :

    All Ls are Js. I + A ⇒ I–type of Conclusion ‘‘Some Zs are Js’’. This is the Conclusion I.

    All Ys are Ls.

    All Ls are Js.

    Ds I

    Gs

    Fs

    Ds Es

    Gs

    or IX

    Ds Es II

    Some Ks are Zs.

    I + E ⇒ O-type of Conclusion “Some Ks are not Zs. (Possibility).

    No Z is Y.

    Fs

    Ds Es

    A + A ⇒ A –type of Conclusion ‘‘All Ys are Js’’. This is the Conclusion II. 60. (3) E * M : Some Es being Ms is a possibility. M # G : Some Ms are not Gs. G @ N : No G is N. Conclusion I and Conclusion II form Complementary Pair. Therefore, either Conclusion I or Conclusion II follows. 61. (5) D & E : All Ds are Es.

    I + A ⇒ I–type of Conclusion ‘‘Some Ts are Xs.’’ This is the Conclusion II. 63. (3) K @ Z : Some Ks being Zs is a possibility. Z * Y : No Z is Y. Y ! P : All Ys being Ps is a possibility.

    No Z is Y.

    Gs

    Some Zs are Ls.

    or

    Some Ts are Ks. E + I ⇒ O1–type of Conclusion ‘‘Some Ks are not Js’’.

    Fs

    No M is G

    Es

    No J is T.

    X

    Some Ms are Gs.

    or

    Fs Ds Es

    All Ys are Ps. E + A ⇒ O1 – type of Conclusion “Some Ps are not Zs.” 64. (1) M % G : Some Ms are Gs. G # L : All Gs are Ls. L & B : Some Ls are not Bs.

    All Gs are Ls. Gs

    I + A ⇒ I-type of Conclusion “Some Ms are Ls”. Venn diagrams of “Some Ls are not Bs” :

    XI Venn diagrams IX and X supports the Conclusion I. Venn diagram VIII supports the Conclusion II. 62. (2) J @ T : No J is T. T % K : Some Ts are Ks. K & X : All Ks are Xs.

    BPRE–230

    Ls Ls

    or

    Bs

    I

    Bs

    II or

    SYMBOLS & NOTATIONS B % L : Some Bs are Ls.

    No T is J. Ls

    Bs

    Bs

    Bs

    Some Js are Ls. E + I ⇒ O1-type of Conclusion

    III Venn diagrams of “Some Ms are Ls” : Ms Ms

    or

    Ls

    Ls

    “Some Ls are not Ts.” This is the Conclusion (5). 66. (2) C % F : Some Cs are Fs. F ! M : All Fs being Ms is a possibility. M # T : All Ms are Ts.

    Ls or III

    Ls Bs V

    L & M : All Ls are Ms.

    V or

    Ls Ms

    IV

    or

    Some Cs are Fs. IV

    Ls

    All Fs are Ms.

    Ms

    I + A ⇒ I-type of Conclusion

    Ls

    Ms or Ls

    “Some Cs are Ms”. (Possibility) VI

    All Fs are Ms.

    VII

    Some Bs are Ls.

    VI After combining the venn diagrams II and V, we get :

    Ms Ls Bs

    VII Venn diagram VII supports the Conclusion (1). 65. (5) Q ! T : All Qs being Ts is a possibility. T * J : No T is J. J % L : Some Js are Ls.

    All Qs are Ts.

    All Ms are Ts. A + A ⇒ A-type of Conclusion “All Fs are Ts”. (Possibility) F ! M supports the Conclusion (2). 67. (1) H % L : Some Hs are Ls. L & B : Some Ls are not Bs. B # N : All Bs are Ns. Conclusion (1) is the Converse of B # N. 68. (3) L & G : All Ls are Gs. G # K : Some Gs are not Ks. K @ M : No K is M.

    All Ls are Gs.

    A + E ⇒ E-type of Conclusion “No Q is J”. (Possibility)

    I + A ⇒ type of Conclusion “Some Bs are Ms.” This is the Conclusion (3). Venn diagram V supports the Conclusion (2). 70. (5) Z * T : Some Zs being Ts is a possibility. T # P : Some Ts are not Ps. P $ Y : All Ps being Ys is a possiblility. Conclusion (5) is the Converse of ‘P $ Y’. 71. (2) P $ K : All being Ks is a possibility.

    Some Gs are not Ks.

    Ks

    A + O ⇒ No Conclusion

    Ps

    Conclusion (3) is the Converse of “All Ls are Gs”. 69. (5) Q $ B : All Qs being Bs is a possibility. Bs

    No T is J.

    All Ls are Ms.

    Qs

    Qs or Bs

    I

    II

    BPRE–231

    Ps or Ks

    I

    II

    K & R : All Ks are Rs.

    Rs Ks

    Ks or Rs

    III

    IV

    SYMBOLS & NOTATIONS R % Y : Some Rs are Ys.

    A @ N : No A is N.

    Ts

    Rs Rs

    Ys or

    Ks

    Ys A

    V

    N

    VI

    III T @ M : No T is M.

    IV

    or

    N & Q : All Ns are Qs.

    Ys

    T

    Rs

    Qs Ns

    IV

    or

    VII

    M & J : All Ms are Js.

    Qs

    All Ps are Ks.

    V

    VI

    Js

    A + A ⇒ A-type of Conclusion “All Ps are Rs”. Conclusion (D) is the Converse of it. Venn diagrams I and II support the Conclusion (E). After combining venn diagrams I, III and VII, we get :

    No A is N. I + E ⇒ O-type of Conclusion “Some Js are not Ns”.

    Ks

    Js VI

    Venn diagram II contradicts the Conclusion (1).

    No T is M.

    No A is N. All Ms are Js.

    E + A ⇒ O1-type of Conclusion “Some Qs are not As”. Venn diagrams of “Some Js are not Ns” :

    Rs

    or

    V

    All Ns are Qs.

    Ys

    Ms

    Ms

    Some Js are As.

    All Ks are Rs.

    M

    Ns

    Ps Js Js

    Ns or

    E + A ⇒ O1 – type of Conclusion “Some Js are not Ts”. Venn diagram V contradicts the Conclusion (3). Conclusion (1) and Conclusion (2) form Complementary Pair. Therefore, either Conclusion (1) or Conclusion (2) follows. 74. (5) N % B : Some Ns are Bs.

    Ns Ns

    VI VIII

    VII or

    Venn diagram VIII supports the Conclusion (C). 72. (4) J % A : Some Js are As.

    Bs or

    Ns I

    As or I

    As II

    or

    N

    Bs Ns

    VIII Venn diagrams VI and VII support the Conclusion (4). 73. (4) K * T : Some Ks being Ts is a possibility.

    Ks

    As Js

    Ts or

    Ks

    III B & Q : All Bs are Qs.

    Qs Bs

    Ts IV

    III

    II or

    J

    Js Js

    Bs

    I

    II or

    BPRE–232

    Qs or Bs V

    Q $ T : All Qs being Ts is a possibility.

    SYMBOLS & NOTATIONS C @ L : No C is L.

    Ts

    77. (4) X $ Y : All Xs being Ys is a possibility.

    Ts

    Qs

    or

    C Qs VII

    VI

    L

    Ys Xs

    VII

    After combining venn diagrams IV and VI, we get :

    After Combining venn diagrams II and V, we get :

    Hs Zs

    Ts Qs

    Ys II

    Y & Z : All Ys are Zs.

    Zs Ys

    Venn diagram VIII supports the Conclusion (1).

    Venn diagram VIII supports the Cnoclusion (1). 76. (5) P & Q : All Ps are Qs.

    Qs

    Some Ns are Bs.

    Ys IV

    Ps I + A ⇒ I-type of Conclusion “Some Ns are Qs.” Conclusion (2) is the Converse of it. 75. (1) H * Z : Some Hs being Zs is a possibility.

    Zs Qs II

    I

    Q # R : Some Qs are not Rs.

    Rs

    Qs

    As or V

    As VI

    or

    As Zs

    III or

    VII

    Qs

    Zs

    I

    Z % A : Some Zs are As.

    Zs

    or

    Hs Zs or

    III

    Ps

    All Bs are Qs.

    Rs

    All Xs are Ys.

    IV or

    All Ys are Zs.

    II or

    A + A ⇒ A–type of Conclusion

    Zs Qs

    Hs

    Z # C : Some Zs are not Cs.

    Zs

    Rs Rs

    Ss or

    Ss

    Cs

    Cs or

    VI IV

    V

    VII

    After combining the venn diagrams IV and VI, we get :

    Zs, Ys As

    or

    Ss Rs

    Venn diagram VIII supports the Conclusion (2).

    VIII

    Venn diagram IV contradicts the Conclusion (3). ❑❑❑

    Cs VI

    Conclusion (1) is the Converse of it.

    VIII

    or

    Zs

    “All Xs are Zs”.

    Rs

    V R % S : Some Rs are Ss.

    III

    Zs

    Zs or

    VIII

    VIII

    Hs

    or

    I

    Cs

    Bs

    Xs

    No Conclusion follows.

    BPRE–233

    DISTANCE & DIRECTION

    8

    DISTANCE & DIRECTION

    QUESTIONS FROM 1999 TO 2010 ARE AVAILABLE ONLINE NATIONALISED BANKS & IBPS PO/MT/SO EXAMS Directions (1–6) : Study the following information to answer the given questions :

    (1) East (2) South-East (3) North-East (4) South (5) None of these

    (Corporation Bank PO Exam. 16.01.2011)

    Point P is 9m towards the East of Point Q. Point R is 5 m towards the South of point P. Point S is 3m towards the West of point R. Point T is 5m towards the North of point S. Point V is 7m towards the South of point S. 1. If a person walks in a straight line for 8m towards west from point R, which of the following points would he cross the first ? (1) V (2) Q (3) T (4) S (5) Cannot be determined 2. Which of the following points are in a straight line ? (1) P, R, V (2) S, T, Q (3) P, T, V (4) V, T, R (5) S, V, T 3. An office bus driver starts from the office, drives 2 km towards North, takes a left turn and drives for 5 km. He then takes a left turn and drives for 8 km before taking a left turn again and driving for 5 km. The driver finally takes a left turn and drives 1 km before stopping. How far and towards which direction should the driver drive to reach the office again? (1) 3 km towards North (2) 7 km towards East (3) 6 km towards South (4) 6 km towards West (5) 5 km towards North (IBPS Bank PO/MT CWE 18.09.2011)

    4. Point B is 3 metres North of point A. Point C is 3 metres West of point B. Point D is 5 metres South of point C. Point E is 7 metres East of point D. In which of the following directions is point E, with reference to point B?

    (IDBI Bank Officer Exam. 16.12.2012)

    5.

    Point R is 10 metres north of point A. Point K is exactly in the middle of the points R and A. Point N is 7 metres east of point A. Point M is 7 metres east of point K. Point S is 6 metres north of point M. What is the distance between points S and N ? (1) 13 metres (2) 16 metres (3) 11 metres (4) 12 metres (5) None of these

    (IBPS Bank PO/MT CWE-III, 26.10.2013)

    6. W walked 30 metres towards South, took a left turn and walked 50 metres, again he took a left turn and walked 30 metres. How far is he from the starting point? (1) 80 metres (2) 100 metres (3) 130 metres (4) 50 metres (5) None of these (Corporation Bank SO (Marketing) Exam, 22.02.2014)

    Directions (7-8) : Study the following information carefully and answer the questions given below : (IBPS Bank PO/MT CWE-IV, 18.10.2014)

    Point J is 6 metres to the west of Point P. Point P is 5 metres to the north of Point L. Point W is 4 metres to the west of point L. Point S is 3 metres to the south of Point W. Point S is 7 metres to the west of Point B. Point X is 8 metres to the north of point B. 7. Which of the three points lie in a straight line ? (1) J, S, W (2) B, S, W (3) B, L, X (4) J, P, X (5) L, S, W

    BPRE–234

    8. Point J is in which direction with respect to point L ? (1) North-West (2) North (3) South-East (4) North-East (5) East Directions (9–10) : Study the following information carefully and answer the questions given below : Amaya starts from point P dacing North. He walks for 5m and reaches point Q. He takes a right turn and walks for 14m and reaches point R. He then takes a left turn, walks for 2.5 m and reaches point S. He takes another left turn, walks for 7m and reaches point T. He takes a left turn again, walks for 7.5m and reaches points U. He takes a left turn, walks for 3.5 to reach point V. (SIDBI Officer Online Exam.24.02.2016)

    9. How far and in which direction is Amaya from point P? (1) 5.5 m West (2) 7 m East (3) 3.5 m North–East (4) 10.5 m East (5) 10.5 m South–East 10. Had Amaya takes a right instead of taking a left from point T, while reaching everything else the same, in what direction would point V be from point T? (1) South (2) North (3) North–West (4) North–East (5) South–East Directions (11–13) : Study the following information carefully and answer the questions given below : (Bank of Baroda Exam, 25.09.2016)

    Roshni starts walking towards north from Point Q. She walks for 15m and reaches Point R. She takes a left turn and walks for 8m to reach Point M. From Point M she takes a right turn and walks for 4m. She then takes a left turn and reaches Point K after walking for 5m. Azhar is standing 8m west of Point M. He walks 6m towards north.

    DISTANCE & DIRECTION He takes a right turn and walks for 3m to reach Point D. Point S is 11m south of Point D. 11. In which direction is Point K with respect to Roshni’s starting position? (1) North-west (2) South (3) North (4) West (4) South-east 12. If Point C is 13m west of Point Q, how far and in which direction is Point S with respect to Point C? (1) 15m to the East (2) 10m to the West (3) 10m to the North (4) 15m to the North (5) 6m to the South 13. How far is Point R from Azhar’s starting position? (1) 10m (2) 18m (3) 12m (4) 16m (5) 13m Directions (14–16) : Study the following information carefully and answer the questions given below : Point M is 15m to the east of Point L. Point C is 3m to the north of Point M. Point Q is 6m to the east of Point C. Point P is 3m to the south of Point Q. Point V is to the north of Point L. A person walks 9m from Point V towards south, reaches Point R, takes a left turn and reaches Point C. 14. In which direction is Point V with respect to Point P ? (1) South-west (2) West (3) South-east (4) North-west (5) North-east (IBPS Bank PO/MT CWE (Pre Exam), 16.10.2016 (First Sitting))

    15. If a person walks 15m towards east from Point V, takes a right turn and walks 4m, how far will he be from Point M ? (1) 5m (2) 9m (3) 12m (4) 6m (5) 8 m 16. What is the difference of the distance between Points V, L and Points R, Q ? (1) 14m (2) 9m (3) 8m (4) 3m (5) 7m Directions (17–19) : Study the following information carefully and answer the questions given below : (IBPS Bank PO/MT CWE-VI (Pre Exam), 16.10.2016 (Second Sitting))

    Point B is 25 m south of Point A. Point C is 10 m east of Point B. Point D is 30 m north of Point C. Point E is 7 m east of Point D. Point X is 18 m south of Point E. Point M is 12 m south of Point X. Point C is 7 m west of Point M. 17. Point B is in which direction from Point D? (1) South (2) South-West (3) North-East (4) South-East (5) North 18. If Point W is 3 m to the north of Point A, then what is the distance between Points B and W? (1) 28 m (2) 15 m (3) 21 m (4) 24 m (5) 17 m 19. What is distance between Points B and M? (1) 19 m (2) 15 m (3) 21 m (4) 17 m (5) 13 m Directions (20–22) : Study the following information carefully and answer the questions given below : (IBPS Bank PO/MT (Pre.) Exam, 23.10.2016)

    Point E is 16m to the south of Point C. Point F is 4m to the west of Point E. Point H is 5m to the south of Point F. Point J is 12m to the east of Point H. Point Y is to the east of Point F. A person walks 15m from Point Y towards west, reaches Point Z, takes a left turn and reaches Point J. 20. What is the difference of the distance between the points Z, J and points Y, F? (1) 26 m (2) 32 m (3) 12 m (4) 8 m (5) 22 m 21. If a person walks 20 m towards north from Point Z, takes a left turn and walks 8 m, how far will he be from Point C? (1) 2 m (2) 6 m (3) 3 m (4) 4 m (5) 5 m 22. In which direction is Point Z with respect to Point H? (1) North–west (2) North–east (3) South–west (4) East (5) South–east Directions (23–25) : Study the following information carefully and answer the questions given below : (Bank of Maharashtra PO Exam, 26.10.2016)

    Point Q is 23m to the west of Point P. Point S is 14m to the north of Point Q. Point T is 2m to the west of Point S.

    BPRE–235

    Point R is 5m to the south of Point T. Point Z is 17m to the east of Point R. A person walks 16m from Point Z towards north, reaches Point C, takes a left turn and walks a certain distance to reach Point Y. Point Y is to the north of Point Q. 23. What is the difference of the distance between Points Y, S and Points Y, C ? (1) 12 m (2) 6 m (3) 4 m (4) 11 m (5) 5 m 24. In which direction is Point T with respect to Point P ? (1) South-west (2) South-east (3) North-east (4) North-west (5) West 25. If a person walks towards south from Point Y, which of the following points would he reach first ? (1) Z (2) Q (3) T (4) R (5) S Directions (26–28) : Study the following information carefully and answer the questions given below : (IBPS RRBs Officer CWE (Pre.) Exam, 14.11.2016 (Shift-I))

    S is 11m east of N. N is 8m North of P. P is 4m west of O. Point O is the midpoint of Points P and R, such that P, O and R form a straight line. Q is 13m south of R. 26. If L is 8m east of R and M is 7m south of L, then what is the distance between M and Q? (1) 6m (2) 10m (3) 7m (4) 5m (5) 3m 27. In which direction is N with respect to Q? (1) North-West (2) West (3) East (4) North East (5) South West 28. Hiten walks 2m towards north from Point P, takes a right turn and walks for 8m. How far will he be from Point Q? (1) 13m (2) 7m (3) 17m (4) 8m (5) 15m Directions (29–31) : Study the following information carefully and answer the questions given below : (IBPS RRBs Officer CWE (Pre.) Exam, 14.11.2016 (Shift-II))

    R is 15m west of Q. J is 6m north of Q. W is 2m west of J. L is 10m south of W. K is 6m west of L.

    DISTANCE & DIRECTION 29. If F is 4m to the south of R and V is 2m east of K, how far is Point F from Point V? (1) 8m (2) 11m (3) 5m (4) 9m (5) 4m 30. Kabir walks 10m towards south from Point J, takes a right turn and walks for 3m. How far will he be from Point K? (1) 4 m (2) 10 m (3) 5 m (4) 6 m (5) 9 m 31. In which direction is R with respect to J? (1) West (2) South east (3) North east (4) North (5) South west Directions (32–35) : Study the following information carefully and answer the questions given below : (IBPS RRBs Officer CWE (Pre.) Exam, 14.11.2016 (Shift-III))

    Point A is 14m east of Point B. Point C is 6m south of Point A. Point P is 4m west of Point C. Point C is the midpoint of Points P and H, such that points P, C and H form a straight line. Point O is 6m south of Point H. 32. Kunal walks 10m towards north from Point H, takes a left turn, and walks for 4m. How far will he be from Point C? (1) 2 m (2) 10 m (3) 4 m (4) 6 m (5) 7 m 33. If Point A is 2m to the north of Point L and Point R is 4m west of Point O, how far is Point L from Point R? (1) 10 m (2) 12 m (3) 7 m (4) 14 m (5) 9 m 34. In which direction is Point B with respect to Point H? (1) North–east (2) South–east (3) North–west (4) North (5) West Directions (35–38) : Study the following information carefully and answer the questions given below : (IBPS RRBs Officer CWE (Main Exam) 05.11.2017)

    ‘A’ walks 10 km north from point Q to reach point H. He takes a left turn and walks 9 km to reach point S. On the other side, ‘B’ walks 5 km north from point Y to reach point J. Point Y is 8 km to the east of point Q. Next ‘B’ turns to his right and walks 4 km to

    reach point D. Also ‘A’ turned left from point S and reached point M after walking 5 km. M is in west direction from J. 35. What is the shortest distance between points M and J? (1) 10 km (2) 17 km (3) 18 km (4) 12 km (5) Cannot be determined 36. If ‘B’ walks 4 km east from point Y, then he is in which direction with respect to point D? (1) South (2) North–west (3) North–east (4) North (5) South–west 37. What is the shortest distance between points Q and D? (1) 16 km

    (2)

    192 km

    (4) 197 km (5) 13 km 38. What is the direction of J with respect to Q? (1) South (2) North–west (3) North–east (4) North (5) South–west Directions (39–41) : Study the information carefully and answer the questions given below : (3) 14 km

    (IBPS SO (IT Officer) CWE (Prelim Exam) 30.12.2017)

    Point A is 12m to the north of Point B. Point C is 8m to the east of Point B. Point D is 4m south of Point C. Madhav who is standing at Point A, walks 10m towards west, takes a left turn and walks for 14m to reach Point S. He takes a left turn again, walks for 5m and stops at Point Q. Aditya who is standing at Point D, walks 6m towards west, takes a right turn, walks for 2m and stops at Point Y. Point Z is 7m away from Point S. 39. In which direction is Point S with respect to Point D? (1) North-West (2) West (3) South-West (4) North-East (5) East 40. If Point L is to the north of Point B such that Point Z and Point L form a horizontal straight line, then which of the following will be true? (1) Distance between Points A and L is 7m (2) Point L is to the north west of Point D. (3) Point Y is to the south east of Point Z

    BPRE–236

    (4) Point Z is to the north of Point S (5) All the given statements are true 41. What is the distance between Point Q and Point Y? (1) 5m (2) 13m (3) 7m (4) Cannot be determined (5) 9m 42. Rajat walks for 2m toward north from Point A and reaches Point B, takes a left turn walks for 3m to reach Point C and turns left again. He walks for 6m to reach Point D and turns left, walks for 7m to reach Point E. In which direction is Point E with respect to Point A ? (1) South-East (2) North-West (3) East (4) West (5) North-East (IBPS SO (Law Officer) CWE (Prelim Exam) 31.12.2017)

    Directions (43–44) : Study the following information carefully and answer the questions given below : Point A is 9 metre to the west of point B. Point G is 9 metre to the east of point B. Point C is 9 metre to the south of point G. Point E is 13 metre to the west of point C. Point W is 2 metre to the south of point E. Point F is 2 metre to the east of point W. (IBPS RRBs Officer CWE (Prelim Exam) 11.08.2018)

    43. What is the direction of point F with respect to point B? (1) East (2) South (3) South-East (4) South-West (5) West 44. If point X is 9 metre to the north of point E, then how far and in which direction is point X with respect to point A? (1) 2 metre–North (2) 5 metre–East (3) 2 metre–West (4) 2 metre–East (5) 4 metre–West Directions (45–47) : Study the following information carefully and answer the questions given below : Point B is 14m east of point A. Point C is 9m north of point B. Point D is 12m east of point C. Point E is 15m south of point D. Point F is 30m west of point E. Point G is 10m north of point F. Point H is 18 m east of point G. (IBPS RRBs Officer CWE (Prelim Exam) 18.08.2018)

    DISTANCE & DIRECTION 45. If point X is 6m south of point A then which point is at shortest distance from point X? (1) E (2) A (3) F (4) B (5) G 46. What is the distance of Point C from Point H? (1) 9 metre (2) 5 metre (3) 4 metre (4) 6 metre (5) 7 metre 47. Point B is in which direction with respect to Point F ? (1) South (2) South-east (3) North (4) North-east (5) North-west Directions (48–50) : Study the following information carefully and answer the questions given below : Point J is 13m to the west of Point K. Point K is 7m to the south of Point L. Point M is 4m to the east of Point L. Point N is 20m to the south of Point M. Sudha starts walking from Point B towards south. She walks for a distance of 9m, takes a right turn and walks for a distance of 8m to reach point N. (IBPS Bank PO/MT CWE (Prelim Exam) 14.10.2018)

    48. In which direction is Point L with respect to Point B? (1) West (2) North (3) South-east (4) North-west (5) North-east 49. Point W is to the west of Point N and also to the south of Point K, what is the difference between WN and WK? (1) 9m (2) 11m (3) 7m (4) 13m (5) 4m 50. Point Z is 8m to the west of Point B, how far and in which direction is Point M with respect to Point Z? (1) 7m towards North (2) 13m towards East (3) 13m towards South (4) 11m towards South (5) 11m towards North Directions (51–53) : Study the following information carefully and answer the questions given below : Point Q is 23m to the west of Point P. Point S is 14m to the north of Point Q. Point T is 2m to the west of Point S. Point R is 5m to the south of Point T. Point Z is 17m to the east of Point R. A person walks 16m from Point Z towards south, reaches Point C, takes a right turn and walks a cer-

    tain distance to reach Point Y. Point Q is to the north of Point Y. (Canara Bank PO Exam, 23.12.2018)

    51. What is the difference of the distance between Points Y, S and Points Y, C ? (1) 12 m (2) 5 m (3) 14 m (4) 11 m (5) 6 m 52. In which direction is Point T with respect to Point P ? (1) South-west (2) South-east (3) North-east (4) North-west (5) West 53. If a person walks towards north from Point Y, which of the following points would he reach first ? (1) Z (2) Q (3) T (4) S (5) R Directions (54–56) : Study the following information carefully and answer the questions given below : Bhanu starts walking from point V towards east. He walks for a distance of 52m and reaches point B and turns right, walks for a distance of 20m to reach point C. From point C he turns right and walks for 16m to reach point W, turns right, walks for 28m to reach Point E. From point E, he turns left, walks for a distance of 24m to reach Point F and stops. (IBPS RRBs Officer CWE Prelim Exam, 04.08.2019)

    54. If Point R is 8m north of point V, what is distance between point F and point R ? (1) 18 metre (2) 12 metre (3) 16 metre (4) 22 metre (5) 36 metre 55. Four of the following five points are alike in a certain way based on their directions as per the given arrangement and thus they form a group. Which one of the following does not belong to that group? (1) BF (2) BE (3) WV (4) VE (5) WF 56. In which direction is Point F with respect to Point C ? (1) North-west (2) North (3) North-east (4) South-east (5) West 57. Vijay walks 10 km towards South. From there he takes a UTurn and walks 6 km. Then, he takes a left turn and walks 3 km. What is the distance and direction of his current location with respect to his starting point?

    BPRE–237

    (1) (2) (3) (4) (5)

    4 km, South-west 5 km, South-west 5 km, South-east 5 km, North-west 6 km, North-east

    (IBPS RRBs Officer CWE Prelim Exam, 17.08.2019)

    58. A ran 15 metre towards South, took a turn towards east and ran 20 metre and again he took a turn towards the north and ran 15 metre. What is the distance between his current position and his starting position? (1) 10 metre (2) 15 metre (3) 30 metre (4) 20 metre (5) 25 metre (IBPS RRBs Officer CWE Prelim Exam, 17.08.2019)

    Directions (59 – 61) : Study the following information carefully and answer the questions given below : Point B is 8m to the south of point A. Point H is 16m to the east of point B. Point C is 12m to the south of point H. Point D is 8m to the west of point C. Point E is 8m to the north of point D. Point F is 12m to the east of point E. Point G is to the north of point F and also to the east of point H. (IBPS Bank PO/MT CWE Prelim Exam, 19.10.2019)

    59. In which direction is point A with respect to point E? (1) North (2) East (3) North-west (4) South-east (5) South-west 60. If point J is to the east of point A and also to the north of point E, which of the given points lie in a straight vertical line? (1) JH (2) BJ (3) CJ (4) DJ (5) JF 61. How far is point H from point G? (1) 16 metre (2) 4 metre (3) 8 metre (4) 2 metre (5) 12 metre 62. Rohan walked 5 km towards south from his house and then 8 km towards west. From there he walked 3 km towards north followed by 4 km towards east. Then he walked 6 km towards south and 2 km towards west to reach the market. What is the shortest between his house and the market ? (1) 12 km (2) 10 km (3) 13 km (4) 17 km (5) 25 km (IBPS Bank PO/MT CWE Main Exam, 30.11.2019)

    DISTANCE & DIRECTION Directions (63–65): Study the following information carefully and answer the questions given below : (IBPS RRBs Officer Scale-I CWE Main Exam, 13.10.2019)

    Point Q is 15 metre towards east of Point P. Point R is 10 metre to the south of Point Q. Point S is 5 metre to the east of Point R. Point T is 5 metre to the north of Point S. Point U is 5 metre to the east of Point T. Point V is 10 metre to the south of Point U. Point W is 5 metre to the west of Point V. 63. What is the shortest distance between points P and W? (1) 17 metre (2) 25 metre (3) 21 metre (4) 13 metre (5) 9 metre 64. What is the distance between points T and W? (1) 10 metre (2) 15 metre (3) 5 metre (4) 7 metre (5) 9 metre 65. In which direction is point U with respect to point R ? (1) North-west (2) South-east (3) West (4) North-east (5) East 66. Point X is 5km to the north of Point Y. Point Z is 3 km to the east of Point Y. Point U is 2 km to the north of Point Z. Point T is 5 km to the west of Point U. Savita starts walking from Point U. She walks 3 km north and reaches Point A. She takes a left turn and walks 5 km to reach Point B. How far and in which direction is Point B with respect to Point T? (1) 5 km East (2) 5 km South (3) 8 km West (4) 2 km North (5) 3 km North (IBPS SO Prelim Exam, 28.12.2019)

    67. Point R is 20 metre to the north of point G. Point G is 12 metre to the east of point D. Point F is 16 metre to the north of point D. Point A is 15 metre to the west of point R. Animesh is standing at the point A. He goes to point F covering the shortest distance. Then he goes to the point G covering the shortest distance. Find the total distance covered by Animesh. (1) 20 metre (2) 25 metre (3) 18 metre (4) 17 metre (5) 15 metre (IBPS Bank PO/MT CWE Prelim Exam, 03.10.2020)

    Directions (68–72) : In each of following questions, some symbols are used with different meanings as illustrated below : (IBPS RBSs Officer CWE Prelim Exam, 07.08.2021)

    X @Y – X is in the north direction of Y or X travels towards north X & Y – X is in the east direction of Y or X travels towards east X $ Y – X is in the west direction of Y or X travels towards west X ! Y – X is in the northeast direction of Y or X travels towards northeast X + Y – X is in the northwest direction of Y or X travels towards northwest X # Y – X is in the south direction of Y or X tracels towards south X * Y – X is in the southeast direction of Y or X travels towards southeast X ^ Y – X is in the south west direction of Y or X travels towards south west The houses of eight friends are in the following manner : M#S@H S$R+K K@L#A H^K&M A&R*P$D 68. In which direction is house of S with respect to house of L? (1) South (2) North–east (3) South–west (4) North–west (5) North 69. In which direction is house of R with respect to house of M? (1) South–east (2) North–east (3) South (4) North (5) North-west 70. In which direction is house of H with respect of house of P? (1) West (2) North (3) South (4) North–west (5) South–east 71. In which direction is house of A with respect to house of R ? (1) South–east (2) North–east (3) South–west (4) North–west (5) East

    BPRE–238

    72. In which direction is house of L with respect to house of P ? (1) South-east (2) North–east (3) South–west (4) North–west (5) North 73. Kislaya travelled from the city A 5 km towards south, then travelled 8 km towards west. From there he travelled 3 km towards north and 4 km towards east. Finally he reached the city B after travelling 6 km towards south and then 2 km towards west. What is the shortest distance between the city A and city B ? (1) 5 km (2) 13 km (3) 10 km (4) 7 km (5) 17 km (IBPS Bank PO/MT CWE Prelim Exam, 04.12.2021)

    SBI PO EXAMS Directions (1–6) : Study the following information and answer the following questions : (SBI Probationary Officer Exam 28.04.2013)

    A, B, C, D, E, G and I are seven friends who study in three different standards namely 5th, 6th and 7th such that not less than two friends study in the same standard. Each friend also has a different favourite subject namely History, Civics, English, Marathi, Hindi, Maths and Economics but not necessarily in the same order. A likes Maths and studies in the 5th standard with only one other friend who likes Marathi. I studies with two other friends. Both the friends who study with I like languages (Here languages include only Hindi, Marathi and English). D studies in the 6th standard with only one person and does not like civics. E studies with only one friend. The one who likes history does not study in 5th or 6th standard. E does not like languages. C does not like English, Hindi or Civics. 1. Which combination represents E’s favourite subject and the standard in which he studies ? (1) Civics and 7th (2) Economics and 5th (3) Civics and 6th (4) History and 7th (5) Economics and 7th

    DISTANCE & DIRECTION 2. Which of the following is I’s favourite subject ? (1) History (2) Civics (3) Marathi (4) Either English or Marathi (5) Either English or Hindi 3. Who amongst the following studies in the 7th standard ? (1) G (2) C (3) E (4) D (5) Either D or B 4. Which of the following combinations is definitely correct ? (1) I and Hindi (2) G and English (3) C and Marathi (4) B and Hindi (5) E and Economics 5. Which of the following subjects does G like ? (1) Either Maths or Marathi (2) Either Hindi or English (3) Either Hindi or Civics (4) Either Hindi or Marathi (5) Either Civics or Economics 6. B is 7 metres away in the north of A. A moves 9 metres towards east from the starting point, takes a right turn and walks 2 metres, then takes a left turn and moves 1 metre and finally he takes a left turn and moves 9 metres. Now, how far and in which direction is A from the B? (1) 5 metres East (2) 10 metres West (3) 5 metres West (4) 10 metres East (5) None of these (SBI Specialist Officer (Law Officer : MMGS Scale-II) Online Exam, 19.04.2014)

    Directions (7–9) : Study the following information carefully and answer the questions given below : (SBI Management Executive Exam, 19.09.2014)

    Point U is 8 metre east of Point B. Point R is 2 metre south of Point B. Point R is 7 metre north of Point S. Point K is between points S and M. Point K is 3 metre away from Point M. Point Y is 4 metre south of Point M. 7. Point M is in which direction with respect to Point R? (1)North east (2) South east (3) South (4) East (5) North 8. Point U is in which direction with respect to Point S\ (1) North west (2)South (3) West (4) Northeast (5) East

    9. Which of the following three points lie in a straight line excluding S K M \ (1) BRS (2) UMK (3) SMY (4) BUR (5) KMY Directions (10–12) : Study the following information carefully and answer the questions given below : (SBI Management Executive Exam. 19.09.2014)

    Point U is 8 metre east of Point B. Point R is 2 metre south of Point B. Point R is 7 metre north of Point S. Point K is between points S and M. Point K is 3 metre away from Point M. Point Y is 4 metre south of Point M. 10. Point M is in which direction with respect to Point R? (1)North east (2) South east (3) South (4) East (5) North 11. Point U is in which direction with respect to Point S\ (1) North west (2)South (3) West (4) Northeast (5) East 12. Which of the following three points lie in a straight line excluding S K M \ (1) BRS (2) UMK (3) SMY (4) BUR (5) KMY 13. Point A is 30 metres to the east of point B. Point C is 10 metres to the south of Point A. Point D is 15 metres to the west of Point C. Point E is exactly in the middle of the Points D and F. Points D, E and F lie in a straight line. The length of the line DEF is 20 metres. Point F is to the north of Point D. Point G is 15 metres to the east of Point F. How far and in which direction is Point G from Point A ? (1) 10 metres, South (2) 15 metres, North (3) 10 metres, North (4) 15 metres, South (5) 10 metres, East (SBI PO Phase–I (Preliminary) Online Exam. 20.06.2015)

    14. A person starts from his house and moves towards the market. He walks 40 metres towards south and takes a right turn. After walking 30 metres he takes a left turn and walks 20 metres. Finally he takes a left turn and reach the market after walking 30 metres. How far and in which direction is his house from the market ?

    BPRE–239

    (1) (2) (3) (4) (5)

    60 metres, South 60 metres, North 70 metres, North 70 metres, South 90 metres, North (SBI PO Phase–I (Preliminary) Online Exam. 20.06.2015)

    15. A person starts walking from his office towards a party hall. He walks for 30m towards East. He takes a 90° right turn and walks for 15m. He again takes a 90° right turn, and walks for another 20m. He then walks for 25m after taking a 90° left turn. Turining 90° towards his right he walks for 10m to reach the party hall. How far and in which direction is the party hall from his office ? (1) 40m towards West (2) 40m towards South (3) 45m towards South (4) 45m towards North (5) 40m towards North (SBI PO Phase–I (Preliminary) Online Exam. 21.06.2015)

    16. Point A is 40m to the north of point B. Point C is 20m to the west of point A. Point D is 30m to the south of point C. Point E is exactly midway between points D and F in such a manner that Point D, E and F form a horizontal straight line of 40m. Point F is to the east of point D. Point G is 30m to the north of point F. How far and in which direction is point G from point A? (1) 30m towards West (2) 40m towards North (3) 20m towards West (4) 20m towards East (5) 30m towards East (SBI PO Phase–I (Preliminary) Online Exam. 21.06.2015)

    17. Point A is 30m to the South of point B. Point C is 20 m to the East of point A. Point D is 15m to the south of point C. Point D is exactly midway between points E and F in such a manner that point E, D and F form a horizontal straight line of 40m. Point E is to west of point D. How far and in which direction is point E from point B ? (1) 45m towards South (2) 25 m towards south (3) 30m towards west (4) 35m towards north (5) 45 m towards north (SBI PO Phase–I (Preliminary) Online Exam. 27.06.2015)

    DISTANCE & DIRECTION 18. A person starts walking from his home towards his friend’s place. He walks for 25m towards West. He takes a 90° right turn and walks for 20m. He again takes a 90° right turn, and walks for 10m. He then walks for another 10m after taking a 90° left turn. Turning 90° towards his right, he walks for 15m to reach his friend’s place. How far and in which direction is the friend’s place from his home ? (1) (2) (3) (4) (5)

    30m towards East 30m towards North 40m towards South 30m towards South 40m towards North (SBI PO Phase–I (Preliminary) Online Exam. 27.06.2015)

    Directions (19–21) : Study the following information carefully and answer the questions given below : (SBI PO Online (Pre.) Exam, 02.07.2016 (Ist Shift))

    Ashish starts running from point P and runs 10 km towards North. He takes a right turn and runs 15 km. He now runs 6 km. after taking a left turn. He takes a left turn, runs 15 km. Finally he takes right turn and runs 5 km. and stops at Ravi’s house. 19. How far is Ravi’s house with respect to point P? (1) 16 km. (2) 25 km. (3) 4 km. (4) 21 km. (5) None of these 20. Towards which direction was the Ashish running before he stopped at Ravi’s house? (1) North (2) East (3) South (4) West (5) North–West 21. Ravi starts walking from point D and walks 20 metres towards north. He takes a left turn and walks 15 metres. Then he takes a right turn and stops at point B after walking 5 metres. Mohit is standing at point Z which is 30 metres to the east of point B. He walks 25 metres towards south. He takes a left turn and stops at point X after walking 10 metres. How far and in which direction is point X from point D? (1) 25 metres West (2) 25 metres South (3) 25 metres East (4) 30 metres East (5) 20 metres West (SBI PO Online (Pre.) Exam, 02.07.2016 (IInd Shift))

    Directions (22–23) : Study the following information carefully and answer the questions given below : (SBI PO Phase-I (Pre.) Online Exam, 03.07.2016 (Ist Shift))

    Points J is 20 metre to the north of point G. Point G is 10 metre to the west of Point K. Point K is 15 metre to the south of Point L. Ashish is standing at Point T which is 30 metre to the east of Point L. He starts walking towards south and walks for 35 metre. He takes a right turn and stops at Point M after walking for 40 metre. 22. How far and in which direction is Point G with respect to Point M? (1) 15 metre towards South (2) 15 metre towards West (3) 20 metre towards South (4) 15 metre towards North (5) 20 metre towards North 23. If Ashish walks for 10 metres towards north from his final position to reach Point D, how much distance will he have to cover in order to reach the Point J? (1) 10 metres (2) 35 metres (3) 25 metres (4) 30 metres (5) 20 metres Directions (24–25) : Study the following information carefully and answer the questions given below : (SBI PO Phase-I (Pre.) Online Exam, 03.07.2016 (IInd Shift))

    Gauri starts walking from Point M and walks 20m towards the south. She then takes a right turn and walks for 30m. She takes a left turn and stops at Point O after walking for 15m. Point R is 10m to the east of Point O. Point J is 35m to the north of Point R. Point S is 5m to the west of Point J. 24. If Pooja is standing at Point W which is 10m to the north of Point S, in which direction will she have to walk in order to reach Point O? (1) North–east (2) West (3) South–west (4) South (5) South–east 25. How far and in which direction is Point S with respect to Point M? (1) 25m towards West (2) 20m towards East (3) 25m towards East (4) 20m towards West (5) 25m towards North Directions (26–27) : Study the following information carefully and answer the questions given below : (SBI PO Online (Preliminary) Exam, 10.07.2016)

    BPRE–240

    Vansh starts walking from Point E and walks 25m towards the north. He then takes a right turn and walks for 15m. He makes a left turn and stops at Point M after walking for 20m, Point K is 30m to the west of Point M. Point K is 45m to the north of Point J. Point J is 10m to the east of Point L. 26. How far and in which direction is Point E with respect to Point L? (1) 30m towards West (2) 25m towards East (3) 20m towards East (4) 20m towards West (5) 25m towards West 27. If Neha is standing at Point D which is 20m to the north of Point L, in which direction will she have to walk in order to reach Point M? (1) North-west (2) East (3) South-east (4) North-east (5) North 28. A person starts from Point A, walks 30 metre towards south and reaches Point B. He then takes a right turn, walks 7 metre, followed by a right turn, and walks for 6 metre. He then takes a right turn and walks 7 metre. He takes a final left turn, walks a certain distance and reaches Point R. Point R is 17 metre to the north of Point B. What is the distance between Point A and Point R? (1) 18 metre (2) 23 metre (3) 21 metre (4) 27 metre (5) 13 metre (SBI PO Online (Preliminary) Online Exam. 30.04.2017)

    29. A person starts from Point A, walks 7m towards north and reaches Point B. He then takes a right turn, followed by a right turn and walks for 10m each time. He then takes a right turn and walks 14m. He takes a final left turn, walks 3m and reaches Point C. Point Z is 10m to the north of Point C. Point Z is in which direction with respect to Point B? (1) North-West (2) South-East (3) South-West (4) North-East (5) South (SBI PO Online (Preliminary) Online Exam. 06.05.2017)

    30. A man walks 12 metre east from point A and reaches point B. From point B he takes left turn and walks 4 metre and then he takes right turn and walked 6 metre and again he takes right

    DISTANCE & DIRECTION turn and walks 7 metre and again takes right turn and reaches point M. If it is given that point B is in north from point M, then what is the distance between B and M? (1) 7 metre (2) 6 metre (3) 5 metre (4) 4 metre (5) 3 metre (SBI PO Online (Preliminary) Exam, 07.05.2017)

    Directions (31–35) : Study the following information carefully and answer the questions given below : (SBI PO Phase-II (Main) Exam 04.06.2017)

    There are six cars – A, B, C, D, E, F – parked in row facing north direction, but not necessarily in the same order. The distances between two adjacent cars are successive multiples of three (i.e., if the distance between the 1st and the 2nd car is 3 m, 1st and the 3rd cars is 6 m and between 1st and 4th cars is 9 m and so on.) Information regarding all the cars : ● The distance between the cars A and B is 33m and car A is to the immediate left of car B. ● The distance between the cars E and F is 99m. The distance between E and D is a multiple of 2. ● The car B is 75m away from car C. Car D is in one position to the left of car C, but not to the immediate left of C. ● Car F starts moving towards north and after going 18m, it turns right, then it moves 63m and then it goes another 7m to its right turn and stops at point Z. ● Car C moves 33m towards south direction and then takes a right turn and goes 75m straight. Then it turns again to its right direction and moves another 17m and halts at point X. ● An another car M is parked at 13m to the west of Point Z. Now M starts moving towards further west and covers 77m and reached point Y. 31. How many cars are there in between cars D and F ? (1) Two (2) None (3) More than three (4) One (5) Three 32. What is the distance between Point Z and Point X ? (1) 25 m (2) 18 m (3) 27 m (4) 36 m (5) They don’t align in the same straight line

    33. What is the position of Car M with respect to Car F? (1) 5 34 m towards north-west (2) 50 m towards south-east (3) 10 17 m towards north-east (4) 5 34 m towards south-east (5) None of these 34. Which car will be met first, if M moves through the shortest distance from Point Y? (1) E (2) F (3) C (4) None of these (5) D 35. What is the position of Car A with respect to Car E ? (1) 75m towards right (2) to the immediate right (3) 36m towards left (4) 69m towards left (5) None of these Directions (36–39) : Study the following information carefully and answer the questions given below : (SBI PO Phase-II (Main) Exam 04.06.2017)

    Mohan is standing at Point G. He walks 12 m towards east, takes a left turn and walks for 9 m to reach Point S. He takes a right turn and walks for 5 m to reach at a point which is 5 m to the south west of Point J. Varun, who is standing at Point J, starts walking towards north. He walks for 4m, takes a left turn and stops at Point D after walking for 14m. He turns left and stops at Point R after walking for 8m. Point Z is 9m north of Point Q and 6m to the west of Point R. 36. If J is 4m north of Y, then what is the distance between Point Y and Mohan’s final position? (1) 5m (2) 16m (3) 10m (4) 3m (5) Cannot be determined. 37. Rohit walks 4m to the north of Point Z, takes a right turn and stops at Point N after walking for 6m. Which of the following points will be nearest to Point N ? (1) Both D and R (2) Only S (3) Only D (4) Both S and J (5) Only Q 38. In which direction is Point D with respect to Point Q? (1) South-east (2) West (3) North-east (4) North (5) Cannot be determined

    BPRE–241

    39. If Point L is 9m south of Point R, what will be the distance between Point Q and Point L ? (1) 12 m (2) 4 m (3) 8 m (4) 6 m (5) 10 m Directions (40–42) : Study the following information carefully and answer the questions given below : There are five friends A, B, C, D and E standing randomly. B is to the northeast of E. D is 2km to the east of E, who is 6km to the west of A. C is to the northwest of D and in the line of EB. D is 4km the south of B. SBI PO (Prelim Exam), 08.07.2018 (Shift-I))

    40. In which direction is C with respect to A ? (1) South–west (2) South–east (3) North–east (4) North–west (5) None of these 41. In which direction is A with respect to B ? (1) South–east (2) South–west (3) North–west (4) North–east (5) None of these 42. What is the distance between D and A ? (1) 5 km (2) 4 km (3) 6 km (4) 3 km (5) None of these Directions (43–46) : Study the following information carefully and answer the questions given below : Ram is standing at Point A. He walks for 5m towards east and reaches Point B. He takes a left turn and walks for 1 m to reach Point C. He then takes a right turn and walks 7m to reach Point D. He again takes a right turn and walks 10m to reach Point E. SBI PO (Prelim Exam), 08.07.2018 (Shift-II))

    43. What is the direction of Point E with respect to Point B? (1) South-West (2) North-East (3) East (4) South (5) South-East 44. From point E, Ram walks 8m towards west to reach point F. In which direction and for how many metres he needs to walk to reach 9 m south of point A? (1) North-West-9m (2) North-4m (3) West-4m (4) North-5m (5) South-5m

    DISTANCE & DIRECTION 45. Ram walks 5m towards south from point C to reach point J and turns left, walks for 3m and reaches point K. What is the direction of point D with respect to point K? (1) West (2) North-East (3) North-West (4) North (5) East 46. If point M is 5m to the west of point C, what is the distance between point A and point M? (1) 2 m (2) 5 m (3) 4 m (4) 1 m (5) 3 m Directions (47–49) : Study the following information carefully and answer the questions given below : Point G is 17m to the north of Point H. Point G is 4m to the east of Point J. Point K is 25m to the south of Point J. Point K is 12m to the west of Point L. Point M is 8m to the north of Point L. (SBI PO Prelim Exam, 14.06.2019)

    47. How far and in which direction is point M with respect to point H? (1) 23m towards South (2) 8m towards East (3) 25m towards West (4) 12m towards East (5) 8m towards West 48. If Point Z is 8m to the west Point L, then which of the following points will form a straight line ? (1) ZKM (2) HZM (3) ZLG (4) GHZ (5) JHZ 49. In which direction is Point G with respect to Point L ? (1) North-West (2) South-East (3) South (4) North-East (5) North Directions (50–54) : Study the following information carefully and answer the questions given below : There are eight friends living around a village in a rectangular form. S’s house is located north-west of F’s house. J’s house is located south-west of M’s house. H’s house is located to the north of J’s house. N’s house is located to the east of H’s house. S’s house is located to the north J’s house. M’s house is located to the north of N’s house. Z’s house is located to the east of J’s house. V’s house is located exactly between S’s and M’s houses. F’s house is located to the south of V’s house and to the west of Z’s house. (SBI PO Main Exam, 20.07.2019)

    50. In which direction is F’s house with respect to M’s house? (1) North–west (2) North–east (3) South–west (4) South–east (5) Between the houses of J and Z 51. Whose house is located third to the right of N’s house? (1) J (2) S (3) F (4) V (5) Z 52. Whose house is located second to the left of H’s house? (1) F (2) M (3) J (4) V (5) Z 53. Whose house is located to the north east of J’s house? (1) Z (2) S (3) F (4) Either V or M (5) V and N 54. What is the direction of N’s house with respect to V’s house? (1) South–west (2) South–east (3) North–west (4) North–east (5) Exactly between M’s and Z’s houses

    RBI GRADE–B/ NABARD GRADE–A OFFICER EXAMS 1. From his house, Lokesh went 15 kms to the north. Then he turned west and covered 10 kms. Then, he turned south and covered 5 kms. Finally turning to east, he covered 10 kms. In which direction is he from his house ? (1) East (2) West (3) South (4) North (5) North east (Nabard Officer Grade ‘A’ Online Exam. 01.03.2015)

    Directions (2–5) : Study the following information carefully and answer the questions given below : (RBI Officer Grade ‘B’ Phase-I Exam 17.06.2017)

    Trishaa starts walking from point A, walks 12 metre to the south and reaches Point B. She then takes a left turn and walks 7 metre to reach Point C. She then takes a left turn, walks 5 metre and reaches Point D. From Point D she turns left, walks 11 metre and stops at point E. Point P is to the west of Point B. Rishi who is standing at

    BPRE–242

    Point P walks for a distance equal to the shortest distance between Points B and E and reaches Point Q. From Point Q he takes a left turn, walks for a certain distance and reaches Point E. 2. Mehul starts walking towards south from Point B. He walks for a certain distance to reach Point K. He then takes a left turn and walks for 10 metre, takes another left turn and stops after walking for 9 metre. If his final position is 3 metre to the east of Point D, what is the distance between Point B and Point K? (1) 4 metre (2) 6 metre (3) 5 metre (4) 2 metre (5) Cannot be determined 3. Point M is 9 metre to the north of Point Q. From Point M, which of the following walking directions would lead to Point A? (A) 12 metre towards west, turn right and walk for 2 metre, then turn right and walk for 7 metre. (B) 4 metre towards east, turn left and walk for 3 metre, then turn right and walk 5 metre. (C) 3 metre towards north, turn left, walk 19 metre, turn left and walk 5 metre. (D) 3 metre towards north, turn right and walk for 4 metre. (1) Both (A) and (D) (2) Only (D) (3) Both (B) and (C) (4) Only (C) (5) Only (B) 4. In which direction is Point C with respect to Point P? (1) North-East (2) West (3) East (4) South-East (5) North-West 5. Point Z is 9 metre to the east of Point A. Arun starts from Point Z, walks towards south, takes a right turn and walks for a certain distance to reach Point D. What is the total distance that Arun has to walk in order to reach Point D? (1) 20 metre (2) 11 metre (3) 19 metre (4) 9 metre (5) 15 metre Directions (6-8) : Study the followng information carefully and answer the questions given below : (NABARD Assistant Manager Online Exam 06.08.2017)

    Point B is 10 m to the north of Point C. Point J is 7 metre to the east of Point B. Point K is 5 metre to the west of Point C. Point S is 12 metre to the south of Point K. Point T is 5 metre to the east of Point S.

    DISTANCE & DIRECTION 6. If Suman walks 7 metre to the east of T, takes a left turn and walks for 10 metre to reach Point X, how far and in which direction should she walk to reach Point J? (1) 10 metre East (2) 12 metre North (3) 8 metre North (4) 10 metre South (5) 12 metre West 7. In what direction is Point S with respect to Point C? (1) South-East (2) North-West (3) South-West (4) West (5) North 8. How far is Point T from Point B? (1) 12 metre (2) 26 metre (3) 22 metre (4) 15 metre (5) 18 metre Directions (9–11) : Study the following information carefully and answer the questions given below : Tara starts walking from point A towards south-east direction. She walks for a distance of 20 metre to reach point B. From point B she walks towards north direction. She walks for a distance of 8 metre to reach point C and again turns left. From point C she walks for 12 metre to reach Point D. Point E forms the mid-point of the straight lines DC and AB. From point D she turns right and walks for a distance of 5 metre to reach point F. From point F she turns left and walks for 6 metre to reach point G. Point K forms the mid-point of the straight line FG. From point G she again turns left walks for 5 metre and reaches point H. From point H she turns left, walks for a certain distance to reach point I. From point I she turns left and walks for 8 metre to reach point J such that points IKJ lie in a straight vertical line. (RBI Officers in Grade ‘B’ Phase-I Exam. 16.08.2018)

    9. In which of the following pairs the distance between two the given points is minimum? (1) JI (2) HE (3) AJ (4) KI (5) EC 10. In which direction is point G with respect to point B? (1) West (2) North-East (3) North-West (4) South-East (5) South 11. What is the distance between point H and point E? (1) 9 metre (2) 6 metre (3) 15 metre (4) 12 metre (5) 18 metre

    Directions (12–16) : Study the following information carefully and answer the questions given below : Point A is 20m towards North of point B. Point B is 30m towards East of point C. Point D is 30m towards North of Point C. Point E is 40m towards East of point D. Point F is 40m towards West of Point A. Point G is 10m towards South of Point F. Point H is 10m towards South of Point C. (SEBI Assistant Manager Exam. 17.11.2018)

    12. In which direction is Point G with respect to Point E? (1) North-West (2) South-West (3) East (4) South-East (5) North-East 13. In which direction is Point A with respect to Point C ? (1) North-West (2) South-West (3) South-East (4) North-East (5) West 14. Which of the following represents a straight line? (1) G-C-B (2) E-C-H (3) H-C-D (4) E-D-F (5) F-A-E 15. If point K is 40m towards south of Point F then, what is the distance between point G and Point K? (1) 30 m (2) 40 m (3) 20 m (4) 10 m (5) 50 m 16. If the Point M is 40 metre towards East of point G, then what is the distance between point B and point M ? (1) 20 metre (2) 30 metre (3) 50 metre (4) 40 metre (5) 10 metre Directions (17–20) : Study the following information carefully and answer the questions given below : Point Q is 17 metre to the west of Point K. Point L is 9 metre to the north of Point Q. Point M is 33 metre to the west of Point L. Point J is 16 metre to the south of Point M. Point P is 49 metre to the east of Point J. Point P is to the south of Point S. Vijay is standing at Point S. He walks towards north for a 14 metre to reach Point Z and then turns left and walks for 16 metre to reach Point A.

    BPRE–243

    He then turns left and walks for a distance of 5 metre to reach Point L. Point B is 21 metre to the South of Point A. (RBI Grade B Officer Exam, 09.11.2019)

    17. In which direction is Point S with respect to Point Q? (1) North-west (2) North (3) South-west (4) East (5) South-east 18. If the distance between Point P and Point B is subtracted from the distance between Point Q and Point K, then the resultant will be________m. (1) 3 (2) 10 (3) 5 (4) 7 (5) 1 19. If Davinder is standing 7m to the north of Point J, then how far and in which direction is he from Point K? (1) 40 metre to the West (2) 50 metre to the North (3) 38 metre to the South (4) 50 metre to the West (5) 40 metre to the North 20. If Point Y is to the west of Point Z and also to the north of Point M, then what is the distance (in metres) between point Y and Point J? (1) 31 (2) 29 (3) 21 (4) 26 Directions (21–22) : Study the following information carefully and answer the questions given below : (RBI Officer Grade 'B' Phase-I Exam, 06.03.2021)

    Anita walks 4 km towards north from her house. She turns to her left and walks 3 km. She turns to her right and walks 6 km. She again turns to her right and walks 4 km. Once again she turns to her right and walks 3 km. Then she turns to her left and walks 4 km. She again turns to her left and walks 3 km. Finally she turns to her right and walks 3 km to reach her school. 21. In which direction is the school of Anita from her house ? (1) North-east (2) North-west (3) South-west (4) North (5) South-east

    DISTANCE & DIRECTION 22. If Anita’s friend Radhika walks 8 km to the east from her house to reach the school, what is the distance between the houses of Anita and Radhika ? (1) 7 km (2) 9 km (3) 10 km (4) 11 km (5) 13 km

    INSURANCE EXAMS Directions (1–5) : Read the following information carefully and answer the questions given below it : (Oriental Insurance Company Exam.08.04.2012)

    P α Q means Q is to the right of P at a distance of one metre. (ii) P β Q means Q is to the North of P at a distance of one metre (iii) P γ Q means Q is to the left of P at a distance of one metre. (iv) P η Q means Q is to the South of P at a distance of one metre. (v) In each of the following questions all persons face South. 1. If A η B γ L β K, then K is in which direction with espect to A? (1) South (2) East (3) North (4) West 2. If G α I η R α M, then M is in which direction with respect to I ? (1) North-East (2) North-West (3) South-East (4) South-West 3. If α B γ C β D, then D is in which direction with respect to A ? (1) North (2) South (3) East (4) West 4. If R β L η S α N γ F, then N is in which direction with respect to L? (1) South (2) South–West (3) North (4) North–East 5. If A γ F β G γ T β Q, then Q is in which direction with respect to A ? (1) South-East (2) South-West (3) North-East (4) NorthWest 6. A man starts from a point, walks 8 km towards north, turns right and walks 12 km, turns left and walks 7 km, turns and walks 24 km towards south, turns right and walks 12 km. In which direction is he from the starting point ? (i)

    (1) North (3) West

    (2) South (4) East

    (NICL (GIC) AO (Finance) Exam, 08.09.2013)

    7. Rohan walks a distance of 3 kms towards north, then turns to his left and walks for 2 kms. He again turns left and walks 3 kms. At this point he turns to his left and walks for 3 kms. How many kms is he from the starting point? (1) 5 kms (2) 3 kms (3) 2 kms (4) 1 km 8. One evening, Raja started to walk towards the Sun. After walking a while, he turned to his right and again to his right. After walking a while, he again turned right. In which direction is he facing ? (1) South (2) East (3) West (4) North (NICL (GIC) AO Exam, 08.09.2013)

    9. A squirrel starts climbing up a tree at the speed of 6 metres per minute but after each 6 metres it slips down 4 metres. It will be able to reach a top of 120 metres high in : (1) 20 minutes (2) 60 minutes (3) 58 minutes (4) 1

    1 hours 2

    (NICL (GIC) AO Exam, 15.12.2013)

    Directions (10-11) : Study the following information carefully and answer the questions given below : Point L is 15m to the east of point K. Point M is 7 m to the north of point L. Point N is 6m to the west of point M. Point O is 13m to the south of point N. Point K is 7 m to the south of point P. 10. If a person walks 6m towards east from point O and then takes a left turn, which of the following points would he reach first ? (1) L (2) M (3) K (4) P (5) N 11. Point P is in which direction with respect to point O ? (1) South (2) North-west (3) South-east (4) North (5) South-west (NIACL Administrative Officer (AO) Exam, 10.01.2015)

    Directions (12–13) : Study the following information carefully and answer the questions given below : (NIACL Administrative Officer (AO) Online Exam, 11.01.2015)

    BPRE–244

    Point A is 17 metres to the south of Point B. Point C is 9 metres to the West of Point B. Point A is 12 metres to the west of Point F. Point D is 7 metres to the north of Point C. Point E is 21 metres to the east of Point D. 12. If a person walks 9 metres towards east from Point D and then takes a right turn, which of the following points would he reach first ? (1) F (2) C (3) E (4) B (5) A 13. Point E is in which direction with respect to Point A ? (1) North-West (2) North-East (3) North (4) South (5) South-East Directions (14–15) : Read the following information carefully and answer the questions given below : (NIACL Administrative Officer (AO) Online Exam, 12.01.2015)

    Point H is 6m towards the east of Point G. Point R is 8m north of Point G. Point Q is exactly midway between Point R and Point G. Point K is 10m to the south of Point Q. Point L is 3m towards the east of Point Q. Point S is exactly midway between Point G and Point H. 14. If a person walks 4m towards the south from Point L, takes right turn and walks for another 3m, which of the following points would he reach ? (1) Q (2) G (3) K (4) H (5) Cannot be determined 15. If a person walks 8m towards north from Point S, which of the following points would he cross and how far will he be from Point R? (1) G, 4m (2) H, 3m (3) L, 6m (4) L, 3m (5) G, 8m 16. Starting from point P, Kishore walks 10 metres towards South. He takes a left turn and walks for 30 metres. He then takes a right turn and walks for 20 metres. Again turning to his right, he walks for 30 metres and reaches a point Q. How far and in which direction is this point Q from point P ? (1) 30 metres towards North (2) 20 metres towards South (3) 20 metres towards North (4) 30 metres towards South (5) Other than those given as options. (OICL Specialist Officer (Finance) Exam. 03.05.2015)

    DISTANCE & DIRECTION (1) South-east (3) East

    Directions (17–18) : Study the following information carefully and answer the questions given below : (LIC Assistant Administrative Officer (AAO) Online Exam. 05.03.2016)

    Meenal starts from Point A, walks 15m to the east and reaches Point B. She then takes a left turn and walks 4m to reach Point C. She finally takes a right turn and walks 5m and stops at Point D. Reena who is standing at Point Z which is 7m to the south of Point B walks 9m towards west takes right turn and walks for 11m and stops at point Y. 17. How far and in which direction is Point Y with respect to Point D? (1) 14m towards west (2) 9m towards east (3) 13m towards east (4) 9m towards west (5) 14m towards east 18. If Meenal walks 11m towards south from Point D, in which direction will she have to walk in order to reach Point Z ? (1) North

    (2) South-east

    (3) West

    (4) North-west

    (5) East Directions (19–20) : Study the following information carefully and answer the questions given below : (LIC Assistant Administrative Officer (AAO) Online Exam. 06.03.2016)

    Kunal starts from Point A, walks 14m to the north and reaches Point B. He then takes a right turn and walks 5m to reach point C. He finally takes a left turn and walks 4m and stops at Point D. Shubhra who is standing at point Z which is 6m to the west of point B walks 7m towards south, takes a left turn and walks for 11m and stops at point Y. 19. How far and in which direction is Point D with respect to Point Y? (1) 9 m towards south (2) 11 m towards north (3) 11 m towards south (4) 10 m towards south (5) 9 m towards north 20. If Shubhra walks 4m towards North from point Z, in which direction will she have to walk in order to reach point D?

    (2) North-west (4) West

    (5) North Directions (21–22) : Study the following information carefully and answer the questions given below : (UIICL A.O. Exam 12.06.2016)

    Hitesh starts walking towards East from Point G. He walks for 15m and then takes a left turn. He then walks for 9m and again takes a left turn. He reaches Point F after walking for 7m. He takes a right turn, walks for 6m and reaches Point K. He finally stops at Point S after walking for 8m to the left of Point K. 21. If, Ankur walks 5m North from Point G, takes a right turn and stops after walking for 18m. Then, in which direction is Point K, with respect to Ankur’s final position ? (1) South

    (2) South-east

    (3) North

    (4) North-west

    (5) East 22.

    How far is Point S from Point G ? (1) 12 m (2) 9 m

    (3) 7 m (4) 18 m (5) 15 m Directions (23-24) : Study the following information carefully and answer the questions given below : (NICL A.O. Exam 05.06.2017)

    Lalit goes to his office from his house with his son by a bike. He drives the bike from point A. He drives 6 km towards south and reaches point B. Then he turns to his right and drives 8 km and reaches a bakery shop. 23. How far is bakery shop from point A and in which direction ? (1) 10 km, south-west (2) 11 km, north-east (3) 12 km, south-west (4) 13 km, south-east (5) Other than those given as options 24. From bakery shop, Lalit turns to his right and drives 4 km and reaches Point C. He then takes a right turn and drives 5 km and reaches school and drops his son. What is the distance between bakery shop and school ? (1) 3.10 km (2) 5.13 km (3) 6.4 km (4) 4.56 km (5) 7.5 km

    BPRE–245

    25. From the school he drives 5 km towards left then turns right and drives 2km and reaches point D. Point D is in which direction of Point A ? (1) North-east (2) South-west (3) South-east (4) North-west (5) North Directions (26–28) : Study the following information carefully and answer the questions given below : Point A is 20m east of Point B. Point D is 15m west of Point C. Point E is 45m south of Point D. Point C is 30m north of Point B. Point F is 15m north of Point A. G is the midpoint of Point B and Point C. (LIC AAO Prelim Exam. 04.05.2019)

    26. If H is 15m south of Point D, then what is the distance between point E and point H? (1) 20m (2) 25m (3) 30m (4) 45m (5) 60m 27. Four of the following five belong to a group as per the given arrangement. Which is the one that does not belong to the group? (1) F, C (2) A, G (3) A, C (4) E, B (5) B, D 28. In which direction and at what distance is point G, with respect to point F? (1) 20m east (2) 30m west (3) 30m east (4) 20m west (5) None of these Directions (29–31) : Study the following information carefully and answer the questions given below : Sohan starts moving from Point T towards east. He walks for a distance of 60 metre and reaches Point Z. From Point Z he takes a right turn and walks for a distance of 54 metre to reach Point R. From Point R he takes a left turn, walks for a distance 50 metre and stops at Point Q. Vipin is standing at Point P. He walks towards west for a distance of 30 metre and reaches Point J, takes a left turn, walks for 54 metre and reaches Point Q. (LIC AAO Prelim Exam. 05.05.2019)

    29. In which direction is Point R with respect to Point P? (1) South (2) North-West (3) East (4) South-West (5) North-East

    DISTANCE & DIRECTION

    (LIC AAO Main Exam, 28.06.2019)

    NATIONALISED BANKS & IBPS PO/MT/SO EXAMS 2. (5)

    3. (5)

    4. (2)

    5. (3)

    6. (4)

    7. (4)

    8. (1)

    9. (4)

    10. (3)

    11. (1)

    12. (3)

    13. (4)

    14. (4)

    15. (5)

    16. (2)

    17. (2)

    18. (1)

    19. (4)

    20. (5)

    21. (4)

    22. (2)

    23. (3)

    24. (4)

    25. (5)

    26. (2)

    27. (1)

    28. (5)

    29. (4)

    30. (3)

    31. (5)

    32. (2)

    33. (1)

    34. (3)

    35. (2)

    36. (1)

    37. (5)

    38. (3)

    39. (1)

    EXPLANATIONS

    1. (3)

    2. (1)

    3. (1)

    4. (3)

    5. (2)

    6. (4)

    7. (2)

    8. (4)

    9. (1)

    10. (2)

    11. (4)

    12. (1)

    13. (3)

    14. (2)

    15. (2)

    16. (4)

    17. (1)

    18. (2)

    19. (4)

    20. (1)

    21. (3)

    22. (5)

    23. (4)

    24. (3)

    25. (1)

    26. (2)

    27. (4)

    28. (5)

    29. (3)

    30. (5)

    31. (2)

    32. (3)

    33. (1)

    34. (5)

    35. (4)

    36. (4)

    37. (1)

    38. (3)

    39. (5)

    40. (4)

    41. (1)

    42. (2)

    43. (5)

    44. (3)

    45. (2)

    46. (4)

    47. (2)

    48. (4)

    49. (1)

    50. (3)

    51. (2)

    52. (4)

    53. (5)

    54. (2)

    (1 – 2) : 9m T

    Q

    P 5m

    S

    3m

    R

    7m

    V North

    2. (1)

    3. (2)

    4. (3)

    5. (4)

    6. (2)

    7. (1)

    8. (3)

    9. (3)

    10. (3)

    11. (4)

    12. (2)

    13. (4)

    14. (3)

    15. (1)

    16. (5)

    17. (4)

    18. (5)

    19. (4)

    20. (3)

    21. (3)

    22. (5)

    INSURANCE EXAMS

    West

    East

    South 1. (4) He would cross Point S. 2. (5) Points S, V and T are in a straight line. 3. (5) 5km

    2km Office N

    6km

    5km 1km

    1. (2)

    2. (4)

    5. (4)

    6. (2)

    3. (1) 7. (4)

    8. (1)

    9. (3)

    10. (1)

    11. (2)

    12. (4)

    13. (2)

    14. (2)

    15. (4)

    16. (4)

    40. (5)

    17. (1)

    18. (3)

    19. (2)

    20. (3)

    21. (4)

    22. (5)

    23. (1)

    24. (3)

    42. (1)

    43. (4)

    44. (2)

    45. (3)

    46. (2)

    47. (4)

    48. (4)

    25. (4)

    26. (3)

    27. (4)

    28. (4)

    49. (1)

    50. (5)

    51. (5)

    52. (4)

    29. (3)

    30. (4)

    31. (2)

    32. (3)

    53. (2)

    54. (2)

    55. (4)

    56. (1)

    57. (2)

    58. (4)

    59. (3)

    60. (4)

    61. (2)

    62. (2)

    63. (2)

    64. (1)

    Kiran’s

    65. (4)

    66. (5)

    67. (2)

    68. (4)

    69. (2)

    70. (3)

    71. (5)

    72. (1)

    ONE LINER APPROACH GENERAL KNOWLEDGE

    Must Read

    Buy Today

    BPRE–246

    W

    E S

    4. (2)

    41. (3)

    73. (3)

    NATIONALISED BANKS & IBPS PO/MT/SO EXAMS

    RBI GRADE–B/ NABARD GRADE–A OFFICER EXAMS 1. (4)

    SHORT ANSWERS

    1. (4)

    SBI PO EXAMS

    8km

    30. What is the total distance between Point P and Point T? (1) 80 metre (2) 140 metre (3) 90 metre (4) 110 metre (5) 114 metre 31. Four of the following five are alike in a certain way based on the directions and thus they form a group. Which one of the following does not belong to that group? (1) TJ (2) ZP (3) RQ (4) ZJ (5) QT 32. Point L is 2m towards north of point C. Point N is 7m towards east of point L. Point O is 8m towards south of point N. Point T is 12m towards west of point O. Point Y is 10m to the east of point C. In which direction is point C with respect to point T ? (1) North (2) South-east (3) North-east (4) North-west (5) South-west

    4. (2)

    C 5m

    D

    3m

    B 3m A E

    7m North

    NW West SW

    NE East

    SE South Point E is in South-East direction with reference to Point B.

    DISTANCE & DIRECTION 5. (3)

    (9–10) : S

    7m

    S 2.5 m

    T

    R 6

    m

    m

    14 m

    Q

    R 7.5 m

    M

    K

    5m

    5

    m

    5 7

    A

    m

    15 m

    V

    m

    W

    3.5 m P

    N

    7m

    U

    4m V

    9m

    Z

    North NE

    NW

    North

    West West

    R 3m

    East

    5m 6m C

    15 m

    3m M

    L

    East NW

    NE

    South

    9. (4) Amaya is (7 + 3.5) metres = 10.5 metres to the East from point P. 10. (3)

    N

    V

    3.5 m

    7.5 m

    S

    50 m

    (7–8) :

    T

    X

    5m

    P

    D

    3m A

    7m

    E

    10m 30m

    NW

    NE East

    3m

    2m

    {

    NE

    B

    South 3m

    North

    SE

    SW

    B

    3m

    X 12m

    L

    25m

    North

    7m NW

    S

    Point V is in North–West direction from point T. (11–13) :

    West

    S

    South

    W

    8m

    4m

    W

    SE

    SW

    14. (4) Point V is in North-West direction with respect to the Point P. 15. (5) Required distance = (5 + 3) metres = 8 metres 16. (2) VL = 12 metres RQ = 21 metres Required difference = (21 – 12) metres = 9 metres (17–18) :

    U

    E

    30 m

    30 m

    W

    East

    West

    5m

    South

    Required distance = (6 + 5) metres = 11 metres 6. (4)

    J

    P

    North

    SE

    SW

    6m

    Q

    18m

    5

    11. (1) Point K is in North–West direction of point Q – the starting point of Roshni. 12. (3) Point S is 10 metre to the north of point C. 13. (4) Point R is 16 metre from Azhar’s starting point. (14–16) :

    10m

    D K

    C

    7m

    M

    North 5m

    NW

    NE

    4m

    West SW

    East

    8m

    West

    R

    5m

    15m 10m

    C

    East

    SW

    S

    SE

    South 7. (4) Points J, P and X lie in a straight line. 8. (1) Point L is in North-West direction of point L.

    M

    Azhar

    13m

    Q Roshni's Starting Point

    BPRE–247

    SE South

    17. (2) Point B is in South-west direction from point D. 18. (1) BW = (25 + 3) metres = 28 metres 19. (4) BM = (10 + 7) metres = 17 metres

    DISTANCE & DIRECTION 26. (2) Required distance MQ

    (20–22): 8m

    M

    X

    2

    = North

    4m

    NW

    C

    West

    a JQf + a JMf

    2

    2

    =

    a 6 f + a 8f

    2

    36 + 64 =

    =

    100 = 10 metres NE 27. (1) N is in North–West direction with respect to Q. 28. (5) DQ = (2 + 13) metres = 15 metres East (29–31) : North

    20 m

    SW

    NE

    NW

    SE South

    West

    16 m

    East 2m

    W

    SE

    SW

    J

    South F

    4m

    8m

    E

    15 m

    Z

    5m

    5m

    H

    R

    20. (5) YF = (4 + 8 + 15) metres = 27 metres ZJ = 5 metres Required difference = (27 – 5) metres = 22 metres 21. (4) MC = 4 metres 22. (2) Point Z is in North–east direction with respect to Point H. (23–25): 15 metre

    F

    16 metre

    West

    5 metre

    M 4 m 1m

    L

    4m

    M

    K

    4m 14m

    A 2m

    East

    10m

    L

    North NW

    6m

    NE

    SE

    SW South

    P

    R

    V

    Kabir

    NE

    NW

    2m

    29. (4) FV = (7 + 2) metres = 9 metres 30. (3) KM = (2 + 3) metres = 5 metres 31. (5) R is in South–West direction with respect to J. (32–34) :

    B

    2 metre S T

    K

    North

    C

    11 metre

    Y

    Q

    7m

    4m

    J

    12 m

    6m

    Y

    4m

    C

    4m

    West

    East

    H

    Z

    6m

    SE

    SW South

    9 metre

    R Q

    P

    23 metre

    23.

    (3) YS = 11 metres; YC = 15 metres Required difference = (15 – 11) metres = 4 metres 24. (4) Point T is in North-West direction with respect to Point P. 25. (5) The nearest point to the south of Point Y is Point S. (26–28) : N

    11 m

    North

    S

    NW

    3m

    4m

    O

    32. (2) Final position of Kunal ⇒ Point M MC = (4 + 6) metres = 10 metres 33. (1) LR = (4 + 6) metres = 10 metres 34. (3) Clearly, Point B is in North–West direction with respect to Point H. (35–37) : 9 km S H

    NE

    8m

    5 km

    8m C Hiten 2 m P O 4m 4m

    West

    D R

    8m

    East

    J

    L

    SW

    SE South

    North

    7m 13 m

    NW 8m

    J 6m Q

    4 km

    D

    M

    M

    West

    East SE

    SW

    BPRE–248

    5 km

    NE

    South

    Q

    8 km

    Y

    K

    DISTANCE & DIRECTION 35. (2) MJ = (9 + 8) km = 17 km. 36. (1) B reaches at point A. Point A is in south direction of point D. 2

    2

    =

    2

    G

    =

    3m

    B 2m

    6m

    North NW

    A

    NE

    West

    East

    42. (1)

    15m

    4m A

    B

    4m

    10m

    14m 6m

    X

    F

    E

    30m

    45. (3) F X = 4 metres A X = 6 metres 46. (2) CH = 5 metres 47. (4) Point B is in North-East direction of Point F. (48–50) : 4m

    L

    South

    E

    7m

    9m

    H

    SE

    SW

    D

    18m

    169 = 13 km.

    38. (3) Point J is in North–east direction of point Q. 39. (1) Clearly, Point S is in North-West direction with respect to Point D. 40. (5) All the given statements are true. 41. (3) Distance between Points Q and Y = (5 + 2) metre = 7 metre C

    D

    5m

    2

    b ADg + bQA g b4g + b12g = 25 + 144

    37. (5) QD =

    12m

    C

    M

    7m

    It is clear that Point E is in South-East direction with respect to Point A. (43–44) :

    North NE

    NW West

    13m

    J

    K

    4m Z

    East

    B

    13m A

    5m

    X

    9m

    B

    4m

    SE

    SW

    G

    9m

    South

    W

    9m

    E

    C

    13 m 2m 2m

    W

    F

    N

    8m

    48. (4) Point L is in North-West direction of Point B. 49. (1) WK = 13 metre WN = 4 metre Required difference = (13–4) metre = 9 metre 50. (5) Point M is 11 metre North to Point Z. (51–53) : T

    2m

    S

    5m

    North NW

    4m

    R

    NE

    15m

    2m

    Z

    North

    West

    9m

    East

    Q

    SE

    SW

    43. (4) Point F is in South–west direction of point B. 44. (2) Point X is 5 metre to the east of point A. (45–47) : North NE

    East

    West

    SE

    SW South

    8m 15m

    7m

    South

    NW

    NW

    Y

    7m

    15m

    NE

    West

    East

    P

    SW

    SE South

    C

    51. (5) YS = (14 + 7) metre = 21 metre YC = 15 metre Required difference = (21 – 15) metre = 6 metre 52. (4) Point T is in North-West direction with respect to Point P. 53. (2) If a person walks towards North from Point Y, he would reach Point Q first.

    BPRE–249

    DISTANCE & DIRECTION (54–56) :

    12 m

    R

    62. (2)

    24 m

    F

    E

    8m

    House

    A

    8m

    V

    B

    36 m

    North

    5 km

    20 m

    C

    3 km

    W

    16 m

    SE

    SW

    C

    North NW

    NW West

    SW

    East SE

    SW

    3 km. C

    B =

    2

    b 4 g + b3 g

    2

    16 + 9 =

    =

    25 = 5 km.

    Direction ⇒ South–West

    2

    =

    b AH g + bGH g b8g + b6g

    =

    64 + 36

    AG =

    South

    b ACg + bCDg

    SE South

    6 km.

    AD =

    East

    NE

    57. (2)

    2

    2

    2

    2

    = 100 = 10 km (63–65):

    D

    A

    15 metre

    15 metre

    NW

    58. (4)

    NE

    West

    North

    2

    H

    G 2 km F

    South

    4 km.

    3 km

    Market

    54. (2) FR = 12 metre 55. (4) Except VE, in all other pairs the first is to the south-east of the second. V is to the south-west of E. 56. (1) Point F is to the North-west of Point C.

    A

    B

    4 km

    East

    West

    D

    E

    3 km

    NE

    NW

    4 km

    D

    B 20 metre AD = 20 metre (59–61) : A

    North NE

    West SW

    C

    P

    East South

    15 m

    Q

    Z

    10 m

    T

    5m

    U

    J R

    8m B

    SE

    8m 8m

    4m E

    4m

    G

    North NW

    West 4m F

    8m

    8m 8m

    SW

    S

    W

    NE

    4m

    8m D

    H

    5m

    East SE South

    C

    63. (2) PW = =

    59. (3) Point A is in the North-West direction with respect to point E. 60. (4) D J is a straight vertical line. 61. (2) H G = 4 metres

    BPRE–250

    5m

    V

    (PZ)2 + (ZW)2

    (20)2 + (15)2

    = PW = =

    10 m

    400 + 225

    625 = 25 metre 64. (1) TW = 10 metre 65. (4) Point U is in North-East direction of Point R.

    DISTANCE & DIRECTION (68–72) :

    66. (5) 2 km B

    3 km

    X

    North

    P

    A

    NW

    D

    NE

    West 3 km T

    S

    M

    2 km

    NW

    3 km North NE

    West

    Z H

    East

    East

    A

    SE South K 68. (4) The house of S is to the North-west of house of L. 69. (2) The house of R is to the North–east of house of M. L 70. (3) The house of H is to the south of house of P. The house of A is to the east of house of R. The house of L is to the South-east of house of P. SW

    U

    Y

    R

    71. (5) 72. (1) 73. (3)

    A SW

    SE South Point B is 3 km to the north of Point T. 67. (2) 15m

    A 4m M

    4 km

    North 5 km

    3 km

    West

    R

    4 km

    F

    3m

    3 km B AB =

    D

    G

    12m

    3 km

    South

    2 km

    20m

    16m

    East

    =

    C (AC)2 + (BC)2 2

    2

    (8) + (6)

    =

    64 + 36

    =

    100 = 10 km

    North NE

    SBI PO EXAMS (1–6) : Friends Standard

    SW AF =

    SE South

    (AM)2 +(MF)2

    =

    (4)2 + (3)2

    =

    16 + 9 = 5 metre (FD)2 + (DG)2

    =

    (16)2 + (12)2

    =

    256 + 144 =

    400

    = 20 metre Total distance = (5 + 20) metre = 25 metre

    A B C D E G I

    5th 7th 5th 6th 6th 7th 7th

    Favourite Subject Maths Hindi/English Marathi Economics Civics Hindi/English History

    1. (3) E’s favourite subject is Civics and he studies in Standard 6th. 2. (1) I’s favourite subject is History. 3. (1) B, G and I study in Standard 7th. 4. (3) C’s favourite subject is Marathi.

    BPRE–251

    6. (4)

    B

    A

    9m 9m

    2m North 1m

    East

    FG =

    5. (2) G likes either Hindi or English.

    7m

    East

    West

    West

    NW

    South Required distance = (9 + 1) metres = 10 metres Direction ⇒ East

    DISTANCE & DIRECTION (7–9) :

    16. (4)

    North NW

    8m

    B

    20 m

    C

    NE

    West

    R

    East

    30 m

    SW

    SE

    20 m

    E

    M

    3m

    4m

    It is clear from the diagram that point G is 10 metres to the north of point A. 14. (2)

    Y

    House 7. (2) Point M is in Southeast direction with respect to Point R. 8. (4) Point U is in Northeast direction with respect to Point S. 9. (1) Points B, R and S lie in a straight line. (10–12) :

    B

    30 m.

    2m R 7m K

    5m

    Market

    30 m

    B

    A

    20m

    20 m

    Y

    C SE

    15m South

    E

    20m

    D

    20m

    F

    BE = (30 + 15) metres = 45 metres Point E is to the south of Point B. 18. (2) 15m

    E

    F Friend’s place

    10m D C 10m

    C

    D

    East

    West SW

    15 m

    4m

    10. (2) Point M is in Southeast direction with respect to Point R. 11. (4) Point U is in Northeast direction with respect to Point S. 12. (1) Points B, R and S lie in a straight line. 13. (3)

    NE

    20 m.

    M

    3m

    North NW

    A S

    B

    30m

    Required distance = (40 + 20) metres = 60 metres Person’s house is to the north of market. 15. (2)

    U

    B Point G is 20 metres to the east of Point A. 17. (1)

    40 m

    30 m 8m

    F

    20 m 10 m

    South K

    30 m

    30 m

    D

    7m

    5m

    G

    U

    2m

    S

    20 m

    A

    15m

    25 m F

    10 m

    B

    E North NE

    NW

    25m

    A

    AF = (20 + 10) metres = 30 metres Point F is to the north of Point A. (19–21) : Ravi’s House

    F

    15 m

    G

    10 m B

    15 m

    E

    10 m 15 m

    A

    10 m D

    15 m

    15 km 6 km

    SE

    SW South

    10 m C

    East

    West

    5 km

    Required distance AF = (15 + 25) metres = 40 metres Party hall is to the south of his office.

    BPRE–252

    15 km 10 km

    P

    DISTANCE & DIRECTION 22. (5) Point G is 20 metres towards North from Point M. 23. (4) The distance between Points D and J = (10 + 20) metres

    North NE East

    West

    = 30 metres (24–25) :

    SE

    SW South

    West

    5 metre

    20 metre

    J

    M

    35 metre

    20 metre

    25 metre

    20 metre

    O

    10 metre

    D

    10 metre

    A

    R

    North

    North

    NW

    NW

    X

    West

    North

    West

    NE

    West

    SE

    SW South

    Point X is 25 metres to the east of Point D. (22–23) :

    East

    SW

    South 24. (3) Point O is in South–West direction with respect to the Point W. 25. (1) Distance between Point S adn Point M = (20 + 5) metres = 25 metres Direction ⇒ West

    L

    NW

    30 metre

    SE South

    B 10 metre

    NE

    7 metre

    Z

    East

    West

    A

    7 metre

    K

    10 metre

    SE

    SW

    35 metre

    South

    D

    10 metre

    C K

    North

    14 metre

    3 metre

    (26-27) : 40 metre

    30 metre

    North

    M

    45 metre

    NW

    NE

    20 metre

    West

    West

    East

    East

    15 metre

    D 20 metre

    South

    B

    AR = 13 metre 29. (3)

    T

    15 metre

    R

    7 metre

    North

    J

    M

    East

    SE

    SW

    East

    20 metre

    NE

    NE

    17 metre

    5 metre

    15 metre

    26. (2) Required distance LE = (10 + 15) metre = 25 metre Direction ⇒ East 27. (4) Neha has to walk in North– East direction to reach the point M. 28. (5)

    13 metre

    S

    South

    20 metre 15 metre

    SE

    SW

    10 metre

    Z

    15 metre

    G

    East

    10 metre

    30 metre

    NW

    NE

    W

    19. (4) Ravi’s house is (10 + 6 + 5) = 21 km. away from point P. 20. (1) Ashish was running towards north before stopping at Ravi’s house. 21. (3) B

    North NW

    6 metre

    NW

    25 metre

    L J 10 metre

    15 metre

    E

    BPRE–253

    SE

    SW South

    Point Z is in South-West direction of Point B.

    DISTANCE & DIRECTION 30. (5)

    North

    6 metre

    NW

    NE

    4 metre B

    3 metre

    12 metre

    A

    M

    East

    West

    39. (5) QL =

    SE

    SW South 63m Car M 13m 7m 27m S L Z Y E 11m Car M F 39m 36m D F A B E C T 33m 27m 30m 16m 33m X 17m

    75m 31. (2) Car D is to the immediate left of Car F. 32. (3) ZX = ZB + BX = (11 + 16) metres = 27 metres 33. (1) Finally Car M is at Y.

    (MQ)2 + (ML)2

    (8)2 + (6)2 =

    =

    64 + 36

    = 100 = 10 metres (40–42) :

    BM = 3 metres (31–35) :

    FY =

    36. (4) Fianlly Mohan is at the Point F. FY = 3 metres. 37. (1) Both D and R are 4 metres away from Point N. DN = 4 metres RN = 4 metres 38. (3) Clearly, Point D is in NorthEast direction of Point Q.

    C

    B

    4 km.

    R I G H T

    E

    2 km.

    D

    A 4 km.

    North NW

    NE

    West

    (FS)2 + (SY)2

    East

    SE

    SW South

    =

    (11)2 + (27)2 = 121 + 729 = 850 = 25 × 34 = 5 34

    40. (4) C is in North–west direction of A. 41. (1) A is in South–east direction of B. 42. (2) DA = 4 km. (43–46) :

    Direction ⇒ North-West 34. (5) YD = 11 metres 35. (4) Car A is second to the left of Car E. AE = (33 + 36) metres = 69 metres R (36–39) :

    A

    D

    1m 5m

    N

    7m

    C

    1m

    4m 6m

    5m

    M

    B 1m 4m

    4m

    10m

    4m Z

    6m

    M

    9m

    D 1m

    J

    4m

    F

    4m 5m

    S

    Q

    K

    4m

    L

    9m F

    3m

    9m

    G

    J 3m

    North

    Y

    NW

    NE

    West

    East

    SE

    SW

    12m

    South

    BPRE–254

    E

    8m

    DISTANCE & DIRECTION 43. (5) Point E is in South-East direction with respect to the Point B. 44. (3) Now, Ram has to walk 4 metres to the west from Point F to reach 9 metre south of Point A. 45. (2) Point D is in North-East direction with respect to the Point K. 46. (4) AM = 1 metre (47-49) :

    J

    RBI GRADE–B/ NABARD GRADE–A OFFICER EXAMS

    North

    G

    NW

    NE East

    West SW

    South

    50. (3) F’s house is in south–west direction of M’s house. 51. (2) The house of S is located third to the right of N’s house. 52. (4) The house of V is located second to the left of H’s house. 53. (5) The house of V, M and N are located to the north–east of J’s house. 54. (2) The house of N is in south– east direction of V’s house.

    1. (4)

    NE East

    West

    SE

    SW South 6. (2) XJ = 12 Metres

    Point J is in North direction of Point X. 7. (1) Point S is in South-West direction of Point C. 8. (3) TB = BC + CT

    10 km

    SE

    17 m

    North NW

    = (10 + 12) Metres

    5 km

    = 22 Metres (9–11) : 10 km

    25 m

    J

    8m

    H

    M

    8m

    15 km

    G 5m

    8m

    3m

    K F 3m 3m

    Z

    L

    8m

    47. (2) Point M is 8 metre east to the Point H. 48. (4) GHZ form a straight line. 49. (1) Point G is in North-West direction with respect to Point L. (50–54) : S

    V

    M

    H

    2. 3.

    4. 5.

    It is clear from the diagram that now he is to the north of his house. (1) BK = 4 metres (2) From Point M, walk 3 metre towards North, turn right and walk 4 metre to reach Point A. (3) Point C is in the east direction with respect to Point P. (4) Arun has to walk (7 + 2) metres = 9 metres. (6-8) :

    N

    B

    7m

    6m

    F

    Z K

    5m

    X

    12 m

    10 m

    SE

    SW South

    S

    5m

    E 10 m

    C

    8m

    B

    North NW

    NE

    West

    East

    SE

    C 2m

    East

    6m

    South

    NE

    West

    D

    SW

    North NW

    I

    J

    10 m J

    10 m

    5m

    H

    K 4m

    A

    T

    7m Suman

    BPRE–255

    Z

    9. (3) JI = 8 metre HE = 12 metre AJ = 3 metre KI = 5 metre EC = 6 metre 10. (3) Point G is in North–West direction of Point B. 11. (4) HE = 12 metre

    DISTANCE & DIRECTION (12–16) : 40m

    D

    10m

    E

    10m F

    A

    30m

    10m

    20m

    M

    G

    12. (2) Point G is in South-West direction of Point E. 13. (4) Point A is in North-East direction of Point C. 14. (3) H-C-D represents a straight line. 15. (1) GK = 30 metre 16. (5) MB = 10 metre (17–20) : A 16 metre Z

    Y

    5 m.

    C 30m 10m

    30m

    B

    33 metre

    M

    9 m.

    9 metre D

    16 m. 7 m.

    7 metre J

    North

    East SW

    B

    33 metre

    SE South

    Vijay S 1 m.K

    SW

    East SE South

    16 metre P

    17. (4) Point S is to the east of point Q. 18. (5) QK – PB = (17 – 16) metre = 1 metre 19. (4) DK = (33 + 16 + 1) metre = 50 metre Direction ⇒ West 20. (3) JY = (7 + 9 + 5) metre = 21 metre

    NE

    West

    Q

    Davinder

    K

    West

    NE

    NW 14 metre

    H

    NW

    North

    L

    (21–22) : > > >S>T P>R >Q> >S>T P>Q>R>S>T 21. (3) R scored less marks than P and Q. 22. (5) T scored the lowest marks.

    5. (4) Q

    7. (4)

    INSURANCE EXAMS K

    B

    L

    NW

    I

    West

    NE

    West SW

    2. (4)

    North East South

    2 km

    6. (2)

    B

    Starting Point

    R

    9 km

    Starting Point

    17 km 9 km

    North

    L

    West

    East South

    F

    N

    8. (1) In the evening the Sun is seen in the West.

    12 km

    A C

    4. (2)

    1 km

    8 km

    D

    3. (1)

    7 km

    12 km 8 km

    M

    South

    SE

    G

    East

    A

    F

    3 km

    A

    North

    G

    T

    1. (2)

    R S

    It is clear from the diagram that the man is in the South direction from the starting point.

    BPRE–256

    It is clear from the diagram that he is facing towards South. 9. (3) In one minutes the squirrel climbs up two metres. Therefore, the squirrel will climb up 120 – 6 = 114 metres in 57 minutes and in 58th minute it will be on the top of the tree.

    DISTANCE & DIRECTION (14–15) :

    (10–11) :

    North

    R

    N

    P

    6m

    NW

    M 4 m.

    7m

    East

    West

    7m

    3 m.

    Q

    K

    NE

    L

    SE

    SW South

    L

    15m

    17. (1) Point Y is (9 + 5) m = 14 metre towards West from the Point D. 18. (3) It is clear from the diagram that Meenal would have to walk towards West to reach Point Z. (19-20) :

    4 m.

    6m

    3 m.

    G

    O

    3 m.

    S

    H

    6 m.

    D

    North NW

    K

    NE East

    West SW

    14. (2) He would reach at the point G. 15. (4) He would cross point L and now he is 3 m from point R. 16. (4)

    SE

    Z

    5m

    B

    C

    10 m

    10. (1) If a person walks 6 metres towards east from point O and takes a left turn, he would reach point L first. 11. (2) Point P is in North–West direction of point O. (12–13) : 21 m

    D

    4m 6m

    7m

    P

    South

    4m

    30 m

    14 m

    11 m

    A

    20 m Q

    Y

    30 m

    E

    North

    7m C

    9m

    B

    West

    24 m

    East

    17 m

    South 12 m

    A

    = 30 metres

    F

    Direction (17–18) :

    North NW

    19. (2) Point D is 11 metre towards north of Point Y. 20. (3) She would walk towards East. (21–22) :

    PQ = (20 + 10) metres

    ⇒ South

    NE

    C 5m

    Y West

    East

    15 m

    A SW

    SE

    S

    8 metre

    K 6m

    4m

    F

    11 m

    7m

    18 m

    South

    12. (4) It is clear from the diagram he would reach Point B first. 13. (2) Point E is in North-East direction with respect to Point A.

    B

    D

    7m

    9m Ankur 5 m

    9m

    Z

    BPRE–257

    G

    15 m

    DISTANCE & DIRECTION 29.(3)

    (26–28) : North

    NW

    NE East

    West SW

    NW

    NE

    West

    21.

    (4) It is clear from the diagram that Point K is in North-West direction from the Ankur’s final position. 22. (5) Required distance = GS = (9 + 6) metres = 15 metres (23–25) :

    N 2m 3m

    C

    Y

    T

    SE

    SW

    15m

    North NW

    C

    SW

    Point C is in North-east direction with respect to Point T.

    F

    North

    15m

    45m

    B

    SE South

    G

    H

    6 km

    20m

    NW

    NE

    A

    West

    B

    8 km

    NE East

    15m

    4 km

    E

    O

    7m

    West

    A

    5 km F School

    5m

    South

    D

    5 km

    6m

    East

    2 km D G

    Bakery Shop

    2m

    SE

    South

    C

    7m

    L

    North

    East

    SE

    SW South

    North

    NE

    NW West

    E

    East

    SW

    23. (1) AE =

    SE

    South 2

    b ABg + bBEg

    2

    2

    2

    =

    b6g + b8g

    =

    36 + 64 =

    100 = 10 km

    Bakery shop is in South-West direction of Point A. 24. (3) EF =

    2

    bCF g + bCEg

    26. (3) EH = (45 – 15) metre = 30 metre 27. (4) Except E, B, in all other pairs, the first point is in South-East direction of the second point. E is in South-West direction of B. 28. (4) Point G is 20 metre to the west of point F. (29–31) :

    T

    60 metre

    Z

    = =

    b5 g + b 4 g

    2

    =

    32. (3) L

    7m

    2m

    N 2m 3m

    C

    Y

    6m

    Vipin P

    T

    5m

    7m

    O

    North 54 metre

    NW

    54 metre

    NE

    2 R

    2

    30 metre

    J

    30. (4) Point R is in South–West with respect to Point P. 31. (2) PT = PJ + JZ + ZT = (30 + 50 + 60) metre = 140 metre

    25 + 16

    41 = 6.4 km

    25. (4) Point D is in North-West direction of Point A.

    50 metre

    West

    Q

    Must Read

    Buy Today

    Kiran’s

    ONE LINER APPROACH GENERAL KNOWLEDGE BPRE–258

    East

    SW

    SE South

    Point C is in North-east direction with respect to Point T. ❐❐❐

    DISTANCE & DIRECTION

    MODEL EXERCISES 1. At my house I am facing East, then I turn left and go 10 m, then I turn right and go 5 m and then I go 5 m towards the South and from there 5 m towards the West. In which direction am 1 from my house? (1) East (2) West (3) North (4) South (5) None of these 2. A and B start walking in opposite directions. A covers 3 km and B covers 4 km. Then, A turns right and walks 4 km while B turns left and walks 3 km. How far is each from the starting point? (1) 5 km (2) 4 km (3) 10 km (4) 8 km (5) None of these 3. Ram walks 10 m South from his house, turns left and walks 25 m, again turns left and walks 40 m, then turns right and walks 5 m to reach to school. In which direction the school is from his house? (1) South-West (2) North-East (3) East (4) North (5) None of these Directions (4-6) : Read the situation given below to answer these questions a, b, c, d, e, f, g, h and i are nine houses.c is 2 km East of b. a is 1 km North of b and h is 2 km South of a. g is 1 km West of h while d is 3 km East of g and f is 2 km North of g. i is situated just in middle of b and c while e is just in middle of h and d. 4. Distance between e and g is (1) 2 km (2) 1 km (3) 5 km (4) 1.5 km (5) None of these 5. Distance between a and f is (1) 1.41 km (2) 3 km (3) 2 km (4) 1 km (5) None of these 6. Distance between e and i is (1) 4 km (2) 2 km (3) 1 km (4) 3 km (5) None of these

    Directions (7-9) : Farmer Batuk Singh has a larger square field divided into nine smaller squares, all equal, arranged in three rows of three fields each. One side of the field runs exactly East West. The middle square must be planted with rice because it is wet. The wheat and barley should be continuous so that they can be harvested all at once by the mechanical harvester. Two of the field should be planted with soyabeans. The North Western most field should be planted with peanuts and the Southern third of the field is suitable only for vegetables. These questions refer to the following squares (1) The square immediately North of the rice (2) The square immediately East of the rice (3) The square immediately West of the rice (4) The square immediately NorthEast of the rice. (5) None of these 7. If Batuk Singh decides to plant the wheat next to the peanuts, in which square will the barley be? 8. Which square cannot be planted with wheat? 9. Which square cannot be planted with soyabeans? 10. ‘A’ is east of ‘B’ and west of ‘C’, ‘H’ is south-west of ‘C’ and ‘B’ is South-East of ‘X’. Who is at the farthest West? (1) C (2) A (3) X (4) B (5) None of these 11. A river flows West to East and on the way turns left and goes in a semicircle round a hillock and then turns left in a right-angles. In which direction is the river finally flowing? (1) North (2) South (3) East (4) West (5) None of these 12. A postman was returning to the post office which was in front of him to the North. When the post office was 100 m away from him, he turned to the left and moved 50 m to deliver the last letter at

    BPRE–259

    Shantivilla. He then moved in the same direction for 40 m, turned to his right and moved 100 m. How many m away he was now from the post office? (1) 0 (2) 150 (3) 90 (4) 100 (5) None of these 13. Standing on a rock, Ravi said that Madurai was more than 3 km but less than 8 km from there. Prabhu said that it was more than 6 km but less than 10 km from there. If both of them are correct, how far is Madurai from the rock? (1) 8 km (2) 6 km (3) 7 km (4) Cannot be determined (5) None of these 14. Ram walks 10 m South from his house, turns left and walks 23 m, again turns left and walks 40 m, then turns right and walks 5 m to reach his school. In which direction is the school from his house? (1) East (2) North-East (3) South-West (4) North (5) None of these 15. A taxi driver commenced his journey from a point and drove 10 km towards North and turned to his left and drove another 5 km. After waiting to meet one of his friends, he turned to his right and continued to drive another 10 km. He has covered a distance of 25 km so far but in which direction may he now be going? (1) North (2) West (3) East (4) South (5) None of these 16. A girl leaves from her home. She first walks 30 m in North-West direction and then 30 m in SouthWest direction. Next, she walks 30 m in South-East direction. Finally, she turns towards her house. In which direction is she moving? (1) South-West (2) South-East (3) North-East (4) North-West (5) None of these

    DISTANCE & DIRECTION 17. Rajesh walked 25 m towards South. Then he turned to his left and walked 20 m. He then turned to his left and walked 25 m. He again turned to his right and walked 15 m. At what distance is he from the starting point and in which direction? (1) 60 m, East (2) 35 m, East (3) 35 m, North (4) 40 m, East (5) None of these 18. Sarita is standing facing North, she walks 10 km straight, turns left and walks another 10 km and turns right and walks 5 km and finally turns left and walks 15 km to reach a park. Which direction is she facing now? (1) East (2) West (3) North (4) South (5) None of these 19. The door of Aditya’s house faces the East. From the back side of his house, he walks straight 50 m, then turns to the right and walks 50 m again. Finally, he turns towards left and stops after walking 25 m. Now, Aditya is in which direction from the starting point? (1) North-East (2) South-East (3) South-West (4) North-West (5) None of these 20. Two ladies and two men are playing cards and are seated at North, East, South and West of a table. No lady is facing East. Persons sitting opposite to each other are not of the same sex. One man is facing South. Which directions are the ladies facing? (1) South and East (2) East and West (3) North and East (4) North and West (5) None of these 21. Bhavika and Sunaina start simultaneously towards each other from two places 100 m apart. After walking 30 m, Bhavika turns left and goes 10 m, then she turns right and goes 20 m and then turns right again and comes back to the road on which she had started walking. If both Bhavika and Sunaina walk with the same speed, what is the distance between them at this point of time? (1) 70 m (2) 10 m (3) 40 m (4) 20 m (5) None of these

    22. A man starts from a point and moves 3 km north, then turns to west and goes 2 km. He turns North and walks 1 km and then moves 5 km towards, East. How far is he from the starting point? (1) 11 km (2) 10 km (3) 5 km (4) 8 km (5) None of these 23. Ruchi’s house is to the right of Vani’s house at a distance of 20 m in the same row facing North. Shabina’s house is in the NorthEast direction of Vani’s house at a distance of 25 m. Determine that Ruchi’s house is in which direction with respect to Shabina’s house? (1) South (2) East (3) North-East (4) West (5) None of these 24. Kailash faces towards North. “Turning to his right, he walks 25 m. He then turns to his left and walks 30 m. Next, he moves 25 m to his right. He then turns to his right again and walks 55 m. Finally, he turns to right and moves 40 m. In which direction is he now from his starting point? (1) South (2) South-West (3) North-West (4) South-East (5) None of these Directions (25-28) : Attempt these questions based on the information given. P, Q, R and S are standing on four corners of a square piece of plot as shown in the given figure. They start moving and the movements are explained in each of the questions. Q

    R N

    S

    P

    E w S

    25. P Q, R and S walk diagonally to opposite corners and from there, Q and R walk one and a half sides anti-clockwise while P and S walk one side clockwise along the sides. Where is S now?

    BPRE–260

    (1) At the South-West corner (2) At the North-East corner (3) At the North-West corner (4) None of the above (5) None of these 26. Q travelled straight to R, a distance of 10 km. He turned right and walked 7 km towards S. Again, he turned right and walked 8 km and then finally turned right and walked 7 km. How far is he from his original position? (1) 2 km (2) 8 km (3) 7 km (4) 3 km (5) None of these 27. P and S walk one and a half lengths of the side clockwise and clockwise respectively. Which one of the following statements is true? (1) P and S are both at the midpoint between Q and R (2) P and S are at the midpoints between R and Sand Q and R respectively (3) P is at midpoint between Q and R and S is at the corner originally occupied by P (4) S is at the midpoint between Q and R and P is at the midpoint between the original side of R and S (5) None of these 28. P, Q, R and S walk one and a half sides clockwise. Who is on the left of Q, if he is facing West? (1) S (2) R (3) P (4) Q (5) None of these 29. A train runs 120 km in West direction, then 30 km in South direction and then 80 km in East direction before reaching the station. In which direction is the station from the train’s starting point? (1) South-East (2) North-West (3) South-West (4) South (5) None of these 30. Manu goes 40 km North, turns right and goes 80 km, turns right again and goes 30 km. In the end, he turns right again and goes 80 km. How far is he from his starting point if he goes straight ahead another 50 km and turns left to go his last 10 km? (1) 40 km (2) 30 km (3) 10 km (4) 50 km (5) None of these

    DISTANCE & DIRECTION 31. From a point, Rajneesh started walking towards East and walked 35 m. He then turned towards his right and walked 20 m and he again turned right and walked 35 m. Finally, he turned to his left and walked 20 m and he reached his destination. Now, how far is he from his starting point? (1) 50 m (2) 55 m (3) 20 m (4) 40 m (5) None of these 32. A directional post is erected on a crossing. In an accident, it was turned in such a way that the arrow which was first showing East is now showing South. A passerby went in a wrong direction thinking it is West. In which direction is he actually travelling now? (1) South (2) North (3) East (4) West (5) None of these 33. A person goes 20 m towards South, then turns left and goes 20 m and another 20 m to the left and then 60 m to the right. In which direction is he going now? (1) North (2) South (3) East (4) West (5) None of these 34. A man was walking with his face towards East. He walked 30 m and then turned to his right by another 30 m. He again turned towards his right by 60 m. He now turned back by 90 m. Which direction was he facing now? (1) North (2) South (3) East (4) West (5) None of these 35. From the given equilateral triangle below, Rakesh, Lokesh and Bhagat run along the sides in clockwise direction and stop af-

    1 sides. Which of 2 the following statements is true?

    ter covering 1

    N Rakesh w

    Lokesh

    Bhagat

    E

    S

    (1) Lokesh is to the west of Bhagat (2) Rakesh is to the South-West of Lokesh (3) Lokesh is to the North-West of Rakesh (4) Bhagat is to the North-East of Rakesh (5) None of these 36. Village A is 20 km to the north of Village B. Village C is 18 km to the East of Village B, Village D is 12 km to the West of Village A. If Raj Gopal starts from Village C and goes to Village D, in which direction is he from his starting point? (1) North-East (2) North-West (3) South-East (4) North (5) None of these 37. If South-East becomes North, North-East becomes West and so on, what will West become? (1) South-West (2) South-East (3) North-East (4) North-West (5) None of these 38. While facing East, I turn to my left and walk 10 m, then turn to my left and walk 10 m. Now, I turn 45 degrees towards my right in North-West direction and cover 25 m. At this point, in which direction am I from my starting point? (1) South-East (2) South-West (3) North-East (4) North-West (5) None of these 39. From my house, I went towards North for 10 m, took left turn for 10 m, right turn for 5 m and walked taking right turn. In what direction was I going at that point? (1) East (2) South (3) North (4) West (5) None of these 40. Rajesh’s car is facing towards South. He drives 10 km and turns left and drives 10 km. Then, he turns right and drives 40 km. He turns right again and drives 30 km. Lastly, he turns right and drives 50 km. How far is Rajesh from the starting point? (1) 10 km (2) 20 km (3) 40 km (4) 60 km (5) None of these

    BPRE–261

    41. Shravan and Bhagat start walking from a point. Shravan walks in West direction and Bhagat in South direction. After covering 20 km, Shravan turns left and walks 15 km. Bhagat walks 10 km, turns left and walks 5 km. Shravan, then turns left again and walks 25 km, whereas Bhagat turns right and walks 5 km. How far are Shravan and Bhagat from each other? (1) They are back at the starting point (2) 5 km (3) They are at the same place at the finishing point (4) Data insufficient (5) None of these 42. Two friends Jack and Bunty start a race and together they run for 50 m. Jack turns right and runs 60 m while Bunty turns left and runs 40 m. Then, Jack turns left and runs 50 m while Bunty turns right and runs 50 m. How far are the two friends now from each other? (1) 130 m (2) 100 m (3) 150 m (4) 140 m (5) None of these 43. Hemant walks from his house and moves 4 m towards North. He turns left at 90° to his right and moves 3 m. He takes another 90° turn to the right and walks 1 m. From here, Hemant turns 90° to the left and moves for another 1 m. He finally turns 90° to the right and moves for another 3 m, his final destination. What is the distance between the starting and destination points? (1) 4 m (2) 7 m (3) 5 m (4) 8 m (5) None of these 44. A child is looking for his mother. He went 80 m in the East before turning to his right. He went 20 m before turning to his right again to trace his mother at his married sister’s house, 20 m from this point. His mother was not there. From there, he went 100 m to his North where he met his mother who was shopping there in the market. How far did the son meet his mother from the starting point? (1) 80 m (2) 60 m (3) 100 m (4) 140 m (5) None of these

    DISTANCE & DIRECTION 45. A person is facing West. He turns 60° in the clockwise direction and then another 180° in the same direction and then turns 270° in the anti-clockwise direction. Which direction is he facing now? (1) North-West (2) West (3) South-West (4) South (5) None of these 46. Rishi walks 20 km towards North. From there, he walks 16 km towards South. Then, he walks 3 km towards East. How far and in which direction is Rishi with reference to his starting point? (1) 5 km North-East (2) 5 km West (3) 7 km East (4) 7 km West (5) None of these 47. A clock is placed in such a manner that at 12 O’clock, its minutehand points towards north-east. In which direction does its hourhand point at 1:30 pm ? (1) West (2) South (3) North (4) East (5) None of these 48. Ram starts from his house on cycle and goes 10 km towards North-East. He turns to the left and cycles 8 km, then he turns towards South-West and cycles for another 10 km. Finally, he turns towards his house. In which direction is he going now ? (1) South (2) South-East (3) West (4) North-West (5) None of these 49. Mahender walked 30 metres towards east, took a right turn and walked another 40 metres. He again took a left turn and moved 30 metres. In which direction is he now from the starting point ? (1) North-East (2) East (3) South-East (4) South (5) None of these 50. Point B is 5 metres to the east of Point A. Point C is 3 metres to the south of Point B. Point D is 4 metres to the west of Point C. Point D is 7 metres to the south of Point E. Point F is 1 metre to the west of Point E. Point G is 1 metre to the south of Point F. If Point H is 1 metre to the east of Point A, then what is the distance between Point H and Point C ? (1) 7 metres (3) 3 metres (5) 8 metres

    (2) 5 metres (4) 4 metres

    Directions (51–52) : Study the following information carefully and answer the questions given below : Amaya starts from point P dacing North. He walks for 5m and reaches point Q. He takes a right turn and walks for 14m and reaches point R. He then takes a left turn, walks for 2.5 m and reaches point S. He takes another left turn, walks for 7m and reaches point T. He takes a left turn again, walks for 7.5m and reaches points U. He takes a left turn, walks for 3.5 to reach point V. 51. How far and in which direction is Amaya from point P? (1) 5.5 m West (2) 7 m East (3) 3.5 m North–East (4) 10.5 m East (5) 10.5 m South–East 52. Had Amaya takes a right instead of taking a left from point T, while reaching everything else the same, in what direction would point V be from point T? (1) South (2) North (3) North–West (4) North–East (5) South–East 53. A is 15 m to the west of R. R is 20 m to the north of G. G is 12 m to the east of D. F is 16 m to the north of D. If A has to meet F through the shortest distance and then has to meet G from F through the shortest distance, then what is the total distance travelled by A? (1) 20 metre (2) 25 metre (3) 30 metre (4) 35 metre (5) 15 metre 54. Statement : Hameed walks “# % + ?” Conclusions : I. The distance between the starting point and end point is 13 metre. II. Hameed is standing in west direction with respect to the starting point. 55. Statement : John walks “! & # $ ?” Conclusions : I. The distance between the starting point and end point is 9 metre.

    BPRE–262

    II. The total distance John travelled is 26 metre. 56. Statement : Lalitha walks “& ? # !” Conclusions : I. Lalitha is standing southwest direction with respect to the starting point. II. The distance between the starting point and end point is 10 metre. 57. Statement : Ravi walks “@ & @ ! $” Conclusions : I. Ravi is standing west direction with respect to the starting point. II. The distance between the starting point and end point is 106 metre. 58. Statement : Santosh “& ! $ >” Conclusions : I. The distance between the starting point and end point is 101 metre II. The distance between the starting point and end point is

    1

    metre Directions (59– 63): In the following questions, some symbols are used with the following meanings as illustrated below. Study the following information and answer the given questions: X@Y X is in the north direction of Y X is in the east direction of X&Y Y X is in the west direction X$Y of Y X is in the northeast direcX!Y tion of Y X+Y X is in the northwest direction of Y X#Y X is in the south direction of Y X is in the southeast direcX*Y tion of Y X is in the south west direcX^Y tion of Y Statement: There are 8 friends residing in a city in their respective homes as in the following directions. (Distance in km) Suman $ Ritu + Komal & Mala @ Shilpa ^ Komal @ Lalita $ Ambika ^ Parvati & Durga

    DISTANCE & DIRECTION 59. In which direction Suman’s house is located with respect to Lalita’s house? (1) South (2) Northeast (3) Southwest (4) Northwest (5) North 60. In which direction Ritu’s house is located with respect to Mala’s house? (1) Southeast (2) Northeast (3) South (4) North (5) Northwest 61. In which direction Shilpa’s house is located with respect to Parvati’s house? (1) West (2) North (3) South (4) Northwest (5) Southwest 62. In which direction Ambika’s house is located with respect to Ritu’s house? (1) Southeast (2) Northeast (3) Southwest (4) Northwest (5) East 63. In which direction Lalita’s house is located with respect to Parvati’s house? (1) Southeast (2) Northeast (3) Southwest (4) Northwest (5) North Directions (64–68) : Study the following information carefully and answer the questions given below : There are eight districts P, Q, R, S, T, U, V and W in a state geographically arranged in a circular manner. There are five archaeologists A, B, C, D and E are sitting around a circular table and marking the state boundaries in the following directional map.

    NW

    N

    W SW

    NE E

    S

    SE

    All the archaeologist are sitting around a circular table facing the centre

    ● B says that the district W is sec-

    ond to the left of a district that is opposite district P ● The district U is to the immediate right of a district that is second to the right of district S. ● A is to the immediate left of an archaeologist who is second right of E says that district T is third to the left of district R ● E who is an immediate neighbour of either B or D says that district V is in either Northwest or in the east. ● C says district Q is in neither southwest nor in west ● B knows that the district R is shortly reachable between the districts P and U ● A who is an immediate neighbour of D says that the district in southwest is second to the right of a district that is second to the right of district R 64. Which of the following district is located second to the right of the district in Northwest direction? (1) District Q (2) District V (3) District T (4) District P (5) District W 65. What is the position of E with respect to D around the circular table? (1) Immediate right (2) Immediate left (3) Second to the right (4) Second to the left (5) None of these 66. Which of the following district is located exactly between Southwest and Southeast ? (1) District P (2) District U (3) District W (4) District T (5) District V 67. If B belongs to district R and D belongs to district S, then what is the position of B with respect to D ? (1) Second to the left (2) Just opposite (3) Immediate right (4) Second to the right (5) Immediate left 68. If each district is placed equi-distant of 15 km, what is the distance between districts W and T ? (1) 30 km (2) 15 km (3) 45 km (4) 60 km (5) 75 km

    BPRE–263

    Directions (69-73) : In the following questions, the symbols #, &, @ and $ are used with the following meanings as illustrated below. Study the following information and answer the given questions : Note : The directions which are given indicate exact directions. ● X # Y - X is in the south direction of Y AND X is the son of Y. ● X @ Y - X is in the north direction of Y AND X is the mother of Y. ● X & Y - X is in the east direction of Y AND X is the father of Y. ● X $ Y - X is in the west direction of Y AND X is the maternal uncle of Y. ● X * Y - X is in the southeast direction of Y AND X is the paternal uncle of Y. ● X ! Y - X is in the northeast direction of Y AND X is the brother of Y. ● X + Y – X is in the northwest direction of Y AND X is the sister of Y. ● X ^ Y – X is in the south west direction of Y AND X is the niece to Y. Study the Conclusions based on the given statement and select the appropriate answer. Give answer (1) if only Conclusion I is true Give answer (2) if only Conclusion II is true Give answer (3) if either Conclusion I or Conclusion II is true Give answer (4) if neither Conclusion I nor Conclusion II is true Give answer (5) if both the Conclusion I and Conclusion II are true 69. Statement : K * M ! T & N # S @ Z, as SN = NZ and SZ = 10 metre Conclusions : M ! Z as per directions N # T as per relations 70. Statement : D&M!T#K+H@Y Conclusions : I. D ^ Y as per directions II. T # D as per relations 71. If H $ B @ S # M & T is true, as MS = 10 metre and MB = BS, then Conclusions : I. B @ T as per relations II. S * T as per directions

    DISTANCE & DIRECTION 72. If A # B @ C & D $ E ! F is true, as BA = AC and DC = CE, then Conclusions : I. F is the grand grand child of B II. A is in northwest of E 73. If E ! F + G # H & I ! J is true, as EG = 10 metre where HG = EG/ 2 and F is in east of I, then Conclusions: I. There are two daughters of H. II. J is in northwest direction of H. 74. X walks 10 metre followed by 5 metre to his left. Then he takes right turn and walks 5 metre followed by 10 metre to his right. He reaches his house after walking 5 metre to his left. In which direction is he standing now from the starting point, if he started towards east? (1) South–west (2) South (3) North–east (4) North–west (5) South–east 75. A School is located to the south of Bus stand. A College is located to the north of Railway station. Bus stand is located to the west of Post Office which is located to the east of College. If a Mall is located to the east of School and south east of College, then in which direction is Post Office with respect to School? (1) North (2) South (3) South–east (4) North–east (5) South–west Directions (76–78) : Read the following information carefully and answer the questions given below: Roshan started from his house and travelled 8 km towards east. Then he turned right and travelled 8 km followed by 6 km to his left. He reached a market after travelling 4 km to his left. He again travelled 6 km towards east. Then he turned left and travelled 4 km followed by 3 km to his right to reach a Bus stand. He then travelled 10 km to his right followed by 3 km to his right. Finally he reached Gagan’s house after travelling 2 km to his right. 76. What is the distance between Roshan’s house and Bus stand? (1) 24 km. (2) 23 km. (3) 17 km. (4) 18 km. (5) 19 km.

    77. In which direction is Market with respect to Gagan’s house? (1) North–East (2) South – East (3) North – West (4) West (5) South 78. If a point Z is 6 km to the east of Market, what is the shortest between point Z and the Bus Stand ? (1) 5 km. (2) 9 km. (3) 13 km. (4) 14 km. (5) 17 km. Directions (79–81) : Study the following information carefully and answer the questions given below : ● There are 9 traffic signals A, B, C, D, E, F, G, H and Z in a town, located in different directions. ● Signal D is located in northwest direction of Signal E. ● Signal A is located 8 km west of Signal B. ● Signal Z is located in northeast of Signal H. ● Signal C is located in Southeast direction of Signal F. ● Signal G is located 16 km in west direction of Signal E. ● Signal H is located in southwest direction of Signal D. ● Signal Z is located 8 km north of Signal B. ● Signal C is located 4 km south of Signal D. ● Signal F is located 8 km north of a Signal which is 8 km west of Signal A. ● Signal E is exactly at midpoint between Signals Z and B. ● Signal D is 8 km north of Signal F which is located to the north of Signal G. 79. In which direction is signal G with respect to signal B? (1) South-west (2) North-west (3) North (4) South-east (5) North-east 80. Which of the following pairs of the signals lies in a straight line? (1) G and A (2) C and H (3) D and G (4) Z and A (5) C and A

    BPRE–264

    81. What is the area covered by the signals G, C, A and H ? (1) 48 sq. km. (2) 32 sq. km. (3) 40 sq. km. (4) 64 sq. km. (5) 54 sq. km. Directions (82 – 84) : Study the following information carefully and answer the questions given below: There are NINE traffic signals P, Q, R, S, T, U, V, W and X in a town, located in different directions. Signal V is located 20 km west to Signal T. Signal W is located to the southwest of Signal S. Signal X is located 10 km north to Signal Q. Signal S is located northwest of Signal T. Signal R is located 5 km south to Signal S. Signal P is located 10 km west to Signal Q. Signal X is located to the northeast of Signal W. Signal T is midpoint between Signals X and Q. Signal S is 10 km east of Signal U which is located to the north of Signals V and W. Signal R is located to the Southeast of Signal U. Signal U is located 10 km north to a Signal which is 10 km to the west of Signal P. 82. In which direction is T with respect to W? (1) Southeast (2) Southwest (3) North (4) Northwest (5) Northeast 83. In which direction is S with respect to Q? (1) Northeast (2) Northwest (3) Southeast (4) South (5) Southwest 84. If a temporary signal A was fixed 2 km to the east of signal P, what is the shortest distance between signal V and signal A ? (1) 5 km (2) 10 km (3) 13 km (4) 17 km

    DISTANCE & DIRECTION 3. (2) School is in North-East direction

    SHORT ANSWERS 1.(3)

    2.(1)

    3.(2)

    5.(4)

    6.(3)

    7.(4)

    8.(3)

    9.(4)

    10.(2)

    11.(3)

    12.(3)

    13.(4)

    14.(2)

    15.(1)

    16.(3)

    17.(2)

    18.(2)

    19.(4)

    20.(4)

    21.(4)

    22.(3)

    23.(1)

    24.(4)

    25.(2)

    26.(1)

    27.(1)

    28.(2)

    29.(3)

    30.(4)

    31.(4)

    32.(2)

    33.(3)

    34.(3)

    35.(2)

    36.(2)

    37.(2)

    38.(4)

    39.(1)

    40.(2)

    41.(3)

    42.(2)

    43.(1)

    44.(3)

    45.(3)

    46.(1)

    47.(4)

    48.(2)

    49.(3)

    50.(2)

    51.(4)

    52.(3)

    53.(2)

    54.(5)

    55.(2)

    56.(5)

    57.(2)

    58.(3)

    59.(4)

    60.(4)

    61.(5)

    62.(1)

    63.(3)

    64.(2)

    65.(4)

    66.(4)

    67.(2)

    68.(1)

    69.(5)

    70.(2)

    71.(5)

    72.(5)

    73.(4)

    74.(5)

    75.(4)

    76.(2)

    77.(3)

    78.(1)

    79.(2)

    80.(5)

    81.(2)

    82.(5)

    83.(2)

    84.(4)

    House

    End point

    W

    10 m

    E

    West

    12. (3)

    (4-6) :

    a

    f

    100

    100

    40

    1

    b

    c

    2

    1

    g

    h

    d

    e

    1

    N

    4. (1) The distance between e and g is 2 km. 5. (4) Distance between a and f is 1 km. 6. (3) The distance between e and I is 1 km (7-9) : F1

    Peanuts

    F2

    Soyabeans

    Wheat

    50

    Hence the postman is 50 + 40 = 90 m from the post office. 13. (4) Let the distance to Madurai be x km. Then, according to Ravi, 3 < x < 8 and according to Prabha, 6 < x < 10 Given, when both of them are correct, the distance to Madurai cannot be determined exactly as it could be any thing between 6 and 8. Note:-The distance may be 6.2 km, 6.8 km, 7.5 km. 14. (2) 5m

    Barley Vegetables

    School

    House

    N

    E

    W 5m

    Rice

    F3

    40 m

    S 5m

    2. (1) Distance of both of them from starting point.

    7. (4) In the square which is North East of rice. 8. (3) The square immediately North West of rice square cannot be planted with wheat. 9. (4) The square immediately North East of the rice square cannot be planted with soyabean. 10. (2) According to figure, B is the farthest distance. X

    5 km

    P

    Q

    The positions of all the nine houses are given below

    Starting House point

    4 km

    East

    S

    25 m

    1. (3) Clearly, end point is to the North of starting point.

    10 m

    N

    40 m

    EXPLANATIONS

    5m

    School

    5m

    4.(1)

    will be B, because X will be North West of C. 11. (3) River, Finally goes in the East direction.

    B

    5 km 3 km

    A H

    C

    N

    W

    W

    E

    10 m S

    23 m

    Hence, the school of Ram is to the North-East from the house. 15. (1)

    N

    10 km

    E

    W

    5 km

    S

    E S

    3 km 4 km Starting point

    =

    4 2 + 32 =

    25 = 5 km

    Here, X is at the most farthest west. If in question it is written that with respect to C who one is at most farthest then the answer

    BPRE–265

    Start

    From the figure, it is clear that he is going in North direction.

    DISTANCE & DIRECTION 16. (3)

    21. (4)

    26. (1) Q

    m

    20 m 30

    30 m

    10 m

    N

    10 m

    E

    W

    Bhavika

    S

    For reaching her house, she should move North-East direction. 17. (2) End N

    25 m W

    25 m

    Sunaina

    100 m

    30 m

    15 m

    S

    18. (2)

    2 km

    ∴ PQ =

    Start 25

    From the figure it is clear that she is facing wast direction. 19. (4)

    R is on the left of Q. 120 km

    V

    m

    W

    E

    25 m

    Man N E

    W

    80 km

    R

    S

    So, ladies are facing North and West direction.

    W

    End

    S

    W

    S

    P

    N

    E

    R

    Start

    A

    B 35 m 20 m

    S

    W

    E

    Q

    So, S is at North-East corner.

    BPRE–266

    31. (4)

    S

    R

    D

    S

    S

    S

    Start

    Required distance = 50 km

    a

    N

    E

    50 km

    10 km

    P

    S

    80 km

    40 km

    E

    55 m W

    40 m

    N

    Lady

    So, the train is in South-West direction from starting point. 30. (4)

    30 km

    So, he is in South-East direction from starting point. 25. (2) Q

    S

    N

    40 m

    End

    50 m

    E

    W

    30 km

    Start

    S

    N

    80 km End

    S

    S

    20 m

    Start

    N

    30 m 25 m

    Lady

    S R

    So, Ruchi’s house is in South direction from Shabina’s house. 24. (4)

    So, Aditya is in North-West direction from the starting point. 20. (4)

    E Q

    29. (3)

    E

    50 m W

    Man

    S W

    25 = 5 KM R

    N

    P and S are both at the midpoint between Q and R. 28. (2) N

    23. (1)

    25m

    E

    QR = 5 km – 2 km = 3 km PQ2 = QR2 + PR2 = 9 + 16

    E S

    R

    W

    P

    S

    10 km

    P S N

    P

    N W

    Q

    27. (1)

    E

    5 km 10 km

    ∴ Required distance = 10 – 8 = 2 km

    3 km W

    15 km

    Q

    R

    N

    Required distance = 20 + 15 = 35 m in the East.

    S

    S

    5 km

    E

    7 km

    8 km

    When Bhavika travels a distance of 50 m (in the straight line), Sunaina travels a distance of 70 m. ∴ Distance between them = (70 – 50) = 20 m 22. (3) 1 km

    20 m

    7 km

    30 m

    Start

    Start

    10 km R

    P

    20 m E

    35 m

    C

    DISTANCE & DIRECTION Thus Raj Gopal is moving in North-West direction. 37. (2)

    Required distance AE = 20 + 20 = 40 m 32. (2) W'

    E

    W

    S'

    N'

    S

    F

    N

    20 km A

    N

    End N

    41. (3)

    W

    10 km

    E

    W

    E

    15 km

    S

    C

    5 km

    E'

    From the figure, it is clear that North is showing West. So that person is going in North direction. 33. (3) Starting point

    60 m

    20 m

    20 m

    25 E

    m

    Z

    25 km

    Now the directions changed by 135° anti-clock direction. So, West become South-East. 38. (4)

    N

    W

    SE

    S

    Let A be the starting point. From the graph, it clear that they meet ar Z point which is thier finishing point.

    N

    10 m

    45

    E

    W S

    42. (2)

    50 m

    10 m S

    20 m

    So, person is now going to East. 34. (3) Starting Point

    Bunty

    Start

    30 m

    So, the person is in North-West direction from starting point. 39. (1)

    N

    30 m

    E

    W

    10 m

    60 m

    E

    W

    50 m

    South

    So man is now facing in the east. 35. (2) Rakesh New diagram After movement

    So, the person is going in East direction. 40. (2)

    Bhagat

    Required distance = 60 + 40 = 100 m 43. (1)

    Lokesh

    Bhagat

    N

    N Rakesh

    A

    Start A

    F

    B

    E

    W

    3m

    C

    W

    E

    1m

    Thus, Rakesh is to the SouthWest of Lokesh. 36. (2) 12 km

    60 m

    S

    90 m

    D

    Jack

    10 m

    S

    Lokesh

    50 m

    N

    5m

    40 m

    20 km B

    10 km

    50 km

    C

    1m D 1m

    S

    40 km

    4m

    E

    S

    3m

    20 km E

    30 km

    D

    A

    B

    18 km

    C

    Required distance AF = DE – BC = 30 – 10 = 20 km

    BPRE–267

    F

    Required distance AF = BC + DE = 4m

    DISTANCE & DIRECTION 44. (3)

    and the direction will be North East.

    80m

    N E

    W

    N

    E 20m B 20m

    60m

    16 km

    AE2 + EF 2

    =

    602 + 802

    =

    3600 + 6400

    =

    10,0 00

    =

    S

    North NE

    East

    Required distance HC

    m

    1

    m 0k

    SE South

    8k

    m 0k

    D

    =

    (HB)2 + (BC)2

    =

    (4)2 + (3)2

    =

    16 + 9

    =

    25 = 5 metre

    (51–52) :

    House

    7m

    T

    49. (3)

    S 2.5 m

    Let the initial position of person be AB. After turning 60°, it becomes AC. After turning 180°, it becomes AD. After turning 270°, in anticlockwise direction, which is 30°(270° – 180° – 60°) ACW the final position of AE. So, he is facing South-West Direction. 46. (1) AC = AB – BC = 20 – 16 = 4 km

    East

    West

    SW

    A

    E

    NE

    SE

    1

    S

    180

    C

    NW

    48. (2)

    SW

    3m

    North

    NW

    According to question the hourhand will point towards East at 1.30 p.m. E

    W

    B

    D

    South

    C

    4m

    H 3m

    SW

    N

    ∴ AD =

    1m

    4 km A Start

    b60g2 + b80g2

    45. (3)

    60

    m

    3m D

    West

    AE 2 + EF 2

    270

    4

    A

    = 10000 = 100 m

    B

    3 km

    C

    G

    E

    W

    47. (4) In normal position the minute hand would point towards North at 12 O’ clock and the hour-hand would point towards North-East at 1.30 pm.

    = 100 m AE = AB – BE = 80 – 20 = 60m EF = DF – DE = 100 – 20 = 80m ∴ Required distance AF =

    E

    1m

    20 km

    D C Sister's 20m house EF = DF – DE = 100 – 20 = 80m

    AF =

    1m

    F

    B

    S

    20m

    A

    50. (2)

    = 16 + 9 = 5km

    F

    Starting Point 30m

    Q

    14 m

    R 7.5 m

    5m 40m

    AC2 + CD2

    3.5 m P

    30m

    BPRE–268

    7m

    U

    V

    DISTANCE & DIRECTION North

    2

    bFDg + bDGg

    FG = NE

    NW

    2

    b16g + b12g West

    East

    2

    =

    256 + 144

    =

    400 = 20 metre Total distance = (5 + 20) metre = 25 metre 54. (5)

    SE

    SW South

    5m E

    4m D 3m

    U

    3.5 m

    AE =

    T

    S

    Point V is in North–West direction from point T. 53. (2) 15 m

    A

    F

    8m

    B

    C 7.5 m

    (AF)2 + (EF)2

    =

    (5)2 + (12)2 =

    5m

    F

    =

    169 = 13 metre

    55. (2)

    AF = =

    (9)2 + (5)2

    =

    81 + 25

    =

    106 metre 4m

    C

    5m

    Case-II 4m B

    4m B

    8m

    D

    North NE

    NW

    =

    82 metre

    56. (5)

    South AF = = =

    2

    b ATg + b TF g 2

    b 4 g + b3 g

    2

    16 + 9 =

    AE = =

    25 = 5 metre

    =

    (1)2 + (10)2

    =

    1 + 10 =

    101 metre

    C

    North 5m

    E 2

    2

    (AF) + (EF)

    =

    4m

    F

    SE

    2

    (59–63) :

    5m SW

    E

    AE =

    81 + 1

    5m

    D

    5m

    E

    (AG)2 + (GF)2

    (9)2 + (1)2 =

    East

    West

    5m

    10m D

    C

    4m

    =

    F

    F

    G

    16 m

    AF =

    A

    5m

    20 m

    G

    5m

    A

    R

    12 m

    (AE)2 + (EF)2

    5m

    5m

    F

    D

    B

    4m

    C

    58. (3) Case-I

    4m 3m

    A

    D

    5m

    5 + 144

    4m T

    End Point E 5m

    A

    51. (4) Amaya is (7 + 3.5) metres = 10.5 metres to the East from point P. 52. (3)

    V

    57. (2)

    2

    NE

    NW

    B

    West 2

    East

    2

    (AF) + (FE)

    (3)2 + (1)2

    =

    9 +1

    10 metre

    BPRE–269

    SW

    SE

    South

    DISTANCE & DIRECTION

    Suman

    59. 60. 61. 62. 63.

    (4) (4) (5) (1) (3)

    Ritu

    Mala

    Komal

    Durga

    Shilpa

    Lalita

    Ambika

    Parvati

    Suman’s house is in North-west of Lalita’s house. Ritu’s house is in North of Mala’s house. Shilpa’s house is in South-west of Parvati’s house. Ambika’s house is in South-east of Ritu’s house. Lalita’s house is in South-west of Parvati’s house.

    P R

    W

    U Q

    S

    B

    T&N T is the father of N. N#S N is the son of S. S is the mother of Z. S@Z Conclusion II N is the son of T. N#T It is true. 70. (2) K M D

    (64–68) : V

    T

    C

    H

    T

    D

    B

    A

    E

    64. (2) District R is in North-west direction. District V is second to the right of district R. 65. (4) E is second to the left of D. 66. (4) Southwest ⇒ S Southeast ⇒ Q T is exactly between S and Q. 67. (2) According to the question B is just opposite D. 68. (1) W S ⇒ 15 km. S T ⇒ 15 km. W T ⇒ (15 + 15) km. = 30 km. 69. (5) S M 5m N

    T

    Conclusion II S*T S is in southeast directon of T. It is true. A is the son of B. 72. (5) A # B B@C B is the mother of C. C is the father of D. C&D D $ E D is the maternal uncle of E. E!F E is the brother of F. Conclusion I B is the paternal grandmother of D. D is the maternal uncle of F. Therefore, F is the grand grand child of D.

    K

    5m Z

    Conclusion I M is in North-east diM!Z rection of Z. It is true. K is the paternal unK*M cle of M. M!T M is the brother of T.

    Y Conclusion I D is in south-west D ^ Y direction of Y. It is not true. D&M D is the father of M. M!T M is the brother of T. T#K T is the son of K. K+H K is the sister of H. H is the mother of Y. H@Y Conclusion II T#D T is the son of D. M and T are sons of D and K. 71. (5) H $ B H is the maternal uncle of B. B is the mother of S. B@S S#M S is the son of M. M$T M is the father of T. Conclusion I B is the other of T. B@T It is true. T

    A

    E

    C

    D

    F A is in north-west of E. E is brother of F. 73. (4) E ! F F+G F is sister of G. G#H G is son of H. H is father of I. H&I I!J I is brother of J. J may or may not be daughter of H. Thus, Conclusion I is not true.

    E

    I

    H

    F

    J

    E

    J is in the south-west direction of H. 74. (5) 5 metre

    M 5 metre

    H

    B

    10 metre

    10 metre 5 metre

    S

    BPRE–270

    House

    DISTANCE & DIRECTION (79–81) :

    North NW

    NE

    West

    East

    8 km

    D

    Z

    4 km

    SE

    SW

    8 km

    F

    4 km 8 km

    G

    8 km

    C

    E

    South Now, X is in South–east direction from the starting point. 75. (4)

    Bus Stand

    College

    4 km

    4 km

    H

    A

    8 km

    B

    8 km

    Post Office North NE

    NW School

    Railway Station

    Mall West

    Post office is in North–east direction with respect to School. (76–78) : Starting Point A

    B

    F 3 km

    8 km

    Bus stand G

    4 km Market E 6 km

    8 km

    4 km K

    Z 4 km

    4 km C

    6 km

    10 km

    Gagan’s J House 2 km

    D

    I

    3 km

    H

    East SE

    SW South

    79. (2) Signal G is to the North-west of Signal B. 80. (5) Signals C and A lie in a straight line. 81. (2) Area of GCAH = 8 × 4 sq. km = 32 sq. km. (82–84) : 10 km

    U

    10 km

    W

    West

    P 2 km A

    10 km

    East

    Q

    8 km

    North NW

    SW

    T 5 km

    5 km

    5 km

    NE

    10 km

    R

    North

    X 5 km

    5 km

    5 km V

    NW

    10 km

    S

    West

    SE South

    NE East

    SE South 82. (5) T is in north-east direction with respect to W. 83. (2) S is in north-west direction with respect to Q. SW

    76. (2) AG = (8 + 6 + 6 + 3) km. = 23 km. 77. (3) Market is to the North-west of Gagan’s House. 78. (1) ZG =

    (ZK)2 + (KG)2

    84. (4) VA =

    (VW)2 + (WA)2

    =

    (3)2 + (4)2

    =

    (5)2 + (12)2 =

    =

    9 + 16

    =

    169 = 13 km

    =

    25 = 5 km.

    25 + 144

    ❑❑❑

    BPRE–271

    RANKING/ARRANGEMENT

    9

    RANKING/ARRANGEMENT

    QUESTIONS FROM 1999 TO 2010 ARE AVAILABLE ONLINE NATIONALISED BANKS & IBPS PO/MT/SO EXAMS 1. In a class of 20 students, Alisha‘s rank is 15th from the top. Manav is 4 ranks above Alisha. What is Manav‘s rank from the bottom? (1) 10th (2) 11th (3) 9th (4) 12th (5) None of these (Oriental Bank of Commerce PO Exam, 26.12.2011)

    Directions (2–4) : Study the following information carefully and answer the questions given below : (IDBI Bank Officer Exam, 22.08.2014)

    Among six persons – U, V, W, X, Y and Z & each one has different weight. Y is heavier than only two persons. U is lighter than only V and X. X is not the heaviest. Z is not the lightest. The person who is the second heaviest weighs 58 kg while the person who is the second lightest weighs 35 kg. 2. Who among the following is the heaviest of all ? (1) W (2) X (3) Z (4) V (5) Y 3. Who among the following may have a weight of 38 kg ? (1) U (2) Y (3) Z (4) W (5) U or Y 4. Who among them is heavier than only W ? (1) Z (2) Y (3) U (4) X (5) Cannot be determined. Directions (5–7) : Study the following information to answer the given questions : (Indian Bank PO Exam. 02.01.2011 (Ist sitting)

    L, M, N, O, P, Q and R are sitting around a circle facing the centre. O is sitting between L and R. Q is second to the right of R and P is second to the right of Q. N is not an immediate neighbour of R.

    5. Which of the following is not correct ? (1) R is second to the right of L (2) M is second to the left of N (3) L sits exactly between O and P (4) P and N are immediate neighbours (5) P sits to the opposite of N 6. How many persons are seated between L and Q if we count anticlockwise from L to Q ? (1) One (2) Two (3) Three (4) Four (5) More than four 7. Who is to the immediate left of P? (1) L (2) N (3) M (4) O (5) None of these Directions (8–9) : Four of the following five are alike in a certain way based on their seating positions in the above arrangement and so form a group. Which is the one that does not belong to the group ? (Indian Bank PO Exam. 02.01.2011 (Ist sitting)

    8. (1) QPN (2) MOR (3) LRO (4) RQM (5) ROL 9. (1) LP (2) RM (3) MQ (4) NP (5) OR Directions (10–14) : Study the following information to answer the given questions : (Indian Bank PO Exam. 02.01.2011 (Ist sitting)

    A, B, C, X, Y, Z are seated in a straight line facing North. C is third to the right of Z and B sits second to the right of C. X sits to the immediate right of A. 10. Which of the following represents the pairs of persons sitting exactly in the middle of the line ? (1) XB (2) ZB (3) BX (4) XC (5) XY

    BPRE–272

    11. What is X’s position with respect to Z ? (1) Immediate right of Z (2) Second to the left (3) Third to the right (4) Second to the right (5) None of these 12. Four of the following five are alike in a certain way based on their seating positions in the above arrangement and so form a group. Which is the one that does not belong to the group ? (1) ZA (2) XC (3) CY (4) YB (5) XA 13. How many persons are seated between A and C ? (1) One (2) Two (3) Three (4) Four (5) None of these 14. If A : X and Z : A, then Y : ? (1) Y (2) B (3) A (4) X (5) None of these Directions (15–20) : Study the following information to answer the given questions : (Indian Bank PO Exam. 02.01.2011 (Ist sitting)

    J, P, Q, R, S, T, U and V are four married couples sitting in a circle facing the centre, The profession of the males within the group are lecturer, lawyer, doctor and scientist. Among the males, only R (the lawyer) and V (the scientist) are sitting together. Each man is seated besides his wife. U, the wife of the lecturer is seated second to the right of V. T is seated between U and V. P is the wife of the doctor. Q is not the doctor. S is a male. 15. Which of the following is P’s position with respect to S ? (1) Second to the right (2) Second to the left (3) Immediate right (4) Immediate left (5) Third to the left 16. Which of the following is J’s position with respect to T ? (1) Third to the left (2) Fourth to the right

    RANKING/ARRANGEMENT

    17.

    18.

    19.

    20.

    (3) Third to the right (4) Opposite T (5) Second to the right Which of the following is not true regarding the couples ? (1) P is the wife of S (2) T is the wife of Q (3) R is the husband of J (4) J and S are seated adjacent to each other (5) All are true The wives of which two husbands are immediate neighbours ? (1) UT (2) SR (3) VQ (4) RV (5) None of these Four of the following are alike in a certain way based on their seating position in the above arrangement and so form a group. Which is the one that does not belong to the group ? (1) RSJ (2) TRV (3) UTV (4) SQP (5) UPQ Who is the Lawyer’s wife ‘? (1) T (2) P (3) J (4) U

    (5) None of these Directions (21–25) : Study the following information carefully and answer the given questions. (UCO Bank PO Exam. 30.01.2011)

    A, B, C, D, E, F, G and H are sitting around a circular table facing the centre. Each one of them has a different profession viz. doctor, engineer, architect, teacher, clerk, shopkeeper, businessman and banker. A sits third to right of teacher. D sits second to left of G. G is not an immediate neighbour of the teacher. Only one person sits between B who is the shopkeeper and teacher. The one who is an architect sits third to right of the shopkeeper. H sits between architect and engineer. E is not an immediate neighbour of H. Engineer sits third to the right of clerk. Only one person sits between businessman and F. E is neither a businessman nor a doctor. 21. Which of the following is true with respect to the given seating arrangement? (1) E is an immediate neighbour of the engineer (2) E is an architect (3) The clerk is an immediate neighbour of the banker

    (4) The teacher sits between H and the engineer (5) Shopkeeper sits second to the right of the teacher 22. What is the profession of H ? (1) Businessman (2) Architect (3) Banker (4) Teacher (5) Shopkeeper 23. What is the position of doctor with respect to the banker? (1) Immediately to the left (2) Third to the left (3) Second to the right (4) Fourth to the left (5) Second to the left 24. Who sit/s exactly between the architect and the businessman? (1) C and H (2) Clerk (3) Banker and shopkeeper (4) Doctor (5) C and teacher 25. Who amongst the following is a clerk? (1) C (2) D (3) E (4) F (5) G Directions (26–30) : Read the following information carefully and answer the questions which follow: (Bank Of Baroda PO Exam. 13.03.2011)

    A, B, C, D, E, F, G and H study in different standards viz. 1st, 2nd, 3rd, 4th, 5th, 6th, 7th and 8th. All of them are seated around a circular table facing the centre. C sits third to right of H. H studies in Std. 8. F sits second to left of E. E is not an immediate neighbour of C or H. The one who studies in Std. 1 is an immediate neighbour of E. Three people sit between H and the student of Std. 7. H, C and also their immediate neighbours do not study in Std. 2. Only one person sits between the student of Std. 2 and G. Students of Std. 3 and 4 are immediate neighbours of each other. C is neither in Std. 4 nor in Std. 3. Only one person sits between A and the student of Std 5. A does not study in Std. 2 or 3. B does not study in Std. 2. 26. Which of the following is true regarding F ? (1) F studies in Std. 5 (2) D and G are immediate neighbours of F (3) One person sits between F and the student of Std. 4 (4) F sits second to right of A (5) None is true

    BPRE–273

    27. Who amongst the following studies in Std. 3 ? (1) C (2) F (3) G (4) B (5) Cannot be determined 28. How many persons sit between D and the student of Std. 4, when counted from the left hand side of student of Std. 4 ? (1) None (2) One (3) Two (4) Three (5) Four 29. Which of the following is true? (1) Only one person is sitting between student of Std. 4 and Std.6 (2) D studies in Std.2 (3) The one studying in Std. 3 is an immediate neighbour of H (4) One person sits between H and the student of Std. 2 (5) None is true 30. Who amongst the following represent immediate neighbours of G? (1) A, B (2) E and the student of Std. 2 (3) H and the student of Std. 6 (4) E, F (5) A, D Directions (31–36) : Study the following information carefully and answer the given questions. (Allahabad Bank PO Exam. 17.04.2011)

    P, Q, R, S, T, V, W and X are captains of eight different Cricket teams namely Australia, New Zealand, India, Pakistan, Sri-Lanka, England, West Indies and South Africa, but not necessarily in the same order. All of them are seated around a circular table and are facing the centre. P is sitting third to the left of the Sri Lankan captain. Only two people sit between T and W. Neither T nor W is an immediate neighbour of P. Neither T nor W is the captain of Sri Lanka. The captain of South Africa sits second to the right of S. S is not an immediate neighbour of P. S is not Sri Lankan captain and P is not the captain of South Africa. The Australian captain sits third to left of V. Australian and Sri Lankan captains are not immediate neighbours of each other. Only one person sits between S and the Indian captain. Captains of Pakistan and New Zealand are immediate neighbours of each other. S is not the captain of New Zealand’s team. Only

    RANKING/ARRANGEMENT one person sits between Q and the captain of England. The captain of England is immediate neighbour of X. W and Q are not immediate neighbours of each other. 31. How many people sit between T and the captain of England when counted in clockwise direction from T ? (1) None (2) One (3) Two (4) Four (5) Five 32. Who amongst the following is the captain of the Australian team? (1) P (2) V (3) W (4) T (5) Q 33. W61hich of the following would come in the place of the question mark based upon the given seating arrangement ? VS XR TV RP ? (1) SW (2) WX (3) QW (4) QX (5) VR 34. Which of the following is true with respect to the given seating arrangement ? (1) R is the captain of South Africa (2) W is an immediate neighbour of V. (3) Captains of Australia and England are immediate neighbours of each other. (4) Four people sit between W and Q (5) X sits second to left of S. 35. Who amongst the following is the Indian captain? (1) Q (2) V (3) X (4) T (5) Cannot be determined 36. What is the position of the captain of West Indies with respect to R ? (1) Immediate left (2) Second to the left (3) Third to the right (4) Second to the right (5) Third to the left Directions (37-43) : Study the following information carefully and answer the given questions : (Indian Overseas Bank PO Exam. 22.05.2011)

    A, B, C, D, E, F, G and H are sitting around a circular table facing the centre. Each one of them has a different profession viz. doctor, engi-

    neer, architect, pilot, banker, teacher, businessman and politician. The politician sits third to right of G. C is an immediate neighbour of G. Architect sits second to right of C. B sits third to right of H. H is neither a politician nor an architect. Only one person sits between C and the teacher. A and F are immediate neighbours of each other. Neither A nor F is a politician. Doctor sits second to right of A. Two people sit between D and the engineer. D is not a politician. Pilot is not an immediate neighbour of the politician. Banker sits second to left of A. 37. Who amongst the following is a businessman ? (1) A (2) H (3) C (4) F (5) D 38. What is the position of F with respect to the politician ? (1) Immediate right (2) Third to the left (3) Second to the right (4) Fourth to the left (5) Second to the left 39. Who sit(s) exactly between the teacher and the engineer ? (1) C and H (2) Only the politician (3) Only the doctor (4) C and B (5) The architect and the banker 40. Which of the following is true with respect to the given seating arrangement ? (1) D is an immediate neighbour of G (2) G is a banker (3) The banker and the teacher are immediate neighbours of each other (4) Pilot sits exactly between architect and the businessman (5) Doctor sits second to the right of the businessman 41. What is the profession of G ? (1) Businessman (2) Pilot (3) Banker (4) Teacher (5) Architect 42. Four of the following five are alike in a certain way based on the given seating arrangement and thus form a group. Which is the one that does not belong to that group? (1) G - Doctor (2) E -Architect (3) H - Businessman

    BPRE–274

    (4) E – Politician (5) D - Pilot 43. What is the profession of E ? (1) Businessman (2) Architect (3) Banker (4) Politician (5) Engineer Directions (44–48) : Study the following information carefully and answer the given questions. (IBPS Bank PO/MT CWE Exam. 18.09.2011)

    Representatives from eight different Banks viz. A, B, C, D, E, F, G and H are sitting around a circular table facing the centre but not necessarily in the same order. Each one of them is from a different Bank viz. UCO Bank, Oriental Bank of Commerce, Bank of Maharashtra, Canara Bank, Syndicate Bank, Punjab National Bank, Bank of India and Dena Bank. F sits second to right of the representative from Canara Bank. Representative from Bank of India is an immediate neighbour of the representative from Canara Bank. Two people sit between the representative of Bank of India and B. C and E are immediate neighbours of each other. Neither C nor E is an immediate neighbour of either B or the representative from Canara Bank. Representative from Bank of Maharashtra sits second to right of D. D is neither the representative of Canara Bank nor Bank of India. G and the representative from UCO Bank are immediate neighbours of each other. B is not the representative of UCO Bank. Only one person sits between C and the representative from Oriental Bank of Commerce. H sits third to left of the representative from Dena Bank. Representative from Punjab National Bank sits second to left of the representative from Syndicate Bank. 44. Four of the following five are alike in a certain way based on the given arrangement and thus form a group. Which is the one that does not belong to that group? (1) H - UCO Bank (2) A - Canara Bank (3) D - Bank of Maharashtra (4) E - Syndicate Bank (5) F - Punjab National Bank 45. Which of the following is true with respect to the given seating arrangement? (1) B is the representative from Bank of Maharashtra

    RANKING/ARRANGEMENT (2) C sits second to right of H (3) The representative from Dena Bank sits to the immediate left of the representative from UCO Bank (4) A sits second to right of the representative from Bank of India (5) The representatives from Bank of Maharashtra and Syndicate Bank are immediate neighbours of each other 46. Who amongst the following sit exactly between B and the representative from Bank of India? (1) A and the representative from UCO Bank (2) F and G (3) H and the representative from Bank of Maharashtra (4) H and G (5) Representatives from Syndicate Bank and Oriental Bank of Commerce 47. Who amongst the following is the representative from Oriental Bank of Commerce? (1) A (2) C (3) H (4) G (5) D 48. Who amongst the following sits second to left of B ? (1) C (2) H (3) The representative from Canara Bank (4) The representative from Punjab National Bank (5) G Directions (49–53) : Study the following information carefully and answer the given questions : (Corporation Bank PO Exam. 16.01.2011)

    Eight friends P, Q, R, S, T, V, W and Y are sitting around a square table in such a way that four of them sit at four corners of the square while four sit in the middle of each of the four sides. The ones who sit at the four corners face the centre while those who sit in the middle of the sides face outside. P who faces the centre sits third to the right of V. T, who faces the centre, is not an immediate neighbour of V. Only one person sits between V and, W. S sits second to right of Q. Q faces the centre. R is not an immediate neighbour of P.

    Who sits second to the left of Q ? (1) V (2) P (3) T (4) Y (5) Cannot be determined 50. What is the position of T with respect to V ? (1) Fourth to the left (2) Second to the left (3) Third to the left (4) Third to the right (5) Second to the right 51. Four of the following five are alike in a certain way and so form a group. Which is the one that does not belong to that group ? (1) R (2) W (3) V (4) S (5) Y 52. Which of the following will come in place of the question mark based upon the given seating arrangement? WP TR QW RS ? (1) YT (2) VY (3) VQ (4) PY (5) QV 53. Which of the following is true regarding R ? (1) R is an immediate neighbour of V (2) R faces the centre (3) R sits exactly between T and S (4) Q sits third to left of R (5) None is true Directions (54–60) : Study the following information to answer the given questions : 49.

    (Corporation Bank PO Exam. 16.01.2011)

    Twelve people are sitting in two parallel rows containing six people each, in such a way that there is an equal distance between adjacent persons. In row-1, P, Q, R, S, T and V are seated and all of them are facing south. In row-2, A, B, C, D, E and F are seated and all of them are facing north. Therefore, in the given seating arrangement each member seated in a row faces another member of the other row. A sits third to right of D. Neither A nor D sits at extreme ends. T faces D. V does not face A and V does not sit at any of the extreme ends. V is not an immediate neighbour of T. B sits at one of the extreme ends. Only two people sit between B and E. E does not face V. Two persons sit between R and Q. R is not an immediate neighbour of T. C does not face V. P is not an immediate neighbour of R.

    BPRE–275

    Who amongst the following sit at extreme ends of the rows ? (1) B, E (2) S, T (3) P, R (4) B, F (5) None of these 55. Who amongst the following faces A? (1) R (2) T (3) P (4) Q (5) S 56. How many persons are seated between T and S ? (1) One (2) Two (3) Three (4) Four (5) None 57. P is related to V in the same way as C is related to F. To which of the following is E related to, following the same pattern ? (1) B (2) D (3) C (4) A (5) None of these 58. Which of the following is true regarding F ? (1) F sits second to right of C (2) F is not an immediate neighbour of A (3) F sits third to left of D (4) F sits at one of the extreme ends of the line (5) F faces V 59. Who amongst the following sits exactly between P and Q ? (1) R (2) V (3) S (4) T (5) Cannot be determined 60. Q types faster than R but not as fast as V. T types faster than R. S types faster than V. Who amongst the five of them types the fastest ? (1) V (2) T (3) S (4) Data Inadequate (5) None of these 54.

    (Punjab & Sind Bank PO Exam. 23.01.2011)

    Directions (61-65) : Study the following information carefully and answer the questions given below : (Punjab & Sind Bank PO Exam. 23.01.2011)

    Eight persons A, B, C, D, E, F, G and H are sitting around a rectangular table in such a way that two persons sit on each of the four sides of the table facing the centre. Persons sitting on opposite sides are exactly opposite to each other.

    RANKING/ARRANGEMENT D faces North and sits exactly opposite to H. E is to the immediate left of H. A and G sit on the same side. G is exactly opposite of B who is to the immediate right of C. A is next to the left of D. 61. Who is sitting opposite to A ? (1) G (2) D (3) E (4) A (5) None of these 62. Who is next to E in clockwise direction? (1) G (2) B (3) F (4) A or F (5) None of these 63. Which of the following pairs of persons has both the persons sitting on the same side with first person sitting to the right of second person ? (1) DF (2) CB (3) FC (4) AG (5) None of these 64. Who is sitting opposite to E ? (1) D (2) A (3) F (4) A or D (5) None of these 65. Which of the following statements is definitely true? (1) A is facing North (2) E is sitting opposite to F (3) F is to the left of G (4) C is to the left of A (5) None of these Directions (66-70) : Study the following information to answer the given questions: (UCO Bank PO Exam. 30.01.2011)

    Twelve people are sitting in two parallel rows containing six people each, in such a way that there is an equal distance between adjacent persons. In row-1 P, Q, R, S, T and V are seated and all of them are facing South. In row-2 A, B, C, D, E and F are seated and all of them are facing North. Therefore, in the given seating arrangement each member seated in a row faces another member of the other row. P sits third to the left of T. Neither P nor T sits at an extreme end of the line. A sits second to the right of E. Neither A nor E faces T or P. A does not sit at an extreme end. R does not face A and R does not sit at an extreme end of the line. Only one person sits between F and C. Neither F nor C faces T. C does not sit at the extreme end. Only one person sits between V and Q. F is not an immediate neighbour of B and A does not face V.

    66. How many persons are seated between B and D ? (1) One (2) Two (3) Three (4) Four (5) None 67. V is related to B in the same way as Q is related to C. To which of the following is P related to, following the same pattern ? (1) F (2) A (3) D (4) E (5) None of these 68. Which of the following is true regarding V ? (1) V faces A (2) T is not an immediate neighbour of V. (3) C faces the one who is second to left of V (4) V sits at one of the extreme ends of the line (5) R sits third to the right of V 69. Who amongst the following sit at extreme ends of the rows? (1) D, F (2) V, S (3) Q, S (4) B, D (5) None of these 70. Who amongst the following faces B? (1) V (2) S (3) Q (4) R (5) T Directions (71–76) : Study the following information to answer the given questions : (Bank Of Baroda PO Exam. 13.03.2011)

    Twelve people are sitting in two parallel rows containing six people each, in such a way that there is an equal distance between adjacent persons. In row – 1 A, B, C, D, E and F are seated and all of them are facing south. In row - 2 P, Q, R, S, T and V are seated and all of them are facing north. Therefore, in the given seating arrangement each member seated in a row faces another member of the other row. V sits third to right of S. S faces F and F does not sit at any of the extreme ends of the line. D sits third to right of C. R faces C. The one facing E sits third to right of P. B and P do not sit at the extreme ends of the line. T is not an immediate neighbour of V and A is not an immediate neighbour of C. 71. Who amongst the following faces D? (1) T (2) P (3) Q (4) R (5) None of these

    BPRE–276

    72. Who amongst the following represent the people sitting at extreme ends of the rows? (1) R, F (2) T, A (3) D, R (4) C, Q (5) S, A 73. Four of the following five are alike in a certain way and thus form a group. Which is the one that does not belong to that group? (1) B – T (2) A – Q (3) C – S (4) F – P (5) D – R 74. Four of the following five are alike in a certain way and thus form a group. Which is the one that does not belong to that group ? (1) D (2) S (3) V (4) T (5) A 75. How many persons are seated between R and T ? (1) One (2) Two (3) Three (4) Four (5) None 76. Who amongst the following faces Q? (1) D (2) B (3) E (4) A (5) None of these 77. Which of the following is true regarding B ? (1) B sits to the immediate left of C (2) B faces Q (3) B sits fourth from the extreme left end of the line (4) D and F are immediate neighbours of B (5) None is true Directions (78–79) : Study the following information carefully and answer the given questions. (Allahabad Bank PO Exam. 17.04.2011)

    Amongst five friends, each got a different percentage of marks in the examination. Poonam scored more than Ben but less than Ajay. Ajay scored 70% marks. Shreya scored less marks only than Kim. The one who scored the minimum marks scored 65% marks and the one who scored the highest, scored 87% marks. 78. Who amongst the following scored the second lowest marks? (1) Ben (2) Kim (3) Shreya (4) Ajay (5) Poonam

    RANKING/ARRANGEMENT 79. Who amongst the following is the most likely to have scored 82% marks ? (1) Ben (2) Poonam (3) Shreya (4) Kim (5) Either Kim or Ben Directions (80–85) : Study the following information to answer the given questions : (Allahabad Bank PO Exam. 17.04.2011)

    Ten people are sitting in two parallel rows containing five people each, in such a way that there is an equal distance between adjacent persons. In row — 1 P, Q, R, S and T are seated and all of them are facing South. In row — 2 A, B, C, D and E are seated and all of them are facing North. Therefore, in the given seating arrangement each member seated in a row faces another member of the other row. D sits third to the left of A. P faces immediate neighbour of D. R sits second to the right of P. Only one person sits between Q and S. B and E are immediate neighbours of each other. E does not face P and Q. 80. How many persons are seated between Q and T ? (1) None (2) One (3) Two (4) Three (5) Cannot be determined 81. Four of the following five are alike in a certain way and thus form a group. Which is the one that does not belong to that group ? (1) R (2) S (3) C (4) T (5) A 82. Who amongst the following represent the people sitting exactly in the middle of the rows ? (1) P, E (2) S, D (3) S, A (4) A, R (5) P, B 83. Which of the following is true regarding B ? (1) A and C are immediate neighbours of B (2) B sits at one of the extreme ends of the line (3) Q faces B (4) T is an immediate neighbour of the person facing B (5) D sits to the immediate left of B 84. Four of the following five are alike in a certain way and thus-form a group. Which is the one that does not belong to that group?

    (1) T – E (2) Q – C (3) S – B (4) R – A (5) P – D 85. Who amongst the following faces S? (1) A (2) B (3) C (4) D (5) E Directions (86-92) : Study the following information carefully and answer the given questions. (Allahabad Bank PO Exam. 17.04.2011)

    Eight friends, Meenal, Rumia, Shikha, Ali, Peter, Harleen, Ketan and Bharat are sitting around a square table in such a way that four of them sit at four corners of the square while four sit in the middle of each of the four sides. The ones who sit at the four corners face the centre while those who sit in the middle of the sides face outside. Bharat sits second to the right of Shikha. Bharat does not sit at any of the corners. Meenal sits third to the right of Peter. Peter is not an immediate neighbour of Shikha. Rumia and Ketan are immediate neighbours of each other but Rumia does not sit at any of the corners of the table. Harleen is neither an immediate neighbour of Peter nor Shikha. 86. Four of the following five are alike in a certain way and so form a group. Which is the one that does not belong to that group ? (1) Peter (2) Rumia (3) Harleen (4) Shikha (5) Bharat 87. Who sits third to the left of Ali ? (1) Bharat (2) Rumia (3) Shikha (4) Peter (5) Cannot be determined 88. What is the position of Peter with respect to Meenal ? (1) To immediate left (2) Second to the left (3) Third to the left (4) Third to the right (5) Second to the right 89. Who amongst the following sits second to the right of Ketan ? (1) Shikha (2) Ali (3) Bharat (4) Harleen (5) Meenal 90. Who amongst the following represent the immediate neighbours of Harleen ? (1) Meenal, Ketan (2) Bharat, Rumia

    BPRE–277

    (3) Bharat, Meenal (4) Ali, Rumia (5) Ali, Ketan 91. Who amongst the following sit/s exactly between Peter and Ali ? (1) Only Bharat (2) Ketan and Rumia (3) Only Harleen (4) Harleen and Meenal (5) No one sits between Peter and Ali 92. Who amongst the following is an immediate neighbour of Meenal? (1) Rumia (2) Ali (3) Ketan (4) Harleen (5) Shikha Directions (93–99) : Study the following information to answer the given questions : (Indian Overseas Bank PO Exam. 22.05.2011)

    P, Q, R, S, T, V and W are sitting in a straight line facing north. Each one of them lives on a different floor in the same building which is numbered from one to seven. Q sits fourth to the left of the person living on the 6th floor. Either Q or the person living on the 6th floor sits at the extreme ends of the line. Only one person sits between Q and W. W lives on the 3rd floor. The person living on 1st floor sits third to right of S. S is not an immediate neighbour of W. Only one person lives between T and the person who lives on the 2nd floor. P and R are immediate neighbours of each other. P does not live on the 6th floor. One who lives on 5th floor sits third to right of the one who lives on the 7th floor. 93. Who amongst the following lives on the 4th floor? (1) P (2) Q (3) R (4) S (5) V 94. On which of the following floors does T live? (1) 1 st (2) 2nd (3) 5th (4) 6th (5) 7th 95. How many floors are threre between the floors on which V and P live ? (1) One (2) Two (3) Three (4) Four (5) None

    RANKING/ARRANGEMENT Directions (96-99) : Four of the following five are alike in a certain way based on the given arrangement and thus form a group. Which is the one that does not belong to that group? (Indian Overseas Bank PO Exam. 22.05.2011)

    96. (1) W (2) T (3) S (4) P (5) Q 97. (1) T - 2nd floor (2) R - 7th floor (3) V - 3rd floor (4) S - 5th floor (5) Q - 6th floor 98. Which of the following is true with respect to the given arrangement? (1) The one who lives on the 5th floor is an immediate neighbour of S (2) V lives on the 1 st floor (3) T sits second to the left of the person who lives on 2nd floor (4) R and V are immediate neighbours of each other (5) The one who lives on the 4th floor sits at one of the extreme ends of the line 99. If all the persons are made to live in alphabetical order from the bottom-most floor to the top-most floor (i .e. P lives on the 1st floor, Q lives on the 2nd floor and finally W lives on the 7th floor) who would still live on the same floor as the original arrangement? (1) R (2) V (3) W (4) T (5) S Directions (100–102) : Study the following the information carefully and answer the questions given below : (Bank of Baroda Junior Management Grade/Scale-I Exam, 18.04.2015)

    In a class, Sameer’s rank is 10th from the top and Anjali’s rank is 4th from the top. Among girls, Anjali’s rank is 3rd from the top and 7th from the bottom. Among boys, Sameer’s rank is 5th from the top and 21st from the bottom. 100. What is the total number of students in that class ? (1) 43 (2) 30 (3) 34 (4) 39 (5) 36 101. How many boys and girls are there in between Sameer and Anjali ? (1) Four boys, two girls (2) Two boys, four girls (3) Three boys, two girls

    (4) Two boys, two girls (5) Two boys, three girls 102. The positions of the first and the fifth digits of the number 81943275 are interchanged. Similarly the positions of the second and the sixth digits are interchanged and so on till the fourth and the eighth digits. Which of the following will be the third digit from the right end after the rearrangement? (1) 1 (2) 9 (3) 2 (4) 4 (5) None of these (IBPS Specialist Officer CWE Exam. 11.03.2012)

    Directions (103 – 107) : Study the following arrangement carefully and answer the questions given below : (Corporation Bank PO Exam. 16.01.2011)

    4261389581651585 39492321572624274 103. If all the even digits are deleted from the above arrangement, which of the following will be seventh from the right end of the arrangement ? (1) 3 (2) 5 (3) 1 (4) 9 (5) 7 104. How many such 2s are there in the above arrangement each of which is immediately preceded by an odd digit and also immediately followed by an odd digit ? (1) None (2) One (3) Two (4) Three (5) More than three 105. How many such 1s are there in the above arrangement, each of which is immediately preceded by a perfect square ? (1) None (2) One (3) Two (4) Three (5) More than three 106. If one is added to each of the even digits and two is added to each of the odd digits given in BOLD in the above arrangement, how many digits will appear twice in the new number thus formed ? (1) None (2) One (3) Two (4) Three (5) More than three 107. Which of the following is seventh to the right of the nineteenth digit from the right end of the above arrangement ?

    BPRE–278

    (1) 3 (2) 5 (3) 1 (4) 7 (5) None of these Directions (108) : The following questions are based on the five three digit numbers given below : (Punjab & Sind Bank PO Exam. 23.01.2011)

    394 632 783 576 895 108. If the positions of the first and the second digits within each number are interchanged, which of the following will be the second highest number ? (1) 632 (2) 783 (3) 576 (4) 895 (5) 394 109. The positions of the first and the fifth digits of the number 81943275 are interchanged. Similarly the positions of the second and the sixth digits are interchanged and so on till the fourth and the eighth digits. Which of the following will be the third digit from the right end after the rearrangement? (1) 1 (2) 9 (3) 2 (4) 4 (5) None of these (IBPS Specialist Office CWE Exam. 11.03.2012)

    Directions (110 – 114) : Study the following arrangement carefully and answer the questions given below : (IBPS Specialist Officer CWE Exam. 11.03.2012)

    M3#RAT1EJ$K@FU5 4VI6©D8★HN7δ2W 110. Four of the following five are alike in a certain way based on their positions in the above arrangement and so form a group. Which is the one that does not belong to that group? (1) TER (2) U4@ (3) 6DV (4) J 1 K (5) Nδ ★ 111. How many such numbers are there in the above arrangement, each of which is immediately preceded by a letter but not immediately followed by a letter? (1) None (2) One (3) Two (4) Three (5) More than three 112. What should come in place of the question mark (?) in the following series based on the above arrangement?

    RANKING/ARRANGEMENT RT1 $@F 4I6 ? (1) 8 ★ H (2) 8 H N (3) 8 H 7 (4) D ★ H (5) None of these 113. How many such symbols are there in the above arrangement, each of which is immediately preceded by a number and immediately followed by a letter? (1) None (2) One (3) Two (4) Three (5) More than three 114. Which of the following is the seventh to the left of the twentieth from the left end of the above arrangement ? (1) V (2) @ (3) δ (4) © (5) None of these Directions (115–121) : Study the following information carefully and answer the given questions : (IBPS Bank PO/MT CWE Exam. 17.06.2012)

    A, B, C, D, E, F, G and H are sitting around a circle facing the centre but not necessarily in the same order. ● B sits second to left of H’s husband. No female is an immediate neighbour of B. ● D’s daughter sits second to right of F. F is the sister of G. F is not an immediate neighbour of H’s husband. ● Only one person sits between A and F. A is the father of G. H’s brother D sits to the immediate left of H’s mother. Only one person sits between H’s mother and E. ● Only one person sits between H and G. G is the mother of C. G is not an immediate neighbour of E. 115. What is position of A with respect to his mother-in-law? (1) Immediate left (2) Third to the right (3) Third to the left (4) Second to the right (5) Fourth to the left 116. Who amongst the following is D’s daughter ? (1) B (2) C (3) E (4) G (5) H 117. What is the position of A with respect to his grandchild ? (1) Immediate right

    (2) Third to the right (3) Third to the left (4) Second to the left (5) Fourth to the left 118. How many people sit between G and her uncle? (1) One (2) Two (3) Three (4) Four (5) More than four 119. Four of the following five are alike in a certain way based on the given information and so form a group. Which is the one that does not belong to that group? (1) F (2) C (3) E (4) H (5) G 120. Which of the following is true with respect to the given seating arrangement ? (1) C is the cousin of E (2) H and H’s husband are immediate neighbours of each other (3) No female is an immediate neighbour of C (4) H sits third to left of her daughter (5) B is the mother of H 121. Who sits to the immediate left of C? (1) F’s grandmother (2) G’s son (3) D’s mother-in-law (4) A (5) G Directions (122–124) : Study the following information carefully to answer the given questions : (IBPS Bank PO/MT CWE Exam. 17.06.2012)

    Each of the six friends, A, B, C, D, E and F scored different marks in an examination. C scored more than only A and E. D scored less than only B. E did not score the least. The one who scored the third highest marks scored 81 marks. E scored 62 marks. 122. Which of the following could possibly be C’s score ? (1) 70 (2) 94 (3) 86 (4) 61 (5) 81 123. Which of the following is true with respect to the given information ? (1) D’s score was definitely less than 60 (2) F scored the maximum marks (3) Only two people scored more than C

    BPRE–279

    (4) There is a possibility that B scored 79 marks (5) None is true 124. The person who scored the maximum, scored 13 marks more than F’s marks. Which of the following can be D’s score ? (1) 94 (2) 60 (3) 89 (4) 78 (5) 81 Directions (125–132) : Study the following information carefully to answer the given questions : (IBPS Bank PO/MT CWE Exam. 17.06.2012)

    Eight persons from different Banks viz. UCO bank, Syndicate Bank, Canara Bank, PNB, Dena Bank, Oriental Bank of Commerce, Indian Bank and Bank of Maharashtra are sitting in two parallel rows containing four people each, in such a way that there is an equal distance between adjacent persons. In row-1 A, B, C and D are seated and all of them are facing south. In row-2 P, Q, R and S are seated and all of them are facing north. Therefore, in the given seating arrangement each member seated in a row faces another member of the other row. (All the information given above does not necessarily represent the order of seating as in the final arrangement) ● C sits second to right of the person from Bank of Maharashtra. R is an immediate neighbour of the person who faces the person from Bank of Maharashtra. ● Only one person sits between R and the person from PNB. Immediate neighbour of the person from PNB faces the person from Canara Bank. ● The person from UCO bank faces the person from Oriental Bank of Commerce. R is not from Oriental Bank of Commerce. P is not from PNB. P does not face the person from Bank of Maharashtra. ● Q faces the person from Dena Bank. The one who faces S sits to the immediate left of A. ● B does not sit at any of the extreme ends of the line. The person from Bank of Maharashtra does not face the person from Syndicate Bank. 125. Which of the following is true regarding A ? (1) The person from UCO Bank faces A

    RANKING/ARRANGEMENT (2) The person from Bank of Maharashtra is an immediate neighbour of A (3) A faces the person who sits second to right of R (4) A is from Oriental Bank of Commerce (5) A sits at one of the extreme ends of the line 126. Who is seated between R and the person from PNB ? (1) The person from Oriental Bank of Commerce (2) P (3) Q (4) The person from Syndicate Bank (5) S 127. Who amongst the following sit at extreme ends of the rows ? (1) D and the person from PNB (2) The person from Indian Bank and UCO Bank (3) The person from Dena Bank and P (4) The person from Syndicate Bank and D (5) C, Q 128. Who amongst the following faces the person from Bank of Maharashtra ? (1) The person from Indian Bank (2) P (3) R (4) The person from Syndicate Bank (5) The person from Canara Bank 129. P is related to Dena Bank in the same way as B is related to PNB based on the given arrangement. To who amongst the following is D related to, following the same pattern ? (1) Syndicate Bank (2) Canara Bank (3) Bank of Maharashtra (4) Indian Bank (5) Oriental Bank of Commerce 130. Four of the following five are alike in a certain way based on the given seating arrangement and thus form a group. Which is the one that does not belong to that group ? (1) Canara Bank (2) R (3) Syndicate Bank (4) Q (5) Oriental Bank of Commerce 131. Who amongst the following is from Syndicate Bank ?

    (1) C (2) R (3) P (4) D (5) A 132. C is from which of the following banks ? (1) Dena Bank (2) Oriental Bank of Commerce (3) UCO Bank (4) Syndicate Bank (5) Canara Bank Directions (133–136) : Study the given information carefully and answer the given questions : (IBPS RRBs Office Assistant CWE Exam. 09.09.2012)

    In a group of six people, P, Q, R, S, T and U, each having a different weight, S is heavier than Q. R is lighter than only T and P. Q is not the lightest. P is not the heaviest. 133. Who amongst the following is the lightest ? (1) T (2) P (3) R (4) U (5) None of these 134. Who amongst the following is the second heaviest ? (1) R (2) S (3) U (4) T (5) P 135. How many people are lighter than S? (1) None (2) Three (3) Two (4) More than three (5) One 136. Fifteen girls are standing in a straight fine facing North. Sudha is standing eleventh from the right end. Radha is standing ninth from the left end. Meena is standing exactly between Sudha and Radha. How many girls are standing to the right of Meena ? (1) Eight (2) Seven (3) Nine (4) Six (5) Cannot be determined (IBPS RRBs Office Assistant CWE Exam. 09.09.2012)

    Directions (137-141) : Study the given information carefully and answer the given questions : (IBPS RRBs Office Assistant CWE Exam. 09.09.2012)

    Eight people P, Q, R, S, T, U, V and W are sitting around a circular table facing the centre not necessarily in the same order. T is sitting third to the right of P. W is sitting second to

    BPRE–280

    the right of S. S is not an immediate neighbour of either P or T. U and Q are immediate neighbours of each other. Q is not an immediate neighbour of P. V is not an immediate neighbour of W. 137. What is R’s position with respect to V ? (1) Second to the left (2) Second to the right (3) Third to the right (4) Third to the left (5) None of these 138. Four of the following five are alike in a certain way based on their seating positions in the above arrangement and so form a group. Which one does not belong to the group ? (1) RQ (2) PV (3) TP (4) US (5) WT 139. Who is sitting second to the right of the one who is sitting to the immediate right of W ? (1) U (2) R (3) V (4) P (5) Q 140. How many people are sitting between U and W when counted from the left side of U ? (1) Three (2) None (3) More than three (4) Two (5) One 141. Who is sitting exactly between T and S ? (1) U (2) Q (3) W (4) V (5) R Directions (142-146) : Study the given information carefully and answer the given questions : (IBPS RRBs Office Assistant CWE Exam. 09.09.2012)

    Eight people – A, B, C, D, E, F, G and H are sitting in a straight line facing North not necessarily in the same order. F is sitting fifth to the right of A. D is sitting fifth to the right of E. E is to the immediate right of A. H is third to the left of D. B is third to the right of G. G is an immediate neighbour of A. 142. What is E’s position with respect to C ? (1) Second to the left (2) Third to the right (3) Fourth to the right

    RANKING/ARRANGEMENT

    143.

    144.

    145.

    146.

    147.

    (4) Third to the left (5) None of these Who is sitting third to the right of the one who is sitting sixth from the right end of the above arrangement ? (1) D (2) C (3) E (4) H (5) None of these Who amongst the following are sitting at the extreme ends of the line ? (1) G and D (2) A and D (3) G and F (4) E and D (5) A and B Four of the following five are alike in a certain way based on their seating positions in the above arrangement and so form a group. Which one does not belong to the group ? (1) HD (2) BE (3) EC (4) AH (5) GB If all eight persons are asked to sit in an alphabetical order from right to left, the positions of how many will remain unchanged as compared to their original seating position ? (1) Three (2) More than three (3) One (4) Two (5) None Consider five people A, B, C, D and E each having different age. A is younger than only B. C is older than D. D is not the youngest. Who amongst the following are older than C ? (1) Only A and B (2) Only E, B and A (3) Only A and E (4) Only E and B (5) None of these (IDBI Bank Officer Exam.16.09.2012)

    148. In a row of children facing north, A stand 15th from left and B stands 9th from left. C who stands exactly between A and B is 7th from right. What is the total number of children standing in the row ? (1) 21 (2) 17 (3) 18 (4) 19 (4) None of these (IDBI Bank Officer Exam.16.09.2012)

    Directions (149-153) : Study the following information carefully and answer the given questions. (IDBI Bank Officer Exam.16.09.2012)

    Seven people P, Q, R, S, T, W and X are sitting in a straight line facing North, not necessarily in the same order. R sits at one of the extreme ends of the line. T has as many people sitting on his right, as to his left. S sits third to the left of X. Q sits to the immediate left of W. Q does not sit at any of the extreme ends of the line. 149. If all the people are made to sit in alphabetical order from right to left, the positions of how many people will remain unchanged ? (1) Five (2) One (3) Three (4) None (5) Two 150. How many people sit to the right of P ? (1) Four (2) Five (3) Two (4) One (5) None 151. Four of the following five are alike in a certain way, based on the information given above and so form a group. Which is the one that does not belong to that group ? (1) S (2) P (3) X (4) Q (5) W 152. Who amongst the following are immediate neighbours of S ? (1) T and R (2) Q and P (3) W and R (4) R and P (5) None of these 153. Who amongst the following sits second to the right of fourth person from the right end of the line ? (1) W (2) Q (3) R (4) P (5) None of these Directions (154-158) : Study the following information carefully and answer the given questions :

    person sits between A and H. C and G are immediate neighbours of each other. Neither C nor G is an immediate neighbour of B. • Only one person sits between C and D. 154. Who amongst the following is an immediate neighbour of both A and H ? (1) F (2) B (3) G (4) D (5) C 155. F is related to D in a certain way based on the seating positions in the given arrangement. Similarly C is related to E in the same way. To whom amongst the following is H related to following the same pattern ? (1) G (2) A (3) F (4) C (5) B 156. Which of the following represents the correct position of A? (1) Second to the left of H (2) Immediate left of C (3) Exactly between F and E (4) Second to the right of E (5) Third to right of D 157. What is the position of D with respect to the position of G ? (1) Third to the left (2) Second of the right (3) Immediate right (4) Fourth to the left (5) Second of the left 158. Which of the following is true with respect to given seating arrangement ? (1) Only two people sit between D and B (2) A is an immediate neighbour of B (3) C sits third to left of H (4) Only one person sits between H and D (5) None is true Directions (159– 160) : Study the following the information carefully and answer the questions given below : •

    (IDBI Bank Officer Exam.16.09.2012)

    (BOB Junior Management Grade/Scale–I Exam. 18.04.2015)

    Eight friends A, B, C, D, E, F, G and H are sitting around a circle (not necessarily in the same order) facing the centre. • B sits third to left of F. • E is an immediate neighbour of both B and H. Only one

    In a class, Sameer’s rank is 10th from the top and Anjali’s rank is 4th from the top. Among girls, Anjali’s rank is 3rd from the top and 7th from the bottom. Among boys, Sameer’s rank is 5th from the top and 21st from the bottom.

    BPRE–281

    RANKING/ARRANGEMENT 159. What is the total number of students in that class ? (1) 43 (2) 30 (3) 34 (4) 39 (5) 36 160. How many boys and girls are there in between Sameer and Anjali ? (1) Four boys, two girls (2) Two boys, four girls (3) Three boys, two girls (4) Two boys, two girls (5) Two boys, three girls 161. In a class of 40 children, Sunetra’s rank is eighth from the top. Sujit is five ranks below Sunetra. What is Sujit’s rank from the bottom? (1) 27 (2) 28 (3) 29 (4) 26 (5) Other than those given as options 162. How many persons are standing in a straight line (Note : All are facing north) ? I. J stands third from the right end of the line. Q stands to the immediate left of J. Only two persons stand between Q and T. No person stands to the left of T. II. R stands third from the left end of the line. Q is one of the immediate neighbours of R. Only two persons stand between Q and S. (1) The data either in statement I alone or statement II alone are sufficient to answer the question. (2) The data in both statements I and II are necessary to answer the question. (3) The data in statement I alone are sufficient to answer the question while the data in statement II alone are not sufficient to answer the question. (4) The data in statement II alone are sufficient to answer the question while the data in statement I alone are not sufficient to answer the question. (5) The data in both statements I and II together are not sufficient to answer the question.

    Directions (163-165) : Study the following information to answer the given questions : (IDBI Bank Officer Exam.16.09.2012)

    Each of the five friends, A, B, C, D and E scored different marks in an examination. Maximum marks of the examination are 100. B scored more than E but less than C. Only two people scored more than A. E did not score the minimum marks. The one who scored second highest marks scored 87%. E scored 72% marks. 163. How many people scored more than E in the examination ? (1) None (2) One (3) Two (4) Three (5) Cannot be determined 164. Who amongst the following possibly scored 81% marks ? (1) C (2) D (3) B (4) A (5) Either C or B 165. Which of the following percentages may correctly represent C’s percentage in the exam ? (1) 65% (2) 74% (3) 87% (4) 83% (5) 89% Directions (166-170) : Study the following information to answer the given questions : (IDBI Bank Officer Exam.16.09.2012)

    Eight people are sitting in two parallel rows containing four people each, in such a way that there is an equal distance between adjacent persons. In row-1, A, B, C and D are seated (but not necessarily in the same order) and all of them are facing South. In row-2, P, Q, R and S are seated (but not necessarily in the same order) and all of them are facing North. Therefore, in the given seating arrangement each member seated in a row faces another member of the other row. R sits second to left of the person who faces A. S is an immediate neighbour of R. Only one person sits between A and D. One of the immediate neighbours of C faces Q. B does not sit at any of the extreme ends of the line. 166. Who amongst the following sits second to the right of the person who faces P ? (1) A (2) B (3) C (4) D (5) Cannot be determined

    BPRE–282

    167. Four of the following five are alike in certain way based on the given seating arrangement and thus form a group. Which is the one that does not belong to the group? (1) C (2) R (3) Q (4) P (5) D 168. Which of the following is true regarding C ? (1) C sits second to right of D (2) A sits to immediate right of C (3) S faces C (4) D is an immediate neighbour of C (5) The person who faces C is an immediate neighbour of R. 169. Who amongst the following faces R? (1) A (2) B (3) C (4) D (5) Cannot be determined 170. Who amongst the following faces B? (1) P (2) Q (3) R (4) S (5) Cannot be determined Directions (171–173) : Read the following information carefully and answer the questions given below : (IBPS Specialist Officer CWE Exam.17.03.2013)

    Among five persons µ P, Q, R, S and T — each has different height. Only two persons are shorter than S. T is shorter than S but taller than R. The one who is the second tallest among them is of 158 centimetre. 171. Which of the following statements is definitely true with respect to the given information ? (1) R is definitely 150 cm (2) None of the given statements is true (3) T is shorter than S (4) Q is shorter than S (5) P is possibly 153 cm 172. Which of the following is possibly the height of Q ? (1) 148 cm (2) 156 cm (3) 152 cm (4) 150 cm (5) 158 cm 173. How many persons are shorter than Q ? (1) Two (2) Three (3) Four (4) Cannot be determined (5) One

    RANKING/ARRANGEMENT Directions (174 – 177) : Study the following information to answer the given questions : (IBPS Specialist Officer CWE Exam.17.03.2013)

    J, P, Q, R, S, T, U and V are four married couples sitting in a circle facing the centre, The profession of the males within the group are lecturer, lawyer, doctor and scientist. Among the males, only R (the lawyer) and V (the scientist) are sitting together. Each man is seated besides his wife. U, the wife of the lecturer is seated second to the right of V. T is seated between U and V. P is the wife of the doctor. Q is not the doctor. S is a male. 174. Which of the following is P’s position with respect to S ? (1) Second to the right (2) Second to the left (3) Immediate right (4) Immediate left (5) Third to the left 175. Which of the following is J’s position with respect to T ? (1) Third to the left (2) Fourth to the right (3) Third to the right (4) Opposite T (5) Second to the right 176. Which of the following is not true regarding the couples ? (1) P is the wife of S (2) T is the wife of Q (3) R is the husband of J (4) J and S are seated adjacent to each other (5) All are true 177. The wives of which two husbands are immediate neighbours? (1) UT (2) SR (3) VQ (4) RV (5) None of these 178. Four of the following are alike in a certain way based on their seating position in the above ar rangement and so form a group. Which is the one that does not belong to the group? (1) RSJ (2) TRV (3) UTV (4) SQP (5) UPQ Directions (179 – 182) : Study the following information to answer the given questions : (IBPS Specialist Officer CWE Exam.17.03.2013)

    Seven friends, T, U, V, W, X, Y and Z are sitting in a straight line facing north. W sits fifth to the right of T. W does not sit at any of extreme ends. Two people sit between Z and X. Y sits third to the left of U. Y sits exactly in the middle. Z is not an immediate neighbour of Y. 179. What is Z’s position with respect to W ? (1) Second to the left (2) Third to the right (3) Fourth to the left (4) Third to the left (5) Fourth to the right 180. Who is second to the right of T ? (1) Y (2) X (3) U (4) V (5) None of these 181. Four of the following five are alike in a certain way besed on their seating positions in the above line and so form a group. Which is the one that does not belong to the group ? (1) UW (2) XV (3) ZT (4) YV (5) WX 182. If all the seven friends are made to sit alphabetically from right to left. positions of how many will remain unchanged ? (1) None (2) One (3) Two (4) Three (5) Four Directions (183-187) : Study the following information carefully answer the given questions : (Indian Overseas Bank PO Online Exam, 01.09.2013)

    Twelve persons are sitting in two parallel rows containing six persons each, in such a way that there is an equal distance between adjacent persons. In row–1, A, B, C, D, E and F are seated (but not necessarily in the same order) and all of them are facing south. In row–2, P, Q, R, S, T and V are seated (but not necessarily in the same order) and all of them are facing north. Therefore, in the given seating arrangement each person seated in a row faces another person of the other row. A sits third to the left of E. The person facing A sits second to the left of T. Two persons are sitting between T and P. C and D are immediate neighbours. C and D do not sit at any of the extreme ends of the line. Only one

    BPRE–283

    person sits between B and C. The person facing D is an immediate neighbour of Q. V is not an immediate neighbour of P. S does not face A. 183. Who amongst the following sits seconds to the right of the person who faces R ? (1) C (2) D (3) B (4) E (5) Cannot be determined 184. Which of the following statements regarding B is true ? (1) B sits second to the left of C (2) A sits to immediate left of B (3) T faces B (4) D is an immediate neighbour of B (5) The person who faces B is an immediate neighbour of S 185. Who amongst the following faces P ? (1) A (2) D (3) C (4) E (5) Cannot be determined 186. Who amongst the following sits exactly between T and R ? (1) V (2) Q (3) S (4) P (5) Cannot be determined 187. Four of the following five are alike in a certain way based on the given seating arrangement and thus form a group. Which is the one that does not belong to the group? (1) F (2) Q (3) T (4) C (5) E Directions (188-192) : Study the following information carefully to answer the given questions : (Indian Overseas Bank PO Online Exam, 01.09.2013)

    Eight persons – A, B, C, D, E, F, G and H – are sitting around a circular table facing towards the centre, but not necessarily in the same order. All of them like different colours viz. yellow, purple, orange, black, white, green, red and blue. B is sitting second to the left of D. D likes blue colour. B does not like yellow colour. D is the immediate neighbour of that two persons who like red and purple colours respectively. Three persons sit between B and the person who likes green colour. F and A are immediate neighbours. E does not like black, yellow or purple colour. H is an immediate neighbour of G. A is immediate neighbour of both who like

    RANKING/ARRANGEMENT red and black colours respectively. E is sitting just opposite to H who likes orange colour. 188. How many persons are seated between H and D, if we go clockwise from H to D ? (1) Two (2) Two (3) Four (4) One (5) None 189. Which of the following pairs of persons represents the immediate neighbours of G ? (1) H and the person who likes black colour (2) F and the person who likes green colour (3) B and H (4) The person who likes orange colour and C (5) None of these 190. What is E’s position with respect to the person who likes orange colour ? (1) Third to the left (2) Fifth to the left (3) Fifth to the right (4) Fourth to the right (5) Second to the right 191. Starting from A, if all the persons are made to sit in the alphabetical order in clockwise direction, the positions of how many persons (excluding A) will remain unchanged ? (1) None (2) One (3) Two (4) Three (5) More than three 192. Who amongst the following is sitting exactly between the person who likes red colour and F ? (1) G (2) H (3) B (4) D (5) A Directions (193-198) % Study the following information carefully and answer the questions given below : (IBPS Bank PO/MT CWE-III, 26.10.2013)

    Eight people – A, B, C, D, E, F, G and H – are sitting around a circular table. A and B are facing towards the centre while other six people are facing opposite to the centre. A is sitting second to the right of H. B sits third to the left of A. D sits second to the right of G. G is immediate neighbour of neither B nor A. E and F are immediate neighbours and are facing outside. 193. What is the position of C with respect to D ? (1) Third to the right

    194.

    195.

    196.

    197.

    198.

    (2) Third to the left (3) Fourth to the left (4) Fourth to the right (5) Second to the left Who is sitting to the immediate right of G ? (1) C (2) D (3) F (4) H (5) None of these Which of the following pairs represents the people who are immediate neighbours of C ? (1) B and G (2) B and H (3) G and H (4) D and G (5) None of these Who is sitting third to the left of G? (1) A (2) D (3) E (4) F (5) Either E or F How many people are there between B and D ? (1) Two (2) Four (3) Three (4) One (5) None Eight people — M, N, O, P, Q, R, S and T – are sitting around a circular table facing the centre but not necessarily in the same order. N sits second to the left of M. Three people are sitting between N and S. P is the immediate neighbour of S. R and N are not immediate neighbours. O sits second to the left of P. Only three people sit between R and O. Who among the following is sitting third to the right of R ? (1) Q (2) N (3) T (4) M (5) O

    (IBPS Bank PO/MT CWE-III, 26.10.2013)

    Directions (199-203) : Study the following information carefully and answer the given questions : Twelve persons are sitting in two parallel rows containing six persons each, in such a way that there is an equal distance between adjacent persons. In Row-I, A, B, C, D, E and F are seated (but not necessarily in the same order) and all of them are facing south. In Row-2, P, Q, R, S, T and U are seated (but not necessarily in the same order) and all of them are facing north. Therefore, in the given sitting arrangement each person seated in a row faces another person of the other row. S sits third to the right of U. S does not sit at any extreme end. Two

    BPRE–284

    persons sit between P and Q. T sits third to the left of U. R is not an immediate neighbour of U. F sits third to the right of D. C does not sit at any extreme end. One person sits between B and D. D sits second to the left of C. C is not an immediate neighbour of B and A. 199. Who among the following are sitting at the extreme ends of any row? (1) ST (2) EB (3) FB (4) TP (5) DE 200. What is the position of S with respect to R? (1) Third to the right (2) Second to the right (3) Third to the left (4) Second to the left (5) Fourth to the right 201. Four of the following five are alike in a certain way based on their sitting arrangement and hence form a group. Which one does not belong to the group? (1) CR (2) BQ (3) FT (4) AS (5) EU 202. What is the position of A with respect to E? (1) Second to the right (2) Second to the left (3) Third to the left (4) Third to the right (5) Immediate left 203. If all the six persons of Row-1 (A, B, C, D, E and F) are asked to sit in an alphabetical order from left to right, the positions of how many will remain unchanged as compared to their original sitting position? (1) None (2) Two (3) Three (4) One (5) More than three Directions (204-208) : Study following information to answer the given questions : (Corporation Bank SO (Marketing) Exam, 22.02.2014)

    Six people – C, D, E, F, G and H– are standing in a straight line facing north not necessarily in the same order. D is standing second to the right of F. C is standing fourth to the left of H and H is not standing on the extreme end of the line. E is standing second to the right of D.

    RANKING/ARRANGEMENT 204. What is the position of G with respect to E ? (1) Immediate left (2) Second to the left (3) Third to the left (4) Third to the right (5) None of these 205. Which of the following pairs represents the people standing at the extreme ends of the line? (1) FH (2) CE (3) DE (4) CH (5) None of these 206. Who is standing second to the right of C ? (1) F (2) D (3) G (4) E (5) None of these 207. Four of the following five are alike in a certain way based on their positions in the above arrangement and so form a group. Which of the following does not belong to the group ? (1) CG (2) GE (3) GH (4) DE (5) FD 208. If all the people are asked to stand in an alphabetical order from left to right, the positions of how many will remain unchanged ? (1) One (2) Two (3) Three (4) None (5) None of these Directions (209–213) : Study the following information carefully and answer the questions given below : (IBPS RRBs Officer Scale–I & II CWE 12.09.2015)

    Eight persons — E, F, G, H, W, X, Y and Z — are sitting in two parallel rows containing four persons each. E, F, G and H are sitting in row–1 facing north and W, X, Y and Z are sitting in row–2 facing south (but not necessarily in the same order.) Thus, each person sitting in row – 1 faces another person sitting in row – 2. Each of the two rows consists of one Doctor, one Engineer, one Pilot and one Scientist (but not necessarily in the same order). ● The Doctor of row–1 sits second to the right of H. X faces one of the immediate neighbours of H. Only one person sits between the X and the Scientist. ● The one who faces the Scientist of row – 2 is an immediate neighbour of E. Only one person sits between E and the Pilot.

    ● W sits second to the right of Z. Y does not face G. The Scientist of row–1 faces the Engineer of row – 2. ● G faces one of the immediate neighbours of the Doctor of row– 2. The Doctor of row–2 does not sit at any of the extreme ends of the line. Z is not a Doctor. 209. Which of the following represents the people sitting at extreme ends of both the lines? (1) F, H and X, Y (2) F, H and Z, W (3) G, E and Z, X (4) E, H and X, Z (5) G, E and W, Y 210. Who amongst the following sits to the immediate left of Pilot of row–1? (1) H (2) The Doctor of row–1 (3) The Engineer of row – 1 (4) G (5) F 211. Which of the following represent both the immediate neighbours of Y? (1) Z and the Scientist of row–2 (2) X and the Engineer of row–2 (3) W and the Doctor of row–2 (4) X and the Pilot of row–2 (5) W and the Pilot of row–2 212. Which of the given statements is true with respect to the given arrangement? (1) G is a Scientist. (2) Y sits to the immediate right of X. (3) F and Z face each other. (4) None of the given statements is true (5) The Engineer of one row faces the Doctor of another row. 213. If Y and X interchange their places, so do H and Z, then who amongst the following will face E? (1) Y (2) H (3) F (4) W (5) Other than those given as options Directions (214–219) : Study the following information carefully and answer the questions given below : (IDBI Bank Officer Exam, 22.08.2014)

    Eight persons– E, F, G, H, I, J, K and L& are standing in a straight line, but not necessarily in the same

    BPRE–285

    order. Some of them are facing north while others are facing south. K is standing third to the left of J. J is at one of the extreme ends of the line. E is not an immediate neighbour of J. Neither I nor L is at the extreme end of the line. L is not an immediate neighbour of K. There is only one person between K and E. I is standing second to the left of L. F is to the immediate right of H. G is standing second to the right of E. The immediate neighbours of K face opposite direction to that of K. The immediate neighbour of J faces opposite direction. Both G and H face the same direction as that of E. E is to the immediate left of I who faces south. 214. The immediate neighbours of L are : (1) E and I (2) E and G (3) G and H (4) F and G (5) E and F 215. How many persons are standing exactly between H and G ? (1) Three (2) Four (3) Five (4) Two (5) None 216. Who among the following is to the immediate right of J ? (1) There is no person (2) H (3) L (4) E (5) J 217. Who among the following is fourth to the right of H ? (1) I (2) L (3) K (4) E (5) F 218. Four of the following five are alike in a certain way based on the above arrangement and hence they form a group. Which is the one that does not belong to that group ? (1) K (2) G (3) E (4) H (5) I 219. Who among the following is third to the right of K ? (1) J (2) E (3) L (4) G (5) F Directions (220–224) : Study the following information carefully and answer the questions given below : (SIDBI Officer Exam, 03.09.2014)

    Seven friends — P, Q, R, S, T, V and W are sitting in a straight line facing North, not necessarily in the same

    RANKING/ARRANGEMENT order. T sits at one of the extreme ends of the line. Q sits third to left of T. Only one person sits between S and W. S is an immediate neighbour of Q. P is not an immediate neighbour of T and S. P and R do not sit at the extreme ends of the line. 220. What is R’s position with respect to P ? (1) Second to the right (2) Fourth to the right (3) Second to the left (4) Third to the left (5) None of these 221. Four of the following five are alike in a certain way based on their seating positions in the above arrangement and so form a group. Which is the one that does not belong to the group ? (1) QS (2) VP (3) TR (4) PW (5) WQ 222. If all the seven friends are made to sit in alphabetical order from right to left, the positions of how many will remain unchanged ? (1) Four (2) Three (3) One (4) Two (5) None 223. Who sits exactly in the middle of the line ? (1) P (2) W (3) S (4) Q (5) R 224. Who amongst the following sits to the immediate left of W ? (1) Q (2) P (3) R (4) S (5) T Directions (225-229) : Study the following information carefully and answer the questions given below : (SIDBI Officer Exam. 03.09.2014)

    Eight persons – A, B, C, D, E, F, G and H – are sitting around a cir cular table facing the centre but not necessarily in the same order. Each of them represents different mobile companies viz, Motorola, LG, Samsung, Apple, Nokia, Sony, Micromax and Reliance but not necessarily in the same order. F is sitting second to the right of the person who represents Reliance. B and the person who represents Nokia are immediate neighbours of the person who represents Reliance. C and E are immediate neighbours of each oth-

    er. Neither C nor E is an immediate neighbour of B. A is to the immediate right of E. A represents Samsung company. Two persons are sitting between B and A. Three persons are sitting between D and H. G is an immediate neighbour of the person who represents Micromax company. There is only one person between C and the person who represents Apple company. H is third to the left of the person who represents Apple company. The person representing Sony is second to the left of the person who represents LG company. 225. Who among the following represents Sony company ? (1) F (2) H (3) D (4) C (5) B 226. What is the position of B with respect to C ? (1) Third to the left (2) Fourth to the left (3) Third to the right (4) Fourth to the right (5) Fifth to the left 227. If all the eight persons are made to sit in alphabetical order in clockwise direction starting from A, positions of how many persons will remain unchanged (excluding A) ? (1) One (2) Two (3) Three (4) Four (5) None 228. Four of the follwoing five are alike in a certain way based on the above arrangement and so form a group. Which is the one that does not belong to the group ? (1) AF (2) AE (3) BE (4) CG (5) DH 229. Which of the following statements is/are true about F ? (1) F represents Apple company (2) F is sitting exactly between B and D (3) F is sitting just opposite to the person who represents Samsung company (4) F is fourth to the left of A (5) All are true Directions (230-234): Study the following information carefully and answer the questions given below : (IBPS RRBs Officer Scale-I CWE, 06.09.2014)

    BPRE–286

    A building has seven floors numbered one to seven, in such a way that ground floor is numbered one, the floor above it, number two and so on such that the topmost floor is numbered seven. One out of seven per sons, viz., P, Q, R, S, T, U and V lives on each floor, but not necessarily in the same order. Each one of them is travelling to different places, viz., Bangalore, Chennai, Delhi, Jaipur, Kolkata, Mumbai and Patna, but not necessarily in the same order. Three persons live on the floors above the floor of P. There is only one person between P and the person travelling to Bangalore. U lives immediately below the person who is travelling to Mumbai. The person who is travelling to Mumbai lives on an even numbered floor. P lives below the person travelling to Mumbai. Two persons are living between the persons who are travelling to Bangalore and Patna respectively. T lives immediately above R. T is not travelling to Patna. Two persons live between Q and the person travelling to Kolkata. The person who is travelling to Delhi is not living immediately above or below the floor of Q. The person who is travelling to Kolkata lives below Q. S does not live immediately above or below the floor of P. V is not travelling to Chennai. The person who is travelling to Delhi does not live on the ground floor. 230. Who among the following lives on the topmost floor ? (1) U (2) Q (3) V (4) T (5) S 231. Four of the following five are alike in a certain way and hence they form a group based on the given arrangement. Which one of the following does not belong to that group? (1) R (2) S (3) V (4) U (5) T 232. Who among the following travels to Delhi ? (1) T (2) U (3) S (4) R (5) P 233. How many persons live between the person who is travelling to Mumbai and S ? (1) Three (2) Four (3) One (4) Two (5) Five

    RANKING/ARRANGEMENT 234. Who among the following does live on the floor immediately above the floor of T ? (1) S (2) Q (3) U (4) P (5) V Directions (235-237) : Study the following information carefully and answer the questions given below : (IBPS RRBs Officer Scale-I CWE, 06.09.2014)

    Among six friends P, Q, R, S, T and U each one has different weight. R is heavier than T. R is lighter than Q. Q is lighter than both P and U. T is not the lightest. T weighs 50 kg. The third heaviest person is of 63 kg. 235. Who among the following is the heaviest ? (1) U (2) P (3) S (4) Q (5) Either P or U 236. Who among following is heavier than S but lighter than R? (1) Q (2) T (3) U (4) Cannot be determined (5) None of these 237. Who among the following may weigh 58 kg ? (1) U (2) Q (3) P (4) R (5) Cannot be determined Directions (238-242): Study the following information carefully and answer the questions given below : (IBPS RRBs Officer Scale-I CWE, 06.09.2014)

    Nine friends, A, B, C, D, E, F, G, H and I are sitting around a circular table facing the centre but not necessarily in the same order. D is sitting second to the right of F. H is an immediate neighbour of E. Two persons are sitting between A and E. B is sitting second to the left of C. Two persons are sitting between D and C. Neither H nor E is immediate neighbour of C and D. G is sitting third to the right of A. Only one person is sitting between C and E. 238. In which of the following combinations is the first person sitting in between the second and the third persons ? (1) ADB (2) HEI (3) FIC (4) GBC (5) FDB 239. Who among the following is to the immediate left of D ?

    (1) B (2) A (3) F (4) G (5) I 240. ‘C’ is related to the ‘D’ in a certain way based on the given seating arrangement. In the same way ‘H’ is related to the ‘C’. To whom amongst the following is E related to, following the same pattern ? (1) B (2) D (3) C (4) A (5) G 241. How many persons are seated between F and G if we go anticlockwise from F to G ? (1) Two (2) Four (3) Three (4) None (5) One 242. Starting from A, if all the persons are made to sit in the alphabetical order in anticlockwise direction, the positions of how many (excluding A) will remain unchanged ? (1) One (2) Two (3) Three (4) Four (5) None Directions (243-247) : Study the following information carefully and answer the questions given below : (IBPS Bank PO/MT CWE-IV, 18.10.2014)

    Eight persons – J, K, L, M, W, X, Y and Z – are standing in a straight line, but not necessarily in the same order. Some of them are facing north while some others are facing south. J is standing at the fourth position to the right of X. X is standing at one of the extreme ends of the line. Both the immediate neighbours of J face north. M is standing at the third position to the right of J. M is facing the same direction as that of J. There is only one person between M and L. L is standing at the third position to the right of Y. Z is standing to the immediate left of L. K is not facing north. Z is facing the same direction as that of W. K is not standing at any of the extreme ends of the line. 243. Who among the following are facing South ? (1) M, J, Y (2) J, K, M, X (3) J, K, M, X, Y (4) J, M, X, Y (5) K, M, X, Y 244. How many persons are standing exactly between Y and Z ? (1) Three (2) Four (3) Two (4) Five (5) None of these

    BPRE–287

    245. Who among the following is to the immediate left of W ? (1) K (2) Y (3) L (4) J (5) None of these 246. Four of the following five are alike in a certain way based on the above arrangement and hence form a group. Which is the one that does not belong to the group ? (1) M (2) J (3) L (4) Y (5) X 247. Who among the following is standing at one of the ends excluding X ? (1) Z (2) M (3) W (4) Y (5) L Directions (248–253) : Study the following information carefully and answer the questions given below : (Bank of Baroda Junior Management Grade/Scale-I Exam, 18.04.2015)

    Eight persons – P, Q, R, S, T, U, V and W — are sitting around a circular table with equal distance between each other, facing the centre, but not necessarily in the same order. Each one of them belongs to a different profession viz. Manager, Engineer, Chef, Pilot, Lawyer, Doctor, Architect and Teacher but not necessarily in the same order. V sits second to the right of the Manager. The Pilot and the Engineer are the immediate neighbours of V. R sits second to the right of T who is a Lawyer. T is an immediate neighbour of the Pilot. Only one person sits between W and U. S sits third to the left of T. P sits exactly between U and S. The Architect sits second to the left of P. The Chef and the Teacher are immediate neighbours of the Architect. W is not a Teacher. 248. Who among the following is a doctor ? (1) R (2) V (3) Other than those given as options (4) W (5) P 249. Who sits exactly between R and the Manager, when counted from the right of R ? (1) W (2) T (3) Q (4) S (5) U

    RANKING/ARRANGEMENT 250. Which of the following statements is not true as per the given information ? (1) R is a Chef. (2) P and V are immediate neighbours of S. (3) All the given statements are true (4) Only three persons sit between U and Q (5) S is an Engineer 251. Four of the following five are alike in a certain way based on their positions in the arrangement and hence form a group. Which one does not belong to that group ? (1) SU (2) RW (3) TQ (4) PU (5) VS 252. Who sits second to the right of Q? (1) S (2) P (3) Other than those given as options (4) W (5) U 253. If all the persons are made to sit in the alphabetical order in clockwise direction starting from P, the position of how many of them will remain unchanged (excluding P) ? (1) Three (2) One (3) Two (4) Four (5) Nonea Directions (254–259) : Study the following information carefully and answer the questions given below : (Bank of Baroda Junior Management Grade/Scale-I Exam, 18.04.2015)

    Seven people – P, Q, R, S, T, U and V – are sitting in a straight line with equal distance between each other, but not necessarily in the same order. Some of them are facing North and some are facing South. Only two people are sitting to the the left of V. Only two people sit between V and Q. P sits second to the left of Q. The immediate neighbours of P face opposite directions (i.e. if one of the neighbours faces South then the other faces North and vice–versa). Only one person sits between P and R. U sits third to the left of R. S is not an immediate neighbour of Q. Both the immediate neighbours of R face the same direction (i.e. if one neighbour faces South then the other neighbour also faces South and vice– versa). P faces the same direction as that of R. T faces North. Q sits to the immediate left of T.

    254. Who amongst the following sits exactly between V and the on who is sitting to the immediate left of Q ? (1) P (2) R (3) Other than those given as options (4) T (5) U 255. Who amongst the following sits exactly in the middle of the line? (1) S (2) P (3) U (4) T (5) R 256. Which of the following statements is TRUE as per the given information ? (1) S faces South (2) V sits third to the left of Q (3) None of the given options is true (4) S sits exactly between R and P (5) U sits to the immediate right of P 257. Which of the following pairs represent the people sitting at the two extreme ends of the line ? (1) QR (2) Other than those given as options (3) TU (4) SQ (5) ST 258. Who amongst the following sits second to the right of S ? (1) U (2) V (3) Q (4) Other than those given as options (5) P 259. Which of the following pairs represent the immediate neighbours of Q ? (1) TU (2) QP (3) PR (4) TV (5) VU Directions (260– 265) : Study the following information carefully and answer the questions given below : (BOB Junior Management Grade/Scale–I Exam. 18.04.2015)

    Eight persons – P, Q, R, S, T, U, V and W — are sitting around a circular table with equal distance between each other, facing the centre, but not necessarily in the same order. Each one of them belongs to a different profession viz. Manager, Engineer, Chef, Pilot, Lawyer, Doctor, Architect and Teacher but not necessarily in the same order.

    BPRE–288

    V sits second to the right of the Manager. The Pilot and the Engineer are the immediate neighbours of V. R sits second to the right of T who is a Lawyer. T is an immediate neighbour of the Pilot. Only one person sits between W and U. S sits third to the left of T. P sits exactly between U and S. The Architect sits second to the left of P. The Chef and the Teacher are immediate neighbours of the Architect. W is not a Teacher. 260. Who among the following is a doctor ? (1) R (2) V (3) Other than those given as options (4) W (5) P 261. Who sits exactly between R and the Manager, when counted from the right of R ? (1) W (2) T (3) Q (4) S (5) U 262. Which of the following statements is not true as per the given information ? (1) R is a Chef. (2) P and V are immediate neighbours of S. (3) All the given statements are true (4) Only three persons sit between U and Q (5) S is an Engineer 263. Four of the following five are alike in a certain way based on their positions in the arrangement and hence form a group. Which one does not belong to that group ? (1) SU (2) RW (3) TQ (4) PU (5) VS 264. Who sits second to the right of Q? (1) S (2) P (3) Other than those given as options (4) W (5) U 265. If all the persons are made to sit in the alphabetical order in clockwise direction starting from P, the position of how many of them will remain unchanged (excluding P) ? (1) Three (2) One (3) Two (4) Four (5) None

    RANKING/ARRANGEMENT Directions (266–271) : Study the following information carefully and answer the questions given below : (BOB Junior Management Grade/Scale–I Exam. 18.04.2015)

    Seven people – P, Q, R, S, T, U and V – are sitting in a straight line with equal distance between each other, but not necessarily in the same order. Some of them are facing North and some are facing South. Only two people are sitting to the the left of V. Only two people sit between V and Q. P sits second to the left of Q. The immediate neighbours of P face opposite directions (i.e. if one of the neighbours faces South then the other faces North and vice–versa). Only one person sits between P and R. U sits third to the left of R. S is not an immediate neighbour of Q. Both the immediate neighbours of R face the same direction (i.e. if one neighbour faces South then the other neighbour also faces South and vice–versa). P faces the same direction as that of R. T faces North. Q sits to the immediate left of T. 266. Who amongst the following sits exactly between V and the on who is sitting to the immediate left of Q ? (1) P (2) R (3) Other than those given as options (4) T (5) U 267. Who amongst the following sits exactly in the middle of the line ? (1) S (2) P (3) U (4) T (5) R 268. Which of the following statements is TRUE as per the given information ? (1) S faces South (2) V sits third to the left of Q (3) None of the given options is true (4) S sits exactly between R and P (5) U sits to the immediate right of P 269. Which of the following pairs represent the people sitting at the two extreme ends of the line ? (1) QR (2) Other than those given as options (3) TU (4) SQ (5) ST

    270. Who amongst the following sits second to the right of S ? (1) U (2) V (3) Q (4) Other than those given as options (5) P 271. Which of the following pairs represent the immediate neighbours of Q ? (1) TU (2) QP (3) PR (4) TV (5) VU Directions (272–276) : Study the following information carefully and answer the questions given below : (IBPS RRBs Officer Scale–I & II CWE 12.09.2015)

    Eight friends — J, K, L, M, N, O, P and Q — are sitting around a circular table but not necessarily in the same order. Some of them are facing the centre and some of them are facing outside. (i.e. in a direction opposite to the centre.) Facing the same direction means if one person faces the centre then the other also faces the centre and vice– versa. Facing the opposite direction means if one person faces the centre then the other faces outside and vice– versa. Immediate neighbours facing the same direction means if one neighbour faces the centre then the other also faces the centre and vice–versa. Immediate neighbours facing the opposite direction means if one neighbour faces the centre then the other faces outside and vice–versa. ● Only one person sits between K and O. Q sits third to the right of O. ● M sits to the immediate right of Q. Q faces outside. ● L sits second to the left of P. P is not an immediate neighbour of O. ● L faces a direction opposite to that of O. Immediate neighbours of L face opposite directions. ● J sits third to the left of N. J is not an immediate neighbour of P nor K. ● M and J face a direction same as that of N. 272. Four of the following five are alike in a certain way based on the directions they are forming and so form a group. Which is the one that does not belong to that group?

    BPRE–289

    (1) KL (2) QM (3) PQ (4) NJ (5) QL 273. Who among the following sit exactly between Q and O when counted from the right of Q? (1) P, J (2) M, P (3) L, K (4) N, L (5) J, M 274. Which of the following is not true regarding K as per the given arrangement? (1) N is an immediate neighbour of K. (2) All the given options are true (3) Only three persons sit between K and M. (4) L sits to the immediate left of K. (5) K faces outside. 275. What is L’s position with respect to N? (1) Immediate right (2) Second to the right (3) Third to the right (4) Third to the left (5) Immediate left 276. Who amongst the following are immediate neighbours of P ? (1) M, N (2) K, M (3) J, Q (4) N, K (5) Q, N Directions (277–281) : Study the following information carefully and answer the questions given below : (IBPS Bank PO/MT CWE–V (Preliminary) 03.10.2015)

    Ten persons are sitting in two parallel rows containing five persons each, in such a way that there is equal distance between adjacent persons. In row–1, J, K, L, M and N are seated (not necessarily in the same order) and all of them are facing south. In row–2, V, W, X, Y and Z are seated (not necessarily in the same order) and all of them are facing north. Therefore in the given seating arrangement each member seated in row faces another member of the other row. Z sits third to the right of W. V sits second to the left of Z. The person facing V sits to the immediate right of K. Only one person sits between K and M. J is not an immediate neighbour of K. Only two persons sit between J and L. Neither K nor J faces Y. 277. Who amongst the following is facing N ? (1) Y (2) V (3) X (4) W (5) Z

    RANKING/ARRANGEMENT 278. Which of the following statements is true regarding M ? (1) M faces one of the immediate neighbours of X. (2) K is one of the immediate neighbours of M. (3) None of the given statements is true (4) L sits to the immediate right of M. (5) Only one person sits between M and N. 279. Who amongst the following is facing X ? (1) K (2) L (3) M (4) J (5) N 280. What is the position of Z with respect to Y ? (1) Third to the right (2) Second to the right (3) Immediate left (4) Immediate right (5) Second to the left 281. Four of the following five are alike in a certain way based on the given arrangement and hence form a group. Which of them does not belong to that group ? (1) M (2) J (3) Y (4) W (5) N Directions (282–286) : Study the following information carefully and answer the questions given below : (IBPS Bank PO/MT CWE–V (Preliminary) 04.10.2015)

    Ten persons are sitting in two parallel rows containing five persons each, in such a way that there is equal distance between adjacent persons. In row 1 — B, C, D, E and F are seated (not necessarily in the same order) and all of them are facing south. In row 2 — R, S, T, U and V are seated (not necessarily in the same order) and all of them are facing north. Therefore, in the given seating arrangement, each member seated in a row faces another member of the other row. R sits second to the right of U. The person facing R sits to the immediate left of D. Only one person sits between D and E. E does not sit at an extreme end of the line. The person facing E is an immediate neighbour of T. S sits third to the left of T. F is not an immediate neighbour of D. B does not face U.

    282. Who amongst the following is facing V ? (1) F (2) D (3) C (4) B (5) E 283. Who amongst the following is facing C ? (1) T (2) S (3) V (4) U (5) R 284. What is the position of D with respect to F ? (1) Immediate left (2) Second to the left (3) Third to the right (4) Immediate right (5) Second to the right 285. Which of the following statements is true regarding B? (1) None of the given statements is true (2) C sits second to the right of B (3) B sits at an extreme end of the row (4) B faces one of the immediate neighbours of T (5) Only two persons sit between B and F 286. Four of the following five are alike in a certian way based on the given arrangement and hence form a group. Which one of the following does not belong to that group ? (1) RU (2) SR (3) VT (4) EB (5) DC Directions (287–291) : Study the following information carefully and answer the questions given below :

    son sits between L and K. The one who faces K sits to the immediate left of S. N sits second to the right of J. Neither K nor L faces U. 287. Who amongst the following is facing M?

    (IBPS Bank PO/MT CWE–V (Preliminary.) 04.10.2015)

    (1) N (2) M (3) K (4) J (5) L Directions (292–296) : Study the following information carefully and answer the questions given below :

    Ten persons are sitting in two parallel rows containing five people each, in such a way that there is equal distance between adjacent persons. In row–1– J, K, L, M and N are seated (not necessarily in the same order) and all of them are facing North. In row–2– R, S, T, U and V are seated (not necessarily in the same order) and all of them are facing South. Therefore in the given seating arrangement each member seated in a row faces another member of the other row. V sits at one of the extreme ends of the line. Only two people sit between V and R. The one who faces R sits to the immediate left of L. Only one per-

    BPRE–290

    (1) U (2) R (3) T (4) V (5) S 288. Four of the following five are alike in a certain way based on the given arrangement and hence form a group. Which of them does not belong to that group? (1) NK (3) ML

    (2) JL (4) RU

    (5) ST 289. What is the position of U with respect to S? (1) Second to the right (2) Immediate left (3) Second to the left (4) Third to the right (5) Immediate right 290. Which of the following statements is true regarding N? (1) N sits at an extreme end of the row. (2) Only two persons sit between N and M. (3) N faces one of the immediate neighbours of R. (4) None of the given statements is true. (5) L sits to the immediate right of N. 291. Who amongst the following is facing T?

    (IBPS Bank PO/MT CWE–V (Preliminary.) 04.10.2015)

    Eight persons — C, D, E, F, P, Q, R and S — are sitting around a circular table facing the centre with equal distance between each other (but not necessarily in the same order). Each one of them is also related to D in some way or the other. P sits third to the right of E. D sits to the immediate left of E. Only one person sits between P and D’s son.

    RANKING/ARRANGEMENT

    (IBPS Bank PO/MT CWE–V (Preliminary) 10.10.2015 Ist Sitting)

    not necessarily in the same order. In row – 2, T, U, V, W and X are seated and all of them are facing north but not necessarily in the same order. Therefore, in the given seating arrangement each member seated in a row faces another member of the other row. V sits exactly in the middle of the row. The one who faces V sits to the immediate left of F. H is an immediate neighbour of F but does not face V. W sits second to left of U. U faces the one who is an immediate neighbour of D. G is not an immediate neighbour of D. G does not sit at the extreme end of the row. X does not face H. 297. Who amongst the following is facing V ? (1) G (2) E (3) F (4) D (5) Cannot be determined 298. Which of the following statements is true regarding T ? (1) T faces F (2) Only two persons sit to the left of T (3) T sits to the immediate left of W. (4) U sits fourth to the right of T. (5) None of the given statements is true 299. What is the position of D with respect to F ? (1) Third to the right (2) Second to the right (3) Immediate left (4) Third to the left (5) Second to the left 300. Four of the following five are alike in a certain way based on the given seating arrangement and hence they form a group. Which of the following does not belong to that group ? (1) H (2) T (3) X (4) D (5) U 301. Who amongst the following is facing F ? (1) T (2) W (3) V (4) X (5) U Directions (302-306) : Study the following information carefully and answer the questions given below :

    Ten persons are sitting in two parallel rows containing five persons each in such a way that there is equal distance between adjacent persons. In row – 1, D, E, F, G and H are seated and all of them are facing south but

    Eight persons – J, K, L, M, Q, R, S, and T – are sitting around a circular table facing the centre but not necessarily in the same order. Each of

    R sits to the immediate right of D’s son. Only three persons sit between D’s husband and R. Only one person sits between D’s husband and C. F sits to the immediate right of Q. D’s father sits second to the right of F. Only three persons sit between D’s father and D’s brother. D’s daughter sits second to the right of S. D’s sister sits third to the right of D’s mother. 292. Which of the following statements is TRUE with respect to the given information? (1) S is the brother of R. (2) Only three persons sit between Q and S. (3) E sits third to the right of D’s daughter. (4) All the given options are true (5) R is an immediate neighbour of D. 293. Who amongst the following is the brother of D? (1) Q (2) E (3) F (4) C (5) R 294. As per the given seating arrangement, Q : P in the same way as R : E. Then following the same pattern D : ? (1) R (2) S (3) Q (4) C (5) F 295. Who sits to the immediate left of D’s son? (1) D’s father (3) Q (5) D’s mother

    (2) R (4) S

    296. How is Q related to P? (1) Sister (2) Sister–in–law (3) Niece (4) Brother–in–law (5) Aunt Directions (297–301) : Study the following information carefully and answer the questions given below :

    (IBPS Bank PO/MT CWE–V (Preliminary) 10.10.2015 Ist Sitting)

    BPRE–291

    them is related to M in some way or the other. Two persons are sitting between Q and L. M is sitting second to the left of Q. Three persons are sitting between L and the wife of M. The son of M is sitting second to the right of the wife of M. Three persons are sitting between the son of M and the brother of M. The daughter of M is sitting second to the left of the brother of M. J is sitting to the immediate right of R. R is neither son nor wife of M. The sister of M is sitting second to the left of R. K is sitting to the immediate right of the sister of M. Two persons are sitting between K and the father of M. T is sitting second to the right of the mother of M. 302. Who amongst the following is the brother of M? (1) L (2) S (3) T (4) R (5) J 303. What is the position of M’s daughter with respect to M’s son? (1) Third to the right (2) Second to the left (3) Third to the left (4) Second to the right (5) Immediate right 304. Who amongst the following is the wife of M ? (1) K (2) S (3) R (4) L (5) T 305. Which of the following statements is true regarding the given seating arrangement ? (1) M’s father is sitting to the immediate left of M’s son (2) Only four persons are sitting between S and J. (3) M is sitting exactly between his daughter and brother (4) M’s wife is sitting just opposite to M’s father. (5) None of the given statements is true 306. Who amongst the following is the sister of M ? (1) R (2) S (3) L (4) J (5) Q Directions (307–311) : Study the following information carefully and answer the questions given below : (IBPS Bank PO/MT CWE–V (Preliminary) 10.10.2015)

    Ten persons are sitting in two parallel rows containing five people each, in such a way that there is equal dis-

    RANKING/ARRANGEMENT tance between adjacent persons. In row-1, A, B, C, D and E are seated (but not necessarily in the same order) and all of them are facing north. In row-2, L, M, N, O and P are seated (but not necessarily in the same order) and all of them are facing south. Therefore, in the given seating arrangement each member seated in a row faces another member of the other row. B sits second to the right of D. The person facing B sits to the immediate left of N. L sits second to the right of N. Only two persons sit between L and P. E is not an immediate neighbour of D. O does not face E. C neither faces N nor sits at an extreme end of the line. 307. Which of the following statements is TRUE regarding O? (1) L sits to the immediate left of O. (2) Only three persons sit between P and O. (3) O sits exactly in the middle of the row. (4) O faces one of the immediate neighbours of B. (5) None of the given statements is true 308. Who amongst the following is facing A? (1) L (2) M (3) O (4) P (5) N 309. Who amongst the following is facing M? (1) E (2) A (3) D (4) B (5) C 310. Four of the following five are alike in a certain way based on the given arrangement and hence form a group. Which is the one that does not belong to the group? (1) NO (2) CE (3) AB (4) PO (5) ML 311. What is the position of C with respect to B? (1) Second to the left (2) Third to the left (3) Immediate left (4) Immediate right (5) Second to the right Directions (312–316) : Study the following information carefully and answer the questions given below. (IBPS RRBs Officer Scale–I & II CWE 13.09.2015)

    Eight persons E, F, G, H, W, X, Y and Z are sitting in two parallel rows containing four persons each in such a way that there is an equal distance between adjacent persons. E, F, G and H are sitting in row-1 facing north and W, X, Y and Z are sitting in row-2 facing south. (but not necessarily in the same order.) Thus, each person sitting in row-1 faces another persons sitting in row-2. Each of the two rows consists of the one Professor, one Leader, one Technician and one Doctor (but not necessarily in the same order). ● The Doctor of row-1 sits second to the right of H. E is an immediate neighbour of H. E faces the Leader of row-2. ● X sits to the immediate right of the Leader. X faces one of the immediate neighbours of the Professor of row-1. The Professor of row-1 does not sit at any of the extreme ends of the line. ● W sits second to the left of Y. Z does not face G. F faces the Professor of row-2. In both the rows, onlye one person sits between the Professor and the Technician. W is not a Doctor. 312. Which of the following represent both the immediate neighbours of W ? (1) X and the Leader of row-2 (2) Y and the Technician of row2 (3) Y and the Doctor of row-2 (4) X and the Professor of row-2 (5) Z and the Doctor of row-2 313. Who amongst the following sits to the immediate left of the Leader of row-1 ? (1) The Professor of row-1 (2) F (3) H (4) G (5) No one 314. Which of the given statements is true with respect to the given arrangement ? (1) None of the given statements is true (2) Y sits to the immediate right of Z. (3) The Technician of one row faces the DOctor of another row. (4) Y and H face each other (5) H is a Technician 315. If E and X interchange their places and so do G and Z, then who amongst the following will face W?

    BPRE–292

    (1) X (2) Other than those given as options (3) A (4) Z (5) H 316. Which of the following represent the people sitting at extreme ends of both the lines ? (1) F, H and Z, W (2) G, H and X, Y (3) E, H and Y, Z (4) G, F and Z, Y (5) G, F and W, Y Directions (317-321) : Study the following information carefully and answer the questions given below : (IBPS RRBs Officer Scale–I & II CWE 13.09.2015)

    Eight friends P, Q, R, S, W, X, Y and Z are sitting around a circular table but not necessarily in the same order. Some of them are facing the centre and some others are facing outside (i.e. in a direction opposite to the centre.) Note : (i) Facing the same direction means if one person faces the centre then the other also faces the centre and viceversa. (ii) Facing the opposite directions means if one person faces the centre then the other faces outside and vice– versa. (iii) Immediate neighbours facing the same direction means if one person faces the centre then the other also faces the centre and vice-versa. (iv) Immediate neighbours facing the opposite directions means if one person faces the centre then the other faces outside and vice-versa. ● R sits second to the right of Y. Only two persons sit between R and W. ● P sits to the immediate right of W. W faces outside. ● Only one person sits between P and Z. Immediate neighbours of P face opposite directions. ● Q sits third to the left of Z. Q is not an immediate neighbour of P. ● X faces a direction opposite to that of Y. X is an immediate neighbour of neither Y nor P. ● Immediate neighbours of S face same direction. P does not face outside. ● R and Q face a direction opposite to that of S.

    RANKING/ARRANGEMENT 317. Four of the following five are alike in a certain way based on the direction they are facing and so form a group. Which is the one that does not belong to that group ? (1) RW (2) QZ (3) PQ (4) YX (5) XP 318. Who among the followings sit exactly between X and P when counted from the left of X ? (1) Z, R (2) S, Z (3) R, Q (4) S, W (5) Q, Y 319. Who among the following are immediate neighbours of Y ? (1) Z, P (2) S, X (3) Q, S (4) W, Q (5) X, Z 320. What is Q’s position with respect to W ? (1) Immediate left (2) Third to the left (3) Second to the left (4) Immediate right (5) Second to the right 321. Which of the following is true regarding S as per the given seating arrangement ? (1) R is an immediate neighnour of S. (2) S faces outside. (3) Only two persons sit between S and Q. (4) X sits second to the right of S. (5) None of the given options is true Directions (322-327) : Study the following information carefully and answer the questions given below : Eight persons — S, T, U, V, W, X, Y and Z are seated in a straight line but not necessarily in the same order, some of them are facing south while some are facing North. S sits fourth to left of X. X sits at one of the extreme end of the line. Both the immediate neighbours of S face south. T sits second to left of Z. Z is not an immediate neighbour of S. Neither Z nor U sits at the extreme end of the line. Both the immediate neighbours of U face north. W sits to immediate left of Y. Immediate neighbours of V face opposite directions (i.e. if one neighbour of V faces north then the other faces south and vice – versa). Immediate neighbours of T face oppo-

    site directions (i.e. if one neighbour of T faces north then the other faces south and vice–versa). People sitting at the extreme ends face the same directions (i.e. if one person faces North then the other also faces north and vice–versa). 322. Which of the following pairs represents immediate neighbours of the persons seated at the two extreme ends of the line? (1) U, Z (2) T, Y (3) W, T (4) Y, Z (5) S, T 323. How many persons are seated between T and X ? (1) Four (2) More than four (3) One (4) Three (5) Two 324. If each of the persons is made to sit in alphabetical order from right to left the positions of how many will remain unchanged as compared to the original seating arrangement? (1) One (2) Two (3) Four (4) None (5) Three 325. Who amongst the following sits exactly between Z and T? (1) U (2) V (3) Y (4) X (5) W 326. In a row of children facing north, Neeta is fifteenth from the left end of the row. If she is shifted towards the right end of the row by four places, she becomes eighth from the right end. How many children are there in the row? (1) 24 (2) 28 (3) Other than those given as options (4) 27 (5) 26 327. Eight friends A, B, C, D, E, F, G and H are sitting around a circle facing the centre. D is sitting between B and G. F is sitting between A and H. E is at second place right to A. What is the position of A? (1) Left to F (2) Right of F (3) Between E and F (4) Cannot be determined (5) Other than those given as options

    BPRE–293

    Directions (328–332) : Study the following information carefully and answer the questions given below : (IBPS Specialist Officer (Marketing) CWE 01.02.2016)

    Eight friends– P, Q, R, S, T, U, V and W – are sitting around a square table in such a way that four of them sit at four corners while four sit in the middle of each of the four sides. The ones who sit at the four corners face the centre while those who sit in the middle of the sides face outside (i.e. opposite to the centre). ● V sits second to the right of R. R sits in the middle of one of the sides of the table. ● Only two persons sit between V and Q. S is one of the immediate neighbours of Q. ● T sits second to the left of S. ● P sits second to the left of U. ● V is not an immediate neighbour of U. 328. How many persons sit between R and T when counted from the right of R? (1) None (2) Four (3) Three (4) Two (5) One 329. Which of the following statements is true regarding P? (1) Both T and R are immediate neighbours of P. (2) Only three persons sit between P and S. (3) P sits at middle of one of the sides. (4) W sits second to the left of P. (5) None of the given options is True 330. What is the position of V with respect to Q? (1) Second to the left (2) Third to the left (3) Second to the right (4) Fifth to the right (5) Fifth to the left 331. Four of the following five are alike in a certain way based on the given arrangement and so form a group. Which is the one that does not belong to the group? (1) Q (2) T (3) S (4) R (5) V 332. Who sits second to the left of W? (1) T (2) Q (3) V (4) S (5) U

    RANKING/ARRANGEMENT Directions (333–337) : Study the following information carefully and answer the questions given below : (IBPS Specialist Officer (Marketing) CWE 01.02.2016)

    Twelve people are sitting in two parallel rows containing six people each in such a way that there is an equal distance between adjacent persons. In row – 1 – J, K, L, M, N and O are seated (but not necessarily in the same order) and all of them are facing South. In row – 2 – U, V, W, X, Y and Z are seated (but not necessarily in the same order) and all of them are facing North. Therefore, in the given seating arrangement, each member seated in a row faces another member of the other row. ● M sits fourth to the left of J. The one facing J sits third to the left of Y. ● Only one person sits between Y and U. U does not sit at any of the extreme ends of the line. ● The one facing Z sits second to the right of K. Z does not sit at any of the extreme ends of the the line. ● Only two persons sit between K and O. ● The one facing K sits second to the left of X. ● V is not an immediate neighbour of Z. L is not an immediate neighbour of M. 333. Which of the following groups represents the persons sitting at extreme ends of both the rows? (1) M, O, X, W (2) M, K, V, W (3) N, K, V, Y (4) J, N, U, V (5) J, O, Z, X 334. Who amongst the following faces V? (1) M (2) L (3) J (4) N (5) K 335. Which of the following statements is true with respect to the given information? (1) K faces one of the immediate neighbours of X. (2) V sits exactly between W and U. (3) None of the given options is true (4) J faces Z. (5) J is an immediate neighbour of K.

    336. Which of the following statements is true regarding N? (1) K sits second to right of N. (2) V is an immediate neighbour of the person who faces N. (3) Both L and O are immediate neighbours of N. (4) Only one person sits between N and J. (5) None of the given options is true 337. Who amongst the following sits second to the right of the person who faces L? (1) V (2) Z (3) Y (4) U (5) W (IBPS Specialist Officer (IT) CWE 14.02.2016)

    Directions (338–342) : Study the following information carefully and answer the questions given below : (IBPS Specialist Officer (IT) CWE 14.02.2016)

    Eight friends L, M, N, O, P, Q, R and S are sitting around a square table in such a way that four of them sit at four corners while four sit in the middle of each of the four sides, but not necessarily in the same order. The ones who sit in the middle of the sides face the centre while those who sit at the four corners face outside (i.e. opposite to the centre.) ● R sits third to the right of P. P sits on one of the corners of the table. ● Only one person sits between O and R. M is one of the immediate neighbours of O. ● N sits second to the right of M. ● Only three persons sit between M and L. ● S sits to the immediate right of L. 338. What is the position of N with respect to L ? (1) Second to the right (2) Third to the right (3) Third to the left (4) Second to the left (5) Fifth to the right 339. Which of the following statements is true regarding R on the basis of given arrangement ? (1) Either two or four persons are sitting between R and S (2) Q sits second to the right of R (3) L sits third to the left of R

    BPRE–294

    (4) R is an immediate neighbour of both N and Q. (5) None of the given statements is true 340. How many persons sit between R and S when counted from the right of R ? (1) Three (2) Four (3) None (4) Two (5) One 341. Who sits second to the left of O ? (1) L (2) R (3) P (4) N (5) S 342. Four of the following five are alike in a certain way based on the given arrangement and so form a group. Which is the one that does not belong to that group ? (1) S (2) Q (3) O (4) R (5) M Directions (343–347) : Study the following information carefully and answer the questions given below : (IBPS Specialist Officer (IT) CWE 14.02.2016)

    Twelve persons are sitting in two parallel rows containing six people each in such a way that there is an equal distance between adjacent persons. In row-1, M, N, O, P, Q and R are seated (but not necessarily in the same order) and all of them are facing south. In row-2, A, B, C, D, E and F are seated (but not necessarily in the same order) and all of them are facing north. Therefore, in the given seating arrangement, each member seated in a row faces another member of the other row. ● B faces one who sits third to the right of Q. ● Only three persons sit between Q and N. ● R is neither an immediate neighbour of N nor Q. One of the immediate neighbours of R faces F. ● Only three persons sit between F and the one who faces O. ● A sits second to the left of the one facing M. ● C is not an immediate neighbour of B. N does not face E. 343. Which of the following statements is true with respect to the given information? (1) D faces N (2) M faces one of the immediate neighbours of F.

    RANKING/ARRANGEMENT (3) F sits exactly between A and D. (4) None of the given options is true (5) O is an immediate neighbour of R. 344. Who amongst the following sits second to the right of the person who faces B ? (1) M (2) O (3) R (4) N (5) P 345. Which of the following groups of persons represents the persons sitting at extreme ends of both the rows ? (1) M, Q, C, F (2) P, R, D, E (3) N, R, E, F (4) O, Q, A, C (5) Q, P, A, C 346. Four of the following five are alike in a certain way based on the given arrangement and so form a group. Which is the one that does not belong to that group ? (1) AD (2) RM (3) PQ (4) BE (5) CF 347. Who amongst the following faces E? (1) Q (2) P (3) O (4) M (5) R Directions (348–352) : Study the following information carefully and answer the questions given below : (BOB Manipal School of Banking Officer Online Exam, 14.08.2014)

    Eight persons – M, N, O, P, Q, R, S and T – are sitting arround a circular table facing the centre, but not necessarily in the same order. Each one of them likes different colours, viz., Red, Green, Blue, Orange, Brown, Purple, Pink and White, but not necessarily in the same order. The person who likes red colour is sitting third to the right of R. There is only one person between the persons who like red and orange colours. The person who likes orange colour is not to the immediate right of R. P is sitting third to the right of O. N is sitting to the immediate left of S. N does not like red, green or orange colour. P is not an immediate neighbour of R. There is only one person between P and the person who likes blue colour. O is sitting to the immediate left of the person who likes red colour. S does not like red or ornage colour. M is an immediate neighbour of both R and O. The person who likes white

    colour is to the immediate right of that person who likes red colour. Q does not like white colour. The person who likes white colour is just opposite to the person who likes pink colour. M likes brown colour. 348. Who among the following likes Green colour ? (1) T (2) P (3) O (4) R (5) Q 349. Starting from M, if all the persons are made to sit in the alphabetical order in anticlockwise direction, the position of how many (excluding M) will remain unchanged ? (1) One (2) Two (3) Three (4) Four (5) None 350. Which of the following statements is NOT TRUE with respect to the given seating arrangement ? (1) The person who likes blue colour is second to the left of M. (2) T is sitting exactly between the person who likes orange colour and Q (3) S is sitting just opposite to Q (4) N likes pink colour (5) The person who likes green colour is sitting third to the left of P. 351. Four of the following five are alike in a certain way based on the above seating arrangement and hence they form a group. Which one of the following does not belong to that group ? (1) MP (2) QR (3) NO (4) SQ (5) TR 352. Who among the following is sitting exactly between R and N ? (1) Q (2) T (3) P (4) O (5) S Directions (353–358) : Study the following information carefully and answer the questions given below : (IDBI Bank Officer Exam, 22.08.2014)

    Eight persons– S, T, U, V, W, X, Y and Z – are sitting around a circular table facing towards the centre, but not necessarily in the same order. Each of them likes different colour, viz, Red, Blue, Green, Yellow, Pink, Orange, Purple and Silver, but not necessarily in the same order.

    BPRE–295

    The person who likes Red colour is sitting third to the right of T. There are three persons between the persons who like Red colour and Purple colour respectively. T is an immediate neighbour of that person who likes Green colour. There are two persons between the person who likes Green colour and S. V is to the immediate left of Z. Z does not like Purple colour or Red colour. Z is an immediate neighbour of S. V does not like Green colour. U is sitting second to the right of Y. U does not like Green colour or Purple colour. W does not like Purple colour or Green colour. The person who likes Blue colour is an immediate neighbour of the person who likes Orange colour. The person who likes Orange colour is second to the left of S. The person who likes Silver colour is an immediate neighbour of both U and the person who likes Green colour. T does not like Yellow colour. 353. If all the eight persons are made to sit in alphabetical order in anticlockwise direction starting from S, positions of how many will remain unchanged (excluding S) ? (1) None (2) One (3) Two (4) Three (5) Four 354. What is the position of W with respect to S ? (1) Immediate left (2) Fourth to the right (3) Third to the left (4) Third to the right (5) Second to the left 355. Which of the following statements is/are true about Z ? (1) Z likes Blue colour (2) Z is sitting exactly between the person who likes Orange colour and S (3) Z is sitting third to the left of the person who likes Pink colour (4) Z is sitting just opposite to that person who likes Green colour (5) All the statements are true 356. Who among the following likes Purple colour ? (1) X (2) V (3) Z (4) T (5) Y

    RANKING/ARRANGEMENT 357. Who among the following is sitting exactly between W and the person who likes Pink colour ? (1) The person who likes Red colour (2) V (3) The person who likes Blue colour (4) The person who likes Green colour (5) None of these 358. Four of the following five pairs are alike in a certain way based on the above arrangement and so form a group. Which is the one that does not belong to that group ? (1) U, X (2) W, Z (3) S, W (4) T, V (5) Y, Z Directions (359–364) : Study the following information carefully and answer the questions given below : (SIDBI Officer Online Exam.24.02.2016) Eight persons – P, Q, R, S, T, U, V and W – are sitting around a circular area at equal distance between each other but not necessarily in the same order. Some people face the centre while some face outside (i.e. in a direction opposite to the centre). Q sits seconds to the right of T. T faces outside. Only two persons sit between Q and W. Both the immediate neighbours of Q faces the centre. Both the immediate neighbours of R face outside. R is neither an immediate neighbour of Q nor V. Only two persons sit between R and V. S sits second to the left of P. Immediate neighbours of W face same direction (i.e. If one neighbour faces the centre then the other also faces centre and vice–versa.) T sits to immediate left of one of his neighbours. W faces the same direction as R (i.e. if R faces the centre, then W also faces centre and vice–versa.) 359. What is R’s position with respect to W? (1) Second to the left (2) Second to the right (3) Fourth to the right (4) Fifth to the right (5) Third to the right

    360. Which of the following is true regarding U as per the given arrangement? (1) None of the given options is true (2) Only three persons sit between U and Q. (3) U faces the centre (4) S is one of the immediate neighbours of U. (5) U sits third to the left of T. 361. Who amongst the following sit exactly between R and V when counted from the right of R? (1) P, U (2) U, W (3) P, Q (4) Q, T (5) S, T 362. Four of the following five are alike in a certain way based on the given seating arrangement and so form a group. Which is the one that does not belong to that group? (1) W (2) P (3) U (4) Q (5) S 363. Who amongst the following are immediate neighbours of T ? (1) S, V (2) U, V (3) R, S (4) P, R (5) V, W 364. Who is sitting to the immediate left of V? (1) W (2) P (3) T (4) Q (5) S Directions (365–369) : Study the following information carefully and answer the questions given below : (SIDBI Officer Online Exam.24.02.2016)

    Nine persons – P, Q, R, S, T, U, V, W and X – are seated in a straight line facing north but not necessarily in the same order. T sits fourth to the left of W. Neither T nor W sits at the extreme ends of the line. Only one person sits between X and R. Neither X nor R is an immediate neighbour of W. P sits second to the right of X. Only two persons sit between P and S. Q is not an immediate neighbour of S or P. V is not an immediate neighbour of X.

    BPRE–296

    365. What is the position of V with respect to Q? (1) Fifth to the right (2) Immediate neighbour (3) Second to the right (4) Third to the right (5) Other than those given as options 366. Which of the following represents persons seated at two extreme ends of the line? (1) S, R (3) R, P

    (2) Q, S (4) X, Q

    (5) V, X 367. How many persons are seated between R and P? (1) One (2) Two (3) Three (4) Four (5) None 368. If T is related to V and Q is related to P in a certain way, to which of the following would X be related to following the same pattern? (1) Q (2) T (3) R (4) S (5) W 369. Who amongst the following sits exactly in the middle of the persons who sits seventh from the left and the person who sits sixth from the right? (1) P (2) Q (3) T (4) S (5) V Directions (370–372) : Study the following arrangement carefully and answer the questions given below : (SIDBI Officer Online Exam.24.02.2016)

    A$H★JRS2&!T6@QBKFG UMI©#%E4N59D3LW7 370. If all the letters are dropped from the arrangement, which of the following will be the ninth from the right end? (1) # (2) @ (3) © (4) 6 (5) None of these 371. Four of the following five are alike in a certain way based on their positions in the above arrangement and so form a group. Which is the one that does not belong to the group?

    RANKING/ARRANGEMENT (1) AJHS (2) G©M% (3) TB@F (4) 5DL7 (5) %54D 372. How many such numbers are there in the above arrangement, each of which is immediately preceded by a vowel and also immediately followed by a consonant? (1) None

    (2) One

    (3) Two

    (4) Three

    (5) More than three Directions (373–375) : Study the following information carefully and answer the questions given below : (United Bank of India PGDBF Manipal Exam,07.08.2016)

    Seven persons – S, T, U, V, W, X and Y – are standing in a straight line, but not necessarily in the same order. Some of them are facing south while some others are facing north. Only three persons are standing between U and V. Neither U nor V is standing at the any extreme end of the line. X is standing second to the right of U. Y is standing third to the right of X. Neither S nor Y is an immediate neighbour of V. T is standing second to the right of S. Both the immediate neighbours of X face opposite directions (i.e., if one neighbour faces north the other neighbour faces south and vice–versa). V faces a direction opposite to that of T. Both the persons standing at the extreme ends face the same directions as that of X. 373. Which of the following pairs represents the persons standing at the extreme ends of the line? (1) T, Y (2) S, T (3) S, Y (4) W, X (5) W, Y 374. What is the position of U with respect to T? (1) Second to the right (2) Third to the right (3) Third to the left (4) Immediate left (5) Second to the left 375. Four of the following five are alike in a certain way based on the given arrangement and hence they form a group. Which one of the following does not belong to that group? (1) W (2) T (3) V (4) X (5) Y

    Directions (376–380) : Study the following information carefully and answer the questions given below: (Bank of Baroda Exam, 25.09.2016)

    Eight friends, A, B, C, D, E, F, G and H are seated in a straight line with equal distance between each other, but not necessarily in the same order. Some of them are facing north while some are facing south. ● B sits fifth to the left of G. Neither B nor G sits at an extreme end of the line. ● A sits second to the right of B. ● Only two people sit between A and F. ● D sits third to the left of H. D is not an immediate neighbour of A. Neither H nor E is an immediate neighbour of B. ● Immediate neighbours of A face opposite directions (i.e., if one neighbour faces north then the other faces south and vice-versa). ● E faces north. Immediate neighbours of B face the same direction (i.e., if one neighbour faces north then the other also faces north and vice-versa). ● Both A and D face a direction opposite to that of F (i.e.. if F faces north then both A and D face south and vice-versa). ● C and E face opposite directions (i.e.. if C faces north then E faces south and vice-versa). 376. What is the position of F with respect to B? (1) Second to the left (2) Third to the right (3) Immediate left (4) Fourth to the left (5) Immediate right 377. Which of the following pairs represents the persons seated at the two extreme ends of the line? (1) E, H (2) F, D (3) F, E (4) C, A (5) D, C 378. Four of the following five are alike in a certain way based on the given arrangement and hence they form a group. Which one of them does not belong to that group? (1) FC (2) BF (3) EH (4) DE (5) AB 379. Based on the given arrangement, which of the following statements is true with respect to C?

    BPRE–297

    (1) B sits to the immediate left of C. (2) H is an immediate neighbour of C. (3) Only two people sit between C and F. (4) None of the given options is true (5) E sits third to the left of C. 380. Which of the following pairs sits, exactly between G and A? (1) C,F (2) H,C (3) F,B (4) H, E (5) B,E Directions (381–385) : Study the following information carefully and answer the questions given below : (IBPS Bank PO/MT CWE (Pre Exam), 16.10.2016 (First Sitting))

    Eight friends – P, Q, R, S, T, U, V and W – are seated in a straight line with equal distance between each other, but not necessarily in the same order. Some of them are facing north while some are facing south. ● V is an immediate neighbour of the person sitting at an extreme end of the line. R sits second to the left of V. ● Only one person sits between R and T. ● As many people sit to the right of T as to the left of P. V and P face the same direction (i.e., if V faces north then P also faces north and vice-versa.) ● Immediate neighbours of P face opposite directions (i.e. if one neighbour faces north then the other faces south and vice-versa.) ● Q sits fourth to the left of S. Q is not an immediate neighbour of V. ● Persons sitting at the extreme ends face opposite directions (i.e. if one person faces north then the other person faces south and viceversa.) ● W faces south. W does not sit at an extreme end of the line. U sits to the immediate right of W. ● U and Q face the same direction (i.e. if U faces north then Q also faces north and vice-versa.) 381. As per the given arrangement, which of the following statements is not true with respect to U ? (1) U sits at an extreme end of the line (2) Only three persons sit between U and R

    RANKING/ARRANGEMENT (3) U sits second to the left of T (4) All the given statements are true (5) U is an immediate neighbour of Q. 382. What is the position of Q with respect to W ? (1) Immediate left (2) Second to the left (3) Third to the left (4) Third to the right (5) Second to the right 383. Four of the following five are alike in a certain way based on the given arrangement and hence form a group. Which one of them does not belong to that group ? (1) WS (2) QT (3) WR (4) UP (5) RV 384. How many persons sit to the left of T ? (1) Two (2) None (3) More than three (4) One (5) Three 385. Which of the following represents the immediate neighbours of P ? (1) R, T (2) S, V (3) W, P (4) V, R (5) T, S Directions (386–390) : Study the following information carefully and answer the questions given below : (IBPS Bank PO/MT CWE-VI (Pre Exam), 16.10.2016 (Second Sitting))

    Eight persons — A, B, C, D, W, X, Y and Z — are sitting around a circular table facing the centre at equal distance from each other, but not necessarily in the same order. Each of them belongs to different areas viz, Dwarka, Karol Bagh, Okhla, Chanakyapuri, Mehrauli, Rohini, Lajpat Nagar and Saket, but not necessarily in the same order. W is sitting third to the left of Y. The person who is from Dwarka is to the immediate right of W and W is not from Okhla. B is sitting fourth to the right of Z. Z is not the immediate neighbour of Y. Neither B nor Z is an immediate neighbour of W. X is from Chanakyapuri and is sitting third to the right of the person from Dwarka. The person from Mehrauli is sitting second to the left of person from Chanakyapuri. The person from Rohini is sitting second to the left of W. A who is from

    Lajpat Nagar is sitting exactly between X and Z. The person from Saket is sitting second to the right of the person from Lajpat Nagar. C is sitting third to the left of X. 386. Who amongst the following persons belongs to Okhla? (1) D (2) Y (3) C (4) B (5) Z 387. What is A’s position with respect to B? (1) Third to the right (2) Second to the right (3) Third to the left (4) Second to the left (5) Fourth to the right 388. How many people are sitting between Z and C when counted in an anticlockwise direction from C? (1) One (2) Two (3) Three (4) Four (5) None 389. Four of the following five pairs are alike in a certain way based on their positions in the above arrangement and so form a group. Which of the following does not belong to the group? (1) B-Rohini (2) Z – Mehrauli (3) D – Okhla (4) Y – Saket (5) X - Dwarka 390. Which of the following statements is false according to the above mentioned arrangement? (1) C is to the immediate right of the person from Karol Bagh (2) The person from Lajpat Nagar is third to right of the person from Mehrauli. (3) The person from Dwarka is sitting exactly between the persons from Karol Bagh and Saket. (4) D is neither from Chanakyapuri nor from Karol Bagh. (5) There are only three people between A and C. Directions (391–395) : Study the following information carefully and answer the questions given below : (IBPS Bank PO/MT CWE-VI (Pre Exam), 16.10.2016 (Second Sitting))

    Eight persons S, T, U, V, W, X, Y and Z are sitting in a straight line equidistant from each other (but not necessarily in the same order). Some of them are facing south while some are facing north.

    BPRE–298

    (Note : Facing the same direction means, if one is facing north then the other also faces north and vice-versa. Facing the opposite directions means, if one is facing north then the other faces south and vice-versa) S faces north. Only two people sit to the right of S. T sits third to the left of S. Only one person sits between T and X. X sits to the immediate right of W. Only one person sits between W and Z. Both the immediate neighbours of T face the same direction. U sits third to the left of X. T faces the opposite direction as S. Y does not sit at any of the extreme ends of the line. V faces the same direction as W. Both Y and U face the opposite direction of Z. 391. How many persons in the given arrangement are facing North? (1) More than four (2) Four (3) One (4) Three (5) Two 392. Four of the following five are alike in a certain way, and so form a group. Which of the following does not belong to the group? (1) W, X (2) Z, Y (3) T, S (4) T, Y (5) V, U 393. What is the position of X with respect to Z ? (1) Second to the left (2) Third to the left (3) Third to the right (4) Fifth to the right (5) Second to the right 394. Who amongst the following sits exactly between Z and W? (1) T (2) Y (3) X (4) W (5) U 395. Who is sitting second to the right of T? (1) Z (2) X (3) W (4) None of these (5) V Directions (396–400) : Study the following information carefully and answer the questions given below : (IBPS Bank PO/MT (Pre.) Exam, 23.10.2016)

    Eight friends – G, H, I, J, K, L, M and N — are seated in a straight line with equal distance between each other, but not necessarily in the same order. Some of them are facing north while some are facing south. ● K is an immediate neighbour of the person sitting at an extreme

    RANKING/ARRANGEMENT













    396.

    397.

    398.

    399.

    end of the line. Only three persons sit between K and M. J sits second to the right of M. J does not sit at an extreme end of the line. N sits to the immediate left of G. N is not an immediate neighbour of M. Immediate neighbours of G face opposite directions (i.e. if one neighbour faces north then the other faces south and viceversa.) Persons sitting at the extreme ends face opposite directions (i.e. if one person faces north then the other faces south and viceversa.) H sits second to the left of L. L faces north. L is not an immediate neighbour of K. Immediate neighbours of L face the same direction (i.e. if one neighbour faces north then the other also faces north and viceversa.) Both K and H face a direction opposite to that of J (i.e. if J faces north then K and H faces south and vice-versa.) As per the given arrangement, which of the following statements is true with respect to I? (1) I sits at an extreme end of the line. (2) K is an immediate neighbour of I. (3) Only four persons sit between I and M. (4) I faces a direction opposite to that of J. (5) All the given statements are true. Which of the following pairs represents the immediate neighbours of J? (1) L, I (2) H, L (3) L, K (4) I, K (5) H, K What is the position of K with respect to L? (1) Immediate right (2) Second to the right (3) Third to the left (4) Third to the right (5) Immediate left Four of the following five are alike in a certain way based on the given arrangement and hence form a group. Which of them does not belong to that group?

    (1) JM (2) HL (3) HI (4) NL (5) KJ 400. How many persons sit to the left of G? (1) One (2) Two (3) More than three (4) None (5) Three Directions (401–405) : Study the following information carefully and answer the questons given below : (Bank of Maharashtra PO Exam, 26.10.2016)

    Eight friends — A, B, C, D, E, F, G and H — are sitting around a circular table with equal distance between them but not necessarily in the same order. Some of them are facing the centre while some face outside. (i.e. opposite to centre). ● D sits third to the left of H. H faces the centre. E sits second to the right of D. ● Only one person sits between E and A. Immediate neighbours of A face opposite directions (i.e. if one neighbour faces the centre then the other neighbour faces outside and viceversa). ● Only three people sit between B and G. B is not an immediate neighbour of A. ● F sits second to the left of B. Immediate neighbours of F face the same direction (i.e. if one neighbour faces the centre then the other also faces centre and vice-versa). ● Immediate neighbours of C face opposite directions (i.e. if one neighbour faces the centre then the other neighbour faces outside and vice-versa). ● Both E and A face a direction same as that of D (i.e. if D faces the centre then both E and A faces the centre and viceversa). 401. Which of the following statements is true regarding C as per the given arrangement ? (1) None of the given options is true (2) Only two people sit between C and E. (3) A sits second to the left of C. (4) G faces the same direction same as that of C. (5) Both G and D are immediate neighbours of C.

    BPRE–299

    402. How many people sit between F and H when counted from the left of H ? (1) None (2) Three (3) More than three (4) One (5) Two 403. Who sits third to the right of E ? (1) C (2) D (3) B (4) G (5) F 404. What is the position of G with respect to A ? (1) Second to the right (2) Third to the right (3) Immediate left (4) Second to the left (5) Immediate right 405. Four of the following five are alike in a certain way based on the given arrangement and so form a group. Which is the one that does not belong to that group ? (1) G (2) D (3) F (4) C (5) E Directions (406–411) : Study the following information carefully and answer the questions given below : (Bank of Maharashtra PO Exam, 26.10.2016)

    Eight people viz. L, M, N, O, P, Q, R and S are sitting on eight different steps of the same flights of stairs, but not necessarily in the dame order. The lowermost step is numbered one, the one above that is numbered two and so on till the topmost step is numbered eight. Each one of them has different years of experience in the field of banking viz., 10 years, 13 years, 15 years, 18 years, 20 years, 23 years, 25 years and 28 years, but not necessarily in the same order. R is the most experienced and is sitting on an even numbered step but not on step number 2. The one who has less experience than only R is sitting on the lowermost step. P is sitting on a step which is immediately below the step on which R is sitting. L is the least experienced. L is sitting on a step which is immediately below the step on which N is sitting. Only three people sit between L and S. S has 20 years of experience. O has more experience than only L. Q is sitting on a step which is immediately below O’s step. The one who has 23 years of experience is sitting on a step which is immediately above the step on which S is sitting. N has more experience than P.

    RANKING/ARRANGEMENT 406. Which of the following statements is/are true as per the given arrangement ? (1) S is sitting on the step number 6 (2) M is sitting on the step number 2 (3) None of the given statements is true (4) N has 18 years of experience (5) Q has 15 years of experience 407. How many people are sitting between the one who has 15 years of experience and M ? (1) Two (2) None (3) Three (4) One (5) Four 408. Who amongst the following is sitting on step number 3 ? (1) Q (2) L (3) M (4) N (5) S 409. Four of the following five are alike in a certain way based on the given arrangement and hence form a group. Which is the one that does not belong to that group ? (1) Step number 3 – Q (2) R — Step number 8 (3) N — Step number 2 (4) O — M (5) Step Number 5 – S 410. Which of the following statements is true with respect to O ? (1) S sits immediately below O (2) Only two persons sit between O and S (3) None of the given statements is tue (4) R sits below O (5) O is sitting on step number 2 411. How much experience does M has ? (1) 13 years (2) 23 years (3) 15 years (4) 18 years (5) 25 years Directions (412–417) : Study the following informaton carefully and answer the questions given below : (IBPS RRBs Officer CWE (Pre.) Exam, 14.11.2016 (Shift-I))

    Twelve people are sitting in two parallel rows containing six people each, in such a way that there is equal distance between adjacent persons. In row 1-G, H, I, J, K and L are seated (but not necessarily in the same order) and all of them are facing south. In

    row 2-R, S, T, U, V and W are seated (but not necessarily in the same order) and all of them are facing north. Therefore, in the given seating arrangement, each member seated in a row faces another member of the other row. I sits third to the left of the person facing V. Neither I nor G sits at any of the extreme ends of the line. Only one person sits between I and G. W sits second to the right of T. T is not an immediate neighbour of V. T does not face I. The one who faces K sits to the immediate left of R. K is not an immediate neighbour of G. Only one person sits between L and the one who faces U. J is not an immediate neighbour of I. 412. What is the position of T with respect to V ? (1) Third to the left (2) Third to the right (3) Second to the right (4) Immediate right (5) Second to the left 413. Who amongst the following sits exactly between L and the one who faces U ? (1) G (2) The one who faces S (3) J (4) The one who faces R (5) K 414. Four of the following five are alike in a certain way based on the given arrangement and hence form a group. Which of them does not belong to that group ? (1) W (2) V (3) L (4) J (5) R 415. Which of the following statements is tue regarding H ? (1) None of the given statements is true (2) H sits third to the right of I. (3) H is an immediate neighbour of K (4) Only two persons sit between H and L (5) H faces one of the immediate neighbours of R 416. Who amongst the following faces K? (1) R (2) W (3) T (4) S (5) V 417. In a queue of fifteen people facing north, Rahul’s position is ninth from the end of the queue. Only four people are standing be-

    BPRE–300

    tween Rahul and Tom, Karan is standing immediately after Tom. Bharti is standing exactly between Karan and Rahul. Sonali is standing before Bharti but after Tom. What is position of Sonali from the beginning of the queue? (Note : All people are standing one behind the other) (1) Cannot be determined (2) Seventh (3) Eleventh (4) Fourth (5) Sixth (Bank of Maharashtra PO Exam, 26.10.2016)

    Directions (418–422) : Study the following information carefully and answer the questions given below : (IBPS RRBs Officer CWE (Pre.) Exam, 14.11.2016 (Shift-I))

    Eight persons–A, B, C, D, M, N, O and P – are sitting around a square table (but not necessarily in the same order) in such a way that four of them sit at four corners while four sit in the middle of each of the four sides. The ones sitting in the middle of the sides are facing the centre and the ones sitting at the corners are facing outside (i.e. opposite to the centre). A sits in the middle of one of the sides. Only one person sits between A and M. A sits third to the right of B. Only three persons sit between B and N. C sits second to the right of N. O and C face the same direction. M is not an immediate neighbour of O. P sits second to the right of D. 418. Who is to the immediate left of M? (1) N (2) B (3) D (4) P (5) C 419. What is the position of O with respect to P? (1) Fourth to the left (2) Third to the left (3) Third to the right (4) Immediate right (5) Immediate left 420. Which of the following statements is true with respect to the given arrangement? (1) None of the given statements is true (2) Only three people sit between O and D. (3) D sits third to the left of A. (4) M sits at one of the corners of the table. (5) M is an immediate neighbour of P.

    RANKING/ARRANGEMENT 421. As per the given arrangement, four of the following five are alike in a certain way and hence form a group. Which of the following does not belong to the group? (1) D (2) A (3) P (4) O (5) M 422. Who amongst the following sit exactly between B and the one who sits to the immediate right of N, when counted from the left of B? (1) O, D (2) M, P (3) C, P (2) M, C (5) A, C Directions (423–427) : Study the following information carefully and answer the questions given below :

    425. Who sits second to the left of H? (1) N (2) K (3) F (4) E (5) No one as H sits at one of the extreme ends of the line 426. Who amongst the following sit exactly between N and O? (1) H, L (2) M, K (3) F, K (4) F, G (5) K, E 427. In which of the given pairs of people, is even number of people sitting between them? (1) G, E (2) K, L (3) F, M (4) O, M (5) K, N Directions (428–433) : Study the following information carefully and answer the questions given below :

    (IBPS RRBs Officer CWE (Pre.) Exam, 14.11.2016 (Shift-I))

    (IBPS RRBs Officer CWE (Pre.) Exam, 14.11.2016 (Shift-II))

    Nine persons – E, F, G, H, K, L, M, N and O – are seated in a straight line facing north, with equal distance between each other but not necessarily in the same order. Only two persons sit between E and the one sitting at extreme ends of the line. K sits second to the right of E. H sits fourth to the left of M. M does not sit at any of the extreme ends of the line. M is not an immediate neighbour of E. The number of people sitting between H and K is double than that between M and O. More than two people sit between G and E. G is not an immediate neighbour of M. F is an immediate neighbour of L but not H. 423. Four of the following five are alike in a certain way based on the given arrangement and thus form a group. Which is the one that does not belong to that group? (1) N, E (2) M, H (3) K, L (4) O, N (5) F, O 424. Which of the following statements is true with respect to N as per the given arrangement? (1) N is an immediate neighbour of H. (2) N sits second to the left of M. (3) N sits at one of the extreme ends of the line. (4) More than two people sit between N and G. (5) None of the given options is true

    Eight persons — E, F, G, H, Q, R, S and T — are sitting around a square table (but not necessarily in the same order) in such a way that four of them sit at four corners while four sit in the middle of each of the four sides. The ones sitting at the corners are facing the centre and the ones sitting in the middle of the sides are facing outside (i.e. opposite to the centre). Q sits in the middle of one of the sides. Only two persons sit between Q and R. Only one person sits between R and E. E is an immediate neighbour of both T and F. S sits to the immediate left of F. Only one person sits between G and S. 428. Who sits third to the left of G? (1) H (2) T (3) F (4) R (5) E 429. What is the position of T with respect to R? (1) Second to the right (2) Third to the left (3) Immediate left (4) Third to the right (5) Immediate right 430. How many persons sit between T and F when counted from the left of T? (1) One (2) None (3) Three (4) Two (5) More than three 431. Which of the following statements is true with respect to the given arrangement? (1) Only three persons sit between E and Q.

    BPRE–301

    (2) Q is an immediate neighbour of S. (3) F sits second to the left of H (4) H sits at one of the corners of the table (5) None of the given statements is true 432. Which of the following pairs represent the persons sitting between Q and the one sitting second to the left of E, when counted from the left of Q? (1) F, S (2) T, S (3) F, G (4) H, G (5) R, T 433. Among five people — A, B, C, D and E — each scoring different marks, only two persons scored more marks than A. D scored more than A. B scored less than D but not the lowest. C scored more than B but not the highest. Who amongst them did score the lowest marks ? (1) Cannot be determined (2) B (3) E (4) C (5) D (IBPS RRBs Officer CWE (Pre.) Exam, 14.11.2016 (Shift-II))

    Directions (434–439) : Study the following information carefully and answer the questions given below : (IBPS RRBs Officer CWE (Pre.) Exam, 14.11.2016 (Shift-II))

    Nine persons – G, H, I, J, K, R, S, T and U — are seated in a straight line facing North, with equal distance between each other but not necessarily in the same order. Only two persons sit to the left of I. Only one person sits between I and U. H sits fourth to the right of R. R is not an immediate neighbour of U. Less than three persons sit between R and U. Number of persons sitting between I and U is half as that between H and J. Only three persons sit between K and T. K is not an immediate neighbour of J. Only two persons sit between T and G. 434. In which of the given pairs of persons, is odd number of persons sitting between them? (1) H, T (2) I, H (3) U, R (4) J, K (5) G, K 435. Which of the following statements is true with respect to S as per the given arrangement? (1) S is an immediate neighbour of U.

    RANKING/ARRANGEMENT

    436.

    437.

    438.

    439.

    (2) None of the given options is true. (3) S sits at one of the extreme ends of the line. (4) More than two persons sit between S and R. (5) S sits second to the left of G. Who amongst the following sit exactly between T and G? (1) I, J (2) H, J (3) R, U (4) H, S (5) J, R Who sits second to the left of J? (1) T (2) No one as J sits at one of the extreme ends of the line. (3) I (4) G (5) S Which of the following pairs represent the persons sitting at the extreme ends of the line ? (1) H, U (2) U, G (3) K, G (4) U, S (5) I, G Among five people – A, B, C, D and E — each scoring different marks, only one person scored less marks than B. D scored more marks than B but less than A. A did not score the highest marks. Who scored the second highest marks? (1) E (2) Cannot be determined (3) A (4) C (5) D (IBPS RRBs Officer CWE (Pre.) Exam, 14.11.2016 (Shift-III))

    Directions (440–444) : Study the following information carefully and answer the questions given below : (IBPS RRBs Officer CWE (Pre.) Exam, 14.11.2016 (Shift-III))

    Nine persons, B, C, D, E, L, M, N, O and P are seated in a straight line facing north, with equal distance between each other, but not necessarily in the same order. As many people sit to the left of E as to the right of E. Only one person sits between E and O. L sits third to the left of P. P is not an immediate neighbour of O. Neither P nor D sits at any of the extreme ends of the line. Only three persons sits between B and C. B is not an immediate neighbour of E. The number of people sitting between B and E is double as that between E and N.

    440. Who sits third to the right of B? (1) O (2) E (3) N (4) L (5) No one as B sits at one of the extreme ends of the line 441. Four of the following five are alike in a certain way based on the given arrangement and thus form a group. Which is the one that does not belong to that group? (1) C, E (2) M, D (3) O, C (4) B, L (5) D, B 442. Which of the following statements is true with respect to M as per the given arrangement? (1) M sits second to the left of O. (2) M is an immediate neighbour of L. (3) More than two people sit between D and M. (4) M sits at one of the extreme ends of the line. (5) None of the given options is true 443. Who amongst the following sit exactly between L and P? (1) E, N (2) M, O (3) B, N (4) C, O (5) D, E 444. In which of the given pairs of people, is odd number of people sitting between them? (1) E, M (2) B, M (3) L, C (4) L, P (5) P, O Directions (445–449) : Study the following information carefully and answer the questions given below :

    (1) Third to the left (2) Second to the left (3) Immediate left (4) Immediate right (5) Second to the right 446. Which of the following pairs represent the people sitting between O and the one sitting to the immediate right of P, when counted from the left of O? (1) N, S (2) R, N (3) S, M (4) T, Q (5) R, T 447. How many people sit between T and M when counted from the right of T? (1) More than three (2) Two (3) One (4) Three (5) None 448. Which of the following statements is true with respect to the given arrangement? (1) None of the given statements is true (2) Q is an immediate neighbour of T. (3) S sits at one of the corners of the table (4) Only two people sit between M and Q. (5) R and N face opposite directions 449. Who sits third to the left of O? (1) R (2) M (3) P (4) N (5) S Directions (450–455) : Read the following information carefully and answer the questions given below :

    (IBPS RRBs Officer CWE (Pre.) Exam, 14.11.2016 (Shift-III))

    (IBPS Bank PO/MT CWE (Main) Exam, 18.11.2016

    Eight people – M, N, O, P, Q, R, S and T — are sitting around a square table (but not necessarily in the same order) in such a way that four of them sit at the corners while four sit in the middle of each of the four sides. The ones sitting at the corners are facing the centre and the ones sitting in the middle of the sides are facing outside. (i.e. opposite to the centre) O sits in the middle of one of the sides. Only two people sit between O and R. T sits to the immediate right of R. T and S face the same direction. N sits to the immediate left of S. P is an immediate neighbour of N. M sits third to the right of P. 445. What is the position of P with respect to T?

    Ten persons — Anupam, Ankur, Abhisar, Abhijeet, Anoop, Abhigyan, Animesh, Abhinav, Ankit and Abhiskek are seated in two parallel rows with equal distance and facing each other but not necessarily in the same order. The persons seated in row I are facing South while the persons seated in row II are facing the North. Among them, each of the persons like different games — Chess, Cricket, Kabbadi, Tennis, Kho-kho, Wrestling Carrom, Football, Badminton and Hockey. Each of them are wearing shirts of different colours — White, Blue, Brown, Green, Violet, Black, Red, Sky-blue, Yellow and Rose but not necessarily in the same order. All are staying in a building having ten floors. They stay on dif-

    BPRE–302

    RANKING/ARRANGEMENT ferent floors but not necessarily in the same order. The ground floor is numbered as first floor and the immediate upper floor is numbered as second floor and so on till the topmost floor is numbered ten. Ankur is sitting in front of the neighbour of the person wearing green shirt. The person who likes khokho is sitting in the middle of the row and he is sitting in front of a person who is wearing Red shirt and he stays in the floor having even number. Only one person is staying in between the floors of Animesh and him. Ankit stays in the immediate upper floor of Anupam. Ankit stays in the floor having even number and a person wearing sky-blue shirt is sitting immediate right to him. Abhigyan is sitting in front of the neighbour of the person wearing yellow shirt and he stays in the floor having odd number. Only two persons are sitting in between Animesh and Ankur. Ankur does not stay in the ground (first) floor. Only one person is sitting between Ankur and the person wearing Red shirt. Anupam does not like cricket. Anupam does not stay either on third or the fifth floor. Two persons are staying between Anupam and the person who likes Badminton. Abhishek is sitting in front of the person wearing blue shirt. Abhishek does not like kho-kho. The person who is not wearing violet colour shirt is sitting in front of Animesh. Anupam does not like wrestling. The person wearing Rose coloured shirt is sitting to the right of the person wearing violet shirt. The person wearing sky-blue colour shirt is facing North and he stays on the first floor. Abhijeet is not wearing skyblue colour shirt and an immediate neighbour of Animesh is sitting in front of him. Only one person in that building is staying in between the floors of the persons who like badminton and chess. The person who likes wrestling stays in the immediate lower floor of the person who likes Badmintion. Animesh is the immediate neighbour of Abhinav and Ankit is the immediate neighbour of Abhishek. The person wearing green shirt is the immediate neighbour of Abhijeet. Anoop stays in the immediate upper floor of Abhisar, only one person is staying in between the floors of the person who likes tennis and Abhisar. Abhigyan is sitting in front of the person wearing brown shirt. Abhigyan likes carrom and he is not wearing white colour shirt. Two

    persons are sitting in between Abhigyan and the person wearing brown shirt. Abhijeet is not sitting next to Abhishek. Ankit is sitting in front of the person wearing black shirt. Abhijeet is not the immediate neighbour of the person wearing brown colour shirt. Anoop is not sitting in front of the person wearing sky-blue shirt. Abhijeet is sitting to the right of the person wearing Rose shirt. The person who likes football is staying in the floor immediate upper to the person who likes Hockey. Two persons are staying in between Anupam and the person who likes Badminton. Ankur is wearing a blue shirt. 450. Who among the following is sitting third to the left of Abhishek ? (1) Animesh (2) Abhigyan (3) Ankit (4) Abhisar (5) None of these 451. Which among the following combinations is sitting at the two ends of any row? (1) Abhishek, Anupam (2) Abhigyan, Animesh (3) Anupam, Abhijeet (4) Ankur, Abhinav (5) None of these 452. Who among the following does like wrestling ? (1) Abhisar (2) Ankit (3) Animesh (4) Ankur (5) None of these 453. Which among the following games are liked by the immediate neighbours of the person wearing black shirt ? (1) Tennis, Badminton (2) Badminton, Chess (3) Chess, Kabbadi (4) Tennis, Kabbadi (5) None of these 454. Based on the given arrangement, there is same relation among Abhishek-Badminton-Black shirt and Chess-Blue-Ankit. How is pink related to in the same way ? (1) Kho-Kho-Abhigyan (2) Abhisar-Kho-Kho (3) Tennis-Anupam (4) White-Football (5) None of these 455. Who among the following is sitting in the middle of the row facing south ? (1) Person wearing yellow shirt (2) Person who stays on the third floor (3) Person who likes cricket

    BPRE–303

    (4) Person who likes badminton (5) Other than the given option. Directions (456–460) : Read the following information carefully and answer the questions given below : (IBPS Bank PO/MT CWE (Main) Exam, 18.11.2016

    Ten persons — A, B, C, D, E, U, W, X, Y and Z — are sitting around a circular table facing the centre and facing each other but not necessarily in the same order. Each of them is related to D in some way or the other. Only two persons are sitting between X and C. The wife of D is sitting third to the right of B. The father of D is not the immediate neighbour of D. C is sitting to the right of X. Only three persons are sitting in between C and the sister of D. A is sitting immediate right to Y. D is sitting second to the the left of X. The son of D is sitting second to the right of D’s sister. Only one person is sitting between Z and the son of D. Y is neither the son nor mother of D. Z is the immediate neighbour of the mother of D. Only three persons are sitting between the mother and brother of D. The daughter of D is sitting second to the left of D’s brother. Four persons are sitting between D and the father of D. 456. How many persons are sitting between B and the mother of B when counted to the right of B? (1) Five (2) Two (3) Six (4) Data not sufficient (5) None of these 457. How is C related to X ? (1) Husband (2) Wife (3) Son (4) Nephew (5) Data not sufficient 458. Who is sitting to the immediate left of Y? (1) A (2) B (3) D (4) Data are inadequate (5) None of these 459. Who is the brother of D ? (1) A (2) B (3) Y (4) Cannot be determined (5) None of these 460. Which of the following statements is NOT TRUE? (1) Z is the aunt of B (2) D has the same relation with A as C has with X.

    RANKING/ARRANGEMENT (3) X is sitting immediate left to her daughter (4) The daughter of D is sitting third to the right of D. (5) None of these Directions (461–465) : Study the following information carefully and answer the questions given below : (IBPS SO (Agriculture) Exam, 29.01.2017)

    Eight family members D, E, F, G, H, I, J and K are sitting around a circular table but not necessarily in the same order. Some of them are females and some are males. All of them are related to each other in the same way or the other. Some of them are facing the centre while some are facing outside (i.e. opposite to the centre.) Only two people sit between E and H. E faces the centre. I sits second to the right of E. H is the wife of D. No females is an immediate neighbour of H. F is not an immediate neighbour of E. F is the daughter of H. Both the immediate neighbours of F face the centre. Only three people sit between D and F’s brother. I is not the brother of F. Neither D nor F’s brother is an immediate neighbour of I. K, the wife of E, sits to the immediate left of G. Both J and D face a direction opposite to that of F (i.e. if F faces the centre then both J and D face outside and vice-versa). U’s husband sits second to the left of J. E’s father sits to the immediate right of H. E sits second to the right of D’s father. Both the immediate neighbours of I are females. 461. How many people sit between E and D’s father when counted from the right of E ? (1) Four (2) Three (3) None (4) One (5) Two 462. Who amongst the following sits exactly between J and H when counted from the right of J? (1) E (2) I (3) D (4) K (5) F 463. Which of the following statements regarding E is definitely true? (1) I and K are immediate neighbours of E. (2) E sits second to the left of I. (3) E is the son of D. (4) None of the given options is correct (5) G is the father of E.

    464. Who amongst the following faces outside (i.e. opposite to the centre)? (1) F (2) G (3) H (4) K (5) E 465. If it is given that J is married to I, then whatis the position of E with respect to J’s daughter-inlaw? (1) Third to the right (2) Second to the right (3) Immediate right (4) Second to the left (5) Third to the left Directions (466–467) : The following questions are based on the five three-digit numbers :

    (A) Fourth to the right (B) Fourth to the left (C) Third to the right (1) Only (A) (2) Only (B) (3) Only (C) (4) Either (A) or (B) (5) None of these 470. In which of the following pairs is the second person sitting second to left of the first person ? (1) RT (2) RW (3) QR (4) PS (5) WZ Directions (471–476) : Study the following information carefully and answer the questions given below :

    (IBPS SO (Agriculture) Exam, 29.01.2017)

    Eight persons – A, B, C, D, E, F, G and H – are sitting around a circular table at equidistance facing towards the centre but not necessarily in the same order. Each one of them also likes different subjects, viz., English, Physics, Mathematics, Chemistry, Biology, History, Hindi and Geography, but not necessarily in the same order. The one who likes Biology sits third to the right of B. Only one person sits between the persons who like Biology and Hindi respectively. The one who likes Hindi is not an immediate neighbour of B. E sits to the immediate right of C. C does not like Biology or Hindi. E likes neither Biology nor Hindi. Only three persons sit between the persons who like Hindi and Chemistry respectively. A sits third to the right of D. A is not an immediate neighbour of B. Only one person sits between A and the one who likes Mathematics. The person who likes Mathematics is an immediate neighbour of the one who likes History. H is an immediate neighbour of D. The one who likes Physics sits second to the left of H, but he is not an immediate neighbour of C. H does not like English. G does not like Hindi. 471. Who among the following does like Hindi? (1) B (2) H (3) C (4) F (5) E 472. What is the position of A with respect to E? (1) Third to the left (2) Second to the left (3) Third to the right (4) Second to the right (5) Immediate left

    458 374 654 487 568 466. If in each of the numbers the positions of the first and the third digits are interchanged and then the numbers so obtained are arranged in descending order from the left to right, which number will be at the fourth position ? (1) 654 (2) 487 (3) 458 (4) 374 (5) 568 467. If in each of the numbers the positions of the first two digits are interchanged and then the numbers so obtained are ar ranged in ascending order from left to right, which number will be at the second position ? (1) 654 (2) 458 (3) 568 (4) 487 (5) 374 Directions (468–470) : Study the following information carefully and answer the questions given below : (IBPS SO (Agriculture) Exam, 29.01.2017)

    P, Q, R, S, T, U, V, W and Z are sitting around a circle facing at the centre. R is third to the right of Z who is second to the right of P. S is not an immediate neighbour of Z and R. T is third to the left of S. Q is third to the right of W who is not an immediate neighbour of S. 468. Which of the pair of persons are the immediate neighbours of P ? (1) VQ (2) VW (3) VS (4) SR (5) None of these 469. What is Q’s position with respect to Z ?

    BPRE–304

    (United Bank of India PGDBF Manipal Exam,07.08.2016)

    RANKING/ARRANGEMENT 473. Four of the following five are alike in a certain way based on the given arrangement and hence they form a group. Which one of the following does not belong to that group? (1) A – Geography (2) C – English (3) B – Physics (4) E – Chemistry (5) F – Biology 474. Which of the following statements is true regarding D as per the given arrangement? (1) The one who likes History sits second to the left of D (2) D sits just opposite to the one who likes Hindi (3) D likes Chemistry (4) D sits third to the right of the one who likes Mathematics (5) All the statements are true 475. As per the given arrangement ‘F’ is related to the one who likes Chemistry and ‘E’ is related to the one who likes Biology following a certain pattern. With which of the following ‘C’ is related following the same pattern? (1) The one who likes Mathematics (2) The one who likes Geography (3) The one who likes English (4) The one who likes Physics (5) The one who likes History 476. How many persons are sitting between H and E when counted from the left of H? (1) Two (2) Three (3) None (4) One (5) More than three Directions (477–481) : Study the following information carefully and answer the questions given below : (Bank of Baroda Exam, 25.09.2016)

    Eight people M, N, O, P, Q, R, S and T are sitting around a circular table facing the centre with equal distance between each other, but not necessarily in the same order. Each person is dressed as a different cartoon character — Scrooge, Dexter, Minion, Nobita, Oswald, Popeye, Yogi and Simpson, but not necessarily in the same order. S sits second to the left of the one who is dressed as Yogi. Only two people sit between the one who is dressed as Yogi and the one who is dressed as Scrooge. O sits to the immediate left

    of the one who is dressed as Scrooge. Only one person sits between O and the one who is dressed as Dexter. S neither is dressed as Dexter nor Simpson. N sits third to the left of the one who is dressed as Simpson. N is neither dressed as Dexter nor Yogi. M sits to the immediate left of the one who is dressed as Simpson. Only three people sit between M and the one who is dressed as Nobita. Q sits second to the left of the one who is dressed as Nobita. T and P are immediate neighbours of each other. T is not dressed as Nobita. The one who is dressed as Oswald sits to the immediate right of P. M is not dressed as Popeye. 477. Which of the followingiis true with respect to the given arrangement? (1) M is dressed as Minion. (2) Only two people sit between R and the one who is dressed as Dexter. (3) The one who is dressed as Popeye sits to the immediate right of O. (4) None of the given options is true. (5) The one who is dressed as Yogi and Q are immediate neighbours of each other 478. How many people sit between M and the one who is dressed as Scrooge, when counted from the right of M? (1) Four (2) One (3) Two (4) More than four (5) None 479. Who amongst the following is dressed as Oswald? (1) T (2) R (3) Q (4) M (5) S 480. Who sits third to the right of the one who is dressed as Minion? (1) The one who is dressed as Scrooge (2) M (3) The one who is dressed as Oswald (4) T (5) Q 481. In the given arrangement, T is related to the one who is dressed as Popeye in the same way as O is related to the one who is dressed as Nobita. Following the same pattern, to who is N related ?

    BPRE–305

    (1) The one who is dressed as Dexter (2) The one who is dressed as Oswald (3) The one who is dressed as Simpson (4) The one who is dressed as Minion (5) The one who is dressed as Yogi Directions (482–486) : Study the following information carefully and answer the questions given below : (IBPS Bank PO/MT CWE (Pre Exam), 16.10.2016 (First Sitting))

    Eight people viz, C, D, E, F, W, X, Y and Z are sitting around a circular table facing the centre but not necessarily in the same order. Each one of them is wearing a watch of a different brand viz, Titan, Rado, Casio, Tissot, Rolex, Swatch, Omega and Longines but not necessarily in the same order. Only two people sit between the one wearing Rado and X. The one wearing Tissot sits second to the left of X. Only three people sit between the one wearing Rado and W. The one wearing Casio sits second to the right of the one wearing Swatch. Neither X nor W is wearing Swatch. The one wearing Swatch is not an immediate neighbour of the one wearing Tissot. Z is not wearing Tissot. The one wearing Titan sits to the immediate right of Z. C is an immediate neighbour of one wearing Titan. Only three people sit between C and Y. Only three people sit between F and the one wearing Omega. Neither F nor E is wearing Rado. Only one person sits between the ones wearing Omega and Rolex. 482. Who amongst the the following sits to the immediate left of the one wearing Rado ? (1) C (2) The one wearing Omega (3) The one wearing Swatch (4) Z (5) D 483. Who amongst the following is wearing Longines ? (1) X (2) Y (3) D (4) E (5) Z 484. Four of the following five are alike in a certain way based on the given arrangement and thus form a group. Which is the one that does not belong to that group ?

    RANKING/ARRANGEMENT (1) Z-Rado (2) E-Longines (3) X-Rolex (4) W-Swatch (5) C-Titan 485. Which of the following represents the brand of watch worn by E ? (1) Casio (2) Omega (3) Longines (4) Rolex (5) Swatch 486. Who amongst the following sit exactly between X and the one wearing Rado when counted from the right of X ? (1) The ones wearing Tissot and Titan (2) Z and the one wearing Longines (3) D and F (4) E and Z (5) E and the one wearing Swatch Directions (487–491) : Study the following information carefully and answer the questions given below : (IBPS Bank PO/MT (Pre.) Exam, 23.10.2016)

    Eight different persons viz. K, L, M, N, V, W, X and Y are sitting around a circular table facing the centre but not necessarily in the same order. Each one of them likes a different brand of clothes viz. Nautica, Chemistry, Zara, Mango, Puma, Adidas, Zodiac and Park Avenue but not necessarily in the same order. Only three persons sit between X and the one who likes Zodiac. V sits second to the right of X. The one who likes Nautica sits third to the left of Y. Y does not like Zodiac. The one who likes Zodiac is not an immediate neighbour of Y. Only three persons sit between Y and the one who likes Chemistry. W does not like Chemistry. The one who likes Puma sits to the immediate left of K. K is not an immediate neighbour of V. Only two persons sit between the ones who like Puma and Zara. M is one of the immediate neighbours of the one who likes Zara. The one who likes Park Avenue sits to the immediate right of L. Only three persons sit between L and the one who likes Adidas. 487. Four of the following five are alike in a certain way based on the given arrangement and thus form a group. Which is the one that does not belong to that group? (1) X – Adidas (2) V – Zodiac (3) Y – Puma (4) N – Park Avenue (5) M – Nautica

    488. Who amongst the following likes Mango? (1) W (2) K (3) Y (4) V (5) N 489. Which of the following represents the brand which W likes? (1) Adidas (2) Puma (3) Park Avenue (4) Zara (5) Nautica 490. Who amongst the following sits second to the left of the one who likes Zodiac? (1) W (2) The one who likes Puma (3) Y (4) The one who likes Adidas (5) X 491. Who amongst the following sit exactly between the ones who like Puma and Zara when counted from the right of the one who likes Zara? (1) The one who likes Adidas and Mango (2) N and the one who likes Park Avenue (3) X and Y (4) K and W (5) V and the one who likes Chemistry Directions (492-497) : Study the following information carefully and answer the given questions : (Corporation Bank SO (Marketing) Exam, 22.02.2014)

    Eight people – A, B, C, D, E, F, G and H– are sitting around a circular table facing towards the centre, but not necessarily in the same order. All of them are at equidistant. Each one of them teaches different subjects viz., English, Hindi, Mathematics, Biology, Psychology, Physics, Chemistry and Accounts, but not necessarily in the same order. The person who teaches Accounts, sits third to the right of G. C is an immediate neighbour of G. The person who teaches Mathematics sits second to the left of C. B sits third to the right of H. H teaches neither Accounts nor Mathematics. Only two persons sit between C and the person who teaches Physics. A and F are immediate neighbours of each other. Neither A nor F teaches Accounts. The person who teaches English sits second to the right of A. Two persons sit between D and the person who teaches Hindi. D

    BPRE–306

    does not teach Accounts. The person who teaches Psychology is an immediate neighbour of the person who teaches Accounts. The person who teaches Physics sits second to the left of A. One of the immediate neighbours of G teaches Chemistry. 492. Who among the following teaches Chemistry? (1) A (2) H (3) D (4) G (5) None of these 493. What is the position of B with respect to the person who teaches Psychology? (1) Second to the left (2) Third to the right (3) Third to the left (4) Second to the right (5) None of these 494. Who among the following sits exactly between the person who teaches Biology and the person who teaches Physics? (1) The person who teaches Mathematics (2) E (3) The person who teaches Accounts (4) Cannot be determined (5) There is no such person 495. Which of the folloing subjects does E teach? (1) Chemistry (2) Hindi (3) Accounts (4) English (5) None of these 496. Which of the following statements is true with regard to the given sitting arrangement? (1) The person who teaches Hindi is an immediate neighbour of both H and D. (2) One of the immediate neighbours of F teaches Biology (3) E is sitting exactly between B and the person who teaches Mathematics (4) The person who teaches Chemistry is second to the right of E (5) All are true 497. Four of the following five are alike in a certain way based on the given sitting arrangement and hence form a group. Which is the one that does not belong to the group? (1) CF (2) GA (3) BD (4) EH (5) BA

    RANKING/ARRANGEMENT Directions (498–502) : Study the following information carefully and answer the questions given below :

    Directions (503–507) : Study the following information carefully and answer the questions given below :

    (IBPS RRBs Officers CWE (Prelim Exam) 09.09.2017 Ist Sitting)

    (IBPS RRBs Officers CWE (Prelim Exam) 09.09.2017 Ist Sitting)

    Twelve people are sitting in two parallel rows containing six people each, in such a way that there is an equal distance between adjacent persons. In row–1, L, M, N, O, P and Q are seated and all of them are facing south. In row–2, D, E, F, G, H and I are seated and all of them are facing north. Therefore in the given seating arrangement, each member seated in a row faces another member of the other row. (Please Note : None of the information given is necessarily in the same order.) The one facing M sits third to the left of E. Neither E nor Q sits at any of the extreme ends of the row. Only three people sit between M and O. The one facing O sits third to the right of I. The one facing D sits second to the right of P. P is not an immediate neighbour of M. Only two people sit between M and L. More than two people sit between F and H. More than one person sits between the one facing H and Q. 498. Who sits second to the left of Q? (1) No one (2) The one facing E (3) O (4) M (5) N 499. How many people sit between M and the one who faces G? (1) Two (2) More than three (3) Three (4) One (5) None 500. Who amongst the following sits third to the left of the person who faces F? (1) L (2) Q (3) P (4) O (5) M 501. Who amongst the following is facing N? (1) F (2) D (3) E (4) G (5) I 502. What is the position of F with respect to I? (1) Third to the right (2) Second to the right (3) Fourth to the left (4) Fourth to the right (5) Other than those given as options

    Eight family members S, T, U, V, W, X, Y and Z are sitting around a circular table but not necessarily in the same order. Some of them are females and some are males. All of them are related to each other in the same way or the other. Some of them are facing the centre while some are facing outside (i.e. opposite to the centre.) Only two people sit between T and W. T faces the centre. X sits second to the right of T. W is the wife of S. No female is an immediate neighbour of W. U is not an immediate neighbour of T. U is the daughter of W. Both the immediate neighbours of U face the centre. Only three people sit between S and U’s brother. X is not the brother of U. Neither S nor U’s brother is an immediate neighbour of X. Z, the wife of T, sits to the immediate left of V. Both Y and S face a direction opposite to that of U (i.e. if U faces the centre then both Y and S face outside and vice–versa). U’s husband sits second to the left of Y. T’s father sits to the immediate right of W. T sits second to the right of S’s father. Both the immediate neighbours of X are females. 503. How many people sit between T and S’s father when counted from the right of T? (1) Four (2) Three (3) None (4) One (5) Two 504. Who amongst the following sits exactly between Y and W when counted from the right of Y? (1) T (2) X (3) S (4) Z (5) U 505. Which of the following statements regarding T is definitely true? (1) X and Z are immediate neighbours of T. (2) T sits second to the left of X. (3) T is the son of S. (4) None of the given options is correct. (5) V is the father of T. 506. Who amongst the following faces outside (i.e. opposite to the centre)?

    BPRE–307

    (1) W (2) V (3) T (4) Z (5) U 507. How many female members are there in the family? (1) Four (2) Three (3) Two (4) Five (5) Cannot be determined Directions (508–512) : Study the following information carefully and answer the questions given below : (IBPS RRBs Officers CWE (Prelim Exam) 09.09.2017 IInd Sitting)

    Eight friends, K, L, M, N, O, P, Q and R are sitting around a circular table with equal distance between them but not necessarily in the same order. Some of them are facing the centre while some face outside (i.e. opposite to the centre). ● K sits third to the right of N. N faces the centre. P sits second to the left of K. ● M sits to the immediate right of P. P faces the centre. ● Only three people sit between M and L. O sits to the immediate right of R. R is not an immediate neighbour of K. ● M sits second to the right of O. M and L face opposite directions (i.e. if M faces the centre then L faces outside and vice-versa). ● Immediate neighbours of N face opposite directions (i.e. if one neighbour faces the centre then the other neighbour faces outside and vice-versa). ● Immediate neighbours of Q face the same direction (i.e. if one neighbour faces the centre then the other neighbour also faces the centre and vice-versa.) 508. How many people sit between R and M when counted from the right of M? (1) More than three (2) Two (3) Three (4) None (5) One 509. Who sits to the immediate right of O? (1) R (2) L (3) Q (4) M (5) K 510. Four of the following five are alike in a certain way based on the given arrangement and so they form a group. Which is the one that does not belong to that group?

    RANKING/ARRANGEMENT (1) M (2) R (3) O (4) K (5) Q 511. What is the position of K with respect to L? (1) Third to the right (2) Third to the left (3) Immediate right (4) Immediate left (5) Second to the right 512. Which of the following statements is true regarding Q? (1) None of the given options is true (2) Only three people sit between Q and O (3) N is an immediate neighbour of Q (4) Only two people sit between Q and K (5) P sits second to the right of Q Directions (513–517) : Study the following information carefully and answer the questions given below : (IBPS RRBs Officers CWE (Prelim Exam) 09.09.2017 IInd Sitting)

    Twelve people are sitting in two parallel rows containing six people each, in such a way that there is an equal distance between adjacent persons. In row–1, L, M, N, O, P and Q are seated and all of them are facing south. In row–2, F, G, H, I, J and K are seated and all of them are facing north. Therefore in the given seating arrangement, each member seated in a row faces another member of the other row (Please Note : None of the information given is necessarily in the same order). The one facing J sits third to the right of O. Neither O nor L sits at any of the extreme ends of the row. Only three people sit between J and G. The one facing Q sits second to the left of I. Q does not face J. I is not an immediate neighbour of G. More than two people sit between L and P. More than two people sit between K and F. The one facing K sits to the immediate left of M. 513. How many people sit between J and the one who faces M? (1) Three (2) None (3) Two (4) More than three (5) One 514. What is the position of L with respect to O?

    (1) Fourth to the left (2) Third to the right (3) Fourth to the right (4) Second to the right (5) Second to the left 515. Who sits second to the right of N? (1) No one (2) L (3) The one facing F (4) O (5) M 516. Who amongst the following is facing K? (1) O (2) M (3) P (4) N (5) Q 517. Who amongst the following sits to the immediate right of the person who faces O? (1) H (2) F (3) K (4) G (5) I Directions (518–522) : Study the following information carefully and answer the questions given below : (IBPS RRBs Officers CWE (Prelim Exam) 10.09.2017)

    Eight friends — O, P, Q, R, S, T, U and V — are sitting around a circular table with equal distance between them but not necessarily in the same order. Some of them are facing the centre while some face outside (i.e. opposite to centre). ● O sits third to the right of R. P sits third to the left of O. S sits to the immediate left of O. ● U sits third to the right of S. S and O face opposite directions (i.e., if S faces the centre then O faces outside and vice-versa). ● Q sits to the immediate right of T. Q is not an immediate neighbour of R. ● V sits third to the right of Q. O faces the centre. ● Immediate neighbours of V face opposite directions (i.e., if one neighbour faces the centre then the other faces outside and vice-versa). ● Immediate neighbours of P face the same direction as T (i.e., if T faces the centre then both the immediate neighbours of P also face the centre and vice-versa.) 518. Four of the following five are alike in a certain way based on the given arrangement and so they form a group. Which is the one that does not belong to that group ?

    BPRE–308

    (1) R (2) V (3) O (4) T (5) U 519. Which of the following statements is true regarding V ? (1) V faces towards centre (2) V sits second to the right of O. (3) Only three people sit between V and U. (4) Only three people sit between V and T. (5) None of the given statements is true. 520. How many people sit between T and O when counted from the right of O ? (1) One (2) Three (3) More than three (4) Two (5) None 521. What is the position of S with respect to Q ? (1) Third to the right (2) Second to the right (3) Third to the left (4) Immediate right (5) Second to the left 522. Who sits second to the left of P ? (1) V (2) U (3) S (4) T (5) R Directions (523–527) : Study the following information carefully and answer the questions given below : (IBPS RRBs Officers CWE (Prelim Exam) 10.09.2017)

    Twelve people are sitting in two parallel rows containing six people each, in such a way that there is equal distance between adjacent persons. In row- 1, B, C, D, E, F and G are seated and all of them are facing south. In row-2, J, K, L, M, N and O are seated and all of them are facing north. Therefore in the given seating arrangement, each member seated in row faces another member of the other row. (Please Note : None of the information given is necessarily in the same order). More than two people sit to the right E. The one facing E sits to the immediate left of N. Only two people sit between N and J. The one facing M sits third to the right of B. More than two people sit betweeen C and D. C does not sit at any of the extreme ends of the row. More than two people sit between the one facing G and L. K is not an immediate neighbour of J.

    RANKING/ARRANGEMENT 523. Who sits second to the left of O ? (1) N (2) The one facing F (3) L (4) K (5) No one 524. What is the position of B with respect to G ? (1) Third to the right (2) Second to the left (3) Fourth to the left (4) Third to the left (5) Second to the right 525. Who amongst the following is facing F ? (1) O (2) K (3) N (4) M (5) L 526. How many people sit between J and the one who faces D ? (1) None (2) More than three (3) Two (4) One (5) Three 527. Who amongst the following sits to the immediate left of the person who faces N ? (1) F (2) B (3) C (4) D (5) G Directions (528–532) : Study the following information carefully and answer the questions given below : (IBPS RRBs Officer CWE (Prelim Exam) 16.09.2017)

    Twelve people are sitting in two parallel rows containing six people each, in such a way that there is equal distance between adjacent persons. In row–1, R, S, T, U, V and W are seated and all of them are facing south. In row–2, E, F, G, H, I and J are seated and all of them are facing north. Therefore in the given seating arrangement each member seated in a row faces another member of the other row. (Please Note : None of the information given is necessarily in the same order.) The one facing H sits second to the right of S. More than one person sits to the left of H. Only two people sit between S and W. More than two people sit between R and the one facing J. R is at one of the positions to the right of W. J does not sit at any of the extreme ends of the row. The one facing U sits to the immediate left of E. E is not an immediate neighbour of J. One of the immediate neighbours of F faces T. G is not an immediate neighbour of E.

    528. Four of the following five are alike in a certain way based on the given arrangement and thus they form a group. Which is the one that does not belong to the group? (1) VU (2) TS (3) FJ (4) EI (5) TV 529. If all the persons in row–2 are made to sit in alphabetical order from left to right, who amongst the following will face S according to the new arrangement? (1) G (2) J (3) E (4) F (5) H 530. Who amongst the following sits exactly between H and the one facing S? (1) I (2) The one facing R (3) F (4) The one facing T (5) G 531. Who amongst the following sit to the immediate left and immediate right respectively of the person facing V? (1) G, F (2) I, J (3) H, E (4) H, F (5) E, J 532. Which of the following statements is true with regard to the given arrangement? (1) Four people sit between F and the one facing U. (2) Only one person sits between V and R. (3) U is an immediate neighbour of the one facing G. (4) None of the given options is true (5) V faces G. Directions (533–537) : Study the following information carefully and answer the questions given below : (IBPS RRBs Officer CWE (Prelim Exam) 16.09.2017)

    Eight people, Q, R, S, T, U, V, W and X are sitting around a circular table with equal distance between each other but not necessarily in the same order. Some of them are facing the centre while some face outside, (i.e., opposite to centre.) ● V sits second to the right of W. W faces the centre. Only two people sit between V and X (from either V’s right or V’s left).

    BPRE–309

    Only three people sit between X and S. S faces the centre. T sits third to the right of S. ● R sits second to the right of T. Q sits third to the left of R. ● Immediate neighbours of R face a direction opposite to that of R (i.e., If R faces the centre then both the immediate neighbours of R face outside and vice-versa.) ● Immediate neighbours of T face the same direction (i.e., if one neighbour faces the centre then the other also faces the centre and viceversa.) ● Immediate neighbours of Q face the same direction (i.e., if one neighbour faces the centre then the other also faces the centre and vice-versa.) 533. In which of the following pairs, do both the people face the same direction (i.e. either both face centre or both face outside)? (1) S, Q (2) X, V (3) T, U (4) T, Q (5) R, V 534. If all the people are made to sit in alphabetical order in clockwise direction starting from Q, then the position of how many people (including Q) will remain unchanged? (1) None (2) Three (3) More than three (4) Two (5) One 535. Which of the following staments is not true as per the given arrangement? (1) T faces the centre. (2) V sits second to the left of T. (3) Only three people sit between Q and U. (4) T is an immediate neighbour of U. (5) All the given statements are true. 536. Who sits to the immediate right of T? (1) Q (2) R (3) X (4) W (5) U 537. Which of the following pairs represents the immediate neighbours of W? (1) S, U (2) U, Q (3) Q, R (4) R, S (5) Q, X ●

    RANKING/ARRANGEMENT Directions (538–542) : Study the following information carefully and answer the questions given below : (IBPS Bank PO/MT CWE-VII (Prelim Exam) 14.10.2017)

    Eight people viz. A, B, C, D, E, F, G and H are sitting around a square table in such a way that four of them sit at four corners of the table while four sit in the middle of each of the four sides, but not necessarily in the same order. The ones sitting in the middle of the sides are facing the centre and the ones sitting at the corners of the table are facing outside (i.e. opposite to the centre). A sits in the middle of one of the sides. C is an immediate neighbour of A. E sits second to the left of C. Only three people sit between E and G. Only two people sit between G and D (either from left or right). D is not an immediate neighbour of C. B sits second to the right of D. F sits second to the right of B. 538. How many people sit between C and H when counted from the left of H? (1) None (2) Two (3) One (4) Three (5) More than three 539. Which of the given statements is not true as per the given arrangement? (1) G sits at one of the corners of the table. (2) All the given statements are true. (3) Only three people sit between F and D. (4) B and F face the centre. (5) E sits second to the right of F. 540. Which of the following pairs represents the people sitting between H and F, when counted from the left of F? (1) G, B (2) C, G (3) C, E (4) A, D (5) B, D 541. As per the given arrangement, four of the following five are alike in a certain way and thus they form a group. Which one of the following does not belong to that group? (1) C (2) D (3) H (4) G (5) E

    542. Who sits third to the left of B? (1) G (2) A (3) E (4) H (5) C Directions (543-547) : Study the following information carefully and answer the questions given below : (IBPS Bank PO/MT CWE-VII (Prelim Exam) 14.10.2017)

    Eight people viz. P, Q, R, S, T, U, V and W are sitting in a straight line facing north with equal distance between each other. Each of them teaches a different subject viz. Chemistry, History, Mathematics, English, Physics, Biology, Geography and Social Science. (Note: None of the given information is necessarily in the same order.) P sits third from the left end of the line. Only one person sits between P and the one who teaches English. Only three people sit between the one who teaches English and R. As many people sit to the right of R as to the left of the one who teaches Social Science. Only three people sit between the one who teaches Social Science and W. No one sits between W and the one who teaches History. As many people sit to the left of the one who teaches History as to the right of T. Only one person sits between T and V. The one who teaches Mathematics sits to the immediate right of the one who teaches Physics. The one who teaches Mathematics is an immediate neighbour of V. Only one person sits between Q and the one who teaches Mathematics. More than four people sit between Q and the one who teaches Chemistry. U sits to the immediate left of S. U does not teach Geography. 543. How many people sit to the right of U? (1) Two (2) None (3) More than three (4) Three (5) One 544. Four of the following five are alike in a certain way based on the given arrangement and thus they form a group. Which one of the following does not belong to that group? (1) Q – Physics (2) T – English (3) S – Chemistry (4) P – Bioiogy (5) W – History

    BPRE–310

    545. Which of the following statements is true about the one who teaches Geography as per the given arrangement? (1) The one who teaches Geography sits second to the left of P. (2) Only one person sits between T and the one who teaches Geography. (3) V sits to the immediate left of the one who teaches Geography. (4) None of the given statements is true. (5) Less than two people sit between the one who teaches Geography and English. 546. What is the position of the one who teaches Chemistry with respect to V? (1) Immediate right (2) Fourth to the right (3) Third to the right (4) Third to the left (5) Fourth to the left 547. Who sits third to the left of S? (1) T (2) The one who teaches Mathematics (3) P (4) The one who teaches Biology (5) The one who teaches Social Science Directions (548–552) : Study the following information carefully and answer the questions given below : (IBPS RRBs Officer CWE (Main Exam) 05.11.2017)

    Eight persons namely P, Q, R, S, W, X, Y and Z are siting around a square table facing the centre but not necessarily in the same order. Four of them are sitting at the four corners of the table while the other four are sitting in the middle of the side of the square. Each of them is of different age. The persons sitting at the corners are having the ages multiple of 3. The persons sitting at the middle of the sides are having the ages multiple of 2. P sits second to the right of the one whose age is 44. P is an immediate neighbour of the one who is 39 years old. Only two persons sit between Q and the one having the age of 39. Q has not the age of 44. W sits second to the left of Q. S is to the immediate right of the person having age of 15. Only three persons sit between Z and the one having age of 15. The difference

    RANKING/ARRANGEMENT between the ages of Z and the one who is second to the left of Z is 6. S is older than Z. Immediate neighbours of S are younger than S. Person having age 22 sits second to the right of the person having age 10. Age of P is not 10. Y sits to the immediate left of the person having age of 22. Age of Y is 51. Age of R is greater than 20. Age of R is 24 years more than X. Age of X is 3 years more than age of P. 548. Who among the following is 9 year old? (1) P (2) S (3) W (4) X (5) No one 549. Who sits second to the right of the one who sits to the immediate left of the one whose age is 51 years? (1) R (2) S (3) Y (4) Q (5) W 550. How many persons sit between the one who is 15 year old and the one who is 12 year old, when counted anticlockwise from the one who is 15 year old? (1) One (2) Two (3) Three (4) More than three (5) None of these 551. Who among the following sits diagonally opposite to Z? (1) X (2) T (3) S (4) P (5) W 552. Who among the following sits to the immediate right of S? (1) P (2) W (3) R (4) Y (5) Z Directions (553–556) : Study the following information carefully and answer the questions given below : (IBPS SO (IT Officer) CWE (Prelim Exam) 30.12.2017)

    A certain number of people are sitting in a straight line facing north. B sits fifth to the right of K. Only two people sit between K and S. M sits seventh to the left of S. R sits at one of the positions to the right of M. The number of people between S and B is one less than that between M and R. Q sits at one of the positions between S and R. Q is not an immediate neighbour of K. L sits seventh to the right of Q. L sits at an extreme end. As many people sit between L and B as to the left of M.

    553. Who sits second to the left of B? (1) L (2) R (3) S (4) M (5) Other than those given as options 554. Which of the following represents the positions of R in the given line? (1) Exactly between Q and K (2) Second to the right of S (3) Eighth from the right end of the line (4) Sixth to the left of L (5) Seventh from the left end of the line 555. Which of the following statements is true as per the given information? (1) K is an immediate neighbour of L (2) None of the given statements is true. (3) Only six people sit between B and L. (4) Only one person sits to the left of M. (5) More than three people sit between M and R 556. How many people are sitting in the given line? (1) 17 (2) 20 (3) 15 (4) 22 (5) Cannot be determined Directions (557–561) : Study the following information carefully and answer the questions given below : (IBPS SO (IT Officer) CWE (Prelim Exam) 30.12.2017)

    Twelve people are sitting in two parallel rows containing six people each, in such a way that there is equal distance between adjacent persons. In row-1, A, B, C, D, E and F are seated (not necessarily in the same order) and all of them are facing north. In row-2, P, Q, R, S, T and U are seated (not necessarily in the same order) and all of them are facing south. Therefore in the given seating arrangement, each member seated in a row faces another member of the other row. B sits third to the right of A. The one who faces A sits second to the right of P. R sits second to the left of P. Only two people sit between C and the one who faces R. As many people sit to the right of E as to the left of C. D faces U. Only one person sits between U and T. The one who faces F sits second to the left of Q.

    BPRE–311

    557. Which of the given statements is TRUE with respect to the given information? (i) C faces S (ii) T sits at an extreme end. (iii) D faces an immediate neighbour of R. (1) Only (iii) (2) All (i), (ii) and (iii) (3) Both (i) and (iii) (4) Both (i) and (ii) (5) Only (iii) 558. Who amongst the following faces T? (1) B (2) D (3) E (4) C (5) A 559. If all the people of Row-1 are made to sit in alphabetical order from left to right, then who amongst the following will face P as per the new arrangement? (1) F (2) C (3) D (4) A (5) B 560. As per the given arrangement, B is related to Q in the same way as P is related to D. Following the same pattern, to whom is E related? (1) U (2) The one who faces A (3) T (4) The one who faces D (5) S 561. Who among the following does not sit at any of the extreme end of the rows ? (1) The one who faces A (2) The one who sits to the immediate left of C (3) U (4) R (5) D Directions (562–567) : Study the following information carefully and answer the questions given below : (IBPS SO (IT Officer) CWE (Prelim Exam) 30.12.2017)

    Eight people D, E, F, G, R, S, T and U are sitting around a square table facing the centre. Four people are sitting at the corners and four in the middle of the sides of the table. The ages of the ones sitting in the middle of the sides are multiples of 3 and that of those sitting at the corners are multiples of 7. D sits second to the right of the 36 year old. One of the immediate

    RANKING/ARRANGEMENT neighbours of D is 28 years old. Only three people sit between E and the 28 year old. The age of the one who sits second to the left of E is less than 15 and also a multiple of 2. G is an immediate neighbour of the 51 year old. G is neither 14 nor 28 year old. Only one person sits between G and U (when counted from left or right). F is 27 years old. Only three people sit between F and T. The one who sits to the immediate left of S is 49 years old. G’s age is less than 10 years. The difference between ages of G and D is 11. 562. How many people sit between T and the 51 year old, when counted from the left of T? (1) One (2) None (3) More than three (4) Two (5) Three 563. HOW many people are younger than D? (1) Three (2) More than four (3) Four (4) Two (5) One 564. Which of the following statements is true as per the given arrangement? (1) T is 51 years old. (2) None of the given statements is true (3) F is an immediate neighbour of U. (4) The differences of the ages of E and F is 8 (5) T sits to the immediate right of the 7 year old. 565. Who amongst the following is 14 years old? (1) None of the given options (2) R (3) T (4) U (5) E 566. What is the age of D(in years)? (1) 48 (2) 15 (3) 18 (4) 27 (5) 9 567. In a straight line of six people (all facing north), Q sits at an extreme end of the line. Only two people sit between Q and G. R sits second to the right of G. L sits third to the right of A. Only one person sits between Q and B. Who sits to the immediate right of B? (1) No one as B sits at the extreme right end (2) L

    (3) G (4) A (5) Cannot be determined (IBPS SO (IT Officer) CWE (Prelim Exam) 30.12.2017)

    Directions (568–572) : Study the following information carefully and answer the questions given below : (IBPS SO (Law Officer) CWE (Prelim Exam) 31.12.2017)

    Eight people viz. M, N, P, Q, R, S, T and U are sitting in a straight line but not necessarily in the same order. Some of them are facing north while some of them are facing south. N sits third from one of the extreme ends of the line. Only two people sit between N and S. Q faces north and sits second to the right of S. Q is not an immediate neighbour of N. Only three people sit between Q and U. M sits second to the right of U. R sits to the immediate right of M. Both the immediate neighbours of M face a direction opposite to that of M. P is not an immediate neighbour of Q and faces north. T sits second to the left of P and faces a direction opposite to that of P. 568. Four of the following five are alike in a certain way based on the given arrangement and thus they form a group. Which one of the following does not belong to the group? (1) U (2) M (3) P (4) N (5) Q 569. What is the position of N with respect to M? (1) Third to the right (2) Second to the left (3) Immediate right (4) Second to the right (5) Immediate left 570. Which of the following statements is not TRUE as per the given arrangement? (1) All the given statements are true (2) N sits second to the left of T. (3) Only two people sit between T and U. (4) S faces the same direction as T. (5) T is an immediate neighbour of S. 571. How many people sit between N and Q? (1) Four (2) One (3) Three (4) More than Four (5) Two

    BPRE–312

    572. Who sits second to the left of S? (1) P (2) M (3) U (4) N (5) T Directions (573–577) : Study the following information carefully and answer the questions given below : (Canara Bank PO Exam 04.03.2018)

    Twelve people are sitting in two parallel rows, containing six people each in such a way that there is an equal distance between adjacent persons. In row–1 R, S, T, U, V and W are seated and all of them are facing north and in row–2 A, B, C, D, E and F are seated and all of them are facing south (but not necessarily in the same order). Each person also teaches a different subjects namely, Accounts, English, Hindi, History, Psychology, Geography, Civics, Economics, Physics, Statistics, Chemistry and Biology (but not necessarily in the same order). C sits at one of the extreme ends of the row and faces one of the immediate neighbours of T. Only two people sit between T and the one who teaches Chemistry. R sits to the immediate right of the one who teaches Chemistry. R faces E. D is neither an immediate neighbour of C nor E. The one who teaches Economics sits third to the left of D. S faces one of the immediate neighbours of E. The one who faces S teaches Statistics. Only two people sit between the one who teaches Statistics and the one who teaches History. Only one person sits between A and the one who teaches History. U and W are immediate neighbours of each other. The one who faces U sits to the immediate left of F. The one who teaches Biology is an immediate neighbour of A. Only two people sit between W and the one who teaches Geography. V faces the one who teaches Hindi. The one who teaches Physics sits third to the right of the one who teaches English. Neither W nor A teaches Civics. The one who teaches Accounts sits in row-2. 573. Which of the following statements is definitely true as per the given information? (1) U faces A. (2) C and F are immediate neighbours of each other. (3) B faces the one who teaches Civics, (4) E teaches History (5) W is an immediate neighbour of the one who teaches Physics.

    RANKING/ARRANGEMENT 574. Who amongst the following is an immediate neighbour of the one who teaches Economics? (1) The one who teaches Biology (2) The one who teaches Chemistry (3) F (4) B (5) None of the given options 575. Which of the following subjects is taught by T? (1) Economics (2) Accounts (3) Civics (4) Hindi (5) Biology 576. Which of the following pairs represent the immediate neighbours of the one who teaches Civics? (1) A, B (2) T, W (3) S, R (4) U, V (5) B, D 577. Who amongst the following teaches Physics? (1) R (2) A (3) D (4) U (5) V Directions (578–582) : Study the following information carefully and answer the questions given below : (Canara Bank PO Exam 04.03.2018)

    Eight persons are sitting in a straight line facing north. They are in different Standards/pursuing different courses and are sitting in such a way that all the seniors of every person is sitting to the right. Therefore, no person who is in a higher standard is sitting to the left of any person. A is in 5th standard and is sitting third from the left end of the line. Only one person is sitting between A and Q. M sits third to the left of the 7th standard student. M is not an immediate neighbour of A. As many people sit between the 5th and the 7th standard students as between Q and the 12th standard student. R is pursuing B Com. As many people are sitting to the right of the B Com student as to the left of the 3rd standard student. B sits to the immediate left of the 9th standard student. More than two people sit between S and the Ph D student. F is younger than Z. 578. Who sits exactly between A and Q? (1) Z (2) The 9th standard student (3) F (4) The 3rd standard student (5) B

    579. Four of the following five are alike in a certain way based on their positions in the arrangement and hence form a group. Which one of the following does not belong to that group? (1) S – 7th standard (2) Q – B.Com (3) M – 5th standard (4) F – 9th standard (5) B – 12th standard 580. Which of the following statements is true as per the given information? (1) M is an immediate neighbour of S (2) B is in 9th standard (3) S sits second to the right of Q (4) R and the 3rd standard student are immediate neighbours of F. (5) All the given statements are true 581. Who amongst the following is younger than M? (1) Z (2) No one (3) S (4) B (5) F 582. Who studies in the 7th standard? (1) Z (2) S (3) B (4) F (5) The person studying in one standard higher than F. Directions (583–585) : Study the following information carefully and answer the questions given below :

    the other faces outside and viceversa.) ● Immediate neighbours of O face opposite directions (i.e., if one neighbour faces the centre then the other faces outside and viceversa.) ● Immediate neighbours of S faces the same direction as S. (i.e., If S faces outside then both the immediate neighbours of S also face outside and vice-versa). 583. What is M’s position with respect to L? (1) Second to the right (2) Third to the left (3) Fourth to the right (4) Second to the left (5) Third to the right 584. Who amongst the following are immediate neighbours of O? (1) P, M (2) M, Q (3) R, M (4) L, Q (5) L, N 585. Four of the following five are alike in a certain way, based on the given seating arrangement and so they form a group. Which is the one that does not belong to that group? (1) P (2) M (3) S (4) R (5) Q Directions (586–590) : Study the following information carefully and answer the questions given below :

    (IDBI Bank PO Exam 29.04.2018)

    Eight persons A, B, C, D E, F, G and H sit on the line and all of them face towards north direction but not necessarily in same order. All of them lives on different floors of eight-storey building viz. 1st, 2nd, 3rd, 4th, 5th, 6th, 7th and 8th but not necessarily in same order. The one who lives on 3rd floor sits second to right of one who lives on 2nd floor. C lives on 5th floor. A sits fourth to left of the one who lives on 8th floor. D does not sit adjacent to H. Neither A nor the person who lives on 8th floor sit on the extreme end of the line. B sits third to left of F. Only one person sits between the G, who lives on 1st floor and the person who lives on 8th floor. Only two persons sit between G and the one who lives on 7th floor. H sits immediate left of one who lives on 7th floor. Only two persons sit between H and F, who lives on 6th floor.

    Eight persons L, M, N, O, P, Q, R and S are sitting around a circular area at equal distance between each other, but not necessarily in the same order. Some people face the centre while some face outside (i.e. in a direction opposite to the centre). ● O faces the centre. S sits third to the right of O. Only one person sits between S and L (either from left or right). ● P sits second to the left of L. ● R sits second to the right of N. N is not an immediate neighbour of S. ● Q sits to the immediate left of M. M and L face the same direction (i.e. M faces centre then L also faces the centre and vice-versa.) ● Immediate neighbours of Q face opposite directions (i.e., if one neighbour faces the centre then

    BPRE–313

    (IDBI Bank PO Exam 29.04.2018)

    RANKING/ARRANGEMENT 586. B lives on which of the following floor? (1) 6th (2) 2nd (3) 7th (4) 5th (5) 3rd 587. How many persons sit between G and B? (1) Two (2) One (3) Three (4) Four (5) None of these 588. D lives on which of the following floor? (1) 2nd (2) 3rd (3) 8th (4) 4th (5) 7th 589. Who among the following sits to the immediate right of the person one who lives on 7th floor? (1) No one (2) F (3) D (4) B (5) A 590. Who among following sits third to right of A? (1) H (2) E (3) F (4) G (5) C Directions (591–595) : Study the following information carefully and answer the questions given below : (IBPS Bank PO/MT CWE-VII (Prelim Exam) 15.10.2017)

    A, B, C, D, E, F, G and H are eight friends and sitting around a circular table but not necessarily in same order. Some of them are facing inside and some of them are facing outside. A sits third to right of H. Only two people sit between H and B. C sits second to left of B. Only three people sit between B and E. D is second to left of F. Immediate neighbours of H face same direction as H. F sits third to left of A, who faces centre. The immediate neighbours of A face opposite to the direction of A. 591. Who is sitting third to the right of F? (1) C (2) B (3) A (4) D (5) E 592. Who are facing the centre ? (1) AB (2) AGH (3) AD (4) ADC (5) None of those given as options 593. Who sits opposite to H? (1) A (2) D (3) F (4) E (5) G

    594. How many people are sitting between C and B, when counted from left of C? (1) Two (2) Three (3) One (4) Four (5) Five 595. Who are sitting exactly between D and C when counted from right of D? (1) HE (2) CE (3) FH (4) BF (4) DF Directions (596–600) : Study the following information carefully and answer the questions given below : (IBPS Bank PO/MT CWE-VII (Prelim Exam) 15.10.2017)

    Ten persons are sitting in two parallel rows containing five persons in each row. In 1st row M, N, O, P and Q are seated and are facing south. In 2nd row, U, V, X, Y and Z are seated and are facing north. Therefore in the given seating arrangement, each member seated in a row faces another member of the other row. They like different colours viz., Red, Orange, Blue, Brown, Black, White, Yellow, Pink, Peach, and Grey (not necessarily in same order). M doesn’t like brown and P likes black. Y sits third to the left of U, who likes yellow. M faces immediate neighbour of Y, who likes orange. The one who likes peach sits at extreme end. O sits second to the right of M. The one who likes red faces the one who likes pink but M doesn’t like pink. Only one person sits between N and P. V and Z are immediate neighbours. Z does not face M and N, and N doesn’t like grey. The one who faces U likes white. The one who faces an immediate neighbour of Y likes brown. 596. How many persons are seated between N and the one who likes white? (1) None (2) One (3) Two (4) Three (5) More than three 597. Who amongst the following faces P? (1) U (2) The one who likes Pink (3) X (4) N (5) The one who likes grey 598. Which of the following statements is true regarding M? (1) N and O are immediate neighbours of M

    BPRE–314

    (2) M sits at one of the extreme ends of the line (3) M likes black (4) P sits to the immediate left of M (5) None of these 599. Who amongst the following pair sits exactly in the middle of the rows? (1) M, V (2) P, Y (3) U, N (4) M, Z (5) Other than those given as options 600. V likes which of the following colour? (1) Brown (2) Pink (3) Black (4) White (5) Other than those given as options Directions (601–603) : Study the following information carefully and answer the questions given below : (IBPS Bank PO/MT CWE-VII (Prelim Exam) 15.10.2017)

    A certain number of persons are seated in a row. The Row is arranged in a horizontal manner and all are facing to north direction. Ranjan sits fourth from the left end of the row. Only two persons sit between Ranjan and Seema. Puja sits to the immediate right of Seema. There are as many persons between Puja and Seema as between Dinesh and Puja. Dinesh does not sit to the left of Ranjan. 601. How many persons sit between Ranjan and Dinesh? (1) Three (2) Five (3) None (4) Four (5) One 602. What is the position of Puja with respect to Ranjan? (1) Immediate right (2) Third to the left (3) Other than those given as options (4) Second to the left (5) Fourth to the right 603. How many persons are seated in a row? (1) Nine (2) Six (3) Ten (4) Eleven (5) Four Directions (604–605) : Study the following information carefully and answer the questions given below : (IBPS Bank PO/MT CWE-VII (Prelim Exam) 15.10.2017)

    There are six persons S, T, U, V, W and X, who got different marks

    RANKING/ARRANGEMENT in the examination. S got more marks than only U and X. T got less marks than W, who did not get the highest marks in the examination. The person who got the second highest marks got 92 marks. 604. V got which of the following possible score? (1) 85 (2) 66 (3) 94 (4) 89 (5) 92 605. How many persons got more marks than U ? (1) Four (2) Two (3) Five (4) Cannot be determined (5) One 606. If s got 69 marks and U got 68 marks, then which of the following statements is true? (1) X got the lowest marks. (2) U got the fifth highest marks (3) Five persons got more marks than X. (4) X got 66 marks is a possibility (5) All are true Directions (607–611) : Study the following information carefully and answer the questions given below : “Numbers given in the figures are considered as their positions.”

    1

    5

    8

    4

    2

    6

    7

    3

    Eight persons Shiva, Veena, Bharat, Sagar, Meghana, Mani, Wasim and Devi are sitting around the circle having eight vacant chairs. Four of them are facing towards the centre while other four are facing away from the centre. Devi sits opposite to Veena who is to the immediate left of Sagar. Shiva is facing away from the centre and Devi is second to the left of Shiva. Wasim is sitting exactly between Veena and Mani. Bharat who is facing the centre is opposite to Sagar. The persons second to the right and left of Shiva are facing away from the centre. Both the imme-

    diate neighbours of Bharat are facing away from the centre. Now they start playing cards game. They shuffled a pack of cards. 1. Meghana draws one card and changes her place according to the given conditions, 2. Mani draws one card and changes his place according to the given conditions. Similarly others draw cards according to the conditions given below. Conditions : 1. If the card drawn is spade, the person who draws first moves to position 5 facing opposite direction of current direction, then the second person who also draws spade moves to position 6, or 7 and so on. 2. If card drawn is heart, person remains on the same position facing the same direction. 3. If the card drawn is diamond, the person who draws first moves to corner 1 facing same direction, then the second person, who also draws diamond moves to corner 2 and so on. 4. If card drawn is club, person remains on the same position facing opposite direction of current direction. Draws : 1. Mani draws Queen of Club 2. Shiva draws Jack of Club 3. Veena draws 3 of Spade 4. Bharat draws a King of Diamond 5. Sagar draws Ace of Heart 6. Wasim draws 9 of Heart 7. Meghana draws 7 of Diamond 8. Devi draws 4 of Spade Note : All the persons draw card in an order given above. (For example – First Mani draws then Shiva draws and so on.....) (IBPS Bank PO/MT CWE (Main Exam) 26.11.2017)

    607. After all the arrangements been done, how many of them are facing away from the centre ? (1) 3 (2) 5 (3) 1 (4) None (5) Cannot be determined 608. After all persons changed their places based on the above conditions, how many persons do remain in the same places of the circle? (1) 2 (2) 3

    BPRE–315

    (3) 4 (4) 5 (5) 6 609. After all persons changed their places based on the above conditions, who among the following is sitting to the immediate right of Meghana? (1) Devi (2) Veena (3) Sagar (4) Bharat (5) Mani 610. After all persons changed their places based on the above conditions, who among the following is sitting at the position 7 of the square? (1) Shiva (2) Sagar (3) Devi (4) Wasim (5) Mani 611. After all persons changed their places based on the above conditions, who is opposite to Bahart? (1) Sagar (2) Shiva (3) Meghana (4) Wasim (5) Mani Directions (612–614) : Study the following information carefully and answer the questions given below : More than 10 persons were sitting in a circular arrangement facing the centre. C is third to the left of D. There are two persons sitting between E and A and between E and C respectively. There are four persons sitting between D and A. A is 5th to the right of D. F is neither between A and D nor between A and E. B and F are immediate neighbours of each other. B is neighbour of neither D nor C. (IBPS Bank PO/MT CWE (Main Exam) 26.11.2017)

    612. How many persons are sitting in the circular arrangement? (1) 14 (2) 11 (3) 12 (4) 13 (5) Cannot be determined 613. What is the position D with respect to F? (1) Third to the left (2) Third to the right (3) Fourth to the left (4) Fourth to the right (5) None of these 614. How many persons are sitting between C and A in anti-clockwise direction? (1) 2 (2) 0 (3) 3 (4) 6 (5) 7

    RANKING/ARRANGEMENT Directions (615–619) : Study the following information carefully and answer the questions given below : Eight students A, B, C, D, P, Q, R and S are sitting in a row. Only four of them are facing north. Each of them is studying different subjects viz. Physics, Chemistry, Biology, Commerce, Maths, Economics, History and Education. A who is studying Chemistry is 3rd to the right of D who faces north. Only two students are sitting to the left of A. The student at the extreme left end is facing north. History student and P are sitting at extreme ends. D is studying either Biology or Physics. Maths student is between B and a student facing south. S, the Economics student is sitting between P and Biology student. C is sitting between the students who are facing towards south. Q is an immediate neighbour of Chemistry student. P studies either Education or History. Maths student is sitting between students facing south. Biology student is sitting between the students facing south. Physics student is sitting second to the right of Q. (IBPS Bank PO/MT CWE (Main Exam) 26.11.2017)

    615. Four of the following five are alike in a certain way based on their positions in the given arrangement and so they form a group. Which is the one that does not belong to the group? (1) PD (2) SB (3) BA (4) AC (5) AR 616. Who among the following is a Biology student and sitting second to the right of whom? (1) D – 2nd to the right of P (2) P – 2nd to the right of D (3) C – 2nd to the right of B (4) B – 2nd to the right of C (5) None of these 617. Who is sitting third to the right of Commerce student? (1) Economics student (2) Education student (3) Physics student (4) Chemistry student (5) None of these 618. Which of the following pairs of students is sitting at the extreme ends of the row ?

    (1) Economics student and R (2) Education student and C (3) Education student and R (4) Biology student and C (5) Commerce student and C 619. Who among the following an arts student is sitting between two science students? (1) S – Economics (2) Q – Commerce (3) C – Physics (4) R – History (5) B – Commerce Directions (620–623) : Study the following information carefully and answer the questions given below : A certain number of people are sitting in a straight line facing north with equal distance between each other. Only three people sit between C and Z. F sits seventh to the left of C. F sits fourth from one of the extreme ends of the line. Only four people sit between F and V. N sits eighth to the right of V. Only three people sit to the right of N. Only two people sit between N and P. P does not sit at an extreme end of the line. More than one person sits between P and Z. (IBPS RRBs Officer CWE (Prelim Exam) 11.08.2018)

    620. What is the position of V with respect to P? (1) Second to the right (2) Fourth to the right (3) Fifth to the left (4) Sixth to the right (5) Ninth to the left 621. How many people sit between C and P? (1) Seven (2) Two (3) Five (4) Nine (5) One 622. If Q sits third to the left of C, what is his position from the left end of the line? (1) Sixth (2) Fifteenth (3) Seventh (4) Eighth (5) Eleventh 623. If J sits fifth from the left end of the line, how many people sit to the right of J? (1) Fifteen (2) Fourteen (3) Eleven (4) Twenty (5) Nineteen Directions (624–627) : Study the following information carefully and answer the questions given below :

    BPRE–316

    Eight people are sitting around a circular table with equal distance between each other. Some of them are facing the centre and some are facing outside (i.e. opposite to the centre). Note : (1) Facing the same direction means if one person faces the centre, then the other also faces the centre and vice-versa. (2) Facing opposite directions means, if one person is facing the centre then the other faces outside and vice versa. B sits second to the left of A. Only three people sit between B and F. D sits second to the right of F. E sits second to the right of G. G is neither an immediate neighbour of D nor F. G faces the centre. C sits second to the left of E. H sits third to the right of D. C faces a direction opposite to that of D. Both B and H face the same direction as C. (IBPS RRBs Officer CWE (Prelim Exam) 11.08.2018)

    624. Which of the following statements is true as per the given arrangement? (1) None of the given statements is true (2) H sits to the immediate right of B. (3) A faces the centre. (4) G and E face the same direction (5) B is an immediate neighbour of E. 625. How many people sit between C and H when counted from the left of C? (1) Two (2) None (3) One (4) Three (5) More than three 626. Who sits to the immediate right of H? (1) B (2) F (3) E (4) C (5) G 627. Four of the following five are alike in a certain way as per the given arrangement and thus they form a group. Which one of the following does not belong to that group? (1) H (2) E (3) F (4) D (5) A Directions (628–632) : Study the following information carefully and answer the questions given below :

    RANKING/ARRANGEMENT Fourteen people are sitting in two parallel rows containing seven people each with equal distance between each other. In row–1, A, B, C, D, E, F and G are seated and all of them are facing south but not necessarily in the same order. In row–2, P, Q, R, S, T, U and V are seated and all of them are facing north but not necessarily in the same order. Thus in the given arrangement each person seated in a row faces another person of the other row. A sits second from one of the extreme ends of the row. P faces the person who sits to the immediate right of A. Only two people sit between P and R. Only one person sits between the one facing R and C. Only two people sit between C and D. Only one person sits between the one facing D and Q. The one facing E sits at one of the extreme ends of the row. Neither P nor S sits at any of the extreme end of the row. As many people sit between U and R as between E and F. The one who faces T sits at one of the positions to the right of the one who faces S. Only two people sit between B and F. F is not an immediate neighbour of a person who sits at any of the extreme ends of the row. (IBPS RRBs Officer CWE (Prelim Exam) 11.08.2018)

    628. Who amongst the following sits second to the right of A? (1) The one who faces T (2) C (3) The one who faces Q (4) B (5) E 629. How many people sit between U and S? (1) Three (2) None (3) One (4) Two (5) More than three 630. Who amongst the following faces the person who sits second to the right of T? (1) B (2) D (3) C (4) G (5) A 631. Who amongst the following sits at one of the extreme ends of the row? (1) The one who faces U (2) B (3) Q (4) The one who faces G (5) D

    632. Who amongst the following faces D? (1) T (2) U (3) V (4) P (5) S Directions (633–637) : Study the following information carefully and answer the questions given below : Eleven boxes A, B, C, D, E, F, G, H, I, J, K are kept one above the other but not necessarily in the same order. Box G is kept at fifth position from the top. Only two boxes are kept between G and H. Box D is kept just above box H. There are as many boxes above Box D as below Box B. Only five boxes are kept between Box F and Box K, which is kept at one of the positions below Box G. Box A is kept at one of the positions above Box F. Only one box is kept between Box G and Box C. Box I is kept above box E but not just above. Box E is not kept immediately above or immediately below Box C. (IBPS RRBs Officer CWE (Prelim Exam) 18.08.2018)

    633. What is the position of box I? (1) 8th from the bottom (2) 7th from the top (3) 3rd from the top (4) 6th from the bottom (5) None of these 634. How many boxes are kept between Box E and Box H? (1) Seven (2) Six (3) Five (4) Four (5) Eight 635. Which among the following statements is true regarding Box J? (1) It is 7th from the bottom (2) Box K is placed above the Box J (3) Only two boxes are kept between Box B and Box J (4) It is kept just below Box H (5) All are true 636. Which of the following represents the boxes kept between Boxes A and I ? (1) C, B (2) A, K (3) F, G (4) J, D (5) None of these 637. Which of the following box is kept just above Box B? (1) K (2) C (3) F (4) D (5) None of these Directions (638–342) : Study the following information carefully and answer the questions given below :

    BPRE–317

    Eight persons A, B, C, D, E, F, G, H are sitting around a circular table such that five of them are facing towards the centre and the rest are facing away from the centre. Three persons are sitting between F and H, who are facing centre. C is 2nd to the right of F and faces opposite direction to F. A sits 3rd to the left of C. G is one of the neighbours of E. Only two persons sit between G and B, who is not neighbour of H. G does not face C. G and A face same direction but opposite to F. (IBPS RRBs Officer CWE (Prelim Exam) 18.08.2018)

    638. What is the position of E with respect to A? (1) Immediate right (2) Fifth to the left (3) Second to the left (4) Second to the right (5) None of these 639. How many persons are sitting between C and H, when counted from the left of C? (1) One (2) Two (3) Three (4) Four (5) None 640. Four of the following five are alike in a certain way and hence they form a group. Which one among the following does not belong to that group? (1) B (2) C (3) F (4) D (5) E 641. Which of the following represents an immediate neighbour of G ? (1) C (2) B (3) F (4) D (5) A 642. Which of the following statements is not true regarding F? (1) It faces towards the centre. (2) E is to the immediate left of F. (3) Two persons sits between F and D, when counted from the right to D. (4) All are true (5) No one sits between F and B Directions (643–647) : Study the following information carefully and answer the questions given below : Certain number of persons are sitting in a row facing north. M sits 4th to the right of S. Only five persons sit between M and X. T sits at one of the positions left to S. The number of

    RANKING/ARRANGEMENT persons sitting between M and U are same as between S and T. Q is 2nd from one of the extreme ends. Only four persons sit between S and U. No one sits to the right of N, who is immediate right to P. X is 3rd left to P. Not more than two persons sit between Q and U. (IBPS RRBs Officer CWE (Prelim Exam) 18.08.2018)

    643. How many persons are sitting in the row? (1) 17 (2) 20 (3) 24 (4) 26 (5) 27 644. How many persons are sitting between S and T? (1) Seven (2) Six (3) Five (4) Four (5) Eight 645. What is the position of U from the left end? (1) 6th (2) 2nd (3) 4th (4) 5th (5) 3rd 646. How many persons are sitting between Q and M? (1) Seven (2) Eleven (3) Ten (4) Nine (5) Eight 647. Which of the following represents the person sitting at extreme end? (1) T (2) U (3) X (4) P (5) M Directions (648–652) : Study the following information carefully and answer the questions given below : Fourteen persons are sitting in two parallel rows such that seven persons are sitting in each row. A, B, C, D, E, F, G are sitting in row-1 facing north while P, Q, R, S, T, U, V are sitting in row-2 facing south. G sits third to the left of A and neither of them sits at an extreme end of the row. The one who faces A sits immediate right to T. Only one person sits between T and Q. The one who faces Q sits third to the right of E. S sits to the immediate left of V. S neither faces G nor E. D is an immediate neighbour of the one who faces S. The one who faces C sits fifth to the left of P. B sits third to the left of F. U sits at one of positions to the right of R. (IBPS RRBs Officer CWE (Prelim Exam) 18.08.2018)

    648. Four of the following are alike in a certain way and so they form a group. Which of the following does not belong to that group?

    (1) U (2) B (3) T (4) C (5) P 649. How many persons sits between F and C? (1) Two (2) One (3) None (4) Three (5) More than three 650. Which of the following statements is not true regarding U? (1) No one sits to the right of U (2) U sits third to the right of Q. (3) P is an immediate neighbour of U. (4) E is an immediate neighbour of the one who faces U. (5) Only two persons sit between U and S 651. What is the position of C with respect to A? (1) Second to the left (2) Third to the right (3) Immediate right (4) Immediate left (5) Second to the right 652. What is the position of B with respect to D? (1) Third to the left (2) Second to the left (3) Third to the right (4) Fourth to the left (5) Fifth to the right Directions (653–657) : Study the following information carefully and answer the questions given below : Ten boxes are kept in a stack one above the other. Only five boxes are kept between M and N. Only three boxes are kept between N and Q. As many boxes are kept between M and Q as between M and S. W is kept immediately above S. The number of boxes kept above W is one less than the number of boxes kept below T. T is not kept at the top of the stack. V is kept immediately above P. Only two boxes are kept between P and O. R is kept at one of the positions above O. (IBPS Bank PO/MT CWE (Prelim Exam) 14.10.2018)

    653. Which of the following boxes is kept immediately above and immediately below box O respectively? (1) P and Q (2) M and W (3) M and Q (4) W and N (5) N and P

    BPRE–318

    654. Which of the following statements is not true as per the given arrangement? (1) All the given statements are true (2) Only two boxes are kept between V and N (3) R is kept immediately above M (4) Only one box is kept above S. (5) P is kept at the third position from the bottom of the stack. 655. What is the position of O in the given stack of boxes? (1) Fifth from the top (2) Second from the bottom (3) Fourth from the top (4) Fifth from the bottom (5) Fourth from the bottom 656. T is related to V and O is related to R following a certain pattern based on the given arrangement. To which of the following is Q related to following the same pattern? (1) N (2) M (3) W (4) T (4) P 657. How many boxes are kept between R and S? (1) Four (2) One (3) Three (4) Two (5) None Directions (658–662) : Study the following information carefully and answer the questions given below : Twelve people are sitting in two parallel rows containing six people each, in such a way that there is equal distance between adjacent persons. In row-1 T, U, V, W, X and Y are seated (not necessarily in the same order) and all of them are facing south. In row–2, A, B, C, D, E and F are seated (not necessarily in the same order) and all of them are facing north. Therefore in the given seating arrangement each member seated in a row faces another member of the other row. V sits second from one of the extreme ends of the row. Only three people sit between the one who faces V and E. As many people sit to the left of E as to the right of B. T sits second to the right of X. Neither X nor T is an immediate neighbour of V. The one who faces A sits to the immediate right of T. Only one person sits between A and

    RANKING/ARRANGEMENT F. Neither U nor Y faces F. W faces an immediate neighbour of D. Y does not sit at any of the extreme ends of the row. (IBPS Bank PO/MT CWE (Prelim Exam) 14.10.2018)

    658. How many people sit between C and the one who faces Y? (1) Three (2) None (3) Four (3) One (5) Two 659. Who amongst the following faces U? (1) E (2) C (3) F (4) A (5) B 660. Who amongst the following sits third to the left of Y? (1) U (2) V (3) The one who faces E (4) W (5) The one who faces B 661. Four of the following five are alike in a certain way based on the given arrangement and so they form a group. Which is the one that does not belong to that group? (1) X (2) U (3) B (4) T (5) E 662. Which of the following statements is true with respect to the given information? (i) Only one person sits between W and Y. (ii) A is an immediate neighbour of C. (iii) D faces an immediate neighbour of U. (1) Both (i) and (ii) (2) Both (i) and (iii) (3) Only (ii) (4) Only (iii) (5) None of the given statements is true 663. Six people — A, B, C, D, E and F — are sitting around a circular table facing the centre. Only two people sit between B and E. C is an immediate neighbour of E. D sits second to the right of C. What is the position of A with respect to D? (1) Cannot be determined (2) Immediate left (3) Second to the right (4) Fourth to the left (5) Second to the left (IBPS Bank PO/MT CWE (Prelim Exam) 14.10.2018)

    Directions (664–667) : Study the following information carefully and answer the questions given below : A certain number of people are sitting in a straight line facing north with equal distance between each other. Only five people sit between A and B. B sits fifth to the right of C. D sits fourth to the right of C. D sits third from one of the extreme ends of the line. The one who likes Red colour sits seventh to the left of D. Only eight people sit between the one who likes Red colour and the one who likes Blue colour. The one who likes Blue colour sits at the extreme left end of the line. The one who likes Yellow colour sits to the immediate right of the one who likes Blue colour. Only seven people sit between the one who likes Yellow colour and E. The one who likes Orange colour sits sixth to the right of E. Only seven people sit between E and F. (IBPS Bank PO/MT CWE (Prelim Exam) 14.10.2018)

    664. How many people sit between A and the one who likes Yellow colour? (1) Six (2) Five (3) Seven (4) Nine (5) Three 665. As per the given arrangement, F is related to the one who likes Blue colour in the same way as B is related to D following a certain pattern. Following the same pattern who is C related to? (1) The one who likes Orange colour (2) A (3) F (4) The one who likes Red colour (5) B 666. What is the position of B with respect to the one who likes Orange colour? (1) Fifth to the left (2) Third to the left (3) Second to the right (4) Seventh to the left (5) Ninth to the right 667. How many people are sitting in the given line? (1) 19 (2) 18 (3) 23 (4) 21 (5) 15 Directions (668-672) : Study the following information carefully and answer the questions given below:

    BPRE–319

    Nine boxes named P, Q, R, S, T, U, V, W and X are placed one above other but not necessarily in the same order. Only five boxes are placed between P and R. T is placed immediate above R. Only three boxes are placed between T and S. As many boxes are placed between P and S as between Q and T. U is placed below Q, but not at bottom most position. More than four boxes are placed between T and U. Only one box is placed between U and V. Box X is placed above box W. (IBPS Bank PO/MT CWE (Prelim Exam) 21.10.2018)

    668. Which box is placed at the bottom most position ? (1) V (2) S (3) T (4) X (5) P 669. How many boxes are placed between X and P ? (1) One (2) Two (3) Three (4) Four (5) More than four 670. If in a certain way S is related to X and P is related to W then to which among the following Q is related ? (1) P (2) V (3) R (4) T (5) S 671. Which boxes are placed immediate above and immediate below Q? (1) R and W (2) X and S (3) U and X (4) S and U (5) T and R 672. What is the position of box W? (1) Fourth from the top (2) Fifth from the top (3) Third from the bottom (4) Sixth from the bottom (5) Fourth from the bottom Directions (673–675) : Study the following information carefully and answer the questions given below: A certain number of persons are seated in a row. The Row is arranged in a vertical manner and all are facing north direction. Ranjan sits fourth from the left end of the row. There are only two persons between Ranjan and Seema. Puja sits to the immediate right of Seema. There are as many persons between Puja and Seema as between Dinesh and Puja. Dinesh does not sit to the left of Ranjan. (IBPS Bank PO/MT CWE (Prelim Exam) 21.10.2018)

    RANKING/ARRANGEMENT 673. How many persons sit between Ranjan and Dinesh? (1) 3 (2) None (3) 5 (4) 4 (5) 1 674. What is the position of Puja with respect to Ranjan? (1) Immediate right (2) Third to the left (3) None of these (4) Second to the left (5) Fourth to the right 675. How many persons sit in a row? (1) 4 (2) 6 (3) 9 (4) 11 (5) 8 Directions (676–680): Study the following information carefully and answer the questions given below: 12 students A, B, C, D, E, F, G, H, I, J, K and L who have secured top 12 ranks are, sitting around a square table facing the centre. ● J is sitting third of a student who is to the immediate right of K. G is sitting second to the right of a student who is sitting to the immediate left of E ● D is sitting exactly between the students who have secured 2nd and 7th ranks. H is sitting immediate right of a person who is sitting opposite to the student with 12th rank ● A is sitting to the immediate left of a student who is sitting second to the left of I who has secured 9th rank. ● C is sitting opposite to a student who is 3rd rank holder and is an immediate neighbour of J and 10th rank holder ● F is sitting to the immediate left of 2nd rank holder and fifth to the left of I. ● C is immediate right of a student who is sitting to the immediate right of E with 4th or 10th rank ● K is sitting third to the left of a student with 6th rank who is sitting to the immediate left of E ● The immediate right neighbour of I is sitting opposite to H who is sitting second to the left of J. ● The person second left of A secured 5th rank and G is either 7th or 8th rank holder.



    The person opposite to L holds 11th rank whose neighbour is the topper.

    (IBPS Bank PO/MT CWE (Main Exam) 18.11.2018)

    676. Who among the following is sitting exactly between 9th and 10th rank holders? (1) F (2) J (3) E (4) K (5) L 677. What is the average rank of the students B, G and E? (1) 5 (2) 4 (3) 3 (4) 6 (5) 7 678. What is the rank of the student who is sitting fifth to the right of C? (1) 12th (2) 10th (3) 3rd (4) 1st (5) 4th 679. What are the respective ranks secured by J and the student who is sitting opposite to J? (1) 10 and 2 (2) 7 and 4 (3) 1 and 9 (4) 6 and 5 (5) 4 and 3 680. Which of the following statements is true with respect to the given arrangement? (1) D secured 5th rank and sits to the immediate right of J (2) C secured 1st rank and sits to the immediate right of F (3) L secured 10th rank and sits to the immediate right of K (4) G secured 4th rank and sits to the immediate left of C (5) None of the statements is true 681. Eight persons A, B, C, D, E, F, G and H are sitting around a circular table facing the centre, but not necessarily in the same order. F sits to the immediate right of D and third to the left of A. G sits third to the left of D who does not sit next to E. B sits next to G but not next to D. C does not sit next to either D or A. What is the position of H with respect to C ? (1) Third to the left (2) Third to the right (3) Second to the right (4) Fifth to the left (5) Fourth to the right (Canara Bank PO Exam, 23.12.2018)

    BPRE–320

    Directions (682–686) : Study the following information carefully and answer the questions given below : Eight friends — A, B, C, D, E, F, G and H — are sitting around a circular table with equal distance between them but not necessarily in the same order. Some of them are facing the centre while some face outside (i.e. opposite to centre). ● D sits third to the left of H. H faces the centre. E sits second to the right of D. ● Only one person sits between E and A. Immediate neighbours of A face opposite directions. (i.e. if one neighbour faces the centre then the other neighbour faces outside and vice-versa.) ● Only three people sit between B and G. B is not an immediate neighbour of A. ● F sits second to the left of B. Immediate neighbours of F face the same direction (i.e. if one neighbour faces the centre then the other also faces centre and viceversa.) ● Immediate neighbours of C face opposite directions (i.e. if one neighbour faces the centre then the other neighbour faces outside and vice-versa). ● Both E and A face a direction same as that of D (i.e. if D faces the centre then both E and A face the centre and vice-versa.) (Canara Bank PO Exam, 23.12.2018)

    682. Which of the following statements is true regarding C as per the given arrangement? (1) None of the given options is true (2) Only two people sit between C and E (3) A sits second to the left of C. (4) G faces the same direction as that of C. (5) Both G and D are immediate neighbours of C. 683. How many people sit between F and H when counted from the left of H? (1) None (2) Three (3) More than three (4) One (5) Two 684. Who sits third to the right of E? (1) G (2) D (3) B (4) C (5) F

    RANKING/ARRANGEMENT 685. What is the position of G with respect to A? (1) Second to the right (2) Third to the right (3) Immediate left (4) Immediate right (5) Second to the left 686. Four of the following five are alike in a certain way based on the given arrangement and so they form a group. Which is the one that does not belong to that group? (1) B (2) D (3) F (4) C (5) G Directions (687–692) : Study the following information carefully and answer the questions given below : Eight people viz., L, M, N, O, P, Q, R and S are sitting on eight different steps of the same flight of stairs, but not necessarily in the same order. The lowermost step is numbered one, the one above that is numbered two and so on till the topmost step is numbered eight. Each one of them has different years of experience in the field of banking viz., 10 years, 13 years, 15 years, 18 years, 20 years, 23 years, 25 years and 28 years but not necessarily in the same order. R is the most experienced and is sitting on an even numbered step but not on step number 2. The one who has less experience than only R is sitting on the lowermost step. P is sitting on a step which is immediately below the step on which R is sitting. L is the least experienced. L is sitting on a step which is immediately below the step on which N is sitting. Only three people sit between L and S. S has 20 years of experience. Q has more experience than only L. Q is sitting on a step which is immediately below O’s step. The one who has 23 years of experience is sitting on a step which is immediately above the step on which S is sitting. N has more experience than O. (Canara Bank PO Exam, 23.12.2018)

    687. Which of the following statement is/are true as per the given arrangement? (1) S is sitting on step number 6 (2) M is sitting on step number 2 (3) None of the given statements is true (4) N has 13 years of experience (5) Q has 15 years of experience

    688. How many people are sitting between the one who has 15 years of experience and M? (1) Two (2) None (3) Three (4) One (5) Four 689. Who amongst the following is sitting on step number 3? (1) Q (2) N (3) M (4) L (5) S 690. Four of the following five are alike in a certain way based on the given arrangement and hence they form a group. Which is the one that does not belong to that group? (1) Step number 5 — Q (2) R — Step number 7 (3) N — Step number 2 (4) O — M (5) The one who has 15 years of experience — Q 691. Which of the following statements is true with respect to O ? (1) S sits immediately below O (2) Only two persons sit between O and S (3) None of the given statements is true (4) R sits below O (5) O is sitting on step number 5 692. How much experience does M has ? (1) 13 years (2) 23 years (3) 15 years (4) 18 years (5) 25 years Directions (693–697) : Study the following information carefully and answer the questions given below : Twelve people are sitting in two parallel rows containing six people each, in such a way that there is equal distance between adjacent persons. In row 1- G, H, I, J, K and L are seated (not necessarily in the same order) and all of them are facing south. In row 2R, S, T, U, V and W are seated and all of them are facing north. Therefore, in the given seating arrangement, each member seated in a row faces another member of the other row. I sits third to the left of the person facing V. Neither I nor G sits at any of the extreme ends of the line. Only one person sits between I and G. W sits second to the right of T. T is not an immediate neighbour of V. T does not face I. The one who faces K sits to the immediate left of R. K is not

    BPRE–321

    an immediate neighbour of G. Only one person sits between L and the one who faces U. J is not an immediate neighbour of I. (Canara Bank PO Exam, 23.12.2018)

    693. What is the position of T with respect to V? (1) Third to the left (2) Third to the right (3) Second to the right (4) Immediate right (5) Second to the left 694. Who amongst the following faces G? (1) V (2) R (3) U (4) T (5) S 695. Who amongst the following sits exactly between L and the one who faces U? (1) G (2) K (3) J (4) The one who faces S (5) The one who faces R 696. Four of the following five are alike in a certain way based on the given arrangement and hence they form a group. Which one of them does not belong to that group? (1) W (2) V (3) L (4) J (5) R 697. Which of the following statements is true regarding H ? (1) None of the given statements is true (2) H sits third to the right of I. (3) H is an immediate neighbour of K (4) Only two persons sit between H and L (5) H faces one of the immediate neighbours of R Directions (698–702) : Study the following information carefully and answer the questions given below : Eight people viz. U, V, P, O, R, J, A and B are sitting around a square table such that four people sit at the four corners of the table and four people sit in the middle of each of the four sides. The ones sitting at the corners of the table are facing the centre and the others sitting in the middle of the sides are facing outside (i.e. opposite to the centre). V is an immediate neighbour of P. P sits at one of the corners of the table. Only three people sit between A

    RANKING/ARRANGEMENT and O. A sits in one of the middle sides of the table. O is not an immediate neighbour of P. Only one person sits between B and O. There are only two persons between R and O. J sits to the immediate right of B. (IBPS Specialist Officer CWE (Prelim Exam) 31.01.2019)

    698. Four of the following five are alike in a certain way based on the given arrangement and thus they form a group. Which one of the following does not belong to the group? (1) V (2) B (3) P (4) A (5) O 699. Who sits to the immediate right of V? (1) A (2) U (3) O (4) P (5) J 700. How many people sit between A and O when counted from the right of A? (1) Three (2) More than three (3) Two (4) None (5) One 701. Which of the following statements is true about J as per the given arrangement? (1) J sits to the immediate right of A. (2) V sits second to the left of J. (3) J faces outside. (4) Only two people sit between J and V. (5) None of the given options is true 702. Which of the given pairs represent the immediate neighbours of B? (1) O, J (2) J, R (3) A, O (4) P, R (5) U, R Directions (703-707) : Study the following information carefully and answer the questions given below : Ten people are sitting in two parallel rows containing five people each in such a way that there is equal distance between adjacent persons. In row-1 A, B, C, D and E are seated (not necessarily in the same order) and all of them are facing north. In row-2 P, Q, R, S and T are seated (not necessarily in the same order) and all of them are facing south.

    Therefore in the given seating arrangement each member seated in a row faces another member of the other row. Each one of them also likes different perfumes- Davidoff, Cheque, Brut, Gucci, Hollister, Titan, Police, Valentino, Yardley and Ferrari, but not necessarily in the same order. S sits at one of the extreme ends of the row. A faces an immediate neighbour of S. Only one person sits between A and the one who likes Brut. T sits third to the right of P. P faces the one who likes Davidoff. C is an immediate neighbour of A. C faces Q. The one who sits second to the right of Q likes Yardley. Only two people sit between the one who likes Yardley and the one who likes Valentino. B sits to the immediate right of E. D faces one of the immediate neighbours of the one who likes Gucci. The one who likes Police is an immediate neighbour of T. The one who likes Ferrari sits second to the right of the one who likes Hollister. P does not like Cheque. (IBPS Specialist Officer CWE (Prelim Exam) 31.01.2019)

    703. As per the given arrangement, which of the given statements is true? (1) T is an immediate neighbour of the one who likes Brut. (2) None of the given statements is true. (3) Only one person sits between A and the one who faces T. (4) B faces the one who likes Titan. (5) A sits second to the right of E. 704. As per the given arrangement, A is related to D in the same way as T is related to Q. Following the same pattern, to which is S related? (1) The one who likes Cheque (2) P (3) R (4) The one who faces A (5) The one who likes Yardley 705. Who amongst the following faces E? (1) Q (2) R (3) S (4) T (5) Other than those given as options 706. Who sits exactly between Q and the one who faces D?

    BPRE–322

    (1) R (2) S (3) T (4) The one who likes Police (5) The one who likes Titan 707. Who is second to the left of C? (1) The one who faces Q. (2) B (3) A (4) The one who likes Ferrari (5) The one who likes Hollister Directions (708–712) : Study the following information carefully and answer the questions given below: (IBPS RRBs Officer CWE Prelim Exam, 04.08.2019)

    Eight people are sitting in a straight line with equal distance between adjacent persons. Some of them are facing north while some of them are facing south. A sits second from one of the extreme ends of the line and faces north. H sits to the immediate right of A. B sits third to the right of H. B and H face opposite directions. G sits to the immediate left of B. D sits third to the left of G. F sits second to the right of D. C sits to the immediate left of F. E sits at one of the positions to the left of C. E faces the same direction as C. 708. As many people sit to the left of C as to the left of _______. (1) B (2) A (3) D (4) H (5) E 709. Which of the following statements is true as per the given arrangement? (1) Only two people sit to the right of F. (2) None of the given statements is true. (3) H sits at one of the positions to the left of G. (4) F sits second to the left of B. (5) C is an immediate neighbour of G. 710. As per the given arrangement, how many people are facing south? (1) Two (2) More than four (3) One (4) Four (5) Three 711. Four of the following five are alike in a certain way based on their direction as per the given arrangement and thus they form a group. Which one of the following does not belong to that group?

    RANKING/ARRANGEMENT (1) GB (2) BD (3) EH (4) DF (5) FG 712. Which of the following pairs represents the immediate neighbours of E? (1) B, A (2) A, H (3) G, A (4) G, H (5) B, C Directions (713–717) : Study the following information carefully and answer the questions given below: (IBPS RRBs Officer CWE Prelim Exam, 04.08.2019)

    Eleven boxes are kept one above the other in a stack. Only five boxes are kept between box F and box K. Box T is kept immediately below box F. More than five boxes are kept above box T. Only two boxes are kept between box T and box S. Only five boxes are kept between box S and box M. Box M is kept at one of the positions above box S. Only two boxes are kept between box M and box R. Only three boxes are kept between box R and box H. Box H is kept at one of the positions below box R. Only one box is kept between box H and box J. Only three boxes are kept above box J. Box W is kept at one of the positions below box D but above box A. 713. What is the position of box R in the given stack of boxes ? (1) Immediately above box W (2) Fifth from the top (3) Third from the bottom (4) Second from the top (5) Immediately below box J 714. Which of the following statements is true as per the given arrangement ? (1) None of the given statements is true (2) Only one box is kept between box D and box S. (3) Box M is kept at one of the positions below box H. (4) Only four boxes are kept below box T. (5) Box S is kept at the bottom of the stack. 715. Four of the following five are alike in a certain way as per the given arrangement and thus they form a group. Which one of the following does not belong to that group? (1) RS (2) DH (3) MT (4) KJ (5) FA

    716. How many boxes are kept between box J and box F ? (1) Four (2) Three (3) Two (4) None (5) Five 717. Which box is kept immediately above and immediately below box A respectively ? (1) H, T (2) W, S (3) T, S (4) W, T (5) S, H Directions (718–723) : Study the following information carefully and answer the questions given below : (IBPS RRBs Officer CWE Prelim Exam, 17.08.2019)

    Eight friends P, Q, R, S, W, X, Y and Z are sitting around a circular table in a café facing the centre but not necessarily in the same order. Each of them orders different types of coffees viz. Vienna, Affogato, Irish, Mochaccino, Macchiato, Espresso, Cappuccino and Latte, but not necessarily in the same order. R orders Cappuccino and sits third to the right of W. Those who order Espresso and Mochaccino are immediate neighbours of each other. Those who order Affogato and Latte are immediate neighbours of each other. Those who order Espresso and Mochaccino are immediate neighbours of neither W nor R. Neither X nor S orders Latte. The one who orders Vienna, sits opposite to the Q, who orders Irish. P orders Affogato but he is not an immediate neighbour of the one who orders Vienna. Z sits fourth to the right of the one who orders Latte. Y sits third to the right of X, who does not order Espresso. 718. Which of the following coffee Y orders? (1) Mochaccino (2) Espresso (3) Latte (4) Vienna (5) Other than those given as options 719. Who among the following orders Macchiato? (1) S (2) X (3) Y (4) W (5) None of these 720. Who among the following sits opposite to Q? (1) Y (2) S (3) W (4) X (5) None of these

    BPRE–323

    721. What is the position of S with respect to Z? (1) Seocnd to the left (2) Second to the right (3) Immediate right (4) Third to the left (5) None of these 722. How many friends sit between X and R, when counted from the left of R? (1) Two (2) Three (3) One (4) Four (5) None Directions (723–727) : Study the following information carefully and answer the questions given below : (IBPS RRBs Officer CWE Prelim Exam, 17.08.2019)

    Eight persons A, B, C, D, W, X, Y and Z are sitting in two parallel rows viz row-1 and row-2. All of them are facing either north or south. Only two persons in each row are facing north direction. C sits opposite to the one who sits second to the left of D. D sits in row2. Y sits second to the left of W. Both W and Y face same direction. X and C sit in opposite rows. W sits opposite to X. Both the immediate neighbours of X face opposite directions. A and Z face same direction. Y does not face north. A and C do not face each other. Z is not the immediate neighbour of X and W. Y is not the immediate neighbour of C. A does not sit opposite to D. 723. Four among the following five are alike in a certain way and thus they form a group. Find the one that does not belong to a group? (1) W, X (2) B, D (3) Y, Z (4) C, X (5) A, W 724. Who among the following sits second to the left of B? (1) X (2) Z (3) D (4) C (5) None of these 725. Which of the following statements is true regarding Z? (1) Z sits second to the right of A. (2) Z sits opposite to B. (3) Z sits at an extreme end of the row. (4) Z faces north direction. (5) All of the above are true

    RANKING/ARRANGEMENT 726. Who among the following sits second to the right of Z? (1) A (2) X (3) D (4) W (5) C 727. Who among the following sits second to the right of the one who sits opposite to A? (1) C (2) X (3) B (4) Z (5) Other than those given as options Directions (728–732) : Study the following information carefully and answer the questions given below : (IBPS Bank PO/MT CWE Prelim Exam, 12.10.2019)

    Eight persons A to H are sitting around the circular table but not necessarily in same order. Some of them are facing centre and some of them are facing opposite to the centre. Note: Persons facing the same direction means if one is facing centre, other person is also facing centre and vice versa. Persons facing opposite directions means if one is facing centre, other person is facing opposite to the centre and vice versa. A sits third to the left of D. Only two persons are sitting between A and B. B sits second to the left of F. C sits third to the left of G. G is not the immediate neighbour of F. H sits second to the left of E. H faces the same direction as F (Either facing centre or opposite to the centre). E is not an immediate neighbour of G. C faces opposite direction of E. Immediate neighbours of E faces same direction. A and B face opposite directions to each other. B doesn’t face opposite to the centre. 728. Who sits second to the left of the one who sits third to the right of A? (1) D (2) F (3) E (4) C (5) None of these 729. Who is an immediate neighbour of F? (1) A (2) C (3) B (4) Both A and C (5) None of these 730. Who sits third to the left of H? (1) A (2) C (3) F (4) B (5) G

    731. Four of the following five are alike in a certain way as per the given arrangement and they form a group. Which of the following does not belong to that group? (1) D (2) G (3) F (4) B (5) A 732. Who amongst the following person is facing B? (1) G (2) A (3) H (4) C (5) D Directions (733–737) : Study the following information carefully and answer the questions given below : (IBPS Bank PO/MT CWE Prelim Exam, 12.10.2019)

    Ten people are sitting in park on a bench, they are in a linear row and they are facing either north or south direction. The number of people facing south is more than the number of people facing north. The person sitting at the ends are facing different directions. Not more than three persons sitting together are facing the same direction. L is five places away from N, who is also sitting at an end. N is closer to L than to M. More than two persons are sitting between M and K. N and O are facing the same direction. K sits third to the right of I. N sits to the left of P and I sits to the right of K. O sits in the middle of R and N. R is an immediate neighbour of L and M and they all are facing the same direction. J sits to the immediate left of Q, who sits at an end. 733. How many people facing north are sitting between P and M? (1) Two (2) One (3) Three (4) None (5) More than three 734. Who is sitting fourth to the left of the one who is sitting third to the left of O? (1) O (2) Q (3) M (4) P (5) K 735. If L is related to P and J in a certain way, in the same way, who is related to O? (1) O (2) Q (3) N (4) P (5) K 736. Which of the following statements is/are true with respect to the given information?

    BPRE–324

    (1) O is two places away from N. (2) L sits second to the left of M. (3) Q faces north and N faces south. (4) More number of people is sitting to the right of L than to the left of L. (5) All the given statements are true 737. How many people are sitting between I and K? (1) Two (2) One (3) Three (4) More than three (5) None Directions (738 – 742) : Study the following information carefully and answer the questions given below : (IBPS Bank PO/MT CWE Prelim Exam, 19.10.2019)

    Nine people are sitting in a straight line with equal distance between adjacent persons. Some of them are facing north while some of them are facing south. More than four people face north. P sits second from one of the extreme ends of the line. Only two people sit between P and R. Q sits third to the right of R. B sits second to the left of Q. Only three people sit between Q and C. D sits to the immediate left of C. D faces a direction opposite to that of C. Immediate neighbours of D face opposite directions. A sits second to the left of E. A does not sit at an extreme end of the line. S sits at one of the positions to the left of A. S sits at one of the positions to the right of B. S faces a direction opposite to that of B. 738. How many people sit between P and B? (1) More than three (2) One (3) Three (4) None (5) Two 739. What is the position of B with respect to E? (1) Third to the right (2) Third to the left (3) Immediate right (4) Second to the right (5) Immediate left

    RANKING/ARRANGEMENT 740. Who amongst the following sit at the extreme ends of the line? (1) E, S (2) B, Q (3) S, Q (4) Q, E (5) S, B 741. Which of the following statements is true as per the given arrangement? (1) B is an immediate neighbour of both A and C. (2) None of the given statements is true (3) Less than two people sit between P and Q. (4) Q sits to the immediate left of A. (5) Only one person sits between R and D. 742. Four of the following five pairs are alike in a certain way as per the given arrangement and thus they form a group. Which one of the following does not belong to that group? (1) SC (2) CR (3) RE (4) PS (5) AR Directions (743 – 747) : Study the following information carefully and answer the questions given below : (IBPS Bank PO/MT CWE Prelim Exam, 19.10.2019)

    Six people are sitting around a circular table facing the centre. Each one of them is of a different age. Only two people sit between the A and the 15 year old. P is an immediate neighbour of the 15 year old. The 20 year old sits second to the right of P. R is 18 years old and is also an immediate neighbour of the 20 year old. The sum of the present ages of R and the one who sits to the immediate left of R is 48 years. J sits second to the right of the 40 year old. S is 15 years younger than W. W’s present age is an even number. 743. What is the present age (in years) of the one who sits to the immediate left of W? (1) 55 (2) 20 (3) 40 (4) 30 (5) 15 744. What is the sum of the present ages of S and R (in years)? (1) 43 (2) 45 (3) 33 (4) 65 (5) 38

    745. Whose present age, from amongst the following is an odd number? (1) Only P (2) Both A and P (3) Both J and A (4) Only A (5) Both S and J 746. Which of the following statements is true with respect to the given information? (1) The sum of the presents ages of A and W is 58 years. (2) R is older than P. (3) None of the given statements is true. (4) W is 40 years old. (5) J is an immediate neighbour of the 20 years old. 747. Who sits second to the right of S? (1) A (2) The 18 year old (3) P (4) W (5) The 15 year old Directions (748-752) : Study the following information carefully and answer the questions given below : (IBPS Bank PO/MT CWE Main Exam, 31.11.2019)

    Twelve friends P, Q, R, S, T, U, V, W, X, Y, Z and B are sitting around a circular table facing the centre. There are three categories of friends : (1) Friends who drink 2 litres of water per day (2) Friends who drink 3 litres of water per day (3) Friends who drink 4 litres of water per day The friends who drink 2 litres, 3 litres and 4 litres of water respectively, sit one by one in ascending order in clockwise direction. ˆ Y sits to the immediate left of a person who sits second to the right of R. ˆ S sits second to the left of a person who sits second to the left of U. ˆ T is an immediate neighbour of either V or S. ˆ V sits third to the right of a person who sits third to the right of Q.

    BPRE–325

    ˆ Z sits to the immediate left of a person who sits second to the right of W. ˆ X sits second to the right of a person who sits third to the right of B. ˆ U sits third to the right of a person who sits third to the right of X. ˆ P sits second to the right of a person who sits third to the left of S. ˆ B drinks 2 litres water per day and Z sits exactly between Q and W. ˆ R is an immediate neighbour of either B or P. 748. Q and S together drink how many liters of water per day? (1) 4 litres (2) 6 litres (3) 8 litres (4) 5 litres (5) 7 litres 749. Who among the following sit exactly between B and T when we proceed to the right of B ? (1) S and P (2) R and Y (3) V and W (4) Q and S (5) X and Z 750. Who among the following sits second to the right of a person who drinks 4 litres of water per day ? (1) T (2) Q (3) R (4) S (5) V 751. How many litres of water are consumed by the immediate neighbours of W per day? (1) 8 litres (2) 5 litres (3) 7 litres (4) 4 litres (5) 6 litres 752. Who among the following drinks more quantity of water? (1) Z (2) B (3) Y (4) R (5) X Directions (753–757) : Study the following information carefully and answer the questions given below : (IBPS Bank PO/MT CWE Main Exam, 31.11.2019)

    A certain number of chairs are arranged in a row from left to right facing the north direction. Some girls are sitting on the chairs.

    RANKING/ARRANGEMENT There is a vacant chair exactly between R and S. M sits fourth to the right a vacant chair which is to the immediate left of G. J sits second to right of the one who sits sixth to left of U. Only three chairs are between A and K. Either I or G sits at the extreme right end. A sits fourth to the right of the one who sits tenth to left of B. Only one chair is vacant between E and J. Either A or K sits at the extreme left end. B sits fifth to right of a vacant chair which is sixth to the left of a vacant chair. A sits 9th to the left of U and only five chairs are occupied between them. There are four more chairs to the right of N than to the left of her. S has only one immediate neighbour to her left and M has only one immediate right neighbour. B sits to the immediate left of the one who sits third to the right of a vacant chair. A sits third to the right of the one who sits sixth to the left of E. There are nine chairs between R and N. 753. How many vacant chairs are there between R and G ? (1) Two (2) Three (3) Four (4) Five (5) One 754. How many girls are sitting to the right of E ? (1) Two (2) Three (3) Four (4) Five (5) Six 755. Who sits exactly between S and M? (1) B (2) N (3) A (4) G (5) Av 756. Who sits to the right of U but to the left of I ? (1) E (2) G (3) M (4) A (5) B 757. How many girls are sitting to the right of A but to the left of M ? (1) Three (2) Four (3) Five (4) Six (5) Seven Directions (758–762) : Study the following information carefully and answer the questions given below : (IBPS Bank PO/MT CWE Main Exam, 31.11.2019)

    Eight family members P, Q, R, S, T, U, V and W are sitting in a row facing north. There are three generations

    and three couples in the family. Each couple has at least one child. U and R are not blood related whereas Q and T are blood related. R does not have blood relation with anyone in the family except P. Q sits to the immediate left of a person who is son of U. V sits to the immediate right of S who is father of Q. S and W are blood related whereas S and U are not blood related. R and Q sit immediately right to their respective spouse. W sits fourth to the right of G’s daughter and S sits third to the left of Q’s sister. U does not have blood relation with anyone in the family except her son W. 758. Who sits second to the right of T’s spouse? (1) U (2) Q (3) R (4) V (5) W 759. Who sits fourth to the right of T ? (1) T’s spouse (2) Q’s Niece (3) R’s daughter (4) U’s Son (5) V’s son 760. Who sits exactly between P and S? (1) P’s grandmother (2) R’s father-in-law (3) U’s brother-in-law (4) W’s cousin (5) Q’s Brother 761. Who sits to the immediate left of W? (1) S’s son-in-law (2) S’s daughter-in-law (3) P’s maternal uncle (4) W’s maternal uncle (5) R’s Mother 762. Who sits at the extreme left end? (1) U’s brother (2) R’s mother-in-law (3) Q’s niece (4) T’s niece (5) V’s spouse Directions (763–767) : Study the following information carefully and answer the questions given below : (IBPS RRBs Officer Scale-I CWE Main Exam, 13.10.2019)

    There are certain number of chairs arranged in a row and all the chairs are facing north. There are few people sitting on few chairs in the row.

    BPRE–326

    l B sits to the immediate left of a

    person who sits third to the right of a vacant chair l A sits third to the right of a person who sits sixth to the left of E and there are only nine chairs between K and H. l One vacant chair is placed exactly between I and C. G sits fourth to the right of a vacant chair which is to the immediate left of F. l J sits second to the right of a person who sits sixth to the left of C. There are only three chairs between A and L. l B sits fifth to the right of a vacant chair which is sixth to the left of a vacant chair. l There are eight chairs between A and C and five chairs are occupied. l The number of chairs to the right side of A is 3 times the number of chairs to the left side of A. l Either K or F sits at the right end and either A or L is at the left end of the row. l A sits fourth to the right of a person who sits tenth to the left of B. There is one vacant chair between E and J. l D has only one immediate left neighbour. G has only one immediate right neighbour. 763. Who among the following sits sixth to the left from the right end of the row? (1) H (2) B (3) J (4) E (5) A vacant chair 764. How many vacant chairs are between E and G? (1) One (2) Two (3) Three (4) Four (5) No vacant chair 765. What is total number of chairs placed in the row? (1) 17 (2) 10 (3) 18 (4) 15 (5) 12 766. Who among the following sits exactly between J and I? (1) E (2) B (3) A (4) H (5) A vacant chair

    RANKING/ARRANGEMENT 767. How many chairs are occupied between D and G? (1) 10 (2) 8 (3) 11 (4) 9 (5) 14 Directions (768–772): Study the following information carefully and answer the questions given below : (IBPS RRBs Officer Scale-I CWE Main Exam, 13.10.2019)

    Twelve students B, K, D, H, G, A, J, L, E, F, C and I are sitting in two rows, one behind the other facing the teacher in such a way that six students sit in each row. The teacher is facing south. l B sits immediate right above diagonally of a student who sits second to the left of L. l K sits opposite to a student who sits immediate left above diagonally of D. l F sits immediate right below diagonally of a student who sits second to the right of G. l J sits to the immediate left of a student who sits immediate right above diagonally of A. l C sits immediate right above diagonally of a student who sits to the immediate left of F. l L sits opposite to a student who sits immediate left above diagonally of A. l C sits at one of the ends of either row. l D sits to the immediate left of a student who sits second to the left of E diagonally. l H sits immediate left above diagonally of a student who sits opposite to B. 768. Who among the following sits exactly between D and A? (1) H (2) B (3) L (4) I (5) F 769. Who among the following sits immediate right below diagonally of the student sitting at the left end of row ? (1) D (2) I (3) B (4) C (5) K 770. Who among the following sits third to the left of F’s immediate neighbour ?

    (1) H (2) D (3) L (4) G (5) J 771. Which of the following pairs of students sits at the ends of either row ? (1) C and D (2) B and K (3) J and E (4) H and F (5) A and K 772. Which of the following pairs of students sits immediate below diagonally of B? (1) G and A (2) C and I (3) H and L (4) E and D (5) K and L Directions (773–777): Study the following information carefully and answer the questions given below : (IBPS RRBs Officer Scale-I CWE Main Exam, 13.10.2019)

    Twelve friends D, E, F, G, H, A, J, K, L, M, N and B who were born in different years, are sitting around a circular table facing the centre. The eldest friend was born in 1990 while the youngest friend was born in 2001. All the friends are sitting according to the descending order of their ages in clockwise direction starting from the friend who was born in 1990. l H sits third to the right of A and J sits third to the right of the eldest friend. l E sits fourth to the left of the friend born in 1991 and K is three years younger to B. l A sits to the immediate left of the friend who sits third to the right of L. l D sits second to the right of the friend who is third to the right of H. l L sits to the immediate left of the friend who is second to the right of the friend born in 1999. l N is an immediate neighbour of both the eldest friend and the friend born in a leap year. l J was born before D but after G. l M is an immediate neighbour of either B or K.

    BPRE–327

    l B sits fourth to the left of the

    friend who is third to the left of the friend born in 1996. l F sits fourth to the left of the friend who is to the immediate right of N. 773. What is the difference between the ages of F and L (in years)? (1) 3 (2) 6 (3) 9 (4) 2 (5) 4 774. What is the position of E with respect ot J? (1) Third to the left (2) Fourth to the right (3) Fifth to the left (4) Second to the right (5) Immediate left 775. Who among the following sits fourth to the right of the youngest friend ? (1) H (2) M (2) K (4) B (5) G 776. Who among the following is four years elder to D? (1) B (2) K (3) N (4) A (5) E 777. Who among the following was born in the lowest leap year ? (1) N (2) A (3) F (4) G (5) H Directions (778–782) : Study the following information carefully and answer the questions given below : (IBPS SO Prelim Exam, 28.12.2019)

    Seven people bought seven different gadgets consecutively one after the other on different occasions. No person other than those given bought a gadget on the given occasion or any other occasion. Only three people bought a gadget between A and the one who bought a phone. Only two people bought a gadget between the one who bought a phone and the one who bought a tablet. Only three people bought a gadget between B and the one who bought a tablet. As many people bought a gadget before B as after C. Only two people bought a gadget between C and the one who bought a TV. G bought a gadget before D as after E. F bought a gadget on one of the occasions before

    RANKING/ARRANGEMENT E. Only three people bought a gadget between F and the one who bought a speaker. The one who bought a watch, bought it on one of the occasions before the one who bought a speaker but after the one who bought headphones. The one who bought a laptop bought on one of the occasions after G. 778. Four of the following five are alike in a certain way based on their positions as per the given arrangement and thus form a group. Which one of the following does not belong to that group? (1) F – TV (2) E – Tablet (3) G – Speaker (4) C – Headphones (5) B – Watch 779. Which of the following gadgets did G buy? (1) TV (2) Speaker (3) Tablet (4) Watch (5) Headphones 780. How many people bought a gadget between F and D? (1) None (2) More than three (3) One (4) Three (5) Two 781. Which of the following statements is true as per the given information? (1) None of the given statements is true (2) No one bought a gadget before B. (3) Only two people bought a gadget between the one who bought a speaker and the one who bought a TV. (4) Only two people bought a gadget after E. (5) D bought a gadget on one of the occasions after A. 782. Who amongst the following bought a laptop? (1) E (2) B (3) C (4) D (5) A Directions (783–787) : Study the following information carefully and answer the questions given below : (IBPS SO Prelim Exam, 28.12.2019)

    Nine people are sitting a straight line. Some of them face north and some face south. P sits sixth from one of the extreme ends of the line and faces north. R sits second to the right of P. Q sits fourth to the left of R. Q and R face opposite directions. The number of people sitting to the right of Q is one less than that sitting to the right of S. Q and S are not immediate neighbours of each other. M sits second to the right of S. Only three people sit between M and B. G sits second to the right of B. More than three people sit to the left of G. W does not sit at any of the extreme ends of the line. Immediate neighbours of W face opposite directions. Both M and X face a direction opposite to that of W. 783. How many people sit between Q and the one who sits third to the right of M? (1) None (2) One (3) Three (4) More than three (5) Two 784. As per the given information, four of the following five are alike in a certain way based on their directions and hence they form a group. Which one of the following does not belong to the group? (1) B (2) X (3) G (4) S (5) Q 785. Which of the following statements is NOT TRUE with respect to the given information? (1) S sits third to the left of R. (2) Q sits second from one of the extreme ends of the line. (3) As many people sit to the right of G as to the right of S. (4) G and M face the same direction. (5) B and R are immediate neighbours of each other. 786. How many people sit to the right of X? (1) One (2) More than three (3) Three (4) Two (5) None

    BPRE–328

    787. Who sits fourth to the left of W? (1) M (2) P (3) G (4) S (5) Q Directions (788–792) : Study the following information carefully and answer the questions given below : (IBPS SO Prelim Exam, 28.12.2019)

    Six people – A, B, C, D, E and F – work in the same office but at a different post – Clerk (CL), Probationary Officer (PO), Assistant Manager (AM), Manager (MG), General Manager (GM) and Executive Director (ED). It is assumed that only the given posts exist in the said office and only one person works on each post. Each one of them is of a different age (in years). (Note: The posts have been given in increasing order of seniority with Clerk (CL) being the junior most position whilst the Executive Director (ED) being the senior most position.) E is an immediate senior of the one who is 38 year old. E is junior than the MG. The sum of present ages of the CL and the GM is 80 years. B is senior than E but junior than the 60 year old. The present age of the ED is an odd number. A is an immediate junior of the 51 year old. D is 9 years older than A. D is the not the 51 year old. C is junior than the 29 year old. 788. Which combination is correct as per the given arrangement? (1) CL – A (2) ED – The 29 year old (3) AM – The 60 year old (4) PO – C (5) MG – B 789. Who amongst the following is the GM? (1) An immediate junior of B (2) An immediate senior of D (3) D (4) The 60 year old (5) C 790. How many posts are there between the ones on which C and the 38 year old work? (1) Two (2) None (3) One (4) More than three (5) Three

    RANKING/ARRANGEMENT 791. Four of the following five are alike in a certain way based on their posts in the given arrangement and hence they form a group. Which one of the following does not belong to the group? (1) PO – F (2) D – The 29 year old (3) B – The 38 year old (4) GM – E (5) AM – C 792. Which of the following statements is NOT TRUE as per the given information? A. The ED is 47 year old. B. C’s present age is an odd number. C. There are only two posts between the ones on which F and A work. (1) All (A), (B) and (C) (2) Only (A) (3) Only (B) (4) Both (A) and (C) (5) Both (B) and (C) Directions (793 – 795) : Study the following information carefully and answer the questions given below : (IBPS SO Prelim Exam, 28.12.2019)

    Six bags – A, B, C, D, E and F – are each of a different weight. Only two bags are heavier than Bag F. Bag E is heavier than Bag F but not the heaviest. Bag A is heavier than Bag B but lighter than Bag C. Bag D is lighter than Bag B. The second lightest bag weighs 3 kg. (Note: All the given weights are in whole numbers only) 793. If Bag E weighs 10 kg, what is the possible weight of Bag F? (1) 5 kg (2) 11 kg (3) 13 kg (4) 2 kg (5) 4 kg 794. As many bags are lighter than D as are heavier than (1) A (2) F (3) B (4) C (5) E 795. How many bags are lighter than Bag A? (1) three (2) two (3) four (4) five (5) one Directions (796–100) : Study the following information carefully and answer the questions given below : (Indian Bank Specialist Officer SO Exam, 08.03.2020)

    Seven people — A, B, C, P, Q, R and S — work in the same office but at a different position — Clerk (CL), Assistant Manager (AM), Manager (MG), Deputy General Manager (DGM), General Manager (GM), Executive Director (ED) and Chief Financial Officer (CFO). It is assumed that only the given positions exist in the said office and only one person works at each position. Each person plays a different sport — Hockey, Cricket, Basketball, Football, Kabaddi, Rugby and Golf. (Note : The positions have been given in increasing order of seniority with Clerk (CL) being the junior most position whilst the Chief Financial Officer (CFO) being the senior most position in the said office). The one who plays Golf is senior than the GM. There are only two positions between the ones on which C and the one who plays Golf work. There is only one position between the ones on which C and the one who plays Football work. The one who plays Football is junior than C. There are only three positions between the ones on which Q and the one who plays Football work. There are only three positions between the ones on which the one who plays Basketball and the one who plays Kabaddi work. The one who plays Baskteball is junior than the DGM. There are only three positions between the ones on which P and A work. A is junior than P. A does not play Basketball. There are only three positions between the ones on which the one who plays Hockey and R work. B is an immediate senior of the one who plays Cricket. 796. As many people are senior than R as are junior than ____________. (1) B (2) P (3) C (4) the one who plays Cricket (5) the one who plays Hockey 797. Who amongst the following is an immediate senior of the one who plays Basketball? (1) The one who plays Rugby (2) B (3) C (4) The one who plays Football (5) Q

    BPRE–329

    798. Based on the given arrangements, which of the following statements is/are true? A. the Manager plays Basketball B. R is Junior than the General Manager. C. S is an immediate junior of Q. (1) Only A (2) Only A and B (3) Only B (4) Only C (5) None of A, B and C 799. As per the given arrangement, which of the following combinations is/are true? A. C – The one who plays Kabaddi B. A – The one who plays Rugby C. P – The one who plays Golf (1) Only A and C (2) Only B and C (3) Only A and B (4) All A, B and C (5) None of A, B and C 800. Who amongst the following works above the Executive Director? (1) The one who plays Golf (2) B (3) Q (4) The one who plays Football (5) The one who plays Cricket Directions (801–803) : Study the following information carefully and answer the questions given below : (Indian Bank Specialist Officer SO Exam, 08.03.2020)

    Ten members of a family A, B, C, D, E, F, G, H, I and J differ in their ages. A list was prepared in which the ages of all the members were arranged in descending order. A is eldest while B is the youngest. J is older than five members. C is younger than A but older than J. G is older than seven members. D is only older than B. H is older than I, F and E but younger than C. F is older than E but younger than I. 801. How many members are older than E? (1) Six (2) Seven (3) Eight (4) Five (5) Nine 802. Whose age is more than that of H but less than that of C? (1) G (2) I (3) D (4) F (5) None of these

    RANKING/ARRANGEMENT 803. Who is the second youngest member in the list? (1) C (2) G (3) D (4) J (5) None of these Directions (804–808) : Study the following information carefully and answer the questions given below : (Indian Bank Specialist Officer SO Exam, 08.03.2020)

    Eight people — A, B, C, D, P, Q, R and S — are sitting around a circular table facing the centre. Each one of them is of a different age. Age of four of the given people is in odd number and age of other four is in even number. Only two people sit between A and the 23 year old (either from left or right). Only three people sit between the 23 year old and P. R is an immediate neighbour of P. Only one person sits between R and the 48 year old (either from left or right). A is not 48 years old. Only two people sit between the 48 years old and S (either from left or right). Only one person sits between S and the 33 year old (either from left or right). B sits third to the right of D. One of the immediate neighbours of B is 19 years old. The sum of present ages of R and S is 46 years. One of the immediate neighbours of the 27 year old is 36 years old. The 20 year old sits second to the right of the 36 year old. The sum of the present ages of A and Q is 84 years. One of the given persons is 52 years old. 804. The difference between the present ages (in years) of which amongst the following pairs is the highest? (1) P – R (2) B – Q (3) Q – C (4) S – A (5) D – C 805. Which of the following statements is true based on the given arrangement? (1) Only three people sit between R and S. (2) None of the given statements is true (3) A sits second to the left of the 23 years old. (4) Only two people sit between the one who is 27 years and the one who is 19 years old. (5) B and C are immediate neighbours of each other.

    806. Who amongst the following is 52 year old? (1) C (2) B (3) P (4) D (5) Q 807. Four of the following five are alike in a certain way as per the given arrangement and hence they form a group. Identify the one which does not belong to the group. (1) P (2) B (3) A (4) D (5) Q 808. What is the present age of C? (1) 27 years (2) 19 years (3) 33 years (4) 52 years (5) 23 years Directions (809–813) : Study the following information carefully and answer the questions given below : (Indian Bank Specialist Officer SO Exam, 08.03.2020)

    Ten people are sitting in two parallel rows containing five people each with equal distance between each other. In row I, A, B, C, D and E are seated and all of them are facing north. In row II, J, K, L, M and N are seated and all of them are facing south. Thus in the given arrangement each member seated in a row faces another member of the other row. Each person belongs to a different city. E sits second from an extreme end of the row. The one who faces J sits to the immediate left of E. The one who belongs to Imphal sits to the immediate left of J. As many people sit to the left of the one who belongs to Imphal as to the left of the one who belongs to Jaipur. B is an immediate neighbour of the one who belongs to Jaipur. B does not face J. The one who belongs to Aizawl sits second to the right of B. The one who belongs to Aizawl faces an immediate neighbour of K. The one who belongs to Delhi sits third to the right of K. More than one person sits between M and the one who belongs to Delhi. M faces an immediate neighbour of the one who belongs to Indore. N sits second to the right of the one who faces A. C sits at one of the positions to the right of the one who belongs to Lucknow but at one of the positions to the left of the one who belongs to Surat. The one who belongs

    BPRE–330

    to Chandigarh sits to the immediate right of the one who belongs to Mumbai. M belongs to the Pune. 809. As per the given arrangement, four of the following five are alike in a certain way based on their positions and hence they form a group. Which one of the following does not belong to that group? (1) B (2) D (3) N (4) The one who belongs to Indore (5) The one who belongs to Pune 810. How many people sit to the right of D? (1) Four (2) None (3) Two (4) Three (5) One 811. Who faces J? (1) The one who belongs to Aizawl (2) D (3) The one who belongs to Chandigarh (4) The one who belongs to Surat (5) B 812. Which of the following statements is not true with respect to the given information? (1) C belongs to Jaipur. (2) D faces the one who belongs to Pune. (3) J and K are immediate neighbours of each other. (4) A faces the one who belongs to Chandigarh. (5) All the given statements are true 813. As per the given arrangement, N belongs to __________. (1) Imphal (2) Chandigarh (3) Delhi (4) Mumbai (5) Pune Directions (814 – 818) : Study the following information carefully and answer the questions given below : (IBPS RRBs Officer CWE Prelim Exam, 13.09.2020)

    Ten boxes are placed in an almirah such that the box which is at lowest place is numbered as 1st, and the box above it is numbered as 2nd and

    RANKING/ARRANGEMENT so on till the topmost box is numbered as 10th position. Only four boxes are placed between J and K which is placed on an odd numbered position. The box K is placed below the box J. The box M is placed below the box K. Two boxes are placed between K and L which is placed just below box Q. Box B is placed below the box M. Box Q is placed on 7th position. The box I is placed on an even numbered position but just above the box P. The box A is placed immediate above the box R. 814. Which box is placed at the bottom of the almirah? (1) J (2) K (3) I (4) Q (5) B 815. Which of the following box is placed at the 9th position ? (1) J (2) P (3) Q (4) M (5) R 816. How many boxes are placed between box L and box B ? (1) Three (2) Two (3) Seven (4) Four (5) One 817. Four of the following are alike in a certain way, based on their positions in the given arrangement and so they form a group. Which of the following does not belong to that group? (1) L (2) B (3) K (4) A (5) Q 818. Which of the following statements is true as per the given information? (1) Box J is placed just above the box L. (2) Two boxes are placed between box P and box Q. (3) Box R is placed at an even numbered position. (4) Box K is placed at fourth number position. (5) None of the given options is true Directions (819–823) : Study the following information carefully and answer the questions given below : (IBPS RRBs Officer CWE Prelim Exam, 13.09.2020)

    Seven friends are sitting around a square table in such a way that three of them sit at four corners of the

    square table while four sit in the middle of each of the four sides. The ones who sit at the corner facets inside while those who sit in the middle face outside. One of the corner seats is vacant. K sits second to the left of O and third to the left of J. Two persons sit between N and P. M sits second to the left of N and adjacent to the vacant seat. One person sits between P and L. 819. How many persons sit between J and O when counted from the right of J? (1) More than three (2) One (3) None (4) Two (5) Three 820. Who sits third to the right of L? (1) J (2) O (3) K (4) N (5) M 821. Four of the following five are alike in a certain way as per the given information and so they form a group. Which of the following does not belong to that group? (1) JL (2) LP (3) PM (4) OK (5) MO 822. Who among the following sits adjacent to O? (1) J (2) P (3) N (4) K (5) L 823. Which of the following statements is true as per the given information? (1) J sits to the immediate left of M (2) Three persons sit between J and P when counted from the left of J. (3) N sits second to the left of K. (4) Both (1) and (2) (5) Both (2) and (3) Directions (824 – 828) : Study the following information carefully and answer the questions given below : (IBPS RRBs Officer CWE Prelim Exam, 13.09.2020)

    Some persons are sitting in a straight line. Some are facing south and some are facing north. C is sitting second to the right of B and both are facing opposite directions. F is sitting fifth to the right of A

    BPRE–331

    and is facing north. G is sitting at ninth position from the left and is facing north. D is sitting third from the left of H and both are facing opposite directions. E is sitting to the immediate right of F and both are facing opposite directions. B is sitting tenth to the right of D and both are facing opposite directions. A is at the extreme left and H is at the extreme right and both of them are facing north. No seat is available between A and B. 824. Who is sitting at the middle of the line? (1) F (2) E (3) G (4) D (5) No one 825. Who is sitting third to the left of C? (1) H (2) A (3) B (4) E (5) F 826. How many seats are there in the row? (1) 15 (2) 9 (3) 12 (4) 16 (5) 7 827. If the positions of C and F are interchanged, then who is immediate neighbour of F? (1) E (2) B (3) A (4) G (5) No one 828. Who is sitting sixth to the left of H? (1) E (2) F (3) G (4) C (5) B Directions (829–833) : Study the following information carefully and answer the questions given below : (IBPS Bank PO/MT CWE Prelim Exam, 03.10.2020)

    Eight persons – A, K, M, V, C, S, G and R – are sitting in a straight line at equal distance but not necessarily in the same order. Three of them are facing South and the remaining persons are facing North. Each of them works in different banks viz., SBI, UCO, PNB, BOB, UBI, RBI, BOI and BOM, but not necessarily in the same order. ● The one who works in PNB sits to the immediate right of the one who works in UCO bank.

    RANKING/ARRANGEMENT ●M

    does not face north direction. G is an immediate neighbour of both M and C. ● V sits second to the right of G. M sits to the immediate left of K. ● The one who works in SBI sits at any of the extreme ends. The one who works in BOB sits exactly between C and A. ● S does not work in PNB. Only four persons sit between R and C who works in RBI. ● S sits to the right of M but not to the immediate right. M works in BOI. G and C face opposite directions (i.e., if one person is facing north direction, the other person is facing south direction and vice-versa). ● The one who works in BOM is sitting to the immediate right of V. ● K sits third from any of the extreme ends of the line. ● R sits third to the right of M. 829. Four of the following five are alike in a certain manner and hence they form a group. Which one of the following does not belong to the group ? (1) K (2) R (3) G (4) S (5) A 830. Who among the following sits second to the left of the one who works in BOB ? (1) A (2) G (3) M (4) R (5) K 831. Who among the following sits exactly between M and S ? (1) A (2) R (3) G (4) K (5) C 832. Which of the following statements is/are correct ? (1) The one who works in UCO Bank sits exactly between K and R. (2) The immediate neighbours of C face the same direction. (3) S is sitting third to the left of G. (4) R and A are sitting at the exteme ends of the line. (5) All the statements are true

    833. What is the position of V with respect to the one who works in BOI ? (1) Second to the right (2) Third to the right (3) Third to the left (4) Second to the left (5) Immediate right Directions (834–838) : Study the following information carefully and answer the questions given below : (IBPS Bank PO/MT CWE Prelim Exam, 03.10.2020)

    Eight persons – A, B, C, D, E, F, G and H – are sitting around a circular table, but not necessarily in the same order. Four of them are facing the centre and the other four are facing away from the centre. Not more than two persons facing the same direction are sitting together. B sits third to the left of A and they are facing opposite directions (i.e., if one person is facing the centre, the other person is facing outside and viceversa). H sits to the immediate right of D. F sits second to the left of G. The immediate neighbours of H are facing the same direction (i.e., if one neighbour is facing the centre, the other neighbour is also facing the centre and vice-versa) but opposite direction with respect to H. H faces the centre. B is an immediate neighbour of E and both are facing the same direction. E sits second to the left of C who is not an immediate neighbour of B. H sits third to the right of C who does not face outside. 834. Who among the following sits opposite to the one who is to the immediate left of H ? (1) G (2) C (3) E (4) B (5) D 835. Who among the following sits exactly between E and D ? (1) C (2) H (3) F (4) G (5) B 836. Who among the following sits third to the right of F ? (1) D (2) B (3) G (4) E (5) H 837. Which of the following statements is not true ?

    BPRE–332

    (1) Both the immediate neighbours of G face outside. (2) E faces the centre while A faces outside. (3) The immediate neighbours of B face opposite directions. (4) G is sitting third to the left of D. (5) B, C, E and H face the centre. 838. Who among the following sits second to the right of C ? (1) F (2) A (3) G (4) H (5) E Directions (839–843) : Study the following information carefully and answer the questions given below : (IBPS RRBs Officer CWE Prelim Exam, 31.12.2020)

    Eight persons – A, B, C, D, E, F, G and H – are sitting in a straight line at equal distance from each other, but not necessarily in the same order. Four of them are facing north while the other four face south. G is not an immediate neighbour of A. The persons sitting at the extreme ends face the same direction (i.e., if one person is facing north, the otehr person also faces north and vice-versa). Both the immediate neighbours of A face opposite direction to that of F (i.e., if F is facing north, both the immediate neighbours of A face south and vice-versa). Both the immediate neighbours of H face opposite directions. F sits third to the right of A. F sits at one of the extreme ends of the line. E sits third to the left of A. D is one of the immediate neighbours of E, and he faces north. B sits exactly between C and F. E sits to the immediate right of D. Both the immediate neighbours of C face north. 839. Who among the following sits third to the right of C ? (1) F (2) B (3) H (4) G (5) E 840. Who among the following sits exactly between A and B ? (1) F (2) D (3) C (4) H (5) E 841. Who among the following is sitting second to the left of B ? (1) C (2) F (3) H (4) E (5) A

    RANKING/ARRANGEMENT 842. Four of the following five are alike in a certain way and hence they form a group. Which one of the following does not belong to that group ? (1) C (2) B (3) G (4) H (5) E 843. Who among the following sits 4th to the left of F ? (1) H (2) A (3) B (4) G (5) D Directions (844–848) : Study the following information carefully and answer the questions given below : (IBPS RRBs Officer CWE Prelim Exam, 31.12.2020)

    Eight persons – P, A, K, H, J, N, R and S – from eight different ministries, namely Agriculture, Finance, Defence, Railways, Health, Education, Sports and Urban Development, are sitting around a circular table facing the centre, but not necssarily in the same order. N is sitting third to the left of Sports Secretary. Neither P nor K is sitting adjacent to N or Sports Secretary. N is not Agriculture Secretary. Sports Secretary and Railways Secretary are immediate neighbours of each other. H is sitting second to the right of Railways Secretary. P and K are immediate neighbours of each other. Defence Secretary nor Sports Secretary. Only one person is sitting between P and Finance Secretary. S is sitting third to the left of Urban Development Secretary. R and Agriculture Secretary are immediate neighbours of each other. Education Secretary is sitting second to the left of Health Secretary. 844. Who among the following sits to the immediate left of Agriculture Secretary ? (1) Urban Development (2) R (3) Railways Secretary (4) A (5) Defence Secretary 845. Who among he following sits exactly between J and Finance Secretary ? (1) K (2) H (3) N (4) R (5) S

    846. The Secretary of which ministry sits third to the right of K ? (1) Railways (2) Urban Development (3) Education (4) Sports (5) Defence 847. Who among the following sits opposite to Education Secretary ? (1) Defence Secretary (2) Sports Secretary (3) Urban Developemtn Secretary (4) Railways Secretary (5) Health Secretary 848. Four of the following five are alike in a certain way and hence they form a group. Which one of the following does not belong to that group ? (1) K and S (2) P and Sports Secretary (3) R and Finance Secretary (4) J and Urban Development Secretary (5) Health Secretary and Agriculture Secretary Directions (849–853) : Study the following information carefully and answer the questions given below : (IBPS RRBs Officer CWE Prelim Exam, 07.08.2021)

    Twelve employees including four team leaders – U, V, W, X, Y, Z, O, P, Q, R, S and T– are sitting around a rectangular table facing the centre. The four team leaders are sitting at the four corners of the table. The sitting arrangement and salary of the employees are given in the following table : Rs. 40,000 Rs. 30,000 Rs. 35,000 Rs. 40,000

    Rs. 29,000

    Rs. 32,000

    Rs.36,000

    Rs. 33,000

    Rs. 40,000 Rs. 38,000 Rs.28,000 Rs. 40,000

    W is sitting second to the left of the one who is sitting third to the left of O. R is sitting third to the left of the one who is sitting diagonally opposite to Y. P is sitting third to the left of the one who is sitting diagonally opposite to X. The immediate neighbour of both

    BPRE–333

    S and X is sitting at one of the corners. Q, a team leader, is sitting at the top left corner. X is sitting second to the right of the one who is sitting to the immediate left of R. U is sitting to the immediate left of the one who is sitting third to the left of Z. Q is sitting opposite to the one who is sitting third to the right of V. T is sitting to the immediate right of the one who is sitting fourth to the left of O. V is sitting to the immediate right of the one who is sitting third to the right of T. 849. Which of the following pairs of employees has the salary difference of Rs. 3,000 ? (1) Y and P (2) T and Z (3) U and V (4) T and R (5) X and S 850. Which of the following employees is sitting fourth to the right of T ? (1) The employee who receives Rs. 33,000 (2) The employee who receives Rs. 38,000 (3) The employee who receives Rs. 30,000 (4) The employee who receives Rs. 28,000 (5) The employee who receives Rs. 40,000 851. Which of the following statements is correct regarding the employee who receives Rs. 32,000 as salary ? (1) He is sitting exactly between P and X. (2) He is sitting opposite to R (3) His salary together with the salary of V is Rs. 73,000 (4) He is sitting third to the right of X (5) He is sitting diagonally opposite to W 852. What is the salary of the employee who is sitting exactly between Z and P ? (1) Rs. 33,000 (2) Rs. 40,000 (3) Rs. 36,000 (4) Rs. 29,000 (5) Rs. 30,000 853. What is the sum of salaries of both R and U ? (1) Rs. 66,000 (2) Rs. 80,000 (3) Rs. 65,000 (4) Rs. 78,000 (5) Rs. 67,000

    RANKING/ARRANGEMENT Directions (854–858) : Study the following information carefully and answer the questions given below : (IBPS Bank PO/MT CWE Prelim Exam, 04.12.2021)

    A certain number of chairs are arranged in a row facing north. Some girls are sitting on some of the chairs. ● B sits to the immediate left of the one who sits third to the right of a vacant chair. ● A sits third to the right of the one who sits sixth to the left of E. ● There are nine chairs between R and H. ● B sits fifth to the right of a vacant chair which is sixth to the left of a vacant chair. ● A sits ninth to the left of N and only five chairs are occupied between them. ● The number of chairs to the right side of H is four more than the number of chairs to the left side of H. ● S has only one immediate neighbour to her left and M has only one immediate neighbour to her right. ● There is only one vacant chair between T and N. ● M sits fourth to the right of a vacant chair which is to the immediate left of G. ● J sits second to the right of the one who sits sixth to the left of N. ● There are only three chairs between A and K. ● Either R or G sits at the extreme right end. ● A sits fourth to the right of the one who sits tenth to the left of B. ● There is one vacant chair between E and J. ● Either A or K sits at the extreme left end. 854. How many girls are sitting to the right of A but to the left M? (1) Three (2) Four (3) Five (4) Six (5) Seven 855. Who among the following sits to the right of N but to the left of R? (1) G (2) M (3) B (4) E (5) S

    856. How many girls are sitting to the right of E ? (1) Four (2) Seven (3) Five (4) Three (5) Six 857. Four of the following five are alike in a certain way and hence they form a group. Which one of the following does not belong to that group? (1) AH (2) EJ (3) MR (4) BG (5) ST 858. How many vacant chairs are there between T and G ? (1) Three (2) Two (3) Four (4) One (5) Five Directions (859–863) : Study the following information carefully and answer the questions given below : (IBPS Bank PO/MT CWE Prelim Exam, 04.12.2021)

    Eight students – A, B, C, D, E, F, G and H – are sitting in a row facing north in the following manner : L E F T

    B

    F

    D

    A

    H

    G

    E

    C

    R I G H T

    These students are to be rearranged around a circlular table as per the following instructions : All the students should face the centre of the circular table. The student who is to the immediate right of D should sit to the immediate left of the one who will sit to the immediate left of C. The student at the right end should sit to the immediate right of the one who will sit second to the left of G. The student who is third to the left of G should sit to the immediate left of the one who will sit second to the right of F. The student who is fourth to the left of H should sit to the immediate right of the one who will sit second to the left of F. The student who is exactly between H and E should sit opposite to the one who will sit exactly between D and A. The student who is to the immediate right of B should sit third to the right of the one who will sit to the immediate right of E. 859. Which of the following statements is correct with respect to H and G? (1) H is second to the left of G

    BPRE–334

    (2) H is third to the right of G (3) G is third to the right of H (4) G is to the immediate left of H (5) H is sitting just opposite to G. 860. Who among the following is sitting exactly between H and F ? (1) G (2) E (3) C (4) D (5) B 861. Four of the following five are alike in a certain way and hence they form a group. Which one of the following does not belong to that group? (1) GD (2) EH (3) CA (4) DB (5) GE 862. In which of the following pairs, the two students are sitting opposite to each other ? (1) A and C (2) D and C (3) F and G (4) B and D (5) B and H 863. Who among the following is sitting third to the right of E ? (1) H (2) D (3) B (4) G (5) F

    SBI PO EXAMS 1. Four of the following five are alike in a certain way based on their positions in the above arrangement and so form a group. Which is the one that does not belong to that group ? (1) J 3 D (2) I 6 P (3) A 5 R (4) Y H $ (5) T © V Directions (2–6) : Study the following information to answer the given questions : (SBI Associate Banks PO Exam. 07.08.2011)

    Twelve people are sitting in two parallel rows containing six people each, in such a way that there is an equal distance between adjacent persons. In row-1 P, Q, R, S, T and V are seated and all of them are facing South. In row-2 A, B, C, D, E and F are seated and all of them are facing North. Therefore, in the given seating arrangement each member seated in a row faces another member of the other row.

    RANKING/ARRANGEMENT S sits third to right of Q. Either S or Q sits at an extreme end of the line. The one who faces Q sits second to right of E. Two people sit between B and F. Neither B nor F sits at an extreme end of the line. The immediate neighbour of B faces the person who sits third to left of P. R and T are immediate neighbours of each other. C sits second to the left of A. T does not face the immediate neighbour of D. 2. Who amongst the following sit at extreme ends of the rows ? (1) S, D (2) Q, A (3) V, C (4) P, D (5) Q, F 3. Who amongst the following faces S? (1) A (2) B (3) C (4) D (5) F 4. How many persons are seated between V and R ? (1) One (2) Two (3) Three (4) Four (5) None 5. P is related to A in the same way as S is related to B based on the given arrangement. To which of the following is T related to, following the same pattern ? (1) C (2) D (3) E (4) F (5) Cannot be determined 6. Which of the following is true regarding T ? (1) F faces T (2) V is an immediate neighbour of T (3) F faces the one who is second to right of T (4) T sits at one of the extreme ends of the line (5) Q sits second to the right of T 7. Four of the following five are alike in a certain way based on the given arrangement and so form a group. Which is the one that does not belong to that group? (1) A–T (2) B–T (3) F–P (4) C–V (5) E – Q Directions (8–14) : Study the following information carefully and answer the given questions. (SBI Associate Banks PO Exam. 07.08.2011)

    Eight colleagues, A, B, C, D, E, F, G and H are sitting around a circular table facing the centre but not nec-

    essarily in the same order. Each one of them holds a different post viz. Manager, Company Secretary, Chairman, President, Vice - President, Group Leader, Financial Advisor and Managing Director. A sits third to right of the Managing Director. Only two people sit between the Managing Director and H. Vice President and the Company Secretary are immediate neighbours of each other. Neither A nor H is a Vice President or a Company Secretary. Vice President is not an immediate neighbour of the Managing Director. Manager sits second to left of E. E is not an immediate neighbour of H. The Manager is an immediate neighbour of both Group Leader and the Financial Advisor. Financial Advisor sits third to right of B. B is not the Vice President. C sits to the immediate right of the Chairman. A is not the Chairman. F is not an immediate neighbour of A. G is not an immediate neighbour of the Manager. 8. Who amongst the following sits third to the left of E ? (1) The Manager (2) G (3) A (4) The Financial Advisor (5) B 9. Four of the following five are alike in a certain way based on the given arrangement and thus form a group. Which is the one that does not belong to that group ? (1) F — Chairman (2) G — President (3) D — Manager (4) A — Financial Advisor (5) B — Managing Director 10. Who amongst the following is the President of the company ? (1) A (2) C (3) H (4) G (5) D 11. Which of the following is true with respect to the given seating arrangement ? (1) The Group Leader of the company is an immediate neighbour of the Vice-president (2) G sits second to right of D (3) The Group Leader and the Company Secretary are immediate neighbours of each other

    BPRE–335

    (4) The Chairman of the company sits to the immediate left of the Manag-ing Director (5) The Group Leader sits second to the left of D 12. Which of the following posts does B hold in the company ? (1) Chairman (2) Manager (3) Company Secretary (4) Vice - President (5) Financial Advisor 13. Who amongst the following sit exactly between the Managing Director and H ? (1) H and the Chairman (2) B and G (3) The Chairman and C (4) F and C (5) E and the Group Leader 14. Who amongst the following is the Group Leader ? (1) C (2) F (3) G (4) H (5) A Directions (15–19) : Study the following information and answer the questions given below : (SBI Probationary Officer Exam 28.04.2013)

    Eight people - E, F, G, H, J, K, L and M are sitting around a circular table facing the centre. Each of them is of a different profession - Chartered Accountant, Columnist, Doctor, Engineer, Financial Analyst, Lawyer, Professor and Scientist but not necessarily in the same order. F is sitting second to the left of K. The Scientist is an immediate neighbour of K. There are only three people between the Scientist and E. Only one person sits between the Engineer and E. The Columnist is to the immediate right of the Engineer. M is second to the right of K. H is the Scientist. G and J are immediate neighbours of each other. Neither G nor J is an Engineer. The Financial Analyst is to the immediate left of F. The Lawyer is second to the right of the Columnist. The Professor is an immediate neighbour of the Engineer. G is second to the right of the Chartered Accountant. 15. Who is sitting second to the right of E ? (1) The Lawyer (2) G (3) The Engineer (4) F (5) K

    RANKING/ARRANGEMENT 16. Who amongst the following is the Professor ? (1) F (2) L (3) M (4) K (5) J 17. Four of the following five are alike in a certain way based on the given arrangement and hence form a group. Which of the following does not belong to that group ? (1) Chartered Accountant - H (2) M - Doctor (3) J - Engineer (4) Financial Analyst - L (5) Lawyer - K 18. What is the position of L with respect to the Scientist ? (1) Third to the left (2) Second to the right (3) Second to the left (4) Third to the right (5) Immediate right 19. Which of the following statements is true according to the given arrangement ? (1) The Lawyer is second to the left of the Doctor (2) E is an immediate neighbour of the Financial Analyst (3) H sits exactly between F and the Financial Analyst (4) Only four people sit between the Columnist and F (5) All of the given statements are true Directions (20–24) : Study the following information carefully to answer the given questions : (SBI Management Executive Exam, 23.02.2014)

    Eight persons – A, B, C, D, E, F, G and H – are sitting around a circular table facing outward, but not necessarily in the same order. They are at equidistant. D is third to the right of H. C is an immediate neighbour of both A and H. C is sitting second to the left of B. H is second to the right of A. He is second to the left of E. F is not an immediate neighbour of A. 20. Who is to the immediate left of D? (1) F (2) E (3) G (4) Cannot be determined (5) None of these 21. How many persons are seated between H and F if we go anticlockwise from H to F ? (1) One (2) Two (3) Three (4) Cannot be determined (5) None of these

    22. Which of the following is A’s position with respect to D ? (1) Third to the right (2) Third to the left (3) Second to the right (4) Fourth to the left (5) None of these 23. Who amongst the following is sitting just opposite to G? (1) E (2) H (3) C (4) B (5) Either B or E 24. Which of the following statements is not true regarding the seating arrangement ? (1) There are only three persons between G and B (2) D is sitting exactly between E and F (3) G is second to the left of D (4) B is third to the right of A (5) All are true Directions (25-31) : Study the following information carefully and answer the questions given below : (SBI Probationary Officer Online Exam, 21.06.2014)

    Eight friends – R, S, T, U, V, W, X and Y – are sitting around a circular table facing the centre, but not necessarily in the same order. Each of the them studies in the different Standards viz, Standard I to Standard VIII, but not necessarily in the same order. T is second to the right of the person who studies in Standard VII. Only one person sits between T and the person who studies in Standard V. X is sitting third to the left of the person who studies in Standard VIII. The person studying in Standard VIII is not an immediate neighbour of the person studying in Standard VII. T does not study in Standard VIII. The person studying in Standard VI is to the left of U. U does not study in Standard V or Standard VIII. The persons studying in Standard VI and VII are immediate neighbours of each other. One of the immediate neighbours of the person studying in Standard VIII, studies in Standard II. S and Y are immediate neighbours of each other. There is one person between S and R. V does not study in Standard I. R studies in Standard V. Y does not study in Standard II. W is sitting between the persons who study in Standard VII and Standard IV. X is second to the right of the person studying in Standard I.

    BPRE–336

    25. Who among the following is sitting third to the right of ‘T’. (1) S (2) The person studying in Standard II (3) The person studying in Standard V (4) Y (5) The person studying in Standard I 26. Who among the following is sitting just opposite to ‘R’ ? (1) The person studying in Standard VI (2) W (3) The person studying in Standard V (4) S (5) The person studying in Standard VII 27. Which of the following pairs represents the immediate neighbours of ‘V’? (1) R and the person studying in Standard VIII (2) S and the person studying in Standard III (3) The person studying in Standard V and Y (4) Cannot be determined (5) None of these 28. What is the position of ‘W’ with respect to ‘S’ ? (1) Fourth to the right (2) Fourth to the left (3) Third to the right (4) Fifth to the right (5) Sixth to the left 29. Starting from R, if all the persons are made to sit in the alphabetical order in clockwise direction, the positions of how many (excluding R) will remain unchanged? (1) Two (2) Three (3) Four (4) One (5) None 30. Who among the following is sitting fourth to the left of the person who studies in Standard II? (1) U (2) W (3) X (4) S (5) Cannot be determined 31. Who among the following is sitting exactly between the person who studies in Standard III and W? (1) T (2) U (3) R (4) V (5) Y

    RANKING/ARRANGEMENT Directions (32-38) % Study the following information carefully and answer the questions given below : (SBI Specialist Officer (Law Officer : MMGS Scale-II) Online Exam, 19.04.2014)

    Seven persons — I, J, K, L, M, N and O — are sitting in a circle at equidistance but not necessarily in the same order. Some of them are facing towards the centre and some are facing outside the centre. O is facing outside. J sits second to the right of O. N is sitting second to the left of P. P is not an immediate neighbour of J or O. N is not an immediate neighbour of J. The immediate neighbour of K faces towards the centre. K and P face the same direction. I is sitting second to the right of N. The immediate neighbour of M faces outside. I and M face the same direction as that of J. 32. Who amongst the following are not facing the centre ? (1) K, N, O and J (2) I, M, J and P (3) J, M, I, O and N (4) P, K, O and N (5) None of these 33. Which of the following is the J’s position with respect to N ? (1) Third to the right (2) Fourth to the right (3) Third to the left (4) Second to the left (5) Fifth to the left 34. Which of the following is the K’s position with respect to M ? (1) Third to the left (2) Third to the right (3) Fourth to the left (4) Second to the right (5) None of these 35. Who among the following is sitting exactly between M and K? (1) N (2) P (3) O (4) I (5) J 36. Which of the following is the P’s position with respect to J ? (1) Third to the right (2) Fourth to the left (3) Fourth to the right (4) Third to the left (5) Fifth to the left 37. How many persons are sitting between M and O if we move clockwise starting from M ? (1) None (2) Two (3) Three (4) One (5) Four

    Which of the following statements is true regarding the seating arrangement ? (1) There are only two persons seated between K and I (2) M, K and I are seated adjacent (3) N, P and J are seated adjacent (4) Three persons are facing the centre (5) One of the neighbours of I is K Directions (39-44) : Study the following information carefully and anaswer the questions given below :

    38.

    (SBI Specialist Officer (Law Officer : MMGS Scale-II) Online Exam, 19.04.2014)

    Eight persons — P, Q, R, S, T, U, V and W – are standing in a straight line at equidistance. Some of them are facing North while some others are facing South. R is third to the left of W. W is facing North. T is third to the right of W. S is second to the left of Q. Q is not an immediate neighbour of W or R. Both the immediate neighbours of P face South. The immediate neighbour of V faces opposite direction. Immediate neighbours of U face opposite direction (If U is facing North, immediate neighbour of U face South and vice-versa). V faces just opposite of Q. U is not an immediate neighbour of P and faces opposite direction with respect to that of P (If U faces South P faces North and vice-vera). T faces the same direction as that of V. 39. Who among the following are facing North ? (1) P, V, W and Q (2) R, W, U and Q (3) U, S, T and Q (4) P, W, S and Q (5) None of these 40. Who among the following is to immediate left of S ? (1) U (2) T (3) V (4) W (5) None of these 41. How many persons are there between P and U ? (1) Three (2) Four (3) Two (4) One (5) There is no person 42. Who among the following are standing on the extreme ends of the line ? (1) P and Q (2) R and Q (3) U and R (4) R and S (5) None of these

    BPRE–337

    43. Who among the following is third to the right of U ? (1) P (2) V (3) R (4) Q (5) None of these 44. Who among the following are the immediate neighbours of V? (1) R and W (2) P and U (3) R and S (4) P and W (5) None of these Directions (45-51) : Study the following information carefully and answer the questions given below : (SBI Probationary Officer Online Exam, 21.06.2014)

    Eight persons – H, I, J, K, L, M, N and O – are standing in a straight line at equidistant. Some of them are facing north while others are facing south. M is standing third to the right to H. M is standing at one of the extreme ends. L is standing third to the left of H. The immediate neighbours of J face north. N is not an immediate neighbour of H. The persons standing at the extreme ends face the same direction (Both are facing either north or south). The immediate neighbours of H face just opposite direction as that of M. The immediate neighbours of O face opposite directions with respect to each other. One of the immediate neighbours of L is K who is facing north. I is standing between J and M. Not more than four persons are facing north. 45. Who among the following is third to the left of N? (1) K (2) J (3) H (4) I (5) O 46. The immediate neighbours of L are : (1) M and N (2) N and O (3) K and N (4) N and H (5) J and H 47. How many persons are standing exactly between I and O? (1) Three (2) Four (3) One (4) Two (5) None 48. Who among the following is to the immediate left of H? (1) O (2) J (3) I (4) L (5) K

    RANKING/ARRANGEMENT 49. Four of the following five are alike in a certain way based on the above arrangement and hence form a group. Which one of the following does not belong to that group? (1) N (2) L (3) O (4) I (5) K 50. Who among the following is exactly between L and J? (1) N (2) O (3) H (4) I (5) None 51. Who among the following is fourth to the right of J? (1) N (2) I (3) H (4) L (5) K Directions (52–58) : Study the following information carefully and answer the questions given below ; (SBI Probationary Officer Online Exam, 28.06.2014)

    Eight persons — L, M, N, O, P, Q, R and S – are standing in a straight line, but not necessarily in the same order. Some of them are facing north while some others are facing east. N is standing third to the right of S. N is standing at one of the extreme ends. O is standing third to the left of S. The immediate neighbours of Q face north. M is not an immediate neighbour of S. The persons standing at the extreme ends face the same direction (Both are facing either north or east). Both the immediate neighbours of S face east unlike N. Oneof the immediate neighbours of L faces east while the other faces north. One of the immediate neighbours of O is P who is facing north. R is standing between Q and N. Not more than four persons are facing north. 52. Who among the following is infront of M ? (1) Q (2) N (3) O (4) L (5) S 53. The immediate neighbours of O are : (1) M and P (2) L and M (3) L and P (4) L and S (5) Q and R 54. How many persons are standing exactly between R and L ? (1) One (2) Three (3) Four (4) Two (5) None

    55. Who among the following is to the immediate left of S ? (1) Q (2) L (3) R (4) O (5) M 56. Four of the following five are alike in a certain way based on the above arrangement and hence form a group. Which is the one that does not belong to the group? (1) S (2) R (3) M (4) N (5) P 57. Who among the following is exactly between O and Q ? (1) None (2) N (3) S (4) M (5) L 58. Who among the following is fifth to the left of R ? (1) O (2) M (3) L (4) S (5) Q Directions (59–64) : Study the following information carefully and answer the questions given below : (SBI Management Executive Exam. 19.09.2014)

    Eight persons — P, Q, R, S, T, U, V and W – are sitting around a circular table such that some are facing towards the centre while some others are facing opposite to the centre, but not necessarily in the same order. P, Q, T and U are facing towards the centre while others are facing opposite to the centre. Q is not an immediate neighbours of R. S and U are immediate neighbours of P. There is only one person between Q and U. There are two persons between T and Q. S is sitting to the immediate left of T. U is to the immediate left of W. R is sitting to the immediate right of T. 59. Who among the following is sitting to the immediate left of V\ (1) R (2) Q (3) T (4) W (5) S 60. How many persons are sitting between T and W a\ (1) None (2) One (3) Two (4) Three (5) More than three 61. Who among the following is sitting exactly between W and V? (1) Q (2) R (3) S (4) U (5) P

    BPRE–338

    62. If all the eight persons are made to sit in alphabetical order in clockwise direction starting from P, the position of how many will remain unchanged ( excluding P)\ (1) One (2) Two (3) Three (4) Four (5) None 63. Who among the following is sitting third to the right of R\ (1) Q (2) S (3) P (4) T (5) W 64. Who among the following is sitting just oppsite to S ? (1) W (2) Q (3) V (4) R (5) T Directions (65–70) : Study the following information carefully and answer the questions given below : (SBI Management Executive Exam, 19.09.2014)

    Eight persons — P, Q, R, S, T, U, V and W – are sitting around a circular table such that some are facing towards the centre while some others are facing opposite to the centre, but not necessarily in the same order. P, Q, T and U are facing towards the centre while others are facing opposite to the centre. Q is not an immediate neighbours of R. S and U are immediate neighbours of P. There is only one person between Q and U. There are two persons between T and Q. S is sitting to the immediate left of T. U is to the immediate left of W. R is sitting to the immediate right of T. 65. Who among the following is sitting to the immediate left of V\ (1) R (2) Q (3) T (4) W (5) S 66. How many persons are sitting between T and W a\ (1) None (2) One (3) Two (4) Three (5) More than three 67. Who among the following is sitting exactly between W and V? (1) Q (2) R (3) S (4) U (5) P 68. If all the eight persons are made to sit in alphabetical order in clockwise direction starting from P, the position of how many will remain unchanged ( excluding P)\

    RANKING/ARRANGEMENT (1) One (2) Two (3) Three (4) Four (5) None 69. Who among the following is sitting third to the right of R\ (1) Q (2) S (3) P (4) T (5) W 70. Who among the following is sitting just oppsite to S ? (1) W (2) Q (3) V (4) R (5) T Directions (71–76) : Study the following information carefully and answer the questions given below : (SBI Associates PO Online Exam, 29.11.2014)

    Eight persons — A, B, C, D, E, F, G and H – are sitting around a circular area at equal distances between each other, but not necessarily in the same order. Some of the people are facing the centre while some others face outside (i.e. in a direction opposite to the centre). Note : Facing the same direction means if one faces the centre then other also faces the centre and vice versa. Facing opposite direction means if one person faces the centre then the other faces outside and vice versa. D sits second to the left of B. B faces outside. A and F are immediate neighbours of D. H sits second to the right of A. C sits third to the left of H. E sits to the immediate right of G. D sits third to the left of E. D faces the same direction as H. F sits to the immediate right of C. F and G take same directions. 71. Which of the following is true regarding F as per the given seating arrangement ? (1) Only one person sits between F and B (2) Only three persons sits between F and H (3) A sits second to the left of F. (4) C sits to the immediate right of F. (5) F faces the centre. 72. How many people in the given arrangement face outside ? (1) Four (2) Two (3) One (4) Three (5) Five

    73. What is C’s position with respect to A ? (1) Fourth to the right (2) Third to the left (3) Second to the left (4) Fifth to the left (5) Third to the right 74. Who sits exactly between G and C when counted from the right of G ? (1) E (2) F (3) H (4) D (5) B 75. Who sits to the immediate left of B? (1) D (2) H (3) Other than those given as options (4) A (5) G 76. Four of the following five are alike in a certain way based on the given seating arrangement and so form a group. Which is the one that does not belong to that group ? (1) F (2) C (3) H (4) G (5) A Directions (77–81) : Study the following information carefully and answer the questions given below : (SBI Associates PO Online Exam. 29.11.2014)

    Ten people are sitting in two parallel rows, containing five people each in such a way that there is equal distance between adjacent persons. In row— 1 : J, K, L, M and N are seated and all of them are facing north and in row—2 : P, Q, R, S and T are seated and all of them are facing south (but not necessarily in the same order). Each person also likes a different cartoon character namely, Batman, Ben 10, Dexter, Donald Duck, Tom, Jerry, Noddy, Oswald, Superman and Spiderman (not necessarily in the same order). N sits exactly in the centre of the row and faces the one who likes Tom. Only one person sits between S and the one who likes Tom. S faces one of the immediate neighbours of the one who likes Jerry. Only two people sit between K and the one who likes Jerry. K faces the one who likes Oswald. P faces one of the immediate neighbours of L. P does not like Oswald. M is an immediate neighbour of L. The one who likes Donald Duck is not an immediate neighbour of S. T

    BPRE–339

    is not immediate neighbour of P and faces the one who likes Ben 10. T is not at the extreme end of the line. R faces the one who likes Superman. R does not like Tom. The one who likes Spiderman faces north. S does not face the one who likes Spiderman. The one who likes Batman sits to the immediate right of the person who likes Noddy. S does not like Batman. 77. Which of the following pairs represent the people sitting at the extreme end of the two rows ? (1) S, M (2) Q, L (3) P, N (4) Q, J (5) R, L 78. Who amongst the following likes Donald Duck ? (1) S (2) P (3) R (4) T (5) Q 79. Who amongst the following is an immediate neighbour of the one who likes Spiderman ? (1) The one who likes Jerry (2) The one who likes Noddy (3) K (4) The one who likes Superman (5) T 80. Which of the following cartoon character does Q like ? (1) Batman (2) Tom (3) Jerry (4) Noddy (5) Oswald 81. Which of the following statements is definitely true as per the given arrangement ? (1) L faces R (2) Q likes Oswald (3) Q is an immediate neighbour of P (4) The one who like Superman is an immediate neighbour of P (5) The one who likes Dexter faces N. Directions (82–87) : Study the following information carefully and answer the questions given below : (SBI Associates PO Online Exam, 30.11.2014)

    Eight people & M, N, O, P, Q, R, S and T – are sitting around a circular are at equal distances between each other, but not necessarily in the same order. Some of the people are facing the centre while some face outside. (i.e., in a direction opposite to the centre).

    RANKING/ARRANGEMENT Note % Facing the same direction means one faces the centre then the other also faces the centre and vice versa. Facing opposite direction means if one person faces centre then the other faces outside and vice versa. S sits second to the right of P. P faces the centre. T sits second to the left of S. T is an immediate neighbour of both O and Q. R sits second to the left of Q. M sits second to the left of R. T sits second to the left of N. M sits second to the left of O. T faces the same direction as Q. N sits third to the right of M. 82. Who sits exactly between S and T, when counted from the left of S? (1) M (2) Q (3) P (4) O (5) N 83. How many people in the given arrangement face the centre ? (1) One (2) Three (3) Five (4) Two (5) Four 84. Who sits second to the right of N? (1) P (2) R (3) T (4) S (5) O 85. Four of the following five are alike in a certain way based on the given sitting arrangement and so form a group. Which is the one that does not belong to the group ? (1) R (2) S (3) Q (4) M (5) O 86. Which of the following is true regarding T as per the given sitting arrangement ? (1) T faces the centre (2) Only two people sit between N and T (3) Only three people sit between T and P (4) S sits second to left of T (5) Only one person sits between T and R 87. What is M’s position with respect to T ? (1) Second to the left (2) Fifth to the right (3) Third to the right (4) Third to the left (5) Fourth to the right

    Directions (88–92) : Study the following information carefully and answer the questions given below : (SBI Associates PO Online Exam. 30.11.2014)

    Ten people are sitting in two parallel rows containing five people each in such a way that there is an equal distance btween adjacent persons. In row – 1 J, K, L, M and N are seated and all of them are facing north and in row-2 P, Q, R, S and T are seated and all facing south (but not necessarily in the same order). Each person also likes different flowers namely Gazania, Plumeria, Tulip, Orchids, Lily, Rose, Daffodil, Daisy, Peony and Dahlia (but not necessarily in the same order) N sits exactly in the centre of the row and faces the one who likes Orchids. Only one person sits between N and the one who likes Lily. S faces one of the immediate neighbours of the one who likes Lily. Only one person sits between P and the one who likes Orchids. P is not an immediate neighbour of S. P faces one of the immediate neighbours of L. M is neither an immediate neighbour of L nor faces S. Q is not an immediate neighbour of S and faces the one who likes Plumeria. M does not face R. M faces one of the immediate neighbours of the one who likes Rose. Only one person sits between the one who likes Rose and the one who likes Gazania. The one who likes Peony and the one who likes Daisy face each other. Only two people sit between the one who likes Daisy and the one who likes Daffodil. J does not like Daffodil. The one who likes Dahlia sits fourth to the right of the one who likes Peony. 88. Who amongst the following is an immediate neighbour of the one who likes Peony ? (1) The one who likes Rose (2) The one who likes Lily (3) The one who likes Gazania (4) K (5) T 89. Which of the following flowers does S like ? (1) Lily (2) Gazania (3) Orchids (4) Rose (5) Daffodil 90. Which of the following is definitely true as per the given arrangement ? (1) L faces R (2) Q is an immediate neighbour of P

    BPRE–340

    (3) The one who likes Daisy is an immediate neighbour of Q (4) Q likes Daffodil (5) The one who likes Dahlia faces N 91. Which of the following pairs represents the people sitting at the extreme ends of the two rows ? (1) P, J (2) Q, L (3) R, K (4) S, K (5) S, L 92. Who amongst the following likes Tulip ? (1) J (2) P (3) S (4) Q (5) N Directions (93–97) : Study the following information carefully and answer the questions given below : (SBI PO Phase–I (Preliminary) Online Exam. 20.06.2015)

    Eight friends — C, D, E, F, N, O, P and Q — are sitting in a straight line facing north but not necessarily in the same order. O is sitting second to the right of P. E is sitting third to the left of Q. F is sitting fourth to the left of P. E does not sit on the extreme end of the line. D is immediate neighbour of neither F nor P. C is not an immediate neighbour of P. 93. Which of the following pairs represents the two persons sitting at the extreme ends of the line ? (1) F, C (2) D, F (3) C, Q (4) N, D (5) O, F 94. Which of the following statements is definitely true as per the given arrangement ? (1) F and E are immediate neighbours of P. (2) C is sitting third to the right of Q. (3) There are only three persons between D and E. (4) N is sitting to the immediate left of P. (5) All the given statements are true 95. Who among the following is sitting third to the right of N ? (1) O (2) D (3) C (4) E (5) Q

    RANKING/ARRANGEMENT 96. What is the position of C with respect to Q ? (1) Fourth to the right (2) Third to the right (3) Fourth to the left (4) Third to the left (5) Second to the right 97. Four of the following five are alike in a certain way based on the given arrangement and hence they form a group. Which one of the following does not belong to that group ? (1) FN (2) PD (3) EQ (4) CP (5) NQ Directions (156–160) : Study the following information carefully and answer the questions given below : (SBI PO Phase–I (Preliminary) Online Exam. 20.06.2015)

    Eight persons — M, N, O, P, Q, R, S and T — are sitting around a circular table at equal distance between each other, but not necessarily in the same order. Some of them are facing the centre while some others are facing outside (i.e., in a direction opposite to the centre) Note : Facing the same direction means if one faces the centre then the other also faces the centre and vice– versa. Facing opposite directions means if one person faces the centre then the other person faces outside and vice–versa. R is sitting second to the right of Q. Only three persons are sitting between R and S. T is sitting second to the right of R. T faces the centre. R and S face opposite directions. P and S face opposite directions. N is sitting second to the left of P. P is not an immediate neighbour of Q. Only one person is sitting between P and O. O is not an immediate neighbour of Q. M is sitting third to the left of T. The immediate neighbours of T face opposite directions. M and R face opposite directions. N faces the same direction as that of O. 98. Which of the following statements is true regarding T according to the given seating arrangement ? (1) T is sitting second to the left of S. (2) T is sitting exactly between O and P. (3) T is sitting just opposite to N. (4) There are four persons between T and Q. (5) T faces the opposite direction as that of M.

    99. How many persons in the given seating arrangement face outside ? (1) Three (2) Four (3) Five (4) Six (5) Two 100. Four of the following five are alike in a certain way based on the given seating arrangement and hence they form a group. Which is the one that does not belong to that group ? (1) O (2) S (3) M (4) Q (5) N 101. Who among the following sits exactly between S and Q when counted from the left of S ? (1) None (2) O (3) M (4) N (5) T 102. What is the position of M with respect to R ? (1) Other than those given as options (2) Third to the right (3) Second to the left (4) Second to the right (5) Third to the left Directions (103 – 107) : Study the following information carefuly and answer the questions given below : (SBI PO Phase–I (Preliminary) Online Exam. 21.06.2015)

    Eight persons A, B, C, D, E, F, G and H are sitting around a circular area of equal distances between each other, but not necessarily in the same order. Some of the people are facing the centre while some face outside. i.e. in a direction opposite to the centre. (NOTE : Same direction means if one person is facing the centre then the other also faces the centre and viceversa. Opposite direction means that if one person is facing the centre then the other person faces outside and viceversa). D sits third to the right of B. E sits second to the left of B. Immediate neighbours of B face the same direction (i.e. if one neighbour faces the centre the other neighbour also faces the centre and vice-versa). C sits second to the left of E. E faces the centre. F sits third to the right of C. G sits second to the left of H. H is not an immediate neighbour of B. G faces the same direction as D. Immediate neighbours of E face opposite directions (i.e.

    BPRE–341

    if one neighbour faces the centre the other neighbour faces outside and viceversa.). Immediate neighbours of F face opposite directions (i.e. if one neighbour faces the centre the other neighbour faces outside and vice-versa). 103. How many persons in the given arrangement face the centre ? (1) Two (2) Three (3) One (4) Four (5) Five 104. Which of the following is true regarding A as per the given seating arrangement ? (1) H sits second to the left of A (2) A faces outside (3) Only two persons sit between A and B. (4) C is one of the immediate neighbours of A. (5) Only three persons sit between A and G. 105. Four of the following five are alike in a certain way based on the given seating arrangement and so form a group. Which is the one that does not belong to that group ? (1) F (2) B (3) H (4) G (5) D 106. What is E’s position with respect to H ? (1) Third to the left (2) Second to the right (3) To the immediate right (4) To the immediate left (5) Third to the right 107. Who sits second to the left of G ? (1) H (2) A (3) B (4) Other than those given as options (5) F Directions (108–112) : Study the following information carefully and answer the questions given below : (SBI PO Phase–I (Preliminary) Online Exam. 21.06.2015)

    Eight persons E, F, G, H, L, M, N and O are seated in a straight line, facing North, but not necessarily in the same order. O sits at the extreme right end of the line. Only four persons sit between O and G. Both F and M are immediate neighbours of G. Only two persons sit between M and L. L is not an immediate neighbour of O. N sits second to left of E.

    RANKING/ARRANGEMENT 108. What is the position of L with respect to G ? (1) Third to the right (2) To the immediate left (3) Second to the right (4) Fourth to the right (5) Second to the left 109. Based on the given arrangement, which of the following is true with respect to N ? (1) Only three persons sit between N and O. (2) None of the given options is true (3) Only one person sits to the right of N. (4) E sits to immediate right of N. (5) Both L and F are immediate neighbours of N. 110. Who amongst the following persons represents the person seated at the extreme left end of the line ? (1) L (2) N (3) E (4) F (5) H 111. How many persons are seated between O and E ? (1) Two (2) Three (3) Four (4) None (5) One 112. Who amongst the following sit exactly between M and L ? (1) E,N (3) F,G

    (2) F,O (4) E,G

    (5) H,N Directions (113–117) : Study the following information carefully to answer the given questions. (SBI PO Phase–I (Preliminary) Online Exam. 27.06.2015)

    Eight friends G, H, I, J, N, O, P and Q are seated in a straight line facing north, but not necessarily in the same order. ● H sits second to right of O. O sits at one of the extreme ends of the line. ● Only three people sit between H and N. ● I sits third to the left of J. Only two people sit between J and G. ● P is not an immediate neighbour of N. 113. Who amongst the following represents the person seated at the extreme right end of the line ?

    (1) P (2) Q (3) O (4) I (5) G 114. Who amongst the following sit exactly between J and G ? (1) O, Q (2) H, O (3) O, P (4) N, Q (5) H, N 115. What is the position of P with respect to N ? (1) Third to the left (2) Second to the right (3) Fourth to the right (4) Third to the right (5) Second to the left 116. Based on the given arrangement, which of the following is true with respect to Q ? (1) Only two persons sit between Q and N. (2) Only two persons sit to the right of Q. (3) None of the given options is true. (4) Both I and G are immediate neighbours of Q. (5) P sits to immediate right of Q. 117. How many persons are seated between P and G ? (1) None (2) One (3) Three (4) Four (5) Two Directions (118–122) : Study the following information carefully and answer the questions given below : (SBI PO Phase–I (Preliminary) Online Exam. 27.06.2015)

    Eight persons N, O, P, Q, R, S, T and U are sitting around a circular area at equal distances between each other, but not necessarily in the same order. Some of the people are facing the centre while some face outside (i.e. in a direction opposite to the centre). (NOTE : Same direction means that if one person is facing the centre then the other person also faces the centre and vice-versa. Opposite direction means that if one person is facing the centre then the other person faces outside and vice-versa.) R sits second to the right of T. T face the centre. O sits third to the left R. R and O face opposite directions. Immediate neighbours of O face the centre. P sits second to the right of O. U sits to the immediate left of P. N sits second to the left of Q. Q faces the

    BPRE–342

    same direction as O. Q is not an immediate neighbour of T. Immediate neighbours of R face opposite directions. (i.e. if one neighbour faces the centre the other neighbour faces outside and vice-versa.) 118. Four of the following five are alike in a certain way based on the given seating arrangement and so form a group. Which is the one that does not belong to that group ? (1) U (2) S (3) P (4) N (5) Q 119. Who sits second to the left of U ? (1) T (2) O (3) Q (4) Other than those given as options (5) S 120. Which of the following is true regarding S as per the given seating arrangement ? (1) U is one of the immediate neighbours of S. (2) S sits third to the right of R. (3) S faces the centre. (4) Only two persons sit between S and T. (5) Only one person sits between S and N. 121. What is P’s position with respect to N ? (1) Second to the left(2) Second to the right (3) Third to the right (4) Third to the left (5) Fourth to the left 122. How many persons in the given arrangement face outside ? (1) Two (2) Three (3) Four (4) Five (5) One Directions (123–127) : Study the following information carefully and answer the questions given below : (SBI Probationary Officer Online Exam, 28.06.2014)

    Eight friends A, B, C, D, E, F, G and H are sitting around a circle facing the centre. Four out of the eight friends drive a car while four drive a bike. No two friends who drive a car are immediate neighbours of each other. A, who drives a car sits third to left of H. G, who drives a bike sits second to right of E. F and C are immediate neighbours of each other. F does not drive a car. B is an immediate neighbour of F but not of H.

    RANKING/ARRANGEMENT 123. Four of the following five are alike in a certain way based on the above arrangement and so form a group. Which is the one that does not belong to the group ? (1) B (2) E (3) A (4) C (5) D 124. If all the eight friends are made to sit in alphabetical order in the anti-clockwise direction starting from A, positions of how many will remain unchanged (excluding A) ? (1) None (2) One (3) Two (4) Three (5) Four 125. In which of the following pairs do both the members drive a bike ? (1) F, D (2) B, D (3) F, C (4) H, B (5) E, H 126. Which of the following is true about D ? (1) D drives a bike (2) H and E are immediate neighbours of D (3) H sits third to left of D (4) D sits second to the left of C (5) None is true 127. What is F’s position with respect to E ? (1) Immediate right (2) Third to the right (3) Second to the left (4) Fourth to the left (5) Second to the right Directions (128–132) : Study the following information carefully and answer the questions given below : (SBI Associates PO Online Exam, 29.11.2014)

    Ten people are sitting in two parallel rows, containing five people each in such a way that there is equal distance between adjacent persons. In row— 1 : J, K, L, M and N are seated and all of them are facing north and in row—2 : P, Q, R, S and T are seated and all of them are facing south (but not necessarily in the same order). Each person also likes a different cartoon character namely, Batman, Ben 10, Dexter, Donald Duck, Tom, Jerry, Noddy, Oswald, Superman and Spiderman (not necessarily in the same order).

    N sits exactly in the centre of the row and faces the one who likes Tom. Only one person sits between S and the one who likes Tom. S faces one of the immediate neighbours of the one who likes Jerry. Only two people sit between K and the one who likes Jerry. K faces the one who likes Oswald. P faces one of the immediate neighbours of L. P does not like Oswald. M is an immediate neighbour of L. The one who likes Donald Duck is not an immediate neighbour of S. T is not immediate neighbour of P and faces the one who likes Ben 10. T is not at the extreme end of the line. R faces the one who likes Superman. R does not like Tom. The one who likes Spiderman faces north. S does not face the one who likes Spiderman. The one who likes Batman sits to the immediate right of the person who likes Noddy. S does not like Batman. 128. Which of the following pairs represent the people sitting at the extreme end of the two rows ? (1) S, M (2) Q, L (3) P, N (4) Q, J (5) R, L 129. Who amongst the following likes Donald Duck ? (1) S (2) P (3) R (4) T (5) Q 130. Who amongst the following is an immediate neighbour of the one who likes Spiderman ? (1) The one who likes Jerry (2) The one who likes Noddy (3) K (4) The one who likes Superman (5) T 131. Which of the following cartoon character does Q like ? (1) Batman (2) Tom (3) Jerry (4) Noddy (5) Oswald 132. Which of the following statements is definitely true as per the given arrangement ? (1) L faces R (2) Q likes Oswald (3) Q is an immediate neighbour of P (4) The one who like Superman is an immediate neighbour of P (5) The one who likes Dexter faces N.

    BPRE–343

    Directions (133–138) : Study the following information carefully and answer the questions given below : (SBI Assistant Manager (System) Exam, 17.01.2016)

    Eight people L, M, N, O, P, Q, R and S are sitting around a circular table with equal distance between each other but not necessarily in the same order. Some of them are facing the centre while some are facing outside (i.e. away from the centre). M sits third to the left of L. Only three people sit between M and S. P sits to the immediate right of S. Immediate neighbours of P face opposite directions (i.e., if one neighbour faces the centre then the other neighbour faces outside and vice-versa.). Only one person sits between P and O. R sits second to the right of O. Both R and N face the same direction as S (i.e., if S faces the centre then R and N also face the centre and viceversa.) Immediate neighbours of Q faces opposite directions (i.e., if one neighbour faces the centre then the other neighbour faces outside and viceversa). P does not face outside. O faces a direction opposite to that of M. 133. How many people sit between L and Q when counted from the left of Q ? (1) Five (2) None (3) Four (4) One (5) Three 134. Which of the following statements is true as per the given arrangement ? (1) Q faces the centre (2) Only three people sit between P and L. (3) R sits to the immediate right of N. (4) None of the given options is true (5) N is an immediate neighbour of O. 135. Who amongst the following sits third to the left of P ? (1) Q (2) N (3) R (4) L (5) M 136. How many people face the centre as per the given arrangement ? (1) Four (2) One (3) Two (4) Three (5) More than four

    RANKING/ARRANGEMENT 137. What will come in place of question mark (?) in the given series based on the positions as given in the arrangement ? PS LR MP SM ? (1) ON (2) SO (3) NQ (4) OL (5) LS 138. As per the given arrangement O is related to L in the same way as M is related to S. Following the same pattern to whom is N related ? (1) P (2) Q (3) M (4) R (5) O Directions (139–141) : Study the following arrangement carefully and answer the questions given below : (SBI Assistant Manager (System) Exam, 17.01.2016)

    58317925128345285 423739624295364612 139. How many such 3s are there in the given arrangement each of which is immediately preceded as well as immediately followed by an even digit ? (1) More than three (2) One (3) Three (4) Two (5) None 140. How many such 2s are there in the given arrangement each of which is immediately followed by a perfect square ? (Note : 1 is also a perfect square) (1) One (2) More than three (3) Two (4) Three (5) None 141. Which of the following is fifth to the left of the sixteenth digit from the left end of the given arrangement ? (1) 5 (2) 7 (3) 2 (4) 8 (5) 3 142. How many 5s are there in the given arrangement each of which is immediately followed by a digit which has a numerical value of less than three ? (1) Two (2) Three (3) One (4) More than three (5) None 143. If all the even digits are deleted from the given arrangement, which of the following will be eighth from the right end of the arrangement ?

    (1) 7 (2) 9 (3) 1 (4) 5 (5) 3 Directions (144–149) : Study the following information carefully and answer the questions given below : (SBI PO Online (Pre.) Exam, 02.07.2016 (Ist Shift))

    Eight persons – H, I, J, K, L, M, N and O – are standing in a straight line at equidistant. Some of them are facing north while others are facing south. M is standing third to the right to H. M is standing at one of the extreme ends. L is standing third to the left of H. The immediate neighbours of J face north. N is not an immediate neighbour of H. The persons standing at the extreme ends face the same direction (both are facing either North or South). The immediate neighbours of H face just opposite direction as that of M. The immediate neighbours of O face opposite direction with respect to each other. K is one of the immediate neighbours of L and is facing north. I is standing between J and M. Not more than four persons are facing north. L is to the immediate right of K. 144. Who among the following is third to the left of N? (1) K (2) J (3) H (4) I (5) O 145. The immediate neighbours of L are : (1) M and N (2) N and O (3) K and N (4) N and H (5) J and H 146. How many persons are standing exactly between I and O? (1) Three (2)Four (3) One (4) Two (5) None 147. Four of the following five are alike in a certain way based on the above arrangement and hence form a group. Which of the following does not belong to that group? (1) N (2) L (3) O (4) J (5) K 148. Who among the following is exactly between L and J? (1) O (2) N (3) H (4) I (5) None

    BPRE–344

    149. Who among the following is fourth to the right of J? (1) N (2) I (3) H (4) L (5) K Directions (150–155) : Study the following information carefully and answer the questions given below : (SBI PO Online (Pre.) Exam, 02.07.2016 (IInd Shift))

    Eight friends viz., G, H, I, J, W, X, Y and Z are sitting in a straight line, but not necessarily in the same order. Some of them are facing North while some others are facing South. ● J sits third to the right of W. W sits at any extreme and of the line. Both the immediate neighbours of J face North. ● Only one person sits between J and H. H is not an immediate neighbour of W. ● I sits second to the left of Y. Y is not an immediate neighbour of J. Both the immediate neighbours of Y face South. ● G does not sit at any extreme end of the line. G is not an immediate neighbour of J. ● More than one person are sitting between I and X. ● The immediate neighbours of Z face a direction opposite to Z (i.e., if Z faces North, the immediate neighbours of Z face South and vice–versa). ● The immediate neighbours of Y face the same direction as that of Y (i.e., if Y faces North, both the immediate neighbours of Y also face North and vice–versa). 150. Who amongst the following sits third to the right of I? (1) W (2) X (3) H (4) G (5) Z 151. How many persons sit between G and H? (1) Three (2) More than three (3) None (4) One (5) Two 152. Which of the following statements is true based on the given arrangement? (1) Only three persons face North. (2) H and J face opposite directions (3) X sits at one of the extreme ends of the line. (4) Z faces South (5) None of the given statements is true

    RANKING/ARRANGEMENT 153. Who amongst the following are immediate neighbours of H? (1) X and Y (2) I and Z (3) Y and Z (4) Other than those given as options (5) Y and I 154. Four of the following five are alike in a certain way based on the given arrangement and hence they form a group. Which one of the following does not belong to that group? (1) Y, J (2) H, G (3) X, I (4) W, J (5) I, G 155. In the given arrangement X is related to H and Z is related to I following a certain pattern, with which of the following W is related to, following the same pattern? (1) H (2) J (3) Y (4) Z (5) G Directions (156–160) : Study the following information carefully to answer the questions given below : (SBI PO Phase-I (Pre.) Online Exam, 03.07.2016 (Ist Shift))

    Eight friends – A, B, C, D, L, M, N and O – are seated in a straight line, but not necessarily in the same order. Some of them are facing north while some are facing south. Only three persons sit to the left of N. B sits second to the right of N. C sits third to the left of O. O is not an immediate neighbour of B. O does not sit at any of the extreme ends of the line. C and O face same direction (i.e. if C faces north then O also faces north and vice–versa.) Both the immediate neighbours of D face north. D does not sit at any of the extreme ends of the line. Persons sitting at extreme ends face opposite directions (i.e. if one person faces north then the other faces south and vice–versa.) Both the immediate neighbours of N face same direction (i.e., if one neighbour faces north then the other also faces north and vice–versa.) A sits second to the left of L. D faces a direction opposite to L. (i.e., if L faces north then D faces south and vice–versa.) 156. How many persons sit exactly between B and O? (1) More than the four (2) Three

    (3) Two

    (4) Four

    (5) One

    157. Who among the following represent the immediate neighbours of the persons sitting at the extreme ends of the line? (1) D, O (2) C, D (3) D, M (4) A, L (5) A, M 158. Who amongst the following faces north? (1) A (2) L (3) O (4) N (5) C 159. Which of the following statements is true based on the given arrangement? (1) B and C face the same direction. (2) None of the given options is true (3) B faces south. (4) Only three persons face south. (5) M sits at one of the extreme ends of the line. 160. Who amongst the following sits second to the right of C? (1) M (2) No one as only one person sits to the right of C. (3) A (4) B (5) L Directions (161–165) : Study the following information carefully and answer the questions given below :

    (1) C (2) N (3) E (4) M (5) No one as less than two people sit to the right of L. 162. Which of the following statements is true, based on the given arrangement? (1) M and E face opposite directions. (2) M sits at one of the extreme ends of the line. (3) Only four people face south. (4) None of the given options is true (5) K faces north. 163. Who among the following represents the persons sitting at extreme ends of the line? (1) L, M (2) L, N (3) F, M (4) M, N (5) F, N 164. How many people sit between C and K? (1) Four (2) Three (3) More than four (4) Two (5) One 165. Who among the following are immediate neighbours of F? (1) Other than those given as options (2) E and M (3) D and K (4) D and N (5) E and D Directions (166–170) : Study the following information carefully and answer the questions given below :

    (SBI PO Online (Pre.) Exam, 03.07.2016 (IInd Shift))

    (SBI PO Online (Preliminary) Exam, 10.07.2016)

    Eight friends, C, D, E, F, K, L, M and N are seated in a straight line, but not necessarily in the same order. Some of them are facing north while some are facing south. ● Only one person sits to the left of K. E sits third to the right of K. ● C sits second to the left of E. C is not an immediate neighbour of K. ● C faces a direction opposite to that of E (i.e. if E faces north then C faces south and vice-versa.) ● Both the immediate neighbours of D face south. D is not at any of the extreme ends of the line. ● F sits third to the right of N. ● Both the immediate neighbours of C face north. L is not an immediate neighbour of E. ● Immediate neighbours of K face opposite directions (i.e. if one neighbour faces north then the other faces south and vice-versa.) 161. Who amongst the following sits second to the right of L?

    Eight friends, C, D, E, F, L, M, N and O are seated in a straight line, but not necessarily in the same order. Some of them are facing north while some face south. ● Only three persons sit to the right of M. E sits second to the left of M. ● F sits third to the right of O. O is not an immediate neighbour of M. O does not sit at any of the extreme ends of the line. ● Both the immediate neighbours of O face south. ● D sits second to the right of N. ● As many people sit between M and D as between M and L. ● Immediate neighbours of F face opposite directions (i.e. if one person faces north then the other person faces south and vice-versa.) ● L and F face direction opposite to C (i.e. If C faces north then both L and F face south and vice-versa.)

    BPRE–345

    RANKING/ARRANGEMENT Persons sitting at extreme ends face opposite directions (i.e., if one person faces north then the other person faces south and vice–versa) 166. Which of the following statements true, based on the given arrangement? ●

    (1) D faces north. (2) Only three persons face south. (3) L sits at one of the extreme ends of the line. (4) O and E face the same direction (5) None of the given options is true 167. How many persons sit to the left of O? (1) Three (2) More than four (3) One (4) Four (5) Two 168. Who amongst the following faces north? (1) E (2) M (3) F (4) N (5) D 169. Who amongst the following sits second to the left of L? (1) O (2) No one as less than two persons sit to the left of L (3) N (4) F (5) D 170. Who among the following represent the persons sitting at extreme ends of the line? (1) D, N (2) O, D (3) L, N (4) D, L (5) D, C Directions (171–175) : Study the following information carefully and answer the questions given below : (SBI PO Online Main Exam, 31.07.2016)

    Twelve persons are sitting in two parallel rows containing six persons each, in such a way that there is an equal distance between adjacent persons. In row – 1, N, Q, P, R, T and S are seated (but not necessarily in the same order) and all of them are facing south. In row – 2, Z, D, F, X, H and V are seated (but not necessarily in the same order) and all of them are facing north. Therefore, in the given seating arrangement, each member seated in a row faces another member of the other row.

    The one facing D sits third to the right of N. D does not sit at any of the extreme ends of the line. ● Only one person sits between N and P. ● The one who faces Q sits third to the left of X. X does not sit at any of the extreme ends of the line. ● Z sits second to the left of X. ● The one facing H sits second to the left of S. S does not face V. ● T is neither an immediate neighbour P nor Q. 171.Who amongst the following faces S? (1) F (2) Z (3) D (4) H (5) X 172. Which of the following groups of persons represents the persons sitting at the extreme ends of both the rows? (1) Q, P, Z, F (2) S, R, V, F (3) V, H, R, T (4) N, R, X, H (5) P, R, F, V 173. Which of the following statements is true as per the given arrangement? (1) S faces Z (2) None of the given statements is true (3) D sits exactly between F and H (4) S is an immediate neighbour of Q (5) P faces one of the immediate neighbours of D 174. As per the given arrangement, four of the following five are alike in a certain way and hence they form a group. Which one of the following does not belong to the group? (1) N – X (2) T – F (3) D – Q (4) R – V (5) Z – P 175. Who amongst the following sits third to the left of person who faces S? (1) F (2) H (3) X (4) V (5) D Directions (176–181) : Study the following information carefully and answer the questions given below : ●

    (SBI PO Online Main Exam, 31.07.2016)

    The Government of State KLM organised a health camp where counters of twelve different branches were arranged around a circular area marked 1 to 12 like the digits of a clock. These

    BPRE–346

    counters were alloted to twelve volunteers and all of them were facing away from the centre of the circular area, The twelve volunteers were respectively, P, T, D, Z, K, Q, M, R, V, B, H and F and the twelve branches were respectively, Diagnosis, Vaccination, Surgery, Orthopaedics, Paediatrics, Neurology, Nephrology, Integrated Services, Dentistry, Blood donation, Ophthalmology and Cardiology. (Note : The information given above is not necessarily in the same order). ● P is sitting at the Counter Number 12. V, who deals with Nephrology, sits just opposite to P. ● Only three volunteers are sitting between P and T. P and T deal with Diagnosis and Ophthalmology respectively. ● M is sitting at the Counter Number 4 and that of R at the Counter Number 10. M deals with Surgery while R deals with Orthopaedic. ● T does not deal with Diagnosis and he is not an immediate neighbour of D, Z, M or R. ● B deals with Neurology and he is sitting exactly between T and V. ● Paediatrics counter is just opposite to Integrated Services counter and these counters were under the control of H and Z respectively. ● R is sitting exactly between F and H. F deals with Cardiology. Cardiology counter is exactly between the counters of Orthopaedics and Diagnosis. ● D sits to the immediate right of P. D does not deal with Vaccination or Blood donation. ● K sits exactly between D and Z and he does not deal with Vaccination. 176. Which of the following statements is not true with respect to Paediatrics counter? (1) H deals with the Paediatrics counter (2) This counter is at the position of 9 of the clock (3) This counter is exactly between the counters of Orthopaedics and Ophthalnology (4) The Dentistry counter is fifth to the right of Paediatrics counter (5) All the given statements are true

    RANKING/ARRANGEMENT 177. Who among the following deals with Dentistry counter? (1) Z (2) K (3) D (4) M (5) Other than those given as options 178. As per the given arrangement four of the following five are alike in a certain way and hence form a group. Which one of the following does not belong to that group? (1) B – Paediatrics (2) Vaccination – Diagnosis (3) Cardiology – D (4) Surgery – Nephrology (5) Z – Q 179. Which one of the following pairs does not represent the volunteers who are sitting just opposite to each other? (1) T – Blood donation (2) R – Surgery (3) K – Q (4) Diagnosis – V (5) F – Neurology 180. Which of the following statements is true regarding the volunteer who deals with Vaccination counter? (1) Q deals with Vaccination counter. (2) The one who deals with Vaccination sits at Counter number 4 (3) The Vaccination counter is exactly between the counters of Surgery and Blood donation (4) The volunteer who deals with Vaccination is third to the left of K. (5) R sits just opposite to the one who deals Vaccination counter 181. In a vertical queue of 13 people, all facing east, K stands exactly at the centre of the queue. No one stands between K and W. Only five people stand between W and P. L stands at one of the positions before P but not at the beginning of the queue. How many people stood after W? (1) Three (2) None (3) Five (4) Six (5) Seven (SBI PO Online (Preliminary) Online Exam. 30.04.2017)

    Directions (182–187) : Study the following information carefully and answer the questions given below : (SBI PO Online (Preliminary) Online Exam. 30.04.2017)

    Eight persons F, G, H, I, O, P, Q and R are seated in a straight line facing north. Each of them works on different floors of an office building viz. 7th, 16th, 18th, 23rd, 31st, 35th, 44th, 47th. None of the given information is necessarily in the same order. ● O sits fourth to the right of the one who works on the 31st floor. The one who works on the 23rd floor sits second to the right of O. ● Q sits third to the left of I. I is not an immediate neighbour of O. Q does not sit at any of the extreme ends of the line. ● Only two people sit between Q and P. The one who works on the 44th floor sits to the immediat right of H. H is not an immediate neighbour of P. ● The difference between the numerical values of floor numbers in which P and the one to the immediate right of P work is 13. ● Only one person sits between F and the one who works on the 35th floor. F is not an immediate neighbour of I. ● More than two people sit between R and the one who works on the 16th floor. O does not work on the 16th floor. ● H works on a floor lower than O. 182. Which of the following pairs represents the persons seated at the two extreme ends of the line? (1) G and the one working on the 7th floor. (2) I, R (3) The ones working on the 18th and 44th floors. (4) P and the one working on 16th floor (5) The ones working on the 31st and 23rd floors 183. What is the difference between the floor numbers on which P and R work? (1) 4 (2) 31 (3) 3 (4) 16 (5) 15

    BPRE–347

    184. F is related to the one working on the 47th floor following a certain pattern based on the given arrangement. In the same pattern, P is related to the one working on the 44th floor. To who amongst the following is H related to following the same pattern? (1) The one working on the 35th floor (2) The one to the immediate left of R. (3) The one sitting second to the left of O. (4) The one working on the 16th floor. (5) O 185. Fill in the blanks (respectively in the same order) in order to make the statement correct based on the given arrangement. G _______ and O __________. (1) works on the 47th floor, sits to the immediate left of Q. (2) works on one of the floors above H, works on one of the floors below F. (3) sits to immediate left of I, works on the 44th floor. (4) sits second to the right of Q, works three floors above I. (5) Other than those given as options. 186. How many people sit to the left of the one working on the 35th floor? (1) One (2) Two (3) None (4) Four (5) Three 187. In a vertical queue of 15 people all facing north with equal distance between each other, P stands third from one of the extreme ends of the queue. Only six people stand between P and A. The number of people standing before A is same as that standing after G. Q stands exactly between A and G. How many people stand after Q? (1) Seven (2) Four (3) Twelve (4) Six (5) Eight (SBI PO Online (Preliminary) Online Exam. 06.05.2017)

    RANKING/ARRANGEMENT Directions (188–192) : Study the following information carefully and answer the questions given below : (SBI PO Online (Preliminary) Online Exam. 06.05.2017)

    Eight persons – A, B, C, D, P, Q, R and S – all of them facing north are sitting in a straight line. Each person is having different age viz., 14, 16, 21, 26, 28, 17, 29 and 31 but not necessarily in the same order. B sits at one of the extreme ends of the line. Q’s age is not a prime number. There are three persons sitting between one whose age is 16 and B. One whose age is 29 sits second to the left of C. C sits to the right of one whose age is 16 but not immediately. D sits at the third position from the left end. The one who is the eldest among all is not an immediate neighbour of D, who is not the youngest among all. P sits to the immediate right of one who is the eldest. C’s age is neither 14 nor 17. Difference of D’s immediate neighbours’ ages is 5. There are two persons sitting between one whose age is 26 and S, who sits to the right of one whose age is 26. R is not to the immediate left of S. Q sits third to the right of A. 188. Who among the following persons sits second to the right of S? (1) B (2) C (3) Q (4) A (5) None of these 189. What is the age of one who sits to the immediate right of P? (1) 17 (2) 28 (3) 14 (4) 26 (5) Cannot be determined 190. Four of the following five are alike in a certain way based on the given arrangement and hence they form a group. Which one of the following does not belong to that group? (1) 14, R (2) P, 29 (3) 26, B (4) Q, 29 (5) S, 21 191. Who among the following persons are immediate neighbours of one whose age is 14? (1) P and Q (2) C and S (3) S and B (4) R and C (5) None of these 192. Which of the following statement is/are definitely true? (1) One who sits third to the right of S is Q, whose age is 14.

    (2) A’s immediate neighbours ages are consecutive numbers. (3) R is an immediate neighbour of S and C. (4) A’s age is 21 and R’s age is a prime number. (5) Both (2) and (4) Directions (193–198) : Study the following information carefully and answer the questions given below : (SBI PO Online (Preliminary) Exam, 07.05.2017)

    Eight people viz. A, B, C, D, P, Q, R and S are sitting in a straight line. They all are facing north. Each one of them has a different age i.e. 14,16,17,19, 21, 23, 26 and 31 year, but not necessarily in the same order. B sits at one of the extreme end of the row. There are three persons sitting between C and Q. Q is of neither 14 nor 19 years old. There are two persons sitting between D and the person whose age is 23 years. Neither Q nor D is the oldest person. Age difference of immediate neighbours of D is 5 years. A sits to the right of R, but not immediate right. There are three persons sitting between B and the one whose age is 16 years. The one whose age is 19 years sits third to the right of C. R sits to the right of B. Q sits second to the right of the person whose age is 23 years. P sits immediate left of the person whose age is 14 years. Q is not youngest person. The one, whose age is 31 years is not an immediate neighbour of the youngest person. C is not the fourth oldest person. 193. Who sits second to the right of D? (1) A (2) S (3) P (4) R (5) None of these 194. How many persons sit between the persons whose age is 31 years and S? (1) Four (2) Five (3) Three (4) One (5) None of these 195. Who among the following persons is 26 years old? (1) R (2) D (3) C (4) S (5) None of these 196. If P is related to 16 years in the same way as B is related to 26 years, then which of the following is R related to, following the same pattern?

    BPRE–348

    (1) 19 year (2) 17 year (3) 21 year (4) 31 year (5) None of these 197. What is the age difference of A’s immediate neighbours? (1) Three (2) Seven (3) Five (4) Six (5) None of these 198. In a vertical row 13 persons are sitting. A is seventh from the beginning and two persons sits between G and A. The number of persons between A and L is same as the number of persons between G and Q. Then what is the position of Q from the beginning? (1) Fourth (2) Eighth (3) Sixth (4) Ninth (5) Cannot be determined (SBI PO Online (Preliminary) Exam, 07.05.2017)

    Directions (199–201) : Study the following information carefully and answer the questions given below : (SBI PO Online (Pre.) Exam, 02.07.2016 (Ist Shift))

    Seven persons – A, B, C, D, K, L and N – are seated in ascending order of their salaries. N earns more than L and D. N earns more than A but he does not earn the highest. A earns more than L. The person who earns the second highest receives a salary of Rs. 35,000 while the third lowest earner receives Rs. 23,000. K earns less than L but more than D. C earns Rs. 18,000. D is not the lowest earner. 199. Who among the following earn(s) more than Rs. 23,000 but less than Rs. 35,000? (1) Only A (2) Only L (3) A and L (4) K and L (5) A and N 200. Who among the following may earn Rs. 21,000? (1) K (2) D (3) L (4) C (5) N 201. Who among the following earns Rs. 35,000? (1) A (2) L (3) B (4) K (5) N Directions (202–204) : Study the following information carefully and answer the questions given below : (SBI PO Phase-II (Main) Exam 04.06.2017)

    A certain number of people are sitting in a straight, horizontal line; facing north. The 17 year old sits fourth

    RANKING/ARRANGEMENT from the left end of the line, Only two people sit between the 17 year old and the 40 year old. There are eight people between the 40 year old and the 19 year old. The 5 year old sits fourth to the left of the 19 year old. The 9 year old sits second to the left of the 5 year old. No one sits to the right of the 19 year old. 202. How many people are sitting in the line ? (1) 9 (2) 16 (3) Cannot be determined (4) 10 (5) 11 203. Which of the following correctly represents the correct position of the 9 year old in the line ? (1) 8th from the right end (2) 2nd from the right end (3) 7th from the left end (4) 3rd to the right of the 40 year old (5) 8th from the left end 204. If Zara sits exactly between the 5 year old and the 9 year old, how many people sit between Zara and the 40 year old ? (1) Four (2) Five (3) Six (4) Two (5) Three Directions (205–209) : Study the following information carefully and answer the questions given below : (SBI PO Phase-II (Main) Exam 04.06.2017)

    Ten people are sitting in two parallel rows containing five people each, in such a way that there is equal distance between adjacent persons. In row-1, D, E, F, G and H are seated (not necessarily in the same order) and all of them are facing north, In row-2 K, L, M, N and O are seated (not necessarily in the same order) and all of them are facing south. Therefore, in the given seating arrangement each member seated in a row faces another member of the other row. People of each row consist one person of the given ages-19, 22, 29, 35 and 56 (all ages are in years). L sits second to the right of K. Neither O nor K sits at any of the extreme ends of the row. The person sitting to the immediate right of the one who faces K is 22 years old. D sits third to the left of the 22 year old. One of the immediate neighbours of the one who faces D is 35 years old. O sits

    second to the left of the 35 year old. The one who sits to the immediate left of O faces the 19 year old. In row-1, only two people sit between the 19 year old and the 35 year old. E sits third to the right of F. E faces the 56 year old. G and N face each other. One of the immediate neighbours of the one who faces M is 56 years old. The 29 year old person of row 2 faces the 56 years old. N’s age is an even;number. 205. Who sits second to the left of E ? (1) G (2) The 56 year old (3) H (4) The one who faces K (5) The 19 year old 206. Which of the following pairs represent the people who are of same age? (1) M, D (2) L, E (3) O, H (4) K, F (5) N, G 207. One of the people whose age is 19 years faces which of the following persons ? (1) G (2) E (3) K (4) N (5) L 208. What is the age of M (in years)? (1) 35 (2) 19 (3) 56 (4) 22 (4) 29 209. Which of the following statements is TRUE with respect to the given information? (i) H is 29 years old. (ii) N faces the 22 year old. (iii) F sits to the immediate right of D. Directions (210–211) : Study the following information carefully and answer the questions given below : (SBI PO Phase-II (Main) Exam 04.06.2017)

    There are two square fields of different size such that the larger one is surrounding smaller field. Four gates are there for each field in the middle of the sides. Eight people A, B, C, D, E, F, G and H are standing at different gates but not necessarily in the same sequence. The persons who are on the sides of larger field facing centre and the persons who are at sides of smaller field facing outside such that inner sides persons and outer sides persons are facing each other. There is one person standing between B and D. C faces B. A is to the immediate

    BPRE–349

    right of C. G is not the immediate neighbour of D. G faces neither D nor F. One person is standing between H and F. E is facing the centre. 210. Which of the following persons are facing each other? (1) BD (2) EB (3) FH (4) DE (4) AH 211. Four of the following five are alike in a certain way based on the certain arrangement. Which one of the following does not belong to the group? (1) EF (2) CH (3) DA (4) FC (5) BH Directions (212–215) : Study the following information carefully and answer the questions given below : Twelve people are sitting in two parallel rows containing six people each in such a way that there is an equal distance between adjacent persons. In row-1, P, Q, R, S, T and V are seated and all of them facing south. In row-2 A, B, C, D, E and F are seated and all of them are facing north. Therefore, in the given seating arrangement each member seated in a row faces another member of the row. A sits third to right of D. Neither A nor D sits at extreme ends. T faces D. V does not face A and V does not sit at any of the extreme ends. V is not an immediate neighbour of T. B sits at one of the extreme ends. Only two persons sit between B and E. E does not face V. Two persons sit between R and Q. R is not an immediate neighbour of T. C does not face V. P is not an immediate neighbour of R. SBI PO (Prelim Exam), 08.07.2018 (Shift-I))

    212. Who amongst the following sit at extreme ends of the row? (1) B, E (2) S, T (3) P, R (4) B, F (5) None of these 213. Who amongst the following faces A? (1) R (2) T (3) P (4) Q (5) S 214. How many persons are seated between T and S? (1) 1 (2) 2 (3) 3 (4) 4 (5) 5

    RANKING/ARRANGEMENT 215. P is related to V in the same way as C is related to F. To which of the following is E related to following the same pattern? (1) B (2) D (3) C (4) A (5) None of these Directions (216–219) : Study the following information carefully and answer the questions given below : A certain number of people are sitting in a straight line facing north. Each one of them has a different number of cookies with them. B sits fifth to the right of G. Only one person sits between B and T. The one who has 7 cookies sits to the immediate right of T. Only three people sit between X and the one who has 7 cookies. X is not an immediate neighbour of B. The one who has 12 cookies sits seventh to the left of X. The one who has 8 cookies sits third to the left of the one who has 12 cookies. The one who has 9 cookies sits third from the left end of the line. Only one person sits between M and the one who has 9 cookies. M sits thirteenth from one of the extreme ends of the line. The one who has 15 cookies sits fifth to the right of M. Only three people sit between W and the one who has 15 cookies. SBI PO (Prelim Exam), 08.07.2018 (Shift-II))

    216. What is the position of W from the right end of the line? (1) Sixth (2) Fourth (3) First (4) Eighth (5) Eleventh 217. How many people are sitting in the given line? (1) 22 (2) 13 (3) 25 (4) 17 (5) 15 218. How many people sit between B and the one who has 15 cookies? (1) More than three (2) One (3) None (4) Three (5) Two 219. If the difference between the number of cookies with T and the one who sits ninth from the left end of the line is 8, then how many cookies does T have? (1) 23 (2) 7 (3) 4 (4) 15 (5) 17

    Directions (220–224) : Study the following information carefully and answer the questions given below : Eight people are sitting in a straight line facing north. Each one of them is of a different age (in years) — 9, 18, 22, 29, 35, 42, 51 and 58. P sits third from the extreme left end of the line. Only three people sit between P and B. P’s age is double the age of B. The one who sits to the immediate right of B is 13 years older than B. G sits third to the left of the 42 year old. G does not sit at an extreme end. V sits to the immediate right of F. Only one person sits between V and the 35 year old. S is 51 years old. Only three people are sitting between the 51 year old and the 9 year old. M is an immediate neighbour of the 22 year old. Z sits at one of the positions to the right of M.

    220. What is the difference in ages of G and B (in years) ? (1) 23 (2) 13 (3) 7 (4) 9 (5) 40 221. What is the position of F with respect to S? (1) Third to the left (2) Fourth to the right (3) Immediate left (4) Fifth to the left (5) Sixth to the right 222. Which of the following statements is true with respect to the given information? (1) The sum of ages of F and V is more than 40. (2) V sits second from the left end of the line. (3) Z is younger than G. (4) B is 9 years old. (5) None of the given statements is true. 223. How many people sit between the 18 year old and V? (1) None (2) Two (3) More than three (4) One (5) Three 224. What is the age of Z? (1) 22 years (2) 29 years (3) 58 years (4) 18 years (5) 42 years Directions (225–229) : Study the following information carefully and answer the questions given below :

    Eight people are sitting in a straight line. Some of them are facing north and some are facing south. B is sitting second from one of the extreme ends of the line. E sits fourth to the right of B. Only two people sit between E and H. Immediate neighbours of H face opposite directions. A sits to the immediate left of D. A is neither an immediate neighbour of E nor H. D and A face opposite directions. People sitting at the extreme ends of the line face the same direction. F sits fourth to the left of C. G sits at one of the positions to the left of both H and E. E faces north. 225. Who is sitting second to the left of H? (1) D (2) F (3) A (4) C (5) E 226. Who amongst the following is an immediate neighbour of D ? (1) C (2) E (3) F (4) B (5) G 227. How many people are sitting between G and B ? (1) More than three (2) None (3) Three (4) Two (5) One 228. Which of the following statements is true with respect to the given information ? (1) Only two people sit between B and C (2) None of the given statements is true (3) A faces south (4) D sits at one of the extreme ends of the line (5) F and H face the same direction 229. Four of the following five are alike in a certain way based on their directions as per the given arrangement and so they form a group. Which one of them does not belong to the group ? (1) A (2) F (3) D (4) C (5) B Directions (230–234) : Study the following information carefully and answer the questions given below :

    (SBI PO Prelim Exam, 14.06.2019)

    (SBI PO Prelim Exam, 14.06.2019)

    SBI PO (Prelim Exam), 08.07.2018 (Shift-II))

    BPRE–350

    RANKING/ARRANGEMENT Ten boxes are kept above one another in a stack. Only three boxes are kept between A and S. Q is kept immediately below S. Only two boxes are kept between S and M. S is kept at one of the positions above M. Only one box is kept between M and B. Only one box is kept below B. C is kept immediately above K but not at the topmost position. R is kept at one of the positions above W but below G. 230. As per the given arrangement, four of the following five are alike in a certain way and hence they form a group. Which one of the following does not belong to the group ? (1) SG (2) MW (3) CA (4) KR (5) BR 231. As per the given arrangement, as many boxes are kept above G as below ________. (1) C (2) W (3) M (4) R (5) Q 232. Which boxes is/are kept exactly between R and A ? (1) Only M (2) Both B and W (3) Only C (4) Both S and Q (5) Only B 233. Which box is kept third from the top of the stack ? (1) A (2) C (3) R (4) G (5) K 234. How many boxes are kept between B and W ? (1) Five (2) Four (3) Three (4) None (5) Two Directions (235–239) : Study the following information carefully and answer the questions given below : (SBI PO Prelim Exam, 09.06.2019)

    Twelve people are sitting in two parallel rows containing six people each, in such a way that there is equal distance between adjacent persons. In row-1, B, C, D, E, F and G are seated and all of them are facing north. In row-2, P, Q, R, S, T and U are seated and all of them are facing south. Therefore in the given seating arrangement, each member seated in a row faces another member of the other row. R sits fourth to the right of T. Only one person sits between F and the person facing R. Only two people

    sit between F and G. As many people sit to the right of G as to the left of P. Only one person sits between P and U. U faces an immediate neighbour of B. E sits second to the left of C. As many people sit between E and F as between T and Q. 235. Who amongst the following faces D? (1) The one who sits to the immediate left of Q (2) P (3) U (4) S (5) The one who sits second to the right of R 236. How many people sit between S and the one who faces B? (1) Three (2) More than three (3) One (4) Two (4) None 237. Which of the following statements is true as per the given arrangement? (1) No one sits between R and S (2) Only three people sit between D and B (3) G faces an immediate neighbour of U (4) C is an immediate neighbuor of B (5) None of the given statements is true 238. Four of the following five are alike in a certain way based on the given arrangement and so they form a group. Which is the one that does not belong to that group? (1) D (2) P (3) G (4) F (5) T 239. Who amongst the following sits second to the right of G? (1) The one who faces Q (2) B (3) D (4) The one who faces R (5) E Directions (240–244) : Study the following innformation carefully and answer the questions given below : (SBI PO Prelim Exam, 09.06.2019)

    Ten boxes – A, B, C, D, E, P, Q, R, S and T are kept one above the other in a stack. Only four boxes are kept between box A and box P. Box B is kept immediately below box P. Only three boxes are kept between box B and box R. Only five boxes are kept between box R and box D. Only four

    BPRE–351

    boxes are kept between box D and box S. Only three boxes are kept between box S and box T. Box T is kept at one of the positions below box S. Only one box is kept between box T and box E. Box Q is kept at one of the positions above box C but below box A. 240. Four of the following five are alike in a certain way based on their positions in the given arrangement and thus they form a group. Identify the one which does not belong to the group. (1) R–B (2) A–C (3) C–D (4) D–Q (5) P–E 241. As per the given arrangement, how many boxes are kept between box A and box Q? (1) None (2) More than three (3) One (4) Two (5) Three 242. Which of the following statements is TRUE based on the given information? (1) As many boxes are kept below the box T as above the box B. (2) Box A is kept at the bottom of the stack. (3) No box is kept between box Q and box R. (4) Only two boxes are kept above box S. (5) None of the given statements is true. 243. As per the given arrangement, which box is kept immediately below box S? (1) R (2) C (3) D (4) P (5) A 244. As per the above arrangement, as many boxes are kept above box E as below box ____ (1) R (2) P (3) S (4) Q (5) A Directions (245–249) : Study the following information carefully and answer the questions given below : (SBI PO Main Exam, 20.07.2019)

    There are eight students sitting in a row facing north as given below :

    B F D A H G E C These students are rearranged around a circular table as per the following instruction.

    RANKING/ARRANGEMENT All the students should face the centre of the circular table. ● The student to the immediate right of D should sit to the immediate left of a person who will sit to the immediate left of C. ● The student at the right end should sit to the immediate right of a person who will sit second to the left of G. ● The student third to the left of G should sit to the immediate left of a person who will sit second to the right of F. ● The student fourth to the left of H should sit to the immediate right of a person who will sit second to the left of F. ● The students exactly between H and E should sit opposite to a person who will sit exactly between D and A. ● The student to the immediate right of B should sit third to the right of a person who will sit to the immediate right of E. 245. In which of the following pairs the two persons are sitting opposite to each other? (1) A and C (2) B and H (3) F and G (4) B and D (5) D and C 246. Who among the following is sitting third to the right of E? (1) H (2) D (3) B (4) G (5) F 247. Who among the following is sitting exactly between H and F? (1) G (2) E (3) C (4) D (5) B 248. Who among the following is sitting second to the right of B? (1) D (2) C (3) F (4) G (5) A 249. Which of the following statements is correct with respect to H and G? (1) H is second to the left of G (2) H is third to the right of G (3) G is third to the right of H (4) G is to the immediate left of H (5) H is sitting opposite to G Directions (250–254) : Study the following information carefully and answer the questions given below : ●

    (SBI PO Main Exam, 20.07.2019)

    There are 13 persons sitting around two circular tables. Five persons are sitting around the small table and the remaining persons are sitting around the larger table. ● All the members are facing the centre and M is sitting to the immediate right of either A or F. ● D is sitting second to the left of J and A is sitting second to the right of K. ● H is sitting to the immediate left of B and L is sitting to the immdiate right of G. ● B is not the immediate neighbour of F and M but I is the immediate neighbour M. ● J is to the immediate right of F and E is sitting to the immediate left of A. ● F is an immediate neighbour of neither B nor K. ● G is sitting fourth to the left of A and C is sitting third to the left of K. 250. Who among the following is sitting second to the left of H? (1) M (2) F (3) C (4) D (5) K 251. Who among the following is sitting to the immediate right of G? (1) B (2) D (3) L (4) M (5) I 252. Who among the following is sitting opposite to C? (1) L (2) H (3) M (4) A (5) E 253. Who among the following is sitting second to the right of F? (1) I (2) G (3) L (4) H (5) A 254. Who among the following is sitting third to the left of M? (1) K (2) D (3) C (4) B (5) L

    RBI GRADE–B/ NABARD GRADE–A OFFICER EXAMS Directions (1-8) : Study the following information carefully and answer the questions given below : (RBI Officer Grade ‘B’ Phase-I Exam, 03.08.2014)

    BPRE–352

    Eight persons – A, B, C, D, E, F, G and H – are sitting around a circular table facing the centre. Each one of them has a different profession viz., Doctor, Lawyer, Painter, Librarian, Architect, Engineer, Teacher and Accountant, but not necessarily in the same order. A sits third to the right of F. Only one person sits between A and C. Accountant is sitting third to the right of C. Accountant is sitting to the immediate left of Engineer. B is sitting to the immediate left of H. Three persons sit between B and Architect. D is an immediate neighbour of G. D is neither an Engineer nor an Architect. Only one person sits beween Librarian and Architect. Painter is to the immediate left of Teacher. D is not a Doctor. G is neither a Librarian nor a Lawyer. Lawyer is an immediate neighbour of Architect. 1. What is the profession of D ? (1) Teacher (2) Architect (3) Painter (4) Engineer (5) Accountant 2. Who amongst the following is a Doctor ? (1) C (2) B (3) G (4) A (5) H 3. What is the position of Lawyer with respect to G ? (1) Third to the left (2) Third to the right (3) Fourth to the left (4) Fourth to the right (5) Second to the right 4. Who sit(s) exactly between the Engineer and Teacher ? (1) Architect and E (2) Doctor and Librarian (3) Painter and H (4) Accountant (5) C and Doctor 5. Which of the following is Not True with respect to the given seating arrangement ? (1) E is an immediate neighbour of Librarian (2) G is a Doctor (3) Accountant is an immediate neighbour of Teacher (4) Lawyer sits third to the left of G (5) Doctor is sitting exactly between B and A

    RANKING/ARRANGEMENT 6. Starting from A, if all the persons are made to sit in the alphabetical order in anticlockwise direction, the position of how many (excluding A) will remain unchanged? (1) Three (2) Two (3) One (4) Five (5) Four 7. Four of the following five are alike in a certain way based on the above seating arrangement and hence form a group. Which one of the following does not belong to that group? (1) AG (2) DH (3) GE (4) EF (5) BC 8. Who among the following is third to the left of Librarian? (1) Engineer (2) Teacher (3) Painter (4) Accountant (5) Architect Directions (9–10) : The following questions are based on the five threedigit numbers : (RBI Officer Grade ‘B’ Phase-I Exam. 21.11.2015)

    458 374 654 487 568 9. If in each of the numbers the positions of the first and the third digits are interchanged and then the numbers so obtained are arranged in descending order from the left to right, which number will be at the fourth position ? (1) 654 (2) 487 (3) 458 (4) 374 (5) 568 10. If in each of the numbers the positions of the first two digits are interchanged and then the numbers so obtained are ar ranged in ascending order from left to right, which number will be at the second position ? (1) 654 (2) 458 (3) 568 (4) 487 (5) 374 Directions (11–12) : Read the following information carefully and answer the questions which follow : (RBI Grade-B Officer’s Exam. 18.12.2011)

    Sudha weighs more than Bharat and Abhishek. ● Rahul weighs less than only Karan. ● Parul weighs as much as Sudha but less than Dana. ● Abhishek does not weigh the minimum. ●

    Who amongst the following is the third heaviest ? (1) Karan (2) Rahul (3) Bharat (4) Dana (5) Cannot be determined 12. Which of the following is true ? (1) Only four people are heavier than Rahul (2) Bharat weighs more than Parul (3) No one weighs less than Bharat (4) Only two people are heavier than Karan (5) All are true Directions (13–15) : Study the following information to answer the given questions. 11.

    (RBI Officer Grade ‘B’ Exam. 25.08.2013)

    V, U and T are seated in a circle facing the centre. A, B and C are also seated in the same circle but two of them are not facing the centre (facing opposite direction of the centre). V is second to the left of C. U is second to the right of A. B is third to the left of T. C is second to the right of T. A is seated next to V. 13. Which of the following is V’s position with respect to C ? (1) Second to the right (2) Third to the left (3) Fourth to the right (4) Fourth to the left (5) Cannot be determined 14. Which of the following is true regarding the seating arrangement? (1) A, B and C are seated adjacent (2) V, U and T are seated adjacent (3) There are two persons whose seating arrangement cannot be ascertained (4) Those not facing the centre are seated adjacent (5) There are only two persons seated between V and T 15. Which of the following is A’s position with respect to U ? (1) Second to the left (2) Second to the right (3) Third to the right (4) Cannot be determined (5) None of these Directions (16–20) : Study the following information carefully and answer the questions given below : (RBI Officer Grade ‘B’ Phase-I Exam. 21.11.2015)

    BPRE–353

    Eight family members S,T, U, V, W, X, Y and Z are sitting around a circular table but not necessarily in the same order. Some of them are females and some are males. All of them are related to each other in the same way or the other. Some of them are facing the centre while some are facing outside (i.e. opposite to the centre.) Only two people sit between T and W. T faces the centre. X sits second to the right of T. W is the wife of S. No females is an immediate neighbour of W. U is not an immediate neighbour of T. U is the daughter of W. Both the immediate neighbours of U face the centre. Only three people sit between S and U’s brother. X is not the brother of U. Neither S nor U’s brother is an immediate neighbour of X. Z, the wife of T, sits to the immediate left of V. Both Y and S face a direction opposite to that of U (i.e. if U faces the centre then both Y and S face outside and vice-versa). U’s husband sits second to the left of Y. T’s father sits to the immediate right of W. T sits second to the right of S’s father. Both the immediate neighbours of X are females. 16. How many people sit between T and S’s father when counted from the right of T? (1) Four (2) Three (3) None (4) One (5) Two 17. Who amongst the following sits exactly between Y and W when counted from the right of Y? (1) T (2) X (3) S (4) Z (5) U 18. Which of the following statements regarding T is definitely true? (1) X and Z are immediate neighbours of T. (2) T sits second to the left of X. (3) T is the son of S. (4) None of the given options is correct (5) V is the father of T. 19. Who amongst the following faces outside (i.e. opposite to the centre)? (1) U (2) V (3) W (4) Z (5) T

    RANKING/ARRANGEMENT 20. If it is given that Y is married to X, then whatis the position of T with respect to Y’s daughter-inlaw? (1) Third to the right (2) Second to the right (3) Immediate right (4) Second to the left (5) Third to the left Directions (21-23) : Study the following information carefully and answer the questions given below : (RBI Officer Grade ‘B’ Phase-I Exam, 03.08.2014)

    Among the six persons – P, Q, R, S, T and U – each has different weight. P is heavier than three persons. R is lighter than T. S is lighter than only Q. R is not the lightest. The second heaviest person is of 68 kg and the second lightest person is of 35 kg. 21. Which of the following represents the desending order of weights of the six persons? (1) S, Q, P, T, R, U (2) Q, S, P, U, T, R (3) Q, S, P, T, R, U (4) Q, S, P, T, U, R (5) S, Q, P, U, R, T 22. Who among the following is heavier than only U? (1) R (2) P (3) T (4) S (5) Q 23. Who among the following may weigh 67 kg? (1) Q (2) P (3) T (4) S (5) Cannot be determined Directions (24–26) : Study the following information carefully and answer the questions given below : (RBI Officer Grade ‘B’ Phase-I Exam. 21.11.2015)

    P, Q, R, S, T, U, V, W and Z are sitting around a circle facing at the centre. R is third to the right of Z who is second to the right of P. S is not an immediate neighbour of Z and R. T is third to the left of S. Q is third to the right of W who is not an immediate neighbour of S. 24. Which of the pair of persons are the immediate neighbours of P ? (1) VQ (2) VW (3) VS (4) SR (5) None of these

    25. What is Q’s position with respect to Z ? (A) Fourth to the right (B) Fourth to the left (C) Third to the right (1) Only (A) (2) Only (B) (3) Only (C) (4) Either (A) or (B) (5) None of these 26. In which of the following pairs is the second person sitting second to left of the first person ? (1) RT (2) RW (3) QR (4) PS (5) WZ Directions (27–31) : Study the following information carefully and answer the questions given below : (RBI Officer Grade ‘B’ Phase-I Exam. 21.11.2015)

    Ten persons from different companies viz. Indigo, Biocon, Nokia, Samsung, Havells, Amul, Tata, Vadilal, Flipkart and Cipla are sitting in two parallel rows containing five people each, in such a way that there is an equal distance between adjacent persons. In row-1 G, H, I, J and K are seated and all of them are south. In row-2 U, V, W, X and Y are seated and all of them are facing north. Therefore, in the given seating arrangement, each member seated in a row faces another member of the other row. (All the information given above does not necessarily represent the order of seating as in the final arrangement). K sits third to the left of the person from Biocon. V is an immediate neighbour of the person who faces K. The person from Havells sits to the immediate left of V. Only two people sit between the person from Nokia and H. The person from Nokia is not an immediate neighbour of K. Only one person sits between the one who faces H and the person from Tata. X sits second to the right of W. W does not sit at an extreme end of the line. The person from Cipla sits third to the left of the one who faces W. U is an immediate neighbour of the person from Samsung. U is not from Tata. Only two people sit between the person from Amul and J. The person from Flipkart is an immediate neighbour of the person who faces the person from Amul. Only one person sits between the one who faces the person from Flipkart and I. Neither I nor U is from Indigo.

    BPRE–354

    27. K is related to the person from Vadilal in the same way as W is related to person from Tata based on the given arrangement. To who amongst the following is, X related to, following the same pattern? (1) The person from Amul (2) The person from Indigo (3) The person from Nokia (4) The person from Havells (5) The person from Flipkart 28. Y is from which of the following companies? (1) Indigo (2) Tata (3) Amul (4) Samsung (5) Biocon 29. Who amongst the following faces the person from Vadilal? (1) The person from Flipkart ? (2) V (3) Y (4) The person from Nokia (5) The person from Havells 30. Which of the following is true regarding G? (1) Only one person sits between G and the person from Cipla. (2) The person from Samsung faces G. (3) G is an immediate neighbour of the person from Biocon. (4) None of the given options is true (5) G sits to the immediate left of L. 31. Who amongst the following sit at extreme and sof the rows? (1) The persons from Indigo and G (2) The person from Cipla and X (3) K, Y (4) J and the person from Flipkart (5) The person from Nokia and U Directions (32–37) : Study the following information carefully and answer the questions given below : (Nabard Officer Grade ‘A’ Online Exam. 01.03.2015)

    Eight persons – A, B, C, D, E, F, G and H — are sitting around a circular table facing the centre but not necessarily in the same order. Each one of them has a different profession viz. doctor, engineer, architect, teacher, clerk, shopkeeper, businessman and banker. A sits third to right of teacher. D sits second to left of G. G is not an immediate neighbour of teacher. Only one person sits between B, who is

    RANKING/ARRANGEMENT shopkeeper, and teacher. The one who is an architect sits third to right of the shopkeeper. H sits between architect and engineer. E is not an immediate neighbour of H. Engineer sits third to the right of clerk. Only one person sits between businessman and F. E is neither a businessman nor a doctor. 32. Which of the following statements is true with respect to the given sitting arrangement? (1) E is an immediate neighbour of the engineer (2) E is an architect (3) The clerk is an immediate neighbour of the banker (4) The teacher sits between H and the engineer (5) Shopkeeper sits second to the right of the teacher 33. What is the profession of H ? (1) businessman (2) architect (3) shopkeeper (4) banker (5) teacher 34. What is the position of doctor with respect to the banker ? (1) immediately to the left (2) third to the left (3) second to the left (4) fourth to the left (5) second to the right 35. Who sits exactly between the architect and the businessman ? (1) C and H (2) Clerk (3) Banker and Shopkeeper (4) Doctor (5) C and Teacher 36. Who amongst the following is the clerk ? (1) G (2) D (3) E (4) F (5) C 37. Who sits immediately right to the businessman ? (1) teacher (2) doctor (3) clerk (4) banker (5) shopkeeper Directions (38–42) : Study the following information carefully to answer the questions given below : (NABARD Assistant Manager Exam, 15.15.2016)

    Eight friends – P, Q, R, S, T, U, V and W — are seated in a straight line, but not necessarily in the same order. Some of them are facing north while some face south. ● S sits at one of the extreme ends of the line. V sits second to the right of S.

    Only two persons sit between P and Q. ● Both the immediate neighbours of V face South. ● R sits third to the left of T. T is an immediate neighbour of Q. ● Immediate neighbours of R face opposite directions (i.e., if one neighbour faces north then the other neighbour faces south and vice–versa.) ● U faces north. ● Only two persons sit between R and W. R faces a direction opposite to W (i.e., if W faces north then R faces south and vice–versa.) ● Both the immediate neighbours of W face south. ● Q faces a direction opposite to W. (i.e., if W faces north then Q faces south and vice–versa.) 38. What is the position of T with respect to U? (1) Second to the right (2) Second to the left (3) Third to the right (4) Fourth to the left (5) Fourth to the right 39. Who amongst the following sit exactly between R and T? (1) P, V (2) S, P (3) Q, S (4) S, V (5) Q, U 40. Who amongst the following, sitting at the extreme end of the line, faces north? (1) No one (2) U (3) T (4) S (5) P 41. In which of the following pairs are both the persons facing north? (1) T, S (2) U, W (3) W, S (4) Q, R (5) R, P 42. Based on the given arrangement, which of the following is true with respect to P? (1) Only one person sits between P and T. (2) None of the given options is true (3) R sits to immediate right of P. (4) No one sits to the left of P. (5) Q is one of the immediate neighbours of P. ●

    BPRE–355

    Directions (43–47) : Study the following information carefully and answer the questions given below : (NABARD Assistant Manager Exam, 15.05.2016)

    Eight persons — P, Q, R, S, T, U, V and W — are sitting around a circular table facing the centre but not necessarily in the same order. Each one of them works in a different company viz. Microsoft, Sony, Dell, P&G, HTC, ONGC, Google and Amazon but not necessarily in the same order. The one who works in Microsoft sits third to the right of V. Only one person sits between the one who works in Microsoft and U. Only three persons sit between T and U. R sits second to the left of U. Both R and T are immediate neighbours of the one who works in P & G. Only three persons sit between the one who works in P & G and S. S is one of the immediate neighbours of the one who works in Dell. The one who works in ONGC sits second to the left of the one who works in Dell. W sits third to the left of the one who works in ONGC. P is one of the immediate neighbours of W. The one who works in Google sits to the immediate left of the one who works in HTC. Only three persons sit between the one who works in Google and the one who works in Sony. 43. V is related to Google in a certain way based on the given arrangement. Following the same pattern, Q is related to Amazon. To which of the following companies is R related to following the same pattern? (1) Microsoft (2) HTC (3) ONGC (4) Dell (5) P & G 44. Q works in which of the following companies? (1) HTC (2) P & G (3) Google (4) Sony (5) Dell 45. What is the position of P with respect to Q? (1) Second to the left (2) Fourth to the left (3) Third to the left (4) Second to the right (5) Third to the right

    RANKING/ARRANGEMENT 46. Which of the following statements is true with respect to the given arrangement? (1) T sits second to right of U. (2) Q is an immediate neighbour of the one who works in Google. (3) P and the one who works in Dell are immediate neighbours of each other. (4) S works in ONGC (5) R sits second to the right of the one who works in HTC. 47. Who amongst the following works in Amazon? (1) R (2) Other than those given as options (3) T (4) S (5) P Directions (48–52) : Study the following information carefully and answer the questions given below : (RBI Officer Grade ‘B’ Phase-I Exam, 04.09.2016 (Shift-I))

    Ten persons from different cities viz. Delhi, Mumbai, Patna, Agra, Surat, Chennai, Kanpur, Bangalore, Ranchi and Lucknow are sitting in two parallel rows containing five people each, in such a way that there is an equal distance between adjacent persons. In row-1 P, Q, R, S and T are seated and all of them are facing south. In row-2 A, B, C, D and E are seated and all of them are facing north. Therefore, in a given seating arrangement, each member seated in a row faces another member of the other row. (All the information given above does not necessarily mean represent the order of seating as in the final arrangement) ● The person from Surat sits second to left of R. Neither R nor P sit at an extreme end of the line. Only two persons sit between the one who faces the person from Surat and D. The person from Delhi sits second to the right of B. ● The person from Ranchi is an immediate neighbour of the one who faces the person from Delhi. C sits second to the left of E. C is neither from Delhi nor sits at an extreme end of the line. ● P is an immediate neighbour of the person from Mumbai. The person from Mumbai does not face the person from Delhi. Only two persons sit between the one who faces P and the person from Agra.

    ●B

    faces the person from Bangalore. More than two people sit between Q and S. Only two persons sit between the one who faces S and the person from Patna. ● The person from Chennai sits second to the right of the person from Lucknow. The person from Lucknow does not face P. 48. Four of the following five are alike in a certain way based on the given arrangement and so form a group. Which is the one that does not belong to that group? (1) T (2) A (3) B (4) P (5) S 49. Who amongst the following sit at extreme ends of the line? (1) The persons from Mumbai and Agra. (2) The persons from Lucknow and D. (3) Q and A (4) S and the person from Kanpur. (5) The person from Delhi and Q. 50. Which of the following statements is true regarding T? (1) T is an immediate neighbour of the person from Surat. (2) T is from Ranchi. (3) The person from Delhi faces T. (4) T sits at an extreme ends of the line. (5) None of the given options is true. 51. B is related to Patna in the same way as S is related to Ranchi based on the given arrangement. To whom amongst the following is C related to, following the same pattern? (1) Mumbai (2) Lucknow (3) Pune (4) Agra (5) Surat 52. Who amongst the following faces the person from Patna? (1) The person from Chennai (2) T (3) The person from Bangalore (4) The person from Surat (5) E Directions (53–58) : Study the following information carefully and answer the questions given below : (RBI Officer Grade ‘B’ Phase-I Exam 17.06.2017)

    BPRE–356

    Twelve people are sitting in two parallel rows containing six people each, in such a way that there is an equal distance between adjacent persons. In row–1 S, T, U, V, W and X are seated and all of them are facing south. In row–2 M, N, O, P, Q and R are seated and all of them are facing north. Therefore in the given sitting arrangement, each member seated in a row faces another member of the other row. No two people with names starting with consecutive alphabet is an immediate neighbour of each other. For example, A is not an immediate neighbour of B. B is not an immediate neighbour of either A or C and so on. Each of them also likes a different movies viz. Vertigo, Cinderella, Twilight, Gladiator, Uninhabited, Inception, Frozen, Watchmen, Tangled, Aladdin, Hero and Wanted. (Please Note : None of the information given is necessarily in the same order.) T sits at an extreme end of the line. Only two people sit between T and the one who likes Frozen. N sits second to the left of the one who faces the one who likes Frozen. The one who likes Twilight sits second to the right of O. T does not face the one who likes Twilight. Only two persons sit between P and R. U does not sit at an extreme end of the line. X is not an immediate neighbour of T. V faces the one who likes Tangled. Only three people sit between the ones who like Tangled and Vertigo. R likes Inception. Only one person sits between R and M. One of the immediate neighbours of M faces the one who likes Hero. The one who likes Cinderella sits to the immediate left of S. P sits second to the right of the one who likes Aladdin. Only one person sits between the ones who like Watchmen and Wanted respectively. T does not like Wanted. N does not like Gladiator. 53. Which of the following statements is true as per the given arrangement? (1) U likes Watchmen, (2) Q faces one of the immediate neighbours of W. (3) P is an immediate neighbour of N. (4) None of the given statements is true. (5) S sits exactly between V and X

    RANKING/ARRANGEMENT 54. Who amongst the following likes Uninhabited? (1) P (2) W (3) S (4) X (5) N 55. Who amongst the following is facing Q? (1) X (2) V (3) The one who likes Watchmen (4) S (5) The one who likes Frozen 56. Four of the following five are alike in a certain way based on the given arrangement and hence they form a group. Which one of them does not belong to that group? (1) The one who likes Tangled (2) N (3) The one who likes Vertigo (4) The one who likes Gladiator (5) V 57. W is related to Cinderella and P is related to Inception in a certain way based on the given arrangement. To which of the following is U related, following the same pattern? (1) The one who faces P (2) The one who faces O (3) T (4) S (5) The one who likes Uninhabited 58. What is the position of Q with respect to the one who likes Inception? (1) Third to the right (2) Second to the right (3) Third to the left (4) Second to the left (5) Fourth to the left Directions (59–62) : Study the following information carefully to answer the given questions : (NABARD Assistant Manager Grade ‘A’ Online Exam 05.08.2017)

    Eight persons — S, T, U, V, W, X, Y and Z — are seated in a straight line but not necessarily in the same order. Some of them are facing south while some are facing North. S sits fourth to left of X. X sits at one of the extreme end of the line. Both the immediate neighbours of S face south. T sits second to left of Z. Z is not an immediate neighbour of S. Neither Z nor U sits at the extreme end of the line. Both the immediate neighbours of U face north. W sits to immediate left of Y. Immediate neighbours of V

    face opposite directions (i.e., if one neighbour of V faces north then the other faces south and vice-versa). Immediate neighbours of T face opposite directions (i.e., if one neighbour of T faces north then the other faces south and vice-versa). People sitting at the extreme ends face the same direction (i.e., if one person faces North then the other also faces north and viceversa). 59. Which of the following pairs represents immediate neighbours of the persons seated at the two extreme ends of the line? (1) Y, Z (2) T, Y (3) W, T (4) U, Z (5) S, T 60. How many persons are seated between T and X ? (1) Four (2) More than four (3) One (4) Three (5) Two 61. If each of the persons is made to sit in alphabetical order from right to left the positions of how many will remain unchanged as compared to the original seating arrangement? (1) None (2) Two (3) Four (4) One (5) Three 62. Who among the following sits exactly between Z and T ? (1) U (2) V (3) Y (4) X (5) W Directions (63–66) : Study the folloiwng information carefully to answer the given questions : (NABARD Assistant Manager Grade ‘A’ Online Exam 05.08.2017)

    Eight family members S, T, U, V, W, X, Y and Z are sitting around a circular table but not necessarily in the same order. Some of them are females and some are males. All of them are related to each other in the some way or the other. Some of them are facing the centre while some are facing outside (i.e., opposite to the centre). Only two people sit between T and W. T faces the centre. X sits second to the right of T. W is the wife of S. No female is an immediate neighbour of W. U is not an immediate neighbour of T. U is the daughter of W. Both the immediate neighbours of U face the centre.

    BPRE–357

    Only three people sit between S and U’s brother. X is not the brother of U. Neither S nor U’s brother is an immediate neighbour of X. Z, the wife of T, sits to the immediate left of V. Both Y and S face a direction opposite to that of U (i.e. if U faces the centre then both Y and S face outside and vice-versa). U’s husband sits second to the left of Y. T’s father sits to the immediate right of W. T sits second to the right of S’s father. Both the immediate neighbours of X are females. 63. How many people sit between T and S’s father when counted from the right of T? (1) Four (2) Three (3) None (4) One (5) Two 64. Who amongst the following sits exactly between Y and W when counted from the left of W? (1) T (2) X (3) S (4) Z (5) U 65. Which of the following statements regarding T is definitely true? (1) X and Z are immediate neighbours of T (2) T sits second to the left of X. (3) T is the son of S (4) None of the given options is correct (5) V is the father of T. 66. Who amongst the following faces outside (i.e. opposite to the centre ? (1) U (2) V (3) W (4) Z (5) T Directions (67-70) : Study the following information carefully and answer the questions given below. (NABARD Assistant Manager Online Exam 06.08.2017)

    Eight people — A, B, C, D, E, F, G and H — are sitting around a circular table facing the centre. Each of them live in a different city namely, Mumbai, Pune, Lucknow, Chennai, Surat, Kochi, Jaipur and Aurangabad, but not necessarily in the same order. The one who lives in Mumbai sits second to the right of H. A is an immediate neighbour of the one who lives in Mumbai. Only three people sit between A and the one who lives in Chennai. Only two people sit (from either right or left) between the one who lives in Chennai and the one who

    RANKING/ARRANGEMENT lives in Lucknow. F is an immediate neighbour of the one who lives in Lucknow. As many people sit between F and the one who lives in Aurangabad as between F and the one who lives in Chennai. D sits second to the left of the one who lives in Aurangabad. C sits to the immediate left of B. The one who lives in Surat sits second to the right of C. D neither lives in Pune nor Kochi. F does not live in Pune. E does not live in Lucknow. 67. Who is sitting second to the right of the one who lives in Kochi? (1) D (2) The one who lives in Pune (3) A (4) B (5) The one who lives in Surat 68. Which of the following statements is true according to the given arrangement? (1) The one who lives in Mumbai is not an immediate neighbour of H. (2) All of the given statements are true. (3) Only two people sit between C and A when counted from the left of C. (4) The one who lives in Aurangabad sits second to the right of one who lives in Jaipur. (5) B lives in Chennai. 69. Four of the following five are alike in a certain way based on the given arrangement and hence they form a group. Which of the following does not belong to that group? (1) G-Surat (2) C-Mumbai (3) H-Pune (4) B-Aurangabad (5) F-Jaipur 70. Who amongst the following lives in Mumbai? (1) E (2) F (3) D (4) G (5) C Directions (71-75) : Study the following information carefully and answer the questions given below : (NABARD Assistant Manager Online Exam 06.08.2017)

    Ten people are sitting in two parallel rows containing five people each in such a way that each person seated in a row faces another person of the other row. In row 1- A, B, C, D and E are seated and all of them are facing north. In row 2– J, K, L, M and N are seated and all of them are facing south. All of them are from ten different

    countries viz. Holland, Argentina, Zaire, Poland, Germany, Venezuela, Russia, Brazil, France and Italy. (Note : None of the given information is necessarily in the same order.) A sits third form the left end of the row. Only one person sits between A and B. Only two people sit between B and the one from Russia. The one who faces the one from Russia sits to the immediate left of K. Only one person sits between K and the one from Argentina. The one from Argentina sits at one of the positions to the left of K. D sits to the immediate left of the one who faces the one from Argentina. The one from Holland sits to the immediate right of the one from Argentina. Only two people sit between the one from Holland and N. The one from Germany is an immediate neighbour of the one who faces N. C is not from Germany. The one from Venezuela faces E. Only three people sit between the ones from Poland and France. Neither B nor C is from either Poland or France. The one from France sits at one of the positions to the left of the one from Poland. J sits to the immediate right of L. J is not an immediate neighbour of the one from Poland. More than one person sits between the one from Italy and Brazil. The one from Brazil sits at one of the positions to the right of the one from Italy. 71. As per the given arrangement, M is related to Holland and E is related to Germany foll-owing a certain pattern. To which amongst the following is B related to following the same pattern? (1) France (2) A r g e n t i n a (3) Poland (4) Zaire (5) Brazil 72. Which of the following statements is true as per the given arrangement? (1) A is from Zaire. (2) None of the given statements is true. (3) E faces an immediate neighbour of M. (4) Only two people sit between K and M. (5) E faces the one from Russia. 73. Who sits second to the right of L? (1) K (2) N (3) The one who faces A (4) The one who faces D (5) The one from France

    BPRE–358

    74. Four of the following five are alike in a certain way based on the given arrangement and thus form a group. Which one of the following does not belong to the group? (1) Holland (2)Venezuela (3) Russia (4) Poland (5) Argentina 75. Who is from Italy? (1) N (2) C (3) B (4) L (5) E Directions (76–80) : Study the following information carefully and answer the questions given below : (RBI Assistant Manager Online Exam 25.03.2017)

    Ten persons are sitting in two parallel rows containing five persons each in such a way that there is equal distance between adjacent persons. In row–1 A, B, C, D and E are seated (not necessarily in the same order) and all of them are facing north. In row–2 P, Q, R, S and T are seated (not necessarily in the same order) and all of them are facing south. Therefore in the given seating arrangement each member seated in a row faces another member of the other row. Each one of them also likes different perfumes – Davidoff, Cheque, Brut, Gucci, Hollister, Titan, Police, Valentino, Yardley and Ferrari, but not necessarily in the same order. S sits at one of the extreme ends of the row. A faces an immediate neighbour of S. Only one person sits between A and the one who likes Brut. T sits third to the right of P. P faces the one who likes Davidoff. C is an immediate neighbour of A. C faces Q. The one who sits second to the right of Q likes Yardley. Only two persons sit between the one who likes Yardley and the one who likes Valentino. B sits to the immediate right of E. D faces one of the immediate neighbours of the one who likes Gucci. The one who likes Police is an immediate neighbour of T. The one who likes Ferrari sits second to the right of the one who likes Hollister. P does not like Cheque. 76. As per the given arrangement, which of the following statements is true? (1) T is an immediate neighbour of the one who likes Brut. (2) None of the given statements is true. (3) Only one person sits between A and the one who faces T. (4) B faces the one who likes Titan. (5) A sits second to the right of E.

    RANKING/ARRANGEMENT 77. As per the given arrangement, A is related to D in the same way as T is related to Q. Following the same pattern, to which is S related? (1) The one who likes Cheque. (2) P (3) The one who faces A. (4) R (5) The one who likes Yardley. 78. Who amongst the following faces E? (1) Q (2) R (3) S (4) T (5) Other than those given as options 79. Who sits exactly between Q and the one who faces D? (1) T (2) S (3) R (4) The one who likes Police. (5) The one who likes Titan. 80. Who is second to the left of C? (1) The one who faces Q. (2) B (3) A (4) The one who likes Ferrari. (5) The one who likes Hollister. Directions (81–85) : Study the following information carefully and answer the questions given below : (RBI Assistant Manager Online Exam 25.03.2017)

    Eight persons, A, B, C, D, E, F, G and H are sitting around a circular table, facing the centre, with equal distance between each other. Each one of them bought a different product viz. laptop, phone, camera, refrigerator, printer, speakers, TV and AC but not necessarily in the same order. ● E sits second to the left of the one who bought a TV. Only two persons sit between E and H. ● The one who bought a phone sits to the immediate right of G. G is neither an immediate neighbour of H nor E. G do not buy a TV. Neither A nor the one who bought a phone is an immediate neighbour of E. ● Only three people sit between A and the one who bought a camera. A did not buy a phone. ● The one who bought a printer sits third to the right of the one who bought a laptop. Only three people sit between the one who bought a laptop and an AC. ● C is an immediate neighbour of the one who bought an AC. ● B sits second to the left of F. B did not buy a refrigerator.

    81. Which of the following products did C buy? (1) Phone (2) Laptop (3) Printer (4) Speakers (5) Camera 82. Which of the following statements is true as per the given arrangement? (1) Both A and B are immediate neighbours of H. (2) A bought a laptop. (3) The one who bought a refrigerator is an immediate neighbour of B. (4) None of the given statements is true. (5) C sits to immediate right of G. 83. Who among the following bought a refrigerator? (1) F (2) H (3) D (4) A (5) E 84. Four of the following five are alike in a certain way based on the given arrangement and hence they form a group. Which one of the following does not belong to that group? (1) A–AC (2) E–Laptop (3) D–Refrigerator (4) C–Camera (5) H–Printer 85. Who sits exactly between E and the one who bought a TV, when counted from the right of E? (1) D (2) G (3) F (4) The one who bought an AC. (5) The one who bought speakers. Directions (86–90) : Study the following information carefully and answer the questions given below : Eight people – A, B, C, D, E, F, G and H – are sitting around a square table in such a way that four of them sit at corners while four sit in the middle of each of the four sides. The ones sitting in the middle of the sides are facing the centre and the ones sitting at the corners are facing outside (i.e. opposite to the centre). Each one of them also likes a different colour viz. Red, Black, Green, Yellow, Purple, Orange, White and Blue. (Please note that none of the information given is necessarily in the same order). E sits at one of the corners of the table. Only two people sit between E and the one who likes Purple (either

    BPRE–359

    from left or right). The one who likes White sits to the immediate right of B. B faces the centre. B is not an immediate neighbour of E. B does not like Purple. Only three people sit between the ones who like White and Black. H sits second to the left of the one who likes Black. Both H and the one who likes Yellow face the same direction (if H faces the centre then the one who likes yellow also faces the centre and vice-versa). F sits third to the right of the one who likes Blue. Neither H nor B likes Blue. D is one of the immediate neighbours of the one who likes Blue. A likes Green. C sits to the immediate right of the one who likes Red. (RBI Officers in Grade ‘B’ Phase-I Exam. 16.08.2018)

    86. Who sits second to the right of G? (1) F (2) The one who likes Orange (3) E (4) The one who likes Green (5) H 87. Who amongst the following likes Orange colour? (1) H (2) B (3) D (4) G (5) E 88. Which of the following statements is true with respect to the given arrangement? (1) None of the given options is true (2) A is an immediate neighbour of H (3) B sits second to the left of C (4) Only three people sits between C and E (5) G sits in the middle of one of the sides of the table. 89. Four of the following five are alike in a certain way based on their positions in the given arrangement and thus they form a group. Which one of the following does not belong to that group? (1) HG (2) BC (3) ED (4) AC (5) FH 90. What is the position of E with respect to the one who likes Black? (1) Third to the left (2) Third to the right (3) Second to the right (4) Immediate left (5) Immediate right

    RANKING/ARRANGEMENT Directions (91–95) : Study the following information carefully and answer the questions given below : Seven people – P, Q, R, S, T, U and V – sit in a straight line facing north (not necessarily in the same order). Each one supports a different team in the cricket world cup viz., India, New Zealand, England, South Africa, Australia, West Indies and Sri Lanka (not necessarily in the same order). The one who supports India sits third from the left end of the line. Only one person sits between the one who supports India and U. Only two people sit between U and P. P supports Sri Lanka. P and Q are immediate neighbours of R. R does not support India. V supports West Indies and sits at one of the extreme ends of the line. Only three people sit between V and S. The one who supports South Africa sits third to the right of T. The one who supports England is an immediate neighbour of S. The one who supports New Zealand sits at one of the positions to the left of the one who supports Australia. (RBI Officers in Grade ‘B’ Phase-I Exam. 16.08.2018)

    91. As per the given arrangement, who amongst the following sits at one of the extreme ends of the line? (1) U (2) R (3) Q (4) S (5) T 92. Which of the following statements is true with respect to the given arrangement? (1) None of the given statements is true (2) Only three people sit between T and the one who supports Australia. (3) S supports Australia. (4) P and R are immediate neighbours of T. (5) U and V are immediate neighbours of each other. 93. As per the given arrangement, V is related to R in the same way as Q is related to P. As per the given pattern, who is S related to? (1) The one who supports South Africa (2) R (3) T (4) U (5) The one who supports India

    94. As per the given arrangement, who sits second to the right of T? (1) V (2) Q (3) U (4) The one who supports West Indies (5) The one who supports Sri Lanka 95. As per the given arrangement, which team does T support? (1) None of the given options (2) New Zealand (3) England (4) Australia (5) India Directions (96–100) : Study the following information carefully and answer the questions given below : Seven people — D, E, F, G, X, Y and Z — are sitting around a circular table facing the centre but not necessarily in the same order. Each one of them is related to each other in the some way or the other. F sits second to the right of G’s brother. G’s mother is an immediate neighbour of F. G sits second to the right of his mother. G sits to the immediate left of Y. G’s son sits third to the right of Y. No one sits between G’s son and G’s father in law. X is the wife of D. X sits to the immediate right of E. Y is the only daughter of D. Only one of the children of E is married. G’s wife has no siblings. (SEBI Assistant Manager Exam. 17.11.2018)

    96. As per the given arrangement, F is the _________ of the one who sits to the immediate left of D (1) Daughter (2) Son (3) Husband (4) Niece (5) Nephew 97. How is X related to Y? (1) Aunt (2) Grandmother (3) Cannot be determined (4) Mother (5) Mother-in-law 98. Which of the following statements is true with respect to the passage? (1) E sits fourth to the right of Y. (2) Z is the son of E. (3) All the given statements are true.

    BPRE–360

    (4) G’s brother sits to the immediate left of Y’s father. (5) G sits third to the left of D. 99. How many people sit between G’s wife and F when counted from the left of F? (1) Two (2) Three (3) More than three (4) One (5) None 100. Who sits second to the right of Z? (1) D (2) E’s grandson (3) Y (4) G’s wife (5) G Directions (101–105) : Study the following information carefully and answer the questions given below : Ten people are sitting in a straight line with equal distance between adjacent persons. Some of them are facing north and some are facing south. Q sits fourth from one of the extreme ends of the line. V sits fifth to the left of Q. R sits second to the left of V. R faces the same direction as V. Immediate neighbours of R face a direction opposite to that of R. More than four people sit between R and S. M sits fourth to the right of S. T sits fifth to the right of M. People sitting at the extreme ends of the line face opposite directions. W faces a direction opposite to that of T. N sits to the immediate left of W and faces north. P sits fourth to the left of N. Y sits at one of the positions to the left of P. (SEBI Assistant Manager Exam. 17.11.2018)

    101. How many people sit between T and Y? (1) Five (2) Two (3) Six (4) One (5) Four 102. Which of the following statements is true as per the given arrangement? (1) Q faces south (2) M sits to the immediate right of P. (3) N sits fourth to the left of V. (4) More than two people sit between M and W. (5) R is an immediate neighbour of Q.

    RANKING/ARRANGEMENT 103. Four of the following five are alike in a certain way as per the given arrangement and thus they form a group. Which one of the following does not belong to that group? (1) V (2) M (3) W (4) R (5) P 104. Who amongst the following sits to the immediate left of P? (1) R (2) Y (3) T (4) M (5) W 105. Which of the following pairs represents the immediate neighbours of the people sitting at the extreme ends of the line? (1) N, Y (2) V, P (3) V, N (4) Y, P (5) Q, P Directions (106–109) : Study the following information carefully and answer the questions given below : (RBI Grade B Officer Exam, 09.11.2019)

    A certain number of people are sitting in a straight line facing north. H sits second to the right of the 13 year old. Only five people sit between H and K. The 25 year old sits third to the left of K. Only four people sit between the 25 year old and the 10 year old. B sits second to the right of the 10 year old. B sits at one of the extreme ends of the line. As many people sit between H and B as between the ones who are 13 and 42 year old. As many people sit to the right of K as to the left of the 42 year old. M sits fifth from one of the extreme ends of the line. G sits sixth to the right of M. 106. How many people sit between G and H? (1) Six (2) Seven (3) Four (4) Eleven (5) Five 107. How many people are sitting in the given line? (1) 25 (2) 27 (3) 17 (4) 29 (5) 20 108. Who amongst the following is the 42 years old? (1) H (2) B (3) Cannot be determined (4) M (5) G

    109. Only two people sit between K and the 30 years old, what is the 30 year old’s position with respect to B? (1) Immediate left (2) Seventh to the left (3) Fourth to the right (4) Second to the left (5) Third to the left Directions (110–114) : Study the following information carefully and answer the questions given below : (RBI Grade B Officer Exam, 09.11.2019)

    Twelve people are sitting in twoparallel rows containing six people each in such a way that there is equal distance between adjacent persons. In row–1 – A, B, C, D, E and F are seated and all of them are facing north. In row–2 – P, Q, R, S, T and U are seated and all of them are facing south. Thus, in the given arrangement each member seated in a row faces another member of the other row. Each person likes a different country. P sits second from one of the extreme ends of the row. The one who likes France sits to the immediate right of P. The one who faces the one who likes France sits second to the left of B. Only two people sit between B and the one who likes Spain. The one who likes Spain faces an immediate neighbour of the one who likes Argentina. The one who likes Argentina faces C. The one who likes Malaysia sits to the immediate left of C. Only two people sit between the one who likes Malaysia and the one who likes Indonesia. An immediate neighbour of the one who likes Indonesia faces R. The one who likes Denmark sits at one of the positions to the right of R. A faces the one who likes Denmark. D sits at one of the positions to the right of E but to the left of F. The one who faces F is an immediate neighbour of the one who likes Switzerland. Only two people sit between the one who likes Switzerland and the one who likes Italy. The one who faces the one who likes Norway sits third to the right of the one who likes Ireland. The one who likes Japan sits at one of the positions to the left of the one who likes Germany. Q does not sit at any of the extreme ends of the row. Q does not face D. S sits at one of the positions to the right of Q. T sits at one of the positions to the left of U.

    BPRE–361

    110. Four of the following five are alike in a certain way based on their positions as per the given arrangement and thus they form a group. Which one of the following does not belong to that group? (1) The one who likes Germany (2) U (3) A (4) T (5) The one who likes Denmark 111. Who amongst the following sits second to the right of U? (1) S (2) The one who likes France (3) The one who likes Italy (4) P (5) The one who likes Argentina 112. Which of the following statements is true as per the given arrangement? (1) E sits to the immediate left of F. (2) R is an immediate neighbour of T. (3) Q faces F. (4) None of the given statements is true (5) R sits at one of the extreme ends of the row. 113. Who amongst the following faces S? (1) The one who likes Malaysia (2) The one who likes Indonesia (3) B (4) C (5) E 114. How many people sit to the left of F? ` (1) None (2) Three (3) One (4) More than three (5) Two Directions (115–119) : Study the following information carefully and answer the questions given below : (RBI Officer Grade 'B' Phase-I Exam, 06.03.2021)

    Twelve persons – P, Q, R, S, T, U, V, W, X, Y, Z and A- are sitting around a square table facing the centre at equal distance, but not necessarily in the same order.

    RANKING/ARRANGEMENT S is sitting second to the right of X. ● W is sitting to the immediate left of U. ● A is sitting exactly between X and Y. ● R is sitting just opposite to Q. ● W is sitting fifth to the left of Y. ● W is sitting just opposite to Y, who is to the immediate left of A. ● V is sitting fourth to the right of X. ● Q is sitting fourth to the right of T and just opposite to R, who is sitting to the immediate left of S. ● P is an immediate neighbour of either A or U. 115. Which of the following pair is sitting just opposite to each other ? (1) T and S (2) P and R (3) T and Z (4) R and V (5) U and A 116. The immediate neighbours of S are (1) R and Z (2) A and T (3) P and X (4) V and R (4) Q and W 117. Which of the following statements is correct as per the given arrangement ? (1) S is sitting fourth to the left of Y. (2) V is sitting to the immediate left of W. (3) Q is sitting third to the right of A. (4) X is sitting second to the right of A (5) U is sitting fifth to the left of Z. 118. Four of the following five are alike in a certain way and hence they form a group. Which one does not belong to that group ? (1) A (2) S (3) P (4) V (5) U 119. Who among the following is sitting just opposite to Z ? (1) U (2) X (3) R (4) T (5) A Directions (120–124) : Study the following information carefully and answer the questions given below : ●

    (RBI Officer Grade 'B' Phase-I Exam, 06.03.2021)

    Twelve letters/numbers – B, 4, E, L, 15, P, R, 19, T, U, 23 and 25 – are placed in a matrix consisting of four rows and three columns. The bottom row is numbered 1, the one just above it is numbered 2 and so on till the topmost is numbered 4. Similarly, the column at the left end is numbered 1 and that at the right end is numbered 3. ● R is just above T. 19 is immediate left above diagonally of P which is immediate right above diagonally of 25. ● U is immediate right above diagonally of the cell which is immediate left above diagonally of T. ● 23 is immediate left above diagonally of the cell which is to the immediate right of 25. L is immediate right above diagonally of the cell which is second to the left of R. ● 4 is two rows above the cell in the same column which is immediate left below diagonally of B. T is immediate right above diagonally of 15. ● P is immediate left above diagonally of the cell which is in the Column-3. ● E is second to the right of the cell which is immediate left below diagonally of P. 25 is at the bottom of Column-1. 120. In which of the following Rows/ Columns there are letters only ? (1) Row-1 (2) Row-3 (3) Column-3 (4) Column-2 (5) Row-4 121. Which of the following letters/ numbers is second to the right of 19 ? (1) L (2) B (3) E (4) 15 (5) R 122. Four of the following five are alike in a certain way and hence they form a group. Which one does not belong to that group ? (1) T (2) 15 (3) P (4) L (5) B 123. Which of the following letters/ numbers is placed exactly between B and 15 ? (1) T (2) 23 (3) P (4) 19 (5) L

    BPRE–362

    124. How many meaningful words can be formed with the letters of the Column-3, using each letter only once in each word ? (1) None (2) One (3) Two (4) Three (5) Four Directions (125–129) : Study the following information carefully and answer the questions given below : (RBI Officer Grade 'B' Phase-I Exam, 06.03.2021)

    Twelve persons – A to L – live in different flats of an apartment having three floors. There are four flats on each floor. The bottom floor is numbered as Row-1 and the top floor is numbered as Row-3. The flats at the left end are in Column-1 and that of at the right end are in Column-4. ● E lives immediate right below diagonally of the one who lives immediate right above diagonally of H. ● G is to the immediate right of the one who lives two flats below the flat of I in the same column. ● D is immediate left below diagonally of the one who lives immediate right below diagonally of B. ● C lives second to the right of the one who lives immediate right above diagonally of F. ● A lives immediate right above diagonally of the one who lives immediate right below diagonally of F. ● B lives immediate left above diagonally of the one who lives immediate left below diagonally of I. ● J lives immediate left below diagonally of the one who lives third to the right of F. ● L lives immediate left below diagonally of the one who lives immediate right below diagonally of K. 125. Four of the following five are alike in a certain way and hence they form a group. Which one does not belong to that group ? (1) I (2) J (3) K (4) C (5) B

    RANKING/ARRANGEMENT 126. Which of the following statements is correct regarding the flat of H? (1) H lives in the Column-3 of the Row-2. (2) H lives exactly between A and E in the Row-2. (3) H lives in the Column-2 of the Row-3. (4) H lives just below K in the Column-2. (5) H lives third to the right of E in the Row-2 127. Who among the following lives second to the right of K ? (1) B (2) I (3) A (4) E (5) C 128. Who among the following does not live in the Row-2 ? (1) J (2) A (3) H (4) F (5) E 129. Who among the following does live in the Column-1 of Row-1? (1) G (2) F (3) D (4) J (5) L Directions (41–45) : Study the following information carefully and answer the questions given below : (RBI Officer Grade 'B' Phase-I Exam, 06.03.2021)

    Twelve persons are sitting in two parallel rows containing six persons in each row, in such a way that there is an equal distance between adjacent persons. In the Row-1, S, Y, T, J, B and N are seated and all of them are facing south. In the Row-2, P, R, L, K, O and A are seated and all of them are facing north. Therefore, in the given seating arrangement, each member seated in a row faces another member of the other row. ● P is sitting second to the right of O. Only one person is sitting between A and L. Neither S nor B sits at the extreme end of the line. ● A is not an immediate neighbour R. P does not face N. Neither A nor L faces B. ● L does not sit at the extreme end of the line. Only one person sits between Y and N. S is sitting third to the left of B. Neither P nor O faces B or S. ● P does not sit at the extreme end of the line. T does not face P. T does not sit at the extreme end of the line.

    41. Four of the following five are alike in a certain way and hence they form a group. Which one does not belong to that group ? (1) K (2) A (3) J (4) O (5) N 42. Who among the following sits exactly between B and T ? (1) J (2) N (3) S (4) Y (5) No one 43. Who among the following sits third to the right of J ? (1) T (2) Y (3) S (4) B (5) N 44. Who among the following does face R ? (1) N (2) S (3) Y (4) J (5) B 45. Who among the following is sitting fourth to the left of K ? (1) R (2) P (3) O (4) L (5) A 135. Among five friends J, K, L, M and N, each has different height. N is the second tallest in the group. J is taller than only M. L and N are taller than K. Who among them is the second shortest friend ? (1) K (2) J (3) M (4) Cannot be determined (5) None of these (RBI Officer Grade 'B' Phase-I Exam, 06.03.2021)

    INSURANCE EXAMS Directions (1–5) : Study the following information carefully and answer the questions given below : (LIC Assistant Administrative Officer (AAO) Online Exam. 22.03.2015)

    Eight persons — M, N, O, P, Q, R, S and T — are sitting around a circular table facing the centre with equal distances between each other (but not necessarily in the same order). Each of them also belongs to a different field of work viz, Author, Designer, Doctor, Engineer, Journalist, Policeman, Singer and Teacher (but not necessarily in the same order).

    BPRE–363

    Q sits second to the left of the Singer. Only two persons sit between S and the Singer. Only one person sits between Q and the Designer. N sits second to the left of the Designer. P is neither an immediate neighbour of N nor Q. P is not a singer. T is an immediate neighbour of P. The Doctor sits second to the right of T. Only three persons sit between the Doctor and the Engineer. R sits second to the left of the Engineer. T is not a Journalist. The Journalist is neither an immediate neighbour of S nor T. M sits to the immediate right of the Journalist. The Teacher sits to immediate right of the Author. 1. Who sits exactly between N and M, when counted from the right of M? (1) S (2) T (3) O (4) Other than those given as options (5) R 2. Which of the following statements is TRUE as per the given arrangement? (1) Only one person sits between S and O. (2) The Engineer sits second to the left of O. (3) S is a Teacher. (4) All the given options are true. (5) The Author sits to the immediate left of T 3. Who amongst the following is a Designer? (1) M (2) P (3) S (4) T (5) R 4. Four of the following five are alike in a certain way, and so form a group. Which one of the following does not belong to that group ? (1) N, Teacher (2) R, Engineer (3) P, Policeman (4) S, Journalist (5) M, Author 5. What does R work as ? (1) Doctor (2) Engineer (3) Singer (4) Journalist (5) Designer

    RANKING/ARRANGEMENT Directions (6–8) : Study the following information and answer the questions given below : (United India Insurance AO Exam. 27.03.2011)

    Twelve people are standing in two parallel rows each row having 6 people. P, Q, R, S, T and U are standing in row 1 facing South while A, B, C, D, E and F are standing in row 2 facing North. They are standing in such a way that there is an equal distance between two adjacent persons. Thus each person from row 1 is facing another person from row 2. D is standing third to the right of E. C is standing third to the right of A. A and E are not immediate neighbours. U is facing the one who is to the immediate right of C. T is standing fourth to the left of S. B is not facing U. R is standing third to the left of P. 6. Who is facing E ? (1) R (2) S (3) Q (4) P (5) None of these 7. Which of the following pairs represents persons standing at the end of the rows ? (1) TB (2) PD (3) PF (4) DS (5) None of these 8. What is F’s position with respect to R ? (1) F is facing the one who is to the immediate left of R. (2) F is to the immediate left of the person facing R. (3) F is facing the one who is second to the right of R. (4) F is second to the left of the person facing R. (5) None of these Directions (9–11) : Four of the following five are alike in a certain way based on their positions in the above arrangement and so form a group. Which one does not belong to the group? (United India Insurance AO Exam. 27.03.2011)

    9. (1) PQE (3) TRC (5) QUD 10. (1) PSBA (3) BEQS (5) RUCF

    (2) BCR (4) FEQ (2) CEQR (4) UTDF

    11. How many persons are standing between B and D ? (1) One (2) Two (3) Three (4) Four (5) None Directions (12–16) : Study the following arrangement of digits, letters and symbols, and answer the questions given below : (Oriental Insurance Company Exam.08.04.2012)

    F ✡ ETN1 [ 8DIP7 ● ■ JU6 KG | ¥ H2M9★S 3 L C ✶ Q 4 R 5 V ✴ AB 12. How many consonants are there each of which is either immediately preceded by a number and/ or immediately followed by a symbol ? (1) Seven (2) Eight (3) Nine (4) Ten 13. Three of the following four are alike in a cetain way based on the above arrangement and form a group. Which is the one that does not belong to the group ? (1) G2★ (2) 8PJ (3) ●UG (4) |2S 14. If the positions of E and F are interchanged, and the positions of M and A are interchanged, how many vowels will be there each of which will be both immediately preceded and immediately followed by a consonant ? (1) Three (2) Two (3) One (4) Zero 15. What should come in place of the question mark in the following series ? EB✡ 1V[ P ✶ ● ? (1) G9 ■ (2) KH ■ (3) KH (4) GH ■ 16. A man performs three consecutive operations. In his first operation he reverses the order of first 10 elements from the left. After that he starts second operation and reverses the order of first 20 elements from the left. After that he performs his last operation and reverses the order of first 30 elements from the same end. After three operations which element will be 4th to the right of the 15th element from the right end of the newly obtained series ? (1) G (2) K (3) F (4) L

    BPRE–364

    17. Six friends P, Q, R, S, T and U are sitting around a circular table facing towards centre. The angle made at the centre of the circle by a straight lines from P and Q is 180°, from Q and R is 120°, from R and S 180°. T is not sitting on the immediate left of R while R is not on the immediate right of P. On the basis of the above information which of the following statements is definitely true ? (1) R is sitting between P and S. (2) T is sitting between P and U. (3) Q is sitting between S and U. (4) T and R are sitting opposite each other. (Oriental Insurance Company Exam.08.04.2012)

    18. 13 students are standing in a horizontal row from let to right. If all the odd-numbered students in a row are shifted to the successive odd-numbered positions, what will be the position of a boy, who was seventh in the row initially ? (1) fifth from left (2) fifth from right (3) eighth from left (4) ninth from right (Oriental Insurance Company Exam.08.04.2012)

    19. 14 students are standing in row from left to right. After inter changing their positions, first student goes at 14th place, second goes at 13th place, third goes at 12th and so on. If ‘A’ was at ninth position before change, then after changing he would be at which place ? (1) Fifth from right (2) Sixth from right (3) Fifth from left (4) Sixth from left (Oriental Insurance Company Exam.08.04.2012)

    Directions (20–25) : Study the following information carefully and answer the questions given below : (United India Insurance AAO Exam.03.06.2012)

    UQAXWZFDILOVSNB MCRGYKHPTJE Note : Each letter gets a numerical value based on its position in the above arrangement starting from U (Take U = 1, Q = 2, A = 3 ... and so on)

    RANKING/ARRANGEMENT 20. The value of which of the following consonants when added to that of the vowel which follows it immediately amount to 27 ? (1) Q (2) S (3) Z (4) V 21. Some of the letters in the above arrangement occupy the same position as they occupy in the English alphabet sequence. What will be the sum of the numeric value assigned to these letters ? (1) 23 (2) 27 (3) 39 (4) 41 22. Three of the following four are alike in a certain way based on their position in the above arrangement and form a group. Which is the one that does not belong to that group ? (1) FNK (2) SGJ (3) ZVR (4) LMH 23. If starting from Q all the consonants in the above arrangement are given sequentially the value of even numbers such as Q = 2, X = 4, W = 6 and so on and all the vowels are given the value of 7 each, then what will be the value of the letters in the word CUSTOM? (1) 98 (2) 116 (3) 122 (4) 140 24. If letters of the word EXAMINATION are given the same numeric values as their numeric values in the above arrangement, then what be its code ? (1) 127 (2) 133 (3) 138 (4) 144 25. In a row of boys, Srinath is 7th from the left and Venkat is 12th from the right. If they interchange their positions, Srinath becomes 22nd from the left. How many boys are there in the row ? . (1) 19 (2) 31 (3) 33 (4) 34 (NICL (GIC) AO (Finance) Exam, 08.09.2013)

    26. While the group photo of a family was taken, the father was found to be sitting to the left of his son and right to the grandfather. Mother was sitting to the right of her daughter but left to grandfather. Who is occupying the central place ? (1) Son (2) Grandfather (3) Father (4) Mother

    27. In a row of trees, a tree is 7th from left end and 14th from the right end. How many trees are there in the row? (1) 18 (2) 19 (3) 20 (4) 21 28. Five girls M, N, O, P and Q are standing in a row. P is on the right of Q. N is on left of Q, but is on the right of M. P is on the left of O. Who is standing on the extreme right? (1) Q (2) N (3) O (4) P (NICL (GIC) AO Exam, 08.09.2013)

    29. Six friends are sitting in a circle and are facing the centre of the circle. Deepa is between Prakash and Pankaj. Priti is between Mukesh and Lalit. Prakash and Mukesh are opposite to each other. Who is sitting opposite to Prakash ? (1) Mukesh (2) Deepa (3) Pankaj (4) Lalit 30. Six friends are sitting in a circle and are facing the centre of the circle. Deepa is between Prakash and Pankaj. Priti is between Mukesh and Lalit. Prakash and Mukesh are opposite to each other. Who is sitting opposite to Priti? (1) Prakash (2) Deepa (3) Pankaj (4) Lalit (NICL (GIC) AO Exam, 15.12.2013)

    31. There are five different houses, A to E, in a row. A is to the right of B and E is to the left of C and right of A. B is to the right of D. Which of the houses is in the middle ? (1) A (2) E (3) D (4) C 32. There are six persons—A, B, C, D, E and F. We have to rank them in such a way that B gets first rank and E gets the last rank. In how many ways this could be done ? (1) 24 (2) 120 (3) 600 (4) 48 33. (i) Six friends A, B, C, D, E and F are seated in a circle facing each other. (ii) A is between D and B and F is between C and E. (iii) C is the third to the left of B

    BPRE–365

    Which of the following is the position of A in relation to F? (1) Second to the left (2) Second to the right (3) Fourth to the right (4) Third to the right (NICL (GIC) AO Exam, 15.12.2013)

    Directions (34–39) : Study the following information carefully and answer the questions given below : (NIACL Administrative Officer (AO) Exam, 10.01.2015)

    J, K, L, M, N, O, P and Q are sitting around a circular area at equal distances between each other, but not necessarily in the same order. Some of the people are facing the centre while some others are facing outside (i.e. in a direction opposite to the centre). (Note : Facing the same direction means if one faces the centre then the other also faces the centre and vice versa. Facing the opposite directions means if one faces the centre then the other faces outside and vice–versa.) Q sits second to the left of P. Q faces outside. Only two people sit between P and K. Both M and J are immediate neighbours of Q. Only three people sit between J and L. N sits third to the right of M. L is not an immediate neighbour of N. Both the immediate neighbours of P face the same direction. O sits third to the left of J. N faces the same direction as M. O sits to the immediate right of K. Both the immediate neighbours of K face opposite directions. Q faces a direction opposite to that of O. 34. Which of the following is true as per the given seating arrangement ? (1) None of the given options is correct. (2) P sits to the immediate left of M. (3) Q sits second to right of N. (4) L faces the same direction as J. (5) O is an immediate neighbour of M. 35. What is L’s position with respect to K ? (1) Immediate left (2) Immediate right (3) Second to the left (4) Second to the right (5) Third to the right

    RANKING/ARRANGEMENT 36. How many people in the given arrangement face outside ? (1) Four (2) Six (3) Five (4) Two (5) Three 37. Who amongst the following sits exactly between P and O when counted from the right of P ? (1) Q (2) U (3) N (4) M (5) L 38. Who is sitting third to the right of Q ? (1) N (2) K (3) J (4) L (5) O 39. Four of the following five are alike in a certain way based on the given seating arrangement and so form a group. Which is the one that does not belong to that group ? (1) O (2) L (3) K (4) Q (5) N Directions (40–46) : Study the following information carefully and answer the questions given below : (NIACL Administrative Officer (AO) Online Exam, 11.01.2015)

    Eight persons — J, K, L, M, P, Q, R and S — are sitting around a circular table at equal distances between each other, but not necessarily in the same order. Some of the people are facing the centre while some others are facing outside (i.e. in a direction opposite to the centre). (Note : Facing the same direction means if one faces the centre then the other also faces the centre and vice versa. Facing the opposite directions means if one faces the centre then the other faces outside and vice–versa.) K sits second to the right of P. K is an immediate neighbour of J. P faces towards centre. P is to the immediate left of M. Neither K nor L is an immediate neighbour of M. S sits second to the right of M. L is to the immediate right of S. R sits third to the right of Q. L and Q face the same direction as that of M. Q sits second to the right of J. Both the immediate neighbours of Q face the same direction. Both the immediate neighbours of M face the same direction.

    40. How many persons in the given arrangement face outside ? (1) Four (2) Six (3) Five (4) Three (5) Two 41. Who amongst the following is sitting third to the left of L ? (1) K (2) P (3) Q (4) R (5) M 42. Four of the following five are alike in a certain way based on the given seating arrangement and hence they form a group. Which is the one that does not belong to that group ? (1) P (2) Q (3) J (4) L (5) S 43. Who amongst the following sits exactly between K and P if one moves clockwise from K ? (1) S (2) L (3) R (4) Q (5) J 44. What is the position of R with respect to Q ? (1) Immediate right (2) Third to the right (3) Third to the left (4) Second to the right (5) Second to the left 45. Who amongst the following is sitting second to the left of S ? (1) R (2) P (3) K (4) J (5) M 46. Which of the following statements is true as per the given seating arrangement ? (1) P sits to the immediate right of M. (2) J sits second to the right of Q. (3) K faces the same direction as R. (4) S and R immediate neighbours of L. (5) None of the given options is correct Directions (47–52) : Study the following information carefully and answer the questions given below : (NIACL Administrative Officer (AO) Online Exam, 12.01.2015)

    Nine friends A, B, C, D, E, F, G, H and J are sitting around a circle facing a centre, but not necessarily in the same order. C is third to the left of A.

    BPRE–366

    E is fourth to the right of A. D is fourth to the left of J who is second to the right of A. F is third to the right of B. G is not an immediate neighbour of A. 47. What is H’s position with respect to E ? (1) Third to the left (2) Fourth to the left (3) Fifth to the right (4) Fifth to the left (5) Sixth to the left 48. Who is third to the right of G ? (1) B (2) D (3) A (4) Data Inadequate (5) None of these 49. Who is fifth to the right of E ? (1) F (2) C (3) H (4) A (5) None of these 50. Who is second to the left of H ? (1) A (2) F (3) D (4) Data Inadequate (5) None of these 51. Who is to the immediate right of D? (1) F (2) C (3) A (4) Data Inadequate (5) None of these 52. In which of the following groups is the third person sitting between the first and second persons ? (1) CDF (2) EBC (3) HFA (4) JGE (5) EGB Directions (53–54) : Study the following information carefully and answer the questions given below : (LIC Assistant Administrative Officer (AAO) Online Exam. 22.03.2015)

    Eight persons — K, L, M, N, O, P, Q and R — are seated in a straight line facing North but not necessarily in the same order. L sits third from the left end of the line. Only three persons sit between L and R. O sits to the immediate left of Q. Only one person sits between M and Q. P sits third to the left of N. L is not an immediate neighbour of K. 53. What is the position of Q with respect to P? (1) Second to the left (2) Third to the left (3) Second to the right (4) Third to the right (5) Fifth to the left

    RANKING/ARRANGEMENT 54. Which of the following pairs represents persons seated at the two extreme ends of the line? (1) L, K (2) Q, R (3) Q, N (4) O, N (5) O, R Directions (55–59) : Study the following information carefully and answer the questions given below : (OICL Specialist Officer (Finance) Exam. 03.05.2015)

    Eight persons — A, B, C, D, P, Q, R and S — are sitting around a circular table with equal distances between each other (but not necessarily in the same order). Some of them face outside and some others face the centre. NOTE : Facing the same direction means if one faces the centre then the other also faces the centre and vice– versa. Facing the opposite directions means if one faces the centre then the other faces outside and vice-versa. Immediate neighbours face the same direction means if one neighbour faces the centre then the other neighbour also faces the centre and vice–versa. Immediate neighbours face the opposite directions means if one neighbour faces the centre then the other neighbour faces outside and vice–versa. P sits to the immediate right of C. Only three persons sit between P and A. R sits third to the left of A. Neither A nor C is an immediate neighbour of B. D sits to the immediate left of B. Q sits second to the left of D. The immediate neighbours of A face the same direction. The immediate neighbours of B face opposite directions. S sits second to the right of P. The immediate neighbours of C face the same direction. S faces outside. 55. Four of the following five are alike in a certain way based on the given sitting arrangement and so form a group. Which is the one that does not belong to that group? (1) P (2) Q (3) D (4) B (5) A 56. What is the position of D with respect to S? (1) Second to the right (2) Third to the right (3) Third to the left (4) Immediate right (5) Fourth to the left

    57. Who sits third to the right of Q? (1) B (2) P (3) A (4) Other than those given as options (5) R 58. How many persons in the given arrangement face the centre? (1) More than four (2) One

    (3) Three

    (4) Two (5) Four 59. Who sits exactly between B and the one who sits to the immediate left of R, when counted from the right of B? (1) D (2) A (3) P (4) R (5) Q Directions (60–64) : Study the following information carefully and answer the questions given below : (LIC Assistant Administrative Officer (AAO) Online Exam. 05.03.2016)

    Eight persons namely, A, B, C, D, E, F, G and H are sitting around a circular area at equal distances between each other, but not necessarily in the same order. Some people face the centre while some face outside (i.e. in a direction opposite to the centre.) ● C sits second to the left of G. G faces the centre. Both the immediate neighbours of C face outside. D sits third to the right of C. B sits third to the right of H. H is neither an immediate neighbour of C nor D. ● Both the immediate neighbours of A face outside. ● E faces same direction as H (i.e. if H faces the centre then E also faces the centre and vice-versa.) ● Immediate neighbours of E face opposite directions (i.e. if one neighbour faces the centre then the other faces outside and viceversa.) 60. Four of the following five are alike in a certain way based on the given seating arrangement and so form a group. Which is the one that does not belong to that group ? ● ●

    (1) B (3) E (5) C

    (2) D (4) A

    BPRE–367

    61. Who amongst the following are immediate neighbours of E ? (1) D, H

    (2) D, F

    (3) C, G (4) A, G (5) A, C 62. Which of the following statements is true regarding F as per the given seating arrangement ? (1) A is one of the immediate neighbours of F. (2) F faces the centre. (3) None of the given options is true (4) Only three persons sit between F and D. (5) F sits third to the left of H. 63. What is H’s position with respect to C ? (1) Fifth to the left (2) Fourth to the right (3) Third to the right (4) Second to the right (5) Third to the left 64. Who is sitting to the immediate right of G ? (1) A

    (2) H

    (3) D (5) E

    (4) F

    Directions (65–69) : Study the following information carefully and answer the questions given below : (LIC Assistant Administrative Officer (AAO) Online Exam. 05.03.2016)

    Twelve persons are sitting in two parallel rows containing six persons each, in such a way that there is equal distance between adjacent persons. In row-1, S, T, U, V, W and X are seated and all of them are facing south, but not necessarily in the same order-I. In row-2, M, N, O, P, Q and R are seated and all of them are facing north, but not necessarily in the same order. Therefore, in the given seating arrangement, each member seated in a row faces another member of the other row. T sits third to the left of X. X does not sit at any extreme ends of the line. P sits second to the left of the one who faces T. Only two persons sit between P and M. V sits second to the right of S. S is not an immediate neighbour of T. O sits second to the left or R. Q is neither an immediate neighbour of M nor R. U is not an immediate neighbour of S.

    RANKING/ARRANGEMENT 65. Four of the following five are alike in a certain way based on the given arrangement and hence form a group. Which one of them does not belong to that group ? (1) N (2) O (3) R (4) U (5) T 66. What is the position of P with respect to Q ? (1) Fourth to the right (2) Third to the left (3) Second to the right (4) Second to the left (5) Third to the right 67. Who amongst the following is facing N ? (1) S (2) T (3) W (4) V (5) X 68. Which of the following statements is true regarding W? (1) T is one of the immediate neighbours of W. (2) Only one person sits between W and V. (3) U sits to second to the right of W. (4) W faces one of the immediate neighbours of M. (5) None of the given statements is ture 69. Who amongst the following is facing U ? (1) R (2) P (3) O (4) Q (5) M Directions (70–74) : Study the following information carefully and answer the questions given below : (LIC Assistant Administrative Officer (AAO) Online Exam. 06.03.2016)

    Eight persons — I, J, K, L, M, N, O and P — are sitting around a circular area at equal distance between each other but not necessarily in the same order. Some people face the centre while some face the outside (i.e. in a direction opposite to the centre). J sits third to the right of M. M faces outside. Only person sits between J and N. Only three persons sit between Q and P. O is not an immediate neighbour of J.

    Both the immediate neighbours of L face the centre. L is not an immediate neighbour of O. Both the immediate neighbours of K face centre. Both the immediate neighbours of M face a direction opposite to that of O (i.e. if O faces the centre then both the immediate neighbours of M face outside and vice-versa.) 70. Which of the following statements is true regarding I as per the given arrangement? (1) P is one of the immediate neighbours of I. (2) I faces the centre. (3) I sits third to the left of L. (4) Only three persons sit between I and N. (5) None of the given options is true 71. Who is sitting to immediate right of L? (1) K (2) M (3) I (4) P (5) J 72. What is O’s position with respect to K? (1) Fifth to the right (2) Second to the right (3) Third to the right (4) To the immediate left (5) Fourth to the right 73. Who amongst the following are immediate neighbours of N? (1) P, M (2) L, M (3) O, P (4) I, O (5) I, K 74. Four of the following five are alike in a certain way based on the given arrangement and so form a group. Which is the one that does not belong to that group? (1) J (2) P (3) O (4) L (5) K Directions (75–79) : Study the following information carefully and answer the questions given below : (LIC Assistant Administrative Officer (AAO) Online Exam. 06.03.2016)

    Twelve people are sitting in two parallel rows containing six people each, in such a way that there is equal distance between adjacent persons. In row-1 B, C, D, E, F and G are seated

    BPRE–368

    and all of them are facing south. In row-2 P, Q, R, S, T and U are seated and all of them are facing north. Therefore, in the given seating arrangement, each member seated in a row faces another member of the other row. (Please Note : None of the information given is necessarily in the same order.) R sits second to the right of S. Only one person sits between R and U. C sits to the immediate right of the one who faces U. Only two persons sit between C and F. Q sits fourth to the left of T. G is an immediate neighbour of the one who faces T. D does not face R. B sits third to the right of D. E is to the immediate right of B. G does not face P. 75. Who amongst the following is facing C? (1) U (2) S (3) P (4) R (5) T 76. Which of the following statements is true regarding P? (1) E is an immediate neighbour of the one who faces P. (2) Only one person sits between P and T. (3) P sits to the immediate right of U. (4) None of the given statements is true. (5) Q is an immediate neighbour of P. 77. Four of the following five are alike in a certain way based on the given arrangement and hence form a group. Which of them does not belong to that group? (1) F (2) D (3) S (4) T (5) G 78. What is the position of B with respect to C? (1) Second to the right (2) Second to the left (3) Fourth to the right (4) Immediate right (5) Third to the left 79. Who amongst the following is facing Q? (1) E (2) B (3) F (4) C (5) D

    RANKING/ARRANGEMENT Directions (80-84) : Study the following information carefully and answer the questions given below : (NICL A.O. Exam 05.06.2017)

    Eight persons — U, V, J, A, X, W, B and M — are sitting around a square table in such a way that four of them sit at four corners of the square while four sit in the middle of each of the four sides. The ones who sit at the four corners face the centre while those who sit in the middle of the sides face outside. A is sitting second to the right of U. U sits at one of the middle side of table. X sits second to the right of B. B is not the neighbour of U and A. There are only three persons between A and M. J is not the neighbour of U. V sits second to the right of W. 80. Which of the following statements is not true with respect to the given sitting arrangement ? (1) Two persons sit between A and U. (2) B and M do not face each other in the sitting arrangement (3) X and A are immediate neighbours of each other. (4) W sits opposite to B. (5) U sits in middle of one of the sides of the square table. 81. Who amongst the following is sitting third to the right of V ? (1) M (2) J (3) X (4) W (5) Cannot be determined 82. How many people sit between X and W when counted in anticlockwise direction from W ? (1) None (2) One (3) Two (4) Three (5) More than three 83. Who amongst the following sits opposite to U ? (1) A (2) W (3) J (4) V (5) Other than those given as options 84. What is the position of X with respect to J ? (1) Immediate right (2) Second to the left (3) Third to the right (4) Immediate left (5) Second to the right Directions (85-89) : Study the following information carefully and answer the questions given below : (NICL A.O. Exam 05.06.2017)

    Eight friends — F, G, H, L, M, N, O and Q — are sitting in a straight line facing North but not necessarily in the same order. ● H is sitting second to the right of N and L is sitting fourth to the right of H. ● M is sitting to the right of F. ● Number of persons can sit between G and L is same as the numbers of persons sitting between O and F. ● O is sitting to the immediate left of G who is not an immediate neighbour of L. ● F is not an immediate neighbour of N. 85. If all the arranged persons are rearranged in alphabetical order from left to right, positions of how many of them will remain unchanged ? (1) None (2) One (3) Two (4) Three (5) More than three 86. Which of the following pairs are sitting at the extreme ends of the line ? (1) N, M (2) Q, L (3) O, Q (4) Other than those given as options (5) Cannot be determined 87. Based on the given arrangement, Q is related to O and H is related to G in a same way as O is related to ? (1) M (2) L (3) F (4) Q (5) Other than those given as options 88. How many persons are sitting between Q and M ? (1) One (2) Two (3) Three (4) Four (5) More than four 89. Who is sitting third to the left of H? (1) Q (2) N (3) L (4) No one, as only two persons are sitting to the left of H (5) Other than those given as options Directions (90–94) : Study the following information carefully and answer the questions given below : (GIC Officer Online Exam 15.05.2017)

    Eight people — A, B, C, D, E, F, G and H — are sitting in a straight line

    BPRE–369

    with equal distance between each other, but not necessarily in the same order. Some of them are facing north and some are facing south. B sits at one of the extreme ends of the line. Only three people sit between B and G. A sits third to the right of G. A is not an immediate neighbour of B. E sits to the immediate left of A. Only two people sit between E and F. Immediate neighbours of F face opposite directions (i.e. if one neighbour faces north then the other neighbour faces south and vice-versa.) D sits to the immediate left of H. Immediate neighbours of D face the same direction (i.e. if one neighbour faces north then the other neighbour also faces north and vice-versa.) Persons sitting at the extreme ends face the same direction (i.e. if one person faces north then the other one also faces north and vice-versa.) C faces north. G sits to the immediate right of C. Immediate neighbours of C face the same direction (i.e. if one neighbour faces north then the other neighbour also faces north and viceversa.) 90. How many people sit between C and A as per the given arrangement? (1) Two (2) Three (3) One (4) More than three (5) None 91. Which of the following statements is true as per the given arrangement? (1) A sits second to the right of C. (2) None of the given statements is true. (3) D faces south. (4) C is an immediate neighbour of E. (5) Only two people sit between G and D. 92. Who amongst the following sits fourth to the left of D? (1) Other than those given as options (2) B (3) C (4) G (5) E 93. Four of the following five are alike in a certain way based on the given arrangement and thus form a group. Which of the following does not belong to the group? (1) F (2) B (3) G (4) E (5) A

    RANKING/ARRANGEMENT 94. Which of the given pairs represents the immediate neighbours of the persons sitting at the extreme ends of the line? (1) F, H (2) H, E (3) E, C (4) C, D (5) D, F Directions (95–99) : Study the following information carefully and answer the questions given below : (GIC Officer Online Exam 15.05.2017)

    Eight friends — A, B, C, D, E, F, G and H — are sitting around a circular table facing the centre but not necessarily in the same order. Each of them visited a State viz., Sikkim, Manipur, Nagaland, Punjab, Rajasthan, Odisha, Kerala and Assam but not necessarily in the same order. A sits third to the left of the one who visited Punjab, Only one person sits between the one who visited Punjab and B. Only three people sit between the one who visited Nagaland and B. Only three people sit between the ones who visited Sikkim and Odisha. A neither visited Sikkim nor Odisha. B is not an immediate neighbour of the one who visited Sikkim. C is an immediate neighbour of A. Only three people sit between C and D. C sits third to the right of G. Only two people sit between C and E. H does not visit Nagaland, Only one person sits between the one who visited Kerala and F. Neither B nor G visited Rajasthan. The one who visited Assam sits third to the right of the one who visited Manipur. 95. Which of the following will come in place of the (?) in the given series based on the given ar rangement? Sikkim B Assam C Odisha ? (1) G (2) E (3) F (4) D (5) H 96. Which of the following statements is not TRUE about D as per the given arrangement? (1) Only three people sit between the one who visited Kerala and D. (2) D sits to the immediate right of the one who visited Assam. (3) F sits second to the right of D. (4) D sits third to the left of H. (5) All the given statements are true.

    97. What is the position of H with respect to the one who visited Nagaland? (1) Second to the right (2) Second to the left (3) Third to the right (4) Immediate left (5) Immediate right 98. Who sits to the immediate left of the one who visited Odisha? (1) The one who visited Manipur (2) A (3) The one who visited Rajasthan (4) The one who visited Nagaland (5) D 99. How many persons are sitting between the one who visited Punjab and H when counted from the left of H? (1) More than three (2) None (3) Three (4) Two (5) One Directions (100–104) : Study the following information carefully and answer the questions given below : (GIC Officer Online Exam 15.05.2017)

    Twelve people are sitting in two parallel rows containing six people each, in such a way that there is equal distance between adjacent persons. In row–1 S, T, U, V, W and X are seated and all of them are facing south. In row–2 J, K, L, M, N and O are seated and all of them are facing north. Therefore in the given seating arrangement, each member seated in a row faces another member of the other row. (Please Note: None of the information given is necessarily in the same order.) L sits second from an extreme end of the row. Only two people sit between L and N. V sits to the immediate right of the one who faces N. W sits third to the right of X. X does not sit at an extreme end of the row. J sits second to the left of the one who faces X. Only one person sits between J and M. One of the immediate neighbours of J faces T. S sits at one of the positions to the left of T. S does not face O. 100. Four of the following five are alike in a certain way based on the given arrangement and hence form a group. Which one of them does not belong to that group? (1) V (2) M (3) K (4) S (5) U

    BPRE–370

    101. Who amongst the following is facing K? (1) T (2) S (3) X (4) W (5) V 102. Which of the following statements is true as per the given arrangement? (1) O faces one of the immediate neighbours of U. (2) None of the given statements is true. (3) L sits second to the left of J. (4) X faces N. (5) N is an immediate neighbour of K. 103. What is the position of S with respect to X? (1) Third to the left (2) Second to the right (3) Immediate left (4) Immediate right (5) Second to the left 104. Who amongst the following is facing U? (1) N (2) M (3) L (4) J (5) O Directions (105–109) : Study the following information carefully and answer the questions given below : Ten persons, i.e., A, B, C, D, E, P, Q, R, S and T are sitting in a single row and all are facing to the North direction, but not necessarily in the same order. A is fourth from the extreme end. Only three persons sit between A and B. C sits to the immediate right of B. Only two persons sit between C and D. Only three persons sit between R and C. R sits to the immediate left of P. E sits to the right of R. Only two persons sit between S and T. S sits to the left of T. (LIC AAO Prelim Exam. 04.05.2019)

    105. Who among the following person sits third to the left of P? (1) Q (2) T (3) C (4) A (5) None of these 106. How many persons sit between R and T? (1) None (2) One (3) Two (4) Three (5) More than three 107. Who among the following persons are sitting at extreme ends? (1) A and B (2) B and C (3) C and D (4) D and E (5) None of these

    RANKING/ARRANGEMENT 108. Who among the following persons sits second to the right of S? (1) A (2) B (3) F (4) E (5) C 109. The number of persons sitting to the left of B is same as the number of persons sitting right to which of the following person? (1) P (2) R (3) Q (4) S (5) None of these Directions (110–114) : Study the following information carefully and answer the questions given below: Nine persons i.e. A, B, C, D, E, F, G, H and J are sitting around a circular table but not necessarily in the same order. All of them are facing towards the centre. A sits second to the right of B. F sits fourth to the left of A. Only two persons sit between C and F. G is an immediate neighbour of J. G sits second to the right of H. D sits second to the left of E. E is not an immediate neighbour of A. (LIC AAO Prelim Exam. 04.05.2019)

    110. Who among the following person sits third to the right of E? (1) B (2) A (3) C (4) F (5) None of these 111. How many persons sit between E and F? (1) Two (2) One (3) Three (4) None (5) More than three 112. Who among the following person sits exactly between C and E? (1) B (2) A (3) D (4) F (5) H 113. Who among the following person sits to the immediate right of A? (1) B (2) C (3) D (4) G (5) H 114. Four of the following five are alike in a certain way based on their pattern. Which of the following does not belong to that group? (1) E – C (2) B – F (3) A – C (4) D – J (5) C – H Directions (115–119) : Study the following information carefully and answer the questions given below : Fourteen people are sitting in two parallel rows containing seven people each with equal distance between each other.

    In row 1- A, B, C, D, E, F and G are seated and all of them are facing north but not necessarily in the same order. In row 2- J, K, L, M, N, O and P are seated and all of them are facing south but not necessarily in the same order. Thus in the given arrangement each person seated in a row faces another person of the other row. C sits at one of the extreme ends of the row. P is an immediate neighbour of the one who faces C. Only one person sits between P and N. The one who faces N sits to the immediate right of G. Only two people sit between G and F. Only two people sit between the one who faces F and M. K sits second to the right of M. The one who faces K is an immediate neighbour of D. More than three people sit between D and E. The one who faces E sits to the immediate left of O. The one who faces O is an immediate neighbour of A. J sits at one of the positions to the left of L. (LIC AAO Prelim Exam. 05.05.2019)

    115. Who amongst the following faces B? (1) K (2) M (3) J (4) L (5) N 116. Which of the following statements is TRUE based on the given arrangement? (1) N faces the one who sits second to the right of A. (2) Only two people sit between N and J. (3) As many people sit between B and D as between L and P (4) All of the given statements are true. (5) J is an immediate neighbour of L. 117. M is related to K in the same way as G is related to A following a certain pattern based on the given arrangement. Following the same pattern who is related to D? (1) C (2) G (3) B (4) E (5) F 118. As per the given arrangement, four of the following five are alike in a certain way based on their positions and thus they form a group. Identify the one which does not belong to the group? (1) J (2) A (3) L (4) P (5) D

    BPRE–371

    119. Who amongst the following sits to the immediate right of K? (1) J (2) N (3) L (4) No one (5) P Directions (120–124) : Study the following information carefully and answer the questions given below : Eight boxes – A, B, C, D, P, Q, R and S—are kept one above the other in a stack. Only five boxes are kept between box A and box P. Box B is kept immediately below box P. Only one box is kept between box B and box R. Box R is kept at one of the positions above box P. Only three boxes are kept between box R and box D. Only two boxes are kept between box D and box S. Box C is kept at one of the positions above box Q. (LIC AAO Prelim Exam. 05.05.2019)

    120. Which of the following statements is true based on the given arrangement? (1) Only two boxes are kept above box S. (2) As many boxes are kept below box R as above box C. (3) No box is kept between box Q and box D. (4) None of the given statements is true. (5) Box A is kept at the bottom of the stack. 121. As per the given arrangement, how many boxes are kept between box A and box Q? (1) Two (2) None (3) One (4) Three (4) More than three 122. As per the given arrangement, as many boxes are kept above box D as below box __________. (1) S (2) A (3) P (4) Q (5) R 123. Four of the following five are alike in a certain way based bn their positions in the given arrangement and thus they form a group. Identify the one which does not belong to the group. (1) S–C (2) C–P (3) P–S (4) A–C (5) R–B 124. As per the given arrangement, which box is kept immediately below box S? (1) D (2) A (3) P (4) R (5) C

    RANKING/ARRANGEMENT Directions (125-130) : Study the following information carefully and answer the questions given below : (LIC AAO Main Exam, 28.06.2019)

    There are eight family members A, B, C, D, E, F, G and H sitting around a circular table facing the centre but not necessarily in the same order. The family has three generation gap with three couples in the family and each couple has at least one child. H is sitting third to the right of his aunt. E is blood related only with H in the family. G is sitting opposite to A’s husband. F is sitting second to the left of a person who is sitting to the immediate right of E. C, who has only one son-in-law, is sitting second to the left of D who is granddaughter of F. B is a female member sitting second to the right of G who is paternal uncle of H. A is sitting second to the right of a person who the is brother-in-law of B. D is sitting opposite to a person who is sitting third to the left of D’s father. The son-in-law of the family is sitting third to the left of his mother-inlaw. E is sitting exactly between G’s father to the F to the left and A’s son to the right. B is blood related only with D in the family. 125. How is A related to F? (1) Mother (2) Mother-in-law (3) Son (4) Grandson (5) Daughter 126. Who among the following is sitting second to the right of E? (1) E’s son (2) G’s wife (3) C’s daughter (4) F’s grandson (5) E’s sister-in-law 127. Who among the following is sitting opposite to D? (1) D’s Mother (2) D’s Grandfather (3) D’s uncle (4) D’s cousin (5) D’s Nephew

    128. How many family members are sitting exactly between Grandfather and Grandmother in the family? (1) Two (2) Three (3) Four (4) Five (5) More than five 129. Which of the following family’s member is sitting to the immediate right of daughter-in-law of the family? (1) Son-in-law (2) Grandson (3) Mother-in-law (4) Grandfather (5) None of these Directions (130-134): Study the following information carefully and answer the questions given below : (LIC AAO Main Exam, 28.06.2019)

    A class teacher makes a group of five students according to their ranks in the final exam. S is 8 ranks behind F. K is 11 ranks ahead of A. L is 6 ranks ahead of F. S is 11th rank behind when compared from the rank of K. F is 10th rank ahead when compared to the rank of A. F is 7th rank behind when compared to L’s rank. S is 6th rank ahead when compared to the last rank holder. The group definitely includes the 1st rank holder. F occupies a prime number rank. 130. How many students are there in the class ? (1) 25 students (2) 20 students (3) 30 students (4) 22 students (5) 8 students 131. What is the rank occupied by F? (1) 11th rank (2) 13th rank (3) 5th rank (4) 7th rank (5) 17th rank 132. What is the rank of K with respect to the last rank holder? (1) 14th rank (2) 15th rank (3) 16th rank (4) 17th rank (5) 18th rank 133. Who among the following in the selected group has secured the least rank? (1) A (2) K (3) S (4) F (5) L 134. What is the rank occupied by S? (1) 8th rank (2) 13th rank (3) 12th rank (4) 16th rank (5) 15th rank

    BPRE–372

    SHORT ANSWERS

    NATIONALISED BANKS & IBPS PO/MT/SO EXAMS 1. (1) 5. (5) 9. (1) 13. (1) 17. (2) 21. (3) 25. (5) 29. (2) 33. (2) 37. (1) 41. (2) 45. (5) 49. (3) 53. (5) 57. (1) 61. (5) 65. (2) 69. (2) 73. (5) 77. (4) 81. (2) 85. (5) 89. (4) 93. (4) 97. (5) 101. (3) 105. (1) 109. (1) 113. (3) 117. (2) 121. (1) 125. (2) 129. (4) 133. (4) 137. (2) 141. (4) 145. (2) 149. (2) 153. (2) 157. (3) 161. (2) 165. (5) 169. (4) 173. (4) 177. (3) 180. (2) 185. (3)

    2. (4) 6. (3) 10. (4) 14. (2) 18. (3) 22. (4) 26. (5) 30. (3) 34. (3) 38. (3) 42. (4) 46. (3) 50. (4) 54. (3) 58. (5) 62. (2) 66. (2) 70. (5) 74. (2) 78. (5) 82. (5) 86. (3) 90. (2) 94. (1) 98. (5) 102. (1) 106. (3) 110. (4) 114. (5) 118. (1) 122. (1) 126. (5) 130. (4) 134. (5) 138. (3) 142. (4) 146. (3) 150. (1) 154. (1) 158. (5) 162. (3) 166. (2) 170. (4) 174. (4) 178. (3) 182. (1) 186. (1)

    3. (5) 7. (2) 11. (4) 15. (4) 19. (3) 23. (2) 27. (4) 31. (3) 35. (4) 39. (3) 43. (4) 47. (5) 51. (4) 55. (5) 59. (4) 63. (4) 67. (1) 71. (1) 75. (2) 79. (3) 83. (5) 87. (1) 91. (5) 95. (2) 99. (5) 103. (4) 107. (1) 111. (5) 115. (4) 119. (2) 123. (5) 127. (4) 131. (3) 135. (3) 139. (5) 143. (2) 147. (1) 151. (5) 155. (1) 159. (3) 163. (4) 167. (3) 171. (3) 175. (1) 179. (3) 183. (2) 187. (4)

    4. (1) 8. (5) 12. (5) 16. (1) 20. (3) 24. (4) 28. (4) 32. (1) 36. (4) 40. (5) 44. (2) 48. (4) 52. (2) 56. (2) 60. (4) 64. (3) 68. (4) 72. (2) 76. (3) 80. (3) 84. (4) 88. (4) 92. (5) 96. (3) 100. (3) 104. (3) 108. (5) 112. (2) 116. (3) 120. (5) 124. (3) 128. (1) 132. (5) 136. (1) 140. (5) 144. (1) 148. (3) 152. (4) 156. (2) 160. (3) 164. (4) 168. (2) 172. (5) 176. (2) 180. (4) 184. (1) 188. (3)

    RANKING/ARRANGEMENT 189. (1) 193. (2) 197. (3) 201. (5) 205. (2) 209. (3) 213. (2) 217. (4) 221. (3) 225. (3) 229. (5) 233. (2) 237. (4) 241. (3) 245. (4) 249. (5) 253. (1) 257. (5) 261. (5) 265. (1) 269. (5) 273. (5) 277. (2) 281. (5) 285. (2) 289. (4) 293. (4) 297. (1) 301. (2) 305. (3) 309. (1) 313. (5) 317. (1) 321. (5) 325. (2) 329. (4) 333. (1) 337. (3) 341. (2) 345. (4) 349. (1) 353. (3) 357. (4) 361. (2) 365. (5) 369. (1) 373. (5) 377. (2) 381. (5) 385. (4) 389. (5) 393. (3) 397. (2) 401. (3)

    190. (4) 194. (4) 198. (2) 202. (2) 206. (3) 210. (4) 214. (2) 218. (5) 222. (5) 226. (1) 230. (3) 234. (4) 238. (4) 242. (1) 246. (3) 250. (1) 254. (1) 258. (2) 262. (1) 266. (1) 270. (2) 274. (1) 278. (4) 282. (5) 286. (1) 290. (2) 294. (3) 298. (3) 302. (2) 306. (5) 310. (4) 314. (3) 318. (2) 322. (4) 326. (5) 330. (5) 334. (5) 338. (4) 342. (5) 346. (3) 350. (4) 354. (2) 358. (2) 362. (3) 366. (2) 370. (2) 374. (2) 378. (5) 382. (2) 386. (2) 390. (3) 394. (1) 398. (3) 402. (2)

    191. (2) 195. (1) 199. (3) 203. (4) 207. (2) 211. (5) 215. (3) 219. (3) 223. (4) 227. (4) 231. (1) 235. (5) 239. (2) 243. (3) 247. (2) 251. (1) 255. (2) 259. (1) 263. (1) 267. (2) 271. (1) 275. (3) 279. (1) 283. (4) 287. (1) 291. (5) 295. (1) 299. (4) 303. (4) 307. (3) 311. (2) 315. (1) 319. (4) 323. (5) 327. (2) 331. (1) 335. (4) 339. (3) 343. (1) 347. (5) 351. (2) 355. (5) 359. (1) 363. (4) 367. (3) 371. (4) 375. (3) 379. (1) 383. (3) 387. (1) 391. (2) 395. (5) 399. (4) 403. (4)

    192. (5) 196. (5) 200. (1) 204. (3) 208. (1) 212. (1) 216. (1) 220. (2) 224. (2) 228. (2) 232. (5) 236. (2) 240. (5) 244. (1) 248. (2) 252. (4) 256. (2) 260. (2) 264. (4) 268. (2) 272. (2) 276. (4) 280. (3) 284. (3) 288. (3) 292. (2) 296. (5) 300. (5) 304. (1) 308. (5) 312. (4) 316. (2) 320. (3) 324. (1) 328. (3) 332. (2) 336. (2) 340. (1) 344. (2) 348. (3) 352. (5) 356. (1) 360. (2) 364. (5) 368. (4) 372. (2) 376. (3) 380. (4) 384. (1) 388. (4) 392. (4) 396. (5) 400. (1) 404. (5)

    405. (1) 409. (3) 413. (5) 417. (4) 421. (4) 425. (3) 429. (5) 433. (3) 437. (3) 441. (5) 445. (5) 449. (4) 453. (1) 457. (1) 461. (4) 465. (5) 469. (4) 473. (3) 477. (1) 481. (3) 485. (2) 489. (2) 493. (4) 497. (5) 501. (1) 505. (3) 509. (5) 513. (5) 517. (4) 521. (2) 525. (4) 529. (2) 533. (5) 537. (1) 541. (2) 545. (3) 549. (4) 553. (3) 557. (3) 561. (4) 565. (2) 569. (5) 573. (2) 577. (1) 581. (2) 585. (2) 589. (1) 593. (2) 597. (5) 601. (4) 605. (4) 609. (2) 613. (4) 617. (2)

    406. (4) 410. (5) 414. (1) 418. (5) 422. (3) 426. (2) 430. (1) 434. (1) 438. (2) 442. (4) 446. (3) 450. (4) 454. (1) 458. (4) 462. (2) 466. (4) 470. (2) 474. (5) 478. (2) 482. (3) 486. (5) 490. (4) 494. (1) 498. (2) 502. (5) 506. (5) 510. (4) 514. (2) 518. (3) 522. (5) 526. (1) 530. (4) 534. (2) 538. (4) 542. (3) 546. (2) 550. (2) 554. (5) 558. (1) 562. (1) 566. (3) 570. (2) 574. (5) 578. (5) 582. (3) 586. (2) 590. (5) 594. (3) 598. (4) 602. (5) 606. (5) 610. (3) 614. (5) 618. (3)

    407. (2) 411. (2) 415. (4) 419. (2) 423. (4) 427. (1) 431. (2) 435. (5) 439. (3) 443. (3) 447. (1) 451. (4) 455. (4) 459. (3) 463. (3) 467. (1) 471. (4) 475. (2) 479. (5) 483. (1) 487. (5) 491. (1) 495. (3) 499. (4) 503. (4) 507. (1) 511. (2) 515. (3) 519. (4) 523. (5) 527. (2) 531. (5) 535. (4) 539. (5) 543. (1) 547. (5) 551. (1) 555. (2) 559. (2) 563. (4) 567. (3) 571. (1) 575. (3) 579. (4) 583. (3) 587. (3) 591. (5) 595. (4) 599. (1) 603. (1) 607. (4) 611. (1) 615. (4) 619. (5)

    BPRE–373

    408. (5) 412. (3) 416. (2) 420. (1) 424. (5) 428. (3) 432. (4) 436. (4) 440. (2) 444. (1) 448. (2) 452. (1) 456. (2) 460. (4) 464. (1) 468. (3) 472. (1) 476. (1) 480. (4) 484. (4) 488. (3) 492. (2) 496. (2) 500. (3) 504. (2) 508. (1) 512. (3) 516. (1) 520. (1) 524. (3) 528. (1) 532. (3) 536. (3) 540. (1) 544. (4) 548. (5) 552. (3) 556. (1) 560. (5) 564. (5) 568. (4) 572. (3) 576. (4) 580. (1) 584. (1) 588. (4) 592. (1) 596. (3) 600. (2) 604. (3) 608. (5) 612. (1) 616. (1) 620. (3)

    621. (2) 625. (3) 629. (3) 633. (4) 637. (1) 641. (5) 645. (4) 649. (1) 653. (3) 657. (5) 661. (4) 665. (2) 669. (4) 673. (4) 677. (4) 681. (1) 685. (4) 689. (2) 693. (3) 697. (4) 701. (5) 705. (2) 709. (4) 713. (4) 717. (2) 721. (2) 725. (3) 729. (4) 733. (1) 737. (1) 741. (5) 745. (5) 749. (2) 753. (2) 757. (4) 761. (3) 765. (1) 769. (1) 773. (2) 777. (3) 781. (2) 785. (1) 789. (4) 793. (1) 797. (4) 801. (2) 805. (4) 809. (4) 813. (3) 817. (1) 821. (3) 825. (4) 829. (1)

    622. (4) 626. (2) 630. (4) 634. (2) 638. (3) 642. (4) 646. (2) 650. (2) 654. (4) 658. (2) 662. (1) 666. (3) 670. (3) 674. (5) 678. (3) 682. (3) 686. (5) 690. (4) 694. (5) 698. (3) 702. (2) 706. (3) 710. (5) 714. (5) 718. (3) 722. (4) 726. (2) 730. (3) 734. (4) 738. (3) 742. (4) 746. (4) 750. (1) 754. (5) 758. (2) 762. (5) 766. (4) 770. (2) 774. (2) 778. (4) 782. (3) 786. (5) 790. (2) 794. (4) 798. (3) 802. (1) 806. (2) 810. (2) 814. (5) 818. (3) 822. (2) 826. (1) 830. (2)

    623. (1) 627. (4) 631. (5) 635. (3) 639. (1) 643. (3) 647. (1) 651. (3) 655. (1) 659. (5) 663. (1) 667. (1) 671. (2) 675. (3) 679. (2) 683. (2) 687. (1) 691. (5) 695. (2) 699. (4) 703. (4) 707. (5) 711. (3) 715. (1) 719. (1) 723. (1) 727. (3) 731. (5) 735. (3) 739. (2) 743. (2) 747. (3) 751. (5) 755. (5) 759. (4) 763. (5) 767. (2) 771. (4) 775. (5) 779. (1) 783. (3) 787. (2) 791. (1) 795. (2) 799. (2) 803. (3) 807. (1) 811. (1) 815. (2) 819. (2) 823. (5) 827. (5) 831. (4)

    624. (5) 628. (2) 632. (1) 636. (3) 640. (2) 644. (5) 648. (5) 652. (4) 656. (2) 660. (3) 664. (4) 668. (1) 672. (5) 676. (5) 680. (5) 684. (1) 688. (3) 692. (5) 696. (1) 700. (1) 704. (4) 708. (2) 712. (1) 716. (3) 720. (3) 724. (4) 728. (1) 732. (2) 736. (2) 740. (1) 744. (1) 748. (3) 752. (4) 756. (3) 760. (1) 764. (3) 768. (3) 772. (5) 776. (4) 780. (5) 784. (4) 788. (5) 792. (3) 796. (5) 800. (1) 804. (3) 808. (5) 812. (4) 816. (4) 820. (5) 824. (5) 828. (3) 832. (5)

    RANKING/ARRANGEMENT 833. (3) 837. (1) 841. (5) 845. (3) 849. (1) 853. (4) 857. (3) 861. (1)

    834. (3) 838. (2) 842. (2) 846. (5) 850. (5) 854. (4) 858. (1) 862. (2)

    835. (5) 839. (4) 843. (1) 847. (4) 851. (4) 855. (2) 859. (5) 863. (3)

    836. (4) 840. (3) 844. (2) 848. (2) 852. (2) 856. (5) 860. (4)

    SBI PO EXAMS 1. (2) 5. (2) 9. (5) 13. (5) 17. (3) 21. (3) 25. (2) 29. (4) 33. (1) 37. (3) 41. (3) 45. (2) 49. (5) 53. (1) 57. (5) 61. (1) 65. (2) 69. (3) 73. (2) 77. (5) 81. (3) 85. (5) 89. (4) 93. (2) 97. (5) 101. (3) 105. (3) 109. (2) 113. (5) 117. (3) 121. (2) 125. (5) 129. (4) 133. (3) 137. (1) 141. (4) 145. (3) 149. (4) 153. (5) 157. (1)

    2. (4) 6. (3) 10. (1) 14. (2) 18. (2) 22. (1) 26. (5) 30. (2) 34. (4) 38. (1) 42. (2) 46. (3) 50. (2) 54. (4) 58. (1) 62. (5) 66. (4) 70. (2) 74. (1) 78. (4) 82. (4) 86. (3) 90. (2) 94. (4) 98. (2) 102. (5) 106. (4) 110. (1) 114. (4) 118. (4) 122. (1) 126. (4) 130. (1) 134. (1) 138. (2) 142. (1) 146. (4) 150. (4) 154. (2) 158. (4)

    3. (1) 7. (5) 11. (4) 15. (2) 19. (1) 23. (4) 27. (1) 31. (1) 35. (2) 39. (4) 43. (1) 47. (4) 51. (4) 55. (2) 59. (2) 63. (3) 67. (1) 71. (2) 75. (4) 79. (1) 83. (2) 87. (3) 91. (1) 95. (1) 99. (1) 103. (5) 107. (1) 111. (4) 115. (1) 119. (5) 123. (2) 127. (3) 131. (2) 135. (5) 139. (2) 143. (5) 147. (5) 151. (1) 155. (4) 159. (5)

    4. (2) 8. (4) 12. (3) 16. (4) 20. (2) 24. (3) 28. (3) 32. (3) 36. (5) 40. (1) 44. (4) 48. (1) 52. (4) 56. (3) 60. (4) 64. (2) 68. (5) 72. (5) 76. (3) 80. (2) 84. (1) 88. (3) 92. (5) 96. (3) 100. (4) 104. (2) 108. (5) 112. (3) 116. (2) 120. (3) 124. (2) 128. (5) 132. (3) 136. (4) 140. (3) 144. (2) 148. (1) 152. (3) 156. (2) 160. (3)

    161. (4) 165. (5) 169. (2) 173. (5) 177. (3) 181. (5) 185. (5) 189. (1) 193. (4) 197. (4) 201. (5) 205. (2) 209. (4) 213. (5) 217. (4) 221. (4) 225. (4) 229. (3) 233. (5) 237. (2) 241. (1) 245. (5) 249. (5) 253. (4)

    162. (3) 166. (4) 170. (5) 174. (2) 178. (2) 182. (2) 186. (3) 190. (2) 194. (1) 198. (5) 202. (2) 206. (5) 210. (4) 214. (2) 218. (5) 222. (2) 226. (2) 230. (3) 234. (4) 238. (4) 242. (3) 246. (3) 250. (2) 254. (1)

    163. (2) 167. (3) 171. (1) 175. (4) 179. (5) 183. (1) 187. (1) 191. (4) 195. (3) 199. (3) 203. (4) 207. (3) 211. (5) 215. (1) 219. (3) 223. (1) 227. (5) 231. (2) 235. (1) 239. (5) 243. (2) 247. (4) 251. (3)

    164. (1) 168. (1) 172. (3) 176. (4) 180. (1) 184. (4) 188. (3) 192. (5) 196. (2) 200. (2) 204. (5) 208. (1) 212. (3) 216. (2) 220. (3) 224. (5) 228. (1) 232. (1) 236. (3) 240. (4) 244. (5) 248. (1) 252. (5)

    RBI GRADE–B/ NABARD GRADE–A OFFICER EXAMS 1. (3) 5. (2) 9. (4) 13. (3) 17. (2) 21. (3) 25. (4) 29. (2) 33. (5) 37. (3) 41. (4) 45. (3) 49. (4) 53. (2) 57. (1) 61. (4) 65. (3) 69. (3) 73. (4) 77. (3) 81. (1) 85. (3)

    2. (5) 6. (5) 10. (1) 14. (4) 18. (3) 22. (1) 26. (2) 30. (3) 34. (2) 38. (2) 42. (3) 46. (5) 50. (1) 54. (5) 58. (4) 62. (2) 66. (1) 70. (1) 74. (3) 78. (2) 82. (5) 86. (4)

    3. (1) 7. (3) 11. (4) 15. (1) 19. (1) 23. (2) 27. (5) 31. (4) 35. (4) 39. (5) 43. (1) 47. (4) 51. (2) 55. (3) 59. (1) 63. (4) 67. (4) 71. (5) 75. (2) 79. (1) 83. (2) 87. (2)

    BPRE–374

    4. (4) 8. (4) 12. (3) 16. (4) 20. (5) 24. (3) 28. (1) 32. (3) 36. (1) 40. (1) 44. (2) 48. (5) 52. (3) 56. (3) 60. (5) 64. (2) 68. (2) 72. (1) 76. (4) 80. (5) 84. (4) 88. (5)

    89. (1) 90. (3) 91. (1) 93. (4) 94. (5) 95. (3) 97. (4) 98. (3) 99. (1) 101. (4) 102. (2) 103. (5) 105. (3) 106. (5) 107. (2) 109. (1) 110. (2) 111. (5)

    92. (2) 96. (5) 100. (2) 104. (1) 108. (4) 112. (4)

    113. (1) 114. (4) 115. (3) 116. (1) 117. (2) 121. (5) 125. (2) 129. (3) 133. (5)

    118. (5) 122. (1) 126. (4) 130. (1) 134. (3)

    119. (4) 123. (4) 127. (5) 131. (4) 135. (2)

    120. (3) 124. (2) 128. (1) 132. (2)

    INSURANCE EXAMS 1. (3)

    2. (5)

    3. (2)

    5. (4)

    6. (3)

    7. (2)

    4. (1) 8. (1)

    9. (5)

    10. (5)

    11. (3)

    12. (2)

    13. (1)

    14. (3)

    15. (4)

    16. (1)

    17. (3)

    18. (2)

    19. (4)

    20. (2)

    21. (4)

    22. (1)

    23. (3)

    24. (1)

    25. (3)

    26. (2)

    27. (3)

    28. (3)

    29. (1)

    30. (2)

    31. (1)

    32. (1)

    33. (4)

    34. (2)

    35. (4)

    36. (3)

    37. (5)

    38. (2)

    39. (1)

    40. (3)

    41. (5)

    42. (1)

    43. (4)

    44. (2)

    45. (5)

    46. (3)

    47. (1)

    48. (5)

    49. (4)

    50. (2)

    51. (1)

    52. (3)

    53. (2)

    54. (4)

    55. (5)

    56. (2)

    57. (1)

    58. (3)

    59. (4)

    60. (3)

    61. (1)

    62. (4)

    63. (5)

    64. (2)

    65. (4)

    66. (3)

    67. (2)

    68. (1)

    69. (5)

    70. (3)

    71. (5)

    72. (4)

    73. (1)

    74. (2)

    75. (3)

    76. (2)

    77. (5)

    78. (4)

    79. (1)

    80. (1)

    81. (2)

    82. (5)

    83. (3)

    84. (4)

    85. (2)

    86. (1)

    87. (3)

    88. (5)

    89. (4)

    90. (3)

    91. (4)

    92. (5)

    93. (1)

    94. (2)

    95. (3)

    96. (2)

    97. (5)

    98. (1)

    99. (4) 100. (5)

    101. (2) 102. (1) 103. (3) 104. (4) 105. (1) 106. (3) 107. (4) 108. (5) 109. (2) 110. (2) 111. (4) 112. (1) 113. (5) 114. (3) 115. (5) 116. (2) 117. (3) 118. (1) 119. (4) 120. (2) 121. (1) 122. (3) 123. (2) 124. (4) 125. (5) 126. (3) 127. (4) 128. (2) 129. (4) 130. (2) 131. (4) 132. (3) 133. (1) 134. (5)

    RANKING/ARRANGEMENT

    NATIONALISED BANKS & IBPS PO/MT/SO EXAMS 1. (1) Manav‘s rank from the top = 15 – 4 = 11th Manav‘s rank from the bottom = 20 – 11 + 1 = 10th 2. (4) V is the heaviest. 3. (5) V > X > U > Y > Z > W ↓ ↓ 58 kg 35 kg U or Y weigh more than 35 kg and less than 58 kg. 4. (1) Z is heavier than only W. (5–7) : Sitting arrangement

    left of the second person. In XA, the first person is to the immediate right of the second person. 13. (1) Only one person X, is seated between A and C. 14. (2) A is to the immediate left of X. Z is to the immediate left of A. Similarly, Y is to the immediate left of B. (15-20) : Sitting arrangement

    M ale, Le cturer Q (W ife of S ) F em a le

    S cie ntist, V M ale

    P

    M ale, D oc to r S

    La wy er, M a le R J (W ife of R ) F em a le

    O R

    L

    M

    P

    Q

    N

    5. (5) P is to immediate right of N. 6. (3) If we count anti-clockwise from L to Q, then three persons– O, R, M – are seated between L and Q. 7. (2) N is the immediate left of P. 8. (5) Except in ROL, in all others the third person is seated between the first and the second persons. In ROL, the second person is seated between the first and the third persons. 9. (1) Except in LP, in all others the first person is to the immediate left of the second person. In LP, the first person is to the immediate right of the second person. (10-14) : L Z E F T

    A

    X

    C

    Y

    (W ife of Q ) U F em a le T (W ife of V ) F em a le

    B R I G H T

    10. (4) X and C are sitting exactly in the middle of the line. 11. (4) X is second to the right of Z. 12. (5) Except in XA, in all others the first person is to the immediate

    15. (4) P is to the immediate left of S. 16. (1) J is third to the left of T. 17. (2) T is the wife of V. 18. (3) Wives of Q and V are immediate neighbours. 19. (3) Except in UTV, in all others the third person is sitting between the first and the second persons. In UTV, the second person is sitting between the first and the third persons. 20. (3) R is a lawyer. J is wife of R. (21-25) : Sitting arrangement A Businessman G Clerk

    D Doctor F Architect

    E Banker B Shopkeeper

    C Engineer

    H Teacher

    21. (3) Statement (3) is true. 22. (4) H is a Teacher. 23. (2) D, Doctor is third to the left of Banker E. 24. (4) Doctor D, sits between F, Architect and A, Businessman. 25. (5) G is a clerk.

    BPRE–375

    (26-30) : Std 5 C Std 7

    B Std 3

    F

    Std 2 D

    E Std 6

    A Std 4

    G Std 1

    H Std 8

    26. (5) None is true. 27. (4) B studies in Std 3. 28. (4) There are three students between A and D when counted from the left hand side of A. 29. (2) D studies in Std 2. 30. (3) E and H are immediate neighbours of G. (31–36) :

    Captain of New Zealand W Captain of South Africa

    EXPLANATIONS

    Captain of Pakistan S

    X

    Captain R of England

    Sri Lankan V Captain

    T

    P Australian Captain

    Indian Captain

    Q Captain of West Indies

    31. (3) There are two persons – Q and P – between T and R, the captain of England when counted in clockwise direction from T. 32. (1) P is the Captain of Australian team. 33. (2) +3 +4 +5 +6 V  → X  → T  → R  → W +3 +4 +5 +6 S  → R  → V  → P  → X

    34. (3) R is the Captain of England. W is immediate neighbour of S or X. Captains of Australia (P) and England (R) are immediate neighbours. Three people sit between W and Q. X sits second to the right of S. 35. (4) T is the Indian Captain. 36. (4) Q is the Captain of West Indies. Q is second to the right of R.

    RANKING/ARRANGEMENT (37–43) : Pilot G Engineer C

    Architect B

    H Doctor

    Banker D

    E Politician

    F Teacher A Businessman

    37. (1) A is a Businessman. 38. (3) E is the Politician. F is second to the right of E. 39. (3) Doctor (H) sits exactly between the Teacher (F) and the Engineer (C). 40. (5) Doctor (H) sits second to the right of Businessman (A). 41. (2) G is a Pilot. 42. (4) Only the combination E-Politician is correct. 43. (4) E is the Politician. (44–48) :

    Dena Bank

    Uco Bank F C

    G Bank of India H Canara Bank

    Punjab E National Bank Oriental Bank of Commerce

    D

    A B Syndicate Bank

    46. (3) H, the representative from Canara Bank and A, the representative from Bank of Maharashtra, sit between B, the representative from Syndicate Bank and G, the representative from Bank of India. 47. (5) D is the representative from Oriental Bank from Commerce. 48. (4) E, the representative from Punjab National Bank sits second to the left from B, the representative from Syndicate Bank. W

    (49-53) : P

    S

    Y

    V

    T

    Q

    R

    49. (3) T sits second to the left of Q. 50. (4) T is third to the right of V. 51. (4) R, W, V and Y are sitting in the middle of the sides of the table. S is sitting at the corner. 52. (2) WP and TR represent neighbours. There are two persons between Q and W as well as R and S. Now, we have to choose such pair in which there would be three persons between the two. 53. (5) None of the statements is true. (54–60) :

    Bank of Maharashtra

    44. (2) Except in A-Canara Bank pair, in all others the first person is second to the left of the second person. 45. (5) B is the representative from Syndicate Bank. C sits third to the right of H. The representative from the Dena Bank, C is to the immediate right of the representative from the UCO Bank, F. A is second to the left of G, the representative from Bank of India. A, the representative from Bank of Maharashtra and B, the representative from Syndicate Bank are immediate neighbours of each other.

    L E F T

    R I G H T P

    T

    Q

    V

    S

    R

    C

    D

    E

    F

    A

    B

    L E F T

    R I G H T

    54. (3) R and P sit at extreme ends of the rows. 55. (5) S faces A. 56. (2) Two persons – Q and V – are seated between T and S 57. (1) There are two persons between P and V. 58. (5) F faces V. 59. (4) T sits exactly between P and Q. 60. (4) V > Q > R T > R and S > V

    BPRE–376

    Now, S > V > Q > R T (61-65) : H

    E

    N

    G

    B

    A

    C

    W

    E S

    D

    F

    61. (5) C is sitting opposite to A. 62. (2) B is next to E in clockwise direction. 63. (4) A and G are sitting on the same side and A is to the right of G. 64. (3) F is sitting opposite to E. 65. (2) E is sitting opposite to F. (66–70) :

    Row-1

    R I G H T

    L E F T

    V E

    T B

    Q A

    R C

    L E F T

    P D

    S F

    R I G H T

    Row-2 66. (2) To persons – A and C – are seated between B and D. 67. (1) V and B are opposite diagonally. Therefore, P is related to F. 68. (4) V sits at one of the extreme ends of the line. 69. (2) V and S are at extreme ends of the Row-1. 70. (5) T faces B. (71-76) : ROW-1 R I G H T L E F T

    L E F T

    D

    B

    F

    C

    E

    A

    T

    P

    S

    R

    Q

    V R

    ROW-1

    I G H T

    71. (1) T faces D. 72. (2) T and A are sitting at extreme ends of the rows. 73. (5) D is far away from R. 74. (2) Except S, all other are sitting at the extreme ends of the rows.

    RANKING/ARRANGEMENT

    Left R

    Q

    P

    S

    T

    C

    D

    B

    E

    A

    Right

    Left

    Right

    80. (3) Two persons – P and S – are seated between Q and T. 81. (2) Except S, all others are seated at the ends. 82. (5) P and B are sitting exactly in the middle of rows. 83. (5) D and E are immediate neighbours of B. B sits exactly in the middle of the row. P faces B. Either Q or S is an immediate neighbour of P, who faces B. 84. (4) R and A are sitting opposite diagonally. 85. (5) E faces S. (86 – 92) : Ketan

    Peter

    Ali

    91. (5) No one sits between Peter and Ali. 92. (5) Shikha and Bharat are immediate neighbours of Meenal. (93–95) :

    Harleen

    Shikha

    Bharat

    Meenal

    86. (3) Except Harleen, all others sit in the middle of the sides. 87. (1) Bharat sits third to the left of Ali. 88. (4) Peter is third to the right of Meenal. 89. (4) Harleen sits second to the right of Ketan. 90. (2) Bharat and Rumia are immediate neighbours of Harleen.

    4th 2 Girls, 3 Boys A

    10th S

    Among girls, Anjali’s rank is 3rd from the top. So, before Anjali there are two girls and one boy. Among boys, Sameer’s rank is 5th from the top. So, there are three boys and two girls in between Anjali and Sameer. 102. (1) 8 1 9 4 3 2 7 5 3 2 7 5 8 1 9 4 103. (4) According to the question, the new sequence would be : 1 3 9 5 1 5 1 5 5 3 9 9 3 1 5 7 7

    7th from the right end

    BPRE–377

    104. (3) Odd digit 2 Odd digit Such combinations are : 923 ; 321

    105. (1) Perfect Square

    93. (4) S lives on the 4th floor. 94. (1) T lives on the 1st floor. 95. (2) V lives on the 2nd floor and P lives on the 5th floor. 96. (3) S is sitting at the extreme left end. 97. (5) Except in the case of Q – 6th floor, in all others the floor number is one more than the actual floor number. In case of Q, the floor number is one less than the actual floor number. 98. (5) S lives on the fourth floor and he sits at the extreme left end. 99. (5) P ⇒ 1st floor Q ⇒ 2nd floor R ⇒ 3rd floor S ⇒ 4th floor T ⇒ 5th floor V ⇒ 6th floor W ⇒ 7th floor (100–102) : The rank of Smeer from the top ⇒ 10th The rank of Anjali from the top ⇒ 4th Total number of girls = 3 + 7 – 1 =9 Total number of boys = 5 + 21 – 1 = 25 100. (3) Total number of students = 9 + 25 = 34 101. (3) 2 Girls, 1 Boy

    Rumia

    1st 3rd 5th 6th W T R P

    Right

    4th 2nd 7th V S Q left

    75. (2) Two persons – P and S – are seated between R and T. 76. (3) E faces Q. 77. (4) D and F are immediate neighbours of B. (78–79) : Ajay > Poonam > Ben Kim > Shreya Kim > Shreya > Ajay > Poonam>Ben 87% 70% 65% 78. (5) Poonam scored the second lowest narks. 79. (3) Shreya may have scored 82% marks. (80-85) :

    1

    There is no such combination. 106. (3)

    4 2 6 1 3 8 +1 +1 +1 +2 +2 +1

    5 3 7 3 5 9 107. (1) 7th to the right of the 19th digit from the right end means 12th from the right end, i.e., 3. 108. (5) 394 ⇒ 934; 632 ⇒ 362; 783 ⇒ 873; 576 ⇒ 756; 895 ⇒ 985 Second Highest Number

    ⇒ 934 ⇒ 394 109. (1) 8 1 9 4 3 2 7 5 3 2 7 5 8 1 +2

    –4

    +2

    –4

    9 4

    → E   → R 110. (4) T   U  → 4   → @ –4 +2 6  → D   → V –2

    +4

    +2

    –4

    J  → 1   → K N  → δ   → ∗ 111. (5)

    Letter

    Number

    Number or Symbol

    Such combinations are :

    M3 # ; D8 ★ ;

    U54 ;

    16 ;

    N 7δ

    112. (2) +6

    +6

    +6

    R  → $  → 4 → 8 +6 +6 +6 T  →@  → I  → H +6 +6 +6 1  → F  → 6  →

    N

    113. (3)

    Number

    Symbol

    Letter

    Such combinations are :

    3#R ;

    8∗H

    114. (5) 7th to the left of 20th from the left end means 13th from the left end, i.e., F.

    RANKING/ARRANGEMENT (115-121) :

    Oriental Bank of Commerce

    (D’s Daughter)

    L E F T

    H

    (122–124) : B, D, F > C > A, E B > D > F > C > A, E B>D>F>C>E>A Score of F = 81 Score of E = 62 122. (1) C scored more than 62 and less than 81. 62 < 70 < 81 123. (5) D’s score was more than 81. B scored the maximum marks. Three people, B, D and F scored more than C. B scored more than 81 marks. 124. (3) Score of F = 81 Socre of B = 81 + 13 = 94 Therefore, score of D = more than 81 and less than 94 81 < 89 < 94

    L E F T

    A

    B

    D

    Q

    S

    R

    9th 7th M

    Girls

    S R 11th Eight girls are standing to the right of Meena. (137–141) : T V

    UCO Bank

    115. (4) A’s wife is H. H’s mother is B. Therefore, B is the motherin-law of A. A is second to the right of B. 116. (3) E is the daughter of D. 117. (2) A is the father of F and G. G is the mother of C. Therefore, C is the grandchild of A. A is third to the right of C. 118. (1) D is brother-in-law of A. A is father of F and G. Therefore, D is maternal uncle of G. There is only one person between G and D is clockwise direction. 119. (2) Except C, all others are females. 120. (5) B is the mother of H. C is the nephew of E. A is the husband of H. A is third to the left of H. Both the neighbours of C are females. F and G are daughters of H. 121. (1) B sits to the immediate left of C. B is grandmother of F.

    C P

    Indian Bank

    (Female) F (Female)

    Punjab National Bank

    (Female) G

    E

    Dena Bank

    (Husband of H) A

    R I G H T

    Canara Bank

    (H’s mother) B C (M ale)

    Syndicate Bank

    (M ale) D

    134. (5) P is the second heaviest. 135. (3) Q and U are lighter than S. 136.(1)

    (125–132) :

    R I G H T

    Q

    S

    U

    R 125. (2) The person from Punjab National Bank, Q faces A. B is from Bank of Maharashtra and he is a immediate neighbour of A. A faces the person who sits second to the left of R. A is from Dena Bank. A sits third from the left and second from the right. 126. (5) S is seated between R and Q, the person from Punjab National Bank. 127. (4) Persons at the extreme ends : C from Canara Bank; D from Oriental Bank of Commerce; P from Syndicate Bank; R from UCO Bank. 128. (1) S from Indian Bank faces B from Bank of Maharashtra. 129. (4) P faces the immediate neighbour of A from Dena Bank. B faces the immediate neighbour of Q from Punjab National Bank. Similarly, D faces immediate neighbour of S from Indian Bank. 130. (4) Except Q, all others are seated at the extreme ends of the lines. 131. (3) P is from Syndicate Bank. 132. (5) C is from Canara Bank. (133–136) : S > Q T, P > R T, P > R > S > Q T>P>R>S>Q>U 133. (4) U is the lightest.

    BPRE–378

    P W

    137. (2) R is second to the right of V. 138. (3) Except TP, in all other pairs, one person is just opposite to the other person. 139. (5) P is sitting to the immediate right of W. Q is sitting second to the right of P. 140. (5) Only P is sitting between U and W, when counted from the left side of U. 141. (4) V is sitting exactly between T and S. (142–146) : L E F T

    G A E B H C F D R

    I G H T

    142. (4) E is third to the left of C. 143. (2) Sixth from the right end ⇒E C is third to the right of E. 144. (1) G and D are sitting at the extreme ends of line. 145. (2) E is the immediate neighbour of B. In all others, the first person is third to the right of the second person. 146. (3)

    G

    A

    E

    B

    H

    C

    F D

    H

    G

    F E

    D

    C

    B

    A

    The position of C is unchanged.

    RANKING/ARRANGEMENT 147. (1) B > A > C, D, E C>D>E B>A>C>D>E A and B are older than C. 148. (3)

    9th B

    12th C 7th

    15th A

    Total number of children = 12 + 7 – 1 = 18 (149–153) :

    L R E F T

    S

    149. (2) R

    X

    P

    S

    W

    T

    P

    X

    T

    T

    S

    Q

    X

    R

    W R I G H T Q

    Q

    W

    P

    150. (1) Four people – T, X, Q and W – sit to the right of P. 151. (5) W is at the extreme right end of the line. 152. (4) R and P are immediate neighbour of S. 153. (2) Fourth person from the right end ⇒ T Second to the right of T ⇒ Q (154–158) :

    2 Girls, 1 Boy

    D

    G

    F

    10th S

    Among girls, Anjali’s rank is 3rd from the top. So, before Anjali there are two girls and one boy. Among boys, Sameer’s rank is 5th from the top. So, there are three boys and two girls in between Anjali and Sameer. 161. (2) The rank of Sunetra from the top = 8th The rank of Sujit from the top = 8 + 5 = 13th Therefore, Sujit’s rank from the bottom = 40 – 13 + 1 = 28th 162. (3) From statement I

    LEFT T | | Q J | | RIGHT

    Q R Q

    LEFT

    H

    4th 2 Girls, 3 Boys A

    There are seven persons in the line. From statement II

    B E

    C sits third to the right of H. Either two or four persons sit between H and D. (159–160) : The rank of Smeer from the top ⇒ 10th The rank of Anjali from the top ⇒ 4th Total number of girls =3+7–1=9 Total number of boys = 5 + 21 – 1 = 25 159. (3) Total number of students = 9 + 25 = 34 160. (3)

    C A

    154. (1) F is an immediate neighbour of both A and H. 155. (1) F is just opposite to D. C is just opposite to E. Similarly, H is just opposite to G. 156. (2) A is to the immediate left of C. 157. (3) D is to the immediate right of G. 158. (5) None of the statements is true. D is an immediate neighbour of B. A is just opposite to B.

    S

    >A>

    R I G H T L E F T

    Row-1 D

    B

    A

    R

    S

    Q

    Row-2

    L E C F T P R I G H T

    166. (2) C faces P. B sits second to the right of C. 167. (3) Except Q, all others are seated at extreme ends of the lines. 168. (2) C sits third to the left of D. P faces C. A is an immediate neighbour of C. P faces C and Q is an immediate neighbour of P. 169. (4) D faces R. 170. (4) S faces B. (171-173) : >S> >S>T>R Either P or Q is of 158 cm. 171. (3) It is clear that T is shorter than S. 172. (5) Q is either 158 cm high or more than 158 cm. 173. (4) Either three or four persons are shorter than Q. P>Q>S>T>R or, Q > P > S > T > R (174-177) : Sitting arrangement

    S RIGHT Male, Lecturer Q

    No answer. (163–165) : C>B>E ,

    (166–170) :

    (Wife of S)

    U

    (Wife of Q) Female T (Wife of V) Female Scientist, V Male

    Female P

    ,

    C>B>A>E>D ↓ ↓ 87% 72% 163. (4) Three people – C, B and A – scored more than E in the Examination. 164. (4) B > A > E ↓ ↓ 87% 72% A scored 81% marks. 165. (5) C scored more than 87% marks.

    BPRE–379

    Male, Doctor S

    Lawyer, Male R J (Wife of R) Female

    174. (4) P is to the immediate left of S. 175. (1) J is third to the left of T. 176. (2) T is the wife of V. 177. (3) Wives of Q and V are immediate neighbours. 178. (3) Except in UTV, in all others the third person is sitting between the first and the second

    RANKING/ARRANGEMENT

    U W

    T U

    182. (1) None (183–187) :

    E

    D

    C

    A

    B

    F

    LEFT

    RIGHT

    ROW-1

    B

    E A A

    S

    P

    R

    V

    T

    RIGHT

    LEFT

    Q

    ROW-2

    183. (2) A faces R. D sits second to the right of A. 184. (1) B sits second to the left of C. A sits to the immediate right of B. V faces B. A and F are immediate neighbours of B. V faces B. R and T are immediate neighbours of V. 185. (3) C faces P. 186. (1) V sits exactly between T and R. 187. (4) Except C, all others are seated at extreme ends of the lines. (188-192) : White B Purple C H Orange Blue D E Red

    G Yellow

    A Green

    F Black

    FH

    192. (5) E likes red colour. A sits exactly between E and F. (193–198) :

    H G

    A

    B

    LEFT P

    U

    S

    Q

    RIGHT

    R

    199. (3) F and B are sitting at the extreme ends of Row-1. 200. (1) S is third to the right of R. 201. (5) Except in EU, in all others one person is just opposite to the other person. 202. (2) A is second to the left of E. 203. (4)

    F

    C

    E

    D

    A

    B

    F

    E

    D

    C

    B

    A

    (204-208) : C F L E F T

    D

    C

    T

    Row-2

    F

    G G

    C D

    R I G H T

    B

    LEFT

    W V Y X

    H

    A

    E/F E/F

    193. (2) C is third to the left of D. 194. (4) H is sitting to the immediate right of G. 195. (1) B and G are immediate neighbours of C. 196. (5) Either E or F is sitting third to the left of G. 197. (3) There are three people between B and D.

    G

    D

    H

    E

    C D (209–213) :

    E

    F

    G

    H

    Row–2

    198. (2)

    N O

    Q/T

    M

    R I G H T

    Q/T P

    R S

    N is sitting third to the right of R.

    BPRE–380

    R I G H T

    204. (3) G is third to the left of E. 205. (2) C and E are standing at the extreme ends of the line. 206. (3) G is standing second to the right of C. 207. (2) Except in GE, in all others there is only one person between the two. 208. (1) C F G D H E

    L E F T

    Pilot

    X V

    C

    D

    X

    W

    Y

    Z

    G

    H

    F

    E

    Row–1

    Doctor

    Y Z

    D

    E B

    E

    Scientist

    Z T

    L E F T

    191. (2)

    C

    Engineer

    179. (3) Z is fourth to the left of W. 180. (4) V is second to the right of T. 180. (2) Except in XV, in all others first person is to the immediate right of the second person.

    F

    Doctor

    U R I G H T

    W

    Pilot

    X

    Engineer

    Y

    Scientist

    V

    Row-1 RIGHT

    Z

    (199-203) :

    LEFT

    L T E F T

    188. (3) There are four persons – G, F, A and E – between H and D, if we go clockwise from H to D. 189. (1) H and F are immediate neighbours of G. F likes black colour. 190. (4) H likes orange colour. E is fourth to the left or right of H.

    RIGHT

    persons. In UTV, the second person is sitting between the first and the third persons. (179–182) :

    L E F T

    R I G H T

    RANKING/ARRANGEMENT (225-229) :

    G and E sit at the extreme ends of the Row–1.

    Motorola

    C

    63kg

    G

    The Engineer of one row faces the Scienctist of another row.

    E LG

    213. (2) Z faces E. H interchanged position with Z. So, H faces E. (214-219) :

    B

    A Samsung

    H Nokia

    225. (3) D represents Sony Company. 226. (1) B is third to the left of C. 227. (4)

    J H F K I E L G

    E F

    214. (2) E and G are immediate neighbours of L. 215. (3) Five persons – F, K, I, E and L – are standing between H and G.

    D D

    B

    F

    P/U P/U Q R T S 235. (5) Either P or U is the heaviest. 236. (2) T is heavier than S but lighter than R. 237. (4) Q > R > T ↓ ↓ 63 kg 50 kg So, R may weigh 58 kg. 238. (4) G is sitting between B and C. 239. (2) A is sitting to the immediate left of D. 240. (5) C is third to right of D. H is third to right of C. E is third to right of G. 241. (3) There are three persons – A, D, B – between F and G. 242. (1) C B

    H

    E B

    P

    W

    Q

    S

    R

    T

    R I G H T

    220. (2) R is fourth to the right of P. 221. (3) Except in TR, in all others the first person is to the immediate left of the second person. 222. (5)

    L V P E F T W V

    W

    T

    Q

    S

    S

    R

    R

    Q

    T

    R I G H P T

    223. (4) Q sits exactly in the middle of the line. 224. (2) P sits to the immediate left of W.

    H H

    A A

    228. (2) Except AE, in all others one person is sitting just opposite to the second person. 229. (5) All statements are true. (230-234) : Floor Number 7 6 5 4 3 2 1

    Person

    Destination

    V Q U P T R S

    Jaipur Mumbai Patna Delhi Kolkata Bangalore Chennai

    230. (3) V lives on the topmost floor. 231. (1) Except R, all others live on odd numbered floors. 232. (5) P is travelling to Delhi. 233. (2) Four persons live between the floors of Q and S. 234. (4) P lives on the floor immediately above the floor of T.

    BPRE–381

    C E

    H

    219. (3) L is third to the right of K.

    L V E F T

    F I

    D G

    217. (4) E is fourth to the right of H.

    (220-224) :

    A A

    B D

    G G

    C C

    216. (1) J is facing south and he is at the extreme end. 218. (5) Except I, all others are facing North.

    50kg

    ax m

    212. (1) Y sits second to the left of X. F faces Y.

    cro Mi

    F

    Sony D

    210. (4) H is a Pilot. G sits to the immediate left of H. 211. (5) W (Doctor) and Z (Pilot) are immediate neighbours of Y.

    (235-237) : Q>R>T P, U > Q

    Apple

    Reliance

    209. (3) X and Z sit at the extreme ends of the Row–2.

    (243–247) : M Z L

    E G

    I F

    J

    W

    Y

    K

    X

    243. (3) M, J, Y, K and X are facing South. 244. (1) There are three persons (L, J, W) between Z and Y. 245. (4) J is to the immediate left of W. 246. (3) Except L, all others are facing South. 247. (2) M is standing at one of the ends. (248–253) : Pilot Q

    Doctor V S Engineer

    Lawyer T

    W Chef

    P Manager U Teacher R Architect

    RANKING/ARRANGEMENT 248. (2) V is the Doctor. 249. (5) U sits between R and P (Manager). 250. (1) W is chef. 251. (1) Except the pair SU, in all other pairs the two persons are immediate neighbours. 252. (4) W sits second to the right of Q. 253. (1)

    U

    V V

    260. (2) V is the Doctor. 261. (5) U sits between R and P (Manager). 262. (1) W is chef. 263. (1) Except the pair SU, in all other pairs the two persons are immediate neighbours. 264. (4) W sits second to the right of Q. 265. (1)

    S W

    Q

    V V

    U

    S W

    Q

    P P

    T T

    P P

    T T S

    U

    W R R

    Q

    U

    W

    S

    R R

    (254–259) :

    S

    Q

    V

    R

    T

    R

    P

    U

    K L

    P

    S Engineer P Manager U Teacher R Architect

    T

    266. (1) U is sitting to the immediate left of Q. P sits between V and U. 267. (2) P sits exactly in the middle of the line. 268. (2) S faces North. S is at one of the ends. U sits to immediate left of P. 269. (5) S and T are sitting at the two extreme ends of the line. 270. (2) V sits second to the right of S. 271. (1) U and T are immediate neighbours of Q. (272–276) :

    Doctor V

    Lawyer T

    W Chef

    Q

    V

    U

    254. (1) U is sitting to the immediate left of Q. P sits between V and U. 255. (2) P sits exactly in the middle of the line. 256. (2) S faces North. S is at one of the ends. U sits to immediate left of P. 257. (5) S and T are sitting at the two extreme ends of the line. 258. (2) V sits second to the right of S. 259. (1) U and T are immediate neighbours of Q. (260–265) :

    Pilot Q

    Q

    (266–271) :

    S

    P

    272. (2) K and L face the same direction. Q and M face the opposite directions. P and Q face the same direction. N and J face the same direction. Q and L face the same direction. 273. (5) When counted from the right of Q, two persons – M and J – are sitting exactly between Q and O. 274. (1) N is an immediate neighbour of both P and Q. 275. (3) N faces towards the centre. L sits third to the right of N. 276. (4) K and N are immediate neighbours of P. (277–281) :

    N

    O

    Q

    J M

    BPRE–382

    R I G H T J L W E F T

    Row–1

    N

    K

    L

    V

    X

    Z

    Row–2

    L E F M T Y R I G H T

    277. (2) V is facing N. 278. (4) M faces one of the immediate neighbours of Z. L is an immediate neighbour of M. Two persons – L and K – sit between M and N. 279. (1) K is facing X. 280. (3) Z is to the immediate left of Y. 281. (5) Except N, all others sit at the extreme ends of the rows. (282–286) : R I G H T L E F T

    Row–1

    L E F T

    C

    D

    B

    E

    F

    U

    S

    R

    V

    T R I

    Row–2

    G H T

    282. (5) E is facing V. 283. (4) U is facing C. 284. (3) D is sitting third to the right of F. 285. (2) B sits exactly in the middle of the line. B faces R. R is an immediate neighbour of both S and V. Only E sits between B and F. 286. (1) R sits second to the right of U.

    RANKING/ARRANGEMENT

    L

    N

    K

    Row–1

    R I G H T

    287. (1) U is facing M. 288. (3) Except in the pair M and L, in all other pairs the first person sits to the immediate left of the second person. M sits second to the left of L. 289. (4) U sits third to the right of S. 290. (2) N sits second from the right end. Only two persons– J and L – sit between N and M. N faces immediate neighbour of T or V. L sits to the immediate left of N. 291. (5) L is facing T. (292–296) :

    F D’s Husband D

    X

    Row–2

    S D’s Son E D’s Father

    292. (2) S is the grandson of R. P is the daughter of D. E sits third to the left of P. R is an immediate neighbour of both P and S. 293. (4) C is the brother of D. 294. (3) Q sits second to the right of P. R sits second to the right of E. D sits second to the right of Q. 295. (1) S is the son of D. E sits to the immediate left of S. E is the father of D. 296. (5) Q is the sister of D. P is the daughter of D. Therefore, Q is the aunt of P.

    R I G H T

    297. (1) G is facing V. 298. (3) T faces H. T sits at the extreme left end of the row. U sits third to the right of T. 299. (4) D sits third to the left of F. 300. (5) Except U, all others are seated at the extreme ends of the rows. 301. (2) W is facing F. (302-306) :

    M

    M’s S brother

    L M ’s daughter T M ’s father

    M ’s Q sister M ’s K wife

    D’s Daughter D’s Brother P C R D’s Mother Q D’s Sister

    D

    U

    J M ’s son

    R M’s mother

    302. (2) S is the brother of M. 303. (4) L is the daughter of M. J is the son of M. L is second to the right of J. 304. (1) K is the wife of M. 305. (3) M’ s father T is sitting to the immediate right of M’s son J. Only three persons are sitting between S and J. M’s wife K is sitting just opposite to M’s daughter L. 306. (5) Q is the sister of M. (307–311) :

    Row-2 R I G H T

    W E

    X H

    L E F T

    Doctor

    J

    E

    V

    Z

    Y

    F

    G

    L E F T

    Technician

    M

    L E F T

    G

    W

    Professor

    V

    F

    T

    Doctor

    S

    L E F T

    H

    L E F T

    Leader

    T

    L E F T

    Row–1

    Professor

    R

    Row–2

    R I G H T

    307. (3) L sits to the immediate right of O. Only one person sits between P and O. O faces one of the immediate neighbours of C or A. 308. (5) N is facing A. 309. (1) E is facing M. 310. (4) Except PO, in all other pairs, the two persons are immediate neighbours of each other. There is one person between O and P. 311. (2) C sits third to the left of B. (312-316) : Technician

    R I G H T U

    (297-301) :

    Leader

    S sits to the immediate left of R. V sits to the immediate left of T. E sits to the immediate left of B. D sits to the immediate left of C. (287–291) :

    Row-1 312. (4) W is a Leader. X (Technician) and Z (Professor) are immediate neighbours of W. 313. (5) H is the Leader of Row-1. H sits at the extreme left end of the row. 314. (3) Y sits to the immediate left of Z. Technician of Row-1, G faces the Doctor of Row-2, Y. X faces H. H is a Leader. 315. (1) E faces W. If E interchanges place with X, then X would face W. 316. (2) G and H sit at the extreme ends of the Row-1. X and Y sit at the extreme ends of the Row-2. (317-321) :

    W R I G H T M E L E F T

    L E F T

    Row-2

    L C

    O D

    N A

    Row-1

    BPRE–383

    B

    Y

    P

    Q

    P R I G H T

    R I G H T

    S

    Z

    R X

    RANKING/ARRANGEMENT 317. (1) Both R and W face outside. In all other pairs, both the persons face opposite directions. 318. (2) When counted from the left of X, S and Z are sitting between X and P. 319. (4) W and Q are immediate neighbours of Y. 320. (3) Q is second to the left of W. 321. (5) P and Z are immediate neighbours of S. S faces towards the centre. Three persons sit between S and Q. X sits second to the left of S. (322-327) : W

    Y

    U

    S

    T

    V

    Z

    X

    322. (4) W and X are seated at the two extreme ends of the line. Y is immediate neighbour of W and Z is immediate neighbour of X. 323. (5) Two persons – V and Z – are seated between T and X. 324. (1) W

    Y

    U

    S

    T

    V

    Z

    X

    Z Y X W V U T S 325. (2) V sits exactly between Z and T.

    14

    326. (5)

    15th

    19th

    N

    N

    7

    8th

    Total number of children in the row = 19 + 8 – 1 = 26 D

    327. (2) B/G

    B/G

    H

    E

    C

    F A

    A is to the immediate right of (328–332) : R P U

    328. (3) When counted from the right of R, three persons – P, V and W – are seated between R and T. 329. (4) R and V are immediate neighbours of P. Either two or four persons sit between P and S. P sits at the corner. 330. (5) V sits fifth to the left or third to the right of Q. 331. (1) Except Q, all others are seated in the middle of the sides. 332. (2) Q sits second to the left of W. (333–337) :

    R I G H O T L W E F T

    Row–1 J

    Z

    L

    K

    U

    V

    N

    Y

    338. (4) N is sitting second to the left of L. 339. (3) Only three persons sits between R and S. Q sits second to the left of R. R is an immediate neighbour of both M and N. 340. (1) Only three persons are sitting between R and S either clockwise or anticlockwise. 341. (2) R sits second to the left of O. 342. (5) Except M, all others are seated in the middle of the sides. (343–347) :

    L E M F T

    R I G H T O

    R I G H T

    L A E F T

    X

    Row–2

    333. (1) O, M, W and X are sitting at the extreme ends of the lines. 334. (5) K faces V. 335. (4) K faces one of the immediate neighbours of U or Y. V sits exactly between U and Y. J is an immediate neighbour of both O and L. 336. (2) K sits to the immediate right of N. Y faces N. V is an immediate neighbour of Y. O and L are immediate neighbours of J. Two persons – L and K – sit between N and J. 337. (3) U faces L. Y sits second to the right of U. (338–342) : S L P

    Row -1

    N

    M

    R

    P

    Q

    D

    B

    E

    F

    C

    Row -2

    Q

    O

    V

    Brown M

    Pink R S Blue

    N Purple

    Red Q

    P Orange T White

    Q

    M

    N T

    W

    R

    BPRE–384

    R I G H T

    343. (1) M faces immediate neighbour of both D and E. F sits exactly between E and C. O is an immediate neighbour of R. 344. (2) M faces B. O sits second to the right of M. 345. (4) O and Q are sitting at the extreme ends of Row–1. A and C are sitting at the extreme ends of Row–2. 346. (3) Except PQ, in all other pairs the first person sits to the immediate left of the second person. 347. (5) R faces E. (348–352) :

    O Green S

    L E F T

    348. (3) O likes green colour.

    RANKING/ARRANGEMENT 349. (1) T R

    M

    S S

    M

    N O

    O

    R

    N Q

    P

    Q

    354. (2) W is sitting fourth to the right or left of S. 355. (5) All the statements are true. 356. (1) X likes Purple colour. 357. (4) T likes Pink colour. Y is sitting exactly between W and T. Y likes Green colour. 358. (2) Except W, Z pair, in all others, one person is sitting just opposite to the other person. (359–364) :

    +

    +4

    +4

    –2

    → M  → % G  →   +

    +2

    +2

    +

    Vowel U

    +

    –2 4 4  → 5  → 4   →D %  372. (2)

    Q

    Number Consonant

    There is only one such combination :

    E4N S

    W

    (373–375) : W V

    T

    X

    S

    U

    Y

    V

    W Silver

    Y Green

    Z Blue

    T Pink X Purple 353. (3) X W

    Y

    Z Z

    359. (1) R sits second to the left of W. 360. (2) U faces outside. R and W are immediate neighbours of U. U sits second to the left of T. 361. (2) When counted from the right of R, two persons U and W sit between R and V. 362. (3) Except U, all others face the centre. 363. (4) P and R are immediate neighbours of T. 364. (5) S is sitting to the immediate left of V. (365–369) : L E F T

    W

    V T

    S

    +

    4 –2 4  → J  → H   →S A 

    +2

    P

    R

    Red U

    S

    9th from the right end 371. (4)

    → D   → L   → 7 5 

    Position of S remains unchanged. 350. (4) N likes purple colour. 351. (2) Except QR, in all others, one person is sitting just opposite to the other person. 352. (5) S is sitting exactly between R and N. (353–358) :

    U

    2 & ! 6 @ © # % 4 5 9 3 7

    +

    P

    Y V

    $

    –2 4 4 → B  →@   →F T  

    T

    T

    V Orange

    370. (2) According to question, the new sequence would be :

    U X T

    The positions of W and Z would remain unchanged.

    Q R

    T X U P W V S

    R I G H T

    365. (5) V sits seventh to the right of Q. 366. (2) Q and S are seated at two extreme ends of the line. 367. (3) Three persons – T, X and U – are seated between R and P. 368. (4) T sits fifth to the left of V. Q sits fifth to the left of P. X sits fifth to the left of S. 369. (1) Seventh from the left ⇒ W Fifth from the right ⇒ U P sits exactly between U and W.

    BPRE–385

    373. (5) W and Y are standing at the extreme ends of the line. 374. (2) T faces north. U is standing third to the right of T. 375. (3) Except V, all others are facing north. (376–380) : D G E H A C B F

    376. (3) F sits to the immediate left of B. 377. (2) D and F are seated at the two extreme ends of the line. 378. (5) Except AB, in all other pairs both the persons face the same direction. A faces north while B faces south. 379. (1) H is an immediate neighbour of E and A. Only B sits between C and F. E sits third to the right of C. 380. (4) H and E sit exactly between G and A. (381–385) : U W T Q R P V S

    RANKING/ARRANGEMENT 381. (5) U sits third to the right of Q. 382. (2) Q sits second to the left of W. 383. (3) Both W and R faces south. In all others, one person faces north while the other person faces south. 384. (1) T faces north. U and W sit to the left of T. 385. (4) R and V are immediate neighbours of P. (386–390) :

    Karol Bagh W

    ka

    ar w C

    Z Rohini

    A Lajpat Nagar

    T

    W

    X

    G

    N

    E

    G

    B

    D

    C F

    386. (2) Y belongs to Okhla. 387. (1) A is sitting third to the right of B. 388. (4) When counted in anticlockwise direction from C, four persons — B, Y, X and A — are sitting between C and Z. 389. (5) Except X – Dwarka, in all others one person is sitting just opposite to the second person. 390. (3) The person from Dwarka is sitting exactly between the persons from Karol Bagh and Mehrauli. (391–395) :

    Z

    M

    A

    X Chanakyapuri

    V

    L

    H

    B

    OkhlaY

    J

    396. (5) All the statements are true. 397. (2) H and L are immediate neighbours of J. 398. (3) K is third to the left of L. 399. (4) Except NL, in all other pairs only one person is sitting between the two given persons. 400. (1) Only N sits to the left of G. (401–405) :

    D Saket

    Mehrauli

    D

    (396–400) : I K H

    S

    Y

    U

    391. (2) Four persons — V, Z, W and S — are facing north. 392. (4) Except T, Y, in all other pairs one person is facing north while the other person is facing south. Both T and Y are facing south. 393. (3) X is sitting third to the right of Z. 394. (1) T sits exactly between Z and W. 395. (5) V is sitting second to the right of T.

    401. (3) Three persons sit between C and E. G and C face opposite directions. G and F are immediate neighbours of C. 402. (2) F sits just opposite H. Therefore, three persons sit between F and H. 403. (4) G sits third to the right of E. 404. (5) G sits to the immediate right of A. 405. (1) Except G, all others face towards the centre. (406–411) : Step Person Number 8 N 7 L 6 R 5 P 4 M 3 S 2 O 1 Q

    Experience 18 10 28 15 23 20 13 25

    years years years years years years years years

    406. (4) S is sitting on the Step Number 3. M is sitting on the Step Number 4. Q has experience of 25 years.

    BPRE–386

    407. (2) P has 15 years of experience. P is sitting immediately above M. 408. (5) S is sitting on the Step Number 3. 409. (3) Q is sitting on Step Number 1 and 1 + 2 = 3. R is sitting on Step Number 6 and 6 + 2 = 8. O is sitting on Step Number 2 and M is sitting on Step Number 4. And, 2 + 2 = 4 S is sitting on Step Number 3 and 3 + 2 = 5. N is sitting on Step Number 8 and 8 – 6 = 2. 410. (5) S sits immediately above O. R sits above O. 411. (2) M has 23 years of experience. (412–417) :

    Row -1

    R I G H T J L V E F T

    G

    H

    I

    K

    L

    S

    T

    U

    W

    R

    Row -2

    L E F T R I G H T

    412. (3) T sits second to the right of V. 413. (5) I faces U. K sits between I and L. 414. (1) Except W, all others are sitting at the extreme ends of the rows. 415. (4) H sits to the immediate right of I. H is an immediate neighbour of both G and I. Two persons – I and K – sit between H and L. H faces neighbour of both S and U. 416. (2) W faces K. 417. (4) ↑ T K S B ↑ R ↑ ↑ ↑ ↑ ↑ ↑ ↑ ↑ Sonali is fourth from the beginning of the queue. (418–422) : A N O

    D

    M

    B

    P

    C

    RANKING/ARRANGEMENT 418. (5) C is to the immediate left of M. 419. (2) O is third to the left of P. 420. (1) O is to the immediate left of D. D sits second to the right of A. M sits in the middle of one of the sides. M is an immediate neighbour of both N and C. 421. (4) Except O, all others are sitting in the middle of the sides. 422. (3) M sits to the immediate right of N. When counted from the left of B, two persons P and C sit between B and M. (423–427) : L E F T

    F L H E O K M N G

    R I G H T

    423. (4) Except O, N, in all other pairs three persons sit between the two given persons. 424. (5) N is an immediate neighbour of both M and G. N sits to the immediate right of M. N sits second from the right end. N sits to the immediate left of G. 425. (3) F sits second to the left of H. 426. (2) K and M sit exactly between N and O. 427. (1) Four persons – O, K, M and N – sit between G and E. Three persons – H, E and O – sit between K and L. Five persons – L, H, E, O and K – sit betweent F and M. Only K sits between O and M. Only M sits between K and N. (428–433) :

    Q G

    S

    H

    F

    R T

    E

    428. (3) Clearly, F sits third to the left of G. 429. (5) T is to the immediate right of R.

    430. (1) When counted from the left of T, only E sits between T and F. 431. (2) Either two or four persons sit between E and Q. F sits fourth to the left or right of H. H sits in the middle of one of the sides. 432. (4) It is clear that R sits second to the left of E. G and H are sitting between Q and R. 433. (3)

    ,

    >A>

    ,

    440. (2) Clearly, E sits third to the right of B. 441. (5) D sits sixth to the right of B. In all other pairs, the first person sits to the immediate right of the second person. 442. (4) M sits second to the right of O. M is an immediate neighbour of L. D sits to the immediate left of M. 443. (3) B and N sit exactly between L and P. 444. (1) Three persons – C, O and D – are sitting between E and M.

    D > A, B C>B D>C>A>B>E (434 – 439) : R L E U K I R J T S H G I G F H T T

    434. (1) Only S sits between H and T. Four persons – R, J, T and S – sit between I and H. Two persons – K and I – sit between U and R. Two persons – I and R – sit between K and J. Six persons – I, R, J, T, S and H – sit between K and G. 435. (5) S is an immediate neighbour of both T and H. S sits third from the right end. Only two persons – J and T – sit between R and S. 436. (4) S and H sit exactly between T and G. 437. (3) I sits second to the left of J. 438. (2) U and G are sitting at the extreme ends of the line. 439. (3)

    ,

    ,

    >B>

    A>D>B E/C > A > D > B > E/C A scored the second highest marks. (440–444) :

    R L L B N P E C O D M I E G F H T T

    BPRE–387

    Six persons – N, P, E, C, O and D – are sitting between B and M. Four persons – B, N, P and E – are sitting between L and C. Two persons – B and N – are sitting between L and P. Two persons – E and C – are sitting between P and O. (445–449) :

    M

    O

    S

    Q

    T

    N P

    R

    445. (5) P sits second to the right of T. 446. (3) N sits to the immediate right of P. When counted from the left of O, two persons – M and S – sits between O and N. 447. (1) When counted from the right of T, four persons – R, P, N and S – sit between T and M. 448. (2) Q is an immediate neighbour of both T and O. S sits in the middle of one side. Either one or five person(s) is/ are sitting between M and Q. Both R and N face towards the centre. 449. (4) N sits third to the left of O.

    RANKING/ARRANGEMENT

    Floors 10 9 8 7 6 5 4 3 2 1

    Person Abhinav Abhigyan Ankit Anupam Abhijeet Ankur Anoop Abhisar Animesh Abhishek

    Favourite Kabbadi Carrom Football Hockey Kho–Kho Tennis Badminton Wrestling Chess Cricket

    Colour of the shirt Brown Violet Green Black White Blue Red Rose Yellow Sky–blue

    Seating Arrangements :

    459. (3) Y is the brother of D 460. (4) The son of D is sitting third to the right of D. Hence, option (4) is wrong. (461–465) : Brother of F E Female J

    Father of D

    (450–455) :

    Husband of F G

    Wife K of E

    I

    Row (2) Person

    Abhinav Animesh

    floor

    10

    games

    Kabbadi Chess

    South direction Colour North direction Colour games Floor

    2

    Brown

    Yellow

    Violet

    Rose

    Anoop

    Anupam Ankur

    4

    7

    5

    Badminton Hockey

    White

    Green

    Sky-blue

    Carrom Wrestling Kho-Kho Football Cricket 9

    3

    Person Abhigyan Abhisar

    6

    8

    1

    Abhijeet

    Ankit

    Abhishek

    Row (1) 450. (4) Abhisar is sitting to the third left of Abhishek. 451. (4) Ankur and Abhinav are sitting at the ends of the row. 452. (1) Abhisar likes wrestling. 453. (1) The immediate neighbours of the person wearing black shirt likes tennis and badminton. 454. (1) The way Abhishek, the person who likes badminton and the person wearing black shirt respectively live in the first, fourth and the seventh floors and the person who likes chess, person who is wearing blue shirt and Ankit respectively: live in the second, fifth and eight floor, the same way; the person wearing rose colour shirt, the person who likes kho-kho and Abhigyan respectively; stay in the third, sixth and Nineth floor. In each of the combinations, there is a difference of three floors in between the floors of each of the persons.

    455. (4) Anoop, who likes badminton is sitting in the middle of the persons who is facing south dircection in row (2). (456–460) : C, the father of D B, the daughter of D

    Not known

    Not known

    Son of D

    Y, the brother of D

    X, the mother of D

    A, the wife of D

    Z, the sister of D

    456. (2) B is the daughter of D and A is the wife of D. So, A is the mother of B. Two persons are sitting in between B and A. 457. (1) C is the husband of X because C and X repectively are the father and mother of D. 458. (4) As per the given information, the person sitting to the left of Y cannot be determined.

    BPRE–388

    D

    Tennis Blue

    Black

    Red

    Daughter F of H

    H

    461. (4) I is the father of D. When counted from the right of E, only J sits between E and I. 462. (2) When counted from the right of J, then I sits exactly between J and I. 463. (3) J and G are immediate neighbours of E. E sits second to the right of I. D is father of E. 464. (1) F faces outside. 465. (5) I is father of D. Therefore, J would be mother of D. H is the wife of D and hence daughter-in-law of J. E is third to the left of H. 466. (4) 458 ⇒ 854 374 ⇒ 473 654 ⇒ 456 487 ⇒ 784 568 ⇒ 865 865 > 854 > 784 >

    473

    > 456

    473 ⇒ 374 467. (1) 458 ⇒ 548 374 ⇒ 734 654 ⇒ 564 487 ⇒ 847 568 ⇒ 658 548
    S > U, X

    V > W > T > S > UX > UX

    ↓ 92 604. (3) V got more marks than 92. Therefore, possible score of V would be 94. 605. (4) Either four or five persons got more marks than U. 606. (5) V > W > T > S > U > X

    ↓ 92





    69 68

    F

    C

    612. (1) It is clear that there 14 persons in the circular arrangement. 613. (4) Clearly, D is fourth to the right of F. 614. (5) Seven persons are sitting between C and A in anticlockwise direction. (615–619) : Q

    A

    C

    R

    History

    B

    Physics

    D

    Maths

    S

    Biloogy

    P

    Chemistry

    >

    B

    Commerce

    >

    D

    Economics

    601. (4) Four persons sit between Ranjan and Dinesh. 602. (5) Puja is fourth to the right of Ranjan. 603. (1) Nine persons are seated in the row. (604–606) :

    E

    Education

    Dinesh

    Seema

    Ranjan

    L E F T

    Puja

    596. (3) Q likes white. Two persons – M and P – are seated between N and Q. 597. (5) Z faces P. Z likes Grey colour. 598. (4) N and P are immediate neighbours of M. M sits exactly in the middle of Row-1. M likes Red colour. 599. (1) M and V sit exactly in the middle of the rows. 600. (2) V likes Pink colour. (601–603) :

    615. (4) Except AC, in all other pairs both persons are facing the same direction. A faces south while C faces north. 616. (1) D is a Biology student. D is sitting second to the right of P. 617. (2) B is a Commerce student. B faces south. P sits third to the right of B. P is an Education student. 618. (3) P and R are sitting at the extreme ends of the row. P is an Education student.

    BPRE–393

    619. (5) B is sitting between Biology and Maths students. (620–623) :

    C

    F

    Z

    P

    N

    V

    J Q 620. (3) V is fifth to the left of P. 621. (2) Two persons sit between C and P. 622. (4) Q is eighth from the left end. 623. (1) There are 20 persons in the line. Therefore, 15 persons sit to the right of J. 624. (5) B faces outside. H sits third to the right of B. A faces outside. G faces the centre while E faces outside. 625. (3) C faces outside. Only F sits between C and H when counted from the left of C. 626. (2) H faces outside. F sits to the immediate right of H. 627. (4) Except D, all others face outside. D faces the centre. (628–632) :

    R I G H T L E F T

    Row–1

    D

    B

    G

    C

    F

    A

    E

    T

    R

    Q

    U

    P

    S

    V

    Row–2

    L E F T R I G H T

    628. (2) C sits second to the right of A. 629. (3) Only P sits between U and S. 630. (4) Q sits second to the right of T. G faces Q. 631. (5) D and E are sitting at the extreme ends of the row–1. D faces T while E faces V. 632. (1) T faces D.

    RANKING/ARRANGEMENT 636. (3) Two boxes – F and G – are kept between Box A and Box I. 637. (1) Box K is kept just above the Box B. (638–642) F

    (633–637) : Number 1 2 3 4 5 6 7 8 9 10 11

    Box Box Box Box Box Box Box Box Box Box Box Box

    D H A F G I C J E K B

    B

    E

    G

    C

    633. (4) Box I is sixth from the top or bottom, i.e., it is at middle in the stack. 634. (2) Six boxes – A, F, G, I, C and J - are kept between Box H and Box E. 635. (3) Box J is fourth from the bottom. Box K is placed below Box J. There are five boxes between Box H and Box J.

    D

    A

    H 638. (3) E sits second to the left of A. 639. (1) When counted form the left of C, only D sits between C and H. 640. (2) Except C, all others are facing the centre. 641. (5) A is an immediate neighbour of G. 642. (4) All the statements are true.

    (643–647) :

    L E F T

    T Q

    U

    S

    M

    643. (3) 24 persons are sitting in the row. 644. (5) Eight persons are sitting between S and T. 645. (4) U is fifth from the left end. 646. (2) Eleven persons are sitting between Q and M. 647. (1) T sits at the extreme left end. (648–652) :

    Row-2

    P N

    X

    650. (2) U sits fourth to the right of Q. 651. (3) C sits to the immediate right of A. 652. (4) B sits fourth to the left of D. (653–657) :

    Row –1

    R I G H U T L E F T

    B

    V

    Y

    T

    W

    X

    C

    A

    D

    F

    E

    Row –2

    10

    W

    Box

    9

    S

    L E F T

    Box

    8

    R

    Box

    7

    M

    Box

    6

    O

    Box

    5

    Q

    L B E G F D A C E F T Row-1

    R I G H T

    Box

    4

    V

    Box

    3

    P

    Box

    2

    T

    Box

    1

    N

    653. (3) Box M is kept immediately above Box O. Box Q is kept immediately below Box O.

    BPRE–394

    L E F T R I G H T

    658. (2) A faces Y. C sits to the immediate left of A. 659. (5) B faces U. 660. (3) X sits third to the left of Y. X faces E. 661. (4) Except T, all others are sitting at the extreme ends of the rows. 662. (1) Only T sits between W and Y. A sits to the immediate right of C. D faces T. T is an immediate neighbour of Y and W. Therefore, only (i) and (ii) are true.

    B

    663. (1)

    A/F

    A/F Box

    R I G H T U P V S Q R T

    648. (5) Except P, all others are seated at the extreme ends of the rows. 649. (1) Two persons – D and A – sit between F and C.

    R I G H T

    654. (4) Box S is kept at the top of stack. 655. (1) Box O is fifth from the top and sixth from the bottom. 656. (2) V is above T. There is only one box between V and T. R is above O. There is only one box between R and O. M is above Q. There is only one box between M and Q. 657. (5) Box S is kept immediately above Box R. (658–662) :

    D

    C E or

    E C

    D

    A/F

    A/F B

    RANKING/ARRANGEMENT (664–667) :

    (676-680):

    F

    E

    A C

    D B

    1 C

    Orange

    Red

    Blue Yellow

    11

    664. (4) F likes yellow colour. Nine persons sit between A and F. 665. (2) F is to the immediate right of D. B is to the immediate right of D. C is to the immediate right of A. 666. (3) B is second to the right of the one who likes Orange colour. 667. (1) 19 persons are sitting in the line. 668. (1) Box V is placed at the bottom. 669. (4) Four boxes – Q, S, W and U – are placed between X and P. 670. (3) There is only one box between S and X. X is above S. There is only one box between P and W. W is above P. There is only one box between Q and R. R is above Q. 671. (2) Box X is kept immediately above Q. Box S is placed immediately below Q. 672. (5) Box W is fourth from the bottom and sixth from the top. (673-675) :

    LEFT

    F

    8 G

    4 E

    2 H

    B 6

    5 D

    I 9

    J 7

    L 12

    K 10

    A 3

    676. (5) I ⇒ 9th; K ⇒ 10th L sits exactly between I and K. 677. (4) B ⇒ 6th; G ⇒ 8th; E ⇒ 4th Required Average =

    6 + 8 + 4 18 = = 6th 3 3

    678. (3) A sits fifth to the right of C. A ⇒ 3rd 679. (2) E sits opposite J. E ⇒ 4th; J ⇒ 7th 680. (5) D secured 5th rank and sits to the immediate left of J. C secured 1st rank and sits to the immediate left of F. L secured 12th rank. G secured 8th rank. 681. (1)

    F

    9

    C

    8

    D

    7 6

    H

    E

    Ranjan

    5

    B

    A

    4

    G

    3

    Seema

    2

    Puja

    1

    Dinesh

    H sits third to the left of C. (682–686) :

    A

    673. (4) Four persons sit between Ranjan and Dinesh. 674. (5) Puja sits fourth to the right of Ranjan. 675. (3) There are nine persons in the row.

    E B

    G D

    C F

    BPRE–395

    Step Number 8 7 6 5 4 3 2 1

    Person

    Experience

    R P S O Q N L M

    28 23 20 15 13 18 10 25

    years years years years years years years years

    687. (1) M is sitting on Step Number 1. N has 18 years of experience. Q has 13 years of experience. 688. (3) O has 15 years of experience. Three persons – Q, N and L – are sitting between O and M. 689. (2) N is sitting on Step Number 3. 690. (4) Except O-M, in all other pairs one is just above or below the other. 691. (5) S sits immediately below P. S sits immediately above O. R sits above O. 692. (5) M has 25 years of experience. (693–697) :

    R I G H J T L V E F T

    H

    RIGHT

    682. (3) Three persons sit between C and E. G faces outside and C faces the centre. G and F are immediate neigghbours of C. 683. (2) When counted from the left of H, three persons sit between F and H. 684. (1) G sits third to the right of E. 685. (4) G sits to the immediate right of A. 686. (5) Except G, all others are facing the centre. (687–692) :

    Row-1

    G

    H

    S

    T

    I

    K

    L E F L T

    U

    W

    R

    Row-2

    R I G H T

    693. (3) T sits second to the right of V. 694. (5) S faces G. 695. (2) I faces U. K sits exactly between L and I.

    RANKING/ARRANGEMENT 696. (1) Except W, all others are sitting at the extreme ends of the lines. 697. (4) H sits to the immediate right of I. H is an immediate neighbour of both G and I. Two persons – I and K – sit between H and L. H faces T. T is an immediate neighbour of both S and U. (698–702) :

    V U

    P

    703. (4) T is an immediate neighbour of S and Q. S likes Yardley while Q likes Police. 704. (4) A sits to the immediate right of D. T sits to the immediate right of Q. S sits to the immediate right of T, who faces A. 705. (2) R faces E. 706. (3) S faces D. T sits exactly between S and Q. 707. (5) D is second to the left of C. D likes Hollister. (708–712) :

    D O

    G

    B

    E

    A

    H

    715. (1) Except RS, in all other pairs only two boxes are kept between the two given boxes. There are eight boxes between Box R and Box S. 716. (3) Only two boxes are kept between Box J and Box F. 717. (2) Box W is immediately above Box A. Box S is immediately below Box A. (718–723) :

    S D

    T A

    Q C

    Row-1

    R E

    P B

    L E F T R I G H T

    5 Box M 6 Box H 7 Box F 8 Box T 9 Box W 10 Box A 11 Box S 713. (4) Box R is second from the top in the given stack. 714. (5) Seven boxes are kept between Box D and Box S.

    BPRE–396

    Y Latte

    Z Espresso X Mochaccino

    W Vienna S Macchiato

    718. (3) Y orders Latte coffee. 719. (1) S orders Macchiato. 720. (3) W sits opposite to Q. 721. (2) S sits second to the right of Z. 722. (4) When counted from the left of R, four friends – P, Y, W and S – sit between X and R. (723–727) :

    C

    W

    B

    Y

    A

    X

    D

    Z

    Row–1

    Row-2

    708. (2) Six persons sit to the left of C. C faces south. Six persons sit to the left of A. A faces north. 709. (4) Five persons sit to the right of F. H sits at one of the positions to the right of G. C is an immediate neighbour of D and F. 710. (5) Three persons – C, E and H – face south. 711. (3) Except EH, in all other pairs both the persons face north. E and H face south. 712. (1) B and A are immediate neighbours of E. (713–717) : 1 Box K 2 Box R 3 Box D 4 Box J

    Cappuccino R Irish Affogato Q P

    Row–2

    R B

    698. (3) Except P, all others are sitting in the middle of the sides. P is sitting at the corner of the table. 699. (4) P sits to the immediate right of V. 700. (1) Three persons sit between A and O. 701. (5) J sits to the immediate right of B. V sits third to the left of J. J faces the centre. Either two or four persons sit between J and V. 702. (2) J and R are immediate neighbours of B. (703–707) :

    L E F T

    F

    Only three boxes are kept below Box T.

    A

    J

    R I G H T

    C

    Box M is kept immediately above Box H.

    A and C does not face each other. The back of C is towards back of A.

    RANKING/ARRANGEMENT 723. (1) Except W, X, in all other pairs both the persons face the same direction. W faces south while X faces north. 724. (4) C sits second to the left of B. 725. (3) Z sits third to the left of A. Z sits opposite Y. Z faces south direction.

    736. (2) O is three places away from N. Q faces south and N faces north. Five persons are sitting to the left of L while four persons are sitting to the right of L. 737. (1) Two persons – O and P – are sitting between I and K.

    726. (2) X sits second to the right of Z. 727. (3) C sits opposite A. B sits second to the right of C. (728–732) :

    (738–742) :

    739. (2) B sits third to the left of E.

    G

    740. (1) S and E sit at the extreme ends of the line.

    F

    H

    C

    B

    D E

    728. (1) B sits third to the right of A. D sits second to the left of B.

    741. (5) B is an immediate neighbour of both A and R. Five persons sit between P and Q. Q sits to the immediate right of A. Only C sits between R and D.

    731. (5) Except A, all others are facing the centre.

    20 years A R 18 years

    W 40 years

    732. (2) G is facing B. (733–737) :

    Q J M R L

    I

    O P K N

    P 30 years

    S 25 years J 15 years

    733. (1) R, L, I and O are sitting between P and M. I and O are facing North. 734. (4) R is sitting third to the left of O. P is sitting fourth to the left of R. 735. (3) There are two persons between L and P. There are two persons between L and J. There are two persons between O and N.

    = 43 years 745. (5) S = 25 years J = 15 years 746. (4) A + W = (40 + 20) years = 60 years R (18 years) is younger than P (30 years). J is an immediate neighbour of both S (25 years) and P (30 years). 747. (3) P sits second to the right of S. (748–752) : 3U

    2B

    4Q

    4R

    Z2

    3Y

    742. (4) Except PS, in all other pairs, both the persons face the same direction. P and S face opposite directions. (743–747) :

    729. (4) Both A and C are immediate neighbours of F. 730. (3) F sits third to the left of H.

    744. (1) S + R = (25 + 18) years

    S P D C R B A Q E

    738. (3) Three persons – D, C and R – sit between P and B.

    A

    743. (2) A sits to the immediate left of W. A is 20 year old.

    W3

    2T

    S4 4V

    X3

    P2

    748. (3) Q + S = (4 + 4) litres = 8 litres 749. (2) R and Y sit between B and T when we proceed to the right of B. 750. (1) Q, S, V and R drink 4 litres of water per day. T sits second to the right of R. 751. (5) Z and S are immediate neighbours of W. Z + S = (2 + 4) litres = 6 litres 752. (4) Q, S, V and R drink 4 litres of water per day

    (753–757) :

    K

    S

    753. (2) 754. (5) 755. (5) 756. (3) 757. (4) left

    R

    A

    N

    E

    J

    B

    G

    U

    M

    I

    Three chairs are vacant between R and G. Six girls – J, B, G, U, M and I – are sitting to the right of E. There is a vacant chair between U and M. M sits to the right of U but to the left of I. Six girls – N, E, J, B, G and U – are sitting to the right of A but to the of M.

    BPRE–397

    RANKING/ARRANGEMENT (758–762) :

    (773–777) :

    S

    L E F T

    V

    P

    T

    U

    Husband-Wife

    Q

    W

    R I G H T

    V

    te gh u Da

    So n

    Wife-Husband

    R

    Daughter

    Wife-Husband

    H E

    J B

    F C

    There are three vacant chairs between E and G. There are altogether 17 chairs in the row. H sits exactly between J and I. Eight chairs are occupied between D and G.

    TEACHER L E (768–772) : F T

    H

    B

    G

    J

    E

    C

    K

    D

    L

    A

    I

    F

    G 1997

    A 1996

    E 1995

    G K

    763. (5) There is a vacant chair at sixth to the left position from the right end. 764. (3) 765. (1) 766. (4) 767. (2)

    K 1994

    F was born in 1992. (778 –782) :

    S is husband of V.

    I A

    H 1993

    F is 6 years elder than L. 774. (2) E is fourth to the right of J. 775. (5) N was born in 2001. G sits fourth to the right of N. 776. (4) D was born in 2000. A was born in 1996. 777. (3) Leap years : 1992, 1996 and 2000.

    758. (2) R is husband of T. Q sits second to the right of R. 759. (4) W sits fourth to the right of T. W is son of Q and U. 760. (1) V sits exactly between P and S. V is maternal grandmother of P. 761. (3) Q sits to the immediate left of W. Q is maternal uncle of P. 762. (5) S sits at the extreme left end.

    L D

    F 1992

    773. (2) 1998 – 1992 = 6

    P

    W

    (763–767) :

    1991 B

    L 1998

    T

    Q

    1990 M

    J 1999

    Son

    U

    N 2001 D 2000

    r

    S

    R

    R I G H T

    768. (3) L sits exactly between D and A. 769. (1) H is sitting at the left end of the first row. D is sitting immediate right below diagonally of H. 770. (2) I is an immediate left neighbour of F. D sits third to the left of I. 771. (4) H and C are at extreme ends of the first row. K and F are at extreme ends of the second row. 772. (5) K sits immediate left below diagonally of B. L sits immediate right below diagonally of B.

    BPRE–398

    S.No. Person Gadget 1 B Headphones 2 F Phone 3 E Watch 4 5 6

    G D A

    TV Tablet Speaker

    7

    C

    Laptop

    778. (4) Except the combination ‘‘C– Headphones’’, in all other combinations the given person bought a gadget before the given gadget. C bought Laptop on one of the occasions after B who bought headphones. 779. (1) G bought a TV. 780. (5) Two person–E and G – bought a gadget between F and D. 781. (2) B was the first to buy a gadget.

    RANKING/ARRANGEMENT Only one person bought a gadget between the one who bought a speaker and the one who bought TV. Four persons – G, D, A and C – bought a gadget after E. D bought a gadget immediately before A. 782. (3) C bought a Laptop. (783–787) : X

    Q

    G

    P

    B

    R

    S

    W

    M

    Now,

    C>E>F>A>B>D 3 kg

    793. (1) C > E > F > A > B > D

    10 kg

    Weight of bag F would be more than 4 kg and less than 10 kg. 794. (4) No bag is lighter than bag D. No bag is heavier than bag C. 795. (2) Two bags – B and D – are lighter than bag A. (796–800) :

    783. (3) R sits third to the right of M. Three persons – G, P and B – sit between Q and R. 784. (4) Except S, all others are facing south. 785. (1) S sits to the immediate right of R. 786. (5) No one sits to the right of X. 787. (2) P sits fourth to the left of W. (788–792) : Person

    Age

    Executive Director (ED)

    D

    47 years

    General Manager (GM)

    F

    60 years

    Post

    Manager (MG)

    B

    29 years

    Assistant Manager (AM)

    E

    51 years

    Probationary Officer (PO)

    A

    38 years

    C

    20 years

    Clerk (CL)

    788. (5) B is a Manager (MG). 789. (4) F is a General Manager (GM). F is 60 year old. 790. (2) C is a Clerk (CL). A is 38 year old who is a Probationary Officer (PO). A is immediate senior of C. 791. (1) Except the combination PO– F, in all others the first person is senior than the second person and there is a gap of only one post. Probationary Officer (PO) is junior than F, the General Manager (GM). 792. (3) C is 20 year old. Therefore, C’s present age is an even number. (793–795) : >

    >F>

    >E>F> C>A>B>D

    > >

    > >

    3 kg

    Post

    Person

    Clerk (CL)

    S

    Basketball

    Assistant Manager (AM)

    R

    Football

    Manager (MG)

    A

    Rugby

    Deputy General Manager (DGM)

    C

    Cricket

    General Manager (GM)

    B

    Kabaddi

    Executive Director (ED)

    Q

    Hockey

    Chief Financial Officer (CFO)

    P

    Golf

    796. (5) Five persons – A, C, B, Q and P – are senior than R. Five persons – B, C, A, R and S – are junior than Q. Q plays hockey. 797. (4) S plays basketball. R is an immediate senior than S. R plays football. 798. (3) Clerk plays Basketball. S is an immediate junior of R. Therefore, only statement (B) is true. 799. (2) C plays cricket. Therefore, only statements B and C are true. 800. (1) P is the Chief Financial Officer (CFO). P plays golf. (801–803) :

    1 2 3 4 5 6 7 8 9 10 A > > > > > > > > >B A>

    >

    Sport

    >

    >J>

    >

    >

    >

    >B

    A > C >G>

    >J>

    >

    >

    >D > B

    A > C >G> H> J > I > F >E >D>B 801. (2) Seven persons – A, C, G, H, J, I and F – are older than E. 802. (1) Age of G is more than that of H but less than that of C. 803. (3) D is the second youngest.

    BPRE–399

    (804–808) :

    27 years S

    36 years A 33 years P

    R 19 years

    D 20 years

    C 23 years Q 48 years

    B 52 years

    804. (3) P – R = (33 – 19) years = 14 years B – Q = (52 – 48) years = 4 years Q – C = (48 – 23) years = 25 years A – S = (36 – 27) years = 9 years C – D = (23 – 20) years = 3 years 805. (4) Either two or four persons sit between R and S. A sits third to the left of C, who is 23 years old. B sits second to the right of C.

    RANKING/ARRANGEMENT 806. (2) B is 52 – year old. 807. (1) Except P, the ages of all other persons are even numbers. P is 33–year old. 808. (5) The persent age of C is 23 years.

    816. (4) Four boxes – A, R, K and M – are placed between box L and box B. 817. (1) Except box L, all other four boxes are placed at the odd numbered positions. 818. (3) Box J is placed just above the box Q.

    (809–813) :

    Only one box is placed between box P and box Q.

    Pune

    Imphal

    Mumbai

    Delhi

    M

    B

    C

    A

    E

    D

    Surat

    K

    Indore

    J

    Aizawl

    L

    Jaipur

    L E F T

    N

    Lucknow

    R I G H T

    Chandigarh

    Row – II L E F T

    R S K M G C V A N

    K

    Vacant

    812. (4) A faces J. J belongs to Mumbai. 813. (3) N belongs to Delhi. (814 – 818) :

    L

    M

    Row – I

    811. (1) A faces J . A belongs to Aizawl.

    P

    O

    819. (2) When counted from the right of J, only M sits between J and O.

    821. (3) Except PM, in all other pairs both the persons sit at the corners of table or in the middle of the sides of the table. P sits at the corner while M sits in the middle of one side.

    G

    822. (2) P sits adjacent to O. 823. (5) J sits to the immediate right of M.

    9

    Box P

    8

    Box J

    7

    Box Q

    When counted from the left of J, three persons – N, L and K – sit between J and P.

    6

    Box L

    N sits second to the left of K.

    5

    Box A

    4

    Box R

    3

    Box K

    2

    Box M

    1

    Box B

    Thus, both (2) and (3) are true. (824 –828) :

    AB

    C

    FE

    G

    D

    H

    824. (5) No one is sitting in the middle of the line as there is a vacant seat.

    BPRE–400

    C

    E

    F

    B

    D

    A H

    Box I

    815. (2) Box P is placed at the 9th position.

    829. (1) Except K, all other persons are facing north direction. 830. (2) V works in BOB. G sits second to the left of V. 831. (4) K sits exactly between M and S. 832. (5) All the statements are true. 833. (3) M works in BOI. V sits third to the left of M. (834 – 838) :

    820. (5) M is sitting third to the right of L.

    10

    814. (5) Box B is placed at the bottom of almirah.

    828. (3) G is sitting sixth to the left of H. (829–833) :

    (819 – 823) :

    R I G H T

    810. (2) D sits at the extreme right end of the Row–I.

    826. (1) There are 15 seats in the row. 827. (5) If C and F interchange positions, there is no one as immediate neighbour of F.

    Box R is placed at fourth position.

    J

    809. (4) E belongs to Indore. M belongs to Pune. Except E, all others are sitting at the extreme ends of the rows.

    825. (4) E is sitting third to the left of C.

    834. (3) A is to the immediate left of H. E sits opposite to A. 835. (5) B sits exactly between E and D. 836. (4) E sits third to the right of F. 837. (1) C and E are immediate neighbours of G. Both C and E face the centre. 838. (2) A sits second to the right of C. (839 – 843) :

    D E G H A C B F 839. (4) G is sitting third to the right of C.

    RANKING/ARRANGEMENT

    P (Urban Development)

    A (Finance) N (Health)

    H (Agriculture)

    J (Defence)

    R (Sports)

    S (Railways)

    844. (2) R sits to the immediate left of Agriculture Secretary (H). 845. (3) A is the Finance Secretary. N sits between J and A. 846. (5) J sits third to the right of K. J is the Defence Secretary. 847. (4) K is the Education Secretary. S sits opposite to K. S is the Railways Secretary. 848. (2) Except P and Sports Secretary (R), all other pairs consist of persons sitting opposite to each other. P sits second to the right of R. (849–853) : Q

    T

    Z

    W

    Rs. 40,000 Rs. 30,000 Rs. 35,000 Rs. 40,000 Y Rs. 29,000

    P Rs. 32,000

    O Rs.36,000

    S Rs. 33,000

    V R X U Rs. 40,000 Rs. 38,000 Rs.28,000 Rs. 40,000

    849. (1) P – Y = Rs. (32,000 – 29,000) = Rs. 3,000 850. (5) V is sitting fourth to the right of T. V receives Rs. 40,000 851. (4) P receives Rs. 32,000. P is third to the right of X. 852. (2) W is sitting exactly between Z and P. W receives Rs. 40,000.

    L E F T

    K S

    T A

    H E

    J B

    GN

    854. (4) Six girls – H, E, J, B, G and N – are sitting to the right of A but to the left of M. 855. (2) M sits to the right of N but to the left of R. 856. (5) Six girls are sitting to the right of E. 857. (3) Except MR, in the other four pairs of girls, there is one vacant chair between the two girls. 858. (1) There are three vacant chairs between T and G. (859–863) :

    1. (2) J

    I A Y T

    G

    C

    A E

    859. (5) H is sitting just opposite to G. 860. (4) D is sitting exactly between H and F. 861. (1) Except G D, in the other four pairs the first student is sitting second to the right of the second student. G is third to the left of D. 862. (2) C and D are sitting opposite to each other. 863. (3) B is third to the right of E.

    FOR L AT E S T VA C A N C IE S and Dates of

    log on our website : w w w .kira n p ra ka s ha n .c o m

    BPRE–401

    –3

    3

    +3

    6

    –3

    5

    –3

    H

    –3

    ©

    +5

    D

    –5

    P

    +5

    R

    +5

    $

    +5

    V

    2. (4) P and D sit at extreme ends of the rows. 3. (1) A faces S. 4. (2) Two persons – S and T – are seating between V and R. 5. (2) P faces the person who sits second to the left of A. S faces the person who sits second to the left of B. Similarly, T faces E who sits second to the left of D. 6. (3) F faces V who sits second to the right of T. 7. (5) A faces the immediate neighbour of T. B faces the immediate neighbour of T. F faces the immediate neighbour of P. C faces the immediate neighbour of V. But E faces the person who is second to the right of Q. (8–14) :

    B

    H

    R I G H T

    SBI PO EXAMS

    F D

    M R

    Fi n Ad a n vi cia so l r

    K (Education)

    853. (4) R + U = Rs. (38,000 + 40,000) = Rs. 78,000 (854 – 858) :

    President

    A

    G Vice-

    D

    Manager

    840. (3) C is sitting exactly between A and B. 841. (5) A is sitting second to the left of B. 842. (2) Except B, all other persons face south. B faces north. 843. (1) H is sitting fourth to the left of F. (844–848) :

    President

    H

    B

    Com pany Secretary

    F

    Group Leader

    C E Chairman

    M anaging Director

    8. (4) D, the Financial Advisor sits third to the left of E.

    RANKING/ARRANGEMENT 9. (5) Except in B–Managing Director, in all others the first person is to the immediate left of the second person. 10. (1) A is the President of the Company. 11. (4) The Group Leader F is the immediate neighbour of H, the Manager or E, the Chairman. G sits second to the left of D. The immediate neighbours of Group Leader, F are Chairman E and Manager H. The Chairman E, is to the immediate left of Managing Director C. The Group Leader F sits second to the right of the Financial Advisor D. 12. (3) B is the Company Secretary of the Company. 13. (5) E, the Chairman and F, the Group Leader sit between C, the Managing Director and H, the Manager. 14. (2) F is the Group Leader.

    19. (1) Lawyer J is second to the left of the Doctor F. Chartered Accountant E is an immediate neighbour of Lawyer J and Columnist M. Scientist H sits exactly between Doctor F and Professor K. Only three persons – E, J, G or L, K, H – sit between Doctor F and Columnist M. (20–24) :

    S

    D E

    T T

    UV

    S X

    G

    30. (2) V studies in Standard II. W is sitting fourth to the right of V. 31. (1) X studies on Standard III. T is sitting between X and W. (32-38) : P

    A

    H

    YW

    V

    F

    B

    W U

    X

    R R

    Y

    C

    20. (2) E is to the immediate left of D.

    M

    21. (3) Three persons – C, A and G – are seated between H and F if we go anticlockwise from H to F. 22. (1) A is third to the right of D.

    (15–19) :

    29. (4)

    J

    K

    N

    23. (4) B is sitting just opposite to G. Scientist H

    Doctor F

    Professor K

    24. (3) G is second to the right of D. Financial G Analyst J Lawyer

    L Engineer

    M Columnist

    (25–29) :

    III

    IV T W I

    X

    E Chartered Accountant

    15. (2) Financial Analyst G is sitting second to the right of E. 16. (4) K is the professor. 17. (3) Chartered Accountant E is sitting just opposite to Scientist H. Columnist M is sitting just opposite of Doctor F. Lawyer J is not sitting just opposite to Engineer L. Financial Analyst G is sitting just opposite of Engineer L. Lawyer J is sitting just opposite of Professor K. 18. (2) Engineer L is second to the right of Scientist H.

    U VII

    V R

    II

    Y VI

    V S VIII

    25. (2) V is sitting third to the right of T. V studies in Standard II. 26. (5) The person studying in Standard VII is sitting just opposite to R. U studies in Standard VII. 27. (1) R and S are immediate neighbours of V. R studies in Standard V and S studies in Standard VIII. 28. (3) W is third to the right of S. W is fifth to the left of S.

    BPRE–402

    I O 32. (3) P and K face the centre while N, O, I, J and M face outward. 33. (1) J is third to right of N. J is fourth to left of N. 34. (4) K is second to the right and fifth to the left of M. 35. (2) P is sitting exactly between M and K. 36. (5) P is second to the right and fifth to the left of J. 37. (3) Three persons – P, K and N – are sitting between M and O if we move clockwise from M. 38. (1) N and O are seated between K and I. Only two persons, P and K are facing the centre. J and O are neighbours of I. (39-44) :

    R P V W U S T Q 39. (4) P, W, S and Q face North.

    RANKING/ARRANGEMENT 40. (1) U is to the immediate left of S.

    T R

    42. (2) R and Q are standing at the extreme ends. 43. (1) P is third to the right of U. 44. (4) P and W are immediate neighbours of V.

    68. (5)

    (59–62) :

    41. (3) There are two persons – V and W – between P and U.

    U R

    S

    V

    L

    Q

    N

    O

    H

    J

    I

    M

    SOUTH

    U

    U R

    46. (3) The immediate neighbours of L are K and N. 47. (4) Two persons – H and J – are standing between I and O.

    V T

    B

    S

    A

    EAST P

    O

    M

    L

    S

    Q

    R

    N

    S

    Q

    U

    Q

    W R

    F

    63. (3) P is sitting third to the right of R. Remember, R is facing outside the centre. 64. (2) Q is sitting just opposite to S. (65–68) : T R

    S

    52. (4) L is infront of M. 53. (1) The immediate neighbours of O are M and P.

    V

    P

    54. (4) Two persons – S and Q – are standing between R and L. 55. (2) L is standing to the immediate left of S. 56. (3) Except M, all others are facing north. 57. (5) L standing is exactly between O and Q. 58. (1) O is fifth to the left of R.

    G

    D

    50. (2) O is sitting exactly between L and J.

    NORTH

    H

    PP

    T V

    49. (5) Except K, all others are facing south. K faces north.

    (52–58) :

    U

    W

    48. (1) O is to the immediate left of H.

    51. (4) L is to the fourth of J.

    Q

    Q W R 69. (3) P is sitting third to the right of R. Remember, R is facing outside the centre. 70. (2) Q is sitting just opposite to S. (71–76) : S

    59. (2) Q is sitting to the immediate left of V. 60. (4) Three persons are sitting between T and W. 61. (1) Q is sitting exactly between W and V. 62. (5)

    45. (2) N is facing south. Therefore, J is third to the left of N.

    W

    PP

    T V

    W

    K

    S

    P

    (45–51) :

    NORTH

    V T

    Q

    U

    W 65. (2) Q is sitting to the immediate left of V. 66. (4) Three persons are sitting between T and W. 67. (1) Q is sitting exactly between W and V.

    BPRE–403

    E C

    71. (2) Either two (D and A) or four (C, E, G and H) persons sit between F and B. A sits second to the right of F. C sits to the immediate left of F. F faces outside. 72. (5) Five persons – B, G, C, F and A – face outside. 73. (2) C is third to the left or fifth to the right of A. 74. (1) E sits exactly between G and C. 75. (4) A sits to the immediate left of B. 76. (3) Except H, all others face outside. Must Read

    Buy Today

    Kiran’s

    ONE LINER APPROACH GENERAL KNOWLEDGE

    RANKING/ARRANGEMENT (88 – 92) :

    (77–81) :

    Row–1

    77. (5) R and L are sitting at the extreme ends of the two lines. 78. (4) T likes Donald Duck. 79. (1) N likes Spiderman. J and M are immediate neighbours of N. M likes Jerry. 80. (2) Q likes Tom. 81. (3) L faces S. Q likes Tom. Q is an immediate neighbour of both P and T. K likes Superman. K is an immediate neighbour of both J and R. L likes Dexter. L faces S. (82–87) :

    P M

    R

    S

    N

    O

    Q T

    82. (4) O sits exactly between S and T. 83. (2) P, N and O face the centre. 84. (1) P sits second to the right of N. 85. (5) Except O, all others face outside. 86. (3) T faces outside. Q sits between N and T. S sits second to the right of T. N and Q sit between T and R. 87. (3) T faces outside. M is third to the right or fifth to the left of T.

    Peony

    Orchid

    Rose

    Q

    P

    N

    L

    J

    L E F T

    Daisy

    R I G H T

    R

    Plumeria

    L

    S K

    Tulip

    M

    T M

    Daffodil

    N

    R I G H T

    Dahlia

    Noddy

    Batman

    Tom

    S

    L E F T

    Lily

    J

    P

    Dexter

    K

    Q

    Jerry

    T Spiderman

    Donald Duck

    Oswal

    R Ben–10

    L E F T

    Row–2

    Superman

    R I G H T

    Gazania

    Row – 2 L E F T

    R I G H T

    Row – 1

    88. (3) P likes Peony. Q is an immediate neighbour of P. Q likes Gazania. 89. (4) S likes Rose. 90. (2) L faces Q. J likes Daisy. L is an immediate neighbour of J. Q likes Gazania. T likes Dahlia. T faces M. 91. (1) P and J are sitting at the extreme ends of the two rows. 92. (5) N likes Tulip. (93–97) :

    F

    C

    E

    N

    P

    Q

    O

    T O

    S

    R

    M

    N Q

    BPRE–404

    B F

    G

    A

    E

    D

    93. (2) D and F are sitting at the extreme ends of the line. 94. (4) N and Q are immediate neighbours of P. C is sitting fourth to the left of Q. There are four persons between D and E. 95. (1) O is sitting third to the right of N. 96. (3) C is sitting fourth to the left of Q. 97. (5) Except in the pair NQ, in all others there are two persons between the given pair of persons. (98–102) :

    P

    98. (2) S faces the centre. T is sitting second to the right of S. T is sitting just opposite to Q. There are only three persons between T and Q. T and M face the same direction, i.e, towards the centre . 99. (1) Three persons – P, Q and R – face outside, i.e., opposite to the centre. 100. (4) Except Q, all others face towards the centre. 101. (3) M sits exactly between S and Q when counted from the left of S. 102. (5) R faces outside, i.e., opposite to the centre. M is sitting third to the left of R. (103 –107) :

    D

    H C

    103. (5) Five persons - B, G, E, D and F - face towards the centre. 104. (2) H sits third to the left of A. Only one person sits between A and B. Immediate neighbours of A are F and D. Either two or four persons sit between A and G. 105. (3) Except H, all others face the centre. 106. (4) H faces outside. E is to immediate left of H. 107. (1) H sits second to the left of G. (108– 112) : L E F T

    L

    F G M N H E O

    R I G H T

    108. (5) L is second to the left of G. 109. (2) Only two persons sit between N and O. Three persons sit to the right of N. E sits second to the right of N. M and H are immediate neighbours of N. 110. (1) L is sitting at the extreme left end of the line. 111. (4) O is to the immediate right of E. 112. (3) F and G sit exactly between M and L.

    RANKING/ARRANGEMENT

    R

    R I G H T

    O

    Row–2

    121. (2) P is second to the right of N. 122. (1) Two persons - N and R - face outside. (123-124) : Car A Bike Bike E G

    B Car

    Car D

    Bike H C Car

    L E F T

    R

    T

    K

    J

    128.

    S

    118. (4) Except N, all others face the centre. 119. (5) S sits second to the left of U. 120. (3) Q and O are immediate neighbours of S. S sits second to the left of R. Three persons sit between S and T. Either two or four persons sit between S and N.

    F Bike

    H

    F F C E The position of only F remains unchanged. 125. (5) E and H drive bikes. 126. (4) D drives a car. G and H are immediate neighbours of D. H sits to the immediate right of D. 127. (3) F is second to the left of E. (128–132) : D

    U

    Q

    BG

    CD

    129. 130. 131. 132.

    Noddy

    P

    H

    Q

    P

    S

    N

    M

    L Dexter

    N

    E

    G

    Batman

    T

    B

    Jerry

    113. (5) G is sitting at the extreme right end of the line. 114. (4) Two persons - Q and N - are sitting between J and G. 115. (1) P is sitting third to the left of N. 116. (2) Q is an immediate neighbour of N. Q is sitting to the immediate left of N. N and G are sitting to the right of Q. J and N are immediate neighbours of Q. J is sitting to the immediate right of Q. 117. (3) Three persons - J, Q and N are seated between P and G. (118– 122) :

    A A

    Tom

    P J Q N G

    Spiderman

    H

    Donald Duck

    I

    Ben–10

    O

    123. (2) Except E, all others drive cars. 124. (2)

    Superman

    L E F T

    R I G H T

    Oswal

    (113– 117) :

    L E F T

    Even digit 3 Even digit R I G H T

    Row–1 (5) R and L are sitting at the extreme ends of the two lines. (4) T likes Donald Duck. (1) N likes Spiderman. J and M are immediate neighbours of N. M likes Jerry. (2) Q likes Tom. (3) L faces S. Q likes Tom. Q is an immediate neighbour of both P and T. K likes Superman. K is an immediate neighbour of both J and R.

    L likes Dexter. L faces S. (133–138) : M O

    133. (3) Four persons – N, R, M and O – sit between L and Q when counted from the left of Q. 134. (1) Either one or five person(s) sit(s) between P and L. R sits to the immediate left of N. N is an immediate neighbour of both R and L. 135. (5) M sits third to the left of P. 136. (4) Three persons – M, P and Q – face the centre. 137. (1) If we move anticlockwise, the number of the persons between the given pair is increasing by one. P and S are immediate neighbours of each other. There is only one person between L and R. There are two persons between M and P. There are three persons between S and M. There are four persons between O and N. 138. (2) O sits just opposite L. M sits just opposite S. N sits just opposite Q. 139. (2)

    5831792512

    8542373962429536 4612 140. (3)

    54237396

    29

    53

    5 8 3 1 7 9 2 5 1 2 8 3 4 5 2...... 11th from left end

    5 1 or 2

    5 8 3 1 7 9 2

    N

    24

    64612 141. (4) 5th to the left of the 16th digit from the left means 11th from the left end

    52 Q

    2 Perfect Square

    5831792512834528

    142. (1)

    R

    834 52

    51

    2 8 3 4

    8542373962429

    5364612 143. (5) According to the question, the new sequence would be : 5 3 1 7 9 5 1 3 5 5 3 7 3 9 9 5 3 1

    P

    L S

    BPRE–405

    8th from the right end

    RANKING/ARRANGEMENT (144–149) : K L

    N

    O

    H

    J

    I

    M

    144. (2) N is facing south. J is third to the left of N. 145. (3) K and N are immediate neighbours of L. 146. (4) Two persons – H and J – are standing between I and O. 147. (5) Except K, all others are facing South. 148. (1) L

    N

    O

    H

    J

    149. (4) J is facing south. L is fourth to the right of J. (150–155) : X Y H I J Z G W

    (161–165) : L C M

    E

    F

    D

    K

    N

    (171–175) :

    R I G H R T

    161. (4) M sits second to the right of L. 162. (3) Both M and E face north. M sits third from one end of line. Four persons – C, F, K and N – face south. 163. (2) L and N are sitting at the extreme ends of the line. 164. (1) Four persons – M, E, F and D – sit between C and K. 165. (5) E and D are immediate neighbours of F. (166-170) :

    D

    O

    N

    M

    F

    E

    L

    L V E F T

    Row–1

    Q

    P

    S

    N

    T

    D

    Z

    F

    X

    H

    Row–2

    L E F T R I G H T

    171.(1) F faces S. 172.(3) R and T sit at the extreme ends of the row–1. V and H sit at the extreme ends of the row–2.

    C

    173. (5) S faces F. 150. (4) ‘I’ faces North. G sits third to the right of I. 151. (1) Three persons – I, J and Z – are sitting between G and H. 152. (3) Only two persons – I and Z – face North. H and G both face South. Z faces North. 153. (5) Y and I are immediate neighbours of H. 154. (2) There are three persons between H and G. In all others, there are only two persons between the given pair of persons.

    D sits exactly between V and Z. 166. (4) D faces south. Five persons face south. L sits second from the extreme end. 167. (3) Only D sits to the left of O. 168. (1) E faces north. 169. (2) Only C sits to the left of L. 170. (5) D and C are sitting at extreme ends of the line. (176–181) :

    155. (4) X sits second to the right of H. Z sits second to the right of I. W sits second to the right of Z. (156–160) : L O A N C B D M

    156. (2) Three persons – A, N and C – sit between B and O. 157. (1) D and O represent the immediate neighbours of the persons sitting at the extreme ends of the line. 158. (4) N, B, D and M face North. 159. (5) B faces North and C faces South. Four persons – L, O, A and C– face South. 160. (3) C faces South. A sits second to the right of C.

    Cardiology F 11

    S is an immediate neighbour of both P and N. P faces Z. Z is an immediate neighbour of D. 174. (2) Except T – P, in all other pairs, one person faces the other. 175. (4) F faces S. V sits third to the left of F.

    Diagnosis P 12 1

    Dentistry D

    Orthopaedic R 10

    Blood Donation 2K

    Integrated Services 3Z

    Paediatrics H9

    4 M Surgery

    8 T Ophthalmology 7 B Neurology

    5 6 V Nephrology

    Q

    Vaccination

    176. (4) The Dentistry counter is fourth to the right of Paediatrics counter. 177. (3) Dentistry counter is operated by D.

    BPRE–406

    RANKING/ARRANGEMENT

    2

    P

    3

    4

    5

    5

    G

    6



    A

    7

    7 181. (5)

    Q

    8

    9

    A

    10 11

    11

    K

    12

    12

    13

    13

    14

    14

    15

    15

    (182–187) : I

    23rd

    G

    16th

    O

    47th

    Q

    44th

    H

    7th

    F

    18th

    P

    31st

    R

    35th

    L E F T

    R I G H T

    P



    In both the cases, seven persons stand after Q. (188–192) : ● B sits at one of the extreme ends of the line. Therefore, B can sit at the extreme left end or extreme right end. ● D sits at the third position from the left end. Case-I D

    A

    S

    R

    Q

    C

    Years

    Years

    Years

    Years

    Years

    P

    Years

    R I G H 31 26 17 21 16 29 14 28 T B

    Years

    L E F T

    Years

    182. (2) R (35th floor) and I (23rd floor) are seated at two extreme ends of the line. 183. (1) P works on 31st floor. R works on 35th floor. Required difference = 35 – 31 =4

    G

    10

    W

    L



    Q

    8

    9

    P





    6

    BPRE–407

    D

    Q

    S

    31 21 26 16 29

    3

    4

    P



    C

    B

    R I G H T

    Years

    2

    A

    Years

    1

    L E F T

    Years

    1

    Case-II

    Years

    184. (4) F sits third to the left of the working on 47th floor. P sits third to the left of the one working on 44th floor. H sits third to the left of the one working on 16th floor. 185. (5) G works on the 16th floor and O works on the 47th floor. 186. (3) R works on the 35th floor. R sits at the extreme left end. 187. (1) Case-I Case-II

    Years

    178.(2) B deals with Neurology. He is second to the left of H, who deals with Paediatrics. F deals with cardiology. F is second to the left of D. M deals with Surgery. V deals with Nephrology. M is second to the left of V. Z is second to the left of Q. Q deals with Vaccination. P deals with Diagnosis. Q is fifth to the right of P. 179. (5) Q sits just opposite to F. Q deals with Vaccination. 180. (1) The one who deals with Vaccination, sits at counter number 5. The Vaccination counter is exactly between the counters of Surgery and Nephrology. Q deals with Vaccination. Q is third to the right of K. F sits just opposite Q, who deals with Vaccination counter.

    Q’s age is not a Prime Number. Therefore, Q’s age may be 14, 16, 21, 26 or 28 years. Three persons are sitting between the one whose age is 16 years and B. In Case-I, the one whose age is 16 years would sit at the fifth position from the left end and in the Case-II at the fourth position from the left end. C sits to the right of one whose age is 16 years but not immediately. The one whose age is 29 years sits second to the left of C. In Case-I, C would sit at the extreme right end. In Case-II, C would sit at the second position from the right end. The eldest one is not an immediate neighbour of D and D is not the youngest. P sits to the immediate right of the eldest person. Thus, in Case–I, B would be the eldest and P would sit at the second position from the left end. In Case–II also, the eldest person would sit at the extreme left end and P at the second position from the left end. Difference between the ages of D’s immediate neighbours is 5. There are two persons between the one whose age is 26 years and S. S sits to the right of one whose age is 26. In Case–I, P’s age would be 26 years and S would sit at the fifth position from the left end. In Case–II, P’s age would be 21 years and D’s age would be 26 years. Thus, in Case–II, S would sit at the third position from the right end. R does not sit to the immediate left of S. Q sits third to the right of A. If A sits at the extreme left end in Case–II, then Q would sit at fourth position from the left end. Thus, R would sit to the immediate left of S. But, it is given that R does not sit to the immediate left of S. Therefore, the arrangement given in Case–II can be discarded.

    RANKING/ARRANGEMENT

    YEARS

    YEARS

    YEARS

    YEARS

    YEARS

    YEARS

    YEARS

    YEARS

    L B P D C R S A Q E F T 31 21 14 26 16 23 19 17

    R I G H T

    193. (4) R sits second to the right of D. 194. (1) B’s age is 31 years. Four persons – P, D, C and R – sit between B and S. 195. (3) C is 26 years old. 196. (2) P is third to the left of the person whose age is 16 years. B is third to the left of the peron whose age is 26 years. R is third to the left of the per son whose age is 17 years. 197. (4) Immediate neighbours of A are S and Q. The age of S is 23 years. The age of Q is 17 years. Required Difference = (23 – 17) years = 6 years

    198. (5) 1

    1

    2

    2

    3

    3

    4

    4

    5

    5

    6

    6

    (205–209) :

    19 years

    56 years

    22 years

    O

    K

    N

    F

    D

    H

    E

    G

    19 years

    22 years

    L E F T

    R I G H T

    8

    8

    Row-1

    9

    9

    205. (2) D sits second to the left of E. D is 56 year old. 206. (5) Both N and G are 22 year old. 207. (3) E and O are 19 year old. E faces K. O faces H. 208. (1) M is 35 year old. 209. (4) H is 29 year old. N faces G. G is 22 year old. F sits to the immediate left of D. (210–211) :

    10

    G

    10

    11

    11

    12

    12

    13

    13

    E

    The positions of L and Q cannot be determined. (199–201) : L, D < N A Parul = Sudha > Abhishek > Bharat 11. (4) Dana is the third heaviest. 12. (3) Bharat weighs minimum. (13–15) : Sitting arrangement

    Daughter U of W S Father of T

    16. (4) X is the father of S. When counted from the right of T, only Y sits between T and X. 17. (2) When counted from the right of Y, then X sits exactly between Y and W. 18. (3) Y and V are immediate neighbours of T. T sits second to the right of X. S is father of T. 19. (1) U faces outside. 20. (5) X is father of S. Therefore, Y would be mother of S. W is the wife of S and hence daughter-in-law of Y. T is third to the left of W. (21-23) :

    >

    >P >

    >

    >

    T>R Q >S > > > > Now, Q > S > P > T > R >U

    68 kg 35 kg 21. (3) Descending order of weights Q>S>P>T>R>U 22. (1) R is heavier than only U. 23. (2) S weighs 68 kg. P is lighter than S. (24–26) : R

    A

    U

    Q

    T

    F Accountant

    D

    Painter

    Wife Z of T

    X

    564 < 658 < 734 < 847

    T Architect

    Husband of U V

    Female Y

    B G

    D

    J

    Brother of U T

    A A

    Father of S

    (250–254) :

    G

    1. (3) D is a Painter. 2. (5) H is a Doctor. 3. (1) E is a lawyer. E is third to the left of G. 4. (4) Only Accountant F is sitting between Engineer B and Teacher G. 5. (2) G is a Teacher.

    W

    S

    Teacher

    C

    V

    B 13. (3) V is second to the left of C or fourth to the right of C. 14. (4) Option (4) is correct. 15. (1) A is second to the left of U.

    BPRE–410

    Z

    P V

    24. (3) V and S are immediate neighbours of P. 25. (4) Q is sitting fourth to the left or fourth to the right of Z. 26. (2) W is sitting second to the left of R.

    RANKING/ARRANGEMENT

    Amul

    Cipla

    K

    H

    V

    X

    U

    L E F T

    R I G H T

    27. (5) K is to the immediate left of the person from Vadilal. W is to the immediate left of the person from Tata. Similarly, X is to the immediate left of the person from Flipkart. 28. (1) Y is from Indigo. 29. (2) I is from Vadilal. V faces I. 30. (3) Two persons sit between G and the person from Cipla. The person from Havells faces G. G is an immediate neighbour of J from Biocon. G sits to the immediate right of I. 31. (4) Persons sitting at the extreme ends of both the rows : J (Biocon) and H (Cipla) Y (Indigo) and U (Flipkart) (32 – 37) :

    T HTC

    Q is an immediate neighbour of T, who works in HTC and R, who works in ONGC. P is an immediate neighbour of W, who works in Google and S, who works in Amazon. R sits second to the right of T, who works in HTC.

    V Sony

    Q P&G

    R ONGC

    Row–1

    Surat

    Ranchi

    Q

    R

    T

    P

    S

    D

    B

    C

    A

    E

    R

    Chennai

    T

    Mumbai

    H

    H

    L

    Bangalore

    F Architect

    U

    46. (5) T sits fourth to the left or right of U.

    47. (4) S works in Amazon.

    I

    I

    E F T

    Teacher

    R

    P

    V

    W

    S

    F

    Delhi

    E

    Lucknow

    L

    Patna

    C Engineer

    32. (3) E is an immediate neighbour of Clerk and Shopkeeper. E is an Architect. Clerk G is an immediate neighbour of Banker E. H is a Teacher. Shopkeeper B sits second to the left of Teacher H. 33. (5) H is a Teacher. 34. (2) Doctor D is third to the left of Banker E. 35. (4) Doctor D sits between Architect F and Businessman A. 36. (1) G is a Clerk. 37. (3) Clerk G is to the immediate right of the Businessman A. (38–42) :

    Q

    45. (3) P sits third to the left of Q.

    Dell U

    G

    T

    just opposite R. 44. (2) Q works in P & G company.

    R D Doctor

    E Banker B Shopkeeper

    just opposite Q. P works in Microsoft. P is sitting

    Amazon S

    (48–52) :

    Businessman A

    43. (1) W works in Google. W is sitting just opposite V. S works in Amazon. S is sitting

    Microsoft P

    Google W

    Agra

    Clerk G

    38. (2) U faces north. T sits secon to the left of U. 39. (5) Q and U sit exactly between R and T. 40. (1) T and S are sitting at the extreme ends of the line. Both T and S face south. 41. (4) Q and R face north. 42. (3) Three person – Q, U and R – sit between P and T. Three persons – V, W and S – sit to the left of P. R and V are immediate neighbours of P. (43–47) :

    Kanpur

    Tata

    Flipkart

    Vadilal I

    W

    Samsung

    Nokia G

    Y

    Havells

    L E F T

    J

    Indigo

    R I G H T

    Biocon

    (27–31) :

    T

    H T

    Row–2

    48. (5) Except S, no one is sitting at the extreme end of any row. 49. (4) In Row–1, Q and S sit at the extreme ends. Q is from Kanpur. 50. (1) T is from Mumbai. C faces T. C is from Lucknow. T sits exactly in the middle of the row–1. 51. (2) B is from Patna. S is from Ranchi. C is from Lucknow. 52. (3) B is from Patna. R faces B. R is from Bangalore.

    BPRE–411

    G

    RANKING/ARRANGEMENT (71-75) : are five person in each of the two parallel rows. Each person sitting in a row faces another person of the other row. ● A sits third from the left end of the row. Only one person sits between A and B. Only two persons sit between B and the one from Russia. As per these information, we may have two arrangements. Further, we may have two arrangements prior to the information : Neither B nor C is from either Poland or France. Observe the diagrams given below :

    (63–66) :

    ● There

    T

    V

    Z

    Mumbai E

    N B

    Y

    U X

    S

    T

    W V

    V U

    Z T

    X S

    62. (2) It is clear tha V sits exactly between Z and T.

    ai

    nn

    he

    C Pune

    C

    hi

    Z

    Y

    oc

    W

    B

    F

    L E F T

    E

    France

    Venezuela

    Argentina

    K A

    Brazil

    Germany

    D

    Argentina

    R I G H T

    H Surat

    K

    60. (5) Two persons – V and Z – are seated between T and X. 61. (4) The position of only Y will remain unchanged

    Lucknow G

    N B R I G H T

    Row-2

    Au

    D

    L E

    L E F T

    Row -1 Diagram-I

    r

    A

    Zaire

    L E F T

    J A

    Holland

    (67-80) :

    X

    59. (1) W and X are sitting at the two extreme ends of the line. Y is the immediate neighbour of W. Z is the immediate neighbour of X.

    M D

    K C

    Russia

    S

    R I G H T

    66. (1) U faces outside.

    ra n

    U

    Row -2

    u ip

    Y

    (i) S is the husband of W. U is the daughter of S and W. T is teh son of S and W. (ii) X is the father of S. U is the wife of V. T is the husband of Z. 63. (4) X is the father of S. When counted from the right of T, only Y sits between T and X. 64. (2) When counted from the left of W, it is clear that X sits exactly between W and Y. 65. (3) Y and V are immediate neighbours of T. T sits second to the right of X. T is the son of S and W. V is the brother-in-law of T.

    Ja

    W

    U r of ghte Dau nd W S Sa Husband of W

    Holland

    53. (2) U likes wanted. Q faces T and T is an immediate neighbour of W. P is an immediate neighbour of M. S sits exactly between U and V. 54. (5) N likes Uninhabited. 55. (3) T is facing Q. T likes Watchmen. 56. (3) Q likes Vertigo. Q sits at the extreme left end of the Row-2. 57. (1) V likes Cinderella. W sits third to the right of V. R likes Inception. P sits third to the right of R. X likes Hero. U sits third to the right of X. X faces P. 58. (4) R likes Inception. Q sits second to the left of R. (59-62) :

    W W ife of S

    Russia

    Row-2

    fT

    Fe m al e

    Twilight

    R I G H T

    Venezuela

    P

    Poland

    M

    Italy

    O

    W ife o

    R

    Z

    ad

    N

    X

    ga b

    Q

    L E F T

    Fath er of S

    X

    Tangled

    V

    Aladdin

    S

    H us of ban U d V

    Brother of U T

    Y

    Hero

    Cinderella

    Frozen

    Wanted U

    Inception

    W

    Vertigo

    L E F T

    T

    Uninhabited

    R I G H T

    Gladiator

    Watchmen

    Row-1

    Germany

    (53-58) :

    L E F T

    C R I G H T

    Row-1

    67. (4) F lives in Kochi. B sits second to the right of F. B lives in Chennai. 68. (2) All the statements are true. 69. (3) Except H–Pune, in all other pairs one person sits just opposite the other person. 70. (1) E lives in Mumbai.

    BPRE–412

    Diagram-II ● Only three persons sit between the ones from Poland and France. Neither B nor C is from either Poland or France. In case of Diagram-II, this condition is not applicable. Therefore, discard Diagram-II and complete the arrangement given in the Diagram-I.

    RANKING/ARRANGEMENT 71. (5) M is from Holland. E is from Germany. B is from Brazil. 72. (1) E faces L. L is an immediate neighbour of J and N. K sits to the immediate right of M. E faces L. L is from Venezuela. 73. (4) M sits second to the right of L. M faces D. 74. (3) D is from Russia. D sits in the Row-1. All other persons are seated in the Row-2. 75. (2) C is from Italy. (76–80) :

    Ferrari

    Row–1

    R I G H T

    76. (4) T is immediate neighbours of S and Q. S likes Yardley while Q likes Police. A faces T. B faces P who likes Titan. A sits second to the left of E. 77. (3) A sits to the immediate right of D. T sits to the immediate right of Q. S sits to the immediate right of T. T faces A. 78. (2) R faces E. 79. (1) S faces D. T sits between S and Q. 80. (5) D sits second to the left of C. D likes Hollister. (81–85) :

    Laptop F

    Camera E

    Speakers B

    D TV

    C Phone

    H Refrigerator

    A Printer

    G AC

    81. (1) C bought a phone.

    F

    D Z

    E

    G Blue

    C Purple

    X

    G

    ●F

    H Red

    86. (4) A sits second to the right of G. A likes green colour. 87. (2) B likes orange colour. 88. (5) A is an immediate neighbour of F and E. B sits second to the right of C. Either two or four persons sit between C and E. 89. (1) Except HG, in all other pairs, both the persons face the same direction. H faces outside while G faces the centre. 90. (3) F likes black colour. E sits second to the right of F. (91–95) : L E F T

    U

    T

    S

    P

    R

    Q

    V

    R I G H T

    91. (1) U and V are sitting at the extreme ends of the line. 92. (2) Q supports Australia. Three persons – S, P and R – sit between T and Q. S supports India.

    BPRE–413

    is the son of G. is the husband of X. D is father of Y. D is father-in-law of G. ● Z is brother of G. Z is son of E. ● Y is the daughter of D and X. Y is the wife of G. ● G is the husband of Y. G is the son of E. G is father of F. ● X is the wife of D. X is mother of Y. X is mother-in-law of G. ● E is mother of G and Z. 96. (5) Z sits to the immediate left of D. Z is the brother of G. F is the son of G. Therefore, F is the nephew of Z. 97. (4) X is the mother of Y. 98. (3) All the given statements are true. 99. (1) Y is the wife of G. when counted from the left of F, two persons – D and Z – sit between F and Y. 100. (2) F sits second to the right of Z. F is grandson of E. (101–105) : ●D

    D White

    B Orange

    West Indies

    B

    Australia

    E

    South Africa

    C

    P is an immediate neighbour of both S and R. R is an immediate neighbour of both P and Q. U and V are sitting at the extreme ends of the line. 93. (4) V sits second to the right of R. Q sits second to the right of P. S sits second to the right of U. 94. (5) P sits second to the right of T. P supports Sri Lanka. 95. (3) T supports England. (96–100) :

    Y

    Sri Lanka

    A

    E Yellow

    India

    D

    A Green

    F Black

    England

    P Davidoff

    R

    Brut

    Q

    L E F T

    New Zealand

    Titan

    Valentino

    Police

    Gucci

    Yardley

    T

    Cheque

    L E F T

    S

    Hollister

    R I G H T

    Row–2

    82. (5) A and D are immediate neighbours of H. A bought a Printer. H bought a Refrigerator. H sits just opposite to B. C sits to the immediate right of G. 83. (2) H bought a Refrigerator. 84. (4) Except C–Camera, in all other pairs, the first person sits to the immediate left of the person who bought the given product. E bought a Camera. C sits second to the left of E. 85. (3) D bought a TV. When counted from the right of E, F sits exactly between E and D. (86–90) :

    T V Y R P M Q W N S

    101. (4) Only V sits between T and Y.

    RANKING/ARRANGEMENT 102. (2) Q faces North. N sits seventh to the left of V. Only Q sits between M and W. R is an immediate neighbour of Y and P. 103. (5) Except P, all others face south. 104. (1) R sits to the immediate left of P. 105. (3) T and S are sitting at the extreme ends. V is an immediate neighbour of T. N is an immediate neighbour of S. (106–109) :

    M 42

    G 13 H 25 K 10 30 B

    Norway

    France

    Italy

    Argentina

    Denmark

    P

    T

    A

    C

    E

    D

    F

    B

    Row–1

    Germany

    U

    Japan

    R

    Indonesia

    Q

    Spain

    S

    Ireland

    L E F T

    Row–2

    Malaysia

    R I G H T

    Switzerland

    106. (5) Five persons sit between G and H. 107. (2) 27 persons are sitting in the given line. 108. (4) M is 42–years old. 109. (1) The 30 years old is to the immediate left of B. (110–114) :

    L E F T

    R I G H T

    110. (2) B likes Germany. S likes Denmark. Except U, all others are sitting at the extreme ends of the rows. 111. (5) Q sits second to the right of U. Q likes Argentina. 112. (4) E sits second to the left of F. R sits third to the right of T. Q faces C. R sits fourth from the left end and third from the right end. 113. (1) A faces S. A likes Malaysia. 114. (4) Four persons – A, C, E and D – sit to the left of F.

    (125–129) :

    (115–119) :

    Q

    T

    Column

    P

    U

    Row 3

    Y

    W

    A

    V

    X

    R

    Z

    S

    115. (3) T is sitting fifth to the left of S. P is sitting sixth to the right or left of R. T sits diagonally opposite to Z. R sits third to the left of V. U sits fifth to the left of A. 116. (1) R and Z are immediate neighbours of S. 117. (2) S is sitting fourth to the right of Y. Q sits third to the left of A. X is sitting to the immediate right of A. U is sitting third to the right of Z. 118. (5) Except U, the other four are sitting on the sides of the square. U is sitting at one corner of the square. 119. (4) T sits just opposite to Z. (120–124) :

    Column Row

    1

    2

    3

    4

    4

    L

    U

    3

    19

    B

    R

    2

    23

    P

    T

    1

    25 15

    E

    120. (3) In the Column-3, there are four letters. 121. (5) R is second to the right of 19 in the Row-3. 122. (1) Except T, the other four letters/numbers are in the Column2. T is in the Column-3. 123. (4) P is exactly between B and 15 in the Column-2. 124. (2) Letters ⇒ U, R, T, E Meaningful ⇒ TRUE

    BPRE–414

    3

    4

    K

    I

    C E G

    1

    2

    B

    2

    F

    H

    A

    1

    D

    L

    J

    125. (2) Except J, the other four persons live in the Row-3. J lives in the Row-1. 126. (4) H lives in the Column-2 of the Row-2, just below K. 127. (5) C lives second to the right of K in the Row-3. 128. (1) J lives in the Row-1. 129. (3) D lives in the Column-1 of the Row-1.

    Row-1 R I G H T L E F T

    N B Y

    T

    S

    J

    P

    L

    K

    A

    O R

    L E F T R I G H T

    Row-2 130. (1) Except K, the other four persons are sitting at the ends of the rows. 131. (4) Y is sitting exactly between B and T. 132. (2) Y is sitting third to the right of J. 133. (5) B is facing R. 134. (3) O is sitting fourth to the left of K. >N> > > 135. (2) >N> >J>M L, N > K L>N>K>J>M J is the second shortest friend.

    INSURANCE EXAMS (1–5) : Policeman Q Journalist Doctor R S Singer M

    Designer P

    O Engineer

    T Teacher N Author

    RANKING/ARRANGEMENT

    and Consonant

    Symbol

    G 8



    |

    +4 +3 +3

    U

    +3

    +4

    J

    +4

    G

    2

    3

    4

    5

    6 7

    U Q

    A

    X

    W

    Z F D

    B M

    B

    –3

    V

    [

    P

    +9 –9

    –5



    L

    O

    V

    S

    R

    G

    Y

    K

    H

    P

    T

    J

    13

    19 +

    P

    R I G H T

    O

    P ra

    sh ka

    La

    lit

    Pa

    Priti

    nk

    aj

    Mu

    sh ke

    30. (2) Pra

    sh ka

    La

    lit

    +6

    16 +6

    22

    Priti

    +6

    → H L  → M   Pa

    H

    23. (3)



    Q X W Z F F L V S N B 2 4 6 8 10 12 14 16 18 20 22

    Consecutive symbols are given.

    Q

    29. (1) Mukesh is opposite to Prakash.

    18

    → V  → R Z  10

    N

    25

    12 +6

    G

    M

    +

    6 6 → G   → J S  

    6

    L E F T

    E

    20. (2) The value of S is 13. If there will be a Vowel following it, then its value will be 14 Now, 13 + 14 = 27 21. (4) I = 9; N = 14; R = 18 9 + 14 + 19 = 41

    Vowel Consonant

    1 +5

    C

    R I G H T

    Grandfather occupied the central place. 27. (3) Total number of trees in the row = 7 + 14 – 1 = 20 28. (3)

    8 9 10 11 12 13 I

    Father

    19. (4) After interchange 1st → 14th 2nd → 13th 3rd → 12th 4th → 11th 5th → 10th 6th → 9th 7th → 8th 8th → 7th 9th → 6th (20–23) :

    L E F T

    Grandfather

    5th from the right

    DIP +3

    6 7 8 9 10 11 12 13

    Mother

    1 2 3 4 5

    Total number of boys in the row = 12 + 22 – 1 = 33 26. (2)

    Daughter

    18. (2)

    N

    22nd V 12th

    U Q

    There is only one such vowel :

    15. (4) E

    7th S

    Son

    S

    14 21 22. (1) 7 +7 +7 F  → N  → K

    +4

    C U S T O M ↓ ↓ ↓ ↓ ↓ ↓ 26 +7 + 18 + 40 + 7 + 24 = 122 24. (1) E X A M I N A T I O N ↓ ↓ ↓ ↓ ↓ ↓ ↓ ↓ ↓ ↓ ↓ 26 4 3 16 9 14 3 24 9 11 14 26+4+3+16+9+14+3+24+9+11+14 = 127 25. (3)

    2 S 14. (3) E ✡ F T ..... H 2 A 9 .... M B

    Consonant

    R

    14 15 16 17 18 19 20 21 22 23 24 25 26

    2 +3 P

    P T

    1

    Such combinations are : F ; 8D ; G| ; 2M ; 3L ; C ; 4R ; 5V ; 13. (1)

    17. (3)

    M C R G Y K H P T J 24 26 28 30 32 34 36 38 40 42

    D ee pa

    Number Consonant

    | ID8 [ 1N TE L3S★ 9M2H K G C✶Q4 ✡ F7 ● ■ J U 6 R 5V ✴ A B

    pa

    12. (2)

    16. (1) After three operations the new sequence would be :

    D ee

    1. (3) O sits exactly between M and N when counted from the right of M. 2. (5) Three persons sit between S and O. O is an Engineer. S is a Doctor. N is an Author. He sits to the immediate left of T, the Teacher. 3. (2) P is a Designer. 4. (1) Except in the pair N – Teacher, in all other pairs the first person sits second to the left of the second person. N sits to the immediate left of Teacher, T. 5. (4) R is a Journalist. 6. (3) Q is facing E. 7. (2) P and D. 8. (1) F faces U, who is to the immediate left of R. 9. (5) Except in Q U D, in all other groups there is one person between the first and the second persons and the second person is facing the third person. 10. (5) Except in RUCF, in all others the first person is just opposite of the fourth person and the second person is just opposite of the third person. In RUCF, the first person is just opposite of the third person and the second person is just opposite of the fourth person. 11. (3) Three persons — E, C and F — are standing between B and D.

    BPRE–415

    nk

    aj

    k es Mu

    h

    Deepa is sitting opposite to Priti.

    RANKING/ARRANGEMENT 31. (1)

    (55–59) :

    (40–46) : P

    L E F T

    D

    B

    A

    E

    C

    R I G H T

    House A is in the middle. 32. (1) B and E will get their rank in only one way. The remaining four persons can be ranked in 4! ways Number of arrangements = 4! = 4 × 3 × 2 × 1 = 24 33. (4)

    C

    E A

    B

    Or C

    D

    F

    A E

    P

    K

    R

    E

    A

    G

    N J

    H

    47. (1) H is third to the left of E. 48. (5) C is third to the right of G. 49. (4) A is fifth to the right of E. 50. (2) F is second to the left of H. 51. (1) F is to the immediate right of D. 52. (3) A is sitting between H and F. (53 – 54) : L E F T

    O

    Q

    F

    H

    C

    L

    M

    P

    K

    R

    N

    R I G H T

    53. (2) Q sits third to the left of P. 54. (4) O and N are seated at the two extreme ends of the line.

    BPRE–416

    A

    D

    J

    34. (2) M faces outside. P sits to the immediate left of M. Q sits second to the left of N. L faces outside and J faces towards the centre. O sits third to the left of M. 35. (4) L sits second to the right of K. 36. (3) K, L, M, N and Q face outside. 37. (5) L sits between P and O. 38. (2) K is sitting third to the right of Q. 39. (1) Except O, all others face outside.

    G

    B

    F

    K

    55. (5) Except A, all others face outside. 56. (2) D sits third to the right of S. 57. (1) B sits third to the right of Q. 58. (3) A, C and R face the centre. 59. (4) P sits to the immediate left of R. R sits between B and P when counted from the right of B. (60–64) :

    E

    D

    Q

    O

    A D

    L

    A is third to the right or left of F. (34–38) :

    L

    Q

    B

    C

    P

    S

    R

    S

    J

    B

    M

    C

    M

    40. (3) Five persons - M, S, L, J, and Q - face outside 41. (5) M is sitting third to the left of L. 42. (1) Except P, all others face outside. 43. (4) Q sits exactly between K and P when one moves clockwise from L. 44. (2) R is third to the right of Q. 45. (5) M is sitting second to the left of S. 46. (3) P sits to the immediate left M. J sits second to the left of Q. J and S are immediate neighbours of L. (47–52) :

    F

    D

    Q

    B 60. (3) Except E, all others face outside. 61. (1) H and D are immediate neighbours of E. 62. (4) G and C are immediate neighbours of F. F faces outside. F sits second to the left of H. 63. (5) H sits third to the left or fifth to the right of C. 64. (2) H is sitting to the immediate right of G. (65–69) :

    R I G H T V L Q E F T

    Row–1

    X O

    S P

    W R

    Row–2

    T N

    L E F U T M R I G H T

    RANKING/ARRANGEMENT 65. (4) Except U, all others are seated away from extreme ends. 66. (3) P is sitting second to the right of Q. 67. (2) T is facing N. 68. (1) Two persons – X, S – sit between W and V. U sits second to the left of W. W faces R. R is an immediate neighbour of both P and N. 69. (5) M is facing U. (70 – 83) :

    M I

    77. (5) Except G, all others are sitting at the extreme ends of the rows. 78. (4) B is sitting to the immediate right of C. 79. (1) E is facing Q. (80–84) :

    U

    V

    W

    A

    M

    90. (3) Only E sits between C and A. 91. (4) A sits second to the left of C. D faces North. Only F sits between G and D. 92. (5) E sits fourth to the left of D. 93. (1) Except F, all others are facing south. 94. (2) E is the immediate neighbour of A. H is the immediate neighbour of B. (95–99) :

    A Rajasthan B C Manipur Kerala

    N

    O

    P

    J

    K L

    70. (3) M and O are immediate neighbours of I. I faces outside Either one or five person(s) sit(s) between I and N. 71. (5) J is sitting to the immediate right of L. 72. (4) O is sitting to the immediate left of K. 73. (1) P and M are immediate neighbours of N. 74. (2) Except P, all others face the centre. P faces outside. (75-79) :

    B

    X

    J

    80. (1) Either one or five person(s) sit(s) between A and U. 81. (2) V faces the centre. J sits third to the right of V. 82. (5) When counted in anti-clockwise direction from W, five persons — U, V, M, B and J — sit between X and W. 83. (3) J sits opposite to U. 84. (4) X is sitting to the immediate left of J. (85–89) : L N Q H O G F L E F T

    85. (2) Alphabetical order

    N Q H O G F L

    F G H L

    75. (3) P is facing C. 76. (2) C faces P. B and G are immediate neighbours of C. Only U sits between P and T. P sits to the immediate left of U. R and U are immediate neighbours of P.

    M R I G H T

    M

    M N O Q

    The position of only H will remain unchanged. 86. (1) N and M are sitting at the extreme ends of the line. 87. (3) Q sits second to the left of O. H sits second to the left of G. O sits second to the left of F. 88. (5) Five persons — H, O, G. F and L — are sitting between Q and M. 89. (4) Only two persons — N, Q — are sitting to the left of H. (90–94) : A E C G F D H B

    BPRE–417

    H Sikkim

    E Odisha D Punjab

    G Assam

    F Nagaland

    95. (3) H visited Sikkim. There are two persons between H and B in anticlockwise direction. G visited Assam. There are two persons between B and G in anticlockwise direction. There are two persons between G and C in anticlockwise direction. E visited Odisha. There are two persons between C and E in anticlockwise direction. There are two persons between E and F in anticlockwise direction. 96. (2) G visited Assam. D sits to the immediate left of G. 97. (5) H sits to the immediate right of F. F visited Nagaland. 98. (1) E visited Odisha. B sits to the immediate left of E. B visited Manipur. 99. (4) D visited Punjab. Two persons – F and G – sit between H and D, when counted from the left of H. (100–104) :

    R I G H T L E F T

    Row–1

    V

    W

    U

    T

    X

    S

    M

    N

    J

    O

    L

    K

    Row–2

    L E F T R I G H T

    RANKING/ARRANGEMENT

    C

    P D

    M

    N

    G

    B

    L A

    Row–1

    O

    J

    F

    E R I G H T

    115. (5) N faces B. 116. (2) N faces B. B sits to the immediate left of A. L and O sit between N and J. Only G sits between B and D. M and N sit between L and P. J sits second to the left of L. 117. (3) M sits second to the left of K. G sits second to the left of A. D sits second to the left of B. 118. (1) J sits at the extreme left end of the Row–2. 119. (4) K sits at the extreme right end of the Row–2. (120-124): Box A

    B C

    E

    F

    A

    Box D Box C Box Q Box S Box R Box P

    H

    D

    J

    G

    110. (2) A sits third to the right of E. 111. (4) F sits to the immediate left of E. 112. (1) B sits exactly between C and E. 113. (5) H sits to the immediate right of A. 114. (3) Except A–C, in all other pairs one person sits between the two given persons.

    Buy Today Quickest Mathematics

    PRACTICE WORK BOOK

    Box B 120. (2) Four boxes – A, D, C and Q – are kept above box S. Two boxes are kept below box R and two boxes are kept above box C. Box C is kept between box D and box Q. Box A is kept at the top of the stack. 121. (1) Two boxes – D and C – are kept between box A and box Q. 122. (3) Only one box is kept above box D. Only one box is kept below box P. 123. (2) Except C–P, in all others there is only one box between the two given boxes. There are three boxes between box C and box P. 124. (4) Box R is kept immediately below box S.

    BPRE–418

    G

    B

    F

    C

    E

    A H

    Husband-Wife

    F

    So n

    K

    D

    L E F T

    B

    Wife-Husband

    C

    G

    Wife-Husband

    A

    E

    Son

    105. (1) Q sits third to the left of P. 106. (3) Two persons – Q and A– sit between R and T. 107. (4) D and E are sitting at the extreme ends. 108. (5) C sits second to the right of S. 109. (2) Two persons sit to the left of B. Two persons sit to the right of R. (110-114):

    L E F T

    Row–2

    er

    R I G H T

    R I G H T

    ht ug Da

    L D S B C T Q A R P E E F T

    (125–129) :

    (115-119) :

    Daughter

    100. (5) Except U, all others are sitting at the extreme ends of the rows. 101. (2) S is facing K. 102. (1) O faces T. T is an immediate neighbour of U. L sits second to the right of J. X faces L. N is an immediate neighbour of both M and J. 103. (3) S sits to the immediate left of X. 104. (4) J is facing U. (105-109) :

    D

    H

    125. (5) A is daughter of F and C. 126. (3) A is sitting second to the right of E. A is daughter of C. 127. (4) H is sitting opposite to D. H is cousin of D. 128. (2) F is the grandfather and C is the grandmother in the family. Only three persons are sitting between F and C. 129. (4) B is the daughter-in-law in the family. F sits to the immediate right of B. F is the grandfather in the family. (130–134) :

    1 L

    2

    3

    4

    5 K

    6

    7 F

    8

    9

    10

    11

    12

    13

    14

    15 S

    16 A

    17

    18

    19

    20

    130. (2) There are 20 students in the class. 131. (4) F occupies 7th rank. 132. (3) K is 16th from the last. 133. (1) A occupies 16th rank. 134. (5) S occupies 15th rank. ❐❐❐

    RANKING/ARRANGEMENT

    MODEL EXERCISES Directions (1-4) : Read the following information carefully and answer these questions. Six boys A, B, C, D, E and F are marching in a line. They are arranged according to their height, the tallest being at the back and the shortest in front. • F is between B and A • E is shorter than D but taller than C who is taller than A. • E and F have two boys between them. • A is not the shortest among them. 1. Where is E ? (1) Between A and B (2) Between C and A (3) Between D and C (4) In front of C (5) None of these 2. Who is the tallest? (1) B (2) D (3) F (4) A (5) None of these 3. If we start counting from the shortest, which boy is fourth one in the line? (1) E (2) A (3) D (4) C (5) None of these 4. Who is the shortest ? (1) C (2) D (3) B (4) F (5) None of these Directions (5-8) : A goldsmith has five gold rings, each having a different weight. They are labelled as D, E, F, G and H. Their peculiarities are given in the following 5 statements. Statement 1 Ring D weighs twice as much as ring E. Statement 2 Ring E weighs four and a half times as much as ring F. Statement 3 Ring F weighs half as much as ring G. Statement 4 Ring G weighs half as much as ring H. Statement 5 Ring H weighs less than ring D but more than ring F. 5. If these rings are sold according to their weights, which ring will fetch the highest value in rupees? (1) D (2) G (3) F (4) H (5) None of these

    6. Ring H is heavier to which of the following two rings ? (1) GE (2) GF (3) DF (4) DE (5) None of these 7. Which of the following is the lightest in weight? (1) D (2) E (3) F (4) G (5) None of these 8. Which of the following represents the descending order of the weights of the rings ? (1) D, E, H, G and F (2) F, D, G, E and H (3) H, F, G, D and E (4) E, G, H, D and F (5) None of these 9. Five newlyborn babies were weighed by the doctor. In her report, she stated that child A is lighter than child B. Child C is lighter than child D. Child B is lighter than child D, but heavier than child E. Which child is the heaviest ? (1) E (2) D (3) C (4) A (5) None of these 10. A, B, C, D and E when arranged in descending order of their weight from the top, A becomes third E, is between D and A while C and D are not at the top. Who among them is the second ? (1) C (2) B (3) E (4) Data inadequate (5) None of these 11. Thirty-six vehicles are parked in a parking lot in a single row. After the first car, there is one scooter. After the second car, there are two scooters. After the third car, there are three scooters and so on. Work out the number of scooters in the second half of the row. (1) 10 (2) 12 (3) 15 (4) 17 (5) None of these 12. In a row at a bus stop, A is 7th from the left and B is 9th from the right. Both of them interchange

    BPRE–419

    their positions and thus A becomes llth from the left. How many people are there in that row ? (1) 18 (2) 19 (3) 20 (4) 21 (5) None of these Directions (13-17) : Study the following information carefully to answer these questions. (I) M, N, P, Q, S and T are six members of a group in which there are three female members. Females work in three departments Accounts, Administration and Personnel and sit in three different floors Ist, IInd and IIIrd. Persons working in the same department are not on the same floor. On each floor, two persons work. (II) No two ladies work in the same department or on the same floor. N and S work in the same, department but not in Personnel. Q works in Administration. S and M are on the 1st and IIrd floors respectively and work in the same department. Q, a lady, does not work on IInd floor. P, a man, works on 1st floor. 13. Which of the following groups of persons is females ? (1) SQT (2) QMT (3) QPT (4) Data inadequate (5) None of these 14. T works in which department ? (1) Accounts (2) Administration (3) Personnel (4) Accounts or Personnel (5) None of these 15. Which of the following pairs of persons works on IInd floor ? (1) PT (2) SM (3) QN (4) None of these (5) All of these 16. If T is transferred to Accounts and S is transferred to Administration, who is to be transferred to Personnel to maintain the original distribution of females on each floor ?

    RANKING/ARRANGEMENT (1) P (2) N (3) Q (4) Data inadequate (5) None of these 17. Which of the following pairs of persons works in Administration? (1) QP (2) QN (3) SP (4) Data inadequate (5) None of these Directions (18-20) : Read the following statements to answer these questions. Nine cricket fans are watching a match in a stadium. Seated in one row, they are J, K, L, M, N, O, P, Q and R. L is at the right of M and at third place at the right of N. K is at one end of the row. Q is immediately next to O and P. O is at the third place at the left of K. J is right next to left of O. 18. Who is sitting in the centre of the row? (1) L (2) O (3) J (4) Q (5) None of these 19. Who is at the other end of the row? (1) R (2) J (3) P (4) N (5) None of these 20. Which of the following statements is true ? (1) R and P are neighbours (2) There is one person between L and O (3) M is at one extreme end (4) N is two seats away from J (5) None of these 21. A, B, C, D, E and F are sitting around a round table. A is between E and F, E is opposite D, and C is not in either of the neighbouring seats of E. Who is opposite to B ? (1) F (2) C (3) D (4) None of these (5) None of these 22. Seven men-A, B, C, D, E, F and G are standing in a queue in that order. Each one is wearing a cap of different colour-violet, indigo, blue, green, yellow, orange and red. D is able to see in front of him green and blue, but not vio-

    let. F can see violet and yellow, but not red,. G can see caps of all the colours other than orange. If E is wearing an indigo coloured cap, then the colour of the cap worn by F is (1) Blue (2) Violet (3) Red (4) Orange (5) None of these 23. In a queue of children, Kashish is fifth from the left and Mona is sixth from the right. When they interchange their places among themselves, Kashish becomes thirteenth from the left. Then, what will be Mona’s position from the right? (1) 4th (2) 14th (3) 8th (4) 15th (5) None of these Directions (24-26) : Read the information given below to answer these questions. (i) There is a group of five girls (ii) Kamini is second in height but younger than Rupa. (iii) Pooja is taller than Monika but younger in age. (iv) Rupa and Monika are of the same age but Rupa is tallest among them. (v) Neelam is taller than Pooja and elder to Rupa. 24. If they are arranged in the ascending order of height, then who will be in the third position ? (1) Monika (2) Monika or Rupa (3) Rupa (4) None of these (5) All of these 25. If they are arranged in the descending order of their ages, then who will be in the third position? (1) Monika or Rupa (2) Monika (3) Kamini (4) None of these (5) All of these 26. To answer the question “who is the youngest person in the group”, which of the given statements is superfluous? (1) Only (i) (2) Only (v) (3) Only (ii) (4) Either (i) or (iv) (5) None of these

    BPRE–420

    Directions (27-31) : Read the information given below and answer the questions that follow. (i) There is a group of seven persons A, B, C, D, E, F and G. (ii) There are four males, three females, two married couples and three unmarried persons in the group. (iii) The seven persons are seated in a row on the bench. (iv) Their professions are engineer, teacher, doctor, psychologist, businessman, architect and student. (v) B is not married and another person, the psychologist, is the most intelligent. (vi) The engineer is married to the teacher, who is the least intelligent of the group. (vii) D is an architect. He is sitting on the leftmost corner (viii) The student is sitting on the rightmost corner of the bench. (ix) The doctor is married to C. C is the second most intelligent of the group followed by her husband. (x) The least intelligent of the group is sitting on the immediate right of D, followed by the most intelligent. (xi) There are as many more intelligent persons than the engineer as there are less intelligent. (xii) On the bench, followed by D, there are three females sitting in succession. (xiii) The psychologist is a female. (xiv) The student is more intelligent than the architect, who is more intelligent than only one person, F. (xv) Neither A nor G is a female. 27. Who is sitting on the immediate right of D? (1) F (2) E (3) C (4) Can’t say (5) None of these 28. Which two are sitting together? (1) D and E (2) E and A (3) Teacher and businessman (4) Engineer and doctor (5) None of these 29. The engineer is not more intelligent than (1) student (2) architect (3) teacher (4) businessman (5) None of these

    RANKING/ARRANGEMENT 30. Which of these cannot be a married couple? (1) A-F (2) A-C (3) G-F (4) None of these (5) None of these 31. Which of these represents the correct order of intelligence (in the decreasing sequence) ? (1) EBF (2) CDF (3) ECB (4) None of the above (5) All of these 32. In a class of 35 students, Kiran is placed 7th from the bottom whereas Mohan is placed 9th from the top. Sohan is placed exactly in between Kiran and Mohan. What is Kiran’s position from Sohan ? (1) 13th (2) nth (3) 10th (4) 9th (5) None of these Directions (33-34) : There are six steps on a staircase leading from the ground floor to the first floor. Denote the first step by 1, second by 2, and so on. There are four people P, Q, R and S. No two people can be on the same step. I. P is two steps below R. II. Q is on the next step to S. ; III. R is two steps below S. 33. If P is on the first step, on which step is Q? (1) Third (2) Fourth (3) Fifth (4) Sixth (5) None of these 34. If P is on the first step, which of the following is true? (1) Q is on the second step (2) R is on the fourth step (3) S is on a lower step than R (4) Step four is empty (5) None of these Directions (35-38) : Study the following information to answer these questions. A Blacksmith has five iron articles A, B, C, D and E, each having a different weight. I. A weighs twice as much as B. II. B weighs four and a half times as much as C. III. C weighs half as much as D. IV. D weighs half as much as E. V. E weighs less than A but more than C.

    35. Which of the following is the highest in weight? (1) A (2) B (3) C (4) D (5) None of these 36. E is heavier than which of the following two articles? (1) D, B (2) D, C (3) A, C (4) A, B (5) None of these 37. Which of the following articles is the heaviest in weight? (1) A (2) B (3) C (4) D (5) None of these 38. Which of the following represents the descending order of weight of the Article? (1) A, B, E, D, C (2) B, D, E, A, C (3) E, C, D, A, B (4) C, A, D, B, E (5) None of these 39. Five students participated in the scholarship examination. “Sudha scored higher than Puja. Kavita scored lower than Suma but higher than Sudha. Mamta scored between Puja and Sudha.” Who scored lowest in the examination? (1) Puja (2) Kavita (3) Mamta (4) Sudha (5) None of these 40. Samant remembers that his brother’s birthday is after fifteenth but before eighteenth of February, whereas his sister remembers that her brother’s birthday is after sixteenth but before nineteenth of February. On which date of February is Samant’s brother’s birthday? (1) 18th (2) 16th (3) 19 th (4) 17th (5) None of these 41. In a class of 40 children, Sunita’s rank is eighth from the top. Sujit is five ranks below Sunita. What is Sujit’s rank from bottom? (1) 26 (2) 27 (3) 28 (4) 29 (5) None of these 42. There are 30 plants of Chiku, Guava, Sitafal and Mango in a row. There is one pair of Mango plants after Chiku and Guava and Mango plants are followed by one Chiku and one Sitafal plant and

    BPRE–421

    so on. If the row begins with a plant of Chiku, then which of the following will be in the last of the row? (1) Chiku (2) Guava (3) Mango (4) Sitafal (5) None of these 43. Mohan is thirteenth from the left end in a row of children. Prabir is twelfth from the right end and eighteenth from the left end. How many children are towards the right of Mohan in that row? (1) 12 (2) 17 (3) 18 (4) None of these (5) None of these Directions (44-47) : Answer these questions based on the information given. Five boys Amit, Suresh, Vishnu, Ujjwal and Rakesh are sitting in a row facing south. Five girls Alka, Jyoti, Veena, Prabha and Nayan are sitting in front of them in a row, each one facing one of them. All these are not necessarily in the same serial order. Vishnu is facing Veena. Suresh and Rakesh are not facing Prabha. Amit is at one of the ends. Nayan is fourth to the right of Alka and is facing Rakesh. Jyoti is between Alka and Veena. 44. Which pair occupies the middle position of the rows? (1) Rakesh, Nayan (2) Suresh, Prabha (3) Veena, Vishnu (4) Ujjwal, Jyoti (5) None of these 45. Who is facing Jyoti? (1) Ujjwal (2) Amit (3) Amit or Ujwal (4) Suresh (5) None of these 46. Which of the following pairs are sitting diagonally opposite? (1) Rakesh, Alka (2) Amit, Alka (3) Nayan, Rakesh (4) Suresh, Nayan (5) None of these 47. Which of the following is Prabha’s position? (1) Between Jyoti and Nayan (2) Immediate left of Nayan (3) Facing Suresh (4) Second to the right of Alka (5) None of these

    RANKING/ARRANGEMENT 48. Five senior citizens are living in a multi-storied building. Mr Bhagat lives in a flat above Mr Ashokan, Mr Lokesh lives in a flat below Mr Gaurav, Mr Ashokan lives in a flat above Mr Gaurav and Mr Rakesh lives in a flat below Mr Lokesh. Who lives in the topmost flat? (1) Mr Bhagat (2) Mr Gaurav (3) Mr Lokesh (4) Mr Rakesh (5) None of these 49. Kittu is in between Mohan and Sohan. Raju is to the left of Sohan and Shyam is to the right of Mohan. If all of the friends are sitting facing North, then who is on their extreme right? (1) Kittu (2) Sohan (3) Mohan (4) Shyam (5) None of these 50. Among five boys, Vineet is taller than Manick, but not as tall as Ravi. Jacob is taller than Dilip but shorter than Manick. Who is the tallest in their group? (1) Ravi (2) Manick (3) Vineet (4) Dilip (5) None of these 51. Persons X, Y, Z and Q live in red, green, yellow or blue coloured houses placed in a sequence on a street. Z lives in a yellow house. The green house isadjacent to the blue house. X does not live adjacent to Z. The yellow house is in between the green and red house. The colour of the house in which X lives is (1) yellow (2) green (3) red (4) None of these (5) All of these 52. Eight friends, namely P, Q, R, S, T, U, V and W are sitting around a circular table facing towards the centre. Q is sitting between V and S. W is third to the left of Q and second to the right of P R is sitting between P and V. Q and T are not sitting opposite to each other. Who is sitting third to the left of S? (1) T (2) P (3) U (4) S (5) None of these 53. Six children B, D, C, M, J and K are split into two groups of three each and are made to stand in

    two rows in such a way that a child in one row is exactly facing a child in the other row. M is not at the ends of any row and is to the right of J, who is facing C. K is to the left of D, who is facing M. Which of the following groups of children are in the same row? (1) MJK (2) BMD (3) BDC (4) None of these (5) All of these 54. If every even numbered letter starting from B is replaced by odd numbers 3, 5, 7, ... etc respectively, then which letter/digit will be 3rd to the right of the letter/ digit which is 10th from the right? (1) 23 (2) 11 (3) 22 (4) None of these (5) 24 55. In a queue, Sadiq is 14th from the front and Joseph is 17th from the end, while Jane is in between Sadiq and Joseph. If Sadiq be ahead of Joseph and there be 48 persons in the queue, how many persons are there between Sadiq and Jane? (1) 5 (2) 6 (3) 7 (4) 8 (5) None of these 56. (i) Six friends A, B, C, D, E and F are seated in a circle facing each other. (ii) A is between D and B and F is between C and E. (iii) C is the third to the left of B. Which of the following is the position of in A relation to F? (1) Second to the left (2) Second to the right (3) Fourth to the right (4) Third to the right (5) None of these 57. Ages are to be computed in whole numbers only and no two persons are of the same age. Mahesh is a year older than Vikas. Vikas is two years older than Jagan. Jagan is a year younger than Suresh. Suresh is two years younger than Mahesh. Akmal is two years younger than Jagan. Which of the following is in the order from the oldest to the youngest? (1) Mahesh, Vikas, Jagan, Suresh, Akmal

    BPRE–422

    (2) Mahesh, Vikas, Suresh, Akmal, Jagan (3) Mahesh, Vikas, Suresh, Jagan, Akmal (4) Mahesh, Jagan, Vikas, Akmal, Suresh (5) None of these 58. There are five different houses A to E in a row. A is to the right of B; E is to the left of C and right of A; B is to the right of D. Which of the houses is in the middle? (1) D (2) B (3) A (4) E (5) None of these 59. A seven member panel is sitting in a row, so that (i) A is in between D and F (ii) C is in between F and G (iii) G is in between C and E (iv) D is in between B and A . Who is sitting in the middle? (1) G (2) F (3) C (4) B (5) None of these 60. Four girls are sitting on chairs for a group photograph. (i) Sheela is to the left of Rani, (ii) Meena is, to the right of Rani, (iii) Rita is in between Rani and Meena. Who would be second from the left in the photograph? (1) Meena (2) Sheela (3) Rani (4) Rita (5) None of these 61. P, Q, R, S, T, U and V are sitting in a circle facing the centre. S, who is second to the right hand side of R, is not to the immediate right to V. U is not between V and T. P is between R and Q. If Q and R interchange places so as to be like T and V then which of the following is true? (1) S is third to the right of R (2) T is second to the left of R (3) Q is fourth to the right of T (4) S is second to the left of R (5) None of these 62. Five boys are standing in a line facing the wall. Each boy is either wearing red, green, yellow, white or blue dress. The boy dressed in yellow is not standing at any end of the line. The boy dressed in red is not standing at any end of the line. What colour

    RANKING/ARRANGEMENT dress has been worn by the boy who is standing to the right side of the boy dressed in yellow? (1) White (2) Green (3) Blue (4) Cannot be determined (5) None of these 63. Five shirts are placed on a shelf in a heap. Red shirt is above the Blue shirt and Green shirt is below the Orange shirt. Blue shirt is above the Orange shirt and White shirt is below the Green shirt. The colour of the shirt between Red and Orange shirts is (1) White (2) Green (3) Blue (4) Red (5) None of these 64. Five persons are standing in a line in a ceremony. One of the two persons standing at the extreme ends is a General Manager and the other one is a Senior Manager. The Deputy General Manager is standing to the right of the Assistant General Manager. A Deputy Manager is to the left of the Senior Manager. The Assistant General Manager is standing between the General Manager and the Deputy General Manager. Counting from the left, what is the position of the Deputy General Manager? (1) First (2) Third (3) Second (4) Fifth (5) None of these 65. Five friends namely L, M, N, R and S are standing in a row facing south, but not necessarily in the same order. ˆ Only M is between L and S. • N is to the immediate right of S. • R is to the immediate left of L. Who is occupying the middle position in the row? (1) L (2) M (3) S (4) R (5) None of these 66. Gurmeet is taller than Jeet but not as tall as Geeta. Mona is taller than Gurmeet but not as tall as Nene. Geeta is shorter than Mona. Who amongst them is the tallest ? (1) Gurmeet (3) Nene

    (2) Mona (4) Jeet

    (5) Cannot be determined

    Directions (67-68) : Study the following information carefully and answer the questions given below : Six persons - C, D, E, F, G and H - are standing in a straight line facing north but not necessarily in the same order. D is standing second to the right of F. C is standing fourth to the left of H. H is not standing at any extreme ends of the line. E is standing second to the right of D. 67. Who among the following are standing at the extreme ends of the line ? (1) DE (2) CF (3) GC (4) CE (5) EG 68. Who among the following is standing third to the right of F ? (1) H (2) C (3) E (4) G (5) D Directions (69–73) : Study the following information carefully and answer the questions given below : Eight persons — M, N, O, P, Q, R, S and T — are sitting in a straight line with equal distances between each other, but not necessarily in the same order. Some of them are facing north and some are facing South. ● M sits at one of the extreme ends of the line. Only three persons sit between M and S. Q sits exactly between M and S. ● T sits third to the right of Q. N is an immediate neighbour of T and faces south. O sits second to the right of R. O is not an immediate neighbour of S. ● Immediate neighbours of S face opposite directions (i.e. if one neighbour faces north then the other neighbour faces south and vice-versa). Immediate neighbours of R face opposite directions (i.e. if one neighbour faces north then the other neighbour faces south and vice-versa). ● M and P face the same direction as Q (i.e. If Q faces north then M and P also face north and viceversa.) Both the immediate neighbours of Q face south. 69. In the given arrangement, if two persons come and sit to the immediate left of Q, how many persons will sit between R and O?

    BPRE–423

    (1) Two (2) Three (3) Four (4) More than four (5) One 70. Who amongst the following sits third to the right of R? (1) S (2) Q (3) Other than those given as options (4) N (5) M 71. How many persons face north as per the given arrangement? (1) Two (2) Three (3) Four (4) More than four (5) One 72. Four of the following five are alike in a certain way based upon their seating arrangement and so form a group. Which of the following does not belong to the group? (1) QO (2) MR (3) NR (4) OS (5) PS 73. How many persons sit between S and N as per the given arrangement? (1) None (2) Two (3) More than three (4) One (5) Three Directions (74–79) : Study the following information carefully and answer the questions given below : Eight persons – P, Q, R, S, T, U, V and W – are sitting around a circular area at equal distance between each other but not necessarily in the same order. Some people face the centre while some face outside (i.e. in a direction opposite to the centre). Q sits seconds to the right of T. T faces outside. Only two persons sit between Q and W. Both the immediate neighbours of Q faces the centre. Both the immediate neighbours of R face outside. R is neither an immediate neighbour of Q nor V. Only two persons sit between R and V. S sits second to the left of P. Immediate neighbours of W face same direction (i.e. If one neighbour faces the centre then the other also faces centre and vice–versa.) T sits to immediate left of one of his neighbours. W faces the same direction as R (i.e. if R faces the centre, then W also faces centre and vice–versa.)

    RANKING/ARRANGEMENT 74. What is R’s position with respect to W? (1) Second to the left (2) Second to the right (3) Fourth to the right (4) Fifth to the right (5) Third to the right 75. Which of the following is true regarding U as per the given arrangement? (1) None of the given options is true (2) Only three persons sit between U and Q. (3) U faces the centre (4) S is one of the immediate neighbours of U. (5) U sits third to the left of T. 76. Who amongst the following sit exactly between R and V when counted from the right of R? (1) P, U (2) U, W (3) P, Q (4) Q, T (5) S, T 77. Four of the following five are alike in a certain way based on the given seating arrangement and so form a group. Which is the one that does not belong to that group? (1) W (2) P (3) U (4) Q (5) S 78. Who amongst the following are immediate neighbours of T ? (1) S, V (2) U, V (3) R, S (4) P, R (5) V, W 79. Who is sitting to the immediate left of V? (1) W (2) P (3) T (4) Q (5) S Directions (80–84) : Study the following information carefully and answer the questions given below : Nine persons – P, Q, R, S, T, U, V, W and X – are seated in a straight line facing north but not necessarily in the same order. T sits fourth to the left of W. Neither T nor W sits at the extreme ends of the line. Only one person sits between X and R. Neither X nor R is an immediate neighbour of W.

    P sits second to the right of X. Only two persons sit between P and S. Q is not an immediate neighbour of S or P. V is not an immediate neighbour of X. 80. What is the position of V with respect to Q? (1) Fifth to the right (2) Immediate neighbour (3) Second to the right (4) Third to the right (5) Other than those given as options 81. Which of the following represents persons seated at two extreme ends of the line? (1) S, R (2) Q, S (3) R, P (4) X, Q (5) V, X 82. How many persons are seated between R and P? (1) One (2) Two (3) Three (4) Four (5) None 83. If T is related to V and Q is related to P in a certain way, to which of the following would X be related to following the same pattern? (1) Q (2) T (3) R (4) S (5) W 84. Who amongst the following sits exactly in the middle of the persons who sits seventh from the left and the person who sits sixth from the right? (1) P (2) Q (3) T (4) S (5) V Directions (85–88) : Study the following arrangement carefully and answer the questions given below : A$H★JRS2&!T6@QBKFG UMI©#%E4N59D3LW7 85. What should come in place of the question mark (?) in the following series based on the above arrangement? H ★ 2 RST !TB BKM ? (1) © # E (2) M I % (3) # % N (4) I © 4 (5) None of these

    BPRE–424

    86. If all the letters are dropped from the arrangement, which of the following will be the ninth from the right end? (1) # (2) @ (3) © (4) 6 (5) None of these 87. Four of the following five are alike in a certain way based on their positions in the above arrangement and so form a group. Which is the one that does not belong to the group? (1) AJHS (2) G©M% (3) TB@F (4) 5DL7 (5) %54D 88. How many such numbers are there in the above arrangement, each of which is immediately preceded by a vowel and also immediately followed by a consonant? (1) None (2) One (3) Two (4) Three (5) More than three Directions (89 – 93) : Study the following information carefully and answer the questions given below : A % B A is sitting third to the right of B and A is facing away from the centre A + B A is immediate right of B and A is facing centre A # B A is second to the right of B and A is facing away from the centre A ! B A is sitting third to the left of B and A is facing the centre A $ B A is second to the left of B and A is facing the centre A @ B A is immediate left of B and A is facing away from the centre A & B A is opposite to B and A is facing the centre A * B A is sitting third to the right of B and A is facing the centre A ^ B A is sitting third to the left of B and A is facing away from the centre Give answer (1) if only Conclusion I is true Give answer (2) if only Conclusion II is true Give answer (3) if either Conclusion I or Conclusion II is true Give answer (4) if neither Conclusion I nor Conclusion II is true Give answer (5) if both the Conclusion I and Conclusion II are true

    RANKING/ARRANGEMENT There are eight friends sitting around a round table. Four of them are facing the centre and four of them are facing away from the centre. K@M*Q+Z%D$T^E$N 89. Statement : With respect to K and Q Conclusions : I. Q is sitting second to the right of D. II. K is sitting second to the right of Q. 90. Statement : With respect to N and T Conclusions : I. N is sitting to the immediate right of T. II. T is sitting to the immediate right of M.

    91. Statement: With respect to Z and D Conclusions : I. Z is sitting third to the right of T. II. D is sitting second to the right of T. 92. Statement : With respect to E and M Conclusions : I. M is sitting opposite to E II. E is sitting exactly between Q and D 93. Statement : With respect to M and N Conclusions : I. M is sitting second to the right of Z II. N is sitting third to the left of Q

    Directions (94–98) : Study the following information carefully and answer the questions that follow : Twelve friends D, E, F, G, H, I, J, K, L, M, N and O were born in different months starting from January to December of the same year. All the friends are sitting around a rectangular table as per the arrangement given below. Each person is recognized with different personality traits assigned according to the month wise astrology. ch ar M

    er er mb tob Ap pte Oc ril Se

    August

    November

    July

    December

    Febr uary

    June

    May

    January

    G is sitting second to the right of a person who is sitting third to the left of L. ● H is sitting to the immediate left of a person who is sitting third to the right of E. ● I is sitting second to the right of a person who is sitting third to the left of N. ● F is sitting opposite to a person who is sitting to the immediate right of a person who is youngest among all. ● E is sitting second to the left of a person who is sitting opposite to the person born in June ● K is sitting to the immediate right of a person who is third to the right of a person born in September. O is an immediate neighbour of either F or N. ● The eldest person is sitting third to the left of a person who is to the immediate left of J. ● The person opposite to D is sitting to the immediate left of person born in November. ● The person born in March is fourth to the right of N. ● The person born in April is second to the right of a person who is sitting to the immediate right of M. Refer the following table for the personality traits according to month wise astrology : ●

    January February March April May June Optimistic Independent Sensible Dynamic Traditional Polite July August September October November December Caring Brave Calm Loyal Empathetic Honest

    BPRE–425

    94. Who among the following is sitting third to the right of a Honest person? (1) G (2) J (3) D (4) H (5) I 95. Who among the following is sitting exactly between a Sensible person and a Caring person? (1) H (2) L (3) N (4) K (5) F 96. Which of the following statements is CORRECT with respect to G, according to the given arrangement? (1) G is second to the right of a Caring person (2) G is opposite to a Loyal person (3) G is to the immediate left of a Honest person (4) G is to the immediate right of a Polite person (5) G is third to the left of a Sensible person 97. Who among the following is just elder to the Brave person and to the immediate left of an Independent person? (1) I (2) H (3) D (4) M (5) E 98. Suppose, F was born in the month in which L was born and L was born in a month in which F was born, then what is the personality trait of F in the given arrangement? (1) Brave (2) Traditional (3) Dynamic (4) Loyal (5) Optimist Directions (99–103) : Study the following information carefully and answer the questions that follow : A%B

    A+B A#B

    A!B

    A$B

    A is sitting third to the right side of B and A is facing away from the centre A is to the immediate right of B and A is facing centre A is second to the right of B and A is facing away from the centre A is sitting third to the left side of B and A is facing the centre A is second to the left of B and A is facing the centre

    RANKING/ARRANGEMENT A@B

    A&B A*B

    A^B

    A is to the immediate left of B and A is facing away from the centre A is opposite to B and A is facing the centre A is sitting third to the right side of B and A is facing the centre A is sitting third to the left side of B and A is facing away from the centre

    Give answer (1) if only Conclusion I is true Give answer (2) if only Conclusion II is true Give answer (3) if either Conclusion I or Conclusion II is true Give answer (4) if neither Conclusion I nor Conclusion II is true Give answer (5) if both the Conclusion I and Conclusion II are true 99. Statement : Q&U$S!N+G*L Conclusions : I. G is second to the right of S. II. L is second to the right of N. 100. Statement : I#B*H$M+G&Q Conclusions : I. G is third to the left of Q. II. G is third to the right of Q. 101. Statement : T#G$L^N$B$K Conclusions : I. N is second to the right of K. II. L is second to the right of G. 102. Statement : U@M$T+H$N+O Conclusions : I. H is to the immediate left of M. II. N is second to the right of H. 103. Statement : F#G$M^H$N$B Conclusions : I. G is second to the left of M II. N is third to the right of G Directions (104–108) : Study the following information carefully and answer the questions given below : There are eight family members Sudha, Sundar, Mohan, Meghna, Raj, Lata, Rohit and Suman are sitting around a circular table facing the centre. There are three couples and each couple, has at least one child.

    Only one couple is sitting next to each other. Sundar is second to the left of Raj. Mohan is uncle of Suman’s son. ● Meghna is opposite to the eldest person in the family. Rohit calls Sudha as aunt. ● Lata is third to the left of Meghna but not an immediate neighbour of Sudha. The person second to the left of Suman is the eldest woman in the family. Raj sits to the immediate left of his wife. ● Sundar sits opposite to Rohit whose father is not Mohan. Sudha is not related with anyone in the family except Meghna. ● The son in law of the family is sitting immediate right to a person who is opposite to Meghna’s father’s mother. ● Sudha is not an immediate neighbour of Meghna but is the neighbour of either Sundar or Suman. 104. How is the person to the immediate left of Suman related to Mohan? (1) Son (2) Father (3) Cousin (4) Nephew (5) Niece 105. Who is sitting third to the left of daughter–in–law of the family? (1) Meghna (2) Rohit (3) Mohan (4) Suman (5) Raj 106. The person between Sundar and granddaughter in the family is sitting third to the right of : (1) Lata (2) Grandfather (3) Grandson (4) Suman (5) Raj 107. Four of the following five are alike in a certain way based on their positions in the given arrangement and so they form a group. Which is the one that does not belong to the group? (1) Mohan – Meghna (2) Suman – Rohit (3) Raj – Rohit (4) Lata – Sundar (5) Sudha – Suman 108. Who among the following is sitting opposite to the person who is to the immediate left of grandfather? ●

    BPRE–426

    (1) Rohit (2) Suman (3) Mohan (4) Lata (5) Meghna 109. There are six persons A, B, C, D, K and L are sitting around a circular table. Only two are facing away from the centre. B is to the immediate right of both K and C. A is facing away from the centre is second to the right of L who faces K. D facing centre is second to the left of C. Who among the following is sitting second to the left of D? (1) K (2) B (3) C (4) L (5) A Directions (110–104) : Study the following information carefully and answer the questions given below : There is an apartment having three floors. Each floor has four flats. The bottom floor is known as row–1 and the top floor is known as row–3. The left end floors are in column 1 and the right end floors are in column 3. Twelve persons D, E, F, G, H, I, J, K, L, M, N and O lives in different flat (not in the same order).

    Columns Rows

    ● All

    the flats are facing the north direction ● H is in immediate right of a flat that is immediate above left diagonally of F’s flat ● D is in immediate right below diagonally of O’s flat ● M is in the same row where J is living. ● K is in the same column where N is living. ● I is immediate right of K who is immediate left below diagonally of M ● N is second right of a flat that is two flats above the flat of L in the same column ● G is two flats above a flat that is second right of F’s flat ● D is immediate left above diagonal of F who is immediate right below diagonally of J ● O is in immediate left of a flat that second right of E’s flat

    RANKING/ARRANGEMENT 110. Who among the following is living in immediate right above diagonally of D’s flat? (1) O (2) I (3) F (4) G (5) M 111. Who among the following is living third to the right of J’s flat in the same row? (1) M (2) D (3) G (4) E (5) I 112. Suppose, Column 1 is directly placed just above the column 3, then who will be 5th from the bottom ? (1) D (2) J (3) E (4) K (5) H 113. Suppose, row 1 is immediately followed by row 3 who will be 7th from the right end ? (1) N (2) F (3) O (4) G (5) I 114. Who among the following is an immediate neighbour of D and also belongs to 2nd column in the apartment? (1) M (2) K (3) N (4) G (5) H Directions (115–117) : Study the following information carefully and answer the questions given below : There are two parallel rows facing each other from north to south having six seats each. Only four persons are sitting in each row. B is to the immediate right of D. K is third to the left of a seat just opposite the seat which is second to the left of A. J does not have immediate neighbours. J does not sit at any extreme end of the row. The person sitting just opposite L does not have immediate neighbours. D faces west and has only one person to his left. K is at the extreme left end of the row facing east. I sits exactly between J and K. I does not face A. No one sits just opposite C. 115. Who is sitting just opposite A ? (1) I (2) K (3) L (4) J (5) C 116. What is the position of D with respect to A ? (1) Third to the left

    (2) Second to the left (3) Immediate left (4) Third to the right (5) Immediate right 117. What is the position of I with respect to J ? (1) Second to the right (2) Immediate right (3) Third to the right (4) Immediate left (5) Second to the left Directions (118–122) : Study the following information carefully and answer the questions that follow : There are eight professionals K, L, M, N, O, P, Q and R sitting around a rectangular table (not necessarily in the same order). Only four of them are facing the centre. All the professionals are seated as given in the figure below :

    Finance

    HR















    Marketing

    Technical

    The person in each corner is the head of respective department. The persons on each side of the table are the subordinates to the concerned overlapping two departments. P, who faces away from the centre, is sitting second to the left of Q. R is subordinate to both N and Technical team head. K is second to the right of M. L receives orders whereas O gives orders. M is second to the right of a person who is to the immediate left of Marketing head’s subordinate. Immediate neighbours of N are facing away from the centre. Both Q and P receive orders from M and both L and R receive orders from N.

    BPRE–427

    R’s immediate neighbours are heads of two different departments. ● N is the head of either marketing or HR department ● All the subordinates are facing away from the centre 118. Who among the following is the head of Finance department? (1) N (2) Q (3) O (4) L (5) M 119. Who among the following is sitting third to the left of the subordinate of both HR and Technical head? (1) O (2) N (3) Q (4) R (5) K 120. Who among the following are subordinates to Finance Department? (1) N and R (2) P and R (3) L and R (4) Q and L (5) Q and P 121. What is the position of Q with respect to Technical head? (1) Third to the left (2) Second to the right (3) Third to the right (4) Second to the left (5) Immediate right 122. What is the position of M with respect to R? (1) Third to the left (2) Third to the right (3) Second to the left (4) Second to the right (5) Opposite to each other Directions (123–127) : Study the following information carefully and answer the questions that follow : Ten professionals B, D, F, H, J, L, N, P, R and T are sitting around a round table (of 20 metre) facing the centre. Six persons out of 10 professionals should have discussion about six different problems – Production, HR, Finance, Marketing, Sales and Advertisement. One person among the 10 professionals is the CEO of the company and another one is MD of the company. Refer the following figure : ●

    RANKING/ARRANGEMENT The person third to the right of CEO should be placed to the immediate left of Sales issues ● The person third to the right of MD should be placed to second to the left of Finance issues ● The immediate neighbour of both F and N should be placed second to the left of Advertisement issues ● The person third to the right of D should be placed to the immediate left of HR issues ● The person third to the left of P should be placed second to the right of Sales issues 123. Who is sitting second to the right of R in figure–2? (1) F (2) H (3) B (4) J (5) R 124. What is the position of CEO with respect to the MD in figure – 2? (1) Immediate left (2) Second to the right (3) Immediate right (4) Second to the left (5) Facing opposite 125. Who among the following is exactly sitting between Marketing and Production issues in figure– 2? (1) J (2) F (3) B (4) R (5) T 126. What is the position of B with respect to MD in the main circle (figure – 1)? (1) Second to the right (2) Second to the left (3) Third to the right (4) Immediate left (5) Immediate right 127. What is the position of CEO with respect to D in the main circle (figure – 1)? (1) Immediate right (2) Third to the right (3) Immediate left (4) Third to the left (5) Second to the left Directions (128–132) : Study the following information carefully and answer the questions that follow : ●

    CEO 2

    2

    2

    2

    2

    2 2

    2 2

    2 MD















    Figure–1 P is sitting third to the right of T and B is sitting third to the left of F. H is sitting to the immediate right of N. J is sitting to the immediate left of a person who is third to the right of R. D is to the immediate left of a person who is third to the right of CEO B is to the immediate right of a person who is second to the right of a person sitting exactly between T and L. R is second to the left of a person who is 6 metre away from D. The distance between B and CEO is equal to distance between D and CEO. Six out of 10 professionals who are sitting around the 20 metre table should be placed around the following round table (facing the centre as given below). Each of them has to represent each of six different problems placed around the following circular arrangement.

    B em tis r ve Ad

    t en

    n tio uc od Pr

    F



    T Marketing HR

    Finance R

    H

    Sales J

    Figure–2 The person second to the left of R from the 20 metre round table should be placed second to the right of Marketing issues.

    A%B

    A is sitting third to the right side of B and A is facing away from the centre

    BPRE–428

    A+B

    A#B

    A!B

    A$B

    A@B

    A&B A*B

    A^B

    A is to the immediate right of B and A is facing centre A is second to the right of B and A is facing away from the centre A is sitting third to the left of B and A is facing the centre A is second to the left of B and A is facing the centre A is to the immediate left of B and A is facing away from the centre A is opposite to B and A is facing the centre A is sitting third to the right of B and A is facing the centre A is sitting third to the left of B and A is facing away from the centre

    Give answer (1) if only Conclusion I is true Give answer (2) if only Conclusion II is true Give answer (3) if either Conclusion I or Conclusion II is true Give answer (4) if neither Conclusion I nor Conclusion II is true Give answer (5) if both the Conclusion I and Conclusion II are true 128. Statement : H @ B # P + K ^ Z # D, where D is facing the centre Conclusions : I. B and D are sitting together. II. K is second to the left of H. 129. Statement : D$M+B#L&Y@P Conclusions : I. D is second to the left of Y. II. D is exactly between P and M. 130. Statement : I#J+Q#M$B@C Conclusions : I. B is second to the left of M. II. B is second to the left of Q. 131. Statement : U*R$N+B&V+G Conclusions : I. U is second to the left of G. II. B is second to the left of U.

    RANKING/ARRANGEMENT 132. Statement : E$P^C+H@Y$V Conclusions : I. H is to immediate left of Y II. P is to immediate left of V Directions (133-137) : Study the following information carefully and answer the questions given below : There are eight people K, L, M, N, O, P, Q and R sitting around a circular table facing the centre. The table has eight seats with equal distance of 5 metre. Among the eight people, four are blood related to each other. ● L is second to the left of M who is 15 metre away from N’s father ● K is 10 metre to the left of Q whereas R is 10 metre to the right of O who is niece of M ● P is 15 metre to the left of K who is to the immediate left of a person related to R as daughter ● Q is opposite to R’s brother who is 5 metre away from both N and P ● O is 10 metre to right of her brother and 10 metre to the left of her father. Q, not blood related with R, is 10 metre to the left of L. Now some people are shifted to another circular arrangement that has 10 seats facing the centre as follows :

    Red Yellow

    Black

    Green

    Violet

    Father of N moves third to the left of Violet seat ● The person 20 metres left of M moves to the immediate right of Red seat ● The person 5 metres left of N’s sister moves third to the right of Yellow seat ● The person between M and L moves to the seat just opposite to Black seat ●

    The person 15 metres to the right of R moves between Green and Violet seats. 133. Who is sitting second to the right of R after the rearrangement ? (1) P (2) K (3) N (4) Black seat (5) Violet seat 134. How is M related to the person sitting to the immediate right of Yellow seat after the rearrangement ? (1) Son (2) Daughter (3) Father (4) Brother (5) Sister 135. Who is sitting 20 metre left of K before rearrangement? (1) M (2) N (3) Q (4) P (5) L 136. What colour seat is located third to the M’s seat after rearrangement? (1) Black (2) Green (3) Red (4) R’s seat (5) Q’s seat 137. After the rearrangement, how many seats are vacant between the seats of N and L ? (1) Two (2) Three (3) One (4) Four (5) None of these Directions (138–142) : Study the following information carefully and answer the questions given below : Seven alphabet are arranged from bottom to top numbered as 1 to 8. E is at even numbered position. M is at odd numbered position. A is at even numbered position. There is only one alphabet between D and Z. Z is above D. There is only one alphabet between G and R. G is above R. The alphabet at fourth position is the same as that of the bottommost alphabet. Z is exactly between M and D. There are only two Vowels from 1st to 3rd positions. D is exactly between two Vowels. There is no vowel at the the topmost position but there is a vowel at the bttommost position. There is only one alphabet above M. 138. What is the position of A ? (1) 4th (2) 5th (3) 2nd (4) 1st (5) 3rd 139. Which alphabet is exactly between G and A ? ●

    BPRE–429

    (1) Alphabet Z (2) Alphabet R (3) Alphabet M (4) Alphabet D (5) Alphabet E 140. What will be the word if the letters at 7-2-5-4 positions are arranged in the same order? (1) RAZE (2) MAZE (3) READ (4) DARE (5) RADE 141. What will be the word if the letters at 8-2-7-1 positions are arranged in the same order? (1) DEED (2) DEAM (3) DEEM (4) GAME (5) MEEZ 142. What will be the word if the letters at 6-4-2-3 positions are arranged in the same order? (1) DEEZ (2) MEEZ (3) REAR (4) REED (5) READ Directions (143 – 147): Study the following information carefully and answer the questions given below : Eight friends are sitting in a row facing the north as given in the following order. O T P M R Q N S These friends are to be arranged around a circular table as given below: • Only two friends face away from the centre • The person third to the left of R from the row sits to the immediate right of both M and Q • The person second to the right of P from the row sits to the immediate right of both N and O • The person fourth to the left of S from the row sits second to the right of the person who is second to the right of O from the row • O sits to the immediate left of both Q and R • N sits second to the left of S. P faces the centre • M sits third to the left of N • The persons at the extreme ends of the row sit opposite each other 143. Who is sitting to the immediate left of N around the circle? (1) R (2) M (3) P (4) O (5) S

    RANKING/ARRANGEMENT 144. Who are the immediate neighbours of M? (1) P and T (2) O and N (3) Q and S (4) R and Q (5) S and T 145. What is the position of R with respect to M? (1) Second to the right (2) Third to the left (3) Immediate right (4) Opposite to M (5) Immediate left 146. What is the position Q with respect to N? (1) Second to the right (2) Second to the left (3) Third to the right (4) Third to the left (5) Opposite to N 147. What is the position of S with respect to Q? (1) Third to the left (2) Third to the right (3) Immediate left (4) Immediate right (5) Second to the right Directions (148–152) : Study the following information carefully and answer the questions given below : There are eight students M, N, O, P, Q, R, S and T from different states viz.,West Bengal, Kerala, Tamil Nadu, Punjab, Uttarakhand, Maharashtra, Andhra Pradesh and Madhya Pradesh who are sitting around a circular table facing the centre but not necessarily in the same order. ● Q sits to the immediate right of a person who sits second to the left of M. ● N sits to the immediate left of a person who sits third to the right of O who is from either Uttarakhand or Madhya Pradesh. ● P sits opposite to a person who sits second to the left of the person from Uttarakhand. ● M is the immediate neighbour of the persons from Kerela and West Bengal. ● R sits opposite to a person who sits second to the left of a person from Kerala.

    S sits to the immediate right of a person who sits opposite to a person from Uttarakhand. ● R sits second to the right of S who sits second to the right of a person from West Bengal. ● T sits to the immediate left of a person who sits exactly between a person from Andhra Pradesh and Q. ● M sits second to the left of a person who sits exactly between a person from Punjab and S. ● The person from West Bengal sits exactly between person from Maharashtra and M. 148. Who among the following sits opposite to a person from Tamil Nadu? (1) T (2) P (3) O (4) R (5) N 149. The person from which city sits third to the left of T? (1) Kerala (2) Maharashtra (3) West Bengal (4) Madhya Pradesh (5) Punjab 150. Who among the following sits exactly between the persons from Andhra Pradesh and Punjab? (1) O (2) P (3) N (4) M (5) S 151. Who among the following are from Madhya Pradesh and Maharashtra respectively? (1) R and M (2) O and P (3) P and N (4) R and O (5) M and T 152. Who among the following sits third to the right of a person from Tamil Nadu? (1) R (2) T (3) Q (4) O (5) N Directions (153–157) : Study the following information carefully and answer the questions given below : ●

    A%B

    A+B

    A is sitting third to the right of B and A is facing away from the centre. A is to the immediate right of B and A is facing the centre.

    BPRE–430

    A#B

    A!B

    A$B A@B

    A&B A*B

    A^B

    A?B

    A is second to the right of B and A is facing away from the centre. A is sitting third to the left of B and A is facing the centre. A is second to the left of B and A is facing the centre. A is to the immediate left of B and A is facing away from the centre. A is opposite to B and A is facing the centre. A is sitting third to the right of B and A is facing the centre. A is sitting third to the left of B and A is facing away from the centre. A is to the immediate left of B and A is facing the centre.

    There are 12 friends sitting around a square table. There are four friends sitting at the four corners of the table. In each side of the table two persons are sitting as immediate neighbours. Six of them are facing the centre and six of them are facing away from the centre. M$N#O*P^Q+R@S*T# U ? V * W, and X is an immediate neighbour of M. 153. Who sits second to the left of X? (1) P (2) R (3) M (4) Q (5) U 154. Who sits opposite to N? (1) R (2) T (3) W (4) S (5) O 155. Who sits to the immediate right of W? (1) N (2) V (3) P (4) R (5) O 156. Who sits exactly between V and S? (1) U (2) T (3) R (4) X (5) P 157. Who sits fourth to the right of V? (1) Q (2) N (3) X (4) U (5) W

    RANKING/ARRANGEMENT Directions (158 – 162) : Study the following information carefully and answer the questions given below : There are eight class mates A, B, C, D, E, F, G and H. All are sitting around a circular table facing the centre but not necessarily in the same order. The immediate neighbours of a mate are friends of the mate while the mate sitting opposite to a mate is his enemy. All the class mates are aged either 15 or 16 or 17 years. There are three mates who are 15 years old. There are two mates who are 17 years old. There are three mates who are 16 years old. The mates in the age order 15, 16 and 17 sit one by one in the same order anticlockwise. G and A are friends but C and D are enemies. H and B are friends but F and A are enemies. C and E are friends. F and H are friends. B and D are friends but G and H are enemies. D is sitting second to the left of G. E and G are friends. D and A are friends but E and B are enemies. B is 15 years old. 158. Who is the enemy of a mate whose age is 16 years? (1) A

    (2) D

    (3) E

    (4) G

    (5) F 159. Who is the friend of D’s enemy? (1) G

    (2) E

    (3) B

    (4) H

    (5) A 160. Who is the enemy of A’s friend? (1) F

    (2) D

    (3) C

    (4) G

    (5) E 161. Who is sitting second to the right of a mate whose age is 17 year? (1) E

    (2) F

    (3) C

    (4) B

    (5) D 162. Who sits third to the left of F? (1) D

    (2) E

    (3) A

    (4) B

    (5) G Directions (163–167) : Study the following information carefully and answer the questions given below:

    There are 16 students sitting in two rows. Each row has eight seats. All the students are facing the teacher. C sits second to the left of a student who sits third to the right of H. E sits to the immediate right of a student who sits immediate right below diagonally of P. H sits immediate left below diagonally of a student who sits second to the right of B. D sits immediate left above diagonally of a student who sits second to the left of H. P sits second to the right of a student who sits fourth to the left of J. L sits second to the left of a student who sits immediate left above diagonally of M. F sits third to the right of a student who sits immediate left above diagonally of N. E sits sixth to the right of G’s immediate right neighbour. J sits fifth to the right of I’s immediate right neighbour. A is in the 2nd row who is facing north direction. K sits exactly between G and H’s immediate neighbour. N sits third to the left of a student who sits immediate left below diagonally of O. 163. Who among the following sits exactly between K and H? (1) B (2) N (3) P (4) A (5) C 164. Who among the following sits third to the right of B? (1) D (2) J (3) H (4) M (5) P 165. Which of the following pairs sits at the extreme ends of the rows? (1) M and L (2) J and H (3) D and E (4) G and B (5) J and C 166. Who among the following are the immediate neighbours of C? (1) P and M (2) H and L (3) L and N (4) A and H (5) A and F

    BPRE–431

    167. Who among the following sits immediate right above diagonally of N? (1) L (2) D (3) H (4) I (5) P Directions (168-172) : Study the following information and answer the questions that follow : Twelve persons are sitting in two parallel lines in such a way that there are six persons in each row at equidistance. A, B, C, D, E and F are sitting in row-1 and they face towards east. P, Q, R, S, T and V are sitting in row-2 and they face towards west. One person of one row faces the other person of the other row. P, who is sitting at one of the ends of the row, is second to the right of T. A does not face P or T. A is third to the left of F. Three are two persons between Q and V. There is only one person between C and D. C and D do not face P. B is neighbour of C. S, who does not face D, is not the neighbour of Q. 168. Who among the following faces F? (1) Q (2) T (3) S (4) R (5) V 169. How many persons are sitting between E and C? (1) One (2) Two (3) Three (4) Four (5) None 170. E is related to R in the same way as B is related to S. On this pattern F is related to (1) V (2) P (3) T (4) Q (5) None of these 171. Which of the following statements is true about B? (1) Q is the neighbour of that person who faces B (2) C is not the immediate neighbour of B (3) E is second to the right of B. (4) T faces B (5) B is second from the left end of the row 172. Who are sitting at the ends of the row? (1) P, V (2) F, A (3) D, E (4) P, S (5) None of these

    RANKING/ARRANGEMENT

    SHORT ANSWERS 1. (3)

    2. (2)

    3. (4)

    EXPLANATIONS 4. (3)

    5. (1)

    6. (2)

    7. (3)

    8. (1)

    9. (2)

    10. (1)

    11. (3)

    12. (2)

    13. (1)

    14. (3)

    15. (4)

    16. (3)

    17. (4)

    18. (3)

    19. (4)

    20. (2)

    21. (1)

    22. (3)

    23. (3)

    24. (4)

    25. (4)

    26. (1)

    27. (1)

    28. (4)

    29. (4)

    30. (4)

    31. (2)

    32. (3)

    33. (4)

    34. (4)

    35. (3)

    36. (2)

    37. (1)

    38. (1)

    39. (1)

    40. (4)

    41. (3)

    42. (3)

    43. (4)

    44. (3)

    45. (4)

    46. (1)

    47. (2)

    48. (1)

    49. (4)

    50. (1)

    51. (4)

    52. (3)

    53. (4)

    54. (4)

    55. (4)

    56. (4)

    57. (3)

    58. (3)

    59. (2)

    60. (3)

    61. (2)

    62. (4)

    63. (3)

    64. (2)

    65. (2)

    66. (3)

    67. (4)

    68. (1)

    69. (2)

    70. (5)

    71. (3)

    72. (1)

    73. (4)

    74. (1)

    75. (2)

    76. (2)

    77. (3)

    78. (4)

    79. (5)

    80. (5)

    81. (2)

    82. (3)

    83. (4)

    84. (1)

    85. (4)

    86. (2)

    87. (4)

    88. (2)

    89. (5)

    90. (2)

    91. (1)

    92. (5)

    93. (2)

    94. (2)

    95. (5)

    97. (3)

    98. (2)

    99. (5) 100. (5)

    96. (1)

    101. (1) 102. (2) 103. (5) 104. (4) 105. (1) 106. (3) 107. (5) 108. (2) 109. (3) 110. (4) 111. (1) 112. (2) 113. (3) 114. (5) 115. (3) 116. (1) 117. (5) 118. (3) 119. (2) 120. (4) 121. (1) 122. (2) 123. (2) 124. (3) 125. (3) 126. (3) 127. (5) 128. (1) 129. (1) 130. (5) 131. (2) 132. (5) 133. (2) 134. (4) 135. (5) 136. (3)

    (1– 4) : D>E>C>A>F>B Front

    Back

    1. (3) E is between D and C. 2. (2) D is the tallest. 3. (4) C is the fourth counting from the shortest. 4. (3) B is the shortest in the row. 5. (1) Ring D is the heaviest, hence will fetch maximum price. 6. (2) G and F. 7. (3) Ring F is the lightest. 8. (1) The correct order is given by D E H G and F (9) : The order in which they stand according to their height is given below. Tallest D

    E

    A

    C

    F B Shortest

    9. (2) On the basis of information, we conclude that D is the heaviest, however we cannot say anything about the child who is lightest. (10) : On the basis of information given in the question we conclude following D = 2E ...(i) E = 4.5 F ...(ii) F=

    G 2

    145. (4) 146. (3) 147. (2) 148. (5) 149. (4) 150. (3) 151. (2) 152. (1) 153. (2) 154. (4) 155. (5) 156. (1) 157. (2) 158. (4) 159. (2) 160. (3) 161. (1) 162. (5) 163. (2) 164. (5) 165. (3) 166. (4) 167. (1) 168. (5) 169. (2) 170. (4) 171. (4) 172. (3)

    C B

    H ...(iv) 2 Suppose H = 4, then G = 2, F = 1 E = 4.5 & D = 9. Now, we see that auto matically, condition (v) is complied with. Hence, order of their weight in decreasing order is as below

    E

    G=

    D

    D

    ...(iii)

    E

    H

    G

    F

    137. (1) 138. (3) 139. (1) 140. (2) 141. (4) 142. (5) 143. (3) 144. (5)

    12. (2) Total number of people in the row = (9 + 11 ) – 1 = 19 13. (1) SQT is a group of female. 14. (3) T works in personnel department. 15. (4) NT work on the IInd floor. 16. (3) Q must be transferred to personnel to maintain the original distribution of females on each floor. 17. (4) Department of P is not known, as per information, only Q works in Administration. (18-20) : On the basis of the information give in the question we get the order of all the nine persons as below NRMLJOPQK 18. (3) J is sitting in the middle of the row. 19. (4) N is at the other end of the row. 20. (2) There is one person between L and Q. 21.(1) The sitting order of the six persons is given below. Thus, F is opposite to B.

    10. (1) If all the five persons are arranged in descending order of their weight, the following order is obtained. BCAED 12345 Hence, C is on the second place. 11. (3) In the second half of the row, there are 18 vehicles. Thus, number of scooters are 15.

    BPRE–432

    F A

    22. (3) On the basis of information given in the question, the colours of cap of all the seven persons is given as under A Green or Blue B Green or Blue C Yellow D Violet E Indigo Red F Red 23. (3) Kailash After

    5th

    6th

    Interchange Kashish

    Mona 13th

    Hence, therefore 8 students between them.

    RANKING/ARRANGEMENT (24-26) : On the basis of information given in the questions following table can be made. Height Rupa Kamini Neelam Pooja Monika

    Age Neelam Rupa = Monika Kamini Pooja

    24. (4) Neelam will be at the third position. 25. (4) Since, positions of Kamini and Pooja keeps verifying. Hence, either Kamini or Pooja will be in the third position. 26. (1) Only (i) 27. (1) Firisting on the immediate right of D. 28. (4) Engineer and drictor are sitting together. 29. (4) The engineer is not more intelligent than businessman. 30. (4) According to information option (4) is true. 31. (2) CDF represents the correct order of intelligence. 32. (3) Mohan’s position = 9th from the top. Kiran’s position = (35 –7 + 1 =) 29th from the top Sohan’s position =

    FG 9 + 29 =IJ H 2 K

    19th from the top Hence, Kiran’s position is 10th from Sohan’s position. (33-34) : Steps of four people are as following Q S R P

    33. (4) If P is on first step then Q is on sixth step. 34. (4) If P is on first step then step four is empty. (35-38) : Let C weighs x kg. B weighs 4.5 x kg. A weighs 9 x kg. D weighs 2 × kg. E weighs 4. x kg. 35. (3) C is the lightest in weight 36. (2) E is heavier than D and C. 37. (1) A is the heaviest in weight.

    38. (1) The descending order is A>B>E>D>E 39. (1) According to given information, Suma > Kavita > Sudha > Mamta > Puja So, Puja scored the lowest marks. 40. (4) According to Samant, his brother’s birthday can be on 16th or 17th February. According to the sister of Samant, their brother’s birthday can be on 17th or 18th February. So, from both the statements, Samant’s brother’s birthday is on 17th February. 41. (3) Sunita’s rank is 8th from top. Sujit’s rank is 13th from top. Sujit’s rank from bottom. = 40 – 13 + 1 = 28th 42. (3) 2C 2G 2M 1C 1S = 8 2C 2G 2M 1C 1S = 8 2C 2G 2M 1C 1S = 8 2C 2G 2M = 6 30 43. (4) Prabir is twelfth from right end and eighteenth from left. Hence, total number of students. = (18 + 12) – 1 = 30 – 1 = 29 ∴ Number of students to the right of Mohan = 29 – 13 = 16 (44-47) : According to given information, their seating plan is as following Amit Suresh Vishnu Ujjwal Rakesh

    Alka

    Jyoti

    50. (1) According to the given information, they can be arranged in the following manner. Ravi > Vineet > Manick > Jacob > Dilip So, Ravi is tallest in their group. 51. (4) Red

    Yellow Green Blue Z

    X

    So, X lives in blue house. 52. (3) According to figure U is sitting third to left of S. T W

    S

    U

    Q

    V

    P R

    53. (4) According to question, Row 1 = B M J Row 2 = K D C 54. (4) The sequence is A 3 C 5 E 7 G 9 I 11 K 13 M 15 O 17 Q 19 S 21 U 23 W 25 Y 27 10th letter from right = Q So, 3rd digit to right = 21 55. (4) Number of persons between Sadiq and Joseph = 48 – (14 + 17) = 17 As, Jane is in between Sadiq and Joseph. So, there are 8 persons between Sadiq and Jane. 56. (4) Arranging the data given in statements. So the position of A in relation to F is third to the right. F

    Veena Prabha Nayan

    44. (3) Veena and Vishnu occupies the middle position. 45. (4) Suresh is facing Jyoti. 46. (1) Rakesh and Alka are sitting diagonally opposite. 47. (2) Prabha is immediate left of Nayan. 48. (1) Mr Bhagat Mr Ashokan Mr Gaurav Mr Lokesh Mr Rakesh So, Mr Bhagat lives in the top most flat. 49. (4) Raju, Sohan, Kittu, Mohan, Shyam. So, Shyam is on the extreme right.

    BPRE–433

    E

    C

    D

    B A

    57. (3) Mahesh > Vikas 1 yr Vikas > Jagan 2 yr Suresh > Jagan 1 yr Mahesh > Suresh 2 yr Jagan > Akmal 2 yr Combining all five equations, Mahesh > Vikas > Suresh > Jagan > Akmal

    RANKING/ARRANGEMENT 58. (3) D B A E C So, house A is in the middle. 59. (2) B D A F C G E So, F is sitting in the middle. 60. (3) Sheela Rani Rita Meena Second from the left is Rani. 61. (2) According to figure T is second to the left of R. U

    V

    Q

    S P R

    If Q and R interchange their places and T and V inter change their places.

    68. (1) H is standing third to the right of F. (69–73) :

    M

    O

    Q

    R

    S

    T

    N

    P

    69. (2) If two more persons are seated to the immediate left of Q, then three persons will sit between R and O. 70. (5) R faces south. M sits third to the right of R. 71. (3) Four persons — M, Q, T and P – face north. 72. (1) Q is the immediate neighbour of O. In all others, there are two persons between the given pair. 73. (4) Only T sits between S and N. (74–79) :

    80. (5) V sits seventh to the right of Q. 81. (2) Q and S are seated at two extreme ends of the line. 82. (3) Three persons – T, X and U – are seated between R and P. 83. (4) T sits fifth to the left of V. Q sits fifth to the left of P. X sits fifth to the left of S. 84. (1) Seventh from the left ⇒ W Fifth from the right ⇒ U P sits exactly between U and W. 85. (4)

    H

    R

    +4

    !

    +5

    +3

    S

    +4

    T

    +5

    +3

    2

    T

    +4

    B

    +5

    B K

    +6

    I

    +6

    ©

    +6

    M

    4

    86. (2) According to question, the new sequence would be :

    $

    U

    +3

    2 & ! 6 @ © # % 4 5 9 3 7

    T R

    S P

    9th from the right end

    P

    R

    V

    87. (4)

    Q

    +4

    T is second to the left of R. 62. (4) The data given in the question is not sufficient. 63. (3) Red Blue Orange Green White So, blue shirt is between red and orange shirts. 64. (2)

    According to question, Deputy General Manager is third from the left end. 65. (2) N S M L R So, M is occupying the middle position in the row. 66. (3) Geeta > Gurmeet > Jeet Nene > Mona > Gurmeet Mona > Geeta Therefore, Nene is the tallest. (67–68) :

    L E F T

    R I G H T

    67. (4) C and E are standing at the extreme ends of the line.

    +4

    +

    +

    –2 4 4 →   → M   →% G  

    Q

    U

    +4

    –2

    +4

    +2

    +2

    +2

    –2

    +4

    →@  → F T  → B   → D   → L   → 7 5  +4

    → 5   → 4  → D %  88. (2)

    S

    W

    Vowel

    V

    General Asst. Deputy Deputy Senior Manager General General Manager Manager Manager Manager

    –2

    → J   → H  → S A 

    74. (1) R sits second to the left of W. 75. (2) U faces outside. R and W are immediate neighbours of U. U sits second to the left of T. 76. (2) When counted from the right of R, two persons U and W sit between R and V. 77. (3) Except U, all others face the centre. 78. (4) P and R are immediate neighbours of T. 79. (5) S is sitting to the immediate left of V. (80–84) : L E F T

    Q R

    T X U P W V S

    BPRE–434

    R I G H T

    Number Consonant

    There is only one such combination :

    E4N

    (89–93): K M

    Z

    T

    Q

    N

    E

    D 89. (5) Both the Conclusion I and Conclusion II are true. 90. (2) N is sitting to the immediate left of T.

    RANKING/ARRANGEMENT 91. (1) D is sitting second to the left of T. 92. (5) Both the Conclusion I and Conclusion II are true. 93. (2) M is sitting second to the left of Z. (94–98) : Calm Loyal ble ns i J H Se O h September October c r Ma

    Empathetic N November

    Caring D July

    Honest I December

    Polite G June



    B



    L

    G





    K

    N

    Dy n am E ic Ap r il

    Brave F August

    Independent K February

    101. (1)

    104.

    T Conclusion I is true. L is second to the left of G. 102. (2)

    T N

    105.

    H

    Traditional Optimist M L January May

    106. M

    94. (2) I is an honest person. J sits third to the right of I. 95. (5) O is a sensible person and D is a caring person. F sits exactly between O and D. 96. (1) G is second to the right of D, who is a caring person. 97. (3) F is a brave person who was born in August. D was born in July. D sits to the immediate left of K, an independent person. 98. (2) F was born in August. L was born in May. Traditional trait is associated with May. 99. (5) U

    N

    L

    O U

    H is to the immediate right of M. Conclusion II is true. 103. (5)

    B G

    107.

    M

    108.

    N H 109.

    F Both the Conclusions are true. (104–108) :

    Meghna is the daughter of Mohan and Sudha. Rohit is the son of Sundar and Suman. Sudha is the daughter–in–law of Raj and Lata. Sundar is the son–in–law of Raj and Lata. (4) Rohit sits to the immediate left of Suman. Rohit is the son of Suman and Sundar. Suman is the sister of Mohan. Therefore, Rohit is the nephew of Mohan. (1) Sudha is the daughter–in–law of Raj and Lata. Meghna sits third to the left of Sudha. (3) Meghna is the granddaughter of Raj and Lata. Mohan is sitting between Sundar and Meghna. Mohan is sitting third to the right of Rohit. Rohit is the grandson of Raj and Lata. (5) Sudha is sitting just opposite to Suman. (2) Raj is the grandfather of Meghna and Rohit. Sudha is sitting to the immediate left of Raj. Suman is sitting just opposite to Sudha. (3)

    C L

    B

    Lata Rohit

    S

    Raj

    G Q

    Suman

    K

    Sudha

    Both the Conclusions are true. 100. (5)

    A

    M

    Mohan

    G

    B



    Q

    Sundar

    Meghna

    H

    ● ●

    I G is third to the left and third to the right of Q.

    Couples Male Female Raj Lata Mohan Sudha Sundar Suman ● Mohan is the son of Raj and Lata. ● Suman is the daughter of Raj and Lata.

    BPRE–435

    (110–114) :

    E O N

    G

    J H D M L

    F

    K

    I

    110. (4) G is living diagonally and immediately right above D’s flat. 111. (1) M lives third to the right of J in the same row. 112. (2) J would be fifth.

    RANKING/ARRANGEMENT 113. (3)

    O is 7th from the right end. 114. (5) H is an immediate neighbour of D and lives in Column 2. (115–117) :

    LEFT

    B

    A

    L

    G

    J

    F

    Production

    Sales

    R

    U

    Finance

    123. (2) H is sitting second to the right of R.

    I

    H is sitting to the immediate right of J.

    D

    V

    132. (5)

    H

    Y

    124. (3) H is the CEO and J is the MD.

    B J

    HR

    H

    N

    RIGHT

    K

    131. (2)

    T

    Advertisement

    R

    B

    Marketing

    E O N G L F K I

    C

    P

    125. (3) B sits between T and F.

    C RIGHT LEFT 115. (3) L is sitting opposite A. 116. (1) D sits third to the left of A. 117. (5) I sits second to the left of J. (118–122) : Finance

    126. (3) B sits third to right of J. 127. (5) H sits second to left of D. P

    L

    P

    Q

    2

    D

    H

    N

    R

    T

    2 2

    2

    MD

    Red

    P Green

    R

    B

    M

    129. (1)

    Y

    Black

    M

    Violet K

    L

    P D

    M

    C

    B 2

    2

    Q Yellow

    130. (5)

    2

    2

    P L

    D

    K

    2

    M R

    Z

    B

    H CEO 2

    N

    K

    R

    HR Technical 118. (3) O is the head of Finance Department. 119. (2) P is the subordinate of the both HR and Technical head. N is sitting third to the left of P. 120. (4) L and Q are subordinates to the Finance Department. 121. (1) K is the Technical head. Q is third to the left of K. 122. (2) M is third to the right of R. (123–127) :

    F

    K O

    N

    Q M

    (133–137) :

    128. (1)

    Marketing

    O

    E

    V

    B

    J P

    L

    Q

    J

    BPRE–436

    I

    R is the father of N and O. R is the brother of M. N is the son of R. O is the daughter of R. 133. (2) After the rearrangement, K is second to the right of R. 134. (4) R is sitting to the immediate right of yellow seat. M is brother of R. 135. (5) L is sitting 20 metre to the left of K before the final rearrangement. 136. (3) Red colour seat is located third to the M’s seat after rearrangement. 137. (1) Two seats are vacant between the seats of N and L.

    RANKING/ARRANGEMENT (148–152) : (138–142) :

    8

    G Uttarakhand N

    7 M 6

    R

    5

    Z

    4

    E

    3

    D

    2

    A

    1

    E

    S Kerala M Tamil Nadu

    P Maharashtra

    Q West Bengal

    139. (1) Z is exactly between G and A. 140. (2) 7

    2

    5

    4

    M

    A

    Z

    E

    141. (4) 8

    2

    7

    1

    G

    A

    M

    E

    142. (5) 6

    4

    2

    3

    E

    Madhya O Pradesh

    Andhra Pradesh T

    138. (3) A is at the 2nd position.

    R

    Punjab R

    A

    D

    Teacher

    148. (5) M is from Tamil Nadu. N sits opposite to M. 149. (4) O sits third to the left of T. O is from Madhya Pradesh. 150. (3) T is from Andhra Pradesh. R is from Punjab. N sits exactly between T and R. 151. (2) O is from Madhya Pradesh. P is from Maharashtra. 152. (1) M is from Tamil Nadu. R sits third to the right of M. (153–157) :

    V

    T

    U

    158. (4) E, D and H are 16 years old. G is enemy of H. 159. (2) C is enemy of D. E and F are friends of C. 160. (3) G and D are friends of A. C is enemy of D. 161. (1) A and C are 17 years old. E sits second to the right of A. 162. (5) G sits third to the left of F. (163–167) :

    S

    O

    R

    W

    Q

    L E F T

    M

    R

    T

    O Q

    143. (3) N faces away from the centre.

    17 years C

    16 years E

    15 years G

    P sits to the immediate left of N. 144. (5) S and T are immediate neighbours of M. 145. (4) R sits opposite to M. 146. (3) Q sits third to the right of N. 147. (2) S sits third to the right of Q.

    A 17 Years

    F 15 years H 16 years

    D 16 years B 15 years

    BPRE–437

    L

    F

    P

    O

    J

    G

    K

    N

    H

    C

    A

    M

    E

    R I G H T

    Row-1

    E

    P

    A

    Q

    B

    T

    C

    S

    F

    V

    D RIGHT

    R

    RIGHT

    Row-2

    N

    S

    B

    LEFT

    N

    X M P 153. (2) X faces away from the centre. R sits second to the left of X. 154. (4) S sits opposite N. 155. (5) O sits to the immediate right of W. 156. (1) U sits exactly between V and S. 157. (2) N sits fourth to the right of V. (158 – 162) :

    I

    163. (2) N sits exactly between K and H. 164. (5) P sits third to the right of B. 165. (3) D, G, J and E sit at the extreme ends of the rows. 166. (4) H and A are immediate neighbours of C. 167. (1) L sits immediate right above diagonally of N. (168–172) :

    (143-147) :

    P

    D

    LEFT

    168. (5) V faces F. 169. (2) Two persons - A and B. 170. (4) E and R at the left ends of the rows. B and S are third from the left end. F is second from the right end. In Row-2, Q is second from the right end. 171. (4) T faces B. 172. (3) E and D are sitting at the ends of the row-1. ❑❑❑

    INPUT

    10

    INPUT

    QUESTIONS FROM 1999 TO 2010 ARE AVAILABLE ONLINE NATIONALISED BANKS & IBPS PO/MT/SO EXAMS Directions (1–7) : Study the following information carefully and answer the questions which follow : (Indian Bank PO Exam. 02.01.2011 (Ist sitting)

    A word and number arrangement machine when given an input line of words and numbers rearranges them following a particular rule. Following is an illustration of input and the rearrangement. (Single digit numbers are preceded by a zero. All other numbers are two digit numbers.) Input : when 19 will you 07 be 40 coming 62 home 100 89 Step I : be when19 will you 07 40 coming 62 home 100 89 Step II : be 07 when 19 will you 40 coming 62 home 100 89 Step III be 07 coming when 19 will you 40 62 home 100 89 Step IV : be 07 coming 19 when will you 40 62 home 100 89 Step V : be 07 coming 19 home when will you 40 62 100 89 Step VI : be 07 coming 19 home 40 when will you 62 100 89 Step VII : be 07 coming 19 home 40 when 62 will you 100 89 Step VIII : be 07 coming 19 home 40 when 62 will 89 you 100 Step VIII is the last step of the arrangement of the above input as the intended arrangement is obtained. As per the rules followed in the above steps. Find out in each of the following questions the appropriate steps for the given input. Input for the questions (1 – 4) : Input : next 57 problem 82 14 trend 02 purchase growth 41 1. How many steps would be needed to complete the arrangement ? (1) V (2) VI (3) VIII (4) VII (5) Cannot be determined 2. Which of the following would be the final arrangement ? (1) growth 02 next 14 problem 41 purchase 57 82 trend. (2) growth next problem purchase trend 02 14 41 57 82.

    (3) growth 02 next 14 problem 41 purchase 57 trend 82. (4) growth 82 next 57 purchase 41 problem 14 next 02. (5) None of these) 3. Which of the following would be step I ? (1) 41 purchase 02 trend 14 82 problem 57 next growth. (2) growth 02 next 57 problem 82 14 trend purchase 41. (3) 41 growth next 57 problem 82 14 trend 02 purchase. (4) growth next 57 problem 82 14 trend 02 pur chase 41. (5) growth next 57 purchase 82 14 trend 02 problem 41. 4. Which word/number would be at the sixth position from the left end in Step III ? (1) Purchase (2) 14 (3) problem (4) 41 (5) trend (5 – 7) : Input : just 14 and value 22 time 05 15 5. Which word/number would be at position 5 from the right end in Step III ? (1) and (2) 15 (3) just (4) 14 (5) time 6. Which of the following would be step II of the arrangement ? (1) and just value 14 22 time 05 15. (2) and 5 just 14 value 22 time 15. (3) and 05 just 14 time 15 22 value. (4) and just value time 14 22 05 15. (5) None of these 7. How many steps would be needed to complete the arrangement and which would be the last step ? (1) Step IV : and 05 just 14 time 15 value 22. (2) Step V : and 05 just 14 time 15 value 22. (3) Step V : 22 and 15 just 14 time 05 just. (4) Step VI: and 05 just 14 time 15 value 22. (5) None of these Directions (8–12) : Study following information to answer the given questions : (Corporation Bank PO Exam. 16.01.2011)

    A word and number arrangement machine when given an input line of words rearranges them following a particular rule. The following is an illustration of input and rearrangement.

    BPRE–438

    INPUT Input Step I Step II Step III Step IV

    : : : : :

    age road own wire tire ink pen uni dice eat uni age road own wire tire ink pen eat dice uni own age road wire tire ink eat pen dice uni own ink age wire tire eat road pen dice uni own ink eat age wire tire road pen dice And step four is the last step of the rearrangement. As per the rules followed in the above steps, find out in each of the following questions the appropriate steps for the given input. Input for the questions Input : gem stat ace cast omit fan rate uncut era input 8. Which of the following would be the final arrangement ? (1) cast gem fan rate stat uncut omit input era ace (2) uncut omit input era ace cast fan gem rate stat (3) uncut omit input era ace stat rate gem fan cast (4) uncut omit input era ace stat fan gem rate cast (5) None of these 9. In Step III, which of the following word would be at 6th position from the left ? (1) rate (2) ace (3) stat (4) gem (5) None of these 10. Which step number would be the following output ? uncut omit gem stat ace rate era input fan cast (1) II (2) III (3) V (4) IV (5) None of these 11. In step IV of the rearrangement, if omit is related to era and rate is related to fan in a certain way, to which of the following would ace be related to, following the same pattern ? (1) rate (2) input (3) stat (4) gem (5) None of these 12. Which of the following would be Step VII ? (1) Uncut omit input era ace stat rate gem fan cast (2) Uncut omit input era ace rate stat fan gem cast (3) Uncut omit input era ace cast fan gem rate stat (4) Uncut omit input era stat ace rate gem fan cast (5) There will be no such step as the input gets rearranged before step VII Directions (13 – 20) : Study the following information to answer the given questions : (UCO Bank PO Exam. 30.01.2011)

    A word and number arrangement machine when given an input line of word and numbers rearranges them following a particular rule. The following is an illustration of input and rearrangement. (All numbers in these questions are two digit numbers) Input : lived 18 a 12 once 93 upon 32 time 46 wolf Step I % a lived 18 12 once 93 upon 32 46 time wolf Step II % a once 18 12 93 upon 32 46 lived time wolf Step III% a once upon 18 12 93 32 46 lived time wolf Step IV% a once upon 93 18 12 32 46 lived time wolf Step V% a once upon 93 46 18 12 32 lived time wolf Step VI% a once upon 93 46 32 18 12 lived time wolf Step VI is the last Step of the arrangement of the above input as the intended arrangement is obtained.

    Directions (13 – 16) : Now, answer the questions based on the following input : Input: unique 84 can 77 open 86 quick 13 base 53 amiss 11 equal 98 start 13. Which of the following would be Step II for the above Input ? (1) amiss equal unique 84 77 open 86 quick 13 base 53 11 98 can start (2) amiss equal open unique 84 77 86 quick 13 base 53 11 98 can start (3) amiss equal open unique 84 77 86 13 53 11 98 base can quick start (4) amiss unique 84 77 open 86 13 base 53 11 equal 98 can quick start (5) None of these 14. Which of these words/numbers would be fifth (from right side) in Step III for the input? (1) 53 (2) 11 (3) equal (4) 98 (5) None of these 15. Which of the following would be the last step for the input? (1) amiss equal open unique 98 86 84 77 53 13 11 base can quick start (2) amiss equal open unique 98 84 86 77 53 13 11 base can quick start (3) amiss equal open unique 98 86 84 77 53 13 11 can base quick start (4) amiss equal open unique 98 86 84 77 53 11 13 base can quick start (5) None of these 16. How many Steps would be needed to complete the arrangement for the above input? (1) VII (2) III (3) V (4) IV (5) None of these 17. The following stands for which step of the rearrangement? amiss equal open unique 98 84 77 86 13 53 11 base can quick start (1) Step III (2) Step V (3) Step VI (4) Step IV (5) None of these Directions (18-20) : Given below are five steps in a jumbled order in the form of (A), (B), (C), (D) and (E) for an input. Arrange them according to the order in which they should appear based on the example given. Then answer the questions that follow. (1) arrival on 16 44 28 66 finish match (2) arrival on 66 44 28 16 finish match (3) arrival 16 44 28 on 66 finish match (4) arrival on 66 44 16 28 finish match (5) arrival on 66 16 44 28 finish match 18. Which of the following will be Step II ? (1) A (2) B (3) C (4) D (5) E 19. Which of the following will be Step III ? (1) A (2) B (3) C (4) D (5) E 20. Which of the following will be Step I ? (1) A (2) B (3) C (4) D (5) E

    BPRE–439

    INPUT Directions (21-25) : Study the following information to answer the given questions — (Allahabad Bank PO Exam. 17.04.2011) A word and number arrangement machine when given an input line of words and numbers rearranges them following a particular rule. The following is an illustration of input and rearrangement. Input : sum 28 have 19 96 48 luck nice 78 rope Step I : have sum 28 19 48 luck nice 78 rope 96 Step II : luck have sum 28 19 48 nice rope 96 78 Step III : nice luck have sum 28 19 rope 96 78 48 Step IV : rope nice luck have sum 19 96 78 48 28 Step V : sum rope nice luck have 96 78 48 28 19 And Step V is the last step of the rearrangement. As per the rules followed in the above steps, find out in each of the following questions the appropriate steps for the input given below : Input : 49 last zen 16 82 yet can vast 33 aim 87 54 21. How many steps would be needed to complete the arrangement ? (1) IV (2) V (3) VI (4) VII (5) None of these 22. Which Step number would be the following output ? vast last can aim zen 16 yet 33 87 82 54 49 (1) III (2) II (3) VII (4) IV (5) There will be no such step 23. Which of the following would be the Step I ? (1) aim 49 can zen 16 yet vast 33 54 87 82 (2) vast last can aim zen 16 yet 33 87 82 54 49 (3) zen 49 last 16 82 yet can vast 33 aim 54 87 (4) aim 49 last zen 82 yet can vast 33 87 54 16 (5) None of these 24. In Step V which of the following word/number would be on the 6th position (from the right) ? (1) 87 (2) 16 (3) 33 (4) zen (5) aim 25. Which of the following would be the final arrangement ? (1) zen yet vast last can aim 16 33 49 54 82 87 (2) aim can last vast yet zen 16 33 49 54 82 87 (3) aim can last vast yet zen 87 82 54 49 33 16 (4) zen yet vast last can aim 87 82 54 49 33 16 (5) None of these Directions (26 – 30) : Study the following information to answer the given questions : (IBPS Bank PO/MT CWE Exam. 18.09.2011)

    A word and number arrangement machine when given an input line of words and numbers rearranges them following a particular rule. The following is an illustration of input and rearrangement. Input : 52 peak 91 snow freeze 46 cold 15 high 31 73 trek Step I : 15 52 peak snow freeze 46 cold high 31 73 trek 91 Step II : 15 31 52 peak snow freeze 46 cold high trek 73 91 Step III : 15 31 46 peak snow freeze cold high trek 52 73 91

    Step IV : 15 31 46 cold peak snow freeze high trek 52 73 91 Step V : 15 31 46 cold freeze peak snow high trek 52 73 91 Step VI : 15 31 46 cold freeze high peak snow trek 52 73 91 Step VI is the last step of the rearrangement. As per the rules followed in the above steps, find out in each of the following questions the appropriate steps for the given input. Input for the questions. Input : 67 hot sun 19 best 83 ice 49 ace 77 cut 37 26. How many steps would be needed to complete the arrangement ? (1) X (2) VIII (3) IX (4) VII (5) None of these 27. Which Step number would be the following output? 19 37 49 ace best hot sun ice cut 67 77 83 (1) II (2) VI (3) V (4) IV (5) None of these 28. Which of the following would be the Step I ? (1) 19 37 49 hot sun best ice ace cut 67 77 83 (2) 83 67 hot sun best ice 49 ace 77 cut 37 19 (3) 19 67 ace best hot sun ice 49 77 cut 37 83 (4) 19 67 hot sun best ice 49 ace 77 cut 37 83 (5) None of these 29. Which of the following would be the final arrangement? (1) 67 77 83 ace best cut hot ice sun 19 37 49 (2) 19 37 49 ace best cut hot ice sun 67 77 83 (3) 19 37 49 67 77 83 ace best cut hot ice sun (4) 19 37 49 ace ice best cut hot sun 67 77 83 (5) None of these 30. In Step IV, which of the following word/number would be on 7th position (from the right) ? (1) sun (2) best (3) 67 (4) cut (5) None of these Directions (31–35) : Study the following information carefully and answer the given questions : (IBPS Specialist Officer CWE Exam. 11.03.2012)

    A word and number arrangement machine when given an input line of words and numbers rearranges them following a particular rule in each step. The following is an illustration of input and rearrangement. (All the numbers given in the arrangement are two digit numbers). Input : gone over 35 69 test 72 park 27 Step I : 27 gone over 35 69 test 72 park Step II : 27 test gone over 35 69 72 park Step III : 27 test 35 gone over 69 72 park Step IV : 27 test 35 park gone over 69 72 Step V : 27 test 35 park 69 gone over 72 Step VI : 27 test 35 park 69 over gone 72 Step VII : 27 test 35 park 69 over 72 gone and Step VII is the last step of the rearrangement of the above input as the desired arrangement is obtained. As per the rules followed in the above steps, find out in each of the following questions the appropriate step for the given input.

    BPRE–440

    INPUT 31. Input : 86 open shut door 31 49 always 45 How many steps will be required to complete the rearrangement? (1) Five (2) Six (3) Seven (4) Four (5) None of these 32. Step III of an input: 25 yes 37 enemy joy defeat 52 46 Which of the following is definitely the input? (1) enemy 25 joy defeat yes 52 37 46 (2) 37 enemy 25 joy yes defeat 52 46 (3) enemy joy defeat 25 52 yes 46 37 (4) Cannot be determined (5) None of these 33. Step II of an Input : 18 win 71 34 now if victory 61 How many more steps will be required to complete the rearrangement? (1) Three (2) Four (3) Five (4) Six (5) More than six 34. Input : where 47 59 12 are they going 39 Which of the following steps will be the last but one? (1) VII (2) IV (3) V (4) VIII (5) None of these 35. Step II of an input: 33 store 81 75 full of goods 52 Which of the following will be step VI ? (1) 33 store 52 of 75 81 full goods (2) 33 store 52 of 75 full 81 goods (3) 33 store 52 of 75 goods 81 full (4) There will be no such step (5) None of these Directions (36–39) : Study the following information carefully and answer the given questions : (IBPS Bank PO/MT CWE Exam. 17.06.2012)

    A word and number arrangement machine when given an input line of words and numbers rearranges them following a particular rule in each step. The following is an illustration of input and rearrangement. (All the numbers are two digit numbers). Input : tall 48 13 rise alt 99 76 32 wise jar high 28 56 barn Step I : 13 tall 48 rise 99 76 32 wise jar high 28 56 barn alt Step II : 28 13 tall 48 rise 99 76 32 wise jar high 56 alt barn Step III : 32 28 13 tall 48 rise 99 76 wise jar 56 alt barn high Step IV : 48 32 28 13 tall rise 99 76 wise 56 alt barn high jar Step V : 56 48 32 28 13 tall 99 76 wise alt barn high jar rise Step VI : 76 56 48 32 28 13 99 wise alt barn high jar rise tall Step VII: 99 76 56 48 32 28 13 alt barn high jar rise tall wise and Step VII is the last step of the above input, as the desired arrangement is obtained. As per the rules followed in the above steps, find out in each of the following questions the appropriate step for the given input.

    Input : 84 why sit 14 32 not best ink feet 51 27 vain 68 92 (All the numbers are two digits numbers). 36. Which step number is the following output? 32 27 14 84 why sit not 51 vain 92 68 feet best ink (1) Step V (2) Step VI (3) Step IV (4) Step III (5) There is no such step 37. Which word/number would be at 5th position from the right in Step V ? (1) 14 (2) 92 (3) feet (4) best (5) why 38. How many elements (words or numbers) are there between ‘feet’ and ‘32’ as they appear in the last step of the output? (1) One (2) Three (3) Four (4) Five (5) Seven 39. Which of the following represents the position of ‘why’ in the fourth step ? (1) Eighth from the left (2) Fifth from the right (3) Sixth from the left (4) Fifth from the left (5) Seventh from the left Directions (40 – 42) : Study the following information to answer the given questions : (IBPS Specialist Officer CWE Exam.17.03.2013)

    A word and number arrangement machine when given an input line of words and numbers rearranges them following a particular rule. The following is an illustration of input and rearrangement. (All numbers in these questions are two digit numbers) Input : 16 today 32 waiting 21 are 11 people 46 bus 66 long Step I : 16 today 32 waiting 21 11 people 46 bus 66 long are Step II : 16 today 32 waiting 21 people 46 bus 66 long 11 are Step III : 16 today 32 waiting 21 people 46 66 long bus 11 are Step IV : today 32 waiting 21 people 46 66 long 16 bus 11 are Step V : today 32 waiting people 46 66 21 long 16 bus 11 are Step VI : today 32 waiting 46 66 people 21 long 16 bus 11 are Step VII : today waiting 46 66 32 people 21 long 16 bus 11 are Step VIII : waiting 46 66 today 32 people 21 long 16 bus 11 are Step IX : waiting 66 46 today 32 people 21 long 16 bus 11 are Step X : 66 waiting 46 today 32 people 21 long 16 bus 11 are Step X is the last Step of the arrangement of the above input as the intended arrangement is obtained. Now, answer the questions based on the following input : Input : 23 you 13 wake 81 me 43 before 72 go 34 up

    BPRE–441

    INPUT 40. Which of these words /numbers would be fourth (from left side) in Step IV for the input ? (1) me (2) 43 (3) 81 (4) wake (5) None of these 41. The following stands for which step of the rearrangement ? you wake 81 43 72 34 up me 23 go 13 before (1) Step IX (2) Step IV (3) Step VI (4) Step V (5) None of these 42. Which of the following would be Step II for the above input ? (1) 23 you wake 81 me 43 72 34 up go 13 before (2) 23 you 13 wake 81 me 43 72 go 34 up before (3) 23 you wake 81 me 43 72 go 34 up before 13 (4) 23 you wake 81 me 43 72 go 34 up 13 before (5) None of these Directions (43–47) : Study the following information carefully and answer the questions given below : (Indian Overseas Bank PO Online Exam, 01.09.2013)

    A word and number arrangement machine when given an input line of words and numbers rearranges them following a particular rule in each step. The following is an illustation of input and various steps of rearrangement. (All the numbers are two digit numbers). Input : more presence required 12 42 70 for 63 37 good work 28 Step I % for 70 more presence required 12 42 63 37 good work 28 Step II % good 63 for 70 more presence required 12 42 37 work 28 Step III % more 42 good 63 for 70 presence required 12 37 work 28 Step IV % presence 37 more 42 good 63 for 70 required 12 work 28 Step V % required 28 presence 37 more 42 good 63 for 70 12 work Step VI % work 12 required 28 presence 37 more 42 good 63 for 70 And Step VI is the last step of the rearrangement as the desired arrangement is obtained. As per rules followed in the above steps, find out in each of the questions the appropriate step for the given input. Inpur for the questions : money 48 24 18 wanted for investment 65 90 lock credit 32 43. Which step number is the following output ? lock 32 investment 48 for 65 credit 90 money 24 18 wanted (1) Step II (2) Step III (3) Step IV (4) Step V (5) There is no such step 44. Which word/number would be fourth from the left in step IV ? (1) for (2) 48 (3) 65 (4) 32 (5) credit 45. Which word/number would be second to the right of the fifth element from the left in step V ? (1) for (2) 65 (3) 48 (4) investment (5) 32

    46. How many elements (words/numbers) are there between “for” and “18” in step III ? (1) Four (2) Three (3) Six (4) Seven (5) Five 47. Which of the following represents the position of “credit” in step V? (1) Third from the right (2) Fifth from the left (3) Sixth from the left (4) Fourth from the right (5) Sixth from the right Directions (48-52) : Study the following information carefully and answer the questions given below : (IBPS Bank PO/MT CWE-III, 26.10.2013)

    A word and number arrangement machine when given an input line of words and numbers rearranges them following a particular rule in each step. The following is an illustation of input and various steps rearrangement. (All the numbers are two digit numbers). Input : 11 day 34 night 93 pace 27 easy 44 joy Step I : 93 11 day 34 night pace 27 easy 44 joy Step II : 93 11 34 night pace 27 easy 44 joy day Step III : 93 44 11 34 night pace 27 easy joy day Step IV : 93 44 11 34 night pace 27 joy day easy Step V : 93 44 34 11 night pace 27 joy day easy Step VI : 93 44 34 11 night pace 27 day easy joy Step VII : 93 44 34 27 11 night pace day easy joy Step VIII : 93 44 34 27 11 pace day easy joy night Step IX : 93 44 34 27 11 day easy joy night pace And Step IX is the last step of the rearrangement as the desired arrangement is obtained. As per rules followed in the above steps, find out in each of the questions the appropriate step for the given input. Input for the questions : class 25 war 15 race 73 heap 58 just 88 take 38 48. What is the position of ‘war’ in the Step ‘VII’ ? (1) Seventh from the left end (2) Eighth from the right end (3) Fifth from the left end (4) Fifth from the right end (5) Sixth from the left end 49. Which of the following is the ninth from the right in Step VI ? (1) race (2) 25 (3) war (4) 58 (5) 15 50. What is the position of ‘15’ in the Step ‘IX’ ? (1) Seventh from the left end (2) Eighth from the left end (3) Fifth from the right end (4) Seventh from the right end (5) Eighth from the right end 51. How many Steps are required to complete this arrangement? (1) Eleven (2) Twelve (3) Ten (4) Nine (5) None of these 52. Which of the following represents the Step X ? (1) 88 73 58 38 25 war 15 race take class heap just (2) 88 73 58 38 25 15 class heap just race take war (3) 88 73 58 38 25 15 war class heap just race take (4) 88 73 58 38 25 15 war take class heap just race (5) There is no such Step

    BPRE–442

    INPUT Directions (53–58) : Study the following information carefully and answer the questions given below :

    Directions (59-62) : Study the following information carefully and answer the questions given below :

    (IDBI Bank Officer Exam, 22.08.2014)

    (SIDBI Officer Exam, 03.09.2014)

    When a word and number arrangement machine is given an input line of words and numbers, it arranges them following a particular rule. The following is an illustration of input and rearrangement : (All the numbers are two digit numbers). Input : nice 54 vide run 11 why 97 68 aim 37 gun 81 Step I : why nice 54 vide run 97 68 aim 37 gun 81 11 Step II : vide why nice 54 run 97 68 aim gun 81 11 37 Step III : run vide why nice 97 68 aim gun 81 11 37 54 Step IV : nice run vide why 97 aim gun 81 11 37 54 68 Step V : gun nice run vide why 97 aim 11 37 54 68 81 Step VI : aim gun nice run vide why 11 37 54 68 81 97 Step VI is the last step of the above arrangement as the intended arrangement is obtained. As per the rules followed in the above steps, find out in each of the following questions the appropriate steps for the given input. Input : 63 45 fog 13 88 can ink 76 51 you hang 27 air end 53. How many steps are required to complete this arrangement? (1) VI (2) VII (3) V (4) Cannot be determined (5) None of these 54. How many elements (words/numbers) are there between ‘ink’ and ‘45’ as they appear in the Step V? (1) Eight (2) Five (3) Six (4) Four (5) Seven 55. Which step number would be the following output ? fog hang ink you 63 88 can 76 air end 13 27 45 51 (1) V (2) VI (3) IV (4) III (5) None of these 56. Which of the following would be third to the right of fifth from the left end in Step III ? (1) 76 (2) 51 (3) can (4) 88 (5) air 57. What will be the position of ‘can’ in Step II ? (1) Eighth from left (2) Seventh from right (3) Sixth from right (4) Seventh from left (5) Ninth from right 58. Which of the following would be Step V ? (1) hang fog end you ink 76 can 88 air 13 27 45 51 63 (2) end fog 88 can 76 hang ink you 13 27 air 45 51 63 (3) end fog hang ink you 88 can 76 air 13 27 45 51 63 (4) end fog 51 63 hang ink 88 you 76 can air 13 27 45 (5) There is no such step

    A word and number arrangement machine when given an input line of words and numbers rearranges them following a particular rule in each step. The following is an illustration of input and various steps of rearrangement. (All the numbers are two digit numbers). Input % sour burden made new 13 25 38 point 67 98 50 weak to find 44 83 Step I : burden sour made new 25 38 point 67 50 to find 44 83 weak 98 Step II : 13 25 burden find sour made new 38 point 67 50 44 to weak 83 98 Step III : 13 25 38 burden find made new point 50 44 sour to weak 67 83 98 Step IV : 13 25 38 44 burden find made new point sour to weak 50 67 83 98 And Step IV is the last step of the rearrangement as the desired arrangement is obtained. As per rules followed in the above steps, find out in each of the questions the appropriate step for the given input. Input : water is required 12 41 65 for all 27 33 89 to 75 90 survive life 59. Which word/number would be fourth to the left of the seventh element from the right in the Step III ? (1) required (2) is (3) 41 (4) 65 (5) for 60. How many elements (words/numbers) are there between ‘all’ and ‘65’ as they appear in Step IV ? (1) Six (2) Eight (3) Five (4) Seven (5) Four 61. At which of the following position ‘required’ would appear from the right in the Step II ? (1) Eleventh (2) Tenth (3) Ninth (4) Eighth (5) Seventh 62. Which of the following represents the position of ‘for’ in the Step I ? (1) Sixth from left (2) Ninth from right (3) Seventh from left (4) Eleventh from right (5) Eighth from left Directions (63–68) : Study the following information carefully and answer the questions given below : (IBPS Bank PO/MT CWE-IV, 18.10.2014)

    When a word and number arrangement machine is given an input line of words and numbers, it arranges them following a particular rule. The following is an illustration of input and rearrangement : (All the numbers are two digit numbers). Input : bike 51 ride 37 11 duke 58 damp line 75 tent 84 Step I : 84 51 ride 37 11 duke 58 damp line 75 tent bike Step II : 75 84 51 ride 37 11 duke 58 line tent bike damp Step III : 58 75 84 51 ride 37 11 line tent bike damp duke Step IV : 51 58 75 84 ride 37 11 tent bike damp duke line Step V : 37 51 58 75 84 11 tent bike damp duke line ride Step VI : 11 37 51 58 75 84 bike damp duke line ride tent Step VI is the last step of the above arrangement as the intended arrangement is obtained.

    BPRE–443

    INPUT As per the rules followed in the above steps, find out in each of the following questions the appropriate steps for the given input. Input : find 64 belt 28 54 lamp 17 give flat 69 real 95 63. Which of the following would be the Step III ? (1) 54 64 69 95 28 lamp 17 real belt find flat give (2) 69 95 64 28 58 lamp 17 give flat real belt find (3) 64 69 95 28 54 lamp 17 give real belt find flat (4) 54 64 69 95 28 give lamp 17 real flat find belt (5) None of these 64. What will be the position of the ‘lamp’ in the Step IV ? (1) Sixth from the left end (2) Seventh from the left end (3) Fifth from the right end (4) Sixth from the left end (5) Fifth from the left end 65. How many elements (words/numbers) are there between “28” and “69” as they appear in the Step I ? (1) Six (2) Seven (3) Four (4) Five (5) Three 66. At which of the following position “real” would appear from the right in the Step V ? (1) Seventh (2) Sixth (3) Fifth (4) Fourth (5) None of these 67. Which word/number would be the second to the left of the fifth element from the right in the Step III ? (1) 54 (2) 17 (3) give (4) 28 (5) lamp 68. Which Step number would be the following output ? 28 54 64 69 95 17 real belt find flat give lamp (1) Step V (2) Step IV (3) Step III (4) Step II (5) Step VI Directions (69-72) : Study the following information carefully and answer the questions given below : (IBPS Bank PO/MT CWE–V Main Exam. 31.10.2015)

    When a word and number arrangement machine is given an input line of words and numbers, it rearranges them following a particular rule in each step. The following is an illustration of input and rearrangement. (All the numbers are two digit numbers) Input % talk 47 12 rise at 99 75 32 wise joke high 28 56 be Step I % 12 talk 47 rise 99 75 32 wise joke high 28 56 be at Step II % 28 12 talk 47 rise 99 75 32 wise joke high 56 at be Step III % 32 28 12 talk 47 rise 99 75 wise joke 56 at be high Step IV % 47 32 28 12 talk rise 99 75 wise 56 at be high joke Step V % 56 47 32 28 12 talk 99 75 wise at be high joke rise Step VI % 75 56 47 32 28 12 99 wise at be high joke rise talk Step VII % 99 75 56 47 32 28 12 at be high joke rise talk wise

    Step VII is the last step of the above rearrangement as the desired arrangement is obtained. As per the rules followed in the given steps, find out the appropriate steps for the given input. Input % 83 why sat 14 32 no be ink feet 50 27 vain 67 92 69. Which step number is the following output? 32 27 14 83 why sat no 50 vain 67 92 be feet ink (1) Step V (2) Step VI (3) Step IV (4) Step I (5) Other than those given as options 70. Which word/number would be at fifth position from the right in Step V? (1) 14 (2) 92 (3) feet (4) be (5) sat 71. How many elements (words or numbers) are there between ‘feet’ and ‘32’ as they appear in the last step of the output? (1) One (2) Three (3) Four (4) Five (5) Two 72. Which of the following represents the position of ‘why’ in the fourth step? (1) Eighth from the left (2) Fifth from the right (3) Sixth from the left (4) Fifth from the left (5) Sixth from the right Directions (73–76) : Study the following information carefully and answer the questions given below : (IBPS Specialist Officer (Marketing) CWE 01.02.2016)

    When a word and number arragement machine is given an input line of words and numbers, it arranges them following a particular rule. The following is an illustration of input and rearrangement. (All the numbers are two digit numbers) Input % 23 kinetic amount 64 nature 71 58 opium verdict 96 elderly 15 Step I : opium 23 kinetic amount 64 nature 71 verdict 96 elderly 15 58 Step II : elderly opium 23 kinetic amount nature 71 verdict 96 15 58 64 Step III : amount elderly opium 23 kinetic nature 71 verdict 15 58 84 96 Step IV : 15 amount elderly opium 23 kinetic nature 71 58 64 96 verdict Step V : 23 15 amount elderly opium kinetic 71 58 64 96 verdict nature Step VI : 71 23 15 amount elderly opium 58 84 96 verdict nature kinetic Step VI is the last step of the above arrangement as the intended arrangement is obtained. As per the rules followed in the given steps, find out the appropriate steps for the given input. Input % adverb 59 36 salient 81 idioms bakery 14 launch 47 umpire 62 73. Which elements come exactly between ‘59’ and ‘bakery’ in Step II of the given input? (1) Only ‘launch’ (2) Only ‘62’ (3) Only ‘idioms’ (4) Both ‘81’ and ‘salient’ (5) Both ‘adverb’ and ‘36’

    BPRE–444

    INPUT 74. Which of the following combinations represents the first two and the last elements of the Step VI of the given input? (1) 81, 59, launch, bakery (2) 81, 62, salient, launch (3) 50, 47, bakery, salient (4) 62, 14, idioms, umpire (5) 62, 81, umpire, launch 75. If in the Step III, ‘idioms’ interchanges its position with ‘81’ and ‘salient’ also interchanges its position with ‘14’ then which element will be third to the left of ‘47’? (1) 14 (2) salient (3) idioms (4) 81 (5) adverb 76. In which step are the elements ‘bakery launch 47 14’ found in the same order? (1) Sixth (2) Third (3) Fourth (4) The given order of elements is not found in any step (5) Fifth Directions (77–81) : Study the following information carefuly and answer the questions given below : (IBPS Specialist Officer (IT) CWE 14.02.2016)

    When a word and number arrangement machine is given an input line of words and numbers, it rearranges them following a particular rule. The following is an illustration of input and rearrangement : (All the numbers are two-digit numbers.) Input % baffled 46 iceberg 19 nation unreal 75 28 kidney 63 abandon 94 Step I : abandon baffled 46 iceberg 19 unreal 75 28 kidney 63 94 nation Step II : iceberg abandon baffled 49 19 unreal 75 28 63 94 nation kidney Step III : unreal iceberg abandon 46 19 75 28 63 94 nation kidney baffled Step IV : 75 unreal iceberg abandon 46 19 63 94 nation kidney baffled 28 Step V : 63 75 unreal iceberg abandon 19 94 nation kidney baffled 28 46 Step VI : 19 63 75 unreal iceberg abandon nation kidney baffled 28 46 94 Step VI is the last step of the above arrangement as the intended arrangement is obtained. As per the rules followed in the given steps, find out the appropriate steps for the given input. Input : offense 71 vaccine 12 39 garage absorb 86 earlier 25 malign 64 77. If in the step II, ‘absorb’ interchanges its position with ‘86’ and ‘39’ also interchanges its position with ‘vaccine’ then which element will be seventh to the left of ‘25’ ? (1) 39 (2) absorb (3) 86 (4) 64 (5) vaccine 78. Which of the following combinations represents the first two and the last two elements in the step IV of the given input ? (1) earlier, 86, vaccine, 71 (2) 39, absorb, earlier, 12

    (3) offense, 25, 12, absorb (4) 25, malign, garage, 86 (5) 71, offense, garage, 12 79. In which step are the elements ‘absorb 86 25 vaccine’ found in the same order ? (1) Third (2) Fifth (3) Fourth (4) Sixth (5) The given order of elements is not found in any step 80. Which elements come exactly between ‘25’ and ‘malign’ in step III of the given input ? (1) Both ‘64’ and ‘vaccine’ (2) Only ‘39’ (3) Both ‘86’ and ‘earlier’ (4) Only ‘garage’ (5) Only ‘offense’ 81. Which element is third to the left of the one which is seventh from the left end in Step VI of the given input ? (1) earlier (2) 71 (3) malign (4) offense (5) 25 Directions (82–87) : Study the following information carefully and answer the questions given below : (United Bank of India PGDBF Manipal Exam,07.08.2016)

    When a word and number arrangement machine is given an input line of words and numbers, it arranges them following a particular rule. The following is an illustration of input and rearrangement. (All the numbers are two-digit numbers.) Input : 23 module ore 84 39 bore specie 52 76 pulse Step I : 85 borate 23 module ore 39 specie 52 76 pulse Step II : 77 modulate 85 borate 23 ore 39 specie 52 pulse Step III : 53 orate 77 modulate 85 borate 23 39 specie pulse Step IV : 40 pulsate 53 orate 77 modulate 85 borate 23 specie Step V : 24 speciate 40 pulsate 53 orate 77 modulate 85 borate Step V is the last step of the above arrangement as the intended arrangement is obtained. As per the rules followed in the given steps, find out the appropriate steps for the given input. Input for questions 41 indice aspire 73 96 probe marine 28 59 chrome 82. In which step of the given arrangement are the elements ‘60 indicate 74 chromate 97’ found in the same order? (1) Fifth (2) Fourth (3) Third (4) Second (5) First 83. What is the position of ‘indice’ from the right of ‘chromate’ in the Step II? (1) Second (2) Third (3) Immediate right (4) Fourth (5) Fifth 84. Which element(s) is/are present exactly between ‘74’ and ‘aspirate’ in the Step III of the given arrangement? (1) Both ‘chromate’ and ‘97’ (2) Only ‘97’

    BPRE–445

    INPUT (3) Only ‘chromate’ (4) Both ‘97’ and ‘probe’ (5) Only ‘probe’ 85. Which element is the third to the right of eighth element from the right end in the Step II? (1) aspirate (2) indice (3) 41 (4) probe (5) marine 86. If in the Step III ‘indicate’ interchanges position with ‘probe’ and ‘74’ interchanges position with ‘marine’, which of the following elements would be to the immediate right of ‘41’? (1) marine (2) 97 (3) 28 (4) aspirate (5) indicate 87. If in the Step IV three is added to each of the numbers, how many numbers so formed would be exactly divisible by ‘3’? (1) None (2) One (3) Two (4) Three (5) More than three Directions (88–92) : Study the following information carefully and answer the questions given below : (Bank of Baroda Exam, 25.09.2016)

    When a word and number arrangement machine is given an input line of words and numbers, it arranges them following a particular rule. The following is an illustration of input and arrangement: (All the numbers are two digit numbers) Input : 48 height marker 13 92 kneel school 29 65 barrier Step I : barriers 48 height marker 92 kneel school 29 65 12 Step II : 28 barriers 48 marker 92 kneel school 65 12 heights Step III : kneels 28 barriers marker 92 school 65 12 heights 47 Step IV : kneels 28 barriers marker 92 school 12 heights 47 markers Step V : schools 64 kneels 28 barriers 12 heights 47 markers 91 Step V is the last step of the above arrangement as the intended output of arrangement is obtained. As per the rules followed in the given steps, find the appropriate steps for the given input Input : 84 layer 97 packet 51 damage narrow 75 32 table 88. Which of the following represents the sum of the fourth element from the left end in Step V and the second element from the right end in Step II? (1) 105 (2) 125 (2) 114 (4) 147 (5) 81 89. ‘damages’ is related to ‘97’ in Step III in the same way as ‘narrow’ is related to ‘table’ in Step I. Following the same pattern to which element is ‘31’ related to in Step IV? (1) 50 (2) 74 (3) packets (4) damages (5) 97 90. Which of the following represents the element that is fifth to the left of ‘layers’ in the last Step ? (1) narrows (2) 50 (3) packets (4) 83 (5) table

    91. In step IV, how many elements are there between ‘50’ and the fourth element from the right end? (1) More than three (2) None (3) Three (4) Two (5) One 92. Which of the following is the third to the left of the ninth element from the left end of Step II? (1) narrow (2) 97 (3) packet (4) damages (5) 75 Directions (93–97) : Study the following information carefully and answer the questions given below : (Bank of Maharashtra PO Exam, 26.10.2016)

    When a word and number arrangement machine is given an input line of words and numbers, it arranges them following a particular rule. The following is an illustration of input and arrangement. (All the numbers are two digit numbers). Input : wait 47 19 rose door 23 aim less 85 year 99 68 Step I : year 19 wait 47 rose door 23 aim less 85 99 68 Step II : year 19 47 rose door aim less 85 99 68 23 wait Step III : rose 47 year 19 door aim less 85 99 68 23 wait Step IV : rose 47 year 19 door aim 85 99 23 wait 68 less Step V : door 85 rose 47 year 19 aim 99 23 wait 68 less Step VI : door 85 rose 47 year 19 23 wait 68 less 99 aim Step VI is the last step of the above arrangement as the intended output of the arrangement is obtained. As per the rules followed in the given steps, find the appropriate steps for the given input. Input : band 85 zen 54 den fit 25 37 home 41 sun 73 93. How many elements are there between ‘band’ and ‘73’ in Step IV ? (1) Five (2) One (3) Two (4) None (5) Four 94. Which is the third element to the left of the ninth element from the left in the third step ? (1) zen (2) band (3) 54 (4) 85 (5) fit 95. In which of the following steps ‘41 73 37’ found consecutively in the same order ? (1) Step II (2) Step IV (3) Step I (4) Step III (5) Step V 96. In Step V, ‘home’ is related to ‘25’ following a certain pattern. Following the same pattern, ‘37’ is related to ‘fit’ in Step VI. In Step I, with which of hte folliowing is ‘band’ related to following the same pattern ? (1) fit (2) den (3) 81 (4) sun (5) home 97. Which of the following combinations represents the first two and the last two elements in the Step V of the given input ? (1) den, 41; 54, fit (2) home, 73; 85 band (3) den, 25; 37, sun (4) den, 85; 54, home (5) den, 73; 54, fit Directions (98–102) : Read the given information carefully and answer the questions given below : (IBPS Bank PO/MT CWE (Main) Exam, 18.11.2016

    A word and number arrangement machine, when given an input line of words and numbers arranges them

    BPRE–446

    INPUT following a particular rule in each step. The following is an illustration of input and rearrangement. (All numbers are two digit numbers) Input : tell 17 weep 25 age 67 still 53 hand 58 vein 90 Step I : 89 weep tell 17 25 age 67 still 53 hand 58 vein Step II : 68 vein 89 weep tell 17 25 age still 53 hand 58 Step III : 57 tell 68 vein 89 weep 17 25 age still 53 hand Step IV : 54 still 57 tell 68 vein 89 weep 17 25 age hand Step V : 26 hand 54 still 57 tell 68 vein 89 weep 17 age Step VI : 18 age 26 hand 54 still 57 tell 68 vein 89 weep and Step VI is the last step for the above rearrangement as the desired arrangement is reached. As per the rules followed in the above steps, find out in each of the following questions, the appropriate step for the given Input. (All numbers are two digits.) Input : store 96 clean 57 take 16 break 29 feet 36 wait 70 ice 72 98. Which Step is the given arrangement? 58 ice 69 store 71 take 95 wait break clean 16 29 feet 36 (1) Step IV (2) Step II (3) Step VI (4) Step VII (5) No such step 99. Which element is fifth to the left of the third from the left in step V ? (1) 95 (2) take (3) 71 (4) feet (5) ice 100. What will be the square root of the product of the seventh element from the right and eighth element from the left? (1) 256 (2) 1024 (3) 65536 (4) 16 (5) None of these 101. What will be the position of ‘wait’ in step III ? (1) fourth from the feft (2) tenth from right (3) fourth from the right (4) tenth from left (5) sixth from the left 102. Which element will be exactly in the middle of ‘store’ and ‘29’ in step II ? (1) 57 (2) take (3) break (4) 16 (5) None of these Directions (103–107) : Study the following information carefully and to answer the questions given below : (Indian Bank PO (Pre.) Exam, 21.01.2017 (Ist Sitting))

    When a word and number arrangement machine is given an input line of words and numbers, it arranges them following a particular rule. The following is an illustration of input and rearrangement. ( All the numbers are two-digit numbers) Input : turn 12 84 mist hike 45 vast 26 site gate 72 56 Step I : gate turn 12 mist hike 45 vast 26 site 72 56 84 Step II : 72 gate turn 12 mist 45 vast 26 site 56 84 hike Step III : mist 72 gate turn 12 45 vast 26 site 84 hike 56 Step IV : 45 mist 72 gate turn 12 vast 26 84 hike 56 site

    Step V :

    turn 45 mist 72 gate 12 vast 84 hike 56 site 26 Step VI : 12 turn 45 mist 72 gate 84 hike 56 site 26 vast Step VI is the last step of the above arrangement as the intended arrangement is obtained. As per the rules followed in the given steps, find the appropriate steps for the given input. Input: 15 role air 96 63 born with 77 like 39 some 52 103. How many elements are there between ‘with’ and ‘born’ in step IV? (1) None (2) One (3) Two (4) Three (5) Five 104. In which of the following steps is ’15 some 52’ found consecutively in the same order? (1) Step IV (2) Step VI (3) Step II (4) Step I (5) Step III 105. Which of the following is second element to the left of the ninth element from the left in the second step? (1) like (2) 39 (3) with (4) role (5) air 106. In step I, ‘some’ is related to ‘77’ following a certain pattern. Following the same pattern, in step V, ‘role’ is related to ‘96’. In step IV, to which of the following is ‘born’ related to? (1) with (2) 63 (3) like (4) role (5) 39 107. In Step II, which element(s) appear(s) exactly between ‘77’ and ‘15’? (1) Only ‘role’ (2) Both ‘born’ and ‘52’ (3) Only ‘air’ (4) Only ‘some’ (5) Both ‘like’ and ‘39’ Directions (108–111) : Study the following information carefully and answer the questions given below : (Indian Bank PO (Pre.) Exam, 21.01.2017 (2nd Sitting))

    When a word and number arrangement machine is given an input line of words and numbers, it arranges them following a particular rule. The following is an illustration of input and rearrangement: (All the numbers are two digit numbers) Input : 48 pabular measles 13 92 tokener riddles 69 25 baldric Step I : toner 14 48 pabular measles 92 riddles 69 25 baldric Step II : roles 26 toner 14 48 pabular measles 92 69 baldric Step III : polar 49 roles 26 toner 14 measles 92 69 baldric Step IV : moles 70 polar 49 roles 26 toner 14 92 baldric Step V : boric 93 moles 70 polar 49 roles 26 toner 14 Step V is the last step for the above arrangement as the intended output of arrangement is obtained. As per the rules followed in the given steps, find out the appropriate steps for the given output. Input : 84 springs 97 damping 51 telomes paddles 75 32 ballots

    BPRE–447

    INPUT 108. In Step IV, how many elements are there between ‘poles’ and the fourth element from the right end ? (1) Three (2) Two (3) One (4) None (5) More than three 109. Which of the following represents the element that is fourth to the left of ‘poles’ in the last Step ? (1) 98 (2) Boots (3) 76 (4) Tomes (5) There are less than four elements to the left of ‘poles’ 110. Which of the following represents the sum of the fourth element from the left end in Step V and the fifth element from the right end in Step IV ? (1) 128 (2) 85 (3) 183 (4) 118 (5) 137 111. ‘damping’ is related to ‘97’ in Step III in the same way as ‘75’ is related to ‘paddles’ in Step I. Following the same pattern to which element is ‘85’ related to in Step IV ? (1) 76 (2) ballots (3) poles (4) doing (5) 52 Directions (112– 116) : Study the following information carefully and answer the questions given below : (IBPS SO (Agriculture) Exam, 29.01.2017)

    When a word and number arrangement machine is given an input line of words and numbers, it arranges them following. a particular rule. The following is an illustration of input and rearrangement : (All the numbers are two digit numbers). Input : bear 24 binders brave 91 17 but 68 bailer 35 be 74 Step I : 19 bear 24 binders brave 17 but 68 bailer 35 74 be Step II : 47 19 bear 24 binders brave 17 68 bailer 35 be but Step III : 86 47 19 24 binders brave 17 bailer 35 be but bear Step IV : 53 86 47 19 24 binders 17 bailer be but bear brave Step V : 42 53 86 47 19 binders 17 be but bear brave bailer Step VI : 71 42 53 86 47 19 be but bear brave bailer binders Step VI is the last step of the above arrangement as the intended arrangement is obtained. As per the rules followed in the given steps, find out the appropriate steps for the given input. Input: tyre 71 toughen 59 tip 82 13 thanks to 68 table 46 112. If in the last step, ‘1’ is added to each of the odd numbers and ‘2’ is subtracted from each of the even numbers, then how many numbers multiple of ‘4’ will be formed? (1) Two (2) Three (3) More than three (4) None (5) One

    113. If in step III, ‘28’ interchanges its position with ‘table’ and ‘59’ also interchanges its position with ‘tip’ then which of element will be to the immediate left of ‘46’? (1) ‘28’ (2) ‘to’ (3) ‘table’ (4) ‘tip’ (5) ‘59’ 114. Which of the following combinations represents the third and the ninth element in second last step of the given input from the left end? (1) ‘95’ and ‘tip’ (2) ‘table’ and ‘86’ (3) ‘17’ and ‘to’ (4) ‘86’ and ‘tip’ (5) ‘tyre’ and ‘17’ 115. Which element comes exactly between ‘59’ and ‘thanks’ in Step II of the given input? (1) toughen (2) 46 (3) 13 (4) table (5) 68 116. Which element is third to the right of the one which is eighth from the right end in Step IV? (1) 13 (2) tip (3) thanks (4) to (5) 46 Directions (117–121) : When a number arrangement machine is given an input line of numbers, it arranges them following a particular rule. (IBPS Bank PO/MT CWE-VII (Prelim Exam) 14.10.2017)

    The following is an illustration of input and rearrangement : (All the numbers are two digit numbers) Input : 81 63 79 42 15 24 86 37 96 19 Step I : 15 19 81 63 79 42 24 86 37 96 Step II : 96 86 15 19 81 63 79 42 24 37 Step III : 24 37 96 86 15 19 81 63 79 42 Step IV : 81 79 24 37 96 86 15 19 63 42 Step V : 42 63 81 79 24 37 96 86 15 19 Step V is the last step of the above arrangement as the intended output of arrangement is obtained. As per the rules followed in the given steps, find the appropriate steps for the given input. Input : 26 69 13 82 55 21 71 34 93 47 117. Which is the fourth element to the left of the seventh element from the left end in Step III of the given arrangement? (1) 82 (2) 93 (3) 13 (4) 26 (5) 55 118. What will be the resultant if the fifth element from right end in Step V is subtracted from the second element from left end in Step II as per the given arrangement? (1) 59 (2) 79 (3) 56 (4) 48 (5) 67 119. In which of the following steps ‘26 69 55’ found consecutively in the same order as per the given arrangement?

    BPRE–448

    INPUT (1) Only II (2) Both I and II (3) Both II and III (4) There is no such step (5) Only III 120. As per the given arrangement, in Step I ‘21’ is related to ‘69’ as per a certain pattern. Following the same pattern, ‘26’ is related to ‘93’ in Step IV. To which of the following is ‘47’ related to following the same pattern in Step V? (1) 34 (2) 21 (3) 71 (4) 69 (5) 55 121. How many elements appear to the right of ‘26’ in Step IV of the given arrangement? (1) Six (2) None (3) Two (4) Five (5) Seven Directions (122–126) : Study the following information carefully and answer the questions given below : (IDBI Bank PO Exam 29.04.2018)

    A word and number arrangement machine when given an input line of words and numbers rearranges them following a particular rule in each step. The following is an illustration of input and rearrangement. Input : cup for hot 34 69 72 tea 27 Step I : 27 cup for hot 34 69 72 tea Step II : 27 tea cup for hot 34 69 72 Step III : 27 tea 34 cup for hot 69 72 Step IV : 27 tea 34 hot cup for 69 72 Step V : 27 tea 34 hot 69 cup for 72 Step VI : 27 tea 34 hot 69 for cup 72 Step VII : 27 tea 34 hot 69 for 72 cup And Step VII is the last step of the above arrangement. As per the rules followed in the above steps, find out in each of the following question, the appropriate step for the given input. 122. Input: kind 12 96 heart water 59 42 yes How many steps will be required to complete the rearrangement? (1) Three (2) Five (3) Four (4) Six (5) None of these 123. Input: jungle 43 mode 25 basket 39 target 19 Which of the following steps will be the last but one? (1) VII (2) VIII (3) IX (4) VI (5) None of these 124. Step III of an input is: 12 world 31 ask cart ball 87 75 Which of the following will definitely be the input? (1) 31 ask cart ball 87 75 world 12 (2) 31 ask cart ball 87 75 12 world (3) 31 ask 12 world cart ball 87 75 (4) Cannot be determined (5) None of these 125. Step II of an input is: 24 year 56 43 last part 64 over How many more steps will be required to complete the rearrangement? (1) Four (2) Six (3) Seven (4) Five (5) None of these

    126. Step III of an input: ‘32 station 46 81 73 march go for’ is: (1) 32 station 46 march 73 go for 81 (2) 32 station 46 march 73 81 go for (3) 32 station 46 march 73 go 81 for 4 There will be no such step. (5) None of these Directions (127-130) : Study the following information carefully and answer the questions given below : (IBPS Bank PO/MT CWE (Main Exam) 26.11.2017)

    When a word and number arrangement machine is given an input line of words and numbers, it rearranges them following a particular rule. The following is an illustration of input and rearrangement :

    Input

    base curl 30

    Step I

    qpoe

    41

    82 base curl

    12

    24 face pond

    30

    12

    24

    face

    Step II qpoe

    82 gbdf 60 base curl 12

    24

    Step III qpoe

    82 gbdf 60 dvsm 48 base

    12

    Step IV qpoe

    82 gbdf 60 dvsm 48 cbtf

    24

    Step IV is the last step of the above arrangement as the intended output of arrangement is obtained. As per the rules followed in the given steps, find the appropriate steps for the given input. Input : dear fake mane 62 34 55 40 hate 127. What will be the resultant if fifth number from the left end of Step I is added to fifth number from right end of Step III ? (1) 164 (2) 148 (3) 158 (4) 144 (5) 95 128. How many elements are there between ‘124’ and ‘dear’ in the Step III ? (1) Four (2) Three (3) One (4) Two (5) More than four 129. Which of the following is sixth from the right end in the Step IV ? (1) efbs (2) gblf (3) ibuf (4) nbof (5) dear 130. What is the difference between the second number from the left and second number from the right in the Step II ? (1) 44 (2) 56 (3) 69 (4) 84 (5) 90 Directions (131–135) : Study the following information carefully and answer the questions given below : (Canara Bank PO Exam, 23.12.2018)

    When a word and number arrangement machine is given an input line of words and numbers, it arranges them following a particular rule. The following is an illustration of input and rearrangement. (All the numbers are two digit numbers)

    BPRE–449

    INPUT Input

    : wait 47 19 rose door 23 aim less 85 year 99 68 Step I : year 19 wait 47 rose door 23 aim less 85 99 68 Step II : year 19 47 rose door aim less 85 99 68 23 wait Step III : rose 47 year 19 door aim less 85 99 68 23 wait Step IV : rose 47 year 19 door aim 85 99 23 wait 68 less Step V : door 85 rose 47 year 19 aim 99 23 wait 68 less Step VI : door 85 rose 47 year 19 23 wait 68 less 99 aim Step VI is the last step of the above arrangement as the intended output of the arrangement is obtained. As per the rules followed in the given steps, find the appropriate steps for the given input. Input : ban 85 zen 54 den fit 25 37 home 41 sun 73 131. What is the position of ‘ban’ in the Step III ? (1) Fourth from the left end (2) Sixth from the right end (3) Sixth from the left end (4) Ninth from the right end (5) Fifth from the left end 132. How many elements are there between ‘ban’ and ‘73’ in step IV ? (1) Five (2) One (3) Two (4) None (5) Four 133. Which is the third element to the left of the ninth element from the left in the third step ? (1) zen (2) ban (3) 85 (4) 54 (5) fit 134. In which of the following steps ‘54 den fit 73 37’ found consecutively in the same order ? (1) Step IV (2) Step I (3) Step II (4) Step III (5) Step V 135. In step V, ‘home’ is related to ‘25’ following a certain pattern and ‘37’ is related to ‘fit’. In step I, with which of the following is ‘ban’ related to following the same pattern ? (1) fit (2) den (3) 81 (4) sun (5) home Directions (136-140) : Study the following information carefully and answer the questions given below : (IBPS Specialist Officer CWE (Prelim Exam) 31.01.2019)

    A word and number arrangement machine when given an input line of words and numbers rearranges them following a particular rule in each step. The following is an illustration of an input and its rearrangement. Input : mobile 34 under trade 42 27 head 58 arrow 62 77 digital

    Step I

    : trade mobile 34 under 42 27 head 58 arrow 62 digital 77 Step II : trade mobile head 34 under 42 27 arrow 62 digital 77 58 Step III : trade mobile head digital 34 under 42 arrow 62 77 58 27 Step IV : trade mobile head digital arrow 34 under 42 77 58 27 62 Step V : trade mobile head digital arrow under 42 77 58 27 62 34 Step VI : trade mobile head digital arrow under 77 58 27 62 34 42 And Step VI is the last step of the above input. As per the rules followed in the given steps, find out the appropriate steps for the given input. Input : inspire 55 jump 86 view 22 74 angle fight 66 ride 14 136. How many steps will be required to complete the rearrangement? (1) V (2) VII (3) VI (4) IV (5) None of these 137. Which of the following would be at the seventh position from the left in step IV? (1) 22 (2) inspire (3) 14 (4) angle (5) None of these 138. Which step number would be the following output? View ride jump inspire 55 22 angle fight 14 86 66 74 (1) IV (2) III (3) V (4) II (5) None of these 139. In the last step, all the words get rearranged in alphabetical order, which of the following words will remain at its original position? (1) Jump (2) Fight (3) Inspire (4) Ride (5) No word 140. How many words/numbers are there between “74” and “ride” in step II? (1) Four (2) Five (3) Two (4) Three (5) Six Directions (141–145) : Study the following information carefully and answer the questions given below : (IBPS Bank PO/MT CWE Main Exam, 30.11.2019)

    A word and number arrangement machine when given an input line of words and numbers rearranges them following a particular rule in each step. The following is an illustration of input and rearrangement : Input

    Cave

    335

    Lake

    533

    Pond

    344

    434

    Canal

    Step-I Canal Cave

    Lake

    Pond

    533

    344

    353

    434

    335

    Step-II Canal

    344

    Cave

    533

    Lake

    434

    Pond

    353

    335

    353

    Step-III

    344

    434

    533

    353

    335

    Canal

    Cave

    Lake

    Pond

    Step-IV

    344

    Pond

    434

    Lake

    533

    Cave

    353

    Canal

    335

    Input for the questions Page

    BPRE–450

    233

    Scale

    223

    Water

    322

    266

    Model

    626

    INPUT 141. What is the difference between element at left end of step III and right end of step IV ? (1) 99 (2) 43 (3) 304 (4) 89 (5) 56 142. Which of the following is fourth to the right of element “233” in Step II ? (1) Page (2) 322 (3) Model (4) 223 (5) Water 143. Which of the following is fifth to the left of elements “scale” in step IV ? (1) 266 (2) 223 (3) 322 (4) Page (5) Model 144. Which of the following elements is placed exactly between “Page and 322” in step I? (1) Scale (2) 233 (3) Model (4) Water (5) 266 145. How many elements are placed exactly between “626 and Water” in step III ? (1) Two (2) Three (3) Four (4) Five (5) Six Directions (146–150) : A word and number arrangement machine when given an input line of words and numbers rearranges them following a particular rule in each step. The following is an illustration of input and the various steps of rearrangement: Input Step - I Step - II Step - III Step - IV

    Notes Royal Royal Royal Royal

    Bank Royal Notes Bank Notes Bank Notes Model 8753 Notes

    8573 8753 8537 8527 Model 8523 8753 8537 Model 8725 8573 8753 Model 8725 8537 Bank 8753 8725 8573 8537 8725 Model 8573 Bank 8537

    Step - V Step - VI

    8753 8753

    Royal 8725

    Notes 8537

    8725 8573

    8573 8527

    Model Royal

    8537 Notes

    8725 8527 8527 8527 8527

    Bank 8527 Model Bank

    INPUT English Clerk Scale 6712 6127 6271 6147 Office 6714

    (IBPS RRBs Officer Scale-I CWE Main Exam, 13.10.2019)

    146. Which element is fourth to the left of “Scale” in Step – VI? (1) Clerk (2) 6127 (3) 6712 (4) Office (5) 6147 147. Which element is fourth to the right of “6271” in Step – V? (1) 6714 (2) Scale (3) Clerk (4) 6127 (5) English 148. Which element is third to the right of “Office” in Step – IV? (1) 6271 (2) Scale (3) Clerk (4) 6127 (5) 6147 149. Which element is exactly between “6147” and “Clerk” in Step – III? (1) Office (2) Scale (3) English (4) 6714 (5) 6712

    150. Which element is at the same position in Step – I and Step – V? (1) 6714 (2) Office (3) 6271 (4) English (5) 6147 Directions (151–155) : Study the following information carefully and answer the questions given below : (Indian Bank Specialist Officer SO Exam, 08.03.2020)

    A word and number arrangement machine when given an input line of words and numbers rearranges them following a particular rule. The following is an illustration of input and rearrangement. (Note : All the given numbers are two digit numbers only) INPUT : emparts 34 75 recoups 48 outlies 67 21 zodiacs 86 lumbers circles Step I : zodiac emparts 34 75 recoups 48 outlies 67 86 lumbers circles 22 Step II : 33 zodiac emparts 75 48 outlies 67 86 lumbers circles 22 recoup Step III : outlie 33 zodiac emparts 75 67 86 lumbers circles 22 recoup 47 Step IV: 68 outlie 33 zodiac emparts 75 86 circles 22 recoup 47 lumber Step V: empart 68 outlie 33 zodiac 86 circles 22 recoup 47 lumber 76 Step VI: 85 empart 68 outlie 33 zodiac 22 recoup 47 lumber 76 circle Step VI is the last step of the rearrangement. As per the rules followed in the above steps, determine the appropriate steps for the given input. Input: frights 32 wallops 51 muddles 15 94 backups 64 heights 44 plutons 151. How many numbers appear between ‘muddle’ and ‘pluton’ in Step IV of the given arrangement? (1) Two (2) None (3) One (4) Three (5) More than three 152. Which is the first element in Step II when counted from the right end of the given arrangement? (1) ‘16’ (2) ‘wallop’ (3) ‘pluton’ (4) ‘heights’ (5) ‘31’ 153. In Step III of the given arrangement, how many elements are there between ‘frights’ and ‘heights’? (1) Four (2) Five (3) One (4) Six (5) Seven 154. In Step V, what is the position of ‘16’ from the right end as per the given arrangement? (1) Fourth (2) Third (3) First (4) Seventh (5) Fifth 155. In which step of the given arrangement does ‘backups 16 pluton 43’ appear consecutively in the same order? (1) Step V (2) Step IV (3) Step I (4) Step II (5) Step III

    BPRE–451

    INPUT

    SBI PO EXAMS Directions (1–3) : Study the following information to answer the given questions : (SBI Associate Banks PO Exam. 07.08.2011)

    A word and number arrangement machine when given an input line of words and numbers rearranges them following a particular rule in each step. The following is an illustration of input and rearrangement. (All the numbers are two digits numbers and are arranged as per some logic based on the value of the number.) Input : win 56 32 93 bat for 46 him 28 11 give chance Step I : 93 56 32 bat for 46 him 28 11 give chance win Step II : 11 93 56 32 bat for 46 28 give chance win him Step III: 56 11 93 32 bat for 46 28 chance win him give Step IV: 28 56 11 93 32 bat 46 chance win him give for Step V : 46 28 56 11 93 32 bat win him give for chance Step VI: 32 46 28 56 11 93 win him give for chance bat and Step VI is the last Step of the arrangement of the above input as the intended arrangement is obtained. As per the rules followed in the above steps, find out in each of the following questions the appropriate steps for the given input, Input for the questions : Input : fun 89 at the 28 16 base camp 35 53 here 68 (All the numbers given in the arrangement are two digit numbers.) 1. Which of the following would be the Step II ? (1) 89 fun at 28 16 base camp 35 53 here 68 the (2) 35 53 28 68 16 89 the here fun camp base at (3) 16 89 at fun 28 camp base 35 53 68 the here (4) 53 28 68 16 89 35 the here fun camp base at (5) None of these 2. Which word/number would be at 7th position from the left in Step IV ? (1) base (2) at (3) 35 (4) the (5) 53 3. Which step number would be the following output ? 53 28 68 16 89 at 35 the here fun camp base (1) There will be no such step (2) Ill (3) II (4) V (5) IV Directions (4–8) : Study the given information and answer the following questions : (SBI Probationary Officer Exam 28.04.2013)

    When a word and number arrangement machine is given an input line of words and numbers,, it arranges them following a particular rule. The following is an illustration of input and rearrangement: (All the numbers are two digit numbers) Input : 40 made butter 23 37 cookies salt extra 52 86 92 fell now 19 Step I : butter 19 40 made 23 37 cookies salt extra 52 86 92 fell now Step II : cookies 23 butter 19 40 made 37 salt extra 52 86 92 fell now Step III : extra 37 cookies 23 butter 19 40 made salt 52 86 92 fell now

    Step IV : fell 40 extra 37 cookies 23 butter 19 made salt 52 86 92 now Step V : made 52 fell 40 extra 37 cookies 23 butter 19 salt 86 92 now Step VI : now 86 made 52 fell 40 extra 37 cookies 23 butter 19 salt 92 Step VII : salt 92 now 86 made 52 fell 40 extra 37 cookies 23 butter 19 Step VII is the last step of the above arrangement as the intended arrangement is obtained. As per the rules followed in the given steps, find out the appropriate steps for the given input. Input : 32 proud girl beautiful 48 55 97 rich family 61 72 17 nice life 4. How many steps will be required to complete the given input ? (1) Five (2) Six (3) Seven (4) Eight (5) Nine 5. Which of the following is the third element from the left end of step VI ? (1) beautiful (2) life (3) 61 (4) nice (5) 17 6. Which of the following is step III of the given input ? (1) proud 72 girl 48 family 32 beautiful 17 55 97 rich 61 nice life (2) life 55 girl 48 family 32 beautiful 17 proud 97 rich 61 72 nice (3) girl 48 family 32 beautiful 17 proud 55 97 rich 61 72 nice life (4) family 32 beautiful 17 proud girl 48 55 97 rich 61 72 nice life (5) girl 48 life 55 family 32 beautiful 17 proud 97 rich 61 72 nice 7. What is the position of “nice” from the left end in the final step? (1) Fifth (2) Sixth (3) Seventh (4) Eighth (5) Ninth 8. Which element is third to the right of ‘family’ in Step V ? (1) beautiful (2) 17 (3) proud (4) 97 (5) 32 Directions (9–15) : Study the following information carefully and answer the questions given below : (SBI Specialist Officer (Law Officer : MMGS Scale-II) Online Exam, 19.04.2014)

    A word and number arrangement machine when given an input line of words and numbers rearranges them following a particular rule in each step. The following is an illustration of input and various steps of rearrangement. (All the numbers are two digit numbers). Input :screen 31 award 93 blind 57 troop 19 Step I : 93 screen 31 award blind 57 troop 19 Step II : 93 awardscreen 31 blind 57 troop 19 Step III : 93 award 57 screen 31 blind troop 19 Step IV : 93 award 57 blind screen 31 troop 19 Step V : 93 award 57 blind 31 screentroop 19 Step VI : 93 award 57 blind 31 screen 19 troop And Step VI is the last step of the rearrangement as the desired arrangement is obtained.

    BPRE–452

    INPUT As per rules followed in the above steps, find out in each of the questions the appropriate step for the given input. Input : vital 54 cards 72 help 24 wall 66 lamp 49 9. How many steps would be required to complete the rearrangement ? (1) Eight (2) Six (3) Seven (4) Nine (5) Cannot be determined 10. How many elements (words/numbers) are there between “lamp” and “66” as they appear in Step V ? (1) Five (2) Six (3) Four (4) Seven (5) Three 11. Which step number is the following output ? 72 cards 66 help 54 lamp vital 24 wall 49 (1) Step III (2) Step IV (3) Step V (4) Step VI (5) There is no such step

    12. Which word/number would be to the immediate right of the fourth element from the right end in Step III ? (1) help (2) wall (3) 24 (4) lamp (5) 54 13. In the final step “66” is related to “lamp” and “72” is related to “help” in a certain way, then “cards” is related to (1) vital (2) 66 (3) 49 (4) wall (5) 54 14. At what position of “wall” would come from the left end in the Step V? (1) Ninth (2) Eighth (3) Sixth (4) Seventh (5) Fifth 15. Which word/number would be at the seventh position from the right end in the Step VI ? (1) 54 (2) lamp (3) cards (4) help (5) 66

    Directions (16–22) : Study the following information carefully and answer the questions given below : (SBI Probationary Officer Online Exam, 21.06.2014)

    A word and number arrangement machine when given an input line of words and numbers rearranges them following a particular rule in each step. The following is an illustration of input and various steps of rearrangement. (All the numbers are two digit numbers). Input : sweet 46 nice 36 friend 26 help 96 bright 76 kind 66 Step I : sweet 46 nice 36 friend 26 help bright 76 kind 66 96 Step II : sweet nice 46 36 friend 26 help bright kind 66 76 96 Step III : sweet nice kind 46 36 friend 26 help bright 66 76 96 Step IV : sweet nice kind help 36 friend 26 bright 46 66 76 96 Step V : sweet nice kind help friend 26 bright 36 46 66 76 96 Step VI : sweet nice kind help friend bright 26 36 46 66 76 96 And Step VI is the last step of the rearrangement as the desired arrangement is obtained. As per rules followed in the above steps, find out in each of the questions the appropriate step for the given input. Input : arrow 98 paint 58 lamb 38 each 78 great 18 most 48 rent 88 16. Which word/number would be fifth to the left of the sixth element from the right in the Step V? (1) great (2) arrow (3) lamb (4) 38 (5) 48 17. Which of the following represents the position of “58” in the Step IV? (1) Eighth from left(2) Third from right (3) Ninth from left (4) Eleventh from left (5) Fifth from right 18. How many elements (words/numbers) are there between “most” and “78” as they appear in the Step VI? (1) Eight (2) Seven (3) Nine (4) Five (5) Four 19. Which step number is the following output? rent paint most arrow 58 lamb 38 each great 18 48 78 88 98 (1) There is no such step (2) Step II (3) Step V (4) Step VI (5) Step III 20. Which element (word/number) would be at the eleventh position from the right in the Step III? (1) lamb (2) arrow (3) 58 (4) 38 (5) each

    21. Which element (word/number) would be at the sixth position from the left in the Step VI? (1) 18 (2) arrow (3) great (4) each (5) 38 22. At which of the following positions “great” would appear from the left in the Step V? (1) Fifth (2) Sixth (3) Fourth (4) Second (5) Third Directions (23–30) : Study the following information carefully and answer the questions given below : (SBI Probationary Officer Online Exam, 28.06.2014)

    When a word and number arrangement machine is given an input line of words and numbers, it arranges them following a particular rule. The following is an illustration of input and rearrangement : (All the numbers are two digit numbers). Input : jar eight till 31 68 73 if after 98 16 25 on Step I : after jar eight till 31 68 73 if 98 25 on 16 Step II : after eight jar till 31 68 73 if 98 on 25 16 Step III : after eight if jar till 68 73 98 on 31 25 16 Step IV : after eight if jar on till 73 98 68 31 25 16 Step V : after eight if jar on till 98 73 68 31 25 16 Step V is the last step of the above arrangement as the intended arrangement is obtained. As per the rules followed in the above steps, find out in each of the following questions the appropriate steps for the given input. Input : site grid 19 53 22 call art main 35 66 fill 93 23. Which step number would be the following output ? art call fill grid site main 66 93 53 35 22 19

    BPRE–453

    INPUT (1) II (2) III (3) V (4) IV (5) None of these 24. What will be the position of ‘53’ in Step IV ? (1) Second from the right end (2) Eighth from the left end (3) Fifth from the right end (4) Third from the right end (5) None of these 25. Which of the following is fourth from the left end of Step III ? (1) 53 (2) grid (3) site (4) fill (5) None of these 26. How many steps are needed to complete this arrangement ? (1) V (2) IV (3) VII (4) Cannot be determined (5) None of these 27. Which of the following would be the final arrangement of the above input ? (1) art call fill grid main 93 site 66 53 35 22 19 (2) art call grid fill main site 93 66 53 35 22 19 (3) art call fill grid main site 93 66 53 35 22 19 (4) art call fill grid main site 93 66 53 35 19 22 (5) None of these 28. Which of the following would be Step II ? (1) art call site grid 53 main 35 66 fill 93 22 19 (2) art call site grid 53 main 35 66 fill 22 19 93 (3) art call site grid main 53 35 66 fill 93 22 19 (4) art call site grid 53 main 35 fill 66 93 22 19 (5) None of these 29. Which of the following would be second to the right of the seventh from the right end of Step IV ? (1) main (2) 66 (3) site (4) grid (5) 93 30. How many elements (words/numbers) are there between ‘fill’ and ‘35’ as they appear in the Step III? (1) Seven (2) Six (3) Five (4) Four (5) Eight Directions (31–35) : Study the following information carefully and answer the questions given below : (SBI Management Executive Exam, 19.09.2014)

    When a word and number arrangement machine is given an input line of words and numbers, it arranges them following a particular rule. The following is an illustration of input and rearrangement : (All the numbers are two digit numbers). Input : more vivid assure 33 65 76 from direct 94 14 23 other Step I : assure more vivid 33 65 76 from direct 94 23 other 14 Step II : assure direct more vivid 33 65 76 from 94 other 23 14 Step III : assure direct from more vivid 65 76 94 other 33 23 14 Step IV : assure direct from more vivid 76 94 other 65 33 23 14 Step V : assure direct from more other vivid 94 76 65 33 23 14

    Step V is the last step of the above arrangement as the intended arrangement is obtained. As per the rules followed in the above steps, find out in each of the following questions the appropriate steps for the given input. Input : watch guide help sky 18 54 26 37 away press 76 42 85 decide 31. Which of the following would be the Step III ? (1) away decide watch guide help sky 54 37 press 76 42 85 26 18 (2) away decide guide watch help sky 54 press 76 42 85 37 26 18 (3) away decide help guide watch sky 54 press 42 76 37 85 26 18 (4) away decide guide help watch 54 sky press 76 42 85 37 26 18 (5) None of these 32. How many elements (words/numbers) are there between ‘guide’ and ‘press’ as they appear in the Step IV ? (1) One (2) Two (3) Three (4) Four (5) None 33. Which of the following would be third to the right of the eigth from the right end of the Step V? (1) watch (2) 76 (3) press (4) 42 (5) 54 34. What will be the position of ‘sky’ from the left end in the Step III ? (1) Fifth (2) Fourth (3) Sixth (4) Eighth (5) Seventh 35. Which of the following elements (word/number) would be tenth from the right end in the Step IV ? (1) watch (2) sky (3) 54 (4) press (5) 76 Directions (36–41) : Study the following information carefully and answer the questions given below : (SBI Associates PO Online Exam, 29.11.2014)

    When a word and number arrangement machine is given an input line of words and numbers, it arranges them following a particular rule. The following is an illustration of input and rearrangement. (All the numbers are two digit numbers) Input : initiators 32 67 of 40 the company 20 are 18 96 humble Step I : 21 initiators 32 67 of 40 the company are 96 humble 18 Step II : company 21 initiators 32 67 of 40 the 96 humble 18 are Step III : 40 company 21 initiators 67 of the 96 humble 18 are 32 Step IV : initiators 40 company 21 67 of the 96 18 are 32 humble Step V : 96 initiators 40 company 21 of the 18 are 32 humble 67 Step VI : the 96 initiators 40 company 21 18 are 32 humble 67 of Step VI is the last step of the above arrangement as the intended arrangement is obtained. As per the rules followed in the given steps, find out the appropriate steps for the given input.

    BPRE–454

    INPUT Input: parenting 16 36 and raising 44 children 21 is 89 very 95 demanding 72 job 65 36. In which step are the elements ‘95 job 16 and’ found in the same order ? (1) The given order of elements is not found in any step (2) Fifth (3) Fourth (4) Second (5) Third 37. What is the position of the ‘72’ from the right end in the fourth step ? (1) Fifth (2) Eighth (3) Ninth (4) Seventh (5) Sixth 38. Which element is fifth to the left of the element which is tenth from the left end of the third step ? (1) raising (2) parenting (3) is (4) 72 (5) job 39. How many steps will be required to complete the given arrangement based on the given input ? (1) Ten (2) Eight (3) Six (4) Nine (5) Seven 40. Which element is exactly between ‘parenting’ and ‘raising’ in the second step of the given arrangement ? (1) 95 (2) 16 (3) 44 (4) 21 (5) 36 41. Which of the following is the third last step of the arrangement based on the given input ? (1) 72 44 21 parenting is raising children 89 very 95 and job demanding 16 36 65 (2) 72 parenting 44 is 21 children raising 89 very 95 and 16 demanding 36 job 65 (3) parenting 72 is 44 children 21 raising 89 very 95 16 and 36 demanding 65 job (4) parenting job is demanding children raising 89 very 95 16 and 36 21 44 65 72 (5) parenting is 72 44 children raising 21 89 very 95 16 and 36 65 demanding job Directions (42–47) : Study the following information carefully and answer the questions given below : (SBI Associates PO Online Exam, 30.11.2014)

    When a word and number arrangement machine is given an input line of words and numbers, it arranges them following a particular rule. The following is an illustration of input and rearrangement. (All the numbers are two digit numbers) Input : 14 36 methodology 21 of research 59 crucial 98 very 62 is Step I : 98 14 36 methodology 21 of research 59 crucial very is 62 Step II : 98 very 14 36 methodology 21 of 59 crucial is research 62 Step III : 59 98 very 14 methodology 21 of crucial is research 62 36 Step IV : 59 98 very of 14 21 crucial is methodology research 62 36 Step V : 21 59 98 very of crucial is methodology research 62 36 14 Step VI : 21 59 98 very of is crucial methodology research 62 36 14

    Step VI is the last step of the above arrangement as the intended arrangement has been obtained. As per the rules followed in the given steps find out the appropriate steps for the given input. Input % 65 work in 23 to be 13 89 managed 48 97 load 34 healthy 55 style 42. Which element is exactly between ‘style’ and ‘be’ in second in last step of the given arrangement ? (1) work (2) healthy (3) 23 (4) load (5) be 43. What is the position of ‘48’ from the right end in the third step ? (1) sixth (2) seventh (3) fifth (4) ninth (5) eighth 44. Which of the following is fifth step of the arrangement based on the given input ? (1) 48 65 97 to managed in 23 be 13 load healthy style work 89 55 34 (2) 48 work 65 style 97 in 23 be 13 load healthy 89 managed 55 to 34 (3) 48 65 97 work style in 23 be 13 load healthy managed to 89 55 34 (4) 34 48 55 65 89 97 work style in 23 be 13 load healthy managed to (5) 89 55 34 work style in 23 be 13 load healthy managed to 48 65 97 45. In which step are the elements ‘be 13 healthy in’ found in the same order ? (1) third (2) sixth (3) fourth (4) the given order of elements is not found in any step (5) fifth 46. How many steps will be required to complete the given arrangement based on the given input ? (1) eight (2) ten (3) seven (4) nine (5) six 47. Which element is eighth to left of the element which is twelfth from the left end of the third last step ? (1) 97 (2) 23 (3) work (4) style (5) to Directions (48–52) : Study the following information carefully and answer the questions given below : (SBI Management Executive Exam. 19.09.2014)

    When a word and number arrangement machine is given an input line of words and numbers, it arranges them following a particular rule. The following is an illustration of input and rearrangement : (All the numbers are two digit numbers). Input : more vivid assure 33 65 76 from direct 94 14 23 other Step I : assure more vivid 33 65 76 from direct 94 23 other 14 Step II : assure direct more vivid 33 65 76 from 94 other 23 14 Step III : assure direct from more vivid 65 76 94 other 33 23 14 Step IV : assure direct from more vivid 76 94 other 65 33 23 14 Step V : assure direct from more other vivid 94 76 65 33 23 14

    BPRE–455

    INPUT Step V is the last step of the above arrangement as the intended arrangement is obtained. As per the rules followed in the above steps, find out in each of the following questions the appropriate steps for the given input. Input : watch guide help sky 18 54 26 37 away press 76 42 85 decide 48. Which of the following would be the Step III ? (1) away decide watch guide help sky 54 37 press 76 42 85 26 18 (2) away decide guide watch help sky 54 press 76 42 85 37 26 18 (3) away decide help guide watch sky 54 press 42 76 37 85 26 18 (4) away decide guide help watch 54 sky press 76 42 85 37 26 18 (5) None of these 49. How many elements (words/numbers) are there between ‘guide’ and ‘press’ as they appear in the Step IV ? (1) One (2) Two (3) Three (4) Four (5) None 50. Which of the following would be third to the right of the eigth from the right end of the Step V? (1) watch (2) 76 (3) press (4) 42 (5) 54 51. What will be the position of ‘sky’ from the left end in the Step III ? (1) Fifth (2) Fourth (3) Sixth (4) Eighth (5) Seventh 52. Which of the following elements (word/number) would be tenth from the right end in the Step IV ? (1) watch (2) sky (3) 54 (4) press (5) 76 Directions (53–58) : Study the following information carefully and answer the questions given below :

    53. In which step are the elements ‘95 job 16 and’ found in the same order ? (1) The given order of elements is not found in any step (2) Fifth (3) Fourth (4) Second (5) Third 54. What is the position of the ‘72’ from the right end in the fourth step ? (1) Fifth (2) Eighth (3) Ninth (4) Seventh (5) Sixth 55. Which element is fifth to the left of the element which is tenth from the left end of the third step ? (1) raising (2) parenting (3) is (4) 72 (5) job 56. How many steps will be required to complete the given arrangement based on the given input ? (1) Ten (2) Eight (3) Six (4) Nine (5) Seven 57. Which element is exactly between ‘parenting’ and ‘raising’ in the second step of the given arrangement ? (1) 95 (2) 16 (3) 44 (4) 21 (5) 36 58. Which of the following is the third last step of the arrangement based on the given input ? (1) 72 44 21 parenting is raising children 89 very 95 and job demanding 16 36 65 (2) 72 parenting 44 is 21 children raising 89 very 95 and 16 demanding 36 job 65 (3) parenting 72 is 44 children 21 raising 89 very 95 16 and 36 demanding 65 job (4) parenting job is demanding children raising 89 very 95 16 and 36 21 44 65 72 (5) parenting is 72 44 children raising 21 89 very 95 16 and 36 65 demanding job Directions (59–64) : Study the following information carefully and answer the questions given below :

    (SBI Associates PO Online Exam. 29.11.2014)

    When a word and number arrangement machine is given an input line of words and numbers, it arranges them following a particular rule. The following is an illustration of input and rearrangement. (All the numbers are two digit numbers) Input : 14 36 methodology 21 of research 59 crucial 98 very 62 is Step I : 98 14 36 methodology 21 of research 59 crucial very is 62 Step II : 98 very 14 36 methodology 21 of 59 crucial is research 62 Step III : 59 98 very 14 methodology 21 of crucial is research 62 36 Step IV : 59 98 very of 14 21 crucial is methodology research 62 36 Step V : 21 59 98 very of crucial is methodology research 62 36 14 Step VI : 21 59 98 very of is crucial methodology research 62 36 14 Step VI is the last step of the above arrangement as the intended arrangement has been obtained. As per the rules followed in the given steps find out the appropriate steps for the given input. Input % 65 work in 23 to be 13 89 managed 48 97 load 34 healthy 55 style

    When a word and number arrangement machine is given an input line of words and numbers, it arranges them following a particular rule. The following is an illustration of input and rearrangement. (All the numbers are two digit numbers) Input : initiators 32 67 of 40 the company 20 are 18 96 humble Step I : 21 initiators 32 67 of 40 the company are 96 humble 18 Step II : company 21 initiators 32 67 of 40 the 96 humble 18 are Step III : 40 company 21 initiators 67 of the 96 humble 18 are 32 Step IV : initiators 40 company 21 67 of the 96 18 are 32 humble Step V : 96 initiators 40 company 21 of the 18 are 32 humble 67 Step VI : the 96 initiators 40 company 21 18 are 32 humble 67 of Step VI is the last step of the above arrangement as the intended arrangement is obtained. As per the rules followed in the given steps, find out the appropriate steps for the given input. Input : parenting 16 36 and raising 44 children 21 is 89 very 95 demanding 72 job 65

    (SBI Associates PO Online Exam. 30.11.2014)

    BPRE–456

    INPUT 59. Which element is exactly between ‘style’ and ‘be’ in second in last step of the given arrangement ? (1) work (2) healthy (3) 23 (4) load (5) be 60. What is the position of ‘48’ from the right end in the third step ? (1) sixth (2) seventh (3) fifth (4) ninth (5) eighth 61. Which of the following is fifth step of the arrangement based on the given input ? (1) 48 65 97 to managed in 23 be 13 load healthy style work 89 55 34 (2) 48 work 65 style 97 in 23 be 13 load healthy 89 managed 55 to 34 (3) 48 65 97 work style in 23 be 13 load healthy managed to 89 55 34 (4) 34 48 55 65 89 97 work style in 23 be 13 load healthy managed to (5) 89 55 34 work style in 23 be 13 load healthy managed to 48 65 97 62. In which step are the elements ‘be 13 healthy in’ found in the same order ? (1) third (2) sixth (3) fourth (4) the given order of elements is not found in any step (5) fifth 63. How many steps will be required to complete the given arrangement based on the given input ? (1) eight (2) ten (3) seven (4) nine (5) six 64. Which element is eighth to left of the element which is twelfth from the left end of the third last step ? (1) 97 (2) 23 (3) work (4) style (5) to Directions (65–69) : Study the following information carefully and answer the questions given below :

    Input

    : legation lemon 82 legible 67 46 let 53 75 lethal 38 literatim 26 lend 65. Which of the following combinations represents the first two and the last two elements in the Step V of the given input? (1) literatim, lethal; 64, 62 (2) literatim, lemon; 46, 62 (3) literatim, legation; 83, 62 (4) literatim, legible; 83, 62 (5) literatim, lethal; 35, 64 66. If in the Step VI, ‘legible’ interchanges position with ‘35’ and ‘lemon’ interchanges position with ‘64’, then which of the following elements will be fifth of the left of ‘let’? (1) lethal (2) legation (3) 82 (4) let (5) 35 67. Which element is fifth to the right of the one which is tenth from the right end in Step IV of the given input? (1) 35 (2) lend (3) 67 (4) 75 (5) lemon 68. Which of the following elements come(s) exactly between ‘lemon’ and ‘46’ in the Step II of the given input? (1) 82 and legible (2) Only 67 (3) legible and 67 (4) 82, legible and 67 (5) Only legible 69. In which of the following steps are the elements ‘lethal lemon 82 let 75’ found in the same order? (1) Step V (2) Step VI (3) Step VII (4) Step IV (5) There is no such step Directions (70–72) : Study the following information carefully and answer the questions given below : (SBI PO Phase-II (Main) Exam 04.06.2017)

    An input-output is given in different steps. Some mathematical operations are done in each step. No mathematical operation is repeated in next step.

    5 3

    1 2

    2 4

    2 1

    4 3

    2 4

    (SBI PO Online Main Exam, 31.07.2016)

    When a word and number arrangement machine is given an input line of words and numbers, it rearranges them following a particular rule. The following is an illustration of input and rearrangement (All the numbers are two–digit numbrs) Input % he 12 house 18 higher 78 63 heat 54 her 15 habitat Step I : habitat he house 18 higher 78 63 heat 54 her 15 21 Step II : habitat higher he house 18 78 63 heat 54 her 51 21 Step III : habitat higher house he 78 63 heat 54 her 81 51 21 Step IV : habitat higher house heat he 78 63 her 45 81 51 21 Step V : habitat higher house heat her he 78 36 45 81 51 21 Step VI : habitat higher house heat her he 87 36 45 81 51 21 Step VI is the last step of the above arrangement as the intended arrangement is obtained. As per the rules followed in the given steps, find out the appropriate steps for the given input.

    5 6

    Step I

    3 8 1 6

    Step II

    3.5

    Step III

    8 8 2 2 2

    1.5 Step IV As per the rules followed in the steps given above, find out in each of the following questions the appropriate step for the given input.

    4 2

    5 1

    2 9

    3 2

    7 1

    1 4

    70. Find the addition of two numbers obtained in Step III : (1) 1.5 (2) 3 (3) 7 (4) 3.5 (5) Other than those given as options 71. Find the difference between sum of numbers which obtained in 1st Step and sum of numbers obtained in all other Steps : (1) 232 (2) 185 (3) 187

    BPRE–457

    INPUT (4) 183.5 (5) Other than those given as options 72. Find the multiplication of the numbers obtained in Step II : (1) 426 (2) 462 (3) 188 (4) 98 (5) Other than those given as options Directions (73–77) : Study the following information carefully and answer the questions given below : SBI PO (Prelim Exam), 08.07.2018 (Shift-I))

    A word and number arrangement machine when given an input line of words and numbers rearranges them following a particular rule in each step. The following is an illustration of an input and rearrangement. Input : 19 ear 24 an 18 nose 25 our 32 mind 9 box Step I : 10 19 ear 24 18 nose 25 our 32 mind box an Step II : box 10 ear 24 18 nose 25 our 32 mind an 20 Step III : 26 box 10 24 18 nose our 32 mind an 20 ear Step IV : mind 26 box 10 24 nose our 32 an 20 ear 17 Step V : 23 mind 26 box 10 our 32 an 20 ear 17 nose Step VI : our 23 mind 26 box 10 an 20 ear 17 nose 31 And Step VI is the last step of the rearrangement of the above input. As per the rules followed in the above steps, find out in each of the following questions the appropriate step for the given input. Input : 17 and 32 on 12 never 29 time 7 put 4 fix 73. In which Step the elements ‘32 12 time’ found in the same order? (1) Step I (2) Step II (3) Step IV (4) Step III (5) Step VI 74. In Step III, which of the following element would be at 2nd to the left of 5th from the right end? (1) 12 (2) time (3) put (4) 4 (5) and 75. How many steps are required to complete the above arrangement? (1) Three (2) Four (3) Five (4) Seven (5) None of these 76. Which of the following would be the Step V after arrangement? (1) 11 on 30 fix 8 32 time 18 and never 3 put (2) on 11 30 fix 8 32 time and 18 never 3 put (3) 11 on 30 fix 8 32 time and 18 never put 3 (4) 11 on 30 fix 8 32 time and 18 never 3 put (5) None of these 77. In Step VI, ‘time’ is related to ‘on’ and ‘never’ is related to ‘put’. In the same way ‘fix’ is related to : (1) time (2) and (3) on (4) 11 (5) None of these Directions (78–81) : Study the following information carefully and answer the questions given below : SBI PO (Prelim Exam), 08.07.2018 (Shift-II))

    When a letter and number arrangement machine is given an input line of letters and numbers, it arranges them following a particular rule. The following is an illustration of input and rearrangement. (All the numbers are two digit numbers) Input : G 33 22 18 A 28 U V 10 M

    Step Step Step Step Step

    I : B 11 G 33 22 18 28 U V M II : H 19 B 11 33 22 28 U V M III : N 23 H 19 B 11 33 28 U V IV : V 29 N 23 H 19 B 11 33 V V : W 34 V 29 N 23 H 19 B 11 Step V is the last step of the above arrangement as the intended output of arrangement is obtained. As per the rules followed in the given steps, find the appropriate steps for the given input. Input : K 50 39 P 15 27 C Y 20 F 78. In Step III, which elements appears exactly between ‘L’ and ‘21’? (1) Only ‘28’ (2) Only ‘K’ (3) None (4) Both ‘21’ and ‘K’ (5) Both ‘28’ and ‘G’ 79. What is the position of ‘40’ from the left of ‘D’ in the second last Step? (1) Third (2) Eighth (3) Seventh (4) Fifth (5) Ninth 80. How many elements are there between ‘39’ and the element which is fifth from the right end in Step II? (1) Four (2) Two (3) None (4) One (5) Five 81. What will be the resultant if the third element from right end of Step IV is added to the second element from the left end of Step I? (1) 32 (2) 55 (3) 91 (4) 37 (5) 66 Directions (82–86) : A number arrangement machine when given an input line of numbers rearranges them following particular calculation rule in each step. Consider the following illustration of input and rearrangement : (SBI PO Main Exam, 20.07.2019)

    Input 6 5 7 6 8 7 9 Step–I 9 7 10 8 11 9 12 Step–II 15 9 16 10 17 11 18 Step–III 33 13 34 14 35 15 36 Step–IV 87 21 88 22 89 23 90 Input 11 10 15 14 18 17 20 82. Which of the following number is fourth from the left end in Step-IV? (1) 22 (2) 42 (3) 30 (4) 18 (5) 89 83. Which of the following number is sixth from the right end in Step-III? (1) 45 (2) 49 (3) 38 (4) 21 (5) 18 84. Which of the following number is in the middle of Step-II? (1) 24 (2) 18 (3) 21 (4) 20 (5) 14 85. Which of the following number is to the immediate left of a number which is 3rd from left end in Step-I? (1) 12 (2) 19 (3) 20 (4) 23 (5) 28 86. What is the sum of output numbers of 10, 14 and 20 from the input? (1) 200 (2) 214 (3) 157 (4) 163 (5) 240

    BPRE–458

    INPUT

    RBI GRADE–B/NABARD GRADE–A OFFICER EXAMS Directions (1–5) : Study the following information carefully and answer the given questions : (RBI Grade-B Officer’s Exam. 18.12.2011)

    A word and number arrangement machine when given an input line of words and numbers rearranges them following a particular rule in each step. The following is an illustration of input and rearrangement. (All the numbers are two digits numbers.) Input : gate 20 86 just not 71 for 67 38 bake sun 55 Step I : bake gate 20 just not 71 for 67 38 sun 55 86 Step II : for bake gate 20 just not 67 38 sun 55 86 71 Step III : gate for bake 20 just not 38 sun 55 86 71 67 Step IV : just gate for bake 20 not 38 sun 86 71 67 55 Step V : not just gate for bake 20 sun 86 71 67 55 38 Step VI : sun not just gate for bake 86 71 67 55 38 20 and Step VI is the last step of the above input as the desired arrangement is reached. As per the rules followed in the above steps, find out in each of the following questions the appropriate step for the given input. Input : 31 rise gem 15 92 47 aim big 25 does 56 not 85 63 with moon

    1. How many steps will be required to complete the rearrangement ? (1) Eight (2) Six (3) Seven (4) Five (5) None of these 2. Which word/number would be at 7th position from the left in Step IV ? (1) rise (2) aim (3) big (4) 15 (5) 47 3. Which step number is the following output ? rise not moon gem does big aim 15 with 92 85 63 56 47 31 25 (1) Step V (2) Step VII (3) Step IV (4) Step VIII (5) There is no such step 4. Which of the following represents the position of ‘92’ in the VIth step? (1) Ninth from the left (2) Fifth from the right (3) Sixth from the right (4) Ninth from the right (5) Seventh from the left 5. Which word/number would be at the 5th position from the right in the last step ? (1) gem (2) 63 (3) 56 (4) 85 (5) does

    Directions (6–11) : Study the following information carefully and answer the questions given below : (RBI Officer Grade ‘B’ Online Exam. 25.08.2013)

    A word and number arrangement machine when given an input line of words and numbers rearranges them following a particular rule in each step. The following is an illustation of input and various steps of rearrangement. (All the numbers are two digit numbers). Input : plan Step I : 92 Step II : 92 Step III : 92 Step IV : 92 Step V : 92 Step VI : 92

    more plan 84 84 84 84 84

    vacation more plan 61 61 61 61

    35 vacation vacation plan 56 56 56

    56 35 35 vacation vacation 35 35

    92 nice holiday 56 nice tours 56 nice tours 35 56 tours 35 tours 12 vacation 12 tours 12 vacation tours

    tours 84 61 12 plan plan plan

    84 61 12 nice nice nice nice

    61 12 more more more more more

    12 holiday holiday holiday holiday holiday holiday

    And Step VI is the last step of the rearrangement as the desired arrangement is obtained. As per rules followed in the above steps, find out in each of the questions the appropriate step for the given input. Input :hard work pays 96 42 in 79 long run 18 25 57 6. Which step number is the following output? 96 79 57 42 work run 18 25 pays long in hard (1) Step II (2) Step III (3) Step IV (4) Step V (5) There is no such step 7. How many elements (words/numbers) are there between “work” and “in” as they appear in Step III? (1) Six (2) Five (3) Seven (4) Four (5) Three 8. Which of the following represents the positions of ‘pays’ in the Step II? (1) Seventh from right (2) Sixth from left (3) Eighth from right (4) Fourth from left (5) Fifth from left

    9. Which word/number would be at the seventh position from the right in the Setp IV? (1) work

    (2) run

    (4) 18

    (5) 25

    (3) long

    10. Which word/number would be at the ninth position from the left in the Step V? (1) long

    (2) 18

    (4) 25

    (5) in

    (3) pays

    11. Which word/number would be third to the left of the fourth element from the right in the Step III? (1) pays

    (2) 42

    (4) 18

    (5) work

    BPRE–459

    (3) run

    INPUT Directions (12–16) : Study the following information carefully and answer the questions given below : (RBI Officer Grade ‘B’ Phase-I Exam, 03.08.2014)

    A word and number arrangement machine when given an input line of words and numbers rearranges them following a particular rule in each step. The following is an illustration of input and various steps of rearrangement. (All the numbers are two digit numbers). Input % 10 sea 25 57 41 rose giraffe 85 hot 32 lost 77 99 beard cost palm Step I : 10 beard 25 57 41 rose giraffe 85 hot 32 lost 77 cost palm sea 99 Step II : 10 25 beard cost 57 41 giraffe hot 32 lost 77 palm rose sea 85 99 Step III : 10 25 32 beard cost giraffe 57 41 hot lost palm rose sea 77 85 99 Step IV : 10 25 32 41 beard cost giraffe hot lost palm rose sea 57 77 85 99 And Step IV is the last step of the rearrangement as the desired arrangement is obtained. As per rules followed in the above steps, find out in each of the questions the appropriate step for the given input. Input : 31 11 win arm blanket zebra 24 81 chip team slip 62 55 dawn 91 78 12. Which of the following represents the position of “slip” in the Step III? (1) Ninth from right (2) Eighth from left (3) Eighth from right (4) Seventh from left (5) Sixth from left 13. How many elements (words/numbers) are there between “dawn” and “81” as they appear in Step IV? (1) Five (2) Six (3) Seven (4) Four (5) Eight 14. Which element (word/number) would be at the ninth position from the left in the Step II? (1) 62 (2) slip (3) 55 (4) team (5) dawn 15. At which of the following position “dawn” would appear from the right in the Step III? (1) Tenth (2) Ninth (3) Seventh (4) Fifth (5) Eighth 16. Which word/number would be third to the left of the sixth element from the right in the Step III? (1) dawn (2) 55 (3) chip (4) slip (5) 62 Directions (17 – 21) : Study the following information carefully and answer the questions given below : (RBI Officer Grade ‘B’ Phase-I Exam. 21.11.2015)

    When a word and number arrangement machine is given an input line of words and numbers, it arranges them following. a particular rule. The following is an illustration of input and rearrangement : (All the numbers are two digit numbers). Input : bear 24 binders brave 91 17 but 68 bailer 35 be 74 Step I : 19 bear 24 binders brave 17 but 68 bailer 35 74 be

    Step II

    : 47 19 bear 24 binders brave 17 68 bailer 35 be but Step III : 86 47 19 24 binders brave 17 bailer 35 be but bear Step IV : 53 86 47 19 24 binders 17 bailer be but bear brave Step V : 42 53 86 47 19 binders 17 be but bear brave bailer Step VI : 71 42 53 86 47 19 be but bear brave bailer binders Step VI is the last step of the above arrangement as the intended arrangement is obtained. As per the rules followed in the given steps, find out the appropriate steps for the given input. Input: tyre 71 toughen 59 tip 82 13 thanks to 68 table 46 17. If in the last step, ‘1’ is added to each of the odd numbers and ‘2’ is subtracted from each of the even numbers, then how many numbers multiple of ‘4’ will be formed? (1) Two (2) Three (3) More than three (4) None (5) One 18. If in step III, ‘28’ interchanges its position with ‘table’ and ‘59’ also interchanges its position with ‘tip’ then which of element will be to the immediate left of ‘46’? (1) ‘28’ (2) ‘to’ (3) ‘table’ (4) ‘tip’ (5) ‘59’ 19. Which of the following combinations represents the third and the ninth element in second last step of the given input from the left end? (1) ‘95’ and ‘tip’ (2) ‘table’ and ‘86’ (3) ‘17’ and ‘to’ (4) ‘86’ and ‘tip’ (5) ‘tyre’ and ‘17’ 20. Which element comes exactly between ‘59’ and ‘thanks’ in Step II of the given input? (1) toughen (2) 46 (3) 13 (4) table (5) 68 21. Which element is third to the right of the one which is eighth from the right end in Step IV? (1) 13 (2) tip (3) thanks (4) to (5) 46 Directions (22–23) : Study the following information carefully and answer the questions given below: (Nabard Officer Grade ‘A’ Online Exam. 01.03.2015)

    When a word and number arrangement machine is given an input line of words and numbers, it arranges them following a particular rule. The following is an illustration of input and rearrangement: (All the numbers are two-digit numbers) Input : 32 prevent 46 a 62 mishap 54 by carefulness 93 Step I : 93 32 46 a 62 mishap 54 by carefulness prevent Step II : 62 93 32 46 a 54 by carefulness prevent mishap Step III : 54 62 93 32 46 a by prevent mishap carefulness Step IV : 46 54 62 93 32 a prevent mishap carefulness by

    BPRE–460

    INPUT Step V : 32 46 54 62 93 prevent mishap carefulness by a Step V is the last step of the above arrangement as the intended arrangement is obtained. As per the rules followed in the given steps, find out the appropriate steps for the given input. Input : regular 46 exercise 31 29 must for 54 all 83 91 now 22. Which element is seventh to left of the element which is eleventh from the left end of the last step ? (1) 83 (2) 54 (3) all (4) 46 (5) regular 23. Which element is exactly between ‘31’ and ‘for’ in third step of the given arrangement ? (1) 29 (2) 54 (3) must (4) 91 (5) regular Directions (24–28) : Study the following information carefully and answer the questions given below : (NABARD Assistant Manager Exam, 15.15.2016)

    When a word and number arrangement machine is given an input line of words and numbers, it arranges them following a particular rule. The following is an illustration of input and arrangement. (All the numbers are two digit numbers) Input : 84 PARTY EAGLE 29 HOUND 16 58 ROAST 71 BEACH Step I : 16 ROAST 84 PARTY EAGLE 29 HOUND 58 71 BEACH Step II : 16 ROAST 84 EAGLE HOUND 58 71 BEACH 29 PARTY Step III : 58 HOUND 16 ROAST 84 EAGLE 71 BEACH 29 PARTY Step IV : 58 HOUND 16 ROAST 84 BEACH 29 PARTY 71 EAGLE Step V : 84 BEACH 58 HOUND 16 ROAST 29 PARTY 71 EAGLE Step V is the last step of the above arrangement as the intended output of arrangement is obtained. As per the rules followed in the given steps, find the appropriate steps for the given input. Input : CHANT 18 SALTY 45 ABACUS WARDEN 30 91 67 KNIGHT 24. Which element appears exactly between the fourth element from the left and fifth element from the right in the second last Step? (1) 67 (2) ABACUS (3) WARDEN (4) KNIGHT (5) 91 25. Which of the following represent the two consecutive elements to the immediate right of ‘KNIGHT’ in the last Step? (1) 45, CHANT (2) 30, SALTY (3) 18, WARDEN (4) 91, SALTY (5) WARDEN, 30 26. What is the position of ‘18’ from the left of ‘SALTY’ in Step III? (1) Seventh (2) Second (3) Fourth (4) Third (5) Sixth 27. In Step III, which element(s) appear(s) exactly between ‘WARDEN’ and ‘ABACUS’? (1) Only ‘91’ (2) Both ‘CHANT’ and ‘67’ (3) Both ‘18’ and ‘SALTY’

    (4) Only ‘KNIGHT’ (5) Only ‘CHANT’ 28. In Step I, ‘ABACUS : WARDEN’ are related in the same way as ‘CHANT : 91’ in Step IV. In the same way ‘WARDEN’ : _____ in Step V? (1) CHANT (2) 91 (3) 18 (4) ABACUS (5) Other than those given as options Directions (29–33) : Study the following information carefully and answer the questions given below : (RBI Officer Grade ‘B’ Phase-I Exam, 04.09.2016 (Shift-I))

    When a word and number arrangement machine is given an input line of words and numbers, it rearranges them following a particular rule. The following is an illustration of input and rearrangement (All the numbers are two digit numbers.) Input : 42 prey burn 78 21 melt gulp 96 83 head Step I : ban 23 42 prey 78 melt gulp 96 83 head Step II : gap 44 ban 23 prey 78 melt 96 83 head Step III : had 80 gap 44 ban 23 prey melt 96 83 Step IV : mat 85 had 80 gap 44 ban 23 prey 96 Step V : pay 98 mat 85 had 80 gap 44 ban 23 Step V is the last step of the above arrangement as the intended output of arrangement is obtained. As per the rules followed in the given steps, find the appropriate steps for the given input. Input : 61 rust 33 colt 86 four torn 28 49 leap 29. Which of the following is the fourth to the left of the eighth element from the left end of Step II? (1) cat (2) far (3) 35 (4) rust (5) 30 30. Which of the following represents the element that is fifth to the right of ‘cat’ in step III? (1) 63 (2) torn (3) lap (4) far (5) 86 31. In step III, how many elements are there between ‘86’ and the third element from the left end? (1) More than three(2) One (3) Three (4) None (5) Two 32. What is the difference between the third element from the right end in step V and the fifth element from the left end in step II? (1) 31 (2) 55 (3) 26 (4) 5 (5) 16 33. ‘torn’ is related to ‘rust’ in step I in the same way as ‘lap is related to ‘tan’ in step V. Following the same pattern to which element is ‘86’ related to in step IV? (1) cat (2) 51 (3) 35 (4) far (5) 30 Directions (34– 38) : Study the following information carefully and answer the questions given below : (RBI Officer Grade ‘B’ Phase-I Exam, 04.09.2016 (Shift-II))

    When a word and number arrangement machine is given an input line of words and numbers, it rearranges them following a particular rule. The following is an illustration of input and rearrangement (All the numbers are two digit numbers.)

    BPRE–461

    INPUT Input : 87 46 part 68 30 burn heat 93 goat mend Step I : 87 46 68 30 burn heat goat mend put 91 Step II : 46 68 30 burn heat goat put 91 mud 85 Step III : 46 30 burn goat put 91 mud 85 hut 66 Step IV : 30 burn put 91 mud 85 hut 66 gut 44 Step V : put 91 mud 85 hut 66 gut 44 bun 28 Step V is the last step of the above arrangement as the intended output of arrangement is obtained. As per the rules followed in the given steps, find the appropriate step for the given input. Input : 61 neon 17 skin 28 four cart 86 42 here 34. What is the difference between the fifth element from the left end in Step II and the fifth element from the right end in Step IV ? (1) 17 (2) 5 (3) 31 (4) 37 (5) 9 35. In Step IV, how many elements are there between ‘84’ and the third element from the right end ? (1) One (2) None (3) Two (4) More than three (5) Three 36. Which of the following is the third to the left of the seventh element from the left end of Step II ? (1) sun (2) cart (3) 28 (4) 84 (5) 42 37. Which of the following represents the element that is fifth to the left of ‘nun’ in Step III ? (1) 35 (2) here (3) 28 (4) cart (5) hue 38. ‘sun’ is related to ‘cart’ in Step I in the same way as ‘hue’ is related to ‘84’ in Step III. Following the same pattern to which element is ‘59’ related to in Step V ? (1) 40 (2) 35 (3) cut (4) sun (5) fur Directions (39–44) : Study the following information carefully and answer the questions given below : (RBI Officer Grade ‘B’ Phase-I Exam 17.06.2017)

    When a word and number arrangement machine is given an input line of words and numbers, it arranges them following a particular rule. The following is an illustration of input and rearrangement : (All the numbers are two-digit numbers before performing the operations) Input : tweet 43 also 13 suit 29 money kite 71 59 Step I : ls tweet 43 13 suit 29 money kite 59 73 Step II : kt ls tweet 43 13 suit 29 money 73 61 Step III : mny kt ls tweet 13 suit 29 73 61 47 Step IV : st mny kt ls tweet 13 73 61 47 31 Step V : twt st mny kt ls 73 61 47 31 17 Step V is the last step of the above arrangement as the intended output of arrangement is obtained. As per the rules followed in the given Steps, find the appropriate Steps for the given input. Input : neat bites 23 11 piles your 37 79 give 47 39. As per the given arrangement, in Step IV, ‘pls’ is related to ‘gv’ following a certain pattern. Following

    the same pattern, ‘83’ is related to ‘41’ in Step III. In Step V, to which of the following is ‘bts’ related to following the same pattern? (1) 29 (2) 41 (3) 53 (4) nt (5) yr 40. Which is the third element to the left of the seventh element from the left end in Step V of the given arrangement? (1) gv (2) 53 (3) nt (4) 23 (5) 83 41. How many elements are there between ‘your’ and ‘83’ in Step III of the given arrangement? (1) None (2) Two (3) Three (4) One (5) More than three 42. Which is the fourth element to the right of ‘11’ in the Step IV of the given arrangement? (1) 29 (2) 41 (3) bts (4) 83 (5) your 43. In which one of the following Steps is ‘bts 11 your’ found consecutively in the same order in the given arrangement? (1) Step II (2) Step III (3) There is no such Step (4) Step V (5) Step IV 44. In Step II of the given arrangement, which elements appear to the immediate right and immediate left of ‘neat’ respectively? (1) ‘37’and ‘piles’ (2) ‘give’ and ‘11' (3) ‘11’ and ‘your’ (4) ‘23’ and ‘bts’ (5) ‘piles’ and ‘gv’ Directions (45–49) : Study the following information carefully and answer the questions given below : (RBI Assistant Manager Online Exam 25.03.2017)

    When a word and number arrangement machine is given an input line of words and numbers, it arranges them following a particular rule. The following is an illustration of input and rearrangement. (All the numbers are two-digit numbers) Input : react 21 east 92 11 basket 68 untidy heart 85 Step I : basket react 21 east 92 68 untidy heart 85 11 Step II : east basket react 92 68 untidy heart 85 11 21 Step III : heart east basket react 92 untidy 85 11 21 68 Step IV : react heart east basket 92 untidy 11 21 68 85 Step V : untidy react heart east basket 11 21 68 85 92 Step V is the last step of the above arrangement as the intended output of arrangement is obtained. As per the rules followed in the given steps, find the appropriate steps for the given input. Input : weight 56 fasten 71 beast 97 49 mould 12 stronger 45. Which Step number is the following output? mould fasten beast weight 71 97 stronger 12 49 56 (1) III (2) IV (3) II (4) I (5) There is no such Step 46. What is the position of ‘mould’ from the left of ‘12’ in Step IV? (1) Seventh (2) Third (3) Fifth (4) Fourth (5) Second

    BPRE–462

    INPUT 47. Which of the following represent the third and the seventh elements from the left of the first Step? (1) 71, mould (2) 56, 49 (3) 56, 97 (4) fasten, 97 (5) fasten, 49 48. How many elements are there between ‘stronger’ and ‘56’ in the last Step? (1) Two (2) Four (3) Three (4) Five (5) More than five 49. Which of the following is fifth element to the right of ‘fasten’ in Step II? (1) 56 (2) 71 (3) mould (4) stronger (5) 97 Directions (50–54) : Study the following information carefully and answer the questions given below : (RBI Grade B Officer Exam, 09.11.2019)

    A number arrangement machine when given an input line of numbers rearranges them following a particular rule. The following is an illustration of input and rearrangement. (Note : All the given numbers are two digit numbers only) Input : 84 79 36 42 17 63 89 95 28 57 Step–I : 20 14 84 79 36 42 63 89 95 57 Step–II : 18 21 20 14 84 79 63 89 95 57 Step–III : 60 66 18 21 20 14 84 79 89 95 Step–IV : 82 42 60 66 18 21 20 14 89 95 Step–V : 92 98 82 42 60 66 18 21 20 14 Step–V is the last step of the above arrangement as the intended output of rearrangement is obtained. As per the rules followed in the above steps, determine the appropriate steps for the given input : Input : 47 87 38 15 56 54 64 71 90 29 50. Which of the element/s appear in the given input as well as in the last step of the given input? (1) 47 and 90 (2) Only 71 (3) 47 and 38 (4) There is no such element (5) Only 90 51. What will be the sum of the third element from the right end of step II and the fifth element from the left end of step I of the given input? (1) 14 (2) 102 (3) 50 (4) 135 (5) 110 52. In step IV, what is the position of ‘87’ from the right end? (1) Fifth (2) Fourth (3) Third (4) Second (5) Sixth 53. Which of the following is the fourth element to the left of the seventh element from the left end of the step III of the given input? (1) 19 (2) 18 (3) 50 (4) 71 (5) 27 54. How many elements are there between ‘50’ and ‘54’ in step II of the given input? (1) One (2) Two (3) None (4) Three (5) More than three

    INSURANCE EXAMS Directions (1–8) : Study the following information and answer the questions given : (United India Insurance AO Exam. 27.03.2011)

    When a word arrangement machine is given an input line of words, it arranges them following a particular rule. The following is an illustration of input and rearrangement : Input : grow after seen craft preach help Step I : grow seen craft preach help after Step II : grow seen preach help after craft Step III : seen preach help after craft grow Step IV : seen preach after craft grow help Step V : seen after craft grow help preach Step VI : after craft grow help preach seen Step VI is the last step of the above arrangement as the intended arrangement is obtained. As per the rules followed in the above steps, find out in each of the following questions the appropriate steps for the given input. (1–4) : Input : dare a beach null wrap steel 1. How many steps will be required to reach the intended arrangement for the given input ? (1) V (2) VII (3) VI (4) IV (5) None of these 2. Which one of the following will be step II for the given input? (1) dare null wrap steel beach a (2) dare null wrap steel a beach (3) dare null steel wrap a beach (4) a beach dare null wrap steel (5) None of these 3. Which of the following will be second from the left end of step V ? (1) beach (2) steel (3) a (4) wrap (5) None of these 4. Which of the following will be fourth from the right end of step III ? (1) wrap (2) a (3) beach (3) steel (5) None of these Directions (5–8) : Following are steps of an input. Rearrange them and answer the questions : (United India Insurance AO Exam. 27.03.2011)

    (A) metal offer arrange blue kite (B) arrange blue kite metal offer (C) kite metal offer arrange blue (D) offer arrange blue kite metal (E) kite metal blue offer arrange 5. Which of the following is step III ? (1) A (2) B (3) C (4) D (5) E 6. Which of the following is first from the right end of step II ? (1) arrange (2) kite (3) metal (4) blue (5) None of these

    BPRE–463

    INPUT 7. Which of the following is step V? (1) A (2) B (3) C (4) D (5) E 8. Which of the following is step IV ? (1) A (2) B (3) C (4) D (5) E Directions (9–13) : A letter/letter combination arrangement machine when given an input of letters/letter combinations rearranges them following a particular rule in each step. The following is an illustration of the input and the steps of rearrangement. (Oriental Insurance Company Exam.08.04.2012)

    Input : Going but for crept te light sir Step I : Crept going but for te light sir Step II : Crept going light but for te sir Step III : Crept going light but for sir te (Step III is the last step for this input.) As per the rules followed in the above steps, find out in the given questions the appropriate step for the given input. 9. Input : the in car as he may me Which of the following will be the third step for this input ? (1) car the in as he may me (2) car may the as in he me (3) car as may he the in me (4) car may the in as he me 10. If the second step of an input is ‘clever remand window sales batch tiger never’ which of the following steps would be last step of that input ? (1) IV (2) V (3) VI (4) VII 11. If the input is ‘true se veto be nuke my like,’ which of the following will be the IV step ? (1) veto true nuke like se be my (2) be my se like true veto nuke (3) like nuke true veto be se my (4) be my like se true veto nuke 12. Input : ‘more fights cats cough sough acts idea’. Which of the following steps would be the last step for this input ? (1) IV (2) V (3) VI (4) VII 13. In how many steps the following input be fully arranged ? Input : amis goes to the bar after dinner everyday (1) Four (2) Five (3) Six (4) Seven Directions (14–18) : Study the following information carefully and answer the given questions :

    Step I :

    85 wheat tent 13 21 ask 63 steal 49 hand 54 vast Step II : 63 vast 85 wheat tent 13 21 ask steal 49 hand 54 Step III : 54 tent 63 vast 85 wheat 13 21 ask steal 49 hand Step IV : 49 steal 54 tent 63 vast 85 wheat 13 21 ask hand Step V : 21 hand 49 steal 54 tent 63 vast 85 wheat 13 ask Step VI : 13 ask 21 hand 49 steal 54 tent 63 vast 85 wheat and Step VI is the last step of the above input, as the desired arrangement is obtained. As per the rules followed in the above steps, find out in each of the following questions the appropriate step for the given input. Input : store 95 clean 56 tape 15 break 28 feet 35 wait 69 ice 71 14. Which step number is the following output? 35 feet 56 ice 69 store 71 tape 95 wait clean 15 break 28 (1) Step V (2) Step III (3) Step VI (4) Step IV (5) There is no such step 15. Which word/number would be at sixth position from the right in Step IV ? (1) 15 (2) wait (3) clean (4) 95 (5) 28 16. How many elements (words/numers) are there between ‘feet’ and ‘15’ as they appear in the second last step of the output ? (1) Six (2) Seven (3) Five (4) Eight (5) Nine 17. Which of the following represents the position of ‘wait’ in the third Step ? (1) Ninth from the left (2) Sixth from the left (3) Eighth from the right (4) Seventh from the right (5) Tenth from the right 18. Which word/number would be at seventh position from the left in the Step VI ? (1) ice (2) store (3) tape (4) 71 (5) 69 Directions (19–24) : Study the following information carefully and answer the questions given below : When a number and number arrangement machine is given an input line of words and numbers, it arranges them following a particular rule. The following is an illustration of input and rearrangement. (NIACL Administrative Officer (AO) Exam, 10.01.2015)

    (United India Insurance AO Exam. 26.05.2013)

    A word and number arrangement machine when given an input line of words and numbers rearranges them following a particular rule in each step. The following is an illustration of input and rearrangement. (All the numbers are two digit numbers). Input : tent 13 wheat 21 ask 63 steal 49 hand 54 vast 85

    (AII the numbers are two digit numbers) Input : 36 24 maintain 87 safe distance 60 12 with other 98 cars Step I : cars 36 24 maintain 87 safe distance 60 with other 98 12 Step II : 24 cars 36 maintain 87 safe 60 with other 98 12 distance

    BPRE–464

    INPUT Step III : maintain 24 cars 87 safe 60 with other 98 12 distance 36 Step IV : 60 maintain 24 cars 87 safe with 98 12 distance 36 other Step V : safe 60 maintain 24 cars with 98 12 distance 36 other 87 Step VI : 98 safe 60 maintain 24 cars 12 distance 36 other 87 with Step VI is the last step of the above arrangement as the intended arrangement is obtained. As per the rules followed in the given steps, find out the appropriate steps for the given input. Input : weight 34 93 and 14 density 84 concept to 49 74 be understood 56 19. Which element is seventh to the right of the element which is eleventh from the right end of the last step of the given arrangement ? (1) 74 (2) to (3) understood (4) 56 (5) density 20. What is the position of ‘concept’ from the right end of the second last step of the given arrangement ? (1) Eighth (2) Eleventh (3) Ninth (4) Seventh (5) Tenth 21. Which element is/are exactly between ‘93’ and ‘to’ in the fourth step of the given arrangement ? (1) Both density and 84 (2) density (3) 84 (4) Both density and concept (5) There is no element 22. What is the position of ‘weight’ from the left of ‘56’ in the third step of the given arrangement ? (1) Sixth (2) Fourth (3) Ninth (4) Seventh (5) Fifth 23. In which step of the given arrangement are the elements ‘to 49 74 understood’ found in the same order ? (1) Fourth (2) Second (3) Third (4) Fifth (5) The given order of elements is not found is any step 24. Which of the following is the third last step of the given arrangement ? (1) to 56 concept 34 and weight 93 84 understood 14 be 49 density 74 (2) to density concept be and weight 93 84 understood 14 34 49 56 74 (3) 14 34 49 56 74 weight 93 84 understood to density concept be and (4) to density concept be and 14 34 49 56 74 weight 93 84 understood (5) to 74 concept 49 14 and weight 93 84 understood be 34 density 56 Directions (25–29) : Study the following information carefully and answer the questions given below : When a number and number arrangement machine is given an input line of words and numbers, it arranges them following a particular rule. The following is an illustration of input and rearrangement. (NIACL Administrative Officer (AO) Online Exam, 11.01.2015)

    (All the numbers are two digit numbers) Input : talk 48 13 rose and 99 76 32 west jug height28 56 bang Step I : 13 talk 48 rose 99 76 32 west jug height 28 56 bang and Step II : 28 13 talk 48 rose 99 76 32 west jug height56 and bang Step III : 32 28 13 talk 48 rose 99 76 west jug 56 and bang height Step IV : 48 32 28 13 talk rose 99 76 west 56 and bang heightjug Step V : 56 48 32 28 13 talk 99 76 west and bang height jug rose Step VI : 76 56 48 32 28 13 99 west and bang heightjug rose talk Step VII : 99 76 56 48 32 28 13 and bang height jug rose talk west Step VII is the last step of the above arrangement as the intended arrangement is obtained. As per the rules followed in the given steps, find out the appropriate steps for the given input. Input : 84 why sit 14 32 not best ink feet 51 27 vain 68 92 25. Which element (s) is/are exactly between ‘why’ and ‘51’ in the Step III ? (1) sit (2) Both ‘not’ and ‘vain’ (3) Both ‘sit’ and ‘not’ (4) not (5) There is no element 26. Which of the following is the Step IV of the given arrangement ? (1) 51 32 27 14 sit why 84 feet 68 92 best not ink vain (2) 68 51 32 27 14 sit why vain 92 not feet best ink 84 (3) 51 32 27 14 68 sit why feet not 84 92 best ink vain (4) 51 32 27 14 vain sit why 92 68 feet best not ink 84 (5) 51 32 27 14 84 why sit vain 68 92 best feet ink not 27. What is the position of ‘ink’ to the left of ‘best’ in the Step II ? (1) Fifth (2) Fourth (3) Sixth (4) Third (5) Seventh 28. Which element is sixth to the left of the thirteenth element from the left in the Step V ? (1) 84 (2) vain (3) 92 (4) why (5) 14 29. In which step of the given arrangement are elements ‘27 14 why vain best’ appear in the same order ? (1) Fourth (2) Sixth (3) Fifth (4) Third (5) Second

    BPRE–465

    INPUT Directions (30–34) : Study the following information carefully and answer the questions given below : (NIACL Administrative Officer (AO) Online Exam, 12.01.2015)

    A word and number arrangement machine when given an input line of words and number rearranges them following a particular rule. The following is an illustration of output and rearrangement. Input : 52 peak 91 snow freeze 46 cold 15 high 31 73 trek Step I : 15 52 peak snow freeze 46 cold high 31 73 trek 91 Step II : 15 31 52 peak snow freeze 46 cold high trek 73 91 Step III : 15 31 46 peak snow freeze cold high trek 52 73 91 Step IV : 15 31 46 cold peak snow freeze high trek 52 73 91 Step V : 15 31 46 cold freeze peak snow high trek 52 73 91 Step VI : 15 31 46 cold freeze high peak snow trek 52 73 91 Step VI is the last step of the rearrangement. As per the rules followed in the above steps, find out in each of the following questions the appropriate steps for the given input. Input for the questions. Input : 67 hot sun 19 best 83 ice 49 ace 77 cut 37 30. How many steps would be needed to complete the rearrangement ? (1) X (2) VIII (3) IX (4) VII (5) None of these 31. Which step number would be the following output ? 19 37 49 ace best hot sun ice cut 67 77 83 (1) II (2) VI (3) V (4) IV (5) None of these 32. Which of the following would be the Step I ? (1) 19 37 49 hot sun best ice ace cut 67 77 83 (2) 83 67 hot sun best ice 49 ace 77 cut 37 19 (3) 19 67 ace best hot sun ice 49 77 cut 37 83 (4) 19 67 hot sun best ice 49 ace 77 cut 37 83 (5) None of these 33. Which of the following would be the final arrangement ? (1) 67 77 83 ace best cut hot ice sun 19 37 49 (2) 19 37 49 ace best cut hot ice sun 67 77 83 (3) 19 37 49 67 77 83 ace best cut hot ice sun (4) 19 37 49 ace ice best cut hot sun 67 77 83 (5) None of these 34. In Step IV, which of the following word/number would be on 7th position from the right ? (1) sun (2) best (3) 67 (4) cut (5) None of these Directions (35 –39) : Study the following information carefully and answer the questions given below : (LIC Assistant Administrative Officer (AAO) Online Exam. 22.03.2015)

    When a word and number arrangement machine is given an input line of words and numbers, it arranges them following a particular rule. The following is an illustration of input and rearrangement.

    (All the numbers are two-digit numbers) Input : 24 method 87 67 of data 34 collection 45 12 specified now Step I : 12 method 87 67 of data 34 collection 45 specified now 24 Step II : 34 12 method 87 67 of data collection specified now 24 45 Step III : 67 34 12 method of data collection specified now 24 45 87 Step IV : collection 67 34 12 method of specified now 24 45 87 data Step V : method collection 67 34 12 of specified 24 45 87 data now Step VI : of method collection 67 34 12 24 45 87 data now specified Step VI is the last step of the above arrangement as the intended arrangement is obtained. As per the rules followed in the given steps, find out the appropriate steps for the given input. Input : chemical 68 11 reaction 87 is 21 hard to 53 92 detect 35. In which step are the elements ‘to 92 detect 21’ found in the same order? (1) Sixth (2) Second (3) The given order of elements is not found in any step. (4) Third (5) Fifth 36. What is the position of ‘21’ from the right end in the last step? (1) Tenth (2) Eighth (3) Fifth (4) Fourth (5) Sixth 37. In which step are the elements ‘87 53 11 reaction’ found in the same order? (1) Sixth (2) Fifth (3) Fourth (4) Fourth and Fifth (5) None of these 38. Which element is fifth to the left of the element which is ninth from the left end of the fourth step? (1) 11 (2) 87 (3) 53 (4) reaction (5) chemical 39. Which element is exactly between ‘chemical’ and ‘87’ in the second step of the given arrangement? (1) 53 (2) hard (3) reaction (4) is (5) 68 Directions (40–44) : Study the following information carefully and answer the questions given below : (UIICL A.O. Exam 12.06.2016)

    When a word and number arrangement machine is given an input line of words and numbers, it rearranges them following a particular rule. The following is an illustration of input and rearrangement : (All the numbers are two-digit numbers.) Input : magic 68 23 value heal 44 92 trick fair 36 sale 75 Step I : magic 68 heal 44 92 trick fair 36 sale 75 value 23 Step II : trick 36 magic 68 heal 44 92 fair sale 75 value 23 Step III : trick 36 magic 68 heal 92 fair 75 value 23 sale 44 Step IV : magic 68 trick 36 heal 92 fair 75 value 23 sale 44 Step V : magic 68 trick 36 92 fair value 23 sale 44 heal 75 Step VI : fair 92 magic 68 trick 36 value 23 sale 44 heal 75

    BPRE–466

    INPUT Step VI is the last step of the above arrangement as the intended output of arrangent is obtained. As per the rules followed in the given steps, find the appropriate steps for the given input. Input for the questions : reach 53 gems busy 84 15 usual 97 power 46 78 decent 40. If in Step IV ‘46’ interchanges its position with ‘usual’ and ‘reach’ interchanges its position with ‘53’, then which element will be to the immediate left of ‘reach’ ? (1) usual (2) 46 (3) 15 (4) power (5) decent 41. Which of the following combinations represents the seventh and ninth elements in Step III of the given input from the right end ? (1) ‘97’ and ‘busy’ (2) ‘78’ and ‘busy’ (3) ‘97’ and ‘gems’ (4) ‘84’ and ‘gems’ (5) ‘decent’ and ‘84’ 42. In Step II, ‘53 : 97’ are related in the same way as ‘busy : usual’ in the Step IV. In the same way, ‘reach’ : _______ in the Step V. (1) 15 (2) 97 (3) usual (4) power (5) 46 43. Which element is fourth to the right of one which is eighth from the right end in the Step V ? (1) gems (2) 78 (3) usual (4) 15 (5) power 44. Which element(s) come(s) exactly between ‘97’ and ‘usual’ in the Step III of the given input ? (1) Only ‘78’ (2) Both ‘78’ and ‘decent’ (3) Both ‘84’ and ‘decent’ (4) Only ‘decent’ (5) Both ‘decent’ and ‘15’ Directions (45–49) : Study the following information carefully and answer the questions given below : (GIC Officer Online Exam 15.05.2017)

    A word and number arrangement machine when given an input line of words and numbers rearranges them following a particular rule. The following is an illustration of input and rearrangement. Input : 93 met 64 ape 72 59 18 top fat 36 bun hot Step I : 18 93 met 64 ape 72 59 fat 36 bun hot top Step II : 93 18 met 64 72 59 fat 36 bun hot top ape Step III : 36 93 18 64 72 59 fat bun hot top ape met Step IV : 72 36 93 18 64 59 fat hot top ape met bun Step V : 59 72 36 93 18 64 fat top ape met bun hot Step VI : 64 59 72 36 93 18 top ape met bun hot fat And Step VI is the last step of the above arrangement as the intended output is obtained. As per the rules followed in the given input, find out the appropriate steps for the given input. Input : bin 92 49 air 17 fin log 61 75 mob 22 row 45. In which of the following steps is ‘log 61 row’ found consecutively in the same order? (1) There is no such Step (2) Step V (3) Step II (4) Step IV (5) Step I

    46. How many elements are there between ‘92’ and ‘row’ in Step V? (1) Two (2) Three (3) Five (4) Four (5) More than five 47. Which is the fourth element to the right of the ninth element from the right end in Step VI? (1) air (2) fin (3) row (4) 22 (5) 17 48. In Step I, ‘49’ is related to ‘log’ in a certain way. Following the same pattern, in Step II, ‘61’ is related to ‘22’. In Step IV, following the same pattern to which of the following is ‘fin’ related to? (1) 49 (2) 17 (3) bin (4) log (5) row 49. In Step II, which elements appear exactly between ‘75’ and ‘22’? (1) Both ‘mob’ and ‘fin’ (2)Both ‘log’ and ‘61’ (3) Only ‘mob’ (4)Both ‘mob’ and ‘61’ (5) Only ‘fin’

    SHORT ANSWERS

    NATIONALISED BANKS & IBPS PO/MT/SO EXAMS 1. (4) 6. (2) 11. (4) 16. (5) 21. (3)

    2. (3) 7. (1) 12. (5) 17. (4) 22. (4)

    3. (4) 8. (3) 13. (5) 18. (1) 23. (5)

    4. (3) 9. (1) 14. (4) 19. (5) 24. (2)

    5. (4) 10. (1) 15. (1) 20. (3) 25. (4)

    26. (4)

    27. (3)

    28. (4)

    29. (2)

    30. (1)

    31. (2)

    32. (4)

    33. (2)

    34. (5)

    35. (3)

    36. (5) 41. (4)

    37. (4) 42. (4)

    38. (2) 43. (3)

    39. (3) 44. (2)

    40. (1) 45. (1)

    46. (5)

    47. (4)

    48. (5)

    49. (2)

    50. (4)

    51. (1) 56. (1) 61. (1) 66. (2) 71. (2) 76. (2) 81. (4) 86. (5) 91. (3) 96. (2) 101. (5) 106. (5) 111. (4) 116. (5) 121. (5) 126. (3) 131. (5) 136. (3) 141. (2) 146. (3) 151. (5)

    52. (3) 57. (4) 62. (3) 67. (5) 72. (3) 77. (3) 82. (2) 87. (3) 92. (1) 97. (5) 102. (2) 107. (3) 112. (2) 117. (2) 122. (2) 127. (4) 132. (3) 137. (4) 142. (4) 147. (4) 152. (3)

    53. (2) 58. (3) 63. (3) 68. (1) 73. (4) 78. (5) 83. (4) 88. (5) 93. (3) 98. (5) 103. (4) 108. (1) 113. (1) 118. (4) 123. (5) 128. (1) 133. (1) 138. (2) 143. (1) 148. (1) 153. (1)

    54. (5) 59. (2) 64. (1) 69. (5) 74. (1) 79. (2) 84. (1) 89. (2) 94. (4) 99. (2) 104. (2) 109. (2) 114. (4) 119. (1) 124. (4) 129. (3) 134. (4) 139. (5) 144. (2) 149. (5) 154. (5)

    55. (3) 60. (4) 65. (4) 70. (4) 75. (3) 80. (1) 85. (2) 90. (4) 95. (1) 100. (3) 105. (1) 110. (5) 115. (3) 120. (3) 125. (1) 130. (5) 135. (2) 140. (1) 145. (4) 150. (3) 155. (1)

    BPRE–467

    INPUT

    SBI PO EXAMS

    EXPLANATIONS

    1. (5)

    2. (3)

    3. (4)

    4. (3)

    5. (4)

    6. (3)

    7. (1)

    8. (2)

    9. (3)

    10. (1)

    11. (4)

    12. (2)

    13. (5)

    14. (2)

    15. (4)

    16. (3)

    17. (4)

    18. (1)

    19. (5)

    20. (2)

    21. (4)

    22. (1)

    23. (4)

    24. (5)

    25. (3)

    26. (1)

    27. (3)

    28. (1)

    29. (5)

    30. (2)

    31. (2)

    32. (4)

    33. (5)

    34. (3)

    35. (1)

    36. (2)

    37. (4)

    38. (1)

    39. (2)

    40. (5)

    41. (3)

    42. (4)

    43. (5)

    44. (3)

    45. (2)

    46. (1)

    47. (3)

    48. (2)

    49. (4)

    50. (5)

    51. (3)

    52. (1)

    53. (2)

    54. (4)

    55. (1)

    56. (2)

    57. (5)

    58. (3)

    59. (4)

    60. (5)

    61. (3)

    62. (2)

    63. (1)

    64. (3)

    65. (3)

    66. (5)

    67. (2)

    68. (4)

    69. (1)

    70. (4)

    71. (3)

    72. (5)

    73. (3)

    74. (1)

    75. (5)

    76. (4)

    77. (2)

    78. (5)

    79. (4)

    80. (3)

    81. (1)

    82. (3)

    83. (5)

    84. (2)

    85. (1)

    86. (3)

    RBI GRADE–B/ NABARD GRADE–A OFFICER EXAMS 1. (1)

    2. (4)

    3. (2)

    4. (3)

    5. (3)

    6. (3)

    7. (1)

    8. (4)

    9. (2)

    10. (3)

    11. (2)

    12. (4)

    13. (2)

    14. (1)

    15. (3)

    16. (5)

    17. (2)

    18. (1)

    19. (4)

    20. (3)

    21. (5)

    22. (2)

    23. (1)

    24. (2)

    25. (3)

    26. (1)

    27. (5)

    28. (4)

    29. (5)

    30. (2)

    31. (1)

    32. (3)

    33. (4)

    34. (1)

    35. (5)

    36. (2)

    37. (3)

    38. (4)

    39. (3)

    40. (1)

    41. (1)

    42. (2)

    43. (5)

    44. (4)

    45. (1)

    46. (3)

    47. (2)

    48. (4)

    49. (5)

    50. (5)

    51. (2)

    52. (4)

    53. (1)

    54. (5)

    INSURANCE EXAMS 1. (3)

    2. (2)

    3. (3)

    4. (4)

    5. (1)

    6. (4)

    7. (2)

    8. (4)

    9. (2)

    10. (1)

    11. (3)

    12. (3)

    13. (2)

    14. (1)

    15. (3)

    16. (4)

    17. (2)

    18. (5)

    19. (5)

    20. (2)

    21. (3)

    22. (4)

    23. (2)

    24. (1)

    25. (3)

    26. (5)

    27. (1)

    28. (4)

    29. (2)

    30. (4)

    31. (3)

    32. (4)

    33. (2)

    34. (1)

    35. (2)

    36. (5)

    37. (4)

    38. (1)

    39. (3)

    40. (4)

    41. (1) 46. (2)

    42. (3) 47. (1)

    43. (5) 48. (5)

    44. (2) 49. (3)

    45. (4)

    NATIONALISED BANKS & IBPS PO/MT/SO EXAMS (1-7) : After careful analysis of the given input and various steps of rearrangement it is evident that words and numbers are rearranged alternately. World are arranged in alphabetical order and the numbers are arranged in ascending order. (1-4) : Input : next 57 problem 82 14 trend 02 purchase growth 41 Step I : growth next 57 problem 82 14 trend 02 purchase 41 Step II : growth 02 next 57 problem 82 14 trend purchase 41 Step III : growth 02 next 14 57 problem 82 trend purchase 41 Step IV : growth 02 next 14 problem 57 82 trend purchase 41 Step V : growth 02 next 14 problem 41 57 82 trend purchase Step VI : growth 02 next 14 problem 41 purchase 57 82 trend Step VII : growth 02 next 14 problem 41 purchase 57 trend 82 Step VII is the last step of arrangement. 1. (4) VII 2. (3) Option (3) is the final arrangement. 3. (4) Option (4) is the Step I. 4. (3) The word ‘problem’ is sixth from the left end in Step III. (5-7) : Input : just 14 and value 22 time 05 15 Step I : and just 14 value 22 time 05 15 Step II : and 05 just 14 value 22 time 15 Step III: and 05 just 14 time value 22 15 Step IV: and 05 just 14 time 15 value 22 5. (4) The number ‘14’ is at the fifth position from the right in Step III. 6. (2) Option (2) is the Step II. 7. (1) Four steps are required to complete the arrangement. (8-12) : After careful analysis of the given input and various steps of rearrangement, it is evident that two words are rearrange in each step. The words starting with Vowels are arranged from the left in the reverse alphabetical order and words starting with consonants are arranged from the right in alphabetical order. Input : gem stat ace cast omit fan rate uncut era input Step I : uncut gem stat ace omit fan rate era input cast Step II : uncut omit gem stat ace rate era input fan cast

    BPRE–468

    INPUT Step III : uncut omit input stat ace rate era gem fan cast Step IV : uncut omit input era stat ace rate gem fan cast Step V : uncut omit input era ace stat rate gem fan stat 8. (3) Option (3) is the final arrangement. 9. (1) The word ‘rate’ is at sixth place from the left in the Step III. 10. (1) It is Step II. 11. (4) There is one word between ‘omit’ and ‘era’. Therefore, ‘ace’ would be related to ‘gem’. 12. (5) Step V is the last step. (13-20) : After careful analysis of the given input and various steps of rearrangement, it is evident that the numbes are rearranged in the middle in descending order and words are arranged in alphabetical order from the left and right. The words beginning with vowels are rearranged from the left in alphabetical order and the words beginning with consonants are rearranged from the right in the reverse alphabetical order. (13-16) : Input : unique 84 can 77 open 86 quick 13 base 53 amiss 11 equal 98 start Step I : amiss unique 84 can 77 open 86 13 base 53 11 equal 98 quick start Step II : amiss equal unique 84 77 open 86 13 base 53 11 98 can quick start Step III : amiss equal open unique 84 77 86 13 53 11 98 base can quick start Step IV : amiss equal open unique 98 84 77 86 13 53 11 base can quick start Step V : amiss equal open unique 98 86 84 77 13 53 11 base can quick start Step VI : amiss equal open unique 98 86 84 77 53 13 11 base can quick start 13. (5) None of these 14. (4) 98 would be fifth from the right in step III. 15. (1) Option (1) is the last step. 16. (5) Six steps 17. (4) It is step IV. (18-20) : Step I : (C) arrival 16 44 28 on 66 finish match Step II : (A) arrival on 16 44 28 66 finish match Step III : (E) arrival on 66 16 44 28 finish match Step IV : (D) arrival on 66 44 16 28 finish match Step V : (B) arrival on 66 44 28 16 finish match 18. (1) A is the step II. 19. (5) E is the step III. 20. (3) C is the step I. (21–25) : After careful analysis of the given input and various steps of rearrangement it is evident that words have been rearranged in the reverse alphabetical order and the numbers have been rearranged in descending order. In the first step the word which will appear first in a dictionary is placed at the extreme left position and the highest number is placed at the extreme right position. In the second step, the word which will appear at the second place is placed at the extreme left position

    and the second highest number is placed at the extreme right position. Similar rearrangement is carried on till we get the final output. Input : 49 last zen 16 82 yet can vast 33 aim 87 54 Step I : aim 49 last zen 16 82 yet can vast 33 54 87 Step II : can aim 49 last zen 16 yet vast 33 54 87 82 Step III : last can aim 49 zen 16 yet vast 33 87 82 54 Step IV : vast last can aim zen 16 yet 33 87 82 54 49 Step V : yet vast last can aim zen 16 87 82 54 49 33 Step VI : zen yet vast last can aim 87 82 54 49 33 16 21. (3) Six steps. 22. (4) This is step IV. 23. (5) None of these 24. (2) ‘16’ is at the sixth position from the right. 25. (4) Option (4) is the final arrangement. (26–30) : There are six numbers and six words in the input. The three numbers are placed in the beginning and the remaining three numbers are placed in the last. The numbers are rearranged in ascending order. The six words are rearranged in alphabetical order in the middle. Input : 67 hot sun 19 best 83 ice 49 ace 77 cut 37 Step I : 19 67 hot sun best ice 49 ace 77 cut 37 83 Step II : 19 37 67 hot sun best ice 49 ace cut 77 83 Step III : 19 37 49 hot sun best ice ace cut 67 77 83 Step IV : 19 37 49 ace hot sun best ice cut 67 77 83 Step V : 19 37 49 ace best hot sun ice cut 67 77 83 Step VI : 19 37 49 ace best cut hot sun ice 67 77 83 Step VII :19 37 49 ace best cut hot ice sun 67 77 83 26. (4) Seven steps are needed to complete the arrangement. 27. (3) It is Step V. 28. (4) Option (4) is Step I. 29. (2) Option (2) is the final arrangement. 30. (1) In Step IV ‘sun’ would be seventh from the right. (31–35) : From the given input and various steps of rearrangement it is evident that in the first step one number is rearranged and in the next step one word is rearranged. These two steps are continued alternately till all the numbers get arranged in ascending order and the words get arranged in reverse alphabetical order. 31. (2) Input : 86 open shut door 31 49 always 45 Step I : 31 86 open shut door 49 always 45 Step II : 31 shut 86 open door 49 always 45 Step III : 31 shut 45 86 open door 49 always Step IV : 31 shut 45 open 86 door 49 always Step V : 31 shut 45 open 49 86 door always Step VI : 31 shut 45 open 49 door 86 always 32. (4) It is not possible to determine the Input from any given step. 33. (2) Step II :18 win 71 34 now if victory 61 Step III : 18 win 34 71 now if victory 61 Step IV : 18 win 34 victory 71 now if 61 Step V : 18 win 34 victory 61 71 now if Step VI : 18 win 34 victory 61 now 71 if 34. (5) Input : where 47 59 12 are they going 39 Step I : 12 where 47 59 are they going 39 Step II : 12 where 39 47 59 are they going Step III : 12 where 39 they 47 59 are going Step IV : 12 where 39 they 47 going 59 are

    BPRE–469

    INPUT 35. (3) Step II :33 store 81 75 full of goods 52 Step III : 33 store 52 81 75 full of goods Step IV : 33 store 52 of 81 75 full goods Step V : 33 store 52 of 75 81 full goods Step VI : 33 store 52 of 75 goods 81 full (36–39) : After careful analysis of the given input and various steps of rearrangement it is evident that in each step one number and one word are rearranged. The lowest number is moved to the left end while the word which will come first in alphabetical order is moved to the right end. In the second step the second lowest number is moved to the left end and the word which will come second in the alphabetical order is moved to the right end. The procedure is continued till all the numbers and words get rearranged accordingly. Input : 84 why sit 14 32 not best ink feet 51 27 vain 68 92 Step Step Step Step Step Step Step

    I : 14 84 why sit 32 not ink feet 51 27 vain 68 92 best II : 27 14 84 why sit 32 not ink 51 vain 68 92 best feet III : 32 27 14 84 why sit not 51 vain 68 92 best feet ink IV : 51 32 27 14 84 why sit vain 68 92 best feet ink not V : 68 51 32 27 14 84 why vain 92 best feet ink not sit VI : 84 68 51 32 27 14 why 92 best feet ink not sit vain VII : 92 84 68 51 32 27 14 best feet ink not sit vain why

    36. (5) There is no such step. 37. (4) The word ‘best’ is at the fifth position from the right in Step V. 38. (2) There are three elements – 27, 14 and best – between ‘feet’ and ‘32’ in the last step of the output. 39. (3) The word ‘why’ is sixth from the left and ninth from the right in the Step IV. (40-42) : After careful analysis of the given input and various steps of rearrangement it is evident that words and numbers are rearranged from right to left. In the first step one word is rearranged and in the second step one number is rearranged. The words are rearranged in alphabetical order and the numbers are arranged in ascending order from right to left. Input : 23 you 13 wake 81 me 43 before 72 go 34 up Step I : 23 you 13 wake 81 me 43 72 go 34 up before Step II : 23 you wake 81 me 43 72 go 34 up 13 before Step III : 23 you wake 81 me 43 72 34 up go 13 before Step IV : you wake 81 me 43 72 34 up 23 go 13 before Step V : you wake 81 43 72 34 up me 23 go 13 before Step VI : you wake 81 43 72 up 34 me 23 go 13 before Step VII : you wake 81 72 43 up 34 me 23 go 13 before Step VIII : you 81 72 wake 43 up 34 me 23 go 13 before Step IX : 81 you 72 wake 43 up 34 me 23 go 13 before 40. (1) me 41. (4) It is step V. 42. (4) Option (4) is step II. (43-47) : After careful analysis of the given input and various steps of rearrangement it is evident that in each step one word and a number are rearranged. In the

    first step the word which comes the first in English alphabetical series and the highest number move to the extreme left positions. In the second step, the word which comes to second in English alphabetical series and the second highest number are moved to the extreme left positions. The same procedure is continued till all the words are arranged in the reverse alphabetical order from left to right and all the numbers get arranged in ascending order. Input % money 48 24 18 wanted for investment 65 90 lock credit 32 Step I % credit 90 money 48 24 18 wanted for investment 65 lock 32 Step II % for 65 credit 90 money 48 24 18 wanted investment lock 32 Step III % investment 48 for 65 credit 90 money 24 18 wanted lock 32 Step IV % lock 32 investment 48 for 65 credit 90 money 24 18 wanted Step V % money 24 lock 32 investment 48 for 65 credit 90 18 wanted Step VI % wanted 18 money 24 lock 32 investment 48 for 65 credit 90 And, Step VI is the last step for the given input. 43. (3) This is step IV. 44. (2) Fourth from left in Step IV ⇒ 48 45. (1) Fifth element from the left in the Step V ⇒ investment Second to the right of ‘investment’ ⇒ for 46. (5) There are five elements (65, credit, 90, money, 24) between ‘for’ and ‘18’ in the Step III. 47. (4) Fourth from the right. (48-52) : After careful analysis of the given input and various steps of rearrangement, it is evident that in the first step the highest number is placed at the extreme left position and in the second step the word which comes first in the alphabetical order is placed at the extreme right position. In the next step the second highest number is placed at the second position from the left. After that step the word which comes second in the alphabetical order is placed at the extreme right position. These two steps are continued alternately till all the numbers get arranged in the descending order from the left and all the words get arranged in alphabetical order after the numbers. Input Step I Step II Step III Step IV Step V Step VI Step VII Step VIII Step IX Step X Step XI

    % % % % % % % % % % % %

    class 25 war 15 race 73 heap 58 just 88 take 38 88 class 25 war 15 race 73 heap 58 just take 38 88 25 war 15 race 73 heap 58 just take 38 class 88 73 25 war 15 race heap 58 just take 38 class 88 73 25 war 15 race 58 just take 38 class heap 88 73 58 25 war 15 race just take 38 class heap 88 73 58 25 war 15 race take 38 class heap just 88 73 58 38 25 war 15 race take class heap just 88 73 58 38 25 war 15 take class heap just race 88 73 58 38 25 15 war take class heap just race 88 73 58 38 25 15 war clas heap just race take 88 73 58 38 25 15 class heap just race take war

    And, Step XI is the last step.

    BPRE–470

    INPUT 48. (5) The word ‘war’ is sixth from the left end in step VIII. 49. (2) ‘25’ is the ninth from the right in Step VI. 50. (4) ‘15’ is seventh from the right end in Step IX. 51. (1) Eleven Steps were required to complete the arrangement. 52. (3) Option (3) is the Step X. (53-58) : After careful analysis of the input and various steps of rearrangement, it is evident that two elements – one word and one number – are rearranged in each step. The word which comes in the last as per dictionary order is moved to the extreme left end and the lowest number is moved to the extreme right end, in the first step. In the next step, the word which comes second last as per dictionary order is moved to the extreme left end and the second lowest number is moved to the extreme right position. The same procedure is continued till all the words get arranged in dictionary order from left to right and all the numbers get rearranged in ascending order after the words. Input : 63 45 fog 13 88 can ink 76 51 you hang 27 air end Step I : you 63 45 fog 88 can ink 76 51 hang 27 air end 13 Step II : ink you 63 45 fog 88 can 76 51 hang air end 13 27 Step III : hang ink you 63 fog 88 can 76 51 air end 13 27 45 Step IV : fog hang ink you 63 88 can 76 air end 13 27 45 51 Step V : end fog hang ink you 88 can 76 air 13 27 45 51 63 Step VI : can end fog hang ink you 88 air 13 27 45 51 63 76 Step VII : air can end fog hang ink you 13 27 45 51 63 76 88 53. (2) Seven steps are required to complete the arrangement. 54. (5) There are seven elements (you, 88, can, 76, air, 13, 27) between ‘ink’ and ‘45’ in Step V. 55. (3) This is Step IV. 56. (1) Third to the right of fifth from the left end means eighth from the left or seventh from the right as there are 14 elements. Eighth from the left in Step III ⇒ 76 57. (4) The element ‘can’ is seventh from the left end in the Step II. The element ‘can’ is eighth from the right end in the step II. 58. (3) Option (3) is Step V. (59-62) : After careful analysis of the given input and various steps of rearrangement, it is evident that in each step four elements (two words and two numbers) are rearranged. In the first step the lowest number is placed at the extreme left position while the highest number is placed at the extreme right position. Again, the word which comes first in the dictionary order is placed at the second position from left and the word which comes last is placed at the second position from the right. In the second step, the second lowest number is placed at the second

    position from the left and the second highest number is placed at the second position from the right. Again, the word which comes second in the dictionary order is placed at the fourth position from the left and the word which comes second last in the dictionary order is placed at the fourth position from the right. The same procedure is continued till all the words get arranged in the dictionary order after four numbers which are in ascending order. The remaining four numbers also get arranged in ascending order after the eight words. Input : water is required 12 41 65 for all 27 33 89 to 75 90 survive life Step I % 12 all is required 41 65 for 27 33 89 to 75 survive life water 90 Step II % 12 27 all for is required 41 65 33 75 survive life to water 89 90 Step III % 12 27 33 all for is required 41 65 life survive to water 75 89 90 Step IV % 12 27 33 41 all for is life required survive to water 65 75 89 90 59. (2) Fourth to the left of the seventh element from the right means eleventh from the right or sixth from the left, i.e, ‘is’. 60. (4) There are seven elements (for, is, life, required, survive, to, water) between ‘all’ and ‘65’ in Step IV. 61. (1) Eleventh 62. (3) Seventh from the left (63–68) : After careful analysis of the input and various steps of rearrangement, it is evident that in each two elements (one number and one word) are rearranged. The word which comes first as per the order in a dictionary is moved to the extreme right position while the highest number is moved to the extreme left position in the step I. In the next step, the word which comes second as per the order in a dictionary is placed at the extreme right position and the second highest number is placed at the extreme left position. The same procedure is continued till all the words get rearranged in dictionary order from the left to the right after the numbers and the numbers get rearranged in ascending order from the left to the right. Input : find 64 belt 28 54 lamp 17 give flat 69 real 95 Step I : 95 find 64 28 54 lamp 17 give flat 69 real belt Step II : 69 95 64 28 54 lamp 17 give flat real belt find Step III : 64 69 95 28 54 lamp 17 give real belt find flat Step IV : 54 64 69 95 28 lamp 17 real belt find flat give Step V : 28 54 64 69 95 17 real belt find flat give lamp Step VI : 17 28 54 64 69 95 belt find flat give lamp real 63. (3) Option (3) is the Step III. 64. (1) The element “lamp” is at the sixth position from the left in the Step IV. Its position from the right end is seventh. 65. (4) There are five elements (54, lamp, 17, give, flat) between “28” and “69” as they appear in the Step I. 66. (2) Element “real” would appear at the sixth position from the right in the Step V. 67. (5) Second to the left of the fifth element from the right means seventh from the right end or sixth from the left end as there are twelve elements. Seventh from the right end in the Step III ⇒ “lamp” 68. (1) This is the Step V.

    BPRE–471

    INPUT (69-72) : After careful analysis of the input and various steps of rearrangement, it is evident that in each step two elements (one word and one number) are rearranged. In the first step the lowest number moves to the extreme left position and the word which comes first in the alphabetical order moves to the extreme right position. In the second step, the second lowest number moves to the extreme left position while the word which comes second in the alphabetical order moves to the extreme right position. The same procedure is continued till all the numbers get rearranged in descending order and all the words get rearranged in alphabetical order after the numbers. Input Step I Step II Step III Step IV Step V Step VI Step VII

    % % % % % % % %

    83 why sat 14 32 no be ink feet 50 27 vain 67 92 14 83 why sat 32 no ink feet 50 27 vain 67 92 be 27 14 83 why sat 32 no ink 50 vain 67 92 be feet 32 27 14 83 why sat no 50 vain 67 92 be feet ink 50 32 27 14 83 why sat vain 67 92 be feet ink no 67 50 32 27 14 83 why vain 92 be feet ink no sat 83 67 50 32 27 14 why 92 be feet ink no sat vain 92 83 67 50 32 27 14 be feet ink no sat vain why

    69. (5) This is the Step III. 70. (4) The element ‘be’ is at the fifth position from the right in the Step V. 71. (2) There are three elements (27, 14, be) between ‘32’ and ‘feet’ in the last Step. 72. (3) The element ‘why’ is at the sixth position from the left in the Step IV. (73–76) : After careful analysis of the given input and various steps of rerrangement, it is evident that rearrangement is based on the odd and the even numbers as well as the words beginning with vowel and consonant. In the first step the word which comes last in the alphabetical order and beginning with a vowel moves to the extreme left position and the lowest even number moves to the extreme right position. In the second step the word which comes second last in the alphabetical order and beginning with a vowel moves to the extreme left position while the second lowest even number moves to the extreme right position. In the third step the word which comes first in the alphabetical order and beginning with a vowel moves to the extreme left position and the highest even number moves to the extreme right position. The process of rearrangement changes from the Step IV onwards. In the fourth step the lowest odd number moves to the extreme left position while the word beginning with a consonant and which comes last in the alphabetical order moves to the extreme right position. In the fifth step the second lowest odd number moves to the extreme left position and the word beginning with a consonant and which comes at the second last position in the alphabetical order moves to the extreme right position. In the sixth and last step the highest odd number moves to the extreme left position and the word beginning with a consonant and which comes first in the alphabetical order moves to the extreme right position. Input % adverb 59 36 salient 81 idioms bakery 14 launch 47 umpire 62 Step I : umpire adverb 59 36 salient 81 idioms bakery launch 47 62 14

    Step II : idioms umpire adverb 59 salient 81 bakery launch 47 62 14 36 Step III : adverb idioms umpire 59 salient 81 bakery launch 47 14 36 62 Step IV : 47 adverb idioms umpire 59 81 bakery launch 14 36 62 salient Step V : 59 47 adverb idioms umpire 81 bakery 14 36 62 salient launch Step VI : 81 59 47 adverb idioms umpire 14 36 62 salient launch bakery 73. (4) The elements ‘81’ and ‘salient’ come between ‘59’ and ‘bakery’ in the Step II. 74. (1) First two elements of Step VI ⇒ 81, 59 The last two elements of Step VI ⇒ launch, bakery 75. (3) In the Step III, third to the left of ‘47’ is ‘81’. But ‘81’ interchanges positions with ‘idioms’. Therefore, ‘idioms’ would be third to the left of ‘47’ in the Step III. 76. (2) The elements ‘bakery launch 47 14’ are found in the same order in the Step III. (77–81) : After careful analysis of the given input and various steps of rearrangement it is evident that only words are rearranged till Step III. From Step IV and onwards only numbers are rearranged. Two elements are rearranged in each step. In Step I, the word beginning with a vowel and which comes first in the alphabetical order moves to the extreme left position while the word beginning with a consonant and which comes last in the alphabetical order moves to the extreme right position. In Step II, the word beginning with a vowel and which comes second in the alphabetical order moves to the extreme left position while the word beginning with a consonant and which comes second last in the alphabetical order moves to the extreme right position. In Step III, word beginning with a vowel and which comes last in the alphabetical order moves to the extreme left position while the word beginning with a consonant and which comes first in the alphabetical order moves to the extreme right position. In Step IV, the highest odd number moves to the extreme left position while the lowest even number moves to the extreme right position. In Step V, the second highest odd number moves to the extreme left position and the second lowest even number moves to the extreme right position. In Step VI, the lowest odd number moves to the extreme left position and the highest even number moves to the extreme right position. Input % offense 71 vaccine 12 39 garage absorb 86 earlier 25 malign 64 Step I : absorb offense 71 12 39 garage 86 earlier 25 malign 64 vaccine Step II : earlier absorb offense 71 12 39 garage 86 25 64 vaccine malign Step III : offense earlier absorb 71 12 39 86 25 64 vaccine malign garage Step IV : 71 offense earlier absorb 39 86 25 64 vaccine malign garage 12 Step V : 39 71 offense earlier absorb 86 25 vaccine malign garage 12 64 Step VI : 25 39 71 offense earlier absorb vaccine malign garage 12 64 86

    BPRE–472

    INPUT 77. (3) In Step II, seventh to the left of 25 ⇒ absorb But ‘absorb’ interchanges position with ‘86’. 78. (5) First two elements in Step IV ⇒ 71, offense Last two elements in Step IV ⇒ garage, 12 79. (2) The elements ‘absorb 86 25 vaccine’ are found in the same order in the Step V. 80. (1) The elements ‘64’ and ‘vaccine’ come exactly between ‘25’ and ‘malign’ in the Step III. 81. (4) 7th from the left end in the Step VI ⇒ vaccine 3rd to the left of ‘vaccine’ ⇒ offense (82–86) : After careful analysis of the given input and various steps of rearrangement, it is evident that two elements – one word and one number – are rearranged in each step. In the first step, the highest number is moved to the extreme left position after adding one to it and the word which comes first in the dictionary order is moved to the second position from the left after deleting the last letter and adding ‘ate’ to it. In the second step, the second highest number is moved to the extreme left position after adding one to it while the word which comes second in the dictionary order is moved to the second position after deleting the last letter and adding ‘ate’ to it. The same procedure is continued till all the letters and numbers get rearranged. Input : 41 indice aspire 73 96 probe marine 28 59 chrome Step I : 97 aspirate 41 indice 73 probe marine 28 59 chrome Step II : 74 chromate 97 aspirate 41 indice probe marine 28 59 Step III : 60 indicate 74 chromate 97 aspirate 41 probe marine 28 Step IV : 42 marinate 60 indicate 74 chromate 97 aspirate probe 28 Step V : 29 probate 42 marinate 60 indicate 74 chromate 97 aspirate 82. (2) The elements ‘60 indicate 74 chromate 97’ are found in the same order in the Step IV. 83. (4) The element ‘indice’ is fourth to the right of ‘chromate’ in the Step II. 84. (1) Step III ..... 74

    chromate 97

    aspirate 41 ......

    85. (2) 8th element from the right in the Step II ⇒ 97 3rd to the right of ‘97’ in the Step II ⇒ indice 86. (5) Step III

    60 indicate

    74 chromate 97 aspirate 41 probe

    marine 28

    Now, ‘indicate’ would be to the immediate right of ‘41’. 87. (3) 42 + 3 = 45 ⇒

    45 = 15 3

    60 + 3 = 63 ⇒

    63 = 21 3

    74 + 3 = 77 ⇒

    77 = 25.666 3

    97 + 3 = 100 ⇒

    100 = 33.333 3

    31 = 10.333 3 (88–92) : After careful analysis of the given input and various steps of rearrangement, it is evident that two elements – one word and one number – are rearranged in each step. In the first step, the word which comes first in the dictionary order is placed at the extreme left position after adding ‘s’ to it while the lowest number is placed at the extreme right position after deducting ‘1’ from it. In the second step, the second lowest number is placed at the extreme left position after deducting ‘1’ from it while the word which comes second in the dictionary order is placed at the extreme right position after adding ‘s’ to it. These two steps are continued alternately to rearrange all the words and the numbers. Input : 84 layer 97 packet 51 damage narrow 75 32 table Step I : damages 84 layer 97 packet 51 narrow 75 table 31 Step II : 50 damages 84 97 packet narrow 75 table 31 layers Step III : narrows 50 damages 84 97 packet table 31 layers 74 Step IV : 83 narrows 50 damages 97 table 31 layers 74 packets Step V : tables 83 narrows 50 damages 31 layers 74 packets 96 88. (5) 4th element from the left end in Step V ⇒ 50 2nd element from the right end in Step II ⇒ 31 Required sum = 50 + 31 = 81 89. (2) In Step III, ‘damages’ is second to the left of ‘97’. In Step I, ‘narrow’ is second to the left of ‘table’. In Step IV, ‘31’ is second to the left of ‘74’. 90. (4) 5th to the left of ‘layers’ in the last Step, i.e., Step V ⇒ 83 91. (3) Step IV 28 + 3 = 31 ⇒

    83 narrows

    50

    damages 97 table

    31 layers

    74 packets There are three elements – damages, 97, table – between ‘50’ and ‘31’. 92. (1) 3rd to the left of the 9th element from the left end means 6th from the left end. 6th from the left end in Step II ⇒ narrow (93–97) : After careful analysis of the given input and various steps of rearrangement it is evident that in each step two elements (one word and one number) are rearranged. In the first step, the word which comes last in the dictionary order is moved to the extreme left position while the lowest number is moved to the second position from the left. In the second step, the word which comes second last in the dictionary order is moved to the extreme right position while the second lowest number is moved to the second position from the right. These two steps are continued alternately till all the numbers and words get rearranged. Input : band 85 zen 54 den fit 25 37 home 41 sun 73 Step I : zen 25 band 85 54 den fit 37 home 41 sun 73 Step II : zen 25 band 85 54 den fit home 41 73 37 sun

    BPRE–473

    INPUT Step Step Step Step 93.

    III : home 41 zen 25 band 85 54 den fit 73 37 sun IV : home 41 zen 25 band 85 den 73 37 sun 54 fit V : den 73 home 41 zen 25 band 85 37 sun 54 fit VI : den 73 home 41 zen 25 37 sun 54 fit 85 band (3) In Step IV, there are two elements – 85, den – between ‘band’ and ‘73’. 94. (4) 3rd to the left of 9th from the left means 6th from the left. 6th from the left in the Step III ⇒ 85 95. (1) In Step II, ‘41, 73, 37’ are found consecutively in the same order. 96. (2) In Step V, ‘home’ is third to the left of ‘25’. Similarly, in Step VI, ‘37’ is third to the left of ‘fit’. In Step I, ‘band’ is third to the left of ‘den’. 97. (5) First two elements in Step V ⇒ den, 73 Last two elements in Step V ⇒ 54, fit 98. (5) There is no such step. 99. (2) From the left (3 + 5) in step V ⇒ ‘take’ will be the 8th element 100. (3) In step II, the seventh to the right and eighth to the left will be 16 (16 × 16)2 = 65536 101. (5) The position of ‘wait’ will be sixth to the left in step III. 102. (2) ‘take’ will be in the middle of ‘store’ and ‘29’ in step I. (103–107) : After careful analysis of the given input and various steps of rearrangement, it is evident that two elements (one word and one number) are rearranged in every step. In the first step, the word which comes first in the dictionary order, is moved to the extreme left position while the highest number is moved to the extreme right position. In the second step, the word which comes second in the dictionary order is moved to the extreme right position while the second highest number is moved to the extreme left position. These two steps are continued alternately till all the words and numbers get rearranged. Input : 15 role air 96 63 born with 77 like 39 some 52 Step I : air 15 role 63 born with 77 like 39 some 52 96 Step II : 77 air 15 role 63 with like 39 some 52 96 born Step III : like 77 air 15 role with 39 some 52 96 born 63 Step IV : 52 like 77 air 15 with 39 some 96 born 63 role Step V : some 52 like 77 air 15 with 96 born 63 role 39 Step VI : 15 some 52 like 77 air 96 born 63 role 39 with 103. (4) Step IV 52 like 77 air 15

    with

    39 some 96

    born

    63 role 104. (2) In Step VI, the elements ‘15 some 52’ are found consecutively in the same order. 105. (1) 2nd element to the left of the 9th element from the left means 7th from the left end. 7th from the left end in Step II ⇒ like 106. (5) In Step I, ‘some is third to the right of ‘77’. In Step V, ‘role’ is third to the right of ‘96’ Similarly, in Step IV, ‘born’ is third to the right of ‘39’.

    107. (3) Step II

    77

    air

    15

    role 63 with like 39 some 52

    96 born (108–111) : After careful analysis of the given input and various steps of rearrangement, it is evident that two elements (one word and one number) are rearranged in each step. In the first step, the word which comes last in the dictionary order is moved to the extreme left position after replacing the second, the third and the fourth letters with ‘O’ and the lowest number is moved to the second position from the left after adding ‘one’ to it. In the second step, the word which comes second last in the dictionary order, is moved to the extreme left position after replacing the second, the third and the fourth letters with ‘O’ and the second lowest number is moved to the second position from the left end after adding ‘one’ to it. The same procedure is continued till all the words and numbers get arranged. Input % 84 springs 97 damping 51 telomes paddles 75 32 ballots Step I : tomes 33 84 springs 97 damping 51 paddles 75 ballots Step II : songs 52 tomes 33 84 97 damping paddles 75 ballots Step III : poles 76 songs 52 tomes 33 84 97 damping ballots Step IV : doing 85 poles 76 songs 52 tomes 33 97 ballots Step V : boots 98 doing 85 poles 76 songs 52 tomes 33 108. (1) Step IV doing 85 poles 76 songs 52 tomes 33 97 ballots 4th from the right end

    109. (2) Last Step (Step V) boots 98 doing 85 poles 76 songs 52 tomes 33 4th to the left

    110. (5) Fourth element from the left in the Step V ⇒ 85 Fifth element from the right end in the Step IV ⇒ 52 Required sum = 85 + 52 = 137 111. (4) ‘damping’ is to the immediate right of ‘97’ in the Step III. ‘75’ is to the immediate right of ‘paddles’ in the Step I. ‘85’ is to the immediate right of ‘doing’ in the Step IV. (112-116) : After careful analysis of the given input and various steps of rearrangement it is evident that in each step two elements (one word and one number) are rearranged. In the first step the highest number moves to the extreme left position and its digits are interchanged while the word which contains least number of letters, moves to the extreme right position. In the second step, the second highest number moves to the extreme left position and its digits are interchanged while the word which contains the second lowest number of letters moves to the extreme right position. The same procedure

    BPRE–474

    INPUT is continued till all the numbers and words get rearranged following the rule mentioned above. Input : tyre 71 toughen 59 tip 82 13 thanks to 68 table 46 Step I : 28 tyre 71 toughen 59 tip 13 thanks 68 table 46 to Step II : 17 28 tyre toughen 59 13 thanks 68 table 46 to tip Step III : 86 17 28 toughen 59 13 thanks table 46 to tip tyre Step IV : 95 86 17 28 toughen 13 thanks 46 to tip tyre table Step V : 64 95 86 17 28 toughen 13 to tip tyre table thanks Step VI : 31 64 95 86 17 28 to tip tyre table thanks toughen 112. (2) 31 + 1 = 32; 64 – 2 = 62; 95 + 1 = 96; 86 – 2 = 84; 17 + 1 = 18; 28 – 2 = 26; The numbers 32, 84 and 96 are multiples of 4. 113. (1) Step III : 86 17 28 tough 59 13 thanks table 46 to tip tyre

    ‘28’ will be to the immediate left of ‘46’. 114. (4) Second last step means Step V : 3rd element from the left in Step V ⇒ 86 9th element from the left in Step V ⇒ tip 115. (3) The element ‘13’ comes exactly between ‘59’ and ‘thanks’ in Step II. 116. (5) 8th from the right end in Step IV ⇒ toughen Third to the right of ‘toughen’ ⇒ 46 (117–121) : After careful analysis of the given Input and various steps of rearrangement, it is evident that two numbers are rearranged in each step. In the first step the lowest and the second lowest numbers are shifted to the first and the second positions respectively from the left. In the second step, the highest and the second highest numbers are shifted to the first and the second positions respectively from the left. These two steps are continued alternately to rearrange all the numbers. Input 26 69 13 82 55 21 71 34 93 47 13 21 26 69 82 55 71 34 93 47 Step I Step II 93 82 13 21 26 69 55 71 34 47 Step III 26 34 93 82 13 21 69 55 71 47 Step IV 71 69 26 34 93 82 13 21 55 47 Step V 47 55 71 69 26 34 93 82 13 21 117. (2) 4th element to the left of the 7th element from the left means 3rd element from the left. 3rd element from the left in the Step III ⇒ 93 118. (4) 5th element from the right end in the Step V ⇒ 34 2nd element from the left end in the Step II ⇒ 82 Required answer = 82 – 34 = 48 119. (1) Step II 93 82 13 21

    26 69 55

    71 34 47

    120. (3) In Step I, ‘21’ is second to the left of ‘69’. In Step IV, ‘26’ is second to the left of ‘93’. In Step V, ‘47’ is second to the left of ‘71’.

    121. (5) Step IV 71 69 26 34 93 82 13 21 55 47 There are seven elements to the right of ‘26’ in the Step IV. (122–126) : After careful analysis of the given input and various steps of rearrangement, it is evident that one element is rearranged in each step. In the first step, the lowest number is placed at the extreme left position and in the second step the word which comes in the last as per dictionary is placed at the second position from the left. In the third step, the second lowest number is placed at the third position from the left and in the fourth step, the word which comes at the second last as per dictionary is placed at the fourth position from the left. These two steps are continued till all the numbers and words get rearranged. 122. (2) Input : kind 12 96 heart water 59 42 yes Step I : 12 kind 96 heart water 59 42 yes Step II : 12 yes kind 96 heart water 59 42 Step III : 12 yes 42 kind 96 heart water 59 Step IV : 12 yes 42 water kind 96 heart 59 Step V : 12 yes 42 water 59 kind 96 heart 123. (5) Input : jungle 43 mode 25 basket 39 target 19 Step I : 19 jungle 43 mode 25 basket 39 target Step II : 19 target jungle 43 mode 25 basket 39 Step III : 19 target 25 jungle 43 mode basket 39 Step IV : 19 target 25 mode jungle 43 basket 39 Step V : 19 target 25 mode 39 jungle 43 basket 124. (4) It is not possible to determine the Input from given step. 125. (1) Step II : 24 year 56 43 last part 64 over Step III : 24 year 43 56 last part 64 over Step IV : 24 year 43 part 56 last 64 over Step V : 24 year 43 part 56 over last 64 Step VI : 24 year 43 part 56 over 64 last Four more steps would be required to complete the rearragnement. 126. (3) Input : 32 station 46 81 73 march go for Step I : 32 station 46 march 81 73 go for Step II : 32 station 46 march 73 81 go for Step III : 32 station 46 march 73 go 81 for (127–130) : After careful analysis of the given input and various steps of rearrangement, it is evident that two elements (one word and one number) are rearranged in each step. In the first step, the word which comes last as per the dictionary order is placed at the extreme left end after replacing each letter with the immediate next letter in the English alphabetical series while the highest number is placed at the second position from the left after multiplying it with two. In the second step, the word which comes second last as per the dictionary order is placed at the third position from the left end after replacing each letter with the immediate next letter in the English alphabetical series while the second highest number is placed

    BPRE–475

    INPUT at the fourth position from the left end after multiplying it with two. The same procedure is continued to arrange all the words and numbers. Input

    dear

    fake

    mane

    62

    34

    55

    40

    hate

    Step I

    nbof

    124

    dear

    fake

    34

    55

    40

    hate

    Step II

    nbof

    124

    ibuf

    110

    dear

    fake

    34

    40

    Step III

    nbof

    124

    ibuf

    110

    gblf

    80

    dear

    34

    Step IV

    nbof

    124

    ibuf

    110

    gblf

    80

    efbs

    68

    127. (4) 5th number of the left end in Step I ⇒ 34 5th number of the right end in Step III ⇒ 110 Required sum = 34 + 110 = 144 128. (1) In the Step III, four elements are there between ‘124’ and ‘dear’ 129. (3) In Step IV, ‘ibuf’ is sixth from the right end. 130. (5) 2nd number from the left end in Step II ⇒ 124 2nd number from the right end in Step II ⇒ 34 Required difference = 124 – 34 = 90 (131–135) : After careful analysis of the given input and various step of rearrangement, it is evident that two elements (one word and one number) are rearranged in each step. The word which comes last in alphabetical order and the lowest number are placed at the first and the second positions from the left in the first step. In the second step, the word which comes second last in alphabetical order and the second lowest number are placed at the first and the second positions from the right. These two steps are repeated alternately to get the output. Input : ban 85 zen 54 den fit 25 37 home 41 sun 73 Step I : zen 25 ban 85 54 den fit 37 home 41 sun 73 Step II : zen 25 ban 85 54 den fit home 41 73 37 sun Step III : home 41 zen 25 ban 85 54 den fit 73 37 sun Step IV : home 41 zen 25 ban 85 den 73 37 sun 54 fit Step V : den 73 home 41 zen 25 ban 85 37 sun 54 fit Step VI : den 73 home 41 zen 25 37 sun 54 fit 85 ban 131. (5) In the Step III, ‘ban’ is fifth from the left end. 132. (3) In Step IV, there are two elements – 85 and den – between ‘ban’ and ‘73’. 133. (1) 3rd to the left of 9th element from the left means 6th from the left end. In the Step III, 6th from the left end ⇒ 85 134. (4) In the Step III, ‘54 den fit 73 37’ found consecutively in the same order. 135. (2) In Step V ‘home’ is third to the left of ‘25’. ‘37’ is third to the left of ‘fit’. In Step I ‘ban’ is third to the left of ‘den’. (136–140) : After careful analysis of the given input and various steps of rearrangement, it is evident that two elements (one word and one number) are rearranged in each step. In the first step, the word beginning with a consonant which comes last in the alphabetical order, is placed at the extreme left position and the number with highest sum of digits is placed at the extreme right position. In the second step, the word beginning with a consonant and which comes second last in the alphabetical order is placed

    at the second position from the left end and the number with second highest sum of digits is placed at the extreme right position. The same procedure is continued and the words beginning with a vowel are arranged in the alphabetical order after the words beginning with a consonant. Input % inspire 55 jump 86 view 22 74 angle fight 66 ride 14s Step I : view inspire 55 jump 22 74 angle fight 66 ride 14 86 Step II : view ride inspire 55 jump 22 74 angle fight 14 86 66 Step III : view ride jump inspire 55 22 angle fight 14 86 66 74 Step IV : view ride jump fight inspire 22 angle 14 86 66 74 55 Step V : view ride jump fight angle inspire 22 86 66 74 55 14 Step VI : view ride jump fight angle inspire 86 66 74 55 14 22 136. (3) Six steps would be required to complete the rearrangement. 137. (4) 7th from the left end in Step IV ⇒ angle 138. (2) This is Step III. 139. (5) Step VI view ride

    jump

    fight angle inspire

    angle fight inspire jump ride

    view

    140. (1) Step II view ride inspire 55 jump 22 74 angle fight 14 86 66 (141–145) : ˆ Step I ⇒ Words are arranged according to the alphabetical order, odd numbers in descending order and even numbers in ascending order alternatively. ˆ Step II ⇒ Words then numbers are arranged alternatively. Words are in alphabetical order. Even numbers in ascending order and odd numbers in descending order alternatively. ˆ Step III ⇒ Numbers are arranged first in such a way, even numbers are in ascending order and odd numbers are in descending order. Words are in alphabetical order ˆ Step IV ⇒ Numbers then words are arranged alternatively as in the same order from step III. Input

    Page

    233

    Scale

    223

    Water

    322

    266

    Model

    626

    Step-I Model Page

    Scale

    Water

    233

    266

    223

    322

    626

    Step-II Model

    266

    Page

    233

    Scale

    322

    Water

    223

    626

    Step-III

    266

    322

    626

    233

    223

    Model

    Page

    Scale

    Water

    Step-IV

    266

    Model

    322

    Page

    626

    Scale

    233

    Water

    223

    141. (2) Element at the left end in the step III ⇒ 266 Element at the right end in the step IV ⇒ 223 Required difference = 266 – 223 = 43 142. (4) Fourth to the right of “233” in step II ⇒ 223 143. (1) Fifth to the left of “scale” in step IV ⇒ 266 144. (2) Step I model 626

    BPRE–476

    page

    scale water

    233

    226 223

    322

    INPUT 145. (4) Step III 266 322 626 (146–150) : Input

    English

    Clerk

    Scale

    6712

    6127

    6271

    6147

    Office

    6714

    Step - I

    Scale

    English

    Clerk

    6712

    6271

    6147

    Office

    6714

    6127

    Step - II Step - III

    Scale Scale

    Office Office

    English English

    Clerk Clerk

    6712 6714

    6271 6712

    6714 6271

    6147 6147

    6127 6127

    Step - IV

    Scale

    6714

    Office

    6712

    English

    6271

    Clerk

    6147

    6127

    Step - V Step - VI

    6714 6714

    Scale 6712

    6712 6271

    Office 6147

    6271 6127

    146. (3) 147. (4) 148. (1) 149. (5)

    English 6147 Clerk 6127 Scale Office English Clerk

    Fourth to the left of “Scale” in Step-VI ⇒ 6712 Fourth to the right of “6271” in Step-V ⇒ 6127 Third to the right of “Office” in Step IV ⇒ 6271 Step - III

    ......Clerk 6714

    6712

    SBI PO EXAMS

    water

    233 223 model page scale

    6271 6147 ......

    150. (3) The element “6271” is fifth from the left end in both the Steps I and V. (151–155) : After careful analysis of the given input and various steps of rearrangement, it is evident that two elements (one word and one number) are arranged in each step. In the first step, the word which comes last as per English dictionary order is moved to the extreme left position and the lowest number is moved to the extreme right position. In the second step, the word which comes second last as per English dictionary order is moved to the extreme right position and the second lowest number is moved to the extreme left position. These two steps are repeated alternately till all the words and numbers get rearranged. Note : The last letter of the word, i.e., ‘s’ is deleted. The odd number is increased by ‘1’ while the even number is decreased by ‘1’. Input : frights 32 wallops 51 muddles 15 94 backups 64 heights 44 plutons Step I : wallop frights 32 51 muddles 94 backups 64 heights 44 plutons 16 Step II : 31 wallop frights 51 muddles 94 backups 64 heights 44 16 pluton Step III : muddle 31 wallop frights 51 94 backups 64 heights 16 pluton 43 Step IV : 52 muddle 31 wallop frights 94 backups 64 16 pluton 43 height Step V : fright 52 muddle 31 wallop 94 backups 16 pluton 43 height 63 Step VI : 93 fright 52 muddle 31 wallop 16 pluton 43 height 63 backup 151. (5) Step IV 52 muddle 31 wallop frights 94 backups 64 16 pluton 43 height In step IV, four numbers – 31, 94, 64 and 16 – appear betwen ‘muddle’ and ‘pluton’. 152. (3) The first element from the right end in the Step II ⇒ pluton 153. (1) In Step III, there are four elements – 51, 94, backups and 64 – between ‘frights’ and ‘heights’. 154. (5) In Step V, ‘16’ is at fifth position from the right end. 155. (1) In Step V, ‘backups 16 pluton 43’ appear in the same order.

    (1–3) : After careful analysis of the given input and various steps of rearrangement it is evident that one word and one number are arranged in each step. The numbers are arranged from the left and the words are arranged from the right. In the first step the word which comes last in the dictionary is shifted to the extreme rightwhile the highest number is shifted to the extreme left. In the second step the word which appears at the second last position in dictionary order is moved to the extreme right while the lowest number is shifted to the extreme left position. The same procedure is followed to get the final output. Input % Step I % Step II % Step III % Step IV % Step V % Step VI %

    fun 89 at the 28 16 base camp 35 53 here 68 89 fun at 28 16 base camp 35 53 here 68 the 16 89 fun at 28 base camp 35 53 68 the here 68 16 89 at 28 base camp 35 53 the here fun 28 68 16 89 at base 35 53 the here fun camp 53 28 68 16 89 at 35 the here fun camp base 35 53 28 68 16 89 the here fun camp base at

    1. (5) None of these 2. (3) 35 would be at 7th positon from the left in Step IV. 3. (4) This is Step V. (4–8) : After careful analysis of the given input and various steps of rearrangement, it is evident that in each step one ward and one number are rearranged. The word which comes first in the dictionary is placed at the extreme left position followed by the lowest number. In the second step, the word which comes second in the dictionary is placed at the extreme left position followed by the second lowest number. The same process is continued till all the words get rearranged in the reverse order of English alphabet and all the numbers get rearranged in descending order. Input : 32 proud girl beautiful 48 55 97 rich family 61 72 17 nice life Step I : beautiful 17 32 proud girl 48 55 97 rich family 61 72 nice life Step II : family 32 beautiful 17 proud girl 48 55 97 rich 61 72 nice life Step III : girl 48 family 32 beautiful 17 proud 55 97 rich 61 72 nice life Step IV: life 55 girl 48 family 32 beautiful 17 proud 97 rich 61 72 nice Step V: nice 61 life 55 girl 48 family 32 beautiful 17 proud 97 rich 72 Step VI: proud 72 nice 61 life 55 girl 48 family 32 beautiful 17 97 rich Step VII: rich 97 proud 72 nice 61 life 55 girl 48 family 32 beautiful 17 4. (3) Seven steps will be required to comptete the given input. 5. (4) The third element from the left end of Step VI is ‘nice’. 6. (3) Option (3) is the step III of the given input. 7. (1) The position of ‘nice’ from the left end in the final step is fifth. 8. (2) ‘17’ is third to the right of ‘family’ in Step V.

    BPRE–477

    INPUT (9–15) : After careful analysis of the given input and various steps of re-arrangement, it is evident that in each step one number or a word get re-arranged. In the first step, the highest number moves to the extreme left position. In the second step the word which comes first in the dictionary moves to the second position from the left. These two steps are repeated alternately till all the numbers get rearranged in descending order and the words in alphabetical order. Input : vital 54 cards 72 help 24 wall 66 lamp 49 Step I : 72 vital 54 cards help 24 wall 66 lamp 49 Step II : 72 cards vital 54 help 24 wall 66 lamp 49 Step III : 72 cards 66 vital 54 help 24 wall lamp 49 Step IV : 72 cards 66 help vital 54 24 wall lamp 49 Step V : 72 cards 66 help 54 vital 24 wall lamp 49 Step VI : 72 cards 66 help 54 lamp vital 24 wall 49 Step VII : 72 cards 66 help 54 lamp 49 vital 24 wall And, Step VII is the last step. 9. (3) Seven steps would be required to complete the rearrangement. 10. (1) There are five elements (help, 54, vital, 24, wall) between “66” and “lamp” in the Step V. 11. (4) This is Step VI. 12. (2) Fourth element from the right end in the Step III ⇒ 24 Immediate to the right of “24” ⇒ wall 13. (5) There are two elements between “66” and “lamp” in the final Step. Similarly, there are two elements between “72” and “help” in the final Step. So, “cards” would be related to “54”. 14. (2) The element “wall” would come at the eighth position from the left end in the Step V. 15. (4) The element “help” would be at the seventh position from the right end in the Step VI. (16–21) : After careful analysis of the input and various steps of rearrangement, it is evident that in each step one word and one number are rearranged. The word which comes last in the dictionary order is moved to the extreme left position while the highest number is moved to the extreme right position in the Step I. In the next step, the word which comes at the second position in dictionary order is placed at the second position from the left and the second highest number is placed at the second position from the right. Such procedure is continued till all the words get rearranged in dictionary order but in reverse manner and all the numbers get rearranged after the words in ascending order. Input : arrow 98 paint 58 lamb 38 each 78 great 18 most 48 rent 88 Step I : rent arrow paint 58 lamb 38 each 78 great 18 most 48 88 98 Step II : rent paint arrow 58 lamb 38 each 78 great 18 most 48 88 98 Step III : rent paint most arrow 58 lamb 38 each great 18 48 78 88 98 Step IV: rent paint most lamb arrow 38 each great 18 48 58 78 88 98 Step V : rent paint most lamb great arrow 38 each 18 48 58 78 88 98 Step VI: rent paint most lamb great each arrow 18 38 48 58 78 88 98

    16. (3) 5th to the left of the sixth from the right means 11th from the right, i.e., 4th from the left as there are 14 elements. 4th from the left in the Step V ⇒ lamb 17. (4) The position of ‘58’ in Step IV is eleventh from the left and fourth from right. 18. (1) There are eight elements (lamb, great, each, arrow, 18, 38, 48, 58) between “most” and “78” in the Step VI. 19. (5) This is Step III. 20. (2) 11th from the right means 4th from the left. 4th from the left in Step III is “arrow”. 21. (4) In the Step VI, “each” is at the sixth position from the left. 22. (1) In Step V, “great” is at the fifth position from the left. (23–30) : After careful analysis of the input and various steps of rearrangement, it is evident that in each step two elements – one number and one word – are rearranged. The word which comes first in the dictionary order is moved to the extreme left position while the lowest number is moved to the extreme right position in the Step I. In the next step, the word which comes second in the dictonary order is placed at the second position from the left while the second lowest number is placed at the second position from the right. The same procedure is continued till all the words get rearranged in dictionary order from the left and the numbers get rearranged in descending order after the words. Input : site grid 19 53 22 call art main 35 66 fill 93 Step I : art site grid 53 22 call main 35 66 fill 93 19 Step II : art call site grid 53 main 35 66 fill 93 22 19 Step III : art call fill site grid 53 main 66 93 35 22 19 Step IV : art call fill grid site main 66 93 53 35 22 19 Step V : art call fill grid main site 93 66 53 35 22 19 23. (4) This is Step IV. 24. (5) ‘53’ is fourth from the right or ninth from the left in Step IV. 25. (3) ‘Site’ is fourth from the left end in Step III. 26. (1) Five steps are needed to complete the arrangement. 27. (3) Option (3) is the final arrangement. 28. (1) Option (1) the Step II. 29. (5) Seventh from the right end of Step IV ⇒ main Second to the right of ‘main’ in Step IV ⇒ 93 30. (2) There are six elements (site, grid, 53, main, 66, 93) between ‘fill’ and ‘35’ in Step III.

    BPRE–478

    INPUT (31–35) : After careful analysis of the input and various steps of rearrangement, it is evident that in each step two elements – one number and one word – are rearranged. The word which comes first in the dictionary order is moved to the extreme left position while the lowest number is moved to the extreme right position in the Step I. In the next step, the word which comes second in the dictonary order is placed at the second position from the left while the second lowest number is placed at the second position from the right. The same procedure is continued till all the words get rearranged in dictionary order from the left and the numbers get rearranged in descending order after the words. Input : watch guide help sky 18 54 26 37 away press 76 42 85 decide Step I : away watch guide help sky 54 26 37 press 76 42 85 decide 18 Step II : away decide watch guide help sky 54 37 press 76 42 85 26 18 Step III: away decide guide watch help sky 54 press 76 42 85 37 26 18 Step IV : away decide guide help watch sky 54 press 76 85 42 37 26 18 Step V : away decide guide help press watch sky 76 85 54 42 37 26 18 Step VI : away decide guide help press sky watch 85 76 54 42 37 26 18 31. (2) Option (2) is the Step III. 32. (4) There are four elements (Words/Numbers) between ‘guide’ and ‘press’ in the Step IV. 33. (5) Third to the right of the eighth from the right means fifth from the right end. Fifth from the right end in Step V ⇒ 54 34. (3) Sixth 35. (1) Tenth from the right end in the Step IV ⇒ watch (36–41) : After careful analysis of the given input and various steps of rearrangement, it is evident that two elements (numbers or words) are rearranged in each step. In the first step the lowest number moves to the extreme right position while the second lowest number moves to the extreme left position. In the

    second step, the first word in alphabetical order moves to the extreme right position while the second word in alphabetical order moves to the extreme left position. These two steps are continued alternately to complete the rearrangement. Input : parenting 16 36 and raising 44 children 21 is 89 very 95 demanding 72 job 65 Step I : 21 parenting 36 and raising 44 children is 89 very 95 demanding 72 job 65 16 Step II : children 21 parenting 36 raising 44 children is 89 very 95 demanding 72 job 65 16 and Step III : 44 children 21 parenting raising is 89 very 95 demanding 72 job 65 16 and 36 Step IV : is 44 children 21 parenting raising 89 very 95 72 job 65 16 and 36 demanding Step V : 72 is 44 children 21 parenting raising 89 very 95 job 16 and 36 demanding 65 Step VI : parenting 72 is 44 children 21 raising 89 very 95 16 and 36 demanding 65 job Step VII : 95 parenting 72 is 44 children 21 raising very 16 and 36 demanding 65 job 89 Step VIII : very 95 parenting 72 is 44 children 21 16 and 36 demanding 65 job 89 raising 36. (2) ‘95 job 16 and’ appear in the same order in the Step V. 37. (4) The position of ‘72’ from the right end in the Step IV is seventh. 38. (1) 10th from the left end in the Step III ⇒ demanding 5th to the left of ‘demanding’ ⇒ raising 39. (2) Eight steps would be required to complete the given arrangement. 40. (5) ‘36’ is exactly between ‘parenting’ and ‘raising’ in the Step II. 41. (3) Third last step means Step VI. Option (3) is Step VI.

    (42–47) : After careful analysis of the given input and various steps of rearrangement it is evident that two elements (numbers or words) are rearranged in each step. In the first step the highest number moves to the extreme left position while the second highest number moves to the extreme right position. In the second step the word which comes last in dictionary order is moved to the second position from the left while the word which comes second from the last is moved to the second position from the right. In the third step the third highest number is moved to the extreme left position while the fourth highest number is moved to the extreme right position. In the fourth step the word which comes third from the last in dictionary order is moved to the fourth position the from the left while the word which comes fourth from the last is moved to the fourth position from the right. The same procedure is continued till all the numbers and words get rearranged. Input : 65 work in 23 to be 13 89 managed 48 97 load 34 healthy 55 style Step I : 97 65 work in 23 to be 13 managed 48 load 34 healthy 55 style 89 Step II : 97 work 65 in 23 be 13 managed 48 load 34 healthy 55 style to 89 Step III : 65 97 work in 23 be 13 managed 48 load 34 healthy style to 89 55 Step IV : 65 97 work style in 23 be 13 48 load 34 healthy managed to 89 55 Step V : 48 65 97 work style in 23 be 13 load healthy managed to 89 55 34 Step VI : 48 65 97 work style load 23 be 13 healthy in managed to 89 55 34 Step VII : 23 48 65 97 work style load be healthy in managed to 89 55 34 13 Step VIII : 23 48 65 97 work style load healthy be in managed to 89 55 34 13

    BPRE–479

    INPUT 42. (4) The element ‘load’ is exactly between ‘style’ and ‘be’ in the second last step, i.e., Step VII. 43. (5) The position of ‘48’ in the Step III is eighth from the right end. 44. (3) Option (3) is Step V. 45. (2) The elements ‘be 13 healthy in’ are found in the same order in the Step VI. 46. (1) Eight Steps would be required to complete the given arrangement. 47. (3) Third last Step means Step VI. 12th from the left end in Step VI ⇒ managed 8th to the left of ‘managed’ ⇒ work TRICK 8th to the left of 12th from the left means 4th from the left. (48 – 52) : After careful analysis of the input and various steps of rearrangement, it is evident that in each step two elements – one number and one word – are rearranged. The word which comes first in the dictionary order is moved to the extreme left position while the lowest number is moved to the extreme right position in the Step I. In the next step, the word which comes second in the dictonary order is placed at the second position from the left while the second lowest number is placed at the second position from the right. The same procedure is continued till all the words get rearranged in dictionary order from the left and the numbers get rearranged in descending order after the words. Input : watch guide help sky 18 54 26 37 away press 76 42 85 decide Step I : away watch guide help sky 54 26 37 press 76 42 85 decide 18 Step II : away decide watch guide help sky 54 37 press 76 42 85 26 18 Step III : away decide guide watch help sky 54 press 76 42 85 37 26 18 Step IV : away decide guide help watch sky 54 press 76 85 42 37 26 18 Step V : away decide guide help press watch sky 76 85 54 42 37 26 18 Step VI : away decide guide help press sky watch 85 76 54 42 37 26 18 48. (2) Option (2) is the Step III. 49. (4) There are four elements (Words/Numbers) between ‘guide’ and ‘press’ in the Step IV. 50. (5) Third to the right of the eighth from the right means fifth from the right end. Fifth from the right end in Step V ⇒ 54 51. (3) Sixth 52. (1) Tenth from the right end in the Step IV ⇒ watch (53–58) : After careful analysis of the given input and various steps of rearrangement, it is evident that two elements (numbers or words) are rearranged in each step. In the first step the lowest number moves to the extreme right position while the second lowest number moves to the extreme left position. In the second step, the first word in alphabetical order

    moves to the extreme right position while the second word in alphabetical order moves to the extreme left position. These two steps are continued alternately to complete the rearrangement. Input : parenting 16 36 and raising 44 children 21 is 89 very 95 demanding 72 job 65 Step I : 21 parenting 36 and raising 44 children is 89 very 95 demanding 72 job 65 16 Step II : children 21 parenting 36 raising 44 children is 89 very 95 demanding 72 job 65 16 and Step III : 44 children 21 parenting raising is 89 very 95 demanding 72 job 65 16 and 36 Step IV : is 44 children 21 parenting raising 89 very 95 72 job 65 16 and 36 demanding Step V : 72 is 44 children 21 parenting raising 89 very 95 job 16 and 36 demanding 65 Step VI : parenting 72 is 44 children 21 raising 89 very 95 16 and 36 demanding 65 job Step VII : 95 parenting 72 is 44 children 21 raising very 16 and 36 demanding 65 job 89 Step VIII : very 95 parenting 72 is 44 children 21 16 and 36 demanding 65 job 89 raising 53. (2) ‘95 job 16 and’ appear in the same order in the Step V. 54. (4) The position of ‘72’ from the right end in the Step IV is seventh. 55. (1) 10th from the left end in the Step III ⇒ demanding 5th to the left of ‘demanding’ ⇒ raising 56. (2) Eight steps would be required to complete the given arrangement. 57. (5) ‘36’ is exactly between ‘parenting’ and ‘raising’ in the Step II. 58. (3) Third last step means Step VI. Option (3) is Step VI. (59 – 64) : After careful analysis of the given input and various steps of rearrangement it is evident that two elements (numbers or words) are rearranged in each step. In the first step the highest number moves to the extreme left position while the second highest number moves to the extreme right position. In the second step the word which comes last in dictionary order is moved to the second position from the left while the word which comes second from the last is moved to the second position from the right. In the third step the third highest number is moved to the extreme left position while the fourth highest number is moved to the extreme right position. In the fourth step the word which comes third from the last in dictionary order is moved to the fourth position the from the left while the word which comes fourth from the last is moved to the fourth position from the right. The same procedure is continued till all the numbers and words get rearranged.

    BPRE–480

    INPUT Input : Step I : Step II : Step III : Step IV : Step V : Step VI : Step VII : Step VIII

    65 work 97 65 97 work 65 97 65 97 48 65 48 65 23 48 : 23 48

    in 23 to be 13 89 managed 48 97 load 34 healthy 55 style work in 23 to be 13 managed 48 load 34 healthy 55 style 89 65 in 23 be 13 managed 48 load 34 healthy 55 style to 89 work in 23 be 13 managed 48 load 34 healthy style to 89 55 work style in 23 be 13 48 load 34 healthy managed to 89 55 97 work style in 23 be 13 load healthy managed to 89 55 34 97 work style load 23 be 13 healthy in managed to 89 55 34 65 97 work style load be healthy in managed to 89 55 34 13 65 97 work style load healthy be in managed to 89 55 34 13

    59. (4) The element ‘load’ is exactly between ‘style’ and ‘be’ in the second last step, i.e., Step VII. 60. (5) The position of ‘48’ in the Step III is eighth from the right end. 61. (3) Option (3) is Step V. 62. (2) The elements ‘be 13 healthy in’ are found in the same order in the Step VI. 63. (1) Eight Steps would be required to complete the given arrangement. 64. (3) Third last Step means Step VI. 12th from the left end in Step VI ⇒ managed

    66. (5) Step IV literatim legation

    lethal lemon 82 67 let 75 lend 35

    64 83 62

    In Step IV ‘legible’ is fifth to the left of ‘let’, But, ‘legible’ interchanges position with ‘35’. 67. (2) 5th to the right of 10th from the right means 5th from the right. 5th from the right in the Step IV ⇒ lend 68. (4) Step II literatim legation

    8th to the left of ‘managed’ ⇒ work TRICK 8th to the left of 12th from the left means 4th from the left. (65–69) : After careful analysis of the given input and various steps of rearrangement, it is evident that two elements (one word and one number) are rearranged in each step. In the first step, the word having the maximum number of alphabet is moved to the extreme left position while the lowest number is moved to extreme right position and the positions of digits are interchanged. In the second step, the word with second highest number of alphabet is moved to the second place from the left end and the second lowest number is moved to the second place from the right end and the positions of digits are interchanged. The same procedure is continued till all the words and numbers get rearranged. Input : legation lemon 82 legible 67 46 let 53 75 lethal 38 literatim 26 lend Step I : literatim legation lemon 82 legible 67 46 let 53 75 lethal 38 lend 62 Step II : literatim legation lemon 82 legible 67 46 let 53 75 lethal lend 83 62 Step III : literatim legation legible lemon 82 67 let 53 75 lethal lend 64 83 62 Step IV : literatim legation legible lethal lemon 82 67 let 75 lend 35 64 83 62 Step V : literatim legation legible lethal lemon 82 let 75 lend 76 35 64 83 62 Step VI : literatim legation legible lethal lemon lend 82 let 57 76 35 64 83 62 Step VII : literatim legation legible lethal lemon lend let 28 57 76 35 64 83 62 65. (3) First two elements in Step V ⇒ literatim, legation Last two elements in Step ⇒ 83, 62

    legible

    lemon

    82 legible 67 46 let 53 75 lethal lend 83 62

    69. (1) The elements ‘lethal lemon 82 let 75’ are found in the same order in the Step V. (70–72) : After careful analysis of the given input and various steps of rearrangement, it is clear that in the Step I, the first digit of the first box is multiplied with the second digit of the fourth box and the second digit is multiplied with the first digit of that box. In the Step II, the sum of the first digit of the first box and both the digits of the second box is equal to the number given in the first box. Similarly, the sum of the second digit of the first box and both the digits of the third box is equal to the number given in the second box. In the Step III, one-half of the sum of the two digits of the first box is given in the first box. Similarly, one-half of the sum of digits of the second box is given in the second box. In the Step IV, the number is obtained by subtracting the second number from the first number.

    Input 4 2 Step I

    5 1

    2 9

    8 6

    5 7

    Step II

    2 0

    Step III

    1

    Step IV

    3 2

    7 1

    1 4

    8 9 2 3 2.5

    –1.5

    70. (4) The two numbers obtained in the Step III ⇒ 1, 2.5 Required Sum = 1 + 2.5 = 3.5 71. (3) The numbers obtained in the Step I ⇒ 86, 57 and 89

    BPRE–481

    INPUT Required sum = 86 + 57 + 89 = 232 The numbers obtained in other Steps ⇒ 20, 23, 1, 2.5 and –1.5 Required sum = 20 + 23 + 1 + 2.5 + (–1.5) = 45 Required difference = 232 – 45 = 187 72. (5) The numbers obtained in the Step II = 20 and 23 Required product = 20 × 23 = 460 (73–77) : After careful analysis of the given input and various steps of rearrangement, it is evident that two elements (one word and one number) are rearranged in each Step. Upto third step odd numbers are rearranged and then even numbers are rearranged. In the first Step, the lowest odd number is placed at the extreme left position after adding one to it while the word which comes first as per dictionary order is placed at the extreme right position. In the second Step, the word which comes second as per the dictionary order is placed at the extreme left position while the second lowest odd number is placed at the extreme right position after adding one to it. In the third Step, the highest odd number is placed at the extreme left position after adding one to it while the word which comes third as per the dictionary order is placed at the extreme right position. In the fourth Step, the lowest even number is placed at the extreme right position after subtracting one from it while the word which comes fourth as per the dictionary order is placed at the extreme left position. In the fifth Step, the second lowest even number is placed at the extreme left position after subtracting one from it while the word which comes fifth as per the dictionary order is placed at the extreme right position. In the sixth Step, the highest even number is placed at the extreme right position after subtracting one from it while the word which comes last as per the dictionary order is placed at the extreme left position. Input Step I Step II Step III Step IV Step V Step VI

    17 8 fix 30 on 11 time

    and 17 8 fix 30 on 11

    32 32 32 8 fix 30 on

    on on on 32 8 fix 30

    12 12 12 on 32 8 fix

    73. (3) Step IV ....... 8 32 12 time put ....... 74. (1) 2nd to the left of 5th from the right end means 7th from the right end. 7th from the right end in Step III ⇒ 12 75. (5) Six Steps 76. (4) Option (4) is the Step V. 77. (2) In Step VI, ‘time’ is second to the left of ‘on’. In Step VI, ‘never’ is second to the left of ‘put’. In Step VI, ‘fix’ is second to the left of ‘and’. (78–81) : After careful analysis of the given input and various steps of rearrangement it is evident that in each step two elements (one letter and one number) are rearranged. In the first step, the letter which comes first as per the English alphabetical series is moved to the extreme left position and it is replaced with the next alphabet. Similarly, the lowest number is moved to the second position from the left after adding one to it. In the second step, the letter which comes second as per the English alphabetical series is moved to the extreme left position and it is replaced with the next alphabet. Similarly, the second lowest number is moved to the second position from the left after adding one to it. The same procedure is continued to rearrange all the letters and numbers. Input K 50 39 P 15 27 C Y 20 F Step I D 16 K 50 39 P 27 Y 20 F Step II G 21 D 16 K 50 39 P 27 Y Step III L 28 G 21 D 16 50 39 P Y Step IV Q 40 L 28 G 21 D 16 50 Y Step V Z 51 Q 40 L 28 G 21 D 16

    never never never 12 12 32 8

    29 29 29 time time time and

    time time time put put and 18

    7 put put 4 and 18 never

    put 4 4 and 18 never 3

    4 fix and 18 never 3 put

    fix and 18 never 3 put 31

    78. (5) In Step III, ‘28’ and ‘G’ appear between ‘L’ and ‘21’. 79. (4) Second last Step ⇒ Step IV In Step IV, ‘40’ is fifth to the left of ‘D’. 80. (3) In Step II, fifth from the right end ⇒ 50 The element ‘50’ is to the immediate left of ‘39’. 81. (1) 3rd element from the right end in Step IV ⇒ 16 2nd element from the left end in Step I ⇒ 16 Required Sum ⇒ 16 + 16 = 32 (82–86) : The numbers in the first column are based on the following pattern : x + 31; x + 32; x + 33; x + 34 The numbers in the second column are based on the following pattern : x + 21; x + 22; x + 23; x + 24 These two steps are applied to all the columns alternately. Input 11 10 15 14 18 17 20 Step–I 14 12 18 16 21 19 23 Step–II 20 14 24 18 27 21 29 Step–III 38 18 42 22 45 25 47 Step–IV 92 26 96 30 99 33 101 82. (3) 4th from the left end in the Step–IV ⇒ 30 83. (5) 6th from the right end in the Step–III ⇒ 18 84. (2) 18 is in the middle in the Step–II. 85. (1) 3rd from the left end in Step–I ⇒ 18 12 is to the immediate left of 18. 86. (3)

    BPRE–482

    10 ⇒ 26 14 ⇒ 30 20 ⇒ 101 Total ⇒ 157

    INPUT

    RBI GRADE–B/ NABARD GRADE–A OFFICER EXAMS (1–5) : After careful analysis of the given input and various steps of rearrangement it is evident that in each step one word and one number are rearranged. The words are rearranged from left in alphabetical order and the numbers are rearranged from the right in descending order but in the final step the words get rearranged in alphabetical order in reverse manner the numbers appear in descending order. Input : 31 rise gem 15 92 47 aim big 25 does 56 not 85 63 with moon Step I : aim 31 rise gem 15 47 big 25 does 56 not 85 63 with moon 92 Step II : big aim 31 rise gem 15 47 25 does 56 not 63 with moon 92 85 Step III : does big aim 31 rise gem 15 47 25 56 not with moon 92 85 63 Step IV : gem does big aim 31 rise 15 47 25 not with moon 92 85 63 56 Step V : moon gem does big aim 31 rise 15 25 not with 92 85 63 56 47 Step VI : not moon gem does big aim rise 15 25 with 92 85 63 56 47 31 Step VII : rise not moon gem does big aim 15 with 92 85 63 56 47 31 25 Step VIII : with rise not moon gem does big aim 92 85 63 56 47 31 25 15 1. (1) Eight 2. (4) ‘15’ is at the 7th position from the left in Step IV 3. (2) It is Step VII. 4. (3) ‘92’ is at the sixth position from the right in Step VI. 5. (3) ‘56’ is at the fifth position from the right in Step the last step. (6–11) : After careful analysis of the given input and various steps of re-arrangement, it is evident that in each step one number and one word are rearranged. In the first step the highest number moves to the extreme left position while the word which comes first in the alphabetical order moves to the extreme right position. In the next step the second highest number moves to the second position from the left and the word which comes second in the alphabetical order moves to second position from the right. The same procedure is continued till all the numbers get arranged in descending order from the left and the words get arranged in alphabetical order from the right. Input : hard work pays 96 42 in 79 long run 18 25 57 Step I : 96 work pays 42 in 79 long run 18 25 57 hard Step II : 96 79 work pays 42 long run 18 25 57 in hard Step III : 96 79 57 work pays 42 run 18 25 long in hard Step IV : 96 79 57 42 work run 18 25 pays long in hard Step V : 96 79 57 42 25 work 18 run pays long in hard Step VI : 96 79 57 42 25 18 work run pays long in hard 6. (3) This is Step IV. 7. (1) There are six elements (pays, 42, run, 18, 25, long) between ‘work’ and ‘in’ in the Step III. 8. (4) The word ‘pays’ is fourth from the left in the Step II. 9. (2) The word ‘run’ is seventh from the right in the Step IV. 10. (3) The word ‘pays’ is ninth from the left in the Step V. 11. (2) Fourth element from the right in the Step III ⇒ 25 Third to the left of 25 ⇒ 42 (12-16) : After careful analysis of the given input and various steps of rearrangement, it is evident that in each step four elements (two words and two numbers) are rearranged. In the first step the lowest number is placed at the extreme left position while the highest number is placed at the extreme right position. Again, the word which comes first in the dictionary order is placed at the second position from left and the word which comes last is placed at the second position from the right. In the second step, the second lowest number is placed at the second position from the left and the second highest number is placed at the second position from the right. Again, the word which comes second in the dictionary order is placed at the fourth position from the left and the word which comes second last in the dictionary order is placed at the fourth position from the right. The same procedure is continued till all the words get arranged in the dictionary order after four numbers which are in ascending order. The remaining four numbers also get arranged in ascending order after the eight words.

    BPRE–483

    INPUT Input %

    31 11 win arm blanket zebra 24 81 chip team slip 62 55 dawn 91 78

    Step I %

    11 arm 31 win blanket 24 81 chip team slip 62 55 dawn 78 zebra 91

    Step II % 11 24 arm blanket 31 chip team slip 62 55 dawn 78 win zebra 81 91 Step III % 11 24 31 arm blanket chip slip 62 55 dawn team win zebra 78 81 91 Step IV % 11 24 31 55 arm blanket chip dawn slip team win zebra 62 78 81 91 12. (4) The position of “slip” in the Step III is seventh from the left. 13. (2) There are six elements (slip, team, win, zebra, 62 78) between “dawn” and “81” in Step IV. 14. (1) “62” is at the ninth position from the left in Step II. 15. (3) In the Step III “dawn” is at the seventh position from the right. 16. (5) Sixth element from the right in the Step III ⇒ team Third to the left of “team”⇒ 62 (17-21) : After careful analysis of the given input and various steps of rearrangement it is evident that in each step two elements (one word and one number) are rearranged. In the first step the highest number moves to the extreme left position and its digits are interchanged while the word which contains least number of letters, moves to the extreme right position. In the second step, the second highest number moves to the extreme left position and its digits are interchanged while the word which contains the second lowest number of letters moves to the extreme right position. The same procedure is continued till all the numbers and words get rearranged following the rule mentioned above. (17–21) : Input : tyre 71 toughen 59 tip 82 13 thanks to 68 table 46 Step I : 28 tyre 71 toughen 59 tip 13 thanks 68 table 46 to Step II : 17 28 tyre toughen 59 13 thanks 68 table 46 to tip Step III : 86 17 28 toughen 59 13 thanks table 46 to tip tyre Step IV : 95 86 17 28 toughen 13 thanks 46 to tip tyre table Step V : 64 95 86 17 28 toughen 13 to tip tyre table thanks Step VI : 31 64 95 86 17 28 to tip tyre table thanks toughen 17. (2) 31 + 1 = 32; 64 – 2 = 62; 95 + 1 = 96; 86 – 2 = 84; 17 + 1 = 18; 28 – 2 = 26; The numbers 32, 84 and 96 are multiples of 4.

    18. (1) Step III : 86 17 28 tough 59 13 thanks table 46 to tip tyre

    ‘28’ will be to the immediate left of ‘46’. 19. (4) Second last step means Step V : 3rd element from the left in Step V ⇒ 86 9th element from the left in Step V ⇒ tip 20. (3) The element ‘13’ comes exactly between ‘59’ and ‘thanks’ in Step II. 21. (5) 8th from the right end in Step IV ⇒ toughen Third to the right of ‘toughen’ ⇒ 46 (22 – 23) : After careful analysis of the given input and various steps of rearrangement it is clear that in each step two elements are rearranged. The highest number moves to the extreme left position while the word which would appear last in the English alphabetical order is moved to the extreme right position in the first step. In the second step, the second highest number is moved to the extreme left position while the word which would appear second last in the English alphabetical order is moved to the extreme right position. The same procedure is continued till all the numbers get rearranged in ascending order from left to right and the words get rearranged in the English alphabetical order after the numbers but in the reverse order. Input : regular 46 exercise 31 29 must for 54 all 83 91 now Step I : 91 46 exercise 31 29 must for 54 all 83 now regular Step II : 83 91 46 exercise 31 29 must for 54 all regular now Step III : 54 83 91 46 exercise 31 29 for all regular now must Step IV : 46 54 83 91 exercise 31 29 all regular now must for Step V : 31 46 54 83 91 29 all regular now must for exercise Step VI : 29 31 46 54 83 91 regular now must for exercise all 22. (2) Eleventh element from the left in the last step ⇒ exercise Seventh to the left of “exercise” ⇒ 54 TRICK Seventh to the left of eleventh from the left in the last step implies fourth from the left (11–7 = 4) Fourth from the left in the last step ⇒ 54 23. (1) ‘29’ is exactly between ‘31’ and ‘for’ in the third step. (24–28) : After careful analysis of the given input and various steps of rearrangement, it is evident that in each step two elements (one word and one number) are rearranged. In the first step, the lowest number

    BPRE–484

    INPUT moves to the extreme left position and the word which comes last in the alphabetical order moves to the second position from the left. In the second step, the word which comes second last in the alphabetical order moves to the extreme right position and the second lowest number moves to the second position from the right. The same procedure is continued till all the numbers and words get rearranged. Input : CHANT 18 SALTY 45 ABACUS WARDEN 30 91 67 KNIGHT Step I : 18 WARDEN CHANT SALTY 45 ABACUS 30 91 67 KNIGHT Step II : 18 WARDEN CHANT 45 ABACUS 91 67 KNIGHT 30 SALTY Step III : 45 KNIGHT 18 WARDEN CHANT ABACUS 91 67 30 SALTY Step IV : 45 KNIGHT 18 WARDEN ABACUS 91 30 SALTY 67 CHANT Step V : 91 ABACUS 45 KNIGHT 18 WARDEN 30 SALTY 67 CHANT 24. (2) Fourth element from the left in the second last step ⇒ WARDEN Fifth element from the right in the second last step ⇒ 91 ‘ABACUS’ lies exactly between ‘WARDEN’ and ‘91’ in the second last step. 25. (3) Two consecutive elements to the immediate right of ‘KNIGHT’ in the last step ⇒ 18, WARDEN 26. (1) ‘18’ is seventh from the left of ‘SALTY’ in the Step III. 27. (5) Only ‘CHANT’ appears exactly between ‘WARDEN’ and ‘ABACUS’ in the Step III. 28. (4) In Step I, ‘WARDEN’ is fourth to the left of ‘ABACUS’. In Step IV, ‘91’ is fourth to the left of ‘CHANT’. In Step V, ‘ABACUS’ is fourth to the left of ‘WARDEN.’ (29–33) : After careful analysis of the given input and various steps of rearrangement, it is evident that in each step two elements — one word and one number — are rearranged. In the first step, the word which comes first in the dictionary order is placed at the extreme left position after replacing the second and the third letters with ‘a’ while the lowest number is placed at the second position from the left end after adding ‘2’ to it. In the second step, the word which comes second in the dictionary order is placed at the extreme left position after replacing the second and the third letters with ‘a’ while the second position from the left end after adding ‘2’ to it. The similar procedure is continued till all the words and the numbers get rearranged. Input Step I Step II Step III Step IV

    : : : : :

    61 rust cat 30 far 35 lap 51 rap 63

    Step V : tan

    88

    33 61 cat far lap

    colt rust 30 35 51

    86 33 61 cat far

    rap

    63

    lap

    four torn 28 86 four torn rust 86 torn 30 61 rust 35 cat 30 51

    far

    35

    49 49 49 86 86

    leap leap leap torn torn

    cat

    30

    29. (5) 4th to the left of the 8th element from left means 4th from the left. 4th from the left in Step II ⇒ 30 30. (2) 5th to the right of ‘cat’ in Step III ⇒ torn 31. (1) Step III 35 cat 30 61 rust 86

    far

    lap 51

    torn

    32. (3) 3rd element from the right end in Step V ⇒ 35 5th element from the left end in Step II ⇒ 61 Required difference = 61 – 35 = 26 33. (4) ‘torn’ is 4th to the right of ‘rust’ in Step I. ‘lap’ is 4th to the right of ‘tan’ in Step V. Similarly, ‘86’ is 4th to the right of ‘far’ in Step IV. (34–38) : After careful analysis of the given input and various steps of rearrangement it is evident that two elements — one word and one number — are rearranged in each step. In the first step, the highest number is moved to the extreme right position after deducting two from it while the word that comes last in the alphabetical series is moved to the second position from the right end after replacing the second and the third letters with the alphabet ‘u’. In the second step, the second highest number is moved to the extreme right position after deducting two from it and the word which comes second last in the alphabetical series is moved to the second position from the right end after replacing the second and the third letters with the alphabet ‘u’. The similar procedure is continued till all the numbers and words get rearranged. Input Step I Step II Step III Step IV

    61 61 17 17 17

    neon neon 28 28 cart

    17 17 four four sun

    skin 28 cart car 84

    28 four 42 sun nun

    four cart here 84 59

    cart 42 sun nun hue

    86 here 84 59 40

    42 sun nun hue fur

    here 84 59 40 26

    Step V

    sun

    84

    nun

    59

    hue

    40

    fur

    26

    cut

    15

    34. (1) Fifth element from the left in the Step II ⇒ 42 Fifth element from the right in the Step IV ⇒ 59 Required difference = 59 – 42 = 17 35. (5) Step IV 17 cart sun

    84

    nun 59 hue

    49

    fur 26

    36. (2) Seventh from the left end of the Step II ⇒ sun Third to the left of ‘sun’ ⇒ cart 37. (3) Fifth to the left of ‘nun’ in the Step III ⇒ 28 38. (4) The element ‘sun’ is third to the right of ‘cart’ in the Step I. The element ‘hue’ is third to the right of ‘84’ in the Step III. The element ‘59’ is third to the right of ‘sun’ in the Step V. (39-43) : After careful analysis of the given input and various steps of rearrangement, it is evident that two elements (one word and one number) are rearranged in each step. In the first step the word which comes first in the alphabetical order is moved to the extreme left position after deleting all the vowels while

    BPRE–485

    INPUT the highest number is moved to the extreme right position and it is replaced with the next prime number. In the second step, the word which comes second in the alphabetical order is moved to the extreme left position after deleting all the vowels while the second highest number is moved to the extreme right position and it is replaced with the next prime number. The same procedure is continued till all the words and numbers get rearranged. Prime Numbers upto 100 2, 3, 5, 7, 11, 13, 17, 19,, 23, 29, 31, 37, 41, 43, 47, 53, 59, 61, 67, 71, 73, 79, 83, 89, 97 Input : neat bites 23 11 piles your 37 79 give 47 Step I : bts neat 23 11 piles your 37 give 47 83 Step II : gv bts neat 23 11 piles your 37 83 53 Step III : nt gv bts 23 11 piles your 83 53 41 Step IV : pls nt gv bts 11 your 83 53 41 29 Step V : yr pls nt gv bts 83 53 41 29 13 39. (3) In Step IV, ‘pls’ is second to the left of ‘gv’. In Step III, ‘83’ is second to the left of ‘41’. In Step V, ‘bts’ is second to the left of ‘53’. 40. (1) 3rd element to the left of the 7th element from the left end means 4th element from the left end. Step V yr pls nt

    gv bts 83 53 41 29 13

    4th from left end

    41. (1) Step III : nt gv bts 23 11 piles

    your

    83

    53 41

    42. (2) Step IV pls nt gv bts 11 your 83 53 41 29 4th to the right

    43. (5) In Step IV, ‘bts 11 your’ are found consecutively in the same order. 44. (4) In Step II, ‘23’ is to the immediate right and ‘bts’ is to the immediate left of ‘neat’. (45–49) : After careful analysis of the input and various steps of rearrangement, it is evident that two elements (one word and one number) are rearranged in each step. In the first step, the word which comes first as per dictionary order moves to the extreme left position while the lowest number moves to the extreme right position. In the second step, the word which comes second as per dictionary order moves to the extreme left position while the second lowest number moves to the extreme right position. The same procedure is continued in the next steps till all the words and the numbers get rearranged. Input : weight 56 fasten 71 beast 97 49 mould 12 stronger Step I : beast weight 56 fasten 71 97 49 mould stronger 12

    Step II

    : fasten beast weight 56 71 97 mould stronger 12 49 Step III : mould fasten beast weight 71 97 stronger 12 49 56 Step IV : stronger mould fasten beast weight 97 12 49 56 71 Step V : weight stronger mould fasten beast 12 49 56 71 97 45. (1) This is the Step III. 46. (3) In Step IV, ‘mould’ is fifth to the left of ‘12’. 47. (2) In the first Step, ‘56’ is third from left and ‘49’ is seventh from the left. 48. (4) There are five elements between ‘stronger’ and ‘56’ in the last Step. 49. (5) In Step II, ‘97’ is fifth to the right of ‘fasten’. (50–54) : After careful observation of the given input and the various steps of rearrangement, it is evident that two numbers are rearranged in each step from the left side in ascending order. The odd number is written after rearrangement adding ‘3’ while the even number is halved. Input Step–I Step–II Step–III Step–IV Step–V

    : : : : : :

    47 18 19 27 32 90

    87 32 50 28 74 45

    38 47 18 19 27 32

    15 87 32 50 28 74

    56 38 87 18 19 27

    54 56 56 32 50 28

    64 54 54 87 18 19

    71 64 64 64 32 50

    90 71 71 71 87 18

    29 90 90 90 90 32

    50. (5) The element ‘90’ appears in the given input as well as in the last step of the given input. 51. (2) 3rd element from the right end of the Step–II ⇒ 64 5th element from the left end of Step–I ⇒ 38 Required sum = 64 + 38 = 102 52. (4) In step IV, the element ‘87’ is second from the right end. 53. (1) 4th element to the left of the 7th element from the left end meand 3rd element from the left end. 3rd element from the left end of the Step–III ⇒ 19 54. (5) Step–II : 19 50 18 32 87 56 54 64 71 90 1 2 3 4

    INSURANCE EXAMS (1–4) : Input : dare a beach null wrap steel Step I : dare beach null wrap steel a Step II : dare null wrap steel a beach Step III : null wrap steel a beach dare Step IV : wrap steel a beach dare null Step V : wrap a beach dare null steel Step VI : a beach dare null steel wrap Step VI is the last step.

    BPRE–486

    INPUT 1. (3) Six steps will be required to reach the intended arrangement for the given input. 2. (2) Option (2) is the same as step II. 3. (3) The word ‘a’ is second from the left end of Step V. 4. (4) The word ‘steel’ is fourth from the right end of step III. (5–8) : (E) : Step I : kite metal blue offer arrange (C) : Step II : kite metal offer arrange blue (A) : Step III : metal offer arrange blue kite (D) : Step IV: offer arrange blue kite metal (B) : Step V : arrange blue kite metal offer 5. (1) (A) is the Step III. 6. (4) The word ‘blue’ is the first from the right end of Step II. 7. (2) (B) is the Step V. 8. (4) (D) is the Step IV. (9–13) : After careful observation of the given input and various steps of rearrangement it is evident that in each step one word is rearranged. The words are rearranged as per alphabetical order and the number of letters in the word. 9. (2) Input : the in car as he may me Step I : car the in as he may me Step II : car may the in as he me Step III : car may the as in he me 10. (1) Step II : clever remand window sales batch tiger never Step III : clever remand window batch sales tiger never Step IV : clever remand window batch never sales tiger 11. (3) Input : true se veto be nuke my like Step I : like true se veto be nuke my Step II : like nuke true se veto be my Step III : like nuke true veto se be my Step IV : like nuke true veto be se my Step V : like nuke true veto be my se 12. (3) Input : more fights cats cough sough acts idea Step I : fights more cats cough sough acts idea Step II : fights cough more cats sough acts idea Step III : fights cough sough more cats acts idea Step IV : fights cough sough acts more cats idea Step V : fights cough sough acts cats more idea Step VI : fights cough sough acts cats idea more 13. (2) Input : amis goes to the bar after dinner everyday Step I : everyday amis goes to the bar after dinner Step II : everyday dinner amis goes to the bar after Step III : everyday dinner after amis goes to the bar Step IV : everyday dinner after amis goes bar to the Step V : everyday dinner after amis goes bar the to

    (14-18) : After careful analysis of the given input and various steps of re-arrangement it is evident that in each step one number and one word are rearranged. In the first step the highest number and the word that comes in the last in alphabetical order move to the extreme left position. In the next step the second highest number and the word that comes last but one in alphabetical order move to the extreme left position. The same procedure is continued till all the numbers get arranged in ascending order and all the words in alphabetical order. Input : store 95 clean 56 tape 15 break 28 feet 35 wait 69 ice 71 Step I : 95 wait store clean 56 tape 15 break 28 feet 35 69 ice 71 Step II : 71 tape 95 wait store clean 56 15 break 28 feet 35 69 ice Step III : 69 store 71 tape 95 wait clean 56 15 break 28 feet 35 ice Step IV : 56 ice 69 store 71 tape 95 wait clean 15 break 28 feet 35 Step V : 35 feet 56 ice 69 store 71 tape 95 wait clean 15 break 28 Step VI : 28 clean 35 feet 56 ice 69 store 71 tape 95 wait 15 break Step VII : 15 break 28 clean 35 feet 56 ice 69 store 71 tape 95 wait And, Step VII is the last step of the above input. 14. (1) The given output is the Step V. 15. (3) ‘Clean’ is at the sixth position from the right in step IV. 16. (4) There are eight elements. 17. (2) ‘wait’ is the sixth from the left in the third Step. 18. (5) ‘69’ is at seventh position from the left in the Step VI. (19–24) : After careful analysis of the given input and various steps of rearangement, it is evident that in each step two elements (one word and one number) are rearranged. In the step I, the word which comes first in the dictionary order is placed at the extreme left position while the lowest number is placed at the extreme right position. In the step II, the second lowest number is placed at the extreme left position while the word which comes second in the dictionary order is placed at the extreme right position. These two steps are continued alternately to complete the rearrangement. Input : weight 34 93 and 14 density 84 concept to 49 74 be understood 56 Step I : and weight 34 93 density 84 concept to 49 74 be understood 56 14 Step II : 34 and weight 93 density 84 concept to 49 74 understood 56 14 be

    BPRE–487

    INPUT Step III Step Step Step Step 19.

    : concept 34 and weight 93 density 84 to 74 understood 56 14 be 49 IV : 56 concept 34 and weight 93 84 to 74 understood 14 be 49 density V : to 56 concept 34 and weight 93 84 understood 14 be 49 density 74 VI : 84 to 56 concept 34 and weight 93 14 be 49 density 74 inderstood VII : weight 84 to 56 concept 34 and 14 be 49 density 74 understood 93 (5) Eleventh element from the right in the last Step ⇒ 56 Seventh to the right of 56 ⇒ density

    TRICK

    20. 21. 22. 23. 24.

    Seventh to the right of eleventh from the right means fourth from the right (11–7 = 4). Fourth from the right ⇒ density (2) The position of ‘concept’ from the right end of the second last Step is eleventh. (3) Step IV : ....... 93 84 to .... (4) The position of ‘weight’ from the left of ‘56’ in the third step is seventh. (2) The elements ‘to 49 47 understood’ are found in the same order in the Step II. (1) Step VII is the last Step. Step V is the third last Step. Option (1) is step V.

    (25–29) : After careful analysis of the given input and various steps of rearrangement, it is evident that two elements (one number and one word) are rearranged in each step. In the first step, the lowest number is moved to the extreme left position while the word which comes first in alphabetical order is moved to the extreme right position. In the second step the second lowest number is moved to the extreme left position and similar ly, the word which comes second in the alphabetical order is moved to the extreme right position. The same procedure is continued till all the numbers get rearranged in descending order from the left and all the words get rearranged in alphabetical order after the numbers. STEP Input Step I Step II Step III Step IV Step V Step VI Step VII

    1 84 14 27 32 51 68 84 92

    2 why 84 14 27 32 51 68 84

    3 sit why 84 14 27 32 51 68

    4 14 sit why 84 14 27 32 51

    5 32 32 sit why 84 14 27 32

    6 not not 32 sit why 84 14 27

    7 best ink not not sit why why 14

    25. (3) In Step III, ‘sit’ and ‘not’ are there between ‘why’ and ‘51’ 26. (5) Option (5) is the Step IV. 27. (1) The word ‘ink’ is fifth to the left of ‘best’ in the Step II. 28. (4) 13th from the left in Step V ⇒ not 6th to the left of ‘not’ in Step V ⇒ why 29. (2) The elements ‘27 14 why vain best’ appear in the same order in Step VI. (30–34) : After careful analysis of the given input and various steps of rearrangement, it is evident that only numbers are rearranged upto step III. Then, one word is rearranged in each step after three numbers in the English alphabetical order. The highest number is placed at the extreme right position while the lowest number is placed at the extreme left position in the first step. In the next step the second highest and the second lowest numbers are rearranged. In the final output the three smaller numbers get arranged from the left in ascending order and the three larger numbers get ar ranged after all the words in ascending order. The words are rearranged in alphabetical order. Input : 67 hot sun 19 best 83 ice 49 ace 77 cut 37 Step I : 19 67 hot sun best ice 49 ace 77 cut 37 83

    8 ink feet ink 51 vain vain 92 best

    9 feet 51 51 vain 68 92 best feet

    10 51 27 vain 68 92 best feet ink

    11 27 vain 68 92 best feet ink not

    12 vain 68 92 best feet ink not sit

    13 68 92 best feet ink not sit vain

    14 92 best feet ink not sit vain why

    Step II : 19 37 67 hot sun best ice 49 ace cut 77 83 Step III : 19 37 49 hot sun best ice ace cut 67 77 83 Step IV : 19 37 49 ace hot sun best ice cut 67 77 83 Step V : 19 37 49 ace best hot sun ice cut 67 77 83 Step VI : 19 37 49 ace best cut hot sun ice 67 77 83 Step VII : 19 37 49 ace best cut hot ice sun 67 77 83 30. (4) Seven steps would be needed to complete the rearrangement. 31. (3) It is Step V. 32. (4) Option (4) is Step I. 33. (2) Option (2) is the final arrangement. 34. (1) The word ‘sun’ is at the 7th position from the right in the Step IV. (35-39) : After careful analysis of the given input and various steps of rearrangement it is evident that only numbers are rearranged upto Step III and then words are rearranged. In the first step the lowest and the second lowest numbers are placed at the extreme left and right positions respectively. In the second step the third lowest and the fourth lowest numbers are placed at the extreme left and right positions respectively. In the third step, the second highest and the highest numbers are placed at the extreme left and

    BPRE–488

    INPUT right positions respectively. In the fourth step the words which appear at the first and the second positions in alphbetical order are placed at the extreme left and right positions respectively. Similarly, in the second step, the words which appear at the third and the fourth positions in alphabetical order are placed at the extreme left and right positions respectively. The same procedure is continued in the next step (s). Input : chemical 68 11 reaction 87 is 21 hard to 53 92 detect Step I : 11 chemical 68 reaction 87 is hard to 53 92 detect 21 Step II : 53 11 chemical reaction 87 is hard to 92 detect 21 68 Step III : 87 53 11 chemical reaction is hard to detect 21 68 92 Step IV : chemical 87 53 11 reaction is hard to 21 68 92 detect Step V : hard chemical 87 53 11 reaction to 21 68 92 detect is Step VI : reaction hard chemical 87 53 11 21 68 92 detect is to 35. (2) The elements ‘to 92 detect 21’ are found in the same order in the Step II. 36. (5) The position of ‘21’ from the right end in the last Step is Sixth. 37. (4) The elements ‘87 53 11 reaction’ are found in the same order in both the fourth and fifth steps. 38. (1) 5th to the left of 9th from the left and means 4th from the left end. 4th elements from the left end in the step IV ⇒ 11 39. (3) The element ‘reaction’ is exactly between ‘chemical’ and ‘87’ in the step II. (40–44) : After careful analysis of the given input and various steps of rearrangement it is evident that two elements (one word and one number) are rearranged in each step. In the first step, the lowest number moves to the extreme right position and the word which comes last in the alphabetical series moves to the second position from the right. In the second step, the word which comes second last in the alphabetical order moves to the exteme left position and the second lowest number moves to the second position from the left. The same procedure is continued till all the numbers and words get rearranged. Input

    %

    reach 53 gems busy 84 15 usual 97 power 46 78 decent

    Step I

    : reach 53 gems busy 84 97 power 46 78 decent usual 15

    Step II

    : reach 46 53 gems busy 84 97 power 78 decent usual 15

    Step III : reach 46 gems busy 84 97 78 decent usual 15 power 53

    Step IV : gems 78 reach 46 busy 84 97 decent usual 15 power 53 Step V : gems 78 reach 46 busy 97 usual 15 power 53 decent 84 Step VI : busy 97 gems 78 reach 46 usual 15 power 53 decent 84 40. (4) Step IV gems 78 reach

    46

    busy 84 97 decent usual 15 power 53

    Now, ‘power’ will be to the immediate left of ‘reach’. 41. (1) 7th from the right end in Step III ⇒ 97 9th from the right end in Step III ⇒ busy 42. (3) In Step II ‘97’ is fourth to the right of ‘53’. In Step IV ‘usual’ is fourth to the right of ‘busy’. In Step V ‘usual’ is fourth to the right of ‘reach’. 43. (5) 8th from the right end in Step V ⇒ busy 4th to the right of ‘busy’ in Step V ⇒ ‘power’ 44. (2) Step III reach 46 gems busy 84

    97 78 decent usual 15 power 53

    (45–49) : After careful analysis of the given input and various steps of rearrangement, it is evident that two elements (one word and one number) are rearranged in each step. In the first step, the lowest number moves to the extreme left position while the word which comes last in the dictionary order moves to the extreme right position. In the second step, the highest number moves to the extreme left position while the word which comes first in the dictionary order moves to the extreme right position. The same procedure is continued in the next steps. Input : bin 92 49 air 17 fin log 61 75 mob 22 row Step I : 17 bin 92 49 air fin log 61 75 mob 22 row Step II : 92 17 bin 49 fin log 61 75 mob 22 row air Step III : 22 92 17 bin 49 fin log 61 75 row air mob Step IV : 75 22 92 17 49 fin log 61 row air mob bin Step V : 49 75 22 92 17 fin 61 row air mob bin log Step VI : 61 49 75 22 92 17 row air mob bin log fin 45. (4) In Step IV, ‘log 61 row’ are found consecutively in the same order. 46. (2) Step V 49 75 22

    92

    17 fin 61

    row

    air mob bin log

    There are three elements between ‘92’ and ‘row’ in the Step V. 47. (1) 4th element to the right of the 9th element from the right end means 5th element from the right end. 5th element from the right end in the Step VI ⇒ air 48. (5) In Step I, ‘49’ is third to the left of ‘log’. In Step II, ‘61’ is third to the left of ‘22’. In Step IV, ‘fin’ is third to the left of ‘row’. 49. (3) In Step II, ‘mob’ appears exactly between ‘75’ and ‘22’. ❐❐❐

    BPRE–489

    INPUT

    MODEL EXERCISES Directions (1-3) : A word arrangement machine, when given a particular input, rearranges it following a particular rule. The following is the illustration of the input and the steps of arrangement Input : put pocket hand watch he for them Step I : put for he watch hand pocket them Step II : put he for watch pocket hand them Step III: put hand pocket watch for he them Step IV: put pocket hand watch for he them And so on goes the machine. Study the logic and answer the questions that follow. 1. If step III of a given input be ‘fly sky birds my su fur say’, what is the seventh step of the input? (1) fly sky birds my su fur say (2) fly birds sky my fur su say (3) fly fur su my birds sky say (4) fly su fur my sky birds say (5) None of these 2. If step VII of an input is ‘slow ran dhurwa pat hak dig vi’, what is step V of that input? (1) slow dig hak pat dhurwa ran vi (2) slow hak dig pat ran dhurwa vi (3) slow dhurwa ran pat dig hak vi (4) slow ran dhurwa pat hak dig vi (5) None of these 3. Given the following Input : ana dhir raj ran san rah aji What step will be the following arrangement? Arrangement : ana san rah ran dhir raj aji (1) IV (2) V (3) VI (4) VIII (5) None of these Directions (4-7) : Read the following information carefully to answer these questions. A word number arrangement machine when given, an input as set of words-numbers, rearranges them following a particular rule and generates stepwise output till the rearrangement is completed following that rule. Following is an illustration of input and steps of rearrangement till the last step. Input : pour ask 57 dear 39 fight 17 28 Step I : ask pour 57 dear 39 fight 17 28 Step II : ask 57 pour dear 39 fight 17 28 Step III: ask 57 dear pour 39 fight 17 28 Step IV: ask 57 dear 39 pour fight 17 28 Step V : ask 57 dear 39 fight pour 17 28 Step VI: ask 57 dear 39 fight 28 pour 17 and Step VI is the last output. As per the rule followed in the above steps, find out the answer to each of the following questions. 4. If Step II of an input is “cut 97 38 end for 29 46 down”, which of the following will be the last step? (1) VI (2) IV (3) V (4) VII (5) None of these

    5. If the Step I of an input is “car 17 vas tiger 92 87 like 52”, which of the following will be the Step IV? (1) car 92 like 87 tiger 17 vas 52 (2) car 92 like 87 17 vas tiger 52 (3) car 92 like 87 tiger 52 17 vas (4) car 92 like 17 vas tiger 87 52 (5) None of these 6. Input: zeal for 49 31 hight 22 track 12. Which of the following will be the Step III? (1) for 49 high zeal 31 22 track 12 (2) for 49 high 31 zeal 22 track 12 (3) for 49 high 31 track 22 zeal 12 (4) for 49 high 31 track zeal 22 12 (5) None of these 7. Input : 19 feet 34 28 dog bag 43. “Bag 43 dog 19 feet 34 28” would be the Step (1) I (2) IV (3) II (4) None of these Directions (8–12) : Study the following information carefully and answer the questions given below : When a word and number arrangement machine is given an input line of words and numbers, it rearranges them following a particular rule. The following is an illustration of input and various steps rearrangement. (All the numbers are two digit numbers) Input : 25 93 as hill stamp bolt 45 34 63 total 13 alter rise Step I : alter 25 93 as hill stamp bolt 45 34 63 total rise 13 Step II : alter as 93 hill stamp bolt 45 34 63 total rise 25 13 Step III : alter as bolt 93 hill stamp 45 63 total rise 34 25 13 Step IV: alter as bolt hill 93 stamp 63 total rise 45 34 25 13 Step V : alter as bolt hill rise 93 stamp total 63 45 34 25 13 Step VI: alter as bolt hill rise stamp total 93 63 45 34 25 13 Step VI is the last step of the above arrangement as the intended arrangement is obtained. As per the rules followed in given steps, find out the appropriate steps for the given input. Input for questions : Input : ropes 12 33 strong 35 19 in blue ample kite 47 77 57 8. Which element is sixth to the right of the tenth form the right end of the last step of the given arrangement ? (1) 47 (2) 35 (3) 57 (4) 33 (5) 77 9. What is the position of the ‘ropes’ from the right end of the second last step of the given arrangement ? (1) Tenth (2) Eighth (3) Eleventh (4) Seventh (5) Ninth 10. Which element(s) is/are exatly between ‘strong’ and ‘47’ in the third step of the given arrangement ? (1) ‘kite’ (2) Both ‘in’ and ‘kite’

    BPRE–490

    INPUT (3) Both ‘35’ and ‘kite’ (4) 77 (5) Both ‘blue’ and ‘35’ 11. What is the position of ‘blue’ from the left of ‘strong’ in the fourth step of the given arrangement ? (1) Fifth (2) Third (3) Second (4) Fourth (5) Sixth 12. In which step of the given ar rangement are the elements ‘35 kite 47 77’ found in the same order ? (1) Step III (2) Step II (3) Step IV (4) Step V (5) The given order of elements is not found in any step Directions (13–17) : Study the following information carefully and answer the questions given below : When a word and number arrangement machine is given an input line of words and numbers, it arranges them following a particular rule. The following is an illustration of input and rearrangement. (All the numbers are two digit numbers) Input : 41 sprain 10 early 97 noble 26 65 ankle death Step I : 97 41 10 early noble 26 65 ankle death sprain Step II : noble 97 41 10 early 26 ankle death sprain 65 Step III : 41 noble 97 10 26 ankle death sprain 65 early Step IV : death 41 noble 97 10 ankle sprain 65 early 26 Step V : 10 death 41 noble 97 sprain 65 early 26 ankle Step V is the last step of the above arrangement as the intended arrangement is obtained. As per the rules followed in the given steps, find out the appropriate steps for the given input. Input: omits 36 59 yards 41 elect train 12 lakes 85

    13. Which element comes exactly between ‘85’ and ‘lakes’ in the Step III of the given input? (1) 59 (2) yards (3) elect (4) omits (5) train 14. If in the first step, ‘59’ interchanges its position with ‘lakes’ and ‘omits’ also interchanges its position with ‘12’ then which element will be to the immediate left of ‘41’? (1) 59 (2) omits (3) 12 (4) elect (5) lakes 15. Which of the following combinations represents the first two and the last two elements in the Step V of the given input? (1) 41, train and omits, 36 (2) 12, lakes and 36, elect (3) 12, lakes and omits, 36 (4) lakes, 41 and 36, elect (5) lakes, 41 and 59, omits 16. Which element is fourth to the right of the one which is ninth from the right in the Step V of the given input? (1) yards (2) 36 (3) 85 (4) elect (5) 12 17. In which step are the elements ‘train 85 yards 59’ found in the same order? (1) Third (2) Fourth (3) Second (4) Fifth (5) The given order of elements is not found in any step Directions (18-22) : A word and number arrangement machine when given an input line of words and numbers rearranges them following a particular rule in each step. The following is an illustration of input and rearrangement. Step–I The lowest odd number should be placed at the right end preceded by a word which comes last as per dictionary order. The largest even number should be placed at the left end followed by a word which comes first as per dictionary order. Step–II All the words should be arranged as per dictionary order from right side. Then all the numbers should be arranged according to ascending order from the left side as even numbers come first then odd numbers. Step–III Each number should be followed by each word alternatively in the same order from left to right. Step–IV 1. The 5th element from the left end should be placed at the second position from the right end. 2. The 6th element from the right end should be placed at the second position from the left end. 3. 2nd element from the right end of step I should be placed at 3rd position from left end. 4. 4th element from the left end of step II should be placed at the 3rd position from the right end. 5. 3rd element from the right end of step III should be placed at 4th position from the right end. 855

    428

    Physics

    735

    Zoology

    640

    Step-I

    640

    Geography

    History

    855

    428

    Physics

    Zoology

    735

    Step-II

    428

    640

    735

    855

    Step-III

    428

    Zoology

    640

    Physics

    Step-IV

    428

    640

    Input

    History Geography

    735

    History

    855

    Geography

    855

    735

    Geography

    Zoology Physics History

    Input for questions : Practice 955 Democracy

    BPRE–491

    Zoology Physics

    History Geography

    882 Judicial

    527

    Federal

    656

    INPUT Give answer (1) if only Conclusion I is true Give answer (2) if only Conclusion II is true Give answer (3) if either Conclusion I or Conclusion II is true Give answer (4) if neither Conclusion I nor Conclusion II is true Give answer (5) if both the Conclusion I and Conclusion II are true 18. Statement : With respect to Step – IV Conclusions : I. “Judicial” is third to the left of “527” II. “Democracy” is exactly between “955” and “527” 19. Statement : With respect to Step – III Conclusions : I. “Practice” is fourth from the left end II. “882” is sixth from the right end 20. Statement : With respect to Step – II Conclusions : I. “Federal” is immediate left of an element that is fifth right of “527” II. “Judicial” is immediate right of an element that is fourth right of “656” 21. Statement : With respect to Step – I Conclusions : I. “656” is exactly between “Federal” and “Practice” II. “955” is exactly between “882” and “Judicial” 22. Statement : With respect to Step – III and step-IV Conclusions : I. The element 3rd left of “955” in the step III is the same as the element 2nd left of “955” in step IV II. The fourth right element from “Practice” in step III is the same as 4th right element from “656” in step IV Directions (23–27): A word and number arrangement machine when given an input line of words and numbers rearranges them following a particular rule in each step. The following is an illustration of input and rearrangement.

    Step–I

    Step–II

    Step–III Step–IV

    From the Input, all the words should be arranged from right side of step – I according to the alphabetical order of first alphabet of each word. Then all the numbers should be arranged in ascending order from the left side of step-I as odd numbers come first then even numbers come next. From step-I, the lowest odd number should be placed at the right end of the step-I preceded by a word which comes last in alphabetical order. The largest even number should be placed at the left end of the step-I followed by a word which comes first in alphabetical order. Each word from the step-II should be followed by each number alternatively in the same order from left to right in the step III. 1. The 7th element from the left end of step III should be placed in 3rd position from the right end of step-IV. (in case the element has been moved already, then do not perform this arrangement). 2. The 5th element from the right end of step III should be placed in 3rd position from the left end of step-IV. (in case the element has been moved already, then do not perform this arrangement). 3. Remaining all the elements occupy in the remaining places as in the order in step-III

    Input

    Career

    Capel

    3256

    Coffee

    Step-I

    3265

    3625

    3256

    3526

    Coffee

    Step-II

    3526

    Capel

    3625

    3256

    Carrom Career

    Step-III

    Capel

    3526

    Carrom

    3625

    Career

    Step-IV

    Capel

    3526

    3625

    Carrom

    Career

    3265

    3625

    Carrom

    Carrom Career

    3526 Capel

    Coffee

    3265

    3256

    Coffee

    3265

    Coffee

    3256

    3265

    Input for questions : Muse Mouse 5683 5863 More 5638 5386 Might Give answer (1) if only Conclusion I is true Give answer (2) if only Conclusion II is true Give answer (3) if either Conclusion I or Conclusion II is true Give answer (4) if neither Conclusion I nor Conclusion II is true Give answer (5) if both the Conclusion I and Conclusion II are true 23. Statement : With respect to Step – IV Conclusions : I. “More” is third to the left of “5683” II. “5863” is exactly between “5638” and “Mouse” 24. Statement: With respect to Step – III Conclusions : I. “Mouse” is second to the right of “Might” II. “5386” is second to the left of “Muse” 25. Statement : With respect to Step – II Conclusions : I. “5683” is third to the right of “5386” II. “5386” is fourth to the right of “5638” 26. Statement : With respect to Step – I Conclusions : I. “Might” is second to the right of “Mouse” II. “5863” is fourth to the left of “More”

    BPRE–492

    INPUT 27. Statement : With respect to Step – IV Conclusions : I. “Mouse” is exactly between “5863” and “5638” II. “Might” is second to the right of “Mouse” Directions (28–32) : Study the following information carefully and answer the questions given below : An input–output is given in different steps. Some mathematical operations are done in each step. No mathematical operation is repeated in the next step. Input :

    5 12 11

    7 18 15

    9 24 7

    3 14 33

    Step–I :

    14 42 24

    12 36 20

    14 20 34

    Step–II :

    4 2

    3 6

    3 4 2

    6

    Step–III :

    1 6 11

    12

    Step–IV :

    As per the rules followed in the steps given above, find out in each of the following questions the appropriate step for the given input. Input : 7 8 9

    9 10 11

    5 12 7

    3 4 9

    1 6 5

    28. What is the sum of the first digit of the first set in Step–II and second digit of the second set in Step– III? (1) 3 (2) 4 (3) 2 (4) 1 (5) 5 29. What is the different between the sum of the all numbers in the Step–II and sum of all the third numbers in the Step–I? (1) 12 (2) 14 (3) 16 (4) 10 (5) 8 30. If the output is 10, what is the number in the third set of Step–II? (1) 18 (2) 22 (3) 20 (4) 12 (5) 15 31. If the third number of the first set in the Step–I is replaced with 26, what will be the first number of Step–III? (1) 4 (2) 6 (3) 8 (4) 10 (5) 12 32. If the numbers in the third set of Input are 8–12–10 respectively, what will be the numbers in the second set of Step–I? (1) 17, 20, 14 (2) 14, 20, 17 (3) 14, 16, 17 (4) 17, 20, 17 (5) 17, 14, 19 Directions (33–37) : A word and number arrangement machine when given an input line of words and numbers rearranges them following a particular rule in

    each step. The following is an illustration of input and rearrangement. Step–I First symbol set from the right end should be placed at the left end in the step–I and the next symbol set from the right side should be placed at the right end in the step–I. Step–II All the words should be arranged from left according to the reverse alphabetical order. Then all the numbers should be arranged in descending order. Step–III Each word should be followed by each number alternatively in the same order. Step–IV 1. The second element from the left end in step– III should be placed at second position from the right end. 2. The second element from the right end in step– III should be placed at second position from the left end. 3. 3rd element from the left end in step–I should be placed at the 3rd position from left end. 4. 4th element from the right end in step–II should be placed at 3rd position from the right end (in case the element has been moved already, do not perform this arrangement) 5. 4th element from the left end in step–III should be placed at the 4th position from the right end (in case the element has been moved already, do not perform this arrangement) Input

    higher

    level

    115

    97

    Step–I

    @ # @ higher caps

    level

    115

    97

    49

    #@#

    Step–II

    @#@

    level higher caps

    115

    97

    49

    #@#

    97

    caps 49

    #@#

    caps

    Step–III @ # @

    level

    115

    higher

    Step–IV @ # @

    49

    caps

    97

    49 #@# @#@

    higher 115 level #@#

    Input for questions : 85 super cooler 75 125 + % + @ + @ turbo 33. Which of the following element is third to the left of “125” in the output? (1) + % + (2) 75 (3) Super (4) Cooler (5) 85 34. Leaving the symbol sets, which of the following word or number does occupy more number of times in all the 8 columns? (1) Turbo (2) Cooler (3) 125 (4) 85 (5) None 35. Which of the following element is 4th from the left end of step–II and also 6th from the right end of step– IV? (1) Super (2) + % + (3) Cooler (4) 75 (5) Turbo 36. Which of the following element is 5th to the left of an element that is second from the right end in step– III? (1) 75 (2) Turbo (3) Super (4) @ + @ (5) 125 37. If step–II is followed by step–IV which of the following element is 12th from the right end?

    BPRE–493

    INPUT (1) 85 (2) (4) 125 (5) Directions (38–42) : tion carefully and answer The following is an rearrangement. Input

    %

    Turbo (3) + % + Super Study the following informathe given question : illustration of input and its

    33 cable right 48 board @ shall 116 # & 75

    Step I : board 33 cable right 48 @ shall 75 # & 116 Step II : board 33 c @bl@ right 48 @ shall 75 # & 116 Step III : board c@bl@ right shall 33 48 75 116 @ #& Step IV : board c@bl@ right shall 3 4 7 11 @ #& Input for questions : 75 moon white 84 grow $ ligght 215 % 66 38. Which of the following is second to the right of third element from the right end in the Step–I ? (1) $ (2) 215 (3) grow (4) 75 (5) white 39. What is the position of 75 in the Step–IV ? (1) As it is turned into 5 and Second to the right of 21 (2) As it is turned into 7 and 3rd to the right of grow (3) As it is turned into 5 and 3rd to the right of moon (4) As it is turned into 7 and 3rd to the right of m@@n (5) As it is turned into 5 and 3rd to the right of % 40. If the elements of Step II are written after the elements of Step I in a single row, which of the following will be 15th from the left end ? (1) 75 (2) 66 (3) white (4) $ (5) m@@n 41. Which of the following is seventh element from the left end in Step III ? (1) 66 (2) $ (3) light (4) 75 (5) 84 42. What is the sum of numbers which are at seventh and eight positions respectively from the left end in Step IV ? (1) 13 (2) 15 (3) 28 (4) 29 (5) 27 Directions (43 - 47): A word and number arrangement machine when given an input line of words and numbers rearranges them following a particular rule in each step. The following is an illustration of input and rearrangement. Step -I From the Input, all the words should be ar ranged from right side according to the alphabetical order. Then all the numbers should be arranged in ascending order from the left side as even numbers come first then odd numbers. Step - II The lowest odd number should be placed at the right end of the Step II. The largest even number should be placed at the left end of the step II. All the remaining elements should occupy the blank positions in the same order.

    Step-III

    Each word from the Step II should be followed by each number alternatively in the same order from left to right in the Step III. Step - IV 1. The 4th element from the left end of Step III should be placed at the second position from the right end of Step IV. (in case the element has been moved already, then do not perform this arrangement) 2. The 6th element from the right end of Step III should be placed at the second position from the left end of Step IV. (in case the element has been moved already, then do not perform this arrangement) 3. 3rd element from the right end of Step III should be placed at the 4th position from the right end of Step IV (in case the element has been moved already, then do not perform this arrangement) 4. All the remaining elements should occupy the blank positions in the same order as in the Step III. March January June July 1234 1324 1243 1423 1324 1243 1423 March June July January 1234 1234 1423 March June July January 1243 Step II 1324 1423 January 1243 Step III March 1324 June 1234 July Step IV March June 1324 July 1423 January 1234 1243 Input

    Step I

    Input for the questions Wednesday Saturday Monday Friday 2538 2358 2853 2583 43. Which element is 5th from the right end of Step -I? (1) Wednesday (2) 2583 (3) 2538

    (4) Monday

    (5) 2853 44. Which element is exactly between 2538 and 2358 in Step - IV? (1) 2583 (2) 2853 (3) Monday (4) Wednesday (5) No middle element 45. Which element is 6th from the left end of Step – II ? (1) Monday

    (2) 2853

    (3) 2538

    (4) Saturday

    (5) 2358 46. Which element is third to the left of Friday in Step – III? (1) Saturday

    (2) Friday

    (3) 2358

    (4) 2853

    (5) 2538 47. Which element from Step – II has the same position in Step – IV? (1) Wednesday (2) 2538 (3) 2358 (5) Friday

    BPRE–494

    (4) 2583

    INPUT

    SHORT ANSWERS 1. (1)

    2. (2)

    3. (3)

    4. (3)

    5. (2)

    6. (1)

    7. (4)

    8. (2)

    9. (5)

    10. (3)

    11. (4)

    12. (1)

    13. (3)

    14. (5)

    15. (2)

    16. (1)

    17. (4)

    18. (1)

    19. (2)

    20. (5)

    21. (1)

    22. (5)

    23. (5)

    24. (1)

    25. (4)

    26. (1)

    27. (4)

    28. (3)

    29. (1)

    30. (5)

    31. (2)

    32. (4)

    33. (4)

    34. (5)

    35. (3)

    36. (2)

    37. (4)

    38. (2)

    39. (3)

    40. (1)

    41. (5)

    42. (4)

    43. (5)

    44. (2)

    45. (1)

    46. (3)

    47. (4)

    EXPLANATIONS 1. (1) fly sky birds my su fur say

    Step III. Fly

    sky birds may

    IV. Fly birds sky

    su

    fur

    say

    may

    fur

    su

    say

    sky birds say

    V. Fly

    su

    fur

    may

    VI. Fly

    fur

    su

    may birds sky say

    VII. Fly

    sky birds may

    su

    fur

    say

    2. (2) slow hak dig pat ran dhurwa vi Step V. Slow hak

    dig

    pat

    VI. Slow dig

    hak

    pat dhurwa ran

    VII. Slow ran dhurwa pat

    ran dhurwa vi

    hak

    dig

    vi vi

    nary is placed at the first position from the left and other terms are kept as they are. In the second step, the highest number is placed at the second place keeping rest of the terms as they are and the same process is continued till the rearrangement ends. 4. (3) Step II : cut 97 38 end for 29 46 down Step III: cut 97 down 38 end for 29 46 Step IV: cut 97 down 46 38 end for 29 Step V : cut 97 down 46 end 38 for 29 Hence, step V is the last step. 5. (2) Step I:car 17 vas tiger 92 87 like 52 Step II : car 92 17 vas tiger 87 like 52 Step III: car 92 like 17 vas tiger 87 52 Step IV: car 92 like 87 17 vas tiger 52 6. (1) Input :zeal for 49 31 high 22 track 12 Step I : for zeal 49 31 high 22 track 12 Step II : for 49 zeal 31 high 22 track 12 Step III: for 49 high zeal 31 22 track 12 7. (4) Input :19 feet 34 28 dog bag 43 Step I : bag 19 feet 34 28 dog 43 Step II : bag 43 19 feet 34 28 dog Step III: bag 43 dog 19 feet 34 28 (8–12) : After careful analysis of the given input and various steps of rearrangement, it is evident that in each step two elements (a word and a number) are rearranged according to a certain rule. In the first step, the word which comes first as per the dictionary order is moved to the extreme left position while the lowest number is moved to the extreme right position. In the second step, the word which comes second as per the dictionary order is moved to the second position from the left while the second lowest number is moved to the second position from the right. The same procedure is continued till all the words get rearranged as per the dictionary order from left to right and all the numbers get rearranged after the words in descending order. Input

    3. (3) VI

    : ropes 12 33 strong 35 19 in blue ample kite 47 77 57

    Step I

    : ample ropes 33 strong 35 19 in blue kite 47 77 57 12

    Step II

    : ample blue ropes 33 strong 35 in kite 47 77 57 19 12

    dhir raj aji

    Step III

    : ample blue in ropes strong 35 kite 47 77 57 33 19 12

    dhir rah

    rah san aji

    Step IV

    Step : IV : ana dhir

    raj rah

    san rah aji

    : ample blue in kite ropes strong 47 77 57 35 33 19 12

    Step : V : ana rah

    san rah

    raj dhir aji

    Step : VI : ana san

    rah rah

    dhir raj aji

    1 2 Input : ana dhir

    3 4 raj rah

    5 6 7 san rah aji

    Step : I : ana rah

    san rah

    raj dhir aji

    Step : II : ana san

    rah rah

    raj

    Step : III : ana

    Step V

    : ample blue in kite ropes strong 77 57 47 35 33 19 12 8. (2) 10th from the right end of the last step ⇒ kite

    (4-7) : After careful study of the given input and various steps of rearrangement , we may conclude that in the first step one word which comes first in dictio-

    6th to the right of ‘kite’ ⇒ 35 9. (5) The position of ‘ropes’ from the right end of the second last step is ninth. 10. (3) In Step III, the elements ‘35, kite’ lie between ‘strong’ and ‘47’.

    BPRE–495

    INPUT 11. (4) In the fourth step ‘blue’ is fourth to the left of ‘strong’. 12. (1) The elements ‘35 kite 47 77’ are found in the same order in the Step III. (13–17) : After careful analysis of the given input and various steps of rear rangement, it is evident that in the first step the highest number moves to the extreme left position while the word which comes last in the alphabetical order moves to the extreme right position. In the second step the word which comes second last in the alphabetical order moves to the extreme left position while the second highest number moves to the extreme right position. These two steps are continued alternately till all the numbers and the words get rearranged. Input : omits 36 59 yards 41 elect train 12 lakes 85 Step I : 85 omits 36 59 41 elect train 12 lakes yards Step II : train 85 omits 36 41 elect 12 lakes yards 59 Step III : 41 train 85 36 elect 12 lakes yards 59 omits Step IV : lakes 41 train 85 elect 12 yards 59 omits 36 Step V : 12 lakes 41 train 85 yards 59 omits 36 elect 13. (3) The element ‘elect’ comes exactly between ‘85’ and ‘lakes’ in the Step III. Step III : .... 85 36 elect 12 lakes ... 14. (5) Step I : 85 omits 36 59 41 elect train 12 lakes yards After interchanging the position of elements : Step I : 85 12 36 lakes 41 elect train omits 59 yards Now, the element ‘lakes’ is to the immediate left of ‘41’. 15. (2) First two elements in the Step V ⇒ 12, lakes Last two elements in the Step V ⇒ 36, elect 16. (1) 9th from the right in the Step V ⇒ lakes 4th to the right ‘lakes’⇒ yards 17. (4) The elements ‘train 85 yards 59’ are found in the same order in the Step V.

    (18–22) : Judicial

    527

    Federal

    656

    Judiccial Federal

    656

    Practice

    527

    Input

    Practice

    955

    Democracy

    Step-I

    882

    Democracy

    955

    Step-II

    656

    882

    527

    955

    Step-III

    656

    Practice

    882

    Judicial

    Step-IV

    656

    882

    Practice

    Input

    Muse

    Mouse

    5683

    5863

    Step-I

    5683

    5863

    5386

    5638

    Step-II

    5638

    Might

    5863

    5386

    Step-III

    Might

    5638

    Mouse

    Step-IV

    Might

    5638

    5863

    882

    Practice Judicial Federal Democracy Federal

    955

    Democracy

    955

    527

    Democracy

    More

    5638

    5386

    Might

    Muse

    Mouse

    More

    Might

    Mouse

    More

    Muse

    5683

    5863

    More

    5386

    Muse

    5683

    Mouse

    More

    Muse

    5386

    5683

    527

    Judicial Federal

    (23–27) :

    23. (5) Step-IV

    5863

    Might 5638

    Mouse More Muse 5386 5683

    Both the Conclusions I and II are true. 24. (1) Conclusion I is true. “5386” is to the immediate left of “Muse” in the Step-III. Therefore, Conclusion II is not true. 25. (4) “5683” is fourth to the right of “5386” in the Step-II. Therefore, Conclusion I is not true. “5386” is third to the right of “5638” is the Step-II Therefore, Conclusion II is also not true. 26. (1) Conclusion I is true. “5863” is fifth to the left of “More” in the Step-I. Therefore, Conclusion II is not true. 27. (4) Step-IV Might 5638 5863

    Mouse

    More Muse 5386 5683

    “Might” is third to the left of “Mouse”. Therefore, neither Conclusion I nor Conclusion II follows. (28–32) :

    Input :

    5 12 11

    7 18 15

    14 42 24

    Step–I :

    9 24 7

    12 36 20

    (18 + 24)

    Step–II :

    4

    2

    Step–III :

    (5 + 7)

    6

    2

    (42 – 36)

    (36 – 34)

    Now,

    BPRE–496

    3

    6

    12 (6 × 2)

    Step–IV :

    6 11

    (14 + 6) (33 + 1) (3 + 11)

    (11 + 9)

    3

    1

    14 20 34

    (12 + 24) (15 + 9)

    (7 + 7)

    3 14 33

    4

    Largest number from Step–I

    INPUT Input :

    7

    8

    9

    9 10 11

    5

    16 22 16

    Step–I :

    Step–II :

    4

    9

    1

    8

    2

    2

    Step–III :

    5

    (4 + 6) (3 + 5)

    (9 + 5)

    (7 + 7)

    6

    10 10

    (8 + 12) (11 + 5)

    2

    3

    14 20 14

    (10 + 12) (9 + 7)

    12 7

    0

    (9 + 1)

    1

    2

    0

    10

    20

    Step–IV :

    28. (3) First digit of the first set in Step–II ⇒ 2 Second digit of the second set in Step–III ⇒ 0 Required sum ⇒ 2 + 0 = 2 29. (1) Sum of all the numbers in Step–II = 22 + 20 + 10 = 52 Sum of all the numbers in Step–I = 16 + 14 + 10 = 40 Required difference = 52 – 40 = 12 30. (5) Step–II : 2 2

    2 0

    1 5

    2

    Step–III :

    5

    (22 – 20) (20 – 15) Step–IV :

    10 (2 × 5)

    31. (2)

    Step–I :

    16 22 26 6

    2

    Step–II : Step–III :

    14 20 14 2

    8

    0

    10 10 1

    0

    6 (26 – 20)

    32. (4)

    Input :

    7

    8

    9

    9

    10 11

    8 12 10

    33. (4) Third to the left of “125” in the Step–IV ⇒ cooler 34. (5) There are many elements which appear twice in the column. 35. (3) 4th from the left end in Step–II ⇒ cooler 6th from the right end in Step–IV ⇒ cooler 36. (2) 2nd from the right end in Step–III ⇒ 75 5th to the left of ‘75’ ⇒ turbo 37. (4) There are eight elements in each step. 12th from right end means 4th from the right end in Step–II ⇒ 125 (38–42) : Step I : If the fifth term from the left end is a word/ an alphabet, it is placed at the extreme left end and remaining elements are placed in the same order except the numbers. The numbers are arranged in ascending order. Otherwise, Step I will be the same as the Input. Step II : If in the Step I, the third element has a vowel in the word or it is an odd number, the vowels of the word are replaced with @ or the number is reduced by 3. All the remaining elements should palced as they are. Step III : First of all the words are arranged in alphabetical order, followed by the numbers in ascending order and then the symbols are placed. Step IV : Delete the unit digits of all the numbers. Input Step-I Step-II Step-III

    75 grow grow grow

    moon white 84 grow 66 moon white 75 66 m@@n white 75 light m@@n white 66

    Step-IV

    grow

    light

    m@@n white

    6

    $ $ $ 75

    light light light 84

    215 84 84 215

    % % % $

    66 215 215 %

    7

    8

    21

    $

    %

    38. (2) 3rd element from the right end in Step I ⇒ 84 2nd to the right of 84 ⇒ 215 39. (3) In Step IV unit digit ‘5’ is deleted. ‘7’ is third to the right of ‘m@@n’. 40. (1) Required element ⇒ 5th element from the left end in the Step II. 5th element from the left end in Step II ⇒ 75 41. (5) 7th element from the left end in Step III ⇒ 84 42. (4) 7th element from the left end in Step IV ⇒ 8 8th element from the left end in Step IV ⇒ 21 Required sum ⇒ 8 + 21 = 29 (43–47) : 2853 2358 2583 2538 Wednesday Saturday Monday Friday 2538 2583 Wednesday Saturday Monday Friday 2358 2853 2358 2583 2853 Wednesday Saturday Monday Friday 2538 2538 2583 Friday Monday Saturday Step-III Wednesday 2853 2358 2358 Friday 2583 2538 2853 Step-IV Wednesday Saturday Monday Input

    Step-I

    Step-II

    Step–I :

    43. (5) 5th from the right end in the Step - I ⇒ 2853 44. (2) Step-IV

    17 20 17

    (33–37) : Input Step–I

    2538 Monday 2853 Friday 2358 85

    @+@

    Step–II @ + @ Step–III @ + @ Step–IV @ + @

    super cooler 85

    75

    super cooler

    turbo super cooler

    125

    +%+

    @+@ turbo

    75

    125

    turbo +%+

    125

    85

    75

    +%+

    turbo

    125

    super

    85

    cooler

    75

    +%+

    75

    cooler

    85

    super

    125

    turbo +%+

    45. (1) 6th from the left end in the Step-II ⇒ Monday 46. (3) Third to the left of Friday in the Step-III ⇒ 2358 47. (4) The element ‘2583’ is at the extreme right end in the Steps II, III and IV. ❑❑❑

    BPRE–497

    SYLLOGISM

    11

    SYLLOGISM

    QUESTIONS FROM 1999 TO 2010 ARE AVAILABLE ONLINE NATIONALISED BANKS & IBPS PO/MT/SO EXAMS Directions (1–7) : In each question below are three statements followed by two conclusions numbered I and II. You have to take the three given statements to be true even if they seem to be at variance from commonly known facts and then decide which of the given conclusions logically follows from the three statements disregarding commonly known facts. (Bank Of Baroda PO Exam. 13.03.2011)

    Give answer (1) if only conclusion I follows. Give answer (2) if only conclusion II follows. Give answer (3) if either conclusion I or conclusion II follows. Give answer (4) if neither conclusion I nor conclusion II follows. Give answer (5) if both conclusion I and conclusion II follow. (1–2) : Statements: Some stars are moons. All moons are planets. No planet is universe. 1. Conclusions : I. All moons being stars is a possibility. II. No universe is star. 2. Conclusions : I. At least some planets are stars. II. No moon is universe. (3–4) : Statements : All sticks are plants. All plants are insects. All insects are amphibians. 3. Conclusions : I. At least some amphibians are plants. II. All sticks are insects. 4. Conclusions : I. All amphibians are sticks. II. All plants are amphibians. (5 – 7) : Statements : All apartments are huts. No hut is a building. All buildings are cottages.

    5. Conclusions : I. No cottage is an apartment. II. Some cottages are apartments. 6. Conclusions : I. No apartment is a cottage. II. Some buildings being apartments is a possibility. 7. Conclusions : I. Some cottages being apartments is a possibility. II. No cottage is a hut. Directions (8–15) : In each question below are two/three statements, followed by two conclusions numbered I and II. You have to take the two/three given statements to be true even if they seem to be at variance from commonly known facts and then decide which of the given conclusions logically follows from the given statements disregarding commonly known facts. (Allahabad Bank PO Exam. 17.04.2011) Give answer (1) if only conclusion I follows. Give answer (2) if only conclusion II follows. Give answer (3) if either conclusion I or conclusion II follows. Give answer (4) if neither conclusion I nor conclusion II follows.· Give answer (5) if both conclusion I and conclusion II follow, (8 – 9) : Statements : All buildings are houses. No house is an apartment. All apartments are flats. 8. Conclusions : I. No flat is a house. II. No building is an apartment. 9. Conclusions : I. All buildings being flats is a possibility. II. All apartments being buildings is a possibility.

    BPRE–498

    (10-11) : Statements: Some oceans are seas. All oceans are rivers. No river is a canal. 10. Conclusions : I. All rivers can never be oceans. II. All canals being oceans is a possibility. 11. Conclusions : I. No ocean is a canal. II. Atleast some seas are rivers. (12-13) : Statements : No day is night. All nights are noon. No noon is an evening. 12. Conclusions : I. No day is noon. II. No day is an evening. 13. Conclusions : I. No evenings are nights. II. All days being noon is a possibility. (14–15) : Statements : Some papers are boards. No board is a card. 14. Conclusions : I. No card is a paper. II. Some papers are cards. 15. Conclusions : I. All cards being papers is a possibility. II. All boards being papers is a possibility. Directions (16–20) : In each question below are two/three statements followed by two conclusions numbered I and II. You have to take the two/three given statements to be true even if they seem to be at variance from commonly known facts and then decide which of the given conclusions logically follows from the given statements disregarding commonly known facts. (IBPS Bank PO/MT CWE Exam. 18.09.2011)

    Give answer (1) if only conclusion I follows

    SYLLOGISM Give answer (2) if only conclusion II follows. Give answer (3) if either conclusion I or conclusion II follows. Give answer (4) if neither conclusion I nor conclusion II follows. Give answer (5) if both conclusion I and conclusion II follow. (16 – 17) : Statements : All gliders are parachutes. No parachute is an airplane. All airplanes are helicopters. 16. Conclusions : I. No helicopter is a glider. II. All parachutes being helicopters is a possibility. 17. Conclusions : I. No glider is an airplane. II. All gliders being helicopters is a possibility. 18. Statements : Some mails are chats. All updates are chats. Conclusions : I. All mails being updates is a possibility. II. No update is a mail. (19 – 20) : Statements : No stone is a metal. Some metals are papers. All papers are glass. 19. Conclusions : I. No glass is a metal. II. Atleast some glass is metal. 20. Conclusions : I. All stones being glass is a possibility. II. No stone is a paper. Directions (21-25) : In each of the questions below, two statements are given followed by two conclusions numbered I and II. You have to take the two statements to be true even if they seem to be at variance from the commonly known facts and then decide which of the given conclusions logically follows from the given statements disregarding the commonly known facts. (IBPS RRBs Office Assistant CWE Exam. 09.09.2012)

    Give answer (1) if only conclusion I follows. Give answer (2) if only conclusion II follows. Give answer (3) if either conclusion I or II follows. Give answer (4) if neither conclusion I nor II follows.

    Give answer (5) if both conclusions I and II follow. 21. Statements : All stones are pebbles. All pebbles are rocks. Conclusions : I. All stones are rocks. II. All rocks are pebbles. 22. Statements : All kings are rulers. Some rulers are queens. Conclusions : I. Some kings are queens. II. Atleast some queens are rulers. 23. Statements : Some gardens are parks. Some parks are areas. Conclusions : I. Atleast some areas are parks. II. No garden is an area. 24. Statements : No letter is a fax. All messages are faxes. Conclusions : I. No message is a letter. II. Atleast some faxes are messages. 25. Statements : Some boards are plains. No plain is a square. Conclusions : I. All squares are boards. II. All plains are boards. Directions (26-30) : In each question/group of questions below are two/three statements followed by two conclusions numbered I and II. You have to take the two/three given statements to be true even if they seem to be at variance from commonly known facts and then decide which of the given conclusions logically follows from the given statements disregarding commonly known facts. (IDBI Bank Officer Exam.16.09.2012)

    Give answer (1) If only conclusion I follows Give answer (2) If only conclusion II follows Give answer (3) If either conclusion I or conclusion II follows Give answer (4) If neither conclusion I nor conclusion II follow Give answer (5) If both conclusion I and conclusion II follow 26. Statements : No bank is a school. Some schools are colleges.

    BPRE–499

    27.

    28.

    29.

    30.

    Conclusions : I. Some colleges are definitely not schools. II. All banks being colleges is a possibility. Statements : Some carts are trolleys. All baskets are trolleys. Conclusions : I. Atleast some baskets are carts II. All trolleys are baskets. Directions (28-29) : Statements : All fruits are vegetables. All vegetables are plants. No plant is a root. Conclusions : I. All fruits are plants. II. No root is a vegetable. Conclusions : I. No fruit is a root. II. Atleast some roots are vegetables. Statements : Some calculators are phones. No phone is an eraser. Conclusions : I. No calculator is an eraser II. Some calculators are definitely not phones.

    Directions (31–35) : In each of the following questions two/three statements are given followed by two Conclusions numbered I and II. You have to take the given statements to be true even if they seem to be at variance from commonly known facts. Read both the conclusions and then decide which of the given conclusions logically and definitely follows from the given statements disregarding commonly known facts. (IBPS Specialist Officer CWE Exam.17.03.2013)

    31. Statements All beans are pulses. All pulses are crops. No crop is seed. Conclusions : I. All crops are pulses. II. All beans are crops. (1) Only Conclusion II follows (2) Neither Conclusion I nor Conclusion II follows

    SYLLOGISM (3) Either Conclusion I or Conclusion II follows (4) Only Conclusion I follows (5) Both Conclusion I and Conclusion II follow 32. Statements No fruit is vegetable All potatoes are vegetables. Some fruits are apples. Conclusions : I. Some apples are potatoes II. Some potatoes being fruits is a possibility. (1) Both Conclusion I and Conclusion II follow (2) Only Conclusion II follows (3) Either Conclusion I or Conclusion II follows (4) Only Conclusion I follows (5) Neither Conclusion I nor Conclusion II follows 33. Statements All books are journals. All diaries are journals. Conclusions : I. All journals are books. II. Some diaries being books is a possibility. (1) Either Conclusion I or Conclusion II follows (2) Only Conclusion I follows (3) Both Conclusion I and Conclusion II follow (4) Neither Conclusion I nor Conclusion II follows (5) Only Conclusion II follows 34. Statements No fruit is a vegetable. All potatoes are vegetables. Some fruits are apples. Conclusions : I. No fruit is a potato. II. Atleast some apples are fruits. (1) Both Conclusion I and Conclusion II follow (2) Either Conclusion I or Conclusion II follows (3) Only Conclusion II follows (4) Neither Conclusion I nor Conclusion II follows (5) Only Conclusion I follows

    35. Statements All beans are pulses. All pulses are crops. No crop is a seed. Conclusions : I. No seed is a bean. II. No bean is a pulse. (1) Both Conclusion I and Conclusion II follow (2) Neither Conclusion I nor Conclusion II follows (3) Only Conclusion I follows (4) Either Conclusion I or Conclusion II follows (5) Only Conclusion II follows Directions (36-40) : In each question below are two or three statements followed by two conclusions numbered I and II. You have to take the two given statements to be true even if they seem to be at variance from commonly known facts and then decide which of the given conclusions logically follows from the given statements disregarding commonly known facts. (Indian Overseas Bank PO Online Exam, 01.09.2013)

    Give answer (1) if only conclusion I follows. Give answer (2) if only conclusion II follows. Give answer (3) if either conclusion I or conclusion II follows. Give answer (4) if neither conclusion I nor conclusion II follows. Give answer (5) if both conclusion I and conclusion II follow. (36-37) : Statements : No cow is a bull. All bulls are animals. Some animals are mammals. 36. Conclusions : I. At least some mammals are animals. II. Some mammals being bulls is a possibility. 37. Conclusions : I. At least some animals are bulls. II. No animal is a cow. 38. Statements : Some pencils are pens. No pen is eraser. All sharpeners are erasers.

    BPRE–500

    Conclusions : I. No eraser is a pencil. II. All pencils can never be sharpeners. (39-40) : Statements : Some stars are planets. Some planets are moons. No moon is a sun. 39. Conclusions : I. No star is a sun. II. All planets being suns is a possibility. 40. Conclusions : I. All suns being stars is a possibility. II. Some stars are planets. Directions (41-45) % In each of the question below are given four statements followed by four conclusions numbered I, II, III and IV. You have to assume everything in the statements to be true even if they seem to be at variance from commonly known facts and then decide which of the four given conclusions logically follows from the statements disregarding commonly known facts and select the appropriate answer. (IBPS Bank PO/MT CWE-III, 26.10.2013)

    41.

    42.

    Statements Some numbers are digits. All digits are alphabets. No alphabet is a vowel. All consonants are vowels. Conclusions I. No digit is a vowel. II. No alphabet is a consonant. III. No vowel is a number. IV. All vowels are numbers. (1) Only I and III follow (2) Only II and IV follow (3) Only I and II follow (4) Only I, II and III follow (5) Only I, II and either III or IV follow Statements All documents are files. Some files are papers. All papers are certificates. No certificate is a manuscript. Conclusions I. At least some documents are papers. II. No manuscript is a document. III. At least some certificates are files. IV. No paper is a document.

    SYLLOGISM

    43.

    44.

    45.

    (1) Only I, II and IV follow (2) Only II and III follow (3) Only I and IV follow (4) Only III follows (5) None of these Statements No group is people. All people are animals. All animals are plants. All plants are roots. Conclusions I. No group is a plant. II. All people are plants. III. At least some groups are plants. IV. All plants are groups. (1) Only II follows (2) Only I and IV follow (3) Only II and III follow (4) Only III and IV follow (5) None of these Statements Some jobs are vacancies. All jobs are works. No work is a trade. All professions are trades. Conclusions I. At least some works are jobs. II. No vacancy is a trade. III. No profession is a work. IV. At least some works are professions. (1) All I, II, III and IV follow (2) Only II, III and IV follow (3) Only I and IV follow (4) None follows (5) Only I and III follow Statements All marks are grades. No grade is a score. All letters are scores. All scores are characters. Conclusions : I. At least some letters are grades. II. At least some scores are grades. III. At least some characters are marks. IV. Some characters are letters. (1) Only II and IV follow (2) Only IV follows (3) Only I and III follow (4) All I, II, III and IV follow (5) None follows

    Directions (46–49) : In each of the questions below, three statements are given followed by two conclusions numbered I and II. You have to take the three statements to be true even if they seem to be at variance from the commonly known facts and then decide which of the given conclusions logically follows from the given statements disregarding the commonly known facts. (Corporation Bank SO (Marketing) Exam, 22.02.2014)

    Give answer (1) if only conclusion I follows. Give answer (2) if only conclusion II follows. Give answer (3) if either conclusion I or II follows. Give answer (4) if neither conclusion I nor II follows. Give answer (5) if both conclusions I and II follow. (46-47) : Statements : All diamonds are stones. All stones are gems. No gem is a diamond. 46. Conclusions : I. All gems are stones. II. All diamonds are gems. 47. Conclusions : I. No gem is a diamond. II. No diamond is a stone. 48. Statements : No day is night. All nights are noons. No noon is evening. Conclusions : I. No day is noon. II. No night is evening. 49. Statements : All jackets are trousers. No trouser is shirt. Some shirts are caps. Conclusions : I. Some caps are jackets. II. Some shirts are jackets. Directions (50–55) : In each question below are three statements followed by two conclusions numbered I and II. You have to take the given statements to be true even if they seem to be at variance from commonly known facts and then decide which of the given conclusions logically follows from the given statements disregarding commonly known facts. (BOB Manipal School of Banking Officer Online Exam, 14.08.2014)

    BPRE–501

    Give answer (1) if only conclusion I follows. Give answer (2) if only conclusion II follows. Give answer (3) if either conclusion I or II follows. Give answer (4) if neither conclusion I nor II follows. Give answer (5) if both conclusions I and II follow. (50–51) : Statements No bottle is jar. All cans are jars. All cans are tumblers. 50. Conclusions I. All tumblers are jars. II. All bottles being tumblers is a possibility. 51. Conclusions I. At least some cans are bottles. II. No tumbler is a bottle. (52–53) : Statements Some prints are designs. All designs are copies. All copies are motifs. 52. Conclusions I. At least some prints are motifs. II. All designs are motifs. 53. Conclusions I. At least some copies are prints. II. All motifs being prints is a possibility. (54–55) : Statements All clouds are vapours. No vapour is gas. All gases are rains. 54. Conclusions I. All rains being clouds is a possibility. II. No cloud is gas. 55. Conclusions I. Some rains are not vapours. II. All vapours are clouds. Directions (56–61) : In each question below are three statements followed by two conclusions numbered I and II. You have to take the three given statements to be true even if they seem to be at variance from commonly known facts and then decide which of the given conclusions logically follows from the two statements disregarding commonly known facts. (IDBI Bank Officer Exam, 22.08.2014)

    SYLLOGISM Give answer (1) if only conclusion I follows. Give answer (2) if only conclusion II follows. Give answer (3) if either conclusion I or conclusion II follows. Give answer (4) if neither conclusion I nor conclusion II follows. Give answer (5) if both conclusion I and conclusion II follow. (56–57) : Statements : No paper is book. All books are words. No word is letter. 56. Conclusions : I. No letter is book. II. All letters being paper is a possibility. 57. Conclusions : I. No letter is paper. II. Some papers are definitely not words. (58-59) : Statements : All answers are questions. All doubts are answers. Some doubts are reasons. 58. Conclusions : I. All doubts are questions. II. All anwers are doubts. 59. Conclusions : I. All reasons being questions is a possibility. II. All answers being reasons is a possibility. (60–61) : Statements : Some traps are plans. All plans are ideas. No idea is design. 60. Conclusions : I. All traps are designs. II. At least some traps are ideas. 61. Conclusions : I. All designs being trap is a possibility. II. No design is plan. Directions (62–68) : In each of the questions below are given two or three statements followed by two conclusions numbered I and II. You have to take the given statements to be true even if they seem to be at variance from commonly known facts. Read all the conclusions and then decide which of the given conclusions logically follows from the given statements disregarding commonly known facts. (SIDBI Officer Exam, 03.09.2014)

    Give answer (1) if only Conclusion I follows. Give answer (2) if only Conclusion II follows. Give answer (3) if either Conclusion I or II follows. Give answer (4) if neither Conclusion I nor II follows. Give answer (5) if both Conclusions I and II follow. (62-63) : Statements All shapes are figures. No figure is digit. Some digits are numbers. 62. Conclusions : I. At least some numbers are figures. II. All numbers being shape is a possibility. 63. Conclusions : I. No number is a shape. II. No shape is a digit. (64-65) : Statements : No magnet is insulator. All conductors are insulators. All resistors are conductors. 64. Conclusions : I. At least some magnets are resistors. II. All resistors are insulators. 65. Conclusions : I. No magnet is a conductor. II. All insulators are conductors. (66-67) : Statements Some reagents are chemicals. All chemicals are elements. Some elements are substances. 66. Conclusions : I. All substances being reagents is a possibility. II. At least some elements are reagents. 67. Conclusions : I. All substances being chemicals is a possibility. II. No substance is a reagent. 68. Statements : Some computers are keyboards. Some keyboards are scanners. Conclusions : I. No scanner is a computer. II. At least some computers are scanners. Directions (69-73) : In each question below are three statements followed by two conclusions numbered

    BPRE–502

    I and II. You have to take the given statements to be true even if they seem to be at variance from commonly known facts and then decide which of the given conclusions logically follows from the given statements disregarding commonly known facts. (IBPS RRBs Officer Scale-I CWE, 06.09.2014)

    Give answer (1) if only Conclusion I follows. Give answer (2) if only Conclusion II follows. Give answer (3) if either Conclusion I or II follows. Give answer (4) if neither Conclusion I or II follows. Give answer (5) if both Conclusions I and II follow. (69-70) : Statements All erasers are sharpeners. All sharpeners are pencils. Some pencils are pens. 69. Conclusions I. At least some sharpeners are pens. II. No sharpener is a pen. 70. Conclusions I. No eraser is a pen. II. All pencils are sharpeners. 71. Statements All railways are trains. No train is station. Some stations are platforms. Conclusions I. All railways being platforms is a possibility. II. No railway is station. (72-73) : Statements All winters are summers. Some summers are springs. No spring is an autumn. 72. Conclusions I. At least some winters are summers. II. Some autumns being summers is a possibility. 73. Conclusions I. All summers can never be autumn. II. At least some summers are winters. Directions (74-79) : In each of the following questions are given four statements followed by five conclusions given as five alternative choic-

    SYLLOGISM es. You have to assume every thing in the statements to be true even if they seem to be at variance from commonly known facts and then decide which of the given conclusions does not logically follows from the information given in the statements disregarding commonly known facts. The conclusion which does not follow is your answer. (IBPS Bank PO/MT CWE-IV, 18.10.2014)

    74. Statements All rivers are oceans. All oceans are ponds. No pond is stream. All streams are canals. (1) At least some rivers are not streams. (2) Some canals are not rivers. (3) All rivers are ponds. (4) No ocean is stream. (5) All rivers being canal is a possibility. 75. Statements All colours are brushes. All paints are brushes. All colours are inks. All inks are dyes. (1) At least some brushes are paints. (2) All colours are dyes. (3) Some paints are not brushes. (4) At least some inks are brushes. (5) At least some dyes are inks. 76. Statements No talk is speech. All lectures are talks. All addresses are speeches. All classes are addresses. (1) All speeches being talks is a possibility. (2) No address is lecture. (3) At least some speeches are classes (4) No lecture is speech. (5) Some addresses are not talks. 77. Statements All shelters are dens. Some dens are houses. All houses are buildings. No building is nest. (1) Some buildings are not nests. (2) At least some building are houses.

    (3) At least some dens are buildings. (4) All shelters are houses. (5) At least some houses are not nests. 78. Statements Some ends are terminals. All terminals are stops. All stops are posts. All posts are locations. (1) All terminals being locations is a possibility. (2) No terminal is end. (3) At least some ends are stops. (4) Some ends are locations. (5) At least some posts are terminals. 79. Statements All cities are towns. All towns are villages. All villages are districts. All lanes are cities. (1) All towns are districts. (2) All lanes are towns. (3) All cities being villages is a possibility. (4) All lanes can never be towns. (5) All cities are districts. Directions (80 – 85) : In these questions, two/three statements followed by two conclusions numbered I and II have been given. You have to take the given statements to be true even if they seem to be at variance from the commonly known facts and then decide which of the given conclusions logically follows from the given statements disregrading commonly known facts. (Bank of Baroda Junior Management Grade/Scale-I Exam, 18.04.2015)

    Give answer (1) If either Conclusion I or conclusion II follows Give answer (2) If both Conclusion I and Conclusion II follow Give answer (3) If only Conclusion II follows Give answer (4) If neither Conclusion I nor Conclusion II follows Give answer (5) If only Conclusion I follows 80. Statements : All drivers are swimmers. Some swimmers are athletes. No athlete is a banker. Conclusions I. All swimmers being bankers is a possibility. II. No driver is a banker.

    BPRE–503

    (81-82) : Statements : All frogs are amphibians. Some turtles are amphibians. All turtles are reptiles. 81. Conclusions : I. Atleast some amphibians are reptiles II. No frog is a turtle. 82. Conclusions : I. All frogs being turtles is a possibility. II. No reptile is a frog. 83. Statements : All kings are warriors. Some dukes are kings. Conclusions : I. All kings are dukes. II. Atleast some dukes are warriors. (84-85) : Statements : Some plants are trees. All trees are weeds. All weeds are shrubs. 84. Conclusions : I. All plants are weeds. II. Some plants are weeds. 85. Conclusions : I. All trees are shrubs. II. All shrubs being plants is a possibility. Directions (86 – 92) : In these questions, two/three statements followed by two conclusions numbered I and II have been given. You have to take the given statements to be true even if they seem to be at variance from the commonly known facts and then decide which of the given conclusions logically follows from the given statements disregrading commonly known facts. (BOB Junior Management Grade/Scale–I Exam. 18.04.2015)

    Give answer (1) If either Conclusion I or conclusion II follows Give answer (2) If both Conclusion I and Conclusion II follow Give answer (3) If only Conclusion II follows Give answer (4) If neither Conclusion I nor Conclusion II follows Give answer (5) If only Conclusion I follows 86. Statements : All drivers are swimmers. Some swimmers are athletes. No athlete is a banker.

    SYLLOGISM Conclusions I. All swimmers being bankers is a possibility. II. No driver is a banker. (87-88) : Statements : All frogs are amphibians. Some turtles are amphibians. All turtles are reptiles. 87. Conclusions : I. Atleast some amphibians are reptiles II. No frog is a turtle. 88. Conclusions : I. All frogs being turtles is a possibility. II. No reptile is a frog. 89. Statements : All kings are warriors. Some dukes are kings. Conclusions : I. All kings are dukes. II. Atleast some dukes are warriors. (90-91) : Statements : Some plants are trees. All trees are weeds. All weeds are shrubs. 90. Conclusions : I. All plants are weeds. II. Some plants are weeds. 91. Conclusions : I. All trees are shrubs. II. All shrubs being plants is a possibility. 92. Which of the following is definitely true if the statements given below are considered to be true? (You have to take the given statements to be true even if they seem to be at variance from commonly known facts and then decide which of the given Conclusions logically follows from the given statements disregarding commonly known facts.) No kite is a bird. All reptiles are birds. All kites are amphibians. All amphibians are plants. (1) All amphibians being birds is a possibility. (2) All plants being reptiles is a possibility. (3) At least some reptiles are amphibians.

    (4) All kites are plants. (5) At least some plants are reptiles (IBPS RRBs Officer Scale–I & II CWE 12.09.2015)

    Directions (93–96) : In each of the following questions three statements followed by two Conclusions numbered I and II have been given. You have to take the given statements to be true even if they seem to be at variance from the commonly known facts and then decide which of the given Conclusions logically follows from the given statements, disregarding commonly known facts. (IBPS RRBs Officer Scale–I & II CWE 12.09.2015)

    Give answer (1) if both the Conclusion I and Conclusion II follow Give answer (2) if neither Conclusion I nor Conclusion II follows Give answer (3) if either Conclusion I or Conclusion II follows Give answer (4) if only Conclusion I follows Give answer (5) if only Conclusion II follows (93-94) : Statements Some ratios are percent. All percent are fractions. No fraction is a section. 93. Conclusions I. No section is a percent. II. All ratios being fractions is a possibility 94. Conclusions I. All sections being ratios is a possibility. II. At least some fractions are ratios. (95-96) : Statements All metals are plastics. All plastics are ores. Some ores are wood. 95. Conclusions I. All wood being metals is a possibility. II. No ore is a metal. 96. Conclusions I. At least some metals are wood. II. All plastics being wood is a possibility. Directions (97–101) : In each of the questions given below two/three statements followed by two Conclusions numbered I and II have been giv-

    BPRE–504

    en. You have to take the two/three given statements to be true even if they seem to be at variance from commonly known facts and then decide which of the given Conclusions logically follows from the given statements disregarding commonly known facts. (IBPS Bank PO/MT CWE–V (Preliminary) 03.10.2015)

    Give answer (1) if only Conclusion II follows Give answer (2) if only Conclusion I follows Give answer (3) if both the Conclusion I and Conclusion II follow Give answer (4) if either Conclusion I or Conclusion II follows Give answer (5) if neither Conclusion I nor Conclusion II follows 97. Statements : All races are sprints. Some races are contests. Conclusions : I. Some contests are sprints. II. All contests are sprints. 98. Statements : No bank is a locker. All banks are stores. No store is panel. Conclusions : I. No store is a locker. II. No panel is a bank. (99–100) : Statements : Some strikes are hits. No strike is a raid. All attacks are raids. 99. Conclusions : I. Some hits are definitely not raids. II. All hits being strikes is a possibility. 100. Conclusions : I. No attack is a strike. II. All attacks being hits is a possibility. 101. Statements : Some equations are formulae. All equations are terms. All terms are symbols. Conclusions : I. All equations are symbols. II. No symbol is a formula. Directions (102–106) : In each of the following questions, two or three statements followed by two Conclusions numbered I and II have been given. You have to take the given State-

    SYLLOGISM ments to be true even if they seem to be at variance from the commonly known facts and then decide which of the given Conclusions logically follows from the given statements disregarding commonly known facts. (IBPS Bank PO/MT CWE–V (Preliminary) 04.10.2015)

    Give answer (1) if both the Conclusion I and Conclusion II follow Give answer (2) if either Conclusion I or Conclusion II follows Give answer (3) if neither Conclusion I nor Conclusion II follows Give answer (4) if only Conclusion I follows Give answer (5) if only Conclusion II follows (102–103) : Statements All calls are mails. Some mails are posts. Some posts are letters. 102. Conclusions I. All posts being calls is a possibility. II. No letter is a mail. 103. Conclusions I. All mails are calls. II. No call is a letter. 104. Statements Some vehicles are cars. Some cars are trucks. All trucks are sedans. Conclusions I. All vehicles being sedans is a possibility. II. At least some cars are sedans. 105. Statements Some bridges are roads. No road is underpass. Conclusions I. Some bridges are underpasses. II. No bridge is an underpass. 106. Statements No unit is a part. All parts are items. Some items are elements. Conclusions I. No unit is an element. II. At least some units are items. Directions (107–111) : In each of the following questions two or three statements followed by two Conclusions numbered I and II are given. You have to take the given statements to be true even if they seem to be at vari-

    ance from the commonly known facts and then decide which of the given Conclusions logically follows from the given statements disregarding commonly known facts. (IBPS Bank PO/MT CWE–V (Preliminary.) 04.10.2015)

    Give answer (1) if neither Conclusion I nor Conclusion II follows. Give answer (2) if either Conclusion I or Conclusion II follows. Give answer (3) if only Conclusion I follows. Give answer (4) if both the Conclusion I and Conclusion II follow. Give answer (5) if only Conclusion II follows. (107–108) : Statements Some tasks are hurdles. All hurdles are jobs. Some jobs are works. 107. Conclusions I. All works being hurdles is a possibility. II. At least some works are tasks. 108. Conclusions I. Some jobs are tasks. II. All jobs are tasks. 109. Statements Some problems are solutions. No solution is a trick. All rules are tricks. Conclusions I. No rule is a solution. II. Some problems are definitely not tricks. 110. Statements All ministers are deans. Some deans are heads. Some heads are principals. Conclusions I. No principal is a minister. II. All heads being ministers is a possibility. 111. Statements No queue is a line. Some queues are rows. Conclusions I. No row is a line. II. Some rows are lines. Directions (112-118) : In each of the following questions, two/three statements followed by two Conclusions numbered I and II are given. You have to take the given statements to be true even if they seem to be at variance from the commonly known facts

    BPRE–505

    and then decide which of the given Conclusions logically follows from the given statements disregarding commonly known facts. (IBPS Bank PO/MT CWE–V (Preliminary) 10.10.2015 Ist Sitting)

    Give answer (1) if only Conclusion I follows Give answer (2) if only Conclusion II follows Give answer (3) if either Conclusion I or Conclusion II follows. Give answer (4) if neither Conclusion I nor Conclusion II follows Give answer (5) if both the Conclusion I and Conclusion II follow 112. Statements Some slides are photos. All photos are images. All images are creations. Conclusions I. At least some images are slides. II. All photos are creations. 113. Statements No space is a gap. All fissures are gaps. No gap is a crack. Conclusions I. No space is crack. II. No fissure is a crack. 114. Statements No loss is a profit. Some profits are gains. Conclusions I. No gain is a loss. II. Some gains are losses. (115-116) : Statements All points are views. No view is an idea. Some ideas are thoughts. 115. Conclusions I. Some thoughts being points is a possibility. II. No view is a thought. 116. Conclusions I. At least some ideas are points. II. All thoughts being ideas is a possibility. Directions (117–121) : In each of the following questions, two/three statements followed by two Conclusions numbered I and II have been given. You have to take the given statements to be true even if they seem to be at variance from the commonly

    SYLLOGISM known facts and then decide which of the given Conclusions logically follows from the given statements disregarding commonly known facts. (IBPS Bank PO/MT CWE–V (Preliminary) 10.10.2015)

    Give answer (1) if both the Conclusion I and Conclusion II follow Give answer (2) if either Conclusion I or Conclusion II follows Give answer (3) if neither Conclusion I nor Conclusion II follows Give answer (4) if only Conclusion I follows Give answer (5) if only Conclusion II follows (117–118) : Statements : No ground is a soil. All soils are basins. Some basins are deltas. 117. Conclusions : I. No delta is a soil. II. Some grounds being deltas is a possibility. 118. Conclusions : I. At least some soils are deltas. II. All basins are soils. 119. Statements : All policies are decisions. No decision is a verdict. No verdict is a result. Conclusions : I. All results being policies is a possibility. II. No verdict is a policy. 120. Statements : Some calculators are machines. No calculator is a phone. Conclusions : I. Some machines are phones. II. No machine is a phone. 121. Statements : All seasons are winters. Some winters are autumns. All autumns are falls. Conclusions : I. At least some falls are winters. II. At least some autumns are seasons. Directions (122–126) : In each of the following questions two or three statements followed by two Conclusions numbered I and II are given. You have to take the given statements to be true even if they seem to be at variance from the commonly known facts and then decide which of the given Con-

    clusions logically follows from the given statements disregarding commonly known facts. (IBPS RRBs Officer Scale–I & II CWE 13.09.2015)

    Give answer (1) if both the Conclusion I and Conclusion II follow Give answer (2) if neither Conclusion I nor Conclusion II follows Give answer (3) if only Conclusion I follows Give answer (4) if either Conclusion I or Conclusion II follows Give answer (5) if only Conclusion II follows (122-123) : Statements : Some letters are digits. All digits are numbers. All symbols are numbers. 122. Conclusions : I. At least some letters are numbers. II. All symbols being digits is a possibility. 123. Conclusions : I. No letter is a symbol. II. All letters being digits is a possibility. (124-125) : Statements : No point is a spot. All spots are marks. No mark is a dot. 124. Conclusions : I. All points being marks is a possibility. II. No spot is a dot. 125. Conclusions : I. All dots being points is a possibility. II. At least some points are marks. 126.Which of the following is definitely true if the statements given below are considered to be true ? (You have to take the given statements to be true even if they seem to be at variance from commonly known facts and then decide which of the given conclusions logically follows from the given statements disregarding commonly known facts.) All farms are houses. All houses are jungles. No jungle is a ranch. All ranches are estates.

    BPRE–506

    (1) No jungle is an estate (2) No jungle is a farm (3) All estates being houses is a possibility (4) No farm is a ranch. (5) All ranches being houses is a possibility (IBPS RRBs Officer Scale–I & II CWE 13.09.2015)

    Directions (127-131) : In each of the following questions three statements followed by two conclusions numbered I and II have been given. You have to consider the given statements to be true even if they seem to be at variance with the commonly known facts and then decide which of the given conclusions logically follows from the given statements disregarding commonly known facts. (IBPS Bank PO/MT CWE–V Main Exam. 31.10.2015)

    Give answer (1) if both the Conclusions I and II follow Give answer (2) if either Conclusion I or Conclusion II follows Give answer (3) if neither Conclusion I nor Conclusion II follows Give answer (4) if only Conclusion I follows Give answer (5) if only Conclusion II follows 127. Statements % No meeting is an argument. All debates are arguments. Some debates are fights. Conclusions : I. No fight is a meeting. II. Some fights are meetings. 128. Statements % All hands are limbs. All limbs are fingers. Some fingers are thumbs. Conclusions : I. Some thumbs being limbs is a possibility. II. All hands are fingers. 129. Statements % All teams are participants. All members are teams. No member is a captain. Conclusions : I. Atleast some participants are members. II. All teams being captains is a possibility.

    SYLLOGISM 130. Statements % Some slopes are mountains. No mountain is a river. Some rivers are ponds. Conclusions : I. All ponds being mountains is a possibility. II. All slopes being rivers is a possibility. 131. Statements % No gate is a door. All doors are walls. No wall is a ceiling. Conclusions : I. At least some gates are ceilings. II. No ceiling is a door. Directions (132–136) : In each of the following questions, two or three statements followed by two Conclusions numbered I and II have been given. You have to take the given statements to be true even if they seem to be at variance from the commonly known facts and then decide which of the given Conclusions logically follows from the given statements disregarding commonly known facts. (IBPS Specialist Officer (Marketing) CWE 01.02.2016) Give answer (1) if both the Conclusion I and Conclusion II follow Give answer (2) if either Conclusion I or Conclusion II follows Give answer (3) if neither Conclusion I nor Conclusion II follows Give answer (4) if only Conclusion I follows Give answer (5) if only Conclusion II follows (132–133) : Statements All crafts are projects. Some projects are missions. No mission is a guide. 132. Conclusions I. Some projects are guides. II. No project is a guide. 133. Conclusions I. No guide is a craft. II. Atleast some missions are crafts. (134–135) : Statements Some outputs are results. All outputs are products. All products are yields.

    134. Conclusions I. No product is a result. II. All yields are products. 135. Conclusions I. All outputs are yields. II. All results being yields is a possibility. 136. Statements No price is a rate. All rates are expenses. Conclusions I. No expense is a price. II. All prices being expenses is a possibility. Directions (137–141) : In each of the following questions, two or three statements followed by two Conclusions numbered I and II have been given. You have to take the given statements to be true even if they seem to be at variance from the commonly known facts and then decide which of the given Conclusions logically follows from the given statements disregarding commonly known facts. Give answer (1) if only Conclusion I follows Give answer (2) if either Conclusion I or Conclusion II follows Give answer (3) if only Conclusion II follows Give answer (4) if neither Conclusion I nor Conclusion II follows Give answer (5) if both the Conclusion I and Conclusion II follow (137–138) : Statements All fares are prices. Some prices are costs. All costs are tariffs. 137. Conclusions I. No fare is a tariff. II. No cost is a fare. 138. Conclusions I. Atleast some prices are tariffs. II. All tariffs being prices is a possibility. 139. Statements No proof is an evidence. No proof is an indication. Conclusions I. All indications being evidences is a possibility. II. No evidence is an indication.

    BPRE–507

    (140–141) : Statements Some tests are exams. No exam is a challenge. Some challenges are wins. 140. Conclusions I. No test is a win. II. Atleast some tests are wins. 141. Conclusions I. All tests can never be challenges. II. No win is an exam. Direction (142–147) : In each of the following questions, two or three statements followed by two Conclusions numbered I and II have been given. You have to take the given statements to be true even if they seem to be at variance from the commonly known facts and then decide which of the given Conclusions logically follows from the given statements disregarding commonly known facts. (SIDBI Officer Online Exam.24.02.2016) Give answer (1) if both the Conclusion I and Conclusion II follow Give answer (2) if either Conclusion I or Conclusion II follows Give answer (3) if neither Conclusion I nor Conclusion II follows Give answer (4) if only Conclusion I follows Give answer (5) if only Conclusion II follows (142–143) : Statements Some pictures are images. All images are memories. Some memories are sketches. 142. Conclusions I. Aleast some pictures are memories. II. Some memories are definitely not pictures. 143. Conclusions I. All images are sketches. II. All pictures being sketches is a possibility. (144–145) : Statements All mountains are peaks. Some peaks are rivers. No mountain is a valley. 144. Conclusions I. All rivers can never be mountains. II. All mountains are rivers. 145. Conclusions I. No valley is a peak. II. Atleast some valleys are rivers.

    SYLLOGISM 146. Statements Some hours are minutes. Some hours are seconds. Conclusions I. Atleast some seconds are minutes. II. No second is a minute. 147. Statements All cars are trucks. All trucks are vehicles. Conclusions I. All vehicles ate trucks. II. All cars are vehicles. Directions (148–153) : In these questions, three statements followed by two Conclusions numbered I and II have been given. You have to take the given Statements to be true even if they seem to be at variance from the commonly known facts and then decide which of the given Conclusions logically follows from the given Statements disregarding commonly known facts. (United Bank of India PGDBF Manipal Exam,07.08.2016)

    Give answer (1) if either Conclusion I or Conclusion II follows Give answer (2) if neither Conclusion I nor Conclusion II follows Give answer (3) if only Conclusion I follows Give answer (4) if both the Conclusion I and Conclusion II follow Give answer (5) if only Conclusion II follows (148–149) : Statements : No drum is a book. All phones are books. All caps are phones. 148. Conclusions : I. Atleast some phones are drums. II. No cap is a drum. 149. Conclusions : I. All caps are books. II. All drums being cap is a possibility. (150–151) : Statements : Some cows are goats. All goats are deer. All deer are buffaloes. 150. Conclusions : I. At least some deer are cows. II. All buffaloes being cow is a possibility.

    151. Conclusions : I. At least some cows are buffaloes. II. All goats are buffaloes. (152–153) : Statements : All pencils are rubbers. No rubber is a pen. All pens are stickers. 152. Conclusions : I. All rubbers being stickers is a possibility. II. All rubbers are pencils. 153. Conclusions : I. All stickers being pencils is a possibility. II. No pencil is a pen. Directions (154–159) : In these questions three statements followed by two conclusions I and II have been given. You have to take the given statements to be true even if they seem to be at variance from commonly known facts and then decide which of the given conclusions logically follows from the given statements disregarding commonly known facts. (Bank of Baroda Exam, 25.09.2016)

    Give answer (1) if only conclusion I is true Give answer (2) if both conclusion I and II are true Give answer (3) if either conlusion I or II is true Give answer (4) if only conclusion II is true Give answer (5) if neither conclusion I nor II is true 154. Statements : Some penguins are peacocks. Some peacocks are ducks. Some ducks are swans. Conclusions : I. All ducks can never be penguins. II. Some swans being peacocks is a possibility. 155. Statements : Some chocolates are pizzas. All pizzas are burgers. All burgers are ice-creams. Conclusions : I. Atleast some chocolates are burgers. II. All ice-creams being chocolates is a possibility.

    BPRE–508

    156. Statements : No wall is floor. All floors are foofs. Some roofs are windows. Conclusions : I. Some floors are definitely not windows. II. Atleast some roofs are walls. 157. Statements : All plugs are wires. Some wires are switches. All switches are batteries. Conclusions : I. All plugs being switches is a possibility. II. Atleast some batteries are plugs. 158. Statements : Some cakes are biscuits. All biscuits are pastries Some pastries are doughnuts. Conclusions : I. All doughnuts being cakes is a possibility. II. Atleast some cakes are pastries. 159. Statements : No green is yellow. All yellow are red. All red are blue. Conclusions : I. Atleast some blues are yellows. II. All blues being greens is a possibility. Directions (160 – 164) : In these questions, two or three statements followed by two Conclusions numbered I and II have been given. You have to take the given statements to be true even if they seem to be at variance from the commonly known facts and then decide which of the given Conclusions logically follows from the given statements disregarding commonly known facts. (IBPS Bank PO/MT CWE (Pre Exam), 16.10.2016 (First Sitting))

    Give answer (1) if either Conclusion I or Conclusion II follows Give answer (2) if both the Conclusion I and Conclusion II follow Give answer (3) if only Conclusion I follows Give answer (4) if only Conclusion II follows Give answer (5) if neither Conclusion I nor Conclusion II follows

    SYLLOGISM (160–161) : Statements No biscuit is a cookie. All cookies are pastries. Some pastries are sandwiches. 160. Conclusions I. All pastries are cookies. II. All biscuits being pastries is a possibility. 161. Conclusions I. Atleast some cookies are sandwiches. II. Some sandwiches are biscuits. (162–163) : Statements Some keys are locks. Some locks are drawers. All drawers are tables. 162. Conclusions I. No key is a drawer. II. Atleast some keys are drawers. 163. Conclusions I. All keys can never be tables. II. Atleast some locks are tables. 164. Statements All frames are pictures. Some pictures are images. Conclusions I. Some frames are images. II. All frames are images. Directions (165-169) : In each question below are given two/three statements followed by two conclusions numbered I and II. You have to take the given statements to be true even if they seem to be at variance with commonly known facts. Read all the conclusions and then decide which of the given conclusions logically follows from the given statements, disregarding commonly known facts. (IBPS Bank PO/MT CWE-VI (Pre Exam), 16.10.2016 (Second Sitting))

    Give answer (1) if only Conclusion I follows. Give answer (2) if only Conclusion II follows. Give answer (3) if either Conclusion I or II follows. Give answer (4) if neither Conclusion I nor II follows. Give answer (5) if both Conclusions I and II follow. (165–166) : Statements: All circles are a triangles. Some triangles are rectangles. All rectangles are squares.

    165. Conclusions : I. All rectangles being triangles is a possibility. II. All circles being square is a possibility. 166. Conclusions : I. Some triangles are not rectangles. II. No square is a circle. (167–168) : Statements: Some chairs are tables. Some beds are tables. No furniture is bed. 167. Conclusions : I. All chairs being furniture is a possibility. II. Some tables are not beds is a possibility. 168. Conclusions : I. Some tables are not furniture. II. All tables being furniture is a possibility. 169. Statements : All arts are theatres. Some arts are dramas. Conclusions : I. All dramas being theatres is a possibility. II. Some dramas are theatres. Directions (170–174) : In these questions two or three statements followed by two Conclusions numbered I and II are given. You have to take the given statements to be true even if they seem to be at variance from commonly known facts and then decide which of the given Conclusions logically follows disregarding commonly known facts. (IBPS Bank PO/MT (Pre.) Exam, 23.10.2016)

    Give answer (1) if neither Conclusion I nor Conclusion II follows Give answer (2) if only Conclusion I follows Give answer (3) if only Conclusion II follows Give answer (4) if both the Conclusion I and Conclusion II follow Give answer (5) if either Conclusion I or Conclusion II follows (170–171) : Statements : All packets are envelopes. No envelop is a gift. Some gifts are boxes. 170. Conclusions : I. All envelopes are packets. II. All boxes can never be envelopes.

    BPRE–509

    171. Conclusions : I. All packets being boxes is a possibility. II. No packet is a gift. (172–173) : Statements : All diaries are novels. All novels are biographies. Some biographies are scripts. 172. Conclusions : I. Atleast some diaries are scripts. II. No diary is a script. 173. Conclusions : I. All diaries are biographies. II. Some scripts are definitely not novels. 174. Statements: Some days are months. Some months are weeks. Conclusions : I. Some weeks are days. II. No week is a day. Directions (175–179) : In these questions, three statements followed by two Conclusions numbered I and II have been given. You have to take the given statements to be true even if they seem to be at variance from the commonly known facts and then decide which of the given Conclusions logically follows from the given statements disregarding commonly known facts : (Bank of Maharashtra PO Exam, 26.10.2016)

    Give answer (1) if either Conclusion I or Conclusion II follows Give answer (2) if only Conclusion I follows Give answer (3) if both the Conclusion I and Conclusion II follow II Give answer (4) if only Conclusion II follows Give answer (5) if neither Conclusion I nor Conclusion II follows (175–176) : Statements : Some pens are erasers. Some erasers are markers. All markers are lighters. 175. Conclusions : I. Atleast some erasers are lighters. II. Some lighters being pens is a possibility. 176. Conclusions : I. All pens are markers. II. Some pens are markers.

    SYLLOGISM (177–179) : Statements : No mail is a notice. All notices are intimations. Some intimations are warnings. 177. Conclusions : I. No mail is a warning. II. Atleast some mails are warnings. 178. Conclusions : I. All intimations are notices. II. Some notices being warnings is a possibility. 179. Statements : All seconds are minutes. All minutes are hours. Some hours are days. Conclusions : I. Some days are definitely not minutes. II. All seconds are hours. Directions (180–184) : In these questions, two/three statements followed by two Conclusions numbered I and II are given. You have to take the given statements to be true even if they seem to be at variance from commonly known facts and then decide which of the given conclusions logically follows disregarding commonly known facts. (IBPS RRBs Officer CWE (Pre.) Exam, 14.11.2016 (Shift-I))

    Give answer (1) if either Conclusion I or Conclusion II follows Give answer (2) if neither Conclusion I nor Conclusion II follows Give answer (3) if only Conclusion I follows Give answer (4) if both the Conclusions I and II follow Give answer (5) if only Conclusion II follows (180–181) : Statements : All copies are books. No book is a pencil. All pencils are rubbers. 180. Conclusions : I. No rubber is a book. II. Some copies are rubbers. 181. Conclusions : I. No copy is a pencil. II. Some books are rubbers. (182–183) : Statements : Some mobiles are calculators. Some calculators are pens. Some pens are scales.

    182. Conclusions : I. No calculator is a scale. II. Atleast some calculators are scales. 183. Conclusions : I. Some mobiles are scales. II. All pens being calculators is a possibility. 184. Statements : Some drinks are eatables. All eatables are sweets. Conclusions : I. Atleast some sweets are drinks. II. All drinks are sweets. Directions (185–189) : In each of the following questions, two/three statements followed by two Conclusions numbered I and II are given. You have to take the given statements to be true even if they seem to be at variance from commonly known facts and then decide which of the given Conclusions logically follows from the given statements disregarding commonly known facts. (IBPS RRBs Officer CWE (Pre.) Exam, 14.11.2016 (Shift-II))

    Give answer (1) if neither Conclusion I nor Conclusion II follows Give answer (2) if either Conclusion I or Conclusion II follows Give answer (3) if both the Conclusion I and Conclusion II follow Give answer (4) if only Conclusion I follows Give answer (5) if only Conclusion II follows (185–186) : Statements: All papers are woods. Some woods are leaves. All leaves are trunks. 185. Conclusions : I. Some leaves are papers. II. Atleast some trunks are woods. 186. Conclusions : I. All trunks being woods is a possibility. II. Some trunks are papers. (187–188) : Statements : No mobile is a band. All bands are pillows. Some pillows are sheets.

    BPRE–510

    187. Conclusions : I. No mobile is a pillow. II. All sheets are bands. 188. Conclusions : I. Some pillows are mobiles. II. All bands are sheets. 189. Statements : All ladders are snakes. Some snakes are frogs. Conclusions : I. No ladder is a frog. II. Atleast some ladders are frogs. Directions (190–194) : In these questions, two/three statements followed by two Conclusions numbered I and II are given. You have to take the given statements to be true even if they seem to be at variance from commonly known facts and then decide which of the given Conclusions logically follows from the statements disregarding commonly known facts. (IBPS RRBs Officer CWE (Pre.) Exam, 14.11.2016 (Shift-III))

    Give answer (1) if both the Conclusion I and Conclusion II follow Give answer (2) if either Conclusion I or Conclusion II follows Give answer (3) if neither Conclusion I nor Conclusion II follows Give answer (4) if only Conclusion I follows Give answer (5) if only Conclusion II follows 190. Statements : No horse is a goat. Some goats are deer. Conclusions : I. No horse is a deer. II. Atleast some horses are deer. (191–192) : Statements : Some trees are branches. Some branches are roads. No road is a sky. 191. Conclusions : I. No tree is a sky. II. Some branches are definitely not skies. 192. Conclusions : I. All roads are trees. II. All skies are branches. (193–194) : Statements : All plates are spoons. All spoons are glasses. Some glasses are mugs.

    SYLLOGISM 193. Conclusions : I. All plates being mugs is a possibility. II. All glasses are plates. 194. Conclusions : I. Some mugs are spoons. II. No mug is a spoon. Directions (195–197) : In each of the questions given below four statements followed by three Conclusions numbered I, II and III are given. You have to consider the given satements to be true even if they seem to be at variance with commonly known facts and then decide which of the given Conclusions logically follows from the given statements, irrespective of commonly known facts. (IBPS Bank PO/MT CWE (Main) Exam, 18.11.2016

    195. Statement : Some pens are erasers. Some erasers are blades. All blades are knives. All knives are peelers. Conclusions : I. Atleast some pens are blades. II. Atleast some pens are peelers. III. Atleast some blades are peelers. (1) Only II and III follow. (2) Only I and II follow. (3) Only I and III follow. (4) All I, II and III follow (5) None follows. 196. Statement : Some doors are windows Some windows are rods. All rods are sticks All sticks are weights. Conclusions: I. Atleast some weights are windows. II. Atleast some sticks are windows. III. All rods are weights. (1) Only conclusion III follows. (2) Only conclusion I and III follow. (3) All Conclusions follow. (4) None of the Conclusions follows (5) None of these 197. Statements : Some leaves are flowers. Some flowers are petals. Some petals are fruits. Some fruits are plants.

    Conclusions : I. Some plants are leaves. II. Some petals are plants. III. Some fruits are flowers. (1) Only Conclusions I and III follow. (2) Only II and III follow. (3) Only I and II follow. (4) None follows. (5) None of these Directions (198–202) : In these questions, two/three statements followed by two Conclusions numbered I and II have been given. You have to take the given statements to be true even if they seem to be at variance from commonly known facts and then decide which of the given Conclusions logically follows from the given statements disregarding commonly known facts. (Indian Bank PO (Pre.) Exam, 21.01.2017 (Ist Sitting))

    Give answer (1) if either Conclusion I or Conclusion II follows Give answer (2) if neither Conclusion I nor Conclusion II follows Give answer (3) if both the Conclusion I and Conclusion II follow Give answer (4) if only Conclusion I follows Give answer (5) if only Conclusion II follows (198–199) : Statements : Some clouds are gates. No gate is a flip-flop. All flip-flops are memories. 198. Conclusions : I. No cloud is a flip-flop. II. All memories can never be gates. 199. Conclusions : I. No gate is a memory. II. Atleast some gates are memory. (200–201) : Statements : All crackers are parcels. Some parcels are gifts. No gift is a box. 200. Conclusions : I. No cracker is a gift. II. Some parcels are definitely not boxes. 201. Conclusions : I. All gifts being parcels is a possibility. II. Some boxes are crackers.

    BPRE–511

    202. Statements : Some chips are diodes. All diodes are transistors. Conclusions : I. Some chips are definitely not diodes. II. Atleast some transistors are chips. Directions (203–204) : In these questions four statements followed by three Conclusions numbered I, II and III have been given. You have to take the given statements to be true even if they seem to be at variance from the commonly known facts and then decide which of the given Conclusions logically follows from the given statements disregarding commonly known facts. Select the appropriate answer. (Indian Bank PO (Pre.) Exam, 21.01.2017 (2nd Sitting))

    203. Statements : All petals are flowers. Some flowers are buds. Some buds are leaves. All leaves are plants. Conclusions : I. Some petals are not buds. II. Some flowers are plants. III. No flower is a plant. (1) Only I follows (2) Either II or III follows (3) I and II follow (4) Only III follows (5) None follows 204. Statements : Some apartments are flats. Some flats are buildings. All buildings are bungalows. All bungalows are gardens. Conclusions : I. All apartments being building is a possibility. II. All bungalows are not buildings. III. No flat is garden. (1) None follows (2) Only I follows (3) Either I or III follows (4) I and II follows (5) Only II follows Directions (205–207) : In these questions, three statements followed by two Conclusions numbered I and II have been given. You have to take the given statements to be true even if they seem to be at variance from the com-

    SYLLOGISM monly known facts and then decide which of the given Conclusions logically follows from the given statements disregarding commonly known facts. (Indian Bank PO (Pre.) Exam, 21.01.2017 (2nd Sitting))

    Give answer (1) if only Conclusion I follows Give answer (2) if either Conclusion I or Conclusion II follows Give answer (3) if only Conclusion II follows Give answer (4) if both the Conclusion I and Conclusion II follow Give answer (5) if neither Conclusion I nor Conclusion II follows 205. Statements : Some stones are shells. All shells are pearls. No stone is fish. Conclusions : I. No fish is a shell. II. All shells being fish is a possibility. 205. Statements : All clerks are assistants. No assistant is an officer. All officers are managers. Conclusions : I. No manager is a clerk. II. All assistants being managers is a possibility. 207. Statements : All gliders are parachutes. No parachute is an airplane. All airplanes are helicopters. Conclusions : I. All parachutes being helicopters is a possibility. II. No helicopter is a glider. Directions (208 – 210) : In each of the following questions, three statements followed by two Conclusions numbered I and II have been given. You have to take the given statements to be true even if they seem to be at variance from commonly known facts and then decide which of the given Conclusions logically follows from the given statements disregarding commonly known facts. (IBPS SO (Agriculture) Exam, 29.01.2017)

    Give answer (1) if either Conclusion I or Conclusion II follows Give answer (2) if only Conclusions I follows Give answer (3) if both the Conclusion I and Conclusion II follows Give answer (4) if only Conclusion II follows Give answer (5) if neither Conclusion I nor Conclusion II follows 208. Statements : No hour is a day. All months are days. Some hours are calendars. Conclusions I. No month is an hour. II. All calendars being days is a possibility. 209. Statements : Some comics are books. All comics are novels. No novel is an article. Conclusions : I Some novels are books. II. All articles being books is a possibility. 210. Statements : No liquid is fluid. All solids are liquids. All gases are solids. Conclusions : I. Atleast some solids are fluids. II. All gases are liquids. Directions (211–212) : In these questions, four statements followed by five conclusions are given as five alternatives (1), (2), (3), (4) and (5). You have to assume the given statements to be true even if they seem to be at variance from commonly known facts and then decide which of the given Conclusions logically follows from the given statements disregarding commonly known facts. (IBPS SO (Agriculture) Exam, 29.01.2017)

    211. Statements : Some flutes are guitars. No guitar is a piano. Some pianos are drums. All drums are violins. Conclusions : (1) All pianos are violins. (2) No flute is a piano. (3) Some drums are definitely not flutes.

    BPRE–512

    (4) All violins being guitars is a possibility. (5) All violins being flutes is a possibility. 212. Statements : All pets are animals. All animals are reptiles. No animal is a bird. Some birds are insects. Conclusions : (1) No reptile is a pet. (2) All insects being animals is a possibility (3) Some pets are insects. (4) No pet is a bird. (5) Some birds are reptiles. Directions (213–214) : The questions given below consist of six statements followed by five options each containing three statement group. You have to select the group in which related statements are logically. (IBPS Bank PO/MT CWE (Main) Exam, 18.11.2016

    213. A B

    All scholars are fools. All those who are fools, are not scholars. C All those who are fools are uneducated. D All scholars are educated. E All those who are fools are educated. F Scholars are uneducated. (1) ABE (2) ADF (3) ACF (4) BCE (5) None of these 214. A Some polishes are made of oil. B All oils are made of combs. C All combs are used for polish. D Some oils are combs. E Some polishes are used for oils. F Some combs are used for oils. (1) CEF (2) CBA (3) ABE (4) BDF (5) None of these Dirctions (215–219) : In these questions, three statements followed by two Conclusions numbered I and II have been given. You have to take the given statements to be true even if they seem to be at variance from the commonly known facts and then decide which of the given Conclusions logically follows from the given statements. (IBPS RRBs Officers CWE (Prelim Exam) 09.09.2017 Ist Sitting)

    SYLLOGISM Give answer (1) if either Conclusion I or Conclusion II follows Give answer (2) if neither Conclusion I nor Conclusion II follows Give answer (3) if both the Conclusion I and Conclusion II follows Give answer (4) if only Conclusion I follows Give answer (5) if only Conclusion II follows 215. Statements : All powers are authorities. All authorities are controls. Some controls are rules. Conclusions : I. All powers are controls. II. No rule is an authority. 216. Statements : No chair is a table. All tables are desks. No desk is a paper. Conclusions : I. No chair is a paper. II. At least some chairs are desks. 217. Statements : All segments are episodes. No episode is a movie. All movies are films. Conclusions : I. All episodes being films is a possibility. II. No segment is a movie. (218–219) : Statements : Some hits are misses. Some misses are losses. No loss is a win. 218. Conclusions : I. Some losses are hits. II. All wins can never be hits. 219. Conclusions : I. All hits being wins is a possibility. II. No win is a miss. Directions (220–224) : In each of the following questions, three statements followed by two Conclusions numbered I and II have been given. You have to take the given statements to be true even if they seem to be at variance from the commonly known facts and then decide which of the given Conclusions logically follows from the given statements. (IBPS RRBs Officers CWE (Prelim Exam) 09.09.2017 IInd Sitting)

    Give answer (1) if only Conclusion I follows Give answer (2) if only Conclusion II follows Give answer (3) if both the Conclusion I and Conclusion II follow Give answer (4) if either Conclusion I or Conclusion II follows Give answer (5) if neither Conclusion I nor Conclusion II follows 220. Statements : All methods are services. All services are demands. Some demands are requests. Conclusions : I. All methods being requests is a possibility. II. Atleast some services are requests. 221. Statements : All resorts are hotels. No hotel is a cottage. All lodges are cottages. Conclusions : I. No lodge is a hotel. II. No cottage is a resort. 222. Statements : Some ideas are plans. All plans are blueprints. Some blueprints are maps. Conclusions : I. Atleast some blueprints are ideas. II. At least some maps are ideas. (223–224) : Statements : Some stocks are bills. No bill is a dividend. All dividends are cuts. 223. Conclusions : I. All dividends can never be stocks. II. All stocks can never be dividends. 224. Conclusions : I. All bills being cuts is a possibility. II. No cut is a stock. Directions (225–229) : In these questions, three statements followed by two Conclusions numbered I and II have been given. You have to take the given statements to be true even if they seem to be at variance from commonly known facts and then decide which of the given Conclusions logically follows from the given statements disregarding commonly known facts. (IBPS RRBs Officers CWE (Prelim Exam) 10.09.2017)

    BPRE–513

    Give answer (1) if either Conclusion I or Conclusion II follows Give answer (2) if both Conclusion I and Conclusion II follow Give answer (3) if only Conclusion I follows Give answer (4) if only Conclusion II follows Give answer (5) if neither Conclusion I nor Conclusion II follows (225–226) : Statements : Some posts are mails. Some mails are letters. No letter is a circular. 225. Conclusions : I. No letter is a post. II. Some letters are posts. 226. Conclusions : I. All circulars being posts is a possibility. II. All mails can never be circulars. 227. Statements : No option is a question. No question is a choice. All alternatives are choices. Conclusions : I. No choice is an option. II. All alternatives can never be options. 228. Statements : All tablets are medicines. All syrups are medicines. Some syrups are vitamins. Conclusions : I. All medicines being vitamins is a possibility. II. All syrups are tablets. 229. Statements : Some planets are stars. All moons are stars. No star is a comet. Conclusions : I. Some moons are definitely not planets. II. All planets can never be comets. Directions (230–234) : In each of the following questions, three statements followed by two Conclusions numbered I and II have been given. You have to take the given statements to be true even if they seem to be at variance from commonly known facts and then decide which of the given Conclusions logically follows from the given statements. (IBPS RRBs Officer CWE (Prelim Exam) 16.09.2017)

    SYLLOGISM Give answer (1) if both the Conclusion I and Conclusion II follow Give answer (2) if neither Conclusion I nor Conclusion II follows Give answer (3) if either Conclusion I or Conclusion II follows Give answer (4) if only Conclusion I follows Give answer (5) if only Conclusion II follows 230. Statements : All plots are themes. Some themes are fictions. No fiction is a biography. Conclusions : I. Some biographies are definitely not plots. II. All themes can never be biographies. (231–232) : Statements : Some trainings are drills. All drills are bars. Some bars are goals. 231. Conclusions : I. Atleast some bars are trainings. II. Atleast some goals are trainings. 232. Conclusions : I. All goals being drills is a possibility. II. All drills can never be trainings. 233. Statements : All numbers are letters. Some letters are posts. All posts are digits. Conclusions : I. No number is a digit. II. Atleast some numbers are digits. 234. Statements : All paints are machines. No machine is a circuit. All circuits are wires. Conclusions : I. All paints being circuits is a possibility. II. At least some wires are machines. Directions (235–239) : In each of the following questions, some statements followed by two Conclusions numbered I and II have been given. You have to take the given statements to be true even if they seem to be at variance from commonly known facts and then decide which of the given Conclusions logically follows from the given statements. (IBPS RRBs Officer CWE (Main Exam) 05.11.2017)

    Give answer (1) if both the Conclusion I and Conclusion II follow

    Give answer (2) if either Conclusion I or Conclusion II follows Give answer (3) if only Conclusion I follows Give answer (4) if only Conclusion II follows Give answer (5) if neither Conclusion I nor Conclusion II follows (235–236) : Statements All mobiles are discs. Some discs are spheres. No sphere is charger. 235. Conclusions : I. At least some mobiles are spheres. II. Some chargers are discs. 236. Conclusions : I. All discs are chargers. II. Some discs are chargers. 237. Statements : All books are pencils. No pencil is pen. Conclusions : I. All pencils being books is a possibility. II. No pen is book. (238–239) : Statements All games are puzzles. All dramas are puzzles. No puzzle is theory. 238. Conclusions : I. All theories being dramas is a possibility. II. All puzzles are games. 239. Conclusions : I. All dramas being games is a possibility. II. No theory is game. Directions (240–243) : In each of the following questions, three statements followed by two Conclusions numbered I and II given below it. You have to take the given statements to be true even if they seem to be at variance from the commonly known facts and then decide which of the given Conclusions logically follows from the given statements. (IBPS SO (IT Officer) CWE (Prelim Exam) 30.12.2017)

    Give answer (1) if both the Conclusion I and Conclusion II follow Give answer (2) if either Conclusion I or Conclusion II follows Give answer (3) if only Conclusion I follows Give answer (4) if only Conclusion II follows Give answer (5) if neither Conclusion I nor Conclusion II follows

    BPRE–514

    (240–241) : 240. Statements : Some juices are fruits. No fruit is a potato. Some potatoes are cakes. Conclusions : I. Some cakes are fruits. II. No fruit is a cake. 241. Conclusions : I. All juices being cakes is a possibility. II. All juices can never be potatoes. 242. Statements : All sketches are paintings. Some paintings are drawings. All drawings are letters. Conclusions : I. All paintings can never be letters. II. Atleast some sketches are drawings. 243. Statements : All cubs are kittens. No kitten is a zebra. All zebras are bears. Conclusions : I. All zebras being cubs is a possibility. II. No kitten is a bear. Directions (244–249) : In each of the following questions, three statements followed by two Conclusions numbered I and II have been given. You have to take the given statements to be true even if they seem to be at variance from commonly known facts and then decide which of the given Conclusions logically follows from the given statements. (IBPS SO (Law Officer) CWE (Prelim Exam) 31.12.2017)

    Give answer (1) if either Conclusion I or Conclusion II follows Give answer (2) if only Conclusion I follows Give answer (3) if only Conclusion II follows Give answer (4) if both the Conclusion I and Conclusion II follow Give answer (5) if neither Conclusion I nor Conclusion II follows (244–245) : Statements : No door is a lamb. All trucks are doors. No ship is a lamb 244. Conclusions : I. Atleast some doors are ships. II. No door is a ship.

    SYLLOGISM 245. Conclusions : I. Some ships being trucks is a possibility. II. No lamb is a truck. (246–247) : Statements : All nodes are tasks. Some steps are nodes. All steps are cabins. 246. Conclusions : I. All tasks being steps is a possibility. II. Some nodes are definitely not cabins. 247. Conclusions : I. All nodes are cabins. II. No step is a task. 248. Statements : All festivals are carnivals. Some exhibitions are festivals. All pictures are exhibitions. Conclusions : I. Atleast some carnivals are exhibitions. II. No picture is a festival. Directions (249–253) : In each of the following question, three statements followed by two Conclusions numbered I and II are given. You have to take the given statements to be true even if they seem to be at variance from commonly known facts and then decide which of the given Conclusions logically follows. (Canara Bank PO Exam 04.03.2018)

    Give answer (1) if neither Conclusion I nor Conclusion II follows Give answer (2) if both Conclusion I and Conclusion II follow Give answer (3) if only Conclusion I follows Give answer (4) if only Conclusion II follows Give answer (5) if either Conclusion I or Conclusion II follows 249. Statements : No whistle is a van. All trollies are vans. Some whistles are carriers. Conclusions : I. No whistle is a trolley. II. All carriers being vans is a possibility. 250. Statements : No cup is a plate. No mug is a cup. All jars are plates. Conclusions : I. No jar is a cup. II. Some mugs are definitely not plates.

    251. Statements : Some palaces are caves. No village is a cave. All resorts are palaces. Conclusions : I. Some palaces are definitely not villages. II. All caves being resorts is a possibility. 252. Statements : Some hotels are rooms. Some keys are hotels. No bungalow is a room. Conclusions : I. All hotels being bungalows is a possibility. II. All keys can never be rooms. 253. Statements : All bridges are camps. All tents are camps. Some hinges are tents. Conclusions : I. All camps can never be hinges. II. Some bridges are definitely not tents. Directions (254–258) : In each of the following questions four statements followed by three Conclusions I, II and III have been given. You have to take the given statements to be true even if they seem to be at variance from the commonly known facts and them decide which of the given Conclusions logically follows from the given statements disregarding commonly known facts. (IDBI Bank PO Exam 29.04.2018)

    254. Statements : Some villages are towns. Some towns are huts. All huts are rivers. Some rivers are tents. Conclusions : I. Some tents are towns. II. Some rivers are towns. III. Some huts are villages. (1) Only Conclusions I and II follow (2) Only Conclusion II follows (3) None follows (4) Only Conclusion III follows (5) Only Conclusions II and III follow 255. Statements : All hotels are buses. Some buses are cars. All cars are trams. Some trams are clouds.

    BPRE–515

    Conclusions : I. Some trams are buses. II. Some trams are hotels. III. Some clouds are cars. (1) Only Conclusion I follows (2) Only Conclusions I and II follow (3) Either Conclusion I or II follows (4) Only Conclusions I and III follow (5) None follows 256. Statements : All flowers are books. All books are carpets. Some carpets are keys. Some keys are locks. Conclusions : I. Some keys are books. II. Some keys are flowers. III. Some locks are books. (1) Only Conclusions II and III follow (2) Either Conclusion I or II and III follow (3) Only Conclusion III follows (4) None follows (5) All follow 257. Statements : All boxes are cups. All chairs are cups. All cups are mirrors. All tables are mirrors. Conclusions : Some tables are chairs. Some mirrors are boxes. Some mirrors are chairs. (1) Only Conclusion III follows (2) Only Conclusions I and II follow (3) Only Conclusions I and III follows (4) None follows (5) Only Conclusions II and III follow 258. Statements : Some pins are needles. All needles are ropes. Some ropes are buckets. All buckets are trees. Conclusions : I. Some buckets are pins. II. Some ropes are pins. III. No bucket is pin. (1) Only Conclusion II follows (2) Only Conclusions I and III follow

    SYLLOGISM (3) Only Conclusion II and either Conclusion I or III follow (4) All follow (5) None follows Directions (259–262) : In each of the following questions, three statements followed by two Conclusions numbered I and II have been given. You have to take the given statements to be true even if they seem to be at variance from commonly known facts and then decide which of the given Conclusions logically follows from the given statements. Give answer (1) if both the Conclusion I and Conclusion II follow Give answer (2) if either Conclusion I or Conclusion II follows Give answer (3) if only Conclusion I follows Give answer (4) if only Conclusion II follows Give answer (5) if neither Conclusion I nor Conclusion II follows (IBPS RRBs Officer CWE Prelim Exam, 11.08.2018)

    (259–260) : Statements : Some orchids are boxes. No car is an orchid. All arrows are boxes. 259. Conclusions : I. All cars being arrows is a possibility. II. All boxes can never be cars. 260. Conclusions : I. Some cars are definitely not boxes. II. No arrow is an orchid. (261–262) : Statements : All logics are posters. Some gates are posters. All blanks are gates. 261. Conclusions : I. Atleast some blanks are logics. II. No logic is a blank. 262. Conclusions : I. No blank being poster is a possibility. II. All logics can never be gates. Directions (263–266) : Each of the questions below consists of some statements followed by two Conclusions numbered I and II. Consider the given statements to be true even if they seem to be at variance with commonly known facts. Read all the Conclusions and then decide which of the given Conclusions does not logically follow from the given statements using all statements together. (IBPS RRBs Officer CWE Prelim Exam, 11.08.2018)

    Give answer (1) if only Conclusion II follows Give answer (2) if either Conclusion I or Conclusion II follows Give answer (3) if neither Conclusion I nor Conclusion II follows Give answer (4) if both Conclusion I and Conclusion II follow Give answer (5) if only Conclusion I follows (263–264) : Statements : All grills are Arrows. Some hats are grills. Some cells are arrows. 263. Conclusions : I. Some cells are definitely not grills. II. Some hats can never be arrows. 264. Conclusions : I. Some hats are arrows. II. Some grills are cells. (265–266) : Statements : Some doors are fans. No door is rose. No fan is shelf. 265. Conclusions : I. Some fans can never be roses. II. Some roses are shelf is a possibility. 266. Conclusions : I. All doors are shelf is a possibility. II. All shelf can be doors. Directions (267–269) : In each of the following questions, three statements followed by three Conclusions numbered I, II and III have been given. You have to take the given statements to be true even if they seem to be at variance from the commonly known facts and then decide which of the given Conclusions logically follows from the given statements : (Indian Bank PO Prelim Exam, 06.10.2018)

    267. Statements : All roads are sides. All gadgets are roads. No frame is a gadget. Conclusions : I. Some frames are definitely not roads. II. All sides being frames is a possibility. III. Atleast some sides are gadgets.

    BPRE–516

    (1) Only III follows (2) Only I and II follow (3) Only I and III follow (4) Only I follows (5) None of I, II and III follows 268. Statements : Some bamboos are clusters. Some orchids are bamboos. All dangers are clusters. Conclusions : I. All orchids being dangers is a possibility. II. All orchids can never be clusters. III. Some bamboos are definitely not dangers. (1) Only II and III follow (2) Only I and III follow (3) All I, II and III follow (4) Only II follows (5) Only I follows 269. Statements : No silt is a clay. All plasters are clays. No hill is a slit. Conclusions : I. No plaster is a slit. II. All hills can never be clays. III. Some hills are definitely not plasters. (1) Only I follows (2) All I, II and III follow (3) Only II follows (4) Only II and III follow (5) Only III follows 270. In this question are given three statements followed by five Conclusions, one of which definitely logically follows from the given statements. That Conclusion is your answer. (NOTE : You have to take the three given statements to be true even if they seem to be at variance from commonly known facts.) Statements : Some pigeons are crows. No duck is crow. All sparrows are pigeons. (1) No duck is a sparrow (2) All sparrows are crows (3) Some pigeons are ducks (4) All crows being sparrows is a possibility (5) Some crows are definitely not pigeons.

    SYLLOGISM Directions (271–273) : In these questions, two/three statements followed by two Conclusions numbered I and II have been given. You have to take the given statements to be true even if they seem to be at variance from commonly known facts and then decide which of the given Conclusions logically follows from the givens statements. Give answer (1) if only Conclusion I follows. Give answer (2) if only Conclusion II follows. Give answer (3) if either Conclusion I or Conclusion II follows. Give answer (4) if neither Conclusion I nor Conclusion II follows. Give answer (5) if both the Conclusion I and Conclusion II follow. (IBPS Bank PO/MT CWE (Prelim Exam) 14.10.2018)

    271. Statements : All ratios are norms. Some norms are tables. Conclusions : I.

    No table is a ratio.

    II. All tables are norms. (272–273) : Statements : No deer is a lion. Some lions are tigers. All lions are horses. 272. Conclusions : I.

    All tigers being deer is a possibility.

    II. Atleast some tigers are horses. 273. Conclusions : I.

    All deer being horses is a possibility.

    II. All horses are lions.

    Give answer (2) if only Conclusion II follows Give answer (3) if either Conclusion I or Conclusion II follows Give answer (4) if neither Conclusion I nor Conclusion II follows Give answer (5) if both Conclusion I and Conclusion II follows (IBPS Bank PO/MT CWE (Prelim Exam) 21.10.2018)

    (274–275): Statements: All papers are woods. Some woods are leaves. All leaves are trunks. 274. Conclusions : I. Some leaves are papers. II. Atleast some trunks are woods. 275. Conclusions : I. All trunks being woods is a possibility. II. Some trunks are papers. (276-277) Statements: No mobile is a band. All bands are pillows. Some pillows are sheets. 276. Conclusions : I.

    No mobile is a pillow.

    II. All sheets are bands. 277. Conclusions : I.

    Some pillows are mobiles.

    II. All bands are sheets. 278. Statements: All ladders are snakes. Some snakes are frogs. Conclusions : I. No ladder is a frog. II. Atleast some ladders are frogs.

    Directions (274–278): In each of the questions below are given two/three statements followed by two Conclusions numbered I and II. You have to take the given statements to be true even if they seem to be at variance with commonly known facts. Read all the Conclusions and then decide which of the given Conclusions logically follows from the given statements disregarding commonly known facts.

    Directions (279–283) : In each of the questions below are given three statements followed by two Conclusions numbered I and II. You have to take the given statements to be true even if they seem to be at variance with commonly known facts. Read all the Conclusions and decide which of the given Conclusions logically follows from the given statements disregarding commonly known facts.

    Give answer (1) if only Conclusion I follows

    Give answer (1) if either Conclusion I or Conclusion II follows.

    BPRE–517

    Give answer (2) if only Conclusion I follows Give answer (3) if both the Conclusion I and Conclusion II follow Give answer (4) if only Conclusion II follows Give answer (5) if neither Conclusion I nor Conclusion II follows (Canara Bank PO Exam, 23.12.2018)

    (279–280) : Statements : Some pens are erasers. Some erasers are markers. All markers are lighters. 279. Conclusions : I. Atleast some erasers are lighters. II. Some lighters being pens is a possibility. 280. Conclusions : I. All pens are markers. II. Some pens are markers. (281–282) : Statements : No mail is a notice. All notices are intimations. Some intimations are warnings. 281. Conclusions : I. No mail is a warning. II. Atleast some mails are warnings. 282. Conclusions : I. All intimations are notices. II. Some notices being warnings is a possibility. 283. Statements : All seconds are minutes. All minutes are hours. Some hours are days. Conclusions : I. Some days are definitely not minutes. II. All seconds are hours. Directions (284–288) : In each of the questions below are given three statements followed by two/three Conclusions numbered I, II and III. You have to take the given statements to be true even if they seem to be at variance with commonly known facts. Read all the Conclusions and decide which of the given Conclusions logically follows from the given statements disregarding commonly known facts. (IBPS Specialist Officer CWE (Prelim Exam) 31.01.2019)

    SYLLOGISM 284. Statements : Some circles are squares. All squares are rectangles. All lines are rectangles. Conclusions : I. All lines are squares. II. Some circles are rectangles. III. Some lines are circles. (1) Only I follows (2) Only II follows (3) Only III follows (4) Only I and II follow (5) None of these 285. Statements : Some friends are trust. All trust are love. Some happy are trust. Conclusions : I. Some love are happy. II. Some friend are love. III. All love are trust. (1) Only I and III follow (2) Only II follows (3) Only I and II follow (4) Only III follows (5) None of these 286. Statements : All ice are winter. Some snows are ice. Some winter are cold. Conclusions : I. All snow being cold is a possibility. II. Some ice are definitely not cold. (1) Only Conclusion I follows (2) Only Conclusion II follows (3) Either Conclusion I or II follows (4) Neither Conclusion I nor Conclusion II follows (5) Both Conclusion I and Conclusion II follow 287. Statements : Some blue are green. Some grey is blue. All green are white. Conclusions : I. Some grey is neither blue nor white is a possibility. II. All green are grey is a possibility.

    (1) Only Conclusion I follows (2) Only Conclusion II follows (3) Either Conclusion I or II follows (4) Neither Conclusion I nor II follows (5) Both Conclusion I and Conclusion II follow 288. Statements : All trees are plants. No fruit is plant. All fruits are flowers. Conclusions : I. Some plants which are flowers are also trees. II. No flower is a tree. (1) Only Conclusion I follows (2) Only Conclusion II follows (3) Either Conclusion I nor Conclusion II follows (4) Neither Conclusion I nor Conclusion II follows (5) Both Conclusion I and Conclusion II follow Directions (289–293) : In these questions, three statements followed by two Conclusions numbered I and II have been given. You have to take the given statements to be true even if they seem to be at variance from the commonly known facts and then decide which of the given Conclusions logically follows from the given statements. (IBPS RRBs Officer CWE Prelim Exam, 04.08.2019)

    Give answer (1) : if either Conclusion I or Conclusion II follows. Give answer (2) : if neither Conclusion I nor Conclusion II follows. Give answer (3) : if both the Conclusion I and Conclusion II follow. Give answer (4) : if only Conclusion I follows. Give answer (5) : if only Conclusion II follows. (289–290) : Statements : All nests are ankles. Only a few pillows are nests. Only a few trollies are ankles. 289. Conclusions : I. Some trollies are definitely not nests. II. All ankles can never be pillows.

    BPRE–518

    290. Conclusions : I. All pillows being ankles is a possibility. II. Atleast some trollies are nests. 291. Statements : Only a few offers are buckets. No can is a bucket. All arrows are buckets. Conclusions : I. Some offers are arrows. II. No can is an arrow. (292–293) : Statements : No dentist is a carpenter. Only a few actors are dentists. No supervisor is an actor. 292. Conclusions : I. All supervisors can never be dentists. II. No carpenter is an actor. 293. Conclusions : I. All supervisors being carpenters is a possibility. II. Some actors are definitely not carpenters. Directions (294–298) : Each of the questions below consists of some statements followed by two Conclusions numbered I and II. Consider the given statements to be true even if they seem to be at variance with commonly known facts. Read all the Conclusions and then decide which of the given Conclusions follow(s) logically from the given statements (IBPS RRBs Officer CWE Prelim Exam, 17.08.2019)

    Give answer (1) if only Conclusion I follows Give answer (2) if only Conclusion II follows Give answer (3) if either Conclusion I or Conclusion II follows Give answer (4) if neither Conclusion I nor Conclusion II follows Give answer (5) if both Conclusion I and Conclusion II follow (294–295) : Statements : Some wins are trophies. Some trophies are cups. No cup is a prize. 294. Conclusions : I. Atleast some cups are wins. II. All prizes being trophies is a possibility.

    SYLLOGISM 295. Conclusions : I. No trophy is a prize. II. No prize is a win. 296. Statements : No layer is a coat. All coats are deposits. All deposits are sheets. Conclusions : I. All coats are sheets. II. All deposits can never be layers. 297. Statements : Some flats are apartments. No apartment is a hall. Some halls are rooms. Conclusions : I. Atleast some rooms are flats. II. No apartment is a room. 298. Statements : Some codes are secrets. All secrets are puzzles. Conclusions : I. All secrets being codes is a possibility. II. Atleast some puzzles are codes. Directions (299 – 303) : In each of the questions below are given some statements followed by two Conclusions numbered I and II. You have to take the given statements to be true even if they seem to be at variance with commonly known facts. Read all the Conclusions and decide which of the given Conclusions logically follows from the given statements disregarding commonly known facts. (IBPS Bank PO/MT CWE Prelim Exam, 19.10.2019)

    Give answer (1) if only Conclusion I follows Given answer (2) if only Conclusion II follows Give answer (3) if either Conclusion I or Conclusion II follows. Give answer (4) if neither Conclusion I nor Conclusion II follows Give answer (5) if both the Conclusion I and Conclusion II follow 299. Statements : All accounts are groups. No group is a collection. Only a few collections are donations.

    Conclusions : I. All donations can never be groups. II. No account is a collection. 300. Statements : Only a few trucks are buses. Only a few buses are cycles. All cycles are planes. Conclusions : I. No truck is a cycle. II. All buses being planes is a possibility. 301. Statements : Only a few bottles are glasses. All glasses are eggs. Conclusions : I. Atleast some bottles are eggs. II. No bottle is an egg. 302. Statements : All rats are tigers. Only a few tigers are cats. No cat is a bear. Conclusions : I. Atleast some rats are cats. II. All tigers can never be bears. 303. Statements : Only a few frogs are reptiles. All reptiles are snakes. All snakes are toads. Conclusions : I. All reptiles are toads. II. All snakes being frogs is a possibility. Directions (304–306) : In the following questions, some statements are given followed by some Conclusions. You have to consider the statements to be true, even if they seem to be at variance from commonly known facts. Read the Conclusions and decide which of the given Conclusions logically follows from the given statements: 304. Statements : (1) Some lions are cats (2) All cats are tigers. (3) Some goats are tigers. Conclusions : I.

    Some goats being cats is a possibility II. Some lions are not tigers III. Some cats are not tigers

    BPRE–519

    (1) Only I and II follow (2) Only II and III follow (3) All the Conclusions I, II and III follow (4) Only Conclusions I and III follow (5) Only Conclusion I follows 305. Statements : (1) All black are dark. (2) All spaces are matters. (3) Some dark are spaces. Conclusions : I. Some black are spaces. II. Some dark are matters. III. Some matters are not black. (1) Only Conclusions I and II follow (2) Only Conclusion II follows (3) All Conclusion I, II and III follow (4) Only Conclusions I and III follow (5) None follows 306. Statements : (1) Some water are hydrogen (2) All water are oxygen. (3) All oxygen are gases. Conclusions : I. Some water are not gases. II. Some hydrogen are oxygen. III. All water are gases. (1) Only Conclusions I and II follow (2) Only Conclusion III follows (3) All Conclusions I, II and III follow (4) Only Conclusions I and III follow (5) None follows Directions (307–309) : In each question below are given three statements followed by three Conclusions numbered I, II and III. You have to take the given statements to be true even if they seem to be at variance from commonly known facts. Read all the Conclusions and then decide which of the given Conclusions logically follows from the given statements. (IBPS RRBs Officer Scale-I CWE Main Exam, 13.10.2019)

    307. Statements 1. All human are animals. 2. All animals are socials. 3. Some socials are philosophers.

    SYLLOGISM Conclusions I. Some socials are not philosophers. II. Some humans are philosophers. III. Some animals being philosophers is a possibility. (1) Only Conclusion I follows (2) Only Conclusion II follows (3) Only Conclusion III follows (4) Only Conclusions II and III follow (5) Only Conclusions I and III follow 308. Statements 1. All politicians are professors. 2. Some citizens are females. 3. Some citizens are politicians. Conclusions I. Some politicians being females is a possibility. II. Some citizens are professors. III. Some politicians are not professors. (1) Only Conclusion I follows (2) Only Conclusion II follows (3) Only Conclusion III follows (4) Only Conclusions I and II follow (5) Only Conclusions I and III follow 309. Statements 1. Some currencies are rupees. 2. Some coins are currencies. 3. All rupees are dollars. Conclusions I. Some coins are rupees. II. Some currencies are dollars. III. Some dollars are not rupees. (1) Only Conclusion I follows (2) Only Conclusion II follows (3) Only Conclusions I and II follow (4) Only Conclusions II and III follow Directions (310–313) : In each of the following questions, three statements followed by two Conclusions numbered I and II have been given. You have to take the given statements to be true even if they seem to be at variance from the commonly known facts and then decide which of the given Conclusions logically follows from the given statements. (IBPS SO Prelim Exam, 28.12.2019)

    Give answer (1) if only Conclusion I follows Give answer (2) if only Conclusion II follow Give answer (3) if either Conclusion I or Conclusion II follows Give answer (4) if neither Conclusion I nor Conclusion II follows Give answer (5) if both Conclusion I and Conclusion II follow 310. Statements :

    Give answer (1) if only Conclusion I follows Give answer (2) if only Conclusion II follows Give answer (3) if either Conclusion I or Conclusion II follows. Give answer (4) if neither Conclusion I nor Conclusion II follows Give answer (5) if both the Conclusion I and Conclusion II follow 314. Statements :

    Only a few malls are banks.

    All companies are offices.

    All banks are shops.

    Only a few offices are buildings. All buildings are stores.

    Only a few shops are halls. Conclusions : I. At least some malls are shops. II. No bank is a hall. 311. Statements : Only a few brooms are sticks. Only a few sticks are docks. All tins are docks. Conclusions : I. At least some docks are brooms. II. No stick is a tin. 312. Statements : All aims are goals. Only a few steps are blocks. Only a few blocks are aims. Conclusions : I. Only a few steps are goals. II. All aims being steps is a possibility. 313. Statements : Only a few tapes are cots. All cots are ships. No ship is a van. Conclusions : I. Some tapes are vans. II. No tape is a van. Directions (314–317) : In each of the following questions, three statements followed by two Conclusions numbered I and II have been given. You have to take the given statements to be true even if they seem to be at variance from commonly known facts and then decide which of the given Conclusions logically follows from the given statements. (Indian Bank Specialist Officer SO Exam, 08.03.2020)

    BPRE–520

    Conclusions : I. At least some offices are stores. II. All companies being buildings is a possibility. 315. Statements : All grounds are fields. Only a few grounds are mats. No mat is a carpet. Conclusions : I. At least some mats are fields. II. No ground is a carpet. 316. Statements : All bins are cones. Only a few cones are arrows. No arrow is a flag. Conclusions I. All bins are flags. II. At least some arrows are bins. 317. Statements : Only a few folders are books. Only a few envelopes are folders. All envelopes are pockets. Conclusions : I. All envelopes can never be books. II. Some folders are packets. Directions (318–320) : In each of the following questions, three statements are followed by three Conclusions numbered I, II and III. You have to take the given statements to be true even if they seem to be at variance from commonly known facts and then decide which of the following Conclusions logically follow(s) from the given statements. (IBPS Bank PO/MT CWE Prelim Exam, 03.10.2020)

    SYLLOGISM 318. Statements : No bird is kite. Some cats are kites. All tigers are cats. Conclusions : I. Some birds being tigers is a possibility. II. Some cats being birds is a possibility. III. All kites being cats is a possibility. (1) Only Conclusion I and Conclusion III follow (2) Only Conclusion II and Conclusion III follow (3) All the Conclusions I, II and III follow (4) Only Conclusion I and Conclusion II follow (5) Only Conclusion III follows 319. Statements : Some politicians are ministers. All officers are engineers. Some politicians are engineers. Conclusions : I. Some officers are not engineers. II. Some ministers being engineers is a possibility. III. Some politicians are officers. (1) Only Conclusion II follows (2) Only Conclusion I and Conclusion II follow (3) Only Conclusion II and Conclusion III follow (4) Only Conclusion I follows (5) All the Conclusions I, II and III follow 320. Statements : All movies are dramas. Some stories are movies. Some keys are stories. Conclusions : \ I. Some keys being movies is a possibility. II. Some stories are dramas. III. All keys being dramas is a possibility. (1) Only Conclusion I and Conclusion III follow (2) Only Conclusion I follows (3) Only Conclusion II and Conclusion III follow

    (4) Only Conclusion I and Conclusion II follow (5) All the Conclusions I, II and III follow Directions (321–325) : Study the following information and the two Conclusions numbered I and II carefully and decide which of the given Conclusion logically follow(s) from the given statements ? ● A & B : All As are Bs. ● A % B : Some As are Bs. ● A # B : Some As are not Bs. ● A @ B : No A is B. ● A $ B : All As being Bs is a possibility. (IBPS RRBs Officer CWE Prelim Exam, 31.12.2020)

    Give answer (1) if only Conclusion I follows Give answer (2) if only Conclusion II follows Give answer (3) if both the Conclusion I and Conclusion II follow Give answer (4) if either Conclusion I or Conclusion II follows Give answer (5) if neither Conclusion I nor Conclusion II follows 321. Statements : J$U&L@F Conclusions : I. All Fs being Us is a possibility. II. All Js being Ls is a possibility. 322. Statements : K$E%Y&G Conclusions : I. Some Es are Gs. II. Some Ks are not Gs. 323. Statements : Z%B&K#G Conclusions : I. All Gs being Zs is a possibility. II. All Zs being Ks is a possibility. 324. Statements : T%G&L@F Conclusions : I. All Gs are Ts. II. Some Fs are not Gs. 325. Statements : G@M%K&F Conclusions : I. Some Ms are Fs. II. Some Ks are not Gs.

    BPRE–521

    Directions (326–327) : Study the following statements and Conclusions based on Syllogism and select the appropriate answer : (IBPS RRBs Officer CWE Prelim Exam, 07.08.2021)

    A#B A%B ● A&B ● A*B ● A@B

    All As are Bs. Some As are Bs. Some As are not Bs. No A is B. Some As being Bs is a possibility. ● A$B : No A being B is a possibility Give answer (1) if only Conclusion I follows Give answer (2) if only Conclusion II follows Give answer (3) if either Conclusion I or Conclusion II follows Give answer (4) if neither Conclusion I nor Conclusion II follows Give answer (5) if both the Conclusion I and Conclusion II follow 326. Statement : T$Z%M#G Conclusions : I. Some Ts are not Gs. II. Some Gs are Zs. 327. Statement : G#Z%K*P Conclusions I. Some Ks are Zs. II. Some Zs are not Ps. Directions (328–331) : In each of the questions below are given three statements followed by three Conclusions numbered I, II and III. You have to take the given statements to be true even if they seem to be at variance from commonly known facts. Read all the Conclusions and then decide which of the given Conclusions logically follows from the given statements. ● ●

    : : : : :

    (IBPS Bank PO/MT CWE Prelim Exam, 07.08.2021)

    328. Statements All cats are tigers. Some lions are cats. Some goats are tigers. Conclusions I. Some cats are not tigers. II. Some lions are tigers. III. Some goats being cats is a possibility. (1) Only I and II follow (2) Only II and III follow (3) All I, II and III follow (4) Only I and III follow (5) None follows

    SYLLOGISM 329. Statements All waters are oxygens. Some waters are hydrogens. All oxygens are gases. Conclusions I. All waters are gases. II. Some hydrogens are oxygens. III. Some oxygens are waters. (1) All I, II and III follow (2) Only I and II follow (3) Only II and III follow (4) None follows (5) Only III follows 330. Statements Some atoms are matters. All matters are molecules. All particles are atoms. Conclusions I. Some molecules are particles. II. Some atoms are particles. III. Some matters are atoms. (1) Only I and III follow (2) None follows (3) Only I follows (4) Only I and II follow (5) Only II and III follow 331. Statements Some pins are tablets. All tablets are needles. No needle is thread. Conclusions I. Some needles are pins. II. Some threads are needles. III. Some needle are tablets. (1) None follows (2) Only I and II follow (3) Only II and III follow (4) Only I and III follow (5) All I, II and III follow

    SBI PO EXAMS Directions (1–6) : In each question below are two/three statements followed by two conclusions numbered I and II. You have to take the two/ three given statements to be true even if they seem to be at variance from commonly known facts and then decide which of the given conclusions logically follows from the given statements disregarding commonly known facts. (SBI Associate Banks PO Exam. 07.08.2011)

    Give answer (1) : if only conclusion I follows. Give answer (2) : if only conclusion II follows. Give answer (3) : if either conclu-sion I or conclusion II follows. Give answer (4) : if neither conclusion I nor conclusion II follows. Give answer (5) : if both conclusion I and conclusion II follow. 1. Statements : All rings are circles. All squares are rings. No ellipse is a circle. Conclusions : I. Some rings being ellipses is a possibility. II. Atleast some circles are squares. 2. Statements : No house is an apartment. Some bungalows are apartments. Conclusions : I. No house is a bungalow: II. All bungalows are houses. 3. Statements : Some gases are liquids. All liquids are water. Conclusions : I. All gases being water is a possibility. II. All such gases which are not water can never be liquids. 4. Statements : All minutes are seconds. All seconds are hours. No second is a day. Conclusions : I. No day is an hour. II. Atleast some hours are minutes. Directions (5–6) : 5. Statements : Some teachers are professors. Some lecturers are teachers. Conclusions : I. All teachers as well as all professors being lecturers is a possibility. II. All those teachers who are lecturers are also professors. 6. Conclusions : I. No professor is a lecturer. II. All lecturers being professors is a possibility.

    BPRE–522

    Directions (7–11) : In each of the questions below, two/three statements are given followed by conclusions/group of conclusions numbered I and II. You have to assume all the statements to be true even if they seem to be at variance from the commonly known facts and then decide which of the given two conclusions logically follows from the information given in the statements. (SBI Probationary Officer Exam 28.04.2013)

    Give answer (1) if only conclusion I follows Give answer (2) if only conclusion II follows Give answer (3) if either I or II follows Give answer (4) if neither I nor II follows Give answer (5) if both I and II follow Directions (7-11) : Statements : Some squares are circles. No circle is a triangle. No line is a square. 7. Conclusions : I. All squares can never be triangles. II. Some lines are circles. 8. Conclusions : I. No triangle is a square. II. No line is a circle. Directions (9-10) Statements : All songs are poems. All poems are rhymes. No rhyme is a paragraph. 9. Conclusions : I. No song is a paragraph. II. No poem is a paragraph. 10. Conclusions : I. All rhymes are poems. II. All songs are rhymes. 11. Statements : Some dews are drops. All drops are stones. Conclusions : I. Atleast some dews are stones. II. Atleast some stones are drops.

    SYLLOGISM Directions (12–16) : In each of the questions below are given four statements followed by four Conclusions numbered I, II, III and IV. You have to take the given statements to be true even if they seem to be at variance from commonly known facts. Read all the Conclusions and then decide which of the given Conclusions logically follows from the given statements disregarding commonly known facts. (SBI Management Executive Exam, 23.02.2014)

    12. Statements : All pens are books. All books are chairs. Some chairs are desks. Some desks are tables. Conclusions : I. Some tables are chairs. II. Some desks are pens. III. Some chairs are pens. IV. All pens are chairs. (1) Only I and II follow (2) Only I and III follow (3) Only I and IV follow (4) Only III and IV follow (5) None of these 13. Statements : Some trains are buses. Some buses are trucks. Some trucks are boats. Some boats are cars. Conclusions : I. Some trucks are trains. II. Some cars are trucks. III. Some boats are buses. IV. Some boats are trains. (1) None follows (2) Only I and II follow (3) Only III follows (4) Only IV follows (5) Only III and IV follow 14. Statements : All hills are roads. All roads are stones. All stones are jungles. All jungles are rivers. Conclusions : I. Some rivers are stones. II. Some jungles are hills. III. Some stones are hills. IV. All rivers are jungles. (1) Only I and II follow (2) Only II and III follow (3) Only I, II and III follow (4) Only II, III and IV follow (5) All follow

    15. Statements : Some books are pens. Some pens are glasses. Some glasses are plates. Some plates are bottles. Conclusions : I. Some bottles are books. II. Some glasses are books. III. Some plates are glasses. IV. Some bottles are pens. (1) Only I and II follow (2) Only III follows (3) Only I, II and III follow (4) Only III and IV follow (5) Only IV follows 16. Statements : Some petals are flowers. All flowers are desks. Some desks are cards. All cards are trains. Conclusions : I. Some desks are flowers. II. Some desks are petals. III. Some petals are cards. IV. Some desks are trains. (1) Only I and IV follow (2) Only II, III and IV follow (3) Only III and IV follow (4) Only I, II and III follow (5) Only I, II and IV follow Directions (17-21) : In each question below are two or three statements followed by two conclusions numbered I and II. You have to take the given statements to be true even if they seem to be at variance from commonly known facts and then decide which of the given conclusions logically follows from the given statements disregarding commonly known facts. (SBI Specialist Officer (Law Officer : MMGS Scale-II) Online Exam, 19.04.2014)

    Give answer (1) if only conclusion I follows. Give answer (2) if only conclusion II follows. Give answer (3) : if either conclusion I or II follows. Give answer (4) if neither conclusion I or II follows. Give answer (5) if both conclusions I and II follow. (17-18) : Statements : All crops are fields. No yield is a field. All fields are harvests.

    BPRE–523

    17.

    Conclusions : I. No field is a crop. II. All crops being harvest is a possibility. 18. Conclusions : I. All harvests being yield is a possibility. II. All harvests are fields. 19. Statements : Some trades are exports. All businesses are trades. Conclusions : I. At least some businesses are exports. II. All businesses being exports is a possibility. (20-21) : Statements : Some countries are towns. All countries are districts. All districts are villages. 20. Conclusions : I. At least some towns are villages. II. All countries are villages. 21. Conclusions : I. At least some districts are towns. II. All towns are villages. Directions (22-26) : In each question below are given three statements followed by two conclusions numbered I and II. You have to assume everything in the statements to be true even if they seem to be at variance from commonly known facts and then decide which of the given conclusions logically follows from the information given in the statements. (SBI Probationary Officer Online Exam, 21.06.2014)

    22. Statements : No star is a cone. Some cones are triangles. All kites and stars. Conclusions : I. All stars are kites. II. At least some triangles and stars. (1) Only Conclusion I follows (2) Only Conclusion II follows (3) Either Conclusion I or Conclusion II follows. (4) There is possibility that some stars are triangles. (5) Neither Conclusion I nor Conclusion II follows.

    SYLLOGISM 23. Statements : All drums are banjos. Some drums are guitars. No banjo is a flute. Conclusions : I. Some guitars are flutes. II. No guitar is flute. (1) Only Conclusion I follows (2) Only Conclusion II follows (3) There is possibility that some flutes are banjos. (4) Either Conclusion I or Conclusion II follows (5) Both Conclusions I and II follow 24. Statements : Some pins are needles. All needles ae swords. Some swords are knives. Conclusions : I. All swords being pins is a possibility. II. No neele being knife is a possibility. (1) Only Conclusion I follows (2) Only Conclusion II follows (3) Either Conclusion I or Conclusion II follows (4) Neither Conclusion I nor Conclusion II follows (5) There is possibility that some pins are knives. 25. Statements : Some schemes are offers. Some offers are discounts. No discount is a loan. Conclusions : I. Those offers which are discounts can never be loans. II. Some loans are definitely schemes. (1) Only Conclusion I follows (2) There is possibility that all schemes are loans (3) Only Conclusion II follows (4) Either Conclusion I or Conclusion II follows (5) Neither Conclusion nor Conclusion II follows 26. Statements : No car is hotel. All lodges are hotels. No lodge is house. Conclusions : I. Some houses are not lodges. II. No lodge is car. (1) There is possibility that all houses are cars.

    (2) Only Conclusion I follows (3) Only Conclusion II follows (4) Neither Conclusion I nor Conclusion II follows (5) Both Conclusion I and Conclusion II follow Directions (27–31) : In each question given below are given three statements followed by two Conclusions numbered I and II. You have to assume everything in the statement to be true even if they seem to be at variance from commonly known facts and then decide which of the given Conclusions logically follows from the information given in the statements disregarding commonly known facts. (SBI Probationary Officer Online Exam, 28.06.2014)

    27. Statements : All papers are wood. Some wood are metals. All metals are glasses. Conclusions I. At least some glasses are wood. II. Some glasses are metals. (1) There is possibility that some papers are glasses. (2) Only Conclusion I follows (3) Only Conclusion II follows (4) Both Conclusion I and Conclusion II follow (5) Neither Conclusion I nor Conclusion II follows 28. Statements Some stones are rocks. Some rocks are diamonds. Some diamonds are gems. Conclusions I. Some gems are stones. II. All diamonds are stones. (1) Neither Conclusion I nor Conclusion II follows (2) Only Conclusion I follows (3) Only Conclusion II follows (4) All stones being gems is a possibility (5) Either Conclusion I or Conclusion II follows 29. Statements All days are nights. All evenings are nights. All nights are mornings. Conclusions I. All days being mornings is a possibility. II. Some mornings are evenings. (1) Only Conclusion I follows

    BPRE–524

    (2) Both Conclusion I and Conclusion II follow (3) Neither Conclusion I nor Conclusion II follows (4) Only Conclusion II follows (5) There is possibility that no day is morning. 30. Statements Some pins are nails. All nails are hammers. All hammers are needles. Conclusions I. All needles are pins. II. All nails are needles. (1) All pins are hammers. (2) Only Conclusion I follows (3) Only Concluision II follows (4) Either Conclusion I or Conclusion II follows (5) Neither Conclusion I nor Conclusion II follows 31. Statements All leaves are roots. No root is a tree. All trees are bushes. Conclusions I. No leaf being a tree is a possibility. II. Some bushes are not leaves. (1) Only Conclusion I follows (2) Only Conclusion II follows (3) Either Conclusion I or Conclusion II follows (4) Neither Conclusion I nor Conclusion II follows (5) Both Conclusion I and Conclusion II follow Directions (32-35) : In each question below are two or three statements followed by two conclusions numbered I and II. You have to take the given statements to be true even if they seem to be at variance from commonly known facts and then decide which of the given conclusions logically follows from the given statements disregarding commonly known facts. (SBI Management Executive Exam, 19.09.2014)

    Give answer (1) if only conclusion I follows. Give answer (2) if only conclusion II follows. Give answer (3) if either conclusion I or II follows. Give answer (4) if neither conclusion I or II follows. Give answer (5) if both conclusions I and II follow.

    SYLLOGISM 32. Statements % All territories are limits. All limits are constraints. No region is constraint. Conclusions % I. Some territories are not regions. II. At least some constraints are territories. 33. Statements % All vapours are droplets. Some clouds are vapours. Conclusions % I. Some clouds are droplets. II. No droplet is cloud. 34. Statements % All mistakes are flaws. All corrections are solutions. No flaw is solution. Conclusions% I. Some flaws are solutions. II. No correction is flaw. 35. Statements % All zeroes are numbers. All digits are alphabets. No alphabet is number. Conclusions % I. No number is digit. II. No zero is digit. Directions (36–41) : In each question given below two/three statements followed by two conclusions numbered I and II have been given. You have to take the given statements to be true if they seem to be at variance from the commonly known facts and then decide which of the given conclusions logically follows from the given statements disregarding commonly known facts. (SBI Associates PO Online Exam, 29.11.2014)

    (36–37) : Statements : All bugs are worms. Some worms are moths. No moth is a fly. 36. Conclusions : I. No fly is a worm. II. All moths being bugs is a possibility. (1) Either Conclusion I or Conclusion II follows (2) Both Conclusions I and II follow (3) Neither Conclusion I nor Conclusion II follows (4) Only Conclusion II follows (5) Only Conclusion I follows

    37. Conclusions : I. All worms are bugs. II. No bug is a fly. (1) Either Conclusion I or Conclusion II follows (2) Both Conclusions I and II follow (3) Neither Conclusion I nor Conclusion II follows (4) Only Conclusion II follows (5) Only Conclusion I follows (38–39) : Statements : No magic is a trick. All charms are tricks. All hoaxes are charms. 38. Conclusions : I. No magic is a hoax. II. No charm is a magic. (1) Either Conclusion I or Conclusion II follows (2) Both Conclusions I and II follow (3) Neither Conclusion I nor Conclusion II follows (4) Only Conclusion II follows (5) Only Conclusion I follows 39. Conclusions : I. All hoaxes are tricks. II. All tricks are charms. (1) Either Conclusion I or Conclusion II follows (2) Both Conclusions I and II follow (3) Neither Conclusion I nor Conclusion II follows (4) Only Conclusion II follows (5) Only Conclusion I follows 40. Statements : Some stars are planets. No planet is a moon. Conclusions : I. No star is a moon. II. Some stars are moons. (1) Either Conclusion I or Conclusion II follows (2) Both Conclusions I and II follow (3) Neither Conclusion I nor Conclusion II follows (4) Only Conclusion II follows (5) Only Conclusion I follows 41. Statements : All watches are clocks. Some clocks are towers. All towers are poles.

    BPRE–525

    Conclusions : I. At least some poles are clocks. II. All towers being watches is a possibility. (1) Either Conclusion I or Conclusion II follows (2) Both Conclusions I and II follow (3) Neither Conclusion I nor Conclusion II follows (4) Only Conclusion II follows (5) Only Conclusion I follows Directions (42–47) : In each question given below two/three statements followed by two conclusions numbered I and II have been given. You have to take the given statements to be true even if they seem to be at variance from the commonly known facts and then decide which of the given conclusions logically follows from the given statements disregarding commonly known facts. (SBI Associates PO Online Exam, 30.11.2014)

    (42–43) : Statements % Some prices are costs. Some costs are amounts. All amounts are expenses. 42. Conclusions : I. At least some amounts are prices. II. All amounts being prices is a possibility. (1) Either Conclusion I or II follows (2) Both Conclusions I and II follow (3) Neither Conclusion I nor II follows (4) Only Conclusion II follows (5) Only Conclusion I follows 43. Conclusions : I. Atleast some expenses are costs. II. No expense is a cost. (1) Either Conclusion I or II follows (2) Both Conclusions I and II follow (3) Neither Conclusion I nor II follows (4) Only Conclusion II follows (5) Only Conclusion I follows

    SYLLOGISM 44. Statements : All invitations are rejections. Some invitations are celebrations. No rejection is an attraction. Coclusions : I. Some celebrations are rejections. II. All celebrations are rejections. (1) Either Conclusion I or II follows (2) Both Conclusions I and II follow (3) Neither Conclusion I nor II follows (4) Only Conclusion II follows (5) Only Conclusion I follows 45. Statements : All grades are scales. All scales are categories. Conclusions : I. All grades are categories. II. All categories are scales. (1) Either Conclusion I or II follows (2) Both Conclusions I and II follow (3) Neither Conclusion I nor II follows (4) Only Conclusion II follows (5) Only Conclusion I follows (46-47) : Statements : Some metals are papers. All papers are alloys. No alloy is a wood. 46. Conclusions : I. All woods being metals is a possibility. II. All metals being woods is a possibility. (1) Either Conclusion I or II follows (2) Both Conclusions I and II follow (3) Neither Conclusion I nor II follows (4) Only Conclusion II follows (5) Only Conclusion I follows 47. Statements : I. No paper is a wood. II. Atleast some meals are alloys. (1) Either Conclusion I or II follows (2) Both Conclusions I and II follow

    (3) Neither Conclusion I nor II follows (4) Only Conclusion II follows (5) Only Conclusion I follows Directions (48-51) : In each question below are two or three statements followed by two conclusions numbered I and II. You have to take the given statements to be true even if they seem to be at variance from commonly known facts and then decide which of the given conclusions logically follows from the given statements disregarding commonly known facts. (SBI Management Executive Exam. 19.09.2014)

    Give answer (1) if only conclusion I follows. Give answer (2) if only conclusion II follows. Give answer (3) if either conclusion I or II follows. Give answer (4) if neither conclusion I or II follows. Give answer (5) if both conclusions I and II follow. 48. Statements % All territories are limits. All limits are constraints. No region is constraint. Conclusions % I. Some territories are not regions. II. At least some constraints are territories. 49. Statements % All vapours are droplets. Some clouds are vapours. Conclusions % I. Some clouds are droplets. II. No droplet is cloud. 50. Statements % All mistakes are flaws. All corrections are solutions. No flaw is solution. Conclusions% I. Some flaws are solutions. II. No correction is flaw. 51. Statements % All zeroes are numbers. All digits are alphabets. No alphabet is number. Conclusions % I. No number is digit. II. No zero is digit.

    BPRE–526

    Directions (52–57) : In each question given below two/three statements followed by two conclusions numbered I and II have been given. You have to take the given statements to be true if they seem to be at variance from the commonly known facts and then decide which of the given conclusions logically follows from the given statements disregarding commonly known facts. (SBI Associates PO Online Exam. 29.11.2014)

    (52–53) : Statements : All bugs are worms. Some worms are moths. No moth is a fly. 52. Conclusions : I. No fly is a worm. II. All moths being bugs is a possibility. (1) Either Conclusion I or Conclusion II follows (2) Both Conclusions I and II follow (3) Neither Conclusion I nor Conclusion II follows (4) Only Conclusion II follows (5) Only Conclusion I follows 53. Conclusions : I. All worms are bugs. II. No bug is a fly. (1) Either Conclusion I or Conclusion II follows (2) Both Conclusions I and II follow (3) Neither Conclusion I nor Conclusion II follows (4) Only Conclusion II follows (5) Only Conclusion I follows (54–55) : Statements : No magic is a trick. All charms are tricks. All hoaxes are charms. 54. Conclusions : I. No magic is a hoax. II. No charm is a magic. (1) Either Conclusion I or Conclusion II follows (2) Both Conclusions I and II follow (3) Neither Conclusion I nor Conclusion II follows (4) Only Conclusion II follows (5) Only Conclusion I follows

    SYLLOGISM 55. Conclusions : I. All hoaxes are tricks. II. All tricks are charms. (1) Either Conclusion I or Conclusion II follows (2) Both Conclusions I and II follow (3) Neither Conclusion I nor Conclusion II follows (4) Only Conclusion II follows (5) Only Conclusion I follows 56. Statements : Some stars are planets. No planet is a moon. Conclusions : I. No star is a moon. II. Some stars are moons. (1) Either Conclusion I or Conclusion II follows (2) Both Conclusions I and II follow (3) Neither Conclusion I nor Conclusion II follows (4) Only Conclusion II follows (5) Only Conclusion I follows 57. Statements : All watches are clocks. Some clocks are towers. All towers are poles. Conclusions : I. At least some poles are clocks. II. All towers being watches is a possibility. (1) Either Conclusion I or Conclusion II follows (2) Both Conclusions I and II follow (3) Neither Conclusion I nor Conclusion II follows (4) Only Conclusion II follows (5) Only Conclusion I follows Directions (58–63) : In each question given below two/three statements followed by two conclusions numbered I and II have been given. You have to take the given statements to be true even if they seem to be at variance from the commonly known facts and then decide which of the given conclusions logically follows from the given statements disregarding commonly known facts. (SBI Associates PO Online Exam. 30.11.2014)

    (58–59) : Statements % Some prices are costs. Some costs are amounts. All amounts are expenses. 58. Conclusions : I. At least some amounts are prices. II. All amounts being prices is a possibility. (1) Either Conclusion I or II follows (2) Both Conclusions I and II follow (3) Neither Conclusion I nor II follows (4) Only Conclusion II follows (5) Only Conclusion I follows 59. Conclusions : I. Atleast some expenses are costs. II. No expense is a cost. (1) Either Conclusion I or II follows (2) Both Conclusions I and II follow (3) Neither Conclusion I nor II follows (4) Only Conclusion II follows (5) Only Conclusion I follows 60. Statements : All invitations are rejections. Some invitations are celebrations. No rejection is an attraction. Coclusions : I. Some celebrations are rejections. II. All celebrations are rejections. (1) Either Conclusion I or II follows (2) Both Conclusions I and II follow (3) Neither Conclusion I nor II follows (4) Only Conclusion II follows (5) Only Conclusion I follows 61. Statements : All grades are scales. All scales are categories. Conclusions : I. All grades are categories. II. All categories are scales. (1) Either Conclusion I or II follows (2) Both Conclusions I and II follow (3) Neither Conclusion I nor II follows (4) Only Conclusion II follows (5) Only Conclusion I follows

    BPRE–527

    (62-63) : Statements : Some metals are papers. All papers are alloys. No alloy is a wood. 62. Conclusions : I. All woods being metals is a possibility. II. All metals being woods is a possibility. (1) Either Conclusion I or II follows (2) Both Conclusions I and II follow (3) Neither Conclusion I nor II follows (4) Only Conclusion II follows (5) Only Conclusion I follows 63. Statements : I. No paper is a wood. II. Atleast some meals are alloys. (1) Either Conclusion I or II follows (2) Both Conclusions I and II follow (3) Neither Conclusion I nor II follows (4) Only Conclusion II follows (5) Only Conclusion I follows Directions (64–68) : In each question given below two or three statements followed by two Conclusions numbered I and II have been given. You have to take the given statements to be true even if they seem to be at variance from the commonly known facts and then decide which of the following Conclusions logically follows from the given statements, disregarding commonly known facts. (SBI PO Phase–I (Preliminary) Online Exam. 20.06.2015)

    Give answer (1) if only Conclusion I follows Give answer (2) if only Conclusion II follows Give answer (3) if either Conclusion I or Conclusion II follows Give answer (4) if neither Conclusion I nor Conclusion II follows Give answer (5) if both the Conclusions I and II follow (64–65) : Statements All magazines are journals. Some journals are periodicals. All periodicals are bulleteins. 64. Conclusions I. Some periodicals are definitely not journals. II. All periodicals being magazines is a possibility.

    SYLLOGISM 65. Conclusions I. At least some bulleteins are journals. II. No bulletein is a magazine. 66. Statements All turns are loops. No loop is a bend. Some bends are curves. Conclusions I. At least some curves are loops. II. No bend is a turn. 67. Statements No country is a village. All villages are districts. Conclusions I. All countries are districts. II. All districts are villages. 68. Statements All progress are growth. All developments are growth. No growth is an evolution. Conclusions I. All developments being progress is a possibility. II. No evolution is a progress. Directions (69 – 73) : In each of the following questions, two/three statements followed by two conclusions numbered I and II have been given. You have to take the given statements to be true even if they seem to be at variance form the cdommonly known facts and then decide which of the given conclusions logically follows from the given statements disregarding commonly known facts. (SBI PO Phase–I (Preliminary) Online Exam. 21.06.2015)

    Give answer (1) if only Conclusion I follows Give answer (2) if only Conclusion II follows Give answer (3) if either Conclusion I or Conclusion II follows. Give answer (4) if both the Conclusion I and Conclusion II follow Give answer (5) if neither Conclusion I nor Conclusion II follows (69 – 73) : Statements : Some wins are trophies. Some trophies are cups. No cup is a prize. 69. Conclusions I. Atleast some cups are wins. II. All prizes being trophies is a possibility.

    70. Conclusions I. No trophy is a prize. II. No prize is a win. 71. Statements No layer is a coat. All coats are deposits. All deposits are sheets. Conclusions I. All coats are sheets. II. All deposits can never be layers. 72. Statements Some flats are apartments. No apartment is a hall. Some halls are rooms. Conclusions I. Atleast some rooms are flats. II. No apartment is a room. 73. Statements Some codes are secrets. All secrets are puzzles. Conclusions I. All secrets being codes is a possibility. II. Atleast some puzzles are coses. Directions (74 – 78) : In each of the following questions, two/three statements followed by two conclusions numbered I and II have been given. You have to take the given statements to be true even if they seem to be at variance from commonly known facts and then decide which of the given conclusions logically follows from the given statements disregarding commonly known facts. (SBI PO Phase–I (Preliminary) Online Exam. 27.06.2015)

    Give answer (1) if only Conclusion I follows Give answer (2) if neither Conclusion I nor Conclusion II follows Give answer (3) if only Conclusion II follows Give answer (4) if both Conclusion I and Conclusion II follow Give answer (5) if either Conclusion I or Conclusion II follows 74. Statements No factory is an industry. All workshops are industries. Some plants are workshops. Conclusions I. No workshop is a factory. II. Atleast some plants are industries.

    BPRE–528

    75. Statements Some sands are particles. Some particles are glasses. Conclusions I. Some glasses are definitely not particles. II. Some glasses being sands is a possibility. 76. Statements Some movies are films. No film is a show. All shows are pictures. Conclusions I. Atleast some pictures are films. II. No show is a movie. (77– 78) : Statements Some actors are singers. All singers are dancers. Some dancers are players. 77. Conclusions I. All actors being dancers is a possibility. II. All dancers are singers. 78. Conclusions I. Atleast some dancers are actors. II. No player is an actor. Directions (79–81) : In each of the following questions, three statements followed by two Conclusions numbered I and II have been given. You have to take the given statements to be true even if they seem to be at variance from commonly known facts and then decide which of the given Conclusions logically follows from the given statements disregarding commonly known facts. (SBI PO Online Main Exam, 31.07.2016)

    Give answer (1) if only Conclusion I follows Give answer (2) if only Conclusion II follows Give answer (3) if both the Conclusion I and Conclusion II follow Give answer (4) if either Conclusion I or Conclusion II follows Give answer (5) if neither Conclusion I nor Conclusion II follows 79. Statements : Some metres are weights. All kilograms are distances. No metre is distance. Conclusions : I. No kilogram is a metre. II. All weights being distances is a possibility.

    SYLLOGISM (80–81) : Statements : All friends are relatives. All relatives are knowns. No known is a guest. 80. Conclusions : I. No friend is a guest. II. No relative is a guest. 81. Conclusions : I. All knowns are relatives. II. All friends are knowns. Directions (82–83) : In each of the following questions four statements followed by five Conclusions are given as five options (1), (2), (3), (4) and (5). You have to take given statements to be true even if they seem to be at variance from commonly known facts and then decide which of the given Conclusions logically follows from the given statements disregarding commonly known facts. (SBI PO Online Main Exam, 31.07.2016)

    82. Statements : Some memoranda are tenders. All circulars are recommendations. No memorandum is a circular. All orders are circulars. Conclusions : (1) Some memoranda are recommendations. (2) Some orders are tenders. (3) All circulars being tenders is a possibility. (4) No recommendation is an order (5) No order is a memorandum 83. Statements : Some pencils are pens. No pen is a computer. Some computers are mobiles. All mobiles are tablets. Conclusions : (1) All tablets being pencils is a possibility. (2) No pencil is a computer. (3) Some mobiles are not pencils. (4) All tablets being pens is a possibility. (5) All computers are tablets Directions (84–87) : In each of the following questions, three statements followed by two Conclusions I and II have been given. You have to take the given statements to be true

    even if they seem to be at variance from the commonly known facts and then decide which of the given Conclusions logically follows from the given statements. Give answer (1) if only Conclusion I follows Give answer (2) if only Conclusion II follows Give answer (3) if either Conclusion I or Conclusion II follows Give answer (4) if neither Conclusion I nor Conclusion II follows Give answer (5) if both the Conclusion I and Conclusion II follow. 84. Statements : All watches are rings. Only a few rings are carrots. Only a few carrots are phones. Conclusions : I. No ring is a phone. II. All watches can never be carrots. 85. Statements : No test is a paper. Only a few papers are registers. No register is a book. Conclusions : I. All tests being registers is a possibility. II. No paper is a book. 86. Statements : Only a few breads are juices. Only a few juices are toasters. All toasters are pans. Conclusions : I. All breads being toasters is a possibility. II. Atleast some juices are pans. 87. Statements : Only a few wires are loops. All loops are circles. No circle is a rectangle. Conclusions : I. Atleast some wires are circles. II. All wires can never be rectangles. Directions (88–91) : In each of the questions below, some statements are given followed by two Conclusions numbered I and II. You have to assume all the statements to be true even if they seem to be at variance from the commonly known facts and then decide which of the given two Conclusions logically follow(s) from the information given in the statements. (SBI PO Prelim Exam, 09.06.2019)

    BPRE–529

    Give answer (1) if either Conclusion I or Conclusion II follows Give answer (2) if both the Conclusion I and Conclusion II follow Give answer (3) if neither Conclusion I nor Conclusion II follows Give answer (4) if only Conclusion I follows Give answer (5) if only Conclusion II follows 88. Statements : All marks are signs. All potatoes are signs. All readers are signs. Conclusions : I. All marks being potatoes is a possibility. II. Atleast some readers are marks. 89. Statements : No potato is a radish. Only a few coconuts are potatoes. Only a few basils are radishes. Conclusions : I. All basils can never be potatoes. II. Some radishes are definitely not coconuts. 90. Statements : No fruit is a cottage. Only a few fruits are toasts. All plums are cottages. Conclusions : I. All toasts being cottages is a possibility. II. Some plums are fruits. 91. Statements : Only a few meetings are luxuries. All posters are meetings. No test is a luxury. Conclusions : I. All luxuries can never be posters. II. All tests being meetings is a possibility.

    RBI GRADE–B/ NABARD GRADE–A OFFICER EXAMS Directions (17-6) : In each question below are two/three statements followed by two conclusions numbered I and II. You have to take the two/ three given statements to be true even if they seem to be at variance from commonly known facts and then de-

    SYLLOGISM cide which of the given conclusions logically follows from the given statements disregarding commonly known facts. (RBI Grade-B Officer’s Exam. 18.12.2011)

    Give answer (1) if only conclusion I follows. Give answer (2) if only conclusion II follows. Give answer (3) if either conclusion I or conclusion II follows. Give answer (4) if neither conclusion I nor conclusion II follows. Give answer (5) if both conclusion I and conclusion II follow. (1-2): Statements : Some colours are paints. All colours are varnishes. No varnish is dye. 1.Conclusions : I. No paint is dye. II. All paints being varnishes is a possibility. 2.Conclusions : I. Some varnishes are paints. II. No dye is colour. (3-4) : Statements : All squares are triangles. No triangle is circle. All circles are rectangles. 3.Conclusions : I. No rectangle is square. II. All rectangles being square is a possibility. 4.Conclusions : I. No square is circle. II. Atleast some circles are squares. 5.Statements : No paper is book. Some books are libraries. Conclusions : I. All libraries being books is a possibility. II. No library is paper. 6.Statements : All hills are mountains. All mountains are rocks. Conclusions : I. All those rocks which are mountains are also hills. II. All hills are rocks.

    Directions (7-12) : In each question below are two or three statements followed by two conclusions numbered I and II. You have to take the two given statements to be true even if they seem to be at variance from commonly known facts and then decide which of the given conclusions logically follows from the given statements disregarding commonly known facts. (RBI Officer Grade ‘B’ Exam. 25.08.2013)

    Give answer (1) if only conclusion I follows. Give answer (2) if only conclusion II follows. Give answer (3) if either conclusion I or II follows. Give answer (4) if neither conclusion I nor II follows. Give answer (5) if both conclusions I and II follow. (7-8) : Statements : Some perfumes are scents. No scent is a bar. No perfume is a can. 7.Conclusions : I. All scents can never be cans. II. Some bars are cans. 8.Conclusions : I. Some cans are scents. II. Some bars are perfumes. 9.Statements : No shop is a factory. Some factories are industries. All industries are machines. Conclusions : I. No industry is a shop. II. At least some machines are factories. (10-11) : Statements : All classes are diamonds. No diamond is store. All rooms are classes. 10. Conclusions : I. All rooms are diamonds. II. At least some diamonds are classes. 11. Conclusions : I. Some stores are classes. II. No room is a store. 12. Statements : Some prizes are winners. All winners are students.

    BPRE–530

    Conclusions : I. At least some winners are prizes. II. At least some students are winners. Directions (13-18) : In each question below are three statements followed by two conclusions numbered I and II. You have to take the given statements to be true even if they seem to be at variance from commonly known facts and then decide which of the given conclusions logically follows from the given statements disregarding commonly known facts. (RBI Officer Grade ‘B’ Phase-I Exam, 03.08.2014)

    Give answer (1) if only conclusion I follows. Give answer (2) if only conclusion II follows. Give answer (3) if either conclusion I or II follows. Give answer (4) if neither conclusion I nor II follows. Give answer (5) if both conclusions I and II follow. (13-14) : Statements : All triangles are squares. No square is rectangle. Some rectangles are cones. 13. Conclusions : I. Some cones are rectangles. II. All cones are rectangles. 14. Conclusions : I. No triangle is rectangle. II. Some cones being triangles is a possibility. (15-16) : Statements : No aim is vision. All visions are objectives. No objective is goal. 15. Conclusions : I. All goals being aim is a possibility. II. All aims being objective is a possibility. 16. Conclusions : I. No goal is vision. II. All objectives are visions. (17-18) : Statements : All years are ages. Some years are eras. All eras are distances.

    SYLLOGISM 17. Conclusions : I. At least some distances are ages. II. Some eras are definitely not years. 18. Conclusions : I. At least some eras are ages. II. All distances being years is a possibility. Directions (19–20) : In each of the following questions, three statements followed by two Conclusions numbered I and II have been given. You have to take the given statements to be true even if they seem to be at variance from commonly known facts and then decide which of the given Conclusions logically follows from the given statements disregarding commonly known facts. (RBI Officer Grade ‘B’ Phase-I Exam. 21.11.2015)

    Give answer (1) if either Conclusion I or Conclusion II follows Give answer (2) if only Conclusions I follows Give answer (3) if both the Conclusion I and Conclusion II follows Give answer (4) if only Conclusion II follows Give answer (5) if neither Conclusion I nor Conclusion II follows 19. Statements : No hour is a day. All months are days. Some hours are calendars. Conclusions I. No month is an hour. II. All calendars being days is a possibility. 20. Statements : Some comics are books. All comics are novels. No novel is an article. Conclusions : I Some novels are books. II. All articles being books is a possibility. 21. Statements : No liquid is fluid. All solids are liquids. All gases are solids. Conclusions : I. Atleast some solids are fluids. II. All gases are liquids.

    Directions (22–27) : In these questions, two/three statements followed by two conclusions numbered I and II have been given. You have to take the given statements to be true even if they seem to be at variance from the commonly known facts and then decide which of the given conclusions logically follows from the given statements disregarding commonly known facts. (Nabard Officer Grade ‘A’ Online Exam. 01.03.2015)

    Give answer (1) if only Conclusion I follows Give answer (2) if only Conclusion II follows Give answer (3) if either Conclusion I or II follows Give answer (4) if neither Conclusion I nor II follows Give answer (5) if both Conclusions I and II follow 22. Statements : All dancers are performers. All performers are singers. Conclusions : I. All dancers are singers. II. At least some singers are dancers. (23 – 24) : Statements : Some pigeons are woodpeckers. All eagles are woodpeckers. 23. Conclusions : I. Atleast some eagles are pigeons. II. No eagle is a pigeon. 24. Conclusions : I. All woodpeckers being pigeons is a possibility. II. Some eagles are definitely not pigeons. (25 – 26) : Statements : All shows are plays. Some shows are movies. No movies is theatre. 25. Conclusions : I. All theatres being play is a possibility. II. All movies are shows. 26. Conclusions : I. All movies are plays. II. Some shows are definitely not theatres.

    BPRE–531

    27. Statements : Some floors are walls. No door is a wall. Conclusions : I. Some floors are doors. II. No floor is a door. Directions (28–32) : In each of the following questions two or three statements followed by two Conclusions numbered I and II have been given. You have to take the given statements to be true even if they seem to be at variance from the commonly known facts and then decide which of the given Conclusions logically follows from the given statements disregarding commonly known facts. (NABARD Assistant Manager Exam, 15.15.2016)

    Give answer (1) if both Conclusion I and Conclusion II follow Give answer (2) if neither Conclusion I nor Conclusion II follows Give answer (3) if only Conclusion I follows Give answer (4) if only Conclusion II follows Give answer (5) if either Conclusion I or Conclusion II follows (28–29) : Statements : Some roads are houses. Some houses are bungalows. No bungalow is an apartment. 28. Conclusions : I. All roads being apartments is a possibility. II. No apartment is a house. 29. Conclusions : I. Some houses are definitely not apartments. II. At least some roads are bungalows. 30. Statements : All guests are hosts. Some guests are relatives. All relatives are friends. Conclusions : I. All guests are friends. II. No host being friend is a possibility. 31. Statements : Some shops are outlets. Some outlets are factories. All factories are industries. Conclusions : I. At least some shops are factories. II. At least some outlets are industries.

    SYLLOGISM 32. Statements : All certainties are beliefs. Some beliefs are fears. Conclusions : I. At least some fears are certainties. II. No fear is a certainty. Directions (33–38) : In each of the following questions, three statements followed by two Conclusions numbered I and II have been given. You have to take the given statements to be true even if they seem to be at variance from commonly known facts and then decide which of the given Conclusions logically follows from the given statements disregarding commonly known facts. (RBI Officer Grade ‘B’ Phase-I Exam, 04.09.2016 (Shift-I))

    Give answer (1) if neither Conclusion I nor Conclusion II follows Give answer (2) if either Conclusion I or Conclusion II follows Give answer (3) if only Conclusion I follows Give answer (4) if both Conclusion I and Conclusion II follow Give answer (5) if only Conclusion II follows 33. Statements : Some coffee is tea. All tea is water. All water is milk. Conclusions : I. All coffee being water is a possibility. II. All milk is tea. 34. Statements : No sea is sky. Some skies are kites. All kites are balloons. Conclusions : I. Some balloons are seas. II. All balloons being skies is a possibility. 35. Statements : Some stars are planets. Some planets are galaxies. Some galaxies are suns. Conclusions : I. All suns being galaxies is a possibility. II. Some galaxies are stars. 36. Statements : All registers are pens. All pens are pencils. No pencil is rubber.

    Conclusions : I. No register is rubber. II. Some pencils are pens. 37. Statements : Some radios are televisions. Some televisions are telephones. All telephones are computers. Conclusions : I. All computers being televisions is a possibility. II. Some radios being telephones is a possibility. 38. Statements : All cats are dogs. All wolves are dogs. All dogs are jackals. Conclusions : I. All wolves are jackals. II. Atleast some jackals are cats. Directions (39–44) : In these questions, three statements followed by two conclusions numbered I and II have been given. You have to take the given statements to be true even if they seem to be at variance from the commonly known facts and then decide which of the given conclusions logically follows from the given statements disregarding commonly known facts. (RBI Officer Grade ‘B’ Phase-I Exam, 04.09.2016 (Shift-II))

    Give answer (1) if neither conclusion I nor II follows Give answer (2) if only conclusion II follows Give answer (3) if either conclusion I or II follows Give answer (4) if both conclusions I and II follow Give answer (5) if only conclusion I follows 39. Statements All bedsheets are cupboards. Some cupboards are buckets. No bucket is a tap. Conclusions I. All taps are cupboards. II. No bedsheet is a bucket. 40. Statements Some penguins are kangaroos. Some kangaroos are dolphins. All dolphins are elephants. Conclusions I. Atleast some dolphins are penguins. II. All kangaroos being penguins is a possibility.

    BPRE–532

    41. Statements Some plants are animals. Some animals are flowers. Some flowers are insects. Conclusions I. All insects being plants is a possibility. II. Atleast some animals are insects. 42. Statements All metals are gases. All gases are wood. All wood are fabric. Conclusions I. Atleast some fabrics are metals. II. No wood is a metal. 43. Statements Some circles are squares. All squares are triangles. All triangles are hexagons. Conclusions I. Atleast some circles are hexagons. II. All hexagons being circles is a possibility. 44. Statements All planets are moons. Some moons are stars. All stars are suns. Conclusions I. Some planets are definitely not stars. II. All suns being moons is a possibility. Directions (45–47) : In these questions, three statements followed by two Conclusions numbered I and II have been given. You have to take the given statements to be true even if they seem to be at variance from commonly known facts and then decide which of the given Conclusions logically follows from the given statements. (RBI Officer Grade ‘B’ Phase-I Exam 17.06.2017)

    Give answer (1) if both the Conclusion I and Conclusion II follow Give answer (2) if either Conclusion I or Conclusion II follows Give answer (3) if neither Conclusion I nor Conclusion II follows Give answer (4) if only Conclusion I follows Give answer (5) if only Conclusion II follows (45-46) : Statements : Some planets are stars. All stars are comets. No comet is an astronaut.

    SYLLOGISM 45. Conclusions : I. No star is an astronaut. II. All astronauts are planets. 46. Conclusions : I. Some planets being astronauts is a possibility. II. At least some comets are planets. 47. Statements : All trains are buses. All buses are rickshaws. No rickshaw is a plane. Conclusions : I. All trains are rickshaws. II. All planes being trains is a possibility. Directions (48–49) : In these questions, four statements followed by five Conclusions are given, one of which definitely does not logically follow (or is not a possibility) from the given statements. That Conclusion is your answer. (RBI Officer Grade ‘B’ Phase-I Exam 17.06.2017)

    (NOTE : You have to take the four given statements to be true even if they seem to be at variance from commonly known facts and then decide which of the given Conclusions logically does not follow from the given statements.) 48. Statements : No performance is an experiment. Some experiments are blueprints. No blueprint is a table. All blueprints are chairs. (1) Conclusion : Some experiments are definitely not tables. (2) Conclusion : At least some experiments are chairs. (3) Conclusion : All chairs being performances is a possibility. (4) Conclusion : All tables being experiments is a possibility. (5) Conclusion : All blueprints can never be performances. 49. Statements : All stations are platforms. All platforms are dividers. Some platforms are roads. No road is a track.

    (1) Conclusion : All stations are dividers. (2) Conclusion : Some platforms are definitely not tracks. (3) Conclusion : All tracks being stations is a possibility. (4) Conclusion : All dividers being tracks is a possibility (5) Conclusion : Some dividers are roads. Directions (50–52) : In these questions, two/three statements followed by two conclusions numbered I and II have been given. You have to take the given statements to be true even if they seem to be at variance from the commonly known facts and then decide which of the given conclusions logically follows from the given statements. (NABARD Assistant Manager Online Exam 06.08.2017)

    Give answer (1) if either Conclusion I or Conclusion II follows Give answer (2) if only conclusion I follows Give answer (3) if both Conclusion I and Conclusion II follows Give answer (4) if only Conclusion II follows Give answer (5) if neither Cconclusion I nor Conclusion II folows 50. Statements : All digs are fruits. Some cherries are digs. Conclusions : I. All cherries are fruits. II. Some cherries are fruits. 51. Statements : Some trumps are yachts. No van is a trump. All kites are yachts. Conclusions : I. All kites being trumps is a possibility. II. All vans can never be yachts. 52. Statements : No river is a candle. All axes are candles. No jack is a river. Conclusions : I. All jacks being candles is a possibility. II. No axe is a river. Directions (53–57) : In these questions, three statements followed by two Conclusions numbered I and II have been given. You have to take the given statements to be true even if they seem

    BPRE–533

    to be at variance from commonly knwon facts and then decide which of the given Conclusions logically follows. (RBI Assistant Manager Online Exam 25.03.2017)

    Give answer (1) if both Conclusion I and Conclusion II follow Give answer (2) if neither Conclusion I nor Conclusion II follows Give answer (3) if only Conclusion I follows Give answer (4) if only Conclusion II follows Give answer (5) if either Conclusion I or Conclusion II follows (53–54) : Statements : All users are benefits. No benefit is a profit. Some profits are gains. 53. Conclusions : I. All gains being benefits is a possibility. II. No profit is a user. 54. Conclusions : I. All gains can never be users. II. All benefits being gains is a possibility. (55–56) : Statements : Some interviews are talks. All talks are meetings. All conferences are meetings. 55. Conclusions : I. Atleast some meetings are interviews. II. All conferences being interviews is a possibility. 56. Conclusions : I. No conference is a talk. II. Some talks are conferences. 57. Statements : No card is a tag. All tags are labels. All labels are marks. Conclusions : I. Some marks are cards. II. No mark is a card. Directions (58–60) : In each of the following questions, three statements followed by two Conclusions numbered I and II have been given. You have to take the given statements to be true even if they seem to be variance from the commonly known facts and then decide which of the given Conclusions logically follows from the given statements.

    SYLLOGISM Give answer (1) if only Conclusion I follows. Give answer (2) if only Conclusion II follows. Give answer (3) if either Conclusion I or Conclusion II follows. Give answer (4) if neither Conclusion I nor Conclusion II follows. Give answer (5) if both Conclusion I and Conclusion II follow. (SEBI Assistant Manager Exam. 17.11.2018)

    58. Statements : All tonics are solids. All biscuits are tonics. All almonds are biscuits. Conclusions : I. Atleast some solids are biscuits. II. All almonds are tonics. 59. Statements : No coil is a pan. Some hats are pans. No quilt is a coil. Conclusions : I. All coils being hats is a possibility. II. No pan is a quilt. 60. Statements : Some lamps are shafts. All roses are lamps. Some shafts are coins. Conclusions : I. Atleast some lamps are coins. II. All roses being shafts is a possibility. Directions (61 – 65) : In each of the questions below are given three statements followed by two Conclusions numbered I and II. You have to take the given statements to be true even if they seem to be at variance with commonly known facts. Read all the Conclusions and decide which of the given Conclusions logically follows from the given statements disregarding commonly known facts. NABARD Grade A Manager Exam, 15.06.2019

    Give answer (1) if only Conclusion I follows Given answer (2) if only Conclusion II follows Give answer (3) if either Conclusion I or Conclusion II follows. Give answer (4) if neither Conclusion I nor Conclusion II follows Give answer (5) if both the Conclusion I and Conclusion II follow

    61. Statements : All clerks are assistants. No assistant is an officer. All officers are managers. Conclusions : I. No manager is a clerk. II. All assistants being managers is a possibility. 62. Statements: All gliders are parachutes. No parachute is an airplane. All airplanes are helicopters. Conclusions : I. No helicopter is a glider. II. All parachutes being helicopters is a possibility. 63. Statements : All grasses are trees. Only a few trees are forests. All forests are lands. Conclusions : I. All grasses being forests is a possibility. II. Atleast some trees are lands. 64. Statements : Only a few paragraphs are stories. No story is a movie. All movies are videos. Conclusions : I. Atleast some movies are paragraphs. II. All videos can never be stories. 65. Statements : No corn is a pie. Only a few pies are biscuits. Only a few biscuits are dumplings. Conclusions : I. All biscuits can never be corns. II. All dumplings being pies is a possibility. Directions (66–69) : Each of the questions below consists of some statements followed by three Conclusions numbered I, II and III. Consider the given statements to be true even if they seem to be at variance with commonly known facts. Read all the Conclusions and then decide which of the given Conclusions follow(s) logically from the given statements. RBI Grade B Officer Exam, 09.11.2019

    66. Statements : All heights are peaks. No height is a mountain. All mountains are landscapes. Only a few mountains are valleys.

    BPRE–534

    Conclusions : I. Some valleys being heights is a possibility. II. No peak is a landscape. III. Atleast some valleys are landscapes. (1) Only I follows (2) Both I and III follow (3) Only III follows (4) Both I and II follow (5) Both II and III follow 67. Statements : Only a few books are tables. All tables are novels. Only a few tables are chairs. All chairs are drawers. Conclusions : I. Some chairs are definitely not books. II. All novels being chairs is a possibility. III. All tables are drawers. (1) None follows (2) Both I and II follow (3) Only II follows (4) Both I and III follow (5) Both II and III follow 68. Statements : No choice is an option. Only a few options are alternatives. All alternatives are plans. No alternative is a goal. Conclusions : I. All goals being choices is a possibility. II. Atleast some options are plans. III. All choices can never be alternatives. (1) All I, II and III follow (2) Both I and II follow (3) Only III follows (4) Only I follows (5) Both I and III follow 69. Statements : All computers are laptops. No laptop is a smartphone. Only a few smartphones are digilockers. All digilockers are smartwatches. Conclusions : I. Some smartwatches are definitely smartphones. II. All laptops being digilockers is a possibility. III. Some digilockers are not computers.

    SYLLOGISM (1) All I, II and III follow (2) Only II follows (3) Both I and III follow (4) Both II and III follow (5) Only I follows Directions (70–74) : Study the following information carefully and answer the questions given below : RBI Officer Grade 'B' Phase-I Exam, 06.03.2021

    Below are given some statements regarding the rules of Syllogism. Select the appropriate Conclusion(s) in each question. ● A @ B : All As are Bs. ● A + B : Some As are Bs. ● A # B : Some As are not Bs. ● A ! B : No A is B. ● A % B : Some As being Bs is a possibility. ● A > B : All As being Bs is a possibility. 70. Statement : L+N>R Conclusions : I. All Ls being Rs is a possibility. II. Some Rs are not Ns. III. Some Ns are Ls. (1) Only Conclusion I and Conclusion III follow (2) Only Conclusion I and Conclusion II follow (3) Only Conclusion I follows (4) Only Conclusion II and Conclusion III follow (5) Only Conclusion III follows 71. Statement : Y@P%T Conclusions : I. All Ps being Ys is a possibility. II. Some Ts being Ys is a possibility. III. Some Ts are not Ps. (1) Only Conclusion I follows (2) Only Conclusion I and Conclusion III follow (3) Only Conclusion II and Conclusion III follow (4) Only Conclusion I and Conclusion II follow (5) Only Conclusion III follows 72. Statement : Z+H@L Conclusions : I. All Zs are Hs. II. Some Hs are Ls. III. Some Ls are Zs. (1) Only Conclusion II follows (2) Only Conclusion III follows (3) Only Conclusion II and Conclusion III follow

    (4) Only Conclusion I and Conclusion II follow (5) Only Conclusion I and Conclusion III follow 73. Statement : M%L>B Conclusions : I. All Ls are Bs. II. Some Ms being Bs is a possibility. III. All Bs are Ls. (1) Only Conclusion I and Conclusion III follow (2) Only Conclusion II follows (3) Only Conclusion II and Conclusion III follow (4) Only Conclusion I follows (5) Only Conclusion III follows 74. Statement : G#K+M Conclusions : I. Some Ks are not Gs. II. All Ms being Gs is a possibility. III. Some Gs being Ks is a possibility. (1) Only Conclusion II follows (2) Only Conclusion III follows (3) Only Conclusion I follows (4) Only Conclusion I and Conclusion II follow (5) Only Conclusion II and Conclusion III follow

    INSURANCE EXAMS Directions (1–6) : In each of the questions below, three statements are given followed by conclusions /group of conclusions numbered I and II. You have to assume all the statements to be true even if they seem to be at variance from the commonly known facts and then decide which of the given two conclusions logically follows from the information given in the statements. (United India Insurance AO Exam. 27.03.2011)

    Give answer (1) if only conclusion I follows. Give answer (2) if only conclusion II follows Give answer (3) if either conclusion I or conclusion II follows. Give answer (4) if neither conclusion I nor conclusion II follows. Give answer (5) if both conclusions I and II follow. (1–2) : Statements : All pens are books. Some books are pages. All pages are papers.

    BPRE–535

    1. Conclusions : I. No paper is a pen. II. At least some pages are pens. 2. Conclusions : I. All books are papers. II. Some books are papers. (3–4) : Statements : Some Ds are Gs. All Gs are Ks. All Ks are Ls. 3. Conclusions : I. At least some Ds are Ls. II. All Gs are Ls. 4. Conclusions : I. Atleast some Ks are Ds. II. All Ds are Ls. (5–6) Statements : Some files are folders. All folders are pockets. No pocket is a bag. 5. Conclusions : I. All pockets are files. II. All files are bags. 6. Conclusions : I. Atleast some bags are folders. II. All folders are files. Directions (7-11) : In each question below are given two statements followed by two conclusions numbered I and II. You have to take the two given statements to be true even if they seem to be at variance for commonly known facts and then decide which of the given conclusions logically follows from the two given statements. Read both the statements and give answer as : (1) If only Conclusion I follows (2) If only Conclusion II follows (3) If either I or II follows (4) If neighter I nor II follows (General Insurance Corporation AAO Exam. 11.12.2011)

    7.Statements : I. Some Art works are paintings. II. All paintings are Master pieces. Conclusions : I. All Master pieces are paintings II. Some Master pieces are Art works. 8.Statements : I. Some men are Genius. II. No brother is Genius. Conclusions : I. Some brothers are men. II. Some brothers are not men.

    SYLLOGISM 9.

    10.

    11.

    Statements : I. All pants are skirts. II. No shirt is a skirt. Conclusions : I. Some skirts are pants. II. All shirts are pants. Statements : I. All Planes are Tyres. II. All Tyres are engines. Conclusions : I. No Engine is a plane II. No tyre is a plane Statements : I. Some cartoons are funny. II. Some cartoons are silly Conclusions : I. All funny are cartoons

    II. Some silly are cartoons Directions (12–16) : In each question below are given two statements followed by four conclusions numbered I, II, III and IV. You have to take the two given statements to be true even if they seem to be at variance from commonly known facts. Read all the conclusions and then decide which of the given conclusions logically follows/follow from the two given statements disregarding commonly known facts. (Oriental Insurance Company Exam.08.04.2012)

    12.

    13.

    Statements : Some men are home-makers. Some home-makers are women. Conclusions : I. Some men are women. II. Some women are men. III. All women are home-makers. IV. All home-makers are men. (1) All follow (2) None follows (3) Only I and III follow (4) Only II and IV follow. Statements : All scholars are teachers. Some teachers are researchers. Conclusions : I. All scholars are researchers. II. Some scholars are researchers. III. Some researchers are teachers. IV. Some teachers are scholars. (1) None follows (2) Only III and IV follow (3) All follow (4) Only III follows.

    14.

    15.

    16.

    Statements : Some men are boys. No boy is a woman. Conclusions : I. No man is woman II. No boy is man III. Some men are women IV. Some boys are men. (1) All follow (2) None follows (3) Only IV and either I or III follow (4) Only I and III follow Statements : No Manager is a teacher. All teachers are researchers. Conclusions : I. No researcher is a teacher. II. No researcher is a manager. III. Some teachers are researchers. IV. Some researchers are teachers. (1) None follows (2) All follow (3) Only II follows (4) Only III and IV follow Statements : All houses are rooms. All rooms are windows. Conclusions : I. All windows are rooms. II. All rooms are houses. III. All houses are windows. IV. Some windows are houses. (1) None follows (2) Only I and II follow (3) Only III and IV follow (4) Only II follows

    Directions (17–22) : In questions given below, statement 1 and 2 are followed by conclusions I and II. Taking the statements to be true although they appear at variance with commonly accepted facts, mark your answer as under : (United India Insurance AAO Exam.03.06.2012)

    (1) Only conclusion I follows from the statements. (2) Only conclusion II follows from the statements. (3) Both I and II follow from the statements. (4) Neither I nor II follows.

    BPRE–536

    17. Statements : 1. All rats are dogs. 2. Some dogs are bulls. Conclusions : I. Some rats are bulls. II. No bull is a dog. 18. Statements : 1. All women are doctors. 2. All doctors are nurses. Conclusions : I. All nurses are women. II. Some nurses are women. 19. Statements : 1. Some trees are birds. 2. All birds are snails. Conclusions : I. Some snails are trees. II. Some birds are not trees. 20. Statements : 1. A is taller than B but shorter than C and D who is taller than E. 2. E is taller than A but shorter than C who is taller than D. Conclusions : I. B is the shortest. II. C is the tallest. 21. Statements : 1. Only those students were admitted to college who were either hockey players or dancers. 2. All players were singers. Conclusions : I. All singers were admitted to the college. II. Some singers were hockeyplayers as well. 22. Statements : 1. All the cancer patients were cured. 2. All the cured patients grew fat. Conclusions : I. All cancer patients grew fat. II. Patients suffering from other diseases were not cured. Directions (23–28) : In questions given below, statement 1 and 2 are followed by conclusions I and II. Taking the statements to be true although they appear at variance with commonly accepted facts, mark your answer as under : (United India Insurance AAO Exam. 03.06.2012)

    (1) Only conclusion I follows from the statements. (2) Only conclusion II follows from the statements. (3) Both I and II follow from the statements. (4) Neither I nor II follows.

    SYLLOGISM 23. Statements : 1. All rats are dogs. 2. Some dogs are bulls. Conclusions : I. Some rats are bulls. II. No bull is a dog. 24. Statements : 1. All women are doctors. 2. All doctors are nurses. Conclusions : I. All nurses are women. II. Some nurses are women. 25. Statements : 1. Some trees are birds. 2. All birds are snails. Conclusions : I. Some snails are trees. II. Some birds are not trees. 26. Statements : 1. A is taller than B but shorter than C and D who is taller than E. 2. E is taller than A but shorter than C who is taller than D. Conclusions : I. B is the shortest. II. C is the tallest. 27. Statements : 1. Only those students were admitted to college who were either hockey players or dancers. 2. All players were singers. Conclusions : I. All singers were admitted to the college. II. Some singers were hockeyplayers as well. 28. Statements : 1. All the cancer patients were cured. 2. All the cured patients grew fat. Conclusions : I. All cancer patients grew fat. II. Patients suffering from other diseases were not Directions (39–34) : In each group of questions below are two/three statements followed by two conclusions numbered I and II. You have to take the given statements to be true even if they seem to be at variance from commonly known facts and then decide which of the given conclusions logically follows from the two/three statements disregarding commonly known facts.

    Give answer (1) if only conclusion I follows. Give answer (2) if only conclusion II follows Give answer (3) if either conclusion I or conclusion II follows. Give answer (4) if neither conclusion I nor conclusion II follows. Give answer (5) if both conclusions I and II follow. 29. Statements : All pens are nibs. All nibs are inks. No ink is colour. Conclusions : I. All inks are nibs. II. All pens are inks. 30. Statements : All pens are nibs. All nibs are inks. No ink is colour. Conclusions : I. No colour is nib. II. No colour is pen. 31. Statements : No dream is project. All ventures are projects.. Conclusions : I. No venture is dream. II. All projects are ventures. 32. Statements : No road is way. All ways are paths. No path is bridge. Conclusions : I. Some bridges are roads. II. All paths are roads. (33–34) : Statements : Some numbers are digits. No digit is alphabet. All alphabets are letters. 33. Conclusions : I. No letter is digit. II. Some letters are digits. 34. Conclusions : I. Some letters are numbers. II. All numbers can never be alphabets. Directions (35–39) : In each of the questions below are given two or more statements followed by two conclusions numbered I and II. You have to take the given statements to be true even if they seem to be a at variance from commonly known facts. Read all the conclusions and then decide which of the given conclsions logically follows from the given statements disregarding commonly known facts.

    (United India Insurance AO Exam. 26.05.2013)

    (LIC Assistant Administrative Officer (AAO) Exam. 12.05.2013)

    BPRE–537

    Give answer (1) if only conclusion I follows. Give answer (2) if only conclusion II follows Give answer (3) if either conclusion I or conclusion II follows. Give answer (4) if neither conclusion I nor conclusion II follows. Give answer (5) if both conclusions I and II follow. 35. Statements : No book is a page. All pages are markers. All markers are articles. Conclusions : I. All markers can never be books. II. All books are definitely articles. 36. Statements : Some fruits are apples. All apples are guavas. No guava is a banana. Conclusions : I. All guavas are fruits. II. Some guavas are fruits. 37. Statements : All locks are keys. Some keys are pockets. Conclusions : I. No pocket is lock. II. At least some pockets are keys. 38. Statements : No books is a page. All pages are markers. All markers are articles. Conclusions : I. All pages are articles. II. At least some articles are markers. 39. Statements : Some fruits are apples. All apples are guavas. No guava is a banana. Conclusions : I. All fruits are bananas. II. No apple is a banana. 40. In each of the following questions two statements are given and these statements are followed by two conclusions numbered (1) and (2). You have to take the given two statements to be true even if they seem to be at variance from commonly known facts. Read the conclusions and then decide which of the given conclusions logically follows from the two given statements, disregarding commonly known facts.

    SYLLOGISM Statements : Some mangoes are yellow. Some tixo are mangoes. Conclusions : 1. Some mangoes are green. 2. Tixo is a yellow. (1) Only (1) conclusion follows (2) Only (2) conclusion follows (3) Either (1) or (2) follows (4) Neither (1) nor (2) follows (NICL (GIC) AO (Finance) Exam, 15.12.2013)

    Directions (41–46) : In these questions, two/three statements followed by two conclusions numbered I and II have been given. You have to take the given statements to be true even if they seem to be at variance from the commonly known facts and then decide which the given conclusions logically follows from the given statements disregarding commonly known facts. (NIACL Administrative Officer (AO) Exam, 10.01.2015)

    41. Statements : All hosts are guests. All visitors are hosts. Some visitors are invitees. Conclusion I : Atleast some guests are invitees. Conclusion II : All invitees being hosts is a possibility. (1) Either conclusion I or II follows (2) Neither conclusion I nor II follows (3) Both conclusions I and II follow (4) Only conclusion I follows (5) Only conclusion II follows 42. Statements : All hosts are guests. All visitors are hosts. Some visitors are invitees. Conclusion I : No visitor is a guest. Conclusion II : All hosts are visitors. (1) Either conclusion I or II follows (2) Neither conclusion I nor II follows (3) Both conclusions I and II follow (4) Only conclusion I follows (5) Only conclusion II follows

    43. Statements : All gadgets are instruments. Some instruments are devices. All devices are tools. Conclusion I : Atleast some tools are instruments. Conclusion II : All tools being gadgets is a possibility. (1) Either conclusion I or II follows (2) Neither conclusion I nor II follows (3) Both conclusions I and II follow (4) Only conclusion I follows (5) Only conclusion II follows 44. Statements : No cup is a medal. All medals are trophies. No trophy is a shield. Conclusion I : Some shields being cups is a possibility. Conclusion II : No trophy is a cup. (1) Either conclusion I or II follows (2) Neither conclusion I nor II follows (3) Both conclusions I and II follow (4) Only conclusion I follows (5) Only conclusion II follows 45. Statements : No cup is a medal. All medals are trophies. No trophy is a shield. Conclusion I : All trophies are medals. Conclusion II : No shield is a medal. (1) Either conclusion I or II follows (2) Neither conclusion I nor II follows (3) Both conclusions I and II follow (4) Only conclusion I follows (5) Only conclusion II follows 46. Statements : Some dreams are aims. Some aims are objectives. Conclusion I : All objectives being dreams is a possibility. Conclusion II : No objective is a dream. (1) Either conclusion I or II follows (2) Neither conclusion I nor II follows (3) Both conclusions I and II follow

    BPRE–538

    (4) Only conclusion I follows (5) Only conclusion II follows Directions (47–52) : In these questions, two/three statements followed by two Conclusions numbered I and II have been given. You have to take the given statements to be true even if they seem to be at variance from the commonly known facts and then decide which the given Conclusions logically follows from the given statements disregarding commonly known facts. (NIACL Administrative Officer (AO) Online Exam, 11.01.2015)

    47. Statements No cone is triangle. All triangles are squares. Conclusions : I. All cones being squares is a possibility. II. All squares are triangles. (1) Both Conclusions I and II follow (2) Either Conclusion I or II follows (3) Only Conclusion I follows (4) Only Conclusion II follows (5) Neither Conclusion I nor II follows 48. Statements Some planes are ship. No ship is a radar. All cars are radars. Conclusions : I. No car is ship. II. All radars being plane is a possibility. (1) Both Conclusions I and II follow (2) Either Conclusion I or II follows (3) Only Conclusion I follows (4) Only Conclusion II follows (5) Neither Conclusion I nor II follows (49-50) : Statements All alphabets are letters. All letters are digits. No digit is a number. 49. Conclusions : I. No alphabet is a digit. II. All alphabets are digits. (1) Both Conclusions I and II follow (2) Either Conclusion I or II follows (3) Only Conclusion I follows (4) Only Conclusion II follows (5) Neither Conclusion I nor II follows

    SYLLOGISM 50. Conclusions : I. No number is an alphabet. II. All numbers being letters is a possibility. (1) Both Conclusions I and II follow (2) Either Conclusion I or II follows (3) Only Conclusion I follows (4) Only Conclusion II follows (5) Neither Conclusion I nor II follows (51-52) : Statements Some troops are soldiers. All soldiers are fighters. No fighter is a warrior. 51. Conclusions : I. All warriors being troops is a possibility. II. Atleast some fighters are troops. (1) Both Conclusions I and II follow (2) Either Conclusion I or II follows (3) Only Conclusion I follows (4) Only Conclusion II follows (5) Neither Conclusion I nor II follows 52. Conclusions : I. Atleast some warriors are soldiers. II. All fighters are soldiers. (1) Both Conclusions I and II follow (2) Either Conclusion I or II follows (3) Only Conclusion I follows (4) Only Conclusion II follows (5) Neither Conclusion I nor II follows Directions (53–60) : In each of these questions, three/four statements followed by two conclusions numbered I and II have been given. You have to take the given statements to be true even if they seem to be at variance from the commonly known facts and then decide which of the given conclusions logically follows from the given statements disregarding commonly known facts. (NIACL Administrative Officer (AO) Online Exam, 12.01.2015)

    53. Statements : All alphabets are letters. All letters are digits. No digit is a number.

    Conclusions % I. No alphabet is a number. II. All alphabets are digits. (1) Either conclusion I or II follows (2) Neither conclusion I nor II follows (3) Both conclusions I and II follow (4) Only conclusion I follows. (5) Only conclusion II follows 54. Statements : All fruits are flowers. No flower is sweet. Some sweets are desserts. Conclusions % I. Some desserts are flowers. II. No dessert is flower. (1) Either conclusion I or II follows (2) Neither conclusion I nor II follows (3) Both conclusions I and II follow (4) Only conclusion I follows (5) Only conclusion II follows 55. Statements : Some planes are ships. No ship is a radar. All cars are radars. Conclusions % I. No car is a ship. II. All radars being planes is a possibility. (1) Either conclusion I or II follows (2) Neither conclusion I nor II follows (3) Both conclusions I and II follow (4) Only conclusion I follows (5) Only conclusion II follows 56. Statements : All locks are keys. All keys are doors. Some doors are windows. Some windows are floors. Conclusions % I. Some keys are windows. II. No floor is door. (1) Either conclusion I or II follows (2) Neither conclusion I nor II follows (3) Both conclusions I and II follow (4) Only conclusion I follows (5) Only conclusion II follows

    BPRE–539

    57. Statements : All rats are cats. No cow is cat. All dogs are cows. Conclusions % I. No dog is rat. II. No dog is cat. (1) Either conclusion I or II follows (2) Neither conclusion I nor II follows (3) Both conclusions I and II follow (4) Only conclusion I follows (5) Only conclusion II follows 58. Statements : Some towers are pillars. Some pillars are buildings. All buildings are flats. No flat is house. Conclusions % I. No building is house. II. Some towers are houses. (1) Either conclusion I or II follows (2) Neither conclusion I nor II follows (3) Both conclusions I and II follow (4) Only conclusion I follows (5) Only conclusion II follows 59. Statements : All cups are bowls. All bowls are trays. Some trays are plates. No plate is spoon. Conclusions % I. Some bowls are plates. II. Some cups are spoons. (1) Either conclusion I or II follows (2) Neither conclusion I nor II follows (3) Both conclusions I and II follow (4) Only conclusion I follows (5) Only conclusion II follows 60. Statements : All books are pens. Some pens are desks. Some desks are chairs. Some chairs are tables Conclusions % I. Some tables are desks. II. Some chairs are pens. (1) Either conclusion I or II follows

    SYLLOGISM (2) Neither conclusion I nor II follows (3) Both conclusions I and II follow (4) Only conclusion I follows (5) Only conclusion II follows Directions (61-65) : In each of the questions given below two/three statements followed by two conclusions numbered I and II have been given. You have to take the given statements to be true even if they seem to be at variance from commonly known facts and then decide which of the given conclusions logically follows from the given statements disregarding commonly known facts. (LIC Assistant Administrative Officer (AAO) Online Exam. 22.03.2015)

    Give answer (1) if neither Conclusion I nor Conclusion II follows Give answer (2) if either Conclusion I or Conclusion II follows Give answer (3) if only Conclusion II follows Give answer (4) if both the Conclusion I and Conclusion II follow Give answer (5) if only Conclusion I follows (61–62) : Statements: Some rooms are stores. All stores are godowns. All godowns are warehouses. 61. Conclusions : I. All rooms are godowns. II. All stores are warehouses. 62. Conclusions : I. All warehouses being rooms is a possibility. II. Atleast some godowns are rooms. 93. Statements: All kittens are cubs. No kitten is a puppy. Conclusions : I. All puppies being cubs is a possibility. II. All cubs are kittens. (64–65) : Statements : No sea is a lake. Some lakes are rivers. All rivers are oceans. 64. Conclusions : I. No sea is a river. II. All oceans are lakes. 65. Conclusions : I. Atleast some oceans are lakes II. All rivers are lakes.

    Directions (66–68) : In each of the following questions, three statements followed by two Conclusions numbered I and II are given. You have to take the given statements to be true even if they seem to be at variance from the commonly known facts and then decide which of the given conclusions logically follows from the given statements disregarding commonly known facts. (OICL Specialist Officer (Finance) Exam. 03.05.2015)

    Give answer (1) if both the Conclusions and conclusion II follow Give answer (2) if either Conclusion I or Conclusion II follows Give answer (3) if only Conclusion I follows Give answer (4) if only Conclusion II follows Give answer (5) if neither Conclusion I nor Conclusion II follows (66–67) : Statements Some villages are cities. All cities are oceans. No ocean is a town. 66. Conclusions : I. All cities being villages is a possibility. II. No town is a village. 67. Conclusions : I. Atleast some oceans are villages. II. No city is a town. 68. Statements Some stones are rocks. No rock is a hill. All hills are mountains. Conclusions : I. Some stones are mountains. II. All mountains are rocks. Directions (69 – 73) : In each of the following questions, two or three statements followed by two Conclusions numbered I and II have been given. You have to assume the given statements to be true even if they seem to be at variance from the commonly known facts and then decide which of the given Conclusions logically follows from the given statements disregarding commonly known facts. (LIC Assistant Administrative Officer (AAO) Online Exam. 05.03.2016)

    Give answer (1) if either Conclusion I or Conclusion II follows Give answer (2) if neither Conclusion I nor Conclusion II follows Give answer (3) if both Conclusion I and Conclusion II follow

    BPRE–540

    Give answer (4) if only Conclusion I follows Give answer (5) if only Conclusion II follows 69. Statements : Some stands are racks. No rack is a box. All boxes are cartons. Conclusions : I. All stands can never be boxes. II. All racks being cartons is a possibility. 70. Statements : All kittens are turtles. Some turtles are puppies. Conclusions : I. Atleast some puppies are kittens. II. No puppy is a kitten. 71. Statements : All papers are mills. All mills are factories. Conclusions : I. All mills are papers. II. All papers are factories. (72–73) : Statements : No perfume is a fragrance. Some perfumes are deodorants. All deodorants are colognes. 72. Conclusions : I. Atleast some perfumes are colognes. II. No fragrance is a deodorant. 73. Conclusions : I. Atleast some deodorants are not fragrances. II. All fragrances being colognes is a possibility Directions (74–78) : In each of the following questions, two/three statements followed by two conclusions numbered I and II have been given. You have to take the given statements to be true even if they seem to be at variance from the commonly known facts and then decide which of the given conclusions logically follow from the given statements disregarding commonly known facts. (LIC Assistant Administrative Officer (AAO) Online Exam. 06.03.2016)

    Give answer (1) if either Conclusion I or Conclusion II follows Give answer (2) if neither Conclusion I nor Conclusion II follows Give answer (3) if both the Conclusion I and Conclusion II follow Give answer (4) if only Conclusion I follows Give answer (5) if only Conclusion II follows

    SYLLOGISM 74. Statements : No cream is a lotion. Some lotions are perfumes. Conclusions : I. All perfumes are lotions. II. No cream is a perfume. (75–76) : Statements: All legumes are pulses. Some pulses are grains. No grain is a crop. 75. Conclusions : I. All legumes are crops. II. All pulses can never be crops. 76. Conclusions : I. All crops being pulses is a possibility. II. Atleast some grains are legumes. 77. Statements : Some oceans are rivers. All oceans are waterfalls. Conclusions : I. All rivers are waterfalls. II. Some rivers are waterfalls. 78. Statements: Some curtains are pillows. All pillows are blankets. All blankets are doormats. Conclusions : I. All doormats being curtains is a possibility. II. All curtains can never be blankets. Directions (79–83) : In each of the questions given below three statements followed by two conclusions numbered I and II have been given. You have to take the given statements to be true even if they seem to be at variance from the commonly known facts and then decide which of the given conslusions logically follows from the given statements disregarding commonly known facts. Give answer (1) if either Conclusion I or Conclusion II follows Give answer (2) if neither Conclusion I nor Conclusion II follows Give answer (3) if only Conclusion I follows Give answer (4) if both Conclusion I and Conclusion II follow Give answer (5) if only Conclusion II follows (79–80) : Statements No cookie is a biscuit. All biscuits are juices. Some juices are fruits.

    79. I. II.

    Conclusions All juices can never be cookies. Some fruits are definitely not cookies. 80. Conclusions I. Atleast some biscuits are fruits. II. All cookies being fruits is a possibility. (81–82) : Statements Some lines are strokes. All strokes are paints. Some paints are colours. 81. Conclusions I. Some paints are definitely not lines. II. All colours being strokes is a possibility. 82. Conclusions I. Some strokes are definitely not colours. II. All lines can never be colours. 83. Statements All days are weeks. Some weeks are months. All months are years. Conclusions I. Atleast some days are years. II. No day is a year. Directions (84–88) : In these questions, two/three statements followed by two Conclusions numbered I and II have been given. You have to take the given statements to be true even if they seem to be at variance from commonly known facts and then decide which of the given Conclusions logically follows from the given statements. (GIC Officer Online Exam 15.05.2017)

    Give answer (1) if only Conclusion I follows. Give answer (2) if only Conclusion II follows. Give answer (3) if either Conclusion I or Conclusion II follows Give answer (4) if neither Conclusion I nor Conclusion II follows Give answer (5) if both the Conclusion I and Conclusion II follow (84–85) : Statements : Some dolphins are tigers. Some bears are dolphins. All elephants are tigers. 84. Conclusions : I. No bear is a tiger. II. All elephants being dolphins is a possibility. 85. Conclusions : I. All tigers are bears. II. No dolphin is an elephant.

    BPRE–541

    (86–87) : Statements : Some kittens are cows. All hyenas are kittens. Some cows are goats. 86. Conclusions : I. All cows being hyenas is a possibility. II. Some kittens are definitely not cows. 87. Conclusions : I. All kittens can never be goats. II. No hyena is a cow. 88. Statements : No rain is a fog. All rains are smogs. Conclusions : I. Atleast some fogs are smogs. II. No smog is fog. Directions (89–93) : In each question below are two/three statements followed by two Conclusions numbered I and II. You have to take the two/three given statements to be true even if they seem to be at variance from commonly known facts and then decide which of the given Conclusions logically follows from the given statements disregarding commonly known facts. Give answer (1) if only Conclusion I follows. Give answer (2) if only Conclusion II follows. Give answer (3) if either Conclusion I or Conclusion II follows. Give answer (4) if neither Conclusion I nor Conclusion II follows. Give answer (5) if both Conclusion I and Conclusion II follow. (LIC AAO Prelim Exam. 04.05.2019)

    89. Statements : Only a few books are readings. All readings are journals. Conclusions : I. All books are journals. II. Some journals are readings. 90. Statements : Only a few woods are trees. Only a few woods are stems. Conclusions : I. Some stems are trees is a possibility. II. Some stems are trees. 91. Statements : All jungles are parks. All parks are lakes. Only a few lakes are rivers. Conclusions : I. All jungles are lakes. II. Some rivers are parks is a possibility.

    SYLLOGISM 92. Statements : No centre is venue. All places are venues. No place is laboratory. Conclusions : I. No place is centre II. No centre is laboratory. 93. Statements : Some guns are steels. All steels are iron. All iron are silvers. Conclusions : I. All guns are silvers. II. Some guns are not silvers. Directions (94–98) : In each of the following questions, three/four statements followed by two Conclusions I and II have been given. You have to take the given statements to be true even if they seem to be at variance from the commonly known facts and then decide which of the given Conclusions logically follows from the given statements. Give answer (1) if only Conclusion I follows Give answer (2) if only Conclusion II follows Give answer (3) if either Conclusion I or Conclusion II follows Give answer (4) if neither Conclusion I nor Conclusion II follows Give answer (5) if both the Conclusion I and Conclusion II follow (LIC AAO Prelim Exam. 05.05.2019)

    (94–95) : Statements : All dresses are coats. Only a few coats are jackets. All jackets are ties. No tie is a shoe. 94. Conclusions : I. Atleast some coats are ties. II. No shoe is a jacket. 95. Conclusions : I. Some dresses are definitely not shoes. II. All dresses being ties is a possibility. 96. Statements : Only a few storms are tornados. Only a few tornados are floods. All floods are rains. Conclusions : I. Some rains being storms is a possibility. II. Atleast some storms are floods. (97–98) : Statements : No page is a book. All books are novels. Only a few novels are scripts. No script is a theory.

    97. I. II. 98. I. II.

    Conclusions : Atleast some pages are scripts. No script is a page. Conclusions : All books can never be theories. All novels being theories is a possibility. Directions (99–101) : In the following questions, some statements followed by some Conclusions are given. You have to consider the statements to be true, irrespective of commonly known facts. Read the Conclusions and decide which of the given Conclusions logically follows from the given statements. LIC AAO Main Exam, 28.06.2019 99. Statements : (1) Some Physics are English. (2) No English is Mathematics. (3) All Mathematics are Economics. Conclusions : I. Some Physics are not Mathematics II. Some English are Economics III. Some Economics are not English (1) Only I and II follow (2) Only II and III follow (3) All I, II and III follow (4) Only I and III follow (5) None follows 100. Statements : (1) All blue are green. (2) Some yellow are green. (3) All red are yellow. Conclusions : I. Some red being blue is a possibility. II. All red are blue III. No blue being yellow is a possibility (1) Only I follows (2) Only II and III follow (3) All I, II and III follow (4) Only I and III follow (5) None follows 101. Statements : (1) All hills are mountains. (2) Some rocks are mountains. (3) All rocks are ropes. Conclusions : I. All hills are ropes. II. No hill being rope is a possibility. III. Some ropes are not hills. (1) Only I and II follow. (2) Only III follows. (3) All I, II and III follow. (4) Only I and III follow. (5) None follows.

    BPRE–542

    SHORT ANSWERS

    NATIONALISED BANKS & IBPS PO/MT/SO EXAMS 1. (4)

    2. (5)

    3. (5)

    4. (2)

    5. (4) 9. (4)

    6. (4) 10. (1)

    7. (4) 11. (5)

    8. (2) 12. (4)

    13. (1)

    14. (3)

    15. (2)

    16. (4)

    17. (1)

    18. (1)

    19. (2)

    20. (4)

    21. (1)

    22. (2)

    23. (1)

    24. (5)

    25. (4)

    26. (5)

    27. (4)

    28. (5)

    29. (1)

    30. (4)

    31. (1)

    32. (5)

    33. (5)

    34. (1)

    35. (3)

    36. (5)

    37. (1)

    38. (4)

    39. (4)

    40. (2)

    41. (3)

    42. (4)

    43. (1)

    44. (5)

    45. (2)

    46. (2)

    47. (1)

    48. (2)

    49. (4)

    50. (2)

    51. (4)

    52. (5)

    53. (5)

    54. (2)

    55. (1)

    56. (1)

    57. (2)

    58. (1)

    59. (5)

    60. (2)

    61. (5)

    62. (1)

    63. (2)

    64. (2)

    65. (1)

    66. (2)

    67. (4)

    68. (3)

    69. (3)

    70. (4)

    71. (2)

    72. (5)

    73. (5)

    74. (5)

    75. (3)

    76. (1)

    77. (4)

    78. (2)

    79. (4)

    80. (4)

    81. (5)

    82. (5)

    83. (3)

    84. (3)

    85. (2)

    86. (4)

    87. (5)

    88. (5)

    89. (3)

    90. (3)

    91. (2)

    92. (4)

    93. (1)

    94. (1)

    95. (4)

    96. (5)

    97. (2)

    98. (1)

    99. (3) 100. (3)

    101. (2) 102. (4) 103. (3) 104. (1) 105. (2) 106. (5) 107. (3) 108. (3) 109. (4) 110. (5) 111. (2) 112. (5) 113. (2) 114. (3) 115. (1) 116. (2) 117. (5) 118. (4) 119. (5) 120. (2) 121. (1) 122. (1) 123. (5) 124. (3) 125. (5) 126. (4) 127. (2) 128. (1) 129. (4) 130. (3) 131. (5) 132. (2) 133. (5) 134. (3) 135. (1) 136. (5) 137. (4) 138. (5) 139. (4) 140. (2) 141. (1) 142. (4) 143. (5) 144. (3) 145. (5) 146. (2) 147. (5) 148. (5) 149. (3) 150. (4) 151. (4) 152. (3) 153. (2) 154. (4) 155. (2) 156. (4)

    SYLLOGISM 157. (2) 158. (2) 159. (1) 160. (4) 161. (3) 162. (4) 163. (4) 164. (5)

    SBI PO EXAMS

    165. (5) 166. (4) 167. (5) 168. (1)

    1. (2)

    2. (4)

    3. (1)

    4. (2)

    169. (5) 170. (3) 171. (2) 172. (2)

    5. (1)

    6. (2)

    7. (4)

    8. (4)

    173. (2) 174. (5) 175. (3) 176. (5)

    9. (5)

    10. (2)

    11. (5)

    12. (4)

    177. (4) 178. (4) 179. (4) 180. (2)

    13. (1)

    14. (3)

    15. (2)

    16. (5)

    181. (3) 182. (1) 183. (5) 184. (3)

    17. (2)

    18. (4)

    19. (4)

    20. (5)

    185. (5) 186. (4) 187. (1) 188. (1)

    21. (1)

    22. (4)

    23. (4)

    24. (1)

    189. (2) 190. (5) 191. (5) 192. (3)

    25. (4)

    26. (5)

    27. (4)

    28. (1)

    193. (4) 194. (2) 195. (2) 196. (3)

    29. (2)

    30. (3)

    31. (5)

    32. (5)

    197. (4) 198. (5) 199. (5) 200. (5)

    33. (1)

    34. (2)

    35. (5)

    201. (4) 202. (5) 203. (2) 204. (4)

    37. (3)

    38. (2)

    205. (5) 206. (3) 207. (1) 208. (3)

    41. (2)

    209. (3) 210. (4) 211. (5) 212. (2)

    45. (5)

    213. (3) 214. (1) 215. (4) 216. (5) 217. (3) 218. (2) 219. (4) 220. (3) 221. (3) 222. (3) 223. (2) 224. (1)

    49. (1) 53. (3)

    65. (5)

    66. (2)

    67. (3)

    68. (2)

    69. (1)

    70. (1)

    71. (4)

    72. (3)

    73. (2)

    74. (5)

    INSURANCE EXAMS 1. (4)

    2. (2)

    3. (5)

    4. (1)

    5. (4)

    6. (4)

    7. (2)

    8. (4)

    9. (1)

    10. (4)

    11. (4)

    12. (2)

    13. (2)

    14. (3)

    15. (4)

    16. (3)

    17. (4)

    18. (2)

    19. (1)

    20. (3)

    36. (4)

    21. (2)

    22. (1)

    23. (4)

    24. (2)

    39. (5)

    40. (1)

    25. (1)

    26. (3)

    27. (2)

    28. (1)

    42. (4)

    43. (5)

    44. (5)

    29. (2)

    30. (1)

    31. (1)

    32. (4)

    46. (3)

    47. (2)

    48. (5)

    33. (3)

    34. (2)

    35. (1)

    36. (2)

    52. (4)

    37. (2)

    38. (5)

    39. (2)

    40. (4)

    56. (1)

    41. (3)

    42. (2)

    43. (4)

    44. (2)

    45. (5)

    46. (2)

    47. (3)

    48. (1)

    49. (4)

    50. (3)

    51. (1)

    52. (5)

    53. (3)

    54. (1)

    55. (4)

    56. (2)

    57. (5)

    58. (4)

    59. (2)

    60. (2)

    61. (3)

    62. (4)

    63. (5)

    64. (1)

    65. (5)

    66. (3)

    67. (1)

    68. (5)

    50. (2) 54. (2)

    51. (5) 55. (5)

    57. (2)

    58. (4)

    59. (5)

    60. (5)

    225. (1) 226. (2) 227. (4) 228. (3)

    61. (5)

    62. (3)

    63. (2)

    64. (2)

    229. (4) 230. (5) 231. (1) 232. (4)

    65. (1)

    66. (2)

    67. (4)

    68. (5)

    69. (2)

    70. (5)

    71. (4)

    72. (5)

    237. (1) 238. (5) 239. (1) 240. (2)

    73. (4)

    74. (4)

    75. (3)

    76. (2)

    241. (1) 242. (4) 243. (5) 244. (1)

    77. (1)

    78. (1)

    79. (1)

    80. (3)

    69. (3)

    70. (1)

    71. (5)

    72. (4)

    81. (2)

    82. (5)

    83. (1)

    84. (4)

    73. (3)

    74. (2)

    75. (5)

    76. (3)

    85. (1)

    86. (5)

    87. (5)

    88. (2)

    77. (5)

    78. (4)

    79. (4)

    80. (4)

    89. (4)

    90. (3)

    91. (5)

    81. (5)

    82. (2)

    83. (3)

    84. (2)

    85. (4)

    86. (1)

    87. (4)

    88. (1)

    89. (2)

    90. (1)

    91. (5)

    92. (1)

    93. (3)

    94. (5)

    95. (5)

    96. (5)

    97. (1)

    98. (1)

    99. (4) 100. (1)

    233. (3) 234. (5) 235. (3) 236. (5)

    245. (3) 246. (2) 247. (5) 248. (2) 249. (3) 250. (2) 251. (2) 252. (1) 253. (1) 254. (2) 255. (1) 256. (4) 257. (5) 258. (3) 259. (1) 260. (5) 261. (3) 262. (5) 263. (3) 264. (5) 265. (5) 266. (1) 267. (1) 268. (5) 269. (2) 270. (4) 271. (4) 272. (2) 273. (1) 274. (2) 275. (1) 276. (4) 277. (4) 278. (3) 279. (3) 280. (5) 281. (4) 282. (4) 283. (4) 284. (2) 285. (3) 286. (1) 287. (2) 288. (4) 289. (2) 290. (3) 291. (5) 292. (2) 293. (2) 294. (5) 295. (4) 296. (5) 297. (1) 298. (5) 299. (5) 300. (2) 301. (1) 302. (5) 303. (5) 304. (5) 305. (2) 306. (4) 307. (3) 308. (4) 309. (2) 310. (1) 311. (1) 312. (2) 313. (3) 314. (5) 315. (1) 316. (2) 317. (2) 318. (3) 319. (1) 320. (5) 321. (2) 322. (1) 323. (3) 324. (2) 325. (3) 326. (2) 327. (5) 328. (2) 329. (1) 330. (5) 331. (4)

    RBI GRADE–B/ NABARD GRADE–A OFFICER EXAMS 1. (2) 5. (1) 9. (2) 13. (1) 17. (1) 21. (4) 25. (1) 29. (1) 33. (3) 37. (4) 41. (4) 45. (4) 49. (4) 53. (4) 57. (5) 61. (2)

    2. (5) 6. (5) 10. (5) 14. (5) 18. (5) 22. (5) 26. (2) 30. (2) 34. (5) 38. (4) 42. (5) 46. (1) 50. (4) 54. (1) 58. (5) 62. (2)

    3. (4) 7. (1) 11. (2) 15. (4) 19. (3) 23. (3) 27. (4) 31. (4) 35. (3) 39. (1) 43. (4) 47. (4) 51. (2) 55. (1) 59. (4) 63. (5)

    BPRE–543

    4. (1) 8. (4) 12. (5) 16. (1) 20. (3) 24. (5) 28. (3) 32. (5) 36. (4) 40. (4) 44. (2) 48. (3) 52. (4) 56. (5) 60. (5) 64. (5)

    101. (2)

    of Kiran Prakashan

    G et b ook s an d m aga zin es o f K ira n Prak ash an at y ou r d oorste p easily, lo g on ou r w e b site : www.kiranprakashan.com

    SYLLOGISM

    EXPLANATIONS

    NATIONALISED BANKS & IBPS PO/MT/SO EXAMS (1-7) : (i) All moons are planets → Universal Affirmative (A–type). (ii) Some stars are moons → Particular Affirmative (I-type). (iii) No planet is universe → Universal Negative (E-type). (iv) Some planets are not universe → Particular Negative (O–type). (1-2) : Some stars are moons. All moons are planets.

    I + A ⇒ I-type of Conclusion “Some stars are planets”. (A) All moons are planets.

    No planet is universe.

    A + E ⇒ E-type of Conclusion “No moon is universe”. (B) Some stars are planets

    No planet is universe.

    I + E ⇒ O-type of Conclusion “Some stars are not universe”. (C) 1. (4) Neither Conclusion I nor II follows. 2. (5) Conclusion I is Converse of Conclusion (A). Conclusion II is Conclusion (B). (3-4) : All sticks are plants.

    A + A ⇒ A-type of Conclusion “All sticks are amphibians”. (C) 3. (5) Conclusion I is Converse of Conclusion (B). Conclusion II is Conclusion (A) 4. (2)Conclusion II is Conclusion (B). (5-7) :

    No hut is a building.

    A + E ⇒ E-type of Conclusion “No apartment is a building”. (A)

    No river is a canal. A + E ⇒ E-type of Conclusion “No ocean is a canal.” (B)

    No hut is a building.

    Some seas are rivers.

    All buildings are cottages.

    E + A ⇒ O1-type of Conclusion “Some cottages are not huts.” (B) 5. (4) Neither Conclusion I nor II follows. 6. (4) Neither Conclusion I nor II follows. 7. (4) Neither Conclusion I nor II follows. (8-15) : (i) All building are houses → Universal Affirmative (A-type). (ii) Some oceans are seas → Particular Affirmative (I-type). (iii) No house is an apartment → Universal Negative (E-type). (iv) Some houses are not apartments → Particular Negative (O-type). (8–9) : All buildings are houses. No house is an apartment.

    A + E ⇒ E-type of Conclusion “No building is an apartment.” (A)

    All insects are amphibians.

    No day is night. All nights are noon.

    E + A ⇒ O1-type of Conclusion “Some noon are not days.” (A) All nights are noon. No noon is an evening.

    A + E ⇒ E–type of Conclusion “No night is an evering.” (B) 12. (4) None follows. 13. (1) Conclusion I is Converse of Conclusion B. (14-15) : Some papers are boards.

    E + A ⇒ O1-type of Conclusion “Some flats are not houses.” (B)

    All insects are amphibians.

    All sticks are insects.

    No river is a canal. I + E ⇒ O1 -type of Conclusion “Some canals are not seas.” (C) 10. (1) All oceans are rivers. Its converse “ Some rivers are oceans”, is true. Thus, Conclusion I is true. 11. (5) Conclusion B is Conclusion I. Conclusion A is Conclusion II. (12-13) :

    All apartment are flats.

    No building is an apartment.

    A + A ⇒ A-type of Conclusion “All plants are amphibians.”. (B)

    All oceans are rivers. I + A ⇒ I-type of Conclusion “Some seas are rivers.” (A) All oceans are rivers.

    All plants are insects.

    All plants are insects.

    Some seas are oceans.

    All apartments are huts.

    No house is an apartment.

    A + A ⇒ A-type of Conclusion “All sticks are insects”. (A)

    (10 – 11) :

    All apartments are flats.

    E + A ⇒ O1-type of Conclusion “Some flats are not buildings.” (C) 8. (2) Conclusion A is Conclusion II. 9. (4) None follows.

    BPRE–544

    No board is a card. I + E ⇒ O-type of Conclusion “Some papers are not cards.” (A) 14. (3) Conclusions I and II form Complementary Pair. Therefore, either I or II follows. 15. (2) Sometimes, Particular Affirmative Premise is considered same as the Universal Affirmative. Therefore, Conclusion II may be true.

    SYLLOGISM (16–20) : (i) All gliders are parachutes → Universal Affirmative (A-type). (ii) Some mails are chats → Particular Affirmative (I-type). (iii) No stone is metal → Universal Negative (E-type). (iv) Some stones are not metals → Particular Negative (O-type). (16–17) :

    Now, Combine figures (2) and (3). Parachutes Airplane

    Thus, all mails being updates is a possibility. (19–20) :

    No stone is a metal.

    Gliders

    Combine figures (3) and (5)

    Some metals are papers. E + I ⇒ O1 -type of Conclusion “Some papers are not stones.”(A)

    Helicopters

    All gliders are parachutes

    Parachute

    Some metals are papers.

    Airplanes

    Some chats are mails. A + I ⇒ No Conclusion Some mails are chats.

    r Pa (1)

    h ac

    s at

    (2)

    Mails

    es

    Ch

    ut

    Ch

    (1)

    Parachutes Gliders

    (2)

    All updates are chats.

    d Up

    a ts

    Chats

    s a te

    Ch

    Airplane

    Up dates

    (4)

    (3) All airplanes are helicopters.

    H

    op ic el

    (4)

    t

    Airplanes (5)

    “Some glasses are not stones”. (21–25) : (i) All stones are pebbles → Universal Affirmative (A-type). (ii) Some rulers are queens → Particular Affirmative (I-type) (iii) No letter is a fax → Universal Negative (E-type) (iv) Some letters are not faxes → Particular Negative (O-type)

    M

    a il

    All pebbles are rocks. A + A ⇒ A-type of Conclusion “All stones are rocks.” This is Conclusion I. 22. (2) All kings are rulers.

    (5)

    Now combine the figures (3) and (5)

    Helicopters s er

    E + I ⇒ O1 -type of Conclusion

    21. (1) All stones are pebbles.

    No parachute is an airplane.

    Parachute

    Some metals are glasses.

    (3)

    Gliders

    Airplanes

    Ma ils

    s

    All airplanes are helicopters. E + A ⇒ O1-type of Conclusion Some helicopters are not gliders.” (C) 16. (4) Neither Conclusion I nor Conclusion II follows. All gliders are parachutes.

    20. (4) No stone is a metal.

    at

    No glider is an airplane.

    18. (1) All updates are chats.

    at s

    All airplanes are helicopters. E + A ⇒ O1-type of Conclusion “Some helicopters are not parachutes.”(B)

    I + A ⇒ I-type of Conclusion “Some metals are glasses.”(B) 19. (2) Conclusion II is Converse of Conclusion (B).

    Ch

    No parachute is an airplane.

    All papers are glass.

    No parachute can be a helicopter. 17. (1) Conclusion I is Conclusion (A). No glider is an airplane and all airplanes are some helicopters. Therefore, no glider can be a helicopter.

    M ai ls

    No parachutes is an airplane. A + E ⇒ E-type of Conclusion “No glider is an airplane.”(A)

    s

    C hat

    d Up

    at

    s

    es

    BPRE–545

    Some rulers are queens. A + I ⇒ No Conclusion Conclusion II is Converse of the second Premise. 23. (1) Both the Premises are Particular Affirmative (I-type). No Conclusion follows from the two Particular Premises. Conclusion I is Converse of the first Premise.

    SYLLOGISM 24. (5) All messages are faxes.

    No fax is a letter. A + E ⇒ E-type of Conclusion “No message is a letter.” This is Conclusion I. Conclusion II is Converse of the second Premise.

    All the ‘colleges’ are outside the limit of ‘Banks’. Therefore, “Some colleges are definitely not schools.” Again, “All banks being colleges is a possibility”. Thus, both the Conclusions follow. 27. (4) All baskets are trolleys.

    25. (4) Some boards are plains.

    No plain is a square. I + E ⇒ O-type of Conclusion “Some boards are not squares”. (26–30) : (i) All baskets are trolleys → Universal Affirmative (A&type). (ii) Some schools are colleges → Particular Affirmative (I&type). (iii) No bank is a school → Universal Negative (E–type). (iv) Some banks are not schools → Particular Negative (O–type).

    Some trolleys are carts. A + I ⇒ No Conclusion (28–29) :

    All fruits are vegetables.

    All vegetables are plants. A + A ⇒ A-type of Conclusion “All fruits are plants”(P)

    All vegetables are plants.

    26. (5) No bank is a school.

    No plant is a root. Some schools are colleges. E + I ⇒ O1-type of Conclusion “Some colleges are not banks.”

    Co

    ge l le

    s

    A + E ⇒ E-type of Conclusion “No vegetable is root.”(Q)

    Banks

    A + E ⇒ E-type of Conclusion “No fruit is a root.”(R) 28. (5) Conclusion I is the Conclusion P. Conclusion II is the Converse of Conclusion Q. 29. (1) Conclusion I is the Conclusion R. No root is a vegetable. Therefore, Conclusion II does not follow. 30. (4)

    Some calculators are phones. It implies that all the ‘banks’ are some ‘colleges’. The derived Conclusion may also be represented as :

    Colleges

    Banks

    or

    P

    P or

    C

    Therefore, Conclusion II does not follow. (31-35) : (i) All beans are pulses ⇒ Univer sal Affirmative (A-type). (ii) Some fruits are apples ⇒ Particular Affirmative (I-type). (iii) No crop is seed ⇒ Universal Negative (E-type). (iv) Some crops are not seeds ⇒ Particular Negative (O-type). 31. (1) All beans are pulses.

    All pulses are crops. A + A ⇒ A–type of Conclusion “All beans are crops.” This is Conclusion II

    All beans are crops.

    No crop is a seed. No plant is a root.

    Colleges

    C

    P

    All fruits are plants.

    Banks

    It implies that some of the ‘colleges’ are outside the limit of ‘banks’. It may also be represented as :

    C

    No phone is an eraser. I + E ⇒ O-type of Conclusion “Some calculators are not erasers”. The first Premise can be illustrated with the help of venn-diagrams :

    BPRE–546

    A + E ⇒ E–type of Conclusion “No bean is a seed.”

    All pulses are crops

    No crop is a seed. A + E ⇒ E–type of Conclusion “No pulse is a seed.” 32. (5) Some apples are fruits.

    No fruit is vegetable. I + E ⇒ O–type of Conclusion “Some apples are not vegetables.”

    All potatoes are vegetables.

    No vegetable is fruit. A + E ⇒ E–type of Conclusion “No potato is fruit”. Neither Conclusion I nor II follows.

    SYLLOGISM 33. (5) All diaries are journals.

    A + E ⇒ E-type of Conclusion “No sharpener is a pen”.(Q) (39–40) :

    (36–37) :

    No cow is a bull.

    Some journals are books.

    Jo u Books

    rn a

    Some planets are moons. All bulls are animals. E + A ⇒ O1-type of Conclusion “Some animals are not cows”.(P)

    A + I ⇒ No Conclusion

    ls

    Diaries

    From the above diagram it is clear that “Some diaries being books is a possibility”. 34. (1)

    All potatoes are vegetables.

    All bulls are animals.

    Some animals are mammals. A + I ⇒ No Conclusion 36. (5) Conclusion I is the Converse of the third Premise. “All bulls are animals”. – its venn diagrams. Animals

    Animals, and

    Bulls

    Bulls

    No vegetable is fruit.

    II

    Venn diagrams of the third Premise.

    als im An

    All pulses are crops.

    No crop is a seed. A + E ⇒ E-type of Conclusion “No bean is a seed.” Conclusion I is Converse of it.

    No seed is a bean.

    All beans are pulses. E + A ⇒ O1-type of Conclusion “Some pulses are not seeds.” (36–40) : (i) All bulls are animals → Univer sal Affirmative (A-type). (ii) Some pencils are pens → Particular Affirmative (I-type). (iii) No cow is a bull → Universal Negative (E-type). (iv) Some cows are not bulls → Particular Negative (O type).

    ;

    All digits are alphabets.

    No alphabet is a vowel.

    III

    A + A ⇒ A-type of Conclusion “All beans are crops.”

    All beans are crops.

    All digits are alphabets. I + A ⇒ I–type of Conclusion “Some numbers are alphabets.”

    m als

    35. (3) All beans are pulses.

    41. (3) Some numbers are digits. I

    Ma m

    A + E ⇒ E-type of Conclusion “No potato is fruit.” Conclusion I is Converse of it. Conclusion II is Converse of the third Premise.

    No moon is a sun. I + E ⇒ O-type of Conclusion “Some planets are not sun”. 39. (4) None follows 40. (2) Conclusion II is the Converse of the first Premise. (41-45) : (i) All digits are alphabets → Universal Affirmative (A–type) (ii) Some numbers are digits → Particular Affirmative (I–type). (iii) No alphabet is a vowel → Universal Negative (E–type). (iv) Some alphabets are not vowels → Particular Negative (O–type).

    Animals

    Mammals

    Mammals

    Animals

    A + E ⇒ E-type of Conclusion “No digit is a vowel”. This is Conclusion I. All consonants are vowels.

    _

    IV

    V

    From the diagrams II and IV, it is clear that “Some mammals being bulls is a possibility”. 37. (1) Conclusion I is the Converse of the second Premise. 38. (4) Some pencils are pens.

    No pen is eraser. I + E ⇒ O-type of Conclusion “Some pencils are not erasers”.(P)

    All sharpeners are erasers.

    No eraser is pen.

    BPRE–547

    No vowel is an alphabet. A + E ⇒ E-type of Conclusion “No consonant is an alphabet”. Conclusion II is Converse of it. Some numbers are alphabets. No alphabet is a vowel. I + E ⇒ O1-type of Conclusion “Some vowels are not numbers.”

    42. (4) Some files are papers.

    All papers are certificates. I + A ⇒ I- type of Conclusion “Some files are certificates. Conclusion III is Converse of it.

    SYLLOGISM All papers are certificates.

    No certificate is a manuscript.

    A + E ⇒ E–type of Conclusion “No paper is a manuscript.” 43. (1) No group is people.

    All people are animals. E + A ⇒ O1–type of Conclusion “Some animals are not groups.”

    All people are animals. All animals are plants. A + A ⇒ A– type of Conlcusion “All people are plants.” This is Conclusion II.

    All people are plants.

    45. (2) All marks are grades.

    No grade is a score.

    49. (4) All jackets are trousers.

    A + E ⇒ E-type of Conclusion “No mark is a score”.

    No trouser is shirt. A + E ⇒ E–type of Conclusion “No jacket is shirt.”

    All letters are scores.

    No trouser is shirt.

    All scores are characters.

    Some shirts are caps.

    A + A ⇒ A–type of Conclusion Conclusion IV is Converse of it. (46-49) : (i) All diamonds are stones → Universal Affirmative (A-type). (ii) Some shirts are caps → Particular Affirmative (I-type). (iii) No gem is diamond → Universal Negative (E-type). (iv) Some gems are not diamonds → Particular Negative (O-type). (46-47) :

    All diamonds are stones.

    All plants are roots.

    All stones are gems.

    A + A ⇒ A-type of Conclusion “All people are roots.”

    A + A ⇒ A–type of Conclusion “All diamonds are gems”.(P)

    All animals are plants.

    All stones are gems.

    All plants are roots.

    No gem is diamond.

    E + I ⇒ O1–type of Conclusions “Some caps are not trousers.”.

    No jacket is shirt.

    Some shirts are caps. E + I ⇒ O1–type of Conclusion “Some caps are not jackets.” (50–231) : (i) All cans are jars → Universal Affirmative (A–type). (ii) Some prints are designs → Particular Affirmative (I–type). (iii) No bottle is jar → Universal Negative (E–type). (iv) Some bottles are not jars → Particular Negative (O–type). (50–51) :

    All cans are jars. A + A ⇒ A-type of Conclusion “All animals are roots.” 44. (5) Some vacancies are jobs.

    All jobs are works. I + A ⇒ I–type of Conclusion “Some vacancies are works.”

    All jobs are works. No work is a trade. A + E ⇒ E–type of Conclusion “No job is trade”.

    All professions are trades.

    A + E ⇒ E – type of Conclusion “No stone is diamond.”(Q)

    No jar is bottle.

    No gem is diamond.

    A + E ⇒ E – type of Conclusion “No can is bottle.” (P)

    All diamonds are stones. E + A ⇒ O1– type of Conclusion “Some stones are not gems.”(R) 46. (2) Conclusion II is Conclusion (P). 47. (1) Conclusion I is the third Premise. 48. (2) No day is night.

    Some tumblers are cans.

    All cans are jars. I + A ⇒ I – type of Conclusion “Some tumblers are jars.” (Q)

    Some jars cans.

    E + A ⇒ O1–type of Conclusion “Some noons are not days.”

    All cans are tumblers. I + A ⇒ I – type of Conclusion “Some jars are tumblers.” (R)

    All nights are nooons.

    No bottle is jar.

    All nights are noons.

    No trade is a work. A + E ⇒ E–type of Conclusion “No profession is a work”. This is Conclusion III. Conclusion I is Converse of the second Premise.

    No noon is evening.

    Some jars are tumblers.

    A + E ⇒ E–type of Conclusion “No night is evening.” This is Conclusion II.

    E + I ⇒ O1 – type of Conclusion “Some tumblers are not bottles.” (S)

    BPRE–548

    SYLLOGISM (54–55) :

    Its venn diagrams

    (58-59) :

    All clouds are vapours. T um

    b ler

    All doubts are answers.

    Tumblers

    s

    Bottles

    ;

    Bottles

    No vapour is gas. A + E ⇒ E – type of Cunclusion “No cloud is gas”.(P)

    II

    I

    50. (2) Venn diagram II supports Conclusion II. 51. (4) Conclusion (P) ⇒ No can is bottle. Therefore, none follows. (52–53) :

    Some prints are designs.

    No vapour is gas.

    All answers are questions. A + A ⇒ A - type of Conclusion “All doubts are questions”. (P)

    Some reasons are doubts. All doubts are answers.

    All gases are rains. E + A ⇒ O1 – type of Cunclusion “Some rains are not vapours.” (Q)

    I + A ⇒ I - type of Conclusion “Some reasons are answers”. (Q)

    Some reasons are doubts.

    No cloud is gas.

    All doubts are questions.

    I + A ⇒ I – type of Conclusion “Some prints are copies.” (P)

    All designs are copies.

    All copies are motifs. A + A ⇒ A – type of Conclusion “All designs are motifs.” (Q)

    Some prints are copies.

    All copies are motifs. I + A ⇒ I – type of Conclusion “Some prints are motifs.” (R) 52. (5) Conclusion (R) is Conclusion I. Conclusion (Q) is conclusion II. 53. (5) The Converse of Conclusion (P) is Conclusion I. Venn diagrams of Conclusion (R)

    Motifs ; I Motifs Motifs

    All gases are rains. E + A ⇒ O1 – type of Conclusion “Some rains are not clouds.” (R) 54. (2) Conclusion (P) is Conclusion II. 55. (1) Conclusion (R) is Conclusion I. (56-61) : (i) All books are words → Universal Affirmative (A-type). (ii) Some doubts are reasons → Particular Affirmative (I-type). (iii) No paper is book → Universal Negative (E-type). (iv) Some papers are not books → Particular Negative (Otype). (56-57) :

    Questions

    Reasons or

    Questions

    All books are words. E + A ⇒ O1 – type of Conclusion “Some words are not papers”. (P)

    All books are words. No word is letter. A + E ⇒ E – type of Conclusion “No book is letter”. (Q) 56. (1) Conclusion I is Converse of Conclusion (Q). 57. (2) Venn diagrams of the Premises

    ; Book Words

    III

    Venn diagram II supports Conclusion II.

    Reasons

    No paper is book.

    Paper II

    I + A ⇒ I- type of Conclusion “Some reasons are questions”. (R) 58. (1) Conclusion I is same as that of Conclusion (P). 59. (5) Some reasons are questions — Conclusion (R) Its Venn diagrams

    Reasons or

    Questions

    All designs are copies.

    Therefore, Conclusion I is true. Similarly, on the basis of Conclusion (Q), we can say that Conclusion II also follows. (60-61) :

    Some traps are plans. All plans are ideas.

    It is clear that Conclusion II is true.

    BPRE–549

    I + A ⇒ I - type of Conclusion “Some traps are ideas”. (P)

    SYLLOGISM E + I ⇒ O1-type of Conclusion “Some numbers are not figures.” (Q)

    All plans are ideas. No idea is design. A + E ⇒ E - type of Conclusion “No plan is design”. (Q)

    Some traps are plans. No plan is design. I + E ⇒ O - type of Conclusion “Some traps are not designs.” (R) 60. (2) Conclusion II is Conclusion (P). 61. (5) Conclusion II is Converse of Conclusion (Q). Venn diagrams of Conclusion (R)

    Traps

    Design

    or

    No shape is digit. Some digits are numbers. E + I ⇒ O1-type of Coclusion “Some numbers are not shapes.” (R) 62. (1) Conclusion (Q) : “Some numbers are not figures.” It implies that some numbers are figures. So, Conclusion I follows. Conclusion (R) : “Some numbers are not shapes.” So, Conclusion II does not follow. 63. (2) Conclusion II is the Conclusion (P). (64-65) :

    All conductors are insulators.

    No insulator is magnet.

    66. (2) Conclusion II is the Converse of the Conclusion (P). 67. (4) Neither Conclusion I nor Conclusion II follows. 68. (3) Both the Premises are Particular Affirmative (I-type). No Conclusion follows from the two Particular Premises. Conclusions I and II form Complementary Pair. Therefore, either Conclusion I or Conclusion II follows. (69-73) : (i) All erasers are sharpeners → Universal Affirmative (A-type). (ii) Some pencils are pens → Particular Affirmative (I-type). (iii) No eraser is a pen → Universal Negative (E-type). (iv) Some erasers are not pens → Particular Negative (O-type). (69-70) :

    All erasers are sharpeners.

    All sharpeners are pencils.

    Traps

    A + E ⇒ E-type of Conclusion “No conductor is magnet”. (P)

    A + A ⇒ A - type of Conclusion “All erasers are pencils.” (P)

    Design

    All resistors are conductors.

    All sharpeners are pencils.

    Therefore, Conclusion I also follows. (62–68) : (i) All shapes are figures → Universal Affirmative (A–type). (ii) Some digits are numbersa → Particular Affirmative (I–type). (iii) No figure is digit → Universal Negative (E-type). (iv) Some figures are not digits → Particular Negative (O–type). (62-63) :

    All shapes are figures. No figure is digit. A + E ⇒ E-type of Conclusion “No shape is digit”. (P)

    No figure is digit. Some digits are numbers.

    No conductor is magnet.

    Some pencils are pens.

    A + E ⇒ E-type of Conclusion “No resistors are magnet.” (Q)

    A + I ⇒ No Conclusion

    All resistors are conductors. All conductors are insulators. A + A ⇒ A-type of Conclusion “All resistors is insulators.” (R) 64. (2) Conclusion (Q) : “No resistor is magnet.” So, Conclusion I does not follow. Conclusion II is Conclusion (R). 65. (1) Conclusion I is the Converse of Conclusion (P). (66-68) :

    Some reagents are chemicals. All chemicals are elements. I + A ⇒ I-type of Conclusion “Some reagents are elements”. (P)

    All erasers are pencils.

    Some pencils are pens. A + I ⇒ No Conclusion 69. (3) Conclusions I and II form Complementory Pair. Therefore, either Conclusion I or Conclusion II follows. 70. (4) Neither Conclusion I nor Conclusion II follows. 71. (2) All railways are trains.

    No train is station. A + E ⇒ E-type of Conclusion “No railway is station.” This is Conclusion II.

    No train is station.

    All chemicals are elements.

    Some stations are platforms. Some elements are substances. A + I ⇒ No Conclusion

    BPRE–550

    E + I ⇒ O1-type of Conclusion “Some platforms are not trains.”

    SYLLOGISM (72-73) :

    All winters are summers. Some summers are springs. A + I ⇒ No Conclusion

    Some summers are springs. No spring is an autumn. I + E ⇒ O-type of Conclusion “Some summers are not autumuns” (P). 72. (5) Conclusion I is the Implication of the first Premise. Conclusion (P) states that “Some summers are not autumns.” It may mean that some autumns are summers. 73. (5) Conclusion (P) states that some summers are not autumns. Therefore, all summers can never be autumn. Conclusion II is the Converse of the first Premise. (74–79) : (i) All rivers are oceans → Universal Affirmative (A-type). (ii) Some ends are terminals → Particular Affirmative (I-type). (iii) No pond is stream → Universal Negative (E-type). (iv) Some ponds are not streams → Particular Negative (O-type).

    A + E ⇒ E – type of Conclusion “No river is stream.” (S) If no river is stream, some rivers would also be not streams. Thus, Option (1) is valid.

    No coean is stream. All streams are canals. E + A ⇒ O1 – type of Conclusion “Some canals are not oceans.” (T)

    No river is stream. All streams are canals. E + A ⇒ O1 – type of Conclusion “Some canals are not rivers.” (U) This is Option (2). 75. (3) All colours are inks.

    All inks are dyes. A + A ⇒ A – type of Conclusion “All colours are dyes.” (P) This is Option (2)

    Some inks are colours. All colours are brushes. I + A ⇒ I – type of Conclusion “Some inks are brushes.” (Q) This is Option (4).

    Some brushes are colours.

    74. (5) All rivers are oceans

    All colours are inks. All oceans are ponds. A + A ⇒ A – type of Conclusion “All rivers are ponds.” (P) This is Option (3).

    All oceans are ponds. No pond is stream. A + E ⇒ E – type of Conclusion “No ocean is stream.” (Q) This is Option (4).

    I + A ⇒ I – type of Conclusion “Some brushes are inks.” (R) Option (1) is Converse of the second Premise. Option (5) is Converse of the fourth Premise. 76. (1) All lectures are talks.

    No talk is speech.

    All addresses are speeches. No speech is talk. A + E ⇒ E – type of Conclusion “No address is talk.” (R) If supports Option (5).`

    All addresses are speeches. No speech is lecture. A + E ⇒ E – type of Conclusion “No address is lecture.” (S) This is Conclusion (2). 77. (4) Some dens are houses.

    All houses are buildings. I + A ⇒ I – type of Conclusion “Some dens are buildings”. (P) This is Conclusion (3).

    All houses are bulidings. No building is nest. A + E ⇒ E – type of Conclusion “No house is nest.” (Q) It supports Option (5).

    No nest is house. All houses are buildings. E + A ⇒ O1 – type of Conclusion “Some buildings are not nests.” (R) This is Option (1). Option (2) is Converse of the third Premise. 78. (2) Some ends are terminals.

    All terminals are stops. I + A ⇒ I – type of Conclusion “Some ends are stops.” (P) This is Option (3).

    All terminals are stops.

    No pond is stream.

    A + E ⇒ E – type of Conclusion “No lecture is speech.” (P) This is Option (4)

    All streams are canals.

    All classes are addresses.

    A + A ⇒ A – type of Conclusion “All terminals are posts.”(Q) Option (5) is Converse of it.

    E + A ⇒ O1 – type of Conclusion “Some canals are not ponds.” (R)

    All rivers are ponds. No pond is stream.

    All addresses are speeches. A + A ⇒ A – type of Conclusion “All classes are speeches.” (Q) Option (3) is Converse of it.

    BPRE–551

    All stops are posts.

    All terminals are posts. All posts are locations.

    SYLLOGISM A + A ⇒ A – type of Conclusion “All terminals are locations.” (R) This is Option (1).

    (81–82) :

    83. (3) Some dukes are kings.

    Some amphibians are turtles.

    Some ends are stops.

    All kings are warriors. All turtles are reptiles.

    All stops are posts. I + A ⇒ I – type of Conclusion “Some ends are posts.” (S) Some ends are terminals. All terminals are locations. I + A ⇒ I – type of Conclusion “Some ends are locations.” (T) This is Option (4). 79. (4) All cities are towns.

    I + E ⇒ I – type of Conclusion “Some amphibians are reptiles.” (P) 81. (5) Conclusion (P) is Conclusion I. 82. (5) Venn diagrams of “All frogs are amphibians.”

    I + A ⇒ I – type of Conclusion “Some dukes are warriors.” This is Conclusion II. (84–85) :

    Some plants are trees.

    All trees are weeds. Amphibians

    Amphibians

    I + A ⇒ I – type of Conclusion “Some plants are weeds.” (P)

    All trees are weeds.

    Frogs

    All towns are villages. A + A ⇒ A – type of Conclusion “All cities are villages.” (P) This is Option (3).

    Frogs I

    Venn diagrams of “Some turtles are amphibians” :

    ph ib ian

    p Am Turtles

    Am

    Turtles

    All cities are towns. II

    All towns are districts. A + A ⇒ A – type of Conclusion “All cities are districts.” (R) This is Option (5). All lanes are cities.

    All weeds are shrubs. A + A ⇒ A – type of Conclusion “All trees are shrubs.” (Q) Some plants are weeds.

    s an bi i h

    s

    All towns are villages. All villages are districts. A + A ⇒ A – type of Conclusion “All towns are districts.” (Q) This is Option (1).

    II

    III Turtles

    All weeds are shrubs. I + A ⇒ I – type of Conclusion “Some plants are shrubs.” (R) 84. (3) Conclusion (P) is Conclusion II. 85. (2) Conclusion (Q) is Conclusion I. Venn diagrams of Conclusion (R)

    Amphibians

    Plants

    All cities are towns. A + A ⇒ A – type of Conclusion “All lanes are towns.” (S) This is Option (2). (80–85) : (i) All drivers are swimmers → Universal Affirmative (A–type). (ii) Some swimmers are athletes → Particular Affirmative (I–type). (iii) No athlete is a banker → Universsal Negative (E–type). (iv) Some athletes are not bankers → Particular Negative (O–type). 80. (4) Some swimmers are athletes.

    IV Combine diagrams II and V

    Turtles Amphibians

    Frogs

    Shrubs

    Shrubs

    Plants

    Plants

    Shrubs

    I II III From statement III it is clear that “shrubs being plants is a possibility.” (86–89) : (i) All drivers are swimmers → Universal Affirmative (A–type). (ii) Some swimmers are athletes → Particular Affirmative (I–type). (iii) No athlete is a banker → Universsal Negative (E–type). (iv) Some athletes are not bankers → Particular Negative (O–type). 86. (4)

    Some swimmers are athletes. No athlete is a banker. I + E ⇒ O – type of Conclusion “Some swimmers are not bankers.”

    Thus, All frogs beings turtles is a possibility.

    BPRE–552

    No athlete is a banker.

    SYLLOGISM I + E ⇒ O – type of Conclusion “Some swimmers are not bankers.” (87 – 88) :

    Some amphibians are turtles.

    All turtles are reptiles. I + E ⇒ I – type of Conclusion “Some amphibians are reptiles.” (P) 87. (5) Conclusion (P) is Conclusion I. 88. (5) Venn diagrams of “All frogs are amphibians.” Amphibians

    89. (3) Some dukes are kings.

    All amphibians are plants. All kings are warriors. I + A ⇒ I – type of Conclusion “Some dukes are warriors.” This is Conclusion II. (90 – 267) :

    Frogs I

    II

    All trees are weeds. I + A ⇒ I – type of Conclusion “Some plants are weeds.” (P) All trees are weeds.

    All weeds are shrubs. A + A ⇒ A – type of Conclusion “All trees are shrubs.” (Q) Some plants are weeds.

    s ph ib ian

    s

    p Am Turtles

    Am

    Turtles

    an

    II

    III

    Some ratios are percent. All percent are fractions. I + A ⇒ I-type of Conclusion “Some ratios are fractions.” (P)

    All percent are fractions. No fraction is a section. A + E ⇒ E-type of Conclusion “No percent is a section.” (Q)

    Some ratios are fractions.

    No fraction is a section.

    Venn diagrams of “Some turtles are amphibians” : bi hi

    A + A ⇒ A – type of Conclusion “All kites are plants.” This is option (4). (93-94) :

    Some plants are trees.

    Amphibians

    Frogs

    All kites are amphibians.

    All weeds are shrubs. I + A ⇒ I – type of Conclusion “Some plants are shrubs.” (R) 90. (3) Conclusion (P) is Conclusion II. 91. (2) Conclusion (Q) is Conclusion I. Venn diagrams of Conclusion (R)

    I + E ⇒ O-type of Conclusion “Some ratios are not sections.” (R) 93. (1) Conclusion I is the Converse of the Conclusion (Q). Venn diagrams of “Some ratios are fractions”: Fr ac

    Ratios Turtles

    Plants

    Shrubs

    Shrubs

    Plants

    Plants

    Shrubs

    I

    IV Combine diagrams II and V

    Turtles Amphibians

    Frogs

    II

    III

    All reptiles are birds.

    Thus, All frogs beings turtles is a possibility.

    No bird is a kite. A + E ⇒ E – type of Conclusion “No reptile is a kite”.

    BPRE–553

    t io

    ns

    Fractions

    or

    I

    Amphibians

    From statement III it is clear that “shrubs being plants is a possibility.” 92. (4) (i) All reptiles are birds → Universal Affirmative (A–type). (ii) Some reptiles are birds → Particular Affirmative (I–type). (iii) No kite is a bird → Universal Negative (E–type). (iv) Some kites are not birds → Particular Negative (O–type).

    Ratios

    II

    Fractions Ratios or

    III Diagram III supports the Conclusion II. 94. (1) Venn diagrams of “Some ratios are not sections” :

    t Ra

    Ratios

    ios ns or io ct e S I

    Sections

    II

    SYLLOGISM or

    Ratios

    Sections

    This diagram supports the Conclusion I. 96. (5) Venn diagrams of “All plastics are ores” :

    Ores

    Ores

    III or

    II

    Venn diagrams of “Some ores are wood” :

    All plastics are ores. Ores

    Ores

    Wood

    Wood

    No store is a panel. A + E ⇒ E – type of Conclusion “No bank is a panel”. Conclusion II is the Converse of it. (99–100) : Some hits are strikes.

    or

    III

    Metals

    No strike is a raid. I + E ⇒ O – type of Conclusion “Some hits are not raids”. (P) All attacks are raids.

    IV

    or

    Metals

    Wood

    Ores I

    es Or

    es o Wo

    d

    or

    Wood

    No raid is a strike. A + E ⇒ E – type of Conclusion “No attack is a strike”. (Q) No attack is a strike.

    Ores

    II

    Venn diagrams of “Some ores are wood” :

    Or

    All banks are stores. E + A ⇒ O1 – type of Conclusion “Some stores are not lockers”. All banks are stores.

    Plastics

    I

    All metals are plastics.

    Ores

    All races are sprints. I + A ⇒ I – type of Conclusion “Some contests are sprints”. This is Conclusion I. 98. (1) No locker is a bank.

    Plastics

    Diagram II supports the Conclusion I. Conclusion II is the Converse of the Conclusion (P). (95–96) :

    A + A ⇒ A-type of Conclusion “All metals are ores.” (P) 95. (4) Venn diagrams of “All metals are ores” :

    97. (2) Some contests are races.

    or

    V Combine diagrams II and V :

    Wood

    Some strikes are hits. E + I ⇒ O1 – type of Conclusion “Some hits are not attacks”. (R) 99. (3) Conclusion I is the Conclusion (P). Venn diagrams of “Some strikes are hits”:

    IV

    III

    Plastics Wood Ores Ores VI

    Metals; Ores Wood

    VI

    or

    I

    or

    V Combine diagrams II and IV :

    r St

    Strikes

    Hits

    es ik

    This diagram supports the Conclusion II. (97–277) : (i) All races are sprints → Universal Affirmative (A–type). (ii) Some races are contests → Particular Affirmative (I–type). (iii) No bank is a locker → Universal Negative (E–type). (iv) Some banks are not lockers → Particular Negative (O–type).

    BPRE–554

    Hits

    II

    or Hits

    Strikes

    III Diagram II supports the Conclusion II.

    SYLLOGISM

    ta ck

    s I

    or

    Trucks

    Mails

    VI

    Cars

    VII

    Vehicles

    After combining the venn diagrams IV and VI, we get :

    II

    or

    Posts

    V

    At

    Hits

    Attacks

    or

    Posts or

    After combining the venn diagrams II and V, we get :

    Posts

    Mails

    ai ls

    Hits

    Venn diagrams of “Some mails are posts” :

    M

    100. (3) Conclusion I is the Conclusion (Q). Venn diagrams of “Some hits are not attacks” :

    VII

    Mails, Calls

    Venn diagrams of “All trucks are sedans” :

    Posts

    Attacks

    Hits

    Sedans

    III

    Diagram II supports the Conclusion II. 101. (2)

    Some formulae are equations.

    Therefore, Conclusion I follows. 103. (3) Neither Conclusion I nor Conclusion II follows. 104. (1) Some cars are trucks.

    All equations are terms.

    All trucks are sedans. I + A ⇒ I – type of Conclusion

    I + A ⇒ I – type of Conclusion “Some formulae are terms”.

    “Some cars are sedans.” This is Conclusion II Venn Diagram Method :

    All equations are terms.

    Trucks

    VIII

    T

    Some formulae are terms.

    Sedans, Trucks Cars Vehicles

    ks

    Cars

    X

    All terms are symbols. I + A ⇒ I – type of Conclusion “Some formulae are symbols”. (102–103) :

    All calls are mails.

    Venn diagrams of “Some vehicles are cars” : s le ic h Ve

    Cars

    s cle hi Ve

    Cars or

    Some mails are posts. A + I ⇒ No Conclusion Venn Diagram Method :

    I

    Mails

    Mails Calls

    Posts or

    C

    ls al

    Vehicles Cars

    III Venn diagrams of “Some cars are trucks” :

    Mails

    ks or uc Tr

    Cars

    Calls III

    Cars

    Trucks

    or

    IV

    IV

    Cars

    or

    V

    BPRE–555

    Therefore, all vehicles being sedans is a possibility. Thus, Conclusion I also follows. 105. (2) Some bridges are roads.

    No road is underpass.

    II

    or

    Posts

    I II 102. (4) Venn diagrams of “All calls are mails” :

    Calls

    IX

    After combining the venn diagrams VII and IX, we get :

    Sedans c ru

    or Trucks

    All terms are symbols. A + A ⇒ A – type of Conclusion “All equations are symbols”. This is Conclusion I.

    Sedans

    Trucks

    VI

    I + E ⇒ O – type of Conclusion “Some bridges are not under passes”. Conclusion I and Conclusion II form Complementary Pair. Therefore, either Conclusion I or Conclusion II follows. 106. (5)

    No unit is a part. All parts are items.

    E + A ⇒ O1– type of Conclusion “Some items are not units.” Venn diagrams of “Some items are not units” :

    SYLLOGISM Items Items

    U

    ni

    I

    ts

    or

    After combining the Venn diagrams II and IV, we get

    Units

    Jobs

    Deans, Ministers

    Works

    II

    or

    After combining the Venn diagrams II and IV, we get :

    Heads

    Hurdles

    Units

    Items III

    Venn diagrams I and II support the Conclusion II, but Venn diagram III contradicts it. However, ‘at least’ is mentioned in the Conclusion II, so there is possibility that the Conclusion is Valid. (107–111) : (i) All ministers are deans → Universal Affirmative (A–type). (ii) Some jobs are works → Particular Affirmative (I–type). (iii) No solution is a trick → Universal Negative (E–type). (iv) Some solutions are not tricks → Particular Negative (O–type). (107–108) :

    Some tasks are hurdles.

    VI Venn diagram VI supports the Conclusion I. 108. (3) Conclusion I is the Converse of the Conclusion (P). 109. (4) All rules are tricks.

    No trick is a solution. A + E ⇒ E–type of Conclusion “No rule is a solution.” This is the Conclusion I.

    Some problems are solutions. No solution is a trick. I + E ⇒ O–type of Conclusion “Some problems are not tricks.” This is the Conclusion II. 110. (5) All ministers are deans.

    All hurdles are jobs.

    Some deans are heads.

    I + A ⇒ I–type of Conclusion “Some tasks are jobs.” (P) 107. (3) Venn diagrams of “All hur dles are jobs” :

    A + I ⇒ No Conclusion Venn diagrams of “All ministers are deans” : Deans

    Jobs

    Jobs

    Hurdles

    Ministers

    or

    Ministers

    or Deans

    Hurdles

    I

    II

    I

    II

    Venn diagrams of “Some jobs are works” :

    Venn diagrams of “Some deans are heads” : Deans

    Jobs Works or

    Jobs

    III

    or

    Works Jobs

    V

    Works

    IV

    Heads or

    Deans

    III

    or

    Heads

    IV

    VI Venn diagram VI supports the Conclusion II. 111. (2) Some rows are queues.

    No queue is a line. I + E ⇒ O–type of Conclusion “Some rows are not lines.” Conclusions I and II form Complementary Pair. Therefore, either Conclusion I or Conclusion II follows. (112-114) : (i) All photos are images → Universal Affirmative (A-type). (ii) Some slides are photos → Particular Affirmative (I-type). (iii) No space is a gap → Universal Negative (E-type). (iv) Some spaces are not gaps → Particular Negative (O-type). 112. (5) Some slides are photos.

    All photos are images. I + A ⇒ I–type of Conclusion “Some slides are images”. Conclusion I is Converse of it.

    All photos are images. All images are creations. A + A ⇒ A-type of Conclusion “All photos are creations”. This is Conclusion II. 113. (2) All fissures are gaps.

    Heads

    No gap is a crack. Deans

    V

    BPRE–556

    A + E ⇒ E-type of Conclusion “No fissure is a crack”. This is Conclusion II.

    SYLLOGISM or

    All fissures are gaps.

    Deltas

    Thoughts

    No gap is a space.

    Ideas

    A + E ⇒ E-type of Conclusion “No fissure is a space”. 114. (3) No loss is a profit.

    Some profits are gains. E + I ⇒ O1 – type of Conclusion “Some gains are not losses”. Conclusions I and II form Complementary Pair. Therefore, either Conclusion I or Conclusion II follows. (115-116) : All points are views.

    Basins

    III Diagram II supports Conclusion II.

    VI After combining the Venn dia-

    (117–118) : (i) All soils are basins → Universal Affirmative (A–type). (ii) Some basins are deltas → Particular Affirmative (I–type). (iii) No ground is a soil → Universal Negative (E–type). (iv) Some grounds are not soils → Particular Negative (O–type). (117–118) :

    grams II and VI, we get :

    Deltas Basins Grounds

    No ground is a soil.

    No view is an idea. A + E ⇒ E – type of Conclusion “No point is an idea”. (P) No view is an idea.

    VII All soils are basins. E + A ⇒ O1 – type of Conclusion

    Basins

    ns

    Basins or

    ts gh ou Th

    Points

    or

    Basins I

    I

    II

    are deltas”:

    or II

    Venn diagrams of “Some basins are deltas”.

    or

    ts gh ou

    II

    IV

    Delta Basins

    Deltas

    De

    Th

    Ba

    lta

    s

    si n s

    Basins

    Thoughts

    Delta

    or

    III

    V

    IV I

    a elt or

    III

    III Diagram I supports Conclusion I. 116. (2) Conclusion (P) contradicts Conclusion I. Venn diagrams of “Some ideas are thoughts” : Ideas

    D

    Grounds

    Thoughts

    or

    Basins

    Ba

    Basins

    Ideas

    Venn diagrams of “Some basins

    Points

    or Points

    II

    si ns

    I

    or

    Grounds

    Gr Thoughts

    Soils

    Soils

    ou

    nd s

    Ba si

    Some ideas are thoughts E + I ⇒ O1 – type of Conclusion “Some thoughts are not views”. (Q) E + I ⇒ O1–type of Conclusion. ‘‘Some thoughts are not points’’. (R) 115. (1) Venn diagrams of “Some thoughts are not points”.

    “Some basins are not grounds”. 117. (5) Venn diagrams of “Some basins are not grounds” :

    Venn diagram VII supports the Conclusion II. 118. (4) Venn diagrams of “All soils are basins” :

    V or

    BPRE–557

    After combining the Venn diagrams II and V, we get :

    SYLLOGISM

    Deltas

    Winters

    Soils

    Seasons

    Seasons

    122. (1) Conclusion I is the Conclusion (P). Venn diagrams of “All digits are numbers” :

    Winters

    Numbers

    or

    I

    Basins

    II

    Venn diagrams of “Some winters

    Digits

    are autumns” :

    VI

    119. (5) All policies are decisions.

    W in te r

    Au tu m ns

    s

    Winters

    Venn diagram VI supports the Conclusion I.

    or

    Autumns

    III

    IV

    or

    No decision is a verdict. A + E ⇒ E–type of Conclusion “No policy is a verdict.”

    Winters

    it. Premises :

    II Venn diagrams of “All symbols are numbers” :

    V

    After combining the Venn dia-

    Numbers

    grams II and V, we get :

    Decisions Policies

    Digits, Numbers

    Autumns

    Conclusion II is the Converse of Venn diagrams of all the three

    I or

    Verdict

    Symbols

    Result Autumns Seasons

    120. (2)

    Some machines are calculators. No calculator is a phone. I + E ⇒ O–type of Conclusion “Some machines are not phones.” Conclusions I and II form Complementary Pair. Therefore, either Conclusion I or Conclusion II follows. 121. (1) Some winters are autumns.

    All autumns are falls.

    I + A ⇒ I – type of Conclusion

    Winters

    III or

    VI

    Venn diagram VI supports the Conclusion II. (122-127) : (i) All digits are numbers → Universal Affirmative (A-type). (ii) Some letters are digits → Particular Affirmative (I-type). (iii) No point is a spot → Universal Negative (E-type). (iv) Some points are not spots → Particular Negative (O-type). (122-123) :

    Symbols, Numbers

    IV After combining the Venn diagrams II and III, we get : Digits, Numbers Symbols

    Some letters are digits.

    “Some winters are falls.” Conclusion I is the Converse of it. Venn diagrams of “All seasons are winters” :

    All digits are numbers. I + A ⇒ I-type of Conclusion “Some letters are numbers”. (P)

    BPRE–558

    V Venn Diagram V supports the Conclusion II.

    SYLLOGISM 123. (5) Venn diagrams of “Some letters are digits”:

    Marks

    A + E ⇒ E – type of Conclusion “No debate is a meeting”.

    Some fights are debates. Points Digits

    II or

    I or

    Marks

    Digits Letters

    Points

    III Venn diagram II supports the Conclusion I. 125. (5) Venn diagrams I and II given in the answer to Question Number 14 support the Conclusion II. 126. (4)

    II or

    All farms are houses.

    Letters

    All debates are arguments. I + A ⇒ I-types of Conclusion “Some fights are arguments”Conclusion I and Conclusion II form Complementary Pair. Therefore, either Conclusion I or Conclusion II follows. 128. (1) All hands are limbs.

    All limbs are fingers. A + A ⇒ A–type of Conclusion “All hands are fingers”This is Conclusion II. Venn diagrams of ‘All limbs are fingers’ :

    rs ge F in Limbs

    Fingers or

    All houses are jungles.

    Digits

    A + A ⇒ A - type of Conclusion “All farms are jungles”.

    All houses are jungles. III Venn diagram II supports the Conclusion II. (124-125) : No point is a spot.

    Limbs I

    II

    Venn diagrams of ‘Some fingers are thumbs’ :

    No jungle is a ranch. A + E ⇒ E - type of Conclusion “No house is a ranch”. All farms are jungles.

    All spots are marks. E + A ⇒ O1 - type of Conclusion “Some marks are not points.” (P)

    All spots are marks.

    No mark is a dot. A + E ⇒ E - type of Conclusion “No spot is a dot.” (Q) 124. (3) Venn diagrams of “Some marks are not points” :

    Points

    Thumb

    Fingers

    III

    No jungle is a ranch. A + E ⇒ E - type of Conclusion “No farm is a ranch”. This is option (4). (127–131) : (i) All hands are limbs → Universal Affirmative (A-type). (ii) Some fingers are thumbs → Particular Affirmative (I-type). (iii) No member is a captain → Universal Negative (E-type). (iv) Some members are not captains → Particular Negative (O-type). 127. (2)

    or Fingers

    Thumbs

    Fingers

    IV

    or

    Thumbs

    V

    After combining the Venn diagrams II and V, we get : Fingers

    Thumbs

    All debates are arguments. I or

    Limbs

    No argument is a meeting.

    BPRE–559

    Thus, Conclusion I also follows.

    SYLLOGISM (132–133) :

    129. (4)

    All members are teams.

    Slopes

    Rivers

    Some projects are missions.

    or

    All teams are participants. A + A ⇒ A – type of Conclusion ¶All members are participants¸Conclusion I is Converse of it.

    No captain is a member.

    No mission is a guide. III

    From Venn diagrams I and II, it is clear that ‘Some slopes are rivers’.

    No mountain is a river.

    All members are participants. E + A ⇒ O1– type of Conclusion “Some participants are not captains”.

    No captain is a member.

    Some rivers are ponds. E + I ⇒ O1 – type of Conclusion ¶Some ponds are not mountains¸Venn diagrams of ‘Some ponds are not mountains’ :

    Ponds

    or

    Mo un

    E + A ⇒ O1– type of Conclusion ¶Some teams are not captains¸Venn diagrams of ‘Some teams are not captains’ :

    Projects

    Projects

    Crafts

    Ponds

    tai ns

    All members are teams.

    I + E ⇒ O–type of Conclusion. “Some projects are not guides”. 132. (2) Conclusion I and Conclusion II form Complementary Pair. Therefore, either Conclusion I or Conclusion II follows. 133. (5) Venn diagrams of “All crafts are projects” :

    I

    or

    M

    II

    Crafts

    I

    II

    Venn diagrams of “Some projects are missions” :

    s T eam Teams

    a in pt a C

    s or

    Captains

    I II

    or

    Ponds

    Mountains

    III

    M iss

    s ct je o Pr

    Projects

    io

    ns

    or

    III

    From Venn diagrams I and II, some ponds are mountains.

    Teams

    or

    Captains

    From Venn diagrams I and II, some (not all) teams are captains. 130. (3)

    Some slopes are mountains. No mountain is a river. I + E ⇒ O – type of Conclusion “Some Slopes are not rivers.” Venn diagrams of ‘Some slopes are not rivers’ :

    Slopes

    Rivers

    Slopes

    or

    Rivers

    II

    or

    Missions Projects

    All doors are walls. No wall is a ceiling. A + E ⇒ E – type of Conclusion “No door is a ceiling”. Conclusion II is Converse of it: (132–136) :

    V After combining venn diagrams II and III, we get :

    (i) All crafts are projects → Universal Affirmative (A–type). (ii) Some projects are missions → Particular Affirmative (I–type). (iii) No mission is guide → Universal Negative (E–type). (iv) Some missions are not guides →

    I

    IV

    131. (5) No gate is a door.

    All doors are walls. E + A ⇒ O1 – type of Conclusion “Some walls are not gates”.

    Missions

    Particular Negative (O–type).

    Projects Missions Crafts VI This venn diagram supports the Conclusion II.

    BPRE–560

    SYLLOGISM 141. (1) Venn diagrams of “Some tests are not challenges” :

    (134–135) :

    Some results are outputs.

    Expenses

    Prices

    Tests

    or

    All outputs are products. I + A ⇒ I–type of Conclusion “Some results are products.”(P)

    All outputs are products.

    III Venn diagram II supports the Conclusion II. (137–138) :

    es ng or le l a Ch

    Tests

    Challenges

    I

    II

    Some prices are costs. All products are yields.

    Yie l

    ds

    Yields

    rif

    fs or

    I

    or

    Prices

    II Prices

    I

    II or

    or Yields

    Tariffs

    III Venn diagram III supports the Conclusion II. 139. (4) No Conclusion follows from two Negative Premises. (140–141) :

    Results

    III Venn diagram III supports the Conclusion II. 136. (5) No price is a rate.

    Some tests are exams. No exam is a challenge.

    All rates are expenses. E + A ⇒ O1 – type of Conclusion “Some expenses are not prices.” Its venn diagrams :

    Some pictures are images.

    I + E ⇒ O – type of Conclusion “Some tests are not challenges” (P)

    No exam is a challenge.

    All images are memories. I + A ⇒ I–type of Conclusion “Some pictures are memories.” (P) 142. (4) Conclusion I is the Conclusion (P). Venn diagrams of “Some pictures are memories” : Pictures

    ie s

    R es ul ts

    Results

    Ta

    mo r

    Prices

    or

    Memories

    Me

    Tariffs

    III Thus, it is clear that all tests can never be challenges. (142–147) : (i) All mountains are peaks → Universal Affirmative (A–type). (ii) Some peaks are rivers → Particular Affirmative (I–type). (iii) No mountain is a valley → Universal Negative (E–type). (iv) Some mountains are not valleys → Particular Negative (O–type). (142–143) :

    es

    All products are yields. I + A ⇒ I–type of Conclusion “Some results are yields.” (R) 134. (3) Neither Conclusion I nor Conclusion II follows. 135. (1) Conclusion I is the Conclusion (Q). Venn diagrams of “Some results are yields” :

    Challenges

    Tests or

    tu r

    Some results are products.

    All costs are tariffs. I + A ⇒ I - type of Conclusion “Some prices are tariffs.” (P) 137. (4) From two Affirmative Premises, no Negative Conclusion can be derived. Therefore, neither Conclusion I nor Conclusion II follows. 138. (5) Conclusion I is the Conclusion (P). Venn diagrams of “Some prices are tariffs” :

    Pi c

    A + A ⇒ A–type of Conclusion “All outputs are yields.” (Q)

    I

    or

    II

    Memories

    Pictures

    Some challenges are wins.

    p en

    s es

    Expenses

    Prices

    Ex

    Prices

    or

    I

    II

    E + I ⇒ O1 – type of Conclusion “Some wins are not exams.” (Q) 140. (2) Conclusion I and Conclusion II form Complementary Pair. Therefore, either Conclusion I or Conclusion II follows.

    BPRE–561

    III Venn diagram II contradicts the Conclusion II. Therefore, only Conclusion I follows.

    SYLLOGISM (144–145) :

    Some memories are sketches. A + I ⇒ No Conclusion Venn diagrams of “Some pictures are memories” :

    Peaks

    ys

    All mountains are peaks. E + A ⇒ O1– type of Conclusion “Some peaks are not valleys.” 144. (3) Venn diagrams of “All mountains are peaks” :

    es

    tu

    M em or i

    Pic

    or

    I

    or

    Memories

    II

    Peaks

    Valleys

    II

    are rivers” :

    Peaks

    Venn diagrams of “Some peaks

    III Venn diagrams of “Some memories are sketches” :

    Rivers

    Peaks Rivers or

    Peaks

    Rivers

    III Sketches

    Rivers or

    Peaks

    Memories

    he s

    III Venn diagrams of “Some peaks

    are rivers” :

    Sk et c

    Peaks

    Mountains

    Pictures

    em or ie s

    II

    or

    Mountains

    I

    M

    Valleys

    or

    Peaks

    Memories

    or

    or

    I

    re s

    Pictures

    No valley is a mountain.

    Va lle

    All images are memories.

    Pe ak s

    143. (5)

    IV or

    IV

    V or

    Rivers Peaks

    Rivers IV

    or

    V

    Peaks

    Sketches Memories

    VI After combining venn diagrams III and VI, we get :

    VI After combining venn diagrams II and VI, we get :

    V After combining venn diagrams II and IV, we get :

    Rivers Peaks Valleys

    Peaks, Mountains

    Sketches Memories

    Rivers VII This diagram supports Conclu-

    Pictures

    VI VII This diagram supports Conclusion II. Therefore, only Conclusion II follows.

    This diagram contradicts Conclusion I. 145. (5) Venn diagrams of “Some peaks are not valleys” :

    BPRE–562

    sion II. 146. (2) Both the Premises are Particular Affirmative (I–type). No Conclusion follows from the two Particular Premises. Conclusion I and Conclusion II form Complementary Pair. Therefore, either Conclusion I or Conclusion II follows.

    SYLLOGISM 147. (5) All cars are trucks.

    (150–151) :

    No rubber is a pen.

    Some cows are goats. All trucks are vehicles. A + A ⇒ A–type of Conclusion “All cars are vehicles.” This is Conclusion II. (148–153) : (i) All phones are books → Universal Affirmative (A–type). (ii) Some cows are goats → Particular Affirmative (I–type). (iii) No drum is a book → Universal Negative (E–type). (iv) Some drums are not books → Particular Negative (O–type). (148–149) :

    All goats are deer.

    No pencil is a pen.

    All deer are buffaloes. A + A ⇒ A–type of Conclusion “All goats are buffaloes.” (Q)

    All pens are stickers.

    Some cows are deer.

    E + A ⇒ O1–type of Conclusion “Some stickers are not pencils. (R) 152. (3) Venn diagrams of “Some stickers are not rubbers” :

    All deer are buffaloes. I + A ⇒ I–type of Conclusion “Some cows are buffaloes.” (R) 150. (4) Conclusion I is the Converse of the Conclusion (P). Venn diagrams of “Some cows are buffaloes” :

    All phones are books.

    i St

    Stickers

    s er ck

    rs

    A + A ⇒ A–type of Conclusion “All caps are books.” (P)

    E + A ⇒ O1–type of Conclusion “Some stickers are not rubbers”. (Q)

    be

    All phones are books.

    I + A ⇒ I–type of Conclusion “Some cows are deer”. (P)

    b Ru

    All caps are phones.

    All pens are stickers. All goats are deer.

    I

    es lo fa

    All caps are books.

    Rubbers

    or

    Cows

    or

    Stickers

    Rubbers

    Buffaloes

    III I

    II or

    Venn diagram II supports the Conclusion I. 153. (2) Venn diagrams of “Some stickers are not pencils” :

    No book is a drum. Buffaloes

    ils nc

    S

    Stickers

    rs ke it c

    Pe

    A + E ⇒ E–type of Conclusion “No cap is a drum.” (R) 148. (5) Conclusion (Q) contradicts the Conclusion I. Venn diagram of “No phone is a drum” :

    II

    Cows uf B

    No book is a drum. A + E ⇒ E–type of Conclusion “No phone is a drum.” (Q)

    or

    or

    Pencils

    Cows I

    II or

    Phone

    Drum

    I Conclusion II is the Conclusion (R). 149. (3) Conclusion I is the Conclusion (P). Conclusion (R) contradicts the Conclusion II. Venn diagram of “No cap is a drum” :

    III Venn diagram II supports the Conclusion II. 151. (4) Conclusion I is the Conclusion (R). Conclusion II is the Conclusion (Q). (152–153) :

    All pencils are rubbers. No rubber is a pen.

    Cap

    Drum

    A + E ⇒ E–type of Conclusion “No pencil is a pen”. (P)

    BPRE–563

    Stickers

    Pencils III

    (154–159) : (i) All pizzas are burgers → Universal Affirmative (A–type). (ii) Some ducks are swans → Particular Affirmative (I–type). (iii) No wall is floor → Universal Negative (E–type). (iv) Some walls are not floors → Particular Negative (O–type).

    SYLLOGISM

    Pe ac oc ks

    s la te co C ho

    Peacocks

    or

    Ice–creams

    s m

    Pe ng ui ns

    Penguins

    ea

    or

    Chocolates cr e–

    Penguins

    After combining venn diagrams II and V, we get : Ic

    154. (4) All the three Premises are Particular Affirmative (I–type). No Conclusion follows from the two Particular Premises. Venn diagrams of “Some penguins are peacocks” :

    Ducks

    I

    II or

    Peacocks

    Ice–creams

    I

    or

    II

    Peacocks Penguins

    X Venn diagram X contradicts the Conclusion I. After combining venn diagrams V and VIII, we get :

    Chocolates

    III

    Peacocks

    Venn diagram II supports the Conclusion II.

    Ducks

    III Venn diagrams of “Some peacocks are ducks” :

    156. (4) No wall is floor.

    Swans

    D uc ks

    Pe ac o

    ck s

    Peacocks

    IV

    or

    or

    All floors are roofs. E + A ⇒ O – type of Conclusion 1 “Some roofs are not walls.” Venn diagrams of “All floors are roofs” :

    Ducks

    V

    Ducks

    XI Venn diagrams XI supports the Conclusion II. 155. (2)

    Roofs

    Floors

    Some chocolates are pizzas. or

    Floors

    Peacocks

    All pizzas are burgers.

    VI Venn diagrams of “Some ducks are swans” :

    Roofs

    I + A ⇒ I–Type of Conclusion “Some chocolates are burgers”. This is the Conclusion I.

    I

    II

    Venn diagrams of “Some roofs are windows” :

    All pizzas are burgers.

    VII

    or

    VIII

    Some chocolates are burgers.

    Swans

    s W in do w

    All burgers are ice–creams. A + A ⇒ A – type of Conclusion “All pizzas are ice–creams.”

    of s

    Swans

    Roofs

    Ro

    an s

    or

    Sw

    D

    uc ks

    Ducks

    III

    or

    or Windows

    All burgers are ice–creams. Ducks

    IX

    I + A ⇒ I–type of Conclusion “Some chocolates are ice– creams.” Venn diagrams of “Some chocolates are ice–creams”:

    BPRE–564

    Roofs

    V

    Windows

    IV

    SYLLOGISM After combining venn diagrams I and V, we get :

    or

    Wires, Plugs

    Switches

    Batteries

    Sw

    Windows

    itc he s

    W ir es

    Wires

    Roofs

    III

    IV

    Floors

    or

    X Venn diagram X supports the Conclusion II.

    Switches

    158. (2) Some cakes are biscuits. Wires

    VI

    All biscuits are pastries.

    Venn diagram VI contradicats the Conclusion I. Venn diagrams of “Some roofs are not walls” :

    V After combining the venn diagrams I and V, we get :

    I + A ⇒ I–type of Conclusion “Some cakes are pastries”. This is the Conclusion II. Venn diagrams of “Some cakes are pastries” : Cakes

    Switches

    Walls

    Wires

    VII

    or

    Pa st ri es

    or

    Ca ke s

    Ro

    W al ls

    of s

    Roofs

    Pastries

    Plugs

    VIII

    I

    or

    II or Pastries

    VI

    Walls

    Cakes

    Venn diagram VI supports the Conclusion I. Venn diagrams of “Some wires are batteries” : Wires

    Wires Plugs

    Plugs

    or

    or

    VII

    VIII or Batteries Wires

    Wires

    I

    II

    Venn diagrams of “Some wires are switches” :

    Pastries

    Batteries s

    Pa st ri es

    B at te ri es

    All switches are batteries. I + A ⇒ I–type of Conclusion “Some wires are batteries.” Venn diagrams of “All plugs are wires” :

    W ir

    es

    157. (2) Some wires are switches.

    III Venn diagrams of “Some pastries are doughnuts” :

    ut

    Venn diagrams VII and VIII support the Conclusion II.

    ou gh n

    IX

    or

    IV

    V or Doughnuts

    Pastries

    IX After combining venn diagrams II and VIII, we get :

    BPRE–565

    Doughnuts

    D

    Roofs

    VI

    SYLLOGISM After combining venn diagrams II and V, we get :

    After combining venn diagrams II and IV, we get :

    Venn diagrams of “Some pastries are sandwiches” :

    Blue Red

    Cakes Pastries

    Sa nd wi

    Pastries

    Pastries ch es or

    Sandwiches

    Green Doughnuts

    III

    IV

    or

    VI VII Venn diagram VII supports the Conclusion I. 159. (1) No green is yellow.

    All yellow are red. E + A ⇒ O –type of Conclusion 1 “Some red are not green”

    All yellow are red.

    Venn diagrams VI contradicts the Conclusion II. (160–164) : (i) All cookies are pastries → Universal Affirmative (A–type). (ii) Some pastries are sandwiches → Particular Affirmative (I–type). (iii) No biscuit is a cookie → Universal Negative (E–type). (iv) Some biscuits are not cookies → Particular Negative (O–type). (160–161) :

    Sandwiches Pastries

    V After combining Venn diagrams II and IV, we get :

    No biscuit is a cookie. All red are blue. A + A ⇒ A – type of Conclusion “All yellow are blue”. Conclusion I is the Converse of it. Venn diagrams of “Some red are not green” :

    or

    All cookies are pastries. E + A ⇒ O1–type of Conclusion “Some pastries are not biscuits.” 160. (4) Venn diagrams of “Some pastries are not biscuits” :

    i str Pa

    Bi sc

    es

    ui ts or

    Biscuits

    I

    I

    Sandwiches

    VI Pastries

    Green

    G

    re en

    R

    ed

    Red

    Pastries, Cookies

    Venn diagram VI supports the Conclusion I. (162–163) :

    Some locks are drawers.

    II or

    II

    All drawers are tables.

    or

    Biscuits

    Pastries

    I + A ⇒ I–type of Conclusion “Some locks are tables”. (P) 162. (4) Venn diagrams of “Some keys are locks” :

    Green

    Red

    Keys

    III Venn diagrams of “All red are blue” : Blue Red

    Red

    III

    Pastries

    or

    Cookies

    V

    Pastries

    or

    Locks

    I

    Locks Keys

    I

    BPRE–566

    II

    or

    Locks

    II

    or

    Cookies

    Blue

    IV

    Keys

    Venn diagram II supports the Conclusion II. 161. (3) Venn diagrams of “All cookies are pastries” :

    III

    SYLLOGISM

    Locks Drawers

    es

    All frames are pictures. IV

    or

    V

    Or

    Some pictures are images.

    Drawers Locks

    VI After combining Venn diagrams III and VI, we get :

    Drawers Locks

    A + I ⇒ No Conclusion (165–169) : (i) All circles are triangles → Universal Affirmative (A-type). (ii) Some triangles are rectangles → Particular Affirmative (I-type). (iii) No furniture is bed → Universal Negative (E-type). (iv) Some furniture are not beds → Particular Negative (O-type). (165–166) :

    Squares

    Sq ua

    or

    ng l

    rs

    Venn diagrams of “Some triangles are squares”:

    Tr ia

    Dr aw e

    Locks

    Venn diagram IV contradicts the Conclusion I. Conclusion II is the Conclusion (P). 164. (5)

    re s

    Venn diagrams of “Some locks are drawers” :

    VI

    VII

    Or

    Triangles

    VIII After combining the Venn diagrams IV and VIII, we get :

    Some triangles are rectangles.

    Keys

    Squares All rectangles are squares.

    VII Venn diagram VII supports the Conclusion II. 163. (4) We have,

    I + A ⇒ I-type of Conclusion “Some triangles are squares”. (P) 165. (5) Venn diagrams of “Some triangles are rectangles” :

    es

    ng l

    I Venn diagrams of “All drawers are tables” : Drawers

    Or

    gl

    Tr ia

    Keys

    Rectangles

    Re ct an

    IX

    I

    II

    Or

    Venn diagram IX supports the Conclusion II. 166. (4) Neither Conclusion I nor Conclusion II follows. (167–168) :

    Triangles

    No furniture is bed.

    or Tables

    Triangles

    Keys

    Triangles Circles

    or

    Chairs

    bl es

    Locks

    irs

    Drawers, Tables

    Some beds are tables. E + I ⇒ O1-type of Conclusion “Some tables are not furniture.” (P) 167. (5) Venn diagrams of “Some chairs are tables” :

    ha

    After combining Venn diagrams I and III, we get :

    III Venn diagram II supports the Conclusion I. Venn diagrams of “All circles are triangles”:

    C

    III

    II

    Circles IV

    IV

    V

    BPRE–567

    or

    Tables

    Ta

    Drawers

    Circles

    es

    Drawers Locks

    Tables

    Triangles

    I

    II

    SYLLOGISM or

    Venn diagrams of “Some dramas are theatres”:

    Tables or

    Dramas

    Dra m

    as

    s at r e The

    Chairs

    III

    or

    III

    II Theatres

    ni tu re

    Ta ble s

    Tables or

    Theatres

    I

    Venn diagrams of “Some tables are not furniture” :

    or

    It is clear that “All boxes can never be envelopes.” Therefore, only Conclusion II follows. 171. (2) Venn diagrams of “Some boxes are not packets” :

    Dramas

    Fu r

    Furniture

    Envelopes

    Boxes

    Boxes

    or

    Furniture

    Tables VI

    After combining Venn diagrams III and V, it may be said that Conclusion I follows. Venn diagrams of “Some beds are tables” :

    s ed

    T ab l

    B

    es

    Beds Tables

    or

    III

    Boxes

    Venn diagram III supports the Conclusion I. (170–174) : (i) All packets are envelopes → Universal Affirmative (A–type). (ii) Some gifts are boxes → Particular Affirmative (I–type) (iii) No envelop is a gift → Universal Negative (E–type). (iv) Some envelopes are not gifts → Particular Negative (O–type). (170–171) :

    All packets are envelopes.

    I

    VIII Tables

    or Beds

    II

    Boxes

    Packets III

    Venn diagram II supports the Conclusion I. (172–173) :

    No envelop is a gift.

    All diaries are novels. All novels are biographies. A + A ⇒ A – type of Conclusion “All diaries are biographies” (P) 172. (2) Venn diagrams of “All diaries are biographies” :

    No envelop is a gift. Some gifts are boxes. E + I ⇒ O –type of Conclusion 1 “Some boxes are not envelopes.” (Q)

    Diaries

    Biographies

    No packet is a gift.

    Diaries

    or

    IX Venn diagrams VII supports the Conclusion II. 168. (1) Conclusion I is the Conclusion (P). Conclusion II does not follow.

    Biographies

    Some gifts are boxes. I

    E + I ⇒ O –type of Conclusion 1 “Some boxes are not packets.” (R) 170. (3) Venn diagrams of “Some boxes are not envelopes” :

    169. (5) Some dramas are arts.

    Venn diagrams of “Some biographies are scripts” :

    “Some drams are theatres.”. This is the Conclusion II.

    lo pe

    s

    Boxes

    or

    Envelopes

    nv e

    Boxes

    II

    io B

    ap gr

    es hi

    Biographies Scripts

    or

    Scripts

    E

    All arts are theatres. I + A ⇒ I-type of Conclusion

    packets

    or

    A + E ⇒ E–type of Conclusion “No packet is a gift.” (P)

    VII

    or

    ke ts

    V

    Pa c

    IV

    I

    II

    BPRE–568

    III

    IV

    SYLLOGISM or

    After combining venn diagrams II and IV, we get :

    Scripts

    Venn diagrams of “Some pens are erasers” :

    Novels, Biographies

    Pens

    Biographies

    Pens

    Scripts

    E

    V After combining venn diagrams II and V, we get :

    Scripts Diaries

    Biographies

    VI Venn diagram VI supports the Conclusion I. 173. (2) Conclusion (P) is the Conclusion I. Venn diagrams of “All novels are biographies” :

    or Biographies

    I

    II

    Venn diagrams of “Some biographies are scripts” :

    ap gr io B

    es hi

    se

    Erasers

    IV

    V

    Erasers Pens or

    VI After combining Venn diagrams II and V, we get :

    Pens Erasers

    Lighters

    Novels

    Biographies

    Novels

    VI Venn diagram VI contradicts the Conclusion II. 174. (5) Both the Premises are Particular Affirmative (I–type). No Conclusion follows from the two Particular Premises. Conclusion I and Conclusion II form Complementary Pair. Therefore, either Conclusion I or Conclusion II follows. (175–179) : (i) All markers are lighters → Universal Affirmative (A–type). (ii) Some pens are erasers → Particular Affirmative (I–type). (iii) No mail is a notice → Universal Negative (E–type). (iv) Some mails are not notices → Particular Negative (O–type). (175–176) : Some erasers are markers.

    ra

    or rs

    All markers are lighters. I + A ⇒ I-type of Conclusion “Some erasers are lighters.” (P) 175. (3) Conclusion I is the Conclusion (P). Venn diagrams of “Some erasers are lighters” : Erasers

    Biographies Scripts

    or

    Scripts

    Er

    as

    e

    rs

    L III

    IV or

    r te igh

    s or

    Lighters

    I

    II

    Scripts

    Lighters

    Biographies

    Erasers or

    V

    III

    BPRE–569

    VII Venn diagram VII supports the Conclusion II. 176. (5) Neither Conclusion I nor Conclusion II follows. (177–178) :

    No mail is a notice. All notices are intimations. E + A ⇒ O1-type of Conclusion “Some intimations are not mails.” (P) 177. (4) Venn diagrams of “Some intimations are not mails” : ns io at m ti In

    I

    Intimations

    Mails

    or

    Mails

    II

    SYLLOGISM

    Intimations

    Mails

    or

    tim In

    a

    Venn diagrams of “Some intimations are warnings” :

    In

    tim

    Hours

    Wa

    III

    rn

    s ing rn a W

    or

    s or i ng

    Warnings

    III

    IV

    Days

    III

    IV

    Days

    Warnings

    Hours or

    Warnings

    Intimations

    IV

    or

    Days

    or

    Intimations

    ns io at

    Hours

    Intimations

    ns tio

    V After combining Venn diagrams II and IV, we get :

    V

    Warnings V After combining Venn diagrams II and V, we get :

    Intimations or

    Hours, Minutes Days

    Warnings After combining Venn diagrams II and VI, we get :

    Notices

    VI

    Warnings

    Intimations

    Intimations Mails

    VI Venn diagram VI supports the Conclusion II. 179. (4) All seconds are minutes.

    VII Venn diagram VII supports the Conclusion II. 178. (4) Venn diagrams of “All notices are intimations” :

    Minutes

    Notices Minutes

    or

    II

    Venn diagrams of “Some intimations are warnings” :

    No book is pencil.

    No book is pencil.

    or

    All pencils are rubbers.

    Intimations Hours

    I

    All copies are books.

    A + E ⇒ E – type of Conclusion “No copy is pencil”. (P)

    Hours

    Intimations Notices

    All minutes are hours. A + A ⇒ A-type of Conclusion “All seconds are hours”. This is Conclusion II.

    Venn diagram VI contradicts the Conclusion I. (180–184) : (i) All copies are books → Universal Affirmative (A–type). (ii) Some pens are scales → Particular Affirmative (I–type). (iii) No book is a pencil → Universal Negative (E–type). (iv) Some books are not pencils → Particular Negative (O–type). (180–181) :

    I

    II

    Venn diagrams of “Some hours are days” :

    BPRE–570

    E + A ⇒ O1–type of Conclusion “Some rubbers are not books”. (Q)

    No copy is pencil. All pencils are rubbers.

    SYLLOGISM E + A ⇒ O1–type of Conclusion “Some rubbers are not copies”. (R) 180. (2) Neither Conclusion I nor Conclusion II follows. 181. (3) Conclusion I is the Conclusion (P). (182–183) : All the three Premises are Particular Affirmative (I–type). No Conclusion follows from the two Particular Premises. 182. (1) Conclusion I and Conclusion II form Complementary Pair. Therefore, either Conclusion I or Conclusion II follows. 183. (5) Venn diagrams of “Some calculators are pens” :

    Some woods are leaves.

    la lcu Ca

    or

    Pens

    Some goats are deer.

    All leaves are trunks. I + A ⇒ I–type of Conclusion “Some woods are trunks”. (P) 185. (5) Conclusion II is the Converse of the Conclusion (P). 186. (4) Venn diagrams of “Some woods are trunks” : Woods Tr un

    Woods

    ks

    or

    Deer Ho r se

    Deer

    s or

    Horses

    I

    II or

    II or

    Pens

    Trunks

    I

    E + I ⇒ O –type of Conclusion 1 “Some deer are not horses.” Venn diagrams of “Some deer are not horses” :

    Trunks

    I Calculators

    s tor

    190. (5) No horse is a goat.

    II

    Calculators

    Horses

    Woods

    III

    or Pens

    Deer

    III Venn diagram II supports the Conclusion I. (187–188) :

    Venn diagrams I and II support the Conclusion II. (191–192) :

    Some benches are roads.

    No mobile is a band. No road is a sky. All bands are pillows.

    III Venn diagram II supports the Conclusion II. 184. (3)

    Some drinks are eatables. All eatables are sweets. I + A ⇒ I – type of Conclusion “Some drinks are sweets”. Conclusion I is the Converse of it. (185–189) : (i) All papers are woods → Universal Affirmative (A–type). (ii) Some woods are leaves → Particular Affirmative (I–type). (iii) No mobile is a band → Universal Negative (E–type). (iv) Some mobiles are not bands → Particular Negative (O–type). (185–186) :

    E + A ⇒ O1–type of Conclusion “Some pillows are not mobiles”. (P) 187. (1) Neither Conclusion I nor Conclusion II follows. 188. (1) Neither Conclusion I nor Conclusion II follows.

    I + E ⇒ O–type of Conclusion “Some benches are not skies.” (P) 191. (5) Conclusion (P) is the COnclusion II. 192. (3) Neither Conclusion I nor Conclusion II follows. (193–194) :

    All plates are spoons.

    189. (2) All ladders are snakes.

    Some snakes are frogs. A + I ⇒ No Conclusion Conclusion I and Conclusion II form Complementary Pair. Therefore, either Conclusion I or Conclusion II follows. (190–194) : (i) All plates are spoons → Universal Affirmative (A–type). (ii) Some glasses are mugs → Particular Affirmative (I–type) (iii) No horse is a goat → Universal Negative (E–type). (iv) Some horses are not goats → Particular Negative (O–type).

    BPRE–571

    All spoons are glasses. A + A ⇒ A – type of Conclusion “All plates are glasses.” (P) 193. (4) Venn diagrams of “All plates are glasses” :

    Plates

    Glasses Plates

    or Glasses

    I

    II

    Venn diagrams of “Some glasses are mugs” :

    SYLLOGISM

    IV or

    All rods are sticks

    Mugs

    All sticks are weights Glasses

    A + A ⇒ A–type of conclusion. “All rods are weights.” [Conclusion III] Hence, Conclusion III follows.

    V After combining venn diagrams II and V, we get

    Some windows are sticks. [P]

    All flip-flops are memories. E + A ⇒ O1–type of Conclusion “Some memories are not gates.”(Q) 198. (5) Venn diagrams of “Some memories are not gates” :

    Memories s te Ga

    I + A ⇒ I–type of Conclusion “Some windows are weights” [Q] Conclusion I is the converse of Conclusion Q Hence, Conclusion I follows. Venn diagrams :

    II

    VI Venn diagram VI supports the Conclusion I. 194. (2) Conclusion I and Conclusion II form Complementary Pair. Therefore, either Conclusion I or Conclusion II follows. 195. (2) Conclusion II follows from the first statement.

    All blades are knives All knives are peeclers A + A ⇒ A–type of conclusion. “All blades are peelers” [P] Conclusion III follows from conclusion P. Venn diagrams :

    Peelers Knives Blade

    Weights Sticks Doors Windows

    Rods

    Gates

    III From the above Venn diagrams it is clear that “All memories can never be gates”. Therefore, Conclusion II follows. 199. (5) Venn diagrams of “Some memories are not gates” :

    Memories 197. (4) All the statements are particular Affirmative. So, no Conclusion can be derived from them. None of the Conclusions is the converse of any statement. Venn diagrams :

    es at G

    I

    or

    or

    Gates

    II

    Leaves Flowers Petals Fruits Plants

    (198–202) : (i) All diodes are transistors → Universal Affirmative (A–type). (ii) Some clouds are gates → Particular Affirmative (I–type). (iii) No gift is a box → Universal Negative (E–type). (iv) Some gifts are not boxes → Particular Negative (O–type). (198–199) :

    Some clouds are gates. From above Conclusion I does not follow.

    Memories

    s

    Glasses

    Gates

    or

    ri e

    Plates

    or

    I All sticks are weights.

    Mugs

    Pen Eraser

    No gate is a flip-flop.

    s

    III

    All rods are sticks I + A ⇒ I-type of conclusion Some windows are sticks. [P] Conclusion II is the converse of conclusion [P], hence follows.

    rie

    Mugs

    mo

    or

    mo

    Mugs

    Me

    es ass Gl

    I + E ⇒ O–type of Conclusion “Some clouds are not flipflops.”(P)

    196. (3) Some windows are rods

    Me

    Glasses

    No gate is a flip-flop.

    BPRE–572

    Memories

    Gates

    III Venn diagrams I and II support the Conclusion II. (200–201) : Some parcels are gifts.

    No gift is a box. I + E ⇒ O-type of Conclusion “Some parcels are not boxes.”(P)

    SYLLOGISM 200. (5) Conclusion (P) supports the Conclusion II. 201. (4) Venn diagrams of “Some parcels are gifts” :

    ls

    Parcels r ce

    or

    Pa

    II or

    Gifts r

    Venn diagrams of “Some flats are buldings” : Flats Flats

    B

    Gifts

    fts Gi

    I

    (iii) No stone is fish → Universal Negative (E–type). (iv) Some stones are not fishes → Particular Negative (O–type). 203. (2) Some buds are leaves.

    Parcels

    All leaves are plants. I + A ⇒ I – type of Conclusion “Some buds are plants.” Conclusion II and Conclusion III form complementary pair. Therefore, either II or III follows. 204. (4) Some flats are buildings.

    All buildings are bungalows. I + A ⇒ I – type of Conclusion “Some flats are bungalows.”

    III Venn diagram II supports the Conclusion I.

    gs i n or

    IV

    Buildings

    V

    or

    Buildings Flats

    VI After combining the venn diagrams III and VI, we get :

    All buildings are bungalows.

    202. (5) Some chips are diodes.

    ld ui

    Buildings Flats

    All bungalows are gardens. All diodes are transistors. I + A ⇒ I-type of Conclusion “Some chips are transistors”. Conclusion II is the Converse of it. Venn diagrams of “Some chips are diodes” :

    Some flats are bungalows.

    All bungalows are gardens. I + A ⇒ I – type of Conclusion “Some flats are gardens.” This Conclusion Contradicts the Conclusion III. “All bungalows are not buildings” implies that “Some bungalows are buildings.” Thus, Conclusion II follows. Venn diagrams of “Some apartments are flats” :

    ips

    Chips

    Ch

    es od Di

    or

    I

    Diodes

    II or

    Diodes Chips

    Ap

    tm ar

    en

    ts

    Apartments

    or

    Flats

    I

    III Venn diagram III contradicts the Conclusion I. (203–204) : (i) All petals are flowers → Universal Affirmative (A–type). (ii) Some flowers are buds → Particular Affirmative (I–type).

    Apartments

    A + A ⇒ A – type of Conclusion “All buildings are gardens.”

    Flats

    II

    or Flats

    VII Venn diagram VII clearly supports the Conclusion I. Therefore, Conclusion I and Conclusion II follow. 205. (5) Some stones are shells.

    All shells are pearls. I + A ⇒ I – type of Conclusion “Some stones are pearls.” Some shells are stones.

    No stone is fish. I + E ⇒ O - type of Conclusion “Some shells are not fish.” Venn diagrams of “Some shell are not fish” : Shells

    Apartments

    III

    BPRE–573

    Fish

    Shells I

    or

    Fish

    II

    SYLLOGISM A + E ⇒ E - type of Conclusion “No glider is an airplane.”

    or

    I + E ⇒ O–type of Conclusion “Some calendars are not days.” Venn diagrams of “Some calendars are not days.” :

    No parachute is an airplane. Shells

    Fish

    Therefore, neither Conclusion I nor Conclusion II follows. 206. (3)

    No glider is an airplane.

    da en al

    Days

    Days

    or

    C

    E + A ⇒ O1 – type of Conclusion “Some helicopters are not parachutes.”

    III

    Cal end ars

    rs

    All airplanes are helicopters.

    I

    II Or

    All clerks are assistants. All airplanes are helicopters. No assistant is an officer. A + E ⇒ E - type of Conclusion “No clerk is an officer.”

    E + A ⇒ O1 – type of Conclusion “Some helicopters are not gliders.” Venn diagrams of “Some helicopters are not parachutes”:

    No assistant is an officer. Helicopters

    All officers are managers. E + A ⇒ O1 - type of Conclusion “Some managers are not assistants.”

    lic He

    Par ach ut

    s ter op

    es

    or

    Parachutes

    I

    II

    Days

    Calendars

    III Venn diagram II states that “All days are calendars.” It can also be represented as :

    Calendars

    or

    No clerk is an officer.

    Days All officers are managers. E + A ⇒ O1 - type of Conclusion “Some managers are not clerks.” Venn diagrams of “Some managers are not assistants” : rs ge na Ma

    Ass ista

    Managers

    nts

    I

    or

    Assistants

    II

    or

    Managers

    Assistants

    III

    Venn diagram II supports the Conclusion II. 207. (1)

    Helicopters

    Parachutes

    III Venn diagram II supports the Conclusion I. (208–210) : (i) All months are days → Universal Affirmative (A–type). (ii) Some hours are calendars → Particular Affirmative (I–type). (iii) No hour is a day → Universal Negative (E–type). (iv) Some hours are not days → Particular Negative (O–type). 208. (3) All months are days.

    No day is an hour. A + E ⇒ E–type of Conclusion “No month is an hour.” This is Conclusion I.

    All gliders are parachutes.

    Some calendars are hours.

    No parachute is an airplane.

    No hour is a day.

    BPRE–574

    i.e., All calendars are days. Therefore, Conclusion II also follows. 209. (3) Some books are comics.

    All comics are novels. I + A ⇒ I–type of Conclusion “Some books are novels.” Conclusion I is the converse of it.

    All comics are novels. No novel is an article. A + E ⇒ E–type of Conclusion “No comics is an article.”

    Some books are novels. No novel is an article. I + E ⇒ O–type of Conclusion “Some books are not articles.” Venn diagram of “Some books are not articles” :

    SYLLOGISM Books Books

    Venn diagrams of “Some flutes are not pianos” :

    Articles

    Articles or

    No bird is an animal.

    Flu tes Flu tes

    I

    “All pets are reptiles.”

    Pianos

    or

    Pianos

    II

    or I

    II

    All animals are reptiles. E + A ⇒ O1–type of Conclusion “Some reptiles are not birds.”

    No animal is a bird.

    or

    Books

    Articles

    III Venn diagram II supports the Conclusion II.

    Some birds are insects. E + I ⇒ O1–type of Conclusion “Some insects are not animals.” Venn diagrams of “Some insects are not animals”:

    Piano

    Flutes

    III

    210. (4) All gases are solids.

    Venn diagrams of “Some pianos are violins” :

    Insects Insects

    Animals or

    Animals

    All solids are liquids. A + A ⇒ A–type of Conclusion “All gases are liquids.” This is Conclusion II.

    Pianos Pianos

    Violins

    or

    I

    II

    Violins

    or

    All solids are liquids. IV

    V

    or

    No liquid is fluid. A + E ⇒ E–type of Conclusion “No solid is fluid.” 211. (5) Some flutes are guitars.

    Violins

    Pianos

    No guitar is a piano. I + A ⇒ O–type of Conclusion “Some flutes are not pianoes.”

    No guitar is a piano.

    After combining Venn diagrams II and V, we get :

    Some pianos are drums.

    Flutes Pianos Violins

    All drums are violins. I + A ⇒ I–type of Conclusion “Some pianos are violins.” Venn diagrams of all the premises :

    Therefore, option (5) may be true. 212. (2) All pets are anim als.

    Flutes

    Guitars

    Pianos

    Drums Violins

    Anim als

    III

    Venn diagrams II states that, “All animals are insects.” It can also be represented as the following diagram :

    VI

    E + I ⇒ O1 – type of Conclusion “Some drums are not guitars.”

    Some pianos are drums.

    Insects

    All anim als are reptiles.

    A + A ⇒ A – type of Conclusion

    BPRE–575

    Insects

    Animals

    Therefore, option (2) is correct. 213. (3) Statement A & B and C & E and D & F are contradictory. Hence, groups ABE, ADF and BCE are not logically related. Statement [A] : All scholars are fools. Statement [C] : All those who are fools are uneducated. A + A ⇒ A–type of Conclusion All scholars are uneducated. [Statement F] Hence, statements A, C, F are logically related.

    SYLLOGISM 214. (1) In group CBA, statement C is of different nature. In group ABE, statement E is of different nature/type. In group BDF all three statements are of different kind. The relationship between statements [C] and [E] can be shown by the following Venn diagram.

    Polish Combs

    Oils

    or

    Desks

    or

    Chairs

    III Venn diagrams I and II support the Conclusion II.

    Venn diagrams of “Some hits are misses” :

    217. (3) All segments are episodes.

    Hits Misses or

    Hits

    A + E ⇒ E–type of Conclusion “No segment is a movie”. This is the Conclusion II.

    IV

    All movies are films. Hits

    Films Ep is

    Films

    od es

    or

    VI After combining the venn diagrams II and V, we get :

    Episodes Hits

    I

    Misses

    II or

    Wins

    Episodes

    Films

    216. (5) No chair is a table.

    VII

    All tables are desks. E + A ⇒ O1–type of Conclusion “Some desks are not chairs.”

    III Venn diagram II supports the Conclusion I.

    All tables are desks.

    (218–219) : Some misses are losses.

    No desk is a paper. A + E ⇒ E–type of Conclusion

    No loss is a win. I + E ⇒ O–type of Conclusion “Some misses are not wins”. (P) 218. (2) Venn diagrams of “Some misses are not wins” :

    “No table is a paper”. Venn diagrams of “Some desks are not chairs” :

    Misses

    Desks Chairs

    II

    es iss M

    I

    V

    Misses

    E + A ⇒ O1–type of Conclusion “Some films are not episodes”. Venn diagrams of “Some films are not episodes” :

    All authorities are controls. A + A ⇒ A–type of Conclusion “All powers are controls”. This is the Conclusion I. Negative Conclusion does not follow from the Affirmative Premises.

    Chairs or

    Misses

    or

    No episode is a movie.

    215. (4) All powers are authorities.

    Desks

    Wins

    III

    No episode is a movie. From the diagram, some polishes are oils (or) some combs used for oils, is a possibility. This is statement F. Hence, statements C, E, F are logically related. (215–219) : (i) All tables are desks → Universal Affirmative (A–type). (ii) Some controls are rules → Particular Affirmative (I–type). (iii) No chair is a table → Universal Negative (E–type). (iv) Some chairs are not tables → Particular Negative (O–type).

    Misses

    Wins or

    I

    Wins

    II

    BPRE–576

    Venn diagram VII contradicts the Conclusion II. 219. (4) We have venn diagrams : Misses Wins

    Misses

    or

    I

    Hits

    II

    After combining venn diagrams I and II, we get Misses Wins

    Hits

    III

    Misses

    or

    Wins

    IV

    Hits

    SYLLOGISM or

    After combining Venn diagrams I and V, we get :

    Misses Wins

    222. (3) Some ideas are plans.

    Requests

    All plans are blueprints.

    Demands

    I + A ⇒ I–type of Conclusion “Some ideas are blueprints”. Conclusion I is Converse of it. Venn diagrams of “Some ideas are blueprints” :

    Hits Methods

    V

    Demands Services

    Id ea

    s

    VI

    Venn diagram VI supports the Conclusion I. Venn diagrams of “All services are demands” :

    or

    ts in

    220. (3) All methods are services.

    Ideas

    pr ue Bl

    Venn diagram V supports the Conclusion I. (220–224) : (i) All methods are services → Universal Affirmative (A–type). (ii) Some demands are requests → Particular Affirmative (I–type). (iii) No hotel is a cottage → Universal Negative (E–type). (iv) Some hotels are not cottages → Particular Negative (O–type).

    I

    Demands

    or

    Blueprints

    II

    Blueprints

    or Ideas Services

    All services are demands. A + A ⇒ A–type of Conclusion “All methods are demands.” Venn diagrams of “All methods are demands” :

    VII

    VIII

    After combining Venn diagrams V and VII, we get :

    III

    Venn diagrams of “Some blueprints are maps” :

    Requests pr in

    ts

    Blueprints

    Bl ue

    Demands

    s ap M

    Methods

    Demands or

    Methods

    Demands

    eq u

    es ts

    em an ds

    IV

    V

    or

    II

    Venn diagrams of “Some demands are requests” :

    IX

    Venn diagram IX supports the Conclusion II.

    Maps

    Blueprints

    221. (3) All resorts are hotels. or

    Requests

    R

    D

    Maps

    Services Demands

    I

    or

    No hotel is a cottage. III

    IV

    or Requests

    Demands

    A + E ⇒ E–type of Conclusion “No resort is a cottage.” Conclusion II is the Converse of it.

    All lodges are cottages.

    VI

    After combining Venn diagrams II and V, we get : Ideas Blueprints Maps

    No cottage is a hotel. A + E ⇒ E–type of Conclusion “No lodge is a hotel”. This is the Conclusion I.

    V

    BPRE–577

    VII

    Venn diagram VII supports the Conclusion II.

    SYLLOGISM (223–224) : Some stocks are bills.

    (225–226) :

    Some mails are letters. No bill is a dividend. I + E ⇒ O–type of Conclusion “Some stocks are not dividends”. (P) No bill is a dividend. All dividends are cuts. E + A ⇒ O1–type of Conclusion “Some cuts are not bills”. (Q) 223. (2) Venn diagrams of “Some stocks are not dividends” :

    or

    Posts Mails or

    Posts

    Mails

    I II

    Option

    Question

    Alternative

    Mails

    or

    Thus, “All alternatives can never be options”. 228. (3)

    Posts Stocks

    No choice is a question. A + E ⇒ E= type of Conclusion “No alternative is a question.” Venn diagrams of “No option is a question” and “No alternative is a question” :

    II or

    I

    All alternatives are choices.

    Dividends

    D

    St o

    iv id en

    ds

    ck s

    Stock

    No letter is a circular. I + E ⇒ O-type of Conclusion “Some mails are not circulars”. (P) 225. (1) Conclusion I and Conclusion II form Complementary Pair. Therefore, either Conclusion I or Conclusion II follows. 226. (2) Venn diagrams of “Some posts are mails” :

    Venn diagram VII supports the Conclusion I. It is clear from the Venn diagrams IV, V and VI that “All mails can never be circulars”. Thus, both the Conclusion I and Conclusion II follow. 227. (4)

    Dividends

    Some vitamins are syrups. III

    Venn diagram II contradicts the Conclusion I, but supports the Conclusion II. 224. (1) Venn diagrams of “Some cuts are not bills” :

    Bi lls

    C ut s

    Cuts or

    Bills

    III Venn diagrams of “Some mails are not circulars” :

    Mails

    C ir c

    Mails ul

    ar or s

    Circulars

    IV

    V

    All syrups are medicines. I + A ⇒ I-type of Conclusion “Some vitamins are medicines”. Venn diagrams of “Some vitamins are medicines” :

    in Vitam

    Me dic ine s

    s

    Vitamins

    or

    Medicines

    or II

    I

    or

    I

    Circulars

    Mails

    II

    or

    Medicines Cuts

    Bills

    III

    Venn diagram II supports the Conclusion I. (225–229) : (i) All alternatives are choices → Universal Affirmative (A–type). (ii) Some posts are mails → Particular Affirmative (I–type) (iii) No letter is circular → Universal Negative (E–type). (iv) Some letters are not circulars → Particular Negative (O–type).

    VI After combining Venn diagrams II and V, we get :

    Posts Mails Circulars

    Vitamins

    III Venn diagram II supports the Conclusion I. 229. (4)

    Some planets are stars.

    VII

    BPRE–578

    No star is a comet.

    SYLLOGISM I + E ⇒ O-type of Conclusion “Some planets are not comets”. All moons are stars.

    No star is a comet. A + E ⇒ E-type of Conclusion “No moon is a comet”. Venn diagrams of “Some planets are stars” :

    Venn diagram VI contradicts Conclusion I. Venn diagrams of “Some planets are not comets” :

    Stars

    or

    Planets

    Comets

    or

    V After combining venn diagrams II and IV, we get :

    Comets

    or

    VII

    VIII

    Plots, Themes

    or

    Stars Biographies

    Planets I

    Biographies

    Themes

    Planets

    Planets Pl anets

    or

    Comets

    II

    Stars Planets

    III

    Venn diagrams of “All moons are stars” :

    Stars Moons

    IX It is clear from the above venn diagrams that Conclusion II follows. (230–234) : (i) All plots are themes → Universal Affirmative (A–type). (ii) Some themes are fictions → Particular Affirmative (I–type). (iii) No fiction is a biography → Universal Negative (E–type). (iv) Some fictions are not biographies → Particular Negative (O–type). 230. (5) Some themes are fictions.

    VI Venn diagram VI contradicts the Conclusion I. Ven diagrams III and IV supports the Conclusion II. (231–232) : Some trainings are drills. All drills are bars. I + A ⇒ I–type of Conclusion “Some trainings are bars.” (P) 231. (1) Conclusion I is the Converse of the Conclusion (P). Venn diagrams of “Some trainings are bars” :

    No fiction is a biography.

    Plots

    Moons V After combining venn diagrams II and IV, we get

    I

    II

    or

    Trainings Themes

    I II Venn diagrams of “Some themes are not biographies” :

    Planets Stars

    III Venn diagrams of “Some bars are goals” : Bars

    Themes

    or

    Biographies

    Ba

    rs

    ls oa

    es

    G

    em

    p ra og

    or

    Gaols

    s ie h

    Th

    Bi

    Moons

    VI

    Bars

    Bars

    Plots

    Themes

    or

    s

    Stars

    Tr

    s

    ar

    or

    Trainings ng ni ai

    B

    I + E ⇒ O–type of Conclusion “Some themes are not biographies”. Venn diagrams of “All plots are themes” :

    IV

    III

    IV

    BPRE–579

    IV

    V or

    SYLLOGISM Goals

    Goals

    Bars

    Bars

    VI After combining venn diagrams II and V, we get :

    VIII After combining the Venn diagrams V and VII, we get :

    Trainings Bars

    Bars, Drills

    Goals

    Goals

    VII Venn diagram VII supports the Conclusion II. 232. (4) Venn diagrams of “Some trainings are drills” :

    IX

    Venn diagram IX supports the Conclusion I. 233. (3) Some letters are posts.

    Trainings s

    D

    or

    ll ri

    Drills

    s

    Tr

    ng ni ai

    I

    or

    II

    Drills

    III Venn diagram II contradicts the Conclusion II. Venn diagrams of “All drills are bars” : Drills

    Drills

    All posts are digits. I + A ⇒ I–type of Conclusion “Some letters are digits”. Conclusion I and Conclusion II form Complementary Pair. Therefore, either Conclusion I or Conclusion II follows. 234. (5) All paints are machines.

    Trainings

    Bars

    Wires

    or Bars

    IV V Venn diagrams of “Some bars are goals” :

    No machine is a circuit. A + E ⇒ E–type of Conclusion “No paint is a circuit”. Thus, Conclusion I does not follow. No machine is a circuit. All circuits are wires. E + A ⇒ O1–type of Conclusion “Some wires are not machines”. No paint is a circuit. All circuits are wires. E + A ⇒ O1–type of Conclusion “Some wires are not paints”. Venn diagrams of “Some wires are not machines” :

    Bars Goals

    s

    or

    hi ne

    i re W

    ac

    ls oa

    or

    III Venn diagrams I and II support the Conclusion II. (235–239) : (i) All mobiles are discs → Universal Affirmative (A–type). (ii) Some discs are spheres → Particular Affirmative (I–type). (iii) No sphere is charger → Universal Negative (E–type). (iv) Some spheres are not chargers → Particular Negative (O–type). (235–236) : Some discs are spheres. No sphere is charger. I + E ⇒ O–type of Conclusion “Some discs are not chargers”. 235. (3) Venn diagrams of “All mobiles are discs” : Discs Mobiles

    Discs

    or Mobiles

    I II Venn diagrams of “Some discs are spheres” : Discs

    scs Di

    Sp he re or s

    III

    or

    Spheres

    IV

    Spheres Discs

    V After combining venn diagrams II and V, we get : Spheres Discs

    M

    rs

    G

    Ba

    Wires

    Machines

    Machines

    Mobiles

    s

    VI

    or

    VII I

    or

    BPRE–580

    II

    VI Venn diagram VI supports the Conclusion I.

    SYLLOGISM 236. (5) Neither Conclusion I nor Conclusion II follows.

    Puzzles

    237. (1) All books are pencils.

    Dramas

    III

    IV

    After combining the venn diagrams II and IV, we get :

    Cakes

    Dramas C ak

    or

    II

    (238–239) : All games are puzzles.

    No puzzle is theory. A + E ⇒ E–type of Conclusion “No game is theory”. (P)

    All dramas are puzzles. No puzzle is theory. A + E ⇒ E–type of Conclusion “No drama is theory”. (Q) 238. (5) Venn diagram of “No drama is theory” :

    Fruits

    Theory

    Games

    or

    Cakes

    Fruits VI

    After combining venn diagrams III and V, we get :

    Cakes Fruits Juices

    No fruit is potato. I + E ⇒ O–type of Conclusion “Some juices are not potatoes”. (P)

    I

    Venn diagram I contradicts the Conclusion I. Neither Conclusion I nor Conclusion II follows. 239. (1) Venn diagrams of “All games are puzzles” :

    V or

    Some juices are fruits.

    No fruit is potato.

    VII Venn diagram VII supports the Conclusion I. Venn diagrams of “Some juices are not potatoes” :

    Some potatoes are cakes. E + I ⇒ O1-type of Conclusion “Some cakes are not fruits”. (Q) 240. (2) Conclusion I and Conclusion II form Complementary Pair. Therefore, either Conclusion I or Conclusion II follows. 241. (1) Venn diagrams of “Some juices are fruits” :

    Juices

    Juices

    P o ta toes

    VIII

    II

    Venn diagrams of “All dramas are puzzles” :

    Fruits or

    Fruits

    I

    II

    BPRE–581

    Potatoe

    IX

    Potatoes

    Juices

    c es Ju i

    or

    or

    Juices Puzzles

    I

    Fruits or IV

    V Venn diagram V supports the Conclusion I. Conclusion II is the Converse of Conclusion (P). (240–243) : (i) All sketches are paintings → Universal Affirmative (A-type). (ii) Some juices are fruits → Particular Affirmative (I-type). (iii) No fruit is potato → Universal Negative (E-type). (iv) Some fruits are not potatoes → Particular Negative (O–type). (240–241) :

    Venn diagram II supports the Conclusion I.

    Games

    es

    Puzzles Pencils

    I

    Puzzles

    III Venn diagrams of “Some cakes are not fruits” :

    Games

    Books

    Drama

    Juices

    Puzzles

    A + E ⇒ E–type of Conclusion “No book is pen”. Conclusion II is the Converse of this Conclusion. Venn diagrams of “All books are pencils” :

    Books

    Fruits

    or

    No pencil is pen.

    Pencils

    or

    Dramas

    X Venn diagrams VIII, IX and X support the Conclusion II.

    SYLLOGISM 242. (4)

    244. (1) Conclusion I and Conclusion II form Complementary Pair. Therefore, either Conclusion I or Conclusion II follows. 245. (3) Conclusion II is the Converse of the Conclusion (P). (246–247) :

    Drawings

    Some paintings are drawings.

    Paintings

    All drawings are letters.

    Sketches

    I + A ⇒ I–type of Conclusion “Some paintings are letters”. Venn diagrams of “Some paintings are letters” : Paintings

    n Pai

    ting

    s

    Letters

    Letters or

    Some steps are nodes. IX Venn diagram IX supports the Conclusion II. 243. (5) All cubs are kittens.

    All nodes are tasks. I + A ⇒ I-type of Conclusion “Some steps are tasks”. (P)

    Some nodes are steps. I

    II or

    Letters Paintings

    III Venn diagrams III contradicts the Conclusion I. Venn diagrams of “All sketches are paintings” : Paintings

    Paintings

    Sketches

    or Sketches

    IV

    V

    Venn diagrams of “Some paintings are drawings” : Paintings

    n Pa i

    ti n

    gs

    D ra

    w ing

    s

    or

    VI

    Drawings

    No kitten is a zebra. A + E ⇒ E–type of Conclusion “No cub is a zebra”. Therefore, Conclusion I does not follow.

    I + A ⇒ I-type of Conclusion “Some nodes are cabins”. (Q)

    No kitten is a zebra.

    Some tasks are steps.

    All zebras are bears. E + A ⇒ O1–type of Conclusion “Some bears are not kittens”. No cub is a zebra. All zebras are bears. E + A ⇒ O1–type of Conclusion “Some bears are not cubs”. (244–249) : (i) All trucks are doors → Universal Affirmative (A–type). (ii) Some steps are nodes → Particular Affirmative (I–type). (iii) No door is a lamb → Universal Negative (E–type). (iv) Some doors are not lambs → Particular Negative (O–type). (244–245) :

    All steps are cabins.

    All steps are cabins. I + A ⇒ I-type of Conclusion “Some tasks are cabins”. (R) 246. (2) Venn diagrams of “Some steps are tasks” :

    Ta s

    Steps

    I

    or Drawings Paintings

    No door is a lamb.

    A + E ⇒ E-type of Conclusion “No truck is a lamb”. (P) Venn diagrams of all the three Premises :

    or

    Tasks Steps III Venn diagram II supports the Conclusion I. Venn diagrams of “Some nodes are cabins” :

    Nodes

    Doors

    Tasks

    II

    All trucks are doors. VII

    Steps

    ks or

    Cab ins or

    Nodes Cabins

    VIII After combining Venn diagrams IV and VIII, we get :

    Trucks

    Lamb

    Ship

    V

    IV or

    BPRE–582

    SYLLOGISM or

    After combining the venn diagrams II and V, we get :

    Cabins Nodes

    Carriers

    Palaces

    Vans

    Caves

    III

    VI

    250. (2) All jars are plates.

    VI Venn diagram VI supports the Conclusion II. 252. (1) No bungalow is a room.

    No plate is a cup. A + E ⇒ E–type of Conclusion. ‘‘ No jar is a cup.’’ This is the Conclusion I. Veen diagrams of ‘‘No cup is plate’’ and ‘‘No mug is a cup’’ :

    All festivals are carnivals.

    Plate

    Mug

    Cup

    H otels Bu

    ng

    al

    All pictures are exhibitions.

    Some rooms are hotels. E + I ⇒ O1–type of Conclusion ‘‘Some hotels are not bungalows’’. Venn diagrams of ‘‘Some hotels are not bungalows’’ : s

    Some exhibitions are festivals.

    I + A ⇒ I-type of Conclusion “Some exhibitions are carnivals”. Conclusion I is the Converse of it.

    Resorts

    The above venn diagrams contradict the Conclusion II.

    ow

    Venn diagram VI contradicts the Conclusion II. 247. (5) Neither Conclusion I nor Conclusion II follows. 248. (2)

    A + I ⇒ No Conclusion (249–253) : (i) All trollies are vans → Universal Affirmative (A-type). (ii) Some whistles are carriers → Particular Affirmative (I-type) (iii) No whistle is a van → Universal Negative (E-type). (iv) Some whistles are not vans → Particular Negative (O-type).

    Some caves are palaces.

    la

    s ce

    r Ca

    rs rie

    r Ca

    Vans

    or

    I

    s

    Vans

    Pa

    Caves

    I

    s ce la

    Caves

    II or

    Ca

    s ve

    II

    or

    Bungalows

    Hotels

    III

    The above venn diagrams contradicts the Conclusion I. Venn diagrams of ‘‘ Some hotels are rooms’’ :

    Palaces Rooms

    Hotels

    III Venn diagrams of ‘‘ All resorts are palaces’’.

    IV

    or

    Hotels

    Palaces

    Rooms

    Palaces

    Rooms

    Resorts

    or

    II

    Bungalows

    or

    Some carriers are whistles.

    r rie

    s tel o H

    E + I ⇒ O1–type of Conclusion ‘‘ Some palaces are not villages’’. This is the Conclusion I. Venn diagrams of ‘‘ Some palaces are caves’’ :

    Pa

    No whistle is a van. I + E ⇒ O–type of Conclusion ‘‘Some carriers are not vans’’. Venn diagrams of ‘‘Some carriers are not vans’’ :

    or

    251. (2) No village is a cave.

    249. (3) All trollies are vans.

    No van is a whistle. A + E ⇒ E - type of Conclusion ‘‘ No trolley is a whistle’’. Conclusion I is the Converse of it.

    I

    These venn diagrams support the Conclusion II.

    Some exhibitions are festivals.

    or

    Hotels

    Resorts

    V

    IV

    BPRE–583

    V

    VI

    SYLLOGISM Venn diagrams of ‘‘ Some keys are hotels’’ : Keys Hotels or

    Keys

    Hotels

    VII

    Venn diagram II contradicts the Conclusion I. Venn diagrams of ‘‘ All bridges are camps’’ : Camps Bridges

    or

    All books are carpets.

    or

    Camps IV

    Keys

    V

    IX After combining the venn diagrams VI and IX, we get :

    All cups are mirrors. Camps

    Tents

    or

    Tents

    VI VII After combining the Venn diagrams V and VII, we get :

    Keys

    Bridges Tents Camps

    X Venn diagram X contradicts the Conclusion II. 253. (1) Some hinges are tents.

    All tents are camps. I + A ⇒ I–type of Conclusion ‘‘ Some hinges are camps.’’ Venn diagrams of ‘‘ Some hinges are camps’’ : Hinges

    or

    Ca mps

    VIII Venn diagram VIII contradicts the Conclusion II. (254–258) : (i) All huts are rivers → Universal Affirmative (A-type). (ii) Some villages are towns → Particular Affirmative (I-type). (iii) No bucket is pin → Universal Negative (E-type). (iv) Some buckets are not pins → Particular Negative (O-type). 254. (2) Some towns are huts.

    I II

    or

    Camps Hinges

    III

    257. (5) All boxes are cups.

    A + A ⇒ A-type of Conclusion “All boxes are mirrors.” Conclusion II is Converse of it.

    All chairs are cups.

    Rooms Hotels

    Camps

    A + A ⇒ A-type of Conclusion “All flowers are carpets.” None of the Conclusions follows.

    Venn diagrams of ‘‘All tents are camps’’ : Camps

    es ng Hi

    256. (4) All flowers are books.

    Bridges

    VIII

    Hotels

    I + A ⇒ I-type of Conclusion “Some buses are trams.” Conclusion I is the Converse of it.

    All huts are rivers. I + A ⇒ I-type of Conclusion “Some towns are rivers.” Conclusion II is the Converse of it. 255. (1) Some buses are cars.

    All cars are trams.

    BPRE–584

    All cups are mirrors. A + A ⇒ A-type of Conclusion “All chairs are mirrors.” Conclusion III is the Converse of it. 258. (3) Some pins are needles.

    All needles are ropes. I + A ⇒ I-type of Conclusion “Some pins are ropes.” Conclusion II is Converse of it.

    Some ropes are buckets.

    All buckets are trees. I + A ⇒ I-type of Conclusion “Some ropes are trees.” Conclusion I and Conclusion III form Complementary Pair. Therefore, either Conclusion or Conclusion III follows. (259–263) : (i) All arrows are boxes → Universal Affirmative (A–type). (ii) Some orchids are boxes → Particular Affirmative (I–type). (iii) No car is an orchid → Universal Negative (E–type). (iv) Some cars are not orchids → Particular Negative (O–type).

    SYLLOGISM (259–260) :

    This venn diagram contradicts the Conclusion I. (261–262) :

    No car is an orchid. Some orchids are boxes. E + I ⇒ O1–type of Conclusion “Some boxes are not cars.” (P) 259. (1) Venn diagrams of “Some boxes are not cars” : Boxes Boxes

    or

    Cars

    Cars

    Venn diagram VIII supports the Conclusion I. 262. (5) We have venn diagrams :

    All blanks are gates. Gates

    Posters

    Some gates are posters. A + I ⇒ No Conclusion 261. (3) Venn diagrams of “All logics are posters” :

    Logics

    Posters Logics

    or

    and

    Gates

    Blanks I

    II

    After combining these two venn diagrams, we get :

    Posters I

    II

    Posters

    or

    Boxes

    Gates, Blanks

    I II Venn diagrams of “Some gates are posters” :

    Cars

    Gates

    III Venn diagrams of “All arrows are boxes” : Boxes

    Boxes

    Arrows

    Pos te r s or

    Gates

    III

    Posters

    III Venn diagram III contradicts the Conclusion I. We have venn diagram :

    IV

    or

    Gates, Blanks

    or

    Posters Arrows

    IV

    Posters

    Gates

    Logics

    V

    After combining the venn diagrams II and V, we get :

    Boxes, Arrows

    V Venn diagrams of “All blanks are gates” : Gates

    Gates

    Cars Blanks

    or Blanks

    VI Venn diagram VI supports the Conclusion I. Venn diagrams I, II and VI support the Conclusion II. 260. (5) We have venn diagram

    VI

    VII

    After combining the venn diagrams I, IV and VII, we get :

    Gates, Blanks Posters

    IV Venn diagram IV contradicts the Conclusion II. (263–266) : (i) All grills are arrows → Universal Affirmative (A-type). (ii) Some hats are grills → Particular Affirmative (I-type). (iii) No door is rose → Universal Negative (E-type). (iv) Some doors are not roses → Particular Negative (O-type). 263. (3) Venn diagrams of “All grills are arrows” :

    Boxes, Arrows Logics

    Arrows Grills

    Cars

    Arrows or Grills

    VIII

    BPRE–585

    I

    II

    SYLLOGISM Venn diagrams of “Some cells are arrows” :

    Some fans are doors.

    Cells

    Ar

    Cells

    ro

    ws or

    Arrows

    IV

    III or

    No door is rose.

    Fans

    Cells

    Roses or

    Fans

    V After combining venn diagrams II and V, we get :

    II or

    Fans

    Cells

    All gadgets are roads. E + A ⇒ O -type of Conclusion 1 “Some roads are not frames”.

    No frame is a gadget.

    All gadgets are sides. E + A ⇒ O -type of Conclusion 1 “Some sides are not frames”. Venn diagrams of “Some roads are not frames” :

    Roses III

    VI Venn diagram VI contradicts the Conclusion I. Venn diagrams of “Some hats are arrows” :

    Ar

    Roses

    I

    Arrows, Grills

    Hats

    No frame is a gadget.

    I + E ⇒ O-type of Conclusion “Some fans are not roses”. (Q) 265. (5) Venn diagrams of “Some fans are not roses” :

    Arrows

    A + A ⇒ A -type of Conclusion “All gadgets are sides”. Conclusion III is the Converse of it.

    The above venn diagrams support the Conclusion I. Conclusion II does not follow. 266. (1) Venn diagrams of “Some doors are not shelves” :

    Sh elv es or

    Doors

    Roads

    Frames Roads

    or

    Frames

    II

    I

    or

    Doors Shelves

    Roads

    Frames

    Arrows

    ro

    ws or

    VII

    Hats

    VIII

    I

    II

    or

    Doors

    Shelves

    or

    Hats Arrows

    IX Venn diagram VIII contradicts the Conclusion II. 264. (5) Conclusion I is the Conclusion (P). (265–266) :

    Some doors are fans.

    No fan is shelf. I + E ⇒ O-type of Conclusion “Some doors are not shelves”. (P)

    III Venn diagram II contradicts the Conclusion I. Venn diagrams of “Some sides are not frames” :

    III The above venn diagrams contradict the Conclusion I. Venn diagram II supports the Conclusion II. (267–269) : Univer(i) All roads are sides sal Affirmative (A-type). (ii) Some bamboos are clusters Particular Affirmative (I-type). (iii) No frame is gadget Universal Negative (E-type). (iv) Some frames are not gadgets Particular Negative (O-type). 267. (1) All gadgets are roads.

    All roads are sides.

    BPRE–586

    Sides Sides

    Frames or

    Frames

    IV

    V

    or

    Sides

    Frames VI

    The above venn diagrams contradict the Conclusion II. Thus, only Conclusion III follows.

    SYLLOGISM 268. (5) Venn diagrams of “Some bamboos are clusters” :

    B

    bo am

    os

    Clus

    ters

    Bamboos or

    Clusters

    Venn diagram IX supports the Conclusion I. Venn diagram IX contradicts the Conclusion II and Conclusion III.

    After combining venn diagrams II and V, we get :

    Pigeons, Sparrows

    269. (2) All plasters are clays.

    Crows No clay is silt.

    II

    I

    or

    A + E ⇒ E-type of Conclusion “No plaster is silt.” This is the Conclusion I. Venn diagrams of all the three Premises :

    Clusters Bamboos

    Clay III Venn diagrams of “Some orchids are bamboos” : Bam b

    Orchids

    oo s

    Orchids

    or

    IV

    or

    Bamboos

    Hill

    The above venn diagrams support the Conclusion II and Conclusion III. 270. (4) No duck is crow.

    271. (4) All ratio are norms.

    V

    Some crows are pigeons. E + I ⇒ O -type of Conclusion 1 “Some pigeons are not ducks.” Thus, Conclusions (1), (2) and (3) do not follow. Venn diagrams of “Some pigeons are crows” :

    Bamboos Orchids

    VI Venn diagrams of “All dangers are clusters” :

    Pigeons

    Crows or

    Pigeons

    Crows

    E + I ⇒ O1 – type of Conclusion. “Some tigers are not deer.” (P)

    I + A ⇒ I–type of Conclusion “Some tigers are horses.” (Q)

    Crows

    VII VIII After combining venn diagrams III, VI and VIII, we get

    Orchids

    Some lions are tigers.

    All lions are horses.

    or

    Clusters

    Bamboos

    No deer is a lion.

    II

    I

    or

    Clusters, Dangers

    Some norms are tables. A + I ⇒ No Conclusion (272–273) :

    Some tigers are lions.

    Dangers

    Clusters Dangers

    Silt

    Plaster

    VI Venn diagram VI supports the Conclusion (4). Venn diagram VI contradicts the Conclusion (5). (271–273) : (i) All ratios are norms Universal Affirmative (A-type). (ii) Some norms are tables Particular Affirmative (I-type). (iii) No deer is a lion Universal Negative (E-type). (iv) Some deer are not lions Particular Negative (O-type).

    No deer is a lion.

    Pigeons

    All lions are horses. III Venn diagrams of “All sparrows are pigeons” : Pigeons

    E + A ⇒ O1–type of Conclusion. “Some horses are not deer.” (R) 272. (2) Venn diagrams of “Some tigers are not deer”.

    Pigeons Tiger

    Sparrows

    or

    Tiger

    Deer

    or

    Deer

    Sparrows

    IX

    IV

    V

    BPRE–587

    I

    II

    SYLLOGISM or

    or

    Woods Tigers

    Deer

    279. (3) Conclusion I is the Conclusion (P). Venn diagrams of “Some pens are erasers” :

    Er

    Trunks III

    Pens

    Conclusion II is the Conclusion Q. 273. (1) Venn diagrams of “Some horses are not deer” :

    Deer

    or

    Deer

    II Erasers

    Trunks

    Pens III Venn diagrams of “Some erasers are lighters” :

    or

    Erasers

    III Deer III

    Venn diagram II supports the Conclusion I. (276-277) :

    No mobile is a band.

    Venn diagram II supports the Conclusion I. (274-278) : (i) All papers are words Universal Affirmative (A-type). (ii) Some woods are leaves Particular Affirmative (I-type). Uni(iii) No mobile is a band versal Negative (E-type). (iv) Some mobiles are not bands Particular Negative (Otype). (274-275) :

    Some woods are leaves.

    Erasers

    or

    II

    Horses

    or

    or

    Woods I

    er s

    I

    II

    Horses Horses

    Pens

    as

    Er

    s er as

    L ig h te rs or

    Lighters

    IV

    V or

    Lighters

    All bands are pillows. E + A → O1 – type of Conclusion ‘‘Some pillows are not mobiles.’’ (P) 276. (4) Neither Conclusion I nor Conclusion II follows. 277. (4) Neither Conclusion I nor Conclusion II follows. 278. (3)

    All ladders are snakes.

    Erasers

    VI After combining venn diagrams II and V, we get : Pens Erasers Lighters

    All leaves are trunks.

    Some snakes are frogs.

    I + A ⇒ I-type of Conclusion ‘‘Some woods are trunks’’. (P) 274. (2) Conclusion II is the Converse of the Conclusion (P). 275. (1) Venn diagrams of ‘‘Some woods are trunks’’ :

    Conclusion I and Conclusion II form Complementary Pair. Therefore, either Conclusion I or Conclusion II follows. (279–283) : (i) All markers are lighters Universal Affirmative (A-type). (ii) Some pens are erasers Particular Affirmative (I-type). (iii) No mail is a notice Universal Negative (E-type). (iv) Some mails are not notices Particular Negative (O-type).

    Woods

    Trunks I

    BPRE–588

    VII Venn diagram VII supports the Conclusion II. 280. (5) Neither Conclusion I nor Conclusion II follows. (281–282) :

    No mail is a notice.

    All notices are intimations.

    SYLLOGISM E + A ⇒ O1-type of Conclusion “Some intimations are not mails.” (P) 281. (4) Venn diagrams of “Some intimations are not mails” :

    ima In t

    t io

    Warnings

    Warnings Intimations

    Intimations

    Notices

    Mails

    ns Mails

    VI Venn diagram VI supports the Conclusion II.

    VII

    I or Intimations

    Venn diagram VII supports the Conclusions II. 282. (4) Venn diagrams of “All notices are intimations” :

    283. (4) All seconds are minutes.

    Mails

    All minutes are hours. Notices

    Intimations

    or

    Notices

    II or

    Intimations

    I

    Intimations

    Mails

    II Venn diagrams of “Some intimations are warnings” :

    A + A ⇒ A-type of Conclusion “All seconds are hours”. This is the Conclusion II. Venn diagrams of “All minutes are hours” : Minutes

    Hours Minutes

    or Hours

    III Venn diagrams of “Some intimations are warnings” :

    im In t

    a t io

    i ma In t

    t io

    ns

    Warnings

    I II Venn diagrams of “Some hours are days” :

    ns Warnings

    Hours

    III or

    IV or

    Intimations

    Intimations

    Warnings

    Warnings

    Hours

    Days or

    III

    or

    Days

    IV

    Days Hours

    IV V or

    or

    Warnings Warnings Intimations Intimations

    V After combining venn diagrams II and IV, we get : Hours, Minutes Days

    V VI After combining venn diagrams II and VI, we get :

    After combining venn diagrams I and V, we get :

    BPRE–589

    VI Venn diagram VI contradicts the Conclusion I.

    SYLLOGISM (284–288) : (i) All squares are rectangles Universal Affirmative (A-type). (ii) Some circles are squares Particular Affirmative (I-type). (iii) No fruit is plant Universal Negative (E-type). (iv) Some fruits are not plants Particular Negative (O-type).

    Blue

    Winter Winter

    Cold

    or

    Cold

    IV

    284. (2) Some circles are squares.

    Blue

    V

    Green or

    Green

    I

    II

    or

    or

    Cold

    Green

    Winter

    Blue

    All squares are rectangles. I + A ⇒ I-type of Conclusion “Some circles are rectangles”. This is the Conclusion II.

    VI After combining venn diagrams III and VI, we get : Cold Winter

    285. (3) Some friends are trust.

    Grey Grey

    Snows

    All trust are love. I + A ⇒ I-type of Conclusion “Some friends are love.” This is the Conclusion II.

    Some happy are trust.

    III Venn diagrams of “Some grey are blue” :

    Blue

    or

    Blue

    IV

    V

    or

    VII Venn diagram VII supports the Conclusion I. Venn diagrams of “All ice are winter” :

    Blue Grey

    All trust are love. I + A ⇒ I-type of Conclusion “Some happy are love”. Conclusion I is Converse of it.

    Winter Ice

    Ice

    286. (1) Some snows are ice.

    Winter VIII

    All ice are winter. I + A ⇒ I-type of Conclusion “Some snows are winter”. Venn diagrams of “Some snows are winter” : Snows Snows

    Winter or

    I

    VI Venn diagrams of “All green are white” :

    or

    IX

    After combining venn diagrams VI and VIII, we get :

    Cold Winter Ice

    White Green

    White or Green

    VII

    VIII

    Venn diagrams of “Some blue are white” :

    Winter

    II or

    Winter Snows

    III Venn diagrams of “Some winter are cold” :

    Blue X Venn diagram X contradicts the Conclusion II. 287. (2) Some blue are green.

    Blue

    White or

    X

    IX or

    All green are white. I + A ⇒ I-type of Conclusion “Some blue are white”. Venn diagrams of “Some blue are green” :

    BPRE–590

    White

    White Blue

    XI

    SYLLOGISM After combining venn diagrams VI and XI, we get :

    “Only a few pillows are ankles”. Venn diagrams of “Only a few pillows are ankles” :

    Or

    Ankles

    White Blue Grey

    Pillows

    Ankles

    Trollies

    XII I Or

    Venn diagram XII contradicts the Conclusion I. After combining venn diagrams II and V, we get :

    VI Venn diagrams of “All nests are ankles” :

    Pillows

    Ankles

    Grey

    Ankles

    Blue

    Nests

    Green

    II Or

    XIII Venn diagram XIII supports the Conclusion II.

    VII Or

    Ankles

    288. (4) All trees are plants.

    Nests

    Pillows No plant is fruit. A + E ⇒ E-type of Conclusion “No tree is fruit”.

    Some flowers are fruits.

    III Venn diagrams of “Only a few trollies are ankles” :

    No fruit is plant. I + E ⇒ O-type of Conclusion “Some flowers are not plants”. (289–293) : (i) All nests are ankles Universal Affirmative (A–type). (ii) Only a few pillows are nests Particular Affirmative (I–type). [Only a few = Some] Univer(iii) No can is a bucket sal Negative (E–type). (iv) Some cans are not buckets Particular Negative (O–type). (289–290) :

    Trollies

    VIII 289. (2) After combining venn diagrams VI and VIII, we get :

    Ankles

    Nests, Ankles IV Or

    Trollies

    Trollies

    Ankles

    Only a few pillows are nests.

    All nests are ankles. I + A ⇒ I-type of Conclusion

    Ankles

    V

    BPRE–591

    IX Venn diagram IX contradicts the Conclusion I. Venn diagram II contradicts the Conclusion II.

    SYLLOGISM 290. (3) Venn diagram III supports the Conclusion I. Venn diagram IX supports the Conclusion II. 291. (5) All arrows are buckets.

    Or

    Or

    Carpenters Supervisors

    Dentists

    Actors

    All arrows are buckets.

    Only a few buckets are offers. A + I ⇒ No Conclusion (292–293):

    Carpenter

    Dentists

    Only a few actors are dentists.

    No dentist is a carpenter. I + E ⇒ O-type of Conclusion “Only a few actors are not carpenters.” (P)

    No supervisor is an actor.

    IV After combining venn diagrams II and IV, we get :

    Some trophies are cups.

    Dentists

    No cup is a prize.

    Carpenter

    Supervisors

    Only a few actors are dentists. E + I ⇒ O1 – type of Conclusion. “Some dentists are not supervisors”. (Q) 292. (2) Venn diagrams of “Some dentists are not supervisors”:

    VIII Venn diagram VIII contradicts the Conclusion II. (294–298) : Uni(i) All coats are deposits versal Affirmative (A–type). (ii) Some wins are trophies Particular Affirmative (I–type). (iii) No cup is a prize Universal Negative (E–type). (iv) Some cups are not prizes Particular Negative (O–type). (294–295) :

    V Thus, Conclusion I does not follow. Venn diagrams of “Only a few actors are not carpenters” :

    I + E ⇒ O–type of Conclusion “Some trophies are not prizes”. (P) 294. (5) Venn diagrams of “Some wins are trophies” : Wins Tr

    s

    “No arrow is a can”. Conclusion II is the Converse of it.

    III Venn diagram II contradicts the Conclusion I. Neither Conclusion I nor Conclusion II follows. 293. (2) Venn diagram of “No dentist is a carpenter” :

    W in

    No bucket is a can. A + E ⇒ E-type of Conclusion

    op

    hie

    s or

    I

    Trophies

    II

    or

    Actors

    Dentists

    Carpenters

    Trophies

    Supervisors

    Wins

    I Or

    VI Or

    III

    Actors

    Venn diagrams of “Some trophies are cups” :

    Dentists

    Carpenters

    Trophies s

    Supervisors Tr

    op h

    ie

    Cu

    II

    VII

    BPRE–592

    IV

    ps

    or

    Cups

    V

    SYLLOGISM or

    or

    This is the Conclusion I. Venn diagrams of “Some deposits are not layers” :

    Cups

    Rooms

    Deposits La y

    ers or

    Halls

    Layers

    D

    ep

    os it s

    Trophies

    VI

    VI I

    After combining venn diagrams II and V, we get :

    Flats

    Deposits

    Trophies

    Venn diagram VII supports the Conclusion I. Venn diagrams of “Some trophies are not prizes” :

    No apartment is a hall.

    Trophies

    Rooms

    VII

    Prizes

    I + E ⇒ O–type of Conclusion “Some flats are not halls”.

    Some codes are secrets.

    No apartment is a hall.

    All secrets are puzzles.

    Fl

    at s

    Ha

    es

    Flats

    Prizes

    Codes Se c re

    od

    or

    lls

    or

    ts or

    I

    Venn diagram IX supports the Conclusion II. 295. (4) Neither Conclusion I nor Conclusion II follows.

    Secrets

    Halls

    X I

    II

    or

    II

    or Secrets Codes

    296. (5) No layer is a coat.

    Flats

    All coats are deposits.

    Halls

    III Venn diagrams of “Some halls are rooms” :

    E + A ⇒ O1–type of Conclusion “Some deposits are not layers”.

    All coats are deposits.

    A + A ⇒ A–type of Conclusion “All coats are sheets”.

    I + A ⇒ I–type of Conclusion “Some codes are puzzles”. Conclusion II is the Converse of it. Venn diagrams of “Some codes are secrets” :

    Some halls are rooms. E + I ⇒ O1 type of Conclusion “Some rooms are not apartments”. Venn diagrams of “Some flats are not halls” :

    IX

    al ls

    Ro

    H

    All deposits are sheets.

    Venn diagram VII supports the Conclusion I. 298. (5)

    C

    s ie op h Tr

    or

    VIII

    Trophies

    Halls

    Some flats are apartments.

    VII

    s

    Layers

    III These venn diagrams support the Conclusion II. 297. (1)

    Cups

    i ze

    After combining venn diagrams II and V, we get :

    or

    Wins

    Pr

    II

    Halls om

    s

    or

    IV

    BPRE–593

    Rooms

    V

    III Venn diagram II supports the Conclusion I. (299–303) : (i) All accounts are groups Universal Affirmative (A-type). (ii) Only a few colletions are donaParticular Affirmative tions (I-type). [Only a few = Some]

    SYLLOGISM (iii) No group is a collection Universal Negative (E-type). (iv) Some groups are not collections Particular Negative (Otype).

    or

    Planes

    E + I ⇒ O1-type of Conclusion “Some donations are not groups”. Venn diagrams of “Some donations are not groups” :

    III

    Groups

    II

    I + A ⇒ I-type of Conclusion “Some bottles are eggs.” This is the Conclusion I. 302. (5) Some tigers are cats.

    VIII After combining venn diagrams IV and VIII, we get : Cats

    No cat is a bear.

    Tigers

    I + E ⇒ O-type of Conclusion “Some tigers are not bears.” Venn diagrams of “Some tigers are not bears” :

    Rats

    IX

    Tigers Bears or

    Tigers

    Bears

    Groups

    303. (5) Some frogs are reptiles.

    III

    or

    All reptiles are snakes.

    Venn diagrams I and II support the Conclusion I. 300. (2) Some buses are cycles.

    I + A ⇒ I-type of Conclusion “Some frogs are snakes”.

    Tigers

    I + A ⇒ I-type of Conclusion “Some buses are planes”. Venn diagrams of “Some buses are planes” : Buses Planes

    Bears

    III

    All cycles are planes.

    Planes or

    Venn diagram IX supports the Conclusion I.

    II

    I

    Buses

    Cats

    All glasses are eggs.

    or

    Donations

    VII

    or

    Venn diagram III supports the Conclusion II.

    Donations

    I

    Cats

    Tigers

    Some collections are donations.

    Groups or

    or

    301. (1) Some bottles are glasses.

    No group is a collection.

    na Do

    Cats

    Tigers

    VI

    No group is a collection.

    ns tio

    Tigers

    Buses

    299. (5) All accounts are groups.

    A + E ⇒ E-type of Conclusion “No account is a collection.” This is the Conclusion II.

    Venn diagrams of “Only a few tigers are cats” :

    All snakes are toads.

    Venn diagrams I and II support the Conclusion II. Venn diagrams of “All rats are tigers” :

    Tigers

    All reptiles are snakes.

    A + A ⇒ A - type of Conclusion “All reptiles are toads”. This is the Conclusion I. Venn diagrams of “Some frogs are snakes” :

    Rats Frogs

    Rats

    or

    Frogs

    Snakes or

    Snakes

    Tigers I

    II

    IV

    V

    BPRE–594

    I

    II

    SYLLOGISM or

    Goats

    Snakes

    Animals

    Socials

    Tigers Animals

    Frogs

    or

    Cats

    Socials

    I

    III Venn diagram II supports the Conclusion II. (304–306) : (i) All cats are tigers Universal Affirmative (A-type). Par(ii) Some lions are cats ticular Affirmative (I-type) Univer(iii) No water is gas sal Negative (E-type) (iv) Some water are not gases Particular Negative (O-type) 304. (5) Some lions are cats.

    II

    Venn diagrams of “Some socials are philosophers” : VI

    Venn diagram VI supports the Conclusion I.

    Socials

    Philosophers

    305. (2) Some dark are spaces.

    or III

    All spaces are matters.

    Socials

    I + A ⇒ I-type of Conclusion “Some dark are matters.” This is the Conclusion II. 306. (4)

    Philosophers

    Some hydrogen are water. All cats are tigers. I + A ⇒ I-type of Conclusion “Some lions are tigers”. Venn diagrams of “All cats are tigers” :

    I + A ⇒ I-type of Conclusion “Some hydrogen are oxygen” This is the Conclusion II.

    Philosophers

    Socials

    All water are oxygen.

    Tigers

    or

    Cats or

    Cats

    All oxygen are gases. Tigers II

    I

    Venn diagrams of “Some goats are tigers” :

    Goats

    or

    Tigers

    III

    A + A ⇒ A - type of Conclusion “All water are gases.” This is the Conclusion III. (307–309) : (i) All humans are animals Universal Affirmative (A-type). (ii) Some socials are philosophers Particular Affirmative (Itype). (iii) No social is philosopher Universal Negative (E-type). (iv) Some socials are not philosophers Particular Negative (O-type). 307. (3)

    All humans are animals.

    Goats Tigers

    IV

    All water are oxygen.

    Tigers or

    V

    After combining venn diagrams I and V, we get : Philosophers Socials Animals

    VI

    Venn diagram VI supports the Conclusion III. 308. (4)

    Some citizens are politicians.

    Goats All animals are socials.

    IV

    V

    After combining the venn diagrams I and IV, we get :

    A + A ⇒ A – type of Conclusion “All humans are socials”. Venn diagrams of “All animals are socials”:

    BPRE–595

    All politicians are professors. I + A ⇒ I-type of Conclusion “Some citizens are professors”. This is the Conclusion II.

    SYLLOGISM Venn diagrams of “Some citizens are females”. Citizens Citizens

    Females

    or

    Females

    I

    II

    Females Citizens

    or

    I + A ⇒ I-type of Conclusion “Some currencies are dollars.” This is the Conclusion II. (310 – 313) : (i) All banks are shops Universal Affirmative (A-type) (ii) Only a few malls are banks Particular Affirmative (I-type) [Only a few = Some] Universal (iii) No ship is a van Negative (E-type) (iv) Some ships are not vans Particular Negative (O–type) 310. (1)

    Docks Sticks

    or VI

    After combining venn diagrams II and V, we get : Brooms Sticks

    Only a few malls are banks. III

    Docks

    Venn diagrams of “Some citizens are politicians”.

    All banks are shops. Citizens Citizens

    Politicians

    or

    Politicians

    IV

    I + A ⇒ I-type of Conclusion “Some malls are shops”. This is the Conclusion I.

    All banks are shops. V

    Politicians Citizens

    or VI

    After combining venn diagrams III and V, we get :

    Only a few shops are halls. A + I ⇒ No Conclusion Thus, only Conclusion I follows. 311. (1) Venn diagrams of “Only a few brooms are sticks” : Brooms

    Brooms

    Sticks

    or

    Sticks

    I

    Females

    VII

    Venn diagram VII supports the Conclusion I. No Negative Conclusion follows from the two Affirmative Premises. Thus, only Conclusion I follows. 312. (2)

    Only a few blocks are aims.

    All aims are goals. I + A ⇒ I-type of Conclusion “Some blocks are goals”. Venn diagrams of “Only a few steps are blocks”:

    II

    Citizens

    Steps

    Steps

    Blocks

    Sticks

    Politicians

    or

    Blocks

    I

    Brooms II

    or VII III

    Venn diagram VII supports the Conclusion I. 309. (2)

    Blocks

    Venn diagrams of “Only a few sticks are docks” :

    Steps Sticks

    Some currencies are rupees. Sticks

    Docks

    or

    Docks

    IV

    All rupees are dollars.

    V

    BPRE–596

    or III

    Venn diagrams of “Only a few blacks are aims” :

    SYLLOGISM Blocks

    Blocks

    Aims

    or

    Aims

    IV

    I + A ⇒ I-type of Conclusion “Some mats are fields”. This is the Conclusion I. Some grounds are mats.

    (iv) Some mats are not carpets Particular Negative (O–type) [Only a few = Some] 314. (5)

    Some offices are buildings. V

    Aims

    All buildings are stores. Blocks

    or VI

    After combining venn diagrams II and V, we get : Steps

    I + A ⇒ I-type of Conclusion “Some offices are stores”. This is the Conclusion I. Venn diagrams of ‘‘All companies are offices’’ : Offices

    Companies

    Aims

    Some cones are arrows.

    No arrow is a flag. I + E ⇒ O-type of Conclusion “Some cones are not flags”. Venn diagrams of “All bins are cones”:

    Companies

    or

    Blocks

    No mat is a carpet. I + E ⇒ O-type of Conclusion ‘‘Some grounds are not carpets’’. 316. (2)

    Offices II

    I

    Venn diagrams of ‘‘Some offices are buildings’’ :

    Cones

    Bins

    Bins VII

    Venn diagrams VII supports the Conclusion II. Thus, only Conclusion II follows. 313. (3)

    Only a few tapes are cots.

    Offices

    or

    Buildings

    or

    I

    II

    Venn diagrams of ‘‘Some cones are arrows’’ :

    III Offices

    Cones

    Buildings

    Cones Building

    Offices

    or

    All cots are ships. I + A ⇒ I-type of Conclusion “Some tapes are ships”.

    All cots are ships.

    Cones

    “No cot is a van. Conclusion I and Conclusion II form Complementary Pair. Therefore, either Conclusion I or Conclusion II follows. (314 – 317) : (i) All companies are offices Universal Affirmative (Atype) (ii) Only a few offices are buildings Particular Affirmative (Itype) (iii) No mat is a carpet Universal Negative (E-type)

    Arrows

    or V

    IV

    III

    After combining venn diagrams I and V, we get :

    IV

    Arrows

    Buildings Offices

    No ship is a van. A + E ⇒ E-type of Conclusion

    Arrows

    Cones or

    Companies

    V After combining venn diagrams II and IV, we get : VI Venn diagram VI supports the Conclusion II. 315. (1)

    Cones, Bins Arrows

    Some mats are grounds.

    All grounds are fields.

    BPRE–597

    VI Venn diagrams VI supports the Conclusion II.

    SYLLOGISM 317. (2)

    Some folders are envelopes.

    All envelopes are packets. I + A ⇒ I-type of Conclusion “Some folders are packets”. This is the Conclusion II. Venn diagrams of ‘‘Some folders are books’’ : Folders Folders

    Books

    Books

    (318 – 320) : Universal (i) All tigers are cats Affirmative (A-type). (ii) Some cats are kites Particular Affirmative (I-type). (iii) No bird is kite Universal Negative (E-type). (iv) Some birds are not kites Particular Negative (O-type). 318. (3) No bird is kite.

    Some kites are cats. E + I ⇒ O1–type of Conclusion “Some cats are not birds”. Venn diagrams of “Some cats are kites” :

    or

    Cats

    Cats, Tigers Birds

    IX Venn diagram IX supports the Conclusion I. Venn diagrams IV and V support the Conclusion II. Venn diagram II supports the Conclusion III. 319. (1) Venn diagrams of “Some politicians are ministers” :

    I II

    Cats

    Kites

    Kites or

    Books I

    Folders

    III Venn diagrams of ‘‘Some envelopes are folders’’:

    Mi

    Folders

    or

    Folders

    IV

    I

    Ministers

    II

    or

    Cats

    Ministers Politicians

    III Venn diagrams of “Some cats are not birds” :

    V

    III

    Cats

    Folders

    Cats

    Birds

    Birds or

    Envelopes

    or

    ter s or

    Kites

    Envelopes

    Envelopes

    nis

    II

    or

    or

    Politicians

    s an ic i lit o P

    IV

    V

    or

    Venn diagrams of “Some politicians are engineers” :

    s an ic i lit o P

    En

    Politicians

    gin

    ee rs or

    Engineers

    VI

    After combining venn diagrams III and VI, we get : Book Folders

    Envelopes

    Cat

    VII

    IV

    V

    oror

    VI Venn diagrams of “All tigers are cats” : Cats Tigers

    Venn diagrams VII contradicts the Conclusion I.

    Bird

    Engineers

    Cats

    Politicians

    Tigers

    VI

    or

    VII

    VIII

    After combining venn diagrams V and VIII, we get

    BPRE–598

    After combining venn diagrams III and V, we get

    SYLLOGISM Ministers

    Stories

    Politicians

    ies Stor

    Dram as

    or

    Js

    Us

    Dramas

    or

    Js

    Engineers

    Us VII

    VIII

    I

    or

    U & L : All Us are Ls.

    VII Venn diagram VII supports the Conclusion II. 320. (5) Some stories are movies.

    All movies are dramas. I + A ⇒ I-type of Conclusion “Some stories are dramas.” Conclusion II is the Converse of it. Venn diagrams of “Some stories are movies” : Stories

    Mov ies

    ies Stor

    or

    I

    Dramas Stories

    Movies Stories

    After combining venn diagrams, II and V, we get :

    III

    L

    IV

    F

    Stories

    V Movies

    X

    Keys

    Keys

    or

    III L @ F : No L is F.

    Keys

    Dramas

    Venn diagrams of “Some keys are stories” :

    IV

    Us

    IX

    Stories

    or

    or

    Us

    Venn diagram X supports the Conclusion I. After combining venn diagrams VI and IX, we get :

    or

    Keys

    Ls

    Ls

    All Js are Us.

    Movies

    II

    Stor ies

    II

    Stories

    V

    Stories Keys

    VI Venn diagrams of “Some stories are dramas” :

    XI Venn diagram XI supports the Conclusion III. (321–325) : (i) All Ps are Ts Universal Affirmative (A-type). Particular (ii) Some Ps are Ts Affirmative (I-type). (iii) No P is T Universal Negative (E-type). Particu(iv) Some Ps are not Ts lar Negative (O-type). 321. (2) J $ U : ALl Js being Us is a possibility.

    All Us are Ls. A + A ⇒ A-type of Conclusion “All Js are Ls”. (Possibility) This is the Conclusion II. All Us are Ls.

    No L is F. A + E ⇒ E-type of Conclusion “No U is F”. Therefore, Conclusion I does not follow. 322. (1) K $ E : All Ks being Es is a possibility. E % Y : Some Es are Ys. Y & G : All Ys are Gs. Some Es are Ys.

    All Ys are Gs. I + A ⇒ I-type of Conclusion “Some Es are Gs”. This is the Conclusion I. 323. (3) Z % B : Some Zs are Bs. Zs Zs

    Bs

    I

    BPRE–599

    or

    Bs

    II

    SYLLOGISM or

    Venn diagram X supports the Conclusion II. 324. (2) T % G : SOme Ts are Gs. G & L : All Gs are Ls. L @ F : No L is F.

    Bs Zs

    Some Ts are Gs. III B & K : All Bs are Ks.

    All Gs are Ls.

    Bs

    Ks

    All Gs are Ls.

    or

    Bs

    Ks IV

    V

    K # G : Some Ks are not Gs.

    Ks Ks

    Gs

    I + A ⇒ I-type of Conclusion “Some Ts are Ls.”

    or

    Gs

    A + E ⇒ E-type of Conclusion “No G is F.” Thus, Conclusion II follows. 325. (3) G @ M : No G is M. M % K : Some Ms are Ks. K & F : All Ks are Fs.

    VII

    or

    K

    Some Ms are Ks.

    G VIII

    Some Zs are Bs.

    All Bs are Ks. I + A ⇒ I-type of Conclusion “Some Zs are Ks”. After combining venn diagrams II, V and VII, we get :

    Zs Ks, Bs Gs

    IX

    Venn diagram IX supports the Conclusion I. After combining venn diagrams III and IV, we get :

    Ks Bs

    Some Zs are Ks.

    No K is P. I + E ⇒ O–type of Conclusion ‘‘Some Zs are not Ps’’. This is the Conclusion II. Conclusion I is the Converse of the second statement. 328. (2)

    E + I ⇒ O1-type of Conclusion “Some Ks are not Gs”. This is the Conclusion II.

    Some lions are cats.

    Some Ms are Ks.

    All cats are tigers. I + A ⇒ I–type of Conclusion ‘‘Some lions are tigers’’. This is the Conclusion II. First statement contradicts the Conclusion I. Venn diagrams of ‘‘All cats are tigers’’ :

    All Ks are Fs. I + A ⇒ I-type of Conclusion “Some Ms are Fs”. This is the Conclusion I. (326–327) : Universal Af(i) All As are Bs firmative (A–type). (ii) Some As are Bs Particular Affirmative (I–type). Universal Nega(iii) No A is B tive (E–type). Par(iv) Some As are not Bs ticular Negative (O–type). 326. (2) T $ Z : No T being Z is a possibility. Z % M : Some Zs are Ms. M # G : All Ms are Gs.

    Cats

    Tigers Cats

    or

    Tigers I II Venn diagrams of ‘‘Some goats are tigers’’ :

    No T is Z. Goats Goats

    Zs

    X

    All Ms are Gs. I + A ⇒ I–type of Conclusion ‘‘Some Zs are Gs’’. Conclusion II is the Converse of it. 327. (5) G # Z : All Gs are Zs. Z % K : Some Zs are Ks. K * P : No K is P.

    No L is F.

    No G is M. VI

    Some Zs are Ms.

    Some Zs are Ms. E + I ⇒ O1–type of Conclusion ‘‘Some Ms are not Ts.’’ (Possibility)

    BPRE–600

    III

    Tigers

    or

    Tigers

    IV

    SYLLOGISM

    All tablets are needles.

    V After combining venn diagrams I and IV, we get : Goats

    No needles is thread. A + E ⇒ E–type of Conclusion ‘‘No tablet is thread’’. Conclusion III is the Converse of the second statement.

    Tigers

    SBI PO EXAMS

    Cats

    (1–6) : (i) All rings are circles a → Universal Affirmative (A-type). (ii) Some gases are liquids → Particular Affirmative (I-type). (iii) No ellipse is a circle → Universal Negative (E-type). (iv) Some ellipses are not circlesa → Particular Negative (Otype).

    VI Venn diagram VI supports the Conclusion III. 329. (1)

    Some hydrogens are waters.

    All seconds are hours. A + A ⇒ A-type of Conclusion “All minutes are hours”. Conclusion II is Converse of it. No day is a second. All seconds are hours. E + A ⇒ O1-type of Conclusion “Some hours are not days.” All minutes are seconds.

    No second is a day. A + E ⇒ E-type of Conclusion “No minute is a day”. (5–6) :

    Teachers

    Goats

    4. (2) All minutes are seconds.

    ors

    I + A ⇒ I –type of Conclusion ‘‘Some pins are needles’’. Conclusion I is the Converse of it.

    Pro fes s

    or

    Tigers

    c Le

    tu

    s r er

    or

    1. (2) All squares are rings.

    All waters are oxygens. I + A ⇒ I –type of Conclusion ‘‘Some hydrogens are oxygens’’. This is the Conclusion II. All waters are oxygens.

    All rings are circles.

    A + A ⇒ A-type of Conclusion “All squares are circles.” Conclusion II is Converse of it.

    Professors, Teachers

    Lecturers

    or

    All rings are circles.

    All oxygens are gases. A + A ⇒ A–type of Conclusion ‘‘All waters are gases’’. This is the Conclusion I. Conclusion III is the Converse of the first statement. 330. (5)

    No circle is an ellipse.

    A + E ⇒ E-type of Conclusion “No ring is an ellipse” 2. (4) No house is an apartment.

    Some atoms are matters. Some apartm ents are bungalow s.

    All matters are molecules. I + A ⇒ I–type of Conclusion ‘‘Some atoms are molecules’’. Conclusion II is the Converse of the third statement. Conclusion III is the Converse of the first statement. 331. (4) Some pins are tablets.

    All tablets are needles.

    E + I ⇒ O1-type of Conclusion “Some bungalows are not houses” 3. (1) Some gases are liquids. All liquids are water

    I + A ⇒ I-type of Conclusion “Some gases are water”. Therefore, Conclusion I may be true. From Affirmative Premises we cannot derive Negative Conclusion.

    BPRE–601

    Pr

    s of e

    so

    rs Teachers, Lecturers

    Remember that No Conclusion follows from the two Particular Premises. 5. (1) Conclusion I may be true. 6. (2) Conclusion II may be true. (7–11) (i) All songs are poems → Universal Affirmative (A-type). (ii) Some squares are circles → Particular Affirmative (I-type). (iii) No circle is a triangle → Universal Negative (E-type). (iv) Some circles are not triangles → Particular Negative (O-type). (7-8) :

    Some squares are circles. No circle is a triangle.

    SYLLOGISM I + E ⇒ O-type of Conclusion “Some squares are not triangles”. (P)

    No line is a square.

    It implies that some parts of circles are outside the limit of lines. or, “All lines are circles”.

    Some squares are circles.

    Squares

    (A)

    Triangles

    E + I ⇒ O1-type of Conclusion “Some circles are not lines.” (Q) 7. (4) The Conclusion (P) does not imply that “All squares can never be triangles”. Some squares are not triangles.

    It implies, that some squares are triangles. Squares

    (B)

    Triangles

    (C)

    Circles

    Lines

    No circle is line. This diagram contradicts Conclusion II. 8. (4) Neither Conclusion I nor II follows. (9-10) :

    All songs are poems. All poems are rhymes. A + A ⇒ A-type of Conclusion “All songs are rhymes”. (P)

    (C)

    Squares

    Triangles

    No square is triangle. Now, take Conclusion (Q).

    No rhymes is a paragraph

    Circles

    (A)

    It implies that some circles are lines.

    Circles Lines (B)

    All songs are rhymes.

    14. (3) All hills are roads.

    All roads are stones. A + A ⇒ A-type of Conclusion “All hills are stones.” Conclusion III is Converse of this Conclusion.

    All hills are stones. All stones are jungles. A + A ⇒ A-type of Conclusion “All hills are jungles.” Conclusion II is Converse of this Conclusion.

    All stones are jungles. No rhyme is a paragraph. A + E ⇒ E-type of Conclusion “No song is a paragraph”. (R) 9. (5) Conclusion I is Conclusion (R). Conclusion II is Conclusion (Q). 10. (2) Conclusion II is Conclusion (P). 11. (5) Some dews are drops.

    All drops are stones. Lines

    All books are chairs. A + A ⇒ A-type of Conclusion “All pens are chairs.” This is Conclusion IV. Conclusion III is Converse of this Conclusion. 13. (1) All the four Premises are Particular Affirmative. No Conclusion follows from the two Particular Premises.

    All poems are rhymes.

    A + E ⇒ E-type of Conclusion “No poem is a paragraph”. (Q) It implies that some parts of squares are outside the limit of triangles. Or, “All triangles are squares”.

    12. (4) All pens are books.

    I + A ⇒ I -type of Conclusion “Some dews are stones”. This is Conclusion I. Conclusion II is Converse of the second Premise. (12–16) : (i) All pens are books → Universal Affirmative (A-type). (ii) Some chairs are desks → Particular Affirmative (Itype). (iii) No cloth is room → Universal Negative (E-type). (iv) Some cloths are not rooms → Particular Negative (Otype).

    BPRE–602

    All jungles are rivers. A + A ⇒ A-type of Conclusion. “All stones are rivers.” Conclusion I is Converse of this Conclusion 15. (2) All the four Premises are Particular Affirmative (I-type). No Conclusion follows from the two Particular Premises. Conclusion III is the Converse of the third Premise. 16. (5) Some petals are flowers.

    All flowers are desks. I + A ⇒ I-type of Conclusion “Some petals are desks.” Conclusion II is the Converse of this Conclusion. Some desks are cards. All cards are trains. I + A ⇒ I-type of Conclusion “Some desks are trains”. This is Conclusion IV. Conclusion I is the Converse of the second Premise.

    SYLLOGISM

    No field is a yield. A + E ⇒ E-type of Conclusion “No crop is a yield”.(P)

    No yield is a field. All fields are harvests. E + A ⇒ O1-type of Conclusion “Some harvests are not yields.” (Q) All crops are fields. All fields are harvests. A + A ⇒ A-type of Conclusion “All crops are harvests.” (R) 17. (2) Conclusion I does not follow. Conclusion (R) supports Conclusion II. 18. (4) Neither Conclusion I nor Conclusion II follows. Conslusion (Q) : Some harvests are not yields. Thus, Conclusion (Q) does not support Conclusion I. 19. (4) All business are trades.

    Some trades are exports. A + I ⇒ No Conclusion (20–21) :

    All countries are districts.

    All districts are villages. I + A ⇒ I-Type of Conclusion “Some towns are districts”. (P)

    All countries are districts.

    All districts are villages. A + A ⇒ A-type of Conclusion

    Some towns are districts.

    All districts are villages. I + A ⇒ I-type of Conclusion “Some towns are villages”. (R) 20. (5) Conclusion (R) is the Conclusion I. Conslusion (Q) is the Conclusion II. 21. (1) Conclusion I is the Converse of Conclusion (P). (22-26) : (i) All kites are stars → Universal Affirmative (A – type). (ii) Some cones are triangles → Particular Affirmative (I-type) (iii) No star is a cone → Universal Negative (E-type) (iv) Some stars are not cones → Particular Negative (O-type) 22. (4) All kites are stars.

    No star is a cone. A + E ⇒ E-type of Conclusion. “No kite is a cone.” No kite is a cone. Some cones are triangles.

    “No drum is a flute.”

    Some guitars are drums. All drums are banjos. I + A ⇒ I-type of Conclusion “Some guitars are banjos.”

    Some guitars are banjos. No banjo is a flute. I + E ⇒ O-type of Conclusion. “Some guitars are not flutes.” Conclusions I and II form Complementary Pair. So, either Conclusion I or Conclusion II follows. 24. (1) Some pins are needles.

    All needles are swords. I + A ⇒ I-type of Conclusion. “Some pins are swords.” All needles are swords. Some swords are knives. A + I ⇒ No Conclusion Venn Diagrams of Premise : Some pins are swords.

    E + I ⇒ O1-type of Conclusion “Some triangles are not kites.”

    No star is a cone.

    Pins Pins

    Sw

    Some cones are triangles. E + I ⇒ O1-type of Conclusion “Some triangles are not stars.” Some (not all) triangles are not stars. Therefore, there is some possibility that some stars are triangles. “Some triangles are not stars” → Its Venn diagram would be :

    s

    All crops are fields.

    “Some countries are villages”. (Q)

    Tr ian gle

    (17-21) : (i) All crops are fields → Universal Affirmative (A–type). (ii) Some trades are exports → Particular Affirmative (I–type). (iii) No yield is a field → Universal Negative (E–type). (iv) Some yields are not fields → Particular Negative (O–type). (17–18) :

    Stars

    Triangles

    Stars

    It implies that some triangles are stars and also all stars triangles. 23. (4) All drums are banjos.

    No banjo is a flute. A + E ⇒ E-type of Conclusion

    BPRE–603

    or

    ds or

    Swords

    Swords

    or

    Pins

    Therefore, all swords being pins is a possibility. Thus, only Conclusion I follows. 25. (4)

    Some offers are discounts. No discount is a loan. I + E ⇒ O-type of Conclusion “Some offers are not loans.” Neither Conclusion I nor Conclusion II follows.

    SYLLOGISM 26. (5) All lodges are hotels.

    30. (3) Some pins are nails.

    No hotel is car. A + E ⇒ E-type of Conclusion “No lodge is a car” This is Conclusion II. No lodge is house : Its Conversion would be “No house is lodge.” If no house is lodge, then some houses are also not lodges. Therefore, Conclusion I also follows. (27–31) : (i) All papers are wood → Universal Affirmative (A-type). (ii) Some wood are metals → Particular Affirmative (I-type). (iii) No root is a tree → Universal Negative (E-type). (iv) Some roots are not trees → Particular Negative (O-type). 27. (4) Some wood are metals.

    All nails are hammers. I + A ⇒ I-type of Conclusion “Some pins are hammers.”

    All nails are hammers.

    I + A ⇒ I-type of Conclusion “Some wood are glasses.” Conclusion I is Converse of this Conclusion. Conclusion II is Converse of the third Premise. 28. (1) All the three Premises are Particular Affirmative. No Conclusion follows from the two Particular Premises.

    All hammers are needles. A + A ⇒ A-type of Conclusion “All nails are needles.” This is Conclusion II.

    Some pins are hammers.

    A + A ⇒ A-type of Conclusion “All days are mornings.” This is Conclusion I.

    All evenings are nights.

    All nights are mornings. A + A ⇒ A-type of Conclusion “All evenings are mornings.” Conclusion II is Converse of it.

    All limits are constraints.

    A + E ⇒ E-type of Conclusion ¶No limit is region.¸

    All territories are constraints. No constraint is region. A + E ⇒ E-type of Conclusion

    All hammers are needles. I + A ⇒ I-type of Conclusion “Some pins are needles.” 31. (5) All leaves are roots.

    A + E ⇒ E-type of Conclusion “No leaf is a tree.” This is Conclusion I.

    No root is a tree.

    All trees are bushes. A + E ⇒ O1-type of Conclusion “Some bushes are not roots.”

    No leaf is a tree.

    ¶No territory is region¸ If ‘‘No territory is region’’, then ‘‘Some territories would also be not regions’’. So, Conclusion I also follows. 33. (1)

    Some clouds are vapours.

    All vapours are droplets. I + A ⇒ I-type of Conclusion “Some clouds are droplets.” This is Conclusion I. 34. (2)

    All mistakes are flaws . No flaw is solution. A + E ⇒ E-type of Conclusion

    ¶No mistake is solution’’.

    29. (2) All days are nights.

    All nights are mornings.

    All limits are constraints. A + A ⇒ A-type of Conclusion ‘‘All territories are constraints’’. Its Converse is Conclusion II.

    No constraint is region.

    No root is a tree. All metals are glasses.

    32. (5) All territories are limits.

    All trees are bushes. E + A ⇒ O1-type of Conclusion “Some bushes are not leaves.” This is Conclusion II. (32–35) : (i) All territories are limits → Universal Affirmative (A-type). (ii) Some clouds are vapours → Particular Affirmative (I-type). (iii) No region is constraint → Universal Negative (E-type) (iv) Some regions are not constraints → Particular Negative (O-type)

    BPRE–604

    All corrections are solutions.

    No solution is flaw. A + E ⇒ E-type of Conclusion

    ¶No correction is flaw¸ This is Conclusion II. 35. (5) All zeroes are numbers.

    No number is alphabet. A + E ⇒ E-type of Conclusion

    ¶No zero is alphabet.”

    SYLLOGISM All digits are alphabets.

    39. (5) Conclusion I is the Conclusion (P).

    (B) Some worms are moths. Moths

    No alphabet is number.

    Worms

    Moths OR

    A + E ⇒E-type of Conclusion

    Worms

    No planet is a moon.

    ¶No digit is number.¸ Its converse is Conclusion I.

    All zeroes are numbers.

    No number is digit.

    III

    IV

    or Worms

    A + E ⇒ E-type of Conclusion

    ¶No zero is digit.¸

    Moths

    It is Conclusion II.

    All digits are alphabets.

    40. (1) Some stars are planets.

    V Combine diagrams II and V :

    I + E ⇒ O–type of Conclusion “Some stars are not moons.” Conclusions I and II form Complementary Pair. Therefore, either Conclusion I or II follows. 41. (2) Some clocks are towers.

    All towers are poles. I + A ⇒ I–type of Conclusion “Some clocks are poles.” Conclusion I is Converse of it. Venn diagrams (A) All watches are clocks.

    No alphabet is zero . A + E ⇒ E- type of conclusion

    ¶No digit is zero.¸ (36–41) : (i) All bugs are worms → Universal Affirmative (A–type). (ii) Some stars are planets → Par ticular Affirmative (I–type). (iii) No planet is a moon → Universal Negative (E–type). (iv) Some planets are not moons → Particular Negative (O–type). (36–37) : All bugs are worms.

    Worms Bugs Moths

    VI This diagram supports Conclusion II. 37. (3) Neither Conclusion I nor Conclusion II follows. (38–39) :

    All hoaxes are charms.

    Some worms are moths. A + I ⇒ No Conclusion

    Some worms are moths.

    All charms are tricks. A + A ⇒ A – type of Conclusion “All hoaxes are tricks.” (P) All charms are tricks.

    Clocks Watches

    I

    or Clocks Watches

    II (B) Some clocks are towers.

    Clocks Towers

    No moth is a fly. I + E ⇒ O–type of Conclusion “Some worms are not flies.” (P) 36. (4) Venn diagrams of the first two Premises : (A) All bugs are worms.

    Worms Bugs

    Worms OR

    Bugs

    I

    II

    No trick is a magic. A + E ⇒ E – type of Conclusion “No charm is a magic.” (Q)

    III

    or

    All hoaxes are tricks. Clocks No trick is a magic. A + E ⇒ E – type of Conclusion “No hoax is a magic.” (R) 38. (2) Conclusion I is the Converse of Conclusion (R). Conclusion II is the Conclusion (Q).

    BPRE–605

    Towers

    IV

    SYLLOGISM (B) Some costs are amounts.

    or

    45. (5) All grades are scales.

    Am

    costs

    Am

    oun

    Towers

    ou nt

    ts

    s

    costs

    Clocks IV

    V

    Some metals are papers.

    Amounts

    V Combine diagrams II and IV

    costs

    All papers are alloys.

    Clocks VI (C) All amounts are expenses.

    Towers

    Expenses

    Expenses

    Watches VI This diagram supports Conclusion II. (42–47) : (i) All amounts are expenses → Universal Affirmative (A-type). (ii) Some prices are costs → Particular Affirmative (I-type). (iii) No alloy is wood → Universal Negative (E-type). (iv) Some alloys are not wood → Particular Negative (O-type). (42–43) :

    nt ou Am

    s

    Amounts

    I + A ⇒ I - type of Conclusion ‘‘Some costs are expenses.’’ (P) 42. (4) Venn Diagrams (A) Some prices are costs. Prices Prices

    Costs

    Costs

    I

    II

    VII VIII Combine diagrams II and V. Prices costs

    No alloy is a wood.

    A + E ⇒ E - type of Conclusion. ‘‘No paper is a wood.’’ (Q)

    No paper is a wood.

    I + E ⇒ O - type of Conclusion. ‘‘Some metals are not woods.’’ (R) Some metals are alloys.

    No alloy is a wood.

    This diagram supports Conclusion II. 43. (5) Conclusion I is the Converse of the Conclusion (P). 44. (5) Some celebrations are invitations.

    All invitations are rejections.

    I + A ⇒ I - type of Conclusion ‘‘Some celebrations are rejections.’’ This is Conclusion I. All invitations are rejections.

    III

    All papers are alloys.

    Amounts

    Costs Prices

    I + A ⇒ A - type of Conclusion. ‘‘Some metals are alloys.’’ (P)

    Some metals are papers.

    Some costs are amounts.

    All amounts are expenses.

    All scales are categories.

    A + A ⇒ A - type of Conclusion ‘‘All grades are categories’’ This is Conclusion I. (46–47) :

    I + E ⇒ O - type of Conclusion ‘‘Some metals are not woods.’’ (S) 46. (3) Neither Conclusion I nor II follows. 47. (2) Conclusion I is Conclusion (Q). Conclusion II is Conclusion (P). (48–51) : (i) All territories are limits → Universal Affirmative (A-type). (ii) Some clouds are vapours → Particular Affirmative (I-type). (iii) No region is constraint → Universal Negative (E-type) (iv) Some regions are not constraints → Particular Negative (O-type) 48. (5) All territories are limits.

    No rejection is an attraction.

    A + E ⇒ E - type of Conclusion ‘‘No invitation is an attraction.’’

    BPRE–606

    All limits are constraints. A + A ⇒ A-type of Conclusion ‘‘All territories are constraints’’.

    SYLLOGISM Its Converse is Conclusion II.

    ¶No digit is number.¸

    All limits are constraints.

    Its converse is Conclusion I.

    All zeroes are numbers. No constraint is region. A + E ⇒ E-type of Conclusion

    ¶No limit is region.¸ All territories are constraints.

    A + E ⇒ E-type of Conclusion ¶No zero is digit.¸

    All digits are alphabets.

    If ‘‘No territory is region’’, then ‘‘Some territories would also be not regions’’. So, Conclusion I also follows. 49. (1)

    Some clouds are vapours.

    All vapours are droplets. I + A ⇒ I-type of Conclusion “Some clouds are droplets.” This is Conclusion I. 50. (2)

    No alphabet is zero . A + E ⇒ E- type of conclusion ¶No digit is zero.¸ (52–57) : (i) All bugs are worms → Universal Affirmative (A–type). (ii) Some stars are planets → Particular Affirmative (I–type). (iii) No planet is a moon → Universal Negative (E–type). (iv) Some planets are not moons → Particular Negative (O–type). (52–53) :

    All corrections are solutions.

    No solution is flaw. A + E ⇒ E-type of Conclusion

    ¶No correction is flaw¸ This is Conclusion II. 51. (5)

    All zeroes are numbers.

    Some worms are moths. A + I ⇒ No Conclusion

    I + E ⇒ O–type of Conclusion “Some worms are not flies.” (P) 52. (4) Venn diagrams of the first two Premises : (A) All bugs are worms.

    Worms Bugs

    Worms OR

    I II (B) Some worms are moths. Moths

    All digits are alphabets. Worms

    Moths OR

    Worms

    No alphabet is number. A + E ⇒E-type of Conclusion

    All hoaxes are charms.

    All charms are tricks. A + A ⇒ A – type of Conclusion “All hoaxes are tricks.” (P) All charms are tricks.

    No trick is a magic. No moth is a fly.

    No number is alphabet.

    ¶No zero is alphabet.”

    VI This diagram supports Conclusion II. 53. (3) Neither Conclusion I nor Conclusion II follows. (54–55) :

    Some worms are moths.

    Bugs A + E ⇒ E-type of Conclusion

    Moths

    All bugs are worms.

    All mistakes are flaws . No flaw is solution. A + E ⇒ E-type of Conclusion ¶No mistake is solution’’.

    V Combine diagrams II and V :

    Worms Bugs

    A + E ⇒ E-type of Conclusion

    ¶No territory is region¸

    Worms Moths

    No number is digit.

    It is Conclusion II.

    No constraint is region.

    or

    III

    IV

    BPRE–607

    A + E ⇒ E – type of Conclusion “No charm is a magic.” (Q)

    All hoaxes are tricks. No trick is a magic. A + E ⇒ E – type of Conclusion “No hoax is a magic.” (R) 54. (2) Conclusion I is the Converse of Conclusion (R). Conclusion II is the Conclusion (Q). 55. (5) Conclusion I is the Conclusion (P). 56. (1) Some stars are planets.

    No planet is a moon. I + E ⇒ O–type of Conclusion “Some stars are not moons.” Conclusions I and II form Complementary Pair. Therefore, either Conclusion I or II follows.

    SYLLOGISM (C) All amounts are expenses.

    57. (2) Some clocks are towers.

    Clocks I + A ⇒ I–type of Conclusion “Some clocks are poles.” Conclusion I is Converse of it. Venn diagrams (A) All watches are clocks.

    Clocks Watches

    I

    or Clocks

    Towers

    o Am

    un

    ts

    Amounts

    Watches VI This diagram supports Conclusion II. (58 – 59) : (i) All amounts are expenses → Universal Affirmative (A-type). (ii) Some prices are costs → Particular Affirmative (I-type). (iii) No alloy is wood → Universal Negative (E-type). (iv) Some alloys are not wood → Particular Negative (O-type). (58 – 59) : Some costs are amounts.

    Watches

    II (B) Some clocks are towers.

    Expenses

    Expenses

    All towers are poles.

    All amounts are expenses. I + A ⇒ I - type of Conclusion ‘‘Some costs are expenses.’’ (P) 58. (4) Venn Diagrams (A) Some prices are costs.

    Clocks

    VII VIII Combine diagrams II and V. Prices costs Amounts

    This diagram supports Conclusion II. 59. (5) Conclusion I is the Converse of the Conclusion (P). 60. (5) Some celebrations are invitations.

    All invitations are rejections.

    I + A ⇒ I - type of Conclusion

    Towers

    Prices Prices

    Costs

    Costs

    ‘‘Some celebrations are rejections.’’ This is Conclusion I. All invitations are rejections.

    III I

    or

    II No rejection is an attraction.

    Costs

    Clocks

    A + E ⇒ E - type of Conclusion Prices

    ‘‘No invitation is an attraction.’’ 61. (5)

    Towers

    All grades are scales.

    III (B) Some costs are amounts. IV

    costs ou nt Am

    oun Am

    costs

    Towers Clocks

    s

    ts

    or

    IV

    V

    All scales are categories.

    A + A ⇒ A - type of Conclusion ‘‘All grades are categories’’ This is Conclusion I. (62– 66) : Some metals are papers.

    Amounts costs

    V Combine diagrams II and IV

    All papers are alloys.

    I + A ⇒ A - type of Conclusion. VI

    BPRE–608

    ‘‘Some metals are alloys.’’ (P)

    SYLLOGISM All papers are alloys.

    Journals

    65. (1) Conclusion I is the Converse of Conclusion (P). 66. (2) All turns are loops.

    No alloy is a wood.

    Periodicals

    A + E ⇒ E - type of Conclusion.

    No loop is a bend.

    ‘‘No paper is a wood.’’ (Q) Some metals are papers.

    II or

    No paper is a wood.

    A + E ⇒ E – type of Conclusion “No turn is a bend.” Conclusion II is the Converse of it.

    No loop is a bend.

    I + E ⇒ O - type of Conclusion.

    Periodicals

    ‘‘Some metals are not woods.’’ (R)

    Some bends are curves.

    Some metals are alloys.

    Journals No alloy is a wood.

    E + I ⇒ O1 – type of Conclusion “Some curves are not loops.” Its venn diagrams

    I + E ⇒ O - type of Conclusion

    III

    Curves

    Loops

    r na

    ls

    Diagram II contradicts Conclusion I.

    J ou

    ‘‘Some metals are not woods.’’ (S) 62. (3) Neither Conclusion I nor II follows. 63. (2) Conclusion I is Conclusion (Q). Conclusion II is Conclusion (P). (64–65) : (i) All journals are periodicals → Universal Affirmative (A–type). (ii) Some journals are periodicals → Particular Affirmative (I–type). (iii) No journal is a periodical → Universal Negative (E–type). (iv) Some journals are not periodicals → Particular Negative (O–type). (64–65) :

    I

    Magazines

    or

    Curves IV

    Loops

    or

    Journals

    Some journals are periodicals.

    II or

    All periodicals are bulleteins. I + A ⇒ I – type of Conclusion “Some journals are bulleteins.” (P) 64. (2) Venn diagrams of “Some journals are periodicals.”

    Magazines Curves

    Loops

    V Combine diagrams II and V : Journals, Magazines

    III Diagram III contradicts the Conclusion I. 67. (4) No country is a village.

    Journals

    Periodicals

    Periodicals

    All villages are districts. I

    or

    This diagram supports Conclusion II.

    BPRE–609

    E + A ⇒ O1– type of Conclusion “Some districts are not countries.”

    SYLLOGISM 68. (5) All progresses are growth.

    No growth is an evolution. A + E ⇒ E – type of Conclusion “No progress is an evolution.” Conclusion II is the Converse of it. Venn diagrams of “All progresses are growth” :

    This diagram suggests that all developments being progress is a possibility. (69 – 70) : (i) All coats are deposits → Universal Affirmative (A - type). (ii) Some wins are trophies → Particular Affirmative (I - type). (iii) No cup is a prize → Universal Negative (E - type).

    G ro

    w th

    (iv) Some cups are not prizes → Particular Negative (O - type). Progresses

    (69 – 70) :

    Some trophies are cups.

    I or Progress

    No cup is a prize. I + E ⇒ O - type of Conclusion “Some trophies are not prizes”. (P) 69. (2) Venn diagrams of Conclusion (P) :

    II “Venn diagrams of “All developments are growth” :

    Gr

    ow

    th

    I or

    III or

    No layer is a coat. All coats are deposits.

    E + A ⇒ O1 - type of Conclusion “Some deposits are not layers” ‘All deposits can never be layers” implies the Conclusion derived above. Therefore, Conclusion II follows.

    All coats are deposits. All deposits are sheets. A + A ⇒ A - type of Conclusion “All coats are sheets”. This is Conclusion I. 72. (5)

    Some flats are apartments. No apartment is a hall. I + E ⇒ O - type of Conclusion “Some flats are not halls.”

    No apartment is a hall. Some halls are rooms. E + I ⇒ O1 - type of Conclusion “Some rooms are not apartments”. Neither Conclusion I nor Conclusion II follows. 73. (4) Some codes are secrets.

    Growth

    Developments

    71. (4)

    Trophies Prizes

    All secrets are puzzles. I + A ⇒ I - type of Conclusion “Some codes are puzzles”. Conclusion II is Converse of it. Venn diagrams of “Some codes are secrets”.

    Growth

    Developments

    II or I or

    IV Combine diagrams II and IV :

    Codes

    Growth Progress Developments

    III Venn diagram II supports Conclusion II. 70. (5) Neither Conclusion I nor Conclusion II follows.

    BPRE–610

    Secrets

    II

    SYLLOGISM or

    Glasses

    Sands

    Secrets

    Particles

    Particles Codes

    Glasses

    III Venn diagram II supports Conclusion I. Therefore, both the Conclusions I and II follow. (74– 78) : (i) All workshops are industries → Universal Affirmative (A - type). (ii) Some plants are workshops → Particular Affirmative (I - type). (iii) No factory is an industry → Universal Negative (E - type). (iv) Some factories are not industries → Particular Negative (O - type). 74. (4)

    All workshops are industries.

    II It implies that : All particles are glasses.

    Particles Glasses

    III It implies that : All glasses are particles. Venn diagrams of “Some sands are particles”:

    Particles

    A + E ⇒ E - type of Conclusion “No workshop is a factory”. This is Conclusion I.

    Particles Sands

    Sands Particles

    I It implies that (i) Some particles are glasses. (ii) Some particles are not glasses. Therefore, Conclusion I does not follow.

    No film is a show. All shows are pictures. E + A ⇒ O1 - type of Conclusion “Some pictures are not films”. Thus, neither Conclusion I nor II follows.

    Some actors are singers. IV or

    V or

    Glasses

    I + E ⇒ O - type of Conclusion ‘‘Some movies are not shows.’’

    (77 – 78) :

    Some plants are workshops.

    I + A ⇒ I-type of Conclusion “Some plants are industries”. This is Conclusion II. Thus, both Conclusions I and II follow. 75. (3) Venn diagrams of “Some particles are glasses”:

    76. (2) Some movies are films.

    No film is a show.

    No industry is a factory.

    All workshops are industries.

    Therefore, some glasses being sands is a possibility. Thus, Conclusion II follows.

    VI After combining the diagrams III and VI, we get

    BPRE–611

    All singers are dancers. I + A ⇒ I - type of Conclusion “Some actors are dancers”. (P). 77. (1) Conclusion (P) is the Conclusion I. The Converse of “All singers are dancers” would be : “Some dancers are singers”. Therefore, Conclusion II does not follow. 78. (1) Conclusion I is the Converse of the Conclusion (P). (79–80) : (i) All friends are relatives → Universal Affirmative (A–type). (ii) Some metres are weights → Particular Affirmative (I–type). (iii) No metre is distance → Universal Negative (E–type). (iv) Some metres are not distances → Particular Negative (O–type).

    SYLLOGISM

    or

    Distances

    I

    Venn diagrams of “Some computers are tablets” :

    I + E ⇒ O–type of Conclusion. “Some tenders are not circulars.”

    All orders are circulars.

    All orders are circulars.

    A + E ⇒ E–type of Conclusion. “No order is a memorandum.” This is Conclusion (5).

    II

    or

    IV

    Tablets

    V or

    No circular is a memorandum.

    or

    Ta bl

    A + A ⇒ A–type of Conclusion. “All orders are recommendations.”

    e ts

    t er

    s

    Computers

    All circulars are recommendations.

    Di

    st

    an ce s

    We i gh ts

    Weights

    III

    No memorandum is a circular.

    All kilograms are distances. No distance is metre. A + E ⇒ E–type of Conclusion “No kilogram is metre.” This is Conclusion I. Venn diagrams of “Some weights are not distances” :

    Computers

    Pencils

    Some tenders are memoranda.

    pu

    No metre is distance. I + E ⇒ O–type of Conclusion “Some weights are not distances.”

    m

    79. (1)

    81. (2) Conclusion II is the Conclusion (P). 82. (5)

    Co

    Some weights are metres.

    Tablets

    Computers

    83. (1) Some pencils are pens.

    Weights

    Distances

    No pen is a computer. I + E ⇒ O–type of Conclusion. “Some pencils are not computers.”

    III The above diagrams shows that Conclusion II is invalid. (80–81) :

    All friends are relatives. All relatives are knowns. A + A ⇒ A–type of Conclusion “All friends are knowns”. (P)

    VI After combining venn diagrams II and V, we get : Pencils

    No pen is a computer.

    Computers

    Some computers are mobiles. E + I ⇒ O1– type of Conclusion “Some mobiles are not pens”.

    Tablets

    Some computers are mobiles.

    All relatives are knowns.

    VII All mobiles are tablets. I + A ⇒ I–type of Conclusion “Some computers are tablets.” Ven diagrams of “Some pencils are not computers” :

    No known is a guest. A + E ⇒ E–type of Conclusion “No relative is a guest.” (Q)

    All friends are knowns.

    s pu

    t er

    nc il

    Computers

    m

    or

    Co

    A + E ⇒ E–type of Conclusion “No friend is a guest.” (R) 80. (3) Conclusion I is the Conclusion (R). Conclusion II is the Conclusion (Q).

    Pe

    No known is a guest.

    s

    Pencils

    I

    II

    or

    BPRE–612

    Venn diagram VII supports the Conclusion (1). (84-87) : Uni(i) All watches are rings versal Affirmative (A-type). (ii) Only a few rings are carrots Particular Affirmative (I-type) [Only a few = Some] Uni(iii) No register is a book versal Negative (E-type). (iv) Some registers are not books Particular Negative (Otype).

    SYLLOGISM Venn diagrams of “Some registers are not tests”.

    or

    Ju

    Tests

    To as

    Watches

    s

    Watches

    R eg is te r

    Rings

    te rs

    ic e

    s

    84. (4) Venn diagrams of “All watches are rings”.

    or

    IV

    Rings

    I

    II

    I

    Juices

    or

    Venn diagrams of “Some rings are carrots” :

    Toasters

    or

    Registers Rings

    V

    ar ro

    ts

    Tests

    C

    Toasters

    III

    Juices

    II

    or or Rings

    Ca rr ots Rings

    Carrots

    V

    Registers

    After combining venn diagrams III and VI, we get :

    Tests

    or

    III IV

    V

    After combining venn diagrams I and V, we get :

    Carrots Rings

    Watches

    Venn diagram II supports the Conclusion I.

    Toasters Juices

    86. (5) Some juices are toasters.

    Breads

    All toasters are pans. I + A ⇒ I-type of Conclusion “Some juices are pans”. This is the Conclusion II. Venn diagrams of “Only a few (Some) breads are juices.”

    VII Venn diagram VII supports the Conclusion I.

    Venn diagram VI contradicts the Conclusion II. Thus, neither Conclusion I nor Conclusion II follows.

    All loops are circles. I + A ⇒ I-type of Conclusion

    Ju ic es

    VI

    Br ea ds

    87. (5) Some wires are loops.

    “Some wires are circles.” This is the Conclusion I.

    I

    All loops are circles. or

    85. (1) No test is a paper. Some papers are registers.

    Breads

    Juices

    E + I ⇒ O1 – type of Conclusion “Some registers are not tests.”

    Juices

    Breads

    No register is a book. I + E ⇒ O-type of Conclusion. “Some papers are not books”.

    Some wires are circles.

    or

    Some papers are registers. II

    III

    Venn diagrams of “Only a few (Some) juices are toasters” :

    BPRE–613

    No circle is a rectangle. A + E ⇒ E–type of Conclusion “No loop is a rectangle.”

    No circle is a rectangle. I + E ⇒ O–type of Conclusion “Some wires are not rectangles.”

    SYLLOGISM Venn diagrams of “Some wires are not rectangles” :

    Signs

    Readers

    Re ct an gl es

    s W ir e

    I or

    No potato is a radish.

    or

    Readers

    Venn diagram II contradicts the Conclusion II.

    Signs

    V VI After combining the venn diagrams II and IV, we get :

    Some radishes are basils. E + I ⇒ O1-type of Conclusion “Some basils are not potatoes”. Venn diagrams of “Some basils are not potatoes” :

    Marks

    Wires Rectangles

    Basils

    s sil Ba

    Signs

    III Venn diagram II supports the Conclusion II. (88–91) : (i) All marks are signs Universal Affirmative (A-type). (ii) Only a few basils are radishes Particular Affirmative (Itype). [Only a few = Some] Uni(iii) No potato is a radish versal Negative (E-type). (iv) Some potatoes are not radishes Particular Negative (O-type). 88. (2) Venn diagrams of “All marks are signs”: Marks or Signs I II Venn diagrams of “All potatoes are signs” : Signs

    Potatoes

    Potatoes VI

    The above venn diagrams support the Conclusion I. 90. (3) Some toasts are fruits.

    Signs

    No fruit is a cottage.

    Readers VIII Venn diagram VIII supports the Conclusion II. 89. (4)

    Some coconuts are potatoes.

    No potato is a radish. I + E ⇒ O-type of Conclusion “Some coconutes are not radishes.” Venn diagrams of “Some coconuts are not radishes” : con Co

    Coconuts

    uts

    Radishes or

    Radishes

    I + E ⇒ O-type of Conclusion “Some toasts are not cottages”. Venn diagrams of “Some toasts are not cottages” : Toasts Co tta g

    ts as To

    es or

    Cottages

    I

    II or

    Toasts

    Cottages III

    The above venn diagrams contradict the Conclusion I.

    All plums are cottages. I

    II

    or

    or Signs

    III

    Basils

    Rectangles Marks

    Potatoes

    V or

    Venn diagram VII supports the Conclusion I. After combining the venn diagrams II and VI, we get :

    or

    Signs

    Potatoes

    VII

    II

    Marks

    es or

    IV

    Potatoes

    Wires

    Po tat o

    IV Venn diagrams of “All readers are signs” :

    No cottage is a fruit.

    Coconuts

    Radishes III

    BPRE–614

    A + E ⇒ E-type of Conclusion “No plum is a fruit.” This Conclusion contradicts the Conclusion II.

    SYLLOGISM 91. (5)

    (3–4) :

    No test is a luxury.

    Posters

    Some luxuries are meetings. E + I ⇒ O1-type of Conclusion “Some meetings are not tests”. Venn diagrams of “Some meetings are not tests” :

    Luxuries

    Meetings

    All squares are triangles.

    No triangle is circle. A + E ⇒ E-type of Conclusion “No square is circle.” (P)

    No triangle is circle.

    IX

    M

    Meetings

    gs tin ee

    Tests

    Tests

    or

    I

    II or

    Meetings

    Tests

    (ii) Some colours are paints → Particular Affirmative (I-type).

    Venn diagrams II supports the Conclusion II. Venn diagrams of “Some meetings are luxuries” : Meetings

    gs

    Lu

    xu r

    RBI GRADE–B/ NABARD GRADE–A OFFICER EXAMS (1–6) : (i) All colours are varnishes → Universal Affirmative (A-type).

    III

    n eti Me

    All circles are rectangles. Venn diagram IX contradicts the Conclusion I.

    (iii) No triangle is circle → Universal Negative (E-type). (iv) Some triangles are not circles → Particular Negative (O-type). (1– 2) :

    Some paints are colours. Luxuries

    ies or

    All colours are varnishes. IV

    V or

    I + A ⇒ I-type of Conclusion “Some paints are varnishes.”(P)

    All colours are varnishes. Luxuries No varnish is dye.

    Meetings

    A + E ⇒ E-type of Conclusion “No colour is dye.” (Q)

    VI

    Some paints are varnishes.

    Venn diagrams of “All posters are meetings” :

    Meetings Posters

    Posters or Meetings

    VII

    VIII

    After combining venn diagrams V and VIII, we get :

    No varnish is dye. I + E ⇒ O-type of Conclusion “Some paints are not dye.” (R) 1. (2) Some paints are varnishes. Therefore, Conclusion II is possible. 2. (5) Conclusion I is Converse of Conclusion (P). Conclusion II is Converse of Conclusion (Q).

    BPRE–615

    E + A ⇒ O1-type of Conclusion “Some rectangles are not triangles.”(Q)

    No square is circle.

    All circles are rectangles. E + A ⇒ O1- -type of Conclusion “Some rectangles are not squares.” (R) 3. (4) None follows. 4. (1) Conclusion I is converse of Conclusion (P). 5. (1) No paper is book.

    Some books are libraries. E + I ⇒ O1- type of Conclusion “Some libraries are not papers.” Conclusion I may be derived from the second Premise. 6. (5) All hills are mountains.

    All mountains are rocks. A + A ⇒ A- type of Conclusion “All hills are rocks.” It is Conclusion II. Conclusion I also follows. (7–8) :

    Some perfumes are scents. No scent is a bar. I + E ⇒ O-type of Conclusion. “Some perfumes are not bars.” (P)

    Some scents are perfumes.

    No perfume is a can. I + E ⇒ O-type of Conclusion “Some scents are not cans.”(Q)

    SYLLOGISM 7. (1) Conclusion (Q) supports the Conclusion I. 8. (4) Neither I nor II follows. 9. (2)

    No shop is a factory.

    Some factories are industries. E + I ⇒ O1-type of Conclusion “Some industries are not shops”.

    (13-18) : (i) All triangles are squares → Universal Affirmative (A–type). (ii) Some rectangles are cones → Particular Affirmative (I–type). (iii) No square is rectangle → Universal Negative (E–type). (iv) Some squares are not rectangles → Particular Negative (O–type). (13-14) :

    (15-16) :

    No aim is vision. All visions are objectives. E + A ⇒ O1 – type of conclusion “Some objectives are not aims”. (P)

    All visions are objectives.

    All triangles are squares.

    Some factories are industries.

    No objective is goal. No square is rectangle. All industries are machines. I + A ⇒ I-type of Conclusion “Some factories are machines.” Conclusion II is Converse of it. (10–11) :

    All classes are diamonds. No diamond is store. A + E ⇒ E-type of Conclusion “No class is store”. (P)

    All rooms are classes. No class is store. A + E ⇒ E-type of Conclusion. “No room is store”.(Q)

    All rooms are classes.

    A + E ⇒ E – type of Conclusion “No triangle is rectangle”. (P)

    No square is rectangle.

    Some rectangles are cones. E + I ⇒ O1 – type of Conclusion “Some cones are not squares”. (Q)

    A + E ⇒ E – type of Conclusion “No visions is goal”. (Q) 15. (4) From Negative Premises, Affirmative Conclusion does not follow. 16. (1) Conclusion I is Converse of the Conclusion (Q). (17-18) :

    Some eras are years.

    No triangle is rectangle.

    All years are ages. Some rectangles are cones. E + I ⇒ O1 – type of Conclusion “Some cones are not triangles.” (R) 13. (1) Conclusion I is Converse of the third Premise. 14. (5) Conclusion I is Conclusion (P). Conclusion (R) Some cones are not triangles. Its Venn diagrams

    I + A ⇒ I – type of Conclusion “Some eras are ages.” (P)

    Some years are eras.

    All eras are distances. I + A ⇒ I – type of Conclusion “Some years are distances.” (Q) Some ages are eras.

    All classes are diamonds. A + A ⇒ A-type of Conclusion “All rooms are diamonds”. (R)

    No store is room.

    All rooms are classes. E + A ⇒ O1-type of Conclusion. “Some classes are not stores”.(S) 10. (5) Conclusion (R) is the Conclusion I. Conclusion II is Converse of the first Premise. 11. (2) Conclusion Q is the Conclusion II.

    Cones

    Triangles

    I OR Cones Triangles

    All eras are distances. I + A ⇒ I – type of Conclusion “Some ages are distances.” (R) 17. (1) Conclusion I is the Converse of Conclusion (R). Venn diagrams of Premise : “Some years are eras.”

    Years or

    II OR

    12. (5) Some prizes are winners.

    I

    Eras

    Years All winners are students. I + A ⇒ I-type of Conclusion “Some prizes are students”. Conclusion I is Converse of the first Premise. Conclusion II is Converse of the second Premise.

    Cones

    Triangles

    III Diagrams I and II support Conclusion II.

    BPRE–616

    Eras

    Years or

    II III From the venn diagrams, it is evident that Conclusion II is not valid.

    SYLLOGISM 18. (5) Conclusion I is Conclusion (P). Converse of Conclusion (Q) is: “Some distances are years.” Its venn diagrams would be :

    Venn diagram II supports the Conclusion II. 21. (4) All gases are solids.

    Days

    Calendars

    All solids are liquids.

    III Distances

    Venn diagram II states that “All days are calendars.” It can also be represented as :

    Years

    All solids are liquids.

    I

    Calendars

    or

    No liquid is fluid. Years Distance

    Years

    or II

    III

    Venn diagram III supports Conclusion II. (19–21) : (i) All months are days → Universal Affirmative (A–type). (ii) Some hours are calendars → Particular Affirmative (I–type). (iii) No hour is a day → Universal Negative (E–type). (iv) Some hours are not days → Particular Negative (O–type). 19. (3) All months are days.

    Days i.e., All calendars are days. Therefore, Conclusion II also follows. 20. (3) Some books are comics.

    All comics are novels. I + A ⇒ I–type of Conclusion “Some books are novels.” Conclusion I is the converse of it.

    A + E ⇒ E–type of Conclusion “No month is an hour.” This is Conclusion I.

    Some calendars are hours.

    All performers are singers.

    No novel is an article. A + E ⇒ E–type of Conclusion “No comics is an article.”

    No novel is an article.

    All eagles are woodpeckers. Some woodpeckers are pigeons.

    Books Book s

    Articles

    Articles or

    I

    en

    da

    rs

    Cal end ars

    al

    A + I ⇒ No Conclusion 23. (3) Conclusions I and II form Complementary Pair. Therefore, either Conclusion I or Conclusion II follows. 24. (5) Venn Diagrams of “Some pigeons are woodpeckers” :

    II

    or

    Days

    Pigeons

    C

    or

    A + A ⇒ A – type of Conclusion “All dancers are singers.” This is Conclusion I. Conclusion II is Converse of it. (23–24) :

    I + E ⇒ O–type of Conclusion “Some books are not articles.” Venn diagram of “Some books are not articles” :

    No hour is a day. I + E ⇒ O–type of Conclusion “Some calendars are not days.” Venn diagrams of “Some calendars are not days.” :

    All dances are performers.

    All comics are novels.

    Some books are novels.

    No day is an hour.

    A + E ⇒ E–type of Conclusion “No solid is fluid.” (22–27) : (i) All shows are plays → Universal Affirmative (A–type). (ii) Some shows are movies → Particular Affirmative (I–type). (iii) No door is a wall → Universal Negative (E–type). (iv) Some doors are not walls → Particular Negative (O–type). 22. (5)

    W oo dp ec ke rs

    Distances

    Days

    A + A ⇒ A–type of Conclusion “All gases are liquids.” This is Conclusion II.

    Books I

    Articles

    II

    I

    Or

    III

    BPRE–617

    or

    SYLLOGISM

    Plays

    Woodpeckers

    Houses Roads

    Pigeons

    Theatres III

    II

    Venn diagrams of “Some houses are not apartments” :

    II

    or

    or Houses

    Plays

    Theatres

    H

    ou

    ts en tm ar

    se s

    Ap

    Pigeons

    or

    Apartments

    Woodpeckers

    IV

    III III According to diagram III, “All woodpeckers being pigeons is a possibility.” Either “Some eagles are pigeaons” or “No eagle is a pigeon” is true. Therefore, “Some eagles are definitely not pigeons.” (25–26) :

    Some movies are shows. All shows are plays. I + A ⇒ I – type of Conclusion “Some movies are plays.” (P) Some shows are movies.

    No movies is theatre. I + E ⇒ O–type of Conclusion “Some shows are not theatres.” (Q) Some plays are movies.

    No movies is theatre. I + E ⇒ O–type of Conclusion “Some plays are not theatres.” (R) 25. (1) Venn diagrams of Conclusion (R) :

    27. (4) No door is a wall.

    Some walls are floors. E + I ⇒ O1 – type of Conclusion “Some floors are not doors.” (28–32) : (i) All guests are hosts → Universal Affirmative (A–type). (ii) Some roads are houses → Particular Affirmative (I–type). (iii) No bungalow is an apartment → Universal Negative (E–type). (iv) Some bungalows are not apartments → Particular Negative (O– type). (28–29) :

    Some houses are bungalows. No bungalow is an apartment. I + E ⇒ O – type of Conclusion “Some houses are not apartments.” (P) 28. (3) Venn diagrams of “Some roads are houses” : Ho u

    Roads

    se s

    or

    or

    Apartments

    Houses

    VI After combining the venn diagrams III and V, we get :

    Houses

    Apartments

    Roads

    VII

    or Houses Roads

    Apartments

    Houses

    Theatres

    I

    Or

    Diagram II supports the Conclusion I. 26. (2) Conclusion II is Conclusion Q.

    Roads Plays

    V

    VIII I

    II

    Or

    BPRE–618

    Venn diagram VIII supports the Conclusion I.

    SYLLOGISM 29. (1) Conclusion (P) supports the Conclusion I. Venn diagrams of “Some roads are houses” :

    30. (2)

    Some guests are relatives. All relatives are friends.

    Ho

    Roads

    I + A ⇒ I – type of Conclusion “Some guests are friends.” Venn diagrams of “All guests are hosts” :

    Roads us es

    or

    Houses

    Hosts

    I

    Guests

    II

    I

    II

    Venn diagrams of “Some guests are friends” :

    Roads

    Guests

    es us Ho

    ow

    or

    s

    Bungalows

    or

    s

    al

    nd

    Hou ses

    ie

    Venn diagrams of “Some houses are bungalows” :

    Gu

    ts es

    Fr

    III

    ng

    Some coffee is tea. All tea is water.

    Hosts

    Houses

    Bu

    33. (3)

    or

    Guests

    or

    (33–38) : (i) All tea is water → Universal Affirmative (A–type). (ii) Some coffee is tea → Particular Affirmative (I–type). (iii) No sea is a sky → Universal Negative (E–type). (iv) Some seas are not skies → Particular Negative (O–type).

    III

    Friends

    IV

    I + A ⇒ I–type of Conclusion “Some coffee is water.”

    All tea is water. All water is milk. A + A ⇒ A–type of Conclusion “All tea is milk.” Venn diagrams of “Some coffee are water” :

    or Coffee Friends

    IV

    V

    Guests

    Water or

    Coffee

    Water

    or I

    Bungalows Houses

    VI After combining the venn diagrams III and VI, we get :

    V If we combine venn diagrams I or II and III or IV or V, we would get that atleast some hosts are friends. Therefore, Conclusion II does not follow. 31. (4) Some outlets are factories. All factories are industries.

    Bungalows Houses Roads

    I + A ⇒ I – type of Conclusion “Some outlets are industries.” This is Conclusion II. 32. (5) All certainties are beliefs.

    II

    or

    Water Coffee

    III Venn diagram III supports the Conclusion I. 34. (5)

    No sea is sky.

    Some skies are kites. E + I ⇒ O – type of Conclusion 1 “Some kites are not seas.” Some skies are kites.

    Some beliefs are fears. VII Venn diagram VII supports the Conclusion II.

    A + I ⇒ No Conclusion Conclusion I and Conclusion II form complementary pair. Therefore, either Conclusion I or Conclusions II follows.

    BPRE–619

    All kites are balloons. I + A ⇒ I – type of Conclusion “Some skies are balloons,” Venn diagrams of “Some skies are balloons” :

    SYLLOGISM

    al lo on s

    or

    Televisions

    Balloons

    I

    II

    or

    Balloons

    No pencil is rubber. A + E ⇒ E – type of Conclusion “No register is rubber.” This is Conclusion I. Conclusion II is the Converse of the second Premise. 37. (4)

    Te le v is io ns

    Skies

    All registers are pencils.

    Skies

    or Telephones

    All telephones are computers.

    Televisions

    I + A ⇒ I–type of Conclusion “Some televisions are computers.” Venn diagrams of “Some televisions are computers” : Televisions

    s on isi

    te

    lev

    pu

    Televisions

    Computers

    Te

    rs

    II

    or

    es la xi

    After combining venn diagrams VI and IX, we get :

    m Co

    or

    I

    Ga

    IX

    Telephones

    Galaxies or

    Telephones

    VIII

    Some televisions are telephones.

    III

    Suns

    or

    VII

    Skies

    Venn diagram II supports the Conclusion II. 35. (3) All the three Premises are Particular Affirmative (I–type). No Conclusion follows from the two Particular Premises. Venn diagrams of “Some galaxies are suns” :

    s ne ho le p Te

    B

    Radios

    Computers

    Suns

    Televisions

    I

    II

    X

    or

    Suns Galaxies

    III Venn diagram II supports the Conclusion I. Venn diagrams of “Some radios are televisions” : Radios

    ns io

    Radios

    s vi le Te

    III Venn diagram II supports the Conclusion I. 36. (4)

    All registers are pens.

    or

    Televisions

    IV

    V

    or

    All pens are pencils. A + A ⇒ A – type of Conclusion “All registers are pencils.”

    Televisions Radios

    All pens are pencils. No pencil is rubber. A + E ⇒ E – type of Conclusion “No pen is rubber.”

    VI Venn diagrams of “Some televisions are telephones” :

    BPRE–620

    Venn diagram X supports the Conclusion II. 38. (4)

    All cats are dogs. All dogs are jackals.

    A + A ⇒ A – type of Conclusion “All cats are jackals.” Conclusion II is the Converse of it.

    All wolves are dogs. All dogs are jackals. A + A ⇒ A – type of Conclusion “All wolves are jackals”. This is Conclusion I. (39–44) : (i) All bedsheets are cupboards → Universal Affirmative (A-type). (ii) Some cupboards are buckets → Particular Affirmative (I-type). (iii) No bucket is a tap → Universal Negative (E-type). (iv) Some buckets are not taps → Particular Negative (O-type).

    SYLLOGISM 39. (1)

    Some cupboards are buckets.

    After combining the Venn diagrams II and V, we get :

    Venn diagrams of “Some flowers are insects : Flowers

    Penguins No bucket is a tap.

    Kangaroos

    I + E ⇒ O–type of Conclusion “Some cupboards are not taps.” Thus, neither Conclusion I nor Conclusion II follows. 40. (4)

    Dolphins

    rs we o l F

    I + A ⇒ I–type of Conclusion “Some kangaroos are elephants.” Venn diagrams of “Some penguins are kangaroos” : Penguins n Pe

    s in gu n Ka

    I

    gar

    s oo

    or

    Kangaroos

    or

    Insects Flowers

    VII

    Venn diagram VII supports the Conclusion I. Venn diagram II supports the Conclusion II. Thus, both the Conclusions I and II follow. 41. (4) All the three Premises are Particular Affirmative (I-type). No Conclusion follows from the two Particular Premises. Venn diagrams of “Some plants are animals” : Plants

    ts Plan

    Penguins

    a im An

    ls or

    I

    III Venn diagrams of “Some kangaroos are dolphins” :

    IV

    Dolphins

    or

    Dolphins

    All gases are wood. III Venn diagrams of “Some animals are flowers” :

    im An

    als Flo

    rs or we

    IV

    Flowers

    V

    or

    Kangaroos

    Insects

    42. (5) All metals are gases.

    Plants

    Animals

    V

    Flowers

    X Venn diagram X supports both the Conclusions I and II.

    Kangaroos or ins l ph o D

    Plants Animals

    II

    Animals

    os ro

    IX After combining Venn diagrams II, V and VIII, we get :

    Animals

    or

    a ng Ka

    VIII

    or

    II

    Kangaroos

    Insects

    VII

    Some kangaroos are dolphins.

    All dolphins are elephants.

    s or ect Ins

    A + A ⇒ A–type of Conclusion “All metals are wood.”

    All gases are wood.

    All wood are fabrics. A + A ⇒ A–type of Conclusion “All gases are fabrics.”

    All metals are wood.

    Flowers

    All wood are fabrics. Animals

    VI VI

    BPRE–621

    A + A ⇒ A–type of Conclusion “All metals are fabrics.” Conclusion I is the Converse of it.

    SYLLOGISM 43. (4)

    Some circles are squares. Suns

    All squares are triangles. I + A ⇒ I–type of Conclusion “Some circles are triangles.”

    Stars Moons

    Moons

    Planets

    All squares are triangles. III All triangles are hexagons. A + A ⇒ A–type of Conclusion “All squares are hexagons.” Some circles are triangles.

    Venn diagram II supports the Conclusion II. Venn diagrams of “All planets are moons” :

    All triangles are hexagons. I + A ⇒ I-type of Conclusion “Some circles are hexagons.” This is Conclusion I. Venn diagrams of “Some circles are hexagons” :

    Moons

    Circles

    g xa He

    s or on

    Venn diagram IX contradicts the Conclusion I. (45-47) : (i) All stars are comets → Universal Affirmative (A–type). (ii) Some planets are stars → Particular Affirmative (I–type). (iii) No comet is an astronaut → Universal Negative (E–type). (iv) Some comets are not astronauts → Particular Negative (O–type). (45–46) :

    or

    Planets

    Circles

    IX

    IV

    Hexagons

    Moons

    Some planets are stars. I

    II

    or

    All stars are comets.

    Planets

    Hexagons

    I + A ⇒ I-type of Conclusion “Some planets are comets”. (P)

    V

    Circles

    All stars are comets.

    Venn diagrams of “Some moons are stars” :

    III Venn diagram II supports the Conclusion II. 44. (2) Some moons are stars.

    Moons Moons

    or

    Stars

    VI

    All stars are suns. I + A ⇒ I–type of Conclusion “Some moons are suns.” Venn diagrams of “Some moons are suns” :

    No comet is an astronaut. A + E ⇒ E-type of Conclusion “No star is an astronaut”. (Q)

    Stars

    Some planets are comets.

    VII

    I + E ⇒ O-type of Conclusion “Some planets are not astronauts”. (R) 45. (4) Conclusion I is the Conclusion (Q). 46. (1) Venn diagram of “Some planets are not astronauts” :

    or

    Stars Moons

    No comet is an astronaut.

    or

    Suns

    Planets

    Suns

    ro na u

    Moons

    ts

    Moons

    I

    II

    or

    After combining Venn diagrams IV and VIII, we get :

    BPRE–622

    As t

    VIII

    I

    SYLLOGISM Or

    Some experiments are blueprints. Chairs

    Planets All blueprints are chairs. Astronauts

    I + A ⇒ I-type of Conclusion “Some experiments are chairs.” This is the Conclusion (2).

    Blueprints

    No table is a blueprint. V

    C

    an

    ce

    s

    Performances

    I

    47. (4) All trains are buses.

    VI

    Or Blueprints

    From the Venn diagram VI, it is clear that Conclusion (3) does not follow. Venn diagrams of “Some experiments are chairs” :

    Performances

    en

    rs

    rim

    All buses are rickshaws.

    Ex pe

    Ch ai

    All buses are rickshaws. A + A ⇒ A–type of Conclusion “All trains are rickshaws”. This is the Conclusion I.

    irs, B luepr i

    r fo rm

    Venn diagram II supports the Conclusion I. Conclusion II is the Converse of the Conclusion (P).

    ha

    s nt

    III

    After combining Venn diagrams II and V, we get :

    Pe

    Astronauts

    E + A ⇒ O1-type of Conclusion “Some chairs are not tables.” Venn diagrams of “Some blueprints are not performances” :

    Bl ue pr in ts

    Planets

    All blueprints are chairs.

    ts

    II Or

    No rickshaw is a plane. A + E ⇒ E–type of Conclusion “No bus is a plane”.

    II

    VII

    Or

    Or

    All trains are rickshaws.

    xp

    e ri m en

    t

    s

    Performances

    E

    Blueprints No rickshaw is a plane. A + E ⇒ E-type of Conclusion “No train is a plane”. 48. (3)

    Chairs III

    No performance is an experiment. Some experiments are blueprints. E + I ⇒ O1-type of Conclusion “Some blueprints are not performances.” Some experiments are blueprints. No blueprint is a table. I + E ⇒ O-type of Conclusion “Some experiments are not tables.” This is the Conclusion (1).

    Venn diagrams of “All blueprints are chairs” :

    VIII Or

    Chairs

    Chairs Blueprints

    Experiments

    IV Or

    BPRE–623

    IX

    SYLLOGISM Venn diagrams of “Some chairs are not tables” :

    “Some platforms are not tracks” This is the Conclusion (2).

    Some roads are platforms.

    Venn diagram VI supports the Conclusion (3). Venn diagrams of “All platforms are dividers” :

    Chairs Tables X Or

    Chairs

    All platforms are dividers. I + A ⇒ I-type of Conclusion “Some roads are dividers.” This is the Converse of the Conclusion (5). Venn diagrams of “All stations are platforms” : Platforms

    Tables

    Platforms

    Stations

    Dividers Platforms

    VII

    VIII

    After combining the Venn diagrams IV and VIII, we get :

    Or

    D

    rs, Platfo ide r iv

    s m

    I II Venn diagrams of “Some platforms are not tracks” :

    Or

    Or Platforms

    Stations

    XI

    Dividers

    Tracks

    Platforms

    Chairs Tables

    Tracks

    XII

    (50-52):

    III

    After combining Venn diagrams VIII and XI, we get :

    (i) All digs are fruits → Universal Affirmative (A-type). (ii) Some cherries are digs → Particular Affirmative (I-type). (iii) No van is a trump → Universal Negative (E-type). (iv) Some vans are not trumps → Particular Negative (O-type).

    Or

    pe riments Ex Chairs

    Platforms Tracks

    Tables

    IV Or

    50. (4) Some Cherries are digs.

    XIII

    All stations are platforms.

    Tracks

    V After combining Venn diagrams II and IV, we get : tf o

    r m s, S ta tio

    Tracks

    ns

    All platforms are dividers. A + A ⇒ A-type of Conclusion “All stations are dividers.” This is the Conclusion (1).

    All digs are fruits.

    Platforms

    P la

    From the Venn diagram XIII, it is clear that Conclusion (4) follows. From the Venn diagram II, it is clear that Conclusion (5) is true. 49. (4)

    IX Venn diagram IX does not support the Conclusion (4).

    I + A ⇒ I-type of Conclusion “Some cherries are fruits”. This is the Conclusion II. 51. (2) No van is a trump.

    Some trumps are yachts E + I ⇒ O1 - type of Conclusion. “Some yachts are not vans”. All kites are yachts.

    Some platforms are roads. No road is a track. I + E ⇒ O-type of Conclusion

    VI

    BPRE–624

    Some yachts are trumps. A + I ⇒ No Conclusion Vann diagrams of “All kites are yachts”:

    SYLLOGISM 52. (4) All axes are candles.

    Yachts

    No candle is a river.

    or

    Yachts II

    I

    Venn diagrams of “Some trumps are yachts” :

    Gains

    This is the Conclusion II.

    or

    Benefits

    I

    II

    Venn diagram of “No river is a

    or

    candle” :

    ps ts

    m

    Gains

    Ya ch

    ru

    A + E ⇒ E – type of Conclusion “No axe is a river.”

    s it ef en B

    Kites

    T

    Gains

    Kites

    River

    III

    III

    I s

    “Venn diagram of “No jack is a

    ht Ya c

    Benefits

    Candle

    river” :

    Trumps

    The above venn diagrams do not support the Conclusion I. Conclusion II is the Conclusion (P). 54. (1) Venn diagrams of “Some gains are not users” :

    or

    I

    grams I and II, we get :

    II

    Kites

    or

    Yachts

    River

    Candle

    Jack Gains

    VI

    Venn diagrams of “Some yachts are not vans”.

    ht

    s

    Yachts Vans

    Vans or

    VII

    VIII

    III

    (53–54) :

    All users are benefits. No benefit is a profit. A + E ⇒ E–type of Conclusion “No user is a profit”. (P)

    No benefit is a profit.

    Gains Gains

    IX Venn diagram VIII contradicts the Conclusion II.

    Benefits

    IV

    V or

    No user is a profit.

    Vans

    or

    Some profits are gains. E + I ⇒ O1–type of Conclusion “Some gains are not benefits.” (Q)

    Yachts

    The above venn diagrams support the Conclusion I. Venn diagrams of “Some gains are not benefits” :

    s it ef en B

    Ya c

    Users

    III

    Venn diagram VI supports the Conclusion I.

    or

    Users

    II After combining the Venn dia-

    ps um

    Users or

    Gains

    After combining Venn diagrams II and IV, we get :

    Tr

    Gains

    River

    Jack

    V

    Some profits are gains. E + I ⇒ O1–type of Conclusion “Some gains are not users.” (R) 53. (4) Venn diagrams of “Some gains are not benefits” :

    BPRE–625

    Gains

    Benefits

    VI Venn diagram V supports the Conclusion II.

    SYLLOGISM (55–56) :

    Some interviews are talks.

    56. (5) Conclusion I and Conclusion II form Complementary Pair. Therefore, either Conclusion I or Conclusion II follows.

    A + I ⇒ No Conclusion Venn diagrams of “Some lamps are shafts” : Lamps

    57. (5) No card is a tag.

    All talks are meetings. I + A ⇒ I–type of Conclusion “Some interviews are meetings”. (P) 55. (1) Conclusion I is the Converse of the Conclusion (P). Venn diagrams of “Some interviews are meetings” :

    Shafts or

    Lamps

    Shafts

    All tags are labels. E + A ⇒ O1–type of Conclusion “Some labels are not cards”.

    II

    I

    or

    All tags are labels. All labels are marks.

    gs tin ee M

    In te rv ie w s

    Interviews

    or

    I

    Meetings

    II or Meetings Interviews

    A + A ⇒ A–type of Conclusion “All tags are marks.” Conclusion I and Conclusion II form Complementary Pair. Therefore, either Conclusion I or Conclusion II follows. (58–60) : (i) All tonics are solids Universal Affirmative (A-type). (ii) Some hats are pans Particular Affirmative (I-type). (iii) No coil is a pan Universal Negative (E-type). (iv) Some coils are not pans Particular Negative (O-type). 58. (5) All biscuits are tonics.

    III Venn diagrams of “All conferences are meetings” : Meetings

    Conferences

    Conferences

    or

    Lamps

    III Venn diagrams of “All roses are lamps” : Lamps Roses

    Lamps or Roses

    IV V Venn diagrams of “Some shafts are coins” : Shafts

    All tonics are solids. A + A ⇒ A -type of Conclusion “All biscuits are solids”. Conclusion I is the Converse of it.

    All almonds are biscuits.

    Coins or

    Shafts

    VI

    Coins

    VII

    or

    Meetings

    IV

    V

    After combining venn diagrams II and IV, we get Interviews

    All biscuits are tonics. A + A ⇒ A -type of Conclusion “All almonds are tonics”. This is the Conclusion II. 59. (4) No coil is a pan.

    Meetings

    Some pans are hats. Conferences

    VI Venn diagram VI supports the Conclusion II.

    E + I ⇒ O -type of Conclusion 1 “Some hats are not coils”. Neither Conclusion I nor Conclusion II follows.

    Shafts

    VIII After combining venn diagrams III and VIII, we get : Coins Shafts Lamps

    60. (5) All roses are lamps.

    Some lamps are shafts.

    BPRE–626

    IX

    SYLLOGISM Venn diagram IX supports the Conclusion I. After combining venn diagrams III and IV, we get :

    Venn diagrams of “Only a few trees are forests” :

    62. (2)

    All gliders are parachutes.

    Trees

    Shafts

    No parachute is an airplane.

    Lamps

    A + E ⇒ E-type of Conclusion “No glider is an airplane”.

    Roses

    No parachute is an airplane.

    For

    Trees

    Venn diagram X supports the Conclusion II. (61–65) : (i) All clerks are assistants Universal Affirmative (A-type). (ii) Only a few trees are forests Particular Affirmative (I-type). [Only a few = Some] (iii) No assistant is an officer Universal Negative (E-type). (iv) Some assistants are not officers Particular Negative (O-type).

    Pa

    s ter

    ch u

    te s

    No assistant is an officer.

    All officers are managers. E + A ⇒ O1-type of Conclusion “Some managers are not assistants”. Venn diagrams of “Some managers are not assistants” :

    Trees

    V After combining venn diagrams I and V, we get :

    Parachutes

    Forests Trees II

    I

    Grases

    or

    61. (2) All clerks are assistants.

    No assistant is an officer. A + E ⇒ E-type of Conclusion “No clerk is an officer”.

    or

    Helicopter

    Parachute VI

    III Venn diagram II supports the Conclusion II.

    Venn diagram VI supports the Conclusion I. 64. (5)

    Only a few paragraphs are stories.

    63. (5) All grasses are trees.

    No story is a movie. Only a few trees are forests.

    I + E ⇒ O-type of Conclusion “Some paragraphs are not movies.” Venn diagrams of “Some paragraphs are not movies” :

    A + I ⇒ No Conclusion

    Only a few trees are forests. e ag an M

    As

    rs

    Managers si s

    ta nt s

    or

    Assistants

    II

    I or

    IV

    Forests

    Helicopters

    ra

    Forests

    or

    E + A ⇒ O1-type of Conclusion “Some helicopters are not parachutes”. Venn diagrams of “Some helicopters are not parachutes” :

    op l ic He

    s or

    III

    All airplanes are helicopters. X

    est

    All forests are lands. I + A ⇒ I-type of Conclusion “Only a few trees are lands”. This is the Conclusion II. Venn diagrams of “All grasses are trees” :

    Paragraphs r Pa

    a

    a gr

    ph

    s Movies

    or

    Movies

    II

    I

    or

    Manager

    Assistant III

    Venn diagram II supports the Conclusion II.

    Trees Grasses

    Grasses or

    Paragraph

    Movie

    Trees II

    I

    BPRE–627

    III

    SYLLOGISM Venn diagrams I and II support the Conclusion I.

    Pies

    No story is a movie.

    Pie

    All movies are videos.

    Biscu it

    Biscuits

    V

    All heights are peaks.

    Biscuits

    E + A ⇒ O1–type of Conclusion “Some peaks are not mountains”.

    Pies

    No height is a mountain.

    Stores

    All mountains are landscapes.

    VI

    Venn diagrams of “Only a few biscuits are dumplings” :

    V

    IV

    or

    Biscuits Dum

    B

    i ts isc u

    gs

    or

    Dumplings

    All mountains are landscapes. VIII

    VII

    VI

    or

    These venn diagrams support the Conclusions II. 65. (5)

    Dumplings

    No corn is a pie.

    Biscuits

    Only a few pies are biscuits. E + I ⇒ O1-type of Conclusion “Some biscuits are not corns.” Venn diagrams of “Some biscuits are not corns” :

    IX

    Biscuits Corns or

    I + A ⇒ I–type of Conclusion “Some valleys are landscapes”. This is the Conclusion III.

    No height is a mountain. Some mountains are valleys. E + I ⇒ O1–type of Conclusion “Some valleys are not heights”. Venn diagrams of “Some valleys are not heights” :

    After combining venn diagrams V and VIII, we get : Pies

    u its Bisc

    E + A ⇒ O1–type of Conclusion “Some landscapes are not heights”.

    Some valleys are mountains. plin

    Story

    Video

    66. (2) No mountain is height.

    or

    Videos Stories or

    or

    IV

    E + A ⇒ O1-type of Conclusion “Some videos are not stories.” Venn diagrams of “Some videos are not stories” :

    os Vide

    s

    (iii) No height is a mountain Universal Negative (E–type). (iv) Some heights are not mountains Particular Negative (O– type).

    Valleys

    He igh ts or

    Valleys

    Heights

    Biscuits

    I

    Corns

    II or

    Dumplings

    II

    I

    Valleys

    or

    X Biscuit

    Corn

    III These venn diagrams support the Conclusion I. Venn diagrams of “Only a few pies are biscuits” :

    Venn diagram X supports the Conclusion II. (66–69) : (i) All heights are peaks Universal Affirmative (A–type). (ii) Only a few mountains are valleys Particular Affirmative (I– type). [Only a few = Some]

    BPRE–628

    Heights

    III Venn diagrams I and II support the Conclusion I. Thus, Conclusion I and Conclusion III follow. 67. (3) Some books are tables.

    All tables are novels. I + A ⇒ I–type of Conclusion

    SYLLOGISM “Some books are novels”. Some tables are chairs.

    All chairs are drawers. I + A ⇒ I–type of Conclusion “Some tables are drawers”. Some chairs are tables.

    Venn diagram VII contradicts the Conclusion I. Venn diagram V supports the Conclusion II. 68. (2)

    No choice is an option.

    Plans Plans

    E + I ⇒ O1–type of Conclusion “Some alternatives are not choices”.

    Some options are alternatives.

    Books

    No v

    Books

    els

    or

    All alternatives are plans. Novels

    II

    I

    Goals

    VII

    VI

    or

    Some options are alternatives. All tables are novels. I + A ⇒ I–type of Conclusion “Some chairs are novels”. Venn diagrams of “Some books are novels” :

    or

    Goals

    I + A ⇒ I–type of Conclusion “Some options are plans”. This is the Conclusion II.

    Plans

    Goals

    VIII After combining venn diagrams II, V and VII, we get :

    rnatives Plans, Alte

    No goal is an alternative.

    or

    Goals

    Choices

    Novels

    All alternatives are plans. Books

    III Venn diagrams of “Some chairs are novels” :

    Choices

    at

    els

    or

    Novels

    or

    Choices

    es iv

    Chairs

    or

    Plans, Alternatives Choices

    I

    IV

    IX

    Alternatives

    rn te Al

    Chairs

    No v

    E + A ⇒ O1–type of Conclusion “Some plans are not goals”. Venn diagrams of “Some alternatives are not choices” :

    II

    Goals

    or

    V or Novels

    Alternatives

    Choices

    X

    Chairs

    III VI After combining venn diagrams II and VI, we get : Books

    Venn diagram II contradicts the Conclusion III. Venn diagrams of “All alternatives are plans” :

    Venn diagram X supports the Conclusion I. 69. (1) All computers are laptops.

    No laptop is smartphone. Plans

    Alternatives

    Novels Alternatives

    or

    A + E ⇒ E–type of Conclusion “No computer is smartphones”.

    No laptop is smartphone.

    Chairs Plans

    IV

    VII

    V

    Venn diagrams of “Some plans are not goals” :

    BPRE–629

    Some smartphones are digiclockers. E + I ⇒ O1–type of Conclusion “Some digilockers are not laptopes”.

    SYLLOGISM Some smartphones are digilockers.

    Rs Ns

    All digilockers are smartwatches.

    Ls

    I + A ⇒ I–type of Conclusion “Some smartphones are smartwatches.” Conclusion I is the Converse of it.

    No computer is smartphone.

    All Ls are Bs. VI Venn diagram VI supports the Conclusion I. Conclusion III is the Converse of “Some Ls are Ns”. 71. (4) Y @ P : All Ys are Ps.

    Ps Ys

    Some smartphones are digilockers. E + I ⇒ O1–type of Conclusion “Some digilockers are not computers”. This is the Conclusion III. Venn diagrams of “Some digilockers are not laptops” :

    Some Ms are Ls.

    I + A ⇒ I-type of Conclusion “Some Ms are Bs”. This is the Conclusion II. 74. (5) G #K : Some Gs are not Ks.

    Gs

    Ys I

    or

    II

    Ps II

    I

    or

    P % T : Some Ps being Ts is a possibility.

    Ps

    Gs Ks

    Ks or

    Ts

    or

    Ts Ps

    G

    K III

    K + M : Some Ks are Ms.

    Digilockers D ig er ck il o

    Laptops or

    Laptops

    s

    II

    I

    or

    Digilockers

    Laptops

    Ns or I

    Ls Ns II

    or

    III N > R : All Ns being Rs is a possibility.

    IV

    IV

    Ps Ts

    Rs or Ns V

    After combining venn diagrams III and IV, we get :

    Ks

    Ms or IV

    Ks Ms V

    or

    Ts Ps Ys

    VI Venn diagram VI supports the Conclusion II. Venn diagram II supports the Conclusion I. 72. (3) Z + H : Some Zs are Hs. H @ L : All Hs are Ls. Some Zs are Hs.

    Ns Ls

    Rs Ns

    or

    V After combining the venn diagrams I and IV, we get :

    III Venn diagram II supports the Conclusion II. 70. (1) L + N : Some Ls are Ns.

    Ls

    III

    All Hs are Ls. I + A ⇒ I-type of Conclusion “Some Zs are Ls”. Conclusion III is the Converse of it. Conclusion II is the Implication of “All Hs are Ls”. 73. (2) M % L : Some Ms being Ls is a possibility. L > B : All Ls being Bs is a possibility.

    BPRE–630

    Ms Ks VI After combining the venn diagrams II and V, we get :

    Gs Ks Ms

    VII Venn diagram VII supports the Conclusion II and Conclusion III.

    INSURANCE EXAMS (1–6) : (i) All pens are books → Universal Affirmative (A-type). (ii) Some books are pages → Particular Affirmative (I-type). (iii) No pocket is a bag → Universal Negative (E-type). (iv) Some pockets are not bags → Particular Negative (O-type).

    SYLLOGISM (1–2) : Some books are pages.

    All pages are papers.

    I + A ⇒ I-type of Conclusion “Some books are papers.” 1. (4) Neither Conclusion I nor Conclusion II follows. 2. (2) Conclusion II is same as the derived Conclusion. (3–4) : Some Ds are Gs.

    All Gs are Ks. I + A ⇒ I-type of Conclusion “Some Ds are Ka.” (A)

    5. (4) Neither Conclusion I nor Conclusion II follows. 6. (4) Neither Conclusion I nor Conclusion II follows. (7–11) : (i) All paintings are master pieces → Universal Affirmative (Atype). (ii) Some art works are paintings → Particular Affirmative (Itype). (iii) No brother is genius → Universal Negative (E-type). (iv) Some brothers are not genius → Particular Negative (Otype). 7. (2) Some art works are paintings.

    (iii) No boy is a woman → Universal Negative (E-type). (iv) Some boys are not women → Particular Negative (O-type). 12. (2) Both the Premises are Particular Affirmative (I-type). No Conclusion follows from the two Particular Premises. 13. (2)

    All scholars are teachers.

    Some teachers are researchers. A + I ⇒ No Conclusion Conclusion III is Converse of the second Premise. Conclusion IV is Converse of the first Premise. 14. (3) Some men are boys.

    All Gs are Ks. All paintings are master pieces. All Ks are Ls. A + A ⇒ A-type of Conclusion “All Gs are Ls.” (B) Some Ds are Ks.

    All Ks are Ls. I + A ⇒ I-type of Conclusion “Some Ds are Ls.” (C) 3. (5) Both Conclusions I and II follow. Conclusion I is Conclusion (C). Conclusion II is Conclusion (B). 4. (1) Conclusion I is Converse of Conclusion (A). (5–6) : Some files are folders.

    All folders are pockets.

    I + A ⇒ I-type of Conclusion “Some files are pockets.” (A) All folders are pockets.

    No pocket is a bag.

    A + E ⇒ E-type of Conclusion “No folder is a bag.” (B) Some files are pockets.

    No pocket is a bag.

    I + E ⇒ O-type of Conclusion “Some files are not bags.” (C)

    I + A ⇒ I-type of Conclusion. “Some art works are master pieces.” Conclusion II is Converse of it. 8. (4) No brother is genius.

    Some genius are men. E + I ⇒ O1–type of Conclusion “Some men are not brothers.” 9. (1) All pants are skirts.

    No skirt is a shirt. A + E ⇒ E–type of Conclusion “No pant is a shirt.” Conclusion I is Converse of the first Premise. 10. (4) All planes are tyres.

    No boy is a woman. I + E ⇒ O-type of Conclusion “Some men are not women.” Conclusion IV is Converse of the first Premise. Conclusions I and III form Complementary Pair. Therefore, either Conclusion I or III follows. 15. (4)

    No manager is a teacher. All teachers are researchers.

    E + A ⇒ O1-type of Conclusion “Some researchers are not managers.” Conclusion III is the implication of the second Premise. Conclusion IV is the Converse of the second Premise. 16. (3) All houses are rooms.

    All rooms are windows. All tyres are engines. A + A ⇒ A–type of Conclusion “All planes are engines.” 11. (4) Both the Premises are Particular Affirmative. No Conclusion follows from the two Particular Premises. (12–16) : (i) All scholars are teachers → Universal Affirmative (A-type). (ii) Some men are home-makers → Particular Affirmative (Itype).

    BPRE–631

    A + A ⇒ A-type of Conclusion “All houses are windows.” This is Conclusion III. Conclusion IV is Converse of this Conclusion. (17–22) : (i) All rats are dogs → Universal Affirmative (A&type). (ii) Some dogs are bulls → Particular Affirmative (I&type). (iii) No bull is a dog → Universal Negative (E–type).

    SYLLOGISM (iv) Some bulls are not dogs → Particular Negative (O–type).

    24. (2) All women are doctors.

    17. (4) All rats are dogs.

    Some dogs are bulls. A + I ⇒ No Conclusion 18. (2) All women are doctors.

    All doctors are nurses. A + A ⇒ A-type of Conclusion “All women are nurses”. Conclusion II is Converse of it. 25. (1) Some trees are birds.

    30. (1) All nibs are inks.

    No ink is colour. A + E ⇒ E-type of Conclusion “No nib is colour”. Conclustion I is Converse of it. 31. (1) All ventures are project.

    No project is dream. All doctors are nurses. A + A ⇒ A-type of Conclusion “All women are nurses”. Conclusion II is Converse of it. 19. (1) Some trees are birds.

    All birds are snails. I + A ⇒ I-type of Conclusion “Some trees are snails”. Conclusion I is Converse of it. 20. (3) From both the statements C>D>E>A>B Both the conclusions are correct. 21. (2) Players form a part of singers and hockey-players are a part of players. Therefore, all singers were not admitted to the college. Only Conclusion II is true. 22. (1)

    All the cancer patients were cured.

    All the cured patients grew fat. A + A ⇒ A-type of Conclusion “All the cancer partients grew fat”. This is Conclusion I. (23 –28) : (i) All rats are dogs → Universal Affirmative (A&type). (ii) Some dogs are bulls → Particular Affirmative (I&type). (iii) No bull is a dog → Universal Negative (E–type). (iv) Some bulls are not dogs → Particular Negative (O–type). 23 (4) All rats are dogs.

    Some dogs are bulls. A + I ⇒ No Conclusion

    All birds are snails. I + A ⇒ I-type of Conclusion “Some trees are snails”. Conclusion I is Converse of it. 26. (3) From both the statements C>D>E>A>B Both the conclusions are correct. 27. (2) Players form a part of singers and hockey-players are a part of players. Therefore, all singers were not admitted to the college. Only Conclusion II is true. 28. (1) All the cancer patients were cured.

    All the cured patients grew fat. A + A ⇒ A-type of Conclusion “All the cancer partients grew fat”. This is Conclusion I. (29–34) : (i) All pens are nibs → Universal Affirmative (A-type). (ii) Some numbers are digits → Particular Affirmative (I-type). (iii) No ink is colour → Universal Negative (E-type). (iv) Some inks are not colours → Particular Negative (O-type). 29.(2) All pens are nibs.

    All nibs are inks. A + A ⇒ A-type of Conclusion “All pens are inks.” This is Conclusion II.

    All nibs are inks.

    No ink is colour. A + E ⇒ E-type of Conclusion “No nib is colour”.

    BPRE–632

    A + E ⇒ E-type of Conclusion “No venture is dream”. This is Conclusion I. 32. (4) No road is a way.

    All ways are paths. E + A ⇒ O1-type of Conclusion “Some paths are not roads”.

    All ways are paths.

    No path is bridge. A + E ⇒ E-type of Conclusion “No way is bridge”. (33-34) :

    Some numbers are digits.

    No digit is alphabet. I + E ⇒ O-type of Conclusion “Some numbers are not alphabets”. (P) No digit is alphabet.

    All alphabets are letters.

    E + A ⇒ O1-type of Conclusion “Some letters are not digits.” (Q) 33. (3) Conclusions I and II form Complementary Pair. Therefore, either Conclusion I or II follows. 34. (2) Conclusion P ⇒ ‘Some numbers are not alphabets’. This implies that ‘All numbers can never be alphabets. (35–39) : (i) All pages are markers → Universal Affirmative (A-type). (ii) Some fruits are apples → Particular Affirmative (I-type). (iii) No book is a page → Univer sal Negative (E-type). (iv) Some books are not pages → Particular Negative (O-type).

    SYLLOGISM 35. (1) No book is a page All pages are markers.

    E + A ⇒ O1-type of Conclusion “Some markers are not books.” This Conclusion implies that “All markers can never be books.” All pages are markers.

    (41–46) : (i) All hosts are guests → Universal Affirmative (A–type). (ii) Some visitors are invitees → Particular Affirmative (I–type). (iii) No cup is a medal → Universal Negative (E–type). (iv) Some cups are not medals → Particular Negative (O–type). (41–42) :

    Some invitees are visitors.

    36. (2) Some fruits are apples.

    All visitors are hosts.

    All apples are guavas.

    All hosts are guests. A + A ⇒ A–type of Conclusion “All visitors are guests.” (Q)

    Some invitees are hosts. No guava is a banana.

    A + E ⇒ E-type of Conclusion “No apple is a banana.” 37. (2) All locks are keys.

    Some keys are pockets.

    A + I ⇒ No Conclusion Conclusion II is Converse of the second Premise.

    All hosts are guests. I + A ⇒ I–type of Conclusion “Some invitees are guests.” (R) 41. (3) Conclusion I is the Converse of Conclusion (R). Venn Diagrams of “Some invitees are hosts”

    I

    39. (2) All apples are guavas.

    No guava is a banana.

    A + E ⇒ E-type of Conclusion “No apple is a banana.” This is Conclusion II. 40. (4) Both the Premises are Par ticular Affirmative (I-type). No Conclusion follows from the two Particular Premises.

    No trophy is a shield. A + E ⇒ E–type of Conclusion “No medal is a shield.” (Q) 44. (2) Neither Conclusion I nor II follows. 45. (5) Conclusion II is the converse of Conclusion (Q). 46. (2) Both the Premises are Par ticular Affirmative (I–type). No Conclusion follows from the two Particular Premises. (47–52) : (i) All triangles are squares → Universal Affirmatve (A-type). (ii) Some planes are ship → Particular Affirmative (I-type). (iii) No cone is rtiangle → Universal Negative (E-type). (iv) Some cones are not triangles → Particular Negative (O-type). 47. (3) No cone is triangle.

    All triangles are squares. E + A ⇒ O1- type of Conclusion “Some squares are not cones”. Its venn diagrams

    Hosts

    38. (5) All pages are markers.

    All markers are articles. A + A ⇒ A-type of Conclusion “All pages are articles.” This is Conclusion I. Conclusion II is Converse of the second Premise.

    All medals are trophies. E + A ⇒ O1 – type of conclusion “Some trophies are not cups.” (P)

    All visitors are hosts. I + A ⇒ I–type of Conclusion “Some invitees are hosts.” (P)

    All apples are guavas.

    I + A ⇒ I-type of Conclusion “Some fruits are guavas”. Conclustion II is Converse of it.

    No cup is a medal.

    All medals are trophies.

    All markers are articles.

    A + A ⇒ A - type of Conclusion “All pages are articles”.

    I + A ⇒ I–type of Conclusion “Some instruments are tools.” Conclusion I is the Converse of this Conclusion. (44–45) :

    or

    Hosts Hosts

    Cones

    Squares

    or

    II

    III

    Diagram III supports conclusion II. 42. (2) Neither Conclusion I nor II follows. 43. (4)

    I or

    Squares Cones

    Some instruments are devices.

    All devices are tools.

    BPRE–633

    II

    SYLLOGISM or

    “All alphabets are digits”. (P)

    All letters are digits. Troops

    Squares

    Cones

    No digit is a number. A + E ⇒ E - type of Conclusion “No letter is a number”. (Q)

    III Diagram II supports Conclusion I. 48. (1) Some planes are ship.

    No ship is a radar. I + E ⇒ O - type of Conclusion “Some planes are not radars”.

    All cars are radars. No radar is a ship. A + E ⇒ E - type of Conclusion “No car is a ship”. This is Conclusion I. Venn diagrams of “Some planes are not radars”.

    Planes

    Radars

    All alphabets are digits. No digit is a number. A + E ⇒ E - type of Conclusion “No alphabet is a number”. (R) 49. (4) Conclusion (P) is Conclusion II. 50. (3) Conclusion I is Converse of Conclusion (R). (51–52) :

    Some troops are soldiers. All soldiers are fighters. I + A ⇒ I - type of Conclusion “Some troops are fighters”. (P)

    All soldiers are fighters. No fighter is a warrior. A + E ⇒ E - type of Conclusion “No soldier is a warrior”. (Q)

    Some troops are fighters. I or

    No fighter is a warrior.

    Planes

    I + E ⇒ O - type of Conclusion “Some troops are not warriors”. (R) 51. (1) Venn diagrams of Conclusion (R)

    Radars

    II or Venn Diagram II supports Conclusion I. Conclusion II is Converse of Conclusion (P). 52. (5) Neither Conclusion I nor II follows. (53–60) : (i) All alphabets are letters → Universal Affirmative (A–type). (ii) Some windows are floors → Particular Affirmative (I–type). (iii) No digit is a number → Univer sal Negative (E–type). (iv) Some digits are not numbers → Particular Negative (O–type). 53. (3) All alphabets are letters.

    All letters are digits. A + A ⇒ A – type of conclusion “All alphabets are digits.” This is Conclusion II.

    All letters are digits. No digit is a number. A + E ⇒ E – type of conclusion “No letter is a number.”

    All alphabets are digits.

    Troops

    Warriors

    No digit is a number. A + E ⇒ E–type of conclusion “No alphabet is a number.” This is Conclusion I.

    II or

    Planes

    Warriors

    Radars

    I or

    Troops III Venn diagram II supports Conclusion II. (49–50) :

    Warriors

    54. (1) All fruits are flowers.

    No flower is a sweet. A + E ⇒ E–type of conclusion “No fruit is sweet.”

    No flower is sweet.

    All alphabets are letters. Some sweets are desserts. All letters are digits. A + A ⇒ A - type of Conclusion

    II or

    BPRE–634

    E + I ⇒ O1 – type of conclusion “Some desserts are not flowers.”

    SYLLOGISM No fruit is sweet. Some sweets are desserts. E + I ⇒ O1–type of conclusion “Some desserts are not fruits.” Conclusions I and II form Complementary Pair. Therefore, either Conclusion I or II follows.

    All buildings are flats. No flat is house.

    59. (2) All cups are bowls.

    All bowls are trays. I +E ⇒ O – type of conclusion “Some planes are not radars.”

    Warehouses

    A + E ⇒ E–type of conclusion “No building is house.” This is Conclusion I.

    55. (4) Some plane are ships.

    No ship is a radar.

    Rooms

    A + A ⇒ A–type of conclusion “All cups are trays.”

    Some trays are plates.

    All cars are radars.

    III According to venn diagram III, all warehouses are rooms. Therefore, all warehouses being rooms is a possibility. This is Conclusion I. Conclusion II is the Converse of the Conclusion (P). 63. (5) No puppy is a kitten.

    All kittens are cubs.

    No plate is spoon. No radar is a ship. A + E ⇒ E–type of conclusion “No car is a ship.” This conclusion I.

    I + E ⇒ O–type of conclusion “Some trays are not spoons.” 60. (2) All books are pens.

    Some doors are windows.

    I + A ⇒ I-type of Conclusion “Some rooms are godowns.” (P)

    A + E ⇒ E–type of conclusion “No dog is cat.” This is Conclusion II.

    I

    Cubs

    Puppies

    II

    or

    Puppies

    III According to venn diagram II, all puppies are cubs. Therefore, Conclusion I follows. (64–65) :

    No sea is a lake. Some lakes are rivers.

    All godowns are warehouses.

    E + I ⇒ O1– type of Conclusion “Some rivers are not seas.” (P)

    I + A ⇒ I–type of Conclusion “Some rooms are warehouses.” (R) 61. (3) Conclusion II is the Conlusion (Q). 62. (4) Venn diagrams of “Some rooms are warehouses”. : Warehouses

    or

    Rooms

    All buildings are flats. I + A ⇒ I–type of conclusion “Some pillars are flats.”

    or

    Some rooms are godowns.

    Ro

    58. (4) Some pillars are buildings.

    A + A ⇒ A–type of Conclusion “All stores are warehouses.” (Q)

    W ar e ho u se s

    No cow is cat.

    All godowns are warehouses.

    s

    All dogs are cows.

    All stores are godowns.

    om

    No cat is cow. A + E ⇒ E–type of conclusion “No rat is cow.”

    s

    Some rooms are stores. All stores are godowns.

    57. (5) All rats are cats.

    Cubs

    A + I ⇒ No conclusion (61–62) :

    All keys are doors.

    A + I ⇒ No conclusion Neither Conclusion I nor II follows.

    s

    ie

    A + A ⇒ A-type of conclusion “All locks are doors.”

    C

    ub

    pp

    All keys are doors.

    Some pens are desks.

    Pu

    56. (2) All locks are keys.

    E + A ⇒ O1 – type of Conclusion “Some cubs are not puppies.” Venn diagrams of “Some cubs are not puppies” :

    II

    I or

    BPRE–635

    Some lakes are rivers. All rivers are oceans. I + A ⇒ I – type of Conclusion “Some lakes are oceans”. (Q) 64. (1) Neither Conclusion I nor Conclusion II follows. 65. (5) Conclusion I is the Converse of the Conclusion (Q). (66—67) : (i) All cities are oceans → Universal Affirmative (A–type). (ii) Some villages are cities → Particular Affirmative (I–type).

    SYLLOGISM (iii) No ocean is a town → Universal Negative (E–type). (iv) Some oceans are not towns → Particular Negative (O–type). (66–67) : Some villages are cities.

    67. (1) Conclusion I is the Converse of Conclusion (P). Conclusion II is same as that of Conclusion (Q).

    No rocks is a hill. I + E ⇒ O – type of Conclusion “Some stones are not hills.”

    “Some villages are oceans.” (P)

    Some villages are oceans. No ocean is a town. I + E ⇒ O – type of Conclusion “Some villages are not towns.” (R) 66. (3) Venn diagrams of “Some villages are cities” :

    All hills are mountains. E + A ⇒ O1 – type of Conclusion “Some mountains are not rocks.” (69–71) : (i) All boxes are cartons → Universal Affirmative (A-type). (ii) Some stands are racks → Particular Affirmative (I-type). (iii) No rack is a box → Universal Negative (E-type). (iv) Some racks are not boxes → Particular Negative (O-type). 69. (3) Some stands are racks.

    No rack is a box. I + E ⇒ O–type of Conclusion “Some stands are not boxes”. Therefore, Conclusion I follows.

    Cities

    Villages

    No rack is a box. I

    All boxes are cartons.

    or

    E + A ⇒ O1–type of Conclusion “Some cartons are not racks”. Venn diagrams of “Some cartons are not racks” :

    Villages Cities

    II

    71. (5) All papers are mills.

    No rock is a hill.

    All cities are oceans.

    A + E ⇒ E – type Conclusion “No city is a town.” (Q)

    Some turtles are puppies. A + I ⇒ No Conclusion Conclusion I and Conclusion II form Complementary Pair. Therefore, either Conclusion I or Conclusion II follows.

    68. (5) Some stones are rocks.

    All cities are oceans. I + A ⇒ I – type of Conclusion

    No ocean is a town.

    70. (1) All kittens are turtles.

    C

    t ar

    Cartons

    s on

    Racks or

    All mills are factories. A + A ⇒ A–type of Conclusion “All papers are factories”. This is Conclusion II. (72–73) : Some perfumes are deodorants. All deodorants are colognes. I + A ⇒ I–type of Conclusion “Some perfumes are colognes”. (P) Some deodorants are perfumes. No perfume is a fragrance. I + E ⇒ O–type of Conclusion ‘‘Some deodorants are not fragrances”. (Q) Some colognes are perfumes.

    No perfume is a fragrance. I + E ⇒ O – type of Conclusion “Some colognes are not fragrances”. (R) 72. (4) Conclusion I is the Conclusion (P). 73. (3) Conclusion I is the Conclusion (Q). Venn diagrams of “Some colognes are not fragrances” : Colognes

    Racks Co

    or

    I

    II or

    Cities

    g lo

    s ne

    c an gr a Fr

    I

    es

    or

    or

    II

    Villages Cartons

    III Venn diagram II supports the Conclusion I.

    Racks

    III Venn diagram II supports the Conclusion II.

    BPRE–636

    Colognes

    Fragrances

    III Venn diagram II supports the Conclusion II.

    SYLLOGISM (74–78) : (i) All legumes are pulses → Universal Affirmative (A-type). (ii) Some lotions are perfumes → Particular Affirmative (I-type). (iii) No cream is a lotion → Universal Negative (E-type). (iv) Some creams are not lotions → Particular Negative (O-type). 74. (2)

    No cream is a lotion.

    Some lotions are perfumes. E + I ⇒ O1–type of Conclusion “Some perfumes are not creams”. Neither Conclusion I nor Conclusion II follows. (75-76) :

    I + A ⇒ I–type of Conclusion “Some curtains are blankets”.

    Venn diagram II supports the Conclusion I. Venn diagrams of “All legumes are pulses” :

    Pulses Legumes

    Pulses or Legumes

    IV

    A + A ⇒ A–type of Conclusion “All pillows are doormats”.

    I + A ⇒ I–type of Conclusion “Some curtains are doormats”. Venn diagrams of “Some curtains are blankets”:

    V

    Venn diagrams of “Some pulses are grains”: or

    or Pulses

    Grains or

    Pulses

    I + E ⇒ O–type of Conclusion “Some pulses are not crops”. (P) 75. (5) Venn diagrams of “Some pulses are not crops” :

    VI

    Grains

    VII

    Venn diagrams III contradicts the Conclusion II. Venn diagrams of “Some curtains are doormats”: After combining the Venn diagrams V and VII, we get: or

    The above venn diagrams show that “All pulses can never be crops”. Therefore, Conclusion II follows. 76. (3) Venn diagrams of “Some pulses are not crops” :

    Venn diagram IX supports the Conclusion II. 77. (5)

    I + A ⇒ I–type of Conclusion “Some rivers are waterfalls”. This is Conclusion II. 78. (4)

    or

    BPRE–637

    or

    Venn diagram V supports the Conclusion I. (79–83) : (i) All biscuits are juices → Universal Affirmative (A–type). (ii) Some juices are fruits → Particular Affirmative (I–type). (iii) No cookie is a biscuit → Universal Negative (E-type). (iv) Some cookies are not biscuits → Particular Negative (O–type).

    SYLLOGISM or

    (79–80) : Fruits

    No cookie is a biscuit.

    Lines

    Juices

    All biscuits are juices.

    Paints

    E + A ⇒ O1 – type of Conclusion “Some juices are not cookies.” (P)

    Cookies

    All biscuits are juices.

    III Venn diagram III contradicts the Conclusion I. Venn diagrams of “All strokes are paints” :

    Some juices are fruits. 79.

    A + I ⇒ No Conclusion (4) Venn diagrams of “Some juices are not cookies” : Juices

    coo

    s ice Ju

    kie s

    or

    VII

    80.

    Cookies

    Venn diagram VII supports the Conclusion II. (4) Venn diagrams of “All biscuits are juices” : Juices

    II

    Biscuits

    VIII

    Fruits

    III

    Juices

    The above venn diagrams show that “All juices can never be cookies”. Therefore, Conclusion I follows. Venn diagrams of “Some juices are fruits” : Juices ces Jui

    Fruits or

    IV

    Fruits

    Venn diagrams of “Some paints are colours” : Paints

    n ts Pai

    r lou Co

    s or

    Colours

    VII

    Colours

    X Venn diagram X supports the Conclusion I. Venn diagram VII supports the Conclusion II. (81–82) : Some lines are strokes. All strokes are paints.

    81.

    I + A ⇒ I – type of Conclusion “Some lines are paints.” (P) (5) Venn diagrams of “Some lines are paints” :

    Juices

    Paints

    VIII After combining Venn diagrams V and VII, we get : Paints, Strokes

    Colours

    Paints

    nt s or Pai

    Lines

    VI diagrams II and VI, we get :

    V

    VI

    Biscuits

    or

    After combining the venn

    Strokes

    or

    V

    Fruits

    or

    IX

    After combining Venn diagrams VI and IX, we get :

    Cookies

    Strokes

    IV

    or

    or

    Juices

    Paints

    Juices

    Biscuits

    I

    Paints

    Lines

    IX I

    II

    BPRE–638

    Venn diagram IX supports the Conclusion II.

    SYLLOGISM 82.

    (2) After combining the Venn diagrams V and VIII, we get :

    or

    Venn diagrams of “All elephants are tigers” :

    Years

    Colours

    Tigers

    Weeks

    Tigers

    Elephants

    Paints Strokes

    or

    V

    Elephants

    After combining Venn diagrams I and V, we get :

    Venn diagram X contradicts the Conclusion I. After combining Venn diagrams II and VIII, we get :

    V

    After combining Venn diagrams II and IV, we get :

    Years

    X

    IV

    Weeks Dolphins Tigers

    Days

    Elephants

    Colours Paints

    VI

    Lines

    XI Venn diagram XI contradicts the Conclusion II. 83.

    (3) Some weeks are months.

    All months are years. I + A ⇒ I – type of Conclusion “Some weeks are years.” Venn diagrams of “All days are weeks” : Weeks Days

    Weeks

    or Days

    I

    Years or

    III

    Venn diagram VI supports the Conclusion I. (84–88) : (i) All elephants are tigers → Universal Affirmative (A–type). (ii) Some dolphins are tigers → Particular Affirmative (I–type). (iii) No rain is a fog → Universal Negative (E–type). (iv) Some rains are not fogs → Particular Negative (O–type). (84–85) :

    All elephants are tigers. Some tigers are dolphins. A + I ⇒ No Conclusion 84. (2) Venn diagrams of “Some dolphins are tigers” : Dolphins

    s in ph l Do

    Tigers or

    Tigers

    II

    Venn diagrams of “ Some weeks are years” :

    eks We

    VI

    I

    II or

    Weeks

    Tigers

    Years

    Dolphins

    IV

    Venn diagram VI supports the Conclusion II. 85. (4) Neither Conclusion I nor Conclusion II follows. (86–87) : All hyenas are kittens.

    Some kittens are cows. A + I ⇒ No Conclusion 86. (1) Venn diagrams of “Some kittens are cows :

    tte Ki

    Cows

    ns

    Cows

    or

    Kittens

    II

    I or Kittens Cows

    III Venn diagrams of “All hyenas are kittens” :

    III

    BPRE–639

    SYLLOGISM Goats

    Hyenas

    Kittens Hyenas

    Cows

    or Kittens

    IV

    VI

    V

    After combining venn diagrams III and V, we get :

    After combining Venn diagrams II and V, we get : Kittens Cows

    Hyenas; Kittens

    Goats

    Venn diagrams I and II support the Conclusion I. (89–93): (i) All readings are journalsa Universal Affirmative (A–type). (ii) Some guns are steel Particular Affirmative (I–type). (iii) No place is laboratory Universal Negative (E–type). (iv) Some places are not laboraParticular Negatories tive (O–type). 89. (2) ‘Only a few’ is equivalent to ‘Some’. Therefore, the first Premise is Particular Affirmative (I–type).

    Some books are readings.

    Cows

    All readings are journals.

    VII

    VI Venn diagram VI supports the Conclusion I. Venn diagram II contradicts the Conclusions II. 87. (4) Venn diagrams of “Some kittens are cows” :

    After combining venn diagrams III and VI, we get : Goats Cows Kittens

    I + A ⇒ I-type of Conclusion ‘‘Some books are journals’’. Conclusion II is the Converse of the second Premise. 90. (1) Both the Premises are Particular Affirmative (I–type). No Conclusion follows from the two Particular Premises. Venn diagrams of ‘‘Some woods are trees’’ :

    Kittens

    s en tt i K

    Cows

    or

    I

    Cows

    II

    or

    VIII Venn diagram VIII contradicts the Conclusion I. Thus, neither Conclusion I nor Conclusion II follows.

    Woods

    Trees

    I

    88. (1) No fog is rain. Cows

    Woods

    All rains are smogs. E + A ⇒ O1–type of Conclusion “Some smogs are not fogs.” Venn diagrams of “Some smogs are not fogs” :

    Kittens

    III Venn diagrams of “Some cows are goats” :

    Goats

    or

    Goats

    or

    Fogs

    Fogs

    V or

    or

    Trees I

    II

    Woods

    or

    IV

    II

    Smogs

    s og Sm

    Cows Cows

    Trees

    Smogs

    Fogs

    III

    BPRE–640

    III

    SYLLOGISM Venn diagrams of ‘‘Some woods are stems’’ :

    Rivers

    Lakes

    Lakes

    Parks Woods

    Parks

    Stems I IV

    or

    or

    VI Parks

    Woods

    Venn diagram VI supports the Conclusion II. 92. (1) All places are venues.

    Stems Lakes

    No venue is centre.

    II V or

    Stems

    or Venn diagrams of ‘‘Some lakes are rivers’’.

    Woods Lakes

    VI After combining venn diagrams III and V, we get : Tree

    Rivers

    III

    Lakes Rivers

    Stems

    IV

    Veen diagram VII supports the Conclusion I.

    or

    Rivers

    91. (5) All gungles are parks.

    Lakes All parks are lakes. A + A ⇒ A-type of Conclusion ‘‘All jungles are lakes’’. This is the Conclusion I. Venn diagrams of ‘‘All parks are lakes’’ :

    93. (3) Some guns are steel.

    All steels are iron. I + A ⇒ I-type of Conclusion ‘‘Some guns are iron’’.

    All steels are iron.

    or

    Woods

    VII

    A + E ⇒ E- type of Conclusion ‘‘No place is centre’’. This is the Conclusion I.

    V After combining Venn diagrams I and V, we get :

    BPRE–641

    All iron are silvers. A + A ⇒ A- type of Conclusion All steels are silvers’’.

    Some guns are iron.

    All iron are silvers. I + A ⇒ I- type of conclusion ‘‘Some guns are silvers’’. Conclusion I and Conclusion II form Complementary Pair. Therefore, either Conclusion I or Conclusion II follows. (94–98) : Uni(i) All dresses are coats versal Affirmative (A–type). (ii) Some coats are ties Particular Affirmative (I–type). (iii) No tie is a shoe Universal Negative (E–type). (iv) Some ties are not shoes Particular Negative (O–type).

    SYLLOGISM (94–95) : ‘Only a few’ means ‘Some’.

    Storms

    I

    I + A ⇒ I–type of Conclusion “Some coats are ties”. (P)

    VI Tornados

    No tie is a shoe. A + E ⇒ E–type of Conclusion “No jacket is a shoe”. (Q)

    Venn diagram VI supports the Conclusion I. Venn diagrams of “Some coats are ties” :

    Storms

    III

    Some coats are ties.

    Venn diagrams of “Some tornados are rains” :

    Coats Ties

    Coats

    No tie is a shoe. I + E ⇒ O–type of Conclusion “Some coats are not shoes”. (R) 94. (5) Conclusion I is the Conclusion (P). Conclusion II is the Converse of the Conclusion (Q). 95. (5) Venn diagrams of “All dresses are coats” : Dresses

    Dresses

    or

    VII

    Ties

    VIII

    To

    a rn

    Tornados

    s do

    Rains

    or

    Rains

    or IV

    V

    Ties

    or

    Coats

    Rains

    Tornados

    IX

    or Coats

    I

    After combining venn diagrams I and IX, we get :

    VI

    II

    Venn diagrams of “Some tornados are floods” :

    Ties

    Venn diagrams of “Some coats are not shoes” :

    Coats Dresses

    Coats Shoes

    Coats

    II

    or

    All jackets are ties.

    Coats

    Tornados

    os

    Shoes

    All jackets are ties.

    or

    ad

    St or

    rn

    m

    s

    To

    Coats, Dresses

    Some coats are jackets.

    or

    ad

    Tornados

    os

    Floods

    or

    Floods

    Shoes

    X III

    IV

    Venn diagram X supports the Conclusion II.

    or

    96. (5) Some tornados are floods. Coats

    T

    n or

    VII

    VIII

    or Floods

    Tornados

    Shoes

    All floods are rains.

    V After combining venn diagrams II and IV, we get :

    I + A ⇒ I–type of Conclusion “Some tornados are rains”. Venn diagrams of “Some storms are tornados” :

    BPRE–642

    IX Venn diagrams of “All floods are rains” :

    SYLLOGISM Rains Floods

    Floods or

    Novels

    Novels N

    els ov

    or

    Pages

    Pages

    Books

    II

    I

    Novels or

    Rains

    X

    Books

    XI

    I

    After combining venn diagrams II and V, we get :

    II

    Venn diagrams of “Some novels are not theories” :

    or

    Novels

    Storms N

    or

    or

    Tornados

    els ov

    e Th

    Pages

    Novels

    ie

    Theories

    s

    III

    Rains

    Venn diagrams of “Some novels are scripts” :

    XII Venn diagram XII supports the Conclusion I. After combining venn diagrams X and XII, we get :

    IV

    or

    Novels N

    Novels

    els ov

    Scripts or

    Theories

    Scripts

    V IV

    Storms

    III

    V

    After combining venn diagrams II and IV, we get :

    or

    Tornados

    Novels, Books

    Rains Scripts

    Theories

    Floods Novels

    VI VI XIII Venn diagram XIII supports the Conclusion II.

    After combining venn diagrams II and VI, we get :

    (97–98) : No page is a book. Scripts

    All books are novels. E + A ⇒ O1–type of Conclusion “Some novels are not pages”. (P)

    Novels Pages

    Some novels are scripts.

    Venn diagram VI supports the Conclusion I. Venn diagrams IV and VI contradict the Conclusion II. (99–101) : (i) All blue are green Universal Affirmative (A-type). Par(ii) Some yellow are green ticular Affirmative (I-type). (iii) No English is Mathematics Universal Negative (E-type) (iv) Some English are not MathematParticular Negative (Oics type). 99. (4) Some Physics are English.

    No script is a theory. I + E ⇒ O–type of Conclusion “Some novels are not theories”. (Q) 97. (1) Venn diagrams of “Some novels are not pages” :

    VII No English is Mathematics. Venn diagram VII supports the Conclusion I. 98. (1) Venn diagrams of “All books are novels” :

    BPRE–643

    I + E ⇒ O-type of Conclusion “Some Physics are not Mathematics”. This is the Conclusion I.

    SYLLOGISM No English is Mathematics.

    Ropes

    Ropes Red, Yellow Green

    All Mathematics are Economics. E + A ⇒ O1-type of Conclusion “Some Economics are not English”. This is the Conclusion III.

    Rocks

    Blue

    or

    VI

    Rocks

    VII

    After combining venn diagrams I, IV and VII, we get :

    100. (1) All red are yellow.

    VIII Ropes, Rocks

    Some yellow are green. A + I ⇒ No Conclusion Venn diagrams of “All blue are green” :

    Venn diagram VIII supports the Conclusion I and contradicts the Conclusion III. 101. (2)

    Mountains Hills

    Some mountains are rocks.

    Blue

    Green

    All rocks are ropes. or

    Blue

    Green

    I II Venn diagrams of “Some yellow are green” :

    I + A ⇒ I-type of Conclusion “Some mountains are ropes”. Venn diagrams of “All hills are mountains” :

    Mountains

    Mountains

    Hills

    Green or

    III

    Green

    I

    II

    Venn diagrams of “Some rocks are mountains” :

    Green

    Yellow or

    Yellow

    Red

    VI

    or

    Rocks

    Yellow

    Mountains

    VII

    IV

    After combining venn diagrams I, IV and VII, we get :

    CLEAR YOU DOUBTS

    III

    Venn diagrams of “All red are yellow” :

    Red

    G et book s a n d m a g a z in e s o f K ira n P ra k a sh a n a t y ou r d o o rs te p e a s ily , lo g o n o u r w e b site :

    Mountains or

    V

    Yellow

    of Kiran Prakashan

    w w w .kiran p rakash an.co m

    Rocks IV

    Venn diagrams VIII supports the Conclusion III and contradicts the Conclusion II. ❐❐❐

    or

    Hills

    Yellow

    VIII

    Mountains or

    Rocks

    V

    Venn diagrams of “All rocks are ropes” :

    BPRE–644

    SYLLOGISM

    MODEL EXERCISES Directions (1-5) : In these questions, there are two statements followed by two conclusions A and B. You have to take the statements to be true even if they seem to be at variance from commonly known facts and then decide which of the conclusions logically follows. Choose (1) If only A follows (2) If only B follows (3) If both A and B follow (4) If neither A nor B follows (5) None of these 1. All chairs are houses. Some shoes are houses. A : Some chairs are shoes. B : Some shoes are chairs. 2. All men are dogs. All dogs are cats. A : All men are cats. B : All cats are men. 3. All men are married. Some men are educated. A : Some married are educated. B : Some educated are married. 4. All boys are girls. No girl is a father. A : All girls are boys. B : No boy is a father. 5. All boys are rivers. Some rivers are girls A : Some girls are boys. B : Some boys are girls. Directions (6-10) : In these questions, there are two statements followed by three conclusions numbered I, II and III. You have to take the given statements to be true even if they seem to be at variance from commonly known facts and then decide which of the given conclusion logically follows from the given statements. 6. Statements Some years are decades. All centuries are decades. Conclusions I. Some centuries are years. II. Some decades are years. III. No century is a year. (1) Only either I or III and II follow (2) Both II & III follow (3) Only I follows (4) Only II follows (5) None of these

    7. Statements All Shares are debentures. No debenture is an equity. Conclusions I. No equity is a share. II. Some debentures are shares. III. No share is an equity. ; (1) Only I follows (2) Only II follows (3) All follow (4) Only III follows (5) None of these 8. Statements All nets are jets. All jets are sets. Conclusions I. All jets are nets. II. Some sets are neither nets nor jets. III. No set is a net. (1) Only I follows (2) Only III follows (3) Only II follows (4) None follows (5) None of these 9. Statements All cities are towns. Some cities are villages. Conclusions I. All villages are towns. II. No village is a town. III. Some villages are towns. (1) Only III follows (2) Only I follows (3) Only II follows (4) None of the above (5) All of the above 10. Statements Some tables are chairs. Some chairs are beds. Conclusions I. Some tables are beds. II. No bed is either a chair or a table. III. All beds are chairs. (1) Only either 1 or II follows (2) Only III follows (3) Only II follows (4) None follows (5) All of the above

    BPRE–645

    Directions (11-20) : In these questions, you are given two statements A and B followed by two conclusions (I) and (II). You have to take the two statements to be true even if they seem to be at variance from commonly known facts. You are to decide which of the given conclusions definitely follows from the given statements. Indicate your answer as (1) If only conclusion I follows (2) If only conclusion II follows (3) If both I and II follow (4) If neither I nor II follows (5) None of these 11. Statements A : Some books are cars . Some cars are boxes B : Conclusions I. Some books are boxes II. Some boxes are books. 12. Statements A : All goats are parrots. B : All parrots are crows. Conclusions I. All crows are goats. II. All goats are crows. 13. Statements A : Some papers are pens. B : Some pencils are pens. Conclusions I. Some pens are pencils. II. Some pens are papery. 14. Statements B : Some kings are queens. A : All queens are beautiful. Conclusions , I. All kings are beautiful. II. All queens are kings. 15. Statements A : Some hens are crows. B : All crows are horses. Conclusions I. Some horses are hens II. Some hens are horses. 16. Statements A : All books are pencils B : All pencils are flowers. Conclusions I. All books are flowers. II. Some pencils are not books.

    SYLLOGISM 17. Statements A : Some books are pencils. B : Some pencils are pens. Conclusions I. Some books are pens ; II. Some pens are books. 18. Statements A : All horses are bullocks. B : All bullocks are parrots. Conclusions I. All horses are parrots. II. All parrots are horses. 19. Statements A : All flowers are leaves. B : Some leaves are birds. Conclusions I. Some birds are flowers. II. Some leaves are flowers. 20. Statements A : All windows are doors. B : No door is wall. Conclusions I. No windows is wall. II. No wall is door. Directions (21-30) : In these questions, two statements are given, followed by two inferences A and B, Assume the statements to be true, mark your answer as (1) If only inference I follows (2) If only inference II follows (3) If both I and II follow (4) If neither I nor II follows (5) None of these 21. All mothers are aunts. All aunts are ladies. So, I. All mothers are ladies. II. All aunts are mothers. 22. Some doctors are fools. Some fools are rich. So, I. Some doctors are rich. II. Some rich are doctors. 23. All goats are cows. Some goats are lambs. So, I. All goats are lambs. II. Some lambs are cows. 24. All pedestrians are poor. All poor are honest. So, I. All honest are pedestrians. II. All pedestrians are honest. 25. All rings are wings. All wings are kings. So, I. All rings are kings. II. All kings are rings.

    26. Some books are hooks. All books are fish. So, I. Some hooks are fish. II. Some fish are hooks. 27. All pens are guns. All guns are inkpots. So, I. All pens are inkpots. II. All inkpots are pens. 28. All P’s are Q’s. All Q’s are R’s. So, I. All P’s are R’s. II. Ail R’s are P’s. 29. Some words are sharp. All swords are rusty. So, I. Some rusty things are sharp. II. Some rusty things are not sharp. 30. All liquor is water. No water is bitter. So, I. No liquor is bitter. II. No bitter things is liquor. Directions (31-35) : In these questions, there are two statements followed by two possible inferences A and B. Assume the statements to be correct even if they vary with facts. Choose your answer as (1) If only inference A follows (2) If only inference B follows (3) If both the inference A and B follow (4) If neither inference A nor B follows (5) None of these 31. All graduates are chairs. All chairs are tables. A : All graduates are tables. B : All tables are graduates. 32. Every minister is a student. Every student is inexperienced. A : Every minister is inexperienced. B : Some inexperienced are students. ; . . 33. Some birds are elephants. Some elephants are white. A : Some birds are white. B : Some white are birds. 34. No bat is ball. No ball is wicket. A : No bat is wicket. B : All wickets are bats. 35. All fish are tortoise. No tortoise is a crocodile. A : No crocodile is a fish. B : No fish is a crocodile.

    BPRE–646

    Directions (36-39) : In these questions, two statements are followed by two conclusions numbered I and II. Assume the given statements to be true, even if they seem to be at variance with commonly known facts and then mark your answer as (1) If only conclusion I follows (2) If only conclusion II follows ‘ (3) If both the conclusions follow (4) If neither I nor II follows (5) None of these 36. Statements All players are smokers. Some smokers are wine-addicts. Conclusions I. All smokers are players. II. Some wine-addicts are smokers. 37. Statements All women are ministers. All ministers are simpleton. Condusions I. All women are simpleton. II. All ministers are simpleton. 38. Statements All cars are not trains. All cars are four-wheeled vehicles. Conclusions I. All trains are not four-wheeled vehicles. II. Some trains are four-wheeled vehicles. 39. Statements All jails are guest houses. All guest houses are comfortable. Conclusions I. All jails are comfortable. II. No jail is comfortable. Directions (40-44) : In each of the questions below, there are two or three statements followed by four conclusions numbered I, II, III and IV. You have to take the given statements to be true even if they seem to be at variance with commonly known facts and then decide which of the given conclusions logically follow(s) from the given statements. 40. Statements Some singers are rockers. All rockers are westerners. Conclusions I. Some rockers are singers. II. Some westerners are rockers. III. Some singers are westerners.

    SYLLOGISM IV. Some singers are not westerners. (1) I, II and III follow (2) 1, II and IV follow (3) II, III and IV follow (4) I, III and IV follow (5) None of these 41. Statements No Indian is an Asian. Some Americans are Indians. Conclusions I. Some Indians are not Asians. II. Some Asians are not Americans. III. Some Americans are not Asians. IV. All Americans are Asians. (1) Either III or IV follows (2) Either I or II follows (3) I and III follow (4) Either II or III follows (5) None of these 42. Statements Some charts are darts. All darts are carts. Some carts are smarts. Conclusions I. Some charts are carts. II. Some carts are darts. III. Some darts are smarts. IV. Some smarts are charts. (1) I and III follow (2) II and III follow (3) I and II follow (4) I, III and IV follow (5) None of these 43. Statements Some big are small. No small is large. Some large are tiny. Conclusions I. Some large are not big. II. No big is large. III. Some small are not tiny. IV. Some big are not tiny. (1) Only I follows (2) Only II follows (3) Only III follows (4) None follows (5) All of these 44. Statements No killer is a sweater. No jacket is a sweater. Some jackets are roses.

    Conclusions I. Some roses are sweaters. II. Some roses are not sweaters. IlI. No killer is a jacket. IV. Some jackets are killers. (1) Either I or II and III follow (2) Either III or IV and II follow (3) Either II or III follows (4) Either I or II and either III or IV follow (5) None of these Directions (45-48) : Each of these questions has three groups of statements marked A, B, C. Identify the group(s) in which the information given is logical. 45. A. To forgive is divine. Divine facts are rare. Forgiveness is rare. B. Pepsi contains added flavour. Coke contains permitted colours. Pepsi and Coke are cold drinks. C. Some beer is wine and some beer is vodka. All wine is vodka. All beer must be vodka or wine. (1) A and C (2) A and B (3) only A (4) only C (5) None of these 46. A. Sensex crashes on Monday, Wednesday and Friday. Nasdaq rises high on Tuesday, Thursday and Saturday. Sensex follows the Nasdaq movement. B. Mt. Everest, the highest peak, is taller than the second highest peak in China. Mt. Everest may be higher than Mt. Sunfest. C. On the highway, going from East to West, town A comes after town B. Town C comes after town D. Towns C and A come after town B. (1) only B (2) B and C (3) only C (4) A and C (5) None of these 47. A. Some substances are crystalline. Marble is crystalline. Marble is a substance. B. All greyhounds are dogs. Some dogs are cows. Some greyhounds are dogs. C. All locks are keys. Some keys do not open. Some locks do not open.

    BPRE–647

    (1) (3) (5) 48. A.

    A and C (2) B and C only A (4) None of these All of these All beautiful things are sad. She is beautiful. She is sad. B. All nice things are flat. TVs are flat. TVs are nice things. C. Potatoes are stems. All stems are fruits. Potatoes are fruits. (1) Only C (2) A and B (3) Only A (4) A and C (5) None of these Directions (49-52) : In each of these questions, two statements are given followed by two conclusions numbered I and II. Consider the two given statements to be true even if they seem to be absurd. Mark answer as (1) if only conclusion II follows (2) if only conclusion I follows (3) if either conclusion I or II follows (4) if neither conclusion I nor II follows 49. Statements No coin is a dollar. Red token is a coin. Conclusions I. Red token is not a dollar. II. Red token may not be a dollar. 50. Statement All bulbs are birds. Some birds are butterflies. Conclusions I. All butterflies are bulbs. II. Some bulbs are butterflies. 51. Statements All lamps are poles. Some poles are pipes. Conclusions I. Some lamps are pipes. II. Some pipes are poles. 52. Statement Some tanks are ponds. Some ponds are buckets Conclusions I. Some buckets are ponds. II. Some tanks are buckets. Direction (53-57) : In each of the questions below are given three statements followed by two Conclusions numbered I and II. You have to take the given statements to be true even if they seem to be at variance from commonly known facts. Read all the Conclusions and then decide which of the given Conclusions logically follows from the given statements disregarding commonly known facts.

    SYLLOGISM Give answer (1) if only Conclusion I follows Give answer (2) if only Conclusion II follows Give answer (3) if either Conclusion I or Conclusion II follows Give answer (4) if neither Conclusion nor Conclusion II follows Give answer (5) If both the Conclusions I and II follow (53–54) : Statements : All sticks are plants. All plants are insects. All insects are amphibians. 53. Conclusions : I. All amphibians are sticks. II. All plants are amphibians. 54. Conclusions : I. At least some amphibians are insects II. All sticks are insects. 55. Statements : All apartments are huts. No hut is a building. All buildings are cottages. Conclusions : I. No cottage is an apartment. II. Some cottages are apartments. (56–57) : Statements : Some stars are moons. All moons are planets. No planet is universe. 56. Conclusions : I. At least some planets are stars. II. No moon is universe. 57. Conclusions : I. All moons being stars is a possibility. II. No universe is a star. Directions (58–62) : In each of the following questions, two/three statements followed by two conclusions numbered I and II have been given. You have to take the given statements to be true even if they seem to be at variance from commonly known facts and then decide which of the conclusions logically follows from the statements disregarding commonly known facts.

    Give answer (1) if only Conclusion I follows. Give answer (2) if only Conclusion II follows. Give answer (3) if either Conclusion I or Conclusion II follows. Give answer (4) if neither Conclusion I nor Conclusion II follows Give answer (5) if both the Conclusion I and Conclusion II follow (58-59) : Statements: No number is a digit. All digits are fractions. Some fractions are integers. 58. Conclusions : I. Atleast some digits are integers. II. All numbers being integers is a possibility. 59. Conclusions : I. All fractions can never be numbers. II. Some integers are definitely not numbers. 60. Statements: Some seconds are minutes. All minutes are hours. Conclusions : I. Some seconds are hours. II. All seconds are hours. (61–62) : Statements : All flaws are errors. Some errors are mistakes. Some mistakes are blunders. 61. Conclusions : I. All flaws can never be mistakes. I. Atleast some blunders are errors. 62. Conclusions : I. All flaws being blunders is a possibility. II. No error is a blunder. Direction (63–68) : In each of the following questions, two or three statements followed by two Conclusions numbered I and II have been given. You have to take the given statements to be true even if they seem to be at variance from the commonly known facts and then decide which of the given Conclusions logically follows from the given statements disregarding commonly known facts. Give answer (1) if both the Conclusion I and Conclusion II follow

    BPRE–648

    Give answer (2) if either Conclusion I or Conclusion II follows Give answer (3) if neither Conclusion I nor Conclusion II follows Give answer (4) if only Conclusion I follows Give answer (5) if only Conclusion II follows (63–64) : Statements Some pictures are images. All images are memories. Some memories are sketches. 63. Conclusions I. Aleast some pictures are memories. II. Some memories are definitely not pictures. 64. Conclusions I. All images are sketches. II. All pictures being sketches is a possibility. (65–66) : Statements All mountains are peaks. Some peaks are rivers. No mountain is a valley. 65. Conclusions I. All rivers can never be mountains. II. All mountains are rivers. 66. Conclusions I. No valley is a peak. II. Atleast some valleys are rivers. 67. Statements Some hours are minutes. Some hours are seconds. Conclusions I. Atleast some seconds are minutes. II. No second is a minute. 68. Statements All cars are trucks. All trucks are vehicles. Conclusions I. All vehicles ate trucks. II. All cars are vehicles. Directions (69 – 73) : In each of the following questions, two/three statements followed by two conclusions numbered I and II have been given. You have to take the given statements to be true even if they seem to be at variance from commonly known facts and then decide which of the given conclusions logically follows from the given statements disregarding commonly known facts.

    SYLLOGISM Give answer (1) if only Conclusion I follows Give answer (2) if neither Conclusion I nor Conclusion II follows Give answer (3) if only Conclusion II follows Give answer (4) if both Conclusion I and Conclusion II follow Give answer (5) if either Conclusion I or Conclusion II follows 69. Statements No factory is an industry. All workshops are industries. Some plants are workshops. Conclusions I. No workshop is a factory. II. Atleast some plants are industries. 70. Statements Some sands are particles. Some particles are glasses. Conclusions I. Some glasses are definitely not particles. II. Some glasses being sands is a possibility. 71. Statements Some movies are films. No film is a show. All shows are pictures. Conclusions I. Atleast some pictures are films. II. No show is a movie. (72 – 73) : Statements Some actors are singers. All singers are dancers. Some dancers are players. 72. Conclusions I. All actors being dancers is a possibility. II. All dancers are singers. 73. Conclusions I. Atleast some dancers are actors. II. No player is an actor. Directions (74–75) : In each of the following questions, three statements are given followed by three Conclusions numbered I, II and III. You have to take the given statements to be true even if they seem to be at variance from commonly known facts and decide which of the following Conclusions logically follow(s) from the statements. Give answer (1) if only Conclusion I and Conclusion II follow

    Give answer (2) if only Conclusion II and Conclusion III follow Give answer (3) if all the Conclusions I, II and III follow Give answer (4) if only Conclusion I and Conclusion III follow Give answer (5) if none of the Conclusions follows 74. Statements :

    EXPLANATIONS 1. (4) Middle term houses is not distributed in the statements hence, none of the conclusion follows. 2. (1) From the figure drawn, conclusion A follows. C D

    All alphabet are symbols. Only a few letters are clauses.

    M

    Only a few symbols are clauses. Conclusions : I. Some letters being alphabet is a possibility. II. Some alphabet are not clauses. III. Some clauses can be alphabet. 75. Statements :

    3. (3) From the figure, we can say that some men are educated and some educated are men. hence, both of them follow. Married Educated

    Only a few cats are pets. No bird is a pet.

    Men

    All tigers are cats. Conclusions : I. Some birds being tigers is a possibility. II. Some pets are not cats. III. Some cats being birds is a possibility.

    4. (2) Conclusion A does not follow. Conclusion B that “No boy is a father” is valid from the figure. Hence, only B follows. G B

    F

    SHORT ANSWERS 1.(4)

    2.(1)

    3.(3)

    5.(4)

    6.(1)

    7.(3)

    8.(3)

    9.(1)

    10.(4)

    11.(4)

    12.(2)

    13.(3)

    14.(4)

    15.(3)

    16.(1)

    17.(4)

    18.(1)

    19.(2)

    20.(3)

    21.(1)

    22.(2)

    23.(2)

    24.(2)

    25.(1)

    26.(3)

    27.(1)

    28.(1)

    29.(1)

    30.(3)

    31.(1)

    32.(3)

    33.(4)

    34.(4)

    35.(3)

    36.(2)

    37.(3)

    38.(4)

    39.(1)

    40.(1)

    41.(3)

    42.(3)

    43.(4)

    44.(2)

    45.(3)

    46.(1)

    47.(4)

    48.(4)

    49.(2)

    50.(4)

    51.(1)

    52.(2)

    53.(2)

    54.(5)

    55.(3)

    56.(5)

    57.(1)

    58.(5)

    59.(4)

    60.(1)

    61.(2)

    62.(1)

    63.(4)

    64.(5)

    65.(3)

    66.(5)

    67.(2)

    68.(5)

    69.(4)

    70.(3)

    71.(2)

    72.(1)

    73.(1)

    74.(4)

    75.(4)

    BPRE–649

    4.(2) 5. (4) Middle terms “rivers” is not distributed in the Premises. Hence, none of conclusions of mediate inference follows. R B

    G

    Conclusion A–X B–X

    Hence hove conclusion follows. 6. (1) Middle terms decades is not distributed hence, no definite conclusion of mediate inference can be drawn. However, conclusions (I) and (III) make a complementary pair or inference, hence either (I) or (III) follows. Conclusion (II) is immediate inference drawn from statement I. Hence, option (1) is the correct answer.

    SYLLOGISM 7. (3) All shares are debentures can be converted into some debentures are shars. Hence, conclusion (II) follows. Now middle term is distributed in the statement, hence mediate inference can be drawn. Conclusion (I) & and (II) are negative conclusions which follow if one of the statements in negative. Hence, all follow. 8. (3) Conclusion I does not follow because there are some jets which are not nets. Conclusion II is true from the given figure but it is not true if all the circles concide hence it does not follow. Conclusion III does not follow. Hence, none of the conclusions follow. S J N

    14. (4) Conclusion (II) is immediate inference and does not follow from the statement (1). Conclusion (I) is mediate inference and does not follow. 15. (3) From the figure, both the conclusions “Some horses are hens” and “some hens are horses” are valid. Horses C

    H

    V C

    Conclusion (III) follows as “some villages are towns.” Hence, option (1) is the correct answer. 10. (4) Middle term “chairs” is not distributed. Hence no conclusions of mediate inference follows. Moreover, conclusion III is immediate inference and does not follow from statement 2. 11. (4) Both the conclusions are mediate inference and do not follow as middle term “cars” is not distributed in the statements. 12. (2) Only conclusion (II) follows. 13. (3) Conclusion (I) is immediate inference drawn from statement (II). Conclusion (II) is immediate inference drawn from statement A. Both the conclusions are valid.

    C L G

    24. (2) From the figure, conclusion I does not follows conclusion follows from the statements. Honest Poor

    16. (1) Only conclusion (I) follows. 17. (4) Middle term “pencils” is not distributed. Hence, none of conclusions of mediate inference follows. 18. (1) From the figure, only conclusion (I) “All horses are parrots’ is valid.

    Ped

    25. (1) Only conclusion (I) follows. 26. (3) From the figure, it can easily been concluded that some books are fish and some fish are books. F H

    P B

    9. (1) From the figure, it is clear that conclusion (I) does not follow. Also, conclusion (II) does not follow. T

    23. (2) Conclusion (I) is immediate inference and does not follow form statement. Conclusion (II) is mediate inference and follows from the diagram.

    B

    H

    19. (2) Conclusion (II) immediate inference drawn from statement A and follows. Conclusion (I) in mediate inference but does not follow because middle term leaves is not distributed in the statement. 20. (3) Both the conclusions are valid from the statements. D W

    Wall

    21. (1) Conclusion (I) “All mothers are ladies” follow from statement. Conclusion (II) does not follow. L A M

    27. (1) Only conclusion (I) follows. 28. (1) Only conclusion (I) follows. 29. (1) If both the statements are positive, Conclusion is always Positive, hence, conclusion (II) is ruled out. From the figure, Conclusion (I) “Some rusty things are sharp” follow. Rusty Sharp Sword

    30. (3) From the figure drawn on the basis of statements, both the conclusions follow. W B L

    31. (1) This can be solved easily with the help of venn-diagrams. tables

    22. (2) Middle term “fools” is not distributed in the statements, hence none of the conclusions (mediate inference) follows from the statements.

    BPRE–650

    Chains graduate

    SYLLOGISM Clearly, statement A follows and B does not follow. 32. (3) From the diagram, it is clear that every minister is inexperienced and some inexperienced are students. Inexperienced Students Minister

    33. (4) None of the conclusions follows as middle term elephants is not distributed in the statements. 34. (4) As per rule of syllogism, if both the statements are negative, no mediate inference follows. This is also clear from the venn-diagram. Bat

    Ball

    Bat

    Ball

    Wicket (II)

    Wicket (I)

    Conclusion (1) is true from figure (I) but false from (II) hence does not follow. Conclusion (II) is false from both the figures. 35. (3) If one of the statements is negative, conclusion will be negative both the conclusions follow. 36. (2) Conclusion (I) is immediate inference and does not follow from statements I. Conclusion (II) is immediate inference drawn from statement II as I type proposition can be converted into I type. 37. (3) Clearly, both the conclusion follow from the statements. 38. (4) None of the conclusions is definitely true. FWW

    FWW T

    T

    C

    C

    39. (1) Clearly all jails are comfortable. Hence, only conclusion I follow. GH J

    40. (1) I follows from some singers are rockers and II follows from all rockers are westerners combining both the statements, we get the conclusion III. 41. (3) Some Americans are Indians and no Indian is an Asian implies that some Americans are not Asians. Also, No Indian is an Asian implies that some Indian are not Asians. 42. (3) Some charts are darts + All darts are Carts ⇒ Some charts are carts. Hence, I follows. Again, All darts are Carts. => Some carts are darts. Hence, II follows. All darts are carts + some carts are smarts, gives no conclusion. Hence, III does not follows, Similarly, we see that IV also does not follow. 43. (4) None of the given conclusion follows. 44. (2) Some Jackets are roses ⇒ Some roses are .Jackets. Now some roses are jackets + No jacket is a sweater = Some roses are not sweaters. Hence, II follows. Also, III and JV form a complementary pair ie, one of them is always true. Hence, either III or IV follows. 45. (3) Only A has statements which are logically related. 46. (1) Only B is true. 47. (4) According to question, Crystalline

    Substance

    Some locks may or may not open. Hence (C) is false only option (B) follows but none of the alter natives has this option. 48. (4) According to question, Sad

    She

    Beautiful things

    Clearly, (1) follows. ie. all beautiful things are sad + she is beautiful = she is sad. Nice things

    Flat

    Clearly, TV may or may not be nice things.

    Potatoes Stems Fruits

    Potatoes are stems + all stems are fruits ⇒ Potatoes are fruits. Hence, A and C follows. 49. (2) According to question,

    Marble

    Red token

    (1) Marble may or may nob be a substance. Hense, (1) is false. (2) All greyhounds are dogs hence some greyhounds will be dogs. Hence, (2) is true. (3)

    Dollar

    Coin

    I. Red token is not a dollar. II. Red token may not be a dollar. Only conclusion I follows. 50. (4) According to question,

    C

    Locks

    Do not open

    Bulbs

    Keys Birds

    BPRE–651

    Butterflies

    SYLLOGISM I. All butterflies are bulbs II. Some butterflies are bulbs. Neither conclusion I nor II follows. 51. (1) According to question,

    54. (5) Conclusion I is the Converse of the third Premise. Conclusion (P) is the Conclusion II. 55. (3) All apartments are huts.

    No hut is a building. Lamps

    A + E ⇒ E – type of Conclusion “No apartment is a building”.

    Pipes

    No hut is a building.

    Poles

    No number is a digit.

    I. Some lamps are poles. II. Some poles are pipes. Only conclusion II follows. 52. (2) According to question,

    Tanks

    Ponds

    (58–62) : (i) All digits are fractions → Universal Affirmative (A–type). (ii) Some fractions are integers → Particular Affirmative (I–type). (iii) No number is a digit → Universal Negative (E–type). (iv) Some numbers are not digits → Particular Negative (O–type). (58–59) :

    All buildings are cottages. E + A ⇒ O1 – type of Conclusion “Some cottages are not huts”. Conclusions I and II form Complementary Pair. Therefore, either Conclusion I or Conclusion II follows. (56–57) :

    Buckets

    Some stars are moons. I. Some buckets are ponds. II. Some tanks are buckets. Only conclusion I follows. (53-57) :

    All moons are planets. I + A ⇒ I – type of Conclusion “Some stars are planets”. (P)

    All digits are fractions. E + A ⇒ O1-type of Conclusion “Some fractions are not numbers.” 58. (5) Venn diagrams of “All digits are fractions” :

    Fractions

    Digits or

    Digits

    Fractions

    All moons are planets.

    (i) All sticks are plants → Universal Affirmative (A - type) (ii) Some stars are moons

    I

    No planet is universe. →

    Particular Affirmative (I - type) (iii) No hut is a building → Universal Negative (E - type) (iv) Some huts are not buildings → Particular Negative (O - type) (53–54) :

    All sticks are plants. All plants are insects. A + A ⇒ A – type of Conclusion “All sticks are insects”. (P)

    All plants are insects.

    A + E ⇒ E – type of Conclusion “No moon is universe”. (Q)

    Some stars are planets. No planet is universe. I + E ⇒ O – type of Conclusion “Some stars are not universe”. (R) 56. (5) Conclusion (P) is the Converse of the Conclusion I. Conclusion (Q) is the Conclusion II. 57. (1) Venn diagrams of “Some stars are moons”.

    Stars Moons or

    Stars

    or

    Fractions

    ers eg t n I

    III

    IV Fractions

    or

    Integers

    V

    I

    II or

    All sticks are insects.

    Moons

    All insects are amphibians.

    Stars

    A + A ⇒ A – type of Conclusion “All sticks are amphibians”.(R) 53. (2) Conclusion (Q) is the Conclusion II.

    Integers

    s ion ac t r F

    Moons

    All insects are amphibians. A + A ⇒ A – type of Conclusion “All plants are amphibians”. (Q)

    II

    Venn diagrams of “Some fractions are integers” :

    After combining the Venn diagrams II and IV, we get :

    Integers Digits Fractions

    III Venn diagram II supports the Conclusion I.

    BPRE–652

    SYLLOGISM Therefore, Conclusion I follows. Venn diagrams of “Some fractions are not numbers” :

    ns tio c a Fr

    Mistakes Errors Numbers

    rs be m or Nu

    or

    VI

    After combining Venn diagrams II and V, we get : It can be illustrated as :

    Fractions

    Numbers

    Mistakes

    Fractions Integers

    or

    Errors

    Fractions or

    Numbers

    Flaws

    Numbers

    VII VIII

    VI Hence, Conclusion I does not follow. Venn diagrams of “Some mistakes are blunders’’ :

    Therefore, Conclusion II does not follow. 60. (1)

    s un de r

    “Some seconds are hours.” This is Conclusion I. (61–62) :

    Mistakes

    M

    Numbers

    All minutes are hours. I + A ⇒ I-type of Conclusion

    is ta ke s

    Some seconds are minutes.

    or

    Blunders

    Bl

    After combining the Venn diagrams, IV and VII, we get :

    VII

    VIII

    All flaws are errors. Blunders

    Therefore, Conclusion II follows. 59. (4)

    Some errors are mistakes. Fractions

    I + A ⇒ No Conclusion 61. (2) Venn diagram of “All flaws are errors ’’ :

    IX

    Numbers Errors

    Errors

    Flaws

    or

    I

    Errors

    II

    Mistakes

    Venn diagrams of “Some errors are mistakes” :

    ke s

    Mistakes

    is ta

    or

    X

    M

    Therefore, Conclusion I does not follow.

    Blunders

    Errors

    Er ro rs

    Numbers

    After combining Venn diagrams IV and VIII, we get :

    Flaws It can be illustrated as :

    Fractions

    Mistakes or

    III

    BPRE–653

    IV

    Therefore, Conclusion II follows.

    SYLLOGISM 62. (1) After combining the Venn diagrams VI and IX given in the answer to Question Number 44, we get :

    Blunders

    Venn diagram II contradicts the Conclusion II. Therefore, only Conclusion I follows. 64. (5)

    Sketches Memories Pictures

    All images are memories.

    Mistakes Flaws

    Some memories are sketches. A + I ⇒ No Conclusion Venn diagrams of “Some pictures

    Therefore, Conclusion I follows. (63–68) : (i) All mountains are peaks →

    Pictures

    es

    tu r

    Memories

    or

    M

    em

    or i

    Pic



    Particular Affirmative (I–type).

    es

    Universal Affirmative (A–type). (ii) Some peaks are rivers

    VII

    are memories” :

    (iii) No mountain is a valley →

    I

    Universal Negative (E–type).

    II

    This diagram supports Conclusion II. Therefore, only Conclusion II follows. (65–66) :

    No valley is a mountain.

    or

    (iv) Some mountains are not valleys

    All mountains are peaks.

    → Particular Negative (O–type). (63–64) :

    E + A ⇒ O1– type of Conclusion “Some peaks are not valleys.” 65. (3) Venn diagrams of “All mountains are peaks” :

    Memories

    Some pictures are images. Pictures All images are memories.

    Peaks

    I + A ⇒ I–type of Conclusion “Some pictures are memories.” (P) 63. (4) Conclusion I is the Conclusion (P). Venn diagrams of “Some pictures are memories” :

    III

    Peaks or

    Mountains

    Mountains

    Venn diagrams of “Some memories are sketches” :

    I s

    ch es

    or ie em

    Venn diagrams of “Some peaks Sketches

    are rivers” :

    Sk

    et

    M

    Memories

    or

    Me

    Pi

    mo

    ct

    rie

    ur

    s

    es

    Pictures

    or

    II

    Memories

    I

    II

    IV

    Peaks

    V

    or

    Rivers or

    Peaks

    Rivers

    or

    Sketches

    Memories

    Memories

    III

    IV or

    Rivers

    Pictures VI After combining venn diagrams III and VI, we get :

    III

    BPRE–654

    Peaks

    V

    SYLLOGISM After combining venn diagrams II and IV, we get :

    I It implies that (i) Some particles are glasses. (ii) Some particles are not glasses. Therefore, Conclusion I does not follow.

    Rivers Peaks

    Peaks, Mountains

    Valleys

    Rivers

    Glasses Particles

    VII VI This diagram contradicts Conclusion I. 66. (5) Venn diagrams of “Some peaks are not valleys” :

    or

    Va lle

    ys

    Pe ak s

    Peaks

    I

    Valleys

    II or

    This diagram supports Conclusion II. 67. (2) Both the Premises are Particular Affirmative (I–type). No Conclusion follows from the two Particular Premises. Conclusion I and Conclusion II form Complementary Pair. Therefore, either Conclusion I or Conclusion II follows. 68. (5) All cars are trucks.

    Glasses

    All trucks are vehicles.

    69. (4)

    All workshops are industries.

    Peaks

    Particles

    A + A ⇒ A–type of Conclusion “All cars are vehicles.” This is Conclusion II.

    Valleys

    II It implies that : All particles are glasses.

    III It implies that : All glasses are particles. Venn diagrams of “Some sands are particles”:

    No industry is a factory. III Venn diagrams of “Some peaks are rivers” :

    Some plants are workshops.

    IV or

    All workshops are industries.

    Particles

    Peaks Rivers or

    Peaks

    IV

    Rivers

    V or

    Rivers

    Particles

    A + E ⇒ E - type of Conclusion “No workshop is a factory”. This is Conclusion I.

    I + A ⇒ I-type of Conclusion “Some plants are industries”. This is Conclusion II. Thus, both Conclusions I and II follow. 70. (3) Venn diagrams of “Some particles are glasses”:

    Sands

    V or

    Sands

    Peaks

    Particles VI

    Glasses

    After combining venn diagrams II and VI, we get :

    VI

    BPRE–655

    SYLLOGISM After combining the diagrams III and VI, we get

    74. (4) All alphabet are symbols.

    Some symbols are clauses.

    Sands Particles

    Letters Clauses Symbols

    A + I ⇒ No Conclusion Venn diagrams of “All alphabet are symbols” :

    Alphabet

    Glasses Symbols Alphabet

    Therefore, some glasses being sands is a possibility. Thus, Conclusion II follows. 71. (2) Some movies are films.

    Alphabet

    or

    IX Symbols

    I

    II

    Venn diagram IX supports the Conclusions I and III and contradicts the Conclusion II. 75. (4) No bird is a pet.

    Venn diagrams of “Some letters are clauses” :

    Some pets are cats. Letters Cl au ses or

    Le tt er s

    No film is a show. I + E ⇒ O - type of Conclusion ‘‘Some movies are not shows.’’

    Clauses

    No film is a show.

    E + I ⇒ O1–type of Conclusion “Some cats are not birds”. Venn diagrams of “Some cats are not birds” : Cats

    E + A ⇒ O1 - type of Conclusion “Some pictures are not films”. Thus, neither Conclusion I nor II follows.

    IV or Clauses

    or

    Birds

    I

    II

    or

    (72 – 73) :

    Letters

    Some actors are singers.

    Cats

    All singers are dancers.

    Birds

    V Venn diagrams of “Some symbols are clauses” :

    Cl au ses or

    Sy m bo ls

    I + A ⇒ I - type of Conclusion “Some actors are dancers”. (P). 72. (1) Conclusion (P) is the Conclusion I. The Converse of “All singers are dancers” would be : “Some dancers are singers”. Therefore, Conclusion II does not follow. 73. (1) Conclusion I is the Converse of the Conclusion (P). (74–75) : (i) All alphabet are symbols Universal Affirmative (A–type). (ii) Only a few letters are clauses Particular Affirmative (I– type). [Only a few = Some] Universal (iii) No bird is a pet Negative (E–type). (iv) Some birds are not pets Particular Negative (O–type).

    Bi rds

    Ca ts

    III

    All shows are pictures.

    III Venn diagrams of “All tigers are cats” :

    Symbols

    Cats

    Clauses

    Tigers

    VII

    IV

    Cats

    or Tigers

    VI or Clauses Symbols

    VIII After combining the venn diagrams I, IV and VIII, we get :

    BPRE–656

    V

    After combining the venn diagrams II and V, we get : Cats, Tigers Birds

    VI

    Venn diagram VI supports the Conclusion I and Conclusion III. ❑❑❑

    PROBLEM SOLVING

    12

    PROBLEM SOLVING

    QUESTIONS FROM 1999 TO 2010 ARE AVAILABLE ONLINE NATIONALISED BANKS & IBPS PO/MT/SO EXAMS Directions (1-7) : Study the following information to answer the given questions : (Indian Bank PO Exam. 02.01.2011 (Ist sitting)

    Six lectures are scheduled in a week starting from Monday and ending on Sunday of the same week. Computer Science is not on Tuesday or Saturday. Psychology is immediately after Organisational Behaviour. Statistics is not on Friday and there is one day gap between Statistics and Research Methods. One day prior to the schedule of Economics there is no lecture (as that day is the ‘off’ day and Monday is not the ‘off’ day.) 1. Which of the following is the last lecture scheduled ? (1) Statistic (2) Research Methods (3) Psychology (4) Cannot be determined (5) None of these 2. If Wednesday is the ‘off ’ day, the code would be 2 – 4, if Thursday was the ‘off ’ day, the code would be 3 – 3. Taking into account the ‘off’ day which of the following code is correct ? (1) 2 – 4 (2) 3 – 3 (3) 4 – 2 (4) Cannot be determined (5) None of these 3. Which lecture is scheduled on Friday ? (1) Economics (2) Psychology (3) Computer Science (4) Cannot be determined (5) None of these 4. How many lectures were scheduled between Economics and Psychology ? (1) One (2) Two (3) Three (4) Cannot be determined (5) None of these

    5. Which day is Computer Science scheduled ? (1) Monday (2) Wednesday (3) Thursday (4) Cannot be determined (5) None of these 6. Which day is the ‘off ’ day ? (1) Tuesday (2) Wednesday (3) Friday (4) Cannot be determined (5) None of these 7. If someone wants to attend only two lectures out of Psychology, Research Methods and Computer Science but wants the two days to be successive (one after the other) then which lecturecombination may be selected ? (1) Research Methods, Computer Science (2) Psychology, Computer Science (3) Psychology, Research Methods (4) Any two of the three is possible (5) With the condition of successive day it is not possible Directions (8–12) : Study the following information carefully and answer the given questions : (Corporation Bank PO Exam. 16.01.2011)

    One of the seven subjects, viz., Maths, Zoology, Botany, Chemistry, Physics, English and Statistics is taught on one day in a week starting from Monday and ending on Sunday. Chemistry is taught on Thursday. English is taught the day immediately next to the day when Zoology is taught. English is neither taught on Tuesday nor Saturday. Only one lecture is held between Chemistry and Botany. Two lectures are scheduled between Maths and Zoology. Statistics is neither taught on Monday nor Sunday. 8. On which of the following days is Physics taught ? (1) Monday (2) Tuesday

    BPRE–657

    (3) Wednesday (4) Thursday (5) Friday 9. How many subjects are taught between Botany and Zoology ? (1) None (2) One (3) Two (4) Three (5) Four 10. Which of the following subjects is taught on Saturday ? (1) Botany (2) Statistics (3) Zoology (4) Maths (5) Physics 11. On which of the following days is statistics taught ? (1) Tuesday (2) Wednesday (3) Thursday (4) Friday (5) Cannot be determined 12. If Statistics is related to Zoology and Physics is related to Botany in a certain way, then to which of the following would Chemistry be related to, following the same pattern ? (1) Maths (2) Statistics (3) Physics (4) English (5) Cannot be determined Directions (13–15) : Study the following information to answer the given questions : (Corporation Bank PO Exam. 16.01.2011)

    A building has seven floors numbered one to seven, in such a way that the ground floor is numbered one, the floor above it, number two and so on such that the topmost floor is numbered seven. One out of seven people viz. A, B, C, D, E, F and G lives on each floor. A lives on fourth floor. E lives on the floor immediately below F’s floor. F does not live on the second or the seventh floor. C does not live on an odd numbered floor. B does not live on a floor immediately above or below C’s floor. D does not live on the topmost floor. G does not live on any floor below E’s floor. 13. Who lives on the topmost floor ? (1) B (2) C (3) E (4) G (5) Cannot be determined

    PROBLEM SOLVING 14.

    Who lives immediately above D’s floor ? (1) A (2) B (3) C (4) F (5) G 15. Four of the following five are alike in a certain way and so form a group. Which is the one that does not belong to that group ? (1) F (2) D (3) B (4) G (5) C Directions (16–20) : Read the following passage carefully and answers the questions given below it. (Punjab & Sind Bank PO Exam. 23.01.2011)

    A group of seven friends; A, B, C, D, E, F and G work as Economist, Agriculture Officer, IT Officer, Terminal Operator, Clerk, Forex Officer and Research Analyst, for Banks L, M, N, P, Q, R and S but not necessarily in the same order. C works for Bank N and is neither a Research Analyst nor a Clerk. E is an IT Officer and works for Bank R. A works as Forex Officer and does not work for Bank L or Q. The one who is an Agriculture Officer works for Bank M. The one who works for Bank L works as a Terminal Operator. F works for Bank Q. G works for Bank P as a Research Analyst. D is not an Agriculture Officer. 16. Who amongst the following works as an Agriculture Officer? (1) C (2) B (3) F (4) D (5) None of these 17. What is the profession of C ? (1) Terminal Operator (2) Agriculture Officer (3) Economist (4) Cannot be determined (5) None of these 18. For which bank does B work? (1) M (2) S (3) L (4) Either M or S (5) None of these 19. What is the profession of the person who works for Bank S ? (1) Clerk (2) Agriculture Officer (3) Terminal Operator (4) Forex Officer (5) None of these 20. Which of the following combinations of person, profession and bank is correct? (1) A - Forex Officer - M

    (2) D - Clerk - L (3) F-Agriculture Officer-Q (4) B-Agriculture Officer-S (5) None of these Directions (21–27) : Study the following information to answer the given questions : (UCO Bank PO Exam. 30.01.2011)

    Six chemicals L, M, N, O, P and Q are kept in bottles of different colours viz, green, red, blue, white, pink and violet. not necessarily in the same order. There bottles are arranged from left to right, again not necessarily in the same order. Chemical M is kept in white bottle. Chemical L is not kept in green bottle and is kept to the immediate left of the violet bottle. Chemical O is kept in the blue bottle and is kept exactly between the bottles containing chemicals L and M. The red bottle is at the extreme left end. The bottles containing chemical Q is not kept at either of the ends. The green bottle is kept at the extreme right end. Chemical P is not kept near the white bottle. 21. Four of the following five are alike in a certain way based on their positions in the above arrangement and so form a group. Which is the one that does not belong to the group ? (1) LM (2) LP (3) QO (4) LQ (5) NO 22. Which bottle contains chemical L? (1) Pink (2) Blue (3) Red (4) Cannot be determined (5) None of these 23. Which of the following combinations of chemical and bottle is correct ? (1) P - Red (2) N - Green (3) P - Green (4) Q- Pink (5) None of these 24. Which bottle contains chemical Q? (1) Pink (2) Green (3) Violet (4) Cannot be determined (5) None of these 25. If all the six chemicals are arranged alphabetically from left to right, positions of how many will remain unchanged ?

    BPRE–658

    (1) None (2) One (3) Two (4) Three (5) Four 26. Which bottle contains chemical N? (1) Green (2) Red (3) Pink (4) Cannot be determined (5) None of these 27. Which chemical is kept in the bottle at the extreme right end ? (1) P (2) N (3) L (4) Cannot be determined (5) None of these Directions (28-32) : Study the following information to answer the given questions : (UCO Bank PO Exam. 30.01.2011)

    Six plays A, B, C, D, E and F are to be staged starting from Monday and ending on Sunday with one of the days being an off day, not necessarily in the same order, Each of the plays has different time duration: ½ hour, 1 hour, 1½ hours, 2 hours, 2 ½ hours and 3 hours, again not necessarily in the same order. Sunday is not an off day and a Play of ½ hour duration is staged on that day. Play A is staged immediately before Play E. There are two plays staged between Play F which is for 3 hours and Play C which is for 1½ hours. The off day is after the staging of Play E and there are two days between the off day and Play A. Play D which is for 2 hours is not staged on Monday. The play staged immediately before the off day is of 3 hours. Play A is for less than 2½ hours. 28. What is the time duration of Play B? (1) 2½ hours (2) 2 hours (3) 1 hour (4) ½ hour (5) None of these 29. Which day is the off day? (1) Tuesday (2) Monday (3) Friday (4) Saturday (5) Cannot be determined 30. Which of the following combinations of Play - Day - Time Duration is correct ? (1) E - Wednesday - 2 hours (2) A - Tuesday - 1 hour (3) C - Thursday - 1½ hours (4) F - Tuesday - 3 hours (5) None is correct

    PROBLEM SOLVING 31. On which day is Play D staged? (1) Wednesday (2) Saturday (3) Tuesday (4) Friday (5) Cannot be determined 32. How many plays are staged before the off day? (1) Two (2) One (3) Five (4) Three (5) None of these Directions (33-34) : Keeping all the other information the same, if D is staged on Monday, then ____ (Bank Of Baroda PO Exam. 13.03.2011)

    33. A play of what time duration would be staged on Thursday? (1) 2 hours (2) 2½ hours (3) 1 hour (4) 3 hours (5) Cannot be determined 34. Which day would be the off day? (1) Tuesday (2) Monday (3) Friday (4) Saturday (5) Cannot be determined Directions (35–41) : Study the following information carefully and answer the given questions. (Bank Of Baroda PO Exam. 13.03.2011)

    Seven lectures are scheduled to be held in a week. There is only one lecture on each of the seven days of the week, starting from Monday and ending on Sunday. Chemistry is taught either on Wednesday or on Saturday. Three lectures are scheduled to be held between Chemistry and English. Two lectures are scheduled to be held between English and Computers. Lecture on Physics is scheduled on the day which is immediately next to the day when lecture on Biology is scheduled. Lecture on Psychology is scheduled to be held after Mathematics (not necessarily immediately after Mathematics). Lecture on Psychology is not scheduled for Saturday or Sunday. 35. Which of the following lectures is scheduled for Thursday? (1) Mathematics (2) English (3) Physics (4) Biology (5) Computers 36. Which of the following combinations of day- lecture is correct? (1) Saturday — Physics (2) Monday — Biology (3) Tuesday — English (4) Thursday — English (5) Sunday — Computers

    37. How many lectures are scheduled between Chemistry and Physics? (1) None (2) One (3) Two (4) Three (5) Five 38. If Chemistry is related to Biology and Psychology is related to Computers in a certain way based upon the given lecture schedule, then Biology will be related to which of the following based upon the same relationship? (1) English (2) Mathematics (3) Physics (4) Chemistry (5) None of these 39. On which of the following days is the lecture on Psychology scheduled? (1) Friday (2) Monday (3) Tuesday (4) Thursday (5) None of these 40. Which of the following lecture(s) is/are scheduled to be held between the lectures on Chemistry and Physics? (1) No lecture is scheduled between these two lectures (2) Only Computers (3) Computers and Psychology (4) Computers and Biology (5) English and Mathematics 41. How many lectures are scheduled to be held between English and Mathematics? (1) None (2) One (3) Two (4) Three (5) Five Directions (42-46) : Study the following information carefully and answer the questions given below : (IBPS Bank PO/MT CWE Exam. 18.09.2011)

    P, Q, R, S, T, V, W and Z are travelling to three destinations Delhi, Chennai and Hyderabad in three different vehicles – Honda City, Swift D’Zire and Ford Ikon. There are three females among them one in each car. There are at least two persons in each car. R is not travelling with Q and W. T, a male, is travelling with only Z and they are not travelling to Chennai. P is travelling in Honda City to Hyderabad. S is sister of P and travels by Ford Ikon. V and R travel together. W does not travel to Chennai. 42. Members in which car are travelling to Chennai ? (1) Honda City

    BPRE–659

    (2) Swift D’ Zire (3) Ford Ikon (4) Either Swift D’ Zire or Ford Ikon (5) None of these 43. In which car are four members travelling ? (1) None (2) Honda City (3) Swift D’zire (4) Ford Ikon (5) Honda City or Ford Ikon 44. Which of the following combinations represents the three female members? (1) QSZ (2) WSZ (3) PSZ (4) Cannot be determined (5) None of these 45. Who is travelling with W ? (1) Only Q (2) Only P (3) Both P and Q (4) Cannot be determined (5) None of these 46. Members in which of the following combinations are travelling in Honda City? (1) PRS (2) PQW (3) PWS (4) Data inadequate (5) None of these Directions (47–51) : Study the following information carefully and answer the questions given below : (IBPS Specialist Officer CWE Exam. 11.03.2012)

    A, B, C, D, E, F, G and H are eight friends travelling in three different cars, viz. X, Y and Z with at least two in one car to three different places, viz. Delhi, Chandigarh and Agra. There is at least one female member in each car. D is travelling with G to Delhi but not in car Y. A is travelling with only H in car Z but not to Chandigarh. C is not travelling with either D or E. F and D are studying in the same only girls’ college. H, B and G are studying in the same only boys’ college. 47. Which of the following represents the group of females among them? (1) F, C, A (2) F, G, A (3) D, C, A (4) Data inadequate (5) None of these

    PROBLEM SOLVING 48. Which of the following combinations is correct? (1) Delhi – X – C (2) Chandigarh – X – F (3) Agra – Z – E (4) Delhi – Y – E (5) None of these 49. In which car are four of them travelling ? (1) X or Z (2) Y (3) X or Y (4) Z (5) None of these 50. In which of the following cars is C travelling? (1) X (2) Y (3) Z (4) Either X or Y (5) Data inadequate 51. Passengers in which car are travelling to Chandigarh ? (1) Y (2) X (3) Either X or Y (4) Data inadequate (5) None of these Directions (52-54) : Study the following information carefully to answer the given questions : (Indian Overseas Bank PO Online Exam, 01.09.2013)

    There are six people – P, Q, R, S, T and U – each having different weight. P is heavier than U but lighter than Q. T is heavier than R but lighter than P. Q is lighter than S. R is not the lightest. The second lightest person weighs 52 kilogram while the third heaviest is of 64 kilogram. 52. Who among the following may be of 58 kilogram in weight ? (1) P (2) Q (3) T (4) R (5) U 53. How many persons are heavier than R ? (1) None (2) One (3) Two (4) Three (5) More than three 54. Which of the following may represent the weight of Q ? (1) 62 kg (2) 66 kg (3) 60 kg (4) 58 kg (5) 56 kg Directions (55-59) % Study the following information carefully and answer the questions given below : (IBPS Bank PO/MT CWE-III, 26.10.2013)

    Seven people – A, B, C, D, E, F and G – are having different hobbies, viz., Travelling, Reading, Dancing, Painting, Sculpting, Singing and Pottery making, but not necessarily in the same order. Each of them belong to different State, viz., Punjab, Odisha, Kerala, Rajasthan, Maharashtra, Gujarat and Karnataka, but not necessarily in the same order. A belongs to Maharashtra. D likes pottery making. The person who likes sculpting is from the State of Odisha. The person who likes dancing is from the State of Gujarat. F does not belong to Gujarat, Odisha, Punjab or Rajasthan. F does not like singing, reading or painting. B does not belong to Kerala, Odisha, Punjab or Rajasthan. B does not like painting, travelling reading or singing. C does not like sculpting and he is not from Rajasthan or Punjab. Neither D nor G belongs to Punjab. A does not like reading. The person from Kerala likes singing. 55. Who among the following likes singing ? (1) A (2) C (3) E (4) G (5) Cannot be determined 56. Which one of the following combinations is true according to the given information ? (1) A – Travelling – Maharashtra (2) C – Dancing – Gujarat (3) E – Reading – Karnataka (4) D – Pottery Making – Rajasthan (5) All are true 57. Who among the following belongs to the State of Karnataka ? (1) B (2) D (3) F (4) E (5) Cannot be determined 58. Which of the following combinations is true about G ? (1) Sculpting – Odisha (2) Pottery making – Karnataka (3) Dancing – Gujarat (4) Singing – Kerala (5) Travelling – Karnataka 59. The person who belongs to Punjab, likes : (1) Travelling (2) Sculpting (3) Painting (4) Pottery making (5) Reading

    BPRE–660

    Directions (60–65) : Study the following information carefully and answer the questions given below : (IBPS Bank PO/MT (Pre.) Exam, 23.10.2016)

    Seven athletes – M, N, O, P, Q, R and S — live on seven different floors of a building but not necessarily in the same order. The lowermost floor of the building is numbered one, the one above that is numbered two and so on till the topmost floor is numbered seven. Each one of them runs for a different distance in marathon – 850 m, 1300 m, 2200 m, 2800 m, 3300 m, 4000 m and 4700 m, but not necessarily in the same order. The one who runs for 2200m lives on floor numbered 3. Only one person lives between O and the one who runs for 2200 m. The one who runs for 4000 m lives immediately above O. Only one person lives between the one who runs for 4000 m and the one who runs for 1300 m. The number of people living between O and the one who runs for 1300 m is same as that between the one who runs for 4000m and R. N lives on an odd numbered floor. N ran for 2000 m more than the one who lives on floor number 4. Only two people live between Q and the one who runs for 3300 m. The one who runs for 2800 m lives on one of the floors below Q but not on the floor number 2. Only two people live between M and S. The one who runs for 850 m lives immediately below M. 60. How many people live between S and N? (1) Three (2) One (3) Five (4) Four (5) One 61. Who amongst the following live/ s between P and the one who runs for 1300 m? (1) Both Q and R (2) Only S (3) Both R and the one who runs for 850 m (4) Only the one who runs for 4000 m (5) Both R and the one who runs for 2200 m 62. As per the given arrangement, four of the following five are alike in a certain way and so form a group. Which one of the following does not belong to the group? (1) Q – 3300 m (2) O –1300 m

    PROBLEM SOLVING (3) Floor number 4 – S (4) Floor number 2 – R (5) Floor number 7 – 1300 m 63. Which of the following statements is true with respect to the given arrangement? (1) Only two people live between P and O. (2) Q runs for 4000 m. (3) N lives on floor number 7. (4) The one who runs for 850m lives immediately above P. (5) None of the given options is true 64. If the total distance covered by B and M is 5300m, then how much did B run alone? (1) 2000 m (2) 4000 m (3) 3100 m (4) 1300 m (5) 600 m 65. Who amongst the following runs for 2200 m? (1) P (2) N (3) Q (4) S (5) R Directions (66–70) : Study the following information carefully and answer the questions given below : (SIDBI Officer Exam. 03.09.2014)

    A building has seven floors numbered one to seven in such a way that the ground floor is numbered one, the floor above it number two, and so on such that the topmost floor is numbered seven. Among the seven per sons L, M, N, O, P, Q and R, each one lives on different floor. O lives on fifth numbered floor. Three persons live between the floors of N and Q. N lives above the floor of O. P lives on the floor immediately above the floor of R. Only one person lives between the floors of L and P. 66. Who among the following lives immediately above M’s floor ? (1) L (2) Q (3) O (4) N (5) R 67. Who among the following lives exactly between the floors of O and Q ? (1) R (2) L (3) N (4) M (5) P 68. Who among the following lives on the topmost floor ? (1) N (2) M (3) L (4) Q (5) R

    69. Who among the following lives on the fourth numbered floor? (1) M (2) Q (3) L (4) R (5) P 70. Four of the following five are alike in a certain way and hence form a group. Which one of the following does not belong to that group ? (1) N (2) O (3) Q (4) R (5) P Directions (71–75) : Study the following information carefully and answer the questions given below : (IBPS Bank PO/MT CWE-IV, 18.10.2014)

    There are seven friends – J, K, L, M, N, O and P. Each one of them has different profession viz., Accountant, Actor, Athlete, Choreographer, Doctor, Engineer and Lawyer, but not necessarily in the same order. They were born in the years 1983, 1984, 1986, 1987, 1990, 1992 and 1994, but not necessarily in the same order. The Lawyer was born in 1986 while the Athlete was born in 1984. K is a doctor and he was not born in the year 1983. P was born in the year 1992. P is neither Choreographer nor Actor. N was not born in the year 1994. N is not an Athlete. L was born in the year 1990. L is neither Accountant nor Choreographer. Doctor was not born in the year 1994. J is an Engineer. J was not born in the year 1994. O is not a Choreographer. 71. In which year O was born ? (1) 1983 (2) 1984 (3) 1990 (4) 1987 (5) None of these 72. Which of the following statements is NOT true on the basis of information given above? (1) Accountant was born in the year 1992 (2) Engineer was born in the year 1983 (3) Doctor war was born in the year 1987 (4) Choreographer was born in the year 1990 (5) All are true 73. Who among the following is a Choreographer ? (1) J (2) K (3) M (4) N (5) Cannot be determined

    BPRE–661

    74. Which of the following combinations of FriendPr ofession-Year of Birth is correct ? (1) O – Athlete – 1984 (2) J – Engineer – 1987 (3) P – Lawyer – 1994 (4) L – Choreographer – 1990 (5) M – Actor – 1992 75. Who among the following is the eldest ? (1) Accountant (2) Choreographer (3) Doctor (4) Actor (5) Engineer Directions (76–80) : Study the following information carefully and answer the questions given below : (Bank of Baroda Junior Management Grade/Scale-I Exam, 18.04.2015)

    Eight people – S, T, U, V, W, X, Y and Z – live on separate floors of an 8 – floor building. Ground floor is numbered 1, first floor is numbered 2 and so on until the topmost floor is numbered 8. ● Y lives on floor numbered 1. Only two people live between Y and T. ● S lives immediately above X. S lives on an even numbered floor. ● Only one person lives between X and W. X lives above W. ● Z lives on an odd–numbered floor above S. ● V does not live on the topmost floor. 76. Which of the following is true with respect to the given infor mation ? (1) Only one person lives between T and X. (2) Only two people live between T and U. (3) W lives immediately above Y. (4) Z lives on floor numbered 7. (5) U lives on an odd–numbered floor. 77. Who amongst the following lives between U and S ? (1) Z (2) V (3) T (4) W (5) No one 78. Who among the following live on floor number 5 ? (1) U (2) X (3) V (4) T (5) Z

    PROBLEM SOLVING 79. Who lives on the floor immediately below T ? (1) S (2) Other than those given as options (3) Y (4) Z (5) W 80. How many people live between the floors on which Z and T live? (1) More than three (2) Three (3) No one (4) One (5) Two Directions (81–82) : Read the following information carefully and answer the questions which follow : (IBPS Bank PO/MT CWE Exam. 18.09.2011)

    A, B, C, D, E and F live on different floors in the same building having six floors numbered one to six (the ground floor is numbered 1, the floor above it , number 2 and so on and the topmost floor is numbered 6). A lives on an even numbered floor. There are two floors between the floors on which D and F live. F lives on a floor above D’s floor. D does not live on floor number 2. B does not live on an odd numbered floor. C does not live on any of the floors below F’s floor. E does not live on a floor immediately above or immediately below the floor on which B lives. 81. Who amongst the following live on the floors exactly between D and F ? (1) E, B (2) C, B (3) E, C (4) A, E (5) B, A 82. On which of the following floors does B live ? (1) 6th (2) 4th (3) 2nd (4) 5th (5) Cannot be determined Directions (83–88) : Study the following information carefully and answer the questions given below : (IBPS RRBs Officer Scale–I & II CWE 12.09.2015)

    Eight persons — K, L, M, N, O, P, Q and R — live on eight different floors of a building but not necessarily in the same order. The lower most floor of the building in numbered one, the one above that is numbered two and so on till the topmost floor is numbered eight. Each of them also likes a different superhero namely, Batman, Superman, Captain America, Thor, Hulk, Wolver-

    ine, Nova and Ironman, but not necessarily in the same order. ● The one who likes Thor lives on an even numbered floor. Only three persons live between the one who likes Thor and M. ● Only two persons live between M and N. N does not live on the lower most floor. Only three persons live between N and the one who likes Hulk. ● O lives immediately above K. O lives on an even numbered floor. K does not like Hulk. K lives neither on floor numbered three nor five. ● Only two persons live between K and the one who likes Nova. ● Only one person lives between the one who likes Nova and Ironman. The one who like Ironman lives below the one who likes Nova. ● L lives immediately above Q. Only one person lives between Q and the one who likes Captain America. ● The one who likes Wolverine lives immediately above the one who likes Batman. ● P does not like Thor. K does not like Superman. 83. Which of the following pairs represent those who lives immediately above and immediately below N? (1) R, L (2) Other than those given as options (3) L, M (4) K, Q (5) R, Q 84. Four of the following five are alike in a certain way as per the given arrangement and so form a group. Which of the following does not belong to that group? (1) K – Floor numbered Five (2) Q – Floor numbered Six (3) L – Floor numbered Two (4) N – Floor numbered Seven (5) R – Floor numbered Eight 85. Which of the following superheroes does R like? (1) Other than those given as options (2) Superman (3) Nova (4) Thor (5) Batman

    BPRE–662

    86. Which of the following superheroes does P like? (1) Ironman (2) Wolverine (3) Batman (4) Captain America (5) Other than those given as options 87. N lives on which of the following floor numbers? (1) Five (2) Three (3) Seven (4) One (5) Other than those given as options 88. As per the given arrangment, P is related to Nova and N is related to Wolverine in a certain way. To which of the following is M related to in the same way? (1) Superman (2) Batman (3) Captain America (4) Thor (5) Hulk Directions (89–91) : Study the following information carefully and answer the questions given below : (IBPS Bank PO/MT CWE–V (Preliminary) 03.10.2015)

    Each of the six buildings P, Q, R, S, T and U houses different number of offices. S has more offices than only T and R. Q has more number of offices than P but less than U. R does not house the least number of offices. The building which houses the least number of offices has 5 offices. The building which has second highest number of offices has 23 offices. S has 11 less number of offices than Q. 89. Which of the following buildings has the second least number of offices ? (1) Q (2) U (3) R (4) P (5) T 90. The number of offices in P is an even number which is divisible by 2 as well as 3. How many offices does P have ? (1) 20 (2) 24 (3) 16 (4) 18 (5) 12 91. Which of the following is the number of offices in the building R ? (1) 25 (2) 12 (3) 13 (4) 14 (5) 11

    PROBLEM SOLVING Directions (92–94) : Study the following information carefully and answer the questions given below : (IBPS Bank PO/MT CWE–V (Preliminary) 04.10.2015)

    Each of the six buildings — E, F, G, H, I and J — has different number of floors. Only three buildings have more number of floors than J. G has more number of floors than I but less than E. I has more number of floors than J. F does not have the least number of floors. The building having least number of floors has 5 floors. The building having third highest number of floors has 26 floors. F has 14 floors less than the number of floors in I. 92. If the number of floors in building G is less than 38 and is an odd number which is divisible by 3 but not 7, how many floors does G have ? (1) 15 (2) 35 (3) 9 (4) 27 (5) 29 93. Which of the following buildings has the second highest number of floors ? (1) I (2) F (3) J (4) H (5) G 94. How many floors does building J possibly have ? (1) 22 (2) 5 (3) 11 (4) 35 (5) 12 Directions (95–97) : Study the following information carefully and answer the questions given below : (IBPS Bank PO/MT CWE–V (Preliminary.) 04.10.2015)

    Each of the six buses–R, S, T, U, V and W– has different number of occupants. T has more number of occupants than R and S but less than V. U has less number of occupants than only W. S does not have the lowest number of occupants. The bus having second lowest number of occupants has 20 occupants and the bus having second highest number of occupants has 64 occupants. T has 21 less number of occupants than U. 95. Which of the following buses has third lowest number of occupants? (1) S (2) T (3) R (4) U (5) V

    96. If the number of occupants of bus R is more than 7 and is an odd number which is divisible by 3 but not 5, how many occupants are there in bus R? (1) 9 (2) 21 (3) 15 (4) 27 (5) 19 97. How many occupants does bus V possibly have? (1) 43 (2) 72 (3) 52 (4) 36 (5) 68 Directions (98–102) : Study the following information carefully and answer the questions given below : (IBPS Bank PO/MT CWE–V (Preliminary.) 04.10.2015)

    Seven persons namely, M, N, O, P, Q, R and S have an anniversary but not necessarily in the same order, in seven different months (of the same year) namely February, March, April, June, September, October and November. Each of them also likes a different flower namely Rose, Jasmine, Lily, Marigold, Daffodil, Sunflower and Orchid but not necessarily in the same order. R has an anniversary in the month which has more than 30 days. Only one person has an anniversary between R and the one who likes Rose. Both S and O have an anniversary in one of the months after the one who likes Rose. S has an anniversary immediately before O. The one who likes Lily has an anniversary in the month which has less than 30 days. Only three persons have an anniversary between the one who likes Lily and the one who likes Orchid. Only two persons have an anniversary between S and the one who likes Marigold. P has an anniversary immediately after the one who likes Marigold. Only two persons have an anniversary between P and Q. M has an anniversary immediately before the one who likes Jasmine. O does not like Sunflower. 98. Which of the following represents the month in which S has an anniversary? (1) Cannot be determined (2) October (3) March (4) April (5) September 99. Which of the following does O like?

    BPRE–663

    (1) Rose (2) Jasmine (3) Marigold (4) Daffodil (5) Orchid 100. As per the given arrangement, Lily is related to April and Marigold is related to September following a certain pattern, with which of the following is Orchid related to following the same pattern? (1) February (2) June (3) October (4) March (5) November 101. Which of the following represents the persons who have an anniversary in April and November respectively? (1) N, O (2) Q, M (3) Q, O (4) N, M (5) N, S 102. How many persons have an anniversary between the months in which Q and M have an anniversary? (1) None (2) One (3) Three (4) Two (5) More than three Directions (103-105) : Study the following information carefully and answer the questions given below : (IBPS Bank PO/MT CWE–V (Preliminary) 10.10.2015 Ist Sitting)

    Six sales persons – U, V, W, X, Y, and Z – sell insurance policies. Each of them sold different number of policies. U sold more policies than both Y and Z but less than X. Z sold more policies than only W. X did not sell the highest number of policies. The third highest number of policies sold is equal to 33. The least number of policies sold is equal to 11. Y sold 13 more policies than that of W. 103. Who among the following did sell exactly 33 policies ? (1) X (2) Y (3) Z (4) U (5) Cannot be determined 104. Which of the following may represent the number of policies sold by Z ? (1) 26 (2) 19 (3) 9 (4) 36 (5) 28 105. Who among the following did sell less policies than only V ? (1) U (2) Y (3) X (4) Z (5) Cannot be determined

    PROBLEM SOLVING Directions (106–108) : Study the following information carefully and answer the questions given below : (IBPS Bank PO/MT CWE–V (Preliminary) 10.10.2015)

    Each of the six persons P, Q, R, S, T and U has lived in India for a different number of days. S has stayed for more number of days than R but less than T. P has stayed for more number of days than only U. T has not stayed for the most number of days. The one stayed for the second least number of days stayed for 14 days in India. The one who stayed for second highest number of days stayed for 47 days, S stayed for 15 days less than T. 106. For how many days did R possibly stay in India ? (1) 56

    (2) 41

    (3) 33 (5) 11

    (4) 25

    107. Who amongst the following stayed for the second highest number of days ? (1) U

    (2) T

    (3) Q (5) P

    (4) R

    108. If the number of days for which Q stayed in India is less than 60 and is an even number which is divisible by 3 but not by 4, for how many days did Q stay in India? (1) 54 (2) 42 (3) 56 (4) 48 (5) 30 Directions (109–115) : Study the following information carefully and answer the questions given below : (IBPS RRBs Officer Scale–I & II CWE 13.09.2015)

    Eight persons S, T, U, V, W, X, Y and Z live on eight different floors of a building but not necessarily in the same order. The lowermost floor of the building is numbered one, the one above that is numbered two and so one till the topmost flooor is numbered eight. Each of them also works at a different banks namely, IDBI, SBI, HDFC, BOI, PNB, TJSB, Axis Bank and SVC, but not necessarily in the same order. ● Z lives on an even numbered floor. Only three persons live between Z and the one who orks at BOI. W lives immediately below the one who works at BOI.

    Only three persons live between W and the one who works at Axis Bank. ● V lives immediately above T. V lives on an odd numbered floor. T does not work at BOI. ● Only two persons live between T and the one who works at SBI. The one who works at SBI does not live on the lowermost floor. ● The one who works at SVC lives immediately above the one who works at PNB. The one who works at SVC live on an even numbered floor but not on floor numbered two. ● Only one person lives between the one who works at SVC and the one who works at IDBI. ● X lives immediately above S. X lives on an even numbered floor. X does not work at TJSB. ● U does not work at PNB and does not live on floor numbered four. 109. Four of the following five are alike in a certain way based on the given arrangement and hence they form a group. Which one of the following does not belong to that group ? (1) S — Floor numbered two (2) T — Floor numbered eight (3) W — Floor numbered six (4) Z — Floor numbered one (5) U — Floor numbered seven 110. V works at which of the following banks ? (1) Axis Bank (2) SVC (3) IDBI (4) SBI (5) Other than those given as options 111. Which of the following pair represent those who live immediately above and immediately below Z? (1) S, T (2) T, V (3) Other than those given as options (4) S, V (5) X, Y 112. W works at which of the following banks ? (1) Other than those given as options (2) PNB (3) Axis Bank (4) BOI (5) SBI 113. U lives on which of the following floor numbers ? (1) One (2) Seven (3) Other than those given as options ●

    BPRE–664

    (4) Five (5) Two 114. As per the given arrangement, U is related to PNB and S is related to HDFC in a certain way. To which of the following is V related to in the same way ? (1) SVC (2) SBI (3) IDBI (4) TJSB (5) Axis Bank 115. Study the following information carefully and answer the question which follows : For the past three years the Government of State X has been organizing awareness programs for the Hoardings and banners have been put across the State promoting women upliftment and gender equality. Which of the following cannot be consequence of the steps taken by the government of State X ? (1) The rate of female foeticide in the state has came down from 280 cases per year to 30 cases per year (2) Dowry and domestic violence cases in the State dropped by 85% in the last couple of years. (3) Cases reported by women of the State regarding chain snatching on the roads have come down by 40% in the last two years. (4) Two years ago minimum 15% fee exemption was made mandatory for girl child in all public and private schools of the State. (5) In all the public organizations of the State, a minimum of 10% female employees has been made compulsory since the past two and a half years. (IBPS RRBs Officer Scale–I & II CWE 13.09.2015)

    Directions (116-119) : Study the following information carefully and answer the questions given below : L, M, N, O and P are five different poles, each of different length. O is not the third shortest pole. N is bigger than only P. L is shorter than only one pole. The size of the shortest pole is 7 ft and that of the second tallest pole is 13 ft. 116. Which of the following poles is the third tallest? (1) M (2) Cannot be determined (3) N (4) P (5) L

    PROBLEM SOLVING 117. According to the given arrangement, which of the following combinations of pole and length is correct? (1) N – 14 ft (2) P – 5 ft (3) O – 12 ft (4) L – 13 ft (5) Other than those given as options 118. If in the number 38564927, first all the even digits are arranged in ascending order and then all the odd digits are arranged in ascending order which digit will be fourth from the right end? (1) 3 (2) 5 (3) 4 (4) Other than those given options (5) 6 119. If ‘4’ is subtracted from each odd digit and ‘1’ is added to each even digit in the number 947 6582, which of the following digits will apear twice in the new number thus formed? (1) Only 2 (2) Both 1 and 7 (3) Both 3 and 5(4) None (5) Only 1 (IBPS Specialist Officer (Marketing) CWE 01.02.2016)

    Directions (120–126) : Study the following information carefully and answer the questions given below : (IBPS Specialist Officer (Marketing) CWE 01.02.2016)

    Eight persons – K, L, M, N, O, P, Q and R – live on eight different floors of a building but not necessarily in the same order. The lowermost floor of the building is numbered one, the one above that is numbered two and so on till the topmost floor is numbered eight. P lives on an even numbered floor but not on the topmost floor. Only three persons live between L and P. O lives immediately below L. N lives immediately below P. K lives immediately above Q. There are as many persons between Q and R as are there between R and O. 120. If L and P interchanges their positions and so do K and M, then who will live between M and L, as per the new arrangement? (1) Q (2) O (3) N (4) R (5) Other than those given as options

    121. Who amongst the following lives on the floor numbered six? (1) M (2) R (3) Q (4) P (5) L 122. On which of the following floor numbers does M live? (1) Eight (2) Six (3) Three (4) Seven (5) Two 123. As per the given arrangement, four of the following five are alike in a certain way and so form a group. Which one of the following does not belong to the group? (1) MR (2) PN (3) ML (4) LO (5) NK 124. Who amongst the following live exactly between K and N? (1) L, P (2) P, Q (3) Q, O (4) R, L (5) Q, R 125. If ‘2’ is added to each odd digit and ‘1’ is subtracted from each even digit in the number ‘2368547’, which of the following digits will appear twice in the new number thus formed ? (1) None (2) Both 3 and 5 (3) Only 1 (4) Both 5 and 7 (5) Only 2 (IBPS Specialist Officer (IT) CWE 14.02.2016)

    126. Study the given information carefully and answer the question given below : Carmakers Kellbahn’s domestic sales have surged as they have reported sales of 12,000 cars this December. This number is more than double the number of cars sold by them in the third quarter (October-December) put together last year. Which of the following can be concluded from the given statement ? (1) If 6000 more cars were sold in the third quarter of last year, the two numbers would have become equal. (2) Kellbahn definitely sold less than 6000 cars in December last year. (3) Addition of International sales figures would have made performance of last quarter better.

    BPRE–665

    (4) If 6000 less cars were sold in December this year, the number of cars sold would have been same as that in December last year. (5) Kellbahn definitely sold less than 12,000 cars in November this year. (IBPS Specialist Officer (IT) CWE 14.02.2016)

    Directions (127–132) : Study the following information carefully and answer the questions given below : (IBPS Specialist Officer (IT) CWE 14.02.2016)

    Seven friends namely, P, Q, R, S, T, U and V visit seven different cities namely, Berlin, Kabul, Jakarta, Madrid, Chicago, Miami and Sydney, but not necessarily in the same order, starting from Monday to Sunday (of the same week.) Q visits a city on Wednesday. Only one person visits between Q and the one who visits Madrid. T visits immediately after the one who visits Madrid. Only four persons visit between T and the one who visits Sydney. Only one person visits between the one who visits Sydney and R. The one who visits Berlin visits immediately before R. The one who visits Chicago visits on one of the days before the one who visits Berlin but not on Wednesday. More than three persons visit between the one who visits Chicago and P. Only two persons visit between S and the one who visits Miami. Only two persons visit between the one who visits Miami and the one who visits Jakarta. V does not visit Berlin. 127. On which of the following days does S visit a city ? (1) Sunday (2) Thursday (3) Tuesday (4) Saturday (5) Friday 128. As per the given arrangement R is related to the one who visits Jakarta in a certain way and T is related to the one who visits Miami in the same way. To which of the following is V related to in the same way ? (1) The one who visits Kabul (2) The one who visits Sydney. (3) The one who visits Madrid (4) The one who visits Berlin. (5) The one who visits Chicago 129. Which of the following cities does P visit ?

    PROBLEM SOLVING (1) Berlin (2) Jakarta (3) Sydney (4) Madrid (5) Miami 130. Which of the following statements is true about U ? (1) All the given statements are true. (2) U visits Berlin. (3) U visits a city on Saturday. (4) Only three persons visit between U and Q. (5) U visits a city immediately before T. 131. Four of the following five are alike in a certain way and thus form a group as per the given arrangement. Which of the following does not belong to that group ? (1) T – Thursday (2) P – Sunday (3) Q – Wednesday (4) R – Friday (5) V – Monday 132. Who amongst the following visits Kabul ? (1) P (2) R (3) S (4) U (5) Q Directions (133–137) : Study the following information carefully and answer the questions given below : (IBPS Specialist Officer (IT) CWE 14.02.2016)

    Eight persons — P, Q, R, S, W, X, Y and Z — live on eight different floors of a building but not necessarily in the same order. The lowermost floor of the building is numbered one, the one above that is numbered two and so on till the topmost floor is numbered eight. P lives on an odd numbered floor below the floor numbered six. Y lives immediately above P. Only three persons live between Y and W. There are as many persons between W and R as are there between R and Y. Z lives on an odd numbered floor below Y but not on the floor numbered three. Only one person lives between Q and X. Q lives above X. 133. Who amongst the following lives on the floor numbered three ? (1) S (2) W (3) X (4) P (5) Q 134. As per the given arrangement, four of the following five are alike in a certain way, and so form a group. Which one of the following does not belong to the group ?

    (1) W – Five (2) Y – One (3) S – Three (4) Q – Four (5) X – Eight 135. Who amongst the following lives exactly between S and X? (1) Only P (2) Only Q (3) Both Z and R(4) No one (5) Both P and Y 136. If Q and Y interchange their places and so do W and S, then who will live between Q and W as per the new arrangement ? (1) P (2) X (3) R (4) S (5) Other than those given as options 137. On which of the following floor numbers does R live ? (1) Two (2) Six (3) Five (4) Seven (5) Three Directions (138–143) : Study the following information carefully and answer the questions given below : (IBPS Specialist Officer (IT) CWE 14.02.2016)

    Seven friends namely, P, Q, R, S, T, U and V visit seven different cities namely, Berlin, Kabul, Jakarta, Madrid, Chicago, Miami and Sydney, but not necessarily in the same order, starting from Monday to Sunday (of the same week.) Q visits a city on Wednesday. Only one person visits between Q and the one who visits Madrid. T visits immediately after the one who visits Madrid. Only four persons visit between T and the one who visits Sydney. Only one person visits between the one who visits Sydney and R. The one who visits Berlin visits immediately before R. The one who visits Chicago visits on one of the days before the one who visits Berlin but not on Wednesday. More than three persons visit between the one who visits Chicago and P. Only two persons visit between S and the one who visits Miami. Only two persons visit between the one who visits Miami and the one who visits Jakarta. V does not visit Berlin. 138. On which of the following days does S visit a city ? (1) Sunday (2) Thursday (3) Tuesday (4) Saturday (5) Friday 139. As per the given arrangement R is related to the one who visits Jakarta in a certain way and T is

    BPRE–666

    related to the one who visits Miami in the same way. To which of the following is V related to in the same way ? (1) The one who visits Kabul (2) The one who visits Sydney. (3) The one who visits Madrid (4) The one who visits Berlin. (5) The one who visits Chicago 140. Which of the following cities does P visit ? (1) Berlin (2) Jakarta (3) Sydney (4) Madrid (5) Miami 141. Which of the following statements is true about U ? (1) All the given statements are true. (2) U visits Berlin. (3) U visits a city on Saturday. (4) Only three persons visit between U and Q. (5) U visits a city immediately before T. 142. Four of the following five are alike in a certain way and thus form a group as per the given arrangement. Which of the following does not belong to that group ? (1) T – Thursday (2) P – Sunday (3) Q – Wednesday (4) R – Friday (5) V – Monday 143. Who amongst the following visits Kabul ? (1) P (2) R (3) S (4) U (5) Q Directions (144-147) : Read the following information carefully and answer the questions which follow : (IDBI Bank Officer Exam.16.09.2012)

    P, Q, R, S, T and V live on different floors in the same building having six floors numbered one to six (the ground floor is numbered 1, the floor above it, number 2 and so on and the topmost floor is numbered 6). Q lives on an even numbered floor. Only two people live between the flloors on which Q and V live. S lives on a floor immediately above the floor on which R lives. S does not live on an odd numbered floor. P does not live on a floor which is immediately above or immediately below the floor on which T lives. P does not live on the lowermost floor i.e. floor no. 1.

    PROBLEM SOLVING 144. Who amongst the following live on the floors exactly between the floors on which Q and V live ? (1) R, S (2) P, R (3) S, T (4) P, T (5) Cannot be determined 145. Who amongst the following lives on floor number 5 ? (1) P (2) Q (3) T (4) V (5) Cannot be determined 146. On which of the following floors does T live ? (1) 3rd (2) 5th (3) 1st (4) 4th (5) Cannot be determined 147. How many people live on the floors above the floor on which P lives? (1) None (2) One (3) Two (4) Three (5) Cannot be determined Directions (148–152) : Study the following information carefully and answer the questions given below : (BOB Manipal School of Banking Officer Online Exam, 14.08.2014)

    A building has seven floors, numbered one to seven in such a way that the ground floor is numbered one, the floor above it numbered two and so on such that the topmost floor is numbered seven. One of the seven per sons, viz, P, Q, R, S, T, U and V, live on each floor. P lives on fourth floor. Two per sons live between the floors of P and R. Two persons live between the floors of Q and S. Q lives on the floor immediately below the V’s floor. Q does not live below the fllor of U and S. Q lives on the floor of P. T lives on the floor immediately above the floor of R. 148. Who among the following lives on fifth floor ? (1) T (2) S (3) V (4) Q (5) U 149. How many persons live between the floors of V and R ? (1) None (2) Two (3) Five (4) Three (5) One 150. Four of the following five are alike in a certain way based on the given arrangement and hence they form a group. Which one of the following does not belong to that group ?

    (1) Q (2) R (3) V (4) U (5) S 151. Who among the following lives exactly between the floors of U and V ? (1) P (2) T (3) S (4) Q (5) None 152. Who among the following lives immediately above the floor of P? (1) Q (2) U (3) T (4) S (5) V Directions (153–157) : Study the following information carefully and answer the questions given below : A building has seven floors numbered one to seven, in such a way that ground floor is numbered one, the floor above it, number two and so on such that the topmost floor is numbered seven. One out of seven persons, viz., P, Q, R, S, T, U and V lives on each floor, but not necessarily in the same order. Each one of them is travelling to different places, viz., Bangalore, Chennai, Delhi, Jaipur, Kolkata, Mumbai and Patna, but not necessarily in the same order. Three persons live on the floors above the floor of P. There is only one person between P and the person travelling to Bangalore. U lives immediately below the person who is travelling to Mumbai. The person who is travelling to Mumbai lives on an even numbered floor. P lives below the person travelling to Mumbai. Two persons are living between the persons who are travelling to Bangalore and Patna respectively. T lives immediately above R. T is not travelling to Patna. Two persons live between Q and the person travelling to Kolkata. The person who is travelling to Delhi is not living immediately above or below the floor of Q. The person who is travelling to Kolkata lives below Q. S does not live immediately above or below the floor of P. V is not travelling to Chennai. The person who is travelling to Delhi does not live on the ground floor. (IBPS RRBs Officer Scale-I CWE, 06.09.2014)

    153. Who among the following lives on the topmost floor ? (1) U (2) Q (3) V (4) T (5) S

    BPRE–667

    154. Four of the following five are alike in a certain way and hence they form a group based on the given arrangement. Which one of the following does not belong to that group? (1) R (2) S (3) V (4) U (5) T 155. Who among the following travels to Delhi ? (1) T (2) U (3) S (4) R (5) P 156. How many persons live between the person who is travelling to Mumbai and S ? (1) Three (2) Four (3) One (4) Two (5) Five 157. Who among the following does live on the floor immediately above the floor of T ? (1) S (2) Q (3) U (4) P (5) V Directions (158–159) : Study the following information carefully and answer the questions given below : (IBPS RRBs Officer Scale–I & II CWE 12.09.2015)

    Each of the six persons, P, Q, R, S, T and U has travelled to different number of countries. R has travelled to more number of countries than only S and T. Q has travelled to more number of countries than U but less than P. T is not the person who has travelled to least number of countries. The one who has travelled to second highest number of countries has travelled to 8 countries. The one who has travelled to the least number of countries has travelled to only two countries. 158. Who amongst the following has possibly travelled to 11 countries? (1) P (2) R (3) T (4) Q (5) S 159. If it is given that R has travelled to four countries less than the number of countries travelled by Q, then to how many countries has U possibly travelled? (1) 4 (2) 9 (3) 3 (4) 6 (5) 10

    PROBLEM SOLVING Directions (160–165) : Study the following information carefully and answer the questions given below : (SIDBI Officer Online Exam.24.02.2016)

    Seven friends namely A, B, C, D, E, F and G learn seven different languages namely Japanese, Hindi, French, Chinese, Spanish, English and German, not necessarily in the same order, starting from Monday to Sunday (of the same week). F learns a language on Wednesday. Only one person learns between F and the one who learns Spanish. Only three persons learn between the one who learns Spanish and B. The one who learns French learns immediately after B. Only two persons learn between the one who learns French and G. A learns immediately before D. Only three persons learn between the one who learns Japanese and the one who learns English. F does not learn Japanese. E learns before the one who learns English. Only one person learns between E and the one who learns Chinese. A does not learn Hindi. 160. As per the given arrangement, B is related to the one who learns French in a certain way and F is related to the one who learns Chinese in the same way. To which of the following is A related to in the same way? (1) the one who learns German (2) the one who learns Hindi (3) the one who learns Japanese (4) the one who learns English (5) the one who learns Spanish 161. Which of the following statements is/are true about C? (1) C learns Hindi (2) C learns a language on Saturday (3) C learns a language immediately after F (4) Only two persons learn between C and G (5) All the given statements are true 162. Four of the following five are alike in a certain way and thus form a group as per the given arrangement. Which of the following does not belong to that group? (1) D – Thursday (2) F – Sunday (3) B – Friday (4) A – Tuesday (5) G – Monday

    163. Which of the following languages does D learn? (1) Spanish (2) Hindi (3) French (4) Japanese (5) English 164. Who amongst the following learns German? (1) E (2) A (3) G (4) D (5) C 165. On which of the following days does E learn a language? (1) Thursday (2) Friday (3) Monday (4) Sunday (5) Tuesday Directions (166–171) : Study the following information carefully and answer the questions given below :

    167.

    168.

    (United Bank of India PGDBF Manipal Exam,07.08.2016)

    Seven persons – A, B, C, D, E, F and G – published their journals in the seven different months of the same year viz., January, February, April, July, August, November and December, but not necessarily in the same order. Each one of them also likes different colours, viz., Blue, White, Red, Orange, Pink, Green and Yellow, but not necessarily in the same order. D published his journal in a month which has less than 31 days. Only two persons published journals between D and the one who likes Orange colour. F published his journal immediately after the one who likes Orange colour. F publisshed his journal before the one who likes Blue colour. The one who likes Blue colour published his journal in a month which has 31 days but not in the month of August. Only three persons published their journals between the one who likes Blue colour and B. The one who likes Pink colour published his journal immediately before B but not in the month which has 31 days. G published his journal immediately before the one who likes Red colour. As many persons published their journals between G and A as between the persons who like Red colour and Orange colour respectively. Only one person published his journal between A and the one who likes White colour. C published his journal before the one who likes White colour. C does not like Green colour. 166. Which of the following colours does C like?

    BPRE–668

    169.

    170.

    171.

    (1) Pink (2) White (3) Orange (4) Yellow (5) Other than those given as options Who among the following likes Blue colour? (1) The one who published his journal in December (2) The one who published his journal in April (3) The one who published his journal in July (4) The one who published his journal in January (5) The one who published his journal in November Four of the following five are alike in a certain way based on the given arrangement and hence they form a group. Which one of the following does not belong to that group? (1) January – Green (2) July – Red (3) D – Pink (4) F – White (5) December – Orange As per the given arrangement, ‘January’ is related to ‘White’ and ‘December’ is related to ‘July’ following a certain pattern, then with which of the following ‘February’ is related following the same pattern? (1) The one who published his journal in November (2) The one who likes Orange colour (3) The one who likes Red colour (4) The one who published his journal in July (5) The one who likes Green colour Which of the following combinations is definitely correct? (1) February – C – Green (2) April – A – Pink (3) August – F – White (4) November – G – Blue (5) January – C – yellow How many persons published their journals between the ones who like Pink and Red colours? (1) None (2) Two (3) Three (4) More than three (5) One

    PROBLEM SOLVING Directions (172–176) : Study the following information carefully and answer the questions given below :

    Directions (177–181) : Study the following information carefully and answer the questions given below :

    Directions (182–186) : Study the following information carefully and answer the questions given below :

    (IBPS RRBs Officer CWE (Pre.) Exam, 14.11.2016 (Shift-I))

    (IBPS RRBs Officer CWE (Pre.) Exam, 14.11.2016 (Shift-I))

    (IBPS RRBs Officer CWE (Pre.) Exam, 14.11.2016 (Shift-II))

    Sagar sells mobiles of seven different companies viz., Samsung, HTC, Lenovo, Intex, Micromax, Nokia and Oppo starting from Monday and ending on Sunday, but not necessarily in the same order. Sagar sold HTC on Monday. He sold only three mobiles between HTC and Samsung. He does not sell any mobile between the day he sold Samsung and Lenovo. He sold only two mobiles between Lenovo and Oppo. He sold Micromax mobiles the day immediately before the day he sold Oppo mobiles. He sold Intex on one of the days after Oppo but not on Sunday. 172. On which day Sagar sold Micromax? (1) Thursday (2) Tuesday (3) Friday (4) Saturday (5) Wednesday 173. How many mobiles did Sagar sell between HTC and Lenovo? (1) One (2) Two (3) Four (4) None (5) Three 174. Which of the following statements is not true as per the given arrangement? (1) Sagar sells mobiles of any one company between Samsung and Nokia. (2) Sagar sells Intex on Thursday (3) Sagar did not sell mobile of any other company between Oppo and Intex (4) All the given statements are true (5) Sagar sells Lenovo on Friday 175. Four of the following five are alike in a certain way based on the given arrangement and thus form a group. Which is the one that does not belong to that group? (1) Nokia–Friday (2) Samsung–Thursday (3) Micromax–Monday (4) Intex–Wednesday (5) Oppo–Tuesday 176. Which mobile did Sagar sell on Sunday? (1) Oppo (2) Lenovo (3) Micromax (4) Nokia (5) None of those given as options

    Nine friends – A, B, C, D, E, F, G, H and I – live on nine different floors of a building but not necessarily in the same order. The lower most floor of the building is numbered one, the one above that is numbered two and so on till the topmost floor is numbered nine. I lives on floor numbered six. E lives on an odd numbered floor above I. Only three people live between E and G. A lives on an even numbered floor immediately below D but not on the floor numbered eight. Only one person lives between A and F. C lives on one of the floors below F. The number of people living above C is equal to the number of people living below H. 177. Four of the following five are alike in a certain way as per the given arrangement and thus form a group. Which of the following does not belong to that group? (1) DE (2) FC (3) HB (4) EI (5) AG 178. How many persons live between A and the person living on the floor numbered seven? (1) More than three (2) Two (3) One (4) None (5) Three 179. H lives on which of the following floor numbers? (1) Eight (2) One (3) Five (4) Nine (5) Other than those given as options 180. Which of the following statements is not true about B as per the given arrangement? (1) B lives on an even numbered floor. (2) All the given statements are true. (3) Only one person lives between B and E. (4) H lives immediately above B. (5) Only one person lives above B. 181. In which of the given pairs of people, is even number of people living between them? (1) G, C (2) H, E (3) E, D (4) B, I (5) I, H

    Rohan speaks about seven different countries viz., Egypt, China, Indonesia, Japan, Malaysia, France and Austria in a seminar held on seven different days of the same week starting from Monday and ending on Sunday but not necessarily in the same order. Thus on one day Rohan speaks about only one country. Rohan speaks about Japan on Friday. He speaks about Egypt on one of the days after Japan. He speaks only about two countries between Egypt and China. He speaks about only one country between China and France. He speaks about France on one of the days before he speaks about China. He speaks only about one country between Japan and Malaysia. He speaks about Austria on one of the days before he speaks about China but not on Monday. 182. Rohan speaks about which country on Thursday? (1) Malaysia (2) Egypt (3) Indonesia (4) Austria (5) China 183. Which of the following statements is not true as per the given arrangement? (1) All the given statements are true (2) Rohan speaks about France on the day immediately before the day he speaks about Austria. (3) Rohan speaks about China on Wednesday (4) Rohan speaks about Egypt on Saturday (5) Rohan speaks about Indonesia on Thursday 184. On which day Rohan speaks about France? (1) Saturday (2) Wednesday (3) Sunday (4) Monday (5) Tuesday 185. On how many countries does Rohan speak about between China and Malaysia? (1) Four (2) Three (3) Two (4) One (5) None

    BPRE–669

    PROBLEM SOLVING 186. Four of the following five are alike in a certain way based on the given arrangement and thus form a group. Which is the one that does not belong to that group? (1) Saturday–Malaysia (2) Tuesday–France (3) Sunday–Egypt (4) Monday–Austria (5) Friday–China Directions (187 – 191) : Study the following information and answer the questions given below : (IBPS RRBs Officer CWE (Pre.) Exam, 14.11.2016 (Shift-II))

    Nine friends – P, Q, R, S, T, U, V, W and X — live on nine different floors of a building but not necessarily in the same order. The lower most floor of the building is numbered one, the one above that is numbered two and so on till the topmost floor is numbered nine. Only two persons live below the floor on which V lives. Only one person lives between V and P. W lives on an odd numbered floor but not on floor number 7. Only two persons live between W and Q. X does not live on the topmost floor. P does not live on the lowermost floor. S lives immediately below R but R does not live on topmost floor. Neither R nor T live on floor number 6. U lives immediately above P. 187. How many persons live between the floors on which P and S live? (1) Three (2) More than three (3) None (4) Two (5) One 188. Who lives on the floor immediately below V? (1) X (2) T (3) S (4) Q (5) U 189. On which of the following floor numbers does X live? (1) Four (2) One (3) Two (4) Five (5) Seven 190. Which of the following statements is true with respect to U as per the given arrangement? (1) Only three persons live between U and Q. (2) Only three persons live above U. (3) Only one person lives between U and S.

    (4) U lives on an odd numbered floor. (5) None of the given options is true 191. Who lives on floor numbered five? (1) U (2) Q (3) S (4) P (5) Other than those given as options Directions (192–196) : Study the following information carefully and answer the questions given below : (IBPS RRBs Officer CWE (Pre.) Exam, 14.11.2016 (Shift-III))

    Nine friends – P, Q, R, S, T, U, V, W and X — live on nine different floors of a building but not necessarily in the same order. The lower most floor of the building is numbered one, the one above that is numbered two and so on till the topmost floor is numbered nine. T lives on an odd numbered floor below the floor numbered five. Only three people live between T and X. Only one person lives between X and V. V lives above X. Q lives on an odd numbered floor immediately below P. Q lives on the floor numbered five. The number of persons living between X and P is equal to the number of people living between T and S. W lives on an even numbered floor immediately above R. 192. S is related to the floor numbered six and R is related to the floor numbered three in a certain way. X is related to which of the following floor numbers following the same way? (1) Seven (2) Nine (3) Five (4) Eight (5) Four 193. U lives on which of the following floor numbers? (1) Five (2) Four (3) Nine (4) Three (5) Other than those given as options 194. Four of the following five are alike in a certain way as per the given arrangement and thus form a group. Which of the following does not belong to that group? (1) QU (2) VW (3) XT (4) PR (5) SR 195. How many persons live below the floor on which W lives? (1) One (2) Two (3) More than three (4) Three (5) None

    BPRE–670

    196. Which of the following statements is true as per the given arrangement? (1) Only three people live between S and V. (2) None of the given statements is true (3) Q lives immediately above T. (4) More than three persons live above R. (5) U lives on the floor numbered nine. Directions (197–201) : Study the following information carefully and answer the questions given below : (IBPS RRBs Officer CWE (Pre.) Exam, 14.11.2016 (Shift-III))

    Gaurav watches seven movies viz., Gladiator, Braveheart, Titanic, Inception, Chinatown, Avatar and Passion on seven different days of the same week, starting from Monday and ending on Sunday, but not necessarily in the same order. Thus on one day he watches only one movie. Gaurav watches Inception on Friday. He watches only one movie between Inception and Titanic. He watches only three movies between Titanic and Gladiator. He watches only two movies between Titanic and Chinatown. Gaurav watches Avatar immediately before the day he watches Titanic. He watches Passion on one of the days after he watches Avatar. 197. How many movies does Gaurav watch between Braveheart and Passion? (1) Two (2) Three (3) One (4) None (5) Four 198. Four of the following five are alike in a certain way based on the given arrangement and thus form a group. Which is the one that does not belong to that group? (1) Wednesday — Avatar (2) Friday — Braveheart (3) Thursday — Titanic (4) Saturday — Inception (5) Sunday — Chinatown 199. Which of the following statements is true as per the given arrangement? (1) Gaurav watches Inception on the day immediately after the day on which he watches Braveheart. (2) None of the given statements are true.

    PROBLEM SOLVING (3) Gaurav watches only two movies between Avatar and Chinatown. (4) Gaurav watches Passion on Wednesday. (5) Gaurav watches Gladiator on Sunday. 200. On which day of the week does Gaurav watch Avatar? (1) Saturday (2) Sunday (3) Tuesday (4) Wednesday (5) Thursday 201. Which movie does Gaurav watch on Monday? (1) Chinatown (2) Gladiator (3) Titanic (4) Braveheart (5) Passion Directions (202–204) : Read the following intermation carefully and answer the questions given below : (IBPS Bank PO/MT CWE (Main) Exam, 18.11.2016

    The seven members of a family are — P, Q, R, S, T, V and W. Two among them are married couple of different generations. Each of the family members works in different banks — IDBI, HDFC, ICICI, PNB, SBI, BOI and UCO but not necessarily in the same order. None of the females work in either HDFC or ICICI bank. In the family the grandfather works in BOI. S is the son of P. and ‘P’ works in IDBI. W is the daughter-in-law of R. ‘R’ works in SBI. Q is the grandfather of V and ‘V’ works in PNB bank. ‘R’ is the mother of T and ‘T’ works in HDFC bank. 202. In which bank the two married females work? (1) UCO and SBI (2) PNB and UCO (3) SBI and PNB (4) ICICI and SBI (5) None of these 203. How is ‘V’ related to ‘T’ ? (1) Sister (2) Daughter (3) Aunt (4) Data not sufficient (5) None of these 204. Which of the following statements is true ? (1) P and T are brothers (2) R is the grandmother of V (3) The members of the family are of three generations (4) T and W are married couple (5) None of these

    Directions (205–206) : Read the following information carefully and answer the questions given below : (IBPS Bank PO/MT CWE (Main) Exam, 18.11.2016

    Six Cricket players — Tendulkar, Dhoni, Dravid, Lakshman, Kumble and Shewag played different matches in a session against different countries. Kumble had played more matches than Shewag and Lakshman only. Dravid had played more matches than Dhoni but less than Tendulkar. Shewag has not played the least number of matches. The player who had played the second highest number of matches has played eight matches. The person who had played the least number of matches has played only two matches. 205. Which of the following players has probably played eleven matches? (1) Tendulkar (2) Kumble (3) Shewag (4) Dravid (5) Lakshman 206. If it is given that Kumble has played four matches less than Dravid, then how many matches Dhoni has played ? (1) 4 (2) 8 (3) 6 (4) 9 (5) 11 Directions (207–211) : Read the following information carefully to answer the questions given below : (IBPS Bank PO/MT CWE (Main) Exam, 18.11.2016

    Seven persons — Ravish, Ravindra, Ranvijay, Ranveer, Ramashankar, Ramendra and Ritesh are to deliver lecture in seven months of a year-such as — January, February, March, June, August, October and December but not necessarily in the same order. The subjects for lecture are — Anthropology, Psychology, Statistics, Organisational behaviour, Economics, Computer Science and Environmental awareness. Each of the persons likes different fruits — Mango, Litchi, Papaya, Banana, Grapes, Oranges and Apple but not necessarily in the same order. The lecture on Computer Science is neither in February nor in October. The lecture of Ranvijay is in the month that has less than 31 days. The lecture of Ravish is immediately before the lecture of Ramashankar. Ritesh likes apples. The lecture of psychology is held immediately before the lecture on environmental awareness. The

    BPRE–671

    person who likes grapes delivers his lecture in the month having less than 31 days. The person who has to deliver lecture in March, does not like oranges. The lecture on statistics is not in August. Only two persons deliver lectures between the lectures of Ranvijay and Ranveer. The lecture on Anthropology is immediately before the lecture on economics. Only one person delivers lecture before the lecture of the person who likes papaya. The lecture of the person who likes bananas is immediately before the lecture of Ramashankar. The lecture on Anthropology is not delivered in January. The lecture of Ravindra is immediately before the lecture of the person who likes papaya. Three lectures were delivered between Ravindra and the person who likes mangoes. Ramashankar likes neither mango nor papaya. 207. The last lecture is delivered on which subject? (1) Computer Science (2) Anthropology (3) Economics (4) Cannot be determined (5) None of these 208. Which of the following combination is NOT TRUE? (1) August — Ravish — Banana — Environmental Awareness (2) December — Ranveer — Oranges — Anthropology (3) June — Ranvijay — Grapes — Computer Science (4) January — Ritesh — Apple — Organisational behaviour (5) March — Ravindra — Litchi — Statistics. 209. Who among the following likes Mangoes ? (1) Ravindra (2) Ramendra (3) Ranveer (4) Cannot be determined (5) None of these 210. In the given arrangement if months are written in reverse order, eg., January is written as December, February as October and so on, then in which of the following months the lecture of the person, who likes grapes, will be delivered ? (1) February (2) October (3) June (4) March (5) None of these

    PROBLEM SOLVING 211. How many lectures are delivered there between Ritesh and Ravish ? (1) 5 (2) 3 (3) 2 (4) Cannot be determined (5) None of these Directions (212–216) : Study the following information carefully and answer the questions given below : (Indian Bank PO (Pre.) Exam, 21.01.2017 (Ist Sitting))

    Seven persons, namely, P, Q, R, S, T, U and V like seven different cities namely, Agra, Mumbai, Delhi, Jaipur, Patna, Chandigarh and Kolkata. Each of them works in either of the three departments of a company viz. Production, HR and IT with at least two of them in a department. (Note: None of the information given is necessarily in the same order). R works in HR department with the one who likes Mumbai. S works with the one who likes Patna. S does not work with R. P works with only the one who likes Delhi. P neither likes Patna nor works in Production Department. The one who likes Agra works with the one who likes Jaipur. Neither R nor Q likes Jaipur. Q works with only T. U likes Chandigarh. U does not work in IT department. 212. Four of the following five are alike in a certain way as per the given arrangement and hence form a group. Which of the following does not belong to that group? (1) V – Kolkata (2) P-Patna (3) Q- Jaipur (4) S- Chandigarh (5) T- Agra 213. Who amongst the following likes Kolkata? (1) S (2) P (3) T (4) R (5) Q 214. Which of the following statements is true as per the given arrangement? (1) None of the given statements is true (2) R likes Agra (3) U works in the same department in which S works. (4) V works with only the one who likes Chandigarh (5) Both Q and T work in Production Department

    215. Which of the following combinations represent the combination of people working in IT department? (1) S, P and the one who likes Kolkata (2) P and V (3) R, the one who likes Mumbai (4) The one who likes Agra and T (5) T, the ones who like Patna and Agra 216. Which of the following combinations represents the department in which V works and the city he likes? (1) IT-Delhi (2) IT-Kolkata (3) Production-Kolkata (4) HR-Patna (5) Production-Patna Directions (217–221) : Study the following information carefully and answer the questions given below : (Indian Bank PO (Pre.) Exam, 21.01.2017 (Ist Sitting))

    Seven persons — P, Q, R, S, T, U and V live on seven different floors of a building but not necessarily in the same order. The lower most floor of the building is numbered one, the one above that is numbered two and so on till the top most floor is numbered seven. Each one of them likes a different watch- Casio, Citizen, Fossil, Seiko, Omega, Tissot and Fastrack, but not necessarily in the same order. U lives on floor number 4. Only two persons live between U and the one who likes Citizen. Only three persons live between the one who likes Citizen and the one who likes Fastrack. S lives on one of the even numbered floors above the one who likes Fastrack. Only one person lives between S and the one who likes Seiko. V lives on one of the odd numbered floors below the one who likes Seiko. Only one person lives between V and the one who likes Fossil. P lives either immediately above or immediately below the one who likes Fossil. T lives immediately above P. Only one person lives between Q and the one who likes Casio. The one who likes Tissot lives on one of the floors above the one who likes Omega. 217. Who lives immediately below S? (1) Q (2) T (3) U (4) The one who likes Omega (5) The one who likes Tissot

    BPRE–672

    218. How many persons live above the one who likes Seiko? (1) More than three (2) Two (3) None (4) Three (5) One 219. Which of the following watches does T like? (1) Omega (2) Fastrack (3) Fossil (4) Tissot (5) Casio 220. On which of the following floor numbers does Q live? (1) 7 (2) 6 (3) 3 (4) 2 (5) 5 221. How many persons live between the one who likes Fastrack and the one who likes Casio? (1) None (2) One (3) More than three (4) Two (5) Three Directions (222–226) : Study the following information carefully and answer the questions given below : (Indian Bank PO (Pre.) Exam, 21.01.2017 (2nd Sitting))

    Nine people, namely P, Q, R, S, T, U, V, W and X like nine different movies namely, Twilight, Gladiator, Frozen, Inception, Cindrella, Titanic, Watchmen, Vertigo and Dread, Each of them works in either of the three states viz., Kerala, Haryana and Punjab. There are two posts in the states viz., IFS and IPS except Kerala which has only the IFS post. Not more than two people work at the same post. (Note : (1) None of the information given is necessarily in the same order. (2) Example, in the given information, if it is stated that X works with Y then it means that both X and Y works in the same state.) R works as an IFS but not in Kerala. The one who likes Frozen works with R within the same post. The one who likes Watchmen works as an IPS with the one who likes Twilight within the same post. The one who likes Watchmen does not work in the state in which R works. T works in the same state in which the one who likes Twilight works but not in the same post. Both X and P work as an IFS in the same state. The one who likes Titanic works in the same post in the same state in which U works. Neither T nor the IPS likes Titanic. S likes Gladiator. S does not work in Haryana.

    PROBLEM SOLVING The one who likes Inception works with S. T does not like Inception. V neither likes Watchmen nor works with S. The one who likes Vertigo works with V. Q does not work with the one who likes Vertigo. X does not like Dread. 222. Which of the following combinations represents the posts for which U works and the movie he likes ? (1) IFS-Dread (2) IFS- Watchmen (3) IPS- Vertigo (4) IPS- Dread (5) IFS- Frozen 223. Which of the following statements is not true as per the given arrangement ? (1) Q works in Punjab with the one who likes Cindrella. (2) All the given statements are true. (3) R works at the same post at which X works. (4) Both T and V work in the same state. (5) R likes Titanic. 224. Who amongst the following likes Cindrella ? (1) Q (2) V (3) X (4) S (5) T 225. Which of the following combinations is true with respect to the given arrangement ? (1) U - IFS Punjab - Inception (2) P - IFS- Kerala - Cindrella (3) T - Haryana- IPS - Vertigo (4) V - IPS- Haryana - Twilight (5) S - IFS- Kerala - Gladiator 226. Which of the following combinations of persons works in the State of Haryana ? (1) Q, R and U (2) T, V and W (3) P, X and W (4) V, W and X (5) T, W and X Directions (227–231) : Study the following information carefully and answer the questions given below : (IBPS Bank PO/MT CWE (Main) Exam, 18.11.2016

    Twelve people, namely R, S, T, U, V, W, F, G, H, I, J and K have exams in one of the months viz. March, April, May, August, September and December of the same year. In each month, the exam will be held on 12th and 27th with each person appearing on a different date.

    ● U has an exam after August. If U’s exam in a month having odd number of days, then this exam is also on an odd numbered date. If his exam is in a month having even number of days, his exam is on an even numbered date. ● Only three people have exams between U and K. ● H’s exam is on an odd number date in a month having even number of days. H’s exam is after K. ● Only three people have exams between R and W. R has his exam before W. R neither has his exam in March nor April. R’s exam is not immediately before U. W’s exam is not in September. ● As many people have an exam between R and U as between T and F. T’s exam is before F. T’s exam is on an even numbered date. ● Only two people have exam between F and S. V’s exam is before S but after J. ● More than three people have exam between V and I. 227. Which of the following correctly represents the date and month on which F has his exam ? (1) 12th April (2) 27th May (3) 12th May (4) 27th December (5) 12th March 228. Who amongst the following has an exam on 27th April? (1) I (2) V (3) G (4) S (5) H 229. How many people have exams between J and S ? (1) Five (2) Four (3) One (4) Three (5) Two 230. Which of the following statements is true regarding I ? (1) Only two people have exams after I. (2) I has an exam in May. (3) I has an exam immediately before W. (4) I has his exam on 12th. (5) None of the given options is true.

    BPRE–673

    231. As per the given arrangement, four of the following five are alike in a certain way and thus form a group. Which is the one that does not belong to that group ? (1) UI (2) RH (3) KS (4) JK (5) VR Directions (232–237) : Study the following information carefully and answer the questions given below : (IBPS SO (Agriculture) Exam, 29.01.2017)

    Seven people namely M, N, O, P, Q, R and S like seven different genres of music namely Classical, Hip-hop, Pop, Jazz, Opera, Rock and Electronics but not necessarily in the same order. Each person also works in the same office but at a different position on the basis of seniority namely Trainee (TE), Assistant Manager (AM), Manager (MG), Senior Manager (SM), Chief Manager (CM), Executive Director (ED) and Director (DR) but not necessarily in the same order. (Please Note: The positions have been given in increasing order of seniority with TE being the junior most position whilst DR being the senior most position.) Only one person is senior than O. The one who is senior than O likes HipHop. Only two people are junior than R. M is junior than R and likes Electronica. M is not the junior most. Q is junior than the CM but senior than the one who likes Jazz. The one who likes is not the senior most. S likes Classical. The one who likes Pop is senior than Q but not the ED. P does not like Pop. 232. As per the given arrangement, R is related to Opera and Q is related to classical in a certain way. To which of the following is P related to in the same way? (1) Pop (2) Jazz (3) Rock (4) Electronica (5) Hip-hop 233. Who amongst the following works as a MG? (1) R (2) Q (3) S (4) P (5) Other than those given as options 234. Which of the following pairs represents the people who have less experience than P and more experience than Q? (1) M, S (2) R, O (3) N, P (4) M, O (5) N, 0

    PROBLEM SOLVING 235. Who amongst the following works as a TE? (1) P (2) N (3) R (4) S (5) O 236. Which combination represents the position at which N works and the music genres that he likes? (1) CM-Rock (2) CM-Pop (3) SM-Classical (4) MG-Opera (5) MG-Jazz 237. Which of the following music genres does R like? (1) Electronica (2) Pop (3) Rock (4) Hip-Hop (5) Jazz Directions (238 – 242) : Study the following information carefully and answer the questions given below : (IBPS SO (Agriculture) Exam, 29.01.2017)

    Eight people P, Q, R, S, T, U, V and W, live on eight different floors of building (but not necessarily in the same order.) The lowermost floor of the building is numbered one, the one above that is numbered two, and so on till the topmost floor is numbered eight. Each one of them plays a different game namely- San Andreas, Roadrash, Contra, Castlevania, Resident Evil, Tekken3, Dragon Quest and Metal Gear (but not necessarily in the same order). V lives on an even numbered floor above the floor numbered two. Only three people live between the V and the one who plays Roadrash. T lives immediately below the one who plays Roadrash. Only two people live between T and the one who plays Tekken3. R lives immediately above the one who plays Tekken3. The one who plays San Andreas lives on an even numbered floor below R. Only two people live between the one who plays San Andreas and the one who plays Dragon Quest. The one who plays Metal Gear lives immediately below Q. Q neither lives on the topmost floor nor plays Roadrash. U lives on odd numbered floor but not the lowermost floor. Only two people live between U and the one who plays Resident Evil. Only one person lives between D and the one who plays Contra. W lives immediately below the one who plays Castlevania.

    238. Which of the following games does S play? (1) San Andreas (2) Roadrash (3) Tekken3 (4) Metal Gear (5) Castlevania 239. How many people live between V and the one who plays Tekken3? (1) One (2) None (3) Two (4) Four (5) Five 240. Which of the following statements is TRUE with respect to the given information? (1) V plays Resident Evil (2) R lives immediately below the one who plays Contra (3) All the give statements are true (4) Q lives immediately above T. (5) Only four people live between U and the one who play roadrash. 241. Who amongst the following live exactly between P and the one who plays Dragon Quest? (1) S, the one who plays Resident Evil (2) S, Q (3) T, W (4) T, The one how plays Tekken 3 (5) U, The one who plays Roadrash 242. Four of the following five are alike in a certain way and so form a group. Which one of the following does not belong to the group? (1) P–Contra (2) W–Castlevania (3) U–Tekken 3 (4) Q–Dragon Quest (5) B–Roadrash Directions (243–247) : Study the following information carefully and answer the questions given below : (IBPS SO (Agriculture) Exam, 29.01.2017)

    Ten persons from different companies viz. Indigo, Biocon, Nokia, Samsung, Havells, Amul, Tata, Vadilal, Flipkart and Cipla are sitting in two parallel rows containing five people each, in such a way that there is an equal distance between adjacent persons. In row-1 G, H, I, J and K are seated and all of them are facing south. In row-2 U, V, W, X and Y are seated and all of them are facing north. Therefore, in the given seating arrangement, each member seated in a row faces another

    BPRE–674

    member of the other row. (All the information given above does not necessarily represent the order of seating as in the final arrangement). K sits third to the left of the person from Biocon. V is an immediate neighbour of the person who faces K. The person from Havells sits to the immediate left of V. Only two people sit between the person from Nokia and H. The person from Nokia is not an immediate neighbour of K. Only one person sits between the one who faces H and the person from Tata. X sits second to the right of W. W does not sit at an extreme end of the line. The person from Cipla sits third to the left of the one who faces W. U is an immediate neighbour of the person from Samsung. U is not from Tata. Only two people sit between the person from Amul and J. The person from Flipkart is an immediate neighbour of the person who faces the person from Amul. Only one person sits between the one who faces the person from Flipkart and I. Neither I nor U is from Indigo. 243. K is related to the person from Vadilal in the same way as W is related to person from Tata based on the given arrangement. To who amongst the following is, X related to, following the same pattern? (1) The person from Amul (2) The person from Indigo (3) The person from Nokia (4) The person from Havells (5) The person from Flipkart 244. Y is from which of the following companies? (1) Indigo (2) Tata (3) Amul (4) Samsung (5) Biocon 245. Who amongst the following faces the person from Vadilal? (1) The person from Flipkart ? (2) V (3) Y (4) The person from Nokia (5) The person from Havells 246. Which of the following is true regarding G? (1) Only one person sits between G and the person from Cipla. (2) The person from Samsung faces G.

    PROBLEM SOLVING (3) G is an immediate neighbour of the person from Biocon. (4) None of the given options is true (5) G sits to the immediate left of L. 247. Who amongst the following sit at extreme and sof the rows? (1) The persons from Indigo and G (2) The person from Cipla and X (3) K, Y (4) J and the person from Flipkart (5) The person from Nokia and U Directions (248–250) : Study the following information carefully and answer the questions given below : (SIDBI Officer Online Exam.24.02.2016)

    There are five jars – A, B, C, D and E – each having different numbers of cookies. Jar A has more cookies than jar E but less than jar B. Jar D has more cookies than jar B. Jar D does not have the most number of cookies. The jar having the third least number of cookies has 11 cookies. The jar having the most number of cookies has 24 cookies. 248. Which of the following jars probably has 17 cookies? (1) B (2) E (3) Either ‘A’ or ‘E’ (4) D (5) Either ‘C’ or ‘B’ 249. Which of the following jars has the second lowest number of cookies? (1) D (2) A (3) E (4) C (5) B 250. How many cookies does jar E possibly have? (1) 22 (2) 11 (3) 5 (4) 19 (5) 12 Directions (251–255) : Study the following information carefully answer the questions given below : (Bank of Baroda Exam, 25.09.2016)

    Seven stores viz, A, B, C, D, E, F and G sold different number of TV’s of a particular brand viz. 4, 6, 9, 12, 15, 18 and 27 on seven different days of the same week starting from Monday and ending on Sunday, but not necessarily in the same order.

    A sold the TV’s on one of the days after Friday. Only three stores sold TV’s between A and the store which sold 12 TV’s. The store which sold TV’s on the day immediately after the store which sold 12 TV’s sold more than 12 TV’s but not odd number of TV’s. ● Only two stores sold TV’s between F and the store which sold 4 TV’s. The store which sold 4 TV’s sold these before F. 4 TV’s were not sold on Monday. ● Only three stores sold TV’s between F and D. The difference between the number of TVs sold on Wednesday and Saturday is more than 10. ● B sold the TVs on a day immediately before the day on which 9 TV’s were sold. 9 TV’s were not sold on Friday. ● Only two stores sold TV’s between B and G. ● Difference between the number of TV’s sold by G and D is less than 5. C did not sell the least number of TV’s. 251. In which of the following pairs did both the stores sell odd number of TV’s? (1) D, E (2) A, B (3) E, F (3) F, G (4) C, D 252. Which of the following stores sold 4 TV’s? (1) F (2) A (3) C (4) G (5) E 253. Based on the given arrangement, which of the following statements true? (1) D sold the TV’s on Wednesday. (2) Only one store sold TV’s between C and F. (3) A sold 12 TV’s on Sunday. (4) C sold 18 TV’s on Thursday. (5) None of the given options is true. 254. How many stores sold less number of TV’s than B? (1) Five (2) Three (3) Six (4) Two (5) One 255. What is the difference between the number of TV’s sold on Monday and Friday? (1) 9 (2) 12 (3) 18 (4) 3 (5) 11 ●

    BPRE–675

    Directions (256–258) : Study the following information carefully and answer the questions given below : (Bank of Baroda Exam, 25.09.2016)

    Six friends Sunny, Saksham, Rishi, Rupali, Preeti and Sandeep won different number of chocolates at an event on the same day. ● Rishi won less chocolates than Sandeep but more than Rupali and Preeti. ● Rishi won less chocolates than only two persons. ● Preeti won less chocolates than Rupali but more than Sunny. Sandeep didn’t win the maximum number of chocolates. ● The one who won second highest number of chocolates won 41 chocolates. Preeti won 25 chocolates. 256. How many chocolates did Saksham probably win? (1) 44 (2) 41 (3) 20 (4) 40 (5) 38 257. Which of the following possibly won 20 chocolates? (1) Rishi (2) Saksham (3) Sandeep (4) Sunny (5) Rupali 258. If Rishi won 32 chocolates, then how many chocolates did Rupali probably win? (1) 23 (2) 27 (3) 21 (4) 33 (4) 37 Directions (259–264) : Study the following information carefully and answer the questions given below : (IBPS Bank PO/MT CWE (Pre Exam), 16.10.2016 (First Sitting))

    Seven athletes – M, N, O, P, Q, R and S – live on seven different floors of a building but not necessarily in the same order. The lower most floor of the building is numbered one, the one above that is numbered two and so on0 till the topmost floor is numbered seven. Each one of them runs for a different distance in a marathon – 750m, 1200m, 2200m, 2900m, 3600m, 4300m and 5000m, but not necessarily in the same order. The one who runs for 4300m lives on floor number 4. Only one person lives between M and the the one who runs for 4300m. Only two people live between M and S. The one who runs for 1200m lives on one of the even

    PROBLEM SOLVING numbered floors above R. Only two people live between the one who runs for 1200m and the one who runs for 2900m. N lives on one of the floors above M. N ran for 2100m more than the one who lives on floor number 3. The number of people living between the one who runs for 5000m and S is same as the number of people living between M and R. Only one person lives between R and Q. The one who runs for the shortest distance lives immediately below Q. Only one person lives between P and the one who runs for 3600m. 259. Which of the following live(s) between O and the one who runs for 2900m? (1) Only the one who runs for 2200m (2) Both M and R (3) Both M and the one who runs for 3600m (4) Both P and the one who runs for 2200m (5) Only R 260. As per the given arrangement, four of the following five are alike in a certain way and so form a group. Which one of the following does not belong to the group? (1) M-4300 m (2) Floor number 7-S (3) Floor number 4-N (4) P-3600m (5) Floor number 5-5000m 261. How many people live between S and O? (1) Five (2) One (3) None (4) Four (5) Three 262. Who amongst the following runs for 2900m ? (1) S (2) M (3) N (4) P (5) Q 263. If the total distance covered by B and M is 4800m, then how much did B run alone ? (1) 4050m (2) 2600m (3) 1200m (4) 3600m (5) 1900m 264. Which of the following statements is true with respect to the given arrangement ? (1) None of the given options is true (2) Only two people live between P and R (3) M lives on floor number 2.

    (4) The one who runs for 750m lives immediately above S (5) Q runs for 3600m Directions (265–267) : Study the following information carefully and answer the questions given below : (IBPS Bank PO/MT CWE (Pre Exam), 16.10.2016 (First Sitting))

    Six books–A, B, C, D, E and F – each of different thickness, are kept on a table. F is thicker than B but thinner than C. A is thinner than both B and E, but not thinnest. E is thinner than F. The second thinnest book is 7 cms thick and the second thickest book is 13cms thick. (Note : The thickness of all the books is in whole numbers.) 265. If E is 12cm thick, then which of the following statements is true about E ? (1) F is 2cms thicker than E. (2) The total thickness of E and C together is 22 cms. (3) E is thinner than D. (4) All the given statements are true (5) E is the third thickest book of all 266. If A is 2cm thicker than D, then how thick is D ? (1) Cannot be determined (2) 15 cm (3) 5 cm (4) 11 cm (5) 3 cm 267. With respect to the thickness of given books, if B + A = 17 cm then F + B = ? (1) 17 cm (2) 23 cm (3) 21 cm (4) 19 cm (5) 24 cm Directions (268–270) : Study the following information carefully and answer the questions given below : (IBPS Bank PO/MT CWE-VI (Pre), 16.10.2016 (Second Sitting))

    There are six wires in a cable A, B, C, D, E and F and they have different length but not necessarily in the same order. E is greater than C but less than D and B. A is greater than D and B. A is not the longest wire. F is 13 cm. long and E is 4 cm. long. 268. If D is 5cm less than F what would be the length of D? (1) 7 (2) 8 (3) 9 (4) Can’t be determined (5) None of these

    BPRE–676

    269. Which wire has least length? (1) B (2) A (3) C (4) E (5) None of these 270. If A is 10 cm. long and B carries 5 cm. length then what would be the length of C? (1) 2 (2) 6 (3) 7 (4) 9 (5) 8 Directions (271–276) : Study the following information carefully to answer the questions given below : (IBPS Bank PO/MT CWE-VI (Pre Exam), 16.10.2016 (Second Sitting))

    M, N, O, P, Q, R and S seven people live on seven different floors of a building but not necessarily in the same order. The lower most floor of the building is numbered 1, the one above that is numbered 2 and so on till the topmost floor is numbered 7. Each one of them have different income i.e. Rs. 3500, Rs. 15000, Rs. 7500, Rs. 9000, Rs. 11000, Rs. 13500 and Rs. 5000. (But not necessarily in the same order.) M lives on an odd numbered floor but not on the floor numbered 3. The one who has income of Rs. 11000 lives immediately above M. Only two people live between M and the one who has income of Rs. 7500. The one who has income of Rs. 15000 lives on one of the odd numbered floors above P. Only three people live between O and the one who has income of Rs. 15000. The one who has income of Rs. 7500 lives immediately above O. The one who has income of Rs. 3500 lives immediately above the one who has income of Rs. 5000. S lives on an odd numbered floor but not on the topmost floor. Only one person lives between N and Q. N lives on one of the floors above Q. Neither Q nor M has income of Rs. 9000. Q does not has income of Rs. 7500. 271. How much income M has? (1) Rs. 13500 (2) Rs. 5000 (3) Rs. 7500 (4) Rs. 15000 (5) Rs. 3500 272. Which of the following combinations is true with respect to the given arrangement? (1) Rs. 13500 – O (2) Rs. 15000 – R (3) Rs. 5000 – O (4) Rs. 11000 – P (5) Rs. 9000 – N

    PROBLEM SOLVING 273. If all the people are made to live in alphabetical order from top to bottom, the positions of how many people will remain unchanged? (1) Four (2) Two (3) None (4) One (5) Three 274. Which of the following statements is true with respect to the given arrangement ? (1) The one who has income of Rs. 5000 lives immediately below M. (2) R has income of Rs. 15000. (3) None of the given options is true. (4) Only four people live between P and S. (5) S lives immediately below Q. 275. Who amongst the following lives on the floor numbered 2? (1) N (2) The one who has income of Rs. 3500 (3) The one who has income of Rs. 5000 (4) P (5) Q 276. How much income R has? (1) Rs. 13500 (2) Rs. 5000 (3) Rs. 7500 (4) Rs. 15000 (5) Rs. 3500 Directions (277–279) : Study the following information carefully and answer the questions given below : (IBPS Bank PO/MT (Pre.) Exam, 23.10.2016)

    Six books – A, B, C, D, E and F — each of different thickness, are kept on a table. C is thicker than A, but thinner than E. A is thicker than both B and D. E is not the thickest. The third thinnest book is 9cms thick and the thickest book is 16cms thick. (Note the thickness of all the books are in whole numbers.) 277. If E is 5cm thicker than A, then how thick is E? (1) 11 cm. (2) 9 cm. (3) 12 cm. (4) 14 cm. (5) Cannot be determined 278. With respect to the thickness of the given books, if C + F = 27, then A + C = ? (1) 20 (2) Other than those given as options (3) 23 (4) 15 (5) 19

    279. If B is 8cm thick, then which of the following statements is true about B? (1) B is the third thinnest book of all. (2) F is 5cms thicker than B. (3) B is thicker than D. (4) All the given statements are true (5) B is the thinnest book of all. Directions (280–282) : Study the following information carefully and answer the questions given below : (Bank of Maharashtra PO Exam, 26.10.2016)

    Each of the six stores - L, M, N, O, P and Q — sold different number of computers in one day. M sold more computers than P but less than both O and Q. L sold less computers than P but more than N. The store which sold third highest number of computers sold 29 computers. The store which sold the minimum computers sold 6 computers. 280. If the number of computers sold by O is 30, then which of the following statements is true ? (1) None of the given option is true (2) O and N together definitely sold less than 35 computers (3) Only two stores sold more computers than O (4) O sold maximum number of computers (5) Q sold more computers than O 281. If the number of computers sold by P is a multiple of three, more than 25, how many computers were sold by P ? (1) 30 (2) 27 (3) 36 (4) 33 (5) Cannot be determined 282. How many stores sold less computers than L ? (1) One (2) Three (3) Two (4) Five (5) None as L sold the lowest number of computers Directions (283–287) : Study the following information carefully and answer the questions given below : (IBPS RRBs Officers CWE (Prelim Exam) 09.09.2017 Ist Sitting)

    Eight boxes — G, H, I, J, K, L, M and N — are stacked above one another but not necessarily in the same order. Only two boxes are kept between

    BPRE–677

    G and H. Only two boxes are kept between G and I. As many boxes are kept between H and M as between H and K. M is kept at one of the positions above K. Only one box is kept between J and L. N is kept immediately below J. 283. What is the position of Box I in the given stack of boxes? (1) Third from the bottom (2) Fourth from the top (3) Third from the top (4) Fourth from the bottom (5) First from the bottom 284. How many boxes are kept between Boxes N and G? (1) One (2) Two (3) More than three (4) Three (5) None 285. As per the given arrangement, N is related to I in the same way as G is related to J. Following the same pattern,to whom is K related? (1) Other than those given as options (2) H (3) L (4) G (5) M 286. Which boxes is/are kept between the box that is third from the top of the stack and Box J? (1) Only H (2) Only M (3) Both N and I (4) None of those given as options (5) Only L 287. Which of the following pairs of boxes is kept at the top and bottom of the stack respectively? (1) L, K (2) M, I (3) L, N (4) K, G (5) G, M Directions (288–292) : Study the following information carefully and answer the questions given below : (IBPS RRBs Officers CWE (Prelim Exam) 09.09.2017 Ist Sitting)

    M, T, D, F, H, R and W are seven students studying in three different Colleges I, II and III with at least two in each college. Each of them has a favourite subject from–English, History, Geography, Mathematics, Physics, Chemistry and Biology, but not necessarily in the same order. D’s favourite subject is Physics and studies in College II with only M. H does not study in College III and he likes English. F studies in College III

    PROBLEM SOLVING and does not like Mathematics. Those who like Geography and Chemistry study in the same college. W likes Biology and does not study in College I. R does not study with H. R does not like Chemistry. M does not like History. 288. What is M’s favourite subject? (1) Mathematics (2) Chemistry (3) Data inadequate (4) Geography (5) Other than those given as options 289. In which college do three of them study? (1) I (2) II or III (3) II (4) III (5) Data inadequate 290. Which of the following groups of students studies in the College III? (1) FTR (2) FWR (3) FM (4) Data inadequate (5) Other than those given as options 291. What is T’s favourite subject? (1) Chemistry (2) Biology (3) Mathematics (4) Data inadequate (5) Other than those given as options 292. Which of the following groups of students study in College I? (1) TR (2) HR (3) HT (4) HF (5) Other than those given as options Directions (293–297) : Study the following information carefully and answer the questions given below : (IBPS RRBs Officers CWE (Prelim Exam) 09.09.2017 IInd Sitting)

    Seven people viz, M, N, O, P, Q, R and S buy a car in seven different months of the same year viz, January, February, March, April, May, June and July but not necessarily in the same order. P buys a car in a month having 31 days after April. Only two people buy a car between P and N. Only three people buy a car between O and S. O buys a car in one of the months before S. S does not buy a car in June. R buys a car in the month immediately before M.

    293. How many people buy a car between R and N? (1) One (2) Three (3) None (4) More than three (5) Two 294. Who buys the car in one of the months before R? (1) Only N (2) Both O and S (3) Only S (4) Both N and P (5) Only O 295. Which of the following statements is true as per the given information? (1) N buys a car in one of the months before R. (2) None of the given statements is true. (3) Only four people buy a car between O and M. (4) P buys a car in May. (5) O buys a car in the month immediately before S. 296. In which month did Q buy a car? (1) April (2) January (3) July (4) June (5) February 297. Four of the following five are alike in a certain way based on the given arrangement and thus they form a group. Which is the one that does not belong to the group? (1) O–April (2) Q–March (3) R–July (4) S–February (5) M–June Directions (298–302) : Study the following information carefully and answer the questions given below : (IBPS RRBs Officers CWE (Prelim Exam) 09.09.2017 IInd Sitting)

    Eight boxes – J, K, L, M, R, S, T and U are stacked one above another. R is kept immediately above J. Only three boxes are kept between J and K. Only two boxes are kept between L and K. L is kept exactly between J and S. Only three boxes are kept between S and T. S is kept at one of the positions above T. M is not kept immediately above T. 298. In the given arrangement, four of the following five are alike in a certain way and hence they form a group. Which one of the following does not belong to the group? (1) RL (2) SJ (3) MT (4) SK (5) UM

    BPRE–678

    299. Which one of the following correctly depicts the position of Box L? (1) Fourth box below R (2) Second box above M (3) Immediately below T (4) Second from the top of the stack (5) First from the top of the stack 300. If all the boxes are arranged in alphabetical order from top to bottom, then the position of how many boxes will remain unchanged? (1) Two (2) Three (3) More than three (4) One (5) None 301. Which box is kept third from the bottom of the stack? (1) K (2) L (3) U (4) J (5) R 302. Which box/es is/are kept between S and U? (1) Both M and T (2) Only M (3) Both J and L (4) Only L (5) Both M and K Directions (303–306) : Study the following information carefully and answer the questions given below : (IBPS RRBs Officers CWE (Prelim Exam) 10.09.2017)

    Seven people viz. M, N, O, P, Q, R and S buy a car in seven different months of the same year viz. April, May, August, September, October, November and December, but not necessarily in the same order. R buys a car in one of the months before July. Only three people buy a car between N and R. Q buys a car in a month having only 30 days immediately before M. P buys a car in a month having only 30 days immediately after S. 303. How many people buy a car between S and M ? (1) None (2) Two (3) One (4) Three (5) Four 304. Four of the following five are alike in a certain way based on the basis of given arrangement and thus they form a group. Which is the one that does not belong to the group ? (1) M-November (2) O-August (3) P-September (4) R-May (5) Q-October

    PROBLEM SOLVING 305. Which of the following statements is not true as per the given information ? (1) All the given statements are true (2) O buys a car in a month having 31 days. (3) N buys a car in a month immediately after P. (4) Only two people buy a car betwen M and P. (5) S buys a car in May. 306. In which month did O buy a car ? (1) October (2) May (3) November (4) September (5) August 307. Who buy(s) a car in the month(s) after N ? (1) Both Q and M (2) Only S (3) Both P and Q (4) Only M (5) Only Q Directions (308–311) : Study the following information carefully and answer the questions given below : Eight boxes — Q, R, S, T, U, W, X and Z — are stacked above one another. Only one box is kept between X and Q. Only three boxes are kept between X and W. X is kept at one of the positions below W. S is kept immediately below W. There are as many boxes between W and Q as between S and R. Only four boxes are kept between R and Z. T is kept at one of the positions above Z. 308. As per the given arrangement, four of the following five are alike in a certain way and hence they form a group. Which one of the following does not belong to the group ? (1) RX (2) ZT (3) WQ (4) RQ (5) UW 309. Which box is kept fourth from the bottom of the stack ? (1) S (2) Z (3) W (4) R (5) Q 310. Which of the following correctly depicts the position of the Box X? (1) Fourth box below Q (2) Second box above Z (3) Third from the top of the stack (4) Immediately below W (5) First from the bottom of the stack.

    311. Which box/boxes is/are kept between Q and X ? (1) Both Z and W (2) Only Z (3) Both R and Z (4) Only R (5) None 312. If all the boxes are arranged in alphabetical order from top to bottom, then the positons of how many boxes will remain unchanged ? (1) More than three (2) None (3) Three (4) One (5) Two Directions (313–316) : Study the following information carefully and answer the questions given below : (IBPS RRBs Officer CWE (Prelim Exam) 16.09.2017)

    Seven people viz. F, G, H, I, J, K and L were promoted in seven different months of the same year viz., February, April, June, July, September, October and December, but not necessarily in the same order. J was promoted in a month having 31 days but not in July. Only three people were promoted between J and L. I was promoted in a month having only 30 days immediately before F. G was promoted immediately before K. 313. In which month was G promoted? (1) October (2) April (3) June (4) September (5) February 314. As per the given arrangement, G is related to April following a certain pattern. To which of the following is I related following the same pattern as per the given arrangement? (1) December (2) October (3) February (4) September (5) June 315. Who was promoted in July? (1) H (2) K (3) L (4) F (5) Other than those given as options 316. Which of the following statements is not true about K as per the given information? (A) K was promoted in a month having 31 days. (B) K was promoted in one of given months after L. (C) Only two people were promoted between K and I.

    BPRE–679

    (1) Only C (2) Both A and C (3) Both B and C (4) Both A and B (5) Only B 317. How many people were promoted before H? (1) Four (2) None (3) Three (4) Two (5) One Directions (318–321) : Study the following information carefully and answer the questions given below : (IBPS RRBs Officer CWE (Prelim Exam) 16.09.2017)

    Eight boxes–A, B, C, D, W, X, Y and Z – are stacked above one another. More than one box is kept between A and B. As many boxes are kept between B and W as between A and B. Only one box is kept between W and X. More than three boxes are kept between X and D. No box is kept between Y and C. Only two boxes are kept between C and Z. C is kept at one of the positions above Z. 318. As per the given arrangement, D is related to A in the same way as X is related to Z. Following the same pattern to whom is C related? (1) V (2) W (3) B (4) D (5) X 319. Which one of the following depicts the position of box Y with respect to box W? (1) Immediately before W (2) Fifth above W (3) Fourth below W (4) Second above W (5) Third above W 320. How many boxes are kept above box W? (1) More than three (2) Three (3) None (4) One (5) Two 321. Which boxes are kept at second and fifth positions from the top of the stack respectively? (1) Z, C (2) Z, W (3) A, D (4) A, B (5) None of those given as options 322. Which box/es is/are kept between D and C? (1) Only Y (2) None (3) Only W (4) Both B and W (5) Both A and Y

    PROBLEM SOLVING Directions (323-326) : Study the following information carefully and answer the questions given below : (IBPS Bank PO/MT CWE-VII (Prelim Exam) 14.10.2017)

    Seven people viz. T, U, V, W, X, Y and Z study in three branches of an engineering college viz. Aeronautical, Chemical and Electrical. Each of them also likes a different sport viz., Hockey, Cricket, Football, Volleyball, Boxing, Archery and Wrestling. Atleast two people study in each branch. (Note : None of the given information is necessarily in the same order). T studies in Aeronautical with only the one who likes Archery. The one who likes Wrestling studies with the one who likes Hockey. U studies with W and the one who likes Cricket. Neither U nor W likes either Wrestling or Hockey. Z studies with the one who likes Boxing. Neither T nor U likes Boxing. Y studies with V. V neither studies Electrical engineering nor likes Wrestling. T does not like Volleyball. 323. Who amongst the following studies Chemical Engineering? (1) Y (2) U (3) The one who likes Volleyball (4) W (5) The one who likes Cricket 324. Which of the following is true as per the given arrangement? (A) Y likes Cricket. (B) X studies Aeronautical Engineering. (C) The one who likes Boxing studies Electrical Engineering. (1) Only C (2) Only A (3) Both A and B (4) Only B (5) Both B and C 325. Which sport does X likes? (1) Wrestling (2) Archery (3) Boxing (4) Volleyball (5) Cricket 326. Four of the following five study in the same branch based on the given arrangement and thus they form a group. Who amongst the following study in different branch as per the given arrangement? (1) T – X (2) Football-Archery (3) Wrestling-Boxing (4) Y – Hockey (5) W – Volleyball

    327. Who amongst the following likes Football? (1) W (2) U (3) V (4) T (5) Y Directions (328-331) : Study the following information carefully and answer the questions given below : (IBPS Bank PO/MT CWE-VII (Prelim Exam) 14.10.2017)

    Seven people viz. J, K, L, M, N, O and P advertise for a brand viz. Fastrack, Biba, Nike, Puma, Marigo, Vivo and Zara in seven different months of the same year viz., February, April, May, June, August, September and November. (Note : None of the given information is necessarily in the same order. No one advertises in any other month of the given year.) Only two people advertise between P and the one who advertises for Puma. P advertises in a month having 31 days. More than three people advertise between the one who advertises for Puma and the one who advertises for Biba. Only three people advertise between the one who advertises for Biba and K. No one advertises between K and the one who advertises for Marigo. Only three people advertise between O and L. L advertises in one of the months before O. The one who advertises for Fastrack advertises in a month immediately before the one who advertises for Nike. Only two people advertise between N and the one who advertises for Vivo. Only one person advertises between M and the one who advertises for Zara. M advertises in one of the months before June. 328. Who advertises in the month of April? (1) The one who advertises for Fastrack (2) N (3) The one who advertises for Puma (4) J (5) M 329. How many people advertise between M and the one who advertises for Nike? (1) Three (2) More than three (3) Two (4) None (5) One 330. Advertisement for Marigo is done in which month?

    BPRE–680

    (1) April (2) September (3) August (4) June (5) May 331. As per the given arrangement, L is related to the one who advertises for Zara following a certain pattern. In the same pattern, N is related to the one whom advertises for Fastrack. Following the same pattern to whom amongst the following is O related to? (1) Biba (2) Puma (3) Marigo (4) Vivo (5) Nike 332. Which of the given combinations is correct as per the given arrangement? (1) J – Zara (2) L – May (3) N – Fastrack (4) O – November (5) M – Vivo Directions (333–336) : Study the following information carefully and answer the questions given below : (IBPS RRBs Officer CWE (Main Exam) 05.11.2017)

    Eight friends A, B, C, D, E, F, G and H were born on 7th, 9th, 16th and 19th in March and July. Each one of them likes either a colour or a fruit. The persons who likes colours were born on that day which is a perfect square and they likes Yellow, Green, Red and Blue. The persons who were born on the day which denotes a prime numbers like fruits – Apple, Cherry, Mango and Banana. (Note : None of the given information is necessarily in the same order). The one who likes yellow colour was born on a day which is a perfect square in the month of March. No person was born between D and the one who likes yellow. D doesn’t like colour. Only three persons were born between D and the one who likes Mango. No person is born between the one who like Mango and Blue colour. The number of people born after the one who likes blue colour is one less than the number of persons born before A. The one who likes Apple was born immediately before B. B does not like Mango. The number of people born before B is same as the number of persons born after G. Only three persons were born between the one who likes Green and H. H and the one who likes Cherry were born in the same month

    PROBLEM SOLVING but not in March. C was born after the one who likes Blue colour. E doesn’t like any colour. F doesn’t like Apple. 333. Who among the following likes green colour? (1) B (2) A (3) G (4) H (5) C 334. Who was born on 16th March? (1) D (2) A (3) G (4) F (5) E 335. Who among the following likes cherry? (1) A (2) G (3) H (4) F (5) B 336. Who was born on 19th July? (1) E (2) A (3) D (4) B (5) C 337. Who among the following likes red colour? (1) A (2) B (3) H (4) C (5) D Directions (338–342) : Study the following information carefully and answer the questions given below : (IBPS RRBs Officer CWE (Main Exam) 05.11.2017)

    Ten persons Q, R, S, T, U, V, W, X, Y and Z are going to four cities on four different dates i.e., 2nd, 4th, 7th and 9th of the month. Each person goes to different cities on different dates. Four cities are namely Pune, Chandigarh, Kochi and Patna, but not necessarily in the same order. At least one person goes on each date. No person goes to Pune on 4th of the month. R does not go to any city on 2nd and 7th of the month. The persons, who go on 7th go to different cities. Y goes to Kochi only with that person who goes on 7th of the month. Maximum four persons can go on the same date. Only one person goes to Chandigarh. Z does not go on 7th of the month. S does not go to Pune but go with U, who goes on 9th of the month. The person going to Kochi, visits on even number date of the month, which is more than 3. R goes to Pune with Q, but does not go with X. X goes to that city to which most of the persons are going. V does not go with S and Z but go with another person. Z goes to Pune. T does not go with U. W does not go on 9th and 2nd of the

    month. X goes on an odd number date of the month and no any person goes on that date on which X goes to that city. The person going to Pune, visits on even number date of the month. S goes on an odd number date of the month. 338. Who among the following goes to Chandigarh? (1) R (2) T (3) X (4) V (5) Q 339. To which of the following city most of the persons are going? (1) Chandigarh (2) Pune (3) Kochi (4) Both Kochi and Pune (5) Patna 340. Four of the following five are alike in a certain way based on their positions in the above arrangement and so form a group. Which is the one that does not belong to that group? (1) S (2) R (3) Z (4) X (5) T 341. Which of the following statements is true regarding U? (1) Only U goes to Chandigarh (2) U goes on 7th of the month (3) None of the option is true (4) U goes to that city in which most of the persons are going (5) U goes with Z 342. Who among the following goes on 2nd of the month? (1) Z (2) Q (3) W (4) Y (5) S Directions (343–348) : Study the following information carefully and answer the questions given below : (IBPS RRBs Officer CWE (Main Exam) 05.11.2017)

    Seven cars A, B, C, D, E, I, H are parked in a linear row facing north in such a way that no two cars parked with each other according to alphabetical order (for example – A is not parked with B, B is not parked with A and C and so on). Some cars are of Petrol and some are of Diesel variant. I is third to the left of A. More than three cars are parked between the petrol cars. E is second to the right of B. H is a diesel car and parked at one of the extreme

    BPRE–681

    ends. C is a diesel car and parked fourth to the right of D. All the cars are arranged in ascending order according to the distance covered by them from left to right. Car B covers 27 km and Car C covers 40 km. H is to the right of E. I is of diesel variant car and no petrol variant car is parked next to it. 343. Which among the following are petrol cars? (1) D and E (2) A, I, B (3) A and D (4) A, I, D, B (5) None of these 344. What can be the distance covered by car I? (1) 45 km (2) 63 km (3) 17 km (4) 31 km (5) 25 km 345. Which among the following cars are parked at the two extreme ends? (1) D, E (2) H, D (3) H, B (4) D, B (5) None of these 346. Which among the following car is parked to the immediate left of car C? (1) B (2) H (3) D (4) A (5) E 347. How many cars are parked between car B and car H? (1) Four (2) Three (3) Five (4) Two (5) None of these 348. Five boxes A, B, C, D and E are stacked above one another but not necessarily in the same order. Only one box is kept between A and B. C is kept at one of the positions above B but not at the top of the stack. Only two boxes are kept between C and D. Which of the following box is kept second from the bottom of the stack? (1) D (2) B (3) A (4) C (5) Cannot be determined (IBPS SO (IT Officer) CWE (Prelim Exam) 30.12.2017)

    Directions (349–354) : Study the following information carefully and answer the questions given below : (IBPS SO (IT Officer) CWE (Prelim Exam) 30.12.2017)

    Seven people A, B, F, G, L, M and X were born on seven different days of the same week starting from

    PROBLEM SOLVING Monday and ending on Sunday. Each one of them works in different companies, viz., Dell, Capegemini, Convergys and IBM. Atleast one person works in each of the given companies. A was born on Thursday. Only two people were born between F and A. F works for Capegemini. One of the persons who works for IBM was born on Tuesday. Only two people were born between L and the one born on Tuesday. X works for Dell, and was born on one of the days after L. The person born immediately after X works for Capegemini. One of the persons who works for Convergys was born immediately before M. Only two people were born between B and one of the persons who work for Convergys. G and M work for the same company. One of the persons born after G works for IBM. 349. Who amongst the following was born on Monday? (1) L (2) F (3) None of the given options (4) One of the persons who work for Convergys (5) One of the persons who work for IBM 350. In which of the given companies, does only one person work? (1) None (2) Only Convergys (3) Both Dell and Capegemini (4) Both Capegemini and Convergys (5) Only Dell 351. X was born on which of the following days? (1) Friday (2) Wednesday (3) Tuesday (4) Other than those given as options (5) Saturday 352. Which of the following represents the people who work for IBM? (1) L, B, A (2) L, G, M (3) A, M (4) G, M (5) L, G 353. How many people were born between M and the one who works for Capegemini ? (1) More than three (2) Three (3) One (4) Two (5) None

    Directions (354–356) : Study the following information carefully and answer the questions given below : (IBPS SO (Law Officer) CWE (Prelim Exam) 31.12.2017)

    Six people viz., P, Q, R, S, T and U each has a different weight, but not necessarily in the same order. P is heavier than only two people. S is heavier than P but lighter than Q. Q is heavier than R but lighter than U. The weight of the one who is second heaviest is 87 kgs. The weight of the one who is second lightest is 70 kgs. (Note: Weights are calculated in whole numbers only) 354. lf R weighs 21 kgs less than Q, then which of the following statements is definitely true? (i) The weight of R is 66 kgs. (ii) R does not weigh the lowest. (iii) T is heavier than R. (1) Only (i) (2) Both (ii) and (iii) (3) Only (iii) (4) All (i), (ii) and (iii) (5) Both (i) and (iii) 355. If the weight of P is 76 kgs, then who amongst the following possibly weighs 83 kgs? (1) T (2) S (3) R (4) Q (5) U 356. Who amongst the following is the second heaviest? (1) U (2) R (3) T (4) Q (5) S Directions (357–361) : Study the following information carefully and answer the questions given below : (IBPS SO (Law Officer) CWE (Prelim Exam) 31.12.2017)

    Seven people viz. Oliver, Richard, Samuel, Michael, Peter, Albert and Tom participated in one of the seven events viz. elocution competition, painting competition, debate, technical fest, athletic meet, cultural fest and science quiz held in college in seven different months of the same year viz. January, February, April, June, July, September and November. (Note: No event was held in any other month of the given year in the college. Each person participated only in one event held in one of the given months.)

    BPRE–682

    Richard participated in an event held in a month which has only 31 days. Only two people participated in an event between Richard and Albert. The one who participated in painting competition had the event before Albert in a month having 31 days. Only one person had an event between the one who participated in painting competition and the one who participated in elocution competition. Only two people participated in an event between Albert and the one who participated in debate. Tom participated in an event immediately before the one who participated in athletic meet. Albert did not participate in athletic meet. Tom did not participate in elocution competition. Only two people participated in an event between Oliver and Tom. Peter participated in an event immediately before the one who participated in technical fest. Only two people participated in an event between the one who participated in cultural fest and Michael. 357. In which of the following month did Michael participate in an event? (1) September (2) November (3) July (4) January (5) June 358. Which of the following statements is TRUE as per the given arrangement? (1) Peter participated in elocution competition. (2) The one who participated in athletic meet participated in a month before July. (3) Oliver participated in an event immediately before the one who participated in elocution competition. (4) Tom participated in an event in a month which has less than 31 days. (5) None of the given statements is true. 359. As per the given arrangement, Peter is related to the one who participated in Elocution competition and Samuel is related to the one who participated in Science quiz in a certain way. To which of the following is Oliver related to in the same way? (1) The one who participated in cultural fest. (2) The one who participated in technical fest.

    PROBLEM SOLVING (3) The one who participated in athletic meet. (4) The one who participated in painting competition. (5) The one who participated in debate. 360. Which of the following pairs represents the people who participated in an event immediately before and immediately after Samuel respectively? (1) Oliver, Richard (2) The one who participated in cultural fest, Tom (3) The one who participated in elocution competition, Michael (4) Albert, Richard (5) Oliver, Tom 361. In which event did Tom participate? (1) Painting competition (2) Debate (3) Science quiz (4) Technical fest (5) Cultural fest Directions (362–366) : Study the following information carefully and answer the questions given below : (IBPS SO (Law Officer) CWE (Prelim Exam) 31.12.2017)

    Five boxes viz. P, Q, R, S and T are kept one above the other. Each box nas a different colour viz. Blue, Red, Yellow, Green and Orange. Only one box is kept between P and the green box. Only two boxes are kept between the green box and T. The orange box is kept immediately above R. R is not green in colour. More than one box is kept between the orange box and S. As many boxes are kept between the orange box and S as between the blue box and the red box. Red box is not kept at the bottommost position. 362. Which of the following represents the colour of box Q? (1) Orange (2) Green (3) Blue (4) Red (5) Yellow 363. What is the position of yellow box in the given stack of boxes? (1) Immediately above the blue box (2) Immediately below the green box (3) Third from the top (4) First from the top (5) Second from the bottom

    364. Which of the following pairs of boxes is kept between T and the green box? (1) R and the blue box (2) P, R (3) Q and the orange box (4) R, Q (5) P and the red box 365. Which of the following is/are true with respect to the given arrangement? (i) There are less than three boxes between S and the orange box. (ii) R is placed at the bottom-most position. (iii) T is yellow in colour. (1) Both (i) and (ii) (2) Only (iii) (3) Only (ii) (4) Both (i) and (iii) (5) Only (i) 366. How many boxes are kept between P and the orange box? (1) Two (2) None (3) One (4) Cannot be determined (5) Three Directions (367–371) : Study the following information carefully and answer the questions given below : (Canara Bank PO Exam 04.03.2018)

    Eight persons, viz., B, C, D, E, L, M, N and O have to attend a seminar in March, June, August and October months of the same year. In each month the seminar will be conducted on either the 16th or 25th of that month. Each person belongs to different city — Deihi, Ahmedabad, Surat, Nasik, Bangalore, Kanpur, Mumbai and Udaipur. Not more than two of the given people have a seminar in the same month. Only one of the given person attends a seminar on each date. M has a seminar on the 16th of a month which has 31 days. Only three people have a seminar between M and B. The one who belongs to Nasik has to attend a seminar on one of the days between M and B but not in June. Only four people have a seminar between the one who belongs to Surat and the one who belongs to Nasik. As many people have to attend a seminar before O as between B and the one who belongs to Surat. The number of people attending a seminar before O is one less than those attending after the one who belongs to Delhi. L belongs to Bangalore but does not have a semi-

    BPRE–683

    nar on 25th. L has to attend a seminar on one of the days after the one who belongs to Mumbai. Only two people have a seminar between N and the one who belongs to Kanpur. B neither belongs to Kanpur nor Udaipur. Only one person has to attend a seminar between E and the one who belongs to the Udaipur. No one has to attend a seminar between C and the one who belongs to Ahmedabad. 367. To which of the following cities does M belong? (1) Udaipur (2) Mumbai (3) Bangalore (4) Delhi (5) None of the given options 368. How many people have a seminar between D and the one who belongs to Surat? (1) None (2) Two (3) Three (4) One (5) More than three 369. Who amongst the following have to attend a seminar in August? (1) E and O (2) M and the one who belongs to Delhi. (3) B and the one who belongs to Kanpur. (4) The one who belongs to Ahmedabad and E. (5) M and D 370. How many people have a seminar after E? (1) None (2) One (3) More than three (4) Two (5) Three 371. As per the given arrangement, which one of the following combinations is not correct? (1) O – 25th (2) L – June (3) N – l6th (4) C – Kanpur (5) D – Ahmedabad 372. Which of the following statements is true with respect to the given information? (1) B has to attend a seminar in March (2) None of the given statements is true (3) No one has a seminar after D. (4) Only three people have a seminar between B and N (5) L belongs to Ahmedabad

    PROBLEM SOLVING Directions (373–378) : Study the following information carefully and answer the questions given below : (Canara Bank PO Exam 04.03.2018)

    Eight people – O, P, Q, R, S, T, U and V — live on eight different floors of a building but not necessarily in the same order, The lowermost floor of the building is numbered one, the one above that is numbered two and so on till the top most floor is numbered eight. Each one of them likes a different game — Chess, Carom, Snooker, Cricket, Football, Tennis, Boxing and Polo, but not necessarily in the same order. U lives on floor number 6. Only one person lives between U and the one who likes Cricket, Only two people live between the one who likes Cricket and the one who likes Polo. The one who likes Polo lives on one of the floors below the one who likes Cricket. S lives on one of the odd numbered floors above the one who likes Polo. The one who likes Carom lives immediately below S but not on floor number 2. Only four people live between S and the one who likes Chess. As many people live between the one who likes Chess and Polo as between S and the one who likes Tennis. Only two people live between V and the one who likes Tennis. Only three people live between T and the one who likes Snooker. V neither likes Snooker nor Football. P lives immediately above R. O lives immediately above the one who likes Football. 373. How many people live above the one who likes Boxing? (1) Three (2) One (3) More than three (4) Two (5) None 374. How many people live between the one who likes Cricket and the one who likes Football? (1) More than three (2) Three (3) Two (4) None (5) One 375. On which of the following floor number does Q live? (1) 7 (2) 8 (3) 6 (4) 3 (5) 2 376. Four of the following five are alike in a certain way based on the given arrangement and hence they form a group. Which one of the following does not belong to the group?

    (1) U–Tennis (2) R–Boxing (3) T–Polo (4) P–Cricket (5) Q–Carom 377. Who lives immediately above P? (1) Q (2) O (3) The one who likes Football (4) U (5) The one who likes Polo 378. Which of the following games does R like? (1) Snooker (2) Boxing (3) Cricket (4) Football (5) Polo Directions (379–383) : Study the following information carefully and answer the questions given below : (Canara Bank PO Exam 04.03.2018)

    Eight people, F, G, H, I, J, K, L and M have lectures in different cities, viz., Delhi, Mumbai and Patna at 1 pm, 2 pm, 3 pm, 4 pm, 5 pm, 6 pm, 7 pm and 8 pm on the same day. Not more than three people have a lecture in the same city. F has a lecture in Mumbai with only the one who has a lecture at 8 pm. K and the person having lecture at 7 pm have lectures in the same city but not in Mumbai. J and L have lectures in the same city but not in the same city as K. I has a lecture at 5 pm in the same city as G. None of those having lectures in Delhi have it at 3 pm or before 3 pm. One of the lectures in Patna is at 6 pm. M does not have a lecture in the same city as L. Difference between the lecture timings of J and F is 3 hours. H’s lecture is before L’s lecture. 379. In which city does I have a lecture? (1) Same city as the one having lecture at 2 pm. (2) Same city as H (3) Patna (4) Same city as M (5) Delhi 380. At what time is L’s lecture? (1) 2 pm (2) 3 pm (3) 4 pm (4) 6 pm (5) 8 pm 381. Based on the given arrangement, which of the following statements is true? (1) F and H have lectures in the same city. (2) M has a lecture before L. (3) None of the given options is true.

    BPRE–684

    (4) G and J have lectures in the same city (5) F has a lecture at 4 pm 382. Who has a lecture at 7 pm? (1) J (2) F (3) M (4) G (5) H 383. What is the difference between the lecture timings of M and J? (1) 4 hours (2) 2 hours (3) 1 hour (4) 6 hours (5) 5 hours Directions (384–388) : Study the following information carefully and answer the questions given below : (IDBI Bank PO Exam 29.04.2018)

    Ten people, A, B, C, D, E, V, W, X, Y and Z appeared for the same exam in one of the years, viz, 1988, 1993, 1996, 2001 and 2008. In each year these people appeared in either March or September, with only one person having an exam in a month of the respective year. C had the exam in September 1993. Only one person had an exam between C and Y. As many people had exams after Y as before V. Only three people had exams between X and D. X had his exam in one of the months before D. X did not have his exam in March. More than four people had exams between D and W. W and Y did not have exams in the same year. Only three people had exams between W and B. A had his exam in one of the months before E but in one of the months after Z. 384. How many people had exams between C and E? (1) Six (2) None (3) Three (4) Five (5) Four 385. When did A have his exam? (1) March 1996 (2) March 2001 (3) September 1993 (4) September 2001 (5) March 2008 386. Which of the following statements is true as per the given arrangement? (1) E had the exam before X. (2) C and A had exams in the same year. (3) Y had the exam in 1996. (4) Z had the exam before all others. (5) C had the exam in one of the months between A and X.

    PROBLEM SOLVING 387. How many people had exams after B? (1) Six (2) Three (3) Four (4) One (5) Two 388. Which of the following combinations is correct based on the given information? (1) V — September (2) X — 1993 (3) W — 1996 (4) E — March (5) D — 2008 Directions (389–393) : Study the following information carefully and answer the questions given below : (IBPS Bank PO/MT CWE-VII (Prelim Exam) 15.10.2017)

    L, M, N, O, P, Q and R are seven employees who are working in the same company. They attend meeting in different department viz; Administrative, Security, Finance and HR department on different days from Monday to Sunday but not necessarily in the same order. One employee attends only one meeting and only one meeting is held on each day. There are two employees who attend meeting in Administrative, Security, HR department and only one employee attends meeting in Finance department. L attends meeting on Thursday. There are two persons who attend meeting between L and the person who attends meeting in HR department. Only three persons attend meeting between the persons who attend meeting in Administrative department and the one who attends meeting in Finance department. The one who attends meeting in administrative department attends before the one who attends in Finance department. The one who attends meeting in Finance department does not attend on Saturday. The number of persons who attend meeting between L and the one who attend meeting in finance department is same as the number of persons who attend meeting between O and the one who attends meeting in Security department. The one who attends meeting in Security department attend before O. O does not attend meeting in HR department. Q attends meeting on the day immediately before the day on which L attends meeting. O does not attend meeting on the day just after the day on which L attends meeting. The number of persons who attend meeting between L and P is same as the number of persons who attend

    meeting between L and R. P attends meeting in one of the day before the day on which R attends meeting. N attends meeting in Administrative department. R does not attend meeting in Security department. 389. Who among the following persons did attend meeting on Friday ? (1) O (2) M (3) P (4) N (5) R 390. Which of the following combinations of “Person – Day” is true with respect to the given arrangement? (1) R — Friday (2) M — Saturday (3) Q — Thursday (4) P — Friday (5) P — Tuesday 391. L attends meeting in which of the following department? (1) Security (2) HR (3) Administrative (4) Finance (5) Either (1) or (2) 392. In this arrangement, Q is related to Tuesday then N is related to : (1) Thursday (2) Wednesday (3) HR (4) None of those given as options (5) Sunday 393. How many persons do attend meeting between P and O ? (1) Three (2) None (3) More than three (4) Two (5) One Directions (394–398) : Study the following information carefully and answer the questions given below : (IBPS Bank PO/MT CWE (Main Exam) 26.11.2017)

    Five topics A, B, C, D and E will be discussed, each topic on one day, from Monday to Saturday. Topic A will be discussed before E and topic B will be discussed before D. Topics B will not be discussed on the first day. There will be one rest day (F). There will be a gap of two days between the days on which topics D and B will be discussed. Topic C will be discussed immediately before the rest day. The rest day will not be the second or the fourth day. 394. Which of the following is the correct sequence of the discussion on the topics including the rest day (F) ?

    BPRE–685

    (1) AEFCDB (2) ABECFD (3) AEBCFD (4) ABCFED (5) None of these 395. Which of the following is the correct statement ? (1) Topic A will be discussed on Tuesday (2) Discussion on topic C will be immediately preceded by discussion on topic B (3) Discussion on topic B will take place before that of A (4) Thursday is the rest day (5) None is true 396. On which day will be the topic C be discussed ? (1) Tuesday (2) Wednesday (3) Friday (4) Thursday (5) None of these 397. How many days gap will be there between the days on which topics E and B will be discussed ? (1) One (2) Nil (3) Two (4) Three (5) None of these 398. With reference to A, the discussion on topic E will take place _______ (1) Immediately next day (2) After a day’s gap (3) Three days after (4) Cannot be determined (5) None of these Directions (399–403) : Study the following information carefully and answer the questions given below : (IBPS RRBs Officer CWE (Prelim Exam) 11.08.2018)

    Nine people – A, B, C, D, E, F, G, H and K – were born in nine different months viz. January, March, April, May, June, August, October, November and December of the same year. Note : No other person was born in any other month. A was born in a month having 31 days after August. Only three people were born between A and G. C was born in one of the months after G in a month having 30 days. Only one person is born between C and D. More than three people were born between the one born in January and H. Only two people were born between H and B. E was born in one of the months after F but not in a month having only 30 days. 399. How many people are younger than E? (1) One

    PROBLEM SOLVING (2) Two (3) More than three (4) Three (5) None 400. Who amongst the following was born in May? (1) F (2) K (3) E (4) D (5) B 401. How many people were born between F and the one born in June? (1) More than three (2) One (3) Two (4) Three (5) None 402. As per the given arrangement G is related to April and H is related to June in a certain way. To which of the following is C related in the same way? (1) May (2) August (3) October (4) January (5) March 403. In which of the following months was K born? (1) April (2) March (3) June (4) August (5) November Directions (404–408) : Study the following information carefully and answer the questions given below : (IBPS RRBs Officer CWE (Prelim Exam) 18.08.2018)

    Movies of different duration were released on different days starting from Monday to Friday. Movie was A was released on Tuesday. No movie was released between A and the one which was of 75-minute duration. Only one movie was released between the one which is of 75-minute duration and the one which is of 100-minute duration. No movie was released between the one which is of 100 minute and B. Only one movie was released after B. B was released immediately after 100-minute duration movie. Movie C was released immediately after the one which is of 130-minute duration. More than two movies were released between C and D. The movie which is of 90-minute duration was released before E. One of movies was of 20 minutes more duration than movie E.

    404. How many movies were released after E? (1) One (2) Two (3) None (4) Three (5) More than three 405. Which of the following movie was of 150-minute duration? (1) E (2) A (3) There is no such movie (4) C (5) D 406. What is the total duration of movies D and E together? (1) 135 (2) 225 (3) 165 (4) 175 (5) 190 407. Which of the following statements is true regarding B? (1) The movie released after B is of 120-minute duration (2) Two movies were released between A and B (3) Movie B is of 100-minute duration (4) Total duration of movie B and A is 225 minutes (5) Movie A was released after B. 408. Which of the following statements is true? (1) The movie released before A is of 130-minute duration (2) Three movies were released between A and E (3) No movie was released between A and E (4) Total duration of movie C and A is 230 minutes (5) Movie C was released immediately after E. Directions (409–411) : Study the following information carefully and answer the questions given below : (IBPS RRBs Officer CWE (Prelim Exam) 18.08.2018)

    There are six persons M, N, O, P, Q, R of different heights. N is shorter than M but taller than Q. Only two persons are taller than M. R is taller than Q and O. Q is not the shortest. The one who is second shortest is 154 cm. P is not the shortest person. 409. If M is 19 cm taller than Q, then what is the height of M? (1) 190 cm (2) 181 cm (3) 175 cm (4) 130 cm (5) 173 cm 410. If P is 181 cm then which of the following statements is true?

    BPRE–686

    I. Only one person is taller than P. II. The difference between the heights of P and Q is 27 cm III. O is the shortest person. (1) Only I (2) Only I and II (3) All are true (4) Only II and III (5) Only I and III 411. How many persons are shorter than N? (1) One (2) Two (3) None (4) Three (5) More than three Directions (412–416) : Study the following information carefully and answer the questions given below : Indian Bank PO Online Prelim Exam, 06.103.2019

    Nine people- J, K, L, M, N, O, P, Q and R – were born in three different months- January, April and July of the same year on either of the three dates5th, 16th and 22nd. Thus only one person was born on each date of each month. It is assumed that only the given people were born in the given year. J was born on 16th of a month having 31 days. Only four people were born between J and N. P was born immediately after N. K was born on an odd numbered date. K was not born in the months in which P and J were born. As many people were born before K as after M. Only one person was born between M and O. L was born immediately before Q. 412. Who amongst the following was born immediately before M? (1) Q (2) J (3) R (4) K (5) N 413. On which of the following dates was R born? (1) 5th January (2) 22nd July (3) 16th January (4) 16th July (5) 22nd April 414. How many people were born between L and P? (1) Three (2) More than three (3) One (4) Two (5) None

    PROBLEM SOLVING 415. As per the given arrangement, four of the following five are alike in a certain way and hence form a group. Which one of the following does not belong to the group? (1) KR (2) RP (3) QJ (4) LO (5) NL 416. Who amongst the following was born on 5th July? (1) K (2) M (3) N (4) O (5) None of those given as options Directions (417–421) : Study the following information carefully and answer the questions given below : (IBPS Bank PO/MT CWE (Prelim Exam) 14.10.2018)

    Each of eight people — A, B, C, D, E, F, G and H–visits Mumbai in one of the given months — January, February, March, April, May, June, July and August of the same year but not necessarily in the same order. Note: Only one person visits Mumbai in each month. No other person visits Mumbai in any other month of the same year. Only three people visit Mumbai before A. Only two people visit Mumbai between A and B. C visits Mumbai immediately after B. More than four people visit Mumbai between C and D. Only two people visit Mumbai between D and E. F visits Mumbai in one of the months before E but not in January. H visits Mumbai immediately after G. 417. Which of the following statements is TRUE as per the given information? (1) Only two people visit Mumbai between D and G. (2) No one visits Mumbai after E. (3) None of the given statements is true (4) A visits Mumbai in one of the months after F. (5) C visits Mumbai in April. 418. As per the given information, who amongst the following visits Mumbai in July? (1) E (2) C (3) G (4) D (5) H 419. As per the given information, how many people visit Mumbai between C and G?

    (1) Two (2) Three (3) One (4) None (5) More than three 420. In which month does D visit Mumbai? (1) August (2) June (3) February (4) May (5) January 421. Four of the following five are alike in a certain way based on the given information and thus they form a group. Identify the one which does not belong to the group? (1) June – E (2) May – A (3) April – C (4) February – B (5) August – H Directions (422–426) : Study the following information carefully and answer the questions given below: IBPS Bank PO/MT CWE Prelim Exam, 21.10.2018

    Eight people— P, Q, R, S, T, U , V and W— are sitting around a circular table facing the centre but not necessarily in the same order. Each of them speaks a different language namely, Marathi, English, Hindi, Bengali, Malyalam, Tamil, Gujarat and Urdu but not necessarily in the same order. The one who speaks Malayalam sits second to the right of W. Only two people sit between the one who speaks Malayalam and the one who speaks Gujarati. V sits to the immediate left of the one who speaks Gujarati. Only two people sit between V and U. U is an immediate neighbour of W. The one who speaks Hindi sits second to the right of U. T sits to the immediate right of the one who speaks Hindi. Only three people sit between T and the one who speaks Marathi. The one who speaks Tamil sits second to the left of the one who speaks Marathi. S sits to the immediate left of P. The one who speaks Urdu sits second to the right of P. Only three people sit between the one who speaks Bengali and the one who speaks English. The one who speaks Bengali is not an immediate neighbour of T. R sits second to the left of the one who speaks English. 422. Which of the following will come in place of question mark (?) in the following series based on the given arrangement ? Bengal V Hindi P English (?)

    BPRE–687

    (1) W (2) S (3) T (4) Q (5) Other than those given as options 423. What is the position of U with respect to the one who speaks Malayalam ? (1) Immediate right (2) Second to the left (3) Third to the right (4) Third to the left (5) Second to the right 424. Who sits second to the left of the one who speaks Gujarati? (1) V (2) W (3) T (4) U (5) Q 425. Four of the following five are alike in a certain way based on their seating arrangement and so they form a group. Which one does not belong to the group? (1) Q-Hindi (2) V-English (3) R-Bengali (4) U-Gujarati (5) P-Tamil 426. Which of the following statements is true regarding Q? (1) Q sits third to the right of U. (2) Only three people sit between Q and P. (3) Q is an immediate neighbour of the one who speaks Tamil. (4) Q sits second to the left of P. (5) Q speaks Malayalam. Directions (427–431) : Study the following information carefully and answer the questions given below: (IBPS Bank PO/MT CWE (Main Exam) 18.11.2018)

    Seven persons M, N, O, P, Q, R and S live on different floors of an apartment. The ground floor of the building is numbered 1 and so as the top floor is numbered as 7th floor. Each person likes different cartoon characters viz., Chipmnuk, Flinstone, Jetson, Popeye, Scooby Doo, Simpson and Tweety (but not necessarily in the same order). Each person also likes different colours namely Yellow, Orange, Red, Black, Blue, Brown and Purple but not necessarily in the same order. ● Only two persons live between P and the one who likes Popeye. The persons living on the ground floor and the topmost floor do not like Orange colour. ● More than three persons live between the one who likes Yel-

    PROBLEM SOLVING

















    427.

    428.

    429.

    low colour and the one who likes Orange. M does not live on the lowermost floor. M lives on any odd numbered floor below the one who likes Popeye. The person who likes Jetson does not like Yellow. Only two persons live between M and the person who likes Tweety. S lives on an even numbered floor but neither immediately above nor immediately below the floor of M. Only one person lives between N and R. The person who likes Orange lives exactly between the persons who like Brown and Blue. Only three persons live between the persons who like Chipmnuk and Jetson respectively. R lives on an even numbered floor and does not like Popeye. The person who likes Chipmnuk live on any floor above the N’s floor. The person who likes Chipmnuk does not live on the topmost floor. The person who likes Scooby Doo lives on the floor immediately above the floor of the person who likes Simpson. O does not like Chipmnuk or Jetson. Only two persons live between the person who likes Purple and the one who likes Brown. N likes neither Purple nor Red. The person living immediately below the floor of S, likes which of the following colour and cartoon character respectively? (1) Black – Scooby doo (2) Brown – Jetson (3) Purple – Tweety (4) Red – Popeye (5) Red – Chipmunk Which of the following person is living exactly between the floors of Q and R and also likes Black colour? (1) P (2) M (3) N (4) S (5) O The person who likes Simpson cartoon character lives exactly between which of the following pair of persons? (1) M and P (2) R and P (3) S and N (4) N and Q (5) R and S

    430. Which of the following pairs of person live on the bottom and top floors of the apartment? (1) S and N (2) Q and R (3) S and P (4) O and P (5) M and O 431. The person who likes Flinstone cartoon character likes which of the following colours? (1) Orange (2) Yellow (3) Black (4) Blue (5) Purple Directions (432–436): Study the following information carefully and answer the questions given below: (IBPS Bank PO/MT CWE (Main Exam) 18.11.2018)

    Seven tenants A, B, C, D, E, F and G live on seven different floors of a building but not necessarily in same order. Lowermost floor of building is numbered 1, so as the top most floor is numbered 7. Each one of them also owns different brands of Car viz., Fiat, Hyundai, Honda, Skoda, Toyota, Ford and Chevrolet. All of them go for a tour on seven different days of the same week starting from Monday to Sunday but not necessarily in the same order. ● The person who goes to tour on Friday owns Fiat Car. G lives on an odd numbered floor. E does not own Honda car ● D goes to tour on Sunday. Only one person lives between B’s and E‘s floors ● The tenant who owns Toyoto car lives immediately above A’s floor ● Neither C nor A owns Skoda car. The tenant who goes to tour on Wednesday is immediately after the tenant who owns Hyundai. ● The tenant who owns Fiat lives immediately above the tenant who owns Chevrolet car. The one who owns Honda car goes to tour immediately before the tenant who lives on floor number 3. ● There are two persons between the floor number 4 and the person who goes to tour on Wednesday. The tenant who goes to tour on Wednesday is not C. ● Tenant A lives on an odd numbered floor but not on floor number three.

    BPRE–688

    Only two tenants live between D’s floor and the one who owns Toyota car ● The tenant who owns Honda car lives immediately above C. The tenant who owns Toyota car goes to tour on one of the days before Thursday ● The one who owns Hyundai car lives in on odd numbered floors above D. B lives on the floor above E. Only three tenants live between C‘s floor and the tenant who owns Hyundai. 432. Who among the following tenant goes to tour on Friday and by which car? (1) A – Honda (2) C – Fiat (3) B – Skoda (4) E – Fiat (5) D – Honda 433. The tenant three floors below the floor of tenant B used which of the following car and on which day did he go to the tour? (1) Hyndai – Friday (2) Chevrolet – Sunday (3) Ford – Monday (4) Honda – Saturday (5) Skoda – Tuesday 434. Which among the following pairs of the tenants go to the tour on the weekends? (1) E and C (2) B and F (3) F and D (4) B and D (5) A and C 435. Who among the following person lives on 5th floor and which of the following car does he use? (1) B – Chevrolet (2) C – Ford (3) A – Hyundai (4) F – Honda (5) G – Toyota 436. Which among the following pairs of tenants go to the tour on Monday and Friday? (1) G and B (2) F and A (3) E and D (4) D and C (5) B and E Directions (437–441) : Study the following information carefully and answer the questions given below : ●

    (IBPS Bank PO/MT CWE (Main Exam) 18.11.2018)

    There are 16 boxes in a rack placed in the form of 4 × 4 matrix. The rows of a matrix are denoted as 1, 2, 3 and 4 from top to bottom and

    PROBLEM SOLVING the columns are denoted as #, &, % and @ from right to left in the same order. The boxes contain 16 different articles Book, Watch, Toy, Mobile, Radio, Ring, Pen, Fruit, Ball, Laptop, Glows, Cap, Money, Clock, Cup and Camera. (All the given information is not necessarily in the same order). Note: When a box is said to be ‘beside/adjacent/between or at a gap of‘ it can be either horizontally or vertically. ● The Ring is in a box that is immediate right below diagonally of the box having camera. Box having books is in the same column of the box having Fruits ● Glows is beside the box having books. The pen is in a box that is immediate below box which is immediate left of a box having Money ● Clock is in a box which is immediate left above diagonally of a box that is immediate right of a box having Laptop ● Watch is in a box that is three boxes above the box (in the same column) which is immediate left of a box having Fruits ● Cap is in a box that is second right of a box having Books. ● Toy is in a box that is immediate right of a box which is immediate left below diagonally of a box having Camera ● The cup is in a box that is third right of a box which is two boxes above the box having Glows ● Mobile is in a box that is immediate left below diagonally of a box that is immediate left below diagonally of a box having Toys ● Radio is in a box which is immediate adjacent left of the box having either Camera or Watch ● The box containing Ball is immediate right below diagonally of a box having Clock 437. Which of the following position correctly represents the box having “Glows”? (1) 2% (2) 4@ (3) 1# (4) 3@ (5) 2& 438. Which of the following position correctly represents the box having “Radio”? (1) 2& (2) 3@ (3) 2% (4) 1# (5) 3%

    439. Which of the following item is in the box that has matrix position “3#”? (1) Pen (2) Fruits (3) Book (4) Cap (5) Cup 440. Which of the following item is in the box that has matrix position “1%”? (1) Radio (2) Cup (3) Laptop (4) Ball (5) Money 441. What is the position of Box having Toy with respect to the Box having Cap? (1) Immediate left above diagonally (2) Immediate right above diagonally (3) Immediate left in the same row (4) Immediate right in the same row (5) Two boxes above in the same column Directions (442–444): Study the following information carefully and answer the questions given below: (IBPS Bank PO/MT CWE (Main Exam) 18.11.2018)

    There are seven friends P, Q, R, S, T, U and V. All of them play a game in which each friend throws a dice having 6 sides, then picks a card and performs a particular activity. A number on the dice is repeated only once. And each activity is performed by not more than 2 people. There are 4 activities singing, dancing, acting and mimicry. There are four cars - Hearts, Spade, Club and Diamond are picked by the people and not more than 2 people pick the same colour card. U gets number 5 and performs same activity as that of P’s. S gets a number between the number claimed by Q and P. P gets a number less than V. ● V performs acting and gets the same number on dice as that of R‘s. The friend who gets 3 does not get a spade. The one who gets the lowest number performs dancing. ● P either performs singing or dancing. T gets number 6 on the dice and opens a card of hearts and performs dancing.

    BPRE–689

    The friend who gets a club does not perform singing. Only one of the persons who sing gets a spade. The person who gets club gets the lowest two numbers. ● Q and S get the same card. The friend who gets a diamond card performs mimicry and is the only one to get this card and perform this activity 442. Which of the following is true about S in the game? (1) S got dice number 2 (2) S got Diamond card (3) S performed Acting (4) 1 and 3 (5) 2 and 3 443. Which of the following is true about P in the game? (1) P got dice number 2 (2) P got Heart card (3) P performed Singing (4) 1 and 2 (5) 2 and 3 444. Which of the following is true about T, U and V in the game? (1) T performed Dancing (2) U got Club card (3) V got dice number 4 (4) 1 and 2 (5) 1 and 3 Directions (445–449) : Study the following information carefully and answer the questions given below : ●

    (IBPS Specialist Officer CWE (Prelim Exam) 31.01.2019)

    Seven women J, K, L, M, N, O and P live on seven different floors of a building, but not necessarily in the same order. Lowermost floor of the building is numbered 1, one above that is numbered 2 and so on till the topmost floor is numbered 7. Each of them likes different brands of clothes viz. Puma, Adidas, HRx, Fila, Nike, Levi’s and Wranglar, but not necessarily in the same order. J lives on an even numbered floor but not on 4th floor. Only two women live between M and the one who likes Nike. The one who likes HRx lives immediately below L. The one who likes Adidas lives on odd numbered floors above M. Neither O nor P likes Fila. Three women live between L and the one who likes Adidas. The one who likes Puma lives immediately below the one who likes Wranglar. Only one woman lives between K and N. P does not like

    PROBLEM SOLVING Puma. The one who likes Nike lives immediately above J. K lives above N. 445. Who among the following likes Fila brand ? (1) J (2) M (3) L (4) N (5) None of these 446. Which of the following brand does P like ? (1) Puma (2) Wranglar (3) Levi’s (4) HRx (5) None of these 447. Who among the following lives on topmost floor? (1) K (2) N (3) M (4) P (5) None of these 448. Four of the following five are alike in a certain way based on the given arrangement and thus they form a group. Which one of the following does not belong to the group? (1) N (2) K (3) L (4) O (5) P 449. Which of the following combination is true? (1) M — Wranglar (2) K — Puma (3) J — Adidas (4) O — Nike (5) K — Adidas Directions (450–454) : Study the following information carefully and answer the questions given below : (IBPS Specialist Officer CWE (Prelim Exam) 31.01.2019)

    A, B, C, D, E, F and G are working in the public sector bank. The rank of every person is different among MD, ED, CGM, GM, DGM, AGM and Officer but not necessarily in the same order. They are appointed in the different cities among Mumbai, Delhi, Pune, Nagpur, Bangalore, Kolhapur, and Solapur but not necessarily in the same order. (Rank decreases moving from left to right as: MD, ED, CGM, GM, DGM, AGM and Officer). There are only 3 persons with the rank higher than C. The rank of C is lower than the one working in the city Pune. A, the DGM is working in Solapur. F is either ED or CGM and is working in Mumbai. The rank of D is higher than B but lower than the one working in Delhi. Neither D nor B is the AGM. E is working in Nagpur. Neither C nor AGM is working in Kolhapur.

    450. Who is the AGM? (1) A (2) D (3) E (4) F (5) G 451. Who works in Solapur? (1) A (2) B (3) C (4) D (5) F 452. Who works in Kolhapur? (1) E (2) D (3) F (4) Officer (5) AGM 453. ______ works in Delhi. (1) ED (2) MD (3) GM (4) D (5) E 454. Who is ED? (1) A (2) The one who works in Pune (3) E (4) The one who works in Delhi (5) The one who works in Nagpur Directions (455–459) : Study the following information carefully and answer the questions given below : (IBPS Specialist Officer CWE (Prelim Exam) 31.01.2019)

    Eight persons P, Q, R, S, T, U, V and W likes different fruits- Banana, Mango, Kiwi, Grapes, Orange, Apple, Guava and Papaya but not necessarily in the same order. Also, they have different salaries in Rupees i.e. 2000, 5000, 3000, 4500, 6533, 5523, 6400 and 7000 but not necessarily in the same order. U’s salary is a perfect square. P likes Kiwi. Salary of the one who likes Mango is more than the one who likes Banana. Salary of the one who likes Guava is 2000. V’s salary is an odd number. T likes Papaya and Q likes Apple. R’s salary is 1500 more than Q’s salary. W doesn’t like Orange and Grapes. P’s salary is 2.5 times of W’s salary. V and U either likes Banana or Mango. R doesn’t like Orange. T’s salary is an odd number which is divisible by 3. 455. Who among the following likes Guava? (1) V (2) P (3) Q (4) R (5) W 456. Difference between the amount of salary of the one who likes Grapes and V is (1) 1200 (2) 2033 (3) 2345 (4) 400 (5) 3000

    BPRE–690

    457. What is the salary of P? (1) 4500 (2) 5523 (3) 7000 (4) 5000 (5) 3000 458. U likes which of the following fruit? (1) Banana (2) Mango (3) Orange (4) Grapes (5) Guava 459. What is the salary of S? (1) 5000 (2) 2000 (3) 7000 (4) 3000 (5) None is true Directions (460–464) : Study the following information carefully and answer the questions given below: IBPS RRBs Officer CWE Prelim Exam, 04.08.2019 Seven different events were held on seven different days of the same week starting from Monday and ending on Sunday. Event R was held on Friday. Only one event was held between event R and event F. Only four events were held between event F and event J. Event C was held immediately before event E. Event Z was held on one of the days after event S. 460. Which of the following statements is not true as per the given arrangement? (A) Event E was held on one of the days before event Z. (B) No event was held after event Z. (C) Only three events were held between event C and event F. (1) Both (A) and (C) (2) Both (B) and (C) (3) Only (C) (4) Only (B) (5) Only (A) 461. On which day was event Z held ? (1) Saturday (2) Wednesday (3) Thursday (4) Tuesday (5) Sunday 462. How many events were held before event F ? (1) Two (2) Three (3) More than three (4) One (5) None 463. The number of events held before event J is one less than the number of events held after event _______. (1) F (2) C (3) Z (4) E (5) R

    PROBLEM SOLVING 464. Which event was held on Wednesday ? (1) Z (2) C (3) S (4) F (5) E Directions (465–469) : Study the following information carefully and answer the questions given below : IBPS RRBs Officer CWE Prelim Exam, 04.08.2019 Eight people live on four different floors of a building where the ground floor is numbered one, the one above that is numbered two and so on till the topmost floor of the building is numbered four. On each floor there are two flats, viz. Flat A and Flat B such that Flat A is exactly to the west of Flat B. Also, Flat A of floor number two is immediately above flat A of floor number one. Similarly, flat B of floor number three is immediately above flat B of floor number two and so on. Thus only one person lives in each flat and only two people live on each floor. The area of each flat on each floor is same. P lives on an even numbered floor. No one lives to the west of P. Q lives on a floor immediately below the floor on which P lives. No one lives to the east of Q. There is only one floor between the floors on which Q and S live. R lives in a flat exactly to the west of S. There are only two floors between the floors on which R and T live. U lives in a flat exactly to the west of V. W lives on one of the floors above V. 465. Who amongst the following lives in flat B on floor number four ? (1) U (2) T (3) V (4) Q (5) S 466. Which of the following statements is true as per the given information ? (A) There is only one floor between the floors on which V and P live. (B) R lives on the lowermost floor. (C) S lives on a floor immediately above U. (1) Only (C) (2) Only (A) (3) Only (A) and (B) (4) Only (B) and (C) (5) Only (B) 467. Four of the following five are alike in a certain way as per the given arrangement and thus they form a group. Which one of the fol-

    lowing does not belong to that group? (1) SP (2) QV (3) PW (4) UR (5) VS 468. Which of the following depicts the flat and the floor number in which U lives? (1) Flat A, Floor number 1 (2) Flat B, floor number 2 (3) Flat B, floor number 1 (4) Flat A, Floor number 3 (5) Flat A, Floor number 2 469. Who lives in a flat immediately below W ? (1) S (2) P (3) Q (4) U (5) V Directions (470–472) : Study the following information carefully and answer the questions given below: IBPS RRBs Officer CWE Prelim Exam, 04.08.2019 Six boxes viz.— A, B, C, D, E and F are each of a different weight. A is heavier than only two boxes. D is heavier than A but lighter than B. B is heavier than C but lighter than F. E is not the lightest. The weight of the second heaviest box is 79 kg. The weight of the second lightest box is 62 kg. (Note: Weights are calculated in whole numbers only) 470. If the weight of A is 68 kg, then which of the following boxes possibly weighs 75 kg? (1) E (2) C (3) B (4) D (5) F 471. If C weighs 24 kg less than B, then which of the following statements is definitely true? (i) The weight of C is 55 kg. (ii) C does not weigh the lowest. (iii) E is 7 kg heavier than C. (1) Only (i) (2) Only (ii) (3) Both (i) and (iii) (4) Only (iii) (5) Both (ii) and (iii) 472. Which of the following is the second heaviest box? (1) C (2) F (3) D (4) E (5) B Directions (473–477) : Study the following information carefully and answer the questions given below : IBPS RRBs Officer CWE Prelim Exam, 17.08.2019

    BPRE–691

    A, B, C, D, E, F and G are seven members of a club. Each of them likes a day of the week viz. Monday, Tuesday, Wednesday, Thursday, Friday, Saturday and Sunday but not necessarily in the same order. Each of them has a different types of car like Swift, Alto, Figo, Beat, SX4, Estilo and Optra but not necessarily in the same order. C likes Wednesday and his favourite car is neither SX4 nor Optra. E does not like Monday and his favourite car is Beat. The one who likes Friday, likes Figo. The one who likes Estilo car likes Tuesday. D likes Saturday and his favourite car is not SX4. Favourite car of G is Alto. F likes Thursday. B does not like Estilo. 473. Which of them likes Tuesday? (1) A (2) B (3) D (4) F (5) G 474. Whose favourite car is Figo? (1) A (2) B (3) F (4) C (5) D 472. Who likes Sunday? (1) A (2) C (3) F (4) E (5) G 476. Who among the following likes SX4? (1) B (2) A (3) F (4) D (5) E 477. Which of the following combinations is true as per the given information? (1) F-Thursday-Estilo (2) C-Wednesday-Alto (3) D-Saturday-Beat (4) G-Monday-Swift (5) None of the given combination is true Directions (478–482) : Study the following information carefully and answer the questions given below : IBPS RRBs Officer CWE Prelim Exam, 17.08.2019

    Six Professors, A, B, C, D, E and F of six different subjects – Economics, Mathematics, Botany, Physics, Accounting and Statistics have taken lectures on different time schedules from 10am to 4 pm. Note : Each lecture is of only one hour. F is neither the professor of Botany nor Economics and did not take lecture on or after 2 pm. A has taken lec-

    PROBLEM SOLVING ture immediately after the lecture of Statistics. D is either the professor of Accounting or Physics but did not lecture at 12:00 pm. B has taken lecture on Mathematics but before the lecture of F. C has taken lecture at 1 pm but not of Economics. E is the professor of Statistics. The lecture of Accounting has been started at 11 am. 478. Who is the professor of Physics? (1) F (2) D (3) C (4) A (5) B 479. Who among the following has taken the lecture at 10 am? (1) D (2) B (3) E (4) Either B or D (5) Either D or E 480. Which of the following subjects is taught by C? (1) Physics (2) Accounting (3) Statistics (4) Economics (5) Botany 481. Which of the following statements is true as per the given information? (1) D has taken lecture at 10 am. (2) A has taken lecture on Economics at 4 pm. (3) F has taken lecture on Physics at 12 pm. (4) E has taken lecture at 11 am. (5) None of the above statements is true 482. Which of the following combination is true regarding the professor and lecture at 2 pm? (1) A, Accounting (2) D, Physics (3) F, Physics (4) E, Statistics (5) G, Mathematics Directions (483–487) : Study the following information carefully and answer the questions given below : IBPS Bank PO/MT CWE Prelim Exam, 12.10.2019 Seven conferences A, B, C, D, E, F and G are to be held on seven different days in the same week, starting from Monday and ending on Sunday. Each conference will be held in different places, viz. Chennai, Bangalore, Hyderabad, Pune, Mumbai, Delhi and Kolkata, but not necessarily in the same order.

    Conference A will be held immediately after the conference which will be held in Chennai. The conference which is held in Mumbai will be held in one of the days before D but not to be held immediately before D. The conference which is held in Hyderabad will be held immediately before C. The conference G will be held immediately after the conference which will be held in Kolkata. Only one conference will be held between B and E. Neither conference B nor conference A is held in Delhi. Conference D will be held on one of the days before the conference to be held in Chennai. Conference A is not to be held in Bangalore. Only two conferences will be held between D and the conference which will be held in Chennai. Conference B is not to be held after D. Only three conferences will be held between C and the conference which is to be held in Mumbai. 483. Which of the following combinations is correct? (1) G – Hyderabad (2) B – Mumbai (3) D – Kolkata (4) F – Bangalore (5) Other than those given as options 484. How many conferences will be held between B and G? (1) Three (2) Four (3) Five (4) Two (5) Other than those given as options 485. Four of the following five are alike in a certain way and thus they form a group as per the given arrangement. Which of the following does not belong to that group? (1) F – Bangalore (2) G – Pune (3) D – Hyderabad (4) B – Chennai (5) E – Kolkata 486. E is related to Saturday in a certain way based on the given arrangement. In the same way C is related to Sunday. Which of the following days is B related to following the same pattern? (1) Tuesday (2) Monday (3) Thursday (4) Friday (5) Other than those given as options

    BPRE–692

    487. Which of the following conference will be held in Mumbai? (1) B (2) F (3) E (4) G (5) Other than those given as options Directions (488–492) : Study the following information carefully and answer the questions given below : IBPS Bank PO/MT CWE Prelim Exam, 12.10.2019 Eight persons A, B, C, D, E, F, G and H have their birthdays in different months of the same year viz. January, April, May and July, such that not more than two persons have their birthdays in the same month. All the birthdays are either on 14th or 23rd of the month. No two persons have their birthdays on the same day of the same month. The following information is also known about them. F does not have birthday in May. E wasn’t born in July. H’s birthday is immediately after B’s. E celebrates his birthday before B.The number of persons who have their birthdays between the birthdays of G and H is equal to the number of persons who have their birthdays between the birthdays of B and D. D was born in July. Birthdays of both E and B are in the same month. There are three birthdays between the birthdays of F and C. 488. Who among the following were born in the same month? (1) A and D (2) B and C (3) C and F (4) D and G (5) C and E 489. Who was born on 23rd July? (1) A (2) B (3) C (4) D (5) Other than those given as options 490. Which of the following combinations is true as per the given information? (1) January 23rd – G (2) April 14th – E (3) May 23rd – H (4) July 14th – C (5) None of these 491. In which month C has his birthday? (1) January (2) April (3) May (4) July (5) Cannot be determined 492. How many people have their birthdays before G?

    PROBLEM SOLVING (1) 7 (2) 3 (3) 5 (4) 1 (5) Other than those given as options Directions (493 – 497) : Study the following information carefully and answer the questions given below : IBPS Bank PO/MT CWE Prelim Exam, 19.10.2019 Ten exams were held in different months of the same year. Each exam was held on either 11th or 16th of one of the given months viz. February, March, April, May and June. Only one exam was held on each date. No other exam was held in the given year. Less than two exams were held before M. Only two exams were held between M and S. As many exams were held before S as after Q. Only two exams were held between Q and V. V was held on one of the dates after Q. N was held immediately before V. Only three exams were held between N and R. T was held immediately before P. Z was held on one of the dates after Y. 493. How many exams were held between M and P? (1) None (2) Two (3) More than three (4) One (5) Three 494. Which of the following statements is true as per the given information? (1) M was held on an even numbered date. (2) Only one exam was held after V. (3) Z was the last exam to be held in the given year. (4) None of the given statements is true (5) Q was held on one of the dates after T. 495. Four of the following five are alike in a certain way as per the given arrangement and thus they form a group. Which one of the following does not belong to that group? (1) Y (2) P (3) R (4) Q (5) N 496. On which of the following dates was exam Z held? (1) 11th April (2) 16th March (3) 11th June

    (4) 16th February (5) 16th May 497. Which of the following exams was held immediately after R? (1) Z (2) Y (3) T (4) Q (5) S Directions (498 – 502) : Study the following information carefully and answer the questions given below : IBPS Bank PO/MT CWE Prelim Exam, 19.10.2019 Six people, L, M, N, X, Y and Z live on six different floors of a six storeyed building where the lowermost floor is numbered one, the one above that is numbered two and so on till the topmost floor of the building is numbered six. Each one of them likes a different colour- Blue, Red, Pink, Yellow, Green and White. L lives on the floor numbered three. No one lives between L and the one who likes Pink. Only two people live between M and the one who likes Pink. N lives immediately above M. Only three people live between N and the one who likes Blue. Only one person lives between X and Y. X does not like Blue. Only two people live between Z and the one who likes Red. The one who likes Yellow lives on one of the floors below the one who likes Red. The one who likes Green does not live on the floor numbered five. 498. Who lives on the floor numbered two? (1) M (2) Z (3) X (4) N (5) Y 499. Who likes Blue colour? (1) The one who lives on the floor numbered four. (2) The one who lives on the floor numbered two. (3) The one who lives immediately above M. (4) Z (5) Y 500. How many people live between Z and the one who likes Green? (1) Two (2) One (3) Three (4) None (5) More than three 501. Who lives immediately above the one who likes White? (1) X (2) Y (3) N (4) L (5) Z

    BPRE–693

    502. Which colour does L like? (1) Green (2) Blue (3) None of the given options (4) Yellow (5) Red Directions (503–507) : Study the following information carefully and answer the questions given below : IBPS Bank PO/MT CWE Main Exam, 30.11.2019 An apartment has four floors. There are three flats on each floor. The bottom floor is numbered as floor number 1 and the top most floor is numbered as floor number 4. The flats of left end are in column 1 while the flats of right end are in column 3. Twelve tenants –N, O, P, B, R, S, T, U, V, A, H and Y – live in different flats and the rents paid by them are shown in the following figure : Rs. 18,000 Rs. L E Rs. 16,000 Rs. F Rs. 14,000 Rs. T Rs. 20,000 Rs.

    15,000 Rs. 18,000

    R I G 20,000 Rs. 15,000 H T 18,000 Rs. 19,000 15,000 Rs. 16,000

    ˆ H lives immediate left below diagonally of the one who lives second to the right of B’s flat. ˆ There is only one flat between the flats of T and the one who lives immediate left below diagonally of N’s flat in the same column. ˆ A lives immediate right below diagonally of the one who lives immediate right below diagonally of O. ˆ B lives immediate left below diagonally of the one who lives immediate left below diagonally of R. ˆ P lives immediate right above diagonally of the one who lives immediate left above diagonally of H. ˆ B lives to the immediate left of the one who lives immediate left above diagonally of U. ˆ Y lives immediae left below diagonally of the one who lives immediate left above diagonally of N. ˆ S lives immediate left below diagonally of the one who lives immediate right below diagonally of Y. ˆ R lives second to the right of the one who lives on the top floor above B in the first column. Only one person lives between O and S.

    PROBLEM SOLVING ˆ N lives on the floor numbered 3. Only one person lives between N and the one who lives immediate right below diagonally of V. 503. What is the sum of rent paid by T and V ? (1) Rs.31,000 (2) Rs.30,000 (3) Rs.35,000 (4) Rs.33,000 (5) Rs.38,000 504. Who lives immediate left above diagonally of the one who pays Rs.19,000 as monthly rent? (1) H (2) P (3) B (4) V (5) A 505. Which of the following pair of tenants pays a sum of Rs.39,000 as monthly rent? (1) T and B (2) O and S (3) R and Y (4) N and B (5) U and S 506. Who among the following pays the least rent at the bottom floor of the apartment? (1) P (2) H (3) V (4) S (5) U 507. Who pays the highest rent in the flats located in the right end column of the apartment? (1) U (2) A (3) V (4) S (5) N Directions (508 - 512) : Study the following information carefully and answer the questions given below: IBPS Bank PO/MT CWE Main Exam, 30.11.2019 There are 12 Government officers M, N, O, P, Q, R, S, T, U, V, W and X belonging to 12 different departments. All of them are sitting around a rectangular table facing the centre but not necessarily in the same order. ˆ R is sitting fourth to the right of an officer who is sitting fourth to the right of the officer who belongs to Medical department. ˆ W is sitting fifth to the left of an officer who is sitting fifth to the left of an officer who belongs to Finance department. ˆ Q is sitting exactly between the officers from Medical and Commerce departments. ˆ U is sitting exactly between the officers from Irrigation and Law departments. ˆ W is sitting second to the right of an officer who is sitting to the immediate right of officer N. ˆ T is sitting second to the left of an officer who is sitting third to the right of officer P.

    ˆ P is sitting to the immediate left of an officer who is sitting third to the right of officer M. ˆ X is sitting third to the right of an officer who is sitting second to the right of officer O. ˆ S is sitting second to the right of an officer who is sitting third to the left of officer V. ˆ U is sitting exactly between the officers P and M and neither of them belongs to Welfare department. ˆ The officers from Finance, Medical, IT and PWD departments are sitting in the same order clockwise. ˆ The officers from Culture, Irrigation, Energy, Housing, Police and PWD departments are sitting in the same order anticlockwise. 508. Who among the following belongs to IT department? (1) N (2) S (3) U (4) V (5) O 509. X belongs to which department? (1) Medical (2) Finance (3) Law (4) Energy (5) Commerce 510. Who among the following sits exactly between officers from Police and Energy Departments? (1) T (2) R (3) P (4) N (5) S 511. Who among the following sits fourth to the right of officer from Cultural department? (1) Officer from Police department (2) Officer from Finance department (3) Officer from PWD department (4) Officer from Medical department (5) Officer from Housing department 512. Who among the following belongs to Welfare department? (1) U (2) S (3) W (4) P (5) T Directions (513–517) : Study the following information carefully and answer the questions given below : IBPS Bank PO/MT CWE Main Exam, 30.11.2019 Eight friends P, B, R, S, T, U, V and M were born in three different months of the same year - March, June and December but not necessarily in

    BPRE–694

    the same order. Each of them is from different cities namely Goa, Puducherry, Bangaluru, Chennai, Mangalore, Ooty, Kolkata and Agra but not necessarily in the same order. ˆ The person from Mangalore was born in a month which has only 30 days. ˆ U was not born in the same month as T. ˆ V is not from Mangalore and B is not from Puducherry. ˆ S is from Chennai and born in the same month as U. ˆ Not less than two people and not more than three people were born in a same month. ˆ Only B and M were born in March. R was not born in December. ˆ The person from Kolkata and the one from Bengaluru were born in the same month. ˆ The one who is from Kolkata was not born in the same month as M. ˆ U is not from Kolkata and T is not from Mangalore. ˆ R is from Agra and was born in the same month as T. ˆ The person from Goa was born in the same month as P. 513. V is from which city? (1) Agra (2) Chennai (3) Puducherry (4) Kolkata (5) Goa 514. The person born in December month belongs to which city? (1) Mangalore (2) Goa (3) Ooty (4) Agra (5) Chennai 515. Who is from Puducherry? (1) S (2) M (3) T (4) R (5) P 516. How many persons are younger to B ? (1) Three (2) Four (3) Five (4) Six (5) Seven 517. How many friends are born between M and U? (1) Two (2) Three (3) Four (4) Five (5) No one Directions (518–522) : Study the following information carefully and answer the questions that follow : (IBPS RRBs Officer Scale-I CWE Main Exam, 13.10.2019)

    There is a housing apartment having four floors. Each floor has three flats. The bottom floor is numbered 1 and so as to the top floor numbered as

    PROBLEM SOLVING 4. The flats located at left end of the building are numbered as Column 1 and so as to the flats located at right end of the building are numbered as Column 3. Twelve different persons A, B, C, D, E, F, G, H, I, J, K and L live in different flats. Each person is from different city, as shown in the table : l D lives in a flat to the immediate left of the flat that is immediate right above diagonally of H’s flat. l E lives second to the right of the flat that is immediate below B’s flat and J is at the bottom floor. l H lives in a flat immediate left above diagonally of a flat that is immediate right above diagonally of F’s flat. l G lives in a flat immediate right above diagonally of a flat that is immediate right above diagonally of F’s flat. l B lives in a flat immediate left below diagonally of a flat that is immediate left below diagonally of C’s flat. l A lives in a flat immediate right above diagonally of a flat that is two flats below I’s flat. l I lives in a flat immediate right above diagonally of a flat that is immediate below flat of D’s flat. l K lives in a flat immediate above a flat that is immediate left above diagonally of E’s flat. Bangalore Chandigarh Patiala

    Mumbai Chennai Delhi

    Hyderabad Bhubaneswar Trivandrum

    Kochi

    Coimbatore

    Patna

    518. Who among the following is from Chennai? (1) D (2) K (3) B (4) A (5) J 519. Who lives second to the left of a person from Hyderabad? (1) B (2) A (3) D (4) F (5) E 520. Who lives in a flat that is exactly between the persons from Kochi and Patna? (1) L (2) K (3) I (4) J (5) C 521. C is second to the right/left of a person from which city? (1) Bangalore (2) Bhubaneswar (3) Patiala (4) Hyderabad (5) Kochi

    522. Who lives second to the left of the person from Patna? (1) L (2) G (3) F (4) J (5) C Directions (523–527) : Study the following information carefully and answer the questions given below : (IBPS SO Prelim Exam, 28.12.2019)

    Eight people – A, B, C, D, P, Q, R and S – conducted a lecture on different dates in four different months viz. January, May, June and August of the same year. The lecture was conducted either on 9th or 18th of each month. Only one person conducted a lecture on each date. No other person conducted a lecture in the given year. Each person conducted the lecture in a different city. A conducted the lecture on 18th of a month in one of the months after May. Only two people conducted the lecture between A and the one who conducted the lecture in Mumbai. The one who conducted the lecture in Delhi conducted in the same month as the one who conducted the lecture in Mumbai. Only three people conducted the lecture between B and the one who conducted in Delhi. C conducted the lecture immediately before B. As many people conducted a lecture before C as after the one who conducted the lecture in Hyderabad. The one who conducted the lecture in Bengaluru conducted in the same month as the one who conducted in Hyderabad. Only three people conducted the lecture between D and the one who conducted in Bengaluru. Only one person conducted the lecture between D and P. Only three people conducted the lecture between P and the one who conducted in Lucknow. Only two people conducted the lecture between the one who conducted in Chennai and the one who conducted in Lucknow. Q conducted the lecture on one of the dates after R but before S. The one who conducted the lecture in Chandigarh conducted on one of the dates before the one who conducted in Pune. 523. How many people conducted the lectures between R and the one who conducted the lecture in Hyderabad? (1) One (2) Three (3) More than three (4) Two (5) None

    BPRE–695

    524. Four of the following five are alike in a certain way as per the given arrangement and hence they form a group. Which one of the following does not belong to that group? (1) The one who conducted the lecture in Mumbai (2) D (3) The one who conducted the lecture in Delhi (4) S (5) C 525. Which of the following statements is TRUE based on the given information? (1) P conducted the lecture on 9th August. (2) Only two people conducted the lecture before B. (3) Only four people conducted the lecture between A and the one who conducted the lecture in Chandigarh. (4) S conducted the lecture in Bengaluru. (5) None of the given statements is true. 526. Who amongst the following conducted the lecture on 9th May? (1) D (2) C (3) B (4) S (5) P 527. Who amongst the following conducted the lecture in Pune? (1) B (2) S (3) Q (4) D (5) P Directions (528–532) : Study the following information carefully and answer the questions given below : (IBPS SO Prelim Exam, 28.12.2019)

    Eight people – P, Q, R, S, T, U, V and W – are sitting around a square table such that four of them sit at four corners of the table and four of them sit in the middle of each of the four sides. The ones sitting at the corners of the table are facing the centre and the ones sitting in the middle of the sides are facing outside (i.e. opposite to the centre). Each one of them likes different colours viz, Red, Blue, White, Yellow, Black, Green, Pink and Orange but not necessarily in the same order. V sits to the immediate right of the one who likes Black. Only three people sit between the ones who like Black and Pink. Q sits second to the left of the one who likes Pink. Only two people sit between P and W (either from left or right). P is neither an

    PROBLEM SOLVING immediate neighbour of the one who likes Black nor the one who likes Pink. P neither likes Black nor Pink. The one who likes Red is an immediate neighbour of W. T faces the centre of the table. Only three people sit between T and the one who likes Orange. W neither likes Orange nor Blue. The one who likes Blue sits third to the right of R. The one who likes Yellow sits second to the right of the one who likes Green. Only one person sits between the one who likes Green and S (either from left or right). 528. Four of the following five are alike in a certain way as per the given arrangement and hence they form a group. Which one of the following does not belong to that group? (1) Q–The one who likes Yellow (2) V–The one who likes Pink (3) U–The one who likes Orange (4) P–The one who likes Blue (5) R–The one who likes Green 529. Who amongst the following likes White colour? (1) P (2) R (3) S (4) Q (5) T 530. How many people sit between U and the one who likes Pink (when counted from the right of U)? (1) None (2) More than three (3) One (4) Three (5) Two 531. Which of the following statements is TRUE as per the given ar rangement? (1) R is an immediate neighbour of the one who likes Black. (2) Only three people sit between S and Q. (3) None of the given statements is true. (4) Both S and the one who likes White face the centre. (5) V likes Orange colour. 532. Who sits third to the left of the one who likes Green? (1) The one who likes Blue (2) Q (3) V (4) P (5) The one who likes Black Directions (533–537) : Study the following information carefully and answer the questions given below : (Indian Bank Specialist Officer SO Exam, 08.03.2020)

    Eight people work in three different countries viz. Hong Kong, Brazil and Argentina with atleast two and not more than three people working in each country. Each of them likes a different colour. (Note : Please note that in the given information if it is stated that ‘X’ works with the one who likes ‘Y’ then it means that ‘X’ and the one who likes ‘Y’ work in the same country and also that ‘X’ does not like ‘Y’) : The one who likes Blue works with C in Hong Kong. A works with the one who likes Orange but not in the same country as C. D works with only E. The one who likes Pink works with the one who likes Red. C neither likes Pink nor Red. A does not like red. The one who likes red does not work with D. G works with the one who likes Yellow. C does not like Yellow. B and the one who likes Green work in different countries. Neither D nor E likes Green. B does not work in Argentina. B does not like Orange. F and the one who likes Purple work in different countries. F and the one who likes Green work in different countries. D does not like Purple. H and the one who likes White works in different countries. 533. Which of the following statements is not true as per the given information? (A) The one who likes Yellow works in Hong Kong. (B) The one who likes Orange works with the one who likes Pink. (C) A works in Argentina. (1) Only A (2) Only C (3) Only A and C (4) All the given statements are true (5) Only B 534. Who amongst the following likes Purple? (1) C (2) The one who works in Hongkong (3) B (4) The one who works with the one who likes White (5) D 535. Which colour does H like? (1) Orange (2) Yellow (3) Blue (4) Green (5) White 536. Four of the following five are alike in a certain way as per the given arrangement and thus they form a group. Which of the following does not belong to that group?

    BPRE–696

    (1) D-Purple (2) B-Orange (3) H-Blue (4) C-Yellow (5) A-Blue 537. Who amongst the following work in Brazil? (1) The one who likes White and E. (2) The one who likes Blue and A. (3) The ones who like Pink, Orange and B (4) The ones who like Green, Blue and Yellow (5) The one who likes Blue, A and F Directions (538 –542) : Study the following information carefully and answer the questions given below. (IBPS RRBs Officer CWE Prelim Exam, 13.09.2020)

    There are ten persons namely J, K, L, M, N, P, Q, R, S and T. They stay in a five floor building. The bottom floor is numbered 1, the one above that is numbered 2 and so on till the topmost floor is numbered 5. Each floor of the building has two flats namely flat A and flat B. Flat A is to the west of flat B. Flat A of floor 2 is immediately above the flat A of floor 1 and immediately below the flat A of floor 3 and flat B of floor 2 is immediately above the flat B of floor 1 and immediately below the flat B of floor 3 and so on. Note : The same flat type means if one person stays in flat A then the other person also stays in flat A if they are staying in the same flat type. Q stays in flat A of an odd numbered floor, but not the topmost floor. There are two floors between the floors of N and R. J stays just above Q. S and N are in different flat types on different floors. S stays on the topmost floor. There is no one to the east of R. M is to the north-east of L. N and R stay in the same flat type. P stays on one of the floors below Q, but not immediately below Q. R’s floor is just below the floor of S. M does not stay on the fifth floor. T stays below K. 538. Who is staying to the west of M? (1) S (2) L (3) J (4) Q (5) R 539. How many floors are there between T and K? (1) One (2) Two (3) Three (4) They are on the same floor (5) Zero 540. Who stays second above L on the same type of flat as that of L? (1) R (2) K

    PROBLEM SOLVING (3) S (4) P (5) J 541. Who stays immediately below K? (1) S (2) M (3) R (4) T (5) P 542. Which of the following combinations stays on the fourth floor? (1) S and K (2) Q and M (3) R and L (4) J and R (5) P and N Directions (543 – 547) : Study the following information carefully and answer the questions given below : (IBPS RRBs Officer CWE Prelim Exam, 13.09.2020)

    Six people Amit, Binod, Chirag, Dheeraj, Emran and Farhan like six different sports — Football, Hockey, Cricket, Tennis, Basketball and Kabaddi, but not necessarily in the same order. They also belong to different cities — Ranchi, Bhubaneshwar, Gurgaon, Kolkata, Delhi and Shimla, but not necessarily in the same order. Amit does not like Basketball and belongs to Ranchi. Dheeraj likes Hockey but does not belong to Bhubaneshwar. The one who belongs to Delhi likes Kabaddi. Emran is not from Kolkata and his favourite sport is Tennis. Binod belongs to Delhi and Farhan likes Cricket. Neither Farhan nor the person who likes Basketball belongs to Bhubaneshwar. The person who likes Hockey belongs to Gurgaon. Chirag does not belong to Kolkata. 543. Who among the following likes Basketball? (1) Dheeraj (2) The one who belongs to Shimla (3) The one who belongs to Ranchi (4) Binod (5) Emran 544. Which of the following statements is not true? (1) The person from Ranchi likes Football. (2) Chirag belongs to Shimla and likes Basketball. (3) Emran belongs to Bhubaneshwar and likes Tennis. (4) The one who likes Hockey belongs to Gurgaon. (5) Binod belongs to Delhi and likes Cricket. 545. Who among the following likes Kabaddi ? (1) Binod (2) Emran (3) Farhan (4) Chirag (5) Dheeraj

    546. Who among the following belongs to Shimla? (1) Amit (2) Binod (3) Chirag (4) Dheeraj (5) Emran 547. Which of the following statements is true? (1) Binod likes Cricket and belongs to Delhi (2) Farhan belongs to Kolkata and likes Kabaddi (3) Dheeraj belongs to Ranchi and likes Hockey (4) Emran belongs to Bhubaneshwar and likes Tennis (5) Amit belongs to Shimla and likes Football Directions (548–552) : Study the following information carefully and answer the questions given below : (IBPS Bank PO/MT CWE Prelim Exam, 03.10.2020)

    Six friends, namely A, B, C, D, E and F have their birthdays on the same date but in different months of the same year viz., January, February, March, April, July and November, but not necessarily in the same order. All friends have different professions viz., Doctor, Lawyer, Soldier, Manager, Accountant and Engineer, but not necessarily in the same order. The difference of ages between the Engineer and A is the same as between D and the Soldier. The one who is older to F is a Manager. The Manager is not the oldest. A is six months older to the Lawyer. B was born in a month having 30 days and he is a Soldier. E is an Engineer and was born in February. F is neither a Doctor nor an Accountant. The youngest friend is an Accountant. 548. Who among the following was born in the month of January? (1) C (2) A (3) D (4) F (5) B 549. Four of the following five are alike in a certain manner and hence they form a group. Which one of the following does not belong to the group ? (1) F (2) D (3) B (4) E (5) C 550. Who among the following is a Lawyer ? (1) A (2) C (3) F (4) D (5) Cannot be determined 551. Who among the following is the youngest ?

    BPRE–697

    (1) F (2) B (3) E (4) A (5) C 552. Which of the following combinations of ‘Friend – Profession – Month’ is correct ? (1) D – Manager – March (2) A – Lawyer – April (3) F – Lawyer – March (4) B – Soldier – January (5) C – Doctor – January Directions (553–557) : Study the following information carefully and answer the questions given below : (IBPS Bank PO/MT CWE Prelim Exam, 03.10.2020)

    An apartment has four floors. There are three flats on each floor. The ground floor of the apartment is numbered as Row-1, the floor above it as Row-2 and so on till the topmost floor is numbered as Row-4. Similarly, the floors at the left end are numbered as Column–1, and the floors at the right end are numbered as Column–3. Twelve persons A to L live in different flats. J lives on the ground floor. E lives second to the right of flat which is just below the flat of B. A lives in a flat immediate right above diagonally of a flat that is two flats below the flat of I. B lives in a flat immediate left below diagonally of a flat that is immediate left below diagonally of the flat of C. D lives in a flat to the immediate left of a flat that is immediate right above diagonally of the flat of H. K lives in a flat immediate above a flat that is immediate left above diagonally of the flat of E. G lives in a flat immediate right above diagonally of a flat that is immediate right above diagonally of the flat of F. H lives in a flat immediate left above diagonally of a flat that is immediate right above diagonally of the flat of F. I lives in a flat immediate right above diagonally of a flat that is immediate below the flat of D. 553. Who among the following lives to the immediate left of L ? (1) A (2) C (3) J (4) K (5) B 554. Who among the following lives exactly between the flats of D and C? (1) H (2) I (3) K (4) B (5) E 555. Who among the following lives immediate right above diagonally of J ?

    PROBLEM SOLVING (1) A (2) E (3) B (4) L (5) F 556. Who among the following lives on the ground floor in the column in which C does live ? (1) J (2) K (3) A (4) E (5) F 557. Who among the following lives immediate left below diagonally of C ? (1) G (2) I (3) K (4) D (5) B Directions (558–562) : Study the following information carefully and answer the questions given below : (IBPS RRBs Officer CWE Prelim Exam, 31.12.2020)

    Seven teachers, namely, G, R, V, S, D, M and J of seven different subjects viz. There is a gap of one day between History class and Mathmatics class. G and J teach one after the other. English class is on the last day of the week. Only two teachers teach between M and S. R teachers Botany on Tuesday. More than two classes are held between the classes of Botany and Zoology. J does not teach History. D and G do not teach immediately beofre or immediately after S. G and S do not teach History or Mathematics. Neither M nor S teach on Sunday or Monday. The number of classes before English class is the same as the number of classes after Chemistry class. 558. On which day does S teach ? (1) Monday (2) Friday (3) Saturday (4) Sunday (5) Wednesday 559. Who among the following does teach History ? (1) M (2) S (3) D (4) J (5) R 560. How many teachers teach between the classes of Chemistry and Zoology ? (1) One (2) Two (3) Three (4) Four (5) Five 561. Which subject is taught on Thursday? (1) History (2) Botany (3) Mathematics (4) English (5) Physics 562. Who among the following teaches immediately after Zoology subject ?

    (1) G (2) V (3) S (4) M (5) R Directions (563–567) : Study the following information carefully and answer the questions given below : (IBPS RRBs Officer CWE Prelim Exam, 31.12.2020)

    Twelve persons from A to L live in a four storey building, each floor having three flats. The ground floor of the building is numbered one, the one above it is numbered two and so on till the topmost floor is numbered four. G lives just below the flat of the one who lives immediate left above diagonally of the flat of E. D lives just above the flat of the one who lives to the immediate left of L. K lives immeidate right above diagonally of the one who lives immeidate right below diagonally of the slat of D. B lives immediate left below diagonally of the one who lives immediate left above diagonally of the flat of E. D lives immediate left above diagonally of the one who lives tow flats below the flat of H. C lives immeidate right above diagonally of the one who lives immediate right above diagonally of the flat of F. J lives second to the left of the one who lives two flats above the flat of I. I lives immediate right above diagonlly of the one who lives two flats below the flat of A. 563. Who among the following lives just below the flat of K ? (1) E (2) I (3) C (4) F (5) L 564. Four of the following five are alike in a certain way and hence they form a group. Which one of the following does not belong to that group ? (1) J (2) C (3) B (4) D (5) G 565. Who among the following lives exactly between the flats of F and I? (1) K (2) B (3) E (4) J (5) L 566. Who among the following lives immediate left above diagonally of A’s flat ? (1) J (2) C (3) H (4) K (5) D 567. Who lives in the same column with H ? (1) E (2) D (3) L (4) B (5) I

    BPRE–698

    Directions (568–572) : Study the following information carefully and answer the questions given below : (IBPS RRBs Officer CWE Prelim Exam, 31.12.2020)

    Seven boxes of seven different colours, viz., Blue, Red, Orange, Yellow, Green, White, and Pink are kept one above the another in a stack, but not necessarily in the same order. Each of the boxes has been given a unique number – 101, 121, 151, 191, 231, 221 and 225, but not necessarily in the same order. White coloured box is kept just below the box number 225, and at one of the positions above the box number 221. Only one box is kept between the White coloured box and the box bearing the number less than the number of White coloured box. White coloured box does not bear the number 191 or 231. Only two boxes are kept between the box number 151 and Blue coloured box. The difference between the numberes of Blue coloured box and the box just below it is less than 80. Yellow coloured box is not kept at the top of the stack. Yellow coloured box does not bear the number 121. Only two boxes are kept between the Yellow coloured box and Red coloured box. Only three boxes are kept between Pink coloured box and the box number 221. Only two boxes are kept between Pink coloured box and Orange coloured box. Orange coloured box is kept at one of the positions below the box number 221. Only one box is kept between Orange coloured box and the box number 121. 568. The box of which colour is kept just above the box number 121 ? (1) White (2) Green (3) Red (4) Pink (5) Blue 569. What is the number of Green coloured box ? (1) 151 (2) 121 (3) 225 (4) 231 (5) 191 570. What is the colour of box number 101 ? (1) Yellow (2) Orange (3) Blue (4) Pink (5) Red 571. Which box is kept just below the box number 221 ? (1) White (2) Pink (3) Blue (4) Orange (5) Red 572. Which box is kept exactly between the box number 151 and Orange coloured box ? (1) Yellow (2) Pink (3) Blue (4) Green (5) Red

    PROBLEM SOLVING Directions (573 – 577) : Study the following information carefully and answer the questions given below : (IBPS RRBs Officer CWE Prelim Exam, 07.08.2021)

    An apartment has four floors. There are three flats on each floor. Twelve tenants – I, J, K, L, M, N, O, D, E, F, G and H – live in different flats. The other details (Rent and profession of tenant) are given in the following table :

    Row

    Column

    1

    3

    2

    4

    Rs. 15,000 Rs. 12,000 Engineer Editor

    Rs. 10,000 Player

    3

    Rs. 16,000 Rs. 13,000 Journalist Doctor

    Rs. 16,000 Police

    2

    Rs. 10,000 Rs. 14,000 Politician Manager

    Rs. 16,000 Writer

    1

    Rs. 11,000 Rs. 15,000 Lawyer Dancer

    Rs. 15,000 Carpenter

    H lives three flats above the one who lives immediate left below diagonally of O. K lives immediate left above diagonally of the one who lives second to the right of J. F lives two flats above diagonally to the right of the one who lives immediate left above diagonally of G. D lives two flats above the flat of the one who lives immediate left below diagonally of I. E lives in the same column in which H lives. K lives on an even numbered floor. E lives exactly between N and J. M and N live in the same row. E lives second to the left of the one who lives immediate right below diagonally of D. L lives two flats below diagonally to the left of the one who lives immediate below of F in the same column. 573. What is the average of the rents paid by tenants M, F, H and L together ? (1) Rs. 13,000 (2) Rs. 11,000 (3) Rs. 10,750 (4) Rs. 12,750 (5) Rs. 14,500 574. Manager and another tenant together pay Rs. 26,000 as rent per month. These two tenants are : (1) O and Editor (2) O and F (3) M and K (4) J and N (5) L and O

    575. Who among the following does not live in the Column–2 ? (1) M (2) D (3) K (4) O (5) J 576. Which of the following combinations of Tenant and Profession is not correct ? (1) L – Dancer (2) K –Editor (3) N – Carpenter (4) H – Engineer (5) E – Politician 577. The total rent paid by L, D and H together is (1) Rs. 37,000 (2) Rs. 39,000 (3) Rs. 42,000 (4) Rs. 40,000 (5) Rs. 41,000 Directions (578–582) : Study the following information carefully and answer the questions given below : (IBPS RRBs Officer CWE Prelim Exam, 07.08.2021)

    Seven Professors A, B, C, D, E F and G are engaged in evaluation of answer papers in three different subjects English, Mathematics and History. At least two persons evaluate the papers in each subject. Each of the evaluators stay in different buildings P, Q, R, S, T, V and W but not necessarily in the same order. A evaluates English papers only with E and stays in building R. D stays in building W and does not evaluate

    BPRE–699

    Mathematics papers. The one who stays in building V evaluates History papers. B and C do not evaluate the papers in the same subject. Those who evaluate English papers do not stay in building Q. F stays in building P but does not evaluate History papers. G evaluates same papers as F. C stays in building T. 578. Who stays in building V ? (1) E (2) F (3) G (4) B (5) Other than those given as options 579. Which of the following combinations of subject, person and building is difinitely correct? (1) Mathematics – F – Q (2) Mathematics – G – Q (3) History – D – T (4) History – E – S (5) Other than those given as options 580. Which of the following groups of persons evaluate the Mathematics papers ? (1) CF (2) EFG (3) CFG (4) FG (5) Other than those given as options 581. Papers in which subject are evaluated by D ? (1) History (2) Mathematics (3) English (4) English or Mathematics (5) History or Mathematics 582. E stays in which building ? (1) P (2) Q (3) T (4) Cannot be determined (5) Other than those given as options Directions (583–587) : Study the following information carefully and answer the questions given below : (IBPS Bank PO/MT CWE Prelim Exam, 04.12.2021)

    Eight friends, A, B, C, D, E, F, G and H, have their birthdays in different months of the same year – January, April, May and July – such that not more than two friends have their birthdays in the same month. The birthdays are either on 14th or 23rd of the month. No two friends have their birthdays on the same date of the same month. D was born in July. The birthday of H is immediately after the birthday of B. The birthday of E is immediately before the birthday of B. The number of friends who have their birthdays between the birthdays of G and H is

    PROBLEM SOLVING equal to the number of friends who have their birthdays between the birthdays of B and D. Both E and B have their birthdays in the same month. Only three friends have their birthdays between the birthdays of F and C. A was born after G. 583. In which of the following months E was born ? (1) January (2) April (3) May (4) July (5) Cannot be determined 584. Who among the following has his birthday in the same month as that of H’s birthday? (1) G (2) D (3) F (4) E (5) C 585. Who among the following has his birthday in the same month as that of F’s birthday? (1) H (2) D (3) E (4) G (5) A 586. Who among the following has his birthday in the same month as that of D’s birthday? (1) A (2) B (3) C (4) E (5) F 587. Who among the following was not born on 23rd of any of the given months ? (1) F (2) D (3) A (4) B (5) C Directions (588–592) : Study the following information carefully and answer the questions given below : (IBPS Bank PO/MT CWE Prelim Exam, 04.12.2021)

    There is an apartment having four floors. There are three flats on each floor. The bottom floor is called Row1, the one above it is called Row-2 and so on till the topmost floor is called Row-4. The flats at the left end are in the Column-1 and the flats at the right end are in the Column-3. Twelve persons, namely, S, T, U, V, A, M, Y, N, O, P, B and R, live in different flats. Y lives immediate left below diagonally of the one who lives immediate left above diagonally of N’s flat. S lives immediate left below diagonally of the one who lives immediate right below diagonally of Y.

    A lives immediate right below diagonally of the one who lives immediate right below diagonally of O. B lives immediate left below diagonally of the one who lives immediate left below diagonally of R. P lives immediate right above diagonally of the one who lives immediate left above diagonally of M’s flat. B lives immediate left of the one who lives immediate left above diagonally of U. R lives second to the right of the flat which is two flats above the flat of B. N lives in the flat which is two flats above the flat that is immediate right below diagonally of V’s flat. T lives immediate left below diagonally of the one who lives second to the right of B’s flat. T lives two flats above the flat in the same column which is immediate left below diagonally of N’s flat. 588. Four of the following five are alike in a certain way and hence they form a group. Which one of the following does not belong to that group? (1) T (2) M (3) R (4) V (5) P 589. Who among the following does live just below Y ? (1) B (2) O (3) V (4) S (5) M 590. A is related to B in the same way as R is related to _______. (1) T (2) Y (3) S (4) V (5) O 591. Who among the following do(es) not live in the Row-2 ? I. S II. V III. A IV. R (1) Only I (2) Only IV (3) Only II and III (4) Only I and IV (5) Only I, II and IV 592. Who among the following does live just above V ? (1) M (3) T (5) Y

    (2) P (4) R

    BPRE–700

    SBI PO EXAMS Directions (1–5) : Study the following information carefully and answer the questions given below : (SBI Management Executive Exam, 23.02.2014)

    In each of the following questions two rows of numbers are given. The resultant number in each row is to be worked out separately based on the following rules and the question below the rows of numbers are to be answered. The operations of numbers progress from left to right. Rules : (i) If an odd number is followed by a two digit even number then they are to be added. (ii) If an odd number is followed by a two digit odd number then the second number is to be subtracted from the first number. (iii) If an even number is followed by a number which is a perfect square of a number then the second number is to be divided by the first number. (iv) If an even number is followed by a two digit even number then the first number is to be multiplied by the second number. (SBI PO Exam. 09.01.2005)

    1. 15 8

    11 20 400 12 10 If the resultant of the second set of numbers is divided by the resultant of the first set of numbers what will be the outcome ? (1) 196 (2) 200 (3) 19 (4) 92 (5) None of these 2. 40 30 3600 15 24 17 What is the sum of the two resultant numbers of the set of numbers given above ? (1) 22 (2) 25 (3) 28 (4) 42 (5) None of these 3. 8 16 16 14 13 11 12 144 What is the difference between the resultant of the first set of numbers and the second set of numbers ? (1) 222 (2) 210 (3) 118 (4) 106 (5) None of these

    PROBLEM SOLVING 4. 13 11 4 17 13 12 If the resultant of the first set of numbers is multiplied by the resultant of the second set of numbers, what will be the outcome ? (1) 48 (2) 96 (3) 69 (4) 75 (5) None of these 5. 19 15 12 15 12 23 16 If the resultant of the second set of numbers is subtracted from the resultant of the first set of numbers what will be the outcome ? (1) 44 (2) 92 (3) 29 (4) 43 (5) None of these Directions (6–10) : Study the following information carefully and answer the given questions : (SBI Management Executive Exam, 23.02.2014)

    Eight persons – A, B, C, D, E, F, G and H – are travelling in three different Cars namely X, Y and Z, but not necessarily in the same order. There are at least two persons in each Car and each Car has persons of both the sexes. Out of eight persons three are females. All of them like different colours viz, Red, Green, Yellow, Blue, Black, White, Grey and Purple but not necessarily in the same order. One female does not like Grey or White colour. D is travelling with G in the Car Z. G likes Black colour. The persons who like Red and Purple colours, are travelling in the same Car. E does not like Purple colour and he is not travelling with H in the same car. A does not like purple nor Red colour. H is travelling in the Car Y. B, C and H are females in the group. B likes Blue colour and travels with the person who likes Black colour. Red and Green colours are liked by female members. The person who is travelling with H, likes Grey colour. One of the persons travelling in Car Z likes Yellow colour. 6. In which of the following Cars only two persons are travelling? (1) Car X (2) Car Y (3) Car Z (4) Cannot be determined (5) None of these 7. The female member who is travelling in the Car X likes which colour ?

    (1) Green (2) Blue (3) Yellow (4) Red (5) None of these 8. Who among the following likes Yellow colour ? (1) D (2) E (3) A (4) F (5) None of these 9. Who among the following is/are travelling with female member C? (1) E and G (2) A and D (3) E and F (4) D and E (5) F and G 10. Which of the following combinations of Person-Car-Colour is not correct ? (1) C – X – Red (2) F – X – Purple (3) G – Z – Black (4) D – Z – Yellow (5) E – Y – Grey Directions (11–13) : Study the following information carefully and answer the questions given below : (SBI Associates PO Online Exam, 30.11.2014)

    Each of the six people U, V, W, X, Y and Z stayed for a different number of days in India. X stayed for less number of days than only V. Y stayed for more number of days than only two people. U did not stay less than Y. W did not stay for the minimum number of days. The one who stayed for the second lowest number of days stayed for eight days. U stayed for 17 days in India. 11. The one who stayed for the maximum number of days stayed for 15 more days than W. Which of the following is possibly the number of days for which it stayed in India ? (1) 5 (2) 32 (3) 23 (4) 20 (5) 17 12. Which of the following is true with respect to Z as per the given information ? (1) The possible number of days for which Z stayed in India is 14 days. (2) Z stayed for the minimum number of days. (3) None of the given options is true (4) Only two people stayed for more number of days than Z. (5) Z definitely stayed for more number of days than Y.

    BPRE–701

    13. Which of the following may be the possible number of days for which Y stayed in India ? (1) 11 (2) 8 (3) 19 (4) 5 (5) 22 Directions (14–19) : Study the following information carefully and answer the questions given below : (SBI Management Executive Exam. 19.09.2014)

    A building has eight floors, numbered one to eight in such a manner that the ground floor is numbered one, the floor above it is numbered two and so on such that the topmost floor is numbered eight. one of the eight persons — P, Q, R, S, T, U, V and W — lives on each floor, but not necessarily in the same order. Each one of them is a professor of different subjects viz, English, Geography, Statistics, Hindi, Mathematics, Economics, Sociology and History but not necessarily in the same order. Three persons live between R and U. R lives on an odd numbered floor. Neither R nor U lives on the ground floor. The professor of Sociology lives immediately below R. Three persons live between the professor of Sociology and the professor of History. P lives immediately below U and immediately above the professor of Mathematics. Q is not the professor of Mathematics. V lives on fourth numbered floor. V is the professor of Hindi . The professor of Economics lives on the topmost floor. Two persons live between V and the professor of English. R is not the professor of English. S lives immediately above the professor of Statistics. Neither S nor Q is the professor of Economics. W is not the professor of Mathematics. 14. Who among the following lives exactly between the floors of T and the professor of Geography? (1) P (2) Professor of Sociology (3) Professor of Hindi (4) W (5) None of these 15. Who among the following lives on the topmost floor? (1) W (2) R (3) U (4) T (5) V

    PROBLEM SOLVING 16. Who among the following lives immediately above the floor of the professor of Statistics? (1) Professor of Geography (2) Professor of Hindi (3) Professor of Mathematics (4) Professor of Sociology (5) Professor of History 17. Who among the following lives on third numbered floor? (1) Professor of Mathematics (2) Professor of Geography (3) Professor of English (4) Professor of Statistics (5) Professor of History 18. How many persons live between the floors of W and the professor of Geography? (1) None (2) Two (3) One (4)Four (5) Three 19. Four of the following five are alike in a certain way and hence form a group. Which one of the following does not belong to that group\ (1) Professor of Economics (2) Professor of Sociology (3) Professor of Hindi (4) Professor of History (5) Professor of English Directions (20–24) : Study the following information carefully and answer the questions given below : (SBI Associates PO Online Exam. 29.11.2014)

    Eight people P, Q, R, S, T, U, V and W live on separate floors of an 8– floor building. Ground floor is numbered–1, first floor is numbered– 2 and so on until the topmost floor is numbered 8. ● P lives on floor numbered five. Only two people live between P and Q. Q lives above P. ● V lives immediately above S. T lives immediately above R. ● Only one person lives between T and W. ● T lives above W. U lives on an odd numbered floor. 20. How many people live between the floors on which S and R live ? (1) More than four (2) Four (3) Two (4) One (5) Three

    21. Who lives on the floor immediately above P ? (1) S (2) W (3) R (4) U (5) Other than those given as options 22. Which of the following statements is true with respect to the given information ? (1) Only one person lives between W and U (2) U lives on floor No.1 (3) V lives on the top most floor (4) Only two people live between T and S (5) P lives immediately above W 23. Who among the following lives on floor No. 2 ? (1) T (2) S (3) V (4) R (5) W 24. Who among the following lives between R and U ? (1) P (2) No–One (3) T (4) W (5) V Directions (25–27) : Study the following information carefully and answer the questions given below : (SBI Associates PO Online Exam. 29.11.2014)

    Each of the six poles P, Q, R, S, T and U has a different height. P is taller than only three poles. S is taller than T but shorter than U. U is shorter than P. Q is not the tallest. The second shortest pole is 2 feet tall. The third tallest pole is 5 feet tall. 25. Which of the following may be the possible height of pole U ? (1) 7 Feet (2) 2 Feet (3) 5 Feet (4) 3 Feet (5) 1 Feet 26. The tallest pole is 11 feet taller than S. Which of the following may be the possible height of Q ? (1) 4 Feet (2) 9 Feet (3) 13 Feet (4) 16 Feet (5) 17 Feet 27. Which of the following is true with respect to the given information ? (1) Only two poles are shorter than Q. (2) Q’s height is definitely less than 2 Feet (3) None of the given options is true (4) S is the second tallest pole (5) T is the shortest pole

    BPRE–702

    Directions (28–30) : Study the following information carefully and answer the questions given below : (SBI Associates PO Online Exam. 30.11.2014)

    Each of the six people U, V, W, X, Y and Z stayed for a different number of days in India. X stayed for less number of days than only V. Y stayed for more number of days than only two people. U did not stay less than Y. W did not stay for the minimum number of days. The one who stayed for the second lowest number of days stayed for eight days. U stayed for 17 days in India. 28. The one who stayed for the maximum number of days stayed for 15 more days than W. Which of the following is possibly the number of days for which it stayed in India ? (1) 5 (2) 32 (3) 23 (4) 20 (5) 17 29. Which of the following is true with respect to Z as per the given information ? (1) The possible number of days for which Z stayed in India is 14 days. (2) Z stayed for the minimum number of days. (3) None of the given options is true (4) Only two people stayed for more number of days than Z. (5) Z definitely stayed for more number of days than Y. 30. Which of the following may be the possible number of days for which Y stayed in India ? (1) 11 (2) 8 (3) 19 (4) 5 (5) 22 Directions (31–35) : Study the following information carefully and answer the questions given below : (SBI Associates PO Online Exam. 30.11.2014)

    Eight people – A, B C, D, E, F, G and H – live on separate floors of an 8 – floor building. Ground floor is numbered 1, first floor is numbered 2 and so on until the top most floor is numbered 8. C lives on Floor No. 3. Only one person lives between C and F. Only one person lives between C and G. E lives immediately above B. H lives immediately above F. H lives on any of the floors below D. Only one person lives between D and A. D lives above A.

    PROBLEM SOLVING 31. Who lives on the floor immediately below D ? (1) Other than those given as options (2) H (3) A (4) G (5) C 32. Which of the following is true with respect to the given infor mation ? (1) Only one person lives between E and G (2) A lives on a even numbered floor (3) A lives immediately below C (4) F lives on Floor number 5 (5) A lives immediately above H 33. How many people live between the floors on which D and C live ? (1) One (2) More than three (3) Two (4) Three (5) No one 34. Who amongst the following lives between B and G ? (1) No one (2) C (3) E (4) H (5) D 35. Who amongst the following lives on Floor number 5 ? (1) G (2) A (3) F (4) D (5) H Directions (36–40) : Study the following information carefully and answer the questions given below : (SBI PO Phase–I (Preliminary) Online Exam. 20.06.2015)

    Seven persons – M, N, O, P, Q, R and S – live on separate floors of a seven–storeyed building, but not necessarily in the same order. The ground floor of the building is numbered 1, the floor above it 2 and so on until the topmost floor is numbered 7. Each person likes differnt cartoon characters, viz, Chipmnuk, Flinstone, Jetson, Popeye, Scooby Doo, Simpson and Tweety, but not necessarily in the same order. The person who likes Popeye lives on floor numbered 4. Only two persons live between P and the one who likes Popeye. M does not live on the lowermost floor. M lives on any odd numbered floor below the one who likes Popeye. S lives on an even numbered floor but neither immediately above nor immediately below the floor of M. Only two persons live between M and the person who likes Tweety. Only one person lives between N and R. R lives on

    an even numbered floor and does not like Popeye. Only three persons live between the persons who like Chipmnuk and Jetson respectively. The person who likes Chipmnuk live on any floor above the N’s floor. The person who likes Chipmanuk does not live on the topmost floor. O does not like Chipmnuk or Jetson. The person who likes Scooby Doo lives on the floor immediately above the floor of the perosn who likes Simpson. 36. How many persons live between the floors on which S and P live ? (1) Three (2) Two (3) Four (4) Five (5) No One 37. Which of the following statements is/are true according to the given information? (1) Q lives on floor numbered 5 and he does not like Popeye (2) M likes Scooby Doo and he does not live on floor numbered 4. (3) O likes Flinstone and he lives on the topmost floor (4) Only two persons live between the floors of Q and R (5) All the statements are true 38. Who among the following lives on the floor immediately above the floor of M ? (1) N (2) R (3) S (4) O (5) No One 39. Who among the following lives exactly between the floors on which S and N live ? (1) P (2) R (3) M (4) Q (5) No one 40. Who among the followimg does like cartoon character Jetson ? (1) R (2) P (3) N (4) Q (5) S Directions (41 – 45) : Study the following information carefully and answer the questions given below : (SBI PO Phase–I (Preliminary) Online Exam. 21.06.2015)

    Seven persons A, B, C, D, K, L and M live on seven different floors of a building but not necessarily in the same order. The lowermost floor of the building is numbered one, the one above that is numbered two and so on till the topmost floor is numbered sev-

    BPRE–703

    en. Each one of them also likes different games namely–Snooker, Badminton, Chess, Ludo, Cricket, Hockey and Polo (but not necessarily in the same order.) Only three persons live between B and K. B lives on one of the floors above K. K does not live on the lowermost floor. Only one person lives between B and the one who like Chess. The one who likes Polo lives on one of the even numbered floors above the one who likes Chess. Only two persons live between M and the one who likes Chess. The one who likes Snooker lives immediately above M. A lives immediately above L. A does not like Chess. The one who likes Ludo lives on one of the odd numbered floors below L. M does not like Ludo. D lives on one of the floors above C. Only one person lives between the one who likes Cricket and the one who likes Hockey. D does not like Cricket. M does not like Badminton. 41. Which of the following games does B like ? (1) Snooker (2) Ludo (3) Polo (4) Badminton (5) Chess 42. Who amongst the following lives on the floor numbered 4 ? (1) The one who likes Hockey (2) The one who likes Chess (3) A (4) L (5) B 43. Which of the following statements is true with respect to the given arrangement ? (1) Only two persons live between K and M. (2) The one who likes Hockey lives immediately above K. (3) C likes Chess (4) C lives on an even numbered floor. (5) None of the given options is true. 44. If all the persons are made to sit in alphabetical order from top to bottom, the positions of how many people will remain unchanged ? (1) None (2) Three (3) Two (4) One (5) Four

    PROBLEM SOLVING 45. Which of the following combinations is true with respect to the given arrangement ? (1) Polo-C (2) Ludo-B (3) Cricket-K (4) Snooker-A (5) Chess-L Directions (46 – 50) : Study the following information carefully and answer the questions given below : (SBI PO Phase–I (Preliminary) Online Exam. 27.06.2015)

    Seven persons J, K, L, M, N, O and P live on seven different floors of a building but not necessarily in the same order. The lower most floor of the building is numbered 1, the one above that is numbered 2 and so on till the topmost floor is numbered 7. Each one of them also likes a different subject namely-English, History, Commerce, Biology, Accounts, Geography and Computer (but not necessarily in the same order.) ● J lives on an odd numbered floor but not on the floor numbered 3. The one who likes Accounts lives immediately above J. Only two persons live between M and the one who likes Accounts. ● The one who likes History lives on one of the odd numbered floors above M. Only three persons live between L and the one who likes History. The one who likes Commerce lives immediately above L. ● The one who likes English lives immediately above the one who likes Computer. P lives on an odd numbered floor. ● Only one person lives between K and N. K lives on one of the floors above N. Neither L nor J likes Biology. N does not like Commerce. 46. Which of the following subjects does J like ? (1) Geography (2) Computer (3) Commerce (4) History (5) English 47. Which of the following combinations is true with respect to the given arrangement ? (1) (2) (3) (4) (5)

    Geography - L History - O Computer - P Accounts - M Biology - K

    48. If all the persons are made to sit in alphabetical order from top to bottom, the positions of how many persons will remain unchanged ? (1) Four (2) Two (3) None (4) One (5) Three 49. Which of the following statements is true with respect to the given arrangement ? (1) The one who likes Computer lives immediately below J. (2) O likes History. (3) None of the given options is true. (4) Only four persons live between M and P. (5) P lives immediately below N. 50. Who amongst the following lives on the floor numbered 2? (1) K (2) The one who likes English (3) The one who likes Computer (4) M (5) O Directions (51-57) % Study the following information carefully and answer the questions given below : (SBI Specialist Officer (Law Officer : MMGS Scale-II) Online Exam, 19.04.2014)

    P, Q, R, S, T, U, V and W live on different floors in the same building having eight floors numbered one to eight (the ground floor is numbered 1, the floor above it, number 2 and so on and the topmost floor is numbered 8). There are only two floors between the floors on which P and R live. R lives on an odd numbered floor. There are four floors between the floors on which P and W live. T lives on a floor immediately above the floor on which U lives. There are only two floors between the floors on which V and S live. V lives on a floor above the floor of S. Q does not live on a floor immediately above or immediately below the floor on which R lives. 51. On which of the following floors does Q live ? (1) Fifth (2) Third (3) Second (4) Fourth (5) Sixth 52. Who amongst the following live on the floors exactly between S and R ?

    BPRE–704

    (1) V, W and Q (2) V, U and W (3) U, V and Q (4) T, U and Q (5) U, Q and W 53. Who amongst the following live on the odd numbered floors excluding R ? (1) U, W and S (2) Q, V and U (3) U, T and S (4) T, W and S (5) None of these 54. Who amongst the following lives on the topmost floor ? (1) T (2) P (3) V (4) Q (5) W 55. On which of the following floors does R live ? (1) Fifth (2) First (3) Seventh (4) Third (5) Either third or fifth 56. How many persons live between the floors of T and Q ? (1) Five (2) Three (3) Two (4) Either three or four (5) Four 57. Who amongst the following live on the floor immediately above the floor of W ? (1) U (2) R (3) V (4) T (5) None of these Directions (58-60) : Study the following information carefully and answer the questions given below : (SBI Probationary Officer Online Exam, 21.06.2014)

    There are five statues – L, M, N, O and P – each of them having different height. Statue L is smaller than only statue M. Statue O is smaller than statue N. Statue O is longer than statue P. The height of the tallest statue is 20 feet. The height of the second smallest statue is 11 feet. 58. What will be the height of statue P? (1) 13 feet (2) 15 feet (3) 9 feet (4) 12 feet (5) 14 feet 59. What will be the height of the third tallest statue? (1) 13 feet (2) 10 feet (3) 19 feet (4) 9 feet (5) 11 feet

    PROBLEM SOLVING 60. If the expressions S = T > O > R and P < O < Z are true, then which of the following is not true? (1) T > O (2) R < S (3) Z > R (4) P > T (5) P < Z Directions (61–63) : Study the following information carefully and answer the questions given below : (SBI Probationary Officer Online Exam, 28.06.2014)

    In a building having five floors above the ground floor, E does not live on the topmost floor. A lives on the ground floor. There are four floors between B and A. The floor immediately below B’s floor is vacant. C lives between D and E. D does not live on the floor immediately above A’s floor. 61. Who lives immediately above A’s floor ? (1) C (2) B (3) E (4) Either D or E (5) Cannot be determined 62. Who lives on the topmost floor ? (1) E (2) C (3) D (4) B (5) Cannot be determined 63. Which of the following floors is vacant ? (1) Fourth (2) Second (3) Third (4) Fifth (5) Cannot be determined Directions (64–68) : Study the following information carefully and answer the questions given below : (SBI Associates PO Online Exam, 29.11.2014)

    Eight people P, Q, R, S, T, U, V and W live on separate floors of an 8– floor building. Ground floor is numbered–1, first floor is numbered– 2 and so on until the topmost floor is numbered 8. ● P lives on floor numbered five. Only two people live between P and Q. Q lives above P. ● V lives immediately above S. T lives immediately above R. ● Only one person lives between T and W. ● T lives above W. U lives on an odd numbered floor. 64. How many people live between the floors on which S and R live ? (1) More than four (2) Four (3) Two (4) One (5) Three

    65. Who lives on the floor immediately above P ? (1) S (2) W (3) R (4) U (5) Other than those given as options 66. Which of the following statements is true with respect to the given information ? (1) Only one person lives between W and U (2) U lives on floor No.1 (3) V lives on the top most floor (4) Only two people live between T and S (5) P lives immediately above W 67. Who among the following lives on floor No. 2 ? (1) T (2) S (3) V (4) R (5) W 68. Who among the following lives between R and U ? (1) P (2) No–One (3) T (4) W (5) V Directions (69–73) : Study the following information carefully and answer the questions given below : (SBI Associates PO Online Exam, 30.11.2014)

    Eight people – A, B C, D, E, F, G and H – live on separate floors of an 8 – floor building. Ground floor is numbered 1, first floor is numbered 2 and so on until the top most floor is numbered 8. C lives on Floor No. 3. Only one person lives between C and F. Only one person lives between C and G. E lives immediately above B. H lives immediately above F. H lives on any of the floors below D. Only one person lives between D and A. D lives above A. 69. Who lives on the floor immediately below D ? (1) Other than those given as options (2) H (3) A (4) G (5) C 70. Which of the following is true with respect to the given information ? (1) Only one person lives between E and G (2) A lives on a even numbered floor (3) A lives immediately below C (4) F lives on Floor number 5 (5) A lives immediately above H

    BPRE–705

    71. How many people live between the floors on which D and C live ? (1) One (2) More than three (3) Two (4) Three (5) No one 72. Who amongst the following lives between B and G ? (1) No one (2) C (3) E (4) H (5) D 73. Who amongst the following lives on Floor number 5 ? (1) G (2) A (3) F (4) D (5) H Directions (74 – 78) : Study the following information carefully and answer the questions given below : (SBI Assistant Manager (System) Exam, 17.01.2016)

    Eight friends namely P, Q, R, S, T, U, V and W live on eight different floors of a building but not necessarily in the same order. The ground floor is numbered 1 and the one above that is numbered 2 and so on till the topmost floor is numbered 8. S lives on an even numbered floor. Only three people live between S and T. Only one person lives between T and V. V lives on one of the floors below T. Only two persons live between V and U. W lives on a floor that is immediately below U. Only two persons live between T and Q. P does not live on the lowermost floor. 74. Who amongst the following lives immediately above Q’s floor ? (1) V (2) S (3) U (4) P (5) W 75. Four of the following groups are alike in a certain way based on the given arrangement and hence they form a group. Which is the one that does not belong to that group ? (1) RS (2) VW (3) TS (4) PU (5) QR 76. Which of the following statements regarding Q is true ? (1) Q live on an even numbered floor. (2) Q lives on the topmost floor (3) Only two persons live above Q’s floor (4) Only one person live between Q and U (5) None of the given statements is true

    PROBLEM SOLVING 77. Who amongst the following lives on the topmost floor ? (1) Q (2) T (3) V (4) W (5) R 78. On which of the following floors does P live ? (1) 1 (2) 3 (3) 7 (4) 5 (5) 4 Directions (79–84) : Study the following information carefully and answer the questions given below : (SBI PO Online (Pre.) Exam, 02.07.2016 (Ist Shift))

    Seven representatives of a company – P, Q, R, S, T, U and V – travelled to three different countries i.e. South Africa, Australia and France. Each of them travelled on different days of the week (no two persons travelled on the same day), starting on Monday and ending on Sunday. Minimum two persons travelled to each country and South Africa is the only country to which three persons travelled. P travelled to South Africa on Monday. U travelled to Australia but neither on Tuesday nor on Saturday. V travelled on Sunday but not to France. The one who travelled to Australia travelled on Tuesday and the one who travelled to France travelled on Saturday. T travelled on Wednesday. R travelled to South Africa but not on Thursday. Q did not travel to France. 79. If everyone’s trip is postponed by one day, who will be travelling on Wednesday? (1) U (2) R (3) Q (4) T (5) None of these 80. Who amongst the seven representatives travelled on Saturday? (1) S (2) Q (3) R (4) Cannot be determined (5) None of these 81. Which one of the following combinations is true according to the given information? (1) U – Thursday – South Africa (2) S – Wednesday – France (3) V – Monday – South Africa (4) R – Friday – Australia (5) None of these

    82. To which country and on which day did Q travel? (1) South Africa, Wednesday (2) Australia, Friday (3) Australia, Thursday (4) Australia, Tuesday (5) None of these 83. Who was the last one to travel? (1) P (2) R (3) V (4) S (5) None of these 84. Who amongst the following representative travel to South Africa? (1) Q (2) V (3) S (4) U (5) None of these Directions (85–89) : Study the following information carefully and answer the questions given below : (SBI PO Online (Pre.) Exam, 02.07.2016 (Ist Shift))

    Seven persons, namely L, M, N, O, P, Q and R will appear for a different exam but not necessarily in the same order, in seven different months (of the same year) namely January, February, April, May, July, September and December. Each of them also likes a different genre of TV shows namely Family, Action, Comedy, Reality, Animated, History and Thriller but not necessarily in the same order. O will appear for an exam in a month which has only 30 days. Only one person will appear between the one who likes Animated Shows and O. The one who likes Action will appear for an exam immediately before the one who likes Animated Shows. The one who likes Thriller will appear for an exam neither in the month which has 31 days nor in the month which has 30 days. Only two persons will appear for an exam between the one who likes Thriller and Q. M will appear for an exam immediately after Q and does not likes Action Shows. R will appear for an exam immediately before N. P likes History Shows and appear for an exam in December. The one who likes Family Shows will appear for an exam in a month which has 31 days. O does not like Comedy Shows. 85. Which of the following genres of TV shows does O like? (1) History (2) Thriller (3) Family (4) Action (5) Reality

    BPRE–706

    86. How many persons will appear for an exam between the months on which N and L will appear for an exam? (1) One (2) None (3) Three (4) Two (5) More than three 87. As per the given arrangement, January is related to Reality Shows and February is related to Action Shows following a certain pattern, with which of the following is July related to following the same pattern? (1) Family Shows (2) Thriller Shows (3) Comedy Shows (4) History Shows (5) Animated Shows 88. Which of the following represents the month in which L will appear for an exam? (1) December (2) May (3) July (4) September (5) Cannot be determined 89. Which of the following represents the persons who will appear for an exam in January and December respectively? (1) N, P (2) N, M (3) R, P (4) R, M (5) M, P Directions (90–95) : Study the following information carefully and answer the questions given below : (SBI PO Online (Pre.) Exam, 02.07.2016 (IInd Shift))

    Seven persons, viz, J, K, L, M, N, O and P attend wedding ceremony in seven different months of the same year, viz., February, March, May, June, August, November and December, but not necessarily in the same order. Each one of them also likes different fruits viz., Apple, Mango, Banana, Strawberry, Cherry, Grapes and Orange, but not necessarily in the same order. J attends a wedding ceremony in a month which has only 30 days. Only two persons attend wedding ceremonies between J and the one who likes Mango. The person who likes Mango does attend a wedding ceremony in a month which has 31 days. K attends a wedding ceremony immediately before the one who likes Mango. Only three persons attend wedding ceremonies between K and the one who likes Cherry. P attends a wedding ceremony before K. P does not attend a wedding

    PROBLEM SOLVING ceremony in the month which has more than 30 days. Only two persons attend wedding ceremonies between P and the one who likes Banana. O attends a wedding ceremony immediately after the one who likes Banana. N attends a wedding ceremony immediately before M. Only one person attends a wedding ceremony between M and the one who likes Apple. The one who likes Apple attends a wedding ceremony between the persons who likes Apple an Strawberry respectively. The one who likes Grapes attends a wedding ceremony in any month before the one who likes Strawberry. 90. Who among the following likes Strawberry? (1) N (2) K (3) O (4) J (5) M 91. Who among the following attends a wedding ceremony immediately before the one who attended a wedding ceremony in the month of June? (1) M (2) P (3) O (4) L (5) K 92. In the given arrangement Apple is related to March and Banana is related to August following a certain pattern, with which of the following is Cherry related to following the same pattern? (1) February (2) June (3) November (4) May (5) December 93. Which of the following combinations is correct as per the given arrangement? (1) N – Grapes (2) June – Banana (3) O – Cherry (4) November – Apple (5) December – K 94. How many persons attended wedding ceremonies between N and the one who attended a wedding ceremony in the month of November? (1) More than three (2) None (3) One (4) Two (5) Three 95. In which of the following pairs, both the persons attended wedding ceremonies in the months each of the which has less than 31 days?

    (1) P and M (2) J and L (3) P and K (4) L and O (5) K and N Directions (96–100) : Study the following information carefully and answer the questions given below :

    (1) P – Silver (2) S – White (3) M – Yellow (4) O – Pink (5) R – Orange Directions (101–106) : Study the following information carefully and answer the questions given below :

    (SBI PO Online (Pre.) Exam, 02.07.2016 (IInd Shift))

    (SBI PO Phase-I (Pre.) Online

    Seven boxes – M, N, O, P, Q, R and S – are kept one above the other and are of seven different colours viz., Yellow, Brown, Pink, Green, Silver, Orange and White, but not necessarily in the same order. Only three boxes are kept between M and S. The orange box is kept immediately below the box M. Only two boxes are kept between orange box and pink box. More than three boxes are kept above the pink box. Box Q is kept immediately below the pink box. Only one box is kept between R and Q. Box R is not of orange colour. Box N is kept below Box R but not immediately below R. Brown box is kept immediately above box P. Only two boxes are kept between brown box and silver box. Yellow box is kept immediately above the white box. 96. How many boxes are kept between Brown Box and Box S? (1) One (2) Two (3) None (4) More than three (5) Three 97. Which of the following boxes is kept immediately below the Box R? (1) White Box (2) Box S (3) Yellow Box (4) Green Box (5) Box P 98. Which of the following boxes is of green colour? (1) Box M (2) Box P (3) Box R (4) Box O (5) Box Q 99. What is the position of Box R in the given stack of boxes? (1) Fourth from the bottom (2) Third from the top (3) Second from the top (4) Third from the bottom (5) Fifth from the top 100. Four of the following five are alike in a certain way and hence form a group. Which of the following does not belong to the group?

    Seven persons namely C, D, E, F, G, H and I like different cities namely, Surat, Kolkata, Bangalore, Mumbai, Ranchi, Delhi and Pune. Each of them studies in either of three schools viz. DAS, RIS and VCS with atleast two of them in a school. (Note : None of the information given is necessarily in the same order.) F studies with the one who likes Bangalore in RIS. The one who likes Delhi studies only with H. H does not like Bangalore. C studies with those who like Surat and Pune. C does not study with F. E studies only with the one who likes Mumbai. The one who likes Mumbai does not study with the one who likes Delhi. More than one person studies with D. D does not like Pune. Both I and the one who likes Ranchi study in the same school but not in DAS. H does not like Ranchi. 101. Which of the following combinations represents the school in which E studies and city he likes? (1) DAS – Bangalore (2) RIS – Bangalore (3) DAS – Kolkata

    BPRE–707

    Exam, 03.07.2016 (Ist Shift))

    (4) VCS – Delhi (5) RIS – Delhi 102. Which of the following cities does G like? (1) Pune (2) Ranchi (3) Mumbai

    (4) Delhi

    (5) Surat 103. Four of the following five are alike in a certain way as per the given arrangement and hence form a group. Which of the following does not belong to that group? (1) DF (2) CI (3) HG (4) FE (5) ID 104. Who amongst the following likes Kolkata? (1) F (2) I (3) H (4) C (5) D

    PROBLEM SOLVING 105. Which of the following combinations represents the group of persons studying in VCS? (1) The ones who like Delhi and Bangalore (2) The ones who like Ranchi and Mumbai (3) The ones who like Delhi, Mumbai and Kolkata (4) The ones who like Pune, Surat and Kolkata (5) The ones who like Surat, Pune and Ranchi 106. Which of the following statements in not true? (1) F studies in the same school in which D studies (2) Both D and C study in the same school (3) G studies with only the one who likes Kolkata (4) C likes Ranchi (5) All of the given statements are true Directions (107–112) : Study the following information carefully and answer the questions given below : (SBI PO Phase-I (Pre.) Online Exam, 03.07.2016 (IInd Shift))

    Seven people, namely, J, K, L, M, N, O and P like seven different cartoon characters namely, Tweety, Wiene, Superman, Garfield, Jerry, Ariel and Popeye. Each of them studies in either of the three standards viz. IV, VIII, XI with atleast two of them in a standard. (Note : None of the information given is necessarily in the same order.) Only one person studies with M in XI standard. K studies with the one who likes Ariel. M does not like Ariel. P studies with only the one who likes Garfield. Neither K nor M likes Garfield. J likes Popeye. J does not study with K. The one who likes Tweety studies with J. N studies with the one who likes Jerry. N does not study in IV standard. Neither K nor O likes Jerry. The one who likes Superman studies with N. 107. Who amongst the following likes Wiene? (1) N (2) O (3) M (4) P (5) K 108. Which of the following cartoon characters does L like? (1) Superman (2) Ariel (3) Tweety (4) Jerry (5) Garfield

    109. Which of the following statements is true? (1) Both M and J study in the same standard (2) K studies in IV standard (3) J studies with the one who likes Wiene (4) More than two people study in the standard in which O studies (5) None of the given statements is true 110. Which of the following combinations represents the combination of people studying in VIII standard? (1) N and the one who likes Wiene (2) J, O (3) K, L, N (4) K and the one who likes Tweety (5) Other than those given as options 111. Four of the following five are alike in a certain way based on the given arrangement and hence form a group. Which of the following does not belong to that group? (1) LK (2) PM (3) KN (4) LN (5) OP 112. Which of the following combinations represents the standard in which O studies and the cartoon character he likes? (1) VIII–Garfield (2) VIII–Superman (3) XI–Tweety (4) IV–Ariel (5) IV–Garfield Directions (113–117) : Study the following information carefully and answer the questions given below : (SBI PO Phase-I (Pre.) Online Exam, 03.07.2016 (IInd Shift))

    Seven people, namely P, Q, R, S, T, U and V have a meeting but not necessarily in the same order on seven different months (of the same year) namely January, February, April, May, July, August and September. Each of them also likes a different fitness activity namely Yoga, Gym Training, Zumba, Mountaineering, Bicycle, Running and Surfing but not necessarily in the same order. The one who likes Gym Training has a meeting on one of the months before May. The one who likes Gym Training has a meeting in the month which has less than 31 days. Only

    BPRE–708

    three people have a meeting between the one who likes Gym training and T. Only two people have a meeting between T and the one who likes running. The one who likes Mountaineering has a meeting on one of the months after the one who likes Running. The one who likes Mountaineering has a meeting in the month which has only 30 days. V has a meeting on one of the months before the one who likes Gym Training. V does not have a meeting in the month which has least number of days. More than three people have a meeting between V and R. Only two people have a meeting between R and Q. The one who likes Yoga has a meeting immediately before U. U does not have a meeting in a month which has 31 days. Only one person has a meeting between the one who likes Surfing and S. S has a meeting after the one who likes Surfing. R does not like Zumba. 113. In the given arrangement Yoga is related to February and Gym Training is related to May following a certain pattern, with which of the following is Zumba related to following the same pattern? (1) July (2) September (3) April (4) August (5) January 114. Who amongst the following has a meeting immediately before the one who has a meeting in July? (1) The one who likes Zumba (2) The one who likes Surfing (3) S (4) R (5) Q 115. Which of the following combinations is correct? (1) S–Zumba (2) V–Yoga (3) R–Running (4) T–Bicycling (5) U–Gym Training 116. Who amongst the following likes Surfing? (1) U (2) Q (3) R (4) T (5) Other than those given as options 117. How many people have a meeting between the one who has a meeting in February and R? (1) More than three (2) None (3) One (4) Two (5) Three

    PROBLEM SOLVING 118. In which of the following pairs, both the persons have a meeting in the months which has less than 31 days? (1) R, S (2) R, Q (3) U, Q (4) T, P (5) S , U Directions (119–124) : Study the following information carefully and answer the questions given below : (SBI PO Online (Preliminary) Exam, 10.07.2016)

    Seven persons, namely A, B, C, D, E, F and G have an appointment but not necessarily in the same order, on seven different months (of the same year) namely January, February, April, June, August, October and December. Each of them also likes a different activity namely Drawing, Singing, Painting, Boxing, Karate, Craft and Running but not necessarily in the same order. The one who likes Craft has an appointment on one of the months before April. Only two persons have an appointment between the one who likes craft and the one who likes painting. Only one person has an appointment .between the one who likes painting and the one who likes running. The one who likes running has an appointment in a month which has 31 days. Only three persons have an appointment between the one who likes running and E. G has an appointment on one of the months before E. G does not have an appointment in the month which has the least number of days. Only three persons have an appointment between G and C. Only one person has an appointment between C and the one who likes Karate. The one who likes Karate has an appointment before C. The one who likes singing has an appointment immediately before B. B has an appointment in a month which has less than 31 days. Only one person has an appointment between A and F. A has an appointment before F. Only one person has an appointment between F and the one who likes drawing. 119. Who amongst the following has an appointment immediately before the one who has an appointment in December? (1) B (2) E (3) C (4) The one who likes Running (5) The one who likes Boxing

    120. In which of the following pairs, both the persons have an appointment in months which have less than 31 days? (1) A, F (2) F, D (3) B, A (4) E, D (5) B, F 121. Which of the following combinations is correct? (1) G-Singing (2) A-Painting (3) F-Running (4) B–Karate (5) D-Painting 122. As per the given arrangement Craft is related to April and Karate is related to June following a certain pattern, which of the following is Drawing related to following the same pattern? (1) February (2) October (3) December (4) August (5) January 123. Who amongst the following likes Running? (1) F (2) D (3) A (4) B (5) Other than those given as options 124. How many persons have an appointment between the one who has an appointment in February and A? (1) Three (2) None (3) More than three (4) One (5) Two Directions (125–135) : Study the following information carefully and answer the questions given below : (SBI PO Online (Preliminary) Exam, 10.07.2016)

    Seven persons namely A, B, C, D, E, F and G teach seven different subjects namely, Mathematics, English, Chemistry, History, Accountancy, Physics and Biology. Each of them works in either of the three institutes viz. Paramount, Brilliant and Embibe with atleast two of them in a institute. (Note : None of the information given is necessarily in the same order.) G works in Brilliant with the one who teaches Accountancy. The one who teaches Biology works in an institute with only the one who teaches Chemistry. B teaches English. B does not work with G. D works with F but not in Embibe. Neither D nor F teaches Accountancy. F does not teach Chemistry. C works with only the one who teaches Mathematics. E works with the one who teaches History.

    BPRE–709

    125. Who amongst the following teaches Physics? (1) F (2) E (3) C (4) D (5) G 126. Which of the following combinations represents the institute in which C works and the subject he teaches? (1) Brilliant–Accountancy (2) Embibe–Accountancy (3) Paramount–Biology (4) Brilliant–History (5) Paramount–Chemistry 127. Which of the following subjects does A teach? (1) Biology (2)Chemistry (3) Mathematics (4) Accountancy (5) History 128. Which of the following statements is true? (1) D teaches Biology (2) None of the given statements is true (3) Only two persons work in Brilliant (4) Both E and G work in the same institute (5) A teaches Mathematics. 129. Which of the following combinations represents the combination of persons working in Paramount? (1) D and the one who teaches English (2) E, A (3) G, C, E (4) F and the one who teaches Chemistry (5) A, B 130. Four of the following five are alike in a certain way based on the given arrangement and hence form a group. Which of the following does not belong to that group? (1) BE (2) GC (3) EA (4) FG (5) AB Directions (131–135) : Study the following information carefully and answer the questions given below : (SBI PO Online Main Exam, 31.07.2016)

    Eight employees – P, Q, T, B, R, J, N and A – work in three different departments, viz., Operation, Personnel and Accounts of an organisation in such a way that at least two of them but not more than three work in the

    PROBLEM SOLVING same department. Each of them likes different cities viz., Bengaluru, Gwalior, Yelahanka, Raipur, Ooty, Warangal, Jodhpur and Palampur, but not necessarily in the same order. Q likes Bengaluru and he works in the Personnel department only with J who likes Palampur. P works in the Operation department and he does not like Gwalior or Ooty. T likes Jodhpur and he works with N who likes Raipur. The ones who work in the Operation department do not like Warangal. A does not work in the Operation department and B does not work in the Accounts department. The ones who like Warangal and Raipur respectively work in the same department. R does not work in the Accounts department and he likes Gwalior. 131. Which of the following combinations of employees does work in the Operation department? (1) Q, the one who likes Ooty and R (2) The one who likes Yelahanka, Q and the one who likes Gwalior (3) P, the ones who like Jodhpur and Ooty respectively (4) The ones who like Yelahanka, Ooty and Gwalior respectively (5) B, the ones who like Ooty and Warangal respectively 132. Which of the following combinations of favourite city and the department is correct? (1) Jodhpur – Accounts (2) Palampur – Personnel (3) Warangal – Accounts (4) Bengaluru – Operation (5) Yelahanka – Accounts 133. As per the given arrangement Q is related to personnel in a certain way and N is related to Accounts in the same way. Who among the following is related to Operation in the same way? (1) A (2) T (3) R (4) J (5) Other than those given as options 134.Who among the following does like Yelahanka? (1) N (2) J (3) T (4) B (5) P 135. Which of the following statements

    is true as per the given arrangement? (1) P works in the Operation department and he likes Jodhpur. (2) J likes Palampur and works in the Personnel department. (3) A works in the Personnel department and he likes Warangal. (4) B works in the Account department and he likes Ooty. (5) N works in the Personnel department and he likes Yelahanka. Directions (136–140) : Study the following information carefully and answer the questions given below : (SBI PO Online Main Exam, 31.07.2016)

    Seven women A, B, C, D, E, F and G (not necessarily in same order) have formed a Self Help Group (SHG) according to their professions (not necessarily in the same order) namely Stitching, Knitting, Colouring, Grinding, Marketing, Gardening and Teaching and according to their experiences they hold different positions namely, Trainee (TE), Assistant Manager (AM), Manager (MG), Senior Manager (SM), Chief Manager (CM), Executive Director (ED) and Director (DR), as per increasing order of experience, where Trainee is junior most position and DR is the senior most position. G is from teaching profession. The woman who is from colouring is not the most experienced. The woman from marketing is more experienced than E but less experienced than Executive Director. D is not from marketing. Only one woman is more experienced than C, and she is from Gardening. A is just less experienced than F and she is from knitting profession. A is not the least experienced. E is just less experienced than Chief Manager but more experienced than woman from grinding. C is not from stitching. 136. Which of the following positions does G hold? (1) Trainee (TE) (2) Manager (MG) (3) Senior Manager (SM) (4) Chief Manager (CM) (5) Director (DR) 137. Which of the following combinations represents the profession and position of B? (1) Grinding – Manager (MG)

    BPRE–710

    (2) Teaching – Senior Manager (SM) (3) Colouring – Manager (MG) (4) Stitching – Executive Director (ED) (5) Marketing – Chief Manager (CM) 138. As per the given arrangement ‘E’ is related to ‘Teaching’ and in the same way ‘F’ is related to ‘colouring’. Following the same pattern which of the following is related to D? (1) Gardening (2) Knitting (3) Grinding (4) Stitching (5) Marketing 139. Who amongst the following does hold the position of Manager (MG)? (1) D (2) G (3) F (4) E (5) Other than those given as options 140. Which of the following combinations represents the respective women who have less experience than F and more experience than B? (1) E, D (2) A, C (3) G, E (4) A, E (5) G, A Directions (141–145) : Study the following information carefully and answer the questions given below : (SBI PO Online (Preliminary) Online Exam. 30.04.2017)

    Eight persons — C, D, E, F, W, X, Y and Z — have to attend a wedding in January, April, September and December months of the same year. In each month the wedding is on either the 11th or the 24th of the month. Not more than two of the given people have to attend a wedding in the same month. W has to attend a wedding on the 11th of the month which has only 30 days. Only three people have to attend a wedding between W and Y. C and Y have to attend a wedding neither on the same date nor in the same month. C does not have to attend a wedding in April. Only two people have to attend a wedding between C and F. X and F does not have to attend a wedding on the same date. D has to attend a wedding on one of the days before X. Only one person has to attend a wedding between D and E. Less than four people have to attend a wedding between E and Z.

    PROBLEM SOLVING 141. How many people have to attend a wedding between F and Z? (1) Two (2) Three (3) None (4) More than three (5) One 142. When does X have to attend a wedding? (1) 24th April (2) Cannot be determined (3) 11th January (4) 24th September (5) 11th December 143. If all the people are made to attend the wedding in alphabetical order starting from 11th January and ending on 24th December, the schedule of how many people will remain unchanged? (1) Three (2) Two (3) Five (4) None (5) One 144. Who among the following has to attend a wedding before Y? (1) Both C and X (2) Only W (3) None (4) Both F and W (5) Only F 145. As per the given arrangement, four of the following five are alike in a certain way and so they form a group. Which of the following does not belong to the group? (1) W (2) Z (3) F (4) Y (5) X Directions (146–150) : Study the following information carefully and answer the questions given below : (SBI PO Online (Preliminary) Online Exam. 30.04.2017)

    Eight people viz. X, G, T, C, P, J, A and M live on different floors of a building. The ground floor of the building is numbered one, the one above that is numbered two and so on till the topmost floor is numbered eight. All of them can perform a different form of dance viz., Kathak, Garba, Dandiya, Bhangra, Lavani, Odissi, Mohiniattam and Sattriya. (Note : None of the given information is necessarily in the same order) T lives on an even numbered floor below floor number five. Only three people live between T and the one who performs Garba. As many people live

    below T as above the one who performs Lavani. Number of people living between the one who performs Garba and Lavani is equal to the number of people living between C and P. C lives on an even numbered floor above P. Neither C nor P performs Garba or Lavani. The one who performs Kathak lives on an odd numbered floor below floor number four. P does not perform Kathak. The number of people between T and the one who performs Kathak is same as the number of people living between X and the one who performs Lavani. X lives on one of the floors above the one who performs Lavani. Number of people living between C and X is equal to the number of people living between T and M. The one who performs Odissi lives on an even numbered floor immediately above the one who performs Bhangra. Only three people live between G and J. G lives on one of the floors above J. The one who performs Bhangra lives immediately above the one who performs Sattriya. C neither likes Odissi nor Dandiya. 146. Which of the following statements is true as per the given arrangement? (1) C performs Odissi. (2) The one who performs Garba lives on floor number six. (3) T lives immediately above J. (4) None of the given statements is true. (5) Only two people live between M and G. 147. Four of the following five are alike in a certain way based on the given arrangement and thus they form a group. Which one of the following does not belong to the group? (1) M–Bhangra (2) C–Dandiya (3) P–Odissi (4) G–Floor number eight (5) T–Floor number five 148. How many people live between the one who performs Lavani and Bhangra? (1) More than three (2) Three (3) One (4) Two (5) None 149. Who performs Dandiya? (1) G (2) A (3) P (4) T (5) C

    BPRE–711

    150. Which dance M performs? (1) Sattriya (2) Lavani (3) Odissi (4) Mohiniattam (5) Kathak Directions (151–155) : Study the following information carefully and answer the questions given below : (SBI PO Online (Preliminary) Online Exam. 06.05.2017)

    Eight persons – A, B, C, D, P, Q, R and S – have a party in either April or December of the same year which will be organised only on the four days viz., 4th, 17th, 20th and 29th of each month. No two people have a party on the same day. A has a party in December on an odd numbered date. Only three people have to attend a party between A and C. The number of people having a party after C is same as those having a party before B. Only one person has a party between P and B. P does not have to attend a party in April. Both S and R have to attend a party on the same date. S has to party on one of the days after R. Only two people have to attend a party between S and Q. 151. How many people have to attend a party between Q and D? (1) None (2) One (3) Three (4) Two (5) More than three 152. When does Q have to attend a party ? (1) 4th April (2) 29th December (3) 17th December (4) Cannot be determined (5) 20th April 153. Who among the following have to attend a party on 4th April and 4th December ? (1) B, C (2) R, S (3) P, B (4) B, D (5) P, C 154. Which of the following statements is true with respect to the given information? (I) S has to attend a party on 4th December. (II) Only two people have to attend a party between A and Q. (III) C has to attend a party on one of the days after R. (1) None of the given statements is true (2) Both (I) and (II) (3) Only (III) (4) Only (I) (5) Both (II) and (III)

    PROBLEM SOLVING 155. In the given arrangement, if Q and A interchange their positions and so do C and P, then how many people will have to attend a party between A and C? (1) Three (2) Two (3) One (4) More than three (5) None Directions (156–160) : Study the following information carefully and answer the questions given below : (SBI PO Online (Preliminary) Online Exam. 06.05.2017)

    Eight people viz. Z, V, N, E, L, Q, H and U live on eight different floors of a building. The ground floor of the building is numbered one, the one above that is numbered two and so on till the topmost floor is numbered eight. All of them play a different video games viz., Batman, Darkstone, Gothic, Magicka, Jurassic Park, Rayman, Payday and Victoria. (Note : None of the given information is necessarily in the same order.) E lives on floor number two. No one lives between E and the one who plays Batman. Only four people live between U and the one who plays Batman. Only one person lives between U and the one who plays Magicka. The one who plays Magicka does not live on the topmost floor. As many people live between the one who plays Batman and Magicka as between L and Q. L lives on an odd numbered floor above Q. As many people live between the one who plays Batman and Q as between the one who plays Darkstone and Q. The one who plays Payday lives on an even numbered floor immediately above the one who plays Victoria but on floor above two. Only two people live between H and Z. H lives on one of the floors above Z. The one who plays Rayman lives immediately below the one who plays Gothic. Only one person lives between V and N. V lives on one of the floors above N. 156. Four of the following five are alike in a certain way based on the given arrangement and thus they form a group. Which one of the following does not belong to the group? (1) V–Floor Number Six (2) Q–Victoria (3) Z–Magicka (4) U–Floor Number Five (5) H–Darkstone

    157. Who plays Jurassic Park ? (1) E (2) Q (3) L (4) H (5) N 158. Which of the following statements is true as per the given arrangement ? (1) Z lives on floor number five. (2) Q plays Magicka. (3) Only one person lives between U and H. (4) V lives immediately above E. (5) All the given statements are true 159. Which game does U play ? (1) Victoria (2) Payday (3) Rayman (4) Gothic (5) Batman 160. How many people live between the one who plays Gothic and U ? (1) Three (2) One (3) Two (4) Four (5) None Directions (161–165) : Study the following information carefully and answer the questions given below : (SBI PO Online (Preliminary) Exam, 07.05.2017)

    In an apartment, eight persons, i.e., D, E, F, G, H, I, J and K live on different floors but not necessarily in the same order. The lowermost floor of the building is numbered 1 and the topmost floor of the building is numbered 8. They are from different stream of Engineering i.e., Chemical Engineering Instrumentation Engineering, Software Engineering, Aeronautical Engineering, Mechanical Engineering, Electrical Engineering, Automobile Engineering, and Civil Engineering. The one who lives on fourth floor is specialized in Mechanical Engineering. D lives on odd numbered floor but above 3rd floor. The number of persons between D and the one who is specialized in Electrical Engineering is same as number of persons between D and I. The one who is specialized in Instrumentation Engineering lives on lowermost floor. K lives on an even numbered floor and he is specialized in Automobile Engineering. There are two floors between E and H and E lives above to H. E is specialized in Aeronautical Engineering. J lives just above the one who is specialized in Aeronautical Engineering. The number of floors between the one who is specialized in Aeronautical Engineering and Mechanical Engineering is two. The one who

    BPRE–712

    is specialized in Civil Engineering lives on odd numbered floor. The number of floors between the one who is specialized in Chemical Engineering and J is four. The one who is specialized in Aeronautical Engineering lives on an odd numbered floor. The number of floors between the one who is specialized in Civil Engineering and the floor on which F lives is same as the number of floors between F and G. I lives below the floor on which D lives. 161. How many persons live between the person who is specialized in Chemical Engineering and the one who is specialized in Electrical Engineering? (1) Six (2) One (3) Four (4) Two (5) None of these 162. J is specialized in which of the following stream of engineering? (1) Aeronautical Engineering (2) Electrical Engineering (3) Civil Engineering (4) Instrumentation Engineering (5) Mechanical Engineering 163. Four of the following five are alike in a certain way and hence they form a group. Which one of the following does not belong to that group? (1) G (2) K (3) H (4) I (5) J 164. D lives on which floor? (1) 1 (2) 3 (3) 4 (4) 5 (5) 7 165. G is related to Mechanical Engineering, in the same way as F is related to Automobile Engineering. Then, which of the following is H related to? (Following the same pattern) (1) Mechanical Engineering (2) Electrical Engineering (3) Civil Engineering (4) Instrumentation Engineering (5) Aeronautical Engineering Directions (166–167) : Study the following information carefully and answer the questions given below : (SBI PO Online (Preliminary) Exam, 07.05.2017)

    There are four boxes i.e. J, K, L and M in which four types of fruits are stored. Fruits are Litchi, Apple, Grapes and Mango. Boxes are arranged in a manner from top to bottom.

    PROBLEM SOLVING There are two boxes between K and L. The box in which Grapes are stored is above L, but not immediately above. The box in which Apple is stored is immediately below M, but not stored in box L. Litchi box is above the Mango box, but not immediately above Apple box. 166. In which of the following boxes, Litchi is stored? (1) J (2) M (3) K (4) L (5) Either J or M 167. Which of the following fruits is stored in second lowest Box? (1) Grapes (2) Apple (3) Mango (4) Litchi (5) Cannot be determined Directions (168–172) : Study the following information carefully and answer the questions given below : (SBI PO Phase-I (Pre.) Online Exam, 03.07.2016 (Ist Shift))

    Seven boxes – J, K, L, M, N, O and P – are kept one above the other, but not necessarily in the same order. Each box contains differernt elements – Cookies, Pencils, Spoons, Diaries, Colours, Jewellery and Watches, but not necessarily in the same order. Only two boxes are kept between M and N. The pencil box is kept immediately below M. Only two boxes are kept between the Pencil box and the Watch box. N is kept above the Watch box. The Diary box is kept immediately below the Watch box. Only three boxes are kept between the Diary box and J. The Jewellery box is kept immediately above the J. O is kept immediately above K. O is not a pencil box. P is kept immediately above the Cookies box. Only one box is kept between the P and the Spoon box. 168. Which of the following boxes is kept immediately above M? (1) P (2) O (3) L (4) The Diary box (5) The Jewellery box 169. What is the position of O in the given stack of boxes? (1) First from the top (2) Second from the top (3) Third from the bottom (4) Fifth from the bottom (5) Fourth from the top

    170. Which of the following boxes contains Spoons? (1) Other than those given as options (2) K (3) M (4) N (5) L 171. Four of the following five are alike in a certain way and hence form a group. Which of the following does not belong to the group? (1) L – Pencils (2) P – Spoons (3) K – Colours (4) N – Diaries (5) O – Spoons 172. How many boxes are kept between K and the Watch box? (1) More than three (2) Two

    (3) Three

    (4) One (5) None Directions (173–177) : Study the following information carefully and answer the questions given below : (SBI PO Phase-I (Pre.) Online Exam, 03.07.2016 (IInd Shift))

    Seven boxes – M, N, O, P, Q, R and S are kept one above the other, but not necessarily in the same order. Each box is of a different shape. Square, Triangular, Rectangular, Hexagonal, Conical, Pentagonal and Octagonal, but not necessarily in the same order. The hexagonal box is kept immediately below M. Only two boxes are kept between the Hexagonal box and the square box. More than two boxes are kept above the square box. Only one box is kept between the square box and R. O is kept immediately above R. Neither O nor S is a hexagonal box. S is kept immediately above the Pentagonal box. S is not a square box. N is kept immediately above Q. Only one box is kept between the N and the triangular box. N is kept above the triangular box. Only two boxes are kept between the Triangular and the Octagonal box. The Rectangular box is kept neither at the top nor at the bottom of the stack. 173. Which of the following is kept immediately below R? (1) S (2) The Triangular Box (3) P (4) N (5) The Conical Box 174. Which of the following boxes is rectangular in shape? (1) R (2) N (3) M (4) O (5) Other than those given as options

    BPRE–713

    175. Four of the following five are alike in a certain way and hence form a group. Which of the following does not belong to that group? (1) M–Triangular (2) M–Hexagonal (3) O–Octagonal (4) S–Square (5) S–Conical 176. What is the position of S in the given stack of boxes? (1) Second from the top (2) Fourth from the top (3) Third from the bottom (4) Third from the top (5) Second from the bottom 177. How many boxes are kept between the P and the Square box? (1) Two (2) One (3) None (4) Three (5) More than three Directions (178–182) : Study the following information carefully and answer the questions given below : (SBI PO Online (Preliminary) Exam, 10.07.2016)

    Seven boxes – A, B, C, D, E, F and G are kept one above the other, but not necessarily in the same order. Each box contains different items- Shoes, Papers, Bands, Medicines, Ribbons, Creams and Phones, but not necessarily in the same order. Only three boxes are kept between D and G, The Ribbon box is kept immediately above G. Only one box is kept between the Ribbon box and A. The Ribbon box is not the second from the bottom of the stack. Only one box is kept between E and A. E is kept above A. The Medicine box is kept immediately above E. Only three boxes are kept between the Medicine box and the Shoe box. The Paper Box is kept immediately above the Phone box. G is not the Paper box. F is kept immediately below the Cream box. Only one box is kept between B and the Cream box. 178. Four of the following five are alike in a certain way and hence form a group. Which of the following does not belong to the group? (1) G–Creams (2) E–Bands (3) C–Shoes (4) D–Papers (5) A–Bands 179. Which of the following boxes contains bands? (1) D (2) C (3) A (4) Other than those given as options (5) G

    PROBLEM SOLVING 180. What is the position of F in the given stack of boxes? (1) Second from the top (2) Third from the bottom (3) First from the top (4) Fifth from the bottom (5) Fourth from the top 181. Which of the following boxes is kept immediately below G? (1) A (2) B (3) C (4) The Shoe box (5) The Paper Box 182. How many boxes are kept between B and the Ribbon box? (1) Two (2) One (3) None (4) Three (5) More than three Directions (183 – 185) : Study the following information carefully and answer the questions given below : (SBI Assistant Manager (System) Exam, 17.01.2016)

    Each of the six people A, B, C, D, E and F buys different number of shirts. C buys more shirts than only F and D. A buys more number of shirts than F but less than E. E does not buy the maximum number of shirts. D does not buy the least number of shirts. The one who has the maximum number of shirts buys 15 shirts. The one who buys the second least number of shirts buys 4 shirts. A buys five more shirts than D. 183. Which of the following may represent the number of shirts bought by C ? (1) 7 (2) 3 (3) 9 (4) 11 (5) 13 184. If the number of shirts bought by E is a number which is divisible by 3, then how many shirts does E buy ? (1) 3 (2) 9 (3) 6 (4) 15 (5) 12 185. Who amongst the following buys the second highest number of shirts ? (1) A (2) E (3) C (4) D (5) B Directions (186–190) : Study the following information carefully and answer the questions given below : (SBI PO Online (Pre.) Exam, 02.07.2016 (Ist Shift))

    T, U, V, W, X, Y and Z are seven different bowls of different fruits i.e. Apple, Banana, Orange, Litchi, Pineapple, Grapes and Cherry but not necessarily in the same order. Bowl which is of Apple fruit is immediately above T. There are only two bowls between W and the bowl which is of Apple fruit. Bowl which is of Orange fruit is above W but not immediately above W. Only three bowls are between V and the bowl which is of Orange fruit. The bowl which is of Cherry fruit is immediately above V. The bowl which is of Litchi fruit is immediately above the bowl Z. Only one bowl is there between U and X. Bowl U is above X. Neither bowl U nor T is of Pineapple fruit. T is not of Banana fruit. 186. How many bowls is/are there between W and Z? (1) One (2) Two (3) Three (4) Four (5) None 187. What is the fruit of ‘Y’? (1) Cherry (2) Apple (3) Litchi (4) Orange (5) Cann’t be determined 188. Find the pair of fruit and bowls which is not correct? (1) U–Litchi (2) Y–Orange (3) T–Grapes (4) Z–Apple (5) None of these 189. Which of the following condition is correct regarding Pineapple fruit with respect to X? (1) There is one bowl between X and Pineapple fruit (2) X is immediately above of Pineapple fruit (3) Pineapple fruit is related to the bowl immediately above X (4) All of the above is true (5) None of the above is true 190. Which of the following fruit belongs to ‘T’? (1) Apple (2) Litchi (3) Pineapple (4) Banana (5) None of these Directions (191–196) : Study the following information carefully and answer the questions given below : (SBI PO Online (Pre.) Exam, 02.07.2016 (IInd Shift))

    Six friends, A, B, C, D, E and F borrowed different number of books from their libraries. D borrowed more number of books than both C and F. D

    BPRE–714

    borrowed less number of books than A. A did not borrowed the highest number of books. C borrowed neither the lowest nor the second lowest number of books. B borrowed neither the lowest nor the highest number of books. The one who borrowed the second highest number of books, borrowed 34 books. The one who borrowed second lowest number of books borrowed 19 books. 191. Who amongst the following did borrow the highest number of books? (1) D (2) E (3) C (4) F (5) None of these 192. What number of books did F possibly borrow? (1) 36 (2) 27 (3) 23 (4) 21 (5) 14 193. Who amongst the following did borrow more number of books than B but less number of books than D? (1) C (2) F (3) A (4) E (5) None of these 194. What number of books did A definitely borrow? (1) 36 (2) 28 (3) 34 (4) 30 (5) 21 195. Who amongst the following did borrow more number of books than only F? (1) E (2) F (3) C (4) B (5) None of these 196. If C borrowed 26 books then D might have borrowed what number of books? (1) 38 (2) 24 (3) 30 (4) 22 (5) 36 Directions (197–205) : Study the following information carefully and answer the questions given below : (SBI PO Online (Pre.) Exam, 02.07.2016 (IInd Shift))

    Seven persons namely, A, B, C, D, E, F and G like seven different fruits viz., Apple, Banana, Guava, Orange, Litchi, Mango and Watermelon, but not necessarily in the same order. Each of them works in either of the three departments of the same company viz., Marketing, Production and Research & Development, with atleast two of them in a department.

    PROBLEM SOLVING B works in the Research & Development department with the one who likes Banana. E works with only one who likes Litchi. E does not like Banana. F works with only G. F does not work with the one who likes Litchi. The one who likes Apple works with the only one who likes Guava. G does not like Guava. A likes orange. The one who likes Mango does not work with the one who likes Guava. C does not work with the one who likes Litchi. D does not work in the Marketing department. The one who likes Watermelon does not work in the Production department. 197. Which of the following represents the combination of that persons who work in the Research & Development department? (1) The ones who like Banana, Apple and Guava (2) The ones who like Litchi, Banana and Watermelon (3) The ones who like Banana, Watermelon and Orange (4) B and the ones who like Banana and Mango (5) The ones who like Mango, Banana and Guava 198. Which of the following statements is not true as per the given arrangement? (1) D works in the Research & Development department and he likes Banana (2) G likes Apple and works in the Marketing department (3) Only two persons work in the Production department (4) C works in the Research & Development department while F works in the Marketing department (5) A works with the ones who like Watermelon and Banana. 199. Which of the following combinations represents the department in which F works and the fruit he likes? (1) Production, Guava (2) Production, Apple (3) Research & Development, Watermelon (4) Marketing, Watermelon (5) Marketing, Guava 200. Four of the following five are alike in certain way as per the given arrangement and hence form a group. Which of the following does not belong to that group?

    (1) A, B (2) D, E (3) F, G (4) C, E (5) B, C 201. Who amongst the following likes Banana? (1) F (2) C (3) G (4) D (5) None of these Directions (202–206) : Study the following information carefully and answer the questions given below : (SBI PO Phase-I (Pre.) Online Exam, 03.07.2016 (Ist Shift))

    Seven persons namely K, L, M, N, O, P and Q have to attend a concert but not necessarily in the same order, in seven different months (of the same year) namely January, February, April, May, June, September and November. Each of them also likes a different movie namely X–Men, Transformers, Frozen, Minions, Shrek, Tangled and Rio but not necessarily in the same order. M will attend a concert in a month which has 31 days. Only two persons will attend a concert between M and the one who likes Frozen. The one who likes Frozen will attend a concert on one of the months after M. Only two persons will attend a concert between the one who likes Frozen and the one who likes Transformers. The one who likes Trasnformers will attend a concert in a month which has 31 days. K will attend a concert immediately after M. Only three persons will attend a concert between K and the one who likes Tangled. Only one person will attend a concert between the one who likes Tangled and L. Only two persons will attend a concert between L and the one who likes Rio. The one who likes X–Men will attend a concert immediately before the one who likes Shrek. Only one person will attend a concert between the one who likes Shrek and P. Only three persons will attend a concert between Q and O. Q will not attend a concert in a month which has 30 days. 202. Who amongst the following likes X–Men? (1) M (2) K (3) O (4) Q (5) L 203. How many persons will attend a concert after M? (1) More than three (2) One (3) None (4) Two (5) Three

    BPRE–715

    204. Which of the following represents the persons who will attend a concert immediatly before and immediately after O? (1) K, N (2) P, L (3) L, N (4) P, N (5) N, M 205. As per the given arrangement January is related to Q and May is related to K following a certain pattern, which of the following is April related to following the same pattern? (1) P (2) O (3) M (4) L (5) N 206. Which of the following represents the month in which K will attend a concert? (1) September (2) June (3) November (4) April (5) Cannot be determined 207. Who amongst the following likes Minions? (1) M (2) L (3) K (4) Other than those given as options (5) P Directions (208–210) : Study the following informations carefully and answer the questions given below : (SBI PO Phase-I (Pre.) Online Exam, 03.07.2016 (Ist Shift))

    Each of the six friends, I, J, K, L, M and N working in an office handles different number of projects in a month. I handled the second lowest number of projects. K handles more projects than L and M but less than J. J did not handle the maximum number of projects. M did not handle the minimum number of projects. The one who handles the third highest number of projects handled 31 projects. L handled 12 projects. 208. How many projects did J possibly handle? (1) 28 (2) 10 (3) 36 (4) 9 (5) 15 209. If the number of projects handled by I + L is only three more than number of projects handled by K, how many projects were handled by I? (1) 32 (2) 14 (3) 19 (4) 9 (5) 22

    PROBLEM SOLVING 210. Which of the following statements is statements true regarding the number of projects handled by N? (1) No one handles more projects than N. (2) Only J handled more number of projects than N (3) N possibly handled 24 projects (4) N handled more number of projects than only three persons (5) The difference between projects handled by N and L is less than 18. Directions (211–213) : Study the following information carefully and answer the questions given below : (SBI PO Phase-I (Pre.) Online Exam, 03.07.2016 (IInd Shift))

    Each of the six translators, L, M, N, O, P and Q translates different number of books in a year. O translated more books than only two people and M is one of them. N translated more books than P but less than L. L did not translate the highest number of books. P translated more books than M. The one who translated second highest number of books translated 32 books. P translated 15 books. 211. How many books has N possibly translated? (1) 33 (2) 36 (3) 23 (4) 12 (5) 14 212. Which of the following statements is true regarding the number of books translated by Q? (1) No other translator has translated less books than Q. (2) Q has translated less number of books than N. (3) Q translated less books than only two persons. (4) Q possibly translated 45 books. (5) The difference between number of books translated by P and Q is less than 17. 213. How many books has M possibly translated? (1) 30 (2) 10 (3) 24 (4) 36 (5) 32 Directions (214–216) : Study the following information carefully and answer the questions given below : (SBI PO Online (Preliminary) Exam, 10.07.2016)

    Each of the six sections, U, V, W, X, Y and Z of the same class has different number of students. Only Z has more number of students than X. V has more number of students than Y but less than U. W has more number of students than both the both Y and U. The section having the third highest number of students has 39 students. Y has 24 students. 214. If the number of students in sections W + Z is sixty–six more than the number of students in section Y, how many of students are there in section Z? (1) 31 (2) 46 (3) 51 (4) 55 (5) 45 215. How many students does section V possibly have? (1) 39 (2) 43 (3) 55 (4) 31 (5) 14 216. Which of the following statements is true regarding the number of students in section U? (1) No other section has less students than U. (2) X has more number of students than U. (3) U possibly has 45 students. (4) U has more number of students than only three sections. (5) None of the given statements is true Directions (217–219) : Study the following information carefully and answer the questions given below : (SBI PO Online (Preliminary) Online Exam. 30.04.2017)

    Each of the six stores P, Q, R, S, T and U sold different number of books in one day. Only three stores sold less books than U. P sold more books than R. T did not sell the highest number of books. S sold more books than R and P but less than U. The store which sold the second highest number of books sold 72 books. 217. How many books did Q probably sell? (1) 43 (2) 58 (3) 71 (4) 65 (5) 89

    BPRE–716

    218. Which of the following stores sold the second lowest number of books? (1) T (2) P (3) S (4) R (5) Q 219. If the total number of books sold by P and T is 125, then how many books did P sell? (1) 51 (2) 76 (3) 53 (4) 45 (5) 68 Directions (220–224) : Study the following information carefully and answer the questions given below : (SBI PO Online (Preliminary) Online Exam. 30.04.2017)

    Eight people – A, B,C, D, E, F, G and H — were born in different years, viz. 1961, 1970, 1974, 1980, 1983, 1987, 1996 and 2000 but not necessarily in the same order. Note : (A) All calculations are done with respect to the present year 2017 assuming the month and date to be same as that of the years of birth as mentioned above. (B) Each person is assumed to born on the same date and same month of the respective years. D was born after 1983 but not in the year 2000. The sum of the present ages of A and D is 64. The difference between the present ages of A and G is less than 5. B was born in an odd numbered year. B is older than G. The sum of present ages of F and C is 68. F is younger than C. H is not the youngest. 220. Four of the following five are alike in a certain way as per the given arrangement and hence they form a group. Which of the following does not belong to the group? (1) E (2) B (3) C (4) F (5) G 221. How many persons is/are younger than E? (1) Two (2) More than three (3) None (4) Three (5) One 222. Who amongst the following was born in the year 1996? (1) F (2) C (3) A (4) G (5) E

    PROBLEM SOLVING 223. If A’s uncle is 22 years older than A, then how old is A’s uncle at present (in years)? (1) 77 (2) 68 (3) 89 (4) 56 (5) 85 224. Which of the following represents the difference between the present ages of B and H (in years)? (1) 22 (2) 9 (3) 25 (4) 32 (5) 13 Directions (225–227) : Study the following information carefully and answer the questions given below : (SBI PO Online (Preliminary) Online Exam. 06.05.2017)

    Each of the six stores D, E, F, G, H and I sold different number of fruits in one day. Only two stores sold less fruits than I. F sold less fruits than D but more than G and I. G sold less fruits than I. H sold more fruits than D. The store which sold the third highest number of fruits sold 62 fruits. (Note : the number of fruits sold is in whole numbers). 225. If the total number of fruits sold by E is 60, with reference to the total number of fruits sold, E + D = 135, then D + F = _______ ? (1) 122 (2) 111 (3) 129 (4) 139 (5) 137 226. If the total number of fruits sold by E is 60, then who amongst the following sold the lowest number of fruits? (1) G (2) D (3) F (4) Cannot be determined (5) E 227. How many fruits did H probably sell ? (1) 61 (2) 54 (3) 81 (4) 49 (5) 30 Directions (228–232) : Study the following information carefully and answer the questions given below : (SBI PO Online (Preliminary) Online Exam. 06.05.2017)

    Eight people – J, K, L, M, N, O, P and Q – were born in different years, viz. 1955, 1963, 1977, 1982, 1988, 1993, 2000 and 2006 but not necessarily in the same order. Note : (A) All calculations are done with respect to the present year, 2017

    assuming the month and date to be the same as that of the years of birth as mentioned above. (B) Each person is assumed to be born on the same date and same month of the respective years. M was born after 1993. The sum of the present ages of M and Q is 46. The difference between the present ages of Q and P is 5. L’s age is exactly divisible by 9. The difference between the present ages of N and J is less than 6. J is older than N. O was born in an even numbered year. O is not 11 year old. 228. How many person(s) is/are younger than M? (1) Three (2) None (3) Two (4) One (5) More than three 229. Which of the following statements is true regarding K as per the given arrangement? (1) The difference between the present ages of K and P is less than 12. (2) K was born in an even numbered year. (3) None of the given statements is true. (4) K is younger than L (5) K is 11 years old. 230. If N is older than O but younger than J, then which of the following can possibly be the present age of N (in years)? (1) 24 (2) 17 (3) 29 (4) 35 (5) 40 231. Who amongst the following was born in the year 1977? (1) J (2) K (3) L (4) N (5) P 232. Which of the following represents the present ages of J and L (in years) respectively? (1) 35, 54 (2) 24, 40 (3) 24, 62 (4) 29, 54 (5) 29, 40 Directions (233–237) : Study the following information carefully and answer the questions given below : (SBI PO Online (Preliminary) Exam, 07.05.2017)

    There are seven persons P, Q, R, S, T, U and V who were born on the same day of the same month of different years i.e., 1984, 1946, 1967, 1972, 1982, 1989 and 1992 but not necessarily in the same order.

    BPRE–717

    Note : (A) All calculations are done with respect to the present year 2017 assuming the month and date to be the same as that of the years of birth as mentioned above. (B) Each person is assumed to be born on the same date and same month of the respective years. The difference between the ages of Q and R is twice the square root of the age of one of the any seven persons. Difference between the ages of R and S is the same as the number obtained by dividing ages of any of the other five persons. Age of P is greatest amongst those whose ages are multiples of five. T is older than V who is not the youngest. R is not youngest person. S was not born in 1992. 233. Who amongst the following persons is the oldest? (1) P (2) V (3) U (4) T (5) None of these 234. What is the age of R? (1) 33 years (2) 35 years (3) 25 years (4) 45 years (5) 50 years 235. How many persons are younger than U? (1) One (2) Two (3) Three (4) Four (5) None of these 236. What is the age of S? (1) 45 years (2) 35 years (3) 33 years (4) 50 years (5) 28 years 237. Who was born in 1989? (1) V (2) U (3) T (4) P (5) Q Directions (238–242) : Study the following information carefully and answer the questions given below : (SBI PO Online (Preliminary) Exam, 07.05.2017)

    Eight persons M, N, O, V, W, X, Y and Z attend seminars on different months of the year viz. March, June, October and November, such that not more than two persons attend their seminars in each of the months. Seminars can be held on either 10th or 27th day of the month. No two seminars can be held on the same day. W and N attend the seminars on the same month. There are three seminars between the seminars of X and O. W does not attend their seminar in November. Z attends his seminar im-

    PROBLEM SOLVING mediately after N. V attends his seminar in the month of November. The number of persons who attend their seminars between the seminars of Y and Z respectively is the same as the number of persons who attend their seminars between the seminars of N and V respectively. X does not attend the seminar on October. W attends his seminar before N. 238. M attends his seminar on which of the following dates? (1) 10th October (2) 27th November (3) 10th November (4) 10th March (5) None of these 239. Which of the following persons attends his seminar on 27th March? (1) W (2) X (3) M (4) N (5) None of these 240. How many persons attend the seminar after W? (1) 5 (2) 4 (3) 6 (4) 3 (5) None of these 241. Who among the following persons attends the seminar on 10th October? (1) W (2) M (3) Z (4) V (5) None of these 242. How many persons attended seminar after V? (1) 5 (2) 4 (3) 7 (4) No one (5) None of these Directions (243–246) : Study the following information carefully and answer the questions given below :

    ple were born between D and the one born in Vellore as between D and the one born in Indore. Only two people were born between D and the one born in Firozabad. A was not born in Firozabad, No one was born between P and Q. Q was born neither in Vellore nor in Firozabad. The one born in Anand was born in one of the years after R but not in 2006. Only one person was born between the one born in Delhi and the one born in Pune. D was not born in Pune. 243. How many people were born between P and the one born in Pune? (1) Three (2) More than three (3) One (4) None (5) Two 244. In which city was A:born? (1) Indore (2) Bhopal (3) Anand (4) Other than those given as options (5) Pune 245. In which year was P born? (1) 1991 (2) 1999 (3) 1988 (4) 2001 (5) 1993 246. As per the given arrangement, R is related to Bhopal in the same way as Q is related to Delhi. Following the same pattern, to which city is D related ? (1) Indore (2) Vellore (3) Anand (4) None of those given as options (5) Pune Directions (247–250) : Study the following information carefully and answer the questions given below :

    (SBI PO Phase-II (Main) Exam 04.06.2017)

    (SBI PO Phase-II (Main) Exam 04.06.2017)

    Seven people A, B, C, D, P, Q and R were born in different years — 1988, 1991, 1993, 1996, 1999, 2001 and 2006 and also in different cities — Firozabad, Pune, Indore, Anand, Bhopal, Vellore and Delhi, but not necessarily in the same order. A was born in an even numbered year. Only three people were born between A and the one born in Vellore. Only one person was born between B and the one born in Vellore. Only four people were born between B and C. No one was born between C and the one born in Indore. C was born after the one born in Indore. As many peo-

    Six people — A, B, C, D, E and F — like six different colours namely, Blue, Green, Silver, Yellow, Orange and Pink, but not necessarily in the same order. Each one of them also studies in one of the three Standards — I, IV and IX. Only two people study in each of the given Standards. F studies in an odd numbered standard with the one who likes Silver. The one who likes Pink is in a higher standard than D. D does not like Silver. F does not like Pink. Only two people are in a higher Standard than A. A does not like Pink.

    BPRE–718

    The one who likes Yellow and the one who likes Blue study in the same Standard. D does not like Yellow. C likes Green. E does not study in the Standard IX. 247. Four of the following five are alike in a certain way based on the given arrangement and hence they form a group, Which one of the following does not belong to the group? (1) B — Green (2) E — Orange (3) A — Blue (4) F — Silver (5) C — Yellow 248. Which of the following statements is true with respect to the given arrangement ? (1) The one who likes Green studies in Standard I. (2) B is in a higher Standard than C. (3) B likes Pink. (4) C and A study in the same Standard. (5) None of the given information is true 249. Who amongst the following likes Orange? (1) B (2) D (3) Other than those given as options (4) F (5) A 250. In which Standard does G study? (1) The one in which E studies (2) IX (3) The one in which A studies (4) The one in which the one who likes Blue studies (5) IV Directions (251–254) : Study the following information carefully and answer the questions given below : (SBI PO Phase-II (Main) Exam 04.06.2017)

    Five boxes viz, , E, F, G, H and I are placed above one another. Each box is of a different colour viz. Pink, Blue, Grey, White and Black. The grey box is kept immediately above H. Only two boxes are kept between the pink and the blue boxes. The pink box is kept at one of the positions above the blue box as well as H. More than one box is kept between the blue box and E. E is not pink in colour. As many boxes are kept between E and the grey box as between G and the white box, H is not white in colour, F Is kept at one of the positions below the pink box but not immediately below it.

    PROBLEM SOLVING 251. How many boxes are kept between I and the black box? (1) Cannot be determined (2) Three (3) None (4) One (5) Two 252. Which of the following statements is/are true with respect to the given arrangement? (i) There are only two boxes between E and the blue box. (ii) F is placed at the bottom-most position (iii) There is only one box between E and grey box. (1) Both (i) and (ii) (2) Both (ii) and (iii) (3) Only (i) (4) Only (ii) (5) Only (iii) 253. Which of the following boxes is black in colour? (1) H (2) I (3) E (4) G (5) Cannot be determined 254. Which of the following represents the colour of I ? (1) Grey (2) White (3) Black (4) Blue (5) Pink Directions (255–258) : Study the following information carefully and answer the questions given below: (SBI PO Phase-II (Main) Exam 05.08.2018)

    Eight persons — P, Q, R, S, T, V, W and X — attend the lecture in different months viz., January, August, November, March, October, April, September and June but not necessarily in the same order. Each one of them likes different ice creams Vanilla, Strawberry, Chocolate, Mango, Butter pecan, Kulfi, Rocky road and Cookies but not necessarily in the same order. More than five persons attend the lecture between P and the one who likes Cookies. W attends the lecture before the one who likes Kulfi. As many persons attend the lecture after X is the same as many persons attend the lecture before the one who likes Butter pecan. Q does not attend the lecture immediately before or after X. Q does not like Butter pecan. As many persons attend the lecture after the one who likes Kulfi is same as many persons attend the lecture before W. X attends the lecture immediately before the one who likes Cookies. W

    does not like Butter pecan. Only one person attends the lecture between T and the one who likes Mango. V attends the lecture immediately before the one who likes Mango. The one who likes Mango attend the lecture before the one who likes Rocky road. Only two persons attend the lecture between Q and the one who likes Strawberry. Only one person attends the lecture between R and the one who likes Vanilla. Q does not like Vanilla. 255. S likes which of the following ice creams? (1) Strawberry (2) Cookies (3) Kulfi (4) Rocky Road (5) None of these 256. Four of the following five are like in a certain way based on the above arrangement and hence they form a group. Find which one does not belong to the group? (1) The one who likes Chocolate (2) T (3) The one who likes Butter pecan (4) The one who likes Strawberry (5) X 257. If P is related to KuIfi, W is related to Cookies in the same way V is related to which of the following? (1) Vanilla (2) Mango (3) Strawberry (4) Cookies (5) None of these 258. Which of the following combinations is true? (1) W – Mango (2) Q – Vanilla (3) X – Strawberry (4) T – Cookies (5) P – Chocolate Directions (259-263) : Study the following information carefully and answer the questions given below : (SBI PO Phase-II (Main) Exam 05.08.2018)

    There are 8 mothers J to Q and 8 children R to Y sitting on the two circular tables but not necessarily in the same order. The circular tables are such that one is small and second is

    BPRE–719

    large with small inside the larger one. The persons sitting on the outer circular table are facing the centre and the persons sitting on the inner circular table are facing outside the centre. So in this way, the mothers are facing the child when they are sitting on the same sides of the table. All the children are aged from 1 to 8 and all the mothers are aged from 31 to 38. No two children or two mothers are adjacent to each other in either of the circle. All the above information is not necessarily in the same order. The one, whose age is 38 doesn’t face inside the circle. L’s son is neither S nor U and he is youngest among eight children. Only three people sit between T’s mother and K, whose age is 34. J is the mother of the child, whose age is 3. N is older than Q and ages of both of them are prime numbers. M doesn’t face inside the circle and L’s age is divisible by 11. N is the mother of X, whose age is 6 and N sits on the immediate left of P’s child. T is the daughter of O, whose age is perfect square and T sits on the immediate right of V’s mother. U is younger than S and difference between their ages is 2. U’s mother is not M. S, who is not an immediate neighbour of L. R is the child of P and faces inside the circle. R’s age is divisible by 5. Only one person sits between W’s mother and Q, whose child is V, whose age is perfect cube. three persons sit between N’s child and W, whose age is half of X. Age of P is a multiple of 7. 259. Which among the following pairs represents the oldest and the youngest mothers respectively? (1) M and Q (2) J and P (3) P and Q (4) Q and K (5) Cannot be determined 260. Four of the following five are alike in a certain way and hence they form a group. Which of the following does not belong to the group? (1) R (2) S (3) U (4) T (5) Y 261. What is the sum of the ages of W’s mother and O’s child? (1) 36 (2) 39 (3) 44 (4) 42 (5) Cannot be determined

    PROBLEM SOLVING 262. Who among the following represent the children of M and K respectively? (1) U and S (2) Y and S (3) X and Y (4) S and U (5) None of these 263. Who among the following are the immediate neighbours of the one, whose age is 4? (1) The one whose age is 33 and the one whose age is 35 (2) The one whose age is 32 and the one whose age is 37 (3) The one whose age is 31 and the one whose age is 32 (4) The one whose age is 37 and the one whose age is 38 (5) None of these Directions (264-268) : Study the following information carefully and answer the questions given below: (SBI PO Phase-II (Main) Exam 05.08.2018)

    Eight persons — G, H, Q, R, T, L, F and M — lives on eight different floors. Lowermost floor is numbered 1, above floor is numbered 2, and so on. The topmost floor is numbered 8. Each one of them are in different ages viz, 20, 23, 24,45, 31, 42, 36 and 17. Persons those live on even number floors are in odd number age. Persons those live on odd number floors are in even number age. All the above information is not necessarily in the same order. Two persons live between F and the one whose age is multiple of 7. Sum of Q’s and L’s age is one less than age of R. Q is younger than L. H is not the second eldest. H lives on odd numbered floor. Only one person lives between H and R. Q lives below G. More than two persons live between M and the one whose age is 31. The one who is the eldest lives below the one who is the second youngest. The one whose age is 24 does not live immediately above or below G. The one whose age is 31 does not live below the one whose age is 23. F does not live on top most floor. T’s age is two more than twice the age of Q. More than three persons live between H and T. 264. How many persons live between M and the one who is the youngest? (1) One (2) Two (3) Three (4) More than three (5) None

    265. If G is related to 42, L is related to 17, in the same way how F is related to : (1) 20 (2) 23 (3) 42 (4) 17 (5) None of these 266. How many persons live above H? (1) None (2) One (3) Two (4) Three (5) More than three 267. Four of the following five are alike in a certain way based on the above arrangement and hence they form a group. Find which one does not belongs to the group? (1) M (2) G (3) F (4) Q (5) L 268. Which of the following statements is true? (1) The one who is youngest lives below M (2) Sum of L’s and H’s age is 6 years more than age of G (3) R lives above Q (4) F lives immediately above T (5) None is true Directions (269–272) : Study the following information carefully and answer the questions given below : (SBI PO Phase-II (Main) Exam 05.08.2018)

    There are four floors in a building as ground floor is numbered 1 and the one above that is numbered 2 and so on till the topmost floor is numbered 4. Each of the floor consists of 3 flats as flat-1, flat-2 and flat-3. Flat-1 of floor-2 is immediately above flat-1 of floor -1 and immediately below flat-1 of floor-3 and so on. In the same way flat-2 of floor-2 is immediately above flat-2 of floor-1 and immediately below flat-2 of floor-3 and so on. Twelve persons A, B, C, D, E, F, G, H, I, J, K and L live in different flats of different floors. Only one person lives in one flat of one floor. Only one person lives between E and G in the same flat number. H lives on an even numbered floor. D lives in the immediate east of E. C lives to the immediate west of K and on an even numbered floor. No one lives to the east of B who lives on an odd numbered floor. Two persons live between F and I in the same flat number. F lives above I. Neither C

    BPRE–720

    nor K is an immediate neighbour of F. E lives immediately above K in same flat number. J does not live on same floor on which F and D live. A lives on one of the floor above L and not in the east of H. 269. K lives on which of the following flat number? (1) Flat-1 (2) Flat-2 (3) Flat-2 (4) Either Flat-1 or FIat-2 (5) Either Flat-2 or Flat-3 270. Who amongst the following live to the west of G? (1) I (2) C (3) K (3) F (5) None of these 271. Who among the following lives immediately above J in the same flat number? (1) B (2) D (3) H (4) F (5) None of these 272. Who among the following lives on 4th floor? (1) E (2) C (3) H (4) G (5) None of these Directions (273–277) : Study the following information carefully and answer the questions given below : SBI PO (Prelim Exam), 08.07.2018 (Shift-I))

    Seven boxes – A, B, C, D, E, F and G – are kept one above the other, but not necessarily in the same order. Each Box has a different number viz. 11, 14, 15, 17, 18, 19 and 22 but not necessarily in the same order. Only three boxes are kept between G and box number 19. Only two boxes are kept between G and B. B is kept at one of the positions below box number 19. Only one box is kept between B and box number 14. E is kept immediately below box number 22. E is kept at one of the places above box number 19. There is only one box between E and the box having number less than E. E’s box number is neither 17 or 18. Only two boxes are kept between box number 15 and F. The difference between F and the box immediately below it is less than four. C is not the topmost box. C’s box number is not 14. Only two boxes are kept between C and A. 273. What is the number of box C? (1) 15 (2) 18 (3) 22 (4) 19 (5) 11

    PROBLEM SOLVING 274. How many boxes are kept between E and box number 14? (1) 4 (2) 1 (3) 2 (4) More than 4 (5) 3 275. What is the position of D in the given stack of boxes ? (1) First from the top (2) Fifth from the top (3) Third from the top (4) Third from the bottom (5) Fourth from the bottom 276. Which of the following boxes is kept immediately above A? (1) B (2) Box number 5 (3) Box number 7 (4) G (5) Box number 4 277. Four of the following five are alike in a certain way and hence they form a group. Which one of the following does not belong to the group? (1) B–4 (2) E–2 (3) C–7 (4) D–1 (5) A–6 Directions (278–282) : Study the following information carefully and answer the questions given below : SBI PO (Prelim Exam), 08.07.2018 (Shift-I))

    Eight friends – A, B, C, D, E, F, G and H – were born in March, June, September and December on either 3rd or 8th (all born on different dates). The ones who were born in a month having 30 days like different fruits – Banana, Apple, Mango and Litchi not necessarily in the same order. The ones who were born in a month having 31 days like different colours – Red, Yellow, Blue and Green not necessarily in the same order. E was born in June. Only one person was born between E and D. D does not like any colour. B likes red colour. Same number of people were born before A as after D. No person was born between the ones who like yellow colour and mango. The one who likes yellow colour was not born on 8th of any month. No person was born between E and one who likes litchi. Only one person was born between B and one who likes Banana. A does not like Banana. The ones who like apple and banana were born either on same date or in same month. Only one person was born between A and H. F does not like any fruit. No person was born

    between C and one who likes blue colour. H does not like fruit. Same number of persons were born between the ones who like green colour and apple and who like blue colour and mango. 278. Who likes Green Colour? (1) C (2) F (3) G (4) H (5) E 279. How many people were born between B and the one who likes Litchi? (1) Four (2) Two (3) Three (4) None (5) One 280. Four of the following five are alike in a certain way and hence they form a group based on a certain pattern. Which one of the following does not belong to that group? (1) A (2) G (3) C (4) D (5) E 281. Who was born on 3rd September? (1) The one who likes Mango (2) E (3) C (4) D (5) The one who likes Litchi 282. C likes which of the following fruit or colour? (1) Yellow (2) Mango (3) Litchi (4) Blue (5) Banana Directions (283–287) : Study the following information carefully and answer the questions given below : Seven phones – C, E, G, L, R, U and Z are of different companies viz. Samsung, Apple and Motorola. Atleast two phones and not more than three phones are of the same company. Each phone is bought in a different city- Alwar, Dehradun, Fatehpur, Gwalior, Jamshedpur, Mumbai and Pune. G is an Apple phone. Only U and Z are of the same company but not of Motorola. Phone C is of the same company as the one bought in Gwalior. Phone E was bought in Alwar but is not of the same company as C. Phones E and R are of the same company. Phone L is of the same company as the one bought in Pune. Neither G nor C was bought in Pune. One of the Motorola phones was bought in Jamshedpur. One of the Apple phones was bought in Dehradun. Z was not bought in Fatehpur.

    BPRE–721

    283. Which of the following phones was bought in Mumbai ? (1) U (2) L (3) C (4) G (5) Other than those given as options 284. As per the given arrangement, four of the following five are alike in a certain way and hence they form a group. Which one of the following does not belong to the group? (1) L – Fatehpur (2) G – Dehradun (3) E – Pune (4) U – Mumbai (5) R – Alwar 285. In which city was phone L bought? (1) Dehradun (2) Pune (3) Jamshedpur (4) Fatehpur (5) Mumbai 286. Which of the following is an Apple phone? (1) E (2) The one bought in Gwalior (3) R (4) The one bought in Pune (5) The one bought in Mumbai 287. Which of the following statements is true with respect to the given information? (1) R was bought in Jamshedpur (2) None of the given statements is true (3) L was bought in Gwalior (4) The phone bought in Mumbai is of Samsung (5) C is a Mototrola Phone Directions (288–291) : Study the following information carefully and answer the questions given below : A showroom launched seven different cars – D, E, F, M, N, O and P consecutively one after the other. Each car was of different colour – Red, Pink, Silver, Black, White, Blue and Orange ● Only three cars were launched between Car D and the Silver car. Car E was launched immediately after the Silver car. ● Only two cars were launched between Car E and the Red car. The Orange car was launched on one of the occasions before the Red car.

    PROBLEM SOLVING ● Only

    three cars were launched between the Red car and the car F. No car was launched between car F and car O. ● More than three cars were launched between car O and the Black car. Only one car was launched between car M and the Pink car. M is not the Black car. ● No car was launched between Pink and the Blue cars. The White car was launched on one of the occasions after car N. 288. How many cars were launched between N and the White car? (1) More than three (2) None (3) Three (4) One (5) Two 289. What is the colour of Car P? (1) Black (2) Red (3) Orange (4) White (5) Blue 290. Which of the following is the Pink car? (1) O (2) D (3) F (4) P (5) E 291. How many cars were launched before the Orange car? (1) One (2) Two (3) None (4) More than three (5) Three Directions (292–296) : Study the following information carefully and answer the questions given below : (SBI PO Prelim Exam, 14.06.2019)

    Six people – D, F, L, N, R and S – were born on six different days of the same week starting from Monday and ending on Saturday but not necessarily in the same order. Each one of them has a different number of books. It is assumed that no person other than those mentioned was born in the said week. R was born after Wednesday but not on Friday. Only two people were born between R and the one who has 40 books. As many people were born after S as before the one who has 40 books. No one was born between S and N. Only two people were born between N and the one who has 35 books. R does not have 35 books. Only one person was born between the one who has 35 books and the one who has 53 books. D has odd number of books. L has 10 more books than D. F has 13 more books than R. One of the given people has 58 books.

    292. Who was born on Wednesday ? (1) The one who has 53 books (2) F (3) N (4) R (5) The one who has 58 books 293. How many books do L and N together have ? (1) 65 (2) 93 (3) 80 (4) 103 (5) 75 294. As per the given arrangement, the one who was born on Saturday has –––––– books. (1) 53 (2) 40 (3) 27 (4) 25 (5) 35 295. How many people were born before the one who has 53 books ? (1) Three (2) Two (3) More than three (4) None (5) One 296. As per the given arrangement, D is related to the one who has 40 books in the same way as S is related the one who has 58 books following a certain pattern. Following the same patter, _______ is related to the one who has 53 books. (1) L (2) None of the given options (3) R (4) N (5) F Directions (297–299) : Study the following information carefully and answer the questions given below : (SBI PO Prelim Exam, 14.06.2019)

    There are three floors in a building such that the ground floor is numbered one, the one above that is numbered two and the topmost floor of the building is numbered three. On each floor there are two flats viz. flat A and flat B such that flat B is exactly to the east of flat A. Also flat A of floor number one is immediately below flat A of floor number two. Similarly, flat B of floor number two is immediately below flat B of floor number three. Only one person lives in each flat. Thus, only two people live on each floor. Q lives on an even numbered floor. No one lives between the floors on which Q and T live. No one lives to the east of T. R lives in a flat immediately above S. No one lives to the west of S. R does not live on the same floor on which Q lives. P lives on one of the floors below U.

    BPRE–722

    297. Which of the following represents the floor number and the flat in which U lives ? (1) Floor number 3, Flat B (2) Floor number 2, Flat B (3) None of those given as options (4) Floor number 3, Flat A (5) Floor number 2, Flat A 298. Which of the following statements is true as per the given information ? (i) R lives in a flat exactly to the west of U. (ii) There is only one floor between the floors on which U and T lives. (iii) S lives on the same floor on which T lives. (1) Only (i) (2) Only (iii) (3) All (i), (ii) and (iii) (4) Both (i) and (ii) (5) Both (ii) and (iii) 299. Who amongst the following lives in flat B on floor number one ? (1) The one who lives in a flat exactly to the west T (2) U (3) P (4) The one who lives in a flat immediately below U (5) T Directions (300–302) : Study the following information carefully and answer the questions given below : (SBI PO Prelim Exam, 14.06.2019)

    Each of the six bags viz. A, B, C, D, E and F is of a different weight. B is heavier than E but lighter than both F and C. C is lighter than D. D is not the heaviest. E is not the lightest. The weight of second lightest bag is 5 kgs. D weighs 21 kgs. (Note : All the given weights are in whole numbers only) 300. With respect to the weights of the given bags if F + E = 29 kgs and F + B = 34 kgs, then B + D = _______ ? (1) 45 kgs (2) 15 kgs (3) Cannot be determined (4) 31 kgs (5) 26 kgs 301. Which is the third heaviest bag ? (1) C (2) D (3) A (4) B (5) E

    PROBLEM SOLVING 302. What is the probable weight of bag C (in kgs) ? (1) 4 (2) Either 2 or 3 (3) 15 (4) 24 (5) Either 22 or 25 Directions (303–307) : Study the following information carefully and answer the questions given below : (SBI PO Prelim Exam, 09.06.2019)

    Eight people were born in eight different months of the same year-January, February, April, June, August, September, October and December. It is assumed that no one other than the given people was born in given year. M was born in a month having only 30 days. Only two people were born bewteen M and P. P was born in one of the months before M. K was born immediately after P. Only three people were born between K and L. As many people were born after L as before Q. Only one person was born between O and J. More than three people were born between J and N. 303. In which month was J born ? (1) June (2) April (3) October (4) December (5) September 304. As per the given arrangement, as many people were born before K as after _____. (1) K (2) O (3) N (4) M (5) P 305. Who amongst the following was born in June ? (1) L (2) Q (3) P (4) K (5) M 306. Which of the followings statements is not true with respect to the given arrangement ? I. Q was born in Apirl. II. J was born immediately before L. III. Only one person was born between O and P. (1) All the given statements are true (2) Both II and III (3) Only II (4) Both I and III (5) Only I 307. How many people were born after O ? (1) Three (2) One (3) Two (4) More than three (5) None

    Directions (308–312) : Study the following information carefully and answer the questions given below : (SBI PO Prelim Exam, 14.06.2019)

    Six people are sitting around a circular table facing the centre with equal distance between adjacent persons. Each of them is pursuing a different course. A sits second to the left of the one who is pursuing B.Tech. The one who is pursuing B.Sc. is an immediate neighbour of the one who is pursuing B. Tech. Only two people sit between the one who is pursuing B.Sc. and the one who is pursuing M.Arch. B sits second to the left of the one who is pursuing M.Arch. Only one person sits between B and D (either from left or right). Only two people sit between D and F. The one who is pursuing B.Pharma sits second to the right of F. E sits second to the left of C. E is not an immediate neighbour of A. The one who is pursuing MBA sits to the immediate right of the one who is pursuing M.Com. 308. Four of the following five are alike in a certain way based on their positions as per the given arrangement and thus they form a group. Which one of the following does not belong to that group? (1) B.Tech–D (2) A–D (3) C–B.Sc. (4) B.Tech–M.Com (5) E–F 309. What is the position of C with respect to the one who is pursuing MBA? (1) Second to the right (2) Immediate right (3) Immediate left (4) Second to the left (5) Third to the right 310. Which of the following statements is true as per the given information? (1) None of the given statements is true. (2) Only one person sits between E and A. (3) The one who is pursuing B.Sc. is an immediate neighbour of D. (4) The one who is pursuing B.Tech sits second to the left of E. (5) A is pursuing MBA.

    BPRE–723

    311. Who amongst the following is pursuing M.Com? (1) The one who sits second to the right of the one who is pursuing M.Arch. (2) The one who sits to the immediate left of A. (3) An immediate neighbour of B. (4) B (5) E 312. Which of the following pairs represents the immediate neighbours of the one who is pursuing B.Pharma? (1) B and the one who is pursuing M.Arch (2) D and C (3) The one who is pursuing B.Tech and C (4) D and the one who is pursuing MBA (5) B and C Directions (313– 317) : Study the following information carefully and answer the questions given below : (SBI PO Main Exam, 20.07.2019)

    Twelve persons A, B, C, D, E, F, G, H, I, J, K and L live in a hotel. The hotel has three floors and there are four rooms on each floor. ● J lives immediate right above diagonally of a person who lives immediate right below diagonally of A. ● C lives immediate left below diagonally of a person who lives immediate left below diagonally of F. ● E lives immediate left above diagonally of a person who lives immediate left above diagonally of L. ● B lives immediate right above diagonally of a person who lives immediate right above diagonally of K. ● C lives to the immediate left of a person who lives two rooms below I in the same column. ● F lives immediate left above diagonally of a person who lives third to the right of H. ● D lives to the immediate right of a person who lives immediate right above diagonally of C. L is living to the immediate left of G who receives Rs. 46,000 as salary. ● The persons who live on one of the floors (left to right) receive salary in the same order Rs. 50,000, Rs. 48,000, Rs. 47,000 and Rs. 46,000. ● The persons who live on one of the floors (right to left) receive salary in the same order Rs. 45,000, Rs. 38,000, Rs. 35,000 and Rs. 40,000.

    PROBLEM SOLVING The persons who live on one of the floors (Left to right) receive salary in the same order Rs. 37,000, Rs. 42,000, Rs. 36,000 and Rs. 43,000. 313. What is the sum of the salaries of K and B? (1) Rs. 85,000 (2) Rs. 90,000 (3) Rs. 89,000 (4) Rs. 93,000 (5) Rs. 87,000 314. What is the salary received by the person who lives second to the right of E? (1) Rs. 36,000 (2) Rs. 47,000 (3) Rs. 38,000 (4) Rs. 42,000 (5) Rs. 43,000 315. What is the sum of the salaries received by the people living at the right end of the hotel? (1) Rs. 1,34,000 (2) Rs. 1,27,000 (3) Rs. 1,25,000 (4) Rs. 1,23,000 (5) Rs. 1,32,000 316. What is the sum of the salaries received by the people living on the top floor of the hotel? (1) Rs. 1,47,000 (2) Rs. 1,58,000 (3) Rs. 1,43,000 (4) Rs. 1,51,000 (5) Rs. 1,59,000 317. Who among the following lives third to the left of G? (1) A (2) F (3) H (4) E (5) C Directions (318–322) : Study the following information carefully and answer the questions given below : ●

    (SBI PO Main Exam, 20.07.2019)

    Seven persons viz., K, A, M, U, R, P and T have taken a Test on different days of the same week starting from Monday and ending on Sunday but not necessarily in the same order. Each of them scored different marks. ● Only twenty four marks in total were scored before Thursday and the one who took the test on Friday scored 4 marks. ● A took the test on Wednesday and the marks scored by him is an odd number.

    Only six marks were scored on Tuesday and K took the test before T. ● The marks scored by each of the persons who took the test on the first four days of the week are multiples of three. ● U took the test on Thursday and P scored twice the marks scored by M. ● The marks scored by U was an even number and maximum marks were scored on Monday. ● R took the test on Saturday and she scored two marks less than U. ● The marks scored by M are one less than the marks scored by A. 318. What is the sum of marks scored by A and R? (1) 15 (2) 20 (3) 13 (4) 17 (5) 12 319. On which day 12 marks were scored? (1) Monday (2) Wednesday (3) Friday (4) Thursday (5) Sunday 320. Which of the following pairs of persons have scored least and highest marks? (1) P and U (2) R and M (3) K and A (4) T and R (5) M and K 321. What marks were scored on Tuesday and Friday respectively? (1) 6 and 4 (2) 3 and 2 (3) 15 and 12 (4) 12 and 4 (5) 6 and 10 322. Who among the following took test on the day on which 6 marks were scored? (1) R (2) T (3) A (4) M (5) K ●

    RBI GRADE–B/ NABARD GRADE–A OFFICER EXAMS Directions (1–4) : Read the following information and five statements given below it carefully and answer the questions which follow. (RBI Grade-B Officer’s Exam. 18.12.2011)

    BPRE–724

    Steve Jobs himself said it: “It’s smart to take an existing idea and enhance it beautifully.” Apple did it and so did some of the most successful brands worldwide. To be successful, it is not essential to innovate every time. Old stuff in new packaging helps as well and is often referred to as ‘imovation’ which is a combination of innovation and imitation. (A) As an innovation is a standalone product in the market, it reaps in all the profit till the ‘imovators’ imitate it, ‘Imovation’, however, is not profitable as there are many imitated products in the market at the same time, all reducing each other’s profits. (B) Brand X duplicated Brand Y’s mobile phones with exactly the same features and same price range but failed to make an impact in the market. (C) No one remembers the innovators after some time because they did not turn their idea into success. On the other hand the imitators did what they had to just at the right time. (D) While Sanfy invented portable MP3 players, it was Apple which enabled MP3 players to play videos and games at a fairly reasonable price, thus capturing the market. (E) ‘Imovation’ is a lot less risky business venture. An imitator is already equipped with the analysis of how market received the innovation. 1. Blind imitation of a product can never be successful and sustainable. Which of the statements numbered (A), (B), (C), (D) and (E) mentioned above proves the above statement most appropriately ? (1) D (2) B (3) C (4) E (5) Both A and C 2. Which of the statements numbered (A), (B), (C), (D) and (E) mentioned above represents an advantage of imitating an existing product ? (1) B (2) Both B and D (3) E (4) Both C and B (5) Both A and D

    PROBLEM SOLVING 3.

    Which of the statements numbered (A), (B), (C), (D) and (E) mentioned above would contradict the fact that imitation of products is a successful market strategy ? (1) A (2) C (3) D (4) E (5) Both B and E 4. ‘Companies must add such values to their imitated products that not only distinguish their imitation but also add to customer satisfaction.’ Which of the statements numbered (A), (B), (C), (D) and (E) mentioned above proves the above statement most appropriately ? (1) A (2) Both A and B (3) Both A and E (4) Both A and C (5) Both B and D Directions (5–10) : Study the following information carefully and answer the questions given below : (RBI Officer Grade ‘B’ Phase-I Exam. 21.11.2015)

    Seven people namely M, N, O, P, Q, R and S like seven different genres of music namely Classical, Hip-hop, Pop, Jazz, Opera, Rock and Electronics but not necessarily in the same order. Each person also works in the same office but at a different position on the basis of seniority namely Trainee (TE), Assistant Manager (AM), Manager (MG), Senior Manager (SM), Chief Manager (CM), Executive Director (ED) and Director (DR) but not necessarily in the same order. (Please Note: The positions have been given in increasing order of seniority with TE being the junior most position whilst DR being the senior most position.) Only one person is senior than O. The one who is senior than O likes HipHop. Only two people are junior than R. M is junior than R and likes Electronica. M is not the junior most. Q is junior than the CM but senior than the one who likes Jazz. The one who likes is not the senior most. S likes Classical. The one who likes Pop is senior than Q but not the ED. P does not like Pop.

    5. As per the given arrangement, R is related to Opera and Q is related to classical in a certain way. To which of the following is P related to in the same way? (1) Pop (2) Jazz (3) Rock (4) Electronica (5) Hip-hop 6. Who amongst the following works as a MG? (1) R (2) Q (3) S (4) P (5) Other than those given as options 7. Which of the following pairs represents the people who have less experience than P and more experience than Q? (1) M, S (2) R, O (3) N, P (4) M, O (5) N, 0 8. Who amongst the following works as a TE? (1) P (2) N (3) R (4) S (5) O 9. Which combination represents the position at which N works and the music genres that he likes? (1) CM-Rock (2) CM-Pop (3) SM-Classical (4) MG-Opera (5) MG-Jazz 10. Which of the following music genres does R like? (1) Electronica (2) Pop (3) Rock (4) Hip-Hop (5) Jazz Directions (11–15) : Study the following information carefully and answer the questions given below : (RBI Officer Grade ‘B’ Phase-I Exam. 21.11.2015)

    Eight people A, B, C, D, E, F, G and H, live on eight different floors of building (but not necessarily in the same order.) The lowermost floor of the building is numbered one, the one above that is numbered two, and so on till the topmost floor is numbered eight. Each one of them plays a different game namely- San Andreas, Roadrash, Contra, Castlevania, Resident Evil, Tekken3, Dragon Quest and Metal Gear (but not necessarily in the same order). G lives on an even numbered floor above the floor numbered two. Only three people live between the G and

    BPRE–725

    the one who plays Roadrash. E lives immediately below the one who plays Roadrash. Only two people live between E and the one who plays Tekken3. C lives immediately above the one who plays Tekken3. The one who plays San Andreas lives on an even numbered floor below C. Only two people live between the one who plays San Andreas and the one who plays Dragon Quest. The one who plays Metal Gear lives immediately below B. B neither lives on the topmost floor nor plays Roadrash. F lives on odd numbered floor but not the lowermost floor. Only two people live between F and the one who plays Resident Evil. Only one person lives between D and the one who plays Contra. H lives immediately below the one who plays Castlevania. 11. Which of the following games does D play? (1) San Andreas (2) Roadrash (3) Tekken3 (4) Metal Gear (5) Castlevania 12. How many people live between G and the one who plays Tekken3? (1) One (2) None (3) Two (4) Four (5) Five 13. Which of the following statements is TRUE with respect to the given information? (1) G plays Resident Evil (2) C lives immediately below the one who plays Contra (3) All the give statements are true (4) B lives immediately above E. (5) Only four people live between F and the one who play roadrash. 14. Who amongst the following live exactly between A and the one who plays Dragon Quest? (1) D, the one who plays Resident Evil (2) D,B (3) E,H (4) E, The one how plays Tekken 3 (5) F, The one who plays Roadrash

    PROBLEM SOLVING 15. Four of the following five are alike in a certain way and so form a group. Which one of the following does not belong to the group? (1) A–Contra (2) H–Castlevania (3) F–Tekken 3 (4) B–Dragon Quest (5) C–Roadrash Directions (16–20) : Study the following information carefully and answer the questions given below : (RBI Officer Grade ‘B’ Phase-I Exam. 21.11.2015)

    Seven flights are bound to fly to different cities viz. Dubai, Sydney, London, Paris, Zurich Madrid and Rome from Monday to Sunday of the same week. Each flight departs at a different time viz. 4 pm, 5 pm, 6 pm, 7 pm, 8 pm, 9 pm and 10 pm in the night on their respective days. None of the information given is necessarily in the same order. The flight to Dubai departs on Wednesday but neither at 5 nor at 8 pm. There is only one day between the flight to Dubai and the flight which departs at 10 pm. There are only three days between the flights which departs at 5 pm and the flight to Rome. The flight which departs at 5 pm departs before the Rome bound flight. The flight which departs on Monday neither departs at 5 pm nor does if fly to Sydney. The flight which departs on Saturday leaves at a time before the flight which departs on Tuesday (i.e. if the Tuesday flight departs at 7 pm then the flight on Saturday departs at 6, 5 or 4 pm.) The flight to London departs on the day immediately before the day on which the 8 pm flight departs. The flights to Paris departs at 7 pm. There is a difference of 2 hours between the Zurich bound flight and the London bound flight (i.e. if a London bound flight departs at 4 pm then the Zurich bound flight departs at either 2 pm or 6 pm on its respective day). 16. Which flight departs on the day immediately after the day on which the Paris bound flight departs?

    (1) The flight which departs at 7 pm. (2) The flight which departs on Thursday. (3) None as the Paris bound flight departs on Sunday (4) The flight which departs at 5 pm (5) The Zurich bound flight 17. At what time does the Sydney bound flight departs (1) 9 pm (2) 10 pm (3) 6 pm (4) 5 pm (5) 4 pm 18. Which flight departs exactly 1 hour after the Sydney bound flight on its scheduled day? (1) The flight which departs on Friday (2) The flight which departs on Thursday (3) The flight which departs at 10 pm (4) The London bound flight (5) The Rome bound flight 19. The Dubai bound flight reaches Dubai in 4 hours after its start. If the flight stops over at New Delhi at exactly half the total time, at what time would the flight arrive in New Delhi? (1) 8 pm (2) 11 pm (3) 7 pm (4) 6 pm (5) 12 midnight 20. Which of the following flights departs at 10 pm? (1) The flight which departs on Friday (2) The Zurich bound flight (3) The flight which departs on Friday (4) The Paris bound flight (5) The flight depending on the day immediately before the Sydney bound flight 21. If ‘2’ is added to each odd digit and ‘1’ is subtracted from each even digit in the number 7243651, which of the following numbers will appear twice in the new number thus formed? (1) Both 5 & 3 (2) Both 3 & 7 (3) Only 1 (4) Only 9 (5) None (Nabard Officer Grade ‘A’ Online Exam. 01.03.2015)

    Directions (22-26) : Study the following information carefully to answer the questions given below : (RBI Officer Grade ‘B’ Phase-I Exam, 03.08.2014)

    A building has seven floors numbered one to seven, in such a way that

    BPRE–726

    the ground floor is numbered one, the floor above it, number two and so on such that the topmost floor is numbered seven. One of the seven people, viz, A, B, C, D, E, F and G lives on each floor. A lives on fourth floor. C lives on the floor immediately below B’s floor. Two people live between the floors of D and G. One people lives between the floors of G and A. Four people live between the floors of C and F. 22. Who among the following lives on the topmost floor? (1) F (2) B (3) G (4) D (5) E 23. Who among the following lives immediately above D’s floor? (1) G (2) E (3) F (4) A (5) C 24. Four of the following five are alike in a certain way and hence form a group. Which one of the following doest not belong to that group? (1) B (2) D (3) G (4) E (5) F 25. Who among the following lives on third numbered floor? (1) E (2) F (3) G (4) C (5) D 26. Who among the following lives exactly between the floors of E and F? (1) C (2) A (3) D (4) G (5) None of these Directions (27 – 32) : Study the following information carefully and answer the questions given below : (Nabard Officer Grade ‘A’ Online Exam. 01.03.2015)

    Eight friends – J, K, L, M, N, O, P and Q – live on eight different floors of a building but not necessarily in the same order. The lower most floor of the building is numbered one, the one above that is numbered two and so on until the topmost floor is numbered eight. J lives on floor numbered six. Only one person lives between J and L. O lives on the floor immediately below L. Only one person lives between O and P. O lives above P. K lives on an even numbered floor but not on the floor numbered two. Only two persons live be-

    PROBLEM SOLVING tween K and Q. Q does not live on the lowermost floor. N lives on one of the floors above Q. 27. Which of the following statements is true about M ? (1) K lives immediately above M (2) All the given statements are true. (3) Only two people live between M and Q. (4) M lives on the lowermost floor (5) M lives on an odd numbered floor. 28. Who amongst the following lives on the floor number seven ? (1) P (2) Cannot be determined (3) O (4) M (5) N 29. Four of the following five are alike in a certain way based on the given arrangement and thus form a group. Which of the following does not belong to that group ? (1) PL (2) MQ (3) OM (4) LN (5) QK 30. Who amongst the following lives on the floor numbered three ? (1) O (2) P (3) N (4) L (5) Q 31. Who lives exactly between the floors on which J and L live ? (1) P (2) Q (3) N (4) O (5) M 32. If P and L interchange their places, who will live between P and M? (1) J (2) No one (2) L (4) O (5) Q Directions (33–37) : Study the following information carefully and answer the questions given below : (NABARD Assistant Manager Exam, 15.15.2016)

    Seven persons namely P, Q, R, S, T, U and V like seven different fruits namely, Apple, Mango, Banana, Orange, Kiwi, Guava and Strawberry. Each of them works in either of the three companies viz. Samsung, Wipro and ITC with at least two of them in a company. (Note : None of the information given is necessarily in the same order.)

    R works in Wipro with the one who likes Banana. U does not work with R. P works in ITC with only the one who likes Kiwi. U does not like Apple. T works in Samsung with the one who likes Apple. T does not like Strawberry. The one who likes Strawberry works with the one who likes Mango but not with R. S works with the one who likes Guava. S does not like Banana. Q does not like Apple. 33. Which of the following combinations represents the persons working in Wipro? (1) S, R (2) Q, V, R (3) U, T, R (4) R, Q (5) T, R, P 34. Which of the following fruits does P like? (1) Banana (2) Strawberry (3) Guava (4) Mango (5) Orange 35. Which combination represents the company in which V works and the fuits he likes? (1) Samsung – Apple (2) Samsung – Banana (3) Wipro – Guava (4) ITC – Mango (5) ITC – Orange 36. Which of the following combinations is correct? (1) S – Strawberry (2) Q – Ornage (3) R – Apple (4) U – Kiwi (5) T – Mango 37. Which of the following statements is true? (1) R works in the company in which P works. (2) U works with the one who likes Mango (3) None of the given statements is true (4) S works in Wipro. (5) Q likes Kiwi. Directions (38–42) : Study the following information carefully and answer the questions given below : (RBI Officer Grade ‘B’ Phase-I Exam, 04.09.2016 (Shift-I))

    Seven people – J, K, L, M, N, O and P – have an interview on seven different days of the same week, starting from Monday and ending on Sunday, but not necessarily in the same order. Each one of them also likes different subjects namely-Statistics, Zoology, Sociology, English, Mathematics, Psychology and Economics, but not necessarily in the same order.

    BPRE–727

    Only four people have their interview between N and the one who likes Zoology. Neither N nor the one who likes Zoology has an interview on Sunday. P has an interview immediately after the one who likes Zoology. Only two people have their interviews between P and J. The one who likes Psychology has an interview on one of the days before J but not on Wednesday. Neither N nor P likes Psychology. Only two people have their interviews between the one who likes Psychology and the one who likes Statistics. The one who likes Economics has an interview immediately before the one who likes Statistics. The number of people having interview between P and the one who likes Economics is same as that of the number of people between J and the one who likes English. N does not like English. Only one person has an interview between the one who likes English and K. The one who likes Sociology has an interview immediately after O. L has an interview on one of the days after M. 38. Four of the following five are alike in a certain way based on the given arrangement and hence form a group. Which of the following does not belong to the group? (1) K–English (2) Wednesday-K (3) Mathematics-Wednesday (4) Sociology-Statistics (5) Tuesday–Zoology 39. How many people have their interviews between L and M? (1) Two (2) Three (3) None (4) One (5) More than three 40. Which of the following statements is TRUE as per the given ar rangement? (1) None of the given statements is true. (2) Only one person has an interview between K and J. (3) O likes Psychology. (4) M has an interview on Friday. (5) The one who likes Zoology has an interview on one of the days before J. 41. Who has an interview immediately after K? (1) The one who likes Economics (2) The one who likes Zoology. (3) J (4) P (5) M

    PROBLEM SOLVING 42. How many people have their interview before the one who likes Mathematics? (1) One (2) More than three (3) Two (4) None (5) Three Directions (43 –47) : Study the following information carefully and answer the questions given below : (RBI Officer Grade ‘B’ Phase-I Exam, 04.09.2016 (Shift-II))

    Seven people, namely, J, K, L, M, N, O and P like seven different movies namely, Twilight, Gladiator, Wanted, Dread, Hero, Jumanji and Signs but not necessarily in the same order. Each person also works in the same office but in a different department (on the basis of experience) namely Administration, Production, Marketing, HR, Finance, R&D and Client Relations (CR), but not necessarily in the same order. (Please Note: Each person has been allocated to a department as per increasing order of experience with the one in Administration being the least experienced whilst the one in Client Relations (CR) being the most experienced). Only two persons have less experience than K. P works in R&D. The one who likes Wanted has more experience than K but less than one who likes Jumanji. P neither likes Wanted nor Jumanji. The one who likes Wanted does not work in Finance. J, who is more experienced than K, likes Twilight. The person who works in Production is less experienced than the person who likes Hero. K does not like Hero. The person who works in HR is more experienced than both L and N. N is not the least experienced person. The one who likes Signs has more experience than N. M is more experienced than J. L does not like Dread. 43. Four of the following five are alike in a certain way based on the given arrangement and so form a group. Which is the one that does not belong to that group ? (1) MO (2) NK (3) PK (4) NJ (5) LO 44. Which combination represents the department in which O works and the movie he likes?

    (1) CR-Signs (2) CR-Gladiator (3) HR-Gladiator (4) Marketing-Wanted (5) HR-Wanted 45. Which of the following movies does M like ? (1) Jumanji (2) Hero (3) Gladiator (4) Signs (5) Dread 46. As per the given arrangement, HR is related to Signs and CR is related to Hero in a certain way. To which of the following is Production related to in the same way ? (1) Dread (2) Jumanji (3) Wanted (4) Gladiator (5) Twilight 47. Which of the following pairs represent the respective people who have more experience than J and less experience than K ? (1) O, J (2) P, O (3) M, N (4) L, N (5) P, M Directions (48–50) : Study the following information carefully and answer the questions given below : (NABARD Assistant Manager Exam, 15.15.2016)

    Six persons — J, K, L, M, N and O — are holding a different number of flowers, but not necessarily in the same order. J holds less than L but more than N. M holds less flowers than N but more than O. L does not hold the most number of flowers. M holds 16 flowers. 48. Who amongst the following holds the third highest number of flowers? (1) N (2) O (3) L (4) J (5) Other than those given as options 49. How many persons hold more flowers than L? (1) Three (2) Two (3) None (4) Four (5) One 50. If the difference between the number of flowers held by M and J is 12, then how many flowers does N possibly hold? (1) 9 (2) 22 (3) 34 (4) 4 (5) 43

    BPRE–728

    Directions (51–56) : Study the following information carefully and answer the questions given below : (RBI Officer Grade ‘B’ Phase-I Exam, 04.09.2016 (Shift-I))

    Seven boxes – A, B, C, D, E, F and G – are kept one above the other, but not necessarily in the same order. Each box has a different number viz. 2, 4, 5, 7, 8, 9 and 12, but not necessarily in the same order. Only three boxes are kept between G and box number 9. Only two boxes are kept between G and B. B is kept at one of the positions below box number 9. Only one box is kept between B and box number 4. E is kept immediately below box number 12. E is kept at one of the places above box number 9. There is only one box between E and the box having number less than E. E’s box number is neither 7 nor 8. Only two boxes are kept between box number 5 and F. The difference between F and the box immediately below it is less than four. C is not the topmost box. C’s box number is not 4. Only two boxes are kept between C and A. 51. What is the number of box C? (1) 5 (2) 9 (3) 12 (4) 8 (5) 2 52. How many boxes are kept between E and Box number 4? (1) Three (2) One (3) Two (4) None (5) More than three 53. What is the position of D in the given stack of boxes? (1) Fourth from the top (2) Fifth from the bottom (3) First from the top (4) Second from the bottom (5) Third from the bottom 54. Which of the following boxes is kept immediately above A? (1) B (2) Box number 5 (3) Box number 7 (4) G (5) Box number 4 55. Four of the following five are alike in a certain way and hence form a group. Which of the following does not belong to the group? (1) B–4 (2) C–7 (3) E–2 (4) D–9 (5) A–8

    PROBLEM SOLVING 56. Which of the following boxes is numbered 8? (1) A (2) F (3) B (4) G (5) D Directions (57–61) : Study the following information carefully and answer the questions given below : (RBI Officer Grade ‘B’ Phase-I Exam, 04.09.2016 (Shift-I))

    Seven people, namely A, B, C, D, E, F and G like seven different pets namely, Dog, Fish, Cat, Rabbit, Parrot, Horse, and Mice but not necessarily in the same order. Each person also works in the same office but in different department (on the basis of experience) namely, Admission, Pharmacy, Emergency, Finance, Laboratory, Physiotherapy and Surgery, but not necessarily in the same order. (Please Note : Each person has been allocated to a department as per increasing order of experience with the one in Admission being the least experienced whilst the one in surgery being the most experienced.) Only one person has more experience than B. E works in Finance. The one who likes Rabbit is more experienced than the one who likes Mice but less experienced than E. The one who likes Rabbit does not work in Pharmacy. D is more experienced than the one who likes Rabbit but less experienced than the one who likes Fish. B does not like Fish. A likes Horse. The one who likes Dog is less experienced than D. Both C and F have less experience than the one who likes Dog. F neither likes Rabbit nor is the least experienced person. The one who likes Cat does not work in Physiotherapy. 57. Which of the following pets does B like? (1) Rabbit (2) Parrot (3) Mice (4) Cat (5) Dog 58. Four of the following five are alike in a certain way based on the given arrangement and so form a group. Which is the one that does not belong to that group? (1) CD (2) AC (3) GD (4) AG (5) FE 59. Which of the following pairs represent respective people who have more experience than E and less experience than F ?

    (1) F, D (2) G, C (3) B, A (4) D, G (5) C, B 60. Which combination represents the department in which G works and the pet he likes? (1) Surgery–Fish (2) Physiotherapy–Fish (3) Surgery–Rabbit (4) Physiotherapy–Rabbit (5) Pharmacy–Cat 61. As per the given arrangement, Admission is related to Mice and Finance is related to Cat in a certain way. To which of the following is Emergency related to in the same way? (1) Rabbit (2) Parrot (3) Horse (4) Fish (5) Dog Directions (62–67) : Study the following information carefully and answer the questions given below : (RBI Officer Grade ‘B’ Phase-I Exam, 04.09.2016 (Shift-II))

    Seven boxes — P, Q, R, S, T, U and V — are kept one above the other, but not necessarily in the same order. Each box has a different number viz. 2, 5, 6, 7, 10, 11 and 15, but not necessarily in the same order. Only two boxes are kept between R and S. Only four boxes are kept between R and box number 10. Box number 2 is kept immediately below V. V is kept at one of the positions below S. R’s number is not 2. Only three boxes are kept between Q and box number 11. Q is neither the topmost nor the lowermost box. The difference between Q and box immediately below it is more than nine. Q’s box number is not 5. Only two boxes are kept between box number 5 and U. The sum of box numbers of U and T is 8. 62. Four of the following five are alike in a certain way and hence form a group. Which of the following does not belong to the group ? (1) V-15 (2) P-5 (3) Q-6 (4) R-7 (5) S-10 63. What is the position of T in the given stack of boxes ? (1) Fifth from the bottom (2) Fourth from the top (3) First from the top (4) Third from the bottom (5) Second from the bottom

    BPRE–729

    64. Which of the following boxes in numbered 6 ? (1) R (2) S (3) U (4) Q (5) V 65. Which of the following boxes is kept immediately below T ? (1) Box number 11 (2) Box number 7 (3) S (4) Q (5) Box number 6 66. What is the number of box V? (1) 11 (2) 5 (3) 6 (4) 15 (5) 7 67. How many boxes are kept between P and box number 15 ? (1) Two (2) One (3) More than three (4) None Directions (68–72) : Study the following information carefully and answer the questions given below : (RBI Officer Grade ‘B’ Phase-I Exam 17.06.2017)

    Eight people — A, B, C, D, E, F, G and H — live on eight different floors of a building but not necessarily in the same order. The lower most floor of the building is numbered one, the one above that is numbered two and so on till the topmost floor is numbered eight. Each one of them ran for a different distance in a marathon — 2300 metre, 3800 metre, 5000 metre, 6400 metre, 7200 metre, 6300 metre, 9100 metre and 10000 metre but not necessarily in the same order. The one who ran for 5000 metre lives on an even numbered floor above floor number 5. Only three people live between the one who ran for 5000 metre and G. The one who ran for 9100 metre lives immediately above E. Only two people live between the one who ran for 9100 metre and A. A does not live on the topmost floor. The total distance run by people living on floor number 3 and floor number 6 is 11400 metre. The one who ran for 6300 metre, lives immediately above the one who ran for 3800 metre. Neither E nor A ran for 6300 metre, Only two people live between B and the one who ran for 6300 metre. The one who lives immediately below D ran for a distance more than that of B, but not the most. No one lives between H and the one who ran for 7200 metre. E did not ran

    PROBLEM SOLVING for 7200 metre. The one who ran for 10000 metre lives immediately above D. C and G together ran for 16,300 metre, C ran more than G. 68. If M ran for 2000 metre more than B, then for how many metres did M run? (1) 5200 metre (2) 5800 metre (3) 4900 metre (4) 7000 metre (5) 4300 metre 69. Which of the following statements is true with respect to the given arrangement? (1) D lives on the topmost floor. (2) Only two people live between G and F. (3) None of the given options is true (4) The one who ran for 2300 metre lives immediately above F. (5) A ran for 9100 metre 70. Who amongst the following lives immediately above the one who ran for 7200 metre? (1) The one who ran for 10000 metre (2) H (3) F (4) The one who ran for 3800 metre (5) G 71. Which of the given combinations is correct as per the given arrangement? (1) Floor number 7–3800 metre (2) Floor number 3–E (3) C–9100 metre (4) D–5000 metre (5) Floor number 2–7200 metre 72. How many metres did F and A together run? (1) 16200 metre (2) 7300 metre (3) 12900 metre (4) 15300 metre (5) 11400 metre Directions (73–78) : Study the following information carefully and answer the question given below : (RBI Officer Grade ‘B’ Phase-I Exam 17.06.2017)

    Twelve Professors — O, P, Q, R, S, T, U, V, W, X, Y and Z — had lectures in different months of the same year viz., January, March, April, May, June and December but not necessarily in the same order. All the lectures are either on 12th or 25th of these

    months. No two professors had lectures on the same day. Each professor also likes a different colour namely, Red, Blue, Green, Yellow, Orange, White, Pink, Silver, Maroon, Violet, Brown and Grey but not necessarily in the same order. (NOTE : No lecture was conducted in any other month of the same year). V had a lecture on 12th April. Only two people had lectures between V and Q. The one who likes red had a lecture in the same month as Q. Q does not like red. The one who likes White had a lecture on an even numbered date of the month which had exactly 30 days. V does not like white. Only three people had a lecture between the one who likes White and the one who likes Violet. As many people had lectures between the one who likes red and O as between Q and the one who likes White. Only two people had lectures between O and the one who likes Blue. U had a lecture on 12th of a month before the one who likes Violet. U likes neither red nor blue. Only two people had lectures between U and the one who likes Orange. Only two people had lectures between the-one who likes Orange and the one who likes Green. X had a lecture in the same month as the one who likes Green. As many people had a lecture after W as before X. Only one person had a lecture between W and Z. Z had a lecture before W. The one who likes Pink had a lecture on an odd numbered day in the same month as Z. P had a lecture on 12th of the same month as R. Only two people had lectures between P and the one who likes Yellow. The one who likes Maroon had a lecture before P. T likes Brown. S does not like Red. W does not like Grey. 73. Which of the following combinations indicates those who had a lecture in March? (1) The one who likes Maroon and R (2) U and X (3) P and R (4) The one who likes Green and the one who likes Pink (5) R and V 74. Which of the following combinations is correct? (1) W-White (2) X- Pink (3) U-Grey (4) V-Violet (5) O-Red

    BPRE–730

    75. If Y is related to White and P is related to Violet based on the given arrangement, then which of the following is related to O following the same pattern? (1) Grey (2) Pink (3) Brown (4) Maroon (5) Silver 76. HOW many people have lectures between the lectures by X and Y? (1) Two (2) Four (3) None (4) One (5) Three 77. Four of the following five are alike in a certain way based on the given arrangement and hence they form a group. Which one of the following does not belong to that group? (1) 12–Y (2) 25–O (3) 12–Green (4) 25–T (5) 12–Pink 78. Who amongst the following had lecture in the same month as S? (1) V (2) The one who likes Grey (3) The one who likes White (4) The one who likes Yellow (5) W Directions (79–83) : Study the following information carefully and answer the questions given below : (RBI Officer Grade ‘B’ Phase-I Exam 17.06.2017)

    Eight people — A, B, C, D, E, F, G and H — live in eight different cities — Varanasi, Nashik, Chennai, Siliguri, Ujjain, Bhopal, Jaipur and Patna, but not necessarily in the same order. Each one of them also studies in one of the four classes — II, V, VIII and XI. Two people study in each of the given classes. ● A studies in an even numbered class. The one who lives in Ujjain studies with A. ● The one who lives in Varanasi and Jaipur study in the same class, but in a class lower than VIII. D studies in a class lower than that of the one who lives in Varanasi. ● The one who lives in Nashik studies with H in the same class, but not in Class XI. D does not live in Nashik. ● B and G study in different classes. B lives neither in Jaipur nor in Nashik. G is senior to both B and H. Neither B nor the one who lives in Siliguri study in the same class as D. E lives in Siliguri.

    PROBLEM SOLVING C is junior to F. The one who lives in Patna studies in an odd numbered class. ● Neither H nor D lives in Bhopal. The one who lives in Chennai is senior to the one who lives in Bhopal. 79. In which of the following cities does C live? (1) Bhopal (2) Chennai (3) Jaipur (4) Nashik (5) Varanasi 80. Four of the following five are alike in a certain way based on the given arrangement and hence they form a group. Which one of the following does not belong to the group? (1) G–Siliguri (2) D–Bhopal (3) C–Varanasi (4) F– Chennai (5) A–Patna 81. In which class does H study? (1) The one in which A studies (2) VIII (3) The one in which E studies (4) V (5) II 82. Who amongst the following lives in Bhopal? (1) A (2) G (3) C (4) H (5) Other than those given as options 83. Which one of the following statements is TRUE with respect to the given information? (1) B lives in Nashik (2) G is senior to E. (3) B and F study in the same class. (4) The one who lives in Jaipur studies in Class V. (5) None of the given statements is true. Directions (84–87) : Study the following information carefully to answer the given questions : ●

    (NABARD Assistant Manager Grade ‘A’ Online Exam 05.08.2017)

    Seven people A, B, C, D, E, F and G were appointed to a company on seven different days of the same week starting from Monday to Sunday (but not necessarily in the same order). Each person also plays a different game namely, Cricket, Hockey, Football, Squash, Volleyball, Tennis and Kho-Kho, but not necessarily in the same order. Only two people were appointed after the one who plays Hockey. E was

    appointed on one of the days after the one who plays Hockey. Only three people were appointed between E and G. Only one person was appointed between G and the one who plays Volleyball. A was appointed immediately after the one who plays Volleyball. Only three people were appointed after the one who plays Kho-Kho. C was appointed immediately after F, but not on Friday. Only two people were appointed between F and the one who plays Cricket. B was appointed immediately before the one who plays cricket. Only two people were appointed between D and the one who plays Tennis. G does not play Football. 84. Who amongst the following was appointed on Wednesday? (1) The one who plays Kho-Kho (2) A (3) B (4) The one who plays Squash (5) F 85. Which of the following statements is true as per the given arrangement? (1) Only one person was appointed between F and the one who plays Squash. (2) Only three people were appointed before C. (3) A plays Tennis. (4) B was appointed on Saturday. (5) None of the given options is true 86. The person who plays Tennis was appointed on which of the following days? (1) Thursday (2) Tuesday (3) Sunday (4) Monday (5) Saturday 87. Which of the following combinations will be definitely true as per the given arrangement ? (1) C–Squash (2) Thursday–D (3) Saturday-Cricket (4) Monday–Football (5) F–Kho-Kho Directions (88-90) : Study the following information carefully and answer the questions given below : Each of the friends, P, Q, R, S, T and U borrowed different number of books. R borrowed more books than U but less than P. Q borrowed more books than P but not the highest. T borrowed less books than U. P borrowed 53 books. The person who borrowed second lowest number of

    BPRE–731

    books borrowed 12 books. (NABARD Assistant Manager Online Exam 06.08.2017)

    88. How many books did T possibly borrow? (1) 55 (2) 15 (3) 10 (4) 25 (5) 70 89. Who among the following borrowed the second highest number of books? (1) Q (2) T (3) P (4) S (5) R 90. If the number of books borrowed by S is 11 more than that of P, then how many books did Q possibly borrow? (1) 53 (2) 61 (3) 64 (4) 66 (5) 50 91. Study the following information carefully and answer the question. College H is known for having its cut-off for admission above 90%. But this year the cut-off was 81%. Which of the following can be a reason for College H having the cut-off at 81% this year? (1) The university to which College H is affiliated has increased the admission fee substantially for all colleges. (2) 2% seats of College H remained vacant last year as some students, migrated to other colleges within the first month of com-mencement of classes. (3) The evaluation of paper was very strict this year thereby reducing the top scorer’s percentage to 65%. (4) Few faculties of College H resigned at short notice this year leading to increase in load of the remaining faculties by two lectures per week. (5) None of those given as options (NABARD Assistant Manager Online Exam 06.08.2017)

    Directions (92–96) : Study the following information carefully and answer the questions given below : Seven movies A, B, C, D, E, F and G were released on seven different months of a year viz. January, Febru-

    PROBLEM SOLVING ary, April, July, September, November and December but not necessarily in the same order. Each movie also represented a different country namely – Spain, India, China, Malaysia, Germany, Peru and Nepal, but not necessarily in the same order. (RBI Assistant Manager Online Exam 25.03.2017)

    (Note : No other movie was released in any other month of the given year.) D was released in one of the months before April. Only three movies were released between D and the one which represented China. B was released immediately after F. Both B and F were released in a month which has 30 days. Only two movies were released between B and the one which represented Peru. Only one movie was released between A and the one which represented Germany. A was released in one of the months before the one which represented Germany. B did not represent Germany. Only three movies were released between the one which represented Spain and the one which represented Germany. As many movies were released between the one which represented Spain and D as between the one which represented G and Malaysia. Only two movies were released between G and the one which represented India. C did not represent India. 92. Which of the following combinations is definitely true as per the given arrangement? (1) B–September (2) A–Peru (3) E–Spain (4) July–Nepal (5) January–C 93. Which of the following statements is true as per the given arrangement? (1) None of the given options is true. (2) Only two movies were released between E and F. (3) Movie D represented China. (4) Movie G was released in January. (5) Three movies were released after the one which represented Germany. 94. Which of the following movies represented Nepal? (1) A (2) G (3) F (4) D (5) C

    95. How many movies were released after the movie E? (1) Three (2) One (3) Four (4) Two (5) None 96. Malaysia is related to F and India is related to C in a certain way based on the given arrangement. To which of the following is Peru related, following the same pattern? (1) The movie which represented Germany. (2) The one which released in December. (3) B (4) D (5) G Directions (97–101) : Study the following information carefully and answer the questions given below : (RBI Officers in Grade ‘B’ Phase-I Exam. 16.08.2018)

    Five boxes viz., P, Q, R, S and T are kept above one another. Each box has a different colour viz. red, blue, white, yellow and pink but not necessarily in the same order. Only two boxes are kept between R and the white box. Only one box is kept between R and S. Only two boxes are kept between the yellow and the blue boxes. The yellow box is kept at one of the positions above the blue box. The yellow box is not kept at the topmost position. As many boxes are kept between the white and the blue boxes as between T and the pink box. T is kept at one of the positions below S. Q is kept at one of the positions above P. 97. What is the position of the pink box in the given stack of boxes? (1) Third from the top (2) First from the top (3) Second from the bottom (4) Immediately below P (5) Immediately above the blue box 98. Which of the following pairs of boxes is kept between the yellow and the blue boxes? (1) The red and the white boxes (2) S and the white box (3) T, R (4) T and the red box (5) P, Q 99. How many boxes are kept between Q and the white box? (1) Two (2) None (3) One (4) Three (5) Cannot be determined

    BPRE–732

    100. Which of the following statements is/are true with respect to the given arrangement? (i) T is kept exactly between R and S. (ii) T is the yellow box. (iii) R is the red box. (1) Both (ii) and (iii) (2) Only (i) (3) Only (ii) (4) Only (iii) (5) Both (i) and (iii) 101. Which of the following represents the colour of box P? (1) Pink (2) White (3) Red (4) Yellow (5) Blue Directions (102–106) : Study the following information carefully and answer the questions given below : (RBI Officers in Grade ‘B’ Phase-I Exam. 16.08.2018)

    Seven people – P, Q, R, S, T, U and V – were born in different years, viz. 1945, 1956, 1968, 1970, 1981, 1996 and 2008. Each of them likes a different company viz. Samsung, TCS, Airtel, Wipro, Infosys, Google and Dell. Note : (A) All calculations are done with respect to the present year, 2018 assuming the month and date to be same as that of the years of birth as mentioned above. (B) Each person is assumed to be born on the same date and same month of the respective years. (C) None of the information given is necessarily in the same order. The difference between the present ages of Q and the one who likes Wipro is 12. Q is elder than the one who likes Wipro. The sum of the present ages of the ones who like Wipro and Dell is 60. P was born in an odd numbered year before the one who likes Wipro. The difference between the present ages of P and the one who likes Samsung is 11. The one who likes Samsung is younger than P. The sum of the present ages of T and V is 98. T neither likes Wipro nor Dell. The one who likes TCS was born in an even numbered year after T. As many persons were born after the one who like TCS as before S. The one who likes Infosys is elder than S. U does not like Wipro. The difference between the present ages of U and the one who likes Airtel is less than 15.

    PROBLEM SOLVING 102. How many persons were born between T and the one who likes Infosys? (1) Three (2) Two (3) One (4) None (5) More than three 103. Who amongst the following likes Google? (1) P (2) S (3) U (4) T (5) V 104. In which of the following years was R born? (1) 1968 (2) 1970 (3) 1981 (4) 2008 (5) 1956 105. Which of the following statements is true as per the given arrangement? (1) V likes Airtel (2) The difference between the present ages of R and the one who likes TCS is 5. (3) P is younger than S. (4) None of the given statements is true (5) Only three persons are elder than T. 106. Who amongst the following is/are younger than the one who likes TCS? (1) Only the one who likes Wipro (2) Both V and U (3) Both S and the one who likes Google (4) Only S (5) Both R and the one who likes Dell Directions (107–111) : Study the following information carefully and answer the questions given below : (RBI Officers in Grade ‘B’ Phase-I Exam. 16.08.2018)

    Nine people, namely A, B, C, D, E, F, G, H and I completed different number of projects in a year, viz. 9, 16, 23, 35, 39, 42, 56, 60 and 71. Each of them works in either of the four departments of a company viz. Production, Marketing, R&D and Finance with atleast two of them in a department. (Note : None of the information given is necessarily in the same order) F works in R&D department with the one who completed 23 projects. Both the ones who completed 35 and 42 projects work in the same department but not in Marketing department.

    F completed even number of projects. Only two people work in production department. The total number of projects completed by both the people in production department was 95. G completed maximum number of projects. B works with only G but not in Finance department. A works with the one who completed 16 and 60 projects. F completed less number of projects than A. The difference between the number of projects completed by D and H is 4. D completed more number of projects than H. D does not work in the department in which A works. C and D work in the same department. I completed more number of projects than the C. 107. Who amongst the following completed 56 projects? (1) A (2) I (3) C (4) D (5) B 108. In which of the following pairs did both the persons complete even number of projects? (1) A, D (2) F, A (3) H, F (4) E, B (5) F, I 109. What is the total number of projects completed by the people in R&D department? (1) 99 (2) 83 (3) 77 (4) 80 (5) 72 110. Which of the following combinations represents the department in which E works and the number of projects completed by him? (1) Production–39 (2) Finance–42 (3) Marketing–9 (4) Finance–35 (5) R&D–42 111. Four of the following five are alike in a certain way as per the given arrangement and hence they form a group. Which of the following does not belong to that group? (1) G–39 (2) I–71 (3) B–23 (4) E–35 (5) C–9 Directions (112–115) : Study the following information carefully and answer the questions given below : (SEBI Assistant Manager Exam. 17.11.2018)

    BPRE–733

    Six ropes P, Q, R, S, T and U are kept on a table. Each rope is of a different length and different colour viz., purple, red, golden, white, brown and silver, but not necessarily in the same order. Q is longer than only red rope. P is longer than Q but shorter than R. White rope is shorter than only brown rope. R is not brown in colour. U is shorter than S but longer than R. Golden rope is longer than Q but shorter than purple rope. 112. Which is the third longest rope? (1) S (2) R (3) P (4) White coloured rope (5) Silver coloured rope 113. What is the colour of rope P ? (1) Purple (2) White (3) Golden (4) Silver (5) Red 114. If length of rope U is 296 cm, what is probably the length of brown coloured rope? (1) 280 cm (2) 293 cm (3) 260 cm (4) 299 cm (5) 275 cm 115. Which of the following statements is true as per the given arrangement? (A) T is the smallest rope. (B) S is brown in colour. (C) Golden rope is shorter than red rope. (1) Only C (2) All A, B and C (3) Only A (4) Only B and C (5) Only A and B Directions (116–120) : Study the following information carefully and answer the questions given below : (SEBI Assistant Manager Exam. 17.11.2018)

    Seven people — H, I, J, K, L, M and O — live on seven different floors of a seven storeyed building where the ground floor is numbered one, the one above that is numbered two and so on till the topmost floor is numbered seven. Each one of them also likes a different metal viz. Gold, Iron, Platinum, Silver, Copper, Aluminium and Rhodium. (None of the given information is necessarily in the same order). Only four people live between L and O. L lives on one of the floors above O. Only two people live between O and the one who likes Platinum. H lives on an even numbered floor below the one

    PROBLEM SOLVING who likes Platinum. As many people live below H as above the one who likes Rhodium. K lives immediately below the one who likes Rhodium. Only three people live between K and the one who likes Gold. L does not like Gold. Only one person lives between the ones who like Gold and Copper. M lives immediately above the one who likes Aluminium. Only one person lives between I and the one who likes Iron. 116. How many people live between M and the one who likes Gold? (1) Two (2) One (3) More than three (4) None (5) Three 117. Which of the following statements is true with respect to the given information? (1) L likes Rhodium. (2) Only one person lives between O and M. (3) J lives on an even numbered floor. (4) No one lives above the one who likes Silver. (5) All of the given statements are true 118. Who amongst the following lives immediately above J? (1) O (2) The one who likes Platinum (3) L (4) The one who likes Iron (5) I 119. I is related to the one who likes Rhodium and H is related to the one who likes Gold following a certain pattern based on the given arrangement. Following the same pattern, L is related to the one who likes (1) Aluminium (2) Silver (3) Iron (4) Copper (5) Platinum 120. Who amongst the following likes Silver? (1) I (2) M (3) H (4) K (5) J Directions (121–125) : Study the following information carefully and answer the questions below : (NABARD Grade A Manager Exam, 15.06.2019)

    Eight people- P, Q, R, S, T, U, V and W live on eight different floors of a building. The lowermost floor of the building is numbered one, the one above that is numbered two and so on till the topmost floor is numbered eight. Each one of them likes a different game- Archery, Chess, Badminton, Tennis, Football, Cricket, Snooker and Rugby but not necessarily in the same order. V lives on floor number five. No one lives between V and the one who likes Snooker. Only three people live between the one who likes Snooker and the one who likes Archery. R lives immediately below the one who likes Archery. Only one person lives between R and the one who likes Tennis. As many people live above the one who likes Tennis as below T. No one lives between W and T. P lives on an odd numbered floor. Only four people live between P and the one who likes Rugby. As many people live between T and the one who likes Rugby as between Q and the one who likes Chess. S lives immediately below the one who likes Badminton. The one who likes Football lives on one of the floors above the one who likes Badminton. 121. The one who likes Rugby is related to V and the one who likes Chess is related to R following a certain pattern based on the given arrangement. To which of the following is the one who likes Snooker related following the same pattern? (1) P (2) U (3) S (4) Q (5) W 122. On which of the following floor numbers does S live? (1) Six (2) Two (3) Four (4) One (5) Five 123. Who amongst the following likes Cricket? (1) W (2) R (3) T (4) P (5) U 124. How many people live between U and the one who likes Football? (1) Two (2) One (3) More than three (4) Three (5) None

    BPRE–734

    125. Which of the following statements is not true as per the given arrangement? (1) Only one person lives between Q and S. (2) R lives on the lowermost floor. (3) T lives immediately below the one who likes Football. (4) V likes Tennis. (5) All the given statements are true Directions (126–130) : Study the following information carefully and answer the questions given below : RBI Grade B Officer Exam, 09.11.2019

    Each of seven people – J, K, L, M, N, O and P – bought a car in seven different months viz. January, March, April, June, August, September and October of the same year. Each person bought a car of a different brand. No person other than the given people bought a car in the given year. J bought a car in one of the months after August. Only two people bought a car between J and the one who bought a Skoda. K bought a car in a month immediately before the one who bought a Skoda. As many people bought a car after K as before the one who bought a Volkswagen. L bought a car in one of the months after the one who bought a Volkswagen. Only two people bought a car between L and the one who bought a Honda. The one who bought a Hyundai bought a car in a month having 31 days in one of the months after the one who bought a Honda but not in August. As many people bought a car before the one who bought a Hyundai as after M. The one who bought a Suzuki bought a car in a month immediately after M. Only three people bought a car between the one who bought a Suzuki and the one who bought a Mahindra. N bought a car in a month having only 30 days. O bought a car in one of the months before N. P bought a car in one of the months after the one who bought a Toyota. 126. In which month did P buy a car? (1) October (2) August (3) March (4) June (5) April 127. How many people bought a car between L and the one who bought a Toyota? (1) Two (2) None (3) Three (4) More than three (5) One

    PROBLEM SOLVING 128. Which of the following statements is true based on the given information? (1) Only three people bought a car between the one who bought a Skoda and O. (2) No one bought a car after L. (3) None of the given statements is true. (4) N bought a Mahindra. (5) As many people bought a car after the one who bought a Suzuki as before L. 129. Four of the following five are alike in a certain way as per the given arrangement and thus they form a group. Which one of the following does not belong to the group? (1) January-Suzuki (2) March-Honda (3) J-Toyota (4) P-Mahindra (5) N-Volkswagen 130. Who amongst the following bought a Honda? (1) The one who bought a car in April. (2) O (3) The one who bought a car in October. (4) J (5) The one who bought a car in June. Directions (131–135) : Study the following information carefully and answer the questions given below : RBI Grade B Officer Exam, 09.11.2019

    Eight people live on four different floors of a four storeyed building where the ground floor is numbered one, the one above that is numbered two and so on till the topmost floor of the building is numbered four. On each floor there are two flats viz. flat M and flat N such that flat M is exactly to the west of flat N. Also flat M of floor number two is immediately above flat M of floor number one. Similarly, flat N of floor number three is immediately above flat N of floor number two and so on. Thus, only two people live on each floor and only one person lives in each flat. Also each one of them is of a different age. A lives on an odd numbered floor in the flat immediately below the one in which the 69 year old lives. No one lives to the west of A. There is only one floor between the floors on which the 69 year old and the 53 year old

    live. The 47 year old lives exactly to the west of the 53 year old. B lives in the flat immediately below the flat in which the 47 year old lives. The one who live exactly to the east of B is 24 years old. There is only one floor between the floors on which C and the 24 year old lives. G lives exactly to the east of E.G is not 53 years old. G is 6 years elder than E. H lives exactly to the east of F. The sum of the present ages of F and C is 109. C lives on one of the floors above F. The difference between the present ages of C and A is 7. D is 12 years younger than the one who lives exactly to his west. 131. Who amongst the following is 55 years old? (1) A (2) The one who lives immediately above C (3) The one who lives immediately below D (4) The one who lives exactly to the east of F (5) C 132. Which of the following represents the floor number and the flat in which D lives? (1) Floor number 1, flat M ' (2) Floor number 4, flat N (3) Floor number 2, flat M (4) Floor number 1, flat N (5) Floor number 4, flat M 133. Which of the following combinations is not correct as per the given information? (1) H–the 53 year old (2) A–the 55 year old (3) G–the 62 year old (4) D–the 24 year old (5) B–the 36 year old 134. Which of the following statements is true as per the given information? (A) D is 45 years younger than E. (B) The sum of the present ages of H and B is 89 years. (C) The 55 year old lives on one of the floors below D. (D) G is younger than H. (1) Both (B) and (C) (2) Both (C) and (D) (3) Only (A) (4) Only (D) (5) Both (A) and (B) 135. What is the difference between the present ages of H and E? (in years)

    BPRE–735

    (1) 23 (2) 16 (3) 31 (4) 39 (5) 20 Directions (136–140) : Study the following information carefully and answer the questions given below : RBI Grade B Officer Exam, 09.11.2019

    Seven people bought seven different things consecutively one after the other on different occasions. No person other than those given bought a thing on the given occasion or any other occasion. Only two people bought a thing between A and the one who bought flowers. B bought a thing immediately after the one who bought flowers. Only one person bought a thing between B and C. Only three people bought a thing between C and the one who bought a watch. The one who bought a watch bought it on one of the occasions before C. G bought a thing immediately before C. Only two people bought a thing between G and F. Only two people bought a thing between F and the one who bought a radio. The one who bought a bag bought it immediately before the one who bought a laptop. More than three people bought a thing before E. The one who bought a calculator bought it on one of the occasions before the one who bought goggles but after D. 136. Four of the following five are alike in a certain way based on their positions as per the given arrangement and thus they form a group. Which one of the following does not belong to that group? (1) C-Flowers (2) A-G (3) Laptop-Calculator (4) Watch-D (5) E-Goggles 137. How many people bought a thing between G and the one who bought Goggles? (1) Three (2) Two (3) None (4) One (5) More than three 138. Who amongst the following bought Goggles? (1) B (2) E (3) A (4) G (5) C

    PROBLEM SOLVING 139. Which of the following statements is true as per the given information? (1) D bought a thing on one of the occasions after B. (2) No one bought a thing before the one who bought a bag. (3) E did not buy flowers. (4) F bought a thing on one of the occasions before the one who bought a calculator. (5) None of the given statements is true 140. Which of the following things did D buy? (1) Watch (2) Flowers (3) Radio (4) Laptop (5) Bag Directions (141–145) : Study the following information carefully and answer the questions given below : RBI Grade B Officer Exam, 09.11.2019

    Nine people viz., L, M, N, O, P, Q, R, S and T work in either of the three countries viz. India, Canada and Mexico. Atleast two and not more than four people work in each country. Each one of them likes a different colour : Black, Blue, Green, Pink, Purple, Red, Silver, Yellow and White. (Note : If it is said that A works with B, then it means A and B work in the same country. If it is said that A works with the one who likes ‘e’, then it means A and the one who likes ‘e’ work in the same country and also that A does not like ‘e’) O works in Canada with only the one who likes Black. M works with the one who likes Blue but not in Canada. The one who likes Green and the one who likes Red work in India. M does not like red. N likes yellow and does not work with the one who likes Blue. T works with the one who likes Red and the one who likes Pink. T does not like Green. R likes Purple and works with the one who likes Silver. L and S works in the same country. L neither likes Red nor Green. P works with the one who likes White. 141. Who amongst the following likes Blue? (1) T (2) L (3) P (4) Q (5) S 142. Which of the following colours does M like? (1) Yellow (2) White (3) Pink (4) None of the given options (5) Green

    143. Who amongst the following works with N? (1) M, L and S (2) Both R and Q (3) Both Q and the one who likes Green (4) Both S and the one who likes Red (5) Both P and the one who likes Blue 144. Which of the following statements is true as per the given arrangement? (1) The one who likes Red works with N. (2) P works with the one who likes Green. (3) Q and the one who likes Purple work in the same country. (4) O likes Silver (5) None of the given statements is true 145. Who amongst the following does not work in Mexico? (1) The one who likes Silver (2) R (3) Q (4) The one who likes Pink (5) The one who likes Yellow Directions (146–147) : Study the following information carefully and answer the questions given below : RBI Officer Grade 'B' Phase-I Exam, 06.03.2021

    Amongst five friends P, Q, R, S and T, each scored different marks in an examination out of a total of 100 marks. S scored more marks than only T. R scored less than P. Only one friend scored more than Q. The one who scored second highest marks scored 87 marks. 146. Who among the following scored marks less than only two friends? (1) P (2) Q (3) R (4) S (5) T 147. Who among the following did score the lowest marks ? (1) P (2) Q (3) R (4) S (5) T

    BPRE–736

    INSURANCE EXAMS Directions (1-4) : A company launches eight products– Q, R, S, T, U, V, W, Y, and Z in one of the four metros of India. The products were launched one after the other over the period of 6 months in 2006. The order in which the products were launched is consistent with the following conditions : (General Insurance Corporation AAO Exam. 11.12.2011)

    V is launched before both Y and Q Q gets launched after Z T gets launched before V but after R S gets launched after V R gets launched before W 1. Which one of the following could be true ? (1) Y is the second product to be launched (2) R is the third product to be launched (3) Q is the fourth product to be launched (4) S is the fifth product to be launched 2. If Z is the seventh product to be launched, then which one of the following could be true? (1) W is the fifth product to be launched (2) T is the fourth product to be launched (3) R is the second product to be launched (4) V is the 6th product to be launched 3. If Q is the 5th product to be launched then each of the following could be true except : (1) Z is the first product to be launched (2) T is the 2nd product to be launched (3) V is the 3rd product to be launched (4) W is the 4th product to be launched 4. If S is the fifth product to be launched, which of the following is correct? (1) Q is the second product to be launched (2) V is the seventh product to be launched (3) T is the sixth product to be launched (4) None of these

    PROBLEM SOLVING Directions (5-7) : Refer to the following table. Given below are the win loss histories of four tennis players in three set matches and number of matches played. (General Insurance Corporation AAO Exam. 11.12.2011)

    Won 2-0 Won 2-1 Lost 0-2 Lost 1-2

    P 23 13 8 6

    Q 18 16 13 8

    R 10 13 16 20

    S 16 14 13 17

    5. If a win carries an award of Rs.1500 and a loss of Rs.1000 which is the largest difference in prize money earned among the four ? (1) Rs. 7000 (2) Rs. 14000 (3) Rs. 5000 (4) Rs. 9000 6. As in the case above, what is the lowest average prize money won per match by any player ? (1) Rs. 1135 (2) Rs. 1300 (3) Rs. 1195 (4) Rs. 1150 7. As in the case above, what is the highest average prize money won per match by any player ? (1) Rs. 1495 (2) Rs. 1360 (3) Rs. 1460 (4) Rs. 1410 Directions (8-15) : (General Insurance Corporation AAO Exam. 11.12.2011)

    Follow the given Instructions : Each Item has a question followed by two statements. Mark (1) if the question can be answered by statement I alone. Mark (2) if the question can be answered with the help of II alone. Mark (3) if the question can be answered with the help of both the statements but not with the help of either statement itself. Mark (4) if the question cannot be answered with the help of both the statements. 8. The average of three quotations for a particular item is Rs.120. Is the highest quotation less than or equal to Rs. 139? I. The lowest quotation is of Rs.90. II. One of the quotation is Rs. 125. 9. How many people, read both Economic Times and Financial Express?

    I. Out of 300 readers, 200 read Financial Express, 220 read Economic Times and 50 read Indian Express. II. Out of total 300 readers 220 read Economic Times, 200 reads Financial Express and 50 read Neither. 10. A bus started from bus stop P, developed engine trouble and reached bus stop Q, 40 minutes late. What is the distance between the bus stops P and Q ? I. The engine trouble developed after travelling 40 kms from bus stop P and the speed reduced to

    1 th of the original 4

    speed. II. The engine trouble developed after travelling 40 kms from bus stop P in 2 hours and the speed reduced to

    1 th 4

    of the original speed. 11. What is the value of prime number x ? I. x + y is a two digit number > 50. II. y is a two digit number. 12. What is the Time difference in Nairobi and Mumbai ? I. The departure time at Nairobi is exactly 6 P.M. local time and the arrival time in Mumbai is next day 3 AM local time. II. Flight time is six hours. 13. Sujata is the eldest daughter. What is the current age of her father ? I. Current age of Sujata is 25 years and current age of her sister Sangeeta is 20 years. II. Sangeeta’s current age is

    1 rd of father’s age five years 3 before. 14. Alka has two sisters and one brother. How many sisters does Sandhya have ? I. Alka and Sandhya are Sisters. II. Sandhya has one brother. 15. Madhav likes car X because its cost Rs. 5,00,000 is cheapest in C segment and Raghav likes car Y because its cost

    BPRE–737

    Rs.7,00,000 is highest in C segment. Which vehicle will be selected by Keshav ? I. Keshav is looking for a car which is average representation of C segment. II. Keshav’s budget is around Rs. 5,50,000 16. ‘A’ went on tour on 15th August, which was the third Sunday of the month and came back on 30th September. So on which day of the month did he return ? (1) Fourth Sunday (2) Fourth Tuesday (3) Fifth Thursday (4) Third Saturday (Oriental Insurance Company Exam.08.04.2012)

    17. If Thursday was the day after the day before Yesterday five days ago, what is the least number of days ago when Sunday was three days before the day after tomorrow ? (1) Two (2) Three (3) Four (4) Five (Oriental Insurance Company Exam.08.04.2012)

    18. Which of the following expressions will not be true if the expression U< V < W < X < Y = Z > A = B > C is definitely true? (1) Z > C (2) U < Y (3) Y > V (4) Z < U (5) B < Y (United India Insurance AO Exam. 26.05.2013)

    Directions (19–23) : Study the following information carefully and answer the questions given below : (NIACL Administrative Officer (AO) Exam, 10.01.2015)

    Eight people J, K, L, M, N, O, P and Q live on separate floors of an 8– floor building. Ground floor is numbered 1, first floor is numbered 2 and so on until the topmost floor is numbered 8. L lives on floor number 5. Only one person lives between L and O. Q lives on an odd numbered floor. Only one person lives between Q and J. Only three people live between P and M. M lives below P. P does not live on the topmost floor. K neither lives immediately above Q nor immediately above O.

    PROBLEM SOLVING 19. How many people live between the floors on which P and J live ? (1) Three (2) Two (3) One (4) None (5) More than three 20. On which of the following floor numbers does K live ? (1) 8 (2) 2 (3) 4 (4) 3 (5) 6 21. Which of the following is true with respect to N as per the given information ? (1) N lives immediately above L. (2) Only five people live below N. (3) N lives on an odd numbered floor. (4) N lives on the topmost floor. (5) Only two people live between N and K. 22. Who among the following lives on floor number 1 ? (1) K (2) J (3) M (4) Q (5) P 23. Who lives on the floor immediately below O ? (1) P (2) Q (3) J (4) K (5) N Directions (24-28) : Study the following information carefully and answer the questions given below : (NIACL Administrative Officer (AO) Online Exam, 11.01.2015)

    Eight persons - S, T, U, V, W, X, Y and Z - live on separate floors of an eight storeyed building. Ground floor is numbered 1, the floor above it is numbered 2 and so on such that the topmost floor is numbered 8. ● T lives on floor number 6. ● W lives on an even numbered floor. ● W lives immediately below the floor of X. ● Only one person lives between S and Z. ● S lives below the floor of Z. ● U lives on the lowermost floor. ● S lives immediately below the floor of T. ● V does not live on the topmost floor. 24. On which of the following floor numbers does V live ? (1) 2 (2) 3 (3) 4 (4) 5 (5) 7

    25. How many persons live between the floors of T and W ? (1) Three (2) Two (3) One (4) Four (5) None 26. Which of the following statements is true with respect to T as per the given information? (1) T lives immediately above X (2) Only three persons live below T (3) T lives on an odd numbered floor (4) T lives immediatley below Z. (5) Only three perrons libe between T and U. 27. Who amongst the following lives on the floor immediately below Z? (1) Z (2) V (3) S (4) Y (5) T 28. Who amongst the following lives on the floor number 5 ? (1) V (2) S (3) X (4) U (5) W Directions (29-32) : Study the following information carefully and answer the questions given below : (NIACL Administrative Officer (AO) Online Exam, 12.01.2015)

    Six lectures are scheduled in a week starting from Monday and ending on Sunday of the same week. Computer Science is not on Tuesday or Saturday. Psychology is immediately after Organisational Behaviour. Statistics is not on Friday and there is one day gap between Statistics and Research Methods. One day prior to the schedule of Economics there is no lecture (as that day is the ‘off’ day and Monday is not the ‘off’ day). 29. Which of the following is the last lecture scheduled ? (1) Statistics (2) Research Methods (3) Psychology (4) Cannot be determined (5) None of these 30. How many lectures were scheduled between Economics and Psychology ? (1) One (2) Two (3) Three (4) Cannot be determined (5) None of these

    BPRE–738

    31. Which day is Computer Science scheduled ? (1) Monday (2) Wednesday (3) Thursday (4) Cannot be determined (5) None of these 32. If Wednesday was the ‘off’ day, the code would be 2–4. If Thursday was the ‘off’ day, the code would be 3–3. Taking account the ‘off’ day, which of the following code is correct ? (1) 2 – 4 (2) 3 – 3 (3) 4 – 2 (4) Cannot be determined (5) None of these Directions (33–37) : Study the following information carefully and answer the questions given below : (OICL Specialist Officer (Finance) Exam. 03.05.2015)

    Seven persons, namely A, B, C, D, E, F and G will be participating in seven different activities for annual college festival namely, Baking, Dancing, Singing, Painting, Acting, Running and Miming (not necessarily in the same order), from Monday to Sunday of the same week). F will participate on Wednesday. Only one person will participate between F and the one who will participate in Acting. The one participating in painting will participate immediately before the one participating in Acting. C will participate immediately before E. Neither C nor E will participate in Acting. Only two persons will participate between C and the one who will participate in Miming. The one who will participate in Baking will participate after C (May or may not be immediately after). B will participate immediately after the one who will participate in Running. Only two persons will participate between G and A. G will participate before A. B will not participate in Singing. 33. Four of the following five are alike in a certain way as per the given arrangement and hence form a group. Which of the following does not belong to that group? (1) E – Saturday (2) G – Monday (3) F – Tuesday (4) A – Wednesday (5) D – Thursday

    PROBLEM SOLVING 34. Which of the following pairs represent those who will participate immediately before and immediately after F? (1) Other than those given as options (2) A, C (3) B, D (4) B, A (5) G, C 35. As per the given arrangement, B is related to painting and D is related to Baking in a certain way. To which of the following is A related in the same way? (1) Running (2) Dancing (3) Other than those given as options (4) Acting (5) Miming 36. On which of the following days will G participate? (1) Monday (2) Saturday (3) Friday (4) Tuesday (5) Thursday 37. In which of the following activities will D participate? (1) Other than those given as options (2) Painting (3) Singing (4) Running (5) Acting Directions (38–41) : Study the following information carefully and answer the questions given below : (LIC Assistant Administrative Officer (AAO) Online Exam. 05.03.2016)

    Eight persons — B, C, D, E, F, G, H and I — live on eight different floors of a building but not necessarily in the same order. The lowermost floor of the building is numbered one, the floor above that is numbered two and so on till the topmost floor is numbered eight. D lives on the floor numbered 5. Only two persons live between D and I. B lives immediately below I. Only four persons live between B and C. H lives immediately above C but not on the topmost floor. F lives on an odd numbered floor. More than two persons live between F and E. 38. Four of the following five are alike in a certain way based on the above arrangement and hence form a group. Which one of them does not belong to the group ? (1) GF (2) EC (3) ID (4) BC (5) HF 39. How many persons live between I and C ? (1) One (2) Three (3) More than three (4) Two (5) None

    40. If all the persons are made to live in alphabetical order from bottom to top, the positions of how many persons will remain unchanged ? (1) More than three (2) None (3) One (4) Two (5) Three 41. Who amongst the following lives immediately below E ? (1) D (2) C (3) G (4) I (5) H Directions (42–43) : Study the following information carefully and answer the questions given below : (LIC Assistant Administrative Officer (AAO) Online Exam. 05.03.2016)

    Five persons namely, A, B, C, D and E buy different number of cookies. C buys more cookies than both B and E. D buys more cookies than B but less than A and E. A does not buy the maximum number of cookies. E does not buy the third highest number of cookies. The one who buys the maximum number of cookies buys 20 cookies. 42. Who amongst the following buys the second lowest number of cookies ? (1) B (3) E

    (2) D (4) C

    (5) A 43. If the difference between the cookies bought by C and A is 8, then which of the following may possibly represent the number of cookies bought by E ? (1) 12 (2) 8 (3) 15 (4) 5 (5) 10 Directions (44–48) : Study the following information carefully and answer the questions given below : (LIC Assistant Administrative Officer (AAO) Online Exam. 05.03.2016)

    Seven persons, namely, A, B, C, D, E, F and G like seven different colours namely, Blue, Green, Orange, Pink, Silver, Red and Yellow, but not necessarily in the same order. They will also attend a wedding, not necessarily in the same order, from Monday to Sunday (of the same week.) E will attend a wedding on Wednesday. Only one person will attend wedding between E and the one who likes pink. Only two persons will attend a wedding between the one who

    BPRE–739

    likes pink and the one who likes yellow. Only three persons will attend a wedding between the one who likes Pink and C. B will attend a wedding immediately after the one who likes pink. Only one person will attend a wedding between B and A. D will attend a wedding immediately before F. Only one person will attend a wedding between F and the one who likes green. F likes neither blue nor orange. The one who likes silver will attend a wedding immediately before the one who likes blue. 44. On which of the following days will G attend a wedding ? (1) Monday (2) Tuesday (3) Saturday (4) Thursday (5) Friday 45. Which of the following colours does D like ? (1) Silver (2) Blue (3) Pink (4) Orange (5) Red 46. Four of the following five are alike in a certain way as per the given arrangement and hence form a group. Which of the following does not belong to that group ? (1) A–Pink (2) D–Blue (3) C–Yellow (4) C–Silver (5) E–Orange 47. As per the given arrangement A is related to Saturday and C is related to Sunday in a certain way. To which of the following is B related to in the same way ? (1) Wednesday (2) Tuesday (3) Friday (4) Monday (5) Thursday 48. Which of the following pairs represent those who will attend a wedding immediately before and immediately after E? (1) D, A (2) B, A (3) A, D (4) C, G (5) G, A Directions (49–52) : Study the following information carefully and answer the questions given below : (LIC Assistant Administrative Officer (AAO) Online Exam. 06.03.2016)

    Eight persons J, K, L, M, N, O, P and Q live on eight different floors of a building, but not necessarily in the same order. The lowermost floor of the building is numbered one, the one above that is numbered two and so on till the topmost floor is numbered eight.

    PROBLEM SOLVING P lives on the floor numbered 3. Only one person lives between J and P. Only three persons live between M and J. O lives immediately above M. There are as many people between O and J as there are between P and Q. K lives on an even numbered floor. Only three persons live between K and L. K lives on any floor above L. 49. Four of the following five are alike in a certain way and hence form a group. Which one of them does not belong to the group? (1) LP (2) OK (3) MO (4) KN (5) NQ 50. Who amongst the following lives immediately below N? (1) Q (2) K (3) P (4) O (5) No one 51. If all the people are made to live in alphabetical order from bottom to top, the positions of how many people will remain unchanged? (1) Two (2) One (3) None (4) Three (5) More than three 52. How many people live between Q and P? (1) One (2) Three (3) None (4) Two (5) More than three Directions (53–54) : Study the following information carefully and answer the questions given below : (LIC Assistant Administrative Officer (AAO) Online Exam. 06.03.2016)

    Five persons A, B, C, D and E won different number of competitions. Only two persons won more number of competitions than E. B won more competitions than D but less than A. C won more competitions than D. C neither won the maximum number of competitions nor second lowest number of competitions. The one who won the second lowest number of competitions won 8 competitions. 53. If the difference between the number of competitions won by B and A is 16, then which of the following may possibly represent the number of competitions won by E? (1) 5 (2) 25 (3) 7 (4) 14 (5) 23

    54. Who among the following won the second highest number of competitions? (1) C (2) E (3) D (4) B (5) A Directions (55–59) : Study the following information carefully and answer the questions given below : (LIC Assistant Administrative Officer (AAO) Online Exam. 06.03.2016)

    Seven persons, namely M, N, O, P, Q, R and S like seven different cars namely, Volkswagen, Audi, BMW, Mercedes, Nissan, Jaguar and Fiat but not necessarily in the same order. They will also attend a screening not necessarily in the same order, from Monday to Sunday (of the same week). M will attend a screening on Friday. Only one person will attend a screening between M and the one who likes Jaguar. Only two persons will attend a screening between R and the one who likes Jaguar. The one who likes Nissan will attend a screening immediately before N. Only two persons will attend a screening between the one who likes Nissan and O. O does not like Jaguar. Only one person will attend a screening between the one who likes Mercedes and O. Q does not like Nissan. S will attend the screening immediately before Q. Only two persons will attend a screening between Q and the one who likes Audi. Only one person will attend a screening between the one who likes Volkswagen and the one who likes BMW. The one who likes BMW will attend the screening before the one who likes Volkswagen. N does not like Audi. 55. Which of the following pairs represent the persons who attend screening immediately before and immediately after M? (1) R, P (2) N, O (3) N, P (4) Q, O (5) Q, N 56. Four of the following five are like as per the given arrangement and hence form a group. Which of the following does not belong to that group? (1) M-BMW (2) Q-Audi (3) S-Mercedes (4) P-Fiat (5) R-Jaguar

    BPRE–740

    57. On which of the following days will N attend a screening? (1) Tuesday (2) Monday (3) Wednesday (4) Thursday (5) Sunday 58. Which of the following cars does P like? (1) Audi (2) Nissan (3) BMW (4) Fiat (5) Cannot be determined 59. As per the given arrangement S is related to Thursday and R is related to Sunday in a certain way. To which of the following is Q related to in the same way? (1) Saturday (2) Monday (3) Wednesday (4) Friday (5) Tuesday Directions (60–63) : Study the following information carefully and answer the questions given below : (UIICL A.O. Exam 12.06.2016)

    Seven different cars- Kwid, Alto, i10, Duster, Corolla, Polo and XUV were sold on different days of the same week, starting from Monday and ending on Sunday, but not necessarily in the same order. Duster was sold on one of the days before Thursday. Only four cars were sold between Duster and XUV. Only one car was sold between XUV and Kwid. Polo was sold on one of the days after Kwid but not on Friday. Only four cars were sold between Polo and Alto. Corolla was sold neither on Wednesday nor on Thurdsay. 60. Which of the following cars were sold before i10 ? (1) Both Duster and Alto (2) Only Alto (3) Both Corolla and Polo (4) Both XUV and Kwid (5) Only Kwid 61. On which of the following days was Kwid sold ? (1) Friday (2) Thursday (3) Monday (4) Saturday (5) Wednesday 62. Four of the following five are alike in a certain way and hence form a group. Which one of the following does not belong to the group ? (1) Corolla-Wednesday (2) Duster-Wednesday (3) Polo-Friday (4) Kwid-Saturday (5) Alto-Sunday

    PROBLEM SOLVING 63.

    How many cars were sold between i10 and Corolla ? (1) None (2) Two (3) One (4) More than three

    Directions (69–73) : Study the following information carefully and answer the questions given below :

    (5) Three

    Seven persons A, B, C, D, E, F and G were appointed to a company on seven different days of the same week starting from Monday to Sunday (but not necessarily in the same order). Each person also plays a different game namely- Cricket, Hockey, Football, Squash, Volleyball, Tennis and Kho-Kho, but not necessarily in the same order. Only two persons were appointed after the one who plays Hockey. E was appointed on one of the days after the one who plays Hockey. Only three persons were appointed between E and G. Only one person was appointed between G and the one who plays Volleyball. A was appointed immediately after the one who plays Volleyball. Only three persons were appointed after the one who plays Kho-kho.C was appointed immediately after F, but not on Friday. Only two persons were appointed between F and the one who plays Cricket. B was appointed immediately before one who plays Cricket. More than two persons were appointed between D and the one who plays Tennis. C does not play Football. 69. Who amongst the following was appointed on Wednesday ? (1) The one who plays Kho-kho (2) A (3) B (4) The one who plays Football (5) F 70. Which of the following statements is true as per the given arrangement ? (1) Only one person was apppointed between F and the one who plays Squash. (2) Only three persons were appointed before C. (3) A plays Tennis. (4) B was appointed on Saturday. (5) None of the given options is true. 71. The person who plays Tennis was appointed on which of the following days ? (1) Thursday (2) Tuesday (3) Sunday (4) Monday (5) Saturday

    Directions (64–68) : Study the following information carefully and answer the questions given below : (UIICL A.O. Exam 12.06.2016)

    Eight persons J, K, L, M, N, O, P and Q live on eight different floors of a building but not necessarily in the same order. Ground floor is numbered 1, first floor is numbered 2 and so on until the topmost floor is numbered 8. ●

    K lives on floor numbered three.



    Only one person lives between







    64.

    65.

    66.

    67.

    68.

    K and Q. P lives immediately above J. P lives on an odd numbered floor. Only two persons live between N and Q. N lives above Q. O lives immediately above L. Which of the following statements is true with respect to the given information ? (1) None of the given statements is true (2) Only two persons live between P and M. (3) L lives on an even numbered floor. (4) J lives immediately above O. (5) Only one person lives between K and O. Who lives on the floor immediately below M ? (1) O (2) P (3) K (4) N (5) Other than those given as options How many persons live between the floors on which P and K live ? (1) Two (2) One (3) None (4) Three (5) More than three Who amongst the following lives on floor number 5 ? (1) N (2) L (3) M (4) P (5) Q Who amongst the following lives between J and M ? (1) Q (2) No one (3) P (4) K (5) N

    (UIICL A.O. Exam 12.06.2016)

    BPRE–741

    72.

    Which of the following combinations is not true as per the given arrangement ? (1) C – Squash (2) Thursday–G (3) Saturday–Squash (4) Monday–Volleyball (5) F–Hockey 73. How many persons were appointed before G ? (1) Two (2) One (3) Three (4) Four (5) None, as G was appointed on Monday Directions (74–78) : Study the following information carefully and answer the questions given below : (UIICL A.O. Exam 12.06.2016)

    Eight executives B, G, H, K, D, F, T and V are working in three departments Engineering, Systems and Marketing of the organization and are posted at different places viz. Chennai, Kozikode, Kolkata, Ranchi, Patna, Bhopal, Nagpur and Hyderabad not necessarily in the same order. At least two and not more than three executives work in any of the three departments. G works in Engineering department at Chennai. H is posted in Ranchi but not in Systems department. No one from Marketing department is posted in Hyderabad. The only other person in same department as that of G is posted in Kolkata. D is posted in Hyderabad and F in Kozikode. V is not posted in Kolkata and works in the same department as that of D. B and T both work in Marketing department. The one who works in Marketing department is not posted in Bhopal. T is not posted in Nagpur. 74. T is posted at which place ? (1) Nagpur (2) Patna (3) Bhopal (4) Cannot be determined (5) Other than those given as options 75. Who is posted in Kolkata ? (1) K (2) T (3) V (4) K or T (5) Other than those given as options 76. Which department has only two Executives ? (1) Systems (2) Marketing (3) Engineering (4) Marketing or Systems (5) Cannot be determined

    PROBLEM SOLVING 77.

    Which of the following group of persons work in Marketing department ? (1) KBT (2) BTF (3) BHD (4) BHT (5) Other than those given as options 78. Which of the following combinations of department, person and place is correct ? (1) Marketing – B – Bhopal (2) Engineering – G – Kolkata (3) Systems – V – Chennai (4) Systems – T – Patna (5) Other than those given as options Directions (79-83) : Study the following information carefully and answer the questions given below : (NICL A.O. Exam 05.06.2017)

    Seven persons, Maya, Pavan, Amit, Arvind, Raghu, Rajeev and Ashok live on different floors of the same building. The lowermost floor of the building is numbered 1, the floor immediately above that is numbered 2 and so on till the topmost floor is numbered 7. Arvind lives on an even number floor. Maya does not live above Arvind’s floor. Pavan lives three floors above Rajeev’s floor. Only one person is living between the floor on which Rajeev and Arvind are living. Ashok lives on an odd numbered floor. Rajeev does not live below Arvind. Amit lives on the floor immediately above Ashok’s floor. 79. Who lives on floor number 3 ? (1) Maya (2) Raghu (3) Rajeev (4) Amit (5) Ashok 80. Who is living immediately above Raghu’s floor ? (1) Maya (2) Ashok (3) Amit (4) Pavan (5) Rajeev 81. Who lives on the topmost floor ? (1) Amit (2) Ashok (3) Pavan (4) Raghu (5) Maya 82. On which floor is Amit living ? (1) 7 (2) 3 (3) 5 (4) 4 (5) 6

    83. If all the people are arranged in the same order as they would appear in the dictionary, starting from top floor to bottom floor, position of how many person(s) will remain unchanged after the rearrangement ? (1) None (2) Three (3) Two (4) One (5) More than three Directions (84-88) : Study the following information carefully and answer the questions given below : (NICL A.O. Exam 05.06.2017)

    Seven students — P, Q, R, S, T, U and W — start taking class from Monday to Sunday of the same week but not necessarily in the same order. T is taking class one of the day before Wednesday. There are only three persons taking class between T and P. Neither Q nor R is taking class on Wednesday. As many students are taking class between P and S as the students taking between S and Q. R has taken class immediately before the day on which U is taking the class. 84. How many students are taking class after W ? (1) One (2) Three (3) Two (4) Cannot be determined (5) Other than those given as options 85. How many students were taking class between T and W ? (1) None (2) One (3) Two (4) Three (5) More than three 86. S was taking class immediately after which day ? (1) Monday (2) Tuesday (3) Wednesday (4) Friday (5) Saturday 87. Who among the following is taking class on Wednesday ? (1) P (2) T (3) W (4) S (5) U 88. Who is taking class exactly between U and Q ? (1) P (2) S (3) T (4) R (5) None Directions (89–90) : Study the following information carefully and answer the questions given below : (GIC Officer Online Exam 15.05.2017)

    Five friends A, J, M, P and Q each scored different marks in an examina-

    BPRE–742

    tion. P scored more than Q but less than A. M scored more than A. J scored less marks than P but J did not score the minimum marks. A scored 86% marks and the one who scored minimum marks scored 70% marks. 89. Who amongst the five friends is most likely to have scored 92% marks ? (1) M (2) P (3) J (4) Q (5) Either P or M 90. How many persons scored more than P? (1) One (2) Two (3) Three (4) Four (5) Cannot be determined Directions (91–95) : Study the following information carefully and answer the questions given below : (GIC Officer Online Exam 15.05.2017)

    Eight persons – A, B, C, D, E, F, G and H – were born in eight different months of a year viz. January, February, April, July, September, October, November and December (but not necessarily in the same order). Each person also likes a different colour namely — Red, Pink, Yellow, Blue, Green, Silver, White and Orange but not necessarily in the same order. (Note : No other person was born in any other month of the given year.) D was born in a month having 31 days after July. Only three people were born between D and the one who likes Yellow. F was born in a month having 30 days immediately before the one who likes Blue. D does not like Blue. F was not born in November. Only two people were born between the ones who like Blue and White. The one who likes White was born in one of the months before the one who likes Blue. Only three people were born between the one who likes White and C. A was born in a month having 30 days. As many people were born before A as after G. Only four people were born between G and the one who likes Green. B was born immediately after the one who likes Orange. E was born in one of the months before B. The one who likes Red was born immediately before the one who likes Silver. 91. H was born in which of the following months? (1) April (2) July (3) February (4) September (5) December

    PROBLEM SOLVING 92. How many people were born between E and the one who likes Orange? (1) None (2) Three (3) One (4) Four (5) Two 93. Yellow is related to F and Silver is related to A in a certain way based on the given arrangement. To which of the following is Blue related, following the same way? (1) The one who likes Orange (2) B (3) G (4) The one born in September (5) H 94. Which of the following combinations is definitely true as per the given arrangement? (1) January–F (2) G–Yellow (3) C–November (4) July–Pink (5) B–White 95. Which of the following statements is TRUE as per the given ar rangement? (1) D was born immediately before the one who likes Green. (2) Only two people were born after G. (3) C likes Pink colour. (4) None of the given options is true. (5) Only two people were born between H and B. Directions (96–97) : Study the following information carefully and answer the questions given below : (GIC Officer Online Exam 15.05.2017)

    Six strings viz. S, T, U, V, W-and Y, each of a different length are kept on a table. T is longer than U and V but shorter than Y. S is longer than Y but shorter than W. The second longest string is 42m long. The third shortest string is 22m long. (Note: The length of all the strings is in whole numbers.) 96. If V is 21 metre long, then which of the following is true about V? (1) V is the third longest of all strings. (2) V is longer than U. (3) The total length of V and Y together is 65 metre. (4) All the given statements are true. (5) V is 30 metre shorter than S.

    97. If W is 14 metre longer than S, then how long is W? (1) 48 metre (2) 56 metre (3) 26 metre (4) Cannot be determined (5) 36 metre Directions (98–102) : Study the following information carefully and answer the questions given below : (GIC Officer Online Exam 15.05.2017)

    Eight people – M, N, O, P, Q, R, S and T — live on eight different floors of a building, but not necessarily in the same order. The lowermost floor of the building is numbered one, the one above that is numbered two and so on till the topmost floor is numbered eight. Each of the persons visits a different city namely, Agra, Mathura, Jaipur, Surat, Jhansi, Delhi, Kolkata and Patna but not necessarily in the same order. P lives on floor number four. Only two people live between P and the one who visits Jaipur. As many people live above the one who visits Jaipur as below the one who visits Delhi. Only four people live between the one who visits Delhi and T. The one who visits Agra lives on an even numbered floor immediately above M. M does not visit Jaipur. P does not visit Agra. Only one person lives between M and R. Only three people live between the ones who visit Jhansi and Kolkata. The one who visits Jhansi lives on one of the floors above the one who visits Kolkata. The one who visits Kolkata does not live on the lowermost floor. The one who visits Mathura lives immediately above S. O lives on one of the floors below S. O does not visit Patna. Q does not live on the topmost floor. 98. How many people live above N? (1) One (2) More than three (3) Three (4) Two (5) None 99. Who amongst the following visits Surat? (1) R (2) O (3) S (4) T (5) N 100. Four of the following five are alike in a certain way as per the given arrangement and so form a group. Which of the following does not belong to the group?

    BPRE–743

    (1) P–Floor number 2 (2) M–Floor number 8 (3) S–Floor number 5 (4) Q–Floor number 3 (5) T–Floor number 4 101. Which of the following statements is true as per the given arrangement? (1) S lives on floor number two. (2) Only one person lives between Q and N. (3) R lives immeditaley below P. (4) All the given options are true. (5) O lives on the lowermost floor. 102. Agra is related to P and Kolkata is related to S in a certain way based on the given arrangement. To which of the following is Jhansi related, following the same pattern? (1) O (2) M (3) Q (4) The one who visits Jaipur (5) The one who lives on floor number three Directions (103– 107) : Study the following information carefully and answer the questions given below : (LIC AAO Prelim Exam. 04.05.2019)

    Six teachers i.e., P, Q, R, S, T and U attend seminar on two different dates 25th and 28th of three given months viz. January, April and July. They all attend the seminar of different subjects i.e., Physics, Economics, History, Chemistry, Biology and Geography, but not necessarily in the same order. T attends the seminar in the month having only 30 days. Only one person attends the seminar between T and the one who attends seminar of the Geography. Only three teachers attend the seminar between History and Geography. Number of persons attending the seminar before Q is same as the number of persons attending the seminar after the one who attends seminar of History. Only two persons attend the seminar between Q and the one who attends the seminar of Physics. R attends the seminar of Biology. U attends the seminar immediately after S. S does not attend the seminar in the month having less than 31 days. Q and P do not attend the seminar of Economics and Geography.

    PROBLEM SOLVING 103. Who among the following teacher attends the seminar on 25th of April? (1) Q (2) U (3) S (4) T (5) None of these 104. Who among the following teacher attends seminar immediately after T? (1) R (2) U (3) S (4) P (5) None of these 105. How many teachers attend seminar between S and P? (1) One (2) Two (3) Four (4) Three (5) None 106. Q attends seminar on which of the following date and month? (1) 25 July (2) 28 January (3) 25 January (4) 28 July (5) None of these 107. Which of the following teachers’ pair attends seminar in the same month? (1) S and U (2) P and U (3) U and R (4) T and P (5) Q and S Directions (108–112) : Study the following information carefully and answer the questions given below : (LIC AAO Prelim Exam. 04.05.2019)

    Seven persons i.e. P, Q, R, S, T, V and W are buying different items one after another but not necessarily in the same order. Only one person buys item between Q and R. Only three persons buy items between S and Q. T buys item immediately before V. S buys before T. More than two persons are buying items between T and S. W buys before P. 108. How many persons buy items before R? (1) None (2) One (3) Two (4) Three (5) More than three 109. Who among the following buys items just before T? (1) P (2) Q (3) R (4) W (5) None of these

    110. Number of persons buying items before W is same as the number of persons who buys items after which of the following? (1) T (2) V (3) P (4) Q (5) S 111. Which of the following statements is true regarding W? (1) No one buys items before W. (2) W buys before S. (3) Two persons buy items between W and Q. (4) One person buys between R and W. (5) None is true 112. Four of the following five are alike in a certain way based on the given arrangement. Which is the one that does not belong to that group? (1) T-P (2) V-Q (3) P-W (4) Q-T (5) S-R Directions (113–117) : Study the following information carefully and answer the questions given below : (LIC AAO Prelim Exam. 05.05.2019)

    A certain number of people are standing in a queue outside a ticket counter, standing one behind the other (all facing the same direction). Each one of them has to buy a different number of tickets. B is standing immediately behind G. Only three people are standing between G and K. The one who has to buy 12 tickets is standing immediately behind K. Only three people are standing between M and the one who has to buy 12 tickets. The one who has to buy 15 tickets is standing at one of the positions in front of M. Only one person is standing between M and the one who has to buy 15 tickets. Only two people are standing between the one who has to buy 15 tickets and the one who has to buy 8 tickets. The one who has to buy 8 tickets is standing third from one of the extreme ends of the queue. Only eight people are standing between P and the one who has to buy 8 tickets. No one stands between V and P. The difference between the number of tickets that V and P have to buy is 3. Only four people

    BPRE–744

    are standing between X and V. X is standing at an extreme end of the queue. The number of people standing between X and K is same as that standing between K and the one who has to buy 20 tickets. X does not have to buy 20 tickets. 113. What is the position of M from the beginning of the queue? (1) Fifth (2) Thirteenth (3) Eighth (4) Tenth (5) Sixth 114. If M has to buy 8 more tickets than V, then how many tickets does M have to buy? (1) Cannot be determined (2) 23 (3) 16 (4) 17 (5) 18 115. How many people are standing in the given queue? (1) 24 (2) 14 (3) 21 (4) 16 (5) 18 116. How many people are standing between the one who has to buy 20 tickets and the one who has to buy 12 tickets? (1) Four (2) Twelve (3) None (4) Seven (5) Nine 117. If C is standing sixth from one of the extreme ends of the queue, then how many tickets do C and P together have to buy? (1) Cannot be determined (2) 27 (3) 28 (4) 24 (5) 20 Directions (118–122) : Study the following information carefully and answer the questions given below : (LIC AAO Prelim Exam. 05.05.2019)

    Nine people viz., L, M, N, O, P, Q, R, S and T work in either of the three cities viz., Mumabi, Delhi and Pune. Atleast two and not more than four people work in each city. (Note : If it is said that a works with b, then a and b work in the same city) O works in Delhi with only S. Q neither works with S nor M. Neither T nor L works in Mumbai. N does not work in the same city in which L works. R neither works with M nor N. More than two people work in Mumbai.

    PROBLEM SOLVING 118. Four of the following five are alike in a certain way as per the given arrangement and thus they form a group. Which one of the following does not belong to that group? (1) N–P (2) Q–L (3) M–N (4) S–M (5) R–T 119. Who amongst the following work in Pune? (1) R, Q, T and L (2) Q,T and P (3) R and P (4) L and M (5) M, P and N 120. Who amongst the following works with P? (1) Only N (2) Both Q and N (3) Both M and N (4) Both M and L (5) Only R 121. Which of the following statements is true as per the given arrangement? (A) L works in Mumbai (B) P and T work in different cities. (C) R works with both T and S. (1) Only A (2) Only B (3) All A, B and C (4) Only A and C (5) Only B and C 122. In the given arrangement, if O and R interchange their city of work and so do P and S, then who amongst the following works with O after interchanging the city of work? (1) S (2) M (3) P (4) N (5) Q Directions (123–127) : Study the following information carefully and answer the questions given below : (LIC AAO Main Exam, 28.06.2019)

    Eight friends S, F, M, N, L, A, J and V were born in different months from March to October in the same year on different dates but not necessarily in the same order. J was born before N and after V but not immediately after V. The one who

    was born in May has hig birthday on 21st. One of the friends was born on 7th August. Only two persons were born between S and F. Only two persons were born between L and V. The person born in July has his birthday on 14th. M is elder than S but younger than L. V is younger than S but elder than A. One of the friends was born on 5th September. N was born on 9th October. One of the friends was born on 11th March. M was born on 8th of the month while V was born on 18th of the month. 123. Who among the following was born on 7th of the month ? (1) M (2) A (3) L (4) J (5) F 124. Who among the following was born in June ? (1) J (2) L (3) S (4) V (5) A 125. How many friends were born between L and A ? (1) Two (2) Three (3) Four (4) Five (5) More than five 126. By how many days F is elder than N? (1) 63 days (2) 33 days (3) 41 days (4) 28 days (5) 83 days 127. Who among the following was born in March ? (1) J (2) F (3) L (4) S (5) A Directions (128–132) : Study the following information carefully and answer the questions given below : (LIC AAO Main Exam, 28.06.2019)

    Nine friends L, R, M, B, K, U, T, S and D are sitting in a row having nine chairs to watch a movie. All of them are facing north and chairs are arranged from left to right serially from 1 to 9. Each of them likes different colours viz., Yellow, Pink, Golden, Silver, Red, Brown, Blue, Green and White. (Note : None of the given information is necessarily in the same order).

    BPRE–745

    R sits to the immediate right of the one who likes Yellow colour. K likes Silver colour and is sitting second to the right of the one who likes Red colour. S sits to the immediate left of T who likes Golden or White colour. L sits exactly between U and R. D does not like Pink or Yellow colour. There is only one person to the left of the one who likes Golden colour. The one who likes Blue colour sits third to the left of T. The one who likes Pink colour sits third to the right of M. B is sitting exactly between the persons who like Silver and Green colours. There is only one person to the right of S who likes Brown or Pink colour. 128. Who among the following is sitting third to the right of a person who likes Pink colour? (1) L (3) D

    (2) R (4) S

    (5) U 129. Who among the following likes Golden and Blue colours ? (1) M and L (2) R and B (3) U and D (5) D and S

    (4) L and T

    130. Which of the following is the favourite colour of the person who is sitting 5th from the right end? (1) Pink (2) Blue (3) Yellow (5) Silver

    (4) Green

    131. How many persons are sitting exactly between the persons who like Golden and Green colours? (1) Two (3) Four

    (2) Three (4) Five

    (5) Six 132. If U likes Brown colour, then who among the following would like Pink colour ? (1) S (2) L (3) D (5) M

    (4) B

    Must Read

    Buy Today

    Kiran’s

    ONE LINER APPROACH GENERAL KNOWLEDGE

    PROBLEM SOLVING

    SHORT ANSWERS

    NATIONALISED BANKS & IBPS PO/MT/SO EXAMS 1. (5) 5. (3) 9. (4) 13. (4) 17. (3) 21. (4) 25. (2) 29. (3) 33. (2) 37. (3) 41. (5) 45. (3) 49. (5) 53. (5) 57. (3) 61. (2) 65. (4) 69. (3) 73. (3) 77. (1) 81. (4) 85. (4) 89. (3) 93. (5) 97. (3) 101. (1) 105. (3) 109. (2) 113. (5) 117. (4) 121. (4) 125. (4) 129. (3) 133. (4) 137. (2) 141. (2) 145. (4) 149. (3) 153. (3) 157. (4) 161. (3) 165. (5) 169. (2) 173. (3) 177. (1)

    2. (5) 6. (5) 10. (3) 14. (3) 18. (1) 22. (1) 26. (2) 30. (2) 34. (4) 38. (1) 42. (3) 46. (2) 50. (2) 54. (2) 58. (1) 62. (3) 66. (4) 70. (5) 74. (1) 78. (2) 82. (1) 86. (5) 90. (4) 94. (1) 98. (2) 102. (3) 106. (4) 110. (1) 114. (3) 118. (1) 122. (3) 126. (5) 130. (2) 134. (3) 138. (4) 142. (1) 146. (3) 150. (1) 154. (1) 158. (1) 162. (1) 166. (4) 170. (5) 174. (5) 178. (2)

    3. (5) 7. (1) 11. (4) 15. (5) 19. (4) 23. (3) 27. (1) 31. (2) 35. (5) 39. (3) 43. (1) 47. (4) 51. (1) 55. (2) 59. (5) 63. (1) 67. (2) 71. (2) 75. (5) 79. (5) 83. (1) 87. (1) 91. (5) 95. (2) 99. (4) 103. (4) 107. (2) 111. (4) 115. (3) 119. (3) 123. (5) 127. (4) 131. (1) 135. (5) 139. (5) 143. (2) 147. (1) 151. (4) 155. (5) 159. (4) 163. (4) 167. (1) 171. (3) 175. (1) 179. (4)

    4. (3) 8. (1) 12. (2) 16. (2) 20. (5) 24. (3) 28. (4) 32. (5) 36. (1) 40. (4) 44. (4) 48. (5) 52. (3) 56. (4) 60. (1) 64. (5) 68. (1) 72. (4) 76. (4) 80. (5) 84. (2) 88. (3) 92. (4) 96. (1) 100. (5) 104. (2) 108. (1) 112. (1) 116. (1) 120. (1) 124. (2) 128. (5) 132. (2) 136. (1) 140. (3) 144. (1) 148. (5) 152. (2) 156. (2) 160. (3) 164. (2) 168. (3) 172. (2) 176. (4) 180. (3)

    181. (5) 185. (2) 189. (3) 193. (5) 197. (1) 201. (4) 205. (1) 209. (3) 213. (1) 217. (1) 221. (2) 225. (4) 229. (4) 233. (1) 237. (5) 241. (3) 245. (2) 249. (2) 253. (4) 257. (4) 261. (5) 265. (5) 269. (3) 273. (3) 277. (4) 281. (2) 285. (3) 289. (4) 293. (1) 297. (3) 301. (1) 305. (5) 309. (1) 313. (5) 317. (3) 321. (4) 325. (2) 329. (2) 333. (1) 337. (3) 341. (4) 345. (2) 349. (4) 353. (1) 357. (2) 361. (3) 365. (1) 369. (4) 373. (1) 377. (3) 381. (3) 385. (1)

    182. (3) 186. (5) 190. (2) 194. (3) 198. (2) 202. (1) 206. (3) 210. (3) 214. (3) 218. (4) 222. (5) 226. (2) 230. (5) 234. (5) 238. (4) 242. (1) 246. (3) 250. (3) 254. (3) 258. (2) 262. (4) 266. (3) 270. (1) 274. (2) 278. (1) 282. (1) 286. (4) 290. (2) 294. (5) 298. (3) 302. (5) 306. (2) 310. (5) 314. (2) 318. (3) 322. (5) 326. (3) 330. (4) 334. (2) 338. (2) 342. (1) 346. (5) 350. (3) 354. (5) 358. (4) 362. (4) 366. (2) 370. (4) 374. (4) 378. (5) 382. (4) 386. (4)

    183. (1) 187. (5) 191. (4) 195. (1) 199. (5) 203. (4) 207. (3) 211. (2) 215. (4) 219. (3) 223. (1) 227. (1) 231. (3) 235. (4) 239. (2) 243. (5) 247. (4) 251. (2) 255. (1) 259. (1) 263. (2) 267. (2) 271. (1) 275. (5) 279. (3) 283. (5) 287. (2) 291. (5) 295. (2) 299. (2) 303. (4) 307. (1) 311. (4) 315. (1) 319. (2) 323. (1) 327. (4) 331. (1) 335. (4) 339. (5) 343. (3) 347. (1) 351. (5) 355. (2) 359. (5) 363. (3) 367. (2) 371. (5) 375. (2) 379. (5) 383. (2) 387. (2)

    BPRE–746

    184. (4) 188. (1) 192. (2) 196. (4) 200. (3) 204. (4) 208. (2) 212. (2) 216. (5) 220. (5) 224. (3) 228. (2) 232. (3) 236. (2) 240. (5) 244. (1) 248. (4) 252. (5) 256. (1) 260. (3) 264. (3) 268. (2) 272. (2) 276. (4) 280. (5) 284. (1) 288. (1) 292. (3) 296. (4) 300. (4) 304. (3) 308. (3) 312. (2) 316. (4) 320. (1) 324. (5) 328. (3) 332. (5) 336. (5) 340. (3) 344. (4) 348. (5) 352. (2) 356. (4) 360. (1) 364. (5) 368. (1) 372. (3) 376. (2) 380. (1) 384. (3) 388. (5)

    389. (4) 393. (3) 397. (2) 401. (4) 405. (3) 409. (5) 413. (3) 417. (4) 421. (3) 425. (2) 429. (1) 433. (2) 437. (4) 441. (1) 445. (2) 449. (5) 453. (2) 457. (4) 461. (1) 465. (2) 469. (4) 473. (1) 477. (5) 481. (3) 485. (4) 489. (5) 493. (4) 497. (2) 501. (3) 505. (5) 509. (5) 513. (4) 517. (2) 521. (1) 525. (4) 529. (1) 533. (2) 537. (3) 541. (3) 545. (1) 549. (4) 553. (5) 557. (3) 561. (5) 565. (5) 569. (3) 573. (4) 577. (2) 581. (1) 585. (4) 589. (1)

    390. (5) 394. (3) 398. (1) 402. (2) 406. (4) 410. (4) 414. (5) 418. (5) 422. (1) 426. (3) 430. (4) 434. (3) 438. (3) 442. (4) 446. (3) 450. (3) 454. (2) 458. (1) 462. (3) 466. (3) 470. (4) 474. (2) 478. (1) 482. (4) 486. (3) 490. (2) 494. (5) 498. (4) 502. (1) 506. (2) 510. (2) 514. (5) 518. (2) 522. (3) 526. (1) 530. (4) 534. (4) 538. (4) 542. (4) 546. (3) 550. (3) 554. (2) 558. (3) 562. (2) 566. (1) 570. (2) 574. (1) 578. (4) 582. (5) 586. (1) 590. (5)

    391. (1) 395. (5) 399. (3) 403. (1) 407. (1) 411. (2) 415. (1) 419. (2) 423. (4) 427. (5) 431. (2) 435. (3) 439. (4) 443. (5) 447. (1) 451. (1) 455. (5) 459. (3) 463. (5) 467. (1) 471. (3) 475. (4) 479. (2) 483. (5) 487. (2) 491. (3) 495. (1) 499. (5) 503. (3) 507. (1) 511. (1) 515. (2) 519. (3) 523. (2) 527. (5) 531. (5) 535. (2) 539. (2) 543. (2) 547. (4) 551. (5) 555. (1) 559. (1) 563. (2) 567. (3) 571. (4) 575. (5) 579. (2) 583. (2) 587. (3) 591. (4)

    392. (2) 396. (4) 400. (5) 404. (2) 408. (3) 412. (2) 416. (4) 420. (1) 424. (5) 428. (3) 432. (4) 436. (5) 440. (5) 444. (5) 448. (4) 452. (4) 456. (2) 460. (4) 464. (2) 468. (5) 472. (5) 476. (3) 480. (5) 484. (1) 488. (4) 492. (1) 496. (5) 500. (2) 504. (4) 508. (4) 512. (3) 516. (4) 520. (4) 524. (3) 528. (2) 532. (5) 536. (5) 540. (5) 544. (5) 548. (2) 552. (1) 556. (4) 560. (4) 564. (4) 568. (5) 572. (1) 576. (3) 580. (3) 584. (5) 588. (3) 592. (2)

    PROBLEM SOLVING

    SBI PO EXAMS 1. (5) 5. (1) 9. (3) 13. (1) 17. (2) 21. (1) 25. (4) 29. (2) 33. (3) 37. (5) 41. (4) 45. (5) 49. (3) 53. (4) 57. (3) 61. (3) 65. (1) 69. (4) 73. (1) 77. (2) 81. (5) 85. (5) 89. (3) 93. (2) 97. (2) 101. (2) 105. (5) 109. (1) 113. (4) 117. (5) 121. (1) 125. (2) 129. (4) 133. (3) 137. (5) 141. (4) 145. (2) 149. (2) 153. (4) 157. (4) 161. (3) 165. (5) 169. (2) 173. (4) 177. (1) 181. (1) 185. (2) 189. (3) 193. (1) 197. (3) 201. (2) 205. (3) 209. (5)

    2. (2) 6. (2) 10. (5) 14. (3) 18. (4) 22. (2) 26. (2) 30. (1) 34. (5) 38. (1) 42. (1) 46. (4) 50. (5) 54. (2) 58. (3) 62. (4) 66. (2) 70. (2) 74. (1) 78. (3) 82. (4) 86. (1) 90. (3) 94. (5) 98. (4) 102. (4) 106. (1) 110. (3) 114. (3) 118. (3) 122. (4) 126. (1) 130. (4) 134. (5) 138. (4) 142. (5) 146. (5) 150. (1) 154. (3) 158. (5) 162. (2) 166. (3) 170. (3) 174. (2) 178. (3) 182. (4) 186. (2) 190. (5) 194. (3) 198. (1) 202. (4) 206. (2) 210. (1)

    3. (4) 7. (4) 11. (3) 15. (1) 19. (5) 23. (5) 27. (5) 31. (4) 35. (1) 39. (4) 43. (2) 47. (1) 51. (3) 55. (1) 59. (1) 63. (1) 67. (5) 71. (3) 75. (1) 79. (3) 83. (3) 87. (4) 91. (1) 95. (3) 99. (1) 103. (1) 107. (4) 111. (2) 115. (2) 119. (5) 123. (2) 127. (5) 131. (4) 135. (2) 139. (3) 143. (1) 147. (4) 151. (2) 155. (1) 159. (2) 163. (1) 167. (2) 171. (1) 175. (5) 179. (5) 183. (1) 187. (4) 191. (2) 195. (4) 199. (5) 203. (5) 207. (3) 211. (3)

    4. (2) 8. (1) 12. (2) 16. (4) 20. (3) 24. (4) 28. (3) 32. (2) 36. (3) 40. (2) 44. (3) 48. (2) 52. (1) 56. (5) 60. (4) 64. (3) 68. (4) 72. (5) 76. (4) 80. (1) 84. (2) 88. (2) 92. (4) 96. (5) 100. (3) 104. (3) 108. (4) 112. (5) 116. (1) 120. (3) 124. (4) 128. (3) 132. (1) 136. (1) 140. (2) 144. (3) 148. (3) 152. (5) 156. (1) 160. (3) 164. (4) 168. (4) 172. (5) 176. (3) 180. (2) 184. (5) 188. (1) 192. (5) 196. (3) 200. (4) 204. (1) 208. (3) 212. (4)

    213. (2) 217. (5) 221. (3) 225. (5) 229. (3) 233. (4) 237. (1) 241. (3) 245. (5) 249. (4) 253. (1) 257. (3) 261. (2) 265. (3) 269. (2) 273. (4) 277. (3) 281. (5) 285. (3) 289. (1) 293. (4) 297. (1) 301. (1) 305. (4) 309. (3) 313. (4) 317. (5) 321. (1)

    214. (3) 218. (2) 222. (1) 226. (1) 230. (1) 234. (2) 238. (4) 242. (5) 246. (3) 250. (2) 254. (5) 258. (5) 262. (4) 266. (2) 270. (1) 274. (5) 278. (4) 282. (1) 286. (2) 290. (5) 294. (3) 298. (4) 302. (3) 306. (5) 310. (5) 314. (3) 318. (3) 322. (2)

    215. (4) 219. (3) 223. (4) 227. (3) 231. (5) 235. (5) 239. (2) 243. (4) 247. (5) 251. (4) 255. (2) 259. (1) 263. (3) 267. (5) 271. (2) 275. (1) 279. (2) 283. (5) 287. (4) 291. (3) 295. (1) 299. (5) 303. (3) 307. (1) 311. (2) 315. (1) 319. (4)

    216. (2) 220. (2) 224. (5) 228. (2) 232. (4) 236. (3) 240. (1) 244. (2) 248. (3) 252. (2) 256. (4) 260. (5) 264. (1) 268. (4) 272. (3) 276. (2) 280. (3) 284. (1) 288. (2) 292. (2) 296. (5) 300. (4) 304. (2) 308. (4) 312. (1) 316. (2) 320. (5)

    RBI GRADE–B/ NABARD GRADE–A OFFICER EXAMS 1. (2) 5. (3) 9. (2) 13. (5) 17. (4) 21. (1) 25. (1) 29. (3) 33. (4) 37. (2) 41. (2) 45. (1) 49. (5) 53. (3) 57. (2) 61. (5) 65. (1) 69. (4) 73. (3) 77. (5) 81. (2)

    2. (3) 6. (1) 10. (5) 14. (3) 18. (4) 22. (2) 26. (4) 30. (1) 34. (3) 38. (5) 42. (4) 46. (4) 50. (2) 54. (3) 58. (4) 62. (2) 66. (5) 70. (1) 74. (4) 78. (3) 82. (1)

    3. (1) 7. (5) 11. (4) 15. (1) 19. (2) 23. (5) 27. (3) 31. (2) 35. (1) 39. (1) 43. (2) 47. (3) 51. (2) 55. (5) 59. (3) 63. (5) 67. (4) 71. (2) 75. (1) 79. (3) 83. (4)

    BPRE–747

    4. (5) 8. (4) 12. (2) 16. (3) 20. (5) 24. (3) 28. (5) 32. (4) 36. (5) 40. (3) 44. (5) 48. (4) 52. (1) 56. (4) 60. (1) 64. (3) 68. (5) 72. (3) 76. (2) 80. (5) 84. (4)

    85. (1) 89. (1) 93. (4) 97. (1) 101. (5) 105. (5) 109. (1) 113. (3) 117. (5) 121. (1) 125. (4) 129. (3) 133. (3) 137. (2) 141. (1) 145. (4) 147. (5)

    86. (3) 90. (2) 94. (5) 98. (2) 102. (2) 106. (1) 110. (2) 114. (4) 118. (4) 122. (3) 126. (2) 130. (1) 134. (5) 138. (1) 142. (5) 143. (3)

    87. (2) 91. (2) 95. (1) 99. (3) 103. (3) 107. (3) 111. (4) 115. (5) 119. (3) 123. (2) 127. (4) 131. (1) 135. (2) 139. (4) 143. (2) 144. (5)

    88. (3) 92. (2) 96. (3) 100. (4) 104. (4) 108. (5) 112. (2) 116. (2) 120. (1) 124. (5) 128. (5) 132. (4) 136. (3) 140. (5) 144. (3) 146. (3)

    INSURANCE EXAMS 1. (4) 5. (1) 9. (2) 13. (3) 17. (2) 21. (4) 25. (1) 29. (5) 33. (2) 37. (5) 41. (5) 45. (4) 49. (2) 53. (4) 57. (5) 61. (2) 65. (3) 69. (4) 73. (1) 77. (4) 81. (3) 85. (3) 80. (1) 93. (4) 97. (2) 101. (4) 105. (4) 109. (2) 113. (3) 117. (2) 121. (2) 125. (2) 129. (2)

    2. (1) 6. (3) 10. (2) 14. (1) 18. (4) 22. (4) 26. (4) 30. (3) 34. (4) 38. (1) 42. (2) 46. (3) 50. (1) 54. (1) 58. (2) 62. (5) 66. (4) 70. (5) 74. (2) 78. (5) 82. (5) 86. (4) 90. (2) 94. (2) 98. (5) 102. (3) 106. (3) 110. (1) 114. (1) 118. (4) 122. (5) 126. (1) 130. (5)

    3. (4) 7. (2) 11. (4) 15. (3) 19. (2) 23. (1) 27. (5) 31. (3) 35. (3) 39. (2) 43. (3) 47. (5) 51. (3) 55. (1) 59. (4) 63. (3) 67. (5) 71. (3) 75. (1) 79. (2) 83. (4) 87. (5) 91. (5) 95. (1) 99. (2) 103. (2) 107. (5) 111. (3) 115. (5) 119. (1) 123. (5) 127. (3) 131. (3)

    4. (4) 8. (3) 12. (3) 16. (3) 20. (3) 24. (3) 28. (2) 32. (5) 36. (1) 40. (4) 44. (1) 48. (2) 52. (4) 56. (3) 60. (1) 64. (2) 68. (1) 72. (2) 76. (3) 80. (5) 84. (2) 88. (1) 92. (3) 96. (2) 100. (1) 104. (1) 108. (3) 112. (4) 116. (4) 120. (3) 124. (4) 128. (3) 132. (1)

    PROBLEM SOLVING

    Days Monday Tuesday Wednesday Thursday Friday Saturday Sunday

    Subject Physics Botany Maths Chemistry Statistics Zoology English

    8. (1) Physics is taught on Monday. 9. (4) Three subjects are taught between Botany and Zoology. 10. (3) Zoology is taught on Saturday. 11. (4) Statistics is taught on Friday. 12. (2) Zoology is taught on the next day on Statistics. Statistics is taught on the next day of Chemistry. (13–15) :

    16. (2) B works as an Agriculture Officer. 17. (3) C is an Economist. 18. (1) B works for bank M. 19. (4) A works for bank S and he is a Forex Officer. 20. (5) None is these. (21-27) : L E F T

    N

    M

    O

    L

    N M O

    25. (2)

    L

    2nd

    E

    1st

    B

    P

    M N O

    P

    Q

    Monday

    C

    1

    Tuesday

    A

    Wednesday

    E F Off day D B

    Sunday

    F

    Q

    Duration

    C

    3rd

    L

    Play

    6th

    D

    R I G H T

    Days

    G

    A

    P

    26. (2) Chemical N is in Red bottle. 27. (1) Chemical P is kept in the bottle at the extreme right. (28-34) :

    7th

    4th

    Q

    21. (4) Except in LQ, in all others thre is a gap of one bottle. 22. (1) Chemical L is in Pink bottle. 23. (3) P– Green is correct. 24. (3) Chemical Q is in Violet bottle.

    Thursday Friday Saturday

    5th

    Bank Post Forex Officer Agriculture Officer Economist Terminal Operator IT Officer Clerk Research Analyst

    S M N L R Q P

    Green

    1. (5) Economics was the last lecture. 2. (5) The code would be 5-1. 3. (5) Research Methods was organised on Friday. 4. (3) There were three lectures between Psychology and Economics. 5. (3) Computer Science was scheduled on Thursday. 6. (5) Saturday was the Off Day. 7. (1) He should attend Computer Science and Research Methods. (8–12) :

    Friend A B C D E F G

    Violet

    Tuesday Wednesday Thursday Friday Saturday Sunday

    Lectures Organisational Behaviour Psychology Statistics Computer Science Research Methods OFF Economics

    Pink

    Days Monday

    Blue

    (1-7) :

    White

    NATIONALISED BANKS & IBPS PO/MT/SO EXAMS

    13. (4) G lives on the topmost floor. 14. (3) C lives immediately above D’s floor. 15. (5) F, D, B and G live on odd numbered floor. C lives on even numbered floor. (16-20) :

    Red

    EXPLANATIONS

    1 Hours 2 1 Hour

    1 Hours 2 3 Hours 2

    2 Hours 1 Hours 2

    28. (4) Duration of Play B is

    1 Hour. 2

    29. (3) Friday is off day. 30. (2) A — Tuesday — 1 Hour is correct.

    BPRE–748

    31. (2) D is staged on Saturday. 32. (5) Four plays were staged before the off day. 33. (2) If D is staged on Monday, 1 Hours duration 2 wil be staged on Thursday. 34. (4) Now, Saturday would be the off day. (35-41) :

    Play E of 2

    Day

    Lecture

    Monday

    Mathematices

    Tuesday

    Psychology

    Wednesday

    Chemistry

    Thursday

    Computers

    Friday

    Biology

    Saturday

    Physics

    Sunday

    English

    35. (5) The lecture on Computers was held on Thursday. 36. (1) The combination Saturday – Physics is correct. 37. (3) Two lectures are scheduled between Chemistry and Physics. 38. (1) There is a gap of one day between the lectures on Chemistry and Biology. Therefore, Biology is related to English. 39. (3) The lect@ure on Psychology was scheduled on Tuesday. 40. (4) The lectu!res on Computers and Biology were held between the lectures on Chemistry and Physics. 41. (5) Five lectures. (42-46) : Person P Q R S T V W Z

    Sex Male M/F Male Female Male Male M/F Female

    Vehicle Honda City Honda City Ford Ikon Ford Ikon Swift D’zire Ford Ikon Honda City Swift D’zire

    Destination Hyderabad Hyderabad Chennai Chennai Delhi Chennai Hyderabad Delhi

    42. (3) R, S and V are travelling to Chennai in Car Ford Ikon. 43. (1) Four members are travelling in no car. 44. (4) S and Z are female members. The third female member is either P, Q or W. 45. (3) P and Q are travelling with W. 46. (2) P, Q and W are travelling in Honda City.

    PROBLEM SOLVING (47–51) : Friend Sex Car A Female Z B Male Y C M/F Y D Female X E M/F X F Female X/Y G H

    Male Male

    Destination Agra Chandigarh Chandigarh Delhi Delhi Delhi or Chandigarh Delhi Agra

    X Z

    47. (4) The sex of C and E is not known. 48. (5) None of the combinations is true. 49. (5) Four of them may be travelling in Car X. 50. (2) C is travelling in Car Y. 51. (1) The passengers of Car Y are travelling to Chandigarh. (52-54): Q > P > U P>T>R S>Q Now, S

    Q

    P

    64 kg

    T

    R

    U

    52 kg

    52. (3)

    P > T > R ↓ ↓ 64 kg 52 kg Therefore, T may be of 58 kg. 53. (5) Four persons – S, Q, P and T – are heavier than R. 54. (2) Q is heavier than P and P’s weight is 64 kg. Therefore, Q’s weight may be 66 kg. (55-59) : Person A B C D E F G

    (71–75) :

    (60–65) :

    Hobby Painting Dancing Singing Pottery making Reading Travelling Sculpting

    State Maharashtra Gujarat Kerala Rajasthan Punjab Karnataka Odisha

    55. (2) C likes singing. 56. (4) The combination D – Pottery Making – Rajasthan is correct. 57. (3) F belongs to Karnataka. 58. (1) G – Sculpting – Odisha is correct. 59. (5) E belongs to Punjab and he likes reading.

    Floor Number 7 6 5 4 3 2 1

    Person N M R Q S P O

    Marathon M 3300 4700 850 1300 2200 4000 2800

    60. (1) Three persons – M, R and Q – live between S and N. 61. (2) Q runs for 1300 metres. Only S lives between P and Q. 62. (3) ● Q lives on floor numbered 4. The one who runs for 3300 metres, lives on floor numbered 7. 4+3=7 ● O lives on floor numbered 1. The one who runs for 1300 metres, lives on floor numbered 4. 1+3=4 ● S lives on floor numbered 3. 4–3=1 ● R lives on floor number 5. 2+3=5 ● The one who runs for 1300 metres, lives on floor numbered 4. 4+3=7 63. (1) P lives immediately above the floor of O. Q runs for 1300 metres. The one who runs for 850 metres lives immediately above Q. 64. (5) M = 4700 metres ∴ B = (5300 – 4700) metres = 600 metres 65. (4) S runs for 2200 metres. (66-70) : Floor Number 7 6 5 4 3 2 1

    Person N M O L Q P R

    66. (4) N lives on the floor immediately above the floor of M. 67. (2) L lives exactly between the floor of O and Q. 68. (1) N lives on the topmost floor. 69. (3) L lives on the fourth numbered floor. 70. (5) Except P, all others live on odd numbered floors.

    BPRE–749

    Friend

    Profession

    J K L M N O P

    Engineer Doctor Actor Choreographer Lawyer Athlete Accountant

    Year of Birth 1983 1987 1990 1994 1986 1984 1992

    71. (2) O is an Athlete and he was born in the year 1984. 72. (4) M is a Choreographer and he was born in the year 1994. 73. (3) M is a Choreographer. 74. (1) The combination O – Athlete – 1984 is correct. 75. (5) J is an Engineer and he was born in the year 1983. So, he is the eldest among the seven friends. (76–80) : Floor Number 8 7 6 5 4 3 2 1

    Person U Z S X T W V Y

    76. (4) X lives immediately above T. Three persons – Z, S and X – live between T and U. W lives immediately above V. U lives on floor number 8. 77. (1) Z lives between U and S. 78. (2) X lives on floor number 5. 79. (5) W lives on the floor immediately below T. 80. (5) Two persons – S and X – live between the floors on which Z and T live. (81–85) : No. 6 5 4 3 2 1

    Floor Fifth Floor Fourth Floor Third Floor Second Floor First Floor Ground Floor

    Person B C F E A D

    81. (4) A and E live on the floors exactly between D and F. 82. (1) B lives on Fifth Floor numbered sixth.

    PROBLEM SOLVING (83–91) :

    E>G>I>J>F>H

    Floor Number 8 7 6 5

    Person Favourite Superhero O Wolverine K Batman R Thor N Captain America L Nova Q Superman M Ironman P Hulk

    4 3 2 1

    83. (1) R lives immediatly above N while L lives immediately below the floor of N. 84. (2) K lives on floor numbered Seven ⇒ (5 + 2) Q lives on floor numbered Three ⇒ (6 – 3) L lives on floor numbered Four ⇒ (2 + 2) N lives on floor numberd Five ⇒ (7 – 2) R lives on floor numbered Six ⇒ (8 – 2) 85. (4) R likes Thor. 86. (5) P likes Hulk. 87. (1) N lives on floor numbered Five. 88. (3) L likes Nova and there are two persons between L and P. O likes Wolverine and there are two persons between O and N. Similarly, N likes Captain America and there are two persons between N and M. (89–91) : P, Q, U > S > T, R U>Q>P U>Q>P>S>R >T







    23 12 5 89. (3) R has the second least number of offices. 90. (4) P has more than 12 but less than 23 offices. 18 is divisible by both 2 and 3. 91. (5) R has more than 5 but less than 12 offices. (92–94) : ,

    ,

    >J>

    E>G>I E>G>I>J>

    ,

    ,





    26 Floors 5 Floors F has 26 – 14 = 12 floors 92. (4) G has second highest number of floors. The probable number of floors G has = 27, 30, 33 27 and 33 are odd numbers and both are divisible by 3. 27 is given as an option. 93. (5) G has the second highest number of floors. 94. (1) J must have more than 12 but less than 26 floors. (95–97) : V > T > R, S W > U > V > T > R, S W>U>V>T>S>R







    64 43 20 95. (2) The bus T has the third lowest number of occupants. 96. (1) The Bus R has less than 20 occupants but more than 7 occupants. The numbers between 7 and 20, divisible by 3 : 9, 12, 15, 18 15 is divisible by 5 also. 9 is an odd number. Therefore, option (1) is the answer. 97. (3) The Bus V has more than 43 but less than 64 occupants. (98–100) : Month February March April June September October November

    Person Q R N P M S O

    Flower Lily Sunflower Marigold Rose Orchid Jasmine Daffodil

    98. (2) S has an anniversary in the month of October. 99. (4) O likes Daffodil. 100. (5) Q likes Lily who has an anniversary in the month of February. But, April is given with Lily. Similarly, N likes Marigold who has an anniversary in the month of April. But, September is given with Marigold. Therefore, Orchid is related to November.

    BPRE–750

    101. (1) N has an anniversary in April. O has an anniversary in November. 102. (3) Three persons – R, N and P – have an anniversary between Q and M. (103-105) : X > U > Y, Z Z>W X did not sell the highest number of policies. V>X>U>Y>Z>W







    33 24 11 103. (4) U sold exactly 33 policies. 104. (2) Z sold more than 11 but less than 24 policies. 105. (3) X sold less policies than only V. (106–108) : T>S>R ,

    ,

    ,

    >P>U

    Q>T>S>R>P>U





    47 days

    14 days

    S stayed for 47 – 15 = 32 days 106. (4) R stayed for more than 14 days but less than 32 days. Thus, R stayed for 25 days. 107. (2) T stayed for the second highest number of days. 108. (1) Q stayed for more than 47 days. Even numbers between 47 to 60 :

    ⇒ 48, 50, 52, 54, 56, 58 54 is divisible by 3 but not by 4. (109-115) : Floor 8 7 6 5 4 3 2 1

    Person X S Z V T Y U W

    Bank HDFC SBI IDBI Axis Bank SVC PNB BOI TJSB

    109. (2) S lives on floor numbered seven. 7–5=2 T lives on floor numbered 4 – 5 = –1 W lives on floor numbered one. 1+5=6 Z lives on floor numbered six. 6–5=1 U lives on floor numbered two. 2+5=7

    PROBLEM SOLVING 110. (1) V works at the Axis Bank. 111. (4) S lives immediately above Z. V lives immediately below Z. 112. (1) W works at the TJSB. 113. (5) U lives on floor numbered two. 114. (3) Y works at the PNB and lives immediately above U. X works at the HDFC and lives immediately above S. Z works at the IDBI and lives immediately above V. 115. (3) Obviously, option (3) cannot be a consequence of the awareness programme. It is related to improved law and order condition in the State X. (116-119) : > > > N > P > L > > N > P O > L > M > N > P





    13 feet 7 feet 116. (1) Pole M is the third tallest. 117. (4) The size of Pole L is 13 feet. 118. (1) 3 8 5 6 4 9 2 7 According to question, 2

    4

    6

    8

    3

    5

    7

    9

    4th from the right end 9

    119. (3)

    -4 5

    4 7 6 5 8 2 +1 -4 +1 -4 +1 +1 5

    3

    7

    1

    9

    (120–125) : Floor Number 8 7 6 5 4 3 2 1

    Person K Q P N R M L O

    120. (1) Floor Number 8 7 6 5 4 3 2 1

    Person K Q P N R M L O

    After interchange M Q L N R K P O

    Now, Q lives between M and L.

    3

    121. (4) P lives on floor numbered six. 122. (3) M lives on floor numbered three. 123. (5) Two persons live between K and N. In all others, one person lives immediately above or below the second person. 124. (2) Q and P live exactly between K and N. 125. (4)

    2 –1 1

    3 +2 5

    6 –1 5

    8 –1

    5 +2

    7

    7

    4 –1

    7 +2

    3

    9

    Digits 5 and 7 appear twice. 126. (5) From the statements it is clear that less than 6000 cars were sold in the third quarter (October-December) put together last year. But it is not clear that how many cars were sold in each month from October to December last year. After surge in the sales, 12,000 cars were sold in December this year. Therefore, Kellbahn definitely sold less than 12,000 cars in November this year. (127–132) : Day Monday Tuesday Wednesday Thursday Friday Saturday Sunday

    Friend V T Q U R S P

    City Madrid Chicago Miami Berlin Kabul Jakarta Sydney

    127. (4) S visits Jakarta on Saturday. 128. (5) R visits Kabul immediately before S who visits Jakarta. T visits Chicago immediately before Q who visits Miami. V visits Madrid immediately before T who visits Chicago. 129. (3) P visits Sydney on Sunday. 130. (2) U visits Berlin on Thursday immediately beofore R. Q visits Miami immediately before U. 131. (1) Except T – Thursday, all the combinations are correct. T visits Chicago on Tuesday. 132. (2) R visits Kabul on Friday.

    BPRE–751

    (133–136) : Floor Number 8 7 6 5 4 3 2 1

    Person W Q R X Y P S Z

    133. (4) P lives on floor numbered three. 134. (3) W lives on floor numbered 8 and 8 – 3 = 5. Y lives on floor numbered 4 and 4–3=1 Q lives on floor numbered 7 and 7–3=4 X lives on floor numbered 5 and 5+3=8 But, S lives on floor numbered 2 and 2 + 1 = 3 135. (5) P and Y live between S and X. 136. (1) Floor Number 8 7 6 5 4 3 2 1

    Person W Q R X Y P S Z

    After interchange S Y R X Q P W Z

    Now, P will live between Q and W. 137. (2) R lives on floor numbered six. (138–143) : Day Monday Tuesday Wednesday Thursday Friday Saturday Sunday

    Friend V T Q U R S P

    City Madrid Chicago Miami Berlin Kabul Jakarta Sydney

    138. (4) S visits Jakarta on Saturday. 139. (5) R visits Kabul immediately before S who visits Jakarta. T visits Chicago immediately before Q who visits Miami. V visits Madrid immediately before T who visits Chicago.

    PROBLEM SOLVING 140. (3) P visits Sydney on Sunday. 141. (2) U visits Berlin on Thursday immediately beofore R. Q visits Miami immediately before U. 142. (1) Except T – Thursday, all the combinations are correct. T visits Chicago on Tuesday. 143. (2) R visits Kabul on Friday. (144–147) : Floor No. 6 5 4 3 2 1

    Person P V S R Q T

    144. (1) S and R live on the floors exactly between the floors on which Q and V live. 145. (4) V lives on Floor Number 5. 146. (3) T lives on Floor Number 1. 147. (1) P lives on the topmost floor. (148–152) : Floor number 7 6 5 4 3 2 1

    Person V Q U P S T R

    148. (5) U lives on floor number five. 149. (3) Five persons – Q, U, P, S and T – live between the floors of V and R. 150. (1) Except Q, all others live on odd numbered floors. 151. (4) Q lives exactly between the floors of U and V. 152. (2) U lives immediately above the floor of P. (153-159) : Floor Number 7 6 5 4 3 2 1

    Person

    Destination

    V Q U P T R S

    Jaipur Mumbai Patna Delhi Kolkata Bangalore Chennai

    153. (3) V lives on the topmost floor. 154. (1) Except R, all others live on odd numbered floors.

    155. (5) P is travelling to Delhi. 156. (2) Four persons live between the floors of Q and S. 157. (4) P lives on the floor immediately above the floor of T. 158. (1) Q travelled to eight countries. Only P has travelled to more number of countries than Q. 159. (4) R has travelled to 8–4 = 4 countries U has travelled to more than four but less than eight countries. So, U ⇒ 5, 6 or 7 (160 –165) : Day

    Friend

    Language

    Monday

    B

    Hindi

    Tuesday

    E

    French

    Wednesday

    F

    English

    Thursday

    C

    Chinese

    Friday

    G

    Spanish

    Saturday Sunday

    A D

    German Japanese

    160. (3) B learns Hindi language immediately before the one who learns French language. Similarly, F learns English language immediately before the one who learns Chinese language. A learns German language immediately before the one who learns Japanese language. 161. (3) C learns Chinese language on Thursday immediately after F who learns English language. C learns Chinese language immediately before G. 162. (1) D learns Japanese language on Sunday (Thursday + 3). F learns English language on Wednesday (Sunday – 4). B learns Hindi language on Monday (Friday – 4). A learns German language on Saturday (Tuesday + 4). G learns Spanish language on Friday (Monday + 4). 163. (4) D learns Japanese language on Sunday. 164. (2) A learns German language on Saturday. 165. (5) E learns French language on Tuesday.

    BPRE–752

    (166–171) : Month January February April July August November December

    Person C D B A G F E

    Colour Yellow Pink Green White Orange Red Blue

    166. (4) C likes Yellow colour. 167. (1) E likes Blue colour. E published his journal in December. 168. (3) D – Pink combination is matched correctly. 169. (2) Two persons published their journals between the one who published his journal in January and the one who likes White colour. Two persons published their journals between the ones who published their journals in December and July respectively. Similarly, two persons published their journals between the one who published his journal in February and the one who likes Orange colour. 170. (5) The combination ‘January – C – Yellow’ is definitely correct. 171. (3) Three persons – B, A and G – published their journals between the ones who like Pink and Red coloured respectively. (172–176) : Day Monday Tuesday Wednesday Thursday Friday Saturday Sunday

    Mobile HTC Micromax Oppo Intex Samsung Lenovo Nokia

    172. (2) Sagar sold Micromax on Tuesday. 173. (3) Sagar sold four mobiles – Micromax, Oppo, Intex and Samsung – between HTC and Lenovo. 174. (5) Sagar sold only Lenovo between Samsung and Nokia. Sagar sold Lenovo on Saturday.

    PROBLEM SOLVING 175. (1) Nokia was sold on Sunday and Sunday – 2 = Friday. Samsung was sold on Friday and Friday – 1 = Thursday. Micromax was sold on Tuesday and Tuesday – 1 = Monday. Intex was sold on Thursday and Thursday – 1 = Wednesday. Oppo was sold on Wednesday and Wednesday – 1 = Tuesday. 176. (4) Sagar sold Nokia on Sunday. (117–181) : Floor Number Floor Number 9 Floor Number 8 Floor Number 7 Floor Number 6 Floor Number 5 Floor Number 4 Floor Number 3 Floor Number 2 Floor Number 1

    Person H B E I D A G F C

    177. (1) Except DE, in all other pairs the first person lives immediately above the second person. 178. (2) E lives on floor numbered 7. Two persons – I and D – live between E and A. 179. (4) H lives on the topmost floor, i.e., floor numbered 9. 180. (3) B lives on floor numbered 8. B lives immediately above E. 181. (5) Only F lives between G and C. Only B lives between H and E. Only I lives between E and D. Two persons – B and E – live between H and I. Only E lives between B and I. (182–186) : Day Monday Tuesday Wednesday Thursday Friday Saturday Sunday

    Country France Austria China Indonesia Japan Egypt Malaysia

    182. (3) Rohan speaks about Indonesia on Thursday. 183. (1) All the statements are true. 184. (4) Rohan speaks about France on Monday.

    185. (2) Rohan speaks about three countries – Indonesia, Japan and Egypt – between China and Malaysia. 186. (5) Rohan speaks about Malaysia on Sunday and Sunday – 1 = Saturday. Rohan speaks about France on Monday and Monday + 1 = Tuesday. Rohan speaks about Egypt on Saturday and Saturday + 1 = Sunday. Rohan speaks about Austria on Tuesday and Tuesday – 1 = Monday. Rohan speaks about China on Wednesday and Wednesday + 2 = Friday. (187 – 191) : Floor Number Floor Number 9 Floor Number 8 Floor Number 7 Floor Number 6 Floor Number 5 Floor Number 4 Floor Number 3 Floor Number 2

    Person T R S U P Q V X

    Floor Number 1

    W

    187. (5) Only U lives between the floors on which P and S live. 188. (1) X lives on the floor immediately below the floor of V. 189. (3) X lives on floor numbered two. 190. (2) Only P lives between U and Q. Only three persons – T, R and S – live above U. S lives immediately above the floor of U. U lives on floor numbered six. 191. (4) P lives on floor numbered five. (192–196) : Floor Number Floor Number 9 Floor Number 8 Floor Number 7 Floor Number 6 Floor Number 5 Floor Number 4 Floor Number 3 Floor Number 2 Floor Number 1

    BPRE–753

    Person V U X P Q S T W R

    192. (2) S lives on floor numbered 4 and 4 + 2 = 6. R lives on floor numbered 1 and 1 + 2 = 3. Similarly, X lives on floor numbered 7 and 7 + 2 = 9. 193. (5) It is clear that U lives on floor numbered 8. 194. (3) Except XT, in all other pairs, even number of persons lives between the two. 195. (1) W lives on floor numbered 2. Only R lives below W. 196. (4) Four persons – U, X, P and Q – live between S and V. Q lives immediately above S. Eight persons live above R. U lives on floor numbered eight. (197–198) : Day Monday Tuesday Wednesday Thursday Friday Saturday Sunday

    Movies Braveheart Avatar Titanic Passion Inception China Town Gladiator

    197. (1) Gaurav watched two movies – Avatar and Titanic – between Braveheart and Passion. 198. (2) Gaurav watched Avatar on Tuesday and Tuesday + 1 = Wednesday. Gaurav watched Titanic on Wednesday and Wednesday + 1 = Thursday. Guarav watched Inception on Friday and Friday + 1 = Saturday. Gaurav watched China Town on Saturday and Saturday + 1 = Sunday. Gaurav watched Braveheart on Monday and Monday + 4 = Friday. 199. (5) Gaurav watched Inception immediately after Passion. Guarav watched three movies between Avatar and China Town. Gaurav watched Passion on Thursday. 200. (3) Gaurav watched Avatar on Tuesday. 201. (4) Gaurav watched Braveheart on Monday.

    PROBLEM SOLVING (202–204) : Person P

    M/F

    Relation

    Q

    M

    R

    F

    S

    M

    T V W

    Bank IDBI

    Inference Male

    Analysis/Remarks S is son of P and W is daughter in law of R ⇒ P & W is married couple.

    V’s grandfather



    BOI

    Given, Grandfather is in BOI.

    T’s mother P’s son

    SBI —

    ICICI

    Remained Bank & Person.

    UCO

    Given that : no woman is in HDFC & ICICI

    HDFC PNB F

    R’s daughterin-law



    Other given information : Both married couoples are of different generations. Hence, possible family-tree will be as follows : Q BOI

    R SBI

    T HDFC

    P IDBI

    W UCO

    S ICICI Male

    Female

    203. (4) The gender of V and T cannot be determined. Hence, relation is not clear. 204. (4) The married couples are Q ↔ R and P ↔ W. Hence, option (4) is definitely false option (1) can not be determined whether true or false. (205–206) : , , , Kumble, Lakshman, Sehwag Tendulkar > Dravid > Dhoni Tendulkar > Dravid > Dhoni >

    ↓ 8 match Kumble > Shewag > Lakshman

    V PNB



    Married couple

    Here sex of T & V is not known. 202. (1) The married females W and R work in UCO and SBI Banks respectively.

    2 match 205. (1) Dravid has played 8 matches. Only Tendulkar has played more matches than Dravid. 206. (3) Kumble has played (8-4) = 4 matches. Dhoni has played more than four but less than eight matches.

    (207-211): Month January February March June August October December

    Person Ritesh Ramendra Ravindra Ranvijay Ravish Ramashankar Ranveer

    Fruit Apple Papaya Litchi Grapes Banana Orange Mango

    Subject Organisational behaviour Psychology Statistics Computer Science Environmental awareness Anthropology Economics

    207. (3) The last lecture is on economics. 208. (2) The combination December-Ranveer-Mango-Economics is true 209. (3) Ranveer likes mango.

    BPRE–754

    210. (3) The lecture delivered to the person who likes grapes will be in June because when the months are written in the reverse order, the middle month i.e, fourth (3 + 3 + 1 ⇒ 7) position will remain unchanged. 211. (2) Three lectures were delivered in between the lectures delivered by Ritesh and Ravish. (212–216) : Person P Q R S T U V

    City Mumbai Agra Delhi Kolkata Jaipur Chandigarh Patna

    Employee HR IT HR Production IT Production Production

    212. (2) Except P – Patna, in all other pairs both are related to the same department. V likes Patna and he works in the Production department. P works in the HR department. 213. (1) S likes Kolkata. 214. (3) R likes Delhi. Both S and U work in the Production department. V works with the ones who like Chandigarh and Kolkata respectively. Both Q and T work in the IT department. 215. (4) Q and T work in the IT department. Q likes Agra. 216. (5) V works in the Production department and he likes Patna. (217–221) : Floor Number 7 6 5 4 3 2 1

    Person

    Watch

    R S Q U T P V

    Casio Tissot Fastrack Seiko Fossil Omega Citizen

    217. (1) Q lives immediately below the floor of S. 218. (4) U likes Seiko. Three persons – Q, S and R – live above U. 219. (3) T likes Fossil. 220. (5) Q lives on floor number 5. 221. (2) Q likes Fastrack and R likes Casio. Only S lives between R and Q.

    PROBLEM SOLVING

    March April May August Septemeber December

    12th 27th 12th 27th 12th 27th 12th 27th 12th 27th 12th 27th

    T J F V K S R G U H W I

    227. (1) F has his exam on 12th April. 228. (2) V has his exam on 27th April. 229. (4) Three persons - F, V and K have exams between the exams of J and S. 230. (5) I has his exam in the last. I has his exam immediately after W. I has his exam on 27th December. 231. (3) Except K S, in all others only two persons have their exams between the given pair of persons. K has his exam immediately before the exam of S.

    232. (3) There is a gap of two persons between R and the one who likes opera. Similarly, there is a gap of two persons between Q and the one who likes classical. There is a gap of two persons between P and the one who likes Rock. 233. (1) R works as Manager (MG). 234. (5) N and O are more experienced than Q but less experienced than P. 235. (4) S works as Trainee (TE). 236. (2) N works as Chief Manager (CM) and likes Pop. 237. (5) R likes Jazz. (238–242) : Floor 8 7 6 5 4 3 2 1

    Person P T W R V U Q S

    Game Roadrash Castlevania Resident Evil Dragon Quest Tekken 3 Contra San Andreas Metal Gear

    238. (4) S plays the game of Metal Gear. 239. (2) V himself plays Tekken 3. 240. (5) V plays Tekken 3. R lives immediately below the one who plays Resident Evil. Q lives immediately above S. 241. (3) T and W live between P and R (Dragon Quest). 242. (1) There are four persons between P and U, who likes Contra. There are only two persons Q and who likes Dragon Quest. Similarly, there are two persons between R–Roadrash. W is immediately below the per-

    BPRE–755

    Amul

    Cipla

    J

    G

    I

    K

    H

    Y

    W

    V

    X

    U Flipkart

    L E F T

    Vadilal

    R I G H T

    Samsung

    222. (5) U works as an IFS and he likes Frozen. 223. (1) Q works in Punjab with ones who like Titanic, Gladiator and Frozen. 224. (3) X likes Cindrella. 225. (4) The combination “V–IPS – Haryana – Twilight” is correct. 226. (2) T, V and W work in the State of Haryana. (227–331) : Exam Date and Month Person

    Position Person Music Director (DR) P Hip–Hop Executive Director (ED) O Opera Chief Manager (CM) N Pop Senior Manager (SM) Q Rock Manager (MG) R Jazz Assistant Manager (AM) M Electronic Trainee (TE) S Classical

    Tata

    Movie Dread Inception Titanic Gladiator Vertigo Frozen Twilight Watchmen Cindrella

    Nokia

    State Kerala Punjab Punjab Punjab Haryana Punjab Haryana Haryana Kerala

    Havells

    Post IFS IPS IFS IPS IFS IFS IPS IPS IFS

    Biocon

    Person P Q R S T U V W X

    son who likes Castlevania. Similarly, U lives immediately below the person who likes Tekken 3. (243–247) :

    Indigo

    (332–237) :

    (222–226) :

    L E F T

    R I G H T

    243. (5) K is to the immediate left of the person from Vadilal. W is to the immediate left of the person from Tata. Similarly, X is to the immediate left of the person from Flipkart. 244. (1) Y is from Indigo. 245. (2) I is from Vadilal. V faces I. 246. (3) Two persons sit between G and the person from Cipla. The person from Havells faces G. G is an immediate neighbour of J from Biocon. G sits to the immediate right of I. 247. (4) Persons sitting at the extreme ends of both the rows : J (Biocon) and H (Cipla) Y (Indigo) and U (Flipkart) (248–250) : B>A>E D>B C>D>B>A>E





    24 11 248. (4) Jar D has more than 11 but less than 24 cookies. 249. (2) Jar A has the second lowest number of cookies. 250. (3) Jar E has less than 11 cookies. (251–255) : Day

    Store

    No. of TVs

    Monday

    D

    Tuesday

    E

    4

    Wednesday

    G

    12

    Thursday

    C

    18

    Friday

    F

    6

    Saturday

    B

    27

    Sunday

    A

    9

    Sold 15

    PROBLEM SOLVING 251. (2) A sold 9 TVs and B sold 27 TVs. 252. (5) E sold 4 TVs. 253. (4) D sold TVs on Monday. C sold 18 TVs immediately before F. A sold 9 TVs on Sunday. 254. (3) B sold maximum number of TVs. Therefore, six stores sold less number of TVs than B. 255. (1) D sold 15 TVs on Monday. F sold 6 TVs on Friday. Required difference = 15 – 6 = 9 (256–258) : Sandeep > Rishi > Rupali, Preeti , Sandeep > Rishi >

    ,

    ,

    Saksham > Sandeep > Rishi >

    ↓ 41 Rupali > Preeti > Sunny

    ↓ 25 256. (1) Sakshan won more than 41 chocolates. 257. (4) Sunny won less than 25 chocolates. 258. (2) Rupali won more than 25 but less than 32 chocolates. (259–264) : Floor Number 7 6 5 4 3 2 1

    Person N Q S R P M O

    Marathon 5000m 1200m 750m 4300m 2900m 2200m 3600m

    259. (1) P ran for 2900 metres. Only M lives between P and O. M ran for 2200 metres. 260. (3) Except ‘Floor Number 4 – N’, in all others there is a gap of one person. 261. (5) Three persons – R, P and M live between S and O. 262. (4) P ran for 2900 metres. 263. (2) M ran for 2200 metres. Therefore, B ran for (4800 – 2200) metres = 2600 metres. 264. (3) R lives immediately above P. S ran for 750 metres. Q ran for 1200 metres.

    (277–279) : E>C>A A > B, D F > E > C >

    (265–267) : C>F>B B, E > A F>E C > F > E, B > A > D







    13 cm 7cm 265. (5) E is 12 cm thick. Therefore, C>F>E>B>A>D







    13 12 7 F is one cm thicker than E. C is thicker than 13 cm. Therefore, E + C > 22 cm D is the thinnest. 266. (3) D = (7 – 2) cm = 5 cm 267. (2) B + A = 17 cm ∴ B = (17 – 7) cm = 10 cm Now, F + B = (13 + 10) cm = 23 cm (268–270) : B, D > E > C A > B, D F > A > B, D > E > C





    13cm 4cm 268. (2) F = 13 cm D = (13 – 5) cm = 8 cm 269. (3) Wire C has the least length. 270. (1) The length of wire C would be less than 4 cm. (271–276) : Floor Person No. 7 R 6 P 5 S 4 N 3 O 2 Q 1 M

    Income Alphabetical (In Rs.) Order 15000 M 3500 N 5000 O 7500 P 9000 Q 11000 R 13500 S

    271. (1) M has an income of Rs. 13500. 272. (2) The combination ‘Rs. 15000 – R’ is true. 273. (3) The position of none will remain unchanged. 274. (2) The one who has income of Rs. 5000 lives immediately below P. P lives immediately above S. 275. (5) Q lives on floor numbered 2. 276. (4) R has an income of Rs. 15000.

    BPRE–756

    A > B, D



    16 cm. 9 cm. 277. (4) E = (9 + 5) cm. = 14 cm. 278. (1) C + F = 27 ⇒ C + 16 = 27 ∴ C = 27 – 16 = 11 Now, A + C = 9 + 11 = 20 279. (3) F > E > C > A > B > D







    16 9 8 B is the second thinnest book of all. F = (B + 8) cm. (280–282) : O, Q > M > P P>L>N Q, O > M > P > L > N





    29 6 280. (5) Q > O > M > P > L > N







    30 29 6 O and N together sold 36 computers. Only Q sold more computers than O. 281. (2) P sold less than 29 computers. 27 is a multiple of 3 and it is more than 25. 282. (1) Only N sold less computers than L. (283–287) :

    Top

    Box M H K L G J N

    Bottom

    I

    283. (5) Box I is first from the bottom. 284. (1) Only Box J is kept between Box N and Box G. 285. (3) Box N is just above the Box I. Box G is just above the Box J. Box K is just above the Box L. 286. (4) Box K is the third from the top of the stack. Two boxes – L and G – are kept between Box K and Box J.

    PROBLEM SOLVING 287. (2) Box M is kept at the top. Box I is kept at the bottom. (288–292) : Student

    College

    M T D F H R W

    II I II III I III III

    Favourite Subject Mathematics History Physics Chemistry English Geography Biology

    288. (1) M’s favourite subject is Mathematics. 289. (4) In College III, three students study. 290. (2) F, R and W study in College III. 291. (5) T’s favourite subject is History. 292. (3) H and T study in College I. (293–297) : Month January February March April May June July

    Person O R M N S Q P

    Box

    J

    K

    Box

    L

    L

    Box

    S

    M

    Box

    M

    R

    Box

    K

    S

    Box

    U

    T

    Box

    T

    U

    Alphabetical order

    293. (1) Only M buys a car between R and N. 294. (5) R buys a car in February and O buys a car in January. 295. (2) N buys a car in April after i.e., R. Only R buys a car between O and M. P buys a car in July. O buys a car in January and S buys a car in May. 296. (4) Q buys a car in June. 297. (3) R buys a car in February and February + 5 = July. Q buys a car in June and June – 3 = March. O buys a car in January and January + 3 = April. S buys a car in May and May – 3 = February. M buys a car in March and March + 3 = June. (298–302) : Box R J

    298. (3) Except MT, in all other pairs only one box is kept between the given two boxes. There are two boxes between M and T. 299. (2) Box L is second above the Box M. 300. (4) The position of only Box L will remain unchanged. 301. (1) Box K is kept third from the bottom of the stack. 302. (5) Two boxes – M and K – are kept between S and U. (303–307) : Month April May August September October November December

    Person R O S P N Q M

    303. (4) Three persons – P, N and Q – buy a car between S and M. 304. (3) The pair ‘P-September’ is correctly matched. In all other pairs, the given person bought a car immediately before or after the given month. 305. (5) S buys a car in August. 306. (2) O buys a car in May. 307. (1) N buys a car in October. Q buys a car in November. M buys a car in December. (308–312) : Box T Box Z Box U Box W Box S Box Q Box R Box X

    Q R S T U W X Z

    308. (3) Except WQ, in all other pairs the first box is kept immediately above or below the second box. 309. (1) Box S is kept fourth from the bottom of the stack. 310. (5) Box X is the first from the bottom of the stack. 311. (4) Only Box R is kept between Q and X. 312. (2) The positions of all the boxes get changed.

    BPRE–757

    (313–317) : Month February April June July September October December

    Person G K L H I F J

    313. (5) G was promoted in February. 314. (2) G was promoted in February, immediately before, K who was promoted in April. Similarly, I was promoted in September, immediately before F, who was promoted in October. 315. (1) H was promoted in July. 316. (4) K was promoted in April – a month having 30 days. K was promoted in April before L. L and H were promoted between K and I. Thus, (A) and (B) are not true. 317. (3) Three persons G, K and L were promoted before H. (318–322) : Box D Box A Box Y Box C Box B Box X Box Z Box W 318. (3) Box D is kept immediately above Box A. Box X is kept immedietely above Box Z. Box C is kept immediately above Box B. 319. (2) Box Y is fifth above Box W. 320. (1) Seven boxes are kept above Box W. 321. (4) Second from the top ⇒ Box A Fifth from the top ⇒ Box B 322. (5) Two boxes – A and Y – are kept between D and C. (323–327) : Person T U V W X Y Z

    Branches Aeronautical Electrical Chemical Electrical Aeronautical Chemical Electrical

    Sport Football Volleyball Hockey Boxing Archery Wrestling Cricket

    PROBLEM SOLVING

    Puma

    May

    K

    Zara

    June

    N

    Marigo

    August

    P

    Fastrack

    September

    O

    Nike

    November

    J

    Biba

    328. (3) L advertises in the month of April. L advertises for Puma. 329. (2) O advertises for Nike. Four persons – L, K, N and P – advertise between M and O. 330. (4) Advertisement for Marigo is done in June. 331. (1) L advertises for Puma immediately before K who advertises for Zara. N advertises for Marigo immediately before P who advertises for Fastrack. O advertises for Nike immediately before J who advertises for Biba. 332. (5) J advertises for Biba. K advertises for Zara. L advertises in the month of April. K advertises in the month of May. N advertises for Marigo. P advertises for Fastrack. O advertises in the month of September. J advertises in the month of November. M advertises for Vivo in the month of February.

    E B A D F H G C

    333. (1) B likes green colour. 334. (2) A was born on 16th March. 335. (4) F likes cherry. 336. (5) C was born on 19th July. 337. (3) H likes red colour. (338–342) : Person Q R S T U V W X Y Z

    Date 7th 9th 9th 7th 9th 7th 4th 7th 4th 2nd

    City Pune Pune Patna Chandigarh Patna Kochi Patna Patna Kochi Pune

    338. (2) T goes to Chandigarh. 339. (5) Four persons S, U, W and X go to Patna. 340. (3) Only Z visits on 2nd of the month. 341. (4) U visits Patna. 342. (1) Z goes to Pune on 2nd of the month. (343–348) : D

    B

    I

    E

    27 km.

    C

    A

    H Diesel

    Vivo

    L

    7 9 16 19 7 9 16 19

    Petrol

    M

    March March March March July July July July

    Colour/ Fruit Apple Green Yellow Banana Cherry Red Blue Mango

    Diesel

    February April

    Person

    Diesel

    Person Brand

    Date

    Diesel

    Month

    Month

    Diesel

    326. (3) T and X study Aeronautical Engineering. T likes Football and X likes Archery. Y likes Wrestling and W likes Boxing. Y studies Chemical Engineering while W studies Electrical Engineering. V likes Hockey. Both V and Y study Chemical Engineering. U likes Volleyball. U and W study Electrical Engineering. 327. (4) T likes Football. (328–332) :

    348. (5) Case-I

    (333–337) :

    Petrol

    323. (1) V and Y study Chemical Engineering. 324. (5) Y likes Wrestling. Both (B) and (C) are true. 325. (2) X likes Archery.

    40 km.

    343. (3) Cars A and D are petrol cars. 344. (4) Car I covered a distance more than 27 km but less than 40 km. 345. (2) Cars D and H are parked at the two extreme ends. 346. (5) Car E is parked to the immediate left of car C. 347. (1) Four cars (I, E, C and A) are parked between Car B and Car H.

    BPRE–758

    Case-II

    A

    D

    C B

    A C

    D

    B

    (349–350) : Day Monday Tuesday Wednesday Thursday Friday Saturday Sunday

    Person B M G A L X F

    Company Convergys IBM IBM Convergys IBM Dell Capegemini

    349. (4) B was born on Monday. He works for Convergys. 350. (3) Only X works for Dell. Only F works for Capegemini. 351. (5) X was born on Saturday. 352. (2) G, L and M work for IBM. 353. (1) F works for Capegemini. Four persons – G, A, L and X were born between M and F. (354–356) : > > >P> > Q>S>P U>Q>R U > Q > S > P > R/T > R/T





    87kg 70kg 354. (5) Weight of Q = 87 kg Weight of R = (87 – 21) kg. = 66 kg. U>Q>S>P>T>R







    87kg 70kg 66kg Both (i) and (iii) are definitely true. 355. (2) Weight of P = 76 kg Weight of S is more than 76 kg but less than 87 kg. 356. (4) Q is the second heaviest. (357–361) : Day January

    Person Peter

    Event Painting Competition February Albert Technical Fest April Oliver Elocution Competition June Samuel Cultural Fest July Richard Debate September Tom Science Quiz November Michael Athletic Meet

    PROBLEM SOLVING 357. (2) Michael participated in an event in November. 358. (4) Peter participated in Painting Competition. Michael participated in Athletic Meet in November. Oliver, himself participated in Elocution Competition. 359. (5) Oliver participated in Elocution Competition. Only one person participated in an event between Peter and Oliver and Peter participated in an event before Oliver. Tom participated in Science Quiz. Only one person participated in an event between Samuel and Tom. Samuel participated in an event before Tom. Only one person participated in an event between Oliver and Richard. Oliver participated in an event before Richard. Richard participated in Debate. 360. (1) Oliver participated in an event immediately before Samuel and Richard participated in an event immediately after Samuel. 361. (3) Tom participated in Science Quiz. (362–366) : Box Colour S Green Q Red P Yellow T Orange R Blue 362. (4) Box Q is red in colour. 363. (3) Yellow box is in the middle, i.e., third from the top or bottom. 364. (5) Box Q (Red) and Box P (Yellow) are kept between T and Green box (S). 365. (1) T is orange in colour. Only (i) and (ii) are true 366. (2) Box T is orange in colour. Box P is kept im7ediately above Box T. (367 – 372) : Month March June

    August October

    City

    367. (2) M belongs to Mumbai. 368. (1) N belongs to Surat. N attends the seminar immediately before D. 369. (4) B and E will have to attend the seminar in August. B belongs to Ahmedabad. 370. (4) Two persons – N and D – have their seminar after E. 371. (5) D belongs to Udaipur. 372. (3) B has to attend a seminar on 16th August. Only E has his seminar between B and N. L belongs to Bangalore. (373 – 378) : Floor Number Person

    379. (5) I has a lecture in Delhi. 380. (1) L’s lecture is at 2 pm. 381. (3) F has a lecture in Mumbai and H in Patna. M has a lecture in the last. G has a lecture in Delhi and J in Patna. F has a lecture at 3 pm. 382. (4) G has a lecture at 7 pm. 383. (2) M has a lecture at 8 pm. J has a lecture at 6 pm. Required difference = 8 pm – 6 pm = 2 hours (384–388) : Year 1988

    Game

    Q

    Tennis

    1993 1996

    S

    Snooker

    U

    Carom

    V

    Boxing

    O

    Cricket

    T

    Football

    P

    Chess

    R

    Polo

    373. (1) V likes Boxing. V lives on floor number five. Three persons live above V. 374. (4) O likes Cricket. O lives on floor number four. T likes Football. T lives on floor number three. 375. (2) Q lives on floor number eight. 376. (2) Except R – Boxing, in all other pairs only one person lives between the two persons. V likes Boxing. Three persons live between V and R. 377. (3) T lives immediately above P. T likes Football. 378. (5) R likes Polo. (379 – 383) : Time

    Person

    1 pm

    H

    Date

    Person

    16th

    M

    2 pm

    L

    25th

    O

    3 pm

    F

    16th

    L

    4 pm

    K

    25th

    C

    5 pm

    I

    16th

    B

    J

    25th

    E

    6 pm

    16th

    N

    7 pm

    G

    25th

    D

    8 pm

    M

    BPRE–759

    City

    2001 2008

    Month March September March September March September March September March September

    Person Z Y W C A X B E V D

    384. (3) Three persons – A, X and B – had exams between C and E. 385. (1) A had his exam in March 1996. 386. (4) E had the exam after X. C had his exam in 1993. A had his exam in 1996. Y had his exam in 1988. C had his exam before both A and X. 387. (2) Three persons – E, V and D – had their exams after B. 388. (5) D had his exam in September 2008. (389–393) : Day Employee Monday M Tuesday P Wednesday Q Thursday L Friday N Saturday R Sunday O

    Department HR Department Security Administrative Security Administrative HR Department Finance

    389. (4) N attends meeting on Friday. 390. (5) ‘P – Tuesday’ combination is true. 391. (1) L attends meeting in Security department. 392. (2) Q attends meeting on Wednesday and Wednesday – 2 = Monday.

    PROBLEM SOLVING L attends meeting Thursday and Thursday – 2 = Tuesday. N attends meeting on Friday and Friday – 2 = Wednesday. 393. (3) Four persons – Q, L, N and R – attend meeting between P and O. (394–398) : Day 1 Monday 2 Tuesday 3 Wednesday 4 Thursday 5 Friday 6 Saturday

    Topic A E B C F (Rest Day) D

    394. (3) Correct sequence : AEBCFD 395. (5) Topic A will be discussed on Monday. Topic C will be discussed immediately after B. Discussion on topic B will take place after that of topic A. Friday is the Rest Day. 396. (4) Topic C will be discussed on Thursday. 397. (2) Topic E will be discussed immediately before Topic B. 398. (1) Topic W will be discussed immediately after that of the Topic A. (399–403) : Month January March April May June August October November December

    Person F E K B G D H C A

    399. (3) Seven persons – K, B, G, D, H, C and A – are younger than E. 400. (5) B was born in May. 401. (4) G was born in June. Three persons – E, K and B – were born between F and G. 402. (2) K was born in April. Only one person was born between K and G. G was born in June. Only one person was born between H and G. D was born in August. Only one person was born between C and D. 403. (1) K was born in April.

    (404–408) : Day Monday Tuesday Wednesday Thursday Friday

    Movie D A E B C

    Duration 75 Minutes 90 Minutes 100 Minutes 130 Minutes 120 Minutes

    404. (2) Two movies – B and C – were released after E. 405. (3) There is no movie of 150minute duration. 406. (4) Duration of movies D and E = (75 + 100) Minutes = 175 minutes 407. (1) Only Statement (1) is true. Only one movie was released between A and B. Movie B is of 130 minutes. Total duration of movies B and A is 200 minutes Movie A was released on Tuesday while movie B was released on Thursday. 408. (3) The movie released before A is of 75 minute duration. Movie A was released immediately before E. Total duration of movies C and A is 210 minutes. Movie C was released immediately after B. (409–411) : M>N>Q , >M>N>Q R > Q, O P/R > M > N > Q > O ↓ 154 cm 409. (5) Q = 154 cm M = (154 + 19) cm = 173 cm 410. (4) P = 181 cm P – Q = (181 – 154) cm. = 27 cm. O is the shortest person. 411. (2) Q and O are shorter than N. (412–416) :

    Month

    Date

    412. (2) J was born immediately before M. 413. (3) R was born on 16th January. 414. (5) L was born immediately after P. 415. (1) Except KR, in all others one person was born between the two given persons. R was born immediately after K. 416. (4) O was born on 5th July. (417–421) :

    Month

    Person

    January

    B

    February

    C

    March

    F

    April

    A

    May

    E

    June

    G

    July

    H

    August

    D

    417. (4) Only H visits Mumbai between G and D. Three persons–G, H and D– visit Mumbai after E. C visits Mumbai in February. 418. (5) H visits Mumbai in July. 419. (2) Three persons–F, A and E– visit Mumbai between C and G. 420. (1) D visits Mumbai in August. 421. (3) E visits Mumbai in May, one month before June. A visits Mumbai in April, one month before May. B visits Mumbai in January, one month before February. H visits Mumbai in July, one month before August. But, C visits Mumbai in February, two months before April.

    Person

    5th January 16th 22nd 5th April 16th 22nd 5th July 16th 22nd

    BPRE–760

    K R N P L Q O J M

    of Kiran Prakashan

    G et book s a n d m a g a z in e s o f K i ra n P r a k a s h a n a t y o u r d o o rs te p e a s il y , lo g o n o u r w e b s i t e : w w w .kira n p rak a s h a n.c o m

    PROBLEM SOLVING

    R di in H

    W Urdu

    (422-426) :

    W

    i al ng e B

    Malayalam T

    P Marathi

    S

    Q

    G

    h is gl n E

    +3

    P

    +3

    V

    Tamil

    Hindi (R)

    English (Q) +3

    W

    423. (4) T speaks Malayalam. U sits third to the left of T. 424. (5) S speaks Gujarati. Q sits second to the left of S. 425. (2) Except V–English, in all other pairs the first sits second to the right of the second. V sits to the immediate right of Q who speaks English. 426. (3) Q sits fourth to the right or the left of U. Either two or four persons sit between Q and P. Q sits third to the left of P. Q speaks English. (427-431) : Floor Person Cartoon Colour Number Character 7 O Flinstone Yellow 6 S Tweety Purple 5 Q Chipmnuk Red 4 N Popeye Black 3 M Scooby Brown Doo 2 R Simpson Orange 1 P Jetson Blue 427. (5) Q lives immediately below S. Q likes Chipmnuk and Red colour.

    i

    Bengali (U)

    +3

    at ar uj

    V

    422. (1) In clockwise direction

    +3

    (432-436) : Floor Per- Car Day Number son 7 G Skoda Wednesday 6 B Toyota Monday 5 A Hyundai Tuesday 4 E Fiat Friday 3 D Chevrolet Sunday 2 F Honda Saturday 1 C Ford Thursday 432. (4) E goes on tour on Friday by Fiat car. 433. (2) D goes on tour on Sunday by Chevrolet car. 434. (3) F ⇒ Saturday D ⇒ Sunday 435. (3) A lives on 5th Floor and he uses Hyundai car. 436. (5) Monday ⇒ B; Friday ⇒ E (437-441) : @ % & # 1 Watch Money Camera Cap 2 Pen Radio Toy Ring 3 Glows Book Clock Cap 4 Mobile Fruits Laptop Ball 437. (4) Glows ⇒ 3@ 438. (3) Radio ⇒ 2% 439. (4) 3# ⇒ Cap 440. (5) 1% ⇒ Money 441. (1) Toy box is immediate above right diagonally of cap box.

    428. (3) N and M live between Q and R. N likes black colour. 429. (1) R likes Simpson character. R lives exactly between M and P. 430. (4) P lives on bottom most floor. O lives on topmost floor. 431. (2) O likes Flinstone cartoon character and Yellow colour. (442-444) :

    P

    Q

    R

    S

    T

    U

    V

    3

    1

    4

    2

    6

    5

    4

    H ea S in

    rt

    g in

    Cl u g

    Da n

    b

    ci n g

    mo D ia

    Mim

    nd

    i cr y

    b C lu A ct

    442. (4) S got dice number 2, Club card and he performed Acting. 443. (5) P got dice number 3. P got Heart Card. P performed Singing. 444. (5) T performed Dancing. U got Spade Card. V got dice number 4. (445–449) : Floor Number 7 6 5 4 3 2 1

    Woman

    Clothes

    K M N O L J P

    Adidas Fila Wranglar Puma Nike HRx Levi’s

    BPRE–761

    ing

    He D an

    a rt g c in

    e S p ad Sin

    g gin

    S pa Ac t

    de

    i ng

    445. (2) M likes Fila brand. 446. (3) P likes Levi’s brand. 447. (1) K lives on topmost floor. 448. (4) Except O, all others live on odd numbered floors. 449. (5) K-Adidas combination is true. (450–454) : Rank MD ED CGM GM DGM AGM Officer

    Person G D F C A E B

    City Delhi Pune Mumbai Bangalore Solapur Nagpur Kolhapur

    PROBLEM SOLVING 450. (3) E is the AGM. 451. (1) A is working in Solapur. 452. (4) B, the officer works in Kolhapur. 453. (2) G, the MD works in Delhi. 454. (2) D is the ED. D works in Pune. (455–459) : Person P Q R S T U V W

    Salary 5000 3000 4500 7000 5523 6400 6533 2000

    Fruit Kiwi Apple Grapes Orange Papaya Banana Mango Guava

    455. (5) W likes Guava. 456. (2) R likes Grapes. His salary is 4500. V ⇒ 6533 Required difference = 6533 – 4500 = 2033 457. (4) P’s salary is 5000. 458. (1) U likes Banana. 459. (3) The salary of S is 7000. (460–464) : Day Monday Tuesday Wednesday C Thursday Friday Saturday Sunday

    Event S J E R Z F

    460. (4) Event E was held on Thursday and Event Z was held on Saturday. Event F was held after Event Z. Events E, R and Z were held between Event C and Event F. Thus, only Statement (B) is not true. 461. (1) Event Z was held on Saturday. 462. (3) Six events were held before Event F. 463. (5) Only one event was held before Event J. Two events were held after Event R. 464. (2) Event C was held on Wednesday.

    (465-469) : Floor Number Flat A Flat B 4 P T 3 W Q 2 U V 1 R S 465. (2) T lives in Flat B on floor number four. 466. (3) P lives on floor number four and V lives on floor number two. R lives on floor number one. S lives on a floor immediately below U. Thus, only statements (A) and (B) are true. 467. (1) Except SP, in all other pairs the first person lives immediately above the second person. Two persons live between the floors of P and S. 468. (5) U lives in Flat A on floor number 2. 469. (4) U lives in a flat immediately below W. (470–472):

    >

    >

    > A >

    >

    B > D > A

    473. 474. 475. 476. 477.

    (1) A likes Tuesday. (2) The favourite car of B is Figo. (4) E likes Sunday. (3) F likes SX4 car. (5) None of the given combinations is true. (478–482) : Time Professor 10.00 AM B 11.00 AM D 12.00 Noon F 1.00 PM C 2.00 PM E 3.00 PM A

    478. (1) F is the Professor of Physics. 479. (2) B took the lecture at 10.00 AM. 480. (5) C teaches Botany. 481. (3) D took the lecture at 11.00 AM. A has taken lecture on Economics at 3.00 PM. E has taken lecture at 2.00PM. 482. (4) E has taken lecture on Statistics at 2.00 PM. (483–487) : Day

    F > B > C E is not the lightest. Therefore, C is the lightest.

    F > B > D > A > E > C 79 kg

    62 kg

    470. (4) A = 68 kg Therfore, D = 75 kg 471. (3) C = (B – 24) kg = (79 – 24) kg = 55 kg E = 62 kg = (55 + 7) kg. Thus, only statements (i) and (iii) are true. 472. (5) Box is the second heaviest. (473–477) : Person A B C D E F G

    Day Tuesday Friday Wednesday Saturday Sunday Thursday Monday

    BPRE–762

    Car Estilo Figo Swift Optra Beat SX4 Alto

    Subject Mathematics Accounting Physics Botany Statistics Economics

    Monday Tuesday Wednesday Thursday Friday Saturday Sunday

    Conference F B D E C G A

    City Mumbai Bangalore Delhi Hyderabad Kolkata Chennai Pune

    483. (5) None of the combinations is correct. 484. (1) Three conferences – D, E and C – will be held between B and G. 485. (4) Except ‘B – Chennai’, in all other pairs the first will be held immediately before the conference held in the given city. 486. (3) Conference E will be held on Thursday and Thursday + 2 = Saturday. Conference C will be held on Friday and Friday + 2 = Sunday. Conference B will be held on Tuesday and Tuesday + 2 = Thursday. 487. (2) Conference F will be held in Mumbai.

    PROBLEM SOLVING (488–492) :

    498. (4) N lives on floor numbered two.

    July

    A

    23rd

    F

    14th

    E

    23rd

    B

    14th

    H

    23rd

    C

    14th

    D

    23rd

    G

    488. (4) D and G were born in July. 489. (5) G was born on 23rd July. 490. (2) The combination 14th April – E is correct. 491. (3) C has his birthday in May. 492. (1) Seven persons have their birthdays before G. (493–497) :

    Month February March April May June

    Date

    Exam

    11th 16th 11th 16th 11th 16th 11th 16th 11th 16th

    M T P S R Y Q Z N V

    493. (4) Only T was held between M and P. 494. (5) M was held on 11th February. V was the last exam. 495. (1) Except Y, all other exams were held on 11th of the months. Y was held on 16th April. 496. (5) Exam Z was held on 16th May. 497. (2) Exam Y was held immediately after R. (498–502) : Floor 6 5 4 3 2 1

    Person Y Z X L N M

    Colour Blue White Pink Green Red Yellow

    Column–1

    Column–2

    Column–4

    Floor No. 4

    Rs. 18,000 O

    Rs. 15,000 T

    Rs. 18,000 R

    Floor No. 3

    Rs. 16,000 Y

    Rs. 15,000 P

    Rs. 16,000 N

    Floor No. 2

    Rs. 14,000 B

    Rs. 20,000 V

    Rs. 15,000 A

    Floor No. 1

    Rs. 20,000 S

    Rs. 18,000 H

    Rs. 19,000 U

    503. (3) T + V = Rs. (15,000 + 20,000) = Rs. 35,000 504. (4) U pays Rs. 19,000 as rent per month. V lives immediate left above diagonally of U. 505. (5) S + U = Rs. (20,000 + 19,000) = Rs. 39,000 506. (2) S, H and U live on the bottom floor. H pays Rs. 18,000 as monthly rent. 507. (1) R, N, A and U live in the right end column flats. U pays Rs. 19,000 as monthly rent. (508–512) :

    g in us Ho R

    Irrigation

    May

    14th

    499. (5) Y likes Blue colour. Y lives on floor numbered six. 500. (2) L likes Green colour. Only X lives between Z and L. 501. (3) Z likes White colour. Y lives immediately above Z. 502. (1) L likes Green colour. (503–507) :

    T

    P

    Culture U

    509. (5) X belongs to the Commerce Department. 510. (2) O belongs to the Police Department. T belongs to the Energy Department. R sits exactly between O and T. 511. (1) U belongs to the Culture Department. O sits fourth to the right of U. O belongs to the Police Department. 512. (3) W belongs to the Welfare Department. (513–517) :

    Person P

    Month June

    City Mangalore

    B

    March

    Ooty

    R

    June

    Agra

    S

    December

    Chennai

    T

    June

    Goa

    U

    December

    Bengaluru

    Police O

    M Law

    V

    December

    Kolkata

    PWD S

    W Welfare

    M

    March

    Puducherry

    IT V

    N

    Q Finance

    April

    Person

    Energy

    January

    Date

    Medical

    Month

    X rce e mm o C

    508. (4) V belongs to the IT Department.

    BPRE–763

    513. (4) V belongs to Kolkata. 514. (5) S, U and V were born in December. S — Chennai U — Bengaluru V — Kolkata

    PROBLEM SOLVING 515. (2) M is from Puducherry. 516. (4) B was born in March. Six persons — P, R, S, T, U and V– are younger to B. 517. (2) M was born in March. U was born in December. Three persons were born between M and U. (518 –522) : Floor Number

    Column –1

    Column –1

    Column –1

    4

    Bangalore D

    Mumbai I

    Hyderabad C

    3

    Chandigarh H

    Chennai K

    Bhubaneswar G

    Person

    Country

    Colour

    2

    Patiala B

    Delhi L

    Trivandrum A

    A

    Brazil

    Pink

    Kochi F

    Coimbatore J

    Patna E

    B

    Brazil

    Red

    1

    C

    Hong Kong

    Green

    D

    Argentina

    White

    E

    Argentina

    Purple

    F

    Brazil

    Orange

    G

    Hong Kong

    Blue

    H

    Hong Kong

    Yellow

    518. (2) K is from Chennai. 519. (3) D lives second to the left of the person from Hyderabad (C). 520. (4) F is from Kochi and E is from Patna. J lives exactly between F and E. 521. (1) C is second to the right of D. D is from Bangalore. 522. (3) E is from Patna. F lives second to the left of E. (523–527) :

    Month January

    May June August

    531. (5) U likes Black colour. R sits opposite U. Either two or four persons sit between S and Q. P likes White colour. S faces the centre and P faces outside. 532. (5) T likes Green colour. U sits third to the left of T. U likes Black colour. (533–537) :

    Date 9 th

    Person

    525. (4) P conducted the lecture on 9th June. Only C conducted the lecture before B. B conducted the lecture in Chandigarh. Five persons – D, R, P, Q and S – conducted the lectures between B and A. 526. (1) D conducted the lecture on 9th May. 527. (5) D conducted the lecture in Pune. (528–532) :

    City

    C

    Lucknow

    18 th

    B

    Chandigarh

    9 th

    D

    Pune

    18 th 9 th

    R P

    Chennai Mumbai

    18 th

    Q

    Delhi

    9 th

    S

    Bengaluru

    18 th

    A

    Hyderabad

    S Blue

    524. (3) P conducted the lecture in Mumbai. Q conducted the lecture in Delhi. Except Q, all others conducted the lecture on 9th of the month. Q conducted the lecture on 18th June.

    V Orange

    Q Red

    P White

    T Green

    523. (2) A conducted the lecture in Hyderabad. Three persons –P, Q and S – conducted lectures between R and A.

    Black U

    R Pink

    W Yellow

    528. (2) Except ‘V–The one who likes Pink’ combination, in all others one person sits to the immediate right of the other person. R likes Pink colour. V sits third to the left of R. 529. (1) P likes White colour. 530. (4) R likes Pink colour. Three persons sit between R and U.

    BPRE–764

    533. (2) A works in Brazil. 534. (4) E likes Purple colour. E works with D who likes White colour. 535. (2) H likes Yellow colour. 536. (5) Except ‘A – Blue’, in all other pairs the first person works with the person who likes the given colour. G likes Blue colour. G works in Hong Kong. A works in Brazil. 537. (3) A likes Pink colour. F likes Orange colour. A, B and F work in Brazil. (538–542) :

    Floor Flat A Flat B Number K S 5 4

    J

    R

    3

    Q

    M

    2

    L

    T

    1

    P

    N

    538. (4) Q stays to the west of M. 539. (2) There are two floors between the floors of K and T.

    PROBLEM SOLVING 540. (5) J stays second above L in Flat A. 541. (3) J and R stay immediately below K. 542. (4) J and R stay on fourth floor. (543 – 547) : Person Sports City Amit Football Ranchi Binod Kabaddi Delhi Chirag Dheeraj Emran

    Basketball Hockey Tennis

    Farhan

    Cricket

    543. (2) Chirag likes Basketball. Chirag belongs to Shimla.

    558. (3) S teaches on Saturday.

    544. (5) Binod belongs to Delhi and likes Kabaddi.

    560. (4) FOur teachers teach between the classes of Chemistry and Zoology.

    545. (1) Binod likes Kabaddi. 546. (3) Chirag belongs to Shimla. 547. (4) Binod likes Kabaddi and belongs to Delhi. Farhan belongs to Kolkata and likes Cricket.

    Shimla Gurgaon Bhubaneshwar Kolkata

    Dheeraj belongs to Gurgaon and likes Hockey. Amit belongs to Ranchi and likes Football.

    (548 – 552) : Month January February

    Days 31 28

    Friend A E

    Profession Doctor Engineer

    March April July November

    31 30 31 30

    D B F C

    Manager Soldier Lawyer Accountant

    548. (2) A was born in the month of January. 549. (4) Except E, all others were born in the months having 30 or 31 days. E was born in February. 550. (3) F is a Lawyer. 551. (5) C is the youngest. 552. (1) The combination ‘D – Manager – March’ is correct. (553–557) : Row Column 1 Column 2 Column 3 4 D I C 3 H K G 2 B L A 1 F J E 553. (5) B lives to the immediate left of L. 554. (2) I lives exactly between the flats of D and C. 555. (1) A lives immediate right above diagonally of J. 556. (4) C lives in Column 3. E lives on the ground floor in the Column 3. 557. (3) K lives immediate left below diagonally of C. (558–562) : Day Monday Tuesday Wednesday Thursday Friday Saturday Sunday

    Teacher D R M G J S V

    Subject Chemistry Botany History Physics Mathematics Zoology English

    BPRE–765

    559. (1) M teaches History.

    561. (5) Physics is taught on Thursday. 562. (2) V teaches immediately after Zoology subject. (563–567) :

    Floor Number 4 3 2 1

    Person I II III J H C D A K F L I B G E

    563. (2) I lives just below the flat of K. 564. (4) Except D, all others live on topmost floor or ground floor. 565. (5) L lies exactly between the flats of F and I on the floor number two. 566. (1) J lives immediate left above diagmally of A’s flat. 567. (3) A, L and G lives with H in the same column. (568–572) : Box number

    Colour

    225

    Green

    151

    White

    221

    Yellow

    101

    Orange

    191

    Blue

    121

    Red

    231

    Pink

    568. (5) Blue coloured box is kept just above the box number 121. 569. (3) The number of Green coloured box is 225. 570. (2) The colour of box number 101 is Orange. 571. (4) Orange coloured box is kept just below the box number 221. 572. (1) Yellow coloured box is kept exactly between the box number 151 and Orange coloured box.

    PROBLEM SOLVING 588. (3) Except R, the other four persons live in Column–2. R lives in Column–3. 589. (1) B lives just below Y. 590. (5) A lives second to the right of B. Similarly, R lives second to the right of O. 591. (4) S and R do not live in Row– 2. 592. (2) P lives just above V.

    (573 – 577) :

    Row

    Column

    4

    Rs. 15,000 Rs. 12,000 Editor Engineer H K

    Rs. 10,000 Player F

    3

    Rs. 16,000 Rs. 13,000 Journalist Doctor J D

    Rs. 16,000 Police N

    Rs. 10,000 Rs. 14,000 Politician Manager E O

    Rs. 16,000 Writer I

    Rs. 11,000 Rs. 15,000 Lawyer Dancer

    Rs. 15,000 Carpenter

    2

    1

    L

    573. (4) =

    M

    M+F+H+L 4

    Rs.(15,000 + 10,000 + 15,000 + 11,000) 4

    Subject

    Building

    A

    English

    R

    B

    History

    V

    C

    Mathematics

    T

    D

    History

    W

    E

    English

    S

    F

    Mathematics

    P

    Mathematics

    Q

    578. (4) B stays in building V. 579. (2) Mathematics–G–Q combination is correct.

    1. (5) First Set of Numbers 15 11 ⇒ 15 – 11 = 4 [Rule (ii)] 4 20 ⇒ 4 × 20 = 80 [Rule (iv)] 80 400

    400 = 5 [Rule (iii)] 80 Second Set of Numbers 8 12 ⇒ 8 × 12 = 96 [Rule (iv)] 96 10 ⇒ 96 × 10 = 960 [Rule (iv)] ⇒

    G

    960 = 192 5 2. (2) First Set of Numbers 40 30 ⇒ 40 × 30 = 1200 [Rule (iv)] 1200 3600 Now,

    Month

    574. (1) Manager O ⇒ Rs. 14,000 Editor K ⇒ Rs. 12,000 575. (5) J lives in Coloum–1. 576. (3) N is a Police. G is a Carpenter. 577. (2) L + D + H = Rs. (11,000 + 13,000 + 15,000) = Rs. 39,000 (578–582) : Professor

    SBI PO EXAMS

    580. (3) C, F and G evaluate Mathematics Papers. 581. (1) D evaluates History Paper. 582. (5) E stays in building S. (583 – 587) :

    Rs.51,000 = Rs. 12,750 = 4

    G

    3

    2

    1

    January

    April May

    July

    Date

    Friend

    14th

    G

    23rd

    F

    14th

    E

    23rd

    B

    14th

    H

    23rd

    C

    14th

    A

    23rd

    D

    3600 = 3 [Rule (iii)] 1200 Second Set of Numbers 15 24 ⇒ 15 + 24 = 39 [Rule (i)] 39 17 ⇒ 39 – 17 = 22 [Rule (ii)] Now, 3 + 22 = 25 3. (4) First Set of Numbers 8 16 ⇒

    583. (2) E was born on 14 th April. 584. (5) H and C were born in May. 585. (4) F and G were born in January. 586. (1) D and A were born in July. 587. (3) A was born on 14th July. (588–592) : Column

    L E F T

    1

    2

    3

    4

    O

    T

    R

    3

    Y

    P

    N

    2

    B

    V

    A

    1

    S

    M

    U

    Row

    BPRE–766

    R I G H T

    16 = 2 [Rule (ii)] 8 2 16



    16 = 2 [Rule (iii)] 2 8 14 ⇒ 8 × 14 = 112 [Rule (iv)] Second Set of Numbers 13 11 ⇒ 13 – 11 = 2 [Rule (ii)] 2 12 ⇒ 2 × 12 = 24 [Rule (iv)] 24 144 ⇒

    144 = 6 [Rule (iii)] 24 Now, 112 – 6 = 106



    PROBLEM SOLVING 4. (2) First Set of Numbers 13 11 ⇒ 13 – 11 = 2 [Rule (ii)] 2 4

    4 ⇒ = 2 [Rule (iii)] 2 Second Set of Numbers 17 13 ⇒ 17 – 13 = 4 [Rule (ii)] 4 12 ⇒ 4 × 12 = 48 [Rule (iv)] Now, 48 × 2 = 96 5. (1) First Set of Numbers 19 15 ⇒ 19 – 15 = 4 [Rule (ii)] 4 12 ⇒ 4 × 12 = 48 [Rule (iv)] Second Set of Numbers 15 12 ⇒ 15 + 12 = 27 [Rule (i)] 27 23 ⇒ 27 – 23 = 4 [Rule (ii)] 4 16 16 ⇒ = 4 [Rule (iii)] 4 Now, 48 – 4 = 44 (6-10) : Person A B C D E F G H

    Gender Male Female Female Male Male Male Male Female

    Car Y Z X Z X X Z Y

    Colour Grey Blue Red Yellow White Purple Black Green

    6. (2) A and H are travelling in the Car Y. 7. (4) C is the female member in the Car X. She likes Red colour. 8. (1) D likes Yellow colour. 9. (3) E and F are travelling with C. 10. (5) The combination E–Y–Grey is not correct. E travels in Car X and he likes white colour. (11–13) : V > X > U > Y > W > Z ↓ ↓ 17 days 8 days 11. (3) W stayed for 8 days. V stayed for maximum number of days. 15 + 8 = 23 days. 12. (2) Z stayed for the minimum number of days.

    13. (1) Y stayed for more than 8 days but less than 17 days. (14–19) : Floor number 8 7 6 5 4 3 2 1

    Person W U P T V R S Q

    Subject Economics English History Mathematics Hindi Geography Sociology Statistics

    14. (3) Professor of Hindi, V lives exactly between the floors of T and professor of Geography R. 15. (1) The professor of Economics W lives on the topmost floor. 16. (4) Professor of Sociology S lives immediately above the floor of professor of Statistics Q. 17. (2) Professor of Geography R lives on the third numbered floor. 18. (4) Four persons - U, P, T and V-live between the floors of W and the professor of Geography R. 19. (5) Except the Professor of English, all others live on even numbered floors. (20–24) : Floor Number 8 7 6 5 4 3 2 1

    Person Q V S P T R W U

    20. (3) Two people – P and T – live between the floors on which S and R live. 21. (1) S lives on the floor immediately above P. 22. (2) W lives immediately above the floor of U. U lives on floor number 1. Q lives on the top most floor. Only one person lives between T and S. P lives immediately above T. 23. (5) W lives on floor number 2. 24. (4) W lives between R and U. (25–27) : ,

    > P >

    ,

    U>S>T

    BPRE–767

    ,

    ,

    >P>U>S>T

    R>Q>P>U>S>T





    5 feet 2 feet 25. (4) The height of pole U should be more than 2 feet but less than 5 feet. 26. (2) R is the tallest Pole. Height of R = (11 + 2) feet = 13 feet The height of Q should be more than 5 feet but less than 13 feet. i.e. 9 feet. 27. (5) Four poles (P, U, S, T) are shorter than Q. Q’s height is more than 5 feet. Q is the second tallest pole. (28 – 30) : V > X > U >

    Y

    > W >





    17 days

    8 days

    Z

    28. (3) W stayed for 8 days. V stayed for maximum number of days. 15 + 8 = 23 days. 29. (2) Z stayed for the minimum number of days. 30. (1) Y stayed for more than 8 days but less than 17 days. (31-35) : Floor Number

    Person

    8

    E

    7 6 5 4 3 2

    B D G A C H

    1

    F

    31. (4) G lives on the floor immediately below D. 32. (2) Two persons – B and D – live between E and G. A lives immediately above C. F lives on floor number 1. 33. (3) Two persons – G and A – live between the floors on which D and C live.

    PROBLEM SOLVING 34. (5) D lives between B and G. 35. (1) G lives on floor number 5. (36–40) : Floor Number 7 6 5 4 3 2 1

    Person O S Q N M R P

    Cartoon Character Flinstone Tweety Chipmunk Popeye Scooby Doo Simpson Jetson

    36.(3) Four persons — Q, N, M and R — live between the floors of S and P. 37. (5) All the statements are true. 38. (1) N lives on the floor immediately above the floor on which M lives. 39. (4) Q lives exactly between the floors on which S and N live. 40. (2) P likes cartoon character Jetson. (41 – 45) : Floor No. Person Games 7 6 5 4 3 2

    B A L D K M

    Badminton Polo Chess Hockey Snooker Cricket

    1

    C

    Ludo

    Alphabetical Order A B C D K L M

    41. (4) B likes Badminton 42. (1) D lives on the floor numbered 4. He likes Hockey. 43. (2) K lives immediately above M. D likes Hockey and he lives immediately above K. C likes Ludo and he lives on an odd numbered floor. 44. (3) The positions of D and K remain unchanged. 45. (5) The combination Chess - L is correct. (46 – 50) : Floor Number 7 6 5 4 3 2

    Person

    Subject

    P K J N M O

    Biology Accounts History English Computer Commerce

    1

    L

    Geography

    46. (4) J likes History. 47. (1) The combination Geography L is correct. 48. (2) Floor Number 7 6 5 4 3 2 1

    Person P K J N M O L

    Alphabetical order J K L M N O P

    The positions of K and O remain unchanged. 49. (3) The one who likes Computer lives immediately below N. O likes Commerce. Only three persons live between P and M. P lives on the topmost floor 50.(5) O lives on the floor numbered 2. (51-57) :

    Floor Number

    Person

    8

    P

    7

    T

    6

    U

    5

    R

    4

    V

    3

    W

    2

    Q

    1

    S

    51. (3) Q lives on the floor number 2. 52. (1) V, W and Q live on floors exactly between the floors of S and R. 53. (4) T → Floor Number 7 W → Floor Number 3 S → Floor Number 1 54. (2) P lives on the topmost floor. 55. (1) R lives on the floor number 5. 56. (5) Four persons – U, R, V and W – live between the floors of T and Q. 57. (3) V lives on the floor immediately above the floor of W.

    BPRE–768

    (58-60) : Statue L is smaller than only statue M. Therefore, M is the tallest statue. N>O>P Now,

    M >

    L > N >

    20 feet

    O > P

    11 feet

    58. (3) Statue P is the smallest. The height of the second smallest statue O is 11 feet. Therefore, the height of statue of P will be less than 11 feet. 59. (1) The tallest statue is 20 feet high. Therefore, the second tallest statue may be 19 feet, 18 feet ..... high. Therefore, the third tallest statue will be less than 19 feet and more than 11 feet high. 60. (4) S = T > O > R PP Z>O> R PR P is smaller than T. (61–63) : 5 Top/Fifth Floor B 4 Fourth Floor Vacant 3 Third Floor D 2 Second Floor C 1 First Floor E Ground Floor A 61. (3) E lives immediately above A’s floor. 62. (4) B lives on the topmost floor. 63. (1) Fourth floor is vacant. (64–68) : Floor Number 8 7 6 5 4 3 2 1

    Person Q V S P T R W U

    64. (3) Two people – P and T – live between the floors on which S and R live. 65. (1) S lives on the floor immediately above P.

    PROBLEM SOLVING 66. (2) W lives immediately above the floor of U. U lives on floor number 1. Q lives on the top most floor. Only one person lives between T and S. P lives immediately above T. 67. (5) W lives on floor number 2. 68. (4) W lives between R and U. (69-73) : Floor Number 8 7 6 5 4 3 2 1

    Person E B D G A C H F

    69. (4) G lives on the floor immediately below D. 70. (2) Two persons – B and D – live between E and G. A lives immediately above C. F lives on floor number 1. 71. (3) Two persons – G and A – live between the floors on which D and C live. 72. (5) D lives between B and G. 73. (1) G lives on floor number 5. (74–78) : Floor Number Floor Number 8 Floor Number 7 Floor Number 6 Floor Number 5 Floor Number 4 Floor Number 3 Floor Number 2 Floor Number 1

    Person T P V Q S U W R

    74. (1) V lives immediately above Q’s floor. 75. (1) Except R S, in all others three persons live between the given pair of persons. Only two persons live between R and S. 76. (4) Q lives on floor numbered five. Three persons live above Q’s floor. Only S lives between Q and U. 77. (2) T lives on the topmost floor. 78. (3) P lives on floor numbered seven.

    (90–95) :

    (79–84) : Day

    Representative P

    Monday

    Country

    South Africa Tuesday Q Australia Wednesday T France Thursday U Australia Friday R South Africa Saturday S France Sunday V South Africa 79. (3) Q is travelling on Tuesday. Therefore, Q will travel on Wednesday. 80. (1) S travelled on Saturday. 81. (5) None of the combinations is correct. 82. (4) Q travelled to Australia on Tuesday. 83. (3) V travelled on Sunday. 84. (2) P, R and V travelled to South Africa. (85–89) : Month

    Person

    January

    R

    February

    N

    April

    O

    May

    L

    July

    Q

    September

    M

    December

    P

    Favourite TV Shows Family Shows Thriller Shows Reality Shows Action Shows Animated Shows Comedy Shows History Shows

    85. (5) O likes Reality Shows. 86. (1) O will appear for an exam between N and L. 87. (4) Only one person will appear in exam between the one who appears in January and the one who likes Reality Shows. Q appears in exam in July and P appears in an exam in December who likes History Shows. 88. (2) L will appear in an exam in May. 89. (3) R will appear in an exam in January and P will appear in an exam in December.

    BPRE–769

    Month February March May June August November December

    Person P N M J O K L

    Fruit Apple Cherry Grapes Banana Strawberry Orange Mango

    90. (3) O likes Strawberry. 91. (1) J attended a wedding ceremony in the month of June. M attended a wedding ceremony in the month of May. 92. (4) The one who likes Apple attended a wedding ceremony in the month of February, immediately before the month of March. The one who likes Banana attended a wedding ceremony in June immediately before the month of August. The one who likes Cherry attended a wedding ceremony in June, in the month of March, immediately before the month of May. 93. (2) June–Banana combination is correct. 94. (5) N attended a wedding ceremony in the month of March. Three persons – M, J and O – attended wedding ceremonies between N and K. 95. (3) P ⇒ February and K ⇒ November. (96–100) : Box Box M Box P Box O Box R Box S Box Q Box N

    Colour Brown Orange Green Silver Pink Yellow White

    96. (5) Box M is of Brown colour. There are three boxes between Box M and Box S. 97. (2) Box S is kept immediately below the Box R. 98. (4) Box O is of green colour. 99. (1) Box R is exactly in the middle of the stack. Therefore, Box R is fourth from top or bottom in the stack.

    PROBLEM SOLVING 100. (3) Box R is of silver colour. There is only one box between Box P and Box R. Box N is of white colour. There is only one box between Box S and Box N. Box S is of pink colour. There is only one box between Box O and Box S. Box P is of orange colour. There is only one box between Box O and Box P. Box Q is of yellow colour. There are four boxes between Box M and Box Q. (101–106) : Person C D E F G H I

    Favourite City Ranchi Surat Bangalore Mumbai Delhi Kolkata Pune

    School VCS VCS RIS RIS DAS DAS VCS

    101. (2) E studies in RIS school and likes Bangalore. 102. (4) G likes Delhi. 103. (1) Except DF, in all others both the persons study in the same school. 104. (3) H likes Kolkata. 105. (5) The ones who likes Surat, Pune and Ranchi study in the VCS school. 106. (1) F studies in the RIS school with E. (107–112) : Person

    Standard

    J K L M N O P

    XI VIII VIII XI VIII IV IV

    Cartoon Character Popeye Superman Jerry Tweety Ariel Garfield Wiene

    107. (4) P likes Wiene. 108. (4) L likes Jerry. 109. (1) Both J and M study in standard XI. K studies in Standard VIII. J studies with the one who likes Tweety. Only P studies with O in Standard IV. 110. (3) K, L and N study in Standard VIII.

    111. (2) Except P M, in all others both the persons study in the same standard. 112. (5) O studies in standard IV. He likes Garfield. (113–118) : Month Person Activity January V Yoga February U Surfing April Q Gym Training May S Running July P Zumba August R Bicycle September T Mountaineering 113. (4) The person who likes Yoga has a meeting in the month immediately before February. The person who likes Gym Training has a meeting in the month immediately before May. The person who likes Zumba has a meeting in the month immediately before August. 114. (3) P has a meeting in July. S has a meeting in May, immediately before July. 115. (2) ‘V–Yoga’ is correct. 116. (1) U likes Surfing. 117. (5) U has a meeting in February. Three persons – Q, S and P – have a meeting between U and R. 118. (3) U has a meeting in February (28 or 29 days). Q has a meeting in April (30 days). T has a meeting in September (30 days). (119-124) : Month Person January G February B April E June A August C October F December D

    Activity Singing Craft Karate Drawing Painting Boxing Running

    119. (5) D has an appointment in December. F has an appointment immediately before D. F likes Boxing. 120. (3) B has an appointment in February (28 or 29 Days). E has an appointment in April (30 Days). A has an appointment in June (30 Days). 121. (1) The combination G – Singing is correct.

    BPRE–770

    122. (4) B likes Craft and he has an appointment in February just before E who has an appointment in April. E likes Karate and he has an appointment in April just before A who has an appointment in June. Similarly, A likes Drawing and he has an appointment in June just before C who has an appointment in August. 123. (2) D likes Running. 124. (4) B has an appointment in February. Only E has an appointment between B and A. (125-128) : Person A B C D E F G

    Subject History English Accountancy Chemistry Physics Biology Mathematics

    Institute Embibe Embibe Brilliant Paramount Embibe Paramount Brilliant

    125. (2) E teaches Physics. 126. (1) C works in Brilliant Institute. He teaches Accountancy. 127. (5) A teaches History. 128. (3) D teaches Chemistry. Only C and G teach in Brilliant. E works with A and B in the Embibe. A teaches History. 129. (4) D and F work in the Paramount Institute. D teaches Chemistary. 130. (4) F and G work in the different institutes. B and E work in the same institute – Embibe. G and C work in the same institute – Brilliant E and A work in the same institute – Embibe. A and B work in the same institute – Embibe. (131–133) : Employee City Department P Yelahanka Operation Q Bengaluru Personnel T Jodhpur Accounts B Ooty Operation R Gwalior Operation J Palampur Personnel N Raipur Accounts A Warangal Accounts

    PROBLEM SOLVING 131. (4) P, B and R work in the Operation department. P likes Yelahanka, B likes Ooty while R likes Gwalior. 132.(1) The combination Jodhpur – Accounts is correct. 133.(3) Q works in the Personnel department. N works in the Accounts department. R works in the Operation department.

    134.(5) P likes Yelahanka. 135.(2) P works in the Operation department and he likes Yelahanka. A works in the Accounts department and he likes Warangal. B works in the Operation department and he likes Ooty. N works in the Accounts department and he likes Raipur.

    (136–140) : Position Trainee (TE) Assistant Manager (AM) Manager (MG) Senior Manager (SM) Chief Manager (CM) Executive Director (ED) Director (DR)

    Profession Teaching Knitting Grinding Stitching Marketing Colouring Gardening

    136. (1) G is a Trainee (TE). 137. (5) B is the Chief Manager (CM) and she is from Marketing. 138. (4) G is from Teaching. There is gap of two women between E and G. C is from Colouring. There is a

    Woman G A F E B C D

    gap of two women between F and C. E is from Stitching. There is a gap of two women between D and E. 139. (3) F is the Manager (MG). 140. (2) A is less experienced than F. C is more experienced than B.

    147. (4) Except G – Floor number eight combination, in all others the first person lives immediately above the second person. 148. (3) G performs Lavani and P performs Bhangra. Only T lives between G and P. 149. (2) A performs Dandiya. 150. (1) M performs Sattriya. (151–155) : Month

    Date

    April

    4th 17th 20th 29th 4th 17th 20th 29th

    December









    (141–145) : Month January April September December

    Date 11th 24th 11th 24th 11th 24th 11th 24th

    141. (4) Four persons – E, W, C and X – have to attend a wedding between F and Z. 142. (5) X has to attend a wedding on 11th December. 143. (1) The schedule of three persons – D, W and Z – will remain unchanged. 144. (3) Y has to attend a wedding on 11th January. 145. (2) Except Z, all others have to attend a wedding on 11th of the month.

    Person Y D F E W C X Z

    Alphabetical order C D E F W X Y Z





    (146–150) : Floor Person Dance Number 8 X Garba 7 A Dandiya 6 C Mohiniattam 5 G Lavani 4 T Odissi 3 P Bhangra 2 M Sattriya 1 J Kathak 146. (5) C performs Mohiniattam. X performs Garba and he lives on floor number 8. Two persons live between T and J.

    BPRE–771







    Person Case-I Case-II D C R Q R C P B A S B P S A

    A has a party in December on an odd numbered date, i.e., either on 17th or 29th December. Only three persons have to attend a party between A and C. Therefore, C will have a party on 17th April or 29th April. In Case-I, six persons have to attend a party after C. Therefore, B will attend a party on 20th December. In Case-II, four persons have to attend a party after C. Therefore, B will attend a party on 4th December. Only one person has a party between P and B. P does not have to attend a party in April. Theredore, in Case-I, P will attend a party on 4th December and in Case-II on 20th December. Both S and R have to attend a party on the same date. S has to attend a party after R. Therefore, in Case-I, R will attend a party on 29th April and that of S on 29th December. In case-II, R will attend a party on 17th April and that of S on 17th December. Only two persons have to attend a party between S and Q. In Case-I, this condition cannot be satisfied. But in Case-II, Q will attend a party on 20th April. Now, there is only one vacant space in the Case-II and the schedule of D has not yet been ascertained. Thus, D will attend a party on 4th April. Therefore, we have to discard the arrangement of Case-I.

    PROBLEM SOLVING 151. (2) Only one person R has to attend a party between Q and D. 152. (5) Q has to attend a party on 20th April. 153. (4) D has to attend a party on 4th April. B has to attend a party on 4th December. 154. (3) (I) S has to attend a party on 17th December. (II) Four persons – C, B, S and P – have to attend a party between Q and A. (III) R has to attend a party on 17th April. C has to attend a party on 29th April. 155. (1) Month April

    Date 4th 17th 20th 29th 4th 17th 20th 29th

    December

    Person H L U Z Q V E N



    Person D R A P B S C Q

    Now, three persons – P, B and S – have to attend a party between A and C. (156–160) : Floor Number 8 7 6 5 4 3 2 1



    D lives on either floor number five of floor number seven as he lives on an odd numbered floor above floor number three. E is specialised in Aeronautical Engineering and he lives on an odd numbered floor. The number of floors between the one who is specialised in Aeronautical Engineering and Mechanical Engineering is two. Thus, E lives on floor number seven as he cannot live on floor number one. Now, D would occupy floor number five. There are two floors between E and H. Therefore, H would occupy floor number four. J lives just above who is specialised in Aeronautical Engineering. Thus, J would occupy floor number eight. The one who is specialised in Chemical Engineering would occupy floor number three as the number of floors between the one who is specialised in Chemical Engineering and J is four. On the basis of above information, we can construct the following table :

    Floor Number 8

    J

    156. (1) Except the combination ‘VFloor Number Six’, in all other combinations, no one lives between the given pair of persons. 157. (4) H plays Jurassic Park. 158. (5) All the given statements are true. 159. (2) U plays Payday. 160. (3) V plays Gothic. Two persons live between U and V. (161–165) : ● The one who lives on fourth floor is specialised in Mechanical Engineering. The one who is specialised in Instrumentation Engineering lives on floor number one.

    7

    E

    6 5 4

    D H

    3 2 1 ●







    The number of persons between D and who is specialised in Electrical Engineering is same as the number of persons between D and I. This statement also confirms that I lives on floor number two. K lives on an even numbered floor and he is specialised in Automobile Engineering. The vacant even number floor is six. Now, only one specialisation, i.e., Software Engineering remains unspecified. Therefore, I has specialisation in Software Engineering. Now, we have information to construct the complete table as follows :

    Floor

    Person Specialisation

    Number 8

    J

    Electrical Engineering Aeronautical Engineering

    Electrical Engineering

    7

    E

    Aeronautical

    6

    K

    Automobile

    5

    D

    Engineering Engineering

    Person Specialisation

    Video Game Jurassic Park Darkstone Payday Victoria Magicka Gothic Rayman Batman



    Civil Engineering

    4

    H

    Mechanical Engineering

    3

    F

    2

    I

    Chemical Engineering

    Mechanical Engineering Chemical Engineering

    Software Engineering

    1

    G

    Instrumentation Engineering

    Instrumentation Engineering

    The one who is specialised in Civil Engineering lives on an odd numbered floor. Thus, D is specialised in Civil Engineering. The number of floors between the one who is specialised in Civil Engineering and the floor on which F lives is same as the number of floors between F and G. Therefore, F would occupy floor number three and G would occupy floor number one. I lives below the floor on which D lives. Therefore, I would occupy floor number two.

    BPRE–772

    161. (3) F is specialised in Chemical Engineering and he lives on Floor Number three. J is specialised in Electrical Engineering and he lives on the topmost floor, i.e., floor number eight. Four persons – E, K, D and H – live between the floors of J and F. 162. (2) J is specialised in Electrical Engineering. 163. (1) Except G, all others live on even numbered floors. 164. (4) D lives on floor number five.

    PROBLEM SOLVING 165. (5) The number of floors between G and the one who is specialised in Mechanical Engineering is two. The number of floors between F and the one who is specialised in Automobile Engineering is two. (166–167) : Box

    K M J L

    Fruit Litchi Grape Apple Mango

    166. (3) Litchi is stored in box K. 167. (2) Second lowest box is J. Apple is stored in the box J. (168–172) : Box

    Element

    Box N

    Colours

    Box O

    Watches

    Box K

    Diaries

    Box M

    Spoons

    Box L

    Pencils

    Box P

    Jewellery

    Box J

    Cookies

    168. (4) Box K is kept immediately above Box M. Box K contains Diaries. 169. (2) Box O is second from the top. 170. (3) Box M contains Spoons. 171. (1) Box L contains Pencils. In all others, there is a gap of one box between the given Box and Elements. Box M contains Spoons. Box L is between Boxes M and P. Box N contains Colours. Box O is between Boxes N and K. Box K contains Diaries. Box O is between Boxes N and K. Box M contains Spoons. Box K is between O and M. 172. (5) Box O contains Watches. Box O is immediately above Box K. (173–177) : Box O R N Q S M P

    Shape Conical Octagonal Rectangular Square Triangular Pentagonal Hexagonal

    173. (4) Box N is kept immediately below Box R. 174. (2) Box N is rectangular in shape. 175. (5) Box M is kept immediately below the Triangular box. Box M is kept immediately above the Hexagonal box. Box O is kept immediately above the Octagonal box. Box S is kept immediately below the Square box. But, there are three boxes between S and Conical box. 176. (3) Box S is third the bottom and fifth from the top. 177. (1) Two boxes – S and M – are kept between P and Square box. (178–182) : Box C E G A F B D

    Item Medicines Ribbons Bands Creams Shoes Papers Phones

    178. (3) Box A contains creams. Box G is immediately above Box A. Box G contains Bands. Box E is immediately above Box G. Box B contains papers. Box D is immediately below Box B. Box G contains Bands. Box A is immediately below Box G. But, Box F contains shoes. There are three boxes between C and F. 179. (5) Box G contains Bands. 180. (2) Box F is third from the bottom. 181. (1) Box A is kept immediately below Box G. 182. (4) Box E contains Ribbons. There are three boxes between B and E. (183–185) :

    ,

    > C > F,D

    ,

    E>A>F B>E>A>C>D>F







    15 9 4 183. (1) C bought more than four but less than nine shirts. 184. (5) E bought more than nine but less than 15 shirts. 12 is a multiple of 3.

    BPRE–773

    185. (2) E bought the second highest number of shirts. (186–190) : Fruit Orange Banana Pineapple Cherry Litchi Apple

    Bowl Bowl Y Bowl U Bowl W Bowl X Bowl V Bowl Z

    Grapes

    Bowl T

    186. (2) There are two bowls – X and V – between W and Z. 187. (4) Bowl Y contains Orange. 188. (1) Bowl U contains Banana. 189. (3) Bowl W contains Pineapple. Bowl W is just above Bowl X. Bowl X is immediately below the Bowl W which contains Pineapple. 190. (5) Bowl T contains Grapes. (191–196) : A > D > C, F E > A > D > C > B > F ↓ ↓ 34 19 191. (2) E borrowed the highest number of books. 192. (5) F borrowed less than 19 books. 193. (1) C borrowed more number of books than B but less number of books than D. 194. (3) A borrowed 34 books. 195. (4) B borrowed more number of books than only F. 196. (3) C borrowed 26 books. D borrowed more than 26 but less than 34 books. (197–201) : Person A

    Fruit Orange

    B C

    Watermelon Banana

    D E F G

    Litchi Mango Guava Apple

    Department Research & Development Research & Development Research & Development Production Production Marketing Marketing

    197. (3) The ones who like Banana, Watermelon and Ornage work in the Research & Development department. 198. (1) D works in the Production department and he likes Litchi.

    PROBLEM SOLVING 199. (5) F works in the Marketing department and he likes Guava. 200. (4) Except C and E, all other pairs works in the same department. 201. (2) C likes Banana. (202–207) : Month

    Person

    January February April May June September November

    Favourite Movie Tangled X–Men Shrek Transformers Minions Rio Frozen

    P Q L M K O N

    202. (4) Q likes X–Men. 203. (5) Three persons – K, O and N – will attend a concert after M. 204. (1) K will attend a concert immediately before O and N will attend a concert immediately after O. 205. (3) P attends a concert in January. Q attends a concert immediately after P. M attends a concert in May. K attends a concert immediately after M. L attends a concert in April. M attends a concert immediately after L. 206. (2) K will attend a concert in June. 207. (3) K likes Minions. (208–210) : >

    >

    >

    >I>

    J > K > L, M N>J>K>M>I>L





    31 12 208. (3) J handles more than 31 Projects. 209. (5) I + L = 31 + 3 ⇒ I + 12 = 34

    ∴ I = 34 – 12 = 22 210. (1) N handles the maximum number of Projects. N handles more than 36 Projects. N handles more number of Projects than remaining five persons. The difference between projects handled by N and L is more than 18.

    (211–213) : ,

    ,

    >O>

    ,M

    L>N>P>M Q>L>N>O>P>M





    32 15 211. (3) N has translated less than 32 but more than 15 books. 212. (4) Q has translated the highest number of books. Q has translated more than 32 books. The difference between number of books translated by P and Q is more than 17. 213. (2) M has translated less than 15 books. (214-216) : ,

    Z>X>

    ,

    ,

    U>V>Y W > Y, U Z>X>W>U>V>Y





    39 24 214. (3) W + Z = Y + 66 ⇒ 39 + Z = 24 + 66

    ,

    ⇒ Z = 90 – 39 = 51 215. (4) Section V has more than 24 but less than 39 students. 216. (2) Sections V and Y have less number of students than Section U. Section U has less than 39 students. Section U has more number of students than only Sections V and Y. (217–219) : ,

    >U>

    ,

    ,

    P>R U > S > P, R Q>T>U>S>P>R

    ↓ 72 217. (5) Q sold more than 72 books. 218. (2) Store P sold the second lowest number of books. 219. (3) P + T = 125

    ⇒ P + 72 = 125 ∴ P = 125 – 72 = 53

    BPRE–774

    (220–224) : Year Person 1961 B 1970 C 1974 H 1980 G 1983 A 1987 D 1996 F 2000 E 220. (2) Except B, all others were born in even numbered years. 221. (3) E was born in 2000. He is youngest. 222. (1) F was born in the year 1960. 223. (4) The difference between ages of B and A is 22 years. Therefore, B is the uncle of A. The present age of B is 56 years. 224. (5) The present age of B = 56 years The present age of H = 43 years Required difference = 56 – 43 = 13 years (225–227) : ,

    >I>

    ,

    D > F > G, I I>G H>D H > D > F > I > G, E

    ↓ 62 225. (5) E + D = 135 ∴ D = 135 – 60 = 75 Now, D + F = 75 + 62 = 137 226. (1) F > I > E > G









    62 61 60 59 or less Clearly, G sold the lowest number of fruits. 227. (3) D sold 75 fruits. H sold more than 75 fruits. (228–232) : ● The one who was born in the year 1955, is 62 years old at present. ● The one who was born in the year 1963, is 54 years old at present. ● The one who was born in the year 1977, is 40 years old at present. ● The one who was born in the year 1982, is 35 years old at present. ● The one who was born in the year 1988, is 29 years old at present.

    PROBLEM SOLVING The one who was born in the year 1993, is 24 years old at present. ● The one who was born in the year 2000, is 17 years old at present. ● The one who was born in the year 2006, is 11 years old at present. ● M was born after 1993. It means M was born in the year 2000 or 2006.



    Year 1955 1963 1977 1982 1988 1993 2000 2006

    Person Case-I Case-II — K L L J P N Q Q J P N M O — M

    O was born in an even numbered year but he is not 11 years old. Therefore, O was born in the year 2000 and not in 2006. Thus, we can reject Case-I. The blank space will be occupied by K. 228. (2) M was born in the year 2006. Therefore, M is the youngest. 229. (3) Present age of K = 62 years Present age of P = 40 years Difference = 62 – 40 = 22 years K was born in an odd numbered year. K is the oldest. K is 62 years old. 230. (1) N is 24 years old. 231. (5) P was born in the year 1977. 232. (4) J = 29 years L = 54 years (233–237) : ●

    Year 1946 1967 1972 1982 1984 1989 1992

    Person T P Q R S V U

    Age in years 71 50 45 35 33 28 25

    Square root of 25 = 5 Therefore, Q – R = 5 × 2 = 10 Q was born in 1972 and R was born in 1982. (Remember R is not the youngest) 50 is the highest multiple of 5. Therefore, P was born in 1967. The age of S may be 71, 33, or

    28 years and the difference between the ages of R and S would be 36, 2 or 7 years. If we divide 28 by 14 we get 2. Therefore, S is 33 years old. 233. (4) T is the oldest. 234. (2) The age of R is 35 years. 235. (5) U is the youngest. 236. (3) The age of S is 33 years. 237. (1) V was born in 1989. (238–242) : Month March June October November

    Date 10th 27th 10th 27th 10th 27th 10th 27th

    Person M X W N Z O V Y

    238. (4) M attends his seminar on 10th March. 239. (2) X attends his seminar on 27th March. 240. (1) Five persons – N, Z, O, V and Y – attend seminar after W. 241. (3) Z attends his seminar on 10th October. 242. (5) Only Y attends seminar after V. (243–246) : Year 1988 1991 1993 1996 1999 2001 2006

    Person B Q P D R C A

    City Firozabad Pune Vellore Delhi Indore Anand Bhopal

    243. (4) Q was born in Pune. No one was born between P and Q. 244. (2) A was born in Bhopal. 245. (5) P was born in 1993. 246. (3) A was born in 2006. Only one person was born between R and A. R was born before A. D was born in Delhi. Only one person was born between Q and D. Q was born before D. Only one person was born between D and C. D was born before C. C was born in Anand.

    BPRE–775

    (247–250) : Person

    Class

    A B C D E F

    IV IX IX IV I I

    Favourite Colour Yellow Pink Green Blue Silver Orange

    247. (5) Except C-Yellow, in all other pairs both the persons study in the same Standard. C studies in Standard IX. A likes yellow colour. A studies in Standard IV. 248. (3) C likes Green colour. C studies in Standard IX. B and C study in the same Standard. A studies in Standard IV. 249. (4) F likes orange colour. 250. (2) C studies in Standard IX. (251–254) : Box Colour Box E White Box I Pink Box G Grey Box H Black Box F Blue 251. (4) Box H is Black in colour. Only one box is kept between I and H. 252. (2) There are three boxes between E and F (Blue box). Statements (ii) and (iii) are correct. 253. (1) Box H is Black in colour. 254. (5) Box I is Pink in colour. (255–258) : Math

    Person

    Ice Cream

    January March

    P V

    Chocolate Butter Pecan

    April

    Q

    Mango

    June August

    W T

    Vanilla Kulfi

    September October

    R X

    Strawberry Rocky Road

    November

    S

    Cookies

    255. (2) S likes Cookies. 256. (4) P likes Chocolate and he attends lecture in January (31 days). V likes Butter Pecan and he attends lecture in March (31 days). T likes Kulfi and he attends lecture in August (31 days).

    PROBLEM SOLVING

    Mother

    31years

    Q

    8 years

    V

    32 years

    J

    3 years

    W

    33 years

    L

    1 years

    Y

    34 years

    K

    2 years

    U

    35 years

    P

    5 years

    R

    36 years

    O

    7 years

    T

    37 years

    N

    6 years

    X

    38 years

    M

    4 years

    S

    ● ●















    ● ● ● ●

    S Q

    T

    J W

    O

    K

    N

    U

    Y

    X

    Age of Child Child

    Age of L’s son is 1 year. Age of K is 34 years. Only three persons sit between O and K. N is older than Q : N > Q. Age of N is 37 years and that of Q’s age is 31 years. Age of L is divisible by 11. Therefore, age of L is 33 years. N is the mother of X. Age of X is 6 years. N sits to the immediate left of R. T is the daughter of O. Age of O is a perfect square, i.e, 36 years. Age of T is 7 years. T sits to the immediate right of Q. U is younger than S and difference between their ages is 2 years. Age U is 2 years and that of S is 4 years. R is the child of P. Age of R is divisible by 5 and hence age of R is 5 years. R faces inside the circle.

    M V

    P

    L

    R

    259. (1) M (38 years) is the oldest and Q (31 years) is the youngest. 260. (5) Except Y, all others are sitting around the larger circular table. 261. (2) J is the mother of W. Age of J is 32 years. T is the child of O. Age of T is 7 years. Required sum = 32 + 7 = 39 years. 262. (4) S is the child of M. U is the child of K. 263. (3) Age of S is 4 years. J and Q are immediate neighbours of S. Age of J is 32 years and age of Q is 31 years. (264–268) : H lives on an odd numbered floor. Only one person lives between H and R. Therefore, R also lives on an odd numbered floor. The sum of the ages of Q and L is one less than the age of R. Q is younger than L. Age of R would be in even number. Therefore, Age of L = 24 years Age of Q = 17 years ∴ Age of R = 24 + 17 + 1 = 42 years Age of T = 17 × 2 + 2 = 36 years Age of H = 20 years

    BPRE–776

    Age of M = 23 or 45 years Age of F or G = 23, 31 or 45 years Floor Number 8

    Person

    Age in years

    G

    31

    7

    H

    20

    6

    Q

    17

    5

    R

    42

    4 3

    M L

    23 24

    2

    F

    45

    1

    T

    36

    264. (1) Q is the youngest (17 years). Only R lives between Q and M. 265. (3) There are two persons between G and R, whose age is 42 years. There are two persons between L and Q, whose age is 17 years. There are two persons between F and R, whose age is 42 years. 266. (2) Only G lives above H. 167. (5) Except L, all others live on the even numbered floors. 268. (4) Q is the youngest. Q lives above M. Age of L is 24 years. Age of H is 20 years. Age of G is 31 years. 24 + 20 = 44; 44 –31 = 13 R lives below Q. (269–272) :

    4 3 2 1

    Flat–1 F Flat–1 L Flat–1 C Flat–1 I

    Flat–2 A Flat–2 E Flat–2 K Flat–2 G

    Flat–3 H Flat–3 D Flat–3 J Flat–3 B

    EAST

    Age of Mother

    V is the child of Q. Age of V is a perfect cube, i.e, 8 years. Age of W is half of X’s age. Therefore, age of W is 3 years. Age of J’s child is 3 years. Therefore, W is child of J. S is not an immediate neighbour of L. Only one person sits between J and Q. Only three persons sit between X and W. Age of P is a multiple of 7. Therefore, age of P is 35 years. Now, the age of J is 32 years.



    WEST

    X like Rocky Road and he attends lecture in October (31 days). R likes Strawberry and he attends lecture in September (30 days). 257. (3) Three persons attend lectures between P and the one who likes Kulfi. Three persons attend lectures between W and the one who likes Cookies. Three persons attend lectures between V and the one who likes Strawberry. 258. (5) P–Chocolate combination is true. (259–263) : ● Mothers are : J, K, L, M, N, O, P and Q ● Children are : R, S, T, U, V, W, X and Y ● Ages of mothers : 31, 32, 33, 34, 35, 36, 37 and 38 years ● Ages of children : 1 year, 2, 3, 4, 5, 6, 7 and 8 years

    269. (2) K lives in Flat –2 on floor number 2. 270. (1) I lives to the west of G. 271. (2) D lives immediately above J in flat – 3. 272. (3) F, A and H live on floor number 4. (273–277) : Position Number 1 2 3 4 5 6 7

    Box

    Number

    D E C B F A G

    22 15 19 11 17 14 18

    PROBLEM SOLVING 273. (4) The number of Box C is 19. 274. (5) The number of Box A is 14. Three boxes – C, B and F – are kept between Box E and Box A. 275. (1) Box D is the first from the top. 276. (2) Box F is kept immediately above the Box A. The position number of Box F is 5. (278–282) : Month March June September December

    Date 3rd 8th 3rd 8th 3rd 8th 3rd 8th

    Person H B A E G D C F

    278. (4) H likes green colour. 279. (2) G likes litchi. Two persons – A and E – were born between B and G. 280. (3) Except C, all others were born in months having 30 days. 281. (5) G was born on 3rd September. G likes litchi. 282. (1) C likes yellow colour. (283–287) : Phone C E G L R U Z

    Company Apple Motorola Apple Motorola Motorola Samsung Samsung

    277. (3) Except the combination C–7, in all other combinations the box and its position number are matched correctly. The position number of Box C is 3.

    City Dehradun Alwar Gwalior Jamshedpur Pune Fatehpur Mumbai

    283. (5) Phone Z was bought in Mumbai. 284. (1) Except ‘L-Fatehpur’, in all other pairs the first is the phone of that company which was also bought in the second (City). But, L is a Motorola phone and Samsung phone was bought in Fatehpur. 285. (3) Phone L was bought in Jamshedpur. 286. (2) C is an Apple phone bought in Dehradun. G is another Apple phone bought in Gwalior. 287. (4) R was bought in Pune. L was bought in Jamshedpur. C is an Apple phone. Z was bought in Mumbai. It is a Samsung phone. (288–291) : Car Colour O Orange F Silver E Pink N Blue M White D Red P Black

    Favourite Colour/Fruit Green Red Apple Banana Litchi Mango Yellow Blue

    288. (2) M is a White car. Car N was launched immediately before Car M. 289. (1) Car P is of black colour. 290. (5) Car E is the Pink Car. 291. (3) Orange car was launched first. (292-296) : Day

    Person

    Monday Tuesday Wednesday Thursday Friday N Saturday

    L D F S 58 R

    No. of Books 45 35 40 53 27

    292. (2) F was born on Wednesday. 293. (4) L ⇒ 45 N ⇒ 58 Required sum = 45 + 58 = 103 294. (3) R was born on Saturday. R has 27 books. 295. (1)S has 53 books. Three persons – L, D and F – were born before S. 296. (5) F has 40 books. D was born immediately before F. N has 58 books. S was born immediately before N. S has 53 books. F was born immediately before S. (297–299) : Floor Number 3 2 1

    Flat A

    Flat B

    R S P

    U Q T

    297. (1) U lives on Floor number 3, Flat B. 298. (4) Both (i) and (ii) are true. S lives on Floor number 2. T lives on Floor number 1.

    BPRE–777

    299. (5) T lives in Flat B of Floor number 1. (300-303) : F, C > B > E D>C D is not the heaviest. E is not the lightest. F>D>C>B>E>A

    21 kg. 5 kg. 300. (4) F + E = 29 kg. ∴ F = (29 – 5) kg. = 24 kg. F + B = 34 kg. ∴ B = (34 – 24)kg = 10 kg. Now, B + D = (10 + 21) kg. = 31 kg. 301. (1) C is the third heaviest bag. 302. (3) The weight of bag C is more than 5 kg. but less than 21 kg. (303–307) : Month January February April June August September October December

    Person Q N P K O M J L

    303. (3) J was born in October. 304. (2) Three persons – P, N and Q – were born before K. Three persons – M, J and L – were born after O. 305. (4) K was born in June. 306. (5) Q was born in January. Only statement I is not true. 307. (1) Three persons – M, J and L – were born after O. (308–312) :

    MBA A C M Com

    F B Sc

    B B Tech

    D M Arch. E B Pharma

    308. (4) Except the pair ‘B Tech – M Com’, in all other pairs, the first person is second to the left or right of the second person. B is pursuing B Tech while C is pursuing M Com. B sits third to the left or right of C.

    PROBLEM SOLVING 309. (3) A is pursuing MBA. C sits to the immediate left of A. 310. (5) Two persons sit between E and A. F is pursuing B Sc. F sits third to the left or right of D. B is pursuing B Tech. B sits to the immediate left of E. 311. (2) C is pursuing M Com. C sits to the immediate left of A. 312. (1) E is pursuing B Pharma. B and D are immediate neighbours of E. D is pursuing M Arch. (313–317) : E I F B Rs. 40,000 Rs. 35,000 Rs. 38,000 Rs. 45,000 H A D J Rs. 37,000 Rs. 42,000 Rs. 36,000 Rs. 43,000 C K L G Rs. 50,000 Rs. 48,000 Rs. 47,000 Rs. 46,000

    313. (4) K + B = Rs. (48,000 + 45,000) = Rs. 93,000 314. (3) F lives second to the right of E. F receives Rs. 38,000. 315. (1) B + J + G = Rs. (45,000 + 43,000 + 46,000) = Rs. 1,34,000 316. (2) E + I + F + B = Rs. (40,000 + 35,000 + 38,000 + 45,000) = Rs. 1,58,000 317. (5) C lives third to the left of G. (318–322) : Day Monday Tuesday Wednesday Thursday Friday Saturday Sunday

    Person K T A U P R M

    Marks 15 6 3 12 4 10 2

    318. (3) A + R ⇒ 3 + 10 = 13 319. (4) U scored 12 marks on Tuesday. 320. (5) M scored least marks (2). K scored the highest marks (15). 321. (1) Tuesday ⇒ 6 Friday ⇒ 4 322. (2) T took Test on Tuesday on which 6 marks were scored.

    RBI GRADE–B/ NABARD GRADE–A OFFICER EXAMS 1. 2. 3. 4.

    (2) Statement B (3) Statement E (1) Statement A (5) Statements B and D

    (16–20) :

    (5–10) : Position Person Music Director (DR) P Hip–Hop Executive Director (ED) O Opera Chief Manager (CM) N Pop Senior Manager (SM) Q Rock Manager (MG) R Jazz Assistant Manager (AM) M Electronic Trainee (TE) S Classical 5. (3) There is a gap of two persons between R and the one who likes opera. Similarly, there is a gap of two persons between Q and the one who likes classical. There is a gap of two persons between P and the one who likes Rock. 6. (1) R works as Manager (MG). 7. (5) N and O are more experienced than Q but less experienced than P. 8. (4) S works as Trainee (TE). 9. (2) N works as Chief Manager (CM) and likes Pop. 10. (5) R likes Jazz. (11–15) : Floor 8 7 6 5 4 3 2 1

    Person A E H C G F B D

    Game Roadrash Castlevania Resident Evil Dragon Quest Tekken 3 Contra San Andreas Metal Gear

    11. (4) D plays the game of Metal Gear. 12. (2) G himself plays Tekken 3. 13. (5) G plays Tekken 3. C lives immediately below the one who plays Resident Evil. B lives immediately above D. 14. (3) E and H live between A and C (Dragon Quest). 15. (1) There are four persons between A and F, who likes Contra. There are only two persons B and who likes Dragon Quest. Similarly, there are two persons between C–Roadrash. H is immediately below the person who likes Castlevania. Similarly, F lives immediately below the person who likes Tekken 3.

    BPRE–778

    Day

    Destination Departure Time Monday Madrid 10 PM Tuesday Sydney 5 PM Wednesday Dubai 9 PM Thursday London 6 PM Friday Zurich 8 PM Saturday Rome 4 PM Sunday Paris 7 PM

    16. (3) None as Paris bound flight departs on Sunday. 17. (4) Sydney bound flight departs at 5 PM. 18. (4) Sydney bound flight departs at 5 PM. London bound flight departs at 6 PM. 19. (2) Dubai bound flight departs at 9 PM. The flight would arrive in New Delhi at 11 PM. 20. (5) Madrid bound flight departs at 10 PM. 21. (1)

    7 2 4 3 6 5 1 9 1 3 5 5 7 3 (22-26) : Floor Number 7 B Floor Number 6 C Floor Number 5 D Floor Number 4 A Floor Number 3 E Floor Number 2 G Floor Number 1 F 22. (2) B lives on the topmost floor. 23. (5) C lives immediately above D’ floor. 24. (3) Except G, all other lives on odd numbered floor. 25. (1) E lives on Floor Number 3. 26. (4) G lives exactly between the floors of E and F. (27 – 32) : Floor Number 8 7 6 5 4 3 2 1

    Person K N J Q L O M P

    27. (3) K lives immediately above N. L and O live between M and Q. M lives on floor number 2. 28. (5) N lives on floor number 7.

    PROBLEM SOLVING 29. (3) Except in pair OM, in all other pairs there are two persons between the given pair of persons. 30. (1) O lives on floor number 3. 31. (2) Q lives between the floors on which J and L live. 32. (4) If P and L interchange their places, O will live between P and M. (33–37) : Person P Q R S T U V

    Company ITC Wipro Wipro ITC Samsung Samsung Samsung

    Fruit Guava Banana Orange Kiwi Mango Strawberry Apple

    33. (4) Q and R work in Wipro company. 34. (3) P likes Guava. 35. (1) V works in the Samsung company and likes Apple. 36. (5) T – Mango is correct. 37. (2) R works in Wipro with Q. U works with T who likes Mango. S works in ITC. Q likes Banana. (38–42) : Day Person Monday N Tuesday O Wednesday M Thursday J Friday K Saturday L Sunday P

    Subject Mathematics Psychology Sociology Economics Statistics Zoology English

    38. (5) Except ‘Tuesday–Zoology’, in all others there is a gap of one day between the given entities. 39. (1) Two persons – J and K – have their interviews between L and M. 40. (3) J has his interview immediately before K. M has his interview on Wednesday. L likes Zoology. L has his interview after J. 41. (2) L has an interview immediately after K. L likes Zoology. 42. (4) N likes Mathematics. N has an interview on Monday.

    (43–47) : Department Administration Production Marketing Human Resource (HR) Finance Research & Development (R & D) Client Relations (CR)

    Person L N K O

    Movie Gladiator Dread Signs Wanted

    J P

    Twilight Hero

    M

    Jumanji

    43. (2) Except NK, in all others there is a gap of two persons. N is just more experienced than that of K. 44. (5) O works in Human Resource (HR) department. He likes the movie Wanted. 45. (1) M likes movie Jumanji. 46. (4) The one who likes movie Signs is just less experienced than that person who works in the HR department. The one who likes Hero is just less experienced than that person who works in the Client Relations (CR) department. Similarly, the one who likes movie Gladiator is just less experienced than that person who works in the Production department. 47. (3) M is more experienced than J. N is less experienced than K. (48–50) : L>J>N N>M>O L does not hold the most number of flowers. Therefore, K holds the most number of flowers. K>L>J>N>M>O

    ↓ 16 48. (4) J holds the third highest number of flowers. 49. (5) Only K holds more flowers than L. 50. (2) M holds 16 flowers. Therefore, J holds 16 + 12 = 28 flowers. N holds more than 16 but less than 28 flowers. (51–56) : Box D E C B F A G

    Box Number 12 5 9 2 7 4 8

    BPRE–779

    51. (2) The number of box C is 9. 52. (1) The number of box A is 4. There are three boxes – C, B and F – between E and A. 53. (3) Box D is first from the top. 54. (3) Box number 7 is kept immediately above A. Box F’s number is 7. 55. (5) Except Box A – Box Number 8, in all others there is gap of one box. Box A is kept immediately above the Box Number 8. 56. (4) Box G is numbered 8. (57–61) : Department Admission Pharmacy Emergency Finance Laboratory Physiotherapy Surgery

    Person A F C E D B G

    Pet Horse Mice Rabbit Dog Cat Parrot Fish

    57. (2) B likes Parrot. 58. (4) A is the least experienced while G is the most experienced. In all others, there is a gap of only one person. 59. (3) D, B and G have more experience than E. Only A has less experience than F. 60. (1) G works in the Surgery department and he likes Fish. 61. (5) A works in the Admission department. F likes Mice. F has just more experience than A. E works in the Finance department. D likes Cat. D has just more experience than E. C works in the Emergency department. E has just more experience than C. E likes Dog. (62–67) : Box Box U Box P Box Q Box S Box V Box T Box R

    Number 6 10 15 5 7 2 11

    62. (2) Except P – 5, in all others there is a gap of one box between the first box and the second box and the first box is kept below the second box. In case of P – 5, there is also a gap of one box between box P and box number 5 but box P is kept above the box number 5.

    PROBLEM SOLVING 63. (5) Box T is second from the bottom. 64. (3) Box U is numbered 6. 65. (1) Box R which is numbered 11 is kept immediately below Box T. 66. (5) The number of box V is 7. 67. (4) B ox P is immediately above the box numbered 15. (68–72) : Floor Number 8 7 6 5 4 3 2 1

    Person

    Marathon

    C D H B F E G A

    10,000 metre 7,200 metre 5,000 metre 2,300 metre 9,100 metre 6,400 metre 6,300 metre 3,800 metre

    68. (5) M ran for (2300 + 2000) metre = 4300 metre 69. (4) C lives on the topmost floor. Only E lives between G and F. B ran for 2300 metre. B lives immediately above F. A ran for 3,800 metre. 70. (1) D ran for 7200 metre. C lives immediately above D. C ran for 10,000 metre. 71. (2) The combination Floor Number 3 – E is correct. 72. (3) F + A = (9100 + 3800) metre = 12900 metre (73-78) : Month January

    Date 12th 25th March 12th 25th April 12th 25th May 12th 25th June 12th 25th December 12th 25th

    Person U X P R V O Y Q Z S W T

    Colour Green Maroon Blue Orange Violet Yellow Red Grey White Pink Silver Brown

    73. (3) P and R had lecturres in the month of March. 74. (4) V–Violet combination is correct. 75. (1) Z likes white. There is a gap of only one lecture between Y and Z. V likes violet. There is a gap of only one lecture between P and V. Q likes grey. There is a gap of only one lecture between O and Q.

    76. (2) Four persons – P, R, V and O – have lectures between the lectures by X and Y. 77. (5) Except 12–Pink, all others are correctly matched. 78. (3) Z has the lecture in the same month as that of S. Z likes white. (79–83) : Person Class City A II Bhopal B V Varanasi C V Jaipur D II Ujjain E XI Siliguri F VIII Nashik G XI Patna H VIII Chennai 79. (3) C lives in Jaipur. 80. (5) Except A–Patna, in all other pairs both the persons study in the same class. 81. (2) H studies in Class VIII. 82. (1) A lives in Bhopal. 83. (4) B lives in Varanasi. G and E study in the same class. B and C study in the same class. C lives in Jaipur and he studies in Class V. (84–87) : Day Monday Tuesday Wednesday Thursday Friday Saturday Sunday

    Person B A G D F C E

    Game Volleyball Cricket Squash Kho-Kho Hockey Football Tennis

    84. (4) G was appointed on Wednesday. G plays Squash. 85. (1) G plays Squash. Only D was appointed between G and F. Five persons – F, D, G, A and B – were appointed before C. A plays Cricket. B was appointed on Monday. 86. (3) E plays Tennis. E was appointed on Sunday. 87. (2) D was appointed on Thursday. C plays Football. A plays Cricket. A was appointed on Tuesday. B plays Volleyball. B was appointed on Monday. F plays Hockey. D plays Kho-kho. (1-3) : P>R>U Q>P U>T S>Q>P>R>U>T





    53

    12

    BPRE–780

    88. (3) T borrowed less than 12 books. 89. (1) Q borrowed the second highest number of books. 90. (2) P borrowed 53 books. Therefore, S borrowed 53 + 11 = 64 books Q borrowed more than 53 but less than 64 books. 91. (2) Obviously, option (2) may be a reason for reducing the cut-off from 90 per cent to 81 per cent. (92–96) : Month January February April July September November December

    Movie G D A E F B C

    Country Spain Malaysia Peru India Germany China Nepal

    92. (2) Movie A represented Peru. Movie B was released in November. Movie E represented India. Movie C represented Nepal and it was released in December. Movie G was released in January. 93. (4) Movie E was released immediately before Movie F. Movie D represented Malaysia. Movie F represented Germany. Two Movies – B and C – were released after Movie F. 94. (5) Movie C represented Nepal. 95. (1) Three Movies – F, B and C – were released after Movie E. 96. (3) There is a gap of two Movies between the one which represented Malaysia and Movie F. There is a gap of two Movies between the one which represented India and Movie C. Similarly, there is a gap of two Movies between the one which represented Peru and Movie B. (97–101) : Box R Q S T P

    Colour Red Yellow Pink White Blue

    97. (1) Box S is Pink. Pink box is third from the top. 98. (2) Box Q is yellow while box P is blue. S and T are kept between Q and P. Box T is white. 99. (3) Box T is white. Only box S is kept between Q and T.

    PROBLEM SOLVING 100. (4) T is kept exactly between S and P. T is the white box. 101. (5) Box P is of blue colour. (102–106) : Year

    Person

    1945 1956 1968 1970 1981 1996 2008

    P S V T U Q R

    Favourite Company Infosys Samsung Dell Airtel Google TCS Wipro

    Difference between the consecutive years : 1956 – 1945 = 11 1968 – 1956 = 12 1970 – 1968 = 02 1981 – 1970 = 11 1996 – 1981 = 15 2008 – 1996 = 12 102. (2) P likes Infosys. Two persons – S and V – were born between T and P. 103. (3) U likes Google. 104. (4) R was born in 2008. 105. (5) V likes Dell. R was born in 2008. Q likes TCS. Q was born in 1996. 2008 – 1996 = 12 P is elder than S. 106. (1) Only R who likes Wipro is younger than Q, who likes TCS. (107–111) : Person Number of Projects A 23 B 9 C 56 D 39 E 42 F 16 G 71 H 35 I 60

    Department R&D Marketing Production Production Finance R&D Marketing Finance R&D

    107. (3) C completed 56 projects. 108. (5) A ⇒ 23; D ⇒ 39 F ⇒ 16; A ⇒ 23 H ⇒ 35; F ⇒ 16 E ⇒ 42; B ⇒ 9 F ⇒ 16; I ⇒ 60 109. (1) A, F and I work in the R & D department. Total number of projects completed by A, F and I = 23 + 16 + 60 = 99 110. (2) E works in the Finance department and he has completed 42 projects.

    111. (4) Except E–35, in all other pairs the two persons work in the different departments. H completed 35 projects. E and H work in the Finance department. (112–115) :

    Red

    T

    Silver

    Q

    Golden

    P

    Purple White

    R U S

    Brown

    112. (2) R is the third longest rope. 113. (3) Rope P is of Golden colour. 114. (4) Rope S is of Brown colour. S>U Therefore, Rope S is longer than 296 cm. 115. (5) Golden rope (P) is longer than red rope (T). (116–120) : Floor Number 7 6 5 4 3 2 1

    Person Metal I L K J M H O

    Silver Rhodium Iron Platinum Copper Aluminium Gold

    116. (2) O likes Gold. Only H lives between M and O. 117. (5) All the given statements are true. 118. (4) K lives immediately above J. K likes Iron. 119. (3) L, who likes Rhodium, lives immediately below I. O, who likes Gold, lies immediately below H. K, who likes Iron, lives immediately below L. 120. (1) I lies Silver. (121–125) : Floor Number 8 7 6 5 4 3 2 1

    Person Game U W T V S P Q R

    Rugby Football Snooker Badminton Chess Tennis Archery Cricket

    BPRE–781

    121. (1) U likes Rugby and he lives on floor numbered eight. V lives on floor numbers five. 8 – 5 = 3. S likes Chess and he lives on floor numbered four. R lives on floor numbered one. 4 – 1 = 3. Similarly, T likes Snooker and he lives on floor numbered six. P lives on floor numbered three. 6 – 3 = 3. 122. (3) S lives on floor number four. 123. (2) R likes Cricket. 124. (5) W likes Football. U lives just above W. 125. (4) V likes Badminton. (126–130) : Month January March April June August September October

    Person M O K N P L J

    Car Toyota Suzuki Honda Skoda Volkswagen Mahindra Hyundai

    126. (2) P bought Volkswagen car in August. 127. (4) M bought Toyota car. Four persons – P, N, K and O – bought cars between L and M. 128. (5) N bought a Skoda car. Only K bought a car between O and N. J bought a car after L. O bought a Suzuki car. Five persons bought cars after O. Five persons bought cars before L. 129. (3) Except ‘J–Toyota’, in all other pairs, the given car was bought immediately after the given month or person. 130. (1) K bought a Honda car in April. (131–135) :

    W E S T

    Floor Number

    M

    N

    Four

    E 69 years

    G 75 years

    Third

    A 55 years

    C 62 years

    Second

    F 47 years

    H 53 years

    First

    B 36 years

    D 24 years

    C – A = 7 years ∴ A = (62 – 7) years = 55 years

    E A S T

    PROBLEM SOLVING [Note : A = C + 7 is not possible as each person is of a different age] F + C = 109 years ∴ C = (109 – 47) years = 62 years B = (D + 12) years = 36 years 131. (1) A is 55 years old. 132. (4) D lives on Floor number 1, Flat N. 133. (3) G is 75 years old. 134. (5) E – D = (69 – 24) years = 45 years (A) is true. H + B = (53 + 36) years = 89 years (B) is true. A is 55 years old. A lives on one of the floors above D. Therefore, (C) is not true. G (75 years) is older than H (53 years). Therefore, (D) is not true. 135. (2) E – H = (69 – 53) years = 16 years (136–140) : Person F D A G C E B

    Thing Watch Bag Laptop Radio Calculator Flowers Goggles

    136. (3) Except ‘Laptop–Calculator’, in all other pairs the first person bought a thing or the first thing was bought immediately before the second. 137. (2) B bought goggles. Two persons – C and E – bought a thing between G and B. 138. (1) B bought goggles. 139. (4) D bought a thing on one of the occasions before B. F bought a watch before the one who bought a bag. E bought flowers. 140. (5) D bought a bag. (141–145) : Person L M N O P Q R S T

    Country India India Mexico Canada Canada Mexico Mexico India India

    Colour Pink Green Yellow White Black Silver Purple Red Blue

    141. (1) T likes Blue colour. 142. (5) M likes Green colour. 143. (2) N, Q and R work in Mexico. 144. (3) S likes Red colour. S works with L, M and T in India. P works with O. O likes White colour. R likes Purple colour. Q and R work in Mexico. 145. (4) N, Q and R work in Mexico. Q likes Silver colour. L likes Pink colour. L works in India. N likes Yellow colour. (146–147) : > > >S>T P>R >Q> >S>T P>Q>R>S>T 146. (3) R scored less marks than P and Q. 147. (5) T scored the lowest marks.

    INSURANCE EXAMS 1. (4) S may be the fifth product to be launched. R > T > V > W > S ....... Z > Q 2. (1)

    1 R R R

    2 T W T

    3 V T V

    4 W V Y

    5 Y Y W

    6 S S S

    7 8 Z Q Z Q Z Q

    3. (4)

    1 2 R T Z R

    3 V T

    4 5 6 7 8 Z Q W Y S V Q W Y S

    4. (4) 1 2 3 4 5 6 7 8 R T V W S Y Z Q 5. (1) Earned prize money P ⇒ 36 × 1500 + 14 × 1000 = Rs. 68000 Q ⇒ 34 × 1500 + 21 × 1000 = Rs. 72000 R ⇒ 23 × 1500 + 36 × 1000 = Rs. 70500 S ⇒ 30 × 1500 + 30 × 1000 = Rs. 75000 ∴ Required difference = 75000 – 68000 = Rs. 7000

    BPRE–782

    6. (3) Average prize money P⇒

    68000 = Rs. 1360 50

    Q⇒

    72000 = Rs. 1309 55

    R ⇒

    70500 = Rs. 1195 59

    S⇒

    75000 = Rs. 1250 60

    7. (2) Maximum average prize money per match won = Rs. 1360 (P) 8. (3) From both the statements Value of three quotations = 120 × 3 = 360 Third quotation 360 – (90 + 125) = 360 – 215 = 145 The highest quotation is more than Rs. 139. 9. (2) From statement II Number of people who read both Economic Times and Financial Express = (220 + 200) – (300 – 50) = 420 – 250 = 170 10. (2) From statement I The time taken to cover 40 km is not known. From statement II If PQ = x km then the distance covered by the troubled engine = (x – 40) km Usual speed =

    Usual time = ∴ 2 hours +

    = ⇒

    40 = 20 kmph 2

    FG x IJ hours H 20 K

    x − 40 x − 5 20

    40 2 = 60 3 2 x x − 40 = = 2− 3 20 5

    x − 4 x + 160 4 = 20 3 ⇒ (–3x + 160) 3 = 80 ⇒ 9x = 480 – 80 = 400



    ⇒x=

    1 400 = 44 km 9 9

    PROBLEM SOLVING 11. (4) From both the statements we can get several values of x. 12. (3) If the time is the same then arrival time would have been 12.00 while it is 3AM ∴ Time interval = 3 hours. 13. (3) From both the statements Let the father age be x years.

    x −5 = 25 3 ∴ x = (25 × 3) + 5 = 80 years 14. (1) From statement I Sandhya has two sisters. 15. (3) From both the statements Keshav will select car X. 16. (3) 15th August ⇒ Sunday Next Sundays ⇒ 22nd, 29th August; 5th, 12th, 19th and 26th September 30th September ⇒ Thursday Other Thursdays in September ⇒ 2, 9, 16 and 23 17. (2) Today is Thursday. Thursday + 5 + 1 + 1 = Thursday Day after tomorrow = Thursday + 2 = Saturday Saturday – 6 = Sunday Now, 6 – 3 = 3 18. (4) U A = B >C Z is greater than U. (19–23) : Floor

    Seventh Floor Sixth Floor Fifth Floor Fourth Floor Third Floor Second Floor First Floor Ground Floor

    Floor Number 8 7 6 5 4 3 2 1

    Person

    N O P L K J M Q

    19. (2) L and K live between the floors of P and J. 20. (3) K lives on floor number 4. 21. (4) N lives on the topmost floor. Seven people live below N. N lives on an even numbered floor, i.e. floor number 8. Three people – O, P and L – live between N and K. 22. (4) Q lives on floor number 1. 23. (1) P lives on the floor immediately below O.

    (24–28) : Floor Seventh Floor Sixth Floor Fifth Floor Fourth Floor Third Floor Second Floor First Floor Ground Floor

    Floor Number 8 7 6 5 4 3 2 1

    Person Y Z T S V X W U

    24. (3) V lives on floor number 4. 25. (1) Three persons - S, V and X live between floors of T and W. 26. (4) T lives immediately above S. Five persons - S, V, X, W and U live below T. T lives on floor number 6. Four persons live between T and U. 27. (5) T lives on the floor immediately below Z. 28. (2) S lives on floor number 5. (29-32) : Day Monday Tuesday Wednesday Thursday Friday Saturday Sunday

    Lecture Organisational Behaviour Psychology Statistics Computer Science Research Methods Off day Economics

    29. (5) Economics is the last lecture scheduled. 30. (3) Three lectures were scheduled between Economics and Psychology. 31. (3) Computer Science was scheduled on Thursday. 32. (5) Saturday was off day. Therefore five lectures were scheduled before Saturday and one lecture was scheduled after Saturday. (33–37) : Day

    Person

    Activity

    Monday

    G

    Running

    Tuesday

    B

    Dancing

    Wednesday

    F

    Miming

    Thursday

    A

    Painting

    Friday

    D

    Acting

    Saturday

    C

    Singing

    Sunday

    E

    Baking

    BPRE–783

    33. (2) G participated in Running on Monday. In all others, the person participated one day after the given day. 34. (4) B participated immediately before F and A participated immediately after F. 35. (3) B participated in dancing two days before the one who participated in painting. Similarly, D participated in Acting two days before the one who participated in baking. A participated in painting two days before the one who participated in singing. 36. (1) G participated in running on Monday. 37. (5) D participated in acting on Friday. (38–41) : Floor

    Person

    Alphabetical

    Number 8 7 6 5 4 3 2 1

    Order E H C D G F I B

    I H G F E D C B

    38. (1) G lives immediately above F. In all others, one, two or more persons live between the given pair of persons. 39. (2) Three persons – D, G and F – live between C and I. 40. (4) The positions of two persons H and B would remain unchanged. 41. (5) H lives immediately below E. (42–43) : C > B, E A, E > D > B A does not buy the maximum number of Cookies and E does not buy the third highest number of cookies. C>E>A>D>B

    ↓ 20 42. (2) D buys the second lowest number of cookies. 43. (3) C bought 20 cookies. Therefore, A bought 20 – 8 = 12 cookies E bought more than 12 but less than 20 cookies.

    PROBLEM SOLVING (53-54) :

    (44-48) : Day

    Person

    Colour

    Monday

    G

    Pink

    Tuesday

    B

    Silver

    Wednesday

    E

    Blue

    Thursday

    A

    Yellow

    Friday

    C

    Green

    Saturday

    D

    Orange

    Sunday

    F

    Red

    44. (1) G would attend a wedding on Monday. 45. (4) D likes orange colour. 46. (3) A would attend a wedding on Thursday. The one who likes Pink colour would attend a wedding three days before A, i.e., on Monday. Similar relation exists between all other combinations except in C-Yellow. 47. (5) A would attend a wedding on Thursday. Thursday + 2 = Saturday C would attend a wedding on Friday. Friday + 2 = Sunday B would attend a wedding on Tuesday. Tuesday + 2 = Thursday 48. (2) B would attend a wedding immediately before E while A would attend a wedding immediately after E. (49-52) : Floor Number 8 7 6 5 4 3 2 1

    Person K N Q J L P O M

    Alphabetical Order Q P O N M L K J

    49. (2) There are five persons between O and K. In all other pairs one person lives immediately above or below the second person. 50. (1) Q lives immediately below N. 51. (3) The position of no person will remain unchanged. 52. (4) Two persons - J and L - live between Q and P.

    ,

    >E> , A >B>D C>D A>C>E>B>D

    ↓ 8 53. (4) A won 8 + 16 = 24 competitions. Therefore, E won more than 8 but less than 24 competitions. C won the second highest number of competitions and hence E cannot won 23 competitions. E won possibly 14 competitions. 54. (1) C won the second highest number of competitions. (55–60) : Day

    Person

    Monday

    S

    (5) Corolla was sold on Friday and Friday – 2 = Wednesday. Duster was sold on Monday and Monday + 2 = Wednesday. Polo was sold on Sunday and Sunday – 2 = Friday Kwid was sold on Thursday and Thursday + 2 = Saturday Alto was sold on Tuesday and Tuesday + 5 = Sunday 63. (3) Only one car Kwid was sold between i10 and Corolla. (64–68) : Floor Number

    Person

    Mercedes

    Floor Number 6

    J

    Floor Number 5

    Q

    Floor Number 4

    M

    Q

    BMW

    O

    Fiat Volkswagen

    Friday

    M

    Audi

    Saturday

    P

    Nissan

    Sunday

    N

    Jaguar

    55. (1) R attended screening immediately before M while P attended screening immediately after M. 56. (3) The combination S-Mercedes is correct. 57. (5) N attended screening on Sunday. 58. (2) P likes Nissan car. 59. (4) S attended screening on Monday. Monday + 3 = Thursday R attended screening on Thursday. Thursday + 3 = Sunday Q attended screening on Tuesday. Tuesday + 3 = Friday (60–64) : Day

    62.

    Car

    Wednesday

    R

    61.

    (1) Duster and Alto were sold before i10. (2) Kwid was sold on Thursday.

    Floor Number 8 Floor Number 7

    Tuesday Thursday

    60.

    64.

    65. 66.

    67.

    N P

    Floor Number 3

    K

    Floor Number 2

    O

    Floor Number 1

    L

    (2) J and Q live between P and M. L lives on floor numbered one. Three persons live between J and O. K lives immediately above O. (3) K lives on the floor immediately below M. (4) Three persons – J, Q and M – live between the floors on which P and K live. (5) Q lives on floor numbered 5.

    68. (1) Q lives between J and M. (69–73) : Day

    Person

    Game

    Monday

    B

    Tuesday

    A

    Volleyball Cricket

    Wednesday

    G

    Football

    Thursday

    D

    Kho-Kho

    Friday

    F

    Hockey

    Car

    Saturday

    C

    Squash

    Sunday

    E

    Tennis

    Monday

    Duster

    Tuesday

    Alto

    Wednesday

    i10

    Thursday

    Kwid

    Friday

    Corolla

    Saturday

    XUV

    Sunday

    Polo

    BPRE–784

    69. 70.

    (4) G was appointed on Wednesday. He plays Football. (5) F was appointed immediately before C, who plays Squash. Five persons were appointed before C. A plays Cricket. B was appointed on Monday.

    PROBLEM SOLVING 71.

    (3) E plays Tennis. He was appointed on Sunday. 72. (2) G was appointed on Wednesday. 73. (1) Two persons – A, B – were appointed before G. (74–78) : Exe.

    Department

    81. (3) Pavan lives on the topmost floor. 82. (5) Amit is living on the floor numbered six. 83. (4) The position of only one person — Ashok — will remain unchanged. (84–88) :

    City

    B

    Marketing

    Nagpur

    G

    Engineeering

    Chennai

    H

    Marketing

    Ranchi

    K

    Engineering

    Kolkata

    Day

    Person

    Monday

    T

    Tuesday

    R

    Wednesday

    U W P

    D

    Systems

    Hyderabad

    Thursday Friday

    F

    Systems

    Kozikode

    Saturday

    S

    T

    Marketing

    Patna

    Sunday

    Q

    V

    Systems Exe : Executive

    Bhopal

    74. 75. 76.

    (2) T is posted in Patna. (1) K is posted in Kolkata (3) Engineering department has only two Executives – G and K. 77. (4) B, H and T works in Marketing department. 78. (5) None of the combinations is correct. (79–83) : ● Arvind lives on an even numbered floor. Even numbered floors are second, fourth and sixth. Pavan lives three floors above Rajeev’s floor. It means from Pavan to Rajeev there would be four floors and Rajeev lives above Arvind’s floor. Also, only one person is living between Rajeev and Arvind. Therefore, Arvind lives on floor numbered two and Rajeev on floor numbered four. Now, Maya will occupy floor numbered one.

    Floor Number 7 6

    Person Pavan

    Alphabetical Order Amit

    Amit

    Arvind

    5

    Ashok

    Ashok

    4

    Rajeev

    Maya

    3

    Raghu

    Pavan

    2

    Arvind

    Raghu

    1

    Maya

    Rajeev

    79. (2) Raghu lives on floor numbered three. 80. (5) Rajeev is living immediately above Raghu’s floor.

    84. (2) Three students — P, S and Q — are taking class after W. 85. (3) Two students — R and U — are taking class between T and W. 86. (4) S was taking class immediately after Friday. 87. (5) U was taking class on Wednesday. 88. (1)

    U W P S Q

    P was taking class exactly between U and Q. (89–90) : A>P>Q M>A P>J>Q M>A>P>J>Q





    86% 70% 80. (1) M scored more marks than A. A scored 86% marks. 90. (2) M and A scored more marks than P. (91–95) : Month

    Person

    January February April July September October November December

    E G F B C D A H

    BPRE–785

    Favourite Colour White Yellow Orange Blue Red Silver Green Pink

    91. (5) H was bron is December. 92. (3) F likes Orange. Only G was born between E and F. 93. (4) G likes Yellow. G was born immediately before F. D likes Silver. D was born immediately before A. B likes Blue. B was born immediately before C. C was born in September. 94. (2) The combination G – Yellow is definitely true. 95. (1) Six persons – F, B, C, D, A and H – were born after G. C likes Red colour. Three persons – C, D and A – were born between H and B. (96–97) : Y > T > U, V W>S>Y W > S > Y > T > U, V





    42 m 22 m 96. (2) V = 21 metre (Given) W>S>Y>T>V>U S – V = (42 – 21) metre = 21 metre V is longer than U. 97. (2) W = (S + 14) metre = (42 + 14) metre = 56 metre (98–102) : Floor Number 8 7 6 5 4 3 2 1

    Person

    City

    N T Q M P R S O

    Jhansi Jaipur Agra Patna Kolkata Mathura Delhi Surat

    98. (5) N lives on the topmost floor. 99. (2) O visits Surat. 100. (1) P lives in floor number 4 and 4–2=2 M lives on floor number 5 and 5 +3=8 S lives on floor number 2 and 2 + 3=5 Q lives on floor number 6 and 6 – 3=3 T lives on floor number 7 and 7 – 3=4 101. (4) All the statements are true.

    PROBLEM SOLVING 102. (3) Q visits Agra. Q lives on floor number 6 and P lives on Floor number 4; 6 – 4 = 2. P visits Kolkata. P lives on floor number 4 and S lives on floor number 2; 4 – 2 = 2. N visits Jhansi. N lives on floor number 8 and Q lives on floor number 6; 8 – 6 = 2. (103–107) :

    R

    Q

    S

    R

    Q or

    Q

    R

    S

    S

    S

    R

    W

    or

    Q

    R

    Q T V

    R

    P

    Q

    T

    V

    108. (3) Two persons–S and W–buy items before R. 109. (2) Q buys items just before T. 110. (1) Only S buys items before W. Only V buys items after T. 111. (3) S buys items before W. R and P buy items between W and Q. R buys items just after W. 112. (4) Except Q–T, in all other pairs, only one person buys items between the two given persons. Q buys items immediately before T. (113 –117) : Suppose all the persons are facing North : 1 2 3 4 5

    — — — — —

    8 Tickets 20 Tickets

    Month March April May June July August September October

    117. (2) C + P = 15 + 12 = 27 Tickets (118–122) : Person City L Pune M Mumbai N Mumbai O Delhi P Mumbai Q Pune R Pune S Delhi T Pune

    Date 11th 8th 21st 18th 14th 7th 5th 9th

    Person L M S V A F J N

    123. (5) F was born on 7th August. 124. (4) V was born in June. 125. (2) Three friends– M, S and V were born between L and A. 126. (1) F was born on 7th August. N was born on 9th October. Number of days from 7th August to 9th October : = 24 + 30 + 9 = 63 days 127. (3) L was born in March.

    (128–132) :

    L E F T

    1

    2

    3

    4

    5

    6

    7

    8

    9

    M

    R

    L

    U

    K

    B

    D

    S

    T White

    103. (2) U attends the seminar on 25th April. 104. (1) R attends seminar immediately after T. 105. (4) Three teachers – U, T and R– attend seminars betwen S and P. 106. (3) Q attends seminar on 25th January. 107. (5) Q and S attend seminars in January. (108 –112):

    Brown

    P

    113. (3) M is at eighth position from the beginning of the queue. 114. (1) V buys 9 or 15 Tickets. Therefore, M = 9 + 8 = 17 or 15 + 8 = 23 115. (5) Eighteen persons are standing in the queue. 116. (4) 20 Tickets ⇒ 4th position 12 Tickets ⇒ 12th position Seven persons are standing between these two.

    Green

    28

    Physics Biology History

    Blue

    Economics

    T R

    Silver

    U

    119. (1) L, Q, R and T work in Pune. 120. (3) M and N work with P in Mumbai. 121. (2) L works in Pune. R works with T, Q and L. 122. (5) After interchange, O ⇒ Pune; R ⇒ Delhi P ⇒ Delhi; S ⇒ Mumbai Now, O works with L, Q and T. (123–127) :

    Pink

    July

    S

    25 28 25

    12 Tickets 9 or 15 Tickets

    Red

    April

    28

    Subject Chemistry Geography

    118. (4) Except S–M, in all other pairs both the persons work in the same city. Q and L work in Pune. M and N work in Mumbai. N and P work in Mumbai. R and T work in Pune. But, S works in Delhi while M works in Mumbai.

    15 Tickets

    Golden

    January

    Date Teacher Q 25

    C — M — — K P V — G B — ✕

    Yellow

    Month

    6 7 8 9 10 11 12 13 14 15 16 17 18

    R I G H T

    128. (3) U likes Pink colour. D sits third to the right of U. 129. (2) R likes Golden colour and B likes Blue colour. 130. (5) K sits fifth from the right end. K likes Silver colour. 131. (3) R likes Golden colour while D likes Green colour. Four persons– L, U, K and B - sit between R and D. 132. (1) U likes Pink colour while S likes Brown colour. If U likes Brown colour then S would like Pink colour. ❑❑❑

    BPRE–786

    PROBLEM SOLVING

    MODEL EXERCISES Directions (1-7) : A management institute organises six once a month lecture series for young entrepreneur as per the following schedule, with no dates conflicting Marketing management August through January Economics April through October Business law January through September Financial management March through June Accounting October through April Personnel management October through December

    1. During which month are the fewest lectures given? (1) January (2) February (3) June (4) September (5) None of these 2. Which is the largest number of lectures that can be attended in a single month? (1) 7 (2) 6 (3) 5 (4) 4 (5) None of these 3. Which two series taken together fill the year without a overleap? (1) Marketing and Financial management (2) Personnel management and Economics (3) Business law and Personnel management (4) Business law and Accounting (5) None of these 4. During how many months of the year must a student attend to hear all the lectures on Marketing, Economics and Financial management? (1) 11 (2) 10 (3) 9 (4) 8 (5) None of these 5. How many lecture series last more than 6 months? (1) 1 (2) 2 (3) 3 (4) 4 (5) None of these 6. How many different series can be attended in September, October and November? (1) 2 (2) 3 (3) 4 (4) 5 (5) None of these 7. How many different lectures can be attended in January, February and March?

    (1) 12 (2) 10 (3) 8 (4) 6 (5) None of these 8. If 21st July, 1999 is a Wednesday, what would have been the day of the week on 21st July, 1947? (1) Monday (2) Sunday (3) Thursday (4) Saturday (5) None of these 9. At an enquiry office at a railway station, a passenger was told A train for Delhi has left 15 min ago, but after every 45 min a train leaves for Delhi. The next train will leave at 8.30 pm. At what time was this information given to the passenger? (1) 7.45 pm (2) 8.00 pm (3) 8.15 pm (4) 8.05 pm (5) None of these 10. Four bells begin to toll together and toll respectively at intervals of 6, 5, 7, 10 and 12. How many times they will toll together in one hour excluding the one at the start ? (1) 7 times (2) 8 times (3) 9 times (4) 11 times (5) None of these Directions (11-14) : These questions are based on the following information. A set of eight candidates A, B, C, D, E, F, G and H are being interviewed by two panels of interviewers : Panel I and Panel II—from 9.30 to 10.50 on a particular day. Each panel will spend about 10 minutes per candidate and at no time during the interview process will a panel be without any candidate. The original schedules of interviews for the eight candidates are shown in the following table Candidate Panel-I Time A 9.30 to 9.40 B 9.40 to 9.50 C 9.50 to 10.00 D 10.00 to 10.10 E 10.10 to 10.20 F 10.20 to 10.30 G 10.30 to 10.40 H 10.40 to 10.50

    Panel-II Time 10.20 to 10.20 10.20 to 10.30 10.30 to 10.40 10.40 to 10.50 9.30 to 9.40 9.40 to 9.50 9.50 to10.00 10.00 to 10.10

    Due to requests from the candidates, the interview schedule was altered for several candidates. The alterations were made in such a way that whenever a change was made, the time schedule for both the panels of a par-

    BPRE–787

    ticular candidate was exchanged in entirety with the time schedule of another candidate. The following alterations were made I. A’s place was taken by G. II. A in turn was accommodated in C’s place III. C in turn was accommodated in E’s place IV. E took H’s place V. H took G’s place 11. Which of the following candidates finished the interviews along with E ? (1) A (2) C (3) D (4) F (5) None of these 12. Which of the following candidates finished the interviews before C ? (1) B (2) D (3) Both of these (4) None of these (5) All of these 13. If G and A had to leave together, then how much time did any of them has to wait? (1) A has to wait 10 min (2) G has to wait 20 min (3) None of them has to wait (4) G has to wait 30 min (5) None of these 14. Which one of the following statements is true? (1) G could leave even before E’s first interview was over (2) F and H left together (3) D was the only candidate who could give company to E (4) All of the above (5) None of these Directions (15-19) : Study the following information carefully to answer these questions. I. Kamal is available at home from 12 noon to 4 pm on Tuesday, Thursday and Sunday. II. His younger brother Navin is available at home on Monday, Thursday, Friday and Sunday between 10 am and 2 pm III. The eldest brother Rajiv is available between 9 am and 12 noon on Monday, Wednesday and Thursday; and 2 pm to 4 pm on Friday, Saturday and Sunday.

    PROBLEM SOLVING 15. At a time; on which day of the week all the three brothers are available at home? (1) None (2) Sunday (3) Thursday (4) Cannot be determined (5) None of these 16. For how many days, only one brother is available at a particular time in a week? (1) One (2) Two (3) Three (4) Four (5) None of these 17. On which day(s) of the week the youngest and the eldest brothers are available at home at the same time? (1) Only Monday (2) Only Thursday (3) Only Friday (4) Both Monday and Thursday (5) None of these 18. A girl earns twice as much in December as in each of the other months. What part of her entire year’s earning does she earn in December? (1) 2/11 (2) 2/13 (3) 3/14 (4) 1/6 (5) None of these 19. A watch is a minute slow at 1 pm on Tuesday and 2 min fast at 1 pm on Thursday. When did it show the correct time? (1) 1.00 am on Wednesday (2) 5.00 am on Wednesday (3) 1.00 pm on Wednesday (4) 5.00 pm on Wednesday (5) None of these 20. A clock gaining 2 min every hour was synchronised at midnight with a clock losing 1 min every hour. How many minutes behind will its minute hand be at eleven the following morning? (1) 23 (2) 27 (3) 22 (4) None of these (5) All of these 21. Suppose it is 3 o’ clock past 20 min the angle between the smaller and bigger hands will be (1) 20° (2) 30° (3) 110° (4) 120° (5) None of these 22. In a month of 31 days, the third Wednesday falls on the 15th.

    What will be the last day of that month? (1) Fifth Thursday (2) Fifth Wednesday (3) Fourth Sunday (4) Fifth Friday (5) None of these 23. When Ranjeev was born, his father was 32 yr older than his brother and his mother was 25 yr older than his sister. If Ranjeev’s brother is 6 yr older than Ranjeev and his mother is 3 yr younger than his father, how old was Ranjeev’s sister when he was born? (1) 15 yr (2) 14 yr (3) 7 yr (4) 10 yr (5) None of these 1 h, the hour hand of a clock 2 rotates through an angle of (1) 90° (2) 140° (3) 120° (4) 75° (5) None of these 25. In 27 March, 1995 was a Monday, then what days of the week was 1 November, 1995? (1) Monday (2) Sunday (3) Tuesday (4) Wednesday (5) None of these 26. 16 January 1997 was a Thursday. What day of the week was 4 January 2000? (1) Tuesday (2) Wednesday (3) Thursday (4) Friday (5) None of these 27. In a year 28th February in Tuesday; if the leap year is excluded, then 28th March will be a (1) Sunday (2) Tuesday (3) Monday (4) Saturday (5) None of these 28. In a class of 60 students, the number of boys and girls participating in the annual sports is in the ratio 3 : 2 respectively. The number of girls not participating in the sports is 5 more than the number of boys not participating in the sports. If the number of boys participating in the sports is 15, then how many girls are there in the class? (1) 20 (2) 30 (3) 25 (4) Data inadequate (5) None of these

    24. In 2

    BPRE–788

    29. Find the time between 3 and 4 o’ clock when the angle between the hahas of a watch is two-third of a right angle. (1) 10

    9 min past 3 11

    (2) 10

    10 min past 3 11

    (3) 11

    9 min past 3. 11

    10 min past 3. 11 (5) None of these 30. A clock is set right at 5 am. If it loses 16 min in 24 h, what will be the true time when the clock indicates 10 am on the 4th day? (1) 11.00 am (2) 8.00 am (3) 10.00 am (4) 9.00 am (5) None of these 31. Ram’s age was square of a number last year and it will be cube of a number next year. How long must he wait before his age is again the cube of a number? (1) 39 yr (3) 10 yr (2) 38 yr (4) 64 yr (5) None of these 32. At a party, there are 43 persons in all. The number of women is two more than men but the number of children is 4 less than men. How many women are there in the party? (1) 17 (2) 13 (3) 11 (4) 25 (5) None of these 33. Six roads lead to a country. They may be indicated by letters X, Y, Z and digits 1, 2, 3. When there is a storm, Y is blocked. When there are floods, X, 1 and 2 will be affected. When road 1 is blocked, Z is also blocked. At a time, when there are floods and a storm, which road can be used? (1) 3 (2) Z (3) Y (4) 2 (5) None of these 34. If the seventh day of a month is three days earlier than Friday, what day will it be on the nineteenth day of the month? (1) Sunday (2) Monday (3) Wednesday (4) Friday (5) None of these (4) 11

    PROBLEM SOLVING 35. If the third day of a month is Monday, which of the following will be the fifth day, from 21st of that month? (1) Wednesday (2) Monday (3) Tuesday (4) Thursday (5) None of these 36. My uncle shall visit me after 64 days of my father’s birthday. If my father’s birthday falls on Tuesday, what shall be the day on my uncle’s visit? (1) Tuesday (2) Sunday (3) Wednesday (4) Monday (5) None of these 37. A worker may claim 15 for each km which he travels by taxi and 5 for each km which he drives his own car. If in one week, he 500 for travelling 80 claimed km, how many kilometers did he travel by taxi? (1) 10 (2) 20 (3) 30 (4) 80 (5) None of these 38. The number of boys in a class is three times the number of girls. Which one of the following numbers cannot represent the total number of children in the class? (1) 48 (2) 44 (3) 42 (4) 40 (5) None of these 3 more than Ramu, 39. Ravi has but then Ramu wins on the horses and trebles his money, so that he now has 2 more than the original amount of money that the two boys had between them. How much money did Ravi and Ramu have between them before Ramu’s win? (1) 9 (2) 11 (4) 15 (3) 13 (5) None of these 40. Simplify and find the value of A’ in the equation given below 2 + 7 × l6 – (8 – 7)–(9 ÷ 3) = A (1) 116 (2) 150 (3) 110 (4) 220 (5) None of these Directions (41–45) : Study the following information carefully and answer the questions given below : Six plays P, Q, R, S, T and U were organised on six different days from Wednesday to Tuesday with one rest day, but not necessarily in the same order. Rest day was not on Friday. Play Q was organised on the first day and then three more plays

    were organised before the rest day. Rest day was between Plays Q and T. Play R was organised immediately after the Play P. S was organised on Thursday. 41. Which of the following Plays was organised immediately after the Play S ? (1) Q (2) U (3) T (4) P (5) R 42. On which of the following days was the rest day ? (1) Sunday

    (2) Tuesday

    (3) Monday

    (4) Thursday

    (5) Saturday 43. Which of the following Plays was organised in the last ? (1) P (3) U (5) T

    (2) R (4) S

    44. Which of the following combinations of the Day-Play is correct ? (1) Tuesday – T (2) Saturday – R (3) Friday – S (4) Monday – U (5) Sunday – P 45. If S is related to Friday in a certain way and T is related to Tuesday in the same way, then P is related to (1) Sunday

    (2) Thursday

    (3) Monday

    (4) Wednesday

    (5) Saturday Directions (46–50) : Study the following information carefully and answer the questions given below : Eight delegates, viz., P, Q, R, S, T, V, W and X, are from eight different cities viz, Hyderabad, Amritsar, Chandigarh, Kolkata, Chennai, Delhi, Patna and Bangalore but not necessarily in the same order. All the eight delegates are sitting around a circular table facing towards the centre but not necessarily in the same order. P is sitting fourth to the left of S. P is from Delhi. X is not an immediate neighbour of S. Delegate W is from Chennai. S does not belong to Amritsar. Delegate from Bangalore is sitting third to the right of the delegate

    BPRE–789

    from Hyderabad. R does not belong to Hyderabad or Bangalore. Delegate V is from Patna and he is sitting second to the right of X. V is an immediate neighbour of P. W is sitting to the immediate left of S. Only one person is sitting between S and the person who belongs to Kolkata. The delegate from Kolkata is not an immediate neighbour of the delegate from Patna. Only two persons are sitting between Q and the delegate from Bangalore. 46. What is the position of R with respect to W ? (1) Second to the right (2) Third to the right (3) Second to the left (4) Third to the left (5) Immediate left 47. To which of the following cities does S belong ? (1) Kolkata (2) Bangalore (3) Hyderabad (4) Chandigarh (5) Other than those given as options 48. Who among the following is sitting exactly between the delegate from Hyderabad and S ? (1) Delegate from Chennai (2) V (3) T (4) Delegate from Amritsar (5) Delegate from Kolkata 49. Which of the following statements is true regarding R ? (1) R belongs to Chandigarh (2) R is sitting second to right of P (3) R is sitting exactly between the delegate from Amritsar and T (4) R is sitting just opposite the delegate from Patna (5) None of the given statements is true 50. Who among the following is sitting third to the right of X ? (1) S (2) Delegate from Chennai (3) Delegate from Bangalore (4) V (5) Delegate from Hyderabad Directions (51–55) : Study the following information carefully and answer the questions given below : Eight persons — P, Q, R, S, T, U, V and W — were born in three different months (of the same year) but not necessarily in the same order, namely

    PROBLEM SOLVING March, June and December such that not less than two persons and not more than three persons were born in a month. Each of them also likes a different fruit namely Guava, Peach, Banana, Cherry, Mango, Orange, Kiwi and Apple but not necessarily in the same order. Only Q and W were born in March. R likes Apple and was born in the same month as T. R was not born in December. The one who likes Mango was born in June. T does not like Mango. U was not born in the same month as T. S likes Cherry and born in the same month as U. V does not like Mango. The one who likes Kiwi and the one who likes Banana were born in the same month. The one who likes Kiwi was not born in the same month as W. U does not like Kiwi. The one who likes Guava was born in the same month as P. Q does not like Peach. 51. As per the given arrangement which of the following combinations represents only the persons who were born in December? (1) T, V (2) U, P, T (3) V, U, S (4) P, T (5) P, V, S 52. As per the given arrangement, which of the following persons represents the one who was born in the same month as the one who likes Orange? (1) W (2) P (3) R (4) T (5) U 53. Which of the following fruits does T like, as per the given arrangement? (1) Orange (2) Guava (3) Peach (4) Banana (5) Other than those given as options 54. Which of the following combinations is correct, as per the given arrangement? (1) December — Peach (2) June — Orange (3) June — Banana (4) March — Guava (5) December — Banana 55. Who amongst the following likes Peach, as per the given arrangement? (1) P (2) V (3) U (4) W (5) T Directions (56–60) : Study the following information carefully and answer the questions given below :

    Seven persons — A, B, C, D, E, F and G — were appointed to a company on seven different days of the same week starting from Monday to Sunday (but not necessarily in the same order). Each person also plays a different game namely- Cricket, Hockey, Football, Squash, Volleyball, Tennis and Kho-Kho, but not necessarily in the same order. Only two persons were appointed after the one who plays Hockey. E was appointed on one of the days after the one who plays Hockey. Only three persons were appointed between E and G. Only one person was appointed between G and the one who plays Volleyball. A was appointed immediately after the one who plays Volleyball. Only three persons were appointed after the one who plays Kho-Kho. F was appointed immediately after C, but not on Friday. Only two persons were appointed between F and the one who plays Cricket. B was appointed immediately after the one who plays cricket. More than two persons were appointed between D and the one who plays Tennis. F does not play Football. 56. Who amongst the following was appointed on Wednesday? (1) The one who plays Kho-Kho (2) The one who plays Cricket (3) F (4) A (5) B 57. Which of the following statements is true as per the given arrangement? (1) Only one person was appointed between F and the one who plays Squash. (2) Only three persons were appointed before C. (3) A plays Tennis. (4) B was appointed on Saturday. (5) None of the given options is true 58. The person who plays Tennis was appointed on which of the following days? (1) Thursday (2) Tuesday (3) Sunday (4) Monday (5) Saturday 59. Which of the following combinations will be definitely true as per the given arrangement? (1) C — Squash (2) Thursday — D (3) Saturday — Cricket (4) Monday — Volleyball (5) F — Kho-Kho

    BPRE–790

    60. How many persons were appointed before G? (1) Two (2) One (3) Three (4) Four (5) None, as G was appointed on Monday Directions (61–66) : Study the following information carefully and answer the questions given below : Seven friends namely A, B, C, D, E, F and G learn seven different languages namely Japanese, Hindi, French, Chinese, Spanish, English and German, not necessarily in the same order, starting from Monday to Sunday (of the same week). F learns a language on Wednesday. Only one person learns between F and the one who learns Spanish. Only three persons learn between the one who learns Spanish and B. The one who learns French learns immediately after B. Only two persons learn between the one who learns French and G. A learns immediately before D. Only three persons learn between the one who learns Japanese and the one who learns English. F does not learn Japanese. E learns before the one who learns English. Only one person learns between E and the one who learns Chinese. A does not learn Hindi. 61. As per the given arrangement, B is related to the one who learns French in a certain way and F is related to the one who learns Chinese in the same way. To which of the following is A related to in the same way? (1) the one who learns German (2) the one who learns Hindi (3) the one who learns Japanese (4) the one who learns English (5) the one who learns Spanish 62. Which of the following statements is/are true about C? (1) C learns Hindi (2) C learns a language on Saturday (3) C learns a language immediately after F (4) Only two persons learn between C and G (5) All the given statements are true

    PROBLEM SOLVING 63. Four of the following five are alike in a certain way and thus form a group as per the given arrangement. Which of the following does not belong to that group? (1) D – Thursday (2) F – Sunday (3) B – Friday (4) A – Tuesday (5) G – Monday 64. Which of the following languages does D learn? (1) Spanish

    (2) Hindi

    (3) French

    (4) Japanese

    (5) English 65. Who amongst the following learns German? (1) E (2) A (3) G

    (4) D

    (5) C 66. On which of the following days does E learn a language? (1) Thursday

    (2) Friday

    (3) Monday

    (4) Sunday

    (5) Tuesday 67. The positions of how many digits will remain the same if the digits in the number 69852731 are rearranged in the ascending order within the number from left to right? (1) One

    (2) More than three

    (3) Three

    (4) Two

    (4) None Directions (68–70) : Study the following information carefully and answer the questions given below : There are five jars – A, B, C, D and E – each having different numbers of cookies. Jar A has more cookies than jar E but less than jar B. Jar D has more cookies than jar B. Jar D does not have the most number of cookies. The jar having the third least number of cookies has 11 cookies. The jar having the most number of cookies has 24 cookies. 68. Which of the following jars probably has 17 cookies? (1) B (2) E (3) Either ‘A’ or ‘E’ (4) D (5) Either ‘C’ or ‘B’

    69. Which of the following jars has the second lowest number of cookies? (1) D (2) A (3) E (4) C (5) B 70. How many cookies does jar E possibly have? (1) 22 (2) 11 (3) 5 (4) 19 (5) 12 Directions (71 – 75): Study the following information carefully and answer the questions given below : There are eight sports academies ranked from 1 to 8. There are eight different players A, B, C, D, E, F, G and H who play different sports viz., Hockey, Football, Rugby, Table Tennis, Cricket, Chess, Basketball and Volleyball but not necessary in the same order. ● The one who plays from the academy having rank 1 plays Hockey and the one who plays from academy having rank 6 plays Football ● B plays from the academy having rank 4 and plays Chess ● Basketball is not played by the player of the academy having rank 2 ● The one who plays from academy having rank 5 plays Cricket and F does not play Cricket ● C’s academy is one spot ahead of D but one spot lower than that of G. ● H plays from an academy whose rank is higher than that of B and plays Basketball ● G does not play Cricket and C plays Volleyball ● A plays Table Tennis and plays from academy having rank 2 71. Who among the following players play volleyball and Football sports? (1) B and C (2) F and A (3) C and G (4) E and H (5) D and F 72. Which of the following sports are played by the academies having ranks 3 and 5 repectively? (1) Rugby and Table tennis (2) Chess and Football (3) Hockey and Basketball (4) Cricket and Rugby

    BPRE–791

    (5) Basketball and Cricket 73. What are the ranks secured by the academies that conduct Chess and Hockey respectively? (1) 2 and 8 (2) 4 and 5 (3) 1 and 3 (4) 4 and 1 (5) 7 and 3 74. Which of the following sport is conducted by the academy having rank exactly between the ranks of academies conducting Football and Rugby respectively? (1) Table tennis (2) Volleyball (3) Basketball (4) Cricket (5) Hockey 75. What are the Ranks of academy of the player D and the academy that conducts Cricket? (1) 8 and 5 (2) 3 and 2 (3) 6 and 3 (4) 1 and 4 (5) 7 and 2 Directions (76–80) : Study the following information carefully and answer the question given below : Twelve Professors — O, P, Q, R, S, T, U, V, W, X, Y and Z — had lectures in different months of the same year viz., January, March, April, May, June and December but not necessarily in the same order. All the lectures are either on 12th or 25th of these months. No two professors had lectures on the same day. Each professor also likes a different colour namely, Red, Blue, Green, Yellow, Orange, White, Pink, Silver, Maroon, Violet, Brown and Grey but not necessarily in the same order. (NOTE: No lecture was conducted in any other month of the same year). V had a lecture on 12th April. Only two people had lectures between V and Q. The one who likes red had a lecture in the same month as Q. Q does not like red. The one who likes White had a lecture on an even numbered date of the month which had exactly 30 days. V does not like white. Only three people had a lecture between the one who likes White and the one who likes Violet. As many people had lectures between the one who likes red and O as between Q and the one who likes White. Only two people had lectures between O and the one who likes Blue. U had a lecture on 12th of a

    PROBLEM SOLVING month before the one who likes Violet. U likes neither red nor blue. Only two people had lectures between U and the one who likes Orange. Only two people had lectures between the-one who likes Orange and the one who likes Green. X had a lecture in the same month as the one who likes Green. As many people had a lecture after W as before X. Only one person had a lecture between W and Z. Z had a lecture before W. The one who likes Pink had a lecture on an odd numbered day in the same month as Z. P had a lecture on 12th of the same month as R. Only two people had lectures between P and the one who likes Yellow. The one who likes Maroon had a lecture before P. T likes Brown. S does not like Red. W does not like Grey. 76. Which of the following combinations indicates those who had a lecture in March? (1) The one who likes Maroon and R (2) U and X (3) P and R (4) The one who likes Green and the one who likes Pink (5) R and V 77. Which of the following combinations is correct? (1) W-White (2) X- Pink (3) U-Grey (4) V-Violet (5) O-Red 78. If Y is related to White and P is related to Violet based on the given arrangement, then which of the following is related to O following the same pattern? (1) Grey (2) Pink (3) Brown (4) Maroon (5) Silver 79. HOW many people have lectures between the lectures by X and Y? (1) Two (2) Four (3) None (4) One (5) Three 80. Four of the following five are alike in a certain way based on the given arrangement and hence they form a group. Which one of the following does not belong to that group? (1) 12–Y (2) 25–O (3) 12–Green (4) 25–T (5) 12–Pink

    Directions (81–85) : Study the following information carefully and answer the questions given below: Eight friends P, Q, R, S, T, U, V and W were born in three different months of the same year, but not necessarily in the same order – March, June and December. Each of them likes a different fruit namely Guava, Peach, Banana, Cherry, Mango, Orange, Kiwi and Apple but not necessarily in the same order. ● Not less than two people and not more than three people were born in a month. ● Only Q and W were born in March. ● R likes Apple and was born in the same month as T. R was not born in December. ● The one who likes Mango was born in the month which has 30 days only. ● U was not born in the same month as T and V does not like Mango. ● S likes Cherry and was born in the same month as U. ● The one who likes Kiwi and the one who likes Banana were born in the same month. ● The one who likes Kiwi was not born in the same month as W. ● U does not like Kiwi and T does not like Mango. ● The one who likes Guava was born in the same month as P. Q does not like Peach. 81. Who among the following was born in December and likes Cherry fruit? (1) P (2) S (3) U (4) W (5) V 82. Who among the following is elder to T? (1) Q (2) R (3) W (4) Q and R (5) Q and W 83. Which one of the followings is CORRECT as per the given arrangement? (1) P – March – Orange (2) R – June – Apple (3) Q – December – Banana (4) U – December – Kiwi (5) T – December –Peach 84. The Kiwi fruit is liked by the person born in which of the following month?

    BPRE–792

    (1) March (2) June (3) December (4) 1 or 2 (5) 2 or 3 85. Suppose, S was born in a month in which R was born, then which fruit does S like? (1) Cheerry (2) Guava (3) Apple (4) Mango (5) Orange Directions (86–90) : Study the following information carefully and answer the questions given below : Eight persons M, N, O, V, W, X, Y and Z attend seminars on different months of the year viz. March, June, October and November, such that not more than two persons attend their seminars in each of the months. Seminars can be held on either 10th or 27th day of the month. No two seminars can be held on the same day. W and N attend the seminars on the same month. There are three seminars between the seminars of X and O. W does not attend their seminar in November. Z attends his seminar immediately after N. V attends his seminar in the month of November. The number of persons who attend their seminars between the seminars of Y and Z respectively is the same as the number of persons who attend their seminars between the seminars of N and V respectively. X does not attend the seminar on October. W attends his seminar before N. 86. M attends his seminar on which of the following dates? (1) 10th October (2) 27th November (3) 10th November (4) 10th March (5) None of these 87. Which of the following persons attends his seminar on 27th March? (1) W (2) X (3) M (4) N (5) None of these 88. How many persons attend the seminar after W? (1) 5 (2) 4 (3) 6 (4) 3 (5) None of these 89. Who among the following persons attends the seminar on 10th October?

    PROBLEM SOLVING (1) W (2) M (3) Z (4) V (5) None of these 90. How many persons attended seminar after V? (1) 5 (2) 4 (3) 7 (4) No one (5) None of these Directions (91 – 95): Study the following information carefully and answer the questions given below: Seven persons namely P, Q, R, S, T, U and V like seven different fruits namely, Apple, Mango, Banana, Orange, Kiwi, Guava and Strawberry. Each of them works in either of the three companies viz. Samsung, Wipro and ITC with at least two of them in a company. (Note : None of the information given is necessarily in the same order.) R works in Wipro with the one who likes Banana. U does not work with R. P works in ITC with only the one who likes Kiwi. U does not like Apple. T works in Samsung with the one who likes Apple. T does not like Strawberry. The one who likes Strawberry works with the one who likes Mango but not with R. S works with the one who likes Guava. S does not like Banana. Q does not like Apple. 91. Which of the following combinations represents the persons working in Wipro ? (1) S, R (2) Q, V, R (3) U, T, R (4) R, Q (5) T, R, P 92. Which of the following fruits does P like ? (1) Banana (2) Strawberry (3) Guava (4) Mango (5) Orange 93. Which combination represents the company in which V works and the fuits he likes ? (1) Samsung – Apple (2) Samsung – Banana (3) Wipro – Guava (4) ITC – Mango (5) ITC – Orange 94. Which of the following combinations is correct ? (1) S – Strawberry (2) Q – Orange (3) R – Apple (4) U – Kiwi (5) T – Mango

    95. Which of the following statements is true ? (1) R works in the company in which P works. (2) U works with the one who likes Mango. (3) None of the given statements is true (4) S works in Wipro. (5) Q likes Kiwi. Directions (96–100): Study the following information carefully and answer the questions given below: Seven students B, C, D, E, F, G and H have to attend workshops conducted by seven different banks CANARA, SBI, ICICI, HDFC, AXIS, PNB and YES but not necessarily in the same order. ● The seven banks have allotted the workshop in the months January, March, April, July, August, September and December but not necessarily in the same order. ● The one who likes CANARA will attend a workshop immediately before B ● The one who likes AXIS will attend a workshop in the month which has less than 31 days ● Only three students will attend a workshop between H and G ● The one who likes PNB will attend a workshop immediately before G ● Only one student will attend a workshop between the one who likes AXIS and B ● The one who likes ICICI will attend a Workshop immediately before the one who likes YES bank ● Only three students will attend a workshop between one who likes AXIS and the one who likes SBI ● C will attend a workshop in a month which has only 30 days. F does not like CANARA ● E will attend a Workshop immediately after the one who likes YES bank. H will attend a workshop immediately after B 96. Which of the following student who does not like Canara bank, attends a workshop after March but before August?

    BPRE–793

    (1) D (2) C (3) E (4) F (5) B 97. Which of the following Bank is liked by student H ? (1) ICICI (2) PNB (3) YES (4) CANARA (5) SBI 98. SBI and PNB conduct their workshop in which of the following months? (1) August and September (2) March and July (3) April and December (4) July and August (5) January and December 99. Who among the following attends his workshop immediately after HDFC’s workshop? (1) C (2) D (3) B (4) E (5) G 100. In which month workshops was conducted by Canara Bank ? (1) April (2) August (3) December (4) July (5) September Directions (101–105) : Study the following information carefully and answer the questions given below : Twelve professors – D, E, F, G, H, I, J, K, L, M, N and O – are to deliver lectures on different dates, but not necessarily in the same order. All of them are sitting around a square table facing the centre as shown below : Thursday 17 20 16

    Tuesday 22

    Sunday 13

    24

    10

    27

    7

    ●J

    5 1 Saturday

    30 Wednesday

    is sitting second to the right of a person who is to the immediate right of D ● I is giving lecture on an odd number day that is seven days later the lecture of O’s lecture ● E gives lecture on a Thursday but after 20th of the month ● F is sitting third to the right of a person who is giving lecture after 16th but before 22nd

    PROBLEM SOLVING ●D

    is to the immediate left of a person who is second to the right of a professor who gives lecture on 10th of the month ● K is fourth to the right of a professor who gives the first lecture of the month ● F gives lecture on a prime number date ● G gives lecture on an even number date and M gives lecture on a Tuesday but not after 10th of the month ● L is an immediate right neighbour of a professor gives lecture on Thursday ● H gives his lecture 12 days before I’s lecture ● I is not a neighbour of E and J ● G is sitting second to the right of a professor who is sitting to the immediate right of N 101. How many days are there between the lectures of Professors “O” and “G”? (1) 2 days (2) 10 days (3) 13 days (4) 15 days (5) 17 days 102. On which day, the professor who is third to the right of professor E is going to give his lecture? (1) Tuesday (2) Wednesday (3) Friday (4) Thursday (5) Saturday 103. Who among the following are giving their lectures on a Thursday? (1) M – F – E (2) D – G – N (3) O – I – E (4) F – K – J (5) D – E – H 104. Who among the following is fourth to the right of a professor gives lecture on 22nd of the month? (1) Professor M (2) Professor F (3) Professor D (4) Professor G (5) Professor E 105. Who among the following is giving his lecture on a Monday? (1) Professor M (2) Professor D (3) Professor G (4) Professor E (5) Professor N

    Directions (106–110) : Study the following information carefully and answer the questions given below : L, M, N, O, P, Q and R are seven employees who are working in the same company. They attend meeting in different department viz; Administrative, Security, Finance and HR department on different days from Monday to Sunday but not necessarily in the same order. One employee attends only one meeting and only one meeting is held on each day. There are two employees who attend meeting in Administrative, Security, HR department and only one employee attends meeting in Finance department. L attends meeting on Thursday. There are two persons who attend meeting between L and the person who attends meeting in HR department. Only three persons attend meeting between the persons who attend meeting in Administrative department and the one who attends meeting in Finance department. The one who attends meeting in administrative department attends before the one who attends in Finance department. The one who attends meeting in Finance department does not attend on Saturday. The number of persons who attend meeting between L and the one who attend meeting in finance department is same as the number of persons who attend meeting between O and the one who attends meeting in Security department. The one who attends meeting in Security department attend before O. O does not attend meeting in HR department. Q attends meeting on the day immediately before the day on which L attends meeting. O does not attend meeting on the day just after the day on which L attends meeting. The number of persons who attend meeting between L and P is same as the number of persons who attend meeting between L and R. P attends meeting in one of the day before the day on which R attends meeting. N attends meeting in Administrative department. R does not attend meeting in Security department. 106. Who among the following persons did attend meeting on Friday ? (1) O (2) M (3) P (4) N (5) R

    BPRE–794

    107. Which of the following combinations of “Person – Day” is true with respect to the given arrangement? (1) R — Friday (2) M — Saturday (3) Q — Thursday (4) P — Friday (5) P — Tuesday 108. L attends meeting in which of the following department? (1) Security (2) HR (3) Administrative (4) Finance (5) Either (1) or (2) 109. In this arrangement, Q is related to Tuesday then N is related to : (1) Thursday (2) Wednesday (3) HR (4) None of those given as options (5) Sunday 110. How many persons do attend meeting between P and O ? (1) Three (2) None (3) More than three (4) Two (5) One Directions (111–115) : Study the following information carefully and answer the questions given below : Eight employees – P, Q, T, B, R, J, N and A – work in three different departments, viz., Operation, Personnel and Accounts of an organisation in such a way that at least two of them but not more than three work in the same department. Each of them likes different cities viz., Bengaluru, Gwalior, Yelahanka, Raipur, Ooty, Warangal, Jodhpur and Palampur, but not necessarily in the same order. Q likes Bengaluru and he works in the Personnel department only with J who likes Palampur. P works in the Operation department and he does not like Gwalior or Ooty. T likes Jodhpur and he works with N who likes Raipur. The ones who work in the Operation department do not like Warangal. A does not work in the Operation department and B does not work in the Accounts department. The ones who like Warangal and Raipur respectively work in the same department. R does not work in the Accounts department and he likes Gwalior.

    PROBLEM SOLVING 111. Which of the following combinations of employees does work in the Operation department? (1) Q, the one who likes Ooty and R (2) The one who likes Yelahanka, Q and the one who likes Gwalior (3) P, the ones who like Jodhpur and Ooty respectively (4) The ones who like Yelahanka, Ooty and Gwalior respectively (5) B, the ones who like Ooty and Warangal respectively 112. Which of the following combinations of favourite city and the department is correct? (1) Jodhpur – Accounts (2) Palampur – Personnel (3) Warangal – Accounts (4) Bengaluru – Operation (5) Yelahanka – Accounts 113. As per the given arrangement Q is related to personnel in a certain way and N is related to Accounts in the same way. Who among the following is related to Operation in the same way? (1) A (2) T (3) R (4) J (5) Other than those given as options 114.Who among the following does like Yelahanka? (1) N (2) J (3) T (4) B (5) P 115. Which of the following statements is true as per the given arrangement? (1) P works in the Operation department and he likes Jodhpur. (2) J likes Palampur and works in the Personnel department. (3) A works in the Personnel department and he likes Warangal. (4) B works in the Account department and he likes Ooty. (5) N works in the Personnel department and he likes Yelahanka.

    Directions (116–120) : Study the following information carefully and answer the questions that follow : There are 12 individuals G, H, I, J, K, L, M, N, O, P, Q and R born in different months (Not in the same order) starting from January to December of a particular year. Each person is given different personality traits according to month wise astrology. All the 12 individuals are sitting around a square table and all are facing the centre. ar nu Ja

    May

    y













    September



    February



    December

    June ● ●

    October

    August

    April

    November

    July

    January Friendly

    February Realistic

    July Tactful

    August Brave

    P is sitting third to the left of G. H is sitting third to the left of I. R, born after October, is sitting opposite to G. M was born after January but before April and is sitting to the immediate left of K. The person born in March is second right of a person who is to the immediate left of P. H is three months elder than I who is four months elder to R. The person born in October is third to the left of K. N was born after October but before December. The person to the immediate right of L is second to the left of O. J is elder to Q but not elder to M. Q was born in either October or March. Refer the following table for the personality traits of each person :

    March

    March Honest

    April Dynamic

    May Traditional

    June Humorous

    September October Calm Assertive

    November Intuitive

    December Interactive

    116. Who among the following is sitting second to the right of a “Dynamic Personality”? (1) P (2) R (3) G (4) H (5) N 117. “H” is sitting exactly between which of the following personalities? (1) Brave and Tactful (2) Assertive and Intuitive (3) Friendly and Calm (4) Realistic and Humorous (5) Honest and Dynamic 118. Who among the following was born before October but after July and also is not a Brave person?

    (1) I (2) K (3) N (4) R (5) G 119. The person who is elder to H but younger to L and is neither Honest nor Dynamic, is (1) M (2) O (3) J (4) K (5) Q 120. Who among the following is 4th to the right of an “Interactive Person”? (1) Tactful person (2) Realistic person (3) Honest person (4) Assertive person (5) Traditional person

    Directions (121-125) : Study the following information carefully and answer the questions given below :

    (1)

    A 1990

    1991

    (2) Q (3)

    X 2000

    1992

    1993

    36 metre

    2001

    (4) D

    BPRE–795

    2002

    1994

    B 1995

    P 2003

    50 metre

    2004

    Y 2005 J

    PROBLEM SOLVING Six people Ravi, Ganesh, Sunil, Gaurav, Mahesh and Keshav are standing in the line 1-AB. All of them are of different ages and are facing north. Ravi is younger to Gaurav. Sunil is standing second to the left of Ganesh. Mahesh is second to the right of Ravi. Keshav is elder to Mahesh. Ganesh is two years elder to a person who is to the immediate left of Mahesh. Similarly, six people Sudha, Radha, Kamala, Malti, Saraswathi and Lakshmi are standing in line4 XY. All of them are of different ages and facing north. Radha is immediate left neighbour of a person who is second to the right of Lakshmi. Malti is immediate neighbour of Sudha. Malti is younger to Saraswathi. Kamala’s immediate neighbor is two years elder to Saraswathi. Sudha’s one immediate neighbour does not have youngsters. 121. In case, both line AB and XY are merged with each other into ABXY, then who is standing 6th to the right of Ganesh? (1) Malti (2) Kamala (3) Radha (4) Saraswathi (5) Cannot be determined 122. If the persons standing on the lines AB and XY are selected as their birth years are ODD numbered and placed on the line QP with 6 metre gap between each of the odd numbered birth year persons, starting from the left end point Q, then who is standing 4th to the right of Gaurav on line QP? (1) Keshav (2) Kamala (3) Lakshmi (4) Mahesh (5) Saraswathi 123. If the persons standing on the lines AB and XY are selected as their birth years are EVEN numbered and placed on the line DJ with 10 metre gap between each of the even numbered birth year persons, starting from the left end point D, then who is standing 3rd to the right of Keshav on line XY? (1) Sudha (2) Gaurav (3) Lakshmi (4) Sunil (5) Radha

    124. If the persons who are standing second to the right of Sunil, Ganesh, Ravi, Lakshmi and Saraswathi are placed on the line QP starting from the right end point P at a gap of 6 meter, then who is standing second to the left of Keshav on the line QP? (1) Malti (2) Ganesh (3) Mahesh (4) Saraswathi (5) Gaurav 125. If people born in 1991, 1994, 2000, 2003 and 2005 are placed on the line DJ with a gap of 10 metre each between them starting from the left point D, then who is standing middle among all on the line DJ? (1) Ravi (2) Kamala (3) Keshav (4) Saraswathi (5) Sudha Directions (126–128) : Study the following information carefully and answer the questions given below : Nine persons – D, E, F, G, H, I, J, K and L – live on different floors of a nine-storeyed building. The ground floor is numbered one, the floor above it is numbered two and so on till the topmost floor is numbered nine. J lives above the floor of K but below the floor of G but not immediately above or below. D lives on an even numbered floor. Only one person lives between floors of D and F. D lives above F. Only three persons live between the floors of E and F. E lives above F. I lives exactly between the floors of K and L. G lives on one of the floors below the floor of H. Only one person lives above the floor of E. F does not live on the floor immediately below the floor of D nor immediately above the floor of L. 126. Who among the following are living on the floors 6-3-1 in the same order ? (1) G – J – K (2) D – K – L (3) D – G – K (4) J – I – K (5) G – E – F 127. Who among the following lives exactly between the floors of G and J ? (1) I (2) K (3) D (4) F (5) H

    BPRE–796

    128. Who among the following live between the floors of D and I ? (1) G, F, K (2) J, F (3) E, J (4) J, F, K (5) F, H Directions (129–133) : Study the following information carefully and answer the questions given below : Seven people – J, K, L, M, N, O and P – have an interview on seven different days of the same week, starting from Monday and ending on Sunday, but not necessarily in the same order. Each one of them also likes different subjects namely-Statistics, Zoology, Sociology, English, Mathematics, Psychology and Economics, but not necessarily in the same order. Only four people have their interview between N and the one who likes Zoology. Neither N nor the one who likes Zoology has an interview on Sunday. P has an interview immediately after the one who likes Zoology. Only two people have their interviews between P and J. The one who likes Psychology has an interview on one of the days before J but not on Wednesday. Neither N nor P likes Psychology. Only two people have their interviews between the one who likes Psychology and the one who likes Statistics. The one who likes Economics has an interview immediately before the one who likes Statistics. The number of people having interview between P and the one who likes Economics is same as that of the number of people between J and the one who likes English. N does not like English. Only one person has an interview between the one who likes English and K. The one who likes Sociology has an interview immediately after O. L has an interview on one of the days after M. 129. Four of the following five are alike in a certain way based on the given arrangement and hence form a group. Which of the following does not belong to the group? (1) K–English (2) Wednesday-K (3) Mathematics-Wednesday (4) Sociology-Statistics (5) Tuesday–Zoology 130. How many people have their interviews between L and M? (1) Two (2) Three (3) None (4) One (5) More than three

    PROBLEM SOLVING 131. Which of the following statements is TRUE as per the given ar rangement? (1) None of the given statements is true. (2) Only one person has an interview between K and J. (3) O likes Psychology. (4) M has an interview on Friday. (5) The one who likes Zoology has an interview on one of the days before J. 132. Who has an interview immediately after K? (1) The one who likes Economics (2) The one who likes Zoology. (3) J (4) P (5) M 133. How many people have their interview before the one who likes Mathematics? (1) One (2) More than three (3) Two (4) None (5) Three Directions (134–138) : Study the following information carefully and answer the question given below : Twelve Professors — O, P, Q, R, S, T, U, V, W, X, Y and Z — had lectures in different months of the same year viz., January, March, April, May, June and December but not necessarily in the same order. All the lectures are either on 12th or 25th of these months. No two professors had lectures on the same day. Each professor also likes a different colour namely, Red, Blue, Green, Yellow, Orange, White, Pink, Silver, Maroon, Violet, Brown and Grey but not necessarily in the same order. (NOTE : No lecture was conducted in any other month of the same year). V had a lecture on 12th April. Only two people had lectures between V and Q. The one who likes red had a lecture in the same month as Q. Q does not like red. The one who likes White had a lecture on an even numbered date of the month which had exactly 30 days. V does not like white. Only three people had a lecture between the

    one who likes White and the one who likes Violet. As many people had lectures between the one who likes red and O as between Q and the one who likes White. Only two people had lectures between O and the one who likes Blue. U had a lecture on 12th of a month before the one who likes Violet. U likes neither red nor blue. Only two people had lectures between U and the one who likes Orange. Only two people had lectures between the-one who likes Orange and the one who likes Green. X had a lecture in the same month as the one who likes Green. As many people had a lecture after W as before X. Only one person had a lecture between W and Z. Z had a lecture before W. The one who likes Pink had a lecture on an odd numbered day in the same month as Z. P had a lecture on 12th of the same month as R. Only two people had lectures between P and the one who likes Yellow. The one who likes Maroon had a lecture before P. T likes Brown. S does not like Red. W does not like Grey. 134. Which of the following combinations indicates those who had a lecture in March? (1) The one who likes Maroon and R (2) U and X (3) P and R (4) The one who likes Green and the one who likes Pink (5) R and V 135. Which of the following combinations is correct? (1) W-White (2) X- Pink (3) U-Grey (4) V-Violet (5) O-Red 136. If Y is related to White and P is related to Violet based on the given arrangement, then which of the following is related to O following the same pattern? (1) Grey (2) Pink (3) Brown (4) Maroon (5) Silver 137. HOW many people have lectures between the lectures by X and Y? (1) Two (2) Four (3) None (4) One (5) Three 138. Four of the following five are alike in a certain way based on the given arrangement and hence they form a group. Which one of the following does not belong to that group?

    BPRE–797

    (1) 12–Y (2) 25–O (3) 12–Green (4) 25–T (5) 12–Pink Directions (139–143) : Study the following information carefully and answer the questions given below : There are four rows in a cupboard and each row has two racks. There are eight different varieties of fruits viz., Apple, Banana, Guava, Orange, Cherry, Plums, Gooseberry and Mango kept in each rack. Each fruit is kept in certain numbers. ● The total number of Cherry and Plum fruits is 24. ● The rack having 10 fruits is immediate left above diagonally of a rack that is immediate right above diagonally of a rack having 12 fruits. ● The rack having 5 fruits is immediate right above diagonally of a rack that is to the left of a rack having 4 fruits. ● The rack having 25 fruits is to the right of a rack that is immediate below a rack having Guava. ● The rack having Cherry is one rack above the fruit that is immediate right below diagonally of a rack having Apples. ● Gooseberry is placed three racks above a rack having Banana diagonally. There are either 15 or 25 Gooseberries. ● Mango is placed to the immediate left of a rack which is two rows below the rack having Apples. ● The rack having 4 fruits is two racks above the rack having 25 fruits and to the immediate right of a rack having 10 fruits. ● The rack having plums fruit is placed to the immediate right above diagonally of a rack having 12 fruits. ● The total number of Guava and Mango fruits is 20. 139. What is the total number of fruits in the top row of the cupboard? (1) 25 (2) 37 (3) 20 (4) 18 (5) 28 140. Which fruit are 20 in number? (1) Gooseberry (2) Plum (3) Mango (4) Banana (5) Apples 141. How many Apples are there in a rack? (1) 5 (2) 25 (3) 8 (4) 12 (5) 10

    PROBLEM SOLVING 142. What fruits are kept in the bottom row of the cupboard? (1) Guava and Plum (2) Mango and Orange (3) Banana and Mango (4) Gooseberry and Cherry (5) Orange and Guava 143. Which fruit is kept exactly between Cherry and Banana? (1) Guava (2) Plum (3) Orange (4) Mango (5) Apple Directions (144–148) : Study the following information carefully and answer the questions given below : Seven professors A, B, C, D, E, F and G from different cities Bangalore, Mumbai, Patna, Pune, Kolkata, Chennai and Jaipur attend seminars on different days starting from Monday and ending on Sunday of the same week. (Note : None of the given information is necessarily in the same order) ● G attends seminar after E but not immediately. ● The person from Patna attends seminar on Wednesday. ● B attends seminar immediately after a person who attends three days before A. ● C attends seminar after Thursday but before Sunday. ● The persons who are from Pune, Bangalore, Patna and Mumbai attend the seminar in the same order. ● E attends seminar on Wednesday and A attends seminar before Friday. ● G attends seminar on Friday. D has seminar before Thursday. ● The persons from Jaipur, Kolkata and Chennai attend the seminars one by one in the same but reverse order. 144. Who among the following attends seminar on Friday? (1) E (2) B (3) G (4) F (5) C 145. The person from Pune attends seminar on which day? (1) Tuesday (2) Monday (3) Saturday (4) Friday (5) Sunday

    146. The person who attends seminar on Wednesday belongs to which city? (1) Patna (2) Kolkata (3) Jaipur (4) Bangalore (5) Mumbai 147. How many persons attend seminars between the person from Pune and from Kolkata?

    (1) One (2) Two (3) Three (4) Four (5) Five 148. The person who attends seminar on Thursday belongs to which city? (1) Patna (2) Bangalore (3) Kolkata (4) Jaipur (5) Mumbai

    Directions (149–153) : Study the following information carefully and answer the questions given below : Eight persons Nitin, Mithun, Piyush, Om, Dinesh, Ravi, Tanu and Shiva are sitting in a row facing the north as follows :

    Om Tanu Piyush Mithun Ravi Dinesh Nitin Shiva All these persons are to be arranged around a circular table according to the ascending order of their ages in clockwise direction. The difference between the ages of adjacent persons is two years. Nitin sits second to the left of Shiva, whereas Mithun sits third to the left of Piyush. Piyush faces the centre. Only two persons are facing opposite the centre. The one who is fourth to the left of Shiva in the row sits second to the right of the one around circular table who is second to the right of Om in the row. The one who is third to the left of Ravi in the row sits to the immediate right of both Mithun and Dinesh. The one who is second to the right of Piyush in the row sits to the immediate right of both Nitin and Om. The persons sitting at the extreme ends of the row are to sit opposite to each other. Om sits to the immediate left of both Dinesh and Ravi. Tanu is 14-year old and is sitting to the immediate right of an eldest person around the circular table. 149. What is the age of the person who sits exactly between Piyush and Ravi? (1) 18 years (2) 26 years (3) 30 years (4) 22 years (5) 16 years 150. What is the age of the person who sits second to the left of Om ? (1) 18 years (2) 28 years (3) 32 years (4) 22 years (5) 14 years 151. What is the age of the person who sits third to right of Nitin? (1) 16 years (2) 18 years (3) 28 years (4) 26 years (5) 30 years 152. What is the age of the person who sits opposite to Mithun? (1) 24 years (2) 18 years (3) 34 years (4) 20 years (5) 16 years 153. What is the sum of the ages of immediate neighbours of Shiva ? (1) 40 years (2) 36 years (3) 44 years (4) 42 years (5) 30 years

    BPRE–798

    Directions (154–158) : Study the following information carefully and answer the questions given below : An IT company functions from a building having four floors. There are three sections on each floor as given in the figure. The bottom floor is numbered as Row–1 and so on as the top floor is numbered as Row–4. The three sections are numbered from left to right as Column–1, Column–2 and Column– 3. Finance Gym Production Security

    HR

    Media

    Canteen Marketing Accounts Technical Parking

    Training

    There are 12 different heads A, B, C, P, Q, R, S, T, U, V, W and X working in each department respectively. ● P’s office is immediately left above diagonally of a department that is second below the A’s office. ● R’s office is immediate left above diagonally of a department that is immediate left below diagonally of C’s office.

    PROBLEM SOLVING B is immediate above department which is immediate left below diagonally of Q’s office. ● T is immediate right below diagonally of a department that is immediate right below diagonally of Gym. ● X’s office is second below the department that is second to the right of Finance department. ● Q’s office is in an even numbered row and S is an immediate neighbour of X. ● C’s office is immediate right above diagonally of an office that is immediately above the W’s office. ● U is immediate left below diagonally of a department that is immediate left below diagonally of marketing department. ● A’s office is immediate right above diagonally of an office that is second department below B’s office. ● V’s office is immediate right above diagonally of a department that is immediate left above diagonally of X’s office. 154. Q belongs to which department? (1) Media (2) Canteen (3) Technical (4) HR (5) Training 155. Production department is headed by : (1) P (2) V (3) B (4) S (5) T 156. Who occupies the office second to the left of Marketing department? (1) A (2) W (3) T (4) C (5) R 157. Who occupies the office that is immediately above R’s department? (1) Q (2) P (3) B (4) X (5) C 158. Which department is located immediate left below diagonally of Media department? (1) HR (2) Canteen (3) Technical (4) Production (5) Training ●

    Directions (159 – 163) : Study the following information carefully and answer the questions given below: There are 10 members A, B, C, D, E, F, G, H, I and J (not necessarily in the same order) sitting in a row from left to right facing the north direction. The seating arrangement has been made according to their weights in descending order from left to right as given in the following arrangement. >

    =

    >

    =

    >

    =

    >

    =

    >

    A sits exactly between a person to the immediate right of C and a person to the immediate left of H. Any immediate right neighbour is either 2 kg less or having equal weight as that of immediate left neighbour. J sits fifth to the right of a person who sits third to the right of a person sitting to the immediate right of I. E sits exactly between the immediate neighbours of F and G. D sits exactly between a person to the immediate right of G and a person fourth to the right of H. C sits fourth to the left of a person who sits third to the right of F. H sits second to the right of a person who sits fourth to the left of B. 159. If H weighs 50 kg, then what is the sum of weights of A and B? (1) 100 kg (2) 84 kg (3) 90 kg (4) 70 kg (5) 92 kg 160. If C weighs 60 kg, then what is the sum of weights of E and H? (1) 106 kg (2) 120 kg (3) 110 kg (4) 114 kg (5) 126 kg 161. If J weighs 45 kg, then what is the sum of weights of F and A? (1) 108 kg (2) 104 kg (3) 100 kg (4) 112 kg (5) 109 kg 162. If I weighs 65 kg, then what is the sum of weights of H and B? (1) 118 kg (2) 114 kg (3) 116 kg (4) 112 kg (5) 111 kg 163. Who sits exactly between immediate neighbours B and A? (1) C (2) H (3) G (4) D (5) E Directions (164–168) : Study the following information carefully and answer the questions given below :

    BPRE–799

    There is a commercial complex having business premises with four floors as given in the following figure. Each floor has three departments. The lowermost floor is numbered as Row 1 and so as to the top floor is numbered as Row 4. The floors located at the left end of the building are numbered as Column 1 and so as to the floors located at the right end of the building are numbered as Column 3. Bank Hospital Parlor Hotel

    IT company Mobile shop Institute Book shop Yoga class Toy shop Gym Car parking

    There are 12 different owners A, B, C, D, E, F, G, H, I, J, K and L occupying different flats. ● H owns a flat immediate left above diagonally of a flat that is immediate right above diagonally of F’s flat. ● I owns a flat immediate right above diagonally of a flat that is immediate below flat of D’s. ● E owns a flat second to the right of a flat that is immediate below flat of B’s and J is at the bottom floor. ● A owns a flat immediate right above diagonally of a flat that is two flats below I’s. ● B owns a flat immediate left below diagonally of a flat that is immediate left below diagonally of C. ● D owns a flat to the immediate left of a flat that is immediate right above diagonally of H’s. ● K owns a flat immediate above a flat that is immediate left above diagonally of E’s. ● G owns a flat immediate right above diagonally of a flat that is immediate right above diagonally of F’s. 164. Which business is owned by L ? (1) Bank (2) Toy shop (3) Parlor (4) Yoga class (5) IT company 165. Who among the following persons own Book shop and Hospital respectively? (1) C and J (2) I and E (3) G and H (4) F and K (5) I and D

    PROBLEM SOLVING 166. Who among the following own the flats in the Row-2 of the Complex ? (1) B, A and L (2) C, B and K (3) D, K and E (4) H, G and J (5) A, G and C 167. Which of the following shops are owned by D and J respectively? (1) Institute and Mobile shop (2) Gym and Hotel (3) IT company and Toy shop (4) Car parking and Toy shop (5) Bank and Gym 168. Who among the following own Car parking and Parlor respectively? (1) C and A (2) E and B (3) L and A (4) G and F (5) D and J Directions (169 – 173) : Study the following information carefully and answer the questions given below: 12 Government officers A, B, C, D, E, F, G, H, I, J, K and L belong to 12 different departments. All of them are sitting around a rectangular table facing the centre as per the seats marked in the following figure. f In

    m or

    at

    io n

    PWD

    Police

    Ho

    us

    in

    g

    Medical

    Energy

    Finance

    Irrigation

    In

    du s

    try

    Welfare

    ● Officer

    Law

    l tu Cu

    re

    F is sitting fourth to the right of an officer who is sitting fourth to the right of officer who belongs to Medical department ● Officer K is sitting fifth to the left of an officer who is sitting fifth to the left of an officer who belongs to Finance department ● Officer I is sitting second to the right of an officer who belongs to Welfare department ● Officer G is sitting second to the right of an officer who is sitting third to the left of officer J ● Officer D is sitting to the immediate left of an officer who is third to the right of officer A

    ● Officer

    L is sitting third to the right of an officer who is sitting second to the right of officer C ● Officer K is sitting second to the right of an officer who is sitting to the immediate right of officer B ● Officer H is sitting second to the left of an officer who is sitting third to the right of officer D ● Officer E is sitting second to the right of an officer who belongs to Information department. ● Officer D is sitting second to the left of an officer who belongs to Housing department 169. Which officer belongs to Finance department? (1) E (2) D (3) C (4) L (5) G 170. Officer C belongs to which department ? (1) Energy (2) Welfare (3) PWD (4) Culture (5) Police 171. The officer belonging to which department sits exactly between K and I? (1) Information (2) Finance (3) Law (4) Irrigation (5) Housing 172. Who sits fourth to the left of an officer belonging to Energy department? (1) J (2) G (3) I (4) K (5) L 173. How many officers sit exactly between B and officer from Housing department counting from the left of B ? (1) Two (2) Three (3) Four (4) Five (5) One Directions (174–178) : Study the following information carefully and answer the questions given below : Eight people — A, B, C, D, E, F, G and H — live in eight different cities — Varanasi, Nashik, Chennai, Siliguri, Ujjain, Bhopal, Jaipur and Patna, but not necessarily in the same order. Each one of them also studies in one of the four classes — II, V, VIII and XI. Two people study in each of the given classes.

    BPRE–800

    ●A

    studies in an even numbered class. The one who lives in Ujjain studies with A. ● The one who lives in Varanasi and Jaipur study in the same class, but in a class lower than VIII. D studies in a class lower than that of the one who lives in Varanasi. ● The one who lives in Nashik studies with H in the same class, but not in Class XI. D does not live in Nashik. ● B and G study in different classes. B lives neither in Jaipur nor in Nashik. G is senior to both B and H. Neither B nor the one who lives in Siliguri study in the same class as D. E lives in Siliguri. ● C is junior to F. The one who lives in Patna studies in an odd numbered class. ● Neither H nor D lives in Bhopal. The one who lives in Chennai is senior to the one who lives in Bhopal. 174. In which of the following cities does C live? (1) Bhopal (2) Chennai (3) Jaipur (4) Nashik (5) Varanasi 175. Four of the following five are alike in a certain way based on the given arrangement and hence they form a group. Which one of the following does not belong to the group? (1) G–Siliguri (2) D–Bhopal (3) C–Varanasi (4) F– Chennai (5) A–Patna 176. In which class does H study? (1) The one in which A studies (2) VIII (3) The one in which E studies (4) V (5) II 177. Who amongst the following lives in Bhopal? (1) A (2) G (3) C (4) H (5) Other than those given as options 178. Which one of the following statements is TRUE with respect to the given information? (1) B lives in Nashik (2) G is senior to E. (3) B and F study in the same class. (4) The one who lives in Jaipur studies in Class V. (5) None of the given statements is true.

    PROBLEM SOLVING

    SHORT ANSWERS

    EXPLANATIONS

    1. (2)

    2. (4)

    3. (2)

    4. (1)

    5. (3)

    6. (4)

    7. (3)

    8. (4)

    9. (2)

    10. (2)

    11. (3)

    12. (4)

    13. (2)

    14. (3)

    15. (3)

    16. (4)

    17. (4)

    18. (2)

    19. (2)

    20. (4)

    21. (1)

    22. (4)

    23. (4)

    24. (4)

    25. (4)

    26. (1)

    27. (2)

    28. (4)

    29. (2)

    30. (1)

    31. (3)

    32. (1)

    33. (1)

    34. (1)

    35. (1)

    36. (3)

    37. (1)

    38. (3)

    39. (3)

    40. (3)

    41. (4)

    42. (1)

    43. (3)

    44. (2)

    45. (5)

    46. (2)

    47. (4)

    49. (3)

    50. (5)

    51. (3)

    52. (1)

    53. (2)

    54. (5)

    55. (4)

    56. (2)

    57. (5)

    58. (3)

    59. (4)

    60. (1)

    61. (3)

    62. (3)

    63. (1)

    64. (4)

    65. (2)

    66. (5)

    67. (4)

    68. (4)

    69. (2)

    70. (3)

    71. (3)

    72. (5)

    73. (4)

    74. (2)

    75. (1)

    76. (3)

    77. (4)

    78. (1)

    79. (2)

    80. (5)

    81. (2)

    82. (5)

    83. (2)

    84. (3)

    85. (3)

    86. (4)

    87. (2)

    88. (1)

    89. (3)

    90. (5)

    91. (4)

    92. (3)

    93. (1)

    94. (5)

    95. (2)

    96. (2)

    97. (3)

    98. (5)

    99. (1) 100. (4)

    48 .(1)

    101. (2) 102. (4) 103. (3) 104. (2) 105. (2) 106. (4) 107. (5) 108. (1) 109. (2) 110. (3) 111. (4) 112. (1) 113. (3) 114. (5) 115. (2) 116. (1) 117. (3) 118. (2) 119. (1) 120. (1) 121. (3) 122. (5) 123. (1) 124. (4) 125. (2) 126. (2) 127. (3) 128. (4) 129. (5) 130. (1) 131. (3) 132. (2) 133. (4) 134. (3) 135. (4) 136. (1) 137. (2) 138. (5) 139. (3) 140. (2) 141. (5) 142. (3) 143. (2) 144. (3) 145. (2) 146. (1) 147. (4) 148. (5) 149. (4) 150. (5) 151. (3) 152. (1) 153. (2) 154. (4) 155. (1) 156. (5) 157. (3) 158. (2) 159. (1) 160. (4) 161. (2) 162. (3) 163. (2) 164. (4) 165. (3) 166. (1) 167. (5) 168. (2) 169. (1) 170. (5) 171. (3) 172. (4) 173. (2) 174. (3) 175. (5) 176. (2) 177. (1) 178. (4)

    (1- 7) : Information given in the question can be presented in the table as below. Aug.

    Jan. Sep.

    Feb. Oct.

    Mar. Nov.

    Apr. Dec.

    May

    June

    July

    Mm

    Mm Mm

    Mm

    Mm Ec

    Mm Ec

    Ec

    Ec

    Ec

    B1

    B1

    B1

    B1

    B1

    Fm Ac

    Fm Ac

    Fm

    Fm

    Ec B1 B1

    B1

    B1

    Ac Pm

    Ac Ac Pm

    Ac aC Pm

    1. (2) Months February and July have only two lectures. Month available in option in February. Hence, option (2) is our answer. 2. (4) Maximum four lectures can be attended in a single month. 3. (2) Business law lasts from Jan. to Sep. and Personnel manage ment lasts from Oct. to Dec. 4. (1) Jan, March, April, May, June, July, August, Sept., Oct., Nov,. & Dec. In all a student has to attend lectures for 11 months. 5. (3) Three lectures-Marketing management, Economics and Business law last more than 6 months. 6. (4) Total number of series in September, October and November is 5 — Marketing, Economics, Accounting, Personnel management and Business law. 7. (3) 8 different lectures can be attended in January, February and March. 8. (4) Counting the number of days after 21st July 1947 to 21 July 1999 = 52 years + 366 days. = (13 leap yr + 39 ordinary years) + 366 days = (13 × 2 odd days + 39 odd days) + 2 odd days = 67 odd days = 4 odd days ie, (7 – 4) = 3 days beyond the day on 21 July 1999. So, the day was Saturday. 9. (2) Time interval of train is 45 min. Next train will leave at 8.30 pm. It means that earlier train had left at 7.45 pm. When the information was given it was 15 min late.

    BPRE–801

    Hence, the information was given at 8.00 pm. 10. (2) They will toll together after the time calculated by taking LCM of 6, 5, 7, 10 and 12 = 420 s or 7 min. Hence, in 1 h or 60 min they will toll 8 times excluding the one at the start. (11 - 14) : The revised schedule of interview is given below A B C D E F G H

    Panel-I Panel-II 9.50 to 10.00 10.30 to 10.40 9.40 to 9.50 10.20 to 10.30 10.00 to 10.20 10.30 to 10.00 10.00 to 10.10 10.40 to 10.50 10.40 to 10.50 10.00 to 10.140 10.20 to 10.30 9.40 to 9.50 10.30 to 9.40 10.10 to10.20 10.30 to 10.40 9.50 to 10.00

    11. (3) E and D finished the interview at 10.50 12. (4) C finished the interview at 10.20 . B finished at 10.30 and finished at 10.50. Hence, none of them finished before C. 13. (2) G finishes at 10.20 and A finishes at 10.40. Hence G has to wait for 20 min. 14. (3) D and E both leave at 10.50. 15. (3) On Thursday all the three brothers are available. 16. (4) For a maximum of four days only one brother is available at a particular time in a week. 17. (4) On Monday and Thursday between 10 am to 12 noon both Navin and Rajiv are available at home. 18. (2) Let the girls earns .x in other month. Then, earning in the

    PROBLEM SOLVING month of December = 2x. ∴ Then Total annual salary = 2x + 11x = 13x

    2x = 2 : 13 13x 19. (2) Time from 1 pm on Tuesdayto 1 pm on Thursday = 48 h The watch gains (1 + 2) = 3 min in 48 h. It will gain 1 min in 16 h. Hence, it will be correct at 5 am on Wednesday. 20. (4) As the given clock is synchronised with a clock losing 1 min every hour, it will now lose 3 min per hour. Hence, in 11 h it will lose 33 min. 21. (1) In 1 min, minute hand travels 6°. ∴ Angle travelled by minute hand in 20 min = 120° Now, 3h 20 min = 200 min ∴ Required ratio =

    1° 2 ∴ Angle travelled by hour hand in 200 min = 100° ∴ Required angle = 120 – 100 = 20° 22. (4) Third Wednesday falls on 15th of the month. Fourth Wednesday will fall on 22nd of the month. Fifth Wednesday will fall on 29th of the month. Fifth Friday will fall on 31st of the month. 23. (4) Suppose, Ranjeev’s age = x yr Ranjeev’s Brother’s Age = x + 6 yr Ranjeev’s Father’s Age = x + 6 + 32 = x + 38 yr Ranjeev’s Mother’s Age = x + 38 – 3 Ranjeev’s Sister’s Age = x + 35 – 25 = x + 10 yr 24. (4) In 1 h, hour hand moves by 30°.

    In 1 min, hour hand travels

    1 So, in 2 h, hour hand moves 2 1 = 75° 2 25. (4) Number of day between 1 November 1995 and 27 March 1995. 4 + 30 + 31 + 30 + 31 + 31 + 30 + 31 + 1 = 219 days. by 30 × 2

    219 Number of odd day = 7 =2 26. (1) 16 January 1997 was a Thursday. 16 January 1998 was a Friday. 16 January 1999 was a Saturday. 16 January 2000 was a Sunday. ∴ 9 January 2000 and 2 January 2000 were also on Sunday. ∴ 4 January was a Tuesday. 27. (2) If 28 February is Tuesday and it is not a leap year. Then, 7 March, 14 March 21 March and 28 March are also Tuesday. 28. (4) From the information given in the question, we can conclude that out of 60 students, number of boys and girls participating in sports are 15 and 10 respectively. Since, number of boys not participating in sports is not given, we cannot solve the question. 29. (2) Two-third of right angle is 60°. Let two hands of watch make 60° angle in x min. x×6–x×

    1 = 60 2



    11x = 60 2



    x

    =

    120 11

    = 10

    10 min 11

    ∴ required time = 10

    10 min 11

    past 3 30. (1) Time interval = 24 × 4 – 7 = 89 h Time lost in 24 h = 16 min ∴ Time measured by clock = 24h – 16 min

    356 44 h h= 60 15 ∴ Actual time when the clock measure = 23

    24 × 15 × 89 356 = 90 h ∴ Required time = 10 am + 1 h = 11 am 31. (3) Go through options, Ram’s present age is 26 yr. Last year, his age was a perfect square. 89 h

    =

    BPRE–802

    Next year, his age will be a perfect cube. Next cube number is 64. Required number of years = 64 26 = 38 yr 32. (1) By option, If women = 17, men = 15 and children =11 So, total = 43 33. (1) When there is a storm, y is blocked. When there are floods, X, 1 and 2 are affected along with Z, So, when both storm and floods are there, then road 3 can be used. 34. (1) 7th day will be Tuesday. 14th day will be Tuesday. So, 19th day will be Sunday. 35. (1) 3rd day is Monday. 10th, 17th and 24th will also be Monday. 5th day from 21st day is 26th day of that month. So, 26th day will be wednesday. 36. (3) If father’s birthday falls on Tuesday. Then, 7th, 14th ....... 63rd day after father’s birthday will also fall on Tuesday. Hence, 64th day from father’s birthday will fall on wednesday. 37. (1) Let the distance covered by taxi = x km Let the distance covered by car = y km So, x + y = 80 .........(i) 15x + 5y = 500 .......(ii) Solving both the equations x = 10 km 38. (3) The ratio of Boys : Girls = 3:1 So, 42 is not possible as it is not divisible by (3 + 1) = 4 39. (3) Go by options, If the money between Ravi and Ramu = 13 Ravi Ramu Before Win 8 5 After win 5 × 3 = 15 = which is 2 more than the original amount. 40. (3) 2 + 7 × l6 – (8 – 7) – (9 ÷ 3) =A 2 + 7 × l6 –l – 3 =A 2 + 112 – 4 = A A = 110 (41–45) : Day Wednesday Thursday Friday Saturday Sunday Monday Tuesday

    Play Q S P R Rest Day T U

    PROBLEM SOLVING 41. (4) Play P was organised after the Play S. 42. (1) Rest Day was on Sunday. 43. (3) Play U was organised in the last. 44. (2) The combination Saturday - R is correct. 45. (5) Play S was organised one day before Friday, i.e., on Thursday. Play T was organised one day before Tuesday, i.e., on Monday. Similarly, Play P was organised one day before Saturday, i.e., on Friday. (46–40) :

    Patna V

    Delhi P Amritsar X Kolkata R

    Hyderabad Q

    W Chennai

    T Bangalore S Chandigarh

    46. (2) R is sitting third to the right of W. 47. (4) S belongs to Chandigarh. 48. (1) Q is from Hyderabad. The delegate from Chennai W is sitting between Q and S. 49. (3) R belongs to Kolkata. R is sitting second to the left of P. R is sitting just opposite Q, the delegate from Hyderabad. 50. (5) The delegate from Hyderabad Q is sitting third to the right of X. (51–55) : Person P Q R S T U V W

    Month June March June December June December December March

    Fruit Mango Orange Apple Cherry Guava Banana Kiwi Peach

    51. (3) S, U and V were born in December. 52. (1) Q likes Orange and he was born in the month of March. W was also known in the month of March. 53. (2) T likes Guava. 54. (5) U likes Banana and he was born in the month of December. 55. (4) W likes Peach.

    (56–60) : Day Monday Tuesday Wednesday Thursday Friday Saturday Sunday

    Person D A G B C F E

    Game Volleyball Football Cricket Kho-Kho Hockey Squash Tennis

    56. (2) G was appointed on Wednesday. He plays cricket. 57. (5) F himself plays squash. Four persons were appointed before C. A plays football. B was appointed on Thursday. 58. (3) E plays tennis. He was appointed on Sunday. 59. (4) Monday – Volleyball combination is correct. 60. (1) Two persons – A and D – were appointed before G. (61 –66) : Day Friend Language Monday B Hindi Tuesday E French Wednesday F English Thursday C Chinese Friday G Spanish Saturday A German Sunday D Japanese 61. (3) B learns Hindi language immediately before the one who learns French language. Similarly, F learns English language immediately before the one who learns Chinese language. A learns German language immediately before the one who learns Japanese language. 62. (3) C learns Chinese language on Thursday immediately after F who learns English language. C learns Chinese language immediately before G. 63. (1) D learns Japanese language on Sunday (Thursday + 3). F learns English language on Wednesday (Sunday – 4). B learns Hindi language on Monday (Friday – 4). A learns German language on Saturday (Tuesday + 4). G learns Spanish language on Friday (Monday + 4). 64. (4) D learns Japanese language on Sunday. 65. (2) A learns German language on Saturday.

    BPRE–803

    66. (5) E learns French language on Tuesday. 67. (4)

    6 9 8 5 2 7 3 1 1 2 3 5 6 7 8 9 (68–70) : B>A>E D>B C>D>B>A>E ↓ ↓ 24 11 68. (4) Jar D has more than 11 but less than 24 cookies. 69. (2) Jar A has the second lowest number of cookies. 70. (3) Jar E has less than 11 cookies. (71-75): Rank 1 2 3 4 5 6 7 8

    Player F A H B E G C D

    Game Hockey Table Tennis Basketball Chess Cricket Football Volleyball Rugby

    71. (3) C plays volleyball and G plays football. 72. (5) 3 ⇒ Basketball 5 ⇒ Cricket 73. (4) Chess ⇒ 4 Hockey ⇒ 1 74. (2) Volleyball is exactly between Football and Rugby 75. (1) D ⇒ 8 Cricket ⇒ 5 (76-80) : Month January

    Date 12th 25th March 12th 25th April 12th 25th May 12th 25th June 12th 25th December 12th 25th

    Person U X P R V O Y Q Z S W T

    Colour Green Maroon Blue Orange Violet Yellow Red Grey White Pink Silver Brown

    PROBLEM SOLVING 76. (3) P and R had lecturres in the month of March. 77. (4) V–Violet combination is correct. 78. (1) Z likes white. There is a gap of only one lecture between Y and Z. V likes violet. There is a gap of only one lecture between P and V. Q likes grey. There is a gap of only one lecture between O and Q. 79. (2) Four persons – P, R, V and O – have lectures between the lectures by X and Y. 80. (5) Except 12–Pink, all others are correctly matched. (81–85) : Month

    March

    June

    December

    Person

    Fruit

    Q

    Orange

    W

    Peach

    P

    Mango

    R

    Apple

    Person P Q R S T U V

    Company ITC Wipro Wipro ITC Samsung Samsung Samsung

    Guava

    Month

    S

    Cherry

    U

    Banana

    V

    Kiwi

    January March April July August September December

    Month

    Date

    Person

    March

    10th

    M

    27th

    X

    10th

    W

    27th

    N

    October

    10th

    Z

    27th

    O

    November

    10th

    V

    27th Y 86. (4) M attends his seminar on 10th March. 87. (2) X attends his seminar on 27th March. 88. (1) Five persons – N, Z, O, V and Y – attend seminar after W. 89. (3) Z attends his seminar on 10th October. 90. (5) Only Y attends seminar after V.

    Fruit Guava Banana Orange Kiwi Mango Strawberry Apple

    91. (4) Q and R work in Wipro company. 92. (3) P likes Guava. 93. (1) V works in the Samsung company and likes Apple. 94. (5) T – Mango is correct. 95. (2) R works in Wipro with Q. U works with T who likes Mango. S works in ITC. Q likes Banana. (96–100)

    T

    81. (2) S was born in December and he likes Cherry. 82. (5) Q and W are elder to T. 83. (2) R–June-Apple combination is true. 84. (3) V likes Kiwi. V was born in December. 85. (3) R likes Apple. So, S would like Apple. (86–90) :

    June

    (91 – 95) :

    Student Bank F G C D B H E

    PNB HDFC AXIS CANARA ICICI Yes SBI

    96. (2) C and D attend workshops after March but before August. C likes Axis Bank. 97. (3) H likes Yes Bank. 98. (5) SBI ⇒ December PNB ⇒ January 99. (1) HDFC ⇒ March C ⇒ April 100. (4) Canara Bank ⇒ July (101–105) : Thursday (Sun) Wednesday 17 20 22 16 Tuesday Sunday L I G K 24 13 F E Thursday O N 27 10 Thursday D H M J (Sunday) 30 7 Monday Wednessday 1 5 (Tues) Saturday

    101. (2) 10 days gap. 102. (4) I is third to the right of Professor E. I gives lecture on Thursday. 103. (3) Professors E, I and O give lectures on Thursday.

    BPRE–804

    104. (2) K gives lecture on 22nd. F is fourth to the right of K. 105. (2) Professor D gives lecture on Monday. (106–110) : Day

    Employee Department

    Monday

    M

    HR Department

    Tuesday

    P

    Security

    Wednesday

    Q

    Administrative

    Thursday

    L

    Security

    Friday

    N

    Administrative

    Saturday

    R

    HR Department

    Sunday

    O

    Finance

    106. (4) N attends meeting on Friday. 107. (5) ‘P – Tuesday’ combination is true. 108. (1) L attends meeting in Security department. 109. (2) Q attends meeting on Wednesday and Wednesday – 2 = Monday. L attends meeting Thursday and Thursday – 2 = Tuesday. N attends meeting on Friday and Friday – 2 = Wednesday. 110. (3) Four persons – Q, L, N and R – attend meeting between P and O. (111–115) : Employee City Department P Yelahanka Operation Q Bengaluru Personnel T Jodhpur Accounts B Ooty Operation R Gwalior Operation J Palampur Personnel N Raipur Accounts A Warangal Accounts 111. (4) P, B and R work in the Operation department. P likes Yelahanka, B likes Ooty while R likes Gwalior. 112.(1) The combination Jodhpur – Accounts is correct. 113.(3) Q works in the Personnel department. N works in the Accounts department. R works in the Operation department. 114.(5) P likes Yelahanka. 115.(2) P works in the Operation department and he likes Yelahanka. A works in the Accounts department and he likes Warangal. B works in the Operation department and he likes Ooty. N works in the Accounts department and he likes Raipur.

    PROBLEM SOLVING

    G

    Humorous June

    Brave I August

    Q

    Assertive October

    J

    Kamala

    Lakshmi

    Malti

    Sudha

    Saraswathi

    Radha

    Lakshmi

    Kamala

    Mahesh

    Keshav

    Ravi

    Ganesh

    Gaurav

    Sunil

    121. (3)

    12

    18

    24

    30

    Mahesh

    Lakshmi

    Saraswathi

    Malti

    Gaurav

    6 Ravi

    Radha is sixth to the right of Ganesh. 122. (5) Q

    36 P

    0

    10

    20

    30

    40

    Ganesh

    Keshav

    Kamla

    50 Y

    Sunil

    Radha

    Sudha

    Saraswathi is fourth to the right of Gaurav. 123. (1) X

    12

    18 Saraswathi

    24 Mahesh

    6

    Malti

    0

    30

    36 P

    Saraswathi

    Malti

    Saraswathi is second to the left of Keshav. 125. (2) 0 10 20 50 30 40 D J

    (126–128) :

    50 metres

    D

    Q

    Kamala

    Mahesh

    2004

    Malti

    Sunil

    2001

    P

    Month January

    124. (4)

    Keshav

    Keshav

    2003

    Sudha

    Ravi

    2002

    Y 2005

    36 metres

    Q X 2000

    B 1995

    1994

    Saraswathi

    1993

    Radha

    1992

    Gaurav

    1991

    Ganesh

    116. (1) P is sitting second to the right of O. 117. (3) H is sitting exactly between K and L. 118. (2) August → I (Brave) September → K (Calm) 119. (1) H was born in May. L was born in January. February → M (Realistic) 120. (1) R is an Interactive Person. P is fourth to the right of R. (121–125) :

    J Honest March

    P July Tactful

    Gaurav

    N November Intuitive

    129. (5) Except ‘Tuesday–Zoology’, in all others there is a gap of one day between the given entities. 130. (1) Two persons – J and K – have their interviews between L and M. 131. (3) J has his interview immediately before K. M has his interview on Wednesday. L likes Zoology. L has his interview after J. 132. (2) L has an interview immediately after K. L likes Zoology. 133. (4) N likes Mathematics. N has an interview on Monday. (134-138) :

    Realistic February M

    Interactive December R

    O Dynamic April

    A 1990

    Calm September K

    Ganesh

    Traditional May H

    January Friendly L

    Keshav

    (116–120) :

    Sudha is third to the right of Keshav.

    Floor Number 9 8 7 6 5 4 3 2 1

    Person H E G D J F K I L

    126. (2) Floor Number 6 ⇒ D Floor Number 3 ⇒ K Floor Number 1 ⇒ L 127. (3) D lives exactly between G and J. 128. (4) J, E and K live between the floors of D and I. (129–133) : Day Person Monday N Tuesday O Wednesday M Thursday J Friday K Saturday L Sunday P

    Subject Mathematics Psychology Sociology Economics Statistics Zoology English

    BPRE–805

    Date 12th 25th March 12th 25th April 12th 25th May 12th 25th June 12th 25th December 12th 25th

    Person U X P R V O Y Q Z S W T

    Colour Green Maroon Blue Orange Violet Yellow Red Grey White Pink Silver Brown

    134. (3) P and R had lectures in the month of March. 135. (4) V–Violet combination is correct. 136. (1) Z likes white. There is a gap of only one lecture between Y and Z. V likes violet. There is a gap of only one lecture between P and V. Q likes grey. There is a gap of only one lecture between O and Q. 137. (2) Four persons – P, R, V and O – have lectures between the lectures by X and Y. 138. (5) Except 12–Pink, all others are correctly matched. (139–143) : Gooseberry 15 Apples 10 Guava 8 Mango 12

    Orange 5 Cherry 4 Plum 20 Banana 25

    139. (3) Goosberry + Orange = 15 + 5 = 20 140. (2) Plum is 20 in number. 141. (5) There are 10 Apples.

    PROBLEM SOLVING 142. (3) Mango and Banana are kept in the bottom row. 143. (2) Plum is kept exactly between Cherry and Banana. (144–148) : Day Monday Tuesday Wednesday Thursday Friday Saturday Sunday

    Person D B E A G C F

    City Pune Bangalore Patna Mumbai Chennai Kolkata Jaipur

    144. (3) G attends seminar on Friday. 145. (2) B belongs to Pune. D attends seminar on Monday. 146. (1) E attends seminar on Wednesday. E belongs to Patna. 147. (4) D belongs to Pune and C belongs to Kolkata. Four persons attend seminars between D and C. 148. (5) A attends seminar on Thursday. A belongs to Mumbai. (149–153) :

    Shiva 18 years

    Piyush 20 years

    Nitin 22 years

    154. (4) Q belongs to HR department. 155. (1) P is the head of Production department. 156. (5) Gym is second to the left of Marketing department. Gym is occupied by R. 157. (3) Finance department is just above R’s department. B is head of Finance department. 158. (2) Canteen is located immediate left below diagonally of Media department. 159. (1) I

    >

    =

    C

    A 52

    >

    F

    I

    C 60

    >

    F 60

    =

    Tarun 14 years

    Om 26 years Dinesh 28 years

    149. (4) Nitin sits exactly between Piyush and Ravi. Nitin is 22 years old. 150. (5) Tarun sits second to the left of Om. Tarun is 14 years old. 151. (3) Dinesh sits third to the right of Nitin. Dinesh is 28 years old. 152. (1) Ravi sits opposite Mithun. Ravi is 24 years old. 153. (2) Mithun and Piyush are immediate neighbours of Shiva. Mithun + Piyush = (16 + 20) years = 36 years (154-158) : Column–1

    Column–2

    Column–3

    Finance B

    HR Q

    Media V

    Row – 3

    Gym R

    Canteen A

    Marketing C

    Row – 2

    Production P

    Accounts S

    Technical X

    Row – 1

    Security U

    Parking W

    Training T

    Row – 4

    A 58

    >

    B 48

    =

    D

    >

    J

    E 58

    =

    H 56

    >

    =

    G

    B

    =

    D

    >

    J

    >

    Sum = (58 + 56) kg. = 114 kg. 161. (2) I

    >

    C

    =

    F 53

    >

    A 51

    =

    E 51

    >

    H 49

    =

    G 49

    >

    B 47

    =

    D 47

    >

    J 45

    Sum = (53 + 51) kg. = 104 kg. 162. (3)

    I > C = F > A

    = E > H = G > B = D > J

    Sum = (59 + 57) kg. = 116 kg. 163. (2)

    I > C = F > A

    = E > H = G > B = D > J

    (164–168) : Column-1 Column-2

    Ravi 24 years

    G > 50

    =

    Sum = (52 + 48) kg. = 100 kg. 160. (4)

    Row-4 Bank

    Mithun 16 years

    E H > 52 50

    =

    Row-3 Hospital Row-2 Parlor

    Column-3

    D IT Company I H B

    Row-1 Hotel

    Institute Yoga Class

    K L

    Gym

    Mobile Shop C Book Shop Toy Shop

    G A

    Car Parking J

    F

    E

    164. (4) L owns Yoga class. 165. (3) G owns Book Shop and H owns Hospital. 166. (1) B, L and A own the flats in the Row-2 of the complex. 167. (5) D owns Bank and J owns Gym. 168. (2) E owns Car Parking and B owns Parlor. (169–173) : ion at m r fo J In

    PWD G

    Police C

    Ho us in F g

    B Medical

    H Energy

    E Finance

    D Irrigation

    In L du st

    ry

    K Welfare

    A Law

    I ure lt Cu

    169. (1) E belongs to the Finance department. 170. (5) C belongs to the Police department.

    BPRE–806

    171. (3) A sits exactly between K and I. A belongs to the Law department. 172. (4) H belongs to the Energy department. K sits fourth to the left of H. 173. (2) F belongs to the Housing department. When counted from the left of B, three officers – J, G and C – sit between B and F. (174–178) : Person A B C D E F G H

    Class II V V II XI VIII XI VIII

    City Bhopal Varanasi Jaipur Ujjain Siliguri Nashik Patna Chennai

    174. (3) C lives in Jaipur. 175. (5) Except A–Patna, in all other pairs both the persons study in the same class. 176. (2) H studies in Class VIII. 177. (1) A lives in Bhopal. 178. (4) B lives in Varanasi. G and E study in the same class. B and C study in the same class. C lives in Jaipur and he studies in Class V. ❑❑❑

    ARGUMENTATION

    13

    ARGUMENTATION

    QUESTIONS FROM 1999 TO 2010 ARE AVAILABLE ONLINE NATIONALISED BANKS & IBPS PO/MT/SO EXAMS Directions (1–4) : Read the following statements carefully and answer the questions which follow. (Indian Bank PO Exam. 02.01.2011 (Ist sitting)

    1. Real estate builders have refused to bring down the property prices as proposed by the government this year. Which of the following arguments would strengthen the stance taken by the builders ? (1) Very few people ventured into buying properties this year owing to exorbitant prices set by the builders. (2) With the ever increasing cost of basic materials such as cement and steel, the profit of the builders has gone down by 48 per cent this year. (3) The builders have earned huge profit in a governmental scheme for building low cost housing societies. (4) In a report published by a national daily, the margin of profit earned by a builder per square foot is as much as eighty percent of its cost price. (5) The builders have to necessarily abide by the decree of the government which controls the prices of the real estate. 2. The ministry of sports has been advised by a committee to take the highest award in the field of sports back from two players who were allegedly-involved in match fixing. Which of the following statements would weaken the argument put forward by the committee to the sports ministry ? (1) A good conduct in the past and a lack of evidence against the players make the case against them very weak.

    (2) The ministry of sports has never declined the recommendations made by the committee earlier. (3) Taking the award back from the players would set a good example to other players for avoiding such actions in the future. (4) There have been past cases where the award had to be taken back from the players owing to some misconduct later on. (5) The committee is constituted of some of the most respected and esteemed members from the fields of sports and politics. 3. Many organizations have been resorting to recruitment based upon performance at graduate/ post-graduate level exams rather than conducting exams for the same purpose. Which of the following statements would strengthen the argument given in the above statement ? (1) A recent study shows no link of past performance with the performance in recruitment exams. (2) The graduate/post-graduate exams are considered to be severely deficient in training in job related environment. (3) Organisations which had undertaken recruitment on the basis of graduate/post graduate exams report a significant drop in the quality of the recruited employees. (4) Such policies would add to unemployment among students having below average performance in graduation or postgraduation. (5) Such policies could save time, money and resources of the organisation which are wasted in the conduct of recruitment examinations.

    BPRE–807

    4. According to a recent government directive, all bank branches in rural areas should be computerized. Which of the following statements would weaken the government’s argument ? (1) Computerisation of bank branches in urban areas has helped in making their performance more efficient and fast. (2) Lack of skilled and qualified manpower has been suitably substituted by computers in banks. (3) Non-computerised bank branches in the rural areas have been proved to be as efficient as their computerized counterparts. (4) The government has introduced a special test for computer knowledge in all recruitment exams for banks. (5) Unemployment in the rural areas could be controlled by training more and more professionals in computers. 5. Read the following information carefully and answer the question which follows. ‘Parents should make strong efforts to curtail the extreme fad for fashion from their children’s minds.’ A research report. Which of the following would weaken the argument presented in the report most appropriately ? (1) Excessive craze of fashion is detrimental to development of a youngster’s personality. (2) Following fashion is not harmful but only till it is done to certain reasonable limits. (3) More fashionable the person, greater are his/her chances to succeed in life. (4) Fashion takes away the time spent in constructive activities by the youngsters.

    ARGUMENTATION (5) Many schools and colleges have uniforms for their students so as to discourage the craze of fashion amongst them. (Indian Overseas Bank PO Exam. 22.05.2011)

    6. A very large number of technically qualified young Indians are coming out of colleges every year though there are not enough opportunities for them to get gainful employment. Which of the following contradicts the views expressed in the above statement? (1) Technically qualified persons are far superior to those with standard degrees like BA/ B.Sc/ B.Com. etc. (2) The Government has not done effective perspective planning for engaging technically qualified personnel while authorising the setting up of technical colleges. (3) A huge gap exists between the level of competence of technically qualified graduates and requirement of the industry. (4) Majority of the technically qualified persons are migrating from India to developed countries for better opportunities. (5) None of these (IBPS Bank PO/MT CWE Exam. 18.09.2011)

    7. Statement : It has been reported in a recent study that intake of moderate quantity of milk chocolate reduces the risk of suffering from central nervous system related illnesses. Which of the following would weaken the findings of the study reported in the above statement? (1) People generally prefer to eat chocolate when they are young. (2) Majority of those not diagnosed with diseases related to central nervous system have stayed away from eating chocolates in their lives. (3) Chocolates contain certain elements which strengthen the functions of the central nervous system

    (4) Majority of those suffering from central nervous system related diseases are middle aged. (5) Many of those who suffer from diabetes also suffer from other major ailments. (IBPS Specialist Officer CWE Exam. 11.03.2012)

    Directions (8–10) : Read the following statements carefully and answer the questions which follow. (IBPS Specialist Officer CWE Exam.17.03.2013)

    8. The ministry of sports has been advised by a committee to take the highest award in the field of sports back from two players who were allegedly-involved in match fixing. Which of the following statements would weaken the argument put forward by the committee to the sports ministry ? (1) A good conduct in the past and a lack of evidence against the players make the case against them very weak. (2) The ministry of sports has never declined the recommendations made by the committee earlier. (3) Taking the award back from the players would set a good example to other players for avoiding such actions in the future. (4) There have been past cases where the award had to be taken back from the players owing to some misconduct later on. (5) The committee is constituted of some of the most respected and esteemed members from the fields of sports and politics. 9. Many organizations have been resorting to recruitment based upon performance at graduate/ post-graduate level exams rather than conducting exams for the same purpose. Which of the following statements would strengthen the argument given in the above statement ? (1) A recent study shows no link of past performance with the performance in recruitment exams. (2) The graduate/post-graduate exams are considered to be severely deficient in training in job related environment.

    BPRE–808

    10.

    11.

    (1) (2)

    (3) Organisations which had undertaken recruitment on the basis of graduate/post graduate exams report a significant drop in the quality of the recruited employees. (4) Such policies would add to unemployment among students having below average performance in graduation or post-graduation. (5) Such policies could save time, money and resources of the organisation which are wasted in the conduct of recruitment examinations. According to a recent government directive, all bank branches in rural areas should be computerized. Which of the following statements would weaken the government’s argument ? (1) Computerisation of bank branches in urban areas has helped in making their performance more efficient and fast. (2) Lack of skilled and qualified manpower has been suitably substituted by computers in banks. (3) Non-computerised bank branches in the rural areas have been proved to be as efficient as their computerized counterparts. (4) The government has introduced a special test for computer knowledge in all recruitment exams for banks. (5) Unemployment in the rural areas could be controlled by training more and more professionals in computers. Statements : Teachers teach reagarding terrorist attacks from books in the colleges. Parents object this because at this tender age if students study about violence it will have negative impact on the students. Teachers say that movies, television, social media etc. expose to violence more than the lessons taught in the classes. Which of the following statements would weaken the parents’ argument ? It has been proved that electronic media do much harm than the benefits associated with them Teachers teach something in broad perspective and also warn

    ARGUMENTATION students regarding the harmful aspect of any event (3) Students of colleges are mature enough to distinguish between good and bad things (4) There are many other factors which shape the attitude of a student. (5) It is our duty to acquaint students about everything whether it is good or bad. (SIDBI Officer Exam. 03.09.2014)

    12. Statements : The Government has enacted new Company Act which envisages that companies should spend at least two per cent of their profit on the activities related to Corporate Social Responsibility. Which of the following statements would strengthen the argument given in the above statement ? (1) It is the duty of entrepreneurs to shoulder the social responsibility and they should spend some money for the deprived classes. (2) The Government should take the responsibility for the welfare of deprived classes. (3) Many companies are already implementing various programmes to help the needy people. (4) The Government should increase the rate of tax on the bigger companies and the extra revenue collected should be spent on the deprived classes (5) The New Company Act has many drawbacks and corporate sector may escape from this responsibility easily. (SIDBI Officer Exam. 03.09.2014)

    13. Statement : Company G terminates contract with company S. Company G starts buying raw material from Company L. Which of the following may be the most plausible reason for the termination of the contract ? (1) Company S was not providing the raw material of good quality. (2) Company G is offered raw material at better discount from Company L. (3) Company L even being a new entrant used an extensive marketing strategy to sell its goods to Company G. (4) Company G wanted to shift to new variant of finished products which can be provided by only Company L.

    (5) Company S was not looking after the problems of Company G. (IBPS Bank PO/MT CWE-IV, 18.10.2014)

    14. Statements : The outbreak of the food-borne illness is causing serious problem in day-to-day lifestyle. Many doctors advise people to reduce the consumption of fast food. It will lower down the health problem to a certain extent. (A) According to a survey about the health issues, people should improve the quality of food. Busy lifestyle and increase in the consumption of fast food is causing mouthful of diseases. (B) Synthetic fertilizers and pesticides used in vegetables and fruits are equally harmful for consumption. Which of the two statements mentioned above would weaken the argument given in the paragraph ? (1) Only (A) (2) Only (B) (3) Either (A) or (B) (4) Neither (A) nor (B) (5) Both (A) and (B) (IBPS Bank PO/MT CWE-IV, 18.10.2014)

    15. Statement : Doctors found that Company M is selling drugs without testing the important aspects of the drugs. Such practice tends the doctor to negate the use of drug. Which of the following arguments would strengthen the stance of the Company M ? (1) The commercial production of any drug is permitted after its repeated trial and verification of result. (2) Some other companies are also selling the drugs which are sold by the Company M. (3) Some people do not agree with the view put forward by the doctors. (4) Any drug is put on test by the pharmacist. Doctors are not competent to approve or disapprove any drug. (5) Doctors promote those drugs which are very costly as they have some share in the profit. (IBPS Bank PO/MT CWE-IV, 18.10.2014)

    16. Study the following information and answer the given question.

    BPRE–809

    Despite strong opposite from the social activists of village X, Pollution Control Board renewed the license of a coal depot to operate coal handing barely one kilometre away from the village. Which of the following statements supports the decision of Pollution Control Board to some extent? (1) Several crops were destroyed due to disposal of waste from the coal depot in the previous year. (2) Maintenance cost of roads in village X has increased due to transportation of coal. (3) The depot provides employment for nearly 70% of people in the neighbouring area. (4) Several complaints have been filed against the depot in the past for violating safety standards. (5) Nine out of ten residents of village X suffer from respiratory problems. (IBPS RRBs Officer Scale–I & II CWE 12.09.2015)

    17. Study the following information carefully and answer the question which follows: Number of deaths due to ABN poisoning has risen in village A from 23 two years ago, to 67 this year. Since the number of patients is the same, it can be concluded that the new medication is the responsible factor for the deaths. Which of the following statements weakens the given statements that the new medication is responsible for increased number of fatalities? (1) While azethorin–1 was being used to treat ABN poisoning since 5 years, azethorin–6 was introduced last year after it obtained necessary clinical approvals. (2) PB released from a new fertilizer being used by the farmers of the village this years, when combined with ABN renders all types of medicines ineffective to treat the disease. (3) Due to low awareness regarding the disease, very low patients have been reporting this disease for many years which proves that the actual numbers could be much higher.

    ARGUMENTATION (4) The new medicines introduced in village A for TQW and RST poisoning has been successful in significantly bringing down the fatality rate. (5) A cement factory set up 5 years ago in the village was responsible for leaching ABN in the soil as well as drinking water way above the danger limit. (IBPS RRBs Officer Scale–I & II CWE 12.09.2015)

    18. Study the following information carefully and answer the question which follows : Farmers should use hydroponics for farming. Hydroponics is growing plants in liquid nutrient solution. Since it does not require soil, farmers can save a large amount of time by avoiding tilling and de-weeding of their fields. Which of the following statements does not weaken the given statement ? (1) Initial set up cost for hydroponics is very high as the necessary equipment and inputs are three times more expensive than that used in traditional methods. (2) Disease causing micro-organisms that are water-based can enter and subsist much more easily in hydroponics based environment as compared to traditional farming. (3) Rather than using only hydroponics or only traditional farming, most of the farmers use both simultaneously. (4) Without soil to serve as a buffer, if the hydroponics system fails, plant death occurs very rapidly leading to loss of 70-85% crop. (5) The production using hydroponics is very limited and is more efficient for house gardens rather than large scale farming. (IBPS RRBs Officer Scale–I & II CWE 13.09.2015)

    19. Study the following information and answer the question. In a recent performance appraisal done by Company X, more than 70% of the employees were found to be under performing. “I think that the restructuring done by the company in the previous year is responsible for the under–performance of the employees”– HR Manager of Company X.

    Which of the following statements weakens the statement of HR manager of Company X? (1) The incentives linked to performance were abolished upon restructuring creating dissatisfaction among employees. (2) After restructring the decision making power was takjen away from employees thus delaying the work by long hours. (3) The number of projects in Company X increased by 60% this year thereby increasing the burden on the existing employees this year. (4) After restructuring the employees were required to report to multiple bosses leading to ambiguity at the work place. (5) Adequate training was not provided to the employees transferred to different departments and locations after restructuring. (IBPS Specialist Officer (Marketing) CWE 01.02.2016)

    20. This question consists of information and two statements numbered I and II given below it. You have to decide which of the given statements weakens or strengthens the information and decide the appropriate answer. Information : Due to increased cases of kidnapping in its vicinity, school M has made it compulsory for parents or legal guardians of the students to give a duly signed authority letter to the person picking up the students from school. I. Most working parents rely on their domestic help, for picking up their children from school, who can easily coerced into forging or misusing the authority letter for monetary plans. II. There is no photograph on the authority letter making it difficult to identify the person who comes to pick up the child. (1) Both statement I and statement II strengthen the information. (2) Statement I strengthens the information while statement II weakens the information. (3) Both statements I and II weaken the information. (4) Both statements I and II are neutral statements.

    BPRE–810

    (5) Statement I weakens the information while statement II strengthens the information. (IBPS Specialist Officer (Marketing) CWE 01.02.2016)

    21. This question consists of an information and two statements numbered I and II given below it. You have to decide which of the given statements weaken/s or strengthen/s the given information and decide the appropriate answer. Information : Examination committee of College X has recommended that the negative marking in the exam should be removed. I. The students tend to give answers simply by guessing when there is no negative marking in the exam. II. Selecting the best candidates from the population becomes difficult in the absence of negative marking. (1) Both Statement I and Statement II weaken the information. (2) Statement I strengthens the information while statement II weakens the information. (3) Statement I weakens the information while statement II strengthens the information. (4) Both Statement I and Statement II are neutral statements. (5) Both Statement I and II strengthen the information. (IBPS Specialist Officer (IT) CWE 14.02.2016)

    22. This question consists of an information and two statements numbered I and II given below it. You have to decide which of the given statements weaken(s) or strengthen(s) the information and decide the appropriate answer. Information : According to a wildlife activist, keeping animals captivated in zoos is a form of cruelty to animals. All such animals should be released in sanctuaries in the wild, i.e., their natural habitats. I. By bringing people and animals together, zoos tend to educate the public and foster appreciation for animals, which in turn, motivates people to protect them.

    ARGUMENTATION II. Animals in captivity suffer from monotonous life and develop stress behaviour. The confinement breaks the inter–generations bonds which are essential for their specific behavioural pattern. (1) Both Statement I and Statement II are neutral Statements (2) Both Statement I and Statement II weakens the information. (3) Statement I strengthens the information while Statement II weakens the information. (4) Statement I weakens the information while Statement II strengthens the information. (5) Both Statement I and Statement II strengthen the information. 23. This question consists of an information and two statements numbered I and II given below it. You have to decide which of the given statements weaken(s) or strengthen(s) the information and decide the appropriate answer. Information : Department X of the Company ABC has been complaining of high work load since the past three months and the complains have not been subsidised despite the fact that around two months ago the Company ABC has employed six new employees specifically for the Department X. I. The works in the Department X requires highly specialised skills and new employees joining the Department have to undergo work specific training for atleast six months. II. The number of employees who resigned from the Department X six months ago is double of the number of employees who have joined the Department X two months ago. (1) Both Statement I and Statement II are neutral statements (2) Both Statement I and Statement II weakens the information. (3) Statement I strengthens the information while Statement II weakens the information. (4) Statement I weakens the information while Statement II strengthens the information.

    (5) Both Statement I and Statement II strengthen the information. (United Bank of India PGDBF Manipal Exam,07.08.2016)

    24. Decision : The authorities of cricket team of country B decided to replace the captain of the team with a relatively new player for the upcoming series to be played against country G. I. Most of the strategies of the old captain failed to help the team with the series held in the past two years. II. The captain of the team was found guilty of ‘ball-tampering’ in one of the major test series held during recent time. (1) Both statement I and statement II weaken the decision. (2) Statement I strengthens the decision while statement II weakens the decision. (3) Both statements I and II are neutral statements. (4) Statement I weakens the decision while statement II strengthens the decision. (5) Both statements I and II strengthen the decision. (Indian Bank PO (Pre.) Exam, 21.01.2017 (2nd Sitting))

    25. This questions consists of a decision and two statements numbered I and II given below it. You have to decide which of the given statements weaken(s) or strengthen(s) the decision and mark the appropriate answer. Decision : ‘Health-Wise’ is a chain of stores of healthcare products in country K. It has decided to keep products of only 20 companies in its store from now onwards. I. These 20 companies have been associated with ‘Health-Wise’ for almost a decade and hence the time between the supply and demand of products is very less. II. 90% of the 20 listed companies have been awarded as the best healthcare products providing companies of the country. (1) Both Statements I and II are neutral statements. (2) Statement I weakens the decision while Statement II strengthens the decision. (3) Both Statements I and II weaken the decision

    BPRE–811

    (4) Statement I strengthens the decision while Statement II weakens the decision. (5) Both Statements I and II strengthen the decision. (IBPS SO (IT Officer) CWE (Prelim Exam) 30.12.2017)

    26. This question consists of a decision and two statements numbered I and II given below it. You have to decide which of the given statements weaken/s or strengthen/s the decision and decide the appropriate answer. Decision : The cricket board of Country V has decided to replace the cricket coach of the team before the onset of its ODI series against Country X which is scheduled after 30 days. Statements : I. The cricket team of Country V had not been performing well in the last one and a half years except for the series against Country H during which the current coach was not well and the team was guided by a temporary coach. II. One of the players of Country V recently had a small tiff with the coach. (1) Statement I strengthens the decision while Statement II weakens the decision (2) Both Statements I and II are neutral Statements. (3) Statement I strengthens the decision while Statement II is a neutral statement. (4) Both Statement I and Statement II weaken the decision (5) Statement I weakens the decision while Statement II strengthens the decision (IBPS SO (Law Officer) CWE (Prelim Exam) 31.12.2017)

    27. This question consists of an information and two statements numbered I and II given below it. You have to decide which of the given statements weaken/s or strengthen/s the information and decide the appropriate answer. Information : Average number of employees resigning from Company A in a year is one-third of that resigning from Company Z. This shows that work environment in Company A is superior to that in Company Z. I. The number of employees working in Company Z is one lakh while that in Company A is 432.

    ARGUMENTATION II. Company A has 23 departments in total whereas Company Z has only 12 departments. (1) Both statements I and II weaken the information. (2) Statement I weakens the information while statement II is a neutral statement. (3) Statement I strengthens the information while statement II is a neutral statement. (4) Statement I weakens the information while statement II strengthens the information. (5) Statement I strengthens the information while statement II weakens the information. (Canara Bank PO Exam 04.03.2018)

    Directions (28–30) : In making decisions about important questions, it is desirable to be able to distinguish between strong arguments and weak arguments in so far as they relate to the question. Strong arguments are those which are both important and directly related to the question. Weak arguments are those which are of minor importance and also may not be directly related to the question or may be related to a trivial aspect of the question (IDBI Bank PO Exam 29.04.2018)

    Give answer (1) if only Argument I is strong Give answer (2) if only Argument II is strong Give answer (3) if either Argument I or Argument II is strong Give answer (4) if neither Agrument I nor Agrument II is strong Give answer (5) if both the Agrument I and Argument II are strong 28. Statement : should the public sector undertakings be allowed to adopt hireand-fire policy? Arguments : I. Yes, as this will help the public sector undertakings to get rid of non-performing employees and will also help to reward the performing employees. II. No, the management may not be able to implement the policy in an unbiased manner and the employees will suffer due to the high-handedness of the management. 29. Statement : Should one close relative of a retiring government employee be given a job in government in India?

    Arguments : I. Yes, where else will the relative get a job like this? II. No, it will close doors of government service to competent and needy youth. 30. Statement : Should the term of the elected members of parliament be reduced to two years in India? Arguments : I. Yes, even otherwise the election are generally held every alternate year in India. II. No, every round of Parliament election needs huge amount of money and it’s a national waste. 31. The Amendment bill intended to insert Section 18A in the SC/ST (Prevention of Atrocities) Act. As per this new Section, a preliminary enquiry will not be required before an FIR is to be registered first. So the Bill says that, any provision for conducting preliminary inquiry will delay the investigation and thus filing of the charge sheet. Which of the following, if true, most severely weakens the argument presented above? (1) As per the new insertion, the FIR can be filed at first instance as after the incident occurred. (2) Preliminary enquiry before filing FIR bundles up the case timings. (3) The enquiry before FIR tampers the charges in case of SC/ST issues. (4) Before the amendment, the Police had to conduct an inquiry even before FIR filing. (5) None of these (IBPS Bank PO/MT CWE (Main Exam) 26.11.2017)

    32. “Around 40 lakh religious and linguistic minority people have been eliminated from NRC (National Register of Citizens) which will have serious ramifications on the demography of different states adjoining Assam”. Few groups of local minority groups have given a threat of social order in this regard. As many as six FIRs have been filed against him in different police stations of Assam for trying to foment trouble and disturb communal harmony. Which of the following, if true, provides the best evidence that

    BPRE–812

    the illegal migrants are threat to the social order? (1) The elimination of 40 lakh minorities have been swept from the list (2) The pro-action of Police followed by the elimination of the junk linguistic minorities from the list. (3) Fewer minorities section of the eliminated community provoked to be indifferent with NRC (4) NRC of Government tries to control illegal immigrants (5) None of these (IBPS Bank PO/MT CWE (Main Exam) 26.11.2017)

    33. In this question, some information and two statements numbered I and II are given below it. You have to decide which of the given statements weaken/s or strengthen/s the information and decide the appropriate answer. Information: As per a recent report by an economic research body, “the income gap between the rich and the poor has continued to widen over the past five years in country X’’. Statement I : About ninety percent of the total income of country X was earned by the top ten percent of the people in the past five years. Statement II : Out of the top ten percent of the wealthy people of country X, the wealth of one tenth has risen faster than that of others in that category over the years. (1) Statement I strengthens the information while Statement II is a neutral statement. (2) Both statement I and statement II strengthen the information (3) Statement I weakens the information while statement II strengthens the information. (4) Statement II weakens the information while Statement I strengthens the information. (5) Both Statement I and Statement II weaken the information. (Indian Bank Specialist Officer SO Exam, 08.03.2020)

    34. Study the following information to answer the given question : “We are commencing the airlines services from City A to City B of Country K at affordable prices

    ARGUMENTATION from the month of April this year. The airline will be routed from City A to City B via City F. The commencement of the airline service will increase tourism in City B.”— Statement by CEO of MGH Airlines. Which of the following can be assumed from the statement of CEO of MGH Airlines? A. MGH Airlines is the first airlines to start air services between City A and City B. B. Unlike MGH Airlines, the airfare for service provided by other airlines between City A and City B is very high. C. City A and City B will now be the most preferred tourist destination of Country K. (1) Only A (2) Only B (3) Only C (4) Only A and B (5) None of these (Indian Bank Specialist Officer SO Exam, 08.03.2020)

    SBI PO EXAMS 1.Statement : Many major auto makers will increase the prices of their vehicles by 7 per cent from next month, mainly to offset higher input and operational costs. Which of the following arguments would weaken the facts given in the statement ? (1) The largest auto maker in the country, Verontee had failed to make any profit when prices for its vehicles were increased by almost 4 per cent (2) Operational and input costs continue to be 85 per cent of the total costs of the vehicles (3) Both input and operational costs were gradually increased by the government to avoid sudden transit losses to the companies (4) At any given time, a price rise of more than 5 per cent definitely leads to substantial decrease in sales of the vehicles (5) Rather than increasing profit, 7 per cent price rise would bring it at the same level which existed before the rise in input/operational costs. (SBI Specialist Officer (Law Officer : MMGS Scale-II) Online Exam, 19.04.2014)

    2. Statement% South African turtles known as Polka turtles, which have dots on their skin, are facing the threat of extinction as they are used as pets by humans. Which of the following steps should be taken by the Government to save these turtles? (1) Poachers should be prevented from poaching these turtles by the forest guards. (2) The term of imprisonment for poaching of Polka turtles should be increased from four years to 10 years. (3) The persons who make these turtles as their pets should be fined heavily. (4) The Government should take step to increase the breeding of Polka turtles. (5) None of these (SBI Management Executive Exam, 19.09.2014)

    3. Statement % The renowned car maker of the Country ‘X’ has observed that its new ‘Zip Zap’ car is very much in demand among the customers. Car maker says that it has provided special type of seat belts in the car. The seat belts are provided even at the rear end of the car. Which of the following is most probable reason for the increase in the sell of Zip Zap cars? (1) The customers are very much aware about their safety (2) Zip Zap cars have special facilities which are not available in the other cars. (3) Zip Zap car has in corporated most sophisticate technology. (4) Zip Zap car is fuel efficient (5) None of these (SBI Management Executive Exam, 19.09.2014)

    4. This question consists of a statement and two conclusions numbered I and II given below it. A conclusion is something which can be directly deduced from the given information in light of the given facts. You have to decide which of the given conclusions logically follows from the given statement and select the appropriate answer.

    BPRE–813

    Statement : At present 32% of our villages receive 24 hours electricity and in the next two years, the state government would strive to add 16% more villages to this list. I. At present 60% villages in the state have no access to electricity at all. II. Even after a couple of years, more than half of the villages lack access to 24 hours electricity. (1) Only II can be concluded from the given information. (2) Only I can be concluded from the given information. (3) Neither I nor II can be concluded from the given information. (4) Both I and II can be concluded from the given information (5) Either I or II can be concluded from the given information. (SBI Associates PO Online Exam. 29.11.2014)

    Directions (5–6) : Each of the following questions consists of a statement followed by two statements numbered I and II given below it. You have to decide which of the given statements numbered I and II weakens or strengthens the statement and mark the appropriate answer. (SBI Associates PO Online Exam. 30.11.2014)

    5. Statement % The world should move towards using bio-fuels in the future as these are more environment friendly as compared to regular fossil- fuels. I.

    While bio-fuels cleaner to burn, the process to produce the fuel, including the necessary machinery leads to very high carbon emission II. As bio-fuels are manufactured from materials such as crop waste, manure and other by products, these, unlike fossil-fuels, are easily renewable. (1) Statement I weakens the information while statements II is a natural statement.

    ARGUMENTATION (2) Both statements I and II weaken the information. (3) Statement I strengthens the information while statement II weakens the statement. (4) Statement I weakens the information while statement II strengthens the statement. (5) Both statements I and II strengthen the information. 6. Statement % Always remain in an air conditioned environment for better health and well-being. I.

    Filters in the air-conditioners lower exposure to allergy creating pollen and other outdoor allergies.

    II. Spending two much time in an air-conditioned environment result in gradual intolerance of body towards natural temperatures. (1) Statement I weakens the information while statements II is a natural statement. (2) Both statements I and II weaken the information. (3) Statement 1 strengthens the information while statement II weakens the statement. (4) Statement I weakens the information while statement II strengthens the statement. (5) Both statements I and II strengthen the information. 7. This question consists of an information and five statements given as five alternatives (1), (2), (3), (4) and (5). You have to decide which of the given alternatives does not strengthen the information. “India’s electronics market will expand rapidly and reach to the tune of $ 75 billion by the end of the year 2017. It was only $ 61.8 billion in 2015” – A report published in the XYZ newspaper in mid 2016. (1) The availability of electronics products in semi-urban and rural areas has been increased in the recent years

    (2) In India, the average labour cost per hour is approximately one–fourth as compared to China (3) Owing to increasing manufacturing costs in China, the manufacturers of electronics goods are migrating to alternative markets (4) Government has announced several incentives for the infrastructure development companies to be implemented in the next 10 years (5) Owing to lower excise duty on different parts of electronics goods in India, cheaper and good quality products can be manufactured in the country. (SBI PO Online Main Exam, 31.07.2016)

    8. Study the following statements carefully and answer the question given below : The Government of the country XYZ claims that presently the country XYZ is the most open economy in the world in term of investments. However, market analysts are of the view that almost all foreign direct investments (FDIs) are related to the service sector, computer software and trade sector only, while FDIs have decreased in the auto, pharma and manufacturing sectors. Which of the following statements strengthen(s) the view of market analysts as compared to the claim of the Government of country XYZ? (A) The Government of the country XYZ has decided to review all the bilateral investment agreements (B) Last year, the Government of the country XYZ prescribed limits for profit margins where FDIs have been made (C) The Government of the country XYZ has decided to review every open trade agreement signed or under consideration (1) Only (A) (2) Only (B) and (C) (3) Only (A) and (C) (4) All (A), (B) and (C) (5) Only (C) (SBI PO Online Main Exam, 31.07.2016)

    BPRE–814

    Directions (9–10) : In making decisions about important questions, it is desirable to be able to distinguish between “strong” arguments and “weak” arguments. “Strong” arguments must be both important and directly related to the question. “Weak” arguments may not be directly related to the question and may be of minor importance or may be related to the trivial aspects of the question. (SBI PO Online Main Exam, 31.07.2016)

    Each question below is followed by three arguments numbered I, II and III. You have to decide which of the arguments is a “strong” argument and which is a “weak” argument. 9. Should the salary and perquisites of public sector undertaking employees be made equivalent to those in the private sector? Arguments : I. Yes, this will help the public sector undertakings to attract and retain competent workforce. II. No, public sector undertakings cannot afford to pay salaries to the level of private sector. III. Yes, otherwise the public sector undertaking will not be able to compete with the private sector organization. (1) None is strong (2) Only III is strong (3) Only I is strong (4) Only II is strong (5) Only I and III are strong 10. Should there be a complete ban on registration of new cars for few months in the big cities in India? Arguments : I. Yes, this will significantly reduce the number of cars on the already overcrowded roads of the big cities in India. II. Yes, the existing car owners will be very happy as they will face less traffic snarls in peak hour. III. No, this is highly discriminatory against those who decide to buy cars now and hence should not be enforced. (1) Only I is strong (2) Only I and III are strong (3) Only III is strong (4) All are strong (5) None of these

    ARGUMENTATION 11. The central bank observed that there is a decline in the growth rate in the 10 core sectors under the governance due to the inappropriate financial policy being carried out by the government. Which of the following argument weakens the above statement? (1) The central bank has the responsibility in country’s economic growth. (2) Financial policy should promote core sectors. (3) Central bank has financial authority to regulate core sectors. (4) The financial policy should be carried out only by government. (5) Appropriate financial policy promotes core sectors. 12. Activities make learning experiences more enjoyable and turn everyday routines into meaningful experiences which encourage children to engage in the creative learning. Which of the following strengthens the above statement? (1) Usually creative children engage in activities. (2) Enjoyable routines encourage children. (3) Meaningful experience gives creative learning. (4) Creative learning is an outcome of activities based learning. (5) Everyday reutilized activities brings creativity.

    RBI GRADE–B/ NABARD GRADE–A OFFICER EXAMS Direction (11) : In making decisions about important questions, it is desirable to be able to distinguish between ‘strong’ arguments and ‘weak’ arguments so far as they relate to the question. ‘Strong’ arguments are those which are both important and directly related to the questions. ‘Weak’ arguments are those which are of minor importance and also may not be directly related to the questions or may be related to a trivial aspect of the question. (RBI Officer Grade ‘B’ Phase-I Exam, 03.08.2014)

    The question below is followed by two arguments numbered I and II. You have to decide which of the arguments is a ‘strong’ argument and which is a ‘weak’ argument. Give answer (1) if only argument I is strong Give answer (2) if only argument II is strong Give answer (3) if either I or II is strong. Give answer (4) if neither I nor II strong. Give answer (5) if both I and II are strong. 1. Should all the power generation and distribution units in the State Y be handed over to the private sector ? Arguments : I. Yes, the State Government are not equipped to handle generation and distribution of electricity efficiently and it is not beneficial too. II. Yes. The private companies handle generation and distribution of electricity efficiently. 2. Two similar buildings were constructed in City X and City Y by ‘Ashiyana’ builders ten years ago. While the building in City X continues to be in good condition, the one in City Y has already started to experience certain structural problems. The residents of the building constructed in City Y have demanded an explanation from the builder regarding lack of uniformity in construction. Which of the following statements would support the builder to some extent ? (1) Cost of construction of building in City X was higer than that of City Y owing labour costs in City X. (2) Minor repair work is always required to be done in all buildings after minimum ten years of construction. (3) Repair work which was done twice earlier in the building in City Y was completed in minimum possible time and within minimum possible budget.

    BPRE–815

    (4) Unlike City X, humidity which is very detrimental to construction material such as iron, cement etc, is very high in City Y owing to its proximity to the sea. (5) Expense of same repair work if done in both the building would be much higher in City X as compared to that in City Y. (Nabard Officer Grade ‘A’ Online Exam. 01.03.2015)

    3. Study the following information carefully and answer the question given below : Company Z and Company F launched similar high end cars last year. However, six months later it was found that the popularity of car manufactured by company Z sky–rocketed while the sales of the one manufactured by company F did not pick up at all. Manager X : “The only reason our car did not succeed was the price of the car. Had our car been priced at a little lower value, the sales of our cars would also have been as good as the one manufactured by Company Z.” Which of the following weakens the statement of the Manager X? (1) Experts have suggested that there is always a segment of population which reviews the car after it has been launched and purchases it only after a certain amount of time has passed after the launch. (2) A few potential buyers had a problem with the fact that the cars manufactured by Company Z had only two airbags while the one manufactured by Company F had six. (3) In the past two years, all cars launched by Company F were of mediocre quality and needed frequent repairs and maintenance thus tarnishing the name of the company to a great extent. (4) The car launched by Company F is available in limited colours, although the owners can request for the colour of their choice. (5) A few other cars launched by Company F several years ago

    ARGUMENTATION had been priced lower than most other cars in the market and they worked very well. (NABARD Assistant Manager Exam, 15.05.2016)

    4. Study the following information carefully and answer the question given below : The commerce department of a college allotted its students to different organisations for completing their two–month internship starting from 05.02.2016. The students were asked to submit their complete internship reports in a specified format along with the original certificates latest by 10.05.2016, failing which they would not be allowed to write their final exams. Following are given cases of 5 students who did not submit their reports by the stipulated deadline. However, one of these was allowed to write his exam. Identify that student. (1) Mehvish had to do her internship under two different project guides. Hence, she found it difficult to adequately allot time to both of them. (2) Sunil Jadhav was absent in the class during which the format of the report was given to the students. (3) Vaibhav Tripathi, who was allotted to an organisation of another city, was given only a provisional certificate by the organisation which was not as per the norms of the college. (4) Akhil Mehta had completed his report before the deadline, but could not submit as it was not written in the specified format. (5) Rachita Saxena was allotted to renowned NGO where the maximum time for internship was of 75 days. (NABARD Assistant Manager Exam, 15.05.2016)

    5. This question consists of information and two statements numbered I and II given below it. You have to decide which of the giv-

    en statements weaken(s) or strengthen(s) the information and decide the appropriate answer. Information : In order to discourage crowd built-up at railway station X, the platform ticket (charged to all such) priced at Rs. 10 should be increased to Rs. 20. I. The price of ticket from X to nearest railway station is Rs. 12. II. On an average, every railway station generates Rs. 24 lacs revenue by charging Rs. 10 for platform ticket while X generates Rs. 28 lacs. (1) Both statement I and statement II weaken the information. (2) Statement I weakens the information while Statement II is a neutral statement. (3) Statement I strengthens the information while statement II weakens the information. (4) Statement I weakens the information while Statement II strengthens the information. (5) Both statement I and statement II strengthen the information. (RBI Officer Grade ‘B’ Phase-I Exam, 04.09.2016 (Shift-I))

    6. This question consists of information and two statements numbered I and II given below it. You have to decide which of the given statements weaken(s) or strengthen(s) the information and decide the appropriate answer. Statement : India has not recorded a win ever in overseas tour to Country A in the last 25 years. With a win in every other overseas tour, India’s record in Country A has been particularly dismal. I. Country Z which has a much stronger team than India, has won most of their matches in the overseas tour to the Country A. II. Due to diplomatic issues, India has toured Country A only twice in the past 25 years as against eight an average to other countries. (1) Statement I is a neutral statement while statement II weakens the information.

    BPRE–816

    (2) Statement I weakens the information while statement II strengthens the information. (3) Both statement I and statement II weaken the information. (4) Both statement I and statement II strengthen the information. (5) Statement I strengthens the information while statement II weakens the information. (RBI Officer Grade ‘B’ Phase-I Exam, 04.09.2016 (Shift-II))

    7. Study the following information carefully and answer the given question : ‘Despite spending huge amount of money, we have not yet been able to find life on other planets. I am personally of the opinion that such research should stop with immediate effect as it is a waste of time and money as no good will ever come out of it. Instead it would be better to use this money to research other elements in space’ – Statement by a Scientist from Space Institute of Country X. Which of the following does not weaken the statement of the scientist of Space Institute of Country X? (1) Although life on other planets has not been discovered yet, such research has widened our knowledge and understanding about other planets and has led to growth and development in science. (2) According to space scientists, if such efforts are continued, the probability of finding life in at least one other planet is much higher as compared to not finding life at all. (3) Various other research projects taken up by the said institute in the past have also failed despite spending huge amount of time as well as money on them. (4) With the amount of time and money that has already been invested in this research, shutting it now would lead to a greater loss than continuing the research.

    ARGUMENTATION (5) While exploring the space and planets, scientists happened to discover many new phenomena which are essential to explain the origin of Earth. (RBI Officer Grade ‘B’ Phase-I Exam, 04.09.2016 (Shift-I))

    8. This question consists of a decision and two Statements numbered I and II given below it. You have to decide which of the given Statements weaken/s or strengthen/s the decision and decide the appropriate answer. Decision : The Government of the State Z decided that no car in the City X shall be allowed to have tinted glasses from next month. I. Most of the crimes reported in the past few months in City X had an involvement of cars with tinted glasses. II. Tinted glasses were responsible for a large number of accidents in City X as they affected visibility during night. (1) Statement I strengthens the decision while Statement II weakens the decision. (2) Both Statement I and Statement II weaken the decision. (3) Both Statement I and Statement II strengthen the decision. (4) Both Statement I and Statement II are neutral statements. (5) Statement I weakens the decision while Statement II strengthens the decision. (RBI Officer Grade ‘B’ Phase-I Exam 17.06.2017)

    9. Study the following information carefully and answer the question given below : School X had been allowing local people to use their basketball court and cricket ground after school hours by charging a fees for the same. This had helped the school generate a significant amount of additional income in the past few years. However, this year the school decided to discontinue the practice.

    Which of the following statements does not strengthen the decision of School X of discontinuing the practice? (1) The number of people using the basketball court and cricket ground of school is slightly decreased last year as other schools in the vicinity also started following the same practice. (2) School X could not win even a single trophy in past few years annual inter-school games competition as the participants did not get adequate time for practice due to access of grounds to local people. (3) Complaints by the nearby residents of School X have increased about the increased disturbance created by the users of the basketball court and cricket ground beyond school hours. (4) The damage caused to the basketball court and cricket ground of School X has increased significantly due to over-use by the outsiders thus not allowing the students to play. (5) The governing board of School X denied the access to school premises in any number by the outsiders (other than parents of students) in this year’s board meeting for security reasons. (RBI Officer Grade ‘B’ Phase-I Exam 17.06.2017)

    10. Study the following information carefully and answer the question given below : These days most of the diseases are waterborne. If we consume only bottled mineral water, it can bring down most of our health issues’- Dr. Patnaik. Which of the following statements weakens the statement of Dr. Patnaik? (A) These days even bottled water is mishandled and reportedly contains a good amount of contaminants.

    BPRE–817

    (B) Around 92% of the people consuming regular/tap water ends up suffering from food poisoning. (C) Bottled mineral water has lesser impurities than regular/tap water. (D) Using unclear cutlery and eating with dirty hands can cause diseases despite having healthy food and water. (1) Both (A) and (D) (2) Both (B) and (C) (3) Only (A) (4) Only (A), (C) and (D) (5) All those given as options (RBI Assistant Manager Online Exam 25.03.2017)

    11. This question consists of a statement and arguments numbered I to IV. You have to pick from the given options the argument/s which is/are strong pertaining to the given statement. Statement : Should the government increase subsidies provided to the farmers in order to bring about improvement in agriculture? Argument (I) : No, since subsidies are provided to only certain crops. It motivates farmers to grow only these crops without rotation for long period leading to increase in pest attack and diminished soil fertility. Argument (II) : Yes, most of the developed countries which provide subsidy in agriculture have been leaders in the production of a certain number of crops. Argument (III) : No, the subsidy being provided to the farmers is already the eighth highest in the world. Argument (IV) : Yes, as farming is a seasonal business it becomes difficult for the farmers to maintain a steady income without subsidies (1) Both I and IV are strong (2) Both II and III are strong (3) Only II is strong (4) Both I and II are strong (5) Both II and IV are strong (RBI Officers in Grade ‘B’ Phase-I Exam. 16.08.2018)

    ARGUMENTATION

    INSURANCE EXAMS Directions (1–5) : These questions are followed by the arguments numbered as I and II. You have to decide and give answer in the following way : (United India Insurance AAO Exam.03.06.2012)

    1.

    2.

    3.

    4.

    5.

    (1) If only argument I is strong. (2) If only argument II is strong. (3) If both arguments I and II are strong. (4) If neither argument I nor argument II is strong. Should Doctors form a union ? Arguments I. No, medical is a noble profession and forming a union is against the ethics of that profession. II. Yes, even a little demand is not paid attention unless you present it as a united group. Should liberalization essential for India ? Arguments I. Yes, to feed huge population of India we require latest technology and in short time this demand can only be fulfilled by liberalization. II. No, liberalization can not fight poverty and unemployment and these problems will increase by this way because advance technology requires less input and manpower. Should film-censorship be abolished ? Arguments I. No, present trend of abolishing established things should be opposed. II. Yes, Art cannot grow and develop in a climate of control. Should sex-education be encouraged in schools ? Arguments I. No, it will disturb the old study pattern of the school. II. Yes, it will give right information at the right time which will help a child to tackle these kinds of problems in future. Should foreign press media enter in India ?

    Arguments I. No, they will give better information which can lead to ruining of India press industry. II. Yes, those which give exact and true news will get better response and in this way the foreign press media help us for reaching to depth of truthness. Directions (6–10) : These questions are followed by the arguments numbered as I and II. You have to decide and give answer in the following way : (United India Insurance AAO Exam.03.06.2012)

    6.

    7.

    8.

    9.

    (1) If only argument I is strong. (2) If only argument II is strong. (3) If both arguments I and II are strong. (4) If neither argument I nor argument II is strong. Should Doctors form a union ? Arguments I. No, medical is a noble profession and forming a union is against the ethics of that profession. II. Yes, even a little demand is not paid attention unless you present it as a united group. Should liberalization essential for India ? Arguments I. Yes, to feed huge population of India we require latest technology and in short time this demand can only be fulfilled by liberalization. II. No, liberalization can not fight poverty and unemployment and these problems will increase by this way because advance technology requires less input and manpower. Should film-censorship be abolished ? Arguments I. No, present trend of abolishing established things should be opposed. II. Yes, Art cannot grow and develop in a climate of control. Should sex-education be encouraged in schools ? Arguments I. No, it will disturb the old study pattern of the school.

    BPRE–818

    II. Yes, it will give right information at the right time which will help a child to tackle these kinds of problems in future. 10. Should foreign press media enter in India ? Arguments I. No, they will give better information which can lead to ruining of India press industry. II. Yes, those which give exact and true news will get better response and in this way the foreign press media help us for reaching to depth of truthness. 11. Read the following information carefully and answer the question which follows. Company X, a Telecom company of country Z, which proposed last week that it would charge its customers for video calls, said that it will not go ahead with the plan. Video call is a facility given free of cost by the company for its existing customers. Which of the following may be a reason for the change in plan by the said company ? (1) The shares of company L, which charges its customers for a similar facility since the beginning, dropped substantially this week. (2) Many people using the services of company X live in rural areas and might not be able to afford an extra charge for certain services. (3) Company X plans to expand its market globally this year. (4) Company X underwent a lot of criticism for the said proposal and also experienced a drop of around 10 per cent customers in just one week. (5) Company Y, a rival to company X setup shop in country Z just two months back and has introduced a number of interesting offers for customers. (NIACL Administrative Officer (AO) Exam, 10.01.2015)

    12. This question consists of an information and two statements numbered I and II given below

    ARGUMENTATION it. You have to decide which of the given statements weaken (s) or strengthen (s) the information and decide the appropriate answer Information: According to a report naval ship ‘AquaEmp’ met with the fatal accident owing to the fact that it was ageing and nearly two decades old. I. The accident took place when the seaman at the artillery section accidently misfired the missile hitting certain crucial engine machinery. II. Average age of all ships, battleships or cargo ships, is 30-35 years, before which these can be operated safely. (1) Both Statement I and Statement II are neutral statements. (2) Both Statement I and Statement II strengthen the information. (3) Both Statement I and Statement II weaken the information. (4) Statement I strengthens the information while Statement II weakens the information. (5) Statement I weakens the information while Statement II strengthens the information. (LIC Assistant Administrative Officer (AAO) Online Exam. 22.03.2015)

    13.

    This question consists of a decision and two statements numbered I and II given below it. You have to decide which of the given statements weakens or strengthens the decision and choose the appropriate answer. Decision : The management department of ASP College has decided that the practice sessions for the upcoming annual fest should not run after 7 pm. Statement I : The props, halo lights, and other materials necessary for the practice have been rented from 10am–9pm till the last day of the fest. Statement II : Recently, cases of accidents on the highway connecting ASP College and the main city have increased during the peak traffic hours. (1) Both statement I and statement II weaken the decision.

    (2) Both statement I and statement II are neutral statements. (3) Statement I weakens the decision while statement II strengthens the decision (4) Statement I strengthens the decision while statement II weakens the decision (5) Both statement I and statement II strengthen the decision. (UIICL A.O. Exam 12.06.2016)

    14. Study the following information carefully and answer the question given below : There has been a threefold increase in the patients suffering from Fugosis — a life threatening communicable disease — in the past few months in Country S. Doctor X — The disease is mainly spreading because of the meat eaters in the country. This can be said because till now the virus causing the disease has been largely found only in meat and meat products. Which of the following strengthens the statement of Doctor X? (1) Fugosis mimics the symptoms of the common cold, as a result by the time the person is hospitalised many around him/her are already affected with the virus. (2) Sate Y of Country S, where around 90% people are vegetarians, has been considerably affected by the virus. (3) Although some people in Country S have restricted their meat consumption, this has not restricted the spread of the virus so far. (4) Out of the total number of persons diagnosed with fugosis, more than 55% of patients are vegetarians but are in contact with meat eaters. (5) Although many samples of packaged and fresh meat were collected for testing from across the country, but not one was found to be contaminated with the virus. (GIC Officer Online Exam 15.05.2017)

    BPRE–819

    15. Study the following information carefully and answer the question given below : ‘Many people had claimed that hygiene level of the food at our two-day annual food fest though good every year many cases of gastroenteritis are reported during and after the festival. It’s two hours for the fest to end this year and not a single visitor has suffered from any food contamination related ailment.’ Statement by the food fest management. Which of the following statements weakens the argument presented by the management of the food fest? (1) It takes a minimum of four to five days for the symptoms of gastroenteritis to manifest. (2) Inspection of all eatables that is carried out by the food and beverage regulation authority every year was not carried out this year. (3) Contaminated eatables and beverages are the main cause of gastroenteritis worldwide. (4) Gastroenteritis not only results from contaminated food but also by using contaminated hands or spoons while eating the food. (5) None of those given as options. (GIC Officer Online Exam 15.05.2017)

    SHORT ANSWERS

    NATIONALISED BANKS & IBPS PO/MT/SO EXAMS 1. (2) 5. (3) 9. (5) 13. (3) 17. (5) 21. (1) 25. (5) 29. (2) 33. (1)

    2. (1) 6. (4) 10. (3) 14. (2) 18. (3) 22. (4) 26. (3) 30. (2) 34. (5)

    3. (5) 7. (5) 11. (2) 15. (1) 19. (3) 23. (5) 27. (2) 31. (3)

    4. (3) 8. (1) 12. (1) 16. (3) 20. (3) 24. (5) 28. (1) 32. (3)

    SBI PO EXAMS 1. (4) 5. (4) 9. (5)

    2. (3) 6. (3) 10. (2)

    3. (1) 7. (4) 11. (3)

    4. (1) 8. (4) 12. (5)

    ARGUMENTATION

    RBI GRADE–B/ NABARD GRADE–A OFFICER EXAMS 1. (4) 5. (1) 9. (5)

    2. (4) 6. (3) 10. (1)

    3. (3) 7. (3) 11. (1)

    4. (3) 8. (3)

    INSURANCE EXAMS 1. (2) 5. (2) 9. (2) 13. (4)

    2. (4) 6. (2) 10. (2) 14. (4)

    3. (4) 7. (4) 11. (4) 15. (1)

    4. (2) 8. (4) 12. (3)

    EXPLANATIONS

    NATIONALISED BANKS & IBPS PO/MT/SO EXAMS 1. (2) Option (2) would strengthen the stance taken by builders. 2. (1) Option (1) would weaken the argument put forward by the committee to the sports ministry. 3. (5) Option (5) would strengthen the argument. 4. (3) Option (3) would strengthen the argument of government. 5. (3) The statement given in option (3) would weaken the argument. 6. (4) Option (4) contradicts the views expressed in the statement. 7. (5) Option (5) would weaken the findings of the study. 8. (1) Option (1) would weaken the argument put forward by the committee to the sports ministry. 9. (5) Option (5) would strengthen the argument. 10. (3) Option (3) would strengthen the argument of government. 11. (2) Obviously, option (2) would weaken the argument of parents. 12. (1) Obviously, option (1) would strengthen the argument made in the statement. 13. (3) Obviously, the most appropriate answer would be Option (3). 14. (2) Obviously statement (B) would weaken the argument given in the question statement. 15. (1) Obviously, Option (1) would strengthen the stance of the Company M.

    16. (3) Obviously, option (3) supports the decision of Pollution Control Board. 17. (5) Obviously, option (5) weakens the given statement that the new medication is responsible for more fatalities. 18. (3) Obviously, option (3) does not weaken the given statement. 19. (3) Obviously, option (3) weakens the statement of HR Manager of Company X. 20. (3) Clearly, both the statements I and II weaken the information. 21. (1) Obviously, both the statements I and II weaken the information. 22. (4) Wild animals in zoos suffer physically and mentally as their complex social, behavioural and physical needs cannot be met in unnatural manmade environment. Thus, Statement I weakens the information and Statement II strengthens the information. 23. (5) Both Statement I and Statement II strengthen the information. 24. (5) Obviously, both the statements I and II strengthen the decision. 25. (5) Obviously both the Statements I and II strengthen the decision. 26. (3) Obviously, Statement I strengthens the decision. Statement II does not highlight major issue. Therefore, it can be taken as a neutral statement. 27. (2) Statement I weakens the information while statement II is a neutral statement. 28. (1) Only argument I seems to be strong. In the recent years in public sector undertakings, the employees who are not performing, are either being fired in form of compulsory retirement or termination. In public sector undertakings, no decision is taken in biased manner. Again, the decision making power is not concentrated in a select few. Therefore, argument II is not strong. 29. (2) Only argument II seems to be strong. The job in Government sector is provided to competent persons after multi-phased selection procesd. 30. (2) Only argument II seems to be strong. Argument I does not make suitable reason. It is based on the exceptional occurrences.

    BPRE–820

    31. (3) Obviously, option (3) weakens the given argument. 32. (3) Obviously, option (3) provides the best evidence that the illegal migrants are threat to social order. 33. (1) Obviously, Statement I strengthens the information and Statement II is a neutral statement. 34. (5) The commencement of the airline service will increase tourism in the City B. Therefore, statement C is invalid. It is mentioned that MGH Airlines would commence service at affordable price. This information does not support statements A and B.

    SBI PO EXAMS 1. (4) Obviously, option (4) would weaken the facts given in the statement. If sales will come down, how profit will increase. 2. (3) From the statement it is clear that Polka turtles are being caught for rearing as pets. But these turtles may die for lack of adequate habitat. So, the main problem is the use of Polka turtles as pets. Hence, option (3) seems to be most plausible step. 3. (1) In any advertisement the point which is highlighted is liked by the people. The statement discusses the safety aspect of the car. So, it may be said that customers are very much aware about their safety. 4. (1) The statement affirms that 32 per cent villages have 24 hours electricity. It does not imply that other villages have no access to electricity at all. Conclusion II may be true. 5. (4) Option (4) is true. 6. (3) Option (3) is true. 7. (4) Options (1), (2), (3) and (5) clearly strengthen the information directly. Option (4) mentions long term policy and its benefit can be realised in the due course while the information predicts the increase in the market of electronics products in the next year. Therefore, option (4) does not strengthen the information directly. 8. (4) Obviously all the three statements (A), (B) and (C) strengthen the view of market analysts.

    ARGUMENTATION 9. (5) Only Arguments I and III are strong. It is not mentioned that why public sector undertakings cannot afford to pay salaries to the level of private sector. Therefore, Argument II is not strong. 10. (2) Clearly, only Arguments I and III are strong. A step should not be taken to favour few people. Therefore, Argument II is not strong. 11. (3) Core sectors should be regulated only by government. Central bank can promote it in financial matters but does not have financial authority. 12. (5) Reutilized activities do not bring creativity. Experience based learning routine brings creativity.

    RBI GRADE–B/ NABARD GRADE–A OFFICER EXAMS 1. (4) Neither argument I nor argument II is strong. None of the arguments cites any fact. 2. (4) Obviously, the option (4) supports the builder. 3. (3) Obviously, option (3) weakens the statement of Manager X. 4. (3) Obviously, Vaibhav Tripathi was allowed to write his exam. He lacks original certificate and it is not his fault. 5. (1) Clearly, both the statements weaken the information. 6. (3) Both the statements weaken the given information. The team of Country Z is much stronger than India and hence it has won most of their matches in the overseas tour to Country A. India has toured Country A only twice in the past 25 years. Therefore, the given information can be weakened by the statement II. 7. (3) Obviously, option (3) does not weaken the statement of the scientist. 8. (3) Both the Statement I and Statement II strengthen the decision. 9. (5) Obviously, option (5) does not strengthen the decision of School X of discontinuing the practice by local people. Local people

    were allowed to use playgrounds after school hours. Therefore, safety and security of the students were not the factors for taking the decision. 10. (1) Obviously, statements (A) and (D) weaken the views expressed by Dr. Patnaik. 11. (1) Argument II is based on an example. We know that citing example is a bad argumentation. Therefore, we can eliminate options (2), (3), (4) and (5). Arguments I and IV seem to be strong.

    INSURANCE EXAMS 1. (2) Only argument II holds strong. A union provides a common platform for the members to voice their genuine demands. There is no doubt that medical is a noble profession, but how forming a union is against the ethics of the profession. 2. (4) Neither argument I nor argument II is strong. The use of tern ‘only’ in the first argument makes it invalid. The second argument is based on an assumption. 3. (4) Argument I seems to be vague. It does not make any substantial point. Therefore, argument I does not hold strong. Argument II does not present a logical reason. 4. (2) Change is the rule of nature. Therefore, argument I does not provide convincing reason. Sex education in schools can help students remove their misconceptions and doubts at a stage, when they would otherwise hesitate to discuss the same with others. Also, sex forms an integral part of the future life of students. Therefore, argument II holds strong. 5. (2) Only argument II seems to be strong. Argument I assumes that Indian press does not privide better information. This fact may not be considered as right. The entry of foreign press media will make competitive environment and the quality of news coverage will be improved. 6. (2) Only argument II holds strong. A union provides a common platform for the members to voice their genuine demands. There is no doubt that medical is

    BPRE–821

    a noble profession, but how forming a union is against the ethics of the profession. 7. (4) Neither argument I nor argument II is strong. The use of tern ‘only’ in the first argument makes it invalid. The second argument is based on an assumption. 8. (4) Argument I seems to be vague. It does not make any substantial point. Therefore, argument I does not hold strong. Argument II does not present a logical reason. 9. (2) Change is the rule of nature. Therefore, argument I does not provide convincing reason. Sex education in schools can help students remove their misconceptions and doubts at a stage, when they would otherwise hesitate to discuss the same with others. Also, sex forms an integral part of the future life of students. Therefore, argument II holds strong. 10. (2) Only argument II seems to be strong. Argument I assumes that Indian press does not privide better information. This fact may not be considered as right. The entry of foreign press media will make competitive environment and the quality of news coverage will be improved. 11. (4) An example cannot be taken as a support to an argument. Therefore, Option (1) may be ruled out. Option (4) seems to be most appropriate reason. 12. (3) Clearly, both the statements weaken the given information. Statement I states that the accident took place when the seaman accidently misfired the missile which hit certain crucial engine machinery. According to statement II, average age of all ships is 30-35 years and naval ship ‘AquaEmp’ was only two decades old. Therefore, the cause of accident was not its age. 13. (4) Obviously, statement I strengthens the decision and statement II weakens the decision. 14. (4) Clearly, option (4) strengthens the statement of Doctor X. 15. (1) Obviously, option (1) weakens the argument presented by the management of the food fest. ❐❐❐

    ARGUMENTATION

    MODEL EXERCISES Directions (1-4) : Each questions consists of five statements A, B, C, D and E followed by four options consisting of three statements put together in a specific order. Choose the option which indicates a valid argument, ie, where the third statement indicated in the answer option is a conclusion drawn from the preceeding two statements. . 1. (A) Oranges are not sweet. (B) Some oranges are sweet. (C) All sweets are tasty. (D) Some oranges are not tasty.

    Directions (5-8) : Each of the questions given below is followed by two arguments numbered I and II You have to decide which of the arguments is a ‘strong’ argument and which is a ‘weak’ argument. Give your answer as (1) If only argument I is strong (2) If only argument II is strong : (3) If either I or II is strong . . (4) If neither I nor II is strong (5) None of these 5. Statement : Should non-vegetarian food be totally banned in our country?

    (E) No orange is tasty. ,

    Arguments :

    (1) CEA

    (2) BDC

    (3) CBD

    (4) EAC

    I. Yes, it is expensive and therefore beyond the means of most people in our country.

    (5) None of these 2. (A) MBAs are in great demand. (B) Sunil and Shobha are in great demand.

    II. No. nothing should be banned in a democratic country. 6. statement : Should India go in for computerisation in industry ?

    (C) Sunil is in great demand.

    Arguments :

    (D) Shobha is in great demand

    I. No, computerisation demands a lot of money. We should not waste money on it.

    (E) Sunil and Shobha are MBAs. (1) ABE

    (2) ECD

    (3) AEB

    (4) EBA

    (5) None of these 3. (A) Some towns in the world are polluted. (B) All polluted towns should be destroyed.

    II. Yes, when advanced countries are introducing computers in various areas, how can we afford to lag behind. 7. Statement : Should personal tax be abolished in India ? Arguments :

    (C) Town Lambusa should be destroyed.

    I. Yes, it will motivate people to earn more.

    (D) Town Lambusa is polluted.

    II. No, individuals must learn to share their wealth with other people.

    (E) Some towns in the world should be destroyed. (1) BDE

    (2) BAE

    (3) ADE

    (4) CDB

    (5) None of these 4. (A) All heroines are pretty. (B) Some heroines are popular. , (C) Sanjana is pretty. (D) Sanjana is a popular heroine. (E) Some popular girls are pretty. (1) ACD

    (2) ABE

    (3) DCA

    (4) EDC

    (5) None of these

    8. Statement : Should judiciary be independent of the executive ? Arguments : I. Yes, this would help to curb the unlawful activities of the executive. II. No, the executive would not be able to take bold measures. 9. Some people have questioned the judges’ objectivity in cases of sex discrimination against women. But the record shows that in sixty per cent of such cases, the judges

    BPRE–822

    have decided in favour of the women. This record demonstrates that the judges have not discriminated against women in cases of sex discrimination against women. The argument above is flawed in that it ignores the possibility that (1) many judges find it difficult to be objective in cases of sex discrimination against women (2) a large number of the judges’ causes arose out of allegations of sex discrimination against women (3) the judges are biased towards women defendants or plaintiffs in cases that do not involve sex discrimination (4) the majority of the causes of sex discrimination against women that have reached the judges’ courts have been appealed from a lower court (5) None of these Directions (10-13) : Each of these has a question followed by two arguments numbered I and II. Decide which of the arguments is ‘strong’ and which is “weak’. Mark answer as (1) if only argument II is strong (2) if only argument I is strong (3) if either argument I or II is strong (4) if neither argument I nor II is strong (5) None of these 10. Should all the professional colleges in India be encouraged to run their own courses without affiliation to any university? Arguments : I. Yes, this is the only way to create more opportunities for those who seek professional training. II. No, this will dilute the quality of professional training as all such colleges may not be equipped to conduct such courses. 11. Should all those who have come in contact with the patients of infectious respiratory disease be quarantined in their houses?

    ARGUMENTATION Arguments : I. No, nobody should be quarantined unless they are tested and found to be infected by the virus causing the disease. II. Yes, this is the only way to control the spread of the dreaded disease. 12. Should India support all the international policies of United States of America? Arguments : I. No, many other powerful countries do not support the same. II. Yes, this is the only way to gain access to US development funds. 13. Should there be a complete ban on advertising of tobacco products in all media? Arguments : I. Yes, this is the only way to save people from suffering from cancer. II. No, this will adversely affect the sale of tobacco products. Directions (14-17) : Each of these has a question followed by two arguments I and II. Decide which of the arguments is a ‘strong’ argument and which is a ‘Weak’ argument. Mark answer as (1) if only argument I is strong (2) if only argument II is strong (3) if neither argument I nor II is strong ; (4) if both arguments I and II are ; strong (5) None of these 14. Should judiciary be independent of the executive? Arguments : I. Yes, this would help to curb the unlawful activities of the executive. II. No, the executive would not be able to take bold measures. 15. Should open book systems be introduced in examinations? Arguments : I. Yes, because it will avoid mass copying. II. No, because then all students will get 100% marks.

    16. Should religion be taught in our schools? Arguments : I. Yes, do the parents not wish to develop their wards into mature individuals. II. No, how can one dream of such a step when we want the young generation to fulfill its role. 17. Should family planning be made compulsory in India? Arguments : I. Yes, looking at the miserable conditions in India, there is no other go. II. No, in India there are people of various religions and family planning is against the tenets of some of religions. 18. Study the following information carefully to answer the given question : ‘Students of Country X go abroad for higher studies despite having some excellent universities in their own country. In fact, there is no benefit but just the hype of studying abroad that encourages students to do so’ — Statement by the General Secretary of students’ association of a college in Country X. Which of the following strengthens the statement of the General Secretary that students do not benefit from studying abroad? (1) Getting some admission in some good universities of Country X is next to impossible as more than 55% of seats are reserved seats and most students compete for the seats which are not reserved. (2) Statistics suggest that more than 55% students of Country X, studying abroad either accept mediocre jobs or return to Country X after completing their course due to lack of job opportunities. (3) Country X provides a number of scholarships for students in almost all the fields in order to ensure that all students get equal opportunity to finish their higher education.

    BPRE–823

    (4) The fees charged by a few universities of Country X is much less than the fees charged by many universities abroad. (5) Some good universities of Country X conduct entrance exams which are more difficult to clear than the entrance exams conducted by universities around the world. 19. Study the following information carefully and answer the question given below : Bus no. 218 is the only bus which plies on the only route between localities A and G of city Haroya. There are three huge factories between these areas. Though the bus has always been very punctual and well maintained, last week, it was decided to discontinue the bus for certain reasons. Which of the following cannot be reason behind discontinuing Bus no. 218? (1) Soon AC buses will start plying between localities A and G, whose fare will be just 2% more than that of Bus no. 218. It is anticipated that Bus no. 218 will not make much profit thereafter. (2) All the given options can be possible reasons for discontinuing the bus. (3) It has been decided to increase the number of e-rickshaws plying between areas A and G by 40% hence making them more easily and frequently available to the customers as compared to the bus. Also, as a result, the additional pollution caused by buses in these areas can brought down to a great extent. (4) Under the recently completed metro rails project of Haroya, there are more frequent trains between localities A and G as compared to the frequency of Bus no. 218. The metro fare is also lower than that of the bus. (5) The number of chain snatching and pick-pocketing cases near locality G of Haroya have drastically increased. As a result, Bus no. 218 has considerably lesser passengers now.

    ARGUMENTATION 20. Study the following information carefully to answer the given question. “The attrition rate of our company has come down significantly in the past one year as compared to last three years” – Statement by HR head of Company X. Which of the following may be a reason for the reduced attrition rate of employees in Company X? (A) The CEO along with the general board of Company X revised its promotion policies this year and had reduced the tenure of promotion from five years to three years. (B) The governing board has sanctioned the annual increment to be increased by 34% for all employees. (C) The number of working days for employees have been reduced from six to five. (D) The monetary benefits are reduced to Rs. 40,000 annually from an amount of Rs. 70,000 per employee. (1) Only A, B and C (2) Only A, C and D (3) Only A and B (5) Only C and D

    SHORT ANSWERS 1. (1)

    2. (3)

    3. (2)

    5. (4)

    6. (2)

    7. (2)

    8. (1)

    9. (1)

    10. (1)

    11. (2)

    12. (4)

    4. (2)

    13. (4)

    14. (1)

    15. (3)

    16. (3)

    17. (2)

    18. (2)

    19. (5)

    20. (1)

    EXPLANATIONS 1. (1) From this figure, we conclude that no oranges are sweet. Hence, CEA is a logical sequence.

    S

    O

    3. (2) BAE is a logical sequence because from statements B and A we get “Some towns in the world should be destroyed.” 4. (2) All heroines are pretty and some heroines are popular imply that some popular heroines are pretty. 5. (4) Since, non-vegetarian food is expensive and beyond the means of most people in the country, it is not necessary that it should be banned completely. 6. (2) Advanced countries are introducing computers in various areas. It means computeri- sation is productive. Hence, argument II is strong. 7. (2) Fact that the income of people is increasing in India implies that people are encouraged to earn more inspite of personal tax. So, I is weak argument but II is strong as it is worthwhile ideal. 8. (1) Judiciary act on a check on the activities of the executive. This requires that the judiciary is independent of the executive. Therefore, argument I is strong but argument II is not strong. 9. (1) The argument in the passage ignores the possibility that many judges find it difficult to be objective in cases of sex determination against women.

    (4) Only B and D

    T

    2. (3) Clearly, AEB gives a logical sequence.

    10. (1) Argument I is not strong because this is not the only way to create more opportunities for those who seek professional training. Argument II is strong because running own courses without affiliation to any university, will dilute the quality of professional training. 11. (2) Argument I is strong because nobody should be quarantined unless they are tested and found to be infected by the virus causing the disease. Argument II is not strong because quaranting in home is not the only way to control the spread of the dreaded disease. 12. (4) Neither argument I nor argument II is strong because India should not support all the inter-

    national policies of United States of America only on the basis that other powerful countries are supporting or not. This is not the only way to gain access to US development funds. There are many other ways. 13. (4) Neither argument I nor argument II is strong because all the patients suffering from cancer are not due to tobacco products. Sale is not a factor in case of ban on advertising of tobacco products. 14. (1) Only argument I is strong because when judiciary will be independent of executive, then it can curb the unlawful activities of the executive. 15. (3) Neither argument I nor II is strong because open book system is itself a mass copying and all the students cannot get 100% marks. 16. (3) Neither argument I nor II is strong because every parents wish to develop their wards into nature individuals and teaching religion is not a wrong step for young generation to fulfill its role. 17. (2) Argument I is not strong because making family planning compulsory in the only way of parent miserable conditions. Argument II is strong because family planning is against the tenents of some religions. 18. (2) Option (2) strengthens the statement of the General Secretary. 19. (5) Obviously, option (5) cannot be reason behind discontinuing Bus No. 218 as it presents law and order problem. 20. (1) ‘Attrition’ refers to a gradual reduction in the number of people who work for an organisation that is achieved by not replacing those who leave. Attrition is the unpredictable and uncontrollable, but normal, reduction of work force due to resignations, retirement, sickness or death. Obviously, statements A, B and C may be reasons for the reduced attrition rate of employees in Company X. ❑❑❑

    BPRE–824

    ASSUMPTION

    14

    ASSUMPTION

    QUESTIONS FROM 1999 TO 2010 ARE AVAILABLE ONLINE NATIONALISED BANKS & IBPS PO/MT/SO EXAMS Directions (1-3) : In each question below is/are given statement(s) followed by two assumptions/inferences numbered I and II. An assumption is something supposed or taken for granted. And an inference can be inferred from the given information. You have to consider the statement(s) and the following assumptions/inferences and decide which of the assumptions/ inferences is implicit in the statement (s). (Indian Bank PO Exam. 02.01.2011 (Ist sitting)

    Give answer (1) if only I is implicit. Give answer (2) if only II is implicit. Give answer (3) if either I or II is implicit. Give answer (4) if neither I nor II is implicit. Give answer (5) if both I and II are implicit. 1. Statement : “Sometimes a person acting out of conviction, pursuing a dream represents the spirit of an entire generation”. I. One gets conviction out of one’s action. II. Expectations of different generations are not the same. 2. Statements : “Take a recruitment system, for example. The company would make an offer to somebody it had rejected three months ago! There was no recruitment database that kept record of that”. I. It is possible to devise a system which would indicate those applicants who were rejected earlier. II. This company considers three months time too short a period to reconsider recruitment decision about a candidate. 3. Statements : “Everyone does not need to build a 100 crore or 1000

    crore company. Small vs big vs superbig is a choice that an entrepreneur makes herself /himself depending upon the ambitions, values and what she/he likes doing”. I. The figures are for the turnover of the company. II. There is hardly any distinction between a small or a big company. Directions (4–8) : In each question below is given a statement followed by two assumptions numbered I and II. An assumption is something supposed or taken for granted. You have to consider the statement and the following assumption and decide which of the assumptions is implicit in the statement. (Corporation Bank PO Exam. 16.01.2011)

    Give answer (1) if only assumption I is implicit. Give answer (2) if only assumption II is implicit. Give answer (3) if either assumption I or assumption II is implicit. Give answer (4) if neither assumption I nor assumption II is implicit. Give answer (5) if both assumptions I and II are implicit. 4. Statement : If parking space is not available in office, park your vehicles in the mall and walk to the office. Assumptions : I. The mall is at a walkable distance from the office. II. The office does not allow visitors’ vehicles in its premises. 5. Statement : Farmers must immediately switch over to organic fertilizers from chemical fertilizers for better yield. Assumptions : I. All the farmers use only chemical fertilizers.

    BPRE–825

    II. Organic fertilizers are readily available to the farmers. 6. Statement : An advertisement by bank X ‘Our interest rates for education loans are lower than any other bank’. Assumptions : I. Some other banks also provide education loans. II. Interest rates charged on education loans are different for different banks. 7. Statement : For any kind of problem with your mobile phone, contact our help desk immediately. Assumptions : I. Help desk has a solution to all kinds of problems related to mobile phones or will guide accordingly. II. Unless the problem is reported immediately, it cannot be solved. 8. Statement : Use our medicine to fight the problem of obesity. Assumptions : I. Other slimming medicines available in the market do not reduce weight. II. Obesity cannot be controlled without medicines. Directions (9–13) : In each question below is given a statement followed by two assumptions/inferences numbered I and II. An assumption is something supposed or taken for granted and an inference is something which can be directly inferred from the given facts. You have to consider the statement and the following assumptions/ inferences and decide which of those is/are implicit in the statement. (UCO Bank PO Exam. 30.01.2011)

    Give answer plicit. Give answer plicit. Give answer is implicit. Give answer II is implicit. Give answer are implicit.

    (1) if only I is im(2) if only II is im(3) if either I or II (4) if neither I nor (5) if both I and II

    ASSUMPTION 9. Statement : Let the secretary prepare PowerPoint presentation for Ashish. Assumptions : I. Ashish is incapable of preparing a PowerPoint presentation himself. II. The secretary is well versed with operations of PowerPoint. 10. Statement : Cars found parked in VIP parking area would be towed away. Assumptions : I. Some people have different parking privileges than others. II. Parking spaces for VIPs and general public are separate. 11. Statement : A flyover would ease the traffic on Motilal Nagar road. Assumptions : I. The Motilal Nagar road usually has a lot of traffic. II. Currently also there is a flyover on Motilal Nagar road. 12. Statement : The newly launched book is making India read like never before. Assumptions : I. India did not have avid readers before the launch of this book. II. This book has not been as successful as other books by the same author. 13. Statement : To avoid standing in long queues, book your tickets on the internet. Assumptions : I. Most people prefer to book tickets on the internet these days. II. Apart from the internet, tickets are available elsewhere as well. Directions (14–18) : In each question below is given a statement followed by two assumptions/inferences numbered I and II. An assumption is something supposed or taken for granted and an inference is something which can be directly inferred from the given facts. You have to consider the statement and the following assumptions/inferences and decide which of those is/are implicit in the statement. (Bank Of Baroda PO Exam. 13.03.2011)

    Give answer (1) if only I is implicit. Give answer (2) if only II is implicit. Give answer (3) if either I or II is implicit.

    Give answer (4) if neither I nor II is implicit. Give answer (5) if both I and II are implicit. 14. Statement : ‘Do not enter – avoid the risk of getting infected with the ABC disease’ – written outside the quarantine ward no. 2 (meant only for ABC disease) of a hospital’ Assumptions : I. Disease ‘ABC’ is contagious. II. All the patients in ward no. 2 suffer from disease ABC. 15. Statement : In city Z, people prefer to buy Car X instead of Car Y as Car X has German technology which is very advanced. Assumptions : I. Cars with German technology are perceived to be better than other cars in city Z. II. Had German technology been present in Car Y also, its sales would have crossed car X’s sales. 16. Statement : Railway does not provide concession to any one for travelling to certain holiday destinations. Assumptions : I. Railway services are available for travelling to these holiday destinations. II. Railways provides concession to certain persons for travelling to places other than these holiday destinations. 17. Statement : “Travellers with a ticket for the second class if found travelling in the first class compartments would be penalized”– Notice in the compartments of a train. Assumptions : I. Travellers with a ticket for the first class are also not allowed to travel in the second class compartments. II. Inspections are carried out in the train to check the tickets. 18. Statement : The prices of petrol and diesel have remained unchanged only in Nigeria since the past three years. Assumptions : I. Petrol and diesel prices have changed elsewhere in the world during these three years. II. Before this three years period, petrol and diesel were

    BPRE–826

    available at a price different from the present rates. Directions (19–21) : Read the following information carefully and answer the questions which follow: Fortunately, more and more countries are shifting their focus away from industrial development to control of climate change these days. (Allahabad Bank PO Exam. 17.04.2011)

    (A) The countries which focus more on controlling climate change than industrial development are only the richer ones which can afford to concentrate on areas other than industrial development. (B) Many countries had once prioritized industrial development which proved to be harmful to the environment in the longer run. (C) Some experts are of the view that climate change is not as alarming an issue as it is made to be because it is a natural phenomenon and has been occurring regularly throughout the history of earth. (D) If climate change continues at the present rate, it would bring in large scale destruction to human habitation in a very short time. (E) Industrial development is one of the biggest but definitely not the only reason behind global warming. 19. Which of the statements numbered (A), (B), (C), (D) and (E) can be an assumed / inferred from the facts/ information given in the statement ? (An assumption is something supposed or taken for granted and an inference is something which is not directly stated but can be inferred from the given facts.) (1) E (2) C (3) A (4) B (5) Either D or A 20. Which of the following can be inferred from statement (E) if it is considered to be true with regard to the given information ? (1) Nations also need-to focus on sources other than those generated due to industrial development (2) Other sources of pollution have more adverse effects as compared to those generated due to industrial development

    ASSUMPTION (3) Unlike older times, industrial development has ceased to be a reason behind global warming these days. (4) If industrial development stops, global warming would automatically come to an end. (5) If sources other than industrial development are identified and controlled, global warming will end completely. 21. Which of the statements numbered (A), (C), (D) and (E) mentioned above represents a reason behind curtailing industrial development by some of the countries? (1) Either C or E (2) D (3) C (4) Either A or C (5) A 22. The Government has appealed to all citizens to use potable water judiciously as there is an acute shortage in supply. Excessive use may lead to huge scarcity in future months. Which of the assumptions is implicit in the above statement? (An assumption is something supposed or taken for granted) (1) People may ignore the appeal and continue using water as per their consideration. (2) Government may be able to tap those who do not respond to the appeal (3) Government may be able to put in place alternate sources of water in the event of a crisis situation. (4) Large number of people may positively respond to the Government’s appeal and help tide over the crisis. (5) Only poor are going to suffer from this shortage of water supply (IBPS Bank PO/MT CWE Exam. 18.09.2011)

    23. Statement : The constable has been recommended for a suitable reward by his superior in recognition of his sincere duty and busting of several gangs of criminals actively involved in the loot and incidents of pick-pocketing. Which of the following can be a possible assumption of the above statement ? (1) The superior is certain that the recommendation would be denied.

    (2) The number of criminals apprehended by this particular constable was exceptionally high. (3) The constable desires to be monetarily compensated for his efforts. (4) The superior wants to set an example for his other juniors by recommending the reward. (5) Rewards recognising the sincerity and accomplishments of policemen are given. (IBPS Specialist Officer CWE Exam.17.03.2013)

    24. Statements : The United Statesbased FMCG company has developed a new deodorant. The company selected a media person couple for its promotion and marketing. They marketed it through television, radio, print, hoarding and social media. Which of the assumptions is implicit in the above statements ? (An assumption is something supposed or taken for granted) (1) People do not pay heed to the advertisements and buy such goods which are available easily (2) Marketing through electronic media has not proved to be very effective (3) People want to see film stars and famous sports personality in the advertisement of an FMCG product (4) Some people may tend to buy the deodorant of the United States-based FMCG company (5) None of these (IBPS Bank PO/MT CWE-III, 26.10.2013)

    25. Statements : The ‘X’ Finance company has launched a new tractor loan scheme. It has brought prosperity among the farmers. Which of the following assumptions is implicit in the above statements? (An assumption is something supposed or taken for granted) (1) The prosperity of farmers solely depends upon the tractor loan scheme. (2) Tractor is an important input for farmers to enhance the farm activities

    BPRE–827

    (3) Earlier there was no tractor loan scheme for farmers (4) Farmers do not prefer to take loans for buying tractors. (5) None of these (IBPS Bank PO/MT CWE-III, 26.10.2013)

    26. Statement : The Government has decided to construct an eight-lane super highway across the state to facilitate fast movement of vehicles. Which of the following can be an assumption which is implicit in the above statement? (1) The Government has adequate resources to construct the proposed super highway (2) The people of the state may protest against the Government’s decision as their farm land will be taken over the Government for constructing the highway. (3) The Government may find it difficult to enrol a suitable contractor for constructing the highway. (4) There is no other highway which can be used for transportation of goods across the state (5) None of these (Corporation Bank SO (Marketing) Exam, 22.02.2014)

    27. Statement : The constable has been recommended for a suitable reward by his superior in recognition of his sincere duty and busting of several gangs of criminals actively involved in the loot and incidents of pick-pocketing. Which of the following can be a possible assumption of the above statement ? (1) The superior is certain that the recommendation would be denied. (2) The number of criminals apprehended by this particular constable was exceptionally high. (3) The constable desires to be monetarily compensated for his efforts. (4) The superior wants to set an example for his other juniors by recommending the reward. (5) Rewards recognising the sincerity and accomplishments of policemen are given. (Corporation Bank SO (Marketing) Exam, 22.02.2014)

    ASSUMPTION 28. Statement : The Ministry of Civil Aviation has bought 35 Aircraft. The decision to buy aircraft was found to be improper. Which of the following may be an assumption that is implicit in the statement ? (1) The Ministry of Civil Aviation is not authorised to buy aircraft. (2) The Ministry of Civil Aviation can buy aircraft. (3) The Ministry of Civil Aviation is facing financial crisis. (4) It was not necessary to replace the old fleet of aircraft (5) The Ministry of Finance did not approve the required amount. (BOB Manipal School of Banking Officer Online Exam, 14.08.2014)

    29. Statement : The Company ‘A’ builds roads and undertakes infrastructure projects. The Company ‘A’ has recently got new infrastructure project. Which of the following may be an assumption that is implicit in the statement ? (1) No company other than Company ‘A’ builds roads and undertakes infrastructure projects. (2) Only Company ‘A’ builds roads and undertakes infrastructure projects (3) The Company ‘A’ has exper tise to build roads and undertake infrastructure projects (4) The Company ‘A’ does not follows the time limit of any project (5) The roads built by the Company ‘A’ are in very poor conditions. (BOB Manipal School of Banking Officer Online Exam, 14.08.2014)

    30. Mosquito bites can be more than just annoying and itchy. They can make you really sick. Protect yourself and your family from mosquitoes during travelling. Because dangerous diseases like Zika, dengue and chikunguniya are spread by mosquitoes. Which of the following can be assumed from the given statement? (1) Mosquitoes are more dangerous than any other animals in the world.

    (2) There is no treatment of diseases like Zika, dengue and chikungunya that are spread by mosquitoes. (3) Forty per cent of the world’s population lives in an area at risk for dengue spread by mosquito and an estimated 390 million people per year are infected with the viruses. (4) There are more chances to get infected from mosquitoes while travelling rather than from staying at home. (5) All travellers are suffering from diseases like Zika, dengue and chikungunya spread by mosquitoes. (IBPS RRBs Officer CWE (Main Exam) 05.11.2017)

    Which of the following can be a right Assumption for the statement? (1) Magstrip ATM/Debit cards have become unsafe for the account holders (2) All the bank-customer information of the user is embedded in chip system of EMV (3) EMV card system probably increases the security of the account holder (4) Master card or visa card is a secured means of payment through plastic money (5) All of the above (IBPS Bank PO/MT Main Exam 26.11.2017)

    33.

    31. In this question is given a statement followed by two assumptions numbered I and II. An assumption is somethingsupposed or taken for granted but not directly given in the statement. You have to consider the statement and the following assumptions and decide which of those is/are implicit in the statement. Statement : These days all flight and train tickets are produced on mobile phone. Therefore, it is safe to predict that in a short time, taking a print out for tickets will completely stop. I. Every passenger in the time to come will be able to afford a mobile phone and travel with it. II. Tickets produced on a mobile phone will have all the essential details as present in a print-out. (1) Only II is implicit (2) Both I and II are implicit (3) Only I is implicit (4) Either I or II is implicit (5) Neither I nor II is implicit (Canara Bank PO Exam 04.03.2018)

    32. Cards with only a magnetic strip at the back, called magstrip cards have been in circulation for a long time now. The EMV chip, named so after its creators Europay, MasterCard and Visa, was introduced to plastic money as an additional secure means of payment. It carries microprocessor data embedded into it which has all important information about the user. These EMV chip cards shield users from ATM frauds like skimming.

    BPRE–828

    In this question, a statement followed by two assumptions numbered I and II has been given. An assumption is something supposed or taken for granted. You are required to consider the statement and the following assumptions and decide which of the assumption is implicit in the statement. Statement: The proposed Constitutional 124th Amendment Bill 2019 has been given assent by the President of India. The Bill now is 103rd Constitutional Amendment Act, 2019. The Government of India has also issued a gazette notification on the assent and the full details are published in the official website. Assumption I : Usually, the Gazette notifications are published in official websites. Assumption II : There is few amendment bills proposed have not yet been enacted. (1) Only assumption I is implicit (2) Only assumption II is implicit (3) Either assumption I or assumption II is implicit (4) Neither assumption I nor assumption II is implicit (5) Both assumptions I and II are implicit

    (IBPS Bank PO/MT CWE (Main Exam) 18.11.2018)

    34.

    Choose the best ASSUMPTION for the statement from the given alternatives. An assumption is something supposed or taken for granted. You are required to consider the statement and the following assumptions and decide which of

    ASSUMPTION the assumption is implicit in the statement. Statement: The Supreme Court has directed the Election Commission to increase random matching of slips with Electronic Voting Machines. But the Election Commission of India says that, the increase in random checks of Voter Verifiable Paper Audit Trail (VVPAT) machines is likely to delay the declaration of official results. (1) The Court observes that the fair election is important than the process of election (2) The Election Commission reinforces the system of election procedures (3) Supreme Court knows that the random audit will delay the result (4) 1 and 2 (5) 2 and 3 (IBPS Bank PO/MT CWE (Main Exam) 18.11.2018)

    Directions (35–36): Consider the following statements in each question and select appropriate ASSUMPTIONS from the choices given below the statement. An assumption is a fact or truth which is taken for granted from the given statement. (IBPS RRBs Officer Scale-I CWE Main Exam, 13.10.2019)

    35. An advertisement in a newspaper – “Wanted married women between 18 and 21 age, able to speak English fluently and type the same in computer, to be hired as typists through interview” A. The women must be able communicate orally apart from typing. B. There are already few women working in the firm. C. More than one woman is required by the firm. D. A 20 year woman might be rejected during interview. (1) Only A and B (2) Only B and C (3) Only A and C (4) Only B and D (5) Only A and D 36. Few socialists in a campaign for healthy children encourage playing in playgrounds rather than video games at home. The scientists across the world also have been warning the negative health impact of modern technology.

    A. Modern technology is spoiling the children’s health. B. Modern technology do not allow children to play in play- grounds. C. The socialists of the campaign know the warnings of the scientists D. There are other socialists who might focus on other aspects favourable to children. (1) Only A and B (2) Only B and C (3) Only A and C (4) Only B and D (5) Only C and D

    SBI PO EXAMS Directions (1–5) : In each of the questions below is given a statement followed by a question. Read the statement carefully and answer the question that follows : (SBI Management Executive Exam, 23.02.2014)

    1. Statement : A combination of factors have seen the sales volume of the top 25 listed real estate companies almost halve to about 11.8 million square feet in the quarter ended September 2013. It was 20.73 million square feet in the year ago period. Which of the following may be a probable reason for the slump in the real estate business ? (1) Housing is not a problem in India now-a-days and every family has got a dwelling unit. (2) The real estate companies have increased the profit margin and hence sales have come down. (3) High cost of funds have dried up liquidity for the cashstarved companies, which in turn log jammed the construction activity across India. (4) Residential and commercial units are being developed at the faster pace in comparison to actual demand. (5) The faulty Housing Policy of the Government is an impediment to the real estate market. 2. Statement : Democracy in the sense of majority rule is not what people are seeking. The middle classes in the Ukraine, Bosnia, Thailand and Venezuela are demanding greater accountability,

    BPRE–829

    and are challenging regimes seen as corrupt, out of touch and which form obstacles to a better future. Which of the following assumptions is implicit in the above statement ? (An assumption is something supposed or taken for granted). (1) The middle classes want a government that is accountable, responsible and effective in moving their country further into the modern world. (2) The importance of middle classes has been enhanced in the governance of a democratic country. (3) The middle classes are capable of overthrowing a corrupt government. (4) Democracy, in true sense, does not mean the rule of majority which takes into consideration only the numbers. (5) Except a small number of countries in the world, other countries do not favour democratic form of government. 3. Statement : Should admission to all professional courses be made on the basis of past academic performances rather than through entrance tests ? Arguments : I. Yes. It will be beneficial for those candidates who are unable to bear the expenses of entrance tests. II. Yes. Many deserving candidates securing high marks in their qualifying academic examinations do not perform well on such entrance tests. III. No. The standard of examinations and assessment conducted by different boards and universities are not comparable and hence there is a need to conduct entrance tests to calibrate them on a common yardstick. Decide which of the arguments is/are ‘strong’ argument(s) and which is/are ‘weak’ argument(s). (1) Only I and II are strong (2) Only II and III are strong (3) Only I and III are strong (4) Only III is strong (5) All I, II and III are strong

    ASSUMPTION 4. Statement : Although the education system has progressed from the point of view of the number of schools, most of them are ill-equipped and have not achieved excellence in imparting education. Which of the following inferences can be drawn from the above statement ? (An inference is something which is not directly stated but can be inferred from the given facts). (1) We need not open any more schools in future. (2) In future, we should provide good teachers and equipment to these schools. (3) It is necessary to open more schools to accommodate the increasing number of students. (4) It is very difficult to manage a large number of schools properly. (5) Any system develops gradually and we should hope for better outcomes. 5. Statement : Satisfaction with coworkers, promotion opportunities, the nature of work, and pay goes with high performance among those with strong growth needs. Among those with weak growth needs, no such relationship is present – and, in fact, satisfaction with promotion opportunities goes with low performance. The statement best support the premise that (1) satisfaction is an inevitable organisational variable (2) job satisfaction and perfor mance are directly and closely related (3) relationship between job satisfaction and performance is moderated by growth need (4) every organisation has few employees having weak growth need (5) high performance is essential for organisational effectiveness. 6. Statement% The Central Government has directed the State Governments to reduce government expenditure in view of the serious resources crunch and it may not be able to sanction any additional grant to

    the States for the next six months. Which of the following may be an assumption implicit in the above statement ? (An assumption is something supposed or taken for granted) (1) The State Governments are totally dependent on Central Government for its expenditure (2) The Central Governments has reviewed the expenditure account of the State Governments (3) The State Government would ignore such directives as these have majority government. (4) The Central Government should allocate more funds to the States (5) None of these (SBI Management Executive Exam, 19.09.2014)

    Directions (7–8) : In each of these questions is given a statement followed by two assumptions numbered I and II. An assumption is something supposed or taken for granted. You have to consider the statement and the following assumptions and decide which of these is/are implicit in the statement. (SBI Associates PO Online Exam, 29.11.2014)

    7. Statement: Send employee XYZ for a training in UK to gain more insight into the project which he is handling at present. I. Some similar training programs are available in the employee’s own country as well. II. With the present skill sets, employee XYZ is incapable of handling the project. (1) Only I is Implicit (2) Either I or II is implicit (3) Only II is implicit (4) Both I and II are implicit (5) Neither I nor II is implicit 8. Statement: Minute traces of nitrate sometimes present in cattle fodder B may be responsible for the poor health of the cattle therefore use fodder G to ensure that your cattle remain healthy. I. Cattle fodder G does not contain nitrate. II. Nitrate is toxic to all living beings.

    BPRE–830

    (1) Only I is implicit (2) Either I or II is implicit (3) Only II is implicit (4) Both I and II are implicit (5) Neither I nor II is implicit 9. Statement% The Central Government has directed the State Governments to reduce government expenditure in view of the serious resources crunch and it may not be able to sanction any additional grant to the States for the next six months. Which of the following may be an assumption implicit in the above statement ?

    (An

    assumption is something

    supposed or taken for granted) (1) The State Governments are totally dependent on Central Government for its expenditure (2) The Central Governments has reviewed the expenditure account of the State Governments (3) The State Government would ignore such directives as these have majority government. (4) The Central Government should allocate more funds to the States (5) None of these (SBI Management Executive Exam. 19.09.2014)

    Directions (10–11) : In each of these questions is given a statement followed by two assumptions numbered I and II. An assumption is something supposed or taken for granted. You have to consider the statement and the following assumptions and decide which of these is/are implicit in the statement. (SBI Associates PO Online Exam. 29.11.2014)

    10. Statement: Send employee XYZ for a training in UK to gain more insight into the project which he is handling at present. I. Some similar training programs are available in the employee’s own country as well. II. With the present skill sets, employee XYZ is incapable of handling the project. (1) Only I is Implicit (2) Either I or II is implicit (3) Only II is implicit (4) Both I and II are implicit

    ASSUMPTION (5) Neither I nor II is implicit 11. Statement: Minute traces of nitrate sometimes present in cattle fodder B may be responsible for the poor health of the cattle therefore use fodder G to ensure that your cattle remain healthy. I. Cattle fodder G does not contain nitrate. II. Nitrate is toxic to all living beings. (1) Only I is implicit (2) Either I or II is implicit (3) Only II is implicit (4) Both I and II are implicit (5) Neither I nor II is implicit 12. This question consists of a statement followed by two assumptions numbered I and II. An assumption is something supposed or taken for granted. You have to consider the statement and the following assumptions and decide which of those is/are implicit in the statement ? Statement : Bank B has requested the Apex Bank of the country to permit them to dilute Government’s stake in the bank to raise funds from the equity market to balance the losses from non-performing assets. Assumption I : The funds generated by diluting the Government’s stake will enable the bank to recover from the losses to some extent. Assumption II : Atleast some equity investors will be ready to purchase the shares of the bank. (1) If neither I nor II is implicit. (2) If only II is implicit. (3) If either I or II is implicit (4) If both I and II are implicit (5) If only I is implicit (SBI Assistant Manager (System) Exam, 17.01.2016)

    13. Study the following information carefully and answer the question given below : Statement : The traffic policeman :has been recommended for a suitable reward by his superior in recognition of his sincere duty and busting of several gangs of bikers actively involved in chain snatching. Which of the following can be an assumption implicit in the above statement?

    (1) Rewards recognizing the sincerity and accomplishments of traffic policeman are given. (2) The traffic police desires to be monetarily compensated for his efforts. (3) The superior is certain that the recommendation would be denied. (4) The superior wants to set an example for his other juniors by recommending the reward. (5) The number of criminals apprehended by this particular traffic policeman was highest ever. (SBI PO Phase-II (Main) Exam 04.06.2017)

    14. In this question, a statement followed by two assumptions numbered I and II has been given. An assumption is something supposed or taken for granted. You are required to consider the statement and the following assumptions and decide which of the assumption is implicit in the statement. Statement : Finance Minister appealed to the officers, field staffs and Insurance agents of Life Insurance Corporation to work for making the schemes of insurance popular among people, especially those living in rural and suburban areas. Assumption I : Generally the insurance is not popular among people. Assumption II : People living in rural and suburban areas are against the Insurance. (1) Only assumption I is implicit (2) Only assumption II is implicit (3) Either assumption I or assumption II is implicit (4) Neither assumption I nor assumption II is implicit (5) Both assumptions I and II are implicit. (SBI PO Phase-II (Main) Exam 05.08.2018)

    15. The term “transgender” typically refers to individuals whose gender identity or gender expression is psychologically different with

    BPRE–831

    the sex that they were assigned at birth naturally. Which of the following is the wrong ASSUMPTION from the above statement? (1) Psychological state completely depends on physiological conditions (2) Usually, a person’s gender identity is compliance with physical nature (3) The gender expression differentiates a transgender from others (4) Physiological nature of a person influences psychological state (5) Nature of sex of a person usually forms gender identity (SBI PO Main Exam, 20.07.2019)

    16. The hypocrite people are the reason for stress and emotional break in personal life since they believe that they are perfect and others are dummies. Which of the following is the wrong ASSUMPTION from the above statement? (1) Human believe certain things about self (2) Perfectionism brings emotional break (3) Hypocrisy brings stress in life (4) People don’t like to be treated dummies (5) Hypocrites are perfect in the endeavors (SBI PO Main Exam, 20.07.2019)

    17. Sex determination is a crime in India where certain people might harm the girl infants. But it is not an offence in other western countries since the attitude is not harmful. Which of the following is the wrong ASSUMPTION from the above statement? (1) All crimes are not offences in some places (2) Some people do not support killing girl infants (3) People have certain intentions behind sex determination (4) Few offences in India are not offence in western countries (5) Sex determination is possible by Indian medical field (SBI PO Main Exam, 20.07.2019)

    ASSUMPTION

    RBI GRADE–B/ NABARD GRADE–A OFFICER EXAMS 1.

    Read the following Statement and answer the question which follows. A massive agitation is brewing in State X against the proposed widening of the National Highway. Which of the following represents an assumption implicit in the given statement? (An assumption is something that is not directly stated but is supposed or taken for granted.) (1) The agitation is carried out by local villagers who would be worst affected by widening of the highway (2) Many other States have much wider highways as compared to State X (3) A national highway already exists in State X (4) Commuters are greatly inconvenienced due to frequent accidents owing to the narrow highway (5) For a road to be declared a highway, a certain width is essential

    3.

    4.

    5.

    6.

    (RBI Grade-B Officer’s Exam. 18.12.2011)

    Directions (2 –7) : In each question below is given a statement followed by two assumptions numbered I and II. An assumption is something supposed or taken for granted. You have to consider the statement and the following assumptions and decide which of the assumptions is implicit in the statement.

    7.

    (RBI Officer Grade ‘B’ Online Exam, 25.08.2013)

    Give answer (1) if only assumption I is implicit. Give answer (2) if only assumption II is implicit. Give answer (3) if either assumption I or assumption II is implicit. Give answer (4) if neither assumption I nor assumption II is implicit. Give answer (5) if both assumptions I and II are implicit. 2. Statement : Job rotation helps employees get an overview of the organisation.

    8.

    Assumptions : I. Job rotation is the only method to get an overview of the organisation. II. It is required to have an overview of the organisation. Statement : Let us appoint Ms. X as the CEO of our Company so that the Company’s products are also perceived to be genuine. Assumptions : I. CEO can change the perception of products. II. Perception is same as the actual reality. Statement : An advertisement— The new model has been launched with K-series engine. Assumptions : I. People know about K-series engine. II. Engine type/series is important for buyers. Statement : Mohan tells Nita, “Let us meet over lunch tomorrow”. Assumptions : I. Lunch timings are known to both. II. Both are aware of the venue for lunch. Statement : You need to be talented to identify talent. Assumptions : I. Talent is acquired and developed. II. Talent is hereditary. Statement : The movie is a super-duper hit and has broken all the records. Assumptions : I. There is no authentic criterion to judge a hit or a flop. II. The performance of earlier movies is known. In this question is given a statement followed by two assumptions numbered I and II. You have to consider the statement and the following assumptions and decide which of those is/are implicit in the statement. Statement : The advanced research agency of defence ministry of Country X has developed bug sized robots that will be tested for deploying in locations that are dangerous and difficult to locate.

    BPRE–832

    Assumption–I : These robots could be used in search and rescue operations after natural disasters or to inspect hazardous environment in Country X. Assumption–II : The miniscule size robots may replace the larger robots for all important tasks of Country X. (1) Both I and II are implicit (2) Only II is implicit (3) Neither I nor II is implicit (4) Either I or II is implicit (5) Only I is implicit (RBI Officers in Grade ‘B’ Phase-I Exam. 16.08.2018)

    9. Study the following information carefully to answer the given question : As per Government orders, all the national banks of Country Y have started introducing electronic cheque books in phased manner. It is believed that this step will reduce manual intervention and hence the time taken by the banks for other processes will decrease by atleast 3-4 days. No physical cheque will be accepted after 15th January 2019. Which of the following can be assumed from the given information? (1) Negligible number of citizens of Country Y have operational accounts in International banks. (2) Before the introduction of electronic cheque books, the manual processing of cheques took a substantial portion of the time of the bankers. (3) The number of transactions through cheques will now exceed the number of transactions through IMPS (4) Most of the other processes and functions of the banks are already digitised. (5) None of these (SEBI Assistant Manager Exam. 17.11.2018)

    10. Study the following information carefully and answer the question given below : Following is an excerpt of a speech given by Albert, a part of which has been omitted. The

    ASSUMPTION

    (United India Insurance AAO Exam.03.06.2012)

    Give answer (4) if both I and II are implicit. 1. Statement : “Though the candidates have been instructed to bring pencils, yet provide some pencils with each invigilator” — An instruction to test administration staff. Assumptions : I. Pencils are in short supply. II. All the candidates will bring the pencil. 2. Statement : “Please drop this letter in the letter-box” — An Officer tells his Assistant. Assumptions : I. The Assistant knows the address where the letter is to be sent. II. The Assistant would follow the instructions. 3. Statement : “If you want timely completion of work- provide independent cabins” — An employee tells the Director of a Company. Assumptions : I. There are not enough cabins. II. Others’ presence hinders timely completion of work. 4. Statement : “Best way to solve this problem of workers’ dissatisfaction is to offer them cash rewards. If this type of incentive can solve the problem in HUDCO company then why not here”.— A Personnel Manager tells the Chairman of a Company. Assumptions : I. The reason for workers’ dissatisfaction in both the Companies was similar. II. Monetary incentives have universal appeal. 5. Statement : “We need to appoint more teachers” — Principal informs the school staff. Assumptions : I. Teachers are available. II. Present teachers are not good.

    Give answer (1) if only assumption I is implicit. Give answer (2) if only assumption II is implicit. Give answer (3) if neither I nor II is implicit; and

    Directions (6–10) : In each question below is given a statement followed by two assumptions numbered I and II. An assumption is something supposed or taken for granted. You have to consider the statement and

    speech is followed by three assumptions. Which of the given assumptions cannot logically fit into the speech given by Albert? ‘Since 1955 ‘GDI’ has conducted 5 researches in every 15 years on gender issue which have proven to be authentic and reliable at many international platforms. As per its latest report, the number of female students enrolling for college/higher education had increased by 40% in the past four decades. As per ‘GDI’ report in 1985, the cases of female foeticide were 76 in every 100 pergnancies which down to 31 in every 100 pregnancies by the time they conducted their next research. Thus, it can be assumed, that the females who were born after the second last experiment of ‘GDI’ ___ (A) are likely to have witnessed atleast some gender discrimination against them. (B) enjoy almost equal opportunities as some of them are at high level management positions. (C) have more chances to be allowed to education as compared to the women born three decades ago. (1) Only (B) (2) Both (A) and (C) (3) Only (A) (4) Both (B) and (C) (5) Only (C) (RBI Grade B Officer Exam, 09.11.2019)

    INSURANCE EXAMS Directions (1–5) : In each question below is given a statement followed by two assumptions numbered I and II. An assumption is something supposed or taken for granted. You have to consider the statement and the assumptions and decide which of the assumptions is implicit in the statement.

    BPRE–833

    the assumptions and decide which of the assumptions is implicit in the statement. (United India Insurance AAO Exam.03.06.2012)

    Give answer (1) if only assumption I is implicit. Give answer (2) if only assumption II is implicit. Give answer (3) if neither I nor II is implicit; and Give answer (4) if both I and II are implicit. 6. Statement : “Though the candidates have been instructed to bring pencils, yet provide some pencils with each invigilator” — An instruction to test administration staff. Assumptions : I. Pencils are in short supply. II. All the candidates will bring the pencil. 7. Statement : “Please drop this letter in the letter-box” — An Officer tells his Assistant. Assumptions : I. The Assistant knows the address where the letter is to be sent. II. The Assistant would follow the instructions. 8. Statement : “If you want timely completion of work- provide independent cabins” — An employee tells the Director of a Company. Assumptions : I. There are not enough cabins. II. Others’ presence hinders timely completion of work. 9. Statement : “Best way to solve this problem of workers’ dissatisfaction is to offer them cash rewards. If this type of incentive can solve the problem in HUDCO company then why not here”. — A Personnel Manager tells the Chairman of a Company. Assumptions : I. The reason for workers’ dissatisfaction in both the Companies was similar. II. Monetary incentives have universal appeal. 10. Statement : “We need to appoint more teachers” — Principal informs the school staff.

    ASSUMPTION Assumptions : I. Teachers are available. II. Present teachers are not good. 11. This question consists of a statement and two assumptions numbered I and II given below it. An assumption is something which is not directly given but can be assumed from the given information. You have to find out which of the given assumptions, numbered I and II are implicit in the given statement? Statement : Exercising does not necessarily mean gymnasium with high technology machines. After all people living in villages are well maintained and are hardly overweight. I. Gymnasiums are not very readily available or used in villages. II. People living in villages do not believe in the use of high technology gymnasiums. (1) Both I and II can be assumed from the given information. (2) Either I or II can be assumed from the given information. (3) Only I can be assumed from the given information. (4) Neither I nor II can be assumed from the given information. (5) Only II can be assumed from the given information. (LIC Assistant Administrative Officer (AAO) Online Exam. 22.03.2015)

    12. Study the following information carefully and answer the question given below : Statement : Students were penalised for submitting their project after the stipulated deadline. Which of the following can be an assumption made by the teacher who penalised the students, according to the given information? (An assumption is something that is supposed or taken for granted.) (1) Students copied the product from each other. (2) Some students submitted the project much before the deadline. (3) The project was not a part of the curriculum of the current year.

    (4) The students were informed well in advance about the importance of maintaining deadline. (5) None of the given options can be an assumption. (GIC Officer Online Exam 15.05.2017)

    13. Study the following information carefully and answer the question given below : (GIC Officer Online Exam 15.05.2017)

    ‘The man behind the revival of SpeedZet Automobiles, the CEO has decided to quit. I believe that the company will soon go into losses due this decision’- statement by a market analyst. Which of the following statements appears to be the perception of the market analyst? (1) The current leadership team of SpeedZet Automobile lacks the capability of running it effectively. (2) SpeedZet Automobiles will try its best to retain the CEO in order to maintain its market position. (3) The strategies adopted by the CEO of SpeedZet Automobiles were exclusive in the industry. (4) SpeedZet Automobiles may decide to shut down the operations due to the CEO’s exit. (5) Any other person can run SpeedZet Automobiles as successfully as the present CEO.

    SHORT ANSWERS

    NATIONALISED BANKS & IBPS PO/MT/SO EXAMS 1. (1) 5. (2) 9. (2) 13. (5) 17. (2)

    2. (1) 6. (5) 10. (5) 14. (5) 18. (5)

    3. (1) 7. (5) 11. (1) 15. (1) 19. (4)

    4. (1) 8. (4) 12. (4) 16. (5) 20. (1)

    21. (2) 25. (2) 29. (3) 33. (2)

    22. (4) 26. (1) 30. (4) 34. (4)

    23. (5) 27. (5) 31. (2) 35. (3)

    24. (4) 28. (2) 32. (3) 36. (5)

    BPRE–834

    SBI PO EXAMS 1. (3) 5. (3) 9. (2) 13. (4) 17. (1)

    2. (1) 6. (2) 10. (5) 14. (1)

    3. (4) 7. (5) 11. (1) 15. (1)

    4. (2) 8. (1) 12. (4) 16. (5)

    RBI GRADE–B/NABARD GRADE–A OFFICER EXAMS 1. (1) 5. (5)

    2. (2) 6. (1)

    9. (4)

    10. (1)

    3. (1) 7. (2)

    4. (5) 8. (5)

    INSURANCE EXAMS 1. (4) 5. (1) 9. (1) 13. (3)

    2. (4) 6. (4) 10. (1)

    3. (4) 7. (4) 11. (3)

    4. (1) 8. (4) 12. (4)

    EXPLANATIONS

    NATIONALISED BANKS & IBPS PO/MT/SO EXAMS 1. (1) Only assumption I is implicit in the statement. 2. (1) Clearly assumption I is implicit in the statement. 3. (1) The strength of a company is measured in terms of investment and turnover. Therefore, inference I is true. 4. (1) Only assumption I is implicit in the statement. Vehicle is parked at a distance which is not far away from the destination. 5. (2) Only assumption II is implicit in the statement. The use of term ‘all’ in the assumption I makes it invalid. 6. (5) Clearly both the assumptions are implicit in the statement. 7. (5) It is mentioned in the statement that for any kind of problem, contact help desk. It implies that help desk suggests solutions to all kinds of problems related to mobile phones. Therefore, both the assumptions are implicit in the statement.

    ASSUMPTION 8. (4) None of the assumptions is implicit in the statement. 9. (2) Only assumption II is implicit in the statement. Secretary is assigned the task assuming that he is well versed with the operations of PowerPoint. 10. (5) Clearly both the assumptions are implicit in the statement. 11. (1) Only assumption I is implicit in the statement. 12. (4) None of the assumptions is implicit in the statement. 13. (5) Clearly, both the assumptions are implicit in the statement. 14. (5) Clearly both the assumptions are implicit in the statement. The notice implies that disease ABC is contagious and it is also mentioned that ward no. 2 is meant only for ABC disease. 15. (1) Only assumption I is implicit in the statement. German technology is very advanced and it is perceived better in the city Z. 16. (5) Clearly both the assumptions are implicit in the statement. 17. (2) Only assumption II is implicit in the statement. 18. (5) Clearly both the assumptions are implicit in the statement. 19. (4) Statement B is an inference. 20. (1) Option (1) can be inferred. 21. (2) Statement D represents a reason behind curtailing industrial development by some of the countries. 22. (4) Obviously, option (4) is an assumption. Any appeal has some effects and people generally respond positively to any appeal. 23. (5) Obviously option (5) is an assumption. 24. (4) We know that some people get influenced by the attractive advertisement. 25. (2) Tractor is very useful for farmers to carry a host of farm activities. 26. (1) If the Government has decided to construct super highway, it implies that the Government has adequate resources to construct it. 27. (5) Obviously option (5) is an assumption. 28. (2) Obviously, option (2) is an assumption. 29. (3) Obviously, option (3) is an assumption.

    30. (4) Option (1) : This can’t be assumed from the given statement because there is no any comparison between mosquitoes and other animals in the statement. Options (2) : This can’t be assumed from the given statement because Zika, Dengue and Chikungunya are considered as dangerous diseases but is not said anything about treatment of these diseases. Options (3) : This can’t be assumed from the given statement as there is no discussion about any stats in the statement. Options (4) : This can be assumed from the given statement as it is mentioned in the given statement that protect yourself and your family from mosquitoes during travelling. So it can be assumed that there are more chances to get infected from mosquitoes while travelling . Options (5) : This can’t be assumed from the given statement because there is only a suggestion that protect yourself and your family from mosquitoes during travelling, it does not mean all travellers are suffering from the mentioned diseases. 31. (2) Obviously, both the assumptions are implicit in the statement. It is clearly mentioned that all tickets are produced on mobile phones these days. Thus, in due course of time print outs of tickets will no longer be required. Every passenger will afford a mobile phone and travel with it. 32. (3) Obviously, Option (3) is a right assumption. 33. (2) Gazette notifications are usually published in the government offices as circulars and news papers for the public. Publishing in website is an auxiliary means. 34. (4) Supreme Court possibly will ensure the fair election which would not consider the electoral procedures. It is Election Commission‘s administration to manage all the issue related to the fair election process. 35. (3) Obviously, only (A) and (C) are valid assumptions. 36. (5) Obviously, only (C) and (D) are valid assumptions.

    BPRE–835

    SBI PO EXAMS 1. (3) Clearly option (3) seems to be most probable cause. High input and labour costs have hit operating margins and higher interest costs are affecting net margins. So, the price of residential and commercial units is hiking unexpectedly. 2. (1) The middle classes are now not merely passive consumers or docile voters, they are demanding that governments not accustomed to accountability and showing deference to popular demands, start acting like true democracies. So, Option (1) may be an assumption. 3. (4) The scheme to select deserving candidates cannot be abolished just because of the expenditure it entails. Therefore, argument I is not strong. Also, students who are intelligent enough to secure good marks in academic examinations have no reason not to perform well in entrance tests. Therefore, argument II also does not hold. Argument III seems to be strong. 4. (2) Clearly, the statement stresses the need to provide good teachers and equipment to schools. Therefore, Option (2) can be inferred from the statement. If a system does not give desired result, we should point out the cause and every effort should be made to address the problem. Therefore, Option (1) does not follow. 5. (3) Obviously the facts mentioned in the statements support the Option (3). 6. (2) Clearly option (2) is a valid assumption. Nothing about the source of income of the State Governments is mentioned in the statement. So option (1) is not implicit in the statement. Since the Central Government has directed the State Governments to reduce expenditure, option (2) is implicit in the statement. 7. (5) Neither Assumption I nor II is implicit in the statement. If training facility is available in one’s own country, he/she would not go to other country for training. It is mentioned in the statement that the employee XYZ

    ASSUMPTION should be sent for a training in United Kingdom to gain more insight into the project. It does not imply that the employee XYZ is incapable of handling the project with the present skill. Moreover, a project is assigned to any person assuming that he/she is capable enough to handle it. 8. (1) Only Assumtion I is implicit in the statement. It is mentioned that the presence of nitrate in fodder B makes it unsuitable for cattle. 9. (2) Clearly option (2) is a valid assumption. Nothing about the source of income of the State Governments is mentioned in the statement. So option (1) is not implicit in the statement. Since the Central Government has directed the State Governments to reduce expenditure, option (2) is implicit in the statement. 10. (5) Neither Assumption I nor II is implicit in the statement. If training facility is available in one’s own country, he/she would not go to other country for training. It is mentioned in the statement that the employee XYZ should be sent for a training in United Kingdom to gain more insight into the project. It does not imply that the employee XYZ is incapable of handling the project with the present skill. Moreover, a project is assigned to any person assuming that he/she is capable enough to handle it. 11. (1) Only Assumtion I is implicit in the statement. It is mentioned that the presence of nitrate in fodder B makes it unsuitable for cattle. 12. (4) Obviously, both the assumptions are implicit in the statement. 13. (4) The statement describes extraordinary accomplishments of the traffic policeman. Therefore, option (4) is the most suitable assumption. 14. (1) Only assumption I is implicit. Finance Minister appealed to the concerned persons to work for making the schemes of insurance popular among people. It implies that insurance is not popular among people.

    15. (1) Psychological state is influenced by physical condition to some extent only. 16. (5) Perfectionism is a false attitude of hypocrites. 17. (1) The reverse is correct ⇒ all offences are not crimes. But all crimes are not offences is wrong.

    RBI GRADE–B/NABARD GRADE–A OFFICER EXAMS 1. (1) Obvioushy option (1) is the assumption. 2. (2) Only assumption II is implicit in the statement. The use of the term ‘only’ in the first assumption makes it invalid. 3. (1) Only assumption I is implicit in the statement. 4. (5) Both the assumptions are implicit in the statement. 5. (5) Clearly both the assumptions are implicit in the statement. 6. (1) Only assumption I is implicit. 7. (2) Only assumption II is implicit in the statement. 8. (5) Obviously, only assumption I is implicit in the statement. The term ‘all’ has been used in the assumption II. The small sized rebots may have some limitations. 9. (4) Electronic cheques require other processes to be digitised. Therefore, it can be assumed that most of the other processes and functions of the banks have already been digitised. 10. (1) Obviously, only Assumption (B) does not logically fit into the speech by Albert.

    INSURANCE EXAMS 1. (4) Assumption I is not appropriate. Pencils are provided to each invigilator for emergency. It does not imply that pencils are in short supply. Any instruction is given assuming that some people will follow that instruction. Therefore, Assumption II is also not appropriate. The use of term ‘all’ makes it unsuitable. 2. (4) Clearly both the assumptions are implicit in the statement. If

    BPRE–836

    the letter is given to Assistant to drop it, it may be assumed that he does know the address. The Assistant will follow the instructions given by Officer. 3. (4) Clerly both the assumptions are implicit in the statement. 4. (1) If similar solution is being applied then it implies that the nature of problem is also same. Therefore, assumption I is implicit in the statement. The use of term ‘universal’ in the second assumption makes it unsuitable. 5. (1) Only assumption I is implicit in the statement. One can appoint teachers only when teachers are available. 6. (4) Assumption I is not appropriate. Pencils are provided to each invigilator for emergency. It does not imply that pencils are in short supply. Any instruction is given assuming that some people will follow that instruction. Therefore, Assumption II is also not appropriate. The use of term ‘all’ makes it unsuitable. 7. (4) Clearly both the assumptions are implicit in the statement. If the letter is given to Assistant to drop it, it may be assumed that he does know the address. The Assistant will follow the instructions given by Officer. 8. (4) Clerly both the assumptions are implicit in the statement. 9. (1) If similar solution is being applied then it implies that the nature of problem is also same. Therefore, assumption I is implicit in the statement. The use of term ‘universal’ in the second assumption makes it unsuitable. 10. (1) Only assumption I is implicit in the statement. One can appoint teachers only when teachers are available. 11. (3) It is clear that only assumption I is implicit in the statement. 12. (4) Only option (4) seems to be an assumption. The students were penalised for not maintaining the deadline. It implies that the students were informed about the importance of maintaining deadline. 13. (3) Obviously, option (3) appears to be the perception of the market analyst. ❐❐❐

    ASSUMPTION

    MODEL EXERCISES Directions (1-4) : In the following five questions, there is a statement followed by two assumptions. You have to choose (1) If only assumption A is implicit (2) If only assumption B is implicit (3) If either A or B is implicit (4) If neither A nor B is implicit (5) None of these 1. Statement : Who is brother of Balu? Assumptions : I. Balu is Arun’s brother. II. Balu’s father is the husband of Arun’s mother. 2. Statement : What will be the total weight of 10 poles, each of the same weight? Assumptions : I. One fourth of the weight of the poles is 5 kg. II. The total weight of three poles is 20 kilograms more than the total weight of two poles. : 3. Statement : When is Manohar’s birthday? Assumptions : I. Manohar’s father was born on 15-8-1947. II. Manohar is 25 years younger to his mother. 4. Statement : Ashok is shorter to Anil. Sunil is taller to Sudhir. Who is the shortest among them? Assumptions : I. Anil is shorter to Sudhir. II. Anil is taller to Sunil. 5. Many business offices are located in buildings having two to eight floors If a building has more than three floors, it has a lift. If the above statements are true, then which of the following must also be true? (1) Second floor does not have lifts (2) Seventh floor have lifts (3) Only floors above the third floors have lifts (4) All floors may be reached by lifts (5) None of these 6. There are many reasons why individuals want to run their own businesses. Some foresee more

    personal satisfaction if they are successful in launching their own business, while others are interested mainly in the prospect of larger financial rewards. Since, 1980s and early 1990s tax regulation and liberal policies have encouraged increasing number of venture capitalists and entrepreneurs to start new enterprises. Since 1990, some one half million new ventures have been started. Not all have succeeded, of course. The above statement makes which of the following assumptions? (1) Success in starting a new business depends in large part on sound financial planning (2) Venture capitalists are motivated by non monetary gains (3) Social incentives motivate investors just as much as financial rewards (4) Most new business ventures succeed initially but fail later on (5) None of these 7. A highly cohesive work group is a prerequisite for high team performance. Sociologists point that the association between group cohesion and success is owing to the support individual team members give to one another and their acceptance of the groups goals and activities. Each of the following, if true, either provides support for or cannot weaken the sociologists assumption about the relationship between cohesive and success except (1) A group of Japanese researchers found that successful work teams were headed by dominant leaders (2) University researchers found that there was a significant correlation between team productivity and the extent to which the team members understood and complied with the groups objectives (3) American researchers found that successful team mem-

    BPRE–837

    bers tended to rate their fellow members more favourably (4) Industrial psychologists in UK found that work groups who tended to participate in after hour social activities were more productive (5) None of these Directions (8-10) : This question is based on the following passage. Read the passage to choose the best answer. Passage If a person reads a lot, he will increase his vocabulary. So, the passage to choose the best answer. 8. The argument is based on the assumption (s). (I) Strong vocabulary is important for writing and speaking (II) People should read books containing unfamiliar words (III)The books to be read should be entertaining . (1) A only (2) B only (3) C only (4) A, B and C (5) None of these 9. In 1950, the average child visited the dentist once a year. But in 1970, the number of visits had increased to two. Today, the average child visits the dentist three times a year. Each of the following, if true could explain this trend except (1) Dentist fees have declined over the period (2) Better home care of teeth has reduced the number of cavities (3) Parents are more aware of the importance of dental care (4) Tax benefits for deducting dental expenses have increased (5) None of these 10. Donors are almost never offended by being asked for too much (in fact, they are usually flattered). And if you ask for too much, your donor can always suggest a smaller amount. On the other hand, donors are frequently offended by being asked for too little, A common reaction is, so that’s all they think I’m worth. The above statement assumes that

    ASSUMPTION (1) Donors are usually never asked for enough (2) A good fund raiser will value the worth of the donor (3) It is worth the gamble to ask for large donations (4) Fund raisers often think that the donors are incapable of giving much (5) None of these Directions (11-12) : Use the statement given below to answer these questions. All good athletes want to win and all atheletes who want to win, eat a wellbalanced diet; therefore, all atheletes who do not eat a well-balanced diet are bad atheletes. 11. If assumptions of the argument above are true, then which of the following statements must be true? (1) No athelete who does not eat a well-balanced diet is a good athelete (2) No bad athelete wants to win (3) Every athelete who eats a well-balanced diet is a good athlete (4) All atheletes who want to win are good athletes (5) None of these 12. Which of the following, if true, would refute the assumptions of the argument above? (1) Bob, the accountant, eats a well-balanced diet, but he is not a good athelete (2) Ann wants to win, but she is not a good athelete (3) All the players on the Burros baseball team eat a well-balanced diet (4) Cindy, the basketball star, does not eat a well-balanced diet, but she is a good athelete (5) None of these Directions (13-17) : In each question, a statement is followed by two assumptions numbered I and II. Consider the statement and the following assumptions to decide which of the assumptions is implicit in the statement. Mark your answer as (1) If only assumption I is implicit (2) If either I or II is implicit (3) If only assumption II is implicit

    (4) If neither I nor II is implicit (5) None of these 13. Statement : Like a mad man, I decided to follow him. Assumptions : I. I am not a mad man. II. I am a mad man. 14. Statement : If it is easy to become an engineer, I don’t want to be an engineer. Assumptions : I. An individual aspires to be professional. II. One desires to achieve a thing which is hard earned. 15. Statement : All the employees are notified that the organisation will provide transport facilities at half the cost from the nearby railway station to the office except those who have been provided with travelling allowance. Assumptions : I. Most of the employees will travel by the office transport. II. Those who are provided with travelling allowance will not read such notice. 16. Statement : An advertisement of a Bank “Want to open a bank account Just dial our ‘room service’ and we will come at your doorsteps.” Assumptions : I. There is a section of people who require such services at their home. II. Now-a-days banking has become very competitive, 17. Statement : I can take you quickly from Kanpur to , Lucknow by my cab but then you must pay me double the normal charges. Assumptions : I. Normally, it will take more time to reach Luknow from Kanpur. II. People want to reach quickly but they will not pay extra money for it Directions (18-22) : Each of these questions has a statement followed by three assumptions numbered I, II and III. Consider the statement and the assumptions to decide which of the assumptions is implicit in the statement. 18. Statement : The residents of the locality wrote a letter to the Cor-

    BPRE–838

    poration requesting to restore normalcy in the supply of drinking water immediately as the supply at present is just not adequate. Assumptions : I. The Corporation may not take any action on the letter. II. The municipality has enough water to meet the demand. III. The water supply to the area was adequate in the past. (1) Only I and III are implicit (2) Only II and III are implicit (3) Only II is implicit . (4) Only III is implicit (5) All of these 19. Statement : We must be prepared to face any eventuality and all the assignments must be completed as per their schedule—Director tells the Faculty members. Assumptions : I. There is possibility of a serious eventuality. II. Dates are fixed for all the assignments. III. Faculty members are supposed to complete all the assignments. (1) Only I is implicit (2) Only III is implicit (3) None is implicit (4) All are implicit (5) None of these 20. Statement : The telephone company informed the subscribers through a notification that those who do not pay their bills by the due date will be charged penalty for every defaulting day. Assumptions : I. Majority of the people may pay their bills by the due date to avoid penalty. II. The money collected as penalty may set off the losses due to delayed payment. III. People generally pay heed to such notices. (1) All are implicit (2) I and III are implicit (3) II and III are implicit (4) None of these (5) None of these 21. Statement : In view of the recent spurt in sugar prices in the open market, the government has asked the dealers to release a vast quantity of imported sugar in the open market.

    ASSUMPTION Assumptions : I. The dealers will follow the government directive. II. The sugar prices will come down. III. The price of indigenous sugar will remain unchanged. (1) I and II are implicit (2) I and III are implicit (3) II and III are implicit (4) None is implicit (5) All of these 22. Statement : In the recently held All India Commerce Conference, the session on ‘ Management of Service Sector in India’ surprisingly attracted large number of participants and also received a very good media coverage in the leading newspapers. Assumptions : I. People were not expecting such an encouraging response for service sector. II. Service sector is not managed properly in India. III. Media is always very positive towards service sector. (1) Only I is implicit (2) I and III are implicit (3) II and III are implicit (4) All are implicit (5) None of these Directions (23-27) : Each question has a statement followed by two assumptions/conclusions. Find the implicit assumption(s)/ conclusion(s). 23. Statement : A good system of education in a country is the flower of economic development; it is also its seed. Assumptions : I. Economic development leads to educational ,; development in a country. II. Educational development leads to economic development in a country. (1) Both I and II are implicit (2) Only II is implicit (3) Only I is implicit ; (4) Neither I nor II is implicit (5) None of these 24. Statement : There is one thing as important as studying; and that is how much is understood.

    Assumptions : I. Studying and understanding go hand in hand. II. Understanding is as important as studying. (1) Both I and II are implicit (2) Only II is implicit (3) Only I is implicit (4) Neither I nor II is implicit (5) All of these 25. Statement : All birds are dogs and some dogs are cats. Conclusions : I. Some cats are not dogs II. All dogs are not birds. (1) Both I and II are implicit (2) Only II is implicit (3) Only I is implicit (4) Neither I nor II is implicit (5) All of these 26. Statement : There are many Indians who are honest. Mohan is an Indian. Conclusions : I. Mohan is honest. II. Mohan is not honest. (1) Both I and II are implicit (2) Only II is implicit (3) Only I is implicit i (4) either I or II is implicit (5) None of these 27. A mail order company recently had a big jump in clothing sales after hiring a copywriter and a graphic artist to give its clothing catalogue a magazine-like format designed to appeal to a more upscale clientele. The company is now planning to launch a housewares catalogue using the same concept. The company’ plan assumes that (1) an upscale clientele would be interested in a housewares catalogue (2) other housewares catalogues with magazine-like formats do not already exist (3) the same copywriter and graphic artist could be employed for both the clothing and housewares catalogues (4) customers to whom the old clothing catalogue appealed will continue to make purchase from catalogues with the new format (5) None of these

    BPRE–839

    Directions (28-31) : A statement is followed by two assumptions numbered I and II. Consider the statement and the assumptions together. Decide which assumption is implicit in the statement. Mark answer as (1) If either assumption I or II is implicit (2) If only assumption II is implicit (3) If only assumption I is implicit (4) If neither assumption I nor II is implicit (5) None of these 28. Statement : A notice : “Use of cell phones and pagers is not allowed inside the auditorium. Please switch off such devices while you are inside the auditorium.” Assumptions : I. All those who have such devices will switch them off before they take their seat in the auditorium. II. Generally, people do not bring such devices when they come to attend functions in the auditorium. 29. Statement : An advertisement : “Fly X airways whenever you decide to go places. Our fares are lesser than train fares.” Assumptions : I. People prefer to travel by air when the fares are reasonable. II. The fares of other airlines are costlier than those of X airways. 30. Statement : Dengue cannot be eradicated from our city unless we create a special ‘Health-squad’ for it. Assumptions : I. Dengue is harmful. II. Creating Health-squad is impossible. 31. Statement : A warning : “Do not smoke in public places as it is a cognizable offence in our country.” Assumptions : I. People often neglect such warnings. II. People do not understand the implications of committing a cognizable offence.

    ASSUMPTION Directions (32-35) : In each question below is given a statement followed by two assumptions I and II. An assumption is something supposed or taken for granted. You have to consider the statement and the following assumptions and decide which of the assumptions is implicit in the statement (1) If only assumption I is implicit (2) If only assumption II is implicit (3) If neither I nor II is implicit (4) If both I and II are implicit (5) None of these 32. Statement : A good book, even if costly, is sold. Assumptions : I. Some books are better than others. II. Most of the books are costly 33. Statement : The integrated steel plants in India would no longer have to depend on imports for continuous casting refractories. Assumptions : I. Continuous casting refractories are needed by India. II. Continuous casting refractories are in demand. 34. Statement : “If you are beautiful” we will catch your beauty. If you are not, we will make you beautiful”. An advertisement of a photo studio. Assumptions : I. How to look beautiful, is a problem of youngsters? II. Nobody desires to be beautiful. 35. Statement : Equality of income throughout a community is the essential condition for maximising the total utility which the total income available could confer on the members of that community. Assumptions : I. If extra income were taken from the rich and given to the poor, the total utility experienced by the community would increase. II. Equal pay for equal work. 36. A famous singer recently won a lawsuit against an advertising firm for using another singer in a commercial to evoke the famous singer’s well-known rendition of a certain song. As a result of the lawsuit, advertising firms

    will stop using imitators in commercials. Therefore, advertising costs will rise, since famous singer’s services cost more than those of their imitators. The conclusion above is based on which of the following assumptions? (1) Most people are unable to distinguish a famous singer’s rendition of a song from a good imitator’s rendition of the same song (2) Commercials using famous singers are usually more effective than commercials using imitators of famous singers (3) The original versions of some well-known songs are unavailable for use in commercials (4) The advertising industry will use well-known renditions of songs in commercials (5) None of these Directions (37-39) : A statement is followed by two assumptions numbered I and II An assumption is something supposed or taken for granted. Consider the statement and the following assumptions and decide which of the assumptions is/are implicit in the statement. Mark answer as (1) if either Assumption I or II is implicit (2) if only Assumption II is implicit (3) if only Assumption I is implicit (4) if neither Assumption I nor II is implicit (5) None of these 37. Statement : In spite of heavy rains, rail services did not get disrupted this year. Assumptions : I. Rail services get disrupted only in rainy season, II. Heavy rains normally affect the rail services. 38. Statement : Majority of the students appearing for X std- final examinations have taken home tuitions. Assumptions : I. Home tuition is the best method of learning. II. There is a shortage of quality home tutors.

    BPRE–840

    39. Statement : Detergents should be used to clean clothes. Assumptions : I. Detergents form more lather II. Detergents help to dislodge grease and dirt. Directions (40-43) : In each of these questions a statement followed by two assumptions numbered I and II Consider the statement and the following assumptions and decide which of the assumptions is implicit in the statement. Mark answer as (1) If only assumption I is implicit (2) If only assumption II is implicit (3) If neither assumption I nor II is implicit (4) If both assumptions I and II are implicit (5) None of these 40. Statement : The patient’s condition would improve after operation. Assumptions : I. The patient can be operated upon in this condition. II. The patient cannot be operated upon in this condition,, 41. Statement : The government has decided to disinvest large chunk of its equity in select public sector undertaking for a better fiscal management. Assumptions : I. The amount generated out of the disinvestments process may reduce substantially the mounting fiscal deficits. II. There will be enough demand in the market for the shares of these undertakings. 42. Statement : All existing inequalities can be reduced, if not utterly eradicated, by action of governments or by revolutionary change of government. Assumptions : I. Inequality is a man-made phenomenon. II. No person would voluntarily part with what he possesses. 43. Statement : “You are hereby appointed as a programmer with a probation period of one year and your performance will be reviewed at the end of the period for confirmation”. A line in an appointment letter.

    ASSUMPTION Assumptions : I. The performance of an individual generally is not known at the time of appointment offer. II. Generally an individual tries to prove his worth in the probation period. Directions (44-45) : Each of these questions has a statement followed by two assumptions I and II Consider the statement and the following assumptions. Decide which of the assumptions is impticit in the statement. Mark answer as (1) If neither assumption I nor II is implicit (2) If only assumption II is implicit (3) If only assumption I is implicit (4) If both assumptions I and II are implicit (5) None of these 44. Statement : Incentives must be given to strengthen the motivation of the employees. Assumptions : I. Incentives are expected to motivate the employees. II. Employees at present are not motivated45. Statement : Despite heavy rains, traffic has not been disturbed. Assumption : I. Rains do not affect traffic movement. II. Adequate precautions were taken for traffic ; ‘, management during rainy season. 46. This question consists of a statement followed by two assumptions numbered I and II. An assumption is something supposed or taken for granted. You have to consider the statement and the following assumptions and decide which of these is/are implicit in the statement. Statement: ‘Due to inadequate rainfall this year, the harvest obtained by farmers of village C is 50% of what was expected. As a result this year we will be buying the crop from village Z as it is been convenient and cheaper for us to buy the crop from the farmers of one village rather than buying it from farmers of different villages.’- Statement by a distributor.

    I. Atleast some farmers in village Z produce the same crop as farmers in village C. II. The harvest in village Z was much more than expected. (1) Both I and II are implicit (2) Either I or II is implicit (3) Only I is implicit (4) Neither I nor II is implicit (5) Only II is implicit 47. This question consists of a statement and four assumptions numbered I to IV. Which of the given assumption/s is/are implicit in the statement. India and the US have agreed to hold their first joint tri services exercise at the inaugural 2+2 dialogue. This is only the second such tri service exercise to be conducted by India with any other country after Russia. The exercise is scheduled for 2019, to be conducted off India’s eastern coast. The two nations have committed to creation of a new, triservices exercise and increase personnel exchanges between the respective militaries and defense organizations. However, this is only the second such exercise to be carried out by India with another nation. The first triservices exercise with Russia was held in October last year. The exercise called INDRA-2017 was India’s way of balancing strategic relations with Russia and the US and sends a message to China. I. Indian is strengthening military ties and defense organizations with both US and Russia at good will manner II. Indian wants to maintain a balanced and strategic relationship with both US and Russia as these both nations not mutual in defense front III. The external threat from the China is supposed to be under control if India is supported by these two developed nations IV. India is welcome in strategic defense relationship in global front (1) Both I and IV are implicit (2) Both II and III are implicit (3) Only II, III and IV are implicit (4) I, III and IV are implicit (5) Both II and IV are implicit

    BPRE–841

    48. In order to boost the Indian Army's firepower and capability enhancement, Indian Government on Friday inducted three major artillery gun systems, including the M777 A2 Ultra Light Howitzers, K-9 Vajra self-propelled gun and a 'Composite Gun Towing Vehicle'. The enhancement push has been brought nearly three decades after the induction of Bofors. It is after 30 years, India is procuring such guns. Under the new government since 2014, procurement of Army equipment has been expedited. Under the Prime Minister’s guidance, negotiations were started and within four years India has inducted these guns and many more will be inducted in the future. Which of the following, if true, provides the best ASSUMPTION from the above statement? (1) India is capable of buying such defense equipments economically and politically (2) Indian had to buy these new guns since Bofors issue is politicized in the country (3) India is procuring such armories to project that India is strong in war force to the neighbor countries (4) India is procuring such armories to tackle the terrorism in the country (5) The old government before 2014 did not attempt to buy such armories

    SHORT ANSWERS 1. (4)

    2. (3)

    3. (4)

    5. (2)

    6. (3)

    7. (1)

    4. (1) 8. (1)

    9. (2)

    10. (2)

    11. (1)

    12. (4)

    13. (1)

    14. (3)

    15. (4)

    16. (1)

    17. (2)

    18. (4)

    19. (4)

    20. (1)

    21. (1)

    22. (3)

    23. (2)

    24. (2)

    25. (1)

    26. (4)

    27. (2)

    28. (3)

    29. (3)

    30. (3)

    31. (2)

    32. (1)

    33. (4)

    34. (1)

    35. (3)

    36. (4)

    37. (2)

    38. (4)

    39. (2)

    40. (1)

    41. (4)

    42. (1)

    43. (4)

    44. (3)

    45. (2)

    46. (1)

    47. (4)

    48. (3)

    ASSUMPTION

    EXPLANATIONS 1. (4) None of the assumptions is implicit. 2. (3) Either of A and B is implicit. 3. (4) Clearly, none of the assumptions is implicit. 4. (1) If we use assumption A in the given statement, then we get the order of the heights of all the four persons as below : Sunil > Sudhir > Anil > Ashok Assumption B does not help to obtain the order, 5. (2) it is not given that how many floors the building has. 6. (3) From the lines of the paragraph “Some foresee more personal satisfaction..,., it can be assumed that social incentives motivate investors just as much as financial rewards. 7. (1) Assumption (1) is the required assumption in the light of given requirements. 8. (1) In the light of the passage, only assumption A is implicit. Hence, option (1) is the correct option. 9. (2) Assumption (2) is absolutely contrary to what has been stipulated in the passage. 10. (2) Assumption A as good fund raiser will value the worth of the donor. 11. (1) No athelete who does not eat a well-balanced diet is a good athelete 12. (4) Cindy, the basketball star, does not eat a well-balanced diet, but she is a good athelete 13. (1) The word ‘like’ makes only assumption I implicit. 14. (3) If only assumption II is implicit 15. (4) If neither I nor II is implicit 16. (1) If only assumption I is implicit 17. (2) If either I or II is implicit 18. (4) It is very clearly implicit in the statement that the water supply to the area was adequate in the past.

    19. (4) All the assumptions are implicit. 20. (1) All the assumptions are implicit. 21. (1) Since order has been delivered by the government hence, it will be attended by the dealers. Moreover, release of vast quantity of imported sugar will bring down the price. 22. (3) Only II and III are implicit in the statement. 23. (2) Only II is implicit. 24. (2) Only II is implicit. 25. (1) Both I and II are implicit. 26. (4) Either I or II is implicit. 27. (2) The company’s plan assumes that other houseware catalogues do not already exist. 28. (3) Since, it is a warning, hence it is assumed that those who have such devices will switch them off before they take their seats in the auditorium. 29. (3) It is assumed that people prefer to travel by air when the fares are reasonable, hence a comparison in fare between train and air is given. 30. (3) It is emphasised in the statement that eradication of Dengue needs attention it is therefore assumed that Dengue is harmful 31. (2) Warning has been displayed because people do not understand the implications of committing a cognizable offence. 32. (1) It is given in the statement that good books, even costly, are sold. This means that there exist some books which are better than others. Nothing certain can be said about assumption II. 33. (4) Statement emphasises the operating situation of continuous costing refractories. Hence, both the assumptions are implicit 34. (1) Since, the advertisement highlights the requirement of being beautiful therefore assumptions I is implicit. 35. (3) None of the assumptions addresses the problem. 36. (4) The advertising industry is using well known renditions of songs in commercials. This can

    BPRE–842

    be assumed because that famous singer’s song was used in the advertisement. 37. (2) Assumption I is not implicit as rail services may be disrupted due to many reasons. Assumption II is, implicit as heavy rains destroys the track due to which rail services may be disrupted. 38. (4) Both assumptions I and II are not implicit as there may be other ways of learning. And any one can contact good teachers for learning. 39. (2) Assumption I is not implicit as more leather does not mean the clothes will be cleaned. Assumption II is implicit as the detergents help to dissolve the grease and dirt. 40. (1) Only assumption I is implicit because patient’s condition would improve after operation, so he can be operated upon in this condition. 41. (4) Both the assumptions are implicit as disinvestment may reduce fiscal deficits. Disinvestment can be done when there is enough demand in market. 42. (1) Only assumption I is implicit because inequality is a manmade phenomenon, 43. (4) Both the assumptions are implicit as the performance of an individual generally is not known at the time of appointment and an individual tries to prove his worth in the probation period. 44. (3) Only assumption I is implicit because incentives are expected to motivate the employees. 45. (2) Only assumption II is implicit because rains affect traffic movement but adequate percautions were taken for traffic movement. So, it has not been disturbed. 46. (1) Clearly, both the assumptions are implicit in the statement. 47. (4) Obviously, I, III and IV are implicit in the statement. 48. (3) Obviously, option (3) is an assumption.

    ❑❑❑

    CONCLUSIONS/INFERENCES

    15

    CONCLUSIONS/ INFERENCES

    QUESTIONS FROM 1999 TO 2010 ARE AVAILABLE ONLINE NATIONALISED BANKS & IBPS PO/MT/SO EXAMS Directions (1–5) : Below is given a passage followed by several possible inferences which can be drawn from the facts stated in the passage. You have to examine each inference separately in the context of the passage and decide upon its degree of truth or falsity. (Indian Bank PO Exam. 02.01.2011 (Ist sitting)

    Mark answer (1) If the inference is “definitely true” i.e. it properly follows from the statement of facts given. Mark answer (2) If the inference is “probably true” though not “definitely true” in the light of the facts given. Mark answer (3) If the data is inadequate i.e. from the facts given, you cannot say whether the inference is likely to be true or false. Mark answer (4) If the inference is “probably false” though not “definitely false” in the light of the facts given. Mark answer (5) If the inference is “definitely false” i.e. it cannot possibly be drawn from the facts given or it contradicts the given facts. (Note : Each of the five questions has only one distinct answer i.e. no two questions can have the same answer. If you get the same answer for more than one question, consider both again and decide which one of the two would more definitely be that answer and in the same way review the others also.) Cardiovascular disease is so prevalent that virtually all businesses are likely to have employees who suffer from, or may develop, this condition. Research shows that between 50–80 per cent of all people who suffer a heart attack are able to return to work. However, this may not be possible if they have previously been involved in heavy physical work. In such cases, it may be possible to move the employee to lighter duties, with appro-

    priate retraining where necessary. Similarly, high-pressure, stressful work, even where it does not involve physical activity, should also be avoided. Human Resource managers should be aware of the implications of job roles for employees with a cardiac condition. 1. Employees who suffer from cardiovascular disease are mostly unable to return to work. 2. Employees suffering from Cardiovascular diseases are unable to handle stressful situations. 3. Employees above the age of 50 are found to suffer from cardiovascular disease. 4. Physical and stressful work definitely leads to a heart attack. 5. Heart disease can affect employees in any type of business. Directions (6-10) : Below is given a passage followed by several possible inferences which can be drawn from the facts stated in the passage. You have to examine each inference separately in the context of the passage and decide upon its degree of truth or falsity. (Punjab & Sind Bank PO Exam. 23.01.2011)

    Mark answer (1) if the inference is ‘definitely true’, i.e. it properly follows from the statement of facts given. Mark answer (2) if the inference is ‘probably true’ though not ‘definitely true’ in the light of the facts given. Mark answer (3) if the ‘data are inadequate’ , i.e. from the facts given you cannot say whether the inference is likely to be true or false. Mark answer (4) if the inference is ‘probably false’, though not ‘definitely false’ in the light of the facts given. Mark answer (5) if the inference is ‘definitely false’, i.e. it cannot possibly be drawn from the facts given or it contradicts the given facts. Aviation is important, both as an industry in its own right and as a fa-

    BPRE–843

    cilitator for the success and competitiveness of other industries. The success of the aviation industry is also integral to national competitiveness, economic development and social progress. More importantly, aviation supports those new economy sectors on which India’s future prosperity relies, like information technology, biotechnology, finance and electronics. That India’s air transport industry is on the rise is an undisputable fact. However the Indian economy will only achieve its true potential if we are able to compete in the industries of the future. We need to be strong in the areas with potential to grow and the role of India’s aviation sector is critical in this regard. 6. India needs to make massive investments in infrastructural development. 7. Growth in India’s aviation sector is important to its economic development. 8. India has made substantial development in the biotechnology sector. 9. India has tremendous potential to grow in the emerging industries. 10. Aviation sector in India has not fulfilled its role in the past in its economic development. 11. Read the following information carefully and answer the question which follows. One of the main reasons behind the lack of applicants for teachers’ training / degree programmes is that teachers have not experienced any improvement in working conditions and their salaries have not kept pace with salaries in other professions. Which of the following can be inferred from the given paragraph? (1) Very tough entrance exam is also one of tile reasons behind plunging number of applicants for teachers’ training programmes.

    CONCLUSIONS/INFERENCES (2) In the years to come, the schools would face a crunch in terms of availability of qualified teachers (3) Training programmes for other professions are also as good as teachers’ training programmes (4) Number of applicants for teachers’ training programmes will improve if the salaries in other professions are reduced (5) No direct relationship can be established between the work conditions of a particular profession and preference for it amongst the qualified candidates (Bank Of Baroda PO Exam. 13.03.2011)

    12. Read the following information carefully and answer the question which follows: Get more involved in our real world. Food inflation is a fantastic way to teach our children mathematics. Try having your child plot a graph of how incomes have grown in India for different jobs between, say, 1950 and today (engineer, doctor, maid, driver). Have them plot how costs have grown in the same period; it will be a fun exercise. By the time they finish the exercise, not only will your children learn a lot, but likely you will realize too that while our incomes have zoomed 300 times in this period, salaries of maids have probably gone up only 50 times, and costs have gone up 100 times. Your maid still has a seven-day week, with no paid vacation or casual leaves. And realizing just how lucky we are is often the first step towards serious philanthropy. All the following can be inferred from the above except for one: (1) Real world problems are a good way to understand other subjects as well as have appreciation for the real world problem (2) Food inflation has affected very differently different professionals over the years (3) Though there are huge salary differences, for all the

    jobs/professions the increase has atleast not been less than the rate of inflation over a longer period of time (4) When we realize that we have been lucky, we tend to think about philanthropy and helping those who have not been so lucky (5) Not only children, even the parents and other adults need to understand and sensitise themselves about the inequality (Bank Of Baroda PO Exam. 13.03.2011)

    Directions (13–16) : Read the following information and five statements given below it carefully and answer the questions which follow : (Indian Overseas Bank PO Exam. 22.05.2011)

    People in the remote village of Kenal are now totally self sufficient and liberal and no longer wait till people from urban areas dish out empowerment as a special privilege or favour. (A) Many of the residents formed a group and initiated a self help micro-credit program which provided help and support to the villagers to start a new business. (B) There was a time when the villagers had to seek favours from their urban counter parts. (C) Kenal lacks the new advancements and technological developments owing to a lack of urban intervention. (D) Many other villages are now approaching the people from Kenal to help them follow the same directions. (E) Women in Kenal too are realizing the importance of empowerment and are taking every possible step to add to their family’s kitty. 13. Which of the statements numbered (A), (B), (C), (D) and (E) can be inferred from the given statement? (An inference is something which can be directly inferred from the given facts) (1) Only A (2) Only B (3) Only C (4) Only D (5) Both A and E 14. Which of the statements numbered (A), (B), (C), (D) and (E)

    BPRE–844

    mentioned above would prove that intervention from their urban counterparts can also be beneficial to a certain extent? (1) A (2) B (3) C (4) D (5) E 15. Which of the statements numbered (A), (B), (C), (D) and (E) mentioned above represents a step which helped the process of liberalization and empowerment of the villagers? (1) A (2) B (3) C (4) D (5) E 16. Which of the following represents a result / repercussion of the success experienced by the residents of the village Kenal? (1) B (2) C (3) Both C and E (4) A (5) D 17. A few travellers were severely beaten up by villagers recently in a remote rural part of the state as the villagers found the movement of the travellers suspicious. The district authority has sent a police team to nab the culprits. Which of the following inferences can be drawn from the above statement? (An inference is something which is not directly stated but can be inferred from the given facts) (1) The villagers dislike presence of strangers in their vicinity. (2) Villagers are generally suspicious in nature. (3) Travellers prefer to visit countryside. (4) The Government generally provides protection to travellers across the country. (5) None of these (IBPS Bank PO/MT CWE Exam. 18.09.2011)

    Directions (18–22) : Read the following information carefully and answer the questions which follow: (IBPS Bank PO/MT CWE Exam. 17.06.2012)

    Small brands are now looking beyond local grocery stores and are tying up with Supermarkets such as Big Bazaar to pull their business out of troubled waters.

    CONCLUSIONS/INFERENCES 18. Which of the following can be inferred from the given information? (An inference is something that is not directly stated but can be inferred from the given information) (1) Merchandise of smaller brands would not be available at local grocery stores in the near future. (2) Smaller brands cannot compete with bigger ones in a supermarket set-up. (3) There is a perception among small brands that sale in a supermarket is higher than that of small grocery stores. (4) Supermarkets generate more revenue by selling products of bigger brands as compared to the smaller ones. (5) Smaller brands have always had more tie-ups with supermarkets as compared to small grocery stores. Directions (19–22) : These questions are based on the information given above and the sentences labeled (A), (B), (C), (D), (E) and (F) as given below. (A) A smaller brand manufacturing a certain product of quality comparable with that of a bigger brand, makes much more profit from the local grocery stores than from the supermarkets. (B) As the supermarkets have been set up only in bigger cities at present, this step would fail to deliver results in the smaller cities. (C) Supermarkets help the smaller brands to break into newer markets without investing substantially in distribution. (D) Supermarkets charge the smaller brands 10% higher than the amount charged to the bigger brands. (E) Being outnumbered by the bigger brands, visibility of the smaller brands at local grocery stores is much lower as compared to the supermarkets. (F) Smaller brands are currently making substantial losses in their businesses.

    19. Which of the statements numbered (A), (B), (C), (D), (E) and (F) can be assumed from the facts/ information given in the statement ? (An assumption is something supposed or taken for granted) (1) Only (A) (2) Only (B) (3) Both (B) and (C) (4) Both (D) and (E) (5) Only (F) 20. Which of the statements numbered (A), (B), (C), (E) and (F) represents a disadvantage of the small grocery stores over the Supermarkets from the perspective of a smaller brand ? (1) Only (A) (2) Only (C) (3) Only (E) (4) Only (F) (5) Both (B) and (C) 21. Which of the statements (A), (B), (C), (D) and (E) mentioned above represents a reason for the shift from local grocery stores to supermarkets by the smaller brands ? (1) Only (A) (2) Only (B) (3) Only (D) (4) Both (A) and (D) (5) Both (C) and (E) 22. Which of the statements numbered (A), (B), (C), (E) and (F) mentioned above would prove that the step taken by the smaller brands (of moving to supermarkets) may not necessarily be correct ? (1) Only (A) (2) Only (C) (3) Only (E) (4) Only (F) (5) Both (B) and (E) Directions (23–27) : Below is given a passage followed by several possible inferences which can be drawn from the facts stated in the passage. You have to examine each inference separately in the context of the passage and decide upon its degree of truth or falsity. (IBPS Specialist Officer CWE Exam.17.03.2013)

    Mark answer (1) If the inference is “definitely true” i.e. it properly follows from the statement of facts given. Mark answer (2) If the inference is “probably true” though not “definitely true” in the light of the facts given.

    BPRE–845

    Mark answer (3) If the data is inadequate i.e. from the facts given, you cannot say whether the inference is likely to be true or false. Mark answer (4) If the inference is “probably false” though not “definitely false” in the light of the facts given. Mark answer (5) If the inference is “definitely false” i.e. it cannot possibly be drawn from the facts given or it contradicts the given facts. (Note : Each of the five questions has only one distinct answer i.e. no two questions can have the same answer. If you get the same answer for more than one question, consider both again and decide which one of the two would more definitely be that answer and in the same way review the others also.) Cardiovascular disease is so prevalent that virtually all businesses are likely to have employees who suffer from, or may develop, this condition. Research shows that between 50–80 per cent of all people who suffer a heart attack are able to return to work. However, this may not be possible if they have previously been involved in heavy physical work. In such cases, it may be possible to move the employee to lighter duties, with appropriate retraining where necessary. Similarly, high-pressure, stressful work, even where it does not involve physical activity, should also be avoided. Human Resource managers should be aware of the implications of job roles for employees with a cardiac condition. 23. Employees who suffer from cardiovascular disease are mostly unable to return to work. 24. Employees suffering from Cardiovascular diseases are unable to handle stressful situations. 25. Employees above the age of 50 are found to suffer from cardiovascular disease. 26. Physical and stressful work definitely leads to a heart attack. 27. Heart disease can affect employees in any type of business. 28. Statements : There are several types of wrist watches in the world. A new type of digital wrist watch has been introduced in the market which can calculate the life span of a person. The wrist watch stops when the person dies.

    CONCLUSIONS/INFERENCES Which of the following can be concluded from the facts given in the statements ? (1) This could lead to chaos and confusion in the world (2) Person can know how long life is left and he could live his life happily (3) The new wrist watch has altered the life pattern of the people (4) The statement implies that death is certain (5) None of these (IBPS Bank PO/MT CWE-III, 26.10.2013)

    29. Statement : Experts define the social entrepreneurship as such that pursues opportunities to serve the mission to give solutions to social problems. Business entrepreneurs typically measure performance in profit and return. Which of the following can be concluded from the above statements ? (1) It is possible to address social problems while making business profitable. (2) Social entrepreneurship is different from the business entrepreneurship. (3) Business entrepreneurship does not care social problems while devising their business policy. (4) Business entrepreneurship is more important than social entrepreneurship. (5) None of these (IBPS Bank PO/MT CWE-III, 26.10.2013)

    30. This question consists of an information followed by two statements numbered I and II. The travel authorities of country Talong have eased the process of acquiring visa for the tourists. Statement I : The economy of country Talong is more dependent of Tourism industry than any other. Statement II : Students will be encouraged to go for higher studies to Talong. Which of the given two statements can be inferred from the given information ? (An inference is something which is not directly stated but can be inferred from the given facts.)

    (1) Neither I nor II can be inferred from the given information. (2) Both I and II can be inferred from the given information. (3) Either I or II can be inferred from the given information. (4) Only I can be inferred from the given information. (5) Only II can be inferred from the given information. 31. This question consists of an information followed by two statements numbered I and II. The travel authorities of country Talong have eased the process of acquiring visa for the tourists. Statement I : The economy of country Talong is more dependent of Tourism industry than any other. Statement II : Students will be encouraged to go for higher studies to Talong. Which of the given two statements can be inferred from the given information ? (An inference is something which is not directly stated but can be inferred from the given facts.) (1) Neither I nor II can be inferred from the given information. (2) Both I and II can be inferred from the given information. (3) Either I or II can be inferred from the given information. (4) Only I can be inferred from the given information. (5) Only II can be inferred from the given information. (BOB Junior Management Grade/Scale–I Exam. 18.04.2015) (Bank of Baroda Junior Management Grade/Scale-I Exam, 18.04.2015)

    32. Statement % The ministry of aviation has ordered to procure 35 new aircraft to be added to the existing fleet this year. Which of the following most appropriately proves that the decision taken by the aviation minister is unrealistic and not based on scientific projections? (1) A recent survey showed that the existing aircraft in the fleet fail to match the international standards and quality norms.

    BPRE–846

    (2) These new aircraft have better technology and greater number of seats as compared to ones already existing in the fleet. (3) The neighbouring country which also procured aircraft lacked enough number of trained pilots. (4) As the number of passengers has declined significantly this year, the existing fleet is not being used to its full potential. (5) As many as 12 pilots are not on flying duties for two straight weeks due to lack of sufficient aircraft to ply on the routes. (IBPS Bank PO/MT CWE–V Main Exam. 31.10.2015)

    33. Study the given information and answer the question : Long term usage of antibiotics causes the disease, ‘Cretosis’ as it decreases the secretion of hormone X. While body can endure the level of hormone X dropping to half the required number micrograms, in levels dropping to 23–micrograms needs immediate medical attention. Which of the following can be concluded from the given statement? (1) A patient can be said to have Cretosis only if his/her hormone ‘X’ levels are 23 micrograms of low. (2) Usage of antibiotics on a short term cannot cause ‘Cretosis’ ever to a minor extent. (3) All micrograms is exactly half of the amount of hormone X required daily by the body. (4) The normal numbers of micrograms of hormone X is more than all micrograms. (5) In a patient with hormone X level of 21 microgram, if administered another 23 micrograms would bring the level to absolute normal. (IBPS Specialist Officer (Marketing) CWE 01.02.2016)

    34. Study the given information carefully to answer the given question : The following are two findings of a one year long survey conducted on the employees of Company K.

    CONCLUSIONS/INFERENCES (A) Every time, an employee is rewarded for his/her work, he/she has performed better for at least next two years. (B) The performance of none of the employees of Company K has improved in the past ten months. Which of the following can be inferred from the given information? (1) More than 90% of Company K’s employees work to their highest potential only when they are rewarded. (2) All the employees of Company K have been rewarded at least once in their work span. (3) The employees receive heavy incentives apart from being rewarded which drive them to perform better. (4) There are factors other than being rewarded which affect the performance of the employees. (5) No employees has been rewarded for his/her work in the last two quarters. (IBPS Specialist Officer (Marketing) CWE 01.02.2016)

    35. Study the given information carefully and answer the question given below : ‘Ever since we started involving our employees in the key decision making process from the last year, the productivity of our employees has been 100%’ - statement by HR manager of Company D. Which of the following statements can be inferred from the given statement ? (An inference is something by which you can logically deduce something to be true based on known premises.) (1) Employees of Company D will always take correct decisions in all the situations. (2) Company D completely relies on its employees for all key decisions. (3) The productivity of employees of Company D has always been satisfactory in the past. (4) Involving employees in decision making process is an effective method to motivate them. (5) Employees of Company D have now become competent enough to give valuable suggestions. (IBPS Specialist Officer (IT) CWE 14.02.2016)

    36. Study the given information carefully to answer the question given below : ‘Kalikaar’ is a famous fabric threadwork that originated in City G of a country. (A) ‘Kalikaar’ was an innovation of ‘Hardali’ tribe of City G. Though the tribe lost its existence long ago, ‘Kalikaar’ is still very much in demand. (B) Only the authentic ‘Kalikaar’ depicts fruits and flowers of a specific tree that was worshipped by ‘Hardali’ tribe. Which of the following can be inferred from the given information ? (Note : An inference is something by which you can logically deduce something to be true based on known premises.) (1) The ‘Hardali’ tribe did not worship Gods and Goddesses. (2) Some people not belonging to the tribe have taken the ‘Kalikaar’ work forward and managed to keep fulfilling its demand. (3) During the existence of ‘Hardali’ tribe, ‘Kalikaar’ was the only means of their income. (4) No city other than city G sells authentic cloth materials of ‘Kalikaar’ threadwork. (5) Since not many people belonged to the ‘Hardali’ tribe, others came to know about the tribe only through its innovation of ‘Kalikaar’. (IBPS Specialist Officer (IT) CWE 14.02.2016)

    37. Study the following information carefully and answer the question given below : Statement : Company X has approached the Government with a proposal to set up a school in the village ABC as the children of this village go to another village for study. Which of the following statements can be inferred from the given statement? (An inference is something by which you can logically deduce something to be true based on known premises)

    BPRE–847

    (1) The Company X has proposed to recruit a team of skilled teachers and made a strong case for why a new school is needed in the village ABC. (2) The Company X considered the difficulty faced by the children of village ABC in attending school in another village. (3) The Company X is a great industry house but it is not interested in undertaking Corporate Social Responsibility programmes (4) Nowadays, setting up a school in the remote areas is very profitable activity (5) The Government has not enough resources to open up a school in the village ABC. (United Bank of India PGDBF Manipal Exam,07.08.2016)

    38. Study the following information carefully and answer the question given below : “The upcoming marathon seems to be an impossible task for me. I can only run 5 km at a stretch. Even if I double that I would still not reach half way through the marathon” — Rohan. Which of the following statements can be inferred from Rohan’s statement? (1) Any runner who can complete the marathon can definitely finish first five kilometres of the marathon faster than Rohan. (2) If Rohan runs four times his capacity, he would finish the marathon. (3) Had the specified time for marathon been one hour, Rohan would have completed it in two hours’ time. (4) The specified distance of the upcoming marathon is definitely more than 20 km. (5) If Rohan runs 5 more kilometres, he will reach exactly half way through the marathon’s specified distance. (Indian Bank PO (Pre.) Exam, 21.01.2017 (Ist Sitting))

    39. Read the following information carefully and answer the question given below : Owing to curfew in some parts of city Seekat, the exam venue for

    CONCLUSIONS/INFERENCES many candidates was changed to Beehal, three days before the exam. Therefore, in Beehal, out of all the candidates, nearly 90% comprised of the ones from Seekat and only 10% had originally registered to appear from Beehal. On the exam day, however, the overall attendance in Beehal (of candidates originally from Beehal as well as shifted from Seekat) was only 23%. In parts of Seekat where there was no curfew, 1400 candidates appeared in the exam. Which of the following can be inferred from the given statement ? (1) Both Beehal and Seekat are definitely located in the same state. (2) Total number of candidates who appeared for exam in Beehal was 1070 more than that in Seekat. (3) Number of candidates present on the exam day in Beehal was definitely less than 1400. (4) Many candidates whose venue was changed from Seekat to Beehal did not appear in the examination (5) The venue of more than 1400 candidates was changed from Seekat to Beehal. (Indian Bank PO (Pre.) Exam, 21.01.2017 (2nd Sitting))

    40. Statement: “Company X has approached they proposal to set up an emergency unit in the only hospital of Village Y.” Which of the following could possibly lead the Government to accept the proposal of Company X ? (A) The emergency cases that come to the hospital of Village Y are not much in number. (B) At present there is no emergency unit in the hospital of Village Y where people can visit in case of emergencies. (C) The emergency unit in one of the nearby hospitals, where people of Village Y visit is overcrowded most of the times. (1) Both B and C (2) Only B (3) Both A and B (4) All A, B and C (5) Only C (Bank of Baroda Exam, 25.09.2016)

    41. Study the following information and answer the question given below : Continuous use of sanitizers should be avoided among young children as they ingest a certain quantity of sanitizer with every use which can increase the per cent content of chemical A in their body to as high as 0.04%. Which of the following can be concluded from the given statement? (A) Chemical A is either present in sanitizers or is formed in the body with the help of an agent present in the sanitizers. (B) Adults and older children are not affected by the sanitizers if ingested in small quantities. (C) Percentage of chemical A in the body under normal conditions is zero. (D) 0.03% of chemical A in the body does not lead to any pathological conditions. (1) Only (C) (2) Both (B) and (D) (3) Only (A) (4) Both (A) and (C) (5) Both (A) and (D) (Bank of Baroda Exam, 25.09.2016)

    42. Study the following information carefully and answer the question given below : (IBPS SO (IT Officer) CWE (Prelim Exam) 30.12.2017)

    “ A variety of duplicate products have snatched our market. Our profits are reducing by almost 10% each passing year. Despite much longer durability of our handmade products, people these days are settling for the second copies only because of their prices and easy availability”- statement by the owner of a handmade products store in City Y. Which of the following can be inferred from the owner’s statement? (An inference is something which you can logically deduce based on the given facts) (1) The market of handmade products will dissolve completely in the coming few years. (2) Manufacturing of second copies or duplicate articles is a recent trend in City Y.

    BPRE–848

    (3) Reducing the price of handmade articles by 10% will attract more customers. (4) Making handmade products more widely available at reduced prices will help its market revive atleast to some extent (5) Durability is the least important factor that affects the choice of customers while buying such items. 43. Study the following information carefully and answer the question given below: Bus no. 218 is the only bus which plies on the only route between localities A and G of city Haroya. There are three huge factories between these areas. Though the bus has always been very punctual and well maintained, last week, it was decided to discontinue the bus for certain reasons. Which of the following cannot be reason behind discontinuing Bus no. 218? (1) Soon AC buses will start plying between localities A and G, whose fare will be just 2% more than that of Bus no. 218. It is anticipated that Bus no. 218 will not make much profit thereafter. (2) All the given options can be possible reasons for discontinuing the bus. (3) It has been decided to increase the number of e-rickshaws plying between areas A and G by 40% hence making them more easily and frequently available to the customers as compared to the bus. Also, as a result, the additional pollution caused by buses in these areas can brought down to a great extent. (4) Under the recently completed metro rails project of Haroya, there are more frequent trains between localities A and G as compared to the frequency of Bus no. 218. The metro fare is also lower than that of the bus. (5) The number of chain snatching and pick-pocketing cases near locality G of Haroya have drastically increased. As a result, Bus no. 218 has considerably lesser passengers now. 44. Study the following information carefully to answer the given question.

    CONCLUSIONS/INFERENCES “The attrition rate of our company has come down significantly in the past one year as compared to last three years” – Statement by HR head of Company X. Which of the following may be a reason for the reduced attrition rate of employees in Company X? (A) The CEO along with the general board of Company X revised its promotion policies this year and had reduced the tenure of promotion from five years to three years. (B) The governing board has sanctioned the annual increment to be increased by 34% for all employees. (C) The number of working days for employees have been reduced from six to five. (D) The monetary benefits are reduced to Rs. 40,000 annually from an amount of Rs. 70,000 per employee. (1) Only A, B and C (2) Only A, C and D (3) Only A and B (4) Only B and D (5) Only C and D (IBPS SO (Law Officer) CWE (Prelim Exam) 31.12.2017)

    45. Study the following information carefully to answer the given question : ‘To increase the sales our new product G, a free gift item worth Rs. 100/- should be made available with the product to attract the customers for the next fifteen days”- Statement by Marketing Head of Company P after one week of launch of product G in the market. Which of the following can be inferred from the statement given by Marketing Head of the Company P? (1) None can be inferred (2) The cost of production of 10 units of product G is definitely less than Rs. 1000. (3) Giving free products with any product will necessarily increase its sales substantially. (4) Company P will be able to make sufficient profits with the sales of product G during next fifteen days only.

    (5) Ever since the product has been launched by the company in the market the sale was below expectation. (IBPS SO (Law Officer) CWE (Prelim Exam) 31.12.2017)

    46. Study the following information carefully to answer the given question : ‘Galax fibre’ is manufactured only in Country A. Though its demand across the world has remained more or less same since the past three years, the profits earned from the fibre has decreased by 20% in the last 8 months causing an overall loss of around Rs. 35 crores. Country A has never made such a huge loss on Galax fibre ever. Which of the following can be inferred from the given statement? (1) The companies that manufacture Galax fibre will have to shut down owing to losses. (2) Galax fibre is exported to other countries at a very high price. (3) Had there been no loss, Country A would have made the highest profit ever through Galax fibre. (4) No other country has adequate machinery to manufacture Galax fibre. (5) Factors other than demand of a commodity also affect its sales and profits. (Canara Bank PO Exam 04.03.2018)

    47. Study the following information carefully and answer the question given below : High amount of ‘trinium’ blocks the absorption of ‘roassium’ by the plants thereby crippling them. Thus, in Village R, the yield of jute is adversely affected. In order to address this situation the use of ground water for irrigation should be completely stopped in the Village R. Which of the following statements CANNOT be concluded from the given information? (A) Jute plants need roassium in order to grow well. (B) The groundwater in Village R contains high amount of ‘trinium’. (C) The groundwater in Village R contains ‘roassium’ to atleast some extent.

    BPRE–849

    (D) For the jute crop to grow well, the concentration of ‘trinium’ in the soil should essentially be zero. (1) Both (C) and (D) (2) Only (D) (3) Both (A) and (D) (4) Only (C) (5) Both (B) and (D) (Canara Bank PO Exam 04.03.2018)

    48. Study the following information carefully to answer the given question : Mallika is a renowned TV artist since past 3 years. She always gets her dresses for all events only from ‘Marvel fashion designers’ based in City X. Recently she opted to wear a dress from a very new designer in the market instead of ‘Marvel fashion designers.’ She had to pay almost double the price of what she usually pays to ‘Marvel fashion designers’. Which of the following cannot be a possible reason behind Mallika’s decision? (1) Unlike the employees of ‘Marvel fashion designers’, those under the new fashion designer have studied in renowned foreign universities specializing in fashion designing. (2) Mallika wore the dress for an international event where world’s best casting directors had come. She wanted to wear something which was unique and she found it with the new designer. (3) The clothes provided by the new fashion designer are of a very superior quality and latest trends which no other designer in the city has. (4) Mallika recently had a fight off with ‘Marvel fashion designers’ for providing the same dress as Mallika’s to another TV star for the same event. (5) All of the given options can be possible reasons. (Canara Bank PO Exam 04.03.2018)

    49. This question consists of a sit uation followed by two statements numbered I and II given below it. Which of the given statements can be a possible reason for the given situation?

    CONCLUSIONS/INFERENCES Situation : “The CEO of an online gift shopping app ‘Gifzo’ has announced to send complimentary chocolate for orders above Rs. 700 and personalized coffee mugs for orders above Rs. 1500 for the next one month” — News article on 23rd February this year. Statement I : Since the inception of Gifzo in March four years ago, it has always made sure to thank its customers on its anniversary with special gestures. Statement II : ‘Gifzo’ launched personalised gifts on 19th February this year, but has only received 100 orders for the same till date. (1) Either I or II can be a possible reason. (2) Only I can be a possible reason. (3) Both I and II can be possible reasons. (4) Only II can be a possible reason. (5) Neither I nor II can be a possible reason. (Canara Bank PO Exam 04.03.2018)

    50. This question consists of a situation followed by two statements numbered I and II given below it. Which of the given statements can be a possible reason for the given situation? Situation : ‘Olivia, a sugar producing factory closed down its operations, facing a loss of Rs. 200 crores.’— a report. Statement I : Olivia was unable to generate enough business from past two years thus losing its clients. Statement II : Olivia was unable to provide full salary to its workers since past two years following which most of the workers resigned leaving only five at present. (1) Only II can be a possible reason. (2) Neither I nor II can be a possible reason. (3) Either I or II can be a possible reason (4) Both I and II can be possible reasons. (5) Only I can be a possible reason. (Canara Bank PO Exam 04.03.2018)

    51. Study the following information carefully to answer the given question : The day we purchased and installed five more machines we were able to cut atleast 1000 more sheets daily thus making the production three times faster on weekly basis, working days being Monday to Saturday’ — Statement by Manager of EVT Metallic Which of the following can be inferred from the given information? (A) After installation of the said machinery, EVT Metallic Works Ltd. was able to produce atleast 6000 sheets more weekly. (B) EVT Metallic Works Ltd. had sufficient funds to purchase five machines simultaneously. (C) Atleast some employees of EVT Metallic Works Ltd. were trained to work on the new machines (D) Previously EVT Metallic Works Ltd. produced 2000 sheets weekly. (1) Only (A) and (D) (2) Only (D) (3) Only (B) and (C) (4) Only (A), (B) and (C) (5) All (A), (B), (C) and (D) (Canara Bank PO Exam 04.03.2018)

    52. Statements : A. Government has recently raised GST slab on few items that are imported from China and Korea in the middle of the financial year. B. In the first quarter of the financial year, few products from Indian manufacturers had declined as per RBI report. Which of the following can be inferred from the above statements? (1) Indian consumers prefer few products that are from China or Korea (2) GST can be used to control the purchasing power over few products (3) The Government has observed the economic plan of the year in the 1st quarter before raising GST slab (4) When consumers purchase more Chinese products, it automatically increases the rate of tax in India (5) None of these (IBPS Bank PO/MT CWE (Main Exam) 26.11.2017)

    BPRE–850

    Directions (53–54) : Study the following information carefully and answer the questions given below : The state government of Tamil Nadu had passed an order on May 23, 2006 allowing appointment as ‘archaka’ any Hindu possessing the requisite qualification and training in temples. Followed by this legislation, the famous Godess Meenakshi Amman temple in Madurai has got a non-Brahmin as ‘archaka’ (priest), 12 years after the Tamil Nadu government passed an order for appointment of people from any caste as priests. The Meenakshi Amman temple has appointed the priest with conditions which included that he could be suspended if he violated the ‘Agama’ rules, a manual for worship of the deity, rituals and construction of temples. These questions are based on the information given above and the statements given below labelled as I, II, III, IV and V. I. The person other than Brahmin can be a priest if he is willing to take a training to be a priest. II. A non Brahmin person can become a priest only if he is well trained in temple Agma rules. III. If a person wants to be trained as a priest then he should spend quite few years to master the Agma rules. IV. The temples other than Meenakshi temple may not appoint other caste people as Archaka V. After a legislation, it is not necessary that the law should be executed immediately. (IBPS Bank PO/MT CWE (Main Exam) 26.11.2017)

    53. Which of the following statements could be a possible Conclusion from the above passage? (1) Only II (2) Both I and II (3) Either I or II (4) Both I and V (5) Only I 54. Which of the following statements could be possible Assumptions from the above passage? (1) Only II (2) Either II or III (3) Either III or IV (4) Both II and V (5) Both III and V

    CONCLUSIONS/INFERENCES 55. Statements : Petrol prices in country Z hovered around Rs. 73 per litre for a long time. Economic downturn in Country A, however, has led to an increase in this price by more than 63% throughout country Z. I. Country Z imports fuel form counry A. II. Economic downturn negatively affects the oil industry of country A. (1) Only I can be conclude from the given information. (2) Neither I nor II can be concluded from the given information. (3) Both I and II can be concluded from the given information. (4) Either I or II can be concluded from the given information. (5) Only II can be concluded from the given information. (IBPS Bank PO/MT CWE (Main Exam) 26.11.2017)

    56. Indore and Bhopal, both in Madhya Pradesh, have emerged as the cleanest cities in the country as per a massive cleanliness survey in 2017, commissioned by the Union Urban Development Ministry. Which of the following can be deduced from the given statement? (I) There cannot be absolute comparison between the rankings of 2016 and for 2017 since the parameters have been tweaked. (II) All the other cities do not have the potential to be cleanest cities. (III) Other cities are not following Swachh Bharat Abhiyan. (1) Only I and II (2) Only III (3) Only II and III (4) All of the above (5) None of these (IBPS Specialist Officer CWE (Prelim Exam) 31.01.2019)

    57. Indian Prime Minister Narendra Modi has vowed his country will go “above and beyond” the 2015 Paris accord on combating climate change.

    Which of the following can be inferred from the above statement? (I) India is not in a condition to accept the Paris accord. (II) India has made a significant progress since 2015. (III) India understands the impact of carbon emissions on climate change and is willing to do more on the issue. (1) Both (I) and (III) (2) Both (II) and (III) (3) All (I), (II) and (III) (4) Only (III) (5) None of these (IBPS Specialist Officer CWE (Prelim Exam) 31.01.2019)

    Directions (58 – 59): Consider the following statements in each question and select an appropriate INFERENCE from the choices given below the statement. An Inference is a range of truth pertaining to the given statement. (IBPS RRBs Officer Scale-I CWE Main Exam, 13.10.2019)

    58. Few people still listen to the radio programs instead of downloading songs. They do not want to miss the excitement of accidentally listening to their favourite songs. A. Some people have different means of entertainment. B. Most people are excited listening to the downloaded songs C. Downloaded songs may not excite the people D. Excited people listen to the radio songs (1) Only A and B (2) Only B and C (3) Only C and D (4) Only A and C (5) Only B and D 59. Rattan is the managing director of XYZ Ltd. This is the third year of his tenure and he has gained a lot of expertise but the company is forcing him to change his department in which he would be having a complete new experience and higher salary. A. Rattan thinks that it is difficult to gain expertise in another department

    BPRE–851

    B. Rattan is more capable and accurate in the existing department C. Company has appointed another person for the existing department D. Company is confident that Rattan would do better in new department (1) Only A and B (2) Only A and C (3) Only B and C (4) Only B and D (5) Only C and D 60. Study the following information carefully and answer the given question : Our brains are designed in such a way that they seek rewards which comes in the form of extra ‘Lomawix’ (a chemical in the brain that makes us feel good). In primitive days, hunting or finding food gave a huge emotional reward as it helped to feed oneself and families. Though the world has changed, a part of brain is still primitive and it still seeks rewards. A lot of industries – alcohol, tobacco, gambling etc are making the most of it. Though there are a lot of people who will get that extra ‘Lomawix’ from certain activities and yet lead normal lives, but not everyone. They get addicted and do not find normal activities rewarding. Though addiction is treatable, it can rarely be uprooted. Four of the following five are either assumptions or inferences drawn from the above information. Which one of the following can neither be inferred nor be assumed from the given information? (1) Brains of people who are addicted to alcohol, tobacco, gambling etc. gradually become used to higher levels of ‘Lomawix’ and cannot function normally with family. (2) All of the given options can either be inferred or be assumed from the given information.

    CONCLUSIONS/INFERENCES (3) If a person who is addicted to something, undergoes treatment and/or therapy, it is not necessary that in future he/she will not get addicted to the same thing or some other thing. (4) Artificially administering ‘Lomawix’ into the body of a person who is addicted to something, can bring down the addiction to the activity (previously addicted to) atleast to some extent. (5) In primitive days not many people were addicted to alcohol, tobacco or gambling as most of them were hunters and thus had higher levels of ‘Lomawix’. (IBPS SO Prelim Exam, 28.12.2019)

    61. Study the following information carefully and answer the given question : Frequent use of ‘mosquito repellant roll-ons’ should be avoided among infants as they inhale a certain quantity of the repellant with every use which can increase the percent content of chemical A in their body to as high as 0.18% leading to serious pathological conditions. Which of the following can be concluded from the given statement? A. Chemical A is either present in mosquito repellant roll-ons or is formed in the body with the help of an agent present in those rollons. B. Adults and older children are not affected by the sanitizers if ingested in small quantities. C. Higher percentage of Chemical A in the body does not lead to any pathological conditions. D. 0.15% of Chemical A in the body does not lead to any serious pathological conditions. (1) Only (A) (2) Only (C) (3) Both (A) and (C) (4) Both (A) and (D) (5) Only (D) (IBPS SO Prelim Exam, 28.12.2019)

    SBI PO EXAMS Directions (1–4) : Study the following information carefully and answer the given questions : (SBI Management Executive Exam, 23.02.2014)

    Following are the conditions for selecting candidates for Post Graduation Diploma in Marketing in an institution. The candidate must (i) have a Graduation Degree with at least 50 percent marks (ii) be at least 22 years as on 01.12.2013 (iii) have secured at least 40 per cent marks in the Entrance Test (iv) have secured at least 50 per cent marks in the Group Discussion and Interview (v) be capable to pay a fee of Rs. 4 lakh per annum at the time of admission. In the case of a candidate who fulfills all the conditions EXCEPT (a) at (iii) above but has secured 30 per cent marks in the Entrance Test and 80 per cent marks in the Graduation, his/her case is to be referred to the Dean of the Institution (b) at (v) above but can deposit an amount of Rs. 2.5 lakh in the beginning of the first year, his/ her case is to be referred to the Director of the Institution. In each question below, details of one candidate are provided. You have to take one of the following courses of actions based on the conditions given above and the information provided in each question and mark the number of that course of action as your answer. You are not to assume anything than the information provided in each question. All these cases are given to you as on 01.12.2013. Marks answer (1) if the case is to be referred to the Dean of the Institution. Marks answer (2) if the case is to be referred to the Director of the Institution. Marks answer (3) if the candidate is to be selected Marks answer (4) if the candidate is not to be selected

    BPRE–852

    Marks answer (5) if the data provided are inadequate to take a decision. Now read the information provided in each question and mark your answer accordingly. 1. Anup Bhatnagar was born on 12th June, 1989. He can deposit an amount of Rs. 4 lakh at the time of admission. He secured 85 per cent marks in the Graduation and 55 per cent marks in the Group Discussion and Interview. He secured 35 per cent marks in the Entrance Test. 2. Nisha Desai was born on 8th April, 1991. She secured 80 per cent marks in the Graduation and 60 per cent marks in Group Discussion and Interview. She can pay an amount of Rs. 4 lakh per annum. 3. Subodh Verma was born on 22nd May, 1990. He secured 50 per cent marks in the Graduation and 40 per cent marks in the Entrance Test. He secured 60 per cent marks in Group Discussion and Interview and can deposit an amount of Rs. 4 lakh as fee. 4. Ravindra Sharma was born on 24th September, 1988. He secured 60 per cent marks in the Entrance Test, 60 per cent marks in the Graduation and 65 per cent marks in the Group Discussion and Interview. He can pay Rs. 3 lakh in the beginning of the first year. 5. Rajiv Batra secured 70 per cent marks in the Entrance Test and was born on 5th June, 1991. He secured 50 per cent marks in the Graduation and can pay Rs. 4 lakh at the time of admission. He secured 60 per cent marks in the Group Discussion and Interview. Directions (6–7) : Each of the following questions consists of a statement followed by two statements numbered I and II given below it. You have to decide which of the given statements numbered I and II weakens or strengthens the statement and mark the appropriate answer. (SBI Associates PO Online Exam, 30.11.2014)

    CONCLUSIONS/INFERENCES 6. Statement % The world should move towards using bio-fuels in the future as these are more environment friendly as compared to regular fossil- fuels. I. While bio-fuels cleaner to burn, the process to produce the fuel, including the necessary machinery leads to very high carbon emission II. As bio-fuels are manufactured from materials such as crop waste, manure and other by products, these, unlike fossil-fuels, are easily renewable. (1) Statement I weakens the information while statements II is a natural statement. (2) Both statements I and II weaken the information. (3) Statement I strengthens the information while statement II weakens the statement. (4) Statement I weakens the information while statement II strengthens the statement. (5) Both statements I and II strengthen the information. 7. Statement % Always remain in an air conditioned environment for better health and well-being. I. Filters in the air-conditioners lower exposure to aller gy creating pollen and other outdoor allergies. II. Spending two much time in an air-conditioned environment result in gradual intolerance of body towards natural temperatures. (1) Statement I weakens the information while statements II is a natural statement. (2) Both statements I and II weaken the information. (3) Statement 1 strengthens the information while statement II weakens the statement. (4) Statement I weakens the information while statement II strengthens the statement. (5) Both statements I and II strengthen the information.

    8. Study the following information carefully and answer the question given below : A few travellers were severely beaten up by people of Country X after they found the movement of the travellers suspicious. The district authority has sent a police team to enquire about the same. Which of the following inferences can be drawn from the above statement? (An inference is something which is not directly stated but can be inferred from the given facts) (1) None of those given as options (2) Country X is the most preferred tourist destination. (3) People of Country X are generally suspicious in nature. (4) The Government of Country X generally provides protection to travellers across the country. (5) The people of Country X dislike presence of strangers in their vicinity. (SBI PO Phase-II (Main) Exam 04.06.2017)

    Directions (9–10) : In each question below is given a statement followed by two Inferences numbered I and II. An Inference is not directly given but can be inferred based on the facts presented. You have to consider the statement and the following Inferences and decide which of those is/ are implicit in the statement. (SBI PO Phase-II (Main) Exam 04.06.2017)

    9. Statement : A leading mobile manufacturer has re-introduced a new model of the existing mobile with modified and peppy looks but with the same processor leading to increase in sales. I. Mobile’s processor is not important as its looks. II. Looks of the mobile are one of the important factors to the buyers. (1) Both I and II are implicit (2) Either I or II is implicit (3) Only I is implicit

    BPRE–853

    (4) Only II is implicit (5) Neither I nor II is implicit 10. Statement : An advertisement in the newspaper–“Join our computer course to get a high paying job”. I. All those who join computer courses get highly paid jobs. II. Only a computer course can get someone a high paying job. (1) Both I and II are implicit (2) Only II is implicit (3) Only I is implicit (4) Either I or II is implicit (5) Neither I nor II is implicit

    RBI GRADE–B/ NABARD GRADE–A OFFICER EXAMS 1. Read the following information carefully and answer the question which follows. A recent report points to the fact that what stands between girls in village Regari and a good education is the lack of transport, to and from the only school in the area. Which of the following can be inferred from the given statement? (An inference is something which is not directly stated but can be inferred from the given facts.) (1) Everyone living in Regari faces problems in commuting (2) Discrimination against girls is more evident in Regari than other villages in India (3) The report had shown Regari in a poor light (4) The school in Regari is not at a walkable distance from the village (5) There is also a lack of institute for higher (earning in Rewari (RBI Grade-B Officer’s Exam. 18.12.2011)

    Directions (2–3) : In each question below is given a statement followed by two “Expectations” numbered I and II. An Expectation is something which can either be an objective or prospect or desired outcome

    CONCLUSIONS/INFERENCES or hope behind the action /statement. You have to consider the statement and the following Expectations and decide which of the Expectations is implicit in the statement. (RBI Officer Grade ‘B’ Online Exam. 25.08.2013)

    Give answer (1) if only Expectation I is implicit. Give answer (2) if only Expectation II is implicit. Give answer (3) if either Expectation I or II Expectation II is implicit. Give answer (4) if neither Expectation I nor Expectation II is implicit. Give answer (5) if both Expectations I and II are implicit. 2. Statement : A promotional campaign – For healthy children encourage them to play in the playgrounds rather than video games at home. Expectations : I. Health of atleast some children would improve following this campaign. II. Most of the parents would not buy video games for their children after this promotional campaign. 3. Statement : ‘We should export the surplus of wheat in order to utilize its over-production this year’. Expectations : I. Knowing that the exported wheat is a part of the surplus of the total production, many countries would not pay the desired amount for it. II. Some countries are willing to import wheat. 4. Expert A says that dinosaurs became extinct due to climatic changes occurred on the Earth due to volcanic eruptions some 65 million years ago. Expert B does not agree with the volcanic eruption theory. According to him dinosaurs became extinct due to the impact of asteroid. Which of the following statements may provide support to the theory propounded by Expert B? (A) The frigid and sweltering climatic extremes caused the extinction of dinosaurs.

    (B) A wide crater lying just off the Yucatan peninsula was created due to the impact of asteroid. (C) Scientists have discovered levels of iridium 30 times greater than average in the Cretaceous/ Tertiary boundary, the layer of sedimentary rock laid down at the time of the dinosaur extinction. (D) Some palaeontologists after analysing the fossil record believe that dinosaurs were doing quite well prior to the end of Cretaceous, when the dinosaurs became extinct. (1) Only (A) (2) Only (A) and (C) (3) Only (C) and (D) (4) Only (A) and (B) (5) Only (B), (C) and (D) (RBI Officer Grade ‘B’ Phase-I Exam, 03.08.2014)

    5. Read the following information and answer the given question. “The man behind the revival of SpeedZet Automobiles, the CEO has decided to quit. I believe that the company will soon go into losses due to this decision”, statement by a market analyst. Which of the following statements appears to be the perception of the market analyst ? (1) No other person can run SpeedZet Automobiles as successfully as the present CEO. (2) SpeedZet Automobiles may decide to shut down the operations due to the CEO’s exit. (3) SpeedZet Automobiles will try its best to retain the CEO in order to maintain its market position. (4) The strategies adopted by the CEO of SpeedZet Automobiles were exclusive in the industry. (5) SpeedZet Automobiles has the highest market share at present. (RBI Officer Grade ‘B’ Phase-I Exam. 21.11.2015)

    6. Study the following information carefully and answer the question given below : The M.B. Road near highway has an average of one pothole or patched piece of asphalt for every yards of road. The primary reason for the condition

    BPRE–854

    of the road isn’t age, weather or rush hour traffic. It’s 18–wheeler trucks. These heavy trucks roar down this route that was never meant to carry weights above 40 tonnes. While regular traffic on the road averages 2 tonnes in terms of weight per vehicle, the average weight per 18–wheeler truck is 35 times this weight. Which of the following can be concluded from the given statement? (A) The highway close to the M.B. Road can definitely carry at least 35 times more number of vehicles than that possible on M.B. road. (B) 18 – wheeler trucks which are 30 tonnes lighter will not cause harm to the mentioned road. (C) Restricting small vehicles while permitting only 18–wheeler trucks may reduce harm caused to the road. (D) A road is constructed considering the type of vehicles plying on it. (1) Only (C) (2) Both (A) and (C) (3) Both (B) and (D) (4) Both (A) and (D) (5) Only (A) (NABARD Assistant Manager Exam, 15.05.2016)

    7. This question consists of a statement and two conclusions numbered I and II given below it. A conclusion is something which can be directly deduced from the given information in light of all the given facts. You have to decide which of the given conclusions logically follows from the given statement and select the appropriate answer. Statement: “We have noticed that private hospitals charge huge amounts for maternity wards irrespective of whether operation procedure is required or not. As a result, we are proposing a cap on the amount that can be charged in such hospitals which will be a win-win situation for both the patients as well as Government hospitals, which are overcrowded most of the times.”— Statement by Health Minister of Country Z.

    CONCLUSIONS/INFERENCES I. With a cap on prices on private hospitals the number of people of Country Z opting for private hospitals instead of Government hospitals is expected to increase. II. The maternity wards of Government hospitals of Country Z are likely to undergo at least some cut in revenue with a cap on prices of private hospitals. (1) Neither I nor II can be concluded from the given statement. (2) Both I and II can be concluded from the given statement. (3) Either I or II can be concluded from the given statement. (4) Only II can be concluded from the given statement. (5) Only I can be concluded from the given statement. 8. Statement : In Country X, government has made the provision of day-care facility mandatory in all such organizations where total number of female employees is 250 or more. FL enterprise, therefore, did not have to provide a day-care facility as only 21% of its staff constitutes female employees. Which of the following can be inferred from the given statement ? (A) No male employees is allowed to avail the day-care facilities as per the mandate. (B) Number of male employees in FL enterprise is definitely more than 950. (C) Had there been 25 less male employees in FL enterprise, they would have had to provide for the day-care facility. (D) The total number of employees in FL enterprise is definitely less than 1250. (1) Only (C) (2) Only (D) (3) None (4) Only (A) and (D) (5) Only (C) and (D) (RBI Officer Grade ‘B’ Phase-I Exam, 04.09.2016 (Shift-II))

    9. Statement : In the past, consumers would rarely walk into an ice cream store and order lowfat ice cream. But that isn’t the case today. An increasing health consciousness combined with a

    much bigger selection of tasty low-fat foods in all categories has made low-fat ice cream a very profitable item for ice cream store owners. Which of the following best supports the statement ? (1) low-fat ice cream produces more revenue than other lowfat foods. (2) ice cream store owners would be better off carrying only low-fat ice cream. (3) ice cream store owners no longer think that low-fat ice cream is an unpopular item. (4) low-fat ice cream is more popular than other kinds of ice cream. (5) consumers are fickle and it is impossible to please them. (RBI Officer Grade ‘B’ Phase-I Exam, 04.09.2016 (Shift-I))

    10. Study the following information carefully and answer the given question : Following are the observations of an experiment on ‘sleep and memory’ conducted on 18 healthy young adults (ages 18 to 25) and 18 healthy older adults (ages 61 to 81). (A) The recall after 8 hours of sleep in younger adults was 65% more than that in the older adults. (B) Night-sleep had higher negative impact on all of the participants as compared to that of day-sleep of equal duration. (C) If a given set of words is memorised immediately before going to sleep, its recall after waking up was found to be better in younger adults than in the older adults. Which of the following can be concluded from the given findings of the research? (I) As per the experiment, there is some correlation between sleep and memory. (II) The part of brain involved in memory is more active during the day as compared to that during the night. (III) A sleep of more than 8 hours can improve the memory in older adults. (IV) Memorising something immediately after waking up from an 8hour long sleep will yield better

    BPRE–855

    results than memorising before sleep. (1) Only IV (2) All the given statements can be concluded from the given findings of the research. (3) Both I and III (4) Both II and IV (5) Both II and III (RBI Officer Grade ‘B’ Phase-I Exam, 04.09.2016 (Shift-I))

    11. Statement : In Country A, it is mandatory for all government organizations to provide transportation facilities (home pick-up and drop) to employees if 75% or more number of total employees working in the organization reside more than 15 kms away from office. The same, however, does not apply to XY enterprises as only 1500 of their employees travel more than 15 kms to work. Which of the following can be inferred from the given statement? (A) The total number of employees in XY enterprises is definitely more than 2000, (B) Only 25% employees of XY enterprises travel less than 15 kms to office. (C) If 25 new recruits who travel more than 15 kms join XY enterprises, the XY enterprises will definitely have to provide transportation facilities. (D) XY enterprise is definitely not a government enterprise. (1) Only (A) (2) Only (C) (3) Both (B) and (D) (4) Only (D) (5) Both (C) and (D) (RBI Officer Grade ‘B’ Phase-I Exam, 04.09.2016 (Shift-I))

    12. This question consists of a Situation and three Statements numbered I, II and III given below it. You have to decide which of the given Statements may be a reason for the given Situation. Situation : Farmers of State D, who traditionally cultivate crop M, bore considerable losses owing to deficiency of nutrient Z in the soil which is vital for adequate growth of crop M. However, this year the farmers registered a 40% growth in production.

    CONCLUSIONS/INFERENCES I. The farmers of State D have started producing crop Y only (which does not require nutrient Z for growth) since the past two years. II. Since the past two years the farmers of State D have been adding a fertiliser to the soil which contains nutrient Z. III. The nutrition this year was 200% more than the previous year in State D. (1) Either I or III (2) Only I (3) All I, II and III (4) Only III (5) Either II or III (RBI Officer Grade ‘B’ Phase-I Exam 17.06.2017)

    13. Study the following information carefully and answer the question given below : ‘If we take a close look at the balance sheets of previous five years, it shows that the amount of loans taken by the company against fixed assets has only increased, the result of which is poor financial health of the company this year’,— Financial report of Company B. Which of the following can be inferred from the statement of the financial report of the Company B? (A) Decreasing the amount of loans against fixed assets will improve the financial health of Company B. (B) The value of fixed assets of Company B has decreased by taking loans against them. (C) An analysis of balance sheets of a Company throws light on its financial health. (D) Company B had not taken any loans in any financial year other than the said five years. (1) None can be inferred (2) Both A and D (3) Both A and C (4) Only D (5) Only B (RBI Officer Grade ‘B’ Phase-I Exam 17.06.2017)

    (5) All those given as options 14. Study the following information carefully and answer the question given below :

    Flyzest Airlines was operating from Terminal D in City Niora ever since it started its operations six years ago. However, recently the airline was shifted to Terminal G. Which of the following statements can be a reason for shifting Flyzest Airlines from Terminal D to Terminal G? (1) Terminal G was recently constructed in City Niora to accommodate the new airlines which are supposed to begin operations next month. (2) Many employees of Flyzest Airlines live in the nearby areas of Terminal G of City Niora. (3) Two airlines were reportedly making continuous losses of Terminal D of City Niora as the passengers preferred Flyzest Airlines over them. (4) Airport authorities have renovated some areas of Terminal D of city Niora in the last month. (5) Flyzest Airlines found it very hard to manage its operations from Terminal D in the past one year due to increased number of passengers.

    16. In this question is given a statement followed by two Conclusions numbered I and II. You have to consider the statement and the following Conclusions and decide which of those can be concluded from the statement. Statement : Last over of the match remains to be bowled but it is definite that Team A has lost to Team B as 29 runs cannot be made in five balls in any possible circumstance. I. In the match being played between Teams A and B, one over contains five balls only. II. In the match being played between teams A and B, the maximum runs possible in one ball is definitely less than six. (1) Either I or II can be concluded (2) Only II can be concluded (3) Neither I nor II can be concluded (4) Both I and II can be concluded (5) Only I can be concluded (RBI Officers in Grade ‘B’ Phase-I Exam. 16.08.2018)

    15. Study the following information carefully and answer the question given below : Although number of literates in Village A is the maximum this year, the highest increase in literacy level was in Village B, with 10% rise this year thus bringing the number of literates to 42000. Which of the following statements can be inferred from the given fact? (1) Percent increase in literacy level in Village A was definitely more than 10%. (2) If 4200 individuals are made literate in village B, it will have more number of literates than village A. (3) The number of literates in village A this year is definitely more than 42000. (4) 4200 individuals in village B are still illiterate. (5) Total number of literates in Village A and Village B together is definitely 88,200.

    17. Study the following information carefully and answer the given question : In order to protect its employees from Apso disease, the management of company X has instructed the head of its canteen to discontinue selling of items having ‘Aabrak’ as an ingredient for the next four months. The canteen has planned to make up for the loss atleast to some extent by replacing ‘Aabrak’ with ‘Mugaar’. Which of the following cannot be inferred from the given information? (1) ‘Mugaar’ is similar to ‘Aabrak’ to quite some extent but does not cause ‘Apso’. (2) The management will hold the head of the canteen responsible if any of the employees is diagnosed with Apso even after such strict actions. (3) Items having Aabrak as an ingredient presently give good profits to the canteen. (4) Consumption of Aabrak in any form can cause Apso disease. (5) After four months from now, the chances of people getting Apso will be negligible.

    (RBI Assistant Manager Online Exam 25.03.2017)

    (RBI Officers in Grade ‘B’ Phase-I Exam. 16.08.2018)

    (RBI Assistant Manager Online Exam 25.03.2017)

    BPRE–856

    CONCLUSIONS/INFERENCES 18. In this question is given a statement followed by two inferences numbered I and II. (An inference is something by which you can logically deduce something to be true based on known premises.) You have to consider the statement and the following inferences and decide which of those follow(s) from the statement. Statement : ‘Around 80 per cent of the employees who reported late for work in the past month were those who reside within 500 metre radius of the company’ – HR Manager of Company B. I. Those who reported late in the past month had recently shifted within 500 metre radius of the company. II. Only 20 per cent of company B reside beyond 500 metre radius of the company. (1) Neither I nor II can be inferred (2) Only I can be inferred (3) Both I and II can be inferred (4) Either I or II can be inferred (5) Only II can be inferred (RBI Officers in Grade ‘B’ Phase-I Exam. 16.08.2018)

    19. Study the following information carefully to answer the given question : Statement : The Central Bank of Country X had asked all payment (wallets, credit/debit card) companies to store data regarding all payments processed by them locally (i.e., within the country) within three months. A majority of local payment companies (which have their head offices in country X) have complied with the orders but maximum global payment companies (which have head offices in countries other than X) have failed to do so. Which of the following can be inferred from the given information? (A) Some global payment companies were already storing the payment data within Country X before the Central Bank asked to do so. (B) The cost of storing payments data is considerably less in countries other than X. (C) The biggest payment company of the world has complied with the orders.

    (1) (2) (3) (4) (5)

    None can be inferred Only A Only B and C Only B Only A and C (SEBI Assistant Manager Exam. 17.11.2018)

    20. Study the following information carefully and answer the question given below : The ‘Fit-Me’ watches have been trending on the online shopping sites since the past few months. As per the market experts, 'These watches would not sustain the ‘Internet publicity’ for long and will be back to their base market losing all the recent attention they have received from the younger population.’ Which of the following statements can be inferred from the given information? (A) The ‘Fit-Me’ watches have started being sold through online site only recently. (B) A major reason of ‘Fit-Me’ being popular among youngsters is its online availability. (C) Once the online craze of ‘Fit-Me’ watches will drastically drop. (1) Only (B) (2) All (A), (B) and (C) (3) Both (B) and (C) (4) Both (A) and (B) (5) Only (A) RBI Grade ‘B’Officer Exam, 09.11.2019

    INSURANCE EXAMS Directions (1–5) : In questions given below, a statement is followed by reasons I and II. Apply the reasons to the statement and mark your answer as under : (Oriental Insurance Company Exam.08.04.2012)

    1.

    (1) Only I is right (2) Only II is right (3) Both I and II are right (4) Neither I nor II is right Statement : The President sometimes pardons the victims who have been destined to be sent to gas chamber. Reasons : I. The President is more kindhearted than the judges. II. The President is constantly interfering with law.

    BPRE–857

    2.

    Statement : After the bomb blasts, the citizens of Mumbai declared that they were not afraid of terrorist activities. Reasons : I. They knew that all the persons involved in such disgraceful activity would be caught by the police. II. They are a brave lot and do not believe in giving up or losing confidence if anti-social elements try to frighten them. 3. Statement : Volunteers often offer their services for scientific and medical experiments that may prove harmful or even fatal. Reasons : I. They do not give importance to their life where advancement of science and help to humanity is concerned. II. They are confident that the scientists would somehow save them. 4. Statement : Some species of birds are on the verge of extinction. Reasons : I. They are being constantly hunted. II. Pollution and destruction of forests have affected their breeding process. 5. Statement : Pesticides are responsible for causing different diseases in human beings, because Reasons : I. They contain some or the other type of poison that kills worms and pests but affects the plants adversely. II. Poisons are harmful to human beings too. 6. A tour and travel agency was sued by three customers as it had promissed a discount on the holiday package but when examined later, it was found that the agency had actually charged 12 percent more than what other agencies were offering. Which of the following is an inference which can be made from the facts stated in the above paragraph ? (1) Cheating may be a ground for filing a case.

    CONCLUSIONS/INFERENCES (2) It is not a grave crime to charge extra money for holiday package. (3) Different tour and travel agencies charge differently. (4) It is not essential to offer what has been promised earlier. (5) None of these (LIC Assistant Administrative Officer (AAO) Exam. 12.05.2013)

    7. Read the following information carefully and answer the question which follows “Pollution level has risen more than four times in matter of six years in city A. In my opinion, if all vehicles more than 20 years old still plying on the road are banned a major part of air pollution in the city can be controlled.” – person A. Which of the following statements most appropriately weakens the person’s opinion ? (1) More than 96% pollution in city A can be attributed to the industries and factories in city A and only 4% to the vehicular, nuclear plants and waste burning together. (2) Around a decade ago, the government of city A had converted all public transport vehicles from diesel-run to CNG-run in order to control pollution levels. (3) Older vehicles emit more pollutants than the newer ones as the newer vehicles are provided with newest carbon reduction technologies. (4) The number of vehicles, for private use as well as public transport use has not increased substantially in the past six years in city A. (5) City B in the neighbouring state which had made similar efforts had managed to reduce pollution levels by half within three years. (NIACL Administrative Officer (AO) Exam, 10.01.2015)

    Directions (8–9) : In each of the following questions, a statement followed by two inferences numbered I and II have been given. You have to consider the statement and the given inferences and decide which of the following can be inferred and select the appropriate answer. (NIACL Administrative Officer (AO) Exam, 10.01.2015)

    8. Statement: During the past year nationwide membership in fitness clubs has declined by about fifteen percent, while sales of fast–food products widely known to contribute to health problems have risen by about the same percent. I. 15% of the total population is unfit. II. The fitness clubs have closed their registrations for new members this year. (1) Either I or II can be inferred. (2) Only II can be inferred. (3) Both I and II can be inferred. (4) Neither I nor II can be inferred. (5) Only I can be inferred 9. Statement: “I would like to cancel hotel booking for todday as my flight to Delhi has been cancelled, thus I would be unable to reach your hotel in three hours’ time”, said Mr. Pandey to the hotel manager over phone. I. There is no other means to reach Delhi other than flight. II. There is no possibility of the fog to clear today. (1) Either I or II can be inferred. (2) Only II can be inferred. (3) Both I and II can be inferred. (4) Neither I nor II can be inferred. (5) Only I can be inferred Directions (10–12) : In each of the following questions, a statement followed by two inferences numbered I and II have been given. You have to consider the statement and the given inferences and decide which of the following can be inferred and select the appropriate answer. (NIACL Administrative Officer (AO) Online Exam, 11.01.2015)

    10. Statement : Rahul read the book written by Professor ‘X’. The book is easy to comprehend and understand the condition of poor in the Country ‘Z’ completely. I. According to Professor ‘X’, Rahul now clearly under stands the condition of poor in the Country ‘Z’. II. No other professor reads this book. (1) Either I or II can be inferred. (2) Only II can be inferred.

    BPRE–858

    (3) Both I and II can be inferred. (4) Neither I nor II can be inferred. (5) Only I can be inferred 11. Statement : The government of Country ‘X’ has fixed the deadline for installing panic button in the public transport. Panic button is used in emergency to activate GPS tracker and find out the points of start and end respectively. I. The Government of Country ‘X’ is serious about the safety of its citizens. II. The public transport will install the panic button. (1) Either I or II can be inferred. (2) Only II can be inferred. (3) Both I and II can be inferred. (4) Neither I nor II can be inferred. (5) Only I can be inferred 12. Statement : The translation of Anand Rathore’s book does not receive as many accolades as the original book. I. The subject matter of Rathore’s book has now become outdated. II. Generally translation works are not liked by people and these are sold less than the original book. (1) Either I or II can be inferred. (2) Only II can be inferred. (3) Both I and II can be inferred. (4) Neither I nor II can be inferred. (5) Only I can be inferred 13. This question consists of a statement and two inferences numbered I and II given below it. An inference is something which is not directly stated but can be inferred from the given facts. You have to decide which of the given inferences logically follows from the given statement and select the appropriate answer. Statement : It has been observed in schools that many students engage in bullying those who are either younger to them or vulnerable despite being punished repeatedly. However, research suggests that, bullying behaviour reduces in students who are sent for counselling as compared to those who are only punished for the same.

    CONCLUSIONS/INFERENCES I. No student who is sent for counselling engages in bullying behaviour. II. Counselling is more effective in modifying behaviour of students as compared to punishment. (1) Only Inference I follows (2) Only Inference II follows (3) Either Inference I or II follows (4) Neither Inference I nor II follows (5) Both the Inference I and II follow (OICL Specialist Officer (Finance) Exam. 03.05.2015)

    14.

    Study the following information carefully and answer the given question : The Government of Country X is being criticized for introducing an additional cess in the service tax for the welfare of micro and small industries. Which of the following statements best explains the stand of the Government in introducing the additional cess ? (1) The percentage of cess introduced by Government of Country X is higher than similar cases in the neighbouring countries. (2) Most of the micro and small industries fall in Country X only because of lack of funds. (3) Additional cess has been introduced by Government of Country X for the third time in a year. (4) Two new schemes worth Rs. 2500 crore fully funded by World Bank have been implemented this year by the Government of Country X for micro and small industries. (5) A recent survey revealed that few cess earlier levied by the Government of Country X did not reach the beneficiaries. (UIICL A.O. Exam 12.06.2016)

    15.

    Study the following information carefully to answer the given question. Mr. Kulkarni worked with GRS telecom for 6 years, after which he decided to start his own venture and hence resigned in March 2012. At present, his venture is successfully running since the past 4 years. However, recently,

    Mr. Kulkarni received a notice from court mentioning that GRS Telecom has filed a case against him for violating their company rules. Which of the following can be a reason behind a case being filed by GRS Telecom against Mr. Kulkarni after so many years ? (1) With the consent of Mr. Kulkarni, GRS telecom had deducted Rs. 55000 per year from his salary as Employee Emergency Fund which Mr. Kulkarni had failed to claim. (2) Mr. Kulkarni’s present venture also is related to the telecom industry and his success is becoming a threat to GRS Telecom. (3) During his service period in GRS Telecom, Mr. Kulkarni was warned atleast 5 times to stop misbehaving with the clerical staff, but they all went ignored by him. (4) Mr. Kulkarni still attempts to lure out internal confidential information of GRS Telecom from its present employees despite being warned twice about the same (5) When Mr. Kulkarni resigned from GRS Telecom, he served a notice period of 83 days as against the company rule of 3 months, but he paid the money for the remaining days. (UIICL A.O. Exam 12.06.2016)

    16. Study the following information carefully and answer the question given below : The commerce department of a college allotted its students to different organisations for completing their two-month internship starting from 31.02.2017. The students were required to submit their internship reports in a specified format along with the original certificates latest by 05.05.2017, failing they would not be allowed to write their final exams. Following options were the list of students who did not submit their reports by the stipulated deadline. However, one of these was allowed to write this exam. Identify that student.

    BPRE–859

    (1) Vaibhav Tripathi, who was allotted to an organization of another city, was given only a provisional certificate by the organisation which was not as per the norms of the organisation. (2) Akhil Mehta had completed his report before the deadline, but could not submit as it was not written in the specified format. (3) Rachit Saxena was allotted to a renowned NGO where the minimum limit for internship was of 75 days. (4) Sunil Jadhav was absent in the class during which the format of the report was given to the students. (5) Mehvish had to do her internship under two different project guides. Hence, she found it difficult to adequately allot time to both of them. (GIC Officer Online Exam 15.05.2017)

    17. Study the following information carefully and answer the question given below : “The high production of groundnut this season can be owed to the fact that pesticide Linethez was applied instead of Piddle.” – An agriculture expert from State A. “Sir, I beg to differ from you! Many farmers have still reported severe attack of ‘Zolf Mervel’ – A farmer from State A. Which of the following cannot be inferred from the given conversation between the expert and the farmer? (1) According to the expert Linethez is more effective than Piddle. (2) Zolf Mervel is definitely one of the groundnut crops. (3) Both Piddle and Linethez are the pesticides used for controlling groundnut pests. (4) The conversation highlights the contrast in the opinions of the expert and the farmer with respect to effectiveness of Linethez. (5) Groundnut cannot be protected from Zolf Mervel using Linethez. (GIC Officer Online Exam 15.05.2017)

    CONCLUSIONS/INFERENCES 18. Study the following information carefully and answer the question given below : Each of the 350 stores of ‘Cottyami’ has its own system of maintaining data records. Recently, the employees’ association of ‘Cottyami’ has requested the CEO to provide a single system where all the stores will submit their data thereby reducing the extra workload every quarter. A ‘No’ as an answer to which of the following questions would have compelled the CEO to Accept the request of the employees’ association? (1) None of those given as options (2) Will installing an integrated system anyway improving the marketing of our products? (3) Will this new system run with less or existing workforce? (4) Will the current data be easily and readily transferable to the new integrated system? (5) Will the smoothening of data management make the employees more dedicated to their job? (GIC Officer Online Exam 15.05.2017)

    19. Study the following information carefully and answer the question given below : The MB road near highway has an average of one pothole or patched piece of asphalt for every 40 yards of road. The primary reason for the condition of the road isn’t age, weather or rush hour traffic. It’s 18-wheeler trucks. These heavy trucks roar down this road that was never meant to carry weights above 40 tonnes. This is because as compared to their prescribed highway, M.B. road saves minimum half an hour time while regular traffic on the road averages 2 tonnes in terms of weight per vehicle, the average weight per 18-wheeler truck is 35 times this weight. Which of the following can concluded from the given statement? (A) The highway close to the M.B. road can definitely carry atleast 35 times more number of vehicles than that possible on M.B. road.

    (B) 18–wheeler trucks which are 30 tonnes heavier will not cause any harm to the mentioned road. (C) Restricting small vehicles while permitting only 18-wheeler trucks may reduce harm to the road. (D) While constructing a road the strength should be decided on the type of vehicles and its weight which are expected to ply. (1) Both (A) and (D) (2) Both (B) and (D) (3) Only (C) (4) Both (C) and (D) (5) Both (A) and (B) (GIC Officer Online Exam 15.05.2017)

    20. Study the following information carefully and answer the given question. Many psychologists and counsellors are of the opinion that the Indian higher education scenario is highly competitive with almost war-like intensity. Nationwide entrance exams are conducted to select a few thousand from the millions competing. The preparation for the myriad competition exams starts almost four years before the exams even begin! Is this right for the students? Which of the following may be the reason behind such observation of psychologists and counsellors? (1) The increase in population leads to increase in competition among students to acquire a seat in the top institutions. (2) The quality of higher education in India is much better than any other Asian countries. (3) The education authorities must think of setting up more universities and colleges so that such stressful competition turns into healthy competition. (4) Most of their patients nowadays are students appearing in entrance examination who are suffering from stress and anxiety. (5) Future potential of the students for attaining higher education must be judged on

    BPRE–860

    the basis of their overall academic achievement and not such entrance exams. (GIC Officer Online Exam 15.05.2017)

    21. Study the following information carefully and answer the question given below : Fighting the disease ‘rencer’ is never easy for anyone. However, finding an assurance to be financially prepared for it, definitely is. For the disease requiring a minimum total of Rs. 60 lakh worth medical expenditure, our insurance scheme offer Rs. 5 lakhs every year for first two years followed by Rs. 10 lakhs every subsequent year — An advertisement by an insurance company. Which of the following statements would prove that the insurance policy is flawed in its approach? (A) The disease although serious and cash intensive, is fatal only in 23% of the cases. (B) 75% of the entire amount for treatment is required in the first two of years of contracting the disease. (C) Expenses for treatment of the disease do not fluctuate much based on the intensity of the disease and the type. (D) If treated within 4 years of contracting the disease, the patient can be completely cured of the disease for life. (1) Both (B) and (D) (2) All (A), (B), (C) and (D) (3) Only (A) (4) Only (C) (5) Both (A) and (C) (GIC Officer Online Exam 15.05.2017)

    22 . Study the following information carefully and answer the question given below : Although two lanes on the flyover are functional, the repair work of the flyover in the city has been stopped since Thursday and the debris have been completely cleared. It has been announced that the repair work will commence after Monday. Which of the following may be a reason for temporary halt of the repair work of the flyover?

    CONCLUSIONS/INFERENCES (1) An international marathon has been planned over the weekend and the flyover is on the route of the same. (2) The flyover in question is not used as much as other flyovers in the city. (3) The city dwellers have blamed the government for not giving them notice before commencing the repair work of the said flyover. (4) The workers have recently been added to work in two shifts–day and night- in order to finish the work in a stipulated period of time. (5) Some people are of the opinion that rather than repairing the flyover, it would be easier to construct a new one. (GIC Officer Online Exam 15.05.2017)

    23. Study the following information carefully and answer the question given below : School B has been running classes upto Class X from the last five years. This year, the school applied for an extension of classes upto XII to the central board. However, the board rejected the application. Which of the following statements cannot be a reason for the rejection of application of extension of classes in school B? (1) School B cannot afford adequately qualified teachers required for Classes XI and XII. (2) School B does not conduct extra classes for the revision of syllabus during the 10 days of winter break. (3) The seats in other schools in the vicinity of School B remain vacant for classes XI and XII despite adopting various marketing gimmicks. (4) The number of admissions in School B for classes VII to X has reduced significantly in the past few years due to poor results. (5) School B lacks infrastructure which is required to conduct classes for XI and XII. (GIC Officer Online Exam 15.05.2017)

    24. Study the following information carefully and answer the question given below : The number of complaints regarding the product of Company F, a renowned shoe manufacturing company, has increased in the past six months across the Country X. What can be an effect of increased number of complaints in Company F? (A) The goodwill of the Company F in the market will reduce. (B) The management of Company F will penalize the quality control department. (C) The stores by Company G will capture major market in the country. (D) The number of customers of Company F may fall. (1) Only (B) and (C) (2) Only (A) and (C) (3) Only (A) and (D) (4) Only (A), (B) and (D) (5) Only (C) (GIC Officer Online Exam 15.05.2017)

    25. This question consist of a situation followed by two statements numbered I and II given below it. Which of the given statements can be a possible reason for the given situation? Situation : Company Y has decided to discontinue the transport facility available for the employees from the next month. Statement I : The number of buses plying on the same route was more than required which created a chaos amongst passengers while boarding the bus thus leading to maximum tragedy. Statement II : The people living in the remote areas were unable to avail to the facility as there were no buses available in these routes. (1) Both I and II can be a possible reasons. (2) Neither I nor II can be a possible reason (3) Either I or II can be a possible reason (4) Only I can be a possible reason (5) Only II can be a possible reason (GIC Officer Online Exam 15.05.2017)

    BPRE–861

    SHORT ANSWERS

    NATIONALISED BANKS & IBPS PO/MT/SO EXAMS 1. (5)

    2. (2)

    3. (3)

    5. (1)

    6. (2)

    7. (1)

    4. (4) 8. (5)

    9. (1)

    10. (3)

    11. (2)

    12. (3)

    13. (2)

    14. (3)

    15. (1)

    16. (5)

    17. (4)

    18. (3)

    19. (5)

    20. (2)

    21. (3)

    22. (1)

    23. (5)

    24. (2)

    25. (3)

    26. (4)

    27. (1)

    28. (4)

    29. (2)

    30. (4)

    31. (4)

    32. (4)

    33. (2)

    34. (5)

    35. (4)

    36. (2)

    37. (2)

    38. (4)

    39. (4)

    40. (1)

    41. (4)

    42. (5)

    43. (5)

    44. (1)

    45. (5)

    46. (5)

    47. (2)

    48. (5)

    49. (2)

    50. (4)

    51. (5)

    52. (2)

    53. (3)

    54. (5)

    55. (5)

    56. (5)

    57. (2)

    58. (4)

    59. (4)

    60. (2)

    61. (5)

    SBI PO EXAMS 1. (1)

    2. (5)

    3. (3)

    4. (2)

    5. (3)

    6. (4)

    7. (3)

    8. (1)

    9. (4)

    10. (5)

    RBI GRADE–B/NABARD GRADE–A OFFICER EXAMS 1. (4)

    2. (5)

    3. (2)

    4. (5)

    5. (4)

    6. (3)

    7. (2)

    8. (3)

    9. (4)

    10. (3)

    11. (1)

    12. (3)

    13. (3)

    14. (5)

    15. (3)

    16. (4)

    17. (4)

    18. (1)

    19. (1)

    20. (3)

    INSURANCE EXAMS 1. (4)

    2. (3)

    3. (1)

    4. (2)

    5. (3)

    6. (1)

    7. (1)

    8. (4)

    9. (4)

    10. (5)

    11. (3)

    12. (5)

    13. (2)

    14. (2)

    15. (4)

    16. (1)

    17. (2)

    18. (1)

    19. (2)

    20. (4)

    21. (1)

    22. (1)

    23. (2)

    24. (4)

    25. (4)

    CONCLUSIONS/INFERENCES

    EXPLANATIONS

    NATIONALISED BANKS & IBPS PO/MT/SO EXAMS 1. (5) It is mentioned in the passage that 50 - 80 per cent of all people who suffer a heart attack are able to return to work. Therefore, the inference is definitely false. 2. (2) It is mentioned in the passage that the persons who have suffered a heart attack should avoid high pressure, stressful work. Therefore, it may be concluded that the inference is probably true. 3. (3) There is no information about this inference. 4. (4) The use of term ‘definitely’ in the inference makes it doubtful. Therefore, the inference is probable false. 5. (1) Consider the very first line of the passage. It is clear from the first line of the passage that the inference is definitely true. 6. (2) Infrastructure plays an important role in economic development. India is a developing country. Therefore, the country needs to make massive investments in infrastructural development. Thus, the inference is probably true. 7. (1) The last line of the passage supports the inference. 8. (5) Biotechnology is mentioned as new economy sector in the case of India. Therefore, the inference is definitely false. 9. (1) From facts stated in the passage, the inference seems to be definitely true. 10. (3) There is no reference about this inference in the passage. 11. (2) From the given information it is clear that the school would face a crunch in terms of availability of qualified teachers in the years to come. 12. (3) It is mentioned that salaries of maids have gone up only 50 times while costs have gone up 100 times 13. (2) Statement (B) can be inferred from the given statement. 14. (3) Statement (C) proves that intervention from their urban counterparts can also be beneficial to a certain extent. 15. (1) Statement (A)

    16. (5) Statement (D) 17. (4) The district authority sent a police team to nab the culprits. It shows that the Government is committed to provide protection to travellers across the country. 18. (3) There is a perception among small brands that sale in a supermarket is higher than that of small grocery stores. 19. (5) The statement clearly implies that smaller brands are currently making substantial losses in their businesses. Consider the term “troubled waters.” 20. (2) Statement (C) represents a disadvantage of the small grocery stores over the Supermarkets from the perspective of a smaller brand. Small grocery stores do not help in distribution of any brand. 21. (3) Statement (D) represents a reason for the shift from local grocery stores to supermarkets by the smaller brands. 22. (1) Statement (A) will prove that that step taken by the smaller brands may not necessarily be correct. 23. (5) It is mentioned in the passage that 50 - 80 per cent of all people who suffer a heart attack are able to return to work. Therefore, the inference is definitely false. 24. (2) It is mentioned in the passage that the persons who have suffered a heart attack should avoid high pressure, stressful work. Therefore, it may be concluded that the inference is probably true. 25. (3) There is no information about this inference. 26. (4) The use of term ‘definitely’ in the inference makes it doubtful. Therefore, the inference is probable false. 27. (1) Consider the very first line of the passage. It is clear from the first line of the passage that the inference is definitely true. 28. (4) It is well known that human being is mortal. 29. (2) Obviously, social entrepreneurship is different from the business entrepreneurship. 30. (4) Only statement I can be inferred from the given information. It is clearly mentioned that the process of acquiring visa for the tourist has been eased. 31. (4) Only statement I can be inferred from the given informa-

    BPRE–862

    tion. It is clearly mentioned that the process of acquiring visa for the tourist has been eased. 32. (4) Obviously, option (4) most appropriately proves that the decision taken by the aviation minister is unrealistic as the existing fleet is not being used for lack of passengers. 33. (2) It has been said that 23 – micrograms level of hormone ‘X’ needs immediate medical attention. It means that cretosis disease becomes intense at that level of hormone ‘X’. It has been clearly mentioned that long term usage of antibiotics causes cretosis. Therefore, option (2) may be concluded from the given information. 34. (5) The performance of the employees of Company ‘K’ is linked to the reward. If the performance of none of the employees has improved, it implies that no employee has been rewarded for his/ her work. Therefore, option (5) is the most appropriate. 35. (4) Obviously, option (4) can be inferred from the given statement. 36. (2) Obviously, option (2) can be inferred from the given information. 37. (2) Obviously, option (2) may be inferred from the given statement. 38. (4) Rohan can run only five kilometre at a stretch. When he doubles it, he would still not reach half way through the marathon. It implies that the upcoming marathon is definitely more than 20 kilometre. 39. (4) Obviously, option (4) can be inferred from the given statements. In Beehal, out of all the candidates, nearly 90 per cent comprised of the ones from Seekat. And, overall attendance in Beehal was only 23 per cent. 40. (1) Obviously, both (B) and (C) may be the cause for the Government to accept the proposal of the Company X. 41. (4) Obviously, both (A) and (C) can be concluded from the given statement. 42. (5) People in City Y are buying second copies of products because of their prices and easy availability. Therefore, option (4) may be an assumption. Option

    CONCLUSIONS/INFERENCES

    43.

    44.

    45.

    46.

    47.

    48.

    49. 50.

    (5) can be inferred from the information given in the owner’s statement. (5) Obviously, option (5) cannot be reason behind discontinuing Bus No. 218 as it presents law and order problem. (1) ‘Attrition’ refers to a gradual reduction in the number of people who work for an organisation that is achieved by not replacing those who leave. Attrition is the unpredictable and uncontrollable, but normal, reduction of work force due to resignations, retirement, sickness or death. Obviously, statements A, B and C may be reasons for the reduced attrition rate of employees in Company X. (5) Obviously, option (5) can be inferred from the statement given by Marketing Head of the Company P. It was decided to launch the scheme of free gift item after one week of the launch of product G in the market. (5) Obviously, option (5) can be inferred from the given statement. Factors other than demand of a commodity also affect its sales and profits. Galax fibre is manufactured only in Country A. It does not imply that no other country has adequate machinery to manufacture Galax fibre. The required machinery can be purchased. However, it is quite possible that the climate of Country A is very specific for manufacturing the Galax fibre. (2) High amount of ‘trinium’ blocks the absorption of ‘roassium’ by the plants. As a result, the jute of yield is adversely affected. It implies that jute plants need roassium. In order to address the problem of high amount of trinium, the use of groundwater for irrigation should be completely stopped. Thus, Conclusion (B) also follows. Conclusion (C) is also approporiate. The very first line of the given information does not support Conclusion (D). (5) Obviously, all the given options can be possible reasons behind Mallika’s decision to wear a dress from new designer. (2) It is clear that only I can a possible reason. (4) Obviously, both I and II can be possible reasons.

    51. (5) Obviously, all (A), (B), (C) and (D) can be inferred from the given information. 52. (2) Obviously, Option (2) can be inferred from the given information. 53. (3) Either I or II could be the possible Conclusion from the given information. 54. (5) Obviously, both III and V are possible assumptions. 55. (5) Only II can be concluded from the given information. 56. (5) Obviously, none of the statements can be deduced. 57. (2) Obviously, both (II) and (III) can be inferred from the given statement. 58. (4) Obviously only (A) and (C) are Inferences of the given statements. The very first statement implies that there are some other means of entertainment. The second statement implies that downloaded songs may not be exciting. 59. (4) Rattan has gained expertise in a particular department. Expertise imparts capability and accuracy. Therefore, (B) is valid. The Company believes that Rattan will do better in the new department also. Thus, (D) is also valid. 60. (2) Obviously, all the given options can be inferred or assumed from the given information. Addiction affects a primitive bit of the brain, a bit of the brain that, from an evolutionary perspective is less advanced and it is more about immediate gains. 61. (5) From the statement, it is clear that frequent use of mosquito repellant roll-ons increases the per cent content of chemical A in children. It means chemical A is present in the body of children. Thus, (A) and (B) are invalid. It is mentioned that 0.18% of chemical A leads to serious pathological conditions. Thus, (C) is invalid and (D) is valid.

    SBI PO EXAMS (1–4) : Candidate

    Conditions

    (i) Anup ✓ Nisha ✓ Subodh ✓ Ravindra ✓ Rajiv ✓

    (ii) ✓ ✓ ✓ ✓ ✓

    (iii) or – × ✓ ✓ ✓

    (a) (iv) (v) or ✓ ✓ ✓ – ✓ ✓ – ✓ ✓ – ✓ – – ✓ ✓

    BPRE–863

    (b) – – – ✓ –

    1. (1) Anup Bhatnagar satisfies conditions (i), (ii), (a), (iv) and (v). Therefore, his case would be referred to the Dean of the Institution. 2. (5) In case of Nisha Desai there is no information about the condition (iii). 3. (3) Subodh Verma satisfies conditions (i), (ii), (iii), (iv) and (v). Therefore, he can be selected. 4. (2) Ravindra Sharma satisfies conditions (i), (ii), (iii), (iv) and (b). Therefore, his case would be referred to the Director of the Institution. 5. (3) Rajiv Batra satisfies all the conditions (i), (ii), (iii), (iv) and (v). Therefore, he can be selected. 6. (4) Option (4) is true. 7. (3) Option (3) is true. 8. (1) None of the given Inferences can be drawn from the given statements. The use of term ‘most’ in the option (2) makes it invalid. From one or two incidents we cannot infer something conclusively. Therefore, options (3), (4) and (5) can be ruled out. 9. (4) Modified look of the mobile has increased the sale of the mobile phones. Therefore, Inference II is implicit in the statement. It is possible that the processor of the previous model was good enough and hence people were attracted towards its modified look. Therefore, Inference I is not implicit. 10. (5) Neither I nor II is implicit. The use of term ‘all’ in the Inference I makes it invalid. Similarly, the use of term ‘only’ in the Inference II makes it invalid.

    RBI GRADE–B/NABARD GRADE–A OFFICER EXAMS 1. (4) It is clear from the statement that the school in Regari is not at a walkable distance from the village. 2. (5) Clearly, both the expectations are implicit in the statement. 3. (2) Only expectation II is implicit in the statement. 4. (5) Clearly, statements (B), (C) and (D) support the theory propounded by Expert B. 5. (4) Obviously, option (4) is the most appropriate answer. 6. (3) Obviuosly, (B) and (D) can be concluded from the statements.

    CONCLUSIONS/INFERENCES 7. (2) Obviously, both the Conclusions follow from the information given in the statements. 8. (3) None of the Conclusions can be inferred from the given statements. Inference (A) is universal premise. Similarly, inference (B) and (C) assert something which cannot be corroborated by the given information. 9. (4) Nowadays, consumers have become health conscious. Therefore, they like low – fat food items including ice–cream. Thus, statement (4) supports the given premise. 10. (3) It is clear from the observations that, there is some relation between sleep and memory. Therefore, Conclusion I follows from the observations of the experiment. It may also be concluded that memory in older adults can improve after a sleep of more than eight hours. Thus, Conclusion III also follows. 11. (1) 1500 is less than 75 per cent of the total number of employees in the XY Enterprises. Therefore, total number of employees in the XY Enterprises is more than 2000. Therefore, statement (A) can be inferred from the given statements. 12. (3) From the given situation, it is not clear that whether farmers of State D cultivated crop M or not this year. All the Statement I, Statement II and Statement III seem to be suitable reasons for the given situation. 13. (3) Obviously, (A) and (C) can be inferred from the statement of the financial report of the Company B. 14. (5) Obviously, option (5) is the most suitable reason for shifting Flyzest Airlines from Terminal D to Terminal G. 15. (3) It is mentioned that highest increase in literacy level was in Village B with 10 per cent rise this year. Therefore, Option (1) is invalid. From the per cent data it is not possible to infer option (2). It is clearly, mentiond that number of literates in village A this year is the maximum. Therefore, the number of literates in village A is definitely more than 42,000. 16. (4) It is clearly mentioned that five balls are to be bowled and only the last over is remaining. Therefore, Conclusion I seems to be true.

    The use of term ‘definitely’ in the Conclusion II makes it doubtful, but it is also true. 17. (4) Obviously, option (4) can be inferred from the given information. 18. (1) Neither I nor II can be inferred from the given statement. The statement talks about the employees who come late for work. “Around 80 per cent of the employees who reported late for work” does not imply the 80 per cent of the total employees. 19. (1) It is mentioned that maximum global payment companies which have head offices in countries other than X, have failed to store data regarding all payments. Therefore, none can be inferred. 20. (3) Both (B) and (C) can be inferred from the given information. It is not possible to infer that ‘Fit–Me’ watches is being sold through online site right from its launch or recently. Therefore, (A) does not follow.

    INSURANCE EXAMS 1. (4) None of the reasons is correct. Judge delivers verdict as per the rule. The President has the power to pardon such persons who have been awarded death sentences. This does not imply that the President is more kind-hearted than the judges. 2. (3) Both the reasons are correct. 3. (1) Only the reason I is correct. 4. (2) Only the reason II is correct. 5. (3) Both the reasons are correct. 6. (1) Clearly, option (1) may be inferred. 7. (1) It has been said that vehicles are major pollutants but the Option (1) discards this notion outrightly. 8. (4) Clearly, neither I nor II can be inferred. The membership of the fitness club cannot be equated with population. 9. (4) Obviously, neither I nor II can be inferred. Mr. Pandey wanted to cancel hotel booking as his flight to Delhi was cancelled and he was unable to reach hotel in three hours. It does not imply that there is no other means to reach Delhi other than flight. 10. (5) Only Inference I can be inferred from the facts given in the question. A book written by some famous writer is read by some other professors too.

    BPRE–864

    11. (3) Clearly, both the inferences seem to be appropriate. 12. (5) Only Inference I can be inferred from the statement. 13. (2) Only Inference II follows. It is clearly mentioned that bullying behaviour reduces in students who are sent for counselling. 14. (2) Obviously, option (2) best explains the stand of the Government in introducing the additional cess. 15. (4) Obviously, option (4) may be a reason for filing a case against Mr. Kulkarni. 16. (1) Obviously, Vaibhav Tripathi was allowed to write his exam. He lacks original certificate and it is not his fault. 17. (2) Zolf Marvel is a type of pest. It is not a groundnut crop. 18. (1) Obviously, a ‘No’ as an answer to none of the questions seems to be right. 19. (2) Clearly, (B) and (D) can be concluded from the given statement. It is mentioned that the highway close to the MB road can carry 18–wheeler trucks with a load of 35 times as compared to the vehicles plying on the MB road which averages two tonnes in terms of weight per vehicle. It does not imply the number of vehicles passing through the highway is 35 time more than the number of vehicles on the MB road. The main cause of dismantled road is heavier trucks. Therefore, restricting small vehicles cannot reduce harm to the road. 20. (4) Obviously, option (4) is the reason behind the mentioned observation of psychologists and counsellors. 21. (1) Obviously, statements (B) and (D) prove that the insurance policy is flawed in its approach. 22. (1) Obviously, option (1) may be a reason for temporary halt of the repair work of the flyover. 23. (2) Obviously, Option (2) cannot be a reason for the rejection of application of extension of classes in School B. 24. (4) Obviously, statements A, B and D would be effects of increased number of complaints against Company F. 25. (4) Obviously, only statement I can be a possible reason for the given situation. ❐❐❐

    CONCLUSIONS/INFERENCES

    MODEL EXERCISES Directions (1-3) : In each of these questions, there is a statement followed by three conclusions. State whether the conclusions are implicit in the given statement. 1. The classic My Fair Lady was based on the play Pygmalion by G. B. Shaw. I. My Fair Lady was the screen adaptation of the play. II. G. B. Shaw was a playwright. III. My Fair Lady tells the story of Liza Doolittle and Henry Higgins (1) Only I is implicit (2) Only II is implicit (3) I and III are implicit (4) None is implicit (5) All of these 2. The Unit Trust of India (UTI) informed SEBI that it would now have to cut the interest rates in its assured return monthly income plans. I. Assured return is a contradiction as far as UTI is concerned. II. Investors would feel cheated if that happens III. SEBI is a regulatory body (1) Only I is implicit (2) Only II is implicit (3) I and II are implicit (4) None is implicit (5) All of these 3. A casual look through the news letters of different mutual funds (MFs) would make one feel that MFs are chock a block full of cash. I. A deeper look would reveal that it is not so. II. MFs are full of cash. III. There are more than one Mutual Funds today. (1) Only I is implicit (2) Only II is implicit (3) Only III is implicit (4) None is implicit (5) All of these Directions (4-8) : Two statements are given followed by two conclusions I & II. Take the statement to be true and then decide which of the conclusions logically follows. Mark your answer as : (1) If only conclusion I follows (2) If only conclusion II follows (3) If either conclusion I or II follows and

    (4) If neither conclusion I nor II follows (5) None of these 4. Statement : Of the ten fishermen caught in a storm, nine managed to return to the shore. Praveen has not yet returned after four days. Conclusions : I. Praveen got killed in the storm. II. Praveen has survived in the storm. 5. Statement : Now you don’t need an import licence to own a VCR. Conclusions : I. VCRs are now manufactured indigenously. II. VCRs are now freely permitted to be imported. 6. Statement : Just about everyone in Germany has been on a diet at one time or the other and millions of them have learned that the weight they lose is all too easily regained. Still despite their frustration, few question the wisdom of deiting. Conclusions : I. Germans should stop dieting. II. Germans do not learn from experience. 7. Statement : A study of planning commission reveals boom in revenues. However, this has been of little avail owing to soaring expenditure. In the event, there has been a high dose of deficit financing, leading to marked rise in prices. Large financial outlays year after year had little impact on level of living. Conclusions : I. A boom in revenues leads to rise in prices. II. Large financial outlays should be avoided. 8. Statement : The average number of students per teacher is 50 in the urban area whereas it is 60 in rural areas. The national average is 55. Conclusions : I. The student teacher ratio in the rural areas is higher than in the urban areas. II. More students study with the same teacher in the rural areas as compared to those in the urban areas.

    BPRE–865

    Directions (9-12) : In each of these questions, a statement is given followed by two conclusions I and II Mark your answer as (1) If only conclusion I follows (2) If oniy conclusion II follows (3) If neither I nor II follows (4) If both I & II follow. (5) None of these 9. Statement : Sealed tenders are invited from competent contractors experienced in executing construction jobs. Conclusions : I. Tenders are invited only from the experienced contractors. II. It is difficult to find competent tenders in construction jobs. 10. Statement : The distance of 900 km by road between Bombay and Jafra will be reduced to 280 km by sea. This will lead to a saving of 7.92 crore per annum on fuel. Conclusions : I. Transportation by sea is cheaper than that by road. II. Fuel must be saved to the greatest extent. 11. Statement : The manager humiliated Sachin in the presence of his colleagues. Conclusions : I. The manager did not like Sachin. II. Sachin was not popular with his colleagues. 12. Statement : Any young man who makes dowry as a condition for marriage discredits himself and dishonours womanhood. Conclusions : I. Those who take dowry in marriages should be condemned by the society. II. Those who do not take dowry in marriages respect womanhood. Directions (13-17) : Each of these questions has an inference drawn out of the passage given below. Mark your answer as (1) if the inference is ‘definitely true’ (2) if the ‘data provided are inadequate’ (3) if the inference is ‘probably true’ (4) if the inference is ‘definitely false (5) None of these

    CONCLUSIONS/INFERENCES Passage Ministry of Environment and forests has granted environmental clearance to the Karakatla open-cast expansion project of the central coal Fields Ltd. in Bihar that envisages exploitation of non-coking coal reserves, the present production level of 0.8 million tonnes is proposed to be expanded to 1.5 million tonnes per annum at an estimated cost of 67. 82 crore under the project. The total land area requirement for the proposed mining activities is about 651 hectares which includes about one-sixth of it as forest land. 13. The expansion plan would require about 100 hectares of forest land. 14. Karakatla open-cast mine is the only one of non-coking coal in the country. 15. There is no demand for non-coking coal. 16. The production cost of one tonne of non-coking coal from Karakatla mine will be about 450. 17. Environmental concern gets less priority over the need of the coal. Directions (18-21) : In each of the questions below there is an inference, which is based on the given passage. Examine each inference separately in the context of the passage and decide upon its degree, of truth or falsity. Mark your answer as (1) if you think the inference is ‘definitely true’ (2) if you think the inference is ‘probably true’ though not definitely true in the light of the facts given (3) if you think the ‘data is inadequate’ i.e., from the facts given, you cannot say whether the inference is likely to be true or false (4) if you think the inference is ‘definitely false’ (5) None of these Passage The reforms are aimed at having an impact over a wide cross-section of the economy. With improvements and greater efficiency in mobilising and allocating resources, the basis will become stronger for promoting economic growth and development. There forms aim at increasing efficiency of the financial sector and the range of financial services available within the economy. The introduction of bank capital adequacy and accounting stan-

    dards, together with improvement in the regulation and supervision of financial institutions and capital market, is aimed at installing greater public confidence, ensuring safety and soundness of the financial system, bringing about greater transparency and accountability in operations and encouraging overall increased resources, mobilisation within the economy. 18. Prior to reforms, the banking sector was running inefficiently although resource allocation was efficient. 19. Bank capital adequacy was not in operation before reforms. 20. Public confidence has some connection with resource mobilisation. 21. The economic reforms have aimed only at the financial sector. Directions (22-25) : In each of these questions, certain statements are given followed by two conclusions numbered I and II. Consider the statements to be true even if they seem to be at variance from commonly known facts. Decide which one of the conclusions logically follows from the statements. Mark answer as (1) if conclusion I and II follow (2) if only conclusion II follow (3) if only conclusion I follows (4) if neither conclusion I nor II follows (5) None of these 22. Statement Industrial cities are highly polluted. Pollution means more diseases. Conclusions I. People living in cities which are not industrial are . healthier than those who live in industrial cities. II. People who live in industrial cities become immune to diseases. 23. Statement My brother sings very well. My sister is a basketball player. I am very inteliigent. Conclusions I. We all are very talented. II. We all are sportsmen. 24. Statement India’s economy depends mainly on forests. Conclusions I. Tree should be preserved to improve Indian Economy. II. India wants only maintenance of forests to improve economic conditions.

    BPRE–866

    25. Statement The best way to escape from a problem is to solve it. Conclusions I. Your life will be dull if you don’t face problem. II. To escape from problem, you should always have some solutions with you. Directions (26-30) : Below is a given a passage followed by several possible inferences which can be drawn from the facts stated in the passage. You have to examine each inference separately in the context of the passage and decide upon its degree of truth or falsify. Mark answer as (1) if the inference is ‘difinitely true’, ie, properly follows from the statement of facts given (2) if the inference is ‘probably false’, though not ‘definitely false’ in the light of the facts given (3) if the ‘data is inadequate’, ie, from the facts given you cannot say whether the inference is likely to be true or false (4) if the inference is ‘definitely false’, ie, it cannot possibly be drawn from the facts given or it contradicts the given facts (5) None of these Passage India’s economic and social progress is forever undermined by constant onslaught of drought followed by devastating floods. There is no denying that the country has registered growth in putting water to productive uses : for irrigation to realize the objectives of food security, of water conservation for crops requiring a considerable amount of water or even underground water for industrial and agricultural purposes. Despite this, the demand for water is visibly more than the supply of available and sustainable water. This is because water resources are not integrated in a systematic way to provide for varied uses. 26. The need of the hour is to manage water resources by integrated method. 27. Unavailability of sufficient water has had an adverse effect on India’s economic progress. 28. India does not lack in the supply of substainable water for various purposes.

    CONCLUSIONS/INFERENCES 29. India has achieved the maximum level possible in the channelling of its water resources. 30. The trend of conservation of water in India is a recent one. Directions (31-33) : In each of the following questions, a statement is given, followed by two conclusions I and II. Consider the statement and the following conclusions. Decide which of the conclusions follows from the statement. Mark answer as (1) if neither conclusion 1 nor II follows (2) if only conclusion II follows (3) if only conclusion I follows (4) if both conclusions I and II follow (5) None of these 31. Statement : Workers feel highly motivated when they get a sense of involvement by participating in the management of companiesConclusions : I. Workers should be motivated to produce more. II. Workers should be allowed to participate in the management of companies. 32. Statement : Power consumption in every family has been doubled during the last five years. Conclusions : I. There is a lot of development in the society. II. Power rates have become cheaper. 33. Statement : This world is neither good nor evil; each man manufactures a world for himself. Conclusions : I. Some people find this world quite good. II. Some people find this world quite bad. Directions (34-37) : In each of these, questions, a statement is followed by two conclusions I and II. Consider the statement to be true even if it seems to be at variance from commonly known facts. Decide which of the given conclusions is/are definitely drawn from the given statement. Mark answer us (1) if only conclusion I follows (2) if only conclusion II follows (3) if neither conclusion I nor II follows (4) if both conclusions I and II follow (5) None of these

    34. Statement : Black cloud follows thunder: rains follow thunder. Conclusions : I. Thunder is the cause of rain. II. Black cloud is the cause of thunder, 35. Statement : The secret of success is constancy of purpose. Conclusions : I. Constant dripping wears the stone. II. Single-minded devotion is necessary for achieving success. 36. Statement : Today, out of the world population of several thousand million, the majority of men have to live under government which refuse them personal liberty and the right to dissent. Conclusions : I. People are indifferent to personal liberty and the right to dissent. II. People desire personal liberty and the right to dissent. 37. Statement : The use of non-conventional sources of energy will eliminate the energy crisis in the world. Conclusions : I. Modern technology is gradually replacing the conventional source of energy. II. The excessive exploitation of environment has led to depletion of conventional sources of energy. Directions (38-42) : The questions given below have a statement followed by two conclusions I and II. Decide which of the conclusions follows from the statement. Mark answer as (1) if neither conclusion I follows nor II follows (2) if conclusion II follows (3) if conclusion I follows (4) if both conclusions I and II follow (5) None of these 38. Statement : The doctor is of the opinion that the patient’s condition would become normal after this operation Conclusions : I. The patient’s condition is such that he can be operated upon. II. Certain costly medicines can be administered to the patient and the operation may not be necessary. 39. Statement : Unemployment allowance should be given to all unem-

    BPRE–867

    ployed Indian youth, above 18 yr of age. Conclusions : I. There are a large number of unemployed young people in India who need monetary assistance. II. The Government of India has sufficient funds to provide unemployment allowance to all the unemployed young people. 40. Statement : If Mr Bhattacharya has gone through the instructions, he can start performing the activities. Conclusions : I. Mr. Bhattacharya shall understand the instructions. II. Mr. Bhattacharya is capable of performing the activities. 41. Statement : Krishna wrote another letter lo his mother after one month as he didn’t receive any reply to his first letter. Conclusions : I. Krishna’s mother didn’t receive the first letter. II. The letter normally reaches within a week. 42. Statement : Gaurav has got the railway reservation done in January this year for the journey he wants to make in April to Calcutta. Conclusions : I. The railways issue reservation 3 months in advance. II. There is more than one train to Calcutta every day. Directions (43-47) : The questions given below have a statement followed by two conclusions I and II. Decide which of the conclusions follows from the statement. Mark answer as (1) if conclusion I follows. (2) if conclusion II follows (3) if both conclusions follow (4) if neither conclusion follows. (5) None of these 43. Statement : Recent trends also indicate that the number of child migrants in large cities is increasing. These children leave their families to join the ranks of urban poor doing odd jobs in markets, workshops, hotels or in service sectors. Conclusions : I. Migration to big cities should be checked. II. The plight of poor children should be thoroughly studied.

    CONCLUSIONS/INFERENCES 44. Statement : The TV programmes, telecast specially for women are packed with a variety of recipes and household hints. A major portion of magazines for women also contains the items mentioned above. Conclusions : I. Women are not interested in other things. II. An average woman’s primary interest lies in home and specially in the kitchen. 45. Statement : Although, the education system has progressed from the point of view of the number of schools, most of them are ill-equipped and have not achieved excellence in imparting education. Conclusions : I. In future, we should provide good teachers and .. equipment to these schools. II. We need not open any more schools in the future. 46. Statement : Nation X faced growing international opposition for its decision to explode eight nuclear weapons at its test site. Conclusions : I. The citizens of the nation favoured the decision. II. Some powerful countries do not want other nations to become as powerful as they are. 47. Statement : Modern man influences his destiny by the choice he makes unlike in the past. Conclusions : I. Earlier, there were less options available to man. II. There was no desire in the past to influence the destiny. Directions (48-52) : These questions have a statement followed by two conclusions I and II. Decide which of the conclusions follows from the statement. Mark answer as (1) if conclusion I follows (2) if neither conclusion I follows nor II follows (3) if conclusion II follows (4) if both conclusions I and II follow (5) None of these 48. Statement : There has been an unprecedented increase in the prices of essential commodities like vegetables, pulses, fruits, sugar, milk, wheat, etc and increase in the prices of diesel and

    petrol has further aggravated the situation this year. Conclusions : I. The supply and demand must match and government must arrest the hoarders and middlemen who are creating an artificial scarcity of these essential items. II. The items in short supply can be imported and their indigenous production be increased. 49. Statement : The Reserve Bank of India has, on 20.3.2010, increased the Repo Rate by 25 basis points to 50. Conclusions : I. This will surely anchor inflation. II. The banks will now raise the lending and deposit rates. 50. Statement : Our parliamentary system is not very successful because most of our countrymen cast their votes based on caste and religion. Conclusions : I. Only educated and principled persons should be given the right to vote. II. We need to create an awareness in our people to rise above caste and religion at the time of casting their vote. 51. Statement : Against the backdrop of surging inflation, the Union Cabinet in March 2010 has decided to increase the Dearness Allowance for Central Government employees and pensioners by 8% with effect from January 2010. Conclusions : I. This will provide some relief to the Central Government employees and pensioners against the surging inflation. II. Increase in Dearness Allowance will neutralize, to some extent, increase in the cost of living. 52. Statement : Some people in this country want the President to be more than a figurehead and to take more active interest in national politics. Conclusions : I. In a parliamentary democracy, the President has to be content with a more or less passive role.

    BPRE–868

    II. The President, under the provisions of the Constitution, is bound by the advice of the Council of Ministers and so, he or she should not come into conflict with the Ministry or Parliament. Directions (53-57) : The questions given below has a statement followed by two conclusions I and II. Consider the statement and the following conclusions. Decide which of the conclusions follows from the statement, Mark answer as (1) if conclusion I follows (2) if conclusions II follows (3) if neither conclusion I follows nor II follows (4) if both conclusion I and II follow (5) None of these 53. Statement : It has been reported by one of the TV channels that the answer scripts of board examination of one state have been evaluated by students studying in the same standard with the help of model answers provided by the paper-setters. Conclusions : I. All such officials/evaluators who have been a part of this malpractice should be seriously dealt with. II. The board should explore the possibilities of getting these answer books evaluated by computerized machines, 54. Statement : Many of those who study in premier engineering, medical and management institutes in India, migrate to developed countries for better prospects in their professional pursuits, after they complete their study. Condusions : I. All the students joining these colleges should be asked to sign a bond at the time of admission, to the effect that they will remain in India, at least for ten years, after they complete their education. II. All those students who desire to settle down in the developed nations of the world should be asked to pay the entire cost of their education which the government has subsidized for them.

    CONCLUSIONS/INFERENCES 55. Statement : There has been a significant drop in the water level of all the water-bodies supplying water to the metro city of Mumbai. Conclusions : I. The water supply authority should continue to impose a partial cut in water supply to the consumers till the situation eases out. II. The government should appeal to all the residents through mass media for the minimal use of water. 56. Statement : The weather department has forecast a warning that a strong cyclonic storm will hit coastal Andhra Pradesh and Orissa in the next 48 hours. Conclusions : I. The local administration should soon send a message that all fishermen of that area avoid going deeper into the sea. II. Local administration should alert the people of coastal areas of both the states to move to safer places and administrative machinery should get ready for meeting out this eventuality. 57. Statement : The brave and alert villagers of Kandli village in Madhya Pradesh caught a group of dreaded dacoits armed with very sophisticated weapons and handed them over to the police. Conclusions : I. The villagers of Kandli village be provided sophisticated weapons to repeat this type of act of bravery in future. II. The villagers should be rewareded for their courage and unity. Directions (58-60) : Each of these questions has a statement followed by two conclusions I and II. Consider the statement and the following conclusions. Decide which of the conclusions follows from the statement. Mark answer as (1) if conclusion I follows (2) if conclusion II follows ; (3) if neither conclusion follows (4) if both conclusions follow (5) None of these 58. Statement : India has great potential for consumer products.

    Conclusions : I. Inflation is curbing demand for consumer products. II. A very large population of the country has a great appetite for consumer products. 59. Statement : Introduction of computers and networking has revolutionized banking services. Conclusions : I. Computers have reduced paperwork to a very large extent. II. All banking functions can be operated and viewed easily on computers. 60. Statement : Productivity of Indian agriculture is very low. Conclusions : I. Indian agriculture is largely dependent on timely rains. II. Indian farmers have not introduced latest technology into the system. 61. Study the following information carefully to answer the given question : A new apparel store has been opened by a telecom company XYZ, from which customers have very high expectations. Which of the following statements can be inferred from the given statement? (An inference is something by which you can logically deduce something to be true based on known/given premises.) (1) XYZ previously operated in telecom industry only. (2) Expansion is the best strategy to have competitive edge in the market. (3) To be successful in an expansion, one needs to be a leader in telecom industry. (4) XYZ has already established brand name in the market through its telecom business. (5) XYZ’s apparel store offers premium products for youngsters only. 62. Study the following information and answer the question given below : Continuous use of sanitizers should be avoided among young children as they ingest a certain quantity of sanitizer with every use which can increase the per cent content of chemical A in their body to as high as 0.04%.

    BPRE–869

    Which of the following can be concluded from the given statement? (A) Chemical A is either present in sanitizers or is formed in the body with the help of an agent present in the sanitizers. (B) Adults and older children are not affected by the sanitizers if ingested in small quantities. (C) Percentage of chemical A in the body under normal conditions is zero. (D) 0.03% of chemical A in the body does not lead to any pathological conditions. (1) Only (C) (2) Both (B) and (D) (3) Only (A) (4) Both (A) and (C) (5) Both (A) and (D) 63. Study the following information carefully and answer the question given below : “A variety of duplicate products have snatched our market. Our profits are reducing by almost 10% each passing year. Despite much longer durability of our handmade products, people these days are settling for the second copies only because of their prices and easy availability”- statement by the owner of a handmade products store in City Y. Which of the following can be inferred from the owner’s statement? (An inference is something which you can logically deduce based on the given facts) (1) The market of handmade products will dissolve completely in the coming few years. (2) Manufacturing of second copies or duplicate articles is a recent trend in City Y. (3) Reducing the price of handmade articles by 10% will attract more customers. (4) Making handmade products more widely available at reduced prices will help its market revive atleast to some extent (5) Durability is the least important factor that affects the choice of customers while buying such items. 64. Study the following information carefully to answer the given question :

    CONCLUSIONS/INFERENCES ‘To increase the sales our new product G, a free gift item worth Rs. 100 should be made available with the product to attract the customers for the next fifteen days” – Statement by Marketing Head of Company P after one week of launch of product G in the market. Which of the following can be inferred from the statement given by Marketing Head of the Company P? (1) None can be inferred (2) The cost of production of 10 units of product G is definitely less than Rs. 1000. (3) Giving free products with any product will necessarily increase its sales substantially. (4) Company P will be able to make sufficient profits with the sales of product G during next fifteen days only. (5) Ever since the product has been launched by the company in the market the sale was below expectation. 65. Statements : I. The Cabinet Committee on Economic Affairs approved strategic sale of government stake in Dredging Corporation of India to consortium of four ports - Vishakhapatnam Port Trust, Paradeep Port Trust, Jawahar Lal Nehru Port Trust and Kandla Port Trust. II. The government currently holds 73.44 per cent in Dredging Corporation of India Ltd (DCIL). The Dredging Corp stock rose up to 9.98% or 34.8 points to 383.15 level on the BSE. The micro cap stock has been gaining during recent times and risen 15.76% during the same period. It opened with a gain of 9.99% at 383.15 in trade as of now against its previous close of 348.35 level on BSE. Which of the following can be inferred from the above statements ? I. The approval is to avoid any further loss in dredging corporation handled by the Government. II. The approval will further facilitate the linkage of dredging activities with the ports. III. This will facilitate the expansion of dredging activity in the country as well as potential diversification of ports.

    IV. Since the growth and BSE rating was in rising trend, the Government could have earned a potentially big amount through the sales of the dredging corporation (1) I and II follow (2) II and III follow (3) I, II and III follow (4) II, III and IV follow (5) I, II, III and IV follow 66. Statements : I. The Zoological Survey of India has come with a new study titled Faunal Diversity of Biogeographic Zones: Islands of India, which proves that the islands, comprising only 0.25% of India’s geographical area, are home to more than 10% of the country’s fauna species. II. The publication, however, also cautions that tourism, illegal construction and mining are posing a threat to the islands’ biodiversity, which is already vulnerable to volatile climatic factors. III. The presence of a large number of species in such a small area makes the Andaman and Nicobar Islands one of the richest ecosystems and biodiversity hot spots in India. Some of the species in A&N Islands are restricted to a very small area and thus more vulnerable to any anthropogenic threat Which of the following can be concluded from the above statements all together? (1) The main motive of the publication is to showcase that Andaman and Nicobar islands of India is rich in biodiversity (2) The main motive of the publication is to showcase that the Andaman and Nicobar islands of India facing anthropogenic threats (3) The Publication also warns that eco tourism and constructions might pose threat to the rich bio-geographic zones of these islands (4) Some rare species are vulnerable to the ecosystems of the islands (5) None of these 67. The world’s largest supercomputer designed to work in the same way as the human brain has been switched on for the first time. The

    BPRE–870

    SpiNNaker machine, designed and built in The University of Manchester in the UK, can model more biological neurons in real time than any other machine on the planet. Biological neurons are basic brain cells present in the nervous system that communicate primarily by emitting ‘spikes’ of pure electro-chemical energy. Neuromorphic computing uses large scale computer systems containing electronic circuits to mimic these spikes in a machine. The SpiNNaker works more like a brain than a traditional computer. Conclusions : I. SpiNNaker can perform the tasks as just like how a human brain perceives and analysis the data and information. II. SpiNNaker can reduce the cluster of network confusions happen in human brain while analyzing complicated biological data. (1) Only Conclusion I Is True (2) Only Conclusion II Is True (3) Either Conclusion I Or Conclusion II Is True (4) Neither Conclusion I Nor Conclusion II Is True (5) Both The Conclusion I And Conclusion II Are True 68. The 9th Defense and Security dialogue was held between India and China at Beijing. Top officials of India and China held the ninth Annual Defense and Security Dialogue here after a one-year gap due to the Doklam standoff, as both countries agreed to enhance military exchanges and interactions. Indian Defense Secretary Sanjay Mitra and Lieutenant General Shao Yuanming from China’s Central Military Commission headed their respective delegations at this year's meeting, which was called off last year following a 72-day stand-off between the Indian Army (IA) and the People’s Liberation Army (PLA) at the Doklam tri-junction, which is situated along the disputed borders between India, China, and Bhutan. Both sides reiterated the importance of maintaining peace and tranquility in the border areas, implementing the consensus reached between Mr. Modi and Mr. Xi and specific additional

    CONCLUSIONS/INFERENCES confidence building measures at the operational level. Which of the following can be inferred from the above given statement as taken to be true? I. The security relationship between India and China was improving before one year II. The Defense security Dialogue is a tool to check the military security breach between the two nations III. The Political heads did not meet in the dialogue since there is strategic security difference between the countries IV. Indian and China have differences of issues only on border security (1) I and II (2) II and IV (3) III and IV (4) I, III and IV (5) II, III and IV

    SHORT ANSWERS 1.(1) 5.(2) 9.(1) 13.(1) 17.(3) 21.(4) 25.(2) 29.(1) 33.(4) 37.(1) 41.(3) 45.(1) 49.(3) 53.(1) 57.(2) 61.(4) 65.(4)

    2.(4) 6.(4) 10.(3) 14.(2) 18.(2) 22.(3) 26.(1) 30.(3) 34.(1) 38.(3) 42.(3) 46.(4) 50.(3) 54.(4) 58.(2) 62.(4) 66.(3)

    3.(3) 7.(4) 11.(1) 15.(4) 19.(1) 23.(4) 27.(3) 31.(2) 35.(4) 39.(1) 43.(2) 47.(1) 51.(4) 55.(4) 59.(1) 63.(5) 67.(2)

    4.(3) 8.(2) 12.(2) 16.(1) 20.(2) 24.(3) 28.(2) 32.(1) 36.(2) 40.(4) 44.(2) 48.(2) 52.(2) 56.(2) 60.(4) 64.(5) 68.(5)

    EXPLANATIONS 1. (1) The statement speaks about the play and writer and not about the story of the play. 2. (4) On the basis of the statement, we can not conclude anything as given in the three conclusions. Hence, none follows. 3. (3) The use of the word ‘different mutual funds’ clarifies the existence of more than one mutual funds today.

    4. (3) Either conclusion I or II follows since there is one of the two possibilities can be possible either Praveen got killed in the storm and not come back or Praveen has survived but he is not getting any route to come back. 5. (2) Statement implies that VCR can be imported but no tax is applicable, it does not mean that VCR are manufactured in the country. 6. (4) Both the conclusions are invalid in the light of the statement given. 7. (4) None of the conclusions follows. 8. (2) From the given figures, it is easily concluded that more students study with the same teacher in the rural areas as compared to those in the urban areas.. 9. (1) Clearly, conclusion 1 can be drawn from the statement. 10. (3) Comparative cost of transportation is not given, hence conclusion 1 is not implicit, conclusion II is a fact but not related with statement. Hence none of the conclusions follows. 11. (1) The manager disliking led to humiliation of Sachin. Hence, I follows. Nothing has been given about the popularity of Sachin hence, II does not follow. 12. (2) We can easily conclude from the statement that those who do not take dowry in marriages respect womanhood. 13. (1) In the last line of the passage, it is very clearly given that forest land requirement is about onesixth of 651 hectares area. 14. (2) No information is given to confirm that karakatla open cast mine is the only one of non-coking coal in the country. 15. (4) Inference is definitely false, 16. (1) It is given in the passage that production cost of 1.5 million tonne per annum is 67.82 crore. It means the production cost of one tonne of non-coking coal will be 450. 17. (3) The inference is probably true 18. (2) With improvements and greater efficiency... implies that the statement is probably true. 19. (1) The introduction of ... implies that the inference is definitely true.

    BPRE–871

    20. (2) The passage voucher for installing greater public confidence through increased resources mobilisation. Therefore, the given inference is probably true. 21. (4) In the first sentence of the passage, is mention of reforms being aimed at having an impact over a wide of cross-section of the economy. Therefore, the inference is definitely false. 22. (3) It cannot be concluded that people living in industrial cities become immune to disease. However, it can be concluded that people living in non-industrial cities are comparatively healthy. 23. (4) None follows. 24. (3) Conclusion I goes with the statement. The use of the word only makes the conclusion doubtful. 25. (2) It cannot be definitely said that life will be dull or not dull if you don’t face problem. However, if we face a problem, in order to escape from it, we should have solution for it. 26. (1) This is a very much clear from the last line of passage. 27. (3) Data is not adequate. 28. (2) Probably false. 29. (1) It is given in the para that India had done everything to channelise its water resources, though not systematic. 30. (3) Nothing is given about its being old or new. 31. (2) Conclusion II follows because for motivating workers, they should be allowed to participate in the management of companies. Conclusion I does not follow because it is not given that motivated workers produce more. 32. (1) Neither conclusion I nor II follows because they are not directly linked with the statement. 33. (4) Both the conclusions follow because some people find this world quite good while some find it quite bad. 34. (1) Conclusion I follows because rains follow thunder so, thunder is the cause of rain. Conclusion II does not follow because thunder is the cause of black cloud. 35. (4) Both the conclusions follow because consistency is required for achieving success.

    CONCLUSIONS/INFERENCES 36. (2) Conclusion II follows because people desire personal liberty and the right to dissent. Conclusion I does not follow because people are not different to personal liberty and the right to dissent. 37. (1) Conclusion I follows because modem technology is gradually replacing the conventional source of energy. Conclusion II does not follow because the given statement is not telling about environment. 38. (3) Conclusion I follows because according to doctor, patient’s condition is such, that it needs operation. Conclusion II is not directly linked with the statement. 39. (1) Unemployed young people need either a job or some monetary assistance. So, conclusion I follows. Conclusion II is not directly linked with statement. 40. (4) Both the conclusions follow because after understanding the instructions Mr Bhattacharya performs the activities because he is capable to do so. 41. (3) Conclusion I follows because Krishna’s mother might not received the letter. Conclusion II does not follow because if a letter normally reaches within a week then Krishna wrote another letter after one week. 42. (3) Conclusion I follows because Gaurav booked the ticket three months in advance. Conclusion II is not directly linked with the statement. 43. (2) Conclusion II follows because it should be thoroughly studied that why poor children are migrating in larger cities, 44. (2) If TV programmes magazines that are mainly for women contains a variety of recipes and household hints then it can be concluded that an average woman’s primary interest lies in household things. So, conclusion II follows. 45. (1) Conclusion I follows because these new schools need good teachers and equipments to improve education system. Conclusion II does not follow because to improve education system, we need to open schools but fully equipped.

    46. (4) Nuclear weapons are dangerous to humanity. So, a nation except those who already have nuclear weapons should not test for a nuclear weapons. A country is said to be powerful when its citizens are happy and satisfied ie, developed countries. So, conclusion II does not follow. Conclusion I is not directly linked with the statement. 47. (1) Conclusion I follows because mordern man has more options than ancient man. Conclusion II does not follow because man has always desire for some thing. 48. (2) In the statement, it is not given that prices are rising because of lesser supply but both the conclusions are telling about only demand and supply. 49. (3) Conclusion II follows because after increasing repo rate, banks will increase their lending and deposit rates. 50. (3) As it is a fundamental right of an adult citizen to cast his vote so conclusion 1 does not follow. Conclusion II follows because people should cast their vote to the most eligible candidate. 51. (4) As inflation is rising so increasing dearness allowance will provide some relief to the central government employees and pensioners and it will neutralize to some extent, increase in the cost of living. So, both the conclusions follow. 52. (2) None of the conclusion is directly linked with the statement. So, neither conclusion I follows nor II follows. 53. (1) Only conclusion I follows because it is a very serious matter and strict action should be taken against all such officials/evaluators. Conclusions II does not linked with statement. 54. (4) Both conclusions I and II follow because India is a developing country and government has subsidized fees in these institutes so, that students of these institutes made India a developed country. 55. (4) Both conclusions I and II follow because a partial cut will not affect the consumers and consumers should minimise the use of water till the situation cases out.

    BPRE–872

    56. (2) Conclusion II follows Administration should be ready so, that damage by cyclone should be lesser. Fishermen of that area should avoid going into the sea. The word ‘deeper’ makes conclusion I meaningless. 57. (2) Conclusion II follows because villagers should be rewarded for their courage and unity but they should not be allowed to take law in their hand. 58. (2) Only Conclusion II follows. A large number of people have strong urge for the consumer products. Therefore, India has great potential for consumer products. 59. (1) Only Conclusion I follows. Introduction of computers in baking sector has revolutionised the banking services. Several new services have been launched by the banks. The use of term ‘all’ in the Conclusion II makes it invalid. 60. (4) Low productivity is an outcome of several factors, viz., lack of irrigation facility, improper size of holdings, good seeds, proper manure etc. Therefore, both the Conclusions follow. 61. (4) Obviously, option (4) can be inferred from the given statement. Famous brand name finds it easier to launch a variety of products. 62. (4) Obviously, both (A) and (C) can be concluded from the given statement. 63. (5) People in City Y are buying second copies of products because of their prices and easy availability. Therefore, option (4) may be an assumption. Option (5) can be inferred from the information given in the owner’s statement. 64. (5) Obviously, option (5) can be inferred from the statement given by Marketing Head of the Company P. It was decided to launch the scheme of free gift item after one week of the launch of product G in the market. 65. (4) Obviously, II, III and IV follow from the given information. 66. (3) Obviously, option (3) can be concluded from the giveninformation. 67. (2) Obviously, only Conclusion II is true. 68. (5) Obviously, II, III and IV are true. ❑❑❑

    COURSES OF ACTION

    16

    COURSES OF ACTION

    QUESTIONS FROM 1999 TO 2010 ARE AVAILABLE ONLINE NATIONALISED BANKS & IBPS PO/MT/SO EXAMS Directions (1-10) : Study the following information carefully and answer the questions which follow: (Punjab & Sind Bank PO Exam. 23.01.2011)

    A Research Institute is recruiting a librarian to digitise its information resources among other duties. Candidates must possess the following criteria. The candidate must — (i) be not less than 35 years and not exceed 40 years as on 01 .11 .2009. (ii) have a Bachelor’s Degree in Library and Information Science with 65 percent marks. (iii) have a Ph.D. in Library Science. (iv) have post qualification experience of at least 4 years in a University Library. However if the candidate fulfills the above mentioned criteria except— (A) at (ii) above but the candidate has a UGC NET certification with all the other above criteria fulfilled he/she may be referred to the Dean. (B) at (iv) above but all the eligibility criteria are met and the candidate has at least one year’s experience in a research institute he/she may be offered contractual appointment for a year. Based on the above criteria study carefully whether the following candidates are eligible for the recruitment process and mark your answer as follows. You are not to assume anything other than the information provided in each question. All cases are given to you as on 01.11.2009. Mark answer (1) if he/she is to be shortlisted. Mark answer (2) if he/she is not to be shortlisted. Mark answer (3) if he/she should be referred to the Dean.

    Mark answer (4) if he/she may be offered contractual appointment if required. Mark answer (5) if the data provided is inadequate to take a decision. 1. Anil Rath has a doctorate in Library Science from Karnataka University in 2003. Born on July 21,1969, he graduated in Library and Information Science from Karnataka University where he was Assistant Librarian for four years since 2005. 2. Dr. Samir Bali has a Ph.D. in Library Science and has been with the Institute of Fundamental Research as Assistant Librarian since October 2008. He graduated with a degree in Library and Information Science in 1994 at the age of 22. He obtained 70 percent in his graduation. 3. Vaishali Shetty has been a librarian at STS University since 2007 when she qualified in the UGC NET examination. She has a degree in Library and Information Science with 60 percent. Her first job was as junior librarian at TRP Institute of Development Research from October 2000 to December 2001. 4. Vivek Jha has a Ph.D. in Library and Information Science. He graduated in Library and Information Science in 1992 with 65 percent. He was born on 01.10.1974. Since July 2005, he has been working as Deputy Librarian at a deemed University. 5. A graduate in Library Science with 69 percent, Dr. M. Puri has been working at Ranchi University for 4 years as Deputy Librarian. She earned her doctorate while working for the Labour Research Institute for 5 years as Assistant Librarian. She is 38 years old as on the required date. 6. Megha Vaidya has a graduate degree in Library Science from Pun-

    BPRE–873

    jab University where she has been a librarian for the past 5 years. In 2002 she obtained her UGC NET qualification at the age of 29. She obtained 72 percent in graduation. 7. Anup Gupta is obtaining his Ph.D. from YCM University where he has been Junior Librarian since 2004. He qualified in the UGC NET exam in June 2000. He has a degree in Library and Information Science with 62 percent. His date of birth is 17.10.1973. 8. Kirti Shukla obtained her doctorate and Bachelor’s degree from Patna University. She obtained 63 percent at graduation. She obtained her UGC NET qualification in 1998 when she was 26. 9. Prakash Sinha has been a librarian for a government institution for three years. Prior to this he was a University Librarian for 7 years after completing his Ph.D. in Library Science. He graduated in 1991 with 68 percent in Library Science. He is exactly 40 years of age on the specified date. 10. Rohan Sachdev obtained his UGC NET qualification after his graduation in 1998 when he began working. He has been Assistant Librarian for the past two years with a research institute. He obtained 65 percent in his graduation in Library Science. He earned his Ph.D. in Library Science in 2007. His date of birth is 22.10.1974. Directions (11-15) : In each question below a statement is given followed by two courses of action numbered I and II, A course of action is a practicable and feasible step or administrative decision to be taken for follow-up, improvement, or further action in regard to the problem, policy, etc. On the basis of the information given in the statement, you have to assume everything in the statement to be true, and decide which of the suggested courses of action logically follow(s) for pursuing. (UCO Bank PO Exam. 30.01.2011)

    COURSES OF ACTION Give answer (1) if only course of action I follows. Give answer (2) if only course of action II follows. Give answer (3) if either course of action I or course of action” follows. Give answer (4) if neither course of action I nor course of action II follows. Give answer (5) if both courses of action I and II follow. 11. Statement : Despite all the efforts by the government, thirty cases of polio were reported in State X as the parents of those children remained uninformed of this drive against polio. Courses of action : I. Strict action should be taken against the parents of children affected from polio. II. Strict action should be taken against the doctors administering polio drops in the state. 12. Statement : Private airliners have increased the fare during festive season by almost 50%. Courses of action : I. People should be advised to travel only by buses and trains during the festive season. II. Airfares should be regulated by the government to a certain limit. 13. Statement : Several passengers were stranded at the airport as all the flights had to be cancelled due to severe snowfall for the third consecutive day. Courses of action : I. All the flights should be immediately resumed. II. Airport authority should provide appropriate stay arrangements for the stranded passengers. 14. Statement : A practice march by the Army is scheduled to be held on one of the busiest roads in the city on Tuesday. Courses of action : I. Traffic should be diverted during the practice march so as to ease traffic congestion during that time.

    II. Public notices should be issued to avoid using the road till the practice march on Tuesday gets over. 15. Statement : Very few students have been opting for academic research as professional courses offer better economic dividends. Courses of action : I. Academic research should be linked to industry so as to improve its economic perspective. II. Economic dividends of professional courses should be made less attractive so as to attract students to academic research as well. Directions (16–20) : In each question below a statement is given followed by two courses of action numbered I and II. A course of action is a practicable and feasible step or administrative decision to be taken for follow-up, improvement, or further action in regard to the problem, policy, etc. On the basis of the information given in the statement, you have to assume everything in the statement to be true, and decide which of the suggested courses of action logically follow(s) for pursuing. (Bank Of Baroda PO Exam. 13.03.2011)

    Give answer (1) if only I follows. Give answer (2) if only II follows. Give answer (3) if either I or II follows. Give answer (4) if neither I nor II follows. Give answer (5) if both I and II follow. 16. Statement : People see tax as a burden and thus devise ways to underpay or avoid it altogether. Courses of Action : I. Government should educate and inform citizens about the ways in which taxes help in development of the nation. II. Tax rates should be increased so that the under-recovery in collection is compensated. 17. Statement : There is no motivation among today’s generation to join the armed forces owing to frequent transfers to risky areas. Perhaps they are not aware of the good side of it. Courses of action : I. Short term internship should be introduced at high school

    BPRE–874

    level to give students a peek into the adventurous life of the forces and provide a more realistic job purview. II. The salary levels of the defense forces should be increased with immediate effect. 18. Statement : A plethora of bodies with no single one which can be held accountable for city’s development is the key reason behind its rather stagnant state. Courses of action : I. Each body should be given a set of duties and there should be no duplication in such allocation. II. An umbrella committee should be appointed with powers to regulate actions of all other bodies. 19. Statement : Number of hospital beds per 10000 people in India, which is a key indicator of healthcare infrastructure, is much lower than many other nations which are thought to be poorer than India. Courses of action : I. The expenditure of importing costly medicines should be borne by the State. II. Incentives should be given to encourage entry of private players into building of healthcare infrastructure. 20. There is an alarming trend of skewed sex ratio against women in India during the past decade and situation may get out of hand if adequate steps are not taken to stop female foeticide. Which of the following can be an effective step to reverse the trend ? (A) The Government should immediately completely ban use of scanners/ sonography on expectant mothers at all health centres (B) The Government should announce a substantial incentive scheme for couples who have at least one girl child. (C) The Government should launch a nationwide campaign to create awareness against female foeticide.

    COURSES OF ACTION (1) (A) only (2) (A) and (B) Only (3) (B) and (C) Only (4) All (A), (B) and (C) (5) None of these (IBPS Bank PO/MT CWE Exam.18.09.2011)

    Directions (21-25) : Study the following information carefully and answer the questions given below : (IBPS Specialist Officer CWE Exam. 11.03.2012)

    Following are the conditions for selecting Manager-HR in an organisation : The candidate must — (i) be at least 30 years and not more than 35 years as on 01.03.2012. (ii) have secured at least 60 percent marks in Graduation in any discipline. (iii) have secured at least 65 percent marks in the Post Graduate Degree/ Diploma in Personnel Management/ HR. (iv) have post qualification work experience of at least five years in the Personnel/ HR Department of an organisation (v) have secured at least 50 percent marks in the selection process. In the case of a candidate who satisfies all the above conditions except(a) at (ii) above, but has secured at least 55 percent marks in Graduation in any discipline and at least 70 percent marks in Post Graduate Degree/Diploma in Personnel Management/HR, the case is to be referred to GM - HR. (b) at (iv) above, but has post qualification work experience of at least four years out of which at least two years as Deputy Manager–HR. the case is to be referred to President - HR. In each question below are given details of one candidate. You have to take one of the following courses of actions based on the information provided and the conditions and sub-conditions given above and mark the num-

    ber of that course of action as your answer. You are not to assume anything other than the information provided in each question. All these cases are given to you as on 01.03.2012. Mark answer (1) if the candidate is not to be selected Mark answer (2) if the data provided are not adequate to take a decision. Mark answer (3) if the case is to be referred to President-HR Mark answer (4) if the case is to be referred to GM-HR Mark answer (5) if the candidate is to be selected 21. Rita Bhatt was born on 25th July 1978. She has secured 62 per cent marks in Graduation and 65 per cent marks in Post Graduate Diploma in Management. She has been working for the past six years in the Personnel Department of an organisation after completing her Post Graduation. She has secured 55 per cent marks in the selection process. 22. Ashok Pradhan was born on 8th August 1980. He has been working in the Personnel Department of an organisation for the past four years after completing his Post Graduate Degree in Personnel Management with 67 percent. Out of his entire experience, he has been working for the past two years as Deputy Manager-HR. He has secured 62 percent marks in Graduation and 58 per cent marks in the selection process. 23. Alok Verma was born on 4th March 1976. He has been working in the Personnel Department of an organisation for the past six years after completing his Post Graduate Diploma in Personnel Management with 66 per cent marks. He has secured 57 per cent marks in the selection process and 63 per cent marks in Graduation. 24. Swapan Ghosh has been working in the Personnel Department of an organisation for the past five years after completing his Post Graduate Degree in HR with 72 per cent marks. He has secured 56 per cent marks in Graduation.

    BPRE–875

    He was born on 12th May 1977. He has secured 58 per cent marks in the selection process. 25. Seema Behl has been working in the Personnel Department of an organisation for the past seven years after completing her Post Graduate Diploma in Personnel Management with 70 per cent marks. She was born on 5th July 1979. She has secured 65 per cent marks in Graduation and 50 per cent marks in the selection process. 26. Statement : The prices of vegetables and other food articles have decreased in the recent months raising hope among policy planners that the RBI’s (Reserve Bank of India) tight grip on supply of liquid money in the market for controlling inflation may be eased. Which of the following may be a possible action of the above situation? (1) The Reserve Bank of India may not reduce its key interest rates in near future. (2) The Government may step in and make certain concessions to the common people on various issues (3) The Reserve Bank of India may consider lowering certain rates to bring in more liquidity in the market. (4) The RBI may wait for at least another year before taking any step. (5) The RBI may collect more data from the market and wait for another four months to ensure they take the correct step. (IBPS Specialist Officer CWE Exam. 11.03.2012)

    27. Statement : A severe cyclonic storm hit the Eastern coastline last month resulting in huge loss of life and property on the entire east coast and the Government had to disburse a considerable amount for relief activities through the district administration machineries. Which of the following may possibly be a follow up measure to be taken up by the Government?

    COURSES OF ACTION (1) The Government may set up a task force to review the post relief scenario in all districts and also to confirm proper end user receipt of the relief supplies. (2) The Government may set up a committee for proper disbursement of relief supplies in future. (3) The Government may empower the District Magistrates to make all future disbursements of relief. (4) The Government may send relief supplies to the affected people in future only after proper assessment of the damage caused by such calamities. (5) The Government may not need to activate any follow up measure. (IBPS Specialist Officer CWE Exam. 11.03.2012)

    Directions (28-32) : Study the following information carefully and answer the questions given below : (IBPS Specialist Office CWE Exam. 11.03.2012)

    Following are the conditions for selecting Manager-HR in an organisation : The candidate must — (i) be at least 30 years and not more than 35 years as on 01.03.2012. (ii) have secured at least 60 percent marks in Graduation in any discipline. (iii) have secured at least 65 percent marks in the Post Graduate Degree/ Diploma in Personnel Management/ HR. (iv) have post qualification work experience of at least five years in the Personnel/ HR Department of an organisation (v) have secured at least 50 percent marks in the selection process. In the case of a candidate who satisfies all the above conditions except(a) at (ii) above, but has secured at least 55 percent marks in Graduation in any discipline and at least 70 percent marks in Post Graduate Degree/Di-

    ploma in Personnel Management/HR, the case is to be referred to GM - HR. (b) at (iv) above, but has post qualification work experience of at least four years out of which at least two years as Deputy Manager–HR. the case is to be referred to President - HR. In each question below are given details of one candidate. You have to take one of the following courses of actions based on the information provided and the conditions and sub-conditions given above and mark the number of that course of action as your answer. You are not to assume anything other than the information provided in each question. All these cases are given to you as on 01.03.2012. Mark answer (1) if the candidate is not to be selected Mark answer (2) if the data provided are not adequate to take a decision. Mark answer (3) if the case is to be referred to President-HR Mark answer (4) if the case is to be referred to GM-HR Mark answer (5) if the candidate is to be selected 28. Rita Bhatt was born on 25th July 1978. She has secured 62 per cent marks in Graduation and 65 per cent marks in Post Graduate Diploma in Management. She has been working for the past six years in the Personnel Department of an organisation after completing her Post Graduation. She has secured 55 per cent marks in the selection process. 29. Ashok Pradhan was born on 8th August 1980. He has been working in the Personnel Department of an organisation for the past four years after completing his Post Graduate Degree in Personnel Management with 67 percent. Out of his entire experience, he has been working for the past two years as Deputy Manager-HR. He has secured 62 percent marks in Graduation and 58 per cent marks in the selection process.

    BPRE–876

    30. Alok Verma was born on 4th March 1976. He has been working in the Personnel Department of an organisation for the past six years after completing his Post Graduate Diploma in Personnel Management with 66 per cent marks. He has secured 57 per cent marks in the selection process and 63 per cent marks in Graduation. 31. Swapan Ghosh has been working in the Personnel Department of an organisation for the past five years after completing his Post Graduate Degree in HR with 72 per cent marks. He has secured 56 per cent marks in Graduation. He was born on 12th May 1977. He has secured 58 per cent marks in the selection process. 32. Seema Behl has been working in the Personnel Department of an organisation for the past seven years after completing her Post Graduate Diploma in Personnel Management with 70 per cent marks. She was born on 5th July 1979. She has secured 65 per cent marks in Graduation and 50 per cent marks in the selection process. Directions (33–37) : In each question below is given a statement followed by two courses of action numbered I and II. A course of action is a practicable and feasible step or administrative decision to be taken for follow-up, improvement, or further action in regard to the problem, policy, etc. On the basis of the information given in the statement, you have to assume everything in the statement to be true, and decide which of the suggested courses of action logically follow (s) for pursuing. (Indian Overseas Bank PO Online Exam, 01.09.2013)

    Give answer (1) if only I follows. Give answer (2) if only II follows. Give answer (3) if either I or II follows. Give answer (4) if neither I nor II follows. Give answer (5) if both I and II follow. 33. Statement : The sales of ballpoint pens manufactured by company Lixus have gone down considerably ever since the same

    COURSES OF ACTION

    34.

    35.

    36.

    37.

    company introduced a gel-ink pen in the market. Courses of action I. Ball point pens should not be manufactured by Lixus any more. II. Lixus should immediately withdraw all gel-ink pens from the market so as to force people to buy ball-point pens. Statement : Nearly 26 per cent of all engineering graduates have been left unemployed owing to severe recession worldwide. Courses of action I. All the unemployed students should be advised to take up jobs in foreign countries. II. The unemployed students should be advised to take up jobs only after the recession is over. Statement : The government airline has been making huge losses even as private airliners continue to prosper and make substantial profit. Courses of action I. All the private airlines should be banned from their operation in the country. II. The government airline should be instructed to increase passenger fare significantly in order to increase its profit. Statement : Many private schools have been violating government directives and have been charging more fees than the specified limits. Courses of action I. The government should identify such schools and take stringent actions against them. II. The government should not limit school fees and keep it flexible for the schools to decide. Statement : A large number of patients admitted in a particular hospital have been diagnosed with a serious contagious disease. Courses of action I. These patients should be immediately evicted from the hospital for fear of spread of the disease.

    II. The hospital should make appropriate quarantine arrangements to control the spread of disease. Directions (38– 42) : In each question below is given a statement followed by three courses of action numbered I, II and III. A course of action is a step or administrative decision to be taken for improvement, follow-up or further action in regard to the problem, policy, etc. On the basis of the information given in the statement, you have to assume everything in the statement to be true, then decide which of the suggested courses of action logically follow(s) for pursuing. (Corporation Bank SO (Marketing) Exam, 22.02.2014)

    38. Statement : The major road connecting the two main parts of the city is inundated due to heavy rains during past two days. Courses of action : I. The government should immediately send a relief team to the affected area. II. The municipal authority should make immediate effort to pump out water from the road. III. The municipal authority should advise the general public to stay indoors till the water is cleared. (1) Only I follows (2) Only I and II follow (3) Only I and III follow (4) Only II and III follow (5) None of these 39. Statement : The prices of essential commodities have gone up substantially during the past few weeks. Courses of action : I. The government should set up an expert committee to study the trend of prices. II. The government should immediately abolish taxes on essential commodities. III. The government should advise the general public to refrain from purchasing essential commodities for few days. (1) None follows (2) Only I follows (3) Only II follows (4) Only III follows (5) Only I and II follow

    BPRE–877

    40. Statement : Large number of students of the local school fell ill after having their midday meal provided by the school. Courses of action : I. The government should immediately suspend the school Principal. II. The school authority should initiate an enquiry to find out the reason for students’ sickness. III. The government should instruct all the schools to suspend their midday meal scheme till further order. (1) Only I follows (2) Only II follows (3) Only III follows (4) Only II and III follow (5) None of these 41. Statement : Many people living in the slums of western part of the town are diagnosed to be suffering from malaria. Courses of action : I. The municipal corporation should immediately make necessary arrangements to spray mosquito repellent in the affected area. II. The municipal authority should immediately make necessary arrangements to provide quick medical help to the affected people. III. The municipal authority should take steps to shift all the people staying in the slums of western part of the town to other areas. (1) Only I follows (2) Only II follows (3) Only I and II follow (4) All I, II and III follow (5) None of these 42. Statement : Many teachers of the local school have rendered their resignation to the Principal to protest the management’s decision for not implementing revised pay scales. Courses of action : I. The school management should accept the resignations and appoint new teachers. II. The school management should persuade the teachers

    COURSES OF ACTION to continue with an assurance of considering the pay hike issue. III. The school management should approach the government for guidelines. (1) None follows (2) Only either I or II follows (3) Only III follows (4) Only I follows (5) Only II follows Directions (43–47) : Study the following information carefully and answer the questions given below : (Corporation Bank SO (Marketing) Exam, 22.02.2014)

    Following are the conditions for selecting Marketing Manager in an organisation : The candidate must : (i) be at least 30 years old as on 01.03.2013 (ii) have secured at least 55 per cent marks in graduation (iii) have secured at least 60 per cent marks in Post graduate Degree/ Diploma in Marketing. (iv) have post qualification work experience of at least five years in the Marketing Division of an organisation. (v) have secured at least 45 per cent marks in the selection process. In the case of a candidate who satisfies all other conditions except — (A) at (iv) above, but has post qualification work experience of at least two years as Deputy Marketing Manager, the case is to be referred to GM-Marketing. (B) at (ii) above, but has secured at least 65 per cent marks in Post graduate Degree/Diploma in Marketing Management, the case is to be referred to Vice PresidentMarketing. In each question below is given details of one candidate. You have to take one of the following courses of actions based on the information provided and the conditions and sub-conditions given above and mark your answer accordingly. You are not to assume anything other than the information provided in each question. All these cases are given to you as on 01.03.2013. Mark answer (1) if the candidate is not to be selected.

    Mark answer (2) if the candidate is to be selected. Mark answer (3) if the data are inadequate to take a decision. Mark answer (4) if the case is to be referred to Vice President – Marketing. Mark answer (5) if the case is to be referred to GM-Marketing. 43. Suresh Mehta has secured 58 per cent marks in graduation. He was born on 19th May, 1979. He has secured 50 per cent marks in the selection process. He has been working for the past seven years in the Marketing division of an organisation after completing his Post Graduation with 62 per cent marks. 44. Sudha Gopalan has secured 50 per cent marks in both selection process and graduation. She has been working for the past six years in the Marketing division of an organisation after completing her Post Graduate Diploma in Marketing with 70 per cent marks. She was born on 14th October, 1982. 45. Divya Kohli has been working for the past five years in Marketing division of an organisation after completing her Post Graduate Diploma in Marketing with 65 per cent marks. She has secured 55 per cent marks in graduation and 50 per cent marks in the selection process. She was born on 2nd April, 1979. 46. Navin Marathe was born on 8th April, 1979. He has secured 60 per cent marks in both graduation and Post-Graduate Degree in Marketing. He has been working for the past six years in the Marketing division of an organisation after completing his PG Degree in Marketing. He has secured 50 per cent marks in the selection process. 47. Varun Malhotra was born on 3rd July, 1980. He has been working as Deputy Marketing Manager in an organisation for the past three years after completing his Post Graduate Degree in Marketing with 65 per cent marks. He secured 55 per cent marks in both graduation and selection process.

    BPRE–878

    48. Statements : The Country ‘X’ has the cheapest domestic airlines. The airline has lost around 40 commanders and senior pilots to Gulf airlines. Senior pilots have started opting Gulf airlines as they were unhappy with the domestic airlines. They complained that the cheapest domestic airline has poor facilities. Which of the following Courses of action may be pursued to solve the problem ? (A course of action is a step or administrative decision to be taken for improvement, follow up or further action in regard to the problem, policy etc.) (1) The Government of the Country ‘X’ should immediately take steps to ban the Gulf airlines so that senior pilots do not move to Gulf airlines. (2) The Government of the Country ‘X’ should immediately cancel the licences of the defiant senior pilots (3) The Government of the Country ‘X’ should immediately take steps to improve the service of the domestic airlines after meeting with the senior pilots (4) The passengers should boycott the services of Gulf airlines as it is the duty of citizens to protect the interests of their country. (5) None of these (IBPS RRBs Officer Scale-I CWE, 06.09.2014)

    49. Statements : Non-Resident Indians invest their money in City ‘A’. About one lakh flats are vacant in the City ‘A’. Still people are struggling hard to get the accommodation. Which of the following Courses of action may be pursued to solve the problem ? (A course of action is a step or administrative decision to be taken for improvement, follow up or further action in regard to the problem, policy etc.) (1) The Government should take over the vacant flats and allot these flats to the needy persons

    COURSES OF ACTION (2) The Government should restrict the investment made by the Non-Resident Indians to check the soaring prices of dwelling units (3) The Government should immediately constitute a task force to review the housing policy (4) The Government should immediately take steps to end the impasse through proper coordination and management (5) None of these (IBPS RRBs Officer Scale-I CWE, 06.09.2014)

    50. Study the following information carefully and answer the question which follows : The farmers of village D have been growing Crop Y for many years as there is hardly any rainfall in the area and Crop Y has minimum water requirements. However, this year many farmers of village D have sown Crop X instead of Crop Y despite the fact that Crop Y yielded a good produce last year. Which of the following statements, if considered true, may not be a reason for the action of the farmers of village D ? (1) In order to reduce the import of Crop X, the government of the State has declared certain incentives for farmers cultivating Crop X this year. (2) Crop X requires only a specific king of fertiliser which is not easily available as it is manufactured only in Village S, whereas any fertiliser with high nitrogen component is suitable for Crop Y. (3) The demand for Crop X has risen considerably over the past one year as a result farmers growing Crop X in the neighbouring villages of village D have made much more profit than expected. (4) The government had introduced a subsidy this year for good quality seeds of Crop X whereas seeds available in the market for Crop Y were of substndard quality. (5) A research committee, set up this year by the government

    has recommended that the soil and climate conditions of village D are the best for Crop X. (IBPS RRBs Officer Scale–I & II CWE 13.09.2015)

    51. Study the following information carefully and answer the question given below : Statement : The vehicular traffic in New Delhi area has increased considerably these days making it difficult for people to reach on time to Railway Station as vehicles frequently face traffic jams at various places. Courses of Action I. The vehicles which are not bound to Railway Station should not be allowed to ply on the road connecting the New Delhi area to the Railway Station. II. The load of vehicular traffic should be diverted through several link roads during the peak hours. III. The time–table of trains should be prepared so as to avoid the congestion during the peak hours. Which of the Courses of Action logically follow(s) for pursuing? (A ‘Course of Action’ is a step or administrative decision to be taken for improvement, follow–up or further action in regard to the problem, policy etc.) (1) Only I and II follow (2) Only I and III follow (3) Only II follows (4) Only II and III follow (5) All I, II and III follow (United Bank of India PGDBF Manipal Exam,07.08.2016)

    52. This question consists of a statement followed by two courses of action I and II given below it. A course of action is an administrative decision to be taken for improvement, follow-up or further action in regard to the problem, policy etc. You have to assume everything in the statement to be true and decide which of the suggested courses of actions logically follows from the given statement. Statement : ‘Global Girl’, a famous store exclusively for woman apparel, hired new designers 16 months ago for a contract of

    BPRE–879

    minimum 3 years. Though ‘Global Girl’ is dissatisfied with the repetitive designs, they cannot fire the designers owing to the contract. Course of Action I : A renowned designer should be hired by ‘Global Girl’ who can guide their designers with new designs as per the current trends. Course of Action II : The salaries of the designers should be considerably reduced and moreover, they should be issued a letter stating their disappointment with the designers. (1) Either I or II follows. (2) Both I and tl follow (3) Only II follows (4) Only I follows (5) Neither I nor II follows (Bank of Baroda Exam, 25.09.2016)

    53. Study the given information carefully to answer the question given below : The “City International School (CIS)” makes all its stationery items and self-published course books available with only ‘The Grand Book Store’. However, the book store has declared to remain closed in the first one and half months of this academic session due to some unavoidable circumstances. Which of the following is a viable course of action to be taken by CIS ? (A) Delay the first term exam of all its classes by two months. (B) Allow the students to attend classes without the books till it is made available. (C) Arrange to set up temporary canopies in the first few days of the session where the books and the stationery can be purchased. (D) Cancel the contract with ‘The Grand Book Store’ and join hands with some other chain of book stores. (1) (2) (3) (4) (5)

    Both B and C Only C Only D Both B and D Both A and C (Indian Bank PO (Pre.) Exam, 21.01.2017 (2nd Sitting))

    COURSES OF ACTION 54. Study the following information carefully and answer the question given below : Despite giving repeated warning to the science students of School Z it has been found in this academic year that students who have not opted for Biology frequently enter the Biology Lab without permission. A few of such students were also found to have damaged the microscopes in the laboratory. Which of the given two options can be a feasible course of action for the given situation? I. Biology students must be given additional ID cards specifically to enter the laboratory. Any student without the card should strictly not be allowed inside the lab unless specified by the teacher. II. Only the teacher should be allowed to touch or use the microscopes. Any student should not be allowed to touch or use the microscopes without prior permission of the concerned authority. (1) Only II is a feasible course of action. (2) Both I and II are feasible course of action. (3) Either I or II is a feasible courses of action. (4) Neither I nor II is a feasible course of action. (5) Only I is a feasible course of action. (IBPS SO (IT Officer) CWE (Prelim Exam) 30.12.2017)

    55. This question consists of a statement followed by two courses of action numbered I and II given below it. A course of action is an administrative decision to be taken for improvement, follow-up or further action in regard to the problem, policy etc. You have to assume everything in the statement to be true and then decide which of the suggested courses of action/s logically follow/s from the given statement. Statement : The cases of thefts in Metro trains in City E have been on rise since the past two months. Courses of Action : I. People should take care of their personal belongings and valuable

    items while travelling in the Metro and Metro police of City E along with the passengers should be alert all the time. II. Metro authorities in City E should restrict the number of passengers travelling in one compartment to only fifty passengers to avoid incidents of theft. (1) Either I or II follows (2) Only II follows (3) Both I and II follow (4) Only I follows (5) Neither I nor II follows (IBPS SO (Law Officer) CWE (Prelim Exam) 31.12.2017)

    56. This question consists of a statement followed by two courses of action numbered I and II given below it. A course of action is an administrative decision to be taken for improvement, follow-up or further action in regard to the problem, policy etc. You have to assume everything in the statement to be true and then decide which of the suggested courses of action/s logically follows from the given statement. Statement : The authorities of “Emperor’s Garden”, a famous tourist spot in City F, which is open for public visit free of cost for two months in a year, have complained about the cleanliness issue in the garden and damage to property caused by the public. Course of Action I : Authorities should restrict entry to a certain age group only. Course of Action II : All those found damaging the property should be penalised in monetary terms. (1) Neither I nor II follows (2) Only I follows (3) Only II follows (4) Both I and II follow (5) Either I or II follows (Canara Bank PO Exam 04.03.2018)

    Directions (57–61) : A company decided to appoint Content Manager (CM) to give a new impetus to its business. Following are the criteria laid down by the company. (IDBI Bank PO Exam 29.04.2018)

    BPRE–880

    The candidate must have (i) a Graduate degree from a recognized university with at least 65% marks. (ii) qualified in at least 5 written examinations of Bank PO. (iii) obtained at least 60% marks in the written test (total marks = 200) conducted by the company. (iv) obtained at least 40% marks out of 75 marks in the interview for the above post conducted by the company. (v) a working knowledge of computers. (vi) completed 30 year of age as on 14th January, 2012. However, in case of a candidate who fulfills all these criteria except (A) (i) above, but is a graduate, is to be referred to the Director of the company. (B) (iii) above, is to be referred to the Assistant Vice President of the company. Based on the above criteria and the information given in each of the following cases, you have to take decision. You are not to assume anything. In case you find that the given data are insufficient to make a decision, give ‘data inadequate’ as your answer. The cases are being given to you as on 14.01.2012. Given answer (1) If the candidate is to be selected as CM (2) If the candidate is not to be selected as CM (3) If the case is to be referred to the Director of the company (4) If the case is to be referred to the Assistant Vice President of the company (5) If the data are inadequate to take any decision 57. Surbhi Gaurav, was born on 12th January, 1981, has done her Graduation with 67% marks. It was the fifth interview for him when she appeared before the interview board constituted for Union Bank of India. She has a working knowledge of computers. She got 121 marks and 38 marks in the written examination and interview respectively, conducted by the company.

    COURSES OF ACTION 58. Vineet Pehal has a working knowledge of computers. He got 125 marks and 32 marks in the written examination and interview respectively, conducted by the company. When he appeared before the interview board for PO for Bank of India, it was his sixth interview for Bank PO. He performed a good dance on 10th January, 2012, on the eve of the birthday of his sister, who is 8 year younger than him. 59. Malik Aanand a talented working computer engineer, is 31 year of age. He has already qualified six exams for Bank POs. He is a Graduate with 65% marks. He got 60% marks in the interview and 40% marks in the written test conducted by the company for CM. 60. Shobha Vijay has a working knowledge of computers. She is a Graduate with 65% marks. She has qualified five written examinations of Bank POs. She has obtained 40% marks and 60% marks in interview and written examinations respectively, conducted by the company for CM. For the last three years her age has been more than 18. 61. Alankrita Mangal is Graduate with 65% marks. She has obtained 30 marks in the interview and 123 marks in the written examination conducted by the company, for CM. She is 32 year old and possesses working knowledge of computers. She has given more than five interview of Bank PO exam.

    SBI PO EXAMS Directions (1–4) : In each question below is given a statement followed by two courses of action numbered I and II. A course of action is a step or administrative decision to be taken for improvement, follow-up or further action in regard to the problem, policy, etc. On the basis of the information given in the statement, you have to assume everything in the statement to be true, then decide which of the suggested courses of action logically follow(s) for pursuing. (SBI Management Executive Exam, 23.02.2014)

    Give answer (1) if only course of action I follows. Give answer (2) if only course of action II follows. Give answer (3) if either course of action I or II follows. Give answer (4) if neither course of action I nor II follows. Give answer (5) if both courses of action I and II follow. 1. Statement : Drinking water supply to many parts of town is disrupted due to loss of water because of leakage in pipes supplying water. Courses of action : I. The government should order an enquiry into the matter. II. The civic body should set up a fact-finding team to assess the damage and take effective step. 2. Statement: There is an alarming increase in the number of people suffering from malaria in many parts of the city. Courses of action : I. The municipal corporation has advised all the government hospitals to store adequate supply of malaria drugs. II. The municipal corporation has urged people to use mosquito repellants and keep their premises clean. 3. Statement : Many people have encroached into the government property and built their houses and business establishments. Courses of action : I. The government should take immediate steps to remove all unauthorised constructions on government land. II. All the encroachers should immediately be put behind bars and also be slapped with a hefty fine. 4. Statement : The meteorological department has predicted normal rainfall throughout the country during the current monsoon. Courses of action : I. The government should reduce the procurement price of foodgrains for the current year.

    BPRE–881

    II. The government should reduce subsidy on fertilizers for the current year. 5. Statement : The number of dropouts in government schools has significantly increased in the urban areas over the past few years. Courses of action : I. The government should immediately close down all such schools in the urban areas where the dropout goes beyond 20 per cent. II. The parents of all the students who dropped out of the government schools in urban areas should be punished. 6. Read the following information carefully and answer the question which follows : Most students from state ‘X’ prefer to move base to other states for higher education. The government of the state ‘X’ is concerned about this problem as these students might not come back even after completing their education. Which of the following statements may be a course of action to deal with this problem ? (1) The government should set up well equipped state of the art universities which will provide quality education (2) The government should not give much importance to this problem as there is high density of population per square km in the state. (3) Scholarships should be declared for those students who perform well in tenth and twelfth standard examinations. (4) All the Colleges and Universities should be asked to emphasize the importance of higher education and benefits of studying in their institute should also be highlighted. (5) The government should make education free for all people of the state. (SBI Associates PO Online Exam, 29.11.2014)

    7. A statement is followed by two courses of action numbered I and II given below it. A course of action is an administrative

    COURSES OF ACTION decision to be taken for improvements, follow-up or further action in the regard to the problems, policy etc. You have to assume everything in the statement to be true and then decide which of the suggested courses of action logically follows from the given statement. Statement % There have been structural shifts from agriculture to industry in Country Z. While industries contribute to 67% of the country’s GDP, it is leading to a steady and undesirable decline in the participaion of labour force in agriculture. I. The government should curtail number of industries growing up in the country in order to promote the agriculture. II. Incentives for working in agriculture should be made competitive with that of other sectos. (1) Either Course of action I or II follows (2) Both Courses of action I and II follow (3) Neither Course of action I nor II follows (4) Only Course of action II follows (5) Only Course of action I follows (SBI Associates PO Online Exam, 30.11.2014)

    8. Read the given information carefully and answer the given question. Five fishermen from Country X were sentenced to imprisonment by the High Court of Country Y on charges of smuggling narcotics from Country X to Country Y. Which of the following supports the legal action taken by Country Y ? (A) Country X had given details of a few fishermen that had no record of involvement in drug related activities. (B) The naval authorities of both the Country X and Country Y have found traces of narcotics in the fish boxes that were shipped from Country X to Country Y by these fishermen.

    (C) All the fish boxes that are shipped between countries undergo a strict process of checking. (D) It was found that two of the five fishermen who were sentenced to death penalty were earlier imprisoned for the same reason a few years back. (1) Both B and C (2) Both A and B (3) Only D (4) None of the given statements (5) Only B (SBI Associates PO Online Exam, 30.11.2014)

    9. Statement% South African turtles known as Polka turtles, which have dots on their skin, are facing the threat of extinction as they are used as pets by humans. Which of the following steps should be taken by the Government to save these turtles? (1) Poachers should be prevented from poaching these turtles by the forest guards. (2) The term of imprisonment for poaching of Polka turtles should be increased from four years to 10 years. (3) The persons who make these turtles as their pets should be fined heavily. (4) The Government should take step to increase the breeding of Polka turtles. (5) None of these (SBI Management Executive Exam. 19.09.2014)

    10. Read the following information carefully and answer the question which follows : (SBI Associates PO Online Exam. 29.11.2014)

    Most students from state ‘X’ prefer to move base to other states for higher education. The government of the state ‘X’ is concerned about this problem as these students might not come back even after completing their education. Which of the following statements may be a course of action to deal with this problem ? (1) The government should set up well equipped state of the art universities which will provide quality education

    BPRE–882

    (2) The government should not give much importance to this problem as there is high density of population per square km in the state. (3) Scholarships should be declared for those students who perform well in tenth and twelfth standard examinations. (4) All the Colleges and Universities should be asked to emphasize the importance of higher education and benefits of studying in their institute should also be highlighted. (5) The government should make education free for all people of the state. 11. A statement is followed by two courses of action numbered I and II given below it. A course of action is an administrative decision to be taken for improvements, follow-up or further action in the regard to the problems, policy etc. You have to assume everything in the statement to be true and then decide which of the suggested courses of action logically follows from the given statement. Statement % There have been structural shifts from agriculture to industry in Country Z. While industries contribute to 67% of the country’s GDP, it is leading to a steady and undesirable decline in the participaion of labour force in agriculture. I.

    The government should curtail number of industries growing up in the country in order to promote the agriculture.

    II. Incentives for working in agriculture should be made competitive with that of other sectos. (1) Either Course of action I or II follows (2) Both Courses of action I and II follow (3) Neither Course of action I nor II follows (4) Only Course of action II follows (5) Only Course of action I follows (SBI Associates PO Online Exam. 30.11.2014)

    COURSES OF ACTION 12. Read the given information carefully and answer the given question. Five fishermen from Country X were sentenced to imprisonment by the High Court of Country Y on charges of smuggling narcotics from Country X to Country Y. Which of the following supports the legal action taken by Country Y ? (A) Country X had given details of a few fishermen that had no record of involvement in drug related activities. (B) The naval authorities of both the Country X and Country Y have found traces of narcotics in the fish boxes that were shipped from Country X to Country Y by these fishermen. (C) All the fish boxes that are shipped between countries undergo a strict process of checking. (D) It was found that two of the five fishermen who were sentenced to death penalty were earlier imprisoned for the same reason a few years back. (1) Both B and C (2) Both A and B (3) Only D (4) None of the given statements (5) Only B (SBI Associates PO Online Exam. 30.11.2014)

    RBI GRADE–B/ NABARD GRADE–A OFFICER EXAMS 1. This question consists of a statement followed by two courses of action numbered I and II given below it. A course of action is an administrative decision to be taken for improvement, follow-up or further action in regard to the problem, policy etc. You have to assume everything in the statement to be true and then decide which of the suggested courses of action logically follow(s) from the given statement. Statement : Most of the people looking for buying/renting prop-

    erties these days complain of being taken to the same property by more than 6-7 brokers. So even after contracting multiple agents, they end up having usually the same options. Course of action I : All the owners should strictly give the responsibility of their properties to only one broker. Course of action II : The brokers should be instructed to mandatorily disclose the list of all the properties they will be showing the customers on a particular day before taking them to the actual site. (1) Both I and II follow (2) Only II follows (3) Only I follows (4) Either I or II follows (5) Neither I nor II follows (RBI Officer Grade ‘B’ Phase-I Exam, 04.09.2016 (Shift-I))

    2. This question consists of a statement followed by two courses of action numbered I and II given below it. A course of action is an administrative decision to be taken for improvement, follow-up or further action in regard to the problem, policy etc. You have to assume everything in the statement to be true and then decide which of the suggested courses of action logically follow(s) from the given statement. Statement : In a competitive exam held across Country ‘Banimia’, more than 8 lakhs candidates appeared. But, about 4,000 candidates were found to be using unfair means, 75% of which were from State K of the Country. Course of Action I : Government of ‘Banimia’ should improve inspection and invigilation in all test centres of State K for all competitive exams. Course of Action II : From now on for any competitive exams in Banimia, any application from State K should not be entertained. (1) Only II follows (2) Only I follows (3) Either I or II follows (4) Neither I nor II follows (5) Both I and II follow (RBI Officer Grade ‘B’ Phase-I Exam, 04.09.2016 (Shift-II))

    BPRE–883

    3. Study the following informa-tion carefully to answer the given question : School ‘X’ had put up the following notice for its students and teachers regard-ing rules of term exams beginning from 20.03.2017 (Monday) • The projects/assignment for any subject of any class should be submitted atleast 40 days prior to the term examination for that particular subject. • Teachers should stop taking the remedial/extra classes before one month of the commencement of term exams. • The students who have not paid their computer lab fess should pay it before 06.03.2017 to be permitted for their computer exam. (Note: Only one exam is conducted per day. The sequence of subjects will be different for different classes.) The following cases were reported to the principal of School X. Which one of these should be punished under the given rules? (1) Geography will be the fourth exam for class 9. Priya Dandekar, a student of class 9 submitted her assignment for the subject on 16.02.2017. (B) Gautam Saxena, a Maths teacher of the school, finished all his extra classes by first week of March. (C) Ranvir Chauhan, paid his computer fees by the mid of February, but submit-ted its receipt to his class teacher on 05.03.2017. (1) Both (A) and (C) (2) Both (B) and (C) (3) Only (C) (4) Only (B) (5) Both (A) and (B) (NABARD Assistant Manager Online Exam 06.08.2017)

    4. The question consists of a statement followed by two courses of action numbered I and II. A course of action is an administrative decision to be taken for improvement, follow-up or further action in regard to the prob-

    COURSES OF ACTION lem, policy etc. You have to assume everything in the statement to be true and then decide which of the suggested courses of action/s logically follows from the given statement. Statement : The Country P is amongst the list of top ten most dirty countries of the world with unhygienic living conditions. Course of Action I : The Government of Country P should start special programs to create awareness amongst people about keeping their surroundings clean. Course of Action II : People who take part in cleanliness drives should be given recognition to motivate them and others. (1) Either I or II follows (2) Only I follows (3) Both I and II follow (4) Only II follows (5) Neither I nor II follows (RBI Assistant Manager Online Exam 25.03.2017)

    5. In this question a statement is given followed by two courses of action numbered I and II. A course of action is a practicable and feasible step or administrative decision to be taken for follow-up, improvement, or further action in regard to the problem, policy, etc. On the basis of the information given in the statement, you have to assume everything in the statement to be true, and decide which of the suggested courses of action logically follow(s) for pursuing. Statement : Company M is an IT company which also involves designing customized computer softwares for their clients once in every two to three months. Recently, two of the four software designers of Company M were on leave on the same days when one of the leading clients of the company gave a big scale project at a short notice. Company M suffered a loss as it could not complete the project on time. Course of Action I : Company M should avoid accepting any project from the mentioned client in future. Course of Action II : Company M should hire two more software designers every quarter to handle such exigencies.

    (1) (2) (3) (4) (5)

    Neither I nor II follows Both I and II follow Only II follows Either I or II follows Only I follows

    (RBI Officers in Grade ‘B’ Phase-I Exam, 16.08.2018)

    INSURANCE EXAMS 1. Below is given a statement followed by three courses of action numbered (A), (B) and (C). A course of action is a step or administrative decision to be taken for improvement, follow - up or further action in regard to the problem, policy etc. On the basis of the information given in the statement, you have to assume everything in the statement to be true, then decide which of the suggested courses of action logically follow(s) for pursuing. (LIC Assistant Administrative Officer (AAO) Exam. 12.05.2013)

    Statement : It was seen that due to the negligence of the caretaker and hostel attendant there was huge shortage of drinking water in the hostel. Courses of action : (A) The authority should immediately make arrangement for drinking water in the hostel. (B) The authority should take proper action against caretaker and hostel attendant. (C) The authority should dismiss the caretaker and hostel attendant. (1) Only A and C (2) Only B & C (3) Only A and B (4) Only C (5) None of these 2. Study the following information carefully and answer the question. “Although the State government has been funding College XYZ for the past 10 years, it has decided not to do so this year.” —Local news report. Which of the following may NOT be a reason for the decision of the government to discontinue the funding of the said college this year? (1) College XYZ has doubled the fees charged from students this year as a result of which it has made three times profit as compared to other State run colleges.

    BPRE–884

    (2) College has received the permission this year to lend its campus for marriages and other social events and this would be an additional source of income for the college. (3) It was recently found out that most of the bills submitted about the usage of the money received by the college as government funds are fabricated and show much more amount than the actual expenditure. (4) Despite repeated warning from the authorities since past two years, the college has continued to reserve some seats for students who are willing to pay huge donation in addition to fees. (5) Although the performance of students of XYZ has improved a little in academics this year, it has deteriorated drastically in sports and other extracurricular activities. (LIC Assistant Administrative Officer (AAO) Online Exam. 22.03.2015)

    3. Study the following information carefully to answer the given question. Even though Country X has developed a new vaccine called Stagerine for the treatment of an earlier incurable disease, the Health Ministry of Country X has recently ordered to discontinue its usage until further refinements. Which of the following strengthens the order given by the Health Ministry of Country X? (1) Country B has also started developing same medicine for the same disease. (2) ‘Cyamine’, the mixture of chemicals which is crucial for the development of Stcagerine, is available only with the scientists of Country B. (3) The vaccine Stagerine, though highly effective for the targeted disease has some long term side effects like skin infection, lever enlargements, swelling of bones etc. (4) Each vaccine costs around $69 which is not affordable by many citizens of the country. (5) The vaccine is made from the pollen of some flower which is

    COURSES OF ACTION available only in the forests in some remote locations of the country. (OICL Specialist Officer (Finance) Exam. 03.05.2015)

    4. In this question is a statement followed by two courses of action numbered I and II. A course of action is a step or administrative decision to be taken for improvement, follow up or further action in regard to the problem, policy etc. On the basis of information given in the statement, you have to assume everything in the statement to be true and then decide which of the suggested course of action logically follows for pursuing. Statement : Railways of country Buroon is facing losses due to reduction in fare by various airlines in the country. Course of actions : I. Railways of country Buroon should improve the quality of rail journey with respect to facilities. II. Number of schemes offering concessional fares should be restricted for airlines of country Buroon. (1) Either course of action I or course of action II follows (2) Only course of action II follows (3) Neither course of action I nor course of action II follows (4) Only course of action I follows (5) Both courses of action I and II follow (OICL Specialist Officer (Finance)

    Give answer (2) if only Course of action II follows Give answer (3) if either Course of action I or II follows Giver answer (4) if neither Course of action I nor II follows Give answer (5) if both the Courses of action I and II follow 5. Statement : The government airline has been making huge losses even as private airlines continue to prosper and make substantial profit. Course of Action I : All the private airlines should immediately be banned from their operation in the country. Course of Action II : The government airline should be instructed to reduce staff strength substantially in order to stop making losses. 6. Statement : A large number of patients admitted in a particular hospital have been diagnosed with a serious contagious disease. Course of Action I : These patients should immediately be discharged from the hospital and sent home. Course of Action II : These patients should immediately be quarantined in a secluded area to control the spread of disease.

    SHORT ANSWERS

    NATIONALISED BANKS & IBPS PO/MT/SO EXAMS

    SBI PO EXAMS 1. (4) 5. (4) 9. (3)

    2. (5) 6. (1) 10. (1)

    3. (1) 7. (4) 11. (4)

    4. (4) 8. (5) 12. (5)

    RBI GRADE–B/ NABARD GRADE–A OFFICER EXAMS 1. (1) 5. (1)

    2. (2)

    3. (5)

    4. (3)

    INSURANCE EXAMS 1. (3) 5. (4)

    2. (5) 6. (2)

    3. (3)

    4. (4)

    EXPLANATIONS

    NATIONALISED BANKS & IBPS PO/MT/SO EXAMS (1–10) : Candidate

    CRITERIA (i) (ii) or (A) (iii) (iv) or (B)

    Anil Rath Dr. Samil Bali Vaishali Shetty Vivek Jha Dr. M Puri Megha Vaidya Anup Gupta Kirty Shukla Prakash Sinha

    × ✓ NG ✓ ✓ ✓ ✓ ✓ ✓

    Rohan Sachedev ✓

    NG ✓ — ✓ ✓ ✓ — — ✓ ✓

    — ✓ ✓ — ✓ — ✓ NG — — ✓ ✓ — ✓ ✓ ✓ NG ✓ ✓ × ✓ — — NG — ✓ ✓

    — ✓ ✓ — ✓ — — — —









    Exam. 03.05.2015)

    Directions (5–6) : In each of the questions given below, a statement is given followed by two courses of action I and II. A course of action is a practicable and feasible step or administrative decision to be taken for follow-up, improvement, or further action in regard to the problem, policy etc. On the basis of the information given in the statement, you have to assume everything in the statement to be true, and decide which of the suggested courses of action logically follow(s) for pursuing. (GIC Officer Online Exam 15.05.2017)

    Give answer (1) if only Course of action I follows

    1. (2) 5. (1) 9. (1) 13. (2) 17. (5)

    2. (4) 6. (5) 10. (4) 14. (5) 18. (5)

    3. (5) 7. (2) 11. (4) 15. (1) 19. (2)

    4. (1) 8. (5) 12. (2) 16. (1) 20. (4)

    21. (5) 25. (5) 29. (3) 33. (4) 37. (2) 41. (4) 45. (1) 49. (4) 53. (1) 57. (1) 61. (1)

    22. (3) 26. (3) 30. (1) 34. (4) 38. (5) 42. (5) 46. (2) 50. (2) 54. (5) 58. (5)

    23. (1) 27. (2) 31. (4) 35. (4) 39. (3) 43. (2) 47. (5) 51. (3) 55. (4) 59. (4)

    24. (4) 28. (5) 32. (5) 36. (1) 40. (2) 44. (4) 48. (3) 52. (4) 56. (3) 60. (5)

    BPRE–885

    1. (2) Anil Rath does not satisfy criterion (I). There is no information about marks in Library and Information Science. 2. (4) Dr. Samil Bali does satisfy criteria (I), (II), (III) and (B). Therefore, he may be offered contractual appointment for one year. 3. (5) Vaishali Shetty does satisfy on ly criteria (A) and (B). 4. (1) Vivek Jha satisfies all the criteria. 5. (1) Dr. M puri satisfies all the criteria. 6. (5) There is no information about criterion (III). 7. (2) Anup Gupta does not satisfy criterion (III).

    COURSES OF ACTION 8. (5) The subject of graduation is not given. There is no information about criterion (IV). 9. (1) Prakash Sinha satisfies all the criteria. 10. (4) Rohan Sachdev satisfies criteria (I), (II), (III) and (B). 11. (4) None of the courses of action is feasible practically. Parents of such children should be motivated to take benefit of Pulse polio drive. The use of term ‘srict’ in both the courses of action makes them invalid. 12. (2) Course of action I seems to be an advice. Therefore, only course of action II seems to be appropriate. 13. (2) It is not possible to resume flights in bad weather conditions. Course of action II is appropriate. 14. (5) Obviously, both the courses of action are appropriate. 15. (1) Only course of action I seems to be appropriate. 16. (1) Only course of action I is suitable for pursuing. 17. (5) Obviously, both the courses of action are suitable. 18. (5) Obviously, both the courses of action are suitable. 19. (2) Only course of action II is suitable. 20. (4) All the three statements can be effective steps to reverse the trend. (21– 25) : Candidate

    (i) (ii) or (a) (iii) (iv) or (b) (v)

    Conditions

    Rita

    ✓ ✓

    — ✓







    Ashok

    ✓ ✓

    — ✓







    Alok

    ×

    — ✓







    Swapan ✓ —











    Seema

    — ✓









    ✓ ✓

    21. (5) Rita Bhatt satisfies all the conditions. Therefore, she would be selected. 22. (3) Ashok Pradhan satisfies conditions (i), (ii), (iii), (b) and (v). Therefore, his case would be referred to President — HR. 23. (1) Alok Verma does not satisfy condition (i). 24. (4) Swapan Ghosh satisfies conditions (i), (a), (iii), (iv) and (v).

    25.

    26. 27. (28–

    Therefore, his case would be referred to GM–HR. (5) Seema Behl satisfies conditions (i), (ii), (iii), (iv) and (v). Therefore, she would be selected. (3) Option (3) may be a possible action. (2) Option (2) may be a follow up measure. 32) :

    Candidate

    Conditions (i) (ii) or (a) (iii) (iv) or (b) (v)

    Rita

    ✓ ✓

    — ✓





    Ashok

    ✓ ✓

    — ✓







    Alok

    ×



    — ✓







    Swapan ✓ — Seema ✓ ✓

    ✓ ✓ — ✓

    ✓ ✓

    — —

    ✓ ✓



    28. (5) Rita Bhatt satisfies all the conditions. Therefore, she would be selected. 29. (3) Ashok Pradhan satisfies conditions (i), (ii), (iii), (b) and (v). Therefore, his case would be referred to President — HR. 30. (1) Alok Verma does not satisfy condition (i). 31. (4) Swapan Ghosh satisfies conditions (i), (a), (iii), (iv) and (v). Therefore, his case would be referred to GM–HR. 32. (5) Seema Behl satisfies conditions (i), (ii), (iii), (iv) and (v). Therefore, she would be selected. 33. (4) None of the courses of action is suitable for pursuing. If the company Lixus would not manufacture Ball point pens, some other company may introduce Ball point pens in the market. Similarly, some other company may introduce gel-ink pens in the market. 34. (4) As the recession is worldwide employment scenario in other countries would also have adversely affected. Therefore, course of action I is not suitable for pursuing. Course of action II lacks practical approach. 35. (4) None of the courses of action is suitable for pursuing. 36. (1) Only course of action I seems to be suitable for pursuing. 37. (2) Only course of action II is suitable for pursuing.

    BPRE–886

    38. (5) All the three courses of action are suitable for pursuing. The Courses of action mentioned here are appropriate to tackle the problem. 39. (3) Course of action II seems to be suitable for pursuing. The government should make effort to point out the cause of price rise instead of setting up an expert committee to study the trend of prices. Course of action III negates the importance of essential commodities. 40. (2) Only Course of action II seems to be suitable for pursuing. The suspension of school Principal cannot solve the problem. Similarly, suspension of the midday meal scheme may cause some other problem. 41. (4) All the three courses of action are suitable for pursuing. 42. (5) Only course of action II is suitable for pursuing. 43. (2) Suresh Mehta satisfies all the conditions (i), (ii), (iii), (iv) and (v). Therefore, he can be selected. 44. (4) Sudha Gopalan satisfies the conditions (i), (B), (iii), (iv) and (v). Therefore, her case would be referred to Vice President – Marketing. 45. (1) Divya Kohli does not satisfy condition (i). Therefore, she cannot be selected. 46. (2) Navin Marathe satisfies all the conditions (i), (ii), (iii), (iv) and (v). Therefore, he can be selected. 47. (5) Varun Malhotra satisfies the conditions (i), (ii), (iii), (A) and (v). Therefore, his case should be referred to GM-Marketing. 48. (3) Clearly, Option (3) is suitable for pursuing. 49. (4) Clearly, Option (4) is suitable for pursuing. 50. (2) Obviously, option (2) may not be a reason for the action of the farmers of village D. 51. (3) Ovbviously, only Course of Action II seems to be suitable for pursuing. 52. (4) Only course of action I seems to be appropriate for pursuing. 53. (1) Course of Action (A) is not a practical step. Only Courses of Action (B) and (C) seem to be suitable.

    COURSES OF ACTION Course of Action (D) seems to be very harsh decision. 54. (5) Obviously, only Course of action I is suitable for pursuing. Course of action II may hinder the proper learning of the students. 55. (4) Only course of action I seems to be appropriate for pursuing. Course of action II is not feasible practically considering the rush in Metros. 56. (3) Course of action I does not seem to be appropriate. Only course of action II seems to be suitable for pursuing. (57–61) :

    Surbhi

    CRITERIA (i) or (A) (ii) (iii) or (B) (iv) (v) (vi) ü – ü ü – ü ü ü

    Vineet

    NG –

    ü ü









    Candiate

    ü ü – Alankrita ü – Malik

    Shobha

    ü ü ü ü ü

    ü ü NG ü ü ü – ü ü NG – ü ü ü

    57. (1) Surbhi Gaurav satisfies all the criteria. Therefore, she can be selected. 58. (5) Vineet Pehal satisfies criteria (ii), (iii) and (iv). However, there is no information regarding the criteria (i) or (A) and (vi). 59. (4) Malik Anand satisfies all the criteria except at (iii). Therefore, his case would be referred to the Assistant Vice-President of the Company. 60. (5) It is given that for the last three years her age has been more than 18. From this information, we cannot determine his exact age. Therefore, the data are inadequate to take any decision. 61. (1) Alankrita Mangal satisfies all the criteria. Therefore, she can be selected.

    SBI PO EXAMS 1. (4) Considering the nature of problem, Courses of Action I is not necessary. Again, it is mentioned in the statement that water supply is disrupted due to loss of water owing to leakage in pipes supplying water. Thus, reason for dis-

    ruption of water supply is known. Then, what is the need of settingup of fact-finding mission. 2. (5) Both the Courses of Action are suitable for pursuing. In case of outbreak of malaria adequate supply of drugs in necessary. Again the people should use mosquito repellants and keep their premises clean to check breeding of mosquitoes. 3. (1) Only Course of Action I seems to be suitable for pursuing. Course of Action II is too harsh to be suitable. 4. (4) Neither of the Courses of Action is suitable for pursuing. A course of action is taken to solve or reduce a problem. 5. (4) Neither of the Courses of Action is suitable for pursuing. The Government should take some other measure to discourage dropout in Government schools. 6. (1) Obviously, option (1) is the appropriate course of action. 7. (4) It is mentioned in the statement that decline in the participation of labour force in agriculture is not desirable. Therefore, Course of action II seems to be suitable for pursuing. 8. (5) Clearly (B) supports the legal action taken by Country Y. 9. (3) From the statement it is clear that Polka turtles are being caught for rearing as pets. But these turtles may die for lack of adequate habitat. So, the main problem is the use of Polka turtles as pets. Hence, option (3) seems to be most plausible step. 10. (1) Obviously, option (1) is the appropriate course of action. 11. (4) It is mentioned in the statement that decline in the participation of labour force in agriculture is not desirable. Therefore, Course of action II seems to be suitable for pursuing. 12. (5) Clearly (B) supports the legal action taken by Country Y.

    RBI GRADE–B/ NABARD GRADE–A OFFICER EXAMS 1. (1) Obviously, both the courses of action are suitable for pursuing.

    BPRE–887

    2. (2) Only course of action I seems to be appropriate for pursuing. Course of action II is too harsh regarding the problem. Furthermore, it is not a practical approach. 3. (5) (1) The term exam of Geography of Class 9 was to be held on 23.03.2017. Priya Dandekar submitted her assignment 33 days prior to the term exam. So, she would be punished. (B) Gautam Saxena, a Maths teacher of the school, finished all his extra classes by first week of March. But he has to finish all his extra classes by 20.02.2017. Therefore, Maths teacher would also be pubished. (C) Ranvir Chauhan paid his computer fees and also submitted its receipt to his class teacher on time. 4. (3) Obviously, both the courses of action are suitable for pursuing. Both the courses of action would improve cleanliness in the Country P. 5. (1) Neither course of action I nor II follows. Course of action I is not according to business ethics and norms. Course of action II is based on the premise that the company M would face the given situation every quarter. It may not be true.

    INSURANCE EXAMS 1. (3) Clearly courses of action (A) and (B) seem suitable for pursuing. 2. (5) Obviously, option (5) may not be a reason for the decision of the government to discontinue the funding of the College XYZ. 3. (3) The vaccine stagerine is highly effective for the targeted disease. But it has serious side effects. So, the Health Ministry of Country X might have ordered to discontinue its use. Thus, option (3) strengthens the order given by the Health Ministry of Country X. 4. (4) Only course of action I seems to be suitable for pursuing. Course of action II is not possible practically. 5. (4) Neither course of action I nor II seems to be suitable for pursuing. 6. (2) Obviously, only course of action II seems to be suitable for pursuing.

    ❐❐❐

    COURSES OF ACTION

    MODEL EXERCISES Directions (1-4) : In each of these questions, there is a statement followed by two courses of action I and II. Assume everything in the statement to be true and then decide which of the two suggested courses of action logically follows. Markyour answer as (1) If only course of action I follows (2) If only course of action II follows (3) If neither I nor II follows (4) If both I and II follow (5) None of these 1. Statement : Exporters in the capital are alleging that commercial banks are violating a Reserve Bank of India directive to oper ate a post-shipment export credit dominated in foreign currency at international interest rates from January this year. Courses of Action : I. The officers concerned in the commercial banks are to he suspended. II. The RBI should be asked to stop giving such directives to commercial banks. 2. Statement : Court take too long in deciding important disputes of various departments. Courses of Action : I. Courts should be ordered to speed up matters. II. Special powers should be granted to officers to settle the disputes concerning their department. 3. Statement : The chairman stressed the need for making education system more flexible and regreted that the curriculum has not been revised in keeping with the pace of the changes taking place. Courses of Action : I. Curriculum should be reviewed and revised periodically. II. System of education should be made more flexible. 4. If an ant were as big as a horse, could it move mountains? No, A giant ant. (1) Would be a structural failure (2) Would never be able to move mountains

    (3) Would be a mere curiosity (4) Would never be able to hunt for food (5) None of these Directions (5-9) : Each of these questions has a statement followed by three suggested Courses of Action numbered I, II and III. Assume everything in the statement to be true, and decide which of the given Courses of Action logically follows for pursuing. 5. Statement : The Department of Education has recommended that the primary level admission to Government and Governmentaided schools should be done purely by random selection and not by admission tests. This is necessitated as the number of admission seekers are much more than the available seats. Courses of Action : I. The Government should instruct the private schools also to follow the same practice. II. The Government should set up an independent body to regulate the primary level admissions. III. The schools should be asked to select student only from those who stay in the neighbouring areas of the school. (1) None follows (2) I and II follow (3) Only I follows (4) Only III follows (5) None of these 6. Statement : The vehicular traffic has increased so much in the recent past that it takes at least two hours to travel between the city and the airport during peak hours. Courses of Action : I. Non-airport bound vehicles should not be allowed to ply on the road connecting the city and the airport. II. The load of vehicular traffic should be diverted through various link roads during peak hours. III. The departure and arrival of flights should be regulated so

    BPRE–888

    as to avoid congestion during peak hours. (1) Only I follows (2) I and II follow (3) Only II follows (4) All follow (5) None of these 7. Statement : Due to cancellation of a huge export order for not adhering to the time frame, the company is likely to get into incurring losses in the current financial year. Courses of Action : I. The officer-in-charge of the production should be immediately suspended. II. The goods manufactured for the export order should be sold to other party. III. The company should change its machinery to maintain the time frame. (1) None follows (2) I and II follow (3) Only II follows (4) All follow (5) None of these 8. Statement : A devastating earthquake has ravaged the city killing hundreds of people and rendering many more homeless. Courses of Action : I. The entry of outsiders into the city should be stopped II. The civic administration should immediately make alternate temporary housing arrangement for the victims. III. The affected people should immediately be shifted to a safer place. (1) Only I follows (2) Only III follows (3) II and III follow (4) Either II or III follows (5) None of these 9. Assertion (A) : India has a tropical monsoon type climate. Reason (R) : India is located exactly between the tropical latitudes. 10. Statement : Drinking water supply to New Bombay has been suspended till further orders from Maharashtra Pollution Control Board following pollution of

    COURSES OF ACTION Patalganga river, caused by discharge of effluents from some chemical industries. Courses of Action : I. The industries responsible for discharging effluents into the river should be asked to close down immediately. II. The river water should immediately be treated chemically before resuming supply. III. The Pollution Control Board should check the nature of effluents being discharged into the river by industries at regular intervals. (1) All follow (2) II and III follow (3) Only I follows (4) Only III follows (5) None of these Directions (11-14) : In each question below is given, a statement followed by two courses of action numbered I and II. You have to assume everything in the statement to be true, then decide which of the two given suggested courses of action logically follows for pursuing. Mark answer as (1) if only II follows (2) if only I follows (3) if neither I nor II follows (4) if both I and II follows (5) None of these 11. Statement : Most of the development plans develop in papers only. Course of action : I. The officials should be instructed to supervise the field. II. The supply of paper to such departments should be cut short. 12. Statement : A large number of engineering graduates in the country are not in a position to have gainful employment at present and the number of such engineers is likely to grow in the future. Course of action : I. The government should launch attractive employment generation schemes and encourage these graduates to opt for such schemes to use their expertise and knowledge effectively. II. This happened due to proliferation of engineering colleges in the country and thereby

    lowered the quality of the engineering graduates. Those colleges which are not equipped to impart quality education should be closed down. 13. Statement : The police department has come under a cloud with recent revelations that at least two senior police officials are suspected to have been involved in the illegal sale of a large quantity of weapons from the state police armory. Courses of action : I. A thorough investigation should be ordered by the State Government to bring out all those who are involved in the illegal sale of arms. II. State police armory should be kept under Central Govern ment’s control. 14. Statement : The Committee has criticized the Institute for its failure to implement a dozen of regular programmes despite an increase in the staff strength and not drawing up a firm action plan for studies and research. Courses of action : I. The broad objectives of the Institute should be redefined to implement a practical action plan. II. The Institute should given a report on reasons for not have implemented the planned programmes. Directions (15-18) : A statement is followed by two courses of action numbered I and II. Assume everything in the statement to be true and decide which course(s) of action logically follow(s) for pursuing. Mark answer as (1) If either course of action I or II follows (2) If only course of action II follows (3) If only course of action I follows (4) If both I and II follow (5) None of these 15. Statement : The condition of the roads at many places has deteriorated due to incessant rains during the past few days resulting in slow movement of vehicular traffic.

    BPRE–889

    Courses of Action : I. All these parts of the road should immediately be repaired by closing these roads from movement of vehicular traffic. II. Motorists should be alerted at various places by putting up sign boards about the bad patches of the roads to enable them to plan their journey accordingly. 16. Statement : Many medical and engineering graduates are taking up jobs in administrative services and in banks. Courses of Action : I. All the professionals should be advised to refrain from taking up such jobs. II. The government should appoint a committee to find out the reasons for these professionals taking up such jobs and to suggest remedial measures. 17. Statement : In the reimbursement of medical expenses as per mutual agreement due on an appointed date, the managers are agitated. They have decided to go on a half-day strike. Courses of Action : I. The managers should not have fixed any date for the repayment of medical expenses. II. If the employees go on strike, then they should be warned about salary cut. 18. Statement : The Transport Ministry said in a report that people should obey the rules of traffic, It will improve transport services and hence the number of road accidents will decrease. Courses of Action : I. The ministry should organise a consciousness campaign to rise awareness about rules relating to traffic. II. The licenses of men who disobey the rules of traffic should be cancelled. Directions (19-21) : In each of these questions a statement is followed by two courses of action numbered I and II. Assume everything in the statement to be true. Decide which of the suggested courses of action logically follow(s) for pursuing. Mark answer as

    COURSES OF ACTION (1) if only course of action I follows (2) if neither course of action I nor II follows (3) if only course of action II follows (4) if both courses of action I and II follow (5) None of these 19. Statement : The Librarian finds some cases in which the pages from certain books issued from the library, are torn. Courses of Action : I. The Librarian should keep a record of books issued to each student and if the pages are found torn, strict measures should be taken against the child who has been issued that book. II. Some funds should be collected from the children collectively to renovate the library. 20. Statement : The availability of imported fruits has increased in the indigenous market and so the demand for indigenous fruits has been decreased. Courses of Action : I. To help the indigenous producers of fruits, the Government should impose high import duty on these fruits, even if these are not of good quality. II. The fruit vendors should stop selling imported fruits so that the demand for indigenous fruits would be increased. 21. Statement : Some serious blunders were detected in the Accounts Section of a factory. Courses of Action : I. An efficient team of auditors should be appointed to check the Accounts. II. A show cause notice should be issued to all the employees involved in the irregularity. Directions (22-25) : In each of these questions, a statement is followed by two courses of action numbered I and II. Assume everything in the statement to be true and decide which of the suggested course(s) of action logically follow(s) for pursuing. Mark answer as

    (1) if only I follows (2) if only II follows (3) if either I or II follows (4) if neither I nor II follows (5) None of these 22. Statement : The prices of foodgrains and vegetables have substantially increased due to prolonged strike call given by the truck owners association. Courses of action : I. The government should immediately make alternative arrangement to ensure adequate supply of foodgrains and vegetable in the market. II. The government should take steps to cancel the licences of all vehicles belonging to the association. 23. Statement : There has been an unprecedented increase in the number of requests for berths in most of the long distance trains during the current holiday season. Courses of action : I. The railway authority should immediately increase the capacity in each of these trains by attaching additional coaches. II. The people seeking accommodation should be advised to make their travel plan after the holiday. 24. Statement : There has been significant drop in the water level of all the lakes supplying water to the city. Courses of action : I. The water supply authority should impose a partial cut in supply to tackle the situation. II. The government should appeal to all the residents through mass media for minimal use of water. 25. Statement : A large number of people visiting India from country ‘X’ has been tested positive for carrying viruses of a killer disease. Courses of action : I. The government of India should immediately put a complete ban on people coming to India from country ‘X’ including those Indians who are settled in country X. II. The government of India should immediately set-up detection centres at all its air-

    BPRE–890

    ports and seaports to identify and quarantine those who are tested positive. Directions (26-30) : Against each of these questions, there is a statement/passage and two courses of action, numbered I and II. Assume everything in the statement to be true. Decide which of the two suggested courses of action logically follows for pursuing. Mark the answer as (1) if only course of action I follows (2) if only course of action II follows (3) if either course of action I or II follows (4) if neither course of action I nor II follows (5) None of these 26. Statement : Love marriages generally end in divorce. So, the children are brought up by one parent. Therefore, the practice of love marriages should be abandoned. Courses of action : I. Arranged marriages are better than love marriages. II. It is undesirable to have a child brought up by one parent. 27. Statement : Properly fed and starved monkeys were made to run through a network of paths designed as a puzzle. It was observed that the starved monkeys could not run faster. This proved that the people with lower intelligence in poor countries are a victim of malnutrition. Courses of action : I. The effects of nutrition on the intelligence of the monkeys is parallel to those on human beings. II. Captive monkeys are more intelligent than wild monkeys. 28. Statement : People in Pratapgarh are not paying their telephone bills in spite of the various verbal warnings and reminders. It has now informed the subscribers through a notification that those who do not pay their bills by the due date will be charged penalty for every defaulting day. Courses of action : I. Majority of people may pay their bills by the due date to avoid penalty.

    COURSES OF ACTION II. People generally pay heed to such statutory notifications. 29. Statement : The civic authorities have decided that all the factories located inside the city in the populated areas be shifted outside at the allocated place to reduce the level of environmental pollution in the city to safeguard the health of people living there. Courses of action : I. The pollution level in the city in future shall certainly reduce after these factories are shifted outside the city limit and hence people will enjoy a better health. II. Enough usable land is available outside the city limit for these factories. 30. Statement : Company ‘A’ has intimated all its present suppliers that tender specification will not be issued to the firms where there is 25% or more default in supplies against the earlier pur chase order placed on them. Courses of action : I. The company ‘A’ will be watching the quality of performance of its suppliers. II. The company ‘A’ expects quality and professional approach from its suppliers. Directions (31-32) : In each question below is given a statement followed by two courses of action numbered I and II. A course of action is a step or administrative decision to be taken for improvement, follow-up or further action in regard to the problem, policy, etc. On the basis of the information given in the statement, you have to assume everything in the statement to be true, then decide which of the suggested courses of action logically follow(s) for pursuing. Give answer (1) if only Course of action I follows Give answer (2) if only Course of action II follows Give answer (3) if either Course of action I or II follows Give answer (4) if neither Course of action I nor II follows Give answer (5) if both Courses of action I and II follow 31. Statement : A large number of patients admitted in a particular hospital have been diagonsed with serious contagious disease.

    Courses of action I. These patients should be immediately evicted from the hospital as there is a chance of spread of the disease. II. The hospital should make appropriate quarantine arrangement to prevent the spread of the disease. 32. Statement : Private schools are charging more fees than the public schools. Courses of action I. The Government should identify such schools and take appropriate actions after proper investigation of the matter. II. The Government should impose ban on such schools with immediate effect. 33. In this question is given a statement followed by two courses of action numbered I and II. A course of action is a step of administrative decision to be taken for improvement, follow up or further action in regard to the problem, policy etc. On the basis of the information given in the statement, you have to assume everything in the statement to be true and then decide which of the suggested courses of action logically follows for pursuing. Statement: Annual results of School G are continuously falling from past three years as the school is unable to hire adequate number of teachers due to lack of funds. Course of Action I : School G should increase the fees of students by 30% in order to generate funds for hiring new teachers. Course of Action II: Classes of School G should be divided into two shifts and should be continued with the existing teachers. (1) Either course of action I or course of action II follows (2) Neither course of action I nor course of action II follows (3) Only course of action I follows (4) Only course of action II follows (5) Both courses of action I and II follow

    BPRE–891

    34. This question consists of a statement followed by two Course of Actions numbered I and II given below it. A Course of Action is an administrative decision to be taken for improvement or the executive decision upon the primary decision. You have to consider the ideas in the statement to be true and then decide which of the following Course of Action logically follow(s) the given statement. Statement : In village X, the general public provoked outburst on a matter that suspecting that two men attempted kidnapping the girl children of that area for human trafficking. The angered mob of the general public lynched on the two men that caused severe injury and death. Courses of Action : I. The General public of the other areas have to learn the responsibility of law & order and hand over them to the police with patients instead of lynching II. The police should come to the crime spot of such incidents at earlier as possible. (1) Only I follows (2) Only II follows (3) Either I or II follows (4) Neither I nor II follows (5) Both I and II follow

    SHORT ANSWERS 1. (3) 5. (2) 9. (1) 13. (2) 17. (2) 21. (4) 25. (2) 29. (4) 33. (3)

    2. (4) 6. (3) 10. (1) 14. (4) 18. (4) 22. (1) 26. (4) 30. (1) 34. (5)

    3. (4) 7. (3) 11. (2) 15. (2) 19. (1) 23. (1) 27. (1) 31. (2)

    4. (3) 8. (4) 12. (4) 16. (2) 20. (3) 24. (3) 28. (3) 32. (1)

    EXPLANATIONS 1. (3) None of the options is a correct course of action. 2. (4) Both the actions can help in speedy disposal of the pending cases. Hence, both the actions follow. 3. (4) Both the actions are related with the solution of problem. Hence both are strong.

    COURSES OF ACTION 4. (3) Would be a mere curiosity 5. (2) Since, the basic problem is that number of admission seekers are much more than the available seats, hence, both courses of action I and II will help to reduce the problem. 6. (3) Actions I and III will create another problems. Action II is the effective way to reduce the problem. 7. (3) Suspension of officer -incharge does not immediately help the situation nor does the charge of machinery help in any way to tackle the present situation. Only II can solve the present crises. 8. (4) Clearly, either II or III action solve the problem. 9. (1) Both A and R are true and R is the correct explanation of A. 10. (1) All the courses of actions are feasable and effective means to mitigate the problem. 11. (2) Course of action I follows because officials should supervise the field. Work in paper means that actual work is not done on site. So, course of action II does not follow. 12. (4) Both courses of action I and II follow because government should launch attractive employment generation schemes for these engineers. Colleges which are not well equipped and not providing proper placement for its students should be closed down. 13. (2) Course of action I follows because weapons were sold from state police armory, so state government, should investigate and bring out all those who are involved in the illegal sale of arms. Course of action II does not follow because state police ar mory is the property of state government. So, it should remain in state government’s control. 14. (4) Both courses of action I and II follow because institute should implement a practical action plan and should give a report on reasons for not implementing the planned program. 15. (2) Closure of roads from vehicular movement makes it a weak action, hence I does not follow.

    However, display of notice will enable motorists to take extra precaution while travelling. 16. (2) Reasons are required to be known for change from core profession to other profession. 17. (2) Warning about the cut in salary will warn them of their action. 18. (4) Both the courses of action follow. 19. (1) Course of action I is an effective course to check the problem. However, collection of funds does not ensure to check the problem. Hence, II does not follow. 20. (3) Imposing high import duty is not a justified course of action. However, if vendors stop selling imported fruits, this will definitely enhance the sale of indigenous fruits. 21. (4) Appointment of auditors will ensure the identification of irregularity and action against erring employees will discourage such blunders in future. Hence, both the courses of action are correct. 22. (1) Course of action I follows because the supply of food grains and vegetable in the market may be affected due to strike. Course of action II does not follow because cancelling the licences of all vehicles belonging to the association is into the solution of the problem. 23. (1) Course of action I follows as railway authority should immediately attach additional coaches. Course of action II does not follows because people plan their travel during holiday. 24. (3) To tackle the situation to authority should impose a partial cut or it can appeal to all the residents for minimal use of water. So, either course of action I or II follows. 25. (2) Course of action I does not follow because a complete ban on people coming to India from country ‘X’ is not the solution of problem. Course of action II follows because detection centres should be set up at airports and seaports to detect the viruses of killer disease.

    BPRE–892

    26. (4) Course of action I does not follow as the statement does not say about arranged marriages. Course of action II also does not follow as the statement does not give any relation between parenting and divorce. Therefore, neither courses of action I nor II follows. 27. (1) The proper nourishment will make the monkeys or human beings work properly. So, the course of action I follows. Course of action II does not follow as there is no information about captive or wild monkeys. 28. (3) Enforcing a penalty will motivate people to pay the bills. So, course of action I follows. It is general tendency to act accordingly when a penalty clause slapped. So, course of action II follows. 29. (4) From the statement, it is not clear that the environment of city is polluted due to these factories or there is some other reason of pollution. So, course of action I does not follow. It is also not clear from the statement that there is some land available or not out side the city limit. So, course of action II also does not follow. 30. (1) Course of action I follows as the company ‘A’ is setting a quality standard. Course of action II does not follow as company A is defining the standard and is selective to provide the contract and not expressing its expectation. 31. (2) Obviously, the Course of action II is suitable for pursuing. Hospital is meant for the treatment of diseases. So, patients should not be evicted from the hospital. 32. (1) Obviously, the Course of action I is suitable for pursuing. Course of action II seems to be a harsh step. 33. (3) From the statement it is clear that the main problem is the lack of funds. Therefore, school management should take appropriate measures to raise funds. Thus, Course of action I seems to be appropriate. 34. (5) Obviously both the courses of ❑❑❑ Action follow.

    DATA SUFFICIENCY

    17

    DATA SUFFICIENCY

    NATIONALISED BANKS & IBPS PO/MT/SO EXAMS Directions (1–5) : Each of the questions below consists of a question and two statements numbered I and II given below it. You have to decide whether the data provided in the statements are sufficient to answer the question. Read both the statements and (Indian Bank PO Exam. 02.01.2011 (Ist sitting)

    Give answer (1) If the data in statement I alone are sufficient to answer the question, while the data in statement II alone are not sufficient to answer the question. Give answer (2) If the data in statement II alone are sufficient to answer the question, while the data in statement I alone are not sufficient to answer the question. Give answer (3) If the data either in statement I alone or in statement II alone are sufficient to answer the question. Give answer (4) If the data even in both statements I and II together are not sufficient to answer the question. Give answer (5) If the data in both statements I and II together are necessary to answer the question. 1. Who is oldest among Peter, Kevin, Joseph and Jason ? I. Jason is older than Peter and Joseph. II. Kevin is younger than Joseph. 2. How is ‘happy’ written in a code language ? I. ‘I happy today’ is written as ‘ke ne que’ and ‘today happy day’ is written as ‘ke joi ne’. II. ‘I play’ is written as ‘que pa’. 3. H is the mother of J. How is J related to V ? I. V is the only daughter of H. II. V is the sister of J. 4. What is the colour of white snow in a colour code ?

    I. Green is called Black, Black is called Blue, and Blue is called Red. II. Red is called White and White is called Orange. 5. Six people P, Q, R, S, T and U are seated around a circular table and are equidistant from each other. Who is second to the right of T ? I. P is to the immediate left of Q and Q sits opposite R. II. S is to the immediate left of U. Directions (6–10) : Each of the questions below consists of a question and two statements numbered I and II given below it. You have to decide whether the data provided in the statements are sufficient to answer the question. Read both the statements and — (Corporation Bank PO Exam. 16.01.2011)

    Give answer (1) if the data in statement I alone are sufficient to answer the question, while the data in statement II alone are not sufficient to answer the question. Give answer (2) if the data in statement II alone are sufficient to answer the question, while the data in statement I alone are not sufficient to answer the question. Give answer (3) if the data either in statement I alone or in statement II alone are sufficient to answer the question. Give answer (4) if the data even in both statements I and II together are not sufficient to answer the question. Give answer (5) if the data in both statements I and II together are necessary to answer the question. 6. Who amongst L, M, N, O and P is the shortest. I. O is shorter than P but taller than N. II. M is not as tall as L.

    BPRE–893

    Are all the five friends viz. Leena, Amit, Arun, Ali and Ken who are seated around a circular table facing the centre ? I. Leena sits second to left of Amit. Amit faces the centre. Arun sits second to right of Leena. II. Ali sits third to the left of Ken. Ken faces the centre. Amit sits to the immediate left of Ali but Ken is not an immediate neighbour of Amit. 8. Is T grandmother of Q ? I. P is the mother of Q. Q is the son of R. R is the son of T. II. L is father of N and N is daughter of T. 9. Point A is towards which direction from point B ? I. If a person walks 4m towards the north from point A, and takes two consecutive right turns, each after walking 4 m, he would reach point C, which is 8m away from point B. II. Point D is 2m towards the east of point A and 4m towards the west of point B. 10. How many brothers does Bharat have ? I. Shiela, the mother of Bharat has only three children. II. Meena, the grandmother of Bharat has only one granddaughter. Directions (11-15) : Each of the questions below consists of a question and two statements numbered I and II given below it. You have to decide whether the data provided in the statements are sufficient to answer the question. Read both the statements and — 7.

    (Punjab & Sind Bank PO Exam. 23.01.2011)

    Give answer (1) if the data in statement I alone are sufficient to answer the question, while the data in statement II alone are not sufficient to answer the question.

    DATA SUFFICIENCY Give answer (2) if the data in statement II alone are sufficient to answer the question, while the data in statement I alone are not sufficient to answer the question. Give answer (3) if the data either in statement I alone or in statement II alone are sufficient to answer the question. Give answer (4) if the data given in both the statements I and II together are not sufficient to answer the question, and Give answer (5) if the data in both the statements I and II together are necessary to answer the question. 11. How is ‘never’ written in a code language? I. ‘never ever go there’ is written as ‘na ja ni ho’ in that code language. II. ‘go there and come back’ is written as ‘ma ho sa ni da’ in that code language. 12. Among M, P, K, J, T and W, who is lighter than only the heaviest? I. P is heavier than M and T. II. W is heavier than P but lighter than J who is not the heaviest. 13. What does ‘$’ mean in a code language? I. ‘5 $ # 3’ means ‘flowers are really good’. II. ‘7 # 3 5’ means ‘good flowers are available’. 14. How is P related to J ? I. M is brother of P and T is sister of P. II. P’s mother is married to J’s husband who has one son and two daughters. 15. How many students are there between Suresh and Mohan in a row of fifty students ? I. Suresh is twelfth from the left end and Mohan is seventeenth from the right end. II. Suresh is six places away from Jayesh who is twentieth from the left end. Directions (16-20) : Each of the questions below consists of a question and two statements numbered I and II given below it. You have to decide whether the data provided in the statements are sufficient to answer the question. Read both the statements and — (UCO Bank PO Exam. 30.01.2011)

    Give answer (1) if the data in statement I alone are sufficient to answer the question, while the data in statement II alone are not sufficient to answer the question. Give answer (2) if the data in statement II alone are sufficient to answer the question, while the data in statement I alone are not sufficient to answer the question. Give answer (3) if the data either in statement I alone or in statement II alone are sufficient to answer the question. Give answer (4) if the data even in both statements I and II together are not sufficient to answer the question. Give answer (5) if the data in both statements I and II together are necessary to answer the question. 16. Point A is towards which direction from point B ? I. If a person walks 5 m towards West from point A, takes a left turn and walk 5 m again, he would be 4 m away from point B. II. Point A is towards the North of point C, point C is towards the East of point D and point B is towards the East of point D. 17. Is S the mother of M ? I. M is sister of Q, Q is sister of R and R is daughter of S. II. M is daughter of Land L is sister of V. 18. Are all the five friends viz. A, B, C, D and E who are seated around a circular table facing the centre? I. A sits third to the right of D. D faces the centre. B sits second to the right of A. II. C sits second to the left of E. E faces the centre. D sits second to the right of C. 19. How is ‘came’ written in the code language? I. ‘we came by car’ is written as ‘4 9 2 8’ and ‘can we buy car’ is written as ’5 8 0 2'. II. ‘can car be cheap’ is written as ‘8 1 5 3’ and ‘came by cheap car’ is written as ‘9 8 4 1’. 20. Who amongst Shreya, Jyoti, Leena, Pia and Kajal is the shortest?

    BPRE–894

    I. Leena is taller than Pia and Pia is taller only than Shreya. II. Kajal is taller than Jyoti and Leena. Leena is taller than Pia. Shreya is not the tallest. Directions (21–25) : Each of the questions below consists of a question and two statements numbered I and II given below it. You have to decide whether the data provided in the statements are sufficient to answer the question. Read both the statements and— (Bank Of Baroda PO Exam. 13.03.2011)

    Give answer (1) if the data in Statement I alone are sufficient to answer the question, while the data in Statement II alone are not sufficient to answer the question Give answer (2) if the data in Statement II alone are sufficient to answer the question, while the data in Statement I alone are not sufficient to answer the question. Give answer (3) if the data in Statement I alone or in Statement II alone are sufficient to answer the question. Give answer (4) if the data in both the Statements I and II are not sufficient to answer the question. Give answer (5) if the data in both the Statements I and II together are necessary to answer the question. 21. In a six storey building (consisting of floors numbered 1, 2, 3,4, 5 and 6. The ground floor is numbered 1, the floor above it is numbered 2 and so on) the third floor is unoccupied. The building houses different people viz. P, Q, R, S and T, each living on a different floor. On which of the floors does T live? I. S lives between the floors on which R and T live. II. There are two floors between T’s floor and Q’s floor. 22. How is ‘see’ written in the code language? I. ‘hope to see you’ is written as ‘3692’, ‘do you see that’ is written as ‘1973’. II. ‘to pray and hope’ is written as ‘0286’ and ‘hope I do well’ is written as ‘5467’. 23. Among five friends A, B, C, D and E sitting around a circular table and facing the centre, who is sitting to the immediate left of A?

    DATA SUFFICIENCY I. A sits third to the right of B. D is not an immediate neighbour of B. II. B is an immediate neighbour of C. 24. Is X the wife of Y? I. X’s daughter M is the only sister of R. R is the son of Y. II. The mother of Y has only one grandson R. 25. Among P, Q, R, S and T, which bag is the lightest? I. P is heavier than Q. R is as heavy as Q. T is lighter than R. II. S is lighter than Q but heavier than T. Directions (26–30) : Each of the questions below consists of a question and two statements numbered I and II given below it. You have to decide whether the data provided in the statements are sufficient to answer the question. Read both the statements and — (Allahabad Bank PO Exam. 17.04.2011)

    Give answer (1) if the data in statement I alone are sufficient to answer the question, while the data in statement II alone are not sufficient to answer the question. Give answer (2) if the data in statement II alone are sufficient to answer the question, while the data in statement I alone are not sufficient to answer the question. Give answer (3) if the data in statement I alone or in statement II alone are sufficient to answer the question. Give answer (4) if the data in both the statements I and II are not sufficient to answer the question. Give answer (5) if the data in both the statements I and II together are necessary to answer the question. 26. How is ‘letter’ written in the code language? I. ‘please write a letter’ is written as ‘7218’ , ‘received a Greek letter is written as ‘7513’. II. ‘write in English please’ is written as ‘2084’, and ‘a letter in Greek’ is written as ‘5714’. 27. Among A, B, C, D and E, seated in a straight line; facing North, who sits exactly in the middle of theline ?

    I. A sits third to left of D. B sits to the immediate right of C. II. B sits second to right of A. E is not an immediate neighbour of D. 28. A six storey building (consisting ‘of an unoccupied ground floor and five floors on top of the ground floor numbered 1, 2, 3, 4 and 5) houses different people viz. A, B, C, D and E. Who lives on the third floor ? I. C lives on an even numbered floor. A lives immediately above D. B lives immediately above A. E does not live on the topmost floor. II. D lives on an odd numbered floor. A and B are immediate neighbours of each other. Similarly, C and E are immediate neighbours of each other. C does not live on an odd numbered floor. 29. Are all the four friends Abhay, Kavita, Prashant and Yasir who are sitting around a circular table facing the centre? I. Kavita sits second to left of Abhay. Abhay faces the centre. Yasir sits to the immediate right of Abhay as Well as Kavita. II. Prashant sits third to the right of Kavita. Abhay sits to immediate right of Prashant as well as Yasir. 30. Is R the granddaughter of C ? I. The only sister of A is the mother of R’s brother, B .. II. C, the mother of A has only one grandson, B. Directions (31-35) : Each of the questions below consists of a question and two statements numbered I and II given below it. You have to decide whether the data provided in the statements are sufficient to answer the question. Read both the statements and — (Indian Overseas Bank PO Exam. 22.05.2011)

    Give answer (1) if the data in statement I alone are sufficient to answer the question, while the data in statement II alone are not sufficient to answer the question. Give answer (2) if the data in statement II alone are sufficient to answer the question, while the data in statement I alone are not sufficient to answer the question.

    BPRE–895

    Give answer (3) if the data either in statement I alone or in statement II alone are sufficient to answer the question. Give answer (4) if the data even in both statements I and II together are not sufficient to answer the question. Give answer (5) if the data in both statements I and II together are necessary to answer the question. 31. Who amongst A, B, C, D and E is the tallest? I. A is taller than B but shorter than C. D is not the tallest. II. Two people are taller than C. 32. Which direction is Ali facing ? I. If Ken who is currently facing East turns 90° towards his right, he would face a direction exactly opposite to the direction which Ali is facing. II. It Priya who is currently facing South turns left, walks 1 m and then takes a left turn again; she would face the same direction as Ali. 33. Did 300 candidates appear for the written examination for admission into college X ? I. The principal of the college correctly mentions that the number of candidates who had appeared for the examination was more than 200. II. According to a statistical report only 175 candidates could qualify the examination 34. How far is point P from point Q ? (All the points lie on a straight line) . I. Point T is exactly midway between points P and Q. Point T is 5 km towards west of point R. II. Point Q is 2 kms towards the east of Point R. 35. How many brothers does A have? I. A who is B’s brother has two siblings. II. D is brother of A and is youngest in the family. Directions (36-40) : Each of the questions below consists of a question and three statements numbered I, II and III given below it. You have to decide whether the data provided in the statements are sufficient to answer the question. (IBPS Bank PO/MT CWE Exam. 18.09.2011)

    DATA SUFFICIENCY 36. How many daughters does P have? I. B and D are sisters of M. II. M’s father T is husband of P. III. Out of the three children which T has, only one is a boy. (1) Only I and III (2) All I, II and III are required to answer the question. (3) Only II and III (4) Question cannot be answered even with all I, II and III (5) Only I and II 37. Who among A, B, C, D, E and F each having a different height, is the tallest? I. B is taller than A but shorter than E. II. Only two of them are shorter than C. III. D is taller than only F (1) Only I and II (2) Only I and III (3) Only II and III (4) All I, II and III are required to answer the question (5) All I, II and III are not sufficient to answer the question 38. Towards which direction is village J from village W ? I. Village R is to the west of Village W and to the north of Village T. II. Village Z is to the east of Village J and to the south of Village T. III. Village M is to the north east of Village J and north of Village Z. (1) Only III (2) Only II and III (3) All I, II and III are required to answer the question (4) Ouestion cannot be answered even with all I, II and III (5) None of these 39. On which day of the week starting from Monday did Manoj visit Chennai ? I. Manoj took leave on Wednesday. II. Manoj visited Chennai the day after his mother’s visit to his house III. Manoj’s mother visited Manoj’s house neither on Monday nor on Thursday

    (1) (2) (3) (4)

    Only II and III Only I and II Only I and III All I, II and III are required to answer the question (5) Ouestion cannot be answered even with all I, II and III 40. How is ‘go’ written in a code language? I. ‘now or never again’ is written as ‘tom ka na sa’ in that code language. II ‘you come again now’ is written as ‘ja ka ta sa’ in that code language III. ‘again go now or never’ is written as ‘na ho ka sa tom’ in that code language (1) Only I and III (2) Only II and III (3) Only I and II (4) All I, II and III are required to answer the question (5) None of these Directions (41–45) : Each of the questions below consists of a question and two statements numbered I and II given below it. You have to decide whether the data provided in the statements are sufficient to answer the question. Read both the statements and— (IBPS Specialist Officer CWE Exam. 11.03.2012)

    Give answer (1) if the data in statement I alone are sufficient to answer the question, while the data in statement II alone are not sufficient to answer the question. Give answer (2) if the data in statement II alone are sufficient to answer the question, while the data in statement I alone are not sufficient to answer the question. Give answer (3) if the data either in statement I alone or in statement II alone are sufficient to answer the question. Give answer (4) if the data given in both the statements I and II together are not sufficient to answer the question, and Give answer (5) if the data in both the statements I and II together are necessary to answer the question. 41. Among M, P, T, R and W each being of a different age, who is the youngest?

    BPRE–896

    I. T is younger than only P and W. II. M is younger than T and older than R. 42. How is ‘gone’ written in a code language? I. ‘you will be gone’ is written as ‘ka pa ni sa’ in that code language. II. ‘he will be there’ is written as ‘ja da ka ni’ in that code language. 43. On which day of the week (starting from Monday and ending on Sunday of the same week) did Sushant visit Chennai ? I. Sushant visited Chennai two days after his brother visited Chennai II. Sushant did not visit Chennai either on Wednesday or on Friday. 44. Towards which direction is P with respect to the starting point? I. P walked 20 metres, took a right turn and walked 30 metres, again took right turn and walked 20 metres towards West. II. P walked 30 metres, took a left turn and walked 20 metres; again took left turn and walked 30 metres towards East. 45. How is K related to Z ? I. Z and P are the only sisters of D. II. D’s mother is wife of K’s father. Directions (46–51) : Each of the questions below consists of a question and three statements numbered I, II and III given below it. You have to decide whether the data provided in the statements are sufficient to answer the question. Read all the three statements and (IBPS Bank PO/MT CWE Exam. 17.06.2012)

    Give answer (1) if the data in Statements I and II are sufficient to answer the question, while the data in Statement III are not required to answer the question Give answer (2) if the data in Statements I and III are sufficient to answer the question, while the data in Statement II are not required to answer the question

    DATA SUFFICIENCY Give answer (3) if the data in Statements II and III are sufficient to answer the question, while the data in Statement I are not required to answer the question; Give answer (4) if the data in either Statement I alone or Statement II alone or Statement III alone are sufficient to answer the question. Give answer (5) if the data in all the Statements I, II and III together are necessary to answer the question. 46. Among six people P, Q, R, S, T and V each lives on a different floor of a six storey building having six floors numbered one to six (the ground floor is numbered 1, the floor above it, number 2 and so on and the topmost floor is numbered 6). Who lives on the topmost floor? (I) There is only one floor between the floors on which R and Q live. P lives on an even numbered floor. (II) T does not live on an even numbered floor. Q lives on an even numbered floor. Q does not live on the topmost floor. (III) S lives on an odd numbered floor. There are two floors between the floors on which S and P live. T lives on a floor immediately above R’s floor. 47. There are six letters W, A, R, S, N and E. Is ‘ANSWER’ the word formed after performing the following operations using these six letters only ? (I) E is placed fourth to the right of A. S is not placed immediately next to either A or E. (II) R is placed immediately next (either left or right) to E. W is placed immediately next (either left or right) to S. (III) Both N and W are placed immediately next to S. The word does not begin with R. A is not placed immediately next to W. 48. Point D is in which direction with respect to Point B ? (I) Point A is to the west of Point B. Point C is to the north of Point B. Point D is to the south of Point C.

    (II) Point G is to the south of Point D. Point G is 4m from Point B. Point D is 9m from Point B. (III) Point A is to the west of Point B. Point B is exactly midway between Points A and E. Point F is to the south of Point E. Point D is to the west of Point F. 49. How is ‘one’ coded in the code language ? (I) ‘one of its kind’ is coded as ‘zo pi ko fe’ and ‘in kind and cash’ is coded as ‘ga to ru ko’ (II) ‘its point for origin’ is coded as ‘ba le fe mi’ and ‘make a point clear’ is coded as ‘yu si mi de’ (III) ‘make money and cash’ is coded as ‘to mi ru hy’ and ‘money of various kind’ is coded as ‘qu ko zo hy’ 50. Are all the four friends viz. A, B, C and D who are sitting around a circular table, facing the centre? (I) B sits second to right of D. D faces the centre. C sits to immediate right of both B and D. (II) A sits to immediate left of B. C is not an immediate neighbour of A. C sits to immediate right of D. (III) D is an immediate neighbour of both A and C. B sits to the immediate left of A. C sits to the immediate right of B. 51. Read the following information carefully and answer the question which follows : Farmers found using chemical fertilizers in the organic-farming area of their farms would be heavily fined. Which of the following statements is an assumption implicit in the given statement? (An assumption is something supposed or taken for granted.) (1) Chemical fertilisers harm the crop. (2) A farm’s area for organic and chemical farming is different. (3) Farmers who do not use chemical fertilizers in the chemical farming area would be penalized as well.

    BPRE–897

    (4) All farmers undertake both these kinds of farming (chemical as well as organic) in their farms. (5) Organic fertilizers are banned in the area for chemical farming. Directions (52-56) : Each of the following questions below consists of a question and two statments numbered I and II given below it. You have to decide whether the date provided in the statements are sufficient to answer the question. Read both the statements and (IDBI Bank Officer Exam.16.09.2012)

    Give answer (1) if the data in Statement I alone are sufficient to answer the question, while the data in statement II alone are not sufficient to answer the question. Give answer (2) if the data in Statement II alone are sufficient to answer the question, while the data in statement I alone are not sufficient to answer the question. Give answer (3) if the data in either Statement I alone or statement II alone are not sufficient to answer the question. Give answer (4) if the data in both the Statements I and II are not sufficient to answer the question. Give answer (5) if the data in both the Statements I and II are together necessary to answer the question. 52. Consider three friends A, B and C and their professions doctor, architect and engineer (not necessarily in the same order). Who is a doctor amongst these three ? I. B is an architect. A is not an engineer. II. Either A or B is a doctor. Etither A or C is a doctor. 53. How many females are there in the family of four people (P, Q, R and S–consisting of a married couple and two children)? I. P is the father of S. S is the sister of Q. II. R is the mother of Q. 54. What is the code for “writing paper”? I. “Read a paper” is written as “817”. “Thinking and writing” is coded as “624”. II. “Writing with pen” is written as “453”, “Paper and pen” is written as “723”.

    DATA SUFFICIENCY 55. A, B, C, D and E are sitting in a straight line facing North. Who sits third from right end of the line? I. A sits second to left of B. B sits second to left of C. II. B is an immediate neighbour of both E and D. Neither E nor D sit at any of the extreme ends of the line. 56. On which day of the same week is Neha’s birthday ? (Monday being the first day of the week)? I. Ramesh correctly remembers that Neha’s birthday is on a day after Tuesday, but before Friday of the same week. II. Kiran correctly remembers that Neha’s birthday is on a day after Monday, but before Thursday of the same week. Directions (57-58) % Each of the questions below consists of a question and three statements numbered I, II and III given below it. You have to decide whether the data provided in the statements are sufficient to answer the question : (IBPS Bank PO/MT CWE-III, 26.10.2013)

    57.

    58.

    Who is the daughter in law of B? I. I is the brother of D. S is the wife of J’s nephew. II. R is the brother of N. T is the son of N. S is the mother of T. III. B is the wife of D. D is the father of N. D has two children. (1) Only I and III (2) All I, II and III are required to answer the question (3) Only II and III (4) Questions cannot be answered even with all I, II and III (5) Only I and II How many students are there in the class ? I. There are more than 22 but less than 36 students in the class. II. If students of the class are divided into groups each group has exactly 11 students. III. There are more than 29 but less than 45 students in the class.

    (1) Only I and II (2) Only II and either I or III are required to answer the question (3) Only II and III (4) All I, II and III are required to answer the question (5) All I, II and III are not sufficient to answer the question. Directions (59–61) : Each of the questions below consists of a question and two statements numbered I and II given below it. You have to decide whether the data provided in the statements are sufficient to answer the question. Read both the statements and — (BOB Manipal School of Banking Officer Online Exam, 14.08.2014)

    Give answer (1) if the data in Statement I alone are sufficient to answer the question, while the data in Statement II alone are not sufficient to answer the question. Give answer (2) if the data in Statement II alone are sufficient to answer the question, while the data in Statement I alone are not sufficient to answer the question. Give answer (3) if the data either in Statement I alone or in Statement II alone are sufficient to answer the question. Give answer (4) if the data even in both Statements I and II together are not sufficient to answer the question. Give answer (5) if the data in both Statements I and II together are necessary to answer the question. 59. Among five persons – M, N, O, P and Q – sitting around a circular table facing towards the centre, who is second to the left of M ? I. There are two persons between P and O. Only one person is sitting between O and Q. II. P is an immediate neighbour of N. Only one person is sitting between N and Q. Only one person is sitting between P and O. 60. There are six persons namely, U, V, W, X, Y and Z. How is X related to Z ? I. U is mother of V. V is husband of W. Y is mother of Z. V is grandson of X.

    BPRE–898

    II. W is mother of X. V is father– in–law of Z. Z is daughter–in– law of W. X is brother of U. U is wife of Y. 61. Seven events – A, B, C, D, E, F and G – were organised on different days of the same week starting from Monday and ending on Sunday but not necessarily in the same order. Which event was organised on Sunday ? I. There were two events between A and B. Event A was organised after event B. Event G was organised immediately before event D. Event E was organised on Wednesday. II. Event F was organised immediately before Event C. There was only one Event between F and A. Event G was organised on Monday. Directions (62–66) : Each of the questions below consists of a question and two statements numbered I and II are given below it. You have to decide whether the data provided in the statements are sufficient to answer the question. Read both the statements and — (IDBI Bank Officer Exam, 22.08.2014)

    Give answer (1) if the data in Statement I alone are sufficient to answer the question, while the data in Statement II alone are not sufficient to answer the question. Give answer (2) if the data in Statement II alone are sufficient to answer the question, while the data in Statement I alone are not sufficient to answer the question. Give answer (3) if the data in Statement I alone or in Statement II alone are sufficient to answer the question. Give answer (4) if the data in both the Statements I and II together are not sufficient to answer the question. Give answer (5) if the data in both the Statements I and II together are necessary to answer the question. 62. Five persons– M, N, O, P and Q – are sitting around a circular table but not necessarily in the same order. Some of them are facing towards the centre while others are facing opposite to the centre.

    DATA SUFFICIENCY

    63.

    64.

    65.

    66.

    Who among them is sitting second to the left of P ? I. P is sitting to the immediate left of N. N faces outside. M is an immediate neighbour of both O and P. Three persons are facing towards the centre. Q is sitting to the immediate right of O. II. P faces towards the centre. O is sitting second to the right of P. O is an immediate neighbour of both Q nad M. Q faces towards the centre. Both the immediate neighbours of P face outside the centre. Is Y mother of W ? I. K has two children J and L. Y is daughter-in-law of K. L is aunt of W. W is brother of S. S is daughter of J. II. S is sister of W. W is son of J. J is son of K. K is married to N. Y is sister-in-law of L. L is daughter of K. How is ‘view’ coded in a certain code language ? I. In that code language ‘‘the nice mount view’’ is coded as ‘‘3246’’ and ‘‘the view was heavenly’’ is coded as ‘‘2568’’. II. In that code language ‘‘he saw the view’’ is coded as ‘‘6721’’ and ‘‘the man admired view’’ is coded as ‘‘0962’’. Thirteen persons are standing in a straight line facing north. What is the position of J with respect to K ? I. O is standing at centre of the line. There are two persons between O and J. There is only one person between J and T. K is an immediate neighbour of T. K is standing at any of the extreme ends of the line. II. K is standing at any of the extreme ends of the line. There are three persons between S and K. J is an immediate neighbour of S. T is standing second to the right of J. How is N related to L ? I. K is the mother of M. M is the wife of O. N is the brother of K. L is the husband ofK.

    II. M is daughter of K. K is the sister of N. O is son-in-law of K. K is wife of L. N is the son of R. Directions (67–71) : Each of the questions below consists of a question and two statements numbered I and II given below it. You have to decide whether the data provided in the statements are sufficient to answer the question. Read both the statements and — (SIDBI Officer Exam, 03.09.2014)

    Give answer (1) if the data in Statement I alone are sufficient to answer the question, while the data in Statement II alone are not sufficient to answer the question. Give answer (2) if the data in Statement II alone are sufficient to answer the question, while the data in Statement I alone are not sufficient to answer the question. Give answer (3) if the data in Statement I alone or in Statement II alone are sufficient to answer the question. Give answer (4) if the data in both the Statements I and II together are not sufficient to answer the question. Give answer (5) if the data in both the Statements I and II together are necessary to answer the question. 67. What is the code for ‘reason’ in a certain code language ? I. In that code language ‘reason to learn better’ is written as ‘xn zt aj ly’ and ‘to learn study better’ is written as ‘zt xn ly rj’. II. In that code language ‘reason to study important’ is written as ‘yk xn aj rj’ and ‘to find reason necessary’ is written as ‘st xn ds aj’. 68. Among the 15 persons standing in a straight line facing north what is the position of T from the right end of the line ? I. L is at the middle. U is to the immediate right of L. There are two persons between U and T. S is to the immediate right of T. II. O is at the extreme left end. There is only one person between M and O. L is standing exactly between M and S. T is an immediate neighbour of both S and R.

    BPRE–899

    69. Among six persons – A, B, C, D, E and F – sitting around a circular table facing the centre, what is the position of D with respect to B ? I. C is sitting third to the right of F. C is an immediate neighbour of both A and B. E is to the immediate right of B. II. F is sitting second to the left of A. There are two persons between C and F. B is an immediate neighbour of both C and E. 70. How is M related to J ? I. A is mother of M. M is married to T. L is daughter of T. L is cousin of J. Y is mother of J. R is mother of T. R has only two daughters and no son. II. R has two daughters T and Y. T is daughter-in-law of A. Y is married to D. J is son of D. A is son of M. 71. Seven different plays – T, U, V, W, X, Y and Z – were staged on seven different days of the same week from Monday to Sunday, but not necessarily in the same order. Which play was staged on Wednesday ? I. Only two plays were scheduled after Play Y. Play Z was staged immediately before the Play V. Play Z was staged immediately after the Play U. Play W was staged on Thursday. II. Four plays were scheduled after the Play Y. Play V was staged immediately before the Play W. Play V was staged immediately after Play Z. Play U was not scheduled after Play Y. Directions (72-76) : Each of the questions below consists of a question and two statements numbered I and II given below it. You have to decide whether the data provided in the statements are sufficient to answer the question. Read both the statements and — (IBPS RRBs Officer Scale-I CWE, 06.09.2014)

    Give answer (1) if the data in Statement I alone are sufficient to answer the question, while the data in Statement II alone are not sufficient to answer the question. Give answer (2) if the data in Statement II alone are sufficient to answer the question, while the data

    DATA SUFFICIENCY in Statement I alone are not sufficient to answer the question. Give answer (3) if the data either in Statement I alone or in Statement II alone are sufficient to answer the question. Give answer (4) if the data even in both Statements I and II together are not sufficient to answer the question. Give answer (5) if the data in both Statements I and II together are necessary to answer the question. 72. How many persons are there in a straight line who are facing North ? I. L is standing exactly in the middle. L is an immediate neighbour of both A and O. Two persons are standing between A and T. T is standing at the second position from the left end of the line. B is standing at the extreme left end of the line. II. J is standing at the second position from the right end of the line. Five persons are standing between J and F. There are two persons between F and K. K is at one of the extreme ends of the line. 73. Who amongst the six friends- M, N, O, P, Q and R - is the heaviest ? I. O is heavier than only two friends. P is heavier than Q. P is lighter than N. II. M is lighter than only two friends. N is heavier than O. N is lighter than R. P is heavier than Q. 74. Six friends, E, F, G, H, I and J are sitting around a circular table facing towards the centre, but not necessarily in the same order. Find the position of G with respect to F. I. E is sitting second to the right of G. Only one person is sitting between E and I. F is an immediate neighbour of G. II. There are two persons between G and H. H is an immediate neighbour of both I and E. F is not an immediate neighbour of I. 75. What is the code for ‘reason’ in a certain code language ? I. In that code language ‘little reason to believe’ is coded as ‘& 4 $ 2’ and ‘reason is never little’ is coded as ‘# & 8 2’.

    II. In that code language ‘little to reason now’ is coded as ‘& 2 % 4’ and ‘believe now is problem’ is coded as ‘% 8 $ @’. 76. How is A related to F ? I. A is mother of B. D is brother of B. R is father of D. R has one son and one daughter. T is father of R. T is married to F. II. F is married to T. T has only two children R and C. R is married to A. A has two children. C is aunt of B and D. Directions (77–82) : Each of the questions below consists of a question and two statements numbered I and II are given below it. You have to decide whether the data provided in the statements are sufficient to answer the question. Read both the statements and — (IBPS Bank PO/MT CWE-IV, 18.10.2014)

    Give answer (1) if the data in Statement I alone are sufficient to answer the question, while the data in Statement II alone are not sufficient to answer the question. Give answer (2) if the data in Statement II alone are sufficient to answer the question, while the data in Statement I alone are not sufficient to answer the question. Give answer (3) if the data in Statement I alone or in Statement II alone are sufficient to answer the question. Give answer (4) if the data in both the Statements I and II together are not sufficient to answer the question. Give answer (5) if the data in both the Statements I and II together are necessary to answer the question. 77. Six friends- A, B, C, D, E and Fare sitting around a circular table. Some of them are facing outside while some others are facing towards the centre. What is the position of C with respect to F ? I. C is sitting second to the left of D. D is facing towards the centre. F is an immediate neighbour of both A and D. E is sitting second to the right of B. B is not an immediate neighbour of A. F faces just opposite to that of B. II. Two persons are sitting between D and E. D and E are facing towards the centre. E

    BPRE–900

    is an immediate neighbour of both C and A. F faces the same direction as that of D. D is an immediate neighbour of both B and F. F is not an immediate neighbour of C. 78. Seven people- P, Q, R, S, T, W and X– are sitting in a straight line facing north but necessarily in the same order. How many people sit to the right of P ? I. R sits at one of the extreme ends of the line. T has as many people sitting on his right, as to his left. II. S sits third to the left of X. Q sits to the immediate left of W. Q does not sit at any of the extreme ends of the line. 79. Point M is towards which direction from point H ? I. If a person walks 6 metres towards west from point M, takes a left turn and walks 6 metres again, he would be 5 metres away from point H. II. Point M is towards the North of point N; point N is towards the East of point T and point H is towards the East of point T. 80. How ‘party’ is written in a certain code language ? I. In that code language ‘going to a party’ is written as ‘la fa gi ne’ and ‘for a party’ is written as ‘fa di ne’. II. In that code language ‘start the party’ is written as ‘ne bs am’ and ‘going to start’ is written as ‘gi bs la’. 81. How is Ranjay related Parvati ? I. Ranjay is son of Parvati’s grandfather’s only daughter. II. Ranjay has no siblings. Parvati has only one brother. 82. Among M, R, H, D and S who scored the highest marks in an Examination ? I. R scored more than D but less than S. II. M scored less than H and D. H has not scored the highest marks. Directions (83–87) : Each of the following questions consists of a question and two statements numbered I and II given below it. You have to decide whether the data given in the

    DATA SUFFICIENCY statements are sufficient to answer the question. Read both the statements and choose the most appropriate option. (Bank of Baroda Junior Management Grade/Scale-I Exam, 18.04.2015)

    Mark answer (1) If the data in statement I alone are sufficient to answer the question, while the data in statement II alone are not sufficient to answer the question. Mark answer (2) If the data in statement II alone are sufficient to answer the question, while the data in statement I alone are not sufficient to answer the question. Mark answer (3) If the data either in statement I alone or in statement II alone are sufficient to answer the question. Mark answer (4) If the data in both statements I and II together are not sufficient to answer the question. Mark answer (5) If the data in both the statements I and II together are necessary to answer the question. 83. Among five friends A, B, C, D and E each studying in a different standard viz. 1st, 4th, 7th, 8th and 10th, in which standard does C study ? (‘Junior’ in the statements implies a lower standard and ‘senior’, higher standard) I. B studies in a standard which is an odd number. C is B’s senior but does not study in 10th standard. II. Only three persons are senior to A. B is senior to A but junior to C. 84. Four friends viz. P, Q, R and S are seated around a circular table. Some are facing the centre while some face outside (i.e. opposite to the centre). Which direction is R facing (centre or outside) ? I. R sits second to the left of S. R sits to the immediate right of Q. II. Q sits to the immediate right of R. P is an immediate neighbour of both R and S. 85. Point M is in which direction with respect to point A ? I. A person starts walking from point A, walks 5 m towards North and then takes a right turn and walks 8m. He then takes a left turn and walks for 3 m before stopping at

    point Z. Point M is 12 m away from point Z. II. A person starts from point A and walks 9 m towards the West. He then takes a right turn and walks 5 m. He then takes a final right turn and stops at point R after walking for 5 m. Point M is towards the North of point R. 86. Is Q the father of A ? I. M is the mother of A and B. C is the only sister of A. R is married to B. R is the daughter–in–law of Q. II. Q has only one daughter C. C is the sister of A and B. B is the son of M. R is the daughter–in–law of M. 87. Six persons are sitting in two parallel rows containing three persons each, in such a way that there is an equal distance between adjacent persons. In row – 1, A, B and C are seated and all of them are facing South. In row – 2, P, Q and R are seated and all of them are facing North. (Therefore, in the given seating arrangement each member seated in a row faces another member of the other row. Who amongst A, B and C faces R ? I. A faces the one who sits to the immediate right of R. A is not an immediate neighbour of C. II. Only one person sits between Q and P. The one who faces B sits to the immediate left of Q. Directions (88 –92) : Each of the following questions consists of a question and two statements numbered I and II given below it. You have to decide whether the data given in the statements are sufficient to answer the question. Read both the statements and choose the most appropriate option. (BOB Junior Management Grade/Scale–I Exam. 18.04.2015)

    Mark answer (1) If the data in statement I alone are sufficient to answer the question, while the data in statement II alone are not sufficient to answer the question. Mark answer (2) If the data in statement II alone are sufficient to answer the question, while the data in statement I alone are not sufficient to answer the question.

    BPRE–901

    Mark answer (3) If the data either in statement I alone or in statement II alone are sufficient to answer the question. Mark answer (4) If the data in both statements I and II together are not sufficient to answer the question. Mark answer (5) If the data in both the statements I and II together are necessary to answer the question. 88. Among five friends A, B, C, D and E each studying in a different standard viz. 1st, 4th, 7th, 8th and 10th, in which standard does C study ? (‘Junior’ in the statements implies a lower standard and ‘senior’, higher standard) I. B studies in a standard which is an odd number. C is B’s senior but does not study in 10th standard. II. Only three persons are senior to A. B is senior to A but junior to C. 89. Four friends viz. P, Q, R and S are seated around a circular table. Some are facing the centre while some face outside (i.e. opposite to the centre). Which direction is R facing (centre or outside) ? I. R sits second to the left of S. R sits to the immediate right of Q. II. Q sits to the immediate right of R. P is an immediate neighbour of both R and S. 90. Point M is in which direction with respect to point A ? I. A person starts walking from point A, walks 5 m towards North and then takes a right turn and walks 8m. He then takes a left turn and walks for 3 m before stopping at point Z. Point M is 12 m away from point Z. II. A person starts from point A and walks 9 m towards the West. He then takes a right turn and walks 5 m. He then takes a final right turn and stops at point R after walking for 5 m. Point M is towards the North of point R. 91. Is Q the father of A ? I. M is the mother of A and B. C is the only sister of A. R is married to B. R is the daughter–in–law of Q.

    DATA SUFFICIENCY II. Q has only one daughter C. C is the sister of A and B. B is the son of M. R is the daughter–in–law of M. 92. Six persons are sitting in two parallel rows containing three persons each, in such a way that there is an equal distance between adjacent persons. In row – 1, A, B and C are seated and all of them are facing South. In row – 2, P, Q and R are seated and all of them are facing North. (Therefore, in the given seating arrangement each member seated in a row faces another member of the other row. Who amongst A, B and C faces R ? I. A faces the one who sits to the immediate right of R. A is not an immediate neighbour of C. II. Only one person sits between Q and P. The one who faces B sits to the immediate left of Q. Directions (93 – 97) : Each of the following questions, consists of a question and two statements numbered I and II given below it. You have to decide whether the data provided in the statements are sufficient to answer the question. Read both the statements and mark the appropriate answer. (IBPS Bank PO/MT CWE–V Main Exam. 31.10.2015)

    Mark answer (1) If the data in statement I alone are sufficient to answer the question, while the data in statement II alone are not sufficient to answer the question. Mark answer (2) If the data in statement II alone are sufficient to answer the question, while the data in statement I alone are not sufficient to answer the question. Mark answer (3) If the data either in statement I alone or in statement II alone are sufficient to answer the question. Mark answer (4) If the data in both statements I and II together are not sufficient to answer the question. Mark answer (5) If the data in both the statements I and II together are necessary to answer the question. 93. P, Q, R, S and T are sitting around a circle facing towards the centre of the circle. Who is to the immediate right of T ?

    I. R, Q and S are in the same respective sequence to the immediate left of T. II. P is between S and T. 94. How many dresses does ‘P’ have? I. P has two dresses less than what T has. II. M has seven dresses, which are thirty percent less than what T has. 95. How is ‘smoking’ written in a code language? I. ‘Thanks for not smoking’ is written as ‘be je we no’ in that code language. II. ‘No Smoking Area’ is written as ‘no se do’ in that code language. 96. Who is the youngest among A, B, C, D and E? I. B is younger than C and D. II. A is younger than C but older than E. 97. How many sons does P have? I. C is the brother of B and A. II. P has three children of which B is a girl. Directions (98–99) : Each of the following questions, consists of a question and two statements numbered I and II given below it. You have to decide whether the data given in the statements are sufficient to answer the question. Read both the statements and mark the appropriate answer. (IBPS Specialist Officer (Marketing) CWE 01.02.2016)

    Mark answer (1) If the data in statement I alone are sufficient to answer the question, while the data in statement II alone are not sufficient to answer the question. Mark answer (2) If the data in statement II alone are sufficient to answer the question, while the data in statement I alone are not sufficient to answer the question. Mark answer (3) If the data either in statement I alone or in statement II alone are sufficient to answer the question. Mark answer (4) If the data in both statements I and II together are not sufficient to answer the question. Mark answer (5) If the data in both the statements I and II together are necessary to answer the question.

    BPRE–902

    98. How many persons are standing in straight line (Note : All are facing North)? I. Q stands third from the right end of the line. Only one person stands between Q and S. S stands at the extreme left end of the line. II. Q stands exactly in the centre of the line. P and M are immediate neighbours of Q. Only one person stands to the left of P. 99. How far is point M from point S? I. Point S is 7 m to the south of point M. Point T is 4 m to the east of point S. Point P is 4m to the north of point T. Point Q is to the west of point P. II. Point R is 4m to the west of point Q. Point S is 4m to the north of point Q. Point X is 4m to the north of point R. Point M is 11m to the east of point X. Directions (100–104) : Each of the following questions consists of a question and two statements numbered I and II given below it. You have to decide whether the data provided in the statements are sufficient to answer the question. Read both the statements and (United Bank of India PGDBF Manipal Exam,07.08.2016)

    Give answer (1) if the data in statement I alone are sufficient to answer the question, while the data in statement II alone are not sufficient to answer the question. Give answer (2) if the data in statement II alone are sufficient to answer the question, while the data in statement I alone are not sufficient to answer the question. Give answer (3) if the data in statement I alone or in statement II alone are sufficient to answer the question. Give answer (4) if the data in both the statements I and II are not sufficient to answer the question. Give answer (5) if the data in both the statements I and II together are necessary to answer the question. 100. Seven persons – M, N, O, P, Q, R and S – are sitting around a circular table. How many persons are facing the centre?

    DATA SUFFICIENCY I. N sits second to the right of P. Only two persons sit between P and Q. M sits to the immediate left of Q. II. R sits third to the left of S. N sits to the immediate right of S. Only one person sits between N and M. 101. How is T related to K? I. K is the mother of M. M is the mother of T. D is the daughter of T. F is the father of D. II. U is the only daughter of L. L is the mother of F. F is married to T. Z is the son of T. K is the grandmother of Z. 102. Six persons J, K, L, M, N and O are on different positions in an organisation with different years of experience, viz, 2, 8, 11, 16, 19 and 23 years, but not necessarily in the same order. Who amongst them is the most experienced? I. L is more experienced than O but less experienced than J. N is more experienced than J. N has the highest number of years of experience amongst the group of even numbers of experiences. II. K is more experienced than both J and N. J is less experienced than M. M has the most experience among the group of odd numbers of experiences. 103. Ten persons are sitting in two parallel rows with five persons in each row at equidistance from one another. In the row – 1, A, B, C, D and E are sitting, but not necessarily in the same order, and all of them are facing south. In the row – 2, P, Q, R, S and T are sitting, but not necessarily in the same order, and all of them are facing north. Therefore, in the given arrangement, the member of one row faces another member of the other row. Who among them faces S? I. R sits third to the right of P. S sits second to the left of T. S does not sit at any extreme ends of the row. S is not an immediate neighbour of P. Only two persons sit between E and D. II. A sits second to the left of C. B sits to the immediate right of C.

    Only two persons sit between A and B. Q is an immediate neighbour of the one who faces D. 104. Point T is in which direction with respect to point G? I. Point G is 15 metre to the north of point R. Point R is 20 metre to the west of point K. Point K is 30 metre to the south of point C. Point C is exactly midway between points S and T such that points S, C and T form a horizontal straight line of 40 metres. II. Point T is 10 metre to the east of point M. Point S is 30 metre to the west of point M. Point M is 10 metre to the east of point C. Point C is 15 metre to the north of point J. Point J is 20 metre to the east of point G. Directions (105–109) : Each of the questions consists of a question and two statements numbered I and II given below it. You have to decide whether the data given in the statements are sufficient to answer the questions. Read both the statements and choose the most appropriate answer : (Bank of Baroda Exam, 25.09.2016)

    Give answer (1) if the data in statement II alone is sufficient to answer the question while the data in statement I alone is not sufficient to answer the question. Give answer (2) if the data in both statements I and II together are necessary to answer the question. Give answer (3) if he data in statement I alone is sufficient to answer the question while data in statement II alone is not sufficient to answer the question. Give answer (4) if the data either in statement I alone or statement II alone are sufficient to answer the question. Give answer (5) if the data even in both the statements I and II together are not sufficient to answer the question. 105. Is P the husband of Q ? I. S is the brother of Q. V is the only brother of S. M is the father of V. M has only three children. P is the son-in-law of M. II. P is married to Q. A is the mother-in-law of Q. A is the mother of E. G is the only sister of E.

    BPRE–903

    106. How is ‘import’ definitely coded in the given code language? (All the given codes are two letter codes only) I. ‘import high quality goods’ is coded as ‘fa ju ra ti’ and ‘monitor quality of goods’ is written as ‘mp fa cd ju’. II. ‘import uranium for plant’ is coded as ‘na ti vo ku’ and ‘plant for uranium access’ is coded as ‘vo zy ku na’. 107. In a five storey building (consisting of floors numbered 1 to 5, wherein the ground floor is numbered 1, the floor above it is numbered 2 and so on till the topmost floor is numbered 5.) each of the five friends namely A, B, C, D and E lives on a different floor. On which floor number does D live ? I. C lives on an even numbered floor. Only two persons live between C and E. B lives on an even numbered floor. A lives immediately below B. II. A lives on an odd numbered floor but not on the lowermost floor. E lives on one of the floors above A. Only two people live between E and C. No one lives between C and D. 108. Five different exams Q, R, S, T and U were conducted in January, February, July, September and December of the same year (but not necessarily in the same order). In which month was exam Q conducted ? (No exam was conducted in any other month in that year) I. U was conducted in a month having less than 31 days. Only two exams were conducted between U and T. No exam was conducted between Q and S. II. R was conducted in a month having 31 days but not July. Only three exams were held between R and T. No exams were held between U and S. 109. Three boxes, A, B and C, each having a different colour, yellow, blue and red are kept one above the other (but not necessarily in the same order). How many boxes (none/one/two) are kept below the blue covered box ?

    DATA SUFFICIENCY I. Only one box is kept between the blue and the yellow box. A is kept immediately below the red box. II. Yellow coloured box is the lowermost. Blue box is kept immediately above C. B is not blue in colour. Directions (110–112) : Each of the following questions consists of a question and two statements numbered I and II given below it. You have to decide whether the data given in the statements are sufficient to answer the question. Read both the satements and choose the most appropriate answer. (Bank of Maharashtra PO Exam, 26.10.2016)

    Give answer (1) if the data in statement I alone are sufficient to answer the question, while the data in statement II alone are not sufficient to answer the question. Give answer (2) if the data in statement II alone are sufficient to answer the question, while the data in statement I alone are not sufficient to answer the question. Give answer (3) if the data in statement I alone or in statement II alone are sufficient to answer the question. Give answer (4) if the data in both the statements I and II are not sufficient to answer the question. Give answer (5) if the data in both the statements I and II together are necessary to answer the question. 110. Five different classes- J, K, L, M and N — were conducted in five different days of the same week starting from Monday and ending on Friday, but not necessarily in the same order. On which day was class M conducted ? I. K was conducted on one of the days before Wednesday. Only two classes were conducted between K and L. M was conducted immediately before L. II. Only three classes were conducted between N and L. Only one class was conducted between L and J. K was conducted immediately before J. 111. Among P, Q, R, S and T sitting in a straight line with equal distance between each other and facing north, how many people sit to the left of Q ?

    I.

    T sits at an extreme end of the line. Only two persons sit between T and P. Q sits to the immediate left of P. II. S sits third to the right of R. Only one person sits between S and Q. T sits second to the left of Q. 112. Three books, A, B and C, each on a different subject, Physics, Mathematics and Sociology are kept one above the other but not necessarily in the same order. How many books (none/ one or two) are kept below the Physics book ? I. Only one book is kept between the Mathematics book and Physics book. B is kept immediately below the Sociology book. B is not a Mathematics book. II. C is kept immediately below the Mathematics book. A is kept immediately above the Sociology book. Only one book is kept between A and B. Directions (113–115) : Each of the questions below consists of a question and two statements numbered I and II given below it. You have to decide whether the data provided in the statements sufficient to answer the questions. (IBPS Bank PO/MT CWE (Main) Exam, 18.11.2016

    Read both the statements and Mark answer (1) if the data in statement I alone are sufficient to answer the question, while the data in statement II alone are not sufficient to answer the question. Mark answer (2) if the data in statement II alone are sufficient to answer the question while the data in statement I alone are not sufficient to answer the question. Mark answer (3) if the data either in statement I alone or in statement II alone are sufficient to answer the question. Mark answer (4) if the data in both statements I and II together are not sufficient to answer the question. Mark answer (5) if the data in both statements I and II together are neccessary to answer the question. 113. What is the position of Yugal from the left in a group of 25 students, each of them facing North?

    BPRE–904

    I. Prakash is exactly in the middle of the row. There are five students between Prakash and Yugal. Yugal is sitting to the right of Prakash. II. There are two students between Sonam and Prakash and there are another two students between Sonam and Yugal. 114. Are Mohan, Sohan and Narendra in a straight line ? I. Narendra is in 6 metres East of Mohan. Om is 4 metres North of Narendra. Tejas is in 3 metres West of Om. Sohan is in 4 metres south of Tejas. II. Mohan is at the distance of 6 metres in North of Prem. Kaushal is exactly in the middle of Prem and Mohan. Ramesh is at the distance of 3 metres in East of Kaushal. Sohan is in 3 metres South of Ramesh, Narendra east of Sohan. 115. What is the code for ‘Sunday’ ? I. ‘Forgiveness is virture’ is coded as ‘12T 18T 44F’ II. ‘Might is right’ coded as ‘M∝20 R∝19 I∝20’. Directions (116 – 120) : Each of the following questions, consists of a question and two statements numbered I and II given below it. You have to decide whether the data given in the statements are sufficient to answer the question. Read both the statements and mark the appropriate answer. (IBPS SO (Agriculture) Exam, 29.01.2017)

    Mark answer (1) If the data in statement I alone are sufficient to answer the question, while the data in statement II alone are not sufficient to answer the question. Mark answer (2) If the data in statement II alone are sufficient to answer the question, while the data in statement I alone are not sufficient to answer the question. Mark answer (3) If the data either in statement I alone or in statement II alone are sufficient to answer the question. Mark answer (4) If the data in both statements I and II together are not sufficient to answer the question. Mark answer (5) If the data in both the statements I and II together are necessary to answer the question.

    DATA SUFFICIENCY 116. How many people are standing between H and U in a straight line of 15 people (Note : All are standing in a straight line facing north.)? I. M stands fifth from the left end of the line. H and K are immediate neighbours of M. Only two people stand between M and U. II. B stands third from the right end of the line. Only six people stand between K and B. Only one person stands between H and K. K stands exactly between H and U. 117. How is B related S? I. N is married to T. B is the sister of N. T has only one daughter K. K is the granddaughter of S. II. N is the daughter of S. N is married to T.T is the only child of Z. B is the sister-in-law of T. 118. Among people A, B, C, D, E and F, each having a different weight, who is the second heaviest? I. C is heavier than only two people. A is heavier than F but lighter than B. D is not the heaviest. II. F is lighter than only two people. C is lighter than F but heavier than D. A is lighter than only B. 119. How many people are standing in a straight line (Note: All are facing north)? I. U stands third from the left end of the line. U is an immediate neighbour of P and W. Only one person stands between W and T. Only two people stand to the right of W. II. S stands at extreme left end of the line. T stands to the extreme right end of the line. Only one person stands between S and U. Only one person stands between T and W. 120. How far is point M from point K? I. Point D is 5m to the south of Point P. Point M is 8m to the west of point D. Point S is 2.5m to the north of point M. Point O is 10m to the east of Point S. Point K is 2.5m to the south of point O. II. Point K is 10m to the east of point M. Point U is 8m to the west of Point M. Point D is to the east of M. Point M is the midpoint of the lines formed by joining points U and D.

    Directions (121–125) : Each of the questons given below consists of a question and two statements numbered I and II given below it. You have to decide whether the data provided in the statements are sufficient to answer the question. Read both the statements and mark the appropriate option : (IBPS RRBs Officer CWE (Main Exam) 05.11.2017)

    Give answer (1) if the data in statement I alone are sufficient to answer the question, while the data in statement II alone are not sufficient to answer the question. Give answer (2) if the data in statement II alone are sufficient to answer the question, while the data in statement I alone are not sufficient to answer the question. Give answer (3) if the data in statement I alone or in statement II alone are sufficient to answer the question. Give answer (4) if the data in both the statements I and II are not sufficient to answer the question. Give answer (5) if the data in both the statements I and II together are necessary to answer the question. 121. What is the code for ‘festival’ in a code language? I. In that language ‘lo ko ni sa’ means ‘celebrate festival light bright’ and ‘jo to ni fa’ means ‘festival surprise candle shine’. II. In the same language ‘bi ya la fa’ means ‘surprise high celebration delight’ and ‘ya la fa ni’ means ‘surprise festival high delight’. 122. Six persons A, B, C, D, E and F are sitting around a circular table. Are they all facing the centre? I. A sits second to the left of F. D sits second to the right of F. Both E and C are immediate neighbours of A. F is facing the centre. II. B is second to the left of E. Only D is between B and E. C is to immediate left of F. B sits opposite to A. C is third to the left of D. E is second to the right of B and second to the left of C. 123. Six persons A, B, C, D, E, F have different heights. Who among them is second tallest?

    BPRE–905

    I. F is taller than only one person. C is taller than A but shorter than E. II. A is taller than both F and B. E is not the tallest. 124. Which bird sits second to the right of Sparrow? I. Five birds myna, parrot, bulbul, crow and sparrow are sitting in a row facing north direction. Myna sits at one of the extreme ends. Parrot sits exactly between Myna and Bulbul. II. Only one bird sits between Bulbul and Sparrow. Sparrow does noy sit to the left of Parrot. All birds are facing north direction. 125. Five persons P, Q, R, S, T live on five different floors of a five storeyed building. The bottom floor is numbered as 1 and top floor is numbered as 5. Who among them lives on fourth floor? I. Only three persons live between Q and T. R lives on third floor. II. Only one person lives between P and S. T does not live above R. Directions (126–128) : Each of the questions given below consists of a question and two statements numbered I and II given below it. You have to decide whether the data provided in the statements are sufficient to answer the question. Read both the statements and (IBPS SO (IT Officer) CWE (Prelim Exam) 30.12.2017)

    Give answer (1) if the data in statement I alone are sufficient to answer the question, while the data in statement II alone are not sufficient to answer the question. Give answer (2) if the data in statement II alone are sufficient to answer the question, while the data in statement I alone are not sufficient to answer the question. Give answer (3) if the data in statement I alone or in statement II alone are sufficient to answer the question. Give answer (4) if the data in both the statements I and II are not sufficient to answer the question. Give answer (5) if the data in both the statements I and II together are necessary to answer the question. 126. How many students are there in class 8 in school K? I. The class teacher of class 8 correctly remembers that the num-

    DATA SUFFICIENCY ber of students in her class is more than 21 but less than 32. The number of students in her class is exactly divisible by 6. II. The principal of school K correctly remembers that the difference between the number of students in class 7 and class 8 is less than nine. There are 33 students in class 7. 127. Among A, B, C, D and E, seated around a circular table, facing the centre, who sits second to the right of C? I. C sits to the immediate right of A. Only two persons sit between B and A (when counted from left or right). II. Both D and C are immediate neighbours of B. E sits second to the right of B. 128. Amongst five bags, L, M, N, O and P, each having a different weight, which is the lightest? I. Only two bags are heavier than O. O is heavier than P but lighter than M. II. N is lighter than O but heavier than P. L is heavier than O, but not the heaviest. Directions (129–130) : Each of the questons given below consists of a question and two statements numbered I and II given below it. You have to decide whether the data provided in the statements are sufficient to answer the question. Read both the statements and mark the appropriate option : (IBPS SO (Law Officer) CWE (Prelim Exam) 31.12.2017)

    Give answer (1) if the data in statement I alone are sufficient to answer the question, while the data in statement II alone are not sufficient to answer the question. Give answer (2) if the data in statement II alone are sufficient to answer the question, while the data in statement I alone are not sufficient to answer the question. Give answer (3) if the data in statement I alone or in statement II alone are sufficient to answer the question. Give answer (4) if the data in both the statements I and II are not sufficient to answer the question.

    Give answer (5) if the data in both the statements I and II together are necessary to answer the question. 129. Five events viz. A, B, C, D and E were held on five different days of the same week starting from Monday and ending on Friday. On which day was event C definitely held? (Note: Only one event was held on each day) Statements : I. Event B was held on one of the days before Wednesday. Only one event was held between Event B and D. Event E was held on one of the days before Event A but after event C. II. Event D was held on one of the days after Tuesday. Event A was held on one of the days after Event D. Event B was held before Event E. Event C was held before Event A. 130. Eight people are sitting in two parallel rows containing four people each with equal distance between each other. In row 1, A, B, C and D are seated and all of them are facing north. In row 2, P, Q, R and S are seated and all of them are facing south. Thus, each member seated in a row faces another member of the other row. Who amongst the following faces P? Statements : I. A sits at an extreme end of the row. Only one person sits between the one who faces A and Q. The one who faces Q sits to the immediate left of D. The one facing D is an immediate neighbour of R. C sits to the immediate left of B. P sits at one of the positions to the right of the S. II. R sits at one of the positions to the left of P but not to the immediate left. The one facing R sits second to the right of B. D sits to the immediate left of A. D does not face R. Q sits to the immediate right of S. Directions (131–134) : Each of the questions given below consists of a question and two statements numbered I and II given below it. You have to decide whether the data given in the

    BPRE–906

    statements are sufficient to answer the question. Read both the statements and mark the appropriate answer. (Canara Bank PO Exam 04.03.2018)

    Give answer (1) if the data in statement I alone are sufficient to answer the question, while the data in statement II alone are not sufficient to answer the question. Give answer (2) if the data in statement II alone are sufficient to answer the question, while the data in statement I alone are not sufficient to answer the question. Give answer (3) if the data in statement I alone or in statement II alone are sufficient to answer the question. Give answer (4) if the data in both the statements I and II are not sufficient to answer the question. Give answer (5) if the data in both the statements I and II together are necessary to answer the question. 131. How far and in which direction is the point B with respect to the point A? Statements : I. Point G is 6m to the east of Point A. Point C is 9m to the north of Point G. Point F is 3m to the west of Point C. Point B is 6m away from point F. II. Point M is 8m to the west of point B. Point R is 8 m to the south of Point M. Point A is 11 m to the east of Point R. Point C is to the north east of Point A. 132. Amongst the six people viz. A, B, C, D, E and F sitting around a circular table facing the centre, who sits second to the right of A? Statements : I. A sits second to the right of F. Only two people sit between A and D. B is neither an immediate neighbour of D nor F. II. Only one person sits between A and F (either from left or right). Only two people sit between F and B. C sits to the immediate left of D. D is not an immediate neighbour of B. 133. Five boxes viz. A, B, C, D and E are kept one above the other in a stack. How many boxes are kept between box E and box B?

    DATA SUFFICIENCY Statements : I. Only two boxes are kept between box C and box D. Only one box is kept between box C and box B. Only two boxes are kept between box E and box A. Box E is kept at one of the positions above box A and below box C. II. No box is kept between box D and box B. Only two boxes are kept between box C and box D. Box E is kept at one of the positions above box A but below Box C. 134. Among six people viz. C, D, E, F, G and H, sitting in a straight line with equal distance between each other and facing north, who sits second to the left of G? Statements : I. C sits third from the left end of the line. Only one person sits between C and H. Only two people sit between E and G. G sits at one of the positions to the right of E. II. E sits third to the left of G. G does not sit at an extreme end of the line. More than three people sit between H and F. F sits at one of the positions to the right of H. Directions (135–138) : Each of the following questions consists of a question and two statements numbered I and II given below it. You have to decide whether the data given in the statements are sufficient to answer the question. Read both the satements and choose the most appropriate answer. (IDBI Bank PO Exam 29.04.2018)

    Give answer (1) if the data in statement I alone are sufficient to answer the question, while the data in statement II alone are not sufficient to answer the question. Give answer (2) if the data in statement II alone are sufficient to answer the question, while the data in statement I alone are not sufficient to answer the question. Give answer (3) if the data in statement I alone or in statement II alone are sufficient to answer the question. Give answer (4) if the data in both the statements I and II are not sufficient to answer the question.

    Give answer (5) if the data in both the statements I and II together are necessary to answer the question. 135. How is ‘also’ written in a code language? I. ‘he also show data’ is written as ‘sx fa mn ca’ and ‘ now many person also’ is written as ‘zb ct sx ya’ in that code language. II. ‘she visit the also’ is written as ‘sx lm nc ty’ and ‘she visit the always’ is written as ‘lm kc ty nc’ in that code language. 136. Point Q is in which direction with respect to Point P? I. Point P is in east of Point L. Point L is in north of Point M. Point M is in west of Point N. Point N is in south of Point O. Point O is in west of Point Q. II. Point L is in west of Point P and north of Point M. Point M is in east of Point N which is south of Point O. Point Q is in east of Point O and north of Point L. 137. Six persons viz. A, B, C, D, E and F are sitting on a circular table for lunch, who among them sits to the immediate left of F (If all the persons are facing towards the centre)? I. A sits third to right of F. One person sits between A and C. E sits third to left of C. II. Only one person sits between E and D. F sits second to right of D. A sits third to left of F. 138. Sharvan’s birthday is in which of the following month in a year? I. Sharvan’s mother correctly remembers that Sharvan’s birthday will come after September but not in the month which has 30 days. II. Sharvan’s father correctly remember that Sharvan’s birthday will not come in first and last month of the year. Directions (139-140) : Each of the questions below consists of a question and two statements numbered I and II given below it. You have to decide whether the data provided in the statements are sufficient to answer the question. Read both the statements and Give answer (1) if the data in statement I alone are sufficient to answer the question, while the data in

    BPRE–907

    (2)

    (3)

    (4)

    (5)

    statement II alone are not sufficient to answer the question. if the data in statement II alone are sufficient to answer the question, while the data in statement I alone are not sufficient to answer the question. if the data either in statement I alone or in statement II alone are sufficient to answer the question. if the data even in both statements I and II together are not sufficient to answer the question. if the data in both statements I and II together are necessary to answer the question.

    (IBPS Bank PO/MT CWE (Main Exam) 26.11.2017)

    139. Five friends A, B, C, D, E were born on different dates 10, 15, 19, 24, 30 but not necessarily in the same order of the month of February and October in the same year. D was born on which date? I. B and E were born in the same month and E is elder to B. Both were born on an even numbered date. Two people A and C were born between E’s and B’s dates. II. B was born in October between 15 and 30. D was born in February after 19th but before 28th. C was born on odd number date in October before 18th. 140. There are six floors in an apartment numbered 1 to 6 from the bottom most floor to the top floor. Six professionals A, B, C, D, E and F are staying on each floor. F stays on which floor ? I. C is on even numbered floor but not the top floor. Only three persons are staying between the floors of B and D. Only three persons are between A and E who is on even numbered floor. II. D needs not to use lift facility to reach his floor. Directions (141–143) : Each of the following questions, consists of a question and two statements numbered I and II given below it. You have to

    DATA SUFFICIENCY decide whether the data given in the statements are sufficient to answer the question. Read both the statements and choose the most appropriate option. Give answer (1) if the data in statement I alone are sufficient to answer the question, while the data in statement II alone are not sufficient to answer the question. Give answer (2) if the data in statement II alone are sufficient to answer the question, while the data in statement I alone are not sufficient to answer the question. Give answer (3) if the data in statement I alone or in statement II alone are sufficient to answer the question. Give answer (4) if the data in both the statements I and II are not sufficient to answer the question. Give answer (5) if the data in both the statements I and II together are necessary to answer the question. (Canara Bank PO Exam, 23.12.2018)

    141. Five different classes — J, K, L, M and N — were conducted in five different days of the same week starting from Monday and ending on Friday, but not necessarily in the same order. On which day was class M conducted? I. K was conducted on one of the days before Wednesday. Only two classes were conducted between L and K. M was conducted immediately before L. II. Only three classes were conducted between N and L. Only one class was conducted between L and J. K was conducted immediately before J. 142. Among P, Q, R, S and T sitting in a straight line with equal distance between each other and facing north, how many people sit to the left of Q? I. T sits at an extreme end of the line. Only two persons sit between T and P. Q sits to the immediate left of P. II. S sits third to the right of R. Only one person sits between S and Q. T sits second to the left of Q. 143. Three books, A, B and C, each on a different subject, Physics,

    Mathematics and Sociology are kept one above the other but not necessarily in the same order. How many books (none/one or two) are kept below the Physics book? I. Only one book is kept between the Mathematics book and Physics book. B is kept immediately below the Sociology book. B is not a Mathematics book. II. C is kept immediately below the Mathematics book. A is kept immediately above the Sociology book. Only one book is kept between A and B. Directions (144–148) : Each of the followig questions, consists of a question and two statements numbered I and II given below it. You have to decide whether the data given in the statements are sufficient to answer the question. Read both the statements and choose the most appropriate question. Give answer (1) if the data in statement I alone are sufficient to answer the question, while the data in statement II alone are not sufficient to answer the question. Give answer (2) if the data in statement II alone are sufficient to answer the question, while the data in statement I alone are not sufficient to answer the question. Give answer (3) if the data in statement I alone or in statement II alone are sufficient to answer the question. Give answer (4) if the data in both the statements I and II are not sufficient to answer the question. Give answer (5) if the data in both the statements I and II together are necessary to answer the question. (IBPS Specialist Officer CWE (Prelim Exam) 31.01.2019)

    144. Among people J, K, L, M, N and O, each having a different weight, who is the second lightest? (I) Only three people are lighter than M. L is lighter than M. L is heavier than N but lighter than J. O is heavier than K. (II) K is lighter than only one person. J is heavier than L but lighter than M. M is not the heaviest. L is not the lightest.

    BPRE–908

    145. Eight people are sitting in two parallel rows containing four people each, in such a way that there is equal distance between adjacent persons. In row-1 W, X, Y and Z are seated and all of them are facing south. In row-2 M, N, O and P are seated and all of them are facing north. Therefore in the given seating arrangement each member seated in a row faces another member of the other row. (Please Note: None of the information given is necessarily in the same order). Who amongst M, N, O and P faces Z? (I) Z sits second to the left of X. Only one person sits between the one who faces Z and P. Only one person sits between N and O. N sits to the immediate right of P. (II) X sits to the immediate right of Y. X sits on one of the extreme ends of the row. O faces W. P sits to the immediate left of N. 146. Three boxes, U, V and W each having a different colour, Blue, Orange and Yellow are kept one above the other (but not necessarily in the same order). How many boxes (none/one/two) are kept above V? (I) Box V is kept immediately above box U. Yellow box is kept immediately above Blue box. V is not the Blue box. U is not the Yellow box. (II) Only one box is kept between Yellow box and W. Orange box is kept immediately below U. Box V is kept on one of the positions above the Orange box. 147. How is ‘silver’ definitely coded in the given code language? (All the given codes are two letter codes only) (I) ‘make silver ornaments today’ is written as ‘ly xt sj br’ and ‘silver ornaments for ceremony’ is coded as ‘br mp zl ly’. (II) ‘ornaments in silver only’ is coded as ‘qf br hk ly’ and ‘only today for preparation’ is coded as ‘mp hk ax sj’. 148. Seven different plays- A, B, C, D, E, F and G have been scheduled on seven different days of

    DATA SUFFICIENCY the same week starting from Monday to Sunday. Which play will be held on Friday? (I) Play A is scheduled on Wednesday. Only two plays are scheduled between plays A and D. Play E is scheduled immediately before B. More than two plays are scheduled before play B. (II) Play C is scheduled on Tuesday. Play D is scheduled on Saturday. Play A is scheduled immediately before Play E. The number of plays scheduled between E and F is equal to the number of plays scheduled between E and G. 149. Study the following information carefully and answer the given question : A new online food ordering and delivering company announced the following offers in the last month to attract customers: Offer 1: Flat 10% off on minimum order of Rs. 1,000. Offer 2 : If the order value ranges between Rs. 500 to Rs. 999 then 20% off on the subsequent order placed on the next day, regardless of the value of the order. Offer 3 : Sweet dish free on all orders placed on Friday, Saturday or Sunday. Offer 4: Free delivery at a cost of Rs. 19 per month instead of a delivery charge on each order. Offer 5: The price for the item which costs the least would be waived for each order placed between 6 p.m. to 8 p.m. None of the offers can be clubbed together. At the end of the month 100 orders were received, the order patterns were observed as follows: • Offer 1 was availed twice the number of times than offer 2. • The number of orders from Friday to Sunday remained unchanged. • Only 5 people availed offer 4. • The number of items per order between 6 p.m. to 8 p.m. increased significantly.

    • The number of orders with cart value above Rs. 500 were only 12. Based on the given pattern of offers, which of the following offers should be discontinued in the next month? (1) Offer 1 and Offer 3 (2) Offer 2 and Offer 4 (3) Offer 2, Offer 3 and Offer 4 (4) Offer 3, Offer 4 and Offer 5 (5) Offer 2 (IBPS SO Prelim Exam, 28.12.2019)

    150. Study the following information carefully and answer the given question: Last month, company P incorporated some new rules regarding over-time in the staff rule book. At present, the company runs on a 5 day (Monday to Friday) model with 10:00 a.m. to 06:00 p.m. as its working hours. The new rules incorporated (to be implemented from next month) are: • On any given working day, no employee should work beyond 8:00 p.m. irrespective of the circumstances. • If an employee wants to do overtime during a holiday, he/she will have to take prior permission from the head of the department. Also, he/she will have to work only between 10:00 a.m. to 04:00 p.m. • No compensation will be paid for over-time on working days from now on. Which of the following instances cannot be a reason for incorporating the mentioned new rules? (1) The employees in support staff department were also required to report on almost every weekend/holiday for assistance to employees without being given any prior intimation. (2) All the given instances can be reasons for incorporating the mentioned new rules. (3) As compared to the previous year, the disbursement towards compensation for

    BPRE–909

    over-time increased by 40% whereas the workload of each employee remained more or less the same (4) During the inspections carried in the last six months, many employees were found to be staying late even if they had reasonable time to finish their targets in normal working hours. (5) The fare for the public transport in the city in which company P is located increases by 10% after 8:00 p.m. (IBPS SO Prelim Exam, 28.12.2019)

    SBI PO EXAMS Directions (1-5) : Each of the questions below consists of a question and two statements numbered I and II given below it. You have to decide whether the data provided in the statements are sufficient to answer the question. Read both the statements and (SBI Associate Banks PO Exam. 07.08.2011)

    Give answer (1) if the data in statement I alone are sufficient to answer the question, while the data in statement II alone are not sufficient to answer the question. Give answer (2) if the data in statement II alone are sufficient to answer the question, while the data in statement I alone are not sufficient to answer the question. Give answer (3) if the data either in statement I alone or in statement II alone are sufficient to answer the question. Give answer (4) if the data even in both statements I and II together are not sufficient to answer the question. Give answer (5) if the data in both statements I and II together are necessary to answer the question. 1. Which bag amongst P, Q, R, S and T is the heaviest ? I. Bag Q is heavier than R and S. Bag T is heavier only than bag P. II. Only three bags are lighter than R. The weight of bag Q is 50 kg which is 2 kg more than bag R.

    DATA SUFFICIENCY 2. Are all the five friends viz. A, B, C, D and E who are seated around a circular table facing the centre ? I. A sits third to the left of B. B faces the centre. D and E are immediate neighbours of each other. C sits second to right of E. II. D sits second to right of C. C faces the centre. Both E and A are immediate neighbours of D. B sits second to right of A. 3. In a college five different subjects, viz. Physics, Chemistry, Botany, Zoology and Mathematics are taught on five different days of the same week, starting from Monday and ending on Friday. Is Chemistry taught on Wednesday ? I. Two subjects are taught between Zoology and Mathematics. Mathematics is taught before Zoology. Chemistry is taught on the day immediately next to the day when Physics is taught. Botany is not taught on Friday. II. Three lectures are scheduled between the lectures of Botany and Zoology. Mathematics is taught immediately before Physics. 4. Is the time in the clock 9 O’clock now ? I. After half an hour, the minute and the hour hands of the clock will make an angle of exactly 90° with each other. II. Exactly 15 minutes back, the hour and the minute’s hand of the clock coincided with each other. 5. Is F the granddaughter of B ? I. B is the father of M. M is the sister of T. T is the mother of F. II. S is the son of F. V is the daughter of F. R is the brother of T. Directions (6–8) : Each of the questions below consists of a question and two statements numbered I and II given below it. You have to decide whether the data given in the statements are sufficient to answer the questions. Read both the statements and — (SBI Probationary Officer Exam 28.04.2013)

    Give answer (1) if the data in statement I alone are sufficient to answer the question, while the data in statement II alone are not sufficient to answer the question. Give answer (2) if the data in statement II alone are sufficient to answer the question, while the data in statement I alone are not sufficient to answer the question. Give answer (3) if the data either in statement I alone or statement II alone are sufficient to answer the question. Give answer (4) if the data neither in statement I nor II together are sufficient to answer the questions. Give answer (5) if the data in both statements I and II together are necessary to answer the questions. 6. Seventeen people are standing in a straight line facing south: What is Bhavna’s position from the left end of the line ? I. Sandeep is standing second to the left of Sheetal. Only five people stand between Sheetal and the one who is standing at the extreme right end of the line. Four people stand between Sandeep and Bhavna. II. Anita is standing fourth to the left of Sheetal. Less than three people are standing between Bhavna and Anita. 7. Five letters - A, E, G, N and R are arranged left to right according to certain conditions. Which letter is placed third? I. G is placed second to the right of A. E is to the immediate right of G. There are only two letters between R and G. II. N is exactly between A and G. Neither A nor G is at the extreme end of the arrangement. 8. Six people - S, T, U, V, W and X are sitting around a circular table facing the centre. What is T’s position with respect to X ? I. Only two people sit between U and W. X is second to the left of W. V and T are immediate neighbours of each other. II. T is to the immediate right of V. There are only two people between T and S. X is an immediate neighbour of S but not of V.

    BPRE–910

    Directions (9-12) : Each of the questions below consists of a question and three statements numbered I, II and III given below it. You have to decide whether the data provided in the statements are sufficient to answer the question. (SBI Management Executive Exam, 23.02.2014)

    9. Who amongst P, Q, R, S, T and V, each securing different marks, secured the second lowest marks ? I. R and T secured more marks than P and Q II. V secured the highest marks III. S secured more marks than P but less than Q. (1) Only I and III (2) All I, II and III are required to answer the question (3) Only II and III (4) Question cannot be answered even with all I, II and III (5) Only I and II 10. Which village is to the North-East of village R ? I. Village S is to the South-East of village N which is to the South-West of village P and village P is to the North of village Q. II. Village T is to the North-West of village Q which is to the south of village P. III. Village R which is to the North of village S, lies between villages N and Q and village N is to the West of village R. (1) Only I and II (2) Only II and III (3) All I, II and III are not sufficient to answer the question (4) All I, II and III are required to answer the question (5) Only I and III or only II and III are required to answer the question 11. What is the rank of Animesh in a class of 17 students ? I. Nirmal who is thirteenth from the bottom is six rank ahead of Bhumika who is two position below Animesh. II. Bhumika is four position ahead of Kamal. III. Bhumika is two position below Animesh and Kamal’s rank is 15th.

    DATA SUFFICIENCY (1) Only I and III (2) Only I and II (3) Only I or II and III together are required to answer the question (4) Only II is required to answer the question (5) All I, II and III are not sufficient to answer the question 12. How is ‘them’ written in a code language ? I. ‘tell them young’ is written as ‘se me ye’ and ‘wise young sharp tell’ is written as ‘me yo na ye’ in that code language. II. ‘clever sharp come tomorrow’ is written as ‘na ki pa lo’ and ‘bring clever young them’ is written as ‘ki po se ye’ in that code language. III. ‘clever sharp come them no’ is written as ‘pa na se ki te’ and ‘yellow come sharp run clever no’ is written as ’ki ni pa be te na’ in that code language. (1) Only III is required to answer the question (2) Only I and II are required to answer the question (3) Only I or II and III together are required to answer the question (4) Only II is required to answer the question (5) All I, II and III are not sufficient to answer the question Directions (13-17) : Each of the questions below consists of a question and two statements numbered I and II given below it. You have to decide whether the data provided in the statements are sufficient to answer the question. Read both the statements and (SBI Specialist Officer (Law Officer : MMGS Scale-II) Online Exam, 19.04.2014)

    Give answer (1) if the data in Statement I alone are sufficient to answer the question, while the data in Statement II alone are not sufficient to answer the question. Give answer (2) if the data in Statement II alone are sufficient to answer the question, while the data in Statement I alone are not sufficient to answer the question. Give answer (3) if the data in Statement I alone or in Statement II alone are sufficient to answer the question.

    Give answer (4) if the data in both the Statements I and II together are not sufficient to answer the question. Give answer (5) if the data in both the Statements I and II together are necessary to answer the question. 13. How is H related to B ? I. H is married to P. P is the mother of T. T is married to D. D is the father of B. II. B is the daughter of T. T is the sister of N. H is the father of N. 14. Among five persons D, E, F, G and H, each of whom having different height, who is the second tallest ? I. D is taller than only G and E. F is not the tallest. II. H is taller than F. G is taller than E but shorter than D. 15. In which direction is J with respect to R ? I. R is to the west of P. P is to the south of G. J is to the west of G. When R, P, G and J are joined by straight lines, a square is formed. II. K is to the south of J. H is to the east of K. R is to the north-west of H. When R, K and H are joined by straight lines, a right angled triangle is formed. 16. What is the position of K from the right end in a row ? I. There are even number of students in the row. There are not more than 11 students in the row. II. C stands at the extreme left end of the line. There are five students between C and B. O is third to the right of B. K is not at the extreme end. There is only one student between O and K. 17. On which day of the week did Arvind visit Mumbai ? I. Arvind visited Mumbai after Monday but before Thursday but not on an odd day of the week. II. Arvind visited Mumbai before Friday but after Monday. Directions (18-22) : Each of the questions below consists of a question and two statements numbered I and II given below it. You have to de-

    BPRE–911

    cide whether the data provided in the statements are sufficient to answer the question. Read both the statements and — (SBI Probationary Officer Online Exam, 21.06.2014)

    Give answer (1) if the data in Statement I alone are sufficient to answer the question, while the data in Statement II alone are not sufficient to answer the question. Give answer (2) if the data in Statement II alone are sufficient to answer the question, while the data in Statement I alone are not sufficient to answer the question. Give answer (3) if the data either in Statement I alone or in Statement II alone are sufficient to answer the question. Give answer (4) if the data even in both Statements I and II together are not sufficient to answer the question. Give answer (5) if the data in both Statements I and II together are necessary to answer the question. 18. What is the code of ‘shine’ in a certain code language? I. In that code language ‘shine was peeled off’ is written as ‘& # @ 9’ and ‘no paint but shine’ is written as ‘7 5 # 8’. II. In that code language ‘try the new shine’ is written as ‘13 # 0’ and ‘we try the new’ is written as ‘6013’, 19. How C is related to H? I. N is son of H. J is mother of Z. N and Z are cousins. C is husband of J. II. L is father of C. A is mother of D. L is married to A. H is wife of D. J is wife of C. 20. In which direction point ‘A’ is located with respect to point ‘B’? I. A man starts walking from point ‘A’ towards east and after walking 3 metres reaches point ‘N’, he turns right and walks 7 metres to reach point ‘M’. Then he turns right and walks 6 metres to reach point ‘O’. He again turns right and walks 7 metres to reach point ‘P’. He, then, turns left and walks 2 metres to reach point ‘B’. II. A man starts walking from point ‘A’ towards east and after walking 3 metres reaches point ‘N’. From point ‘N’ he

    DATA SUFFICIENCY walks 7 metres towards south and reaches point ‘M’. From point ‘M’ he walks 6 metres towards west and reaches Point ‘O’. From point ‘O’ he walks 7 metres towards north and reaches point ‘P’. From point ‘P’ he walks towards west and reaches point ‘B’. The distance between points A and B is 8 metres. 21. How many students scored more than B in a class of 25 students? I. More than four but less than ten students scored more than that of B. B’s rank is an odd number. Seventeen students scored less than D. II. The rank of C is 16th from the last. B got more marks than C. Only two students are there between B and C. 22. Five persons – A, B, C, D and E – are sitting around a circular table. Some of them are facing towards centre while others are facing outside. Who is sitting second to the left of A? I. B is facing outside. C is to the immediate left of B. D is second to the right of C. A is to the immediate left of E. II. D is to the immediate right of A. Both D and A are facing towards the centre. D and B are immediate neighbours of each other. Directions (23–27) : Each of the questions below consists of a question and two statements numbered I and II are given below it. You have to decide whether the data provided in the statements are sufficient to answer the question. Read both the statements and — (SBI Probationary Officer Online Exam, 28.06.2014)

    Give answer (1) if the data in Statement I alone are sufficient to answer the question, while the data in Statement II alone are not sufficient to answer the question. Give answer (2) if the data in Statement II alone are sufficient to answer the question, while the data in Statement I alone are not sufficient to answer the question. Give answer (3) if the data in Statement I alone or in Statement II alone are sufficient to answer the question.

    Give answer (4) if the data in both the Statements I and II together are not sufficient to answer the question. Give answer (5) if the data in both the Statements I and II together are necessary to answer the question. 23. Who is the lightest amongst A, B, C, D and E I. D is heavier than C and lighter than A. II. B is heavier only than E. 24. On which date of the month was Rajiv born ? I. Rajiv’s mother correctly remembers that he was born after 12th and before 18th of May. II. Rajiv’s father correctly remembers that he was born before 23rd and after 16th of May. 25. How many brothers does Sonal have ? I. Nisha, the mother of Sonal’s brother has only two children. II. Sonal’s brother has only one sister. 26. Town M is towards which direction of Town L ? I. Driving 50 Km in a straight line from Town M leads to Town L. II. Towns M and L are located between towns K and P. 27. How is ‘late’ coded in the language ? I. ‘came late to office’ is coded as ‘so ti ly ja’ and late in the night’ is coded as ‘fo pa ti me.’ II. ‘it was late summer’ is coded as ‘ru ki ne ti’ and ‘reached two hours late’ is coded as ‘ti le di co.’ Directions (28-31) % Each of the questions below consists of a question and two statements numbered I and II given below it. You have to decide whether the data provided in the statements are sufficient to answer the question. Read both the statements and — (SBI Management Executive Exam, 19.09.2014)

    Give answer (1) if the data in Statement I alone are sufficient to answer the question, while the data in Statement II alone are not sufficient to answer the question.

    BPRE–912

    Give answer (2) if the data in Statement II alone are sufficient to answer the question, while the data in Statement I alone are not sufficient to answer the question. Give answer (3) if the data either in Statement I alone or in Statement II alone are sufficient to answer the question. Give answer (4) if the data even in both Statements I and II together are not sufficient to answer the question. Give answer (5) if the data in both Statements I and II together are necessary to answer the question. 28. Ten persons are sitting in two parallel rows containing five persons each, in such a way that there is an equal distance between adjacent persons. In Row1, P, Q, R, S and T are seated and all of them are facing south. In Row-2, A, B, C, D, and E are seated and all of them are facing north. Therefore, in the given siting arrangement each member seated in a row faces another member of the other row. Who among them is sitting second to the right of Q and is facing E \ I. C is not an immediate neighbour of E. S is not facing C. B is sitting second to the left of D. II. Only one person is sitting between A and E. Q faces A. Two persons are sitting between T and P. P is an immediate neighbour of both Q and R. 29. A building has six floors, numbered one to six in such a manner that the ground floor is numbered one, the floor above it is numbered two and so on such that the topmost floor is numbered six. One of the six persons, A, B, C, D, E and F , lives on each floor, but not necessarily in the same order. Who among them lives immediately above the floor of A ? I. B lives on three numbered floor. Two persons live between B and A . C lives immediately above the floor of F. E does not live immediately above or below the floor of A. II. D lives on floor number two. There are three persons between the floors of D and A. C does

    DATA SUFFICIENCY

    30.

    I.

    II.

    31. I.

    II.

    32.

    I.

    II.

    not live immediately above or below the floor of A. Five persons -P, Q, R, S and T are sitting around a circular table facing the centre but not necessarily in the same order. Who among them is sitting third to the right of Q? P is sitting second to the right of S. T is sitting to the immediate left of P. Q is an immediate neighbour of S. R is second to the right of Q. T is not an immediate neighbour of R. P is sitting to the immediate left of T. What is the position of C with respect to A? A person walks 5 metres towards north from point A. Then he walks 5 metres towards east. Again he walks 5 metres towards north to reach point B. From point B he walks 5 metres towards east and reaches point C. A person walks 4 metres towards east from point A. Then he walks 9 metres towards north to reach point B. Again he walks 6 metres towards east and reaches point D. C is 7 metres west of D. This question consists of a statement and two conclusions numbered I and II given below it. A conclusion is something which can be directly deduced from the given information in light of the given facts. You have to decide which of the given conclusions logically follows from the given statement and select the appropriate answer. Statement : At present 32% of our villages receive 24 hours electricity and in the next two years, the state government would strive to add 16% more villages to this list. At present 60% villages in the state have no access to electricity at all. Even after a couple of years, more than half of the villages lack access to 24 hours electricity. (1) Only II can be concluded from the given information. (2) Only I can be concluded from the given information. (3) Neither I nor II can be concluded from the given information.

    (4) Both I and II can be concluded from the given information (5) Either I or II can be concluded from the given information. (SBI Associates PO Online Exam, 29.11.2014)

    Directions (33–37) : Each question below consists of a question and two statements numbered I and II given below it. You have to decide whether the data given in the statements are sufficient to answer the question. Read both the statements and choose the most appropriate option. (SBI Associates PO Online Exam, 29.11.2014)

    33. Among V, W, X. Y and Z, seated in a straight line, facing north, who sits exactly in the middle of the line ? I. W sits third to the right of Z. X is an immediate neighbour of W. V sits to immediate left of Y. II. X sits at one of the extreme ends of the line. Only two people sit between X and V. Y sits to immediate right of V. Y does not sit at any of the extreme ends of the line. (1) The data either in statement I alone or statement II alone are sufficient to answer the question. (2) The data even in both statements I and II together are not sufficient to answer the question. (3) The data in statement II alone are sufficient to answer the question while the data in statement I alone are not sufficient to answer the question. (4) The data in both the statements I and II together are necessary to answer the question. (5) The data in statement I alone are sufficient to answer the question while the data in statement II alone are not sufficient to answer the question. 34. Out of P, Q, R, S and T, does R earn more than Rs. 40,000/- ? I. S earns more than only R and T. P earns the maximum. T earns Rs. 50,000/-.

    BPRE–913

    II. Q earns less than only P. R earns less than Q. Q earns Rs. 80,000/-. (1) The data either in statement I alone or statement II alone are sufficient to answer the question. (2) The data even in both statements I and II together are not sufficient to answer the question. (3) The data in statement II alone are sufficient to answer the question while the data in statement I alone are not sufficient to answer the question. (4) The data in both the statements I and II together are necessary to answer the question. (5) The data in statement I alone are sufficient to answer the question while the data in statement II alone are not sufficient to answer the question. 35. Among the six friends A, B, C D, E and F, sitting around a circular table facing the centre, who sits to immediate left of C ? I. A sits second to left of C and third to right of F. B is an immediate neighbour of E. II. E is an immediate neighbour of both B and F. Only three people sit between B and D. F is an immediate neighbour of C. (1) The data either in statement I alone or statement II alone are sufficient to answer the question. (2) The data even in both statements I and II together are not sufficient to answer the question. (3) The data in statement II alone are sufficient to answer the question while the data in statement I alone are not sufficient to answer the question. (4) The data in both the statements I and II together are necessary to answer the question. (5) The data in statement I alone are sufficient to answer the question while the data in statement II alone are not sufficient to answer the question.

    DATA SUFFICIENCY 36. How is ‘success’ written in a certain code language ? I. In that code language ‘success is necessary evil’ is written as ‘8145’ and ‘evil is amongst us’ is written as ‘3874’. II. In that code language ‘marketing is magic mantra’ is written as ‘4629’ and ‘success mantra is marketing’ is written as ‘2564’ (1) The data either in statement I alone or statement II alone are sufficient to answer the question. (2) The data even in both statements I and II together are not sufficient to answer the question. (3) The data in statement II alone are sufficient to answer the question while the data in statement I alone are not sufficient to answer the question. (4) The data in both the statements I and II together are necessary to answer the question. (5) The data in statement I alone are sufficient to answer the question while the data in statement II alone are not sufficient to answer the question. 37. A five storey building (consisting of a ground floor and four floors on top of it such that the ground floor is numbered 1, the floor above it, numbered 2 and so on till the topmost floor is numbered 5) houses different people viz. H, I, J, K and L. Who lives on the lowermost floor (i.e. floor No. 1) ? I. J lives on floor No. 3. K lives immediately below L. K lives on an even numbered floor. II. Only two people live between L and H. L lives above H. J lives immediately below K. J lives on an odd numbered floor. (1) The data either in statement I alone or statement II alone are sufficient to answer the question. (2) The data even in both statements I and II together are not sufficient to answer the question.

    (3) The data in statement II alone are sufficient to answer the question while the data in statement I alone are not sufficient to answer the question. (4) The data in both the statements I and II together are necessary to answer the question. (5) The data in statement I alone are sufficient to answer the question while the data in statement II alone are not sufficient to answer the question. Directions (38–42) : Each question below consists of a question and two statements numbered I and II given below it. You have to decide whether the data given in the statements are sufficient to answer the question. Read both the statements and choose the most appropriate answer. (SBI Associates PO Online Exam, 30.11.2014)

    38. Are all the five friends F, G, H, I and J sitting in a straight line facing north ? I. F sits at one of the extreme ends of the line. Only two people sit between F and H. J sits to immediate left of H. G and J are immediate neighbours of each other. II. J faces north and sits at one of the extreme ends of the line. Only two people sit beween J and I. H sits second to the right of I. H sits to immediate left of G. (1) The data in Statement II alone are sufficient to answer the question, while the data in Statement I alone are not sufficient to answer the question. (2) The data in Statement I alone are sufficient to answer the question, while the data in Statement II alone are not sufficient to answer the question. (3) The data either in Statement I alone or Statement II alone are sufficient to answer the question (4) The data even in both the Statements I and II together are not sufficient to answer the question.

    BPRE–914

    (5) The data in both the Statements I and II together are necessary to answer the question. 39. Are all the five friends A, B, C, D and E who are sitting around the circular table facing the centre ? I. D sits to immediate right of E. E faces the centre. C sits second to the left of the D. A is not an immediate neighbour of B. II. B sits to immediate right of C. A sits to immediate left of E. E is not an immediate neighbour of B. E faces the centre. D is not an immediate neighbour of C. (1) The data in Statement II alone are sufficient to answer the question, while the data in Statement I alone are not sufficient to answer the question. (2) The data in Statement I alone are sufficient to answer the question, while the data in Statement II alone are not sufficient to answer the question. (3) The data either in Statement I alone or Statement II alone are sufficient to answer the question (4) The data even in both the Statements I and II together are not sufficient to answer the question. (5) The data in both the Statements I and II together are necessary to answer the question. 40. Is C the mother of B ? I. P is the father of A and B. R has only the one brother P. C is sister-in-law of R. R is unmarried. F is mother of R. F has only two children. II. H has only two children P and R. P is the father of A. B is the only brother of A. R is unmarried. H is the father-in-law of C. (1) The data in Statement II alone are sufficient to answer the question, while the data in Statement I alone are not sufficient to answer the question.

    DATA SUFFICIENCY (2) The data in Statement I alone are sufficient to answer the question, while the data in Statement II alone are not sufficient to answer the question. (3) The data either in Statement I alone or Statement II alone are sufficient to answer the question (4) The data even in both the Statements I and II together are not sufficient to answer the question. (5) The data in both the Statements I and II together are necessary to answer the question. 41. Five movies P, Q, R, S and T are scheduled to be screened from Monday to Friday of the same week. Which movie will be screened on Friday ? I. Movie P will be screened on Tuesday. Only two movies will be screened between movies R and S. R will be screened before S. II. Movie Q will be screened on Wednesday. Only one movie will be screened between movies Q and R. S will be screened immediately before movie T. (1) The data in Statement II alone are sufficient to answer the question, while the data in Statement I alone are not sufficient to answer the question. (2) The data in Statement I alone are sufficient to answer the question, while the data in Statement II alone are not sufficient to answer the question. (3) The data either in Statement I alone or Statement II alone are sufficient to answer the question (4) The data even in both the Statements I and II together are not sufficient to answer the question. (5) The data in both the Statements I and II together are necessary to answer the question. 42. How far is point R from point P? I. A person starts from Point P, walks 15 metre to the

    II.

    (1)

    (2)

    (3)

    (4)

    (5)

    south, takes right turn and walks 4 metres. He then takes left turn and walks 7 metres. He takes left turn again, walks for 4 metre and reaches point Q. If the person takes left turn and walks 7 metre, he will rach point R. A person starts from Point P, walks 14 metre towards the east, takes a left turn and walks 3 metre. He then takes a left turn again and walks for 14 metre to reach Point N. If he takes a right turn from Point N and walks 5 metre, he will be 27 metre away from the Point R. The data in Statement II alone are sufficient to answer the question, while the data in Statement I alone are not sufficient to answer the question. The data in Statement I alone are sufficient to answer the question, while the data in Statement II alone are not sufficient to answer the question. The data either in Statement I alone or Statement II alone are sufficient to answer the question The data even in both the Statements I and II together are not sufficient to answer the question. The data in both the Statements I and II together are necessary to answer the question.

    Directions (43-46) % Each of the questions below consists of a question and two statements numbered I and II given below it. You have to decide whether the data provided in the statements are sufficient to answer the question. Read both the statements and — Give answer (1) if the data in Statement I alone are sufficient to answer the question, while the data in Statement II alone are not sufficient to answer the question. Give answer (2) if the data in Statement II alone are sufficient to answer the question, while the data in Statement I alone are not sufficient to answer the question.

    BPRE–915

    Give answer (3) if the data either in Statement I alone or in Statement II alone are sufficient to answer the question. Give answer (4) if the data even in both Statements I and II together are not sufficient to answer the question. Give answer (5) if the data in both Statements I and II together are necessary to answer the question. 43. Ten persons are sitting in two parallel rows containing five persons each, in such a way that there is an equal distance between adjacent persons. In Row1, P, Q, R, S and T are seated and all of them are facing south. In Row-2, A, B, C, D, and E are seated and all of them are facing north. Therefore, in the given siting arrangement each member seated in a row faces another member of the other row. Who among them is sitting second to the right of Q and is facing E \ I. C is not an immediate neighbour of E. S is not facing C. B is sitting second to the left of D. II. Only one person is sitting between A and E. Q faces A. Two persons are sitting between T and P. P is an immediate neighbour of both Q and R. 44. A building has six floors, numbered one to six in such a manner that the ground floor is numbered one, the floor above it is numbered two and so on such that the topmost floor is numbered six. One of the six persons, A, B, C, D, E and F, lives on each floor, but not necessarily in the same order. Who among them lives immediately above the floor of A ? I. B lives on three numbered floor. Two persons live between B and A . C lives immediately above the floor of F. E does not live immediately above or below the floor of A. II. D lives on floor number two. There are three persons between the floors of D and A. C does not live immediately above or below the floor of A.

    DATA SUFFICIENCY 45. Five persons -P, Q, R, S and T are sitting around a circular table facing the centre but not necessarily in the same order. Who among them is sitting third to the right of Q? I. P is sitting second to the right of S. T is sitting to the immediate left of P. Q is an immediate neighbour of S. II. R is second to the right of Q. T is not an immediate neighbour of R. P is sitting to the immediate left of T. 46. What is the position of C with respect to A? I. A person walks 5 metres towards north from point A. Then he walks 5 metres towards east. Again he walks 5 metres towards north to reach point B. From point B he walks 5 metres towards east and reaches point C. II. A person walks 4 metres towards east from point A. Then he walks 9 metres towards north to reach point B. Again he walks 6 metres towards east and reaches point D. C is 7 metres west of D. Directions (47–51) : Each question below consists of a question and two statements numbered I and II given below it. You have to decide whether the data given in the statements are sufficient to answer the question. Read both the statements and choose the most appropriate answer. (SBI Associates PO Online Exam. 30.11.2014)

    47. Are all the five friends F, G, H, I and J sitting in a straight line facing north ? I. F sits at one of the extreme ends of the line. Only two people sit between F and H. J sits to immediate left of H. G and J are immediate neighbours of each other. II. J faces north and sits at one of the extreme ends of the line. Only two people sit beween J and I. H sits second to the right of I. H sits to immediate left of G. (1) The data in Statement II alone are sufficient to answer the question, while the data in Statement I alone are not sufficient to answer the question.

    (2) The data in Statement I alone are sufficient to answer the question, while the data in Statement II alone are not sufficient to answer the question. (3) The data either in Statement I alone or Statement II alone are sufficient to answer the question (4) The data even in both the Statements I and II together are not sufficient to answer the question. (5) The data in both the Statements I and II together are necessary to answer the question. 48. Are all the five friends A, B, C, D and E who are sitting around the circular table facing the centre ? I. D sits to immediate right of E. E faces the centre. C sits second to the left of the D. A is not an immediate neighbour of B. II. B sits to immediate right of C. A sits to immediate left of E. E is not an immediate neighbour of B. E faces the centre. D is not an immediate neighbour of C. (1) The data in Statement II alone are sufficient to answer the question, while the data in Statement I alone are not sufficient to answer the question. (2) The data in Statement I alone are sufficient to answer the question, while the data in Statement II alone are not sufficient to answer the question. (3) The data either in Statement I alone or Statement II alone are sufficient to answer the question (4) The data even in both the Statements I and II together are not sufficient to answer the question. (5) The data in both the Statements I and II together are necessary to answer the question.

    BPRE–916

    49. Is C the mother of B ? I. P is the father of A and B. R has only the one brother P. C is sister-in-law of R. R is unmarried. F is mother of R. F has only two children. II. H has only two children P and R. P is the father of A. B is the only brother of A. R is unmarried. H is the father-in-law of C. (1) The data in Statement II alone are sufficient to answer the question, while the data in Statement I alone are not sufficient to answer the question. (2) The data in Statement I alone are sufficient to answer the question, while the data in Statement II alone are not sufficient to answer the question. (3) The data either in Statement I alone or Statement II alone are sufficient to answer the question (4) The data even in both the Statements I and II together are not sufficient to answer the question. (5) The data in both the Statements I and II together are necessary to answer the question. 50. Five movies P, Q, R, S and T are scheduled to be screened from Monday to Friday of the same week. Which movie will be screened on Friday ? I. Movie P will be screened on Tuesday. Only two movies will be screened between movies R and S. R will be screened before S. II. Movie Q will be screened on Wednesday. Only one movie will be screened between movies Q and R. S will be screened immediately before movie T. (1) The data in Statement II alone are sufficient to answer the question, while the data in Statement I alone are not sufficient to answer the question. (2) The data in Statement I alone are sufficient to answer the question, while the data in

    DATA SUFFICIENCY Statement II alone are not sufficient to answer the question. (3) The data either in Statement I alone or Statement II alone are sufficient to answer the question (4) The data even in both the Statements I and II together are not sufficient to answer the question. (5) The data in both the Statements I and II together are necessary to answer the question. 51. How far is point R from point P ? I. A person starts from Point P, walks 15 metre to the south, takes right turn and walks 4 metres. He then takes left turn and walks 7 metres. He takes left turn again, walks for 4 metre and reaches point Q. If the person takes left turn and walks 7 metre, he will rach point R. II. A person starts from Point P, walks 14 metre towards the east, takes a left turn and walks 3 metre. He then takes a left turn again and walks for 14 metre to reach Point N. If he takes a right turn from Point N and walks 5 metre, he will be 27 metre away from the Point R. (1) The data in Statement II alone are sufficient to answer the question, while the data in Statement I alone are not sufficient to answer the question. (2) The data in Statement I alone are sufficient to answer the question, while the data in Statement II alone are not sufficient to answer the question. (3) The data either in Statement I alone or Statement II alone are sufficient to answer the question (4) The data even in both the Statements I and II together are not sufficient to answer the question. (5) The data in both the Statements I and II together are necessary to answer the question.

    Directions (52–56) : Each of the following questions, consists of a question and two statements numbered I and II given below it. You have to decide whether the data given in the statements are sufficient to answer the question. (SBI PO Online Main Exam, 31.07.2016)

    Read both the statements and mark appropriate answer : Mark answer (1) if the data in statement I alone are sufficient to answer the question, while the data in statement II alone are not sufficient to answer the question. Mark answer (2) if the data in statement II alone are sufficient to answer the question, while the data in statement I alone are not sufficient to answer the question. Mark answer (3) if the data either in statement I alone or in statement II alone are sufficient to answer the question. Mark answer (4) if the data in both statements I and II together are not sufficient to answer the question. Mark answer (5) if the data in both the statements I and II together are necessary to answer the question. 52. How is T related to V? I. T is wife of W. P is grandson of W. The father of V is husband of only sister of P. II. P, Q, R, S, T, U, V and W are eight members of a family in which T, U, S and V are females while others are male members. 53. What are the given two digits between 1 to 9? I. The number obtained by multiplying both digits is the same as obtained by subtracting the second digit and one more than first digit from the square of second digit? II. The number obtained by adding both digits is more than twice of the number obtained by subtracting one digit from the other. 54. Amongst six persons namely, P, Q, R, S, T and U standing around a circular path, facing the centre, how many female members are there? I. P is standing just opposite to Q. R is a male member and he is an immediate neighbour of U, a female member. S does not stand opposite to T and one of them is a female member.

    BPRE–917

    II. The two persons standing opposite to each other are not of same sex. 55. How long and in which direction Shubham’s friend has to cover to meet Shubham at a particular location? I. Shubham’s friend moves 4 km. towards east to reach his office. Then he moves 2.5 km towards south, turns left and moves 3.5 km. Again he turns left and moves 5.5 km. to reach his hotel. Now he proceeds towards destination. II. From hotel, Shubham’s friend moves with a speed of 5 km per hour and after covering exactly 3.5 km., he gets a call from his office. He decides to wait at that place to meet Shubham. 56. Who amongst P, Q, R, S, T, U and V will be in the middle if all of them are arranged in ascending order of heights? I. P is taller than S but shorter than U. U is shorter than both R and T. V is taller than R but shorter than T. Q is older than R and S but younger than P. II. Q is the shortest among them. On 18th birthday of T, R was 10 days younger than his 22nd birthday. 57. A person can purchage three articles in Rs 49. What is the price of costliest article? Statement I : The cost price of two articles each is Re. 1 less than the cost price of costliest article. Statement II : The cost price of two articles is same. Statement III : The cost price of costliest articles is 6.25% more than the cost price of cheapest article. (1) Data in either statement I alone or statements II and III together are sufficient to answer the question (2) Data in only statement III is sufficient to answer the question (3) Data in only statement I and II together are sufficient to answer the question (4) Data in only statement I and III together are sufficient to answer the question (5) None of these (SBI PO Phase-II (Main) Exam 05.08.2018)

    DATA SUFFICIENCY 58. Shatabdi Express leaves Patna at 8:00 am for Delhi. At what time will it reach Delhi? Statement I : For the first 100 km it travels at the speed of 250 km per hour and maintains the same speed during the entire journey. Statement II : It has 5 stoppages between Delhi and Patna Statement III : Before every stoppage, it covers a same distance of 240 km. (1) Data in either statement III alone or statments I and II together are sufficient to answer the question (2) Data in only statment III is sufficient to answer the question (3) Data in only statements I and II together are sufficient to answer the question (4) Data in all statements I, II and III together are sufficient to answer the question (5) None of these (SBI PO Phase-II (Main) Exam 05.08.2018)

    59. Seven persons A, B, C, D, E, F and G are sittine around a circular table. Are all the persons facing towards the centre of the table? I. A sits third to the right of C, who sits second to the left of D. Only two persons sit between G and A. B is not the immediate neighbour of C. C faces inside. II. D sits third to the left of B and second to the right of E. C sits third to the right of F, who is not the immediate neighbour of E. III. F sits third to the right of B. A and G faces same direction. F sits second to the left of A. E sits third to the right of F and to the immediate left of B. E and B faces same direction. B is not an immediate neighbour of A. (1) Data in statements I and II together are sufficient to answer the question while the data in statement III are not required to answer the question.

    (2) Data in statements I and III together are sufficient to answer the question while the data in statement II are not required to answer the question. (3) Data in statements II and III are sufficient to answer the question, while the data in statement I are not required to answer the question. (4) Data in all three statements I, II and III together are necessary to answer the question. (5) Data in all the statements, I, Il and III even together are not sufficient to answer the question. (SBI PO Phase-II (Main) Exam 05.08.2018)

    60. A metal block of density ‘D’ and mass ‘M’, in the form of a cuboid, is beaten into a thin square sheet of thickness ‘t’, and rolled to form a cylinder of the same thickness. Find the inner radius of the cylinder— Statement I : Cuboid has dimensions 10 cm × 5 cm × 12 cm Statement II : Thickness ‘t’ = 1.5 cm Statement III : Mass of block, M = 216kg (1) Data in either statement III alone or statements I and II together are sufficient to answer the question. (2) Data in only statement III is sufficient to answer the question. (3) Data in statement I and statement II together are sufficient to answer the question. (4) Data in all statements I, II and III together are sufficient to answer the question. (5) None of the given statements is sufficient to answer the question. (SBI PO Phase-II (Main) Exam 05.08.2018)

    Directions (61–63) : Each of the questions below consists of a question and two statements numbered I and II given below it. You have to decide whether the data provided in the state-

    BPRE–918

    ments are sufficient to answer the question. Read both the statements and give answer. Give answer (1) if the data in statement I alone are sufficient to answer the question while data in statement II alone are not sufficient to answer the question. Give answer (2) if the data in statement II alone are sufficient to answer the question while the data in statement I alone are not sufficient to answer the question. Give answer (3) if the data either in statement I alone or statement II alone are sufficient to answer the question. Give answer (4) if the data even in both the statements I and II together are not sufficient to answer the question. Give answer (5) if the data in both statements I and II together are necessary to answer the question. SBI PO (Prelim Exam), 08.07.2018 (Shift-I))

    61. Statement : Six boys J, K, L, M, N, O are there in a classroom each of them is of different heights. Who among them is the tallest? I. M is taller than N and K. J is taller than M but not as tall as O. L is taller than K. II. M is taller than only three boys. J is taller than K. 62. Statement : Six persons R, S, T, U, V, W lives on different floors of a six storey building such as ground floor is numbered as 1 and above it 2 and so on … upto top floor numbered as 6. How many persons live between R and T? I. T lives on an even numbered floor but not on top floor. Only two persons live between W and T. R lives below W. II. Four persons live between S and U. No one lives between S and T. V lives immediately above R. 63. Statement : Six persons A, B, C, D, E, F are sitting in row. All of them are facing north direction. Who among them sits second from the right end? I. B sits at an extreme end of the row. A sits second to the right of

    DATA SUFFICIENCY B. Only one person sits between A and C. E sits to the immediate right of C. II. E sits third to the right of D. Only one person sits between E and A. F sits to the right to E. C is an immediate neighbour of E. 64. In the given coding system ‘Now they live for’ is coded as ‘gn mu sy fd’ and ‘go now run for’ is coded as “gn sy mo lt”. Which of the following statements is/are required to code ‘go there now’? I. ‘Give it for’ is coded as ‘la sa sy’. II. ‘Go there get ready’ is coded as ‘ht mo ga sx’ III. ‘Now there fall’ is coded as ‘za ga gn’ (1) Only I (2) Both II and III (3) Only II (4) Both I and II (5) Either I or II SBI PO (Prelim Exam), 08.07.2018 (Shift-I))

    RBI GRADE–B/ NABARD GRADE–A OFFICER EXAMS Directions (1–5) : Each of the questions below consists of a question and two statements numbered I and II given below it. You have to decide whether the data provided in the statements are sufficient to answer the question. Read both the statements and (RBI Grade-B Officer’s Exam. 18.12.2011)

    Give answer (1) if the data in Statement I alone are sufficient to answer the question, while the data in Statement II alone are not sufficient to answer the question. Give answer (2) if the data in Statement II alone are sufficient to answer the question, while the data in Statement I alone are not sufficient to answer the question. Give answer (3) if the data in either Statement I alone or in Statement II alone are sufficient to answer the question. Give answer (4) if the data in neither Statement I nor II are sufficient to answer the question. Give answer (5) if the data in both the Statements I and II together are necessary to answer the question.

    Among P, Q, R, S and T, sitting in a straight line, facing North, who sits exactly in the middle of the line ? I. P sits third to left of S. T is an immediate neighbour of P as well as R. II. T sits second to left of S. Q is not an immediate neighbour of either T or S. 2. Among five friends A, B, C, D and E, sitting around a circular table, facing the centre, who sits to the immediate right of A ? I. E sits third to right of D. A is not an immediate neighbour of D. II. C sits second to left of B. A is not an immediate neighbour of C. 3. Who amongst P, Q, R, S and T is the tallest ? I. P is taller than S and T but shorter than R. Q is taller than S. II. T is taller than S. P is not the tallest. 4. Is ‘EAST’ the word formed after performing the following operations on a word containing these four letters ? I. There is only one letter between A and T. E is to the left of A. II. The word does not begin with T. There is only one letter between E and S. T is not an immediate neighbour of E. 5. Is C the grandmother of M ? I. C is the mother of D. D is the brother of M’s father. II. E is the mother of S. S is the sister of M. F, the aunt of S is the only daughter of C. Directions (6-8) : Each of the questions below consists of a question and two statements numbered I and II given below it. You have to decide whether the data provided in the statements are sufficient to answer the question. Read both the statements and — 1.

    (RBI Officer Grade ‘B’ Phase-I Exam, 03.08.2014)

    Give answer (1) if the data in Statement I alone are sufficient to answer the question, while the data in Statement II alone are not sufficient to answer the question.

    BPRE–919

    Give answer (2) if the data in Statement II alone are sufficient to answer the question, while the data in Statement I alone are not sufficient to answer the question. Give answer (3) if the data either in Statement I alone or in Statement II alone are sufficient to answer the question. Give answer (4) if the data even in both Statements I and II together are not sufficient to answer the question. Give answer (5) if the data in both Statements I and II together are necessary to answer the question. 6. Seven different exams – P, Q, R, S, T, U and V – are conducted on different days of the same week, i.e., from Monday to Sunday. Which exam was conducted on Wednesday? I. Two exams were conducted after exam P. Exam Q took place immediately after exam U. Exam Q was not conducted on Sunday or on Thursday. Exam T was not conducted immediately after exam P. II. Three exams were conducted before exam R. Exam S was conducted after exam P but before exam V. Exam Q was conducted after exam U and before exam T. Exam P was not conducted on Monday. 7. What is the position O among 20 persons standing in a straight line ? I. There are five persons between Y and L. L is at one of the extreme ends. T is sixth to the left of Y. There are three persons between O and T. II. R is at one of the extreme ends of the line. There are two persons between O and J. There are nine persons between R and U. J is fourth to the left of U. 8. How is ‘X’ related to ‘R’ ? I. The brother of ‘X’ is married to ‘D’. The father-in-law of D is the grandfather of R. R is the only daughter of D. X is brother-inlaw of D. II. R is the only daughter of P. D is the daughter-in-law of M. P and X are two sons of M. X is not married to D.

    DATA SUFFICIENCY Directions (9–13) : Each of the following questions, consists of a question and two statements numbered I and II given below it. You have to decide whether the data given in the statements are sufficient to answer the question. Read both the statements and mark the appropriate answer. (RBI Officer Grade ‘B’ Phase-I Exam. 21.11.2015)

    Mark answer (1) If the data in statement I alone are sufficient to answer the question, while the data in statement II alone are not sufficient to answer the question. Mark answer (2) If the data in statement II alone are sufficient to answer the question, while the data in statement I alone are not sufficient to answer the question. Mark answer (3) If the data either in statement I alone or in statement II alone are sufficient to answer the question. Mark answer (4) If the data in both statements I and II together are not sufficient to answer the question. Mark answer (5) If the data in both the statements I and II together are necessary to answer the question. 9. How many people are standing between H and U in a straight line of 15 people (Note : All are standing in a straight line facing north.)? I. M stands fifth from the left end of the line. H and K are immediate neighbours of M. Only two people stand between M and U. II. B stands third from the right end of the line. Only six people stand between K and B. Only one person stands between H and K. K stands exactly between H and U. 10. How is B related S? I. N is married to T. B is the sister of N. T has only one daughter K. K is the granddaughter of S. II. N is the daughter of S. N is married to T.T is the only child of Z. B is the sister-in-law of T. 11. Among people A, B, C, D, E and F, each having a different weight, who is the second heaviest? I. C is heavier than only two people. A is heavier than F but lighter than B. D is not the heaviest. II. F is lighter than only two people. C is lighter than F but heavier than D. A is lighter than only B.

    12. How many people are standing in a straight line (Note: All are facing north)? I. U stands third from the left end of the line. U is an immediate neighbour of P and W. Only one person stands between W and T. Only two people stand to the right of W. II. S stands at extreme left end of the line. T stands to the extreme right end of the line. Only one person stands between S and U. Only one person stands between T and W. 13. How far is point M from point K? I. Point D is 5m to the south of Point P. Point M is 8m to the west of point D. Point S is 2.5m to the north of point M. Point O is 10m to the east of Point S. Point K is 2.5m to the south of point O. II. Point K is 10m to the east of point M. Point U is 8m to the west of Point M. Point D is to the east of M. Point M is the midpoint of the lines formed by joining points U and D. Directions (14–15) : Each of the questions below consists of a question and two statements numbered I and II given below it. You have to decide whether the data provided in the statements are sufficient to answer the question. Read both the statements and (Nabard Officer Grade ‘A’ Online Exam. 01.03.2015)

    Give answer (1) if the data in statement I alone are sufficient to answer the question, while the data in statement II alone are not sufficient to answer the question. Give answer (2) if the data in statement II alone are sufficient to answer the question, while the data in statement I alone are not sufficient to answer the question. Give answer (3) if the data either in statement I alone or in statement II alone are sufficient to answer the question. Give answer (4) if the data even in both statements I and II together are not sufficient to answer the question. Give answer (5) if the data in both statements I and II together are necessary to answer the question. 14. How is ‘plan’ coded in the given code language ? (Note : All the codes are two–letter codes only)

    BPRE–920

    I. In the given code language, ‘plan it now’ is coded as ‘ta na se’ and ‘a plan now’ is coded as ‘na ta ra’. II. In the given code language, ‘plan is good’ is coded as ‘zi ju na’ and ‘good to plan’ is coded as ‘fe ju na’. 15. Among friends, P, Q, R, S and T each having a different height, who is the shortest ? I. P is shorter only than three people. Q is taller than S but shorter than T. II. Q is shorter than only one person. P is shorter than T. Directions (16–18) : Each of the questions given below consists of a question and two statements numbered I and II given below it. You have to decide whether the data provided in the statements sufficient to answer the question. Read both the statements and (NABARD Assistant Manager Exam, 15.15.2016)

    Give answer (1) if the data in statement I alone are sufficient to answer the question, while the data in statement II alone are not sufficient to answer the question. Give answer (2) if the data in statement II alone are sufficient to answer the question, while the data in statement I alone are not sufficient to answer the question. Give answer (3) if the data in statement I alone or in statement II alone are sufficient to answer the question. Give answer (4) if the data in both the statements I and II are not sufficient to answer the question. Give answer (5) if the data in both the statements I and II together are necessary to answer the question. 16. How many persons are sitting in a circle, where all the persons are facing the centre? I. K sits third to the right of L. M is an immediate neighbour of K. Only three persons sit between L and K. II. N sits third to the left of M. Only one person sits between N and K. K is an immediate neighbour of M. 17. In a six storey building (consisting of floors numbered 1 to 6, wherein the topmost floor is num-

    DATA SUFFICIENCY bered 6 and the ground floor is numbered 1) each of the six friends namely A, B, C, D, E and F lives on a different floor. On which floor number does E live? I. A lives on floor numbered 3. Only one person lives between A and F. E lives on any odd numbered floor below F. II. B lives on floor numbered 4. A lives on one of the floors below B but not on floor numbered 1. D lives on any even numbered floor above A. C lives immediately above E. 18. How is ‘buyer’ definitely coded in the given language? I. In the given code language, ‘help find a buyer’ is coded as ‘ni ka po sn’ and ‘a small help given’ is coded as ‘tk sn ru ni’. II. In the given code language ‘must find small buyer’ is coded as ‘ka hp tk po’ and ‘find a help soon’ is coded as ‘mj ni sn ka’ Directions (19–23) : Each of the questions given below consists of a question and two statements numbered I and II given below it. You have to decide whether the data provided in the statements are sufficient to answer the question. Read both the statements and (RBI Officer Grade ‘B’ Phase-I Exam, 04.09.2016 (Shift-I))

    Give answer (1) if the data in statement I alone are sufficient to answer the question, while the data in statement II alone are not sufficient to answer the question. Give answer (2) if the data in statement II alone are sufficient to answer the question, while the data in statement I alone are not sufficient to answer the question. Give answer (3) if the data in statement I alone or in statement II alone are sufficient to answer the question. Give answer (4) if the data in both the statements I and II are not sufficient to answer the question. Give answer (5) if the data in both the statements I and II together are necessary to answer the question. 19. In a building, the ground floor is numbered one, first floor is numbered two and so on till the topmost floor is numbered five. Amongst five people- M, N, O, P

    and Q – each living on a different floor, but not necessarily in the same order, on which floor does Q live? I. O lives on an odd numbered floor. M lives immediately below O. Only two people live between M and P. N lives neither immediately below M nor immediately below P. II. N lives on an even numbered floor. Only two people live between N and O. Only one person lives between O and Q. 20. Among six people A, B, C, D, E and F, each having a different height, who is the second shortest? I. Only two people are taller than A. E is taller than both B and C. F is shorter than E. F is taller than C. II. Only two people are shorter than D. A is taller than D but shorter than E. F is neither the tallest nor the shortest. B is taller than C. 21. How many people are standing between A and D. (Note: All are standing in a straight line facing north)? I. K stands second from the left end of the line. Only four people stand between K and T. Y is an immediate neighbour of T. A stands second to the right of Y. As many people stand between K and D as between A and D. II. A stands second from the right end of the line. Z stands third from the left end of the line. D stands exactly in the centre of the line. As many people stand between A and T as between D and Z. 22. How far and in which direction is Point M from Point S? I. Point E is 2m to the east of Point S. Point B is 4m to the south of Point E. Point L is 10m to the east of Point B. Point L forms a midpoint of the vertical straight line of 8m formed by joining points Q and D. Point M is 5m to the west of Point Q. Point Q is to the East of Point E. II. Point M is 8m to the north of Point A. Point M forms the midpoint of the horizontal straight line formed by joining points Q and E. Point E is 8m to the west of Point Q. Point S is 4m to the west of point E.

    BPRE–921

    23. Amongst six people–P, Q, R, S, T, U– standing around a circle, some facing the centre while some facing outside (i.e. opposite to the centre) but not necessarily in the same order, what is the position of T with respect to U? I. P stands second to the right of R. R faces the centre. Q stands second to the left of P. Q is an immediate neighbour of both U and T. U and P face opposite directions (i.e. if U faces the centre then P faces outside and viceversa.). Only two people stand between P and T. II. Only two people stand between R and U. P stands to the immediate left of U. P faces outside. R is an immediate neighbour of T. Directions (24–28) : Each of the questions given below consists of a question and two statements numbered I and II given below it. You have to decide whether the data provided in the statements are sufficient to answer the question. Read both the statements and (RBI Officer Grade ‘B’ Phase-I Exam, 04.09.2016 (Shift-II))

    Give answer (1) if the data in statement I alone are sufficient to answer the question, while the data in statement II alone are not sufficient to answer the question. Give answer (2) if the data in statement II alone are sufficient to answer the question, while the data in statement I alone are not sufficient to answer the question. Give answer (3) if the data in statement I alone or in statement II alone are sufficient to answer the question. Give answer (4) if the data in both the statements I and II are not sufficient to answer the question. Give answer (5) if the data in both the statements I and II together are necessary to answer the question. 24. How many people are standing in a straight line (Note: All are standing in a straight line facing north.) ? I. D stands fifth from the left end of the line. Only one person stands between D and X. S stands fourth from the right end of the line. X stands exactly in the centre of the line.

    DATA SUFFICIENCY II. Y stands second from the left end of the line. R stands second to the right of Y. D is an immediate neighbour of R. Only four people stand between D and S. As many people stand to the right of S as to the left of R. 25. Among six people- M, N, O, P, Q and R standing around a circle, while some facing the centre, some facing outside (i.e. opposite the centre) but not necessarily in the same order, what is the position of O with respect to R ? I. Only two people stand between P and Q. Q is an immediate neighbour of both N and O. Q faces the centre. R stands second to the left of Q. R is not an immediate neighbour of N. II. O stands second to the right of P. P faces the centre. N stands second to the left of O. R is an immediate neighbour of P. N is an immediate neighbour of both Q and M. R and O face opposite directions (i.e. if R faces the centre then O faces outside and viceversa.) 26. In a building, the ground floor is numbered one; first floor is numbered two and so on till the topmost floor is numbered five. Amongst five people- A, B, C, D and E, each living on a different floor but not necessarily in the same order, on which floor does E live ? I. B lives on an even numbered floor. D lives immediately above B. Only one person lives between D and E. E does not live on the topmost floor. II. C lives on an odd numbered floor. A lives either immediately above or immediately below C. Only one person lives between A and B. E lives on one of the floors below B. 27. Among people S, T, U, V, W and X, each having a different weight, who is the second heaviest ? I. V is heavier than only one person. T is heavier than S but lighter than W. S is heavier than both U and X. U is heavier than V. II. S is heavier than only three people. V is heavier than X but lighter than U. U is lighter than both W and T. T is not the heaviest. 28. How far and in which direction is point D from point H ? I. Point V is the midpoint of the 10m horizontal straight line formed by

    joining points U and T. Point U is to the west of Point V. Point L is 8m to the north of Point U. Point D forms the midpoint of the straight line formed by joining Points L and U. Point H is 4m to the north of Point T. II. Point K is 5m to the east of Point L. Point M forms the midpoint of the 8m vertical straight line formed by joining points K and V. Point V is to the south of Point K. Point D is 4m to the south of Point L. Point H is 5m away from the Point M. Directions (29–31) : Each of the following questions, consists of a question and two statements numbered I and II given below it. You have to decide whether the data given in the statements are sufficient to answer the question. Read both the statements and choose the most appropriate option. (RBI Officer Grade ‘B’ Phase-I Exam 17.06.2017)

    Give answer (1) if the data in both the Statements I and II together are necessary to answer the question. Give answer (2) if the data either in Statement I alone or Statement II alone are sufficient to answer the question. Give answer (3) if the data even in both the Statements I and II together are not sufficient to answer the question. Give answer (4) if the data in Statement I alone are sufficient to answer the question while the data in Statement II alone are not sufficient to answer the question. Give answer (5) if the data in Statement II alone are sufficient to answer the question while the data in Statement I alone are not sufficient to answer the question. 29. Six persons A, B, C, D, E and F are sitting around a circular table facing the centre. What is E’s position with respect to F? I. A sits second to the right of D. Only two people sit between A and F. B sits to the immediate right of E. II. Only two people sit between D and E. E is an immediate neighbour of both B and A. C is not an immediate neighbour of B. 30. How is ‘dwarf definitely coded in a language? I. In a certain code language, ‘the dwarf planet’ is coded as ‘cq sd

    BPRE–922

    ap’ and ‘the solar energy’ is coded as ‘kb og sd’. II. In a certain code language, ‘only dwarf planet’ is coded as ‘cq ap st’, and ‘the coldest planet’ is coded as ‘sd cq yt’. 31. Fifteen people are standing in a straight row facing north. What is M’s position with respect to S? I. P stands third from the left end of the row. Only two people stand between P and M. N stands third to the right of M. S is an immediate neighbour of N. II. Q stands second from the right end of the row. Only two people stand between Q and O. O is an immediate neighbour of S. As many people stand to the left of M as to the right of S. Directions (32–36) : Each of the given questions, consists of a question and two statements numbered I and II given below it. You have to decide whether the data provided in the statements are sufficient to answer the question. Read both the statements and mark the appropriate option. (RBI Assistant Manager Online Exam 25.03.2017)

    Give answer (1) if the data in Statement I alone or in Statement II alone are sufficient to answer the question. Give answer (2) if the data in both the Statements I and II are not sufficient to answer the question. Give answer (3) if the data in both the Statements I and II together are necessary to answer the question. Give answer (4) if the data in Statement I alone are sufficient to answer the question, while the data in Statement II alone are not sufficient to answer the question. Give answer (5) if the data in Statement II alone are sufficient to answer the question, while the data in Statement I alone are not sufficient to answer the question. 32. Is T the son of K? Statements : I. K is the father of M. M is the sister of T. D is the son of T. F is the mother of D. II. U is the only son of L. L is the father of F. F is married to T. Z is the daughter of T. Y is the grandfather of Z. 33. Point D is in which direction with respect to Point C? Statements : I. Point C is 14 metre to the north of Point B. Point B is 19 metre to

    DATA SUFFICIENCY

    II.

    34.

    I.

    II.

    35.

    I.

    II.

    36.

    I.

    the west of Point A. Point A is 23 metre to the south of Point E. Point E is exactly between Points Q and D, such that Points Q, E and D form a straight line of 36 metre. Point D is 10 metre to the east of Point H. Point Q is 28 metre to the west of Point H. Point H is 9 metre to the east of Point E. Point E is 15 metre to the north of Point T. Point T is 19 metre to the east of Point C. Six different classes viz. A, B, C, D, E and F were conducted on six different days of the same week starting from Monday and ending on Saturday but not necessarily in the same order. On which day was class D conducted? Statements : Only three classes were conducted between E and A. E was conducted after A. C was conducted immediately before E. D was conducted immediately before F but not on Tuesday. Class B was conducted on one of the days before Wednesday. Only three classes were conducted between B and C. Only one class was conducted between D and C. Five people – P, Q, R, S and T are standing in a straight line facing the north but not necessarily in the same order. Who stands to the immediate right of P? Statements : Q stands at an extreme end of the line. Only two people stand between Q and S. R stands at one of the positions to the right of S. P is an immediate neighbour of R. S stands second from the left end of the line. Q stands at extreme right end of the line. R is an immediate neighbour of P. Five friends – A, B, C, D and E live on five different floors of a building (but not necessarily in the same orde). The lowermost floor of the building is numbered one, the one above that is numbered two and so on till the topmost floor is numbered five. On which floor does D live? Statements : A lives on an odd numbered floor but not on floor numbered one.

    Only one person lives between A and B. As many people live above B as below C. D lives on even numbered floor. II. C lives on an odd numbered floor but not on floor numbered three. Only one person lives between C and A. B lives on an odd numbered floor above A. D lives on one of the floors below A. Directions (37–41) : Each of these questions consists of a question and two statements numbered I and II. You have to decide whether the data given in the statements are sufficient to answer the questions. Read both statements and choose the most appropriate option. Give answer (1) if the data in statement I alone are sufficient to answer the question, while the data in statement II alone are not sufficient to answer the question. Give answer (2) if the data in statement II alone are sufficient to answer the question, while the data in statement I alone are not sufficient to answer the question. Give answer (3) if the data in statement I alone or in statement II alone are sufficient to answer the question. Give answer (4) if the data in both the statements I and II are not sufficient to answer the question. Give answer (5) if the data in both the statements I and II together are necessary to answer the question. (RBI Officers in Grade ‘B’ Phase-I Exam. 16.08.2018)

    37. Six boxes — A, B, C, D, E and F — are kept one above the other. What is the position of box C? I. No box is kept between A and B. Only four boxes are kept between B and C. F is kept at one of the positions above C. II. D is kept immediately above F. More than two boxes are kept between F and B. More than two boxes are kept between B and C. 38. A certain number of people are sitting in a straight line. What is the position of C from the right end of the line? I. C sits third to the left of F. Only one person sits between V and F. V sits ninth from the right end of the line.

    BPRE–923

    II. Q sits at the extreme right end of the line. Only four persons sit between Q and W. R sits second to the left of W. As many people sit between W and R as between C and R. 39. What is the bus fare from city A to city B? (Note : The only available denominations of the currency are – 10, 50, 100, 500 and 1000) I. For two tickets Dev gave four notes out of which three were of the same denomination lower than 100 and one was of denomination higher than 100. The conductor returned four notes of same denomination which was higher than 50. The balance amount was exact multiple of 13. II. For four tickets Suzie gave three notes all of which were of the same denomination above 50. The conductor returned four notes all of which were of the same denomination below 100. 40. Five people – J, K, L, M and N – were born in five different months of the same year – January, April, July, August and November. In which month was N born? I. J was born in a month having only 30 days. More than two people were born between J and K. M was born in one of the months before L. II. L was born in a month having 31 days. Only two people were born between L and K. N was born immediately after K. 41. Ranjit makes 100 pancakes daily. How many pancakes did he sell on Thursday? I. Ranjit’s brother correctly remembers that he sold more than 80 pancakes but less than 95 pancakes on Thursday. Ranjit’s sister correctly remembers that he sold more than 90 pancakes on Thursday and the number of pancakes sold was a multiple of 7. II. Ranjit’s mother correctly remembers that he sold more than 85 pancakes on Thursday but not all 100 pancakes on sale. Ranjit’s wife correctly remembers that less than 10 pancakes were unsold on Thursday and the number of unsold pancakes was a multiple of 3.

    DATA SUFFICIENCY Directions (42–44) : Each of the questions given below consists of a question and two statements numbered I and II given below it. You have to decide whether the data provided in the statements are sufficient to answer the question. Read both the statements and Give answer (1) if the data in statement I alone are sufficient to answer the question, while the data in statement II alone are not sufficient to answer the question. Give answer (2) if the data in statement II alone are sufficient to answer the question, while the data in statement I alone are not sufficient to answer the question. Give answer (3) if the data in statement I alone or in statement II alone are sufficient to answer the question. Give answer (4) if the data in both the statements I and II are not sufficient to answer the question. Give answer (5) if the data in both the statements I and II together are necessary to answer the question. (SEBI Assistant Manager Exam. 17.11.2018)

    42. Four boxes D, E, F and G are kept one above the other (but not necessarily in the same order). Which box is kept at the bottom? (I) Only one box is kept below F. No box is kept between F and D. E is kept at one of the positions above D. (II) Only one box is kept between D and G. G is kept at one of the positions below E. Only one box is kept between E and F. 43. Five different exams Q, R, S, T and U were conducted in January, April, July, September and December of the same year (but not necessarily in the same order). In which month was exam T conducted? (No exam was conducted in any other month in that year) (I) R was conducted in a month having only 30 days. Only one exam was conducted between R and U. T was conducted in one of the months before U. (II) S was conducted in a month having 31 days but not in January. R was conducted immediately before S. Only one exam was conducted between S and T.

    44. Eight people are sitting in two parallel rows containing four people each, in such a way that there is equal distance between adjacent persons. In row-1 J, K, L and M are seated and all of them are facing south. In row-2 A, B, C and D are seated and all of them are facing north. Therefore in the given seating arrangement each member seated in a row faces another member of the other row. What is the position of A with respect to D? (I) L sits at an extreme ends of the row. Only two people sit between D and the person facing J. A sits second to the left of B. (II) C sits second to the right of D. J sits to the immediate left of the person facing C. A does not face J. Directions (45–46) : Each of the questions given below consists of a question and two statements I and II given below it. You have to decide whether the data provided in the statements are sufficient to answer the question. Read both the statements and mark the appropriate option as your answer. Give answer (1) if the data in statement I alone are sufficient to answer the question, while the data in statement II alone are not sufficient to answer the question. Give answer (2) if the data in statement II alone are sufficient to answer the question, while the data in statement I alone are not sufficient to answer the question. Give answer (3) if the data in statement I alone or in statement II alone are sufficient to answer the question. Give answer (4) if the data in both the statements I and II are not sufficient to answer the question. Give answer (5) if the data in both the statements I and II together are necessary to answer the question. (RBI Grade ‘A’ Manager Exam, 15.06.2019)

    45. Five people-A, B, C, D and E are sitting around a circular table facing the centre (not necessarily in the same order). Who sits to the immediate left of E? Statements : I. A sits second to the right of B. Only one person sits between A

    BPRE–924

    and C (either from left or right). D is an immediate neighbour of C. II. Only two people sit between B and D (either from left or right). C sits to the immediate right of D. E sits to the immediate left of A. 46. Five boxes P, Q, R, S and T are kept one above another in a stack. Which box is kept second from the top of the stack? Statements : I. Only three boxes are kept between boxes Q and S. Box T is kept immediately below box Q. Only one box is kept between boxes T and P. II. Only two boxes are kept between boxes P and Q. Only one box is kept between boxes Q and R. Only one box is kept between boxes R and S. Directions (47–48) : The given questions consists of a questions and two statements numbered I and II given below it. You have to decide whether the data provided in the statements are sufficient to answer the question. Read both the statements and mark the appropriate option. Give answer (1) if the data in statement I alone are sufficient to answer the question, while the data in statement II alone are not sufficient to answer the question. Give answer (2) if the data in statement II alone are sufficient to answer the question, while the data in statement I alone are not sufficient to answer the question. Give answer (3) if the data in statement I alone or in statement II alone are sufficient to answer the question. Give answer (4) if the data in both the statements I and II are not sufficient to answer the question. Give answer (5) if the data in both the statements I and II together are necessary to answer the question. (RBI Grade ‘B’ Officer Exam, 09.11.2019)

    47. Six people – P, Q, R, S, T and U are sitting around a circular table facing the centre. How many people sit between U and S when counted from the left of U? Statements : I. S sits second to the right of T. Only two people sit between T and P. Only one person sits between U and P (either from left or right).

    DATA SUFFICIENCY II. Only one person sits between R and U (either from left or right). S sits to the immediate right of R. P is an immediate neighbour of S. 48. Five people viz, J, K, L, M and N joined a company on five different days of the same week starting from Monday and ending on Friday. On which day did M join? Statements : I. Only two people joined between K and L. K joined on one of the days before L. As many people joined after L as before M. II. N joined on one of the days after Wednesday. No one joined between N and J. Only one person joined between J and M.

    INSURANCE EXAMS Directions (1–6) : Each of the questions below consists of a question and two statements numbered I and II given below it. You have to decide whether the data given in the statements are sufficient to answer the questions. Read both the statements and—. (United India Insurance AO Exam. 27.03.2011)

    Give answer (1) if the data in statement I alone are sufficient to answer the question, while the data in statement II alone are not sufficient to answer the question. Give answer (2) if the data in statement II alone are sufficient to answer the question, while the data in statement I alone are not sufficient to answer the question. Give answer (3) if the data either in statement I alone or statement II alone are sufficient to answer the question. Give answer (4) if the data even in both statements I and II together are not sufficient to answer the questions. Give answer (5) if the data in both statements I and II together are necessary to answer the question. 1. D, E, F, G and H are sitting around a circle. Are all of them facing the centre ? I. D is sitting second to the left of G. H is facing the centre and is not an immediate neighbour of G.

    II. F is sitting second to the right of E. E is sitting second to the left of D. 2. How many floors are there in the building (including the ground floor) ? I. The area in which the building stands has a rule that no building can have more than six floors (including the ground floor). II. Three families reside in the building. No two families live immediately above or below each other. 3. Which direction is Sunny facing ? I. If Sunny turns 90° to his left, he will be facing North West. II. If Sunny turns 45° to his right he will be facing a direction that is exactly opposite to that of Sunil. 4. How is B related to E ? I. R is the mother of B and E is the husband of R. II. R has only one daughter who is J. 5. Among J, K, L, M and N each of a different age, who is the youngest ? I. J is younger than N but elder to K. II. L is younger than M but elder to K. 6. In which month of the year was the exam held ? I. Suzan correctly remembers that the exam was held before July but after March. II. Gita correctly remembers that the exam was held after February but before June. Directions (7–11) : Each of the questions below consists of a question and two statements numbered I and II given below it. You have to decide whether the data given in the statements are sufficient to answer the questions. Read both the statements and— (United India Insurance AO Exam. 26.05.2013)

    Give answer (1) if the data in statement I alone are sufficient to answer the question, while the data in statement II alone are not sufficient to answer the question. Give answer (2) if the data in statement II alone are sufficient to answer the question, while the data

    BPRE–925

    in statement I alone are not sufficient to answer the question. Give answer (3) if the data either in statement I alone or statement II alone are sufficient to answer the question. Give answer (4) if the data even in both statements I and II together are not sufficient to answer the questions. Give answer (5) if the data in both statements I and II together are necessary to answer the question. 7. What is the position of Yogesh from the left in the group of 21 students all of them facing north ? I. Praveen is exactly in the middle of the row. There are five persons between Praveen and Yogesh. Yogesh is fifth from the right end. II. There are two persons between Seema and Praveen and another two persons between Seema and Yogesh. 8. In which year did Ranjana complete her graduation ? I. According to Ranjana’s father she graduated after March 1989 but before March 1993. II. The brother of Ranjana remembers correctly that she did her graduation after March 1991 but before March 1994 and the year of her graduation was an even number. 9. Are M, S and N in a straight line ? I. N is 6 metres East of M. O is 4 metres North of N. T is 3 metres West of O and S is 4 metres South of T. II. P is 6 metres North of M. Q is exactly in the middle of P and M. R is 3 metres East of Q. S is 3 metres South of R. N is 3 metres East of S. 10. Whether it was 4 O’s clock two hours ago ? I. At present, both the hands of the clock face the opposite direction along a straight line. II. Two hours ago, both the hands of the clock together formed an angle of 120 degree. 11. Among I, A, G, K, J and V who is the tallest ? I. V is taller than J and G. K is taller than I and G. K is not the tallest.

    DATA SUFFICIENCY II. V is taller than K and I. J is taller than G. V is not the tallest. Directions (12–14) : Each of the questions below consists of a question and two statements numbered I and II given below it. You have to decide whether the data given in the statements are sufficient to answer the questions. Read both the statements and— (LIC Assistant Administrative Officer (AAO) Exam. 12.05.2013)

    Give answer (1) if the data in statement I alone are sufficient to answer the question, while the data in statement II alone are not sufficient to answer the question. Give answer (2) if the data in statement II alone are sufficient to answer the question, while the data in statement I alone are not sufficient to answer the question. Give answer (3) if the data either in statement I alone or statement II alone are sufficient to answer the question. Give answer (4) if the data even in both statements I and II together are not sufficient to answer the questions. Give answer (5) if the data in both statements I and II together are necessary to answer the question. 12. In which year did Avani start her business ? I. According to Avani’s mother, Avani has started her business after 2006 but before 2009. II. According to Avani’s father, she started her business before 2010 but after 2007 and the year was a Leap Year. 13. Who among Meera, Jyoti, Pinki and Sadhana is the tallest ? I. Meera is not as tall as Pinki or Sadhana. II. Jyoti is taller than Sadhana. 14. How is N related to M ? I. O is the husband of N. I. Z who is the sister of brother M is the daughter of N. 15. The logic problems in this set present you with three true statements: Fact 1, Fact 2, and Fact 3. Then, you are given three more statements (labelled I, II, and III), and you must determine which of these, if any, is also a

    fact. One or two of the statements could be true; all of the statements could be true; or none of the satements could be true. Choose your answer based solely on the informaiton given in the first three facts. Fact 1 : All hats have brims. Fact 2 : There are black hats and blue hats. Fact 3 : Baseball caps are hats. If the first three statements are facts, which of the following statements must also he a fact? I. All caps have brims. II. Some baseball caps are blue. III. Baseball caps have no brims. (1) I Only (2) II Only (3) II and III only (4) None of the statements is a known fact.

    17.

    I.

    II.

    (NICL (GIC) AO (Finance) Exam, 15.12.2013)

    Directions (16–20) : Each of the following questions consists of a question and two statements numbered I and II given below it. You have to decide whether the data given in the statements are sufficient to answer the question. Read both the statements and choose the most appropriate answer. (NIACL Administrative Officer (AO) Exam, 10.01.2015)

    16. How is ‘self’ coded in the given code language ? I. In the given code language, ‘help self to win’ is coded as ‘& $ * ’+’ and ‘help to win him’ is coded as ‘+ $ % &’. II. In the given code language ‘make plan with self’ is coded as ‘@ * # >’ and ‘connect to self important’ is coded as ‘&/? *’. (1) The data either in statement I alone or statement II alone are sufficient to answer the question. (2) The data even in both statements I and II together are not sufficient to answer the question. (3) The data in statement II alone are sufficient to answer the question while the data in statement I alone are not sufficient to answer the question.

    BPRE–926

    18.

    I.

    II.

    (4) The data in both the statements I and II together are necessary to answer the question. (5) The data in statement I alone are sufficient to answer the question while the data in statement II alone are not sufficient to answer the question. Among students M, N, O, P and Q, each having a different weight, who weights the most ? P weights more than M but less than N. N does not weigh the most. M weighs more than O. P weighs less than only two people. M weighs more than O but less than Q. (1) The data either in statement I alone or statement II alone are sufficient to answer the question. (2) The data even in both statements I and II together are not sufficient to answer the question. (3) The data in statement II alone are sufficient to answer the question while the data in statement I alone are not sufficient to answer the question. (4) The data in both the statements I and II together are necessary to answer the question. (5) The data in statement I alone are sufficient to answer the question while the data in statement II alone are not sufficient to answer the question. On which day, of the same week–from Monday to Sunday — was Naina born ? Naina’s aunt correctly remembers that she was born after Tuesday but before Sunday and that she was not born on a Thursday. Naina’s uncle correctly remembers that she was born after Monday and she was not born on a Wednesday. (1) The data either in statement I alone or statement II alone are sufficient to answer the question. (2) The data even in both statements I and II together are not sufficient to answer the question.

    DATA SUFFICIENCY

    19.

    I.

    II.

    20. I.

    II.

    (3) The data in statement II alone are sufficient to answer the question while the data in statement I alone are not sufficient to answer the question. (4) The data in both the statements I and II together are necessary to answer the question. (5) The data in statement I alone are sufficient to answer the question while the data in statement II alone are not sufficient to answer the question. How many people stand between H and L in a straight line of 8 people (Note : All are facing North) ? B stands second from the right end of the line. Only two people stand between B and M. Only one person stands between M and L. H stands to the immediate right of S. Y stands at the extreme left end of the line. Only three people stand between Y and S. S is an immediate neighbour of both H and M. L stands second to the left of M. (1) The data either in statement I alone or statement II alone are sufficient to answer the question. (2) The data even in both statements I and II together are not sufficient to answer the question. (3) The data in statement II alone are sufficient to answer the question while the data in statement I alone are not sufficient to answer the question. (4) The data in both the statements I and II together are necessary to answer the question. (5) The data in statement I alone are sufficient to answer the question while the data in statement II alone are not sufficient to answer the question. Is W the uncle of E ? M is the sister of W. B has only one daughter, M. D is the daughter of M. E is the sister of D. S has only two children, M and W. E is the daughter of M. J is married to M. W is married to K.

    (1) The data either in statement I alone or statement II alone are sufficient to answer the question. (2) The data even in both statements I and II together are not sufficient to answer the question. (3) The data in statement II alone are sufficient to answer the question while the data in statement I alone are not sufficient to answer the question. (4) The data in both the statements I and II together are necessary to answer the question. (5) The data in statement I alone are sufficient to answer the question while the data in statement II alone are not sufficient to answer the question. Directions (21–25) : Each of the following questions consists of a question and two statements numbered I and II given below it. You have to decide whether the data given in the statements are sufficient to answer the question. Read both the statements and choose the most appropriate answer. (NIACL Administrative Officer (AO) Online Exam, 11.01.2015)

    21. How many persons stand between N and B in a straight line of nine persons, all facing north ? I. J is at the extreme right end of the line. Only two persons stand between J and F. B stands exactly between U and F. Only three persons stand between K and B. N is an immediate neighbour of K. II. K stands at the extreme left end of the line. Only one person stands between K and R. R stands between N and U. Only three persons stand between N and F. B is an immediate neighbour of F. (1) The data in statement II alone are sufficient to answer the question while the data in statement I alone are not sufficient to answer the question. (2) The data in both the statements I and II together are necessary to answer the question.

    BPRE–927

    (3) The data in statement I alone are sufficient to answer the question while the data in statement II alone are not sufficient to answer the question. (4) The data either in statement I alone or statement II alone are sufficient to answer the question. (5) The data even in both statements I and II together are not sufficient to answer the question. 22. Is J grandfather of T ? I. M is mother of S and T. L is father of S. K is mother-in-law of L. K is married to J. II. M and L are married couple. T is only daughter of L. M is daughter of J. K is wife of J. (1) The data in statement II alone are sufficient to answer the question while the data in statement I alone are not sufficient to answer the question. (2) The data in both the statements I and II together are necessary to answer the question. (3) The data in statement I alone are sufficient to answer the question while the data in statement II alone are not sufficient to answer the question. (4) The data either in statement I alone or statement II alone are sufficient to answer the question. (5) The data even in both statements I and II together are not sufficient to answer the question. 23. In which year Sania was born ? I. Sania’s cousin remembers that she was born after year 1982 but before 1989 and she was born in an even number year. II. Sania’s aunt remembers that she was born after 1984 but before 1990 and that was a leap year. (1) The data in statement II alone are sufficient to answer the question while the data in statement I alone are not sufficient to answer the question. (2) The data in both the statements I and II together are necessary to answer the question.

    DATA SUFFICIENCY (3) The data in statement I alone are sufficient to answer the question while the data in statement II alone are not sufficient to answer the question. (4) The data either in statement I alone or statement II alone are sufficient to answer the question. (5) The data even in both statements I and II together are not sufficient to answer the question. 24. Among A, B, C, D and E, each having different height, who is the shortest ? I. E is taller than C. E is shorter than A. A is not the tallest among them. B is taller than only C. II. A is taller than only three persons. C is shorter than A and E. (1) The data in statement II alone are sufficient to answer the question while the data in statement I alone are not sufficient to answer the question. (2) The data in both the statements I and II together are necessary to answer the question. (3) The data in statement I alone are sufficient to answer the question while the data in statement II alone are not sufficient to answer the question. (4) The data either in statement I alone or statement II alone are sufficient to answer the question. (5) The data even in both statements I and II together are not sufficient to answer the question. 25. How is ‘most’ coded in a certain code language ? I. In that code language ‘most of the time’ is written as ‘# & % ?’ and ‘of the regular time’ is written as ‘& # ? >’. II. In that code language ‘most people make it’ is written as ‘@ * % $’ and ‘it most special tough’ is written as ‘% £ + $’. (1) The data in statement II alone are sufficient to answer the question while the data in statement I alone are not sufficient to answer the question.

    (2) The data in both the statements I and II together are necessary to answer the question. (3) The data in statement I alone are sufficient to answer the question while the data in statement II alone are not sufficient to answer the question. (4) The data either in statement I alone or statement II alone are sufficient to answer the question. (5) The data even in both statements I and II together are not sufficient to answer the question. Directions (26-30) : Each of the following questions below consists of a question and two statements numbered I and II. You have to decide whether the data provided in the statements are sufficient to answer the question. (NIACL Administrative Officer (AO) Online Exam, 12.01.2015)

    26. Who amongst L, M, N, O and P is the shortest ? I. O is shorter than P but taller than N. II. M is not as tall as L. (1) The data in statement I alone are sufficient to answer the question, while the data in statement II alone are not sufficient to answer the question. (2) The data in statement II alone are sufficient to answer the question, while the data in statement I alone are not sufficient to answer the question. (3) The data either in statement I alone or in statement II alone are sufficient to answer the question. (4) The data in both statements I and II together are not sufficient to answer the question. (5) The data in both the statements I and II together are necessary to answer the question. 27. Point A is towards which direction from point B ? I. If a person walks 4m towards the north from point A, and takes two consecutive right turns, each after walking 4m, he would reach point C, which is 8m away from point B.

    BPRE–928

    II. Point D is 2m towards the east of point A and 4m towards the west of Point B. (1) The data in both the statements I and II together are not sufficient to answer the question. (2) The data in statement II alone are sufficient to answer the question, while the data in statement I alone are not sufficient to answer the question. (3) The data in statement I alone are sufficient to answer the question, while the data in statement II alone are not sufficient to answer the question. (4) The data in both the statements I and II together are necessary to answer the question. (5) The data either in statement I alone or in statement II alone are sufficient to answer the question. 28. How many brothers does Bharat have ? I. Shiela, the mother of Bharat has only three children. II. Meena, the grandmother of Bharat has only one granddaughter. (1) The data in either in statement I alone or statement II alone are sufficient to answer the question. (2) The data in statement I alone are sufficient to answer the question, while the data in statement II alone are not sufficient to answer the question. (3) The data in both the statements I and II together are necessary to answer the question. (4) The data in both the statements I and II together are not sufficient to answer the question. (5) The data in statement II alone are sufficient to answer the question, while the data in statement I alone are not sufficient to answer the question 29. Is T the grandmother of Q ? I. P is the mother of Q. Q is the son of R. R is son of T. II. L is the father of N and N is the daughter of T.

    DATA SUFFICIENCY (1) The data in either statement I alone or statement II alone are sufficient to answer the question. (2) The data in statement II alone are sufficient to answer the question, while the data in statement I alone are not sufficient to answer the question. (3) The data in both statements I and II together are sufficient to answer the question. (4) The data in statement I alone are sufficient to answer the question, while the data in statement II alone are not sufficient to answer the question. (5) The data in both the statements I and II together are not sufficient to answer the question. 30. Are all the five friends viz. Leena, Amit, Arun, Ali and Ken who are seated around a circular table facing the centre ? I. Leena sits second to the left of Amit. Amit faces the centre. Arun sits second to the right of Leena. II. Ali sits third to the left of Ken. Ken faces the centre. Amit sits to the immediate left of Ali but Ken in not an immediate neighbour of Amit. (1) The data in statement I alone are sufficient to answer the question, while the data in statement II alone are not sufficient to answer the question. (2) The data in both the statements I and II are not sufficient to answer the question. (3) The data in both the statements I and II are necessary to answer the question. (4) The data in either statement I alone or statement II alone are sufficient to answer the question. (5) The data in statement II alone are sufficient to answer the question, while the data in statement I alone are not sufficient to answer the question. Directions (31–35) : Each of the following questions consists of a question and two statements numbered I and II given below it. You have to decide whether the data provided in the statements are sufficient to answer the question. Read both the statements and (UIICL A.O. Exam 12.06.2016)

    Give answer (1) if the data in statement I alone are sufficient to answer the question, while the data in statement II alone are not sufficient to answer the question. Give answer (2) if the data in statement II alone are sufficient to answer the question, while the data in statement I alone are not sufficient to answer the question. Give answer (3) if the data in statement I alone or in statement II alone are sufficient to answer the question. Give answer (4) if the data in both the statements I and II are not sufficient to answer the question. Give answer (5) if the data in both the statements I and II together are necessary to answer the question. 31. Seven persons A, B, C, D, E, F and G are standing in a straight line facing North, not necessarily in the same order. What is the position of D with respect to G ? I.

    F stands fifth to the right of B. Only one person stands between F and G. D is neither an immediate neighbour of B nor F.

    II.

    C stands at one of the extreme ends of the line. Only two persons stand between C and G. D stands second to the right of B. Among five friends, S, T, U, V and W, who owns the maximum number of houses ? (each owning a different number of houses)

    32.

    I.

    II.

    33.

    I.

    T owns more houses than V. V owns more houses than S but less than W. U does not own the least number of houses. Only one person owns more number of houses than W. S and V owns less number of houses than U. T owns more houses than S. A certain number of boxes have been stacked one above the other. How many boxes are there in the stack ? Only three boxes are kept between U and B. M is kept immediately below B but not at the bottom of the stack. Only five boxes are kept between M and D.

    BPRE–929

    II.

    34.

    I.

    II.

    35.

    I.

    R is kept exactly in the middle of the stack. Only two boxes are kept between R and U. S is kept immediately above U. R is kept above U. A certain number of boxes have been stacked one above the other. How many boxes are there in the stack ? G is kept fifth from the bottom of the stack. Only two boxes are kept between G and Y. Only three boxes are kept above Y. G is kept above Y. F is kept third from the top of the stack. Y is kept immediately below F. Only two boxes are kept between Y and B. B is kept above F. Seven people P, Q, R, S, T, U and V are standing in a straight line facing north, not necessarily in the same order. What is the position of R with respect to T ? S stands second to the right of R. P stands third to the left of R. Only one person stands between P and T.

    II.

    Q stands second from the left end of the line. T is an immediate neighbour of Q. R stands to the immediate left of U. Directions (36–37) : Each of the questions below consists of a question and two statements numbered I and II are given below it. You have to decide whether the data provided in the statements are sufficient to answer the question. Read both the statements and — (GIC Officer Online Exam 15.05.2017)

    Give answer (1) if the data in Statement I alone are sufficient to answer the question, while the data in Statement II alone are not sufficient to answer the question. Give answer (2) if the data in Statement II alone are sufficient to answer the question, while the data in Statement I alone are not sufficient to answer the question. Give answer (3) if the data in Statement I alone or in Statement II alone are sufficient to answer the question. Give answer (4) if the data in both the Statements I and II are not sufficient to answer the question. Give answer (5) if the data in both the Statements I and II together are necessary to answer the question.

    DATA SUFFICIENCY 36. How is ‘success’ definitely coded in the given code language? (Note: All the given codes are two letter codes only) I. ‘ways to achieve success’ is coded as ‘ja tp fa rg’ and ‘success difficult to achieve’ is coded as ‘rg ja ko tp’. II. ‘difficult times before success’ is coded as ‘nb ko tp li’ and ‘times fly before ways’ is coded as ‘fa li nb dy’. 37. Five cars – P, Q, R, S and T — were bought on Monday, Tuesday, Wednesday, Thursday and Friday of the same week (But not necessarily in the same order). On which day was S bought? (No car was bought on any other day of the week) I. R was bought on Wednesday. Only one car was bought between R and T. S was bought on one of the days before T but not on Tuesday. II. Only three cars were bought between T and Q. P was bought immediately before R but not on Wednesday. Directions (38–39) : Each question given below consists of a question and two statements numbered I and II given below it. You have to decide whether the data given in the statements are sufficient to answer the questions. Read both the statements and choose the most appropriate option. Give answer (1) if the data in statement I alone are sufficient to answer the question, while the data in statement II alone are not sufficient to answer the question. Give answer (2) if the data in statement II alone are sufficient to answer the question, while the data in statement I alone are not sufficient to answer the question. Give answer (3) if the data in statement I alone or in statement II alone are sufficient to answer the question. Give answer (4) if the data in both the statements I and II are not sufficient to answer the question. Give answer (5) if the data in both the statements I and II together are necessary to answer the question. (LIC AAO Main Exam, 28.06.2019)

    38. There are seven letters W, M, V, N, Q, R, and J arranged in an order from left to right. What is the fifth letter from the left end? Statement–I : Q is placed fifth

    to the right of V. W and J are not placed immediately next to either Q or V. Q is placed to the immediate left of R Statement–II : M is placed to the immediate left of W but is not an immediate neighbour of Q. N is placed second to the left of R. 39. Who is oldest among P, K, J, R, S and T ? Statement–I: R is older than P and J. R is younger than K. S is older than only T. Statement–II : S is younger than J and P. T is younger than S. R is not the oldest.

    SHORT ANSWERS

    NATIONALISED BANKS & IBPS PO/MT/SO EXAMS 1. (5) 5. (5) 9. (2) 13. (5) 17. (4) 21. (5)

    2. (4) 6. (4) 10. (5) 14. (5) 18. (3) 22. (4)

    3. (1) 7. (3) 11. (4) 15. (1) 19. (4) 23. (1)

    4. (2) 8. (4) 12. (5) 16. (4) 20. (1) 24. (4)

    25. (5)

    26. (4)

    27. (5)

    28. (1)

    29. (3)

    30. (5)

    31. (1)

    32. (3)

    33. (4)

    34. (5)

    35. (4)

    36. (2)

    37. (4) 41. (5) 45. (5) 49. (5) 53. (4) 57. (3) 61. (5) 65. (5) 69. (3) 73. (5) 77. (3) 81. (5) 85. (2) 89. (4) 93. (1) 97. (2) 101. (1) 105. (4) 109. (2) 113. (1) 117. (2) 121. (3) 125. (4) 129. (4)

    38. (5) 42. (4) 46. (5) 50. (4) 54. (5) 58. (1) 62. (2) 66. (3) 70. (1) 74. (5) 78. (5) 82. (5) 86. (1) 90. (2) 94. (5) 98. (3) 102. (2) 106. (1) 110. (2) 114. (3) 118. (2) 122. (4) 126. (5) 130. (3)

    39. (4) 43. (4) 47. (2) 51. (2) 55. (3) 59. (4) 63. (3) 67. (1) 71. (3) 75. (4) 79. (4) 83. (5) 87. (2) 91. (1) 95. (5) 99. (3) 103. (5) 107. (3) 111. (2) 115. (3) 119. (1) 123. (5) 127. (5) 131. (2)

    40. (1) 44. (3) 48. (2) 52. (1) 56. (5) 60. (2) 64. (4) 68. (2) 72. (3) 76. (3) 80. (5) 84. (4) 88. (5) 92. (2) 96. (4) 100. (4) 104. (2) 108. (5) 112. (3) 116. (2) 120. (3) 124. (5) 128. (2) 132. (2)

    BPRE–930

    133. (1) 137. (5) 141. (2) 145. (1) 149. (3)

    134. (5) 138. (5) 142. (2) 146. (3) 150. (2)

    135. (3) 139. (2) 143. (3) 147. (4)

    136. (2) 140. (5) 144. (2) 148. (3)

    SBI PO EXAMS 1. (3) 5. (4) 9. (2) 13. (3) 17. (1) 21. (2) 25. (1) 29. (3) 33. (1) 37. (3) 41. (5) 45. (3) 49. (3) 53. (4) 57. (1) 61. (5)

    2. (2) 6. (1) 10. (5) 14. (1) 18. (3) 22. (5) 26. (4) 30. (3) 34. (2) 38. (1) 42. (3) 46. (3) 50. (5) 54. (2) 58. (4) 62. (5)

    3. (4) 7. (3) 11. (3) 15. (3) 19. (3) 23. (2) 27. (3) 31. (3) 35. (5) 39. (3) 43. (5) 47. (1) 51. (3) 55. (4) 59. (5) 63. (1)

    4. (2) 8. (5) 12. (1) 16. (2) 20. (3) 24. (5) 28. (5) 32. (1) 36. (3) 40. (3) 44. (3) 48. (3) 52. (1) 56. (5) 60. (3) 64. (2)

    RBI GRADE–B/NABARD GRADE–A OFFICER EXAMS 1. (5) 5. (1) 9. (2) 13. (3) 17. (3) 21. (5) 25. (2) 29. (4) 33. (5) 37. (3) 41. (1) 45. (3)

    2. (1) 6. (2) 10. (2) 14. (5) 18. (5) 22. (3) 26. (4) 30. (1) 34. (4) 38. (2) 42. (4) 46. (3)

    3. (4) 7. (4) 11. (2) 15. (5) 19. (3) 23. (1) 27. (3) 31. (1) 35. (2) 39. (1) 43. (5) 47. (2)

    4. (5) 8. (3) 12. (1) 16. (1) 20. (4) 24. (5) 28. (1) 32. (4) 36. (5) 40. (5) 44. (2) 48. (5)

    INSURANCE EXAMS 1. (2) 5. (5) 9. (3) 13. (4) 17. (1) 21. (2) 25. (3) 29. (3) 33. (2) 37. (2)

    2. (5) 6. (4) 10. (1) 14. (5) 18. (2) 22. (4) 26. (4) 30. (4) 34. (3) 38. (5)

    3. (1) 7. (1) 11. (5) 15. (2) 19. (3) 23. (1) 27. (2) 31. (1) 35. (1) 39. (1)

    4. (5) 8. (2) 12. (2) 16. (1) 20. (2) 24. (3) 28. (3) 32. (5) 36. (5)

    DATA SUFFICIENCY

    5m Amit

    NATIONALISED BANKS & IBPS PO/MT/SO EXAMS 1. (5) From both the statements Jason > Peter, Joseph > Kevin 2. (4) From both the statements I

    happy today

    ne

    ke

    today happy day

    que

    que

    play

    B 5m

    Ali is facing outward. 8. (4) From statement I Q is the grandson of T. But the gender of T is not given. 9. (2) From statement I 4m

    joi

    ke

    A

    T

    S

    R

    Q is second to the right of T. 6. (4) From both the statements P>O>N L>M 7. (3) From statement I Leena

    Eeast

    The position of B cannot be ascertained. From statement II 2m A D B 4m A is towards west of B. 10. (5) From both the statements Bharat has one brother and one sister. 11. (4) From both the statements go there → na ja

    go there

    U

    B

    Arun sits second to the right of Leena. If Leena faces the centre then Arun cannot be second to the right of Leena. It means Leena faces outward. From statement II

    NE

    ni ho

    and come back → ma ho sa

    West

    ni

    da

    The code for ‘never’ is either ‘na’ or ‘ja’.

    12. (5) From both the statements,

    East SE

    SW South

    From statement II A

    D

    B

    C

    B

    17. (4) From statement I M, Q and R are daughters of S. The sex of S is not known. From statement II There is no information about S. 18. (3) From statement I A

    K > J > W > P > M, T

    13. (5) From both the statements, 5

    $ # 3



    flowers are

    7

    # 3 5

    really

    good

    → good flowers are available

    Amit

    NW

    South

    C

    never ever

    Q

    B

    North West

    4m

    pa

    P

    B

    North

    ne

    The code for ‘happy’ is ‘ke’ or ‘ne’. 3. (1) From statement I H is mother of J and V. V is the only daughter of H. Therefore, J is brother of V. 4. (2) From statement II The colour of white snow is orange. 5. (5) From both the statements

    A

    Ken

    4m

    I

    16. (4) From Statement I

    Ali

    EXPLANATIONS

    Therefore, $ ⇒ really 14. (5) From both the statements, P and T are daughters of J. M is son of J. 15. (1) From statement I,

    D If A is facing the centre, B cannot be second to the right of A. It means, A is not facing the centre. From statement II C

    12th S

    M 17th

    50 – 29 = 21

    BPRE–931

    E

    DATA SUFFICIENCY If D is second to the right of C, then, C is facing outward. 19. (4) From both the statements car

    4 9

    2

    8

    can we buy car

    5 8

    0

    2

    can car be cheap

    8 1

    5

    3

    we

    came by

    came by cheap car

    9

    8

    4

    20. (1) From statement I

    a letter

    C>A> B D 32. (3) From statement I

    7

    2

    1

    8

    received a Greek letter

    7

    5

    1

    3

    write in English please

    2

    0

    8

    4

    a letter

    5

    7

    1

    4

    please write

    1

    31. (1) From statement I

    25. (5) From both the statements P>Q=R>S>T Bag T is the lightest. 26. (4) From both the statements

    in Greek

    Leena > Pia > Shreya

    Left

    Shreya is the shortest. From statement II

    E

    A

    C

    B

    D

    Right

    27. (5) From both the statements

    Jyoti, Kajal

    Ken

    North West

    East South

    Ken is now facing south. Therefore, Ali is facing north. From statement II Priya

    Kajal > Jyoti, Leena > Pia

    28. (1) From statement I

    Shreya

    21. (5) From both the statements.

    Fifth Floor

    R

    Fourth Floor

    A

    5

    S

    4

    T

    Third Floor Second Floor

    D C

    First Floor

    E

    6

    3 Unoccupied 2

    P

    1

    Q

    29. (3) From statement I

    3 6 9 2

    do you see that

    1 9 7 3

    I

    do well

    0 2 8 6 5 4 6 7

    The code for ‘see’ is either ‘3’ or ‘9’. 23. (1) From statement I

    C/E

    Kavita

    Prashant

    hope to see you

    hope

    P

    Ground Floor

    22. (4) From both the statements

    to pray and hope

    Priya is now facing north. 33. (4) No definite conclusion can be derived even with the data provided in both the statements. 34. (5) From both the statements

    B

    Yasir

    T 5 km R

    Abhay

    Kavita is facing outward. From statement II

    2 km

    >

    > > > >D>F From all the three statements E>B>A>C>D>F 38. (5) Statement I

    R

    W

    A Kavita

    Abhay

    T C/E

    Statement II

    D

    D is sitting to the immediate left of A. 24. (4) From both the statements X and Y are parents of M and R. But, it is not possible to determine the gender of X or Y.

    T

    Yasir

    Prashant is facing outward. 30. (5) From both the statements C is grandmother of B and R. R is granddaughter of C.

    BPRE–932

    Q

    35. (4) From both the statements The gender of B is not known. 36. (2) All, I, II and III are required to answer the question. P is the mother of B, D and M. B and D are daughters of P. 37. (4) Statement I E>B>A Statement II > > >C> Statement III

    Prashant

    B

    7 km

    J

    Z

    From statements I and II

    DATA SUFFICIENCY Starting Point

    30 m

    R

    47. (2) From statements I and III

    20 m

    W

    1 2

    3

    A

    S

    4

    5 E

    A N

    S

    W E

    6 R

    48. (2) From statements I and III 30 m

    T J

    45. (5) From both the statements Z and P are sisters of D and K. K is brother of Z. 46. (5) From statements I, II and III

    Z

    J is to the south-west of W. 39. (4) From all the three statements Manoj’s mother visited his house on Tuesday and Manojdid visit Chennai on Wednesday. 40. (1) From statement I and III

    now or never again

    Floor No. 6 5 4 3 2 1

    Person P T R S Q V

    North C

    East

    West South

    A

    B

    E

    D

    F

    49. (5) From statements I and II

    → tom ka na sa

    again go now or never → na ho ka sa tom

    one

    of

    its

    in

    kind

    its

    point

    kind

    and

    cash

    zo

    pi

    ga

    to

    ko

    fe

    ru

    ko mi

    The code for ‘go’ is ‘ho’. 41. (5) From both the statements

    for

    origin

    ba

    le

    fe

    point

    clear

    yu

    si

    mi

    P, W > T > M > R 42. (4) From both the statements

    you will be gone

    ka pa ni sa

    he will be there

    ja da ka ni

    a

    It is not given, the brother of Sushant visited Chennai on which day. From statement II Sushant visited Chennai on Monday, Tuesday, Thursday, Saturday or Sunday. 44. (3) From statement I

    in

    of

    its

    kind

    kind

    and

    zo pi

    cash

    ga

    make money and cash money

    of various kind

    ko

    to

    fe

    ru

    mi

    ru hy

    qu

    ko

    zo

    From statements I, II and III

    kind

    zo

    pi

    ko

    fe

    in

    kind and cash

    ga

    to

    ru

    ko

    its

    point for origin

    ba

    le

    fe

    mi

    make

    a point clear

    yu

    si

    mi

    de

    make

    money and cash

    to

    mi ru

    hy

    of various kind

    qu

    ko

    zo

    hy

    one of West

    East

    30 m

    South

    20 m

    money From statement II

    its

    BPRE–933

    ko

    to

    North

    Starting Point

    de

    From statements I and III

    one

    43. (4) From statement I

    20 m

    make

    hy

    DATA SUFFICIENCY 50. (4) From statement I

    D/E B D/E

    P

    B

    A

    56. (5) From statement I Neha’s birthday is on Wednesday or Thursday. From statement II Neha’s birthday is on Tuesday or Wednesday. From both the statements Neha’s birthday is on Wednesday. 57. (3) From statements II and III it is clear that S is the daughterin-law of B. B is the mother of N and R. T is the son of N and S is the mother of T. So, S is the wife of N. 58. (1) From statements I and II, there are 33 students in the class. From statements II and III there may be 33 or 44 students in the class. 59. (4) From statement I,

    C

    D From statement II

    A

    B

    D

    C From statement III

    B

    A

    C

    D

    a paper

    8

    1

    7

    Thinking and writing

    6

    2

    4

    4

    5

    3

    7

    2

    3

    Writing

    with Pen

    Paper and Pen

    B

    Q

    O

    From both the statements

    Q

    P

    M

    N

    O

    N

    P

    O

    Q

    O M

    Q

    Q

    P

    or

    O

    Monday Tuesday

    Q

    P

    Either N or P is second to the left of M. 60. (2) From statement I Y is mother of Z. V is grandson of X. It is not possible to determine the relation between X and Z. From statement II Z is daughter–in–law of W. X is husband of Z. 61. (5) From statement I Day

    From statement II

    Wednesday

    Q

    C N

    From statement II

    Q

    or

    or

    55. (3) From statement I

    A

    O

    P

    51. (2) It is clear from the statement that a farm’s areas for organic and chemical farming are different. 52. (1) From statement I B is an architect. A is a doctor. C is an engineer. From statement II Either A or B or C is a doctor. 53. (4) From both the statements The gender of Q is not given. Q is either son or daughter of P and R. 54. (5) From both the statements Read

    N

    O

    Event

    B

    G

    B

    D

    E

    Thursday

    A

    B

    G

    Friday

    A

    G

    D

    G

    D

    Saturday O

    BPRE–934

    Sunday

    A

    D

    DATA SUFFICIENCY J is son of K. J is husband of Y.

    From statement II Day Monday

    W is son of J and Y. Y is mother of W.

    Event G

    N is brother of K. So, N is brother-in-law of L. From statement II L is the husband of K.

    From statement II

    Tuesday

    F

    Wednesday

    C

    F

    Thursday

    A

    C

    F

    Friday

    F

    A

    C

    Saturday

    C

    A

    Sunday

    A

    J is brother of L. Y is sister-inlaw of L. So, J is husband of Y. W is son of J and Y. Y is mother of W. 64. (4) From statement I 3 2 4 6

    the nice mount view the view was heavenly

    2 5 6 8

    From statement II

    From both the statements Day Monday Tuesday Wednesday Thursday Friday Saturday Sunday

    66. (3) From statement I K is wife of L.

    the man admired view

    3 2 4 6

    the nice mount view the view was heavenly

    2 5 6 8 6 7 2 1

    he saw the view the man admired view

    Q

    O

    reason to learn better

    xn zt aj ly

    to learn study better

    zt xn ly rj

    The code for ‘reason’ is ‘aj’. From statement II

    0 9 6 2

    From both the statements

    Event A was organised on Sunday. 62. (2) From statement I

    So, N is brother-in-law of L. 67. (1) From statement I

    6 7 2 1

    he saw the view

    Event G D E B F C A

    N is brother of K.

    reason to study important

    yk xn aj rj

    to find reason necessary

    st xn ds aj

    The code for ‘reason’ may be ‘xn’ or ‘aj’. 68. (2) From statement I

    0 9 6 2

    7 persons

    The code for ‘view’ may be either ‘2’ or ‘6’. 65. (5) From statement I O

    J

    T

    T

    K T

    T

    S

    S

    L

    6 persons

    U

    From statement II

    K

    M L

    S

    T

    R

    8 Persons

    T is 10th from the right end. 69. (3) From statement I

    or

    M

    U

    or

    O

    N

    L

    J

    O

    A

    C

    P

    It is not clear whether P is facing towards the centre or outside. From statement II N

    D

    B

    From statement II T K

    J S J

    T

    E

    F

    D is third to the left or right of B. From statement II

    Q

    or T J S J

    F

    E

    T K

    O

    P

    B

    D From both the statements M

    O

    S J

    T K

    A

    Q is sitting second to the left of P.

    C

    63. (3) From statement I L is daughter of K. L is sister of J.

    J is third to the left of K.

    BPRE–935

    D is third to the left or right or B.

    DATA SUFFICIENCY

    E F

    I

    G

    F

    From statement II

    Play U Z V W Y T/X T/X

    I/E

    I/E

    I/E

    I/E

    From both the statements

    B

    H

    F

    I

    G

    &

    4

    $

    2

    reason is never little

    #

    &

    8

    2

    From statement II little to reason now

    2

    %

    4 &

    believe now is problem

    %

    8

    $ @

    reason

    to

    believe

    &

    4

    $

    is

    never

    little

    #

    &

    8

    2

    little

    to

    reason

    now

    &

    2

    %

    4

    now

    is

    problem

    %

    8

    $

    @

    believe

    BPRE–936

    S

    Q

    W

    X

    Several arrangements are possible From both the statements

    2

    The code for ‘reason’ may be ‘&’ or ‘2’.

    R

    From statement II

    From both the statements

    reason

    T

    RIGHT

    little reason to believe

    R

    RIGHT

    C is third to the left of F. 78. (5) From statement I

    Thus, G is to the immediate left of F. 75. (4) From statement I

    little

    A

    R

    S

    P

    T

    X

    Q

    W

    RIGHT

    K F J Thus, there are 11 persons in the line. 73. (5) From statement I > > >O > > N>P>Q There is no information about M and R. From statement II > >M R>N>O P>Q From both the statements R>N>M>O>P>Q

    E F

    J

    Middle

    C

    D

    O .......

    Thus, there are 11 persons in the line. From statement II

    A

    E

    LEFT

    L

    E

    C is third to the left of F. From statement II

    G/H

    Play U U Y Z Z/V V/W W

    A

    D

    F

    72. (3) From statement I

    B T

    C

    G/H

    From statement II Day Monday Tuesday Wednesday Thursday Friday Saturday Sunday

    B

    LEFT

    Day Monday Tuesday Wednesday Thursday Friday Saturday Sunday

    76. (3) From statement I R is the husband of A. R is the son of T and F. F is the mother of R. A is the daughter-in-law of F. From statement II C is the aunt of B and D. So, C is a female. T has only two children. So, R is a male and is husband of A. Now, A is the daughter in law of F. 77. (3) From statement I

    74. (5) From statement I

    LEFT

    70. (1) From statement I T is daughter of R. T is wife of M. L is daughter of M and T. L is cousin of J. So, M is uncle of J. From statement II Y is sister of daughter in-law of A. J is son of D and Y. A is father-in-law of T. M is grandfather or great grandmother of J. 71. (3) From statement I

    Four people-T, X, Q and W- sit to the right of P.

    DATA SUFFICIENCY 79. (4) From statement I

    Form both of statements

    6m

    H

    M

    From statement II

    > R > D >M

    M H

    H

    Clearly, S scored the highest marks in the examination. 83. (5) From statement I B studies in 1st or 7th standard C studies in 4th or 8th standard From statement II

    5m

    H

    S

    M

    5m

    R

    5m A

    9m

    C>B>A> H

    North

    From statement II

    From both the statements 10th 8th 7th 4th 1st D/E C B A D/E 84. (4) From statement I

    M

    T

    H

    N

    H

    going to a party

    la

    fa

    gi

    a party

    fa

    di ne

    ne

    Q ne bs

    going to start

    gi

    Q

    am

    bs la

    From both the statements going to a party

    la

    for

    fa

    start

    a

    party the

    party

    ne

    fa di bs

    gi

    ne

    S From statement II

    ne

    R

    am

    East SE

    SW

    From statement II start the party

    NE

    West

    R

    80. (5) From statement I

    for

    NW

    South

    It is clear from the diagram that point M is in North–West direction of point A. 86. (1) From statement I M is mother of A, B and C. B is the husband of R. R is daughter–in–law of Q. Therefore, Q is the father of A, B and C. From statement II A, B and C are children of M and Q 87. (2) From statement I,

    R

    Row – 1

    OR going to

    start

    gi

    bs

    la

    81. (5) From Statement I Ranjay may be brother or cousin of Parvati. From statement II No relation can be established between Ranjay and Parvati. From both the statements Ranjay is the son of Parvati’s Aunt. Ranjay has no siblings. So, Parvati is child of maternal uncle of Ranjay. Thus, Ranjay is cousin of Parvati. 82. (5) From statement I

    S

    P

    Q

    Q

    A S

    S

    R

    Row – 2

    M

    From statement II

    Row – 1

    M

    Z

    M 3m

    M, H From statement II

    R, S

    R

    A

    85. (2) From statement I

    > R > D

    H, D > M

    P

    A/C P

    B R

    A/C Q

    8m 5m

    Row – 2 It is clear that B faces R.

    A

    BPRE–937

    DATA SUFFICIENCY 88. (5) From statement I B studies in 1st or 7th standard C studies in 4th or 8th standard From statement II

    From statement II

    M

    C>B>A>

    5m From both the statements 10th 8th 7th 4th 1st D/E C B A D/E 89. (4) From statement I

    93. (1) From statement I T R

    P

    R

    5m A

    9m

    Q

    S P is to the immediate right of T. From statement II P S/T

    North R

    NW

    NE East

    West

    Q

    S From statement II R

    R

    OR P

    Q

    Q

    P

    SE

    SW

    Q

    South

    It is clear from the diagram that point M is in North–West direction of point A. 91. (1) From statement I M is mother of A, B and C. B is the husband of R. R is daughter–in–law of Q. Therefore, Q is the father of A, B and C. From statement II A, B and C are children of M and Q 92. (2) From statement I,

    Row – 1 S

    S

    A

    90. (2) From statement I

    R

    A

    Row – 2 From statement II

    Row – 1 Z

    M 3m

    8m

    A/C P

    B R

    A/C Q

    5m A

    Q/R

    Q/R 94. (5) From statement I P has two dresses less than what T has. No answer. From statement II M has seven dresses and T has 10 dresses. From both the statements P has 10 – 2 = 8 dresses 95. (5) From both the statements

    Thanks for not smoking → be je we no No smoking area → no se do 96. (4) From statement I C, D > B From statement II C>A>E From both the statements C>A>E

    C, D > B A>E

    R

    M

    M

    S/T

    Row – 2 It is clear that B faces R.

    BPRE–938

    97. (2) From statement I C is the brother of B and A. It is not clear whether they are children of P or not. From statement II P has three children of which B is a girl. Therefore, P has two sons and one daughter. 98. (3) From statement I,

    L S E F T

    Q

    R I G H T

    There are five persons in the line.

    DATA SUFFICIENCY From statement II,

    From statement II

    Row–1

    R I G H T

    P Q M

    L E F T

    From statement II,

    R

    There are five persons in the line.

    S

    C/B

    B

    Row–2 From both the statements

    From both the statements

    R

    Row-1

    P M

    East

    SE South

    From statement II,

    4m

    7m

    M

    4m R

    Q

    4m

    Point M is 7 metre away from point S. 100. (4) From statement I

    T







    Row-2 C faces S. 104. (2) From statement I S/T

    20 metre

    R

    20 metre

    C

    G

    20 metre



    S/T

    K

    North NW

    16 11 8 2 From statement II, K > J, N M >J

    NE

    West

    East

    ↓ 23 103. (5) From statement I,

    D/E

    D/E

    P

    Q

    S

    SE

    SW South

    Row–1

    P

    From statement II

    D/E

    D/E

    R

    T

    M

    S

    G

    N Q

    A

    R

    30 metre

    NE

    S

    E

    S

    15 metre

    North

    SW

    C

    Q

    N 101. (1) From statement I, K is the mother of M. M is the mother of T. D is the daughter of T. F is the father of D. Therefore, T is the mother of D. Thus, T is the granddaughter of K. From statement II, Z is the son of T. The sex of T is not known. K is the grandmother of Z. Thus, T is either son or daughter of K. 102. (2) From statement I, N > J > L > O

    15 metre

    point S.

    West

    B

    P S

    Point M is 7 metre away from

    NW

    D Q

    T

    S

    P

    Row–2

    BPRE–939

    20 metre

    C

    10 metre

    M

    10 metre

    T

    15 metre

    Q

    4m

    A

    N

    M

    X

    A

    M

    99. (3) From statement I,

    7m

    C

    20 metre

    J

    Point T is in North–East direction of point G.

    DATA SUFFICIENCY 105. (4) From statement I V and S are brothers of Q. M is the father of V, S and Q. P is the son–in–law of M. Therefore, P is the husband of Q. From statement II,

    P is married to Q. A is the mother–in–law of Q. A is the mother of E. G is the only sister of E. Therefore, P is the son of A. Thus, P is the husband of Q.

    106. (1) From statement I

    fa ju

    monitor quality of goods

    mp fa cd ju

    ra ti

    T

    From statement II

    for

    uranium

    plant

    na

    access

    ti vo ku

    vo zy ku

    Q

    Floor Number Floor Number 5

    E

    Floor Number 4

    B

    Floor Number 3

    A

    Floor Number 2

    C

    Floor Number 1

    D

    Exam (I) R U S Q T

    Person

    (I)

    (II)

    Floor Number 5

    B

    E

    Floor Number 4

    E

    B

    Floor Number 3

    A

    A

    Floor Number 2

    D

    C

    Floor Number 1

    C

    D

    108. (5) From statement I

    Colour (I) Blue Red Yellow

    Colour (II) Yellow Red Blue

    Colour Box Red B Blue A Yellow C

    OR

    Exam

    (I)

    (II)

    January

    T

    R

    Colour

    Box

    February

    Q/S

    U

    Blue

    B

    Red

    C

    September

    U

    Q/S

    Yellow

    A

    December

    R

    T

    Month

    Exam

    Exam

    (I)

    (II)

    January

    R

    T

    February

    U/S

    Q

    July

    U/S

    U/S

    September

    Q

    U/S

    December

    T

    T

    112. (3) From statement I Mathematics Sociology Physics

    A C B

    113. (1) From statement I, Colour Box Blue A Red C Yellow B

    12 students

    110. (2) From statement I Class-I Class-II K K M L M L

    BPRE–940

    5 students 13th Yugal Prakash

    The position of Yugal is 19th from the left. From statement II, 13th Prakash

    16th Sonam

    19th 6 students Yugal

    (or) 6 students

    Two boxes are kept below the blue covered box.

    Day Monday Tuesday Wednesday Thursday Friday

    B

    Mathematics Sociology Physics

    12 students

    Q/S

    From statement II

    Box B/C B/C A

    From both the statements

    Q/S

    S

    From statement II

    From statement II

    Exam

    July

    T R Q Exam (II) T Q S U R

    109. (2) From statement I

    Person

    Month

    Month January February July September December

    From statement II

    T

    From statement II From both the statements

    Person

    P

    na

    Thus, the code for ‘import’ is ‘ti’. 107. (3) From statement I

    Floor Number

    Q P

    or

    uranium

    plant for

    Class L/N K J M L/N

    111. (2) From statement I

    import high quality goods

    import

    From statement II Day Monday Tuesday Wednesday Thursday Friday

    7th Yugal

    10th Sonam

    13th 12 students Prakash

    From the left the position of Yugal will be either 19th or 7th. Hence, only statement I alone is sufficient to give answer while statement II alone is not sufficient to give answer. 114. (3) From statement I, Tejas

    3 metre

    4 metre Mohan 3 m Sohan

    3m

    6 metres

    Om 4 metre

    Narendra

    DATA SUFFICIENCY Clearly; Mohan, Sohan and Narendra are in a straight line. Hence, statement I alone is sufficient. From statement II, Prem

    Hence; Sunday

    H µ 25 [S = 19th from left] Place value [H = 19th from the right] 116. (2) From statement I

    3 metre

    U

    Kaushal 3 metre Ramesh 3 metre 3 metre

    Mohan

    3 metre 3 metre

    Sohan

    Narendra

    Clearly; Mohan, Sohan and Narendra are in a straight line. Hence, statement II alone is sufficient. 115. (3) From statement I, Place value in alphabets

    For giveness

    The next (+1) letter

    6 Þ 6 × 2 = 12 T Place value in alphabets

    i s

    (+1) Next letter

    9 Þ 9 × 2 = 18 T

    Virtue

    From statements II

    Hence, Place value in alphabets Place value in alphabets

    Sunday

    117. (2) From statement I K is the daughter of N and T. B is the sister of N. K is the granddaughter of S. S is either father or mother of N or T. Therefore, B is either daughter of S or sister of son–in–law or daughter–in–law of S. From statement II N is the wife of T. B and N are daughters of S. 118. (2) From statement I

    19 Þ 19 × 2 = 38Z

    13th letter from left 9th letter from left 9th letter from right 18th letter from left 18th letter from right

    >C>

    P 2m S

    >F>

    ,

    5m

    M

    s

    Place value R µ 19

    D

    ,

    West

    East

    , South

    E A is the second heaviest. 119. (1) From statement I |PUWIT Thus, these are six persons in the line.

    U

    Point M is 10 metre away from point K. From statement II 8m 8m 2m M D K Point M is 10 metre away from point K.

    121. (3) From statement I

    lo ko ni sa

    celebrate festival

    jo to ni fa

    festival

    light bright

    surprise candle shine

    From statement II

    right Place value I µ 20

    bi ya la fa

    surprise high celebration delight

    ya la fa ni

    surprise festival high delight

    BPRE–941

    K

    North

    B>A>F>C>D

    Place value M µ 20

    8m

    2.5m

    ,

    ,

    ,

    O

    2.5m

    B>A>F D is not the heaviest. From statement II

    B>A>

    Might

    i

    ,

    From statement II S | U -------- W | T 120. (3)

    F>C>D

    From statement II,

    13th letter from left

    B

    H/U

    Thus, three persons are standing between H and U.

    ,

    (+1)

    K

    H/U

    ,

    Place value 22 Þ 22 × 2 = 44 F

    U

    H/K M H/K

    DATA SUFFICIENCY or

    A

    From both the statements

    Bulbul/Sparrow

    E/C D

    B

    E/C

    From both the statements

    D

    F

    123. (5) From statement I >

    >

    >F>

    Bulbul

    Parrot

    Myna

    Parrot

    or

    Bulbul

    C

    B From statement II C/D Bulbul/Sparrow

    Bulbul/Sparrow

    A D E

    It is clear that, D sits second to the right of C. 128. (2) From statement I > O > P, M, From statement II L>O>N>P M>L>O>N>P Thus, bag P is the lightest. 129. (4) From statement I Day Monday Tuesday Wednesday Thursday Friday

    Event (I) B C D E A

    Event (II) C B E D A

    From statement II Day

    Event Event Event (I) (II) (III) Monday B/C B/C B/C Tuesday B/C B/C/E B/C/E Wednesday D C/E D Thursday A D C/E Friday E A A 130. (3) From statement I

    R I G H P T

    Row-2 Q

    S

    R

    B

    D

    A

    L E F T R I G H T

    Row-1

    From statement II We get the same arrangement as obtained from the data given in the Statement I. Thus, C faces P.

    B

    Sparrow

    Bulbul

    B

    L C E F T

    A

    From statement II

    or

    Sparrow

    Person Q P/S R P/S T

    126. (5) From statement I According to the Class teacher the number of students in her class = 22 to 31 which is divisible by 6 The numbers between 22 and 31 which are divisible by 6 = 24 and 30 From statement II According to Principal the number of students in class 8 = 33 – 8 = 25; 33 – 7 = 26; 33 – 6 = 27; 33 – 5 = 28; 33 – 4 = 29; 33 – 3 = 30; 33 – 2 = 31; 33 – 1 = 32 From both the statements The number of students in class 8 of School K = 30 127. (5) From statement I

    E>C>A From statement II A > F, B E is not the tallest. From both the statements D>E>C>A>F>B E is the second tallest. 124. (5) From statement I

    Myna

    Crow

    Floor Number 5 4 3 2 1

    A

    >

    R I G H T

    125. (4) From both the statements

    C

    E

    Bulbul

    B

    Myna

    L E F T

    Parrot

    F

    From statememt II

    C

    Bulbul/Sparrow

    122. (4) From statement I

    131. (2)

    North NW

    NE

    B A

    East

    West SW

    C/D E

    BPRE–942

    South

    SE

    DATA SUFFICIENCY From statement I

    133. (1) From statement I

    B

    B

    Box C 3m

    F

    6m

    C

    9m

    B

    8m

    C 8m

    R

    AB =

    bBDg 2

    A

    D 3m

    8m 2

    b8g

    =

    64 + 9 =

    + (AD)

    L E F T

    Box E

    Box E

    Box B

    Box A

    L

    B

    F

    H

    E

    North NW

    G

    R I G H T

    H

    L E F T

    D

    From statement II

    A

    E

    B

    H

    West

    East SE

    SW South

    C

    R I G H T

    G

    E

    G

    Q

    O

    L M

    N F

    R I G H T

    P

    It is clear that Point Q is in the North-West direction of the Point P. 137. (5) From statement I A E

    From both the statements

    F

    L E F T

    H

    E

    C

    D

    G

    R F I G H T

    C sits second to the left of G. 135. (3) From statement I

    C

    NE

    From statement II

    From statement II

    C/E

    Q

    N

    M

    or L E F T

    P

    Box D Box B

    C/E

    Q

    O

    or

    C

    E

    D

    C sits second to the right of A.

    he

    also

    ty nc

    kc

    O

    134. (5) From statement I

    73 metre

    lm

    136. (2) From statement I

    Box C

    Box A

    2

    Point B is in North-West direction with respect to the Point A. 132. (2) From statement I

    A



    Box C

    Box D

    + (3)2

    =

    she visit the always

    From statement II

    B

    lm nc ty

    → sx

    B ox A

    From statement II M

    she visit the also

    B ox D

    G

    6m

    ya

    From statement II

    B ox E

    3m B

    also

    → zb ct sx

    B ox B

    6m

    A

    now many person

    B/D B/D F

    C or B/D A

    C

    show data E

    → sx

    BPRE–943

    fa mn ca

    F

    DATA SUFFICIENCY 142. (2) From statement I

    From statement II

    A

    D

    Q

    T

    145. (1) From statement I

    R I G H T

    P

    E

    B/C

    or

    B/C

    F

    From both the statements E A

    B

    D

    F C C is sitting to the immediate left of F. 138. (5) From statement I Shravan’s birthday is in October or December. From statement II Shravan’s birthday is in February, March, April, May, June, July, August, September, October or November From both the statements Shravan’s birthday is in October. 139. (2) From statement II D was born on 24th February. 140. (5) From both the statements Floor number 6 5 4 3 2 1

    Person E/A B C F E/A D

    141. (2) From statement I

    Monday Tuesday

    K

    Wednesday

    M

    Thursday

    L

    M L

    Friday

    P

    From statement II Day

    Classes

    Monday Tuesday Wednesday Thursday Friday

    L/N K J M L/N

    X P

    O

    L E F T

    T

    R Q

    T

    z M R I G H T

    N

    Row-2 or

    From statement II

    Row-1

    P S

    X P

    143. (3) From statement I

    N

    z M

    O

    Mathematics Sociology Physics

    B

    Row-2

    Clearly, M faces Z. From statement II Row-1

    From statement II

    Mathematics

    A C

    X P

    Sociology Physics

    B

    Y N

    144. (2) From statement I >

    >M>

    >

    W O

    Z M

    Row-2 or

    >

    Row-1

    J>M>L>N O>K O, J > M > L > N

    X M

    K

    Y P

    Z N

    W O

    From statement II >K>

    >

    >

    >

    M>J>L

    Classes I II K

    Day

    Q

    L E F T

    Row-1

    M is not the heaviest.

    Row-2 146. (3) From statement I or statement II

    L is not the lightest. O/N > K > M > J > L > O/N L is the second lightest.

    Box Box Box

    V U W

    Yellow Blue Orange

    147. (4) From statement I

    make silver ornaments today

    ly xt sj br

    silver ornaments for ceremony

    br mp zl ly

    From statement II

    ornaments in silver only only today for preparation

    BPRE–944

    qf br hk ly mp hk ax sj

    DATA SUFFICIENCY

    silver

    only

    qf

    for

    mp

    preparation

    hk

    ly

    hk ax sj

    silver ⇒ ly/br 148. (3) From statement I Day Monday Tuesday Wednesday Thursday Friday Saturday Sunday

    Play

    2. (2) From statement I A C

    A E B D

    D

    B

    E

    From statement II Day

    Play

    Monday

    F/G

    Tuesday

    C

    Wednesday

    A

    Thursday

    E

    Friday

    B

    Saturday

    D

    Sunday F/G 149. (3) Only five persons availed offer 4. Therefore, offer 4 may be discontinued in the next month. Now, we can eliminate Options (1) and (5). The number of orders from Friday to Sunday remained unchanged. It is the offer 3 and it may also be discontinued in the next month. Only 12 persons availed offer 2. Therefore, offer 2 may be discontinued in the next month. 150. (2) Obviously, all the given instances can be reasons for the incorporation of new rules.

    SBI PO EXAMS 1. (3) From statement I Q > R, S Q > R, S > T >P Thus, the bag Q is the heaviest. From statement II Q > R > S, T, P Thus, the bag Q is the heaviest.

    It is not clear A, D and C are facing the centre or outward. From statement II C

    Sandeep

    today

    br

    Bhavna is the fourth from the left end. From statement II

    Bhavna

    in

    zl ly

    mp

    B

    Sheetal

    br

    br

    Sheetal

    ceremony

    sj

    xt

    Bhavna

    ornaments

    ly

    for

    silver ornaments

    only

    today

    silver ornaments

    make

    Thus, Chemistry was not taught on Wednesday. 4. (2) From Statement I At 9.30 the minute and the hour hands will make an angle slightly more than 90° From statment II At 8.45, the hour and the minute hands would coincide with each other. 5. (4) From statement I B is the father of M and T. B is the grandfather of F. The gender of F is not known. From statement II There is no mention of B From both the statements The gender of F is not known. 6. (1) From statement I

    Anita

    From both the statements

    7. (3) From statement I A

    R

    A

    N

    G

    E

    From statement II D

    G

    N

    A

    A

    N

    G

    E

    A is facing outward. 3. (4) From statement I Day Monday Tuesday Wednesday Thursday Friday

    Subject Botany Mathematics Physics Chemistry Zoology

    Thus, Chemistry was not taught on Wednesday. From statement II Day

    Subject

    Subject

    Monday

    Botany/ Zoology

    Botany/ Zoology

    Tuesday

    Mathematics Chemistry

    Wednesday Physics

    8. (5) From statement I U V/T

    X

    S

    V/T W From statement II

    T U/W

    V

    Mathematics

    Thursday

    Chemistry

    Physics

    Friday

    Botany/

    Botany/

    Zoology

    Zoology

    X

    U/W S

    BPRE–945

    DATA SUFFICIENCY From both the statements

    From statements I and III

    U

    P

    X

    T

    N

    S

    R

    It is clear from the diagram that village P is to the North-East of village R. 11. (3) From statement I The position of Nirmal from the top = 17 – 13 + 1 = 5th

    Q

    4

    V W

    T is second to the left of X. 9. (2) From statement I R, T > P, Q From statement II V > R, T, P, Q, S From statement III Q>S>P From statements I and II V > R, T > P, Q From all the three statements V > R, T > Q > S > P Clearly, S secured the second lowest marks. 10. (5) From statement I

    NW

    Q

    North NE

    West SW

    S

    East

    South

    SE

    12

    From statement II

    B

    K

    From statement III

    P

    The rank of Kamal = 15th Bhumika is two position below Animesh From statements II and III

    Q

    8

    9th A

    11th B

    15th 2 K

    The rank of Animesh is 9th.

    T

    12. (1) From statement I tell

    them young

    wise young sharp tell

    → se me ye → me yo na ye

    From statement II

    There is no information about the village R in the statement I. From statement II

    clever sharp come tomorrow

    bring clever young them From statement III

    → na ki pa lo → ki po se ye

    P

    Q There is no information about the village R in the statement II. From statement III N

    R

    11th 6 B

    S

    P

    N

    N

    7

    9th 8 A

    The rank of Animesh from the either end is 9th.

    S It is clear from the diagram that village P is to the North-East of village R. From statements II and III T

    5th N 6 13th

    R

    Q

    clever sharp come them no

    pa na se ki te

    yellow come sharp run clever no

    ki ni pa be te na

    It is clear that ‘se’ stands for ‘them’. 13. (3) From statement I P is the wife of H. P is the mother of T. H is the father of T.

    S There is no information about the village R in the statements I and II. Therefore, we cannot arrive at the answer even with the statements I and II taken together.

    14. (1) From statement I

    H > F > D > G , E

    T is the wife of D.

    Clearly, H is the tallest and F is the second tallest.

    T is the mother of B.

    From statement II

    Therefore, H is the grandfather of B.

    H > F

    From statement II H is the father of T and N. B is the daughter of T. Therefore, H is the grandfather of B.

    BPRE–946

    D > G > E No answer.

    DATA SUFFICIENCY 15. (3) From statement I

    J

    North

    G

    NE

    NW West SW

    R

    P

    East South

    SE

    It is clear from the diagram that J to is the north of R. From statement II

    From statement II A is mother-in-law of H. L is father-in-law of H. C is son of A and L. Therefore, C is brother-in-law of H. 20. (3) From statement I

    2m

    B

    P

    3m

    A

    B D C/E

    N

    7m

    7m

    J

    From statement II

    A

    C/E

    From both the statements

    B O

    R

    M

    6m

    D

    North

    K

    H

    West

    It is clear from the diagram that J is to the north of R. 16. (2) From statement I The number of students may be 4, 6, 8 or 10. From statement II

    C

    B

    K

    O

    It is clear that there are 10 students in the row. K is third from the right. 17. (1) From statement I Arvind visited Mumbai on Tuesday or Wednesday. Wednesday is an odd day. Therefore, Arvind visited Mumbai on Tuesday. From statement II Arvind visited Mumbai on Tuesday or Wednesday or Thursday. 18. (3) From statement I shine was peeled off

    &

    #

    @

    9

    no paint but shine

    7

    5

    #

    8

    C East

    South It is clear from the diagram that Point A is to the east of Point B. From statement II From the information given in statement II, we get the same diagram given above. 21. (2) From statement I Five to nine students scored more than that of B. The rank of D is 8th from the top. From statement II Rank of C from the beginning = 25 – 16 + 1 = 10th

    10th E

    7th B

    Thus, six students scored more than that of B. 22. (5) From statement I

    B From statement II try

    the

    new shine

    1

    3

    #

    0

    we

    try

    the new

    6

    0

    1

    3

    19. (3) From statement I C is father of Z. Son of H is N, who is cousin of Z. Therefore, C is brother of H.

    A

    E

    D C

    A/E

    A/E

    BPRE–947

    C is sitting second to the left of A. 23. (2) From statement I A>D>C There is no information about B and E. From statement II B is heavier only than E. Therefore, A, C and D are heavier than B and E. Thus, E is the lightest. 24. (5) From statement I According to his mother, Rajiv was born on 13th, 14th, 15th,16th or 17th May From statement II According to his father, Rajiv was born 17th, 18th, 19th, 20th, 21st or 22nd May From both the statements Common date ⇒ 17th May 25. (1) From statement I Sonal has only one brother as her mother has only two children. From statement II It is not clear how many brother does Sonal have. 26. (4) From statement I

    M

    L

    L

    M

    DATA SUFFICIENCY Floor Number 6 5 4 3 2 1

    North West

    East South

    L

    Person A

    D

    A lives on the topmost floor. So, no one lives above the floor of A. 30. (3) From statement I

    M

    T

    P

    M

    L Y sits exactly in the middle of the line. From statement II X Y V

    From statement II

    K

    M

    L

    P

    P

    L

    M

    K

    S and

    R Q

    so on. 27. (3) From statement I come late to office

    so ti ly ja

    late in the night

    fo pa ti me

    P is sitting third to the right of Q. From statement II R

    S

    From statement II

    ti le di co

    reached two hours late

    28. (5) From both the statements.

    L E E F T

    L E F T

    Row-1

    P

    Q

    S

    T

    B

    A

    D

    C R I G H T

    Row-2

    T P is siting third to the right of Q. 31. (3) From statement I

    D NW

    A

    NE East

    SW

    South

    C

    SE

    It is clear from the diagram that Point C is in Northeast direction with respect to Point A. From statement II

    Person

    C

    1m

    B/E

    North

    West

    29. (3) From statement I 6 A 5 4 3 B 2 1 A lives on the topmost floor. So, no one lives above the floor of A. From statement II

    C

    5m

    5m

    Rs. 80,000 From both the statements It is not possible to determine that R earns more than Rs. 40,000. 35. (5) From statement I

    A

    5m

    5m

    Clearly, R is sitting second to the right of Q and is facing E. Floor Number

    S, T

    P

    B

    Y sits exactly in the middle of the line. 34. (2) From statement I P > Q > S > R, T It is not clear R earns more or less than T. From statement II

    P>Q>R

    Q

    ru ki ne ti

    It was late summer

    R I G H T R

    It is clear from the diagram that Point c is in North-east direction with respect to Point A. 32. (1) The statement affirms that 32 per cent villages have 24 hours electricity. It does not imply that other villages have no access to electricity at all. Conclusion II may be true. 33. (1) From statement I Z V Y W X

    B

    6m

    B/E F

    D sits to the immediate left of C. From statement II B

    D E

    A

    F

    D

    9m

    A

    4m

    BPRE–948

    C

    DATA SUFFICIENCY It is clear that all the five persons are not facing the centre. 40. (3) From statement I P is the father of A and B. R is unmarried and C is sisterin-law of R. P is brother of R. So, P is the husband of C. Now, it is clear that C is the mother of B. From Statement II P is the father of A and B. H is the father of P and R. H is the father-in-law of C. R is unmarried. So, C is the wife of P. Now, it clear that C is the mother of B. 41. (5) From statement I

    38. (1) From statement I

    or

    F

    G

    J

    H

    I

    G

    F

    F E

    C

    B

    D

    or

    I

    J

    I H From statement II

    A

    J

    H

    G

    F

    Either D or F sits to the immediate left of C. 36. (3) From statement I

    success is

    necessary evil

    I

    F

    H

    J

    Day Monday Tuesday Wednesday Thursday Friday

    or

    8 1 4 5 F

    I

    evil is amongst us 3 8 7 4 The code for ‘success’ is either ‘1’ or ‘5’. From statement II

    marketing is

    magic mantra 4 6 2

    success

    9

    F

    G

    It is clear from the statement II that all the five friends are not facing north. 39. (3) From statement I D E

    mantra is marketing

    Floor Number 5 4 3 2 1

    From statement II Day Movie Monday S Tuesday T Wednesday Q Thursday P Friday R

    Movie R P Q S T

    From both the statements

    2 5 6 4 The code for ‘sucess’ is ‘5’. 37. (3) From statement I

    Movie R P Q/T S Q/T

    A/B

    A/B

    Person L K J H/I H/I

    C It is clear that all the five persons are not facing the centre. From statement II A C

    Day Monday Tuesday Wednesday Thursday Friday

    Movie R P Q S T

    Movie T will be screened on Friday. 42. (3) From statement I

    North

    From statement II Floor Number

    Person

    5

    L

    4

    K

    3

    J

    2 1

    H I

    B

    E

    D

    BPRE–949

    West

    East

    South

    DATA SUFFICIENCY A lives on the topmost floor. So, no one lives above the floor of A.

    P

    From statement II

    From statement II Floor Number

    15 m 4m

    C

    7m

    7m Q

    Point R is 15 metre from the point P. From statement II

    B

    6m

    D

    Person

    6 5 4 3 2 1

    R

    1m

    A

    9m

    D

    A

    A lives on the topmost floor. So, no one lives above the floor of A. 45. (3) From statement I

    T

    P

    4m

    It is clear from the diagram that Point c is in North-east direction with respect to Point A. 47. (1) From statement I F G J H I

    R or

    S

    27 m

    I

    J

    G

    F

    R Q

    5m 14 m

    N

    I H From statement II

    P is sitting third to the right of Q.

    3m

    J

    H

    G

    From statement II

    P

    14 m

    R

    S

    Point R is 35 metre away from point P.

    L E E F T

    P

    Q

    S

    T

    B

    A

    D

    C R I G H T

    Row-2

    T 46. (3) From statement I B

    A

    5m

    J

    C

    E

    North NW

    5m

    G

    D

    5m

    NE

    West A

    B

    H

    It is clear from the statement II that all the five friends are not facing north. 48. (3) From statement I

    5m

    44. (3) From statement I 6 5 4 3 2 1

    I

    F

    P is siting third to the right of Q.

    right of Q and is facing E. Person

    F

    P

    Clearly, R is sitting second to the

    Floor Number

    F

    Q

    L E F T

    Row-1

    I or

    43. (5) From both the statements.

    R I G H T R

    F

    SW

    East South

    SE

    It is clear from the diagram that Point C is in Northeast direction with respect to Point A.

    BPRE–950

    A/B

    A/B

    C

    DATA SUFFICIENCY It is clear that all the five persons are not facing the centre. From statement II

    A

    C

    B

    E

    From both the statements Day

    Movie

    Monday

    R

    Tuesday

    P

    Wednesday

    Q

    Thursday

    S

    Friday

    T

    Movie T will be screened on Friday. 51. (3) From statement I

    North

    D It is clear that all the five persons are not facing the centre. 49. (3) From statement I

    West

    East

    P is the father of A and B. R is unmarried and C is sisterin-law of R. P is brother of R. So, P is the husband of C.

    South

    P

    Now, it is clear that C is the mother of B.

    15 m

    From Statement II P is the father of A and B. H is the father of P and R. H is the father-in-law of C.

    4m

    R is unmarried. So, C is the wife

    7m

    of P. Now, it clear that C is the mother of B. 50. (5) From statement I Day

    Movie

    Monday

    R

    Tuesday

    P

    Wednesday

    R 7m Q

    Point R is 15 metre from the point P. From statement II

    52. (1) From statement I : T is wife of W. P is grandson of T and W. Only sister of P is the mother of V. Thus, granddaughter of T and W is the mother of V. Therefore, V is great grandmother of T. 53. (4) From statement I (Suppose the two digits are x and y) x × y = y2 – [y + (x + 1)] ⇒ xy = y2 – x – y – 1 There are two variables and hence two equations would be required to get the value of x and y. From statement II (x + y) > 2 (x – y) It is an in equality and not an equation. 54. (2) From statement I The gender of P and Q is not known. R is a male member. U is a female member. Either S or T is a female member. Therefore, the data in statement I alone are not sufficient to answer the question. From statement II Three members are females. Thus, the data in statement II alone are sufficient to answer the question. 55. (4) From statement I

    North

    R

    West

    Q/T

    Thursday

    S

    Friday

    East

    27 m

    South

    Q/T

    From statement II Day Monday

    E

    Movie

    Movie

    S

    R

    Tuesday

    T

    P

    Wednesday

    Q

    Q

    Thursday

    P

    S

    Friday

    R

    T

    5m N

    4 km.

    14 m A

    3m P

    14 m

    Point R is 35 metre away from point P.

    BPRE–951

    O B

    5.5 km. 2.5 km.

    C 3.5 km. D

    It is not clear the destination is in which direction and how far from the the point E.

    DATA SUFFICIENCY From statement II

    or

    The distance is given as 3.5 km. But, there is no information about direction.

    G

    F

    C D

    Hence, the data in even statements I and II together are not sufficient to answer the question. 56. (5) From statement I U>P>S R, T > U T>V>R T>V>R>U>P>S The height of Q is not given. From statement II Q is the shortest. From both the statements T>V>R>U>P>S>Q Thus, U is in the middle.

    B

    A

    B

    B A

    E

    From statment II

    Data in all the three statements I, II and III together are not sufficient to answer the question. 60. (3) The data given in statements II and II together are sufficient to answer the question.

    D

    C

    57. (1) From statement I x – 1 + x – 1 + x = 49 ⇒ 3x = 49 + 2 ⇒x=

    61. (5) From statement I M > N, K O>J>M L>K

    E

    51 = 17 3

    ∴ Costliest article is worth Rs. 17.

    O > J > M > N, K

    B

    From statements II and III together we can calculate the price of costliest article.

    F

    L From statement II

    or

    58. (4) From all the three statements Distance between Patna and Delhi

    >

    F

    D

    The train will reach Delhi at 8.00 AM + 5 : 45 : 6

    Floor Number

    C

    = 13 : 45 : 6

    B

    = 1 : 45 : 6 PM 59. (5) From statement I

    From statement III

    A

    B

    >

    Person (I) (II)

    6 5 4 3 2 1

    E

    G

    C

    >

    J>K From both the statements O > J > M > N, L > K or, O > J > M > L > K, N Clearly, O is the tallest. 62. (5) From statement I

    = 240 × 6 = 1440 km At the speed of 250 km per hour the train would cover 1440 km in 5 hours 45 minutes 6 seconds.

    >M>

    W R T

    W R T

    From statement II

    F E or

    B

    D

    B A

    BPRE–952

    Floor Number 6 5 4 3 2 1

    (I) S T V R W U

    Person (II) (III) S U T V W R V W R T U S

    (IV) U W V R T S

    DATA SUFFICIENCY From both the statements Floor Number 6 5 4 3 2 1

    RBI GRADE–B/NABARD GRADE–A OFFICER EXAMS

    Person U W V R T S

    1. (5) From both the statements Q

    63. (1) From statement I L E F T

    D/F

    B

    A

    D/F

    C

    R I G H T

    E

    P

    T

    R

    E

    D

    B

    A

    C

    E

    F

    B/C

    A

    B/C

    E sits to the immediate right of A. From statement II C

    or

    D

    B

    C

    E

    F

    A

    Day

    Exam

    Monday

    T/U

    Tuesday

    Q/U

    Wednesday

    Q

    Thursday

    T

    Friday

    P

    Saturday

    V/S

    Sunday

    V/S

    From statement II

    D R I G H T

    S

    T sits exactly in the middle of the line. 2. (1) From statement I

    C sits second from the right end. From statement II L E F T

    6. (2) From statement I

    Day

    Exam

    Monday

    U

    Tuesday

    Q

    Wednesday

    T

    Thursday

    R

    Friday

    P

    Saturday

    S

    Sunday

    V

    7. (4) From statement I

    D/E

    D/E

    O B

    or

    D

    A

    C

    E

    F

    B

    they

    live

    for

    gn

    mu

    sy

    fd

    go

    now

    run

    for

    sy

    mo

    lt

    gn

    go ht

    there

    get

    ready

    mo

    ga

    sx

    now

    there

    fall

    za

    ga

    gn

    R

    O

    J

    OU

    L

    9 O J OU R 8. (3) From statement I R is daughter of X’s brother. So, X is uncle of R. From statement II P is the husband of D. P is the brother of X. R is the daughter of D and P. So, X is the uncle of R.

    E A S T 5. (1) From statement I D is the uncle of M. Therefore, C is the grandmother M. From statement II F is the aunt of S and M. Therefore, C is the grandfather of grandmother of M. The gender of C is not clear. 9. (2) From statement I

    U

    H/K M H/K

    H/U

    K

    U

    H/U

    B

    Thus, three persons are standing between H and U.

    BPRE–953

    9

    or

    From statements II go ⇒ mo there ⇒ ga now ⇒ gn

    5

    From statement II

    3. (4) From both the statements Q 4. (5) From both the statements

    now

    O Y

    A

    R>P>T>S

    64. (2)

    T

    DATA SUFFICIENCY 10. (2) From statement I K is the daughter of N and T. B is the sister of N. K is the granddaughter of S. S is either father or mother of N or T. Therefore, B is either daughter of S or sister of son–in–law or daughter–in–law of S. From statement II N is the wife of T. B and N are daughters of S. 11. (2) From statement I ,

    ,

    >C>

    ,

    B>A>F D is not the heaviest. From statement II ,

    >F>

    U

    8m

    8m

    M

    2m

    D

    K

    16. (1) From statement I, K

    Point M is 10 metre away from point K.

    M Three persons

    14. (5) From statement I plan it now

    ta na se

    a plan now

    na ta ra

    L

    Thus, there are seven persons. From statement II

    From statement II

    N

    plan is good

    zi ju na

    good to plan

    fe ju na

    K

    From both the statements

    ,

    ,

    ,

    plan it now

    ta na se

    a plan now

    na ta ra

    plan is good

    zi ju na

    Either four or six persons are there. 17. (3) From statement I

    B>A>F>C>D

    good to plan

    E A is the second heaviest. 12. (1) From statement I |PUWIT Thus, these are six persons in the line. From statement II S | U -------- W | T 13. (3) P

    5m

    M

    8m

    D

    2.5m K

    15. (5) From statement I ,

    East

    Floor Number 5

    F

    >P>

    A

    Floor Number 2

    T>Q>S

    Floor Number 1

    T>Q>S>P>R

    From statement II,

    or, R > T > Q > P > S

    Floor Number

    From statement II

    Floor Number 6

    ,

    E Person D

    Floor Number 5

    ,

    Floor Number 4

    B

    Floor Number 3

    A

    From both the statements

    Floor Number 2

    C

    T>Q>S>P>R

    Floor Number 1

    E

    18. (5) From statement I,

    a West

    Person

    Floor Number 6

    Floor Number 3 ,

    help

    North

    Floor Number

    Floor Number 4

    T>P

    O

    2.5m

    fe ju na

    >Q>

    2m S

    M

    K

    ,

    ,

    N

    M or

    F>C>D B>A>

    M

    find a

    small

    buyer

    help given

    ni

    ka

    po sn

    tk

    sn

    ru

    ni

    The code for ‘buyer’ is either ‘ka’ or ‘po’. From statement II,

    South

    Point M is 10 metre away from point K. From statement II

    must find

    find small buyer a

    help soon

    BPRE–954

    ka

    hp tk

    po

    mj

    ni

    ka

    sn

    DATA SUFFICIENCY From both the statements

    23. (1) From statement I S

    help

    a

    find

    small

    must

    a

    buyer

    help

    find

    ni

    given

    ka

    tk

    small buyer

    po

    sn

    P

    R

    U

    T

    sn ru ni

    ka

    Q

    hp tk po

    From statement II

    find

    a

    help

    soon

    mj

    ni

    sn

    ka

    T

    Thus, the code for ‘buyer’ is ‘po’.

    Q lives on floor number 3. From statement II Floor Number

    Person (A) Floor Number 5 M/P Floor Number 4 N Floor Number 3 Q Floor Number 2 M/P Floor Number 13 O

    Person (B) O M/P Q N M/P

    Q lives on floor number 3. 20. (4) From statement I, ,

    >A>

    ,

    21. (5) From statement I, ↑K↑↑↑DTY↑A or ↑K↑↑DYTA From statement II ↑↑Z↑DT↑A↑ or ↑ ↑ Z D T A ↑ From both the statements ↑KZ↑↑DTY↑A↑ 22. (3) From statement I

    S

    ,

    >D>

    E

    5 metre

    M

    5 metre

    B

    L

    10 metre North

    NW

    P

    OR T

    Q

    4 metre

    NE

    D

    East

    West

    ,

    E > B, C E>F>C E > F > B, C or, E > B, F > C From statement II ,

    2 metre

    SE

    SW South

    Point M is 7 metre to the east of point S. From statement II ,

    E>A>D B>C Now, E > A > F > D > B > C or, B > E > A > D > F > C or, E > A > B > D > F > C From both the statements E>F>A>D>B>C or, E > B > A > D > F > C

    S

    4 metre

    E

    4 metre

    M

    U

    T

    R

    P

    T

    U

    4 metre

    Person O M Q N P

    4 metre

    Q

    24. (5) From Statement I ↑↑↑↑D ↑X↑↑S↑↑↑ or ↑SX↑D From statement II ↑Y↑RD↑↑↑↑S↑↑↑ or ↑YDR↑↑↑S↑↑↑ From both the statements ↑Y↑RD ↑X↑↑S↑ ↑↑ Thus, there are 13 pepple in the line. 25. (2) From statement I

    P

    8 metre

    Floor Number Floor Number 5 Floor Number 4 Floor Number 3 Floor Number 2 Floor Number 1

    4 metre

    19. (3) From statement I

    R

    M

    R

    N

    A

    Point M is 8 metre to the cost of Point S.

    BPRE–955

    O

    Q

    DATA SUFFICIENCY From statement II

    North

    M/Q

    NW N

    R

    O

    East

    West

    SE

    SW

    M/Q

    South

    O stands to the immediate right of R 26. (4) From statement I Floor Number 5 4 3 2 1

    NE

    Person I D B E

    Point D is 10 metre to the west of Point H. From statement II

    Person II

    1m K

    5m

    4m

    4m

    D

    I C A

    II

    III

    IV

    A C B E

    B C A E

    B E A C

    4m V

    B E

    1m H

    29. (4) From statement I

    From both the statements Floor Number 5 4 3 2 1

    Person I D B E A C

    D

    Person II C A D B E

    ,

    ,

    >S>

    ,

    kb og sd

    energy

    sd ap

    only dwarf planet

    cq ap st

    the coldest planet

    sd

    cq yt

    dwarf ⇒ ap 31. (1) From statement I

    P

    M

    S

    S

    From statement II

    M

    S O S

    Q

    Q C

    B

    18 metre

    Q A

    >V>

    W > T > S > U, X U>V W>T>S>U>V>X T is the second heaviest. From statement II ,

    F

    18 metre

    E

    18 metre

    D

    E

    E sits second to the left of F. From statement II

    27. (3) From statement I ,

    H

    M

    Floor Person Person Person Person

    No. 5 4 3 2 1

    the solar

    M

    L

    From statement II

    cq

    From both the statements Q P M S O M is fourth to the left of S. 32. (4) From statement I K is the father of M and T. D is the son of T. F is the mother of D. Therefore, T is the father of D. Thus, T is the son of K. From statement II L is the father of F and U. U is the only son of L. Therefore, F is the daughter of L. T is the husband of F. There is no mention of K. 33. (5) From statement I

    H

    D B E

    the dwarf planet

    D

    C/F

    18 metre

    C 14 metre

    P

    B C/F

    D 5 metre

    A

    19 metre

    A/B North NE

    NW

    ,

    U>V>X W>T>U W>T>S>U>V>X T is the second heaviest. 28. (1) From statement I L

    A/B

    E

    West

    30. (1) From statement I

    SE

    SW

    the dwarf planet

    cq sd ap

    the solar

    kb og sd

    energy

    East

    South

    From statement II

    Q

    19 metre

    E

    9 metre

    H

    10 metre

    D

    From statement II

    4m D

    H 4m

    4m U

    5m

    V

    5m

    T

    only dwarf planet

    cq ap st

    the coldest planet

    sd cq yt

    15 metre C

    From both the statements

    BPRE–956

    19 metre

    T

    Point D is in North–East direction with respect to Point C.

    DATA SUFFICIENCY 34. (4) From statement I Day Class Monday B Tuesday A Wednesday D Thursday F Friday C Saturday E

    Box C is at the bottommost position. From statement II Position 1 2 3 4 5 6

    From statement II Day

    Class (I)

    Monday Tuesday Wednesday Thursday Friday Saturday

    Class (II) B

    B D D C C

    35. (2) From statement I L E F T

    T

    S

    R/P

    R/P

    Q

    P/R

    Q

    R I G H T

    Box C is at the bottommost position. 38. (2) From statement I C V ↑ F ↑ ↑ ↑ ↑ ↑ ↑ or C↑ ↑ F ↑ V ↑ ↑ ↑ ↑ ↑ ↑ ↑↑ From statement II C ↑ R ↑ W ↑ ↑ ↑ ↑ Q C is 10th from the right end. 39. (1) From statement I

    From statement II L E F T

    T

    S

    P/R

    R I G H T

    36. (5) From statement I Floor Person Person Number (I) (II) 5 B C 4 D/E D/E 3 A A 2 D/E D/E 1 C B From statement II Floor Number Person 5 B 4 E 3 A 2 D 1 C D lives on floor number two. 37. (3) From statement I Position 1 2

    Boxes Box B Box A

    3

    Box F

    4

    Box F

    5

    Box F Box C

    6

    Boxes Box B Box A/E Box A/E Box D Box F Box C

    (A)

    (B)

    (C)

    40.

    10 × 3 = 30 + 500 × 1 = 500 530 Conductor returned four notes of same denomination 100 × 4 = 400 Balance amount = 530 – 400 = Rs. 130 = 13 × 10 From statement II 100 × 3 = 300 50 × 4 = 200 10 × 4 = 40 Now, 300 – 200 = Rs. 100 (Not Possible) 300 – 40 = Rs. 260 500 × 3 = 1500 50 × 4 = 200 10 × 4 = 40 Now, 1500 – 200 = Rs. 1300 1500 – 40 = Rs. 1460 1000 × 3 = 3000 50 × 4 = 200 10 × 4 = 40 Now, 3000 – 200 = Rs. 2800 3000 – 40 = Rs. 2960 (5) From statement I

    Month January April July August November

    (I) K M N L J

    Person (II) K N M L J

    BPRE–957

    (III) K M L N J

    From statement II Month

    Person

    January April July August November

    (I) L

    (II) K N

    K N

    L

    From both the statements Month January April July August November

    Person K N M L J

    41. (1) From statement I According to Ranjit’s brother he sold 81 to 94 pancakes on Thursday. According to Ranjit’s sister he sold 91 or 98 pancakes on Thursday. 91 = 7 × 13 98 = 7 × 14 Common number ⇒ 91 From statement II According to Ranjit’s mother he sold 86 to 99 pancakes on Thursday. Mutiples of 3 between 1 and 9 : = 3, 6, 9 According to Ranjit’s wife he sold 91, 94 or 97 pancakes on Thursday. 42. (4) From statement I

    Box Box Box Box

    E Box D Box F or Box G Box

    Box Box Box Box

    E G F D

    G E F or D

    From statement II

    Box Box Box Box

    Box E D Box F or Box G Box

    F D E or G

    DATA SUFFICIENCY or

    Box Box Box Box

    E G F D

    Row-1 S

    From both the statements

    Box Box Box Box

    (47–48) : 47. (2) From statement I

    Box E D Box F or Box G Box

    E G F D

    L D

    Exam I II

    January

    T

    T

    April

    R

    U

    July

    T

    September

    U

    R

    December

    Exam I II

    January

    T

    April

    R S

    September December

    Row-1

    U J

    D

    A

    From statement II

    C

    S

    Row-2

    P

    A sits to the immediate right of D. (45–46) : 45. (3) From statement I

    U

    D E

    T

    S

    C

    B

    B sits to the immediate left of E. From statement II

    From both the statements Month January April July September December

    Exam T R S U Q

    Exam T was conducted in January. 44. (2) From statement I

    Row-1 J A

    B Row-2

    L D

    R

    Two persons are sitting between U and S. 48. (5) From statement I Day

    R T

    T

    A

    Month

    U

    From statement II

    From statement II

    July

    P

    Row-2

    43. (5) From statement I

    Month

    J B

    A

    Person I K M

    Monday Tuesday Wednesday Thursday Friday

    L L

    From statement II

    B

    Day E C

    A

    II M K

    D

    B sits to the immediate left of E. 46. (3) From statement I or II Box Q Box T Box R Box P Box S

    BPRE–958

    Monday Tuesday Wednesday Thursday Friday

    I

    Person II III M

    M N J

    J N M

    M J N

    From both the statements Day Monday Tuesday Wednesday Thursday Friday

    IV

    Person M K J N L

    J N

    DATA SUFFICIENCY

    INSURANCE EXAMS 1. (2) No answer can be found from the data given in the Statement I. G

    H

    From statement II In order to satisfy the given arrangement E or D must face outward. F is sitting second to the right of E. It means E is second to the left of F. Therefore, either D or E must face outward. 2. (5) From both the statements There are six floors in the building. 6

    A B

    2 1

    11th P Y Yogesh is 17th from the left. From statement II

    10 students

    D

    5 4 3

    There is no information about J and N. From both the statements K is the youngest. 6. (4) From Statement I, Exam was held in April, May or June. From Statement II, Exam was held in March, April or May. From both the statements, 7. (1) From statement I

    Y

    S

    P

    P

    S

    Y

    8. (2) From statement I Ranjana did her graduation in 1990, 1991 or 1992. From statement II Ranjana did her graduation in 1992. 9. (3) From statement I T

    M

    C

    S 6 metres

    3. (1) From statement I,

    North East

    North NE

    West SW

    N

    West

    East SE

    South

    M, S and N are in a straight line. From statement II

    South

    It is clear from the diagram that Sunny was facing North-East. From Statement II It is not clear Sunil is facing which direction. 4. (5) From both the statements E is the husband of R. E is the father of B and J. J is the daughter and B is the son of E. 5. (5) From Statement I N>J>K There is no information about L and M. From Statement II, M>L>K

    Q

    Avani did start her business in 2007 or 2008. From statement II : Avani did start her business in 2008. 13. (4) From both the statements

    Jyoti > Sadhana > Meera Pinki 14. (5) From both the statements N is mother of M and Z. 15. (2) All hats (black and blue) have brims. Baseball caps are hats. So, some baseball caps have brims. Some baseball caps may be blue. Therefore, only II is true. 16. (1) From statement I

    help self to win → & $

    help to win

    R

    S 3 metres N

    M, S and N are in a straight line. 10. (1) From statement I At 6 O’clock both the hands of clock remain in straight line. From statement II Both the hands of clock make an angle of 120° at 4 O’clock and 8 O’clock

    BPRE–959

    *

    +

    him → + $ % &

    The code for ‘self’ is

    *

    From statement II

    # >

    @

    connect to self important

    3 metres M

    12. (2) From statement I :

    make plan self

    P 6 metres

    NW

    3 metres O

    11. (5) From statement I V > J, G K > I, G From statement II V > K, I J>G From both the statements V > K, I, J > G V is not the tallest. So, A is the tallest.

    +$ %&

    The code for ‘self’ is ‘*’.

    17. (1) From statement I N>P>M>O N does not weigh the most. Therefore, Q weighs the most. From statement II Q>M>O ,

    > P >

    ,

    There is no information about N. 18. (2) From statement I Naina was born on Wednesday, Friday or Saturday.

    DATA SUFFICIENCY From statement II Naina was born on Tuesday, Thursday, Friday, Saturday or Sunday. From both the statements Naina was born on Friday or Saturday. 19. (3) From statement I S

    H/S H

    M

    L

    B

    or L

    M

    S

    H

    B

    J is husband of K. J is father of M. Therefore, J is grandfather of T. From statement II M is wife of L. T is daughter of L and M. M is daughter of J and K. Therefore, J is grandfather of T. 23. (1) From statement I Sania was born in 1984, 1986 or 1988. From statement II Sania was born in 1988 24. (3) From statement I >A>E>B>C From statement II

    From statement II Y

    L

    M

    S

    > A, E > C

    H

    25. (3) From statement I most of the time

    Three persons stand between H and L. 20. (2) From statement I D is daughter of M. E is sister of D. E is daughter of M. M is sister of W. The gender of W is not known. W is either aunt or uncle of E. From statement II M and W are children of S. E is daughter of M. 21. (2) From statement I N

    K

    N

    F

    B

    U

    J

    or K

    N

    U

    B

    F

    J

    of the

    # &% ?

    regular time

    &#

    >

    The code for ‘most’ is ‘%’. From statement II most people make it

    @*%$

    it most special tough

    % £+ $

    The code for most is either ‘%’ or ‘$’ 26. (4) From statement I P>O>N There is no information about M and L. From statement II L>M From both the statements P>O>N;L>M 27. (2) From statement I

    North NW

    NE

    West

    East

    SW

    SE South

    From statement II

    A 2 m.

    D

    4 m.

    B

    A is in west direction from B. 28. (3) From statement I Bharat has two siblings. But it is not clear they are brothers or sisters. From statement II Bharat may have one, two or more brothers. From both the statements Bharat has one brother. 29. (3) From statement I Q is grandson of T. The gender of T is not known. From statement II T is mother of N. T is wife of L. From both the statements T is mother of R and N. Q is son of R. So, T is grandmother of Q. 30. (4) From statement I Amit

    Arun

    Leena

    Thus, all friends are not facing the centre. From statement II

    B

    From statement II K N/U R N/U B

    F

    B

    F

    B 4 m.

    From both the statements K

    N

    R

    U

    B

    F

    Ken

    4 m.

    B

    A

    4 m. C

    8 m.

    J

    B

    8 m.

    22. (4) From statement I L is father of S and T L is husband of M.

    Amit Ali B

    BPRE–960

    Thus, all friends are not facing the centre.

    DATA SUFFICIENCY 31.

    (1) From statement I B

    D

    34.

    G

    F

    (3) From statement I

    35.

    P

    G Y There are five boxes in the satck.

    From statement II

    From statement II

    G

    B

    D

    B

    D

    D G

    T

    R

    S

    R

    S

    R stands to the immediate right of T. From statement II

    B F

    B

    T

    P

    D is standing to the immediate left of G. C

    (1) From statement I

    T

    Q

    T

    U

    R

    U

    R

    Y C

    There are four boxes in the stack.

    No answer.

    36. (5) From statement I

    B 32.

    D

    (5) From statement I

    W, T > V > S

    From statement II >

    ja tp fa rg

    success difficult to achieve

    rg ja ko tp

    From statement II

    U

    >W>

    ways to achieve success

    >

    U > S, V T>S

    difficult times before success

    nb ko tp li

    times fly before ways

    fa

    li nb

    dy

    From both the statements

    ways to achieve success

    ja tp

    fa rg

    success difficult to achieve

    rg ja ko tp

    difficult times before

    nb ko tp

    From both the statements U>W>T>V>S 33.

    (2) From statement I

    U

    success

    li

    B M

    times fly before

    ways

    fa

    li nb dy

    B M or U D

    D

    More than one arrangement is possible. From statement II

    R S U There are seven boxes in the stack.

    Thus, success

    tp

    37. (2) From statement I Day Car Monday

    S

    Tuesday Wednesday

    R

    Thursday

    S

    Friday T From statement II Day Car Monday Q/T Tuesday P Wednesday R Thursday S Friday Q/T

    BPRE–961

    38. (5) From both the statements

    L E F T

    V

    M

    W

    J

    N

    Q

    R

    R I G H T

    39. (1) From statement I K > R > P, J > S > T From statement II J/P > S > T R is not the oldest. ❐❐❐

    DATA SUFFICIENCY

    MODEL EXERCISES Directions (1) : Both of these questions are followed by two statements giving certain information Decide whether the information given in the statements is sufficient for answering these questions. Mark your answer as (1) If statement ‘A’ alone is needed to answer the problem (2) If statement ‘B’ alone is needed to answer the problem (3) If both the statements ‘A’ and ‘B’ are needed to answer the problem (4) If both the statements ‘A’ and ‘B’ are not sufficient to answer the problem (5) None of these 1. Can one go-to A without touching B ? I. To go to B, one has to touch A. II. It is not possible to go to A without touching D or C, both of which require you to touch B. Directions (2–6) : Each of the questions below consists of a question and two statements numbered I and II given below it. You have to decide whether the data provided in the statements are sufficient to answer the question. Read both the statements and — Give answer (1) if the data in Statement I alone are sufficient to answer the question, while the data in Statement II alone are not sufficient to answer the question. Give answer (2) if the data in Statement II alone are sufficient to answer the question, while the data in Statement I alone are not sufficient to answer the question. Give answer (3) if the data in Statement I alone or in Statement II alone are sufficient to answer the question. Give answer (4) if the data in both the Statements I and II together are not sufficient to answer the question. Give answer (5) if the data in both the Statements I and II together are necessary to answer the question. 2. What does ‘Pe’ mean in the code language ? I. ‘Na Si La Lo’ means ‘you may go now’ and ‘Ne Si Na Pe’ means ‘he may go there’ in that code language.

    II. ‘Ki Se Pe Bo’ means ‘come there and see’ and ‘Se Ni Bo Ki’ means ‘come here and see’ in that code language. 3. What is Keshav’s rank in the class of 50 ? I. Amit, ranking 18th in the class from the top is 7 ranks below Vivek, who is 5 ranks above Keshav. II. Saurav, the 10th from the bottom, is 20 ranks below Suresh, who is 5 ranks below Keshav. 4. How is Ram related to Nitin ? I. Revati, Nitin’s mother, is cousin of Sukesh, the uncle of Ram. II. Pravin, Ram’s father-in-law is the grandfather of Sachin, the nephew of Nitin. 5. Who among M, N, O, P and Q is the youngest ? I. N, the 2nd youngest, is younger than Q, O and M. II. O, the 2nd oldest, is older than N. 6. Who among P, Q, R, S and T is the shortest ? I. R, though not the shortest, is shorter than only Q. II. S, though not as tall as P, is not the shortest. Directions (7–11) : Each of the following questions, consists of a question and two statements numbered I and II given below it. You have to decide whether the data given in the statements are sufficient to answer the questions. Read both the statements and choose the appropriate option. Mark answer (1) If the data in statement I alone are sufficient to answer the question, while the data in statement II alone are not sufficient to answer the question. Mark answer (2) If the data in statement II alone are sufficient to answer the question, while the data in statement I alone are not sufficient to answer the question. Mark answer (3) If the data either in statement I alone or in statement II alone are sufficient to answer the question.

    BPRE–962

    Mark answer (4) If the data in both statements I and II together are not sufficient to answer the question. Mark answer (5) If the data in both the statements I and II together are necessary to answer the question. 7. Among buildings J, K, L, M, N and O (each building is of different height), which is the tallest? I. M is taller than J and O but shorter than K. N is taller than M but not the tallest. K is not the tallest. II. K is shorter than only two buildings. J is shorter than M and K but taller than O. N is taller than J. 8. What is the direction of M with respect to K ? I. K is 8m to the East of S. Q is to the north of S. P is 4m away from Q. M is 2m to the north of P. II. M is 12m to the west of J. Q is 6m to the South of M. K is to the South-East of Q. 9. How is P related to M? I. M is the wife of C. M and I are the only children of R. G is the only daughter of C. P is the granddaughter of R. II. G is married to L. M is the mother -in-law of L. M is the only daughter of R and A. P is the grandchild of R. 10. Five persons — M, N, O, P and Q — live on five different floors of a building but not necessarily in the same order. The lowermost floor of the building is numbered one, the one above that is numbered two and so on till the topmost floor is numbered five. How many persons live between M and Q? I. M lives on floor numbered three. Only one person lives between M and P. N lives on an odd numbered floor immediately above Q. II. O lives on an even numbered floor immediately below M. Only one person lives between O and Q. N lives on the topmost floor. 11. Six persons are sitting in two parallel rows containing three persons each, in such a way that there is an equal distance between adjacent persons. In row-

    DATA SUFFICIENCY 1, U, V and W are seated and all of them are facing south. In row2, P, Q and R are seated and all of them are facing north. (Therefore, in the given seating arrangement, each member seated in a row faces another member of the other row.) Who sits at extreme left end of row-2? I. V sits at one of the extreme ends of the line. Only one person sits between Q and the one who faces V. P is an immediate neighbour of Q. II. Only one person sits between Q and R. W faces one of the immediate neighbours of R. V is an immediate neighbour of W. Directions (12–16) : Each of the questions given below consists of a question and three statements numbered I, II and III given below it. You have to decide whether the data provided in the statements are sufficient to answer the question. Read all the three statements and Give answer (1) If the data in statements I and II are sufficient to answer the question, while the data in statement III are not sufficient to answer the question. Give answer (2) If the data in statements II and III are sufficient to answer the question, while the data in statement I is not sufficient to answer the question. Give answer (3) If the data in statement I alone or in the statement II alone or in the statement III alone is sufficient to answer the question. Give answer (4) If the data in all the statements I, II and III together are sufficient. Give answer (5) If the data in statements I and III are sufficient to answer the question, while the data in statement II is not sufficient to answer the question. 12. There are two rows facing each other. Each row has three seats. One row is facing north and another south direction. Six friends Ram, Rahul, Sita, Sonu, Mahesh and Mohan are sitting in the seats (not necessarily in the same order). Is Ram sitting to the immediate left diagonal of Sita? I. Rahul, the immediate left neighbour of Sita is facing north

    II. Mohan is second to the right of Sonu facing south direction III. Mahesh is to the immediate left of Rahul. 13. Six friends A, B, C, D, E and F are sitting around a circular table. Only three of them are facing the centre. Is C second to the right of A? I. D facing away from the centre is second to the left of E who is facing the centre. II. F, who is facing the centre is to the immediate right of both B and C. III. E, who is facing the centre, is opposite to B and to the immediate left of A. 14. There are three friends Radha, Rupa and Rekha of different ages. Who is the second eldest among the three friends? I. Rupa was born before 1989 but after 1985 and in an odd number year II. Rekha was born before 1983 but after 1979 and in an odd number year III. Radha was born after 1982 but before 1986 and in an even number year 15. Six persons B, C, D, E, F and G are living in an apartment that has six floors. Each person lives on different floor. The bottom most floor is numbered 1 and top floor is numbered 6. Who lives on 4th floor? I. F is in an even number floor and only one person lives between the floors of B and F. B lives above F. II. E lives on the floor which is exactly in the middle of the floors of D and G. G lives above D. Only three persons live between G and D. III. F lives just above the person living on the bottom most floor. 16. There are eight family members S, T, U, V, W, X, Y and Z. Each couple has at least one child. Is U aunt of V? I. S is the brother of T who is uncle of Z II. X is the father in law of W who is wife of T. III. Z is the daughter of U who is daughter in law of Y

    BPRE–963

    SHORT ANSWERS 1. (1) 5. (1) 9. (1) 13. (4)

    2. (2) 6. (5) 10. (3) 14. (4)

    3. (3) 7. (1) 11. (4) 15. (5)

    4. (4) 8. (3) 12. (4) 16. (5)

    EXPLANATIONS 1. (1) To go to B, one has to touch A 2. (2) From statement I Na Si La Lo

    you may go now

    Ne Si Na Pe

    he may go there

    Pe ⇒ he/there From statement II Ki Se Pe Bo

    come there and see

    Se Ni Bo Ki

    come here and see

    Pe ⇒ there 3. (3) From statement I Thr rank of Amit from the top ⇒ 18th The rank of Vivek from the top

    ⇒ 11th (18 – 7) Therefore, the rank of Keshav from the top = 11 + 5 = 16th From statement II The rank of Saurav from the top = 50 – 10 + 1 = 41st The rank of Suresh from the top = 41 – 20 = 21st Therefore, the rank of Keshav form the top = 21 – 5 = 16th 4. (4) From statement I Revati is mother of Nitin. Revati is cousin of Sukesh. Sukesh is uncle of Ram. Therefore, Ram is cousin of Nitin. The gender of Ram is not known. From statement II Pravin is father-in-law of Ram. Pravin is grandfather of Sachin. Sachin is nephew of Nitin. Therefore, Ram may be brotherin-law or sister-in-law of Nitin. Again, Ram may be brother or sister of Nitin. 5. (1) From statement I Q, O, M > N

    DATA SUFFICIENCY From both the statements

    Therfore, P is the youngest. From statement II

    M

    ,O>N No answer 6. (5) From statement I Q > R > P, S, T From statement II P>S From both the statements

    Q

    M, K > J > O N 8. (3) From statement I

    M

    2m P 4m M

    2m P

    4m

    2m

    4m

    Q

    P

    M

    2m P 8m

    K

    It is clear from the diagram that M is in North-West direction with respect to K. 9. (1) From statement I P is the granddaughter of R. M is the daughter of R. M is the wife of C. G is the only daughter of C and M. R is the daughter of I. M is the sister of I. Therefore, P is the niece of M. From statement II M is the only daughter of R and A. P is the grandchild of R. P may be the child of M. M may have brothers. 10. (3) From statement I Floor Number

    Person

    5 4 3 2 1

    N Q M O P

    Floor Number

    Person

    5 4 3 2 1

    N Q M O P

    East

    W

    V/U

    Q/R

    P

    Q/R

    Row-2 12. (4)

    Right

    P

    V Q/R

    Left

    Mohan

    Ram

    Sonu

    Left

    Right

    Mahesh Rahul

    Sita

    C

    13. (4)

    F

    E

    B

    A D

    14. (4) Rupa was born in 1987. Rekha was born in 1981. Radha was born in 1984. 15. (5) C 6

    G B E F D

    Row-1

    5 4 3 2 1

    16. (5)

    X (Father)

    NE

    West

    V/U

    11. (4) From statement I,

    V Q/R

    North NW

    Row-1

    K

    From statement II

    4m

    S

    J

    6m

    Q>R>P>S>T Therefore, T is the shortest. 7. (1) From statement I K > M > J, O N>M L > K, N > M > J, O Clearly, building L is the tallest. From statement II , >K> , ,

    M

    12m

    Y (Mother)

    Row-2

    From statement II Row-1

    S (Son)

    SE

    SW South

    It is clear from the diagram that M is in North-West direction with respect to K. From statement II

    V/U

    W

    V/U

    Q/R

    P

    Q/R

    Row-2

    BPRE–964

    U (DIL)

    Z (Grand Daughter)

    T (Son)

    W (DIL)

    V (Grand Son) ❑❑❑

    DATA ANALYSIS

    18

    DATA ANALYSIS

    QUESTIONS FROM 1999 TO 2010 ARE AVAILABLE ONLINE NATIONALISED BANKS & IBPS PO/MT/SO EXAMS Directions (1-4) : Study the following information carefully and answer the questions given below : (IBPS Bank PO/MT CWE-III, 26.10.2013)

    Following are conditions for selecting Human Resources (HR) Manager in a company : The candidate must (i) have secured at least 55 per cent marks in the Entrance Examination (ii) have a Graduate Degree with at least 60 per cent marks (iii) have a post graduate degree/Diploma in the Human Resources Management/Management (iv) be less than 30 years of age as on 01.09.2013 (v) have secured at least 55 per cent marks in the Higher Secondary School Examination In the case of a candidate who satisfies all other conditions Except (a) at (i) above, but has secured 65 per cent marks in the final semester in the Management and 45 per cent marks in the Entrance Examination, will be recruited as Head HR (b) at (ii) above, but has post qualification work experience of one year in a company and has 50 per cent marks in the Higher Secondary School Examination will be recruited as CEO of the Company. In each question below are given details of one candidate. You have to take one of the following courses of action based on the information provided and the conditions and sub-conditions given above and mark the appropriate course of action as your answer. You are not to assume anything other than the information provided in each question. All these cases are given to you as on 01.09.2013. 1. Shiva Kumar Kamath was born on August 13, 1987 and he has completed his Post Graduate in

    2.

    3.

    the Human Resources Management. He has secured 65 per cent marks in the Entrance Examination and 60 per cent in the Higher Secondary School Examination. He has completed his Graduation with 62 per cent marks. (1) can be recruited as Head HR of the company (2) can be recruited as Manager HR of the company (3) Data are inadequate to take a decision (4) can be recruited as CEO of the company (5) cannot be recruited in the company Rahil Khan is a Post Graduate in the Business Management System. He was born on April 9, 1988. He has secured 64 per cent marks in Graduation and 70 per cent marks in the Higher Secondary School Examination. (1) can be recruited as Head HR of the company (2) can be recruited as Manager HR of the company (3) Data are inadequate to take a decision (4) can be recruited as CEO of the company (5) cannot be recruited in the company Nakul Bhatnagar was born on May 17, 1985 and is a Post Graduate in Business Management System. He has secured 71 per cent marks in the final semester and 50 per cent marks in the Entrance Examination. He has secured 60 per cent marks in the Higher Secondary School Examination. He has completed Graduation with 63 per cent marks. (1) can be recruited as Head HR of the company

    BPRE–965

    4.

    5.

    (2) can be recruited as Manager HR of the company (3) Data are inadequate to take a decision (4) can be recruited as CEO of the company (5) cannot be recruited in the company Manohar Tambe, a Graduate with 58 per cent marks, was born on March 3, 1984. He has a Diploma in the Human Resources Management. He has been working in a company for the past two years and has 60 per cent marks in the Higher Secondary School Examination. He secured 60 per cent marks in the Entrance Examination. (1) can be recruited as Head HR of the company (2) can be recruited as Manager HR of the company (3) Data are inadequate to take a decision (4) can be recruited as CEO of the company (5) cannot be recruited in the company Swati Shinde was born on January 1, 1986. She has work experience of five years in Human Resources Team of Law. She has secured 60 per cent marks in the Higher Secondary School Examination and 65 per cent marks in the Entrance Examination. She has secured 58 per cent marks in the Graduation. She has completed her Diploma in Management with distinction. (1) can be recruited as Head HR of the company (2) can be recruited as Manager HR of the company (3) Data are inadequate to take a decision (4) can be recruited as CEO of the company (5) cannot be recruited in the company

    DATA ANALYSIS Directions (6–11) : Study the following information carefully and answer the questions given below : (IDBI Bank Officer Exam, 22.08.2014)

    In the State ‘X’ near Reyansh Vihar, a new school is being established. The school requires different items viz, Books, Stationery items, Uniforms, Sports Equipments, Furniture, Laptops and Eatables. These items are to be bought on seven different days of the same week from Monday to Sunday, but not necessarily in the same or der. Stationery items will be purchased on Friday. Only one item will be purchased between Stationery items and Laptops. Furniture will be purchased immediately after or before Stationery items and Laptops. Only one item will be puchased between Furniture and Books. Uniforms will be purchased a day before on which Books will be purchased. Sports equipments will be purchased a day before on which Uniforms will be purchased. 6. Which of the following items will be purchased on Monday ? (1) Books (2) Uniforms (3) Furniture (4) Eatables (5) Cannot be determined 7. Which of the following combinations of Day and Item is definitely correct ? (1) Sunday – Laptops (2) Thursday – Eatables (3) Tuesday – Furniture (4) Wednesday – Books (5) Saturday – Uniforms 8. Which of the following items will be purchased on Thursday ? (1) Eatables (2) Uniforms (3) Books (4) Sports Equipments (5) Laptops 9. How many items will be purchased between Sports equipments and Stationery items ? (1) One (2) Two (3) Three (4) Four (5) None 10. Four of the following five are alike in a certain way on the basis of given information and hence they form a group. Which is the one that does not belong to that group ?

    (1) Eatables – Wednesday (2) Books – Saturday (3) Sports Equipments–Thursday (4) Stationery items – Sunday (5) Laptops – Friday 11. How many items will be purchased after the Books ? (1) Two (2) Three (3) Four (4) One (5) Five

    (1) The computers and other electronic equipment in organisation have not been upgraded since past five years and consume one–third more electricity than the modern modes. (2) Owing to Tuesday being the final closing day, most of the employees work for extra hours on Monday.

    12. Study the following information carefully and answer the given question. “Last week, employees of Organisation X were retracted from working beyond regular office hours in order to cut back on extra consumption of electricity. However, I do not notice any change in the number of units of electricity consumed today,” — Manager of Organisation ‘X’ on Monday. Which of the following statements except ONE, may best explain the failure to reducing the electricity consumption?

    (3) The maximum consumption of electricity for every workpiece including computers take place after 5 O’clock. (4) The employees start all the equipments simultaneously and do not care whether all of these are being used or not. (5) Most of the employees use some personal equipments during office hours continuously. (SIDBI Officer Online Exam.24.02.2016)

    Directions (13–17) : Study the following information carefully and answer the questions given below : (IBPS Bank PO/MT CWE (Main Exam) 26.11.2017)

    L 42m

    (1) K Y 26m

    (2) X (3) B

    C 21m Q 52m

    (4) P

    In the given figure, the four-line segments 1, 2, 3, 4 are KL, XY, BC and PQ respectively. The lengths of the lines are 42 metre, 26 metre, 21 metre and 52 metre respectively. Six people Lakshmi, Bhat, Shilpa, Shiva, Anu and Bharat are standing in the line 1-KL. All of them are facing north, with distance between them increasing in multiples of 6 from the left end (i.e. suppose Lakshmi is at the left end of the line at point K then the remaining people will stand at a distance as 6 metre, 12 metre, 18 metre ..... from the left end). Two persons are standing between Shilpa and Bharat. Shilpa is standing at the left end of the line. Lakshmi is an immediate neighbour of Shiva whereas Bharat is second to the right of Shiva. Only one person stands between Lakshmi and Bhat. Similarly, six people Sagar, Meghana, Mangesh, Ravi, Shekhar and Suman are standing in line-4 PQ. All of them are facing south, with distance between them increasing in multiples of 8 from the left end (i.e. if first person is left end of the line at point P and the remaining will be stand at the distance as 8m, 16m, 24 metre..... from point P) More than three persons stand between Mangesh and Shekhar. Suman is an immediate neighbour of Sagar. Meghana stands to the immediate right of Mangesh. Two people are standing between Suman and Mangesh. Ravi and Suman are immediate neighbours of each other. Sagar is standing third from the right end. 13. In the line KL, between which of the following pairs the distance is 18 metre ?

    BPRE–966

    DATA ANALYSIS (1) Lakshmi – Anu (2) Shiva – Bharat (3) Anu – Bharat (4) Shilpa – Lakshmi (5) None of these 14. If in line-1 KL, if people stand at a distance multiples of 10 in the same order, and rest of them move to line-2 XY and stands from left end in the same order. And if in line 4-PQ, people stand at a distance at multiples of 10 in the same order and rest of them move to line-3 BC, stand from left end in the same order, then how many people are standing in both XY and BC lines? (1) 0 (2) 1 (3) 3 (4) 4 (5) 5 15. If in line 4-PQ, people stand in multiples of 12 in the same order and rest of them move to line3 BC and stand from left end in the same order, then what is the distance between Mangesh and point B ? (1) 6 metre (2) 10 metre (3) 9 metre (4) 8 metre (5) 14 metre 16. If line-1 KL is joined to line 2 XY to form a new straight-line KY in such a way that distance between only Bharat and Bhat is 18 metre. If the Anu is 58 metre away from K, then what is the distance between other people on the line KY? (1) 18 metre (2) 8 metre (3) 10 metre (4) 12 metre (5) 14 metre 17. If in line KL the people are arranged with a gap of 25 metre between them, then who among the following from KL will sit in the PQ line? (1) Bharat (2) Lakshmi (3) Anu (4) Shilpa (5) Bhat and Anu Directions (18–22) : Study the following information carefully and answer the questions given below : (IBPS RRBs Officer CWE (Prelim Exam) 11.08.2018)

    Seven people, namely A, B, C, D, E, F and G work in the same office but at a different position on the basis of seniority namely Sub Staff (SS), Clerk (CL), Executive (EX), Senior Executive (SE), Team Leader (TL), Revenue Manager (RM) and Vice President (VP).

    Note : The positions have been given in increasing order of seniority with Sub Staff (SS) being the junior most position while the Vice President (VP) being the senior most position. Only three people are senior to D. F works at one of the positions junior to D but does not work as SS. There is only one position between the ones at which D and B work. B is senior to D. More than one person is junior to C. C does not work as VP. The person working as EX is junior to G. E is senior to A but does not work as CL. E works at one of the positions junior to C. 18. As per the given arrangement, G is related to TL and B is related to SE in a certain way. To which of the following is E related to in the same way? (1) RM (2) EX (3) CL (4) VP (5) SS 19. All the employees of the office were asked to give donations to an NGO on Diwali. Each of the employees gives donation in exact multiples of 100 increasing consecutively on the basis of seniority starting from Rs. 100 by SS. What will be the sum of amount donated by F and E together? (1) Rs. 700 (2) Rs. 300 (3) Rs. 600 (4) Rs. 500 (5) Rs. 400 20. Who amongst the following works as RM? (1) C (2) B (3) G (4) E (5) A 21. Which of the following combinations is correct based on the given arrangement? (1) C – TL (2) G – SE (3) E – SE (4) A – CL (5) B – TL 22. How many positions are there between the positions at which A and D work? (1) One (2) More than three (3) Two (4) Three (5) None Directions (23–25) : Study the following information carefully and answer the questions given below : (IBPS Bank PO/MT CWE (Prelim Exam) 14.10.2018)

    BPRE–967

    Each of six friends — P, Q, R, S, T and U — is of a different age. P is elder than Q but younger than T. Q is elder than U. U is not the youngest. R is elder than Q but not the eldest. The one who is second eldest is 42 years old and the one who is second youngest is 24 years old. (Note: All ages are in whole numbers only.) 23. If the sum of the ages of U and P is 52 years then what will be the sum of the ages of R and P? (in years) (1) 64 (2) 60 (3) 66 (4) 70 (5) 72 24. As per the given information, what may be the probable age of T? (in years) (1) 40 (2) 28 (3) 44 (4) 30 (5) 36 25. Who amongst the following is the third youngest? (1) Q (2) U (3) S (4) T (5) P Directions (26–28): Study the following information carefully and answer the questions given below: There are six persons S, T, U, V, W and X, who got different marks in the examination. S got more marks than only U and X. T got less marks than W, who did not get the highest marks in the examination. The person who got second highest got 92 marks. (IBPS Bank PO/MT CWE (Prelim Exam) 21.10.2018)

    26. How many persons got more marks than U? (1) Four (2) Two (3) Five (4) One (5) Cannot be determined 27. If S got 69 marks and U got 68 marks, then which of the following statements is true? (1) X got the lowest marks (2) U got the fifth highest marks (3) Five persons got more marks than X (4) All are true (5) X got 66 marks is a possibility 28. V got which of the following possible score? (1) 85 (2) 66 (3) 92 (4) 89 (5) 94

    DATA ANALYSIS Directions (29–31) : Study the following information carefully and answer the questions given below : (Canara Bank PO Exam, 23.12.2018)

    Each of the six stores — L, M, N, O, P and Q — sold different number of computers in one day. M sold more computers than P but less than both O and Q. L sold less computers than P but more than N. The store which sold third highest number of computers sold 29 computers. The store which sold the minimum computers sold 6 computers. 29. If the number of computers sold by O is 30, then which of the following statements is true? (1) None of the given options is true (2) O and N together definitely sold less than 35 computers (3) Only two stores sold more computers than O (4) O sold maximum number of computers (5) Q sold more computers than O 30. If the number of computers sold by P is a multiple of three, more than 25, how many computers were sold by P? (1) 30 (2) 27 (3) 36 (4) 33 (5) Cannot be determined 31. How many stores sold less computers than L? (1) One (2) Three (3) Two (4) Five (5) None as L sold the lowest number of computers Directions (32–36) : Study the following information carefully and answer the questions given below : (Indian Bank Specialist Officer SO Exam, 08.03.2020)

    Eight matches — A, B, C, D, P, Q, R and S — were organized in four months — January, July, November and December — of the same year. Every match was organized either on 3rd or 16th of the month. Only two matches were organized in each month and only one match was organized on each date. No other match was organized on any other date of the given year. Also, each match was organized in a different city. Match A was organized on an even numbered date in one of the months after July. Only three matches were organized between A and the match in Chandigarh. As many matches were

    organized before the match in Chandigarh as after the match in Kolkata. S was organized immediately after the match in Kolkata. Only three matches were organized between S and B. The match in Chennai was organized in the same month as B. B was not organized in Chennai. Only two matches were organized between the match organized in Chennai and C. The match in Delhi was organized on an even numbered date in one of the months before C. As many matches were organized before the match in Delhi as after R. Only one match was organized between R and D. The match in Hyderabad was organized on one of the dates before D. Q was organized on one of the dates before P. The match in Mumbai was organized on an odd numbered date. The match in Pune was organized on one of the dates after the match in Ahmedabad. 32. As per the given information, on which date was Q organized? (1) 16th July (2) 3rd November (3) 3rd December (4) 3rd January (5) 16th January 33. As per the given information, how many matches were organized between P and the match organized in Mumbai? (1) Three (2) One (3) None (4) Two (5) More than three 34. As per the given information, which match was organized in Ahmedabad? (1) The match organized on 3rd November (2) The match organized on 16th December (3) The match organized on 16th January (4) The match organized on 3rd December (5) The match organized on 16th November 35. Four of the following five are alike in a certain way based on the given information and thus form a group. Identify the one which does not belong to the group. (1) P – The match organized on 3rd November (2) C – Chennai (3) S – The match organized on 16th December (4) R – Kolkata (5) Q – Hyderabad

    BPRE–968

    36. Which of the following statements is true based on the given information? (1) No match was organized between P and A. (2) None of the given statements is true (3) C was organized on 16th July. (4) Only two matches were organized between the matches organized in Pune and Delhi (5) P was organized in Delhi

    SBI PO EXAMS Directions (1-5) : Study the following information carefully and answer the questions given below : (SBI Probationary Officer Online Exam, 21.06.2014)

    A college planned to conduct an education fair. The college asked foreign universities to participate in the education fair so that students could know about the foreign universities. Even foreign universities can tell which course is good in which university. The courses offered by these universities are, Social Science, Psychology, Medicine, Commerce, Astronomy, Computer Science and Architecture. The fair will start on Monday and will end on Sunday of the same week. Social Science stall will be set up immediately before the Medicine stall. Medicine stall will not be set up on Tuesday or Thursday. There will be a gap of one day between the stalls of Astronomy and Medicine. Architecture stall will be set up immediately after Astronomy stall. Computer Science stall will be set up on Monday. 1. Which of the following stalls will be set up on Thursday? (1) Astronomy (2) Psychology (3) Medicine (4) Architecture (5) None of these 2. Which of the following stalls will be set up immediately after the Computer Science stall? (1) Medicine (2) Commerce (3) Psychology (4) Social Science (5) None of these

    DATA ANALYSIS 3. On which of the following days of the week the stall on Architecture will be set up? (1) Saturday (2) Tuesday (3) Thursday (4) Friday (5) None of these 4. Which of the following pairs of stalls will be set up between the stalls on Computer Science and Psychology? (1) Social Science and Astronomy (2) Medicine and Astronomy (3) Social Science and Medicine (4) Medicine and Commerce (5) None of these 5. Which of the following stalls will be set up on Friday? (1) Commerce (2) Medicine (3) Psychology (4) Architecture (5) None of these 6. Which of the following stalls will be set up on Wednesday? (1) Medicine (2) Architecture (3) Psychology (4) Social Science (5) None of these Directions (7-11) : Study the following information carefully and answer the questions given below : (SBI PO Phase-II (Main) Exam 04.06.2017)

    There are 10 shelves numbered 1, 2. ....10, They are arranged in two rows one above the other. The shelves 1, 2......5 are in row 1 and rest in row 2 which is above row 1. The shelves are arranged in increasing order of number given to them. Like the shelf number 1 is placed on extreme left of row 1, then shelf number 2 and so on. Similarly the shelf number 6 is placed on extreme left of row 2, and so on. Each shelf contains a certain number of glass slabs and photo frames. There is at least one glass slab in each shelf. The length of each glass slab is 15 cm and that of each photo frame is 6 cm. The shelf 3 has a length of 33 cm. There is one shelf between shelf 3 and yellow shelf. The yellow shelf contains 1 glass slab and 6 photo frames more than that in shelf 3. The silver shelf is just above the yellow

    shelf. The silver shelf contains same number of glass slabs as yellow shelf and 1 photo frame. There are 2 shelves between silver and green shelves. The length of green shelf is 3 cm greater than the silver shelf. The blue shelf is immediate next in number to green shelf. The blue shelf contains 1 glass slab more than that in silver shelf and 1 photo frame less than that in green shelf. There is one shelf between blue and orange shelves. The white shelf is just below the orange shelf. There is one shelf between white and red shelves. Black shelf is in row 2. The pink shelf is just below the black shelf. The black shelf has same number of photo frames and glass slabs. The orange shelf has 1 glass slab more than black shelf. The length of orange shelf is 24 cm more than the length of pink shelf. The length of violet shelf is half the length of yellow shelf. The red shelf has greater than or equal to four glass slabs. The length of pink shelf is 6 cm less than the shelf immediate next in number. The length of row 1 is 267 cm and that of row 2 is 249 cm. 7. How many more photo frames can the row 2 adjust? (1) 1 (2) 2 (3) 3 (4) 4 (5) Other than those given as options 8. The colour of shelf 2 is : (1) Violet (2) White (3) Red (4) Pink (5) Cannot be determined 9. How many total glass slabs do the silver, red and black shelves contain? (1) 7 (2) 9 (3) 10 (4) 12 (5) Other than those given as options 10. What is the total length of the pink, orange and blue shelves ? (1) 146 cm (2) 134 cm (3) 126 cm (4) 133 cm (5) 141 cm 11. If all photo frames of silver and white shelves are removed and added in black shelf then what will be the length of the black shelf ?

    BPRE–969

    (1) 66 cm (2) 67 cm (3) 61 cm (4) 69 cm (5) 62 cm 12. Study the following information carefully and answer the question given below : There has been a spate of bus accidents in Country C in the recent months killing large numbers of passengers and injuring many. This has raised serious doubts about the capability of road transport department of the country of providing safety to travellers. Which of the following statements substantiates the views expressed in the above statement? (1) Local residents are always the first to provide a helping hand to the passengers in the event of such disasters (2) None of those given as options (3) People have no option other than travelling by bus over long distances. (4) Most of the bus routes of Country C have been found to be stressed due to wear and tear in the recent times. (5) The road transportation of the country has recently come to be known as the safest mode by a national survey. (SBI PO Phase-II (Main) Exam 04.06.2017)

    Directions (13–15) : Study the following information carefully and answer the questions given below : Twelve boxes marked A to L as in alphabetical order are placed in the form of 3 × 4 matrix. The rows of a matrix are denoted as 1, 2 and 3 from top to bottom and the columns are denoted as #, &, % and @ from right to left in the same order. The blocks contain twelve different articles viz, Pen, Watch, LED, Toy, Bat, Book, Ring, Banana, Cherry, Ball, Rose and Mango. All the given information is not necessarily in the same order. Note : When a box is said to be ‘beside/adjacent/between or at a gap of’ it can be either horizontally or

    DATA ANALYSIS vertically. The box having rose is kept second to left of box L. The box having Ring is kept just below box L. Box J is kept beside the box having Ring. The box having Mango is kept exactly between the box C and the box B. Box D is kept second to the left of box E. The box J has five neighbouring boxes. Box having Banana is kept adjacent to box B. The box having Banana is kept exactly below box D. Box I is kept at a gap of one from box G. Box H is kept adjacent to box A. Box J having Pen is kept exactly between box having Ball and box C. Box B neither contains Ball nor Rose. The box having Bat is kept adjacent to box H, which neither contains Mango nor kept adjacent to box having Ball. The box having LED is kept exactly between box having Toy and box B. The box having Watch is kept at a gap of one from box G. Three boxes are neighbours of box I. Box K is kept exactly between the box having Book and box A. 13. Which of the following position correctly represents the box having Rose? (1) 1% (2) 2& (3) 1@ (4) 3% (5) None of these 14. Which of the following box contains LED? (1) L (2) K (3) A (4) G (5) None of these 15. Which of the following statements is/are true? I. Box G which contains Mango is kept to the immediate left of the one which contains Cherry. II. The box having Pen is kept second to left of the box which contains Watch. III. The box which contains Book and the box having Banana are kept together. IV. The box having Ball is kept exactly between box H and the one which contains Pen. (1) Only I and IV (2) Only III (3) Only III and IV (4) Only I and III (5) None of these

    RBI GRADE–B NABARD GRADE–A OFFICER EXAMS 1. Read the following paragraph and answer the question which follows : Bookings for the dream project of Ashiana builders’ new building are underway. The building for which construction is due to start next month has flats costing a minimum of Rs. 80 lakhs. Bank B offers 100% loan scheme in which Rs. 15 lakhs is provided by the bank at the start of construction, Rs. 25 lakhs after one year of the start of construction work and Rs. 10 lakhs every subsequent year till the completion of the project. Which of the following statements would prove that the bank’s loan disbursement plan may be flawed in its approach ? (A) Cost of construction undertaken by Ashiana builders varies depending on the area and labour cost even within a city. (B) Ashiana builders take a minimum of three years to complete a project by which time the entire payment must be made by the buyers. (C) The purchase a flat from Ashiana builders it is mandary to pay 75% of the entire amount before the second year of start of construction. (D) Many banks do not offer flexibility in payment scheme if construction of a project does not near competition. (1) All (A), (B), (C) and (D) (2) Only (A) (3) Both (B) and (C) (4) Only (C) (5) Both (B) and (D) (RBI Officer Grade ‘B’ Phase-I Exam. 21.11.2015)

    2. Study the following information carefully and answer the question given below : ‘Goldkart’, a leading gold jewellery chain in country ‘Rikmaya’ is famous for good jewellery with intricate designs. However, it re-

    BPRE–970

    cently chose to import jewellery from Company X for sale besides selling just intricate designs. Which of the following cannot be a reason behind the decisions taken by ‘Goldkart’? (1) Gold jewellery with intricate designs was a brain child of ‘Goldkart’ in order to have an edge over other jewellery companies. Since these designs are now overused, they wish to bring a newer concept of designs. (2) The new concept of layered gold jewellery recently started by Company X is widely in news worldwide. (3) The Jewellery designers of ‘Goldkart’ haven’t updated their skills since 5-6 years making them incompetent as compared to other designers of the country. (4) Since Company X has quoted high prices to export gold jewellery to ‘Goldkart’, the profit gained by ‘Goldkart’ is just 18% more than the usual profit. (5) None of those given as options (RBI Officer Grade ‘B’ Phase-I Exam 17.06.2017)

    3. Study the following information carefully and answer the question given below : A road construction project has been going on in State Y and was due to be completed two months ago. The government of that time had extended the deadline of the project by three months and also allowed truck carrying the raw material to pass through City Z in order to speed up the process. Yet the construction work is further delayed and is not likely to get over in the extended deadline as well. Which of the following may NOT be a reason for the delay of construction work in State Y? (1) The workers have started going on strike and off since the past four months as they are demanding higher wages. (2) Some part of the land on which the road is to be constructed

    DATA ANALYSIS is owned privately and a dispute is going on in the court regarding the same. (3) Monsoon have approached State Y and due to heavy rainfall the construction work is three times slower than the usual pace (4) Due to several complaints by the residents living in the vicinity of the construction site, the construction work is done for four hours less than the usual construction hours since past four months (5) Most roads of City Z have traffic jam in the morning as well as evening for two hours each as most people in the city travel to work by road. (RBI Officer Grade ‘B’ Phase-I Exam 17.06.2017)

    4. Study the following information carefully and answer the question given below : The tourism sector of Country G has emerged out to be the highest contributor to GDP of the economy of Country G for this financial year. The numbers of foreign as well as domestic visitors to various tourist spots of the country were found to be relatively high as compared to last financial year. Which of the following may be a reason for increased tourism in Country G? (A) The tourism packages offered by various travel agencies for visiting places in Country G were much cheaper and attractive. (B) The government of Country G had started special buses, trains and flights at highly subsidized rates for passengers during holidays. (C) The various hotels and travel agencies of City F in Country G started holiday packages of 18% lesser price than any other tourist spot in the country. (D) Country G was found to be the most preferred tourist destination for the last three years globally

    (1) All A, B, C and D (2) Only A and D (3) Only B and C (4) Only A, B and D (5) Only A and B (RBI Officer Grade ‘B’ Phase-I Exam 17.06.2017)

    INSURANCE EXAMS Directions (1–5) : Read the information given below and then answer the questions that follow : (Oriental Insurance Company Exam.08.04.2012)

    An Insurance Agent must schedule his appointment with eight clients A, B, C, D, E, F, G and H during one week, Monday through Friday. He must schedule two appointments for Monday, Tuesday and Wednesday each, and one each for Thursday and Friday. He must see C on Thursday. He must see B on a day before the day on which he sees D. He must see E on a day before the day on which he sees G. He must see A on a day before the day on which he sees G. He must see F and A on the same day. 1. If the Insurance Agent sees D and G on the same day, which of the following is a complete and accurate listing of the days on which he could see them ? (1) Monday (2) Tuesday (3) Wednesday (4) Monday and Wednesday 2. If he sees F on Wednesday, which of the following must be true ? I. He sees D on Tuesday. II. He sees H on Monday. III. He sees G on Friday. (1) I only (2) III only (3) I and III only (4) II and III only 3. If the Insurance Agent sees D on Tuesday, then which of the following must be true ? (1) He sees E on Monday. (2) He sees F on Tuesday. (3) He sees H on Friday. (4) He sees G on Friday.

    BPRE–971

    4. Which of the following, if true, povides sufficient additional information to determine on which day each client will have his appointment? (1) H’s appointment is schedule for Monday. (2) B’s appointment is scheduled for Tuesday. (3) B’s appointment is two days before D (4) F’s appointment is two days before B. 5. Which of the following is an acceptable schedule for the week’s appointments ? Mon. Tue. Wed. Thur. Fri. (1) B, H D, G F, A C E (2) B, H D, E F, A C G (3) B, D H, G E C F, A (4) G, B D, E F, H C A

    SHORT ANSWERS

    NATIONALISED BANKS & IBPS PO/MT/SO EXAMS 1. (2) 5. (4) 9. (2) 13. (1) 17. (5) 21. (1) 25. (1) 29. (5) 33. (1)

    2. (3) 6. (4) 10. (5) 14. (2) 18. (5) 22. (3) 26. (5) 30. (2) 34. (5)

    3. (1) 7. (1) 11. (2) 15. (4) 19. (4) 23. (4) 27. (4) 31. (1) 35. (2)

    4. (4) 8. (3) 12. (3 16. (3) 20. (2) 24. (3) 28. (3) 32. (4) 36. (3)

    SBI PO EXAMS 1. (2) 5. (5) 9. (2) 13. (3)

    2. (4) 6. (1) 10. (5) 14. (2)

    3. (1) 7. (3) 11. (1) 15. (4)

    4. (3) 8. (4) 12. (4)

    RBI GRADE–B/ NABARD GRADE–A OFFICER EXAMS 1. (3)

    2. (5)

    3. (5)

    4. (1)

    INSURANCE EXAMS 1. (3) 5. (2)

    2. (3)

    3. (4)

    4. (4)

    DATA ANALYSIS 11. (2) Three items– Stationery items, Furniture and Laptop– will

    8. (3) Books will be purchased on Thursday.

    13. (1)

    – ✓

    – ✓ ✓ ✓

    Rahil

    NG

    – ✓

    – ✓ ✓ ✓

    Nakul



    ✓ ✓

    – ✓ ✓ ✓

    after Monday. Similarly, Books

    Manohar ✓





    ✓ ✓ ✓ ✓

    will be purchased on Thursday

    Swati





    ✓ ✓ ✓ ✓

    and Saturday is given with it.

    Shilpa

    it is Wednesday, i.e., two days

    NG : Not Given

    L 6m 12m 18m 24m 30m 36m 42m

    K 0

    Distance between Lakshmi and Anu

    In case of Laptop, the day given

    1. (2) Shiva Kumar Kamath satisfies all the conditions. Therefore, he can be recruited as Manager HR of the company.

    Sunday

    Laptops

    6. (4) Eatables will be purchased on Monday.

    Bharat

    Bhat

    Lakshmi

    Mangesh

    15. (4)

    P

    12m

    24m

    16. (3)

    K

    36m

    4m Q 48m

    10m

    20m

    30m

    7. (1) The combination SundayLaptop is correct.

    B

    8m

    C 12m

    Y 48m 58m 68m 18m

    BPRE–972

    Mangesh

    Furniture

    40m

    Anu

    Saturday

    30m

    Bhat

    Stationery

    Sagar

    Friday Items

    20m

    2m Q 50m

    Meghana

    Books

    10m

    Bharat

    Uniforms

    Thursday

    C 21 metre

    No person Suman

    P

    Y 26 metre

    Sagar

    B

    6m 1m

    Lakshmi

    Wednesday

    8m

    Ravi

    Sports Equipments

    30m

    Suman

    Tuesday

    20m

    Shiva

    Eatables

    10m

    2m L 42 metre 40m

    Meghana

    X

    Shiva

    K

    Shekhar

    Items

    Monday

    Shilpa

    14. (2)

    (6-11) : Day

    = 18 metre

    day.

    Shekhar

    2. (3) There are no information regarding conditions (i) or (a). 3. (1) Nakul Bhatnagar satisfies conditions (a), (ii), (iii), (iv) and (v). Therefore, he can be recruited as the Head HR of the company. 4. (4) Manohar Tambe satisfies conditions (i), (b), (iii), (iv) and (v). Therefore, he can be recruited as the CEO of the company. 5. (4) Swati Shinde satisfies conditions (i), (b), (iii), (iv) and (v). Therefore, she can be recruited as the CEO of the company.

    = (30 – 12) metre

    with it is two days before Sun-

    Ravi



    Monday. But, the day given with

    Bhat

    or (b) (iii) (iv) (v)



    Shiva

    (i) or (a) (ii)

    Shiva

    Lakshmi

    10. (5) Eatables will be purchased on

    Anu

    Stationery items.

    Shilpa

    idate

    Conditions

    tween Sports equipments and

    12. (3) Obviously, option (3) does not explain the failure to reduce the electricity consumption.

    Books– will be purchased be-

    (1-5) : Cand-

    be purchased after Books.

    9. (2) Two items– Uniform and

    NATIONALISED BANKS & IBPS PO/MT/SO EXAMS

    Bharat

    EXPLANATIONS

    21 metre

    DATA ANALYSIS 17. (5) Shilpa

    K

    L 42m

    25m Lakshmi

    X

    (29–31) : O, Q > M > P P>L>N Now, O, Q > M > P > L > N

    Shiva

    Y 26 metre

    8m

    29

    (17+8) metre = 25 metre Bharat

    B

    29. (5) O sold 30 computers. Q>O>M>P>L>N

    C 21 metre

    7m

    (18+7) metre = 25 metre

    Bhat P

    6

    Anu

    11m (14+11) metre = 25 metre

    (18–22) : Position Sub Staff (SS) Clerk (CL) Executive (EX) Senior Executive (SE) Team Leader (TL) Revenue Manager (RM) Vice President (VP)

    Person A F E D C B G

    18. (5) G is Vice President. There is only one position between G and C (TL). B is Revenue Manager. There is only one position between B and D (SE). E is Executive. There is only one position between E and A. A is a Sub Staff (SS). 19. (4) SS ⇒ Rs. 100 (A) CL ⇒ Rs. 200 (F) EX ⇒ Rs. 300 (E) SE ⇒ Rs. 400 TL ⇒ Rs. 500 RM ⇒ Rs. 600 VP ⇒ Rs. 700 F + E ⇒ Rs. (200 + 300) = Rs. 500 20. (2) B works as Revenue Manager. 21. (1) C is a Team Leader (TL). 22. (3) Two positions are there between the positions at which A and D work.

    36m

    Q 52 metre

    (23–25) : T>P>Q R>Q>U U is not the youngest. Therefore, S is the youngest.

    T > R / P > Q > U > S

    30 29 6 O + N = 30 + 6 = 36 computers Only Q sold more computers than O. O sold second highest number of computers. 30. (2) P sold less computers than M. Therefore, P sold 27 computers (3 × 9 = 27). 29 > 27 > 25 31. (1) Only N sold less computers than L. (32–36) : Month

    42 24 23. (4) U + P = 52 years ∴ P = 52 – 24 = 28 years Thus, T>R >P>Q>U>S 42 28 24 Now, R + P = 42 + 28 years = 70 years 24. (3) T > R Therefore, age of T would be more than 42 years. 25. (1) Q is the third youngest. (26–28): > > > S > U, X W>T V > W > T > S > U, X

    92 26. (5) Either four or five persons got more marks than U. 27. (4) According to question V >W > T > S > S > U > X

    92 69 68 All the statements are true. 28. (3) V got more than 92 marks.

    BPRE–973

    January

    July November December

    Date

    Match

    3rd

    Q

    City

    16th

    B

    Delhi

    3rd

    P

    Hyderabad

    16th 3rd

    C D

    Chandigarh Kolkata

    16th 3rd

    S R

    16th

    A

    Chennai

    Ahmedabad Mumbai Pune

    32. (4) Match Q was organised on 3rd January. 33. (1) Match R was organised in Mumbai. Three matches – C, D and S – were organised between P and R. 34. (5) Match S was organised on 16th November in Ahmedabad. 35. (2) Except ‘C – Chennai’, in all other pairs, only one match was organised between the given details of the matches. Match Q was organised in Chennai. Two matches were organised between Q and C. 36. (3) Four matches were organised between P and A. Five matches were organised between the matches organised in Pune and Delhi Match P was organised in Hyderabad.

    DATA ANALYSIS

    SBI PO EXAMS (1–6) : Day Monday Tuesday Wednesday Thursday Friday Saturday Sunday

    Course Computer Science Social Science Medicine Psychology Astronomy Architecture Commerce

    1. (2) The stall of Psychology will be set up on Thursday. 2. (4) The stall of Social Science will be set up immediately after the stall of Computer Science. 3. (1) The stall of Architecture will be set up on Saturday. 4. (3) The stalls of Social Science and Medicine will be set up between the stalls of Computer Science and Psychology. 5. (5) The stall of Astronomy will be set up on Friday. 6. (1) The stall of Medicine will be set up on Wednesday. 11. (1) There is one photo frame in Silver shelf. There are three photo frames in White shelf. The length of Black shelf is 42 cm. Length of four photo frames = 6 × 4 = 24 cm Required length = (42 + 24) cm = 66 cm 12. (4) Clearly, Option (4) substantiates the views expressed in the given statement. (13 –15) : @ % & # H A L F Rose Toy Watch Ball D K E J 2 Bat LED Ring Pen I B G C 3 Banana Book Mango Cherry

    1

    13. (3) H– Rose ⇒ 1 @ 14. (2) Box K contains LED. 15. (4) Statements I and III are true.

    (7–10) :

    Row-2 249 cm

    Row-1 267 cm

    6

    7

    8

    Silver 2 Glass Slabs 1 Photo Frame

    Black 2 Glass Slabs 2 Photo Frames

    Orange 3 Glass Slabs 1 Photo Frame

    9

    10

    36 cm

    42 cm

    51 cm

    39 cm

    1

    2

    3

    4

    5

    Yellow 2 Glass Slabs 9 Photo Frames

    Pink 1 Glass Slab 2 Photo Frames

    White 1 Glass Slab 3 Photo Frames

    Violet 2 Glass Slabs 2 Photo Frames

    Red 5 Glass Slabs 1 Photo Frame

    84 cm

    27 cm

    51 cm

    39 cm

    63 cm

    Green Blue 1 Glass Slab 3 Glass Slabs 4 Photo Frames 3 Photo Frames 63 cm

    7. (3) The total length of the five shelves in the Row – 2 = (36 + 42 + 51 + 39 + 63) cm = 231 cm It is given that length of Row-2 is 249 cm Difference =(249 – 231) cm = 18cm Therefore, 3 Photo Frames (3 × 6 = 18 cm) can be adjusted in Row-2. 8. (4) The colour of Shelf 2 is Pink. 9. (2) 2 Glass Slabs in Silver shelf. 5 Glass Slabs in Red shelf. 2 Glass Slabs in Black shelf. Total number of Glass Slabs in these three shelves =2+5+2=9 10. (5) Total length of Pink, Orange and Blue shelves = (27 + 51 + 63) cm = 141 cm

    RBI GRADE–B/ NABARD GRADE–A OFFICER EXAMS 1. (3) Obviously, statements (B) and (C) prove that the bank’s loan disbursement plan is flawed in its approach. 2. (5) All the four options (1), (2), (3) and (4) may be reasons behind the decisions taken by ‘Goldkart’. 3. (5) Obviously, option (5) may not be a reason for the delay of construction work in the State Y. 4. (1) Obviously, all the four statements may be reasons for increased tourism in the Country G.

    2. (3)

    3. (4)

    4. (4)

    INSURANCE EXAMS 1. (3)

    Day

    Clients

    Monday Tuesday Wednesday Thursday Friday

    AF/BE AF/BE D, G C H

    BPRE–974

    5. (2)

    Day

    Clients

    Monday Tuesday Wednesday Thursday Friday

    B, H or E H or E, D AF C G

    Day

    Clients

    Monday Tuesday Wednesday Thursday Friday

    B, H or E D, H or E AF C G

    Day

    Clients

    Monday Tuesday Wednesday Thursday Friday

    AF EH BG C D

    Day

    Clients

    Monday Tuesday Wednesday Thursday Friday

    B, H D, E F, A C G ❑❑❑

    DATA ANALYSIS

    MODEL EXERCISES Directions (1-4) : These questions are to be answered on the basis of given data. A team of 3 reports is to be chosen from amongst two groups. Group X has as its members M, N, O and P and group Y has as its members A, B and C. The selected team must have one member from each group. In addition, O refuses to work unless A works. If M works then B works. C will not work, If M works. If O does not work, M will not work. 1. Which of the following teams cannot be selected? (1) MOA only (2) MBC only (3) NPC only (4) MOA, MBC and NPC (5) None of these 2. Who else can be there in the team of which A is a member? (1) AB (2) ON (3) OPC (4) NB (5) None of these 3. Which of the following cannot work with M? (1) N (2) O (3) C (4) A (5) None of these 4. A team having B and C will also have (1) Only N (2) Only O (3) N, O and P (4) O and P (5) None of these Directions (5-10) : These questions are based on the following statements. Asha and Charu are good in Mathematics and Athletics. Deepa and Asha are good in Atheletics and Studies, Charu and Beena are good in General Knowledge and Mathematics. Deepa, Beena and Ela are good in Studies and General Knowledge. Ela and Deepa are good in Studies and Art. 5. Who is good in Studies, General Knowledge Atheletics and Art? (1) Asha (2) Beena (3) Charu (4) Deepa (5) None of these 6. Who is good in Studies, General knowledge and Mathematics?

    (1) Asha (2) Beena (3) Charu (4) Ela (5) None of these 7. Who is good in Studies, Mathematics and Atheletics? (1) Asha (2) Beena (3) Charu (4) Deepa (5) None of these 8. Who is good in Atheletics, General Knowledge and Mathematics? (1) Asha (2) Beena (3) Charu (4) Deepa (5) None of these 9. Who is good in Studies, General Knowledge and Art but not in Atheletics? (1) Asha (2) Beena (3) Charu (4) Ela (5) None of these 10. A is shorter than B but much taller than E. C is the tallest and D is little shorter than A. Who is the shortest? (1) A (2) E (3) C (4) D (5) None of these Directions (11-14) : There four questions are to be answered on the basis of the following information. A five members research group is to be chosen from the mathematicians A, B, C and D and the physicists E, F, G and H. At least 3 Mathematicians must be in the group. However, A refuses to work with D. B refuses to work with E. F refuses to work with G. D refuses to work with F. 11. If B is chosen, who else would have to be in the group ? (1) F (2) G (3) A (4) C (5) None of these 12. If B and C are chosen, which of the following is definitely true ? P: A is chosen, Q: D is chosen R: Either F or G is chosen (1) P only (2) Q only (3) R only (4) Q and R only (5) None of these 13. If G is rejected, which other member could not work with the group?

    BPRE–975

    (1) A (2) B (3) D (4) F (5) None of these 14. If H is chosen, which of the following must be true? P : A must be chosen. Q : B must be chosen. ; R : G must be chosen. (1) P only (2) Q only (3) R only (4) P, Q and R. (5) None of these Directions (15-20) : Refer to the data below. Four couples, the Bascos, the Jeffersons, the Pardas and the Roberts, are learning a new dance. The dancers must perform this dance in pairs. At the beginning of the dance, each of the married couples is such a pair, but there are three calls for partners changes calls X, Y and Z with the following results : When call X is made, Mr Basco and Mr Parda exchange partners. When call Y is made, Mrs Roberts and Mrs Jefferson exchange partners. When call Z is made, Mr Jefferson and Mr Parda exchange partners. When more than one call is made, each change of partners is based on the arrangement of partners produced by the preceding call. 15. If, since the beginning of the dance, there has been one partner change, made in response to call X, then which of the following must be a pair of partner? (1) Mr Basco and Mrs Parda (2) Mr Basco and Mrs Roberts (3) Mr Jefferson and Mrs Parda (4) Mr Jefferson and Mrs Basco (5) None of these 16. If, after the first two calls of the dance, each dancer is again paired with his or her own spouse, the calls made could have been (1) X followed by Y (2) Z followed by X (3) X followed by X or Y followed by Y or Z followed by Z (4) X followed by X or Y followed by Y, but not Z followed by Z (5) None of these

    DATA ANALYSIS 17. If, since the beginning of the dance, two partner changes have happened, the first in response to call X and the second in response to call Y, then which of the following must be true? (1) Mrs Basco is paired with Mr Roberts (2) Mrs Basco is paired with Mr Jefferson (3) Mrs Parda is paired with Mr Roberts (4) No man is paired with his wife. (5) None of these 18. If Mr Basco is paired with Mrs Basco after the first two calls of the dance, and if the second call was Y, then which of the following is true of the first call? (1) It must have been X (2) It must have been Y (3) It must have been Z (4) It could have been any of the two calls except X (5) None of these 19. If Mr Jefferson is paired with Mrs Basco after the first two calls of the dance, and if the second call was Z, then which of the following is true of the first call? (1) It must have been X (2) It must have been Y (3) It must have been Z (4) It could have been any of the three calls except Y. (5) None of these 20. If the dance begins with the three calls Z, X and Y in the order given, then it must be true that after the third call (1) Mr Parda partners with Mrs Jefferson (2) Mr Parda partners with Mrs Roberts (3) Mr Basco partners with Mrs Basco (4) Mi Basco partner with Mrs Roberts. (5) None of these Directions (21-23) : Study the following information carefully to answer the. questions. Students joining a certain university can choose from among seven courses-Biology. Astronomy, English, Mathematics, History, Chemistry and Psychology. A student cannot take both English and Astronomy. Chemistry is a prerequisite for biology and cannot be taken concurrently with biology. A

    student must take at least two of the three courses, English, History and Psychology. No course may be repeated. 21. What is the maximum number of courses that a student can take without violating any of the conditions? (1) 3 (2) 4 (3) 5 (4) 6 (5) None of these 22. If a student is taking Astronomy and Mathematics and wishes to take four courses, then in how many different ways can the other two courses be taken ? (1) One (2) Two (3) Three (4) Four (5) None of these 23. A student, who studied Biology in his 10 + 2 and chooses chemistry, psychology and other courses in the university, could be violating which of the following conditions? I. A student cannot take both English and Astronomy. II. A student must take at least two of three courses, English, History and Psychology. III. No course can be repeated. (1) 1 only (2) 3 only (3) 1 & 2 only (4) 2 & 3 only (5) None of these Directions (24-28) : Answer these questions based on the, following paragraph. Five golfers C, D, E, F and G play a series of matches in which the following are always true of the results. Either C is the last and G is the 1st or C is the first and G is the last. D finishes ahead of E. Every golfer plays in and finishes every match. There are no ties in any match, ie, no two players ever finish in the same position in a match. 24. Which of the following cannot be true? (1) E finishes second (2) F finishes second (3) E finishes ahead of F (4) F finishes ahead of D (5) None of these 25. If D finishes third, then which of the following must be true ? (1) G finishes first (2) E finishes ahead of F

    BPRE–976

    (3) F finishes ahead of E (4) F finishes behind D (5) None of these 26. If C finishes first, then in how many different orders is it possible for the other golfers to finish? (1) 1 (2) 2 (3) 3 (4) 4 (5) None of these 27. Which of the following additional conditions make it certain that F finishes second ? (1) C finishes ahead of D (2) D finishes ahead of F (3) F finishes ahead of D (4) D finishes behind G (5) None of these 28. If exactly 1 golfer finishes between C and D, then which of the following must be true ? (1) C finishes first (2) G finishes first (3) F finishes third (4) E finished fourth (5) None of these Directions (29-31) : Six products U, V, W, X, Y and Z are to be placed in display windows of a shop. There are six display windows-numbered 1, 2, 3,4, 5, 6 and one product is to be put in one window. Moreover, U cannot be immediately to the left or immediately to the right of V. W must be immediately to the left of X. Z cannot be in window number 6. 29. Which of the following products cannot be placed in window no. 1? (1) U (2) V (3) W (4) X (5) None of these 30. If X is placed in window no. 3, then W must be placed in which window? (1) 1 (2) 2 (3) 4 (4) 5 (5) None of these 31. If U is placed in window no. 5, then which of the following products must be placed in window no. 6 ? (1) V (2) W (3) X (4) Y (5) None of these Directions (32-34) : Answer these questions based on the following information.

    DATA ANALYSIS A team of experts for conducting interviews consists of seven experts Bhushan, Cyriac, Pramila, Ram, Suresh, Shekhar and Unni. Of these Bhushan, Cyriac and Pramila are experts in Social Sciences while Suresh and Unni are experts in Basic Sciences. Ram and Shekhar had exposure in both basic sciences and social sciences. Three panels have to be formed for the interview with a restriction that a panel should have representation from experts with social sciences and basic sciences back ground. Moreover, at least one member should be an expert of only one area. 32. If Cyriac does not like to be a member of panel with Ram, and Unni was in a panel with Shekhar, then the expert who did not participate in the interview was (1) Pramila (2) Cyriac (3) Bhushan (4) Any of these (5) None of these 33. If Pramila did not participate in the interview, then who was the person most likely to be with Unni? (1) Suresh (2) Bhushan (3) Pramila (4) Any of these (5) None of these 34. Unni does not like to be with Shekhar; Ram does not like to be with Pramila and Bhushan had Suresh as the partner. If both Ram and Pramila attended the interview, then who was the partner to Cyriac? (1) Cyriac did not attend the interview (2) Shekhar (3) Pramila (4) Unni (5) None of these Directions (35-36) : These questions are based on the information given below. Two union representatives and one management representative are seated together at an octagonal table with only one seat to a side of the table. No pair of either union or management representatives may be seated together. Two additional management representatives are seated. 35. Seated between the 2 union representatives are (1) At most 2 management representatives

    (2) Only 2 management representatives (3) Not more than one management representative (4) 3 management representatives (5) None of these 36. Seated opposite of the first management representative (1) Must be a union person (2) May be a union person (3) May be a management person (4) Must be a management person (5) None of these Directions (37-39) : Study the following information carefully to answer these questions. At an Electronic Data Processing Unit, five out of the eight program sets P, Q, R, S, T, U, V and W are to be operated daily. On any one day except for the first day of a month, only three of the program sets must be the ones that were operated on the previous day. The program operating must also satisfy the following conditions I. If program P is to be operated on a day, V cannot be operated on that day. II. If Q is to be operated on a day, T must be one of the programs to be operated after Q. III. If R is to be operated on a day, V must be one of the programs to be operated after R. IV. The last program to be operated on any day must be either S or U. 37. Which of the following could be the set of programs to be operated on the first day of a month? (1) U, Q, S, T, W (2) V, Q, R, T, S (3) T, U, R, V, S (4) Q, S, R, V, U (5) None of these 38. Which of the following is true of any day’s valid program set operation? (1) Q cannot be operated at third place (2) P cannot be operated at third place (3) R cannot be operated at fourth place (4) T cannot be operated at third place (5) None of these

    BPRE–977

    39. If R is operated at third place in a sequence, which of the following cannot be the second program in that sequence? (1) S (2) Q (3) T (4) U (5) None of these Directions (40-42) : Read the following information to answer these questions. For getting selected as a Marketing Manager in a company, the candidate must: I. Be a graduate with at least 50% marks. II. Have secured at least 40% marks in the written test. III. Not be less than 24 years and more than 29 years as on 10 October, 1997. IV. Should have work experience of at least two years as an officer. However if a candidate : V. Fulfils all other criteria except at (iv) above but has diploma in marketing management, his/her case is to be referred to General Manager, marketing. Based on the above criteria and the information given in each of the following questions, you have to take the decision with regard to each case. You are not to assume anything. These cases are given to you as on 10 October, 1997. Mark your answer as (1) if the candidate is not to be appointed (2) if the candidate is to be appointed (3) if the data given are not sufficient to take decision (4) if to be referred to General Manager, Marketing (5) None of these 40. Amit Khanna, born on 5 June, 1973 has done his post graduation in Marketing Managering with first class. He has secured 50% marks in the written test. He has been working in an organization as a Marketing Officer for the last four years. 41. Rohit Verma has been working in an organization as Officer for the last ten years. His date of birth is 17 February, 1964. He has secured 60% marks in the degree

    DATA ANALYSIS examination and 40% marks in the written test. 42. Manju Sharma is a first class graduate and has done a diploma in Marketing Management. She has secured 50% marks in the written test. She was 23 years old as on 5 September, 1996. Directions (43-47) : Read the information given below carefully and answer the questions that follow. From a group of six boys M, N, O, P, Q, R and five girls G, H, I, J, K; a team of six is to be selected. Some of the criteria of selections are as follows : • M and J go together. • O cannot be placed with N. • I cannot go with J. • N goes with H. • P and Q have to be together. • K and R go together. Unless otherwise stated, these criteria are applicable to all the following questions. 43. If the team consists of two girls and I is one of them, the other members are (1) GMRPQ (2) KOPQR (3) HNOPQ (4) KRMNP (5) None of these 44. If the team has four boys including O and R, the members of the team other than O and R are (1) HIPQ (2) GJPQ (3) GKPQ (4) GJMP (5) None of these 45. If four members are boys, which of the following cannot consitute the team? (1) GJMOPQ (2) JKMNOR (3) HJMNPQ (4) JKMPQR (5) None of these 46. If both K and P are members of the team and three boys in all are included in the team, the members of the team other than K and P are (1) GIRQ (2) HIRQ (3) GJRM (4) IJRQ (5) None of these 47. If the team has three girls including J and K, the members of the team other than J and K are (1) GHNR (2) MORG (3) MNOG (4) NHOR (5) None of these

    Directions (48-52) : Study the information given below to answer these questions. • Six plays A, B, C, D, E and F are to be organised from Monday to Saturday, ie from 5th to 10th— one play on each day. ˆ There are two plays between C and D and one play between A and C. • There is one play between F and E is to be organised before F. ˆ B is to be organised before A, not necessarily immediately. The organisation does not start with B. 48. The organisation would start from which play? (1) A (2) F (3) D (4) Cannot be determined (5) None of these 49. On which date, the play E is to be organised? (1) 5th (2) 7th (3) 6th (4) Cannot be determined (5) None of these 50. The organisation would end with which of the following plays? (1) A (2) F (3) D (4) Cannot be determined (5) None of these 51. On which day, the play B is organised? (1) Tuesday (2) Friday (3) Thursday (4) None of these (5) None of these 52. Which of the following is the correct sequence of organising plays? (1) AECFBD (2) DFECBA (3) BDEFCA (4) None of these (5) All of these Directions (53-56) : Read the following information and then answer the questions that follow. Group Captain Malhotra is choosing the last part of his crew for the spaceship COSMOS, with which he plans to land on the moon. He needs 4 more crew members of whom at least two must be pilots, the others being engineers. The pilots are to be selected from Dalbir, Eric and Farid and the engineers from Lal, Monty, Naveen and Paul. Eric will not crew with Lal and Dalbir will not crew with Naveen.

    BPRE–978

    53. If Naveen is chosen, which of the following must be other members of the crew? (1) Farid, Lal and Monty (2) Dalbir, Eric and Monty (3) Eric, Farid and Monty (4) Eric, Farid and Paul (5) None of these 54. If Paul is chosen, which candidates will not be chosen to be on the crew? (1) Dalbir, Eric and Monty (2) Dalbir, Eric and Farid (3) Dalbir, Farid and Lal (4) Eric, Farid and Lal (5) None of these 55. Given the above statements about the relationships among the potential crew members, which of the following must be true? A : If Dalbir is rejected, then Monty must be chosen. B : If Dalbir is rejected, then Farid must be chosen. C : If Dalbir is chosen, then Paul must also be chosen. (1) B only (2) C only (3) A and B only (4) A and C only (5) None of these 56. If Lal is chosen as an engineer, which of the following could be the other members of the crew? A : Dalbir, Farid and Monty B : Dalbir, Farid and Naveen C : Dalbir, Farid and Paul (1) A only (2) B only (3) C only (4) A and C only (5) None of these Directions (57-59) : Study the information below to answer these questions. • Seven types of paints A, B, C, D, E, F and G of different colours red, green, yellow, black, blue, white and orange are filled in different tins P, Q, R, S, T, U and V, • The order of the types of paints, colours and the tins is not necessarily in the same order. • Paint B is not kept in tins Q or R and is white in colour. • Paint D is kept in tin V and is not blue or red. • F is kept in tin P and is green in colour.

    DATA ANALYSIS • Paints C and E are kept in tins T and U respectively and are black and yellow in colour respectively. • G is not blue in colour. • Red paint is not kept in tin Q. 57. Which of the following paints is orange in colour? (1) C (2) A (3) D (4) G (5) None of these 58. Paint B is kept in which tin? (1) S (2) T (3) U (4) Q (5) None of these 59. Which tin contains blue paint? (1) R (2) p (3) S (4) None of these (5) All of these Directions (60-63) : Study the following information to answer these questions. A blacksmith has five iron articles A, B, C, D and E each having a different weight. • A weighs twice as much as B. • B weighs four and-a-half times as much as C. • C weighs half as much as D. • D weighs half as much as E. • E weighs less than A but more than C. 60. Which of the following is the lightest in weight? (1) C (2) B (3) A (4) D (5) None of these 61. E is lighter in weight than which of the other two articles? (1) A, C (2) D, C (3) A, B (4) D, B (5) None of these 62. E is heavier than which of the following two articles? (1) A, C (2) D, C (3) D, B (4) A, B (5) None of these 63. Which of the following articles is the heaviest in weight? (1) C (2) B (3) A (4) D (5) None of these Directions (64-67) : Attempt these questions based on the information given against each. 64. In order to qualify in an examination having 6 subjects, a student has to get at least 50% and above

    marks separately in any 4 subjects and 35% and above in each of the 6 subjects. If a total of 25% candidates have qualified in the examination, then which of the following is definitely true? (1) 50% of the students got 50% and above in 4 subjects but only half of them could get 35% and above in all the subjects (2) 75% of the students could not get at least 35% marks in all the 6 subjects taken together (3) 25% of the students have secured 50% and above in all the 6 subjects (4) At least 25% of the students could get at least 35% and above marks in each of the subjects (5) None of these 65. “Cases of food-poisoning have been reported from village X. After a dinner party arranged for 100 people, 68 were admitted to the hospital and 36 were reported to be out of danger. The food, which was cooked and stored in an open space for almost 12 hours, was served after reheating. Investigation is going on.”-A news report Which of the following can be hypothesised from the above information? (1) Cases of food poisoning need to be handled carefully (2) Stale food is likely to be the cause of food poisoning (3) Late night dinner parties for a large number of people result in food-poisoning. (4) Cases of food poisoning are not reported in urban dinner parties (5) None of these 66. “If you want a hassle-free holiday package for city M, then join only our tour. Hurry up; only a few seats available” An advertisement of XYZ Tourist Company. If the above statement is true, then which of the following has been assumed while making the statement? (1) No seats may be available with other tour operators for city M

    BPRE–979

    (2) Now-a-days people have a lot of money to spend on their comforts (3) Travel packages offered by other tour operators are neither cheap nor comfortable (4) Many people desire convenience and comfort while going for a holiday (5) None of these 67. The State Government’s agency ‘Housewell’ has constructed 500 flats for the middle class but in spite of a shortage of houses, it has not even received 100 applications. Which of the following if true, could explain this? (1) A private builder’s scheme which has come up on the adjacent plot is overbooked inspite of higher cost and 100% advance payment (2) The flats are not accessible either by bus or by train (3) The quality of construction of ‘Housewell’ is reported to be very poor (4) The cost and conditions of payment are quite demanding and are slightly higher than the usual government housing schemes (5) None of these Directions (68-71) : Read the following information to answer these questions. Ankit is decorating his room and trying to arrange six paintings on the east and west walls in his room. The paintings are each multicolour representations of one of the letters of the alphabets E, H, M, O, R, T. Ankit does not want the three letters on each wall to make any common English word. Also, the colours of the O and E do not look good next to each other, nor do the T and O go well together. 68. If Ankit puts E, H and M on the east wall, which of the following must be true? (1) R and M cannot face each other (2) O cannot be in the centre of the west wall (3) E cannot be in the centre of the east wall (4) T and M cannot face each other (5) None of these

    DATA ANALYSIS 69. If Ankit’s mother is coming to visit and he decides to celebrate the visit by having his paintings spell ‘MOTHER’, starting with the leftmost painting on the 39 east wall and going around the room, which of the following will be false? (1) O is opposite to E (2) H is next to E (3) T is next to O (4) T is opposite to R (5) None of these 70. Which of the following is not possible? (1) T and O to be opposite to each other (2) T, H and E to be on the same wall. (3) H, M and R to be on the same wall (4) M and O to be opposite to each other (5) None of these 71. If Ankit trades his M painting for another O painting just like the one he has now, which of the following must be false? (1) O can be on opposite walls in the middle (2) Either R or H will be next to an O (3) Either R or H will be next to either T or E (4) T will be opposite to either O or E ‘ (5) None of these Directions (72-75) : Read the following information to answer these questions. In a school, there were five teachers. A and B were teaching Hindi and English. C and B were teaching English and Geography. D and A were teaching Mathematics and Hindi. E and B were teaching History and French. 72. Who among the teachers was teaching maximum number of subjects? (1) A (2) C (3) B (4) D (5) None of these 73. Which of the following pairs was teaching both Geography and Hindi? (1) A and B (2) C and A (3) B and C (4) None of these (5) None of these

    74. More than two teachers were teaching which subjects? (1) History (2) French (3) Hindi (4) Geography (5) None of these 75. D, B and A were teaching which of the following subjects? (1) English only (2) Hindi only (3) Hindi and English (4) English and Geography (5) None of these Directions (76-79) : Use the information given below to answer these questions. After months of talent searching for an Administrative Assistant to the president of the college the field of applicants has been narrowed down to five (A, B, C, D and E). It was announced that the finalist would be chosen after holding a series of all day group personal interviews. The examining committee agreed upon the following procedure (i) The interviews will be held once a week, (ii) No more than 3 candidates will appear at any all day interview session. (iii) Each candidate will appear at least once. (iv) If it becomes necessary to call applicants for additional interviews, no more than one such applicant should be asked to appear the next week. (v) Because of a detail in the written application, it was agreed that whenever Candidate B appears, A should also be present. (vi) Because of travel difficulties, it was agreed that C will appear for only one interview. 76. At the first interview, the following candidates appear A, B and D. Which of the following combinations can be called for the interview to be held the next week? (1) CDE (2) BCD (3) ABE (4) ABC (5) None of these 77. Which of the following is a possible sequence of combinations for interviews in two successive weeks? (1) ABD; ABE (2) ADC; BDE (3) ADE; ABE (4) BDE; ACD (5) None of these

    BPRE–980

    78. If A, B and D appear at the interview and D is called for an additional interview the following week, which two candidates may be asked to appear with D? I. A II. B III. C IV. E (1) I and III (2) I and II (3) II and III (4) III and IV (5) None of these 79. Which of the following correctly states the procedure followed by the search committee? I. After the second interview, all applicants have appeared at least once. II. The committee sees at least one applicant a second time. III. If a third session is held it is possible for all applicants to appear at least twice. (1) II only (2) I only (3) I and II (4) III only (5) None of these Directions (80-82) : Read the following information carefully to answer these questions. Mayor, Avinash, Govind, Alok, Mahesh, Nikhil and Mohit are seven friends. They are studying engineering in different branches of Mechanical, Chemical, Electronics, Computer Science, IT, Biotech and Electrical, not necessarily in the same order. Each of them also excels in one of the following games-football, cricket, hockey, badminton, squash, volleyball and lawn tennis, again not necessarily in the same order. Govind plays hockey and he neither studies Electrical nor IT. Mahesh studies Electronics and plays volleyball. Mayor studies Biotech and does not play neither football nor squash. The one who studies Chemical plays cricket. The one who plays football studies Computer Science. Nikhil plays squash, Mohit plays badminton and studies Electrical. Alok does not study Chemical. 80. Which of the following games does Avinash play? (1) Cricket (2) Football (3) Football or Cricket (4) Data inadequate (5) None of these 81. What does Nikhil study? (1) Computer Science (2) Chemical

    DATA ANALYSIS (3) IT (4) Data inadequate (5) None of these 82. What does Govind study? (1) IT (2) Electrical (3) Chemical (4) Mechanical (5) None of these Directions (83-84) : Read the following information to answer these questions. There are four friends Ravi, Hari, Manu and Jatin. One of them lives at Kanpur and his hobbies are reading and writing. Hari and Jatin live at Lucknow. Hari’s hobby is stamp collection. Common hobby of two friends staying at Lucknow is coin collection. Ravi lives at Allahabad. One boy staying at Lucknow also .loves to listen music. The boy staying at Allahabad loves to travel and read comics. All the boys have two hobbies each. 83. Coin collection and listening to music are the hobbies of (1) Hari (2) Ravi (3) Manu (4) Jatin (5) None of these 84. Which of the following is not the hobby of Ravi? (1) Travelling (2) Reading (3) Reading comics (4) Staying (5) None of these Directions (85-88) : Read the following information to answer these questions. Five friends A, B, C and D and E wore shirts of green, yellow, pink, red and blue colours and shorts of black, white, grey, blue and green colours. • Nobody wore shirt and shorts of same colour. • D wore blue shirt and C wore green shorts. • The one who wore green shirt, wore black shorts and the one who wore blue shorts, wore red shirt. • A wore white shorts and pink shirt. • E did not wear red shirt. 85. What was the colour combination of D’s shirt and shorts? (1) Blue and grey (2) Red and blue (3) Green and black (4) Blue and white (5) None of these

    86. Who wore white shorts? (1) B (2) A (3) E (4) Data inadequate (5) None of these 87. Which colour shirt did C wear? (1) Green (2) Blue (3) Yellow (4) Pink (5) None of these 88. Which colour of shorts did B wear? (1) White (2) Blue (3) Grey (4) Black (5) None of these 89. A, B, C and D play a game of cards. A says to B, “If I give you 8 cards, you will have as many as C has and 1 shall have 3 less than what C has. Also if I take 6 cards from C, I shall have twice as many as D has”. If B and D together have 50 cards, how many card has A got? (1) 23 (2) 27 (3) 37 (4) 40 (5) None of these Directions (90-92) : Answer these questions based on the information below. Marwah and Ramesh are in the same team of hockey players. Prem has defeated Ramesh in badminton but lost to Saurabh in tennis. Naresh is a team mate of Samrat in football and with Saurabh in hockey. Ramesh defeated Saurabh in chess. Those who play cricket, don’t play badminton, volleyball or tennis. Marwah and Prem are in opposite teams of basketball. Naresh represents his state in cricket while Samrat does so at the district level. Boys who play chess don’t play football, basketball or volley ball. Marwah and Prem are together in the volleyball team. Boys who play football play hockey also. 90. Name the boys who don’t play football. (1) Ramesh and Saurabh (2) Ramesh and Samrat (3) Saurabh and Naresh (4) Ramesh and Naresh (5) None of these 91. Which is the most popular game with this group? (1) Hockey (2) Cricket (3) Chess (4) Volleyball (5) None of these

    BPRE–981

    92. Which boy plays both badminton and hockey? (1) Naresh (2) Ramesh (3) Saurabh (4) Prem (5) None of these Directions (93-95) : Answer these questions based on the information given below. ‘TNQ’ is an organization imparting training to people to touch the different dimensions of life and be like God. It was planning to organize series of eight lectures A, B, C, D, E, F, G and H, not necessarily in the same order, for three subjects X, Y and Z. Subject X was to be covered first in three lectures followed by Z and then subject Y in two lectures. Lectures A, C and D have to be on separate days. Lectures B and F have to be kept on separate days. Lecture B cannot be clubbed with A or G or D. Lectures G and H should happen on one day. 93. Which of the following pairs of lectures can go along with lecture A on subject X? (1) B, C (2) G, H (3) D, E (4) F, H (5) None of these 94. Which combination of lectures was arranged on the second day of the series? (1) C, G, H (2) B, D, E (3) C, A, G (4) Data inadequate (5) None of these 95. Which of the following lectures were for subject Y ? (1) D, F (2) G, H (3) B, C (4) Data inadequate (5) None of these Directions (96-98) : Answer these questions based on the information given below. A sports week was organized in an institute from 9th to 16th of a month, 9th being a Wednesday. During this period, various sports were organized like cricket, badminton, table-tennis, kabaddi, hockey and football and they were played, one game being played on each day. The following conditions had to be complied with. • Hockey was not played on the closing day ie, on 16th. • Table-tennis was played on the previous day of the cricket game.

    DATA ANALYSIS • Football was not played, either on Wednesday or Saturday. • No game was played on Thursday and Sunday. • Kabaddi was played on Monday. • There was a gap of 2 days between cricket and football. 96. The sports week started with which game? (1) Table-tennis (2) Hockey (3) Cricket (4) Badminton (5) None of these 97. How many days gap is there between the game of hockey and football? (1) Three (2) Five (3) Four (4) Two (5) None of these 98. Which pair of games was played on Wednesday? (1) Cricket and tennis (2) Hockey and badminton (3) Football and cricket (4) Hockey and cricket (5) None of these Directions (99-101) : Answer these questions based on the information given below. There is an office block for six officers namely Anil, Brijendar, Chander, Dharmendar, Alexander and Farmaan. The three offices are on to the left hand side of the entry point in a row and another three on the right hand side of the entry gate in a row. In between these two rows of offices, there is a corridor and all these offices have the doors opening towards the corridor. • Both Brijendar and Chander have their offices to the right of the corridor (as one enters the office block and Anil occupies an office to the left of the corridor. • Alexander and Farmaan occupy offices on opposite sides of the corridor but their offices don’t face each other. • The offices of Chander and Dharmendar face each other. • Alexander does not have a corner office. Farmaan’s office is further down the corridor than Anil’s but on the same side. 99. If Alexander sits in his office facing the corridor, whose office is to his left? (1) Chander (2) Brijendar (3) Anil (4) Dharmendar (5) None of these

    100. Who is/are Farmaan’s immediate neighbour(s)? (1) Anil and Chander (2) Anil only (3) Chander only (4) Dharmendar only (5) None of these 101. Dharmendar was heard telling someone to go further down the corridor to the last office on the right side. To whose office room was he trying to direct that person? (1) Anil (2) Brijendar (3) Chander (4) Alexander (5) None of these Directions (102 – 106) : Study the following information carefully and answer the questions given below: There is a four floor apartment as in the figure given below. Each floor has three flats. The bottom floor is known as row – 1 and the top floor is known as row– 4. The left end floors are in column 1 and the right end floors are in column 3. There are 12 flats having 12 different tenants – A, B, C, D, E, F, G, H, I, J, K and L living in each flat. Each flat is charged with different monthly rents as given in the below figure. Each flat is occupied by different professionals as in the given figure. Left Rs. 18000 Engineer Rs. 13000 Journalist Rs. 15000 Politician Rs. 14000 Dancer

    Rs. 17000 Editor Rs. 15000 Doctor Rs. 16000 Manager Rs. 15000 Lawyer

    Right Rs. 15000 Player Rs.12000 Police Rs.10000 Winter Rs. 12000 Artist

    ● L is living immediate left above di-

    agonally of a tenant who is living immediate left below diagonally of tenant K ● Tenant G is living immediate left above diagonally of a tenant who is living immediate right above diagonally of H’s flat ● I is living immediately above a tenant in the same column who is living immediate right above diagonally of A’s flat ● B is living two floors above a tenant in the same column who is living immediate left of A’s flat

    BPRE–982

    ● D is living immediate left below

    diagonally of a tenant who is living immediate left above diagonally of C’s flat ● J is living immediate below flat of a tenant who is living immediate right above diagonally of H’s flat ● F is living immediate left below diagonally of a tenant who is living immediate left below diagonally of L’s flat ● D is living immediate left a person in the same row who is living one floor above A’s flat in the same column ● E is living immediate right of a person who is living three floors above the flat of F’s flat 102. What is the total rent paid by Manager and D? (In Rupees) (1) 30,000 (2) 22,000 (3) 31,000 (4) 29,000 (5) 27,000 103. What is the monthly rent paid by the person who stays second to the right of tenant F? (In Rupees) (1) 15,000 (2) 12,000 (3) 10,000 (4) 14,000 (5) 13,000 104. Which of the following combinations is correct according to the given arrangement? (1) G – Politician (2) L – Doctor (3) E – Police (4) C – Manager (5) J – Artist 105. Which of the following statements is correct with respect to the tenant who pays lowest monthly rent? (1) He lives second to the right of A’s flat (2) He lives immediate above of G’s flat (3) He lives immediate right below diagonally of G’s flat (4) He lives immediate below of I’s flat (5) He lives immediate left of D’s flat 106. What is the ratio of monthly rents paid by the tenants I, H and L respectively? (1) 1 : 1 : 1 (2) 2 : 1 : 4 (3) 17 : 15 : 12 (4) 1 : 1 : 2 (5) 2 : 2 : 3

    DATA ANALYSIS Directions (107–111) : Study the following information carefully and answer the questions given below : There is an apartment having four floors as given below. Each floor has three flats. The bottom floor is known as row – 1 and the top floor is known as row -4. The left end floors are in column 1 and the right end floors are in column 3. There are 12 flats having 12 different tenants – A, B, C, D, E, F, G, H, I, J, K and L living in each flat but not necessarily in the same order. Each flat is charged with different monthly rents as given below. Left side

    Columns

    Rs. 15000

    Rs. 12000

    Right side Rs. 10000

    Rs. 16000

    Rs. 13000

    Rs. 16000

    Rs. 10000

    Rs. 14000

    Rs. 16000

    Rs. 11000

    Rs. 15000

    Rs. 15000

    ●D

    is living immediate left below diagonally of a tenant who is living to the immediate right below diagonally of B’s flat ● G is living to the immediate right of a tenant who is living three flats above flat of K ● E is living to the immediate left of a tenant who is living to the immediate right below diagonally of L ● H is living to the immediate right below diagonally of a tenant who is living to the immediate left below diagonally of J ● C is living two flats above the flat of a person in the same column who is living second to the right of the K ● F is living to the immediate left below diagonally of a tenant who is living to the immediate right above diagonally of C ● L is in the same floor where K’s immediate above flat neighbour is living. ● J is living to the immediate right above diagonally of a tenant who is living to the immediate right diagonally above the D ● A is living to the immediate above diagonally of both D and H’s flats ● I is living on an odd numbered floor.

    107. What is the total rent paid by the F and H? (1) Rs. 28,000 (2) Rs. 32,000 (3) Rs. 30,000 (4) Rs. 26,000 (5) Rs. 35,000 108. What is the ratio between the rents paid by the J, D and I in the same order? (1) 1 : 2 : 3 (2) 2 : 2 : 3 (3) 2 : 3 : 1 (4) 1 : 3 : 2 (5) 2 : 3 : 3 109. What is the rent paid by the tenant who is living two flats below the flat of A in the same column? (1) Rs. 16,000 (2) Rs. 14,000 (3) Rs. 11,000 (4) Rs. 15,000 (5) Rs. 13,000 110. Whose rent is the average rent paid by F, C, D and J? (1) G (2) B (3) I (4) D (5) A 111. Which set of the tenants has same rents amount compared with other tenants? (1) B-A-C (2) F-C-H (3) L-K-I (4) J-F-B (5) H-G-F Directions (112–116) : Study the following information carefully and answer the questions given below: There is a four floor apartment as given below. Each floor has three flats. The bottom floor is known as row – 1 and the top floor is known as row -4. The left end floors are in column 1 and the right end floors are in column 3. There are 12 flats having 12 different tenants – D, E, F, G, H, I, J, K, L, M, N and O living in each flat (NOT IN THE SAME ORDER). Each flat is charged with different monthly rents as given below. Each flat is occupied by different professionals as given below. Left

    Right

    Rs. 15,000 Engineer

    Rs. 12,000 Editor

    Rs. 10,000 Player

    Rs. 16,000 Jounralist

    Rs. 13,000 Doctor

    Rs. 16,000 Police

    Rs. 10,000 Politician

    Rs. 14,000 Manager

    Rs. 16,000 Writer

    Rs. 11,000 Dancer

    Rs. 15,000 Lawyer

    Rs. 15,000 Carpenter

    BPRE–983

    Tenant M is living to the immediate right of a person who is living immediate left below diagonally of I ● Tenant J is living immediate left below diagonally of a tenant who is living immediate left above diagonally of N ● Tenant E is living second to the left of a tenant who is living immediate right below diagonally of tenant D ● Tenant L is living two flats left below diagonally of a tenant who is living immediate below flat of F in the same column ● Tenant H is living three flats above a tenant who is living immediate left below diagonally of O ● Tenant K is living immediate left above diagonally of a tenant who is living second to the right of tenant J ● Tenant F is living two flats above of a person who is living immediate left above diagonally of G ● Tenant D is living two flats above the flat of a person who is living immediate left below diagonally of I. ● Tenant E is living in the same column in which H is living ● K is living on an even number floor 112. What is the rent paid by L, D and H together? (1) Rs.37,000 (2) Rs.42,000 (3) Rs.39,000 (4) Rs.40,000 (5) Rs.41,000 113. Which of the following pair pays Rs.32,000 as their monthly rent together? (1) I and M (2) J and L (3) K and D (4) G and F (5) N and I 114. Who among the following are Journalist and Writer respectively ? (1) D and G (2) M and H (3) J and I (4) N and L (5) F and M 115. Manager and another tenant pay rent Rs.29,000 together per month. Who is that person? ●

    DATA ANALYSIS (1) I (2) D (3) E (4) F (5) H 116. What is the average of the rent paid by tenants M, F, H and L together? (1) Rs.13,000 (2) Rs.11,500 (3) Rs.10,750 (4) Rs.12,750 (5) Rs.14,500 Directions (117–121) : Study the following information carefully and answer the questions given below: There are 12 different products A, B, C, D, E, F, G, H, I, J, K and L listed in a bar diagram as given below. These products are marketed from 4 different countries India, USA, UK and France. Each country markets 3 different products from the list given above. The products are rated with INDEX from 10 points to 80 points. 80

    Index 80 70

    70 60

    60 50

    50

    50

    30

    30

    40

    40

    30

    30 20

    50

    20 10

    10 0 India ●











    USA

    UK

    France

    The product L is immediate right below diagonally of a product that is marketed from India Product A is immediate left above diagonally of a product that is marketed from France Product B is in the top index and second to the left of a product in the same index Product H is in middle index which is immediate right above diagonally of product F Product C is at the bottom index which is third to the right of either F or L Product K is immediate left below diagonally of a product in middle index marketed from France

    Product D from India is immediate left below diagonally of a top index product ● Product J from USA is immediate left above diagonally of a middle index product ● Product E is immediate right above diagonally of a middle index product ● Product I has the index value with minimum 10 points and maximum 30 points ● Product A is in one index above the product L in the bar diagram ● Product G is in top index marketed either from USA or France 117. Which of the following middle indexed product is marketed from UK? (1) H (2) I (3) G (4) D (5) A 118. Which of the following product has got index minimum 30 and maximum 40 points? (1) G (2) B (3) D (4) H (4) I 119. What is the total maximum index points earned by the products J and E? (1) 110 (2) 140 ●

    (3) 120

    (4) 160

    (5) 170 120. What is the index points earned by the product K? (1) 20

    (2) 10

    (3) 30

    (4) 50

    (5) 40 121. Which of the following product from UK has got maximum 80 index points? (1) E

    (2) J

    (3) H

    (4) G

    (5) D Directions (122–126) : Study the following information carefully and answer the questions given below : There are seven friends Vishnu, Mani, Mangesh, Bharat, Anu, Lakshmi and Sagar staying in Mega home stay cottage which has five separate buildings Alankar, Prestige, Royal, Comforts and Octane. Each cottage has three floors.

    BPRE–984

    There are three friends staying in three different cottages. Two friends of same gender are in one cottage and another two friends of same gender are in another cottage. ● Lakshmi is in Either Royal cottage or Prestige cottage in third floor. Vishnu is in one of the cottages that is to the immediate left of cottage where Lakshmi stays. Mangesh stays in neither Comforts nor Octane. Bharat is in either Comforts or Alankar. Mani does not use lift facility to reach his room ● Sagar stays in second floor of a cottage that is to the immediate right of Comforts. Anu does not use lift facility to reach her room where as Bahart and Magesh have to use lift facility to reach their rooms. ● Mani stays in a cottage where Vishnu also stays. Prestige cottage has only one left immediate cottage. Anu stays in exactly middle cottage where Lakshmi also stays. Octane has no right immediate cottage. ● Sagar is in a cottage at the extreme end. There is one room vacant between Vishnu’s room and Mani’s room. There is a mini theatre in between Royal cottage where two female friends stay. Lakshmi stays second to the right of Mangesh. ● Bharat stays one floor above Sagar’s floor whereas Mani stays one floor below Mangesh’s floor. 122. In which of the following cottages, does Bharat stay? (1) Prestige (2) Royal (3) Comforts (4) Octane (5) Alankar 123. Who stays in the cottage located third to the right of a cottage where Vishnu stays and in which floor that person stays? (1) Bharat – 2nd floor (2) Sagar – 2nd floor (3) Mani – 1st floor (4) Mangesh – 3rd floor (5) Anu – 2nd floor 124. Who does stays in top floor that is immediate left of Comforts cottage? (1) Vishnu (2) Mangesh (3) Mani (4) Bahart (5) Lakshmi ●

    DATA ANALYSIS 125. Four of the following five are alike in a certain way based on their positions in the given arrangement and so they form a group. Which is the one that does not belong to the group? (1) Mani – Sagar (2) Vishnu – Bharat (3) Mangesh – Sagar (4) Mani – Anu (5) Vishnu – Lakshmi 126. If Bharat stays with Sagar, then which cottage and with which floor will be vacant? (1) Prestige – 2nd floor (2) Alankar – 3rd floor (3) Comforts – 3rd floor (4) Octane – 2nd floor (5) None of these Directions (127 – 131): Study the following information carefully and answer the questions given below : Twelve professionals O, P, Q, R, S, T, U, V, W, X, Y and Z sitting around a rectangular table facing the centre. There are 4 team leaders sitting in each corner of the table. Except team leaders, other employees receive different salary package as given in the following figure.

    40K

    30K

    35K

    40K

    29K

    32K

    36K

    33K

    40K ●











    38K

    28K

    40K

    [K = Thousand] X is sitting second to the right of a person who is sitting to the immediate left of R U is sitting to the immediate left of a person who is sitting third to the left of Z Q is sitting opposite to a person who is sitting third to the right of V T is sitting to the immediate right of a person who is sitting fourth to the left of O V is sitting to the immediate right of a person who is sitting third to the right of T W is sitting second to the left of a person who is sitting third to the left of O

    ● R is sitting third to the left of a

    person who is who is sitting diagonally opposite to Y ● P is sitting third to the left of a person who is sitting diagonally opposite to X ● The immediate neighbour of both S and X is at one of the corners ● Q, one of the team leaders is sitting at top left of the table 127. What is the salary of both R and U? (1) Rs.66,000 (2) Rs.80,000 (3) Rs.65,000 (4) Rs.78,000 (5) Rs.67,000 128. What is the salary received by the person sitting exactly between Z and P? (1) Rs.33,000 (2) Rs.40,000 (3) Rs.36,000 (4) Rs.36,000 (5) Rs.30,000 129. Which of the following statements is true about the person who receives salary Rs. 32,000? (1) He is sitting exactly between P and X (2) He is sitting opposite to R (3) His salary with V sums up to Rs. 72,000 (4) He is third to the right of T (5) He is diagonally opposite to W 130. Who among the following employee is sitting fourth to the right of T? (1) The person who receives Rs. 33,000 (2) The person who receives Rs. 38,000 (3) The person who receives Rs. 30,000 (4) The person who receives Rs. 28,000 (5) The person who receives Rs. 40,000 131. Which of the following pairs have their salary difference of Rs. 3,000 ? (1) T and Z (2) U and V (3) Y and P (4) and R (5) X and S Directions (132–136) : Study the following information carefully and answer the questions given below : Eight people — A, B, C, D, E, F, G and H — live on eight different floors of a building but not necessarily in the same order. The lower most floor of

    BPRE–985

    the building is numbered one, the one above that is numbered two and so on till the topmost floor is numbered eight. Each one of them ran for a different distance in a marathon — 2300 metre, 3800 metre, 5000 metre, 6400 metre, 7200 metre, 6300 metre, 9100 metre and 10000 metre but not necessarily in the same order. The one who ran for 5000 metre lives on an even numbered floor above floor number 5. Only three people live between the one who ran for 5000 metre and G. The one who ran for 9100 metre lives immediately above E. Only two people live between the one who ran for 9100 metre and A. A does not live on the topmost floor. The total distance run by people living on floor number 3 and floor number 6 is 11400 metre. The one who ran for 6300 metre, lives immediately above the one who ran for 3800 metre. Neither E nor A ran for 6300 metre, Only two people live between B and the one who ran for 6300 metre. The one who lives immediately below D ran for a distance more than that of B, but not the most. No one lives between H and the one who ran for 7200 metre. E did not ran for 7200 metre. The one who ran for 10000 metre lives immediately above D. C and G together ran for 16,300 metre, C ran more than G. 132. If M ran for 2000 metre more than B, then for how many metres did M run? (1) 5200 metre (2) 5800 metre (3) 4900 metre (4) 7000 metre (5) 4300 metre 133. Which of the following statements is true with respect to the given arrangement? (1) D lives on the topmost floor. (2) Only two people live between G and F. (3) None of the given options is true (4) The one who ran for 2300 metre lives immediately above F. (5) A ran for 9100 metre 134. Who amongst the following lives immediately above the one who ran for 7200 metre? (1) The one who ran for 10000 metre (2) H (3) F

    DATA ANALYSIS

    Co lu m n– 1 Co lu m n– 2 Co lu m n– 3

    (4) The one who ran for 3800 metre (5) G 135. Which of the given combinations is correct as per the given arrangement? (1) Floor number 7–3800 metre (2) Floor number 3–E (3) C–9100 metre (4) D–5000 metre (5) Floor number 2–7200 metre 136. How many metres did F and A together run? (1) 16200 metre (2) 7300 metre (3) 12900 metre (4) 15300 metre (5) 11400 metre Directions (137–141) : Study the following information carefully and answer the questions given below : Nine different books on Economics, Commerce, Physics, Chemistry, Psychology, Anthropology, Botany, History and Mathematics are placed in different cells but not necessarily in the same order. All the cells are facing north direction.

    Row–3 Row–2

    Economics book is in immediate right of a book that is immediate left below diagonal of Botany book. 137. Which book is in two cells above the Psychology book in the same column? (1) Chemistry (2) Commerce (3) Anthropology (4) Botany (5) Mathematics 138. Which book is in immediate left below diagonal of Economics book? (1) Commerce (2) Mathematics (3) Anthropology (4) Botany (5) History 139. Which of the following book is in 2nd row and in 1st column? (1) Commerce (2) History (3) Chemistry (4) Psychology (5) Mathematics 140. Which of the following book is in 1st row and 2nd column? (1) Economics (2) Psychology (3) Physics (4) Chemistry (5) Commerce 141. Which of the following book is in 3rd column and 3rd row? (1) Mathematics (2) History (3) Physics (4) Chemistry (5) Commerce ●

    SHORT ANSWERS Row–1

    65. (2) 69. (4) 73. (4) 77. (3) 81. (3) 85. (1) 89. (4) 93. (4) 97. (2) 101. (2) 105. (3) 109. (4) 113. (5) 117. (5) 121. (1)

    66. (2) 70. (3) 74. (3) 78. (4) 82. (4) 86. (2) 90. (1) 94. (4) 98. (2) 102. (4) 106. (1) 110. (5) 114. (3) 118. (4) 122. (3)

    67. (4) 71. (1) 75. (2) 79. (2) 83. (4) 87. (3) 91. (1) 95. (4) 99. (1) 103. (2) 107. (2) 111. (2) 115. (5) 119. (2) 123. (2)

    68. (2) 72. (3) 76. (1) 80. (1) 84. (2) 88. (2) 92. (2) 96. (2) 100. (2) 104. (5) 108. (2) 112. (3) 116. (4) 120. (3) 124. (5)

    125. (1) 129. (3) 133. (4) 137. (4) 141. (3)

    126. (3) 130. (5) 134. (1) 138. (3)

    127. (4) 131. (3) 135. (2) 139. (3)

    128. (2) 132. (5) 136. (3) 140. (2)

    EXPLANATIONS 1. (2) MBC does not form a team because C cannot work with M. 2. (2) If A work then O also works hence, On can work with A. 3. (3) C cannot work with M. 4. (1) B and C cannot have O because O works with M. However B and C can have N with them. (5-10) : Maths Athletics Studies Gk Art Asha





    Charu





    Deepa

    Psychology book is in immediate below cell of a book that is to the immediate right of Chemistry book ● Physics book is immediate right above diagonal cell of a book that is in immediate diagonal right below Commerce book ● Anthropology book is in the same column of Chemistry book ● Botany book is in immediate left of a book that is in the same column of Mathematics book ● History book is in immediate right above diagonal cell of a book that is in immediate right of Anthropology book ● Commerce book is second left of a book that is in two cells above Mathematics book in the same row. ●

    1. (2) 5. (4) 9. (4) 13. (3) 17. (4) 21. (3) 25. (3) 29. (4) 33. (1) 37. (3) 41. (3) 45. (2) 49. (2) 53. (3) 57. (3) 61. (3)

    2. (2) 6. (2) 10. (2) 14. (2) 18. (2) 22. (1) 26. (3) 30. (2) 34. (2) 38. (3) 42. (4) 46. (1) 50. (1) 54. (4) 58. (1) 62. (2)

    3. (3) 7. (1) 11. (2) 15. (1) 19. (4) 23. (2) 27. (3) 31. (4) 35. (3) 39. (2) 43. (2) 47. (2) 51. (1) 55. (3) 59. (4) 63. (3)

    BPRE–986

    4. (1) 8. (3) 12. (3) 16. (3) 20. (4) 24. (1) 28. (4) 32. (4) 36. (4) 40. (2) 44. (3) 48. (3) 52. (4) 56. (4) 60. (1) 64. (1)

    Beena Ela



    ✔ ✔







    ✔ ✔







    5. (4) From the above table, it is clear that Deepa is good in Studies, General Knowledge, Atheletics and Art. 6. (2) Beena is good in Studies, General Knowledge and Mathematics. 7. (1) Asha is good in Studies, Mathematics and Atheletics. 8. (3) Charu is good in Atheletics, General Knowledge and Mathematics. 9. (4) Ela is good in studies, GK and Art but not in Atheletics. 10. (2) Height wise ranking in the decreasing order is given below C→B→A→D→E

    DATA ANALYSIS 11. (2) Keeping in view all the conditions given in the question, B can be chosen in the following two ways : Mathematicians Physicists (I) ABC FH or GH (II) BCD GH From (I) Two physicists are FH and GH From (II) Only one way of choosing physicists exists. Hence, If B is chosen then G will also be chosen. 12. (3) If A is chosen then mathematician chosen are ABC and physicists chosen are F and H or G and H. If D is chosen, mathematicians chosen are B, C and D and physicists chosen are GH. From the above two cases, it is clear that whether A is chosen or D is chosen either F or G is chosen. 13. (3) If G is rejected then, the member selected would be ABC-FH. Hence, if G is rejected D cannot be in the group. 14. (2) If H is chosen then the group of mathematician will definitely include B. 15. (1) On call X, Mr Basco and Mr Parda exchange partners hence, the pair of partner would be Mr Basco and Mrs Parda 16. (3) Clearly if X is followed by X or Y followed by Y or Z followed by Z, then each dancer is paired with his or her own spouse after such two calls. 17. (4) Call X : Mr Basco Mrs Parda Mr Parda Mrs Basco Call Y : Mr Roberts Mrs Jefferson Mr Jefferson Mrs Roberts 18. (2) It must have been y is true of the first call. 19. (4) Call X : Mr Basco Mrs Parda Mr Parda Mrs Basco Call Z : Mr Jefferson Mrs Basco Mr Parda Mrs Jefferson 20. (4) Call Z Mr Jefferson Mrs Parda Mr Parda Mrs Jefferson Call X : Mr Basco Mrs Jefferson Mr Parda Mrs Basco

    Call Y : Mrs Roberts Mr Basco Mrs Jefferson Mr Roberts 21. (3) A student can take maximum five courses namely— History, Psychology, Chemistry, Mathematics and Astronomy. 22. (1) Only in one way, other two courses can be taken. .Astronomy, Mathematics, History, Psychology. 23. (2) Could be violating conditions (3)—Chemistry, Psychology and one more subject (History), because English cannot be taken. Chemistry is a prerequisite for Biology, Hence, Chemistry is supposed to be repeated 24. (1) Option (1) that E finishes second is not true because either C or G has to be first and D has to come before E. 25. (3) If D finishes third then F will finish second. Hence, F finishes ahead of E. Hence, option (3) is the correct option. 26. (3) If C finishes first, then G will finish last then there will be following three orders. C D E F G, C F D E G and C D F EG 27. (3) If F finishes ahead D, then F will definitely finish at second place. 28. (4) If there is one golfer between C and D then E finishes at fourth place. 29. (4) Since W must be immediately to the left of X, therefore X cannot be displayed in window no. 1. 30. (2) Since W most be immediately to the left of X, therefore W must be displayed in window no. 2. 31. (4) If U is placed in window no, 5, then Y will be placed in window number 6. 32. (4) Three panels can be as given below (I) Unni — Shekhar Cyriac—Suresh Bhushan — Ram Pramila does not participate. OR (II) Unni — Shekhar Cyriac — Suresh Pramila — Ram Bhushan does not participate. (III) Unni — Shekhar

    BPRE–987

    Bhushan — Suresh Pramila — Ram Cyriac does not participate. 33. (1) Suresh and Unni are experts in Basic sciences. 34. (2) The possible panels are (I) Cyriac — Shekhar (II) Suresh — Ram (III) Unni — Bhushan (35-36) : 2 union representatives and 3 management representatives can sit along the octagonal table as below U M U

    M M

    35. (3) Since, no two union and management representatives can sit together., hence there can be not more than one management representative. 36. (4) According to information, related opposite of the first management representative must be a management person. 37. (3) The last program to be operated on any day must be either S or U. Hence, option (i) is ruled out. From conditions (ii) and (iii), options (4) and (i) are ruled out respectively. 38. (3) If R is operated at fourth place, V must be operated at fifth place (conditional (iii). This violates condition (iv). 39. (2) If R is operated at third place, Q cannot be operated at second place because T must be operated after that which is nor possible. 40. (2) The candidate satisfies all the four essential conditions. Hence, he is to be appointed. 41. (3) The candidate does not satisfy condition III. But whether he has worked as Marketing Officer for at least three years is not given. 42. (4) The candidate does not satisfy condition IV. But she has a diploma in Marketing Management. Hence, her case is to be referred to General manager, Marketing.

    DATA ANALYSIS 43. (2) K and R go together. Hence, option (1) ruled out. O cannot be placed with N. Hence, option (3) is ruled out. 44. (3) K and R go together. Hence, options (1), (2), and (4) are ruled out. 45. (2) O cannot be placed with N. 46. (1) N goes with H. Hence option (2) ruled out. P and Q have to be together. Hence option (4) is ruled out. 47. (2) If the team has three girls including J and K, the members of the team other than J and K are MORG. (48-52) : On the basis of information given in the question, the following arrangement can be made. 5 6 7 8 9 10 Mon Tue Wed Thu Fri Sat D B E C F A 48. (3) The organisation would start from D. 49. (2) E is to be organised on 7th. 50. (1) The organisation would end with A. 51. (1) B is to be organised on Tuesday. 52. (4) None of These. 53. (3) Since, Naveen has been chosen, Dalbir and Paul cannot be chosen. Therefore, Pilots chosen are Eric and Farid. Lan cannot be chosen because of Eric. Therefore, Montly is the 2nd Engineer. 54. (4) Eric and Lal cannot be chosen together. Hence, option (4) is not possible. 55. (3) If Dalbir is rejected, either of Naveen or Paul and Monty must be chosen. Also, Farid must be in the group If Dalbir is chosen, then Farid, Lal and Monty can be chosen. Hence, A and B are possible choices while C is not. 56. (4) When Lal is chosen, Eric cannot be in the group. Therefore, Dalbir and Farid must be chosen and then Naveen cannot be chosen. Therefore, B is not a possible combination. If Lal is chosen, Eric cannot be chosen and so Farid is to be definitely chosen. Therefore, A is true.

    If Monty is chosen. then it is not necessary that Naveen must also be chosen, Dalbir, Eric and Farid can be chosen with Monty. Therefore, B is not true. C is also true because one out of Dalbir, and Eric will definitely be chosen. And thus, if Dalbir is chosen, Naveen cannot be chosen and if Eric is chosen, Lal cannot be chosen. (57-59) : Paint A B C D E F G Tin Q S T V U P R Colour Blue White Black Orange Yellow Green Red 57. (3) Paint D is orange in colour. 58. (1) Paint B is kept in tin S. 59. (4) Tin Q contains blue paint. (60-63) : We have A:B = 2:1 B:C = 9:2 C:D = 1:2 D:E = 1:2 A:B :C:D:E 2 : 1 9 : 2 1 : 2 1 : 2 2 × 9 × 1 × 1 : 1 × 9 × 11 : 1 × 2 × 1 ×1:21×2×2×1:1×2×2×2 = 18 : 9 : 2 : 4 : 8 ∴ A > B > E > D > C. 60. (1) C is the lightest in weight. 61. (3) E is lighter in weight than A, B articles. 62. (2) E is heavier than D, C articles. 63. (3) A is the heaviest in the weight. 64. (1) Wherever candidate B appears, A should also be present. Hence option (2) is ruled out. If it becomes necessary to call applicants for additional interviews, no more than one such applicant should be asked to appear the next week. Hence, options (3) and ( B = H > N > P’ is definitely true? (1) P < B (2) H < V (4) E > N (3) N > V (5) E > P (IBPS RRBs Officer Scale–I & II CWE 12.09.2015)

    Directions (2-4) : Read the following paragraph and answer the questions which follow : (UCO Bank PO Exam. 30.01.2011)

    It is true that the green revolution has succeeded in transforming the Indian economy from a situation of severe food shortage into one where the country has not only become selfreliant in food production but has also been able to generate a sizable surplus for export. However, the high doses of chemical fertilizers and pesticides, which have been used under this ‘conventional’ farming technology (along with ‘high yielding varieties of seeds’ and irrigation) are now reversing the trend and causing severe environmental and health hazards, including the contamination of ground water, 2. Which of the following statements would strengthen the author’s argument? (1) India faced severe food shortages before the inception of green revolution. (2) Organic farming i.e. farming without the use of chemical fertilizers makes the food products too expensive at retail level. (3) Many farmers have reported that the soil in their fields became infertile within five years of switching over to chemical fertilizers. (4) Farmers not relying on the hybrid and high yielding variety of seeds as propagated by green revolution do not make as much profit as the ones using methods of green revolution.

    (5) High yielding varieties of seeds as propagated by green revolution do not require much water for irrigation purposes and are also resistant to pest attacks. 3. Which of the following can be inferred? (An inference is something which is not directly stated but can be inferred from the given facts.) (1) Many countries have banned use of chemical fertilizers (2) Green revolution failed miserably to do any good to India’s needs. (3) The negative effects of techniques of green revolution were not anticipated at its inception (4) India will never face food shortage again (5) The main focus of green revolution was to help international fertilizer manufacturers sell their products in India. 4. Which of the following statements represents a possible effect of the conventional Green Revolution techniques of farming if it continues in the days to come ? (1) India would never face severe food shortage again in the future (2) Export of food products would increase manifold (3) Only ground water would be used for irrigation purpose (4) Cases of chemical poisoning would increase substantially (5) Farmers not following methods of Green Revolution would face severe health hazards Directions (5–9) : In each of the questions below is given a statement, followed by a question. Read the statement carefully and answer the question that follows. (IBPS Specialist Office CWE 11.03.2012)

    5. Statement : The State Government granted special status to the seminar on the importance

    BPRE–1013

    of protecting the environment around us organised by the local academic body to create awareness among the citizens. Which of the following may be a probable reason for the State Government’s decision? (1) The Central Government has accorded highest priority to issues related to protection of environment, over all other issues. (2) In the past the State Government had granted special status to such seminars (3) The State Government has failed to implement measures for protecting the environment (4) The local academic body made a fervent appeal for the special status (5) None of these 6. Statement : Most car manufacturers have marginally reduced the price of their products despite higher input costs and increased Government duties and have promised to keep the prices at the present level for at least the next couple of months. Which of the following can be a best possible reason for the above step by car manufacturers? (1) Car manufacturers have not been able to meet their sales target for the current year so far. (2) The Government is planning to review taxes on the raw materials for manufacturing cars. (3) The car sales market is going through a jubilant phase as the volume of sales has picked up considerably in the recent past and profit per car has also gone up. (4) Car buyers are still hesitant to make their purchases and may postpone for another few months. (5) Oil marketing companies are contemplating reviewing the petrol and diesel prices in order to decide future market prices.

    MISCELLANEOUS 7. Statement : The prices of vegetables and other food articles have decreased in the recent months raising hope among policy planners that the RBI’s (Reserve Bank of India) tight grip on supply of liquid money in the market for controlling inflation may be eased. Which of the following may be a possible action of the above situation? (1) The Reserve Bank of India may not reduce its key interest rates in near future. (2) The Government may step in and make certain concessions to the common people on various issues (3) The Reserve Bank of India may consider lowering cer tain rates to bring in more liquidity in the market. (4) The RBI may wait for at least another year before taking any step. (5) The RBI may collect more data from the market and wait for another four months to ensure they take the correct step. 8. Statement : A severe cyclonic storm hit the Eastern coastline last month resulting in huge loss of life and property on the entire east coast and the Government had to disburse a considerable amount for relief activities through the district administration machineries. Which of the following may possibly be a follow up measure to be taken up by the Government? (1) The Government may set up a task force to review the post relief scenario in all districts and also to confirm proper end user receipt of the relief supplies. (2) The Government may set up a committee for proper disbursement of relief supplies in future. (3) The Government may empower the District Magistrates to make all future disbursements of relief. (4) The Government may send relief supplies to the affected people in future only after proper assessment of the damage caused by such calamities.

    (5) The Government may not need to activate any follow up measure. 9. Statement : It has been reported in a recent study that intake of moderate quantity of milk chocolate reduces the risk of suffering from central nervous system related illnesses. Which of the following would weaken the findings of the study reported in the above statement? (1) People generally prefer to eat chocolate when they are young. (2) Majority of those not diagnosed with diseases related to central nervous system have stayed away from eating chocolates in their lives. (3) Chocolates contain certain elements which strengthen the functions of the central nervous system (4) Majority of those suffering from central nervous system related diseases are middle aged. (5) Many of those who suffer from diabetes also suffer from other major ailments. 10. How many 3-digit numbers can be formed using the 4th, 7th and 9th digits of the number 937862541 each of which is completely divisible by 7 ? (1) None (2) One (3) Two (4) Three (5) More than three (IBPS Bank PO/MT CWE-III, 26.10.2013)

    11. Many sociologists have argued that there is a functional relationship between education and economic systems. They point to the fact that mass formal education began in industrial society and is an established part of all industrial societies. Which of the following inferences can be drawn from the above statements ? (1) Formal education has suffered a setback due to industrial society (2) Industrial society is responsible for expansion of education of the mass level (3) Formal education can be traced to industrial society. (4) Industrial society changed the pattern of education (5) None of these (IBPS Bank PO/MT C WE-III, 26.10.2013)

    BPRE–1014

    12. Statement : According to a survey conducted by a magazine Pulse, College XYZ ranked best among all the MBA institutes in the country. Which of the following statements substantiates the fact expressed in the above statement ? (1) A large number of students of College XYZ find better placement after completing their course (2) There are many institutes which have better infrastructure than the College XYZ (3) The rate of enrolment in the College XYZ has come down in the recent years (4) Majority of the institutes have contradicted the findings of the magazine Pulse (5) None of these (IBPS Bank PO/MT CWE-III, 26.10.2013)

    13. Statement : The luxury car manufacturer Phantom has increased the price of its stunning, fastest Limousine Car by about 45 per cent. Which of the following can be a best possible reason for the above step taken by car manufacturer ? (1) The government is planning to review taxes on the raw materials for manufacturing cars (2) The car sales market is going through a jubilant phase as the volume of sales has picked up considerably in the recent past. (3) The Limousine car is the best in its segment and no other car is posing any challenge to this car (4) Car buyers prefer to buy Limousine car as it provides efficient service after sale. (5) None of these (IBPS Bank PO/MT CWE-III, 26.10.2013)

    14. Statement : A first past the post election is one that is won by the candidate with more votes than others. Highest vote, majority vote and plurality voting system use a first past the post voting method but there are subtle differences in the method of execution.

    MISCELLANEOUS Which of the following can be the best reason for the assertion made in the above statement ? (1) It is not possible to execute the plurality voting system. (2) The plurality voting system uses very complex procedure so it is not a practical approach (3) A system which is simple gets wide acceptance. (4) The method is not so important in electing a candidate (5) None of these (IBPS Bank PO/MT CWE-III, 26.10.2013)

    15. Study the following information carefully and answer the question given below : The government of State Y is promoting organic farming this year onwards. Thus, the raw material needed for it is made available at a subsidised rate. Free regular classes are also arranged to train the farmers in organic farming methods and to solve their problems. The farmers of village B however are still continuing with the industrial farming methods despite several requests from the government. Which of the following statements, if considered true, may support the action of farmers of village B? (1) Industrial farming leads to more productivity in the short term, however, in the long term, it diminishes the soil life and production becomes heavily dependent on chemicals. (2) Organic farming requires considerably more skill as compare to industrial farming. (3) Although a few farmers across State Y have tried their hand at organic faming earlier, none of the farmers of village B have participated in it before. (4) Organic food is expensive and not many people in State Y may be able to afford it, however with the subsidies, provided by the government, it will become affordable. (5) Organic farming methods are very labour intensive as regular extensive observation is required whereas finding la-

    bour at affordable costs is a challenge for farmers of village B. (IBPS RRBs Officer Scale–I & II CWE 12.09.2015)

    Directions (16–18) : Ten flyovers are built to join two places X and Y of a city. The flyovers are — A, B, C, D, E, F, G, H, I and J. These flyovers are joined at four junctions — P, Q, R and S. All are of single lane. The distances between all the routes are also shown. The fuel used by the vehicles on these routes is 6 per kilometre. The whole arrangement is shown by the diagram given below : (IBPS Bank PO/MT CWE (Main) Exam, 18.11.2016))

    P 1 B km 1 km km C 3 2 1 1 2 km km I 2 km 2 3 X Q R Y D J F 1 1 km 3 1 H 3 G1 2 km 1 km 2 3 E S A

    16. If the routes QS and PQ are under-construction and unavailable for transportation, then what should be the toll taxes for each of junctions P, Q, R, S respectively so that the fuel cost of each vehicle must be the same upto Y ? (1) 2, 3, 4, 0 (2) 0, 6, 5, 2 2, 2, 3, 4 (3) (4) 5, 2, 3, 4 (5) None of these 17. If the route PQ is unavailable for transportation and the toll taxes for the junctions P, Q, R and S are 4, 10, 5 and 6 respectively, then from which of the following routes the least number of the vehicles will pass? (1) X Q R Y (2) X S Y (3) X Q S R Y (4) X Q S Y (5) None of these 18. If all the functions are made tollfree, then which of the following routes will be the busiest one ? (1) X P Y (2) X Q S Y (3) X Q S R Y (4) X S Y (5) None of these 19. Read the following information and answer the given question. “The man behind the revival of S-Cross Automobiles, the CEO

    BPRE–1015

    has decided to quit. I believe that the company will soon go into losses due to this decision”, statement by a market analyst. Which of the following statements appears to be the perception of the market analyst ? (1) No other person can run SCross Automobiles as successfully as the present CEO. (2) S-Cross Automobiles may decide to shut down the operations due to the CEO’s exit. (3) S-Cross Automobiles will try its best to retain the CEO in order to maintain its market position. (4) The strategies adopted by the CEO of S-Cross Automobiles were exclusive in the industry. (5) S-Cross Automobiles has the highest market share at present. (IBPS SO (Agriculture) Exam, 29.01.2017))

    20. Read the following paragraph and answer the question which follows : Bookings for the dream project of Floraville builders’ new building are underway. The building for which construction is due to start next month has flats costing a minimum of Rs. 80 lakhs. Bank B offers 100% loan scheme in which Rs. 15 lakhs is provided by the bank at the start of construction, Rs. 25 lakhs after one year of the start of construction work and Rs. 10 lakhs every subsequent year till the completion of the project. Which of the following statements would prove that the bank’s loan disbursement plan may be flawed in its approach ? (A) Cost of construction undertaken by Floraville builders varies depending on the area and labour cost even within a city. (B) Floraville builders take a minimum of three years to complete a project by which time the entire payment must be made by the buyers. (C) The purchase a flat from Ashiana builders it is mandary to pay 75% of the entire amount before the second year of start of construction.

    MISCELLANEOUS (D) Many banks do not offer flexibility in payment scheme if construction of a project does not near competition. (1) All (A), (B), (C) and (D) (2) Only (A) (3) Both (B) and (C) (4) Only (C) (5) Both (B) and (D)

    21.

    22.

    (IBPS SO (Agriculture) Exam, 29.01.2017))

    Directions (21–24) : Study the following information carefully and answer the questions given below : (Bank of Maharashtra PO Exam, 26.10.2016)

    Statement : Currently, eight out of ten software programs installed in computers owned by most of the businesses in Country X are pirated. This unauthorized software use occurs in otherwise legal businesses that buy very few software licenses for their employees’ computers. Criminal enterprises sell counterfeit copies of software programs at cut-rate prices. A. A survey, for small and middle-level enterprises, shows that buying and updating most of these authentic software programs, if installed in all the computers, would bring down their profits by about 37%. B. Most of the people currently involved in developing and selling the authentic software programs are facing a hard time as far as the market is concerned. C. Many market analysts and experts are of the view that curbing this kind of piracy will create more jobs in the software development field and also increase the tax revenues. D. The regulatory authorities of Country X should levy heavy fines on those businesses which are found to be buying/ using pirated versions of software. E. The income generated by selling pirated software is not declared by the criminal enterprises, thus no tax is paid against it. F. Individual users prefer to buy/ update software programs only from authorized sellers. G. 95% of the pirated copies of software programs are sold without any guarantee.

    23.

    24.

    Which of the following strengthen the idea that use of pirated software in businesses should be put to an end ? (1) Both (D) and (F) (2) Only (D) (3) Only (A) (4) Only (F) (5) Both (C) and (E) Which of the following can be considered as the consequence of the issue given in the statement ? (1) Only (F) (2) Both (B) and (E) (3) Only (D) (4) Both (C) and (G) (5) Only (A) Assuming that the given statement is a problem, which of the following can be a course of action ? (1) Only (D) (2) Only (E) (3) Both (F) and (G) (4) Only (A) (5) Only (G) Which of the following is in favour of those enterprises that sell pirated software programs ? (1) Only (G) (2) Only (D) (3) Only (B) (4) Only (A)

    (5) Only (F) 25. If two is subtracted from each even digit and one is added to each odd digit of the number 7631528, then which of the following digits will appear twice in the new number thus formed? (1) Both 6 and 4 (2) Only 1 (3) Both 6 and 2 (4) Only 2 (5) None (IBPS RRBs Officers CWE (Prelim Exam) 09.09.2017 Ist Sitting)

    Directions (26–28) : Following questions are based on the five three digit numbers given below : (IBPS RRBs Officers CWE (Prelim Exam) 09.09.2017 Ist Sitting)

    496 528 317 824 795 26. If ‘1’ is added to the last digit of every odd number and ‘1’ is subtracted from the last digit of every even number, what will be difference between the highest odd number and the lowest even number thus formed ? (1) 27 (2) 505 (3) 209 (4) 37 (5) 328 27. The positions of the first and the second digits of each of the numbers are interchanged. What will be the resultant if first digit of

    BPRE–1016

    highest number is divided by the second digit of the lowest number thus formed? (1) 9 (2) 2 (3) 4 (4) 3 (5) 6 28. If all the numbers are arranged in ascending order from left to right, which of the following will be sum of all the three digits of the number which is fourth from the right ? (1) 15 (2) 11 (3) 19 (4) 14 (5) 21 29. If one is subtracted from the first three digits and two is added to the last four digits of the number 6394217, then which of the following digits will appear twice in the number thus formed? (1) Both 2 and 6 (2) Only 8 (3) None (4) Both 5 and 8(5) Only 5 (IBPS RRBs Officers CWE (Prelim Exam) 09.09.2017 IInd Sitting)

    30. How many such pairs of digits are there in the number 5312647 each of which has as many digits between them in the number (in both forward and backward directions) as they have between them in arithmetic numerals? (1) Four (2) One (3) Three (4) Two (5) More than four (IBPS RRBs Officers CWE (Prelim Exam) 09.09.2017 IInd Sitting)

    31. If three is added to the first four digits and two is subtracted from the last three digits of the number 6324589, then which of the following digits will appear twice in the new number thus formed ? (1) Only 6 (2) Both 6 and 7 (3) Both 3 and 9 (4) Only 3 (5) None (IBPS RRBs Officers CWE (Prelim Exam) 10.09.2017)

    32. How many such pairs of digits are there in the number 1468572 each of which has as many digits between them in the number (in both forward and backward directions) as they have between them in arithmetic numerals ?z (1) One (2) None (3) Three (4) Two (5) More than three (IBPS RRBs Officers CWE (Prelim Exam) 10.09.2017)

    MISCELLANEOUS 33. If three is added to each even digit and two is added to each odd digit of the number 7 1 4 5 3 6 2, then which of the following digits will not appear twice in the new number thus formed? (1) Only 3 (2) Only 9 (3) Both 3 and 5 (4) Both 7 and 9 (5) Only 5 (IBPS RRBs Officer CWE (Prelim Exam) 16.09.2017)

    34. How many such pairs of digits are there in the number 6 9 7 3 5 2 4 each of which has as many digits between them in the number (in both forward and backward directions) as they have between them in arithmetic numerals? (1) Three (2) One (3) More than four (4) Four (5) Two (IBPS RRBs Officer CWE (Prelim Exam) 16.09.2017)

    35. If 2 is added to all odd digits of the number 4738261 and 1 is subtracted from all the even digits, then how many digits will be greater than 5 in the new number thus formed? (1) Five (2) Two (3) Four (4) Three (5) One (IBPS SO (IT Officer) CWE (Prelim Exam) 30.12.2017)

    36. A correctly remembers that B’s birthday is in the month of September. C correctly remembers B’s birthday is after 18th September. D correctly remembers that B’s birthday is on a Friday and before 28th September. If the first day of September was a Wednesday that year then on which date is B’s birthday? (1) 27th (2) 21st (3) 19th (4) 22nd (5) 24th 37. Six people S, T, U, V, W and X were born on six different days of the same week starting from Tuesday and ending on Sunday. V was born on Thursday. Only one person was born between V and X. S was born immediately after X. As many people were born after S as before T. Only two people were born between T and U. How many people were born after U? (1) More than three (2) One

    (3) Three (4) None (5) Two (IBPS Bank PO/MT CWE (Prelim Exam) 14.10.2018)

    38.

    Who among U, V, W, X and Z is heaviest? Statement I: W is shorter than V and equal to X, Z is not taller than U and V Statement II: X is taller than Z and equal to W. V and U are not shorter than W (1) Data in either statement I alone or statements II alone is sufficient to answer the question (2) Data in only statement II is sufficient to answer the question (3) Data in only statement I is sufficient to answer the question (4) Data in statement I and statement II together are not sufficient to answer the question (5) Data in statement I and statement II together are not sufficient to answer the question

    (IBPS Bank PO/MT CWE (Main Exam) 18.11.2018)

    39. If A < B > C > D; M < B = K; T > C > Y then which of the following definitely holds true? (1) T > M (2) M < A (3) T > A (4) D > K (5) Y < K (IBPS SO Prelim Exam, 28.12.2019)

    40. In which of the given expressions does the expression S > Z definitely holds true? (1) P > Q < R < S; Z > Y > Q (2) P = Q < R = S; Z = Y > Q (3) P > Q < R < S = E; Z=Y Q = R < S; Z < Y > Q (5) P = Q > R = S; Z < Y < Q (IBPS SO Prelim Exam, 28.12.2019)

    41. What should come in place of @ and # respectively in the given expressions X > Y = Z @ T > U; X # R < J, so that the expression J > U definitely holds true? (1) =, < (2) >, < (3) (4) >, > (5) =, > (IBPS SO Prelim Exam, 28.12.2019)

    42. Which of the following symbols should be placed in the blank spaces respectively (in the same

    BPRE–1017

    order from left to right) in order to complete the given expression in such a manner that “S > U” definitely holds true? S–T–P–U (2) , >, < (3) >, =, = (4) B = H > N > P P < B : True H < V : Not True N > V : Not True E > N : Not True E > P : Not True 2. (5) Statement (5) would strengthen the author’s argument. 3. (3) It is clear from the paragraph that the negative effect of techniques of green revolution were not anticipated in the beginning. 4. (4) Cases of chemical poisoning would increase substantially. 5. (1) Option (1) may be a probable reason for the State Government’s decision. 6. (1) Option (1) may be a best possible reason. 7. (3) Option (3) may be a possible action. 8. (2) Option (2) may be a follow up measure. 9. (5) Option (5) would weaken the findings of the study. 10. (3) 9 3 7

    8

    62

    5

    4 1

    581 518 = 74; = 83 7 7 11. (2) From the given statements, it is clear that industrial society is responsible for expansion of education of the mass level. 12. (1) Obviously option (1) substantiates the fact given in the statement. 13. (3) The company decided to increase the price of car as there

    BPRE–1025

    2

    SY = 1

    2 ×6= 3

    10

    1 ×6= 3 2 16. (2) In case of unavalilability of the routes QS and PQ the following may be the different route arrangements : XPY, XQRY, XSRY and XSY From answer option (2), Fuel cost for XPY = 12 + 3 + 6 = 21 Fuel cost for XQRY = 3 + 6 + 4 + 5 + 3 = 21 Fuel cost for XSRY = 9 + 2 + 2 + 5 + 3 = 21 Fuel cost for XSY = 9 + 2 +10 = 21 17. (1) Fuel cost of XPY = 12 + 4 + 6 = 22 Fuel cost for XQRY = 3 + 10 + 4 + 5 + 3 = 25 Fuel cost for XSY = 9 + 6 + 10 = 25 Fuel cost for XQSRY = 3 + 10 2 + 5 + 2 + 5 + 3 = 30 Fuel cost XQSY = 3 + 10 + 2 + 6 + 10 = 32 Due to more fuel cost number of vehicles will be the least on the route XQSY. RY =

    MISCELLANEOUS 18. (3) Fuel cost for XPY = 12 + 6 = 18 Fuel cost for XQSY = 3 + 2 + 10 = 15 Fuel cost for XQSRY = 3 + 2 + 2 + 3 = 10 Fuel cost XSY = 9 + 10 = 19 Fuel cost of XQRY = 3 + 4 + 3 = 10 Due to the least fuel cost for the route XQSRY & XQRY, it will be the busiest. From given options, option (3) is correct. 19. (4) Obviously, option (4) is the most appropriate answer. 20. (3) Obviously, statements (B) and (C) prove that the bank’s loan disbursement plan is flawed in its approach. 21. (5) Obviously, both (C) and (E) strengthen the idea that use of pirated software in businesses should be put to an end. 22. (2) Obviously, both (B) and (E) are the consequences. 23. (1) Obviously, only (D) can be a course of action. 24. (4) Obviously, only (A) is in favour of those enterprises that sell pirated software programs. 25. (1) 7 +1

    6 –2

    3 +1

    1 +1

    5 +1

    2 –2

    6 –1

    3 –1

    5

    9 –1

    2

    4 +2

    8

    2

    1

    +2

    6

    +2

    4

    7 +2

    3

    9

    4

    7

    30. (5)

    5

    3

    1

    2

    6

    31. (2)

    6 3 2 4 5 8 9 +3 +3 +3 +3 –2 –2 –2 9

    6

    5

    7

    3

    6

    7

    32. (4)

    1 4 6 8 5 7 2 33. (1)

    7 +2

    1 +2

    9

    4 +3

    3

    5 +2

    7

    3

    6

    +2

    7

    +3

    5

    2 +3

    9

    5

    Digit ‘3’ appears only once. 34. (5)

    8

    6

    9

    7

    3

    5

    2

    4

    –2

    8 4 4 2 6 0 6 26. (2) 317 ⇒ 318 795 ⇒ 796 496 ⇒ 495 528 ⇒ 527 824 ⇒ 823 Highest odd number ⇒ 823 Lowest even number ⇒ 318 Required difference ⇒ 823 – 318 = 505 27. (4) 496 ⇒ 946 528 ⇒ 258 317 ⇒ 137 824 ⇒ 284 795 ⇒ 975 Highest number ⇒ 975 First digit of 975 ⇒ 9 Lowest number ⇒ 137 Second digit of 137 ⇒ 3

    9 =3 3 28. (3) 317 < 496 < 528 < 795 < 824 Fourth number from the right ⇒ 496 Required sum ⇒ 4 + 9 + 6 = 19 Required result =

    29. (3)

    35. (2)

    4 7 3 8 2 6 1 –1 +2 +2 –1 –1 –1 +2

    3 9 5 7 1 5 3 Numbers gerater than 5 ⇒ 9 and 7 36. (5) First day of September was a Wednesday. First Friday of that September ⇒ 3rd Other Fridays ⇒ 10, 17 and 24 According to C, B’s birthday is after 18th September. According to D, B’s birthday is before 28th September. Therefore, B’s birthday is on 24th September. 37. (5) Day Person Tuesday T Wednesday W Thursday V Friday U Saturday X Sunday S Two persons– X and S– were born after U.

    BPRE–1026

    38. (4) From statement I V>W=X U, V > Z From statement II W=X>Z V, U > W 39. (5) A < B > C > D MC>Y MCD K=B>C>Y Option (1) T > M : Not True Option (2) M < A : Not True Option (3) T > A : Not True Option (4) D > K : Not True Option (5) Y < K : True 40. (3) Option (1) P>QQ Z>Y>QQ Z : Not True Option (3) P>Q U 42. (3) S > T = P = U 43. (2)

    3 2 1 7 5 4 –1 +2 –1 –1 –1 +2 2 4 0 6 4 6 4 and 6 are repeated more than once.

    MISCELLANEOUS

    SBI PO EXAMS 1. (5) Statements (B) and (C) are true. Charity stores do not impose unnecessary burden on the customers. Charity stores receive some amount as donations. It is clear that VAT increases the price of things and stores collect taxes from the customers. 2. (5) Statements (B) and (C) are true. Charity stores do not impose unnecessary burden on the customers. Charity stores receive some amount as donations. It is clear that VAT increases the price of things and stores collect taxes from the customers. 3. (5) Only statement (iv) is not in line with the given statement. 4. (3) Only statement III abrogates the given statement.

    RBI GRADE–B/NABARD GRADE–A OFFICER EXAMS 1. (1) Option (1) is an assumption. 2. (5) Obviously, option (5) is the appropriate reason of the given facts. People generally consume the quantity contained in a sachets at a time. They do not want to store the ingredient of the sachet after opening it. They think it better to consume the ingredient at once. This necessarily enhances the sale of the products. 3. (1) Only option (1) seems to be a proper follow–up measure to be taken by the Government. 4. (2) Only II and IV can be concluded from the given findings of the research. 5. (4) Obviously, statement (4) weakens the decision of the Government of Country of reducing the allocation for the welfare of agriculture sector. 6. (4) Obviously, statement (4) does not weaken the statement of the tourism minister of country X. 7. (3) Obviously, statement (3) does not strengthen the given statement that neograss is a good alternative source of energy. 8. (3) Obviosly, option (3) strengthens the decision of Company P. 9. (4) Obviously, option (4) proves the decision of Railway Ministry for State X was wrong. State X has got less than Rs. 40 crore.

    10. (5) Both the statements I and II strengthen the decision. The captain of the team was guilty of multiple faults. Therefore, it was right to replace the captain. 11. (4) Obviously, option (4) supports the demand of the commercial spaces to revoke limitations on electricity usage. 12. (1) Obviously, option (1) supports the program introduced in the Country B. 13. (5) Obviously, only option (5) can be assumed from the given information. 14. (3) Debater 1 suggests punishment for only those new channels that present or spread fake news. Therefore, A is not valid. The authencity of news does not depend on perspective of the viewer. Thus, only B seems to be valid. (15–17) : G > D > B, E G, F > C > B F>G B does not run the least. F does not run the most. Thus, E runs the least and A runs the most.

    A > F > G > C/D > B > E 17 km. 8 km. 15. (5) F + B = 23 km. ∴ B = (23 – 17) km. = 6 km. B + D = 19 km. ∴ D = (19 – 6) km. = 13 km. Therefore, A > F > G > D > C > B > E 17 km.

    13 km. 6 km. 8 km. Statement (C) is not true as A + C = More than 25 km. Statements (A) and (B) are true.

    be stopped completely. It implies that the groundwater in village R contains high amount of trinium. The source of roassium may be other than groundwater. Therefore, statement (C) does not follow. It is mentioned in the very first line that high amount of ‘trinium’ blocks the absorption of ‘roassium’. Therefore, statement (D) does not follow.

    INSURANCE EXAMS 1. (2) All fathers are males and all males are human beings.

    HB M F

    HB = Human Being M = Male F = Father 2. (1) The required region should be present only in the triangle. Such region is marked ‘9’. 3. (2) The required region should be common to the circle and the ellipse. Such region is marked ‘1’. 4. (1) Only Indian who know English is represented by ‘10’. 5. (4) Doctors who know English is represented by ‘8’. 6. (3) Only Asian Post Graduates are represented by ‘2’. 7. (1) Asian Post Graduates who know English is represented by ‘5’.

    16. (2) A runs the highest kilometres. 17. (2) G runs 14, 15 or 16 kilome-

    20 = 16 100 16 families own a car each. Remaining families = 80 – 16 = 64 Number of families having a motor cycle each

    tres. 18. (3) Only (A) and (B) can be concluded from the given information. The first line implies that jute plants need roassium in order to grow well. In order to address the problem of trinium, the use of groundwater for irrigation

    50 = 32 100 Total number of families having a car or motor cycle = 16 + 32 = 48 The number of families having no vehicle = 80 – 48 = 32

    BPRE–1027

    8. (2) 80 ×

    = 64 ×

    MISCELLANEOUS 9. (2) Suppose the present age of Ashok is x years. Present age of mother is y years. 5 years ago 3 (x – 5) = (y – 5) ⇒ 3x – 15 = y – 5 ⇒ 3x – y = 10 ......(i) 5 years hence 2 (x + 5) = y + 5 ⇒ 2x + 10 = y + 5 ⇒ 2x – y = –5 ......(ii) From equations (i) and (ii) x = 15 years and y = 35 years 10. (1) 11. (3) Meaningful order of the words :

    2. Word 4. Phrase 1. Sentence

    14. (2) Meaningful order of the words : 3. Infant

    ↓ 2. Child

    ↓ 4. Boy

    ↓ 1. Adult 15. (4) Meaningful sequence of the words : 4. Measure ↓ 3. Mark ↓ 1. Cut ↓ 5. Tailor ↓ 2. Put on 16. (4) Boys are different from girls. Some boys may be students. Some girls may be students.

    Boys

    Girls

    Stu

    3. Chapter

    de n

    ts

    5. Paragraph

    19. (2) Some whimsical people are frustrated and frustrated people are prone to be drug addicts. Therefore, some whimsical people may be drug addits. Therefore, conclusion (2) follows. 20. (2) Out of the 27 small cubes, the cubes having only one side painted are those which lie at the centre of each face of the big cube. Since there are six faces of a cube, therefore, the required number of cubes is six.

    21. (2) Total number of possible outcomes 9 = C =9 1 Favourable number of outcomes = selecting one marble out of 2 red and 3 green marbles 5 = C =5 1

    5 9

    Required probability = B

    17. (3)

    O

    N

    Q

    G

    J

    I

    169 + 64 + 81 = 13 + 8 + 9 = 30 Third Row

    C

    J H

    1296 + 576 + 100

    D

    H

    G D

    I

    A

    P

    22. (1) First Row

    B

    M

    L K

    F

    E

    12. (3) A

    C

    The rectangles are : AEML; LMOK; KPJD; POIJ; EFNO; FBGN; OGHQ; QHCI; AEOK; KOID; EBGO; OGCI; AFNK; ABGK; LMID; AEID; EBHQ; EBCI; PGCJ; KGCD; ABCD 13. (2) 5th January, 1966 ⇒ Wednesday 5th January, 1967 ⇒ Thursday 5th January, 1968 ⇒ Friday 1968 was a leap year. Therefore, 5th January, 1969 ⇒ Sunday 5th January, 1970 ⇒ Monday And, 5th January, 1971 ⇒ Tuesday

    F

    E

    The triangles are : ∆ABI ; ∆BIC ; ∆AIJ ; ∆CIJ ; ∆AHJ ; ∆CDJ ; ∆JHG ; ∆JDE ; ∆GJF ; ∆EJF ; ∆ABC ; ∆BJC ; ∆JCA ; ∆AJB ; ∆JEC ; ∆JGE ; ∆ACG ; ∆CGE ; ∆ACE ; ∆AGE ; ∆AJG 18. (3) Meaningful sequence of the words : 1. Key ↓ 3. Lock ↓ 2. Door ↓ 4. Room ↓ 5. Switch on

    BPRE–1028

    = 36 + 24 + 10 = 70 Second Row

    625 + ? + 49 = 50 ⇒ 25 + ? + 7 = 50 ⇒ ? = 50 − 32 = 18 ∴ ? = (18)2 = 324 23. (3) First Column 4 × 7 – 1 = 27 Second Column 5 × 8 – 2 = 38 Third Column 6 × 9 – 3 = 51 24. (1)

    4

    8 +4

    16 +8

    +4

    28 +12

    +4

    +4

    44 +16

    64 +20

    +4

    MISCELLANEOUS 25. (1) 22 × 3 = 66 66 × 3 = 198

    Similarly,

    B7

    198 × 3 = 594

    B5

    26. (3)

    B3 4

    8 +4

    16 +8

    +4

    27. (2)

    (1)3

    28 +12

    +4

    44

    +20

    +16

    +4

    64

    +4

    =1

    B1

    B8 B6

    225 + 196 + 169 ⇒ 15 + 14 + 13 = 42 36. (4)

    1089 × 289 × 169 ⇒ 33 × 17 × 13 = 7293

    B4

    Similarly,

    B2

    81 × 529 × 361 ⇒ 9 × 23 × 19 = 3933

    Layer Q I

    49 + 25 + 9 ⇒ 7 + 5 + 3 = 15 ⇒ (15)3 = 3375

    37. (3)

    Layer Q II Layer Q III

    (2)3 = 8

    Similarly,

    (3)3 = 27

    Layer Q IV

    (4)3 = 64

    4 + 36 + 100 ⇒ 2 + 6 + 10 = 18 ⇒ (18)3 = 5832

    Stack B

    28. (3) 5 × 3 + 4 = 19 7 × 5 + 6 = 41 6 × 4 + 5 = 29 29. (3) There are one, two, three and four dots on the faces adjacent to the five dots. Therefore, six dots lie on the face opposite to five dots. If we rotate the first dice upward two dots would be hidden and four dots would be on the top. In this position one dot would be on the face adjacent to four dots. Therefore, three dots lie on the face opposite to four dots. 30. (4) The two stacks formed by cutting each of the two blocks, obtained from the cube, into 32 cubes of the same size and colouring them in different patterns as mentioned in the question can be analysed separately in the following manner :

    A7 A5 A3 A1

    A2

    Layer P I Layer P II

    A4

    A8 A6

    Stack A : Two larger faces – the right hand side and the left hand side - are coloured red and remaining faces coloured green Stack B : Two smaller adjacent faces – top and front - are coloured green and remaining faces coloured red. There is no cube in Stack A which has two red faces. Two cubes in Columns B7 and B8 in Layer Q I and two cubes in columns B1 and B2 in Layer Q IV in Stack B have two red faces and one green face. Thus, there are 2 + 2 = 4 such cubes. 31. (2) It is clear from the diagram when c is at top, a will be at the bottom. 32. (1)Grasshoppers destroy crops on a large scale. So, it is necessary to protect crops from grasshoppers. Obviously, Option (1) seems to be appropriate. 33. (4) 16 + 1 = 17; 17 + 2 = 19; 19 + 3 = 22; 22 + 4 = 26; 26 + 5 =

    31 + 6 = 37; 37 + 5 = 42 34. (2) 6 × 6 = 9 × 4 = 36 24 × 3 = 9 × 8 = 72 15 × 6 = 9 × ?

    ∴ ?=

    Layer P III Layer P IV

    90 = 10 9

    35. (3) Stack A

    31

    841 + 784 + 729 ⇒ 29 + 28 + 27 = 84

    BPRE–1029

    841 + 784 + 729 ⇒ 29 + 28 + 27 = 84

    38. (3)

    Similarly,

    225 + 196 + 169 ⇒ 15 + 14 + 13 = 42 39. (4)

    1089 × 289 × 169 ⇒ 33 × 17 × 13 = 7293 Similarly,

    81 × 529 × 361 ⇒ 9 × 23 × 19 = 3933 49 + 25 + 9 ⇒ 7 + 5 + 3 = 15 ⇒ (15)3 = 3375

    40. (3)

    Similarly,

    4 + 36 + 100 ⇒ 2 + 6 + 10 = 18 ⇒ (18)3 = 5832 41. (4) T is heavier than R but lighter than P. Maximum weight of Q would be 84 kg. as P’s weight is 85 kg. (All weights are in whole numbers). Q is heavier than T. Therefore, T’s weight could not be 84 kg. ∴ Weight of T could be 70 kg. 42. (3) Five persons are lighter than U. 43. (1) P + S = 175 kg. ∴ S = (175 – 85) kg. = 90 kg. P + Q = 160 kg. ∴ Q = (160 – 85) kg. = 75 kg. Now, S + Q = (90 + 75) kg. = 165 kg. ❐❐❐

    MISCELLANEOUS

    MODEL EXERCISES Directions (1-5) : In these questions choose, one of the figures labelled 1, 2, 3 or 4 which best represents the relationship among the items given. (1)

    (3)

    (2)

    (4)

    (5) None of these 1. Mangoes, Apples, Fruits 2. Coffee, Tea. Beverages 3. Musicians, Men, Women 4. Parrots, Birds, Mice 5. Fish, Herring, Animals living in water Directions (6-15) : Read the following information carefully to answer these questions. A sample poll of 200 votes revealed the following information concerning three candidates A, B and C of a certain party who were running for three different offices. 28 in favour of both A and B. 98 in favour of A or B but not C. 42 in favour of B but not A or C. 122 in favour of B or C but not A. 64 in favour of C but not A or B 14 in favour of A and C but not B. 6. How many voters were in favour of all the three candidates? (1) 14 (2) 8 (3) 20 (4) 16 (5) None of these 7. How many voters were in favour of A irrespective of B or C ? (1) 78 (3) 64 (2) 42 (4) 56 (5) None of these 8. How many voters were in favour of B irrespective of A or C ? (1) 78 (2) 64 (3) 42 (4) 56 (5) None of these 9. How many voters were in favour of C irrespective of A or B ? (1) 78 (2) 102 (3) 88 (4) 86 (5) None of these

    10. How many voters were in favour of A and B but not in C ? (1) 8 (2) 20 (3) 14 (4) 16 (5) None of these 11. How many voters were in favour of only one of the candidates? (1) 58 (2) 78 (3) 106 (4) 142 (5) None of these 12. How many voters were in favour of A and C but not B? (1) 22 (2) 14 (3) 16 (4) 20 (5) None of these 13. How many voters were in favour of C alone? (1) 36 (2) 42 (3) 64 (4) 38 (5) None of these 14. How many voters were in favour of B and C but not A? (1) 16 (2) 14 (3) 42 (4) 64 (5) None of these 15. How many voters were in favour of A and C but not B? (1) 16 (2) 14 (3) 36 (4) 42 (5) None of these Directions (16-19) : Each question has a set of 4 statements. Each statement has 3 segments. Choose the alternative where the third segment can be logically deduced, using both the preceding two, but not just from one of them. 16. (A) To forgive is divine. Divine facts are rare. Forgiveness is rare. (B) G is the brother of A and the father of L. B is the wife of A. L is the daughter of G. (C) Pepsi contains added flavour. Coke contains permitted colours. Pepsi and Coke are cold drinks. (D) Some beer is wine and some beer is vodka. All wine is vodka. All beer must be vodka or wine. (1) Only A (2) A & B (3) C & A (4) Only D (5) None of these 17. (A) All Jadoo is Magic. Some magic is witchcraft. Some Jadoo is witchcraft. (B) Floor C is two storeys above floor D. Floor E & A are ad-

    BPRE–1030

    jacent and above floor C. In the Storey complex, Floor D is the ground Floor. (C) In the kingdom of Lemon Grass, Tom Yarn is 400 km from Jom Yarn. Tom Kha is 300 km from Jom Yorn and away from Tom Yarn. Tom Kha and Tom Yarn are 700 km apart. (D) Ford 1 km is a josh machine. Hyundai Santro is the complete family car. There is little josh in Santro. (1) A & C (2) Only B (3) A & D (4) B & C (5) None of these 18. (A) Some silver is gold. Some gold is platinum and some platinum is silver. Some metal is silver, gold and platinum. (B) Pooja likes to read and write. All who read can dance well. Pooja can write while dancing. (C) DDLJ was released before KNPH and HAHK. HAHK was released after PBDHH. DDLJ was released after PBDHH. (D) Roses are blue. Blue roses are gladiolas. All blue is gladiolas, (1) A & C (2) Only C (3) B & D (4) Only A (5) None of these 19. (A) Sensex crashes on Monday, Wednesday and Friday, Nasdaq raises high on Tuesday, Thursday and Saturday, Sensex follows the Nasdaq movement. (B) Rahui has 50% grey hair. Rahul has 30% black hair. Rahul is 10% bald. (C) Mt. Everest, the highest peak in India, is taller than the second highest peak in China. Mt. Sunfest is the highest peak in China. Mt. Everest may be higher than Mt. Sunfest. (D) On the high way, going from East to West, town A comes after town B. Town C comes after town D. Town C & A comes after town B. (1) Only D (2) C & D (3) Only C (4) A & D (5) None of these

    MISCELLANEOUS Directions (20-21) : In both of these questions, rearrange the given sentences so as to form a coherent paragraph. 20. (A) Commitment to acquiring and building a new skill as well as the allocation of time in the work day schedule are evidence of an attempt to change work behaviour. (B) Reinforcement by upper management and job success should lead to changes in priorities. (C) The changes that are the most difficult and least understood are changes in work values. (D) The new manager must believe that making time for ,,,,.,,, others, planning, coaching and other managerial activities are both necessary and important. (1) CDBA (2) ABCD (3) CBDA (4) ACBD (5) None of these 21. (1) Peter Milner of De Montfart, an automotive consultancy in Lichfield, Britain, proposes to change that by using an unconventional mirror one based on a Fresnel prism. (2) But if fate and the markets smile, one source of interference with clean, aerodynamic lines the wing mirror could soon be smoothed over. (3) Of course, there are other constraints on its design (such as not looking hideous) that prevent this ideal from being realised. (4) A car should, for aerodynamic purposes, be shaped like a tear drop (1) DABC (2) ACBD (3) DCBA (4) ABCD (5) None of these Directions (22-24) : In each of these questions, a sentence is followed by four alternatives (1), (2), (3) and (4). Select from among those alternatives the one which most logically complements the idea contained in the given sentence. 22. Civilised people are not alone in having grasped the idea of superstitious beliefs and practices that are superseded but (1) are still widely cherished (2) are universally followed

    (3) are widely recommended (4) that may still evoke compliance (5) None of these 23. We would all lead more contented and satisfying lives if we judged people not by the symbols they display but by (1) the display they symbolise (2) their individual worth (3) the simple brilliance they emit (4) the human warmth they display (5) None of these 24. Has Smith ever won an award for literature? I. Smith won the only award that existed during his time. II. During Smith’s time, writing was looked down upon, let alone merit an award. Directions (25-32) : Staff employed in a UNESCO office in Paris are represented by four intersecting circles as in the given diagram. Each circle represents people who can read and write English, French, Spanish and Russian. Strength of people in each circle is also shown alongside. Study the diagram to answer these questions. Russian

    English

    B C J H LM D K A

    I

    G

    F

    E

    French A = 40 F=1 A 2

    Spanish

    27. People who can read and write all the languages except Spanish are represented by (1) K (2) G (3) B (4) I (5) None of these 28. People who cannot read and write Russian, English and French are represented by (1) L (2) J (3) H (4) E (5) None of these 29. People who cannot read and write Spanish and French but are conversant with English and Russian are represented by (1) B (2) J (3) M (4) K (5) None of these 30. Which of the following languages is known by the maximum number of people as per the diagram? (1) Spanish Only (2) French only (3) English Only (4) Russian only (5) None of these 31. How many people know only Spanish? (1) 10 (2) 20 (3) 40 (4) 60 (5) None of these 32. How many people can read and write any one language except French? (1) 100 (2) 160 (3) 140 (4) 120 (5) None of these Directions (33-37) : In the diagram given below, the circle represents professors in a Medical college, the triangle stands for Surgical Specialists while the rectangle represents the Medical Specialists

    C = 2A G = 2E

    P S A x

    25. People who can read and write English, French and Spanish are represented by (1) A (2) D (3) K (4) F (5) None of these 26. People who can read and write all the four languages can be represented by (1) J (2) M (3) L (4) K (5) None of these

    BPRE–1031

    M

    D y

    B z

    c

    33. Professors who are also surgical specialists are represented by : (1) Y (2) C (3) D (4) X (5) None of these 34. Surgical specialists who are also medical specialists but no professors are represented by (1) X (2) Y (3) Z (4) B (5) None of these

    MISCELLANEOUS 35. College professors who are also medical specialists are represented by (1) Y (2) X (3) Z (4) A (5) None of these 36. ‘B’ represents (1) professors who are not medical specialists (2) professors who are not sur gical specialists (3) professors who are neither medical specialists nor surgical specialists (4) Medical specialists who are neither professors nor surgical specialists (5) None of these 37. ‘C’ represents (1) professors (2) medical specialists (3) surgical specialists (4) medical and surgical specialists (5) None of these Directions (38-41) : Each of these questions is based on a set of given propositions I to IV. Select the best answer to each of these questions. (I) All P are Q. (II) Some P are not Q. (III)Some, but not all, P are Q. (IV)No P is Q. 38. Considered only by themselves, which of the following pairs of statements might either both be true or both be false? (1) I and II (2) II and III (3) I and III (4) I and IV (5) None of these 39. Considered only by themselves, all of the following pairs of statements might both be false but could not both be true EXCEPT. (1) I and II (2) I and III (3) I and IV (4) II and IV (5) None of these 40. Which of the following pairs of statements considered by themselves cannot both be true and cannot both be false? (1) I and II (2) II and III (3) I and III (4) II and IV (5) None of these 41. Which of the following correctly describes the relationships among the above statements? (1) I, II and III could all be false (2) I, II and III could all be true (3) II, III and IV could all be true (4) II, III and IV could all be false (5) None of these

    42. An application was received by inward clerk in the afternoon of a week day. Next day, he forwarded it to the table of the senior clerk, who was on leave that day. The senior clerk put up the application to the desk officer next day in the evening. The desk officer studied the application and disposed off the matter on the same day ie, Friday. Which day was the application received by the inward clerk? (1) Monday (2) Wednesday (3) Tuesday (4) Previous week’s Saturday (5) None of these Directions (43-47) : Each of these questions has a set of Assertion (1) and Reason (R). Mark your answer as (1) If both A and R are true and R is the correct explanation of A (2) A is true but R is false (3) If both A and R are true but R is not the correct explanation of A (4) A is false but R is true (5) None of these 43. Assertion (A) : For the production of aluminium, cheap electricity is essential. Reason (R) : Extraction of aluminium from its ore requires abundant supply of electricity. 44. Assertion (A) : Winds are deflected to their right in Northern Hemisphere and to the left in the Southern Hemisphere. Reason (R) : Rotation of earth causes the changes in wind direction. 45. Assertion (A) : Noise pollution is an unwanted accumulation of noise in the atmosphere. Reason (R) : It interferes with communication. 46. Assertion (A) : Forest cutting is undesirable from the point of view of soil erosion, Reason (R) : Cutting of forests reduces the percolation of rain water. 47. Statement : Every year, at the beginning or at the end of the monsoons, we have some cases of conjectivities, but this year, it seems to be a major epidemic witnessed after nearly four years.

    BPRE–1032

    Courses of Action : I. Precautionary measures should be taken after every four years to check this epidemic, II. People should be advised to drink boiled water during rainy season. Directions (48-52) : Answer these questions based on the diagram given below. Christians

    1 12

    2

    Professionals Females

    5 8

    9

    7 10 6

    6

    11 3

    Asians

    48. Asian Non-Christian females who are Professionals are represented by (1) 3 (2) 10 (3) 8 (4) 11 (5) None of these 49. Asian females who are neither Professionals nor Christians are denoted by (1) 6 (2) 9 (3) 10 (4) 3 (5) None of these 50. Non-Asian professional Christian males are represented by (1) 10 (2) 11 (3) 12 (4) 9 (5) None of these 51. Christian females who are Non-Professionals and Asians are represented by (1) 5 (2) 10 (3) 9 (4) None of these above (5) All of these 52. The shaded portion depicts (1) Asian Professional Christian males (2) Non-Asian Professional female Christians (3) Non-Christian Professional female Asians (4) Non-Asian Unprofessional female Christians (5) None of these Directions (53-56) : A situation and the outcome are presented. Four statements follow thereafter. Each statement is to be separately evaluated in relation to the situation and outcome. Mark answer as

    MISCELLANEOUS (1) If the statement is deducible from the situation, the outcome or both together (2) If the statement presents a possible adequate explanation of the outcome (3) If the statement is inconsistent with or contradicts, the situation, the outcome or both together (4) If the statement does not support a possible explanation of the outcome (5) None of these Situation : Abhijit Roy is training for a national swimming meet. His event is 800 m freestyle. In winning the last five races, his time has never exceeded 8 min. His practice per formances, in which he studiously attempts to duplicate all actual racing conditions, have been better. He is a strong favourite among local sportswriters to win the meet and his coach Prabhu Dayal predicts that he will win in record time. Speed Swimming Gear, in the hope of capitalising on his upcoming victory, has persuaded Roy to be photographed with their goggles, which he always wears in competition. Outcome : Roy clocks his worst time ever and finishes fourth. 53. The Speed Swimming Gear company was confident that Roy would win the race. 54. Roy’s coach had cautioned him not to expect to do as well as he had in past races. 55. After the race, several swimmers complained about the high chlorine content of the water in the pool. 56. It was revealed after the race that the national swimming meet was fixed. Directions (57-59) : In each of these questions, a group of three related words is given. The relationship among the words can best be represented by one of the four diagrams (1), (2), (3) and (4) given below. Choose the diagram that represents the group of words. (1)

    (3)

    (2) (2)

    (4) (4)

    (5) None of these 57. Teacher, College, Student 58. Parrots, Birds, Mice 59. Nitrogen, Ice, Air 60. Which combination figure best represents the relationship between mosquitoes, ants and insects?

    (1)

    (2)

    (3)

    (4)

    (5) None of these 61. Which of the following diagrams best depicts cousins, nephews and males?

    (1)

    (2)

    (3)

    (4)

    (5) None of these 62. Which of the following combinations of circles best represents atheletes, sprinters and marathon runners?

    (1)

    (2)

    (3)

    (4)

    (5) None of these 64. Whenever a major airplane accident occurs, there is a dramnatic increase in the number of airplane mishaps reported in the media, a phenomenon that may last for as long as a few months after the accident. Airline officials assert that the publicity given the gruesomeness of major airplane accidents focuses media attention on the airline industry and the increase in the number of reported accidents is caused by an increase in the number of news sources covering airline accidents, not by an increase in the number of accidents. Which of the following if true, would seriously weaken the assertions of the airline officials? (1) The publicity surrounding airline accidents is largely limited to the country in which the crash occurred (2) Airline accidents tend to occur far more often during certain peak travel months (3) News organizations do not have any guidelines to help them decide how severe an accident is (4) Airplane accidents receive coverage by news sources only when the news sources find it advantageous to do so 65. In the Venn diagram below, circle represents sports per sons, square represents unmarried persons, triangle represents women and rectangle represents educated persons. Every section is numbered. Study the diagram and answer the question. Which sections are represented by number 11? Sports

    (5) None of these 63. Which of the following combination of figures best represents males, dogs and pets?

    6 5 1 2

    3 4

    7 8

    12

    11 10 9

    Unmarried

    15 14

    Educated

    (1)

    (3)

    BPRE–1033

    13

    Women

    MISCELLANEOUS (1) Married, Educated, Sportsmen (2) Unmarried, Uneducated, Women, Sportsperson (3) Married, Educated, Women, Sportswomen (4) Unmarried, Educated, Sportswomen (5) None of these 66. Which of the options is true based on the Venn diagram given below?

    -Educated -Honest -Urban -Hardworking

    (1) Some honest people are also hardworking and are educated (2) Uneducated people are either honest or hardworking (3) All educated people are urban (4) No person is urban, educated, honest and hardworking (5) None of these Directions (67-68) : In each question given below, there are two statements labelled as Assertion (A) and the other labelled as Reason (R). For these statements, which one of the following is correct? (1) A is true but R is false (2) Both A and R are true but R is not the correct explanation of A (3) Both A and R are true and R is the correct explanation of A (4) A is false but R is true (5) None of these 67. Assertion (A) : Downpour of rain lessens the humidity in the atmosphere. Reason (R) : Rains are caused when atmosphere cannot hold more moisture. 68. Assertion (A) : Eggs of mosquitoes being very light in weight, float on water. Reason (R) : Surface tension of water balances the weight of eggs.

    69. Out of a total of 120 musicians in a club, 5% can play all the three instrumentsio guitar, violin and flute. It so happens that the number of musicians who can play any two and only two of the above instruments is 30. The number of musicians who can play the guitar alone is 40. What is the total number of those who can play violin alone or flute alone? (1) 30 (2) 38 (3) 44 (4) 45 (5) None of these 70. There are 50 students admitted to a nursery class. Some students can speak only English and some can speak only Hindi. Ten students can speak both English and Hindi. If the number of students who can speak English is 21, then how many students can speak Hindi, how many can speak only Hindi and how many can speak only English respectively? (1) 39, 29 and 11 (2) 37, 27 and 13 (3) 28, 18 and 22 (4) 21, 11 and 29 (5) None of these Directions (71-74) : For the Assertion (A) and Reason (R) below, choose the correct alternative from the following. Mark answer as (1) if both A and R are true and R is the correct explanation of A (2) if both A and R are true but R is not the correct explanation of A (3) if A is true but R is false (4) if A is false but R is true (5) None of these 71. Assertion (A) : India should not enter into a ‘No-War -Pact’ with Pakistan. Reason (R) : Because Pakistan’s intentions are not clear. 72. Assertion (A) : India should adopt presidential type of government. Reason (R) : Yes, it will bring political order in the country. 73. Assertion (A) : States of India should be given more autonomy/ freedom. Reason (R) : No, because it will encourage separatist tendencies.

    BPRE–1034

    74. Assertion (A) : Divorce laws should be made more favourable to women in India. Reason (R) : No, it will disrupt the family life, as more women will seek divorce. 75. The diagram given below represents the artists who are singers, dancers and story-writers. Scan the diagram and identify the region which represents the artists who are both story-writers and singers, but not dancers. A

    Singers

    C

    V T

    U

    Dancers

    R

    B Story-writers

    (1) T + U + R + B (2) T (3) A + T + B (4) A + T + U + B (5) None of these 76. In the following figure, which of the regions marked 1-12 represents the bank employees who are post-graduates, probationers and working in urbank bank branches? Stands for Probationers Stands for Postgraduates Stands for employees working in urban bank branches Stands for bank employees

    4 8 7

    2 6 3

    9

    12 5

    10 11 2

    (1) 9 (2) 8 (3) 6 (4) 10 (5) None of these 77. Given below is a figure with four intersecting circles, each representing a group of persons having the quality written against each group. Study the figure carefully and indicate the region represented by persons who are hard-working and disciplined but not studious and enterprising.

    MISCELLANEOUS Hardworking

    Stands for English speaking area

    L

    J

    K O R M N P V Q S

    T

    Enterprising

    Stands for Urdu speaking area Stands for Bhojpuri speaking area

    U

    Stands for Hindi speaking area

    Studies

    (1) P (2) N (3) M (4) K (5) None of these 78. In the following figures, the professors, the politicians, the postgraduates and the corporates are represented as shown. Who among the following corporates is a post-graduate as well as a professor? Small triangle stands for professors

    1 2

    4

    E

    C B

    F G

    H

    Coroporated

    9

    8

    Bhojpuri

    (1) 9 (2) 5 (3) 4 (4) 3 (5) None of these Directions (80-81) : In these questions, choose the appropriate number for the quadrant in which the question mark appears. 80.

    A Postgraduates

    (1) C (2) F (3) G (4) H (5) None of these 79. The following figure shows a part of our country where people speak four languages ie, Hindi, English, Urdu and Bhojpuri. Each such like area where a prominent language is spoken is depicted through a triangle, a square, a circle and a rectangle respectively. Which number depicts Bhojpuri, Hindi and English speaking area but not the Urdu speaking area?

    7

    13

    Stands for coroporates

    Politicians

    11

    6

    5

    10

    Stands for postgraduates

    D

    Urdu

    3

    Hindi

    12

    I

    (1) 7 (2) 9 (3) 3 (4) 5 (5) None of these 83. Compare both the boxes below. Which number can replace the question mark (?) in the box shown below?

    English

    Big triangle stands for politicians

    Professors

    82. Which number would replace question mark (?) in the series?

    Disciplined

    (1) 30 (2) 24 (3) 18 (4) 12 (5) None of these 81.

    (1) 36 (2) 117 (3) 52 (4) 26 (5) None of these

    BPRE–1035

    (1) 5 (2) 4 (3) 6 (4) 7 (5) None of these 84. The Chinese President Hu Jintao on an official visit to India, told President Pratibha Patil that China looks at its relationship with India as one of its most important bilateral ties. He specifically mentioned the growing relationship suggesting that China was ready to look ahead without being influenced by the 1962 war. He also called for strategic cooperative partnership for peace and prosperity. India’s President Pratibha Patil responded with similar sentiments saying that India wants to take the relationship to a new high. Which of the following results would most seriously weaken the theory being tested? (1) The growing ties between India and China are conducive to the peace and stability of Asia and the World at large (2) The constantly growing relationship between India and China is of global and strategic significance

    MISCELLANEOUS (3) China and India are facing common challenges, and broadly share a similar set of interests and responsibilities (4) China has forcibly occupied a big chunk of Indian territory at the border during the 1962 Indo-China war and is claiming Arunachal Pradesh to be a part of China. Normalcy in relationships is not possible under these circumstances (5) None of these Directions (85-88) : For the Assertion (1) and Reason (R) below, choose the correct alternative from the following (1) ‘A’ is true but ‘R’ is false (2) Both ‘A’ and ‘R’ are true but ‘R’ is not the correct explanation of ‘A’ (3) Both ‘A’ and ‘R’ are true and ‘R’ is the correct explanation of ‘A’ (4) ‘A’ is false but ‘R’ is true (5) None of these 85. Assertion : (A) Harmonious industrial relations are necessary for faster economic development of a country. Reason (R) : Good relations keep the wheels of production rolling. 86. Assertion : (1) Stress is a common occurrence in modern life. Reason (R) : Stress is due to absence of meditation and religious belief. 87. Assertion : (A) There appears to be a conflict between work-life and family life. Reason (R) One is more money minded. 88. Assertion : (A) Managers should manage their work-force effectively. 89. Study the following information carefully to answer the given question : Three Aeroplanes of Airline-L had to make an emergency landing in the past four months due to technical glitches. As a result the popularity of the airline declined drastically. Also many people cancelled their existing bookings

    at a short notice and are paying double the fare to the competitor companies instead. Airline-L has to choose one course of action from the options given below. Which amongst the following options would be the best decision for the Airline in the present scenario? (1) Making its fare cheaper than the market rate to ensure that its popularity rises again. (2) Reducing the number of planes flying per day, to ensure that optimum work load per employee is maintained. (3) Reducing the number of cancellation of tickets by charging a very huge amount as cancellation fee. (4) Adding additional checks for ensuring safety and advertising the same. (5) Firing its old technical staff and hiring new staff instead. 90. Study the following information carefully and answer the given question. “Last week, employees of Organisation X were retracted from working beyond regular office hours in order to cut back on extra consumption of electricity. However, I do not notice any change in the number of units of electricity consumed today,” — Manager of Organisation ‘X’ on Monday. Which of the following statements except ONE, may best explain the failure to reducing the electricity

    (4) The employees start all the equipments simultaneously and do not care whether all of these are being used or not. (5) Most of the employees use some personal equipments during office hours continuously. 91. The following diagram shows 10 match-sticks forming three squares. What is the minimum number of matches that must be removed to make two squares?

    (1) 3 (2) 2 (3) 1 (4) 4 (5) None of these 92. In the same diagram given above, what is the minimum number of extra match sticks needed so that by adding them at suitable places, five squares can be formed? (1) 5 (2) 4 (3) 3 (4) 2 (5) None of these Directions (93-95) : In these questions, choose the appropriate number for the quadrant in which the question mark appears : 93.

    consumption? (1) The computers and other electronic equipment in organisation have not been upgraded since past five years and consume one–third more electricity than the modern modes. (2) Owing to Tuesday being the final closing day, most of the employees work for extra hours on Monday. (3) The maximum consumption of electricity for every workpiece including computers take place after 5 O’clock.

    BPRE–1036

    –3

    (1) 13 (2) 18 (3) 30 (4) –30 (5) None of these 94.

    (1) 7 (2) –12 (3) 12 (4) 9 (5) None of these

    MISCELLANEOUS 95.

    Directions (100 – 104): The following table consists of 16 numbers in a 4 × 4 matrix.

    Column 1 Column 2 Column 3 Column 4

    (1) 8 (2) 11 (3) 7 (4) 9 (5) None of these Directions (96-98) : In these questions, find the missing number. 96.

    ROW ROW ROW ROW

    -1 -2 -3 -4

    15 25 30 9 30 50 36 25 36 64 16 50 16 9 15 64 Few numbers are picked from the matrix, as in the exact order as first the ROW is considered and then the COLUMN is considered. For example–21 (2nd row and 1st column) is coded as 30 from the matrix and 32 (3rd row and 2nd column) is coded as 64 from the matrix The numbers picked from the matrix are involved in certain mathematical calculations and the result of which should be decoded as per the table given below : 1. If the result is a perfect square 2. If the result is a perfect square from one of the numbers of matrix given above 3. If the result is a prime number

    (1) 240 (3) 84 (2) 195 (4) 196 (5) None of these 97. 3

    4. If the result is a prime number from the matrix 5. If the result is an odd number

    ?

    6561 81

    6. If the result is an odd number from the matrix

    (1) 18 (2) 27 (3) 24 (4) 9 (5) None of these

    7. If the result is an even number

    98. 54

    24

    30 112

    70

    42

    ?

    28

    38

    (1) 76 (2) 66 (3) 10 (4) 65 (5) None of these Directions (99) : Find out the missing letter/number in place of ?

    8. If the result is an even number from the matrix

    100. What is the code for “32 + 23 – 12 + 41”? (1) 93

    (2) 83

    (3) 34

    (4) 38

    (5) 91

    (1) 94 (3) 100

    (2) 110 (4) 82

    (5) 90 102. What is the code for “12 – 24 + 42 + 33”? (1) 11 (2) 5 (3) 9 (4) 13 (5) None of these

    Square the number and add with the prime number Subtract the prime number from the square of the same prime number Mention the immediate next odd number Mention the MATRIX number as row comes first and then column comes next Mention the immediate preceding even number Mention the MATRIX number as row comes first and then column comes next

    The resultant number should be treated in the same order as given in the above table. For example: if the result is 49, rule number 1 applies not rule number 5

    101. What is the code for “43 – 14 + 32 + 21”?

    99.

    Add the square root of the number with the square number Subtract the square root of the number from the square number

    (1) 16

    (2) 19

    (3) 20

    (4) 18

    (5) 21

    BPRE–1037

    103. What is the code for “34 + 43 – 12 – 24” (1) 11 (2) 43 (3) 33 (4) Either 1 or 2 (5) Either 1 or 3 104. What is the code for “43 + 11 – 33 + 41”? (1) 11 (2) 13 (3) 21 (4) Either 1 or 2 (5) Either 2 or 3

    MISCELLANEOUS 105. There are five brothers A, K, L, M, P and T in different ages. T is elder than M but not L. A has only one younger brother. Either L or K is eldest. T is third or fifth brother. A is elder to P. Who among the following is fourth elder brother? (1) L

    (2) T

    (3) A

    (4) M

    (5) P

    EXPLANATIONS 1. (3) Both Mangoes and Apples are Fruits.

    1. (3)

    2. (3)

    3. (4)

    4. (3)

    5. (3)

    6. (2)

    7. (1)

    8. (4)

    9. (2)

    10. (2)

    11. (4)

    12. (2)

    13. (3)

    14. (1)

    15. (2)

    16. (1)

    17. (4)

    18. (4)

    19. (3)

    20. (4)

    21. (3)

    22. (4)

    23. (2)

    24. (4)

    25. (3)

    26. (2)

    27. (4)

    28. (4)

    29. (1)

    30. (3)

    31. (2)

    32. (3)

    33. (3)

    34. (3)

    35. (2)

    36. (4)

    37. (3)

    38. (2)

    39. (4)

    40. (1)

    41. (1)

    42. (2)

    43. (1)

    44. (1)

    45. (3)

    46. (3)

    47. (3)

    48. (2)

    49. (1)

    50. (3)

    51. (4)

    52. (2)

    53. (1)

    54. (3)

    55. (4)

    56. (2)

    57. (1)

    58. (4)

    59. (4)

    60. (2)

    61. (2)

    62. (1)

    63. (2)

    65. (4)

    66. (1)

    69. (3) 73. (2)

    3. (4) Some men may be musician and some women may be musician and men and women are not related.

    5. (3) Fish comes under the category of animals living in water and herring is not related to it. (6-15) : On the basis of information given in question following venndiagram can be prepared.

    A

    36 20 42

    B

    14 8 16 64 C

    6. (2) 8 votes were in favour of all the three candidates A, B and C. 7. (1) The number of voters in favour of A irrespective of B or C =36+20+8+14 = 78

    17. (4) Option (2) The Pattern of storeys

    E

    of five flats is given

    A

    as per figure formed on

    C

    the basis of 1 and 2 statements B Hence, clearly 3 is deducted D from first two. Option (3) 400

    300

    TOM YAM

    JOM YOM

    TOM Kha

    From the first two statements, it is clearly drawn that Tom kha is 700 km from Tom Yarn. 18. (4) Clearly, the shaded portion of the circle is silver, gold and platinum. S

    G

    P

    8. (4) Number of voters in favour of B irrespective of A and C = 42 + 20 + 16 + 8 = 86

    64. (1)

    9. (2) Number of voters in favour of C irrespective of A and B = 64 + 14 + 8 + 16 =102

    19. (3) It is to be noted that the use of the word may be makes the conclusion true.

    67. (3)

    68. (2)

    10. (2) The number of voters in favour of A and B = 20.

    20. (4) ACBD makes a logical paragraph.

    70. (1)

    71. (1)

    72. (4)

    21. (3) DCBA form a coherent paragraph.

    74. (3)

    75. (2)

    76. (1)

    11. (4) The number of voters in favour of only one of the candidates = 36 + 42 + 64 = 142

    77. (4)

    78. (2)

    79. (4)

    80. (2)

    81. (1)

    82. (2)

    83. (1)

    84. (4)

    12. (2) The number of voters in favour of A and C but not B = 14

    85. (3)

    86. (1)

    87. (1)

    88. (2)

    13. (3) 64 voterx were in favour of C done.

    89. (4)

    90. (3)

    91. (2)

    92. (4)

    93. (3)

    94. (2)

    95. (3)

    96. (2)

    14. (1) Number of voters in favour of B and C but not A = 16

    97. (4)

    98. (2)

    99. (3) 100. (1)

    15. (2) Number of voters in favour of A and C but not B = 14

    101. (2) 102. (3) 103. (4) 104. (5) 105. (4)

    R D F

    2. (3) Both Coffee and Tea are Beverages.

    4. (3) Parrots are birds and men are not related to the section of birds.

    SHORT ANSWERS

    ness is automatically a subset of Rare. Therefore, forgiveness is rare.

    16. (1) From the figure, forgiveness is subset of divine and divine is subset of rare. Hence, forgive-

    BPRE–1038

    22. (4) that may still evoke compliance 23. (2) their individual worth 24. (4) Both the statements do not say that Smith has either won or not won the award for literature. 25. (3) K represents those who can read and write French, English and Spanish, 26. (2) M is common to all the four circles. Hence, it represents those who can read and write all the four languages.

    MISCELLANEOUS 27. (4) I can read and write all the languages except Spanish. 28. (4) E cannot read and write Russian, English and French. 29. (1) B is common area of those who can read and write Spanish and French. 30. (3) A = 40 C = 2A = 80

    1 E= A = 20 2 G = 2E = 40 = A Hence, maximum number of people used English. 31. (2) Spanish language is used by the people who belongs to E and E = 20 32. (3) The number of people who can read and write any one language except French = A + C + E = 40 + 80 + 20 = 140 33. (3) D is common to circle and triangle. Hence, it represents Professors who are also surgical specialists. 34. (3) Z is common to triangle and rectangle. Hence, Z represents surgical specialists who are also medical specialists. 35. (2) X represents professors who are also medical specialists as it is common to circle and rectangle, 36. (4) B represents those who are medical specialists only. 37. (3) C represents surgical specialists only. 38. (2) II and III

    not reduce the problem to any extent. Hence, none of the courses of action follows. 48. (2) Asian Non-christian females who are Professionals are represented by 10. 49. (1) Asian females who are neither Professionals nor Christians are denoted by 6. 50. (3) Non-Asian Professional Christian males are represented by 12. 51. (4) All Asian Christian females are professionals. Hence, there is no such required area. 52. (2) The shaded portion depicts non-Asian Professionals female Christians. 53. (1) If the statement is deducible from the situation, the outcome or both together 54. (3) If the statement is inconsistent with or contradicts, the situation, the outcome or both together 55. (4) If the statement does not support a possible explanation of the outcome 56. (2) If the statement presents a possible adequate explanation of the outcome 57. (1) According to question Venndiagram is shown.

    41. (1) I, II and III could all be false

    64. (1) Airplane accident can occur in any part of world. So, the number of airplane mishaps reported in media may not limited to the country in which the crash occurred. 65. (4) Number 11 represents unmarried, educated, sportswomen. 66. (1) Some honest people are also hardworking and are educated. 67. (3) Both assertion and reason are true and reason is the correct explanation of assertion. 68. (2) Assertion and reason both are true but reason is not the correct explanation of assertion. Guitar

    Teacher Student

    Violin

    Flute

    Number of musicians who can play all the three instruments = 5% of 120 = 6

    College

    58. (4) According to question Venndiagram is shown,

    The number of musicians who can play any two and only two of the three instruments = a+ b+ c = 30

    Parrot

    Mice

    Total number of those who can play violin alone or flute alone = 120 – (40 + 30 + 6)

    59. (4) According to question Venndiagram is shown,

    46. (3) Both A and R are correct but R is not correct explanation of A. 47. (3) Precautionary measures after every four years does not have any impact on the problem. Likewise drinking boiled water does

    63. (2) All the three groups — males, dogs and pets may or may not share the common relationship. Hence, figure (2) best represents the relationship.

    b6 c

    44. (1) Explanation as above. 45. (3) Both Assertion and Reason are correct but R is not correct explanation of A.

    62. (1) Marathon runners and sprinters are sub-groups of atheletes. Also, marathon runners and sprinters may share a common relationship.

    69. (3)

    42. (2) The senior clerk got the application on Wednesday. 43. (1) Explanation as above.

    61. (2) All the three categories cousins, nephews and males are interconnected.

    40 a

    39. (4) II and IV 40. (1) I and II

    60. (2) Both mosquitoes and ants are subsets of insects.

    = 120 – 76 = 44 70. (1)

    Nitrogen

    English Hindi 11 10 29 39 21

    Ice

    Air

    BPRE–1039

    So, Number of students who can speak Hindi = 39

    MISCELLANEOUS Number of students who can speak only Hindi = 29 Number of students who can speak only English =11 71. (1) Both assertion and reason are true as India should first analyse her security concerns, then sign any type of war pact. 72. (4) Adopting presidential type of government is not the way to correct political order in the country. There are various ways in democratic type of government to improve political order. So, assertion is not true but the reason is true. 73. (2) Both assertion and reason are true but reason is not the correct explanation of assertion as freedom does not mean encouragement of separatists. 74. (3) Assertion is true as the women who are suffering in their married life, may lead the nor mal life after divorce. But reason is not true as the women will not seek divorce frequently. 75. (2) T represents the artists who are both story-writers and singers but not dancers. 76. (1) 9 is included in all the figures. So, 9 represents the bank employees who are post-graduates, probationers and working in urban bank branches. 77. (4) K is included in hard-working and disciplined but not in studious and enterprising. 78. (2) F represents the corporates who is a post-graduate as well as a professor. 79. (4) 3 is included in square, rectangle and triangle but not in circle which represents Urdu speaking area. 80. (2) 6 × 7 = 42, 4 × 9 = 36, 8 × 3 = 24 81. (1) 5 × 4 = 20, 3 × 8 = 24, 9 × 4 = 36 82. (2)

    93. (3) 2 × 3 = 6 and 7 × 6 = 42, 1 × 2 = 2 and 9 × 2 = 18

    6 1

    ?

    2

    3

    Now, 6 × 2 – 3 × 1=12 – 3 = 9 83. (1) In the first figure, 6=1+5 2 =1 + 1

    ∴ ˜2 x – 3 = 6 and 5 × 6 = 30 Hence, 30 will come in place of ? 94. (2) 2 × 5 = 10 and 3 × 10 = 30, – l × 5 = –5 and – 5 × l=–5 4 × –l = –4 and 3 × – 4 = –12, Hence, 12 will come in place of ?. 95. (3) l + (–l) = 0, 3 + 2 = 5 and 4 + 3 = 7 Hence, 7 will come in place of ?

    So, in second figure,

    96. (2) (6+2+3+2)2=169-l=168 (5+l + 2+3)2=121-l=120 (3+ 5+4+2)2 = 196-l=195

    5=3+2 7=1 + 6 8 = 3+5

    97. (4) (3) → (3)2 = 9, (9)2 = 81, (8I)2

    4+2+2

    84. (4) India and China are now looking for peace and stability. Considering the part issues will weaken the given theory. 85. (3) Both (A) and (R) are true and (R) is the correct explanation of A. 86. (1) (A) is true but (R) is false because stress because of some pressure or problem.

    = 6561 Hence, the number 9 will come in place of ?. 98. (2) 24 + 30 = 54, 70 + 42 = 112, 38 + 28 = 66 99. (3) W – T = 23 – 20 = 3 S – D = 19 – 4 = 15 T – J = 20 – 10 = 10 P – G = l6 – 7 = 9

    87. (1) (A) is true but (R) is false and not related to (A).

    100. (1) 32 + 23 – 12 + 41

    88. (2) Both (A) and (R) are true but (R) is not the correct explanation of (A).

    ⇒ 116 – 25 = 91

    Reason : (R) Mismanaged workforce is unproductive. 89. (4) From the statements it is clear that the main problem is technical glitches which forced the emergency landing of the aeroplanes. Therefore, Airline L should add additional checking for ensuring safety and advertising the same so that it may regain its reputation. 90. (3) Obviously, option (3) does not explain the failure to reduce the electricity consumption. 91. (2) Two match-sticks in the following ways, if removed, will make two squares.

    ⇒ 64 + 36 – 25 + 16 Rule 5 is applicable. Therefore, required answer would be 93. 101. (2) 43 – 14 + 32 + 21 ⇒ 15 – 9 + 64 + 30 ⇒ 109 – 9 = 100 Rule 1 is applicable.

    100 + 100 = 110 102. (3) 12 – 24 + 42 + 33 ⇒ 25 – 25 + 9 + 16 = 25 Rule 2 is applicable. 25 –

    25

    ⇒ 25 – 5 = 20 103. (4) 34 + 43 – 12 – 24 ⇒ 50 + 15 – 25 – 25 = 15 Rule 6 is applicable 15 ⇒ 11 or 43 104. (5) 43 + 11 – 33 + 41 ⇒ 15 + 15 – 16 + 16

    If we find out the difference between adjacent digits.

    92. (4) By adding two match-sticks, four small and one large square can be formed.

    BPRE–1040

    = 30 ⇒ 21 or 13 105. (4) K > L > T > M > A > P

    ❑❑❑